Sunteți pe pagina 1din 1792

DANS LA MEME SERIE

Toujours exclusivement sur DOC-DZ

RESIDANAT EN POCHE 400 COURS - 2011

Lien : http://doc-dz.forumactif.com/f173-outils-de-preparation-du-concours

RESIDANAT EN POCHE Tome I QCM – QCS – 2011

Lien : http://doc-dz.forumactif.com/f173-outils-de-preparation-du-concours

RESIDANAT EN POCHE Tome II CAS CLINIQUES EN QCM – 2011

Lien : http://doc-dz.forumactif.com/f173-outils-de-preparation-du-concours

RESIDANAT EN POCHE Tome III DOSSIERS - 2011

Lien : http://doc-dz.forumactif.com/f173-outils-de-preparation-du-concours

BONNE CHANCE

NADJI 85
IBN-SINA
RESIDANAT EN POCHE TOME II
Cas Clinique en QCM

1
Exclusivement sur DOC - DZ : www.doc-dz.com NADJI 85
RESIDANAT EN POCHE TOME II
Cas Clinique en QCM

2
Exclusivement sur DOC - DZ : www.doc-dz.com NADJI 85
RESIDANAT EN POCHE TOME II
Cas Clinique en QCM

3
Exclusivement sur DOC - DZ : www.doc-dz.com NADJI 85
RESIDANAT EN POCHE TOME II
Cas Clinique en QCM
Un patient de 50 ans traité pour lombalgies rebelles depuis 4 semaines et ayant pris divers médicaments consulte pour un
ictère d'apparition rapide sans douleur.
Le foie est de taille normale. Il n y a pas de prurit. L'échographie ne révèle pas de dilatation des voies biliaires, les
transaminases sont augmentées à 5 fois la normale. Les phosphatases sont à 1,5 fois la normale. La recherche du virus B est
négative. On note une éosinophilie à 800/mm3.

Dans la liste suivante, quel médicament vous paraît être le plus directement en cause chez ce malade ?
A - Chlorpromazine
B - Stéroïdes alkylés en C17
C - Indométhacine
D - Phénobarbital
E - Acide acétyl salicylique
Bonne(s) réponse(s) : C

C E - Sont les seuls médicaments que le patient a pu prendre logiquement pour ces lombalgies. Les salicylés donnent très
rarement des hépatites cytolytiques.
B - Donnent surtout des cholestases
D - Inducteur enzymatique pur.

La biopsie hépatique pourrait montrer des signes de souffrance hépato-cellulaire. Dans la liste suivante, tous les
signes peuvent s'observer sauf un. Lequel ?
A - Stéatose microvasculaire
B - Prédominance centro-lobulaire des lésions
C - Stéatose macro vasculaire
D - Présence d'éosinophiles
E - Présence de corps de Mallory
Bonne(s) réponse(s) : E

E - Retrouvé exclusivement (quasi) dans les hépatites alcooliques aiguës, savoir cependant que les lésions de l'Amiodarone®
peuvent s'accompagner de corps de Mallory.
B D - Ces signes sont évocateurs d'une atteinte médicamenteuse.

Après une hépatite médicamenteuse cytolytique, indiquez la(les) proposition(s) exacte(s) :


A - Il est possible de réintroduire le médicament sans risque
B - Il faut interdire ce médicament au malade concerné
C - On peut prescrire le médicament en diminuant la dose
D - L'évolution cirrhogène est fréquente
E - Ces hépatites favorisent la survenue secondaire d'une lithiase biliaire
Bonne(s) réponse(s) : B

A - Il y a un risque dans ce type d'hépatite fulminante mortelle.


D - De rares médicaments peuvent être responsables d'hépatite chronique cirrhogène citons Aldomet®, Oxyphénicétine®,
Isoniazide®, Amiodarone®, Dupéran®.

Certaines drogues dites inductrices peuvent faciliter la survenue d'un ictère médicamenteux. Indiquez dans la
liste suivante, celle qui se comporte comme inductrice :
A - Rifampicine
B - Isoniazide
C - Alphaméthyldopa
D - Cimétidine
E - Diurétiques thiazidiques
Bonne(s) réponse(s) : A

D - Est au contraire un "inhibiteur".

Le traitement d'une telle hépatite médicamenteuse fait appel à :


A - L'administration systématique de corticoïdes
B - L'administration systématique de cholestyramine
C - L'administration d'anti-histaminique
D - La prescription de Phénobarbital
E - L'abstention thérapeutique
Bonne(s) réponse(s) : E

Le retrait du médicament en cause amènerait la normalisation des troubles dans la majorité des cas.
D - La prescription d'inducteurs est interdite car dangereuse.

4
Exclusivement sur DOC - DZ : www.doc-dz.com NADJI 85
RESIDANAT EN POCHE TOME II
Cas Clinique en QCM
Une femme de 55 ans vient consulter pour un ictère apparu progressivement, sans fièvre, ni douleur abdominale. Dans ses
antécédents, on note une hypertension artérielle traitée depuis deux ans par de l'Aldomet® (alphaméthyl-dopa), et pour une
arthrose cervico-dorsale du Dupéran (clométacine). Elle est en outre constipée chronique et prend depuis plusieurs années
des laxatifs (Bisacodyl, Séné, Dioctylsulfosuccinate). Elle consomme en moyenne 20 g d'alcool par jour. A l'examen clinique,
on note une ascite modérée, des oedèmes des membres inférieurs ; elle se plaint d'une asthénie avec somnolence diurne et
parfois désorientation. Biologiquement, il existe un bloc bêta-gamma à l'électrophorèse des protides, une vitesse de
sédimentation à 35 mm à la première heure, des triglycérides à 1, 2 mmol/l ; un fer sérique normal, une ferritinémie à une fois
et demie la normale. L'alpha-foetoprotéine est normale.

Le diagnostic de cirrhose est retenu : parmi les signes cliniques suivants, quel(s) est(sont) celui(ceux) qui est
(sont) lié(s) à l'insuffisance hépato-cellulaire ?
A - La circulation collatérale abdominale
B - La splénomégalie
C - Les angiomes stellaires
D - Les oedèmes des membres inférieurs
E - L'érythrose palmaire
Bonne(s) réponse(s) : C E

A.B.D - Signes d'hypertension portale.

Le syndrome confusionnel est attribué à une encéphalopathie hépatique : que recherchez-vous à l'examen
clinique en faveur de ce diagnostic ?
A - Une hypertonie extra-pyramidale
B - Flapping trémor des extrémités
C - Une odeur cétonique de l'haleine
D - Foetor hepaticus
E - Un hippocratisme digital
Bonne(s) réponse(s) : A B D

A - Est un signe fréquent au cours de l'encéphalopathie hépatique.


B et D - Spécifique et peut se rencontrer dans les encéphalopathies métaboliques et rénales.

Une analyse du liquide d'ascite est effectuée : quel résultat affirmerait la présence d'une infection du liquide
d'ascite ?
A - Taux de protides supérieur à 30 g/l
B - Taux de polynucléaires neutrophiles supérieur à 100/mm3
C - Présence d'un germe en culture
D - Taux d'amylases supérieur à 10 fois la normale
E - pH inférieur à 7,34
Bonne(s) réponse(s) : C

Seule la présence d'un germe affirme l'infection, cependant un taux de polynucléaires supérieur ou égal à 250 mm3 chez un
patient présentant une suspicion d'infection du liquide d'ascite suffit à faire débuter le traitement antibiotique.

A partir des éléments de cette observation, quelle(s) étiologie(s) pouvez-vous retenir pour cette cirrhose :
A - L'alpha-méthyl-dopa
B - L'acide tiénilique
C - Les laxatifs
D - L'alcoolisme
E - Une hémochromatose primitive
Bonne(s) réponse(s) : A

B - La patiente ne prend pas de médicament qui peut par ailleurs entraîner des hépatites cytolitiques d'évolution cirrhogène.
C - Seul l'oxyphénacétine et la phénolphtaline sont des laxatifs responsables d'hépatites chroniques.
D - La sensibilité hépatique féminine à l'alcool est plus faible que celle de l'homme mais la dose 20 g/24 h est très faible.
E - L'hémocromatose peut être éliminée sur la normalité de la ferritine.

Une sérologie du virus B est effectuée : parmi les résultats suivants, lequel(lesquels) suggère(nt) fortement une
infection ancienne guérie ?
A - Antigène HBs, antigène HBe positifs seuls
B - Anticorps HBs, anticorps HBe positifs seuls
C - Anticorps HBc de type IgM positif seul
D - Anticorps HBc de type IgG positif seul
E - Aucun de ces résultats
Bonne(s) réponse(s) : B D

L'antigène HBs est nécessairement négatif en cas de guérison.


D - L'AC HBc de type IgG peut être le seul marqueur d'une infection ancienne car l'AC Hbs peut disparaître.
5
Exclusivement sur DOC - DZ : www.doc-dz.com NADJI 85
RESIDANAT EN POCHE TOME II
Cas Clinique en QCM

Parmi les médicaments suivants, quel(s) est (sont) celui(ceux) que vous êtes éventuellement autorisés à
prescrire chez cette patiente ?
A - Phénobarbital
B - Diazépam
C - Erythromycine
D - Rifampicine
E - Glafénine
Bonne(s) réponse(s) : C E

Réponse fausse (C D E)
A.B.D - Sont des Inducteurs enzymatiques et doivent être évités chez cette patiente
E - N'est pas hépatotoxique.
C - L'érythrocine est hépatotoxique ( cholestase et peut être utilisé si nécessaire sous surveillance).

L'ascite de cette patiente n'est pas infectée, l'encéphalopathie hépatique a disparu. Vous allez traiter cette
ascite par des diurétiques de type "antialdostérone" ; quelle(s) contre-indication(s) à ce traitement recherchez-
vous ?
A - Une hypokaliémie
B - Une insuffisance rénale
C - Une hypo-albuminémie
D - Une hyponatrémie
E - Une hyperbilirubinémie
Bonne(s) réponse(s) : B D

Après restriction hydrosalée.


D - L'insuffisance rénale majore l'hyponatrémie.

Un homme de 30 ans a depuis 4 ans une diarrhée faite de 3 à 5 selles par jour, abondantes et souvent décolorées. Il n'y a pas
d'alternance avec une constipation et pas de douleurs abdominales. Il a perdu environ 6 kg ces dernières années malgré la
constatation d'un oedème malléolaire vespéral récent. La rectoscopie est normale et on ne découvre pas de parasites à
l'examen des selles. Le lavement baryté est normal.
Le poids des selles est de 600 g avec un taux de lipides de 30 g par 24 heures.
Des biopsies du 3ème duodénum au cours d'une endoscopie digestive haute permettent de faire le diagnostic d'atrophie
villositaire.
Le diagnostic de maladie coeliaque est porté.

Les examens biologiques peuvent découvrir dans ce cas :


A - Une anémie ferriprive
B - Un allongement du temps de Quick
C - Une hyposécrétion biliaire
D - Une hypocalciurie dans les urines de 24 heures
E - Un défaut d'absorption du D-xylose
Bonne(s) réponse(s) : A B D E

A B D - Par malabsorption du fer, de la vitamine K et de la vitamine D.


E - Test d'exploration d'une malabsorption duodénojéjunale .

Le régime sans gluten doit comporter la suppression du ou des aliment(s) suivants :


A - Le poisson
B - Le pain
C - Les oeufs
D - La viande de boeuf
E - Les pâtes alimentaires
Bonne(s) réponse(s) : B E

Sans commentaire.

L'oedème des membres inférieurs est dans ce cas causé par :


A - Une insuffisance rénale fonctionnelle
B - Une insuffisance cardiaque
C - Une sténose hépatique
D - Une hypoalbuminémie
E - Une carence en vitamines
Bonne(s) réponse(s) : D

Par fuite digestive d'albumine.

6
Exclusivement sur DOC - DZ : www.doc-dz.com NADJI 85
RESIDANAT EN POCHE TOME II
Cas Clinique en QCM

La stéatorrhée de ce patient peut résulter du mécanisme physio-pathologique suivant :


A - Une accélération du transit
B - Une insuffisance pancréatique exocrine
C - Une fistule grêlo-grêlique
D - Un intestin congénitalement court
E - Une réduction du nombre des entérocytes matures
Bonne(s) réponse(s) : E

E - Lié à l'atrophie villositaire totale duodénojéjunale, parfois étendue à l'ensemble du grêle.

L'affection aurait également pu être affirmée par :


A - Une biopsie hépatique
B - Une biopsie jéjunale
C - Un transit baryté du grêle
D - Une biopsie rectale
E - Une analyse des selles
Bonne(s) réponse(s) : A

B - Cf Question précédente.
C - Le transit peut montrer des signes évocateurs : fractionnement de la baryte, dilatation jéjunale, rareté des plis muqueux,
voire absence de ces plis.

Un homme de 60 ans, sans autre antécédent qu'une intoxication alcoolique nettement ralentie, consulte pour une dysphagie
d'apparition récente. Il se plaint de douleurs lombaires depuis trois mois.
La fibroscopie montre une lésion du tiers inférieur de l'oesophage assez régulière qui n'a pu être franchie. A la biopsie :
adénocarcinome.
La biologie montre des phosphatases à 420 UI/l (normale 180) ; des gamma GT et une bilirubine strictement normales. Les
phosphatases acides et la calcémie sont également normales.

Pour apprécier l'extension du processus tumoral, il est nécessaire de demander :


A - Transit oesogastroduodénal
B - Scintigraphie oesophagienne
C - Radiographie pulmonaire
D - Echographie hépatique
E - Lymphographie
Bonne(s) réponse(s) : A C D

A - Précise l'extension en hauteur du processus terminal.


C et D - Recherche une atteinte métastatique et pour C un deuxième cancer.
Nb : Des radios de rachis lombaire une scintigraphie osseuse sont nécessaires dans ce contexte.

Chez ce malade compte-tenu des données fournies, l'augmentation des phosphatases alcalines est évocatrice
de :
A - Métastase hépatique
B - Alcoolisme chronique
C - Métastase osseuse
D - Cancer de la prostate
E - Hyperparathyroïdie
Bonne(s) réponse(s) : C

La normalité des gamma GT permet de rattacher l'élévation des phosphatases alcalines à une ou des métastases osseuses.

Quels sont le(les) examen(s) à demander pour préciser la cause des douleurs lombaires chez ce patient :
A - Radiographie du rachis
B - Scintigraphie osseuse
C - Biopsie de prostate
D - Electrophorèse des protéines
E - Urographie intra-veineuse
Bonne(s) réponse(s) : A B

Sans commentaire.

7
Exclusivement sur DOC - DZ : www.doc-dz.com NADJI 85
RESIDANAT EN POCHE TOME II
Cas Clinique en QCM

Parmi les méthodes thérapeutiques suivantes non chirurgicales, la dysphagie de ce malade est susceptible
d'être améliorée par :
A - Radiothérapie du bas oesophage
B - Chimiothérapie par cisplatinium
C - Mise en place d'une prothèse oesophagienne sous endoscopie
D - Chimiothérapie par le 5 fluoro-uracile
E - Traitement endoscopique par le laser
Bonne(s) réponse(s) : C E

Au niveau du 1/3 inférieur on peut avoir un adénocarcinome non radiosensible ou une tumeur épidermoïde qui, elle, est
radiosensible.
B D - Chimiothérapie en règle inefficace dans les cancers de l'oesophage.

Une femme de 62 ans consulte pour des brûlures rétrosternales, à point de départ épigastrique, survenant principalement
après les repas, mais réveillant quelquefois la patiente la nuit. Il n'existe pas de dysphagie.
On relève dans les antécédents une hystérectomie pour fibrome à l'âge de 38 ans, et une HTA modérée traitée par
Catapressan 1 c/jour.
A l'examen clinique. il existe une surcharge pondérale (73 kg pour 1,65 m). l'abdomen est souple. Pas
d'hépatosplénomégalie.
Les caractéristiques de la symptomatologie vous permettent de suspecter un reflux gastro-oesophagien.

Ce reflux gastro-oesophagien acide peut être prouvé et quantifié par le résultat de :


A - Oesogastroscopie
B - pH métrie oesophagienne
C - Scintigraphie oesophagienne
D - Transit baryté oesogastroduodénal
E - Manométrie oesophagienne
Bonne(s) réponse(s) : B C

En fait seul la PHmétrie peut quantifier un reflux oesophagien acide.


La scintigraphie ne permet pas de différencier reflux acide et alcalin.
A et D - Peuvent visualiser des conséquences du reflux acide (oesophagite).
E - Permet de visualiser des circonstances favorisantes : hypotonie du sphincter inférieur de l'oesophage.

En cas de reflux gastro-oesophagien, quels sont dans le liquide de reflux le(les) constituant(s) toxique(s)
agressifs pour la muqueuse oesophagienne ?
A - Pepsine
B - Sels biliaires
C - Bicarbonates
D - Acide chlorhydrique
E - Mucus
Bonne(s) réponse(s) : A B D

Dans de très rares cas (estomacs opérés), on observe des reflux alcalins très toxiques pour la muqueuse oesophagienne.

Le principal mécanisme antireflux est constitué par le sphincter inférieur de l'oesophage (SIO) Chez le sujet
normal, cochez la(les) proposition(s) exacte(s) concernant ce sphincter :
A - Le SIO se relâche à la déglutition
B - Le SIO a une pression constante de 1 à 5 cm d'eau
C - Le SIO est constitué par un anneau musculaire palpable
D - La pression du SIO augmente après injection de gastrine
E - Le SIO subit des influences médicamenteuses
Bonne(s) réponse(s) : A D E

Sans commentaire.

Cochez l'examen pouvant permettre de mettre en évidence une oesophagite peptique chez cette patiente
présentant cliniquement une symptomatologie de reflux gastro-oesophagien.
A - Oesophagoscopie avec biopsie
B - pH métrie oesophagienne
C - Manométrie oesophagienne
D - Transit baryté oesogastroduodénal
E - Scintigraphie oesophagienne
Bonne(s) réponse(s) : A

Evident, non commentée.

8
Exclusivement sur DOC - DZ : www.doc-dz.com NADJI 85
RESIDANAT EN POCHE TOME II
Cas Clinique en QCM

Cochez la(les) complication(s) possibles au cours de l'évolution d'une oesophagite peptique par reflux :
A - Ulcère oesophagien
B - Rupture de l'oesophage
C - Hernie hiatale
D - Pleurésie gauche
E - Sténose oesophagienne
Bonne(s) réponse(s) : A E

Sans commentaire.

Un endobrachyoesophage peut venir compliquer un reflux gastro-oesophagien. Cochez la(les) proposition(s)


exactes concernant cette complication :
A - Il est constitué par un raccourcissement congénital de l'oesophage
B - Il favorise l'apparition d'un carcinome épidermoïde au niveau du bas oesophage
C - Le transit baryté oesophagien permet le diagnostic d'endobrachyoesophage
D - La muqueuse du bas oesophage est remplacée par une muqueuse glandulaire de type gastrique
E - Il favorise l'apparition d'un adénocarcinome au niveau du bas oesophage.
Bonne(s) réponse(s) : D E

Une surveillance endoscopique s'impose avec biopsies.

En cas de reflux gastro-oesophagien acide, quels sont le(les) moyen(s) thérapeutique(s) efficace(s) sur la
symptomatologie ?
A - Prise d'alginates après les repas
B - Cimétidine (Tagamet®) 1 600 mg/24 h
C - Régime sans résidus
D - Anticholinergiques (dérivés atropiniques) aux 3 repas
E - Cholestyramine (Questran®) 1 sachet à chaque repas
Bonne(s) réponse(s) : A B

A - Transforme l'acidité du contenu gastrique.


B - Tarit la sécrétion gastrique acide.
D - N'est indiqué qu'en cas d'oesophagite peptique.

Si une intervention chirurgicale devient nécessaire pour corriger ce RGO, quelle(s) technique(s) vous paraissent
appropriées ?
A - Myotomie extra-muqueuse de Heller
B - Intervention de Nissen
C - Gastrectomie des 2/3 avec anastomose gastro-duodénale
D - Oesogastrectomie polaire supérieure
E - Vagotomie supra-sélective
Bonne(s) réponse(s) : B

A - Traitement du mégaoesophage idiopathique.


B - Fundoplicature et gastropexie.
C E - Traitement de l'ulcère gastro-duodénal.
D - Traitement d'un cancer du bas oesophage ou du cardia.

Vous êtes amené à examiner une femme de 28 ans souffrant depuis 4 ans d'une dysphagie localisée à la région rétro-sternale
basse, capricieuse, survenant principalement lors de la déglutition des liquides. Cette dysphagie ne s'accompagne pas de
douleur ni de symptôme respiratoire. On retrouve des facteurs psychologiques déclenchants
A l'examen clinique, on ne constate aucune anomalie.
Le transit baryté de l'oesophage montre un rétrécissement progressif et régulier de l'oesophage abdominal ; l'oesophage
thoracique est dilate et atone.
Le diagnostic de mégaoesophage idiopathique est alors confirmé par la fibroscopie et la manométrie oesophagiennes.

Quel(s) est (sont) le(s) mécanisme(s) de la dysphagie chez cette malade ?


A - Compression extrinsèque de l'oesophage
B - Anomalie du péristaltisme du corps de l'oesophage
C - Reflux gastro-oesophagien
D - Défaut de relaxation du sphincter inférieur de l'oesophage
E - Hyperpression intra-abdominale
Bonne(s) réponse(s) : B D

Connaissance.

9
Exclusivement sur DOC - DZ : www.doc-dz.com NADJI 85
RESIDANAT EN POCHE TOME II
Cas Clinique en QCM

Parmi les médicaments suivants, quel est celui susceptible d'améliorer la dysphagie de cette maladie ?
A - Un inhibiteur des récepteurs H2 à l'histamine tel que la Cimétidine (Tagamet®)
B - Un modificateur du comportement du tube digestif tel que le Métoclopramide (Primpéran®)
C - Un antiacide tel que le phosphate d'alumine (Phosphalugel®)
D - Un inhibiteur calcique tel que la Nifédipine (Adalate®)
E - Un chélateur des sels biliaires tel que la Cholestyramine (Questran®)
Bonne(s) réponse(s) : D

Hors programme, il y a aussi les dérivés nitrés.

Quelle(s) autre(s) possibilité(s) thérapeutique(s) pouvez-vous envisager chez cette malade ?


A - Dilatations oesophagiennes
B - Résection de l'oesophage avec interposition d'une anse iléale
C - Intervention de Nissen (fundoplication)
D - Radiothérapie
E - Intervention de Heller (myotomie extra-muqueuse)
Bonne(s) réponse(s) : A E

Voir cas clinique précédent

Dix ans après un traitement bien conduit bien que le résultat fonctionnel soit très satisfaisant, quel examen
complémentaire de surveillance vous apparaît indispensable ?
A - pH métrie oesophagienne
B - Transit baryté oesophagien
C - Coloration vitale per endoscopique de la muqueuse oesophagienne par le Lugol
D - Manométrie oesophagienne
E - Tubage gastrique
Bonne(s) réponse(s) : C

A - La recherche d'un cancer de l'oesophage, complication du mégaoesophage.

Un homme de 35 ans est hospitalisé en urgence pour des douleurs abdominales et un ictère. Dans les antécédents on note
une intoxication alcoolique de 180 g par jour depuis plus de 5 ans. L'examen à l'entrée montre un malade en mauvais état
général, avec ictère cutanéo-muqueux. Il a maigri de 3 kg depuis 2 mois L'examen de l'abdomen montre une hépatomégalie
de 18 cm sur la ligne médio-claviculaire et une splénomégalie. Il n'y a pas d'ascite.
Le bilan biologique montre une anémie macrocytaire, une hyperleucocytose, une hyperamylasémie à 4 fois la normale, une
cholestase avec des phosphatases alcalines à 3 fois la normale et une hyperbilirubinémie à 10 fois la normale
Les radiographies d'abdomen sans préparation ne montrent ni pneumopéritoine, ni niveaux liquides, mais des calcifications en
regard de la 1ère vertèbre lombaire. Une échotomographie réalisée en urgence montre une dilatation importante des voies
biliaires et une augmentation de volume de la tête du pancréas.

Indiquez le diagnostic le plus probable :


A - Une pancréatite aiguë nécrotico-hémorragique
B - Une angiocholite
C - Une cholécystite
D - Une poussée aiguë sur pancréatite chronique
E - Un cancer de la tête du pancréas
Bonne(s) réponse(s) : D

A - L'évolution chronique est peu en faveur de ce diagnostic.


B - L'absence de fièvre élimine l'angiocholite.
D - Les calcifications pancréatiques et le terrain font que ce diagnostic est le premier à évoquer.

Précisez le mécanisme probable de l'ictère :


A - Infectieux
B - Compression du bas-cholédoque
C - Obstacle lithiasique de la voie biliaire principale
D - Atteinte du parenchyme hépatique par l'alcool
E - Lithiase du canal de Wirsung
Bonne(s) réponse(s) : B

B - En rapport avec l'oedème de la tête du pancréas et/ou la sclérose qui comprime la voie biliaire intra-pancréatique.
D - On ne nous donne aucune donnée sur les transaminases, une hépatite chronique aiguë ne peut être éliminée. L'absence
de fièvre et l'élévation des phosphatases alcalines à 4 fois la valeur normale sont peu en faveur de ce diagnostic.

10
Exclusivement sur DOC - DZ : www.doc-dz.com NADJI 85
RESIDANAT EN POCHE TOME II
Cas Clinique en QCM

Indiquez le traitement que vous conseillez pour supprimer cet ictère ?


A - La ponction-drainage per-cutanée des voies biliaires
B - La sphinctérotomie endoscopique
C - La prescription de vitamine B1 associée à l'arrêt de l'intoxication alcoolique
D - La dérivation chirurgicale de la voie biliaire
E - L'instillation endoscopique de cholérétiques
Bonne(s) réponse(s) : D

La dérivation de la voie biliaire est rendue par la distension des voies biliaires qui exposent au risque de cirrhose biliaire
secondaire.
B - C'est le traitement de certaines lithiases cholédociennes.

Indiquez parmi les diagnostics suivants celui qu'évoque chez ce patient la présence d'une splénomégalie :
A - Une septicémie
B - Une hépatite à virus
C - Une hypertension portale segmentaire
D - Une atteinte hématologique alcoolique
E - Une thrombose de l'artère splénique
Bonne(s) réponse(s) : C

Compression de la veine splénique par le parenchyme pancréatique calcifié.

Un malade de 75 ans, emphysémateux, est hospitalisé pour un ictère cutanéomuqueux qui a débuté il y a huit jours.
L'interrogatoire apprend :
- que ce malade se plaint d'un prurit depuis un mois
- que l'ictère a été précédé de douleurs de l'hypochondre droit qui se sont estompées depuis et qui irradiaient vers l'arrière en
hémi-ceinture droite
- que l'ictère a été suivi d'une élévation thermique à 38 degré
- qu'existe une tendance à la diarrhée depuis 6 semaines
- que s'y associent des pesanteurs et une sensation de difficultés d'évacuation épigastrique.
L'examen :
- découvre un foie augmenté de volume. à bord mousse, de surface régulière de consistance normale
- sous lequel on perçoit une grosse vésicule
- un Ictère Intense
- une splénomégalie modérée
Bio : anémie microcytaire. hyperleucocytose 8 000/mm3 avec 78 % de polynucléaires, 198000 plaquettes/mm3., bilirubine
totale 148 mg, pH. alcalines 418 UI (x4n), SGOT 84, (x2n), SGPT 76, TP 65 %, alb.27 g/I, marqueurs viraux 0.

Dans ce cas le prurit :


A - Est dû à l'élévation des phosphatases alcalines
B - Est dû à la présence de pigments biliaires
C - Est secondaire à l'accumulation sous cutanée des acides biliaires
D - Est un signe d'hypercholestérolémie
E - Est proportionnel avec l'élévation des transaminases
Bonne(s) réponse(s) : C

Le Questran® (cholestyramine) permet de chélater les acides biliaires et de les éliminer.

Les douleurs de l'hypochondre droit que décrit le malade évoquent :


A - Une douleur radiculaire
B - Une pleurodynie
C - Une douleur biliaire
D - Un syndrome douloureux solaire
E - Un syndrome ulcéreux
Bonne(s) réponse(s) : C

(sociologie élémentaire).

La triade symptomatique clinique la plus typique au cours de la lithiase cholédocienne est :


A - Douleur, fièvre, ictère dans l'ordre chronologique et en moins de 72 h environ
B - Douleur, hémorragie digestive, occlusion intestinale
C - Douleur, fièvre, ictère dans le désordre
D - Céphalées, urticaire, arthro-myalgies
E - Ictère progressif, apyrétique, indolore
Bonne(s) réponse(s) : A

Cette triade n'est présente que dans 40 à 50 % des lithiases cholédociennes symptomatiques.

11
Exclusivement sur DOC - DZ : www.doc-dz.com NADJI 85
RESIDANAT EN POCHE TOME II
Cas Clinique en QCM

Devant ce cas clinique, quel examen complémentaire proposez-vous en premier ?


A - Transit oeso-gastro-duodénal
B - Artériographie coelio-mésentérique
C - Cholangiographie intra-veineuse
D - Echotomographie
E - Duodénoscopie
Bonne(s) réponse(s) : D

C - La cholangiographie est inutile au cas d'ictère. L'échographie permet de visualiser le parenchyme hépatique (recherche de
métastase), la vésicule biliaire, la voie biliaire extrahépatique (dilatation), la tête du pancréas.

Parmi ces diagnostics, quels sont les deux qui vous paraissent les plus probables chez ce malade ?
A - Hépatite virale non A non B
B - Ampullome vatérien
C - Cancer de la tête du pancréas
D - Cancer secondaire du foie
E - Hépatite alcoolique
Bonne(s) réponse(s) : B C

C Est plus vraisemblable par argument de fréquence


A D E - Ne donnent jamais de grosse vésicule.

Si les radiographies digestives ne mettent pas en évidence de sténoses ou d'ulcérations duodénales. que
conseillez-vous chez ce malade âgé, insuffisant respiratoire ?
A - Rien si ce n'est un régime excluant les boissons alcoolisées
B - Un traitement médical par les antispasmodiques
C - Un traitement par la cholestyramine (Questran®)
D - L'exploration chirurgicale d'emblée
E - Mise en place d'une endoprothèse biliaire
Bonne(s) réponse(s) : E

Le malade étant inopérable et en l'absence de sténose digestive, la pose d'une prothèse transtumérale par voie endoscopique
(ou radiologique transpariétale) peut amener un certain confort à ce type de malade.

Une jeune fille âgée de 19 ans vient vous consulter pour un ictère. Elle n'a pas d'antécédent notable, ne prend ni
médicaments, ni moyen contraceptif. Cet ictère est apparu il y a 48 heures après une grippe qui a duré près d'une semaine.
Elle revenait alors de Bretagne où elle avait passé un mois de vacances. Cet ictère est apparu sans douleur, mais par contre il
faut noter une température à 38°5 pendant 4 jours.
A l'examen clinique, vous notez uniquement une sensibilité de l'hypochondre droit.
Vous avez demandé un bilan biologique qui montre :
. bilirubine totale 40 micro-moles/litre
. bilirubine conjuguée 36 micro-moles/litre
. TGO 1200 UI/I (N 20) . TGP 1400 UI/I (N 20)
. phosphatases alcalines 60 (N : 45)
. numération formule sanguine normale
. antigène Hbs négatif
L'échographie abdominale n'a pas révélé de pathologie bilio-pancréatique.

Quel est le diagnostic étiologique le plus probable :


A - Hépatite A
B - Hépatite B
C - Hépatite mononucléosique
D - Hépatite à cytomégalovirus
E - Aucun de ces diagnostics
Bonne(s) réponse(s) : A

B - Diagnostic rendu très probable par la négativité de l'antigène Hbs. Néanmoins, seul la négativité de l'IgM HBc éliminera
définitivement ce diagnostic.
C D - Ne donnent jamais une telle cytolyse.

Quel est le délai d'incubation de cette hépatite ?


A - 0 à 15 jours
B - 15 à 50 jours
C - 50 à 100 jours
D - 100 à 150 jours
E - Plus de 150 jours
Bonne(s) réponse(s) : B

Sans commentaire.
12
Exclusivement sur DOC - DZ : www.doc-dz.com NADJI 85
RESIDANAT EN POCHE TOME II
Cas Clinique en QCM

Quel est le résultat biologique qui permet d'identifier le virus responsable de cette hépatite ?
A - Anticorps anti-virus A de type IgG
B - Anticorps anti-Hbc positifs
C - Anticorps anti-cytomégalovirus positifs
D - MNI test positif
E - Aucun
Bonne(s) réponse(s) : E

Seuls les IgM-HAV sont élevés à un stade précoce de la maladie.

Dans l'hypothèse où une biopsie hépatique serait effectuée chez cette patiente, quels sont le(les) élément(s)
histologique(s) permettant de confirmer le diagnostic ?
A - Granulome inflammatoire
B - Infiltrat inflammatoire portal
C - Stéatose
D - Nécrose hépatocytaire
E - Fibrose péri-portale
Bonne(s) réponse(s) : B D

En fait, on ne fait jamais de PBH au cours d'une hépatite virale A banale (ce qui est la règle).

Quel est le mode de contamination le plus fréquent de cette hépatite ?


A - Digestif
B - Salivaire
C - Sexuel
D - Parentéral
E - Cutané
Bonne(s) réponse(s) : A

Mode de contamination féco-orale.

Le personnel d'un service d'hémodialyse subit un bilan sérologique d'hépatite B avant mise en route d'un programme de
vaccination par le vaccin contre l'hépatite B. L'une des aide-soignantes, enceinte de 5 mois, vous montre ses résultats actuels
:
Antigène HBs -
Anticorps anti-HBc +
Anticorps anti-HBs -
et ceux effectuées 6 mois auparavant où les mêmes marqueurs étaient négatifs
En l'interrogeant vous apprenez que le début de sa grossesse a été marqué pendant une dizaine de jours par un état
nauséeux, des céphalées et un prurit suivis d'une asthénie jusqu' à la fin du premier trimestre.
Vous demandez immédiatement :
- Anticorps anti-HBe = Positifs
- Dosage des transaminases (ALAT. ASAT) = normal

Que préconisez-vous ?
A - Abstention thérapeutique
B - Vaccination par vaccin anti-hépatite B
C - Injection de globulines anti-HBS jusqu'en fin de grossesse
D - Ponction biopsie du foie
E - Surveillance de l'apparition des anticorps anti-HBs
Bonne(s) réponse(s) : A E

L'absence d'antigène HBs, l'existence d'anti HBe 50 est en faveur d'une guérison proche, on attend la montée des anticorps
antiHbs.

La mère redoute l'atteinte de son enfant. Que lui répondez-vous ?


A - Il est prudent de réaliser un avortement thérapeutique
B - Votre enfant ne court aucun risque
C - Votre enfant sera porteur chronique de l'antigène HBS
D - Votre enfant développera une hépatite 3 à 6 mois après la naissance
E - Une séro-vaccination devra être réalisée à la naissance
Bonne(s) réponse(s) : B

La mère n'est plus antigène HBs et est anticorps anti-E, le foetus ne court aucun risque. En cas de contamination par le virus
B, le risque au 3e trimestre est celui d'accouchement prématuré, de mort in utero et surtout de transmission au nouveau né de
l'antigène HBs surtout si la mère est antigène HBe (+).

13
Exclusivement sur DOC - DZ : www.doc-dz.com NADJI 85
RESIDANAT EN POCHE TOME II
Cas Clinique en QCM

L'infection par le virus de l'hépatite B est un risque statistiquement plus important pour :
A - Homosexuels
B - Ecoliers
C - Drogués par voie intraveineuse
D - Familles des porteurs chroniques de l'antigène HBs
E - Polytransfusés
Bonne(s) réponse(s) : A C D E

Evident.

Quel(s) rôle(s) joue(nt) les facteurs immunologiques dans le déterminisme des lésions hépatiques et
extrahépatiques dues au virus B ?
A - La nécrose hépatique correspond au rejet à médiation cellulaire des hépatocytes infectés
B - L'hépatite fulminante résulte d'une faillite de l'immunité à médiation cellulaire
C - Les prodromes de l'hépatite B s'apparentent à la maladie sérique.
D - L'infection chronique est la conséquence d'une réaction hyperimmune
E - Les symptômes extra-hépatiques pouvant s'observer chez les porteurs chroniques relèvent d'un dépôt de
complexes immuns.
Bonne(s) réponse(s) : A C E

A - Le virus B n'est pas directement cytopathogène.


B - Les lésions hépatiques aiguës sont moins sévères chez les immunodéprimés.

Une femme, obèse, âgée de 72 ans, présente des douleurs de la région inguinocrurale gauche. Celles-ci sont apparues
brutalement 6 h avant l'admission et s'accompagnent de vomissements bilieux. Le transit intestinal est conservé.
Dans les antécédents, on relève une hystérectomie pour myome utérin pratiquée il y a 10 ans.
L'examen clinique de cette femme en bon état général, montre un abdomen souple non distendu. Au niveau de la région
crurale gauche existe une petite tuméfaction douloureuse de 3 cm de diamètre.
Le diagnostic évoqué est celui de hernie crurale étranglée.

En présence d'une hernie crurale, le diagnostic d'étranglement est étayé par la constatation de :
A - Irréductibilité
B - Absence d'impulsion à la toux
C - Douleur vive au collet
D - Augmentation chaleur locale
E - Existence d'une psoïtis
Bonne(s) réponse(s) : A B C

Evident.

Comment peut-on différencier une hernie crurale étranglée d'une adénopathie crurale inflammatoire ?
A - Recherche d'une porte d'entrée
B - Transillumination
C - Ponction exploratrice
D - Aspect des téguments
E - Exploration chirurgicale
Bonne(s) réponse(s) : A E

A - Une porte d'entrée serait en faveur d'une adénopathie inflammatoire (E), en cas de doute, l'exploration chirurgicale
s'impose.

Quel examen complémentaire demandez-vous pour confirmer le diagnostic ?


A - Lavement baryté en double contraste
B - Transit du grêle
C - A. S. P. debout
D - Coloscopie
E - Echographie
Bonne(s) réponse(s) : C

Mise en évidence de niveaux hydroaériques.

14
Exclusivement sur DOC - DZ : www.doc-dz.com NADJI 85
RESIDANAT EN POCHE TOME II
Cas Clinique en QCM

Quel traitement proposez-vous chez cette patiente ?


A - Aspiration digestive avec surveillance de l'évolution locale
B - Intervention en urgence
C - Essai de réduction manuelle
D - Infiltration locale à la procaïne
E - Intervention en cas d'aggravation du tableau d'occlusion
Bonne(s) réponse(s) : B

Risque de sphacèle intestinale à rechercher lors de l'exploration chirurgicale et imposant une résection partielle.

Chez une patiente de cet âge. une occlusion par strangulation peut être également causée par :
A - Iléus biliaire
B - Occlusion sur bride
C - Volvulus du sigmoïde
D - Infarctus intestino-mésentérique
E - Cancer du sigmoïde
Bonne(s) réponse(s) : B C

A E - Sont des occlusions par obstacle.


D - Est une occlusion fonctionnelle par ischémie.

Un homme de 30 ans, sans antécédent médical particulier, est hospitalisé en urgence pour vomisssements, arrêt des
matières et des gaz, accompagnés de douleurs abdominales.
La température est à 38°5. L'examen montre une défense sous-ombilicale prédominante à droite. Le toucher rectal est
douloureux.
Les clichés de l'abdomen montrent des niveaux liquides au niveau du grêle et une aérocolie. La numération objective une
hyperleucocytose à 18 000 GB/mm3 avec 86 % de polynucléaires neutrophiles, les globules rouges sont à 4 800 000/mm3.
avec 11 g d'hémoglobine/100 ml. Le ionogramme sanguin est normal.

Quel est le diagnostic le plus probable ?


A - Diverticulite de Meckel
B - Sigmoïdite
C - Appendicite aiguë méso-coeliaque
D - Perforation de l'ulcère duodénal
E - Pyélonéphrite droite
Bonne(s) réponse(s) : C

Toute occlusion fébrile impose la recherche d'une appendicite méso-coeliaque

Quel(s) geste(s) sera(seront) pratiqué(s) par le chirurgien ?


A - Toilette péritonéale
B - Prélèvements bactériologiques aérobie, anaérobie, du pus
C - Colectomie segmentaire
D - Résection d'un diverticule de Meckel
E - Appendicectomie
Bonne(s) réponse(s) : A B E

Evident.

Au 8e jour post-opératoire, ce patient se plaint de dysurie, de pallakiurie, d'un ralentissement du transit qui avait
repris au 3e jour post-opératoire. On note l'apparition de glaires rectales. La numération montre 20 000 globules
blancs par mm3 avec 90 % de polynucléaires neutrophiles.
Quel diagnostic évoquez-vous ?
A - Abcès de paroi
B - Fistule coecale
C - Hémopéritoine
D - Abcès du Douglas
E - Abscès méso-coeliaque
Bonne(s) réponse(s) : D

Confirmé par une échographie, indispensable pour localiser le siège de cette suppuration profonde.

15
Exclusivement sur DOC - DZ : www.doc-dz.com NADJI 85
RESIDANAT EN POCHE TOME II
Cas Clinique en QCM

Quelle(s) thérapeutique(s) proposez-vous ?


A - Adaptation de l'antibiothérapie d'après l'antibiogramme des germes retrouvés au prélèvement
B - Irrigation de la cavité abdominale
C - Evacuation et drainage
D - Lavements évacuateurs
E - Mise à plat de l'incision cutanée
Bonne(s) réponse(s) : AB

Evident.

Un homme de 30 ans a, depuis 4 ans, une diarrhée faite de 3 à 5 selles par jour, abondantes et souvent décolorées. Il n'y a
pas d'alternance avec une constipation et pas de douleurs abdominales. Il a perdu environ 6 kg ces dernières années, malgré
la constatation d'un oedème malléolaire vespéral récent Il n'a aucun antécédent pathologique, ni médical ni chirurgical.
La rectoscopie est normale et on ne découvre pas de parasites à l'examen des selles Le lavement baryté est normal
Le poids quotidien des selles est de 70 g et le taux des lipides est de 30 g par 24 heures.
Des biopsies du 3è duodénum au cours d'une endoscopie digestive haute permettent de faire le diagnostic d'atrophie
villositaire. Le diagnostic de maladie coeliaque est porté

Les examens biologiques peuvent révéler dans ce cas :


A - Une anémie ferriprive
B - Un allongement du temps de Quick
C - Une hyperbilirubinémie
D - Une hypocalciurie dans les urines de 24 heures
E - Un défaut d'absorption du D-xylose
Bonne(s) réponse(s) : A B D E

Voir corrections autres interrégions même sujet

Le régime sans gluten doit comporter la suppression :


A - Du poisson.
B - Du pain.
C - Des oeufs.
D - De la viande de boeuf.
E - Des pâtes alimentaires.
Bonne(s) réponse(s) : B E

Sans commentaire

L'oedème des membres inférieurs est dans ce cas causé par :


A - Une insuffisance rénale fonctionnelle
B - Une insuffisance cardiaque
C - Une stéatose hépatique
D - Une hypoalbuminémie
E - Une carence en vitamines
Bonne(s) réponse(s) : D

L'hypoalbuminémie entraîne une baisse de pression oncotique vasculaire, une fuite du liquide vers le milieu extracellulaire et
oedème.

Dans cette affection, on pourrait trouver des anomalies en demandant :


A - Biopsie gastrique
B - Biopsie jéjunale
C - Transit baryté du grêle
D - Biopsie rectale
E - Phosphatases alcalines sériques
Bonne(s) réponse(s) : B C E

Voir corrections autres interrégions sur le même sujet.

16
Exclusivement sur DOC - DZ : www.doc-dz.com NADJI 85
RESIDANAT EN POCHE TOME II
Cas Clinique en QCM
Un adolescent de 17 ans, sans antécédent, est hospitalisé pour syndrome abdominal aigu douloureux.
A 9 h du matin, il a présenté une douleur épigastrique vive avec malaise et nausées. Après une accalmie de quelques heures,
la douleur s'est localisée dans la fosse iliaque droite.
La température est à 38°C, le pouls à 110, la tension artérielle à 110 maxima.
Le malade se plaint de nausées, ballonnement, arrêt des gaz.
A l'examen, on retrouve une douleur de la fosse iliaque droite et un psoïtis : il y a une défense au niveau de la fosse iliaque
droite ; le toucher rectal est douloureux à droite. Le reste de l'abdomen est souple, la matité hépatique est conservée, les
fosses lombaires sont libres. La radiographie de l'abdomen sans préparation est normale.
Biologie : GR 4 800 000 - GB 14 500 - PN 83 %. Culot urinaire normal.

Quels sont les 2 signes les plus spécifiques du diagnostic d'appendicite aiguë ?
A - Ballonnement
B - Défense de la fosse iliaque droite
C - Nausées
D - Douleur à droite au toucher rectal
E - Pouls accéléré
Bonne(s) réponse(s) : B D

B - Prédominant point de Mac Burney.

Contre le diagnostic d'ulcère perforé vous retenez :


A - Abdomen sans préparation normal
B - Matité hépatique conservée
C - Douleur de la fosse iliaque droite
D - GB à 14 500
E - Arrêt des gaz
Bonne(s) réponse(s) : B

A - Les conditions d'examen de l'ASP ne permettent pas d'éliminer un pneumopéritoine.

Parmi les éléments suivants, lequel est spécifique du diagnostic d'appendicite aiguë rétro-coecale ?
A - Fièvre à 38,5°C
B - Ballonnement
C - Psoïtis
D - Défense du flanc droit
E - Pouls à 110
Bonne(s) réponse(s) : C

(Connaissance non commentée)

En cas de septicémie d'origine appendiculaire, quels sont les 2 germes le plus souvent rencontrés aux
hémocultures ?
A - Staphylocoque doré
B - Salmonelle
C - Streptocoque bêta hémolytique
D - Escherichia coli
E - Bactéroïdes fragilis
Bonne(s) réponse(s) : D E

D - La flore aérobie est dominée par E.coli.


E - Bacteroide fragilis est un anaérobie non sporulé responsable d'état septicémique et de suppuration.

Parmi les traitements suivants, vous devez retenir dans ce cas :


A - Laparotomie exploratrice
B - Appendicectomie par Mac Burney
C - Drainage par rectotomie
D - Couverture antibiotique péri-opératoire par Flagyl
E - Traitement médical par glace + antibiotiques
Bonne(s) réponse(s) : B E

Le Flagyl® diminuerait la fréquence des abcès de paroi.

17
Exclusivement sur DOC - DZ : www.doc-dz.com NADJI 85
RESIDANAT EN POCHE TOME II
Cas Clinique en QCM
Un sujet de 45 ans vient consulter pour une rectorragie de sang rouge de petite abondance qui s'est reproduite à plusieurs
reprises. Il est en bonne santé apparente et n'a pas maigri. Il se sait porteur d'un ulcère de bulbe, actuellement traité par 400
mg de Cimétidine le soir au coucher, après un traitement d'attaque terminé il y a deux mois. Il est plutôt constipé, allant à la
selle environ un jour sur deux et n'a pas de diarrhée, ni de douleurs à la défécation
L'examen physique révèle un foie palpable, débordant de 4 cm le rebord costal sur la ligne mamelonnaire, lisse, régulier, un
peu dur et à bord inférieur tranchant. Il n'y a ni ascite ni splénomégalie. Il n'y a aucune autre anomalie à l'examen de
l'abdomen et pas d'ictère, pas d'angiomes stellaires. Le toucher rectal ne découvre rien d'anormal.

La rectorragie peut être liée à :


A - L'ulcère duodénal
B - L'insuffisance hépato-cellulaire
C - La rupture de varices oesophagiennes
D - Un ulcère gastrique
E - Aucune des causes ci-dessus
Bonne(s) réponse(s) : E

B - N'a rien à voir avec la question.


A C D - Ne donne de rectorragie qu'en cas d'hémorragie importante avec accélération du transit.

Votre démarche diagnostique comportera par priorité :


A - Une nouvelle gastroscopie
B - Un test hémoculture
C - Un examen proctologique
D - Une échographie hépato-biliaire
E - Une exploration des facteurs de l'hémostase
Bonne(s) réponse(s) : C

Recherche d'une cause locale par TR, examen protologique avec anuscopie, rectoscopie.

Pour rechercher la cause de l'hépatomégalie l'interrogatoire de ce patient devra porter sur :


A - La consommation quotidienne de boissons alcoolisées
B - La notion d'hépatite dans les antécédents
C - La consommation de cigarettes
D - La durée de la prise de Cimétidine
E - L'ancienneté de la maladie ulcéreuse
Bonne(s) réponse(s) : A B

Sont les causes prédominantes d'hépatopathie chez l'homme.

Le toucher rectal normal élimine :


A - Des hémorroïdes
B - Une tumeur du rectum
C - Une hypertension portale
D - Un fécalome
E - Une ulcération thermométrique
Bonne(s) réponse(s) : D

A B E - Ne peuvent être éliminé que par un examen anuscopie + restocospie.

Le traitement d'entretien par la Cimétidine :


A - Réduit le risque de récidive ulcéreuse
B - Entraîne une achlorhydrie qui persiste un mois après l'arrêt du médicament
C - Peut provoquer une anémie par carence en facteur intrinsèque
D - Peut se compliquer chez ce patient d'une gynécomastie
E - Evite chez ce patient le risque de transformation cancéreuse de son ulcère
Bonne(s) réponse(s) : A D

A - Le traitement réduit la fréquence des récidives tant qu'il est administré, cependant le taux de récidive à l'arrêt du traitement
d'entretien est identique à celui des patients n'ayant eu qu'un traitement d'attaque.
D - Complication fréquente de la cimétidine exceptionnelle avec la ranitidine.

18
Exclusivement sur DOC - DZ : www.doc-dz.com NADJI 85
RESIDANAT EN POCHE TOME II
Cas Clinique en QCM
Un homme de 42 ans est admis en urgence pour polytraumatisme. Il est conscient mais son haleine traduit une imprégnation
alcoolique importante. Il est agité.
Sa tension artérielle est stable à 12/8. Ses culs de sac conjonctivaux sont bien colorés. Il est
polypnéique : l''auscultation et la radiographie pulmonaire sont normales.
La palpation de l'abdomen est difficile en raison de l'agitation, mais elle déclenche une douleur nette. Le toucher rectal
également. Il a une fracture de la jambe droite et une fracture du cotyle droit. Seul un cliché d'abdomen sans préparation
couché est possible : il ne montre rien de particulier.

Quel(s) diagnostic(s) évoquez-vous au niveau de l'abdomen ?


A - Rigidité pariétale due à un pré D.T.
B - Hémopéritoine
C - Traumatisme pancréas
D - Rupture d'un viscère creux
E - Contusion pariétale
Bonne(s) réponse(s) : B C E

Le tableau abdominal évoque un syndrome abdominal aigu avec irritation péritonéale. Ceci est compatible avec l'atteinte de
ces 3 organes.

Quel(s) examen(s) simple(s) pratiquez-vous pour raccourcir le temps de doute diagnostique ?


A - Une ponction lavage du péritoine
B - Une échographie abdominale
C - Une artériographie tronc coeliaque + Art. Mésent. Sup.
D - Un scanner abdominal
E - Une amylasémie
Bonne(s) réponse(s) : A B E

B - L'échographie simple rapide a une grande valeur d'orientation.


A - La ponction lavage pourrait être réalisée, en fait la laparotomie en urgence s'impose devant ce tableau. En pratique on fera
:
- échographie immédiate
- ponction dialyse
- la paratomie exploratrice.

Une femme de 62 ans consulte pour des brûlures rétrosternales, à point de départ épigastrique, survenant principalement
après les repas, mais réveillant quelquefois la patiente la nuit. Il n'existe pas de dysphagie. On relève dans les antécédents
une hystérectomie pour fibrome à l'âge de 38 ans, et une HTA modérée traitée par Catapressan 1 c/jour.
A l'examen clinique, il existe une surcharge pondérale (73 Kg pour 1,63m). L'abdomen est souple. Pas
d'hépatosplénomégalie.
Les caractéristiques de la symptomatologie vous permettent de suspecter un reflux gastro-oesophagien.

Parmi les examens paracliniques suivants, un reflux gastro-oesophagien acide peut être démontré et quantifié
par :
A - Oesogastroscopie
B - pH métrie oesophagienne
C - Scintigraphie oesophagienne
D - Transit baryté oesogastroduodénal
E - Manométrie oesophagienne
Bonne(s) réponse(s) : B

Déjà commentée.

Le principal mécanisme antireflux est constitué par le sphincter inférieur de l'oesophage (SIO).
Chez le sujet normal, cochez la(les) proposition(s) exacte(s) concernant ce sphincter :
A - Le SIO se relâche à la déglutition
B - Le SIO a une pression de 1 à 5 cm d'eau
C - Le SIO est constitué par un anneau musculaire palpable
D - La pression du SIO augmente après injection de gastrine
E - Le SIO subit des influences médicamenteuses
Bonne(s) réponse(s) : A D E

(Idem 59).

19
Exclusivement sur DOC - DZ : www.doc-dz.com NADJI 85
RESIDANAT EN POCHE TOME II
Cas Clinique en QCM

Cochez la(les) complication(s) possible(s) au cours de l'évolution d'une oesophagite peptique par reflux :
A - Ulcère oesophagien
B - Rupture de l'oesophage
C - Hernie hiatale
D - Pleurésie gauche
E - Sténose oesophagienne
Bonne(s) réponse(s) : A E

Sans commentaire.

Un endobrachyoesophage peut venir compliquer un reflux gastro-oesophagien. Cochez la(les) proposition(s)


exacte(s) concernant cette complication :
A - Il est constitué par un raccourcissement congénital de l'oesophage
B - Il favorise l'apparition d'un carcinome épidermoïde au niveau du bas oesophage
C - Le transit baryté oesophagien permet le diagnostic d'endobrachyoesophage
D - La muqueuse du bas oesophage est remplacée par une muqueuse glandulaire de type gastrique
E - Le cardia anatomique et le cardia muqueux ne sont pas situés au même niveau
Bonne(s) réponse(s) : D E

L'endobrachyoesophage est une lésion acquise secondaire à la cicatrisation d'une oesophagite ulcérée. Cette cicatrisation se
fait par extension de la muqueuse gastrique qui remplace la muqueuse oesophagienne détruite. Le diagnostic est
endoscopique avec biopsie, le risque de dégénescence à type d'adénocarcinome du bas oesophage est de 10 %.

En cas de reflux gastro-oesophagien acide, quels sont le(les) moyen(s) thérapeutique(s) efficace(s) sur la
symptomatologie ?
A - Prise d'alginates après les repas
B - Cimétidine (Tagamet®) 1200 mg/24 h
C - Surélévation de la tête du lit pendant le sommeil
D - Anticholinergiques (dérivés atropiniques) aux 3 repas
E - Cholestyramine (Questran®) 1 sachet à chaque repas
Bonne(s) réponse(s) : A B C

Le but du traitement est double ; améliorer les mécanismes antireflux :


- règle Hygiénodiététiques
- métoclopranide
- éviction des médicaments diminuant le tonus du SIO.
Réduire l'aggressivité pour l'oesophage du liquide de reflux en utilisant des alginates isolées ou associée aux anti-acides et
aux anti-sécrétoires lorsqu'il existe (pour ces derniers) une oesophagite.

Une malade de 50 ans, obèse,1,56m 88 kgs, sans antécédents pathologique consulte pour un syndrome douloureux
abdominal existant depuis plus de 10 ans, prédominant surtout au niveau de l'hypochondre gauche et de la fosse iliaque
gauche, accompagné d'une constipation modérée (2 à 3 selles par semaine). Il n'existe aucune altération de l'état général
mais la malade se plaint d'un ballonnement abdominal quasi-permanent.
Elle apporte un dossier volumineux comportant :
-Un transit gastroduodénal qui a mis en évidence une petite hernie hiatale. Un lavement baryté de qualité médiocre sans
double contraste qui a décelé il y a deux mois une image polypoïde.`
Le compte-rendu d'une coloscopie faite il ya a une semaine qui n'a pas retrouvé de polype ou d'autre lésion organique.
- Une échographie du foie, du pancréas et des voies biliaires qui montre un volumineux calcul vésiculaire de 2 cm de
diamètre. Ce calcul n'est pas retrouvé sur l'abdomen sans préparation.
L'examen clinique met en évidence une douleur provoquée par la palpation dans la partie gauche de l'abdomen. Le toucher
rectal est normal. La formule numération, la VS et le bilan hépatique sont normaux.
Apparemment le diagnostic de trouble fonctionnel intestinal peut être retenu.

les signes fonctionnels présents dans cette observation peuvent être expliqués par :
A - Des troubles de la motricité colique
B - Des troubles de la sensibilité colique avec diminution de la tolérance à la distension gazeuse
C - Une inflammation motrice de la muqueuse colique
D - Un état psychotique
E - Une hypersécrétion du mucus
Bonne(s) réponse(s) : A B

B - Les colopathes présentent souvent une intolérance importante à la distension colique.

20
Exclusivement sur DOC - DZ : www.doc-dz.com NADJI 85
RESIDANAT EN POCHE TOME II
Cas Clinique en QCM

Parmi les examens complémentaires qui ont étés demandés chez cette malade, quels(s) est (sont) celui (ceux)
qui vous parai(ssen)t superflu(s) ?
A - Transit gastroduodénal
B - Echographie du foie et des voies biliaires
C - Radiographie de l'abdomen sans préparation
D - Lavement baryté sans double contraste
E - Coloscopie
Bonne(s) réponse(s) : A B C D

Seule la coloscopie sera effectuée afin d'éliminer une pathologie associée surtout s'il existe des modifications de la
syptomatologie.

Quelle attitude allez-vous adopter devant la lithiase vésiculaire qui vient d'être découverte ?
A - Cholécystectomie de principe
B - Tentative de dissolution du calcul par l'acide Ursodésoxycholique
C - Abstention thérapeutique
D - Sphinctérotomie endoscopique
E - Prescription prolongée de cholérétiques
Bonne(s) réponse(s) : C

Cette patiente étant obèse et asymptomatique , il n'est pas déraisonnable de proposer une abstention thérapeutique.

Parmi les conseils hygéno-diététiques souvent demandés, vous retiendrez dans le cas de cette malade :
A - Suivre un régime sans résidu
B - Eliminer tous les fruits crus
C - Eviter la consommation de féculents (pour diminuer l'apport calorique)
D - Prendre des fibres alimentaires (15 g de Son de Blé par jour en commençant très progressivement)
E - Ne rien boire entre les repas
Bonne(s) réponse(s) : C D

On conseille également des boissons abondantes, un régime riche en fibre, l'exclusion des excès de graisses, de viande et
alcool et si possible des défécations régulières.

Un homme de 43 ans consulte pour des douleurs épigastriques associées à un amaigrissement récent de 4 kg et une diarrhée
faite de 4 selles par jour. semi-liquides, graisseuses. Ces douleurs surviennent par crises, depuis 5 ans, poste-prandiales
tardives, à irradiation dorsale transfixiante, calmées par l'antéflexion et parfois par l'alimentation. Il avoue la consommation
quotidienne de 2 litres de vin ordinaire en moyenne. A l'xamen clinique, le poids est de 62 kg pour une taille de 1m70.
L'épigastre est tendu et douloureux à la palpation. Le poids des selles est de 600 g/24H ; la stéatorrhée est à 20g/24H le
temps d'apparition du carmin est de 9 H avec un temps de disparition de 20 H. L'abdomen sans préparation montre des
calcifications de l'aire pancréatique.

Le diagnostic de pancréatite chronique est retenu. Parmi les caractères de la douleur, quel(s) est (sont)
celui(ceux) en faveur de son origine pancréatique :
A - Siège épigastrique
B - Post-prandiale tardive
C - Calmée par l'antéflexion
D - Calmée par l'alimentation
E - Irradiation dorsale transfixiante
Bonne(s) réponse(s) : A C E

Connaisssance

Quel est le mécanisme le plus probable de la diarrhée :


A - Accélération du transit intestinal
B - Maldigestion liée à un déficit luminal en lipases
C - Malabsorption liée à une atrophie villositaire jéjunale
D - Pullulation microbienne luminale
E - Malabsorption liée à une neuropathie intestinale
Bonne(s) réponse(s) : B

La lipase est une enzyme dont l'origine est (quasi) exlusivement pancréatique en cas de maladie évoluée du pancréas, il
existe une diminution de la sécrétion du suc pancréatique avec une maldigestion lipidique (stéatorrhée > ou égal à 6g/24 h 3
jours de suite) et protidique (créatorhée).

21
Exclusivement sur DOC - DZ : www.doc-dz.com NADJI 85
RESIDANAT EN POCHE TOME II
Cas Clinique en QCM

Parmi les examens complémentaires suivants, lequel permet d'objectiver un déficit pancréatique exocrine :
A - Hyperglycémie provoquée par voie orale
B - Test au D-xylose
C - Tubage duodénal après stimulation pancréatique
D - Dosage de la lactoferrine dans le liquide pancréatique
E - Dosage de la lipasémie
Bonne(s) réponse(s) : C

B - Explore une malabsorption proximale (duodénojéjunale).


E - Elevée en cas de poussée n'a aucune valeur.
D - L'augmentation du taux de lactoferrine est en faveur d'une pancréatite chronique.
C - Après stimulation par sécrétine et pancréozymine retrouvant une augmentation de la concentration du suc pancréatique en
protéine et une baisse du pH par diminution de la sécrétion d'eau et de bicarbonate et diminution de la concentration
d'enzyme.

Vous suspectez la survenue d'un faux kyste du pancréas : que recherchez vous à l'examen clinique en faveur
de ce diagnostic ?
A - Une splénomégalie
B - Yne hépatomégalie
C - Une masse palpable au creux épigastrique
D - Une encéphalographie pancréatique
E - Un souffle systolique au creux épigastrique
Bonne(s) réponse(s) : C

Survenant dans les semaines suivant une poussée aiguë, les faux kystes sont visualisés au mieux par l'échographie.

Quel(s) est (sont) le(s) résultat(s) des examens complémentaires qui doi(ven)t faire suspecter l'apparition de ce
faux kyste ?
A - La présence de calcification pancréatiques à l'abdomen sans préparation
B - Le refoulement de la face postérieure de l'estomac au transit baryté
C - La survenue d'une hyperamylasémie
D - L'anémie macrocytaire présente à l'hémogramme
E - L'élévation des transaminales à plus de 3 fois la normale
Bonne(s) réponse(s) : B

Les risques de compression sont un risque de compression gastroduodénale, biliaire (ictère avec dilatation des voies
biliaires), colique (lavement baryté de la veine splénique, HTP segmentaire). C'est la persistance d'une hyperamylasémie qui
évoque le diagnostic.

Quel examen compléméntaire effectuerz-vous en première intention pour confirmer la présence de ce faux
kyste ?
A - La fibroscopie gastro-duodénale
B - Le cathétérisme rétrograde per-endoscopique du Wirsung
C - L'artériographie sélective du tronc coeliaque et de l'artère mésentérique
D - L'échotomographie abdominale
E - La tomodensitométrie abdominale
Bonne(s) réponse(s) : D

Sans commentaire

22
Exclusivement sur DOC - DZ : www.doc-dz.com NADJI 85
RESIDANAT EN POCHE TOME II
Cas Clinique en QCM
Une infirmière âgée de 29 ans devient ictérique. Dans ces antécédents il faut noter :
- à l'age de 12 ans, une hépatite ictérique survenue dans un contexte épidémique
- depuis trois ans, la prise régulière d'un contraceptif oral
- un changement d'affectation hospitalière depuis 3 mois
Cet ictère franc, au 5ème jour de son évolution. s'accompagne :
- de fatigue intense,
- d'une élevation des transaminases Alat (Sgpt) x 50, d'une numération formule sanguine montrant
4 500 000 globules rouges, 4 200 globules blancs dont 61 % de polynucléaires neutrophiles. Il n'y a ni hépatomégalie, ni
splénomégalie, ni érythème palmaire. ni angiomes stellaires.

Parmi les propositions suivantes de diagnostic, laquelle retenez-vous ?


A - Hépatite médicamenteuse
B - Hépatite chronique en poussée évolutive
C - Hépatite virale B aiguë
D - Hépatite de la mononucléose infectieuse
E - Hépatite de la toxoplasmose
Bonne(s) réponse(s) : C

A - La prise au long cours des contraceptifs oraux et le caractère très cytolytique de l'hépatite permettent d'éliminer cette
hypothèse.Les ostroprogestatifs donnent des hépatites cholestatiques.
B - L'hépatite "épidémique à l'âge de 12 ans" doit certainement être une hépatite A. Celle-ci ne donne jamais de forme
chronique.
C - En faveur, la profession et le changement récent d'affectation.

Lequel des résultats sérologiques suivants permet l'affirmation du diagnostic ?


A - Ac anti A +
B - Ag HBs +, Ac anti HBc +
C - Séro diagnostic de Wright -
D - Anticorps anti Epstein-barr = VCA = 1/160, EBNA 1/160
E - Sérologie syphilitique TPHA
Bonne(s) réponse(s) : B

Connaissance.

Parmi les signes cliniques suivants, quel est celui qui fait redouter le passage vers une forme grave ?
A - Hyperthermie
B - Augmentation de volume du foie
C - Signes méningés
D - Troubles de la conscience
E - Syndrome polyurique
Bonne(s) réponse(s) : D

L'apparition de troubles de la conscience, des manifestations hémorragiques, la diminution de la taille du foie font craindre une
évolution défavorable traduite sur le plan biologique par une chute des facteurs de coagulation.

Quelle est probablement la spécialité du service dans laquelle cette infirmière avait été affectée ?
A - Pneumologie
B - Hémodialyse
C - Protection maternelle et infantile
D - Rhumatologie
E - Dermatologie
Bonne(s) réponse(s) : B

La transmission du virus B se fait par voie parentérale ou par les sécrétions humaines. Les sujets à haut risque sont les
personnels du centre d'hémodialyse, des centres de transfusions sanguines, les dentistes, les homosexuels, les toxicomanes,
les cirrhotiques alcooliques, les conjoints et les enfants de porteurs chroniques de l'antigène Hbs. L'ensemble des personnels
médicaux et paramédicaux en contact potentiel avec le sang des malades.

Au sixième mois de l'évolution, on suspectera une hépatite chronique si :


A - Une splénomégalie est apparue
B - La biopsie hépatique montre une stéatose de plus de 50 % des hépatocytes
C - Un syndrome myélo-prolifératif se fait jour
D - Une hypogammaglobulinémie apparait
E - La sérologie montre Ag HBe +, Ac anti HBc +
Bonne(s) réponse(s) : A E

A - Témoignant du développement d'une hypertension portale en rapport avec une hépatite chronique active.
E - Témoignant d'une réplication virale persistante. La DNA polymérase et l'HBV-DNA sérique sont des marqueurs de
réplication plus sensible que l'antigène HBe.
23
Exclusivement sur DOC - DZ : www.doc-dz.com NADJI 85
RESIDANAT EN POCHE TOME II
Cas Clinique en QCM
Un homme de 80 ans est hospitalisé en raison de douleurs abdominales évoluant par crises, s'accompagnant d'un arrêt des
matières et des gaz et d'un état nauséeux. Chez ce malade, constipé chronique, les troubles ont débuté deux à trois jours
auparavant. Plusieurs crises analogues relevées dans les antécédents avaient cédé spontanément ou après un lavement. A
l'examen existe un ballonnement abdominal important. Oblique et asymétrique. Le malade est apyrétique, l'état général est
médiocre. Le diagnostic de syndrome occlusif est évoqué.

Le siège colique de cette occlusion est évoqué devant :


A - Précocité de l'arrêt des gaz
B - Ballonnement oblique et asymétrique
C - Contracture abdominale
D - Vacuité de l'ampoule rectale au toucher
E - Altération de l'état général
Bonne(s) réponse(s) : A B D

A - Témoigne d'une occlusion d'origine basse.


B - Le météorisme est plus important en cas d'occlusion colique, il est volontiers oblique.
D - Est en faveur d'une occlusion mécanique (volvulus du côlon pubien).

Vous avez fait effectuer une radiographie d'abdomen sans préparation qui confirme votre impression clinique
d'un volvulus du colon pelvien.
Indiquez parmi les suivants le(s) signe(s) radiologique(s) compatible(s) avec ce diagnostic :
A - Anse dilatée unique occupant la majeure partie de l'abdomen
B - Niveaux liquides multiples sur le grêle
C - Présence d'un épanchement péritonéal réactionnel
D - Grand niveau liquide au niveau de l'anse dilatée
E - Refoulement de l'ombre hépatique
Bonne(s) réponse(s) : A D

Connaissance.

La survenue d'un volvulus du côlon pelvien est favorisé par :


A - Existence d'un dolicho-côlon
B - Présence de diverticules
C - Antécédents de poussées antérieures de sigmoïdite
D - Existence de brides fibreuses du mésocolon
E - Association d'une tumeur de la charnière recto-sigmoïdienne
Bonne(s) réponse(s) : A D

B C - En rapport avec des occlusions coliques par obstruction, par sténose sur sigmoïdite diverticulaire.

Chez ce patient, laquelle des mesures thérapeutiques suivantes conseillez-vous en urgence ?


A - Lavement évacuateur
B - Détorsion par manoeuvres externes
C - Coloscopie
D - Colostomie sigmoïdienne
E - Colectomie sigmoïdienne en urgence
Bonne(s) réponse(s) : C

La coloscopie dans un premier temps permet souvent de "passer le pied de l'Anse" et de dévolvuler le colon. En cas d'échec
ou de suspicion de sphacèle, la chirurgie s'impose.

24
Exclusivement sur DOC - DZ : www.doc-dz.com NADJI 85
RESIDANAT EN POCHE TOME II
Cas Clinique en QCM
Un homme de 23 ans est vu en consultation pour un ictère : son père a remarqué cet ictère au niveau des conjonctives en
dehors de toute symptomatologie. Un de ses frères, plus jeune, a eu des poussées ictériques similaires à 2 reprises.
L'examen clinique est négatif.
Le bilan biologique hépatique montre une bilirubinémie totale à 59 micromol/I dont 9 de conjuguée. Les transaminases sont à
la limite supérieure de la normale. Les phosphatases alcalines sont à la limite supérieure de la normale.
De même, la N.F.S. et l'échotomographie hépatique sont normales.
NORMALE : Bilirubine totale < ou = 17 micromol/l,
Bilirubine conjuguée, traces.

Le diagnostic le plus probable est :


A - Maladie de Rotor
B - Anémie hémolytique congénitale
C - Syndrome de Dubin-Johnson
D - Maladie de Gilbert
E - Lithiase biliaire
Bonne(s) réponse(s) : D

Cf Autres interrégions.

Parmi les signes suivants, quel est celui (ou ceux) que le malade pourrait également signaler ?
A - Douleurs de l'hypochondre droit
B - Prurit
C - Hypercoloration des urines
D - Selles décolorées
E - Aucune des anomalies ci-dessus
Bonne(s) réponse(s) : E

La maladie de Gilbert donne un ictère à bilirubine libre par anomalie de la conjugaison. Les
malades en dehors de l'ictère sont asymptomatiques.

Parmi les examens ci-dessus, quel est celui qui donnera le plus certainement un résultat anormal ?
A - Survie des hématies
B - Test de Coombs
C - G6PD
D - Elévation des Gamma GT
E - Aucun des examens ci-dessus
Bonne(s) réponse(s) : E

Seul le taux de bilirubine libre est augmenté.

La biopsie hépatique montrerait :


A - Un dépôt pigmentaire centro-lobulaire
B - Une cholestase
C - Une architecture hépatique normale
D - Une nécrose discrète focalisée
E - Une inflammation des espaces porte
Bonne(s) réponse(s) : C

La PBH n'est jamais indiquée dans la maladie de Gilbert.

Un homme éthylique de 40 ans a un ictère fébrile à 38°C. A l'examen, on note un foie sensible débordant de 10 cm le rebord
costal. Les TGO sont à 70 Ul/l (normale à 30), les TGP à 40 Ul/l ( normale inférieure à 25), les phosphatases alacalines à 140
Ul/l (normale inférieure à 170), les globules blancs à 20.000/mm3. Le taux de prothrombine est à 60 %, les plaquettes à
110.000/mm3. L'antigène HBs est absent dans le sérum.

Quel est le diagnostic le plus probable ?


A - Un carcinome hépatocellulaire
B - Une angiocholite lithiasique
C - Une hépatite alcoolique aiguë
D - Un kyste hydatique du foie
E - Une hépatite virale aiguë
Bonne(s) réponse(s) : C

L'association, chez un alcoolique ictérique, d'une fièvre avec foie sensible augmenté de volume avec élévation des ASAT
supérieurs à l'élévation des ALAT est en faveur du diagnostic d'hépatite alcoolique aiguë.

25
Exclusivement sur DOC - DZ : www.doc-dz.com NADJI 85
RESIDANAT EN POCHE TOME II
Cas Clinique en QCM

Quel est le meilleur moyen pour confirmer le diagnostic ?


A - Une échographie abdominale
B - Une cholangiographie rétrograde
C - Une biopsie hépatique
D - Un scanner
E - Un abdomen sans préparation
Bonne(s) réponse(s) : C

Celles-ci montrent :
1 - Une nécrose hépatocytaire, des corps de Malory évocateurs
2 - Un infiltrat inflammatoire portal
3 - Une fibrose
Les lésions prédominent dans la région centroglobulaire.

Quelle mesure thérapeutique proposez vous ?


A - Ampicilline 2 g intraveineux par jour pendant 7 jours
B - Flubendazole 2 g par 24 heures pendant 3 semaines
C - Sphinctérotomie endoscopique
D - Sevrage alcoolique
E - Hépactectomie partielle
Bonne(s) réponse(s) : D

Sans commentaire.

Cette affection est due à :


A - L'alcoolisme
B - L'alcoolisme et la dénutrition
C - Au déficit en alcool-deshydrogénase
D - L'excès de fer
E - L'hypoalbuminémie
Bonne(s) réponse(s) : A

A En l'absence de toute anomalie nutritionnelle. Rôle direct de l'alcool.

Une femme de 30 ans non vaccinée contre l'hépatite B, se plaint depuis 10 jours d'asthénie, d'anorexie intense, d'arthralgies
L'examen clinique révèle un subictère et des traces d'injections intraveineuses en rapport avec une toxicomanie. Vous
évoquez une hépatite virale aiguë.

L'examen le plus utile pour confirmer ce diagnostic est :


A - Le dosage des transaminases sériques
B - La détermination du taux de prothrombine
C - L'électrophorèse des protéines plasmatiques
D - Le dosage dans le sang de la bilirubine libre et conjuguée
E - La numération formule sanguine
Bonne(s) réponse(s) : A

Le contexte épidémiologique et le tableau sont très en faveur d'une hépatite aiguë débutante.

Vis-à-vis de quel(s) virus les drogués, partageant leurs seringues, constituent-ils un groupe à risque accru
d'infection ?
A - Le virus de l'hépatite A
B - Le virus de l'hépatite B
C - L'agent Delta
D - Le LAV
E - Le virus de l'Herpès simplex virus type II
Bonne(s) réponse(s) : B C D

Ces trois virus ont un mode de contamination parentérale prédominant et sexuel pour les virus B et le LAV. Le virus delta est
un virus à ARN qui nécessite la présence d'antigène HBs (qui lui sert de manteau) pour se développer.

26
Exclusivement sur DOC - DZ : www.doc-dz.com NADJI 85
RESIDANAT EN POCHE TOME II
Cas Clinique en QCM

Le diagnostic d'hépatite aiguë étant porté chez cette femme, la recherche d'antigène HBs dans le sang est
négative. Quelle recherche doit être positive pour rapporter au virus HB l'hépatite aiguë observée ?
A - La recherche d'anticorps delta
B - Le recherche d'anticorps HBs
C - La recherche d'antigène HBe
D - La recherche d'anticorps HBe
E - La recherche d'anticorps IgM HBc
Bonne(s) réponse(s) : E

A - Témoin d'une surinfection pour le virus Delta.


B - Signe de guérison
C - Témoin d'une réplication virale intense toujours associée à l'antigène HBs.
D - Témoin d'une réplication virale arbotive.

Après avoir porté le diagnostic d'hépatite aiguë à virus HB, vous estimez que le risque de passer à l'infection
chronique est de :
A - 1/1000
B - 1/100
C - 1/10
D - 50 %
E - 95 %
Bonne(s) réponse(s) : C

10 % des hépatites virales B évoluent vers la chronicité (0 % des HVA et 30 à 40 % des hépatites non A non B).

Pour parler d'infection chronique à virus HB il faut, par définition, observer dans le sang :
A - La présence d'antigène HBs au-delà de 6 mois
B - La présence d'antigène HBe
C - La présence d'antigène HBc
D - La présence d'anticorps HBs
E - L'absence d'anticorps HBc
Bonne(s) réponse(s) : A

Connaissance.

Cette femme étant à l'issue de son hépatite aiguë, devenue porteuse chronique de virus HB, quel(s) est (sont)
lors de l'analyse de ces marqueurs sériques le ou les résultat(s) qui témoigne(nt) d'une multiplication virale
particuIièrement intense ?
A - La présence d'anticorps HBs
B - La présence d'antigène HBe
C - La présence d'anticorps HBe
D - La présence d'ADN viral
E - La présence d'ADN polymérase virale
Bonne(s) réponse(s) : B D E

Sans commentaire.

Le développement d'une hépatite chronique active vous fait discuter d'une thérapeutique antivirale. Quel(s)
produit(s) est-il raisonnable d'envisager ?
A - La Vira A
B - Le Marboran
C - L'Amantadine (Mantadix®)
D - Le Virustat
E - L'Inosinoplex
Bonne(s) réponse(s) : A

Sans commentaire.

27
Exclusivement sur DOC - DZ : www.doc-dz.com NADJI 85
RESIDANAT EN POCHE TOME II
Cas Clinique en QCM

Le mari de cette femme exempt, jusqu'alors d'infection à virus HB, a été vacciné contre l'hépatite B lorsque sa
femme a développé son hépatite aiguë. Que peut-on s'attendre à trouver dans son sang quatre mois après la
vaccination, à supposer qu'entre temps, il n'ait toujours pas été infecté par le virus HB ?
A - Anticorps HBs
B - Anticorps HBe
C - Anticorps H Bc
D - Anticorps IgM HBc
E - Aucun des marqueurs cités
Bonne(s) réponse(s) : A

Le vaccin est préparé à partir seulement de l'antigène HBs.

Un homme de 60 ans est hospitalisé pour une première poussée d'ascite. La cirrhose éthylique a été découverte trois mois
plus tôt après une hématémèse rapportée à des varices oesophagiennes. Il était sorti de l'hôpital sans traitement
médicamenteux. On constate une ascite de grande abondance responsable d'une dyspnée et associée à une volumineuse
hernie ombilicale avec une peau amincie. La température du patient est de 38°C. Après un épisode de diarrhée il y a 15 jours,
il a un transit intestinal régulier. L'hémogramme découvre une hyperleucocytose à polynucléaires mais ni anémie ni
thrombopénie. La natrémie est à 120 mmoI/I et la kaliémie à 3,5 mmol/l.
Il n'y a pas d'anomalies parenchymateuses sur la radiographie du thorax.
Après une ponction évacuatrice et exploratrice, l'ascite est considérée comme infectée.

Sur quel(s) argument(s) effectue-t-on dès l'arrivée du patient une ponction évacuatrice ?
A - Les antécédents d'hémorragie digestive
B - La hernie ombilicale
C - Les varices oesophagiennes
D - La notion de diarrhée
E - La dyspnée
Bonne(s) réponse(s) : B E

Témoignant d'une mauvaise tolérance de l'ascite. La hernie risque d'entraîner une rupture
ombilicale avec risque majeur de porte d'entrée infectieuse péritonéale.

Le diagnostic d'ascite infectée est affirmé sur :


A - Protides à 18 g/l
B - Liquide hémorragique
C - 120 leucocytes/mm3
D - 100 polynucléaires/mm3
E - Présence de bacilles Gram négatif à la culture du liquide d'ascite
Bonne(s) réponse(s) : C D E

Sans commentaire.

Le traitement de l'infection de l'ascite comporte :


A - L'injection intra-péritonéale d'un anti-inflammatoire
B - Une antibiothérapie par voie intra-veineuse
C - Une dérivation porto-cave
D - Un antiseptique intestinal
E - Un régime hypo-protidique
Bonne(s) réponse(s) : B

D E - Ont un intérêt pour traiter l'encéphalopathie hépatique qui peut survenir après infection du liquide d'ascite.

Pour réduire le volume de l'ascite, quel traitement proposez-vous à ce patient ?


A - Régime désodé strict
B - Restriction hydrique
C - Un anti-aldostérone
D - Un diurétique thiazidique
E - Un corticoïde
Bonne(s) réponse(s) : A

- Restriction des apports sodés.


- Restriction modérée des apports hydriques.
- Diurétique si la natrémie le permet en l'absence d'insuffisance rénale.

28
Exclusivement sur DOC - DZ : www.doc-dz.com NADJI 85
RESIDANAT EN POCHE TOME II
Cas Clinique en QCM
Un sujet de 60 ans. ayant vécu 30 ans au Cameroun et aux antécédents d'alcoolisme, est venu consulter pour une asthénie et
une anorexie évoluant depuis 3 mois et des oedèmes des membres inférieurs plus récents.
A l'examen physique, on constate des angiomes stellaires et un subictère. Le coeur est normal ainsi que la tension artérielle.
On palpe un foie à bord inférieur dur et tranchant. de surface irrégulière dans la région de l'épigastre. Il existe une circulation
veineuse collatérale abdominale et les signes physiques d'une ascite de moyenne abondance. Il n'y a pas d'astérixis.
Les examens biologiques révèlent une cholestase sans cytolyse, un taux de Quick à 45 % et un antigène HBs positif.
L'ionogramme sérique et la créatininémie sont normaux. Le dosage immuno-enzymatique de l'alpha-foeto-protéine est normal.
L'albumine est à 23 g pour 1000, et les gamma globulines à 20 g pour
1000. L'échographie hépatique met en évidence une zone hyperéchogène de 5 cm de diamètre du lobe gauche évocatrice
d'un processus tumoral et une petite image kystique à parois minces dans le lobe droit.

Chez ce patient, la cholestase peut être due à :


A - La cirrhose
B - Un hépatome
C - Un kyste solitaire du lobe droit
D - Une hépatite aiguë récente
E - Une hépatite paludéenne
Bonne(s) réponse(s) : A B

B - La normalité de l'alfa foeto-protéine n'élimine pas formellement le diagnostic.

Chez ce patient, l'(les) argument(s) du diagnostic d'insuffisance hépato-cellulaire est (sont) :


A - Le taux d'albumine
B - La circulation veineuse collatérale abdominale
C - Les angiomes stellaires
D - Le temps de Quick
E - La cholestase
Bonne(s) réponse(s) : A C D

B - Est un signe d'hypertension portale.

Le traitement de l'ascite comporte chez ce patient :


A - Régime sans sel
B - L'usage des bêta-bloqueurs
C - Une chimiothérapie anti-cancéreuse
D - Une dérivation porto-cave
E - L'usage des anti-aldostérone
Bonne(s) réponse(s) : A E

E - N'est pas contre indiqué car il s'agit d'un épargneur potassique.

Le(les) argument(s) clinique(s) ou épidémiologique(s) en faveur du diagnostic de carcinome hépato-cellulaire


est ou sont les suivants :
A - Cirrhose du foie
B - Antigène HBs positif
C - La zone hétérogène à l'échographie
D - Le kyste intra-hépatique
E - Aucune des propositions ci-dessus
Bonne(s) réponse(s) : A B C

Sans commentaire.

Quel examen complémentaire permettrait d'affirmer le diagnostic de carcinome hépatocellulaire ?


A - Un nouveau dosage de l'alpha F.P par une méthode plus sensible
B - Une artériographie du tronc coeliaque et de ses branches
C - Un test thérapeutique par chimiothérapie anti-cancéreuse
D - Une ponction cytologie dirigée sous échographie
E - Un cathétérisme rétrograde des voies biliaires
Bonne(s) réponse(s) : D

Sans commentaire.

29
Exclusivement sur DOC - DZ : www.doc-dz.com NADJI 85
RESIDANAT EN POCHE TOME II
Cas Clinique en QCM
Une malade de 40 ans est admise en urgence pour des douleurs de l'hypochondre droit. A l'examen, chez cette femme
obèse, vous trouvez un empâtement douloureux de l'hypochondre droit, avec réaction de défense localisée. La température
est de 38°5, l'état général bien conservé.

Quel(s) diagnostic(s) évoquez-vous ?


A - Colique hépatique
B - Pancréatite aiguë
C - Cholécystite aiguë
D - Colique néphrétique
E - Appendicite sous hépatique
Bonne(s) réponse(s) : A C E

La question est mal posée car la colique hépatique, qui est une douleur biliaire, a plusieurs étiologies dont la cholécystite
aiguë.

Quel(s) examen(s) demanderez-vous en urgence ?


A - Ionogramme sanguin
B - Cholécystographie orale
C - Echographie
D - Lavement baryté
E - Abdomen sans préparation
Bonne(s) réponse(s) : A C E

La cholécystographie n'est pas demandée dans ce contexte d'urgence.

Les examens complémentaires mèttent en évidence une lithiase vésiculaire.


Quel examen montre qu'elle peut être responsable de la symptomatologie observée ?
A - Dosage bilirubinémie
B - Dosage amylasémie
C - Echographie
D - Cholécystographie orale
E - Scanner
Bonne(s) réponse(s) : C

L'échographie peut montrer, outre la lithiase, une vésicule distendue, douloureuse au passage de la sonde, à paroi épaisse.

Si la lithiase vésiculaire est responsable de la symptomatologie observée, quelle attitude thérapeutique faut-il
proposer ?
A - Antibiothérapie à visée curative
B - Antibiothérapie préparatoire à l'intervention
C - Sphinctérotomie endoscopique en urgence
D - Antispasmodiques + glace sur le ventre jusqu'à disparition de la symptomatologie
E - Antibiothérapie associée à traitement dissolvant de la lithiase
Bonne(s) réponse(s) : B

Le traitement de la cholécystite aiguë lithiasique est chirurgical. L'intervention est souvent effectuée vers la 48ème heure,
après antibiothérapie.

La lithiase vésiculaire expose a des complications. Parmi les suivantes vous retenez :
A - Migration dans la VBP
B - Péritonite biliaire
C - Cancer vésiculaire
D - Abcès sous hépatique
E - Iléus biliaire
Bonne(s) réponse(s) : A B C E

Le cancer de la vésicule est associé à une lithiase dans 80 à 90 % des cas.

30
Exclusivement sur DOC - DZ : www.doc-dz.com NADJI 85
RESIDANAT EN POCHE TOME II
Cas Clinique en QCM
Une malade âgée de 72 ans se plaint depuis 12 heures de crises de coliques abdominales douloureuses accompagnées de
vomissements alimentaires puis fécaloïdes. L'existence d'une lithiase vésiculaire est connue depuis une dizaine d'années.
Celle-ci s'est manifestée par plusieurs épisodes de colique hépatique : le dernier, il y a un mois environ, avait nécessité des
injections répétées d'antispasmodiques et s'était accompagné de fièvre à 38-38,5 degrés pendant 4 jours. A l'examen vous
retrouvez un abdomen météorisé et sensible mais souple. Le TR est normal.

Quel diagnostic vous paraît le plus probable ?


A - Cholécystite aiguë
B - Angiocholite aiguë
C - Iléus biliaire
D - Péritonite biliaire
E - Aucun des diagnostics cités
Bonne(s) réponse(s) : C

L'iléus biliaire est une complication rare de la lithiase biliaire. Il est observé après fistule bilio digestive.

Quel(s) examen(s) complémentaire(s) demandez-vous aussitôt ?


A - Radiographie de l'abdomen sans préparation en position couchée
B - Radiographie de l'abdomen sans préparation en position debout
C - Echographie hépatique
D - Lavement baryté
E - Artériographie mésentérique supérieure
Bonne(s) réponse(s) : A B C

L'ASP peut montrer le calcul, des signes d'occlusion et/ou une aérobilie. Les renseignements supplémentaires apportés par
un ASP en position couché sont discutés et une réponse BC ne serait pas criminelle ! Enfin, il s'agit d'une échographie
hépatobiliaire et pas seulement hépatique.

Le traitement médical que vous proposez comporte :


A - Aspiration gastrique
B - Sonde rectale
C - Sphinctérotomie endoscopique
D - Rééquilibration hydroélectrolytique
E - Dialyse péritonéale
Bonne(s) réponse(s) : A D

Ces 2 éléments sont la base du traitement médical de toute occlusion.

Si le diagnostic évoqué se confirme et que vous décidez d'intervenir, que faut-il faire au cours de l'intervention ?
A - Cholécystectomie systématique
B - Entérotomie
C - Anastomose bilio-digestive
D - Pancréatectomie
E - Sphinctérotomie
Bonne(s) réponse(s) : B

Le geste essentiel est la levée de l'occlusion, donc l'ablation du calcul après sphinctérotomie.

A quelle complication grave cette patiente serait-elle particulièrement exposée en l'absence d'intervention
chirurgicale ?
A - Péritonite biliaire
B - Péritonite par perforation intestinale
C - Hémorragie digestive
D - Pancréatite nécrosante
E - Angiocholite ictéro-urémigène
Bonne(s) réponse(s) : B

Il s'agit d'une occlusion. Le risque est donc celui d'une perforation intestinale.

31
Exclusivement sur DOC - DZ : www.doc-dz.com NADJI 85
RESIDANAT EN POCHE TOME II
Cas Clinique en QCM
Un homme de 52 ans vous consulte pour fatigue et nausées. Il consomme 2 litres de vin à 11° par jour depuis au moins 20
ans. A l'examen clinique vous remarquez un tremblement fin des doigts et un ictère conjonctival. Le foie mesure 14 cm sur la
ligne médio-claviculaire : sa consistance est souple. Le reste de l'examen clinique est normal.
Vous évoquez une hépatite alcoolique de moyenne gravité.

Un litre de vin à 11° représente une consommation d'alcool d'environ :


A - 20 g
B - 50 g
C - 80 g
D - 130 g
E - 200 g
Bonne(s) réponse(s) : C

Le degré alcoolique d'une boisson exprime le volume d'alcool pur qu'elle contient. 11 g d'alcool pur, dont la densité est de 0,8,
contient : 11 x 0,8 = 88 g d'alcool pur.

Au cours des hépatites alcooliques, on peut observer cliniquement :


A - Fièvre
B - Hépatomégalie
C - Hépatalgies
D - Arthralgies
E - Ictère conjonctival
Bonne(s) réponse(s) : A B C E

Les arthralgies s'observent au cours des hépatites virales.

Au cours des hépatites alcooliques, on peut observer biologiquement :


A - Augmentation de l'activité sérique de la gamma GT
B - Hyperleucocytose
C - Bilirubine normale ou modérément élevée
D - Taux de Quick normal ou abaissé
E - Activité sérique Asat/Alat < 1
Bonne(s) réponse(s) : A B C D E

Une hépatomégalie douloureuse avec fièvre et hyperleucocytose n'est pas toujours une angiocholite !

On souhaite pratiquer une biopsie hépatique transcutanée. Cet examen est contre indiqué en cas de :
A - Ictère franc
B - Taux de Quick 30 %
C - Plaquettes 60000/mm3
D - Voies biliaires intra-hépatiques dilatés
E - Circulation collatérale.
Bonne(s) réponse(s) : B C D

La cholestase intrahépatique (responsable d'une dilatation des voies biliaires intrahépatiques) est une contre indication car
elle accroît le risque de cholépéritoine.

L'examen histologique, au cours des hépatites alcooliques aiguës. peut montrer tous les signes ci-dessous sauf
un. Lequel ?
A - Une stéatose
B - Des corps de Mallory
C - Un infiltrat à polynucléaires neutrophiles
D - Une nécrose hépatocytaire acidophile
E - Des corps de Councilman
Bonne(s) réponse(s) : E

Les corps de Mallory (hépatocyte transformé en un corps hyalin acidophile) s'observent en cours des hépatites virales aiguës.

32
Exclusivement sur DOC - DZ : www.doc-dz.com NADJI 85
RESIDANAT EN POCHE TOME II
Cas Clinique en QCM
Une malade de 65 ans est adressée aux urgences pour un syndrome abdominal aigu. Elle n'a pas d'antécédents notables en
dehors d'une crise douloureuse de la fosse iliaque gauche, sans irradiation, avec fièvre et constipation, il y a deux ans. Cette
crise avait cédé en une semaine sous régime alimentaire pauvre en fibres.
L'épisode actuel a débuté il y a 48 heures par des douleurs abdominales diffuses avec diarrhée. Elle a appelé son médecin
traitant ce jour car elle a vomi deux fois, n'a pratiquement plus de gaz, alors que la douleur abdominale persiste, localisée
dans le flanc gauche.
A l'examen, la tension artérielle est à 160-80 mmHg, le pouls à 120/mn, la température à 38°5 sans frisson.
L'abdomen est ballonné. souple, sauf dans la fosse iliaque gauche où il se défend nettement. Les fosses lombaires sont
libres. Il existe une douleur à gauche au toucher rectal. Il n'y a pas de masse perceptible. L'appareil génital est normal. Il n'y a
aucun trouble urinaire. Les orifices herniaires sont libres.
Les radiographies de l'abdomen sans préparation éliminent un pneumopéritoine.
En revanche, on observe une dilatation importante du colon, sans niveau liquide du caecum jusqu'à la région lombaire
gauche. Le sigmoïde et le rectum ne sont pas visibles. La radiographie pulmonaire est normale. Les globules blancs sont à
13.000/mm3.

La crise douloureuse fébrile de la fosse iliaque gauche présentée par la malade il y a 2 ans, évoque en premier
un diagnostic. Lequel ?
A - Une colique néphrétique gauche
B - Une poussée de pancréatite caudale
C - Une complication d'un polype colique
D - Une poussée de sigmoïdite diverticulaire
E - Aucune des propositions précédentes n'est exacte
Bonne(s) réponse(s) : D

La poussée modérée de sigmoïdite réalise classiquement un tableau "d'appendicite à gauche".

L'épisode actuel peut correspondre à :


A - Un cancer du colon gauche non compliqué
B - Un volvulus du colon pelvien
C - Une sigmoïdite diverticulaire
D - Un phlegmon péri-néphrétique
E - Un abcès péri-néoplasique du colon
Bonne(s) réponse(s) : C E

En cas de volvulus du côlon pelvien, il existe à l'ASP une volumineuse image hydroaérique.

Un examen est indiqué dans ce cas. Lequel ?


A - Lavement baryté
B - Lavement baryté avec double contraste
C - Lavement avec produits hydrosolubles
D - Coloscopie
E - Echographie
Bonne(s) réponse(s) : C

Le lavement baryté est formellement contre-indiqué car la perforation est à craindre en permanence. La question est un peu
ambiguë car une échographie peut aider au diagnostic d'abcès.

En l'absence de traitement, la malade pourrait présenter :


A - Une péritonite généralisée
B - Une occlusion aiguë
C - Une pneumaturie
D - Une septicémie
E - Aucune des propositions précédentes n'est exacte
Bonne(s) réponse(s) : A B C D

La question est mal posée car la malade semble déjà en occlusion (vomissements ; arrêt des gaz).

En cas d'intervention chirurgicale, une colostomie sera :


A - Une éventualité dont on préviendra la malade
B - Parfois le seul geste effectué sur le colon
C - Parfois effectuée en amont d'une anastomose
D - Parfois effectuée en aval d'une anastomose
E - Parfois terminale
Bonne(s) réponse(s) : A B C E

Il peut s'agir d'une colostomie terminale mais transitoire, (rétablissement secondaire de la continuité). Prévenir le malade est
indispensable.

33
Exclusivement sur DOC - DZ : www.doc-dz.com NADJI 85
RESIDANAT EN POCHE TOME II
Cas Clinique en QCM
Homme de 20 ans, toxicomane occasionnel à l'héroïne intra-veineuse depuis un an, mais sevré depuis un mois, se plaint
d'une asthénie, de céphalées depuis 10 jours avec installation en 5 jours d'un ictère modéré sans douleur ni fièvre. L'examen
clinique est normal en dehors de l'ictère. Il n'y a pas d'adénopathie, pas de souffle cardiaque, pas d'hépatomégalie et l'état
général est conservé. Biologiquement, on note : NFS normale, ionogramme normal, bilirubine totale 50 micromol/l, bilirubine
conjuguée 39 micromol/l, transaminases SGOT 2250 Ul/l (NI : 45), SGPT 2440 Ul/l (NI : 45), élévation des phosphatases
alcalines à 2 fois la normale. Temps de Quick 100 %. La sérologie virale confirme l'hépatite virale B en retrouvant : Ag HBs
positif, Ac anti-HBs négatif, anticorps anti HBc positif, Ag HBe positif, Ac anti-delta négatif.

Dés lors vous prescrivez :


A - Repos strict au lit au moins 10 jours
B - Arrêt de toute boisson alcoolisée
C - Régime sans graisse
D - Vidarabine (VIRA-A) en traitement de 15 jours
E - Arrêt de tout médicament hépatotoxique
Bonne(s) réponse(s) : B E

Le repos est conseillé mais il parait inutile qu'il soit strict et prolongé.

Afin d'éviter la transmission de l'hépatite, vous recommandez :


A - Isolement strict en chambre seul à l'hôpital
B - Isolement au domicile en déconseillant toute visite
C - Produits de toilette et couverts individuels et jetables
D - Désinfection du linge et de la literie
E - Utilisation de préservatifs en cas de rapports sexuels
Bonne(s) réponse(s) : C E

La recommandation essentielle est E car la transmission, du VHB est surtout parentérale.

L'amie du malade vit avec lui depuis 2 mois. Son examen clinique et son bilan biologique hépatique sont
normaux. La sérologie du virus B est négative. Que lui proposez-vous ?
A - Une vaccination contre le virus B par Hevac B : une injection par an
B - Une vaccination par Hevac B : 2 injections en 2 mois puis rappel tous les 10 ans
C - Une vaccination par Hevac B, 3 injections en 3 mois puis rappel un an plus tard puis tous les 5 ans
D - Une sérothérapie par immunoglobulines polyvalentes Gamma 16 ou Gamma TS
E - Un dosage hebdomadaire des transaminases
Bonne(s) réponse(s) : C

Il s'agit en fait d'une réponse incomplète car la meilleure proposition serait d'associer la vaccination et une sérothérapie par
immoglobulines spécifiques anti-hépatite B.

Lors de la surveillance de l'hépatite du malade, quel élément est le seul à ne pas imposer une hospitalisation
d'urgence devant le risque d'hépatite fulminante :
A - Diminution du temps de Quick (inférieur à 50 %)
B - Troubles de la conscience
C - Agitation
D - Flapping tremor
E - Transaminasémie supérieure à 5 000 Ul/l
Bonne(s) réponse(s) : E

Une élévation importante des transaminases n'a pas de valeur pronostique.

34
Exclusivement sur DOC - DZ : www.doc-dz.com NADJI 85
RESIDANAT EN POCHE TOME II
Cas Clinique en QCM
Une femme de 55 ans vient consulter pour un ictère apparu progressivement, sans fièvre, ni douleur abdominale. Dans ses
antécédents, on note une hypertension artérielle traitée depuis deux ans par de l'Aldomet® (alphaméthyldopa), et pour une
arthrose cervico-dorsale par Dupéran (clométacine). Elle est en outre constipée chronique et prend depuis plusieurs années
des laxatifs (Bisacodyl, Séné, Dioctylsulfosuccinate). Elle absorbe en moyenne 20 g d'alcool par jour. A l'examen clinique, on
note une ascite modérée, des oedèmes des membres inférieurs; elle se plaint d'une asthénie avec somnolence diurne et
parfois désorientation. Biologiquement, il existe un bloc bêta-gamma à l'électrophorèse des protides. une vitesse de
sédimentation à 35 mm à la première heure, des triglycérides à 1,2 mmol/l, un fer sérique normal, une ferritinémie à une fois
et demie la normale. L'alpha-foeto-protéine est normale.

Le diagnostic de cirrhose est retenu. L'insuffisance hépato-cellulaire se traduit par :


A - La circulation collatérale abdominale
B - La splénomégalie
C - Les angiomes stellaires
D - Les oedèmes des membres inférieurs
E - L'érythrose palmaire
Bonne(s) réponse(s) : C E

La question est ambiguë car le diagnostic d'insuffisance hépato cellulaire ne peut pas être porté devant des oedèmes isolés
des membres inférieurs. Cependant, l'insuffisance hépatocellulaire peut être responsable d'une rétention hydrosodée se
traduisant par des oedèmes. La réponse pourrait donc être CDE.

Le syndrome confusionnel est attribué à une encéphalopathie hépatique : que recherchez-vous à l'examen
clinique en faveur de ce diagnostic ?
A - Une hypertonie extra-pyramidale
B - Flapping tremor des extrémités
C - Une odeur cétonique de l'haleine
D - Un signe de Babinski variable
E - Un hippocratisme digital
Bonne(s) réponse(s) : A B D

La question est encore mal posée car il faudrait préciser Babinski bilatéral. La réponse pourrait donc être AB.

Une analyse du liquide d'ascite est effectuée : quel résultat affirmerait la présence d'une infection du liquide
d'ascite ?
A - Taux de protides supérieur à 30 g/l
B - Taux de polynucléaires neutrophiles supérieur à 50/mm3
C - Présence d'un germe en culture
D - Taux d'amylases supérieur à 10 fois la normale
E - pH inférieur à 7,34
Bonne(s) réponse(s) : C

Le taux de protides peut être très variable au cours d'une infection d'ascite, mais ce sont les ascites avec faible taux de
protides qui sont particulièrement sujettes au risque d'infection.

A partir des éléments de cette observation, quelle(s) étiologie(s) pouvez-vous retenir pour cette cirrhose ?
A - L'alpha-méthyl-dopa
B - La clométacine
C - Les laxatifs
D - L'alcoolisme
E - Une hémochromatose primitive
Bonne(s) réponse(s) : A B D

La question est difficile car la prise d'Aldomet® est relativement courte. La consommation d'alcool est faible mais, chez la
femme, dès 20 g/j, le risque d'hépatopathie n'est pas nulle. De plus, l'existence d'un bloc bêta des IgA est en faveur de
l'étiologie alcoolique traduisant une élévation.

Parmi les médicaments suivants, quel(s) est (sont) celui(ceux) que vous êtes éventuellement autorisés à
prescrire chez cette patiente :
A - Phénobarbital
B - Diazépam
C - Erythromycine
D - Rifampicine
E - Glafénine
Bonne(s) réponse(s) : A B C D E

Aucun médicament n'est formellement contre-indiqué mais il vaudrait mieux éviter le diazépam qui peut éventuellement
entraîner des troubles de la conscience sur ce terrain ainsi que l'érythromycine et la glafénine qui peuvent causer des
hépatites allergiques.
35
Exclusivement sur DOC - DZ : www.doc-dz.com NADJI 85
RESIDANAT EN POCHE TOME II
Cas Clinique en QCM

L'ascite de cette patiente n'est pas infectée, l'encéphalopathie hépatique a disparu. Vous allez traiter cette
ascite par des diurétiques de type "antialdostérone" ; le cas échéant, ce traitement serait contre indiqué par :
A - Une hypokaliémie
B - Une insuffisance rénale
C - Une hypo-albuminémie
D - Une hyponatrémie
E - Une hyperbilirubinémie
Bonne(s) réponse(s) : B D

L'hyperkaliémie et non l'hypokaliémie est une contre-indication.

Homme de 49 ans, éthylo-tabagique, présentant depuis plusieurs années des douleurs épigastriques et rétro-sternales
persistantes sans atteinte de l'état général. Traitement sporadique par la prise d'anti-acides.
Le transit oeso-gastrique fait antérieurement était dans les limites de la normale, en-dehors d'un reflux gastro-oesophagien
L'intéressé présente depuis quelques mois une gêne à la déglutition sans atteinte de l'état général. L'endoscopie objective
une sténose concentrique au tiers inférieur de l'oesophage.

Compte tenu des données anamnestiques et cliniques, quelles sont les deux hypothèses diagnostiques que
vous placez en premier rang ?
A - Achalasie
B - Cancer de l'oesophage
C - Syndrome de Plummer-Wilson
D - Sténose peptique de l'oesophage
E - Diverticule épiphrénique
Bonne(s) réponse(s) : B D

Une dysphagie chez un alcoo-lotabagique doit faire systématiquement évoquer un cancer de l'oesophage et donc faire
pratiquer une endoscopie. Toutefois ici, les caractères de la dysphagie, la notion de reflux et l'aspect endoscopique sont très
évocateurs d'une sténose peptique.

Le diverticule de Zencker correspond à la (aux) caractéristique(s) suivante(s) :


A - Très généralement associé à un reflux gastro-oesophagien
B - Situé au niveau de l'espace crico-pharyngien
C - Risque élevé de dégénérescence cancéreuse
D - En rapport avec l'attraction de l'oesophage par des adénopathies
E - Fréquent chez l'adulte jeune
Bonne(s) réponse(s) : B

La diverticule de Zencker, ou diverticule pharyngo oesophagien s'observe surtout chez l'homme après 60 ans.

Le cancer de l'oesophage peut être en rapport avec les facteurs étiopathogéniques suivants, sauf un :
A - Brûlures caustiques de l'oesophage
B - Ethylo-tabagisme
C - Diverticule épiphrénique
D - Endo-brachyoesophage
E - Méga-oesophage idiopathique
Bonne(s) réponse(s) : C

L'éthylo-tabagisme est en France le principal facteur étiopathogénique.

Quelle caractéristique évolutive n'est pas observée dans le cancer de l'oesophage thoracique ?
A - Envahissement des ganglions coeliaques
B - Fistule oeso-aérienne
C - Atteinte du nerf récurrent droit
D - Métastases hépatiques
E - Hématémèse massive
Bonne(s) réponse(s) : C

Le récurrent droit ne descend pas en-dessous de la sous clavière.


Par contre, une paralysie récurrentielle gauche peut s'observer dans le cancer de l'oesophage thoracique car le récurrent G a
un trajet thoracique (il passe sous la crosse de l'aorte).

36
Exclusivement sur DOC - DZ : www.doc-dz.com NADJI 85
RESIDANAT EN POCHE TOME II
Cas Clinique en QCM
Une malade obèse, âgée de 55 ans, vous appelle pour un ictère apparu le lendemain d'une crise douloureuse de
l'hypochondre droit. La température est à 38,2 degrés. L'examen de l'abdomen est sensiblement normal. Les selles sont
partiellement décolorées. Il n'existe pas de tare associée.

Quel est le premier diagnostic que vous évoquez ?


A - Cancer de la tête du pancréas
B - Cholécystite aiguë
C - Lithiase de la voie biliaire principale
D - Poussée de pancréatite aiguë
E - Hépatite virale choléstatique
Bonne(s) réponse(s) : C

La triade douleur-fièvre-ictère est évocatrice de lithiase de la voie biliaire principale.

Quel est l'examen complémentaire que vous demandez aussitôt ?


A - Scanner abdominal
B - Echographie abdominale
C - Cholangiographie intra-veineuse
D - Cholécystographie orale
E - Scintigraphie hépatique
Bonne(s) réponse(s) : B

L'échographie est le premier examen radiologique à demander en cas de suspicion de pathologie biliaire.

En l'absence de traitement, vous devez redouter la survenue de :


A - Pancréatite aiguë
B - Insuffisance rénale
C - Ictère hémolytique
D - Septicémie à gram négatif
E - Poussées ictériques à rechute
Bonne(s) réponse(s) : A B C D E

L'ictère peut être hémolytique, en particulier lors d'une infection à clostridium perfringens.

Pour traiter une douleur d'origine biliaire, il faut administrer :


A - De l'aspirine
B - Des morphiniques
C - Des antispasmodiques
D - De la Cimétidine
E - Du Glucagon
Bonne(s) réponse(s) : C

Les morphiniques sont contre-indiqués car ils entraînent un spasme du sphincter d'Oddi.

Une femme de 42 ans. ayant comme unique antécédent une hystérectomie pour fibrome à 39 ans, consulte pour une
diarrhée. Celle-ci est faite de 6 à 8 selles/24h, diurnes et nocturnes, liquides ne contenant pas d'éléments anormaux, ni sang,
ni glaires. La diarrhée a débuté 4 mois avant la consultation et la malade a perdu 5 kg. L'examen clinique est normal. Le poids
des selles est de 700 g/24 h avec une stéatorrhée à
13 g/24h. Le tubage gastrique montre un débit acide horaire basal non stimulé de 42 mEq/H. Un syndrome de Zollinger-
Ellison est suspecté.

Quel(s) examen(s) vont vous permettre de confirmer ce diagnostic ?


A - Dosage de la gastrinémie à jeun et post-prandiale
B - Dosage de la sérotonine sanguine à jeun et post-prandiale
C - Dosage de la gastrinémie après perfusion IV de sécrétine
D - Tubage gastrique après stimulation à la pentagastrine
E - Tubage gastrique après stimulation à l'insuline
Bonne(s) réponse(s) : A C

Les tubages gastriques après stimulation (y compris à la pentagastrine) du Zollinges Ellison(ZE). Le test à l'insuline emploie la
stimulation vagale.

37
Exclusivement sur DOC - DZ : www.doc-dz.com NADJI 85
RESIDANAT EN POCHE TOME II
Cas Clinique en QCM

Quel est parmi les mécanismes physiopathologiques suivants, celui qui peut expliquer la stéatorrhée de cette
malade ?
A - Déconjugaison des sels biliaires par pullulation microbienne
B - Atrophie villositaire totale par hypersécrétion gastrique acide
C - Inactivation de la lipase pancréatique par un chyme trop acide
D - Inhibition de la formation des chylomicrons par accélération du transit intestinal
E - Inactivation de la pepsine gastrique
Bonne(s) réponse(s) : C

La composante hydroélectrolytique de la diarrhée du ZE est toutefois prédominante.

Une tumeur pancréatique à l'origine de cette maladie est alors recherchée. Parmi l'(les) examen(s) suivant(s),
quel(s) est (sont) celui (ceux) qui peut (peuvent) localiser cette tumeur ?
A - Tomodensitométrie pancréatique
B - Fibroscopie gastrique
C - Artériographie sélective coelio-mésentérique
D - Transit baryté du grêle
E - Urographie intraveineuse
Bonne(s) réponse(s) : A C

La fibroscopie gastroduodénale (et pas seulement gastrique) est réalisée de toute façon dans le bilan (recherche) d'ulcères
mais seule une éventuelle tumeur pancréatique développée dans la paroi duodénale serait ainsi visualisée. En préopératoire,
la tumeur n'est localisée que dans 30 à 50 %.

Parmi les médicaments suivants, quel(s) est (sont) celui (ceux) sui peut (peuvent) entraîner une sédation de la
diarrhée ?
A - Lopéramide (Imodium)
B - Somatostatine
C - Codéine
D - Cimétidine
E - Ranitidine
Bonne(s) réponse(s) : B D E

Le but du traitement est de faire diminuer la sécrétion acide soit par diminution de la libération de gastrine (somatostatine) soit
par action sur la cellule pariétale (anti-H2).

Un homme de 60 ans a une dysphagie évoluant depuis trois mois. Il s'y associe un amaigrissement de cinq kilos sans
anorexie. Le malade se plaint également d'épigastralgies depuis plusieurs années. L'examen clinique de l'abdomen et
l'examen pleuropulmonaire sont normaux.

L'hypothèse d'un cancer de l'oesophage serait appuyée au plan épidémiologique par la notion de :
A - Antécédents de tuberculose pulmonaire
B - Alcoolisme chronique
C - Tabagisme
D - Contact avec l'amiante
E - Antécédent de cancer ORL
Bonne(s) réponse(s) : B C E

L'association entre cancer de l'oesophage et cancer ORL est statistiquement significative.

Parmi les symptômes suivants, indiquez celui(ceux) qui plaide(nt) en faveur d'un reflux gastro-oesophagien :
A - Pyrosis postural
B - Dysphagie plus marquée pour les liquides
C - Régurgitations acides
D - Toux nocturne récidivante
E - Hypersialorrhée
Bonne(s) réponse(s) : A C D

La dysphagie "paradoxale" évoque plutôt un trouble moteur de l'oesophage.

38
Exclusivement sur DOC - DZ : www.doc-dz.com NADJI 85
RESIDANAT EN POCHE TOME II
Cas Clinique en QCM

Indiquez, parmi les suivantes, la(les) affection(s) qui peuvent entraîner une dysphagie douloureuse
accompagnée de régurgitations :
A - Oesophagite peptique par reflux
B - Dyskinésie de la bouche oesophagienne
C - Cancer de l'oesophage
D - Diverticule épiphrénique de l'oesophage
E - Sténose pylorique
Bonne(s) réponse(s) : A C

Les diverticules épiphréniques sont habituellement asymptomatiques.

Indiquez, parmi les suivants, l'examen à visée diagnostique que vous demandez en priorité lors de la première
consultation de ce patient.
A - Transit baryté oesogastroduodénal
B - Echotomographie abdominale
C - Tomodensitométrie thoraco-abdominale
D - Endoscopie oeso-gastrique
E - Manométrie oesophagienne
Bonne(s) réponse(s) : D

L'endoscopie oesophagienne est indispensable devant toute dysphagie.

Parmi les complications suivantes, indiquez celle que l'on suspecte prioritairement chez ce malade en cas de
reflux gastro-oesophagien :
A - Hémorragie digestive haute
B - Rupture spontanée de l'oesophage
C - Sténose peptique
D - Ulcère de l'oesophage
E - Survenue de diverticule de pulsion de l'oesophage
Bonne(s) réponse(s) : C

Typiquement, la sténose peptique est centrée, régulière, courte et bas située.

Un homme de 65 ans a subi, il y a 15 ans, une gastrectomie sub-totale Finsterer pour ulcère duodénal chronique. Les suites
immédiates ont été simples, ce patient a reçu quelques prescriptions d'hygiène alimentaire (fractionnement des repas..) et n'a
présenté jusqu'alors aucun trouble.
Il vient vous consulter pour des pesanteurs épigastriques, une asthénie, un léger amaigrissement. L'examen clinique ne révèle
rien de bien spécial.
Une formule numération montre une légère anémie normocytaire, normochrome, une V.S à 20-60.

Quel est le diagnostic le plus vraisemblable ?


A - Dumping syndrome
B - Ulcère peptique
C - Cancer sur moignon gastrique
D - Maladie de Biermer
E - Syndrome du Petit Estomac
Bonne(s) réponse(s) : C

La question est mal posée car B est aussi une réponse possible.

Quel examen proposeriez-vous pour affirmer le diagnostic ?


A - Transit oeso-gastro duodénal
B - Echographie abdominale
C - Dosage de l'acidité gastrique
D - Fibroscopie gastrique
E - Myélogramme
Bonne(s) réponse(s) : D

La fibroscopie avec biopsies est l'élément clé de la démarche diagnostique.

39
Exclusivement sur DOC - DZ : www.doc-dz.com NADJI 85
RESIDANAT EN POCHE TOME II
Cas Clinique en QCM

Parmi les examens complémentaires suivants, quel est le plus utile pour la conduite thérapeutique ?
A - Echographie hépatique
B - Dosage des antigènes carcino-embryonnaires
C - Dosage de l'acidité gastrique
D - Manométrie oesophagienne
E - Cholangiographie intraveineuse
Bonne(s) réponse(s) : A

Le bilan d'extension comprend la recherche de métastases hépatiques.

Le traitement proposé sera :


A - Traitement médical par anti acides
B - Remise en circuit du duodénum
C - Dégastrogastrectomie
D - Vagotomie thoracique
E - Aucune de ces propositions n'est valable
Bonne(s) réponse(s) : C

La dégastrogastrectomie peut être un geste techniquement difficile.

Un homme de 60 ans présente depuis un mois des rectorragies de sang rouge, accompagnées de ténesme et de faux
besoins. Son état général est bon, il n'a pas maigri. Le toucher rectal est normal. L'examen de l'abdomen ne montre pas de
masse palpable. Le foie est dans ses limites normales.
Une rectosigmoïdoscopie montre une lésion bourgeonnante, irrégulière,saignant au contact, sur la face antérieure du rectum,
à 12 cm de la marge anale, de 3 cm de diamètre. Des biopsies sont pratiquées au niveau de cette lésion.

Quel est le résultat anatomopathologique le plus probable de cette biopsie ?


A - Polype adénovilleux
B - Adénocarcinome Lieberkühnien
C - Adénocarcinome colloïde muqueux
D - Carcinome malpighien
E - Sarcome
Bonne(s) réponse(s) : B

Cette forme représente environ 80 % des cancers du rectum.

Le bilan diagnostique de ce malade comportera :


A - Une coloscopie totale
B - Un dosage antigènes carcino embryonnaires
C - Une échographie hépatique
D - Une fibroscopie oeso-gastrique
E - Une artériographie mésentérique inférieure
Bonne(s) réponse(s) : A B C

La coloscopie totale recherche un autre cancer et/ou un polype.

Quel traitement envisagez-vous ?


A - Amputation abdomino-périnéale
B - Résection antérieure du rectum
C - Chirurgie limitée par rectotomie
D - Opération de Hartman
E - Radiothérapie de contact
Bonne(s) réponse(s) : B

Il s'agit d'une tumeur du 1/3 supérieur du rectum, l'amputation abdomino-périnéale serait donc inutilement mutilante.

40
Exclusivement sur DOC - DZ : www.doc-dz.com NADJI 85
RESIDANAT EN POCHE TOME II
Cas Clinique en QCM

La surveillance ultérieure de ce patient comportera systématiquement :


A - Coloscopie de contrôle
B - Scanner pelvien
C - Echographie hépatique
D - Dosage de l'Alpha foeto-protéine
E - Dosage de l'antigène carcino-embryonnaire
Bonne(s) réponse(s) : A C E

Le dosage de l'alpha foeto-protéine est demandé dans la surveillance des carcinomes hépatocellualires et des cancers du
testicule.

Il est admis qu'un cancer colo-rectal est hautement induit par l'existence préalable de :
A - Polype juvénile
B - Tumeur villeuse
C - Rectocolite hémorragique
D - Diverticulose colique
E - Polypose familiale diffuse
Bonne(s) réponse(s) : B C E

Le risque de cancérisation est maximal avec la polypose familiale diffuse.

Un homme de 28 ans ressent brutalement des douleurs épigastriques intenses survenues à midi à jeun, et accompagnées de
vomissements. Il est admis à 14 h au service des urgences.
L'interrogatoire relève la notion d'un traitement médical correctement suivi depuis 2 ans et mis en oeuvre pour plusieurs
poussées d'ulcère duodénal.
L'examen note une température à 37°, une contracture abdominale à prédominance épigastrique. Le toucher rectal est
douloureux. Le pouls est à 100/mn, la tension artérielle à 12/8 cmHg.
Le diagnostic porté est celui d'ulcère duodénal perforé.

Quels sont les trois signes cliniques qui vous paraissent les plus caractéristiques de la perforation ?
A - Douleurs épigastriques brutales
B - Vomissements
C - Douleur au toucher rectal
D - Contracture épigastrique
E - Accélération du pouls
Bonne(s) réponse(s) : A C D

Les vomissements sont très peu spécifiques.

Quel examen complémentaire demanderez-vous aussitôt ?


A - Fibroscopie gastro-duodénale
B - Echographie abdominale
C - Radiographie de l'abdomen sans préparation en position debout
D - Ponction-lavage de l'abdomen
E - Transit gastro-duodénal
Bonne(s) réponse(s) : C

L'ASP peut visualiser le pneumopéritoine qui signe la perforation d'un organe creux.

La thérapeutique a mettre en oeuvre immédiatement comporte :


A - Mise en place d'une aspiration gastrique
B - Administration d'opiacés
C - Rééquilibration hydroélectrolytique
D - Irrigations gastriques de solutions anti-acides
E - Ponaion-lavage du péritoine
Bonne(s) réponse(s) : A C

Les irrigations gastriques antiacides sont indiquées en cas d'hémorragie.

41
Exclusivement sur DOC - DZ : www.doc-dz.com NADJI 85
RESIDANAT EN POCHE TOME II
Cas Clinique en QCM

Chez ce malade, vous retiendrez, le cas échéant en faveur d'un traitement non chirurgical (méthode de Taylor) :
A - Malade à jeun lors de la perforation
B - Mise en oeuvre précoce du traitement
C - Malade jeune
D - Existence d'une contracture épigastrique
E - Echec du traitement anti-ulcéreux antérieur
Bonne(s) réponse(s) : A B C

L'existence d'un bon état général (malade jeune) peut être retenue en faveur du Taylor. Ici
cependant, l'échec du traitement antiulcéreux antérieur est un très fort argument en faveur de l'intervention.

Une femme de 62 ans consulte pour des brûlures rétrosternales à point de départ épigastrique, survenant principalement
après les repas, mais réveillant quelquefois la patiente la nuit. Il n'existe pas de dysphagie. On relève dans les antécédents
une hystérectomie pour fibrome à l'âge de 38 ans et une HTA modérée traitée par Catapressan 1 c/jour.
A l'examen clinique, il existe une surcharge pondérale (74 kg pour 1,65m).
L'abdomen est souple. Pas d'hépatosplénomégalie.
Les caractéristiques de la symptomatologie vous permettent de suspecter un reflux gastro-oesophagien.

Parmi les examens paracliniques suivants, un reflux gastro-oesophagien acide peut être mis en évidence et
quantifié par :
A - Oesogastroscopie
B - pH métrie oesophagienne
C - Scintigraphie oesophagienne
D - Transit baryté oeso-gastroduodénal
E - Manométrie oesophagienne-
Bonne(s) réponse(s) : B

La question est mauvaise car le caractère acide ne peut être démontré que par la pHmétrie mais TOGD et scintigraphie
permettent la mise en évidence des reflux sans préjuger de son pH. L'endoscopie ne met pas en évidence le reflux.

A propos du sphincter inférieur de l'oesophage, il est exact que normalement :


A - Il se relâche à la déglutition
B - Il constitue le principal mécanisme anti-reflux
C - Il est constitué par un anneau musculaire palpable
D - Sa pression augmente après injection de gastrine
E - Il subit des influences médicamenteuses
Bonne(s) réponse(s) : A B D E

Les anticholinergiques en particulier diminuent sa pression.

Cochez le(s) examen(s) même minime(s), pouvant permettre de mettre en évidence une oesophagite peptique
chez cette patiente présentant cliniquement une symptomatologie de RGO :
A - Oesophagoscopie avec biopsie
B - pH métrie oesophagienne
C - Manométrie oesophagienne
D - Transit baryté oeso-gastro-duodénal
E - Scintigraphie oesophagienne
Bonne(s) réponse(s) : A D

L'endoscopie est le meilleur examen et doit être demandée d'emblée.

Cochez la(les) complication(s) possible(s) au cours de l'évolution d'une oesophagite peptique par reflux :
A - Ulcère oesophagien
B - Rupture de l'oesophage
C - Hernie hiatale
D - Pleurésie gauche
E - Sténose oesophagienne
Bonne(s) réponse(s) : A E

Ulcère et sténose sont souvent associés. L'ulcère peut se perforer mais il ne s'agit pas véritablement d'une "rupture" de
l'oesophage.

42
Exclusivement sur DOC - DZ : www.doc-dz.com NADJI 85
RESIDANAT EN POCHE TOME II
Cas Clinique en QCM

Un endobrachyoesophage peut venir compliquer un RGO. Cochez la(les) proposition(s) exacte(s) concernant
cette complication :
A - Il est constitué par un raccourcissement congénital de l'oesophage
B - Il favorise l'apparition d'un carcinome épidermoïde au niveau du bas oesophage
C - Le transit baryté oesophagien permet le diagnostic d'endobrachyoesophage
D - La muqueuse du bas oesophage est remplacée par une muqueuse glandulaire de type gastrique
E - Le cardia anatomique et le cardia muqueux ne sont pas situés au même niveau
Bonne(s) réponse(s) : D E

Les cancers développés sur endobrachyoesophage sont des adénocarcinomes.

En cas de RGO acide, quel(s) est(sont) le(s) moyen(s) thérapeutique(s) proposé(s) sur la
symptomatologie ?
A - Prise d'alginates après les repas
B - Cimétidine (Tagamet®) 800 mg/24 h
C - Surélévation de la tête du lit pendant le sommeil
D - Anti-cholinergiques (dérivés atropiniques) aux trois repas
E - Cholestyramine (Questran®), 1 sachet a chaque repas
Bonne(s) réponse(s) : A B C

La question pourrait être indiquée dans un reflux alcalin (rôle des sels biliaires).

Si une intervention chirurgicale devient nécessaire pour corriger le RGO, quelle technique vous parait-elle
appropriée ?
A - Myotomie extra-muqueuse de Heller
B - Intervention de Nissen
C - Gastrectomie des 2/3 avec anastomose gastroduodénale
D - Oesogastrectomie polaire supérieure
E - Vagotomie supra-sélective
Bonne(s) réponse(s) : B

L'intervention de Nissen réalise une fundoplicature (la grosse tubérosité est entourée autour de l'oesophage).

Une femme de 64 ans est admise en urgence à l'hôpital pour douleurs abdomino-pelviennes. La douleur s'est installée
progressivement depuis 24 h,.d'abord sous-ombilicale diffuse, puis prédominant dans la fosse iliaque gauche (F.I.G). En
même temps la malade a présenté des nausées et un vomissement ainsi qu'un arrêt du transit. L'interrogatoire permet de
retrouver une constipation ancienne, avec parfois des épisodes douloureux transitoires de la F.I.G. avec ballonnement. La
température est à 39°5 avec quelques frissons, le pouls à 110/mn, la T.A à 16-8 cmHg. Le poids est de 75 kg pour 150 cm. A
l'examen, on note un météorisme abdominal, un empâtement et une défense de la F.I.G. Le TR. et le T.V. sont normaux. Les
examens complémentaires montrent : Hb =15g/100ml Ht = 48 %, G.B = 19 000 avec 82 % de P.N.. , urée sanguine =10
mmol/l, créatininémie 130 micromol/l, natrémie = 146 mEg/l, kaliémie = 4.2 mEg/l, protides 73 g/l.

Parmi les examens à demander en urgence, quels sont les deux qui vous paraissent plus particulièrement
intéressants ?
A - Transaminases
B - Radiographie de l'abdomen sans préparation debout, de face
C - Hémocultures
D - Gazométrie
E - Amylasémie
Bonne(s) réponse(s) : B C

Ces 2 examens s'imposent puisqu'il existe un syndrome infectieux et un syndrome subocclusif.

Parmi les mesures suivantes, il est contre-indiqué de réaliser en urgence :


A - Administration d'un laxatif
B - Mise en place d'une sonde naso-gastrique
C - Lavement évacuateur
D - Injection intra-musculaire d'une ampoule de Dolosal
E - Diète alimentaire complète
Bonne(s) réponse(s) : A C D

Le Dolosal® est contre-indiqué initialement car il risque de majorer le syndrome occlusif (favorise la constipation).

43
Exclusivement sur DOC - DZ : www.doc-dz.com NADJI 85
RESIDANAT EN POCHE TOME II
Cas Clinique en QCM

Le traitement initial fait appel à :


A - Antibiothérapie à large spectre par voie parentérale
B - Glace sur le ventre (F I G )
C - Mise en route d'une réhydratation par voie I.V
D - Mise en place d'une sonde rectale
E - Ponction-lavage du péritoine
Bonne(s) réponse(s) : A B C

Cette question aurait du être placée en dernier car le traitement est bien sûr adapté au
diagnostic retenu.

Après 48 heures de traitement et amélioration des signes locaux et généraux, quel examen complémentaire
vous parait le plus utile au diagnostic ?
A - Cystoscopie
B - Lavement opaque aux hydrosolubles
C - Scanner abdominal
D - Artériographie coelio-mésentérique
E - Laparoscopie
Bonne(s) réponse(s) : B

Le lavement baryté est contre-indiqué en raison du risque de perforation.

Parmi les diagnostics suivants, lequel retenez-vous en priorité ?


A - Pyélonéphrite
B - Infarctus du mésentère
C - Sigmoïdite aiguë diverticulaire
D - Cancer du sigmoïde
E - Typhoïde
Bonne(s) réponse(s) : C

Un cancer du sigmoïde compliqué est également un diagnostic possible.

Un alcoolique chronique est découvert dans le coma.


On note :
- des varicosités des pommettes
- une hypertrophie parotidienne bilatérale
- un ictère franc
- un érythrose palmaire
- des angiomes stellaires
- une circulation veineuse collatérale abdominale.
Il existe une hépatomégalie dure, une ascite, des oedèmes des membres inférieurs. Il n'y a pas de défense abdominale.
La température est à 37°5, le pouls à 100, la tension artérielle à 14-9.
Le coma est calme, profond, sans signe de localisation.
La biologie montre :
- une bilirubinémie conjuguée à 60 micro moles/I
- ASAT (SGOT) à 30 UI/litre (normale inférieure à 20), ALAT (SGPT) à 15 (normale inférieure à 20);
- taux de prothrombine : 40 %
- gamma GT : 60 UI/litre
- urémie : 7 millimoles par litre
- glycémie : 4,5 millimoles par litre
- créatininémie : 155 micromoles par litre
- 13,5 g/100 ml d'hémoglobine, VGM 102 mm3, 3 200 G.B, 120000 plaquettes.
La radiographie de l'abdomen sans préparation : opacité de l'hypochondre gauche abaissant le rein gauche.
Echographie : volumineux calculs vésiculaires, paroi vésiculaire épaissie, voie biliaire principale non vue.
Ponction d'ascite : liquide contenant 40 g par litre de protides, 3000 polynucléaires par mm3, sans germe à l'examen direct.

Parmi les éléments cliniques suivants, quel est celui (quels sont ceux) qui sont très évocateur(s) de cirrhose ?
A - Hypertrophie parotidienne bilatérale
B - Hépatomégalie dure
C - Erythrose palmaire
D - Angiomes stellaires
E - Aucun des éléments précédents
Bonne(s) réponse(s) : B C D

L'érythrose palmaire et les angiomes stellaires sont des signes d'insuffisance hépato-cellulaire.

44
Exclusivement sur DOC - DZ : www.doc-dz.com NADJI 85
RESIDANAT EN POCHE TOME II
Cas Clinique en QCM

Parmi les éléments biologiques suivants, quel(s) est(sont) celui(ceux) évocateur(s) d'insuffisance
hépatocellulaire ?
A - Taux sérique de gamma GT élevé
B - Urémie élevée
C - Chute du taux de prothrombine à 40 %
D - Rapport ASAT (SGOT) sur ALAT (SGPT) élevé
E - Macrocytose
Bonne(s) réponse(s) : C

Le rapport ASAT/ALAT élevé est en faveur de l'étiologie alcoolique de l'hépatopathie.

On peut dire que cette cirrhose alcoolique est compliquée d'hypertension portale car :
A - Il existe une circulation veineuse collatérale abdominale
B - Il existe une splénomégalie
C - Il existe une ascite
D - Il existe des angiomes stellaires
E - Le taux de prothrombine est à 40 %
Bonne(s) réponse(s) : A B C

L'ascite est habituellement un signe d'hypertension portale et d'insuffisance hépatocellulaire.

Quel est le facteur déclenchant le plus probable de ce coma ?


A - Une hépatite alcoolique aiguë surajoutée à la cirrhose
B - Une infection spontanée du liquide d'ascite
C - Une cholécystite aiguë
D - Un hématome intra-cérébral
E - Une hémorragie digestive
Bonne(s) réponse(s) : B

La présence de 3000 polynucléaires/mm3 dans le liquide d'ascite doit faire porter le diagnostic d'infection d'ascite même en
l'absence de germes au début.

Une jeune femme âgée de 26 ans est hospitalisée pour une diarrhée faite d'une dizaine de selles; sans glaire ni sang, ni
douleurs abdominales diurnes ou nocturnes. Cette diarrhée est apparue il y a six mois. Parallèlement, il y a un
amaigrissement de 6 kg malgré une boulimie. Le poids est de 45 kg pour 1,65m. Le pouls est à 80/mn, la tension artérielle à
120/80 mmHg, il y a une dyspnée d'effort sans orthopnée et de discrets oedèmes des membres inférieurs. L'abdomen est
météorisé, indolore sans hépato ou splénomégalie. On note enfin une pâleur importante et une amyotrophie, ainsi que des
douleurs fugaces au niveau des poignets et des mains, des douleurs vertébrales et une éruption cutanée eczématiforme au
niveau du visage. Les examens biologiques réalisés montrent :
‫ ر‬ionogramme sanguin : Na : 138 micromol/l ; K : 2,8 micromol/l
‫ ر‬urée : 1,3 mM, calcémie : 1,86 mM
‫ ر‬numération formule sanguine : GR : 3,14 millions, GB 4.200 dont 69 % de polynucléaires neutrophiles, hémoglobine : 101
g/l, V.G.M : 101 micro3
- taux de prothrombine à 38 %
‫ ر‬électrophorèse des protides sanguins : protides totaux 50 g/l dont 51 % d'albumine, et un dosage pondéral normal des
immunoglobulines,
‫ ر‬les folates sanguins sont à 0,42 microg/l (Nle 5 microg/l), la vitamine B 12 à 168 ng/l (Nle 300 ng/l), le fer sérique à 31
gamma %
Enfin, on trouve 17 g/24 h de graisse dans les selles, dont le poids quotidien est de 1.800 g/24 h.
Le transit baryté de l'intestin grêle montre une importante dilatation de l'ensemble des anses grêles, avec une hypotonie des
anses, des plis épaissis et un élargissement de l'espace interplis. Une biopsie de l'intestin grêle a donc été effectuée montrant
une atrophie villositaire totale, sans autre anomalie.

Le syndrome clinique et biologique réalisé est celui :


A - D'une anorexie mentale
B - D'un syndrome de carence par abus de laxatifs
C - D'un syndrome de malabsorption dont on ne peut préciser l'origine pré-entérocytaire ou entérocytaire
D - D'un syndrome de malabsorption dû à une maladie coeliaque
E - D'une insuffisance cardiaque
Bonne(s) réponse(s) : D

La maladie coeliaque est définie par :


1) un syndrome de malabsorption
2) une lésion histologique caractéristique, mais non spécifique de l'intestin grêle proximale : l'atrophie villositaire totale ou
subtotale
3) une amélioration clinique et histologique sous régime sans gluten.

45
Exclusivement sur DOC - DZ : www.doc-dz.com NADJI 85
RESIDANAT EN POCHE TOME II
Cas Clinique en QCM

Les explorations biologiques chez cette malade :


A - Indiquent une carence mais pas forcément une malabsorption
B - Indiquent une malabsorption mais pas de carence
C - Indiquent une carence et une malabsorption
D - Devraient comprendre un test au xylose
E - Indiquent un taux de folate bas, ce qui est normal puisque la vitamine B12 l'est aussi
Bonne(s) réponse(s) : A D

L'ensemble de ces résultats est évocateur d'une malabsorption ; néanmoins, pour mettre directement en évidence celle-ci, il
conviendrait de réaliser un test au D-xylose (grêle proximal) et un test de Schilling avec facteur intrinsèque (absorption de la
vitamine B12 dans l'iléon).

La survenue de douleurs ostéo-articulaires :


A - Témoigne de l'origine infectieuse de cette malabsorption
B - Doit faire prescrire un traitement antibiotique pour éliminer une maladie de Whipple
C - N'a aucun rapport avec la maladie du tube digestif
D - Peut être mise en relation avec une ostéomalacie qui reste à prouver
E - Est tout à fait normale puisqu'il y a une hypocalcémie
Bonne(s) réponse(s) : D

La carence en vitamine D et calcium explique les douleurs osseuses. Le tableau biologique le plus habituel est celui de
l'ostéomalacie : hypocalcémie, hypophosphorémie, hypocalciurie, augmentation des phosphatases alcalines.

Le traitement électif chez cette malade est :


A - Un régime pauvre en gluten
B - Un traitement corticoïde initial
C - Un régime sans gluten strict
D - Un traitement antibiotique
E - Une exérèse segmentaire du grêle malade
Bonne(s) réponse(s) : C

Un régime sans gluten strict amène habituellement une amélioration clinique spectaculaire en quelques semaines.
La normalisation du test au D-xylose demande plusieurs mois. La réparation histologique est en règle générale, confirmée au
bout d'un an.

Sous traitement approprié, l'évolution est :


A - Nécessairement favorable
B - Parfois marquée par une résistance au traitement
C - Parfois marquée par la survenue de lymphomes
D - Marquée par une normalisation rapide de l'histologie mais moins de l'état clinique
E - Une normalisation rapide des signes cliniques mais plus lente de l'histologie
Bonne(s) réponse(s) : B C E

En cas de résistance (rare) au régime sans gluten, il faut rechercher avant tout des erreurs diététiques.
Un lymphome ou des ulcérations intestinales constituent des causes classiques de résistance clinique au régime sans gluten
au cours de la maladie coeliaque.
Enfin, il existe de rares patients intolérants non seulement au gluten mais à d'autres protéines, des oeufs ou du poisson.

Un homme de 58 ans, a une cirrhose éthylique prouvée histologiquement, avec hypertension portale. Un premier épisode
d'ascite a été traité médicalement lors d'une hospitalisation de 3 semaines. Une nouvelle ascite apparaît six mois plus tard
alors que le malade ne boit plus de boissons alcoolisées et ne prend aucun médicament.
Cette nouvelle ascite conduit à une nouvelle hospitalisation. On note alors cliniquement : une ascite de moyenne abondance
libre, sans douleurs abdominales, associée à une circulation veineuse collatérale abdominale, une dizaine d'angiomes
stellaires. Il n'y a ni fièvre, ni tachycardie, ni ictère, ni pâleur, ni dyspnée, ni signe d'encéphalopathie hépatique.
Biologiquement : NFS normale, Na+ 137 mmol/l, K +3,7 mmol/l, glycémie 5 mmol/l, SGOT 40 UI/l (N:45), SGPT 32 UI/l (N:45),
bilirubine 17 micromol/l, temps de quick 73% albumine 32g/l, gammaglobulines 18g/l, créatinémie 90 micromol/l.

Avant même de débuter un éventuel traitement diurétique, vous proposez pour traiter cette ascite :
A - Evacuation totale par ponction
B - Repos strict au lit
C - Lactulose (Duphalac®) 60 ml/jour
D - Expansion volémique (plasma frais congelé, albumine)
E - Régime sans sel strict
Bonne(s) réponse(s) : B E

Le rpose au lit et le régime sans sel sont le traitement de base d'une poussée ascitique.
Il est parfois suffisant, notamment lors d'une première décompensation ascitique.

46
Exclusivement sur DOC - DZ : www.doc-dz.com NADJI 85
RESIDANAT EN POCHE TOME II
Cas Clinique en QCM

Quelques jours plus tard, on décide d'y adjoindre un traitement diurétique par un diurétique anti-aldéostérone
(Aldactone® 2 comprimés/jour). Vous allez apprécier l'efficacité de ce traitement sur l'évolution de :
A - Courbe de poids
B - Courbe de diurèse
C - Ionogramme urinaire
D - Ionogramme sanguin
E - Périmètre abdominal
Bonne(s) réponse(s) : A B C E

L'efficacité de ce traitement sera jugé sur l'obtention d'une bonne natriurèse, précédant la diminution du poirds et du périmètre
ombilical.

Comment apprécier la tolérance à ce traitement diurétique ?


A - Ionogramme sanguin
B - Ionogramme urinaire
C - Créatininémie
D - Créatininurie des 24 heures
E - Protidémie
Bonne(s) réponse(s) : A C

Ce traitement diurétique doit être arrêté si la natrémie est inférieure à 130 mEq/l ou s'il apparaît une insuffisance rénale.
Le ionogramme sanguin doit être surveillé de façon rapprochée au début du traitement diurétique chez un patient cirrhotique
décompensé.

Après 15 jours de ce traitement, l'ascite a disparu. L'ordonnance de sortie comportera dès lors :
A - Aldactone®, 2 comprimés par jour
B - Régime sans sel strict
C - Repos au lit
D - Moduretic® (hydrochlorothiazide + amiloride), 1 comprimé par jour
E - Régime riche en protides
Bonne(s) réponse(s) : B D

Le régime sans sel doit être bien expliqué au patient : il est très important.
Le traitement diurétique d'entretien peut très bien comporter un diurétique anti-aldostérone (Aldactone®) à doses réduites.

Le malade ne suit pas correctement le traitement prescrit. Un mois plus tard, l'ascite réapparaît entraînant une
nouvelle hospitalisation. Le ou les schémas thérapeutiques applicables dès lors est ou sont :
A - Moduretic® 2 comprimés par jour et régime désodé
B - Poncition évacuatrice avec concentration et réinjection de l'ascite
C - Shunt péritonéo-jugulaire (valve de Le Veen)
D - Le même schéma que pour l'hospitalisation précédente
E - Traitement bêta-bloquant (propranolol, Avlocardyl® 1 comprimé par jour)
Bonne(s) réponse(s) : D

Il ne faut pas confondre ascite récidivante du fait d'une erreur thérapeutique (arrêt du régime sans sel ou des diurétiques) et
ascite réfractaire, rebelle au traitement médical bien conduit, qui pourrait nécessiter d'autres thérapeutiques (ponctions
itératives, ponction-concentration-réinjection, valve de Le Veen)

47
Exclusivement sur DOC - DZ : www.doc-dz.com NADJI 85
RESIDANAT EN POCHE TOME II
Cas Clinique en QCM
Un homme de 55 ans, éthylique chronique, présente brutalement à 16 heures, un vomissement de sang rouge mêlé de débris
alimentaires. Cet épisode se répète à 16h15 et à 17h. Le médecin, appelé d'urgence, constate une pâleur des téguments et
des muqueuses, des sueurs, une tachycardie à 120. La TA est à 90/60 mmHg. A l'arrivée à l'hôpital, l'interne de garde
constate une hépatomégalie ferme, non douloureuse, débordant le rebord costal de 5 cm, une circulation collatérale
abdominale. Il n'y a pas de matité dans les flancs ni de splénomégalie. A l'interrogatoire, on ne retrouve aucun antécédent
digestif. Le malade est placé d'urgence en réanimation chirurgicale. Vers 21h l'hémorragie digestive se reproduit, le malade
émet un méléna. Les premiers examens de laboratoire montrent un hématocrite à 24 %, une hémoglobine à 7 g pour 100, un
taux de prothrombine à 35 %.

Indiquez le diagnostic étiologique le plus vraisemblable de cette hémorragie digestive d'après les données de
l'anamnèse, de l'interrogatoire et de l'examen :
A - Cancer gastrique
B - Hypertension portale
C - Ulcère gastroduodénal
D - Gastrite hémorragique
E - Syndrome de Mallory-Weiss
Bonne(s) réponse(s) : B

Chez ce patient éthylique chronique, une cirrhose est probable : hépatomégalie ferme, circulation veineuse collatérale. La
cause principale d'hématémèse chez le cirrhotique est la rupture de varices oesophagiennes. Il n'en demeure pas moins
qu'une hémorragie d'un ulcère gastroduodénal est possible ; seule une endoscopie haute permettra d'établir avec certitude
l'origine de l'hémorragie.

Parmi les attitudes suivantes, indiquez celle qui vous paraît la plus importante et la plus urgente à mettre en
oeuvre :
A - Injection intraveineuse de vasoconstricteurs
B - Mise en décubitus tête basse
C - La mise en place d'une sonde gastrique
D - La mise en place d'une transfusion sanguine
E - L'injection de facteurs de coagulation en particulier de PPSB
Bonne(s) réponse(s) : D

La mise en place d'une bonne voie d'abord veineux est le geste le plus urgent chez un patient présentant une hémorragie
digestive. La transfusion s'impose d'emblée chez ce patient récidivant une hémorragie, avec une hémoglobine à 7 g.

Parmi les examens complémentaires suivants, indiquez celui que vous demandez en premier lieu pour affirmer
le diagnostic en urgence :
A - Echotomographie hépatique
B - Transit gastroduodénal
C - Fibroscopie oesogastroduodénale
D - Artériographie coeliomésentérique
E - Spléno-portographie
Bonne(s) réponse(s) : C

La fibroscopie oesogastroduodénale doit être réalisée pour établir l'origine de l'hématémèse ; il faut la pratiquer au mieux sur
un estomac préparé, lavé à l'eau glacée à l'aide d'une sonde gastrique.

Parmi les attitudes thérapeutiques suivantes, indiquez celle(s) qui peu(ven)t


permettre de prévenir une récidive hémorragique :
A - Gastrectomie subtotale
B - Sclérose des varices oesophagiennes
C - Vagotomie hypersélective
D - Dérivation porto-cave tronculaire
E - Dérivation spléno rénale distale
Bonne(s) réponse(s) : B E

Le traitement par sclérose des varices oesophagiennes est en général entrepris de prime abord.
Il a une efficacité rapide pour une faible morbidité.

48
Exclusivement sur DOC - DZ : www.doc-dz.com NADJI 85
RESIDANAT EN POCHE TOME II
Cas Clinique en QCM

Quelle(s) complication(s) peut-on redouter chez ce malade ?


A - La récidive de l'hémorragie
B - La perforation d'un ulcère duodénal
C - L'encéphalopathie hépatique
D - La sténose oesophagienne
E - L'apparition d'une ascite
Bonne(s) réponse(s) : A C

L'encéphalopathie hépatique doit être prévenue dans la mesure du possible par du lactulose ou de la néomycine, introduits de
façon systématique.

Une femme de 32 ans vient vous consulter en raison d'un problème de consommation excessive d'alcool avec dépendance.
Elle effectue cette démarche à la suite d'importantes difficultés professionnelles (menace de licenciement de la mairie où elle
travaille comme employée municipale) et de la pression de son milieu familial. Il n'y a pas d'antécédents particuliers. Cette
patiente est mariée et mère de deux enfants. A l'examen clinique, vous trouvez une hépatomégalie isolée.

Concernant l'alcoolisation chez la femme, il est exact que :


A - La consommation de vin est proportionnellement plus forte chez la femme alcoolique que chez l'homme
alcoolique
B - L'organisme de la femme est plus vulnérable que celui de l'homme vis à vis de l'alcool
C - Les facteurs psychonévrotiques sont moins importants que les facteurs socioculturels dans l'alcoolisme
féminin
D - Le syndrome d'alcoolisme foetal comporte la présence chez le nouveau-né d'anomalies morphologiques
crânio-faciales
E - La croissance de l'enfant né de mère alcoolique est habituellement retardée
Bonne(s) réponse(s) : B D

Une cirrhose peut être réalisée chez la femme avec des prises alcooliques de l'ordre de 20 g/jour durant plusieurs années.
Le syndrome d'alcoolisme foetal est d'autant plus marqué que la consommation alcoolique est importante est qu'elle a lieu
durant le 1er trimestre de la grossesse.

Lors d'un bilan effectué récemment à la médecine du travail, il a été constaté chez cette patiente une élévation
de la gamma-glutamyl-transpeptidase à 117 Ul/l. La gamma G.T. :
A - Est une enzyme exclusivement hépatique
B - Peut être normale chez le consommateur excessif d'alcool
C - Peut être élevée au cours des cirrhoses
D - Peut être majorée par la prise de phénobarbital
E - Est un test utile de servage en boissons alcoolisées
Bonne(s) réponse(s) : B C D E

La gamma G.T. a une sensibilité de 75% dans le dépistage de l'alcoolisme chronique. Elle peut s'élever après 2 à 3 semaines
d'intoxication éthylique et diminuer 10 jours environ à l'arrêt de cette intoxication.

Cette patiente accepte une hospitalisation pour cure de déshabituation. Après le début de son sevrage en
alcool, peuvent apparaître des manifestations cliniques en rapport avec l'arrêt de l'alcool. Une dépendance à
l'alcool peut se traduire par :
A - Tremblements des extrémités
B - Sueurs abondantes
C - Irritabilité
D - Hypothermie
E - Sensation de malaise général
Bonne(s) réponse(s) : A C

Les manifestations de sevrage témoignent de la dépendance à l'alcool ; elles peuvent réaliser un delirium tremens à prévenir
par une bonne hydratation associée à des tranquillisants et une vitaminothérapie parentérale B1.

49
Exclusivement sur DOC - DZ : www.doc-dz.com NADJI 85
RESIDANAT EN POCHE TOME II
Cas Clinique en QCM

Dans le programme thérapeutique proposé à cette patiente, un traitement par disulfirame (Espéral®) à la dose
de un comprimé par jour est entrepris. Il est exact que :
A - La réaction disulfirame-alcool s'accompagne habituellement de réactions vasomotrices importantes
B - Elle peut provoquer des troubles digestifs (nausées, vomissements)
C - Ce type de réaction disulfirame-alcool peut survenir avec d'autres médicaments que le disulfirame
D - La réaction disulfirame-alcool peut entraîner des complications cardio-vasculaires graves
E - La prise prolongée de disulfirame peut entraîner des complications neurologiques
Bonne(s) réponse(s) : A B C D E

Le disulfirame est une inhibiteur de l'acétaldéhyde déshydrogénase ; l'ingestion d'alcool durant un traitement par disulfirame
donne lieu à des manifestations liées à la toxicité de l'acétaldéhyde, produit de l'oxydation de l'éthanol.
Ces manifestations comportent : nausées, vomissements, flush vasomoteur, hypotension artérielle ; d'où sa contre-indication
chez le patient de plus de 60 ans, chez l'insuffisant cardiaque. Il est également contre-indiqué en cas de troubles
neurologiques.

Un malade de 75 ans emphysémateux est hospitalisé pour un ictère cutanéomuqueux qui a débuté il y a 8 jours.
L'interrogatoire apprend :
- que cet homme se plaint d'un prurit depuis un mois
- que l'ictère a été précédé de douleurs de l'hypochondre droit qui se sont estompées depuis et qui irradient vers l'arrière en
hémiceinture droite
- que l'ictère a été suivi d'une élévation thermique à 37°5
- que ce patient présente des épisodes diarrhéiques depuis 6 semaines
- que s'y associent des pesanteurs et une sensation de difficulté d'évacuation gastrique.
L'examen met en évidence :
- un foie augmenté de volume, à bord mousse, de surface irrégulière, de consistance normale, sous lequel on palpe une
grosse vésicule
- un ictère intense
- une splénomégaIie modérée.
Les examens biologiques donnent les résultats suivants :
bilirubine totale 180 micromol/l (10 fois la normale), phosphatases alcalines 418 UI (4 fois les valeurs normales), gamma GT
168 Ul (entre 3 et 4 fois les valeurs normales), SGPT 76 (une fois et demi la valeur normale), SGOT 84 U (2 fois la normale),
TP 65 %, albuminémie 27g/l, cholestérolémie 8,67 mmol/l (3,30 g/l) marqueurs viraux de l'hépatite = O.Hb 10,7 g/100ml, VGM
76, leucocytes 9700 (78 % de polynucléaires neutrophiles).

Parmi les 5 propositions suivantes, quelle est celle que vous retenez ? Le prurit :
A - Est dû à l'élévation des phosphatases alcalines
B - Est dû à la présence des pigments biliaires
C - Est dû a l'accumulation sous-cutanée des acides biliaires
D - Est un signe d'hypercholesterolémie
E - Est proportionnel à l'élévation des transaminases
Bonne(s) réponse(s) : C

Il s'agit d'un prurit lié à la cholestase.

Les douleurs de l'hypochondre droit que décrit le malade évoquent :


A - Une douleur radiculaire
B - Une pleurodynie
C - Une douleur biliaire
D - Un syndrome douloureux solaire
E - Un syndrome ulcéreux
Bonne(s) réponse(s) : C

Les douleurs de l'hypochondre droit irradiant en hémiceinture droite sont le plus souvent biliaires ou pancréatiques.

L'examen complémentaire à demander en premier est :


A - Un transit oesogastroduodénal
B - Une artériographie coelio-mésentérique
C - Une échotomographie du foie et des voies biliaires
D - Une duodénoscopie
E - Une cholangiographie intraveineuse
Bonne(s) réponse(s) : C

La cholestase clinique et biologique existant chez ce patient incitent à explorer avant tout les voies biliaires, le foie et le
pancréas, et donc à demander une échographie abdominale.

50
Exclusivement sur DOC - DZ : www.doc-dz.com NADJI 85
RESIDANAT EN POCHE TOME II
Cas Clinique en QCM

Le(s) diagnostic(s) qui parai(ssen)t le(s) plus probable(s) est(sont) :


A - Hépatite virale A non B
B - Ampullome vatérien
C - Cancer de la tête du pancréas
D - Cancer secondaire du foie
E - Hépatite alcoolique
Bonne(s) réponse(s) : C

Le prurit précédant l'ictère


- la grosse vésicule palpable
- le foie de surface irrégulière (métastases?)
évoquent au premier chef le diagnostic de cancer de la tête du pancréas.

Une femme âgée de 45 ans, en bonne santé apparente vient consulter pour l'apparition récente de rectorragies de sang rouge
émises après les selles, éclaboussant la cuvette. Elle n'a aucune douleur anale. Elle a par ailleurs une constipation de très
longue date alternant avec de la diarrhée et associée à de fréquentes douleurs abdominales et à des ballonnements. Elle
consomme régulièrement des laxatifs qui déclenchent souvent les douleurs et la diarrhée. Elle apporte un lavement baryté
datant de 3 ans où l'on ne voit aucune anomalie dans le côlon plein de baryte (technique standard) en dehors de quelques
diverticules du sigmoïde. L'abdomen est normal à l'examen physique. A l'examen proctologique on observe des hémorroïdes
non procidentes et une anite rouge. La rectoscopie met en évidence un polype de 5 mm situé à 6 cm de la marge anale. Le
reste de la muqueuse est normal jusqu'à 16 cm de la marge anale.

L'émission de sang rouge par l'anus peut être dû chez cette patiente à :
A - L'anite hémorroïdaire
B - Une autre tumeur rectale
C - La diverticulose sigmoïdienne
D - Une rectocolite hémorragique
E - Une iléite terminale
Bonne(s) réponse(s) : A

C'est la cause la plus fréquente des rectorragies ; qui ne doit pas faire méconnaître une cause sus-jacente, en particulier
tumorale.

A cause des rectorragies, vous prescrivez à cette patiente, une fois connue le résultat de la rectoscopie :
A - Une artériographie de la mésentérique inférieure
B - Une consultation auprès d'un chirurgien
C - Une coloscopie totale
D - Une radiographie barytée de l'intestin grêle
E - Une recherche de parasites dans les selles
Bonne(s) réponse(s) : C

La coloscopie est impérative, à la recherche de polypes ou de tumeurs coliques.

Le diagnostic de trouble fonctionnel intestinal est évoqué dans ce cas en raison de :


A - Existence de douleurs abdominales
B - Existence de troubles du transit
C - Présence de diverticules sigmoïdiens
D - Présence d'hémorroïdes
E - Présence d'un polype rectal
Bonne(s) réponse(s) : A B

L'ancienneté de la constipation et des douleurs abdominales sont en faveur d'un trouble fonctionnel, mais ne permettent pas
d'éliminer une tumeur associée, responsable des rectorragies.

Les symptômes de troubles fonctionnels intestinaux devraient être améliorés chez cette patiente par :
A - L'exérèse des diverticules du côlon sigmoïde
B - L'exérèse du polype rectal
C - Une normalisation du transit intestinal
D - Des antispasmodiques musculotropes
E - Le traitement chirurgical des hémorroïdes
Bonne(s) réponse(s) : C D

Une régularisation du transit par des fibres alimentaires ou des mucilages est utile, et pourrait permettre l'arrêt des laxatifs
dont on connaît les risques à long terme. Des antispasmodiques sont utiles pour combattre les phénomènes douloureux.

51
Exclusivement sur DOC - DZ : www.doc-dz.com NADJI 85
RESIDANAT EN POCHE TOME II
Cas Clinique en QCM

Le diagnostic de cancer colique est exclu chez cette patiente à cause de :


A - La conservation d'un bon état général
B - Le lavement baryté normal qu'elle a apporté
C - La rectoscopie normale
D - L'examen physique normal
E - Aucune des propositions précédentes
Bonne(s) réponse(s) : E

Seule une coloscopie totale normale (ou un lavement baryté en double contraste de bonne qualité) permet d'éliminer le
diagnostic de cancer colique.

Monsieur X..., 56 ans, éthylique chronique (160 g d'alcool par jour) est hospitalisé pour une poussée d'ascite et d'oedèmes
des membres inférieurs associés à une diarrhée aiguë survenue il y a quelques jours constituée de 6 à 8 selles fécales
quotidiennes sans émissions anormales, et accompagnées de douleurs abdominales diffuses. L'examen clinique met en
évidence, outre le syndrome oedémato-ascitique, une circulation collatérale, une hépatomégalie avec signe du glaçon, et un
astérixis. La température est normale, le pouls à 80/mn, la tension artérielle à 13/8, la diurèse est à 400 ml/24 heures. Vous
êtes en possession des examens biologiques suivants : hémogramme, ionogramme et créatinine plasmatique sont normaux,
phosphatases alcalines 650 nK, transaminases SGOT 350 nK, SGPT 280 nK, taux de prothrombine 48 %, bilirubine totale.70
mmol/l (bilirubine directe 35 mmol/l). Le liquide d'ascite est citrin : l'examen cytologique du liquide de ponction met en
évidence 800 éléments blancs par mm3 dont 70 % de polynucléaires, et ne retient aucune cellule maligne. La culture est en
cours. Les valeurs normales sont :
S.G.O.T. : de 60 à 430 nK
S.G.P.T. : de 110 à 450 nK
phosphatases alcalines : de 250 à 500 nK.

Quel diagnostic pouvez-vous évoquer ?


A - Une décompensation oedémato-ascitique d'une cirrhose
B - Une décompensation oedémato-ascitique avec infection spontanée du liquide d'ascite
C - Une poussée d'hépatite alcoolique sur cirrhose
D - Une tuberculose péritonéale
E - Un syndrome hépato-rénal
Bonne(s) réponse(s) : B

Un taux de polynucléaires supérieur à 250/mm3 dans l'ascite doit faire considérer cette ascite comme étant infectée. Un seuil
de sensibilité plus grand est obtenu dans le diagnostic d'infection du liquide d'ascite si l'on utilise 75/mm3 comme valeur limite
supérieure du nombre de polynucléaires.

Une exploration complémentaire est à réaliser avant la mise en route du traitement, laquelle ?
A - Une échographie abdominale
B - Une série d'hémocultures
C - Une coloscopie
D - Une mise en culture de l'ascite sur milieu de Loewenstein
E - Une ponction biopsie de foie
Bonne(s) réponse(s) : B

Des hémocultures sont justifiées même en l'absence de fièvre, ce qui est fréquent. Une coproculture est également justifiée.

Le traitement à instituer doit comporter une ou plusieurs des thérapeutiques suivantes :


A - Une antibiothérapie par voie générale
B - Une ponction réinjection de liquide d'ascite
C - Un traitement diurétique
D - Des perfusions d'acide glutamique
E - Des corticoïdes
Bonne(s) réponse(s) : A

Le traitement diurétique n'est pas impératif ; il peut être inutile notamment en cas de 1ère poussée ascitique.

L'astérixis est :
A - Un signe d'encéphalopathie hépatique
B - Un signe d'hypertension portale
C - Un signe rencontré dans les carences en vitamines
D - Un signe d'intoxication alcoolique chronique
E - Un symptôme du syndrome de Korsakoff
Bonne(s) réponse(s) : A

L'astérixis n'est pas spécifique de l'encéphalopathie hépatique ; il peut être observé au cours des encéphalopathies
hypoxiques, toxiques, et l'insuffisance rénale notamment.

52
Exclusivement sur DOC - DZ : www.doc-dz.com NADJI 85
RESIDANAT EN POCHE TOME II
Cas Clinique en QCM
Un malade de 52 ans est ré-hospitalisé pour la 3ème décompensation d'une cirrhose. On note dans ses antécédent 5 ans
auparavant, un accident de la voie publique, ayant nécessité une laparotomie. A l'examen l'état général est altéré avec une
fonte des masses maigres, l'abdomen est tendu, mat à la percussion.

Parmi ces éléments biologiques, quel est ou quels sont ceux qui témoignent d'une insuffisance hépatocellulaire
?
A - Hypoalbuminémie
B - Baisse du TP (temps de Quick)
C - Augmentation des phosphatases alcalines
D - Diminution du facteur V
E - Hyponatrémie
Bonne(s) réponse(s) : A B D

L'insuffisance hépatocellulaire se traduit par une diminution des synthèses protéiques hépatiques et notamment des facteurs
de la coagulation et de l'albumine.

Parmi ces éléments, quel est ou quels sont ceux qui doivent faire évoquer un cancer du foie sur cirrhose ?
A - Altération de l'état général
B - Ascite sanglante se reproduisant rapidement
C - Existence d'une thrombose porte
D - Foie siège d'un souffle systolique
E - Fièvre en plateau
Bonne(s) réponse(s) : A B C D

En fait, les manifestations d'un carcinome hépatocellulaire greffé sur cirrhose peuvent être très réduites ou absentes : dans de
tels cas, le diagnostic repose sur 2 examens effectués systématiquement chez tout patient porteur d'une cirrhose :
l'échographie et le dosage de l'alphafoetoprotéine.

Le malade ayant été transfusé lors de sa laparotomie, quel(s) marqueur(s) demandez-vous pour savoir s'il est
porteur d'une hépatite chronique ?
A - Anticorps anti-HA de type IgM
B - Cytomégalovirus
C - Ag HBs
D - Ag HBe
E - Ac anti HBs
Bonne(s) réponse(s) :

QUESTION ANNULEE.

La transfusion sanguine peut transmettre :


A - Le virus du SIDA
B - L'agent Delta
C - Le virus B
D - Le virus non A non B
E - Le virus A
Bonne(s) réponse(s) :

QUESTION ANNULEE.

En cours d'hospitalisation le malade fait une hématémèse. Celle-ci est suivie de coma avec astérixis, rigidité
extra-pyramidale, foetor hepaticus, respiration calme sans pause. En dehors de l'hémorragie digestive quelle
est ou quelles sont les autres causes déclenchantes d'une telle encéphalopathie chez un cirrhotique ?
A - Insuffisance rénale
B - Hépatite alcoolique aiguë
C - Réime hypoprotidique
D - Prescription de Valium®
E - Anastomose porto-cave
Bonne(s) réponse(s) : B D E

Les autres causes déclenchantes (en dehors de l'hémorragie digestive) sont : les infections, les déosrdres hydro-
électrolytiques provoqués par les diurétiques.
Lorsqu'aucun facteur déclenchant n'a pu être mis en évidence, le pronostic est sombre.

53
Exclusivement sur DOC - DZ : www.doc-dz.com NADJI 85
RESIDANAT EN POCHE TOME II
Cas Clinique en QCM

L'état du malade reste précaire ; la recherche de l'alphafoetoprotéine est positive. Par quel(s) examen(s)
pouvez-vous rechercher le cancer du foie sur cirrhose chez ce malade ?
A - Artériographie coelio-mésentérique
B - Cholangiographie percutanée transhépatique
C - Echographie hépatique
D - Portographie transhépatique
E - Dosage des ACE
Bonne(s) réponse(s) : A C

L'artériographie est en fait rarement utile, essentiellement demandée si une intervention chirurgicale est envisageable ;
l'échographie et le dosage de l'alphafoetoprotéine suffisent généralement, avec une ponction biopsie guidée sous
échographie si l'hémostase le permet et si une histologie est nécessaire.

Un homme de 45 ans consulte pour des douleurs épigastriques rythmées par les repas, sans irradiation particulière, à type de
crampes. Ces douleurs sont apparues il y a 2 semaines environ mais ne s'accompagnent d'aucune altération de l'état général
et d'aucun amaigrissement. Dans les antécédents personnels de ce malade on note l'existence il y a 4 ans d'un ulcère du
bulbe traité médicalement par les antihistaminiques H2 pendant 2 mois avec un résultat spectaculaire : disparition totale de la
symptomatologie douloureuse en quelques jours. Cet homme prend 2 à 3 fois par mois de l'aspirine en raison de maux de
tête. Il fume 20 cigarettes par jour et boit en moyenne 1/2 litre de vin par jour (40 g d'alcool par jour). L'éventualité d'une
récidive de la maladie ulcéreuse est des plus probables.

Quel examen allez-vous demander pour faire ce diagnostic ?


A - Transit oesogastroduodénal
B - Fibroscopie gastroduodénale
C - Chimie gastrique
D - Dosage de la gastrinémie
E - PHmétrie oesophagienne
Bonne(s) réponse(s) : B

La récidive fait partie de l'histoire naturelle de la maladie ulcéreuse.

Parmi les médicaments suivants, vous considérez comme anti-sécrétoire ?


A - Phosphate d'alumine (Phosphalugel®)
B - Sucralfate (Ulcar®)
C - Cimétidine (Tagamet®)
D - Ranitidine (Azantac®)
E - Alginates (Gaviscon®)
Bonne(s) réponse(s) : C D

Les médicaments antisécrétoires gastriques sont : les inhibiteurs des récepteurs H2 de l'histamine, les anticholinergiques,
l'oméprazole (inhibiteur de la pompe à proton).

L'aspirine peut être responsable de lésions aiguës ou hémorragiques de la muqueuse gastroduodénale. Cette
gastrotoxicité peut être également le fait :
A - Du paracétamol
B - De la noramidopyrine
C - De l'indométhacine
D - Des benzodiazépines
E - De la phénylbutazone
Bonne(s) réponse(s) : C E

Le mécanisme de cette gastrotoxicité fait appel essentiellement à une inhibition de la synthèse des prostaglandines.

Parmi les propositions suivantes, la guérison de la poussée ulcéreuse peut être favorisée par :
A - Arrêt du tabac
B - Régime sans graisses
C - Prescription de Tagamet® 800 mg le soir pendant 6 semaines
D - Prescription d'une seule dose d'antiacides avant le repas à midi pendant 5 semaines
E - Suppression de l'utilisation d'aspirine
Bonne(s) réponse(s) : A C E

L'arrêt du tabac est essentielle pour diminuer l'incidence des récidives de la maladie ulcéreuse.

54
Exclusivement sur DOC - DZ : www.doc-dz.com NADJI 85
RESIDANAT EN POCHE TOME II
Cas Clinique en QCM

Si au bout d'un traitement de 2 mois le malade continue à souffrir et s'il existait toujours un ulcère évolutif, quels
sont les 2 examens que vous demanderiez d'abord pour rechercher un syndrome de Zollinger-Ellison ?
A - PHmétrie gastrique
B - Chimisme gastrique avec étude de la sécrétion basale
C - Dosage de la gastrinémie
D - Exploration fonctionnelle pancréatique
E - Artériographie des artères abdominales
Bonne(s) réponse(s) : B C

Un débit acide basal supérieur à 26 mEq/h et une gastrinémie basale supérieur à 221 pg/ml sont quasi spécifiques d'un
syndrome de Zollinger-Ellison.
Un débit acide basal inférieur à 11 mEq/h et une gastrinémie basale inférieure à 93 pg/ml permettent d'exclure
raisonnablement un Zollinger-Ellison.
En cas de suspicion clinique, des valeurs intermédiaires sont une indication du test à la sécrétine.

Une femme de 60 ans, pesant 67 kilos pour 1,50 mètre est adressée pour angiocholite. Son seul antécédent est une
hystérectomie totale pratiquée il y a 18 ans pour une indication inconnue. L'examen clinique retrouve un empâtement
douloureux de l'hypochondre droit. un signe de Murphy. La fièvre est 38°. Il existe une oligurie. La biologie montre des signes
de cholestase. Il existe une hyperleucocytose à polynucléaires.

L'angiocholite est définie par la séquence :


A - Douleur, fièvre, ictère
B - Fièvre, douleur, ictère
C - Fièvre, ictère, oligurie
D - Ictère, prurit, hépatomégalie
E - Ictère, douleur, fièvre
Bonne(s) réponse(s) : A

Cette séquence caractérise les formes typiques.


Un autre élément sémiologique important est l'existence de frissons.

La cholestase biologique comporte :


A - Augmentation des transaminases
B - Augmentation des phosphatases alcalines
C - V.S. élevée
D - Alpha 2 globulines élevées
E - Bilirubine libre élevée
Bonne(s) réponse(s) : B

L'élévation de la gamma G.T. permet, en pratique courante, de rattacher une élévation des phosphatases alcalines à une
cholestase.

Quel examen complémentaire demandez-vous en première intention ?


A - Cholécystographie orale
B - Echographie abdominale
C - Cholangiographie rétrograde
D - Cholangiographie transpariéto-hépatique
E - Laparoscopie
Bonne(s) réponse(s) : B

L'échographie abdominale montre typiquement une dilatation de la voie biliaire principale, une vésicule lithiasique.

Statistiquement ce tableau est dû le plus souvent à :


A - Une métastase hépatique d'un cancer utérin
B - Un cancer de la tête du pancréas
C - Un ampullome vatérien
D - Une lithiase vésiculaire
E - Une lithiase cholédocienne
Bonne(s) réponse(s) : E

Une lithiase cholédocienne réalise un obstacle intermittent à l'écoulement biliaire ; il en résulte une stase biliaire et une
contamination par les germes intestinaux ; d'où l'infection de la voie biliaire principale ou angiocholite.

55
Exclusivement sur DOC - DZ : www.doc-dz.com NADJI 85
RESIDANAT EN POCHE TOME II
Cas Clinique en QCM
Un homme de 30 ans a depuis 4 ans une diarrhée faite de 3 à 5 selles par jour, abondantes et souvent décolorées. Il n'y a pas
d'alternance avec une constipation et pas de douleurs abdominales. Il a perdu environ 6 kg ces dernières années malgré la
constatation d'un oedème malléolaire vespéral récent.
La rectoscopie est normale et on ne découvre pas de parasites à l'examen des selles. Le lavement baryté est normal. Le
poids des selles est de 600g avec un taux de lipides de 30 g par 24 heures.
Des biopsies du 3ème duodénum au cours d'une endoscopie digestive haute permettent de faire le diagnostic d'atrophie
villositaire. Le diagnostic de maladie coeliaque est porté.

Les examens biologiques peuvent découvrir dans ce cas :


A - Une anémie ferriprive
B - Un allongement du temps de Quick
C - Une hyposécrétion biliaire
D - Une hypocalciurie dans les urines de 24 heures
E - Un défaut d'absorption du D-xylose
Bonne(s) réponse(s) : A B D E

Ces anomalies sont le reflet du syndrome de malabsorption.


Le test au D-xylose (sucre à absorption duodéno-jéjunale) est anormal dans 90 à 95% des maladies coeliaques de l'adulte.

Le régime sans gluten doit comporter la suppression du ou des aliment(s) suivant(s) :


A - Le poisson
B - Le pain
C - Les oeufs
D - La viande de boeuf
E - Les pâtes alimentaires
Bonne(s) réponse(s) : B E

Le régime sans gluten permet habituellement une amélioration rapide : réduction de la diarrhée et reprise pondérale en
quelques semaines.

L'oedème des membres inférieurs est dans ce cas causé par :


A - Une insuffisance rénale fonctionnelle
B - Une insuffisance cardiaque
C - Une stéatose hépatique
D - Une hypoalbuminémie
E - Une carence en vitamines
Bonne(s) réponse(s) : D

L'hypoalbuminémie reflète à la fois la réduction des ingesta, la malabsorption protéique et l'entéropathie exsudative ; elle est
présente dans près de 40% des cas chez l'adulte.

La stéatorrhée de ce patient peut résulter en priorité du mécanisme physiopathologique suivant :


A - Une accélération du transit
B - Une insuffisance pancréatique exocrine
C - Une fistule grêlo-grêlique
D - Un intestin congénitalement court
E - Une réduction du nombre des entérocytes matures
Bonne(s) réponse(s) : B

Une malabsorption peut entraîner une insuffisance pancréatique exocrine fonctionnelle, par atteinte de la phase intestinale de
la sécrétion pancréatique.

L'affection aurait également pu être affirmée par :


A - Une biopsie hépatique
B - Une biopsie jéjunale
C - Un transit baryté du grêle
D - Une biopsie rectale
E - Une analyse des selles
Bonne(s) réponse(s) : B

Seules des biopsies duodénales ou jéjunales permettent d'affirmer une atrophie villositaire totale ou subtotale du grêle
proximal, condition nécessaire (mais non suffisante) au diagnostic de maladie coeliaque.

56
Exclusivement sur DOC - DZ : www.doc-dz.com NADJI 85
RESIDANAT EN POCHE TOME II
Cas Clinique en QCM
Monsieur R. âgé de 32 ans, maçon de profession, est hospitalisé en urgence dans un service d'ORL pour un traumatisme
facial secondaire à une chute. L'examen initial montre un hématome péri-orbitaire bilatéral et une fracture ouverte du
maxillaire inférieur. L'examen ne montre pas de signe de localisation neurologique. Il n'y a pas de trouble de conscience. De
plus, l'examen révèle l'abolition des achilléens et une hépatomégalie ferme et régulière de 14 cm sans reflux hépato-jugulaire.
Il n'y a pas d'ascite mais une discrète circulation collatérale de type porto-cave. Il existe une légère trémulation. Les
radiographies de la boîte crânienne sont normales, l'alcoolémie est à 2,6 g/l, les premiers résultats du bilan hépatique
montrent des TGO à 90, des TGP à 60, un TP à 30 %, des plaquettes à 130 g/l, le fibrinogène à 4,15 g/l, La recherche de
complexes solubles est négatives.

Chez ce patient alcoolique chronique que vous suspectez être alcoolo-dépendant. quel traitement du syndrome
de sevrage mettez-vous en oeuvre ?
A - Réhydratation par voie veineuse à l'aide de glucosé isotonique
B - Vitamine B1 par voie intramusculaire
C - Vitamine B1 en perfusion
D - Sédatifs par voie intramusculaire
E - Sédatifs par voie intraveineuse
Bonne(s) réponse(s) :

QUESTION ANNULEE.

Quelle est la proposition exacte concernant la vitamine B1 ?


A - La réhydratation glucosée chez un éthylique chronique doit être associée à la vitamine B1 en raison du
risque d'encéphalopathie de Gayet-Wernicke
B - On ne doit pas l'administrer par voie intraveineuse
C - L'utilisation de la vitamine B1 est contre-indiquée en cas d'insuffisance hépatique majeure
D - L'utilisation de la voie orale est aussi efficace que la voie parentérale
E - L'administration de vitamine B1 est contre-indiquée chez la femme enceinte
Bonne(s) réponse(s) :

QUESTION ANNULEE.

Parmi les sédatifs suivants, vous pouvez utiliser chez ce patient ?


A - Tranxene® (ampoule à 20 mg)
B - Equanil® l ampoule à 400 mg)
C - Valium® (ampoule à 10 mg)
D - Atrium® 300 (comprimé)
E - Haldol® (ampoule à 5 mg/ml)
Bonne(s) réponse(s) :

QUESTION ANNULEE.

L'intervention en urgence étant indispensable, quelle thérapeutique utilisez-vous pour corriger les troubles de la
coagulation ?
A - Sulfate de protamine
B - Vitamine K en perfusion
C - Facteur VIII
D - Plasma frais congelé
E - Cryoprécipité
Bonne(s) réponse(s) :

QUESTION ANNULEE.

Quelques jours après l'intervention alors que le traitement per-opératoire des troubles de coagulation était
arrêté, apparaît une ascite avec hyponatrémie à 125 mmol. Le patient ne présente plus de signes de sevrage
éthylique et le bilan hépatique est inchangé. Vous pouvez prescrire :
A - Régime sans sel strict
B - Restriction hydrique
C - Aldactone® 75 micronisée
D - Antibiothérapie à long spectre
E - Repos au lit
Bonne(s) réponse(s) :

QUESTION ANNULEE.

57
Exclusivement sur DOC - DZ : www.doc-dz.com NADJI 85
RESIDANAT EN POCHE TOME II
Cas Clinique en QCM

Pour traiter l'hyponatrémie quelle(s) thérapeutique(s) pouvez-vous utiliser ?


A - Supplémentaire en NaCl par voie IV
B - Diurétiques thiazidiques
C - Plasma frais congelé
D - Diurétiques de l'anse (furosémide à dose massive en flash)
E - Soluté salé à 9 % (500 ml par jour)
Bonne(s) réponse(s) :

QUESTION ANNULEE.

Une femme de 62 ans consulte pour des brûlures rétrosternales, à point de départ épigastrique, survenant principalement
après les repas, mais réveillant quelquefois la patiente la nuit. Il n'existe pas de dysphagie. On relève dans les antécédents
une hystérectomie pour fibrome à l'âge de 38 ans, et une HTA modérée traitée par Catapressan® 1 c/jour. A l'examen
clinique, il existe une surcharge pondérale (73 kg pour 1,65 m), l'abdomen est souple, pas d'hépatosplénomégalie. Les
caractéristiques de la symptomatologie vous permettent de suspecter un reflux gastro-oesophagien.

Ce reflux gastro-oesophagien acide peut être prouvé et quantifié par le résultat de :


A - Oesogastroscopie
B - PHmétrie oesophagienne
C - Scintigraphie oesophagienne
D - Transit baryté oesogastroduodénal
E - Manométrie oesophagienne
Bonne(s) réponse(s) : B

La pH-métrie oesophagienne est le seul examen posant avec certitude le diagnostic d'un reflux gastro-oesophagien acide.

En cas de reflux gastro-oesophagien, quel(s) est(sont) dans le liquide de reflux le(s) constituant(s) toxique(s)
agressif(s) pour la muqueuse oesophagienne ?
A - Pepsine
B - Sels biliaires
C - Bicarbonates
D - Acide chlorhydrique
E - Mucus
Bonne(s) réponse(s) : A B

En cas de reflux gastro-oesophagien (RGO) acide, le rôle majeur toxique pour la muqueuse oesophagienne est tenu par la
pepsine (en milieu acide).
En cas de reflux alcalin (reflux duodéno-gastrique) ce rôle est tenu par les sels biliaires déconjugués et en particulier les
acides biliaires secondaires tels le désoxycholate.

Le principal mécanisme antireflux est constitué par le sphincter inférieur de l'oesophage (SIO). Chez le sujet
normal, cochez la ou les propositions exactes concernant ce sphincter :
A - Le SIO se relâche à la déglutition
B - Le SIO a une pression constante de 1 à 5 cm d'eau
C - Le SIO est constitué par un anneau musculaire palpable
D - La pression du SIO augmente après injection de gastrine
E - Le SIO subit des influences médicamenteuses
Bonne(s) réponse(s) : A D E

Le SIO est une entité physiologique, identifiable par la manométrie qui découvre sur 2 à 4 cm une zone de haute pression
généralement comprise entre +20 et +40 cm d'eau par rapport à la pression gastrique. Le SIO possède 2 propriétés
essentielles :
1) Il est le siège d'un tonus permanent.
2) Il se relaxe lors de la déglutition et de la distension oesophagienne, pendant toute la durée du parcours de l'onde
péristaltique.

Quel est l'examen pouvant permettre de mettre en évidence une oesophagite peptique chez cette patiente
présentant cliniquement une symptomatologie de reflux gastro-oesophagien ?
A - Oesophagoscopie avec biopsie
B - PHmétrie oesophagienne
C - Manométrie oesophagienne
D - Transit baryté oesogastroduodénal
E - Scintigraphie oesophagienne
Bonne(s) réponse(s) : A

La fibroscopie oesogastrique est le seul examen qui permette la mise en évidence d'une oesophagite peptique, l'appréciation
de son stade de gravité, et la pratique de biopsies.

58
Exclusivement sur DOC - DZ : www.doc-dz.com NADJI 85
RESIDANAT EN POCHE TOME II
Cas Clinique en QCM

Cochez la ou les complications possibles au cours de l'évolution d'une oesophagite peptique par reflux :
A - Ulcère oesophagien
B - Rupture de l'oesophage
C - Hernie hiatale
D - Pleurésie gauche
E - Sténose oesophagienne
Bonne(s) réponse(s) : A E

Les oesophagites peptiques superficielles sont de loin les plus fréquentes, d'évolution habituellement bénigne. Ce n'est que
dans environ 3 % des cas que l'oesophagite érosive se complique, complications qui surviennent d'ailleurs plutôt d'emblée
qu'au cours du suivi des patients.

Un endobrachyoesophage peut venir compliquer un reflux gastro-oesophagien. Cochez la ou les propositions


exactes concernant cette complication :
A - Il est constitué par un raccourcissement congénital de l'oesophage
B - Il favorise l'apparition d'un carcinome épidermoïde au niveau du bas oesophage
C - Le transit baryté oesophagien permet le diagnostic d'endobrachyoesophage
D - La muqueuse du bas oesophage est remplacée par une muqueuse glandulaire de type gastrique
E - Il favorise l'apparition d'un adénocarcinome au niveau du bas oesophage
Bonne(s) réponse(s) : D E

L'endobrachyoesophage (EBO) ou oesophage de Barrett est caractérisé par le remplacement de la muqueuse malpighienne
normale du bas oesophage par une muqueuse glandulaire. Cette métaplasie est généralement acquise et résulte d'agressions
répétées de la muqueuse malpighienne par un RGO pathologique. La prévalence de l'adénocarcinome lié à l'EBO est de
l'ordre de 10 à 15%, par contre son incidence est beaucoup plus faible, de l'ordre de 2 à 20 pour 1000 patients/année.

En cas de reflux gastro-oesophagien acide, quels sont le ou les moyens thérapeutiques efficaces sur la
symptomatologie ?
A - Prise d'alginates après les repas
B - Cimétidine (Tagamet®) 1 600 mg/24 h
C - Régime sans résidus
D - Anticholinergiques (dérivés atropiniques) aux 3 repas
E - Cholestyramine (Questran®) 1 sachet à chaque repas
Bonne(s) réponse(s) : A B D

Ces moyens sont relativement efficaces sur la symptomatologie du RGO ; mais en ce qui concerne les lésions d'oesophagite
peptique, seuls les anti H2 et l'oméprazole ont démontré leur efficacité.

Si une intervention chirurgicale devient nécessaire pour corriger ce RGO, quelle technique vous paraît
appropriée ?
A - Myotomie extra-muqueuse de Heller
B - Intervention de Nissen
C - Gastrectomie des 2/3 avec anastomose gastroduodénale
D - Oesogastrectomie polaire supérieure
E - Vagotomie supra-sélective
Bonne(s) réponse(s) : B

Une autre intervention est possible ; il s'agit de la diversion duodénale totale qui comporte :
a) une vagotomie tronculair
b) une antrectomie
c) une anastomose gastro-jéjunale termino-latérale sur une anse en Y de 70 cm.
Cette intervention supprime la possibilité de RGO acide ou alcalin.

59
Exclusivement sur DOC - DZ : www.doc-dz.com NADJI 85
RESIDANAT EN POCHE TOME II
Cas Clinique en QCM
Une femme de 50 ans, célibataire, infirmière, ayant présenté une tuberculose pulmonaire en 1969 guérie par les antibiotiques
antituberculeux, se plaint depuis plusieurs mois d'une asthénie, d'une anorexie, d'une augmentation modérée du volume de
l'abdomen. Elle a maigri de 4 kgs. Depuis deux jours, elle présente des douleurs abdominales assez vives, des vomissements
fétides. Une selle diarrhéique a été émise. A l'examen, on constate des signes discrets de déshydratation extracellulaire, un
abdomen météorisé symétrique et immobile sans ondulations péristaltiques. La percussion retrouve des zones de sonorité
alternant avec des zones mates. La matité hépatique est préservée. Il n'y a pas de circulation collatérale, ni d'oedème des
membres inférieurs. Les touchers pelviens semblent normaux. La palpation des fosses lombaires est normale. Les éléments
biologiques ne témoignent que d'une hémoconcentration avec cependant une VS élevée à 45 à la première heure.

Ce tableau évoque une hypothèse. Laquelle ?


A - Une rupture d'anévrysme de l'aorte
B - Une perforation d'ulcère duodénal
C - Une carcinose péritonéale
D - Une ascite cirrhotique
E - Un volvulus du côlon pelvien
Bonne(s) réponse(s) : C

Les formes subaiguës de carcinose péritonéale peuvent simuler une urgence chirurgicale.
L'ancienneté des troubles, l'amaigrissement, les données de l'examen clinique (zones mates et zones sonores, palpation de
masses tumorales épiploïques) permettent d'évoquer le diagnostic.

Quel examen complémentaire demandez-vous en premier ?


A - Coloscopie
B - Echographie abdominale
C - Cliché sans préparation de l'abdomen
D - Transit aux hydrosolubles
E - Lavement opaque
Bonne(s) réponse(s) : C

L'ASP peut permettre de révéler, outre des aspects non spécifiques de niveaux hydro-aériques, des signes en rapport avec la
carcinose péritonéale, infiltrant ou comprimant les anses intestinales (aspect figé et irrégulier, segmentaire et plurifocal).

Devant ce tableau, votre attitude thérapeutique comporte :


A - Faire opérer la malade sans délai et sans diagnostic
B - Mettre une sonde d'aspiration digestive
C - Réhydrater
D - Prescrire des morphiniques
E - Faire un lavement évacuateur
Bonne(s) réponse(s) : B C

Dans ce contexte, il faut éviter d'opérer d'emblée et sans diagnostic.


L'aspiration gastrique et la réhydratation permettent le plus souvent de passer un cap critique, avec retour à un transit
intestinal quasi normal.
En fonction du cancer primitif en cause, un traitement chimiothérapique pourra être instauré.

Le lendemain à l'examen, la malade étant en décubitus dorsal, il existe des zones de matité médio-abdominales
et dans les flancs. Ces zones de matité :
A - Sont physiologiques
B - Evoquent des nodules de carcinose péritonéale
C - Peuvent témoigner d'une ascite cloisonnée
D - Peuvent témoigner d'un pneumopéritoine
E - Peuvent témoigner de dilatation des anses intestinales
Bonne(s) réponse(s) : C

Ces zones de matité pourraient également correspondre à des masses tumorales épiploïques.

60
Exclusivement sur DOC - DZ : www.doc-dz.com NADJI 85
RESIDANAT EN POCHE TOME II
Cas Clinique en QCM
Un homme de 35 ans est hospitalisé en urgence pour des douleurs abdominales et un ictère. Dans les antécédents on note
une intoxication alcoolique de 180 g par jour depuis plus de 5 ans. L'examen à l'entrée montre un malade en mauvais état
général, avec ictère cutanéo-muqueux. Il a maigri de 3 kg depuis 2 mois. L'examen de l'abdomen montre une hépatomégalie
de 18 cm sur la ligne médio-claviculaire et une splénomégalie. Il n'y a pas d'ascite. Le bilan biologique montre une anémie
macrocytaire, une hyperamylasémie à 4 fois la normale, une cholestase avec des phosphatases alcalines à 3 fois la normale
et une hyperbilirubinémie à 10 fois la normale. Les radiographies d'abdomen sans préparation ne montrent ni
pneumopéritoine ni niveaux liquides, mais des calcifications en regard de la 1ère vertèbre lombaire. Une échotomographie
réalisée en urgence montre une dilatation importante des voies biliaires et une augmentation de volume de la tête du
pancréas.

Indiquez le diagnostic le plus probable :


A - Une pancréatite aiguë nécrotico-hémorragique
B - Une hépatite alcoolique aiguë
C - Une cholécystite
D - Une poussée aiguë sur pancréatite chronique
E - Un cancer de la tête du pancréas
Bonne(s) réponse(s) : D

L'intoxication éthylique chronique, mais surtout la présence sur l'aire pancréatique de calcifications à l'ASP sont fortement
évocatrices de pancréatite chronique.

Précisez le mécanisme probable de l'ictère :


A - Infectieux
B - Compression du bas cholédoque
C - Obstacle lithiasique de la voie biliaire principale
D - Atteinte du parenchyme hépatique par l'alcool
E - Lithiase du canal de Wirsung
Bonne(s) réponse(s) : B

L'augmentation de volume de la tête du pancréas est vraissemblablement à l'origine d'une compression du bas cholédoque.
Une lithiase de la voie biliaire principale ne peut cependant être formellement exclue que par une cholangiographie rétrograde.

Indiquez le traitement que vous conseillez pour supprimer cet ictère ?


A - La ponction-drainage percutanée des voies biliaires
B - La sphinctérotomie endoscopique
C - La prescription de vitamine B1 associée à l'arrêt de l'intoxication alcoolique
D - La dérivation chirurgicale de la voie biliaire
E - L'instillation endoscopique de cholérétiques
Bonne(s) réponse(s) :

QUESTION ANNULEE.

Indiquez parmi les suivants le diagnostic qu'évoque chez ce patient la présence d'une splénomégalie :
A - Une septicémie
B - Une hépatite à virus
C - Une hypertension portale segmentaire
D - Une atteinte hématologique alcoolique
E - Une thrombose de l'artère splénique
Bonne(s) réponse(s) : C

Ce patient a plusieurs raisons pour présenter une hypertension portale responsable d'une splénomégalie :
- une compression du tronc porte par le pancréas augmenté de volume (hypertension portale segmentaire)
- une possible cirrhose alcoolique associée
- une possible thrombose portale ou de la veine splénique.

61
Exclusivement sur DOC - DZ : www.doc-dz.com NADJI 85
RESIDANAT EN POCHE TOME II
Cas Clinique en QCM
Un homme de 54 ans, porteur d'une lithiase biliaire connue,éthylique chronique présente depuis 24 heures un syndrome
douloureux épigastrique d'apparition brutale accompagné de vomissements.
Lorsque vous l'examinez vous notez :
- Une TA systolique à 8 cmHg
- Une tachycardie = 11 2/mn
- Une polypnée
- L'auscultation cardiaque est normale
- L'examen de l'abdomen montre que ce dernier respire mais vous notez quelques tâches ecchymotiques péri-ombilicales.
La palpation met en évidence un météorisme, une légère défense diffuse à tout l'abdomen sans contracture. La température
est à 37°8 .
Les touchers pelviens sont normaux.
Vous évoquez le diagnostic de pancréatite aiguë.

Quel(s) argument(s) est(sont) en faveur du diagnostic de pancréatite aiguë ?


A - Lithiase biliaire connue
B - Absence de contracture abdominale
C - Baisse de la T.A.
D - Ethylisme avéré
E - Taches ecchymotiques péri-ombilicales
Bonne(s) réponse(s) : A D E

L'absence de contracture abdominale et la baisse de la TA peuvent parfaitement accompagner un infarctus mésentérique ou


un infarctus du myocarde qui sont des diagnostics différentiels importants.

A quel(s) examen(s) complémentaire(s) pouvez-vous faire appel pour étayer votre diagnostic ?
A - Echographie abdominale
B - Artériographie coelio-mésentérique
C - Abdomen sans préparation
D - Fibroscopie oesogastroduodénale
E - Transit gastroduodénal aux hydrosolubles
Bonne(s) réponse(s) : A C

Sur le plan morphologique, le scanner abdominal est également un examen utile, qui permet la classification des lésions
observées en stades de gravité.

L'évolution de la maladie pancréatique fait appel à la surveillance de :


A - Transaminases sériques
B - Ionogramme sanguin
C - Créatininémie
D - Calcémie
E - Gamma-glutamyl-transpeptidases
Bonne(s) réponse(s) : B C D

Sur le plan biologique, ne pas omettre la surveillance de l'amylasémie, de l'amylasurie, éventuellement de la lipasémie.

Dans ce cas, vous préconisez systématiquement :


A - Surveillance de l'électro-encéphalogramme afin de dépister les troubles de la conscience
B - Mise en place d'un cathéter veineux
C - Mise en place d'une sonde nasogastrique
D - Mise en place d'une sonde vésicale à demeure
E - Mise en place d'un cathéter intra-artériel pour surveillance continue de la tension artérielle
Bonne(s) réponse(s) : B C D

Il s'agit de traiter et de surveiller au mieux ce patient en état de choc, en milieu de réanimation chirurgicale.

La surveillance étant en route, les principes du traitement incluent de façon systématique :


A - Le remplissage vasculaire pour lutter contre le choc et les troubles ioniques
B - La prescription de morphinomimétiques pour lutter contre la douleur
C - Des vasopresseurs
D - Une corticothérapie importante
E - Des anti-histaminiques
Bonne(s) réponse(s) :

QUESTION ANNULEE.

62
Exclusivement sur DOC - DZ : www.doc-dz.com NADJI 85
RESIDANAT EN POCHE TOME II
Cas Clinique en QCM

L'intervention chirurgicale sera décidée :


A - De principe en urgence
B - Devant l'apparition d'un tableau péritonéal aigu
C - Devant la constitution d'un tableau angiocholitique typique avec calcul du cholédoque à la
cholangiographie
D - Dans la crainte de laisser se constituer une pancréatite chronique
E - Devant l'apparition d'un épanchement pleural gauche
Bonne(s) réponse(s) :

QUESTION ANNULEE.

Un homme de 30 ans est hospitalisé pour une altération de l'état général (amaigrissement de 6 kg), installée en deux mois,
accompagnée de nausées et d'un endolorissement de l'hypochondre droit.
Dans les antécédents, on relève une gastrectomie des deux tiers, il y a 5 ans, pour un ulcère gastrique. L'interrogatoire
rapporte la notion d'une alcoolisation excessive (2 litres de vin, 2 à 4 apéritifs, 2 verres de bière par jour). A l'examen clinique,
vous constatez la présence d'une hépatomégalie ferme lisse, sensible, accompagnée d'un subictère et d'une fébricule à 38
degrés. Il existe quelques angiomes stellaires thoraciques. Il n'y a pas d'autres anomalies cliniques.

Vous retenez en faveur du diagnostic d'hépatite alcoolique aiguë :


A - L'âge du malade
B - La fébricule
C - Les angiomes stellaires
D - L'hépatomégalie sensible
E - Le subictère
Bonne(s) réponse(s) :

QUESTION ANNULEE.

Le vin est la boisson alcoolisée la plus consommée par ce patient et il est exact que :
A - 1 litre de vin à 12° contient 96 g d'alcool pur
B - 1 litre de vin à 10° apporte 320 kilocalories
C - La teneur en glucides du vin rouge ordinaire est élevée, de l'ordre de 40 g/l
D - Le principal composant toxique du vin est l'éthanol
E - La consommation régulière de vin stimule la vitesse d'oxydation de l'éthanol au niveau du foie
Bonne(s) réponse(s) : A C D E

La valeur énergétique de l'alcool est de 7,1 kcal/g ; 1 litre de vin à 10° apporte 80 g d'alcool, soit 560 kcal.
En faite, cette valeur n'est que théorique, l'utilisation énergétique de l'alcool étant bien moindre.

L'hémogramme initial est le suivant :


- hématies:4 850.000/mm3
- hémoglobine:130 g/l
- VGM:107 microns cubes
- leucocytes:11 000/mm3 dont 85 % de PN
- plaquettes:50 000/mm3.
Vous en concluez que :
A - La macrocytose est un stigmate biologique fréquent de l'alcoolisation excessive
B - Le déficit nutritionnel habituellement incriminé dans la macrocytose observée dans l'alcoolisme est la
carence en fotales
C - La macrocytose observée chez ce malade peut être en rapport avec ses antécédents de gastrectomie
D - Lors du sevrage, le VGM revient à la normale, rapidement, en moins d'une semaine
E - La thrombopénie est une complication possible de l'ingestion aiguë d'alcool
Bonne(s) réponse(s) : A C E

La carence en folates constatée chez de nombreux alcooliques est liée au défaut d'apport secondaire au régime, et à l'action
directe de l'alcool qui inhibe l'absorption intestinale des folates. Cette carence en folates n'est cependant pas toujours en
cause dans la macrocytose observée.

63
Exclusivement sur DOC - DZ : www.doc-dz.com NADJI 85
RESIDANAT EN POCHE TOME II
Cas Clinique en QCM

Le dosage des transaminases donne le résultat suivant :


<SGOT (ASAT) : 110 Ul/l -
<SGPT (ALAT) : 90 Ul/l.
Une biopsie hépatique est pratiquée. Vous retenez en faveur d'une hépatite alcoolique aiguë ?
A - Une élévation modérée des transaminases avec un taux de SGOT supérieur à SGPT
B - La baisse des lymphocytes circulants
C - Des lésions hépatiques prédominant dans la région péri-lobulaire
D - Un infiltrat inflammatoire à polynucléaires
E - La présence de corps hyalins de Mallory
Bonne(s) réponse(s) : A D E

Les lésions histologiques de l'hépatite alcoolique comporte typiquement :


- des altérations hépatocytaires
- des corps de Mallory
- une infiltration à polynucléaires neutrophiles
- une fibrose péricellulaire.
Ces lésions tendant à être maximales dans la région centrolobulaire.

Au terme du bilan initial, le diagnostic d'hépatite alcoolique aiguë est posé chez ce malade. L'évolution peut se
faire vers :
A - Aggravation durant les premiers jours d'hospitalisation malgré le traitement
B - Survenue d'une encéphalopathie hépatique mortelle
C - Hémorragie digestive par hypertension portale
D - Guérison totale, sans séquelles, y compris au plan histologique
E - Evolution certaine vers la cirrhose en cas de poursuite de l'alcoolisation excessive
Bonne(s) réponse(s) : A B D E

Les formes majeures d'hépatites alcooliques peuvent entraîner la mort par insuffisance hépatocellulaire.
En cas d'évolution favorable, le foie peut redevenir normal ou présenter une fibrose ou une cirrhose à distance de l'épisode
d'hépatite.
Les formes mineures, plus fréquentes, ont un potentiel évolutif immédiat beaucoup moins grave.

Une femme de 44 ans, obèse et multipare, porteuse d'une lithiase vésiculaire connue depuis un an, vient consulter pour une
douleur sévère de l'hypochondre droit évoluant depuis 6 heures environ. Sa température est à 38° et, à l'examen, son état
général paraît très altéré, vous notez un ictère conjonctival et à la palpation de l'hypochondre droit une défense et une violente
douleur.
A l'examen, il n'y a pas de syndrome occlusif. Sa tension artérielle, habituellement à 14/8 est à 12.5/8. Le reste de l'examen
est sensiblement normal.

Parmi les examens suivants, quel(s) est(sont) celui(ceux) qui est(sont) utile(s) en urgence ?
A - Amylasémie
B - N.F.S.
C - Abdomen sans préparation
D - Cholécystographie per os
E - Artériographie coelio-mésentérique
Bonne(s) réponse(s) : A B C

La cholécystographie per os ne garde pour seule indication que la vérification du caractère fonctionnel de la vésicule biliaire.

Les examens que vous avez demandés ne permettent pas de faire un diagnostic. Comme vous pensez à une
cholécystite aiguë lithiasique, vous demandez une échotomographie. Parmi les signes suivants quel est celui
qui permettra de confirmer ce diagnostic ?
A - Elargissement de la tête du pancréas
B - Dilatation des voies biliaires intra-hépatiques
C - Epaississement de la paroi vésiculaire
D - Lacunes hétérogènes dans le foie droit
E - Aucune des propositions ci-dessus
Bonne(s) réponse(s) : C

L'épaississement de la paroi vésiculaire peut également s'observer en cas d'hypoalbuminémie importante, par oedème de la
paroi (diagnostic différentiel entre cholécystite aiguë et hépatite alcoolique aiguë avec hypoalbuminémie).

64
Exclusivement sur DOC - DZ : www.doc-dz.com NADJI 85
RESIDANAT EN POCHE TOME II
Cas Clinique en QCM

Le diagnostic de cholécystite aiguë étant confirmé, quelles sont la ou les possibilité(s) thérapeutique(s) ?
A - Glace + antispasmodiques
B - Cholécystectomie en urgence
C - Antibiotique puis acide ursodésoxycholique
D - Cathétérisme rétrograde suivi de sphinctérotomie endoscopique
E - Anastomose cholédoco-duodénale
Bonne(s) réponse(s) : B

La cholécystectomie est impérative, elle peut se réaliser après quelques heures de traitement antibiotique et antispasmodique.

La malade refuse l'hospitalisation et tout traitement. Trois jours après, elle est hospitalisée en urgence pour état
de choc toxi-infectieux avec contracture abdominale diffuse. Quel diagnostic peut-on évoquer ?
A - Pancréatite aiguë
B - Ulcère perforé
C - Péritonite biliaire
D - Iléus biliaire
E - Angiocholite ictéro-urémigène
Bonne(s) réponse(s) : C

La péritonite biliaire est due à une perforation de la vésicule dans la cavité péritonéale.
Elle se traduit par une contracture abdominale et des signes généraux sévères.

Un homme atteint de maladie coeliaque est réhospitalisé pour diarrhée et amaigrissement fin septembre 1989. Il a perdu 9 kg
en 3 mois. Agé de 36 ans, mesurant 1, 74 m, il a toujours été un gros mangeur. Son poids maximum, avant sa maladie
coeliaque, a été 61 kg à l'âge de 26 ans, 6 mois après avoir arrêté la compétition (il jouait au tennis de table). Sa mère pèse
environ 50 kg pour 1,68 m et son père 66 kg pour 1,76 m.
La maladie coeliaque à été découverte il y a 4 ans et le malade a suivi depuis très étroitement son régime sans gluten. A
l'interrogatoire il ne prend aucun médicament. Il est manifestement un peu déprimé et met ceci sur le compte de son échec à
ses examens de Faculté de la session de Juin. La diarrhée dure depuis 3 semaines, faite de 2 à 3 selles molles et collantes,
assez abondantes, par jour.
L'examen clinique ne met en évidence aucun élément particulier en dehors de la maigreur évidente. Il n'y a pas de fièvre. Le
poids corporel de cet homme est de 50 kg. Les débits fécaux quotidiens (recueil de 3 jours) sont les suivants : poids = 560g
azote = 3g, lipides = 15g, glucides (fibres alimentaires non dosées) = 40g par jour.
Biologiquement on note les faits suivants : VS = 3 (1ère h), fibrinémie = 2,5 g/l, protéine C réactive = 0,001 g/l, natrémie = 140
mmol/l, kaliémie = 3,5 mmol, protidémie = 60g/l, albuminémie = 30g/l, pré-albuminémie (thyroxine binding pré-albumine) =
0,23 g/l (N > 0,26 g/l).
Interrogatoire alimentaire : protides = 60 g, lipides = 81 g, glucides = 250g par jour (dont fibres alimentaires = 20g/l, lactose
(lait) = 20g/l).

Ce malade a perdu du poids pour la ou les raisons suivantes :


A - Il a une maladie coeliaque
B - Il a actuellement une malabsorption gravissime portant uniquement sur les graisses
C - Il a actuellement une malabsorption gravissime portant sur les 3 principaux macro-nutriments
D - Il ne suit pas du tout son régime sans gluten
E - Il ne mange pas assez
Bonne(s) réponse(s) : C

La malabsoption porte sur les trois principaux macro-nutriments :


- protides : créatorrhée à 3 g/24 heures (N < 1,8 g)
- lipides : stéatorrhée à 15 g/24 heures (N < 3 g)
- glucides : existence de glucides dans les selles, en dehors des fibres alimentaires.
NB : Le patient ne suit vraisemblablement pas son régime sans gluten, mais on ne peut l'affirmer avec certitude sur les
données de l'observation.

Les ingesta énergétiques présents sont de :


A - 3 642 Kcal/j
B - 3 673 Kcal/j
C - 2 698 Kcal/j
D - 1 969 Kcal/j
E - 1 814 Kcal/j
Bonne(s) réponse(s) : D

Calories glucidiques : 250 x 4 Kcal = 1000.


Calories lipidiques : 81 x 9 Kcal = 729.
Calories protidiques : 60 x 4 Kcal = 240.
Total calorique : 1969 Kcal/jour.

65
Exclusivement sur DOC - DZ : www.doc-dz.com NADJI 85
RESIDANAT EN POCHE TOME II
Cas Clinique en QCM

Les entrées énergétiques, eu égard à la déperdition énergétique fécale, sont de :


A - 3 114
B - 1 600
C - 2 999
D - 2 841
E - 1 968
Bonne(s) Réponse(s) : B

La déperdition énergétique fécale est de :


3 X 4 + 15 X 9 + 40 X 4 = 12 + 135 + 160 = 307 Kcal et 1969 (entrées) - 307 (sorties) = 1662 Kcal

Un homme atteint de maladie coeliaque est réhospitalisé pour diarrhée et amaigrissement fin septembre 1989. Il a perdu 9 kg
en 3 mois. Agé de 36 ans, mesurant 1, 74 m, il a toujours été un gros mangeur. Son poids maximum, avant sa maladie
coeliaque, a été 61 kg à l'âge de 26 ans, 6 mois après avoir arrêté la compétition (il jouait au tennis de table). Sa mère pèse
environ 50 kg pour 1,68 m et son père 66 kg pour 1,76 m.
La maladie coeliaque à été découverte il y a 4 ans et le malade a suivi depuis très étroitement son régime sans gluten. A
l'interrogatoire il ne prend aucun médicament. Il est manifestement un peu déprimé et met ceci sur le compte de son échec à
ses examens de Faculté de la session de Juin. La diarrhée dure depuis 3 semaines, faite de 2 à 3 selles molles et collantes,
assez abondantes, par jour.
L'examen clinique ne met en évidence aucun élément particulier en dehors de la maigreur évidente. Il n'y a pas de fièvre. Le
poids corporel de cet homme est de 50 kg. Les débits fécaux quotidiens (recueil de 3 jours) sont les suivants : poids = 560g
azote = 3g, lipides = 15g, glucides (fibres alimentaires non dosées) = 40g par jour.
Biologiquement on note les faits suivants : VS = 3 (1ère h), fibrinémie = 2,5 g/l, protéine C réactive = 0,001 g/l, natrémie = 140
mmol/l, kaliémie = 3,5 mmol, protidémie = 60g/l, albuminémie = 30g/l, pré-albuminémie (thyroxine binding pré-albumine) =
0,23 g/l (N > 0,26 g/l).
Interrogatoire alimentaire : protides = 60 g, lipides = 81 g, glucides = 250g par jour (dont fibres alimentaires = 20g/l, lactose
(lait) = 20g/l).

Parmi les propositions suivantes, il est exact que :


A - Il a une malabsorption protidique, et la maladie coeliaque ne peut pas en rendre compte
B - Il n'y a pas de malabsorption lipidique, eu égard à l'énorme quantitié de graisses ingérées par le patient
C - Il a une malabsorption glucidique modérée et la maladie coeliaque peut en rendre compte
D - La déperdition énergétique fécale, nettement supérieure à 500 Kcal/jour prouve que le malade ment et
qu'il ne suit pas son régime sans gluten
E - Il n'y a pas, à priori, de pathologie inflammatoire (cancer, infection profonde) car les apports énergétiques
sont insuffisants

Bonne(s) réponse(s) : C

B et E - Il n'y a pas de relation de cause à effet.


D - La déperdiiton énergétique fécale est de 307 Kcal, et suggère fortement (mais ne prouve pas) que le malade ne suit pas
son régime.

66
Exclusivement sur DOC - DZ : www.doc-dz.com NADJI 85
RESIDANAT EN POCHE TOME II
Cas Clinique en QCM
Un homme atteint de maladie coeliaque est réhospitalisé pour diarrhée et amaigrissement fin septembre 1989. Il a perdu 9 kg
en 3 mois. Agé de 36 ans, mesurant 1, 74 m, il a toujours été un gros mangeur. Son poids maximum, avant sa maladie
coeliaque, a été 61 kg à l'âge de 26 ans, 6 mois après avoir arrêté la compétition (il jouait au tennis de table). Sa mère pèse
environ 50 kg pour 1,68 m et son père 66 kg pour 1,76 m.
La maladie coeliaque à été découverte il y a 4 ans et le malade a suivi depuis très étroitement son régime sans gluten. A
l'interrogatoire il ne prend aucun médicament. Il est manifestement un peu déprimé et met ceci sur le compte de son échec à
ses examens de Faculté de la session de Juin. La diarrhée dure depuis 3 semaines, faite de 2 à 3 selles molles et collantes,
assez abondantes, par jour.
L'examen clinique ne met en évidence aucun élément particulier en dehors de la maigreur évidente. Il n'y a pas de fièvre. Le
poids corporel de cet homme est de 50 kg. Les débits fécaux quotidiens (recueil de 3 jours) sont les suivants : poids = 560g
azote = 3g, lipides = 15g, glucides (fibres alimentaires non dosées) = 40g par jour.
Biologiquement on note les faits suivants : VS = 3 (1ère h), fibrinémie = 2,5 g/l, protéine C réactive = 0,001 g/l, natrémie = 140
mmol/l, kaliémie = 3,5 mmol, protidémie = 60g/l, albuminémie = 30g/l, pré-albuminémie (thyroxine binding pré-albumine) =
0,23 g/l (N > 0,26 g/l).
Interrogatoire alimentaire : protides = 60 g, lipides = 81 g, glucides = 250g par jour (dont fibres alimentaires = 20g/l, lactose
(lait) = 20g/l).

Parmi les propositions suivantes, il est exact que :


A - Cet homme a une certaine maigreur et il existe une note familiale de maigreur constitutionnelle
B - La déponse énergétique de repos de cet homme est très certaiement inférieure à 900 Kc/j, senon il ne
maigrirait pas
C - La dépense énergétique globale de la journée doit être très certainement supérieure à 3 400 Kcal/j, pour
que cet homme ait perdu 9 kg en 3 mois.

D - Cet homme est bien un gros mangeur, comme le confirme l'interrogatoire alimentaire actuel

E - Ses apports glucidiques représentent plus de 50 % de l'apport énergétique total

Bonne(s) réponse(s) : A E

A - Exact, si l'on considère les poids et tailles des parents.


B - NB : Si l'on utilise les équations de Harris et Benedict (dépense de repos pour un homme = 66,47 + 13,75 x P + 5 X T -
6,67 A ; P = Poids en kg, T = taille en cm, A = âge en années) en multipliant par 1,6 pour une activité modérée, on a 961
Kc/jour.
C - NB : Il est exceptionnel pour un sujet ayant une activité modérée d'avoir une dépense globale de 3 400 Kc/jour.
D - Une ingestion de 1900 Kcal/jour est normale, voire un peu basse.

Parmi les propositions suivantes, il est exact que :


A - Une potidémie basse est un excellent marqueur de la dénutrition
B - Une albumine basse est un bon marqueur de dénutrition prolongée
C - Un bon témoin de la longue durée de dénution est aussi la protéine de transport du rétinol (rétinal-binding
protein)
D - La pré-albumine basse prouve qu'il n'y a pas de syndrome inflammatoire
E - La transferrine est un marqueur nutritionnel de demi-vie d'environ 8 jours chez l'homme
Bonne(s) Réponse(s) : B E

A - La protidémie est trop dépendante de l'état d'hydratation.


B - Exact, avec les restrictions suivantes :
- l'albuminémie dépend de l'hydratation, de l'existence d'un 3e secteur
- il peu exister une fuite protidique notamment par entéropathie exsudative en cas de maladie coeliague, rendant ce marqueur
inopérant.
C - RBP : courte demi-vie.

67
Exclusivement sur DOC - DZ : www.doc-dz.com NADJI 85
RESIDANAT EN POCHE TOME II
Cas Clinique en QCM
Une malade âgée de 38 ans a fait une crise douloureuse paroxystique de l'hypocondre droit, irradiant en arrière à la base
thoracique droite et à l'épaule droite. On a découvert 2 ans plus tôt une micro-lithiase vésiculaire à l'occasion d'une première
crise. Elle est calmée par la prise de Spasfon. Le lendemain la température est à 39° et elle est hospitalisée. On constate à
l'entrée un subictère conjonctival et une douleur provoquée de la région sous-costale augmentant à l'inspiration profonde sans
véritable défense. L'épigastre est également douloureux et la malade signale que la douleur irradie maintenant à la base
thoracique gauche. Les urines sont foncées et rares. L'échographie retrouve une micro-lithiase vésiculaire et un cholédoque
de 10 mm de diamètre sans dilatation des voies biliaires intra-hépatiques et sans image de calcul.
La biologie montre :
- Bilirubine totale : 95 micromol/l
- Bilirubine directe : 80 micromol/l
- Phosphatases alcalines : 380 UI/l (Normale < ou.= à 85) f
- Gamma GT 650 UI/I (Normale < ou = à 47)
- Transaminases (SGOT) : 90 UI/l (Normale < ou = à 40)
- Amylasémie : 600 UI/l (Normale < ou = à 150)
- Créatinémie : 250 mcmol/1 (Normale < ou = à 88
- Leucocytose : 15 000 x 106/l (85 % de polynucléaires)
- Terrain : il s'agit d'une femme active, qui s'occupe de ses quatre enfants, 1,60 m, 65 kg

Le tableau que présente cette malade correspond :


A - A une cholécystite aiguë
B - A une angiocholite
C - A une pancréatite nécrosante
D - A une hépatite
E - A un iléus biliaire
Bonne(s) réponse(s) : B

L'infection bactérienne de la voie bilaire principale est une angiocholite et se traduit principalement par la fièvre et les frissons
( qui ne sont pas mentionnés dans l'observation). Une angiocholite peut se commliquer comme c'est le cas dans l'observation,
d'insuffisance rénale. Il peut y avoir une élévation de l'amylasémie, voire une pancréatite aiguë clinique, lorsque le calcul
détermine une obstruction du canal de Wirshung.

Parmi les éléments biologiques suivants de l'énoncé, quel est ou quels sont celui ou ceux en faveur du
caractère cholestatique de l'ictère ?
A - L'augmentation des Gamma GT
B - L'augmentation de l'amylasémie
C - L'augmentation des transaminases
D - L'augmentation des phosphatases alcalines
E - L'augmentation de la bilirubine directe
Bonne(s) réponse(s) : A D E

Phosphatases alcalines et gamma GT sont augmentées en cas de cholestase. En cas d'ictère cholestatique, il y a une
élévation de la bilirubine directe ou conjuguée.

Concernant le bilan radiologique et échographique quelle est ou quelles sont la, ou les propositions exactes ?
A - L'absence de calcul cholédocien visible en échographie signe la vacuité de la voie biliaire principale
B - L'échographie peut être utilement complétée ici par une cholangiographie
C - La cholangiographie rétrograde s'impose pour préciser le diagnostic
D - L'échographie doit être complétée par un scanner pancréatique
E - Les éléments apportés au dossier sont suffisants pour porter une indication thérapeutique
Bonne(s) réponse(s) : E

Le diagnostic, apporté ici par les éléments cliniques, biologiques, et échographiques est formel ; il n'y a pas de diagnostic
différentiel.

Cette malade présente un ou des signes de gravité, lequel ou lesquels ?


A - La température à 39°
B - L'irradiation scapulaire droite
C - L'élévation de la créatinine sérique
D - L'oligurie
E - L'augmentation des phosphatases alcalines
Bonne(s) réponse(s) : C D

C et D - Témoignent d'une insuffisance rénale aiguë, de mauvais pronostic dans ce contexte.

68
Exclusivement sur DOC - DZ : www.doc-dz.com NADJI 85
RESIDANAT EN POCHE TOME II
Cas Clinique en QCM

Le ou les risques évolutifs est ou sont chez cette malade :


A - La pancréatite aiguë
B - La septicémie
C - La péritonite per perforation vésiculaire
D - L'iléus biliaire
E - L'insuffisance rénale aiguë
Bonne(s) réponse(s) : A B E

La septicémie peut également se compliquer de choc septique.

Parmi les propositions thérapeutiques suivantes, laquelle vous paraît la plus adaptée ?
A - Dialyse rénale - antibiothérapie - intervention chirurgicale
B - Antibiothérapie et surveillance
C - Antibiothérapie - réanimation - intervention chirurgicale
D - Antibiothérapie - réanimation - sphinctérotomie endoscopique
E - Antibiothérapie - aspiration gastrique - réanimation - surveillance
Bonne(s) réponse(s) : C

A - Il n'y a pas d'indication de dialyse rénale à 250 µ mol/l de créatininémie.


D - La sphinctérotomie endoscopique, pourrait permettre un drainage biliaire et une intervention ultérieure (si elle est possible)
en cas de terrain à risque opératoire important (mais ce n'est pas le cas ici).

Si une intervention chirurgicale est décidée, le chirurgien réalisera obligatoirement :


A - Une cholécystectomie
B - Une pancréatectomie
C - Une anastomose bilio-digestive
D - Une désobstruction de la voie biliaire principale
E - Une exploration radiologique de la voie biliaire principale
Bonne(s) réponse(s) : A D E

Après cholédocotomie et extraction des calculs de la voie biliaire principale, le cholédoque est habituellement refermé et un
drainage biliaire externe (drain de Kehr) est réalisé. Une anastomose biliodigestive, en général cholédocoduodénale, n'est
justifiée que s'il existe de très nombreux calculs dans une voie biliaire large, chez un malade âgé.

Une femme de 78 ans, autonome et alerte, est hospitalisée pour une tuméfaction douloureuse de l'aine à droite.
L'interrogatoire retrouve la notion ancienne d'une tuméfaction de même siège, qui s'est réduite jusqu'à présent avec des bruits
hydro-aériques. Elle présente des vomissements deux heures après le début des troubles et elle signale un arrêt des gaz
après l'émission d'une selle diarrhéique.
L'examen retrouve une tuméfaction douloureuse, située sous la ligne de Malgaigne non réductible, non expansive à la toux.
Les radiographies d'abdomen sans préparation montrent quelques niveaux liquides avec un grêle modérément dilaté.

Le tableau présenté par cette patiente correspond à :


A - Une occlusion du grêle sur bride
B - Un adéno-phlegmon de l'aine
C - Une hernie crurale étranglée
D - Une hernie inguinale étranglée
E - Une épiplocèle
Bonne(s) réponse(s) : C

Le tableau clinique présenté par cette patiente est une occlusion intestinale aiguë. L'étranglement herniaire en est la cause
(tuméfaction douloureuse irréductible située sous la ligne de Malgaigne).

La ligne de Malgaigne :
A - Sépare l'étage inguinal de l'étage crural
B - Correspond au bord inférieur du tendon conjoint
C - Est étendue de l'épine iliaque antéro-supérieure à l'épine du pubis
D - Correspond au trajet des vaisseaux fémoraux
E - Correspond à l'arcade crurale (réflexion des fibres du muscle oblique externe)
Bonne(s) réponse(s) : A C E

Sans commentaire.

69
Exclusivement sur DOC - DZ : www.doc-dz.com NADJI 85
RESIDANAT EN POCHE TOME II
Cas Clinique en QCM

Parmi les structures suivantes, le canal crural est délimité par :


A - Le tendon conjoint
B - Les vaisseaux fémoraux
C - Le ligament de Cooper (crête pectinéale)
D - Le fascia transversalis
E - Le muscle droit de l'abdomen
Bonne(s) réponse(s) : B C

C - En arrière.
B - En dehors ; veine, puis artère.
- en dedans : le ligament de Gimbernat au bord tranchant
- en avant : l'arcade crurale.

La hernie crurale externe :


A - Se rencontre souvent chez le jeune enfant
B - Est une hernie congénitale
C - Est la forme anatomo-clinique qui se complique le plus d'étranglement
D - Se rencontre surtout chez l'homme
E - Peut être oblique externe
Bonne(s) réponse(s) : C

C - Ce fait explique que l'indication opératoire d'une hernie crurale soit formelle.

Dans le cas clinique proposé la conduite pratique comportera une des attitudes suivantes :
A - La mise en place d'une sonde naso-gastrique et une surveillance
B - Une intervention chirurgicale immédiate
C - Une intervention après 48 heures de préparation
D - Une tentative de réduction par manoeuvres externes
E - Une laparotomie exploratrice par voie médiane à cheval sur l'ombilic
Bonne(s) réponse(s) : B

C'est une urgence chirurgicale, expliquée par le risque de nécrose du grêle étranglé.

Parmi les propositions techniques suivantes, laquelle est la plus adaptée au cas clinique présenté ?
A - Abaissement conjoint-arcade crurale par voie inguinale (Bassini)
B - Pièce de Mersilène par voie inguinale
C - Abaissement conjoint Cooper par voie inguinale (Mac Vay)
D - Fermeture de l'orifice crural par voie crurale
E - Fermeture de l'orifice inguinal profond par voie inguinale
Bonne(s) réponse(s) : D

Il s'agit d'une hernie crurale étranglée habituelle, justifiant d'un traitement classique.

Monsieur X..., 49 ans, consulte pour une perte de poids récente (15 kg en trois mois), une anorexie et des douleurs
abdominales. Il s'agit de douleurs de siège épigastrique, irradiant dans l'hypochondre gauche et dans le dos. Ces douleurs ont
une intensité forte, elles sont continues, sans paroxysme. Elles sont diurnes et nocturnes. Il n'y a pas de position antalgique.
Le patient boit régulièrement 90 g d'alcool par jour depuis 15 ans, et fume régulièrement deux paquets de cigarettes par jour.
A l'examen clinique, il existe un point douloureux sous-costal gauche. Le malade est ictérique. Il signale que ses urines sont
foncées et ses selles décolorées. Le reste de l'examen clinique est normal. Les transaminases, et l'électrophorèse des
protéines sont normales. La bilirubine totale est à dix fois le taux normal. Le TP est à 60 %.

Le type sémiologique des douleurs décrites dans ce tableau est celui des douleurs :
A - De la colique hépatique
B - Du syndrome ulcéreux, duodénal
C - Du syndrome ulcéreux par maladie de Zollinger-Ellison
D - De la pancréatite
E - De la colique néphrétique gauche
Bonne(s) réponse(s) : D

Le caractère sémiologique le plus typique de douleurs pancréatiques est l'irradiation transfixiante dans le dos.

70
Exclusivement sur DOC - DZ : www.doc-dz.com NADJI 85
RESIDANAT EN POCHE TOME II
Cas Clinique en QCM

Parmi les examens paracliniques suivants, quel est celui qui permet en premier lieu de préciser le mécanisme
de l'ictère ?
A - Biopsie hépatique
B - Transit gastro-duodénal
C - Cholangio-wirsungographie endoscopique
D - Cholangiographie intra-veineuse sous perfusion
E - Echotomographie hépatobiliaire et pancréatique
Bonne(s) réponse(s) : E

L'échographie hépatobiliaire permet de distinguer les ictères avec voie biliaire principale dilatée (obstacle biliaire extra-
hépatique) et les ictères avec voie biliaire principale non dilatée (en général, ictère dont l'origine est une maladie hépatique).

L'aspect décoloré des selles chez ce malade est expliqué par :


A - Une diminution de la concentration des sels biliaires dihydroxylés dans les selles
B - Une diminution de la concentration de l'acide cholique dans les selles
C - Une augmentation du stercobilinogène fécal
D - Une maldigestion lipido-protidique
E - La diminution du stercobilinogène fécal
Bonne(s) réponse(s) : E

L'ictère cholestatique explique la diminution du stercobilinogène fécal en raison de l'absence de bilirubine dans la lumière
intestinale.

L'abaissement du taux de prothrombine est vraisemblablement secondaire à :


A - L'éthylisme chronique
B - L'augmentation de la la bilirubine conjuguée dans le sang
C - L'amaigrissement de 15 kg en trois mois
D - Une carence alimentaire
E - La cholestase extra hépatique
Bonne(s) réponse(s) : E

Le mécanisme en est une malabsorption de vitamine K en raison de la carence d'acides biliaires dans la lumière digestive.

Une femme de 50 ans, consulte pour l'apparition depuis 6 mois de crampes abdominales et de diarrhées post-prandiales
impérieuses. L'interrogatoire note par ailleurs la notion d'une perte de poids inférieure à 2 kg, d'une ménopause récente avec
bouffées de chaleur. Pas d'altération de l'état général mais il existe une hépatomégalie ferme.
Examens complémentaires :
Hémogramme :
- hémoglobine 130 g/l
- érythrocytes 4 x 10 exposant 6/mm3
- calcémie 100 mg/l
- VGM 85 micro 3
- albuminémie 40 g/1
Poids des selles 400 g/24 h; le temps de transit oro-anal du carmin se fait en 4 heures,
Les examens complémentaires ont conclu à un syndrome carcinoïdien,

Quels sont les 2 examens complémentaires qui vont permettre chez cette patiente d'orienter le diagnostic ?
A - Gastrinémie
B - Sérotoninémie
C - 5 HIAA urinaire
D - Clairance de l'alpha-1-antitrypsine
E - Breath-test au glycocholate marqué
Bonne(s) réponse(s) : B C

Les tumeurs carcinoïdes se caractérisent par des taux de 5 HIAA urinaires (métabolite terminal de la sérotonine) supérieurs à
10 mg/24 heures (52 µ mol/24 heures), de sérotonine sanguine supérieurs à 0,25 mg/l (1,42 µ mol/l). Le dosage de la
sérotonine sanguine est d'un meilleur apport diagnostique ; il existe en effet des tumeurs carcinoïdes, souvent débutantes, où
seule la sérotonine sanguine est élevée.

71
Exclusivement sur DOC - DZ : www.doc-dz.com NADJI 85
RESIDANAT EN POCHE TOME II
Cas Clinique en QCM

Quelle est la localisation la plus fréquente des tumeurs carcinoïdes avec syndrome carcinoïdien ?
A - Estomac
B - Duodénum
C - Grêle
D - Colon
E - Appendice
Bonne(s) réponse(s) : C

Les carcinoïdes appendiculaires sont les plus fréquents et sont découverts au cours de 0,6 p.100 des appendicectomies, mais
ils ne s'associent qu'exceptionnellement à un syndrome carcinoïde. L'intestin grêle représente la localisation principale de la
tumeur associée au syndrome carcinoïde.

Quel est chez cette patiente le premier examen à réaliser à la recherche de métastases ?
A - Artériographie coelio-mésentérique
B - Radio du crâne
C - UIV
D - Echographie abdominale
E - Scanner abdominal
Bonne(s) réponse(s) : D

L'échographie et la tomodensitométrie sont les méthodes de choix pour la recherche de métastases. Le premier examen à
réaliser, du fait de sa simplicité, est l'échographie.

Les 2 anomalies biologiques évocatrices de métastases hépatiques chez cette patiente sont l'augmentation de :
A - Transaminases
B - 5-nucléotidases
C - Phosphatases alcalines
D - Gamma GT
E - Bilirubine conjuguée
Bonne(s) réponse(s) : C D

L'élévation du taux de Gamma GT et des phosphatases alcalines sont, dans l'ordre, les anomalies biologiques les plus
sensibles évocatrices de métastases hépatiques.

Les diarrhées motrices sont caractérisées par :


A - Des selles graisseuses
B - L'absence de retentissement sur l'état général
C - La présence dans les selles de débris végétaux
D - L'échec des ralentisseurs du transit
E - Le caractère post-prandial
Bonne(s) réponse(s) : B C E

Les principaux caractères des diarrhées motrices sont : le nombre élevé d'émissions contrastant avec un volume fécal
quotidien modéré ; l'horaire matinal et post prandial précoce des selles ; leur caractère souvent impérieux précédé par des
coliques ; la présence dans les selles de débris végétaux ; l'efficacité des ralentisseurs du transit ; l'absence habituelle de
retentissement sur l'état général.

Les causes des diarrhées d'origine endocrinienne comportent :


A - L'hyperthyroïdie
B - Le syndrome de Zollinger-Ellison
C - Les phéochromocytomes
D - Le syndrome de Verner Morrisson (choléra pancréatique)
E - Le cancer médullaire de la thyroïde
Bonne(s) réponse(s) : A B D E

La principale étiologie des diarrhées d'origine endocrinienne est l'hyperthyroïdie. Plus rarement en cause sont les tumeurs
carcinoïdes (sécrétion de sérotonine) et le syndrome de Zollinger-Ellison (hypersécrétion de gastrine). Le syndrome de Verner-
Morrisson (sécrétion de VIP) et le cancer médullaire de la thyroïde (thyrocalcitonine) sont exceptionnels.

72
Exclusivement sur DOC - DZ : www.doc-dz.com NADJI 85
RESIDANAT EN POCHE TOME II
Cas Clinique en QCM
Une jeune fille de 18 ans est amenée par son fiancé alors qu'elle vient d'ingérer, dans un but de suicide, un produit caustique
utilisé pour déboucher les éviers. Elle est agitée, dyspnéique et légèrement cyanosée. Elle a une sialorrhée hémorragique et
des douleurs thoraciques. Son pouls est à 120/min, sa tension artérielle à 10/7 cm Hg, sa température est normale.

A ce stade, l'attitude thérapeutique comporte :


A - Une trachéotomie
B - Une transfusion sanguine
C - Des antalgiques
D - Une aspiration gastrique
E - Une oxygénothérapie
Bonne(s) réponse(s) : C E

L'attitude thérapeutique à la phase initiale, dépend du degré de gravité, la constatation :


- d'une absorption d'un verre (150 ml) ou davantage d'acide fort ou de Destop®
- de troubles psychiques confusionnels
- de troubles de l'hémostase avec CIVD
- d'une dyspnée liée à une acidose (voir la radio-pulmonaire)
- d'une perforation de l'oesophage (pneumomédiastin) ou de l'estomac (pneumopéritoine).
amènent rapidement le patient en salle d'opération.

Le(s) geste(s) à éviter inclue(nt) :


A - Faire vomir
B - Lavage gastrique
C - Donner des pansements gastriques
D - Neutraliser le caustique par de l'HCl
E - Mettre la malade demi-assise
Bonne(s) réponse(s) : A B C D E

Tous gestes susceptibles d'aggraver les lésions ou de gêner l'exploration endoscopique.

On doit rechercher :
A - Un emphysème sous-cutané
B - Une contracture abdominale
C - Une diarrhée sanglante
D - Des râles bronchiques
E - Un oedème laryngé
Bonne(s) réponse(s) : A B D E

A - Témoin d'un pneumomédiastin par perforation oesophagienne.


B - Témoin d'une perforation gastrique.
D - Témoins d'une pneumopathie d'inhalation associée, ou d'une fistule trachéo-bronchique.
E - Le simple oedème laryngé répondra aux corticoïdes ; une destruction du carrefour obligera à une trachéotomie difficile.

L'endoscopie digestive haute en urgence :


A - Est utile aux décisions thérapeutiques
B - Ne doit pas dépasser l'oesophage
C - A pour but de rechercher une perforation
D - Permet de dilater l'oesophage
E - Sert à évacuer le caustique
Bonne(s) réponse(s) : A E

L'endoscopie haute est indispensable et doit être réalisée en urgence. Elle permet l'évaluation des lésions : de l'érythème et
des pétéchies, à la nécrose, en passant par les ulcérations (et peut servir à évacuer le produit caustique restant le cas
échéant dans l'estomac).

Le traitement est assuré par :


A - Un régime sans résidus
B - Un régime lacté
C - Un régime pauvre en protéines
D - Une gastrostomie d'alimentation
E - Une nutrition parentérale totale
Bonne(s) réponse(s) : E

E - Est de toute façon indispensable, qu'une indication opératoire soit posée ou non, mais elle pourra être remplacée par une
nutrition entérale par jéjunostomie.

73
Exclusivement sur DOC - DZ : www.doc-dz.com NADJI 85
RESIDANAT EN POCHE TOME II
Cas Clinique en QCM

Le risque de sténose ultérieure de l'oesophage est d'autant plus grand que :


A - Les lésions oesophagiennes sont plus étendues
B - Les lésions oesophagiennes sont plus profondes
C - Les lésions oesophagiennes sont circonférencielles
D - Les lésions buccales initiales sont plus importantes
E - La tentative de suicide a eu lieu à jeun
Bonne(s) réponse(s) : A B C

Une corticothérapie parentale administrée d'emblée semble utile pour prévenir les sténoses sauf dans les brûlures graves ; où
elle majore les risques de perforation et d'infection.

Une femme de 28 ans, victime 1/2 heure plus tôt d'un accident d'automobile, arrive à l'hôpital conduite par le SAMU. Elles est
pâle, couverte de sueurs, se plaint d'avoir soif, et décrit une douleur sous-costale gauche irradiant en bretelle. Il y a une
certaine défense à la palpation de l'hypocondre gauche le toucher rectal est modérément douloureux.
La patiente n'a pas d'antécédent pathologique notable, elle n'a jamais été opérée; elle a mené trois grossesses à terme, ses
trois enfants sont en bonne santé. Une carte de groupe sanguin établi à la fin de sa première grossesse porte les mentions :
"groupe A1, rhésus positif - 1ère détermination".

On évoque le diagnostic d'hémopéritoine par rupture de la rate. En dehors de l'examen local, l'appréciation de
l'importance et du retentissement général de cette hémorragie se base à l'instant de l'arrivée sur :
A - Les caractères du pouls
B - Les chiffres tensionnels
C - L'hématocrite
D - Le taux d'hémoglobine
E - Le dosage du fer sérique
Bonne(s) réponse(s) : A B

Au début d'une hémorragie, les pertes de globules rouges et de plasma étant parallèles, l'hématocrite se modifie peu ; et le
taux de l'hémoglobine ou de l'hématocrite sous-estime donc l'anémie. Ultérieurement, l'hémodilution compensatrice se
produisant, il y a chute progressive de ces chiffres. Cela explique que la gravité d'une hémorragie se détermine initialement,
essentiellement sur des critères cliniques.

Dès l'arrivée, on prélève du sang en vue de divers examens biologiques. Parmi ces examens, il faut
nécessairement inclure :
A - Une mesure isotopique de la volémie
B - Une mesure de la vitesse de sédimentation globulaire
C - Une deuxième détermination du groupe ABO et rhésus
D - Une recherche d'agglutinines irrégulières
E - Un test de Coombs direct
Bonne(s) réponse(s) : C D

En vue d'une transfusion.

Après avoir maintenu la volémie par perfusion de macromolécules, on transfuse en même temps qu'on pratique
la splénectomie. Quel type de transfusion choisissez-vous ?
A - Sang total conservé
B - Sang total frais
C - Concentrés érythrocytaires
D - Concentrés érythrocytaires et de leucocytes
E - Plasma frais congelé
Bonne(s) Réponse(s) : C

A partir du moment où la volémie est assurée, il faut compenser le déficit en globules rouges.

Aucun incident notable n'est survenu pendant l'interventio. L'opérée entre en salle de réveil avec une tension
artérielle à 14/8. Un dernier sac de sang a été branché à ce moment. Une demi-heure plus tard, surviennent de
violents frissons et une élévation thermique à 40,2°C ; la tension artérielle est à 15/9. Il ne vient pas de sang par
les drains. La diurèse est conservée. L'hypothèse la plus probable est celle :
A - D'une incompatibilité ABO
B - De la présence chez la patiente d'agglutinines irrégulières anti-érythrocytes
C - De la présence d'anticorps anti-HLA
D - D'une thrombose aiguë post splénectomie
E - D'une souillure bactérienne du sac
Bonne(s) Réponse(s) : E

Une incompatibilité ABO se manifesterait plutôt par une hypotension accompagnée d'un saignement anormal du champ
opératoire (CIVD).

74
Exclusivement sur DOC - DZ : www.doc-dz.com NADJI 85
RESIDANAT EN POCHE TOME II
Cas Clinique en QCM

Devant cet incident, il faut :


A - Contrôler les groupes HLA du donneur et du receveur
B - Recontrôler les groupes érythrocytaires du sac et de l'opérée
C - Injecter un antibiotique

D - Injecter du chlorure de potassium

E - Injecter de l'héparine

Bonne(s) Réponse(s) : B C

Tout incident au cours d'une transfusion impose, de toutes façons, l'arrêt immédiat de celle-ci et le contrôle de la compatibilité
transfusionnelle.

Une femme de 60 ans est hospitalisée pour une anémie à 60 g d'hémoglobine/l découverte devant une asthénie et dyspnée
d'effort d'apparition récente.
L'interrogatoire ne retrouve aucun antécédent, en dehors de brûlures rétrosternales anciennes et intermittentes. Il n'y a pas eu
de saignement extériorisé.
Une dysphagie récente, douloureuse est apparue depuis 8 jours. La fibroscopie gastrique révèle une oesophagite sur 5 cm,
au dessus d'une hernie hiatale.

En faveur d'un saignement chronique, vous retenez :


A - La microcytose
B - L'abaissement de la capacité totale de fixation de la sidérophiline
C - L'effondrement de la ferritinémie
D - La réticulocytose élevée
E - L'abaissement du taux de fer sérique
Bonne(s) réponse(s) : A C E

B - La CTF est élevée en cas de saignement chronqiue


D - La réticulocytose, initialement élevée au début d'un saignement s'abaisse avec l'épuisement des réserves en fer.

Parmi les aliments ou médicaments suivants, qui peuvent favoriser le reflux gastro-oesophagien et doivent donc
être déconseillés, vous retenez :
A - Anticholinergiques
B - Le café
C - Le chocolat
D - Le métoclopramide
E - Les béta-bloquants
Bonne(s) réponse(s) : A B C

A,B,C - Diminuent la pression du sphincter de l'oesophage, de même que les repas gras, l'alcool, le tabac, la théophylline, la
nifédipine, les alpha-bloquants, les bêta sitmulants, les prostaglandines, la progestérone...
D - Augmente, au contraire la pression du SIO.

Un traitement antisécrétoire est prescrit pour 3 mois. L'endoscopie de contrôle à 3 mois montre la disparition de
l'oesophagite. Mais, rapidement à l'arrêt du traitement, la symptomatologie clinique et les lésions
oesophagiennes récidivent. Les risques évolutifs possibles comportent :
A - La sténose peptique de l'oesophage
B - L'ulcère de l'oesophage
C - L'achalasie
D - Varices oesophagiennes
E - L'endobrachyoesophage
Bonne(s) réponse(s) : A B E

- Ulcère oesophagien (à la jonction muqueuse malpighienne - muqueuse cylindrique) et sténose peptique sont les
conséquences rares mais graves du reflux gastro-oesophagien (RGO).
- Le RGO joue un rôle pathogénique dans la survenue d'un endobrachy-oesophage, mais on ne sait pas pourquoi seulement
10 % des patients atteints de RGO ont un endobrachyoesophage et si le RGO est initiateur ou seulement révélateur d'une
anomalie préexistante.

75
Exclusivement sur DOC - DZ : www.doc-dz.com NADJI 85
RESIDANAT EN POCHE TOME II
Cas Clinique en QCM

Quel type d'intervention chirurgicale pourrait être proposé ?


A - La vagotomie tronculaire
B - La vagotomie suprasélective
C - L'antrectomie-vagotomie tronculaire
D - La résection du 1/3 inférieur de l'oesophage
E - La réduction chirurgicale de la hernie hiatale avec montage anti-reflux
Bonne(s) réponse(s) : E

A, B, C - La vagotomie ne règle en aucun cas le problème du reflux gastro-oesophagien, et elle risque d'altérer la vidange
gastrique.

Un homme de 65 ans, est hospitalisé pour la troisième décompensation ictéro oedémato ascitique d'une cirrhose connue
d'étiologie imprécise. Le patient est calme et obnubilé. L'examen clinique retrouve un astérixis, une ascite abondante, des
oedèmes des membres inférieurs remontant jusqu'aux bourses, un ictère franc cutanéo-muqueux, une circulation collatérale
cave inférieure, un épigastre douloureux à la palpation, un souffle cardiaque systolique 2/6. Au toucher rectal, les selles sont
noires et gluantes(melena). La température est à 37,8°C.

Quel est, parmi les suivantes, la cause de la décompensation qui est la plus probable chez ce patient ?
A - Poussée d'hépatite alcoolique aiguë
B - Carcinome hépato cellulaire
C - Hémorragie digestive
D - Septicémie à point de départ digestif
E - Thrombose portale
Bonne(s) réponse(s) : A

Il ne faut pas confondre décompensation (ictéro-ascitique) d'une cirrhose et encéphalopathie hépatique. La cause la plus
probable de décompensation est une poussée d'hépatite alcoolique aiguë ; une autre cause possible pourrait être un
carcinome hépato-cellulaire ou une thrombose portale.
C et D - Ne peuvent être responsable que d'une aggravation clinique aiguë ; et notamment d'une encéphalopathie hépatique.

Quelle est la cause la plus probable des troubles de conscience de ce patient ?


A - Hémorragie cérébroméningée
B - Méningo encéphalite tuberculeuse
C - Bas débit cérébral
D - Encéphalopathie hépatique
E - Encéphalopathie hyperazotémique
Bonne(s) réponse(s) : D

L'encéphalopathie hépatique ayant pour cause déclenchante une hémorragie digestive (méléna) et vraisemblablement une
infection (37°8) (infection du liquide d'ascite au premier chef).

Quel examen s'impose en urgence ?


A - Ponction lombaire
B - Echographie abdominale
C - Fibroscopie oeso-gastro-duodénale
D - Coloscopie
E - Scanner cérébral
Bonne(s) réponse(s) : C

Le méléna témoigne d'une hémorragie digestive ; la fibroscopie oeso-gastro-duodénale doit en déterminer l'origine (la rupture
de V.O. est la plus probable).

Une ponction d'ascite a été faite. Quel(s) examen(s) biochimique(s) demander pour avoir une orientation
étiologique vers une pathologie néoplasique ?
A - Amylase
B - Protides
C - Mesure du pH de l'ascite
D - Cytologie
E - Acide lactique
Bonne(s) réponse(s) : D

L'analyse du liquide d'ascite est en fait de peu d'intérêt pour le diagnostic de greffe d'un hépatocarcinome sur cirrhose, sauf
lorsqu'elle révèle un liquide hémorragique.

76
Exclusivement sur DOC - DZ : www.doc-dz.com NADJI 85
RESIDANAT EN POCHE TOME II
Cas Clinique en QCM
Une femme de 36 ans consulte pour une pâleur apparue depuis un mois associée à des douleurs abdominales. Aucun
antécédent médical. Elle fume depuis plusieurs années 20 cigarettes par jour. Depuis 8 mois elle prend de l'aspirine par
intermittence pour des douleurs dentaires. Les douleurs abdominales ont les caractères d'une douleur ulcéreuse. L'examen
clinique est normal hormis une pâleur et la découverte d'une denture en mauvais état.
Hémogramme :
- 4.18 10 exposant 12/l GR
- Hb 88 g/l
- VGM 71 fl
- Formule leucocytaire normale
- fer sérique : 5 micromol/l (N = 9 à 30 micromol/l)
- coefficient de saturation de la sidérophiline à 8 %
- la fibroscopie gastrique objective un ulcère de la petite courbe de 1 cm de diamètre d'aspect bénin ;10 biopsies montrent
l'absence de signe de malignité
Le diagnostic d'ulcère gastrique compliqué d'anémie ferriprive par saignement chronique est porté.

Quel(s) traitement(s) anti-anémique(s) prescrivez-vous ?


A - Transfusion de 1 litre de sang total
B - Transfusion de 2 culots globulaires
C - Traitement martial par fer ferreux per os
D - Traitement martial par fer ferrique l.M.
E - Vitaminothérapie B12
Bonne(s) réponse(s) : C

L'anémie étant bien tolérée, transfuser cette jeune femme serait une faute professionnelle.

Quel(s) conseil(s) hygiéno-diététique(s) donnez-vous, à cette patiente ?


A - Suppression de l'aspirine
B - Régime lacté
C - Soins dentaires
D - Arrêt du tabac
E - Repas fractionnés
Bonne(s) réponse(s) : A C D

Ces conseils, notamment la suppression de l'aspirine et l'arrêt du tabac, sont essentiels pour éviter les récidives ulcéreuses
après traitement.

Parmi les traitements suivants, lequel ou lesquels est (sont) actif(s) dans l'ulcère gastrique ?
A - Cimétidine 800 mg par jour
B - Sulfate d'atropine 2 g par jour
C - Métoclopramide 30 mg/j
D - Ranitidine 300 mg par jour
E - Sucralfate 4 g par jour
Bonne(s) réponse(s) : A D E

L'oméprazole est également un traitement dont l'efficacité est remarquable ; aux doses de 40 mg/jour ou 20 mg/jour, en
comparaison avec 150 mg x 2/jour de ranitidine ; on a les pourcentages respectifs de cicatrisation suivants :
- à 4 semaines : 80 %, 69 %, 59 %
- à 8 semaines : 96 %, 89 %, 85 %.

Quelle surveillance allez-vous proposer vis-à-vis de l'ulcère ?


A - Fibroscopie à 6 mois avec biopsies
B - Transit radiologique gastrique à 6 semaines
C - Fibroscopie à 6 semaines avec biopsies
D - Recherche de sang dans les selles
E - Surveillance clinique
Bonne(s) réponse(s) : C

Une fibroscopie de contrôle est indispensable après traitement d'un ulcère gastrique, pour ne pas méconnaître un cancer à
forme ulcéreuse.

77
Exclusivement sur DOC - DZ : www.doc-dz.com NADJI 85
RESIDANAT EN POCHE TOME II
Cas Clinique en QCM

Parmi les éléments suivants observés dans l'évolution de cette malade, lesquels ou lequel amènerai(en)t à
poser une indication chirurgicale ?
A - Persistance de l'ulcère à 6 mois malgré un traitement bien appliqué
B - Perforation gastrique
C - Présence d'un carcinome intra-muqueux sur les biopsies
D - Présence d'une gastrite atrophique
E - Association à un reflux duodéno-gastrique
Bonne(s) réponse(s) : A B C

B et C - L'indication opératoire est évidente.


A - L'indication est posée en raison de l'échec du traitement médical, et pour ne pas laisser évoluer une éventuelle lésion
dysplasique.

Un homme de 45 ans, sans antécédents personnels, mais avec des antécédents familiaux de cancer colique, bénéficie d'une
coloscopie à titre systématique Un polype, dont la bénignité est confirmée à l'examen histologique, est enlevé à l'anse
diathermique. Dix ans après, une nouvelle coloscopie motivée par des rectorragies met en évidence un cancer de l'angle
colique gauche qui sera traité chirurgicalement. Il s'agit d'un adénocarcinome classé Dukes C.

Parmi les facteurs de risques de cancer colique reconnus, vous incluez :


A - Tabagisme
B - Travail dans l'amiante
C - Antécédents familiaux de cancer colique
D - Antécédents personnels de polype
E - Aucun facteur de risque n'est clairement identifié
Bonne(s) réponse(s) : C D

Plusieurs études ont montré la prévalence familiale des cancers du colon et du rectum. Une étude américaine a montré
notamment que le risque de polype était de 21 % dans la fratrie des cancers coliques (étude faite sur trois générations) contre
9 % dans le groupe témoin.

Quelle surveillance aurait dû être faite, le cas échéant, après exérèse du polype ?
A - Aucune surveillance n'est utile
B - Examen clinique tous les 6 mois
C - Echographie abdominale tous les ans
D - Coloscopie tous les ans ou les 2 ans
E - Dosage d'ACE tous les 3 mois
Bonne(s) réponse(s) : D

Après détection et exérèse de polypes coliques, une coloscopie doit être prévue tous les ans, et ceci tant que des polypes
sont retrouvés. Après la première coloscopie de contrôle normale, les coloscopies suivantes sont faites tous les 3, puis tous
les 5 ans, à condition de ne pas retrouver de nouveaux polypes.

Que signifie Dukes C ?


A - Tumeur limitée à la sous-muqueuse
B - Envahissement de la musculeuse
C - Dépassement de la musculeuse
D - Ganglions Iymphatiques épicoliques métastatiques
E - Métastases à distance
Bonne(s) réponse(s) : D

A - Dukes A.
B - Dukes B ; B1 d'Astler Coller.
C - Dukes B ; B2 d'Astler Coller.

Existe-t-il, à ce stade et pour cette localisation un traitement complémentaire dont l'efficacité est démontrée ?
A - Radiothérapie du lit tumoral
B - Radiothérapie hépatique
C - Chimiothérapie par fluoro-uracile
D - Immunothérapie
E - Aucun
Bonne(s) réponse(s) : C

Une étude publiée récemment, portant sur 1296 patients opérés d'un cancer du colon au stade B2 ou C, a montré qu'un
traitement chimiothérapique associant 5 FU et Lévamisole pendant 1 an diminuait la mortalité de 33 % et le taux de récidives
de 41 % avec une médiane de survie de 3 ans, pour les stades C.

78
Exclusivement sur DOC - DZ : www.doc-dz.com NADJI 85
RESIDANAT EN POCHE TOME II
Cas Clinique en QCM

Une métastase unique est diagnostiquée au niveau du lobe gauche du foie. Quel traitement proposez-vous ?
A - Hépatectomie gauche
B - Abstention
C - Chimiothérapie par voie générale
D - Chimiothérapie par voie locale
E - Radiothérapie à visée antitumorale
Bonne(s) réponse(s) : A

Ce traitement augmente en effet la médiane de survie, de façon significative.

Une femme âgée de 45 ans, en bonne santé apparente vient consulter pour l'apparition récente de rectorragies de sang rouge
de petite abondance, émise à la fin des selles. Elle n'a aucune douleur anale. Elle a par ailleurs une constipation de très
longue date alternant avec la diarrhée et associée à de fréquentes douleurs abdominales et à des ballonnements. Elle
consomme régulièrement des laxatifs qui déclenchent souvent les douleurs et les diarrhées. Elle apporte un lavement baryté
datant de trois ans où l'on ne voit aucune anomalie dans le colon plein de baryte (technique standard) en dehors de quelques
diverticules du sigmoïde. L'abdomen est normal à l'examen physique. A l'examen proctologique on observe des hémorroïdes
non procidentes et une anite rouge. La rectoscopie met en évidence un polype de 5 mm situé à 6 cm de la marge anale : le
reste de la muqueuse est normal jusqu'à 16 cm de la marge anale.

Le diagnostic de colopathie fonctionnelle est évoqué dans ce cas en raison de :


A - L'existence de douleurs abdominales
B - L'existence de troubles du transit
C - La présence de diverticules sigmoïdiens
D - La présence d'hémorroïdes
E - La présence d'un polype rectal
Bonne(s) réponse(s) : A B

C'est surtout l'ancienneté de ces troubles qui fait évoquer ce diagnostic.

L'émission de sang rouge par l'anus peut être dû chez cette patiente à :
A - L'anite hémorroïdaire
B - Un cancer du sigmoïde
C - La diverticulose sigmoïdienne
D - Une recto-colite hémorragique
E - Une iléite terminale
Bonne(s) réponse(s) : A B C

Le polype rectal, quant à lui, est trop petit (5 mm), pour se compliquer de rectorragies.
Les causes les plus probables sont A ou LB ; les diverticules sigmoïdiens sont très rarement à l'origine de rectorragies qui
sont alors très abondantes.

A cause des rectorragies, vous prescrivez à cette patiente, une fois connu le résultat de la rectoscopie :
A - Une artériographie de l'artère mésentérique inférieure
B - Une consultation auprès d'un chirurgien
C - Une coloscopie
D - Une radio barytée de l'intestin grêle
E - Une recherche de parasites dans les selles
Bonne(s) réponse(s) : C

La coloscopie est indispensable pour rechercher une cause aux rectorragies (cancer colique ?) et pour dépister et enlever
d'autres polypes.

Les symptômes de la colopathie fonctionnelle seront améliorés, chez cette patiente par :
A - L'exérèse des diverticules du colon sigmoïde
B - L'exérèse du polype rectal
C - Une normalisation du transit intestinal
D - Des anti-spasmodiques musculotropes
E - Le traitement des hémorroïdes
Bonne(s) réponse(s) : C D

Cette régularisation du transit, très importante à obtenir dans les colopathies fonctionnelles avec constipation, doit exclure
l'utilisation par la patiente de laxatifs agressifs pour la muqueuse colique.

79
Exclusivement sur DOC - DZ : www.doc-dz.com NADJI 85
RESIDANAT EN POCHE TOME II
Cas Clinique en QCM

Chez cette patiente, la constipation peut être aggravée par la prise de :


A - Sirop de codéine
B - Gel d'alumine
C - Anti-histaminique H2
D - Gel de magnésium
E - Clomipramine (Anafranil® )
Bonne(s) réponse(s) : A B E

Sans commentaire.

Le diagnostic de cancer colique est exclu chez cette patiente à cause de :


A - La conservation d'un bon état général
B - Le lavement baryté normal qu'elle à apporté
C - La rectoscopie normale
D - L'examen physique normal
E - Aucune de ces propositions
Bonne(s) réponse(s) : E

Chez cette patiente, il faut considérer qu'il existe un cancer colique jusqu'à preuve du contraire !...

Une femme de 42 ans sous contraception orale, en bon état général, sans antécédent notable en particulier abdominal,
présente en quelques heures : Une douleur aiguë de l'hypochondre droit à irradiation vers la pointe de l'omoplate, un
subictère, une fièvre à 38°5 sans frissons, des vomissements, un arrêt des gaz, L'examen clinique retrouve une douleur
provoquée de l'hypochondre droit et une douleur à la décompression de la fosse iliaque droite. Les valeurs d'amylases
sanguines et urinaires sont normales.

Vous évoquez par priorité un diagnostic. Lequel ?


A - Cholécystite aiguë
B - Pancréatite oedémateuse
C - Appendicite sous hépatique
D - Adénome hépatique fissuré sous pilule
E - Ampullome vatérien
Bonne(s) réponse(s) : A

Sans commentaire.

Parmi les examens paracliniques cités ci-dessous, lequel vous paraît le plus à même de confirmer le diagnostic
évoqué d'emblée ?
A - Numération formule sanguine
B - Dosage de la calcémie
C - Clichés d'abdomen sans préparation
D - Echographie vésiculaire
E - Cholangiographie intraveineuse
Bonne(s) réponse(s) : D

Sans commentaire.

L'examen physique vous a donc révélé l'existence d'une douleur à la décompression localisée en fosse iliaque
droite. Quelle conclusion en tirez-vous ?
A - C'est à coup sûr une appendicite
B - C'est un début de péritonite diffuse quelle qu'en soit l'origine
C - C'est la conséquence de la distension caecale par arrêt des matières et des gaz
D - L'existence d'une coulée de nécrose pancréatique rétro-péritonéale est probable
E - Une laparoscopie est souhaitable dans les 24 heures
Bonne(s) réponse(s) : B

Sans commentaire.

80
Exclusivement sur DOC - DZ : www.doc-dz.com NADJI 85
RESIDANAT EN POCHE TOME II
Cas Clinique en QCM

Parmi les examens paracliniques suivants, quel est celui qui est inutile et dangereux dans le cas présenté ?
A - Dosage antigène carcino-embryonnaire
B - Dosage de l'alpha-foeto-protéine
C - Cholangiographie endoscopique rétrograde
D - Cholangiographie intraveineuse
E - Cholécystographie orale
Bonne(s) réponse(s) : C

Sans commentaire.

Quelle attitude pratique adoptez-vous ?


A - Laparotomie
B - Antibiothérapie
C - Sphinctérotomie endoscopique
D - Surveillance clinique et paraclinique
E - Intervention dans les 48 heures
Bonne(s) réponse(s) : A

Sans commentaire.

Une femme de 70 ans, obèse, hypertendue artérielle avec ulcère variqueux droit est hospitalisée en urgence avec
vomissements, arrêt des matières et des gaz, sans fièvre. Appendicectomie 25 ans plus tôt.
Examen : Petite masse douloureuse dans la région crurale droite, irréductible et non impulsive à la toux.

Vous retenez comme preuve(s) de l'étranglement d'une hernie crurale :


A - Petite masse arrondie dans la région crurale
B - Non impulsive à la toux
C - Irréductible
D - Pulsatile
E - Gargouillante
Bonne(s) réponse(s) : A B C

Le syndrome occlusif avec vomissements, arrêt des matières et des gaz, complète le tableau clinique.

Dans l'hypothèse d'une hernie crurale étranglée, le siège anatomique du sac herniaire se situe :
A - En dedans des vaisseaux fémoraux
B - Au contact direct de l'artère fémorale
C - Au dessous de l'arcade crurale
D - Au dessous du trou obturateur
E - En dehors du nerf crural
Bonne(s) réponse(s) : A C

Il s'agit ici d'une forme habituelle où la hernie est engagée dans l'anneau crural, entre l'arcade crurale en avant, le ligament de
Gimbernat en dedans, le ligament de Cooper en arrière et les vaisseaux fémoraux en dehors. Le sac herniaire est alors situé
en-dessous de l'arcade crurale.

Parmi les attitudes thérapeutiques suivantes, quelle est celle qui s'impose dans l'immédiat :
A - Cure chirurgicale
B - Réduction par manipulation simple
C - Application d'une vessie de glace
D - Abstention thérapeutique
E - Perfusion de prostigmine
Bonne(s) réponse(s) : A

Une hernie étranglée est une urgence chirurgicale.

81
Exclusivement sur DOC - DZ : www.doc-dz.com NADJI 85
RESIDANAT EN POCHE TOME II
Cas Clinique en QCM

Parmi les gestes chirurgicaux qu'on peut être amené à pratiquer, il y a une place pour :
A - Réduction de la hernie et fermeture de l'orifice
B - Résection du grêle et fermeture de l'orifice
C - Refoulement du sac sans ouverture
D - Résection épiploïque
E - Laparotomie exploratrice
Bonne(s) réponse(s) : A B D

A - Si le grêle contenu dans la hernie est indemne.


B - Si le grêle est nécrotique ou reste douteux.
C - A ne jamais pratiquer...
D - En cas de fragment épiploïque contenu dans la hernie.
E - Inutile, car le diagnostic est certain.

Monsieur X., 59 ans, aux antécédents d'infarctus du myocarde il y a 3 ans, est hospitalisé à la suite d'une première
hémorragie digestive (hématémèse + melaena) avec chute tensionnelle (systolique à 80 mm/Hg).
G.R. 2,38 x 10 exposant 6/mm3 ; Hb. 7g/100 ml ; hématocrite 26 %
On note un éthylisme à deux litres de vin à 10 degrés par jour, depuis plus de 20 ans. Pas de facteurs déclenchants; pas de
douleurs épigastriques. A l'admission, l'examen met en évidence un ictère conjonctival modéré, un gros foie dur à bord
inférieur tranchant, dépassant le rebord costal sur la ligne médio-claviculaire et la ligne médiane de 12 cm. Il n'y a pas de
splénomégalie, mais légère poussée ascitique.
L'oesogastroscopie montre que le saignement était lié à une rupture de varice oesophagienne du tiers inférieur de
l'oesophage. Bon état de conscience. Pas d'astérixis.
La prothrombine est à 50%, l'albuminémie à 25 g/l ; la bilirubinémie totale à 54 micromol/l (normale = 17) ; phosphatases
alcalines, ALAT et ASAT sont normales.

Sur quel(s) argument(s) évoque-t-on le diagnostic de cirrhose ?


A - Ethylisme à 2 litres par jour de vin à 10 degrés depuis plus de 20 ans
B - Gros foie dur à bord inférieur tranchant
C - La bilirubinémie totale à 54 micromol/l
D - La prothrombine à 50 %
E - La présence de varices oesophagiennes
Bonne(s) réponse(s) : A B E

C et D - Ne sont pas des arguments car ils manquent trop de spécificité.

Le traitement immédiat doit nécessairement inclure :


A - Une hyperalimentation parentérale exclusive
B - La prévention du coma post-hémorragique
C - Une transfusion de plasma associée à des concentrés de globules rouges
D - Une antibiothérapie de principe
E - Une vitaminothérapie K parentérale
Bonne(s) réponse(s) : B C

E - Une vitaminothérapie K n'a pas d'indication en l'absence de déficit en vitamine K ; cependant elle peut être réalisée de
principe, en attendant les résultats de l'hémostase complète.

Si l'hémorragie récidive; vous pouvez être amené à utiliser chez ce malade :


A - La somatostatine
B - La sonde de Blakemore
C - La sclérose de varices oesophagiennes
D - L'anastomose porto cave d'urgence
E - Les antihistaminiques H2
Bonne(s) réponse(s) : A B C

La sclérose de varices oesophagismes en urgence est la méthode la plus efficace. Si elle ne peut être réalisée en urgence
dans de bonnes conditions, on a le choix entre sonde de Blakemore, ou selon les équipes, somatostatine ou vasopressine.

82
Exclusivement sur DOC - DZ : www.doc-dz.com NADJI 85
RESIDANAT EN POCHE TOME II
Cas Clinique en QCM

Après arrêt de l'hémorragie vous décidez de mettre ce malade à un traitement médical préventif de la récidive
hémorragique à long terme. Le médicament à prescrire est :
A - La somatostatine
B - La vasopressine
C - Le propranolol
D - Le polidocanol
E - Le lactulose
Bonne(s) réponse(s) : C

En fait, le traitement prophylactique des secondes récidives d'hémorragies digestives par rupture de varices oesophagisme
est au mieux représenté par les scléroses endoscopiques des varices oesophagiennes. Le propranolol a montré une efficacité
supérieure au traitement par scléroses endoscopiques seulement en prophylaxie primaire.

Vous voulez classer ce sujet selon CHILD. Pour ce faire, vous avez besoin de :
A - La taille des varices oesophagiennes
B - La bilirubinémie
C - L'albuminémie
D - La présence ou non d'encéphalopathie
E - La pression portale
Bonne(s) réponse(s) : B C D

La sévérité et le pronostic d'une cirrhose est grossièrement évaluée à l'aide de la classification de Child-Pugh prenant en
compte la bilirubine, l'albumine, la présence d'une ascite, d'une encéphalopathie et le T.P.

Une femme de 66 ans, en bonne santé apparente vient consulter pour une rectorragie de sang rouge accompagnant des
selles ou un peu de glaire, apparue et se reproduisant depuis 1 mois environ. On apprend qu'elle a subi, il y a 6 ans, une hémi-
colectomie droite en urgence pour une occlusion due à un adénocarcinome du côlon ascendant. A la suite de l'intervention,
elle a souffert d'une diarrhée faite de 4 à 6 selles liquides par jour. La diarrhée a été très améliorée par la prise de
cholestyramine (Questran®) sur prescription du service de chirurgie à la sortie de l'hôpital. Après sa sortie du service de
chirurgie, elle n'a cependant consulté son médecin de famille que pour faire renouveler ses ordonnances de Questran. Elle n'a
pas revu le chirurgien et n'a pas consulté un gastro-entérologue. L'examen physique est normal y compris le toucher rectal.

La diarrhée chronique de cette patiente est due à l'intervention chirurgicale par un ou plusieurs mécanismes
suivants :
A - Reflux des germes dans l'iléon
B - Malabsorption des sels biliaires
C - Réduction du pool des sels biliaires
D - Accélération du transit colique
E - Malabsorption des glucides
Bonne(s) réponse(s) : A B

La résection iléale entraîne une rupture du cycle entéro-hépatique des sels biliaires. Les sels biliaires non réabsorbés par
l'iléon réséqué exercent une action "irritante" au niveau du colon. Par ailleurs, la suppression de la valvule iléo-caecale
perturbe les conditions normales d'évacuation du grêle. On observe une accélération du transit global (et non uniquement
colique, réponse D) ; de plus, la colonisation bactérienne rétrograde du grêle a des conséquences propres sur les sels
biliaires.

La cholestyramine a agi sur la diarrhée par le ou les mécanismes suivant(s) :


A - Chélation des sels biliaires
B - Pansement intestinal
C - Réduction de la motricité du grêle
D - Désinfection intestinale
E - Augmentation de la résorption d'eau dans le grêle
Bonne(s) réponse(s) : A

La cholestyramine, administrée avant les repas, peut absorber les sels biliaires, dès leur arrivée en excès dans la dernière
colique ; elle représente ainsi un moyen thérapeutique parfaitement adapté.

83
Exclusivement sur DOC - DZ : www.doc-dz.com NADJI 85
RESIDANAT EN POCHE TOME II
Cas Clinique en QCM

Le toucher rectal est fait chez cette patiente pour rechercher une ou plusieurs anomalies suivantes :
A - Un cancer du bas-rectum
B - Un cancer du côlon sigmoïde prolabé dans le cul de sac de Douglas
C - Une carcinose péritonéale
D - Des hémorroïdes
E - Une recto-colite hémorragique
Bonne(s) réponse(s) : A B

C - Le toucher rectal permet parfois de palper des nodules de carcinose péritonéale, mais ce n'est en aucun cas un moyen de
la rechercher.
D - Le toucher rectal n'est d'aucune utilité pour le diagnostic d'hémoroïdes.

Le test à utiliser pour confirmer l'accélération du transit est :


A - Un transit baryté du grêle
B - Une radio de l'abdomen sans préparation
C - Un lavement baryté
D - Une durée du transit oro-anal du carmin
E - Une scintigraphie au technétium
Bonne(s) réponse(s) : D

Le test au carmin évalue grossièrement la durée du transit intestinal. Après ingestion de deux comprimés de carmin à 0,50 g,
une coloration rouge des selles apparaît normalement au bout de 20-24 heures et disparaît au bout de 48 heures. Ce test est
utile pour confirmer une accélération du transit, en cas de diarrhée motrice, où la coloration rouge des selles apparaît en
moins de 18 heures.

Indiquer l'examen ou les examens justifiés par les symptômes et les antécédents de cette patiente :
A - Oeso-gastro-duodénoscopie
B - Anuscopie
C - Coloscopie
D - Dosage de l'antigène carcino-embryonnaire
E - Dosage de la sérotonine sérique
Bonne(s) réponse(s) : C D

C - Coloscopie à la recherche d'une récidive tumorale ou d'un nouveau cancer.


D - Le dosage de l'ACE ne s'impose qu'en raison de l'antécédent de cancer colique.

Un homme de 67 ans, ancien éthylique, ne consommant actuellement plus d'alcool, mais ayant présenté au cours de ces
dernières années plusieurs poussées d'hépatite alcoolique aiguë, est hospitalisé pour le bilan d'un ictère. Dans ses
antécédents, on note également l'existence d'une affection coronarienne ayant été traitée pendant deux mois, il y a 5 ans, par
perhexilline (Pexid®) et maintenant prenant régulièrement du diltiazem (Tildiem®). L'ictère est apparu de façon très
progressive et évolue irrémédiablement en s'accentuant de jour en jour. Il est indolore, apyrétique et s'accompagne depuis
peu d'un prurit localisé aux mains. L'amaigrissement depuis un mois se chiffre à 5 kilogrammes. Une anorexie en particulier
pour les viandes et les graisses est présente. L'examen clinique retrouve essentiellement une vésicule tendue, déformant la
paroi abdominale à jour frisant, facilement palpable. Les premiers résultats biologiques mettent en évidence une
hyperbilirubinémie conjuguée : très élevée à 520 micromol/l, des phosphatases alcalines à 600 UI, des transaminases ASAT
(TGO) 160 UI, ALAT (TGP) 300 UI, un taux de prothrombine à 45 %, un bilan électrolytique sanguin et urinaire normal. La
numération formule montre : hémoglobine 9,5 g/100 ml, fer sérique 10 micromol/l, réticulocytes 50 000/mm3. La VS est à 50
mm à la 1ère heure.

Quel est le diagnostic que vous évoquez en premier devant ce tableau ?


A - Cirrhose éthylique décompensée
B - Obstacle sur la voie biliaire principale par cancer de la tête du pancréas
C - Hépatite cholestatique
D - Hépatite alcoolique
E - Pancréatite chronique
Bonne(s) réponse(s) : B

Un ictère apparaissant progressivement et sans rémission, sans douleur ni fièvre, avec une altération de l'état général, une
anorexie marquée, un prurit et une vésicule augmentée de volume évoque au premier chef un cancer de la tête du pancréas.

84
Exclusivement sur DOC - DZ : www.doc-dz.com NADJI 85
RESIDANAT EN POCHE TOME II
Cas Clinique en QCM

Parmi les examens suivants quel est celui qui vous paraît devoir être demandé en première intention ?
A - Echographie abdominale
B - Gastroscopie
C - Tomodensitométrie abdominale
D - Radio de l'abdomen sans préparation, debout, de face
E - Cathétérisme de la papille avec opacification rétrograde des voies biliaires
Bonne(s) réponse(s) : A

L'échographie abdominale doit être l'examen demandé de première intention devant un ictère cholestatique, afin de
déterminer si les voies biliaires sont dilatées.

La chute du taux de prothrombine peut être expliquée chez ce malade par certaines étiologies. Quelle est la
plus probable ?
A - Insuffisance hépato-cellulaire chronique
B - Malabsorption de la vitamine K secondaire à l'ictère
C - Atrophie villositaire intestinale chez un éthylique
D - Carence alimentaire simple
E - Consommation intraluminale de la vitamine K au cours d'un dysmicrobisme intestinal
Bonne(s) réponse(s) : B

La cholestase, du fait de l'absence d'acides biliaires dans la lumière intestinale, entraîne une malabsorption des graisses et
des vitamines liposolubles, dont la vitamine K.

Si ce malade n'avait pas de grosse vésicule palpable, le tableau clinique serait alors compatible avec :
A - Une cirrhose avec ictère
B - Une hépatite médicamenteuse après Pexid®
C - Des métastases hépatiques
D - Une hépatite B de rencontre
E - Un cancer de la tête du pancréas
Bonne(s) réponse(s) : E

A - Une cirrhose n'est à l'origine d'un ictère que si elle est compliquée (hépatite alcoolique ou virale, greffe carcinomateuse,
infection...).
B - La prise de Pexid a été brève, et elle est ancienne.
C - Les métastases hépatiques donnent le plus souvent une cholestase anictérique, mais ce tableau est possible.
D - Une hépatite B donnerait plus volontiers un ictère cytolytique que cholestatique.
E - Une grosse vésicule n'est pas indispensable au diagnostic de cancer du pancréas, qui resterait le diagnostic le plus
probable.

Au cours d'une poussée d'hépatite alcoolique, antérieure à l'ictère actuel quels sont les signes biologiques qui
pouvaient être présents ?
A - Elévation des Transaminases glutamiques oxalo acétiques (ASAT) à trois fois la normale
B - Hyperleucocytose
C - Chute du taux de prothrombine à moins de 10 %
D - Elévation de la bilirubine conjuguée
E - Baisse de la calcémie
Bonne(s) réponse(s) : A B D E

A B D - Sont des signes biologiques habituels de l'hépatite alcoolique.


C - Signerait une hépatite alcoolique majeure, évoluant en général vers la mort.
E - Peut se voir en raison d'une fréquente baisse de l'albuminémie.

Parmi les médicaments suivants quels sont ceux qui sont classiquement connus pour provoquer une hépatite
médicamenteuse ?
A - Sintrom®
B - Largactil®
C - Tagamet®
D - Paracétamol®
E - Tétracycline® intraveineuse
Bonne(s) réponse(s) : B D

A - Parmi les anticoagulants oraux, c'est la phénindione (pindione) qui est le plus souvent responsable d'hépatite.
B - Hépatites cholestatiques, avec manifestations allergiques.
C - Peut en fait donner une cytolyse discrète ± cholestase.
D - Hépatites cytolytiques sévères en cas d'intoxications aiguës.

85
Exclusivement sur DOC - DZ : www.doc-dz.com NADJI 85
RESIDANAT EN POCHE TOME II
Cas Clinique en QCM
Une malade de 50 ans, consulte pour un problème de constipation chronique apparue il y a plusieurs années, se traduisant
actuellement par des difficultés à la défécation avec seulement 2 selles par semaine après utilisation pratiquement
quotidienne de laxatifs à base de sennosides.
Elle se plaint de bouffées de chaleur, de céphalées fréquentes et de crampes au niveau des membres inférieurs. La
coloscopie met en évidence une mélanose recto-colique.

Quels sont les laxatifs dont l'usage excessif peut entraîner une maladie des laxatifs ?
A - Lactulose
B - Mucilages
C - Phénolphtaléine
D - Lubrifiants
E - Anthraquinones
Bonne(s) réponse(s) : C E

Les classes de laxatifs exposant à la maladie des laxatifs sont : les anthraquinones : bourdaine, séné, aloès, rhubarbe..., les
laxatifs issus du diphényl-méthane : à base de phénophtaléine ou de bisacodyl.

Quels sont les signes en faveur d'une maladie des laxatifs ?


A - Hyperkaliémie
B - Bouffées de chaleur
C - Crampes musculaires
D - Asthénie
E - Céphalées
Bonne(s) réponse(s) : C

A - L'hypokaliémie constitue un signe éventuel de la maladie des laxatifs.


B - Sont plus en rapport avec l'âge de la patiente et son statut hormonal.
C - En raison de l'hypokaliémie.
D - Trop peu spécifique pour être évocatrice.

Quels sont les signes en faveur d'une constipation terminale ?


A - Difficulté d'expulser les selles avec parfois manoeuvres digitales pour évacuer les féces
B - Sensation de défécation incomplète
C - Douleur après le passage des selles
D - Présence de matières au toucher rectal après la défécation
E - Douleurs au niveau de la fosse iliaque gauche
Bonne(s) réponse(s) : A B D

Au cours des constipations terminales, les patients éprouvent le besoin d'aller à la selle, mais ne peuvent évacuer ou
évacuent avec difficulté. A l'opposé, au cours des constipations par inertie colique, les patients n'éprouvent pas le besoin
d'aller à la selle pendant plusieurs jours.

Quels sont les médicaments susceptibles d'aggraver la constipation de cette patiente ?


A - Sirop de codéine
B - Comprimés de codéthylline
C - Pansements gastriques à base de gel d'alumine
D - Antidépresseurs
E - Pansements gastriques à base de gel de magnésium
Bonne(s) réponse(s) : A B C D

E - Les pansements gastriques à base de gel de magnésium, tendent plutôt à donner une diarrhée, à fortes doses.

Parmi les items suivants quels sont ceux qui peuvent se rencontrer dans un syndrome du périnée descendant ?
A - Rectorragies
B - Cancérisation
C - Incontinence
D - Constipation terminale
E - Prolapsus rectal
Bonne(s) réponse(s) : A C D E

Le syndrome du périnée descendant est un trouble de la statique rectale caractérisé par la descente de l'angle ano-rectal à
plus de 2 cm au-dessous de la ligne pubo-coccygienne. Il est responsable essentiellement d'une incontinence anale (C).
Fréquemment, il est associé à d'autres troubles de la statique rectale : prolapsus rectal (E), procidence interne du rectum ou
rectocèle à l'origine de constipation terminale (D) ou d'ulcère solitaire du rectum responsable de rectorragies (A).

86
Exclusivement sur DOC - DZ : www.doc-dz.com NADJI 85
RESIDANAT EN POCHE TOME II
Cas Clinique en QCM

La mélanose recto-colique :
A - Peut être provoquée par l'usage excessif des laxatifs anthraquinoniques
B - Peut être parfaitement asymptomatique
C - Peut s'accompagner de spasmes oesophagiens
D - Se rencontre surtout chez la femme
E - Est une variété de tumeur bénigne du rectum
Bonne(s) réponse(s) : A B D

La mélanose recto-colique réalise un aspect cuivré de la muqueuse, en plages plus ou moins confluentes et témoigne de la
prise régulière d'anthraquinone. Sur le plan histologique, le chorion comporte des histiocytes contenant des pigments
mélaniques et lipofuschinique. Cet aspect peut régresser à l'arrêt de l'intoxication.

Un homme de 60 ans, maigre, actif, artéritique, ayant subi il y a trois mois un pontage aorto-coronarien, actuellement sous
traitement anticoagulant, présente un épisode de rectorragie.

Quel(s) est (sont) celui(ceux) parmi ces gestes celui (ceux) qui dans une optique étiologique vous parait(ssent)
indispensable(s) ?
A - Palpation des pouls fémoraux
B - Palpation de la fosse iliaque gauche
C - Auscultation abdominale
D - Examen de la marge anale
E - Toucher rectal
Bonne(s) réponse(s) : D E

Examen de la marge anale et toucher rectal sont les gestes indispensables de l'examen clinique à la recherche d'une lésion
locale ayant saigné.

Il est diagnostiqué un cancer de l'ampoule rectale situé à 7 cm de la marge anale. L'atteinte en profondeur de la
paroi rectale sera au mieux appréciée par quelle ou quelles explorations ?
A - Toucher rectal
B - Echographie endorectale
C - Cliché de profil de l'ampoule rectale lors du lavement baryté
D - Tomodensitométrie pelvienne
E - Rectoscopie
Bonne(s) réponse(s) : A B D

L'échographie endorectale est l'examen le plus performant pour objectiver une atteinte des différentes couches de la paroi
rectale, un envahissement de la graisse périrectale ainsi que des ganglions ou des organes de voisinage. C'est un temps
essentiel du bilan des cancers du rectum. Le toucher rectal n'en donne pas moins l'importants renseignements : degré de
mobilité de la tumeur, existence d'une infiltration de la graisse avec induration périrectale...

Le bilan d'extension est négatif et une anesthésie générale acceptée par les anesthésistes. Parmi ces
interventions chirurgicales, laquelle sera vraisemblablement réalisée ?
A - Colostomie iliaque gauche
B - Amputation abdomino-périnéale du rectum
C - Résection antérieure du rectum
D - Intervention de Hartmann
E - Intervention de Bouilly-Wolkman
Bonne(s) réponse(s) : B

Question un peu discutable :


Lorsque le pôle inférieur de la tumeur est situé à plus de 8 cm de la marge anale, une résection antérieure avec
rétablissement de la continuité peut être réalisée. Lorsque le pôle inférieur de la tumeur est situé à moins de 6 cm,
l'amputation abdomino-périnéale s'impose. Entre 6 et 8 cm, en fait, on peut envisager une résection rectale suivie
d'anastomose très basse à la pince ou suivie d'anastomose colo-anale transanale (Parks), sauf si la tumeur est trop
volumineuse (mais la radiothérapie pré-opératoire peut permettre de la réduire).

87
Exclusivement sur DOC - DZ : www.doc-dz.com NADJI 85
RESIDANAT EN POCHE TOME II
Cas Clinique en QCM

L'examen anatomopathologique révèle une lésion rectale classée A dans la classification de Duke, cela signifie
que le néoplasme est :
A - Limité à la muqueuse et à la sous-muqueuse
B - Etendu dans la musculeuse
C - Etendu jusque dans la séreuse
D - Etendu jusque dans la séreuse avec ganglions péri-rectaux envahis
E - Etendu à travers la séreuse avec des ganglions péri-rectaux envahis
Bonne(s) réponse(s) : A

A - Le cancer ne dépasse pas la musculeuse. Il n'y a pas d'envahissement ganglionnaire.


B - Le cancer s'étend au-delà de la musculeuse, sans envahir les ganglions.
C - Envahissement ganglionnaire.

Un jeune homme de 23 ans, marié, un enfant, vient consulter pour un ictère d'apparition récente, de faible intensité (il avait
simplement les yeux jaunes) mais authentifié par un dosage de la bilirubine libre à 30 micromol/l. Les transaminases sont
normales, l'hémogramme et les réticulocytes également.
Dans ses antécédents il signale, au cours des 3 années précédentes, quelques épisodes plus ou moins identiques non
fébriles accompagnés d'une asthénie passagère.
Le caractère libre de la bilirubine vous fait évoquer le diagnostic de syndrome de Gilbert.

Quels sont les autres diagnostics qu'il faut savoir évoquer devant une hyperbilirubinémie non conjuguée ?
A - Hémolyse chronique
B - Stéatose hépatique
C - Maladie de Dubin Johnson
D - Maladie de Criggler-Najar
E - Maladie de Rotor
Bonne(s) réponse(s) : A C D

A - A évoquer de principe.
C - Affection rare, autosomale récessive, liée à un trouble de l'excrétion biliaire de bilirubine conjuguée. La proportion de
bilirubine conjuguée varie entre 40 et 80 %.
D - Type I : maladie des premiers jours de la vie (déficit total en glucuronyl transférase). Type II : affection rare, autosomale
dominante due à un déficit profond en glucuronyl transférase (en quelque sorte, forme très sévère de maladie de Gilbert).
E - Rare, ictère à bilirubine conjuguée (défaut de captation et de stockage de la bilirubine dans l'hépatocyte).

Quels sont les conditions qui peuvent augmenter chez ce malade cette hyperbilirubinémie ?
A - Administration d'Equanil® (méprobramate)
B - Jeûne prolongé
C - Ingestion de glucose
D - Traitement par la Néomycine®
E - Cholécystographie par voie orale
Bonne(s) réponse(s) : B

Le jeûne entraîne une élévation de la bilirubine sérique (épreuve de provocation). L'administration d'un inducteur enzymatique
comme le phénobarbital (mais aussi à un moindre degré le méprobamate) entraîne une diminution de la bilirubine sérique en
augmentant l'activité de la glucuronyl transférase existante.

Une hyperbilirubinémie non conjuguée prédominante(80 % de la bilirubine totale) peut se rencontrer au cours :
A - De la maladie de Minkowski Chauffard
B - De l'hépatite virale B
C - Des formes cholestatiques des hépatites médicamenteuses
D - Des hémolyses chroniques
E - De la cirrhose biliaire primitive
Bonne(s) réponse(s) : A D

A - Maladie hémolytique (sphérocytose).


B C E - Elévation de la bilirubine conjuguée.

88
Exclusivement sur DOC - DZ : www.doc-dz.com NADJI 85
RESIDANAT EN POCHE TOME II
Cas Clinique en QCM

La bilirubine non conjuguée liée à l'albumine :


A - Donne généralement un ictère acholurique (à urines claires)
B - Ne peut traverser la barrière méningée
C - Est filtrée par le glomérule rénal
D - Peut donner un ictère nucléaire chez les sujets de plus de 60 ans
E - Est normalement présente dans le sang
Bonne(s) réponse(s) : A E

A - Les ictères à bilirubine non conjuguée n'entraînent jamais de bilirubinurie. L'urobilirubinurie est augmentée en cas
d'hémolyse pouvant donner des urines oranges ; elle est normale en cas d'anomalie de la glucuronyl transférase.
B - La bilirubine peut passer la barrière méningée, et du fait de sa liposolubilité, pénétrer dans les cellules cérébrales du
nouveau-né exclusivement, et être ainsi à l'origine de l'ictère nucléaire.
E - La bilirubine sérique est normalement de 5 à 17 mmol/l, presque entièrement sous forme non conjuguée.

Chez cet homme jeune :


A - Un traitement par le Gardénal® à fortes doses doit être instauré et poursuivi à vie
B - Une surveillance des transaminases doit être conseillée tous les ans
C - Une évolution cirrhogène est possible
D - La ponction biopsie hépatique n'est pas nécessaire pour confirmer le diagnostic
E - On doit le rassurer sur l'absence de risque de transmission à son épouse
Bonne(s) réponse(s) : D E

A - Le retentissement de la maladie de Gilbert est trop minime pour justifier ce traitement.


D - Le diagnostic est clinique et biologique : élévation isolée de la bilirubine libre sans stigmates d'hémolyse.
E - Transmission autosomale dominante.

Une femme de 82 ans, coronarienne et insuffisante cardiaque, est hospitalisée pour un ictère apparu progressivement en 8
jours.
L'ictère a été précédé de douleurs de l'hypochondre droit, rapidement régressives Depuis 48 heures, il s'accompagne d'une
fièvre à 38°C et d'un prurit. La patiente a eu une cholecystectomie pour lithiase vésiculaire à l'âge de 65 ans. Aucun des
médicaments prescrits pour la coronarite ou l'insuffisance cardiaque chez cette patiente n'est hépatotoxique.
A l'examen : ictère franc, température : 37,8°C, foie augmenté de volume de surface régulière, de consistance normale et
indolore.
Anémie microcytaire avec hémoglobine à 10 g/dl, hyperleucocytose à 13 000/mm3 avec 80 % de polynucléaires, plaquettes à
150 000/mm3, bilirubine totale = 250 micromol/l, phosphatases alcalines = 3 fois la normale, gamma-GT = 6 x N, SGOT = 2 x
N, SGPT = 3 x N, taux de prothrombine = 65%,albumine = 30 g/l, marqueurs sériques du virus de l'hépatite B : négatifs.

Parmi les signes biologiques suivants, quels sont les 3 qui vous permettent d'affirmer la nature cholestatique
d'un ictère ?
A - Augmentation de la bilirubine conjuguée
B - Augmentation des transaminases 20 fois la valeur normale
C - Augmentation du taux sérique de la phosphatase alcaline
D - Augmentation du taux sérique de la gamma-GT
E - Diminution du facteur V
Bonne(s) réponse(s) : ACD

B - Signe une cytolyse hépatique.


E - Signe habituellement une insuffisance hépato-cellulaire.

Devant ce cas clinique, quel examen morphologique proposez-vous en premier lieu ?


A - Transit oeso-gastro-duodénal
B - Cholangiographie intra-veineuse
C - Echotomographie abdominale
D - Duodénoscopie
E - Artériographie coelio-mésentérique
Bonne(s) réponse(s) : C

L'échographie hépatique et des voies biliaires est l'examen le plus simple et le plus utile, il répondra d'abord à une question
simple : les voies biliaires sont-elles dilatées ?

89
Exclusivement sur DOC - DZ : www.doc-dz.com NADJI 85
RESIDANAT EN POCHE TOME II
Cas Clinique en QCM

Parmi les diagnostics suivants, quels sont les deux qui vous paraissent les plus probables chez cette patiente ?
A - Hépatite virale non A non B
B - Ampullome vatérien
C - Cancer de la tête du pancréas
D - Cancer secondaire du foie
E - Lithiase de la voie biliaire principale
Bonne(s) réponse(s) : BC

A - Non, car il s'agit d'un ictère cholestatique.


D - Non, car le foie est de consistance normale et car un tel ictère est rare dans ces cas.
E - Non, car la patiente a été cholecystectomisée il y a plusieurs années.

Parmi les examens suivants, quel est celui qui vous permet de confirmer une des 2 hypothèses diagnostiques ?
A - Artériographie coelio-mésentérique
B - Tomodensitométrie abdominale
C - Duodénoscopie et cholangiographie rétrograde
D - Mesure de la concentration sérique de l'alpha foeto-protéine
E - Ponction biopsie du foie
Bonne(s) réponse(s) : C

C est le seul moyen de confirmer un éventuel ampulome vatérien, en le visualisant et en effectuant des biopsies.

En l'absence de traitement, quelles sont les deux complications immédiates à craindre chez cette patiente ?
A - Angiocholite
B - Cirrhose biliaire secondaire
C - Encéphalopathie hépatique
D - Occlusion intestinale
E - Septicémie à colibacille
Bonne(s) réponse(s) : AE

En fait cette patiente présente déjà très vraisemblablement une angiocholite (infection des voies biliaires). Il est à craindre une
septicémie à colibacille et un choc septique.

Monsieur X... vient consulter pour altération de l'état général.


Vous retenez le diagnostic d'alcoolisation chronique.

Parmi les signes que révèle l'interrogatoire ou l'examen clinique, quel(s) est (sont) celui ou ceux qui est ou sont
attribuable(s) à cette alcoolisation chronique ?
A - Crampes nocturnes des membres inférieurs
B - Cauchemars
C - Dyschromatopsie
D - Tremblements des extrémités
E - "Brûlures" gastriques
Bonne(s) réponse(s) : AD

A - Signe d'une polynévrite éthylique.


B E - Non spécifique.
C - La dyschromatopsie d'une névrite optique rétro-bulbaire, est en fait plus liée au tabac qu'à l'alcool.

Le bilan biologique vous révèle une hypertriglycéridémie. Quelle mesure prenez-vous ?


A - Prescription d'un régime pauvre en graisses
B - Prescription d'un régime pauvre en hydrates de carbone
C - Prescription d'un hypolipémiant
D - Contrôle du bilan lipidique à distance du sevrage
E - Contrôle du bilan thyroïdien
Bonne(s) réponse(s) : D

Il s'agit d'une hyperlipidémie de type IV (triglycérides VLDL) liée à l'alcoolisme.


L'alcool inhibe l'oxygénation des acides gras, qui sont donc formés en excès et estérifiés pour former des triglycérides.
L'arrêt de l'intoxication éthylique doit permettre de normaliser le bilan lipidique ; il n'y a pas d'autre mesure à prendre.

90
Exclusivement sur DOC - DZ : www.doc-dz.com NADJI 85
RESIDANAT EN POCHE TOME II
Cas Clinique en QCM

Dès le premier examen, la tension artérielle (T.A.) est un peu élevée. Quelle mesure prenez-vous ?
A - Prescription d'un antihypertenseur
B - Prescription d'un régime désodé
C - Attendre 6 mois après le sevrage
D - Contrôle de la T.A. 15 jours après le sevrage
E - Prescription d'un diurétique
Bonne(s) réponse(s) : D

La constatation d'une tension artérielle élevée à une prise tensionnelle n'est pas suffisante pour débuter un traitement
antihypertenseur. Il faudra contrôler la tension artérielle à plusieurs reprises.

Si vous utilisez la grille de LE GO, qu'espérez-vous apprécier ?


A - L'état de dépendance vis-à-vis de l'alcool
B - Le retentissement clinique de l'alcoolisation chronique
C - La consommation quotidienne en boissons alcoolisées
D - Le type d'alcoolisation
E - Le retentissement fonctionnel d'une éventuelle polynévrite
Bonne(s) réponse(s) : A

Cette grille comporte 12 cases disposées sur 2 lignes. La première regroupe 6 signes objectifs : aspect du visage, conjonctive,
langue, signes du tremblement (bouche, langue, extrémités). La deuxième ligne estime les troubles subjectifs intéressant
l'appareil nerveux (insomnie, cauchemar...), les troubles digestifs (soif, brûlures gastriques...), l'appareil moteur (crampes...).
Les trois autres cases permettent de noter l'état du foie, de la T.A., du poids.
L'évolution dans le temps permet de fixer un certain nombre de stades de dépendance.

Vous recevez une femme de 72 ans qui présente une anémie : elle se plaint d'une asthénie, d'une dyspnée d'effort; elle
signale également une anorexie progressive en rapport avec de vives douleurs linguales causées par l'absorption d'aliments
chauds ou de préparations vinaigrées. Il existe un amaigrissement de 4 kilos depuis deux mois.
L'examen physique montre une décoloration cutanéo-muqueuse et un subictère conjonctival ; il n'y a aucune tuméfaction des
organes hématopoïétiques. On note une diminution de la sensibilité au chaud et froid et au diapason aux membres inférieurs.
La langue est rouge et dépapillée, d'aspect vernissé.
Vous demandez un examen de sang dont les résultats sont les suivants :
GR : 1,8 x 10 exposant 12/l Hb : 7,3 g/dl Ht : 22 %
GB : 3,4 x 10 exposant 9/l Plaquettes : 90 x 10 exposant 9/l Réticulocytes : 30 x 109/l
Fer sérique : 17 micromoles/l Saturation de la transferrine 0,38 N
Groupe sanguin : A Rh positif
Acide folique : 4 ng/ml de sérum (normale : 1 à 14)
Vitamine B12 : 100 Pg/ml de sérum (normale : 330 à 950).

Quel diagnostic évoquez-vous ?


A - Anémie hémolytique
B - Anémie ferriprive
C - Aplasie médullaire
D - Anémie de Biermer
E - Carence en folates
Bonne(s) réponse(s) : D

Il s'agit d'une anémie de Biermer typique : anémie macrocytaire arégénérative (VGM en µ3 = f(Ht;GR en millions) x 10 = 122
µ3) avec baisse des autres lignées sanguines.
La vitamine B12 est basse.
De plus, on a une clinique évocatrice avec une vraisemblable glossite de Hunter et des signes neurologiques sensitifs.

Indiquez le ou les examens dont le résultat serait modifié par une injection intempestive de vitamine B12 reçue
48 heures auparavant :
A - Volume Globulaire Moyen
B - Dosage sérique de la vitamine B12
C - Gastrinémie
D - Myélogramme
E - Test de Schilling
Bonne(s) réponse(s) : BDE

D - Les mégaloblastes peuvent disparaître en moins de 48 heures. Les myélocytes et métamyélocytes géants persistent.
E - Le test de Schilling est faussé en raison du cycle entéro-hépatique de la vitamine B12 injectée, qui est excrétée par la bile
et qui se trouve ensuite en compétition avec la vitamine B12 marquée dans l'iléon.

91
Exclusivement sur DOC - DZ : www.doc-dz.com NADJI 85
RESIDANAT EN POCHE TOME II
Cas Clinique en QCM

Dans ce cas particulier, quelle mesure thérapeutique envisagez-vous ?


A - Sels ferreux per os
B - Transfusions de concentrés érythrocytaires
C - Acide folique per os
D - Vitamine B12 per os
E - Vitamine B12 injectable
Bonne(s) réponse(s) : E

Il s'agit d'un traitement à vie ; on pourrait rédiger l'ordonnance ainsi :


Vitamine B12, 1000 gamma IM tous les jours pendant 10 jours puis une injection tous les mois de 1000 gamma IM (en fait 100
gamma suffiraient).

Sous traitement spécifique, indiquez les deux anomalies qui regresseront le plus rapidement :
A - Achlorhydrie
B - Anémie
C - Thrombopénie
D - Signes neurologiques
E - Atrophie gastrique
Bonne(s) réponse(s) : BC

A E - Ne sont pas modifiés par le traitement.


D - Les signes neurologiques régressent très lentement.

Quel examen vous semble le plus important à surveiller périodiquement ?


A,- Créatininémie
B - Fibroscopie gastrique
C - Hémogramme
D - Test de Shilling
E - Myélogramme
Bonne(s) réponse(s) : B

Une fibroscopie gastrique devra être faite régulièrement tous les 6 mois à 1 an pour dépister un cancer gastrique favorisé par
la gastrite atrophique fundique.

92
Exclusivement sur DOC - DZ : www.doc-dz.com NADJI 85
RESIDANAT EN POCHE TOME II
Cas Clinique en QCM
Une femme de 47 ans, nulligeste, jusque là normalement réglée, présente depuis 3 semaines des métrorragies génitales
persistantes d'abondance moyenne. Elle n'a aucun rapport depuis 3 mois.
L'examen clinique montre un utérus bosselé, indolore, discrètement augmenté de volume et il n y a pas de sensibilité
particulière à la mobilisation. Les culs-de-sac vaginaux sont souples et libres, sans masse latéro-utérine. L'examen au
spéculum avait montré un col sain en apparence. La température est à 38°2 C.
Le reste de l'examen somatique est normal.

Parmi les examens complémentaires suivants, quel est celui qui vous paraît immédiatement le plus utile et non
contre-indiqué ?
A - Frottis cervico-vaginaux de cytodétection
B - Biopsie d'endomètre
C - Radiographie de l'abdomen sans préparation
D - Numération - formule sanguine et taux d'hémoglobine
E - Hystérosalpingographie
Bonne(s) réponse(s) : D

Compte-tenu du contexte infectieux (hyperthermie), la biopsie d'endomètre et l'hystérosalpingographie sont contre-indiqués.


La Numération Formule Sanguine et l'hémoglobinémie renseignent sur l'importance de l'anémie par spoliation (probable) et
l'état infectieux.

Quel est votre choix parmi ces différents traitements pour arrêter rapidement l'hémorragie ?
A - Corticothérapie par voie orale
B - Antiseptiques locaux
C - Oestrogénothérapie parentérale
D - Curetage hémostatique
E - Prescription de bromocriptine per os (Parlodel)
Bonne(s) réponse(s) : C

On utilise classiquement les oestrogènes injectables (ex : Prémarin®) dans un premier temps afin de réserver le curetage (le
risque de perforation est augmenté par le contexte infectieux) en cas d'échec.

Cette femme a été hospitalisée et mise sous antibiotique. Après 21 jours, elle ne saigne plus et n'a plus de
fièvre. Quels sont le (les) examen(s) complémentaire(s) qui vous parai(ssen)t indispensable(s) pour définir la
cause de l'hémorragie : la NFS montre 3.600.000 G R, 6 500 G B et la VS est à 6-14.
A - Hystéroscopie
B - Coelioscopie
C - Scannographie pelvienne
D - Dosage plasmatique des stéroïdes sexuels
E - Hystérosalpingographie
Bonne(s) réponse(s) : A E

L'infection étant jugulée, il est possible, en l'absence de saignement, de pratiquer selon les possibilités une
hystérosalpingographie et/ou une hystéroscopie afin de visualiser la cause du saignement (fibrome sous-muqueux probable).
Le scanner est un mauvais examen pour apprécier une lésion endo-utérine à fortiori la coelioscopie.

Quelle(s) origine(s) pensez-vous retenir pour expliquer la fièvre à 38°2 du début, alors qu'il n'y a aucune
infection intercurrente ?
A - Endométrite
B - Nécrobiose d'un noyau fibromateux sous- séreux
C - Dysfonction cortico-hypothalamo-hypophysaire s'intégrant dans le cadre d'un syndrome pré-ménopausique
D - Salpingite
E - Toutes ces propositions sont vraies
Bonne(s) réponse(s) : A D

La nécrobiose aseptique d'un fibrome sous-séreux aurait été retrouvée lors de l'examen initial (douleur exquise à la palpation).
Une endométrite et une salpingite sont compatibles avec ce type de tableau car ne sont pas toujours douloureuses.

Quelle(s) origine(s) pourriez-vous retenir pour expliquer les métrorragies ?


A - Cancer de l'endomètre
B - Fibrome sous-muqueux
C - Nécrobiose d'un noyau myomateux sous-séreux
D - Choriocarcinome
E - Synéchie corporéale
Bonne(s) réponse(s) : A B

Le choriocarcinome ne peut se discuter qu'en cas d'antécédent de grossesse. Les synéchies corporéales ne sont pas des
causes de métrorragie. La nécrobiose de fibrome est douloureuse. Le cancer de l'endomètre doit de principe être évoqué et
recherché par un curetage biopsique. L'étiologie vraisemblable est le fibrome sous-muqueux.
93
Exclusivement sur DOC - DZ : www.doc-dz.com NADJI 85
RESIDANAT EN POCHE TOME II
Cas Clinique en QCM

94
Exclusivement sur DOC - DZ : www.doc-dz.com NADJI 85
RESIDANAT EN POCHE TOME II
Cas Clinique en QCM
Une femme de 47 ans, nulligeste, jusque là normalement réglée, présente depuis 3 semaines des métrorragies génitales
persistantes d'abondance moyenne. Elle n'a aucun rapport depuis 3 mois.
L'examen clinique montre un utérus bosselé, indolore, discrètement augmenté de volume et il n y a pas de sensibilité
particulière à la mobilisation. Les culs-de-sac vaginaux sont souples et libres, sans masse latéro-utérine. L'examen au
spéculum avait montré un col sain en apparence. La température est à 38°2 C.
Le reste de l'examen somatique est normal.

Parmi les examens complémentaires suivants, quel est celui qui vous paraît immédiatement le plus utile et non
contre-indiqué ?
A - Frottis cervico-vaginaux de cytodétection
B - Biopsie d'endomètre
C - Radiographie de l'abdomen sans préparation
D - Numération - formule sanguine et taux d'hémoglobine
E - Hystérosalpingographie
Bonne(s) réponse(s) : D

Compte-tenu du contexte infectieux (hyperthermie), la biopsie d'endomètre et l'hystérosalpingographie sont contre-indiqués.


La Numération Formule Sanguine et l'hémoglobinémie renseignent sur l'importance de l'anémie par spoliation (probable) et
l'état infectieux.

Quel est votre choix parmi ces différents traitements pour arrêter rapidement l'hémorragie ?
A - Corticothérapie par voie orale
B - Antiseptiques locaux
C - Oestrogénothérapie parentérale
D - Curetage hémostatique
E - Prescription de bromocriptine per os (Parlodel)
Bonne(s) réponse(s) : C

On utilise classiquement les oestrogènes injectables (ex : Prémarin®) dans un premier temps afin de réserver le curetage (le
risque de perforation est augmenté par le contexte infectieux) en cas d'échec.

Cette femme a été hospitalisée et mise sous antibiotique. Après 21 jours, elle ne saigne plus et n'a plus de
fièvre. Quels sont le (les) examen(s) complémentaire(s) qui vous parai(ssen)t indispensable(s) pour définir la
cause de l'hémorragie : la NFS montre 3.600.000 G R, 6 500 G B et la VS est à 6-14.
A - Hystéroscopie
B - Coelioscopie
C - Scannographie pelvienne
D - Dosage plasmatique des stéroïdes sexuels
E - Hystérosalpingographie
Bonne(s) réponse(s) : A E

L'infection étant jugulée, il est possible, en l'absence de saignement, de pratiquer selon les possibilités une
hystérosalpingographie et/ou une hystéroscopie afin de visualiser la cause du saignement (fibrome sous-muqueux probable).
Le scanner est un mauvais examen pour apprécier une lésion endo-utérine à fortiori la coelioscopie.

Quelle(s) origine(s) pensez-vous retenir pour expliquer la fièvre à 38°2 du début, alors qu'il n'y a aucune
infection intercurrente ?
A - Endométrite
B - Nécrobiose d'un noyau fibromateux sous- séreux
C - Dysfonction cortico-hypothalamo-hypophysaire s'intégrant dans le cadre d'un syndrome pré-ménopausique
D - Salpingite
E - Toutes ces propositions sont vraies
Bonne(s) réponse(s) : A D

La nécrobiose aseptique d'un fibrome sous-séreux aurait été retrouvée lors de l'examen initial (douleur exquise à la palpation).
Une endométrite et une salpingite sont compatibles avec ce type de tableau car ne sont pas toujours douloureuses.

Quelle(s) origine(s) pourriez-vous retenir pour expliquer les métrorragies ?


A - Cancer de l'endomètre
B - Fibrome sous-muqueux
C - Nécrobiose d'un noyau myomateux sous-séreux
D - Choriocarcinome
E - Synéchie corporéale
Bonne(s) réponse(s) : A B

Le choriocarcinome ne peut se discuter qu'en cas d'antécédent de grossesse. Les synéchies corporéales ne sont pas des
causes de métrorragie. La nécrobiose de fibrome est douloureuse. Le cancer de l'endomètre doit de principe être évoqué et
recherché par un curetage biopsique. L'étiologie vraisemblable est le fibrome sous-muqueux.
95
Exclusivement sur DOC - DZ : www.doc-dz.com NADJI 85
RESIDANAT EN POCHE TOME II
Cas Clinique en QCM
Une jeune femme de 19 ans consulte pour des douleurs pelviennes apparues depuis 24 heures s'accompagnant de nausées
et de fièvre à 38°. Il n'y a pas de trouble du transit. Il n'y a aucun antécédent pathologique notable et depuis quatre ans, cette
patiente prend une pilule oestro-progestative combinée normodosée sans surveillance médicale.
A l'examen physique : absence d'ictère, météorisme abdominal discret, palpation de la région hypogastrique normale mais
hypochondre droit sensible avec discrète défense à ce niveau. Il n'y a aucun trouble du transit intestinal. L'examen pleuro-
pulmonaire est normal. L'examen au spéculum montre l'existence de leucorrhées jaunâtres mêlées de sang et après
prélèvements bactériologiques et nettoyage local, le col apparaît légèrement inflammatoire sans autre anomalie.
Les touchers pelviens sont douloureux : cul de sac latéral droit empâté, très sensible et douleur à la mobilisation de l'utérus
ainsi qu'au niveau du cul de sac postérieur. Le cul de sac gauche paraît souple et peu sensible.
Le reste de l'examen clinique est normal ; en particulier, la rate n'est pas perçue, les fosses lombaires sont souples et
indolores et il n'y a aucun signe d'accompagnement.
Parmi les examens complémentaires qui ont été demandés en urgence, on note les résultats suivants : NFS, 11 000 GB dont
78 % de polynucléaires-neutrophiles.
Transaminases SGOT : 25 Ul (N 40), 200 Ul (N 300)
ECBU : 25.000 leucocytes altérés par ml et 1000 colibacilles.

Parmi les diagnostics suivants, ce tableau clinique et paraclinique est compatible avec :
A - Appendicite aiguë
B - Pneumopathie de la base droite
C - Pyélonéphrite aiguë
D - Pancréatique aiguë
E - Salpingite aiguë
Bonne(s) réponse(s) : A E

Les signes orientent vers une pathologie infectieuse prédominant à droite avec des signes de pelvipéritonite compatible avec
une appendicite ou une salpingite.

Parmi les éléments cliniques, quel(s) est(sont) celui(ceux) qui vous oriente(nt) vers le diagnostic de salpingite ?
A - Leucorrhées jaunâtres
B - Douleur à la mobilisation utérine
C - Fièvre à 38°
D - Cul de sac vaginal gauche souple et peu sensible
E - Etat nauséeux
Bonne(s) réponse(s) : A B C E

Seule l'unilatéralité des renseignements fournis par le toucher vaginal ne correspondent pas à la description classique de
salpingite.

Parmi les examens complémentaires suivants, lequel vous paraît le plus adapté pour confirmer le diagnostic de
salpingite ?
A - Coelioscopie
B - Hystérographie
C - Biopsie de l'endomètre
D - Hémoculture
E - Radiographie de l'abdomen sans préparation
Bonne(s) réponse(s) : A

La coelioscopie est préconisée d'emblée chez la femme jeune soucieuse de sa fertilité ultérieure afin de :
- confirmer le diagnostic
- faire des prélèvements bactériologiques au siège de l'infection,
- décrire les lésions et apprécier le pronostic,
- éventuellement libérer quelques adhérences.
N.B.: L'hystérographie et la biopsie d'endomètre sont contre-indiquées en terrain infectieux.

Parmi les résultats suivants des prélèvements bactériologiques, quels sont les agents pathogènes qui sont
susceptibles d'être à l'origine de la salpingite ?
A - Staphylocoques blancs
B - Colibacilles
C - Bacilles de Doderlein
D - Candida albicans
E - Gonocoques
Bonne(s) réponse(s) : B E

Les candida albicans, bacilles de Doderleïn et staphylocoques blancs ne sont jamais responsables d'infection génitale haute
en l'absence de déficit immunitaire sévère. Le gonocoque est le germe le plus fréquemment retrouvé (avec les chlamydia).

96
Exclusivement sur DOC - DZ : www.doc-dz.com NADJI 85
RESIDANAT EN POCHE TOME II
Cas Clinique en QCM

Au vu de ces résultats, quel traitement préconisez vous parmi ceux qui sont proposés ?
A - Ampicilline 3 g par jour
B - Gentalline® 160 mg par jour
C - Flagyl® 15 g par jour
D - Mycostatine® ovule
E - Ces quatre prescriptions sont valables en association
Bonne(s) réponse(s) : E

Afin de préserver au maximum le pronostic de fertilité, il faut être actif au départ sur l'ensemble de la flore bactérienne
susceptible d'être responsable de l'infection, par une antibiothérapie bactéricide à large spectre secondairement adaptée au
résultat des prélèvements. Il est bon de prévenir une candidose vaginale secondaire à cette antibiothérapie par antimycotique
local.

Madame X., 40 ans, est enceinte pour la 3ème fois, car elle a arrêté sa pilule en raison d'une hypertension artérielle. Elle pèse
60 kg pour 1 m 55, les deux premières grossesses se sont déroulées sans problèmes. Les cycles de Madame X sont
réguliers. Les dernières règles sont du 12 Mars 1983.

La date la plus probable de l'accouchement est :


A - 12 Décembre 1983
B - 12 Janvier 1984
C - 24 Décembre 1983
D - 5 Janvier 1984
E - 15 Décembre 1983
Bonne(s) réponse(s) : C

Une grossesse normale accouche en moyenne 9 mois après le début de cette grossesse, soit 39 semaines après les
dernières règles.

Quelle doit être la hauteur théorique de l'utérus à six mols révolus ?


A - 18 cm
B - 20 cm
C - 22 cm
D - 24 cm
E - 28 cm
Bonne(s) réponse(s) : D

L'utérus croit environ de 4 cm par mois à partir du 3ème mois et de 2 cm à partir du 7ème mois de grossesse.

Quelle est la durée du congé prénatal de cette patiente ?


A - 4 semaines avant l'accouchement
B - 6 semaines avant l'accouchement
C - 8 semaines avant l'accouchement
D - 10 semaines avant l'accouchement
E - 12 semaines avant l'accouchement
Bonne(s) réponse(s) : C

En l'absence de complications, la loi prévoit 6 semaines de congé prénatal auquel on peut rajouter 2 semaines pour
grossesse dite "pathologique". Il existe ici une HTA préalable à la grossesse qui constitue un facteur de risque.

L'étude des antécédents de Madame X. vous permet de dire qu'elle appartient à une population à risque :
A - De malformation
B - De diabète gestationnel
C - D'accouchement prématuré
D - De pathologie vasculo-rénale
E - Thrombo-embolique
Bonne(s) réponse(s) : A B D

A 40 ans, le risque de trisomie 21 est augmenté et l'amniocentèse proposée systématiquement. La surcharge pondérale
entraîne un risque accru de diabète gestationnel. L'HTA favorise la toxémie gravidique surajoutée.

97
Exclusivement sur DOC - DZ : www.doc-dz.com NADJI 85
RESIDANAT EN POCHE TOME II
Cas Clinique en QCM
Une femme de 30 ans, mère de 3 enfants, est porteuse d'un D.I.U. (stérilet) vérifié 5 jours auparavant. Au cours d'une de ses
grossesses, elle aurait réalisé une colique néphrétique. Elle vous appelle pour une fièvre à 40° avec frissons, vous notez une
douleur à la percussion de la fosse lombaire gauche, il existe une leucocytose de
22 000/mm3 (83 % de polynucléaires) et trois hémocultures reviennent fertiles pour un protéus mirabilis.

Le diagnostic le plus probable est :


A - Une infection endométriale
B - Une pyélonéphrite aiguë
C - Une diverticulite sigmoïdienne
D - Une cystite aiguë
E - Une poussée de maladie de Crohn
Bonne(s) réponse(s) : B

Le diagnostic le plus probable est une pyélonéphrite aiguë gauche compte-tenu du siège de la douleur, de la bactériémie, du
type de germe retrouvé (1er germe avec E. coli des infections urinaires) et de l'antécédent de colique néphrétique.

La cause la plus vraisemblable de cette affection est :


A - La présence du corps étranger (D.I.U.)
B - Un obstacle mécanique acquis
C - Une malformation congénitale
D - Un déséquilibre hormonal ovarien
E - Aucune des causes ci-dessus
Bonne(s) réponse(s) : B

L'antécédent de colique néphrétique oriente à priori vers une pathologie lithiasique surinfectée,
mais ne permet d'éliminer une malformation.

Tous les moyens suivants peuvent aider au diagnostic, sauf un. Indiquer lequel ?
A - Calcémie
B - Echotomographie abdominale
C - Lavement baryté
D - Cytobactériologie urinaire
E - Radio de l'abdomen sans préparation
Bonne(s) réponse(s) : C

Le lavement baryté n'est ici d'aucun intérêt. La calcémie est en général normale et doit être complétée par une calciurie.

En l'absence de traitement adapté, deux évolutions spontanées sont éventuellement à craindre,


lesquelles ?
A - Arthrite du genou
B - Choc toxi-infectieux
C - Stérilité ultérieure
D - Sténose colique cicatricielle
E - Une récidive à plus ou moins long terme
Bonne(s) réponse(s) : B E

La septicémie précède le choc toxi-infectieux à Gram -


La récidive à plus ou moins long terme est toujours à craindre (persistance de foyers quiescents).

Parmi les antibiothérapies suivantes, compte tenu du germe et de la patiente, laquelle doit être exclue ?
A - Cotrimoxazole (Bactrim®)
B - Erythromycine
C - Tobramycine (Nebcine®)
D - Ampicilline (Totapen®)
E - Céphalosporine orale (Vélocef®)
Bonne(s) réponse(s) : B

Il est impératif de traiter cette patiente par un antibiotique bactéricide à élimination urinaire et à bonne diffusion
parenchymateuse à spectre étroit. L'érythromycine doit être exclue car spectre large et élimination hépatique.

98
Exclusivement sur DOC - DZ : www.doc-dz.com NADJI 85
RESIDANAT EN POCHE TOME II
Cas Clinique en QCM

Parmi les antibiothérapies proposées dans la question précédente, laquelle (lesquelles) peu(ven)t déterminer
une éruption allergique ?
A - Cotrimoxazole (Bactrim®)
B - Erythromycine
C - Tobramycine (Nebcine®)
D - Ampicilline (Totapen®)
E - Céphalosporine orale (Vélocef®)
Bonne(s) réponse(s) : A B C D E

Les produits les plus allergisants sont les sulfamides (Bactrim®) et les béta lactamines (ampicilline, céphalosporine) mais des
éruptions cutanées allergiques ont été décrites pour l'érythomycine et la tobramycine.

Une primigeste à la 28ème semaine de grossesse présente une hauteur utérine à 24 cm, les bruits du coeur sont présents, le
col est long fermé, la tension artérielle est à 15/9 et on note une albuminurie à 50 g/litre. Au cours de la surveillance, on
constate une élévation de la TA à 18/10,5 et une augmentation de l'albuminurie. A la 34ème semaine, la hauteur utérine est à
27 cm, le foetus est en siège, la TA est instable et l'enregistrement des bruits du coeur de l'enfant montre un tracé plat.

Lors de la première consultation, vous préconiserez :


A - Une hospitalisation immédiate
B - Consultation une fois par semaine sans traitement
C - Une consultation une fois par semaine avec repos au lit à domicile
D - Surveillance une fois par semaine avec traitement hypotenseur
E - Surveillance une fois par semaine avec traitement diurétique
Bonne(s) réponse(s) : C

A la première consultation la hauteur utérine est en rapport avec le terme (24 cm pour 6 mois de grossesse), la TA à la limite
supérieure de la normale et l'albuminurie modérée justifiant une consultation hebdomadaire avec repos au lit strict.

Parmi !es examens biologiques, quel est celui qui a la plus grande valeur prédictive de complications ultérieures
?
A - Cholestérolémie
B - Azotémie
C - Béta 1 Glycoprotéine
D - Uricémie
E - Créatininémie
Bonne(s) réponse(s) : D

L'uricémie augmente parallèlement à la gravité du syndrome vasculorénal. Elle a la valeur prédictive la meilleure comparée à
la créatininémie et l'azotémie.

Quelles sont, parmi les prescriptions suivantes, les deux plus utiles ?
A - Diurétiques
B - Régime sans sel
C - Repos au lit
D - Tranquillisants
E - Hypotenseurs
Bonne(s) réponse(s) : C E

Le repos au lit strict est indispensable et permet d'améliorer la perfusion foeto-placentaire. Il est associé aux hypotenseurs de
type centraux dans un premier temps. Les diurétiques et le régime sans sel sont formellement contre-indiqués car aggravant
l'hypovolémie plasmatique.

Quels sont les deux examens qui vont vous permettre de mieux dépister la souffrance foetale ?
A - Dosage de l'oestriol urinaire
B - Dosage de l'hormone lactogène placentaire
C - Dosage du pregnandiol urinaire
D - L'échographie foetale
E - Enregistrement des bruits du coeur de l'enfant
Bonne(s) réponse(s) : D E

La souffrance foetale est dépistée par la répétition des tracés du rythme cardiaque foetal qui doit rester bien fluctuant avec
des phases d'accélération et sans décélération. L'échographie foetale permet de surveiller la croissance foetale (biométrie) et
l'aspect trophique du placenta (zones calcifiées, infarcies). Le dosage de l'oestriol urinaire est moins couramment utilisé
(insuffisance rénale, collecte incorrecte des urines, variabilité extrême du taux de cette hormone).

99
Exclusivement sur DOC - DZ : www.doc-dz.com NADJI 85
RESIDANAT EN POCHE TOME II
Cas Clinique en QCM

A la 34ème semaine, devant les données de l'examen, que préconisez-vous ?


A - Poursuite de la grossesse sous surveillance stricte jusqu'à la 38ème semaine puis extraction du foetus
B - Déclenchement immédiat du travail pour extraire l'enfant par voie basse
C - Test à l'ocytocine pour tester la vitalité foetale
D - Prescription de vasodilatateurs pour améliorer la circulation placentaire
E - Extraction immédiate de l'enfant par césarienne
Bonne(s) réponse(s) : E

Devant le tableau de souffrance foetale chronique majeure :


- retard de croissance intra-utérin (3 cm en 1 mois 1/2)
- tracé cardiaque foetal plat (non fluctuant).
Compte-tenu des données obstétricales :
- foetus en siège
- prématurité (mais viabilité)
- mère primigeste (voie basse exclue dans ce contexte), on préconise la césarienne de sauvetage maternel et surtout foetal
immédiate.

Madame X., serveuse de bar, a 30 ans et consulte pour métrorragies minimes. Dans les antécédents, on retient une
tuberculose pulmonaire traitée sans séquelles à l'âge de 20 ans. Une première grossesse à l'age de 17 ans terminée par une
I.V.G. puis 3 grossesses avec accouchement normal à terme. Madame X a eu un stérilet pendant 2 ans 1/2. Il a été retiré lors
d'une salpingite à gonocoques en 1982. Elle prend actuellement Adépal®, les dernières régles normales sont terminées
depuis 2 jours. A l'examen, le vagin est un peu rouge, Ie col parait remanié au spéculum, le toucher vaginal montre un utérus
un peu gros avec un myome du fond de 3 cm de diamètre, sensible. Les ovaires sont normaux.

L'étude des antécédents de Madame X permet de penser qu'elle appartient à une population à risque de :
A - D'accouchement prématuré
B - Cancer du sein
C - Cancer du col
D - D'accident thrombo-embolique
E - Dysplasie de la vulve
Bonne(s) réponse(s) : A C

Madame X. appartient à une population à risque de cancer du col :


- milieu socio-économique bas
- 1ers rapports sexuels dans l'adolescence (avant 17 ans)
- infection génitale
- profession à risque de partenaires sexuels multiples
- précocité de la première grossesse, multiparité (supérieur à 3).
Et à risque d'accouchement prématuré :
- travail en station debout prolongée
- 3 enfants à charge
- milieu socio-économique bas.
Par-contre, elle n'entre pas dans la population à risque de cancer du sein :
- multiparité (plus de 3 enfants)
- 1ère grossesse jeune (avant 30 ans).

Quel(s) examen(s) complémentaire(s) demandez-vous ?


A - Une échographie pour préciser la topographie du fibrome
B - Un frottis cervico-vaginal
C - Une hystérographie pour rechercher un fibrome intra-cavitaire
D - Un curetage
E - Une N F S
Bonne(s) réponse(s) : B E

Il est impératif de pratiquer un frottis cervico-vaginal compte-tenu du terrain à risque et de l'aspect remanié du col. La
topographie du myome est appréciée par l'examen clinique et rend inutile l'échographie. L'hystérographie et le curetage sont
contre-indiqués dans un contexte infectieux (antécédents de salpingite, utérus sensible, métrorragies), la numération apprécie
les conséquences de la spoliation sanguine répétée et l'éventuelle hyperleucocytose.

Les examens complémentaires pratiqués sont normaux, mais le frottis revient classe III, que faut-il
faire ?
A - Une conisation
B - Un curetage biopsique
C - Un nouveau frottis après un traitement anti-inflammatoire
D - Une biopsie du col
E - Un prélèvement bactériologique endocervical
Bonne(s) réponse(s) : C E

Le frottis classe III (inflammatoire) nécessite une désinfection locale et générale suivie d'un nouveau frottis. Le prélèvement
bactériologique endocervical permet d'adapter le traitement.

100
Exclusivement sur DOC - DZ : www.doc-dz.com NADJI 85
RESIDANAT EN POCHE TOME II
Cas Clinique en QCM

Le second frottis revient classe IV. Quelle(s) attitude(s) proposez-vous ?


A - Hystérectomie totale
B - Conisation du col
C - Nouveau frottis cervical dans 6 mois
D - Curiethérapie cervico vaginale
E - Colposcopie et biopsie cervicales dirigées
Bonne(s) réponse(s) : E

La présence de cellules suspectes sur le frottis nécessite une confirmation anatomopathologique faite au mieux par la
colposcopie avec biopsies dirigées. Selon les résultats (invasifs ou in situ), on pratiquera l'hystérectomie ou la conisation.

Une jeune femme, à la 30ème semaine de grossesse, développe un état infectieux d'allure grippale comportant fièvre à 39°,
frissons, myalgies. L'examen clinique montre une tuméfaction modérée des ganglions cervicaux La vitalité foetale ne semble
pas compromise. Compte tenu du terrain, vous suspectez une infection à listeria monocytogènes.

Parmi les propositions suivantes, laquelle (lesquelles) peut (peuvent) s'appliquer à ce germe ?
A - Bacille anaérobie strict
B - Bacille à gram positif
C - Sporulé
D - Immobile
E - Peu résistant dans le milieu extérieur
Bonne(s) réponse(s) : B

La listéria monocytogène est un bacille gram positif non capsulé, non sporulé mais pourvu de cils périphériques qui le rendent
mobile. C'est un germe résistant que l'on trouve dans le milieu extérieur (réservoir tellurique).

Quelle(s) caractéristique(s) épidémiologique(s) peut (peuvent) s'appliquer à la listériose ?


A - Contamination digestive
B - Maladie sexuellement transmise
C - Transmission par un arthropode vecteur
D - Germe hydrotellurique
E - Réservoir strictement humain
Bonne(s) réponse(s) : A D

Ce germe est ubiquitaire (réservoir tellurique) et donc susceptible de contaminer les aliments et l'eau par opposition aux
germes transmis par un vecteur ou par rapports sexuels.

Parmi les agents suivants, lequel (lesquels) pourrai(en)t être responsable(s) d'un tableau identique chez cette
jeune femme ?
A - Streptocoque B
B - Yersinia entérocolitica
C - Eschérichia coli
D - Toxoplasma gondii
E - Tréponèma pallidum
Bonne(s) réponse(s) : D

Il faut penser systématiquement à la toxoplasmose, souvent asymptomatique mais parfois responsable d'adénopathies
cervicales et de syndrome grippal grave par le risque de transmission foetale. Les autres germes sont spécifiques d'une
localisation infectieuse particulière qui prédomine par rapport au syndrome grippal.

Chez la femme enceinte, quelle(s) autres traduction(s) clinique(s) peut (peuvent) avoir la listériose ?
A - Abcès gynécologiques
B - Latence totale
C - Méningite purulente
D - Discospondylite
E - Pleuro-pneumopathie
Bonne(s) réponse(s) : B C E

La listériose est parfois totalement latente et explique certaines morts foetales in utéro (prélèvements bactériologiques
placentaires systématiques). Elle peut provoquer une méningite purulente (fréquemment à liquide clair). Il existe des formes
pleuropulmonaires pseudotyphoïdiennes chez des patients immunodéprimés dont les femmes enceintes font partie.

101
Exclusivement sur DOC - DZ : www.doc-dz.com NADJI 85
RESIDANAT EN POCHE TOME II
Cas Clinique en QCM

Quel(s) examen(s) faut-il mettre en oeuvre pour assurer rapidement le diagnostic chez cette jeune femme ?
A - Prélèvement de gorge
B - Hémogramme
C - Ponction lombaire
D - Sérologie spécifique
E - Hémocultures
Bonne(s) réponse(s) : E

Les hémocultures retrouvent le plus souvent le germe en période fébrile. La sérologie donne des résultats inconstants et trop
tardifs. La ponction lombaire ne se discute qu'en cas de signes méningés, ce qui n'est pas le cas. L'hémogramme est non
spécifique et le prélèvement de gorge exceptionnellement positif (contamination digestive).

Quelle(s) peut (peuvent) être la (les) conséquence(s) de la listériose maternofoetale non traitée chez cette jeune
femme ?
A - Accouchement prématuré
B - Malformations cardiaques
C - Risque de stérilité ultérieure
D - Septicémie néonatale
E - Malformations oculaires
Bonne(s) réponse(s) : A D

Le foetus contaminé risque une septicémie responsable de mort in utéro, accouchement prématuré, ou avortement selon le
terme et des localisations méningées ou hépatiques responsables de lourdes séquelles. Les malformations oculaires sont
dues particulièrement à la toxoplasmose et les malformations cardiaques évoquent la rubéole. Des avortements successifs
ont été attribués dans quelques observations à la listériose sans que l'on puisse parler de stérilité.

Quelle attitude thérapeutique vous paraît la plus appropriée au cas de cette jeune femme ?
A - Spiramycine pendant 3 semaines
B - Gentamicine jusqu'à l'accouchement
C - Ampicilline pendant 3 semaines
D - Ampicilline jusqu'à l'accouchement
E - Acide nalidixique jusqu'à l'accouchement
Bonne(s) réponse(s) : C

La listéria est sensible à l'ampicilline, aux céphalosporines et à la gentamicine. Le trait classique se fait par 3 à 6 g/24 heures
d'ampicilline pendant 3 semaines.

Quelle est la meilleure prophylaxie de la listériose néonatale ?


A - Prélèvement vaginal lors de chaque visite prénatale
B - Surveillance régulière de la sérologie
C - Hémocultures systématiques à chaque épisode fébrile maternel
D - Prélèvement de gorge systématique à chaque épisode fébrile maternel
E - Vaccination sous contraceptifs de toute femme séronégative
Bonne(s) réponse(s) : C

Seule la pratique systématique d'hémocultures a chaque épisode maternel permet de dépister et guérir une listériose
maternelle. Il n'existe pas de vaccin (contrairement à la rubéole) et les autres examens sont très peu sensibles.

Une femme de 63 ans, pesant 78 kg pour 1,60 m, consulte pour des saignements par le vagin. On ne trouve pas d'antécédent
particulier sauf la prise d'oestrogènes depuis la ménopause intervenue à l'âge de 56 ans. L'examen gynécologique révèle un
col normal, l'utérus est de taille normale.

Le symptôme pour lequel consulte cette patiente s'appelle :


A - Hémorragies vaginales
B - Ménorragies
C - Méno-métrorragies
D - Métrorragies
E - Leucorrhées
Bonne(s) réponse(s) : D

Il s'agit de saignements survenus en dehors des règles (femme ménopausée) ce qui définit les métrorragies. Les ménorragies
sont l'augmentation du volume des règles.

102
Exclusivement sur DOC - DZ : www.doc-dz.com NADJI 85
RESIDANAT EN POCHE TOME II
Cas Clinique en QCM

Quel diagnostic évoquez-vous en premier ?


A - Cancer de la vulve
B - Atrésie vaginale
C - Cancer du vagin
D - Cancer du col utérin
E - Cancer de l'endomètre
Bonne(s) réponse(s) : E

Compte-tenu des facteurs de risque : obésité, ménopause tardive, oestrogénothérapie isolée et de l'aspect normal du col à
l'examen, le 1er diagnostic à évoquer à cet âge est le cancer de l'endomètre.

Quel examen demandez-vous pour confirmer ce diagnostic ?


A - Frottis cervical
B - Colposcopie
C - Curetage biopsique
D - Urographie intraveineuse
E - Scanner pelvien
Bonne(s) réponse(s) : C

Seule l'anatomopathologie permet d'affirmer le diagnostic par curetage endo-utérin biopsique. Les autres examens
participeront éventuellement au bilan préthérapeutique avec l'hystérographie.

Parmi les cancers suivants, un ou plusieurs sont épidémiologiquement liés à la prise d'oestrogènes :
A - Cancer malpighien de la vulve
B - Cancer de l'ovaire
C - Sarcome embryonnaire du vagin
D - Cancer malpighien du col utérin
E - Adénocarcinome de l'endomètre
Bonne(s) réponse(s) : E

Seuls les cancers du sein et de l'endomètre sont épidémiologiquement en rapport avec une
hyperoestrogénie.

Chez une femme de 32 ans, sans antécédent particulier, un frottis cervicovaginal systématique met en évidence des cellules
classe IV. L'examen gynécologique complet montre un col utérin inflammatoire, le vagin est normal de même que les
paramètres. Le reste de l'examen clinique ne montre aucune anomalie. La biopsie du col conclut à l'existence de cellules
malignes épidermoïdes qui restent limites à l'épithélium sans dépasser sa membrane basale.

Quelle est la conclusion de ce compte rendu anatomopathologique ?


A - Carcinome malpighien invasif
B - Carcinome glandulaire
C - Carcinome para-malpighien
D - Carcinome in situ
E - Carcinome micro-invasif
Bonne(s) réponse(s) : D

L'absence de dépassement de la membrane basale définit le carcinome in situ ou non invasif.


Il s'agit d'un carcinome épidermoïde (du revêtement) par opposition au carcinome glandulaire
(adénocarcinome).

Le traitement de choix en est :


A - Une hystérectomie simple
B - Une colpohystérectomie avec Iymphadénectomie
C - Une hystérectomie avec annexectomie bilatérale
D - Une colpectomie
E - Une amputation du col ou une conisation
Bonne(s) réponse(s) : E

Dans un premier temps, la conisation (amputation intra-vaginale du col), si elle passe à distance des limites du foyer tumoral
en zone saine, permet de guérir 100% des carcinomes in situ (sous réserve de la découverte de foyers d'invasion à l'examen
de la pièce).

103
Exclusivement sur DOC - DZ : www.doc-dz.com NADJI 85
RESIDANAT EN POCHE TOME II
Cas Clinique en QCM

L'examen de la pièce opératoire montre qu'il s'agit d'un cancer invasif. Quel est le 1er relai de drainage
Iymphatique du col utérin ?
A - Iliaque interne
B - Lombo-aortique
C - Obturateur
D - Iliaque primitif
E - Inguinal
Bonne(s) réponse(s) : C

Le premier relais Iymphatique du col est le ganglion obturateur au niveau des chaînes iliaques
externes. Ensuite le drainage se fait vers les ganglions iliaques primitifs puis les chaînes
lombo-aortiques. Il existe aussi quelques voies de drainage iliaque interne accessoires. Les
ganglions inguinaux ne sont jamais atteints.

Il s'agit d'un T1. Que doit comporter le bilan pré-thérapeutique ?


A - Une lymphographie pédieuse bilatérale
B - Une coelioscopie
C - Une urographie intra-veineuse
D - Un dosage de béta H.C.G.
E - Une radiographie pulmonaire
Bonne(s) réponse(s) : A C E

Le bilan préthérapeutique comporte une radiographie de thorax afin de rechercher une métastase pulmonaire (exceptionnelle
à ce stade) et de servir de référence pour la surveillance ultérieure. Une urographie intra-veineuse cherche une compression
urétéro-vésicale (qui signerait un stade plus tardif) mais a surtout pour but de localiser les uretères avant la chirurgie
(intervention de Wertheim). Enfin, certaines équipes pratiquent encore une lymphographie pédieuse bilatérale afin de juger de
l'envahissement ganglionnaire.

Le risque d'atteinte ganglionnaire à ce stade de la maladie est :


A - Inférieur à 5%
B - 10 à 15 %
C - 25 à 40 %
D - 45 à 65 %
E - Nul
Bonne(s) réponse(s) : C

Le stade T1 est un stade clinique correspondant à un cancer sans envahissement viscéral ex cervical (cancer limité au col)
mais n'excluant pas l'envahissement ganglionnaire qui est retrouvé à ce stade sur les pièces opératoires (curage iliaque) dans
25 à 40 % des cas.

Mme R., enceinte de 6 mois, vous appelle en consultation. La veille au soir, elle se sentait fatiguée, fiévreuse, et se plaignait
d'une douleur lombaire droite à laquelle elle n'avait pas porté beaucoup d'attention car elle souffrait déjà, depuis plusieurs
semaines, de lombalgies bilatérales mises sur le compte de sa grossesse. Au cours de la nuit, elle a été réveillée par un
frisson suivi de sueurs profuses. A l'examen, vous constatez une fosse lombaire droite sensible à la palpation, un abdomen
souple. La température est de 39°C. Les bruits du coeur foetal sont audibles et Mme R. percoit bien les mouvements du
foetus. Le TV montre un col fermé, un utérus conforme à l'âge de la grossesse. La TA est à 12/6, aux bandelettes, vous
décelez des traces d'albumine et une réaction aux nitrites.

Quel diagnostic évoquez-vous en priorité ?


A - Colique néphrétique
B - Hématome rétro-placentaire
C - Toxémie gravidique
D - Pyélonéphrite aiguë
E - Abcès du rein
Bonne(s) réponse(s) : D

Le tableau clinique évoque en premier lieu une pyélonéphrite aiguë droite (plus fréquente
pendant la grossesse). La protéinurie est dans ce contexte rattachée à l'infection urinaire et non
à une toxémie débutante (TA normale).
La colique néphrétique est non fébrile.

104
Exclusivement sur DOC - DZ : www.doc-dz.com NADJI 85
RESIDANAT EN POCHE TOME II
Cas Clinique en QCM

Quels sont les 2 examens complémentaires à demander dans l'immédiat ?


A - Echographie pelvienne
B - Hémoculture
C - Hématies-leucocytes/minute
D - Examen cyto-bactériologiques des urines
E - Urographie intraveineuse
Bonne(s) réponse(s) : B D

L'urgence est commandée par le diagnostic bactériologique qui permettra d'adapter au mieux le traitement (commencé dès
les prélèvements). L'urographie intra-veineuse ne sera faite qu'à distance de l'accouchement (3 mois). L'échographie sera
faite dans un deuxième temps, la clinique étant ici très rassurante sur l'état foetal.

Que prescrivez-vous ?
A - Quinolones
B - Ampicillines
C - Antispasmodiques
D - Corticoïdes
E - Restriction hydrique
Bonne(s) réponse(s) : B

On débute une antibiothérapie bactéricide à large spectre, à élimination urinaire et bonne diffusion parenchymateuse, non
contre-indiquée par la grossesse, en pratique l'ampicilline est couramment utilisée dans cette indication. On augmentera les
apports hydriques.

La fièvre chute en 24 heures mais la douleur lombaire persiste spontanément et à la palpation. Quel examen
peut vous être utile ?
A - Echographie rénale
B - Uréteropyélographie rétrograde
C - Cliché sans préparation de l'abdomen
D - Nouvel examen cyto-bactériologique
E - Recherche d'anticorps fixés sur les germes
Bonne(s) réponse(s) : A

En cas de persistance des douleurs, malgré la régression des signes infectieux on recherche un obstacle persistant sur les
voies urinaires. Dans le contexte de grossesse, afin d'éviter l'irridiation foetale, et bien que peu performante, on préfère dans
un premier temps l'échographie rénale.

Après l'accouchement, quel sera votre attitude ?


A - Faire une UIV immédiatement après l'accouchement
B - Faire une UIV 2 mois après l'accouchement
C - Faire une scintigraphie au DMSA
D - Surveiller pendant 3 mois les hématies-leucocytes/mn
E - Ne rien faire
Bonne(s) réponse(s) : B

Afin de laisser passer la période d'imbibition progestative responsable d'une hypotonie des voies excrétrices, on ne fera
l'U.I.V. de contrôle que 2 à 3 mois après l'accouchement.

105
Exclusivement sur DOC - DZ : www.doc-dz.com NADJI 85
RESIDANAT EN POCHE TOME II
Cas Clinique en QCM
Une patiente âgée de 47 ans consulte pour ménorragies. Dans ses antécédents on trouve une stérilité primaire d'origine
ovarienne avec de longues périodes de spanioménorrhée.
A l'examen, la malade pèse 80 kg pour une taille de 1m60. L'examen clinique retrouve un col sain, un utérus un peu gros,
indolore et une tumeur pelvienne de 6 cm de diamètre antéro-latérale droite par rapport à l'utérus, séparée de ce dernier par
un sillon, mobilisable avec l'utérus de consistance assez ferme.
Un bilan paraclinique nous donne les renseignements suivants :
- frottis cervicaux de dépistage : aucune cellule suspecte.
- échotomographie pelvienne : utérus de volume normal, ovaires mis en évidence ; le gauche est porteur d'un follicule
translucide de 18 mm de diamètre au 20ème jour du cycle. Il existe en avant et à droite de l'utérus une tumeur de nature
solide homogène, mesurant 7 cm de diamètre.
- l'hystérographie objective une cavité utérine non déformée avec un aspect en double contour plus ou moins régulier de
l'endomètre au niveau de la corne droite.

Quel(s) est(sont) le(s) diagnostic(s) plausible(s) ?


A - Hyperplasie atypique de l'endomètre
B - Polype endométrial
C - Sarcome de l'endomètre
D - Adénocarcinome de l'endomètre
E - Fibrome interstitiel
Bonne(s) réponse(s) : A D

Les fibromes interstitiels et les sarcomes de l'endomètre sont des tumeurs intra-pariétales donc ne peuvent pas être séparées
de l'utérus par un sillon . Le polype donnerait une image de lacune endo-utérine, l'adénocarcinome de l'endomètre est le
diagnostic à éliminer formellement, l'extension extra-cavitaire (annexielle) est probable. L'hyperplasie atypique peut lui être
associée ou expliquer l'image en double contour. Il peut enfin s'agir d'un fibrome sous-séreux.

Dans un but diagnostique vous demandez :


A - Hystéroscopie
B - Scanner
C - Curetage biopsique
D - Coelioscopie
E - Dosages hormonaux
Bonne(s) réponse(s) : A C

L'hystéroscopie avec curetage biopsique permet de faire le diagnostic anatomopathologique, ce qui est dangereux et aléatoire
sous coelioscopie (biopsies contre-indiquées). Le scanner est demandé dans le bilan d'extension mais non pour affirmer le
diagnostic. Les dosages hormonaux ont peu d'intérêt diagnostique.

En dehors de toute pathologie maligne, le traitement peut comporter :


A - Hystérectomie
B - Myomectomie
C - Progestatifs
D - Simple surveillance avec rendez-vous 6 mois plus tard
E - Estrogènes
Bonne(s) réponse(s) : A C

L'hyperplasie utérine et le fibrome utérin sont favorisés par un climat d'hyperoestrogénie et traités par progestatifs. Compte-
tenu de l'âge et de la stérilité, il est licite de proposer une hystérectomie d'emblée d'autant qu'il existe un facteur de risque
important de dégénérescence cancéreuse.

Parmi les éléments ci-dessous, vous retenez comme facteur(s) de risque de cancer de l'endomètre ?
A - Polype endocol
B - Ménopause précoce
C - Obésité diabète
D - Multiparité
E - Ovaires micropolykystiques
Bonne(s) réponse(s) : A C E

Les facteurs de risques du cancer de l'endomètre sont ceux qui favorisent ou révèlent un climat d'hyperoestrogénie relative :
insuffisance lutéale des ovaires micropolykystiques, ménopause tardive, obésité, diabète, polype de l'endomètre, hyperplasie
endométriale, nullipare, traitements oestrogéniques isolés.

106
Exclusivement sur DOC - DZ : www.doc-dz.com NADJI 85
RESIDANAT EN POCHE TOME II
Cas Clinique en QCM

Parmi les éléments suivants vous retenez comme intervenant(s) dans le pronostic du cancer de l'endomètre :
A - Age
B - Pénétration dans le myomètre
C - Envahissement ganglionnaire
D - Situation par rapport à l'isthme
E - Embols vasculaires
Bonne(s) réponse(s) : B C D E

Le pronostic du cancer de l'endomètre dépend comme pour toute tumeur de la différenciation cellulaire et de l'envahissement
ganglionnaire et à distance (présumé sur la présence d'embols vasculaires). Compte-tenu de la richesse du drainage
Iymphatique de l'isthme, son atteinte est de mauvais pronostic ainsi que la pénétration myométriale. L'âge n'intervient pas
directement.

La localisation métastatique la plus fréquente du cancer de l'endomètre est :


A - Poumons
B - Foie
C - Ovaires
D - Vagin
E - Cerveau
Bonne(s) réponse(s) : D

L'extension est d'abord locorégionale vers le vagin (15%), puis aux annexes (10%), enfin par voie hématogène au foie et aux
poumons.

Madame D., âgée de 24 ans, consulte pour aménorrhée secondaire et stérilité primaire. Réglée à 14 ans, elle a présenté des
cycles irréguliers de 40 à 60 jours jusqu'à l'âge de 18 ans. Elle a alors débuté une contraception orale par oestro-progestatifs
par Stéridril®, poursuivie pendant 4 ans sans interruption. Depuis l'arrêt de cette contraception, elle n'est plus réglée.
A l'examen clinique, on note :
- taille 1m60, poids 60 kilos
- hypertrichose modérée et peau acnéique
- col sain avec absence de glaire
- au toucher vaginal, l'utérus est hypoplasique, les ovaires un peu gros.
Différents examens complémentaires sont pratiqués :
- bilan hormonal plasmatique :
F S.H. normale (15 ng/ml), L.H. élevée (4 ng/ml)
Oestradiol bas (50 pg/ml), progestérone basse (0,6 ng/ml)
Delta-4 androstènedione élevée (2800 pg/ml) ainsi que la testostérone (800 pg/ml), 17 OHP normale (0,4 ng/ml).
Prolactine normale (300 uU/ml).
- Caryotype : 46, XX
- H.S.G. : petite cavité utérine, perméabilité tubaire bilatérale
- Radiographie de selle turcique normale.

On peut parler d'aménorrhée secondaire parce que l'absence de règles dure plus de :
A - 3 mois
B - 6 mois
C - 9 mois
D - 12 mois
E - 18 mois
Bonne(s) réponse(s) : A

L'aménorrhée secondaire est définie par l'absence de règles depuis plus de 3 mois.

Parmi les propositions diagnostiques suivantes, laquelle retenez-vous ?


A - Aménorrhée psychogène
B - Syndrome de Rokitanski
C - Syndrome de Stein-Leventhal
D - Ménopause précoce
E - Aménorrhée ovarioplégique
Bonne(s) réponse(s) : C

Le tableau décrit le syndrome de Stein-Leventhal avec ovaires polykystiques (blancs nacrés et augmentés de volume) avec
spanioménorrhée puis anovulation et hyperandrogénie relative. Il existe une hypersensibilité hypophysaire à la LHRH et LH
est élevée.

107
Exclusivement sur DOC - DZ : www.doc-dz.com NADJI 85
RESIDANAT EN POCHE TOME II
Cas Clinique en QCM

Parmi les examens complémentaires suivants, lequel convient-il de pratiquer pour le confirmer ?
A - Coelioscopie
B - Test au TRH
C - Courbe ménothermique
D - Test au LH-RH
E - Dosage des bêta-HCG
Bonne(s) réponse(s) : D

Le meilleur examen est le test au LHRH qui montre une réponse explosive du LH La coelioscopie n'est pas pratiquée
systématiquement.

Quel traitement proposez-vous si la patiente désire une grossesse ?


A - Clomid®
B - Acétate de cyprostérone
C - Résection cunéiforme des ovaires
D - Parlodel®
E - Séquence H.M.G - H.C.G
Bonne(s) réponse(s) : A

La résection cunéiforme des ovaires était le traitement classique peu efficace visant à favoriser l'ovulation ; on utilise
actuellement le Clomid® qui est compétitif avec l'oestradiol au niveau hypothalamique et supprime le rétro-contrôle
hypophysaire d'où libération de FSH et maturation folliculaire.

La normalité du dosage de 17 hydroxyprogestérone permet d'éliminer le diagnostic de :


A - Hyperplasie surrénale congénitale
B - Stein-Leventhal
C - Ménopause précoce
D - Dysgénésie gonadique
E - Tumeur ovarienne audio-séro-sécrétante
Bonne(s) réponse(s) : A

La 17 hydroxyprogestérone est élevée dans les déficits congénitaux en 21 béta, 11 béta et 3 béta OH déshydrogénase de
l'hyperplasie congénitale des surrénales (responsable d'impubérisme, ambiguité sexuelle et aménorrhée primaire).

Parmi les examens rapportés dans l'énoncé, il n'était pas nécessaire de pratiquer en première intention :
A - Progestérone plasmatique
B - Caryotype
C - Radio de selle turcique
D - LH plasmatique
E - Prolactine plasmatique
Bonne(s) réponse(s) : A B C

En l'absence d'ovulation, la progestérone est toujours basse donc de peu d'intérêt et le caryotype normal dans les
aménorrhées secondaires sans malformation. La prolactinémie est normale donc la radiographie de la selle turcique a des
chances de l'être aussi.

Madame S. , 23 ans, consulte le 14 février 1986 son médecin traitant pour une douleur hypogastrique intense apparue assez
brusquement le matin même ; la température est à 39°C depuis l'apparition de la douleur qui est continue. L'interrogatoire ne
met en évidence aucun antécédent notable en dehors de suites de couches fébriles après un premier accouchement le 25
Janvier 1986. Il existe une défense sus-pubienne, le toucher vaginal est très douloureux et objective un empâtement diffus et
hyperalgique des deux culs de sac latéraux du vagin.

Quel diagnostic évoquez-vous en premier ?


A - Nécrobiose aseptique
B - Appendicite
C - Torsion de kyste de l'ovaire
D - Pyélonéphrite bilatérale
E - Infection génito-pelvienne
Bonne(s) réponse(s) : E

Il s'agit d'une infection pelvi génitale bilatérale. La nécrobiose aseptique et la torsion de kyste sont asymétriques au départ.

108
Exclusivement sur DOC - DZ : www.doc-dz.com NADJI 85
RESIDANAT EN POCHE TOME II
Cas Clinique en QCM

Parmi les examens complémentaires, suivants citer celui(ceux) qui est(sont) utilisé(s) pour confirmer votre
hypothèse diagnostique :
A - Coelioscopie
B - Biopsie d'endomètre
C - Hystérosalpingographie
D - Hémogramme
E - Urographie intra-veineuse
Bonne(s) réponse(s) : A D

La coelioscopie est un examen à faire de 1ère intention chez une femme jeune désireuse de grossesse afin :
- de faire un prélèvement bactériologique dans de bonnes conditions
- de confirmer le diagnostic et d'apprécier l'étendue des lésions (pronostic)
- de lever des adhérences éventuelles.
L'hémogramme fait partie du bilan pour apprécier l'hyperleucocytose.
Biopsie d'endomètre et hystérographie sont contre-indiquées dans un tableau infectieux et non utiles dans ce cas.

Le diagnostic est établi : que prescrit-on généralement ?


A - Une antibiothérapie double ou triple
B - Un anti-inflammatoire
C - Un progestatif de synthèse en continu
D - Une laparotomie immédiate
E - Un désinfectant urinaire
Bonne(s) réponse(s) : A B

Une antibiothérapie à large spectre est entreprise (les associations bactériennes sont fréquentes) comprenant entre autre un
anti-anaérobie et anti-chlamydia, associés dans de nombreuses équipes à un traitement anti-inflammatoire dès l'apyrexie
obtenue. La laparotomie ne se discute qu'en cas d'abcès.

Pour affirmer la guérison, que vérfier ?


A - La disparition des germes sur les prélèvements
B - La disparition de la leucocytose
C - La normalisation de l'hystérosalpingographie
D - La disparition de la douleur pelvienne
E - La normalisation de l'échographie rénale bilatérale
Bonne(s) réponse(s) : B D

La surveillance de l'efficacité du traitement repose sur la disparition des signes cliniques (douleurs),la disparition de
l'hyperleucocytose et la normalisation de la V.S. Une coelioscopie est pratiquée dans certaines équipes après un laps de
temps variable. Les prélèvements endo-utérins ne sont pas pratiqués.

Quelle(s) séquelle(s) est (sont) retrouvée(s) électivement dans les suites de cette affection lorsqu'elle est non
ou mal traitée ?
A - Urétéro-hydronéphrose
B - Insuffisance lutéale
C - Algie pelvienne
D - Hémorragie génitale
E - Stérilité
Bonne(s) réponse(s) : C E

Le risque majeur de la salpingite est la stérilité par obturation tubaire. Les algies pelviennes séquellaires sont fréquentes
(adhérences), la survenue d'une hémorragie génitale doit faire rechercher une récidive ou une autre pathologie.

Une femme enceinte de groupe sanguin AB rhésus négatif (éé dd ee) présente dans son sérum un allo-anticorps anti-D (Rho)
résultant d'une immunisation antérieure par grossesse.

Au cours de cette grossesse, les examens légaux à réaliser chez la mère comprendront :
A - Une vérification du groupe sanguin ABO et Rhésus
B - Une recherche des anticorps irréguliers anti-érythocytaires
C - Un titrage de ces anticorps si la recherche précédente est positive
D - Une recherche des anticorps immuns anti-A et/ou anti-B
E - Un dosage de la bilirubine
Bonne(s) réponse(s) : A B C

Il faut systématiquement revérifier le groupe sanguin, rhésus, et les agglutinines irrégulières qu'il faut doser (référence pour la
surveillance ultérieure). La bilirubinémie n'a d'intérêt que chez le foetus.

109
Exclusivement sur DOC - DZ : www.doc-dz.com NADJI 85
RESIDANAT EN POCHE TOME II
Cas Clinique en QCM

Un dosage pondéral de l'anticorps anti-D sérique naturel sera envisagé si on observe :


A - Une augmentation significative du titre de l'anticorps anti-D naturel
B - L'apparition d'autres spécificités (AC) anti-érythrocytaires dans le sérum de la mère
C - Une augmentation du taux de la bilirubinémie maternelle
D - Que le père est homozygote pour le gène D
E - L'apparition d'anticorps immuns anti-A dans le sérum de la mère
Bonne(s) réponse(s) : A

Le dosage pondéral se fait en cas d'augmentation significative du titre d'anticorps anti D naturel tous les 15 jours à partir du
5ème mois.

Le dosage de la bilirubinémie est classiquement réalisé par :


A - Technique radio-immunologique
B - Le test de Kleihauer
C - Technique biochimique (Diazoréaction)
D - Etude spectrophotométrique du liquide amniotique
E - Technique Elisa
Bonne(s) réponse(s) : D

Le dosage de la bilirubinémie est réalisé par étude spectrophotométrique ce qui implique que le tube doit être transporté à
l'abri de la lumière.

A la naissance, les examens pratiqués sur le sang du nouveau-né destinés à porter le diagnostic d'allo-
immunisation foeto-maternelle comportent :
A - Groupage sanguin ABO-Rhésus
B - Test de Coombs direct
C - Test de Kleihauer
D - Numération formule sanguine
E - Dosage de la bilirubine libre
Bonne(s) réponse(s) : A B D E

Le diagnostic de l'autoimmunisation rhésus repose sur la mise en évidence d'agglutinines sur les hématies foetales par le test
de Coombs direct, après le groupage vérifiant la positivité rhésus. L'hémogramme et la bilirubinémie apprécient le
retentissement foetal par le degré d'hémorragie et l'importance de la régénération.

Le groupe ABO Rhésus du sang utilisé pour l'exsanguino-transfusion pourrait être (enfant du groupe A Rhésus
positif) :
A - A Rhésus négatif
B - B Rhésus positif
C - O Rhésus négatif
D - O Rhésus positif
E - A Rhésus positif
Bonne(s) réponse(s) : A C

Il est préférable de transfuser avec un sang isogroupe rhésus négatif (les hématies rhésus + risquent d'être détruites par les
agglutinines), le sang O positif n'est pas incompatible.

Cette femme de 32 ans, enceinte de 3 mois, est hospitalisée pour un syndrome méningé modérément fébrile, un état sub-
comateux et une paralysie oculomotrice. La ponction lombaire montre un liquide opalescent dont l'examen bactériologique
révèlera la présence de Listéria Monocytogènes.

Outre la grossesse, d'autres terrains peuvent favoriser la pathogénicité de ce germe relativement rare ; ainsi
vous retenez :
A - Enfance
B - Adulte jeune
C - Cirrhose éthylique
D - Cancer digestif
E - Asthme
Bonne(s) réponse(s) : C D

Toutes les situations d'immunodépression cellulaire favorisent le développement de ce germe ubiquitaire qui devient alors
virulent.

110
Exclusivement sur DOC - DZ : www.doc-dz.com NADJI 85
RESIDANAT EN POCHE TOME II
Cas Clinique en QCM

Au cours de cette méningite, quelle est l'anomalie la plus évocatrice de l'examen cytochimique du LCR ?
A - Hyper-albuminorachie avec hypercytose lymphocytaire pure
B - Hypoglycorachie avec hypercytose lymphocytaire pure
C - Glycorachie normale ou abaissée et hypercytose "panachée" (présence de polynucléaires et de
Iymphocytes en quantité à peu près égale)
D - Hyperchlorurachie avec hypercytose lymphocytaire pure
E - Hypoglycorachie avec hypercytose à polynucléaires
Bonne(s) réponse(s) : C

Malgré sa nature bacillaire, la listéria peut donner des méningites à liquide clair lymphocytaires mais le tableau d'hypercytose
panachée est le plus fréquemment retrouvé.

Quelle(s) est (sont) la (les) possibilité(s) prévisible(s) d'évolution de la grossesse ?


A - Accouchement à terme d'un enfant normal
B - Accouchement à terme d'un enfant porteur d'une cardiopathie
C - Accouchement proche du terme d'un enfant porteur d'une listériose néonatale
D - Avortement au cours de l'évolution de la méningite
E - Aucune des manifestations précédentes
Bonne(s) réponse(s) : A D

La listériose est grave pour le foetus responsable d'avortement et de septicémie foetale en l'absence de traitement précoce
adapté. Elle n'est pas responsable de malformation foetale (cardiopathie) et l'absence d'avortement signifie que le foetus est
indemne lorsque le traitement est adapté.

Quel traitement proposez-vous chez cette patiente ?


A - Ampicilline
B - Chloramphénicol
C - Céphalosporine, type Cefotaxime®
D - Erythromycine
E - Tétracycline
Bonne(s) réponse(s) : A

Le traitement classique est l'ampicilline 3 à 6 g/24 h pendant 3 semaines. Les aminosides et céphalosporines peuvent
également être utilisées.

Quelle(s) est (sont) la(les) autre(s) localisation(s) fréquente(s) de ce germe ?


A - Hépatiques
B - Pleuro-pulmonaires
C - Rénales
D - Infection placentaire
E - Aucune d'entre elles
Bonne(s) réponse(s) : B D

Les atteintes classiques sont l'infection placentaire à rechercher systématiquement par prélèvement bactério-placentaire en
cas de mort foetale in utéro ou souffrance foetale inexpliquée, lors de la délivrance. Chez la mère elle, peut réaliser un tableau
pseudogrippal, pseudotyphoïdien.

Une patiente de 30 ans, aux cycles réguliers et désirant un enfant, vous consulte pour une aménorrhée de 6 semaines avec
fébricule à 38°C, douleurs pelviennes et épisode de métrorragies. On note dans ses antécédents plusieurs épisodes
d'annexite et une intervention tubaire par microchirurgie il y a un an. Vous retrouvez à l'examen clinique un utérus globuleux à
la mobilisation, des annexes sensibles sans masse percue.

Quel(s) examen(s) complémentaire(s) proposez-vous pour préciser le diagnostic ?


A - Abdomen sans préparation
B - Hystérosalpingographie
C - Echographie pelvienne
D - Prélèvements bactériologiques de l'endocol
E - Dosage des Béta-HCG plasmatiques
Bonne(s) réponse(s) : C D E

Toute aménorrhée secondaire impose la recherche d'une grossesse par dosage des BHCG. Dans ce contexte infectieux, les
prélèvements bactériologiques sont nécessaires et l'hystérographie contre-indiquée. L'échographie renseigne sur la présence
d'un sac intra-utérin et/ou d'une image annexielle
(G.E.U., hydrosalpinx).

111
Exclusivement sur DOC - DZ : www.doc-dz.com NADJI 85
RESIDANAT EN POCHE TOME II
Cas Clinique en QCM

Quel(s) diagnostic(s) évoquez-vous d'emblée ?


A - Endométrite ou salpingite
B - Torsion de kyste de l'ovaire
C - Endométriose pelvienne
D - Grossesse extra-utérine
E - Menace d'avortement spontané fébrile
Bonne(s) réponse(s) : A D E

La grossesse extra-utérine doit bien sûr être évoquée en premier lieu (aménorrhée, antécédent de microchirurgie tubaire)
avant de soulever celui d'endométrite ou salpingite ou d'avortement spontané fébrile. L'endométriose et la torsion de kyste
sont aseptiques au début.

L'échographie révèle une masse latéro-utérine hétérogène de 3 x 4 cm de diamètre, un épanchement liquidien


du Douglas, et une lacune anéchogène intra-utérine, arrondie, de 1 cm de diamètre. Les bêta-HCG
plasmatiques sont à 250 mUl/ml. Quelle est votre attitude ?
A - Abstention thérapeutique et contrôle dans 8 jours pour échographie et bêta-HCG
B - Laparotomie exploratrice d'emblée
C - Curetage et surveillance clinique
D - Coelioscopie
E - Biopsie d'endomètre et coelioscopie en fonction du résultat
Bonne(s) réponse(s) : D

La coelioscopie est impérative pour affirmer ou infirmer la grossesse extra-utérine. L'image intra-utérine peut correspondre à
la caduque.

Au terme de votre démarche diagnostique, vous décidez d'une coelioscopie, au cours de laquelle vous
découvrez une grossesse extra utérine droite de 1 cm de diamètre, ampullaire ; la trompe controlatérale est
recouverte d'adhérences et présente un phimosis complet du pavillon. Quel geste thérapeutique vous semble le
plus adapté ?
A - Conservation tubaire par césarienne tubaire
B - Salpingectomie droite
C - Salpingectomie bilatérale
D - Salpingectomie droite et plastie tubaire gauche
E - Salpingectomie droite et ligature tubaire controlatérale
Bonne(s) réponse(s) : A

Compte-tenu de la petite taille de la grossesse et du mauvais état de la trompe controlatérale, on propose une chirurgie
conservatrice.

Vous recevez un frottis cervico-vaginal classe IV (classification de Papanicolaou) concernant une patiente de 34 ans que vous
suivez régulièrement. Elle n'a aucun signe fonctionnel.

Indiquez le(s) examen(s) que vous devez effectuer :


A - Nouveaux frottis
B - Hystérographie
C - Hystéroscopie
D - Colposcopie
E - Biopsie cervicale sur chaque lèvre du col
Bonne(s) réponse(s) : D

La présence de cellules suspectes impose la confirmation anatomopathologique par colposcopie avec biopsie dirigée (et non
à l'aveugle) d'où découlera l'indication thérapeutique. En l'absence d'anomalie, on pratiquera alors une hystéroscopie avec
biopsie.

Il s'agit de lésions de dysplasie sévère situées sur la lèvre antérieure du col, dont la limite supérieure n'est pas
vue.
Indiquez le point de départ le plus fréquent de ces lésions :
A - La muqueuse de l'endocol
B - Les glandes de l'endocol
C - La muqueuse de l'exocol
D - La muqueuse du cul-de-sac vaginal antérieur
E - La zone de jonction des muqueuses pavimenteuse et cylindrique
Bonne(s) réponse(s) : E

C'est au point le plus haut situé dans l'endocol (zone de jonction) que démarrent les dysplasies, ce qui impose de voir cette
zone lors de la colposcopie et le cas échéant de compléter l'examen par la microcolposcopie.

112
Exclusivement sur DOC - DZ : www.doc-dz.com NADJI 85
RESIDANAT EN POCHE TOME II
Cas Clinique en QCM

Quel geste diagnostique et thérapeutique devez-vous proposer ?


A - Hystérectomie totale
B - Conisation du col
C - Vaporisation laser
D - Curiethérapie vaginale
E - Electrocoagulation des lésions
Bonne(s) réponse(s) : B

La conisation s'impose. Les limites de l'exérèse doivent être en zone saine (confirmation
anatomopathologique) compte-tenu de l'absence de visibilité de la jonction lors de la colposcopie.

L'examen anatomo-pathologique de la pièce découvre un foyer d'épithélioma intra-épithélial. Les limites de la


pièce d'exérèse sont en zone saine. Que proposez-vous ?
A - Une chimiothérapie complémentaire
B - Une radiothérapie complémentaire
C - Une curiethérapie complémentaire
D - Une hystérectomie totale de principe
E - Une simple surveillance
Bonne(s) réponse(s) : E

La conisation enlevant l'ensemble de la lésion non invasive guérit dans 100 % des cas mais impose une surveillance par
frottis ultérieur.

Parmi les agents sexuellement transmissibles suivants, quel(s) est (sont) celui (ceux) corrélés au diagnostic de
dysplasie sévère ?
A - Mycose
B - Trichomonas
C - Papilloma virus
D - Herpès
E - Streptocoque du groupe B
Bonne(s) réponse(s) : C D

Les virus herpès (du type 2) et papilloma virus (responsable des condylomes) ont été retrouvés dans des lésions dysplasiques
et sont correlés avec le diagnostic.

Madame A. présente à 37 semaines un ictère cutanéo-muqueux net avec prurit modéré. L'ictère s'est installé progressivement
avec un syndrome pseudo-grippal et des vomissements. La température est à 37,6°. Sur le plan obstétrical, la H.U. est à 32
cm et le foetus bouge bien. Le col n'est pas modifié. La T.A. est à 125/70. Au Labstix : albumine -, glycosurie -, acétone +,
nitrite -.

Quel est le diagnostic le plus probable ?


A - Cholestase gravidique
B - Hépatite virale
C - Hépatite listérienne
D - Hépatite carentielle
E - Péri hépatite à chlamydia
Bonne(s) réponse(s) : B

Il s'agit en l'absence de signes infectieux du tableau classique de la cholestase gravidique. Dans ce contexte pseudogrippal,
c'est l'hépatite virale qui est évoquée en premier lieu.

Quel(s) examen(s) biologique(s) allez-vous demander ?


A - Transaminases
B - Phosphatases alcalines
C - Bilirubine
D - Cholestérol
E - Electrophorèse des protides sanguins
Bonne(s) réponse(s) : A B C

Le bilan hépatique recherche une cytolyse (transaminases) et apprécie le degré de la cholestase


(phosphatases alcalines, bilirubine).

113
Exclusivement sur DOC - DZ : www.doc-dz.com NADJI 85
RESIDANAT EN POCHE TOME II
Cas Clinique en QCM

Le(s) risque(s) encouru(s) par ce foetus est(sont) :


A - Prématurité
B - Hypotrophie
C - Mort in utéro
D - Malformation cardiaque
E - Hépatite chronique
Bonne(s) réponse(s) : C E

La prématurité est définie par l'accouchement avant 37 S.A. donc est ici exclue, de même pour les malformations et
l'hypotrophie. L'atteinte foetale peut entraîner une hépatite chronique ou la mort foetale in utéro (rare). On pratiquera une
séroprophylaxie systématique à la naissance.

Le signe de gravité de cet ictère gravidique est :


A - Prurit
B - Hypercholestérolémie à 4 g/l
C - Pouls à 55
D - Taux prothrombine à 50 %
E - Présence d'acétone + au Labstix
Bonne(s) réponse(s) : D

La gravité essentielle est liée à l'insuffisance hépatique avec baisse des facteurs de coagulation,
augmentant le risque maternel et obstétrical (hémorragie de la délivrance).

En plus des règles hygiéno-diététiques que prescrivez-vous systématiquement ?


A - Cholestéramine (Questran®)
B - Bétamimétiques
C - Ampicilline
D - Héparine
E - Aucune des propositions ci-dessus
Bonne(s) réponse(s) : A

Le prurit peut être combattu par le Questran® après échec des antihistamiques car il est théoriquement non absorbé.

Madame X. , âgée de 45 ans, vient consulter pour des méno-métrorragies capricieuses. La patiente a eu 2 enfants sans
incidents. Elle était bien réglée jusqu'à l'apparition de ses métrorragies. Elle n'a pas maigri. L'examen de l'abdomen le montre
un peu tendu. Le toucher vaginal trouve dans le cul de sac droit une masse arrondie, séparée de l'utérus par un sillon, mobile.

Parmi les examens suivants, lequel(lesquels) demanderez vous pour asseoir le diagnostic ?
A - Abdomen sans préparation
B - Echotomographie du petit bassin
C - Hystérosalpingographie
D - Coelioscopie
E - Biopsie de la masse sous coelioscopie
Bonne(s) réponse(s) : B D

L'échographie dans un premier temps permet de préciser la nature solide ou liquide de la masse et d'orienter sur sa nature. La
coelioscopie, en cas de confirmation de la nature ovarienne.

La patiente, sur les conseils de son chirurgien, a accepté une laparotomie. Vous assistez à l'opération. Le
chirurgien trouve un gros ovaire droit sans végétation en surface. Si vous étiez à sa place, laquelle(lesquelles)
de ces attitudes choisiriez-vous ?
A - Ponctionner l'ovaire tumoral in situ
B - Faire une biopsie chirurgicale de cet ovaire
C - Enlever l'ovaire en question pour faire une examen histologique extemporané en prenant garde de ne pas
le rompre
D - Faire une hystérectomie totale avec annexectomie d'emblée
E - Faire une ovariectomie bilatérale
Bonne(s) réponse(s) : C

Toute tumeur organique de l'ovaire impose son exérèse pour étude anatomopathologique de l'ensemble de la pièce.

114
Exclusivement sur DOC - DZ : www.doc-dz.com NADJI 85
RESIDANAT EN POCHE TOME II
Cas Clinique en QCM

En admettant que la patiente examinée ait un cancer de l'ovaire au stade III, auquel de ces éléments
attacheriez-vous une importance pronostique ?
A - Présence d'ascite libre
B - Sténose localisée et partielle du grêle
C - Atteinte des coupoles diaphragmatiques
D - Présence d'un gros utérus envahi
E - Volume des lésions restant en fin d'intervention
Bonne(s) réponse(s) : E

La chirurgie réductrice précède la chimiothérapie et radiothérapie. Seul le volume des lésions restant en fin d'intervention
conditionne le pronostic ultérieur (l'ensemble des autres propositions font partie de la description possible d'un stade 3). Des
lésions de plus de 2 cm ne peuvent pas être stérilisées par la radiothérapie.

L'espérance de survie actuelle à 5 ans des femmes atteintes de cancer de l'ovaire est actuellement :
A - 80 %
B - De l'ordre de 75 %
C - de l'ordre de 50 %
D - De l'ordre de 25 %
E - De 0 %
Bonne(s) réponse(s) : D

Tous stades confondus, l'espérance de vie globale est de 25 % à 5 ans. Il s'agit d'un cancer de mauvais pronostic.

Madame T..., âgée de 34 ans consulte pour l'apparition depuis 2 mois environ d'une pesanteur pelvienne. A l'interrogatoire, on
ne retrouve pas la notion d'aménorrhée ou de métrorragies mais celle d'un amaigrissement de 4 kg en 2 mols. L'examen
révèle une augmentation de l'abdomen localisée en sous-ombilical. Le toucher vaginal avec palpation montre la présence
d'une masse annexielle à droite, irrégulière et non mobile. L'utérus parait de taille normale. L'auscultation pulmonaire est
normale.

Quel est le diagnostic le plus probable de la maladie ?


A - Kyste folliculaire ovarien droit
B - Choriocarcinome placentaire
C - Cancer épithélial ovarien
D - Cancer colique sigmoïdien
E - Cancer de l'endomètre
Bonne(s) réponse(s) : C

L'utérus est de taille normale (élimine B, E), le sigmoïde est à gauche et se cancérise plus tard, on est donc orienté vers une
pathologie maligne (élimine A) de l'annexe droite (augmentation de volume rapide, masse irrégulière et fixée, amaigrissement)
et par argument de fréquence vers un cancer épithélial ovarien.

Parmi les examens paracliniques suivants, vous devez demander :


A - Lymphographie bipédieuse
B - Frottis du col utérin
C - Echographie abdomino pelvienne
D - Dosage de l'Ag Ca 125
E - U.I.V.
Bonne(s) réponse(s) : B C D E

L'échographie permet de préciser la nature (liquide ou solide).


L'urographie intra-veineuse précise la position des uretères avant l'intervention chirurgicale et le retentissement éventuel de la
tumeur sur l'appareil urinaire.
Le CA 125 est un marqueur tumoral souvent élevé dans les tumeurs ovariennes.
La lymphographie est inutile car les métastases ganglionnaires sont tardives et leur présence ne modifie pas la conduite
thérapeutique.
Le frottis du col est conseillé bien que l'hystérectomie totale soit faite si elle est possible.

Un traitement chirurgical à visée curatrice est entrepris dans un premier temps. On constate une carcinose
péritonéale. Comment classez-vous cette maladie ?
A - Stade I a
B - Stade I b
C - Stade II
D - Stade III
E - Stade IV
Bonne(s) réponse(s) : D

Le stade III est défini par l'atteinte intra-péritonéale extra-pelvienne en l'absence de métastase extra-abdominale clinique.

115
Exclusivement sur DOC - DZ : www.doc-dz.com NADJI 85
RESIDANAT EN POCHE TOME II
Cas Clinique en QCM

L'examen histologique montre des cellules indifférenciées de grade III. Devant une exérèse tumorale
incomplète, on décide de débuter une chimiothérapie qui comporte du Cisplatinum. Parmi les effets secondaires
suivants, vous attribuez au Cisplatinum :
A - Alopécie
B - Surdité
C - Insuffisance rénale
D - Pancytopénie
E - Fibrose pulmonaire
Bonne(s) réponse(s) : B C D

La toxicité rénale de ce produit nécessite une diurèse provoquée > ou égale à 3 l/24 h. Lors de son administration, cette
toxicité est cumulative (liée à la dose totale).
L'ototoxicité porte sur les hautes fréquences, la plupart du temps se réduit à des bourdonnements d'oreilles.
La pancytopénie est plus rare (myélodépression).

Il s'agit d'une patiente de 62 ans, mère de 3 enfants, ménopausée à 50 ans qui consulte pour des pertes rosées ayant duré
une semaine. Elle mesure 163 cm, pèse 69 kilos et est en bon état général. L'examen général, l'examen des aires
ganglionnaires, l'examen des seins et du col est normal. L'utérus est de volume normal, les annexes ne sont pas percues.
L'hystérographie met en évidence une lacune régulière du fond utérin.

Chez cette patiente, quel est le meilleur examen pour éliminer ou affirmer l'existence d'un cancer de l'endomètre
?
A - Cytologie endo-utérine
B - Biopsie à la canule de Novak
C - Echographie
D - Curetage sous anesthésie générale
E - Coelioscopie
Bonne(s) réponse(s) : D

Sans commentaire.

Un adénocarcinome infiltrant limité à la cavité utérine est confirmé, l'hystérométrie est de 7 cm. Quel est le
stade de ce cancer ?
A - Adénocarcinome in situ
B - IA
C - IB
D - II
E - IV
Bonne(s) réponse(s) : B

(IA) : tumeur limitée au corps utérin avec hystérométrie inférieure ou égale à 8 cm.

Cette patiente bénéficie d'une curiethérapie vaginale avant son intervention. Quel(s) est(sont) le(s) but(s) de
cette irradiation ?
A - Stériliser une éventuelle extension cervicale
B - Stériliser les paramètres
C - Stériliser la lésion endométriale
D - Prévenir une récidive vaginale
E - Permettre de limiter le geste chirurgical
Bonne(s) réponse(s) : D E

Elle vise à stériliser les cellules en migration vaginale, pelvienne et lymphathique.

Sur le compte rendu de l'examen anatomo-pathologique de la pièce opératoire figurent les 5 informations
suivantes. Quelle(s) est(sont) celle(s) qui a(ont) une valeur pour le pronostic ultérieur ?
A - Adénocarcinome du fond utérin
B - Dysplasie cervicale modérée
C - Infiltration du myomètre sur 3 mm
D - Adénocarcinome bien différencié
E - Absence d'atteinte ganglionnaire sur les 20 ganglions examinés
Bonne(s) réponse(s) : C D E

L'atteinte limitée à la muqueuse endométriale est le meilleur pronostic que l'invasion pariétale (myomètre...).

116
Exclusivement sur DOC - DZ : www.doc-dz.com NADJI 85
RESIDANAT EN POCHE TOME II
Cas Clinique en QCM

Compte-tenu du compte-rendu anatomo-pathologique, dans quelle catégorie de survie à 5 ans peut-on classer
cette patiente ?
A - 90 %
B - 75 %
C - 60 %
D - 50 %
E - 30 %
Bonne(s) réponse(s) : B

Connaissance.

Vous examinez une femme enceinte (premier examen prénatal à 2 mois de grossesse) :

Dans le cadre de la prévention de la prématurité, il est licite de rechercher dans les antécédents de cette femme
:
A - Un accouchement prématuré
B - Une mort périnatale
C - Un avortement tardif
D - Une fausse couche curetée
E - Une césarienne
Bonne(s) réponse(s) : A C D

(Pour D = plusieurs IVG).


Les IVG répétées par le risque de traumatisme du col (béance secondaire) ou de synéchies internes séquellaires sont un
facteur de risque de prématurité.

Dans le cadre de la prévention de la prématurité, il est licite de rechercher à l'examen :


A - Un souffle cardiaque
B - Une hypotension orthostatique
C - Une béance isthmique
D - Une anémie
E - Une exocervicite
Bonne(s) réponse(s) : A C D E

La béance cervico-isthmique est une des principales causes mécaniques de prématurité. L'insuffisance cardiaque
décompensée par la grossesse et son risque de prématurité sont à craindre lors de la découverte d'un souffle cardiaque.
L'anémie doit etre recherchée et compensée. Les cervicites comportent un risque théorique d'infection des membranes et de
rupture prématurée.

Cette femme vous demande des renseignements sur la réglementation de la protection prénatale. A propos de
l'allocation du jeune enfant, vous lui répondez qu'elle est versée actuellement sous forme :
A - De mensualités
B - D'allocations prénatales et postnatales
C - D'allocations prénatales exclusivement
D - D'allocations postnatales exclusivement
E - D'allocations familiales
Bonne(s) réponse(s) : B

Les allocations prénatales sont versées par la Caisse d'Allocations Familiales en trois fractions : 2 mensualités après le 1er, 4
mensualités après le 2ème et 3 mensualités après le 3ème examen prénatal (donc reçues après la naissance).

A propos du congé post-natal, vous lui répondez qu'il ouvre droit à une indemnité journalière, et que sa durée
est de :
A - 6 semaines
B - 8 semaines
C - 10 semaines
D - 12 semaines
E - 14 semaines
Bonne(s) réponse(s) : C

Si l'accouchement a lieu à la date prévue, cette durée est de 10 semaines pour une femme ayant moins de 3 enfants. Il est de
12 semaines pour une grossesse multiple, de 18 semaines pour un 3ème enfant. Si l'accouchement a lieu plus tôt que prévu,
le congé post-natal est prolongé d'autant.

117
Exclusivement sur DOC - DZ : www.doc-dz.com NADJI 85
RESIDANAT EN POCHE TOME II
Cas Clinique en QCM

A propos du remboursement des frais de soins, vous lui répondez que les examens prénataux sont pris en
charge par l'assurance maternité, ainsi que tous les frais de soins à partir du :
A - 3ème mois
B - 4ème mois
C - 5ème mois
D - 6ème mois
E - 7ème mois
Bonne(s) réponse(s) : D

Les soins et hospitalisations au cours des 6ème, 7ème, 8ème et 9ème mois de grossesse sont pris en charge à 100% par la
Sécurité Sociale.

Mademoiselle X..., 52 ans, secrétaire de direction, a subi il y a 10 ans une mastectomie totale gauche avec curage axillaire
ayant révélé l'envahissement néoplasique d'un des six ganglions prélevés. Elle se plaint de douleurs nocturnes de la région
dorsolombaire apparues progressivement depuis deux mois. Elles sont exacerbées par les efforts de toux.
L'état général est conservé. Il existe un nodule au niveau de la cicatrice de mastectomie; son prélèvement confirme la
récidive. Le dosage des récepteurs hormonaux est positif (oestradiol et progestérone). Le sein controlatéral est d'aspect
normal. Il n'existe pas d'adénopathie axillaire, ni sus-claviculaire.
L'examen neurologique et cardio-respiratoire est normal.
Le foie n'a pas augmenté de volume.

Vous retiendriez comme facteur(s) de risque de cancer du sein chez cette malade :
A - Père décédé d'un infarctus du myocarde
B - Mère décédée d'un tumeur du sein
C - Célibataire
D - Cycles réguliers
E - Mastodynite prémenstruelle
Bonne(s) réponse(s) : B C E

Les antécédents familiaux (ascendant ou collatéraux) de cancer du sein et la nulli ou pauciparité ont été démontrés alors que
les mastodynies prémenstruelles (hyperoestrogénie relative) sont un facteur de risque encore controversé.

Cette malade est suspecte de métastases osseuses. Pour les rechercher, quel examen estimez-vous le plus
sensible ?
A - L'examen clinique
B - Le dosage de la calcémie
C - L'exploration radiologique
D - La scintigraphie squelettique
E - Le dosage des phosphatases alcalines
Bonne(s) réponse(s) : D

La scintigraphie est plus sensible que l'exploration radiologique classique mais il existe des
faux positifs qui peuvent être levés par les tomographies ou le scanner.

Parmi les signes biologiques suivants, quel est celui qui doit le plus faire suspecter l'existence de métastases
dans la moelle osseuse ?
A - Une anémie
B - Une myélémie
C - Une hypercalcémie
D - Une thrombopénie
E - Une élévation de l'ACE
Bonne(s) réponse(s) : B

L'élévation de l'ACE est non spécifique et l'hypercalcémie signe une métastase osseuse ostéolytique.
La myélémie avec érythroblastémie sont les 1ers signes. La thrombopénie est très tardive ainsi que l'anémie et sont peu
spécifiques.

Le présence de récepteurs hormonaux justifie la prescription d'une hormonothérapie médicale. Dans quel
pourcentage, une réponse thérapeutique peut-elle être espérée ?
A - 5 à 15 %
B - 20 à 30 %
C - 40 à 50 %
D - 65 à 80 %
E - 100 %
Bonne(s) réponse(s) : D

Les formes sans récepteurs ne répondent que dans 4 à 10 % des cas.

118
Exclusivement sur DOC - DZ : www.doc-dz.com NADJI 85
RESIDANAT EN POCHE TOME II
Cas Clinique en QCM

Le pronostic de ce cancer est désormais lié à un ou plusieurs des facteurs suivants :


A - A l'âge de cette malade
B - A la précocité du traitement des métastases
C - A la nature histologique de la récidive
D - A la présence de récepteurs hormonaux
E - A la rapidité de sédation des douleurs osseuses
Bonne(s) réponse(s) : A B C D E

L'âge de la malade et son état général interviennent dans la discussion de la chimiothérapie. La rapidité de sédation des
douleurs indique de façon directe la sensibilité au traitement. Les autres facteurs sont toujours valables.

Une patiente de 35 ans consulte pour un nodule de 2 cm à l'union des quadrants supérieurs et à 8 cm du mammelon.
L'examen montre un ganglion axillaire bas, palpable de 1 cm, élastique cliniquement banal. Le reste de l'examen est normal.
Après ponction, le cytodiagnostic montre des cellules carcinomateuses. A la mammographie, opacité de même taille, mal
limitée, polylobée.
Thermographie : pas de foyer hyperthermique en regard.
Le diagnostic probable est celui de cancer du sein.
L'examen anatomopathologique de la pièce opératoire montre un carcinome galactophorique infiltrant, grade 2, un ganglion
axillaire de niveau bas envahi, sans rupture capsulaire. Quatre autres ganglions présentent une réticulo-histiocytose sinusale.

Quel type histologique est le plus souvent associé à des microcalcifications ?


A - Carcinome galactophorique infiltrant
B - Colloïde
C - Maladie de paget
D - Médullaire à stroma lymphoïde
E - Comédocarcinome
Bonne(s) réponse(s) : E

L'obstruction par des cellules tumorales de la lumière canalaire avec des remaniements nécrotiques du centre sont le
substratum des calcifications en batônnets retrouvées sur la mammographie.

Quelle est la classification clinique TNM ?


A - T1 N0 MX
B - T1 N1a MX
C - T1 N1b M0
D - T2 N1a M0
E - T2 N1b MX
Bonne(s) réponse(s) : B

T1 correspond à une tumeur inférieure ou égale à 2 cm cliniquement. N1a correspond à un ou plusieurs ganglions axillaires
homolatéraux mobiles considérés comme non envahis.

Les 3 localisations le plus souvent envahies par des métastases du cancer du sein sont :
A - Squelette distal des membres
B - Le foie
C - Le cerveau
D - Le poumon
E - Le squelette axial
Bonne(s) réponse(s) : B D E

Le squelette distal est exceptionnellement atteint et le cerveau est atteint plus tardivement.

Le bilan d'extension comporte de manière indispensable :


A - Electrocardiogramme
B - Radio pulmonaire
C - Recherche des récepteurs hormonaux
D - Scintigraphie cérébrale
E - Scintigraphie du corps entier
Bonne(s) réponse(s) : B E

L'électrocardiogramme est la recherche de récepteurs hormonaux sont utiles mais ne font pas partie du bilan d'extension
proprement dit. La scintigraphie cérébrale, remplacée actuellement par le scanner, n'est pas systématique.

119
Exclusivement sur DOC - DZ : www.doc-dz.com NADJI 85
RESIDANAT EN POCHE TOME II
Cas Clinique en QCM

L'approche chirurgicale habituellement proposée est :


A - Contre-indication chirurgicale
B - Tumorectomie simple
C - Mastectomie totale
D - Mastectomie élargie
E - Segmentectomie, curage axillaire
Bonne(s) réponse(s) : E

Dans des tumeurs de moins de 3 cm cliniquement, le traitement est conservateur, associant tumorectomie et curage axillaire.

Le facteur essentiel du pronostic est :


A - Le type histologique
B - Le grade d'histopronostic
C - L'envahissement ganglionnaire
D - Le niveau bas de l'envahissement
E - La réticulo-histiocytose sinusale
Bonne(s) réponse(s) : B

Les 3 caractères les plus discriminants sont par ordre hiérarchiques décroissant:
- le grade histopronostique de Scarff et Bloom,
- le stade clinique (T),
- le nombre de ganglions axillaires histologiquement envahis.

Lequel des traitements suivants est indiqué pour diminuer le risque de récidive locale ?
A - Chimiothérapie
B - Tamoxifène
C - Chimiothérapie - hormonothérapie
D - Radiothérapie externe
E - Progestatifs
Bonne(s) réponse(s) : D

La radiothérapie externe permet de diminuer la récidive dans le reste de la glande et sur la cicatrice. Elle est effectuée
systématiquement (en moyenne, elle utilise 45 gray sur le sein et 15 gray de surimpression du lit tumoral) en cas de chirurgie
conservatrice.

Une jeune femme de 25 ans, primigeste, antérieurement bien réglée, du groupe A Rh+, mesurant 1m56, vous consulte à 33
semaines d'aménorrhée pour l'examen prénatal du 8ème mois. Elle était, au préalable, suivie par différents confrères qui
n'auraient, jusque là, rien noté d'anormal.
Elle était, avant la grossesse, immunisée contre la rubéole et la toxoplasmose et pesait 69 kg. Elle n'a subi qu'un examen
échographique à 21 semaines, qui s'était avéré normal.
Vous constatez lors de votre examen une prise de poids de 15 kg depuis le début de la grossesse, des jambes infiltrées, une
tension artérielle à 14/9 cm de Hg, un pouls à 80/mn, une hauteur utérine à 34 cm, une présentation céphalique fixée, un col
peu modifié au toucher vaginal.
Les bruits du coeur sont bien perçus.
Cette patiente exerce le métier de vendeuse dans un grand magasin, se plaint de fatigue intense en fin de journée. Elle n'a
aucun antécédent pathologique particulier et n'a fait, jusqu'à ce jour, l'objet d'aucune prescription thérapeutique.

Parmi les différents éléments signalés, lequel aurait dû, jusque là, mériter plus d'attention, au cours de la
surveillance prénatale ?
A - La parité
B - La profession
C - La taille
D - Le poids
E - Le terme de la grossesse
Bonne(s) réponse(s) : D

Il existe une surcharge pondérale importante de 14,5 kg (normes du poids idéal pour la taille, facteur de risque pour la
grossesse de décompensation d'un diabète latent). La profession est facteur de risque de prématurité mais celle-ci n'apparaît
pas dans le cas décrit.

120
Exclusivement sur DOC - DZ : www.doc-dz.com NADJI 85
RESIDANAT EN POCHE TOME II
Cas Clinique en QCM

Parmi les examens complémentaires suivants, lequel aurait-il été souhaitable de réaliser ?
A - Une échographie vers 8 semaines
B - Une recherche d'agglutinines irrégulières à la 17ème semaine
C - Une protéinurie à la 20ème semaine
D - Un contrôle du métabolisme glucidique
E - Une radiopelvimétrie à 30 semaines
Bonne(s) réponse(s) : D

Le contrôle du métabolisme glucidique n'est pas rendu obligatoire par la loi. Il doit donc être demandé par le médecin en cas
d'ATCD personnels ou familiaux d'intolérance au glucose et devant une obésité ou un gros bébé.
La radiopelvimétrie se discute chez les femmes de taille inférieure à 1,55 m.

Au cours de votre consultation, quel diagnostic allez-vous évoquer ?


A - Une erreur de terme
B - Une grossesse sur utérus fibromateux
C - Une phlébite débutante
D - Une malformation foetale
E - Un syndrome vasculo-rénal débutant
Bonne(s) réponse(s) : E

Le texte laisse entendre qu'il existe une macrosomie diabétique associée à un syndrome vasculorénal débutant (tension
artérielle limite, oedèmes des membres inférieurs et prise de poids importante).
Rappel : on parle d'HTA gravidique pour des chiffres > 14/9.

Sur les résultats suivants d'examens complémentaires, lequel vous paraît anormal, à 33 semaines ?
A - L'existence d'un point de Béclard visible à l'échographie
B - Une protéinurie à 0,10 g/24 h
C - Une uricémie à 450 micromoles par litre
D - Une hémoglobinie à 112 g/l
E - Un ECBU avec quelques leucocytes et 103 bacilles gram négatif
Bonne(s) réponse(s) : C

L'uricémie est normalement inférieure à 240 µmol/l et est le meilleur témoin biologique de
l'importance du syndrome toxémique.
Un point de Béclard est visible à l'échographie à partir de 33 S.A.
Une bactériurie < à 103 sans pyurie est non significative et doit faire recommencer l'ECBU.
Il existe une anémie physiologique de la femme enceinte par hémodilution, Hb > 11 g/l.
Une protéinurie à 0,10 g/24 h est fréquemment une souillure et doit faire répéter l'examen dans de bonnes conditions de
prélèvement. Elle est significative si > à 0,30 g/l.

Vous demandez un enregistrement du rythme cardiaque foetal. Quel résultat paraîtrait le plus rassurant ?
A - Une fréquence cardiaque à 180 battements par minute (bpm)
B - Un rythme à 160 bpm avec des oscillations à 2 bpm
C - Un tracé plat, sans ralentissement et sans contractions utérines avec rythme à 140 bpm
D - Un rythme à 140 bpm, avec un ralentissement à 80 bpm lors de chaque contraction utérine
E - Un rythme à 140 bpm avec oscillation à 10 bpm, sans ralentissement
Bonne(s) réponse(s) : E

Le tracé normal associe un niveau de base compris entre 120 et 140 b.p.m, des fluctuations > 5 b.p.m, et des accélérations
lors des mouvements à l'exclusion de tout ralentissement.

Avant d'envisager une prescription, parmi les renseignements suivants, lequel vous parait inutile ?
A - Hyperglycémie provoquée
B - Enquête alimentaire
C - Mensurations foetales échographiques
D - Recherche de la maturité pulmonaire foetale
E.- Etude du rythme cardiaque foetal
Bonne(s) réponse(s) : D

La recherche de la maturité pulmonaire foetale ne se discute qu'en cas d'indication d'extraction prématurée pour sauvetage
maternel et/ou foetal, ce que ne laisse pas entendre l'énoncé.

121
Exclusivement sur DOC - DZ : www.doc-dz.com NADJI 85
RESIDANAT EN POCHE TOME II
Cas Clinique en QCM
Une femme enceinte de 8 mois, mal surveillée pendant la grossesse, est hospitalisée en urgence pour un hématome
rétroplacentaire. Parmi les examens biologiques que vous avez demandés, le bilan d'hémostase est le suivant :
- Temps de Quick 45 %
- Temps de céphaline activé : 42 sec (35 sec)
- V : 40 %
- VII + X : 85 %
- II : 45 %
- Fibrinogène : 0,50 g/l
- Plaquettes : 110000/mm3.

Compte-tenu du contexte clinique, quel est le diagnostic le plus probable de ce trouble de la coagulation ?
A - Déficit en vitamine K de la grossesse
B - Hépatite fulminante
C - Fibrinolyse aiguë
D - Thrombocytopénie par hémodilution
E - Coagulopathie de consommation
Bonne(s) réponse(s) : C

Qualitativement, on ne peut distinguer sur ces seules données la coagulopathie de consommation de la fibrinolyse qui sont les
principales complications de l'hématome rétroplacentaire, ce n'est que quantitativement devant l'importance de la baisse du
fibrinogène que l'on suspecte cette dernière.

Parmi les examens complémentaires suivants, lequel(lesquels) permettrai(en)t, le plus sûrement, de confirmer
ce diagnostic ?
A - Dosage du facteur Willebrand (VIII R :Ag)
B - Dosage du facteur antihémophilique B
C - Dosage des produits de dégradation du fibrinogène
D - Recherche de complexes solubles (test à l'éthanol)
E - Mesure du temps de saignement
Bonne(s) réponse(s) : C D

Le dosage des produits de dégradation du fibrinogène (détruit par la plasmine qui est augmentée dans la fibrinolyse) dont
certains fragments peuvent former des complexes avec le fibrinogène ou la fibrine monomère, détectés par le test à l'éthanol,
permettent d'étayer le diagnostic.

L'utérus de cette femme doit être évacué, mais vous craignez une hémorragie de la délivrance. Parmi les
éléments de son bilan d'hémostase, lequel permet effectivement de prévoir ce risque ?
A - Thrombocytopénie à 110000/mm3
B - Fibrinogène à 0,50 g/l
C - L'allongement du TCA
D - Taux d'accélérine à 40 %
E - Taux de prothrombine à 45 %
Bonne(s) réponse(s) : B

Le critère de gravité est ici la diminution importante du fibrinogène < 1 g/l (Nale = 5 g/l). Les autres critères sont thrombopénie
< 50 000.

Parmi les mesures thérapeutiques suivantes destinées à éviter l'hémorragie de la délivrance, laquelle
choisissez-vous ?
A - Cinq flacons de 20 ml de PPSB
B - Perfusion intraveineuse d'un antifibrinolytique
C - Injection immédiate de 100 u/kg d'héparine standard
D - Quatre unités de plasma frais congelé plus 4 grammes de fibrinogène purifié
E - Un flacon de 250 000 unité d'urokinase plus 100 u/kg d'héparine standard
Bonne(s) réponse(s) : D

Le plasma frais congelé apporte l'ensemble des facteurs de la coagulation et les inhibiteurs physiologiques marquant. La dose
habituelle est de 10 à 15 ml/kg. Les antifibrinolytiques ne s'utilisent pas en préventif. Le PPSB contient souvent des facteurs
activés, donc est déconseillé. L'héparine et l'urokinase ne sont pas utilisées en raison du risque hémorragique.

122
Exclusivement sur DOC - DZ : www.doc-dz.com NADJI 85
RESIDANAT EN POCHE TOME II
Cas Clinique en QCM

L'évacuation utérine se déroule normalement, l'hémostase est normale et 5000 u d'héparine SC 2 fois/j sont
prescrites ; à J 5 une phlébite fémoro-poplitée se déclare ; les plaquettes sont à 450.000/mm3. Dès lors vous
choisissez de :
A - Ajouter de l'aspirine (500 mg/jour) à l'héparine sous cutanée
B - Augmenter la posologie de l'héparine sous cutanée : 5 000 unités x 3 par jour
C - Arrêter l'héparine sous cutanée et passer aux antivitamines K
D - Arrêter l'héparine sous cutanée et prescrire de l'héparine intra veineuse à la dose de 200 unités/kg/24
heures
E - Arrêter l'héparine sous-cutanée et prescrire de l'héparine intra-veineuse à la dose de 600 unités/kg/24
heures
Bonne(s) réponse(s) : E

En raison des taux plasmatiques élevés de facteur 4 plaquettaire antihéparinique, de fortes doses d'héparine sont nécessaires.

Quel(s) est (sont) le(s) examen(s) le(s) plus utile(s) pour surveiller l'efficacité biologique du traitement prescrit ?
A - Mesure du temps de saignement
B - Dosage de l'antithrombine III
C - Temps de Quick
D - Temps de céphaline activé
E - Détermination de l'héparinémie circulante
Bonne(s) réponse(s) : D E

Le taux d'héparinémie circulante doit être mesuré compte-tenu de la dégradation de l'héparine par le facteur 4 plaquettaire.

Cette patiente de 32 ans a accouché trois fois dans des conditions normales. Depuis deux ans, il a été mis en place un
dispositif intra-utérin qui était bien supporté jusqu'à ces derniers mois. Des douleurs pelviennes sont apparues, majorées en
période menstruelle puis des métrorragies qui sont permanentes depuis un mois. Les dernières règles normales sont
apparues 15 jours avant la période de saignement. Il existe d'autre part une fébricule à 38° et quelques nausées. L'examen
clinique retrouve : au spéculum : des métrorragies d'origine endo-utérine, les fils du stérilet sont visibles, un utérus un peu
gros, légèrement sensible à la mobilisation, les deux annexes sont légèrement sensibles mais le cul de sac de Douglas est
indolore.

Dans l'hypothèse d'un bêta-HCG plasmatique positif à 250 mUI/ml, si l'échographie met en évidence des
images intra-cavitaires inhomogènes non structurées, il peut s'agir :
A - D'une grossesse intra-utérine avec nidation abortive précoce
B - D'une grossesse molaire
C - D'une grossesse extra-utérine
D - D'une endométriose
E - D'un myome intra-cavitaire
Bonne(s) réponse(s) : A B C

La présence du stérilet n'exclut pas la possibilité de grossesse intra-utérine, ni de grossesse môlaire qui donne des images
échographiques en "tempête de neige". Une grossesse extra-utérine peut s'accompagner d'images intra-utérines trompeuses
correspondant à la caduque.

Si l'échographie met en évidence une grossesse intra-utérine évolutive avec un stérilet en situation isthmique, il
faut :
A - Enlever le stérilet
B - Réaliser un avortement médical
C - Prescrire une antibiothérapie
D - Réaliser une cerclage du col utérin
E - Prescrire un traitement hormonal estroprogestatif
Bonne(s) réponse(s) : A C

Le risque morbide est supérieur lorsque le stérilet est laissé en place et entretient l'infection. Il faut, quand c'est possible,
l'enlever en sachant que le risque abortif est important. L'avortement thérapeutique n'est pas justifié car si elle évolue, la
grossesse évoluera normalement. Toute fièvre en cours de grossesse doit être traitée par une antibiothérapie adaptée.

Dans l'hypothèse d'un bêta-HCG plasmatique négatif, vous devez demander :


A - Echographie
B - Vitesse de sédimentation
C - Bactériologie endocervicale
D - Hystéroscopie
E - Hystérographie
Bonne(s) réponse(s) : A B C

En l'absence de grossesse on suspecte une infection génitale haute. Toute manoeuvre endo-utérine invasive est contre-
indiquée.
123
Exclusivement sur DOC - DZ : www.doc-dz.com NADJI 85
RESIDANAT EN POCHE TOME II
Cas Clinique en QCM

Toujours dans l'hypothèse d'un bêta-HCG plasmatique négatif, la coelioscopie peut être réalisée afin de
rechercher certaine(s) étiologie(s) possible(s) :
A - Grossesse extra-utérine
B - Salpingite
C - Perforation utérine
D - Synéchie
E - Syndrome de Fitz-Hugh Curtis (Péri-Hépatite)
Bonne(s) réponse(s) : B C E

La négativité des béta HCG exclut dans ce contexte la grossesse extra-utérine. Une synéchie utérine n'est pas visible par la
coelioscopie. Le syndrome de Fitz-Hugh-Curtis doit toujours être recherché dans le bilan coelioscopique d'une salpingite, les
germes retrouvés sont chlamydia et gonocoque.

Une femme de 58 ans consulte pour un bilan mammaire systématique. La mère de la patiente a été traitée pour cancer du
sein à l'âge de 50 ans. Sur le plan personnel, les premières règles sont survenues à l'âge de 15 ans, la patiente a eu 2 enfants
à l'âge de 28 et 30 ans qu'elle n'a pas allaités. Deux ans plus tôt (56 ans), une hystérectomie totale avec castration a été
réalisée pour de volumineux fibromes ; la patiente avait des cycles réguliers avant l'intervention. Le bilan mammaire est
demandé, car cette patiente bénéficie d'une hormonothérapie substitutive. L'examen clinique note un bon état général. La
palpation des seins est strictement normale. La mammographie met en évidence du coté droit sur le rayon de 3 H un amas de
microcalcifications nombreuses, denses.

Quel(s) facteur(s) de risque de cancer du sein retrouvez-vous chez cette patiente ?


A - Antécédents familiaux
B - L'âge aux premières règles
C - L'âge à la première grossesse
D - Hystérectomie pour fibrome
E - L'âge à la ménopause
Bonne(s) réponse(s) : A E

Les antécédents familiaux et la ménopause tardive > 55 ans augmentent le risque, qui est augmenté également lorsque la
1ère grossesse survient après 35 ans et si les 1ères règles sont précoces avant 13 ans.

Parmi les critères radiologiques significatifs de malignité en ce qui concerne les microcalcifications, vous
retenez :
A - Différence de densité
B - Groupement étroit en amas
C - Situation rétromammelonnaire
D - Nombre important
E - Toutes les propositions sont exactes
Bonne(s) réponse(s) : E

Le caractère malin est suspecté sur les nombreuses microcalcifications (0,1 à 0,3 mm) groupées en foyer, denses et en
situation rétromamelonnaire (cancer intra-galactophorique, comédocarcinome).

Quelle attitude thérapeutique adopteriez-vous pour cette patiente parmi les propositions suivantes ?
A - Mastectomie sous-cutanée
B - Prélèvement de la zone de microcalcifications après repérage radiologique et examen histopathologique
C - Tumorectomie + curage mammaire interne
D - Mastectomie partielle avec curage axillaire
E - Mastectomie totale avec curage axillaire
Bonne(s) réponse(s) : B

Toute zone radiologiquement suspecte doit être prélevée pour examen anatomopathologique afin d'affirmer le diagnostic
avant de pratiquer une chirurgie radicale partielle ou totale.

124
Exclusivement sur DOC - DZ : www.doc-dz.com NADJI 85
RESIDANAT EN POCHE TOME II
Cas Clinique en QCM

Vous avez opéré cette patiente et le compte-rendu anatomo-pathologique vous indique le score histo-
pronostique de Scarff-Bloom et Richardson (SBR). Ce score de SBR :
A - Est un des facteurs de pronostic permettant d'apprécier l'évolutivité d'un épithélioma mammaire
B - Est fiable à lui seul pour apprécier l'évolutivité d'un épithélioma mammaire
C - Est basé sur l'examen macroscopique de la pièce opératoire
D - Un grade III de SBR implique la présence d'envahissement ganglionnaire axillaire
E - Un cancer de grade I de SBR est hormono dépendant
Bonne(s) réponse(s) : A

Le score histopronostique de Scarf et Bloom est le meilleur critère permettant d'apprécier le


pronostic de la tumeur avec le T (T.N.M.) de la tumeur et le nombre de ganglions envahis.
Il est basé sur l'analyse microscopique qui apprécie :
- le degré de différenciation (tube),
- le degré d'anisonucléose,
- l'importance de l'activité mitotique.

Le bi!an d'extension de ce cancer du sein comporte dès lors en première intention :


A - Le dosage de l'ACE
B - Un scanner cérébral
C - Des clichés de squelette axial
D - Une échographie hépatique
E - Des tomographies pulmonaires
Bonne(s) réponse(s) : A C D E

L'A.C.E. est positif dans 20 % des formes non métastasées initialement.


L'échographie hépatique et le bilan biologique hépatique (phosphatases alcalines, gamma GT) élimine une métastase muette
cliniquement.
Les clichés du squelette axial remplacés actuellement par la scintigraphie osseuse au technétium et les tomographies
pulmonaires recherchent les 2 autres localisations métastatiques fréquentes.
Le scanner cérébral n'est pratiqué qu'en cas d'anomalie clinique.

Une patiente de 28 ans a accouché il y a 8 mois, et présente une hémorragie de la délivrance. La lactation fut bloquée par la
prescription immédiate de bromocriptine et une contraception oestro-progestative commencée à la sortie de la maternité fut
poursuivie 4 mois. Elle consulte pour aménorrhée depuis l'arrêt de la pilule. L'interrogatoire relève la notion de "ballonnements
abdominaux" et constipation et l'examen général et gynécologique est normal en dehors d'une pâleur.

En plus du dosage de la prolactinémie, vous demandez également :


A - Echographie pelvienne
B - Biopsie d'endomètre
C - Test au LHRH
D - Dosage HCG
E - Scanner crânien
Bonne(s) réponse(s) : C D E

Une 2ème grossesse doit être éliminée par la recherche d'HCG plasmatiques.
Compte-tenu de l'hémorragie de la délivrance (état de collapsus possiblement responsable d'une nécrose ischémique
hypophysaire), on évoque un syndrome de Sheehan et on explore les fonctions hypophysaires par le test au TRH (négatif
dans ce cas) et le scanner crânien visualisant la nécrose ; la constipation et la pâleur sont compatibles avec l'hypopituitarisme.
Une étiologie mécanique (synéchie) ne serait pas diagnostiquée par l'échographie ou la biopsie d'endomètre mais par l'H.S.G.

Le dosage de la prolactine révèle un taux élevé à 200 ng/ml. Ce résultat :


A - Est pathognomonique d'un adénome à prolactine
B - N'évoque pas l'adénome à PRL car les valeurs sont trop basses
C - N'a pas forcément de signification pathologique 8 mois après l'accouchement
D - Peut se voir au cours de perturbations thyroïdiennes
E - Commande la réalisation de radiographies et scanner hypophysaires
Bonne(s) réponse(s) : E

Le syndrome d'aménorrhée, surtout s'il est associé à une galactorrhée, doit faire rechercher une hyperprolactinémie. Les
adénomes à prolactine se développent électivement dans le post-partum mais il peut aussi s'agir d'une hyperprolactinémie
fonctionnelle. La normale est > 30 mg/ml et < 100 mg/ml. Au delà de 350 mg/ml, le taux est quasi pathognomonique d'un
adénome à P.R.L. En dessous de ce chiffre, il existe des cas d'hyperprolactinémie fonctionnelle qui néanmoins commande
l'exploration de la selle turcique.

125
Exclusivement sur DOC - DZ : www.doc-dz.com NADJI 85
RESIDANAT EN POCHE TOME II
Cas Clinique en QCM

Les notions de constipation, pâleur et hypercholestérolémie constatée lors d'un bilan bio!ogique réalisé en ville,
font réaliser des dosages hormonaux thyroïdiens qui donnent des valeurs basses de T3 et T4 avec valeurs
élevées de TSH. Ce résultat :
A - Est évocateur d une nécrose hypophysaire du post-partum (syndrome de Sheehan)
B - Peut être en relation avec la prise d'oestroprogestatifs dans les mois précédents
C - Evoque la présence d'un adénome à prolactine entraînant un retentissement hypophysaire thyréotrope
D - Signe une insuffisance thyroïdienne périphérique
E - Traduit une maladie de Basedow
Bonne(s) réponse(s) : D

Les valeurs élevées de la TSH signe l'origine basse de l'hypothyroïdie. Il n'existe pas d'hypothyroïdie secondaire à la prise
d'oestroprogestatifs.

Le scanner hypophysaire est normal. Le diagnostic proposé est celui de thyroïdite du post-partum et l'on
propose un traitement par :
A - Progestatifs normodosés 21 jours par mois
B - Hormonothérapie thyroïdienne substitutive
C - Thyroïdectomie
D - Anti-thyroïdiens de synthèse
E - Des oestrogènes seuls 14 jours par mois
Bonne(s) réponse(s) : B

Les thyroïdites peuvent entraîner un dysfonctionnement hypothalamo-hypophysaire. L'insuffisance thyroïdienne nécessite un


traitement hormonal substitutif. Le rétablissement de cette fonction devrait permettre la reprise de cycle ovulatoire.

Si une grossesse survient sous traitement, il faut :


A - Réaliser une interruption médicale de grossesse
B - Interrompre le traitement en cours, même si la patiente n'est pas euthyroïdienne
C - Prescrire de la progestérone dès la constatation de la grossesse
D - Adapter le traitement en appréciant cliniquement et biologiquement l'état thyroïdien maternel
E - Contre-indiquer la prescription de bromocriptine dans le post-partum
Bonne(s) réponse(s) : D

L'insuffisance thyroïdienne ne contre-indique pas la grossesse mais doit être parfaitement équilibrée afin d'éviter un
retentissement foetal, en particulier cérébral. Les doses doivent en pratique être augmentées par rapport à la période
précédent la grossesse.

Une femme de 40 ans, mère de 5 enfants, consulte pour des saignements en dehors des règles et provoqués par les
rapports. L'examen au spéculum met en évidence un col saignant au contact et porteur d'un bourgeon sur la lèvre postérieure
évoquant un cancer.

Par lequel des moyens suivants confirmez-vous le diagnostic ?


A - Conisation
B - Curetage étagé
C - Hystérographie
D - Colposcopie
E - Biopsie
Bonne(s) réponse(s) : E

L'examen anatomopathologique permet seul d'affirmer le diagnostic de malignité et permet d'adapter le geste thérapeutique.
En l'absence de lésion évidente, la biopsie se pratique sous colposcopie.

Parmi les différents types histologiques citez quel est celui qui a le plus de chances d'être retrouvé chez cette
patiente ?
A - Adénocarcinome
B - Carcinome épidermoïde
C - Leiomyosarcome
D - Cancer à cellules claires
E - Cancer spino-cellulaire
Bonne(s) réponse(s) : B

Le revêtement de l'exocol est de type malpighien non kératinisé. Les adénocarcinomes du col se développent à partir du
contingent glandulaire ou de l'épithélium cylindrique de l'endocol.

126
Exclusivement sur DOC - DZ : www.doc-dz.com NADJI 85
RESIDANAT EN POCHE TOME II
Cas Clinique en QCM

Parmi les examens suivants, lequel(lesquels) est(sont) indispensable(s) pour préciser le stade de ce cancer ?
A - Lymphographie
B - Urographie intra-veineuse
C - Cystoscopie
D - Examen sous anesthésie générale
E - Scintigraphie osseuse
Bonne(s) réponse(s) : A B C D

Le stade clinique nécessite l'examen sous A.G. afin d'apprécier l'infiltration des paramètres et
des cloisons vaginales antérieures et postérieures.
La cystoscopie détermine l'envahissement éventuel de la muqueuse vésicale (stade 4) et l'urographie une compression
urétérale éventuelle. Les métastases osseuses sont tardives.
La lymphographie apprécie l'extension ganglionnaire (N) iliaque externe et lomboaortique.
Le caractère indispensable de ces examens est apprécié différemment par les équipes.

Au terme de ces examens, le cancer apparait limité au seul massif cervical. Quel stade devra lui être attribué ?
A - Ia
B - Ib
C - IIa
D - IIb
E - III
Bonne(s) réponse(s) : B

Il s'agit d'un cancer invasif clinique (1b) limité au col.


1a correspond à un cancer invasif préclinique (découverte de dépistage).
On parle du stade 2 proximal ou distal et non 2a et 2b.

Parmi les options thérapeutiques suivantes laquelle(lesquelles) est(sont) possible(s) chez cette patiente ?
A - Colpohystérectomie élargie et Iymphadénectomie
B - Curiethérapie puis colpohystérectomie élargie et Iymphadénectomie
C - Radiothérapie exclusive
D - Hystérectomie totale sans conservation des ovaires
E - Curiethérapie et conisation
Bonne(s) réponse(s) : A B

Dans les stades 1 et 2 proximal, le protocole habituel associe une curiethérapie première suivie
6 semaines après d'une lymphadénocolpohystérectomie élargie type Wertheïm, suivie d'une
radiothérapie externe en cas d'envahissement ganglionnaire.
Certaines équipes préconisent la chirurgie d'emblée.
La conisation et l'hystérectomie ne se discutent qu'au stade de carcinome in situ. La radiothérapie exclusive est réservée aux
stades 2 distaux et 3.

Madame Simone S. a 32 ans, elle vit dans un hameau de campagne. Sa première grossesse, il y a 3 ans, a été normale
jusqu'au 7ème mois. A ce moment, elle a présenté des oedèmes, une protéinurie à 4 g/24 heures et une élévation
tensionnelle. Elle a alors été mutée au C.H.U. voisin et a accouché à 34 semaines par césarienne d'une fille actuellement en
bonne santé. La protéinurie a disparu dans la semaine qui a suivi l'accouchement. La patiente a reçu pendant deux mois
encore clonidine (Catapressan®) pour sa tension artérielle ; traitement qu'elle a spontanément abandonné.

Le diagnostic de cet état est :


A - Glomérulonéphrite aiguë
B - Néphrose lipidique
C - Hypertension artérielle primitive
D - Pyélonéphrite aiguë
E - Etat pré-éclamptique
Bonne(s) réponse(s) : E

L'association de l'HTA, des oedèmes et de la protéinurie chez une primipare au 3ème trimestre est caractéristique de la
toxémie gravidique ou préclampsie.

127
Exclusivement sur DOC - DZ : www.doc-dz.com NADJI 85
RESIDANAT EN POCHE TOME II
Cas Clinique en QCM

Parmi les propositions suivantes, choisissez la(les) proposition(s) exacte(s) :


A - L'évolution se fait habituellement vers une disparition de la protéinurie
B - En l'absence de traitement, le risque de crise convulsive est important
C - La mort foetale amène souvent une sédation des symptômes maternels
D - Les oedèmes traduisent dans ces cas la compression de la veine cave par l'utérus gravide
E - L'hypertension dans ce cadre s'accompagne souvent d'un fond d'oeil de stade III ou IV
Bonne(s) réponse(s) : B

En l'absence de traitement, le tableau évolue vers l'aggravation avec risque de mort foetale et de mort maternelle par
éclampsie (HTA maligne). En l'absence d'anomalie antérieure, le retentissement oculaire de l'HTA dépasse rarement le stade
2. Les oedèmes sont retentionnels par extravasation plasmatique périphérique. La mort foetale aggrave souvent le tableau
avec le risque surajouté de fibrinolyse.

La décision d'accouchement prématuré dans un tel cas est prise :


A - Sur la constatation d'une maturité foetale suffisante
B - Sur un ralentissement du rythme cardiaque foetal
C - Sur la notion d'antécédents familiaux de toxémie gravidique
D - Sur la constatation de variation tensionnelle importante malgré le traitement
E - Sur la notion d'une activité rénine élevée
Bonne(s) réponse(s) : A B D

L'extraction foetale est impérative en cas d'échappement au traitement ou de souffrance foetale aiguë. En l'absence d'urgence
maternelle ou foetale, on temporisera jusqu'à obtention de la maturité pulmonaire.

La patiente est revue actuellement au 4ème mois d'une deuxième grossesse.


Les chiffres de pression artérielle sont de 140/100 mm de mercure. Elle ne présente aucun symptôme. Il n'y a
pas de protéinurie.
Au plan du pronostic, il serait défavorable de constater :
A - Hypo-uricémie
B - Elévation modérée de la filtration glomérulaire, signe d'une hyperfiltration
C - Volume plasmatique élevé par rapport à Ia mesure théorique avant grossesse
D - Activité rénine plasmatique augmentée de 20 % par rapport à la valeur théorique avant grossesse
E - Baisse du flux plasmatique rénal
Bonne(s) réponse(s) : E

La baisse du flux plasmatique rénal (augmenté ainsi que le volume plasmatique dans une grossesse normale) manifesté par
une hyperuricémie est le mécanisme physiopathologique de la toxémie gravidique et serait donc de mauvais pronostic dans le
cadre d'une hypertension précoce.

Une bactériurie est constatée alors que la patiente n'a aucun antécédent d'infection urinaire. Cette bactériurie :
A - Est pratiquement le signe d'une malformation rénale dans ce contexte
B - Doit être traitée du fait du risque de pyélonéphrite gravidique
C - Peut être négligée s'il s'agit d'une infection à germe gram -
D - Explique l'absence d'atonie physiologique des voies excrétrices
E - Accompagne souvent un défaut de concentration à l'exploration fonctionnelle rénale
Bonne(s) réponse(s) : B

Les bactériuries sont fréquentes en cours de grossesse et doivent toujours être traitées lorsqu'elles sont significatives. Seule
une infection urinaire haute doit faire rechercher une malformation urinaire en sachant que l'imbibition hormonale entraîne une
atonie urétérale et que la compression par l'utérus gravide altère le système anti-reflux.

Devant une hypertension artérielle dans ce contexte :


A - Le traitement est inutile, l'élévation de la minima est physiologique au cours du 2ème trimestre
B - Le risque de complication foetale placentaire augmente avec le chiffre tensionnel
C - L'abaissement tensionnel requis rend inutile le repos, sauf exception
D - L'interruption thérapeutique de grossesse est inévitable à ce stade
E - L'accouchement doit être prévu à proximité d'un centre de néonatologie
Bonne(s) réponse(s) : B E

La grossesse peut être poursuivie à condition d'être étroitement surveillée et traitée. L'hospitalisation en milieu spécialisé
s'impose à terme pour programmer l'accouchement. Le repos est impératif jusqu'à l'accouchement

128
Exclusivement sur DOC - DZ : www.doc-dz.com NADJI 85
RESIDANAT EN POCHE TOME II
Cas Clinique en QCM
Madame B., 35 ans, consulte pour leucorrhées abondantes, blanchâtres, crémeuses, existant depuis 2 jours, s'accompagnant
d'un prurit vulvaire, de brûlures vaginales et de dyspareunie.
Vous évoquez une vulvo-vaginite candidosique.

Que pouvez-vous constater lors de l'examen au spéculum ?


A - A la mise en place du spéculum : inflammation de la vulve et du méat urinaire
B - Muqueuse érythémateuse et exulcérée recouverte d'un dépôt grumeleux d'aspect cailleboté
C - Cervicite
D - Muqueuse avec piqueté hémorragique
E - Odeur nauséabonde
Bonne(s) réponse(s) : A B C

La vulve est souvent rouge oedèmateuse avec des granulations blanchâtres sur la muqueuse vaginale, plus rarement
associée à une exocervicite, odeur de "plâtre frais". Les lésions sont prurigineuses et douloureuses (dyspareunie, dysurie). Le
piqueté muqueux hémorragique évoque le trichomonas. L'odeur nauséabonde est caractéristique des infections à anaérobies.

Quel(s) est (sont) le(s) examen(s) pouvant permettre d'affirmer le diagnostic de vulvo-vaginite à Candida
albicans ?
A - Prélèvement à l'écouvillon stérile et examen direct après coloration au May-Grunwald-Giemsa
B - Ensemencement sur milieu gélosé et test de blastèse
C - Ensemencement sur milieu RAT ou PCB
D - Mise en culture et pratique d'auxanogramme et de zymogramme
E - Aucun des examens ci-dessus
Bonne(s) réponse(s) : A B C D

L'examen direct après coloration permet de visualiser les filaments mycéliens et des blastospores. Le diagnostic est confirmé
par la culture sur milieux usuels ou milieux plus simples, type Sabouraud, complétée par l'auxanogramme et zymogramme qui
sont des tests d'identification qui permettent d'affirmer l'espèce candida albicans (seule pathogène).
Les cultures sur milieu R.A.T. ou P.C.B. sont moins spécifiques. Le test de Blastèse permet une identification présomptive de
l'espèce albicans (rapide, moins spécifique).

Devant une candidose muqueuse, quel(s) médicament(s) doit (doivent) être recherché(s) en tant que cause
favorisante possible ?
A - Progestatifs
B - Oestrogènes
C - Tétracyclines
D - Psychotropes anticholinergiques
E - Antimitotiques
Bonne(s) réponse(s) : B C E

Les terrains immunodéficients consécutifs aux traitements antimitotiques, les antibiotiques à


large spectre (tétracyclines) qui déséquilibrent la flore vaginale et les oestrogènes qui
augmentent l'acidité vaginale favorisent les infections candidosiques.

Quel(s) conseil(s) allez-vous prodiguer à cette patiente dans l'immédiat ?


A - Utilisation de savon type "savon de Marseille"
B - Toilette suivie d'une application de Bétadine gynécologique (R) (polyvinyl iodé)
C - Changement quotidien de lingerie
D - Simple surveillance du partenaire
E - Abstinence sexuelle tant qu'il persiste des signes fonctionnels
Bonne(s) réponse(s) : A C D E

Les savons acides entretiennent l'infection et favorisent les récidives. Il faut les remplacer par
le savon de Marseille.

129
Exclusivement sur DOC - DZ : www.doc-dz.com NADJI 85
RESIDANAT EN POCHE TOME II
Cas Clinique en QCM

Quel schéma thérapeutique adoptez-vous ?


A - .Mycostatine®(12 dragées/jour en 3 prises)
B - Gynodaktarin® (1 ovule le soir) pendant 14 jours
B' - Daktarin® seul (4 comprimés/jour) pendant 10 jours

C - Gynopévaryl® (1 ovule le soir) pendant 21 jours


D - Gynodaktarin® (1 ovule le soir) pendant 14 jours
E - Traitement minute : Fazol G® (1 ovule 3 soirs de suite)
Bonne(s) réponse(s) : E

Il s'agit d'une première manifestation.


On peut proposer en 1ère intention un traitement court par voie locale.
Le traitement par voie générale ne se discute qu'en cas de récidive ou de persistance des
lésions.

Une jeune femme de 20 ans, enceinte de 3 mois, vous consulte pour une suspicion d'herpès génital. Elle a depuis 2 jours trois
lésions vésiculeuses, actuellement ulcérées, de 2 mm de diamètre chacune, sur la face externe de la petite lèvre droite. C'est
la troisième fois depuis 1 an qu'elle souffre de pareilles lésions à cet endroit.

Vous pourrez confirmer le diagnostic d'herpès par :


A - La recherche d' herpès simplex virus (HSV) dans les lésions par isolement en culture de cellules in vitro
B - La recherche d'antigènes HSV par cytodiagnostic en immunofluorescence sur des cellules prélevées à la
base des ulcérations
C - Le dosage des anticorps HSV par neutralisation ou fixation du complément dans un sérum prélevé à cette
patiente
D - Le dosage des anticorps par neutralisation ou fixation du complément dans 2 sérums prélevés, le 1er jour
de la consultation et 2 à 3 semaines après
E - Le dosage des IgM spécifiques de l'HSV type II dans un sérum prélevé à cette patiente 5 à 10 jours après
la consultation
Bonne(s) réponse(s) : A B D

La culture in vitro permet le diagnostic par mise en évidence de l'effet cytopathogénique caractéristique (cellules ballonisées).
Elle permet le typage, mais les résultats sont tardifs et le coût élevé. L'immunofluorescence est rapide et sensible à ce stade
de l'infection. Les sérologies sont moins sensibles et moins spécifiques et nécessitent 2 sérums prélevés à 1 jours d'intervalle.
La mise en évidence d'IgM est inutile puisqu'il s'agit d'une récurrence.

Le diagnostic d'herpès génital récurrent étant acquis, il est logique de conseiller à cette femme :
A - De réfléchir à l'éventualité d'une IVG
B - De prendre de l'aciclovir par voie orale jusqu'à l'accouchement qui pourra se faire alors par voie basse
C - D'accoucher systématiquement par césarienne
D - De débuter la recherche hebdomadaire d'une excrétion asymptomatique d'HSV dans les sécrétions
génitales par inoculation à des cultures de cellules
E - Aucune des propositions précédentes
Bonne(s) réponse(s) : E

Si les lésions ont disparu à la 33ème SA, on pratiquera tous les 15 jours une recherche de virus dans les voies génitales
basses. En cas de positivité du dernier prélèvement, la césarienne est impérative avant tout début de travail.

Cette femme ne revient vous voir que quelques jours avant l'accouchement au neuvième mois de grossesse.
Depuis la dernière visite, elle n'a pas eu de récidives manifestes d'herpès et l'examen gynécologique ne montre
pas de lésions. La poche des eaux n'est pas rompue. Si cette femme accouche par voie basse, le risque
d'herpès pour le nouveau-né peut être estimé à :
A - 85 %
B - 50 %
C - 20 %
D-5%
E - 1/1000
Bonne(s) réponse(s) : B

Le risque est de 10 % si l'on pratique une césarienne avant ou dans les 4 heures suivant la
rupture des membranes.

130
Exclusivement sur DOC - DZ : www.doc-dz.com NADJI 85
RESIDANAT EN POCHE TOME II
Cas Clinique en QCM

Cette femme ayant décidé d'accoucher par voie basse, quelle(s) mesure(s) allez-vous préconiser ?
A - Le monitorage du foetus par électrodes de scalp
B - Le traitement systématique du nouveau-né par aciclovir oral pendant 10 jours
C - Le prélèvement des sécrétions oculaires et pharyngées de l'enfant pour recherche d'HSV
D - La séparation de l'enfant et de la mère
E - L'interdiction d'allaiter l'enfant au sein
Bonne(s) réponse(s) : B C

L'électrode céphalique est formellement contre-indiquée.


L'enfant peut rester avec sa mère et être allaité moyennant des précautions aseptiques (hors du
lit maternel, lavage soigneux des mains, blouse à usage unique...).
La recherche d'H.S.V. est systématique à l'accouchement chez la mère et le foetus et le traitement conseillé jusqu'aux
résultats. L'herpès néonatal est mortel dans 60 % des cas et 50 % des survivants ont des séquelles neurologiques ou
oculaires.

Une femme de 40 ans, non ménopausée, consulte pour un nodule du sein gauche. L'interrogatoire retrouve dans les
antécédents personnels : 2 grossesses menées à terme à 32 et 34 ans, avec allaitement prolongé, la prise de contraceptifs
oraux pendant 5 ans, un fibroadénome opéré il y a 10 ans ; des antécédents familiaux : mère décédée d'un cancer du col de
l'utérus.
L'examen note une lésion du quadrant supéro-externe de 3 cm, dure avec dépression cutanée en regard, non fixée et une
adénopathie axillaire homolatérale dure. La mammographie objective une opacité irrégulière, étoilée, mesurant 2 cm,
entourée d'un halo d'oedème et associée à un épaississement cutané. Des micro-calcifications sont notées dans l'opacité.
Une intervention chirurgicale est pratiquée. Voici les résultats :
- adénocarcinome infiltrant grade 3 de Scarff et Bloom, avec lésions multifocales
- 12 ganglions sur 18 sont envahis dont certains avec rupture capsulaire et embols Iymphatiques
- récepteurs hormonaux sur la tumeur : oestrogène 200 fentomoles, progestérone 8 fentomoles (seuil de positivité pour la
méthode : 10).

Quel est le facteur de risque de cancer du sein présenté par cette malade ?
A - Première grossesse à l'âge de 32 ans
B - Allaitement prolongé
C - Prise de contraceptifs oraux pendant 5 ans
D - Fibroadénome
E - Mère décédée d'un cancer
Bonne(s) réponse(s) : D

Les antécédents de mastopathie bénigne surtout de découverte tardives (30 ans) augmentent le risque de cancer du sein.
Les grossesses tardives > 35 ans multiplient par 2 ce risque et certains auteurs ont montré que le risque relatif était plus élevé
chez des primipares de plus de 25 ans que chez celles de moins de 25 ans. Seuls les antécédents familiaux de cancer du
sein augmentent le risque.

La confrontation radioclinique laisse suspecter des critères de malignité, le(s)quel(s) ?


A - Lésion de 3 cm
B - Dépression cutanée en regard
C - Lésion étoilée plus petite en mammographie qu'à la palpation
D - Epaississement cutané
E - Existence de microcalcifications
Bonne(s) réponse(s) : B C D E

La taille de la tumeur intervient dans la stadification et le pronostic mais n'est pas considéré , comme critère de malignité. La
différence de taille clinique et mammographique correspond à l'oedème péritumoral.

Quelle est la seule classification TNM possible dans ce cas :


A - T3b N1a M+
B - T2 N1b Mx
C - T4b N1b Mx
D - T2a N1a M+
E - T2 N2 Mx
Bonne(s) réponse(s) : B

Lésion > 2 cm = T2.


Lésion < ou = à 5 cm =T2.
Sans fixation au pectoral = a.
Adénopathie axillaire homolatérale suspecte non fixée = N1 b.
L'épaississement cutané à la mammographie et la dépression cutanée en regard de la tumeur sont des signes de mauvais
pronostic mais ne signifient pas l'envahissement cutané.

131
Exclusivement sur DOC - DZ : www.doc-dz.com NADJI 85
RESIDANAT EN POCHE TOME II
Cas Clinique en QCM

Parmi les attitudes diagnostiques et thérapeutiques chirurgicales possibles, vous retenez :


A - Cytoponction - évidemment axillaire
B - Tumorectomie + évidement axillaire
C - Mastectomie + évidement axillaire
D - Mastectomie + évidement axillaire + évidement sus-claviculaire
E - Mastectomie simple
Bonne(s) réponse(s) : B C

La taille clinique de la tumeur est de 3 cm et on se trouve à la limite des indications de chirurgie conservatrice. Le curage
axillaire est systématique mais ne doit pas dépasser la loge en l'absence d'envahissement sus-claviculaire patent.

Les résultats anatomopathologiques connus, il serait de mauvais pronostic de constater chez cette patiente :
A - Adénocarcinome canalaire infiltrant
B - Grade 3 Scarff et Bloom
C - 12 N+ sur 18
D - Embols Iymphatiques tumoraux
E - Récepteurs oestrogènes positifs
Bonne(s) réponse(s) : B C D

Les éléments de mauvais pronostic sont en premier lieu le grade élevé et la présence de plus de 3 ganglions envahis. Les
embols Iymphatiques tumoraux sont à un moindre degré un élément de plus pour juger de l'agressivité de cette tumeur. La
présence de récepteurs aux oestrogènes est de bonne augure pour la sensibilité à l'hormonothérapie.

Dans le cadre du bilan d'extension, il est indispensable de pratiquer :


A - Tomographies pulmonaires
B - Scintigraphie osseuse au Technétium
C - Echographie hépatique
D - Echographie pelvienne
E - Scanner cérébral
Bonne(s) réponse(s) : A B C

Les métastases sont principalement osseuses sur le squelette axial, pulmonaires et hépatiques. Le scanner cérébral ne sera
demandé qu'en cas de symptomatologie clinique d'appel.
L'échographie pelvienne n'a aucun intérêt.

Madame X., âgée de 34 ans, vient consulter pour une aménorrhée de deux mois. Elle a trois enfants âgés de 12, 8 et 4 ans.
Elle a subi à l'âge de 20 ans une ovariectomie pour kyste dermoïde de l'ovaire gauche. Elle est depuis cette intervention, bien
réglée, en bonne santé et ne prend aucun traitement particulier. L'examen objective un utérus grand, de la taille d'une
mandarine, dans le cul de sac droit, une masse régulière, indolore, de la taille d'une balle de ping-pong.

Parmi les propositions suivantes, laquelle vous parait indispensable ?


A - Dosage de FSH
B - Coelioscopie
C - Dosage de FSH bêta HCG
D - Dosage d'oestradiol
E - Dosage de progestérone
Bonne(s) réponse(s) : C

Le dosage des HCG plasmatiques permet de diagnostiquer une grossesse qui est la première éventualité dans ce contexte.
La coelioscopie n'est discutable qu'en cas d'utérus vide à l'échographie et si les HCG sont positives, afin de rechercher une
grossesse extra-utérine.

N'ayant aucune notion sur les résultats d'examens de ses précédentes grossesses, le(s)quel(s) allez-vous
demander ?
A - Groupe sanguin rhésus
B - Réactions syphilitiques
C - Sérologie rubéole
D - Sérologie toxoplasmose
E - Agglutinines irrégulières
Bonne(s) réponse(s) : A B C D E

En l'absence d'examens antérieurs il est obligatoire de prescrire un groupe sanguin, rhésus, phénotype complet. La recherche
d'agglutinines irrégulières n'est obligatoire que chez les femmes rhésus négatif mais est conseillée chez toute femme en début
de grossesse afin de rechercher une immunisation aux autres sous-groupes.
Les sérologies de syphilis rubéole toxoplasmose et la recherche de glycosurie et d'albuminurie viendront compléter ce bilan.

132
Exclusivement sur DOC - DZ : www.doc-dz.com NADJI 85
RESIDANAT EN POCHE TOME II
Cas Clinique en QCM

Revue 15 jours plus tard, à la suite de métrorragies modérées, l'examen clinique apparait identique. Que lui
direz-vous ?
A - Il faut attendre et revoir la situation dans 15 jours
B - Il s'agit sûrement d'une grossesse interrompue
C - Il s'agit d'un oeuf clair
D - Il faut faire une échographie pour savoir
E - Vous avez fait une fausse couche
Bonne(s) réponse(s) : D

Seule l'échographie permet dans l'immédiat de confirmer l'évolutivité ou non de la grossesse. Il existe environ 25 % de risque
de fausse couche spontanée (grossesse interrompue - oeuf clair).

Le diagnostic de grossesse arrêtée est confirmé, apparaissent des douleurs pelviennes régulières dans les
heures qui suivent. Que proposez-vous ?
A - Une coelioscopie
B - Une mise en observation pour aspiration éventuelle
C - Le repos alité avec traitement progestatif
D - Une ponction du Douglas
E - Une laparotomie
Bonne(s) réponse(s) : B

L'expulsion spontanée est préférable en toute circonstance, néanmoins il est, en régle, nécessaire à ce terme de pratiquer un
curetage pour vérifier la vacuité utérine.
En aucun cas on ne cherche à maintenir une grossesse anormale ou à empêcher une fausse-couche spontanée du premier
trimestre car le risque d'anomalie chromosomique est majeur.

Une III° geste de 30 ans est hospitalisée à 34 semaines d'aménorrhée pour une hyperthermie à 39 degrés avec une douleur
costovertébrale droite sans brûlures à la miction. La hauteur utérine est à 29 centimètres, le foetus est en présentation
céphalique, les bruits du coeur sont positifs. Le col est normal ; postérieur, ferme. L'enregistrement montre une contraction
utérine toutes les 10', le rythme cardiaque foetal est à 140 avec oscillations normales.

Une fièvre de ce type évoque quel(s) diagnostic(s) ?


A - Appendicite aiguë
B - Colique néphrétique
C - Listériose
D - Pyélonéphrite gravidique
E - Infection amniotique
Bonne(s) réponse(s) : C D E

Le tableau évoque une pyélonéphrite aiguë droite (les signes urinaires sont inconstants) mais ne permet pas d'éliminer une
listériose à rechercher et traiter systématiquement à ce terme en raison de sa gravité.
L'infection amniotique peut être isolée ou associée à l'un des diagnostics précédents.

Quel(s) examen(s) paraclinique(s) doit-on demander pour faire le diagnostic ?


A - Ponction de sang foetal
B - Amnioscopie
C - Examen cytobactériologique urinaire
D - Hémoculture
E - Amniocentèse
Bonne(s) réponse(s) : C D

L'ECBU recherche une infection urinaire.


L'hémoculture est l'examen le plus sensible pour diagnostiquer une listériose.
L'infection amniotique étant secondaire à un foyer génito-urinaire ou à une infection générale, on ne pratique pas de
prélèvement de liquide amniotique en l'absence de rupture des membranes, le risque de complication de ce geste y serait
considérable.

133
Exclusivement sur DOC - DZ : www.doc-dz.com NADJI 85
RESIDANAT EN POCHE TOME II
Cas Clinique en QCM

Les complications gravidiques possibles comportent :


A - Hydrocéphalie
B - Souffrance foetale
C - Accouchement prématuré
D - Décollement placentaire
E - Hypotrophie foetale
Bonne(s) réponse(s) : B C E

A court terme les risques sont l'accouchement prématuré (avant 37 SA) et la souffrance foetale par infection.
A long terme en cas de traitement insuffisant il peut exister une hypotrophie foetale.
Le décollement placentaire n'est pas une complication liée à l'infection.
L'hydrocéphalie est parfois la conséquence d'infection virale ou toxoplasmique du premier trimestre.

Quel(s) est(sont) le(s) antibiotique(s) que vous pouvez utiliser ?


A - Vibramycine
B - Clamoxyl
C - Rufol®
D - Erythromycine
E - Streptomycine
Bonne(s) réponse(s) : B D

Le traitement de la listériose repose sur l'ampicilline (Clamoxyl®).


En cas d'allergie on peut utiliser l'érythromycine, également sans danger pendant la grossesse.

Quelle attitude obstétricale conseillez-vous ?


A - Progestérone
B - Repos
C - Bêta-mimétiques
D - Ocytocine
E - Cerclage
Bonne(s) réponse(s) : A B C

La présence de contractions régulières signifie la menace d'accouchement prématuré. Le premier traitement est le repos au
lit. La tocolyse repose sur l'utilisation des bêta-mimétiques auxquels certains associent systématiquement les progestatifs.

Une femme de 40 ans, mère de 10 enfants, le premier à l'âge de 17 ans, des antécédents personnels de cancer du sein et
familiaux de cancers du côlon et de l'endomètre, se présente chez son gynécologue pour la réalisation de frottis
cervicovaginaux. La surveillance des frottis est régulière depuis une vingtaine d'années à raison d'un frottis tous les 3 à 4 ans
environ, ceux-ci étant toujours normaux (classe 1).

Quel est, éventuellement, le facteur de risque de cancer du col utérin chez cette femme ?
A - 1ère grossesse précoce
B - Grande multiparité
C - Antécédent personnel de cancer du sein
D - Antécédent familiaux d'adénocarcinomes
E - Aucun
Bonne(s) réponse(s) : A B

Première grossesse précoce et grande multiparité sont les facteurs de risques classiques de cancer du col, en fait discutés
sur les dernières études...

Quel(s) type(s) de virus est(sont) impliqué(s) dans l'histoire naturelle du cancer du col utérin ?
A - Virus d'Epstein Barr
B - Virus de la rougeole
C - Herpès virus
D - Papillomavirus
E - Aucun
Bonne(s) réponse(s) : C D

C'est le papilloma virus qui est considéré comme un facteur causal des dysplasies et cancers du col utérin. L'herpès virus est
souvent retrouvé et est considéré par certains comme un "co-carcinogène".

134
Exclusivement sur DOC - DZ : www.doc-dz.com NADJI 85
RESIDANAT EN POCHE TOME II
Cas Clinique en QCM

La surveillance des frottis chez cette patiente vous paraît :


A - De fréquence suffisante
B - De fréquence correcte
C - De fréquence exagérée
D - De fiabilité correcte
E - De fiabilité incorrecte
Bonne(s) réponse(s) :

QUESTION ANNULEE

Le dernier frottis est de classe IV. Quelle est la signification de classe IV ?


A - Frottis inflammatoire
B - Cellules typiquement tumorales
C - Frottis d'interprétation difficile, à refaire
D - Cellules très suspectes
E - Frottis ne comportant pas de cellules d'origine cervicale
Bonne(s) réponse(s) : D

Seul l'examen anatomopathologique des fragments biopsiques, prélevés au mieux lors d'un examen colposcopique, permet
d'affirmer la nature cancéreuse de la lésion.

La colposcopie a permis d'individualiser une plage douteuse, près de la zone de jonction, qui n'a pas été vue en
totalité. Une biopsie réalisée permet de conclure à un épithélioma in situ. En aucun point le membrane basale
ne parait envahie. Quel traitement proposez-vous ?
A - Aucun
B - Conisation
C - Hystérectomie totale
D - Hystérectomie totale avec ovariectomie bilatérale et lymphadénectomie
E - Curie + radiothérapie externe
Bonne(s) réponse(s) : C

Compte tenu de l'âge et de la grande multiparité, il est raisonnable de proposer d'emblée une hystérectomie totale ; la
conisation est le traitement proposé aux femmes désirant à tout prix une grossesse ultérieure.
La lymphadénectomie et ovariectomie est proposée dans les cancers invasifs ainsi que la curiethérapie et la radiothérapie
externe en fonction du degré d'extension.

Une primigeste de 25 ans, sans antécédent pathologique, est admise d'urgence à 36 semaines d'aménorrhée pour une H.T.A.
à 18/12, avec une protéinurie à 1 gramme/litre. Elle mesure 160 centimètres. Son poids initial était de 54 kgs, elle pèse
actuellement 70 kgs. La hauteur utérine est à 26 cm : il s'agit d'un foetus en présentation céphalique, dos à gauche, bruit du
coeur positif.
Elle est du groupe A Rh+.
Les examens de surveillance ne montrent qu'un test toxoplasmique toujours négatif. Le col est normal, postérieur, fermé.

Quel est le diagnostic à poser ?


A - HTA antégravidique
B - Syndrome néphrotique
C - Protéinurie orthostatique
D - Diabète + HTA
E - Dysgravidie
Bonne(s) réponse(s) : E

Il s'agit du tableau classique de la toxémie gravidique encore appelée dysgravidie ou néphropathie gravidique.

La hauteur utérine est à 26 centimètres. Quelle est la hauteur utérine normale à 36 S.A. ?
A - La valeur de l'HTA
B - L'échographie
C - 34 centimètres
D - 26 centimètres
E - 33 centimètres
Bonne(s) réponse(s) :

QUESTION ANNULEE

135
Exclusivement sur DOC - DZ : www.doc-dz.com NADJI 85
RESIDANAT EN POCHE TOME II
Cas Clinique en QCM

On évalue le retentissement foetal grâce à certains examens. Lequel ou lesquels ?


A - L'importance de l'HTA
B - L'échographie
C - L'albuminurie
D - Le RCF
E - La H.U.
Bonne(s) réponse(s) : B D E

La hauteur utérine insuffisante (normale à ce terme = 31 cm) évoque une hypotrophie que l'on confirme par la biométrie
foetale échographique. La souffrance foetale chronique et sa décompensation aiguë sont surveillés par les RCF répétés et
l'examen doppler.
L'importance de l'H.T.A. et de l'albuminurie évaluent le retentissement maternel.

Quel(s) examen(s) paraclinique(s) évalue(nt) le retentissement et le pronostic maternels ?


A- Uricémie
B - F.O.
C - Plaquettes
D - Kaliémie
E - Albuminurie des 24 heures
Bonne(s) réponse(s) : A B C E

Les facteurs de gravité sont pour la mère :


- l'apparition d'une protéinurie > 1g/l
- une uricémie > 60 mg/l = 360 mmol/l
- une thrombopénie < 100 103/mm3 (risque de CIVD).
- une anomalie du F.O. : risque de lésions rétiniennes irréversibles par H.T.A. maligne.

Quel(s) traitement(s) appliquer à la mère ?


A - Repos
B - Antispasmodiques
C - Anti hypertenseurs
D - Diurétiques
E - Bêta mimétiques
Bonne(s) réponse(s) : A C

Le repos est le premier temps du traitement et vise à améliorer la vascularisation foeto-placentaire.


Les antihypertenseurs sont utilisés dès que la minima est > 10 mmHg afin d'éviter des poussées hypertensives.
Diurétiques et béta mimétiques sont contre-indiqués et sans intérêt.
Les antispasmodiques n'ont pas d'indication systématique à ce stade.

Les critères d'extraction foetale incluent :


A - RCIU (Rythme Cardiaque Intra-Utérin)
B - Uricémie normale
C - Anomalies du RCF
D - HTA maternelle difficilement contrôlée
E - Augmentation de poids de 3 kgs en 7 jours
Bonne(s) réponse(s) : A C D

Les critères d'extraction sont :


- la stagnation pondérale appréciée par la surveillance biométrique foetale
- la souffrance foetale aiguë visible sur les RCF
- la difficulté de contrôler l'HTA maternelle en raison du risque maternel lié à l'éclampsie (principale cause de mort maternelle).

136
Exclusivement sur DOC - DZ : www.doc-dz.com NADJI 85
RESIDANAT EN POCHE TOME II
Cas Clinique en QCM
Madame B., 2ème geste, âgée de 39 ans, de groupe A Rh-, antérieurement bien réglée, est immunisée contre la rubéole et
contre la toxoplasmose. Elle est porteuse d'un utérus cloisonné, affirmé à la suite d'une fausse couche à 12 semaines
d'aménorrhée, par une hystérographie. Vous la voyez pour la première fois, avec un retard de règles de trois semaines.
L'examen est compatible avec un début de grossesse d'environ un mois. Elle ne se plaint d'aucun trouble particulier, en
dehors de crises de tachycardie paroxystique.

Parmi les examens suivants, le(s)quel(s) vous apparai(ssen)t indispensable(s) ?


A - Un dosage de bêta HCG
B - Un Dye-test
C - Une recherche d'agglutinines irrégulières
D - Un VDRL - TPHA
E - Une échographie pelvienne
Bonne(s) réponse(s) : C D

Les examens obligatoires en vue de la déclaration de grossesse sont :


- la recherche d'agglutinines irrégulières
- la recherche d'une syphilis (TPHA-VDRL).
L'échographie ne sera demandée qu'en cas de doute sur le terme et les BHCG en cas de suspicion de grossesse extra
utérine lorsque la RIG est négative.
La femme étant immunisée contre rubéole et toxoplasmose il est inutile de recontrôler sa sérologie.

Si les examens précédemment demandés sont normaux, qu'allez-vous proposer ?


A - Une amniocentèse à 17 SA
B - Une amniocentèse à 28 SA
C - Un arrêt de travail vers la 28ème SA
D - Le repos complet, alité
E - Le traitement chirurgical de la malformation
Bonne(s) réponse(s) : A C

Etant donné l'âge de 39 ans il est raisonnable de proposer une amniocentèse systématique à 17 SA pour rechercher une
anomalie chromosomique. La période idéale se situe entre 16 et 18 SA.
Compte tenu de cette malformation utérine il est justifié de la faire bénéficier d'un arrêt de travail à partir de la période de
viabilité foetale (28 SA), le traitement chirurgical ne serait envisagé qu'en cas de FC spontanées répétées.

A quel(s) risque(s) serait exposée cette grossesse ?


A - Fausse couche tardive
B - Anomalie chromosomique
C - Retard de croissance intra-utérin
D - Prématurité
E - Mort périnatale
Bonne(s) réponse(s) : A B D

La malformation utérine est un facteur de risque de FC précoces, tardives et de prématurité. L'âge maternel est un facteur de
risque d'anomalie chromosomique.

Devant l'apparition de métrorragies minimes à 30 SA, associées à quelques contractions utérines, qu'allez-vous
proposer ?
A - L'évaluation du rapport L/S
B - Une vérification échographique du placenta
C - L'injection de gammaglobulines anti-D
D - Un cerclage tardif
E - Un traitement par la progestérone micronisée
Bonne(s) réponse(s) :

QUESTION ANNULEE

Revue à 36 SA, apparaissent des contractions utérines et une TA à 15/10, le foetus est en siège décomplété.
Que proposez vous ?
A - Une radiographie de contenu utérin
B - Une radiopelvimétrie
C - Une tocolyse par bêta mimétiques
D - L'association alphaméthyldopa - bêta mimétiques
E - L'association alphaméthyldopa - césarienne
Bonne(s) réponse(s) : A B D

L'acceptation de l'accouchement par voie basse de ce siège est fonction du contenue utérin (flexion de la tête) et de la
radiopelvimétrie (normalité du bassin), mais compte tenu de l'association d'une toxémie gravidique, d'une prématurité et d'une
présentation du siège, la sagesse à ce terme est de proposer la césarienne et un traitement antihypertenseur.
137
Exclusivement sur DOC - DZ : www.doc-dz.com NADJI 85
RESIDANAT EN POCHE TOME II
Cas Clinique en QCM
Madame X..., primipare de 25 ans, consulte parce qu'elle n'a plus ses règles depuis quelques mois, sans pouvoir préciser
exactement la date de ses dernières règles. Elle pense qu'elle est enceinte puisqu'elle "sent des mouvements intra-utérins"
depuis dix jours.
L'examen général est normal. L'examen obstétrical révèle un utérus gravide, atteignant l'ombilic, les bruits du coeur foetal sont
perçus. Au toucher, le col est long, fermé et postérieur. La prise de poids a été de 10 kgs. L'analyse d'urine ne montre pas
d'albumine mais une glycosurie à trois croix.

Il s'agit donc d'une grossesse dont le point de départ est imprécis. Parmi les examens suivants, quel est
celui(ceux) qui va(vont) permettre de connaître approximativement la date de l'accouchement ?
A - Dosage plasmatique d'oestradiol
B - L'échographie avec mesure du bipariétal, du biabdominal et du fémur
C - L'appréciation échographique des mouvements foetaux
D - L'enregistrement du rythme cardiaque foetal
E - La mesure de la hauteur utérine et du périmètre ombilical
Bonne(s) réponse(s) : B C E

Les mouvements apparaissent à la 9ème SA.

Parmi les examens biologiques suivants, vous devez prescrire dans ce cas :
A - Albuminurie de 24 heures
B - Glycémie à jeun - glycémie post-prandiale
C - Hyperglycémie provoquée par voie intraveineuse
D - Glycosurie des 24 heures
E - Ionogramme sanguin
Bonne(s) réponse(s) :

QUESTION ANNULEE

La glycémie post-prandiale est trop élevée. Un état de diabète gestationnel est donc fort probable. Cet état
expose à un certain nombre de complications obstétricales. Laquelle ou lesquelles ?
A - Augmentation de la fréquence des malformations foetales
B - Excès de liquide amniotique
C - Diminution du volume du liquide amniotique
D - Macrosomie foetale
E - DPPNI (Décollement Prématuré du Placenta Normalement Inséré)
Bonne(s) réponse(s) : A B D E

Sans commentaire.

En cas de diabète maternel, la maturité pulmonaire foetale :


A - Est acquise plus tôt au cours d'une grossesse normale
B - Est liée à la synthèse du surfactant par le poumon foetal
C - Peut être appréciée par l'étude des phospholipides du liquide amniotique
D - Est toujours acquise à 30 semaines d'aménorrhée
E - Peut être appréciée par l'étude du volume du liquide amniotique à l'échographie
Bonne(s) réponse(s) : B C

Sans commentaire.

Le nouveau-né de mère diabétique est particulièrement exposé à certaine(s) manifestation(s) néo-natale(s) :


A - Hyperglycémie due à une diminution de la sécrétion d'insuline par le pancréas foetal
B - Hypercalcémie par hyperparathyroïdie
C - Hyperthyroïdie
D - Hypoglycémie par excès d'apport de glucose par le lait et hyperinsulinisme réactionnel
E - Hypoglycémie par hyperinsulinisme néo-natal secondaire à l'excès d'apport de glucose maternel, pendant
la vie intra-utérine
Bonne(s) réponse(s) : A E

Sans commentaire.

138
Exclusivement sur DOC - DZ : www.doc-dz.com NADJI 85
RESIDANAT EN POCHE TOME II
Cas Clinique en QCM
Il s'agit d'une femme de 25 ans, sous contraception estroprogestative combinée depuis 5 ans, nullipare, dont les dernières
règles sont normales qui présente des douleurs abdominales basses depuis 8 jours avec une température à 38,5 degrés, des
leucorrhées. A l'examen, les cul-de-sacs vaginaux sont très douloureux, la mobilisation utérine est douloureuse.

Quel est le premier diagnostic que vous évoquez ?


A - Endométrite
B - Bartholinite
C - Salpingite
D - Cervicite
E - Ovarite sclérokystique
Bonne(s) réponse(s) : C

Il s'agit d'une infection génitale haute en raison de la fièvre, des douleurs abdominales et des leucorrhées. La douleur des cul-
de-sacs vaginaux et à la mobilisation utérine oriente en premier lieu vers une salpingite aiguë.

Quel(s) examen(s) systématique(s) demandez-vous ?


A - Prélèvements bactériologiques de l'endocol
B - Radiographie de l'abdomen sans préparation (ASP)
C - Hémogramme, vitesse de sédimentation
D - Dosage de bêta HCG plasmatiques
E - Hystérographie
Bonne(s) réponse(s) : A C

L'hémogramme et la vitesse de sédimentation sont surtout utiles comme base de surveillance de la guérison sous
antibiotiques, par la décroissance de la V.S. et des polynucléaires.
Les prélèvements bactériologiques de l'endocol permettent en règle le diagnostic bactériologique.
L'hystérographie est contre-indiquée en période infectieuse.
HCG et ASP ne sont pas systématiques.

Une coelioscopie a été pratiquée qui montre du liquide dans le Douglas avec des adhérences visqueuses
péritubo-ovariennes. Vous évoquez une infection :
A - A anaréobies
B - A mycoplasme
C - A chlamydiae
D - A gonocoque
E - A streptocoque du groupe B
Bonne(s) réponse(s) : C

Par argument de fréquence et d'après l'aspect coelioscopsique la salpingite à chlamydiae est probable.

La technique la plus sûre pour mettre en évidence ce germe est :


A- La mise en culture
B - La sérologie par méthode Elisa
C - L'hybridation moléculaire
D - L'inhibition de l'hémagglutination
E - L'injection au cobaye
Bonne(s) réponse(s) : A

L'immunofluorescence sur culture cellulaire type Mac-Coy est la technique la plus courante (mise en évidence d'inclusions
spécifiques).

A long terme cette patiente présente un risque plus élevé de :


A - Stérilité
B - Grossesse extra-utérine
C - Douleurs chroniques
D - Récidive
E - Aucune des réponses ci-dessus
Bonne(s) réponse(s) : A B C D

Les foyers latents peuvent être à l'origine de rechutes de la maladie.


Les adhérences sont souvent responsables de douleurs pelviennes chroniques.
Enfin les lésions tubaires sont responsables de stérilité et de GEU.

139
Exclusivement sur DOC - DZ : www.doc-dz.com NADJI 85
RESIDANAT EN POCHE TOME II
Cas Clinique en QCM

A propos de la prévention de cette maladie, toutes les propositions suivantes sont justes sauf une. Laquelle ?
A - Information auprès des jeunes
B - Dépistage des infections génitales basses
C - Traitement par une pénicilline une semaine tous les 6 mois chez les femmes à risque
D - Traitement des partenaires
E - Dépistage bactériologique annuel chez les femmes à risque
Bonne(s) réponse(s) : C

Les pénicillines sont inefficaces sur la majorité des MST (chlamydiae, gonocoque).

Chez une patiente de 64 ans. nullipare et obèse, ménopausée à 50 ans, dont les premières règles sont apparues à 12 ans, on
note de nombreux épisodes de spanioménorrhée dans ses antécédents, avec une notion de métrorragies, il y a 6 mois.
On lui a prescrit des oestrogènes depuis 1 mois qui ont aggravé les hémorragies.

Quels sont les 2 premiers examens à demander chez cette patiente ?


A - Frottis du col et endomètre
B - Coelioscopie
C - Hystérographie
D - Lymphographie
E - Curetage
Bonne(s) réponse(s) :

QUESTION ANNULEE

La conduite à tenir est :


A - La revoir dans trois mois après arrêt des oestrogènes
B - Ajouter des progestatifs pendant 10 jours
C - Pratiquer une hystérectomie
D - Doser les androgènes
E - Aucune de ces propositions
Bonne(s) réponse(s) :

QUESTION ANNULEE

Le diagnostic est celui d'adénocarcinome de l'endomètre avec atteinte de l'endocol, l'hystérométrie est de 9
cms. Il s'agit d'un stade :
A - Ia
B - Ib
C - II
D - III
E - IV
Bonne(s) réponse(s) :

QUESTION ANNULEE

Chez cette femme vous retenez comme facteur(s) de risque de l'adénocarcinome endométrial :
A - Premières règles à 12 ans
B - Obésité
C - Antécédents de spanioménorrhée
D - Traitement oestrogénique prescrit
E - Ménopause à 50 ans
Bonne(s) réponse(s) :

QUESTION ANNULEE

140
Exclusivement sur DOC - DZ : www.doc-dz.com NADJI 85
RESIDANAT EN POCHE TOME II
Cas Clinique en QCM
Une femme de 72 ans, obèse, consulte pour des métrorragies. Dans ses antécédents on relève :
- premières règles à l'âge de 14 ans
- trois enfants
- long passé de dysménorrhées avec ménométrorragies préménopausiques
- ménopausée à l'âge de 52 ans
- diabète gras léger, connu depuis huit ans et traité par sulfamide hypoglycémiant sans régime suivi
L'examen gynécologique montre que le sang vient de la cavité utérine. Le corps utérin est légèrement augmenté de volume,
avec une hystérométrie à 8 cm. Le col apparaît sain.

Parmi les propositions suivantes le ou les diagnostics à discuter est ou sont :


A - Hémorragies fonctionnelles
B - Prise d'oestrogènes
C - Cancer de l'exocol
D - Chorioépithéliome
E - Complications du diabète
Bonne(s) réponse(s) :

QUESTION ANNULEE

Quel(s) élément(s) du tableau clinique peu(ven)t être associé(s) au cancer de l'endomètre ?


A - Premières règles à 14 ans
B - Trois enfants
C - Obésité
D - Ménopause à 52 ans
E - Diabète
Bonne(s) réponse(s) :

QUESTION ANNULEE

Parmi les propositions suivantes qui caractérisent un cancer du corps utérin il est exact que :
A - L'incidence de ce cancer augmente
B - Les frottis en permettent le dépistage
C - Le type habituel est un épithélioma épidermoïde
D - L'hyperplasie adénomateuse est un état prédisposant
E - Des récepteurs à la progestérone sont retrouvés dans les cellules tumorales
Bonne(s) réponse(s) :

QUESTION ANNULEE

Quel(s) examen(s) complémentaire(s) permet(tent) d'affirmer le diagnostic de cancer de l'endomètre ?


A - Biopsie au novack
B - Echographie pelvienne
C - Frottis vaginal
D - Curetage utérin
E - Scanner pelvien
Bonne(s) réponse(s) :

QUESTION ANNULEE

Parmi les examens suivants le(s)quel(s) est(sont) indispensable(s) au bilan préthérapeutique ?


A - Tomodensitométrie abdominale
B - Echographie hépatique
C - Cystoscopie
D - Scintigraphie osseuse
E - Aucun de ces examens
Bonne(s) réponse(s) :

QUESTION ANNULEE

141
Exclusivement sur DOC - DZ : www.doc-dz.com NADJI 85
RESIDANAT EN POCHE TOME II
Cas Clinique en QCM

Lequel ou lesquels des organes suivants comporte(nt) un risque notable d'extension des cancers de
l'endomètre ?
A - Les trompes
B - Le vagin
C - Les ganglions iliaques
D - Le foie
E - Le péritoine
Bonne(s) réponse(s) :

QUESTION ANNULEE

Les critères reconnus comme ayant une influence sur le pronostic de cette affection comportent :
A - Le degré de différenciation
B - L'âge de la patiente
C - L'invasion du myomètre
D - L'importance des métrorragies
E - L'extension à l'isthme
Bonne(s) réponse(s) :

QUESTION ANNULEE

Une femme enceinte de 8 mois, mal surveillée pendant la grossesse, est hospitalisée en urgence pour un hématome
rétroplacentaire. Parmi les examens biologiques que vous avez demandés, le bilan d'hémostase est le suivant :
- temps de Quick : 45 %
- temps de céphaline activé : 42 sec (35 sec)
- V : 40 %
- VII +X : 85 %
- II : 45 %
- fibrinogène : 0,50 g/l
- plaquettes : 110 000/mm3.

Compte tenu du contexte clinique, quel est le diagnostic le plus probable de ce trouble de la coagulation ?
A - Déficit en vitamine K physiologique de la grossesse
B - Hépatite fulminante
C - Fibrinolyse aiguë
D - Thrombocytopénie par hémodilution
E - Coagulopathie de consommation
Bonne(s) réponse(s) : C

La chute du fibrinogène est le signe d'une fibrinolyse aiguë compliquant une CIVD secondaire à l'hématome rétroplacentaire.

Parmi les examens complémentaires suivants, le ou lesquels permettrai(en)t le plus sûrement de confirmer ce
diagnostic ?
A - Dosage du facteur Willebrand (VIII R ag)
B - Dosage du facteur antihémophilique B
C - Dosage des produits de dégradation du fibrinogène
D - Test de paracoagulation (test à l'éthanol)
E - Mesure du temps de saignement
Bonne(s) réponse(s) : C

Le dosage des PDF permet de confirmer le diagnostic et d'évaluer la gravité.

L'utérus de cette patiente doit être évacué, et parmi les mesures thérapeutiques suivantes destinées à éviter
l'hémorragie de la délivrance laquelle choisissez-vous ?
A - Cinq flacons de 20 ml de PPSB
B - Perfusion intraveineuse d'un antifibrinolytique
C - Injection immédiate de 500 U/kg d'héparine standard
D - Quatre unités de plasma frais congelé plus 4 grammes de fibrinogène
E - Un flacon de 250 000 unités de streptokinase
Bonne(s) réponse(s) : D

Il faut apporter en urgence les facteurs de coagulation présents dans le plasma frais congelé supplémenté en fibrinogène.

142
Exclusivement sur DOC - DZ : www.doc-dz.com NADJI 85
RESIDANAT EN POCHE TOME II
Cas Clinique en QCM

Moyennant le traitement adéquat, l'évacuation utérine s'est bien passée. Vingt quatre heures, après le bilan
d'hémostase est normal et il a été prescrit à cette patiente 5 000 unités d'héparine par voie sous-cutanée, deux
fois par jour. Malgré cette prophylaxie elle présente une phlébite fémoropoplitée cinq jours après. Parmi les
solutions thérapeutiques suivantes, laquelle choisissez-vous ?
A - Ajouter de l'aspirine (500 mg/jour) à l'héparine sous-cutanée
B - Augmenter la posologie de l'héparine sous cutanée : 5 000 unités x 3 par jour
C - Arrêter l'héparine sous-cutanée et passer aux antivitamines K
D - Arrêter l'héparine sous-cutanée et prescrire de l'héparine intraveineuse à la dose de 200 unités/kg/24 h
E - Arrêter l'héparine sous-cutanée et prescrire de l'héparine intraveineuse à la dose de 600 unités/kg/24 h.
Bonne(s) réponse(s) : E

Le traitement de la phlébite est l'héparine IV à doses efficaces (à la pompe).

Quel(s) est(sont) le(s) examen(s) le(s) plus utile(s) pour surveiller l'efficacité biologique du traitement prescrit ?
A - Mesure du temps de saignement
B - Dosage de l'antithrombine III
C - Temps de Quick
D - Temps de céphaline activée
E - Détermination de l'héparinémie circulante
Bonne(s) réponse(s) : C D

Sans commentaire.

Madame X., 41 ans, 2 enfants, sans antécédent médicochirurgical particulier, consulte pour un allongement de la durée de
ses règles, qui, depuis un an, sont plus abondantes et accompagnées de caillots. Elle ne présente aucune perte entre les
règles. L'examen général est normal.
L'examen du col au spéculum ne révèle aucune anomalie. Le frottis cervicovaginal est classe II.
Au toucher vaginal, l'utérus est nettement augmenté de volume, du volume d'un pamplemousse, irrégulier, avec masse
arrondie, développée aux dépens de la face antérieure, faisant corps avec l'utérus. On ne perçoit pas de masse latéro-utérine.
Le toucher vaginal est indolore. Le diagnostic de fibrome utérin est porté.

Les troubles des règles présentés par la patiente sont des :


A - Ménorragies
B - Métrorragies
C - Hyperménorrhées
D - Spanioménorrhées
E - Hypoménorrhées
Bonne(s) réponse(s) : A C

Les ménorragies sont l'augmentation de la durée des règles avec modification qualitative : sang plus rouge et caillots.
Les hyperménorrhées sont l'augmentation du volume des règles.
Les métrorragies sont des saignements endo-utérins survenant en dehors des règles.
Les spanioménorrhées sont l'espacement progressif des règles et leur diminution.

Un certain nombre de signes cliniques simples sont susceptibles de distinguer un kyste de l'ovaire d'un fibrome
utérin interstitiel. En cas de fibrome interstitiel :
A - La masse perceptible est mobilisable
B - Il existe un sillon net de séparation entre l'utérus et la masse perçue
C - Les mouvements donnés au col de l'utérus ne sont jamais transmis à la masse palpée
D - La masse perçue est soit antérieure soit postérieure mais jamais latéro-utérine
E - Le volume de la masse perçue est toujours inférieur à 10 cms
Bonne(s) réponse(s) :

QUESTION ANNULEE

Une échographie pelvienne a été demandée. Habituellement, les fibromes utérins non compliqués se traduisent
à l'échographie par :
A - Une image arrondie juxta-utérine, anéchogène
B - Une image arrondie, peu échogène, contenant des cloisons
C - Une image arrondie, peu échogène, avec renforcement des échos postérieurs
D - Une image arrondie, échogène avec atténuation des échos postérieurs
E - Une image arrondie très échogène avec absorption des ultra-sons et "cône d'ombre" postérieur
Bonne(s) réponse(s) : D

Les fibromes se traduisent par une image arrondie de même nature que le myomètre donc échogène.
Il y a atténuation des faisceaux par absorption tissulaire donc atténuation des échos postérieurs.
L'absorption totale serait en rapport avec une structure gazeuse ou osseuse.
Le renforcement postérieur en rapport avec un kyste liquidien.
143
Exclusivement sur DOC - DZ : www.doc-dz.com NADJI 85
RESIDANAT EN POCHE TOME II
Cas Clinique en QCM

Un certain nombre d'examens (complémentaires) permettent de faire le diagnostic de localisation exacte du


fibrome, en particulier, il est possible de faire le diagnostic de fibrome sous-muqueux à l'aide de :
A - Echographie
B - Hystéroscopie
C - Coelioscopie
D - Colposcopie
E - Hystérosalpingographie
Bonne(s) réponse(s) : B E

L'échographie est peu sensible pour la localisation de petits fibromes sous-muqueux mieux vus en hystéroscopie ou
hystérographie et invisibles en coelioscopie ou colposcopie.

Une hystérographie est envisagée. Cet examen reconnait un certain nombre de contre-indications et il est
nécessaire d'observer un certain nombre de précautions. Chez quelle(s) patiente(s), cet examen n'est-il pas
possible ?
A - Patientes dans les 10 premiers jours du cycle
B - Patientes présentant une douleur pelvienne et une VS à 60 à la 1ère heure
C - Patientes en cours de ménométrorragies
D - Patientes en deuxième partie du cycle (J14 à J28)
E - Augmentation récente du volume d'un fibrome utérin
Bonne(s) réponse(s) : B C D

Les contre-indications sont :


- l'infection (risque de dissémination)
- les saignements (difficultés d'interprétation)
- le risque de grossesse (radiation ovulaire)

L'hystérosalpingographie a pu être réalisée et plaide en faveur d'un myome utérin sous-muqueux. Parmi les
signes hystérographiques suivants, vous retenez en faveur de ce type de fibrome :
A - Cavité utérine de petite taille et régulière
B - Lacune intracavitaire, irrégulière
C - Cavité utérine agrandie, ballonnisée
D - Lacune intracavitaire à bords réguliers
E - Latérodéviation globale de la cavité utérine sans déformation
Bonne(s) réponse(s) : C D

La régularité de la lacune et la modification des contours intra-utérins permettent de diagnostiquer un fibrome sous-muqueux.

L'hystérosalpingographie confirme l'existence d'un fibrome intracavitaire. Quelle(s) option(s) thérapeutique(s)


proposez-vous à votre patiente qui ne souhaite plus d'enfants ?
A - Hystérectomie totale avec conservation ovarienne
B - Myomectomie
C - Surveillance simple
D - Traitement martial visant à corriger l'anémie associée
E - Traitement oestroprogestatif
Bonne(s) réponse(s) : A C D

La myomectomie est sans intérêt compte tenu de l'âge, de l'absence de désir de grossesse et du risque de récidive.
Le traitement oestroprogestatif est une contre-indication.

144
Exclusivement sur DOC - DZ : www.doc-dz.com NADJI 85
RESIDANAT EN POCHE TOME II
Cas Clinique en QCM
Une patiente de 26 ans, enceinte de 20 semaines, vient en consultation pour sa visite du 5 ème mois. Le sérodiagnostic de
toxoplasmose pratiqué 15 jours auparavant objective un taux d'lgG à 450 U.I., présence d'lgM. Le précédent sérodiagnostic
pratiqué en début de grossesse à 6 semaines d'aménorrhée était négatif. Il s'agit d'une séroconversion. Un traitement à la
Spiramycine (3 g/j) est instauré.

Les risques de toxoplasmose congénitale pour le foetus sont de l'ordre de :


A - 90%
B - 70%
C - 50%
D - 20%
E - 0,5%
Bonne(s) réponse(s) : D

Le risque d'infection foetale au 1er trimestre d'une toxoplasmose non traitée est, en pourcentage :
- Non infectés 75.
- Mort in utéro 11.
- Forme grave 8.
- Forme bénigne 1,5
- Infection inapparente 4,5
(d'après Desmonts G.)

La toxoplasmose congénitale peut entraîner à cet âge de grossesse :


A - Hydrocéphalie
B - Anomalies des membres
C - Choriorétinite
D - Ascite
E - Atrésie duodénale
Bonne(s) réponse(s) : A C

Les formes à localisation oculo-céphalique (10 %) comportent hydrocéphalie ou microcéphalie avec choriorétinite. Les formes
évolutives polyviscérales sont rares, 1 %, mais en aucun cas ne sont responsables de malformations des membres ou du tube
digestif.

Un prélèvement de sang foetal à été réalisé. Les paramètres sanguins foetaux sont : IgG, 15 UI, IgM, LDH et
gamma-GT normaux, inoculation à la souris négative. La conduite à tenir est :
A - Arrêt de tout traitement
B - Poursuite du traitement par Spiramycine
C - Traitement par Spiramycine + Sulfadiazine-Adiazine
D - Attente des résultats définitifs pour juger
E - Discussion d'une interruption de grossesse
Bonne(s) réponse(s) : B

La présence d'IgM spécifiques et l'inoculation positive à la souris sont spécifiques de l'infection foetale (les IgG sont
transmises passivement) mais leur absence n'élimine pas l'infection foetale qui peut se faire après le prélèvement. En
conséquence, le traitement doit être poursuivi. La sulfadiazine est réservée aux cas d'infection foetale prouvée.

Si la grossesse allait à terme, à la naissance, que proposez-vous ?


A - Dosage des anticorps antitoxoplasmiques au cordon
B - Inoculation à la souris du sang foetal
C - Poursuite du traitement par Rovamycine
D - Radio de crâne
E - Mise en culture du placenta
Bonne(s) réponse(s) : A B C D E

Sont réalisés systématiquement à la naissance : NFS, plaquettes, radio de crâne, fond d'oeil, échographie transfontanellaire
prélèvement sanguin avec dosage des IgG, IgM spécifiques en parallèle avec le dosage maternel et inoculation à la souris de
sang foetal et de LCR. Le sang du cordon peut être souillé par des IgM maternelles et doit être contrôlé. Mise en culture du
placenta (frais stérile) par inoculation à la souris systématique. Si le nouveau né est cliniquement sain on poursuivra le
traitement par Rovamycine en attendant les résultats.

145
Exclusivement sur DOC - DZ : www.doc-dz.com NADJI 85
RESIDANAT EN POCHE TOME II
Cas Clinique en QCM
Une patiente de 25 ans, sans antécédent notable, a cessé sa contraception orale le 1er janvier 1989, elle a été réglée du 4 au
8 Janvier 1989. Elle consulte le 7 Mars 1989 pour un syndrome douloureux pelvien vague et de petites pertes rouges.
L'examen général est normal, la tension artérielle est à 12/8, la fréquence cardiaque à 70, la température à 36°9. L'examen
gynécologique révèle un utérus un peu augmenté de volume, plus petit que ne le voudrait la durée de l'aménorrhée. Le col est
fermé, l'annexe droite est sensible, la mobilisation utérine est indolore.

Quel(s) diagnostic(s) peut-on évoquer ?


A - Grossesse môlaire
B - Grossesse intra-utérine plus jeune que prévu
C - Grossesse extra-utérine
D - Fibrome utérin associé à une grossesse
E - Grossesse intra-utérine arrêtée
Bonne(s) réponse(s) : A B C E

La reprise spontanée des cycles à l'arrêt d'une contraception est très aléatoire et ne permet pas le calcul du terme. Dans ce
cas la taille de l'utérus est de peu d'intérêt et ne permet pas d'éliminer A ou B.
C et E sont les 1ers diagnostics à évoquer. Il n'existe aucun argument évoquant D.

Le dosage plasmatique d'HCG bêta est positif à 1000 UI/ml, l'échographie révèle un utérus légèrement
augmenté de volume, contenant une petite image anéchogène de 7 mm de diamètre. A droite l'échographie met
en évidence une image kystique de 3 cm de diamètre, il peut s'agir de :
A - Grossesse intra-utérine arrêtée
B - Grossesse intra-utérine évolutive de 5 à 6 semaines d'aménorrhée environ
C - Grossesse intra-utérine évolutive de 7 à 8 semaines d'aménorrhée
D - Grossesse extra-utérine droite
E - Grossesse môlaire
Bonne(s) réponse(s) : A B D

En l'absence d'écho embryonnaire visible il peut s'agir soit d'une grossesse jeune évolutive (activité cardiaque à 7 SA), d'une
grossesse arrêtée, d'une image de caduque avec grossesse extra-utérine. L'image kystique peut correspondre à un kyste du
corps jaune ou à une grossesse extrautérine. Il n'existe aucun argument clinique, biochimique (HCG basses) ni
échographique ("tempête de neige") en faveur d'une grossesse môlaire.

10 jours plus tard les pertes ont pratiquement cessé. Le nouveau dosage d'HCG bêta est à 200 UI/ml.
L'échographie montre une image intra-utérine persistante, l'image latéro utérine droite a par contre disparu. Il
s'agit donc de :
A - Grossesse intra-utérine évolutive de 9 semaines d'aménorrhée
B - Grossesse extra-utérine
C - Grossesse môlaire
D - Grossesse intra-utérine arrêtée
E - Grossesse intra-utérine évolutive de 7 à 8 semaines d'aménorrhée
Bonne(s) réponse(s) : D

La diminution des Bêtas HCG montre qu'il s'agit d'une grossesse arrêtée (élimine A-C-E). L'absence de douleur et la
disparition de l'image échographique droite permettent raisonnablement d'éliminer B.

Quelle(s) attitude(s) thérapeutique(s) pouvez-vous proposer ?


A - Expectative jusqu'à l'expulsion spontanée
B - Aspiration du contenu utérin
C - Traitement progestatif par voie intramusculaire
D - Coelioscopie
E - Laparotomie d'emblée
Bonne(s) réponse(s) : A B

En fonction des souhaits de la patiente, on peut proposer l'expectative avec traitement utérotonique ou l'aspiration sous
anesthésie. Le traitement progestatif est censé maintenir une grossesse en place et n'est donc pas indiqué.

146
Exclusivement sur DOC - DZ : www.doc-dz.com NADJI 85
RESIDANAT EN POCHE TOME II
Cas Clinique en QCM
Une patiente se présente, au terme connu de trente deux semaines d'aménorrhée de sa première grossesse, pour une
menace d'accouchement prématuré se traduisant par l'apparition de contractions utérines douloureuses, régulières,
accompagnées d'une modification du col utérin qui devient court et largement perméable au doigt. Aucun écoulement de
liquide n'est constaté.
Le développement foetal est cliniquement normal. La tension artérielle est à 12/8. On ne note aucun antécédent pathologique
particulier.

Parmi les examens suivants, le(s)quel(s) est (sont) indispensable(s) avant l'institution d'un traitement tocolytique
par béta2 sympathomimétiques ?
A- Ionogramme sanguin
B - Auscultation cardiaque de la mère
C- Monitoring foetal
D - Numération formule sanguine
E - Recherche d'agglutinines irrégulières
Bonne(s) réponse(s) : A B C

La mise en route d'un traitement Bêta2 mimétique doit se faire après élimination de ses contre indi-
cations :

- Souffrance foetale (C).


- Troubles du rythme cardiaque maternel, cardiopathie (B).
- Hyperkaliémie (A).

On décide d'installer un traitement bêta-mimétique (Ritodrine®). Celui-ci serait contre-indiqué par :


A - Une infection urinaire
B - Une anémie
C - L'existence de troubles du rythme cardiaque maternel
D - Un hydramnios
E - Une fièvre, quelle qu'en soit l'étiologie
Bonne(s) réponse(s) : C D

La fièvre maternelle, à condition qu'elle soit traitée et en l'absence de souffrance foetale, n'est pas une contre-indication aux
Bêta+. L'anémie et l'infection urinaire doivent être traitées parallèlement. L'hydramnios fait préférer un traitement par
antiprostaglandines.

Parmi les examens suivants, le(s)quel(s) fait (font) partie du bilan étiologique à réaliser devant cette menace
d'accouchement prématuré (M.A.P.) ?
A - L'échographie obstétricale
B - Prélèvement cervico-vaginaux
C - Le dosage de l'uricémie
D - L'amniocentèse
E - L'amnioscopie
Bonne(s) réponse(s) : A B

L'échographie recherche une cause (malformation, hydramnios...) et permet d'apprécier la maturité foetale. Les prélèvements
urinaires et cervico-vaginaux recherchent une cause infectieuse. L'hyperuricémie n'est pas une cause directe de MAP.

Les différents examens complémentaires pratiqués permettent de mettre en évidence une infection urinaire à
colibacilles. Celle-ci doit être considérée comme :
A - Un élément pathologique intercurrent, sans lien avec la M.A.P.
B - Un facteur déclenchant possible de cette M.A.P.
C - Un élément conduisant à discuter l'interruption du traitement bêtamimétique
D - Un élément conduisant à déclencher l'accouchement sans tarder
E - Un facteur de risque d'infection néo-natale
Bonne(s) réponse(s) : B E

L'infection urinaire doit être recherchée et traitée systématiquement en raison du risque de déclenchement d'une MAP et de
contamination foetale. Traitée efficacement, elle permet la poursuite normale de la grossesse.

147
Exclusivement sur DOC - DZ : www.doc-dz.com NADJI 85
RESIDANAT EN POCHE TOME II
Cas Clinique en QCM

Au bout de 24 h, on adjoint, au traitement bêta mimétique, de la progestérone naturelle par voie orale. Cela a
pour but(s) :
A - D'accélérer la croissance foetale en améliorant la perfusion placentaire
B - D'accélérer la maturation pulmonaire foetale
C - De diminuer la contractilité de la fibre musculaire lisse utérine
D - D'améliorer la diurèse maternelle
E - De mûrir le col utérin
Bonne(s) réponse(s) : C

La progestérone naturelle est myorelaxante sur l'utérus et est souvent utilisée en association avec le repos et les
thérapeutiques Bêta+ dans la MAP.

Madame M..., âgée de 30 ans, consulte pour des leucorrhées à répétition. Elle a une fille de quatre ans et depuis la naissance
prend une pilule estroprogestative séquentielle. Elle est ouvrière, mariée avec un chauffeur routier de 35 ans.
HISTOIRE DE LA MALADIE :
Les symptômes ont commencé il y a deux ans au moins et depuis ce temps elle a eu une dizaine d'épisodes leucorrhéiques ;
ils n'ont pas tous eu les mêmes caractères et globalement, on a retrouvé deux types d'épisodes :
(1) - Leucorrhées blanches, épaisses, comme du yaourt, très prurigineuses
(2) - Leucorrhées glaireuses, abondantes, mouillant le linge, plus importantes dans les huit jours suivants les règles.
LE JOUR DE VOTRE EXAMEN :
La patiente décrit depuis un mois quelques métrorragies ; à l'examen les leucorrhées sont abondantes, grisâtres, finement
bulleuses, un peu odorantes avec test à la potasse positif ; le col est rouge ; le frottis est classe III, très inflammatoire et la
biopsie montrera un condylome associé à une dysplasie modérée.

Quel est l'agent pathogène probablement responsable de la leucorrhée (1) décrite dans l'histoire de la maladie ?
A - Trichomonas
B - Mycoplasme
C - Gardnerella
D - Candida albicans
E - Aucun
Bonne(s) réponse(s) : D

Le candida albicans est responsable d'une vulvovaginite très prurigineuse avec leucorrhée épaisse. Les trichomonas donnent
une leucorrhée verdâtre spumeuse et les gardnerella une leucorrhée nauséabonde grisâtre avec test à la potasse+.

Quel est l'agent pathogène probablement responsable de la leucorrhée (2) décrite dans l'histoire de la maladie ?
A - Trichomonas
B - Mycoplasme
C - Gardnerella
D - Candida albicans
E - Aucun
Bonne(s) réponse(s) : E

Les pillules séquentielles (oestrogènes 15ers jours, suivi pendant 10 jours d'une association oestroprogestative) sont
responsables de la sécrétion d'une glaire cervicale abondante en 1ère partie (leucorrhées n°2). Les oestrogènes favorisent les
mycoses à répétition (leucorrhée n°1).

Quel est (sont) le(s) risque(s) encouru(s) ultérieurement par cette patiente ?
A - Stérilité
B - Cancer du col
C - Avortements à répétition
D - Algies pelviennes
E - Dysménorrhée
Bonne(s) réponse(s) : A B C D E

IGH = infection génitale haute. Il existe des facteurs de risque d'IGH.


- Mari : chauffeur routier.
- Infection génitale basse.
- Condylome.
Les salpingites sont elles-mêmes responsables de stérilité, dysménorrhée, algies pelviennes, avortements à répétition
(endométrite-synéchies). Le cancer du col est plus fréquent dans ce contexte (HPV).

148
Exclusivement sur DOC - DZ : www.doc-dz.com NADJI 85
RESIDANAT EN POCHE TOME II
Cas Clinique en QCM

Quelle démarche diagnostique vous parait la meilleure avant de traiter ?


A - Examen cytobactériologique (E.C.B.) urinaire et vaginal NFS dosage HCG
B - N.F.S. V.S. hystérographie
C - Examen cytobactériologique urinaire et sérologie H.I.V.
D - N.F.S., V.S., sérologie chlamydia, examen cytobactériologique vaginal
E - Aucun examen complémentaire n'est nécessaire
Bonne(s) réponse(s) : D

- Il faut avant tout rechercher une association infectieuse et une infection génitale haute.
- L'hystérographie est contre indiquée en climat infectieux.
- L'ECBU est de peu d'intérêt en l'absence de symptomatologie.
- La sérologie HIV est judicieuse sur ce terrain à risque.

Quelle démarche thérapeutique vous paraît la meilleure vis à vis du condylome ?


A - Une abstention avec contrôle du frottis dans 1 an
B - Une conisation
C - Une vaporisation laser
D - Une hystérectomie simple inter-annexielle
E - Un traitement par l'interféron alpha
Bonne(s) réponse(s) : A

La vaporisation laser est indiquée en présence de condylomes multiples. Un seul condylome cervical peut justifier l'abstention
et surveillance régulière des frottis cervicaux. Conisation et hystérectomie ne se discutent que pour une lésion dysplasique
sévère CIN 3.

Madame U...C..., 25 ans. Premières règles à 16 ans, irrégulières survenant entre 30 et 90 jours. Mariage à 20 ans.
Contraception par estroprogestatifs entraînant la régularisation des cycles. Arrêt à 23 ans en vue d'une grossesse. Exploration
à partir de décembre 1986.
Taille 1,62m, poids 55 kg, T.A. 11/7, pilosité normale, pas de galactorrhée. A la suite de ces explorations, administration de
CLOMID qui permet une grossesse lors du premier cycle de traitement.
Dernières règles : 25 mars 1987.
Premier examen de grossesse le 16 juin 1987.
L'immunité vis à vis de la rubéole et de la toxoplasmose n'a jamais été recherchée. Les premiers résultats sont les suivants :
rubéole + 50 UI toxoplasmose : Immunofluorescence + 50 UI - Remington négatif - groupe sanguin O Rh +.
L'examen clinique note un corps utérin ramolli, antéversé, plus gros que l'âge théorique de la grossesse. Le col est fermé.
Activité cardiaque foetale perçue à l'auscultation Doppler.

Quelle(s) pouvait(ent) être la (les) cause(s) des troubles des règles de cette patiente avant la grossesse ?
A - Syndrome de Stein-Leventhal
B - Cause psychogène
C - Dystrophie ovarienne
D - Hyperprolactinémie
E - Bloc enzymatique surrénalien
Bonne(s) réponse(s) : A B C D

Il existe des hyperprolactinémies modérées sans galactorrhée responsables d'hypofertilité et de troubles des règles.

Quelle est l'interprétation de la sérologie de la toxoplasmose :


A - Il s'agit d'une séro-conversion
B - Un 2ème dosage est nécessaire 3 semaines plus tard
C - Il s'agit d'une immunité ancienne certaine D - Elle n'est pas immunisée
E - Une surveillance mensuelle de la sérologie est souhaitable
Bonne(s) réponse(s) : B

Il s'agit probablement d'une immunité ancienne mais en l'absence de dosage antérieur une nouvelle sérologie après 3
semaines est nécessaire pour ne pas ignorer une infection récente en cours avant la montée des IGM.

149
Exclusivement sur DOC - DZ : www.doc-dz.com NADJI 85
RESIDANAT EN POCHE TOME II
Cas Clinique en QCM

Quelle est la cause probable de l'augmentation de volume de l'utérus supérieure à l'âge théorique de grossesse
?
A - Une erreur de terme
B - Un môle hydatiforme
C - Une grossesse gémellaire
D - Un hydramnios
E - Une malformation foetale
Bonne(s) réponse(s) : A

L'erreur de terme est la cause la plus fréquente d'absence de corrélation clinique avec la hauteur utérine. En 2ème lieu on
évoquera la grossesse gémellaire plus fréquente sous Clomid. Il s'agit plus exceptionnellement d'une môle hydatiforme
(l'activité cardiaque serait négative) ou d'un hydramnios précoce, toujours associé à une malformation.

La grossesse évolue normalement et se termine le 2 décembre 1987 par une césarienne. Suites normales.
Allaitement artificiel. Consulte le 15 mars 1988 pour absence de retour de couches. Examen clinique et
échotomographique normaux. A quelle date aurait dû théoriquement se produire le retour de couches ?
A - 2 février 1988
B - 2 mars 1988
C - 20 janvier 1988
D - 2 janvier 1988
E - 20 février 1988
Bonne(s) réponse(s) : C

En l'absence d'allaitement, le retour de couche se produit théoriquement au 40ème jour après l'accouchement. Il faut
s'inquiéter de l'absence de retour de couche après 8 semaines de post-partum.

Qu'allez-vous conseiller ?
A - Faire le relevé de la température
B - Prendre du Clomid®
C - Un curetage
D - Un dosage de prolactine
E - La prise de micro-pilule (progestative) après retour de couches
Bonne(s) réponse(s) : A D

- Il convient d'abord d'éliminer une nouvelle grossesse (courbe de température).


- L'aménorrhée du post partum est fréquente lors d'ATCD de cycles irréguliers et de prise de Clomid.
- Le dosage de prolactine permet d'éliminer un adénome à PRL éventuellement responsable.
- Le curetage est formellement contre-indiqué, responsable de synéchie. La prise de micropilule n'est pas une bonne
indication dans les dystrophies ovariennes.
- Le Clomid ne se discute que s'il y a désir de grossesse.

Une femme de 26 ans sans antécédents est enceinte de 7 mois et demi. On trouve une protéinurie à 0,30 g/24 h., une
glycosurie à 3 +++ à jeun, une créatininémie à 50 micromol/l, une protidémie à 58 g/l, une prise de poids de 10 kg avec des
oedèmes vespéraux des chevilles, une pression artérielle à 10/6,5 cmHg, une bactériurie à 105 germes par ml confirmée à un
deuxième examen sans leucocyturie, sans cystite, sans douleur dans les lombes, une glycémie à 5 mmol/l.

Du fait de cette glycosurie :


A - On doit suspecter un diabète
B - Il faut considérer que la patiente n'était en fait à jeun
C - On doit demander un dosage de l'insuline
D - On porte le diagnostic de tubulopathie distale
E - Le seuil de réabsorption du glucose est certainement trop bas par rapport à la quantité du glucose filtré
Bonne(s) réponse(s) : A E

Toute glycosurie doit faire rechercher un diabète même si le seuil de réabsorption du glucose est diminué pendant la
grossesse et entraîne parfois une glycosurie physiologique. Celle-ci est indépendante de l'état de jeûne. L'insulinémie n'a pas
d'intérêt diagnostique.

La créatininémie basse est due à :


A - Un syndrome d'hyperfiltration
B - Une erreur de laboratoire
C - Un syndrome néphrotique
D - Un état de dénutrition
E - A la consommation de créatinine par le foetus
Bonne(s) réponse(s) : A

La clairance de la créatinine est physiologiquement augmentée pendant la grossesse par hyperfiltration.

150
Exclusivement sur DOC - DZ : www.doc-dz.com NADJI 85
RESIDANAT EN POCHE TOME II
Cas Clinique en QCM

Du fait de la protéinurie et de la protidémie ainsi que de la prise de poids :


A - On conclut à un syndrome néphrotique
B - On conclut à une pré-éclampsie
C - Une volémie plus élevée que chez une femme normale non enceinte est probable
D - L'administration de 40 mg de Lasilix est souhaitable
E - On soupçonne une dilution plasmatique
Bonne(s) réponse(s) : C E

La prise de poids est correcte pour le 8ème mois, les oedèmes vespéraux sont banals en fin de grossesse, la protidémie est
abaissée par hémodilution physiologique et la protéinurie, ici faible, peut être en rapport avec la bactériurie, en l'absence
d'HTA gravidique. Les diurétiques sont contre-indiqués pendant la grossesse lorsqu'on soupçonne un syndrome toxémique.

La bactériurie :
A - Doit être traitée
B - Est ininterprétable du fait de la leucocyturie normale
C - Conduit à l'indication d'une urographie intra-veineuse (réduite à deux clichés)
D - Est probablement limitée au bas appareil
E - Fait courir un risque de pyélonéphrite
Bonne(s) réponse(s) : A D E

Toute bactériurie doit être traitée car risque d'évoluer vers une infection urinaire authentique (avec leucocyturie positive)
rapidement propagée au haut appareil pendant la grossesse (atonie urétérale physiologique).

Chez cette femme on s'attend à ce qu'il existe :


A - Une réninémie basse
B - Une hypervolémie
C - Un flux sanguin rénal élevé
D - Une pression artérielle plus élevée le mois suivant
E - Une aldosteronémie élevée
Bonne(s) réponse(s) : B C E

Comme dans toute grossesse normale, il existe une augmentation de la volémie avec bilan sodé positif, augmentation du flux
sanguin rénal et de la filtration glomérulaire, compensée par une réabsorption tubulaire du sodium grâce à une
hyperaldostéronémie.

Vous dépistez chez une femme de 65 ans. en bon état général, sans autre maladie associée, un carcinome de l'endomètre,
strictement limité au corps utérin.

Parmi les facteurs de risque vous retenez :


A - Obésité
B - Port du stérilet
C - Traitement aux estrogènes
D - Multiplicité de partenaires
E - Grossesses multiples
Bonne(s) réponse(s) : A C

Le cancer de l'endomètre est favorisé par l'hyperoestrogénie relative (obésité) ou iatrogène. Grossesses multiples et
partenaires multiples sont des facteurs de risque de cancer du col.

A quel stade est-elle vue ?


A - Stade 0
B - Stade 1
C - Stade 2
D - Stade 3
E - Stade 4
Bonne(s) réponse(s) : B

Le stade 0 est le cancer limité à l'endomètre, non invasif, et le stade II correspond à l'atteinte du col.

151
Exclusivement sur DOC - DZ : www.doc-dz.com NADJI 85
RESIDANAT EN POCHE TOME II
Cas Clinique en QCM

Le traitement sera :
A - Chimiothérapie pré-opératoire + hystérectomie
B - Radiothérapie + chimiothérapie
C - Hystérectomie avec annexectomie
D - Hystérectomie + chimiothérapie
E - Curetage + hormonothérapie
Bonne(s) réponse(s) : C

Le stade 1 est classiquement traité par une curiethérapie vaginale suivie d'une hystérectomie totale non conservatrice avec
prélèvements ganglionnaires sous veineux, complétés en cas de ganglions envahis par une radiothérapie pelvienne. C'est un
cancer chirurgical peu sensible à la chimiothérapie.

L'examen histologique révèle un envahissement des 2/3 internes du myomètre. Faut-il faire un traitement
complémentaire ? Si oui, lequel ?
A - Non
B - Chimiothérapie
C - Hormonothérapie
D - Chimiothérapie + hormonothérapie
E - Radiothérapie pelvienne
Bonne(s) réponse(s) : E

Le risque d'envahissement ganglionnaire en l'absence de renseignement histologique sur les ganglions est une indication à la
radiothérapie en raison de l'envahissement des 2/3 internes du myomètre.

La surveillance sera faite essentiellement par :


A - Radio du thorax
B - Examens gynécologiques répétés
C - Echographie hépatique
D - Examens biologiques
E - Scanner pelvien
Bonne(s) réponse(s) : B

La surveillance clinique est fondamentale, associée à la surveillance des marqueurs, éventuellement positifs en préopératoire,
et des frottis vaginaux. La radio de thorax, l'échographie hépatique et le scanner seront faits à la demande.

Une patiente de 53 ans. sans antécédent médical particulier, consulte pour une aménorrhée depuis 6 mois et des bouffées de
chaleur. Elle n'est pas enceinte. Le diagnostic de ménopause est envisagé.

En cas d'hésitation diagnostique, la certitude de la ménopause peut être confirmée par la mise en évidence :
A - D'une augmentation du taux de FSH et d'un taux de LH effondré
B - D'une diminution des taux de FSH et de LH
C - D'une augmentation des taux de FSH et de LH
D - D'une augmentation du taux d'HCG bêta
E - D'une augmentation du taux de prolactine
Bonne(s) réponse(s) : C

L'élévation du LHRH, de la LH et FSH correspondent à la chute du taux des stéroïdes sexuels et à la raréfaction des follicules.
La sous-unité Bêta des HCG est spécifique de la grossesse contrairement à l'alpha HCG qui peut entraîner des réactions
croisées avec la LH.

La ménopause s'accompagne souvent :


A - D'un risque accru de maladie cardio-vasculaire
B - D'un risque accru de tassements vertébraux
C - D'une déminéralisation osseuse par carence en calcium et en vitamine D
D - D'une ostéoporose par accélération des phénomènes de résorption osseuse liée à la carence
oestrogénique
E - De l'apparition d'une hyperplasie endométriale
Bonne(s) réponse(s) : A B D

L'absence de protection vasculaire par les oestrogènes naturels entraîne un risque accru de maladie cardiovasculaire. Il existe
une atrophie endométriale par carence oestrogénique et une augmentation de la résorption ostéoclasique.

152
Exclusivement sur DOC - DZ : www.doc-dz.com NADJI 85
RESIDANAT EN POCHE TOME II
Cas Clinique en QCM

Un traitement estroprogestatif est envisagé. Ce type de traitement serait contre-indiqué par :


A - Antécédent récent de cancer du sein
B - Antécédent d'ostéoporose chez la mère de la patiente
C - Hypertension artérielle traitée
D - Antécédent de dysplasie cervicale guérie
E - Incontinence urinaire d'effort
Bonne(s) réponse(s) : A

Le cancer du sein contrairement aux dysplasies cervicales est hormono-dépendant et contre-indique donc les traitements
oestrogéniques.

Le traitement doit comporter :


A - Un traitement estrogénique par voie injectable
B - Un traitement d'estriol par voie percutanée
C - Un traitement d'estradiol par voie percutanée, 10 jours par mois, suivi d'un traitement progestatif 20 jours
par mois
D - Un traitement d'estradiol par voie percutanée 20 jours par mois, associé du 11ème au 20ème jour à un
traitement progestatif
E - De l'estradiol percutanée 1 jour/2 en association avec des progestatifs per os un jour sur deux
Bonne(s) réponse(s) : D

L'association de progestatifs en 2ème partie de cycle est nécessaire pour contre balancer les effets mitotiques des
oestrogènes.

Une jeune femme de 31 ans, sans antécédents particuliers, présente une séroconversion toxoplasmique durant sa grossesse.
En effet, alors qu'elle a toujours été séronégative, sa sérologie toxoplasmique du 6ème mois s'est positivée à 400 UI, avec
présence d'lgM spécifiques. L'examen clinique est strictement normal. L'état général est satisfaisant, en dehors d'une légère
asthénie depuis 3 semaines, mise sur le compte de sa grossesse. Depuis son enfance cette jeune femme a toujours été
entourée d'animaux de compagnie ; actuellement deux chats.

Au début de sa grossesse, elle a reçu les conseils hygiéno-diététiques suivants ; l'un d'entre eux est-il superflu
vis à vis de la toxoplasmose ?
A - Ne pas consommer de viande saignante
B - Bien laver salades et crudités
C - Eviter le contact avec les chats
D - Stériliser la litière des chats à l'eau de javel
E - Faire vacciner les chats contre la toxoplasmose
Bonne(s) réponse(s) : E

Il n'existe pas de vaccination contre la toxoplasmose. Le parasite se rencontre dans la viande ou dans les déjections des
chats, susceptibles de souiller la terre.

Cette patiente présente une toxoplasmose invasive asymptomatique, qui est la forme la plus fréquente chez les
adultes. Toutefois, elle aurait pu exprimer cliniquement sa toxoplasmose par tous les symptômes suivants, sauf
un. Lequel ?
A - Fébricule persistant
B - Exanthème fugace
C - Adénopathie cervico-faciale
D - Asthénie
E - Amblyopie
Bonne(s) réponse(s) : E

Il peut exister également des céphalées, un syndrome mononucléosique avec éosinophilie.

Le titre de 400 UI avec présence d'lgM spécifiques permet de penser que la date de contamination présumée
remonte à quelle période de la grossesse ?
A - Au cours du 1 er mois de grossesse
B - Au début du 2ème mois de grossesse
C - Au début du 3ème mois de grossesse
D - Au début du 4ème mois de grossesse
E - Au début du 5ème mois de grossesse
Bonne(s) réponse(s) : E

La sérologie toxoplasmique est dosée tous les mois chez les femmes séronégatives. De plus, les IGM sont fugace : 3
semaines en moyenne, et signent une infection récente surtout en cas d'élévation des taux sur 2 prélèvements.

153
Exclusivement sur DOC - DZ : www.doc-dz.com NADJI 85
RESIDANAT EN POCHE TOME II
Cas Clinique en QCM

Parmi les traitements suivants, l'un est particulièrement indiqué dans le cas de contaminations per gravidiques.
Lequel ?
A - Fansidar® 3 comprimés par semaine
B - Bactrim® 4 comprimés par jour
C - Rovamycine® 500 : 2 comprimés par jour
D - Rovamycine® 500 : 6 comprimés par jour
E - Adiazine® 4 comprimés par jour
Bonne(s) réponse(s) : D

La Rovamycine® est efficace à la dose de 3 g par jour. Les sulfamides sont efficaces mais contre-indiqués pendant la
grossesse et l'Adiazine® réservée au traitement de l'infection foetale prouvée.

Chez cette jeune femme, le traitement antitoxoplasmique instauré jusqu'à la fin de la grossesse, diminue le
risque de contamination foetale approximativement dans quel pourcentage ?
A - 10%
B - 25%
C - 50%
D - 75 %
E - 100%
Bonne(s) réponse(s) : D

Le traitement antitoxoplasmique fait passer le taux d'infection foetale de 25 % à 8 %, d'après l'étude de Desmorts.

Lorsque cette jeune femme accouchera, le bilan néonatal à orientation toxoplasmique devra comporter :
A - Sérologie mère et sang du cordon
B - Dans le sang du cordon : dosage des IgM non spécifiques, des gamma-GT, et de la L.D.H.
C - Echographie transfontanellaire
D - Inoculation du sang du cordon, et du L.C.R.
E - Radiographie du rachis
Bonne(s) réponse(s) : A C D

Il est préférable de prélever directement le sang foetal car le sang du cordon peut être souillé par les IGM maternelles. L'étude
sérologique doit être menée en parallèle chez l'enfant et la mère.

Dans le cas où son nouveau-né est cliniquement sain, quel est l'argument qui permettra d'éliminer toute atteinte
congénitale :
A - Inoculation du placenta, du caillot du sang du cordon à la souris : négative
B - Evolution normale de la courbe de poids et du périmètre crânien
C - Absence de toute lésion du F.O. à l'âge d'un an
D - Négativation des sérologies au cours de la 1 ère année
E - Développement psychomoteur normal
Bonne(s) réponse(s) : D

Il existe des formes infracliniques à risque d'atteinte ophtalmologique à l'adolescence, qui nécessitent une surveillance
clinique avec F.O. et une surveillance des sérologies au cours de la 1ère année, même lorsque les résultats initiaux sont
négatifs.

Une étudiante, 19 ans, se présente au service de médecine préventive en raison d'une irritation vulvo-vaginale, ayant débuté
2 jours auparavant, avec leucorrhées jaune-verdâtres et dyspareunie. Elle ne signale aucun épisode antérieur et son
partenaire ne se plaint d'aucun symptôme. Après examen gynécologique, très pénible pour elle, l'examen microscopique des
leucorrhées met en évidence l'agent causal. Le traitement prescrit est efficace en 48 h.

Quel est votre diagnostic ?


A - Enterobius vermicularis
B - Trichomonas vaginalis
C - Candida albicans
D - Trichomonas hominis
E - Torulopsis glabrata
Bonne(s) réponse(s) : B

Le trichomonas est un parasite flagellé (mobile) visible à l'examen direct, au microscope, des leucorrhées.

154
Exclusivement sur DOC - DZ : www.doc-dz.com NADJI 85
RESIDANAT EN POCHE TOME II
Cas Clinique en QCM

Le traitement de choix est :


A - Amphotéricine B (comprimés gynécologiques)
B - Metronidazole
C - Flubendazole
D - Tinidazole
E - Miconazole (comp. gynéco)
Bonne(s) réponse(s) : B

Le métronidazole a l'avantage sur le tinidazole, tout aussi efficace, d'exister sous forme d'ovules gynécologiques permettant
un traitement local.

Son partenaire doit-il être traité ?


A - Oui par traitement général
B - Non
C - Seulement si l'examen est positif
D - Seulement s'il présente des symptômes
E - Oui par instillations locales
Bonne(s) réponse(s) : A

Le traitement minute par voie générale des partenaires est conseillé en raison de la fréquence des formes asymptomatiques,
de la difficulté du diagnostic chez l'homme et de la transmission sexuelle fréquente.

L'examen permettant d'arriver le plus sûrement et le plus rapidement au diagnostic est :


A - Une sérologie parasitaire
B - Un hémogramme
C - Une recherche du parasite dans les urines
D - La recherche du parasite à l'examen direct entre lame et lamelle
E - La recherche du parasite après coloration sur un frottis de sécrétion vaginale
Bonne(s) réponse(s) : D

La mobilité et la taille du parasite fait qu'il est facilement et rapidement repérable sur des prélèvements frais, mais il existe des
colorations (M.G.G, acridine orange) sur frottis fins. Il n'existe pas de sérologie fiable.

Une femme de 25 ans, enceinte de 3 mois, consulte pour une suspicion d'herpès génital. Elle a depuis 2 jours trois lésions
vésiculeuses, actuellement ulcérées, de 2 mm de diamètre chacune, sur la face externe de la petite lèvre droite. C'est la
troisième fois depuis 1 an qu'elle souffre de pareilles lésions à cet endroit.

Vous pourrez confirmer avec quasi certitude le diagnostic d'herpès par :


A - Recherche d'herpès simplex virus (HSV) dans les lésions par isolement en culture de cellules in vitro
B - Recherche d'antigènes HSV par cytodiagnostic en immunofluorescence sur des cellules prélevées à cette
patiente
C - Dosage des anticorps HSV par neutralisation ou fixation du complément dans un sérum prélevé de la
patiente
D - Dosage des anticorps par neutralisation ou fixation du complément dans 2 sérums de la patiente,
à J 1 et J21

E - Dosage des IgM spécifiques de l'HSV type II dans un sérum de la patiente 5 à 10 jours après la
consultation
Bonne(s) réponse(s) : A

La recherche d'HSV sur culture de cellules est la plus fiable, l'immunofluorescence est rapide, sensible mais moins spécifique.
La spécificité des méthodes immunologiques est médiocre, la réaction de fixation du complément est abandonnée.

Le diagnostic d'herpès génital récurrent étant acquis, quelle attitude raisonnable conseillez-vous à cette femme ?
A - De réfléchir à l'éventualité d'une IVG
B - De prendre de l'Acyclovir par voie orale jusqu'à l'accouchement qui pourra se faire alors par voie basse
C - D'accoucher systématiquement par césarienne
D - De rechercher une excrétion asymptomatique D'HSV dans les sécrétions génitales par inoculation à des
cultures de cellules in vitro
E - Aucune des propositions précédentes
Bonne(s) réponse(s) : D

Il existe actuellement 2 tendances :


- La plus classique consiste à prélever toutes les semaines après la 34ème SA les sécrétions cervicovaginales, la césarienne
n'étant pratiquée qu'en cas de prélèvement positif à terme ou de lésions.
- La plus moderne consiste, en l'absence de lésion ou de poussée récente à rechercher le virus systématiquement en début
de travail chez la mère et l'enfant et à traiter en cas de prélèvement positif par l'Acyclovir®.

155
Exclusivement sur DOC - DZ : www.doc-dz.com NADJI 85
RESIDANAT EN POCHE TOME II
Cas Clinique en QCM

Cette femme ne revient vous voir qu'au moment du travail, à 8 mois 1/2. Depuis la dernière visite, elle n'a pas
eu de récidives manifestes d'herpès et l'examen gynécologique ne montre pas de lésions. La poche des eaux
n'est pas rompue. Si cette femme accouche par voie basse, le risque d'herpès pour le nouveau-né peut être
estimé à :
A - 50%
B-2%
C-5%
D - 1%
E - Moins de 1/1000
Bonne(s) réponse(s) : E

Le risque est de 75 % lorsque la primo-infection survient dans le mois précédent l'accouchement et de 2 à 5 % en cas de
récurrence herpétique proche de l'accouchement.

Cette femme ayant décidé d'accoucher par voie basse, quelle(s) mesure(s) allez-vous préconiser ?
A - Le dosage des Ac anti HSV chez l'enfant
B - Le traitement systématique du nouveau-né par acyclovir oral pendant 10 jours
C - La séparation de l'enfant et de la mère
D - L'interdiction d'allaiter l'enfant au sein
E - Aucune des propositions ci-dessus
Bonne(s) réponse(s) : E

Le traitement n'est institué qu'en cas de positivité des prélèvements faits en début de travail. Des mesures d'hygiène simples
(ex : lavage des mains à la Betadine® avant manipulation du bébé) suffisent.

Une femme de 20 ans, originaire d'Afrique du Nord, est à 9 semaines d'aménorrhée gravidique. Depuis une semaine, elle
présente des métrorragies de sang rouge, de faible abondance. Il n'y a pas de douleurs pelviennes. La tension artérielle,
strictement normale avant la grossesse, atteint 14/9. A l'examen clinique, le col est sain, violacé, et le frottis pratiqué quinze
jours auparavant est normal. L'utérus est plus gros que ne le voudrait la durée de l'aménorrhée et atteint la taille de 14 cm à
15 semaines d'aménorrhée gravidique. Dans les culs de sacs latéraux, on retrouve des kystes ovariens bilatéraux, de la taille
d'une orange.

Parmi les diagnostics suivants, quel est le plus vraisemblable ?


A - Menace d'avortement spontané par aberration chromosomique
B - Grossesse ectopique
C - Grossesse môlaire
D - Cancer ovarien bilatéral
E - Grossesse gémellaire
Bonne(s) réponse(s) : C

En faveur de la grossesse môlaire on retrouve :


- L'origine d'Afrique du Nord.
- Les métrorragies persistantes.
- L'augmentation de volume de l'utérus.
- Les kystes ovariens bilatéraux.
- L'HTA gravidique précoce.

Parmi les examens suivants, lequel (ou lesquels) demanderez-vous ?


A - Dosage de l'HCG plasmatique
B - Coelioscopie
C - Echographie pelvienne
D - Biopsie ovarienne per-coelioscopique
E - Dosage de l'antigène carcino-embryonnaire
Bonne(s) réponse(s) : A C

Les HCG sont très élevés pour le terme de la grossesse. L'échographie constate l'absence de structure embryonnaire et les
images "en flocon de neige" des vésicules.

Parmi les attitudes thérapeutiques suivantes, laquelle (ou lesquelles) pourriez-vous choisir ?
A - Expectative sous surveillance échographique des kystes ovariens
B - Evacuation utérine à l'aide de prostaglandine
C - Traitement médical anti-hypertenseur
D - Evacuation utérine par aspiration
E - Evacuation utérine par curetage
Bonne(s) réponse(s) : D

Une môle diagnostiquée doit être évacuée mécaniquement (les prostaglandines sont inefficaces) l'aspiration limite le risque de
synéchie. Le contenu est examiné en anatomopathologie.
156
Exclusivement sur DOC - DZ : www.doc-dz.com NADJI 85
RESIDANAT EN POCHE TOME II
Cas Clinique en QCM

Dans la suite, quelle sera votre attitude ?


A - Poursuite de la surveillance clinique seule
B - Surveillance par échographies, et dosage de béta-HCG plasmatique pendant un an
C - Institution d'une contraception pendant un an
D - Chimiothérapie systématique pendant 6 mois
E - Laparotomie pour "second look" après 6 mois de chimiothérapie
Bonne(s) réponse(s) : B C

La surveillance des Bêtas HCG doit être continuée un an, même en cas d'évolution favorable. La contraception
oestroprogestative est impérative pendant cette durée. La chimiothérapie n'est indiquée qu'en cas de môle invasive ou de
choriocarcinome.

Une femme de 27 ans, institutrice de classe maternelle, mère de trois enfants, est à 31 semaines d'aménorrhée gravidique.
Elle habite à 40 kms de son lieu de travail et doit chaque jour accomplir ce trajet en voiture. On retrouve dans ses antécédents
: trois accouchements qui se sont normalement déroulés, à terme, une appendicectomie dans l'enfance, deux interruptions
volontaires de grossesse à 25 et 26 ans. Depuis quarante-huit heures, elle se plaint de douleurs lombaires droites, de
nausées et depuis quelques heures elle ressent des brûlures mictionnelles. Sa température est à 38°. Enfin, elle se plaint
depuis deux heures de contractions utérines survenant toutes les dix minutes, de plus en plus longues (trente secondes) et
douloureuses depuis une heure. Le col utérin est long, ouvert à l'orifice externe, déhiscent à l'orifice interne. La hauteur utérine
est de 27 cm. Le foetus, dont les dimensions échographiques ont toujours été concordantes avec le terme, est vivant :
Mouvements actifs et bruits du coeur sont présents. Il n'y a jamais eu d'hypertension artérielle, ni de glycosurie, Mais la
recherche de protéinurie avec des bandelettes est positive.

Devant cette menace d'accouchement prématuré, quelle doit être parmi les décisions suivantes, celle qui
s'impose ?
A - Prescription d'un repos simple avec arrêt de travail
B - Traitement à domicile par repos et progestatif type rétro-progestérone
C - Repos simple à domicile avec auscultations bi-hebdomadaires par un service de sages-femmes à domicile
D - Hospitalisation d'urgence pour examens complémentaires et traitement tocolytique
E - Hospitalisation d'urgence pour cerclage du col
Bonne(s) réponse(s) : D

La menace d'accouchement prématurée est sévère (contractions rapprochée et modification du col) et impose l'hospitalisation
pour bilan étiologique et traitement tocolytique. Le cerclage du col est inefficace au 3ème trimestre.

Parmi les examens suivants le(s)quel(s) demanderez-vous ?


A - Monitorage avec enregistrement du rythme cardiaque et des contractions utérines deux fois par jour
B - Examen cyto-bactériologique des urines
C - Amnioscopie
D - Electrocardiogramme de la patiente
E - Test de O'Sullivan
Bonne(s) réponse(s) : A B D

La surveillance de la tolérance foetale et de l'efficacité de la tocolyse est impérative. L'électrocardiogramme fait avant
traitement recherche une contre-indication à la tocolyse par Bêta+ (cardiopathie, trouble du rythme). L'ECBU est systématique
et d'autant plus nécessaire qu'il existe une pyélonéphrite clinique.

Une menace d'accouchement prématuré est favorisé par :


A - Longs trajets quotidiens en voiture
B - Multiparité et charges de famille
C - Métier d'institutrice de maternelle
D - Infection urinaire
E - Antécédents d'avortements provoqués
Bonne(s) réponse(s) : A B C D

Les avortements provoqués favorisent les avortements spontanés tardifs par béance du col du synéchie utérine.

En cas d'accouchement prématuré le nouveau-né est particulièrement menacé par :


A - Maladie des membranes hyalines
B - Hémorragie de Benkiser
C - Ictère néo-natal
D - Infection septicémique néo-natale acquise in utero
E - Hypotrophie
Bonne(s) réponse(s) : A C D

La fièvre maternelle fait craindre l'infection néonatale. A 31 SA la maladie des membranes hyalines est à redouter (immaturité
pulmonaire). L'ictère néo-natal est plus fréquent en cas de prématurité. Le petit poids n'est pas synonyme d'hypotrophie enfin
l'hémorragie de Ben Kiser se voit lors de la rupture des membranes en cas d'insertion vélamenteuse du cordon.

157
Exclusivement sur DOC - DZ : www.doc-dz.com NADJI 85
RESIDANAT EN POCHE TOME II
Cas Clinique en QCM
Une femme âgée de 28 ans est agricultrice. En ramenant à l'étable son troupeau de vaches, elle reçoit un coup de sabot. Elle
est hospitalisée avec plaie profonde et contuse du genou droit et ouverture de l'articulation. La radiographie ne décèle aucune
fracture. On apprend qu'elle est enceinte, ses dernières règles remontant à deux mois. Le traitement chirurgical comporte un
parage de la plaie et une immobilisation de l'articulation par une gouttière plâtrée. Comme la patiente n'est pas à jour dans sa
vaccination antitétanique, elle reçoit des gamma globulines spécifiques et la première injection de vaccin. Enfin une
antibiothérapie est prescrite avec Oxacilline (Bristopen), 5 g par jour.

Pendant la grossesse, on est autorisé d'effectuer la vaccination contre :


A - La grippe
B - La tuberculose
C - L'hépatite B
D - La rubéole
E - La coqueluche
Bonne(s) réponse(s) : A C

Les vaccination contre la grippe et l'hépatite B sont réputés sans risque en cours de grossesse. Les autres vaccins
comportent des virus ou bactéries vivantes et sont à éviter.

Le risque tératogène d'un médicament est minimum :


A - Pendant les six premiers jours du développement de l'oeuf
B - De la deuxième à la quatrième semaine
C - De la cinquième à la huitième semaine
D - De la neuvième à la douzième semaine
E - pendant le dernier trimestre
Bonne(s) réponse(s) : A

Avant la nidation, l'oeuf est protégé de l'action des médicaments grâce à son autonomie (absence de connexion vasculaire).

Pour diminuer la douleur après l'intervention vous pouvez prescrire sans risque pour l'embryon :
A - Du paracétamol
B - De l'aspirine
C - De la morphine
D - De la phénacétine
E - De la glafénine
Bonne(s) réponse(s) : A B C D E

La morphine est à éviter en cure prolongée. Glafénine et phénacétine ne sont pas contre-indiquées mais se méfier
des associations. L'aspirine n'est pas contre-indiquée mais à utiliser en 2ème intention et à éviter au 3ème trimestre.

Pour prévenir la phlébite sous plâtre il est déconseillé de prescrire chez cette femme enceinte :
A - De l'héparine
B - De la Fraxiparine
C - De la Calciparine
D - Un antivitamine K tel que le Sintrom
E - Un anti-inflammatoire
Bonne(s) réponse(s) : D E

Les antivitamines K sont contre-indiquées pendant la grossesse et les anti-inflammatoires ne sont pas suffisants face aux
risques cumulés de la grossesse et de l'immobilisation.

Si cette femme est allergique aux pénicillines, quel antibiotique prescrire ?


A - Un macrolide
B - Une tétracycline
C - Un aminoside
D - Un imidazolé
E - Un polypeptide
Bonne(s) réponse(s) : A

Tétracyclines et imidazolés sont contre-indiqués ; les aminosides sont responsables d'ototoxicité et les polypeptides ont un
spectre étroit.

158
Exclusivement sur DOC - DZ : www.doc-dz.com NADJI 85
RESIDANAT EN POCHE TOME II
Cas Clinique en QCM
Cette patiente de 42 ans consulte pour des douleurs lombaires droites irradiant à la fesse et à la face externe de la cuisse. A
l'examen, on note un point douloureux paravertébral en regard de L3 et au niveau de l'épine iliaque postéro-supérieure droite.
Il n'y a pas de signe de Lasègue. Le reste de l'examen est normal. La patiente a été traitée trois ans auparavant pour un
adénocarcinome du sein droit par mastectomie élargie, suivie de radiothérapie.

L'examen habituellement le plus sensible pour faire le diagnostic de métastase osseuse est :
A - Scintigraphie du corps entier au calcium 125
B - Scintigraphie du corps entier aux polyphosphates marquées au technetium
C - Radiographie du squelette
D - Tomographie du rachis
E - Tomodensitométrie vertébrale
Bonne(s) réponse(s) : B

La scintigraphie au pyrophosphate de technétium permet de dépister les métastases osseuses cliniquement muettes mais il
existe des faux positifs. Il faut donc étayer ce diagnostic par des tomographies ou une tomodensitométrie de la région
suspecte.

L'examen biologique destiné à apprécier le risque métabolique de la maladie métastatique osseuse est :
A - Calcémie
B - Numération formule
C - Phosphatases acides
D - Antigène carcino-embryonnaire
E - Electrophorèse des protéines
Bonne(s) réponse(s) : A

L'hypercalcémie qui se voit dans le cadre de métastases ostéolytiques multiples, outre son caractère très péjoratif, entraîne un
risque métabolique immédiat qui peut être gravissime.

Le bilan d'extension des métastases dans les autres organes cibles doit faire demander en plus de la radio
pulmonaire et de l'antigène carcino-embryonnaire :
A - Scintigraphie hépatique
B - Scintigraphie cérébrale
C - Echotomographie pelvienne + Scanner hépatique
D - Mammographie du sein restant
E - Echographie hépatique
Bonne(s) réponse(s) : D

La mammographie du sein restant doit être systématique en raison de la fréquence des cancers controlatéraux. L'échographie
hépatique est réservée pour certains aux modifications du bilan hépatique (gamma GT, P.A.) systématique pour d'autres.

Au cours d'une castration chirurgicale, une métastase est découverte sur l'ovaire droit et une biopsie confirme la
métastase et la présence de récepteurs hormonaux dans les cellules tumorales. Le traitement hormonal
complémentaire sera :
A - Pas de traitement complémentaire
B - Corticothérapie (2mg/kg)
C - Tamoxifène
D - Anti-prolactiniques
E - Oestrogènes à faibles doses
Bonne(s) réponse(s) : C

Le tamoxifène (Nolvadex®) est un antioestrogène utilisé essentiellement chez les femmes ménopausées ou castrées avec
récepteurs + aux oestrogènes. Il se fixerait sur ces récepteurs sans les activer.

La patiente continue à souffrir de lombalgies un mois après l'intervention, le traitement proposé est alors :
A - Epiphysiolyse
B - Immunothérapie par le BCG
C - Surrénalectomie chimique
D - Radiothérapie sur la colonne lombaire
E - Antalgiques type morphine
Bonne(s) réponse(s) : D

L'irradiation rachidienne, éventuellement complétée de chirurgie de consolidation en cas de risque de fracture, est une bonne
indication dans les métastases osseuses entraînant des troubles de compression. L'effet antalgique est bon dans 90 % des
cas en 3 à 4 semaines.

159
Exclusivement sur DOC - DZ : www.doc-dz.com NADJI 85
RESIDANAT EN POCHE TOME II
Cas Clinique en QCM

Une chimiothérapie est envisagée. Quelle combinaison de drogues est habituellement utilisée :
A - 5 FU - endoxan - cis-platine
B - Cyclophosphamide-oncovin-bléomycine
C - Adriamycine - 5 fluoro-uracile-cyclophosphamide
D - Nitrosouré - bléomycine
E - Aucune des propositions ci-dessus
Bonne(s) réponse(s) : C

Les chimiothérapies d'association contenant de l'Adriamycine sont les plus utilisées. Le "FAC" ici proposé est le plus classique
et donne un taux de réponse positive dans 60 à 70 % des cas.

Une femme de 65 ans, mariée, consulte pour une incontinence d'urines survenant à la toux, au rire, à la marche rapide, au
passage de la position assise à la position debout et nécessitant le port permanent de garnitures. Dans ses antécédents, on a
la notion d'une plaie vésico-vaginale après hystérectomie totale qui a fait l'objet d'une intervention chirurgicale secondaire. En
outre elle a eu 4 accouchements difficiles. De temps en temps, elle a des cystites qui aggravent son incontinence. La nuit et
en position assise, elle est parfaitement sèche. L'examen de l'abdomen et des fosses lombaires est normal. L'examen du
périnée met en évidence un raccourcissement de la distance ano-vulvaire, une rectocèle et une cystocèle. A la toux, on voit
une grosse fuite d'urines par le méat uréthral. Au toucher, la vessie est souple, les releveurs sont de médiocre qualité.
L'examen neurologique est normal. Les urines sont limpides.

Parmi les causes d'incontinence d'urine chez la femme, indiquez à laquelle correspond le tableau clinique décrit :
A - Vessie neurologique post-opératoire
B - Abouchement ectopique de l'uretère
C - Incontinence par déficit de la sangle pelvipérinéale
D - Fistule vésico-vaginale
E - Impériosité mictionnelle
Bonne(s) réponse(s) : C

Il s'agit d'une incontinence urinaire d'effort typique liée à la chute du col vésical hors de l'enceinte de pression abdominale, par
déficit du plancher périnéal.

Parmi les épreuves cliniques proposées, indiquez celle qui permet d'affirmer simplement le diagnostic du
mécanisme de l'incontinence :
A - Absorption de bleu de méthylène per os couplée à l'examen au spéculum
B - Injection de bleu de méthylène dans la vessie couplée au spéculum
C - Manoeuvre de réduction de la cystocèle
D - Manoeuvre de reposition du col vésical par 2 doigts vaginaux
E - Manoeuvre de réduction de la rectocèle
Bonne(s) réponse(s) : D

La remise en place du col vésical dans l'enceinte de pression abdominale corrige le défaut initial et permet de présumer des
chances de succès d'une intervention chirurgicale.

Parmi les explorations complémentaires proposées, indiquez la plus utile pour poser l'indication thérapeutique,
dans ce cas particulier :
A - La cystoscopie
B - La cystographie rétrograde
C - L'urographie intraveineuse
D - L'étude urodynamique du bas appareil urinaire
E - L'échographie pelvienne
Bonne(s) réponse(s) : D

L'étude urodynamique mesure les pressions vésicales cervicales et uréthrales et lors de la toux et permet d'apprécier la bonne
transmission des pressions à la toux. Dans ce cas, un défaut de transmission important est prévisible et permet de poser
l'indication opératoire. Cet examen rechercher d'autres anomalies (hypercontractilité vésicale, instabilité uréthrale, insuffisance
sphinctérienne) qui modulent l'indication.

160
Exclusivement sur DOC - DZ : www.doc-dz.com NADJI 85
RESIDANAT EN POCHE TOME II
Cas Clinique en QCM

Au terme de l'examen et des explorations complémentaires, indiquez le ou les procédés thérapeutiques que
vous préconisez :
A - La gymnastique périnéale à l'exclusion de tout autre geste
B - La suspension chirurgicale du col vésical par voie abdominale pure
C - Le port d'un pessaire
D - La colpectomie totale par voie vaginale
E - La périnéorraphie avec soutènement du col vésical
Bonne(s) réponse(s) : E

La rééducation périnéale avant et après l'acte chirurgical est indiquée mais est insuffisante à elle seule dans les incontinences
sévères. Dans le cas, énoncé l'association à la cervicocystoptose d'une rectocèle et d'une distance anovulvaire courte est
l'indication à la cure chirurgicale conjointe du prolapsus et a une périnéoraphie.

Une femme de 55 ans, non ménopausée, consulte pour un nodule du quadrant supéro-externe du sein droit apparu 2 mois
auparavant. L'interrogatoire retrouve la notion d'un cancer du sein chez la grand-mère maternelle, une puberté à l'âge de 10
ans, une première grossesse à l'âge de 32 ans sans allaitement en raison d'un abcès du sein et enfin une cholecystectomie
pour lithiase à 49 ans. Cette malade a régulièrement fumé 1/2 paquet de cigarettes par jour pendant 30 ans. A l'examen : la
tumeur est mobile mais s'accompagne d'une attraction cutanée bien visible à jour frisant ; elle mesure 2,5 cm dans son plus
grand diamètre. A la palpation de l'aisselle homolatérale, on retrouve un ganglion de 1 cm de diamètre les autres territoires
ganglionnaires sont normaux ; il n'existe pas d'hépatomégalie, il n'y a pas de douleurs osseuses et l'état général est conservé.

Chez cette malade, parmi les antécédents suivants, on retient comme facteur de risque reconnu pour le cancer
du sein ?
A - Puberté précoce-ménopause tardive
B - Antécédent d'abcès du sein
C - Première grossesse tardive
D - Tabagisme
E - Cholecystectomie
Bonne(s) réponse(s) : A C

L'hyperoestrogénie relative est le point commun de tous les facteurs de risque du cancer du sein auquel s'ajoute un facteur
héréditaire ainsi que l'effet protecteur d'une maturation précoce du sein lors d'une grossesse jeune.

Dans la classification TNM (UICC - 1987), comment classez-vous ce cancer ?


A - T1 N1
B - T2 N0
C - T2 N1
D - T4 N0
E - T4 N1
Bonne(s) réponse(s) : C

L'attraction cutanée n'est pas un signe d'envahissement mais de rétraction des tractus fibreux (ligaments de Cooper). Seul
l'oedème cutané ("peau d'orange") fait classer la tumeur dans les stades IV.

Lequel des examens complémentaires est indispensable pour affirmer la malignité du nodule et autoriser la
mise en route du traitement spécifique ?
A - L'échographie mammaire
B - La mammographie
C - Le dosage de l'antigène carcino-embryonnaire
D - L'examen histopathologique par biopsie
E - La thermographie
Bonne(s) réponse(s) : D

Seul l'examen anatomo-pathologique (la cytologie ne suffit pas) permet d'affirmer le caractère malin de la tumeur et autorise le
traitement radical.

Compte tenu des caractères de la tumeur, laquelle des attitudes thérapeutiques suivantes permet
statistiquement d'obtenir un contrôle local satisfaisant avec des séquelles minimes ?
A - Intervention de Patey
B - Tumorectomie et curage axillaire suivi d'irradiation
C - Mammectomie simple suivie d'irradiation
D - Irradiation exclusive
E - Intervention de Halsted
Bonne(s) réponse(s) : B

Les essais randomisés ont prouvé que le traitement conservateur (B) était aussi efficace que la chirurgie large (A) dans les
tumeurs de moins de 3 cm, mobiles, excentrées, lorsque le volume mammaire est suffisant pour obtenir un bon résultat
esthétique.
161
Exclusivement sur DOC - DZ : www.doc-dz.com NADJI 85
RESIDANAT EN POCHE TOME II
Cas Clinique en QCM

Parmi les facteurs suivants, lequel ou lesquels vous inciteraient à proposer une chimiothérapie adjuvante ?
A - Grade II de l'index de Scarff-Bloom et Richardson
B - Adhérence cutanée de la tumeur
C - Positivité des récepteurs hormonaux tumoraux
D - Atteinte ganglionnaire histologique
E - Antécédents familiaux de cancer du sein
Bonne(s) réponse(s) : A D

L'atteinte ganglionnaire ( > ou égal à 3N+) chez les femmes non ménopausées est une indication à la chimiothérapie
adjuvante. Si il y a moins de 3 ganglions envahis, le grade histopronostic intervient alors dans l'indication de chimiothérapie.

Une femme enceinte dont la grossesse a débuté le 01/12/1989 a subi régulièrement des sérologies de la toxoplasmose dont
voici les résultats qu'elle vous présente le 10/05/90 :

- 01/03/90 IgE (ELISA) : Négatif


IgM (immunocapture) : Négatif

- 01/04/90 IgG (ELISA) : Négatif


IgM (immunocapture) : Positif

- 01/05/90 IgG (ELISA) : 150 UI/ml


IgM (immunocapture) : Positif

Comment interprétez-vous ces résultats ?


A - Toxoplasmose antérieure à la conception
B - Toxoplasmose postérieure à la conception
C - Risque de toxoplasmose congénitale
D - Absence de toxoplasmose congénitale
E - Contamination au cours du 2ème trimestre de la grossesse
Bonne(s) réponse(s) : B C E

La séroconversion survenant après la conception permet d'affirmer la contamination maternelle mais ne permet que de poser
l'indication de prélèvements foetaux, le foetus n'étant pas toujours atteint.

En fonction de votre interprétation, vous proposez d'emblée la conduite suivante :


A - Ponction de sang foetal
B - Interruption de grossesse
C - Surveillance échographique
D - Traitement par Rovamycine® (Spiramycine)
E - Abstention
Bonne(s) réponse(s) : A C D

Le traitement est instauré d'emblée et réduit le risque de contamination foetale sans diminuer le tableau clinique du foetus
infecté, qu'il faut donc surveiller échographiquement (calcifications cérébrales, hydrocéphalie) et prélever au cordon pour
pouvoir évaluer le risque de contamination et de séquelles, en sachant qu'un résultat n'égatif n'élimine pas l'infection foetale.

Dans la toxoplasmose de la femme enceinte :


A - Le risque de contamination foetale est maximal si la contamination a lieu au 3e trimestre de la grossesse
B - Le risque de lésions foetales est minimal si la contamination a lieu au cours du 1er trimestre de la
grossesse
C - Le risque de contamination foetale est diminué par la prise de Rovamycine® (Spiramycine)
D - Le risque de contamination foetale est totalement écarté par les résultats négatifs d'une éventuelle
ponction de sang du cordon
E - Le risque de lésions foetales est plus important si la mère a présenté des adénopathies
Bonne(s) réponse(s) : A C

Le risque de contamination est d'autant plus élevé et l'atteinte d'autant moins sévère que la grossesse est avancée.

Parmi les résultats des examens biologiques suivants pratiqués sur le sang foetal lequel ou lesquels est (sont)
en faveur d'une contamination foetale ?
A - Présence d'IgG anti-toxoplasmes
B - Présence d'IgM anti-toxoplasmes
C - Hyper gamma GT
D - Hyperplaquettose
E - Hyperéosinophilie
Bonne(s) réponse(s) : B C

Les IgG sont transmises par la mère et donc ininterprétables contrairement aux IgM spécifiques. Les signes indirects
d'infection sont l'augmentation des IgM globales, la thrombopénie, l'augmentation des gamma GT et des LDH.
162
Exclusivement sur DOC - DZ : www.doc-dz.com NADJI 85
RESIDANAT EN POCHE TOME II
Cas Clinique en QCM
Chez une femme de trente ans, sans antécédent particulier, un frottis cervico-vaginal systématique met en évidence des
cellules classe IV.
L'examen gynécologique complet montre un col utérin inflammatoire : le vagin est normal, de même que les paramètres. Le
reste de l'examen clinique ne montre aucune anomalie. La biopsie du col conclut à "L'existence de cellules malignes
épidermoïdes qui restent limitées à l'épithélium sans dépasser sa membrane basale".

Quelle est la conclusion de ce compte rendu anatomopathologique ?


A - Carcinome malpighien invasif
B - Carcinome glandulaire
C - Carcinome para-malpighien
D - Carcinome in situ ou intra épithélial
E - Carcinome micro-invasif
Bonne(s) réponse(s) : D

L'absence de franchissement de la membrane basale définit le carcinome intra-épithélial.

Le traitement de choix est :


A - Une hystérectomie simple
B - Une colpohystérectomie avec Iymphadénectomie
C - Une hystérectomie avec annexectomie bilatérale
D - Une colpectomie
E - Une amputation du col ou une conisation
Bonne(s) réponse(s) : E

Le traitement conservateur peut être proposé chez une femme jeune nullipare. L'intégrité de la pièce opératoire est nécessaire
afin de contrôler histologiquement les limites des lésions et leur extension en profondeur.

L'examen de la pièce opératoire montre qu'il s'agit d'un cancer invasif. Quel est le 1er relais de drainage
Iymphatique du col utérin ?
A - Iliaque interne
B - Lombo-aortique
C - Obturateur
D - Iliaque primitif
E - Inguinal
Bonne(s) réponse(s) : C

Le premier relai lymphatique est la chaîne ganglionnaire iliaque externe (ganglion obstruateur le plus souvent).

Il s'agit d'un T1. Que doit comporter le bilan pré-thérapeutique ?


A - Une échographie hépatique
B - Une coelioscopie
C - Une urographie intra-veineuse
D - Un dosage de bêta H.C.G.
E - Une rectoscopie
Bonne(s) réponse(s) : A C

L'UIV est nécessaire en pré-opératoire pour repérer les uretères et leur normalité. L'échographie hépatique et la radiographie
du thorax sont systématiques et servent de référence (les métastases d'emblée sont exceptionnelles).

Une jeune femme de 31 ans, sans antécédent particulier, présente une séroconversion toxoplasmique durant sa grossesse.
En effet, alors qu'elle a toujours été séronégative, sa sérologie toxoplasmique du 6ème mois s'est positivée à 400 UI, avec
présence d'IgM spécifiques. L'examen clinique est strictement normal. L'état général est satisfaisant, en dehors d'une légère
asthénie depuis 3 semaines, mise sur le compte de sa grossesse.

Au début de sa grossesse, elle a reçu les conseils hygiéno-diététiques ci-dessous. A votre avis lesquels sont
licites ?
A - Ne pas consommer de viande saignante
B - Bien laver salades et crudités
C - Eviter le contact avec les chats
D - Stériliser les litières du chat à l'eau de javel ou à l'ammoniaque
E - Faire bouillir le lait de vache
Bonne(s) réponse(s) : A B C D

Le parasite se rencontre dans la viande rouge ou dans la terre ou les endroits souillés par les déjections des chats.
Le parasite est détruit par la cuisson et la congélation.

163
Exclusivement sur DOC - DZ : www.doc-dz.com NADJI 85
RESIDANAT EN POCHE TOME II
Cas Clinique en QCM

Cette patiente présente une toxoplasmose invasive asymptomatique, qui est la forme la plus fréquente chez les
adultes. Toutefois, elle aurait pu exprimer cliniquement sa toxoplasmose par tous les symptômes suivants. sauf
un, lequel ?
A - Fébricule persistant
B - Exanthème fugace
C - Adénopathie cervicale
D - Asthénie sévère
E - Amblyopie
Bonne(s) réponse(s) : E

L'atteinte oculaire est la choriorétinite.

Le titre de 400 UI avec présence d'IgM spécifiques permet de penser que la date de contamination présumée
remonte à quelle période de la grossesse ?
A - Au cours du 1er mois de grossesse
B - Au début du 2ème mois de grossesse
C - Au début du 3ème mois de grossesse
D - Au début du 4ème mois de grossesse
E - Au début du 5ème mois de grossesse
Bonne(s) réponse(s) : E

Les taux ascendants d'Ac témoignent d'une infection de moins de 2 mois.

Parmi les traitements suivants, l'un est particulièrement indiqué dans le cas de contaminations per-gravidiques.
Lequel ?
A - Fansidar® : 3 comprimés par semaine
B - Bactrim® : 4 comprimés par jour
C - Rovamycine® : 2 comprimés par jour
D - Rovamycine® : 6 comprimés par jour
E - Adiazine® : 4 comprimés par jour
Bonne(s) réponse(s) : D

Le Bactrim® à la dose de 2 à 3 g/jour peut être proposé mais entraîne un risque d'ictère nucléaire néonatal.
La Rovamicine® est efficace à la dose de 6 cp par jour (3 grammes) et diminue le risque de contamination.
L'Adiazine® est indiquée en cas d'infection foetale prouvée associée à la pyriméthamine.

Lorsque cette jeune femme accouchera, le bilan néonatal à orientation toxoplasmique devra comporter 3 des 5
propositions suivantes : lesquelles ?
A - Sérologie mère et sang du cordon
B - Bilan biologique hépatique
C - Radiographie du crâne
D - Inoculation à la souris blanche du placenta, du caillot du sang du cordon et du L.C.R.
E - Radiographie du rachis
Bonne(s) réponse(s) : A C D

Les examens systématiques à réaliser chez le nouveau-né sont l'hémogramme (thrombopénie), le fond d'oeil (choriorétinite),
la radiographie du crâne (calcifications), l'échographie transfontanellaire (dilatation ventriculaire), la ponction lombaire (IgM), le
prélèvement sanguin chez le nouveau-né et la mère en parallèle, et dans les 5 jours après la naissance. Inoculation à la
souris de sang, placenta et LCR.

Dans le cas où son nouveau-né est cliniquement indemne, sur quel élément peut-on éliminer toute infection
congénitale ?
A - Inoculation à la souris : négative
B - Evolution normale de la courbe de poids et du périmètre crânien
C - Absence de toute lésion du F.O. à l'âge d'un an
D - Négativation des sérologies au cours de la 1ère année
E - Absence de signe clinique à la naissance
Bonne(s) réponse(s) : D

Les consultations doivent être multipliées la première année avec examen du F.O. et sérologies répétées même chez un
nouveau-né sain dont le bilan initial est négatif.

164
Exclusivement sur DOC - DZ : www.doc-dz.com NADJI 85
RESIDANAT EN POCHE TOME II
Cas Clinique en QCM

Dans le cas où son nouveau-né présente une lésion de chorio-rétinite au F.O., le traitement à base de
Rovamycine puis Rovamycine-Fansidar, devra être poursuivi pendant combien de temps après la naissance ?
A - 6 mois
B - 12 mois
C - 24 mois
D - Jusqu'à abaissement des titres sérologiques à des niveaux faibles, avec cicatrisation des lésions oculaires
E - Jusqu'à la négativation des titres sérologiques
Bonne(s) réponse(s) : D

Les sérodiagnostics resteront positifs à taux faible et stable.

Une femme enceinte de 8 mois, mal surveillée, pendant la grossesse, est hospitalisée en urgence pour un hématome
rétroplacentaire. Parmi les examens biologiques que vous avez demandés, le bilan d'hémostase est le suivant : - Temps de
Quick : 45 % - Temps de Céphaline Activé : 42 sec (35 sec) - V : 40 % - VII + X : 85 % - II : 45 % - Fibrinogène : 0,50 g/l -
Plaquettes 110 000/mm3.

Compte tenu du contexte clinique, quel est le diagnostic le plus probable de ce trouble de la coagulation ?
A - Déficit en vitamine K de la grossesse
B - Hépatite fulminante
C - Fibrinolyse aiguë
D - Thrombocytopénie par hémodilution
E - Coagulopathie de consommation
Bonne(s) réponse(s) : E

Les signes biologiques de CIDVD sont observés dans 5 à 10 % des HRP, même si les syndromes hémorragiques sont rares.
Ils associent une thrombopénie, une diminution du taux de fibrinogène, une augmentation des PDF avec présence de
complexes solubles.

Parmi les examens complémentaires suivants, le ou lesquels permettrai(en)t, le plus sûrement, de confirmer ce
diagnostic ?
A - Dosage du facteur Willebrand (VIII R :Ag)
B - Dosage du facteur antihémophilique B
C - Dosage des produits de dégradation du fibrinogène
D - Recherche de complexes solubles (test à l'éthanol)
E - Mesure du temps de saignement
Bonne(s) réponse(s) : C D

Le dosage des PDF sur sérum est long (> 1 heure) dans cette situation d'urgence et l'utilisation des Ac monoclonaux permett
leur recherche dans le plasma plus rapidement. La répétition de ces examens permet de suivre l'évolution.

L'utérus de cette femme doit être évacué, mais vous craignez une hémorragie de la délivrance. Parmi les
éléments de son bilan d'hémostase, lequel, plus que d'autres, fait craindre ce risque ?
A - Thrombocytopénie à 110 000/mm3
B - Fibrinogène : 0,50 g/l
C - L'allongement du TCA
D - Taux d'accélérine à 40 %
E - Taux de prothrombine à 45 %
Bonne(s) réponse(s) : B

La thrombopénie n'est inquiétante qu'au dessous de 50000/mm3 et doit alors faire prescrire des transfusions de concentrés
plaquettaires avant l'évacuation utérine. En revanche, l'effondrement du fibrinogène doit être compensé.

Parmi les mesures thérapeutiques suivantes destinées à éviter l'hémorragie de la délivrance, laquelle
choisissez-vous ?
A - Cinq flacons de 20 ml de PPSB
B - Perfusion intraveineuse d'un antifibrinolytique
C - Injection immédiate de 100 U/kg d'héparine standard
D - Quatre unités de plasma frais congelé plus 4 grammes de fibrinogène purifié
E - Un flacon de 250000 unités d'urokinase plus 100 U/kg d'héparine standard
Bonne(s) réponse(s) : D

La fibrinolyse est une défense contre la CIVD et la prescription d'un antifibrinolytique peut être désastreuse.
L'héparine doit être preoscrite avant l'accouchement en raison de l'augmentation du risque hémorragique. Le plasma frais
congelé est habituellement suffisant. L'urokinase n'a pas sa place en préventif.

165
Exclusivement sur DOC - DZ : www.doc-dz.com NADJI 85
RESIDANAT EN POCHE TOME II
Cas Clinique en QCM

L'évacuation utérine se déroule normalement, l'hémostase est normale et 5 000 U d'héparine s/c 2 fois/j sont
prescrites ; à J5 une phlébite fémoro-poplitée se déclare : les plaquettes sont à 450 000/mm3. Dès lors vous
choisissez de :
A - Ajouter de l'aspirine (500 mg/jour) à l'héparine sous cutanée
B - Augmenter la posologie de l'héparine sous cutanée : 5 000 unités x 3 par jour
C - Arrêter l'héparine sous cutanée et passer aux antivitamines K
D - Arrêter l'héparine sous cutanée et prescrire de l'héparine intra-veineuse à la dose de 200 unités/kg/24
heures
E - Arrêter l'héparine sous cutanée et prescrire de l'héparine intra-veineuse à la dose de 600 unités/kg/24
heures
Bonne(s) réponse(s) : E

Le traitement curatif de la thrombose constituée repose sur l'héparinothérapie intraveineuse jusqu'à des taux d'héparinémie
efficace.

Quel(s) est (sont) le(s) examen(s) le(s) plus utile(s) pour surveiller l'efficacité biologique du traitement prescrit ?
A - Mesure du temps de saignement
B - Dosage de l'antithrombine III
C - Temps de Quick
D - Temps de céphaline activé
E - Détermination de l'héparinémie circulante
Bonne(s) réponse(s) : C D E

Sans commentaire.

Une femme de 63 ans, pesant 78 kg pour 1,60m consulte pour des saignements par le vagin. On ne trouve pas d'antécédent
particulier sauf la prise d'oestrogènes depuis la ménopause intervenue à l'âge de 56 ans. L'examen gynécologique révèle un
col normal; l'utérus est de taille normale.

Quel diagnostic évoquez-vous en premier ?


A - Cancer de la vulve
B - Atrésie vaginale
C - Cancer du vagin
D - Cancer du col utérin
E - Cancer de l'endomètre
Bonne(s) réponse(s) : E

L'âge, l'obésité, et surtout la prise d'oestrogène sans compensation progestative font évoquer en premier lieu le cancer de
l'endomètre.

Quel examen demandez-vous pour confirmer ce diagnostic ?


A - Frottis cervical
B - Colposcopie
C - Curetage biopsique
D - Urographie intraveineuse
E - Scanner pelvien
Bonne(s) réponse(s) : C

Seule la confirmation anatomopathologique permet le diagnostic et est indispensable avant toute discussion thérapeutique.

Parmi les cancers suivants, un ou plusieurs est ou sont épidémiologiquement lié(s) à la prise d'oestrogènes par
la malade :
A - Cancer malpighien de la vulve
B - Cancer de l'ovaire
C - Sarcome embryonnaire du vagin
D - Cancer malpighien du col utérin
E - Adénocarcinome de l'endomètre
Bonne(s) réponse(s) : E

Seuls les adénocarcinomes du sein et de l'utérus sont oestrogéno-dépendants.

166
Exclusivement sur DOC - DZ : www.doc-dz.com NADJI 85
RESIDANAT EN POCHE TOME II
Cas Clinique en QCM

Parmi les examens suivants, lesquels sont indispensables pour préciser


l'extension loco-régionale ?
A - Lymphographie
B - Rectoscopie
C - Hystérographie
D - Echotographie pelvienne
E - Urographie intra-veineuse
Bonne(s) réponse(s) :

QUESTION ANNULEE.

Une femme de 22 ans, primipare, sans problème particulier vous consulte pour une contraception.

Vous prescrivez en première intention :


A - Une pilule séquentielle
B - Une minipilule
C - Un stérilet
D - Des préservatifs + spermicides locaux
E - Une pilule normodosée
Bonne(s) réponse(s) :

QUESTION ANNULEE.

En fait son médecin lui a posé un stérilet. Que risque t-elle ?


A - Une infection génitale
B - Une stérilité tubaire à long terme
C - Conséquences plus graves après une maladie sexuelle transmissible
D - Une phlébite
E - Une hypertension artérielle
Bonne(s) réponse(s) : A B C

Tout geste endo-utérin entraîne un risque infectieux dont les conséquences peuvent être la stérilité tubaire, ce qui fait contre-
indiquer le stérilet chez les nullipares.
En cas d'infection sexuellement transmissible, l'endométrite est plus sévère s'il existe un stérilet. Phlébite et hypertension
artérielle sont les effets indésirables des oestroprogestatifs.

Quelques mois plus tard, cette jeune femme présente des métrorragies et des douleurs abdominales. Quelles
hypothèse(s) diagnostique(s) peut-on évoquer ?
A - Un cancer du col de l'utérus
B - Une salpingite
C - Une appendicite
D - Une grossesse extra-utérine
E - Une endométrite
Bonne(s) réponse(s) : B D E

Le stérilet prévient la nidation endo-utérine mais n'empêche pas la fécondation et la nidation ectopique. Le cancer du col ne
provoque des douleurs que dans les stades avancés, ce qui est peu compatible avec un examen génital normal quelques
mois auparavant.

Quel(s) examen(s) demandez-vous en première intention ?


A - Des dosages des HCG plasmatiques
B - NFS
C - Des prélèvements bactériologiques endocervicaux
D - Une échographie
E - Une hystérographie
Bonne(s) réponse(s) : A B C D

L'hystérographie est contre-indiquée lorsque l'on suspecte une infection génitale. L'échographie permet de vérifier la présence
et la situation du stérilet et peut montrer des signes d'infection pelvienne ou de grossesse ectopique.

167
Exclusivement sur DOC - DZ : www.doc-dz.com NADJI 85
RESIDANAT EN POCHE TOME II
Cas Clinique en QCM

168
Exclusivement sur DOC - DZ : www.doc-dz.com NADJI 85
RESIDANAT EN POCHE TOME II
Cas Clinique en QCM
Monsieur G., 56 ans, vient vous consulter pour des douleurs du genou droit. L'interrogatoire vous apprend que ces douleurs
sont apparues progressivement depuis 3 ou 4 ans, survenant après 1200 à 1500 mètres de marche, bien calmées par le
repos. Elles se sont brusquement aggravées depuis 2 mois et depuis existent quelques épisodes de dérobements douloureux.
Ce malade pèse 90 kg pour 1,65 m. A l'examen, le genou est le siège d'un léger épanchement ; le quadriceps présente 2 cm
d'atrophie ; on ne note aucun signe de la série rotulienne ; la palpation de l'interligne interne est sensible ; les différents tests
méniscaux sont négatifs ; les mouvements de flexion sont douloureux en position extrême ; l'axe diaphysaire de la jambe
semble dans le prolongement de celui de la cuisse ; il existe une petite latéralité en valgus mais pas de tiroir. Enfin, on note
l'existence d'un kyste poplité. Le reste de l'examen somatique est négatif.

A quelle lésion rattacher, en 1ère analyse de ce tableau clinique, les douleurs de ce malade ?
A - Instabilité chronique du genou
B - Tumeur de la synoviale
C - Kyste poplité
D - Gonarthrose fémoro-tibiale
E - Monoarthrite rhumatoïde du genou

Quel(s) examen(s) complémentaire(s) demanderez-vous en première intention ?


A - Radiographie des genoux face en charge
B - Radiographie des genoux de profil
C - Scannographie des genoux
D - Arthrographie opaque
E - Ponction et examen du liquide synovial

Le patient revient vous voir très inquiet du résultat de l'analyse de son liquide synovial qui a mis en évidence :
hématies 80/mm3, leucocytes : 1 200/mm3, Iymphocytes 85 %. Ces résultats sont en faveur d'un ou plusieurs
des diagnostics suivants :
A - D'une arthrite tuberculeuse
B - D'une arthrite rhumatoïde
C - Une tumeur de la synoviale
D - Une arthrite virale
E - Votre premier diagnostic

Quelques jours après, le malade vous apporte les radiographies que vous avez prescrites. Votre diagnostic est
confirmé par la constatation de :
A - Pincement d'environ 50 %, localisé a l'interligne fémoro-tibiale interne
B - Densification sous-chondrale de la glène interne
C - Aspect effilé de l'épine tibiale interne
D - Deux microgéodes sous-chondrales au niveau du condyle interne
E - Les axes diaphysaires du fémur et du tibia forment un varus de 5 degrés

Quelle attitude thérapeutique proposez-vous ?


A - Abstention
B - Rééducation du quadriceps et physiothérapie
C - Arthroplastie du genou
D - Synovectomie du genou + ablation du kyste poplité
E - Ostéotomie tibiale de valgisation

Un homme de 40 ans a été victime d'une luxation traumatique antéro-interne de l'épaule, il y a cinq jours. Cette luxation n'a
pas été réduite, mais a fait l'objet d'une tentative de réduction sans anesthésie par un rebouteux La radiographie montre la
luxation et une fracture du trochiter associée Le patient présente une hypoesthésie de la face externe du moignon de l'épaule
à sa jonction avec le niveau du quart supérieur du bras. Un nouvel essai de réduction sous anesthésie générale s'avère
infructueux et prouve l'irréductibilité de la luxation.

Le blessé se présente à l'inspection :


A - En abduction réductible
B - En abduction irréductible
C - En rotation interne irréductible
D - En rotation interne réductible
E - En adduction

169
Exclusivement sur DOC - DZ : www.doc-dz.com NADJI 85
RESIDANAT EN POCHE TOME II
Cas Clinique en QCM

La fracture associée du trochiter :


A - Peut être l'équivalent d'une rupture de la coiffe des rotateurs
B - Se traduit constamment et automatiquement par mise en abduction du bras
C - Justifie toujours une intervention chirurgicale
D - Peut laisser comme séquelle un conflit entre l'acromion et le trochiter
E - Traduit un arrachement du long biceps

Le tableau neurologique fait soupçonner :


A - Une lésion du plexus-brachial
B - Une lésion du nerf sus-scapulaire
C - Une lésion du circonflexe
D - Une lésion du musculo-cutané
E - Une lésion du brachial cutané interne

Le traitement conseillé sera :


A - Rééducation en l'état
B - Une réduction sanglante
C - Une traction progressive au lit, le bras au zénith
D - Une traction progressive sur appareil thoraco-brachial
E - Une résection simple de la tête humérale

Monsieur X. 25 ans, consulte pour des douleurs lombaires apparues 1 mois auparavant, sans fièvre, sans altération de l'état
général.

Parmi les caractéristiques suivantes laquelle (lesquelles) est (sont) en faveur du diagnostic de spondylarthrite
ankylosante ?
A - Impulsivité des douleurs a la toux
B - Evolution par crises, déclenchées par des efforts ou faux mouvements
C - Plusieurs réveils chaque nuit, du fait des douleurs en dehors de tout changement de position
D - Prédominance vespérale des douleurs
E - Absence de soulagement par les anti inflammatoires non stéroïdiens

Parmi les antécédents suivants, quel(s) est (sont) celui (ceux) qui est (sont) en faveur du diagnostic de
spondylarthrite ankylosante ?
A - Urticaire allergique
B - Uvéite
C - Angines à répétition
D - Talalgies bilatérales
E - Céphalées fréquentes

Parmi les propositions d'examens radiographiques suivantes, quelles sont les deux qui vous paraissent les plus
utiles pour confirmer le diagnostic de spondylarthrite ankylosante ?
A - Radiographie du bassin de face
B - Radiographie du rachis dorso-lombaire de face
C - Radiographie dorso-lombo-pelvi-fémorale de face (De Seze)
D - Radiographie du rachis dorso-lombaire de profil
E - Radiographie du rachis lombaire de 3/4 droit et gauche

Sur quel segment rachidien avez-vous le plus de chances d'observer les premiers signes radiologiques de la
spondylarthrite ankylosante ?
A - Rachis cervical
B - Charnière cervico-dorsale
C - Rachis dorsal
D - Charnière dorso-lombaire
E - Rachis lombaire

170
Exclusivement sur DOC - DZ : www.doc-dz.com NADJI 85
RESIDANAT EN POCHE TOME II
Cas Clinique en QCM

Parmi les traitements suivants, quel est celui qui est habituellement le plus efficace dans la spondylarthrite
ankylosante ?
A - Corticothérapie par voie générale
B - Antimalariques de synthèse
C - Anti inflammatoires non stéroïdiens
D - Sels d'or
E - D-Pénicillamine

Après un accident de voiture important, un homme de 40 ans est transporté à l'hôpital en état de choc avec une déformation
importante du membre inférieur au niveau de la hanche. L'abdomen est douloureux, ainsi que la région fessière. Des
radiographies de face et de profil révèlent une luxation postérieure de la hanche et une fracture de l'os coxal.

L'examen clinique doit rechercher avant tout une complication fréquente. Laquelle ?
A - Une paralysie sciatique
B - Une paralysie crurale
C - Une thrombose fémorale
D - Une ouverture cutanée postérieure
E - Un hématome de la fesse

Les radiographies réalisées :


A - Sont suffisantes
B - Doivent être complétées par un faux profil de Léquesne
C - Doivent être complétées par une incidence en 3/4 alaire du côté blessé
D - Doivent être complétées par un incidence en 3/4 obturateur
E - Doivent être complétées par des tomographies

Une rupture sous-péritonéale de vessie se diagnostiquerait chez ce malade par :


A - Une émission spontanée d'urine sanglante
B - Un empâtement sus-pubien avec miction spontanée d'urine et de sang
C - De l'urine et du sang au sondage
D - Une contracture abdominale avec état de choc
E - L'association de toutes les propositions ci-dessus

Le premier geste thérapeutique sur les lésions osseuse est :


A - La réduction de la luxation
B - L'ostéosynthèse du cotyle
C - La mise en extension continue
D - La mise en place d'une prothèse de hanche
E - Immobilisation plâtrée

La(les) complication(s) la(les) plus fréquente(s) à long terme est(sont) :


A - La nécrose de la tête fémorale
B - La suppuration profonde
C - Une ankylose spontanée de la hanche
D - Une fracture du col du fémur
E - Une coxarthrose

Un homme de 65 ans entre à l'hôpital pour une tuméfaction douloureuse du tiers supérieur de la jambe Les radiographies de
la jambe montrent au niveau du tibia, un épaississement de la corticale avec aspect feuilleté des tables externes et internes.
L'opacité se poursuit au niveau des parties molles.

Le diagnostic de l'affection en cours évoque :


A - Une dysplasie fibreuse des os
B - Un cancer de la prostate avec métastases osseuses
C - Une maladie de Hand Schuller Christian
D - Une ostéomalacie
E - Un sarcome ostéogénique

171
Exclusivement sur DOC - DZ : www.doc-dz.com NADJI 85
RESIDANAT EN POCHE TOME II
Cas Clinique en QCM

S'il existe une affection osseuse chronique sous-jacente, la plus probable est :
A - Granulome éosinophile
B - Dysplasie fibreuse
C - Syphilis tertiaire
D - Maladie de Paget
E - Kyste solitaire de l'os

Un de ces examens ne doit pas être pratiqué par principe. Lequel ?


A - Biopsie osseuse transcutanée
B - BW et Nelson
C - Radiographie pulmonaire
D - Bilan radiologique du squelette
E'- Scintigraphie osseuse au pyrophosphate de TC 99m

La thérapeutique la plus appropriée sera :


A - Chimiothérapie-radiothérapie
B - Radiothérapie-amputation de la jambe
C - Chimiothérapie-amputation de la jambe-chimiothérapie
D - Traitement par la vitamine D
E - Oestrogénothérapie

Un adulte de 30 ans a été victime d'une fracture ouverte de type II du quart inférieur des deux os de la jambe, traitée
orthopédiquement. Il a consolide en cal vicieux, comportant 20 degrés de varus, 25 degrés de flexum et 2 cm
raccourcissement. L'ouverture cutanée interne a laissé une cicatrice adhérente de médiocre qualité. L'état vasculo-nerveux du
pied est satisfaisant ; le sujet ne présente pas de tare viscérale particulière.

L'élément le plus péjoratif sur le plan fonctionnel est :


A - Le raccourcissement
B - Le varus
C - La cicatrice cutanée adhérente
D - Le flessum
E - Le siège au 1/4 inférieur de la jambe

Indiquez l'articulation au niveau de laquelle se fera le retentissement fonctionnel le plus important ?


A - Hanche
B - Genou
C - Tibio-tarsienne homolatérale
D - Rachis lombaire
E - Tibio-tarsienne du côté opposé

Pour juger au mieux de l'importance de la déformation, le meilleur cliché radiographique sera :


A - Un cliché de face
B - Un cliché de profil
C - Un cliché dont le rayon incident est perpendiculaire au plan dans lequel la jambe apparaît non déformée
D - Des tomographies face et profil
E - La stéréoradiographie

Vous conseillez au patient la conduite à tenir suivante :


A - Abstention
B - Ostéotomie d'ouverture transversale sus-malléolaire
C - Ostéotomie de fermeture antéro-externe par voie externe
D - Port d'une chaussure orthopédique
E - Arthrodèse de cheville

172
Exclusivement sur DOC - DZ : www.doc-dz.com NADJI 85
RESIDANAT EN POCHE TOME II
Cas Clinique en QCM
Un patient de 47 ans, est hospitalisé pour explorer une vitesse de sédimentation augmentée à 107 mm, de découverte
systématique. Le patient signale des paresthésies distales des 2 mains. L'examen clinique retrouve un syndrome du canal
carpien et une hypotension orthostatique. Hémogramme : 4 000 000 GR, hématocrite 38 %, hémoglobine 12 g/100 ml,
leucocytes 4 000, formule normale, 300 000 plaquettes/mm3. Myélogramme riche avec 25 % de cellules à noyaux excentrés,
à chromatine dense et cytoplasme basophile. Protéines : 89 g/l dont albumine : 40 g/l. alpha-1 : 2 g/l, alpha-2 : 9 g/l,
bêtaglobulines 6 g/l, gammaglobulines : 32 g/l, bande croisée au niveau des gamma-globulines, protéinurie 3 g/24 h.

Un ou plusieurs des examens suivants est (sont) indispensable(s) pour confirmer le diagnostic. Lequel(lesquels)
?
A - Immuno-électrophorèse des protéines sériques
B - Créatinine plasmatique
C - Uricémie
D - Fond d'oeil
E - Test de Coombs direct

Parmi les examens suivants, indiquez celui (ceux) qui est (sont) utile(s) pour établir un pronostic ?
A - Clichés de squelette
B - Calcémie
C - Lymphographie
D - Créatinine plasmatique
E - Fond d'oeil

Une des complications suivantes peut être suspectée d'emblée chez ce patient :
A - Sténose de ia carotide
B - Compression médulaire cervicale
C - Amylose
D - Hyperviscosité plasmatique
E - Leucémie aiguë

L'immuno-électrophorèse des protéines urinaires peut montrer un ou plusieurs des aspects suivants chez un tel
patient :
A - 100 % de chaînes légères kappa
B - 100 % de chaînes lourdes alpha
C - 50 % d'albumine et 50 % de chaînes légères kappa
D - 100 % d'IgG lambda
E - 100 % d'IgM kappa

L'apparition d'une lombalgie aiguë fait proposer les explorations suivantes : quelle(s) est(sont) celle(s) qui
peu(ven)t être proposée(s) sans risque de déclencher une complication liée a la maladie ?
A - Scanner avec injection
B - Scanner sans injection
C - Urographie intraveineuse
D - Radiographie du rachis
E - Artériographie mésentérique

173
Exclusivement sur DOC - DZ : www.doc-dz.com NADJI 85
RESIDANAT EN POCHE TOME II
Cas Clinique en QCM
Mr A., âgé de 51 ans (en 1984), a eu une hématurie macroscopique a l'âge de 31 ans (en 1964). A cette occasion l'urographie
intra-veineuse a découvert une polykystose rénale. On avait conseille a Mr A., en 1964, une surveillance régulière de son état
rénal et de sa pression artérielle mais il n'a pas suivi ces conseils. Depuis quelques semaines, il se sent fatigué et nauséeux.
La pression artérielle est de 170-110 mm Hg. Il n'a pas d'oedème ni de pli cutané. On palpe deux gros reins bosselés Les
examens complémentaires montrent les résultats suivants, dans le sang veineux :
- urée = 33 mmol/l
- K = 4,7 mmol/l
- Cl = 108 mmol/l
- Ca = 1,60 mmol/l
- protides = 70 g/l
- créatinine = 650 micromol/l
- Na = 138 mmol/l
- Co2 total = 21 mmol/l
- phosphore = 2,6 mmol/l
- hémoglobine = 13 g/100 ml

Chez ce malade un ou plusieurs mécanismes contribuent à l'hypocalcémie. Lequel(lesquels) ?


A - Un défaut de synthèse du 1-25 (OH2 D3)
B - Un défaut d'absorption digestive du calcium
C - Une hypercalciurie.
D - L'hyperphosphorémie.
E - L'hyperchlorémie.

Les anomalies phospho-calciques, si elles ne sont pas corrigées, risquent de conduire à deux complications
osseuses principales :
A - Ostéomalacie
B - Ostéoporose
C - Ostéosclérose
D - Ostéité fibreuse due à l'hyperparathyroïdie
E - Maladie de Paget de l'os

Le traitement au long cours à mettre en oeuvre chez ce malade pour corriger


les anomalies phospho-calciques comporte :
A - Apport per os de carbonate de calcium
B - Apport intraveineux de gluconate de calcium
C - Apport per os de phosphate d'alumine
D - Apport per os d'hydroxyde d'alumine
E - Apport per os de métabolites de la vitamine D

L'absence d'anémie est expliquée par :


A - L'insuffisance rénale chronique
B - L'acidose chronique
C - L'hémoconcentration
D - La polykystose
E - L'hypertension artérielle

Un homme de 64 ans est hospitalisé pour des douleurs abdominales, s'accompagnant d'une polyurie et d'une polydipsie. Son
interrogatoire est impossible car il est très obnubile, mais par la famille on apprend que le patient était fatigué depuis plusieurs
semaines et se plaignait de rachialgies diffuses ; dans les antécédents, on retrouve une insuffisance coronarienne avec
épisodes paroxystiques de tachyarythmie, une cataracte et un ulcère gastrique pour l'instant asymptomatique. A l'arrivée en
service de chirurgie, on constate que l'abdomen est souple, mais douloureux. Le pli cutané est marque, la langue est sèche.
Les réflexes osteo-tendineux sont très faibles. Les examens biologiques donnent les résultats suivants : ionogramme sanguin
natrémie = 148 meq, chlorémie = 105 l. Bilan phospho-calcique, calcémie = 4,2 mmoles, phosphorémie = 2,6 mmoles,
phosphatases alcalines = 182 U (N170). Le malade est transféré en Unité de Soins Intensifs.

Parmi les signes cliniques suivants, le(s)quel(s) n'(ne)est(sont) pas lié(s) à l'hypercalcémie :
A - Fasciculations musculaires
B - Douleurs abdominales
C - Polyurie
D - Rachialgies
E - Obnubilation

174
Exclusivement sur DOC - DZ : www.doc-dz.com NADJI 85
RESIDANAT EN POCHE TOME II
Cas Clinique en QCM

En fonction du tableau clinique et biologique, l'hypercalcémie peut être rapportée à :


A - Métastases osseuses
B - Hyperparathyroïdie
C - Intoxication à la vitamine D
D - Hypothyroïdie
E - Syndrome du lait et des poudres alcalines

La découverte d'une vitesse de sédimentation élevée conduit à évoquer la possibilité d'un myélome. Dans la
situation d'urgence présente, ce diagnostic peut être affirmé par le résultat de :
A - Radiographies du squelette complet
B - Electrophorèse du sérum
C - Myélogramme
D - Immuno-électrophorèse du sérum
E - Scintigraphie osseuse

En service de Soins Intensifs, quel traitement mettez-vous en oeuvre, en plus du rétablissement de l'équilibre
hydro-électrolytique ?
A - Calcitonine
B - Mithramycine
C - Diurèse forcée au Lasilix®
D - Dialyse péritonéale
E - Prednisone à la dose de 1 mg/kg

Chez ce malade, il est contre-indiqué d'utiliser :


A - Cédilanide
B - Aminodarone
C - Quinidine
D - Nifédipine
E - Dopamine

Le diagnostic de myélome est confirmé. A, la suite du traitement mis en oeuvre, la calcémie atteint 2,7
micromol/l. Le patient est transféré en médecine, quel traitement préconisez-vous alors pour ramener la
calcémie à la normale ?
A - Mithramycine
B - Association melphalan-prednisone
C - Phosphate de sodium par voie orale
D - Diphosphonates
E - Indométacine

A la suite d'un accident de voiture, un jeune militaire sans antécédent particulier, est transporté à l'hôpital. Le bilan révèle un
traumatisme crânien avec perte de conscience brève et une fracture fermée transversale de la diaphyse fémorale droite.
L'examen clinique est normal ainsi que les radiographies du crâne. Dans l'attente d'une ostéosynthèse, le membre inférieur
droit est placé sous une traction de 8 kg. Le lendemain, il est très agité et présente un subictère, une polypnée à 35/mn, un
pouls à 140 batt/mn et une hyperthermie à 39°. Biologiquement, sont notés une bilirubinémie totale à 56 micromoles/l, directe
à 8 micromoles/l et à l'hémogramme : érythrocytes = 3,4 T/l, hémoglobine = 102 g/l, hémacrotite = 0,29 G, leucocytes = 9,8
G/l, thrombocytes = 90 G/l. Le bilan d'hémostase révèle un taux de prothrombine à 65 %, un fibrinogène à 2,6 g/l et un test à
l'éthanol positif. Le diagnostic retenu est celui d'embolie graisseuse et le tableau clinique régresse en quelques jours sous
traitement.

Les signes cliniques suivants sont constants lors d'une embolie graisseuse, sauf un. Lequel ?
A - Hyperthermie
B - Tachycardie
C - Agitation
D - Tachypnée
E - Frottement pleural

175
Exclusivement sur DOC - DZ : www.doc-dz.com NADJI 85
RESIDANAT EN POCHE TOME II
Cas Clinique en QCM

Le rash pétéchial de l'embolie graisseuse a une(des) localisation(s) spécifique(s), la(lesquelles) ?


A - Deux membres supérieurs
B - Face interne des paupières
C - Plancher buccal
D - En regard de la fracture
E - Creux axillaires

Quelle est l'image radiologique caractéristique observée sur ia radiographie thoracique parmi ces cinq
propositions ?
A - Syndrome interstitiel bilatéral
B - Turgescence hilaire bilatérale
C - Opacités floues, confluentes et diffuses
D - Syndrome de comblement alvéolaire des sommets
E - Discrète hypervascularisation pulmonaire

Que déduisez-vous du bilan biologique ?


A - Hémodilution
B - Anémie hémolytique
C - Aplasie médullaire partielle
D - Insuffisance hépatique
E - Coagulopathie de consommation

La confirmation du diagnostic peut être obtenue par :


A - Electroencéphalogramme
B - Fond d'oeil
C - Bilan sanguin lipidique
D - Scintigraphie osseuse
E - Myélogramme

Les traitements suivants sont recommandés sauf un. Lequel ?


A - Héparinothérapie sous-cutanée
B - Ostéosynthèse fémorale
C - Tranquillisants
D - Transfusion de concentrés plaquettaires
E - Oxygénothérapie nasale

Un homme de 70 ans se plaint depuis un an de céphalées persistantes et d'une surdité progressive. Il souffre également
depuis 15 jours de lombalgies initialement intenses, puis lentement régressives, diurnes, augmentées à la marche, soulagées
par le décubitus. A l'examen, outre l'hypoacousie, on note un rachis lombaire enraidi, peu douloureux, la jambe gauche est
discrètement incurvée avec genu varum et augmentation de la température cutanée en regard. Au toucher rectal, la prostate
est globalement augmentée de volume, régulière avec conservation du sillon médian Les examens biologiques montrent une
NFS, une VS. une électrophorèse des protides sanguins normales; calcémie à2,5 mmoles/l;
phosphorémie : 1.20mmoles/l, calciurie : 3,5 mmoles/24 h,
phosphatases acides totales : 5. 5 Ul/l (normale 3 à 8 )
phosphatases acides prostatiques : 0.2 Ul/l (normale 2)
phosphatases alcalines : 940 Ul/l (normale 30 à 90)
hydroxyprolinurie 3870 micromoles/24 h (normale 115 à 270)
Les radiographies objectivent des discopathies lombaires étagées avec un baillement postérieur L4-L5, une ostéophytose
lombaire, une hypertrophie avec élargissement des corticales du fémur droit, et du tibia gauche, une fissure de la convexité du
fémur droit. un aspect fibrillaire anarchique de l'aile iliaque droite, une condensation avec hypertrophie des os de la base du
crâne, une ostéoporose circonscrite fronto-pariétale droite. Le diagnostic de maladie osseuse de Paget est posé.

En rapport avec la maladie de Paget chez ce malade, vous retenez :


A - Céphalées persistantes
B - Hypoacousie
C - Lombalgies
D - Genu varum
E - Augmentation de la température cutanée de jambe

176
Exclusivement sur DOC - DZ : www.doc-dz.com NADJI 85
RESIDANAT EN POCHE TOME II
Cas Clinique en QCM

Parmi les examens biologiques suivants, vous retenez en faveur du caractère évolutif de la maladie de Paget ?
A - Calcémie : 2,5 mmoles/l
B - Phosphorémie : 1,2 mmol/l
C - Calciurie : 3,5 mmoles/24 h
D - Phosphatases alcalines : 940 Ul/l
E - Hydroxyprolinurie : 3870 micromoles/24 h

Parmi les signes radiologiques suivants, vous retenez en faveur de la maladie de Paget ?
A - Discopathies lombaires étagées
B - Ostéophytose lombaire
C - Hypertrophie des corticales
D - Aspect fibrillaire anarchique
E - Ostéoporose circonscrite du crâne

Une biopsie osseuse de l'aile iliaque droite est pratiquée : l'analyse histologique peut montrer en faveur de la
maladie de Paget :
A - Infiltrat Iympho-plasmocytaire
B - Hyperactivité des ostéoclastes
C - Hyperactivité des ostéoblastes
D - Diminution de la vascularisation
E - Absence de tissu ostéoïde

Quel(s) médicament(s) anti-ostéoclastique(s) pouvez-vous proposer chez ce malade ?


A - Calcium
B - Phosphore
C - Calcitonine
D - Vitamine D
E - Diphosphonates

Un jeune garçon de 13 ans, sans antécédents particuliers présente des douleurs du genou gauche, de survenue assez
brutale, sans origine traumatique. Ce garçon mesure 165 cm et pèse 68 kg. Son membre inférieur a une attitude en discret
flexum de hanche et rotation externe de tout le membre inférieur. Son genou est sec et stable, sans point douloureux précis à
la palpation, et sa mobilité est 0-15°. La mobilité des deux hanches est la suivante (en degrés) :

DROITE GAUCHE
- Flexion 110 80
- Extension 0 0
- Abduction 40 20
- Adduction 20 20
- Rotation externe 50 70
- Rotation interne 40 10

Il existe une esquive du pas du côté gauche avec attaque en adduction et rotation interne. Ce garçon a un examen
neurologique normal, il n'a pas de fièvre et aucun traitement n'est en cours.

Quel diagnostic pouvez-vous évoquer cliniquement :


A - Une arthrite septique du genou
B - Une sciatique
C - Une épiphysiolyse de hanche
D - Une luxation traumatique de rotule spontanément réduite
E - Une ostéomyélite de l'extrémité inférieure du fémur

Un certain nombre d'examens est indispensable, le(s)quel(s) à votre avis :


A - Radiographie du bassin de face et en Lauenstein
B - Radiographie du genou face-profil et fémoro-patellaire
C - Radiographie du rachis face et profil de la charnière lombo-sacrée
D - Numération globulaire formule et vitesse de sédimentation
E - Electromyographie des rotateurs externes de hanche

177
Exclusivement sur DOC - DZ : www.doc-dz.com NADJI 85
RESIDANAT EN POCHE TOME II
Cas Clinique en QCM

Devant un tel tableau, l'attitude thérapeutique à adopter en urgence comporte :


A - Décubitus strict sur un plan dur
B - Traction sur les membres inférieurs
C - Ponction articulaire du genou
D - Chirurgie de l'épiphysiolyse
E - Traitement antibiotique avec immobilisation plâtrée

Dans le cas ci-dessus, quelle est l'évolution spontanée à court terme en l'absence de traitement :
A - Paralysie radiculaire flasque
B - Raideur et trouble de croissance du genou
C - Bilatéralisation de l'épiphysiolyse
D - Ostéïte aiguë
E - Septicémie

Indiquez la complication à craindre à long terme chez ce malade après traitement :


A - Arthrose lombo-sacrée
B - Nécrose de la tête fémorale
C - Ostéité chronique
D - Ankylose du genou
E - Luxation pathologique de la hanche

Un homme de 24 ans souffre depuis 3 semaines d'une douleur du genou droit d'abord rapportée à un traumatisme indirect au
cours d'un match de volley ball. Malgré le repos et un traitement antiinflammatoire non stéroïdien, la douleur s'aggrave, gêne
le sommeil, le genou est chaud, augmenté de volume par un épanchement et a un flexum de 15°. La température qui n'avait
jamais été prise est à 37 °C au moment de l'examen. Il a sur la peau des lésions cutanées évocatrices de gonococcie
disséminée. On soupçonne chez cet homme jeune une arthrite septique et plus spécialement une arthrite gonoccique bien
qu'aucune notion d'urétrite ne soit retrouvée à l'interrogatoire Des rapports homosexuels ne sont pas exclus.

Parmi les lésions cutanées suivantes, quelle est celle qui a fait évoquer le diagnostic de gonococcie disséminée :
A - Kératodermie palmoplantaire
B - Macules roses pâles du décolleté
C - Papulopustules sur les deux cuisses
D - Lésions érythématosquameuses des coudes
E - Nodules dermohypodermiques de la face antérieure des jambes

En dehors de la ponction articulaire, quel(s) prélèvement((s) devraien(t) être fait(s) immédiatement pour isoler le
gonocoque ?
A - Hémoculture
B - Prélèvement urétral
C - Prélèvement des lésions cutanées
D - Prélèvement rectal
E - Prélèvement oropharyngé

Parmi les propositions suivantes concernant le gonocoque, laquelle(lesquelles) est(sont) exacte(s) ?


A - C'est un germe fragile nécessitant sa mise en culture immédiate
B - Ce germe pousse facilement sur les milieux usuels
C - Le germe est habituellement résistant aux bêta lactamines
D - C'est un diplocoque gram négatif
E - Des réactions sérologiques sont nécessaires pour son identification

Le genou gauche a été ponctionné. Quel(s) résultat(s) vous paraîtrai(en)t incompatible(s) avec le diagnostic
d'arthrite gonococcique ?
A - 500 éléments/mm3 dont 70 % de Iymphocytes
B - 7 500 éléments/mm3 dont 90 % de polynucléaires neutrophiles
C - 15300 éléments/mm3 dont 70 % de polynucléaires neutrophiles
D - 6000 éléments/mm3 dont 70 % de Iymphocytes
E - 2000 éléments/mm3 dont 60 % de polynucléaires eosinophiles

178
Exclusivement sur DOC - DZ : www.doc-dz.com NADJI 85
RESIDANAT EN POCHE TOME II
Cas Clinique en QCM

Les prélèvements bactériologiques faits, il faut 48 heures pour obtenir les résultats. Dans cette attente, votre
attitude thérapeutique sera :
A - Pénicilline G : 10 millions d'Unités par jour
B - Expectative et surveillance
C - Traitement antalgique
D - Indométacine 100 mg par jour
E - Indométacine 100 mg par jour, pénicilline G 10 millions d'unités par jour

Sur quel(s) élément(s) parmi les suivants, surveilleriez-vous l'efficacité du traitement ?


A - Courbe thermique
B - Taux des anticorps antigonococciques
C - Vitesse de sédimentation
D - Pouvoir bactéricide du sérum
E - Dosage hebdomadaire des IgA sériques

Parmi les propositions suivantes, quelle(s) est(sont) celle(s) qui s'applique(nt) a ce malade ?
A - L'arthrite gonococcique confère une immunité durable
B - Le traitement prescrit doit guérir une syphilis primaire associée
C - La maladie est à déclaration obligatoire non nominale
D - Une abstinence sexuelle de 30 jours est conseillée après guérison
E - Il faut rechercher des maladies sexuellement transmissibles chez les partenaires

Un malade de 45 ans, ouvrier imprimeur, souffre depuis 3 mois, après un effort de soulèvement, d'une douleur lombaire
irradiant à la fesse gauche, à la face postérieure de la cuisse et du mollet gauche pour gagner le bord externe du pied gauche.
Cette douleur, d'horaire mécanique calmée par le repos, est impulsive à la toux et s'accompagne de paresthésies plantaires.
Antécédents : lombalgies chroniques depuis 10 ans avec épisodes de lumbagos, intervention pour hernie inguinale gauche il
y a 4 ans.

Parmi les signes cliniques suivants, indiquez celui(ceux) en rapport avec cette atteinte radiculaire :
A - Parésie du moyen fessier
B - Impossibilité de marcher sur la pointe du pied
C - Diminution du réflexe rotulien
D - Hypoesthésie de la face dorsale du pied et du gros orteil
E - Abolition du réflexe crémastérien

Parmi les signes radiologiques suivants, indiquez celui(ceux) qui serai(en)t compatible(s) avec atteinte
radiculaire dont souffre le malade :
A - Ostéophytose antérieure somatique de L4 et L5
B - Séquelles d'épiphysite vertébrale dorsale
C - Hypertransparence osseuse sans tassement vertébral
D - Pincement discal avec baillement électif gauche du disque L5-S1
E - Arthrose inter-apophysaire postérieure en L4-L5

Malgré un traitement médical approprié, la douleur sciatique persiste La pratique d'urgence d'une
radiculographie sera commandée par la constatation clinique récente de :
A - Intolérance gastrique aux AINS (Anti-inflammatoires non Stéroïdiens)
B - Sensation de refroidissement du pied gauche
C - Persistance des lombalgies
D - Nécessité de pousser pour uriner
E - Hypoesthésie périnéale

Une radiculographie est faite, le liquide céphalo-rachidien a été analysé à cette occasion : indiquez la(les)
formule(s) qui est(sont) compatible(s) avec une origine discale de cette douleur :
A - Protéinorachie à 0,30 g/l, moins de 1 élément par mm3
B - Protéinorachie 0,30 g/l, 300 éléments par mm3
C - Hypergammaglobulinorachie
D - Abaissement des sucres et des chlorures
E - Protéinorachie à 0,90 g/l, moins de 1 élément par mm3

179
Exclusivement sur DOC - DZ : www.doc-dz.com NADJI 85
RESIDANAT EN POCHE TOME II
Cas Clinique en QCM

Après la réalisation de la radiculographie, les signes suivants persistent ; intolérance aux AINS; sensation de
refroidissement du pied gauche ; persistance des lombalgies; le malade est obligé de pousser pour uriner ;
hypoesthésie périnéale. Indiquez quelle serait votre attitude thérapeutique :
A - Infiltration intra-rachidienne de corticoïdes
B - Tractions vertébrales
C - Pose d'une ceinture plâtrée sous surveillance médicale
D - Intervention chirurgicale rapide
E - Corticothérapie générale à haute dose pendant 5 jours

Une femme de 75 ans a des douleurs osseuses diffuses depuis plusieurs mois. Ces douleurs siègent aux hanches et irradient
dans les cuisses ; il existe également des dorsalgies, des lombalgies et une faiblesse musculaire. La malade vit dans un
hospice, elle n'a pas d'autre antécédent qu'une diarrhée chronique . A l'examen, l'état général est bon. Il existe une douleur à
la palpation des pièces osseuses superficielles; sans tumeur décelable. L'examen articulaire est normal ainsi que le reste de
l'examen somatique. Les radiographies montrent une déminéralisation squelettique diffuse avec tassements de D7, D9, et
D11. La VS est à 1Omm a la 1ère heure, la NFS révèle seulement une anémie normochrome normocytaire arégénérative à 10
g d'hémoglobine/100 ml. Le bilan phosphocalcique est le suivant :
- Calcémie : 2,03 mmol/l (81 mg/l)
- Calciurie : 0,5 mmol/24 h (20 mg/24 h)
- Phosphorémie : 0,61 mmol/24 h (21 mg/l)
- Phosphatases alcalines : 360 UI (N 200)
- Hydroxyprolinurie : 160 micromol/24 h (N 270).
Pour confirmer le diagnostic, on réalise une ponction biopsie de la crête iliaque et on complète le bilan radiographique et
biologique.

Parmi les éléments suivants de l'observation, le(s)quel(s) oriente(nt) vers le diagnostic d'ostéomalacie ?
A - Dorsalgie
B - Faiblesse musculaire
C - Tassements vertébraux
D - Calciurie à 0,5 mmol/24 h
E - Hydroxyprolinurie à 160 micromol/24 h

Parmi les éléments suivants le(s)quel(s) peu(ven)t vous apporter une certitude diagnostique chez cette malade :
A - Phosphatases alcalines à 360 UI
B - Découverte d'une strie de Looser Milkmann sur les radios
C - Diminution du volume osseux calcifié à la ponction biopsie osseuse
D - Hyperostéoïdose avec disparition du front de calcification à la ponction biopsie osseuse
E - Calcémie à 2,03 mmol/l, phosphorémie à 0,61 mmol/l

Parmi les résultats d'examens biologiques suivants, le(s)quel(s) est (sont) compatible(s) avec l'observation de
cette malade ?
A - Effondrement du taux de PTH
B - Augmentation du taux plasmatique de 1 - 25 (OH)2 D3
C - Diminution du taux plasmatique de 25 (OH) D3
D - Augmentation de la clairance du phosphore
E - Diminution du taux de l'AMPc néphrogénique

Chez cette malade, quelle(s) peu(ven)t être la (les) cause(s) de l'ostéomalacie ?


A - Diabète phosphate en rapport avec une tumeur mésenchymateuse
B - Diabète phosphate en rapport avec un myélome
C - Ostéomalacie par carence d'apport en vitamine D
D - Syndrome de malabsorption
E - Hyperthyroïdie

Chez cette malade, la cause de cette ostéomalacie peut être précisée par le(s) résultat(s) suivant(s) :
A - Anémie normochrome hormocytaire
B - Abaissement de l'AMP cyclique urinaire
C - Elévation de la clairance de l'anti-trypsine
D - Découverte d'une stéatorrhée supérieure à 5 g/24 h
E - Diminution de l'élimination urinaire du D-xylose

180
Exclusivement sur DOC - DZ : www.doc-dz.com NADJI 85
RESIDANAT EN POCHE TOME II
Cas Clinique en QCM

Quel médicament allez-vous prescrire à cette malade pour la guérir de son ostéomalacie ?
A - Vitamine D2
B - Anabolisants de synthèse
C - Diphosphonates
D - Calcitonine
E - Fluorure de sodium

Une jeune femme de 26 ans consulte pour l'apparition rapide sur trois jours, sans traumatisme récent, de douleurs et
d'impotence du genou droit. L'examen montre que ce genou est chaud et contient un épanchement intra-articulaire tendu
assez abondant. La mobilisation est très douloureuse ; on ne peut obtenir l'extension complète, la flexion atteint à peine 60
degrés. La température rectale est a 38°2. La malade est hospitalisée. Au 3ème jour d'hospitalisation, l'état clinique est
toujours aussi sévère. L'interrogatoire n'a fourni aucune piste étiologique. L'examen du liquide synovial a donné les résultats
suivants : protéines = 40g/l, éléments figurés = 38000/mm3, formule : polynucléaires 92 %, cellules mononuclées 8 %. Waaler-
Rose et latex négatifs, pas de germes à l'examen direct, culture sur milieux ordinaires stérile, pas de cristaux, soit intra-, soit
extra-cellulaires.

Parmi les hypothèses diagnostiques suivantes, désignez celle(s) qui, étant la(les) plus plausible(s), doi(ven)t
être examinée(s) en premier :
A - Rhumatisme inflammatoire à début mono-articulaire aigu
B - Crise de goutte du genou
C - Arthrite infectieuse
D - Poussée inflammatoire sur arthrose
E - Bursite pré-rotulienne

Parmi les gestes cliniques utiles pour parvenir au diagnostic étiologique, choisissez dans la liste suivante les
deux plus utiles au diagnostic :
A - Interrogatoire approfondi sur les antécédents pathologiques récents
B - Interrogatoire approfondi sur les antécédents pathologiques lointains
C - Examen attentif de la totalité des téguments
D - Examen attentif du coeur
E - Examen attentif des aires ganglionnaires

Quel examen radiographique souhaitez-vous faire faire en priorité ?


A - Genou droit : face. profil, interligne fémoro-patellaire en incidence axiale à 30°, 45° et 60° de flexion
B - Les deux genoux, face et profil
C - Le bassin de face
D - Le thorax, face etprofil
E - Toutes ces radiographies sont nécessaires d'emblée

Parmi les examens sanguins énumérés ci-dessous, quel est chez cette malade, celui qui vous parait le plus
urgent :
A - Vitesse de sédimentation
B - Uricémie
C - Electrophorèse des protides sériques
D - Réaction de Waaler-Rose et du Latex
E - Recherche des anticorps antinucléaires

Vous ponctionnez l'épanchement pour étudier le liquide articulaire. Dans la liste suivante d'examens, quels sont
les deux que vous considérez en première analyse comme les plus rentables pour le diagnostic :
A - Dosage des protéines
B - Numération et formule des éléments figurés
C - Recherche de cristaux
D - Recherche du facteur rhumatoïde
E - Examen bactériologique

181
Exclusivement sur DOC - DZ : www.doc-dz.com NADJI 85
RESIDANAT EN POCHE TOME II
Cas Clinique en QCM

Vous poussez l'interrogatoire sur les antécédents pathologiques de la malade et de son entourage. Parmi les
données suivantes, quelle est celle qui pèserait le plus sur votre orientation diagnostique :
A - Une angine récente chez la malade
B - Une bronchite récente chez la malade
C - Une diarrhée récente chez la malade
D - Une furonculose chez le mari de la malade
E - Une urétrite récente chez le mari de la malade

Quelle attitude choisissez-vous à titre provisoire au soir du 3ème jour d'hospitalisation ?


A - Traitement anti inflammatoire non stéroïdien par voie intramusculaire
B - Traitement antibiotique à spectre large par voie orale
C - Traitement par la pénicilline G par voie veineuse
D - Injection intra-articulaire de corticoïdes en suspension microcristalline
E - Synoviorthèse à l'acide osmique

Un couvreur de 35 ans est tombé de 6 mètres de haut sur ses pieds. Il souffre de son arrière-pied gauche mais peut marcher
à petits pas. Lorsque vous le voyez le lendemain, tout l'arrière-pied est tuméfié. Au palper, son talon vous semble élargi et la
palpation en est sensible. La mobilisation de la tibio-tarsienne est possible mais les amplitudes extrêmes sont douloureuses.
La recherche des mouvements d'inversion et d'éversion du pied est très douloureuse.

Quel diagnostic évoquer devant ce tableau clinique ?


A - Fracture du pilon tibial
B - Luxation sous-astragalienne du pied
C - Fracture du calcanéum
D - Entorse grave tibio-tarsienne
E - Rupture du tendon d'Achille

Quel(s) cliché(s) radiologique(s) de débrouillage demandez-vous en premier ?


A - Tibio-tarsienne de face
B - Tibio-tarsienne de face en inversion
C - Tibio-tarsienne de face en éversion
D - Tibio-tarsienne et pied de profil
E - Incidence rétro-tibiale

En ce qui concerne le calcanéum, que pouvez-vous analyser sur le cliché de profil, si vous soupçonnez une
fracture ?
A - La grande apophyse
B - La petite apophys
C - Le thalamus
D - La tubérosité postérieure
E - La corticale inférieure

A propos de l'angle de Bohler, il est exact que :


A - Sa valeur normale est de 40 degrès
B - Cette valeur diminue dans les enfoncements thalamiques
C - Cette valeur ne diminue que dans les enfoncements thalamiques horizontaux
D - Sa valeur diminue dans les fractures en bec de canard de la tubérosité
E - Sa valeur augmente dans les enfoncements thalamiques verticaux

La radiographie de profil de notre accidenté met en évidence : un angle de Bohler à 0 degrè, un éclat est
soulevé à la faveur d'un trait de fracture sous-thalamique allant du sinus au tarse à la tubérosité postérieure du
calcénéum. La corticale inférieure de l'os n'est pas fracturée. Vous pouvez en déduire une ou plusieurs des
propositions suivantes :
A - Fracture isolée de la tubérosité postérieure
B - Fracture thalamique avec enfoncement vertical
C - Fracture à deux fragments
D - Fracture à trois fragments
E - La petite apophyse est fracturée

182
Exclusivement sur DOC - DZ : www.doc-dz.com NADJI 85
RESIDANAT EN POCHE TOME II
Cas Clinique en QCM

Les traits de fracture, séparation pré ou transthalamiques, ne sont bien visibles que sur certains clichés.
Le(s)quel(s) ?
A - Le cliché de face de l'arrière-pied
B - Le cliché en incidence plantaire rétro-tibiale
C - Le cliché de profil
D - Les tomographies frontales du calcanéum
E - Les tomographies sagittales du calcanéum

Les fractures du calcanéum exposent à une ou plusieurs des complications suivantes. Laquelle(lesquelles) ?
A - Une pseudarthrose
B - Un cal vicieux
C - Une nécrose du calcanéum
D - Un enraidissement douloureux de la sous astragalienne
E - Une tendinite des péroniers

Une femme de 35 ans consulte pour une synovite inflammatoire des 2 poignets et des 2 genoux d'apparition récente. Il n'y a
aucun antécédent particulier, la malade ne signale ni angine ni trouble intestinal. L'examen clinique met en évidence une
raideur des deux poignets, un léger épanchement des genoux avec épaississement de la synoviale. On ne constate aucune
éruption cutanée. Les radiographies des articulations inflammatoires révèlent seulement un gonflement des parties molles aux
poignets. Les examens biologiques montrent :
- V.S. = 35 - 78
- NFS : 4 350 000 GR, 2 800 GB, Hby 12,3 g, 63 % PN - 1 % E - 0 % b - 22 % L - 14 % M
- Test au latex : 1/160ème ; Waaler Rose : 1/128 ; B.W.positif
- complément total : 32 u. (N 50 ± 10)
- examen cytobactériologique des urines : négatif.

Votre diagnostic s'oriente vers :


A - Une polyarthrite rhumatoïde
B - Une syphilis
C - Un lupus
D - Une maladie périodique
E - Aucune de ces affections

Vous complétez le bilan par :


A - Un test de Nelson
B - Un test de Farr
C - Une biopsie cutanée
D - Une ponction-biopsie rénale
E - Une radiographie de la charnière cervico-occipitale

Le liquide de ponction peut vous montrer :


A - 500 éléments par mm3
B - 4500 éléments par mm3
C - Des cristaux de pyrophosphate de calcium
D - Un liquide hémorragique
E - Un liquide très visqueux

Votre préférence ira au traitement suivant :


A - Sels d'Or
B - Pénicilline
C - D-Penicillamine
D - Antimalariques de synthèse
E - Immunodépresseurs

Vous rechercherez une(des) complication(s) éventuelle(s) :


A - Luxation de la charnière cervico-occipitale
B - Troubles psychiques
C - Atteinte rénale
D - Glaucome
E - Nodules sur la face postérieure des coudes

183
Exclusivement sur DOC - DZ : www.doc-dz.com NADJI 85
RESIDANAT EN POCHE TOME II
Cas Clinique en QCM

La malade désire une grossesse et vous demande conseil :


A - Vous pensez qu'elle risque d'aggraver la maladie
B - Vous estimez qu'elle sera bénéfique sur les poussées inflammatoires articulaires
C - Il y a un risque non négligeable d'avortement spontané
D - Vous conseillez d'attendre une stabilisation de la maladie pendant au moins un an
E - Aucun conseil particulier n'est à donner à cette malade

Un ouvrier du bâtiment, à la suite d'une chute d'un lieu élevé, est tombé sur une verrière. Outre un traumatisme crânien, il
présente une plaie à la face externe du genou.

La lésion du nerf sciatique poplité externe est suspectée devant :


A - Une paralysie du triceps sural
B - Une paralysie du jambier antérieur
C - Une paralysie des extenseurs des orteils
D - Une anesthésie du dos du pied
E - Une anesthésie de la plante du pied

La section du nerf sciatique poplité externe est accompagnée de lésions des parties molles. Indiquez sur quel(s)
élément(s) vous faites le diagnostic d'ouverture articulaire :
A - L'existence d'une large plaie cutanée
B - Une image hydroaérique intra-articulaire à la radiographie standard
C - La ponction du genou
D - Une fracture des épines tibiales
E - L'exploration chirurgicale

Ce blessé va être opéré en urgence : indiquez l'(es) examen(s) paraclinique(s) que vous demandez :
A - Groupe sanguin
B - Arthrographie du genou
C - Echographie du creux poplité
D - Electromyogramme
E - Hémoculture

En cas de section complète franche, sans perte de substance du nerf sciatique poplité externe. Indiquez la
conduite à envisager :
A - Une greffe nerveuse sous microscope
B - Une suture directe sous microscope
C - Une suture cutanée avec greffe nerveuse secondaire
D - Une transplantation tendineuse
E - Une arthrodèse de la cheville

Une jeune femme de 27 ans est enceinte de 7 mois. Depuis 5 mois, elle se plaint de violentes douleurs que l'analyse clinique
permet d'identifier comme d'origine sciatique. Ces douleurs l'obligent à s'aliter en permanence. L'examen montre une abolition
du réflexe achiléen gauche et 2 nodules mammaires. La gravité est normale. Les deux radiographies (face + profil) de la
colonne dorso-lombaire montrent un tassement de L5. Les radiographies du crâne et des mains sont normales, la
radiographie pulmonaire également. La calcémie est à 3,25 mmol/l.

Parmi les symptômes et signes suivants, l'hypercalcémie peut être à l'origine de :


A - Abolition d'un réflexe osteotendineux
B - Polyurie
C - Confusion mentale
D - Raccourcissement de l'espace QT sur l'électrocardiogramme
E - Douleur névralgique

Quelle(s) étiologie(s) possible(s) de l'hypercalcémie retenez-vous chez cette malade ?


A - Etat gravidique
B - Métastase osseuse d'un néoplasme
C - Immobilisation prolongée
D - Acromégalie
E - Sarcoïdose

184
Exclusivement sur DOC - DZ : www.doc-dz.com NADJI 85
RESIDANAT EN POCHE TOME II
Cas Clinique en QCM

Quelle(s) mesure(s) thérapeutique(s) peu(ven)t être envisagée(s) pour traiter l'hypercalcémie ? (sans tenir
compte de la grossesse)
A - Diurèse forcée induite par le furosémide + chlorure de sodium
B - Diurèse forcée induite par les thiazidiques + chlorure de sodium
C - Administration de thyrocalcitonine
D - Corticothérapie. prednisone 100 mg/kg/jour
E - Résine échangeuse d'ions type Kayexalate®

Comment s'expliquent les hypercalcémies associées à des cancers ?


A - Synthèse d'un facteur activant l'ostéoclastose
B - Production de protides inhibant la parathormone
C - Destruction osseuse par les métastases
D - Production de prostaglandines
E - Synthèse de vitamine D (1-25 OH D3)

Un sportif de 25 ans a fait une chute sur le moignon de l'épaule il y a six mois. Le patient a été traité par un bandage pendant
8 jours. Il présente des douleurs de l'épaule survenant essentiellement lorsqu'il fait de la gymnastique (barres parallèles et
vélo). Ses douleurs sont localisées a la partie supérieure du moignon de l'épaule. Il n'y a pas de douleur au repos et pas de
sensation de ressaut, ni d'instabilité. L'examen met en évidence : à l'nspection une saillie osseuse en haut et en arrière au
niveau de la partie supérieure du moignon de l'épaule - une mobilité active et passive de l'épaule normale - une douleur
provoquée au cours de l'abduction de l'épaule à partir de 90 degrés jusqu'au zénith - la palpation montre une saillie osseuse
située en dedans de l'extrémité distale de l'acromion. Cette saillie est mobile de haut en bas et d'avant en arrière.

Les douleurs alléguées évoquent :


A - Un syndrome inflammatoire
B - Une lésion tumorale
C - Une lésion post-traumatique avec conséquences mécaniques
D - Une lésion dégénérative
E - Une lésion neurologique

La douleur survenant au-delà de 90 degrés d'abduction et jusqu'au zénith évoque :


A - Une lésion intéressant la clavicule
B - Un syndrome du bourrelet glénoïdien
C - Une tendinite du sus-épineux
D - Un syndrome des arcs douloureux par conflit acromio-huméral
E - Une lésion de l'articulation scapulo-thoracique

La position conseillée pour mettre en évidence la lésion au cours de la radiographie de face sera :
A - En abduction au-delà de 90 degrés
B - En rotation externe
C - En rotation interne
D - En rétropulsion
E - Avec un poids de quelques kilogrammes dans la main

Le traitement proposé est :


A - Une immobilisation en bandage de Velpeau pendant 3 semaines
B - Une rééducation fonctionnelle
C - Une infiltration locale de corticoïdes
D - Des anti-inflammatoires non stéroïdiens per os
E - Une intervention chirurgicale

185
Exclusivement sur DOC - DZ : www.doc-dz.com NADJI 85
RESIDANAT EN POCHE TOME II
Cas Clinique en QCM
Une femme de 30 ans présente depuis 6 mois des douleurs articulaires bilatérales au niveau des poignets, des
métacarpophalangiennes, des genoux et des chevilles. Les douleurs sont permanentes avec une raideur matinale d'environ 1
heure. Elle est asthénique, a perdu quelques kilos et présente une température à 38°C. L'examen montre une tuméfaction
chaude des jointures atteintes et un petit souffle systolique de pointe. Elle aurait eu un rhumatisme articulaire aigu dans
l'enfance Elle a accouché 6 mois plus tôt d'un enfant normal et au moment de sa grossesse, on aurait trouvé une sérologie
syphilitique positive.

Ces données vous paraissent suffisantes pour éliminer a priori :


A - Une polyarthrose
B - Une pseudopolyarthrite rhizomélique
C - Une goutte polyarticulaire
D - Une polyarhrite
E - Une polymyosite

La topographie des atteintes articulaires est à priori évocatrice :


A - D'un syndrome de Reiter
B - D'une spondylarthrite ankylosante
C - D'un rhumatisme psoriasique
D - D'une polyarthrite rhumatoïde
E - D'une tuberculose articulaire

Quel(s) examen(s) biologique(s) demandez-vous chez cette malade dans le seul but de confirmer l'existence
d'un syndrome inflammatoire ?
A - Vitesse de sédimentation
B - Numération formule-plaquettes
C - Uricémie
D - Anticorps anti-nucléaires
E - Albuminémie

Parmi les examens suivants, quel est celui qui vous paraît le plus à même de confirmer le syndrome
inflammatoire si les examens de sang laissaient un doute ?
A - Scintigraphie osseuse
B - Radiographies des jointures atteintes
C - Antigène HLA B 27
D - Numération des éléments du liquide articulaire
E - Thermographie

Quel(s) examen(s) biologique(s) demandez-vous pour préciser l'étiologie de ce tableau de polyarthrite ?


A - Latex et Waaler Rose
B - Myélogramme
C - Anticorps anti-nucléaires
D - Phosphatases alcalines
E - Albuminurie

Le sexe, l'âge, la survenue de la maladie après une grossesse évoquent plus particulièrement :
A - Polyarthrite rhumatoïde
B - Lupus érythémateux disséminé
C - Spondylarthrite ankylosante
D - Maladie de Horton
E - Chondrocalcinose

Vous voyez la malade en consultation et vous ne disposez encore d'aucun examen complémentaire Vous
prescrivez :
A - Sels d'or
B - Corticoïdes per os
C - Antimalariques de synthèse
D - Colchicine
E - Anti inflammatoires non stéroïdiens

186
Exclusivement sur DOC - DZ : www.doc-dz.com NADJI 85
RESIDANAT EN POCHE TOME II
Cas Clinique en QCM
Un homme de 34 ans. conducteur d'engins, est hospitalisé à la demande de son médecin traitant, pour sciatique droite, en
vue d'une probable intervention chirurgicale. Dans les antécédents de ce patient, on trouve plusieurs épisodes lombalgiques
généralement provoqués par des efforts. Il n'y a jamais eu de douleur des membres inférieurs avant la troisième semaine
précédant l'hospitalisation. Est alors apparue une douleur du membre inférieur droit, intéressant la face postérieure de la
cuisse, la face externe de la jambe, le dos du pied et le premier orteil, ainsi qu'une douleur du creux inguinal. L'examen
clinique montre un signe de Lasegue droit à 60°. une raideur lombaire avec distance doigts-sol à 50 cm dans l'antéflexion
maximale ; l'examen neurologique est normal. La mobilité des hanches est également normale. Le patient apporte des
radiographies pratiquées un mois et demi plus tôt à loccasion de la lombalgie, qui montrent un spina bifida de S1 et une
hémisacralasisation de L5. Ce malade, qui mesure 178 cm et pèse 86 kg a une vitesse de sédimentation normale, un
hémogramme normal, une uricémie à 80 mg/l, (480 mmol).

En faveur d'un conflit discoradiculaire, vous retenez dans ce cas :


A - Les antécédents lombalgiques
B - Le spina bifida
C - L'examen neurologique normal
D - La douleur inguinale
E - Le signe de Lasègue

Il importe de demander :
A - De nouveaux clichés lombaires de face et de profil
B - Une tomodensitométrie des derniers disques
C - Une radiculographie
D - Une scintigraphie
E - Aucun des examens précédents

La douleur décrite par le patient évoque :


A - Un conflit discoradiculaire L3-L4
B - Un conflit discoradiculaire L4-L5
C - Un conflit discoradiculaire L5 S1
D - Une sténose du canal lombaire
E - Aucune de ces lésions

L'état de ce patient justifie particulièrement :


A - La mise au repos
B - L'usage des antiinflammatoires non stéroïdiens
C - La pratique d'une injection péridurale de corticoïde
D - Une kinésithérapie active
E - Une exploration chirurgicale

L'hyperuricémie justifie :
A - L'usage de la colchicine
B - L'usage de l'allopurinol (Zyloric®)
C - Un régime hypocalorique
D - Un régime pauvre en alcool
E - Une urographie intraveineuse

Une femme de 30 ans se plaint depuis 6 mois d'une douleur mécanique siégeant au pli inguinal droit et à la face antérieure de
la cuisse droite. Dans les antécédents, on retient la notion d'une immobilisation plâtrée du bassin et du membre inférieur droit
pendant un an, à l'âge de 2 ans. Au terme de la consultation, on retient le diagnostic de coxarthrose.

Une origine dysplasique à cette coxarthrose est rapidement envisagée. Quel(s) élément(s) plaide(nt) en faveur
de cette hypothèse ?
A - La douleur survenue électivement à la marche
B - Le siège inguino-crural de la douleur
C - L'âge de la malade
D - L'antécédent d'immobilisation plâtrée dans l'enfance
E - Le sexe

187
Exclusivement sur DOC - DZ : www.doc-dz.com NADJI 85
RESIDANAT EN POCHE TOME II
Cas Clinique en QCM

Des radiographies sont demandées pour affirmer le diagnostic de coxarthrose sur dysplasie de hanche. Quels
sont les 2 clichés suffisants à ce diagnostic ?
A - Un cliché de profil de la hanche droite
B - Un cliché de face de la hanche droite
C - Un cliché de bassin, debout, de face, les membres inférieurs en rotation interne de 20°
D - Un cliché de bassin en oblique à 25° (faux-profil)
E - Des tomographies de la hanche droite

Les radiographies montrent les signes d'une coxarthrose débutante. Quel est, à votre avis, le signe compatible
avec ce diagnostic ?
A - Perte de l'arrondi de la tête fémorale
B - Condensation hétérogène de la tête fémorale
C - Flou du contour de la tête fémorale
D - Géode du col fémoral
E - Pincement de 1 mm de l'interligne dans le quadrant supéro-externe

Une coxométrie est réalisée. Elle montre les résultats suivants : angle cervico-diaphysaire CCD 130° - angle de
couverture extême de la tête VCE 15° - angle de couverture antérieure de la tête VCA 25° - angle d'obliquité
HTE 23°. A votre avis, cette dysplasie correspond à :
A - Une coxa valga
B - Une couverture externe insuffisante de la tête
C - Une couverture antérieure de la tête insuffisante
D - Une pente exagérée du toit du cotyle
E - Une protrusion acétabulaire

Le diagnostic de coxarthrose secondaire débutante est affirmé. Quelle est l'attitude thérapeutique qui seule peut
prévenir l'aggravation de cette coxarthrose ?
A - L'emploi de la balnéothérapie et de la bicyclène
B - Une rééducation élective des adducteurs et du moyen fessier
C - L'emploi d'anti inflammatoires non stéroïdiens
D - L'emploi d'une canne pour favoriser la décharge de la hanche douloureuse
E - L'intervention chirurgicale rapide

Une vieille dame de 85 ans, qui conservait une activité de ménagère dans sa maison et faisait seule quelques sorties dans
son village, fait une chute de sa hauteur et ne peut plus se relever. Elle présente une douleur aiguë dans l'aine droite Elle est
transportée par des voisins jusqu'à sa maison toute proche où on l'allonge sur un lit. Vous êtes appelé en urgence et
constatez une tumeur molle douloureuse dans l'aine et une attitude en rotation externe du membre inférieur droit. L'essai de
mobilisation entraîne de vives douleurs.

A quel diagnostic pensez-vous le plus volontiers, compte-tenu de l'âge et des signes cliniques ?
A - Luxation traumatique de la hanche
B - Fracture du cotyle
C - Fracture du cadre obturateur
D - Fracture du col du fémur
E - Rupture musculaire du droit antérieur

L'attitude du membre inférieur atteint de fracture transcervicale du fémur type 3 de Garden comporte :
A - Rotation interne
B - Allongement
C - Raccourcissement
D - Rotation externe
E - Adduction

Le risque le plus important encouru par un vieillard atteint de fracture du fémur est-il ?
A - La raideur de la hanche
B - La pseudarthrose
C - Un risque vital
D - La nécrose post traumatique de la tête fémorale
E - Le cal vicieux

188
Exclusivement sur DOC - DZ : www.doc-dz.com NADJI 85
RESIDANAT EN POCHE TOME II
Cas Clinique en QCM

Le traitement de principe d'une fracture du col du fémur à 85 ans chez une femme encore valide est-il ?
A - L'abstention thérapeutique
B - L'immobilisation simple au lit
C - Le traitement chirurgical
D - L'immobilisation en plâtre pelvipédieux
E - La rééducation précoce avec kinésithérapie

Le traitement le plus adapté à une fracture transcervicale du fémur du vieillard est-il ?


A - La suspension traction par broche transosseuse
B - La synthèse de la fracture
C - La mise en place d'une prothèse cervico-céphalique
D - La mise en place d'une cupule sur la tête du fémur
E - L'enclouage élastique de Ender à travers la diaphyse fémorale

Si cette femme avant sa fracture avait déjà perdu la fonction de la marche depuis longtemps et devait avoir
recours à une tierce personne pour aller du lit au fauteuil, quel traitement aurait été logique ?
A - Synthèse chirurgicale
B - Prothèse de hanche
C - Suspension traction au lit
D - Mise au fauteuil (Lucas Championnière)
E - Résection simple de la tête fémorale

Une femme de 60 ans présente des paresthésies et des douleurs intéressant les trois premiers doigts de la main droite. Ces
manifestations surviennent parfois dans la journée mais surtout dans la deuxième partie de la nuit. La symptomatologie est
réveillée lors de l'examen par la flexion du poignet et la pression exercée à la face palmaire du poignet. L'examen
neurologique ne fait pas apparaître de modifications des réflexes ostéo-tendineux des membres supérieurs non plus que de
déficit sensitif distal. L'éventualité d'un syndrome du canal est évoquée.

Parmi les muscles suivants quel(s) est (sont) celui (ceux) dont le déficit peut être compatible avec un tel
diagnostic ?
A - Court abducteur du pouce
B - Long abducteur du pouce
C - Adducteur du pouce
D - Opposant du pouce
E - Court extenseur du pouce

Parmi les circonstances étiologiques suivantes, ce syndrome du canal carpien peut être la conséquence
de :
A - Amylose
B - Hypothyroïdie
C - Diabète
D - Polyarthrite rhumatoïde
E - Fracture de l'extrémité inférieure du radius

Une infiltration est envisagée chez cette patiente. Parmi les éléments suivants quel(s) est (sont) celui (ceux) qui
doit (doivent) faire renoncer à ce geste ?
A - Polyarthrite rhumatoïde
B - Déficit moteur important
C - Echec de deux infiltrations antérieures récentes
D - Ténosynovite tuberculeuse des fléchisseurs
E - Bilatéralité de la symptomatologie

Avec laquelle(esquelles) des affirmations suivantes concernant le traitement infiltratif êtes-vous d'accord ?
A - L'infiltration peut constituer un test thérapeutique
B - L'injection s'effectue dans le canal de Guyon
C - Le produit utilisé est un corticoïde
D - L'infiltration peut être répétée régulièrement même en cas de rechutes fréquentes
E - Un délai de 6 mois minimum est nécessaire entre deux infiltrations

189
Exclusivement sur DOC - DZ : www.doc-dz.com NADJI 85
RESIDANAT EN POCHE TOME II
Cas Clinique en QCM

Parmi les éléments suivants, quel(s) est(sont) celui(ceux) susceptible(s) de constituer une complication du
traitement infiltratif du syndrome du canal carpien ?
A - Douleur immédiate par injection défectueuse dans la gaine du médian
B - Rupture d'un tendon fléchisseur
C - Déclenchement d'une crise de goutte
D - Atrophie cutanée en regard de la zone d'infiltration
E - Infection locale

Monsieur R. André, 47 ans. est attaché commercial dans une entreprise de chaussures caractérisée par ses prix élevés et ses
méventes récentes. Ses soucis professionnels bien surmontés jusqu'alors ont entraîné des gastralgies d'intensité croissante
qu'il a pris l'habitude de calmer par des prises de lait alternées avec une poudre de carbonate de calcium dont il prend 2 gr à
la fois jusqu'à six fois par jour. Il consulte car il a présenté coup sur coup plusieurs fols par semaine depuis trois semaines,
des crises douloureuses violentes à type de colique néphrétique droite ou gauche. La calcémie est variable entre 2,5 et 2,8
mmol. La phosphorémie est normale. La calciurie varie entre 12 et 15 mmol/24 h. Une radiographie montre de multiples
calcifications dans la zone corticomédullaire des deux reins.

Sans tenir compte des données cliniques et biologiques. le tableau radiologique est compatible avec le(s)
diagnostic(s) suivant(s) :
A - Hypoparathyroïdie
B - Hypercalciurie par excès d'apport calcique
C - Néphrocalcinose par maladie kystique de la médullaire (maladie de Cacchi-Ricci)
D - Hypervitaminose D
E - Malabsorption intestinale

Parmi les hypothèses suivantes, laquelle vous parait la plus recevable en tenant compte de l'ensemble clinique,
radiologique et biologique ?
A - Néphrocalcinose par tubulopathie
B - Maladie polykystique héréditaire
C - Hypercorticisme
D - Iléite terminale de type maladie de Crohn
E - Syndrome de Burnett par prise de lait alcalin

D'autres causes peuvent théoriquement entraîner une néphrocalcinose. Citez la(les) hypothèse(s) possible(s)
devant la seule image radiologique :
A - Sarcoïdose (maladie de BBS)
B - Hyperthyroïdie
C - Insuffisance surrénale lente
D - Prise excessive de chelateurs de phosphore
E - Tubulopathie de type Toni Debré Fanconi

Malgré les précautions et les conseils donnés, le sujet élimine à plusieurs reprises de petits calculs de
phosphate de calcium et un calcul du bassinet droit est constaté. Quel(s) est(sont) le(s) facteur(s) favorisant ce
type de lithiase ?
A - Urines alcalines
B - Boissons abondantes
C - Soucis professionnels
D - Consommation excessive de légumes (oseille,.épinard,.artichaut)
E - Infection urinaire à proteus mirabilis

Parmi les mesures thérapeutiques envisagées chez ce sujet en particulier, laquelle(lesquelles) vous parai(ssen)t
adéquate(s) ?
A - Changement radical du traitement des gastralgies
B - Abord chirurgical pour réduction des parathyroïdes de 7/8ème (on laisse une 1/2 parathyroïde sur 4)
C - Prise d'inhibiteur de synthèse de l'acide urique
D - Prescription prolongée de Kayexalate®
E - Traitement de l'infection urinaire à proteus

190
Exclusivement sur DOC - DZ : www.doc-dz.com NADJI 85
RESIDANAT EN POCHE TOME II
Cas Clinique en QCM
M. B.... 48 ans, salarie peintre en bâtiment, souffre, depuis 3 ans d'épigastralgies survenant par poussées. La seule
fibroscopie pratiquée, qui date de 2 ans, a montre un ulcère duodénal bulbaire ; le traitement est suivi très irrégulièrement ;
des douleurs sont survenues récemment. Par ailleurs, depuis 4 ans, il existe des rachialgies rapportées à ses activités
professionnelles, lesquelles sollicitent fortement les segments rachidiens mobiles. A début lombaire, ces douleurs, qui
surviennent par poussées de quelques semaines, ont ensuite concerné également l'étage dorsal et la colonne cervicale. M.
B... signale aussi des douleurs thoraciques latérales et antérieures et a vu enfin apparaître, depuis 1 mois, des douleurs
inguinales bilatérales. Toutes ces algies du tronc ont une composante nocturne prolongée, en fin de nuit, et entraînent un
enraidissement matinal de plusieurs heures. L'examen clinique réalisé l'après-midi montre un test de Schober à 10+1 cm, une
distance occiput-mur de 10 cm, une ampliation thoracique de 3,5 cm ; les hanches sont douloureuses à la mobilisation ; la
flexion, l'abduction et les rotations sont modérément limitées. Les radiographies montrent des coxofémorales normales, des
anomalies des 2 sacro-iliaques avec élargissement et irrégularité de l'interligne flou et condensation de ses berges. Il existe
des ossifications sous-ligamentaires unissant les corps vertébraux de D11 à L4.VS : 48 mm à la 1ère heure.

Pour affirmer, chez ce malade, le diagnostic de spondylarthrite ankylosante, aquelle (lesquelles) des 5
propositions suivantes est (sont) exacte(s) ?
A - Un groupage tissulaire à la recherche du B27 est indispensable
B - Une scintigraphie osseuse est nécessaire
C - La mise en évidence dans le sérum d'anticorps antinucléaires et d'une hypocomplémentémie sera décisive
D - Une ponction biopsie de sacro-iliaque s'impose
E - Les données rapportées dans l'observation suffisent

Voici 5 propositions concernant le test de SCHOBER; laquelle (lesquelles) est (sont) exacte(s) ?
A - Il explore la flexion du rachis lombaire et de la charnière lombosacrée
B - A l'etat normal, la longueur du trait passe de 10 à 16 cm au minimum
C - Il est toujours pathologique au cours de la spondylarthrite ankylosante
D - Il est toujours normal dans les lombalgies communes, d'origine discale
E - Sa valeur dans la spondylarthrite ankylosante, peut varier au cours de la journée

Des 5 propositions suivantes concernant ce cas, laquelle (lesquelles) est (sont) exacte(s) ?
A - Une distance occiput-mur de 10 cm n'est pas pathologique chez l'adulte
B - Une ampliation thoracique de 3.5 cm est pathologique et traduit une atteinte des articulations
costovertébrales
C - La limitation du jeu des hanches ne peut pas être expliquée par la spondylarthrite ankylosante
D - Les images radiologiques vues sur la charnière dorsolombaire et le rachis lombaire correspondent à des
syndesmophytes
E - Une nouvelle fibroscopie gastro-duodénale est justifiée avant un éventuel traitement anti-inflammatoire

Pour le traitement de ce rhumatisme inflammatoire, voici 5 propositions. Laquelle (lesquelles) est (sont)
exacte(s) ?
A - Une déclaration du diagnostic à la Sécurité Sociale permettra une prise en charge à 100 %
B - Le risque de complication de l'ulcère duodénal contre-indique définitivement les AINS
C - Le paracétamol peut être utilise sans restriction
D - Un arrêt de travail sera justifié lors de poussées sévères
E - Une kinésithérapie est devenue inutile

Chez un homme de 38 ans, on découvre une uricémie à 85 mg/l ( méthode a l'uricase). Il n'a jamais fait de crises goutteuses,
son poids est de 83 kg, sa taille de 168 cm, ses chiffres tensionnels de 170/100 mm Hg ; le reste de l'examen clinique est
sans particularités.
Il n'a pas de protéinurie ; l'examen du fond d'oeil est normal ; l'hémogramme est sans particularité. La créatinine plasmatique
est à 10 micromol/l. Il n'y a pas de lithiase urique ; l'uraturie est demandée. Ce sujet à une alimentation hyperglucidique, riche
en fruits (fructose), l'apport alcoolique est également important.

Parmi les affections suivantes, laquelle(lesquelles) vous parai(ssen)t susceptible(s) d'exprimer l'hyperuricémie ?
A - Déficit en hypoxanthine guanine phosphoribosyltransgérase
B - Glycogénose de type I
C - Syndrome myéloprolifératif
D - Néphroangiosclérose
E - Hyperuricémie idiopathique (goutte commune)

191
Exclusivement sur DOC - DZ : www.doc-dz.com NADJI 85
RESIDANAT EN POCHE TOME II
Cas Clinique en QCM

Vous conclurez que le sujet est hypo-uraturique si son uraturie est :


A - Comprise entre 10 et 50 mg/24 h
B - Comprise entre 50 et 200 mg/24 h
C - Comprise entre 300 et 600 mg/24 h
D - Comprise entre 800 et 2000 mg/24 h
E - Comprise entre 3000 et 6000 mg/24 h

L'enquête diététique est en faveur d'une alimentation apportant quotidiennement 3200 Kcal dont 63 % d'hydrate
de carbone, 8 % de protides. 130 g d'alcool. Il boit quotidiennement 2,5 litres d'eau. Quel(s) facteur(s)
alimentaire(s) vous parai(ssen)t susceptible(s) d'avoir favorisé l'hyperuricémie
A - Alimentation hypercalorique
B - Alimentation hyperglucidique
C - Alimentation hyperprotidique
D - Excès d'apport en alcool
E - Insuffisance d'apport hydrique

Quelle(s) mesure(s) thérapeutique(s) vous parai(ssen)t actuellement justifiée(s) ?


A - Régime normocalorique équilibré
B - Evictions des aliments purinophores
C - Majoration de l'activité physique
D - Prescription d'un diurétique (Thiazidique)
E - Prescription de benziodarone (Amplivix®)

Un homme de 60 ans, mesurant 1 m 70 et pesant 68 kg, souffrait depuis 10 ans d'accès de goutte. Depuis quelques mois,
sont apparues des tuméfactions blanchâtres siégeant sur la pulpe des doigts. Depuis trois semaines. il est traité par 300 mg
de Zyloric®. Avant la mise au traitement, l'uricémie était de 600 micromol/l(101 mg) et l'uraturie de 24 h à 60 mmol (1010mg),
cholestérol à 6 mmol/l, triglycérides à 2 mmol, uraturie : 3 mmol/24 h, créatinine : 78 micromol/l. Un contrôle de l'uricémie et de
l'uraturie à la 3ème semaine de traitement par 300 mg de Zyloric donne les résultats suivants : uricémie = 320 micromol,
uraturie = 3 mmol/24 h, créatinine = 78 micromol/l. Le soir même de ce dosage, le malade souffre brutalement d'une douleur
du genou droit et du poignet gauche. Les deux articulations sont gonflées, chaudes, le poignet est rouge et le genou distendu
par un épanchement intra-articulaire. Le diagnostic d'accès de goutte bi-articulaire est rapidement affirmé.

Quel examen a permis d'affirmer le diagnostic d'accès de goutte dans ce contexte ?


A - Radiographie des mains et poignets de face
B - Etude du liquide synovial obtenu par ponction du genou
C - Biopsie de synoviale
D - Radiographie du genou face et profil
E - Dosage de la xanthinémie

L'accès de goutte bi-articulaire dans ce contexte est dû à :


A - Une intolérance médicamenteuse à l'allopurinol
B - Une posologie insuffisante d'allopurinol
C - L'augmentation de la xanthinémie sous allopurinol
D - L'absence de prescription de colchicine ou d'un antinflammatoire non stéroïdien
E - Une mauvaise observance du malade au traitement

Parmi les traitements suivants, quel est celui qui vous parait le plus approprie à cette situation :
A - Arrêt définitif de l'allopurinol, colchicine 3 mg. par jour pendant 3 jours, puis 1 mg chaque soir en
permanence
B - Colchicine : 3 mg. par jour pendant 2 jours, 2 mg pendant 2 jours puis 1 mg, par jour Allopurinol 400
mg/jour en traitement de fond
C - Un antiinflammatoire non stéroïdien pendant 5 jours et benzéromarone (Désuric®) 100 mg. par jour en
traitement de fond
D - Cortancyl ® (Deltacortisone) 15 m par jour pendant 5 jours et poursuite de l'allopurinol 300 mg. par jour
en traitement de fond
E - Un antinflammatoire non stéroïdien pendant 5 jours et allopurinol 300 mg. et colchicine 1 mg par jour en
traitement de fond

192
Exclusivement sur DOC - DZ : www.doc-dz.com NADJI 85
RESIDANAT EN POCHE TOME II
Cas Clinique en QCM

Quelle est la nature vraisemblable des tuméfactions blanchâtres de la pulpe des doigts observées chez ce
malade ?
A - Concrétion calcaire d'un syndrome de Thibierge Weissenbach
B - Nodule rhumatoïde
C - Nodule d'une vascularite cutanée induite par l'allopurinol
D - Concrétion d'urate de sodium
E - Dépôt de xanthine

La goutte, dont souffre ce patient, peut-elle être considérée, en fonction des éléments fournis, comme :
A - Une goutte secondaire à une insuffisance rénale
B - Une goutte secondaire à une hémopathie
C - Une goutte saturnine
D - Une goutte par déficit en hypoxanthine guanine phosphoribosyltransférase
E - Une goutte primitive (hyperformatrice)

L'uricémie après 3 semaines de traitement par 300 mg d'allopurinol est passée de 600 micromol/l à 320
micromol/l. Cet effet hypouricémiant est dû :
A - A une inhibition de la réabsorption tubulaire de l'acide urique
B - A une diminution de l'absorption intestinale des purines
C - A une diminution de la dégradation des purines
D - A une inhibition de la xanthine oxydase et à une diminution de la purinosynthèse de novo
E - A la transformation de l'acide urique en allantoïne par stimulation de l'uricase

Un homme de 30 ans alcoolo-tabagique chronique souffre depuis 2 ans de sciatalgies bilatérales à bascule, de cervicalgies,
de talalgies, d'intensité maximale le matin au réveil, évoluant par poussées et depuis 15 jours d'une arthrite du genou droit.
L'interrogatoire retrouve des antécédents d'oligoarthrite fixe pendant deux mois à l'âge de 25 ans, de diarrhée chronique avec
plusieurs épisodes de rectorragie, d'uvéite antérieure récidivante. L'examen clinique révèle une raideur lombaire globale, une
douleur à la mobilisation des sacro-iliaques, un herpès labial, un souffle diastolique 2/6 parasternal gauche et une
hépatomégalie ferme. La température est à 37°, lexamen ophtalmologique révèle une névrite optique. Les radiographies
objectivent des érosions des articulations sacro-iliaques, une érosion des angles anterosupérieurs des 1ères et 2èmes
vertèbres lombaires, une discopathie L5 -S1 avec ostéophytose, une coxa plana et un aspect hérissé des calcanéums. Les
examens biologiques montrent une NFS normale, une VS à 10 mm à la 1ère h. Les hémocultures sont négatives. Le
diagnostic de pelvispondylite rhumatismale est posé.

Parmi les antécédents suivants retrouvés à l'interrogatoire, vous retenez en faveur du diagnostic de
pelvispondylite rhumatismale :
A - Oligoarthrite fixe
B - Hépatite virale
C - Talalgies
D - Uvéite antérieure récidivante
E - Urticaire récidivante

Parmi les signes cliniques suivants, vous retenez en faveur du diagnostic de pelvispondylite rhumatismale ?
A - Sciatiques bilatérales à bascule
B - Herpès labial
C - Souffle diastolique
D - Hépatomégalie
E - Névrite optique

Parmi les signes radiologiques suivants, vous retenez en faveur du diagnostic de pelvispondylite rhumatismale ?
A - Erosions des articulations sacro-iliaques
B - Erosions des angles antéro-supérieurs des 1ère et 2ème vertèbres lombaires
C - Discopathie L5-S1 avec ostéophytose
D - Coxa plana
E - Aspect hérisse des calcanéums

193
Exclusivement sur DOC - DZ : www.doc-dz.com NADJI 85
RESIDANAT EN POCHE TOME II
Cas Clinique en QCM

Pour rechercher une affection pouvant être associée à la spondylarthropathie chez ce patient, vous demandez :
A - Recherche de l'antigène HLA B 27
B - Wirsungographie
C - Ponction-biopsie sacro-iliaque
D - Coloscopie
E - Transit du grêle

Quels(s) traitement(s) pouvez-vous proposer pour la pelvispondylite rhumatismale chez ce patient ?


A - Antiinflammatoires non stéroïdiens
B - Sels d'or
C - D-pénicillamine
D - Antimalariques de synthèse
E - Infiltration de corticoïdes du genou

Un homme de 76 ans, 65 kg et 1.65 m souffre depuis 2 mois de lombalgies sans irradiation et de douleurs de la région de la
hanche droite irradiant à la face antérieure de la cuisse, aggravées à la station debout et à la marche. A l'examen clinique, il
n'existe ni limitation des mouvements du rachis et de la hanche, ni signe, neurologique objectif, l'état général est bien
conserveé Sur les radiographies du rachis lombaire et du bassin, on constate une ostéocondensation de la quatrième vertèbre
lombaire, de l'aile iliaque et de la ligne innominée droite. Un diagnostic de maladie de Paget est retenu.

Parmi les diagnostics suivants, lequel (lesquels) aurai(en)t pu être initialement discuté(s) devant ces plages de
condensation osseuse :
A - Métastase de cancer de la prostate
B - Ostéodystrophie rénale
C - Spondylarthrite ankylosante
D - Maladie de Hodgkin
E - Intoxication fluorée chronique

Quel(s) est(sont) parmi les nuances radiologiques suivantes, celle(s) qui orienterait(ent) plutôt vers une maladie
de Paget ?
A - Aspect en cadre de la 4ème vertèbre lombaire
B - Absence d'élargissement du corps de cette vertèbre
C - Aspect homogène de la condensation osseuse
D - Début de protrusion acétabulaire
E - Epaississement de la branche ilio-pubienne

Dans le bilan biologique de ce malade, quel(s) est(sont) le(s argument(s) qui écarteraient à priori le diagnostic
de la maladie de Paget ?
A - Vitesse de sédimentation à 72 millimètre à la 1ère heure
B - Calcémie à 2,10 mmol/l (84 mg pour mille)
C - Phosphatases alcalines à 184 unités/l (N inférieure à 50u)
D - Phosphatases acides à 56 unités/l (N inférieure à 5u)
E - Calciurie a 250 mg par 24 heures

Le diagnostic de maladie de Paget étant affirmé, quelle(s) complication(s) est(sont) à redouter à long terme,
compte tenu des localisations observées chez ce patient ?
A - Coxopathie droite
B - Insuffisance cardiaque
C - Paraplégie spasmodique
D - Dégénérescence sarcomateuse
E - Fracture spontanée

Parmi les médicaments suivants, lequel (lesquels) peut (peuvent) être utilisé(s) pour soulager rapidement les
douleurs osseuses de ce malade ?
A - Acide acétylsalicylique (2 g par jour)
B - Prednisone (30 mg par jour)
C - Vitamine B12 (15000 unités par jour)
D - Calcium (1 g par jour)
E - Indométacine (100 mg par jour)

194
Exclusivement sur DOC - DZ : www.doc-dz.com NADJI 85
RESIDANAT EN POCHE TOME II
Cas Clinique en QCM

Comme traitement de fond. on pourra retenir chez ce malade :


A - Calcitonine : 2 unités, 3 fois par semaine pendant 4 mois
B - Calcitonine : 100 unités, 3 fois par semaines pendant 4 mois
C - Stérogyl® (vit D2) : 15 mg/jour pendant 1 mois
D - Didronel® (Etidronate) : 2 comprimés par jour pendant 6 mois
E - Didronel® (Etidronate) : 6 comprimés par jour pendant 6 mois

L'efficacité de ce traitement sera jugée sur :


A - La diminution des douleurs
B - La régression de l'ostéocondensation radiologique
C - L'augmentation de la calciurie
D - La diminution des phosphatases alcalines
E - L'augmentation de l'hydroxyprolinurie

Une femme de 60 ans, sans antécédents particuliers, tombe à la suite d'un faux pas dans sa salle de bain sur la paume de la
main droite, le bras en avant. Elle ressent une vive douleur du poignet et une impotence fonctionnelle de cette articulation.
Vous la voyez une heure après le traumatisme, elle maintient son membre supérieur droit à l'aide de son avant-bras gauche;
le poignet droit est gonflé.

Vous suspectez le diagnostic de fracture de Pouteau-Colles sur :


A - L'âge de la blessée
B - La chute sur la paume
C - Le sexe féminin
D - La déformation en ventre de fourchette du poignet droit
E - L'inclinaison en valgus du poignet droit

A l'examen clinique, vous maintenez votre suspicion en trouvant :


A - Une tuméfaction du poignet droit
B - Une douleur métaphysaire
C - Une verticalisation de la ligne bistyloïdienne
D - Une dépression sus-articulaire dorsale
E - La possibilité de quelques mouvements discret

La radiographie face et profil du poignet droit confirme le diagnostic de fracture de Pouteau-Colles parce que :
A - Le trait de fracture est articulaire
B - Il existe une bascule postérieure du fragment distal
C - Le profil montre une comminution antérieure
D - De face on trouve une translation externe du fragment distal
E - Il existe une fracture de la styloïde cubitale

Le traitement orthopédique va être réalisé de la façon suivante :


A - Réduction avant la sixième heure
B - Réduction sans aucune anesthésie
C - Réduction par traction douce sur le pouce
D - Réduction par pression manuelle à direction antéro-postérieure
E - Contention plâtrée brachio-antébrachiale avec légère extension du poignet

Le traitement chirurgical primaire de cette fracture de Pouteau-Colles :


A - Est indiqué dans tous les cas
B - Peut être réalisé par embrochage
C - Peut être réalisé avec un fixateur externe
D - Peut être réalisé par cerclage
E - Doit comporter une ostéotomie radiale systématique

195
Exclusivement sur DOC - DZ : www.doc-dz.com NADJI 85
RESIDANAT EN POCHE TOME II
Cas Clinique en QCM
Un enfant de 4 ans, quelques heures après un traumatisme bénin du genou, accuse une douleur lancinante de l'extrémité
inférieure du fémur associée à une élévation thermique à 39°5. Vous soupçonnez une ostéomyélite aiguë

Quel(s) signe(s) clinique(s) recherchez-vous ?


A - Impotence fonctionnelle
B - Un flexum irréductible du genou
C - Une douleur localisée sur le condyle interne
D - Une douleur diaphyso-épiphysaire circulaire et transfixiante
E - La présence d'adénopathies inguinales

Quel(s) examen(s) demandez-vous en urgence ?


A - Radiographies standard du genou
B - Hémoculture
C - Dosage des antistreptolysines
D - Formule leucocytaire
E - Vitesse de sédimentation globulaire

Quel(s) traitement(s) proposez-vous ?


A - Anti inflammatoires non stéroïdiens
B - Antibiotique intraveineux
C - Corticothérapie par voie générale
D - Immobilisation plâtrée
E - Intervention d'urgence

Après une sédation d'une dizaine de jours, des douleurs pulsatiles nocturnes apparaissent localisées au fémur.
Quelle(s) complication(s) recherchez-vous ?
A - Une arthrite du genou
B - Une phlébite
C - Un abcès sous périosté
D - Un décollement épiphysaire
E - Une escarre sous plâtre

Un garçon de cinq ans en parfaite santé rentre de l'école en boitillant : il se couche et refuse de se lever pour dîner. Il accuse
une douleur diffuse de l'aine et du tiers supérieur de la cuisse gauche. On n'a noté aucun traumatisme particulier, aucune
épidémie a l'école, aucun antécédent familial orthopédique particulier. A l'examen : pas de fièvre, pas de céphalée, pas de
troubles digestifs, O.R.L. ou autres. Pas d'attitude vicieuse, pas de modification locale, mobilité de la hanche gauche un peu
limitée et douloureuse.

Votre première prescription est :


A - Corticothérapie adaptée au poids de l'enfant
B - Antibiothérapie à spectre large
C - Vitaminothérapie D
D - Kinésithérapie
E - Mise au repos absolu en attendant le résultat des examens complémentaires

Votre diagnostic de probabilité est :


A - Arthrite rhumatismale
B - Arthrite septique de la hanche
C - Poliomyélite
D - Ostéochondrite de la tête fémorale
E - Epiphysiolyse de la hanche

La radiologie du bassin est normale, quel autre examen demandez-vous en priorité ?


A - Bilan électrophorèse sanguin
B - Examen scintigraphique
C - Echographie du petit bassin
D - Tomographie de l'extrémité supérieure du fémur
E - Recherche du groupe HLA B27

196
Exclusivement sur DOC - DZ : www.doc-dz.com NADJI 85
RESIDANAT EN POCHE TOME II
Cas Clinique en QCM

Si le diagnostic est confirmé, quel sera le traitement à cette phase initiale ?


A -Abstention thérapeutique
B - Traitement antalgique et anti-inflammatoire
C - Ponction articulaire, antibiothérapie et immobilisation plâtrée
D - Mise en décharge
E - Traitement chirurgical

Une femme de 45 ans consulte pour une polyarthrite d'apparition récente depuis quelques mois. Elle n'a reçu que des
thérapeutiques non spécifiques symptomatiques. Les douleurs siègent aux poignets et aux mains,aux avant-pieds et plus
modérément aux genoux. Les signes d'épanchement intra-articulaire sont nets, il n'y a pas de déformation apparente. La V.S.
est à 60 mm/85 mm.

Parmi ces éléments, le(s)quel(s) représente(nt) des arguments d'orientation diagnostique en faveur d'une
polyarthrite rhumatoïde ?
A - Atteintes symétriques des interphalangiennes proximales
B - Douleurs à la pression des têtes métatarsiennes
C - Ténosynovite des muscles extenseurs des doigts
D - Radiologiquement : érosion sur la tête de la phalange distale du Vème orteil
E - Sérologie rhumatoïde : Waaler-Rose au 1/16

Avant d'affirmer le diagnostic de polyarthrite rhumatoïde, il est nécessaire d'éliminer une maladie lupique. Parmi
ces propositions, quelle(s) est (sont) celle(s) qui vous oriente(nt) en ce sens dans la mesure ou elle(s) se
voie(nt) essentiellement dans la maladie lupique et beaucoup plus exceptionnellement dans la polyarthrite
rhumatoïde ?
A - Présence d'anticorps antinucléaires à la dilution de 1/50
B - Découverte d'une leucopénieà la numération formule sanguine
C - Baisse du complément sérique mesuré par la valeur du CH50
D - Taux élevé de complexe immuns circulants
E - Positivité nette de l'immunofluorescence sur Crithidia Luciliae

Chez cette patiente, vous pourrez redouter un certain nombre de manifestations extra articulaires compliquant
la polyarthrite rhumatoïde. Laquelle(lesquelles) ?
A - Insuffisance ventriculaire gauche
B - Pleurésie séro-fibrineuse
C - Amylose
D - Pancréatite aiguë nécrotico hémorragique
E - Neuropathie périphérique sensitivomotrice

Vous décidez de la prescription d'une chrysothérapie. Quel(s) est (sont) le(s) éléments qui représentent une
contre-indication à son instauration ?
A - Hyperglycémie
B - Présence d'anticorps antiDNA
C - Protéinurie
D - Hypertriglycériémie
E - Hyperleucocytose

La chrysothérapie ne pouvant être instaurée, quelle(s) autre(s) mesure(s) thérapeutique(s) vous semble(nt)
correcte(s) dans le traitement de cette polyarthrite ?
A - Traitement par la D-Penicillamine
B - Prescription d'anti inflammatoires non stéroïdiens
C - Corticothérapie générale
D - Immunodépresseurs
E - Infiltrations intra-articulaires cortisoniques hebdomadaires

197
Exclusivement sur DOC - DZ : www.doc-dz.com NADJI 85
RESIDANAT EN POCHE TOME II
Cas Clinique en QCM
Vous êtes interne aux urgences, vous recevez une dame âgée de 70 ans ayant fait une chute sur le poignet en
hyperextension. Son poignet est déformé et douloureux; l'impotence fonctionnelle est totale.

Votre prescription radiologique d'emblée comporte :


A - Un poignet de face
B - Une incidence du canal carpien
C - Des cliches fonctionnels du poignet
D - Un profil strict du poignet
E - Des 3/4 du poignet

En faveur d'une fracture de Pouteau-Colles, vous retenez :


A - Une bascule postérieure de l'épiphyse radiale
B - Une ascension de l'épiphyse radiale
C - Un trait de fracture sus-articulaire
D - Une translation latérale interne de l'épiphyse radiale
E - Une comminution postérieure du trait de fracture

Il est fréquent qu'une telle fracture se complique immédiatement de :


A - Une rupture du nerf médian
B - Un syndrome de Volkmann
C - Une ouverture du foyer de fracture
D - Une compression de l'artère radiale
E - Aucune de ces propositions

Les 3 complications secondaires les plus fréquentes sont :


A - Une pseudarthrose
B - Un cal vicieux
C - Une raideur de la main et des doigts
D - Une algodystrophie réflexe de la main
E - Une ostéite

Quelle est la fracture de l'extrémité inférieure du radius la plus fréquente chez une femme âgée ?
A - Fracture sus-articulaire de Pouteau-Colles
B - Fracture comminutive, de l'extrémité inférieure du radius
C - Fracture décollement épiphysaire du radius
D - Fracture articulaire marginale antérieure
E - Fracture en bois vert de l'extrémité inférieure du radius

Une femme de 64 ans souffre depuis 12 ans d'une polyarthrite rhumatoïde. Le traitement a uniquement comporté des
corticoïdes et des anti inflammatoires non stéroïdiens : elle reçoit actuellement 15 mg/j de Prednisone et 125 mg/j
d'indométacine. Toute tentative de réduction des corticoïdes s'accompagne d' une reprise des douleurs. Les signes
d'hypercorticisme sont évidents. Deux motifs à son hospitalisation : des difficultés de préhension de la main droite, des
douleurs du membre inférieur gauche à la marche depuis deux mois. Examen clinique :
1 ) Les difficultés de préhension s'expliquent par une déformation fixée des 2 et 3èmes doigts : hyperextension des
interphalangiennes proximales (IPP), flexion des métacarpophalangiennes (MCP) et des interphalangiennes distales.
2) La douleur gênant la marche, siège à la face antérieure de la cuisse et s'arrête au dessus du genou. Les genoux sont secs,
froids et stables depuis une synoviorthèse à Lynrium 90 faite trois ans auparavant. L'abduction et les rotations de la cuisse
réveillent la douleur. Les radiographies : arthrites radiocarpiennes et des métacarpophalangiennes bilatérales. Les IPP et IPD
sont radiologiquement normales.
- genoux : discrets pincements des interlignes.
- bassin de face : les interlignes coxofémoraux sont normaux ; à gauche, la structure de la tête fémorale est hétérogène, le
contour a perdu sa sphéricité.
- rachis : tassement de D7 et L3.

Quelle déformation des doigts gêne la fonction de la main chez cette malade ?
A - Coup de vent cubital
B - Main de singe
C - Doigts en boutonnière
D - Doigts à ressorts
E - Doigts en col de cygne

198
Exclusivement sur DOC - DZ : www.doc-dz.com NADJI 85
RESIDANAT EN POCHE TOME II
Cas Clinique en QCM

Les difficultés et douleurs à la marche sont dues à :


A - Une neuropathie par vascularite rhumatoïde
B - Une coxite rhumatoïde
C - Une algodystrophie
D - Une ostéonécrose de la tête fémorale
E - Une ostéoporose

Quel(s) examen(s) parmi les suivants a(ont) un intérêt pour orienter le traitement médical de la polyarthrite
rhumatoïde chez cette malade ?
A - Waaler Rose, Latex
B - Scintigraphie osseuse
C - Biopsie de synoviale
D - Recherche d'immuns complexes circulants
E - Protéinurie

Cette malade souffre d'un hypercorticisme iatrogène. Quel est parmi les programmes thérapeutiques suivants
celui qui vous parait le plus à même de le pallier sans dommage pour son état articulaire ?
A - Arrêt progressif des corticoïdes en augmentant les doses d'indométacine
B - Substitution des corticoïdes oraux par des injections d'ACTH de synthèse
C - Arrêt progressif des corticoïdes et chirurgie réparatrice des mains
D - Mise en route d'un traitement antirhumatismal à action lente (Sels d'Or ou D-pénicillamine) puis réduction
progressive des corticoïdes
E - Prescription d'anabolisant de synthèse (androgènes non virilisants)

Une femme de 54 ans consulte pour une sciatique droite de topographie S1 apparue un mois plus tôt sans effort déclanchant
net. Elle a des antécédents de lombalgies d'effort. Elle a subi 7 ans plus tôt une hystérectomie totale pour fibrome. La douleur
est mal calmée par le repos et elle réveille même la malade. L'état général est conservé et il n'y a pas de fièvre. Il n'y a pas de
syndrome rachidien ni de signe de Lasègue. Le réflexe achilléen droit est faible sans autre déficit neurologique. Le reste de
l'examen clinique est normal.Les radiographies du rachis lombaire de face et de profil, et du bassin de face sont normales. La
vitesse de sédimentation est à 8 mm à la première heure. l'hémogramme est normal. La calcémie est a 3 mmol/l (120 mg/l).
La créatininémie est normale.

L'absence de syndrome rachidien oriente le diagnostic vers :


A - Une hernie discale non exclue
B - Une ostéoporose vertébrale
C - Une tumeur intrarachidienne
D - Une métastase osseuse sacrée
E - Une spondylodiscite infectieuse

En faveur d'une hernie discale vous retiendriez :


A - Début progressif sans effort
B - Absence de signe de Lasègue
C - Antécédents fréquents de lombalgie d'effort
D - Vitesse de sédimentation normale
E - Absence de syndrome rachidien

La constatation d'une hypercalcémie peut orienter le diagnostic vers :


A - Hernie discale
B - Maladie de Kahler
C - Métastase d'un cancer viscéral
D - Tumeur intrarachidienne
E - Spondylodiscite infectieuse

Contre le diagnostic de maladie de Kahler vous retiendrez :


A - Immuno-électrophorèse du sérum normale
B - Immuno-électrophorèse des urines normale
C - Myélogramme normal
D - Absence d'insuffisance rénale
E - Vitesse de sédimentation normale

199
Exclusivement sur DOC - DZ : www.doc-dz.com NADJI 85
RESIDANAT EN POCHE TOME II
Cas Clinique en QCM
Une femme de 70 ans accuse depuis plusieurs mois une altération progressive de l'état général avec douleurs osseuses
diffuses. L'examen clinique est par ailleurs normal. Un bilan est alors pratique dont les principaux éléments sont les suivants :
- images ostéolytiques à l'emporte pièce au niveau du crâne, du bassin et des côtes ; déminéralisation du rachis dorso-
lombaire
- VS à 70 mm à la 1 ère heure
- myélogramme 40 % de plasmocytes, dont certains atypiques
- protidémie = 95 g/1
- electrophorèse des protéines sériques : pic d'allure monoclonale au niveau des gamma-globulines (40 g/l)
- immunoélectrophorèse des protéines sérique : identification du pic à une immunoglobuline monoclonale de type IgG kappa
- protéinurie : 1 g/l. presque exclusivement constituée par une protéinurie de Bence Jones
- créatininémie = 340 micromol/l
- calcémie = 3,2 micromol/l
Le diagnostic de maladie de Kahler ou myélome multiple est posé et la malade hospitalisée.

En dehors de la maladie de Kahler, la maladie de Waldenstrِm représente une autre variété de dysglobulinémie
maligne. Elle peut être éliminée chez notre malade devant :
A - Son âge
B - Le degré d'accélération de la VS
C - Le type de la prolifération cellulaire médullaire
D - Le type de l'immunoglobuline
E - L'existence d'une protéinurie de Bence Jones

On peut aussi éliminer chez cette malade une gammapathie monoclonale dite bénigne devant :
A - Le degré d'accélération de la VS
B - Nature IgG de l'immunoglobuline monoclonale
C - L'importance de la plasmocytose médullaire
D - Le taux de la calcémie
E - La présence de lésions ostéolytiques

La protéinurie de Bence Jones signalée :


A - Est constituée de chaînes lourdes d'immunoglobuline monoclonale
B - Est constituée de chaînes légères d'immunoglobuline monoclonale
C - Est constituée de molécules complètes d'immunoglobuline monoclonale
D - Est la conséquence de l'insuffisance rénale
E - Est nécessaire au diagnostic de maladie de Kahler

L'hypercalcémie de la maladie de Kahler :


A - Est un élément permettant de suspecter une masse tumorale élevée
B - S'accompagne d'ordinaire d'une phosphorémie abaissée
C - Ne s'observe pas au cours du myélome à chaînes légères
D - Ne s'observe que lorsqu'existent des lésions ostéolytiques visibles radiologiquement
E - Peut être réduite par la chimiothérapie du myélome

Le traitement de fond de cette maladie de Kahler peut faire appel a :


A - Melphalan (Alkeran®)
B - Cyclophosphamide (Endoxan®)
C - Chlorambucil (Chloraminophène®)
D - Prednisone (Cortancyl®)
E - Radiophosphore

200
Exclusivement sur DOC - DZ : www.doc-dz.com NADJI 85
RESIDANAT EN POCHE TOME II
Cas Clinique en QCM
M. V..., 74 ans, est hospitalisé pour des douleurs inguino-crurales tenaces, d'aggravation progressive et maintenant
responsables de sévères troubles de la marche. Dans les antécédents, on note une claudication intermittente qui a fait
découvrir à58 ans une artériopathie oblitérante des membres inférieurs ; une sympathectomie lombaire gauche, réalisée à 61
ans, n'a eu qu'un effet partiel ; la persistance de douleurs à la marche a conduit le malade à limiter de plus en plus ses sorties
; ces dernières années, il n'a pratiquement pas quitté son domicile (appartement en ville). Depuis que l'affection artérielle a été
décelée M. V... s'est astreint à un régime très restrictif en matières grasses d'origine animale. Les douleurs actuelles ont
débuté dans la région inguinale gauche ; un cliché a mis en évidence une fissure du col fémoral gauche qui a fait parler de
fracture de fatigue et à conduit a la suppression de l'appui pendant 3 mois. Mais les douleurs sont réapparues des la reprise
de la marche ; elles se sont bilatéralisées et s'associent à des douleurs thoraciques et scapulaires depuis quelques mois.
Elles sont maintenant très sévères mais restent de type mécanique, elles ne permettent que quelques petits pas
précautionneux. Outre les troubles de la marche, l'examen clinique montre un état général assez médiocre (45 kg pour 160
cm; perte de 8 kg) ; il n'y a pas de limitation nette des mouvements des hanches et du rachis; la pression du thorax est
douloureuse. Les pouls fémoraux sont bien perçus. L'examen neurologique et digestif est normal ainsi que le reste de
l'examen clinique. Les examens sanguins suivants donnent des résultats normaux : VS, hémogramme, créatininémie.,
glycémie, électrophorèse des protides. Il existe une hyperphosphatasémie alcaline modérée. Calcémie : 84 mg/l (21 mmol/l) ;
phosphorémie : 22 mg/l (0,70 mmol), calciurie des 24 heures : 29 mg (0.82 mmol). Sur les radiographies du rachis lombaire,
les vertèbres ont des contours flous ; le cliché du bassin décèle, sur le bord supérieur des deux cols fémoraux, une fissure
avec condensation périphérique et périostose ; les articulations coxofémorales ont un aspect radiologique normal.

Lequel de ces examens permettra de confirmer le diagnostic d'ostéomalacie ?


A - Scintigraphie osseuse
B - Biopsie osseuse iliaque au trocart
C - Dosage de la 25 OH - vitamine D sérique
D - Radiographie de l'ensemble du squelette
E - Mesure de la clairance rénale du phosphore

Parmi les données suivantes concernant ce cas, laquelle(lesquelles) n'est(ne sont) jamais recontrée(s) dans
l'ostéoporose commune ?
A - L'âge de 74 ans
B - Le sexe masculin
C - L'artériopathie oblitérante des membres inférieurs
D - Les stries de Looser
E - L'hypocalcémie marquée

Le(s)quel(s) de ces 5 éléments peu(ven)t être incriminé(s) dans la survenue de cette ostéomalacie ?
A - Le régime alimentaire très pauvre en lipides d'origine animale
B - Un traitement par antivitamine K suivi 3 ans pour l'artériopathie, quelques années auparavant
C - Un traitement hypolipémiant par clofibrate
D - Un défaut d'exposition solaire
E - Une réduction de l'activité physique

Le(s)quel(s) des 5 traitements suivant(s) est(sont) indiqué(s) chez ce malade ?


A - Calcithérapie isolée, 1.5 g/jour per os
B - Fluorure de sodium (50 mg/jour) + calcium 1 g/jour
C - Vitamine D2 : 10 000UI/jour
D - Calcitonine de saumon (Calsyn®) : 50 UI par vois sous-cutanée
E - Fluorure de sodium (50 mg/jour) + vitamine D2 8 000 UI/jour

Une femme de 42 ans souffrait depuis un an. Au début, elle se plaignait d'un engourdissement nocturne des doigts puis sont
apparus des douleurs et un gonflement des poignets et une difficulté à fléchir les doigts, des douleurs et gonflement du genou
droit et des métatarsophalangiennes droites et gauches. Le diagnostic retenu 6 mois après le début de l'affection sur des
examens cliniques biologiques et radiologiques est celui de polyarthrite rhumatoïde. Les douleurs et les signes objectifs
d'arthrite étant insuffisamment soulagés par 3 grammes par jour d'aspirine ou par 150 mg par jour d'indométacine, un
traitement par les sels d'or a été prescrit à la dose de 5 centigrammes par semaine. Le traitement a été bien toléré. Au 5ème
mois l'effet de ce traitement est favorable. La raideur matinale est passée de 45 a 15 minutes. Les poignets ont retrouve un
aspect normal. La vitesse de sédimentation initialement à 65 mm à la 1ère heure est revenue à 15 mm à la 1 ère heure

Quelle est l'origine des engourdissements nocturnes des doigts qui ont inauguré la maladie ?
A - Une névralgie cervico-brachiale
B - Une neuropathie amyloïde
C - Une synovite des fléchisseurs
D - Une ostéonécrose du semi-lunaire
E - Une spasmophilie

201
Exclusivement sur DOC - DZ : www.doc-dz.com NADJI 85
RESIDANAT EN POCHE TOME II
Cas Clinique en QCM

Une ponction du genou a été effectuée avant la mise au traitement par l'allochrysine. Parmi les formules du
liquide articulaire, quelle est celle qui s'adapte le mieux au diagnostic retenu de polyarthrite rhumatoïde ?

Eléments/mm3 Polynucl Lymphoc. Monoc. Cristaux. Rapport


compl.articul.
compl.sérique
A - 8 700 75 % 15 % 10 % - 0,15
B - 8 700 75 % 15 % 10 % - 0,50
C - 8 700 75 % 15 % 10 % + 0,25
D - 8 700 10 % 10 % 75 % - 0,50
E - 500 10 % 65 % 25 % - 0,25

Quelle est parmi les associations suivantes, celle qui est la plus compatible avec le diagnostic de polyarthrite
rhumatoïde avant la mise en route du traitement de fond ?
A - VS : 65, leucocytes : 3 200, Waaler Rose : 1/128, Latex : 1/20000, anticorps antinucléaires : 1/1000,
homogène : protéinurie absente
B - VS : 65., leucocytes : 7 200, Waaler Rose : négatif, Latex : négatif anticorps antinucléaires : absent,
protéinurie : absente
C - VS : 65, leucocytes : 7 200, Waaler Rose négatif, Latex négatif anticorps antinucléaires : absent,
protéinurie 2 g/24 h
D - VS : 65, leucocytes : 7 200, Waaler Rose : négatif, Latex : négatif. anticorps antinucléaires : 1/1000,
mouchète, protéinurie : absente
E - VS : 10, pincement et condensation des interlignes articulaires des 2 et 3ème articulations
métacarpophalangiennes droites

Sur les radiographies de l'avant pied faites au 7ème mois d'évolution, une image très évocatrice de PR. est
retrouvée : laquelle ?
A - Déminéralisation diffuse des os de l'avant pied
B - Géode cerclée de la diaphyse du 5ème métatarsien
C - Erosion de la 5ème tête métatarsienne
D - Pincement du 5ème interligne métatarsophalangien et ostéophytose
E - Périostose du 5ème métatarsien

Le traitement par les sels d'or étant décidé. quelle surveillance vous parait la plus adaptée ?
A - Electrorétinogramme tous les six mois - protéinurie tous les mois
B - NFS - VS toutes les semaines - protéinurie tous les deux mois
C - Protéinurie avant chaque injection d'or, NFS, VS, Waaler Rose, Latex tous les deux mois
D - Protéinurie avant chaque injection d'or, NFS, VS, Waaler Rose, Latex facteurs antinucléaires tous les 3
mois
E - Protéinurie avant chaque injection d'or, NFS, V S, tous les mois

Quelle suite pensez-vous donner au traitement de cette malade, compte-tenu de l'amélioration actuelle ?
A - Allochrysine® : cinq centigrammes par semaine
B - Arrêt de l'Allochrysine® et surveillance clinique et biologique
C - Arrêt de l'Allochrysine® remplacée par 600 mg de D-Penicillamine par jour
D - Allochrysine : dix centigrammes par mois tant que ce médicament reste efficace et bien supporté
E - Arrêt de l'Allochrysine®, indométhacine : 75 mg par jour

202
Exclusivement sur DOC - DZ : www.doc-dz.com NADJI 85
RESIDANAT EN POCHE TOME II
Cas Clinique en QCM
Un homme de 32 ans, peintre en bâtiment, souffre d'une sciatique droite. Celle-ci, s'est installée à la suite d'un effort, survenu
sur son lieu de travail. L'interrogatoire retrouve des épisodes de lumbagos aigus de durée brève et d'intensité modérée, mais
le patient n'avait jamais eu de sciatique. Il n'a pas d'antécédents pathologiques notables, en particulier pas de problèmes
digestifs. La douleur n'est pas très intense, mais elle est exacerbée par la position debout ou assise prolongée. Elle est mal
calmée par le repos et tient le malade réveille la nuit. Il n'a pas de troubles sphinctériens. A l'examen, il existe une attitude
antalgique croisée : le rachis est raide avec un signe de la sonnette. La douleur suit un trajet L5 : la manoeuvre de Lasègue
devient douloureuse à 60 degrés Le réflexe achilléen est conservé, la force musculaire est normale, il n'y a pas d'anesthésie
en selle. L'examen clinique général est normal. Des radiographies standard du rachis mettent en évidence une diminution
nette de la hauteur du disque L4-L5, à l'exclusion de toute autre anomalie.

Parmi les possibilités thérapeutiques suivantes, vous retenez initialement :


A - Mise au repos complet
B - Kinésithérapie lombaire
C - Diclofanac (Voltarène®) : 150 mg par jour per os
D - Paracétamol : 1,5 g par jour
E - Prednisone : 1 mg par kilo et par jour

Après 8 jours de ce traitement, aucune amélioration n'est obtenue. Vous proposez alors :
A - Mise en place d'un lombostat plâtré
B - 3 infiltrations épidurales de corticoïdes à 4 jours d'intervalle
C - Manipulations vertébrales
D - Tractions vertébrales
E - Injections IM de vitamine B12 (Dibencozan 20 000®) une par jour pendant trente jours

Cette attitude est la bonne, puisque le patient s'améliore et s'achemine vers la guérison Que prescrivez-vous
alors ?
A - Arrêt de travail de 10 jours
B - Arrêt de travail de 45 jours
C - 20 séances de massages et de rééducation du rachis lombaire
D - Un lombostat
E - Jusqu'à disparition complète des symptômes, de la vitamine B1 (1 g par jour), par voie orale

Lors de la reprise du travail, il faut régulariser la position du malade vis à vis des organismes sociaux. Plusieurs
attitudes sont possibles, sauf une. Laquelle ?
A - Reprise avec soin trois mois
B - Guérison
C - Demande de mi-temps thérapeutique
D - Consolidation avec I.P.P. à fixer
E - Aucune formalité n'est nécessaire

Dès la reprise du travail, une récidive survient, avec diminution importante de la force musculaire des péroniers
latéraux, persistante après une semaine de traitement médical bien conduit. Les explorations neuro-
radiologiques confirment la présence d'une hernie discale exclue L4-L5. Que proposez-vous ?
A - Reprise du traitement médical : repos au lit, infiltration
B - La mise en place d'un lombostat plâtré
C - Une chimionucléolyse
D - Un traitement chirurgical de la hernie discale
E - La mise en tractions continue

Un blessé de 45 ans, transporteur routier, éthylique, pesant 87 kilos, a été victime 15 ans auparavant d'un polytraumatisme
avec traumatisme crânien, fracture de la diaphyse fémorale droite et gauche, fracture ouverte comminutive de la jambe droite,
au tiers moyen-tiers inférieur traitée par plâtre. IL a eu des suites opératoires mouvementées, il a repris son travail deux ans
après l'accident. IL consulte pour des douleurs du genou droit de type mécanique apparues il y a quelques mois. A l'examen
clinique et radiographique, les deux fémurs sont consolides et bien alignes. mais le tibia droit présente un cal vicieux avec
angulation de 10° en varus. Il existe des signes d'ostéite avec une fistule qui s'ouvre deux ou trois fois par an mais qui est
actuellement cicatrisée.

Les séquelles auraient été éventuellement évitées par un traitement local de la fracture ouverte comportant :
A - Parage méticuleux
B - Immobilisation stricte par fixateur externe
C - Réduction correcte
D - Fermeture de la peau
E - Conservation des esquilles osseuses dépériostées
203
Exclusivement sur DOC - DZ : www.doc-dz.com NADJI 85
RESIDANAT EN POCHE TOME II
Cas Clinique en QCM

Une désorientation temporospatiale était apparue 24 heures après l'accident, il existait des pétéchies sous-
conjonctivales et des infiltrats au fond d'oeil. S'agissait-il ?
A - D'un delirium tremens
B - D'un coma hépatique
C - D'une hémorragie méningée
D - D'une embolie graisseuse
E - D'un hématome extra-dural

Vers le cinquième jour après l'accident, devant une élévation thermique à 38,5 degrés avec douleur de la
jambe, il fallait évoquer :
A - Compression par plâtre trop serré
B - Déplacement secondaire des fragments
C - Infection du foyer de fracture
D - Thrombo-phlébite
E - Escarre sous plâtre

Pour connaître la cause des douleurs actuelles du genou, quel(s) examen(s) complémentaire(s) apporte(nt) le
plus simplement la réponse
A - Radiographies standard du genou en charge
B - Tomodensitométrie du genou
C - Radiographies des membres inférieurs en totalité et en charge
D - Arthrographie
E - Scintigraphie osseuse

Parmi les conseils thérapeutiques suivants, un est à exclure de façon impérative :


A - La rééducation fonctionnelle
B - Les infiltrations intra-articulaires de corticoïdes
C - Le traitement chirurgical de l'ostéïté
D - Le changement de métier
E - La prescription d'anti inflammatoires non stéroïdiens

Un garçon de 23 ans a noté une modification de ses traits depuis 3 ans. Ses extrémités se sont épaissies. Il a du changer de
paires de chaussures. Il a remarqué que son visage s'était modifié. Il présente des arthralgies au niveau des deux poignets : il
se plaint de céphalées à prédominance matinale.

Le diagnostic le plus vraisemblable est :


A - Une hypoparathyroïdie
B - Une hypothyroïdie
C - Une acromégalie
D - Un hypercortisolisme
E - Un hypogonadisme

Ce syndrome est souvent associé avec :


A - Une phosphorémie basse, une hypoglycémie, une cholestérolémie élevée
B - Un diabète, un épaississement des parties molles, une hémianopsie bitemporale
C - Une ostéoporose, des vergetures, une hypertension artérielle
D - Une augmentation de la densité osseuse, une hypocalcémie
E - Aucun de ces groupements symptomatiques

Les radiographies osseuses révèlent habituellement dans ce type d'affection :


A - Une résorption sous-périostée, des kystes osseux, un effondrement vertébral
B - Une calcification des ganglions de la base, une densité osseuse augmentée des calcifications ectopiques
C - Une ostéoporose, un amaincissement du crâne
D - Un allongement du maxillaire inférieur, une selle turcique augmentée de volume, un épaississement des
parties molles
E - Aucune anomalie osseuse

204
Exclusivement sur DOC - DZ : www.doc-dz.com NADJI 85
RESIDANAT EN POCHE TOME II
Cas Clinique en QCM

Ces patients présentent souvent de manière associée :


A - Un goître, une hypersudation, une hyperséborrhée
B - Une captation élevée d'iode radioactif, une hypercalcémie avec lithiase rénale
C - Une hyperplasie surrénalienne
D - Une hypoglycémie, une poly-adénomatose endocrinienne avec histoire familiale
E - Une phosphorémie normale avec phosphatase alcaline élevée

Un malade est suivi pour une spondylarthrite ankylosante. A la suite d'une chute, il se plaint d'une douleur lombaire surtout
vive en position debout et aux mouvements. Cette douleur n'est pas améliorée par les antiinflammatoires non stéroïdiens
(AINS) alors que sous AINS, il n'a pas les douleurs nocturnes qui le réveillaient chaque nuit auparavant. L'examen montre que
la distance C7-mur qui était de 8 cm un an avant est passée à 12 cm. Mains-sol : 32 cm inchangée. Les radiographies
montrent comme l'année précédente une sacroiliite bilatérale stade II, des syndesmophytes dorsolombaires et une fusion des
articulaires postérieures dorsales basses et lombaires. Sont apparus un pincement du disque L1-L2, une condensation des
plateaux en regard et une solution de continuité sur l'arc postérieur

L'augmentation de la distance C7-mur traduit l'aggravation de quelle déformation ?


A - Projection du cou en avant
B - Lordose lombaire
C - Cyphose dorsale
D - Luxation cervico-occipitale
E - Scoliose à court rayon

Quelle(s) mesure(s) faut-il prendre pour corriger cette déformation ?


A - Postures à plat ventre
B - Traitement de fond par la Salazopyrine®
C - Port d'un collier cervical
D - Gymnastique intensive
E - Port d'un corset plâtré de Swain

Chez ce malade, l'atteinte sacro-iliaque bilatérale classée stade II, se traduit radiologiquement par :
A - Un pincement irrégulier de l'interligne
B - Une fusion des deux os
C - Un pseudoélargissement de l'interligne
D - Une condensation des berges sacrées et iliaques
E - Des géodes sous chondrales dans une condensation de la berge iliaque

Quelle est la nature de la lésion discale L1-L2 apparue depuis un an ?


A - Arthrose pseudopottique
B - Spondylodiscite à Klebsiella Pneumoniae
C - Spondylodiscite inflammatoire de la Sp A
D - Pseudospondylodiscite sur une zone hypermobile
E - Mal de Pott

Une femme de 70 ans, vivant seule chez elle, est retrouvée sur le sol dans sa cuisine par ses voisins, 24 heures après une
chute d'un escabeau. Elle est consciente mais déshydratée. Elle est incapable de se relever du fait d'une impotence
fonctionnelle totale du membre inférieur droit. Celui ci paraît raccourci, en adduction et rotation externe. La température est à
38.5°.
Le médecin habituel de cette malade, appelé en urgence, demande l'hospitalisation et note sur son certificat ses antécédents :
- Varices volumineuses bilatérales
- Insuffisance respiratoire chronique
- Coxarthrose droite évoluée

Quel est le diagnostic clinique le plus probable ?


A - Luxation postéro-supérieure de hanche
B - Fracture de l'ischion
C - Fracture du cotyle
D - Fracture par arrachement du grand trochanter
E - Fracture du col du fémur

205
Exclusivement sur DOC - DZ : www.doc-dz.com NADJI 85
RESIDANAT EN POCHE TOME II
Cas Clinique en QCM

Le bilan radiologique que vous demandez en urgence comporte :


A - Bassin de face
B - Grands axes des membres inférieurs
C - Hanche droite face et profil
D - Clichés dynamiques de hanche droite en abduction et adduction
E - Radiographie pulmonaire

Quel(s) autre(s) examen(s) vous parai(ssen)t utile(s) parmi ceux-ci ?


A - Epreuve cardiaque d'effort
B - Gaz du sang
C - Recherche d'un facteur rhumatoïde
D - Tests de coagulation
E - Scintigraphie osseuse

Parmi ces prescriptions, il faut mettre immédiatement en oeuvre :


A - Une transfusion sanguine
B - Une rééquilibration ionique
C - Un anticoagulant
D - Une traction du membre
E - Une antibiothérapie à large spectre

Si un acte chirurgical est nécessaire, il devrait être logiquement pour vous :


A - Une abstention chirurgicale
B - Une ostéosynthèse par vis
C - Une ostéosynthèse par clous de Ender
D - Une prothèse totale
E - Une prothèse céphalique

Si un acte chirurgical est effectué, il devient alors justifié d'instituer :


A - Lutte anti-escarre
B - Kinésithérapie respiratoire
C - Traitement anticoagulant
D - Bandages élastiques des membres
E - Lever précoce

Madame D.25 ans, est hospitalisée pour des polyarthralgies touchant les coudes et les genoux. L'interrogatoire ne retrouve
aucun antécédent, ni prise médicamenteuse. L'examen clinique met en évidence des ulcérations buccales et un frottement
péricardique. Le bilan biologique objective une anémie normochrome normocytaire régénérative avec Coombs direct positif et
une protéinurie. Le bilan hépatique est normal. L'électrophorèse des protides montre une hyperalpha2globulinémie et une
hypergammaglobulinémie. La VS est augmentée. Le diagnostic de lupus érythémateux disséminé est évoqué.

Ce diagnostic pourrait également être évoqué devant :


A - Hypocomplémentémie
B - Présence d'anticorps anti-IgG
C - Immunofluorescence anti-noyau positive au 1/10 000
D - Diminution du pourcentage des lymphocytes T8+
E - Présence de cryoglobuline

Quel(s) est(sont) celui(ceux) dont la positivité est en faveur du diagnostic ?


A - Présence d'anticorps anti-RNP
B - Présence d'anticorps anti-DNA natif
C - Présence d'anticorps anti-Sm
D - Présence d'anticorps anti-DNA dénaturé
E - Présence d'anticorps anti-centromère

206
Exclusivement sur DOC - DZ : www.doc-dz.com NADJI 85
RESIDANAT EN POCHE TOME II
Cas Clinique en QCM

Le diagnostic étant confirmé, indiquez celui ou ceux qui vous parai(ssen)t utile(s) pour suivre l'évolution :
A - Complément hémolytique sérique total CH 50
B - Anticorps anti-muscle lisse
C - Anticorps anti-microsomes thyroïdiens
D - Facteur rhumatoïde
E - Anticorps anti-DNA natif

Citez la(les) localisation(s) clinique(s) pouvant mettre en jeu la vie du malade :


A - Cutanée
B - Rénale
C - Pleurale
D - Articulaire
E - Cérébrale

Un malade de 45 ans, conducteur de poids lourds, souffre depuis trois mois, après un effort de soulèvement, d'une douleur
lombaire irradiant à la fesse gauche, à la face postéro-latérale de la cuisse, à la face externe de la jambe, pour gagner le
dessus du pied gauche. Cette douleur, d'horaire mécanique, calmée par le repos, est impulsive à la toux et s'accompagne de
paresthésies plantaires. Antécédents : lombalgies chroniques depuis dix ans avec épisodes de lumbagos, intervention pour
hernie inguinale gauche il y a quatre ans.

Parmi les signes cliniques suivants, indiquez celui ou ceux qui seraient en rapport avec les atteintes
monoradiculaires présentées par ce malade ?
A - Parésie du moyen fessier
B - Impossibilité de marcher sur la pointe du pied
C - Diminution du réflexe rotulien
D - Hypoesthésie de la face dorsale du pied et du gros orteil
E - Abolition du réflexe crémastérien

Parmi les signes radiologiques suivants, lequel(lesquels) vous ferait(feraient) douter du diagnostic de sciatique
commune par hernie discale ?
A - Disques L4-L5 et L5-S1 normaux
B - Baillement postérieur L4-L5
C - Effacement sur plateau inférieur de L4
D - Pincement L4-L5 avec érosion de l'angle antérosupérieur de L5
E - Tassement du corps vertébral L4

Malgré la mise en oeuvre d'un traitement médical approprié, la douleur sciatique persiste, bien que la lombalgie
se soit améliorée. L'indication d'une sacco-radiculographie d'urgence sera décidée en raison de la constatation
clinique récente de :
A - Intolérance et inefficacité des anti-inflammatoires
B - Sensation de refroidissement du pied gauche
C - Persistance des lombalgies
D - Nécessité de pousser pour uriner
E - Hypoesthésie périnéale

Une sacco-radiculographie est faite. Le liquide céphalo-rachidien est analysé à cette occasion. Indiquez la ou
les formule(s) compatible(s) avec une origine discale de cette sciatique ?
A - Protéinorachie à 0,30 g/l, moins de un élément par mm3
B - Protéinorachie à 0,30 g/l, 300 éléments par mm3
C - Protéinorachie à 0,15 g/l, moins de un élément par mm3
D - Abaissement des sucres et des chlorures
E - Protéinorachie à 0,90 g/l, moins de un élément par mm3

Après la réalisation de la radiculographie, les signes ayant justifiés la réalisation en urgence de cet examen
persistent. Indiquez quelle sera votre attitude thérapeutique :
A - Infiltration intra-rachidienne de corticoïdes
B - Elongations vertébrales
C - Pose d'un corset plâtré et surveillance médicale
D - Intervention chirurgicale rapide
E - Chimionucléolyse
207
Exclusivement sur DOC - DZ : www.doc-dz.com NADJI 85
RESIDANAT EN POCHE TOME II
Cas Clinique en QCM
A la suite d'un accident, le conducteur, âgé de 40 ans, est retrouvé éjecté de son véhicule, conscient, mais incapable d'un
mouvement volontaire de ses quatre membres. Il n'avait, dans son véhicule, ni appui-tête, ni ceinture de sécurité.
A un premier examen rapide de ses membres, on ne retrouve aucune plaie ni aucune désaxation majeure.

Vous êtes de garde et vous recevez ce blessé. Si le SAMU l'a correctement transporté, vous devez constater
que :
A - Il a le cou en hyper-extension
B - Il est en position latérale de sécurité
C - Il est sur le dos avec un coussin sous la tête
D - Il est à plat ventre avec un coussin sous le menton
E - Il est en alignement tête-cou-tronc

Parmi ces propositions, laquelle ou lesquelles acceptez-vous ?


A - Vous le retournez pour palper les apophyses épineuses
B - Vous recherchez des signes de section médullaire complète
C - Vous vérifiez les amplitudes de mobilité du rachis cervical
D - Vous demandez des clichés standard du crâne et face et profil de tout le rachis
E - Vous demandez des clichés bouche ouverte du rachis cervical supérieur

Les signes cliniques présents vont orienter votre diagnostic topographique vers :
A - Les masses latérales de l'atlas
B - L'odontoïde
C - Le rachis cervical inférieur de C3 à C7
D - La charnière cervico-dorsale
E - La charnière dorso-lombaire

Les premiers clichés effectués ne montrent pas de fracture sur la localisation anatomique suggérée par
l'examen neurologique. Il faut alors rechercher :
A - Un traumatisme crânien passé inaperçu avec lésions neurologiques
B - Une entorse grave C1-C2 réduite
C - Une luxation D12-L1
D - Une entorse grave entre C3 et C7 réduite
E - Une thrombose des artères vertébrales

Pour affirmer votre diagnostic, lequel ou lesquels de ces examens complémentaires radiologiques vous
paraissent utile ?
A - Un scanner crânien
B - Une artériographie des troncs supra-aortiques
C - Un scanner rachidien
D - Des épreuves dynamiques
E - Une myélographie

Une femme de 68 ans mesurant 1,60m et pesant 48 kg se plaint depuis dix jours d'une douleur dorsale basse, apparue
brutalement sans cause évidente, très vive, rendant la station debout presque impossible, soulagée en partie par le décubitus.
Elle a souffert dans l'adolescence d'une pleurésie tuberculeuse, a subi 30 ans auparavant une thyroïdectomie pour Basedow
et peu après une hystérectomie pour fibrome ; enfin deux ans plus tôt, elle a été traitée pour une fracture de l'extrémité
inférieure du radius.
L'examen clinique montre une douleur localisée vers la région dorso-lombaire ainsi qu'une raideur rachidienne à ce niveau. Le
reste de l'examen clinique est normal (température à 37°C).
Après un certain nombre d'examens para-cliniques, le diagnostic d'ostéoporose commune est établi.

Ce diagnostic serait compatible avec :


A - Une hypophosphorémie
B - Une vitesse de sédimentation normale
C - Un pic monoclonal à l'électrophorèse
D - Des phosphatases alcalines à 250 U (N< ou = à 70 U)
E - Une calcémie à 2,8 micromoI/I

208
Exclusivement sur DOC - DZ : www.doc-dz.com NADJI 85
RESIDANAT EN POCHE TOME II
Cas Clinique en QCM

Les clichés radiologiques devraient montrer :


A - Une vertèbre borgne
B - Une vertèbre en cadre
C - Un tassement vertébral
D - Un plateau vertébral estompé
E - Des géodes intra-somatiques

La biopsie osseuse iliaque confirmerait le diagnostic en montrant :


A - Des îlots plasmocytaires
B - Une hyperostéoïdose avec ralentissement de la minéralisation
C - Une augmentation des surfaces de résorption ostéoclastique
D - Une diminution du volume trabéculaire osseux
E - Une structure osseuse lamellaire en mosaïque

Quel(s) facteur(s) de risque pour l'ostéoporose chez cette patiente retenez-vous dans l'observation ?
A - L'âge = 68 ans
B - Les antécédents de maladie de Basedow
C - L'hystéro-ovariectomie à 38 ans
D - Le poids pour la taille
E - Les antécédents de tuberculose pleurale

Pour affirmer le diagnostic d'ostéoporose, vous pensez qu'il est nécessaire de demander en plus :
A - Une détermination du calcium total de l'organisme
B - Un examen tomodensitométrique du rachis
C - Une radiographie du bassin
D - Une scintigraphie osseuse
E - Aucun des examens précédents

Pour traiter l'ostéoporose de cette malade vous donnerez :


A - Une corticothérapie per os 0,1 mg/kg
B - Du fluorure de sodium 5 mg/j
C - De la vitamine D 400 U/j
D - Du calcium 1 g/j
E - Du disphosphonate 1 mg/24h

Non traitée, l'évolution de cette ostéoporose comporte un risque de :


A - Fracture du col fémoral
B - Rétrécissement du canal rachidien
C - Nouveaux tassements de vertèbres
D - Fractures de côtes
E - Compression médullaire

Un homme de 70 ans se plaint de céphalées persistantes et d'une surdité progressive. Il souffre depuis trois mois de
lombalgies mécaniques modérées et depuis 15 jours de douleurs importantes de la cuisse droite. A l'examen le rachis
lombaire est enraidi, peu douloureux. La jambe gauche est le siège d'une incurvation dans les deux plans, avec genu varum.
La température cutanée en regard est augmentée.
Les examens biologiques montrent une N.F.S., une V.S., une électrophorèse sanguine et urinaire normales, une calcémie à
100 mg/l, une phosphorémie à 32 mg/l, une calciurie à 155 mg/24 h, des phosphatases acides à 5,5 UI/l (normale 3-8), des
phosphatases alcalines à 450 UI/l (normale 80-220 UII/l); une hydroxyprolinurie à 95 mg/24 h (normale 40 mg/24 h).
Les radiographies objectivent des discopathies lombaires étagées avec pincement discal L4-L5, une ostéophytose lombaire,
une hypertrophie avec élargissement des corticales du fémur droit et du tibia gauche, une fissure de la convexité du fémur
droit, un aspect fibrillaire anarchique de l'hémi-bassin droit, une condensation avec hypertrophie des os de la base du crâne,
une ostéoporose circonscrite fronto-pariétale droite.
Le diagnostic de maladie de Paget est retenu.

Quel(s) élément(s) clinique(s) peut(peuvent) être en rapport avec la maladie de Paget ?


A - Céphalées persistantes
B - Hypo-acousie
C - Lombalgies
D - Augmentation de la température locale de la jambe gauche
E - Douleurs de la cuisse droite 209
Exclusivement sur DOC - DZ : www.doc-dz.com NADJI 85
RESIDANAT EN POCHE TOME II
Cas Clinique en QCM

Quel(s) paramètre(s) biologique(s) témoigne(nt) de l'évolutivité de la maladie de Paget ?


A- Calcémie à 100 mg/l
B - Phosphorémie à 2 mg/l
C - Hydroxyprolinurie à 95 mg/24 h
D - Phosphatases acides à 5,5 UI/l
E - Phosphatases alcalines à 450 UI/l

Parmi les signes radiologiques, quel(s) est celui ou ceux en faveur de la maladie de Paget ?
A - Discopathies lombaires étagées
B - Fissure fémorale droite
C - Hypertrophie des corticales
D - Aspect fibrillaire anarchique de la trame
E - Ostéoporose circonscrite du crâne

Quel(s) médicament(s) anti-ostéoclastique(s) pouvez-vous proposer chez ce patient ?


A- Calcium
B - Phosphore
C - Calcitonine
D - Vitamine D
E - Diphosphonate

Quel(s) paramètre(s) biologique(s) devez-vous surveiller pour juger l'efficacité de votre traitement ?
A - V.S.
B - Calcémie
C - Phosphatases alcalines
D - Phosphorémie
E - Hydroxyprolinurie

Monsieur A..., 60 ans, est hospitalisé pour des lombalgies invalidantes depuis 15 jours, et résistantes aux antalgiques
habituels. Il y a deux mois, il a subi deux cystoscopie pour une hématurie. Un polype a été enlevé et l'examen histologique a
conclu à sa bénignité. Une U.I.V. élimine toute cause rénale. Quelques jours après la seconde exploration endoscopique,
monsieur A... a présenté des frissons, un accès fébrile à 38°C, régressif sous antiseptique urinaire.
Lors de son hospitalisation, la température est à 40°C, le faciès altéré, il se mobilise mal. La palpation réveille une douleur en
L4-L5. Le reste de l'examen s'avère normal.
La V.S. est à 100 mm à la première heure, les globules blancs sont à 18 000 dont 80 % de Polynucléaires Ionogrammes
sanguin et urinaire sont normaux.

La radiographie standard du rachis lombaire montre une rectitude rachidienne. Le plateau supérieur de L5 est
effacé. Le premier diagnostic à évoquer est :
A - Fracture sur ostéome décalcifiante
B - Maladie de Kahler
C - Spondylodiscite à germes banals
D - Pott lombaire
E - Métastase vertébrale

Une exploration biologique s'impose d'emblée. Laquelle ?


A - Sérodiagnostic de Wright
B - Dosage des phosphatases acides prostatiques
C - Dosage de la calcémie
D - Hémoculture
E - Dosage de l'hydroxyprolinurie

Les examens apportent peu de renseignements. Quel examen complémentaire allez-vous prescrire pour
confirmer votre diagnostic ?
A - Echotomographie
B - Exploration tomodensitométrique
C - Scintigraphie au pyrophosphates marqués au TC99m
D - Ponction discale
E - Xéroradiographie

210
Exclusivement sur DOC - DZ : www.doc-dz.com NADJI 85
RESIDANAT EN POCHE TOME II
Cas Clinique en QCM

Pendant la période diagnostique, on doit proposer au malade :


A - Un repos strict au lit
B - L'entretien de la trophicité des membres par une rééducation appropriée
C - Un plâtre
D - Une coquille plâtrée
E - L'obligation de marcher

Quel(s) examen(s) complémentaire(s) allez-vous demander, avant toute décision thérapeutique et quel que soit
le germe en cause ?
A - Créatinine
B - Transaminases
C - Electrophorèse des protides
D - Sérologie rhumatoïde
E - Electromyogramme

Une femme de 65 ans active, présente à la suite d'une chute de mobylette, une fracture cervicale vraie du fémur dont le trait
est oblique en bas et en dedans. Le déplacement ouvre l'angle cervico-céphalique (20°).
De profil, le déplacement est pratiquement nul. Il n'y a pas d'arthrose associée.
L'impotence fonctionnelle est d'ailleurs modérée et permet à cette femme de décoller le talon du lit. La marche cependant est
très douloureuse. L'état général de la blessée est bon et permet une anesthésie générale de longue durée.

Il s'agit d'une fracture :


A - Garden I Pauwels I
B - Garden I Pauwels II
C - Garden II Pauwels I
D - Garden II Pauwels II
E - Garden III Pauwels III

Cette fracture est d'un bon pronostic :


A - Car elle est stable
B - Car elle prédispose a la consolidation
C - Car elle ne se complique habituellement pas de nécrose
D - Car la réduction est facile sur table orthopédique
E - Car elle survient chez une femme en bon état général

La fracture de cette blessée pourrait faire l'objet :


A - D'une ostéosynthèse en place par vis
B - D'une ostéosynthèse après réduction par vis
C - D'une arthroplastie d'emblée
D - D'une abstention thérapeutique sans autoriser l'appui avec surveillance radiologique régulière
E - D'une ostéosynthèse en place par clous de Ender

Dans l'hypothèse de la survenue d'une nécrose aseptique de la tête fémorale fonctionnellement très gênante :
A - Il faut réaliser une arthroplastie par prothèse totale
B - Il faut réaliser une ostéotomie de Stewart
C - Il faut réaliser une résection tête et col
D - Il faut réaliser une simple ablation du matériel et attendre l'évolution
E - Il faut attendre la consolidation de la fracture et surveiller l'évolution

Un menuisier de 25 ans, droitier, fait une fausse manoeuvre et est blessé à la main droite par une scie circulaire. Vous êtes de
garde et vous faites un premier bilan des lésions :

Au niveau du pouce, vous constatez une plaie palmaire en regard de l'articulation métacarpo-phalangienne
avec une perte de la flexion de P2 sur P1. Vous pensez à une section :
A - Du rameau thénarien du médian
B - Du court fléchisseur du pouce
C - Du fléchisseur superficiel du pouce
D - Du long fléchisseur propre du pouce
E - Du premier interosseux palmaire
211
Exclusivement sur DOC - DZ : www.doc-dz.com NADJI 85
RESIDANAT EN POCHE TOME II
Cas Clinique en QCM

Une autre plaie traverse la paume à la base de l'index et du médius. Ceux-ci sont insensibles à la piqûre. Vous
pensez à une atteinte :
A - De la branche profonde du nerf cubital
B - Des rameaux récurrents de la branche superficielle du nerf radial
C - Des nerfs interosseux et/ou de leurs branches collatérales
D - De l'arcade radio-cubitale superficielle
E - Aucune des réponses précédentes

Il y a une plaie en regard de la face palmaire P2. Le médius a gardé une flexion active de P2 sur P1 mais a
perdu celle de P3 sur P2. Vous évoquez une section :
A - Du fléchisseur superficiel du médius
B - Du fléchisseur profond du médius
C - Des deux tendons fléchisseurs du médius
D - Des interosseux et lombricaux du médius
E - Des nerfs collatéraux

L'annulaire a été sectionné par le coup de scie au niveau de P2. Pour envisager une réimplantation par
microchirurgie, il faut :
A - Un délai écoulé de moins d'une heure
B - Un conditionnement du doigt dans un bain de sérum glacé
C - Un conditionnement dans un sachet isolant et de la glace
D - Une section franche
E - La possibilité de réparer au moins une veine

Une radiographie montre que le trapèze présente une fracture comminutive. Vous prévoyez des conséquences
sur :
A - La stabilité du deuxième métacarpien
B - L'ouverture de la première commissure
C - La pince pouce-index
D - L'inclinaison radiale de la main
E - La flexion de la métacarpo-phalangienne du pouce

A la suite d'un accident de la circulation, le conducteur, âgé de 40 ans, est retrouvé éjecté de son véhicule, conscient, mais
incapable d'un mouvement volontaire de ses quatre membres. Il n'avait dans son véhicule ni appui-tête ni ceinture de sécurité.
A un premier examen rapide de ses membres, on ne retrouve aucune plaie,.ni aucune désaxation majeure.

Comment doit-on transporter ce blessé jusqu'à l'hôpital voisin ?


A - En position latérale de sécurité
B - Sur le dos
C - Le cou en extension
D - Le cou en traction
E - Immobilisé par une minerve

C'est vous qui êtes de garde et qui recevez ce blessé. Vous allez :
A - Le retourner avec précaution pour examiner tout son rachis
B - Vous limiter à un examen neurologique succint
C - Rechercher avec précision des signes de section médullaire complète
D - Demander un bilan radiologique complet du rachis
E - Vous limiter à l'étage rachidien suggéré par l'examen clinique

Lorsque vous examinerez les clichés, la présence d'une tétraplégie vous poussera à rechercher tout
particulièrement :
A - Une fracture des masses latérales de l'atlas
B - Une fracture de l'odontoïde
C - Une entorse grave C1 C2
D - Une luxation-fracture du rachis cervical inférieur
E - Une entorse grave du rachis cervical inférieur

212
Exclusivement sur DOC - DZ : www.doc-dz.com NADJI 85
RESIDANAT EN POCHE TOME II
Cas Clinique en QCM

Ce bilan radiologique est insuffisant pour conclure. Vous devez demander complémentairement :
A - Des tomographies
B - Un scanner
C - Une myélographie
D - Des épreuves dynamiques
E - Aucun des examens ci-dessus

Le diagnostic d'entorse grave cervicale ayant été retenu comme cause des troubles neurologiques, quel
traitement proposez-vous ?
A - Mettre le blessé en observation et suivre l'évolution neurologique
B - Mettre une minerve plâtrée et fixer secondairement le rachis
C - Tenter une réduction en urgence par traction
D - Traiter chirurgicalement la lésion
E - Aucune de ces propositions n'est valable

Un homme de 28 ans souffre depuis huit jours de douleurs polyarticulaires avec céphalées, asthénie, fièvre à 38°C. L'examen
montre un gonflement douloureux du genou gauche et une éruption morbilliforme fugace apparaissant le soir à l'acmé de la
fièvre.

Si vous envisagez le diagnostic d'hépatite virale, vous retiendrez en sa faveur :


A - L'éruption vespérale
B - La fièvre
C - La découverte d'une grosse rate
D - La découverte d'adénopathies
E - L'apparition d'une urticaire

Si vous envisagez le diagnostic de F.L.R. (Fiessinger-Leroy-Reiter) vous retiendrez en sa faveur :


A - L'éruption
B - L'apparition d'une conjonctivite
C - La découverte d'une splénomégalie
D - La découverte d'adénopathies
E - La découverte d'une balanite

Si vous envisagez le diagnostic de syndrome de Lِfgren (dans le cadre d'une sarcoïdose), vous retiendrez en sa
faveur :
A - L'éruption vespérale
B - La découverte de nodosités des membres inférieurs
C - La découverte d'une opacité pulmonaire segmentaire
D - La découverte d'un ganglion hilaire
E - La découverte d'un frottement péricardique

Si vous envisagez le diagnostic de rhumatisme gonococcique vous retiendrez en sa faveur :


A - L'eruption morbilliforme du soir
B - L'apparition de pustules à centre hémorragique du tronc
C - L'apparition d'une kératose palmoplantaire
D - La constatation de placards érythrosquameux des coudes
E - L'apparition d'une urticaire

Si vous envisagez le diagnostic de RAA vous retiendrez en sa faveur :


A - L'éruption vespérale
B - L'apparition d'adénopathie
C - L'apparition d'un frottement péricardique
D - L'apparition d'une splénomégalie
E - L'apparition d'une iritis

213
Exclusivement sur DOC - DZ : www.doc-dz.com NADJI 85
RESIDANAT EN POCHE TOME II
Cas Clinique en QCM

Si vous envisagez le diagnostic de polyarthrite lupique au début, vous retiendrez en sa faveur :


A - La découverte d'une splénomégalie
B - La survenue d'une lucite
C - La découverte d'un frottement péricardique
D - La découverte d'une kératose palmoplantaire
E - La survenue d'une iridocyclite

Un homme de 50 ans, après avoir souffert à plusieurs reprises d'un torticolis, se plaint d'une douleur qui part de la fosse sus
épineuse, et irradie de la face postérieure de l'épaule, au bord externe du bras et de l'avant bras, pour atteindre le pouce et
l'index.

Le siège de la douleur permet un diagnostic topographique. Lequel ?


A - Nerf médian
B - Racine C5
C - Racine C6
D - Racine C7
E - Racine C8

L'examen pourrait montrer un déficit moteur :


A - Du biceps
B - Du long supinateur
C - Du triceps
D - Du court abducteur du pouce
E - Des interosseux

On recherchera aussi une modification (diminution ou abolition) :


A - Du réflexe bicipital
B - Du réflexe stylo-radial
C - Du réflexe cubito-pronateur
D - Du réflexe tricipital
E - Du réflexe palmo-mentonnier

En fait, il n'y a ni déficit moteur ni modification des réflexes dans ce cas particulier. Les radiographies simples du
rachis cervical montrent des images de cervicarthrose, banales à cet âge. Quel diagnostic vous paraît le plus
probable ?
A - Syndrome du canal carpien
B - Névralgie cervico-brachiale vertébrale commune
C - Syndrome du canal de Guyon
D - Neurinome cervical
E - Syndrome du défilé costo-claviculaire

Un diagnostic de myélome est porté chez un homme de 65 ans.


Les examens pratiqués montrent :
- V.S. = 120 1ère heure ;
- électrophorèse du sérum à la : bande étroite identifiée comme IgG Kappa à l'immunoélectrophorèse. L'évaluation de la
dysglobulinémie est de 42 g/l. Les autres Ig sont à : IgA 0,20 g/l et IgM 0,15 g/l
- dans les urines, on trouve une protéinurie de Bence Jones : excrétion de chaîne Kappa à 4,3 g/24 heures
- sur l'hémogramme on relève : hémoglobine = 81 g/l ; VGM = 88 fl, réticulocytes 12.10 exposant 9/l, leucocytes et plaquettes
normaux
- par ailleurs :
créatininémie : 180 micromoI/l
uricémie : 660 micromoI/l
calcémie : 2.3 millimoI/l
- le myélogramme montre une moelle riche avec une plasmocytose à 34 %.

Parmi les examens suivants, lequel ou lesquels vous paraît(paraissent) nécessaire(s) pour compléter les
explorations ?
A - Scintigraphie osseuse
B - Radiographie du squelette
C - Lymphographie
D - Echographie abdominale
E - Ponction biopsie hépatique

214
Exclusivement sur DOC - DZ : www.doc-dz.com NADJI 85
RESIDANAT EN POCHE TOME II
Cas Clinique en QCM

Quel examen supplémentaire vous paraît le plus important pour expliquer l'anémie ?
A - Test de Coombs
B - Durée de vie des globules rouges
C - Mesure isotopique du volume globulaire total
D - Dosage du fer sérique
E - Recherche d'agglutinine irrégulière

Parmi les critères suivants, lequel ou lesquels a(ont) une valeur pronostique ?
A - VS = 120
B - Uricémie à 660 micromoI/l
C - Hémoglobine : 81 g/l
D - Créatininémie : 180 micromoI/l
E - Protéinurie de Bence Jones : 4,3 g/24 h

Un traitement associant Alkéran® et Prednisone® est donné en cures mensuelles de 4 jours. Parmi les critères
suivants, lequel vous paraît le plus valable pour juger de l'efficacité du traitement ?
A - Mesure de l'immunoglobuline monoclonale
B - Recalcification osseuse
C - Créatininémie
D - Dosage de l'hémoglobine sur l'hémogramme
E - Taux de plasmocytes dans la moelle osseuse

Parmi les affirmations suivantes concernant l'évolution sous ce traitement proposé, laquelle ou lesquelles
est(sont) exacte(s) ?
A - La guérison ne peut être attendue
B - Une réponse objective est attendue dans 2/3 des cas
C - Le traitement fait courir un risque de leucémie aiguë
D - La durée de survie médiane probable est de 10 ans
E - Le traitement augmente le risque d'amylose

Madame X., 70 ans est traitée depuis 3 ans pour une insuffisance cardiaque par 1 comprimé de Digoxine® les jours pairs et 1
comprimé d'un diurétique thiazidique (Rénèse® : polythiazide) les jours impairs. Son état cardiaque est globalement stabilisé
avec ce traitement. Cette malade vient consulter pour des douleurs des épaules et des cuisses avec gêne fonctionnelle
importante, apparues progressivement il y a 3 mois. Ces douleurs sont permanentes mais plus marquées la nuit et la raideur
est surtout matinale.
Depuis 15 jours, existent des céphalées lancinantes et pulsatiles. L'amaigrissement est d'environ 10 kg en trois mois.
L'examen clinique met en évidence une limitation douloureuse des mouvements des épaules et des hanches, mais la
palpation de l'artère temporale gauche conduit à déceler un cordon indure et douloureux au toucher. Le reste de l'examen
clinique est normal.
La V.S. est à 100 mm à la 1ère heure. Il existe une anémie à 90 g/l d'hémoglobine, normochrome, normocytaire et
arégénérative. Le bilan biologique est par ailleurs normal. Les radiographies du thorax, des épaules et des coxofémorales ne
mettent en évidence aucune anomalie.
Ainsi, chez cette malade atteinte d'une insuffisance cardiaque, l'on retient le diagnostic de pseudo-polyarthrite rhizomélique,
associée à une maladie de Horton, confirmée par la biopsie de l'artère temporale gauche,
Vous optez pour la corticothérapie à 1 mg/kg/jour.

En l'absence du traitement précité, quel est le risque classiquement redouté encouru par cette patiente ?
A - Accentuation des douleurs des épaules et des hanches
B - Surdité gauche
C - Oblitération artérielle iliaque
D - Amaurose
E - Hémiplégie droite

Parmi les effets adverses suivants, quel(s) est(sont) celui(ceux) qu'il y a lieu de redouter chez cette patiente
ainsi traitée ?
A - Protéinurie
B - Neutropénie
C - Hypokaliémie
D - Hyperglycémie
E - Augmentation du pH gastrique

215
Exclusivement sur DOC - DZ : www.doc-dz.com NADJI 85
RESIDANAT EN POCHE TOME II
Cas Clinique en QCM

Au bout d'un an de traitement, la corticothérapie entraîne des effets secondaires : pour y pallier on opte pour
une première association aux antimalariques de synthèse. Quel(s) nouveau(x) effet(s) secondaire(s) faut-il
redouter avec l'ensemble de son traitement ?
A - Rétinopathie
B - Bloc auriculo-ventriculaire
C - Myopathie
D - Pigmentation cutanée
E - Hyponatrémie

Votre choix s'est porté sur un anti-inflammatoire non stéroïdien (Indometacine-Indocid®) après avoir interrompu
les antimalariques. Quel(s) nouveau(x) effet(s) secondaire(s) faut-il redouter avec l'ensemble de son traitement ?
A - Oedèmes des membres inférieurs
B - Elévation de la créatinine
C - Reprise des céphalées
D - Aggravation de l'anémie
E - Syndrome de Raynaud

Votre choix s'est porté sur la Disulone® (Dapsone) après avoir interrompu les AINS. Quel(s) nouveau(x) effet(s)
secondaire(s) faut-il redouter avec l'ensemble de son traitement ?
A - Hirsutime
B - Méthémoglobinémie
C - Anémie hémolytique
D - Cyanose des extrémités
E - Phemphigus bulleux

Un skieur fait une chute qui entraîne son bras en abduction rotation externe. La douleur est violente, il ressent un
engourdissement du moignon de l'épaule : au cours du transfert au poste de secours, sa main enfle et se cyanose.

Le diagnostic de luxation antéro-interne de la scapulo-humérale est orienté par :


A - Il est possible de ramener le coude contre le corps
B - Il est impossible d'effectuer une rotation interne
C - Une rotation imprimée au coude ne se transmet pas à la tête humérale
D - Il y a un vide sous-acromial externe
E - Il y a précocement une importante ecchymose de la paroi thoracique

Vous retrouvez lors d'un examen systématique régional, une insensibilité à la piqûre au moignon de l'épaule,
sur sa face externe. Vous évoquez alors une atteinte :
A - Du nerf sus-scapulaire
B - De la racine C7 du plexus brachial
C - Du nerf circonflexe
D - Du nerf musculo-cutané
E - De la branche superficielle du nerf spinal

La constatation d'un oedème avec cyanose fait rechercher :


A - Un arrachement du confluant veineux jugulo sous-clavier
B - Une compression de la veine céphalique
C - Une ischémie par rupture de l'artère axillaire
D - Une compression de la veine axillaire
E - Un réflexe vaso-constricteur distal

Quelle (quelles) association(s) lésionnelle(s) fréquente(s) allez-vous rechercher sur la radiographie de face de
l'épaule droite ?
A - Une fracture de la clavicule
B - Une fracture du trochiter
C - Une fracture de l'acromion
D - Une fracture de la coracoïde
E - Une fracture de la glène de l'omoplate

216
Exclusivement sur DOC - DZ : www.doc-dz.com NADJI 85
RESIDANAT EN POCHE TOME II
Cas Clinique en QCM

Après réduction, vous conseillez :


A - Une semaine d'immobilisation
B - Trois semaines d'immobilisation
C - Une rééducation immédiate en rotation externe
D - Un plâtre thoraco-brachial pour un mois
E - Une écharpe coude au corps

Ce blessé est suivi régulièrement. Au 4e mois, son deltoïde bien rééduqué se contracte très correctement,
l'abduction active reste limitée à 15°, alors qu'en passif, elle atteint 90° sans difficulté.
Quel diagnostic évoquez-vous ?
A - Une capsulite rétractile inférieure
B - Une rupture du tendon du sus-épineux
C - Une rupture du bourrelet glénoïdien
D - Une rupture du tendon du long biceps
E - Une séquelle de paralysie circonflexe

Une femme de 58 ans vient consulter pour des troubles de la marche avec une grande fatigue et des douleurs diffuses des
membres inférieurs à la marche. Elle a beaucoup maigri et souffre d'une diarrhée. L'examen clinique révèle une cyphose
dorsale mais le rachis est souple. Malgré une démarche en canard la mobilité des articulations des membres inférieurs est
normale. La malade a des difficultés à se lever d'une chaise. La radiographie vertébrale et du bassin met en évidence une
importante déminéralisation, avec aspect biconcave des corps vertébraux. Le diagnostic d'ostéomalacie est retenue. Elle a été
traitée pendant 6 mois par 50 mg/24 h de fluorure de calcium.

Un diagnostic d'ostéomalacie a pu être porté sur le résultat de :


A - Une vitesse de sédimentation
B - Un dosage de la calcémie et de la phosphorémie,dans le sang et sur urines de 24 heures
C - Un dosage de la parathormone
D - Un test d'hypocalcémie à la calcitonine
E - Un dosage de la vitamine D

Chez cette malade diarrhéique, on a évoqué le diagnostic de maladie coeliaque. Quels sont les 2 examens
nécessaires au diagnostic ?
A - Une coloscopie
B - Un dosage de la vitamine D (250H D 3) circulante
C - Un test au D-Xylose
D - Une fibroscopie gastrique avec biopsie jéjunale
E - Une ponction biopsie du foie

Parmi les situations suivantes, laquelle ou lesquelles peut(vent) se compliquer d'ostéomalacie ?


A - Ulcère gastrique traité par Cimétidine (Tagamet®)
B - Obstruction biliaire prolongée
C - Réaction étendue du grêle
D - Rectocolite hémorragique
E - Cancer colique

On peut attribuer à une affection néoplasique ou paranéoplasique :


A - L'amaigrissement
B - La diarrhée
C - Les douleurs osseuses
D - La calcémie à 28 mml/l, la phosphorémie à 0,7 mmol/l
E - Une hydroxyprolinurie normale

Le fluor utilisé 6 mois chez cette patiente peut être responsable :


A - De l'ostéomalacie
B - De l'amaigrissement
C - De la diarrhée profuse
D - D'une calcémie à 2 mm/l
E - D'une phosphorémie à 0. 7 mml

217
Exclusivement sur DOC - DZ : www.doc-dz.com NADJI 85
RESIDANAT EN POCHE TOME II
Cas Clinique en QCM

Les fissures de Looser et Milkmann sont à rechercher préférentiellement chez cette malade sur :
A - Le 1/3 externe des clavicules
B - Le calcanéum
C - Les branches ilio-pubiennes
D - Les métacarpiens
E - La branche montante du maxillaire inférieur

Une ostéomalacie par entéropathie au gluten étant retenue quel(s) traitement(s) va-t-on proposer ?
A - Calcium à fortes doses en perfusion intraveineuse
B - Phosphore 1 g par jour
C - Dédrogyl® 30 gouttes par jour
D - Régime sans gluten
E - Régime pauvre en graisses

Un menuisier de 25 ans, droitier, fait une fausse manoeuvre et est blessé à la main droite par une scie circulaire. Vous êtes de
garde et vous faites un premier bilan des lésions.

Au niveau du pouce, vous constatez une plaie palmaire en regard de l'articulation métacarpophalangienne avec
une perte de la flexion de P2 sur P1. Vous pensez à une section :
A - Du rameau thénarien du médian
B - Du court fléchisseur du pouce
C - Du fléchisseur du pouce
D - Du long fléchisseur propre du pouce
E - Du premier interosseux palmaire

Une autre plaie traverse la paume à la base de l'index et du médius. Ceux-ci sont insensibles à la piqûre. Vous
pensez à une atteinte :
A - De la branche profonde du nerf cubital
B - Des rameaux récurrents de la branche superficielle du nerf radial
C - Des nerfs interosseux et/ou de leurs branches collatérales
D - De l'arcade radio-cubitale superficielle
E - Aucune des réponses précédentes

Il y a une plaie en regard de P1 du médius et le médius à gardé une flexion de P2 sur P1 mais à perdu celle de
P3 sur P2. Vous évoquez une section :
A - Du fléchisseur superficiel du médius
B - Du fléchisseur profond du médius
C - Des deux tendons fléchisseurs du médius
D - Des interosseux et lombricaux du médius
E - Des nerfs collatéraux

L'annulaire a été sectionné par le coup de scie au niveau de P1. Pour envisager une réimplantation par
microchirurgie, il faut :
A - Un délai écoulé de moins d'une heure
B - Un conditionnement du doigt dans un bain de sérum glacé
C - Un conditionnement dans un sachet isolant et de la glace
D - Une section franche
E - La possibilité de réparer au moins une veine

Une radiographie montre que le trapèze présente une fracture comminutive. Vous prévoyez des conséquences
sur :
A - La stabilité du deuxième métacarpien
B - L'ouverture de la première commissure
C - La pince pouce-index
D - L'inclinaison radiale de la main
E - La flexion de la métacarpo-phalangienne du pouce .

218
Exclusivement sur DOC - DZ : www.doc-dz.com NADJI 85
RESIDANAT EN POCHE TOME II
Cas Clinique en QCM
Une femme de 78 ans, pesant 80 kilos pour 1,68m, diabétique traitée par 2 comprimés quotidiens de Metformine
(Glucophage®) consulte pour une douleur inguinale droite apparue spontanément 3 jours plus tôt, lancinante non calmée par
le décubitus, d'intensité croissante, s'accompagnant d'une fébricule à 38° le matin, 38°5 le soir, sans frisson. Elle a pour
antécédents une coxarthrose bilatérale peu invalidante et une crise unique de chondrocalcinose articulaire ayant-touché le
poignet gauche 2 mois plus tôt, et ayant évolué favorablement sous anti-inflammatoires non stéroïdiens. L'examen clinique
permet de noter : 1) L'existence d'une douleur à la pression de la tête fémorale droite. 2) Une limitation douloureuse des
mouvements passifs de la hanche, par comparaison au côté opposé. Il n'y a ni tuméfaction locale, ni rougeur, ni exagération
du lacis veineux. 3) Il existe à la plante du pied droit une excoriation cutanée que la patiente a provoquée en procédant à
l'ablation énergique d'une hyperkératose banale. Les radiographies du bassin et de la hanche droite sont normales. La
numération formule sanguine donne : 13,4 g d'hémoglobine/100 ml, 11 500 blancs dont 78 % de polynucléaires neutrophiles,
3 % d'éosinophiles, 19 % de Iymphocytes. La vitesse de sédimentation est à 34 mm à la première heure.

Quel(s) diagnostic(s) peu(ven)t être envisagé(s) ?


A - Crise de chondrocalcinose
B - Monoarthrite rhumatismale de début aigu
C - Poussée de coxarthrose
D - Arthrite septique
E - Ostéonécrose aseptique de la tête fémorale

Parmi les examens suivants lequel vous semble utile pour préciser en urgence l'étiologie ?
A - Test au Latex et réaction de Waaler-Rose
B - Scintigraphie osseuse
C - Tomographies de l'articulation coxo-fémorale
D - Dosage de la calcémie
E - Ponction articulaire de la hanche droite

Si la patiente vous demande un délai de réflexion de 48 heures, vous pouvez prescrire parmi les médicaments
suivants :
A - Sels d'or
B - Antibiotiques anti-staphylococciques
C - Antibiotiques à large spectre
D - Antalgiques purs
E - Corticoïdes

Pour ce délai de 48 heures, quelle attitude faut-il recommander à la patiente ?


A - Faire de la rééducation en décharge
B - Faire de l'électrothérapie
C - Marcher avec des cannes anglaises sans appuyer le membre inférieur droit
D - Marcher à sa convenance
E - S'obliger à marcher

Une femme de 72 ans est adressée par son médecin traitant en milieu hospitalier pour altération récente de l'état général
avec asthénie, amaigrissement d'environ 6 kg au cours des 3 derniers mois, état subfébrile quasi permanent (37,8° - 38° C)
depuis plusieurs semaines, céphalées à type de brûlures superficielles prédominant dans la région temporale gauche et algies
massétérines gauches déclenchées par la mastication. S'y associent quelques algies scapulaires bilatérales avec
enraidissement douloureux de tout le rachis cervical, ainsi que des douleurs des hanches. A l'examen clinique, l'humeur est
dépressive, l'état général apparaît altéré, le teint pâle. Discret souffle systolique apexien à l'auscultation cardiaque.
Auscultation pleuro-pulmonaire normale. Pas de signe de phlébite. Pas d'oedème. T.A. 160/90. Absence d'adénopathie.
Abdomen souple : pas d'hépatosplénomégalie. La mobilisation des épaules et des hanches est douloureuse. L'examen
neurologique est normal. La pression de la région temporale gauche déclenche une douleur. Le revêtement cutanéo-muqueux
est d'aspect normal. Les examens biologiques récemment effectués à la demande de son médecin avaient donné les résultats
suivants : VS : 82/110, globules blancs : 12 400/mm3 dont 81 % de polynucléaires. Globules rouges : 3 300 000/mm3.
Hémoglobine 8,7 %, VGM : 73 micron3; uricémie : 82 mg/l ; alpha-foeto-protéines 3 ng/ml (normale inférieure à 15 ng/ml) ;
LDH : 200 micron/l (normale inférieure à 300), SGOT : 18 unités, SGPT : 22 unités, phosphatases alcalines : 240 unités,
A.S.L.O. : 200 unités, fibrinogènémie : 7,8 g °/oo ; CRP : 60 mg °/oo (normale : 15 mg °/oo), Haptoglobinémie : 3,5 g °/oo
(normale inférieure à 1, 5 9 °/oo)

Parmi les diagnostics évoqués, quel est celui que vous suggère ce tableau clinique et biologique ?
A - Thyroïdite subaiguë
B - Lupus érythémateux disséminé
C - Syndrome de Gougerot-sjِgren
D - Maladie de Horton
E - Sarcoïdose

219
Exclusivement sur DOC - DZ : www.doc-dz.com NADJI 85
RESIDANAT EN POCHE TOME II
Cas Clinique en QCM

Parmi les complications suivantes, retenez celle qui est susceptible d'être en rapport avec cette affection ?
A - Greffe d'une infection opportuniste à Pneumocystis carinii
B - Constitution progressive d'une fibrose pulmonaire interstitielle
C - Survenue d'une cécité par obstruction de l'artère centrale de la rétine ou des artères ciliées postérieures
D - Apparition d'un lymphome malin
E - Développement d'une hypertension artérielle maligne

Parmi les examens complémentaires suivants, une aide au diagnostic peut provenir du résultat de :
A - Biopsie des glandes salivaires accessoires
B - Recherche de l'antigène d'histocompatibilité HLA-B27
C - Examen Doppler des artères céphaliques superficielles
D - Biopsies pulmonaires transbronchiques
E - Biopsie de l'artère temporale

Parmi les propositions thérapeutiques figurant ci-après, retenez celle qui vous paraît appropriée au cas de cette
patiente ?
A - Corticothérapie : 0,5 à 1 mg/kg de Prednisone/jour puis réduction progressive ultérieure de la posologie
B - Antipaludéen de synthèse : Hydroxychloroquine 400 à 600 mg/jour
C - Immunosuppresseurs : cyclophosphamide 1 à 4 mg/kg/jour
D - Colchicine : 1 mg/jour
E - D-Pénicillamine : 600 mg/jour

Un homme de 55 ans, obèse, en pré-retraite, présente depuis plusieurs mois une douleur mécanique de l'aine, irradiant au
genou, limitant à moins d'un kilomètre son périmètre de marche. Il monte difficilement les escaliers et boite à la fatigue.

Le diagnostic de coxarthrose primitive est compatible avec la constatation de :


A - Limitation des amplitudes en rotation externe
B - Adénopathie de l'aine
C - Flexum de hanche
D - Absence de tout antécédent traumatique ou médical de cette articulation
E - VS, CRP, formule sanguine, normales

Quel est le meilleur examen complémentaire pour affirmer le diagnostic de coxarthrose ?


A - Arthrographie de hanche
B - Scanner
C - R.M.N.
D - Radiographies du bassin de face et centrées sur l'articulation de face et de faux profil
E - Scintigraphie au technetium

Chez ce patient quelle est l'attitude thérapeutique immédiate ?


A - Arthroplastie totale
B - Ostéotomie fémorale pour retarder au maximum l'heure de l'arthroplastie
C - Cure d'amaigrissement, hygiène orthopédique de hanche, rééducation, traitement médical
D - Infiltration de corticoïdes dans la coxo-fémorale
E - Ostéotomie du bassin

L'évolution peut se faire vers :


A - Stabilisation
B - Attitude vicieuse
C - Restitution ad integrum
D - Coxopathie destructrice
E - Luxation antérieure de la tête fémorale

220
Exclusivement sur DOC - DZ : www.doc-dz.com NADJI 85
RESIDANAT EN POCHE TOME II
Cas Clinique en QCM
Une femme de 70 ans, vivant seule chez elle, est retrouvée sur le sol dans sa cuisine par ses voisins 24 heures après une
chute d'un escabeau. Elle est consciente mais déshydratée. Elle est incapable de se relever du fait d'une impotence
fonctionnelle totale du membre inférieur droit. Celui-ci parait raccourci, en abduction et rotation externe. La température est de
38°5. Le médecin habituel de cette malade, appelé en urgence, demande son hospitalisation et note sur son certificat ses
antécédents : varices volumineuses bilatérales, insuffisance respiratoire chronique, coxarthrose droite évoluée.

Quel est de diagnostic clinique le plus probable ?


A - Luxation postéro-supérieure de hanche
B - Fracture de l'ischion
C - Fracture du cotyle
D - Fracture par arrachement du grand trochanter
E - Fracture du col du fémur

Le bilan radiologique que vous demandez en urgence comporte :


A - Bassin de face
B - Grands axes des membres inférieurs
C - Hanche droite face et profil
D - Clichés dynamiques de hanche droite en abduction et adduction
E - Radio pulmonaire

Quels) autre(s) examen(s) vous parai(ssen)t utile(s) ?


A - Epreuve cardiaque d'effort
B - Gaz du sang
C - Recherche du facteur rhumatoïde
D - Tests de coagulation
E - Scintigraphie osseuse

Parmi ces prescriptions, vous mettez immédiatement en oeuvre :


A - Une transfusion sanguine
B - Une rééquilibration ionique
C - Un anticoagulant
D - Une traction du membre
E - Une antibiothérapie à large spectre

La décision thérapeutique devrait être logiquement :


A - Une abstention chirurgicale
B - Une ostéosynthèse par vis
C - Une ostéosynthèse par clous de Ender
D - Une prothèse totale
E - Une prothèse céphalique

Si un acte est effectué, il paraît justifié d'envisager en post-opératoire :


A - Lutte anti-escarre
B - Kinésithérapie respiratoire
C - Traitement anticoagulant
D - Bandages élastiques des membres
E - Lever précoce

A la suite d'un effort de soulèvement d'une charge lourde, Mr X..., 35 ans ressent une douleur aiguë dans la région lombaire
bloquant le mouvement. Il doit s'aliter et 12 heures plus tard apparaît une douleur du membre inférieur droit intéressant la
fesse, la cuisse, la face externe de la jambe et atteignant le dos du pied. Le réflexe achilléen est conservé. On note un signe
de Lasègue à 20 degrés. Ce patient n'a jamais souffert auparavant de lombalgies.

Devant ce tableau quel est votre diagnostic ?


A - Sciatique tronculaire
B - Lombo-radiculite L 4
C - Lombo-sciatique L 5
D - Lombo-sciatique S 1
E - Sciatique cordonale

221
Exclusivement sur DOC - DZ : www.doc-dz.com NADJI 85
RESIDANAT EN POCHE TOME II
Cas Clinique en QCM

A ce stade quels sont les 2 examens complémentaires utiles ?


A - Radiographie standard du bassin et du rachis lombaire
B - Scintigraphie osseuse
C - Electromyogramme
D - Vitesse de sédimentation
E - Tomographies du rachis lombaire

Quelle mesure thérapeutique est essentielle pour obtenir une amélioration ?


A - Manipulations vertébrales
B - Corticothérapie
C - Gymnastique vertébrale précoce
D - Repos au lit strict
E - Vitamines B injectables

Si 15 jours de traitement médical n'améliorent pas la douleur, quels sont les deux examens complémentaires
capable de reconnaître la lésion responsable ?
A - Myélographie opaque dorsale
B - Scanner X lombaire
C - Potentiels évoqués somesthésiques
D - Sacco-radiculographie
E - Examen cyto-chimique du LCR

Quelles sont les deux complications neurologiques motrices possibles ?


A - Paralysie de la loge antéro-externe de jambe
B - Paralysie de la loge postérieure de jambe
C - Paralysie du quadriceps
D - Syndrome de la queue de cheval
E - Syndrome de compression médullaire

Une femme de 80 ans vient en consultation en urgence pour une baisse brutale de l'acuité visuelle de l'oeil droit. Elle se plaint
de céphalées et d'une altération de l'état général. L'examen ophtalmologique va révéler au fond d'oeil droit un oedème
ischémique de la papille optique. L'examen de l'oeil gauche est normal. L'examen des artères temporales superficielles les
montre indurées et non battantes, beaucoup plus grosses qu'habituellement. Vous posez le diagnostic de maladie de Horton
très probable.

Les complications oculaires de la maladie de Horton comportent :


A - L'ischémie aiguë de la tête du nerf optique
B - L'oedème papillaire de stase
C - L'occlusion de l'artère centrale de la rétine
D - L'occlusion de la veine centrale de la rétine
E - L'occlusion des capillaires rétiniens

Quel(s) examen(s) complémentaire(s) ophtalmologique(s) demandez vous pour préciser votre diagnostic ?
A - Champ visuel Goldmann
B - Test de Lancaster
C - Angiographie en fluorescence
D - Fluorométrie du vitré
E - Electrorétinographie

Quel examen faut-il demander en urgence ?


A - Numération Formule Sanguine
B - Vitesse de sédimentation
C - Glycémie
D - Cholestérolémie
E - Clearance de la créatinine

222
Exclusivement sur DOC - DZ : www.doc-dz.com NADJI 85
RESIDANAT EN POCHE TOME II
Cas Clinique en QCM

L'examen histopathologique de la biopsie d'artère temporale superficielle peut révéler une atteinte :
A - De l'intima
B - De la média
C - De l'adventice
D - De la limitante élastique interne
E - De la limitante élastique externe

Sans attendre ce résultat, le traitement d'urgence repose sur :


A - Les vasodilatateurs
B - Les anticoagulants
C - Les anti-agrégants plaquettaires
D - Les corticoïdes
E - Les antalgiques

A plus long terme, le traitement repose sur :


A - Les vasodilatateurs
B - Les anticoagulants
C - Les anti-agrégants plaquettaires
D - Les corticoïdes
E - Les antalgiques

Une femme de 50 ans, consulte pour une douleur de l'épaule survenant à l'effort pendant la journée, réveillant la patiente lors
du décubitus latéral prolongé. Un récent bilan biologique est normal.

Une pathologie de coiffe des rotateurs se traduit par :


A - Amyotrophie du deltoïde
B - Arc douloureux dans l'abduction vers 60°
C - Ascension de l'épaule lors de l'abduction active
D - Décollement unilatéral de l'omoplate
E - Limitation globale des mouvements passifs de l'épaule

Quel signe radiologique permet de détecter précocément une rupture de la coiffe des rotateurs ?
A - Déminéralisation hétérogène de la tête humérale
B - Calcification se projetant dans l'espace sous-acromial
C - Ascension de la tête humérale en abduction contre résistance
D - Trop bonne visualisation du trochiter
E - Subluxation inférieure de la tête humérale

Vous réalisez une arthrographie. Quel(s) signe(s) radiographique(s) témoigne(nt) d'une rupture de coiffe des
rotateurs ?
A - Une rupture du récessus interne sous-scapulaire
B - Une fuite de produit de contraste dans la gouttière bicipitale
C - Une opacification de la bourse sous-acromio-deltoïdienne
D - Une disparition du récessus articulaire inférieur
E - Une déchirure du bourrelet glénoïdien

La patiente vous apprend que quelques mois plus tôt, elle a consulté un autre rhumatologue pour des douleurs
scapulaires très importantes. Celui-ci aurait parlé "d'épaule aiguë hyperalgique" que désigne ce terme ?
A - Rupture franche de la coiffe des rotateurs
B - Bursite aiguë sous-deltoïdienne microcristalline
C - Fissure des bourrelets glénoïdiens
D - Arrachement trochitérien
E - Hémarthrose

223
Exclusivement sur DOC - DZ : www.doc-dz.com NADJI 85
RESIDANAT EN POCHE TOME II
Cas Clinique en QCM

En faveur d'une lésion ancienne de rupture de coiffe des rotateurs, vous pouvez retrouver sur les radios
standard :
A - Déminéralisation hétérogène de la tête humérale
B - Calcification se projetant dans l'espace sous-acromial
C - Ascension permanente de la tête humérale
D - Remaniement trochitérien avec condensation et structure hétérogène
E - Rupture du cintre scapulo-huméral

Vous proposez une thérapeutique médicale. Quelle est la modalité non acceptable ?
A - Anti-inflammatoires non stéroïdiens lors des poussées douloureuses
B - Infiltrations de dérivés cortisonés retard dans l'espace sous-acromio-deltoïdien
C - Mobilisation active de l'épaule en abduction
D - Rééducation dans les mouvements de décoaptation
E - Antalgiques simples en dehors des poussées douloureuses

Une jeune femme de 29 ans, enceinte de 8 mois, se plaint depuis 1 mois de douleurs de la hanche et de la cuisse droites
survenant à la marche et calmées par le repos. Elles irradient sur la face antérieure de la cuisse jusqu'au genou. Sa mère et
sa soeur ainée auraient été traitées pour un syndrome douloureux comparable. Tous les mouvements de la hanche sont
douloureux mais les amplitudes sont normales. La palpation révèle un point douloureux inguinal et fessier. La fosse iliaque
droite est souple et l'utérus gravide est mobile. Le signe de Lasègue droit est positif à 80°. L'état général est bon. La
température est à 37°5.

Parmi les résultats des examens complémentaires systématiquement demandés. lequel ou lesquels vous
paraissent pathologiques :
A - Radio de la hanche droite de face - angle de couverture de la tête : 5° - angle d'obliquité du toit : 45°
B - Radio de la hanche droite de profil - angle de couverture de la tête : 5°
C - Vitesse de conduction sensitive du nerf sciatique : ralentissement droit de 5 % par rapport au côté gauche
D - Analyse du liquide de ponction : 500 éléments/mm3
E - Angle d'inclinaison extrémité supérieure du fémur droit : 135°

En fait, on aurait donc du se contenter de demander le ou les examens suivants :


A - Radio du bassin de face
B - Hanche droite face et profil
C - Faux profil de Lequesne
D - Ponction de hanche
E - EMG et mesure des vitesses de conduction du nerf crural

Quelle orientation diagnostique pouvez-vous en déduire ?


A - Sciatique gravidique à irradiation inguinale
B - Hernie crurale de la femme enceinte
C - Décompensation d'une dysplasie de hanche
D - Arthrite aiguë de hanche
E - Chondro-calcinose de la femme enceinte

Quelle conduite thérapeutique doit-on adopter ?


A - Proposer des antalgiques et un repos relatif jusqu'à la fin de la grossesse
B - Effectuer une infiltration épidurale de corticoïdes
C - Effectuer rapidement une ostéotomie de varisation de l'extrémité supérieure du fémur droit
D - Programmer la réalisation d'une intervention chirurgicale après l'accouchement
E - Effectuer rapidement une arthrotomie de la hanche droite

224
Exclusivement sur DOC - DZ : www.doc-dz.com NADJI 85
RESIDANAT EN POCHE TOME II
Cas Clinique en QCM
Un garçon de 18 ans consulte le lendemain d'un accident survenu lors d'un match de football. Il vous explique avoir ressenti,
sans choc sur le genou, une vive douleur dans le genou, accompagnée d'une sensation fugace de déboîtement. Une tentative
de reprise de jeu, quelques instants plus tard, a entraîné un accident identique. Le genou a gonflé en quelques minutes.
L'examen révèle :
- Un épanchement intra-articulaire avec choc rotulien
- Une perte douloureuse de l'extension complète
- Une limitation douloureuse de la flexion à 120°
- Une sensibilité de l'interligne articulaire externe
- L'absence de douleur sur le trajet du LLI
- L'absence de laxité interne et externe en légère flexion
- L'absence de signe de Smillie
- L'absence de signe de Trillat Lachmann

Au terme de cet examen, vous concluez :


A - A la possibilité d'une lésion méniscale externe
B - Au diagnostic d'entorse bénigne puisque la stabilité du genou est conservée
C - A la possibilité d'une rupture isolée du ligament croisé antérieur
D - A la possibilité d'une luxation rotulienne spontanément réduite
E - A la possibilité d'une entorse grave du LLE

L'épanchement intra-articulaire actuel :


A - Justifie une ponction articulaire pour affirmer sa nature
B - Est certainement de nature hémorragique (hémarthrose) car il s'est rapidement constitué
C - Peut suffire pour expliquer la perte douloureuse de l'extension complète
D - Peut suffire pour expliquer la limitation de la flexion à 120°
E - Traduit l'existence d'une lésion intra-articulaire grave

Il n'y avait pas de signe de Smillie, mais comment l'avez-vous recherché ?


A - En flexion du genou à 90° en exerçant une traction antérieure du tibia
B - En légère flexion et en luxant la rotule en dehors
C - En recherchant un point douloureux au bord interne de la rotule
D - A plat ventre genou fléchi à 600 en imprimant des mouvements de rotation au tibia
E - En appréciant la continuité de la rotule, du tendon rotulien et de la tubérosité tibiale antérieure

Votre recherche du tiroir antérieur à 90° de flexion à été gênée par la douleur et la difficulté d'obtenir un
relâchement musculaire correct. Parmi les manoeuvres suivantes laquelle permet de tester plus sûrement le
ligament croisé antérieur ?
A - La manoeuvre de Trillat Lachmann
B - La manoeuvre de Oudart
C - La recherche d'un ressaut rotatoire
D - La manoeuvre de Lasègue
E - La manoeuvre de Cabot

Vous faites préciser par l'interrogatoire, le mécanisme de l'accident. Parmi les mécanismes suivants,
le(s)quel(s) aurai(en)t été susceptible(s) d'avoir entraîné une lésion isolée du ligament croisé antérieur ?
A - Valgus
B - Varus
C - Rotation externe du tibia sous le fémur
D - Rotation interne du tibia sous le fémur
E - Shoot dans le vide

Pour compléter votre bilan clinique, quelle est la première prescription à effectuer aux urgences ?
A - Une ponction articulaire
B - Une radiographie face et profil du genou et un défilé fémoro-patellaire à 30° de flexion
C - Une épreuve radiographique dynamique en varus-valgus
D - Une arthrographie
E - Une arthroscopie

225
Exclusivement sur DOC - DZ : www.doc-dz.com NADJI 85
RESIDANAT EN POCHE TOME II
Cas Clinique en QCM
Monsieur X..., 60 ans, vous à consulté pour des douleurs lombaires basses apparues progressivement depuis 3 mois, de
rythme mécanique, sans irradiation dans les membres inférieurs, calmées par l'Aspirine. Les radiographies que vous avez fait
effectuer ont montré un aspect anormal du corps vertébral de L4 qui apparait hétérogène, non tassé, avec une
ostéocondensation à prédominance périphérique, sans atteinte de l'arc postérieur. Le diamètre antéro-postérieur de ce corps
vertébral est, sur le cliché de profil, augmenté par rapport à celui des vertèbres sus et sous-jacentes. Vous avez porté le
diagnostic de maladie de Paget.

Quel(s) argument(s) radiologique(s) vous a (ont) permis d'écarter le diagnostic de métastases osseuses
condensantes ?
A - Atteinte isolée de la vertèbre L4
B - Augmentation du diamètre antéro-postérieur du corps vertébral
C - Aspect hétérogène de la trame osseuse
D - Prédominance périphérique des lésions d'ostéocondensation
E - Absence de tassement vertébral

Parmi les anomalies biologiques suivantes, la maladie de Paget peut expliquer :


A - Augmentation des phosphatases acides sériques
B - Augmentation des phosphatases alcalines sériques
C - Augmentation de la Gamma GT sérique
D - Hypophosphorémie
E - Hypergammaglobulinémie

Une scintigraphie osseuse au pyrophosphate de technétium :


A- Montre une hyperfixation constante des lésions osseuses de la maladie de Paget
B - Permet d'apprécier l'extension de la maladie sur l'ensemble du squelette
C - Permet d'éliminer l'éventualité de métastases osseuses
D - Permet de juger de l'évolutivité des lésions osseuses de la maladie de Paget
E - Permet d'apprécier globalement l'état de la minéralisation squelettique

Parmi les complications osseuses suivantes, vous devez envisager au cours de la maladie de Paget ?
A - Déformation des os longs
B - Stries de Looser - Milkmann
C - Fractures vraies des os longs
D - Platybasie de la base du crâne
E - Dégénérescence sarcomateuse

Toutes les complications neurologiques suivantes peuvent survenir au cours de la maladie de Paget, sauf une.
Laquelle ?
A - Surdité
B - Anosmie
C- Compression médullaire
D - Polyradiculonévrite
E - Syndrome de la queue de cheval

L'évolution de la maladie de Paget peut être freinée par la prise de :


A - Anti-inflammatoires non stéroïdiens
B - Corticoïdes
C - Calcitonine
D - Diphosphonates
E - Fluorure de Sodium

Parmi les examens biologiques suivants, lequel ou lesquels est(sont) utile(s) pour la surveillance à court terme
de l'efficacité du traitement ?
A - Clairance du phosphore
B - Taux sérique de la parathormone
C - Taux sérique des phosphatases alcalines
D - Vitesse de sédimentation globulaire
E - Taux de la calcémie

226
Exclusivement sur DOC - DZ : www.doc-dz.com NADJI 85
RESIDANAT EN POCHE TOME II
Cas Clinique en QCM
Une femme de 65 ans est hospitalisée pour des lombalgies hautes de début brutal, survenues à la suite d'un faux
mouvement. Dans les antécédents, on note une hernie inguinale gauche opérée à l'âge de 55 ans. Elle a été ménopausée à
l'âge de 48 ans. Les lombalgies sont mécaniques, sans fièvre, ni altération de l'état général. L'examen clinique retrouve une
douleur exquise à la palpation de L2. L'examen neurologique est normal. Les radiographies du rachis montrent un aspect
peigné et un tassement de L2. La vitesse de sédimentation est à 10 mm à la première heure ; calcémie : 2,30 mmol/l;
phosphorémie : 1,15 mmol/l ; calciurie et phosphaturie normales ; Phosphatases alcalines : 143 UI/l (normale 105 UI/l).

Parmi les antécédents suivants, le(s)quel(s) est (sont) un argument en faveur d'une ostéoporose commune ?
A - Coliques néphrétiques
B - Absence de traitement hormonal substitutif à la ménopause
C - Prise de Dihydan® au long cours
D - Résection étendue du grêle
E - Corticothérapie au long cours

Parmi les signes radiographiques suivants, le(s)quel(s) remet(tent) en cause ce diagnostic ?


A - Recul du mur postérieur de L2
B - Fissure de la diaphyse fémorale
C - Tassements de D8, L1, L4
D - Lacunes crâniennes
E - Résorption des houppes phalangiennes

Comment intégrez-vous dans cette histoire l'augmentation des phosphatases alcalines ?


A - Lithiase du cholédoque
B - Tassement vertébral récent
C - Maladie de Paget associée
D - Ostéomalacie
E - Cirrhose biliaire primitive

Votre attitude thérapeutique comprendra un ou plusieurs des actes suivants :


A - Ostéosynthèse par vis et plaques
B - Repos allongé pendant les premiers jours
C - Antalgiques
D - Mise en route d'un traitement par fluorure de sodium 50mg/jour
E - Kinésithérapie active des muscles abdominaux et paravertébraux

Un homme de 60 ans est hospitalisé pour une crise d'arthrite aiguë du genou droit. Il s'agit d'un hypertendu traité par un
diurétique thiazidique et mal équilibré. Il est asthénique, a une soif intense alors qu'il n'est pas diabétique. Sa température est
à 38° sans frisson. La ponction du liquide articulaire du genou ramène un liquide trouble contenant 8 000 éléments par mm3 à
prédominance de polynucléaires (80 %). Il contient des cristaux de pyrophosphate de calcium et d'acide urique. Il n'y a pas de
germe à l'examen direct. La NFS donne les résultats suivants : 14000 blancs, 70 % de polynucléaires, l'uricémie est à 500
micromol/l. La radiographie du genou droit face et profil montre une calcification méniscale et du contour des condyles
fémoraux.

Quels éléments du cas présent permettent d'affirmer le diagnostic de chondrocalcinose ?


A - Radiographies des genoux
B - L'existence de cristaux de pyrophosphates de calcium à la ponction articulaire
C - L'hyperleucocytose sanguine
D - L'hypercytose du liquide articulaire
E - L'hyperuricémie

Dans la pathogénie de la crise articulaire actuelle de ce malade, quelles autres hypothèses diagnostiques
peuvent être discutées dans ce cas ?
A - Une hémarthrose
B - La goutte
C - Une hydarthrose d'origine arthrosique (poussée congestive d'arthrose)
D - Une infection associée à la chondrocalcinose
E - Une lésion méniscale

227
Exclusivement sur DOC - DZ : www.doc-dz.com NADJI 85
RESIDANAT EN POCHE TOME II
Cas Clinique en QCM

Parmi ces associations avec la chondrocalcinose, quelles sont celles qui peuvent être rencontrées ?
A - L'hémochromatose
B - L'hyperparathyroïdisme
C - L'ostéoporose
D - L'ostéomalacie
E - L'hypothyroïdie

Parmi les hypothèses suivantes, quelle(s) est (sont) celle(s) qui peut (peuvent) être à l'origine de la
chondrocalcinose ?
A - Une hyperthyroïdie
B - Un hyperparathyroïdisme
C - Un diabète insipide
D - Une septicémie
E - Une polyarthrite rhumatoïde

Parmi ces examens pour prouver un hyperparathyroïdisme, lesquels demandez-vous ?


A - Un bilan phosphocalcique sanguin et urinaire
B - Un ionogramme
C - Des radiographies des mains
D - Un dosage de la PTH
E - Une échographie de la région thyroïdienne

Un skieur fait une chute qui entraîne son bras en adduction rotation externe.
La douleur est violente, il ressent un engourdissement du moignon de l'épaule ; au cours du transfert au poste de secours, sa
main enfle et se cyanose.

Le diagnostic de luxation antéro interne de la scapulo-humérale est orienté par :


A - Il est possible de ramener le coude contre le corps
B - Il est impossible d'effectuer une rotation interne
C - Une rotation imprimée au coude ne se transmet pas à la tête humérale
D - Il y a un vide sous acromial externe
E - Il y a précocement une importante ecchymose de la paroi thoracique

Vous retrouvez lors d'un examen systématique régional une insensibilité à la piqûre au moignon de l'épaule, sur
sa face externe. Vous évoquez une atteinte :
A - Du nerf sus scapulaire
B - De la racine C7 du plexus brachial
C - Du nerf circonflexe
D - Du nerf musculo-cutané
E - De la branche superficielle du nerf spinal

La constatation d'un oedème avec cyanose fait rechercher :


A - Un arrachement du confluent veineux jugulo sous-clavier
B - Une compression de la veine céphalique
C - Une ischémie par rupture de l'artère axillaire
D - Une compression de la veine axillaire
E - Un réflexe vasoconstricteur distal

Quelle ou quelles association(s) lésionnelle(s) fréquente(s) allez-vous rechercher sur les radiographies de
l'épaule droite ?
A - Une fracture de la clavicule
B - Une fracture du trochiter
C - Une facture de l'acromion
D - Une fracture de la coracoïde
E - Une fracture de la glène de l'omoplate

228
Exclusivement sur DOC - DZ : www.doc-dz.com NADJI 85
RESIDANAT EN POCHE TOME II
Cas Clinique en QCM

Après réduction, vous conseillez :


A - Une semaine d'immobilisation
B - Trois semaines d'immobilisation
C - Une rééducation immédiate en rotation externe
D - Un plâtre thoraco-brachial pour un mois
E - Une écharpe coude au corps

Lors d'un match de rugby, un joueur de 24 ans, pilier droit est victime lors d'un placage d'un traumatisme de l'épaule droite
entraînant une luxation antéro-interne. Sa réduction immédiate sur le terrain est réalisée puis une écharpe simple est mise en
place pour 3 semaines coude au corps. Aucun bilan radiographique n'est réalisé. 1 an après, en se coiffant, il présente une
récidive de la luxation antérieure qui fait l'objet d'une nouvelle réduction, et consulte ultérieurement.

Lors de l'examen clinique, devant ce tableau d'instabilité antéro-interne récidivante, quels signes recherchez-
vous ?
A - Test d'appréhension
B - Abduction irréductible
C - Une douleur provoquée par l'abduction activé contrariée
D - Tiroir antérieur
E - Amyotrophie du grand pectoral

Un bilan radiographique est réalisé pour le bilan de la luxation récidivante, il est utile de demander :
A - Face en double obliquité
B - Profil axillaire
C - Profil glénoïdien
D - Incidence de l'encoche de Strycker
E - Abduction contrariée

Le bilan radiographique standard est normal, mais l'interrogatoire retrouve plusieurs épisodes de subluxation
antérieure spontanément réductibles avec impression de ressaut laissant suspecter une atteinte du bourrelet
glénoïdien Celle-ci sera affirmée par :
A - Arthrographie avec incidence de Bernageau
B - Echographie
C-IRM
D - Arthro-scanner
E - Tomographie

Cet examen complémentaire montre une lésion du bourrelet avec décollement sous-capsulo périosté antérieur.
Votre proposition thérapeutique doit être :
A - Rééducation de la sangle musculaire axée sur les rotateurs externes
B - Physiothérapie locale anti inflammatoire
C - Stabilisation antérieure chirurgicale
D - Acromioplastie chirurgicale
E - Abstention et surveillance

Une femme de 70 ans souffre depuis 3 mois de douleurs des épaules et des hanches et, depuis peu, d'un genou qui est le
siège d'un épanchement. Les artères temporales sont bien battantes et non indurées. La V.S. est de 76/110. Il existe une
anémie de type inflammatoire.

Quels sont les diagnostics que suggère en premier lieu cette symptomatologie ?
A - Une pseudo-polyarthrite rhizomélique
B - Une PR à début rhizomélique
C - Une arthrite réactionnelle
D - Un RAA
E - Une péri-artérite noueuse

229
Exclusivement sur DOC - DZ : www.doc-dz.com NADJI 85
RESIDANAT EN POCHE TOME II
Cas Clinique en QCM

Une biopsie de l'artère temporale superficielle met en évidence une artérite giganto-cellulaire. Cette artérite :
A - Permet d'affirmer le diagnostic de maladie de Horton
B - Permet d'exclure une polyarthrite rhumatoïde
C - Permet d'exclure une périartérite noueuse
D - Peut se rencontrer dans une arthrite réactionnelle
E - Est constante dans la pseudo-polyarthrite rhizomélique

On procède à la recherche de facteurs rhumatoïdes. Ces facteurs sont retrouvés dans la population générale à
cet âge dans
A - 75 % des cas
B - 50 % des cas
C - 25 % des cas
D - 5 % des cas
E - 0 % des cas

On recherche l'antigène HLA B27. Cet antigène est retrouvé dans la population générale dans environ :
A - 95 % des cas
B - 50 % des cas
C - 25 % des cas
D - 5 % des cas
E - 0 % des cas

On prescrit au stade initial dans ce cas, un traitement comportant :


A - Prednisone 5 mg/j
B - Phénylbutazone 500 mg/j
C - Colchicine 1 mg/j
D - Méthotrexate 10 mg/j
E - Prédnisone 30 mg/j

Monsieur X. âgé de 64 ans a des douleurs lombaires basses depuis 1 mois qui l'empêchent de se mettre debout. Des clichés
osseux montrent une discrète déminéralisation. A l'examen, vous relevez une douleur à la pression de la 4ème et 3ème
lombaire. Il n'y a rien d'anormal au niveau du coeur, du poumon, du foie, des aires ganglionnaires de, la rate, de la peau.
Sur l'hémogramme, on retrouve :
GR : 3 400 000 par mm3 Plaquettes : 210 000 par mm3
Hb : 10,4 g/100 ml GB : 4 100 par mm3
Hte : 30,9 % PN : 41 %; Eo : 2 % ; L : 500 %;Mono :
VGM : 90,5 fl VS : 98 mm à la première heure
CCMH : 33,7 %
Protéinurie + + +; Créatinine : 116 micromol/l (50-120 ; Calcémie : 2,55 millimol/l (2,24-2,6) ; Uricémie : 589 micromol/l (150-
450).
Protides totaux : 80 g/l
Albumine : 39,8 g/l
Alpha 1 : 1,49 g/l
Alpha 2 : 4,4 g/l
Bêta : 30,4 g/l
Gamma : 5 g/l

Le diagnostic de myélome est évoqué. Pour faire le bilan d'éventuelles lésions osseuses, liées à cette maladie,
vous demandez des radiographies. Pour quels os ces radios vous semblent-elles utiles ?
A - Crâne
B - Colonne cervicale
C - Bassin
D - Pieds
E - Mains

Sur ces radiographies, vous pouvez retrouver dans les cas habituels les anomalies suivantes :
A - Une fracture pathologique
B - Une déminéralisation diffuse
C - Des lacunes ostéolytiques multiples et arrondies
D - Un tassement vertébral
E - Des images ostéolytiques diffuses avec plages de condensation

230
Exclusivement sur DOC - DZ : www.doc-dz.com NADJI 85
RESIDANAT EN POCHE TOME II
Cas Clinique en QCM

La preuve formelle du myélome vous sera apportée le plus souvent avec certitude par un examen. Lequel ?
A - La bêta 2 microglobuline sérique
B - Immuno-électrophorèse (ou immuno fixation)
C - Scintigraphie osseuse
D - Myélogramme
E - Biopsie rénale

L'existence de la protéinurie dans cette affection vous amène à demander que soit recherchée une protéinurie
de Bence Jones. Quelles affirmations sont-elles vraies concernant cette anomalie ?
A - Protéinurie définie habituellement par la présence exclusive de chaînes lourdes d'immunoglobulines
B - Protéinurie thermo-soluble à 56°C
C - Protéinurie parfois non détectée par la méthode des bandelettes
D - Protéinurie jouant un rôle décisif dans la constitution des lésions rénales du myélome
E - Protéinurie massive dans certains myélomes

Ce patient peut développer une infection bactérienne grave.


Quelle en est la raison la plus probable ?
A - Neutropénie
B - Anomalies des fonctions des polynucléaires
C - Hypogammaglobulinémie polyclonale
D - Déficit en Iymphocytes CD4
E - Insuffisance rénale chronique

L'anémie présentée dans ce cas par ce malade peut avoir plusieurs explications. Lesquelles ?
A - Insuffisance rénale
B - Insuffisance médullaire
C - Hémodilution favorisée par l'hyperprotidémie
D - Saignement chronique
E - Carence en acide folique (dénutrition)

Le traitement de la maladie fait habituellement appel à différents produits. Lesquels ?


A - Melphalan (Alkeran®)
B - Corticoïdes
C - Busulfan (Misulban®)
D - Cyclophosphamide (Endoxan®)
E - 5 fluoro-uracile

Sous l'effet de ce traitement l'amélioration se jugera sur la surveillance de certains examens appréciant la :
A - Diminution du nombre des leucocytes
B - Diminution du pic visible dans la zone des bêta-globulines
C - Diminution du nombre de lésions osseuses
D - Diminution de la protéinurie
E - Diminution de la plasmocytose médullaire

Certains événements précipitent la survenue d'une insuffisance rénale. Lesquels ?


A - Hypercalcémie
B - Déshydratation
C - Prescription d'une ampicilline par voie IV
D - Réalisation d'une UIV
E - Prescription de bêta-bloquants

231
Exclusivement sur DOC - DZ : www.doc-dz.com NADJI 85
RESIDANAT EN POCHE TOME II
Cas Clinique en QCM
Au cours d'un accident de la circulation, le conducteur d'un véhicule automobile qui ne portait pas de ceinture, présente :
- un traumatisme crânien avec perte de connaissance brève,
- une fracture du cotyle droit,
- une fracture transversale ouverte stade II de Cauchois de la jambe gauche,
- une fracture de l'humérus gauche au tiers inférieur.
Il est ramené par les services de secours et vous le recevez une heure après l'accident, au service des urgences.

A l'arrivée aux urgences, ce malade est choqué :


A - Le choc est du à une hypovolémie
B - Le choc est du à la douleur
C - Le choc doit être prévenu sur les lieux de l'accident par remplissage veineux et antalgiques per os
D - Ce malade répond à la définition du polyfracture
E - L'immobilisation des fractures est mise en oeuvre sur les lieux de l'accident

Le diagnostic radiologique des fractures du crâne et du bassin repose sur :


A - Des incidences radiologiques de face, en 3/4 alaire et en 3/4 obturateur du bassin
B - Une échographie pelvienne
C - Une tomodensitométrie du bassin
D - Une radiographie du crâne de face front plaque et de profil
E - Une tomodensitométrie crânienne

L'incidence en 3/4 obturateur du bassin permet de dégager :


A - La colonne antérieure
B - La paroi antérieure
C - La colonne postérieure
D - La paroi postérieure
E - Le cadre obturateur

La fracture de l'humérus peut se compliquer de paralysie radiale qui :


A - Entraîne une perte de l'extension des doigts
B - Entraîne une perte de l'extension du poignet
C - Entraîne une perte de l'adduction du pouce
D - Traduit une section du nerf radial
E - Nécessite un complément électromyographique initial

La fracture transversale de jambe ouverte stade II :


A - Comporte un risque de nécrose cutanée secondaire
B - Impose en urgence une antibiothérapie prophylactique
C - Doit être en urgence parée
D - Contre-indique une ostéosynthèse interne
E - Peut être temporairement immobilisée par une broche transcacalnéenne

Une femme de 55 ans, 78 kg pour 1,65 m, se plaint depuis trois jours d'une impotence fonctionnelle du membre inférieur droit
avec température à 38,2°C, sans notion de traumatisme.
Dans ses antécédents, on note un diabète traité par régime et glucophage retard®, une hystérectomie pour fibrome à l'âge de
50 ans, une hypertension artérielle modérée traitée depuis 2 ans par Moduretic® 1 cp/j, une cure chirurgicale d'un ongle
incarné un mois avant l'épisode actuel.
A l'examen, le genou est chaud, douloureux spontanément et à la mobilisation, il existe un choc rotulien, le mollet est souple, il
n'y a pas d'oedème, le reste de l'examen est sans particularité. Biologie : leucocytes = 11 500/mm3, poly.neutro.= 78 %, VS =
105 mm à la première heure.

Parmi les hypothèses diagnostiques suivantes, laquelle doit être évoquée en priorité ?
A - Polyarthrite rhumatoïde à début monoarticulaire
B - Crise de goutte
C - Arthrite septique
D - Poussée de gonarthrose
E - Arthrite réactionnelle

232
Exclusivement sur DOC - DZ : www.doc-dz.com NADJI 85
RESIDANAT EN POCHE TOME II
Cas Clinique en QCM

Quel examen vous permet de confirmer votre hypothèse ?


A - Test au latex-réaction de Waaler-Rose
B - Ponction articulaire
C - Dosages de l'uricémie et de l'uraturie
D - Arthroscopie
E - Groupage HLA

Quelle(s) exploration(s) complémentaire(s) devez-vous proposer ?


A - Radiographies du genou
B - Scintigraphie osseuse
C - Hémocultures
D - Séro-diagnostic des chlamydiae
E - Etude de l'immunité cellulaire

Le traitement doit comporter, outre l'antibiothérapie générale :


A - Ponction et lavage de l'articulation
B - Antibiothérapie intra-articulaire
C - Immobilisation
D - Rééducation statique du quadriceps
E - Corticothérapie par voie générale

Parmi les éléments suivants, quel(s) est (sont) celui (ceux) qui a (ont) favorisé la survenue de cette affection ?
A - Hypertension artérielle
B - Hystérectomie
C - Ablation de l'ongle incarné
D - Traitement par Moduretic
E - Diabète

Une femme de 66 ans souffre depuis une semaine d'une douleur du genou gauche d'apparition brutale et d'horaire nocturne. Il
n'y a pas d'antécédents articulaires. La malade est traitée par le Furosemide (Lasilix®) 40 mg/jour pour hypertension artérielle.
A l'examen le genou est gonflé, chaud, douloureux au palper et limité dans la flexion. Le reste de l'examen articulaire est
normal. La tension artérielle est à 16-9 cm Hg. Il n'y a pas de signe d'atteinte viscérale. Le poids est de 50 kg pour 1,60 m et
l'alimentation est équilibrée.
La VS est à 100 mm à la l ère heure. Le taux d'hémoglobine à 9 g/100 ml, l'uricémie à 90 mg/l (540 micromol/l) la
créatininémie à 140 micromol/l, la calcémie à 120 mg/l (3 mmol/l), la phosphorémie à 31 mg/l (1 mmol/l), la sidérémie est
normale ainsi que la férritinémie. La radiographie des genoux montre des calcifications des ménisques et un genu varum.

Quels sont les arguments pour la goutte ?


A - Sensibilité à la colchicine
B - Traitement par le furosémide
C - Absence de tophus
D - Uricémie à 90 mg/l
E - Présence de cristaux d'urate de sodium dans le liquide synovial

Quels sont les arguments pour la chondrocalcinose ?


A - Liquide synovial mécanique
B - Il s'agit d'un sujet âgé
C - Présence de calcifications méniscales
D - Présence d'un genu varum
E - Présence de microcristaux de pyrophosphate de calcium dans le liquide synovial

Quelles sont les causes ou facteurs prédisposants possibles de la chondrocalcinose de cette malade ?
A - L'âge
B - Insuffisance rénale chronique
C - Hyperparathyroïdisme primitif
D - Hypercalcémie paranéoplasique
E - Hémochromatose

233
Exclusivement sur DOC - DZ : www.doc-dz.com NADJI 85
RESIDANAT EN POCHE TOME II
Cas Clinique en QCM

Quels sont les arguments pour un adénome parathyroïdien à l'origine de la chondrocalcinose ?


A - Hypercalcémie
B - Phosphorémie normale
C - Lithiase urinaire
D - Sidérémie normale
E - Echographie cervicale normale

Un jeune homme de 25 ans souffre d'une oligoarthrite touchant les genoux et les chevilles, apparue une semaine après une
urétrite d'origine vénérienne. Son médecin lui prescrit un traitement par pénicilline G (10 millions d'U. par jour pendant 10
jours) sans aucune efficacité. On évoque le diagnostic de syndrome de Fiessinger-Leroy-Reiter.

Dans l'affection en cause, on trouve l'antigène HLA B27 dans environ :


A - 98 % des cas
B - 75 % des cas
C - 40 % des cas
D - 25 % des cas
E - 5 % des cas

A l'origine de l'urétrite, on soupçonne une infection à chlamydia trachomatis. Pour confirmer cette étiologie, on
procède à :
A - Une culture de prélèvements urétraux
B - Une recherche d'anticorps anti-chlamydiae
C - Un examen en immunofluorescence du prélèvement urétral
D - Une hémoculture
E - Une culture du liquide articulaire

En raison de sa maladie, le sujet risque plusieurs des complications suivantes. Lesquelles ?


A - Arthropathie destructrice
B - Séquelles d'uvéite
C - Kératodermie palmo-plantaire
D - Calcanéïte
E - Spondylarthrite ankylosante

Le traitement des arthrites réactionnelles fait appel en première intention :


A - Aux immunosuppresseurs
B - Aux anti-inflammatoires non stéroïdiens
C - Aux céphalosporines
D - A la colchicine
E - Aux corticoïdes

Une femme de 80 ans vient en consultation en urgence pour une baisse brutale de l'acuité visuelle de l'oeil droit.
Elle se plaint de céphalées et d'une altération de l'état général.
L'examen ophtalmologique va révéler au fond d'oeil droit un oedème ischémique de la papille optique.
L'examen de l'oeil gauche est normal.
L'examen des artères temporales superficielles les montre indurées et non battantes, beaucoup plus grosses
qu'habituellement.

Les complications oculaires de la maladie de Horton comportent :


A - L'ischémie aiguë de la tête du nerf optique
B - L'oedème papillaire de stase
C - L'occlusion de l'artère centrale de la rétine
D - L'occlusion de la veine centrale de la rétine
E - L'occlusion des capillaires rétiniens

Quels examens complémentaires ophtalmologiques demandez-vous pour préciser votre diagnostic ?


A - Champ visuel Goldmann
B - Test de Lancaster
C - Angiographie en fluorescence
D - Fluorométrie du vitré
E - Electrorétinographie

234
Exclusivement sur DOC - DZ : www.doc-dz.com NADJI 85
RESIDANAT EN POCHE TOME II
Cas Clinique en QCM

Quel examen faut-il demander en urgence ?


A - Numération Formule Sanguine
B - Vitesse de sédimentation
C - Glycémie
D - Cholestérolémie
E - Clearance de la créatinine

L'examen histopathologique de la biopsie d'artère temporale superficielle révèle une atteinte :


A - De l'intima
B - De la média
C - De l'adventice
D - De la limitante élastique interne
E - De la limitante élastique externe

Sans attendre ce résultat, le traitement d'urgence repose sur :


A - Les vasodilatateurs
B - Les anticoagulants
C - Les anti-agrégants
D - Les corticoïdes
E - Les antalgiques

A plus long terme, le traitement repose sur :


A - Les vasodilatateurs
B - Les anticoagulants
C - Les anti-agrégants
D - Les corticoïdes
E - Les antalgiques

Une femme de 68 ans a dans ses antécédents lointains un ulcère bulbaire. Elle se plaint depuis 2 mois de douleurs
inflammatoires des ceintures scapulaire et pelvienne avec impotence fonctionnelle majeure. Il existe une asthénie, un
amaigrissement de 4 kg; depuis une semaine des céphalées bitemporales sont apparues.
La mobilisation passive des articulations est peu douloureuse : il existe une tuméfaction inflammatoire des 2 artères
temporales dont les pouls ne sont pas perçus. Elle pèse 60 kg, les radiographies sont normales. vitesse de sédimentation : 90
mm à la première heure. NFS : anémie inflammatoire modérée.

Quel est le diagnostic le plus probable ?


A - Polyarthrite rhumatoïde
B - Polymyosite
C - Pseudo-polyarthrite rhizomélique
D - Maladie de Horton
E - Métastases osseuses

Quel examen allez-vous demander pour le confirmer ?


A - Sérologie rhumatoïde
B - Electromyogramme
C - Scintigraphie osseuse
D - Biopsie d'artère temporale
E - Enzymes musculaires

Quel traitement allez-vous instituer ?


A - Diclofenac 150 mg/jour
B - Dapsone 100 mg/jour
C - Prednisone 60 mg/jour
D - D-Pénicillamine 300 mg/jour
E - Méthotrexate 10 mg/semaine

235
Exclusivement sur DOC - DZ : www.doc-dz.com NADJI 85
RESIDANAT EN POCHE TOME II
Cas Clinique en QCM

Comment allez-vous apprécier l'efficacité du traitement ?


A - Symptomatologie douloureuse
B - Electromyogramme
C - Biopsie d'artère temporale de contrôle
D - Vitesse de sédimentation
E - Sérologie rhumatoïde

Quelle sera la durée moyenne du traitement ?


A - 15 jours
B - 6 mois
C - 18 mois
D - Jusqu'à normalisation de la vitesse de sédimentation
E - A vie

236
Exclusivement sur DOC - DZ : www.doc-dz.com NADJI 85
RESIDANAT EN POCHE TOME II
Cas Clinique en QCM

237
Exclusivement sur DOC - DZ : www.doc-dz.com NADJI 85
RESIDANAT EN POCHE TOME II
Cas Clinique en QCM
Une jeune femme de 27 ans se plaint depuis trois ans de troubles urinaires. Trois à quatre fois, il y a 2 ans, elle a présenté,
par crises, des douleurs sus-pubiennes avec sensation de pesanteur vésicale, des mictions très fréquentes et douloureuses.
Ont été alors pratiquées une cystocopie et une urographie intraveineuse jugées normales. L'année précédente a été indemne
de tout épisode et une uroculture systématique a été trouvée stérile. A son retour de vacances marquées par une grande
activité sexuelle, elle présente les mêmes symptômes qui durent depuis deux jours. La veille, elle signale une hématurie
macroscopique. La température est normale, il n'y a pas de protéinurie sur les urines filtrées, la cytologie montre des hématies
non déformées, un grande quantité de cellules vésicales et de leucocytes. L'uroculture trouve 10 exposant 6 germes par ml.
La patiente a réduit volontairement la quantité de boissons car les mictions sont très douloureuses. La densité urinaire est à
1015.

Vous pratiquez une uroculture. Dans ce contexte, quel est le germe le plus fréquemment mis en évidence ?
A - Escherichia coli
B - Streptocoque hémolytique
C - Staphylocoque
D - Bacille pyocyanique
E - Protéus hauseri
Bonne(s) réponse(s) : A

Sans commentaire.

Vous notez en faveur d'une infection urinaire strictement limitée à l'appareil urinaire inférieur (infection urinaire
basse), le ou les éléments suivants :
A - La densité urinaire à 1015
B - Existence d'une hématurie macroscopique
C - Absence de fièvre
D - Leucocyturie importante
E - Urographie intraveineuse normale
Bonne(s) réponse(s) : C

Sans commentaire.

Quelle attitude allez-vous adopter ?


A - Gentalline® IM 2 fois 40 mg par jour pendant 8 jours
B - Irrigation intravésicale d'anti-inflammatoires ou de corticoïdes
C - Boissons abondantes, 2 litres d'eau minérale par 24 heures
D - Anti-infectieux pendant 5 jours (acide pipémidique ou Pipram®)
E - Consultation gynécologique
Bonne(s) réponse(s) : C D E

Sans commentaire.

L'uroculture a découvert une infection à Protéus mirabilis. La malade ne suit pas le traitement et l'infection va
persister sous forme d'une bactériurie.
Vous la revoyez alors et vous devez répondre à ses questions. Retenez la ou les propositions exactes :
A - Au cours d'une grossesse, il existe un risque important de pyélonéphrite
B - Même en l'absence de malformation, il existe un risque important d'insuffisance rénale d'évolution rapide
C - Il existe un risque de diabète
D - Il existe un risque de lithiase rénale
E - La patiente doit interrompre la prise de pilule contraceptive
Bonne(s) réponse(s) : A D

Il s'agit d'une infection urinaire récidivante chez une femme jeune.


On rappelle les germes les plus fréquemment rencontrés au cours des infections urinaires : E. coli 80%, proteus 10%,
klebsielle 3%, entérocoque 2%.

238
Exclusivement sur DOC - DZ : www.doc-dz.com NADJI 85
RESIDANAT EN POCHE TOME II
Cas Clinique en QCM
Monsieur A. 27 ans, d'origine nigérienne, se présente à la consultation pour une hématurie microscopique dépistée à la
médecine universitaire et confirmée a un centre d'examens de la Sécurité Sociale. Il raconte avoir fait plusieurs épisodes
d'hématurie macroscopique. Dans l'intervalle de ces épisodes, l'hématurie microscopique persiste et vous la retrouvez à la
bandelette à l'examen des urines.
Il n'a pas d'antécédent particulier, il se plaint seulement de quelques signes dysuriques au moment des épisodes
hématuriques.

L'âge et la clinique peuvent évoquer :


A - Une glomérulonéphrite
B - Une tuberculose rénale
C - Une lithiase rénale indolore
D - Une polykystose rénale
E - Une prostatite chronique
Bonne(s) réponse(s) : A B C D E

Sans commentaire.

A plusieurs reprises, l'hématurie macroscopique a revêtu un caractère terminal. Ce caractère évoque surtout :
A - Une origine rénale haute
B - Une glomérulonéphrite hématurique
C - Une malformation vasculaire intrarénale
D - Une tumeur du rein
E - Une cause vésicale
Bonne(s) réponse(s) : E

Sans commentaire.

Dans le contexte clinique décrit, quelle parasitose évoquez-vous comme possible à l'origine de ces hématuries ?
A - Filariose loa loa
B - Ankylostomose
C - Lambliase ou giardiase
D - Schistosomose
E - Distomatose
Bonne(s) réponse(s) : D

Sans commentaire.

Parmi les examens suivants, quel(s) est(sont) celui(ceux) qui prouve(nt) de façon précise et exacte le diagnostic
de parasitose ?
A - Sérodiagnostic
B - Cystoscopie avec biopsie vésicale
C - Artérographie rénale
D - Biopsie rénale sous échographie
E - Numération formule et bilan de coagulation
Bonne(s) réponse(s) : A B

Sans commentaire.

Quel traitement vous paraît-il particulièrement adapté à ce cas ?


A - Déhydroémétine
B - Niridazole (Ambilhar®)
C - Métronidazole (Flagyl®)
D - Nivaquine (Chloroquine®)
E - Acide éthacrynique (Edécrine®)
Bonne(s) réponse(s) : B

Compte tenu du contexte, le diagnostic de bilharziose urinaire est évidemment le plus vraisemblable (Schistosoma
haematobium). Toutefois aucune des étiologies citées lors du premier QCM ne pouvait être d'emblée éliminée, en particulier
la tuberculose. La déhydroémétine et le métronidazole sont des traitements de l'amibiase, la nivaquine du paludisme et l'acide
éthacrynique un diurétique de l'anse de Henlé, concurrent malheureux du furosémide.

239
Exclusivement sur DOC - DZ : www.doc-dz.com NADJI 85
RESIDANAT EN POCHE TOME II
Cas Clinique en QCM
Un homme de 40 ans sans antécédent pathologique présente depuis quelques semaines une asthénie importante. A
l'examen, on découvre une hypertension artérielle à 180/100 mmHg aux quatre membres. Le fond d'oeil est normal,
l'électrocardiogramme trouve une hypertrophie ventriculaire gauche grade I et une onde U. La clairance de la créatinine est à
100 ml/mn. La kaliémie mesurée à 3 reprises est à 3 mmol/l.

Chez cet hypertendu ayant une hypokaliémie, l'interrogatoire doit éliminer la prise de produits hypokaliémiants.
Ainsi, vous recherchez la prise de :
A - Laxatifs
B - Diurétiques thiazidiques
C - Diurétiques distaux
D - Analgésiques
E - Sympathicolytiques
Bonne(s) réponse(s) : A B

Et la prise de réglisse (responsable de tableau d'hyperminéralcortcisme).

Le(le) diagnostic(s) qu'il convient de discuter est(sont) :


A - Hyperaldostéronisme primaire
B - Hypertension rénovasculaire
C - Phéochromocytome
D - Tumeur secrétant de la rénine
E - Coarctation aortique
Bonne(s) réponse(s) : A B D

Ces 3 items entraînent une sécrétion élevée d'aldostérone.

L'examen biologique permettant de différencier hyperaldostéronisme primaire et secondaire est :


A - Activité rénine plasmatique
B - Glycémie
C - Kaliurèse
D - Bicarbonatémie
E - Aucun de ces examens.
Bonne(s) réponse(s) : A

Sans commentaire.

En cas d'hyperaldostéronisme primaire, l'activité rénine plasmatique est :


A - Normale
B - Basse
C - Elevée
D - Stimulable
E - Non stimulable
Bonne(s) réponse(s) : B D

Sans commentaire.

L'(les) examen(s) permettant de différencier hyperplasie bilatérale et adénome de Conn est(sont) :


A - Dosage de la rénine dans les veines surrénales
B - Dosage de l'aldostérone dans les veines surrénales
C - Test de freination par le sérum sale isotonique
D - Test de freination par l'inhibiteur de l'enzyme de conversion
E - Test à la métopirone
Bonne(s) réponse(s) : B C D

La rénine est produite par les reins et non par les surrénales.

Le médicament le plus logique pour traiter l'hypertension artérielle par hyperaldostéronisme primaire est :
A - Thiazidique
B - Spironolactone
C - Bêta-bloquant
D - Dihydralazine
E - Furosémide
Bonne(s) réponse(s) : B

Sans commentaire.
240
Exclusivement sur DOC - DZ : www.doc-dz.com NADJI 85
RESIDANAT EN POCHE TOME II
Cas Clinique en QCM
Le jeune John T. âgé de 17 ans présente un syndrome néphrotique dont un bilan complet a été fait 8 mois auparavant. Le
bilan étiologique était négatif et la biopsie rénale avait montré qu'il s'agissait d'un hyalinose segmentaire et focale. Trois essais
de corticothérapie ont été des échecs et ce garçon lassé des impératifs diététiques et thérapeutiques, a tout abandonné
depuis trois mois. Il est parti en camping il y a 15 jours. Il se présente à la consultation, il a d'énormes oedèmes des membres
inférieurs remontant jusqu'à la racine de la cuisse, il existe une infiltration de la paroi abdominale et des lombes. La pression
artérielle est à 92/65 en position couchée et le pouls est à 96 par minute. L'électrocardiogramme est normal, il existe un
épanchement pleural gauche d'importance moyenne et un comblement du cul-de-sac diaphragmatique droit. L'examen des
urines montre un énorme précipité à I'addition d'acide, sans signe d'hématurie à la bandelette Labstix®. Un ionogramme
plasmatique en urgence montre : créatinine normale, protéines 47 g/l, albumine 17 g/l, potassium 3,3 mmol/l, sodium 133
mmol/l. Dans les urines : sodium 8 mmol/l, potassium 55 mmol/l.

Les oedèmes peuvent être rapportés en première hypothèse :


A - A la baisse de la pression oncotique des protéines plasmatiques
B - A une compression veineuse
C - A une poussée d'insuffisance cardiaque
D - A une hépatite anictérique
E - A une symphyse péricardique
Bonne(s) réponse(s) : A

Voir question [19]

Dans un contexte de ce type, deux complications précoces sont à redouter. Lesquelles ?


A - Collapsus hypovolémique
B - Poussée d'hypertension maligne
C - Hyperkaliémie par hyperaldostéronisme
D - Embolie pulmonaire sur thrombose veineuse latente
E - Compression urétérale et iliaque
Bonne(s) réponse(s) : A D

Voir question [19]

Parmi les propositions thérapeutiques suivantes, vous pouvez au cours des premiers jours avoir recours à :
A - Perfusion de 1,5 litre d'un mélange de CINa isotonique et de glucose isotonique
B - Perfusion d'albumine humaine
C - Prescription de l'inhibiteur de l'enzyme de conversion de l'angiotensine 1
D - Prescription d'antialdostérones
E - Prescription d'anti-inflammatoires non stéroïdiens
Bonne(s) réponse(s) : B D

Voir question [19]

Parmi les évolutions suivantes, laquelle vous paraît la plus vraisemblable ?


A - Rémission complète spontanée
B - Rémission complète après perfusion de 1 mg/jour de prednisone pendant 3 semaines
C - Rémission complète après héparine intraveineuse à la pompe pendant 15 jours
D - Evolution marquée par des hématuries répétées
E - Evolution vers l'insuffisance rénale
Bonne(s) réponse(s) : E

On rappelle que la hyalinose segmentaire et focale est l'histologie la plus souvent rencontrée au cours des syndromes
néphrotiques de l'adulte, et que son évolution est caractérisée par une corticorésistance et une progression de l'insuffisance
rénale.
On rappelle également que les syndromes néphrotiques se compliquent de thromboses veineuses, dont la fréquence
augmente avec l'importance de la protéinurie et de l'hypoprotidémie.

Un jeune appelé de 20 ans consulte pour douleurs lombaires évoluant depuis trois mois. L'examen clinique met en évidence
une masse au niveau du creux épigastrique de (10 x 12 cm) et un petit nodule du testicule gauche.

Parmi les examens biologiques suivants, le(s)quel(s) doi(vent) être pratiqué(s) impérativement avant tout
traitement ?
A - Alphafoetoprotéine
B - Phosphatases acides
C - Bêta HCG
D - V.S.
E - Testotérone
Bonne(s) réponse(s) : A C

Sans commentaire.
241
Exclusivement sur DOC - DZ : www.doc-dz.com NADJI 85
RESIDANAT EN POCHE TOME II
Cas Clinique en QCM

Le taux de bêta HCG vous revient à 10000 UI. La V.S. est à 25 mm à la 1ère heure, les phosphatases alcalines
et acides sont normales. Quel(s) est(sont) le(s) diagnostic(s) à évoquer ?
A - Tératome mature
B - Séminome
C - Choriocarcinome
D - Tératocarcinome à composante choriale
E - Tumeur du sac vitellin
Bonne(s) réponse(s) : B

Sans commentaire.

Parmi les examens suivants, citez les deux indispensables au bilan d'extension :
A - Scanner thoracique et abdominal
B - Radiographie pulmonaire
C - Scintigraphie osseuse
D - Echographie pelvien
E - Transit gastro-intestinal
Bonne(s) réponse(s) : A B

Sans commentaire.

Le bilan d'extension montre comme seule anomalie des adénopathies lombo-aortiques droites mesurant 10 x 10
cm. Quel sera le traitement ?
A - Orchidectomie - curage lombo-aortique
B - Orchidectomie - chimiothérapie
C - Orchidectomie - radiothérapie
D - Orchidectomie - curage - radiothérapie
E - Chimiothérapie et radiothérapie
Bonne(s) réponse(s) : B

La choriocarcinome est une tumeur très chimiosensible et peu radiosensible.

Dans le suivi post-thérapeutique, quel est l'examen le plus important ?


A - Scanner
B - Radiographie du Thorax
C - Echographie
D - Lymphographie
E - Dosage de la bêta HCG
Bonne(s) réponse(s) : E

Sans commentaire.

242
Exclusivement sur DOC - DZ : www.doc-dz.com NADJI 85
RESIDANAT EN POCHE TOME II
Cas Clinique en QCM
Monsieur X. est "un bon vivant" de 55 ans qui vous consulte pour épisodes hématuriques évoluant depuis 2 mois. Un certain
nombre d'examens complémentaires ont été réalisés chez ce patient dans les antécédents duquel sont signalées des crises
de goutte articulaires typiques non traitées.
Dans les urines :
- le pH urinaire à jeun le matin est à 4,5
- l'uraturie des 24 heures à 900 mg (5 355 micromoles)
- le rapport uraturie (mg/l) / créatininurie (mg/l) = 0,7
- la calciurie est à 3 mg/kg/24 heures
- l'examen bactériologique des urines est normal.
Dans le plasma :
- l'urée, la créatininémie sont normales
- l'uricémie est à 80 mg/l la calcémie est à 95 mg/l (12,37 mmoles/l)
- la phosphorémie est à 25 mg/l (0,8 mmoles/l).
Une urographie intraveineuse récente objective la présence d'une lacune intrapyélique gauche.

Quelles sont les 2 hypothèses diagnostiques possibles chez ce patient au vu de l'U.IV. ?


A - Lithiase oxalo-calcique
B - Tumeur de la voie excrétrice
C - Lithiase xanthique
D - Lithiase urique
E - Nécrose papillaire
Bonne(s) réponse(s) : B D

On rappelle que les lithiases radiotransparentes sont les lithiases uriques et xanthiques, les lithiases radio opaques sont
oxalocalciques, phosphocalciques et cystiniques.
L'item C est éliminé en raison de sa rareté et de sa survenue dans un contexte différent (enfance). La xanthinurie, enfin,
s'accompagne d'une hypouricémie et d'une hypouraturie.

Quel examen paraclinique supplémentaire vous paraît indispensable pour trancher entre ces deux hypothèses ?
A - Rapport calcium/créatinine dans les urines
B - Epreuve d'acidification au NH4CL
C - Scanner
D - Artériographie rénale gauche sélective
E - Urétéropyélographie rétrograde à gauche
Bonne(s) réponse(s) : C

Sans commentaire.

L'hypothèse diagnostique d'une lithiase reconnaît comme élément(s) caractéristique(s) :


A - Le pH urinaire
B - La calciurie
C - La phosphorémie
D - L'uraturie
E - L'absence d'infection urinaire
Bonne(s) réponse(s) : A D

L'acidité favorise la précipitation de l'acide urique.


L'uraturie élevé (> 750 mg par jour) aussi...

Quel traitement proposez-vous ?


A - Diurèse alcaline
B - Abord chirurgical du bassinet gauche
C - Montée de sonde par voie basse
D - Thiazidiques
E - Anti-infectieux urinaires
Bonne(s) réponse(s) : A

Cette diurèse devra s'accompagner d'une modification des habitudes alimentaires et on doit envisager la mise en route d'un
traitement hypouricémiant par inhibiteurs de la xanthine oxydase. Ce traitement devra bien sûr être débuté sous couvert d'un
traitement adjuvant par colchicine.

243
Exclusivement sur DOC - DZ : www.doc-dz.com NADJI 85
RESIDANAT EN POCHE TOME II
Cas Clinique en QCM
Une femme de 30 ans, sans antécédents, présente une douleur brutale de la fosse iliaque droite et du flanc droit, avec
nausées, une fébricule à 38°C, sans trouble mictionnel.

Vous l'examinez. Bien que n'ayant pas de valeur spécifique, quel signe évoque une colique néphrétique ?
A - Douleur au toucher pelvien
B - Douleur au point urététal moyen droit
C - Ebranlement douloureux lombaire droit
D - Défense abdominale droite
E - Aucune des réponses précédentes
Bonne(s) réponse(s) : B C

A et D - Sont évocateurs d'une pathologie digestive.


B et C -Sont deux réponses possibles...

Quel examen radiologique demander pour affirmer le diagnostic de colique néphrétique et sa cause au moment
de la crise ?
A - Echographie rénale
B - Radiographie d'abdomen sans préparation
C - Urographie intraveineuse
D - Urétéropyélographie rétrograde
E - Scintigraphie rénale
Bonne(s) réponse(s) : A

Au cours d'une colique néphrétique simple, aucun examen ou la simple pratique d'un ASP peuvent suffire, l'enquête
étiologique étant reprise ultérieurement. Dans un contexte fébrile, la pratique d'une échotomographie permettra au mieux de
préciser la présence ou l'absence d'une stase d'urine infectée. A la phase aiguë de la colique néphrétique, I'UIV ne montre
souvent qu'un aspect de rein muet.

Quel examen biologique demander en urgence ?


A - Numération formule sanguine
B - Cytologie urinaire
C - Examen cytobactériologique des urines
D - Uricémie
E - Calcémie et phosphorémie
Bonne(s) réponse(s) : C

Sans commentaire.

Le diagnostic d'obstacle par un calcul de 5 mm de diamètre au niveau de l'uretère lombaire droit a été posé .
Vous décidez de :
A - Prescrire une antibiothérapie
B - Instituer immédiatement une cure de diurèse
C - Opérer d'urgence
D - Monter une sonde urétérale par voie endoscopique
E - Restriction hydrique
Bonne(s) réponse(s) : A E

La cure de diurèse s'impose au décours et non au cours d'une colique néphrétique. L'antibiothérapie sera débutée après
prélèvement.

Dans quel(s) cas l'indication chirurgicale devient-elle nécessaire ?


A - Hématurie microscopique persistante
B - Poussées fébriles
C - Distention persistante des cavités après 2 mois
D - Douleurs quotidiennes après 3 mois d'évolution
E - Mictions impérieuses
Bonne(s) réponse(s) : B C D

Il s'agit d'indications retardées à la chirurgie.


On rappelle les indications en urgence, au cours d'une colique néphrétique :
- anurie par obstacle
- rétention d'urine infectée avec syndrome infectieux sévère
- anurie
- douleurs rebelles aux antalgiques

244
Exclusivement sur DOC - DZ : www.doc-dz.com NADJI 85
RESIDANAT EN POCHE TOME II
Cas Clinique en QCM
Un homme de la cinquantaine consulte pour une hématurie totale avec caillots suivie d'une douleur de colique néphrétique
droite. C'est le premier épisode de ce type. Il n'a jamais émis de calcul. Il n'a pas d'antécédent médicaux ni chirurgicaux
remarquables mais c'est un gros fumeur et un gros mangeur. L'état général est bien conservé.
Au moment de l'examen il ne souffre plus. Les fosses lombaires sont souples et indolores. On ne perçoit pas de gros rein à
droite. Les organes génitaux externes sont normaux. Les urines sanglantes sont émises avec un bon jet. Au toucher rectal, la
prostate est discrètement hypertrophiée et souple. La vessie est souple. La tension artérielle est normale. Le cliché sans
préparation ne montre pas de calcification. En urographie, on découvre comme seule anomalie une lacune isolée de 10 mm
non obstructive du calice inférieur du rein droit. Les urines sont stériles elles ne contiennent pas de pus ni de cellules
anormales.

Indiquez le ou les diagnostic(s) qui peu(ven)t être évoqué(s) :


A - Un cancer du rein
B - Une tuberculose rénale
C - Une lithiase urique
D - Une billharziose urinaire
E - Une tumeur de la voie excrétrice
Bonne(s) réponse(s) : A C E

Bien que non classique, le cancer du rein peut produire des images de lacune à l'UIV par envahissement.

Pour préciser la nature de la lacune calicielle et la rapporter à sa cause, il est inutile de demander :
A - Une étude du pH urinaire
B - Une échographie rénale
C - Une artériographie
D - Une tomodensitométrie abdominale
E - Une cystoscopie
Bonne(s) réponse(s) : A B C D

S'il s'agit d'une tumeur de la voie excrétrice du calice ou du bassinet, la pratique de E sera utile pour en apprécier l'extension.
Cependant ce QCM est discutable car il ne propose par de hiérarchie dans la stratégie diagnostique. Ainsi, l'artériographie,
qui auparavant occupait une place de choix est actuellement abandonnée pour le scanner. De même la pratique simultanée
de l'échographie et du scanner est redondante.

Lorsque les examens complémentaires habituels n'ont pas permis de trancher en faveur d'une étiologie,
indiquez l'attitude que vous adoptez :
A - Traitement antituberculeux d'épreuve et nouvelle U.I.V. 6 mois plus tard
B - Abstention avec répétition des cytologies urinaires tous les 3 mois
C - Traitement antibilharzien et nouvelle U.I.V. 1 mois plus tard
D - Alcalinisation des urines en permanence et nouvelle U I V. 2 mois plus tard
E - Exploration chirurgicale immédiate par abord direct du rein et pyélotomie
Bonne(s) réponse(s) : D

Compte tenu du faible taux de faux négatifs du scanner pour les cancers du rein (0,5%), une attitude d'expectative est justifiée.

Si vous avez opté pour une attitude d'expectative et une urographie de contrôle vous montre que la lacune, non
seulement n'a pas disparu, mais s'est aggravée et déborde dans le bassinet. Le diagnostic devient alors
évident. Indiquez la sanction thérapeutique qui en découle :
A - Montée de sonde intrarénale pour perfusion d'une substance lytique
B - Pyélotomie avec exérèse de la lésion
C - Néphro-urétérectomie totale élargie
D - Néphrectomie polaire inférieure
E - Irradiation isolée de la région
Bonne(s) réponse(s) : C

Sans commentaire.

245
Exclusivement sur DOC - DZ : www.doc-dz.com NADJI 85
RESIDANAT EN POCHE TOME II
Cas Clinique en QCM
Un épithélioma glandulaire prostatique est découvert après résection endoscopique chez un homme suivi depuis plusieurs
années pour un adénome. Cette résection avait été motivée par la modification des signes cliniques au toucher rectal et
l'aggravation des troubles mictionnels.

La découverte de cet épithélioma prostatique impose la réalisation d'un bilan d'extension. Pouvez vous indiquer
les deux localisations métastatiques les plus fréquentes dans ce type de cancer ?
A - Ganglions
B - Parenchyme pulmonaire
C - Parenchyme hépatique
D - Squelette
E - Cerveau
Bonne(s) réponse(s) : A D

Sans commentaire.

L'existence de localisations métastatiques fonctionnellement gênantes, voire dangereuses conduit à proposer


un traitement hormonal. Pouvez-vous indiquer la (les) thérapeutique(s) hormonale(s) utilisée(s) dans le cancer
de la prostate parmi celles qui sont énumérées ci-dessous ?
A - Pulpectomie
B - Androgènes
C - Anti-androgènes
D - OEstrogènes
E - Antioestrogènes
Bonne(s) réponse(s) : A C D

Sans commentaire.

La surveillance du traitement hormonal implique la réalisation régulière de dosages sériques, témoins de


l'efficacité thérapeutique. Pouvez-vous indiquer parmi ceux proposés, le ou les éléments essentiels de cette
surveillance ?
A - Testostérone
B - Phosphatases acides
C - T.S.H.
D - Alphafoetoprotéine
E - Bêta HCG
Bonne(s) réponse(s) : A B

Sans commentaire.

En dépit de l'hormonothérapie entreprise, des signes neurologiques déficitaires apparaissent au niveau des
membres inférieurs et le bilan paraclinique précise qu'il existe un blocage en D 12. Que faut-il faire ?
A - Prescrire du méthotrexate intrarachidien
B - Prescrire des corticoïdes en injection intrarachidienne
C - Prescrire une chimiothérapie par voie générale
D - Faire pratiquer une laminectomie décompressive
E - Prescrire du strontium radioactif
Bonne(s) réponse(s) : D

Toutefois on doit discuter l'indication dans ce cas d'une radiothérapie locale palliative.

246
Exclusivement sur DOC - DZ : www.doc-dz.com NADJI 85
RESIDANAT EN POCHE TOME II
Cas Clinique en QCM
Monsieur G. est né en 1916. Il est adressé à la consultation pour une insuffisance rénale. C'est un sujet maigre, 52 kg, ancien
gros fumeur. Les derniers taux communiqués sont les suivants : urée 14,5 mmol pour mille, créatinine 458 micromoles pour
mille, potassium 5,6 mmol pour mille. 15 jours avant ce résultat, il est entré en clinique pour une crise douloureuse
abdominale droite avec une poussée hypertensive à 200/140 (chiffre habituel en moyenne depuis 10 ans : 155/100). A cette
date, on notait : urée 8,26 mmoI/l, créatinine 170 micromoles et potassium 4,4 mmol pour mille. Ce sujet ne présente pas
d'antécédent rénal. Un bilan digestif s'avère normal.
En raison d'une infection urinaire à 10 exposant 5/ml E.coli, un traitement par Gentalline® 120 mg 2 fois par jour a été institué
pendant une semaine. L'électrophorèse est normale, il n'y a pas de protéinurie.

Si vous aviez à choisir un premier traitement de l'infection urinaire, avant antibiogramme et autres données,
quel serait votre choix ?
A - Le traitement adopté
B - Une dose identique administrée toutes les 4 heures
C - La même dose en administration intraveineuse
D - La même dose associée à une céphalosporine : céphaloridine (Céporine®) 8 g par jour
E - Amoxycilline (Clamoxyl®) un gramme toutes les 6 heures
Bonne(s) réponse(s) : E

Il convient de souligner les doses beaucoup trop importantes utilisées chez ce sujet : 4,5 mg/kg/j chez un insuffisant rénal...

Une échographie est pratiquée. Le rein droit a un diamètre bipolaire de 63 mm, le rein gauche 108 mm, il n'y a
pas de dilatation des cavités excrétrices. Avant les examens complémentaires, quelles sont les deux
hypothèses recevables parmi les suivantes pour le diagnostic étiologique ?
A - Pyélonéphrite ascendante avec reflux vésico-urétéral droit
B - Glomérulonéphrite chronique hypertensive
C - Thrombose veineuse rénale droite
D - Thrombose artérielle rénale droite
E - Hydronéphrose par anomalie de la jonction
Bonne(s) réponse(s) : A D

Sans commentaire.

Une urographie pratiquée pendant le séjour révèle une morphologie pyélocalicielle normale à gauche et à droite
l'absence de temps néphrographique et d'excrétion. Ces données :
A - Evoquent une lithiase rénale droite méconnue
B - Confirment l'hypothèse d'une pyélonéphrite sur reflux
C - Confirment l'hypothèse d'une thrombose veineuse rénale
D - Confirment l'hypothèse d'une hypotrophie rénale congénitale
E - Confirment l'hypothèse d'une thrombose artérielle rénale droite
Bonne(s) réponse(s) : E

Sans commentaire.

Le rein gauche est selon vous certainement :


A - Fonctionnellement normal
B - Porteur de lésions de néphro-angiosclérose
C - Porteur de lésions de rein de choc
D - Porteur de lésions de glomérulonéphrite mésangiale à dépôts d'lgA
E - Porteur de lésions de pyélonéphrite chronique
Bonne(s) réponse(s) : B C

Sans commentaire.

On apprend qu'au cours du bilan hospitalier une artériographie a également été pratiquée. Choisissez deux
propositions exactes. La poussée d'insuffisance rénale peut être due à :
A - Un obstacle excrétoire
B - Au traitement antibiotique
C - Aux examens radiologiques
D - A l'infection urinaire
E - A la précipitation de protéines myélomateuses
Bonne(s) réponse(s) : B C

Produit iodé et gentamicine sont toxiques pour le rein.

247
Exclusivement sur DOC - DZ : www.doc-dz.com NADJI 85
RESIDANAT EN POCHE TOME II
Cas Clinique en QCM
Un homme de 62 ans, bien suivi médicalement présente depuis trois ans une hypertension artérielle d'importance moyenne
qui a été contrôlée par une association diurétique : Cyclotériam® (triamtérène + cyclothiazide) un comprimé le matin et un
hypotenseur d'action centrale : Catapressan® (clonidine) un comprimé le soir. Ce sujet présente l'hiver, depuis 15 ans au
moins, quelques épisodes de bronchite asthmatiforme régulièrement traitée par des antibiotiques et parfois des corticoïdes.
Entre ces épisodes, il garde une dyspnée d'effort légère. Il n'y a pas d'autres antécédents sauf une crise de colique
néphrétique droite à l'âge de 25 ans, au cours d'un voyage en Grèce au mois d'août.
Une urographie intraveineuse minutée a été réalisée, à la découverte de l'hypertension, montrant deux reins de taille égale, de
morphologie normale et de fonctionnement symétrique. La prise médicamenteuse avait été interrompue après deux ans en
raison de crampes musculaires et de somnolence. Très récemment, alors que survenaient à nouveau des céphalées, la
pression artérielle qui était presque normale a été trouvée à 180/125. Une échographie montre un rein gauche diminué de
taille et un rein droit apparemment normal.

L'asymétrie constatée de la taille des reins peut s'expliquer le plus probablement par :
A - Une lithiase rénale méconnue
B - Une rétention urinaire par obstacle prostatique
C - Une thrombose veineuse rénale
D - Une sténose ou une thrombose artérielle rénale
E - Une pyélonéphrite chronique
Bonne(s) réponse(s) : D

La réapparition de l'hypertension artérielle associée à l'asymétrie rénale, évoque de principe une sténose de l'artère rénal.

Dans ce contexte, un ionogramme sanguin et urinaire est pratiqué. La créatinine plasmatique est trouvée à 118
mmol/l. Parmi les anomalies biologiques suivantes, laquelle est la plus vraisemblablement retrouvée chez ce
malade ?
A - Une acidose hyperchlorémique
B - Une hyponatrémie
C - Une diminution du débit urinaire de potassium
D - Une hypokaliémie
E - Une fuite sodée urinaire de type diabète sodé
Bonne(s) réponse(s) : D

La sténose de l'artère rénale entraîne un hyperaldostéronisme.

Vous voulez préciser la valeur fonctionnelle du rein gauche. Quel moyen allez-vous choisir ?
A - Une clairance de la créatinine endogène des 24 heures
B - Une urographie intraveineuse
C - Une biopsie rénale de chaque rein
D - Un néphrogramme isotopique
E - Une clairance à l'inuline avec recueil urinaire par miction volontaire
Bonne(s) réponse(s) : D

Sans commentaire.

Quel est selon vous le meilleur moyen diagnostique pour préciser la cause de l'aggravation récente de
l'hypertension ?
A - Une artériographie rénale
B - Un scanner corps entier
C - Une phlébographie
D - Un débit cardiaque isotopique
E - Un enregistrement de pression artérielle à I'effort
Bonne(s) réponse(s) : A

Sans commentaire.

Des examens complémentaires ont montré une hypersécrétion de rénine au niveau du rein gauche et une
rénine basse dans la veine rénale droite. En tenant compte du contexte, quel traitement médical vous parait le
plus adéquat ?
A - Dihydralazine
B - Aténolol®
C - Clonidine®
D - Hydrochlorothiazide
E - Régime désodé enrichi en potassium
Bonne(s) réponse(s) : B

Il s'agit en fait d'une bonne indication à un traitement par les inhibiteurs de l'enzyme de conversion. Les autres thérapeutiques
ici proposées ne sont pas satisfaisantes. Toutefois l'utilisation d'un bêtabloquant cardiosélectif (Aténolol®), éventuellement
associé à la dihydralazine...donc B, est possible, mais ce n'est pas satisfaisant.
248
Exclusivement sur DOC - DZ : www.doc-dz.com NADJI 85
RESIDANAT EN POCHE TOME II
Cas Clinique en QCM
Une femme de 75 ans, sans passé néphrologique est traitée pour insuffisance cardiaque depuis deux mois par Digoxine® 1
comprimé par jour, Lasilix® 1 comprimé par jour et un régime désodé parfaitement suivi. Depuis une semaine elle présente
des vomissements, une diarrhée et la diurèse est de 500 ml par jour depuis 48 heures. Devant ce tableau clinique, elle est
adressée en réanimation néphrologique. A son entrée, la pression artérielle est à 10/6 couchée, le pouls à 50/mn, la
conscience est normale. Le bilan biologique montre :
créatininémie 320 micromol/l, natrémie 120 mmol/l, kaliémie 6 mmol/l, réserve alcaline 15 mmol/l, protidémie 79 g/l, azotémie
45 mmol/l, globules rouges 5 millions, hématocrite 49%.

Quel(s) est(sont) le(s) trouble(s) de l'hydratation que l'on peut évoquer devant ce tableau clinique et biologique ?
A - Déshydratation intracellulaire
B - Déshydratation extracellulaire
C - Déshydratation globale
D - Hyperhydratation intracellulaire
E - Hyperhydratation globale
Bonne(s) réponse(s) : B D

Sans commentaire.

Comment peut-on expliquer la bradycardie ?


A - Elle est la conséquence de l'hyponatrémie
B - Elle est la conséquence de l'hyperazotémie
C - Elle est la conséquence de la diarrhée
D - Elle est la conséquence de l'intoxication digitalique
E - Elle est la conséquence des vomissements
Bonne(s) réponse(s) : D

Sans commentaire.

Quelle(s) est(sont) le(s) mesure(s) thérapeutique(s) à prendre ?


A - Perfusion de soluté glucosé isotonique
B - Injection quotidienne d'une ampoule de furosémide pendant 3 jours
C - Apport sodé
D - Restriction hydrique
E - Arrêt des diurétiques
Bonne(s) réponse(s) : C D E

Sans commentaire.

Parmi les propositions suivantes, quelle(s) est(sont) celle(s) qui permettrait(ent) d'évoquer une insuffisance
rénale aiguë fonctionnelle ?
A - Rapport urée urinaire/urée sanguine 10
B - Rapport créatininurie/créatininémie 10
C - Fraction d'élimination urinaire du sodium inférieur à 1
D - Natrémie inférieure à 130
E - Urée sanguine/créatininémie 10
Bonne(s) réponse(s) : A B C

Sans commentaire.

Un homme de 30 ans présente brusquement un syndrome hyperalgique du flanc droit, avec irradiation dans le testicule et
s'accompagnant de pollakiurie. Sa température est à 37°C. Un épisode identique mais moins intense a eu lieu trois mois
auparavant. On ne retrouve par ailleurs aucun antécédent remarquable sur le plan personnel ou familial. Le diagnostic de
colique néphrétique est le plus vraisemblable.

Pour étayer votre diagnostic vous demandez après avoir soulagé votre malade, une urographie intraveineuse.
Quel libellé vous paraît le meilleur ?
A - UIV avec compression
B - UIV minutée avec clichés précoces
C - UIV avec wash-out
D - UIV avec clichés tardifs
E - UIV standard
Bonne(s) réponse(s) : D

Sans commentaire.

249
Exclusivement sur DOC - DZ : www.doc-dz.com NADJI 85
RESIDANAT EN POCHE TOME II
Cas Clinique en QCM

On constate I'existence d'une lithiase transparente de la taille d'une cerise siègeant dans le bassinet droit sans
distension marquée des cavités sus-jacentes. Quelle est la nature chimique probable de ce calcul ?
A - Oxalocalcique
B - Phospho-amoniaco-magnésienne
C - Urique
D - Cystinique
E - Phosphocalcique
Bonne(s) réponse(s) : C

On rappelle que les lithiases radiotransparentes sont les lithiases uriques et xanthiques, les lithiases radio-opaque sont
oxalocalciques, phosphatocalciques et cystiniques. Les lithiases phosphoamoniaco-magnèsiennes sont radio-opaques de
façon variable.

Quelle attitude devra-t-on adopter vis-à-vis de cette lithiase ?


A - Réaliser rapidement une pyélotomie
B - Attendre une migration spontanée
C - Proposer une alcalinisation prolongée
D - Mettre le malade sous diurétiques
E - Monter une sonde urétérale
Bonne(s) réponse(s) : C

Sans commentaire.

Quelle(s) complication(s) évolutive(s) amènerai(en)t à modifier l'attitude précédente ?


A - Hématurie
B - Expulsion de débris de calculs
C - Infection urinaire avec fièvre à 39°C
D - Distension pyélocalicielle
E - Leucocyturie augmentée
Bonne(s) réponse(s) : C D

Sans commentaire.

Vous proposez, pour éviter, en cas d'hyperuricémie, la récidive lithiasique chez ce patient en dehors des cures
de diurèse :
A - Diurétiques thiazidiques
B - Uricoéliminateurs
C - Prescription d'un inhibiteur de la xanthine oxydase
D - Désinfection urinaire continue
E - Alcalinisation
Bonne(s) réponse(s) : C E

Sans commentaire.

Un patient de 60 ans, en bon état général, présente depuis 2 ans une dysurie objectivée par la débitmétrie (débit maximum =
8 ml/s pour une miction de 150 cc) et une pollakiurie nocturne (4 à 5 fois). Il signale, par ailleurs, des épisodes d'hématurie
terminale. Le toucher rectal trouve un adénome évalué en échographie à 80 g. Les urines sont troubles à l'émission et
l'examen cytobactériologique des urines met en évidence une infection à colibacille (105 germes/ml). La fonction rénale est
normale. L'urographie intraveineuse montre un haut appareil fin avec un syndrome tumoral pur du pôle inférieur du rein
gauche, une lithiase vésicale radio-opaque de 3 cm de diamètre, une vessie de lutte avec large empreinte prostatique bilobée
et un résidu post-mictionnel d'environ 100 cc.

L'hématurie terminale de ce patient peut a priori être rapportée à :


A - L'adénome de prostate
B - La lithiase vésicale
C - Au syndrome tumoral du rein gauche
D - A une éventuelle tumeur de vessie associée
E - Aucune de ces causes
Bonne(s) réponse(s) : B D

Sans commentaire.

250
Exclusivement sur DOC - DZ : www.doc-dz.com NADJI 85
RESIDANAT EN POCHE TOME II
Cas Clinique en QCM

Le syndrome tumoral radiologique pur du pôle inférieur du rein gauche :


A - Explique l'hématurie terminale
B - Nécessite une urétéropyélographie rétrograde
C - Nécessite, comme 1er examen complémentaire, une échographie
D - Nécessite une simple surveillance si la masse est liquide en échographie
E - Nécessite une artériographie si la masse est liquide en échographie
Bonne(s) réponse(s) : C D

Sans commentaire.

La conséquence sexuelle habituelle de l'adénomectomie prostatique est :


A - L'impuissance
B - L'anéjaculation
C - La disparition de la libido
D - L'éjaculation rétrograde
E - Aucune des propositions précédentes
Bonne(s) réponse(s) : D

Complication constante !, sauf en cas de ligature des déférents associée, ce qui se pratique pour les interventions par voie
haute, où il y a alors anéjaculation.

D'une façon générale, la(es) conséquence(s) possible(s) de l'obstacle représenté par un adénome de prostate
et visible(s) sur les clichés d'U.I.V. est(sont) :
A - La vessie de lutte
B - La lithiase vésicale
C - La distension urétéropyélocalicielle bilatérale
D - Le diverticule de vessie
E - Le résidu post-mictionnel
Bonne(s) réponse(s) : A B C D E

C'est devenu très rare actuellement !

En faveur de l'indication opératoire on retient :


A - L'existence d'une lithiase vésicale
B - Le volume de l'adénome
C - L'intensité des signes fonctionnels
D - L'âge du patient
E - L'existence d'une infection urinaire récidivante
Bonne(s) réponse(s) : A C E

Sans commentaire.

251
Exclusivement sur DOC - DZ : www.doc-dz.com NADJI 85
RESIDANAT EN POCHE TOME II
Cas Clinique en QCM
Ce patient de 62 ans sans autre antécédent pathologique qu'un excès pondéral ancien et une probable crise de goutte du
gros orteil droit 4 ans auparavant présente de violentes douleurs de l'hypochondre droit et de la fosse iliaque droite qui, se
prolongeant pendant plusieurs heures, motivent son hospitalisation. Le bilan biologique systématique révèle alors comme
seule anomalie une insuffisance rénale avec azotémie à 37 mmol/l, créatininémie à 620 micromol/l, sans perturbations
phosphocalciques, sans hyperkaliémie, sans acidose. L'ASP ne montre aucune image pathologique. Sous traitement
antispasmodique et antalgique les douleurs cèdent et le lendemain se produit une débacle urinaire de 5,2 litres suivie en 6
jours du retour à la normale des taux de l'azotémie et de la créatininémie. L'UIV, qui est alors pratiquée, révèle 2 reins de taille
et de contour normaux et symétriques, des cavités excrétrices normales à droite mais à gauche la présence d'un volumineux
calcul coralliforme radiotransparent qui occupe pratiquement l'ensemble des calices et du bassinet. L'uricémie est trouvée à
625 micromol/l. On retient alors le diagnostic d'insuffisance rénale aiguë obstructive sur lithiase coralliforme urique gauche et
probable migration calculeuse à droite. Quelques examens complémentaires sont alors pratiqués :
- uricémies : 650 - 590 - 640 micromol/l
- uricuries : 2450 - 2300 - 2130 micromol/l
- pH urinaire : 25 déterminations, toutes comprises entre 5 et 5,6
- calcémies : 2,27 - 2,30 - 2,32 mmol/l
- oxaluries/24 h : normales.

L'analyse de l'ensemble de ces données cliniques et paracliniques vous permet de retenir comme principe
thérapeutique :
A - Traitement chirurgical de la lithiase coralliforme gauche
B - Traitement médical seul
C - Traitement médical et chirurgical
D - Traitement purement diététique
E - Mesures diététiques et traitement médical
Bonne(s) réponse(s) : E

Sans commentaire.

Vous allez recommander et prescrire :


A - Traitement alcalinisant permanent des urines
B - Traitement antiseptique urinaire prolongé
C - Cure de diurèse permanente
D - Traitement uricosurique par Amplivix®
E - Traitement urico-inhibiteur par Zyloric®
Bonne(s) réponse(s) : A C E

Sans commentaire.

Vous retiendrez comme critère(s) de surveillance et d'efficacité de votre prescription :


A - Diurèse de 24 h constamment supérieure à 1,5 litre
B - Diurèse de 24 h constamment supérieure à 2,5 litre
C - PH urinaire se situant en permanence à 5,5
D - PH urinaire se situant en permanence à 6,5
E - Normalisation de l'uricémie
Bonne(s) réponse(s) : B D E

Sans commentaire.

Six mois après le début du traitement que vous avez prescrit, vous désirez faire le point sur son efficacité. Parmi
les investigations suivantes, laquelle faites-vous pratiquer ?
A - Urétéropyélographie rétrograde
B - Urographie intraveineuse
C - Cliché de l'arbre urinaire sans préparation
D - Examen des urines à la recherche de cristaux
E - Epreuve de cristallurie provoquée
Bonne(s) réponse(s) : B

Sans commentaire.

252
Exclusivement sur DOC - DZ : www.doc-dz.com NADJI 85
RESIDANAT EN POCHE TOME II
Cas Clinique en QCM
Un homme de 70 ans bénéficie d'une intervention pour arthrose de hanche. Dans ces antécédents on retient un tabagisme (1
paquet de cigarettes/jour) et une hypertension artérielle apparue il y a 20 ans, stabilisée à 180/100 mm de Hg par un
traitement quotidien qui associe 5 mg de chlorydrate d'amiloride et 50 mg d'hydrochlorothiazide (Modurétic®).
L'examen clinique préopératoire est sans particularité. Des examens paracliniques, on retient les éléments suivants :
créatinine 105 micromol/l, cardiomégalie modérée sur la radiographie thoracique et un ECG sans particularité à l'exception
d'un indice de Sokolow à 45.
En post-opératoire le malade est laissé au même traitement hypotenseur. Il reçoit de plus du kétoproféne 5 gélules à 50 mg/j
(Profénid®).
Au 10ème jour post-opératoire l'examen clinique et l'électrocardiogramme ne sont pas modifiés. La tension artérielle est à
140/80 en position couchée et à 90/60 en position assise. Le bilan biologique met en évidence les éléments suivants :
créatinine à 610 mic mol/l, Na+ 128 mmol/l, K+ 6,1 mmol/l, protidémie 80 g/l, Cl 92 mmol/l, bicarbonates 32 micromol/l,
uricémie 582 micromol/l, hématocrite 0,52. La protéinurie des 24 heures est à 0,58 g. Il n'existe pas d'hématurie.

Ce malade présente une insuffisance rénale aiguë. Parmi les étiologies suivants, vous retenez :
A - Néphroangiosclérose
B - Toxicité rénale du kétoprofène
C - Toxicité rénale de l'hydrochlorothiazide
D - Toxicité rénale du chlorydrate d'amiloride
E - Hypovolémie
Bonne(s) réponse(s) : A B E

Il s'agit vraisemblablement d'une insuffisance rénale multifactorielle sur fond d'insuffisance rénale chronique par
néphroangiosclérose ; survenue d'une insuffisance rénale par la conjugaison d'une déshydratation sévère en rapport avec
l'utilisation de diurétiques et d'inhibiteurs de la cyclooxygénase.

Le traitement de cette insuffisance rénale repose dans les 12 premières heures sur :
A - Perfusion du soluté salé isotonique
B - L'hémodialyse
C - L'arrêt du kétoprofène
D - L'arrêt de l'hydrochlorothiazide
E - La prescription de furosémide (Lasilix®)
Bonne(s) réponse(s) : A C D

Sans commentaire.

L'hyperkaliémie impose :
A - L'alcalinisation rapide par lactate de soude molaire en perfusion
B - L'utilisation de Kayexalate®
C - Le régime sans potassium
D - L'arrêt du chlorydrate d'amiloride
E - Le recours à l'hémodialyse
Bonne(s) réponse(s) : C D

L'hyperkaliémie doit d'abord être appréciée en fonction des chiffres du pH.


Le traitement devra alors tenir compte de la reprise ou non de la diurèse. Si la kaliémie ne diminuait pas avec la correction de
l'hypovolémie et avec la reprise de la diurèse, on serait alors amené à utiliser du kayexalate®.

Le malade est revu en consultation au 45ème jour post-opératoire. La tension artérielle est à 180/110 mm Hg en
position couchée et debout. La créatininémie est à 310 - micromol/l, la kaliémie à 5,6 mmol/l. Le malade est
sous Modurétic®.
Quelle attitude thérapeutique retenez-vous ?
A - Adjonction de 100 mg/j de spironolactone (Aldactone®) au traitement en cours
B - Poursuite de l'hydrochlorothiazide seul
C - Non modification du traitement
D - Remplacement du Moduretic® par 100 mg de chlortalidone (Hygroton®)
E - Aucune de ces hypothèses ne vous semble exacte
Bonne(s) réponse(s) : E

Il convient de suspendre chez ce patient l'administration de tout diurétique épargneur du potassium. Les thiazides sont
inefficaces dans l'insuffisance rénale.

253
Exclusivement sur DOC - DZ : www.doc-dz.com NADJI 85
RESIDANAT EN POCHE TOME II
Cas Clinique en QCM
Un homme de 32 ans est hospitalisé pour hyperazotémie de mise en évidence récente. Il ne présente comme antécédent
connu qu'une crise de colique néphrétique non compliquée 10 ans auparavant.
Le début de son affection a été marquée par un épisode hyperthermique aigu d'allure grippale, traité par anti-inflammatoires
non stéroïdiens et ampicilline. Quelques jours plus tard apparait un épisode d'hématurie macroscopique sans caillots ni
douleurs lombaires. Une oligurie est d'emblée signalée par le malade.
A L'examen clinique :
- tension artérielle 16/10, petit galop de pointe.
- discrets oedèmes des membres inférieurs.
Dans les urines :
- protéinurie +++
- hématurie +++ à la bandelette
- test aux nitrites négatif.
Dans le plasma :
- urée : 1,5 g/l
- créat : 50 mg/l
- Na : 128 mEq/l
- protidémie : 58 g/l
- albuminémie : 26 g/l
- K : 5,7 mEq/l.

A ce stade vous évoquez :


A - Glomérulonéphrite aiguë
B - Glomérulonéphrite chronique en poussée
C - Nécrose tubulaire aiguë
D - Insuffisance rénale aiguë par obstacle
E - Néphrite interstitielle aiguë immuno-allergique
Bonne(s) réponse(s) : A B

C'est le seul item que l'on puisse raisonnablement discuter devant l'apparition, au décours d'un syndrome infectieux, d'une
insuffisance rénale, avec HTA, hématurie, protéinurie, hypoalbuminémie. La seule autre hypothèse que l'on pourrait discuter
serait l'acutisation d'une glomérulopathie chronique antérieure.

A ce stade la démarche diagnostique doit faire intervenir le résultat de :


A - Echographie rénale
B - Complément sérique
C - Antistreptolysines
D - Urée urinaire
E - Fond d'oeil
Bonne(s) réponse(s) : A B E

Sans commentaire.

Chez ce patient vous pouvez déjà conclure à l'existence de :


A - Hyperhydratation globale
B - Déplétion sodée
C - Infection urinaire
D - Hyperkaliémie
E - Hémorragie digestive
Bonne(s) réponse(s) : A D

Sans commentaire.

Une fois affirmé le caractère aigu de I'atteinte rénale, quel(s) diagnostic(s) étiologique(s) pouvez-vous retenir
comme possible(s) a priori ?
A - Glomérulonéphrite rapidement progressive ou glomérulonéphrite aiguë maligne
B - Nécrose tubulaire aiguë
C - Néphroangiosclérose maligne
D - Néphrite interstitielle aiguë
E - Nécrose corticale rénale
Bonne(s) réponse(s) : A

L'item A correspond à des lésions diffuses prolifératives extracapillaires. Les items B et E ne correspondent pas au tableau
clinique. L'HTA n'est pas suffisamment élevée pour l'item C. L'item D ne s'accompagne pas de protéinurie aussi importante,
avec hypoalbuminémie.

254
Exclusivement sur DOC - DZ : www.doc-dz.com NADJI 85
RESIDANAT EN POCHE TOME II
Cas Clinique en QCM

Quel examen parmi les suivants vous paraît alors fondamental dans la démarche diagnostique ?
A - Artériographie rénale
B - Recherche d'immuns complexes circulants
C - Biopsie rénale
D - Recherche d'anticorps antinucléaires
E - Recherche d'anticorps antibasale glomérulaire (anticorps anti-GMB)
Bonne(s) réponse(s) : C

Seule la ponction biopsie rénale permet la certitude diagnostique. Elle permettra une évaluation pronostique et guidera les
options thérapeutiques.

Un homme de 73 ans est opéré d'une cholécystectomie pour lithiase vésiculaire. En pré-opératoire, le poids est de 70 kg, la
pression artérielle à 180/110 mmHg. L'interrogatoire retrouve une pollakiurie avec trois mictions nocturnes, tous les autres
éléments cliniques et paracliniques et le toucher rectal sont normaux. Au deuxième jour post-opératoire, survient un accès
fébrile avec frissons. Deux hémocultures sont positives à colibacille. Le traitement immédiatement entrepris comprend
quotidiennement l'administration de 240 mg de gentamicine par voie IM et la perfusion de 2000 ml de soluté glucosé
isotonique. Alors que la diurèse est à 1800 ml/24 h au 2ème jour post-opératoire, elle chute progressivement pour n'être qu'à
300 ml/24 h le 5ème jour. Le poids est alors de 73 kg, il n'y a eu ni vomissements, ni diarrhée, la pression artérielle est a
150/90 mmHg. Le bilan biologique révèle les résultats suivants :
Sang (mmol/l) urée 40, chlore 92, sodium 126, potassium 7,5, bicarbonates 18, paCO2 28 mmHg, créatinine 700 umol/l,
protides totaux 52 g/l, pH artériel 7,27.
Urines volume/24 h 300 ml, sodium 80, potassium 30, urée 100 (mmol/litre).

Cette insuffisance rénale aiguë pourrait avoir pour origine :


A - Un choc toxi-infectieux
B - L'administration d'un médicament néphrotoxique
C - L'hyperkaliémie
D - L'acidose métabolique
E - Aucune des causes ci-dessus
Bonne(s) réponse(s) : A B

Il convient de noter la surestimation fréquente de la fonction rénale chez les sujets âgés dont la masse musculaire est réduite,
ce qui conduit à de fréquents surdosages de médicaments dont la posologie doit être très scrupuleusement ajustée à la
fonction rénale.

Pour éliminer un mécanisme excrétoire il faut demander :


A - Radiographie de l'arbre urinaire sans préparation
B - Echotomographie rénale
C - Urétéropyélographie
D - Artériographie rénale
E - Scintigraphie rénale au 99 TC
Bonne(s) réponse(s) : B

C'est actuellement l'examen clé de la démarche diagnostique devant une insuffisance rénale.

La natrémie à 126 mmol/l :


A - Est le reflet d'une dépletion sodée
B - Est le reflet d'une hydratation intracellulaire par hémodilution
C - Influe directement sur le taux de la kaliémie
D - Peut entraîner des symptômes cliniques
E - Justifie des mesures thérapeutiques
Bonne(s) réponse(s) : B D E

La kaliémie à 7,5 mmol/l :


A - Est accentuée par l'acidose métabolique
B - Peut être à l'origine de troubles psychiques aigus
C - Doit être surveillée à l'aide d'électroencéphalogrammes répétés
D - Représente un péril vital à court terme
E - Justifie un traitement spécifique immédiat
Bonne(s) réponse(s) : A D E

Le principal risque de l'hyperkaliémie est la survenue de troubles du rythme cardiaque. Les complications neurologiques des
hyperkaliémies sont des paralysies flasques.

255
Exclusivement sur DOC - DZ : www.doc-dz.com NADJI 85
RESIDANAT EN POCHE TOME II
Cas Clinique en QCM

La gentamicine :
A - A une toxicité accrue chez les sujets âgés
B - Peut être à l'origine d'un syndrôme hépato-rénal
C - Peut être à l'origine d'une glomérulonéphrite aiguë
D - Reste longtemps stockée au niveau du cortex rénal
E - Peut être prescrite chez un sujet ayant une néphropathie connue
Bonne(s) réponse(s) : A D E

La toxicité des aminosides chez les sujets âgés découle de la surestimation de leur fonction rénale souvent altérée (cf
question 78/59). Les aminosides sont réabsorbés et accumulés au niveau du tube contourné proximal. Ils ne sont
néphrotoxiques que si la posologie est excessive ; ils peuvent être prescrits chez l'insuffisant rénal, à condition d'adapter les
doses et de suivre le traitement sur les dosages sériques.

Le traitement de cette insuffisance rénale aiguë comprend :


A - Une restriction des boissons à 500 ml/24 h
B - La perfusion de 2500 ml de soluté bicarbonate isotonique par jour pendant 48 h
C - La perfusion de 500 ml de soluté glucosé à 30 % + 20 unités d'insuline
D - La baisse de la posologie de la gentamicine à 180 mg/jour
E - Transport dans une unité d'épuration (extra) rénale
Bonne(s) réponse(s) : A C E

L'item C est une des possibilités thérapeutiques permettant une baisse rapide de la kaliémie, autorisant ainsi le transport.

Mme M. 31 ans, consulte pour hypertension artérielle. Celle-ci a d'abord été trouvée au cours de l'adolescence et alors
qualifiée de labile. Son père et son grand-père sont hypertendus. Une soeur vivant en Bretagne aurait présenté une toxémie
gravidique.
Une première grossesse a été marquée par une élévation modérée au cours du dernier mois ayant fait prescrire de la
clonidine (Catapressan®) par l'obstétricien. Une deuxième grossesse a dû être interrompue au 4ème mois pour hypertension
incontrôlée et arrêt de croissance foetale. La patiente est vue six mois après l'intervention. La pression artérielle est en
moyenne à 178/118 mm de mercure. Il existe une discrète hypertrophie ventriculaire gauche, des séquelles rétiniennes des
hémorragies et exsudats constatés au cours de la grossesse. Le fond d'oeil est de stade II. Un premier bilan a apporté les
résultats suivants : créatininémie à 100 micromoles/l ; dans les urines des 24 heures : sodium 107 mmol, potassium 98 mmol.
On sait qu'il n'est pas trouvé de souffle vasculaire et que la taille du rein gauche sur la radiographie simple est réduite de 2 cm
dans son grand axe par rapport au droit. Il n'y a pas d'image de calcul. La cytologie urinaire est normale.

Le tableau d'hypertension maligne déclenchée au cours de la deuxième grossesse, évoque à I'origine de


l'hypertension, dans ce cas :
A - Un décollement prématuré du placenta
B - Une incompatibilité rhésus
C - Une malformation utérine
D - Une hypertension gravidique compliquant une hypertension préexistante
E - Aucune des possibilités ci-dessus
Bonne(s) réponse(s) : D

Sans commentaire.

En faveur d'une hypertension secondaire d'origine rénale éventuelle, on note dans l'énoncé clinique un élément
important :
A - La différence de taille des reins
B - La natriurèse
C - La kaliurèse
D - Le terrain familial
E - Le fond d'oeil
Bonne(s) réponse(s) : A

Sans commentaire.

256
Exclusivement sur DOC - DZ : www.doc-dz.com NADJI 85
RESIDANAT EN POCHE TOME II
Cas Clinique en QCM

Une urographie intraveineuse est pratiquée et montre une atrophie du pôle supérieur du rein gauche avec les
calices du groupe supérieur affleurant le bord externe du rein. Le(s) diagnostic(s) suivant(s) paraî(ssen)t alors
plausible(s) compte tenu de la situation déjà décrite :
A - Thrombose de la veine rénale gauche
B - Thrombose cave
C - Hypoplasie segmentaire rénale (ou atrophie rénale segmentaire)
D - Infarctus rénal
E - Fibrose rétropéritonéale
Bonne(s) réponse(s) : C

L'atrophie rénale segmentaire est une cause assez fréquente d'HTA chez le sujet jeune et chez l'enfant. Elle résulte souvent
d'un reflux vésico-urétéral.

Quel peut être selon vous l'examen complémentaire le plus utile pour le diagnostic ?
A - Cystographie rétrograde
B - Scanner
C - Dosage d'activité rénine dans les veines rénales
D - Artérographie rénale
E - Dosage des catécholamines urinaires
Bonne(s) réponse(s) : A

Sans commentaire.

D'après les lésions radiologiques observées, on peut penser que la néphropathie :


A - Est secondaire à l'abus d'analgésiques
B - Accompagne un reflux vésico-urétéral
C - Complique une lithiase
D - Est une néphroangiosclérose
E - Est la séquelle d'une glomérulopathie
Bonne(s) réponse(s) : B

Sans commentaire.

Une jeune femme de 27 ans est adressée à la consultation pour insuffisance rénale. Elle est mariée, sans enfant, elle est
accompagnée de sa mère et elle n'a aucun antécédent pathologique dans sa famille. Elle-même n'a pas d'antécédent
pathologique. On trouve un épisode de cystite non compliquée à l'âge de 20 ans, quelques épisodes d'infection
rhinopharyngée de l'enfance sans jamais de protéinurie. L'examen des urines ne montre ni protéinurie, ni hématurie, ni
bactériurie. Au cours d'un examen biologique systématique pour de multiples symptômes dont la variabilité est importante on
trouve une créatinine plasmatique à 340 micromoles pour mille. La patiente avait eu, deux heures avant cet examen, une
injection intraveineuse de Calcibronate® pour des troubles nerveux et une injection intramusculaire d'une céphalosporine. On
apprend de plus qu'elle consomme des tranquillisants et des neuroleptiques. La pression artérielle est basse : 130/64 mm de
mercure.

Compte tenu des constatations cliniques et biologiques, il s'agit :


A - D'une glomérulonéphrite subaiguë évolutive (mésangioproliférative)
B - D'une glomérulonéphrite mésangiale à dépôts d'lgA (maladie de Berger)
C - D'une glomérulonéphrite extramembraneuse, syndrôme néphrotique à dépôts d'lgG et de C3
extramembraneux
D - D'une glomérulonéphrite maligne, 80 % de croissants épithéliaux
E - Aucune des hypothèses ci-dessus n'est valable
Bonne(s) réponse(s) : E

L'absence de protéinurie et d'hématurie permet a priori d'éliminer une pathologie glomérulaire.

La patiente prend une pilule oestroprogestative faiblement dosée. La recherche d'anticorps antinucléaires et
d'anticorps anti-ADN natif est négative. Vous concluez qu'il s'agit :
A - D'une maladie lupique à localisation rénale
B - D'une amylose
C - D'une angiopathie systémique (périartérite noueuse)
D - D'une glomérulopathie héréditaire
E - Aucune des hypothèses ci-dessus n'est valable
Bonne(s) réponse(s) : E

Même réponse que précédemment

257
Exclusivement sur DOC - DZ : www.doc-dz.com NADJI 85
RESIDANAT EN POCHE TOME II
Cas Clinique en QCM

Par précaution, une échographie a été pratiquée. La taille des reins est normale et symétrique avec un index
corticomédullaire normal. Vous concluez qu'il s'agit :
A - D'une néphrite intestitielle chronique
B - D'une pyélonéphrite chronique ascendante
C - D'une polykystose
D - D'une nécrose papillaire chronique
E - Aucune des hypothèses ci-dessus n'est recevable
Bonne(s) réponse(s) : E

Il existe des petits reins dans les cas A et B, des gros reins dans le cas C, des reins irréguliers dans le cas D.

La calcémie et la phosphorémie sont trouvées normales. Vous concluez à :


A - Une hyperparathyroïdie primaire
B - Une hyperparathyroïdie secondaire
C - Une hyperparathyroïdie tertiaire autonomisée
D - Un traitement à la vitamine D
E - Aucune des hypothèses ci-dessus
Bonne(s) réponse(s) : E

Sans commentaire.

La kaliémie et la kaliurèse sont normales. La natriurèse est basse. La chlorémie et la natrémie sont normales.
Le poids est normal. Vous concluez que :
A - La malade prend des diurétiques, Lasilix® ou furosémide pour garder son poids ou maigrir
B - La malade prend des anti-aldostérones
C - La malade consomme une association de diurétique thiazide et épargneur de potassium (Cyclotériam® ou
Aldactazine®)
D - Il existe une intoxication aux laxatifs
E - La malade suit un régime désodé
Bonne(s) réponse(s) : E

La natriurèse est le reflet des apports sodés.

La pression artérielle basse s'explique par :


A - La prise de neuroleptiques
B - Un diabète sodé par tubulopathie congénitale
C - Un diabète sodé par tubulopathie acquise
D - Une insuffisance surrénalienne
E - Une hypoparathyroïdie
Bonne(s) réponse(s) : A

Les neuroleptiques sont hypotenseurs.

Les parents d'un jeune garçon de 4 ans, constatent une bouffissure du visage avec prise de poids de 2,2 kg en l'espace de 6
jours.
Examen clinique : les oedèmes sont très marqués au visage avec des paupières gonflées aux deux membres inférieurs et il
existe une hydrocèle bilatérale : température : 37°; pression artérielle : 11,5/7 cm Hg.
Examens complémentaires : hémogramme : normal.
Sang : VS 88/132, fibrinogène 6,5 g/l ; urée 5 mmol/l (0,30 g/l), créatinine 48 micromoles/l (4,8 mg/l), protéines totales 45 g/l,
albumine 15 g/l, électrophorèse (albumine 24 %, alpha1 5 %, alpha 2 43, bêta 18 %, gamma 10 %), cholestérol 9,5 mmol/l
(3,7 g/l).
Urines 4 GR/mm3 et 2 GB/mm3, protéinurie 3,8 g/24 heures avec 85 % d'albumine à l'électrophorèse, volume des urines de
24 heures 300 ml. Autres examens : antistreptolysines 50 U/ml, complément total CH 50 normal.

Compte tenu des signes cliniques et biologiques présents chez ce garçon de 4 ans, quel est le diagnostic ?
A - Glomérulonéphrite aiguë
B - Syndrome néphrotique
C - Cardiopathie décompensée
D - OEdème de Quincke
E - Protéinurie tubulaire
Bonne(s) réponse(s) : B

Sans commentaire.

258
Exclusivement sur DOC - DZ : www.doc-dz.com NADJI 85
RESIDANAT EN POCHE TOME II
Cas Clinique en QCM

En faveur d'un bon pronostic vous retenez :


A - Absence d'hématurie
B - Protéinurie sélective
C - Absence d'hypertension artérielle
D - Absence d'insuffisance rénale
E - Taux de complément normal
Bonne(s) réponse(s) : A B C D E

Tous les critères permettent d'évoquer le diagnostic de syndrome néphrotique pur dont le pronostic est bon. Le taux de
complément normal est en particulier un élément important.

Votre prescription thérapeutique va comporter :


A - Aspirine 100 mg/kg/jour
B - Régime pauvre en sodium
C - Prednisone 2 mg/kg/jour
D - Gentamycine 1 mg/kg/jour
E - Indométacine 1 mg/kg/jour
Bonne(s) réponse(s) : C

Pendant 4 à 6 semaines, complété par les mesures adjuvantes habituelles à toute corticothérapie : régime sans sel, diurétique
et apport enrichi en protides.

Pour conduire correctement ce traitement, vous préconisez :


A - Restriction hydrique en fonction de la diurèse
B - Régime pauvre en sodium
C - Suppléments de calcium
D - Limiter les apports de protéines
E - Suppléments de vitamine D
Bonne(s) réponse(s) : A B C E

Sans commentaire.

Vis a vis d'une éventuelle antibiothérapie vous envisagez :


A - Céphalosporine IV
B - Bactrim® per os
C - Chloramphénicol
D - Gentamicine IM
E - Abstention
Bonne(s) réponse(s) : E

Sans commentaire.

Un homme de 38 ans, célibataire, consulte pour une grosse bourse droite ; il rapporte que l'augmentation de volume a été
progressive en deux à trois mois sans douleur et sans fièvre. L'examen clinique attentif permet de bien différencier l'épididyme
et d'affirmer qu'il s'agit d'une tumeur testiculaire déformant le testicule droit qui est deux fois plus gros que le gauche.
L'examen clinique général est normal.

Parmi les éléments suivants quel est celui qui est considéré comme facteur de risque de cancer du testicule ?
A - Cancer testiculaire chez le père
B - Antécédent d'oreillons
C - Antécédent d'irradiations
D - Antécédent de cryptorchidie
E - Antécédent de traumatisme testiculaire
Bonne(s) réponse(s) : D

La fréquence est alors multipliée par un facteur compris entre 10 et 40.

259
Exclusivement sur DOC - DZ : www.doc-dz.com NADJI 85
RESIDANAT EN POCHE TOME II
Cas Clinique en QCM

La fréquence du cancer testiculaire est par rapport à l'ensemble des cancers masculins de :
A - Environ 1 %
B - Environ 5 %
C - Environ 10 %
D - Environ 15 %
E - Environ 20 %
Bonne(s) réponse(s) : A

Il s'agit néanmoins de la plus fréquente des tumeurs solides chez les sujets de 20 à 30 ans (6%).

Quel(s) est(sont) le(s) marqueur(s) biologique(s) le plus souvent rencontré(s) chez un sujet présentant un
cancer du testicule ?
A - Alphafoetoprotéine
B - Thyrocalcitonine
C - Substance ACTH like.
D - Fraction bêta de l'hormone gonadotrophine chorionique
E - Antigène carcino-embryonnaire
Bonne(s) réponse(s) : A D

Sans commentaire.

Le premier relais ganglionnaire du testicule est :


A - Lombo aortique
B - Iliaque interne
C - Iliaque externe
D - Obturateur
E - Inguinal
Bonne(s) réponse(s) : A

Sans commentaire.

Les moyens suivants sont utiles pour la détection des adénopathies métastatiques d'un cancer du testicule sauf
un. Lequel ?
A - L'examen clinique.
B - La tomodensitométrie
C - L'échographie
D - La lymphographie
E - Le lavement baryté
Bonne(s) réponse(s) : E

Sans commentaire.

L'une des formes histologiques de cancer du testicule est plus radiosensible que les autres et régulièrement
radiocurable. Laquelle ?
A - Le choriocarcinome
B - Le séminome pur
C - Le carcinome embryonnaire pur
D - L'association carcinome embryonnaire séminome
E - Le tératome bien différencié
Bonne(s) réponse(s) : B

Sans commentaire.

Les chances de guérison définitive après traitement d'un malade atteint d'un séminome du testicule au stade I
sont de l'ordre de :
A - 20 %
B - 40 %
c - 60 %
D - 80 %
E - 100 %
Bonne(s) réponse(s) : E

En fait 95 %.

260
Exclusivement sur DOC - DZ : www.doc-dz.com NADJI 85
RESIDANAT EN POCHE TOME II
Cas Clinique en QCM
Monsieur Albert P., âgé de 27 ans, vous est envoyé par le médecin du travail (entrerise de fabrication de nougats) car celui-ci
a découvert une hématurie microscopique lors de l'examen systématique des urines au cours de la visite annuelle de
médecine du travail. L'année précedente, les urines étaient normales. La tension artérielle est à 125/75 mmHg, l'examen
clinique est normal. Il n'y a aucun antécédent médical ou chirurgical. La numération des éléments figurés dans les urines
confirme l'hématurie (260 hématies/s). Il n'y à pas d'infection urinaire. La protéinurie est minime (inférieure a 50 mg/24 h).

Quel examen demandez-vous en première intention ?


A - Artériographie rénale
B - Urographie
C - Cystoscopie
D - Etude du sédiment en contraste de phase
E - Tomodensitométrie de l'espace rétropéritonéal
Bonne(s) réponse(s) : D

Permet la recherche de cylindres hématiques traduisant l'origine glomérulaire de l'hématurie.

Quand une étiologie glomérulaire est trouvée à ces hématuries microscopiques isolées. Il s'agit le plus souvent
d'une glomérulopathie. De quel type ?
A - Extramembraneux
B - Membranoprolifératif avec dépôts d'lgA
C - Prolifératif extracapillaire
D - Mésangial à dépôts d'IgA et/ou de C3
E - Changements glomérulaires minimes
Bonne(s) réponse(s) : D

Ou maladie de Berger.

D'une façon générale un mauvais pronostic dans ce type de glomérulopathie est fonction de :
A - Sexe feminin
B - Hypertension artérielle
C - Protéinurie de taux élevé
D - Episodes d'hématurie macroscopique
E - Taux sérique élevé d'immunoglobulines E
Bonne(s) réponse(s) : B C

Sans commentaire.

Une hypertension artérielle, si elle survient dans ce contexte :


A - Survient seulement au stade d'insuffisance rénale avancée
B - Survient seulement en cas de syndrome néphrotique associé
C - Est fréquente à n'importe quel stade de cette néphropathie
D - Traduit une autre étiologie rénale (sténose artère rénale)
E - Ne mérite pas de traitement
Bonne(s) réponse(s) : C

Elle est en effet rare au début de la maladie, et fréquente à la phase terminale.

261
Exclusivement sur DOC - DZ : www.doc-dz.com NADJI 85
RESIDANAT EN POCHE TOME II
Cas Clinique en QCM
Mr Z., 56 ans, agent d'assurance, vient consulter pour des hématuries d'apparition récente. C'est la veille que le malade a
constaté, en allant aux toilettes, que ses urines étaient uniformément rouge foncé, et depuis, toutes ses mictions indolentes et
normalement espacées sont colorées, avec quelques caillots. Mr Z. est très inquiet mais ne décrit aucun autre trouble
fonctionnel. Dans les antécédents vous notez :
- une appendicectomie à 20 ans
- une crise de colique néphrétique droite à 42 ans suivie de l'expulsion d'un petit calcul d'acide urique
- à 52 ans une artérite des membres inférieurs ayant nécessité un pontage ilio-fémoral, avec un résultat fonctionnel excellent
Depuis l'intervention, il prend régulièrement des antivitamines K (Sintrom® 0,5 à 1.5 cp/j)
- il s'agissait d'un grand fumeur qui a définitivement arrêté de fumer après l'intervention.
A l'examen clinique :
- l'état général est floride
- les urines recueillies dans deux verres, en début de consultation, sont uniformément colorées de sang
- l'abdomen est souple : il n'y a pas de masse perceptible abdominale ou lombaire
- les organes génitaux externes sont normaux
- au TR, la prostate est ferme, de taille et de consistance normales
- fréquence cardiaque 70, TA 18/10 cm Hg.
Mr Z. apporte le résultat de ses derniers examens sanguins effectués l'avant veille de l'apparition du saignement : taux de
prothrombine 43 %, urée sanguine 6 mmol/l, créatinine 90 micromol/l, hématies 5 800 000, leucocytes 6 500, hémoglobine 46
g/l, glycémie 5 mmol/l.

Quel élément clinique prouve qu'il s'agit bien d'une hématurie ?


A - Les urines rouges sont émises pendant la miction
B - Les mictions sont indolentes
C - La coloration persiste à ébullition
D - La présence de caillots
E - Les urines sédimentent au repos
Bonne(s) réponse(s) : D

Sans commentaire.

Vous pouvez envisager comme hypothèse(s) diagnostique(s) :


A - Hypocoagulabilité excessive sous anticoagulants
B - Maladie de Berger
C - Tumeur de vessie
D - Calcul urinaire
E - Cancer du rein
Bonne(s) réponse(s) : C D E

L'item A ne doit jamais être retenu comme étiologie isolée d'une hématurie et la présence de caillot permet d'éliminer une
cause "néphrologique".

L'UIV ayant montré une image d'encorbellement du groupe caliciel inférieur et une déformation du pôle inférieur
du rein, vous prescrivez une échographie qui montre une masse vide d'écho, à contours réguliers et à
renforcement postérieur. Quel examen demandez-vous ?
A - Une angiographie numérisée
B - Une cystoscopie
C - Une biopsie percutanée de la masse rénale
D - Une urétéropyélographie rétrograde
E - Une cavographie
Bonne(s) réponse(s) : A

Sans commentaire.

Ces examens ne retrouvant rien de plus que l'échographie, il est alors logique de :
A - Faire une biopsie prostatique
B - Rechercher régulièrement une hématurie microscopique
C - Rassurer le malade et ne pas pousser plus loin les investigations dans l'immédiat
D - Refaire une nouvelle urographie au bout de 3 mois
E - Arrêter le traitement anticoagulant
Bonne(s) réponse(s) : B C D

Sans commentaire.

262
Exclusivement sur DOC - DZ : www.doc-dz.com NADJI 85
RESIDANAT EN POCHE TOME II
Cas Clinique en QCM
Une femme de 28 ans, sous oestroprogestatifs, présente depuis 8 mois une altération progressive et modérée de l'état
général, avec fébricule persistante à 37°9. La survenue brutale d'un oedème des membres inférieurs avec oligurie, sans
hypertension artérielle ni signes cliniques d'insuffisance rénale, fait pratiquer des examens qui révèlent :
- une protéinurie à 5 g/24 h
- 15 000 hématies/mn au compte d'Addis, avec cylindrurie
- une hypoprotidémie
- 2400 leucocytes/mm3 sans anémie
- VS à 30 mm à la première heure
- Antistreptolysines O dans les limites de la normale.

Parmi les examens suivants, quel est le plus utile pour le diagnostic étiologique ?
A - Test de Coombs
B - Myélogramme
C - Immunoélectrophorèse du sérum
D - Recherche d'anticorps anti-ADN natif
E - Test de Waaler Rose
Bonne(s) réponse(s) : D

La première cause à évoquer devant la survenue d'un syndrome néphrotique impur chez une jeune femme est un lupus
érythémateux aigu disséminé. L'histologie la plus probable étant une glomérulite extramembraneuse ou une hyalinose
segmentaire et focale.

Une ponction biopsie rénale est réalisée, révélant un épaississement diffus et régulier des parois des capillaires
glomérulaires, sans prolifération cellulaire, avec dépôts granuleux régulièrement répartis d'lgG et de C3 à
l'immunofluorescence. Ces aspects indiquent l'existence :
A - D'une glomérulonéphrite segmentaire et focale
B - D'une glomérulonéphrite extramembraneuse
C - D'une glomérulonéphrite mésangiale
D - D'une glomérulonéphrite à lésions glomérulaires minimes
E - D'une tubulopathie interstitielle
Bonne(s) réponse(s) : B

Sans commentaire.

Le mécanisme de l'atteinte rénale est assimilable à :


A - Une réaction d'hypersensibilité immédiate
B - Une réaction d'hypersensibilité à médiation cellulaire
C - Un phénomène d'Arthus local
D - La glomérulonéphrite de la maladie sérique chronique
E - Une réaction de cytotoxicité de type cellulaire dépendant d'anticorps
Bonne(s) réponse(s) : D

Sans commentaire.

Quel que soit le traitement entrepris, la surveillance que vous adoptez comporte :
A - Protéinurie
B - Auto-anticorps anti ADN natif
C - Taux de complément sérique
D - Auto-anticorps anti-Sm
E - Albuminurie
Bonne(s) réponse(s) : A B C E

Sans commentaire.

263
Exclusivement sur DOC - DZ : www.doc-dz.com NADJI 85
RESIDANAT EN POCHE TOME II
Cas Clinique en QCM
Un homme de 23 ans est hospitalisé pour oedèmes apparus 3 jours plus tôt. Son médecin traitant a fait effectuer quelques
examens biologiques : créatininémie 141 mmol/l (16 mg/l), protéinurie 1,60 g/24 heures.
A l'examen clinique on note la présence d'oedèmes des paupières et des membres inférieurs. La diurèse est le premier jour
de 500 ml/24 h, le poids est de 60,500 kg. La pression artérielle est à 16/10 cmHg. Les examens biologiques montrent :
- dans les urines : protéinurie des 24 heures 1,60 grammes, hématies 125000/mn, leucocytes 3 500/mn, natriurèse 51
mmol/24 heures
- dans le sang : creatininémie 123 umol/l (14 mg/l) azotémie 13.2 mmol/l (0.80 g/l), sodium 130 mmol/l, kaliémie 4 mmol/l,
protidémie 60 g/l hématocrite 37 %.
Lors de sa sortie, après 15 jours d'hospitalisation le poids est de 53,500 kg, la pression artérielle de 15/9, la diurèse de 1 500
ml/24 heures, la protéinurie est à 0,5 g/24 heures. Il persiste une hématurie microscopique (35 000/mn). La créatininémie est
à 106 micromol/l (12 mg/l).
Six mois plus tard, la protéinurie est nulle, I'hématurie à 450/mn, la T.A. à 13/8.

Quel hypothèse diagnostique formulez-vous à partir de l'ensemble des données ?


A - Syndrome néphrotique impur
B - Gloménulonéphrite aiguë
C - Maladie de Berger
D - Néphropathie tubulo-interstitielle aiguë
E - Poussée aiguë sur glomérulonéphrite chronique
Bonne(s) réponse(s) : B

On ne peut pas parler de syndrome néphrotique puisque la protéinurie est < à 3 g/j et que la protidémie est égale à 60 g/l.

Citez l'examen complémentaire qui est le plus utile pour le diagnostic :


A - Urographie intraveineuse
B - Dosage du complément (total et C3)
C - Recherche de complexes immuns circulants
D - Dosage pondéral des immunoglobulines plasmatiques
E - Examen cytobactériologique des urines
Bonne(s) réponse(s) : B

L'abaissement du complément est un des éléments les plus importants du syndrome néphritique, puisqu'il traduit la
physiopathologie de la maladie et permet d'en suivre l'évolutivité.

Quel(s) est(sont) le(s) type(s) de néphropathie qui représente(nt) une indication indiscutable de corticothérapie ?
A - La glomérulonéphrite aiguë
B - La maladie de Berger
C - Le syndrome néphrotique pur à lésions glomérulaires minimes
D - L'amylose rénale
E - La néphropathie diabétique
Bonne(s) réponse(s) : C

Sans commentaire.

Parmi les risques lointains de cette atteinte on reconnaît habituellement :


A - Transmission héréditaire de la maladie
B - Développement d'une tumeur du rein
C - Répétition d'infections urinaires
D - Risque d'HTA
E - Risque de lithiase rénale
Bonne(s) réponse(s) :

Aucune réponse.

264
Exclusivement sur DOC - DZ : www.doc-dz.com NADJI 85
RESIDANAT EN POCHE TOME II
Cas Clinique en QCM
Monsieur Georges V., 34 ans. est hospitalisé pour une hypokaliémie inférieure à 3 mmol/l, persistant depuis près de trois mois
malgré un apport régulier de potassium par la bouche.
Depuis 10 ans, ce sujet de petite taille présente une fatigabilité musculaire importante, des paresthésies des extrêmités, des
céphalées, une constipation, une polyuro-polydipsie et un état nettement dépressif.
Il n'y a pas d'antécédents uro-néphrologique, cardiaque ou hépatique. On ne note pas de prise de médicament
hypokaliémiant. L'examen montre un signe de Chvostek. Il n'y a pas d'insuffisance cardiaque. La pression artérielle est de
120/80 mmHg.
A l'E.C.G : signes manifestes d'hypokalémie. Biologiquement on note une alcalose hypokalémique avec perte urinaire de K et
de Cl.

Parmi les anomalies électrocardiographiques suivantes, donnez celle(s) qui peu(ven)t être liée(s) à une
hypokaliémie ?
A - Allongement de PR
B - Elargissement de QRS
C - Allongement de QT
D - Extrasystoles ventriculaires
E - Torsades de pointe
Bonne(s) réponse(s) : D E

L'allongement de QT qui est souvent rapporté n'est pas un allongement réel mais un pseudo allongement lié à la diminution
de l'onde T voire même à sa disparition avec l'apparition d'une onde U avec intervalle QU.

Parmi les situations suivantes, donnez celle(s) capable(s) d'entraîner par elle(s)-même une hypokaliémie ?
A - Acidose métabolique
B - Diarrhée
C - Traitement les spironolactones
D - Syndrome de Bartter
E - Alcalose métabolique
Bonne(s) réponse(s) : B D E

Le syndrome de Bartter est une hyperplasie de l'appareil juxtaglomérulaire avec un tableau d'hyperaldostéronisme et
hyperréninémie.
Cette affection se manifeste dans l'adolescence et se caractérise par une alcalose avec hypokaliémie. Sa transmission est
autosomique récessive.
Son traitement comporte actuellement les inhibiteurs de la synthèse des prostaglandines.

Devant le tableau de ce patient, quel est l'étiologie la plus probable ?


A - Un syndrome de Conn (hyperaldostéronisme primaire)
B - Un hyperaldostéronisme secondaire
C - Une néphropathie tubulo-interstitielle
D - La prise de laxatifs
E - un syndrome de Bartter
Bonne(s) réponse(s) : E

Sans commentaire.

Vous pouvez envisager de traiter ce patient par :


A - Lasilix®
B - Tériam®
C - Modamide®
D - Corticoïdes per os
E - Indocid®
Bonne(s) réponse(s) : B C E

Sans commentaire.

265
Exclusivement sur DOC - DZ : www.doc-dz.com NADJI 85
RESIDANAT EN POCHE TOME II
Cas Clinique en QCM
Une femme de 58 ans se présente à la consultation. Elle ne connait aucun de ses parents et n'a pas, quant à elle, de passé
pathologique connu. Elle n'a pas, depuis sa seule et unique grossesse marquée par une protéinurie, été examinée par un
médecin. Elle a tenu pendant 20 ans un petit commerce et a pris régulièrement pendant cette période un à plusieurs
comprimés par jour d'une préparation antalgique et calmante. Durant cette période elle a eu à trois reprises un épisode de
douleurs lombaires droites avec fièvre et infection urinaire. Convoquée à un examen par un centre de Sécurité Sociale, les
découvertes ont été les suivantes : la pression artérielle est à 175/115 mm de mercure, la créatinine plasmatique est de 270
micromoles pour mille. L'examen des urines (à bandelette réactive) ne montre ni protéinurie, ni hématurie. Le fond d'oeil est
de stade II, il existe une hypertrophie ventriculaire gauche modérée. Il y a un an. une crise douloureuse du gros orteil a été
interprêtée comme crise de goutte.

Parmi les diagnostics suivants, le(s)quel(s) est (sont) compatible(s) avec le tableau clinique ci-dessus avant tout
examen ?
A - Pyélonéphrite chronique
B - Néphropathie interstitielle par prise abusive d'analgésiques
C - Glomérulonéphrite chronique
D - Hypertension artérielle maligne
E - Néphropathie du lupus érythémateux disséminé
Bonne(s) réponse(s) : A B

Sans commentaire.

L'examen clinique confirme les chiffres tensionnels et trouve deux gros reins non douloureux à la palpation.
Dans ce contexte clinique le(s) diagnostic(s) peut ou peu(ven)t être envisagé(s) comme probable(s) ou
possible(s) :
A - Pyélonéphrite chronique par reflux vésico-urétéral
B - Hydronéphrose bilatérale par anomalie de la jonction pyélo-urétérale
C - Thrombose cave et des veines rénales
D - Sténose ou thrombose des artères rénales
E - Polykystose rénale
Bonne(s) réponse(s) : B E

Sans commentaire.

Il vous paraît justifié de compléter le bilan par l'investigation suivante :


A - Artériographie
B - Scanner cranien
C - Tomographie et scintigraphie pulmonaire
D - Echographie rénale et hépatique
E - Ponction biopsie du parenchyme rénal
Bonne(s) réponse(s) : D

L'examen clef de l'exploration de toute insuffisance rénale est l'échographie rénale.


L'exploration couplée hépatique et rénale est intéressante dans la polykystose qui se caractérise par une atteinte hépatique et
rénale.

Parmi les traitements suivants de l'hyperuricémie, lequel vous paraît le plus adéquat ?
A - Colchicine 3 mg/j pendant six mois
B - Uricozyme® (uricolytique) 1 injection intramusculaire par mois pendant 6 mois
C - Amplivix® (benzodiarone) ou Désuric® (benzobromarone) 300 mg par jour pendant six mois
D - Lasilix® (furosémide) 200 mg par jour pendant six mois
E - Zyloric® (allopurinol) 200 mg par jour pendant six mois
Bonne(s) réponse(s) : E

Le traitement devra s'accompagner pendant les trois premières semaines d'une couverture par colchicine. Les uricosuriques
sont contre-indiqués en cas d'insuffisance rénale.

Dans le contexte décrit, il est indispensable pour surveiller et traiter correctement l'hypertension d'obtenir le
résultat de :
A - Dosages étagés de l'activité rénine au niveau de la veine cave et des veines rénales et hépatiques
B - Dosages étagés des catécholamines plasmatiques
C - Fond d'oeil
D - Electrocardiogramme
E - Mesure du débit cardiaque et des résistances périphériques
Bonne(s) réponse(s) : C D

Sans commentaire.

266
Exclusivement sur DOC - DZ : www.doc-dz.com NADJI 85
RESIDANAT EN POCHE TOME II
Cas Clinique en QCM

Si d'autres examens sont faits. vous vous attendez à trouver :


A - Hypocalcémie
B - Hyperphosphorémie
C - Hypocalciurie
D - Baisse de l'urée plasmatique
E - Elévation de la clairance de l'inuline
Bonne(s) réponse(s) : A B

Sans commentaire.

Chez un homme de 55 ans apparaissent des oedèmes des membres inférieurs sans aucune anomalie clinique
antérieurement remarquée par le malade. La pression artérielle est à 180/120 mmHg alors qu'elle était encore normale trois
mois plus tôt et il n'y avait pas de protéinurie. Il n'y a pas de fièvre. L'examen clinique trouve des râles fins des bases
pulmonaires et une tachycardie à 100 par minute avec galop gauche. Le reste de l'examen clinique (fosses lombaires,
abdomen, toucher rectal, revêtement cutané) est normal. Les examens biologiques montrent les résultats suivants :
Sang : urée 15 mmol/l, créatinine 160 micromol/l, ionogramme (mmol/l) : Na 140, K 4,5 ; Cl 98, bicarbonates 22, protides
totaux 66 g/l (albumine 36. globulines 30), hématocrite 37 %, calcémie 2,3 mmol/l, phosphorémie 1,2 mmol/l. Urines des 24
heures (600 ml) : protéines 6 g, Na 20 mmol, K 30 mmol, urée 150 mmol, compte d'Addis (/mn) : hématies 300.000,
leucocytes 10.000.

L'association des anomalies observées est compatible avec :


A - Un syndrome néphrotique
B - Un syndrome néphritique aigu
C - Un syndrome de néphropathie glomérulaire chronique
D - Un syndrome de néphropathie interstitielle aiguë
E - Un syndrome d'obstacle sur les voies excrétrices
Bonne(s) réponse(s) : B

Il existe tous les éléments du syndrome néphritique : hématurie, insuffisance rénale, hypertension artérielle, protéinurie.
A noter qu'en dépit de l'importance de la protéinurie il n'y a pas ou du moins pas encore d'hypoprotidémie, donc pas de
syndrome néphrotique. Les oedèmes des membres inférieurs sont plutôt à rapporter à une défaillance cardiaque.

Parmi les affections susceptibles de donner le tableau clinique et biologique observé chez ce patient, vous
pouvez retenir :
A - Une fibrose rétropéritonéale
B - Une septicémie à staphylocoque doré
C - Une amylose généralisée
D - Un diabète
E - Une infection à streptocoque
Bonne(s) réponse(s) : B E

Les septicémies à staphylocoque doré peuvent également être responsables de la survenue d'un syndrome néphritique. Il
s'agit cependant d'une éventualité rare.

L'examen complémentaire le plus utile pour orienter le diagnostic, est :


A - L'urographie intraveineuse
B - Le dosage du complément sérique
C - L'urétéropyélographie rétrograde
D - La biopsie rectale
E - L'artériographie rénale
Bonne(s) réponse(s) : B

La baisse du complément sérique est un bon marqueur des syndromes néphritiques, le taux complément et de ses
composants est un bon marqueur de l'évolutivité de la maladie.

Parmi les gestes thérapeutiques à envisager pour les premières 24 heures suivant l'hospitalisation de ce
malade, vous envisagez :
A - La prescription de furosémide
B - Un traitement antibiotique
C - La prescription de Kayexalate®
D - Un traitement corticoïde
E - Un traitement anticoagulant
Bonne(s) réponse(s) : A

Les autres thérapeutiques (corticothérapie, plasmaphérèse, immunosuppresseurs) ne peuvent se discuter qu'en fonction de
l'histologie. L'antibiothérapie n'est indiquée que si l'on trouve un foyer infectieux.

267
Exclusivement sur DOC - DZ : www.doc-dz.com NADJI 85
RESIDANAT EN POCHE TOME II
Cas Clinique en QCM
Madame B. Pauline, 48 ans, consulte pour une hématurie. Elle aurait présenté il y a cinq ans une symptomatologie du même
type : hématurie macroscopique indolore, totale, se poursuivant pendant plusieurs jours. Pendant les intervalles, les d'urines
claires, la bandelette Labstix® met en évidence sang ++. A la suite de plusieurs épisodes de ce type, la malade a subi une
néphrectomie gauche il y a cinq ans. Le dossier a été entièrement perdu dans un incendie des archives hospitalières. La
patiente ne se plaint de rien, elle est restée sans surveillance pendant quatre ans. Elle est inquiète, à la reprise des
hématuries, depuis trois mois. Dans le choix des investigations, la localisation de l'origine des hématuries est importante.

A la cytologie urinaire, vous retenez en faveur d'une origine vésicale :


A - L'absence de cylindres granuleux et hématiques
B - Le caractère indemne des hématies
C - La présence d'une protéinurie à plus de 1 g/24h
D - La lymphocyturie importante
E - L'absence de cellules de desquamation
Bonne(s) réponse(s) : A B

Les hématies d'origine glomérulaire sont souvent déformées.

La cytologie vous oriente vers une origine vésicale. Quel est alors le moyen d'investigation décisif de cette
origine ?
A - Artériographie pelvienne
B - Scanner
C - Lymphographie
D - Echographie
E - Cystoscopie
Bonne(s) réponse(s) : E

Sans commentaire.

Un des examens ci-dessus vous apporte la preuve d'une lésion vésicale étendue de type papillomateux. Il est
probable que la cause de la néphrectomie initiale ait été :
A - Une lithiase rénale
B - Un angiome rénal
C - Une tuberculose rénale
D - Une tumeur du bassinet
E - Un angiomyolipome
Bonne(s) réponse(s) : D

L'évolution des tumeurs des voies excrétrices est marquée par la survenue de métastases en aval sur les voies excrétrices.

Vous apprenez que la néphrectomie pratiquée a comporté une urétérectomie. Vous estimez que, dans cette
observation, cette intervention a été pratiquée :
A - Pour éviter un reflux dans le moignon urétéral restant
B - Pour éviter la greffe de cellules malignes
C - Pour améliorer la contractibilité de l'uretère restant
D - Pour prévenir la survenue de douleurs pelviennes
E - Pour éviter une hypertension secondaire
Bonne(s) réponse(s) : B

Sans commentaire.

On peut estimer qu'il y a eu dans ce cas un défaut de surveillance post-opératoire. Il aurait du être pratiqué, six
mois à un an après, l'examen suivant :
A - Phlébographie rénale droite
B - Scintigraphie pulmonaire
C - Echographie péricardique
D - Biopsie de la plèvre diaphragmatique gauche
E - Cystoscopie
Bonne(s) réponse(s) : E

Sans commentaire.

268
Exclusivement sur DOC - DZ : www.doc-dz.com NADJI 85
RESIDANAT EN POCHE TOME II
Cas Clinique en QCM
Monsieur G. 58 ans a été néphrectomisé à droite pour lithiase rénale compliquée douze ans auparavant. La dernière
urographie intraveineuse faite à titre de surveillance date de trois ans et peut être jugée normale en dehors d'une surélévation
du plancher vésical sur les clichés de réplétion. La créatinine plasmatique est à 100 mg pour mille. Le sujet ne présente
qu'une symptomatologie réduite depuis ces trois ans et seulement une pollakiurie à deux mictions par nuit. En dehors de cet
accident rénal, aucun antécédent médical ou chirurgical. Le sujet à fait 48 heures auparavant une crise douloureuse violente
qui a duré 8 heures malgré des antalgiques. Depuis persistent des douleurs plus vagues de même topographie et une anurie
totale. L'examen clinique retient une vessie vide, un adénome prostatique d'importance moyenne, un rein gauche douloureux,
l'absence de fièvre. L'échographie rénale montre des cavités dilatées sans cône d'ombre au niveau du rein.

Le type d'anurie le plus probable est dans ce cas :


A - Anurie prérénale par déshydratation
B - Anurie rénale par rein de choc
C - Anurie post-rénale de nature excrétoire
D - Rétention aiguë d'urine par adénome prostatique
E - Occlusion artérielle rénale aiguë
Bonne(s) réponse(s) : C

Sans commentaire.

S'il s'agit d'une anurie excrétoire, citez la(es) cause(s) théoriquement possible(s) en faisant abstraction des
données fournies :
A - Lithiase urinaire migrant dans la voie excrétrice
B - Compression néoplasique
C - Sclérose tuberculeuse de l'uretère
D - Hématome de la paroi urétérale
E - Choc infectieux par infection à Gram négatif
Bonne(s) réponse(s) : A B C D

Il s'agit de cause d'anurie par obstacle.

Citez l'examen que vous choisissez pour localiser un arrêt excrétoire éventuel :
A - Scanographie
B - Phlébographie iliaque gauche avec cavographie
C - Lymphographie
D - Angiographie digitalisée
E - Ponction du bassinet et pyélographie descendante
Bonne(s) réponse(s) : E

La ponction du bassinet et la mise en place d'une sonde constituent également le traitement immédiat de cette anurie.

Les clichés sans préparation d'excellente qualité n'ont pas montré d'image opaque urétérale. S'il s'agit d'une
récidive lithiasique :
A - Il s'agit d'oxalate de calcium
B - Il s'agit de phosphate de calcium
C - Il s'agit de précipitation médicamenteuse d'antalgique
D - Il s'agit de carbonate de calcium
E - Il s'agit d'acide urique
Bonne(s) réponse(s) : E

On rappelle que les lithiases radiotransparentes sont les lithiases uriques et xanthiques, les lithiases radio-opaques sont
oxalocalciques, phosphatocalciques et cystiniques.

Une irrigation permanente de soluté alcalin permet la dissolution de :


A - Oxalate de calcium
B - Phosphate de calcium
C - Calcul d'antalgique
D - De calcul de carbonate de calcium
E - De calcul d'acide urique
Bonne(s) réponse(s) : E

Sans commentaire.

269
Exclusivement sur DOC - DZ : www.doc-dz.com NADJI 85
RESIDANAT EN POCHE TOME II
Cas Clinique en QCM

Une pyélotomie de décharge a permis d'obtenir une polyurie immédiate importante, six litres dans les douze
premières heures. Compte tenu des données fournies :
A - Une hémodialyse doit être faite immédiatement
B - Une hémodialyse sera indispensable car la diurèse sera inefficace
C - Furosémide 500 mg pour éviter une insuffisance rénale fonctionnelle secondaire
D - Pas de perfusion, pas d'apport liquidien pour réduire la polyurie
E - Dans les 1ères 24h : apport liquidien massif, 8 l minimum avec électrolytes
Bonne(s) réponse(s) : E

Il s'agit d'un syndrome de levée d'obstacle, dont le principal danger résulte de sa sous-estimation, avec risque de collapsus
par déshydratation. Les autres items constituent des erreurs thérapeutiques graves mettant en jeu la vie du patient.

Parmi les mesures préventives d'une récidive du même type, vous retenez :
A - Etablir un traitement avec un ou deux grammes de phosphore par jour
B - Suivre un régime appauvri en calcium
C - S'astreindre à une restriction de boisson
D - En cas de résidu vésical programmer une résection prostatique
E - Etablir un traitement avec allopurinol (Zyloric®) 100 à 300 mg/jour
Bonne(s) réponse(s) : D E

Il est également indispensable d'y associer un traitement par diurèse alcaline. Toutefois l'utilisation d'allopurinol ne peut se
justifier que s'il existe une hyperuricémie importante.

Madame C. Hortense, cultivatrice n'a pas de passé pathologique. Elle a été convoquée il y a six mois pour un bilan de santé
qui s'est avéré satisfaisant, en particulier pas d'anomalie urinaire. A 62 ans, elle continue à élever des volailles de Bresse et
quelques vaches. Un matin, elle fait une chute verticale de 3 mètres en passant par la trappe laissée ouverte de son grenier à
foin. Elle ressent à l'instant une violente douleur lombaire. Un bilan radiologique osseux est négatif. Une semaine après,
existent de très volumineux oedèmes symétriques remontant jusqu'aux cuisses, indolores, et gardant le godet. Il persiste un
endolorissement lombaire. Dans les urines : protéinurie ++++, sang : traces à la bandelette Labstix®. Créatinine plasmatique
8 mg pour mille (normale), pression artérielle normale.

A ce stade, choisissez le diagnostic le plus compatible avec les faits énoncés :


A - Rupture rénale incomplète
B - Rupture de l'intima de l'artère rénale
C - Blessure urétérale
D - Syndrome néphrotique aigu
E - Thrombose des veines sus-hépatiques
Bonne(s) réponse(s) : D

Sans commentaire.

Dans la clinique où elle a été hospitalisée, les examens radiologiques montrent deux gros reins fonctionnels
avec quelques images d'empreintes polycycliques sur la voie excrétrice. Parmi les examens suivants, lequel
vous paraît le plus adéquat à ce stade pour faire progresser le diagnostic ?
A - Immunoélectrophorèse des urines
B - Dosage du complément sérique et de la fraction C3
C - Recherche et dosage d'immuns complexes circulants
D - Angiographie digitalisée
E - Phlébographie cavorénale
Bonne(s) réponse(s) : E

A la recherche d'une thrombose des veines caves et rénales, qui pourrait être l'origine ou la conséquence du syndrome
néphrotique. Il faudra toutefois pratiquer au préalable une electrophorèse des protides urinaires afin de s'assurer qu'il ne s'agit
pas d'une protéinurie monoclonale. Il faudra également s'assurer d'une hydratation correcte de la patiente ( deux examens
utilisant des produits de contraste iodés rapprochés).

Avec les constatations des examens choisis, plusieurs propositions thérapeutiques sont avancées. Vous
choisissez :
A - Traitement fibrinolytique (Urokinase®) ou anticoagulant
B - Embolisation de l'artère rénale
C - Mise en place d'un drain de néphrostomie par ponction du bassinet sous échographie
D - Corticothérapie en flash (1 g de prednisone chaque jour pendant 15 jours)
E - Traitement immunosuppresseur, Caryolysine® ou ciclosporine
Bonne(s) réponse(s) : A

Il faut toutefois souligner les problèmes que peuvent souligner la mise en route d'un traitetement par fibrinolytiques à proximité
d'un épisode traumatique.

270
Exclusivement sur DOC - DZ : www.doc-dz.com NADJI 85
RESIDANAT EN POCHE TOME II
Cas Clinique en QCM

Après trois mois d'évolution, le tableau clinique et biologique est le suivant : oedèmes importants remontant
jusqu'aux lombes, protéinurie 24 gr/24 h, hématurie microscopique discrète, albuminémie 18 gr pour mille, pas
d'insuffisance rénale.

Une biopsie rénale étant pratiquée, quel type de lésion sera trouvé ?
A - Glomérulonéphrite proliférative endocapillaire
B - Glomérulonéphrite intercapillaire à dépôts d'lgA
C - Artériolite avec lésions nécrosantes
D - Glomérulonéphrite extramembraneuse
E - Rein optiquement normal
Bonne(s) réponse(s) : D

Par argument de fréquence, c'est parmi les items proposés celui qui est le plus souvent responsable de syndrome
néphrotique, souvent impur chez l'adulte. A noter que l'histologie la plus fréquemment retrouvée au cours des syndromes
néphrotiques de l'adulte est la hyalinose segmentaire et focale.

Devant une protéinurie massive de ce type avec oedème, un certain nombre de complications sont
envisageables. Laquelle ou lesquelles dans ce contexte ?
A - Embolie pulmonaire
B - Insuffisance cardiaque
C - Choc hypovolémique
D - Ascite et épanchement pleural
E - Hypolipémie importante
Bonne(s) réponse(s) : A C D

Sans commentaire.

La patiente a reçu une corticothérapie à dose importante qui est restée inefficace. La patiente étant hospitalisée
demande à être soulagée de ses oedèmes. Quelle solution thérapeutique vous paraît à la fois sans danger et
efficace ?
A - Dialyse péritonéale avec liquide hypo-osmotique
B - Séance de rein artificiel courte avec ultrafiltration massive
C - Administration de digitalique et d'anti-inflammatoires non stéroïdiens
D - Perfusion d'albumine et de mannitol
E - Cure déclive avec antiagrégant plaquettaire
Bonne(s) réponse(s) : D

Sans commentaire.

La patiente a été traitée. Au bout de dix ans, la créatinine est à 125 micromoles pour mille, la protéinurie est à
3,5 g/24 heures, les oedèmes sont discrets sans traitement. Devant cette notion évolutive, quel est votre choix
diagnostic (il peut être différent ou identique du diagnostic initial) ?
A - Rupture rénale incomplète
B - Rupture intima des artères rénales
C - Blessure, puis sténose urétérale
D - Syndrome néphrotique
E - Thrombose des veine sus-hépatiques
Bonne(s) réponse(s) : D

Sans commentaire.

271
Exclusivement sur DOC - DZ : www.doc-dz.com NADJI 85
RESIDANAT EN POCHE TOME II
Cas Clinique en QCM
Une femme de 38 ans est hospitalisée en 1985 pour bilan et traitement d'une hypertension artérielle, un mois après
thrombose de l'artère centrale de la rétine gauche. L'hypertension artérielle est connue depuis 1971. Les chiffres initiaux
étaient de I'ordre de 220/100 mmHg. Au cours de ces années, à plusieurs reprises la malade se plaint de paresthésies des
extrémités, de crises tétaniformes, d'une constipation rebelle et d'un syndrome polyuro-polydysique marqué. Différents
traitements anti-hypertenseurs ont été utilisés : bêta-bloqueurs, diurétiques (association de thiazidiques et antialdostérone),
anti-hypertenseurs centraux. La pression artérielle n'est jamais descendue au dessous de 170/100 mmHg. A l'entrée dans le
service, un mois après arrêt de tout traitement, en régime normosodé, la pression artérielle est de 200/100 mmHg aux deux
bras. L'examen clinique est normal par ailleurs. Il n'y a pas de protéinurie. Les examens biologiques montrent une kaliémie à
2,4 mEq, une natrémie aà 146 mEq, une kaliurèse à 160 mEq/jour, une natriurèse à 57 mEq/jour. L'ECG montre une
repolarisation anormale avec une onde U caractéristique. L'aldostéronémie est à 4000 pmol/l (normale 10 à 530). L'activité
rénine plasmatique est à 0,3 ng/ml (normales 0,1 à 2).

Le tableau clinique et biologique de cette HTA évoque :


A - Une HTA par néphropathie glomérulaire
B - Une HTA par sténose de l'artère rénale
C - Une HTA d'origine endocrinienne
D - Une HTA par néphropathie interstitielle
E - Une HTA par coarctation de l'aorte
Bonne(s) réponse(s) : C

A - L'absence de protéinurie est un argument important contre l'existence d'une néphropathie glomérulaire qui d'ailleurs
n'expliquerait pas les anomalies hyoroélectrolytiques de cette patiente.
B - L'absence de souffle, de facteurs de risque d'athéromatose plaident en défaveur de cette étiologie qui n'explique pas les
troubles métaboliques.
C - L'association d'une hypertension artérielle et d'une hypokaliémie doit faire évoquer de principe une HTA d'origine
endocrinienne et en particulier un hyperaldostéronisme primaire, après avoir toutefois éliminé la prise de dérivés de la réglisse.
D - L'HTA est rare et tardive dans les néphropathies interstitielles que rien par ailleurs dans l'histoire de cette patiente ne
permet d'évoquer.
E - L'HTA par coarctation de l'aorte se révèle le plus souvent chez le sujet jeune, associant souffle, asymétrie des pouls. La
coarctation de l'aorte n'entraîne pas de telles perturbations hydroélectrolytiques.

Parmi les examens ci-dessous, avant mise en route d'une éventuelle thérapeutique, le(s)quel(s) vous
parai(ssen)t essentiel(s) devant toute hypertension artérielle ?
A - L'urographie intraveineuse
B - Le dosage de l'acide vanyl-mandélique
C - Le dosage de l'aldostérone urinaire
D - Le dosage de la créatinine sanguine
E - Le dosage du potassium plasmatique
Bonne(s) réponse(s) : D E

Il faut comprendre la question dans le sens du bilan minimum devant toute hypertension artérielle sans signe d'orientation, ce
qui n'est pas le cas du ... cas. Le bilan alors proposé ne peut pas comporter, pour des raisons économiques et pour des
raisons de risque de iatrogénie, la pratique systématique d'une IUV, de dosage du VMA ou de l'aldostérone.

Tois heures après la prise de 50 mg de Captopril®, la pression artérielle systolique a baissé de 20 mm Hg,
l'aldostéronémie est à 2700 pmol/l, l'activité rénine plasmatique est à 0,086 ng/ml. Ceci évoque :
A - Une hyperaldostéronisme primaire
B - Une sténose de l'artère rénale
C - Une intoxication à l'acide glycerrhizinique
D - Un phéochromocytome
E - Un hyperaldostéronisme secondaire
Bonne(s) réponse(s) : A

La prise d'un inhibiteur de l'enzyme de conversion a permis une diminution modérée de l'HTA, un effondrement de l'activité
rénine plasmatique et n'a que très peu affecté la sécrétion d'aldostérone. Tout cela milite en faveur d'un hyperaldostéronisme
primaire (A) où la sécrétion d'aldostérone est indépendante ou quasi indépendante du système rénine angiotensine. La faible
diminution de la pression artérielle est à mettre sur le compte de l'activité hypertensive intrinsèque de l'angiotensine II et de
l'activité hypertensive intrinsèque des IEC et des bradykinines. La chute de tension eut été beaucoup plus brutale dans le cas
d'une HTA par sténose de l'artère rénale ou l'HTA dépend de l'hyperactivité du système rénine angiotensine. Dans une
intoxication à l'acide glycérrhizique, l'aldostérone n'est pas élevée, ainsi que dans le phéochromocytome. Enfin, dans un
hyperaldostéronisme secondaire, la sécrétion d'aldostérone est dépendante du système rénine angiotensine et aurait donc été
très diminuée par la prise d'un IEC.

272
Exclusivement sur DOC - DZ : www.doc-dz.com NADJI 85
RESIDANAT EN POCHE TOME II
Cas Clinique en QCM

Dans cette affection, parmi les examens complémentaires ci-dessous, quel(s) est(sont) celui(ceux) que vous
demandez dans un but diagnostique ?
A - L'artériographie sélective des artères rénales
B - Les prélèvements étagés pour dosage de l'activité rénine
C - Une échographie des loges rénales et surrénales
D - Un examen par scanner des loges rénales et surrénales
E - Une biopsie rénale avec examen en immunofluorescence
Bonne(s) réponse(s) : C D

Dans la recherche de la localisation de la source de l'hyperaldostéronisme, la pratique d'une biopsie rénale est bien sûr inutile
(E). La pratique d'une artériographie rénale (A) ou d'un rétropneumopéritoine sont des méthodes agressives, peu rentables
quant à la visualisation d'un adénome et qui ne doivent pas être proposées d'emblée. La méthode du dosage veineux étagé
au cours de cathétérisme (B) est une bonne méthode de localisation de l'adénome et permet de différencier adénome et
hyperplasie. Mais il faut doser l'adostérone et non la rénine (lire un item jusqu'au bout !). L'échographie et le scanner sont
parfois utiles mais leur apport est souvent limité par la taille réduite de l'adénome. Toutefois ils sont peu agressifs (C et D). En
fait les deux examens les plus performants ne sont pas cités ici. Il s'agit des dosages étagés d'aldostérone par cathétérisme
veineux et de la scintigraphie au cholestérol marqué.

Quel traitement médical doit être mis en route dans l'attente d'un éventuel traitement chirurgical ?
A - Spironolactone (Aldactone®)
B - Bêtabloqueur cardiosélectif
C - Inhibiteur de l'enzyme de conversion
D - Vasodilatateur artériel
E - Alphabloqueur
Bonne(s) réponse(s) : A

Le traitement des HTA par hyperminéralocorticisme primaire repose sur l'Aldactone® à la dose de 3 à 5 mg/kg/jour, soit des
doses supérieures aux doses usuelles, ce qui explique l'échec de la première tentative.

Un sujet de 58 ans présente depuis 5 ans, régulièrement, des crises de goutte du gros orteil qu'il s'est contenté de traiter par
le repos et des anti-inflammatoires non stéroïdiens (AINS) pris 2 à 3 jours à chaque crise. Le sujet a présenté en moyenne 4 à
5 crises par an. Il n'a pas de manifestation évoquant une pathologie rénale, sauf une hématurie signalée 3 ans auparavant et
non retrouvée ensuite. La pression artérielle est à 155/105 mm de mercure en moyenne. Les données biologiques révèlent
une hyperuricémie importante, une créatinine sanguine à 195 micromol pour mille stable depuis 4 mois. La cytologie urinaire
est normale et il n'est pas noté de protéinurie. Le taux de phosphore est légèrement élevé, le potassium est normal, il n'y a
pas d'hyperaminoacidurie, ni de glycosurie.

Quelles sont parmi les suivantes les deux causes les plus probables d'insuffisance rénale ?
A - Hypertension avec néphroangiosclérose
B - Glomérulonéphrite chronique
C - Néphropathie toxique due à l'usage des AINS
D - Syndrome héréditaire de tubulopathie
E - Rein goutteux chronique par hyperuricémie
Bonne(s) réponse(s) : A E

E - Est le diagnostic à évoquer de principe, l'antécédent hématurique pouvant être rattaché à un problème lithiasique.
A - Peut être évoqué devant une insuffisance rénale modérée associée à une HTA ancienne.
On devra pratiquer un fond d'oeil afin de rechercher d'autres stigmates d'angiopathie. On peut toutefois être surpris de
l'absence de protéinurie. En effet la néphroangiosclérose de l'HTA modérée chronique s'accompagne d'une protéinurie, de
l'ordre ou inférieure à 1 g/24 h.
B - L'absence de protéinurie est un puissant argument contre cette hypothèse.
C - La toxicité des AINS se manifeste par des insuffisances rénales aiguës. Il faut bien distinguer toxicité rénale des Ains et
toxicité rénale des antalgiques comportant de la phénacétine.
D - L'absence d'hyperaminoacidurie, de glycosurie, l'âge et l'HTA sont autant d'arguments contre cette hypothèse.

Quel traitement hypouricémiant au long cours allez-vous établir ?


A - Prescription de furosémide (Lasilix®)
B - Urico-éliminateur (Amplivix®-benzodiarone)
C - Traitement anti-inflammatoire par corticoïdes : 20 mg de prednisone/jour
D - Uncolytique (Uricozyme® : 2 injections intramusculaires par semaine)
E - Zyloric® (allopurinol)
Bonne(s) réponse(s) : E

A - N'est pas un hypouricémiant


B - Augmente l'uraturie, ce qui est contre-indiqué en cas de néphropathie urique.
C - N'est pas un traitement hypouricémiant et augmente même l'uricémie.
D - Est indiquée dans les grandes hyperuricémies des hémopathies.

273
Exclusivement sur DOC - DZ : www.doc-dz.com NADJI 85
RESIDANAT EN POCHE TOME II
Cas Clinique en QCM

Quel est le type d'atteinte rénale entraîné par l'hyperuricémie ?


A - Néphropathie interstitielle
B - Nécrose tubulaire proximale
C - Nécrose tubulaire distale
D - Sclérose mésangiale
E - Troubles de perméabilité glomérulaire
Bonne(s) réponse(s) : A

Plusieurs facteurs contribuent à l'atteinte interstitielle : l'hyperuricémie, les lithiases, les surinfections urinaires.

En cas de décision de traitement hypotenseur, quel(s) est(sont) le(s) médicament(s) qui peu(ven)t être choisi(s)
en premier lieu dans ce cas ?
A - Polythiazide (Rénèse®)
B - Hydrochlorothiazide (Esidrex®)
C - Spironolactone (Aldactone®)
D - Aténolol (Ténormine®)
E - Nifédipine (Adalate®)
Bonne(s) réponse(s) : D E

A et B - Sont des thiazides hyperuraturiants donc non indiqués ici.


C est un épargneur du potassium, peu efficace dès qu'existe une insuffisance rénale et surtout contre-indiqué en raison du
risque d'hyperkaliémie majeure, même pour une insuffisance rénale modérée.
D - Est un bon traitement d'attaque de l'HTA, surtout chez un sujet encore relativement jeune.
E - L'Adalate® tend de plus en plus à faire partie de l'arsenal thérapeutique de l'HTA chronique. Sa bonne tolérance et la
quasi absence de contre-indications en font un bon médicament de première indication.

Pour prévenir la survenue d'une lithiase urique, vous conseillez :


A - Une diminution des apports calciques
B - Une diminution des apports glucidiques
C - Une alcalinisation des urines
D - Des gels d'alumine pour abaisser le phosphore
E - Des boissons abondantes
Bonne(s) réponse(s) : C E

Le traitement préventif de la lithiase urique (c'est la forme neutre acide qui précipite) repose sur l'hyperdiurèse alcaline.

On assiste à une aggravation de l'insuffisance rénale. Pour prévenir les troubles osseux, vous conseillez :
A - Une diminution des apports calciques
B - Une diminution des apports glucidiques
C - Une alcalinisation des urines
D - Des gels d'alumine pour abaisser le phosphore
E - Une prescription de vitamine D (dérivé dihydroxylé)
Bonne(s) réponse(s) : C D E

En cas d'insuffisance rénale chronique sévère, plusieurs facteurs sont responsables de l'atteinte osseuse : la diminution de la
1 alpha hydroxylation de la vitamine D avec induction d'une ostéomalacie et diminution de l'absorption intestinale du calcium.
L'inhibition de la 1 alpha hydroxylation rénale et une tendance à l'acidose des insuffisants rénaux aggrave l'altération osseuse
de ces patients en mobilisant les tampons osseux. Tout ce qui peut contribuer à réduire la carence en 1,25 dihydroxylase vit.
D, à augmenter les apports calciques, à réduire la phosphorémie, à lutter contre l'acidose de ces patients prévient les troubles
osseux.

274
Exclusivement sur DOC - DZ : www.doc-dz.com NADJI 85
RESIDANAT EN POCHE TOME II
Cas Clinique en QCM

Les anti-inflammatoires non stéroïdiens peuvent entraîner :


A - Une insuffisance rénale aiguë surtout chez les sujets hypovolémiques
B - Des oedèmes par leur action antiprostaglandine
C - Un blocage de la sécrétion de rénine
D - Une inhibition de l'angiotensine 2
E - Des syndromes néphrotiques
Bonne(s) réponse(s) : A B C E

A - Les prostaglandines, dérivées de l'acide arachidonique par la voie de la cyclooxygénase, sont d'importants modulateurs de
l'hémodynamique et des mouvements d'eau et de sodium du rein.
Dès que les systèmes de régulation rénale sont fortement stimulés (cas de l'hypovolémie), l'inhibition de la cyclooxygénase
bloque les systèmes de contrôle, et en particulier bloque la modulation des effets du système rénine angiotensine avec risque
d'insuffisance rénale aiguë. L'association chez un même sujet, d'un régime sans sel, de diurétiques, d'inhibiteurs de l'enzyme
de conversion et d'AINS expose à la survenue de façon quasi systématique d'une insuffisance rénale aiguë sévère, purement
"fonctionnelle" et réversible, mais néanmoins grave en raison des troubles métaboliques et de l'importance de l'hypovolémie
pouvant être atteinte.
B - Les prostaglandines exercent une action natriurétique au niveau tubulaire. Leur inhibition est susceptible d'induire des
rétentions hydrosodées importantes. Cet effet a été bien observé pour la phénylbutazone.
C - L'inhibition de la synthèse des prostaglandines ne bloque pas la sécrétion de rénine mais diminue la réponse à certains
stimuli tel l'orthostatisme.
E - Enfin, certains cas de syndromes néphrotiques à lésions glomérulaires minimes ont été rapportés à l'usage des AINS.

Mademoiselle G, est suivie depuis l'enfance dans un service de néphrologie pédiatrique. Elle a suivi les indications
thérapeutiques jusqu'à l'âge de 15 ans où elle a fait une fugue. Elle refuse depuis 2 ans l'avis de sa mère pour consulter en
médecine d'adulte. Elle présente une insuffisance rénale : dernier dosage il y a 1 an à 196 micromol pour mille de créatinine.
La protéinurie est de faible taux : 20 cg pour mille, diurèse moyenne de 2500/24 heures.

Quel(s) est(sont) à votre avis le(s) élément(s) qui est(sont) en faveur d'une protéinurie tubulaire ?
A - Faible taux par litre et par 24 heures
B - Présence d'une hématurie microscopique associée
C - Abondance de bêta 2 microglobuline
D - Protéinurie non sélective
E - Présence d'haptoglobine et d'immunoglobuline
Bonne(s) réponse(s) : A C

Les protéinuries tubulaires se caractérisent par leur taux relativement modeste (A) et par la large prédominance de protéines
de bas poids moléculaire, qui normalement, après avoir été filtrées par le glomérule doivent être réabsorbées par le tubule.
Leur présence dans l'urine traduit l'altération de cette fonction tubulaire. La bêta 2 microglobuline est une de ces protéines de
poids moléculaire inférieur à celui de l'albumine (C).
La présence d'une hématurie associée est un élément militant en faveur d'une origine glomérulaire avec sans doute des
lésions de prolifération. Le caractère sélectif ou non de la protéinurie, quand celle-ci est composée en majorité d'albumine est
un indice permettant de différencier lésions glomérulaires minimes ou lésions glomérulaires histologiquement décelables.
Mais il s'agit toujours de protéinurie glomérulaire (D). L'item E est également un indice de non sélectivité.

Ne font pas partiedu tableau habituel d'une insuffisance rénale sévère le(s) élément(s) suivant(s) :
A - Elévation de l'azote uréique
B - Elévation de la créatinine
C - Hypophosphorémie
D - Hypokaliémie
E - Anémie hypochrome
Bonne(s) réponse(s) : C D E

Au cours de l'insuffisance rénale chronique sévère, il y a élévation de l'urée et de la créatinine (A,B), tendance à
l'hyperkaliémie et à l'hyperphosphorémie par diminution de leur excrétion rénale (C,D). Une anémie est souvent associée,
multifactorielle (syndrome inflammatoire, hémolyse, carences multiples et en particulier martiale, déficit en érythropoïétine).
Cependant cette anémie est classiquement normochrome et non hypochrome (E).

275
Exclusivement sur DOC - DZ : www.doc-dz.com NADJI 85
RESIDANAT EN POCHE TOME II
Cas Clinique en QCM

A ce stade de l'insuffisance rénale, quelle(s) est(sont) parmi les suivantes la(les) mesure(s) diététique(s) qui
vous parai(ssen)t recevable(s) ?
A - Diminution modérée de l'apport protéique
B - Supplémentation calcique
C - Restriction de fruits et de légumes
D - Restriction liquidienne à 500 ml par 24 heures
E - Malgré le poids normal restriction calorique globale
Bonne(s) réponse(s) : A B

QCM très litigieux ! On ne dispose pas de la clearance actuelle de la créatinine ni même du taux sérique correspondant à
l'intitulé de la question, en effet le seul élément fourni est le taux mesuré il y a un an. Si l'on se réfère alors à cette donnée,
une insuffisance rénale modérée ne requiert alors aucune mesure diététique particulière sauf en cas de particularité liée à la
néphropathie responsable. Toutefois certains auteurs préconisent, mais pour des stades plus avancés de l'insuffisance rénale,
une diminution des apports protidiques, ce qui ralentirait l'évolution vers l'insuffisance rénale. En pratique ces mesures doivent
intervenir pour des clairances inférieures à 20 ml/min. A partir de ce stade, tous les auteurs s'accordent à conseiller un apport
quotidien de protides de l'ordre de 0,7 à 1 g/kg/j de protides.
En raison de l'atteinte du métabolisme de la vitamine D, il est conseillé, pour des clearances inférieures à 10-20 ml/min,.une
supplémentation.
L'item C vise à limiter les apports en potassium et ne s'applique que pour les insuffisances rénales avancées, dont la
clearance est inférieure à 10-20 ml/min.
L'item D ne s'applique que pour les patients anuriques. Chez les patients non anuriques où il y a au contraire perte de
capacités d'adaptation, une telle restriction peut conduire à un état de déshydratation intracellulaire sévère.
L'item E est strictement hors de propos. La seule indication d'un régime hypocalorique en néphrologie est l'association
néphropathie...obésité ! ! !

Alors que cette patiente avait une tension artérielle à la limite supérieure de la normale, elle présente une
hypotension orthostatique et une tension artérielle moyenne basse. Dans ce contexte, vous évoquez pour
l'expliquer :
A - Une surcharge hydrique
B - Un hyperfonctionnement de l'appareil juxtaglomérulaire
C - Un hyperaldostéronisme
D - Une aggravation progressive de l'insuffisance rénale
E - Une fuite sodée urinaire
Bonne(s) réponse(s) : E

Cette réponse est peu satisfaisante. Le tableau exposé évoque une hypovolémie efficace, indépendante de son étiologie. Le
seul item pouvant produire une déplétion volémique est E. Les items A, B et C induisent une rétention hydrosodée. L'item D
ne permet pas de préjuger l'évolution du secteur extracellulaire.

Une brutale augmentation du taux d'urée dans ce contexte peut être due à :
A - La prise de tétracycline pour une infection banale
B - Un diabète sodé et un régime désodé
C - Un apport excessif en potassium
D - Un traitement diurétique
E - Une augmentation de la consommation hydrique
Bonne(s) réponse(s) : B D

Ces deux éléments sont susceptibles d'induire une hypovolémie et d'aggraver ainsi l'insuffisance rénale par un facteur
fonctionnel.
La prise de tétracyclines, si ces produits sont périmés, est susceptible d'induire une néphropathie tubulaire. Les items C et E
sont susceptibles d'induire des perturbations métaboliques, hyperkaliémie ou hyponatrémie, mais non une majoration de
l'insuffisance rénale.

La radiographie osseuse montre une raréfaction de la trame, une diminution de la densité, des images de
résorption sous périostée. Des examens spécialisés ont des chances de montrer dans ces conditions :
A - Un taux abaissé de 1.25 OH D3
B - Un taux de calcium osseux mesuré par radioactivation élevé
C - Un taux de parathormone abaissé
D - Un bilan calcique négatif
E - Un calcium ionisé élevé
Bonne(s) réponse(s) : A D

Il y a déficit de la 1 alpha hydroxylation rénale de la vitamine D au cours de l'insuffisance rénale. Il y a alors diminution de
l'absorption calcique et atteinte osseuse ; il y a aussi hyperparathyroïdie secondaire avec atteinte osseuse. Cette
hyperparathyroïdie n'entraîne toutefois pas d'hypophosphorémie à cause de la diminution de l'excrétion rénale du phosphore.
L'hyperphosphorémie est partiellement responsable de l'inhibition de la 1 alpha hydroxylation.
Le taux de 125 vitamine D3 est donc abaissé (A), le calcium osseux diminué (B), le taux de parathormone élevé (C), le bilan
calcique négatif (D) et le calcium ionisé abaissé (E).

276
Exclusivement sur DOC - DZ : www.doc-dz.com NADJI 85
RESIDANAT EN POCHE TOME II
Cas Clinique en QCM
Un homme de 65 ans est hospitalisé pour dyspnée d'apparition récente et troubles confusionnels. Sont également
cliniquement notés :
- des antécédents de bronchite chronique sur un tabagisme ancien
- une HTA évoluant depuis 10 ans et traitée par Modurétic® et Catapressan®
- une altératIon récente de l'état général
- la T.A. est à 130/85, la T° à 37°
- à l'auscultatIon pulmonaire, diminution du murmure vésiculaire à gauche et râles bronchiques.
Le reste de l'examen clinique est normal. Dans le sang : hémoglobine 11,8 g/100ml, hématocrite 37 %, G.B 9 800, urée 0,20
g/l, créat 10 mg/l, uricémie 30 mg/l, protidémie 70 g/l, cholestérol 2,6 g/l, natrémie 115 mEq/l, chlore 80 mEq/l, kaliémie 4,5
mEq/l, osmolalite plasmatique 230 mOsm/kg, cortisolémie normale, bicar 28 mEq/l.
Dans les urines : prot 0, glycosurie 0, densité 1020, osmolalité 496 mOsm/kg.
La radiographie du thorax montre une atélectasie lobaire supérieure gauche avec épanchement pleural du même côté.

Dans quelle(s) circonstance(s) parmi les suivantes peut on rencontrer une hyponatrémie ?
A - Hyperlipidémie
B - Syndrome de Conn
C - Glycémie élevée
D - Hypercalcémie
E - Traitement par thiazides
Bonne(s) réponse(s) : A C E

A C - Représentent des cas de "fausses hyponatrémies", où la natrémie n'est plus le reflet de l'osmolarité plasmatique en
raison de l'existence d'autres osmoles. Dans le cas des hyperglycémies, ce tableau se rencontre dans les comas
hyperosmolaires. Au cours du traitement par un diurétique il peut y avoir hyponatrémie lorsque la perte en sel excède la perte
en eau. Cela correspond au tableau d'hyponatrémie par déplétion.
Dans le syndrome de Conn, il y a rétention hydrosodée avec hyperhydratation extracellulaire pure et le secteur intracellulaire
est normal ; il peut même y avoir hypernatrémie, en raison de l'activité "hydro-urétique". c'est à dire anti ADH des corticoïdes.

Quel est le diagnostic le plus vraisemblable chez ce patient ?


A - Syndrome de secrétion inappropriée d'A.D.H.(SIADH)
B - Insuffisance surrénale aiguë
C - Néphropathie avec perte de sel
D - Hyperhydratation par insuffisance cardiaque
E - Cirrhose hépatique
Bonne(s) réponse(s) : A

L'analyse des examens biologiques et du tableau clinique permet de conclure à une hypoosmolarité sanguine, c'est-à-dire à
une hyperhydratation intracellulaire, associée à une hydratation extracellulaire normale et à une osmolarité urinaire normale,
c'est-à-dire trop élevée en égard à l'hypo-osmolalité sanguine. Ce tableau correspond à une sécrétion inappropriée d'ADH (A).
L'insuffisance surrénale aiguë s'accompagne d'une acidose hyperkaliémique avec diminution du secteur extracellulaire (B).
Une néphropathie avec perte de sel entraîne également une diminution du secteur extracellulaire (C). Il n'y a enfin ni signe
d'insuffisance cardiaque (D) ni de cirrhose (E). L'origine de cette sécrétion inappropriée d'ADH est vraisemblablement un
syndrome paranéoplasique, satellite d'un cancer bronchopulmonaire. On rappelle, que les cancers bronchopulmonaires les
plus fréquemment à l'origine d'une sécrétion inappropriée d'ADH sont les cancers anaplasiques à petites cellules.

Dans un syndrome de Schwartz-Bartter (hypersécrétion inappropriée d'ADH) on note typiquement :


A - Une hypervolémie
B - Une perte potassique urinaire
C - Un hyperaldostéronisme
D - Une osmolalité urinaire basse
E - Une hypoglycémie
Bonne(s) réponse(s) : A

Il existe de fait une augmentation du secteur extracellulaire dans le syndrome de Schwarz Bartter. Cette hypervolémie,
cliniquement latente est mise en évidence par la mesure des espaces de dilution isotopique. Les autres items sont faux, en
particulier il existe une osmolarité urinaire élevée ou tout au moins supérieure à l'osmolarité plasmatique.

L'osmolalité urinaire chez ce patient vous paraît ?


A - Adaptée à son hyponatrémie
B - Anormalement élevée
C - Affirmer un mécanisme de diurèse osmotique
D - Concordante avec le diagnostic de SIADH
E - Aucune des propositions précédentes n'est exacte
Bonne(s) réponse(s) : B D

L'osmolarité urinaire est élevée pour ce patient car supérieure à l'osmolarité plasmatique qui est abaissée.

277
Exclusivement sur DOC - DZ : www.doc-dz.com NADJI 85
RESIDANAT EN POCHE TOME II
Cas Clinique en QCM

Parmi ces traitements lequel vous paraît nécessaire dans tous les cas ?
A - Furosémide
B - Thiazide
C - Restriction hydrique
D - Perfusion de sérum salé hypertonique
E - Lithium
Bonne(s) réponse(s) : C

La restriction hydrique est le premier élément du traitement de ces patients qui sont toujours exposés à la survenue d'une
intoxication par l'eau.

Une jeune femme de 27 ans se plaint depuis trois ans de troubles urinaires. A trois ou quatre reprises, il y deux ans, elle a
présenté, par crises, des douleurs sus-pubiennes avec sensation de pesanteur vésicale, des mictions très fréquentes et
douloureuses. Ont été alors pratiquées une cystoscopie et un urographie intraveineuse jugées normales. L'année précédente
a été indemne de tout épisode et une uroculture systématique a été trouvée stérile.
Depuis deux jours, elle présente les mêmes symptômes, avec, la veille, une hématurie macroscopique. La température est
normale, il n'a pas de protéinurie, la cytologie montre des hématies non déformées, une grande quantité de cellules et de
leucocytes. L'uroculture trouve 106 germes par ml. La patiente a réduit volontairement la quantité de boissons car les mictions
sont très douloureuses. La densité urinaire est de 1015.

Vous pratiquez une uroculture. Dans ce contexte, quel est le germe le plus fréquemment mis en évidence ?
A - Escherichia coli
B - Streptocoque hémolytique
C - Staphylocoque
D - Bacille pyocyanique
E - Proteus mirabilis
Bonne(s) réponse(s) : A

Escherichia coli est le germe le plus fréquemment retrouvé à l'origine d'infections urinaires basses (80%) ; le Protéus mirabilis
étant derrière (10 à 15%). Lors de réinfections à court terme, le "tiercé" est le suivant : E. Coli (60%), Klebsiella Pneumoniae
(30%) et Proteus (15%). Chez les patients opérés ou sondés : Klebsiella Pneumoniae (50%), Proteus (toutes espèces
confondues) (30%), E. coli (10%).
Le streptocoque D est un contaminant fréquent, qui pour être pris en compte, devra être retrouvé à plusieurs reprises et
s'accompagnant d'une leucocyturie élevée. Le staphylocoque doré se retrouve chez les patients souffrant d'une affection
prostatique, d'une infection urinaire, après manoeuvre instrumentale ou souffrant d'un abcès du rein. Le staphylocoque blanc
est retrouvé dans 2% des infections urinaires basses et surtout chez la jeune femme.

Vous retenez en faveur d'une infection urinaire strictement limitée à l'appareil urinaire inférieur (infection urinaire
basse) :
A - Densité urinaire à 1015
B - Existence d'une hématurie macroscopique
C - Absence de fièvre
D - Leucocyturie importante
E - Présence cylindres leucocytaires
Bonne(s) réponse(s) : C D

A - La densité urinaire n'a aucun rapport avec l'infection.


B - Une hématurie macroscopique peut tout à fait accompagner une infection urinaire sans valeur localisatrice. Elle devra
néanmoins faire suspecter une autre cause de saignement.
C - Evident.
D - Aucune valeur localisatrice de l'infection.
E - La présence de cylindres milite en faveur de l'origine tubulaire des leucocytes.

L'attitude que vous adoptez comporte :


A - Gentalline® IM 2 fois 40 mg par jour pendant 8 jours
B - Irrigation intravésicale d'anti-inflammatoires ou de corticoïdes
C - Boissons abondantes, 2 litres d'eau par 24 heures
D - Anti-infectieux pendant 5 jours (acide pipémidique ou Pipram®)
E - Consultation gynécologique
Bonne(s) réponse(s) : C D E

A - La Gentalline® n'est pas une antibiothérapie de première intention d'une infection urinaire basse.
B - Inconcevable.
C - Premier élément du traitement anti-infectieux : assurer un flux vésical important.
D - Evident.
E - Il s'agit d'une infection urinaire récidivante chez une femme. Une enquête étiologique doit donc être entreprise, passant en
particulier par un examen gynécologique, à la recherche, en particulier, d'une bride hyménéale résiduelle.

278
Exclusivement sur DOC - DZ : www.doc-dz.com NADJI 85
RESIDANAT EN POCHE TOME II
Cas Clinique en QCM

Dans l'hypothèse où il s'agirait d'un Proteus mirabilis résistant au traitement, vous devriez craindre que :
A - Au cours d'une grossesse, il existe un risque important de pyélonéphrite
B - Même en l'absence de malformation, il existe un risque important d'insuffisance rénale d'évolution rapide
C - Il existe un risque de diabète
D - Il existe un risque de lithiase rénale
E - La patiente doit interrompre la prise de pilule contraceptive
Bonne(s) réponse(s) : D

La résistance au traitement doit faire rechercher un facteur favorisant. Protéus mirabilis favorise la survenue de lithiase
phosphato-ammoniaco-magnésienne en alcalinisant les urines par l'action de son uréase (transformation de l'urée avec
libération d'ammonique).

Un homme de 75 ans, 65 kg, 175 cm, a une fièvre à 40° avec frissons. Il est obnubilé, ininterrogeable, mais l'entourage dit que
la fièvre a commencé il y a trois jours, avec une vive douleur du flanc droit, et qu'il a des difficultés à uriner depuis 2 mois.
C'est un diabétique hypertendu, qui prend du Diamicron®, 2cp/j, et de l'Avlocardyl®, 2 cp/j.
A l'examen, la fosse lombaire droite est sensible, l'abdomen ballonné. Le pannicule adipeux et le ballonnement gênent
l'examen de l'hypogastre. Au toucher rectal, la prostate est un peu grosse, avec un effacement du sillon médian, mais bien
limitée. Pression artérielle 14/8 cmHg. Pas de raideur de la nuque, pas de sueurs, pas de signes de localisation neurologique.
Test au nitrite des urines = positif. Créatininémie = 180 micromoles/l. Natrémie = 148 mmoles/l. Chlorures = 110 mmoles/l.
Natriurèse = 12 mmoles/l. Créatininurie = 18 mmoles/l.

Quelle hypothèse étiologique retenez vous en priorité pour expliquer l'obnubilation de ce patient ?
A - La créatininémie à 180 micromoles/l
B - Une encéphalopathie toxique des bêta-bloqueurs
C - Une hypoglycémie
D - Un accident vasculaire cérébral
E - La fièvre à 40°C
Bonne(s) réponse(s) : C

Il s'agit d'un patient âgé, présentant :


a) un syndrome infectieux sévère à point de départ urinaire
b) des troubles de la conscience sans signe de localisation
c) une insuffisance rénale avec chute de la natriurèse et déshydratation intracellulaire modérée.
Il s'agit enfin d'un patient diabétique et hypertendu sous bêta-bloquants et sulfamides hypoglycémiants. Le syndrome
infectieux, les troubles hydroélectrolytiques ne peuvent expliquer les troubles de la conscience. L'absence de signes de
localisation milite contre un accident vasculaire cérébral.
Par contre, la survenue d'un trouble de la conscience chez un diabétique doit faire systématiquement évoquer un "coma du
diabétique" (tiroir) : hypoglycémie, acidocétose, acidose lactique, coma hyperosmolaire. Il n'y a pas d'acidose évidente,
l'hyperglycémie ou une importante glycosurie ne sont pas mentionnées. Par contre de nombreux éléments militent en faveur
de l'hypoglycémie : prise d'hypoglycémiant, jeûn vraisemblable du patient, insuffisance rénale.

Concernant une créatininémie à 180 micromoles/l chez un homme de 70 ans, vous pouvez conclure que :
A - Sa filtration glomérulaire (FG) est supérieure à 100 ml/mn/1,73m2 car ce chiffre de créatininémie est
habituel à cet âge
B - Sa FG est réduite, comme souvent à cet âge
C - Sa FG est certainement inférieure à 50 ml/mn
D - Sa FG est de l'ordre de 80 ml/mn
E - Sa créatininémie est seulement due à ses erreurs diététiques
Bonne(s) réponse(s) : C

Le calcul de la clearance de la créatinine est identique de 7 à 77 ans et compte tenue de la masse musculaire réduite du
sujet, une clearance de 120 ml/min correspondrait chez ce sujet de 65 kg à une créatininémie de l'ordre de 60 mcM ce qui
correspond à une clearance de l'ordre de 120/3 = 40 ml/min. Enfin la créatininémie n'est pas influencée par le régime mais
uniquement par la masse musculaire du patient et sa filtration glomérulaire.

En faveur de la nature fonctionnelle de l'insuffisance rénale. Vous retenez :


A - La natriurèse à 12 mmoles/l
B - Le rapport U/P créatinine élevé
C - La fièvre
D - La créatininémie à 180 micromoles/l
E - Les données du toucher rectal
Bonne(s) réponse(s) : A B C

Les items A et B ne posent pas de problème et sont de connaissance pure. La fièvre n'est pas un des éléments du tableau
d'insuffisance rénale fonctionnelle mais a contribué à la déshydratation, facteur d'insuffisance rénale fonctionnelle.

279
Exclusivement sur DOC - DZ : www.doc-dz.com NADJI 85
RESIDANAT EN POCHE TOME II
Cas Clinique en QCM

Quelle(s) information(s) tirez-vous de la natrémie à 148 mmoles/l ?


A - C'est un bon signe d'hypervolémie
B - C'est un bon signe de déshydratation cellulaire
C - Ce chiffre peut être normal chez le vieillard
D - La soif est habituelle avec ce chiffre
E - Ce chiffre montre que le patient n'a pas suivi sa restriction sodée
Bonne(s) réponse(s) : B D

Il y a équilibre entre les osmolarités intracellulaire et extracellulaire. Le nombre d'osmoles intracellulaires étant fixe, les
variations de l'osmolarité intracellulaire et donc extracellulaire sont déterminées par les variations de l'hydratation
intracellulaire. L'osmolarité extracellulaire évoluant de façon parallèle à la natrémie sauf en cas d'autres agents
osmotiquement actifs.

Une urographie intraveineuse a été proposée. Indiquez la(les) proposition(s) juste(s) :


A - L'UIV fait courir un risque notable d'aggravation de l'insuffisance rénale chez ce sujet âgé et diabétique
B - L'échographie rénale montrera une lithiase éventuelle, quelque soit son siège sur les voies urinaires
C - Le diagnostic d'obstacle n'est pas urgent dans ce contexte
D - Il n'y a aucun signe clinique en faveur d'un obstacle justifiant l'échographie ou l'UIV
E - L'âge et la consistance de la prostate sont suffisants pour porter le diagnostic de rétention sur adénome
prostatique
Bonne(s) réponse(s) : A C

Ce Q.C.M. est particulièrement ambigu et mal posé.


A - Est exact et surtout en raison de l'état de déshydratation importante probable de ce patient.
B - L'échographie peut montrer une lithiase ou son éventuel retentissement, mais pas systématiquement, en particulier si la
lithiase se trouve au niveau du tiers moyens de l'uretère.
C - Le plus urgent dans ce contexte est l'administration de glucose afin de traiter l'hypoglycémie vraisemblable de ce sujet, la
recherche d'un obstacle, en particulier blocage d'urines, après la correction de l'hypoglycémie et de la déshydratation de ce
patient.
D - La sensibilité de la fosse lombaire, le ballonnement abdominal, l'insuffisance rénale doivent faire discuter un obstacle.
L'existence d'une grosse prostate également ; en effet, plusieurs hypothèses peuvent être discutées pouvant expliquer la
constitution d'un obstacle.
Soit il y a rétention d'urine et retentissement sur le haut appareil, soit il y a envahissement du trigone par un cancer de la
prostate (remarquez l'effacement du sillon médian) avec obstruction urétérale unilatérale. Il peut tout à fait y avoir anurie par
obstacle sur un rein et insuffisance rénale fonctionnelle pour le rein épargné.
E - L'absence de globe, généralement bien retrouvé chez un sujet relativement maigre en dépit de la mention d'un pannicule
adipeux dans l'énoncé, et l'absence de besoin mictionnel, l'absence d'état d'agitation, militent contre l'existence d'une
rétention. Si tous les sujets âgés présentant une prostate discrètement augmentée de volume étaient en rétention !...

Une jeune femme âgée de 25 ans consulte pour des infections urinaires à répétition évoluant depuis plusieurs années. Le
germe responsable est constamment un Escherichia coli multirésistant aux antibiotiques usuels. Dans ses antécédents, on
note qu'elle a présenté des infections urinaires très tôt dès l'enfance, et qu'elle a subi à l'âge de 9 ans une intervention
antireflux bilatérale en raison d'un reflux vésico-urétéral bilatéral avec duplicité urétérale gauche. De plus, des épisodes
fréquents de pyélonéphrite gauche ont motivé une néphrectomie polaire inférieure gauche 8 mois auparavant, mais sans
réduction de la fréquence des infections. Elle signale même !a persistance, lors de certains épisodes infectieux, de douleurs
lombaires gauches, sans fièvre. Une nouvelle cystographie rétrograde montre, à gauche, un reflux vésicorénal et un reflux
dans le moignon urétéral inférieur restant.

Avant de prendre une décision thérapeutique, quel examen complémentaire vous paraît souhaitable pour
éliminer la possibilité d'un reflux éga!ement à droite ?
A - Echographie rénale
B - Cystoscopie
C - Scintigraphie rénale
D - Exploration urodynamique
E - Urographie intraveineuse
Bonne(s) réponse(s) : E

Question annulée lors du concours.


La réponse aurait été E : répétition du temps cystographique et en particulier du temps permictionnel qui sera réalisé sur de
grands clichés englobant les reins pendant la miction.

280
Exclusivement sur DOC - DZ : www.doc-dz.com NADJI 85
RESIDANAT EN POCHE TOME II
Cas Clinique en QCM

Typiquement et sur une nouvelle urographie, le reflux pourrait se traduire par :


A - Gros rein gauche muet
B - Dilatation globale des cavités pyélocalicielles
C - Déformation des calices en massue prédominant aux pôles supérieurs
D - Hypertrophie corticale en regard des calices déformés
E - Calcifications péripapillaires en flammèches
Bonne(s) réponse(s) : B C

A - On peut effectivement avoir une image de rein muet si la destruction parenchymateuse est importante. Cela ne constitue
pas néanmoins un aspect typique.
B - Il est classique de pouvoir observer une hypotonie des cavités excrétrices avec un aspect de dilatation.
C - L'aspect en "massue" des calices devenus convexes est classique.
D - Il existe, et c'est l'image séquellaire de reflux la plus évocatrice, une atrophie corticale en regard des calices déformés et
non une hypertrophie.
E - Aspect non décrit dans la littérature.

Avant de décider du traitement il est indispensable de disposer de :


A - Cytobactériologie urinaire
B - Antibiogramme
C - Détermination des CMI aux antibiotiques usuels
D - Dosage de créatinine plasmatique
E - Précisions sur ces traitements antérieurs
Bonne(s) réponse(s) : A B D E

A - Est indispensable avant de débuter le traitement.


B - Est nécessaire pour adapter le traitement.
C - C'est un antibiogramme.
D - Indispensable pour adapter la posologie des antibiotiques.
E - Utile.mais non strictement indispensable pour préciser d'éventuels effets secondaires, l'efficacité habituelle du traitement.

Le germe urinaire s'avère résistant à tous les antibiotiques usuels sauf aux aminosides comme la Gentalline®.
Quel schéma de traitement médical vous parait le plus adapté ?
A - Association bêtalactamine IM et aminoside IM 2 fois par jour pendant 7 jours
B - Aminoside seul IM en infusion vésicale
C - Aminoside seul IM 1 fois par jour pendant 15 jours
D - Aminoside seul IM 1 fois par jour pendant 6 mois
E - Pas de traitement symptomatique de cette infection chronique
Bonne(s) réponse(s) :

Réponse impossible.
Aucun des items proposés n'est satisfaisant. Devant une infection urinaire récidivante à germe multirésistant survenant sur
une lésion urologique (type IV de la classification de Wallace et Petersdorf) on proposera un traitement d'attaque par exemple
par l'association bêta-lactamine-aminoside, relayé par une prophylaxie anti-infectieuse continue par une quinolone par
exemple. Ce traitement ne se conçoit enfin que dans l'optique d'un traitement chirurgical.

Ce traitement médical doit être associé à :


A - Une néphrectomie gauche sans tarder
B - A une nouvelle intervention antireflux gauche sous couvert d'antibiotiques
C - A une intervention antireflux bilatérale systématique
D - A une exérèse du moignon urétéral gauche
E - A aucun traitement chirurgical tant que les urines ne sont pas durablement stérilisées
Bonne(s) réponse(s) : B D

Là encore la réponse est difficile en l'absence de données précises. Il s'agit d'indications délicates qui doivent être examinées
cas par cas. Il semble toutefois raisonnable de proposer a priori et en l'absence d'autres données les items B et D.

281
Exclusivement sur DOC - DZ : www.doc-dz.com NADJI 85
RESIDANAT EN POCHE TOME II
Cas Clinique en QCM

En dehors des infections urinaires, la correction du reflux à un effet favorable sur :


A - Lithiase coralliforme
B - Hypertension artérielle
C - Pyélonéphrite chronique
D - Perte de sel
E - Insuffisance rénale
Bonne(s) réponse(s) : A C E

QCM ambigu à nouveau : la solution A est récusée.


La correction du reflux peut prévenir la survenue d'une lithiase coralliforme, mais cet effet bénéfique est dû à la disparition des
infections urinaires. L'item A est intrinsèquement exact, mais doit être récusé compte tenu de l'énoncé précisant "en dehors
des infections urinaires". L'intérêt majeur de la chirurgie du reflux est de stopper le processus de dégradation de la fonction
rénale par néphropathie interstitielle chronique. Soulignons à nouveau l'ambiguïté de ce QCM. En effet stricto sensu, la
pyélonéphrite chronique correspond à l'infection chronique des cavités pyéliques et du parenchyme. Compte tenu de l'énoncé,
l'item devrait être récusé. Cependant, le terme de pyélonéphrite chronique est souvent abusivement utilisé pour désigner toute
néphropathie interstitielle chronique, et alors l'item est exact. Il existe en effet dans le reflux deux mécanismes de lésions
rénales : la survenue d'infections urinaires récidivantes, éventuellement compliquées par la survenue d'une lithiase
coralliforme, et, d'autre part, les à-coups urodynamiques engendrés par le reflux.

Monsieur C. Christian, 45 ans, de profession libérale se présente à la consultation pour hypertension. Cette hypertension est
de découverte récente, d'environ 10 mois, elle a été l'objet il y a 8 mois d'un court bilan : il existait une hypertrophie
ventriculaire gauche d'importance moyenne à cette époque, pas de protéinurie, ni d'hématurie. On lui a conseillé l'achat d'un
autotensiomètre. Depuis un mois il accuse des céphalées et les chiffres qu'il mesure le matin sont en moyenne de 180/120
mmHg. Il a présenté 4 ans auparavant une lithiase urétérale obstructive à droite dont le traitement radical par cathétérisme
ascendant a été tardif et compliqué d'une pyélonéphrite dont la guérison a nécessité deux mois d'antibiothérapie. Il n'a pas eu
de récidive. Ses parents directs et sa fratrie sont normotendus. A votre examen, la pression artérielle est en moyenne à
195/130, le fond d'oeil est de stade II-III ; dans les urines protéines traces, sang +, cultures stériles.

Parmi les données fournies vous retenez en faveur d'une hypertension artérielle secondaire dans le contexte
cité :
A - L'âge
B - L'existence d'une protéinurie le jour de l'examen
C - L'absence d'antécédent familial
D - Les antécédents "rénaux"
E - L'existence de signes rétiniens
Bonne(s) réponse(s) : A C D E

Voir question précédente.

Les dosages sanguins montrent : créatinine 180 micromoles et potassium 3,1 mmol par litre. Cette hypokaliémie
est à rapporter selon vous à :
A - Une récidive lithiasique
B - Une insuffisance rénale chronique
C - Une tubulopathie congénitale
D - Une néphropathie tubulo-interstitielle due au traitement antibiotique prolongé
E - Un hyperaldostéronisme
Bonne(s) réponse(s) : E

Voir questions précédentes.

Choisissez un examen complémentaire parmi les suivants que vous ferez rapidement et en premier lieu :
A - Electronystasmogramme
B - Un enregistrement continu de la pression artérielle
C - Une échographie rénale
D - Une artériographie aortique et rénale
E - Un cathétérisme separateur des uretères
Bonne(s) réponse(s) : C

Voir questions précédentes.

282
Exclusivement sur DOC - DZ : www.doc-dz.com NADJI 85
RESIDANAT EN POCHE TOME II
Cas Clinique en QCM

Le patient ne suit pas vos conseils et ne se traite pas. Il voit un ami radiologue qui pratique une urographie
intraveineuse. Celle-ci révèle un petit rein droit de 7.5 cm de diamètre vertical. Le rein gauche est apparemment
de taille normale. Il existe un retard de sécrétion des deux reins. L'insuffisance rénale s'explique par :
A - Une tuberculose rénale
B - Une gloménulonéphrite chronique
C - Une tubulopathie au niveau du rein droit
D - Une lithiase obstructive
E - L'atrophie du rein droit avec néphroangiosclérose du rein gauche
Bonne(s) réponse(s) : E

Compte tenu de la sévérité de l'HTA ayant déjà entraîné des signes cardiaques et ophtalmologiques, l'item E est le plus
vraisemblable.

Si vous décidez d'employer un traitement médical, quel est celui qui vous apparaît adapté au contexte et
adéquat en monothérapie ?
A - Réserpine
B - Polythiazide
C - Acébutolol
D - Dihydralazine
E - Isoniazide
Bonne(s) réponse(s) :

QUESTION ANNULEE.
Dans l'HTA par atrophie rénale unilatérale, la stratégie doit suivre la démarche habituelle du traitement de l'HTA. A noter que
l'atteinte cardiaque est une hypertrophie et non une insuffisance congestive. Elle ne contre-indique donc pas l'utilisation d'un
bêta-bloquant, intéressant par ailleurs car dépresseur du système rénine angiotensine. Le thiazide est ici non indiqué en
raison de l'existence d'une insuffisance rénale. De surcroît, au cas où il conserverait quelque activité, il est déconseillé de
l'utiliser en raison de l'hypokaliémie qu'il pourrait majorer.

Madame Hortense F. est une paysanne de 54 ans : elle élève dans une ferme de Gimont dans le Gers quelques vaches et
gave soigneusement ses oies tout au long de l'année pour une entreprise de foie gras. Elle est en bonne santé, ne se
souvient pas avoir été malade. Il y a 2 mois elle est tombée d'une trappe de son grenier à foin et a été plâtrée un mois pour
une fracture malléolajre droite. Elle ressent quelques douleurs à cette cheville et au mollet droit, cette jambe est d'ailleurs
restée discrètement enflée. La patiente se plaint également de quelques douleurs lombaires vagues non systématisées.
Depuis trois semaines, elle a pris plusieurs kilos, se sent boudinnée, les jambes, le visage et les mains sont très enflés. La
protéinurie est trouvée à 12 g/24 heures.

Dans un tel tableau, la(les) constatation(s) biologique(s) suivante(s) est(sont) fréquente(s) :


A - Baisse du calcium total
B - Baisse du calcium non ultrafiltrable
C - Taux de protéines totales sanguines entre 50 et 60 g/l
D - Taux d'albumine plasmatique au-dessus de 30 g/l
E - Cholestérol abaissé
Bonne(s) réponse(s) : A C D

Le diagnostic évoqué par ce cas est celui de syndrome néphrotique.


A - Il y a baisse du calcium par diminution de la fraction liée aux protéines.
B - Cf réponse en A.
C - Souvent la protidémie est comprise entre 50 et 60 g/l, elle est parfois plus abaissée encore.
D - Critère de la définition du syndrome néphrotique.
E - Il y a au contraire hypercholestérolémie. L'hyperlipidémie du syndrome néphrotique étant du type IIa, IIb ou IV.

L'échographie rénale révèle deux reins de taille augmentée de 14 et 15 cm de diamètre vertical, de densité et
de structure homogènes, aux cavités non dilatées. Parmi les propositions suivantes, cochez la(les) propositions
correcte(s) :
A - Cet aspect est pathognomonique d'une amylose
B - Le diagnostic est celui de polykystose rénale
C - C'est l'aspect d'une glomérulonéphrite chronique ancienne avec insuffisance rénale
D - C'est l'aspect d'une sténose artérielle rénale bilatérale
E - C'est un aspect habituel dans un syndrome néphrotique au début
Bonne(s) réponse(s) : E

Sans commentaire, connaissance pure.

283
Exclusivement sur DOC - DZ : www.doc-dz.com NADJI 85
RESIDANAT EN POCHE TOME II
Cas Clinique en QCM

Parmi les propositions suivantes concernant ce cas, cochez la(les) proposition(s) exacte(s) :
A - La corticothérapie a moins de chance d'être active que chez l'enfant
B - Les formes à rein optiquement normal sont moins fréquentes que chez l'enfant
C - Les formes à rein optiquement normal sont possibles même à cet âge
D - Un taux de protéinurie aussi important traduit une hyalinose segmentaire et focale
E - Un tel taux de protéinurie est incompatible avec une fonction rénale normale
Bonne(s) réponse(s) : A B C

La corticothérapie est essentiellement efficace sur les formes à glomérules optiquement normaux, forme moins fréquente chez
l'adulte, ce qui justifie la PBR systématique, mais pouvant néanmoins survenir chez un adulte.

La pression artérielle étant basse, vous pensez trouver dans ce contexte :


A - Une activité rénine basse
B - Une volémie plasmatique élevée
C - Un taux d'aldostérone élevé
D - Une kaliémie haute
E - Une hypotension orthostatique
Bonne(s) réponse(s) : C E

Il y a inflation du secteur extracellulaire, mais au seul profit du secteur interstitiel. Ceci s'accompagne d'une hypovolémie
circulante, avec hyperaldostéronisme secondaire, avec donc ARP élevée, aldostérone élevée et fuite potassique.
L'hypotension orthostatique est un stigmate de cette hypovolémie efficace.

La biopsie rénale montre qu'il existe des dépôts extramembraneux d'immunoglobulines. Dans ce contexte, il est
exact que :
A - L'évolution vers l'insuffisance rénale terminale se fera en trois mois
B - Il faut rechercher une maladie générale associée
C - L'évolution se fait vers 70 % de rémission spontanée
D - La corticothérapie est efficace dans 80 % des cas
E - Un traitement comportant ultrafiltration et plasmaphérèse doit être entrepris
Bonne(s) réponse(s) : B C

En présence d'une glomérulite extramembraneuse, il faut rechercher une pathologie générale associée, schématiquement
une collagénose type lupus chez l'adulte jeune, et une pathologie maligne chez le sujet âgé. L'évolution se fera de façon
progressive et très lentement vers l'insuffisance rénale chronique, à moins que la pathologie responsable soit traitée. Cette
évolution est entrecoupée de phases de rémissions +/- spontanées. Les divers traitements proposés n'ont jamais démontré
leur efficacité.

Parmi les complications possibles, on retient :


A - Une nécrose papillaire rénale
B - Une nécrose corticale rénale
C - La constitution rapide d'une hypertension artérielle maligne
D - Des infarctus rénaux
E - Une thrombose veineuse cave et rénale
Bonne(s) réponse(s) : E

La survenue d'accidents thromboemboliques constitue l'une des complications les plus fréquentes, avec la survenue
d'épisodes infectieux, du syndrome néphrotique.

Si le tableau d'anasarque se complétait et que vous décidiez de prescrire un traitement diurétique, compte tenu
du contexte, vous choisiriez en première intention :
A - Perfusion d'albumine et de mannitol
B - Perfusion de sérum salé isotonique 2 litres
C - Perfusion de sérum glucosé isotonique 2 litres
D - Injection d'aldostérone
E - Perfusion de 500 mg de Lasilix®
Bonne(s) réponse(s) : A

Réponse peu satisfaisante, le mannitol n'étant pas employé. On utilise en fait l'association perfusion d'albumine, Lasilix®.

284
Exclusivement sur DOC - DZ : www.doc-dz.com NADJI 85
RESIDANAT EN POCHE TOME II
Cas Clinique en QCM
Un jeune homme de 21 ans a eu un accident de vélomoteur il y a 4 mois. Celui-ci a entrainé une fracture des os propres du
nez et une abrasion cutanée de toute la face antérieure de la cuisse droite. Il a subi une réduction chirurgicale qui laisse
subsister une déviation de la cloison nasale et une greffe cutanée. Chaque induction anesthésique entraine des
manifestations d'intolérance avec rougeurs cutanées, prurit, tachycardie. De la même façon, un traitement initial à la
pénicilline a été mal toléré avec réaction prurigineuse et oedémateuse. Il existe par ailleurs un terrain familial de bronchites
asthmatiformes, de coryza allergique et d'eczéma.
Depuis un mois, il existe un léger oedème palpébral et des doigts. Les mollets sont un peu douloureux. Un léger oedème est
trouvé à la cheville droite. La recherche de la protéinurie était négative au moment de l'accident : il y a un mois la bandelette
révélait de + à ++. Depuis une semaine le visage est bouffi, les oedèmes des membres inférieurs sont nets, les mains sont
boudinées et le sujet a pris 5 kg.

Le diagnostic le plus probable est :


A - Insuffisance ventriculaire droite
B - Insuffisance cardiaque globale
C - Thrombose de la veine cave
D - Syndrome néphrotique
E - OEdèmes allergiques
Bonne(s) réponse(s) : D

Il n'y a pas de signes d'IVG ni d'IVD (items A et B). Le territoire des oedèmes concerne aussi bien le territoire cave supérieur
que le territoire cave inférieur. Il n'y a pas de signes allergiques au moment de l'installation des oedèmes ; en particulier ils ne
sont pas prurigineux.

Le taux d'albumine est de 24 g/l, le taux de protéines de 52 g/l et la kaliémie est à 3,2 Meq/l. Le(s) facteur(s)
suivant(s) peu(ven)t être impliqué(s) dans le mécanisme des oedèmes :
A - Baisse de la pression oncotique des protéines
B - Hyperaldostéronisme
C - Hyperpression veineuse
D - Capillarité allergique
E - Hypersécrétion d'ADH
Bonne(s) réponse(s) : A B

Remarque : si vous aviez des doutes pour répondre au 1, il fallait lire la question 2 !
Au cours du syndrome néphrotique, la génèse des oedèmes résulte de la fuite du sérum du compartiment circulant vers le
compartiment interstitiel à cause de la chute de la pression oncotique des protides intravasculaires. Il y a alors
hyperaldostéronisme secondaire, car il y a hypovolémie efficace, et la rétention hydrosodée ainsi obtenue contribue à
augmenter le volume des oedèmes, car il n'y a toujours pas de protides pour maintenir l'eau et le sel dans le compartiment
vasculaire.

Compte tenu des constatations, le(s) traitement(s) suivant(s) vous paraî(ssent) adéquat(s) :
A - Digitalique et diurétique thiazique
B - Perfusion de sérum bicarbonaté isotonique
C - Perfusion de soluté protéique : albumine ou plasma
D - Antialdostérone
E - Mannitol 10 %, 250 ml en perfusion
Bonne(s) réponse(s) : C D

A - Il n'y a pas de problème cardiaque justifiant la mise sous digitaliques, qui seraient d'ailleurs à manipuler avec la plus
extrême prudence en raison de l'hypokaliémie.
B - Il n'y a pas d'acidose et l'apport de sel contribue à accentuer la formation des oedèmes.
C - C'est effectivement une attitude thérapeutique justifiée, car c'est permettre un retour vers le compartiment vasculaire du
liquide interstitiel.
D - Constitue un des traitements idéaux de la situation, par son action antialdostérone.
E - Difficile à manipuler, ce d'autant que le sujet est en hypovolémie efficace.

Il a été décidé sans examen complémentaire de faire un traitement actif. Quel est le schéma choisi parmi les
suivants ?
A - Ciclosporine 10 mg/kg/jour pendant six mois
B - Cyclophosphamide 3 mg/kg/jour pendant trois mois
C - Prednisone 0,10 mg/kg/jour pendant six mois
D - Prednisone 0,10 mg/kg/jour pendant quatre semaines
E - Prednisone 1,5 mg/kg/jour pendant quatre semaines
Bonne(s) réponse(s) : E

S'agissant d'un patient adulte, une ponction biopsie rénale devra être toutefois systématiquement pratiquée. Le traitement
sera en particulier suivi sur l'évolution de la protéinurie.

285
Exclusivement sur DOC - DZ : www.doc-dz.com NADJI 85
RESIDANAT EN POCHE TOME II
Cas Clinique en QCM

Le traitement s'est avéré efficace. Il survient trois rechutes dans les six mois suivants. La derniere rechute
s'avère résistante. La protéinurie et les oedémes persistent. Quelle est l'attitude à adopter ?
A - Traitement symptômatique des oedèmes
B - Traitement symptômatique des oedèmes et antiagrégants plaquettaires
C - Poursuite du traitement corticoïde à faible dose
D - Poursuite du traitement corticoïde à forte dose
E - Néphrectomie bilatérale
Bonne(s) réponse(s) : B

La poursuite de la corticothérapie étant inefficace, le traitement étiopathogénique devra être guidé par les résultats de la PBR.
Par ailleurs il convient de poursuivre un traitement symptomatique (restriction sodée, alimentation hyperprotidique et enrichie
en potassium, sauf si insuffisance rénale, diurétique) associé à un traitement préventif de la survenue d'accidents
thromboemboliques, car une des grandes complications des syndromes néphrotiques est la survenue de phlébothrombose, le
risque étant corrélé à l'importance de la protéinurie et de l'hypoprotidémie.

Une biopsie révèle une hyalinose segmentaire et focale. Quelle(s) est(sont) la(les) complications envisagée(s)
en l'absence de traitement ?
A - Survenue d'une insuffisance rénale dans les cinq ans
B - Thrombose veineuse à bas bruit
C - Thrombose veineuse extensive et risque d'embolie pulmonaire
D - Hypocalcémie
E - Troubles trophiques
Bonne(s) réponse(s) : A B C D E

Sans commentaire, connaissance pure.

Une femme de 57 ans est adressée à la consultation pour la découverte, au cours d'un bilan systématique à un centre de
sécurité sociale, d'une insuffisance rénale modérée à 180 micromol/l de créatinine. Sa mère a présenté une glomérulonéphrite
après une scarlatine et a vécu jusqu'à 76 ans avec une protéinurie. Son père est décédé d'accident vasculaire cérébral peu de
temps après avoir présenté un infarctus myocardique. Un frère a présenté une lithiase rénale. Elle-même a eu deux
grossesses normales et trois enfants. Elle se souvient avoir présenté, il y a 20 ans, à la suite de la 2ème, une crise de colique
néphrétique droite avec fièvre et frissons qui ont récidivé pendant trois semaines. Depuis aucune manifestation rénale, elle est
traitée depuis 4 ans par clonidine (Catapressan®) 3 comprimés par jour pour une hypertension modérée. Elle n'a pas de
protéinurie.

Dans un tel tableau clinique et à ce stade, il est plausible d'envisager le ou les diagnostics étiologiques suivants :
A - Néphroangiosclérose
B - Pyélonéphrite chronique
C - Nécrose corticale
D - Glomérulonéphrite chronique
E - Toxicité rénale du Catapressan®
Bonne(s) réponse(s) : B

A - Le caractère modéré et récent de l'hypertension artérielle, l'absence d'une discrète protéinurie milite contre le diagnostic de
néphroangiosclérose.
B - Plus précisément le tableau est évocateur d'HTA sur atrophie corticale segmentaire, survenue au décours d'un épisode de
pyélonéphrite, sur obstacle vraisemblable.
C - Il ne faut pas confondre nécrose corticale et atrophie corticale segmentaire. Le tableau de la nécrose corticale est celui
d'une insuffisance rénale aiguë sévère : une nécrose tubulaire anurique qui ne régresse pas...
D - L'absence de protéinurie est en défaveur de ce diagnostic.
E - Il n'y a pas de toxicité rénale du Catapressan®, qui par contre est éliminé par le rein.

Quel est, parmi ceux-ci, l'examen à réaliser sans délai ?


A - Scanner
B - Etude cinétique par R.M.N.
C - Artériographie conventionnelle
D - Urographie intraveineuse
E - Immunoélectrophorèse urinaire
Bonne(s) réponse(s) : E

C'est l'examen de choix permettant le diagnostic de la question 55. Toutefois, cet examen peut être utilement remplacé par
l'échographie.

286
Exclusivement sur DOC - DZ : www.doc-dz.com NADJI 85
RESIDANAT EN POCHE TOME II
Cas Clinique en QCM

Une échographie montre un petit rein droit, 6,5 cm de longueur, avec une image dense dessinant un cône
d'ombre. Le rein gauche est normal. Votre diagnostic est :
A - Tumeur de la voie excrétrice
B - Glomérulonéphrite atrophiante
C - Thrombose de la veine rénale
D - Séquelle de néphropathie gravidique
E - Pyélonéphrite chronique obstructive
Bonne(s) réponse(s) : E

L'existence d'une image hyperdense associée à un cône d'ombre évoque une lithiase ou une calification. C'est un argument
en faveur d'une pyélonéphrite chronique obstructive.

L'hypertension peut éventuellement être rapportée au petit rein. Pour le prouver, la meilleure méthode paraît
être :
A - Dosage séparé d'activité rénine
B - Biopsie rénale bilatérale
C - Artériographie
D - Etude de l'élimination rénale des métabolites et des prostaglandines
E - Cathétérisme des uretères et calcul de la clairance de l'inuline de chaque rein
Bonne(s) réponse(s) : A

Comme dans l'hypertension artérielle rénovasculaire, le dosage séparé de l'ARP permet d'évaluer la responsabilité du rein
atteint dans la génèse de cette HTA.

Le taux de rénine périphérique est trouvé à 5 fois la valeur normale sans médicament. Quel(s) médicament(s),
parmi les suivants, peu(ven)t l'abaisser ?
A - Hypotenseur d'action centrale (clonidine, Catapressan®)
B - Inhibiteur de l'enzyme de conversion
C - Antialdostérone
D - Bêtabloqueur
E - Vasodilatateur (dihydralazine, Népressol®)
Bonne(s) réponse(s) : A B

Attention, il ne faut pas confondre activité rénine plasmatique et taux d'angiotensine II !


On rappelle que les diurétiques augmentent tous l'ARP, que le système rénine-angiotensine est sous la dépendance du
système sympathique bêta au niveau rénal, il est donc déprimé par les bêta bloquants et par les antihypertenseurs d'action
centrale, que les vasodilatateurs stimulent le système rénine-angiotensine et qu'en inhibant la production d'angiotensine II, les
inhibiteurs de l'enzyme de conversion stimulent de façon très importante l'ARP.

L'hypokaliémie constatée à 3,2 mmol peut être corrigée dans ce cas, au moins en partie, par :
A - Diurétique thiazidique
B - Inhibiteur de l'enzyme de conversion
C - Antialdostérone
D - Bêtabloqueur
E - Vasodilatateur (dihydralazine, Népressol®)
Bonne(s) réponse(s) : B C

QUESTION ANNULEE.
Ces deux médicaments ont une action antialdostérone directe ou indirecte susceptible de favoriser l'épargne potassique.

Un homme de 41 ans, employé de bureau sans antécédent pathologique particulier, consulte son médecin traitant pour une
asthénie croissante depuis plusieurs semaines. L'examen clinique est normal, en dehors d'une hypertension artérielle à
180/100 mmHg aux quatre membres. Le fond d'oeil est normal. L'électrocardiogramme montre une hypertrophie ventriculaire
gauche discrète et une onde U. La kaliémie est à 3 mmol/l. La clairance de la créatinine est à 100 ml/mm.

Chez cet hypertendu ayant une hypokaliémie, l'interrogatoire doit éliminer la prise de produits hypokaliémants.
Ainsi vous recherchez la prise de :
A - Laxatifs
B - Diurétiques thiazidiques
C - Diurétiques distaux
D - Inhibiteurs de l'enzyme de conversion
E - Réglisse
Bonne(s) réponse(s) : A B E

Respectivement responsable de perte potassique digestive et rénale. Seul l'item E entraîne également une HTA associée
réalisant un tableau d'hyperminéralcorticisme primaire.
C et D induisent plutôt des hyperkaliémies.
287
Exclusivement sur DOC - DZ : www.doc-dz.com NADJI 85
RESIDANAT EN POCHE TOME II
Cas Clinique en QCM

Le(s) diagnostic(s) qu'il convient de discuter en priorité est(sont) :


A - Hyperaldostéronisme primaire
B - Hypertension rénovasculaire
C - Phéochromocytome
D - Tumeur sécrétant de la rénine
E - Coarctation aortique
Bonne(s) réponse(s) : A D

Une HTA avec hypokaliémie évoque de principe un hyperminéralocorticisme primaire ou secondaire. L'absence de toute signe
vasculaire associé permet dans un premier temps d'écarter l'item B.

L'examen biologique permenant de différencier hyperaldostéronisme primaire et secondaire est :


A - Activité rénine plasmatique
B - Ionogramme sanguin
C - Kaliurèse
D - Bicarbonatémie
E - Aucun de ces examens
Bonne(s) réponse(s) : A

Sans commentaire, connaissance pure.

En cas d'hyperaldostéronisme primaire, l'activité rénine plasmatique est :


A - Normale
B - Basse
C - Elevée
D - Stimulable
E - Non stimulable
Bonne(s) réponse(s) : B D

B - Evident, l'ARP est faiblement stimulable, et des différences existent entre les différents types d'hyperaldostéronisme
primaire.

Le médicament le plus logique pour traiter l'hypertension artérielle par hyperaldostéronisme primaire est :
A - Thiazidique
B - Spironolactone
C - Bêta-bloquant
D - Dihydralazine
E - Furosémide
Bonne(s) réponse(s) : B

Evidence, tout en sachant qu'il faut avoir recours à des doses supérieures aux doses courantes.

Un patient de 45 ans est atteint d'une polykystose rénale. On palpe deux gros reins, bosselés, dont l'aspect échographique est
caractéristique. Il a atteint le stade de l'insuffisance rénale terminale et un traitement par hémodialyse périodique est
nécessaire.
L'hémogramme est alors le suivant : hématies 2.000.000, hématocrite 18 %, hémoglobine 6 g/100 ml, leucocytes 6 000 dont
neutrophiles 60 %, lymphocytes 30 %, monocytes 10 %, plaquettes 250.000/ mm3, réticulocytes 4%.

Cette anémie est :


A - Normochrome macrocytaire non régénérative
B - Normochrome macrocytaire régénérative
C - Normochrome normocytaire non régénérative
D - Normochrome normocytaire régénérative
E - Normochrome microcytaire régénérative
Bonne(s) réponse(s) : C

Normocytaire car 18%/2.000.000 = 90 cm3 et arégénative car 4%/2.000.000 = 80 000 (< 100.000).

288
Exclusivement sur DOC - DZ : www.doc-dz.com NADJI 85
RESIDANAT EN POCHE TOME II
Cas Clinique en QCM

Parmi les mécanismes suivants, lequel vous paraît prépondérant pour expliquer une telle anémie chez ce
patient ?
A - Hémodilution
B - Hématuries répétées (polykystose)
C - Hémolyse extracorpusculaire
D - Insuffisance de production érythroblastique
E - Hypersplénisme
Bonne(s) réponse(s) : D

On rappelle que l'anémie de l'insuffisance rénale est multifactorielle et classiquement normocytaire. Elle est arégénérative ou
hyporégénérative à cause du déficit en érythropoïétine.
Cependant le cas présenté est curieux et doit éveiller la méfiance et la recherche d'une autre cause d'anémie, car la
polykystose s'accompagne habituellement d'une polyglobulie, même au stade terminal, ou tout au moins très souvent de
l'absence d'anémie. Par ailleurs, même pour une insuffisance rénale chronique "habituelle", le chiffre est trop bas. On devra
en particulier rechercher un saignement digestif fréquent chez les urémiques.
A - Le chiffre est trop abaissé pour un signe d'inflation volémique extracellulaire bruyant (OAP...).
B - Il est exceptionnel qu'une hématurie entraîne une anémie.
C - C'est un des mécanismes de l'anémie des urémiques, mais il vient derrière la carence en érythropoïétine.
E - Il n'y a pas de splénomégalie dans la polykystose et il n'en est pas fait mention dans l'énoncé.

Après plusieurs mois d'hémodialyse, l'aggravation et la répétition des épisodes hématuriques fait proposer une
binéphrectomie. Le bilan d'hémostase pré-opératoire peut montrer en rapport avec l'insuffisance rénale :
A - Allongement du temps de saignement
B - Allongement du temps de céphaline activée
C - Allongement du temps de Quick
D - Hypofibrinogénémie
E - Raccourcissement du temps de Iyse des euglobulines
Bonne(s) réponse(s) : A

Les troubles de l'hémostase des urémiques sont essentiellement liés à une atteinte de l'hémostase primaire.

Le patient a été traité par binéphrectomie et transplantation rénale. Plusieurs mois plus tard, le transplant est
fonctionnel. L'hématocrite devrait se stabiliser, en l'absence de toute transfusion :
A - Entre 10 et 20 %
B - Entre 20 et 30 %
C - Entre 30 et 40 %
D - Entre 40 et 50 %
E - Au dessus de 50 %
Bonne(s) réponse(s) : D

Retour à la normale. En pratique, il existe très souvent une discrète inflation hydrosodée due à la corticothérapie de la greffe,
un état inflammatoire dû aux réactions de rejet ou aux infections intercurrentes, une carence nutritionnelle due à
l'accumulation de ces facteurs (hospitalisation fréquente, régime désodé pauvre en sucre, donc insipide...etc) et l'hématocrite
est souvent de fait entre 35 et 45%...

Mademoiselle Christine D., 18 ans, est suivie depuis l'enfance pour protéinurie et hématurie : une biopsie rénale a été
pratiquée 5 ans auparavant montrant des lésions dites minimes, quelques micro-dépôts hyalins et aucun dépôt significatif en
immunofluorescence. Depuis deux ans, existe une protéinurie plus importante entre 3 et 5 g/24 heures et une élévation
tensionnelle. Au mois de février, l'état clinique est inchangé, la pression artérielle trop élevée pour l'âge ne donne pas lieu à
un traitement, la créatinine plasmatique est trouvée à 142 micromoles pour mille et les protéines sanguines à 58 g pour mille.
Elle ne s'est pas présentée à la consultation d'avant l'été. Elle a trouvé un petit emploi de 4 heures par jour, un T.U.C. Depuis
un mois, elle est fatiguée, bouffie, se plaint de céphalées, elle vomit depuis une semaine et présente une épistaxis depuis
deux jours. On note encore qu'il y a un mois elle a présenté un syndrome grippal négligé avec des urines sombres. Enfin, il est
rappelé qu'il y a un an on avait évoqué une insuffisance thyroïdienne devant une hyperplasie thyroïdienne diffuse avec des
taux d'hormonémie normaux. Elle est normalement règlée depuis l'âge de 15 ans, sa tension est à 200/120 mmHg et vous
notez encore une odeur particulière de l'haleine. Les extrêmités sont trémulantes et les réflexes extrêmement vifs. Elle dit
uriner très peu depuis 48 heures.

Votre hypothèse diagnostique est :


A - Insuffisance thyroïdienne aiguë
B - Diabète sucré décompensé méconnu
C - OEdèmes idiopathiques
D - Hépatite au début
E - Décompensation aiguë d'une insuffisance rénale chronique
Bonne(s) réponse(s) : E

L'histoire est très évocatrice : notion d'une néphropathie, évoluant progressivement vers l'insuffisance rénale chronique,
découverte dans l'enfance (existence de dépôts de hyalinose cours d'un syndrome néphrotique impur. Notion d'une évolutivité
de cette néphropathie (Cr = 142 + protéinurie). Enfin apparition de signe d'inflation volémique intra et extracellulaire au
décours d'un épisode évolutif récent. Pour anecdote, citons l'odeur ammoniacale de l'haleine. Les autres items ne décrivent
pas le tableau.
289
Exclusivement sur DOC - DZ : www.doc-dz.com NADJI 85
RESIDANAT EN POCHE TOME II
Cas Clinique en QCM

La découverte d'un potassium à 7,5 mmol/l s'interprète dans ce contexte comme dû à :


A - Une insuffisance rénale avancée
B - Une insuffisance thyroïdienne
C - Une insuffisance surrénale
D - Une intoxication à la glycirizhine (réglisse-antésite)
E - La prise de laxatifs
Bonne(s) réponse(s) : A

Compte tenu du contexte... notons l'oligoanurie qui accompagne l'étape ultime de cette néphropathie. Cette oligurie est à
l'origine de la rétention hydrique et hydrosodée de cette patiente, ainsi que de son hyperkaliémie. Les items B, D et E
n'entraînent pas d'hyperkaliémie, voire même une hypokaliémie (D et E). L'item C entraîne une hyperkaliémie, mais on ne
peut évoquer ce diagnostic devant ce tableau de rétention hydrosodée manifeste...

La découverte d'une natrémie à 119 mmol pour mille dans ce contexte s'interprète comme due à :
A - Une insuffisance surrénale
B - Une insuffisance thyroïdienne
C - La prise de diurétique
D - Un excès de boisson
E - Une restriction sodée trop sévère
Bonne(s) réponse(s) : D

De même que pour les autres réponses les items A et B sont éliminés. Il n'y a pas de notion de prise de diurétique et la
patiente est anurique... Une restriction sodée ne peut suffire à elle seule à induire une telle hyponatrémie, et ne s'accompagne
pas de signes de rétention hydrosodée précisément. Il y a hyperatrémie car il y a excès d'apport hydrique chez une patiente
dont la perte rénale d'eau est quasi nulle.

La correction de l'hyponatrémie dans ce contexte nécessite :


A - La prise d'antialdostérone
B - Un apport de soluté salé à 9 g pour mille intraveineux (deux litres)
C - Un apport de soluté bicarbonaté a 14 g pour mille intraveineux (deux litres)
D - Des coticoïdes en flash intraveineux, 1 g de prednisone
E - Une ultrafiltration par une technique d'hémodialyse
Bonne(s) réponse(s) : E

Le traitement de l'insuffisance rénale chronique arrivée au stade ultime de son évolution. Notons que l'administration
d'antialdostérone est formellement contre-indiquée en raison du risque d'hyperkaliémie... qui est de toute façon déjà
constituée. L'apport de soluté salé isotonique ne ferait qu'accroître l'hypervolémie extracellulaire de cette patiente. Le soluté
bicarbonaté isotonique aurait le même effet. L'administration de prednisone n'a jamais constitué le traitement d'une
hyponatrémie.

La correction de l'hyperkaliémie dans ce contexte peut reposer sur :


A - Perfusion de 2 litres de soluté de bicarbonate de sodium à 42 g pour mille
B - Prise de Kayexalate® par la bouche ou en lavement
C - Hémodialyse
D - Injection intramusculaire d'antialdostérone (Soludactone®)
E - Prescription d'un diurétique thiazide à la dose habituelle
Bonne(s) réponse(s) : B C

B - Peut permettre une rémission transitoire le temps de trouver une place dans une réanimation équipée d'un rein artificiel.
A - Peut effectivement diminuer de façon transitoire la kaliémie mais au prix d'une inflation supplémentaire du secteur
extracellulaire. Remarquons d'ailleurs que la prescription de deux litres de bicarbonate de sodium semi-molaire correspond à
une augmentation de 6 litres du secteur extracellulaire chez cette patiente anurique, sans compter le transfert du secteur
intracellulaire vers le secteur extracellulaire.
D - Est contre-indiqué.
E - Inefficace.

On trouve également une hypocalcémie à 1,8 mmol pour mille. Elle s'explique, au moins en partie, par :
A - L'insuffisance rénale
B - Une insuffisance parathyroïdienne associée
C - L'hémodilution
D - Le jeune âge
E - L'hypoprotéinémie
Bonne(s) réponse(s) : A E

On rappelle qu'au cours de l'insuffisance rénale il existe une altération de la 1 alpha hydroxylation de la vitamine D,
responsable d'une négativation du bilan calcique et d'un hyperparathyroïdisme secondaire.

290
Exclusivement sur DOC - DZ : www.doc-dz.com NADJI 85
RESIDANAT EN POCHE TOME II
Cas Clinique en QCM

La malade restant subfébrile, il est décidé de donner un traitement antibiotique par sécurité. Dès lors vous
choisissez :
A - Gentalline® (gentamicine) 80 mg en injection intraveineuse lente toutes les six heures
B - Ampicilline ou amoxilline 1,5 g toutes les deux heures
C - Propionate d'érythromycine (Propiocine®) 500 mg toutes les six heures
D - Vancomycine (Vancocine®) 2 gr toutes les 24 heures per os
E - Tétracycline 1 g toutes les 4 heures
Bonne(s) réponse(s) : C

A - Posologie non adaptée et de toute façon excessive même chez le sujet normal.
B - Idem.
C - Option raisonnable tant dans le choix de l'antibiotique que dans la posologie qui n'est pas à modifier car le métabolisme
est hépatique.
D - Posologie à adapter (une fois par semaine ou par quinzaine chez l'insuffisant rénal au stade de la dialyse), s'administrant
exclusivement par voie parentérale et ne devant être administrée que s'il y a forte suspicion d'infection staphylococcique.
E - Posologie excessive même chez le sujet sain.

Un homme de 75 ans consulte pour des troubles mictionnels à type de dysurie et de pollakiurie. Il dit avoir des fuites d'urines
nocturnes depuis 1 an. On note dans les antécédents un infarctus du myocarde datant de moins de 3 mois. L'examen clinique
met en évidence une masse sus-pubienne sensible et mate. La créatininémie est à 600 mmoles/l.

Les fuites d'urine que ce malade présente signifient :


A - Mictions par regorgement
B - Rétention vesicale chronique incomplète sur vessie distendue
C - Mictions impérieuses
D - Déficit sphinctérien
E - Aucune de ces réponses
Bonne(s) réponse(s) : A B

On relève dans l'observation l'âge, le globe, l'association d'incontinence et de troubles mictionnels (dysurie, pollakiurie).

Le toucher rectal pourrait permettre d'identifier :


A - Prostate dure et irrégulière
B - Petit adénome
C - Gros adénome
D - Sténose uréthrale
E - Lobe médian prostatique
Bonne(s) réponse(s) : A B C

Le TR permet l'exploration de la prostate et non de l'urètre.


Le lobe médian n'est perçu que s'il est très hypertrophié. Enfin adénome et cancer sont les deux hypothèses diagnostiques à
envisager. Rappelons que le développement postérieur apprécié par le TR n'est pas forcément corrélé à l'importance des
troubles mictionnels.

Quelle mesure thérapeutique doit être prise en urgence quel que soit le diagnostic évoqué ?
A - Cathéter sus pubien
B - OEstrogénothérapie
C - Cystostomie chirurgicale
D - Résection endo-uréthrale
E - Uréthrotomie interne
Bonne(s) réponse(s) : A

Il faut lever l'obstacle et soulager le patient. Si l'insuffisance rénale est en rapport (anurie par obstacle sur rétention négligée),
il faudra surveiller la survenue d'un syndrome de levée d'obstacle.

Parmi les examens morphologiques suivants, lequel vous paraît le plus adapté en vue du traitement ?
A - Echographie rein vessie prostate
B - Urographie intraveineuse avec clichés permictionnels
C - Uréthrographie rétrograde et mictionnelle
D - Cystographie par ponction sus-pubienne
E - Angiographie numérisée vésicoprostatique
Bonne(s) réponse(s) : A

A - Permet le diagnostic de l'atteinte rénale (dilatation ou non des cavités pyélocalicielles, petits reins ou gros reins),
l'exploration de la vessie et de la prostate.
B - Ne montrera rien car le patient est en insuffisance rénale importante. Il faudra réaliser des clichés très tardifs pour
entr'apercevoir un néphrogramme de mauvaise qualité.
C D - N'explore pas le rein.
E - N'existe pas.
291
Exclusivement sur DOC - DZ : www.doc-dz.com NADJI 85
RESIDANAT EN POCHE TOME II
Cas Clinique en QCM

Parmi les traitements suivants lequel peut-on envisager ?


A - Chimiothérapie
B - OEstrogènes
C - Pulpectomie sous anesthésie locale
D - Radiothérapie
E - Résection endoscopique de la prostate
Bonne(s) réponse(s) :

QUESTION ANNULEE.
On ne pouvait y répondre ne connaissant pas la nature de l'obstacle. En cas d'adénome on propose une résection ; en cas de
cancer et en fonction de l'extension on discute une résection ou surtout une hormonothérapie (B ou C).

Un patient âgé de 35 ans a vu progressivement augmenter le volume d'une bourse. Il n'a pas d'antécédent particulier.
L'examen révèle une absence de transillumination. La peau de la bourse est intacte. Sur la masse observée, I'épididyme se
repère aisément.

Parmi les attitudes suivantes, vous choisissez de :


A - Envisager avec le patient l'éventualité d'une castration unilatérale
B - Le mettre à un traitement antibiotique
C - Faire un dosage d'HCG et d'alphafoetoprotéine
D - Proposer une biopsie
E - Ponctionner la tumeur
Bonne(s) réponse(s) : A C

Il s'agit d'une tumeur testiculaire. On doit donc pratiquer un examen anatomopathologique de I'orchidectomie. Il ne s'agit pas
d'une biopsie mais d'une orchidectomie complète avec examen de toute la pièce.

En admettant que la castration ait été décidée, l'examen rapide de la pièce montre qu'il s'agit d'un
dysembryome malin non mature. Dans le bilan complémentaire à faire, s'il n'a pas été fait avant l'intervention,
l'un ou plusieurs de ces examens est(sont) nécessaire(s). Lequel ou lesquels ?
A - Radiographie thoracique
B - Scanner abdominal
C - Biopsie médullaire
D - Dosage d'HCG et d'alphafoetoprotéine
E - Dosage de la FSH et de la testostérone
Bonne(s) réponse(s) : A B D

A B - Permettent de rechercher une extension aux ganglions lomboaortiques ou au médiastin et aux poumons.
D - Si les résultats sont positifs et même si l'histologie ne trouve pas de lésions, on doit considérer la tumeur comme un
chroriocarcinome, et envisager une chimiothérapie d'emblée, forte même si le bilan d'extension est négatif.

Les examens complémentaires ont révélé la présence d'une masse probablement ganglionnaire pré-aortique
droite. Parmi les options suivantes, laquelle retiendriez-vous immédiatement ?
A - Surveiller
B - Faire un curage des ganglions périaortiques bilatéral
C - Faire une chimiothérapie
D - Faire une radiothérapie des chaines iliaques et lombo-aortiques, du creux sus-claviculaire gauche et du
médiastin
E - Entreprendre une hormonothérapie par un agoniste de la LHRH
Bonne(s) réponse(s) : B

Ce traitement sera suivi d'une chimio et d'une radiothérapie, si l'histologie confirme le diagnostic d'extension.

Grâce au traitement, une rémission complète a été obtenue et le patient a repris son travail. Outre l'examen
clinique, vous vous baserez pour surveiller ce patient sur :
A - Radiographie thoracique
B - Scintigraphie osseuse
C - Urographie intraveineuse
D - Dosage d'HCG et d'alphafoetoprotéine
E - Dosage de FSH
Bonne(s) réponse(s) : A D

Sans commentaire.

292
Exclusivement sur DOC - DZ : www.doc-dz.com NADJI 85
RESIDANAT EN POCHE TOME II
Cas Clinique en QCM

Si l'examen anatomo-pathologique avait montré un séminome pur avec adénopathies lombo-aortiques, à


quelle(s) technique(s) thérapeutique(s) aurez-vous fait appel ?
A - Curage ganglionnaire iliaque et lombo-aortique
B - Polychimiothérapie
C - Irradiation de l'abdomen in toto
D - Irradiation des chaines lombo-aortiques et du creux sus-claviculaire
E - Hormonothérapie
Bonne(s) réponse(s) : A D

Eventuellement complété par une chimiothérapie légère type chlorambucil seul.

Un sujet de 48 ans, chauffeur routier de transports internationaux, se présente avec une insuffisance rénale importante :
créatinine plasmatique à 350 micromol/l. Sur un cliché sans préparation ancien, les reins sont gros et polylobés. Son père et
sa grand-mère paternelle sont morts, l'un d'insuffisance rénale, l'autre d'accident vasculaire cérébral et d'urémie. Le diagnostic
de polykystose rénale est porté. On note par ailleurs que la P.A. est élevée à 182/124 mm de mercure. Le pouls est à 58/mn,
le sujet prend par jour 600 mg de Sectral® (acébutolol). Il n'a aucune prescription diététique et n'a pas d'antécédent
pathologique notable. Il souhaite pouvoir poursuivre le plus longtemps possible son activité professionnelle.

Pour traiter l'hypertension artérielle, vous choisissez de :


A - Ne pas augmenter la thérapeutique antihypertensive pour maintenir une bonne fonction rénale
B - Améliorer le chiffre tensionnel car le risque de rupture vasculaire encéphalique est important
C - Remplacer le bêtabloqueur par un diurétique thiazidique
D - Remplacer le bêtabloqueur choisi par un bêtabloqueur à élimination rénale
E - Ajouter un autre antihypertenseur
Bonne(s) réponse(s) : A E

Il est fondamental de bien contrôler l'HTA pour éviter la survenue d'accidents vasculaires cérébraux. Les thiazidiques sont
inefficaces en cas d'insuffisance rénale.
Les bêta bloquants sont une bonne indication dans ce cas mais ils sont manifestement insuffisants : en effet le pouls est à
58/mn, la posologie est donc suffisante. Il faut donc rajouter un second traitement.

Devant l'hypertension, si vous choisissez de renforcer le traitement, il est contre-indiqué de prescrire dans ce
cas :
A - Anti-aldostérone (200 mg par jour)
B - Lasilix® (furosémide) (40 mg par jour)
C - Régime désodé (2 à 4 g de ClNa par jour)
D - Catapressan® (clonidine) (4 comprimés à 0,15 mg par jour)
E - Népressol® (dihydralazine) (3 comprimés à 75 mg par jour)
Bonne(s) réponse(s) : A B C

A - Est à éviter du fait de l'insuffisance rénale et du risque d'hyperkaliémie.


Les autres items sont possibles, mais :
- le régime désodé ne doit être prescrit qu'en fonction de la natriurèse, car il existe fréquemment une fuite sodée urinaire dans
la polykystose rénale
- le Lasilix® risque d'aggraver la fuite urinaire de sodium et de favoriser l'apparition d'un facteur fonctionnel surajouté. Le
diurétique, même s'il n'est pas formellement contre-indiqué, n'est à l'évidence pas une bonne indication dans ce cas.

Le taux d'urée sanguine paraît trop élevé par rapport à la créatinine plasmatique : vous envisagez les mesures
thérapeutiques suivantes. Dans la situation décrite il serait contre-indiqué de prescrire :
A - Perfusion quotidienne sur trois heures pendant trois semaines de 300 ml de sérum glucosé hypertonique
B - Régime glucidolipidique exclusif définitif
C - Apport protéique de 0,8 à 1 g/kg/jour
D - Restriction de boissons
E - Suppression des bêtabloquants
Bonne(s) réponse(s) : A B D E

Si le taux d'urée sanguine est trop élevé par rapport à la créatininémie, cela signifie qu'il existe un catabolisme azoté. Il faut
donc prescrire un régime normoprotidique. Les boissons doivent être adaptées à la sensation de soif, sans restriction abusive.
Les bêta-bloquants ne sont pas contre-indiqués.

293
Exclusivement sur DOC - DZ : www.doc-dz.com NADJI 85
RESIDANAT EN POCHE TOME II
Cas Clinique en QCM

A la question : existe-t-il un risque d'atteinte de mes enfants par la maladie


et que faire ? Vous répondez :
A - Il n'y a aucun risque de transmission de la maladie
B - Le risque existe et est important (transmission dominante)
C - Dépistage précoce de l'atteinte rénale par une artériographie annuelle dès l'âge de 2 ans
D - Ponction biopsie hépatique et rénale
E - Surveillance par échographie rénale
Bonne(s) réponse(s) : B E

La transmission est autosomique dominante. L'échographie est un examen fiable et reproductible. Elle permet de
diagnostiquer les cas asymptomatiques et permet de prendre les mesures pour prévenir l'HTA, les lithiases et les infections.

Dans le but de prévenir les complications ostéo-articulaires de l'insuffisance rénale, la ou les mesure(s)
suivante(s) vous paraît (paraissent) adéquate(s) :
A - Régime appauvri en phosphore
B - En cas d'hyperphosphorémie, prescription de gels d'alumine
C - Supplémentation calcique per os
D - Correction de l'hyperkaliémie
E - Prescription de dérives actifs de la vitamine D
Bonne(s) réponse(s) : A B C E

Il faut corriger l'hyperphosphorémie et l'hypocalcémie, responsables de l'ostéodystrophie rénale.


D - Aucun rapport.

Dans ce contexte, la ou les affirmation(s) suivante(s) est ou sont plausible(s) :


A - Un traitement de suppléance (hémodialyse) est à envisager dans les cinq ans
B - Il faut conduire le sujet à envisager une reconversion professionnelle
C - Il surveillance biologique doit comporter en priorité celle de la créatininémie
D - L'insuffisance rénale de ce type d'affection n'étant pas évolutive, aucune surveillance n'est nécessaire
E - La protection du capital veineux est essentielle
Bonne(s) réponse(s) : A B C E

A - L'insuffisance rénale dans l'évolution de la PKR est d'apparition lente et progressive, dans un délai de 10 à 20 ans.
Cependant, le déficit rénal est assez avancé dans ce cas. La dialyse est souvent nécessaire aux alentours de 50 ans.
B - En effet, la profession de chauffeur routier est difficilement compatible avec une hémodialyse.
C - Evident.
E - En prévision de l'hémodialyse.

Un sujet de 64 ans est adressé à la consultation pour une protéinurie de découverte récente : 2,6 g/24 h sans hématurie
microscopique ni bactériurie. Il existe une hypertrophie prostatique à l'examen clinique non compliquée.
Il n'y a pas de symptômes cliniques à l'interrogatoire, le sujet est obèse, sa glycémie à jeun est normale. L'échographie rénale
montre une lithiase pyélique gauche sans dilatation en amont, il n'y a pas d'anomalie de la voie excrétrice ou de la
morphologie rénale, le sujet est traité depuis trois ans pour hypertension modérée par association Aldactone® + diurétique
thiazique. Ce jour, à l'examen, la pression artérielle est à 158/94 mm de mercure. Le taux de protéines sanguines est à 81g/l,
la VS est élevée, la créatinine à 178 micromol/l.

La protéinurie dans le tableau fourni est compatible avec le diagnostic suivant :


A - Syndrome néphrotique pur (néphrose lipoïdique)
B - Hypertension artérielle malignisée
C - Glomérulosclérose diabétique
D - Toxicité des diurétiques thiazidiques
E - Trouble de perméabilité glomérulaire
Bonne(s) réponse(s) : E

Par définition, un syndrome néphrotique comporte une protéinurie > 3 g/24h. On ne peut donc cocher A.
Aucun argument pour une HTA maligne dans cette observation.
La glycémie à jeun est normale, ce qui élimine un rein diabétique qui survient lors de diabètes anciens déséquilibrés.
La toxicité rénale des thiazides ne donne pas de protéinurie.

294
Exclusivement sur DOC - DZ : www.doc-dz.com NADJI 85
RESIDANAT EN POCHE TOME II
Cas Clinique en QCM

Le tableau évoque l'(les) hypothèse(s) suivante(s) :


A - Complication rénale de la lithiase
B - Insuffisance rénale par sténose de l'artère rénale gauche
C - Protéinurie due à une hémoconcentration
D - Protéinurie révélant un myélome
E - Protéinurie due à une oxalose
Bonne(s) réponse(s) : D

Il ne s'agit pas de complications rénales de la lithiase, car il n'y a pas de retentissement urographique de la lithiase sur le rein
sus-jacent.
Il n'y a pas de leucocyturie apparemment.
L'existence d'une hyperprotidémie, d'une VS accélérée chez un homme de 64 ans, doit faire évoquer un myélome dans ce
contexte, d'autant qu'on ne nous précise pas la nature de la protéinurie.

Parmi les examens suivants, quel est ou quels sont celui ou ceux qui vous paraît(ssent) utile(s) au diagnostic
étiologique de la protéinurie ?
A - Cytologie en contraste de phase
B - Dosage de l'hémoglobine glycosylée
C - Immuno-électrophorèse des protéines plasmatiques et urinaires
D - Echographie hépatique
E - Myélogramme
Bonne(s) réponse(s) : C E

C - Permet la caractérisation du pic monoclonal et sa quantification.


E - Pose le diagnostic de myélome en montrant plus de 15 % de plasmocytes dystrophiques.

Toujours dans ce contexte, et avec les précisions que vous ont apporté les examens choisis, vous discutez la
réalisation d'une urographie. Quel est l'argument parmi les suivants qui est déterminant ?
A - Oui, une urographie pour préciser le retentissement de la lithiase
B - Non, l'urographie est un examen trop coûteux
C - Oui, une urographie pour préciser le retentissement de l'hypertrophie prostatique
D - Oui, une urographie pour objectiver la néphro-angiosclérose
E - Non, car elle risque de déclencher une insuffisance rénale aiguë
Bonne(s) réponse(s) : E

En effet, la déshydratation entraînée par l'UIV risque de faire apparaître une insuffisance rénale aiguë par précipitation
intratubulaire de chaînes légères.

Dans ce contexte, la ou les proposition(s) suivante(s) sont recevable(s) :


A - Une hypercalcémie est possible malgré l'insuffisance rénale
B - Une hypercalcémie est certaine due à l'insuffisance rénale modérée
C - Une hyperkaliémie est possible due à l'insuffisance rénale et à l'Aldactone®
D - L'insuffisance rénale est d'origine mixte fonctionnelle et organique
E - L'hyperprotéinémie est due exclusivement à la prise de diurétiques
Bonne(s) réponse(s) : A C D

A - Une hypercalcémie est fréquente lors du myélome.


B - L'insuffisance rénale en elle-même entraîne plutôt une hypocalcémie.
C - Evident.
D - Le facteur fonctionnel pouvant résulter de l'hypercalcémie, de la prise de diurétiques. Après correction de ces facteurs seul
restera le facteur organique.
E - Bien que l'hémoconcentration due à la prise de diurétiques joue dans l'évaluation de la protidémie, il existe par ailleurs une
hyperimmunoglobulinémie monoclonale liée au myélome.

295
Exclusivement sur DOC - DZ : www.doc-dz.com NADJI 85
RESIDANAT EN POCHE TOME II
Cas Clinique en QCM
Un homme de 40 ans est hospitalisé pour insuffisance rénale aiguë. Cet homme jusque là en bonne santé a vu apparaître dix
jours plus tôt :
- une hématurie totale
- une oligurie
- de discrets oedèmes des membres inférieurs.
Le médecin traitant, consulté, lui trouve une tension artérielle à 160/100 mm de Hg. Un bilan biologique est réalisé qui
objective :
- NFS : normale
- VS : 30 mm à la première heure
- urée : 10 mmol/l
- créatininémie : 200 micromol/l
Le patient est alors hospitalisé. A l'entrée :
- aucune prise médicamenteuse récente
- discrets oedèmes indolores
- T.A. : 170/110
- F.O. : normal
- T.R. : normal
- fosses lombaires : normales
- auscultation pulmonaire : crépitants deux bases.
Dans le plasma : Dans les urines = diurèse 1000 cc/24h
- Urée . : 20 mmol/l - Protéinurie : 3 g/l
- Créat. : 300 micromoI/l - Urée : 200 mmol/l
- Natrémie : 132 mEq/l - Na : 20 mEq/l
- Kaliémie : 5,7 mEq/l -K : 40 mEq/l
- Cl : 95 mEq/l - Cl : 20 mEq/l
- Hématurie : 500 000 H/ml
L'échographie rénale ne met pas en évidence de distension, les reins sont de taille normale.

Quelle hypothèse diagnostique retenez-vous comme probable chez ce patient ?


A - Nécrose tubulaire aiguë
B - Néphrite interstitielle aiguë allergique
C - Glomérulonéphrite aiguë
D - Insuffisance rénale aiguë fonctionnelle
E - Syndrome néphrotique
Bonne(s) réponse(s) : C

Il s'agit d'un syndrome néphritique aiguë typique associant des oedèmes, une HTA, une protéinurie avec hématurie, une
oligurie et une insuffisance rénale aiguë.

Parmi le ou les éléments suivants, lequel(lesquels) est(sont) en faveur de votre hypothèse diagnostique ?
A - HTA récente
B - Syndrome oedémateux
C - Hyperkaliémie
D - Protéinurie
E - Hématurie microscopique
Bonne(s) réponse(s) : A B D E

Tous ces éléments font partie du syndrome néphritique aiguë. L'hypercalcémie ne témoigne que de l'insuffisance rénale.

Quel ou quels examen(s) envisagez-vous de réaliser rapidement chez ce patient ?


A - Radiographie du thorax
B - Complément sérique
C - Urétéropyélographie ascendante
D - U.I.V.
E - Artériographie rénale
Bonne(s) réponse(s) : A B

A - La radio de thorax s'impose car il existe manifestement une surcharge pulmonaire.


B - Le complément sérique est abaissé et constitue un bon facteur d'évolution, son taux devant remonter rapidement en cas
d'évolution favorable.
C D E - N'ont aucun intérêt puisqu'il s'agit d'une atteinte glomérulaire.

296
Exclusivement sur DOC - DZ : www.doc-dz.com NADJI 85
RESIDANAT EN POCHE TOME II
Cas Clinique en QCM

Quelle(s) anomalie(s) associée(s) à l'insuffisance rénale aiguë paraît(ssent) présente(s) chez ce patient ?
A - Déshydratation intracellulaire
B - Hyperhydratation globale
C - Hyperkaliémie
D - Hypertension artérielle maligne
E - Oedème pulmonaire
Bonne(s) réponse(s) : B C E

Il existe une hyperhydratation intracellulaire dont témoigne l'hyponatrémie, et une hyperhydratation extracellulaire dont
témoignent les oedèmes et l'HTA.
Il n'y a pas d'HTA maligne, le fond d'oeil ne montrant pas d'oedème papillaire. Enfin il existe un oedème pulmonaire dont
témoignent les crépitants aux deux bases.

Dans le cas où votre hypothèse diagnostique principale serait vérifiée, quel(s) élément(s) parmi les suivants
vous parai(ssen)t être déterminant(s) pour le pronostic ?
A - Aspect histologique rénal
B - Absence d'anurie
C - HTA modérée
D - Oedèmes discrets
E - Créatininémie 40 mg/l
Bonne(s) réponse(s) : A B E

Le type histologique est très important pour le pronostic et la PBR doit être faite lors d'une G.N.A de l'adulte. En effet, elle peut
montrer des signes de glomérulonéphrite subaigüe maligne avec croissants diffus extracapillaires. Dans ce cas, il existe
fréquemment une anurie et l'insuffisance rénale s'aggrave rapidement. Enfin, l'item C est discutable, car lors de ces
glomérulonéphrites malignes, l'HTA est souvent modérée par rapport à la G.N.A banale. On peut donc discuter de répondre A,
B, C ou E.

Dans l'hypothèse où existerait une prolifération extra capillaire diffuse dans la chambre de Bowman des
glomérules de ce patient, quel(s) traitement(s) est(sont) envisageable(s) ?
A - Corticothérapie à fortes doses
B - Plasma frais congelé
C - Immunosuppresseurs
D - Abstention thérapeutique
E - Echanges plasmatiques
Bonne(s) réponse(s) : A C E

Dans quelques cas on a pu arrêter l'évolution vers l'insuffisance rénale terminale en associant ces 3 thérapeutiques.

Madame Hortense F. est une paysanne de 54 ans ; elle élève quelques vachesdans une ferme de Gimont dans le Gers, et
gave soigneusement ses oies tout au long de l'année pour une entreprise de foie gras. Elle est en bonne santé, ne se
souvient pas avoir été malade. Il y a 2 mois, elle est tombée d'une trappe de son grenier à foin et a été plâtrée un mois pour
une fracture malléolaire droite. Elle ressent quelques douleurs à cette cheville et au mollet droit, cette jambe est d'ailleurs
restée discrètement enflée. La patiente se plaint également de quelques douleurs lombaires vagues non systématisées.
Depuis trois semaines, elle a pris plusieurs kilos, se sent boudinée, les jambes, le visage et les mains sont très enflés, la
protéinurie est trouvée à 12 g/24 heures.

Dans un tel tableau, la ou les constatation(s) biologique(s) suivante(s) est(sont) fréquente(s) :


A - Baisse du calcium total
B - Elévation du calcium non ultrafiltrable
C - Taux de protéines totales sanguines entre 50 et 60 g/0/000
D - Taux d'albumine plasmatique au-dessous de 30 gr/0/000
E - Cholestérol abaissé
Bonne(s) réponse(s) :

ANNULEE

297
Exclusivement sur DOC - DZ : www.doc-dz.com NADJI 85
RESIDANAT EN POCHE TOME II
Cas Clinique en QCM

L'échographie rénale révèle deux reins de taille augmentée de 14 et 15 cm de diamètre vertical, de densité et
de structure homogènes, aux cavités non dilatées. Parmi les propositions suivantes, laquelle(lesquelles)
est(sont) correct(s) ?
A - Cet aspect est pathognomonique d'une amylose
B - Le diagnostic est celui de polykystose rénale
C - C'est l'aspect d'une glomérulonéphrite chronique ancienne avec insuffisance rénale
D - C'est l'aspect d'une sténose artérielle rénale bilatérale
E - C'est un aspect habituel dans un syndrome néphrotique au début
Bonne(s) réponse(s) : E

A - Les causes d'hypertrophie rénale sont l'amylose, le diabète, la polykystose rénale et l'hydronéphrose bilatérale.
B - Les reins sont homogènes, donc il ne s'agit pas d'une polykystose rénale.
CD - Les reins sont petits dans ce cas.

La pression artérielle étant basse, vous pensez trouver dans ce contexte :


A - Une activité rénine basse
B - Une volémie plasmatique élevée
C - Un taux d'aldostérone élevé
D - Une kaliémie haute
E - Une hypotension orthostatique
Bonne(s) réponse(s) : C E

Il existe dans ce cas un hyperaldostéronisme secondaire avec élevation de l'ARP.


La pression artérielle est basse du fait de l'existence d'une hypovolémie.

La biopsie rénale montre qu'il existe des dépôts extra-membraneux d'immunoglobulines. Dans ce contexte, il
est exact que :
A - L'évolution vers l'insuffisance rénale terminale se fera en trois mois
B - Il faut rechercher une maladie générale associée
C - L'évolution se fait vers 70 % de rémission spontanée
D - La corticothérapie est efficace dans 80 % des cas
E - Un traitement comportant ultrafiltration et plasmaphérèse doit être entrepris
Bonne(s) réponse(s) : B

A - L'évolution vers l'insuffisance rénale est lente et relativement rare (20% à 30% des cas).
B - Il faut rechercher une cause sous jacente : lupus, cancer profond, médicaments (sels d'or,
D-penicillamine, captopril, triméthadione...), infection (syphilis, paludisme, hépatite B...).
C - Environ 20% de rémission spontanée.
D, E il n'y a pas de traitement véritablement efficace.

Parmi les complications possibles, on retient :


A - Une nécrose papillaire rénale
B - Une nécrose corticale rénale
C - La constitution rapide d'une hypertension artérielle maligne
D - Des infarctus rénaux
E - Une thrombose veineuse cave et rénale
Bonne(s) réponse(s) : C E

A - Les causes de nécroses papillaires sont : pyélonéphrite aiguë, diabète, drépanocytose, abus d'analgésiques.
B - Complique parfois une nécrose tubulaire aiguë.
C - Le mot "rapide" suggère de ne pas cocher cet item.
En effet, l'HTA est rarement importante au cours des syndromes néphrotiques du fait de l'existence d'une hypovolémie.
D'autre part, l'HTA liée à une insuffisance rénale organique pourrait être discutée puisque la survenue est tardive par les GN
extramembraneuses.
D'un autre côté, c'est une éventualité toujours possible...
E - Par le biais des modifications plaquettaires et de la coagulation, notamment la fuite urinaire d'antithrombine III.

298
Exclusivement sur DOC - DZ : www.doc-dz.com NADJI 85
RESIDANAT EN POCHE TOME II
Cas Clinique en QCM
Un homme de 50 ans est hospitalisé en urgence pour insuffisance rénale aiguë et H.T.A. Dans ses antécédents on note la
notion d'une H.T.A. traitée depuis 10 ans par diurétiques et Aldomet®. Son père et un de ses frères sont également
hypertendus.
Dans les semaines précédant l'hospitalisation, le patient avait noté :
-une perte de poids de 5 kg environ
- une polyurie nocturne
- des céphalées matinales fréquentes et une baisse progressive de l'acuité visuelle.

A l'entrée on note :
T.A. 22/14, pas d'oedèmes, tachycardie à 100 et galop gauche :
au F.O. : hémorragies, exsudats, oedème papillorétinien.
Au plan paraclinique :
Sur la radio de thorax, cardiomégalie sans oedème pulmonaire.
Dans le sang :
Na 132 mEq/l, K 4,1 mEq/l, Cl 97 mEq/l
bicar 17 mEq/l, urée 1,30 g/l
créat 117 mg/l, Glycémie 1 g/l.

Dans les urines : hématurie + + ; prot. +.

Le diagnostic d'HTA maligne ou accélérée est porté.

L'HTA accélérée est caractérisée par :


A - T.A. systolique > 20 cm Hg
B - Le fond d'oeil décrit
C - Existence d'une protéinurie
D - Céphalées
E - Antécédents familiaux
Bonne(s) réponse(s) : B

L'HTA maligne n'est pas définie par des chiffres tensionnels mais par son retentissement oculaire (oedème papillaire)
cardiaque (insuffisance ventriculaire gauche aiguë) et rénal (insuffisance rénale).

L'HTA maligne peut compliquer :


A - L'hypoplasie rénale segmentaire
B - L'HTA rénovasculaire
C - Le syndrome de Conn
D - Le phéochromocytome
E - Une hypertension essentielle
Bonne(s) réponse(s) : A B C D E

L'HTA maligne peut compliquer toutes les HTA, essentielle ou secondaires, et peut même être révélatrice et apparaître de
novo.

Si l'activité rénine plasmatique et l'aldostéronémie étaient dosées chez ce patient on pourrait voir :
A - Une A.R.P. basse
B - Une A.R.P. élevée avec aldostérone basse
C - Une A.R.P. et une aldosteronémie toutes deux élevées
D - Une A.R.P. basse avec aldostéronémie haute
E - Aucune des propositions précédentes n'est exacte
Bonne(s) réponse(s) : C

Il existe dans ce cas une hyperactivité de l'axe rénine-angiotensine avec hyperaldostéronisme secondaire. La soif est liée à
l'hyper-angiotensinémie.

Les signes histopathologiques rénaux de la néphroangiosclérose maligne comportent :


A - Nécrose fibrinoïde des artérioles rénales
B - Une prolifération extra capillaire glomérulaire
C - Une ischémie glomérulaire
D - Une prolifération endocapillaire pure au niveau des glomérules
E - Aucune des lésions ci-dessus
Bonne(s) réponse(s) : A B C

La P.B.R.dans l'HTA maligne montre :


- une nécrose fibrinoïde
- une endartérite proliférante bouchant les artérioles intralobulaires
- des altérations glomérulaires de type ischémique.

299
Exclusivement sur DOC - DZ : www.doc-dz.com NADJI 85
RESIDANAT EN POCHE TOME II
Cas Clinique en QCM

Quel(s) traitement(s) d'urgence pouvez-vous proposer à ce patient ?


A - Binéphrectomie
B - Traitement anti-hypertenseur
C - Epuration extra-rénale
D - Spironolactone
E - Aucun des traitements ci-dessus
Bonne(s) réponse(s) : B

Il faut utiliser dans ce cas des vasodilatateurs type népressol ou diazoxide, ou des inhibiteurs de l'enzyme de conversion, en
veillant à corriger l'hypovolémie démasquée par une vasodilatation brutale.

Monsieur D. Jean est âgé de 41 ans. Son père est décédé à l'âge de 45 ans d'une hémorragie cérébrale consécutive à une
HTA sévère. Lui-même présente une HTA sévère depuis plusieurs années. Il a fait plusieurs crises de colique néphrétique
tantôt à droite tantôt à gauche et une crise de goutte un an auparavant.
Il est vu pour la première fois dans le service pour une nouvelle crise de colique néphrétique avec hématurie macroscopique.
Le ionogramme sanguin montre : créatinine 250 micromoles/l, urée 128 mmol/l, sodium 135 mmol/l potassium 5 mmol/l, chlore
95 mmol/l, bicarbonates 21 mmol/l, protéines 77 g/l, calcémie 2 mmol/l, phosphorémie 1,80 mmol/l, uricémie 475 micromoles/l,
protéinurie 0,50 g/24 h. Dans l'urine : innombrables hématies. Examen bactériologique des urines : stérile.
La TA est à 200/110 mm Hg en position couchée. L'UIV montre deux gros reins à 15 cm de longueur avec retard de sécrétion
et déformation des tiges calicielles.

Quel diagnostic doit-on évoquer en premier ?


A - Pyélonéphrite chronique
B - Amylose
C - Polykystose rénale de l'adulte
D - Hyperparathyroïdie primaire
E - Tuberculose urogénitale
Bonne(s) réponse(s) : C

Il s'agit d'une insuffisance rénale chronique à gros reins. On peut donc ne retenir que B et C.
La notion d'antécédent d'HTA familiale précoce, le patient présentant lui-même une HTA précoce,
l'image urographique, font retenir le diagnostic de polykystose rénale de l'adulte.

Quel examen paraclinique doit-on pratiquer en premier pour confirmer le diagnostic ?


A - Echotomographie rénale
B - Ponction biopsie rénale
C - Artériographie rénale
D - Néphrogramme isotopique
E - Pyélographie ascendante
Bonne(s) réponse(s) : A

Evident.

Le malade devra diminuer la consommation de :


A - Miel
B - Riz
C - Chocolat
D - Fruits secs
E - Viande
Bonne(s) réponse(s) : C D

C - Du fait de sa richesse en potassium.


D - Très riche en potassium.
E - Est discutable, non pas tant à cause de l'insuffisance rénale (la ration protidique devant être réduite pour une clairance aux
alentours de 30 à 40 ml/mn) mais plutôt pour la richesse de la viande en purines dans le cadre de son hyperuricémie (éviter
surtout les abats, la cervelle).
Donc la réponse est un peu malaisée.
Par contre A et B peuvent être consommés sans problème.

Quel(s) médicament(s) pourrait(pourraient) être prescrit(s) à ce malade ?


A - Vitamine D
B - Sels de potassium
C - Supplémentation en phosphore
D - Inhibiteur de la xanthine-oxydase
E - Bêta-bloquant
Bonne(s) réponse(s) :

ANNULEE

300
Exclusivement sur DOC - DZ : www.doc-dz.com NADJI 85
RESIDANAT EN POCHE TOME II
Cas Clinique en QCM

Quel(s) diurétique(s) pourrait(pourraient) éventuellement être prescrit(s) à ce malade ?


A- Spironolactone
B - Amiloride
C - Furosémide
D - Hydrochlorothiazide
E - Triamtérène
Bonne(s) réponse(s) : C

A B E - Sont hyperkaliémiants, donc à éviter.


D - Est un thiazidique, inefficace en cas d'insuffisance rénale.
Seul le furosémide est utilisable comme diurétique au cours de l'insuffisance rénale chronique.

Madame R. Juliette, née en 1933, est adressée en novembre 80 par son médecin traitant. Dans les antécédents, on retient la
notion d'une polykystose rénale de l'adulte chez la mère et chez une soeur. La malade a eu cinq grossesses (deux fausses
couches et trois enfants), a subi une gastrectomie des 2/3 pour ulcère en 1974, une ligature de trompes en 1972 et se plaint
de céphalées frontales anciennes. En 1974 la créatininémie est à 18 mg/l, l'urée à 0,50 g/l, et en avril 80, la créatininémie est
à 27 mg/l, l'urée est à 0,69 g/l. Elle a eu plusieurs U.I.V., l'actuelle montrant un retard bilatéral de sécrétion, un rein droit de
12,5 cm et un rein gauche de 12 cm. Il n'y a ni protéinurie, ni hématurie, ni HTA, ni oedème et ni infection urinaire. Elle se
plaint seulement de quelques douleurs lombaires basses.

Quel est le diagnostic rénal le plus vraisemblable ?


A - Glomérulonéphrite chronique
B - Néphro-angiosclérose
C - Polykystose rénale de l'adulte
D - Néphrite interstitielle chronique
E - Syndrome néphrotique
Bonne(s) réponse(s) : C

Evident : il existe des antécédents familiaux (la transmission de la polykystose rénale est autosomique dominante).
Par ailleurs, il s'agit d'une insuffisance rénale à gros reins. En l'absence de diabète et d'amylose. Le diagnostic à retenir est
donc polykystose rénale de l'adulte.
Les autres items A B D donnent une insuffisance rénale à petits reins.
E - Pas de protéinurie donc faux.

Le meilleur examen supplémentaire à pratiquer chez cette malade dans un but diagnostique est :
A - Une cystographie par voie rétrograde
B - Une échotomographie rénale
C - Un scanner abdominal
D - Une artériographie rénale
E - Une ponction biopsie rénale
Bonne(s) réponse(s) : B

Evident : l'échographie rénale fait le diagnostic en montrant deux gros reins occupés par des cavités anéchogènes. Elle
permet en outre de rechercher d'autres kystes, en particulier hépatiques (30 à 60 % des cas).

L'on observe de façon constante au cours d'une polykystose rénale de l'adulte au stade de l'insuffisance rénale
chronique :
A - Des petits reins avec kystes bilatéraux
B - Des gros reins avec kystes bilatéraux
C - Une hypertension artérielle
D - Une protéinurie
E - Une note familiale de polykystose
Bonne(s) réponse(s) : B

A - Non, les reins restent hypertrophiés malgré l'insuffisance rénale chronique.


C - Parfois révélatrice, elle s'accompagne d'une fuite sodée urinaire
D - Elle est modérée et intermittente le plus souvent
E - Il existe des cas sporadiques.

L'on observe très fréquemment au cours d'une glomérulonéphrite chronique avec insuffisance rénale chronique :
A- Une protéinurie
B - Une hématurie microscopique
C - Une hypertension artérielle
D - Une hypercalcémie
E - Une infection urinaire
Bonne(s) réponse(s) : A B C

A - Protéinurie importante.
D - C'est le contraire : hypocalcémie.
301
Exclusivement sur DOC - DZ : www.doc-dz.com NADJI 85
RESIDANAT EN POCHE TOME II
Cas Clinique en QCM

Les néphrites interstitielles chroniques peuvent être secondaires à :


A - Reflux vésico-urétéral
B - Lithiase coralliforme infectée
C - Hypercalcémie persistante
D - Prise de ciclosporine
E - Consommation abusive et prolongée d'analgésiques
Bonne(s) réponse(s) : A B C D E

Les causes classiques de néphropathies interstitielles sont :


- les infection récidivantes, favorisées par des malformations urinaires ou des lithiases
- l'abus d'analgésiques type phénacétine
certains désordres métaboliques : hypercalcémie, hyperuricémie, hypokaliémie pour certains.

Une femme de 72 ans est admise en urgence dans le service d'urologie pour syndrome infectieux sévère et douleurs
lombaires droites. Dans ses antécédents, on retrouve quelques épisodes récents de cystite, une kystérectomie totale pour une
raison non connue réalisée dix ans auparavant et une cholécystectomie pour lithiase vésiculaire il y a cinq ans. A l'examen
clinique, outre une importante altération de l'état général et une hyperthermie supérieure à 39° évoluant depuis plusieurs
jours, il existe un empâtement douloureux de la fosse lombaire droite, une tuméfaction sus iliaque droite, douloureuse et
chaude. Sur le plan biologique, une hyperleucocytose, une très forte augmentation de la VS, la créatininémie est à 15 mg/l. Un
cliché d'abdomen sans préparation révèle une grisaille du flanc droit et une lithiase de type coralliforme droite

Quel diagnostic peut-on avancer pour expliquer l'ensemble du tableau clinique et paraclinique ?
A - Pyélonéphrite aiguë
B - Abcès du rein
C - Pyonéphrose fistulisée
D - Pyonéphrose
E - Pyélonéphrite xanthogranulomateuse
Bonne(s) réponse(s) : NNULEE

Les deux examens les plus utiles au diagnostic sont :


A - Echographie
B - Urographie intraveineuse
C - Scanner rénal
D - Urétéro-pyélographie rétrograde
E - Cystographie rétrograde
Bonne(s) réponse(s) :

ANNULEE

Quel(s) traitement(s) proposez-vous ?


A - Antibiothérapie générale
B - Drainage immédiat par sonde urétérale
C - Drainage immédiat par néphrostomie percutanée
D - Traitement chirurgical immédiat
E - Néphro-lithotomie pour exérèse du calcul coralliforme
Bonne(s) réponse(s) :

ANNULEE

Des hémocultures ont été réalisées et se révèlent positives. A votre avis, quel est le germe que l'on a le plus de
chance de mettre en évidence ?
A - Escherichia coli
B - Serratia
C - Klebsiella
D - Proteus
E - Staphylocoque
Bonne(s) réponse(s) :

ANNULEE

302
Exclusivement sur DOC - DZ : www.doc-dz.com NADJI 85
RESIDANAT EN POCHE TOME II
Cas Clinique en QCM

Quelles sont en l'absence de traitement la ou les complications possibles de ce tableau ?


A - Septicémie
B - Bactériémie
C - Néphrite tubulo-interstitielle aiguë contro latérale
D - Choc toxi-infectieux
E - Fistulisation à la peau
Bonne(s) réponse(s) :

ANNULEE

Une femme de 75 ans, sans passé néphrologique est traitée pour insuffisance cardiaque depuis deux mois par Digoxine® : 1
comprimé par jour, Lasilix® : 40 mg par jour et un régime désodé parfaitement suivi. Depuis une semaine elle présente des
vomissements et une diarrhée, la diurèse a chuté autour de 500 ml par jour depuis 48 heures. Devant ce tableau clinique, elle
est adressée en réanimation néphrologique. A son entrée, la pression artérielle est à 10-6 couchée, le pouls à 50/mn, la
conscience est normale. Le bilan biologique montre : Créatininémie : 320 micromol/l, natrémie : 120 mmol/l, kaliémie 6 mmol/l,
réserve alcaline 15 mmol/l, protidémie 79 g/l, azotémie 45 mmol/l, globules rouges 5 millions, hématocrite à 49 %.

Quel(s) est (sont) le(s) trouble(s) de l'hydratation que l'on peut évoquer devant ce tableau clinique et biologique ?
A - Déshydratation intracellulaire
B - Déshydratation extra-cellulaire
C - Déshydratation globale
D - Hyperhydratation intracellulaire
E - Hyperhydratation globale
Bonne(s) réponse(s) : B D

B - En raison de la protidémie et de l'hématocrite élevés.


D - En raison de l'hyponatrémie.

Comment peut-on expliquer la bradycardie ?


A - Elle est la conséquence de l'hyponatrémie
B - Elle est la conséquence de l'hyperazotémie
C - Elle est la conséquence de la diarrhée
D - Elle est la conséquence de l'intoxication digitalique
E - Elle est la conséquence des vomissements
Bonne(s) réponse(s) : D

D - L'intoxication digitalique peut entraîner un blocage de la conduction auriculaisre ou auriculoventriculaire.

Quelle(s) est(sont) le(s) mesure(s) thérapeutique(s) à prendre ?


A - Perfusion de glucosé isotonique
B - Injection quotidienne d'une ampoule de Furosemide pendant 3 jours
C - Apport sodé
D - Restriction hydrique
E - Arrêt des diurétiques
Bonne(s) réponse(s) : C D E

L'hyponatrémie à 120 impose l'arrêt du diurétique et un apport sodé associé à la restriction hydrique puisqu'il s'agit d'une
hyponatrémie de déplétion.

Parmi les propositions suivantes, quelle(s) est (sont) celle(s) qui permettrait(ent) d'évoquer une insuffisance
rénale aiguë fonctionnelle ?
A - Rapport urée urinaire/urée sanguine > 15
B - Rapport créatininurie/créatininémie > 15
C - Fraction d'élimination urinaire du sodium inférieure à 1
D - Natrémie inférieure à 130
E - Urée sanguine/créatininémie > 10
Bonne(s) réponse(s) : A B C

Sans commentaire.

303
Exclusivement sur DOC - DZ : www.doc-dz.com NADJI 85
RESIDANAT EN POCHE TOME II
Cas Clinique en QCM
Un insuffisant rénal chronique par polykystose rénale vous consulte pour aggravation récente et brutale de son
hyperazotémie. Le malade avait été vu en consultation deux mois auparavant avant son départ en vacances. Poids : 68 kilos,
pour 1,75 m T.A. à 140/80, créatinine à 3 50 micromol/l, urée à 0,80 g/l, natrémie à 140 mmol/l, kaliémie à 3,8 mmol/l, diurèse
: 2 litres par 24 h, pas d'oedème des membres inférieurs.
En vacances, il consulte pour un syndrome grippal et se voit prescrire :
- Totapen 2 g par jour pendant 8 jours
- Aspirine 500 mg par jour 2 fois
- Régime désodé strict avec 60 g de protides
Prescription que ce malade méticuleux, suit à la lettre.
Lorsqu'il consulte 8 jours plus tard, il paraît fatigué : Poids : 63 kilos, T.A. : 10/6, pas d'oedème, signe du pli,
biologiquement : Créatinine à 475 micromol/l, urée à 2 g par litre, natrémie à 130, kaliémie à 5 mEq par litre.
Diurèse des 24 dernières heures : 500 cc.

Une insuffisance rénale chronique peut être aggravée par :


A - Une déplétion calcique
B - Une déplétion sodée
C - Une insuffisance cardiaque
D - Un obstacle sur la voie excrétrice
E - Une hypovolémie
Bonne(s) réponse(s) : B C D E

Une insuffisance cardiaque avec un bas débit, par l'hypoperfusion rénale qu'il entraîne, peut aggraver une IRC. De même, une
hypovolémie (pertes digestives, saignement etc...), ou une déplétion sodée.

Comment définissez-vous l'état d'hydratation de ce patient ?


A - Déshydratation globale
B - Déshydratation extracellulaire pure
C - Hyperhydratation intracellulaire pure
D - Déshydratation extracellulaire avec hyperhydratation intracellulaire
E - Etat normal
Bonne(s) réponse(s) : D

L'amaigrissement, un pli cutané, l'hypotension et l'oligurie définissent la deshydratation extracellulaire. L'hyponatrémie ;


l'hyperhydratation intra-cellulaire.

Afin de confirmer le mécanisme vraisemblablement en cause chez ce patient, quel examen vous parait le plus
pertinent ?
A - Clairance de la créatinine
B - Natriurèse des 24 heures
C - Complément sérique
D - Biopsie rénale
E - Examen tomodensitométrique des reins
Bonne(s) réponse(s) : B

Le syndrome grippal avec probablement de la fièvre, a entraîné une perte de NaCl, encore aggravée par le régime désodé,
d'où la majoration de l'insuffisance rénale, de mécanisme fonctionnel. La natriurèse va être effondrée.

Quelle mesure thérapeutique préconisez-vous ?


A - Epuration extra rénale d'urgence
B - Régime à 30 g de protides sans sel
C - Furosémide (Lasilix®) à fortes doses
D - Apports hydriques de 2 litres par 24 heures minimum
E - Apports sodés per os ou I.V.
Bonne(s) réponse(s) : E

Sans commentaire.

Quel(s) élément(s) de surveillance parmi les suivants, vous parait(ssent) indispensables pendant le temps du
traitement ?
A - Poids
B - Tension artérielle
C - Phosphorémie
D - Diurèse
E - Calcémie
Bonne(s) réponse(s) : A B D

Sans commentaire.

304
Exclusivement sur DOC - DZ : www.doc-dz.com NADJI 85
RESIDANAT EN POCHE TOME II
Cas Clinique en QCM
Une patiente de 25 ans a, depuis quelques mois, une lithiase calicielle inférieure droite, radio-opaque de 12 x 10 mm. Depuis
48 heures, elle présente des crises de coliques néphrétiques droites, avec lors de l'admission une température à 40° C.

Il convient de réaliser en urgence devant cette lombalgie droite fébrile :


A - Une urographie intraveineuse
B - Une cystographie rétrograde
C - Un sondage
D - Des hémocultures
E - Un examen cytobactériologique des urines
Bonne(s) réponse(s) : A D E

- L'UIV permet de localiser le calcul.


- Les hémocultures et l'ECBU recherchent une bactériémie, une septicémie, une infection urinaire, qui compliquent le
traitement de la lithiase et imposent en urgence un traitement antibiotique, et si besoin, l'ablation du calcul et/ou le drainage
des urines purulentes au-dessus de l'obstacle.
- B et C sont formellement contre-indiqués car ils risquent d'introduire un germe souvent résistant dans les voies urinaires.

Les urgences chirurgicales de la lithiase des voies excrétrices supérieures comprennent :


A - L'anurie
B - La pyurie
C - L'hématurie
D - La colique néphrétique
E - La rétention purulente d'urine
Bonne(s) réponse(s) : A E

- L'anurie s'observe lorsqu'il y a un obstacle sur un rein unique, ou lorsque l'obstacle est bilatéral et expose à l'hyperkaliémie.
- La rétention purulente expose au choc septique à bacille gram négatif avec hypotension artérielle et insuffisance
cardiocirculatoire.

La rétention purulente sur l'obstacle nécessite de réaliser en urgence :


A - Une lithotripsie extra-corporelle
B - Une montée de sonde urétérale
C - Une néphrostomie
D - Une urétérostomie
E - Une antibiothérapie adaptée, avec anti-inflammatoires et anti-spasmodiques sans autre geste
Bonne(s) réponse(s) : B

- Ce n'est que en cas d'échec de la montée de sonde urétérale, que l'on fera une néphrostomie percutanée.
- La lithotripsie ne permet pas obligatoirement de lever l'obstacle, puisque les fragments peuvent être transitoirement
obstructifs.
- L'urétérostomie peut se compliquer de sténose ou de fistule lorsqu'elle est faite en contexte septique.

Quel(s) aurait (auraient) pu être le(s) traitement(s) de cette lithiase calicielle avant son enclavement ?
A - Une lithotripsie extra-corporelle
B - Une urétéroscopie
C - Une néphrolithotomie
D - Une alcalinisation des urines
E - La surveillance
Bonne(s) réponse(s) : A C

L'alcalinisation des urines serait une erreur, puisqu'il ne s'agit pas d'un calcul d'acide urique (lithiase radiotransparente). La
surveillance simple expose à la migration calculeuse et aux complications infectieuses.

Un homme de 25 ans vient vous consulter pour une grosseur du testicule gauche apparue depuis 2 mois. L'examen clinique
montre un testicule bosselé de 3 x 2 cm. Le reste de l'examen est sans particularité. L'interrogatoire révèle une cryptorchidie
traitée à l'âge de 9 ans.

Parmi les examens biologiques indispensables à demander avant tout geste thérapeutique, vous retenez :
A - C E A (ACE)
B - Alpha-foeto protéines
C - Ca 125
D - Bêta H CG
E - CA 19-9
Bonne(s) réponse(s) :B D

Sans commentaire.

305
Exclusivement sur DOC - DZ : www.doc-dz.com NADJI 85
RESIDANAT EN POCHE TOME II
Cas Clinique en QCM

Quel est le premier geste thérapeutique ?


A - Chimiothérapie
B - Hémicastration par voie haute
C - Biopsie testiculaire
D - Radiothérapie
E - Aucune proposition n'est juste
Bonne(s) réponse(s) : B

Le terme d'hémicastration par voie haute pourrait être remplacé par : castration G par voie haute avec clampage premier du
cordon spermatique.

L'examen histologique montre qu'il s'agit d'un carcinome embryonnaire. Le bilan d'extension comprend :
A - Scanner lombo-aortique
B - Lymphographie pelvienne
C - Echographie abdominale
D - Radiographie du thorax
E - Biopsie ostéo-médullaire
Bonne(s) réponse(s) : A B C D

- La lymphographie bipédieuse, et non pas pelvienne, reste classique, mais elle est de moins en moins faite, au profit du
scanner abdominal avec et sans injection.
- L'échographie abdominale donne une meilleure imagerie (que le scanner) pour les métastases.

Le bilan ne retrouve que des adénopathies lombo-aortiques dont le plus grand diamètre est de 6 cm. A quel
stade est vu le patient ?
A - Stade III
B - Stade IIa
C - Stade IV
D - Stade IIc
E - Stade IIb
Bonne(s) réponse(s) : D

Sans commentaire.

Quel est le traitement à envisager ?


A - Radiothérapie lombo-aortique
B - Chimiothérapie
C - Chimiothérapie + radiothérapie
D - Curage ganglionnaire lombo-aortique d'emblée
E - Hormonothérapie
Bonne(s) réponse(s) : B

Sans commentaire.

Le pourcentage de guérison est de :


A - 10 à 20 %
B - 20 à 40 %
C - 40 à 60 %
D - 60 à 80 %
E - 80 à 100 %
Bonne(s) réponse(s) : C

Il ne s'agit là que de valeurs moyennes, puisque le pronostic dépend non seulement de l'extension, mais du type histologique
et de la chimiosensibilité.

Le suivi thérapeutique comporte :


A - Dosage des marqueurs
B - Scanner lombo-aortique
C - Radiographie du thorax
D - Echographie abdominale
E - Scintigraphie osseuse
Bonne(s) réponse(s) : A B C D

L'échographie abdominale complète le scanner abdominal pour l'exploration du foie.

306
Exclusivement sur DOC - DZ : www.doc-dz.com NADJI 85
RESIDANAT EN POCHE TOME II
Cas Clinique en QCM
Un homme de 22 ans consulte pour des oedèmes des membres inférieurs. L'examen cardiaque et pulmonaire est normal. Les
urines sont claires. L'examen par bandelettes réactives (Albustix) montre une protéinurie importante et le dosage révèle une
protéinurie à 8 g/24 heures. Le diagnostic envisagé est donc celui de syndrome néphrotique.

Pour confirmer le diagnostic de syndrome néphrotique il est nécessaire et suffisant de demander un dosage :
A - De l'albuminémie
B - Du cholestérol sanguin
C - Des triglycérides sanguins
D - De la créatinine
E - De la kaliémie
Bonne(s) réponse(s) : A

La définition du syndrome néphrotique repose sur l'hypoalbuminémie et la protéinurie supérieure à 3 g par 24 heures.
L'hyperlipidémie ne fait pas partie de la définition, il s'agit d'un signe associé.

Quel(s) signe(s) pourrai(en)t faire douter du diagnostic de syndrome néphrotique à lésions glomérulaires
minimes :
A - Hypertension artérielle
B - Insuffisance rénale organique
C - Hématurie supérieure à 100 000/ml
D - Hypercholestérolémie
E - Hypertriglycéridémie
Bonne(s) réponse(s) : A B C

Sans commentaire.

En l'absence d'insuffisance rénale le traitement symptomatique des oedèmes peut comporter :


A - Repos au lit
B - Régime sans sel
C - Spironolactone (Aldactone)
D - Diurétique de l'anse (Lasilix)
E - Diurétique thiazidique (Diurilix )
Bonne(s) réponse(s) : A B C D E

Les oedèmes sont la conséquence de l'hyperaldostéronisme secondaire et de la natriurèse effondrée par réabsorption
excessive de Na par le tube distal du néphron. Le repos au lit, le régime sans sel et l'aldactone agissent directement contre
l'hyperaldostéronisme secondaire. Le Lasilix® et les thiazidiques, en augmentant le sodium délivré au tube distal, augmentent
l'effet diurétique anti-aldostérone de l'aldactone.

En cas de syndrome néphrotique à lésion glomérulaires minimes le traitement initial, lorsqu'il est décidé,
comporte dans tous les cas :
A - Corticoïdes (Cortancyl®)
B - Anti-inflammatoires non stéroïdiens
C - Immunodépresseurs (Imurel®, Endoxan® ou Chloraminophène®)
D - Anticoagulants
E - Antibiotiques
Bonne(s) réponse(s) : A

- Les antibiotiques n'ont aucune indication.


- Les anticoagulants ne sont prescrits qu'en cas d'hypoalbuminémie très sévère (< 10 g/l) ou de complication
thromboemboliques.
- Les immunosuppresseurs ne sont jamais prescrits d'emblée.
- Les AINS peuvent entraîner au cours du syndrome néphrotique une insuffisance rénale aiguë.

En cas de traitement par prednisone (Cortancyl®) la dose quotidienne initiale est le plus souvent de :
A - 1/4 mg/kg
B - 1/2 mg/kg
C - 1 mg/kg
D - 5 mg/kg
E - 10 mg/kg
Bonne(s) réponse(s) : C

Sans commentaire.

307
Exclusivement sur DOC - DZ : www.doc-dz.com NADJI 85
RESIDANAT EN POCHE TOME II
Cas Clinique en QCM

Cet homme a un syndrome néphrotique pur. Un traitement corticoïde est institué. Le syndrome néphrotique
persiste après 4 semaines de traitement. Une ponction biopsie rénale est alors faite. Que peut-elle montrer :
A - Lésions mésangiales avec dépôts IgA
B - Lésions glomérulaires de hyalinose segmentaire et focale
C - Glomérulopathie avec prolifération cellulaire endo-capillaire diffuse
D - Glomérulopathie avec prolifération cellulaire extra-capillaire diffuse
E - Glomérulopathie extra-membraneuse
Bonne(s) réponse(s) : B E

Les lésions de hyalinose sont théoriquement corticorésistantes. Les glomérulopathies extramembraneuses ne sont pas
sensibles à la corticothérapie.
A, C et D - Ne donnent pas de syndrome néphrotique pur.

Si le cyclophosphamide (Endoxan) est utilisé per os quotidiennement pendant plusieurs mois, Quelle(s)
complication(s) peut-on voir survenir ?
A - Aplasie médullaire
B - Alopécie
C - Cystite hémorragique
D - Atteinte des cellules germinatives testiculaires
E - Hyperplasie gingivale
Bonne(s) réponse(s) : A B C D

A - Surtout leucopénie et thrombopénie.

Une femme de 55 ans est hospitalisée pour oligurie. Depuis trois jours elle a une diarrhée importante et de la fièvre. A
l'examen asthénie importante et signes évidents de déshydratation. Les premiers examens sanguins montrent :
- urée sanguine 29 mmol/l
- créatinine 250 micromol/l
- protidémie 82 g/l
- natrémie 132 mmol/l
- kaliémie 3,2 mmol/l
- bicarbonatémie 13,5 mmol/l
- hématocrite 56 %

Une déshydratation extra-cellulaire peut se traduire par :


A- Pli cutané
B - Vomissements
C - Hypotension artérielle
D - Dyspnée
E - Sécheresse des muqueuses de la cavité buccale
Bonne(s) réponse(s) : A C

E - Caractérise la deshydratation intra-cellulaire.

Une hémoconcentration peut se traduire par :


A - Hyponatrémie
B - Hypokaliémie
C - Baisse des bicarbonates plasmatiques
D - Augmentation de la protidémie
E - Augmentation de l'hématocrite
Bonne(s) réponse(s) : D E

Sans commentaire.

308
Exclusivement sur DOC - DZ : www.doc-dz.com NADJI 85
RESIDANAT EN POCHE TOME II
Cas Clinique en QCM

L'étude des gaz du sang artériel de cette malade donne les résultats suivants :
- pH 7,33
- PC02 30 mm (4KPa)
- P02 97 mmHg (1 3KPa)
- SaO2 98 %
- C03H 13,5 mmol/l
A quelle anomalie acido-basique concluez-vous ?
A - Alcalose métabolique
B - Acidose mixte
C - Alcalose mixte
D - Alcalose métabolique + acidose gazeuse
E - Acidose métabolique
Bonne(s) réponse(s) : E

E - En raison de la baisse des bicarbonates plasmatiques et de l'absence d'hypoventilation alvéolaire.

Avant tout traitement, quelle est parmi ces 5 compositions d'urines celle qui est la plus appropriée à l'état de
cette malade ?

Na urinaire en mmol/l Urée urinaire en mmol/l


A- 6 45
B- 6 350
C- 125 45
D- 125 350
E- 60 150
Bonne(s) réponse(s) : B

B - Est la plus appropriée puisqu'il existe un état de deshydratation extracellulaire par pertes digestives et cutanées. Il en
résulte un hyperaldostéronisme secondaire, alors que l'excrétion urinaire d'urée est conservée.

Quel est parmi ces différents traitements celui qui vous parait le plus logique pour cette malade ?
A - Soluté bicarbonaté + soluté salé + chlorure de potassium
B - Antidiarrhéique et diète hydrique
C - Soluté glucosé + chlorure de potassium
D - Epuration extra-rénale
E - Soluté salé + diurétique de l'anse (Lasilix®) + chlorure de potassium
Bonne(s) réponse(s) : A

A - Bicarbonate isotonique à 14/1000 en perfusion ou eau de Vichy Célestin pour corriger la baisse des bicarbonates
sanguins, associés à du NaCl et du KCl car il existe une hyponatrémie (fièvre, diarrhée) et une hypokaliémie (diarrhée). La
correction de l'acidose va encore aggraver l'hypokaliémie.

Un homme de 55 ans a eu une colique néphrétique droite sans fièvre. Il n'a aucun antécédent. Les douleurs ont disparu. La
radiographie sans préparation centrée sur les reins est normale. Sur l'urographie intraveineuse on constate l'existence d'un
calcul clair; arrondi, de 15 mm de diamètre siégeant dans le bassinet droit sans distension marquée des cavités sus-jacentes.

Quelle est la nature chimique la plus probable de ce calcul ?


A - Urique
B - Cystinique
C - Oxalo-calcique
D - Phospho-calcique
E - Phospho-ammoniaco-magnésien
Bonne(s) réponse(s) : A

Il faut comprendre radio-transparent pour calcul clair.


B, C, D et E donnent des calculs radio-opaques à des degrés divers.

309
Exclusivement sur DOC - DZ : www.doc-dz.com NADJI 85
RESIDANAT EN POCHE TOME II
Cas Clinique en QCM

Il est utile de demander chez ce patient ?


A - Magnésémie
B - Recherche de cristaux de cystine dans les urines du matin
C - pH urinaire à chaque miction pendant 24 heures
D - Uricémie
E - Uricurie des 24 heures
Bonne(s) réponse(s) : C D E

- Le Ph urinaire est inférieur à 6 (acide) lorsqu'il y a une lithiase urique. L'uraturie est souvent augmentée. L'uricémie peut être
normale ou augmentée.
- Il serait très curieux qu'une cystinurie se révèle à 55 ans par une 1ère crise de colique néphrétique. La lithiase cystinique
n'est pas radio transparente.

Quelle attitude initiale devra-t-on adopter vis-à-vis de ce calcul ?


A - Réaliser rapidement une pyélotomie
B - Proposer une chirurgie percutanée
C - Proposer une destruction par ondes de choc (Lithotripsie)
D - Monter une sonde urétérale droite
E - Proposer une alcalinisation prolongée des urines
Bonne(s) réponse(s) : E

L'alcalinisation des urines suffit le plus souvent à dissoudre le calcul d'acide urique (elle diminue la cristallisation de l'acide
urique à Ph acide).

Quelle complication évolutive amènerait impérativement à modifier l'attitude précédente ?


A - Hématurie macroscopique totale
B - Reprise des douleurs
C - Distension du bassinet avec des calices fins (sur l'U.I.V.)
D - Infection urinaire avec fièvre à 40°
E - Leucocyturie isolée augmentée à plus de 100 000/ml
Bonne(s) réponse(s) : D

La survenue d'une infection urinaire avec fièvre (pyélonéphrite) sur l'obstacle inciterait à lever l'obstacle en urgence.

Quel(s) conseil(s) ou traitement(s) peut-on proposer pour éviter la récidive lithiasique chez ce patient en dehors
des cures de diurèse ?
A - Diurétiques thiazidiques
B - Urico-éliminateurs (Désuric®...)
C - Hypo-uricémiants par inhibition de la xanthine oxydase (Zyloric®)
D - Régime riche en protides
E - Boissons alcalines
Bonne(s) réponse(s) : E

- L'uricémie n'est pas connue, donc il est difficile de prescrire, a priori, du Zyloric.
- L'alcalinisation permanente des urines réduit au maximum le risque de formation de lithiase uratique.

Un malade ayant subi une intervention chirurgicale sur les voies urinaires présente dans les suites opératoires un tableau
septicémique.

Trois hémocultures sont positives en 48 heures à Enterobacter cloacae. Un traitement antibiotique est adapte selon la
sensibilité du germe à l'antibiogramme et associe cefotaxime (Claforan®) et amikacine (Amiklin®).

Enterobacter cloacae est :


A - Une corynébactérie commensale
B - Un cocci à Gram positif
C - Un bacille à Gram négatif appartenant à la famille des entérobactéries
D - Un bacille à Gram négatif appartenant à la famille des Pseudomonadaceae
E - Une mycobactérie
Bonne(s) réponse(s) : C

Sans commentaire.

310
Exclusivement sur DOC - DZ : www.doc-dz.com NADJI 85
RESIDANAT EN POCHE TOME II
Cas Clinique en QCM

Les bactéries responsables d'infections du tractus urinaire après manoeuvre instrumentale incluent :
A - Serratia marcescens
B - Streptococcus pneumoniae
C - Pseudomonas aeruginosa
D - Actineobacter
E - Brucella
Bonne(s) réponse(s) : A C D

Sans commentaire.

Le cefotaxime (Claforan®) est :


A - Une ampicilline
B - Une uréido-pénicilline
C - Une céphalosporine de 1ère génération
D - Une céphalosporine de 3ème génération
E - Un macrolide
Bonne(s) réponse(s) : D

Le Céfotaxime® est un antibiotique à large spectre, très actif sur les bacilles gram négatif, le plus souvent impliqués dans les
infections urinaire.

L'amikacine (Amiklin®) est :


A - Une béta-lactamine
B - Un aminoside
C - Une cycline
D - Un macrolide
E - Une quinolone
Bonne(s) réponse(s) : B

Sans commentaire.

La concentration minimale inhibitrice (CMI) :


A - Est la plus forte concentration d'antibiotique capable d'inhiber toute culture visible en milieu liquide
B - Est la plus faible concentration d'antibiotique capable d'inhiber toute culture visible en milieu liquide
C - Est la concentration d'antibiotique qui laisse subsister moins de 0,01% de survivants
D - Peut être déterminée par la méthode des disques en milieu gélosé (antibiogramme)
E - Ne répond à aucune de ces définitions
Bonne(s) réponse(s) : B D

Sans commentaire.

Un pouvoir bactéricide des antibiotiques et associations donne les résultats suivants :


amikacine : 0,01% de survivants ; céfotaxime : 1% de survivants ; céfotaxime + amikacine : 0,01% de
survivants, vous pouvez conclure :
A - Que l'amikacine seule est bactéricide
B - Que le céfotaxime seul est bactéricide
C - Que l'association est bactéricide
D - Que l'association est bactériostatique
E - Aucune conclusion n'est possible
Bonne(s) réponse(s) : A C

L'intérêt de l'étude du pouvoir bactéricide est très grand, puisqu'il assure qu'un antibiotique ou une association est
parfaitement synergique et donc efficace, sur le plan thérapeutique, sauf s'il y a une mauvaise diffusion tissulaire.

311
Exclusivement sur DOC - DZ : www.doc-dz.com NADJI 85
RESIDANAT EN POCHE TOME II
Cas Clinique en QCM
Une femme de 31 ans a depuis deux mois des oedèmes des membres inférieurs associés à des arthralgies fugaces et une
fébricule. L'examen révèle un oedème malléolaire modéré. La pression artérielle est à 120/70 mmHg. Les examens
biologiques suivants ont été pratiqués :
Examen des urines par les bandelettes : protéinurie +++, hématurie +, créatininémie 82 micromol/l, protides 56 g,
albuminémie 26 g, gammaglobulines à 22 9.
Formule numération : leucocytes 3 800/mm3, polynucléaires 29 %.

Définissez l'état rénal de cette malade ?


A - Syndrome néphrétique
B - Protéinurie orthostatique
C - Tubulopathie ancienne
D - Syndrome néphrotique impur
E - Syndrome néphrotique pur
Bonne(s) réponse(s) : D

D - Au cours du lupus, l'hypogammaglobulinémie habituellement observée lors d'un syndrome néphrotique, est remplacé par
une hypergammaglobulinémie.

L'hypergammaglobulinémie peut être en faveur de :


A - Une glomérulonéphrite à dépôts d'IgA
B - Une microangiopathie thrombotique
C - Une tubulopathie acquise
D - Une amylose associée à une gammapathie monoclonale
E - Un LED
Bonne(s) réponse(s) : D E

D - L'amylose AL est très rare à 31 ans.


A - L'augmentation des IgA sériques, au cours d'une maladie de Berger (glonérulonéphrite à dépôt d'IgA), se rencontre dans
moins de 50 % des cas.

La découverte d'un complément (CH50) effondré à 15% (normal 35 à 65) évoquerait à priori :
A - Une glomérulonéphrite aiguë
B - Une glomérulonéphrite intercapillaire à dépôts d'IgA
C - Une amylose
D - Une glomérulonéphrite toxique
E - Une glomérulonéphrite lupique
Bonne(s) réponse(s) : A E

Un syndrome néphrotique au cours du lupus s'accompagne d'une chute du CH50, à condition qu'il ne s'agisse pas d'une
gloménulonéphrite extra-membraneuse lupique, au cours de laquelle le CH50 peut être normal.

Les examens logiques dans un tel contexte comportent :


A - Recherche d'immuns complexes circulants
B - Artériographie
C - Scanner
D - Biopsie rénale
E - Biopsie médullaire
Bonne(s) réponse(s) : A D

A - Recherche des complexes immuns circulants, mais surtout facteurs anti-nucléaires, anticorps anti-histones et anti DNA
natifs.
D - La biopsie rénale permet :
- La confirmation du lupus.
- La définition du type de néphropathie : proliférative ou non.
- De préciser son degré d'activité et de chronicité.
- De fixer le pronostic.
- De préciser les indications thérapeutiques.

312
Exclusivement sur DOC - DZ : www.doc-dz.com NADJI 85
RESIDANAT EN POCHE TOME II
Cas Clinique en QCM
Monsieur D.... âgé de 75 ans, est hospitalisé pour une insuffisance rénale. Cet homme vit seul. Dans ses antécédents on note
:
- Une péritonite appendiculaire qui a nécessité trois interventions abdominales vers l'âge de 35 ans ; il a maintenant une
éventration.
- Un diabète sucré connu depuis 15 ans et traité par des sulfamides hypoglycémiants.
- Une hypertension artérielle connue depuis 20 ans et traitée par un bêtabloquant (Ténormine® 1 comprimé) et une
spironolactone (Aldactone® 75 mg 1 comprimé).
- Une crise d'angor récente qui a conduit son médecin à prescrire 3 comprimés par jour de diltiazem (Tildiem®). Depuis
quelques semaines Monsieur D. se plaint d'une asthénie, d'insomnie, de nausées. La diurèse est d'environ 2 litres par jour. A
l'examen clinique : poids 77 kgs, taille 1 m 68, pression artérielle en décubitus 190/120 mmHg, dyspnée de repos et nocturne,
tachycardie, bruit de galop à l'auscultation du coeur, oedèmes des deux chevilles, râles crépitants aux deux bases
pulmonaires, toucher rectal normal, FO : signe de croisement, pas de lésion rétinienne diabétique à l'angiographie. Les
examens biologiques sanguins montrent : Créatinine 800 micromol/l
- Urée 40 mmol/l
- Hémoglobine 7,8 g/100 ml
- Calcium 1,65 mmol/l
- Protides 60 g/l
- Potassium 5,1 mmol/l
- Sodium 140 mmol/l
- Glucose 9 mmol/l
Dans les urines :protéinurie 0,30 g/24 heures. Examen cytobactériologique normal.

Le(s) médicament(s) suivant(s) doi(ven)t être impérativement arrêté(s) :


A - Sulfamides hypoglycémiants
B - Tildiem®
C - Ténormine®
D - Aldactone®
E - Non, le traitement doit être maintenu
Bonne(s) réponse(s) : A C D

A - En raison du risque d'accumulation du sulfamide qui a une élimination rénale.


C - En raison de l'insuffisance cardiaque.
D - En raison de l'insuffisance rénale, l'aldactone est inefficace et risque d'aggraver l'hyperkaliémie.

Ce malade a :
A - Une hyper-hydratation globale
B - Une hyper-hydratation intra-cellulaire isolée
C - Une hyper-hydratation extra-cellulaire isolée
D - Une déshydratation extra-cellulaire isolée
E - Une déshydratation intra-cellulaire isolée
Bonne(s) réponse(s) : C

En raison des oedèmes et de la normalité de la natrémie.

Quels sont les arguments biologiques en faveur du caractère chronique de l'insuffisance rénale ?
A - Protidémie à 60 g/l
B - Natrémie à 140 mmol/l
C - Calcémie à 1,65 mmol/l
D - Hémoglobinémie à 7.8 g/100 ml
E - Kaliémie à 5,1 mmol/l
Bonne(s) réponse(s) : C D

Sans commentaire.

Pour préciser l'état de ses reins il est raisonnable de demander chez ce malade comme premier examen :
A - Une échographie rénale
B - Une urographie intra-veineuse
C - Une urétéro-pyélographie rétrograde
D - Une artériographie rénale
E - Un examen tomodensitométrique avec injection de produit iodé
Bonne(s) réponse(s) : A

C'est l'examen de choix pour préciser la taille, les contours, la différenciation cortico-médullaire des deux reins, pour
rechercher des lithiases, une dilatation des cavités pyélo-calicielles.

313
Exclusivement sur DOC - DZ : www.doc-dz.com NADJI 85
RESIDANAT EN POCHE TOME II
Cas Clinique en QCM

Les reins sont diminués de taille (8cm x 4cm), réguliers et symétriques. Il n'y a pas de dilatation pyélocalicielle.
L'étiologie la plus vraisemblable de l'insuffisance rénale de Monsieur D. est :
A - Une polykystose rénale
B - Une glomérulopathie diabétique
C - Une amylose
D - Une pyélonéphrite chronique
E - Une néphro-angiosclérose
Bonne(s) réponse(s) : E

En raison de l'HTA ancienne, de l'absence de protéinurie abondante, de la régularité des contours des deux reins et de
l'insuffisance coronarienne associée.

Monsieur D. a une insuffisance rénale chronique arrivée au stade terminal. Après avoir résolu les problèmes
actuels il va falloir envisager la méthode de suppléance la mieux adaptée à son cas. Quelles sont les deux
méthodes vraisemblablement les mieux appropriées ?
A - Dialyse péritonéale continue ambulatoire
B - Transplantation rénale
C - Hémodialyse en centre
D - Hémodialyse à domicile
E - Il est beaucoup trop âgé pour bénéficier d'une des méthodes précédentes
Bonne(s) réponse(s) : C E

A - Impossible en raison des interventions chirurgicales abdominales dans les antécédents, sources d'adhérences
péritonéales.
B - Cet homme est trop âgé pour la greffe.
D - Cet homme vit seul et ne pourra assurer seul son hémodialyse à domicile. Mais, l'âge n'est pas une contre-indication à la
dialyse en centre, même si c'est une contre-indication à la greffe.
Il faut donc cocher E dans cet état d'esprit (trop âgé pour la greffe).

Une femme de 49 ans est adressée à la consultation pour la découverte, au cours d'un bilan systématique d'une insuffisance
rénale modérée à 140 micromol/l de créatinine. Sa mère a vécu jusqu'à 76 ans avec une protéinurie. Son père est décédé
d'accident vasculaire cérébral. Un frère a présenté une lithiase rénale. Elle-même a eu deux grossesses normales et trois
enfants. Elle se souvient avoir présenté, il y a 20 ans, à la suite de la 2ème, une crise de colique néphrétique droite avec
température et frissons qui ont récidivé pendant trois semaines. Elle est traitée depuis 4 ans par clonidine (Catapressan® 3
comprimés par jour) pour une hypertension artérielle modérée. Elle n'a pas de protéinurie.

Dans un tel tableau clinique et à ce stade, il est plausible d'envisager le ou les diagnostics étiologiques suivants :
A - Néphro-angiosclérose
B - Pyélonéphrite chronique
C - Nécrose corticale
D - Glomérulonéphrite chronique
E - Toxicité rénale du Catapressan
Bonne(s) réponse(s) : A B

A - Donne une insuffisance rénale modérée sans protéinurie abondante.


B - Cette patiente a fait au décours de sa deuxième grossesse une pyélonéphrite aiguë sur obstacle : on peut imaginer, en
l'absence d'émission de calcul, qu'elle a fait plusieurs infections urinaires à bas bruit, et qu'elle a éventuellement des lithiases
rénales bilatérales, ou un reflux bilatéral expliquant l'insuffisance rénale.
C - Donnerait un syndrome d'insuffisance rénale aiguë, ce qui n'est pas le cas et ce, d'autant plus que les circonstances
favorisant la nécrose corticale ne sont pas présentes.
D - Donnerait une protéinurie abondante.
E - Le Catapressan n'a pas de toxicité rénale.

Quel est, parmi ceux-ci, l'examen à réaliser sans délai ?


A - Scanner
B - Etude cinétique par l.R.M.
C - Artériographie conventionnelle
D - Urographie intraveineuse
E - Immuno-électrophorèse urinaire
Bonne(s) réponse(s) :

QUESTION ANNULEE.

314
Exclusivement sur DOC - DZ : www.doc-dz.com NADJI 85
RESIDANAT EN POCHE TOME II
Cas Clinique en QCM

Une échographie montre un petit rein droit, 6,5 cm de longueur, avec une image dense dessinant un cône
d'ombre. Le rein gauche est normal. Votre diagnostic est :
A - Tumeur de la voie excrétrice
B - Glomérulonéphrite atrophiante
C - Thrombose de la veine rénale
D - Séquelle de néphropathie gravidique
E - Pyélonéphrite chronique obstructive
Bonne(s) réponse(s) : E

Il existe à l'évidence une lithiase puisqu'il y a un cône d'ombre.

L'hypertension peut éventuellement être rapportée au petit rein. Pour le prouver, la meilleure méthode paraît
être :
A - Dosage séparé d'activité rénine
B - Biopsie rénale bilatérale
C - Artériographie
D - Etude de l'élimination rénale des métabolites et des prostaglandines
E - Cathétérisme des uretères et calcul de la clairance de l'inuline de chaque rein
Bonne(s) réponse(s) : A

Sans commentaire.

Le taux de rénine périphérique est trouvé à 5 fois la valeur normale sans médicament. Quel(s) médicament(s),
parmi les suivants, peu(ven)t l'abaisser ?
A - Hypotenseur d'action centrale (clonidine, Catapressan®)
B - Inhibiteur de l'enzyme de conversion
C - Anti-aldostérone
D - Bêtabloqueur
E - Vasodilatateur (dihydralasine, Népressol®)
Bonne(s) réponse(s) : D E

Sans commentaire.

L'hypokaliémie constatée à 3,2 mmol/l peut être corrigée dans ce cas, au moins en partie, par :
A - Diurétique thiazidique
B - Inhibiteur de l'enzyme de conversion
C - Anti-aldostérone
D - Bêtabloqueur
E - Vasodilatateur
Bonne(s) réponse(s) : B C

C - Les anti-aldostérone sont théoriquement contre-indiqués en cas d'insuffisance rénale en raison de leur inefficacité.

315
Exclusivement sur DOC - DZ : www.doc-dz.com NADJI 85
RESIDANAT EN POCHE TOME II
Cas Clinique en QCM
Ce patient de 62 ans sans autre antécédent pathologique qu'un excès pondéral ancien et une probable crise de goutte du
gros orteil droit 4 ans auparavant présente de violentes douleurs de l'hypochondre droit et de la fosse iliaque droite qui, se
prolongeant pendant plusieurs heures, motivent son hospitalisation. Le bilan biologique systématique révèle alors comme
seule anomalie une insuffisance rénale avec azotémie = 37 mmol/l, créatininémie = 620 micromol/l sans perturbations
phosphocalciques, sans hyperkaliémie, sans acidose. L'ASP ne montre aucune image pathologique. L'échographie montre
des cavités pyélocalicielles droites légèrement dilatées et des cavités pyélocalicielles gauches dilatées et occupées par une
formation lithiasique. Sous traitement antispasmodique et antalgique les douleurs cèdent et le lendemain se produit une
débacle urinaire de 5,2 l suivie en 6 jours du retour à la normale des taux de l'azotémie et de la créatinémie. L'UlV, qui est
alors pratiquée, révèle 2 reins de taille et de contours normaux et symétriques, des cavités excrétrices normales à droite mais
à gauche la présence d'un volumineux calcul coralliforme radiotransparent qui occupe pratiquement l'ensemble des calices et
du bassinet. L'uricémie est trouvée à 625 micromol/l. On retient alors le diagnostic d'insuffisance rénale aiguë obstructive sur
lithiase coralliforme urique gauche et probable migration calculeuse à droite. Quelques examens complémentaires sont alors
pratiqués :
- pH urinaire : 25 déterminations, toutes comprises entre 5 et 5,6
- Calcémies : 2,27 - 2,30 - 2,32 mmol/l
- Oxaluries/24h : normales
- Uricémies : 650 - 590 - 640 micromol/l
- Uricuries : 2450- 2300 - 2130 micromol/l

L'analyse de l'ensemble de ces données cliniques et paracliniques vous permet de retenir comme principe
thérapeutique :
A - Traitement chirurgical de la lithiase coralliforme gauche
B - Traitement médical seul
C - Traitement médical et chirurgical
D - Traitement purement diététique
E - Mesures diététiques et traitement médical
Bonne(s) réponse(s) : E

Grâce à l'alcalinisation des urines par l'eau de Vichy par exemple ou par Alcaphor, les lithiases d'acide urique vont
progressivement être dissoutes. Parallèlement, l'allopurinol va corriger l'hyperuricémie. On n'oublie pas, outre l'alcalinisation
permanente des urines, les boissons abondantes.

Vous allez recommander et prescrire :


A - Traitement alcalinisant permanent des urines
B - Traitement antiseptique urinaire prolongé
C - Cure de diurèse permanente
D - Traitement uricosurique par benziodarone (Amplivix®)
E - Traitement urico-inhibiteur par allopurinol (Zyloric®)
Bonne(s) réponse(s) : A C E

Sans commentaire.

Vous retiendrez comme critère(s) de surveillance et d'efficacité de votre prescription :


A - Diurèse de 24h constamment supérieure à 1,5 l
B - Diurèse de 24 h constamment supérieure à 2,5 l
C - pH urinaire se situant en permanence < 5,5
D - pH urinaire se situant en permanence > 6,5
E - Normalisation de l'uricémie
Bonne(s) réponse(s) : B D E

Sans commentaire.

Six mois après le début du traitement que vous avez prescrit, vous désirez faire le point sur son efficacité. Parmi
les investigations suivantes, laquelle faites-vous pratiquer ?
A - Urétéropyélographie rétrograde
B - Urographie intraveineuse
C - Cliché de l'arbre urinaire sans préparation
D - Examen des urines à la recherche de cristaux
E - Epreuve de cristallurie provoquée
Bonne(s) réponse(s) : B

L'UIV va permettre de suivre l'évolution de la lithiase coralliforme et d'en apprécier son caractère obstructif ou non.

316
Exclusivement sur DOC - DZ : www.doc-dz.com NADJI 85
RESIDANAT EN POCHE TOME II
Cas Clinique en QCM
Une jeune femme âgée de 25 ans consulte pour des infections urinaires à répétition évoluant depuis plusieurs années. Le
germe responsable est constamment un Escherichia Coli multirésistant aux antibiotiques usuels. Dans ses antécédents, on
note qu'elle a présenté des infections urinaires très tôt dès l'enfance, et qu'elle a subi à l'âge de 9 ans une intervention anti-
reflux bilatérale en raison d'un reflux vésico-urétéral bilatéral avec duplicité urétérale gauche. De plus, des épisodes fréquents
de pyélonéphrite gauche ont motivé une néphrectomie polaire inférieure gauche 8 mois auparavant, mais sans réduction de la
fréquence des infections. Elle signale même la persistance, lors de certains épisodes infectieux, de douleurs lombaires
gauches, sans fièvre. Une nouvelle cystographie rétrograde montre, à gauche, un reflux vésicorénal et un reflux dans le
moignon urétéral inférieur restant.

Avant de prendre une décision thérapeutique, quel examen complémentaire vous parait souhaitable pour
éliminer la possibilité d'un reflux également à droite ?
A - Echographie rénale
B - Cystoscopie
C - Scintigraphie rénale
D - Exploration urodynamique
E - Urographie intra-veineuse
Bonne(s) réponse(s) : E

Sans commentaire.

Typiquement et sur une nouvelle urographie, le reflux pourrait se traduire par :


A - Gros rein gauche muet
B - Dilatation globale des cavités pyélocalicielles
C - Déformation des calices en massue prédominant aux pôles supérieurs
D - Hypertrophie corticale en regard des calices déformés
E - Calcifications péri-papillaires en flammèches
Bonne(s) réponse(s) : B C

Sans commentaire.

Avant de décider du traitement il est indispensable de disposer de :


A - Cyto-bactériologie urinaire
B - Antibiogramme
C - Détermination des CMI aux antibiotiques usuels
D - Dosage de la créatinine plasmatique
E - Précisions sur ses traitements antérieurs
Bonne(s) réponse(s) : A B C D E

Sans commentaire.

Le germe urinaire s'avère résistant à tous les antibiotiques usuels sauf aux aminosides. Quel schéma de
traitement médical vous paraît le plus adapté ?
A - Association béta-lactamine (IM) et aminoside (IM) 2 fois par jour pendant 7 jours
B - Aminoside seul (IM) en instillation vésicale
C - Aminoside seul (IM) 1 fois par jour pendant 15 jours
D - Aminoside seul (IM) 1 fois par jour pendant 6 mois
E - Pas de traitement symptomatique de cette infection chronique
Bonne(s) réponse(s) : A

Une monothérapie par un aminoside seul risquerait de sélectionner une souche bactérienne résistante.

Ce traitement médical doit être associé à :


A - Une néphrectomie gauche sans tarder
B - A une nouvelle intervention anti-reflux gauche sous couvert d'antibiotiques
C - A une intervention anti-reflux bilatérale systématique
D - A une exérèse du moignon urétéral gauche
E - A aucun traitement chirurgical tant que les urines ne sont pas durablement stérilisées
Bonne(s) réponse(s) : B D

Sans commentaire.

317
Exclusivement sur DOC - DZ : www.doc-dz.com NADJI 85
RESIDANAT EN POCHE TOME II
Cas Clinique en QCM

En dehors des infections urinaires, la correction du reflux à un effet favorable sur :


A - Lithiase coralliforme
B - Hypertension artérielle
C - Pyélonéphrite chronique
D - Perte de sel
E - Insuffisance rénale
Bonne(s) réponse(s) : A B C D E

Sans commentaire.

Une femme de 54 ans, cuisinière, a eu un accident de vélomoteur qui a entraîné, il y a 10 mois, une fracture de la malléole
externe droite et une abrasion cutanée de toute la face externe de la cuisse gauche. Elle a subi une réduction chirurgicale qui
laisse subsister une laxité articulaire et une greffe cutanée. Les inductions anesthésiques se sont accompagnées de rougeur
cutanée, prurit, tachycardie.
Un traitement initial à la pénicilline a entraîné une réaction urticarienne.
Il existe par ailleurs un terrain familial d'eczéma et de rhume des foins.
Depuis un mois, les paupières et les doigts sont enflés le matin surtout.
Les mollets sont un peu douloureux, à l'examen un léger oedème est trouvé à la cheville droite. La recherche de la protéinurie
était négative au moment de l'accident : il y a un mois la bandelette révélait de + à + +. Depuis une semaine le visage est
bouffi, les oedèmes des membres inférieurs sont nets. Les mains sont boudinées et la patiente a pris 5 kgs.

Ce tableau clinique et son mode d'installation évoquent en premier lieu :


A - Insuffisance ventriculaire droite
B - Insuffisance cardiaque globale
C - Thrombose de la veine cave
D - Syndrome néphrotique
E - OEdèmes allergiques
Bonne(s) réponse(s) : D

L'association oedèmes et protéinurie évoque en premier lieu un syndrome néphrotique.

Le taux sérique d'albumine est de 24 g/l, le taux de protéines de 52 g/1 et la kaliémie à 3,2 mEq/l. Le(s)
facteur(s) suivant(s) peut ou peuvent être impliqué(s) dans le mécanisme des oedèmes :
A - Baisse de la pression oncotique des protéines
B - Hyperaldostéronisme
C - Hyperpression veineuse
D - Capillarité allergique
E - Hypersécrétion d'ADH
Bonne(s) réponse(s) : A B

Sans commentaire.

Sur les premiers éléments cliniques biologiques, le(s) traitement(s) suivant(s) vous parait(ssent) adéquat(s) à
titre symptomatique :
A - Digitalique et diurétique thiazidique
B - Perfusion de sérum bicarbonaté isotonique
C - Perfusion de soluté protéique : albumine ou plasma
D - Diurétique anti-aldostérone
E - Mannitol 10 %, 250 ml en perfusion
Bonne(s) réponse(s) : C D

C - En pratique, en l'absence d'oedèmes "réfractaires", c'est-à-dire qui résisteraient au repos, associé au régime désodé strict
et aux diurétiques, les perfusions d'albumine et encore plus de plasma (risques transfusionnels) n'ont pas d'indication.

La malade refusant toute hospitalisation, il a été décidé en l'absence d'hématurie ou d'insuffisance rénale
associées de faire un traitement actif.
Quel est le schéma choisi parmi les suivants ?
A - Cyclosporine 10 mg/kg/jour pendant six mois
B - Cyclophosphamide 3 mg/kg/jour pendant trois mois
C - Prednisone 0,10 mg/kg/jour pendant six mois
D - Prednisone 0,10 mg/kg/jour pendant quatre semaines
E - Prednisone 1,5 mg/kg/jour pendant quatre semaines
Bonne(s) réponse(s) : E

E - Il s'agit du traitement d'un syndrome néphrotique pur. Théoriquement, chez l'adulte, il faut faire une ponction-biopsie rénale
pour faire le diagnostic du type histologique, car il ne s'agit pas le plus souvent d'une néphrose lipoïdique, cortico-sensible,
comme chez l'enfant.

318
Exclusivement sur DOC - DZ : www.doc-dz.com NADJI 85
RESIDANAT EN POCHE TOME II
Cas Clinique en QCM

Le traitement s'est avéré inefficace. La protéinurie et les oedèmes persistent


Quelle est l'attitude à ce moment là ?
A - Traitement symptomatique des oedèmes
B - Traitement symptomatique des oedèmes et anti-agrégants plaquettaires
C - Poursuite du traitement corticoïde à faible dose et hypolipémiant si nécessaire
D - Poursuite du traitement corticoïde à forte dose
E - Ponction biopsie rénale
Bonne(s) réponse(s) : A E

Sans commentaire.

En cas de syndrome néphrotique chronique par glomérulonéphrite extra-membraneuse, quelle(s) est (sont) les
complications envisagées en l'absence de traitement ?
A - Survenue d'une insuffisance rénale dans 1/3 des cas
B - Thrombose veineuse profonde à bas bruit
C - Thrombose veineuse extensive et risque d'embolie pulmonaire
D - Hypocalcémie
E - Troubles trophiques
Bonne(s) réponse(s) : A B C E

A noter que les glomérulonéphrites extra-membraneuses primitives ne sont pas toujours accessibles à un traitement
spécifiques.
D - En l'absence d'insuffisance rénale, il n'y a pas en règle générale d'hypocalcémie vraie (la calcémie diminue parallèlement
à l'albuminémie ; la calcémie corrigée reste normale.

Un homme de 68 ans consulte pour des troubles mictionnels mal systématisés. Il n'a pas d'antécédent pathologique
particulier.
A l'examen clinique, il existe une induration nodulaire marquée du lobe gauche de la prostate, qui respecte parfaitement les
contours de la glande.
La prostate est par ailleurs souple dans son ensemble, elle est globalement hypertrophiée et son poids est évalué à une
quarantaine de grammes.

Quel examen permet de confirmer le diagnostic d'adénocarcinome de la prostate avec certitude ?


A - Dosage des phosphatases acides prostatiques radio-immunologique
B - Examen cytologique des sécrétions recueillies après massage prostatique
C - Dosage de l'antigène prostatique spécifique sérique
D - Examen cytologique des urines après massage de la prostate
E - Biopsie prostatique
Bonne(s) réponse(s) : E

Sans commentaire.

Le diagnostic d'adénocarcinome prostatique étant confirmé, le meilleur examen pour déterminer l'intégrité de la
capsule prostatique est :
A - Le taux d'antigène prostatique spécifique sérique
B - L'échographie prostatique endorectale
C - La détermination du score de Gleason
D - La tomodensitométrie pelvienne
E - L'imagerie par résonance magnétique nucléaire pelvienne
Bonne(s) réponse(s) : B

L'échographie prostatique endorectale n'est malgré tout pas formelle puisqu'un envahissement microscopique ne pourra pas
être diagnostiqué.

Les examens ont confirmé que le cancer de la prostate était limité à la glande. Parmi les options chirurgicales
possibles, laquelle devez-vous retenir ?
A - Résection trans-uréthrale de la prostate avec orchidectomie sous albuginée
B - Adénomectomie trans-vésicale avec orchidectomie sous albuginée
C - Prostatectomie radicale avec orchidectomie sous albuginée
D - Prostatectomie radicale avec traitement par les agonistes du LH-RH
E - Prostatectomie radicale précédée d'un curage ganglionnaire llio-obturateur
Bonne(s) réponse(s) :

Aucune réponse ne convient. Seule conviendrait la prostatectomie radicale sans traitement complémentaire.

319
Exclusivement sur DOC - DZ : www.doc-dz.com NADJI 85
RESIDANAT EN POCHE TOME II
Cas Clinique en QCM

La pièce opératoire révèle seulement un envahissement de la vésicule séminale, quel est le stade de la maladie
selon la classification UICC ?
A - T1 N0M0
B - T2 N0M0
C - T3 N0M0
D - T4 N0M0
E - T4 N1M0
Bonne(s) réponse(s) : C

Sans commentaire.

Le malade refusant toute intervention chirurgicale a opté pour un traitement de radiothérapie prostatique. Les
complications potentielles de la radiothérapie comportent :
A - Impuissance sexuelle
B - Cystite radique
C - Rectite radique
D - Persistance de la tumeur intraprostatique
E - Ejaculation rétrograde
Bonne(s) réponse(s) : A B C D

A - L'impuissance sexuelle n'est pas une complication fréquente de la radiothérapie.

Une femme de 30 ans se plaint depuis trois ans de troubles urinaires.


Trois à quatre fois, il y a 2 ans, elle a présenté, par crises, des douleurs sus-pubiennes avec sensation de pesanteur vésicale,
des mictions très fréquentes et douloureuses. Ont été alors pratiquées une cystoscopie et une urographie intra-veineuse
jugées normales. L'année précédente a été indemne de tout épisode et une uroculture systématique a été trouvée stérile. A
son retour de vacances marquées par une grande activité sexuelle, elle présente les mêmes symptômes qu'il y a deux ans
associés à une hématurie macroscopique ; la température est normale, il n'y a pas de protéinurie sur les urines filtrées, la
cytologie montre des hématies non déformées, une grande quantité de cellules vésicales et de leucocytes. L'uroculture trouve
106 germes par ml. La patiente a réduit volontairement la quantité de boissons car les mictions sont très douloureuses.

Vous pratiquez une uroculture. Dans ce contexte, quel est le germe le plus fréquemment mis en évidence :
A - Escherichia Coli
B - Streptocoque hémolytique
C - Staphylocoque
D - Bacille pyocyanique
E - Protéus Hauseri
Bonne(s) réponse(s) : A

Sans commentaire.

Vous notez en faveur d'une infection urinaire strictement limitée à l'appareil urinaire inférieur (infection urinaire
basse), le ou les éléments suivants :
A - Absence de protéinurie
B - Existence d'une hématurie macroscopique
C - Absence de fièvre
D - Leucocyturie importante
E - Urographie intra-veineuse normale
Bonne(s) réponse(s) : A C E

Sans commentaire.

Quelle(s) mesure(s) allez-vous adopter ?


A - Gentalline IM 4 fois 40 mg par jour pendant 8 jours
B - Irrigation intra-vésicale d'anti-inflammatoires ou de corticoïdes
C - Boissons abondantes, 2 litres d'eau minérale par 24 heures
D - Anti-infectieux pendant 7 jours (acide pipémidique ou Pipram®)
E - Recherche d'une maladie sexuellement transmissible
Bonne(s) réponse(s) : C D

Sans commentaire.

320
Exclusivement sur DOC - DZ : www.doc-dz.com NADJI 85
RESIDANAT EN POCHE TOME II
Cas Clinique en QCM

L'uroculture a découvert une infection à protéus mirabilis.


Une bactériurie au même germe est retrouvée à deux reprises pendant les quatre mois suivants. On envisage
alors :
A - La prescription d'une échographie rénale
B - Un risque important d'insuffisance rénale d'évolution rapide
C - Un risque de diabète
D - Un risque de lithiase rénale
E - La patiente doit interrompre la prise de pilule contraceptive
Bonne(s) réponse(s) : A D

Sans commentaire.

Monsieur X. se présente à la consultation pour hypertension de découverte récente. Elle a été l'objet il y a 8 mois d'un court
bilan :
hypertrophie ventriculaire gauche d'importance moyenne; ni protéinurie, ni hématurie. Depuis un mois il accuse des céphalées
et les chiffres qu'il mesure le matin sont en moyenne de 180/120 mmHg (autotensiomètre).
Il a présenté 4 ans auparavant une lithiase urétérale obstructive à droite dont le traitement radical par cathétérisme ascendant
a été tardif et compliqué d'une pyélonéphrite dont la guérison a nécessité deux mois d'antibiothérapie.
Ses parents directs et sa fratrie sont normotendus.
A votre examen, la pression artérielle est en moyenne à 195/130, le fond d'oeil est de stade
II - III ; dans les urines : protéines traces, sang +, cultures stériles.

Parmi les données fournies, vous retenez en faveur d'une hypertension artérielle secondaire dans le contexte
cité :
A - L'âge
B - L'existence d'une protéinurie le jour de l'examen
C - L'absence d'antécédent familial
D - Les antécédents "rénaux"
E - L'existence de signes rétiniens
Bonne(s) réponse(s) : C D

Sans commentaire.

Les dosages sanguins montrent : dans le sang, créatinine 180 micromoles et potassium 3,1 mmol par litre.
Cette hypokaliémie est à rapporter selon vous à :
A - Une récidive lithiasique
B - Une insuffisance rénale chronique
C - Une tubulopathie congénitale
D - Une néphropathie tubulo-interstitielle due au traitement antibiotique prolongé
E - Un hyperaldostéronisme
Bonne(s) réponse(s) : E

Sans commentaire.

Choisissez un examen complémentaire parmi les suivants que vous ferez rapidement et en premier lieu :
A - Electronystasmogramme
B - Un enregistrement continu de la pression artérielle
C - Une échographie rénale
D - Une artériographie aortique et rénale
E - Un cathétérisme séparateur des uretères
Bonne(s) réponse(s) : C

L'échographie rénale va préciser la taille, les contours des reins, la morphologie des cavités pyélocalicielles, et recherchera
une ou plusieurs lithiases.

Le patient ne suit pas vos conseils et ne se traite pas. Il voit un ami radiologue qui pratique une urographie
intraveineuse. Celle-ci révèle un petit rein droit, 7,5 cm de diamètre vertical. Le rein gauche est apparemment
de taille normale. Il existe un retard de sécrétion des deux reins. L'insuffisance rénale s'explique par :
A - L'atteinte du rein droit
B - Une glomérulonéphrite chronique
C - Une tubulopathie au niveau du rein droit
D - Une lithiase obstructive
E - L'atrophie du rein droit avec néphro-angiosclérose du rein gauche
Bonne(s) réponse(s) : E

Pour qu'il y ait une insuffisance rénale, il faut obligatoirement que les deux reins soient atteints par un même ou plusieurs
processus pathologiques.

321
Exclusivement sur DOC - DZ : www.doc-dz.com NADJI 85
RESIDANAT EN POCHE TOME II
Cas Clinique en QCM

Si vous décidez d'employer un traitement médical, quel est celui qui vous apparaît adapté, au contexte et
adéquat en monothérapie ?
A - Réserpine
B - Rénèse®
C - Sectral®
D - Népressol® (Dihydralazine)
E - Isoniazide
Bonne(s) réponse(s) : C

Sans commentaire.

Une femme de 61 ans se présente à la consultation. Elle n'a pas de passé pathologique connu.
Une unique grossesse a été marquée par une protéinurie, non retrouvée après l'accouchement. Elle a tenu pendant 20 ans un
petit commerce. Elle a pris régulièrement pendant cette période un à plusieurs comprimés par jour d'une préparation
antalgique et calmante. Elle a présenté trois épisodes de douleurs lombaires droites avec fièvre et infection urinaire.
Convoquée à un examen par un centre d'examen, les découvertes ont été les suivantes : la pression artérielle est à 175/115
mm de Hg, la créatinine plasmatique est de 300 micromoles/l.
L'examen des urines ne compte ni protéinurie, ni hématurie.
Le fond d'oeil est de stade
II y a un an, une crise douloureuse du gros orteil a été interprétée comme une crise de goutte.

Parmi les diagnostics suivants, lequel ou lesquels, est ou sont, compatible(s) avec le tableau clinique ci-dessus
et à évoquer avant tout examen complémentaire ?
A - Pyélonéphrite chronique
B - Néphropathie interstitielle par prise abusive d'analgésiques
C - Glomérulonéphrite chronique
D - Hypertension artérielle maligne
E - Néphropathie du lupus érythémateux disséminé
Bonne(s) réponse(s) : A B

Sans commentaire.

L'examen clinique confirme les chiffres tensionnels et trouve deux gros reins non douloureux à la palpation.
Dans ce contexte clinique, le ou les diagnostic(s) peut ou peuvent être envisagé(s) comme probable(s) ou
possible(s) :
A - Pyélonéphrite chronique par reflux vésico-urétéral
B - Hydronéphrose bilatérale par anomalie de la jonction pyélo-urétérale
C - Thrombose cave et des veines rénales
D - Sténose ou thrombose des artères rénales
E - Polykystose rénale
Bonne(s) réponse(s) : B E

Puisqu'il existe deux gros reins à la palpation, il ne peut s'agir de pyélonéphrite chronique. L'absence d'hématurie rend
néanmoins le diagnostic de polykystie rénale peu probable.

Il vous paraît justifié de compléter le bilan par l'investigation suivante :


A - Artériographie
B - Scanner crânien
C - Tomographie et scintigraphie pulmonaire
D - Echographie rénale et hépatique
E - Ponction biopsie du parenchyme rénal
Bonne(s) réponse(s) : D

C'est l'échographie rénale qui va permettre de faire le diagnostic entre polykystose et hydronéphrose (dilatation des cavités
pyélocalicielles). De plus, l'échographie hépatique, si elle objective des kystes hépatiques associés aux kystes rénaux, viendra
conforter le diagnostic de polykystose.

Parmi les traitements suivants de l'hyperuricémie, lequel vous paraît le plus adéquat ?
A - Colchicine® 3 mg/j pendant six mois
B - Uricozyme (uricolytique 1 injection intramusculaire par mois pendant 6 mois)
C - Amplivix® (Benzodiarone) ou Désuric® (Benzobromarone) 300 mg par jour pendant six mois
D - Lasilix® (furosémide) 200 mg par jour pendant six mois
E - Zyloric® (allopurinol) 200 mg par jour pendant six mois
Bonne(s) réponse(s) : E

Néanmoins, il faudra se méfier de l'emploi du zyloric chez l'insuffisant rénal, car il donne beaucoup plus souvent des effets
indésirables sévères, et préférer le traitement au coup par coup de l'accès goutteux par la Colchicine, associé à l'alcalinisation
prudente des urines.
322
Exclusivement sur DOC - DZ : www.doc-dz.com NADJI 85
RESIDANAT EN POCHE TOME II
Cas Clinique en QCM

Dans le contexte décrit, il est indispensable pour surveiller et traiter correctement l'hypertension d'obtenir le
résultat de :
A - Dosages étagés de l'activité rénine au niveau de la veine cave et des veines rénales et hépatiques
B - Dosages étagés des catécholamines plasmatiques
C - Dosage périphérique de l'aldostérone
D - Electrocardiogramme
E - Mesure du débit cardiaque et des résistances périphériques
Bonne(s) réponse(s) : D

Les réponses A, B, C et E ne sont pas raisonnables dans le contexte.

Si d'autres examens complémentaires sont faits, vous vous attendez à trouver :


A - Hypocalcémie
B - Hyperphosphorémie
C - Hypocalciurie
D - Hypokaliémie
E - Hypochlorémie
Bonne(s) réponse(s) : A B C

A, B et C sont les conséquences de l'insuffisance rénale chronique.

Monsieur B. âgé de 58 ans est hospitalisé pour une douleur lombaire droite s'accompagnant d'hématurie intermittente. Une
urographie intraveineuse met en évidence une lacune radiotransparente dans le bassinet du rein droit.

Quels sont le(s) diagnostic(s) à envisager devant cette urographie ?


A - Calcul d'acide urique
B - Tumeur du bassinet
C - Caillot sanguin
D - Calcul muco-protéique
E - Calcul d'oxalate mono-hydraté
Bonne(s) réponse(s) : A B C D

Sans commentaire.

Parmi les examens suivants quel est ou quels sont ceux qui sont utiles au diagnostic différentiel ?
A - Une cytologie urinaire
B - Dosage uricurie
C - pH urinaire
D - Dosage oxalurie
E - Echographie rénale
Bonne(s) réponse(s) : A C E

A - Pour le diagnostic de tumeur du bassinet.


C - Pour la lithiase uratique (pH < 5).
E - Pour la lithiase.

En supposant que le diagnostic différentiel n'ait pu être établi par les examens précédemment cités,
l'identification de la nature de cette lacune pyélique pourrait être faite grâce à :
A - Artériographie rénale
B - Cystoscopie
C - Tomodensitométrie rénale
D - Scintigraphie rénale
E - Pyéloscopie
Bonne(s) réponse(s) : C E

Sans commentaire.

323
Exclusivement sur DOC - DZ : www.doc-dz.com NADJI 85
RESIDANAT EN POCHE TOME II
Cas Clinique en QCM

En supposant que cet homme soit porteur d'un calcul d'acide urique quels sont le ou les traitements possibles
chez cet homme qui est hypertendu et doit observer un régime peu salé :
A - Absorption de bicarbonate de sodium (6 à 8 g/j)
B - Absorption d'un autre alcalinisant (THAM)
C - Colchicine
D - Lithotritie extracorporelle
E - Diurétique thiazidique
Bonne(s) réponse(s) : A B

A - Le bicarbonate de sodium ne donne pas d'HTA, et si la surcharge en NaCl doit absolument être évitée, un diurétique
pourra être alors prescrit en association.

Dans le cas où les examens complémentaires seraient en faveur d'une tumeur maligne, quel(s) traitement(s)
préconisez-vous ?
A - Chimiothérapie
B - Radiothérapie externe
C - Néphrectomie élargie
D - Néphro-urétérectomie
E - BCG endocavitaire
Bonne(s) réponse(s) : D

Une seule réponse est possible, c'est la néphro-urétérectomie.

Madame R. 38 ans, est hospitalisée pour des oedèmes des membres inférieurs apparus un mois auparavant, associés à une
protéinurie. Elle se plaint d'asthénie et a pris 4 kgs en un mois. Récemment son médecin a découvert un nodule du sein
gauche pour lequel des explorations sont en cours. Depuis plusieurs années elle a de vagues douleurs articulaires pour
lesquelles elle a pris de multiples drogues dont elle ignore les noms. Il n'y a pas d'autre antécédent.
A l'examen clinique les oedèmes des membres inférieurs sont bilatéraux, mous, prenant le godet. Il n'y a pas de
splénomégalie, pas d'hépatomégalie, pas de circulation veineuse collatérale abdominale. La pression artérielle est de 140/80
mmHg. Les premiers examens montrent : protéinurie des 24 heures à 11 g - Protidémie 54 g/l - Albuminémie 18 g/l -
Créatinémie 100 micromol/l Examen cytologique urinaire : hématies 4 000/ml d'urine, leucocytes 2 000/ml d'urine.

Dans cette observation pour affirmer le diagnostic de syndrome néphrotique il manque le dosage sanguin :
A - Du cholestérol
B - Des triglycérides
C - Du calcium
D - Du complément total
E - Il ne manque aucun dosage
Bonne(s) réponse(s) : E

Sans commentaire.

S'il se confirmait que le nodule du sein était un cancer, quel serait le diagnostic de néphropathie glomérulaire
qu'il faudrait évoquer en premier ?
A - La néphropathie glomérulaire extra-membraneuse
B - La néphropathie glomérulaire à lésions minimes en microscopie optique
C - La hyalinose segmentaire et focale
D - La néphropathie glomérulaire proliférative endo-capillaire diffuse
E - L'amylose
Bonne(s) réponse(s) : A

Sans commentaire.

Le nodule du sein n'est pas un cancer. Dès lors, comme dans tout syndrome néphrotique on doit rechercher
chez cette femme comme cause possible :
A - Une maladie lupique
B - La prise d'aspirine
C - La prise de médicaments contenant de la phénacétine
D - La prise d'aldactone
E - La prise d'oestro-progestatifs
Bonne(s) réponse(s) : A

L'acide acétylsalicylique, la phénacétine, l'aldactone et les oestroprogestatifs ne donnent pas de syndrome néphrotique
(glomérulonéphrite extra-membraneuse).

324
Exclusivement sur DOC - DZ : www.doc-dz.com NADJI 85
RESIDANAT EN POCHE TOME II
Cas Clinique en QCM

Si le complément sérique était abaissé on devrait évoquer chez cette malade la possibilité d'un des diagnostics
suivants :
A - Maladie lupique
B - Péri-artérite noueuse
C - Amylose
D - Spondylarthrite ankylosante
E - Polyarthrite rhumatoïde
Bonne(s) réponse(s) : A

Sans commentaire.

Le complément sérique est normal. La biopsie rénale montre l'absence de lésion glomérulaire en microscopie
optique. Un traitement corticoïde est commencé. On donne de la prednisone (Cortancyl®) à la dose suivante les
premières semaines :
A - 0,10 mg/kg/jour
B - 0, 50 mg/kg/jour
C - 1 mg/kg/jour
D - 5 mg/kg/jour
E - 10 mg/kg/jour
Bonne(s) réponse(s) : C

Il faut néanmoins savoir qu'une glomérulonéphrite extra-membraneuse lupique ne s'accompagne pas le plus souvent
d'abaissement du complément sérique. Il manque aux données de la PBR, les résultats de l'étude en immuno-fluorescence.

Une femme âgée de 45 ans n'a pas d'autre antécédent qu'une hypertension artérielle apparue dans les deux derniers mois
d'une deuxième grossesse il y a 20 ans et un épisode de cystite à l'âge de 18 ans. Elle dit ne prendre aucun médicament et
observer un régime peu salé.

Elle se plaint de brusques accès de fatigue.


Un électrocardiogramme a montré un allongement de QT.
Un ionogramme sanguin est pratiqué :
- Bicarbonates totaux 38 mmol/l
- Sodium 140 mmol/l
- Chlore 102 mmol/l
- Potassium 2,6 mmol/l
- Créatinine 91 micromol/l
L'examen clinique est négatif.
Les réflexes ostéotendineux sont présents. La pression artérielle est à 117/70 mmHg sans baisse significative à la mise en
orthostatisme. Il n'y a ni oedème, ni protéinurie.

Quel signe électrocardiographique compatible avec le tableau décrit est fréquemment observé ?
A - Retard de déflexion intrinsécoïde
B - Onde T aplatie ou inversée
C - Hypertrophie ventriculaire gauche
D - Episode de tachycardie sinusale
E - Augmentation de voltage de l'onde P
Bonne(s) réponse(s) : B

Sans commentaire.

Le lendemain, la patiente a marché rapidement pour se rendre à votre consultation. Vous refaites l'examen
clinique et vous pouvez trouver :
A - Des réflexes rotuliens polycinétiques
B - Une perte du sens de la position
C - Des vertiges de type labyrinthique
D - Un trismus
E - Des signes de tétanie (signe de Chvosteck)
Bonne(s) réponse(s) : E

Sans commentaire.

325
Exclusivement sur DOC - DZ : www.doc-dz.com NADJI 85
RESIDANAT EN POCHE TOME II
Cas Clinique en QCM

Pour expliquer le trouble ionique présenté, vous recherchez chez la patiente :


A - Prise excessive de gel d'alumine pour douleurs gastriques
B - Consommation excessive de laitages
C - Consommation excessive de légumes
D - Prise de médicaments anti-aldostérone
E - Prise de médications laxatives
Bonne(s) réponse(s) : E

E - Les laxatifs sont pour beaucoup d'entre eux, hormis l'huile de paraffine et le lactulose, responsables d'hypokaliémie parfois
sévère.

Vous décidez de faire une supplémentation potassique. Vous augmentez dans la ration de préférence :
A - Le poisson
B - Le gruyère
C - L'eau minérale alcaline
D - Les agrumes
E - Le pain
Bonne(s) réponse(s) : D

D - Plus généralement les fruits sont riches en potassium.

Malgré cet apport supplémentaire et l'absence de troubles digestifs avoués, la kaliémie reste au-dessous de 3
mmol/l. Le diagnostic probable est :
A - Achylie gastrique
B - Adénome corticosurrénal (syndrome de Conn)
C - Tumeur sécrétante de rénine
D - Tubulopathie avec fuite potassique
E - Médullosurrénalome
Bonne(s) réponse(s) : B

Sans commentaire.

Les examens suivants ont été pratiqués : aldostérone plasmatique, huit fois la valeur normale, activité rénine
plasmatique, 20 fois la valeur normale. Une artériographie rénale ne montre ni lésion artérielle, ni
parenchymateuse. Deux diagnostics restent possibles parmi les suivants :
A - Hyperplasie corticosurrénalienne
B - Tubulopathie : syndrome de Bartter
C - Thrombose veineuse rénale
D - Fibrose rétropéritonéale
E - Prise de diurétique : hydrochlorothiazide (Esidrex®) à visée "esthétique"
Bonne(s) réponse(s) : A E

A - N'est pas possible car l'activité rénine plasmatique doit être effondrée lorsqu'il existe un hyperaldostéronisme primaire.
B - Le syndrome de Bartter associe hypokaliémie et hyperréninémie.
E - S'accompagne d'un hyperaldostéronisme secondaire à la déshydratation extra-cellulaire liée au diurétique.

Les éléments suivants peuvent être considérés en faveur d'une tubulopathie :


A - Glycosurie avec glycémie normale
B - Protéinurie thermosoluble
C - Augmentation du rapport potassium excrété/potassium filtré
D - Très faible natriurèse
E - Hypophosphorémie avec clairance du phosphore élevée
Bonne(s) réponse(s) : A E

Sans commentaire.

326
Exclusivement sur DOC - DZ : www.doc-dz.com NADJI 85
RESIDANAT EN POCHE TOME II
Cas Clinique en QCM
Un homme de 48 ans, présente brusquement un syndrome douloureux du flanc droit, avec irradiation dans le testicule.
Sa température est à 37,2°C.
Un épisode identique mais moins intense a eu lieu trois ans auparavant. On ne retrouve par ailleurs aucun antécédent sur le
plan personnel ou familial. Le diagnostic de colique néphrétique est le plus vraisemblable.

Pour étayer votre diagnostic vous demandez après avoir soulagé votre malade une urographie intraveineuse,
quel libellé vous paraît le meilleur ?
A - U.I V. avec compression
B - U.I.V. minutée avec clichés précoces
C - U.I.V. avec wash-out
D - U.I.V. avec clichés tardifs
E - U.I V. standard
Bonne(s) réponse(s) : D

D - En effet, s'il existe un important retard de sécrétion à droite, seuls les clichés tardifs vont montrer l'obstacle urinaire (siège,
nature).

On constate l'existence d'une lithiase transparente de la taille d'une cerise siègeant dans le bassinet droit sans
distension marquée des cavités sus-jacentes. Quelle est la nature chimique probable de ce calcul ?
A - Oxalo-calcique
B - Phosphoamoniaco-magnésienne
C - Urique
D - Cystinique
E - Phospho-calcique
Bonne(s) réponse(s) : C

Sans commentaire.

Quelle attitude devra-t-on adopter vis-à-vis de cette lithiase ?


A - Réaliser rapidement une pyélotomie
B - Attendre une migration spontanée
C - Proposer une alcalinisation prolongée
D - Mettre le malade sous diurétiques
E - Monter une sonde urétérale
Bonne(s) réponse(s) : C

C - L'alcalinisation va solubiliser le calcul uratique, à condition que le pH urinaire soit maintenu en permanence entre 7 et 8.

Quelle(s) complication(s) évolutive(s) amènerai(en)t à modifier l'attitude précédente ?


A - Hématurie
B - Expulsion de débris de calculs
C - Infection urinaire avec fièvre à 39°
D - Distension pyélo-calicielle
E - Leucocyturie augmentée
Bonne(s) réponse(s) : C D

QUESTION ANNULEE.
Dans les deux cas (C et D), il y a un risque majeur de pyélonéphrite, de septicémie, voire de choc infectieux en particulier à
bacille Gram négatif.

Pour éviter (en cas d'hyperuricémie) la récidive lithiasique chez ce patient vous allez proposer en dehors des
cures de diurèse :
A - Diurétiques thiazidiques
B - Uricoéliminateurs
C - Prescription d'un inhibiteur de la xanthine oxydase
D - Désinfection urinaire continue
E - Alcalinisation
Bonne(s) réponse(s) : C E

C - Zyloric® = allopurinol.

327
Exclusivement sur DOC - DZ : www.doc-dz.com NADJI 85
RESIDANAT EN POCHE TOME II
Cas Clinique en QCM
Madame B. Pauline, 48 ans, consulte pour une hématurie. Elle aurait présenté il y a cinq ans une symptomatologie du même
type : hématurie macroscopique indolore, totale, se poursuivant pendant plusieurs jours. Pendant les intervalles d'urines
claires, la bandelette Labstix met en évidence sang ++. A la suite de plusieurs épisodes de ce type, le malade a subi une
néphrectomie gauche il y a cinq ans. Le dossier a été entièrement perdu dans un incendie des archives hospitalières. La
patiente ne se plaint de rien, elle est restée sans surveillance pendant quatre ans. Elle est inquiète, à la reprise des
hématuries, depuis trois mois. Dans le choix des investigations, la localisation de l'origine des hématuries est importante.

A la cytologie urinaire, vous retenez en faveur d'une origine vésicale :


A - L'absence de cylindres granuleux et hématiques
B - Le caractère indemne des hématies
C - La présence d'une protéinurie à plus de 1 g/24h
D - La leucocyturie importante
E - L'absence de cellules de desquamation
Bonne(s) réponse(s) : A B D

Sans commentaire.

La cytologie vous oriente vers une origine vésicale. Quel est alors le moyen d'investigation décisif de cette
origine ?
A - Artériographie pelvienne
B - Tomodensitométrie
C - Lymphographie
D - Echographie
E - Cystoscopie
Bonne(s) réponse(s) : E

Sans commentaire.

Un des examens ci-dessus vous apporte la preuve d'une lésion vésicale étendue de type papillomateux. Il est
probable que la cause de la néphrectomie initiale ait été :
A - Une lithiase rénale
B - Un angiome rénal
C - Une tuberculose rénale
D - Une tumeur du bassinet
E - Un angiomyolipome
Bonne(s) réponse(s) : D

A - Est le plus souvent dangereuse.


B - Un angiome rénal n'aurait théoriquement pas entraîné de néphrectomie.
C - Aurait conduit à un traitement anti-tuberculeux.
E - N'est pas une indication opératoire.
Surtout, on peut imaginer une double localisation d'une tumeur urothéliale.

Vous apprenez que la néphrectomie pratiquée a comporté une urétérectomie. Vous estimez que, dans cette
observation, cette intervention a été pratiquée :
A - Pour éviter un reflux dans le moignon urétéral restant
B - Pour éviter la greffe de cellules malignes
C - Pour améliorer la contractilité de l'uretère restant
D - Pour prévenir la survenue de douleurs pelvienne
E - Pour éviter une hypertension secondaire
Bonne(s) réponse(s) :

QUESTION ANNULEE.

On peut estimer qu'il y a eu dans ce cas un défaut de surveillance postopératoire. Il aurait du être pratiqué, six
mois à un an après, l'examen suivant :
A - Phlébographie rénale droite
B - Scintigraphie pulmonaire
C - Echographie péricardique
D - Biopsie de la plèvre diaphragmatique gauche
E - Cystoscopie
Bonne(s) réponse(s) : E

E - Dans l'optique de rechercher une deuxième localisation maligne urothéliale (cytoscopie comportant des frottis).

328
Exclusivement sur DOC - DZ : www.doc-dz.com NADJI 85
RESIDANAT EN POCHE TOME II
Cas Clinique en QCM
M. Z. , 56 ans, agent d'assurance, vient consulter pour des hématuries d'apparition récente. C'est la veille que le malade a
constaté, en allant aux toilettes, que ses urines étaient uniformément rouge foncé, et, depuis, toutes ses mictions indolores et
normalement espacées sont colorées, avec quelques caillots; M. Z. est très inquiet, mais ne décrit aucun autre trouble
fonctionnel. Dans les antécédents vous notez :
- une appendicectomie à 20 ans
- une crise de colique néphrétique droite à 42 ans suivie de l'expulsion d'un petit calcul d'acide urique
- à 52 ans une artérite des membres inférieurs ayant nécessité un pontage ilio-fémoral, avec un résultat fonctionnel excellent.
Depuis l'intervention, il prend régulièrement des antivitamines K (Sintrom® : 0,5 à 1,5 c/j)
- il s'agissait d'un grand fumeur qui a définitivement arrêté de fumer après l'intervention.
A l'examen clinique :
- l'état général est floride
- les urines recueillies dans deux verres, en début de consultation, sont uniformément colorées de sang ;
- l'abdomen est souple : il n'y a pas de masse perceptible abdominale ou lombaire.
- les organes génitaux externes sont normaux.
- au TR, la prostate est ferme, de taille et de consistance normales
- fréquence cardiaque 70, TA 18/10 cm Hg M. Z. apporte le résultat de ses derniers examens sanguins effectués l'avant veille
de l'apparition du saignement : taux de prothrombine 43 %, urée sanguine 6 mmol/l, créatinine 90 micromol/l, hématies 5 800
000/mm3, leucocytes 6500/mm3, hémoglobine 14,6 g/dl, glycémie 5 mmol/l.

Quel élément clinique prouve qu'il s'agit bien d'une hématurie ?


A - Les urines rouges sont émises pendant la miction
B - Les mictions sont indolores
C - La coloration persiste à ébullition
D - La présence de caillots
E - Les urines sédimentent au repos
Bonne(s) réponse(s) :

QUESTION ANNULEE.

Vous pouvez envisager comme hypothèse(s) diagnostique(s) ?


A - Hypocoagulabilité excessive sous anticoagulants
B - Maladie de Berger
C - Tumeur de vessie
D - Calcul urinaire
E - Cancer du rein
Bonne(s) réponse(s) :

QUESTION ANNULEE.

L'UIV ayant montré une image d'encorbellement du groupe caliciel inférieur et une déformation du pôle inférieur
du rein, vous prescrivez une échographie qui montre une masse vide d'écho, à contours réguliers et à
renforcement postérieur. Quel examen demandez-vous ?
A - Une angiographie numérisée
B - Une cystoscopie
C - Une biopsie percutanée de la masse rénale
D - Une urétéropyélographie rétrograde
E - Une cavographie
Bonne(s) réponse(s) :

QUESTION ANNULEE.

Ces examens ne retrouvant rien de plus que l'échographie, il est alors logique de :
A - Faire une biopsie prostatique
B - Rechercher régulièrement une hématurie microscopique
C - Rassurer le malade et ne pas pousser plus loin les investigations dans l'immédiat
D - Refaire une nouvelle urographie au bout de 3 mois
E - Arrêter le traitement anticoagulant
Bonne(s) réponse(s) :

QUESTION ANNULEE.

329
Exclusivement sur DOC - DZ : www.doc-dz.com NADJI 85
RESIDANAT EN POCHE TOME II
Cas Clinique en QCM
Carine, âgée de 3 ans, est hospitalisée pour un oedème des paupières évoluant depuis 8 jours, sans fièvre, sans altération de
l'état général. A l'interrogatoire, on apprend que les vaccinations ont été faites sans problème (Tetracoq et ROR). Depuis 6
mois, Carine a fait 3 épisodes de rhinopharyngite non compliquée. Les parents sont en bonne santé. A l'examen clinique, on
note une taille de 95 cm, un poids de 16 kg (14 kg un mois auparavant). Il existe un oedème de paupières et des 2 membres
inférieurs prenant le godet. L'examen pulmonaire est normal de même que l'examen abdominal. L'auscultation cardiaque est
normale ; la tension artérielle est à 9/5 cm Hg. Le reste de l'examen clinique est normal.
Sur le plan biologique, on constate :
- une VS à 90 mm à la première heure
- une protidémie à 46 g/l avec une augmentation des alpha 2 et des bêta-globulines, effondrement des gammaglobulines,
albuminémie à 20 g/l.
- une créatininémie à 60 micromoles/l
- une natrémie à 128 millimoles/l
- une kaliémie à 4,6 millimoles/l
- une calcémie à 2,01 millimoles/l
- la cholestérolémie est à 15,1 mmol/l (N=4,60-7)
- le taux du complément est normal.
Dans les urines, on découvre une protéinurie à 4 g/24 h. une natriurèse inférieure à 2 mmol/l, une kaliurèse à 110 mmol/l. La
protéinurie est sélective. L'ECBU révèle 800 leucocytes avec 50 000 hématies/ml. Il n'y a pas de germe en culture.
La radiographie du thorax et de l'abdomen sans préparation sont normaux.

Dans cette observation, quels sont les arguments qui entrent dans la définition du syndrome néphrotique ?
A - Protidémie à 46 g/l
B - Protéinurie à 4 9/24 h
C - Sélectivité de la protéinurie
D - Elévation de la VS
E - Natrémie basse
Bonne(s) réponse(s) : A B

Sans commentaire.

Un élément essentiel du tableau clinique de syndrome néphrotique n'a pas été indiqué, lequel ?
A - Oedème des séreuses
B - Oligurie
C - Douleurs abdominales
D - Troubles de la conscience
E - Anorexie
Bonne(s) réponse(s) : B

L'oligurie est la conséquence de l'hyperaldostéronisme secondaire, observé au cours du syndrome néphrotique.

Quel est ou quels sont les arguments en faveur d'un syndrome néphrotique pur ?
A - TA normale
B - Cholestérolémie élevée
C - Hypocalcémie
D - Natriurèse effondrée
E - Créatininémie normale
Bonne(s) réponse(s) : A E

Sans commentaire.

Les perturbations du ionogramme sanguin et urinaire témoignent :


A - D'une hémoconcentration
B - D'une sécrétion inappropriée d'ADH
C - D'un hyperaldostéronisme secondaire
D - D'une insuffisance rénale organique
E - D'une perte du pouvoir de concentration des urines
Bonne(s) réponse(s) : C

Sans commentaire.

330
Exclusivement sur DOC - DZ : www.doc-dz.com NADJI 85
RESIDANAT EN POCHE TOME II
Cas Clinique en QCM

Le traitement initial de ce cas comprendra :


A - Cortancyl 5 mg : 7 comprimés un jour sur deux
B - Cortancyl 5 mg : 7 comprimés tous les jours
C - Aldactone 50 mg : 2 comprimés par jour
D - Régime normal
E - Sirop de Potassium : 3 cuillères à café par jour
Bonne(s) réponse(s) : A C

A - Chez l'enfant, la prescription de Cortancyl pour un syndrome néphrotique est de 2 mg/kg, un jour sur deux.
C - Le régime désodé et les diurétiques de la classe des anti-aldostérone sont prescrits conjointement, car il faut diminuer le
plus possible le syndrome oedémateux.

Une fillette de 4 ans est adressée pour une incontinence des urines.
Malgré l'incontinence elle présente des mictions normales. Au cours des deux premières années de la vie, plusieurs poussées
fébriles à 40°, se sont produites. Sa mère a remarqué dans les couches des traces de pus fréquentes. L'examen des fosses
lombaires et de l'abdomen est négatif.
La vulve est rouge, un peu de pus fait issue par le vagin. On ne remarque aucune fuite d'urine à la toux.
L'examen des urines montre une bactériurie à colibacilles.
L'urographie intra-veineuse révèle à droite un rein normal, à gauche une masse qui s'opacifie légèrement au cours de
l'examen et qui refoule en bas et en dehors un rein dont la sécrétion est normale mais dont le pôle supérieur est amputé et
auquel il manque le système caliciel supérieur. L'uretère de ce rein présente un trajet en baïonnette dans sa portion lombaire
basse. La vessie ne présente pas de signes de lutte. La miction s'effectue par un urèthre normal. Il n'y a pas de résidu post-
mictionnel.

Indiquez la cause vraisemblable de ce type d'incontinence :


A - Un néphroblastome avec métastase vertébro-médullaire
B - Une duplicité avec urétérocèle
C - Une duplicité avec abouchement ectopique extravésical de l'uretère
D - Un reflux vésico-rénal avec destruction du pôle supérieur du rein
E - Une dysplasie uréthro-trigonale congénitale associée à un kyste du rein
Bonne(s) réponse(s) : C

Sans commentaire.

Parmi les explorations ci-dessous, il est essentiel de pratiquer pour confirmer le diagnostic :
A - La cystographie rétrograde
B - La cystoscopie et la vaginoscopie
C - L'artériographie rénale
D - L'étude urodynamique de la vessie
E - L'échographie rénale et vésicale
Bonne(s) réponse(s) : B E

Une I.R.M. serait l'examen de choix pour compléter l'échographie rénale et vésicale.

Parmi les lésions énumérées ci-dessous, indiquez celle qui est le plus souvent la cause d'une fuite d'urine chez
l'enfant avant l'âge de 4 ans :
A - Le myélo-méningocèle
B - Les valves de l'urètre postérieur
C - L'abouchement ectopique de l'uretère
D - La dysplasie uréthrocervico-trigonale
E - L'instabilité vésicale par immaturité neurologique
Bonne(s) réponse(s) : E

Sans commentaire.

Indiquez, parmi les suivantes, la ou les anomalie(s) caractéristique(s) ou fréquente(s) qui peu(vent) s'associer
aux duplicités complètes de la voie excrétrice :
A - Le rein en fer à cheval
B - Le reflux vésico rénal
C - La cryptorchidie
D - L'urétérocèle
E - L'atrésie ano-rectale
Bonne(s) réponse(s) : B D

Sans commentaire.

331
Exclusivement sur DOC - DZ : www.doc-dz.com NADJI 85
RESIDANAT EN POCHE TOME II
Cas Clinique en QCM

Dans le cadre de l'observation proposée, indiquez l'attitude thérapeutique que vous préconisez :
A - Une kinésithérapie périnéale avec rééducation vésicale
B - Une néphrectomie élargie gauche
C - Une réimplantation urétérovésicale avec un mécanisme antireflux
D - L'ablation chirurgicale d'un pyélon supérieur surnuméraire
E - La prescription pharmacologique de modificateurs du comportement vésical
Bonne(s) réponse(s) : D

Sans commentaire.

Un homme de 26 ans, sans antécédents, présente une douleur brutale de la fosse iliaque droite et du flanc droit, avec
nausées, sans trouble mictionnel, ni élévation de la température.

Bien que n'ayant pas de valeur spécifique, quel signe d'examen évoque une colique néphrétique droite ?
A - Douleur au toucher pelvien
B - Douleur à la palpation abdominale controlatérale
C - Ebranlement douloureux lombaire droit
D - Défense abdominale droite
E - Aucune des réponses précédentes
Bonne(s) réponse(s) : E

Sans commentaire.

Quel examen permet le diagnostic étiologique au décours de la crise ?


A - Echographie rénale
B - Radiographie d'abdomen sans préparation
C - Urographie intraveineuse
D - Urétéro-pyélographie rétrograde
E - Scintigraphie rénale
Bonne(s) réponse(s) : C

En sachant que s'il s'agit de la migration d'un microcalcul, l'UIV peut être normale au décours de la crise.

Le diagnostic d'obstacle par un calcul de 5 mm de diamètre au niveau de l'uretère lombaire droit a été posé ;
vous décidez de :
A - Prescrire une antibiothérapie
B - Instituer immédiatement une cure de diurèse
C - Opérer d'urgence
D - Monter une sonde urétérale par voie endoscopique
E - Prescrire une restriction hydrique et des anti-inflammatoires
Bonne(s) réponse(s) : B E

B - Si le patient ne souffre plus, il faut l'inciter à boire abondamment.


E - S'il souffre de nouveau, il faut le mettre en restriction hydrique pour éviter la distension des cavités pyélocalicielles. Dans
les deux cas, la prescription d'anti-inflammatoire est très utile pour diminuer l'oedème urétéral autour du calcul et faciliter son
élimination.

Quel(s) examen(s) biologique(s) demander à titre de premier bilan ?


A - Numération formule et myélogramme
B - Cytologie urinaire en contraste de phase
C - Examen cytobactériologique des urines
D - Uricémie
E - Calcémie et phosphorémie
Bonne(s) réponse(s) : C D E

Sans commentaire.

332
Exclusivement sur DOC - DZ : www.doc-dz.com NADJI 85
RESIDANAT EN POCHE TOME II
Cas Clinique en QCM

Dans quel(s) cas l'indication chirurgicale devient-elle nécessaire :


A - Hématurie microscopique persistante
B - Poussées fébriles
C - Distention persistante des cavités
D - Douleurs quotidiennes après 3 mois d'évolution
E - Mictions impérieuses
Bonne(s) réponse(s) : B C D

Sans commentaire.

Mr. A âgé de 55 ans, a eu une hématurie macroscopique à l'âge de 36 ans, A cette occasion a été découverte une
polykystose rénale. Il n'a depuis eu aucune surveillance.
Depuis quelques semaines il se sent fatigué et nauséeux, La pression artérielle est de 170-110 mm Hg. On palpe deux gros
reins bosselés. Les examens complémentaires montrent les résultats suivants, dans le sang veineux :

- Urée : 33 mmol/l - Créatinine : 650 micromol/l


-K : 4,9 mmol/l - Na : 138 mmol/l
- CI : 108 mmol/l - C02 total : 21 mmol/l
- Ca : 1,80 mmol/l - Phosphore : 2,4 mmol/l
- Protides : 70 g/l - Hémoglobine : 13 g/100 ml

Chez ce malade un ou plusieurs mécanismes contribuent à l'hypocalcémie. Lequel ou lesquels ?


A - Un défaut de synthèse du 1-25 (OH)2 D3
B - Un défaut d'absorption digestive du calcium
C - Une hypercalciurie
D - L'hyperphosphorémie
E - L'hyperchlorémie
Bonne(s) réponse(s) : A B

Au cours de l'insuffisance rénale chronique, il existe un défaut de synthèse de 1-25 vitamine D, qui est la forme active de la
vitamine D, responsable de l'absorption active digestive du calcium.

Les anomalies phospho-calciques, si elles ne sont pas corrigées, risquent de conduire à deux complications
osseuses principales :
A - Ostéomalacie
B - Ostéoporose
C - Ostéosclérose
D - Ostéite fibreuse due à l'hyperparathyroïdie
E - Maladie de Paget de l'os
Bonne(s) réponse(s) : A D

A - Le défaut de vitamine D conduit à l'ostéomalacie.


D - L'hyperphosphorémie et l'hypocalcémie entraînent une hyperparathyroïdie secondaire.

Le traitement au long cours à mettre en oeuvre chez ce malade pour corriger les anomalies phospho-calciques
comporte :
A - Apport per os de carbonate de calcium
B - Apport intraveineux de gluconate de calcium
C - Apport per os de phosphate d'alumine
D - Apport per os d'hydroxyde d'alumine
E - Apport per os de métabolites de la vitamine D
Bonne(s) réponse(s) : A D E

A - Le carbonate de Ca corrige partiellement l'hypocalcémie et l'hyperphosphorémie.


D - L'hydroxyde d'aluminium corrige l'hyperphosphorémie.
E - La 25 OH Vit. D (Dédrogyl®), la Un Alpha® et la 1-25 OH Vit. D ( Rocaltrol®) corrigent l'hypocalcémie. Les 2 derniers
corrigent l'hyperparathyroïdie secondaire.

L'absence d'anémie est sans doute expliquée par :


A - L'insuffisance rénale chronique
B - L'acidose chronique
C - L'hémoconcentration
D - La polykystose
E - L'hypertension artérielle
Bonne(s) réponse(s) : D

Sans commentaire.

333
Exclusivement sur DOC - DZ : www.doc-dz.com NADJI 85
RESIDANAT EN POCHE TOME II
Cas Clinique en QCM
Madame Josette C., 62 ans, n'a pas de passé pathologique, et a été convoquée il y a six mois pour un bilan de santé qui n'a
pas montré d'anomalie.
Elle continue à vendre ses fromages sur le marché.
Un matin, elle fait une chute de son escabeau, en essayant de changer une ampoule au plafond. Elle a ressenti sur le
moment, une violente douleur lombaire, mais le bilan radiologique osseux est resté négatif. Une semaine après, apparaissent
de très volumineux oedèmes symétriques remontant jusqu'aux cuisses, indolores, et gardant le godet. Il persiste un
endolorissement lombaire. Dans les urines : protéinurie ++++, sang : traces à la bandelette labstix. Créatinine plasmatique 82
micromol/l (normale), pression artérielle 126/74 mmHg.

A ce stade, choisissez le diagnostic le plus compatible avec les faits énoncés :


A - Rupture rénale incomplète
B - Rupture de l'intima de l'artère rénale
C - Blessure urétérale
D - Syndrome néphrotique aigu
E - Thrombose des veines sus-hépatiques
Bonne(s) réponse(s) : D

Un traumatisme rénal ou du pédicule rénal n'expliquerait pas des oedèmes des membres inférieurs associés à une protéinurie
abondante. L'apparition du syndrome néphrotique est indépendante de la chute.

Dans la clinique où elle a été hospitalisée, l'urographie intra veineuse montre deux gros reins fonctionnels.
Parmi les examens suivants, lequel vous parait le plus adéquat à ce stade pour faire progresser le diagnostic ?
A - Immuno-électrophorèse des urines
B - Dosage du complément sérique et de la fraction c3
C - Recherche et dosage d'immuns complexes circulants
D - Angiographie digitalisée
E - Phlébographie cavorénale
Bonne(s) réponse(s) : B

Le dosage du complément est utile pour faire le diagnostic de lupus, mais ce n'est sûrement pas le premier examen à faire.

Au vu des résultats des examens choisis, plusieurs propositions thérapeutiques sont avancées. Vous choisiriez :
A - Traitement fibrinolytique (urokinase) ou anticoagulant
B - Embolisation de l'artère rénale
C - Mise en place d'un drain de néphrostomie par ponction du bassinet sous échographie
D - Corticothérapie en flash (1 9 de Prednisone chaque jour pendant 15 jours)
E - Traitement immunosuppresseur, chloraminophène ou cyclosporine
Bonne(s) réponse(s) :

QUESTION ANNULEE.

Après trois mois d'évolution, sous traitement le tableau clinique et biologique est le suivant : oedèmes
importants remontant jusqu'aux lombes, protéinurie 24 g/24 h hématurie microscopique discrète, albuminémie
18 gr 0/00, pas d'insuffisance rénale.
Une biopsie rénale étant pratiquée, quel type de lésion peut-on s'attendre à trouver ?
A - Glomérulonéphrite proliférative endocapillaire
B - Glomérulonéphrite intercapillaire à dépôts d'IgA
C - Vascularite avec lésions nécrosantes
D - Glomérulonéphrite extramembraneuse
E - Lésions glomérulaires minimes
Bonne(s) réponse(s) : D

La glomérulonéphrite extramembraneuse est le plus souvent corticorésistante.

Devant une protéinurie massive de ce type avec oedème, un certain nombre de complications sont prévisibles.
Laquelle ou lesquelles peuvent survenir dans ce contexte ?
A - Embolie pulmonaire
B - Insuffisance cardiaque
C - Choc hypovolémique
D - Ascite et épanchement pleural
E - Hypolipémie importante
Bonne(s) réponse(s) : A C D

QUESTION ANNULEE.
Le syndrome néphrotique expose aux complications thromboemboliques, à l'hypovolémie par hypoalbuminurie et aux
épanchements séreux.

334
Exclusivement sur DOC - DZ : www.doc-dz.com NADJI 85
RESIDANAT EN POCHE TOME II
Cas Clinique en QCM

La patiente a reçu une corticothérapie à 2 mg/kg/j qui est restée inefficace. La patiente étant hospitalisée
demande à être soulagée de ses oedèmes. Quelle solution thérapeutique vous parait à la fois sans danger et
efficace, ajoutée au repos et à la restriction sodée ?
A - Dialyse péritonéale avec liquide hyposmotique
B - Séance de rein artificiel courte avec ultrafiltration massive
C - Administration de digitalique et d'anti-inflammatoire non stéroïdien
D - Perfusions d'albumine et de mannitol
E - Cure déclive avec anti-agrégant plaquettaire
Bonne(s) réponse(s) : E

QUESTION ANNULEE.
La surélévation des pieds va entraîner une diminution des oedèmes des membres inférieurs. Le traitement anti-agrégant
plaquettaire peut diminuer le risque thrombo-embolique.

La patiente a été traitée. Au bout de dix ans, la créatinine est à 125 micro- moles par litre, la protéinurie est à
3,5 g/24 heures, les oedèmes sont discrets, sans traitement. Devant cette notion évolutive, quel est votre choix
diagnostique ? (il peut être différent ou identique du diagnostic initial).
A - Rupture rénale incomplète
B - Rupture intima des artères rénales
C - Blessure, puis sténose urétérale
D - Syndrome néphrotique
E - Thrombose des veines sus-hépatiques
Bonne(s) réponse(s) : D

La persistance du syndrome néphrotique après 10 ans d'évolution et l'apparition d'une petite insuffisance rénale sont très en
faveur d'une glomérulonéphrite extramembraneuse.

Un homme de 40 ans, célibataire, consulte pour une grosse bourse droite ; il rapporte que l'augmentation de volume a été
progressive en deux à trois mois, sans douleur et sans fièvre. L'examen clinique attentif permet de bien différencier
l'épididyme et d'affirmer qu'il s'agit d'une tumeur testiculaire déformant le testicule droit qui est deux fois plus gros que le
gauche. L'examen clinique général est normal.

Parmi les éléments suivants quel est celui qui est considéré comme facteur de risque de cancer du testicule ?
A - Cancer testiculaire chez le père
B - Antécédent d'oreillons
C - Antécédent d'irradiations
D - Antécédent de cryptorchidie
E - Antécédent de traumatisme testiculaire
Bonne(s) réponse(s) : D

Sans commentaire.

La fréquence du cancer testiculaire est par rapport à l'ensemble des cancers masculins de :
A - Environ 1%
B - Environ 5%.
C - Environ 10%.
D - Environ 15%.
E - Environ 20%.
Bonne(s) réponse(s) : A

Sans commentaire.

Quel (s) est (sont) le (s) marqueur (s) biologique(s) le (s) plus souvent rencontré(s) chez un sujet présentant un
cancer du testicule ?
A - Alpha-foetoprotéine
B - Thyrocalcitonine
C - Substance ACTH like
D - Fraction bêta de l'hormone gonadotrophine chorionique
E - Antigène carcino-embryonnaire
Bonne(s) réponse(s) : A D

Sans commentaire.

335
Exclusivement sur DOC - DZ : www.doc-dz.com NADJI 85
RESIDANAT EN POCHE TOME II
Cas Clinique en QCM

Le premier relais ganglionnaire du testicule est :


A - Lombo-aortique
B - lliaque interne
C - lliaque externe
D - Obturateur
E - Inguinal
Bonne(s) réponse(s) : A

Sans commentaire.

L'une des formes histologiques de cancer du testicule est plus radiosensible que les autres et régulièrement
radiocurable :
A - Le choriocarcinome
B - Le séminome pur
C - Le carcinome embryonnaire pur
D - L'association carcinome embryonnaire-séminome
E - Le tératome bien différencié
Bonne(s) réponse(s) : B

Sans commentaire.

Les chances de guérison définitive après traitement d'un malade atteint d'un séminome du testicule au stade I
sont de l'ordre de :
A - 20 %.
B - 40 %.
C - 60 %.
D - 80%.
E - 100 %.
Bonne(s) réponse(s) : E

E - 95 %.

336
Exclusivement sur DOC - DZ : www.doc-dz.com NADJI 85
RESIDANAT EN POCHE TOME II
Cas Clinique en QCM

337
Exclusivement sur DOC - DZ : www.doc-dz.com NADJI 85
RESIDANAT EN POCHE TOME II
Cas Clinique en QCM

338
Exclusivement sur DOC - DZ : www.doc-dz.com NADJI 85
RESIDANAT EN POCHE TOME II
Cas Clinique en QCM

339
Exclusivement sur DOC - DZ : www.doc-dz.com NADJI 85
RESIDANAT EN POCHE TOME II
Cas Clinique en QCM
Un jeune couple vous amène pour sa première visite systématique, un garçon âgé de trois semaines, dont ils souhaitent vous
confier la surveillance médicale. Il s'agit du premier enfant, la grossesse et l'accouchement ont été normaux; les mensurations
à la naissance étaient les suivantes : poids = 3100 gg, taille = 50 cm, périmètre crânien = 35 cm. Le test d'Apgar était à 10.
Depuis la naissance, l'enfant est nourri au sein. Les mensurations actuelles sont : poids = 3 600 g, taille = 51,5 cm, périmètre
crânien := 36 cm. L'examen clinique est normal. Les parents vous soumettent toute une série d'interrogations concernant leur
enfant pour lesquelles ils n'ont pas reçu d'informations jusqu'à présent.

Quelle doit être la prise de poids quotidienne moyenne, actuelle de ce nourrisson ?


A - 10 à 15 grammes
B - 15 à 20 grammes
C - 25 à 30 grammes
D - 30 a 40 grammes
E - Plus de 40 grammes
Bonne(s) réponse(s) : C

A savoir (question souvent posée).

Après avoir expliqué l'intérêt de prévenir le rachitisme, vous rédigez l'ordonnance. Parmi ces prescriptions,
quelles sont celles adaptées à ce nourrisson. Pour assurer la prophylaxie au rachitisme chez ce nourrisson, la
posologie quotidienne de vitamine D à lui donner est de :
A - 200 U.I.
B - 1000 U I.
C - 500 U.I.
D - 10 000 U.I.
E - 20 000 U.I.
Bonne(s) réponse(s) : B

Classiquement 800 à 2 000 U.I, suivant la coloration de la peau et l'exposition au soleil, en moyenne 1200 U.I = 3 gouttes de
stérogyl®.

Quelle sera la vaccination à pratiquer à trois mois et demi ?


A - Vaccin antivariolique
B - BCG
C - Vaccin antirougeoleux
D - Vaccin D.T.C.P.
E - Vaccin anticoquelucheux
Bonne(s) réponse(s) : D

A - N'est plus recommandé


B - Possible mais légalement : avant l'entrée en collectivité
C - Après 12 mois ; rarement seul le plus souvent associé au D.T.P : item D

Afin de prévenir les risques de toute hyperthermie supérieure à 39°, quelle(s) thérapeutique(s) peu(vent) être
réalisée(s) par la mère en attendant la visite du médecin ?
A - Administration de diazepam
B - Bain à 37°
C - Administration d'acide acétyl-salicylique
D - Administration de paracétamol
E - Administration d'antibiotiques
Bonne(s) réponse(s) : B C D

A - Un peu litigieux (peut être utilisé en cas d'antécédent de convulsion)


B - En fait 2 degrés en dessous de la température rectale
E - Un antibiotique n'est pas un antipyrétique
N.B : la question est t"raitement de l'hyperthermie" et non pas "prévention des convulsions" (le valium n'est pas un
antithermique)

340
Exclusivement sur DOC - DZ : www.doc-dz.com NADJI 85
RESIDANAT EN POCHE TOME II
Cas Clinique en QCM
Vous voyez en consultation un enfant de trois ans, amené par ses parents, pour une tumeur du flanc droit, découverte la
veille. La tumeur est postérieure, non mobilisable, elle mesure 8 x 10 cm, elle ne souffle pas, elle n'est pas pulsatile. Le reste
de l'examen somatique est normal, le bilan paraclinique ne montre pas d'anomalie à la numération et au ionogramme :
l'échographie abdominale retrouve une masse hyperéchogène correspondant à la description clinique, cette masse est située
au pôle supérieur du rein droit, mais la mauvaise préparation de l'enfant ne permet pas de trancher formellement entre le
caractère rénal ou extrarénal de la tumeur. L'ASP retrouve des microcalcifications au niveau de la masse. L'UIV prouve que
cette tumeur est développée au niveau du pôle supérieur du rein droit qu'elle refoule en avant ; la radiographie pulmonaire est
normale.
Les HVA et VMA sont normaux, la scintigraphie osseuse retrouve une hyperfixation au niveau de la voûte crânienne, du
sternum, des deux épines iliaques. Le myélogramme, effectué en zone pathologique, retrouve des cellules indifférenciées,
groupées en rosettes.

Vous évoquez en priorité :


A - Un lymphome métastatique
B - Un néphroblastome métastatique
C - Un neuroblastome métastatique non sécrétant
D - Un médulloblastome métastatique
E - Un rhabdomyosarcome
Bonne(s) réponse(s) : C

Le diagnostic se fait entre les items B (néphroblastome) et C (neuroblastome). Les arguments en faveur de C sont : les
microcalcifications, la masse extra rénale refoulant le rein. Le piège est la normalité des H.V.A et V.M.A. (environ 3 % des cas).

Vous proposez d'emblée :


A - Une laparotomie exploratrice et thérapeutique
B - Une radiothérapie abdominale et osseuse
C - Une chimiothérapie
D - Une plasmaphérèse
E - Une greffe de moelle
Bonne(s) réponse(s) : C

Question très spécialisée, la chimiothérapie est indiquée ici en raison de l'importance de la masse, de sa fixité et surtout de
l'existence de métastases. Une exérèse serait à pratiquer secondairement si possible (très mauvais pronostic).

Après le traitement initial, l'enfant reçoit plusieurs cures de chimiothérapie ; 15 jours après l'une d'elles, les
parents ramènent l'enfant parce qu'il est hyperthermique avec des frissons depuis 24 h. L'examen somatique et
la radio pulmonaire sont normaux: quelle(s) est(sont) la(les) décisions() à prendre en urgence ?
A - Des hémocultures
B - Une uroculture
C - Une UIV
D - Une ponction lombaire
E - Un traitement antibiotique
Bonne(s) réponse(s) : A B E

Ce tableau évoque une infection sur une "neutropénie" post chimiothérapie. La conduite à tenir repose sur les prélèvements
bactériologiques et l'antibiothérapie d'urgence à spectre large. Une N.F.S. serait utile. La P.L. n'est pas indiquée dans un
premier temps en raison de l'absence de syndrome méningé.

Deux mois plus tard, après une nouvelle cure de chimiothérapie, l'enfant est ramené pour une toux sèche,
quinteuse, avec dyspnée d'apparition récente, et d'aggravation rapide, sans hyperthermie. La radiographie
pulmonaire montre un syndrome interstitiel diffus. La gazométrie montre une hypoxie et une hypocapnie. Vous
évoquez :
A - Une pneumonie à bacille de Friedlander
B - Une localisation tumorale
C - Une infection par le pneumocystis carinii
D - Une bronchite banale
E - Une aspergillose invasive
Bonne(s) réponse(s) : C

L'aspect radiologique (syndrome interstitiel diffus) dans le contexte d'immunodépression évoque 2 diagnostics et l'item E.
L'item C est compatible avec l'absence de fièvre ( quoique celle-ci soit fréquente). Enfin l'argument de "fréquence" est en
faveur du pneumocystis.

341
Exclusivement sur DOC - DZ : www.doc-dz.com NADJI 85
RESIDANAT EN POCHE TOME II
Cas Clinique en QCM
Les parents du jeune Jean-Louis contastent chez leur fils unique (âgé de 4 ans) une bouffissure du visage alors que l'enfant
est traité pour une rhinopharyngite non fébrile par Soframycine® Nasale simple (gouttes nasales). La bouffissure du visage
persiste et l'on note aussi une prise de poids brutale de 2 kg en l'espace de 6 jours (16,8 kg au lieu de 14,6 kg). Il n'existe pas
d'antécédents familiaux.
Examen clinique : les oedèmes sont très marqués au visage avec des paupières gonflées, aux deux membres inférieurs et il
existe une hydrocèle bilatérale. Il existe quelques râles bronchiques à l'auscultation pulmonaire. Examens complémentaires :
- sang 5 100 000 GR/mm3, Hb 13.9g/100 ml, 9 200 GB/mm3 dont 55% de neutrophiles 2% d'éosinophiles, 391 000 plaquettes
/mm3, VS 88/132 mm, fibrinogène 6,5 g/l, urée 5 mmol/l (0.30 g/l) créatinine 48 micromoles/l (4,8 mg/l), protéines totales 45
g/l, albumine 15 g/l, électrophorèse (albumine 24 %, alpha 1=5 %, alpha 2=43%, bêta= 18%, gamma= 10%), cholestérol 9,5
mmol/l (3,7 g/l). urines : 4 GR/mm3, (normal 10 GR/mm3) et 2 GB/mm3 (normal 10 GB/mm3), protéinurie 3.8 g/24 h avec 58%
d'albumine à l'électrophorèse, volume des urines de 24 heures = 300 ml.
autres examens : antistreptolysines 50 U/ml, complément total CH 50 = 47 U/ml (normal de 47 à 83 U/ml) et fraction C3 =
180mg (normal de 14 à 195 mg)

Compte tenu des signes cliniques et biologiques présents chez ce garçon de


4 ans, quel est le diagnostic ?
A - Glomérulonéphrite aiguë
B - Syndrome néphrotique
C - Cardiopathie décompensée
D - Oedème de Quincke
E - Protéinurie tubulaire
Bonne(s) réponse(s) : B

Il s'agit d'un syndrome néphrotique: protéinurie > 3 g/24 h, hypoalbuminémie, hypogammaglobulinémie, hyper ¶ 2, oedème et
prise de poids. Le diagnostic le plus probable est celui de néphrose lipoïdique (lésion glomérulaire minime).

Parmi les paramètres suivants, quel(s) est(sont) le(s) paramètre(s) en faveur d'un bon pronostic ?
A - Absence d'hématurie
B - Protéinurie sélective
C - Absence d'hypertension artérielle
D - Absence d'insuffisance rénale
E - Taux de complément normal
Bonne(s) réponse(s) : A B C D E

Les items A,C,D excluent l'existence d'un syndrome néphrotique impur (de beaucoup moins bon pronostic).
Les items B,E sont effectivement facteurs de bon pronostic.

Vis-à-vis de ce garçon, l'attitude thérapeutique comporte :


A - Aspirine : 100 mg/kg/jour
B - Régime pauvre en sodium
C - Prédnisone : 2 mg/kg/jour
D - Gentamicine : 1 mg/kg/jour
E - Indométhacine : 1 mg/kg/jour
Bonne(s) réponse(s) : B C

Question de cours: certains associent aussi à la phase initiale des diurétiques (type Furosémide ou spironolactone).

Il s'agit de respecter des conseils hygiéno-dietétiques pour conduire correctement ce traitement. Le(s)quel(s) ?
A - Restriction hydrique en fonction de la diurèse
B - Régime pauvre en sodium
C - Suppléments de calcium
D - Limiter les apports de protéines
E - Suppléments de vitamine D
Bonne(s) réponse(s) : A B C E

A - Fait partie du traitement


B,C,E - Sont des mesures accompagnants la corticothérapie
D - Au contraire la protéinurie avec hypoalbuminerie ainsi que la corticothérapie imposent un régime riche en prolides

342
Exclusivement sur DOC - DZ : www.doc-dz.com NADJI 85
RESIDANAT EN POCHE TOME II
Cas Clinique en QCM
Un nourrisson âgé d'un an, eutrophique et sans antécédent pathologique, est amené en urgence par ses parents parce qu'ils
l'ont trouvé pâle et respirant rapidement. A l'examen clinique, il existe une polypnée à 80/minute, une pâleur et une cyanose,
une tachycardie à 180/mn et un bruit de galop, une hépatomégalie. Les pouls sont mal perçus, la tension artérielle systolique
est à 40 mm Hg aux membres supérieurs et aux membres inférieurs. La radiographie de thorax faite en urgence montre des
opacités hilifuges bilatérales et une cardiomégalie avec rapport cardiothoracique à 0,70.

Quel est le diagnostic à envisager :


A - Bronchiolite aiguë
B - Dyspnée laryngée
C - Choc septique
D - Insuffisance cardiaque
E - Pneumothorax
Bonne(s) réponse(s) : D

L'association : tachycardie, hépatomégalie, cardiomégalie, signe l'insuffisance cardiaque. Les opacités thoraciques sont dans
ce contexte un oedème pulmonaire.

Quelle(s) est(sont) la(les) mesure(s) d'urgence à prendre pour traiter l'enfant ?


A - Perfusion de macromolécules
B - Injection de bicarbonate semi-molaire
C - Oxygénation par sonde nasale
D - Injection de furosémide (Lasilix®)
E - Injection de dexaméthasone
Bonne(s) réponse(s) : C D

A - Aggraverait l'insuffisance cardiaque


B - Dépend du degré d'acidose.
E - Est inutile.
On pourrait ajouter un traitement cardiotonique (par exemple par Digoxine®).

Quelle peut être la cause déclenchante de cette détresse vitale ?


A - Tachycardie paroxystique supraventriculaire
B - Myocardite aiguë virale
C - Une communication inter-auriculaire
D - Une insuffisance rénale aiguë
E - Méningococcémie
Bonne(s) réponse(s) : B

Surtout coxsackies du groupe B.


Pour l'item A il faudrait une fréquence cardiaque plus élevée (supérieur à 200 - 220/mm).

Quel(s) examen(s) paraclinique(s) demanderez-vous en urgence ?


A - Cathétérisme cardiaque
B - Echocardiographie
C - Electroencéphalogramme
D - Electrocardiogramme
E - Urographie intraveineuse
Bonne(s) réponse(s) : B D

Les items C et E sont bien entendu faux.


L'item A ne serait intéressant que dans une cardiopathie congénitale éliminée sur l'eutrophie, l'absence d'antécédents
pathologiques, l'absence de souffle.

343
Exclusivement sur DOC - DZ : www.doc-dz.com NADJI 85
RESIDANAT EN POCHE TOME II
Cas Clinique en QCM
R... Xavier, né à terme, avec un score d'Apgar à 9, dont le développement a été normal jusque là, est hospitalisé en urgence à
2 ans, en raison d'une perte de connaissance brutale, accompagnée de secouses musculaires du bras, de la jambe et de
l'hémiface gauche.
Sa température est à 40°C, son pouls à 160/min, sa TA. à 9/6. La crise cède après 15 minutes
Il persiste un déficit moteur gauche qui ira en s'atténuant. La natrémie est à 130 mEq/l, la kaliémie à 4,5 mEq/l, la calcémie à
95 mg/l, la glycémie à 1,80 g/litre.
Quatre jours après la crise, l'électroencéphalogramme est correctement organisé, composé de fréquences à 5-6 c/seconde,
bien voltées, symétriques, de bonne réactivité.

Indiquez la cause de la crise observée ?


A - Thrombo-phlébite cérébrale
B - Hyperthermie.
C - Hyponatrémie.
D - Hyperglycémie.
E - Aucune des causes précédentes
Bonne(s) réponse(s) : B

A - Très rare.
C - Natrémie insuffisamment abaissée pour expliquer les convulsions.
D - L'hyperglycémie est souvent une conséquence des convulsions (par décharge de catécholamines).

Quel traitement devait être fait pour interrompre la crise ?


A - Injection IM de gardénal
B - Injection IV d'aspirine
C - Injection de diazepam intrarectal
D - Injection IV de soluté salé
E - Enveloppements frais
Bonne(s) réponse(s) : C

A - L'absorption du Gardenal® par voie I.M. n'étant pas très rapide, celui-ci ne peut être utilisé pour stopper une convulsion.
B,C,E - Ne sont pas des traitements anticonvulsivants.

Quel(s) est(sont) le(s) risque(s) encouru (s) à l'occasion d'une telle crise ?
A - Hémiplégie définitive
B - Etat de mal épileptique
C - Epilepsie séquellaire
D - Hématome sous-dural
E - Aucun d'entre-eux
Bonne(s) réponse(s) : A B C

A - C'est le syndrome hémiplégie hémiconvulsion plus ou moins épilepsie (C).


B - Est évident (si la convulsion ne s'arrête pas).

Quel(s) traitement(s) faut-il poursuivre après la crise ?


A - Antibiothérapie adaptée
B - Héparinothérapie
C - Synacthène
D - Phénobarbital
E - Aucun traitement
Bonne(s) réponse(s) : D

L'existence d'une crise longue (15 mn) hémicorporelle avec déficit post critique invite fortement à donner un traitement
préventif par phénobarbital ou acide valproïque (Dépakine®).

Un garçon de 5 ans, sans antécédents, présente subitement des vomissements. Le médecin appelé découvre lors de
l'examen une volumineuse masse du flanc et de la fosse iliaque droite donnant le contact lombaire. On ne retouve pas de
notion d'hématurie ni d'infection urinaire, ni à fortiori d'infection urinaire fébrile. L'enfant est pâle, asthénique, grognon. Il est
immédiatement hospitalisé dans un service de Pédiatrie. Le premier examen réalisé est une échographie abdominale qui
confirme le caractère solide de la masse perçue cliniquement. L'ECBU est normal.

A ce stade, quel(s) diagnostic(s) peut-on déjà évoquer ?


A - Hydronéphrose droite
B - Pyélonéphrite xanthogranulomateuse
C - Tumeurs de Wilms (néphroblastome)
D - Sympathoblastome
E - Polykystose de type infantile
Bonne(s) réponse(s) : C D

344pas de syndrome de masse ni d'ECBU normal.


Les items A et E ne donnent pas de masse solide. L'item D ne donne
Exclusivement sur DOC - DZ : www.doc-dz.com NADJI 85
RESIDANAT EN POCHE TOME II
Cas Clinique en QCM

Quel(s) examen(s) faut-il alors demander ?


A - Radio de thorax
B - Urographie intra-veineuse
C - Scanner abdominal
D - Dosage de catécholamines urinaires
E - Scintigraphie isotopique rénale
Bonne(s) réponse(s) : A B C D

L'item A recherche une métastase, l'item B différencie une masse rénale d'une masse extra rénale, l'item C le confirme et
précise l'extension voir l'existence de métastases hépatiques. L'item D est élevé dans 95 % des neuroblastomes.

Les examens précédents permettent de confirmer l'origine rénale de la tumeur, quel(s) autre(s) élément(s) faut-
il s'efforcer de préciser ?
A - Intégrité du rein contro-latéral
B - Existence d'adénopathies
C - Intégrité de la vessie
D - Etat de la veine rénale droite
E - Etat de la veine cave inférieure
Bonne(s) réponse(s) : A B D E

Il n'y a jamais d'atteinte vésicale dans les néphroblastomes, l'item A est indispensable avant chirurgie d'exérèse.

A priori, quel est le traitement qui sera proposé à cet enfant ?


A - Radiothérapie
B - Chimiothérapie
C - Néphrectomie élargie droite
D - Radiothérapie et néphrectomie élargie
E - Chimiothérapie et néphrectomie élargie
Bonne(s) réponse(s) : E

C'est le traitement de choix exérèse chirurgical encadrée par une chimiothérapie éventuellement suivie d'une radiothérapie.

Quel est actuellement le pronostic de survie à 5 ans des néphroblastomes sans métastase initiale ?
A - Inférieur à 10 %
B - Environ 25 %
C - Environ 50 %
D - Supérieur à 80 %
E - Imprévisible
Bonne(s) réponse(s) : D

C'est une tumeur fréquente de "bon" pronostic.

Une fille de 4 ans, jusque là bien portante, est hospitalisée en urgence le 2 janvier 1984 à 8 heures parce-qu'elle souffre du
ventre depuis 36 heures. Les douleurs évoluaient par crises : elles sont devenues permanentes. L'enfant a vomi
(vomissements non sanglants), n'a pas de selles : la température est à 36,8 degrés, la tension artérielle est à 80/60, le pouls à
140, le visage est pâle, la langue est sèche, le ventre respire mal, il est douloureux ; la défense abdominale généralisée est
évidente. Au toucher rectal, l'ampoule rectale est vide, pas de tumeur palpable. Le doigtier ramène du sang noir La
radiographie sans préparation de l'abdomen montre, dans une grisaille généralisée, quelques niveaux liquides (sur le grêle),
pas de pneumopéritoine.

En urgence, vous jugez nécessaire de demander :


A - Numération formule avec hématocrite
B - Culot urinaire
C - Groupe sanguin
D - Coproculture
E - Tous les examens précédents
Bonne(s) réponse(s) : A C

Ce tableau est celui d'une invagination intestinale aiguë. Le bilan demandé ici est un bilan pré-chirurgical.

345
Exclusivement sur DOC - DZ : www.doc-dz.com NADJI 85
RESIDANAT EN POCHE TOME II
Cas Clinique en QCM

Dans l'énoncé du texte, vous auriez souhaité voir préciser :


A - La taille de l'enfant
B - La coloration des urines
C - La présence d'une cicatrice opératoire sur l'abdomen
D - L'état des orifices herniaires
E - L'existence d'une infection rhinopharyngée récente
Bonne(s) réponse(s) : C D

Devant un tableau d'occlusion, ce sont deux éléments à rechercher systématiquement.


C - Est en faveur d'une occlusion sur bride et D en faveur d'un étranglement herniaire.

Indiquez, parmi les suivants, le diagnostic que vous posez :


A - Perforation d'organe creux
B - Infection entérale
C - Torsion d'une tumeur annexielle
D - Sténose hypertophique du pylore
E - Invagination intestinale aigue
Bonne(s) réponse(s) : E

L'existence de douleurs abdominales évoluant par crises avec vomissements, syndrome occlusif et sang au T.R. ne peut
évoquer que le seul diagnostic d'I.I.A. (à 1 stade supplémentaire elle peut se compliquer de l'item A).

Indiquez votre choix thérapeutique :


A - Observation avec surveillance du pouls et de la tension artérielle pendant 24 heures
B - Aspiration-perfusion pendant 24 heures
C - Traitement antalgique et antiinflammatoire
D - Exploration chirurgicale de l'abdomen
E - Traitement antibiotique et anticoagulant
Bonne(s) réponse(s) : D

L'existence de sang noir au T.R. et d'un état de choc impose l'intervention. Si ces facteurs n'existaient pas on pourrait
proposer un lavement opaque à visée diagnostique et thérapeutique. Les items A B C E sont faux (et dangereux).

Vous êtes appelé en pleine nuit pour une dyspnée survenue brutalement chez un enfant de 7 ans. Vous êtes en présence
d'un enfant non hypoxique, non fébrile, qui présente des sibilances diffuses ; c'est le 3e épisode de ce type, le premier étant
survenu il y a un an. Cet entant à des antécédents personnels de dermite atopique. Son père a une pollinose. Vous portez le
diagnostic d'asthme probablement de nature allergique.

Quels sont les deux médicaments utilisables en première intention pour traiter cette crise ?
A - Sirop de théopnylline
B - Ampoule de corticoïde injectée par voie intraveineuse
C - Sirop antihistaminuque (H1)
D - Capsule de cromoglycate de sodium à inhaler
E - Aérosol de bronchodilatateur bêta 2 mimétique
Bonne(s) réponse(s) : A E

B - Peut se discuter mais n'est pas un traitement de première intention.


C D - N'ont aucun intérêt.
E - Attention ne pas confondre aérosol et spray.

Quel(s) médicament (s) pouvez vous utiliser au long cours pour prévenir la
récidive de ces crises :
A - Sirop anti-histaminique (H 1 )
B - Capsule de cromoglycate de sodium à inhaler
C - Sirop d'ampicilline
D - Corticoïdes per os
E - Gammaglobulines IM
Bonne(s) réponse(s) : B

A C E - N'ont aucun intérêt dans la prévention de l'asthme D, outre sa "toxicité" au long cours, n'est jamais utilisé chez l'enfant
sans traitement bronchodilatateur associé.

346
Exclusivement sur DOC - DZ : www.doc-dz.com NADJI 85
RESIDANAT EN POCHE TOME II
Cas Clinique en QCM

Parmi les examens suivants, le(s)quel(s) explore(nt) l'hypersensibilité de type I ?


A - Dosage des IgA sériques
B - Test de transformation Iymphoblastique en présence de PHA
C - Dosage des IgE spécifiques
D - Chimiotaxie des polynucléaires neutrophiles
E - Histaminolibération spécifique des polynucléaires basophiles
Bonne(s) réponse(s) : C E

Question de cours : l'item E est rarement pratiqué.

Parmi les propositions suivantes, indiquez les trois allergènes responsables de crises d'asthme les plus
fréquemment mis en évidence :
A - Candida albicans
B - Salive de chat
C - Streptocoque A
D - Dermatophagoïdes ptéronyssinus
E - Pollen de dactyle
Bonne(s) réponse(s) : B D E

D E - Sont classiques.
B - Un peu moins (mais l'allergène majeur au chat (cat 1 ) est surtout contenu dans la salive.

Antoine, 9 mois, est amené à votre consultation pour l'examen obligatoire du 9 ème mois, et la mise à jour des vaccinations.
Antécédents :
- Familiaux : mère âgée de 22 ans, 2ème geste, 2ème pare, un frère âgé de 2 ans, ancien prématuré (36 semaines), et ancien
dysmature (P. N. = 1 500g), actuellement en bonne santé.
- Personnels : comme son frère, Antoine est né prématurément (34 semaines), mais il n'a pas souffert de dysmaturité (P. N. =
1 900g). Grossesse marquée par :
- une anémie ferriprve ayant justifié un traitement martial
- la notion d'un travail fatigant (travail à la chaîne sur une machine industrielle)
Le carnet de santé révèle que le développement staturo-pondéral et somatique de l'enfant est normal, de même que
l'évolution psycho-motrice.

Parmi les éléments suivants, lequel (lesquels) peut (peuvent) être impliqué (s) dans le déterminisme de la
prématurité ?
A - L'âge de la mère (22 ans)
B - La notion d un accouchement prématuré à la précédente grossesse
C - La notion d'une dysmaturité à la précédente grossesse
D - L'anémie ferriprive
E - La notion d'un travail fatigant durant la grossesse
Bonne(s) réponse(s) : B E

Question de cours, l'âge "normal" de la mère et les items C et D ne sont pas des causes de prématurité.

L'examen obligatoire du 9ème mois comporte un dépistage sensoriel, auditif et visuel. Pour réaliser ce
dépistage, on peut utiliser le (les) moyen(s) suivant(s) :
A - Impédancemétrie
B - Jouets de Moatti
C - Potentiels évoqués
D - Test de l'écran
E - Test des reflets cornéens
Bonne(s) réponse(s) : B D E

Les items A et C ne sont pas des examens de dépistage mais de "confirmation".

Antoine a eu les deux premières injections du vaccin DTCP (diphtérie, tétanos, coqueluche, poliomyélite). La
3ème injection n'a pas été réalisée à cause de la survenue d'infections rhinopharyngées récidivantes. Quelle
conduite adoptez-vous ?
A - Vous faites la 3ème injection de DTCP en ne tenant pas compte du retard
B - Vous faites une injection de ROUVAX, en prévoyant de recommencer la vaccination DTCP un mois après
C - Vous faites le BCG, en prévoyant de recommencer la vaccination DTCP un mois après
D - Vous recommencez sans attendre la totalité de la vaccination DTCP
E - Vous attendez l'âge d'un an pour recommencer la totalité de' la vaccination DTCP
Bonne(s) réponse(s) : A

En fait Q.C.M. litigieuse. Il faudrait savoir la durée de ce retard, si celui-ci est supérieur à 3 - 4 mois l'item D serait sans doute
plus adapté.

347
Exclusivement sur DOC - DZ : www.doc-dz.com NADJI 85
RESIDANAT EN POCHE TOME II
Cas Clinique en QCM

En même temps que la vitamine D, vous conseillez l'administration quotidienne à l'enfant d'un comprimé de
fluor. Les comprimés de Fluor® sont habituellement dosés à :
A - 0,01 mg
B - 0,02 mg
C - 0,05 mg
D - 0,10 mg
E - 0,25 mg
Bonne(s) réponse(s) : E

A savoir, mais question d'un intérêt discutable.

Parallèlement, vous expliquez à la mère d'Antoine que l'administration quotidienne de Fluor ne remplace pas la
brosse à dents, dont il conviendra d'enseigner le bon usage dans les mois et années à venir. Actuellement en
France, quelle est la part des soins dentaires dans l'ensemble de la consommation médicale ?
A - 0,5 à 2 %
B-2à4%
C - 4 à 6%
D-6à8%
E - 8 a 10 %
Bonne(s) réponse(s) : E

Sans commentaire.

Vous êtes appelé auprès d'un enfant de 7 ans pour fièvre, vomissements, érythème maculopapuleux prédominant sur le tronc
et angine érythémateuse diffuse. Vous envisagez le diagnostic de scarlatine.

Quel(s) examen(s) devez-vous demander si vous désirez confirmer ce diagnostic ?


A - Prélèvement de gorge
B - Hémoculture
C - Prélèvement cutané
D - Coproculture
E - MNI test
Bonne(s) réponse(s) : A

Le streptocoque bêta hemolytique du groupe A n'est retrouvé qu'au niveau de la gorge. Les autres manifestations de la
maladie sont dues à une toxine.

Quel est l'agent responsable de la scarlatine ?


A- Staphylococcus aureus
B - Streptocoque bêta-hémotylique du groupe A
C - Coxsackie du groupe B
D - Streptocoque alpha-hémolytique
E - Adénovirus
Bonne(s) réponse(s) : B

Celui-ci produit une exotoxine érythrogène sous l'influence d'un patrimoine génétique apporté par un bactériophage.

Quelle(s) est(sont) la(les) complication(s) de cette maladie ?


A - Adénite mésentérique
B - Rhumatisme articulaire aigu
C - Acné
D - Glomérulonéphrite aiguë
E - Ostéomyélite aiguë
Bonne(s) réponse(s) : B D

On pourrait citer aussi la chorée de sydenham et l'érythème noueux.

348
Exclusivement sur DOC - DZ : www.doc-dz.com NADJI 85
RESIDANAT EN POCHE TOME II
Cas Clinique en QCM

Quelle est l'attitude à envisager ?


A - Ne rien faire et attendre
B - Traiter par pénicilline
C - Traiter par gentamicine (Gentalline®)
D - Traiter par oxacilline (Bristopen®)
E - Prescrire uniquement des soins locaux
Bonne(s) réponse(s) : B

En fait pénicilline V (orale).


C - Est inactif sur le streptocoque
D - N'est pas le meilleur traitement.
A E - Exposent aux complications décrites à la question n°80.

Vous êtes appelé auprès d'un nourrisson de 3 mois, jusque là en bonne santé, et chez lequel est survenue, sans prodromes,
une crise douloureuse, apparemment abdominale, avec vomissement, puis retour au calme complet. Lorsque vous examinez
ce nourrisson, survient une autre crise identique à celle que vous a décrite sa mère, qui vous précise de plus qu'il existe
depuis le premier épisode une intolérance alimentaire absolue.

Parmi les signes cliniques suivants, indiquez celui( ceux) qui doi(ven)t faire envisager le diagnostic
d'invagination intestinale aiguë :
A - Caractère paroxystique des troubles
B - Existence de diarrhée
C - Vomissement initial
D - Refus de l'alimentation
E - Existence d'une fistule ombilicale
Bonne(s) réponse(s) : A D

Les items A et D sont classiques.


C - Les vomissements surviennent plus tardivement (occlusion).

Chez cet enfant, le diagnostic d'invagination a été affirmé par la découverte de sang au toucher rectal. Indiquez
parmi les suivantes, le(s) affection(s) qui peu(ven)t également provoquer une hémorragie intestinale chez les
nourrissons :
A - Ulcère gastrique
B - Mucocèle appendiculaire
C - Ulcère du diverticule de Meckel
D - Etranglement herniaire
E - Polypose rectocolique
Bonne(s) réponse(s) : A C E

Question de cours, l'item D pourrait éventuellement donner des saignements.

L'examen de l'abdomen entre deux crises a permis de sentir le boudin d'invagination, ce boudin a pour
caractéristique(s) clinique(s) :
A - Tuméfaction allongée, cylindrique
B - Sonorité à la percussion
C - Présence à son niveau d'ondulations péristaltiques
D - Consistance plus dure au moment des crises paroxystiques
E - La tête peut être perçue au toucher rectal
Bonne(s) réponse(s) : A D E

Question de cours (en fait la palpation d'un boudin d'invagination est rare et sa palpation au T.R. encore plus).

Une désinvagination au cours du lavement baryté se traduira radiologiquement par :


A - Absence d'image lacunaire endoluminale
B - Opacification complète du caecum
C - Absence d'encoche du bord interne du caecum
D - Opacification de l'appendice
E - Reflux franc de baryte dans le grêle
Bonne(s) réponse(s) : B C E

Question de cours : l'absence d'un de ses critères en particulier de E impose la laparotomie à pour désinvagination
chirurgicale.

349
Exclusivement sur DOC - DZ : www.doc-dz.com NADJI 85
RESIDANAT EN POCHE TOME II
Cas Clinique en QCM
Rémi est âgé de 3 ans. Il est hospitalisé le 15 janvier en urgence parce qu'en quelques heures s'est constitué le tableau
suivant : température à 39°, dyspnée grave, refus alimentaire total, par dysphagie semble-t-il. A l'examen, l'enfant demeure
assis (sa mère précise qu'il refuse d'être allongé) : la dyspnée est intense avec important tirage sus-sternal, sus-claviculaire et
inter-costal; il demeure la bouche ouverte, la langue tirée avec hypersialorrhée.Dans les antécédents personne!s, on note de
fréquentes rhinopharyngites, sans complication et dans les antécédents familiaux, une tante maternelle asthmatique.

Quelle précaution devez-vous observer en cas de transport ?


A - Allonger l'enfant en décubitus ventral tête de côté
B - Calmer l'enfant avec un sédatif
C - Administrer à l'enfant un bronchodilatateur d'action rapide
D - Maintenir l'enfant en position assise
E - Coucher l'enfant sur le dos, la tête en hyperextension
Bonne(s) réponse(s) : D

Ce tableau est celui d'une épiglottite, les items A et E sont interdits (risque d'arrêt respiratoire réflexe).
B - Est angereux : risque de dépression respiratoire.
C - Est inutile.

Quel(s) examen(s) est-il impératif de pratiquer en urgence avant toute décision thérapeutique parmi les suivants
:
A - Examen soigneux de la gorge
B - Laryngoscopie
C - Bronchoscopie
D - Radiographie pulmonaire
E - Aucun
Bonne(s) réponse(s) : E

L'épiglottite est une très grande urgence, les items A B et C sont interdits (risque d'arrêt cardio respiratoire réflexe). L'item D
inutile ferait perdre du temps.

En dehors des éventuels examens ci-dessus, indiquez parmi les suivantes, la proposition correcte relative à la
conduite à tenir :
A - Mise en observation au repos sous sédatifs et oxygène
B - Corticothérapie d'emblée + oxygène
C - Théophylline + oxygène
D - Sérum antidiphtérique + oxygène
E - Intubation d'emblée ou, en cas d'échec, trachéotomie
Bonne(s) réponse(s) : E

C'est en effet le meilleur traitement, les items A C et D n'ont aucun intérêt et font perdre du temps, l'item A est indéfendable.
Les traitements associés à l'item E sont l'antibiothérapie et l'item B.

Concernant l'affection présumée en cause, quelle est, parmi les suivantes, la proposition exacte ?
A - Elle est habituellement due au virus respiratoire syncitial
B - Elle est habituellement due au mycoplasma pneumoniae
C - Elle est habituellement due à l'hémophilus influenzae
D - Elle ne comporte, malgré son caractère impressionnant, aucun risque vital
E - Il s'agit de la forme la plus fréquente des dyspnées laryngées infectieuses de l'enfant
Bonne(s) réponse(s) : C

Question de cours : l'épiglottite est un abcès épiglottique à hémophilus influenzae le plus souvent.

350
Exclusivement sur DOC - DZ : www.doc-dz.com NADJI 85
RESIDANAT EN POCHE TOME II
Cas Clinique en QCM
Un enfant de 6 ans se plaint brutalement de céphalées et il a de la fièvre (39,5 degrés). Le médecin appelé constate une
raideur méningée et quelques éléments purpuriques sur les jambes. Il n'y a aucun signe de choc. La tension artérielle est
normale Le liquide céphalo-rachidien est purulent et montre 900 polynucléaires par mm3, une protéinorachie à 1 g pour mille,
une glycorachie à 2 mmoles/l. Il y a dans la famille, 2 autres enfants de 8 et 10 ans fréquentant la même école, qui n'a pas
d'internat.

Parmi les germes suivants, quel est celui qui est le plus fréquemment en
case à cet âge et en l'absence de traumatisme ?
A - Méningocoque.
B - Staphylocoque
C - Hémophilus
D - Pneumocoque
E - Listeria
Bonne(s) réponse(s) : A

Evident (de toute façon tout purpura fébrile est méningococcique jusqu'à preuve du contraire). Donc l'existence de taches
purpuriques lors d'une méningite purulente signe le méningocoque.

Le germe n'étant par identifié, vous désirez commencer le traitement. Quel sera votre choix :
A - Céphalotine 100 mg/kg/24 h IV
B - Ampicilline 200 mg/kg/ 24 h IV
C - Gentamicine 3 mg/kg/24 h IM
D - Oracilline 2 millions d'U/24 h IM
E - Erythromycine 100 mg/kg/24 h per os.
Bonne(s) réponse(s) : B

A C E - Ne passent pas la barrière hémato méningée


D - L'oracilline n'existe pas en I.M.

A l'examen direct du LCR si l'on vous répond présence de cocci gram négatif, le(s)quel(s) pourrai(en)t être en
cause parmi les suivants :
A - Méningocoque
B - Listéria
C - Pneumocoque
D - Hémophilus
E - Colibacille
Bonne(s) réponse(s) : A

Question de cours :
B - Bacille gram+.
C - Cocci cram+.
D E - Bacilles gram -

Si c'est une méningite à méningocoque, quelle(s) est (sont) la (les) mesure(s) d'hygiène à prendre
obligatoirement :
A - Désinfecter l'habitation
B - Faire une déclaration de maladie contagieuse
C - Fermer l'école
D - Faire un prélèvement de gorge à tous les élèves de la même école
E - Ne prendre aucune mesure particulière
Bonne(s) réponse(s) : B

Déclaration obligatoire n°13. A C - Aucun intérêt : contamination interhumaine directe, le germe est très fragile. Le portage
pharyngé n'est pas forcément pathologique.

Vous avez choisi de faire une chimio-prophylaxie de l'entourage par la Rovamycine®, choisir parmi les cinq
propositions suivantes celle(s) vous paraissant exacte(s) :
A - Elle s'applique aux frères et soeurs vivant sous le même toit
B - Elle ne sera faite que si le prélèvement de gorge des frères et soeurs est positif pour le méningocoque
C - Cette mesure est à étendre à toute l'école
D - Cet antibiotique a été choisi parce qu'il passe bien la barrière méningée
E - La dose usuelle est de 50 mg/kg/jour durant 5 jours
Bonne(s) réponse(s) : A E

Risque maximum pour les sujets ayant un contact fréquent et prolongé. La spiramycine comme tous les macrolides ne passe
pas la barrière hémato méningée et se prescrit à la dose de 50 mg/kg et par jour.

351
Exclusivement sur DOC - DZ : www.doc-dz.com NADJI 85
RESIDANAT EN POCHE TOME II
Cas Clinique en QCM
L'enfant Sébastien, 4 ans, souffre depuis une quinzaine de jours de céphalées de plus en plus intenses, survenant tôt le
matin, réveillant l'enfant. Depuis trois jours, sont apparus des vomissements qui se répètent une ou deux fois surtout en
première partie de journée, après quoi les céphalées s'atténuent Il n'existe aucun antécédent particulier. Le développement
psychomoteur a été normal.

Vous pensez à la possibilité d'une hypertension intra-crânienne débutante. Quel(s) examen(s) permettrai(en)t
de confirmer votre diagnostic ?
A - Un électroencéphalogramme
B - Un fond d oeil
C - Un prélèvement de liquide céphalo rachidien
D - Une radiographie de crâne (face, profil)
E - Une échographie trans-fontanellaire
Bonne(s) réponse(s) : B D

A - N'a aucun intérêt dans le diagnostic d'H.T.I.C.


C - Est interdit, risque d'engagement
D - Peut montrer une disjonction des sutures et/ou des impressions digitiformes
E - A 4 ans la fontanelle est fermée.

Les parents signalent, par ailleurs, que Sébastien paraît manquer d'équilibre et chute anormalement souvent à
la marche Quel(s) élément(s) clinique(s) vous fait(font) évoquer une lésion cérébelleuse, parmi les suivants ?
A - "Danse des tendons" en position garde à vous
B - Dysmétrie lors de la préhension
C - Mauvaise exécution des marionnettes
D - Signe de Babinski bilatéral
E - Strabisme interne de l'oeil droit
Bonne(s) réponse(s) : A B C

Question de cours :
Item D - Est un signe de syndrome pyramidal.
Item E - Est un signe de paralysie du moteur oculaire externe droit (VI)

Le scanner cérébral met en évidence des signes d'hypertension intra-crânienne et une formation tumorale au
niveau du vermis cérébelleux. Quel diagnostic évoque-t-on en priorité ?
A - Un pinéalome
B - Un astrocytome cérébelleux
C - Un médulloblastome
D - Un épendymome
E - Un oligodendrogliome
Bonne(s) réponse(s) : C

Les items A D et E ne sont pas des tumeurs cérébelleuses. L'item B est une tumeur des hémisphères cérébelleux. L'item C
est la seule tumeur des vernix cérébelleux chez l'enfant.

Compte tenu que la tumeur obstrue les voies d'écoulement du liquide cephalo-rachidien, quelle attitude doit-on
adopter ?
A - Une chimiothérapie d'urgence
B - Une radiothérapie centrée sur la tumeur
C - Une intervention d'emblée sur la fosse postérieure
D - Une dérivation ventriculo-péritonéale suivie de l'intervention sur la fosse postérieure
E - Une dérivation ventriculo-péritonéale suivie d'une radiothérapie centré sur la tumeur
Bonne(s) réponse(s) : D

L'urgence est ici l'hypertension intra-crânienne d'où la dérivation comme premier geste, celle-ci est suivie au mieux d'une
exérèse complétée d'une radiothérapie et/ou d'une chimiothérapie.

Dans l'évolution ultérieure apparait un syndrome de la queue de cheval. Vous l'interprétez comme :
A - Le séquelle d'une hypertension intra-crânienne prolongée
B - La traduction d'une métastase osseuse
C - La traduction d'une métastase dans le cul de sac lombo-sacré
D - Le signe d'une ostéoposore secondaire à la thérapeutique
E - Le signe d'une atteinte neurogène périphérique secondaire à la thérapeutique
Bonne(s) réponse(s) : C

Le médulloblastome donne des métastases loco réginales et à distance au sein du système nerveux central mais rarement
des métastases par voie générale. L'item A ne donne pas de syndrome de la queue de cheval, les items D et E sont des
diagnostics "d'élimination".

352
Exclusivement sur DOC - DZ : www.doc-dz.com NADJI 85
RESIDANAT EN POCHE TOME II
Cas Clinique en QCM
Des parents vous consultent pour leur fille de 7 ans. Cette enfant sans antécédent néonatal, a eu, jusque là, une croissance et
un développement psychomoteur normal. Elle est scolarisée en CE1. Depuis 15 jours à l'école comme à la maison,
l'entourage remarque qu'elle présente, plusieurs fois par jour, de brusques épisodes au cours desquels elle paraît, tout à
coup, dans la lune, avec une altération profonde de la conscience pendant un court instant, mais sans modifications du tonus,
ni mouvements anormaux. Vous pensez immédiatement à la possibilité d'une épilepsie débutante de type "petit mal-absence".

Un des arguments cliniques qui va conforter votre diagnostic de petit mal est :
A - L'amaurose survenant immédiatement avant chaque absence
B - La brièveté de la perte de conscience (inférieure à 15 secondes)
C - Le caractère prolongé de la perte de conscience (supérieure à 30 secondes)
D - La reprise progressive de la conscience après l'absence
E - L'apparition progressive entre les absences, d'une dégradation intellectuelle
Bonne(s) réponse(s) : B

A - Evoque plutôt une migraine accompagnée.


B - Et par exclusion C.D.E question de cours.

Vous souhaiteriez assiter au déroulement d'une crise ; quel moyen pouvez-vous utiliser pour provoquer une
absence ?
A - Administration d'un hypnotique
B - Compression des globes oculaires
C - Epreuve de jeûne
D - Epreuve d'hyperpnée
E - Stimulation lumineuse intermittente
Bonne(s) réponse(s) : D

A B C - Ne déclenchent pas de crise type petit mal.


E - Est discutable, plus souvent utilisé dans les crises type grand mal.

Quel(s) élément(s) peut-on trouver sur le tracé électroencéphalographique d'un enfant atteint de petit mal pur ?
A - Pointes-ondes lentes généralisées, diffuses, permanentes
B - Rythme de base normal pour l'âge
C - Paroxysmes brefs, de polypointes-ondes, en région temporale
D - Pointes brèves, isolées, en région rolandique
E - Decharges critiques de pointes-ondes rythmiques à 3 c/s, bilatérales, symétriques d'une durée de
quelques secondes
Bonne(s) réponse(s) : B E

Question de cours : l'absence de l'item E évoque une cause organique ou une épilepsie complexe.
A - Evoque une encéphalopathie convulsivante permanente.
C - Evoque une épilepsie partielle temporale.
D - Evoque une épilepsie partielle bénigne (à pointes rolandiques).

En plus de l'électroencéphalogramme, vous estimez indispensable l'examen suivant :


A - Potentiels évoqués visuels
B - Scanner (tomodensitométrie computérisée) cérébral
C - Dosage des acides aminés sanguins
D - Radiographie de crâne des parents
E - Aucun de ces examens n'est indispensable
Bonne(s) réponse(s) : E

Le petit mal pur (critères E.E.G) n'est jamais associé à une anomalie morphologique ou organique.

Parmi les antiépileptiques suivants, celui (ceux) qui est (sont) actif(s) dans le petit mal est (sont) :
A - Carbamazépine (Tegretol®)
B - Phénobarbital (Gardenal®)
C - Diphénylhydantoïne (Dihydan®)
D - Ethosuximide (Zarontin®)
E - Valproate de Sodium Depakine®)
Bonne(s) réponse(s) : D E

Question de cours : les médicaments régulièrement actifs en monothérapie dans le petit mal sont le Zarontin®, la Dépakine®,
et les benzodiazépines.

353
Exclusivement sur DOC - DZ : www.doc-dz.com NADJI 85
RESIDANAT EN POCHE TOME II
Cas Clinique en QCM

Parmi les évolutions pronostiques suivantes, quelle(s) est (sont) celle(s) qui peu(ven)t s'appliquer à l'épilepsie
petit mal ?
A - La guérison est certaine à la puberté
B - La guérison est possible à la puberté
C - La guérison ne survient jamais même chez l'adulte
D - Des crises grand mal peuvent survenir ultérieurement
E - La dégradation intellectuelle est inéluctable
Bonne(s) réponse(s) : B D

Question de cours : la guérison est fréquente à l'adolescence l'évolution vers une épilepsie "grand mal" est cependant
possible (voire fréquente pour certains).

Vous êtes appelé auprès d'un enfant de 10 ans qui présente depuis deux jours des arthralgies intenses; qui se sont
manifestées au niveau des grosses articulations : poignet, genoux, chevilles, de façon successive, certaines articulations sont
fluxionnaires et leur mobilisation est extrêmement douloureuse. Depuis le début, la fièvre irrégulière atteint 39°. Cet enfant a
présenté 3 semaines auparavant une angine érythémateuse non traitée.
Ce tableau clinique évoque essentiellement un syndrome post-streptococcique.

Parmi les symptômes suivants, quel(s) est(sont) celui(ceux) que l'on peut observer au cours d'un syndrome post-
streptococcique ?
A - Exanthèmes
B - Taches purpuriques
C - Douleurs abdominales
D - Hépatosplénomégalie
E - Souffle systolique de pointe
Bonne(s) réponse(s) : A C E

Question de cours.

Chez ce malade, parmi les examens biologiques suivants, citez celui(ceux) qui est(sont) indispensable(s) pour
confirmer le diagnostic de syndrome post-streptococcique :
A - VS supérieure à 50 mm à la première heure
B - Leucocytose et polynucléose
C - Anémie inférieure à 10 g d'Hb/1OOml
D - Présence de streptocoques hémolytiques du groupe A au prélèvement pharyngé
E - Taux d'ASLO supérieur à 800 unités
Bonne(s) réponse(s) : A E

En fait il faut l'association d'un syndrome inflammatoire et d'une étiologie streptococcique.


A - Fait partie du syndrome inflammatoire mais n'est pas le seul élément et n'est pas constant.
E - Evoque une origine streptococcique (d'autant plus qu'il existe une ascension de ce taux).
Item D - A la troisième semaine le streptocoque n'est plus retrouvé dans le pharynx.

Parmi les affections suivantes, quelle(s) est(sont) celle(s) qui fait(font) partie des syndromes post-
streptococciques ?
A - Adénites cervicales
B - Erythème noueux
C - Glomérulonéphrite aiguë
D - Arthrite chronique
E - Chorée
Bonne(s) réponse(s) : B C E

Question de cours classique.

La péricardite fait partie des atteintes cardiaques du syndrome post-streptococcique. Cette péricardite :
A - Peut entraîner une douleur précordiale
B - Se traduit à l'auscultation par un frottement péricardique
C - Se traduit à l'ECGpar un trouble de la repolarisation
D - Est résistante à la corticothérapie
E - Entraîne souvent une symphyse du péricarde
Bonne(s) réponse(s) : A B C

Ces items évoquent en effet une péricardite. La péricardite rhumatismale est sensible à la corticothérapie (traitement
d'élection) et ne symphyse jamais (contrairement à la péricardite tuberculeuse).

354
Exclusivement sur DOC - DZ : www.doc-dz.com NADJI 85
RESIDANAT EN POCHE TOME II
Cas Clinique en QCM

Sur le plan prophylactique, chez un enfant guéri sans atteinte cardiaque, il faut :
A - Instituer un traitement préventif de longue durée par la pénicilline
B - Instituer un traitement préventif de longue durée par les sulfamides
C - Faire des injections de gamma-globulines à intervalles réguliers
D - Surveiller l'état dentaire et le traiter si nécessaire
E - Interdire la pratique des sports pendant 2 ans
Bonne(s) réponse(s) : A D

A - Est classique, les sulfamides ne sont pas les meilleurs traitements prophylactiques des streptocoques
C E - N'ont aucun intérêt
Item D - Est vrai chez n'importe qui (enfant ou adulte).

Vous êtes appelé au chevet d'un enfant de 5 ans qui présente une température à 39,5 degrés, installée brutalement dans un
contexte épidémique viral saisonnier Sa mère vous précise qu'il est perpétuellement enrhumé: son nez est bouché et coule
pendant la journée. Il ronfle la nuit et dort la bouche ouverte. Il a déjà fait plusieurs épisodes de fièvre depuis l'automne avec
toux et bronchites. Son appétit est capricieux, et il est en retard sur le plan staturo-pondéral. Il n'y a pas d'autres antécédents
pathologiques L'examen de cet enfant blond, d'apparence chétive et hébété, respirant par la bouche, mais tout à fait sthénique
vous montre une obstruction nasale bilatérale complète des tympans d'aspect rosé, mais non bombants et aux reliefs
conservés : sa gorge est rouge, la paroi postérieure du pharynx tapisée d'un enduit mucopurulent. La palpation cervicale met
en évidence une polyadénopathie bilatérale peu douloureuse prédominant au niveau de la nuque, l'examen général est par
ailleurs normal. Les urines sont claires Vous évoquez une adénoïdite aiguë.

Le diagnostic peut être évoqué sur la constatation de :


A - Hyperthermie d'installation brutale
B - Hypotrophie
C - Polyadénopathies au niveau de la nuque
D - Mucopus sur la paroi postérieure du pharynx
E - Obstruction nasale
Bonne(s) réponse(s) : A D E

L'hypotrophie n'est pas un signe de rhinopharyngite aiguë (ni d'adénoïdite). Les polyadénopathies des adénoïdites sont le plus
souvent cervicales latérales et antérieures.
Les signes A,D sont classiques, souvent associés à des troubles digestifs.

Quel(s) examen(s) allez-vous pratiquer pour confirmer ce diagnostic ?


A - Toucher du cavum
B - Radiographie du cavum
C - Hémogramme
D - Examens cytobactériologiques des urines
E - Rhinoscopie postérieure
Bonne(s) réponse(s) : E

C'est en effet un examen simple non invasif confirmant le diagnostic. En général aucun examen complémentaire n'est
pratiqué. Si la question était quels examens pouvez-vous pratiquer : la N.F.S. montrerait une formule virale et la radiographie
de cavum apprécierait l'obstruction.

Quelle(s) est(sont) parmi les complications suivantes celle(s) qui est(sont) susceptible(s) de survenir ?
A - Otite moyenne aiguë
B - Kératite
C - Laryngite
D - Glomérulonéphrite
E - Méningite
Bonne(s) réponse(s) : A C D E

Question litigieuse : les items A et C sont classiques.


La kératite est exceptionnelle (mais la conjonctivite fréquente).
Item E - Cette adénoïdite peut être une "porte d'entrée", enfin parfois due (ou surinfectée) à un streptocoque 13 hémolytique
du groupe A. Elle peut se compliquer d'une glomérulo-néphrite.

Parmi les propositions thérapeutiques suivantes, quelle est (ou quelles sont) celle(s) que vous allez retenir ?
A - Lavage de nez
B - Corticoïdes
C - Antithermiques
D - Antibiotiques par voie générale
E - Gammaglobulines
Bonne(s) réponse(s) : A C D

Les items A et C sont indiscutables. L'item D est litigieux, devant ce tableau d'adénoïdite virale les antibiotiques ne sont pas
indispensables mais préviennent et traitent les surinfections (en pratique péni V ou macrolides), B et E n'ont aucun intérêt.
355
Exclusivement sur DOC - DZ : www.doc-dz.com NADJI 85
RESIDANAT EN POCHE TOME II
Cas Clinique en QCM

Parmi les mesures thérapeutiques suivantes, quelle est (ou quelles sont) celle(s) que vous proposerez pour
éviter au mieux les récidives de cette affection ?
A - Adénoïdectomie
B - Amygdalectomie
C - Antibiotiques locaux au long cours
D - Antibiothérapie générale au long cours
E Corticothérapie
Bonne(s) réponse(s) : A

Les mesures B,C,D,E n'ont aucun intérêt.

Un nourrison de 12 mois est amené aux urgences pour une détresse respiratoire. L'examen révèle une fréquence respiratoire
à 46/mn, un tirage sus-sternal et intercostal modéré, une expiration active, des râles sibilants à l'auscultation La température
est à 37,6 degrés C.

Quel(s) est(sont) l'(les) examen(s) complémentaire(s) nécessaire(s) pour confirmer le diagnostic de bronchiolite :
A - Radiographie pulmonaire
B - Gaz du sang veineux
C - Hémoculture
D - Numération formule sanguine
E - Prélèvement de gorge
Bonne(s) réponse(s) : A

Celle-ci montrerait une distension pulmonaire et des opacités alvéolaires (parfois aussi interstitielles et bronchiques). Les
items B,C,D,E ne confirment pas le diagnostic de bronchiolite (qui est essentiellement clinique).

Si vous prescrivez un bronchodilatateur de type théophylline, quelle posologie utilisez-vous ?


A - 1 mg/kg/jour
B - 5 mg/kg/jour
C - 10 mg/kg/jour
D - 20 mg/kg/jour
E - 30 mg/kg/jour
Bonne(s) réponse(s) : C

Question de cours : la théophylline n'est d'ailleurs pas un traitement de la bronchiolite.

Parmi les thérapeutiques suivantes, il est inutile de prescrire :


A - Salbutamol
B - Aspirine
C - Pristinamycine
D - Kinésithérapie
E - Aérosol fluidifiant
Bonne(s) réponse(s) : A C

Item A : Discutée mais classique : pas de bronchoconstriction dans la bronchiolite. Item B - Peut être utile en cas de fièvre.
Item C - L'étiologie est le virus respiratoire syncitial, la prévention des infections se fait surtout par macrolides (si elle est
pratiquée).
D E - Font partie du traitement avec le maintien d'un état nutritionnel correct et la position demi-assise.

La surveillance repose sur :


A - Fréquence respiratoire
B - Régularité respiratoire
C - Calcémie
D - Diurèse horaire
E - Fréquence cardiaque
Bonne(s) réponse(s) : A B

La surveillance porte sur les fonctions cardiaques, respiratoires, le poids et la température.

356
Exclusivement sur DOC - DZ : www.doc-dz.com NADJI 85
RESIDANAT EN POCHE TOME II
Cas Clinique en QCM
Un enfant de 5 ans a une fièvre à 39°C, des adénopathies cervicales, une rougeur intense du pharynx, sans coryza, des
douleurs à la déglutition.

Quel diagnostic évoquez-vous d'emblée ?


A - Une mononucléose infectieuse
B - Une angine à Streptocoques
C - Une diphtérie
D - Une rougeole
E - Un herpès
Bonne(s) réponse(s) : B

C'est en effet le tableau classique. L'item A bien que donnant le plus souvent une angine pseudomembraneuse peut être
responsable du même tableau.

Quelle attitude adoptez-vous ?


A - Vous faites faire un prélèvement de gorge et vous attendez le résultat avant d'administrer des antibiotiques
B - Vous demandez des ASLO et vous prescrivez des antibiotiques
C - Vous prescrivez des antibiotiques sans faire de prélèvement de gorge
D - Vous prescrivez des anti inflammatoires et vous observez l'évolution
E - Vous attendez le résultat des ASLO avant de traiter
Bonne(s) réponse(s) : C

Le risque le plus important de cette maladie est l'existence de syndromes post streptococciques. Ceci justifie l'abstention des
examens et le traitement par péni G ou macrolides. Le seul examen intéressant dans ce cas serait un M.N.I. test.

Quel antibiotique prescrivez vous en première intention sachant que cet enfant a des antécédents d'allergie ?
A - Pénicilline M
B - Pénicilline V
C - Erythromycine
D - Tétracycline
E - Gentamicine
Bonne(s) réponse(s) : C

A - N'est pas l'antibiotique le plus efficace et de plus il existe une allergie


B - Est l'antibiotique de référence mais les antécédents d'allergie font préférer l'item C
D - Est efficace mais contre-indiqué avant 8 ans
E - Est inefficace sur le streptocoque

Pour apprécier les suites de cet épisode infectieux, vous demandez qu'un bilan biologique soit effectué 15 jours
plus tard. Un examen est indiqué Lequel ?
A - Recherche d'une protéinurie
B - Numération et formule sanguine
C - Recherche des anti-streptolysines O
D - Recherche des anticorps anti-herpès
E - Recherche des anticorps anti-rougeole
Bonne(s) réponse(s) : A

Les complications des infections au streptocoques sont principalement la glomézulonéphrite (d'où l'item A), le rhumatisme
articulaire aigüe, la chorée de Sydenham et l'érythème noueux (surveillance clinique).

Rémi est examiné le 1er décembre à l'âge de 2 mois pour l'émission, au cours des 12 dernières heures, de 5 selles
abondantes liquides, claires, sans sang ni glaires (mucus). Sa température est de 38, 5 degrés, il ne présente pas de signes
patents de déshydratation : l'abdomen n'est pas météorisé. Il a présenté il y a 6h un vomissement. Il pèse 4kg 500. Vous
l'aviez examiné le 15 novembre, son poids était de 4kg 500 ; il était nourri depuis la naissance par un lait 1er âge maternisé ;
votre examen avait été négatif et la maman vous avait signalé pour seul trouble que l'enfant émettait, depuis pratiquement les
premiers jours de la vie, 4 à 6 selles par jour, un peu étendues, émises toujours au cours même de la tétée ou dans le 1/4h
qui suivait ; la prise de poids avait été jusqu'alors de 30g/j.

Compte tenu des données de l'examen du 15 novembre, vous deviez évoquer prioritairement à cette date :
A - Une intolérance aux proteines du lait de vache
B - Une intolérance congénitale au lactose
C - Une entérocolite infectieuse chronique
D - Une diarrhée prandiale non pathologique
E - Une mucoviscidose
Bonne(s) réponse(s) : D

L'emission de selles a lieu au moment de chaque tétée : pendant ou juste après. L'absence d'anomalie clinique et la prise de
poids normale évoquent l'absence de pathologie.
357
Exclusivement sur DOC - DZ : www.doc-dz.com NADJI 85
RESIDANAT EN POCHE TOME II
Cas Clinique en QCM

Quel examen, parmi les suivants, auriez-vous du demander à votre examen du 15 novembre ?
A - Une coproculture
B - Une parasitologie des selles
C - Un test de sueur
D - Un examen radiologique du grêle
E - Aucun examen
Bonne(s) réponse(s) : E

Découle de la réponse précédente.

Concernant la diarrhée des 12 heures le 1er décembre, l'hypothèse étiologique la plus vraisemblable est :
A - La décompensation aiguë d'une intolérance au lait
B - La décompensation aiguë d'une entérocolite chronique
C - Une gastro-entérite à rotavirus
D - Une infection bactérienne invasive
E - Une diarrhée liée à une infection parentérale
Bonne(s) réponse(s) : C

C'est la cause la plus fréquente des diarrhées du nourisson. Mais il faut systématiquement éliminer une infection parentérale :
surtout méningée ou urinaire à cet âge.

Concernant le traitement pour les 24 premières heures de la diarrhée récente, quelle est parmi les suivantes la
prescription la mieux adaptée :
A - Prescription d'un modificateur de la motricité intestinale
B - Hospitalisation d'urgence dans un service de pédiatrie générale la plus proche du domicile pour perfusion
C - Hospitalisation d'urgence dans un service de pédiatrie pour examens complémentaires indispensables
avant de pouvoir fixer la nature du traitement
D - Prescription d'un soluté type OMS
E - Mise en oeuvre d'emblée et pour 3 mois minimum d'un régime d'intolérance aux protéines du lait de vache
Bonne(s) réponse(s) : B

La perte du poids peut être estimée à 10%, valeur à partir de laquelle l'hospitalisation est systématique (de même en cas de
vomissements incoercibles ou de diarrhée très grave). L'attitude thérapeutique initiale est la réhydratation per os avec le
soluté O.M.S., peut être tentée à domicile devant l'examen clinique négatif et si le milieu parait compétent.

Un nourrisson de 6 mois est adressé aux urgences de l'hôpital par son médecin traitant parce que, depuis 24h, il tousse, est
dyspnéique avec polypnée à dominante expiratoire et tirage, fébrile à 39°C. Le thorax est distendu. L'auscultation pulmonaire
perçoit des râles sibilants dans les 2 champs, Le foie est plus abaissé qu'augmenté de volume. La radiographie du thorax
montre une distension pulmonaire bilatérale isolée; le coeur est de volume normal. On est en période hivernale.

Parmi les diagnostics suivants, indiquez celui que vous retenez en premier lieu :
A - Staphylococcie pleuro-pulmonaire
B - Bronchiolite probablement virale
C - Myocardite
D - Corps étranger endobronchique
E - Laryngite
Bonne(s) réponse(s) : B

C'est la cause la plus fréquente, donnant 1 tableau d'asthme fébrile.


Un diagnostic à toujours évoquer devant une dyspnée très fébrile chez 1 nourisson est la staphylococcie pleuro pulmonaire :
percussion à la recherche d'une masse pleurale ; radio pulmonaire si doute.

Parmi les agents suivants, indiquez celui qui est le plus souvent responsable de l'affection en cause :
A - Adénovirus
B - Staphylocoque
C - Virus respiratoire syncitial
D - Cox Bordet Gengou
E - Virus para influenzae
Bonne(s) réponse(s) : C

Rarement adénomes.

358
Exclusivement sur DOC - DZ : www.doc-dz.com NADJI 85
RESIDANAT EN POCHE TOME II
Cas Clinique en QCM

Cette affection peut se compliquer de :


A - Retard staturo-pondéral
B - Asthme
C - Pneumothorax
D - Abcès du poumon
E - Troubles de ventilation pulmonaire
Bonne(s) réponse(s) : B C E

B - Complication fréquente, recherche des antécédents familiaux


C - Complication mécanique
E - Dû à l'hypersécrétion et à l'encombrement par des bouchons muqueux

Quelles sont les 2 mesures thérapeutiques initiales à mettre en oeuvre ?


A - Corticothérapie
B - Antibiothérapie
C - Humidification de l'air ambiant
D - Kinésithérapie respiratoire
E - Traitement par anti inflammatoires non stéroïdiens
Bonne(s) réponse(s) : C D

La kinésithérapie respiratoire est l'élément le plus important, devant être entrepris le plus rapidement.
L'étiologie est la plus souvent virale (crainte d'une surinfection).

Devant des bronchopneumopathies récidivantes du nourrisson, quelle(s) est (sont) les facteurs favorisants que
l'on doit systématiquement rechercher ?
A - Mucoviscidose
B - Troubles de déglutition
C - Allergie
D - Reflux gastro-oesophagien
E - Un déficit en immunoglobulines
Bonne(s) réponse(s) : A B C D E

Egalement déficit en antitrypsine, corps étranger intrabronchique, compression par un arc vasculaire anormal.

Vous êtes appelé par les parents de Julien, 18 mois, parce qu'il vient de présenter des mouvements saccadés des membres
supérieurs. Il est fébrile. A l'examen, un quart d'heure plus tard, vous trouvez, un nourrisson en bon état général, conscient, il
a 40° de température et vous trouvez des signes de rhino-pharyngite. On note de plus à l'anamnèse, que l'enfant a eu la veille
le rappel du tétracoq. Il reçoit depuis sa naissance 1200 unités par jour de vitamine D.

Le diagnostic le plus probable est :


A - 1ère crise convulsive inaugurale d'une épilepsie
B - Convulsion hyperthermique
C - Convulsion hypocalcémique
D - Méningite purulente avec convulsions
E - Encéphalite post-vaccinale
Bonne(s) réponse(s) : B

Pathologie la plus fréquente entre 9 mois et 5 ans ; toujours éliminer une méningite.

Le traitement que vous proposez pour la rhinopharyngite fébrile est le suivant :


A - Instillation nasale de sérum physiologique
B - Aspirine par voie orale à raison de 80 mg/kg/24h
C - Ampicilline par voie parentérale
D - Corticothérapie par voie orale
E - Chloramphénicol par voie orale
Bonne(s) réponse(s) : A B

Le plus important est le traitement antithermique : découvrir l'enfant, bain de 2 degrés en dessous de la température de
l'enfant, aspirine (60 à 80 mg/kg/j) en 4 à 6 prises per os et/ou paracétamol (30 à 50 mglkg/j) en 4 prises. Pas d'antibiotique
durant une rhinopharyngite simple.

359
Exclusivement sur DOC - DZ : www.doc-dz.com NADJI 85
RESIDANAT EN POCHE TOME II
Cas Clinique en QCM

Si la crise se répète, le traitement doit comprendre :


A - Mise en position de sécurité
B - Instillation intrarectale de diazepam (0.30 - 0,50mg/kg)
C - Injection IM de phénobarbital
D - Hospitalisation immédiate dans un service de réanimation
E - Injection intraveineuse de gluconate de calcium
Bonne(s) réponse(s) : A B

Valium® à une dose de 0,5 mg/kg domine la lutte contre la fièvre (Cf supra).

Si vous avez posé le diagnostic de convulsions hyperthermiques, cela implique :


A - Une possibilité de récidive dans 30 à 40 % des cas
B - La possibilité d'un rachitisme hypocalcémique associé
C - La prophylaxie de récidive chez les enfants ayant une crise unilatérale
D- La prescription de Gardenal® ou d'acide valproïque en cas de prophylaxie jusqu'au moins l'âge de 4 ans
E - Un examen tomodensitométrique systématique
Bonne(s) réponse(s) : A C D

D - Uniquement en cas de facteur de risques : crise longue, unilatérale, survenue avant 12 mois, antécédents neurologiques
personnels ou familiaux. En cas de récidive, on peut soit traiter au coup par coup la fièvre en y associant du valium, soit faire
cette prophylaxie.

Jean Yves D.., âgé de dix huit mois, est hospitalisée en pédiatrie pour un volumineux hématome frontal consécutif à une
chute. C'est un deuxième enfant, il a une soeur âgée de 3 ans, il est né à terme, sans problème obstétrical ni néonatal, et son
développement staturo-pondéral et psychomoteur a été normal. On a cependant noté chez lui très tôt, l'apparition fréquentes
d'ecchymoses ou d'hématomes provoqués par des traumatismes souvent minimes, et cette tendance s'aggrave depuis les
premiers pas.
L'enquête ne permet pas de découvrir aucune prédisposition familiale hémorragique.
Le premier bilan d'hémostase a donné les résultats suivants :
- Temps de saignement (W=incision) : 6 minutes
- plaquettes : 380x10 exposant 9/l. (380.000/mm3)
- temps de Quick = 88 %
- temps de céphaline kaolin = 98 secondes pour un témoin à 35 secondes.

Ce premier bilan d'hémostase peut être complété et il faudra faire en priorité :


A - Un temps de coagulation
B - Un temps de Howell
C - Un thrombo-élastogramme
D - Une étude de la rétraction du caillot
E - Aucun des examens ci-dessus
Bonne(s) réponse(s) : E

Le temps de saignement étudie l'hémostase primaire dans son ensemble (D est inutile) ; quant au T.C.K. il remplace
avantageusement A,B et C.

Compte tenu des donées biologiques et du tableau clinique, quelle(s) anomalies de l'hémostase est-il possible
d'écarter d'emblée ?
A - Prothrombine (II)
B - Proaccélérine (V)
C - Proconvertine (VII)
D - Facteur Stuart (X)
E - Aucun des déficits ci-dessus
Bonne(s) réponse(s) : A B C D

T.C.K. allongé seul : l'anomalie est sur la voie endogène, en amont de la phase commune aux deux voies : soit sur l'un des
facteurs antihémophiliques, soit un facteur de la "phase contact" (XII,XI, facteur Fletcher, facteur Flaugeac).

Le déficit constitutionnel sévère en facteur Hageman (XII) a une expression biologique de base analogue à celle
de cette observation. Lequel des arguments ci-dessous peut-il être retenu contre ce diagnostic ? Le déficit en
facteur Hageman (XII) :
A - Est inexistant en Europe
B - Ne se révèle qu'à l'âge adulte
C - Ne s'exprime que par un purpura pétéchial spontané
D - N'entraîne d'hémorragies qu' en cas d'intervention chirurgicale
E - Ne s'accompagne d'aucune manifestation hémorragique
Bonne(s) réponse(s) : E

Un déficit en facteur XII n'entraine pas d'hémorragie ; par contre les thromboses sont fréquentes.
360
Exclusivement sur DOC - DZ : www.doc-dz.com NADJI 85
RESIDANAT EN POCHE TOME II
Cas Clinique en QCM

Une hémophilie A peut expliquer l'ensemble de ce tableau. Quelle(s) donnée(s), déjà recueillies ou restant à
recueillir est (sont) en faveur de ce diagnostic plutôt que de celui de la maladie de Willebrand ?
A - Temps de saignement normal
B - Absence de thrombopénie
C - Résistance capillaire normale
D - Activité coagulante du facteur VIII (VIII c) à 1 %
E - Normalité du taux d'antigène lié au facteur VIII (VIII R Ag)
Bonne(s) réponse(s) : A D E

La maladie de Willebrand s'accompagne d'un allongement du T.S., d'un déficit en facteur VIII, et d'une très forte diminution ou
absence du facteur de Willebrand. Par contre dans l'hémophilie T.S. et taux d'antigène lié au facteur VIII (= déterminant
antigénique de la molécule Willebrand) sont normaux.

Parmi les régles suivantes, indiquer celle(s) qu'il conviendra d'appliquer chez ce sujet hémophile :
A - Interdiction de toute activité sportive, même réduite
B - Interdiction des antalgiques salicylés
C - Interdiction des injections intramusculaires
D - Contre-indication de toute vaccination
E - Surveillance clinique articulaire et musculaire à intervalles réguliers
Bonne(s) réponse(s) : B C E

Eviter tout ce qui peut favoriser un saignement (traumatisme, médicament, injection intramusculaire, attention au traumatisme
thermométrique).

Un nouveau né de 2 950g est opéré d'une atrésie de l'oesophage. Au cours de suites opératoires, simples par ailleurs, la
perfusion veineuse saphène interne (au pied droit) a pu être retirée au 6ème jour. Au 15ème jour, la puéricultrice remarque
que l'enfant ne bouge. pas le membre inférieur droit qui est bloqué en légère flexion et dont la mobilisation est douloureuse.
L'hémogramme montre une polynucléose avec déviation de la formule d'Arneth vers des formes jeunes.

Quelle est l'investigation qui permettra le mieux le diagnostic d'atrésie oesophagienne ?


A - Radiographie pulmonaire
B - Introduction d'une petite sonde souple dans l'oesophage
C - Introduction d'une sonde rigide dans l'oesophage
D - Echotomographie
E - Radiographie abdominale
Bonne(s) réponse(s) : C

Manoeuvre systématique à la naissance, à compléter par intérêt à la seringue. Une petite sonde souple peut s'enrouler dans
le cul de sac oesophagien mais le test est alors négatif.

Quel est le signe observé au cours de la grossesse et qui aide au diagnostic d'atrésie oesophagienne à la
naissance ?
A - Anasarque foeto-placentaire
B - Hydramnios modéré
C - Dilatation gastrique à l'échotomographie
D - Obstacle oesophagien à l'échotomographie
E - Poussées d'éclampsie
Bonne(s) réponse(s) : B

Un hydramnios doit toujours faire chercher une malformation digestive.

Dans une atrésie de type III, on trouve sur la radiographie thoracique et abdominale :
A - Une ascension du diaphragme gauche
B - De l'air dans l'estomac et l'intestin
C - De l'air uniquement dans l'estomac
D - Un abdomen totalement opaque
E - Une dextro-cardie
Bonne(s) réponse(s) : B

Interruption avec cul de sac oesophagien d'amont borgne, distendu par la salive, le cul de sac oesophagien d'aval
communique avec la trachée par un trajet fistuleux permettent passage liquide gastrique dans le poumon.

361
Exclusivement sur DOC - DZ : www.doc-dz.com NADJI 85
RESIDANAT EN POCHE TOME II
Cas Clinique en QCM

La complication citée évoque pour vous :


A - Une arthrite septique de la hanche droite
B - Une phlébite à partir de la dénudation
C - Un abcès sur une perfusion sous-cutanée de sérum
D - Une rétention d'urine
E - Aucun de ces diagnostics
Bonne(s) réponse(s) : A

Complication à évoquer systématiquement, favorisée par les gestes septiques, le signal plus évocateur est un aspect de
"pseudo-paralysie" après intervalle libre.

Quel examen permet d'affirmer le diagnostic ?


A - Scintigraphie osseuse
B - Ponction de hanche
C - Radiographie du bassin
D - Phlébographie
E - Arthrographie opaque
Bonne(s) réponse(s) : B

C'est le seul qui affirme le diagnostic ; permet en outre l'isolement du germe et l'évacuation du pus.

Vous êtes amené à voir à votre consultation un enfant de 8 ans Les parents sont inquiets car en dépit d'un traitement par
gentamicine (Gentalline®) : 40 mg matin et soir par voie IM prescrit depuis 4 jours par un confrère antérieurement consulté, il
reste fébrile En effet la température que vous mesurez est à 39°.
L'enfant est abattu, se plaint d'une dysphagie douloureuse et de douleurs abdominales. Le poids est de 25 kg, le pouls à 130.
Les amydales rouges et oedématiées sont recouvertes par un enduit que votre abaisse-langue enlève aisément. Il existe un
purpura du voile du palais. Vous palpez des ganglions sous-maxillaires sensibles. Il n'y a pas de coryza ni de signes oculaires.
Les tympans sont discrètement congestifs, le ventre sensible mais souple. L'auscultation du coeur montre une tachycardie
régulière, un souffle systolique mésocardiaque 1/6 disparaissant en position debout. Vous évoquez une angine
streptococcique.

Quel antibiotique préconisez-vous ?


A - Péniciline G
B - Acide Nalidixique (Negram®)
C - Cotrimoxazole (Bactrim®)
D - Doxycycline (Vibramycine®)
E - Céfalotine (Keflin®)
Bonne(s) réponse(s) : A

L'antibiotique du choix est la pénicilline de préférence par voie buccale : pénicilline V 100 000 µkg/j pendant 10 jours.

Quelle est la cause de l'échec de la gentamicine ?


A - Mauvaise indication bactériologique
B - Durée insuffisante du traitement
C - Posologie insuffisante
D - Absence de concentration amygdalienne
E - Voie d'administration non adaptée
Bonne(s) réponse(s) : A

Les aminosides ayant une action cellulaire ne peuvent avoir d'action sur le gram (+) que si la paroi est altéré. Seuls, ils sont
donc inefficaces sur les streptocoques.

Quel risque cherchez-vous avant tout à prévenir ?


A - Maladie inflammatoire post-streptococcique
B - Pneumonie de déglutition
C - Otite aiguë
D - Sinusite aiguë
E - Laryngite aiguë
Bonne(s) réponse(s) : A

Glomérulonéphrite aigue et rhumatisme articulaire aiguë sont deux complications classiques et graves de l'angine
streptococcique non traitée.

362
Exclusivement sur DOC - DZ : www.doc-dz.com NADJI 85
RESIDANAT EN POCHE TOME II
Cas Clinique en QCM

Au moment où vous voyez l'enfant, quel examen peut, le plus sûrement, confirmer la nature streptococcique de
l'angine ?
A - Dosage des ASLO
B - Hémoculture
C - Prélèvement de gorge
D - NFS
E - Recherche d'une protéinurie
Bonne(s) réponse(s) : C

Le diagnostic biologique d'une infection est avant tout la mise en évidence du germe au niveau du foyer infectieux.

Trois semaines après la guérison clinique, quel examen complémentaire devriez-vous faire pratiquer ?
A - Protéinurie
B - Prélèvement de gorge
C - ASLO
D - Vitesse de sédimentation
E - Numération formule sanguine
Bonne(s) réponse(s) : A

Examen systématique, à faire à la recherche d'une atteinte rénale.

Vous êtes appelé d'urgence au chevet d'un enfant de 4 ans qui présente depuis 2 jours un syndrome infectieux avec
température à 39°C et un catarrhe rhinopharyngé dans un contexte épidémique viral. Depuis quelques heures se sont
installées progressivement des difficultés respiratoires; surtout lors de l'inspiration qui est bruyante. Toute tentative
d'alimentation est très difficile et douloureuse, même pour les liquides. Vous le trouvez dans son lit, en position demi-assise,
anxieux, respirant lentement et difficilement. Sa voix est étouffée. A chaque inspiration, les creux sus-claviculaires et sus-
sternal se dépriment. L'examen de la cavité buccale montre une rougeur pharyngée diffuse sans autres éléments. Le nez est
encombré de sécrétions muqueuses. A l'auscultation thoracique, le murmure vésiculaire est perceptible à droite comme à
gauche, symétrique. On entend quelques ronchus. Le pouls est à 110, le faciès coloré, les lèvres et les extrémités digitales un
peu cyanosées.

L'origine laryngée de cette dyspnée est évoquée devant :


A - Respiratoire lente
B - Prédominance inspiratoire
C - Dépression des creux
D - Dysphagie douloureuse
E - Voix étouffée
Bonne(s) réponse(s) : A B C

Une dyspnée laryngée est une bradypnée inspiratoire bruyante (cornage) et active (dépression des creux). Dysphagie et voix
étouffée évoquent un obstacle susglottique.

Le degré de gravité de cette dyspnée est apprécié par :


A - Installation depuis plusieurs heures
B - Cyanose discrète des lèvres et des extrémités
C - Température à 39°C
D - Rythme respiratoire lent
E - Prédominance inspiratoire
Bonne(s) réponse(s) : A B

Le diagnostic de gravité est clinique : installation aigue, signes d'hypoxie, signes d'hypercapnie.

Quels sont les 3 éléments essentiels de cette observation qui vous permette de porter le diagnostic
topographique précis de cette dyspnée laryngée ?
A - Dysphagie douloureuse
B - Hyperthermie
C - Sialorrhée
D - Position demi-assise
E - Cyanose des extrémités
Bonne(s) réponse(s) : A C D

Ces 3 éléments à eux seuls évoquent une épiglottite et contre-indiquent l'examen endo-buccal.

363
Exclusivement sur DOC - DZ : www.doc-dz.com NADJI 85
RESIDANAT EN POCHE TOME II
Cas Clinique en QCM

Quel est ce diagnostic parmi les propositions suivantes :


A - Laryngite oedémateuses sous-glottitique
B - Corps étranger laryngé
C - Epiglottite
D - Laryngite striduleuse (faux croup)
E - Croup
Bonne(s) réponse(s) : C

Cf questions précédentes.

Parmi les propositions suivantes concernant la conduite à tenir, quelle est celle que vous devez adopter ?
A - Surveillance stricte à domicile sous antibiothérapie intensive pendant quelques heures et hospitalisation
secondaire en cas d'aggravation
B - Hospitalisation immédiate avec transport par un moyen non médicalisé
C - Surveillance après un traitement sédatif
D - Hospitalisation immédiate par transport médicalisé, sous oxygène et surveillance médicale permanente
(SAMU) après injection de corticoïdes
E - Surveillance à domicile sous corticothérapie
Bonne(s) réponse(s) : D

Doit être transporté sous surveillance médicale continue, rapidement en position assise. Les corticoïdes peuvent avoir une
action sur l'oedème. Le geste d'urgence est l'intubation, en ayant à proximité le matériel de trachéotomie.

L'enfant A.B, est une fille âgée de 10 ans. Depuis 3 ans, elle a des infections urinaires à répétition (3 à 4 par an. La
symptomatologie clinique est toujours la même: pas de fièvre, brûlures mictionnelles, pollakiurie parfois hématurie terminale.
Le bilan urologique pratiqué six mois auparavant est normal (UIV et cystographie rétrograde) Le précédent bilan biologique
avait montré :
- NFS : 4 600 000 GR, 8 500 GB, 46 % PN
- VS : 8/20
- immunofluorescence de la bactériurie négative.
L'enfant vient consulter pour la même symptomatologie clinique avec le résultat du dernier culot urinaire, qui montre 10
exposant 5 colibacilles/ml, 1O exposant 6 polynucléaires altérés/ml.

Le diagnostic évoqué est :


A - Pyélonéphrite aigue
B - Cystite
C - Colique néphrétique
D - Vaginite
E - Autre réponse
Bonne(s) réponse(s) : B

Aucun signe d'infection urinaire haute (notamment présence de fièvre).

Le bilan biologique à pratiquer pour ce nouvel épisode comporte :


A - Electrophorèse des protides sanguins
B - Dosage du complément
C - Electrophorèse des protides urinaires
D - Antigène HBs
E - Aucune des mesures précédentes
Bonne(s) réponse(s) : E

Le seul examen nécessaire est l'examen des urines à la recherche du germe causal, et non antibiogramme.

Le bilan radiologique à pratiquer pour ce nouvel épisode comporte :


A - UIV
B - Cystographie rétrograde
C - Echographie rénale
D - Angiographie numérisée
E - Aucune des mesures précédentes
Bonne(s) réponse(s) : E

Le bilan consiste en une U.I.V. faite. Aucune indication à la refaire devant une symptomatologie identique.

364
Exclusivement sur DOC - DZ : www.doc-dz.com NADJI 85
RESIDANAT EN POCHE TOME II
Cas Clinique en QCM

En admettant que l'antibiogramme montre une sensibilité à toutes les drogues énumérées, le traitement à
mettre en route est :
A - Gentallin®e IM pendant 15 jours
B - Ampicilline et Gentalline® pendant 15 jours
C - Chloramphénicol pendant 8 jours
D - Furadoïne® pendant 10 jours
E - Aucune des mesures précédentes
Bonne(s) réponse(s) : E

Une cystite nécessite un traitement de 8 jours par un antiseptique urinaire ou un antibiotique.

Plusieurs jours après un contage avec un enfant atteint d'oreillons, une fillette de 6 ans développe un état fébrile à 40°
rapidement suivi d'une parotidite unilatérale. En 48h, l'atteinte salivaire se bilatéralise et la température se normalise. Vers le
8e jour cependant, apparaissent des céphalées intenses et des vomissements incoercibles.

Quelle a été la durée d'incubation la plus probable des oreillons ?


A - 3 à 6 jours
B - 1 semaine
C - 12 à 14 jours
D - 18 à 21 jours
E - 1 mois
Bonne(s) réponse(s) : D

La durée moyenne d'incubation est de 21 jours.

Quelle(s) proposition(s) peut (peuvent) s'appliquer à l'épidémiologie de la maladie ?


A - Transmission manuportée
B - La phase de contagiosité dure 8 jours
C - Maladie strictement humaine
D - Confère une immunité
E - Transmission indirecte possible
Bonne(s) réponse(s) : C D

Maladie contagieuse conférant une immunité solide (l'immunité de la mère assure celle de l'enfant), transmise par les
gouttelettes de Flügge. La contagiosité débute 8 jours avant et dure les 9 premiers jours de la maladie.

A la période d'invasion de la maladie, quel(s) symptôme(s) possède(nt) une bonne valeur d'orientation ?
A - Conjonctivite
B - Arthralgies fugaces
C - Douleurs à la pointe de la mastoïde
D - Turgescence du canal de Sténon
E - Pharyngite érythémateuse
Bonne(s) réponse(s) : C D

Egalement un peu de fièvre et quelques otalgies. En 24 à 36 heures phase d'état.

Quelle(s) proposition(s) peut (peuvent) s'appliquer à la parotidite ourlienne ?


A - Souvent bilatérale
B - Tuméfaction fluctuante
C - S'accompagne d'une asialie
D - S'accompagne d'otalgies
E - Peut être responsable d'une lithiase salivaire
Bonne(s) réponse(s) : A D

La parotidite d'abord unilatérale, se bilatéralise le plus souvent. Sa consistance est ferme. La douleur siège en avant du
conduit auditif et irradie vers l'oreille.

365
Exclusivement sur DOC - DZ : www.doc-dz.com NADJI 85
RESIDANAT EN POCHE TOME II
Cas Clinique en QCM

D'après l'énoncé ci-dessus, quelle(s) complication(s) doit-on craindre chez cette fillette ?
A - Pancréatite aiguë
B - Gastro-duodénite
C - Réaction méningée lymphocytaire
D - Sinusite maxillaire
E - Néphrite hypertensive
Bonne(s) réponse(s) : C

Localisation neuroméningée la plus fréquente, il s'agit d'une méningite lymphocytaire, elle est parfois muette cliniquement.

Le traitement de la forme régulière d'oreillons peut comporter :


A - Antithermiques
B - Macrolides pendant 8 jours
C - Injection de gammaglobulines spécifiques
D - Sédatif nerveux
E - Corticothérapie brève
Bonne(s) réponse(s) : A

Les salicylés sont actifs sur les douleurs ourliennes. On y associe l'isolement!

Quelle(s) est (sont) la (les) proposition(s) exacte(s) concernant la vaccination antiourlienne ?


A - Virus inactivé par le formol
B - Nécessite une injection de rappel tous les 3 ans
C - Obligatoire chez l'adolescente séronégative
D - Est contre-indiquée chez le diabétique
E - Peut être associée aux vaccins contre la rougeole et la rubéole
Bonne(s) réponse(s) : A E

Vaccin vivant atténué, éventuellement associé aux vaccins contre la rougeole et la rubéole (R.O.R.).

Un petit garçon de 13 mois est trouvé grognon, vers 15h30, après sa sieste. Il refuse son goûter. Au moment du change,sa
mère le trouve très chaud. Quelques minutes plus tard, il présente des secousses des 4 membres. Cet épisode dure environ
30 secondes, puis, après une brève phase de somnolence, l'enfant se réveille. La mère prend alors la température : 39° C.
Elle fait avaler à l'enfant un demi-comprimé d'aspirine à 0,5 g, et appelle son médecin traitant. Celui-ci diagnostique une crise
convulsive et adresse l'enfant à l'hôpital :

Une crise convulsive du nourrisson peut comporter :


A - Une modification du tonus
B - Une révulsion oculaire
C - Un changement de couleur de l'enfant
D - Un trouble respiratoire
E - Des mouvements saccadés des 4 membres
Bonne(s) réponse(s) : A B C D E

La majorité des crises communes du nourrisson sont des crises motrices bilatérales presque toujours cloniques dans les
convulsions hyperpyrétiques et toniques dans les lésions cérébrales.

Dans le cas de ce nourrisson, le diagnostic de convulsion hyperthermique simple pourra être posé si :
A - Il n'y a pas d'antécédent neurologique important
B - Si le LCR est normal
C - Si l'EEG montre une hypsarythmie
D - Si la glycémie est normale
E - Si la calcémie est basse
Bonne(s) réponse(s) : A B D

La ponction lombaire est faite au moindre doute clinique, ou si l'enfant est d'âge inférieur à 18 mois.

366
Exclusivement sur DOC - DZ : www.doc-dz.com NADJI 85
RESIDANAT EN POCHE TOME II
Cas Clinique en QCM

Les convulsions dites hyperthermiques :


A - Touchent l'enfant de 6 mois à 5 ans
B - Sont toujours des clonies
C - Peuvent se prolonger en un état de mal convulsif
D - Peuvent être dues à une encéphalite aiguë
E - Comportent un risque d'hémiplégie séquellaire
Bonne(s) réponse(s) : A C E

Ce type de convulsion peut se prolonger en état de mal convulsif, quand les convulsions généralisées ou latéralisées durent
plus de 30 minutes. Les convulsions de longue durée (supérieures à 15 minutes) peuvent être à l'origine de séquelles.

Dans la liste suivante, indiquez le facteur qui a une certaine influence sur le taux de récidive des convulsions
hyperthermiques de l'enfant :
A - Le caractère partiel ou généralisé de la crise
B - Le degré de l'hyperthermie
C - L'âge inférieur à 1 an
D - La longueur de la crise
E - La normalité de l'EEG intercritique
Bonne(s) réponse(s) : C

La récurrence des crises existe dans 40% des cas, surtout chez les filles et d'autant plus que la première crise a été plus
précoce.

Face à une crise convulsive fébrile qui se prolonge plus de 10 minutes chez un nourrisson, quelle(s) mesure(s)
thérapeutique(s) mettez-vous en oeuvre ?
A - Mise en position latérale de sécurité
B - Injection IM de gardénal
C - Bain
D - Administration orale d'aspirine
E - Injection intrarectale de Valium®
Bonne(s) réponse(s) : A C D E

En urgence A et E sont nécessaires.


Mais dès la fin de la crise : bain et administration orale d'aspirine.

Une jeune femme de 30 ans vient d'accoucher d'un enfant porteur d'une dysmorphie cranio-faciale: tête petite et ronde, faciès
plat, grosse langue sortie de la bouche, nez écrasé, obliquité des fentes oculaires en haut et en dehors, épicanthus, oreilles
peu ourlées.
L'enfant est cyanosé et polypnéique, on perçoit un souffle systolique 4/6 au milieu du sternum.
La radiographie thoracique montre un gros coeur globuleux et des poumons bien aerés.
Au 2ème jour, le méconium n'était pas évacué et l'enfant s'est mis à vomir avec de la bile. Il a été transféré aussitôt dans le
secteur de pédiatrie.

Quel diagnostic vous fait évoquer ce tableau ?


A - Un syndrome de Turner
B - Un syndrome de Noonan
C - Une mucoviscidose
D - Une trisomie 21
E - Une hypothyroïdie
Bonne(s) réponse(s) : D

Ce tableau est très évocateur de trisomie 21, d'autant que s'y associe une anomalie cardiaque et digestive.

Le diagnostic évoqué peut être confirmé :


A - Immédiatement par le test à la sueur
B - Ultérieurement par la recherche du corpuscule de BARR
C - Ultérieurement par la recherche de l'activité de la trypsine dans les selles
D - Ultérieurement par le caryotype
E - Ultérieurement par le dosage de la T4
Bonne(s) réponse(s) : D

A noter qu'en cas de suspicion de mucoviscidose c'est la réponse C qu'il faut répondre.

367
Exclusivement sur DOC - DZ : www.doc-dz.com NADJI 85
RESIDANAT EN POCHE TOME II
Cas Clinique en QCM

L'état de l'enfant fait supposer une détérioration grave de deux fonctions ou organes. Lesquels ?
A - Le coeur
B - Le cerveau
C - Les poumons
D - Le tube digestif
E - Les reins
Bonne(s) réponse(s) : A D

Malformation cardiaque et atteinte digestive (sténose duodénale) sont souvent associéess au diagnostic.

Le premier organe atteint mettant la vie de l'enfant en danger immédiat est :


A - Le cerveau
B - Les reins
C - Le foie
D - Le tube digestif
E - L'appareil respiratoire
Bonne(s) réponse(s) : D

Sans commentaire.

En fonction de cet organe, quelle attitude thérapeutique préconisez-vous ?


A - Une hormonothérapie
B - L'intervention exploratrice immédiate
C - L'oxygénothérapie contrôlée
D - La dialyse péritonéale
E - L'administration d'antibiotiques
Bonne(s) réponse(s) : B

C'est une évidence chirurgicale !

Quel type de lésion vous paraît la plus probable ?


A - Une hépatite nécrosante
B - Une atrésie intestinale haut située
C - Une volvulus gastrique
D - Un mégacôlon
E - Une athyréose
Bonne(s) réponse(s) : B

Etant donné l'association connue avec la trisomie.

Quel est le deuxième organe gravement atteint ?


A - Le cerveau
B - Le rein
C - Le foie
D - Le coeur
E - L'appareil respiratoire
Bonne(s) réponse(s) : D

Cf questions précédentes : le canal atrioventriculaire est la malformation la plus caractéristique.

Vous prescrivez le régime alimentaire d'un enfant âgé de 8 mois. Son poids de naissance était de 3 kg. Sa croissance
pondérale a toujours été normale jusqu'à ce jour, il n'a jamais été malade.

Parmi les laits suivants, quel est celui qui est le mieux adapté à son âge ?
A - Lait de vache pur
B - Lait de vache demi-écrémé
C - Lait maternisé
D - Lait 2ème âge
E - Lait concentré sucré
Bonne(s) réponse(s) : D

Dès le 5ème mois le lait du 1er âge peut être remplacé par du lait du 2ème âge.
Le lait de vache peut être introduit à 9 mois.

368
Exclusivement sur DOC - DZ : www.doc-dz.com NADJI 85
RESIDANAT EN POCHE TOME II
Cas Clinique en QCM

Cet enfant reçoit 4 prises alimentaires par 24 heures dont 2 biberons. Quelle quantité de lait exprimée en ml,
doit-il recevoir par biberon ?
A - 100 ml
B - 120 ml
C - 150 ml
D - 250 ml
E - 300 ml.
Bonne(s) réponse(s) : D

Nécessite recevoir 500 ml/jour de lait ou d'équivalents lactés.

Cet enfant ne reçoit de la viande qu'au repas de midi. Quelle quantité, exprimée en gr. de viande doit lui être
donnée au maximum ?
A-5g
B - 10 g
C - 20 g
D - 100 9
E - 200 g
Bonne(s) réponse(s) : C

Les quantités de viande, poisson, oeuf sont limitées par la nécessité d'un apport calcique suffisant.

Vous devez donner une supplémentation en vitamine D. Quelle sera la dose par 24 heures ?
A - 200 unités
B - 1200 unités
C - 3000 unités
D - 5000 unités
E - 10 000 unités
Bonne(s) réponse(s) : B

Soit 3 gouttes de Stérogyl® par jour jusqu'à 18 mois.

Un enfant de 8 ans présente, depuis 48 heures, une tuméfaction volumineuse intéressant les deux régions prétragiennes
soulevant les lobules de l'oreille, une otalgie et un tableau général d'infection modérée.

Quel diagnostic évoquez-vous ?


A - Rhinopharyngite suppurée
B - Oreillons
C - Mononucléose infectieuse
D - Sinusite aiguë
E - Aucune des propositions précédentes
Bonne(s) réponse(s) : B

Le diagnostic est évident !

Quel signe local recherchez-vous pour conforter votre diagnostic ?


A - Erythème pharyngé
B - Ulcérations linguales
C - Orifice du sténon oedématié
D - Tuméfaction du plancher buccal
E - Aucune des propositions précédentes
Bonne(s) réponse(s) : C

Rougeur du canal de Sténon.

369
Exclusivement sur DOC - DZ : www.doc-dz.com NADJI 85
RESIDANAT EN POCHE TOME II
Cas Clinique en QCM

L'affection en cause peut se compliquer de :


A - Orchite
B - Pancréatite
C - Méningite lymphocytaire bénigne
D - Rhumatisme articulaire aigu
E - Aucune des propositions précédentes
Bonne(s) réponse(s) : A B C

Il existe un double tropisme du virus ourlien :


- glandulaire responsable d'ourlite, de pancréatite
- neurologique dont l'atteinte la plus fréquente et la méningite lymphocytaire bénigne.

Quel traitement préconisez-vous dans la forme habituelle ?


A - Anti inflammatoire
B - Antibiotique
C - Abstention thérapeutique
D - Vaccination
E - Aucune des propositions précédentes
Bonne(s) réponse(s) : A

Les salicytés ont une bonne action contre les douleurs observées au cours des oreillons.

Marc, âgé de 5 ans, est hospitalisé pour un syndrome douloureux abdominal, qui a été précédé 24 heures auparavant par
l'apparition d'un purpura siègeant essentiellement aux membres inférieurs, avec quelques éléments aux membres supérieurs.
A l'entrée, l'enfant présente des douleurs à type de coliques, avec des vomissements répétés. Le diagnostic de forme
compliquée de purpura rhumatoïde est porté.

Le diagnostic de Purpura rhumatoïde est essentiellement clinique. Parmi les signes suivants, quel(s) est(sont)
le(s) signe(s) important(s) pour ce diagnostic ?
A - Pétéchies pouvant confluer en placards parfois infiltrés
B - Notion de topographie précise de l'éruption purpurique
C - Association fréquente à des phénomènes douloureux
D - Fréquence des épistaxis associés au purpura
E - Association fréquente avec une crise d'asthme.
Bonne(s) réponse(s) : A B C

Le purpura en principe pétéchial est favorisé par l'orthostatisme (se localise avec prédilection aux membres inférieurs), Il
s'associe fréquemment à des douleurs auriculaires et abdominales. Il n'y a pas de troubles de la coagulation.

Un bilan biologique est utile dans le purpura rhumatoïde, bien qu'aucun signe ne soit pathognomonique de
l'affection. Sont habituellement retrouvés un ou plusieurs des signes biologiques suivants :
A - VS modérément ou franchement accélérée
B - Une absence d'anomalie des tests explorant l'hémostase plasmatique
C - Une thrombopénie franche
D - Une augmentation des transaminases
E - Une augmentation des IgA sériques
Bonne(s) réponse(s) : A B C

L'accélération de la V.S. est inconstante. Augmentation des IgA sériques dans 50% des cas.

Le syndrome douloureux abdominal que présente Marc s'est accompagné. dans les heures qui ont suivi
l'hospitalisation, de l'apparition de selles hémorragiques. Ce tableau peut correspondre à plusieurs types de
lésions digestives. Laquelle(lesquelles) ?
A - Invagination intestinale
B - Appendicite aiguë
C - Poussée de rectocolite hémorragique
D - Purpura simple oedémateux et hémorragique de la paroi intestinale.
E - Aucun des propositions précédentes n'est exacte
Bonne(s) réponse(s) : A D

Les hémorragies digestives peuvent relever :


1) D'une invagination intestinale
2) D'une perforation intestinale
3) D'une hémorragie gastro-intestinale massive
4) D'une nécrose étendue de l'intestin
5) D'une occlusion par adhérence

370
Exclusivement sur DOC - DZ : www.doc-dz.com NADJI 85
RESIDANAT EN POCHE TOME II
Cas Clinique en QCM

Une atteinte rénale complique assez fréquemment le purpura rhumatoïde. Parmi les manisfestions suivantes,
cette atteinte rénale peut se traduire par :
A - Hématurie isolée
B - Une tubulonéphrite aiguë
C - Un syndrome néphritique avec hypertension artérielle.
D - Une protéinurie de faible importance transitoire
E - Un syndrome néphrotique
Bonne(s) réponse(s) : A C D E

L'atteinte rénale peut se réduire à une hématurie avec ou sans protéinurie. La biopsie rénale retrouve en général une
glomérulonéphrite segmentaire et focale, les formes très sévères répondent à une glomérulonéphrite membrano-proliférative.

Sur le plan physiopathologique, laquelle des propositions suivantes peut-être considérée comme exacte ?
A - Le purpura rhumatoïde correspond à une vascularite infectieuse
B - Le primum movens est un trouble fonctionnel des plaquettes
C - Il s'agit d'une vascularite toxique
D - L'affection est classée comme une vascularite immuno-allergique
E - Il s'agit d une vasculopathie hypertensive
Bonne(s) réponse(s) : D

Vascularite nécrosante la plus fréquente de l'enfant.

Dans la mesure où aucun traitement médical n'est franchement efficace dans cette affection, quelle est
l'évolution la plus habituelle dans le cas de Marc ?
A - Passage à la chronicité
B - Guérison rapide après la poussée qui a permis le diagnostic
C - Guérison après plusieurs poussées de purpura
D - Mort par insuffisance rénale aiguë
E - Evolution vers une collagénose
Bonne(s) réponse(s) : C

L'évolution est marquée par des poussées favorisées par l'orthostatisme. Le pronostic de la maladie est dû à l'atteinte rénale
imposant une surveillance rapprochée.

Un nourrisson de 8 mois, pesant 8 kg, a une diarrhée aiguë qui comporte de la fièvre (39° C le premier jour), des
vomissements et des selles aqueuses, l'examen clinique est normal.

Devant ce tableau, quel(s) argument(s) épidémiologique(s) plaide(nt) en faveur d'une infection à rota-virus :
A - L'origine ethnique de l'enfant
B - La survenue en période hivernale
C - La région où vit l'enfant
D - La notion d'épidémie
E - La notion de récidive chez le même enfant
Bonne(s) réponse(s) : B D

Les diarrhées à rotavirus sont les plus fréquentes, elles surviennent principalement en hiver et sont très contagieuses.

En supposant que l'enfant accepte de boire un soluté de réhydratation, lequel des 5 éléments suivants doit
conduire à l'hospitaliser :
A - 2 selles liquides en 3 heures
B - Otite bilatérale
C - Vomissements initiaux
D - Perte de poids de 800 g
E - Fièvre à 38°5 C
Bonne(s) réponse(s) : D

Toute perte de poids de 10 % au moins impose l'hospitalisation (traduit l'intensité de la déshydratation). Une diarrhée
incoercible ou des vomissements incessants imposent également l'hospitalisation.

371
Exclusivement sur DOC - DZ : www.doc-dz.com NADJI 85
RESIDANAT EN POCHE TOME II
Cas Clinique en QCM

Dans le bilan pratiqué à l'admission à l'hôpital parmi les altérations sanguines suivantes, indiquez celle(s) qui
tradui(sen)t une hémoconcentration par déshydratation :
A - Elévation de la calcémie
B - Hypernatrémie
C - Hyperprotidémie
D - Elevation de l'hématocrite
E - Hyperchlorémie
Bonne(s) réponse(s) : C D

B - Traduit une déshydratation intracellulaire.

Au plan thérapeutique, indiquez la (les) mesure(s) principale(s) à mettre en oeuvre chez ce nourrisson :
A - Administration d'un pansement intestinal
B - Supplémentation par l'eau sucrée
C - Administration orale d'un antibiotique
D - Suppression de l'apport lacté
E - Administration d'une solution type OMS
Bonne(s) réponse(s) : D E

Compensation des pertes grâce à une solution riche en électrolytes = soluté O.M.S. La reprise de l'apport lacté sera
progressive, ne devant pas commencer avant la 24ème heure.

Un enfant de 8 ans présente 8 jours après une angine, un oedème de la face et des urines anormalement colorées. A
l'examen, on note une prise de poids modérée n'excédant pas un kg et une TA à 130/85 mm Hg. Le diagnostic de
glomérulonéphrite aiguë post-angineuse parait le plus plausible.

Quelle est la caractéristique symptômatique habituelle de l'angine précédant la glomérulonéphrite aiguë (GNA) ?
A - Erythémateuse
B - Ulcéro-nécrotique
C - Pseudo-membraneuse
D - Vésiculaire
E - Hémorragique
Bonne(s) réponse(s) : A

Le plus souvent l'angine est érythémateuse, dûe à un streptocoque bêta hémolytique du groupe A.

Quel germe en est habituellement responsable ?


A - Staphylocoque
B - Entérocoque
C - Streptocoque
D - Germes gram négatif
E - Fuso-spirilles
Bonne(s) réponse(s) : C

Cf ci-dessus. Seul un petit nombre d'angines entraîne une glomérulonephrite A.

Quelle(s) perturbation(s) est(sont) habituelle(s) au début de la GNA ?


A - Hématurie
B - Hypocomplémentémie
C - Eosinophilie
D - Altération de la fonction rénale
E - Protéinurie
Bonne(s) réponse(s) : A B D E

La perturbation la plus importante et la diminution du CH50 et de C3,C4


A,D,E - Témoignent de l'atteinte rénale.

372
Exclusivement sur DOC - DZ : www.doc-dz.com NADJI 85
RESIDANAT EN POCHE TOME II
Cas Clinique en QCM

Le traitement que vous allez proposer devant ce tableau clinique comporte :


A - Corticoïdes
B - Corticoïdes + immuno-dépresseurs
C - Corticoïdes + pénicilline G
D - Pénicilline G
E - Régime désodé
Bonne(s) réponse(s) : D E

S'y associe le repos au lit pendant 10 jours.


Les corticoïdes sont utilisés dans le syndrome néphrotique.

Il est de mauvais pronostic de constater 3 mois après le début de cette maladie :


A - Taux élevé des antistreptolysines
B - Hypocomplémentémie
C - Hypertension artérielle
D - Fonction rénale altérée
E - Syndrome oedémateux
Bonne(s) réponse(s) : B C D E

Normalement le syndrome néphritique commence à régresser au bout de 48 heures.


B - Doit se corriger dans un délai d'un mois.

Dans une glomérulonéphrite aiguë bénigne, quelle(s) lésion(s) peut-on observer ?


A - Prolifération extracapillaire
B - Prolifération endocapillaire
C - Présence de polynucléaires dans les anses capillaires
D - Présence de thrombus dans les anses capillaires
E - Présence de nodules hyalins périphériques
Bonne(s) réponse(s) : B C

Glomérulonéphrite proliférative endocapillaire et exsudative.

Il est admis, dans une unité de Néonatologie, un nouveau-né de 33 semaines, d'une mère primipare, primigeste. La
grossesse, correctement suivie, a été marquée par une fièvre associée à un syndrome pseudogrippal, 10 jours avant
l'accouchement. L'enfant est né par voie basse. L'Apgar était à 7 à 1 minute et à 10 à 10 minutes. Le poids est 1 750 g. La
survenue d'une détresse respiratoire précoce (1ère heure de vie) a justifié son transfert en unité de Néonatologie.

Quel(s) est(sont) parmi les signes cliniques suivants, celui (ceux) qui
oriente(nt) le diagnostic vers celui d'infection néonatale ?
A - Hépatomégalie
B - Sclérème
C - Fontanelle déprimée
D - Splénomégalie
E - Hypothermie
Bonne(s) réponse(s) : A B D E

B - Signe très évocateur mais tardif et péjoratif.


A D - Signes observés le plus couramment.

Quel(s) est (sont), parmi les signes biologiques suivants, celui (ceux) qui permet(tent) d'évoquer le diagnostic
d'infection néonatale ?
A - Leucopénie à 5 000/mm3
B - Hyperfibrinémie supérieure à 4 9/1
C - Hypocalcémie
D - Thrombocytose supérieure à 500 000/mm3
E - Protéinorrachie à 0, 80 g/l
Bonne(s) réponse(s) : A B

La leucopénie ou au contraire l'hyperleucocyptose > 30 000/mm2 l'hyperfibrinémie avant 48 heures.


On insiste actuellement sur l'élévation de la C.R.P., mais celle-ci peut être retardée de quelques heures.

373
Exclusivement sur DOC - DZ : www.doc-dz.com NADJI 85
RESIDANAT EN POCHE TOME II
Cas Clinique en QCM

Parmi les arguments anamnestiques suivants, le (s) quel (s) plaide(nt) en faveur de l'infection néonatale ?
A - Infection urinaire maternelle
B - Rupture prématurée de la poche des eaux
C - Température maternelle supérieure à 38° lors du travail
D - Naissance par forceps
E - Leucorrhées
Bonne(s) réponse(s) : A B C E

B - Risque majeur si ouverture supérieure à 12 heures.

Le germe le plus probablement responsable de l' infection bactérienne suspectée chez ce nouveau-né est :
A - Staphylocoque
B - Streptocoque B
C - Pneumocoque
D - Colibacille
E - Listéria monocytogénes
Bonne(s) réponse(s) : E

Responsable d'un syndrome pseudo-grippal chez la femme enceinte : impose hémocultures et traitement par ampicilline.

Parmi les traitements antibiotique suivant, lequel est le plus adapté à la situation décrite ?
A - Ampicilline-gentamicine
B - Céphalotine-gentamicine.
C - Céfotaxine
D - Gentamicine
E - Methicilline-tobramycine
Bonne(s) réponse(s) : A

Association bactéricide sur la listéria. Traitement à poursuivre 10 jours.

Quelques jours après un contage avec un enfant atteint d'oreillons, une fillette de 6 ans développe un état fébrile à 40°,
rapidement suivi d'une parotidite unilatérale. En 48 heures, l'atteinte salivaire se bilatéralise et la température se normalise.
Vers le 8ème jour cependant, apparaissent des céphalées et des vomissements incoercibles.

Combien de temps avant le début des symptômes chez la fillette, s'est produit le contage entre les 2 enfants ?
A - 3 à 6 jours
B - 1 semaine
C - 12 à 14 jours
D - 18 à 21 jours
E - 1 mois
Bonne(s) réponse(s) : D

Durée moyenne = 16 j (12 - 24): la réponse C pourrait être valable.

Quelle(s) proposition(s) peut (peuvent) s'appliquer à l'épidémiologie de la maladie ?


A - Transmission par voie aérienne
B - La phase de contagiosité dure 8 jours
C - Maladie strictement humaine
D - Confère une immunité solide
E - Maladie très contagieuse
Bonne(s) réponse(s) : A B C D E

Tout est vrai.

Quelle(s) est(sont) la(les) localistion(s) des oreillons en dehors de la parotidite ?


A - Glandes sous-maxillaires et sublinguales
B - Testicule
C - Méningo encéphalite
D - Nerfs crâniens
E - Ovaires
Bonne(s) réponse(s) : A B C D E

D - Des surdités ont pu être observées.

374
Exclusivement sur DOC - DZ : www.doc-dz.com NADJI 85
RESIDANAT EN POCHE TOME II
Cas Clinique en QCM

Quelle(s) proposition(s) peut(peuvent) s'appliquer à la parotidite ourlienne ?


A - Rarement bilatérale
B - Tuméfaction fluctuante
C - S'accompagne d'adénopathies prétragiennes et sous-angulo-maxillaires
D - S'accompagne d'otalgies
E - Peut être responsable d'une lithiase salivaire
Bonne(s) réponse(s) : D

La tuméfaction fluctuante doit faire évoquer une pathologie O.R.L. suppurée (phlegmon).

D'après l'énoncé ci-dessus, quelle complication doit-on craindre chez cette fillette ?
A - Pancréatite aiguë
B - Gastro-duodénite
C - Méningite lymphocytaire
D - Sinusite maxillaire
E - Néphrite hypertensive
Bonne(s) réponse(s) : C

Mais elle est toujours bénigne.

Le traitement de la forme régulière d'oreillons comporte :


A - Antipyrétiques
B - Macrolides pendant 8 jours
C - Injection de gamma-globulines spécifiques
D - Sédatif nerveux
E - Corticothérapie
Bonne(s) réponse(s) : A

Maladie bénigne, aucun traitement dans la forme commune.

Quelle(s) est (sont) la (les) proposition(s) exacte(s) concernant la vaccination antiourlienne ?


A - Virus inactivé
B - Nécessite une injection de rappel tous les 3 ans
C - Est conseillée chez les enfants après l'âge d'un an
D - Est contre-indiquée chez le diabétique
E - Peut être associée aux vaccins contre la rougeole et la rubéole
Bonne(s) réponse(s) : C E

Comme tous les virus vivants le vaccin s'administre après 1 an d'âge.

François est âgé de 3 ans. Il est hospitalisé en urgence parce qu'en quelques heures s'est constitué un tableau associant :
température à 39°, dyspnée grave, refus alimentaire total par dysphagie semble-t-il. A l'examen, l'enfant demeure assis (sa
mère précise qu'il refuse d'être allongé), la dyspnée est intense avec important tirage sus-sternal, sus-claviculaire et iInter-
costal ; il demeure la bouche ouverte, la langue tirée avec hypersialorrhée. Dans les antécédents personnels, on note de
fréquentes rhino-pharyngites sans complication et dans les antécédents familiaux, une tante maternelle asthmatique.

Quelle précaution devez-vous observer en cas de transport ?


A - Assurer une vidange gastrique
B - Calmer l'enfant avec un sédatif
C - Administrer à l'enfant un bronchodilatateur d'action rapide
D - Maintenir l'enfant en position assise
E - Coucher l'enfant sur le dos la tête en hyperextension
Bonne(s) réponse(s) : D

Ne jamais coucher un enfant atteint d'épiglottite.

375
Exclusivement sur DOC - DZ : www.doc-dz.com NADJI 85
RESIDANAT EN POCHE TOME II
Cas Clinique en QCM

Quel(s) examen(s) est-il impératif de pratiquer en urgence avant toute décision thérapeutique parmi les suivants
:
A - Examen soigneux de la gorge
B - Laryngoscopie
C - Bronchoscopie
D - Radiographie du larynx
E - Aucun
Bonne(s) réponse(s) : E

Le diagnostic est évident et les examens dangereux.

En dehors des éventuels examens ci-dessus, indiquez parmi les suivantes la proposition correcte relative à la
conduite à tenir :
A - Mise en observation au repos et oxygénothérapie
B - Corticothérapie d'emblée + oxygène
C - Théophylline + oxygène
D - Traitement sédatif + oxygène
E - Intubation d'emblée ou, en cas d'echec, trachéotomie
Bonne(s) réponse(s) : E

Menace vitale à court terme.

Concernant l'affection présumée en cause, quelle est parmi les suivante,s la proposition exacte ?
A - Elle est habituellement due au virus respiratoire syncitial
B - Elle est habituellement due au mycoplasma pneumoniae
C - Elle est habituellement due à l'hemophilus influenzae
D - Elle ne comporte, malgré son caractère impressionnant. aucun risque
E - Il s'agit de la forme la plus fréquente des dyspnées laryngées infectieuses de l'enfant
Bonne(s) réponse(s) : C

20 % des hémophilus influenzae sont ampicilliné résistants.

Vous êtes appelé dans une famille qui compte trois jeunes enfants âgés respectivement de 6 ans, 4 ans, 20 jours. La mère et
l'îiné, âgé de 6 ans, sont atteints d'une toux sèche quinteuse depuis une dizaine de jours, sans reprise, ni expectoration, ni
vomissements, accompagnée d'un fébricule de 37,8 à 38,2°C. L'examen clinique ne révèle que quelques râles bronchiques
bilatéraux. Les deux enfants de 6 et 4 ans ont été vaccinés contre la coqueluche dans leur première année, avec rappel à 18
mois. Vous hésitez entre trois diagnostics: coqueluche chez les sujets partiellement immunisés, pneumonie atypique, et
trachéobronchite non spécifique.

Parmi les 5 examens suivants, choisissez celui(ceux) qui a(ont) valeur d'orientation entre ces trois hypothèses
diagnostiques :
A - Radiographie pulmonaire
B - Hémogramme
C - Sérologie coxiella burneti
D - Sérologie bordetella pertussis
E - Sérologie mycoplasma pneumoniae
Bonne(s) réponse(s) : A B E

C.D - Sont des sérologies qui ne se font pas en pratique courante.

Parmi les cinq traitements antibiotiques suivants, quel est celui que vous choisissez pour traiter les deux
patients, en l'absence de diagnostic de certitude ?
A - Pénicilline V
B - Amoxicilline
C - Tobramycine
D - Association pénicilline G + Tobramycine
E - Erythromycine
Bonne(s) réponse(s) : E

Son spectre couvre tous les micro-organismes évoqués ci-dessus.

376
Exclusivement sur DOC - DZ : www.doc-dz.com NADJI 85
RESIDANAT EN POCHE TOME II
Cas Clinique en QCM

Dans l'hypothèse d'une coqueluche, quel est le risque de la contamination du nouveau-né âgé de 20 jours
allaité au sein ?
A - Risque nul à cause de l'immunité transmise par les immunoglobulines d'origine maternelle
B - Risque nul à cause de l'immunité liée à l'allaitement maternel
C - Risque nul à cause de l'antibiotique prescrit à la mère et transmis par le lait maternel
D- Risque de coqueluche atténuée à cause de la faible réceptivité de la muqueuse respiratoire dans la
période néonatale
E - Risque de coqueluche mortelle
Bonne(s) réponse(s) : E

Risque de coqueluche mortel car pas d'Ac. transmis par la mère à l'enfant.

Quelle prescription faites-vous pour le nouveau-né ?


A - Aucune prescription
B - Séparation du milieu familial pendant 14 jours
C - Vaccination anticoquelucheuse
D - Injection de gammaglobulines anticoquelucheuses
E - Antibiothérapie prophylactique par spiramycine (Rovamycine®)
Bonne(s) réponse(s) : D

Les autres mesures n'ont pas d'intérêt.

Quelle est la durée moyenne de l'incubation de la coqueluche chez cet enfant nouveau-né ?
A - 3 jours
B - 10 jours
C - 20 jours
D - 4 semaines
E - 6 semaines
Bonne(s) réponse(s) : B

Connaissances.

Quelle est la durée prévisible de la période contagieuse des deux patients après le début du traitement
antibiotique ?
A - 2 jours
B - 7 jours
C - 14 jours
D - Pendant la durée de la toux quinteuse
E - Le traitement antibiotique n'a aucune action sur la contagiosité de la maladie
Bonne(s) réponse(s) : B

Connaissances.

Enfant Thierry, 6 ans, sans antécédents et au développement parfait jusqu'à maintenant. Motif de consultation : suspension
d'activité brève, au moment du repas, de la classe, jamais dans le jeu actif.
A l'examen, vous retrouvez l'absence totale d'anomalies, tant concernant le développement psycho-moteur et somatique que
concernant signes fonctionnels ou physiques. L'enfant ne peut décrire ses malaises qui sont uniquement remarqués par
l'entourage, brefs, mais stéréotypés. L'enfant a un comportement agité à la maison, beaucoup plus calme en classe.

Quelle exploration recommandez-vous pour affirmer le diagnostic ?


A - Calcémie-magnésémie
B - EMG avec recherche de tétanie
C - EEG de veille
D - EEG de veille et de sommeil
E - Examen psychologique
Bonne(s) réponse(s) : C

Un EEG de veille est normal en dehors des crises.

377
Exclusivement sur DOC - DZ : www.doc-dz.com NADJI 85
RESIDANAT EN POCHE TOME II
Cas Clinique en QCM

Des pointes-ondes à trois cycles par seconde par paroxysmes brefs généralisés sont identifiées à l'EEG. Le
diagnostic posé est :
A - Epilepsie astatomyoclonique de l'enfant
B - Epilepsie à paroxysme rolandique de l'enfant
C - Petit mal
D - Spasmophilie
E - Syncopes réflexes d'origine vagale
Bonne(s) réponse(s) : C

Sans commentaires.

Dans l'hypothèse que vous avez retenue, citez le(s) médicament(s) efficace(s) :
A - Magnésium per os
B - Phénobarbital
C - Ethosuccimide (Zarontin®)
D - Acide valproïque (Dépakine®)
E - Neuroleptiques du groupe des phénothiazines
Bonne(s) réponse(s) : C D

Sans commentaires.

La question des parents va immédiatement porter sur la longueur du traitement et le pronostic à long terme.
Vous pouvez leur répondre :
A - Que le traitement est à poursuivre indéfiniment
B - Que le pronostic est excellent et que les chances de guérison à la puberté s'établissent à 90%
C - Que le traitement est probablement en place pour 3 à 5 ans
D - Que les chances de guérison définitive à la puberté sont de 20 à 30%
E - Que les crises de grand-mal ont 20% de chances de survenir en cours d'évolution
Bonne(s) réponse(s) : B C

Sans commentaires.

Une femme, secondipare de 24 ans, met au monde par césarienne, avant le début du travail, prescrite en raison d'une
cicatrice utérine, un garçon de 4100g, au terme de 36 semaines. Le score d'Apgar est normal à une et 5 minutes. L' enfant est
laissé auprès de sa mère qui décide de l'allaiter. Au cours des premières heures de vie, il présente une tachypnée et un
discret tirage. Il est placé en incubateur. Son comportement neurologique est normal. Le pH est à 7.30, la PaC02 à 42 mmHg,
la PaO2 à 60 mmHg. La FiO2 égale à 0,3 à la naissance est ramenée rapidement à 0,21. A la 12ème heure, il présente des
accès de cyanose : calcémie 90 mg/l, glycémie 0,18 g/l, EEG normal. Mise en place d'une perfusion de glucose à 10% (60
ml/kg/24h) ; poursuite de l'alimentation ; à J3 ; bilirubine indirecte 150 micromol/l, Coombs direct négatif, mère O rh+, enfant B
rh-, hématocrite 69%, N.G. 6,2 millions/mm3, hémoglobine = 220 g/l.
Après 48 heures de photothérapie : bilirubine indirecte = 100 micromol/l.
Etat clinique satisfaisant. Retour auprès de la maman à J5.

Quelle est la nature la plus vraisemblable de la détresse respiratoire ?


A - Maladie des membranes hyalines
B - Inhalation méconiale
C - Retard de résorption du liquide pulmonaire
D - Infection pulmonaire
E - Pneumothorax
Bonne(s) réponse(s) : C

Il s'agit sans doute d'un retard de résorption, fréquent en cas de naissance par césarienne.

Quelle perturbation biologique est la plus vraisemblable pour expliquer l'acces de cyanose ?
A - Hypocalcémie
B - Acidose
C - Hypoglycémie
D - Hypokaliémie
E - Hypoxémie
Bonne(s) réponse(s) : C

Les hypoglycémies néonatales peuvent donner toutes sortes de manifestations cliniques.

378
Exclusivement sur DOC - DZ : www.doc-dz.com NADJI 85
RESIDANAT EN POCHE TOME II
Cas Clinique en QCM

Quelle pathologie maternelle doit être évoquée en premier lieu ?


A - Insuffisance placentaire
B - Diabète gestationnel
C - Hypertension artérielle
D - Hypocalcémie
E - Anémie
Bonne(s) réponse(s) : B

Probable hypoglycémie d'un nouveau-né de mère diabétique.

Quelle(s) est(sont) la(les) cause(s) favorisante(s) probable(s) de la détresse respiratoire ?


A - Césarienne
B - Prématurité
C - Insuffisance de surfactant
D - Dépression ventilatoire médicamenteuse
E - Extraction avant le début du travail
Bonne(s) réponse(s) : A

Cf question [191].

Quel est le mécanisme principal de l'ictère ?


A - Prématurité
B - Alimentation au sein
C - Incompatibilité ABO
D - Polyglobulie
E - Déficit en glycuronyl-transférase
Bonne(s) réponse(s) : A

L'ictère de B survient plus tard (3 à 5 j).

Quelle(s) complication(s) peut provoquer la photothérapie :


A - Hyperthermie
B - Déshydratation
C - Atteinte oculaire
D - Diarrhée par intolérance au lactose
E - Anémie
Bonne(s) réponse(s) : A B C

Les accidents sont fréquents et nécessitent une surveillance régulière.

On découvre, à l'occasion d'un examen de santé scolaire, une protéinurie à 3 + chez David, âgé de 6 ans.
Cet enfant a présenté, depuis le premier âge, des infections ORL répétées et, à 4 ans, une scarlatine. Sa mère présente une
protéinurie par glomérulonéphrite chronique. Cliniquement, David a de discrets oedèmes des membres inférieurs et un
hydrocèle. Sa tension artérielle est à 11 /6. Il ne présente aucun foyer infectieux évolutif, et son examen est par ailleurs normal.
Biologiquement :
- protéinurie : 70 mg/Kg/24 h
- Ectrophorèse des protéines urinaires : tracé très sélectif
- protidémie : 51 g/l
- albuminémie : 25 g/l
- créatininémie : 8 mg/l
- numération des leucocytes et des hématies/ml urine : leucocytes 1 000/ml ; hématies 5000/ml.

Quel est le diagnostic le plus probable :


A - Une malformation urinaire
B - Une glomérulonéphrite aiguë streptococique
C - Une néphrose lipoldique
D - Une polykystose rénale
E - Une protéinurie orthostatique
Bonne(s) réponse(s) : C

Il s'agit d'un tableau complet de syndrome néphrotique pur de l'enfant.

379
Exclusivement sur DOC - DZ : www.doc-dz.com NADJI 85
RESIDANAT EN POCHE TOME II
Cas Clinique en QCM

Dans ce cas, le diagnostic positif est établi sur :


A - L'existence d'oedèmes
B - Les antécédents ORL répétés
C - L'antécédent familial de glomérulopathie
D - Les résultats de l'étude quantitative et qualitative de la protéinurie
E - L'absence d'hématurie
Bonne(s) réponse(s) : D E

L'absence d'hématurie élimine une glomérulonéphrite aigüe.

Quel(s) autre(s) examen(s) biologique(s) est-il utile de pratiquer dans l'immédiat pour étayer le diagnostic et/ou
conduire le traitement ?
A - Mesure de la diurèse des 24 heures
B - Dosage des électrolytes urinaires des 24 heures
C - Ponction-biopsie rénale
D - Dosage du complément et de ses fractions C3 et C4
E - Echocardiographie
Bonne(s) réponse(s) : A B D

B - Permet la surveillance et l'équilibration des apports sodés, D n'est utile que pour éliminer toute autre pathologie
glomérulaire.

Parmi les traitements suivants, lequel retenez-vous ?


A - Aspirine 5 cg par kg par 24 h en 4 prises pendant 6 à 8 semaines
B - Prednisone 2 mg/kg/24 h pendant 4 à 6 semaines
C - Prednisone 4 mg/kg/24 h pendant 6 à 8 semaines
D - Repos au lit strict et régime hypoprotidique pendant 4 semaines
E - Aucun des traitements précédents
Bonne(s) réponse(s) : B

Le syndrome néphrotique pur de l'enfant réagit vite aux corticoïdes.

En plus du traitement déjà mentionné, il est justifié de prescrire :


A - Suppression du blanc d'oeuf
B - Régime désodé strict
C - Régime enrichi en calcium et en potassium
D - Régime enrichi en protéines
E - Aucune des propositions ci-dessus
Bonne(s) réponse(s) : B

Il existe une oligo-anurie ainsi qu'une rétention qui doivent être traitées par un régime peu ou pas salé.

On vous amène en consultation un nourrisson de 8 mois, sa famille le trouve retardé par rapport aux performances qu'avait
réalisées à son âge sa soeur ainée âgée de 2 ans 1/2. Il n'existe pourtant aucun facteur pathologique identifié à la naissance
hormis une prématurité (naissance à 32 semaines) ayant nécessité un séjour en incubateur, sans phénomènes pathologiques
notés à cette période. Depuis son retour à domicile, l'enfant n'a posé aucun problème de santé, s'est développé normalement
sur le plan somatique.
Lorsque vous l'examinez, le retard psychomoteur est manifeste : l'enfant tient sa tête mais ne peut maintenir la position
assise, se soulève sur ses avant-bras en position ventrale mais ne se retourne pas ; s'il réussit le "tiré-assis" c'est de façon
fragmentaire et peu puissante, la tête ayant encore tendance à rester un peu en arrière du plan du thorax dans cette
manoeuvre. Il suit des yeux convenablement, gazouille un peu, sourit en réponse, mais ne différencie pas visiblement les
personnes dans ses relations d'échange. Il ne saisit les objets que de façon élémentaire, et semble surtout utiliser la main
droite.
L'examen du tonus des différents segments des membres retrouve une asymétrie : tendance à l'hypertonie des extenseurs et
adduction au membre inférieur gauche; poing fermé et plutôt pronation du membre supérieur gauche. Le périmètre crânien est
normal ; la fontanelle, ouverte reste souple et battante. Les réflexes ostéotendineux sont plus vifs à gauche qu'à droite.

Chiffrez approximativement l'âge de développement de l'enfant :


A - 1 mois
B - 2 mois
C - 4 mois
D - 6 mois
E - 8 mois
Bonne(s) réponse(s) : C

Connaissances neurologiques.

380
Exclusivement sur DOC - DZ : www.doc-dz.com NADJI 85
RESIDANAT EN POCHE TOME II
Cas Clinique en QCM

Identifiez, parmi les syndromes suivants, celui qui semble correspondre à l'état clinique :
A - Syndrome de Little
B - Psychose précoce
C - Hémiplégie cérébrale infantile
D - Retard psychomoteur simple
E - Neuropathie périphérique congénitale asymétrique
Bonne(s) réponse(s) : C

Apparition d'une hémiplégie avec syndrome pyramidal.

Vous décidez de réaliser un bilan étiologique. L'examen tomodensitométrique cérébral a les plus grandes
chances de montrer :
A - Leucomalacie périventriculaire
B - Malformations de la ligne médiane : agénésie du corps calleux
C - Atrophie cérébelleuse et méga grande citerne
D - Asymétrie des hémisphères et des ventricules latéraux
E - Atrophie corticale globale et topographie bifrontale
Bonne(s) réponse(s) : D

A - Traduisant des séquelles d'hémorragies intraventriculaires massives.

Parmi les causes suivantes, désignez celle qui représente le facteur favorisant de ces troubles :
A - Petit poids de naissance
B - Prématurité
C - Hypocalcémie néonatale
D - Convulsion du 5e jour de vie
E - Craniosténose asymétrique
Bonne(s) réponse(s) : B

A.C - Ont été également incriminés.

Pour prendre en charge un tel enfant, il faut :


A - Surveiller attentivement l'EEG, même en l'absence de crises convulsives
B - Promouvoir la rééducation
C - Placer systématiquement l'enfant sous anticonvulsivants
D - Faire surveiller radiologiquement l'articulation de la hanche
E - Assurer une surveillance régulière du scanner cérébral
Bonne(s) réponse(s) : B D

Ce sont des mesures pratiques et efficaces; Elles améliorent le pronostic fonctionnel.


E - Peut éventuellement être pratiqué.

Un nouveau-né, prématuré, dont l'âge gestationnel est estimé à 36 semaines, présente une maladie des membranes hyalines
et nécessite une réanimation. L'évolution en est favorable mais à trois semaines de vie, à minuit, lors d'un change, on
constate une absence de gesticulation du membre inférieur gauche non observée précédemment ; l'examen clinique révèle
une limitation douloureuse de la mobilité de la hanche gauche par rapport à la droite.
L'enfant n'est pas fébrile,il existe une hyperleucocytose avec polynucléose.

Devant ce tableau, vous devez évoquer par principe :


A - Une ostéoarthrite septique de la hanche gauche
B - Une luxation congénitale de la hanche gauche
C - Un étranglement herniaire
D - Un décollement épiphysaire obstétrical fémoral supérieur
E - Une paralysie sciatique par injection médicamenteuse intrafessière
Bonne(s) réponse(s) : A
L'absence de fièvre n'élimine pas le diagnostic.

381
Exclusivement sur DOC - DZ : www.doc-dz.com NADJI 85
RESIDANAT EN POCHE TOME II
Cas Clinique en QCM

Parmi les examens complémentaires suivants, vous devez demander en priorité pour étayer votre diagnostic :
A - Une radiographie du bassin de face
B - Une scintigraphie osseuse
C - Un électromyogramme
D - Des hémocultures
E - Une radiographie d'abdomen sans préparation
Bonne(s) réponse(s) : A D

La radio de bassin peut, quelquefois montrer un écartement et une ascension de la tête fémorale (argument en faveur d'une
collection intra-articulaire).

Sur la radiographie du bassin de face faite en urgence, vous pourriez constater :


A - Latéralisation de la métaphyse fémorale supérieure gauche
B - Apposition périostée de la métaphyse fémorale supérieure gauche
C - Opacité accrue des parties molles de la hanche gauche
D - Lacune du noyau d'ossification épiphysaire supérieur du fémur
E - Absence de signes pathologiques
Bonne(s) réponse(s) : A C D E

Cette réponse montre la variabilité des signes radiologiques possibles (comme leur absence d'ailleurs), tout dépend du stade
évolutif.

Parmi les gestes suivants, quel(s) est(sont) celui(ceux) qui doit(vent) être réalisé(s) dans l'heure ?
A - Langeage en abduction
B - Prescription d'une antibiothérapie à large spectre associant deux drogues
C - Ponction articulaire de la hanche gauche
D - Laparotomie
E - Immobilisation platrée
Bonne(s) réponse(s) : C B E

Bien entendu, on commence par C.

Parmi les séquelles suivantes, vous pourriez constater à l'évolution :


A - Luxation de la hanche gauche
B - Inégalité de longueur des membres inférieurs
C - Destruction de la tête et du col du fémur gauche
D - Coxa-vara
E - Aucune
Bonne(s) réponse(s) : A B C E

Même remarque que pour la question 8 : tout dépend du stade évolutif.

Une fille de 2 ans est hospitalisée parce qu'elle souffre du ventre depuis 48 heures. Les douleurs évoluaient par crises ; elles
sont devenues permanentes. L'enfant a vomi, n'a pas de selles ; la température est à 37,3 degrés, le pouls à 140. Le visage
est pâle, la langue est sèche, le ventre respire mal, il est douloureux; la défense abdominale généralisée est évidente. Au
toucher rectal, l'ampoule rectale est vide, le doigtier ramène du sang noir. La radiographie sans préparation de l'abdomen
montre quelques niveaux liquides.

En urgence, vous jugez nécessaire de demander :


A - Numération formule sanguine
B - Lavement baryté
C - Groupe sanguin
D - Coproculture
E - Electrocardiogramme
Bonne(s) réponse(s) : A C

Pas de lavement baryté (risque de perforation digestive).

382
Exclusivement sur DOC - DZ : www.doc-dz.com NADJI 85
RESIDANAT EN POCHE TOME II
Cas Clinique en QCM

Dans l'énoncé du texte, vous auriez souhaité voir préciser :


A - La tension artérielle
B - La coloration des urines
C - La présence d'une cicatrice opératoire sur l'abdomen
D - L'état des orifices herniaires
E - L'existence d'une otite récente
Bonne(s) réponse(s) : A C D E

C D E - Font partie de l'examen complet que l'on doit faire, face à un enfant qui vomit.

Indiquez, parmi les suivants, le diagnostic que vous posez :


A - Péritonite appendiculaire
B - Gastro-entérite aiguë
C - Torsion d'une tumeur annexielle
D - Syndrome néphro-anémique
E - Invagination intestinale aiguë
Bonne(s) réponse(s) : E

Vu le jeune âge, c'est le diagnostic le plus préférable.

Indiquez votre choix thérapeutique :


A - Observation en milieu chirurgical 36 heures
B - Aspiration digestive et perfusion intraveineuse pendant 24 heures
C - Traitement antalgique et anti inflammatoire
D - Exploration chirurgicale de l'abdomen
E - Traitement diététique (solution glucose-électrolytes) et antibiotique (Bactrim®)
Bonne(s) réponse(s) : D

Il s'agit d'une invagination déjà ancienne, avec probable perforation.

Un garçon de 12 ans est hospitalisé pour la survenue depuis 48 heures d'une asthénie intense, de douleurs articulaires
siègeant au genou droit et aux deux chevilles avec impotence fonctionnelle, d'un fébricule à 38 degrés.
L'interrogatoire retrouve :
- que les douleurs articulaires ont débuté 48 heures auparavant, leur topographie se limitant alors au genou droit et au coude
gauche
- que 8 jours auparavant, il a présenté une dysphagie modérée, ayant duré 48 heures, avec fièvre à 38°5C, évolution
spontanément favorable n'ayant pas justifié de traitement médical.
L'examen clinique note un enfant eutrophique, anxieux, avec température à 38°4C ; l'examen articulaire montre effectivement
une atteinte de trois articulations : genou droit, et les deux chevilles qui sont chaudes, douloureuses à la mobilisation,
gonflées, avec choc rotulien droit. L'examen cardiovasculaire est normal, en dehors d'une tachycardie régulière à 120/mn; le
reste de l'examen est normal.

Parmi les diagnostics suivants, lequel vous parait le plus vraisemblable ?


A - Arthrite chronique juvénile à forme polyarticulaire
B - Adéno-virose
C - Rhumatisme post-streptococcique
D - Arthrite chronique juvénile à forme systémique
E - Méningococcémie
Bonne(s) réponse(s) : C

L'angine, les douleurs articulaires avec les arthrites des grosses articulations doivent faire évoquer le diagnostic.

Quels sont les deux examens complémentaires à réaliser en priorité pour argumenter votre diagnostic ?
A - Vitesse de sédimentation
B - Formule sanguine
C - Dosage des anticorps anti-nucléaires
D - Dosage des anticorps anti-DNA natif
E - Dosage des anti-streptolysines
Bonne(s) réponse(s) : A E

La V.S. affirme l'aspect inflammatoire, les antistreptolysines l'origine streptococcique.

383
Exclusivement sur DOC - DZ : www.doc-dz.com NADJI 85
RESIDANAT EN POCHE TOME II
Cas Clinique en QCM

Quel vous parait le risque évolutif le plus sévère chez cet enfant ?
A - Néphrite aiguë
B - Péricardite aiguë
C - Insuffisance mitrale
D - Chorée
E - Méningite bactérienne
Bonne(s) réponse(s) : B

La question est difficile mais le risque péricardique semble plus important que le risque rénal.

Quelle thérapeutique proposerez-vous une fois confirmé le diagnostic par les examens complémentaires ?
A - Pénicilline 100.000 U/kg/j en deux prises
B - Ampicilline 200 mg/kg/j en deux prises
C - Acide acétyl, salicylique 100 mg/kg/j en 4 prises
D - Pénicilline 100 000 U/kg/j en deux prises + acide acétyl salicylique 100 mg/kg/j en 4 prises
E - Pénicilline 100 000 U/kg/j en 2 prises + prednisone 2 mg/kg/j en deux prises
Bonne(s) réponse(s) : E

Il faut envisager l'association d'une antibiothérapie (pour traiter les foyers streptococciques) et d'un traitement anti-
inflammatoire (corticoïdes pour le R.A.A.).

Un enfant naît à terme par voie basse après une rupture des membranes de 48 heures, en présentation céphalique, le liquide
amniotique est clair. La grossesse s'est déroulée normalement, mais la mère est fébrile (39°C) au moment de l'accouchement.
L'Apgar est à 5 à une minute de vie, et l'enfant développe une détresse respiratoire immédiate, d'aggravation rapide,
nécessitant une intubation trachéale dès la 10ème minute. Après transfert en milieu spécialisé, l'examen clinique trouve une
hépatomégalie. L'hémodynamique centrale et périphérique est conservée. L'examen est normal par ailleurs. Les gaz du sang
mesurés sous ventilation assistée mettent en évidence une acidose métabolique profonde.

Parmi les diagnostics suivants, vous devez évoquer prioritairement chez cet enfant :
A - Inhalation méconiale
B - Maladie des membranes hyalines
C - Hernie diaphragmatique congénitale
D - Infection néonatale
E - Maladie métabolique
Bonne(s) réponse(s) : D

L'hyperthermie maternelle au moment de l'accouchement, l'acidose métabolique, font évoquer le diagnostic.

Parmi les examens complémentaires qui suivent, quel(s) est (sont) celui (ceux) qui vous parai(ssen)t
essentiel(s) et que vous devez pratiquer en urgence dès l'arrivée de l'enfant en Unités de Soins Intensifs
A - Radiographie du thorax
B - Prélèvements bactériologiques
C - Numération formule sanguine
D - Bilan d'hémostase
E - Chromatographie des acides aminés plasmatiques
Bonne(s) réponse(s) : A B C D

La chromatolographie n'a aucun intérêt immédiat.

Quel(s) est(sont) le(s) germe(s) susceptible(s) d'être trouvé(s) chez cet enfant :
A - Colibacille
B - Listéria monocytogènes
C - Streptocoque du groupe B
D - Pseudomonas aeruginosa
E - Serratia
Bonne(s) réponse(s) : A B C

A l'intérieur de ce groupe, les 2 plus vraisemblables sont colibacille et streptocoque B.

384
Exclusivement sur DOC - DZ : www.doc-dz.com NADJI 85
RESIDANAT EN POCHE TOME II
Cas Clinique en QCM

Outre la ventilation assistée, la thérapeutique que vous devez instituer initialement comporte :
A - Antibiothérapie d'emblée
B - Antibiothérapie débutée après résultats des prélèvements bactériologiques
C - Remplissage Vasculaire
D - Exsanguino-transfusion
E - Perfusion de bicarbonates
Bonne(s) réponse(s) : A E

L'antibiothérapie est rapidement débutée ainsi que l'alcalinisation.

Un nourrisson de 6 mois et demi est hospitalisé pour une toux, de la dyspnée et de la fièvre. On ne trouve pas d'antécédents
pathologiques particuliers. Cinq jours auparavant, l'enfant était grognon, sa température était à 38°C, on notait une toux, une
rhinorrhée puis l'état de l'enfant s'aggrave avec dyspnée, apathie et difficultés d'alimentation. A l'examen, le nourrisson est
inquiétant, sa température est à 40°C, il est abattu, et a un aspect grisâtre. La dyspnée avec polypnée est manifeste, elle
s'accompagne d'un tirage modéré et de toux. Le poids est de 6 kg, il n'y a pas de signe de déshydratation. L'examen du thorax
est normal à droite, à gauche, on met en évidence à la base, une matité avec disparition du murmure vésiculaire, Le foie et la
rate ne sont pas palpables, l'examen cardio-vasculaire et l'examen neurologique sont normaux. La radiographie du thorax
montre les anomalies suivantes :
- à droite, on observe deux zones d'opacités non systématisées aux limites imprécises, il s'y associe une image claire arrondie
de 2 cm de diamètre
- à gauche, il existe une opacité nette de la base comblant le cul-de-sac costo-diaphragmatique et à la partie supérieure du
poumon, il existe une bande claire entre le contour externe du poumon, et la paroi ; la silhouette cardiaque est normale.
L'hémogramme montre 20 000 globules blancs/mm3 avec 85 % de polynucléaires neutrophiles. L'examen des urines
n'objective ni glucosurie, ni protéinurie.

Quel est le diagnostic le plus probable ?


A - Méningite purulente
B - Pyélonéphrite aiguë
C - Bronchiolite virale
D - Pneumopathie à mycoplasme
E - Staphylococcie pleuro-pulmonaire
Bonne(s) réponse(s) : E

Le diagnostic est évoqué :


- sur l'aspect radiologique (opacité, bulles pulmonaires, épanchement pleural),
- l'auscultation pulmonaire,
- le contexte fébrile et la N.F.S.

Quel(s) examen(s) complémentaire(s) pourrai(en)t le confirmer ?


A - Numération formule sanguine
B - Hémoculture
C - Coproculture
D - Uroculture
E - Ponction pleurale
Bonne(s) réponse(s) : B E

Seul le prélèvement local (plèvre) et général (hémoculture), à visée bactériologique, aideront au diagnostic.

Dans la description clinique, un symptôme pratiquement constant dans cette affection n'a pas été mentionné,
précisez lequel ?
A - Herpès labial
B - Raideur de la nuque
C - Ballonnement abdominal
D - Hématurie
E - Purpura pétéchial ecchymotique
Bonne(s) réponse(s) : C

Le ballonement abdominal accompagne les manifestations pulmonaires.

Quelle association d'antibiotiques utiliserez-vous, sur les seules données de la clinique et de la radiologie ?
A - Association de deux bêta lactamines
B - Association d'une bêta lactamine et d'un aminoside
C - Association d'une cycline et d'un aminoside
D - Association d'un macrolide et d'un aminoside
E - Association d'une cycline et d'un macrolide
Bonne(s) réponse(s) : B

L'association bêta lactamine-aminoside est synergique et rapidement bactéricide.

385
Exclusivement sur DOC - DZ : www.doc-dz.com NADJI 85
RESIDANAT EN POCHE TOME II
Cas Clinique en QCM

Comment nomme-t-on les lésions dont la plèvre gauche est le siège ?


A - Foyer pneumonique
B - Miliaire pulmonaire
C - Pleurésie purulente
D - Pneumothorax
E - Pyopneumothorax
Bonne(s) réponse(s) : E

Il existe un épanchement liquidien à la base pleurale gauche et un épanchement gazeux à la partie supérieure : le risque
d'inoculation septique est maximum.

Quelle attitude proposez-vous pour le traitement de cette lésion ?


A - Abstention thérapeutique
B - Exsufflation pleurale à l'aiguille
C - Drainage pleural avec aspiration
D - Fibroscopie bronchique
E - Thoracotomie évacuatrice
Bonne(s) réponse(s) : C

Un double drainage avec un éventuel rinçage doit permettre d'éviter une thoracotomie.

Nicolas, âge de 10 ans, est hospitalisé dans le service de pédiatrie par son médecin traitant pour une éruption purpurique
apparue des jours auparavant au niveau des fesses et des genoux. Cette éruption s'accompagne de douleurs avec
gonflement articulaire au niveau des genoux, chevilles et mains, et d'une fébricule à 38°. Dans les jours qui ont précédé, on
note simplement une rhinopharyngite. Son médecin traitant ne trouvait rien d'anormal à l'examen clinique en dehors de
l'éruption et des manifestations articulaires et avait demandé un bilan simple dont les résultats sont les suivants :
- GR : 4 500 000/mm3, GB 80000/mm3 dont PN 0,51, Eo 0,01, L 0,43, M 0,05
- Plaquettes : 300 OOO/mm3
- Temps de saignement : 3 mn, temps de coagulation : 8 mn, fibrinémie: 5 g/l, TP 90 %
- Hg : 120 g/l, Ht : 38 %, VGM : 94 ng, TGMH : 29 ng, CCMH : 31 %
- VS : 66 mm à la 1ére heure
ASLO : 250 Ul/l
Mais l'enfant reprenant une vie normale, on constate la réapparition du purpura sans arthralgies ; l'examen retrouve une
douleur à la mobilisation des genoux. A l'entrée dans le service motivée par cette récidive, on constate :
Sur le plan clinique :
- examen cardiopulmonaire normal
- foie et rate non perçus
- absence d'adénopathies
- pétéchies des membres Inférieurs
- abdomen souple et dépressible.

Le purpura peut être affirmé en raison du(des) caractère(s) suivant(s) :


A - Apparition spontanée ou provoquée par un traumatisme minime
B - Caractère fébrile
C - Ne s'efface pas à la vitropression
D - Passe par les teintes de la biligénèse locale
E - Survenue dans les suites d'une rhinopharyngite
Bonne(s) réponse(s) : A C

La définition du purpura est clinique et simple.

Parmi les examens déjà pratiqués, quel est celui dont le résultat est le plus important ?
A - Taux de prothrombine
B - Temps de coagulation
C - Numération des plaquettes
D - Dosage ASLO
E - Fibrinémie
Bonne(s) réponse(s) : C

Il s'agit donc d'un purpura non thrombopénique.

386
Exclusivement sur DOC - DZ : www.doc-dz.com NADJI 85
RESIDANAT EN POCHE TOME II
Cas Clinique en QCM

Au plan étiologique, le purpura de cet enfant est caractérisé par :


A - L'influence de l'orthostatisme
B - Le contexte fébrile
C - La VS accélérée à 66 mm à la 1ère heure
D - La localisation préférentielle aux membres inférieurs
E - Les poussées récidivantes
Bonne(s) réponse(s) : A C D E

Ces caractères caractérisent le purpura rhumatoïde.

Parmi les complications de ce type de purpura, laquelle conditionne le pronostic à distance ?


A - Invagination intestinale aiguë
B - Crises convulsives
C - Glomérulopathie
D - Localisation testiculaire
E - Défaillance cardiaque
Bonne(s) réponse(s) : C

La glomérulopathie est la complication la plus sévère.

Parmi les solutions thérapeutiques suivantes, vous devez envisager :


A - Corticothérapie
B - Repos au lit
C - Diurétiques
D - Perfusion de plasma frais
E - Abstention médicamenteuse
Bonne(s) réponse(s) : A B

La corticothérapie peut cependant être discuté dans ce cas précis (pour certains, elle serait abusive et sans intérêt).

L'enfant G Manuel, 7 ans, a depuis l'âge de 2-3 mois des lésions prurigineuses qui ont débuté à la face et qui se sont
progressivement généralisées ; les phases d'aggravation ont surtout été hivernales et marquées par des surinfections
cutanées staphylococciques et herpétiques. A partir de l'âge de 2 ans, l'enfant a également eu des accès de dyspnée et de
fréquentes infections ORL et bronchiques. Son état cutané et respiratoire s'améliore nettement en été, surtout en altitude. Il
n'a pas eu d'autres maladies graves et a eu toutes les vaccinations obligatoires sans complications. L'examen clinique montre
: une peau uniformément sèche et squameuse ; des lésions d'eczéma de la face et du cou, des plis des coudes, des mains et
des creux poplités ; de nombreuses cicatrices de grattage ; des adénopathies périphériques axillaires et inguinales ; une
respiration bruyante avec des ronchus et des sibilances à l'auscultation. L'enfant est nerveux et ne cesse de se gratter
pendant l'examen clinique. L'interrogatoire apprend que sa mère avait des lésions d'eczéma pendant l'enfance; il n'y a pas
d'autres antécédents familiaux. Manuel est fils unique ; ses parents sont très préoccupés par son état de santé ; le prurit rend
l'enfant insomniaque ; son développement staturo-pondéral est ralenti malgré un bon appétit et une alimentation normale ; sa
fréquentation scolaire est irrégulière du fait des poussées d'eczéma et des surinfections bactériennes nécessitant des soins à
domicile.

A ce stade, votre sélection d'hypothése(s) diagnostique(s) comporte:


A - Une dermatite atopique
B - Une ichtyose
C - Une pollinose
D - Une dermatite herpétiforme
E - Un asthme
Bonne(s) réponse(s) : A

A la limite, l'ichtyose comme terme évolutif d'un eczéma vieilli aurait pu être retenue.

La biologie devrait, significativement, mettre en évidence :


A - Une leucopénie inférieure à 3000 G B /mm3
B - Une éosinophilie supérieure à 800/mm3
C - Une hypergammaglobulinémie polyclonale
D - Une augmentation isolée des IgE sériques
E - Une hypocalcémie
Bonne(s) réponse(s) : B D

Ce sont des signes biologiques accompagnant souvent l'eczéma.

387
Exclusivement sur DOC - DZ : www.doc-dz.com NADJI 85
RESIDANAT EN POCHE TOME II
Cas Clinique en QCM

Le bilan diagnostique immunologique peut utilement comporter :


A - Une immunoélectrophorèse des protéines sériques
B - Des tests cutanés à lecture immédiate (20 min.) pour chercher une sensibilisation réaginique aux
pneumallergènes
C - Des RAST-lgE pour les mêmes pneumallergènes
D - Un dosage de l'histaminémie
E - Un dosage du complément sérique
Bonne(s) réponse(s) : A B C

Certains déficits en Ig peuvent s'accompagner d'eczéma.

En cas de sensibilisation réaginique aux pneumallergènes, quel est le peumallergène qui donnera le plus
probablement des tests cutanés ou des RAST spécifiques positifs chez cet enfant ?
A - Les pollens de graminées
B - Les squames humaines
C - Les poils d'animaux
D - Les acariens dermatophagoïdes de la poussière
E - Les moisissures
Bonne(s) réponse(s) : D

L'absence d'acariens en altitude explique l'amélioration de son état respiratoire.

Quelle est parmi les proposiotns thérapeutiques suivantes, celle qu'il faut rejeter à priori chez cet enfant ?
A - La corticothérapie générale
B - L'antibiothérapie générale adaptée aux staphylocoques
C - La théophylline
D - Les antihistaminiques anti-H1
E - Le cromoglycate disodique oral ou en inhalations
Bonne(s) réponse(s) : A

Pas de corticothérapie.

Un nourrison de 12 mois est amené aux urgences pour une détresse respiratoire. L'examen révèle une fréquence respiratoire
à 46/mn, un tirage sus-stermal et intercostal modéré, une expiration active, des râles sibilants à l'auscultation. La température
est à 37°6C.

Quel est l'examen complémentaire utile pour confirmer le diagnostic de bronchiolite :


A - Radiographie pulmonaire
B - Gaz du sang veineux
C - Hémoculture
D - Numération formule sanguine
E - Prélèvement de gorge
Bonne(s) réponse(s) : A

Connaissances.

Si vous prescrivez un bronchodilatateur de type théophylline, quelle posologie utilisez-vous ?


A - 0,5 mg/kg/jour
B - 5 mg/kg/jour
C - 10 mg/kg/jour
D - 15 mg/kg/jour
E - 20 mg/kg/jour
Bonne(s) réponse(s) : D

La dose habituelle varie de 10 à 20 mg/kg/j.

388
Exclusivement sur DOC - DZ : www.doc-dz.com NADJI 85
RESIDANAT EN POCHE TOME II
Cas Clinique en QCM

Parmi les thérapeutiques suivantes, il est inutile de prescrire :


A - Salbutamol
B - Corticoïdes
C - Tétracyclines
D - Kinésithérapie
E - Aerosol fluidifiant
Bonne(s) réponse(s) : C

Seules, les tétracyclines sont contre-indiquées.


Pour certains, le salbutamol est inefficace à cet âge.

La surveillance repose sur :


A - Fréquence respiratoire
B - Régularité respiratoire
C - Signes de lutte
D - Etat de conscience
E - Fréquence cardiaque
Bonne(s) réponse(s) : A B C D E

La bronchiolite peut donner des hypoxies majeures : donc surveillance.

Un jeune garçon de 13 ans, sans antécédent notable, se plaint d'une douleur de la cuisse gauche, récente, de survenue
spontanée, et sans notion de traumatisme antérieur. Il mesure 1 m 65 et pèse 75 kg.
A l'examen il se présente en position vicieuse du membre inférieur gauche en discret flexum de hanche et en rotation externe.
La mobilité articulaire des hanches sur le plan du lit est de :

________________________________________________________________
| | Flex | Ext. | Abd. | Add. | Rot. ext. | Rot. int. |
|________|________|_______|_________|______|____________|____________|
| Droite | 130 | 0 | 45 | 45 | 50 | 40 |
| Gauche | 80 | -10 | 20 | 20 | 70 | 10 |
|________|________|_______|_________|______|____________|____________|

Cette immobilisation s'accompagne à gauche, de douleurs modérées de la hanche en fin d'amplitude.


Le genou gauche est sec, non inflammatoire, de mobilité normale, indolore. La rotule, mobile et indolore est bien axée. La
cuisse gauche est souple, froide. Le rachis est mobile et non douloureux. La température est normale.
Enfin la marche s'effectue avec esquive du pas du côté gauche et attaque du sol en rotation externe.

Devant ce tableau clinique, quel diagnostic évoqueriez-vous ?

A - Arthrite septique de hanche


B - " Rhume" de hanche
C - Ostéochondrite de hanche
D - Epiphysiolyse de la tête fémorale
E - Luxation de hanche
Bonne(s) réponse(s) : D

Sans commentaire.

Quel ou quels examens vont étayer ce diagnostic ?


A - Radiographie du bassin de face
B - Radiographie en faux profil de Lequesne
C - Numération formule sanguine et vitesse de sédimentation
D - Tomographie de la hanche gauche
E - Radiographie des 2 hanches de profil
Bonne(s) réponse(s) : A E

Sans commentaire.

Quelle attitude thérapeutique impose ce tableau ?


A - Immobilisation stricte sur le plan du lit
B - Immobilisation en traction des membres inférieurs
C - Ponction articulaire de la hanche
D - Immobilisation du membre inférieur gauche par plâtre pelvi-pédieux
E - Traitement chirurgical
Bonne(s) réponse(s) : E

Sans commentaire.
389
Exclusivement sur DOC - DZ : www.doc-dz.com NADJI 85
RESIDANAT EN POCHE TOME II
Cas Clinique en QCM

L'évolution spontanée possible à court terme peut être marquée par :


A - Raideur articulaire de la hanche gauche
B - Destruction de la hanche gauche
C - Luxation de la hanche gauche
D - Douleur de la hanche droite
E - Aucune des propositions précédentes
Bonne(s) réponse(s) : A D

Sans commentaire.

Quelle complication doit on craindre à terme chez ce patient ?


A - Arthrose lombo-sacrée
B - Coxarthrose
C - Raideur du genou
D - Inégalité de longueur des membres inférieurs
E - Aucune des propositions précédentes
Bonne(s) réponse(s) : B

Sans commentaire.

Une fille de 2 ans, a présenté pendant 3 jours une fièvre en clochers à 39-40 degrés, associée à quelques troubles digestifs,
des douleurs abdominales, des difficultés urinaires, pleurs à la miction. Dans ses antécédents, on retrouve il y a 6 mois un
premier épisode similaire.
L'examen cytobactériologique et chimique des urines montre : nombreux leucocytes altérés, quelques hématies , E. coli 10
exposant 7/ml, protéine : 0,30 g/l.
A la fin du traitement, une urographie intraveineuse est pratiquée qui montre l'absence d'uropathie obstructive, un uretère droit
atone.
La cystographie réalisée confirme qu'il s'agit d'un reflux vésico-urétéral droit stade III (reflux complet avec dilatation des voies
excrétrices).

L'antibiogramme montre que le germe E. coli est sensible aux médicaments suivants. Lequel choisirez-vous
pour traiter à domicile l'infection urinaire de ce nourrisson ?
A - Colistine
B - Ampicilline + Acide. clavulanique (Augmentin®)
C - Carbénicilline
D - Acide nalidixique
E - Furanes
Bonne(s) réponse(s) : B

Sans commentaire.

Quelle exploration complémentaire est indispensable pour aider à l'indication thérapeutique ?


A - Echographie rénale
B - Scintigraphie rénale
C - Ponction biopsie rénale
D - Epreuve de concentration urinaire
E - Aucun de ces examens
Bonne(s) réponse(s) : E

Sans commentaire.

Quel est le risque évolutif à long terme d'un reflux vésico-urétéral stade III non traité ?
A - Cancérisation
B - Trouble de croissance du rein
C - Pyonéphrose
D - Phlegmon périnéphrétique
E - Lithiase rénale
Bonne(s) réponse(s) : B

Sans commentaire.

390
Exclusivement sur DOC - DZ : www.doc-dz.com NADJI 85
RESIDANAT EN POCHE TOME II
Cas Clinique en QCM

Devant un reflux vésico-urétéral de stade III, avec orifice urétéral béant et latéralisé, quelle est la conduite à
tenir ?
A - Néphrectomie
B - Traitement antiseptique continu sans traitement chirurgical
C - Abstention thérapeutique
D - Réfection chirurgicale du trajet urétéral intravésical
E - Contrôle bactériologique des urines mensuel et traitement de chaque infection urinaire au coup par coup
Bonne(s) réponse(s) : D

Sans commentaire.

Vous êtes appelé d'urgence, un dimanche à 20 H 00, auprès d'un nourrisson de 10 mois pesant 8 Kg, pour une fièvre durant
depuis 2 jours et demi à 39 degrés. La fièvre est alors à 39.7 degrés. L'interrogatoire ne révèle pas de contage infectieux dans
l'entourage, mais l'enfant fréquente la crèche. Pas de notion de diarrhée, ni de vomissement. L'auscultation cardiaque et
pulmonaire est normale. Le pouls est à 130/mn. L'abdomen est souple, sans tuméfaction perceptible : le foie dépasse de 1 cm
le rebord costal et la rate n'est pas perçue. Pas d'éruption. L'examen de gorge ne montre ni pharyngite, ni muco-pus dans le
cavum. Les tympans sont clairs. L'état général et l'appétit sont bien conservés.

Quel(s) est(sont) le(s) examen(s) paraclinique(s) qui vous paraît(paraissent) indispensable(s) immédiatement ?
A - NFS
B - Hémoculture
C - Compte de germes urinaires
D - Radiographie pulmonaire
E - Aucun examen
Bonne(s) réponse(s) : E

Sans commentaire.

Quel(s) traitement(s) peut on prescrire ?


A - Température maximum à 20 degrés dans la chambre
B - Bain à 38 degrés pendant 10 mn
C - Aspirine : 125 à 150 mg toutes les 6 heures
D - Paracétamol : 75 mg toutes les 6 heures
E - Amoxicilline : 100 mg toutes les 6 heures
Bonne(s) réponse(s) : A B C D

Sans commentaire.

La mère vous rappelle le lendemain lundi soir, car si la fièvre a disparu le matin même, l'enfant a présenté au
même moment, une apparition de boutons. L'éruption intéresse thorax et cou, respecte le visage, est faite de
macules rosées, avec intervalles de peau saine. La température est à 37,2 degrés. Quel diagnostic évoquez-
vous ?
A - Allergie à l'aspirine
B - Rougeole
C - Roséole infantile
D - Scarlatine
E - Rubéole
Bonne(s) réponse(s) : C

Sans commentaire.

Si une convulsion était survenue lors de l'épisode fébrile à 39.7 degrés quel(s) traitement(s) auriez-vous fait à
domicile ?
A - Gardenal® : 6 cg per os
B - Valium® : 15 mg intrarectal
C - Valium® : 4 mg intrarectal
D - Valium® : 4 mg intramusculaire
E - Gardenal® : 3 cg IM
Bonne(s) réponse(s) : C

Sans commentaire.

391
Exclusivement sur DOC - DZ : www.doc-dz.com NADJI 85
RESIDANAT EN POCHE TOME II
Cas Clinique en QCM
Vous êtes appelé à domicile pour examiner une petite fille de 2 ans ayant eu un développement staturo-pondéral et
psychomoteur normal, sans antécédent pathologique particulier, et qui présente depuis 2 heures une fièvre à 39°C et une
gène respiratoire intense.

Vous suspectez une dyspnée laryngée. Parmi les propositions suivantes, laquelle est une caractéristique
erronée ?
A - Une bradypnée
B - Expiratoire
C - Accompagnée d'un tirage
D - Accompagnée d'un cornage
E - Souvent d'origine virale
Bonne(s) réponse(s) : B

Sans commentaire.

Parmi les caractères suivants, le(s)quel(s) plaide(nt) en faveur d'un épiglottite ?


A - Fièvre
B - Hypersialorrhée
C - Début brutal
D - Dysphagie
E - Vomissements
Bonne(s) réponse(s) : A B C D

Sans commentaire.

Quel est l'agent pathogène le plus souvent responsable des épiglottites ?


A - Pneumocoque
B - Hémophilus
C - Streptocoque
D - Branhamella catarrhalis
E - Virus respiratoire syncitial
Bonne(s) réponse(s) : B

Sans commentaire.

Si vous suspectez le diagnostic d'épiglottite, quelle attitude thérapeutique devez-vous adopter en premier lieu ?
A - Soumettre l'enfant à une humidification des voies aériennes
B - Prescrire une forte dose de corticoïdes
C - Prescrire un traitement antitussif
D - Prescrire une forte dose de corticoïdes et un traitement antibiotique
E - Hospitaliser l'enfant d'urgence
Bonne(s) réponse(s) : E

Sans commentaire.

Parmi les caractères suivants, quel(s) est(sont) celui(ceux) qui plaident en faveur d'une laryngite oedémateuse
sous-glottique ?
A - Début progressif
B - Fièvre modérée
C - Précession par un catarrhe rhinopharyngé
D - Hypersalivation
E - Vomissements
Bonne(s) réponse(s) : A B C

Sans commentaire.

392
Exclusivement sur DOC - DZ : www.doc-dz.com NADJI 85
RESIDANAT EN POCHE TOME II
Cas Clinique en QCM
Cet enfant de sept mois, sans antécédent particulier,.a fait une otite paracentésée 3 jours plus tôt. Il a été traité par
Josacine®, mais il vomit, reste fébrile. Le médecin appelé constate un bombement de la fontanelle antérieure. L'interne de
garde effectue une rachicentèse qui retire un liquide franchement trouble. En urgence, le laboratoire donne les résultats
suivants : cytologie innombrable, avec 90 % de polynucléaires, présence à l'examen de petits bacilles à Gram négatif.

Un L.C.R. de méningite purulente peut avoir un ou plusieurs des caractères suivants :


A - Hyperprotéinorachie
B - Hypochlorurachie
C - Hypoglycorachie
D - Abaissement de l'acide lactique
E - Augmentation de la LDH et de ses iso-enzymes
Bonne(s) réponse(s) : A C E

Sans commentaire.

Le bacille à Gram négatif découvert à l'examen direct peut être :


A - Neisseria meningitidis
B - Listeria monocytogènes
C - Streptococcus pneumoniae
D - Hémophilus influenzae
E - Staphyloccus aureus
Bonne(s) réponse(s) : D

Sans commentaire.

Parmi les germes cités précédemment, quels sont les trois germes les plus fréquemment responsables de
méningite purulente chez un tel nourrisson de sept mois ?
A - Neisseria meningitidis
B - Listeria monocytogènes
C - Streptococcus pneumoniae
D - Hémophilus influenzae
E - Staphyloccus aureus
Bonne(s) réponse(s) : A C D

Sans commentaire.

Parmi les antibiotiques suivants. cochez ceux dont le passage dans le liquide céphalo-rachidien est suffisant
pour permettre le traitement de première intention d'une méningite purulente:
A - Erythromycine ou Erythriocine® I.V.
B - Gentamicine ou Gentalline® I.M.
C - Cefalotine ou Keflin® I.V.
D - Cefotaxime ou Claforan® I.V.
E - Ampicilline ou Totapen® I.V.
Bonne(s) réponse(s) : D E

Sans commentaire.

La petite Stéphanie M... est âgée de 4 ans. Lors de l'examen scolaire systématique, on lui a découvert une hypertension
artérielle à 180 mmHg pour la maxima, 110 pour la minima au membre supérieur.
Il n'y a pas d'antécédent pathologique personnel ni familial.

Quelle(s) es (sont), parmi les signes cliniques suivant. celui(ceux) qui est(sont) en faveur d'une coarctation
aortique ?
A - Hyperpulsalité artérielle au niveau du cou
B - Abolition des pouls fémoraux
C - Oedème prétibial
D - Souffle systolique de l'espace inter-omovertébral gauche
E - Dédoublement constant du 2ème bruit au foyer aortique
Bonne(s) réponse(s) : A B D

Sans commentaire.

393
Exclusivement sur DOC - DZ : www.doc-dz.com NADJI 85
RESIDANAT EN POCHE TOME II
Cas Clinique en QCM

Quel qualificatif s'applique à l'hypertension artérielle de cette enfant ?


A - Labile
B - Limite
C - Confirmée
D - Menaçante
E - Aucun des qualificatifs proposés
Bonne(s) réponse(s) : D

Sans commentaire.

Quelles(s) complication(s) révélatrice(s) aurai(en)t pu survenir chez cette enfant ?


A - Etat de mal convulsif
B - Choc cardiogénique
C - Oedème aigu du poumon
D - Syndrome hémolytique et urémique
E - Amaurose
Bonne(s) réponse(s) : A B C D E

Sans commentaire.

Ayant vous-même retrouvé chez cette enfant des valeurs identiques de pression artérielle, vous devez adopter
l'une des attitudes suivantes. Laquelle ?
A - Nouveau contrôle de la pression artérielle un mois plus tard
B - Prescription d'un régime désodé
C - Prescription d'un régime désodé et d'un diurétique
D - Prescription d'un anti-hypertenseur de type bêta-bloqueur
E - Hospitalisation immédiate
Bonne(s) réponse(s) : E

Sans commentaire.

Pour affirmer le diagnostic de coarctation aortique et conduire à sa cure chirurgicale, quel(s) examen(s)
complémentaire(s) est(sont) indiqué(s) ?
A - Radiographie du thorax de face
B - Doppler carotidien
C - Scintigraphie cardiaque
D - Echocardiographie
E - Angiocardiographie
Bonne(s) réponse(s) : A D E

Sans commentaire.

Compte tenu du fait que cette enfant atteinte d'une coarctation aortique ne mesure que 88 cm, quel examen.
parmi les suivants, doit être pratiqué ?
A - Test de la sueur
B - Test de stimulation de l'hormone de croissance
C - Recherche d'une entéropathie exsudative
D - Caryotype
E - Dosage des hormones thyroïdiennes plasmatiques (T3, T4, TSH)
Bonne(s) réponse(s) : D

Sans commentaire.

394
Exclusivement sur DOC - DZ : www.doc-dz.com NADJI 85
RESIDANAT EN POCHE TOME II
Cas Clinique en QCM
Vous êtes médecin responsable du service médical d'un établissement scolaire comportant un externat et un internat. Vous
posez le diagnostic de méningite chez un enfant de 12 ans, en internat, et vous soupçonnez une méningite cérébrospinale à
méningocoque.

Les parents de l'enfant sont absents pour 48 heures et vous ne pouvez les joindre. Quelle(s) mesure(s) prenez-
vous immédiatement ?
A - Transport de l'enfant à l'hôpital
B - Transport à l'hôpital seulement après autorisation des parents
C - Déclaration obligatoire dès la suspicion de l'infection
D - Déclaration seulement après confirmation bactériologique
E - Déclaration anonyme
Bonne(s) réponse(s) : A D E

Sans commentaire.

L'infirmière de l'établissement vous demande des renseignements sur l'épidémiologie de la méningite à


méningocoques. Vous répondez que :
A - Les méningocoques résistent bien dans l'air ambiant
B - Ils colonisent le rhinopharynx
C - La transmission se fait par voie aérienne
D - La transmission peut être manuportée
E - En France, 80 % des méningocoques sont de type C
Bonne(s) réponse(s) : B C

Sans commentaire.

La direction de l'établissement vous demande quelle(s) mesure(s) de désinfection il faut prendre pour les
locaux. Vous préconisez de procéder, après le départ du malade, à une désinfection :
A - De l'internat, par le formol
B - De tout l'établissement, par le formol
C - Du linge de l'internat, par javellisation
D - Du matériel de cuisine par javellisation
E - Aucune de ces mesures n'est utile
Bonne(s) réponse(s) : E

Sans commentaire.

Quelle(s) mesure(s) prophylactique(s) devez-vous prendre vis-à-vis des sujets contacts proches de l'enfant, et
vivant dans l'établissement ?
A - Désinfection rhinopharyngée par pulvérisations
B - Recherche de méningocoques sur prélèvements pharyngés
C - Antibioprophylaxie par spiramycine (si méningocoque B)
D - Antibioprophylaxie et vaccination (si méningocoque C)
E - Vaccination seule (si méningocoque C)
Bonne(s) réponse(s) : C

L'antiobioprophylaxie est souhaitable pour le méningocoque C.


La vaccination dans le cas de figure du QCM est discutable, voire inutile.

Vis-à-vis de la collectivité, quelle(s) autre(s) mesure(s) obligatoire(s) devez-vous faire appliquer ?


A - Fermeture de l'école pendant 1 semaine
B - Informer les familles de la survenue d'un cas de méningite
C - Réadmission de l'enfant dès sa guérison
D - Réadmission de l'enfant 20 jours après sa guérison
E - Réadmission de l'enfant quand 2 prélèvements pharyngés sont négatifs
Bonne(s) réponse(s) : B C

Sans commentaire.

395
Exclusivement sur DOC - DZ : www.doc-dz.com NADJI 85
RESIDANAT EN POCHE TOME II
Cas Clinique en QCM
Un nourrisson de deux mois, vivant dans des conditions défavorables, non vacciné, tousse depuis 15 jours et de plus en plus
la nuit. La toux est émétisante, et retentit fâcheusement sur son état général, car l'enfant vomit presque tous les biberons. Il
n'y a pas de fièvre. L'auscultation des poumons est normale. La radiographie du thorax révèle une accentuation discrète péri-
hilaire bilatérale, sans adénopathie médiastinale. Le timbre tuberculinique est négatif. Vous avez été appelé d'urgence car à
l'occasion d'une secousse de toux, l'enfant s'est cyanosé et n'a pas repris immédiatement sa respiration. Il est devenu
hypotonique, a perdu connaissance en cessant de respirer. La reprise inspiratoire est survenue après une trentaine de
secondes.

Quelle affection diagnostiquez-vous ?


A - Rhinopharyngite saisonnière compliquée
B - Phase pré-éruptive de la rougeole
C - Méningite avec atteinte encéphalitique
D - Pneumonie atypique
E - Coqueluche
Bonne(s) réponse(s) : E

Coqueluche.

Comment interprétez-vous l'épisode ayant motivé l'appel en urgence ?


A - Coma post-épileptique
B - Quinte asphyxiante
C - Fausse route après vomissement
D - Ictus laryngé
E - Apnée syncopale
Bonne(s) réponse(s) : B

Quinte asphyxiante.

Parmi les examens complémentaires suivants, le ou lesquels sont une aide au diagnostic ?
A - Numération sanguine formule leucocytaire
B - Hémoculture
C - Prélèvement nasopharyngé
D - Sérodiagnostic
E - Intradermoréaction
Bonne(s) réponse(s) : A C D

A. La numérotation montre typiquement une hyperlymphocytose.


C. Le prélèvement nasopharyngé retrouve inconstamment le bacille de Bordet et Gengou.
D. Le sérodiagnostic est classiquement tardivement positif. Actuellement, un nouveau sérodiagnostic avec mise en évidence
d'anticorps anti-F.H.A. donne un diagnostic rapide précoce et fiable.

La numération formule sanguine montre typiquement :


A - Une anémie hypochrome
B - Une hyperleucocytose avec hyperleucopolynucléose
C - Un syndrome mononucléosique
D - Une hyperlymphocytose absolue et relative
E - Une hyperéosinophilie
Bonne(s) réponse(s) : D

Une hyperlymphocytose absolue et relative.

La soeur jumelle du nourrisson élevée par la grand-mère a regagné le domicile maternel depuis dix jours. Elle
paraît en excellente santé, n'a aucun signe de maladie respiratoire. Quelle attitude adoptez-vous ?
A - Il n'y a rien à faire
B - Vous isolez et séparez le frère et la soeur
C - Vous vaccinez la soeur
D - Vous prescrivez un macrolide
E - Vous prescrivez des gammaglobulines hyperimmunes spécifiques à la soeur
Bonne(s) réponse(s) : B D

B. Le frère et la soeur se séparent obligatoirement du fait de l'hospitalisation qui sera longue.


D. Donner un macrolide comme la Rovamycine pendant 5 à 10 jours.
L'action prophylactique des immunoglobulines anticoquelucheuses n'a pas été démontrée.
La vaccination ne protégera pas la soeur.

396
Exclusivement sur DOC - DZ : www.doc-dz.com NADJI 85
RESIDANAT EN POCHE TOME II
Cas Clinique en QCM
Un enfant de 3 ans 1/2 est amené à consulter pour des troubles de prononciation d'apparition brutale.
Dans les antécédents, on relève des épisodes fébriles lors des éruptions dentaires, une rougeole compliquée d'otite à 3 ans,
des rhinopharyngites fréquentes. Le développement psychomoteur est par ailleurs normal. Selon l'entourage, l'enfant paraît
bien entendre mais fait répéter ses parents, est actuellement plus instable, moins obéissant.
Il s'agit d'un premier enfant, en bon état général, présentant une rhinite séreuse banale, une respiration buccale, de grosses
amygdales saines, des adénopathies cervicales bilatérales jugulo-carotidiennes, d'allure banale. Les tympans sont mats,
laiteux immobiles au spéculum de Siegle.
A l'examen audiométrique la perte est de 45 dB des deux côtés en conduction aérienne. La conduction osseuse est normale.

De quel type de surdité s'agit-il ?


A - Surdité de transmission
B - Surdité de perception par atteinte cochléaire
C - Surdité de perception par atteinte du nerf auditif
D - Surdité mixte
E - Cophose
Bonne(s) réponse(s) : A

Sans commentaire.

Le tympanogramme est aplati : quel diagnostic évoquez-vous ?


A - Surdité post-infectieuse virale
B - Otite séromuqueuse
C - Surdité héréditaire tardive
D - Malformation auriculaire
E - Mastoïdite
Bonne(s) réponse(s) : B

Devant toute modification du comportement chez l'enfant petit, penser à l'otite séreuse.

Parmi ces propositions thérapeutiques, laquelle(lesquelles) choisirez vous ?


A - L'appareillage de l'enfant
B - La mise en place d'aérateurs transtympaniques
C - Un traitement antibiotique
D - L'adénoïdectomie
E - Une consultation pédopsychiatrique
Bonne(s) réponse(s) : B C

Il faut rompre le cercle vicieux créé par l'obstruction tubaire.

Cette affection guérit habituellement ; cependant elle peut entraîner des séquelles ou complications. Quelles
sont les deux principales ?
A - Acouphènes
B - Cholestéatome
C - Cophose
D - Vertiges
E - Lésions tympano-ossiculaires
Bonne(s) réponse(s) : B E

Sans commentaire.

397
Exclusivement sur DOC - DZ : www.doc-dz.com NADJI 85
RESIDANAT EN POCHE TOME II
Cas Clinique en QCM
Un nourrisson de 6 mois est amené à la consultation pour une éruption du visage et des membres extrêmement prurigineuse,
cause d'insomnies de l'enfant et des parents, ayant débuté six semaines auparavant. L'examen découvre un enfant en bon
état général, mais défiguré par des lésions érythémateuses, suintantes et croûteuses du visage, épargnant la région médio-
faciale, avec intertrigo rétroauriculaire. Le dos des mains, la face dorsale des avant-bras et les jambes sont couverts de
lésions érythématosquameuses avec des érosions dues au grattage et des croûtes jaunâtres. Les fesses sont rouges avec
quelques érosions et les plis inguinaux et périnéaux légèrement suintants. Un traitement institué depuis un mois et comportant
l'application de crème Parfenac® et l'administration de Célestène(® gouttes (bêtaméthasone) 4mg/jour puis à doses
dégressives a été suivi d'amélioration passagère mais n'a pas empêché la poussée actuelle. L'anamnèse nous apprend que
son frère âgé de 8 ans a présenté plusieurs crises d'asthme, le père est affligé d'une rhinite pollinique, il n'y a aucune autre
dermatose ou maladie générale dans la famille.

L'affection dont souffre ce nourrisson est caractérisée par :


A - Début généralement à la période néonatale
B - Atteinte fréquente du pouce sucé
C - Le respect de la région centro-faciale
D - Le prurit insomniant
E - Le caractère vésiculeux et suintant de la poussée éruptive
Bonne(s) réponse(s) : C D E

Sans commentaire.

Quelle(s) affection(s) parmi les suivantes pouvez-vous éliminer chez ce nourrisson, à la lecture de l'énoncé
clinique ?
A - Une gale acarienne
B - Une dermite séborrhéique
C - Une dermatite atopique
D - Des lésions impétiginisées
E - Une varicelle
Bonne(s) réponse(s) : A B E

Sans commentaire.

Pour confirmer votre diagnostic il est indispensable de disposer du résultat de :


A - Recherche d'une éosinophilie sanguine
B - Recherche de parasites dans les squames
C - Tests cutanés allergologiques
D - Dosage des IgE
E - Aucun de ces examens
Bonne(s) réponse(s) : E

Sans commentaire.

Parmi les mesures thérapeutiques permettant de prévenir les récidives, vous retenez :
A - Utilisation de pommades hydratantes
B - Désensibilisation aux pneumallergènes
C - Régime d'exclusion lactée
D - Injection d'immunoglobulines
E - Eviction des facteurs d'irritation cutanée
Bonne(s) réponse(s) : A E

Sans commentaire.

Un enfant de 2 ans 1/2 présente une dyspnée inspiratoire avec bradypnée évoluant depuis plusieurs heures. Il est enrhumé
depuis plusieurs jours. Les parents vous appellent le soir parce que la dyspnée s'aggrave. A l'examen, vous trouvez un enfant
couché dans son lit, conscient, fébrile à 38°C, présentant un tirage sus-sternal et intercostal avec cornage. La dyspnée est
inspiratoire, la fréquence respiratoire est de 15/mn. De temps en temps, l'enfant présente des accès de toux rauque. La mère
vous montre le carnet de santé de l'enfant, sur lequel vous notez qu'il a été vacciné correctement.

Le type de dyspnée observée traduit un obstacle :


A - Nasal
B - Laryngé
C - Trachéal
D - Bucco-pharyngé
E - Bronchiolaire
Bonne(s) réponse(s) : B

Sans commentaire.

398
Exclusivement sur DOC - DZ : www.doc-dz.com NADJI 85
RESIDANAT EN POCHE TOME II
Cas Clinique en QCM

Quel diagnostic pouvez-vous poser avec le plus de probabilité ?


A - Rhinopharyngite
B - Laryngite aiguë sous glottique
C - Epiglottite
D - Corps étranger dans les voies aériennes
E - Bronchiolite
Bonne(s) réponse(s) : B

Sans commentaire.

Quel est le geste thérapeutique que vous entreprenez immédiatement ?


A - Mise en atmosphère humide
B - Injection intramusculaire de 4 mg de dexaméthasone
C - Injection intramusculaire d'ampicilline (25 mg/kg)
D - Injection intrarectale de diazépam (0,5 mg/kg)
E - Administration d'aspirine (25 mg/kg) per os
Bonne(s) réponse(s) : B

Sans commentaire.

Par la suite, quelle conduite adoptez-vous ?


A - Hospitalisation immédiate systématique
B - Rester au chevet du malade pendant une 1/2 heure afin d'apprécier l'effet du traitement
C - Hospitalisation d'urgence si non amélioration dans la 1/2 heure
D - Si la dyspnée cède totalement : corticothérapie à doses dégressives pendant 5 jours
E - Prescription d'un antitussif
Bonne(s) réponse(s) : B C D

Sans commentaire.

Dans le cas où vous auriez suspecté une épiglottite, votre attitude aurait comporté :
A - Hospitalisation immédiate en transportant l'enfant en position assise
B - Faire prévenir le service d'accueil
C - Injection intramusculaire de Lasilix®
D - Un traitement uniquement à domicile par ampicilline
E - Repos au lit
Bonne(s) réponse(s) : A B

Sans commentaire.

Un garçon de 11 ans, sans antécédent pathologique, vous est adressé pour syndrome hémorragique et hyperthermie. La
température est entre 39 et 40° C depuis 48 heures et l'examen retrouve un purpura pétéchial et ecchymotique au niveau des
membres, des gingivorragies et des bulles hémorragiques endobuccales. Il existe une splénomégalie de 2 travers de doigt,
une pâleur cutanéomuqueuse, un souffle systolique de pointe à l'auscultation cardiaque. L'étude du fond d'oeil montre une
petite hémorragie près de la papille de l'oeil droit.
L'hémogramme révèle :
- Hémoglobine : 6 g/dl
- Leucocytes : 2,5 . 10 exposant 9/l dont : - polynucléaires neutrophiles 20 %
- lymphocytes 75 %
- monocytes 5 %
- Plaquettes : 10 . 10 exposant 9/l
Le taux de prothrombine, le temps de céphaline activée, le dosage du fibrinogène sont normaux. Les explorations biologiques
usuelles hépatiques, rénales et métaboliques sont normales. Le myélogramme révèle, dans une moelle riche, 85 % de
cellules peu différenciées, à rapport nucléocytoplasmique élevé, sans granulations cytoplasmiques.

Le diagnostic retenu est celui de leucémie aiguë lymphoblastique (LAL).


Un autre diagnostic pourrait-il être évoqué, au vu du tableau clinique, de l'hémogramme et du myélogramme ?
A - Aplasie médullaire
B - Mononucléose infectieuse
C - Etat d'hypersplénisme
D - Leucémie lymphoïde chronique
E - Aucune de ces affections
Bonne(s) réponse(s) : E

Dans certains cas très rares, la mononucléose infectieuse peut simuler une leucose myélomonocytaire.

399
Exclusivement sur DOC - DZ : www.doc-dz.com NADJI 85
RESIDANAT EN POCHE TOME II
Cas Clinique en QCM

Certain(s) geste(s) s'impose(nt) chez ce malade :


A - Réalisation de 6 à 8 hémocultures sur 24 heures puis mise en route d'une antibiothérapie double par voie
veineuse
B - Même antibiothérapie, mais mise en route en moins de 3 heures pendant lesquelles on réalise 2 ou 3
hémocultures
C - Transfusion de concentrés plaquettaires
D - Transfusion de plasma frais
E - Transfusion de concentrés érythrocytaires
Bonne(s) réponse(s) : B C E

Sans commentaire.

Au cours d'une LAL commune de l'enfant, en première poussée, on retrouve aussi fréquemment :
A - Des localisations cutanées spécifiques
B - Des douleurs osseuses, avec aspect radiologique en bandes claires métaphysaires
C - Une hyperplasie gingivale
D - Une positivité de la réaction cytochimique des myéloperoxydases
E - La présence de corps d'Auer dans quelques blastes
Bonne(s) réponse(s) : A B C

L.A.L. : péroxydases négatifs.


L.A.M. : pas corps positif d'Auer.

Chez cet enfant, le relativement "bon" pronostic à terme s'appuie sur:


A - L'absence d'hyperleucocytose
B - La présence d'une splénomégalie
C - L'absence de tuméfaction médiastinale
D - L'absence de localisation neuroméningée
E - L'absence d'hépatomégalie et d'adénopathies superficielles
Bonne(s) réponse(s) : A C D E

Sans commentaire.

Le protocole de chimiothérapie d'induction associera chez cet enfant : prednisone, vincristine et daunorubicine.
Une rémission complète est obtenue. Les autres mesures essentielles après l'obtention de la rémission
comporteront :
A - Une irradiation en mantelet
B - Arrêt de toute chimiothérapie
C - Une prévention systématique des localisations neuroméningées par méthotrexate intrarachidien et
irradiation encéphalique
D - Une chimiothérapie d'entretien
E - Immunothérapie
Bonne(s) réponse(s) : C D

Sans commentaire.

Certaines complications iatrogènes sont à redouter à plus ou moins long terme, compte tenu des médicaments
utilisés en induction chez ce malade :
A - Polynévrite
B - Fibrose pulmonaire
C - Insuffisance rénale
D - Myocardiopathie
E - Surdité
Bonne(s) réponse(s) : A D

Les polynévrites sont des complications classiques de la vincristine.


La myocardiopathie liée à la daunorubicine est classique.

400
Exclusivement sur DOC - DZ : www.doc-dz.com NADJI 85
RESIDANAT EN POCHE TOME II
Cas Clinique en QCM

Quels sont les trois modes habituels de rechute de LAL de l'enfant ?


A - Médullaire
B - Neuroméningée
C - Testiculaire
D - Gastrique
E - Gastrique
Bonne(s) réponse(s) : A B C

Sans commentaire.

Vous êtes appelé le 13 mars dans la matinée par la mère d'un enfant de 4 ans. Il a les yeux rouges et gonflés, une rhinite
aqueuse et une toux sèche et douloureuse. La température est à 39 degrés. L'enfant est abattu.
Vous notez une pharyngite et des microadénopathies cervicales.
A l'interrogatoire vous retrouvez un contact avec un camarade rougeoleux en classe le 1er mars précédent. Vous suspectez
une rougeole.

Où recherchez-vous le signe de Kِplick qui affirme la rougeole ?


A - Sur les piliers des amygdales
B - Sur le voile du palais
C - Sur la paroi antérieure du pharynx
D - A la face antérieure de la langue
E - A la face interne des joues
Bonne(s) réponse(s) : E

Sans commentaire.

Où recherchez-vous l'éruption débutante ?


A - A la paume des mains
B - Aux plis de flexion
C - A la racine du cou
D - Derrière les oreilles
E - Aux joues
Bonne(s) réponse(s) : D

Sans commentaire

Au 5ème jour après la sortie de l'éruption l'enfant demeure fébrile.


Vous soupçonner une complication de surinfection. Laquelle ou lesquelles recherchez-vous en priorité ?
A - Angine streptococcique
B - Méningite lymphocytaire
C - Otite aiguë
D - Bronchopneumonie
E - Adénophlegmon cervical
Bonne(s) réponse(s) : C D

Sans commentaire.

Quel(s) germe(s) est(sont) habituellement en cause au cours de ces complications de surinfection ?


A - Escherichia coli
B - Staphylococcus aureus
C - Haemophilus influenzae
D - Streptococcus pneumoniae
E - Klebsiella pneumoniae
Bonne(s) réponse(s) : C D

Sans commentaire.

401
Exclusivement sur DOC - DZ : www.doc-dz.com NADJI 85
RESIDANAT EN POCHE TOME II
Cas Clinique en QCM

Vous décidez de prescrire une antibiothérapie. Parmi les antibiotiques suivants, lequel vous parait le mieux
adapté à cette situation ?
A - Colistine (Colimycine®)
B - Amoxicilline (Clamoxyl®)
C - Rifampicine (Rimactan®)
D - Doxycycline (Vibramycine®)
E - Gentamicine (Gentalline®)
Bonne(s) réponse(s) : B

Ceci dit 20% environ des hémophilus sont résistants à l'Amoxilline.

Si vous aviez vu l'enfant le 5 mars, quelle mesure prophylactique aurait alors permis d'éviter la survenue de
cette rougeole ?
A - Isolement
B - Vaccination
C - Gammaglobulines standard
D - Vaccination + gammaglobulines standard
E - Prescription d'acyclovir (Zovirax®)
Bonne(s) réponse(s) : C

Les gammaglobulines standard sont à faire dans les 5 jours suivant le contage. Après elles sont inutiles.

Un garçon de 13 mois est hospitalisé pour oedèmes des membres inférieurs et du visage apparus depuis trois jours. On
retrouve à l'interrogatoire la notion de selles molles et abondantes depuis plusieurs semaines, ainsi que d'une anorexie
modérée ; l'alimentation cependant est restée normale, complétée par l'administration quotidienne de 1200 unités de vitamine
D2 ; depuis un mois la prise de poids est nettement moins bonne. Il faut noter en outre une otite purulente rebelle qui a fini par
guérir une dizaine de jours avant l'hospitalisation.
A son admission cet enfant pèse 8260 g, mesure 76 cm, il est fébrile (aux environs de 38 degrés).
On trouve à l'examen en dehors des oedèmes, une amyotrophie, une pâleur des téguments, le foie déborde de 1 à 2 cm,
l'auscultation du coeur est normale, la tension artérielle maximale est à 9 avec un brassard de moyenne dimension.
Les résultats des premiers examens complémentaires comportent notamment un taux d'hémoglobine de 11,5 g/dl, 3.800.000
globules rouges, une protidémie à 32 g/l, la calcémie est à 1,95 mmol/l, la natrémie à 135 mmol/l, la créatininémie à 40
micromol/l.
La recherche de protéinurie est négative.

Parmi les symptômes observés chez ce malade et notés ci-dessous, quel(s) est(sont) celui(ceux) qui vous
parai(ssen)t être en faveur d'une malabsorption :
A - L'anorexie
B - Le caractère abondant des selles
C - La cassure de la courbe de poids
D - L'amyotrophie
E - La température notée à l'entrée
Bonne(s) réponse(s) : A B C D

Sans commentaire.

Les oedèmes de ce malade sont dûs à :


A - L'anémie
B - La fièvre
C - Une insuffisance cardiaque
D - Une hypoprotidémie
E - Une insuffisance rénale
Bonne(s) réponse(s) : D

Sans commentaire.

L'hypocalcémie est probablement dûe à :


A - Une carence alimentaire
B - Une hypoprotidémie
C - Une hypoparathyroïdie
D - Une dilution plasmatique
E - Une hypercalciurie
Bonne(s) réponse(s) : C

Sans commentaire.

402
Exclusivement sur DOC - DZ : www.doc-dz.com NADJI 85
RESIDANAT EN POCHE TOME II
Cas Clinique en QCM

Parmi les anomalies suivantes, laquelle ou lesquelles plaide(nt) en faveur d'une malabsorption ?
A - Une hyposidérémie
B - Allongement du temps de Quick
C - Elévation du taux plasmatique de l'acide folique
D - Une ostéoporose
E - Une stéatorrhée
Bonne(s) réponse(s) : A B D E

Sans commentaire.

Si vous pratiquez une biopsie jéjunale et que le laboratoire vous répond qu'il existe une atrophie villositaire sub-
totale, vous devez évoquer :
A - Des lymphangiectasies
B - Une maladie coeliaque
C - Une intolérance primitive au lactose
D - Une mucoviscidose
E - Une intolérance aux protéines du lait de vache
Bonne(s) réponse(s) : B

Sans commentaire.

Thomas G....est né au terme d'une grossesse normale de 39 semaines (poids de naissance 3300g). L'accouchement était
eutocique et l'examen néonatal était normal, hormis un petit souffle cardiaque systolique sans retentissement clinique. Il reçoit
un allaitement maternel complet.
A partir de la 4ème semaine de vie, des difficultés alimentaires apparaissent, avec polypnée et sueurs au cours des biberons,
et la courbe pondérale s'infléchit.
L'enfant est hospitalisé à l'âge de 6 semaines pour fièvre à 38°C, toux et polypnée permanente.
A l'examen : poids 3800g, enfant pâle, non cyanosé, souffle holosystolique d'intensité 4/6ème aux 3ème-4ème espaces
intercostaux gauches, irradiant en rayons de roue : râles sous-crépitants dans les deux champs pulmonaires, hépatomégalie
(débord de 4 cm ; absence de splénomégalie: temps de recolo ration cutané 2 secondes, pouls huméraux et fémoraux
présents, pression artérielle au flush 70 mmHg aux membres supérieurs et inférieurs. La radiographie thoracique montre une
cardiomégalie (RCT = 0,65) et un aspect d'hypervascularisation artérielle pulmonaire.

Parmi les 5 propositions diagnostiques suivantes, laquelle retenez-vous ?


A- Péricardite
B - Endocardite
C - Laryngite aiguë
D - Insuffisance cardiaque
E - Myocardite aiguë
Bonne(s) réponse(s) : D

Sans commentaire.

Quelle cardiopathie l'examen clinique cardiovasculaire de cet enfant vous permet-il de retenir ?
A - Communication interauriculaire
B - Communication interventriculaire
C - Persistance du canal artériel
D - Coarctation de l'aorte
E - Tétralogie de Fallot
Bonne(s) réponse(s) : B

Sans commentaire.

Quelle est la conséquence physiopathologique de cette cardiopathie ?


A - Shunt gauche-droite
B - Shunt droite-gauche
C - Obstacle à l'éjection du ventricule droit
D - Obstacle à l'éjection du ventricule gauche
E - Mixage des deux circulations
Bonne(s) réponse(s) : A

Sans commentaire.

403
Exclusivement sur DOC - DZ : www.doc-dz.com NADJI 85
RESIDANAT EN POCHE TOME II
Cas Clinique en QCM

Quel traitement prescrivez-vous ?


A - Pénicilline G - isoprotérénol
B - Furosémide - digoxine - amoxicilline
C - Théophylline - gentamicine - digoxine
D - Terbutaline - érythromycine - digoxine
E - Prednisone - furosémide - tétracycline
Bonne(s) réponse(s) : B

Sans commentaire.

Quelle est la posologie orale quotidienne de la Digoxine® (en traitementd' entretien) chez un enfant de 3 mois ?
A - 3 à 5 microgrammes/kg
B - 10 à 15 microgrammes/kg
C - 25 à 30 microgrammes/kg
D - 35 à 40 microgrammes/kg
E - 45 à 50 microgrammes/kg
Bonne(s) réponse(s) : B

Sans commentaire.

Une petite fille naît à terme, après une grossesse sans problème, au poids de 3 350 g. Le premier mois de vie se déroule
normalement. Elle est alimentée au biberon avec un lait maternisé.
A l'âge de 6 semaines, l'enfant débute un épisode rhinopharyngé marqué par un coryza et des éternuements. La température
est normale. L'appétit fléchit un peu.
Cinq jours plus tard apparaît une toux qui devient rapidement préoccupante car répétitive, émétisante, et réalisant au bout de
48 heures des quintes au terme desquelles l'enfant est cyanosée. De plus est apparue une polypnée à 65/mn.
Devant l'aggravation rapide de ces symptômes respiratoires, le médecin de famille fait hospitaliser l'enfant au 8ème jour de sa
maladie. L'état général est bon. La température est à 37.8°.

A l'admission de l'enfant en milieu hospitalier, quel(s) diagnostic(s) doi(ven)t être discuté(s) ?


A - Infection respiratoire de nature virale
B - Coqueluche
C - Pneumonie à pneumocoque
D - Staphylococcie pleuropulmonaire
E - Infection respiratoire à Chlamydia trachomatis
Bonne(s) réponse(s) : A B

Sans commentaire.

Quel(s) élément(s) clinique(s) plaiderai(en)t contre une coqueluche du nourrisson ?


A - Une conjonctivite
B - Une diarrhée
C - Une splénomégalie
D - Un ballonnement abdominal
E - Des râles sibilants diffus à l'auscultation du thorax
Bonne(s) réponse(s) : A B C D

Sans commentaire.

Quel(s) élément(s) paraclinique(s) plaiderai(en)t en faveur d'une coqueluche du nourrisson ?


A - Une éosinophilie
B - Une thrombopénie
C - Une leucocyto-lymphocytose
D - Une leucopénie
E - Un aspect radiologique de pneumopathie interstitielle
Bonne(s) réponse(s) : C

L'aspect radiologique de pneumopathie interstitielle peut se voir dans la coqueluche mais aussi dans d'autres affections
pulmonaires (virus, chlamydia).

404
Exclusivement sur DOC - DZ : www.doc-dz.com NADJI 85
RESIDANAT EN POCHE TOME II
Cas Clinique en QCM

Quel examen complémentaire pourrait affirmer rapidement l'origine virale de cette symptomatologie respiratoire
?
A - Une sérologie plasmatique
B - Une contre-immunoélectrophorèse sur les urines
C - Une culture du virus dans les selles
D - Un dosage des IgM spécifiques
E - Un examen en immunofluorescence sur les sécrétions nasopharyngées
Bonne(s) réponse(s) : E

Sans commentaire.

Quelle(s) complication(s) peu(ven)t entraîner une forme sévère de coqueluche chez le petit nourrisson ?
A - Des quintes asphyxiantes
B - Des apnées en dehors de toute secousse de toux
C - Des convulsions
D - Un coma
E - Un pyopneumothorax
Bonne(s) réponse(s) : A B C D

Sans commentaire.

Dans l'immédiat, après l'admission, la surveillance et le traitement comportent :


A - Surveillance continue par moniteur cardiorespiratoire
B - Erythromycine 50 mg/kg/j per os
C - Alimentation duodénale continue ou perfusion si les vomissements se répètent
D - Prednisone 2 mg/kg/j per os
E - Kinésithérapie respiratoire douce
Bonne(s) réponse(s) : A B C

On préfère la spiramycine à l'érythromycine en pratique.


La kinésithérapie respiratoire risque de déclencher des quintes de toux. Que veut dire douce ?
La corticothérapie peut être efficace sur les quintes asphyxiantes ; on utilise plus la bétaméthasone. La corticothérapie est
discutée, sa posologie aussi.

Un petit garçon naît par voie basse, en présentation céphalique, au terme de 34 semaines. Le liquide amniotique est clair. Le
score d'Apgar est côté à 5 à la première minute et à 9 à la cinquième minute de vie, après 2 minutes de ventilation au
masque. Le poids de naissance est de 2000 g. Au bout d'une heure de vie est constatée l'existence d'une détresse
respiratoire qui motive le transfert de l'enfant dans un service de néonatalogie.

Une détresse respiratoire néonatale débutante comporte typiquement :


A - Une accélération de la fréquence respiratoire audessus de 60/mn
B - Des signes objectifs de lutte respiratoire dits signes de tirage
C - Des apnées
D - Une cyanose à l'air ambiant (FiO2 0,21)
E - Aucune des propositions précédentes
Bonne(s) réponse(s) : A B D

Sans commentaire.

Le score de Silverman (ou indice de rétraction) cote de 0 à 2 :


A - Le geignement expiratoire
B - Le battement des ailes du nez
C - Le tirage sus-sternal
D - Le balancement thoraco-abdominal
E - Le creusement xiphoïdien
Bonne(s) réponse(s) : A B D E

Le 5ème paramètre est le tirage intercostal.

405
Exclusivement sur DOC - DZ : www.doc-dz.com NADJI 85
RESIDANAT EN POCHE TOME II
Cas Clinique en QCM

Dans le cas de ce petit garçon, tous les diagnostics suivants peuvent a priori être évoqués sauf un. Lequel?
A - Maladie des membranes hyalines
B - Détresse respiratoire transitoire (retard de résorption)
C - Inhalation méconiale
D - Infection pulmonaire
E - Pneumothorax
Bonne(s) réponse(s) : C

Sans commentaire.

Votre prescription initiale d'examen(s) complémentaire(s) comporte :


A - Une radiographie du thorax de face
B - Une radiographie pulmonaire de profil
C - Une mesure de pH sanguin
D - Un ECG
E - Une mesure de la TcPO2 ( PO2 cutanée)
Bonne(s) réponse(s) : A C E

Sans commentaire.

Le traitement symptomatique initial comporte :


A - Le réchauffement en incubateur
B - La désobstruction régulière (par aspiration) des voies aériennes et digestives
C - La perfusion de sérum glucosé et de calcium
D - L'oxygénothérapie sous enceinte céphalique
E - L'administration de stimulants respiratoires (théophylline ou caféine)
Bonne(s) réponse(s) : A B C D

Sans commentaire.

A posteriori, le diagnostic de maladie des membranes hyalines pourra être établi sur :
A - Rapport Lécithine/sphingomyéline > 2 dans les sécrétions pharyngées prélevées à l'admission
B - Durée d'évolution < 3 jours de la détresse respiratoire
C - Positivité des prélèvements bactériologiques initiaux
D - Aspect microgranité des 2 champs pulmonaires à la radiographie
E - Absence de toute acidose plasmatique au cours de l'évolution
Bonne(s) réponse(s) : D

Sans commentaire.

R . Xavier, né à terme, avec un score d'Apgar à 9, dont le développement a été normal jusque là, est hospitalisé en urgence à
2 ans, en raison d'une perte de connaissance brutale, accompagnée de secousses musculaires du bras, de la jambe et de
l'hémiface gauche. Sa température est à 40°C; son pouls à 160/min, sa T.A. à 9/6.
La crise cède après 15 minutes. Il persiste un déficit moteur gauche qui ira en s'atténuant.
La natrémie est à 130 mEq/l la kaliémie à 4,5 mEq/l, la calcémie à 95 mg/l, la glycémie à 1,80 g/l.
Quatre jours après la crise, l'électroencéphalogramme est correctement organisé, composé de fréquences à 5-6 c/seconde,
bien voltées, symétriques, de bonne réactivité.

Indiquez la cause de la crise observée ?


A - Epanchement sous dural
B - Hyperthermie
C - Hyponatrémie
D - Hyperglycémie
E - Aucune des causes précédentes
Bonne(s) réponse(s) : B

Sans commentaire.

406
Exclusivement sur DOC - DZ : www.doc-dz.com NADJI 85
RESIDANAT EN POCHE TOME II
Cas Clinique en QCM

Quel traitement devait être fait pour interrompre la crise ?


A- Injection IM de gardénal
B - Injection IV d'aspirine.
C - Injection de diazépam intra rectal.
D - Injection de diazépam intramusculaire
E - Injection IM de Dihydan®
Bonne(s) réponse(s) : C

Sans commentaire.

Quel(s) est(sont) le(s) risque(s) encouru(s) à l'occasion d'une telle crise ?


A - Hémiplégie définitive
B - Etat de mal épileptique
C - Epilepsie séquellaire
D - Hématome sous-dural
E - Surdité
Bonne(s) réponse(s) : A B C

Sans commentaire.

Quel(s) traitement(s) faut-il poursuivre après la crise ?


A - Valproate de sodium (Dépakine®)
B - Ethosuximide (Zarontin®)
C - Synacthène
D - Phénobarbital
E - Aucun traitement
Bonne(s) réponse(s) : A D

Sans commentaire.

Un enfant de 3 ans, 4ème de la fratrie, présente depuis 2 jours une fièvre élevée avec toux, rhinite et conjonctivite. Appelé en
consultation vous prescrivez un traitement par aspirine et désinfection nasale.
Deux jours plus tard, les parents demandent une nouvelle visite. L'examen retrouve une éruption érythémateuse derrière les
oreilles. Le soir les lésions érythémateuses se sont étendues sur le visage et le tronc. L'auscultation pulmonaire retrouve des
râles bronchiques dans les deux champs pulmonaires.

Quel est le diagnostic que vous retenez en première intention ?


A - Rubéole
B - Scarlatine
C - Exanthème subit
D - Rougeole
E - Varicelle
Bonne(s) réponse(s) : D

Sans commentaire.

Retenez le ou les examen(s) paraclinique(s) que vous réalisez en pratique courante pour confirmer votre
diagnostic :
A - Sérodiagnostic
B - Recherche de l'agent pathogène dans la gorge
C - Radiographie de thorax
D - Recherche de l'agent pathogène dans l'expectoration
E - Aucune de ces propositions
Bonne(s) réponse(s) : E

Sans commentaire.

407
Exclusivement sur DOC - DZ : www.doc-dz.com NADJI 85
RESIDANAT EN POCHE TOME II
Cas Clinique en QCM

Retenez le ou les élément(s) exact(s) de votre attitude thérapeutique immédiate :


A - Hospitalisation
B - Antibiothérapie systématique par gentamycine
C - Traitement antipyrétique par aspirine 100 mg 3 fois par jour
D - Injection immédiate d'immunoglobulines spécifiques
E - Eviction scolaire des frères et soeurs pendant 10 jours
Bonne(s) réponse(s) : E

Sans commentaire.

Retenez la ou les complication(s) évolutive(s) que vous pouvez craindre dans cette infection :
A - Otite moyenne
B - Encéphalite
C - Convulsions
D - Surinfection cutanée
E - Bronchopneumonie
Bonne(s) réponse(s) : A B C E

Sans commentaire.

Parmi les propositions suivantes retenez celle(s) qui justifie(nt) l'hospitalisation immédiate de ce patient :
A - Râles bronchiques à l'auscultation
B - Enfant de 3 ans et moins
C - Laryngite sous glottique
D - Existence d'une mucoviscidose
E - Misère sociale
Bonne(s) réponse(s) : D E

Sans commentaire.

Au cours d'une épidémie de grippe à Myxovirus influenzae à H3N2, un enfant de 2 ans 1/2 fait une fièvre à 40,5°C avec crise
convulsive et épistaxis. 3 heures plus tard apparaissent des signes de détresse respiratoire avec polypnée à 60 par minute,
battement des ailes du nez, cyanose, toux, râles fins dans les deux poumons. Vous suspectez une grippe maligne.

En quel lieu préférentiel pouvez-vous trouver le virus de la grippe ?


A - Les sécrétions nasophanryngées
B - Les selles
C - Les urines
D - Le sang
E - Le liquide céphalorachidien
Bonne(s) réponse(s) : A

Sans commentaire.

A partir du prélèvement retenu vous pouvez espérer un résultat positif en moins de 24 heures grâce à l'une ou
plusieurs des techniques suivantes :
A - Recherche d'antigène viral par immunofluorescence
B - Inoculation d'oeufs de poule embryonnés
C- Inoculation de cultures primaires de cellules de rein de singe
D - Inoculation de souriceaux nouveau-nés
E - Recherche d'IgM spécifiques
Bonne(s) réponse(s) : A

Sans commentaire.

Parmi les signes cliniques suivants, lequel ou lesquels peut-on observer au cours d'une grippe commune du
jeune enfant ?
A - Une fièvre à 40 degrés durant trois jours
B - Une crise convulsive hyperpyrétique
C - Un syndrome méningé
D - Une polypnée avec cyanose
E - Des courbatures diffuses
Bonne(s) réponse(s) : A B C E

Sans commentaire.
408
Exclusivement sur DOC - DZ : www.doc-dz.com NADJI 85
RESIDANAT EN POCHE TOME II
Cas Clinique en QCM

Vous voulez donner une chimioprophylaxie antigrippale dans l'entourage de cet enfant. Quel produit choisissez-
vous ?
A - L'acycloguanosine (Acyclovir ou Zovirax®)
B - La moroxydine (Assur®)
C - L'isoprinosine (Inosiplex®)
D - La méthisazone (Marboran®)
E - L'amantadine (Mantadix®)
Bonne(s) réponse(s) : E

Sans commentaire.

Bernard, un garçon de 8 ans, est le 3ème enfant d'une famille de 5 enfants présente une protéinurie découverte il y a un an à
l'occasion d'un examen scolaire (protéinurie +++ 0,8 g/l).
On note un examen clinique normal, une TA à 11/7 cmHg, une croissance satisfaisante, mais un sujet longiligne avec une
hyperlordose. Il existe un phimosis.
Sang : urée 4 mmol/l, créatinine 60 micromoles/l, VS 6 mm/20 mm, protéines 65 g/l, ASLO 50 U/ml, GR 5 100.000/mm3, (Hb
13 g %, 6200 GB/mm, CH 50, normal, C3 = 109 mg %.(Nl : 130 à 180 mg %).
Urines : volume de 24 heures 500 ml ; comptage de Malassez 10 GB/mm3 et 50 GR/mm3, uroculture avec 30 000
colibacilles/ml, protéinurie intermittente hautement sélective.

Quels sont en général les 2 arguments en faveur d'une erreur de dosage de la protéinurie ?
A - Nettoyage insuffisant des organes génito-urinaires externes chez la fille
B - Recueil des urines dans un récipient sale
C - Présence de 30 000 colibacilles/ml d'urine
D - Présence d'un phimosis
E - Présence de 50 GR/mm3
Bonne(s) réponse(s) : A D

Sans commentaire.

Quel est chez cet enfant, l'élément en faveur d'une protéinurie pathologique ?
A - Protéinurie +
B - Protéinurie 0,8 g/l
C - Protéinurie hautement sélective
D - Protéinurie intermittente
E - Protéinurie matinale
Bonne(s) réponse(s) : C

Sans commentaire.

Vous retenez également en faveur d'une protéinurie orthostatique :


A - Protéinurie intermittente
B - Complément abaissé
C - Sujet avec hyperlordose
D - Protéinurie permanente
E - Sujet longiligne
Bonne(s) réponse(s) : A C E

Sans commentaire.

Vous retenez en faveur d'un bon pronostic :


A - Le complément normal
B - Absence d'insuffisance rénale glomérulaire
C - TA à 11/7 cm Hg
D - Protéinurie hautement sélective
E - Absence d'infection urinaire associée
Bonne(s) réponse(s) : B

Sans commentaire.

409
Exclusivement sur DOC - DZ : www.doc-dz.com NADJI 85
RESIDANAT EN POCHE TOME II
Cas Clinique en QCM

Sur quel(s) argument(s) faut-il effectuer des examens complémentaires à la recherche d'une uropathie
malformative ?
A - Absence d'infection urinaire
B - Présence de 50 GR/mm3 (urines)
C - Notion d'une lithiase urinaire chez un oncle maternel
D - Existence de malformations des pavillons des oreilles
E - Protidémie à 65 g/l
Bonne(s) réponse(s) : B D

Sans commentaire.

Vous êtes appelé auprès d'un enfant de 15 mois qui présente une crise convulsive depuis 10 minutes. Le tableau que vous
constatez est celui d'une crise hémiclonique droite typique avec perte de conscience et révulsion oculaire. Spontanément,
quelques minutes après votre arrivée, la crise cède et l'examen postcritique montre, l'enfant restant inconscient, une
hémiplégie droite, une hyperthermie à 39° avec, comme seul élément infectieux, une otite purulente spontanément fistulisée,
évoluant depuis 3 jours : l'hyperthermie qui l'accompagne est rendue oscillante par le traitement en cours, uniquement
antithermique. Les antécédents personnels et familiaux sont absolument libres le développement de l'enfant a été normal, il
n'a jamais présenté de crises auparavant.

Parmi les sémiologies suivantes, indiquez celle qui ne fait pas partie du tableau des crises convulsives fébriles ?
A - Crise hémiclonique
B - Crise tonique pure généralisée
C - Crise hypotonique
D - Spasmes en flexion
E - Etat de mal généralisé tonicoclonique
Bonne(s) réponse(s) : D

Sans commentaire.

Une crise hémiconvulsive durant moins d'1/4 d'heure ne fait en général pas courir de risques. Au-delà d'une
demi-heure (état de mal) les risques se multiplient. Cochez celui ou ceux que vous pensez réel(s) ?
A - Hématome sous-dural
B - Oedème cérébral et engagement
C - Auto-entretien convulsif rebelle au traitement
D - Séquelles hémiplégiques
E - Aucune proposition n'est exacte
Bonne(s) réponse(s) : C D

Sans commentaire.

Le tableau clinique réalisé en référence peut a priori faire discuter diverses causes. Cochez celle ou celles qui
est ou qui sont possible(s) .
A - Thrombophlébite cérébrale
B - Abcès du cerveau
C - Convulsion hyperthermique
D - Méningite purulente
E - Epanchement sous-dural
Bonne(s) réponse(s) : A B C D E

Sans commentaire.

Les signes de focalisation neurologique disparaissent en 1 heure, sans récidive critique. Parmi les examens
paracliniques suivants, cochez celui qui est à réaliser immédiatement :
A - Echographie transfontanellaire
B - Analyse cytobactériologique du L.C.R
C - Electroencéphalogramme
D - Radiographie du crâne
E - Ponction des espaces sous-duraux
Bonne(s) réponse(s) : B

Sans commentaire.

410
Exclusivement sur DOC - DZ : www.doc-dz.com NADJI 85
RESIDANAT EN POCHE TOME II
Cas Clinique en QCM

Connaissant les risques et les effets secondaires des vaccinations cochez celle ou celles qui étant
dangereuse(s) devra(devront) être dorénavant proscrite(s) chez cet enfant :
A - Vaccination antirougeoleuse
B - Vaccination antidiphtérique, antitétanique, antipoliomyélitique (D.T.P.)
C - Vaccination anticoquelucheuse
D - Vaccination antivariolique
E - Vaccination antirubéolique
Bonne(s) réponse(s) : C D

Sans commentaire.

La décision éventuelle du traitement préventif des récidives convulsives chez ce malade pourrait découler d'un
seul élément de son histoire clinique. Indiquez-le :
A - L'âge
B - Le caractère de première crise fébrile
C - Le caractère hémiconvulsif de la crise
D - La durée de la crise
E - Signes déficitaires postcritiques transitoires
Bonne(s) réponse(s) : E

Sans commentaire.

Un enfant de 7 ans. vit dans une ferme où l'on trouve beaucoup d'animaux (volailles, bovins, caprins, chat). Il est fatiguée
depuis 15 jours, avec un discret décalage thermique (37,5 - 37,8°C). Vous découvrez des adénopathies cervicales, occipitales
notamment. Le diagnostic de toxoplasmose acquise est envisagé.

Parmi les causes de contamination suivantes, indiquez la ou les plus vraisemblable(s) :


A - Contact avec les chiens
B - L'enfant dort avec le chat
C - Consommation de crûdités tirées du potager
D - Le frère cadet a fait 3 mois plus tôt une toxoplasmose semblable
E - Consommation de volailles issues de l'élevage familial
Bonne(s) réponse(s) : B C

Sans commentaire.

Parmi les résultats suivants de l'hémogramme, indiquez celui le plus fréquemment observé dans ce diagnostic :
A - Leucopénie avec formule leucocytaire normale
B - Leucocytose élevée avec syndrome mononucléosique
C - Leucopénie avec syndrome mononucléosique et légère éosinophilie
D - Leucocytose normale avec lymphocyte élevée
E - Absence d'anomalie
Bonne(s) réponse(s) : E

On peut voir une leucopénie avec syndrome mononucléosique et légère éosinophilie.

Parmi les réponses sérologiques suivantes, indiquez celle(s) qui témoigne(nt) d'une toxoplasmose en cours
évoluant cliniquement depuis 15 jours :
A - Test de Remington positif
B - Absence d'anticorps après mercaptoéthanol
C - Test de lyse (dye test) seul positif
D - Sérologie encore négative à ce stade
E - Aucune réponse parmi celles ci-dessus
Bonne(s) réponse(s) : A

Sans commentaire.

411
Exclusivement sur DOC - DZ : www.doc-dz.com NADJI 85
RESIDANAT EN POCHE TOME II
Cas Clinique en QCM

Quelle(s) attitude(s) thérapeutique(s) vous paraît(ssent) correcte(s) ?


A - Abstention
B - Sulfadiazine + pyriméthamine
C - Spiramycine
D - Doxycycline
E - Erythromycine
Bonne(s) réponse(s) : C

Sans commentaire.

Naïm, de race noire, né et domicilié en France (logement exigu), est hospitalisé le 13 octobre 1981 à l'âge de 4 ans.
Mère : rougeole à 10 ans.
Fratrie : garçon 4 ans, fille 2 ans, fille 3 mois.
Les deux ainés ont reçu les vaccins B.C.G. et D.T.Coq.polio. avec rappel.
Aucun d'eux n'a fait la rougeole. Naïm a eu plusieurs otites.
Le 1er octobre, Naïm (mais non ses soeurs) est en contact avec un enfant rougeoleux.
Le 11 au soir et le 12 octobre, Naïm présente fièvre à 39°, asthénie, anorexie, et catarrhe oculonasal.
Le 13 octobre, il est hospitalisé pour fièvre à 40°, toux et voix rauques, conjonctivite et rhinite purulentes.
Le médecin traitant a posé le diagnostic de rougeole.

Quel(s) est(sont) le(s) élément(s) qui, au cours des journées du 14 et 15 octobre va(vont) permettre la
confirmation du diagnostic de rougeole ?
A - Desquamation de la langue
B - Eruption vésiculeuse
C - Exanthème débutant derrière les oreilles
D - Exanthème plus marqué au niveau des plis
E - Persistance de la fièvre
Bonne(s) réponse(s) : C

Sans commentaire.

Quelle(s) complication(s) peut-on craindre à court terme chez cet enfant ?


A - Laryngite sous-glottique oedémateuse
B - Pneumopathie
C - Otite
D - Pyurie
E - Herpès oculaire
Bonne(s) réponse(s) : A B C

Sans commentaire.

Quelle est la complication grave pouvant survenir plusieurs années après la rougeole ?
A - Sclérose latérale amyotrophique
B - Leucoencéphalite subaiguë sclérosante
C - Myocardite
D - Acrodynie
E - Diabète insipide
Bonne(s) réponse(s) : B

Sans commentaire.

Quelle est votre opinion au sujet de l'antibiothérapie dans le cas particulier de Naïm ?
A - Non justifiée
B - Erythromycine per os (Erythrocine®)
C - Tétracycline per os (Tétracyne®)
D - Gentamïcine intramusculaire (Gentalline®)
E - Céphalosporine de 3ème génération par voie intraveineuse (Claforan®)
Bonne(s) réponse(s) : B

Sans commentaire.

412
Exclusivement sur DOC - DZ : www.doc-dz.com NADJI 85
RESIDANAT EN POCHE TOME II
Cas Clinique en QCM

Quelle sera votre conduite immédiate vis à vis de la soeur de 2 ans ?


A - Traitement prophylactique par érythromycine per os (Erythrocine®)
B - Gammaglobulines humaines standard en I.M.
C - Vaccin antirougeoleux
D - Hospitalisation ce jour
E - Compte tenu de son âge, risque de rougeole nul
Bonne(s) réponse(s) : B

Sans commentaire.

Quelle sera votre conduite immédiate vis a vis de la soeur de 3 mois ?


A - Traitement prophylactique par érythromycine per os (Erythrocine®)
B - Gammaglobulines humaines standard en I.M.
C - Vaccin antirougeoleux
D - Hospitalisation ce jour
E - Compte tenu de son âge, risque de rougeole nul
Bonne(s) réponse(s) : E

Sans commentaire.

Quel est votre opinion au sujet du vaccin anti-rougeoleux ?


A - C'est un vaccin viral, tue, inactivé
B - Il comporte trois injections à un mois d'intervalle, rappel un an après
C - A faire dès l'âge de 3 mois
D - A réserver pour certaines indications individuelles (enfants à risque)
E - A conserver à + 4 degrés
Bonne(s) réponse(s) : E

Sans commentaire.

Vous êtes appelé auprès d'un garçon de 6 mois, chez lequel est survenue une crise douloureuse, apparemment abdominale,
avec vomissement, puis retour calme complet. Lorsque vous l'examinez, survient une autre crise identique à celle décrite par
sa mère : elle vous précise de plus qu'il existe depuis le premier épisode un refus de boire.

Parmi les signes cliniques suivants, indiquez celui ou ceux qui doi(ven)t faire envisager le diagnostic
d'invagination intestinale aiguë :
A - Evolution par crises
B - Selles liquides
C - Vomissement initial
D - Refus de boire
E - Fosse iliaque droite vide à la palpation
Bonne(s) réponse(s) : A D E

B - N'élimine pas le diagnostic mais non classique.


C - Typiquement tardif.

Indiquez, parmi les suivantes, I'(es) affection(s) qui peu(ven)t provoquer des selles sanglantes chez les
nourrisson :
A - Ulcère gastro-duodénal
B - Invagination intestinale
C - Ulcère du diverticule de Meckel
D - Mucoviscidose
E - Polypose rectocolique
Bonne(s) réponse(s) : A B C E

D - Il s'agit d'une stéatorrhée.

413
Exclusivement sur DOC - DZ : www.doc-dz.com NADJI 85
RESIDANAT EN POCHE TOME II
Cas Clinique en QCM

L'examen de l'abdomen entre deux crises à permis de sentir le boudin d'invagination, ce boudin à pour
caractéristique(s) clinique(s) :
A - Tuméfaction cylindrique allongée
B - Tympanisme à la percussion
C - Présence d'ondulations péristaltiques
D - Siège habituel, sous hépatique
E - Tête peut être perçue au toucher rectal
Bonne(s) réponse(s) : A D E

B - Non, mat.
C - Non, il existe une occlusion.
D - Au niveau du colon transverse.
E - Dans les formes évoluées.

Une désinvagination complète au cours du lavement baryté se traduira radiologiquement par :


A - Rejet rapide de la baryte par l'anus
B - Opacification complète du caecum
C - Absence d'encoche du bord interne du caecum
D - Opacification de l'appendice
E - Reflux franc de baryte dans les dernières anses grêles
Bonne(s) réponse(s) : B C E

Signes à bien connaître car une désinvagination lors du lavement baryté peut permettre d'éviter une sanction chirurgicale.

Un enfant de 12 ans est atteint de diarrhée aiguë depuis hier matin. Il a eu 10 - 12 selles glaireuses, la fièvre est à 38,5°C.
L'abdomen est ballonné et douloureux dans son ensemble, sans splénomégalie. La coproculture qui avait été prescrite
aussitôt, montre la présence de Salmonella typhimurium.

Quel(s) est (son) le ou les risque(s) éventuel(s) d'une telle infection digestive chez cet enfant ?
A - Etat septicémique
B - Cholecystite aiguë
C - Métastases septiques ostéo-articulaires
D - Perforation digestive
E - Hépatite aiguë
Bonne(s) réponse(s) : A C D

A C D - Complications classiques.
B - Surtout dans les fièvre thyphoïdes.

Quel traitement préconisez-vous en première intention chez cet enfant ?


A - Chloramphénicol per os
B - Ampicilline IM
C - Gentamicine IM
D - Antidiarrhéïques opiacés per os
E - Réhydratation orale
Bonne(s) réponse(s) : E

Toujours penser à réhydrater un enfant déshydraté !

Si cet enfant fréquente habituellement la cantine scolaire, quelle mesure préconisez-vous ?


A - Antibiothérapie orale de l'enfant à visée de stérilisation digestive rapide
B - Antibiothérapie parentérale de l'enfant dans le même but
C - Eviction scolaire de l'enfant pendant 14 jours
D - Enquête épidémiologique parmi les enfants fréquentant la même cantine
E - Ensemencement immédiat des selles du personnel de la cantine
Bonne(s) réponse(s) : D

A B - Inutile.
C - Un peu sévère.
D - Réponse très discutable.

414
Exclusivement sur DOC - DZ : www.doc-dz.com NADJI 85
RESIDANAT EN POCHE TOME II
Cas Clinique en QCM

Quelle prescription proposez-vous pour l'entourage familial de l'enfant ?


A - Coproculture
B - Antibiothérapie prophylactique
C - Immunoglobulines injectables
D - Vaccination antitypho-paratyphique
E - Aucune mesure prophylactique autre que l'hygiène des mains
Bonne(s) réponse(s) : E

Réponse toujours valable.

Un enfant de 10 ans est atteint d'une angine aiguë érythémato-pultacée accompagnée d'une fièvre à 38,5°C, d'apparition
brutale, avec adénopathies sous angulo-maxillaires sensibles.

Parmi les cinq signes cliniques suivants, relever celui ou ceux qui est (sont) en faveur de la nature
streptococcique de l'angine ?
A - Eruption cutanée scarlatiniforme
B - Coryza
C - Adénopathies sous angulo-maxillaires
D - Douleurs abdominales
E - Toux de laryngo-trachéite
Bonne(s) réponse(s) : A C D

B - Aucun rapport.
E - Evoque plutôt un virus.
A C D - Evident, tableau de scarlatine (angine émétisante).

Quel(s) est (sont) l(es) examen(s) biologique(s) dont le résultat permettra, à breve échéance, d'orienter la
décision thérapeutique ?
A - Hémogramme
B - Prélèvement de gorge pour inoculation sur gélose du sang
C - Dosage des antistreptolysines O sériques
D - Dosage des anticorps antidiphtériques circulants
E - Réaction de Paul et Bunnel
Bonne(s) réponse(s) : A

Très discutable car en pratique, la décision thérapeutique ne dépend pas d'examen complémentaire.

Dans l'hypothèse d'une étiologie, quel traitement préconisez-vous ?


A - Pénicilline V orale pendant 10 jours
B - Gentamicine injectable IM pendant 5 jours
C - Rifampicine per os pendant 5 jours
D - Spiramycine (Rovamycine®) pendant 5 jours
E - Aucun traitement antibiotique
Bonne(s) réponse(s) : A

Sans commentaire.

Dans l'hypothèse d'une cause streptococcique, les complications dont le risque est réduit ou prévenu par le
traitement antibiotique appliqué dès le 2e jour de la maladie comportent :
A - Otite suppurée
B - Adénophlegmon cervical
C - Cardite rhumatismale
D - Chorée aiguë
E - Glomérulo-néphrite aiguë
Bonne(s) réponse(s) : A B C D

Classiquement, la glomérulonéphrite aiguë est mal prévenue par l'antibiothérapie.

415
Exclusivement sur DOC - DZ : www.doc-dz.com NADJI 85
RESIDANAT EN POCHE TOME II
Cas Clinique en QCM
Un garçon âgé de 4 ans est hospitalisé parce que, depuis huit jours, la maman à constaté des oedèmes des paupières le
matin au réveil et devant la persistance de cet état, avec prise de poids de 4 kg, le médecin traitant à demandé une recherche
de protéinurie dans les urines de 24 heures et il a découvert une protéinurie à 10 g/24 heures. Dans les antécédents de cet
enfant, la maman signale de nombreuses rhinopharyngites depuis la naissance mais l'état général a toujours été bien
conservé. A l'entrée dans le service, il pèse 20 kgs alors que son poids de référence était de 15 kgs, présente des oedèmes
des paupières, une ascite et des oedèmes des membres inférieurs. La TA est à 11/7. Il n'a pas de foyer infectieux ORL.
L'auscultation cardiaque est normale. L'examen pulmonaire retrouve une matité de la base gauche. Le reste du bilan clinique
est normal. Le bilan biologique montre une urée à 6 mmol/l, créatinine sanguine à 50 micromol/l, un ionogramme avec chlore
à 102 mmol/l, sodium 136 mmol/l, potassium 4 mmol/l. La V.S. est à 100 à la première heure. La numération formule sanguine
montre 15 000 globules blancs, 60 % de polynucléaires neutrophiles, 40 % de Iymphocytes. L'hémoglobine est à 12 g/100 ml,
le cholestérol à 11 mmol/l. L'électrophorèse des protides montre une protidémie à 33 g/l avec albuminémie à 20 g/l. Le débit
HLM montre 1 000 hématies/minute, 1 000 leucocytes par minute. La protéinurie est à 5 g/24 heures.

Le diagnostic de néphrose lipoïdique est évoqué. Sur quel(s) argument(s) ?


A - Protéinurie à 50 mg/kg/24 h
B - Hypoalbuminémie < 30 g/l
C.- Absence d'hématurie microscopique
D - Oedèmes
E - Prise de poids
Bonne(s) réponse(s) : A B C D E

A B - Définition du syndrome néphrotique.


C - La néphrose lipoïdique entraîne en général un syndrôme néphrotique pur, donc sans HTA, ni hématurie ni insuffisance
rénale.
D et E sonr dus à l'hypoprotidémie.

Dès la première prise en charge de l'enfant il est justifié de demander :


A - Un index de sélectivité de la protéinurie
B - Une urographie intra-veineuse
C - Une cystographie rétrograde
D - Une ponction biopsie rénale
E - Une artériographie rénale
Bonne(s) réponse(s) : A

A - Classiquement, protéinurie sélective.


D - Non, pas initialement.
B C E - Sans rapport.

Il est recommandé d'entreprendre immédiatement :


A - Une restriction hydrique
B - Un régime sans sel
C - Une injection de Lasilix à la dose de 10 mg/kg/24 h
D - Perfusion d'albumine de 20 g/jour pendant trois jours
E - L'administration d'Oracilline : 2 M/jour
Bonne(s) réponse(s) : A B

A - Surtout en cas d'hyponatrémie.


B - Evident.
D - Discutable, mais non justifié dans ce cas.

Parmi les complications suivantes, vous devez craindre chez cet enfant :
A - Péritonite purulente
B - Méningite purulente
C - Syndrome sub-occlusif
D - Thrombose des veines rénales
E - Invagination intestinale aiguë
Bonne(s) réponse(s) : A C D

A - Complication infectieuse.
B - Discutable.
D - Troubles de la coagulation avec risque de thrombose.

416
Exclusivement sur DOC - DZ : www.doc-dz.com NADJI 85
RESIDANAT EN POCHE TOME II
Cas Clinique en QCM

Parmi les évolutions possibles du syndrome néphrotique de cet enfant, quelle est la plus fréquente ?
A - La rémission complète suivie d'une rechute
B - L'absence d'amélioration
C - La guérison sans rechute
D - L'insuffisance rénale chronique
E - L'hypertension artérielle maligne
Bonne(s) réponse(s) : A

Sans commentaire.

Vincent, d'origine française est né le 15 mars 1986 à terme, il pesait 3 Kg 250, mesurait 49,5 cm.

Vous revoyez cet enfant pour un examen de routine à 3 mois. Il pèse 5 Kg 475 et mesure 59 cm. Vous
interrogez la mère sur l'alimentation de l'enfant . Quel ou quels élément(s) vous parait ou vous paraissent
inapproprié(s) dans ce régime ?
A - 5 biberons de 150 g
B - Jus de fruits
C - Deux cuillerées à soupe de farine sans gluten dans chaque biberon
D - Un des biberons comporte l'apport de légumes verts
E - De l'eau sucrée est administrée entre chaque repas
Bonne(s) réponse(s) : C E

C - L'utilisation dans chaque biberon est excessive.

L'enfant à reçu régulièrement jusqu'alors de la vitamine D. Vous prescrivez la poursuite de ce traitement ; quelle
ou quelles est sont la ou les modalité(s) possible(s) ?
A - 200 000 unités tous les mois
B - 1 200 unités chaque jour
C - 200 000 unités tous les 4 à 6 mois
D - 2 000 unités chaque jour
E - Une dose unique de 600 000 unités non renouvelable
Bonne(s) réponse(s) : B C

Sans commentaire.

La mère demande de la conseiller au sujet des vaccinations. Parmi les propositions suivantes laquelle ou
lesquelles est ou sont fausse(s) ?
A - La vaccination contre la variole est obligatoire
B - Le BCG doit être obligatoirement pratiqué avant 1 an
C - La vaccination anti coquelucheuse est obligatoire
D - La vaccination contre la poliomyélite doit de préférence utiliser le vaccin vivant par voie buccale lors de la
primo vaccination
E - La vaccination contre la rubéole ne doit être pratiquée que chez les filles
Bonne(s) réponse(s) : A B C D E

A - Ne se fait plus.
B - Avant 6 ans.
D - C'est lors des rappels qu'on propose le vaccin vivant.

15 jours plus tard vous voyez l'enfant en urgence en raison de l'apparition de selles liquides et nombreuses (7 à
8 par jour) il pèse 5 Kg 200. Quel(s) élément(s) doit ou doivent figurer sur votre ordonnance ?
A - Restreindre les boisson, ne donner qu'une alimentation épaisse
B - Interrompre tout apport de lait pendant 24 à 48 heures
C - Faire pratiquer chaque jour une injection intra musculaire de Colimycine
D - Peser l'enfant 2 fois par jour
E - Donner à boire fréquemment un soluté hydroélectrolytique type OMS
Bonne(s) réponse(s) : B D E

D - La surveillance du poids est fondamentale.


E - Soluté avec un rapport sodium sur glucose permettant de réamorcer la pompe à sodium.

417
Exclusivement sur DOC - DZ : www.doc-dz.com NADJI 85
RESIDANAT EN POCHE TOME II
Cas Clinique en QCM

Vous prévenez les parents qu'il sera peut être nécessaire d'hospitaliser l'enfant; dans quel ou quels cas ?
A - Si le poids devient inférieur à 4 Kg 900
B - Si les selles prennent une coloration verte
C - Si la diarrhée s'accompagne de vomissements incoercibles
D - Si la température atteint 39 degrés
E - Si apparition de troubles de la conscience
Bonne(s) réponse(s) : A C E

A - Déshydratation sévère de plus de 10 %.


C - Réhydratation per os impossible.
D - Non justifié.
E - Evident.

Julien âgé de 6 mois présente depuis 48 h des vomissements en jets, une somnolence, une fièvre à 40 degré. A l'examen, le
développement staturopondéral est normal de même que le périmètre crânien, mais la fontanelle est tendue. L'enfant est
douloureux et hypotonique. Il existe une mauvaise hémodynamique. En dehors de la somnolence, l'examen neurologique est
normal.

La mauvaise hémodynamique se traduit par :


A - Temps de recoloration capillaire au talon à 5 secondes
B - Pouls à 130 battements par minute
C - Fréquence respiratoire à 40 par minute
D - Tension artérielle systolique à 90 mm Hg
E - Diurèse de 0,5 ml/Kg/heure
Bonne(s) réponse(s) : A C E

- La polypnée est un excellent signe de collapsus.


- Diurèse normale : > 1 ml/kg/h.

Vous suspectez une méningite. Une ponction lombaire est pour l'instant contre-indiquée par :
A - Calcémie à 50 mg/l
B - Taux de plaquettes à 150 000/mm3
C - Tension artérielle systolique à 40 mm Hg
D - Anémie à 9,5 g HB % Ml
E - Tension de la fontanelle
Bonne(s) réponse(s) : C

QUESTION ANNULEE.
En ce cas, l'urgence est au remplissage.

Dans une méningite purulente on observe couramment dans le LCR :


A - 250 éléments blancs/mm3
B - Présence de polynucléaires altérés
C - Protéinorrachie à 1,80 g/l
D - Glycorrachie à 0,9 mmol/l
E - Diminution de l'acide lactique
Bonne(s) réponse(s) : A B C D

Connaissances.

A l'examen direct du LCR, le laboratoire répond : présence de nombreux diplocoques gram positif. Quel est le
germe responsable à priori chez ce nourrisson ?
A - Méningocoque
B - Pneumocoque
C - Hémophilus influenzae
D - Listeria monocytogénès
E - Eschérichia coli
Bonne(s) réponse(s) : B

A - Coque Gram -
C - Bacille Gram -
D - Bacille Gram +
E - Bacille Gram -

418
Exclusivement sur DOC - DZ : www.doc-dz.com NADJI 85
RESIDANAT EN POCHE TOME II
Cas Clinique en QCM

Quel(s) antibiotique(s) peu(ven)t être utilisé(s) en monothérapie dans ce cas ?


A - Ampicilline
B - Colimycine
C - Amoxicilline
D - Amikacine
E - Erythromycine
Bonne(s) réponse(s) : A C

Evident.

Un enfant de 7 ans, a depuis l'âge de 2-3 mois des lésions prurigineuses qui ont débuté à la face et qui se sont
progressivement généralisées ; les phases d'aggravation ont surtout été hivernales et marquées par des surinfections
cutanées staphylococciques et herpétiques. A partir de l'âge de 2 ans, l'enfant a également eu des accès de dyspnée et de
fréquentes infections O.R.L. et bronchiques. Son état cutané et respiratoire s'améliore nettement en été, surtout en altitude. Il
n'a pas eu d'autres maladies graves et a eu toutes les vaccinations obligatoires sans complications. L'examen clinique montre
: une peau uniformément sèche et squameuse : des lésions d'eczéma de la face et du cou, des plis des coudes, des mains et
des creux poplités ; de nombreuses cicatrices de grattage ; des adénopathies périphériques axillaires et inguinales ; une
respiration bruyante avec des ronchus et des sibilances à l'auscultation.
L'enfant est nerveux et ne cesse de se gratter pendant l'examen clinique. L'interrogatoire apprend que sa mère avait des
lésions d'eczéma pendant l'enfance ; il n'y a pas d'autres antécédents familiaux et l'enfant est fils unique. Ses parents sont
très préoccupés par son état de santé : le prurit le rend insomniaque; son développement staturo-pondéral est ralenti malgré
un bon appétit et une alimentation normale ; sa fréquentation scolaire est irrégulière du fait des poussées d'eczéma et des
surinfections bactériennes nécessitant des soins à domicile.

A ce stade, votre sélection d'hypothése(s) diagnostique(s) comporte :


A - Une dermatite atopique
B - Une ichtyose
C - Une pollinose
D - Une dermatite herpétiforme
E - Un asthme
Bonne(s) réponse(s) : A B E

B - Lésions cutanées sèches et squameuses.

La biologie devrait, significativement, mettre en évidence :


A - Une leucopénie inférieure à 3 000 G.B./mm3
B - Une éosinophile supérieure à 800/mm3
C - Une hypergammaglobulinémie polyclonale
D - Une augmentation isolée des IgE sériques
E - Une hypocalcémie
Bonne(s) réponse(s) : B D

Connaissance.

Le bilan diagnostique immunologique peut utilement comporter :


A - Une immuno électrophorèse des protéines sériques
B - Des tests cutanés à lecture immédiate (20 min.) pour chercher une sensibilisation réaginique aux
pneumallergènes
C - Des RAST-IgE pour les mêmes pneumallergènes
D - Un dosage de l'histaminémie
E - Un dosage du complément sérique
Bonne(s) réponse(s) : B C

A - Non, dosage pondéral des IgE.


D E - Inutile.

En cas de sensibilisation réaginique aux pneumallergènes, quel est le pneumallergène qui donnera le plus
probablement des tests cutanés ou des RAST spécifiques positifs chez cet enfant ?
A - Les pollens de graminées
B - Les squames humaines
C - Les poils d'animaux
D - Les acariens dermatophagoïdes de la poussière
E - Les moisissures
Bonne(s) réponse(s) : D

Amélioration en altitude où il n'y a pas d'acarien.

419
Exclusivement sur DOC - DZ : www.doc-dz.com NADJI 85
RESIDANAT EN POCHE TOME II
Cas Clinique en QCM

Quelle est parmi les propositions thérapeutiques suivantes celle qu'il faut rejeter à priori chez cet enfant ?
A - La corticothérapie générale
B - L'antibiothérapie générale adaptée aux staphylocoques
C - La théophylline
D - Les antihistaminiques anti-H 1
E - Le cromoglycate disodique oral ou en inhalations
Bonne(s) réponse(s) : A

Evident.

Vous êtes appelé auprès d'un enfant de 18 mois qui présente une crise convulsive depuis 10 minutes. Le tableau que vous
constatez est celui d'une crise clonique, hémicorporelle droite, avec perte de connaissance et révulsion oculaire.
Spontanément, à votre arrivée, la crise cède et l'examen postcritique montre un enfant inconscient, une hémiplégie droite, une
hyperthermie à 39 degrés. Par ailleurs, l'examen clinique montre comme seul élément infectieux, une otite purulente. Les
antécédents personnels et familiaux sont absolument libres et le développement de l'enfant a été normal. Il n'a jamais
présenté de crise convulsive auparavant.

Parmi les présentations suivantes, indiquer celle ou celles qui ne fait ou ne font pas partie du tableau des crises
convulsives fébriles :
A - Crises hémicloniques
B - Crises toniques pures généralisées
C - Crises hypotoniques
D - Spasmes en flexion
E - Etat de mal, généralisé tonico-clonique
Bonne(s) réponse(s) : D

Les formes sensitives pures ne font pas partie non plus du tableau.
B - Les crises toniques pures éliminent les convulsions du 5ème jour mais pas les convulsions hyperthermiques.

Une hémicrise convulsive durant 10 mn ne fait en général. pas courir de risques. Au-delà d'une demi-heure, les
risques se multiplient. Cocher celui ou ceux que vous pensez réel(s) ?
A - Hématome extra-dural
B - Hyperthermie maligne
C - Oedème cérébral et engagement
D - Auto-entretien convulsif rebelle aux traitements
E - Séquelles hémiplégiques
Bonne(s) réponse(s) : C D E

A - Cause possible et non conséquence.


B - Sans rapport.
E - Se voit lors du syndrome hémiplégie-hémiconvulsion et épilepsie.

Le tableau clinique réalisé en référence peut, à priori, faire discuter diverses causes. Cocher celle ou celles qui
est ou qui sont possible(s) :
A - Thrombophlébite cérébrale
B - Abcès du cerveau
C - Encéphalite herpétique
D - Convulsions hyperthermiques
E - Méningite purulente
Bonne(s) réponse(s) : A B C D E

Toutes ces causes peuvent donner le tableau décrit. On rappelle que le diagnostic de convulsion hyperthermique est un
diagnostic "d'élimination".

Les signes de localisation neurologique disparaissent en 1 h, sans récidive critique ; parmi les examens
paracliniques suivants. Cocher celui qu'il vous paraît fondamental de réaliser immédiatement.
A - Dosage des électrolytes sanguins
B - Analyse cyto-bactério-immunologique du L.C.R.
C - Electroencéphalogramme
D - Examen scannographique cérébral
E - Ponction des espaces sous-duraux
Bonne(s) réponse(s) : B

B - Evident, il faut éliminer en urgence une méningite.


C - Apportera peu d'éléments.

420
Exclusivement sur DOC - DZ : www.doc-dz.com NADJI 85
RESIDANAT EN POCHE TOME II
Cas Clinique en QCM

Connaissant les risques et les effets secondaires des vaccinations, cocher celle qui devra être dorénavant
interdite chez cet enfant.
A - Vaccination anti-rougeoleuse
B - Vaccination anti-diphtérique, anti-tétanique
C - Vaccination anti-coquelucheuse
D - Vaccination anti-poliomyélitique
E - B.C.G.
Bonne(s) réponse(s) : C

- Le vaccin anticoqueluche est responsable d'une hyperthermie majeure.


- A noter, pour certains il faut aussi contre-indiquer le vaccin antirougeole.

La décision éventuelle du traitement préventif des récidives convulsives chez ce malade pourrait découler d'un
seul élément de son histoire clinique. Indiquez-le :
A - L'âge
B - Le caractère clonique de la 1ère crise fébrile
C - Le caractère hémi-convulsif de la crise
D - La durée de la crise
E - L'importance de l'hyperthermie
Bonne(s) réponse(s) : C

A - Il aurait fallu un âge inférieur à 9 mois.


D - Il aurait fallu une crise d'une durée supérieure à 15-30 minutes.
Les autres critères de mise sous traitement préventif sont : existence d'un déficit post-critique et récidives.

Un nourrisson de 6 mois est amené pour une éruption du visage et des membres extrêmement prurigineuse, cause
d'insomnies de l'enfant et des parents, ayant débuté six semaines auparavant. L'examen découvre un enfant en bon état
général, mais défiguré par des lésions érythémateuses, suintantes et croûteuses du visage, épargnant la région médio-faciale,
avec intertrigo rétro-auriculaire. Le dos des mains, la face dorsale des avant-bras et les jambes sont couverts de lésions
érythématosquameuses avec des érosions dues au grattage et des croûtes jaunâtres. Les fesses sont rouges avec quelques
érosions et les plis inguinaux et périnéaux légèrement suintants. Un traitement institué depuis un mois et comportant
l'application de crème Parfenac® et l'administration de Celestene® gouttes (bétaméthasone) 4 mg/jour puis à doses
dégressives a été suivi d'amélioration passagère mais n'a pas empêché la poussée actuelle. L'anamnèse nous apprend que
son frère âgé de 8 ans a eu plusieurs crises d'asthme, le père à une rhinite pollinique, il n'y a aucune autre dermatose ou
maladie générale dans la famille.

L'affection dont souffre ce nourrisson est théoriquement caractérisée par :


A - Début généralement à la période néonatale
B - Atteinte fréquente du pouce sucé
C - Le respect de la région centro-faciale
D - Le prurit insomniant
E - Le caractère suintant de la poussée éruptive
Bonne(s) réponse(s) : B C D E

A - Non, début à 3 mois.


B C D E - Classique.

Quelle(s) affection(s) parmi les suivantes pouvez-vous éliminer chez ce nourrisson, à la lecture de l'énoncé
clinique ?
A - Une gale acarienne
B - Une dermite séborrhéique
C - Une dermatite atopique
D - Des lésions impétiginisées
E - Une varicelle
Bonne(s) réponse(s) : A B E

Evident.
D - Rien dans ce dossier n'élimine l'impétigination.

Pour confirmer le diagnostic, vous attendez le résultat de :


A - Recherche d'une éosinophilie sanguine
B - Recherche de parasites dans les squames
C - Tests cutanés allergologiques
D - Dosage des IgE
E - Aucun de ces examens
Bonne(s) réponse(s) : E

Le diagnostic est clinique.


421
Exclusivement sur DOC - DZ : www.doc-dz.com NADJI 85
RESIDANAT EN POCHE TOME II
Cas Clinique en QCM

Parmi les mesures thérapeutiques permettant de prévenir les récidives, vous retenez :
A - Utilisation de préparations hydratantes
B - Désensibilisation aux pneumallergènes
C - Régime d'exclusion lactée
D - Injection d'immunoglobulines
E - Eviction des facteurs d'irritation cutanée
Bonne(s) réponse(s) : A E

B C - L'eczéma atopique n'est pas allergique.

Un nouveau-né de 10 jours, prématuré, dont l'examen à la naissance était normal présente un aspect de paralysie du membre
inférieur droit, avec gesticulation spontanée asymétrique. Depuis 24 heures, il refuse de s'alimenter, et perdu 150g. A
l'examen clinique : pas de fièvre ; oedème de la racine de cuisse droite ; l'enfant pleure dès qu'on le touche. Radiographie du
bassin de face : flou des parties molles, aspect d'excentration de la tête fémorale.

Quel diagnostic évoquez-vous ?


A - Luxation congénitale de hanche
B - Paralysie obstétricale
C - Décollement épiphysaire fémoral supérieur
D - Hémiplégie cérébrale
E - Aucune des hypothèses ci-dessus
Bonne(s) réponse(s) : E

On évoque une arthrite septique de hanche. La fièvre est rare chez le nouveau-né, même septicémique.

Quel(s) examen(s) faut-il pratiquer dans un but diagnostique ?


A - N.F., V.S., protéine C réactive
B - Echographie de hanche
C - Electromyogramme
D - Arthrographie de hanche
E - Ponction de hanche
Bonne(s) réponse(s) : A B E

A - La VS est très discutable chez le nouveau-né, la CRP est un bien meilleur examen pour juger du syndrôme inflammatoire.
B - Permet de voir un épanchement.
C D - Sans rapport.
E - Geste diagnostique et thérapeutique.

Quelle(s) sera(ont) la (les) base(s) du traitement ?


A - Ponction-lavage articulaire
B - Double antibiothérapie associée
C - Immobilisation plâtrée
D - Langeage en abduction
E - Anti-inflammatoires
Bonne(s) réponse(s) : A B C

A - Evite la destruction des cartilages de conjugaison.


B - Dirigé contre le gène en cause, souvent le staphylocoque doré.
C - Classique dans les arthrites.
D - C'est le traitement de la luxation de hanche, insuffisant ici.
E - Pas d'intérêt chez le nouveau-né.

Quelle(s) séquelle(s) peut-on craindre ?


A - Aucune
B - Luxation congénitale de hanche
C - Destruction de la tête fémorale
D - Inégalité de longueur des membres inférieurs
E - Paralysie du membre inférieur
Bonne(s) réponse(s) : C D

B - Luxation oui, mais pas congénitale.


C - En fait, destruction du cartilage de conjugaison.
D - Découle de C.
E - Pas d'atteinte des racines nerveuses.

422
Exclusivement sur DOC - DZ : www.doc-dz.com NADJI 85
RESIDANAT EN POCHE TOME II
Cas Clinique en QCM
Marc, un garçon de 3 ans, est adressé à l'hôpital pour une toux rebelle fébrile. L'histoire a commencé il y a 15 jours dans
l'après-midi, brutalement, par une quinte de toux très violente pendant laquelle l'enfant était très rouge. Puis, la toux s'est
calmée. 8 jours après l'épisode initial, la fièvre est apparue, 38° le matin, 39° le soir, et n'a pas cédé malgré la prise
d'Hiconcil® sirop. Le carnet de santé révèle que les vaccins sont à jour mais que le B.C.G. n'a pas été vérifié.

Parmi les diagnostics proposés ci-dessous, quel est celui qui vous parait le plus probable ?
A - Coqueluche
B - Tuberculose pulmonaire
C - Inhalation de corps étranger
D - Pneumonie franche lobaire aiguë
E - Pneumopathie virale
Bonne(s) réponse(s) : C

A - Vaccins à jour, et ce n'est pas la symptomatologie.


C - Syndrome de pénétration 15 jours auparavant.
B D E - N'explique pas la quinte de toux initiale et brutale.

Parmi les examens suivants, lequel doit apporter la clé du diagnostic ?


A - Intradermoréaction à la tuberculine
B - Bactériologie des crachats
C - Radio standard du thorax de face
D - Endoscopie bronchique à tube rigide
E - Tubage gastrique
Bonne(s) réponse(s) : D

Evident.

Le ou les élément(s) déterminant(s) pour le diagnostic est ou sont ?


A - L'âge
B - La quinte de toux initiale
C - L'absence initiale de fièvre
D - La résistance aux antibiotiques
E - L'absence de BCG
Bonne(s) réponse(s) : A B C D

Sans commentaire.

Vous pratiquez une radio pulmonaire. Quel cliché demandez-vous ?


A - Un simple cliché de face
B - Un cliché de face et un de profil
C - Un cliché en inspiration et un cliché en expiration
D - Un cliché du larynx de profil
E - Une opacification de l'oesophage
Bonne(s) réponse(s) : C

Recherche l'air piégé en expiration du fait de l'obstruction par le corps étranger.

Dès lors vous décidez que :


A - L'enfant est rendu à sa famille avec une ordonnance d'un autre antibiotique
B - L'enfant est adressé à un pneumologue
C - L'enfant est hospitalisé immédiatement
D - L'enfant est isolé de ses frères et soeurs
E - Radiographies pulmonaires et IDR à la tuberculine sont prescrites à toute la famille
Bonne(s) réponse(s) : C

Dans le but d'effectuer la bronchoscopie et d'extraire le corps étranger avec un traitement antibiotique adapté aux différents
prélèvements.

423
Exclusivement sur DOC - DZ : www.doc-dz.com NADJI 85
RESIDANAT EN POCHE TOME II
Cas Clinique en QCM
Un enfant de 4 ans, qui ne présente pas d'antécédent particulier et dont le développement psychomoteur a été normal se
plaint depuis une quinzaine de jours de céphalées de plus en plus intenses, survenant au réveil. Depuis trois jours, sont
apparus des vomissements d'horaire matinal, se répétant une à deux fois et soulageant la céphalée.

Ce tableau évoque une hypertension intracrânienne (HTIC). Quel(s) examen(s) permette(nt) de la confirmer ?
A - Un fond d'oeil
B - Une échographie trans-fontanellaire
C - Une radiographie du crâne (face-profil)
D - Une ponction lombaire
E - Un électro-encéphalogramme
Bonne(s) réponse(s) : A C

A - Oedème papillaire.
B - Pas à cet âge.
C - Disjonction des sutures, empreintes digitiformes...
D - Certainement pas.
E - Sans rapport.

Le scanner cérébral met en évidence des signes d'hypertension intracrânienne et une formation tumorale au
niveau du vermis-cérébelleux. Quels sont les 2 diagnostics à évoquer en priorité ?
A - Un astrocytome cérébelleux
B - Un médulloblastome
C - Un gliome infiltrant
D - Un épendymome
E - Un pinéalome
Bonne(s) réponse(s) : A B

Sans commentaire.

Compte tenu des signes d'HTlC, quelles sont les 2 attitudes possibles ?
A - Une chimiothérapie d'urgence
B - Une radiothérapie centrée sur la tumeur
C - Une intervention neurochirurgicale d'emblée sur la fosse postérieure
D - Une dérivation ventriculaire-externe suivie de l'intervention sur la fosse postérieure
E - Une dérivation ventriculo-péritonéale suivie d'une radiothérapie centrée sur la tumeur
Bonne(s) réponse(s) : C D

A - Est très discutable car une chimiothérapie peut être active rapidement, notamment sur l'oedème associé.
B - Effet retardé.

Dans l'évolution ultérieure apparaît un syndrome de la queue de cheval. Vous l'interprêtez comme :
A - La séquelle d'une hypertension intracrânienne prolongée
B - La traduction d'une métastase osseuse
C - La traduction d'une métastase dans le cul de sac lombosacré
D - Le signe d'une atteinte neurogène périphérique secondaire à la thérapeutique
E - Une séquelle de la chirurgie d'exérèse
Bonne(s) réponse(s) : C

Evident.

Un petit garçon naît par voie basse, en présentation céphalique, au terme de 34 semaines. Le liquide amniotique est clair. Le
score d'Apgar est coté à 5 à la première minute et à 9 à la cinquième minute de vie, après 2 minutes de ventilation au
masque. Le poids de naissance est de 2 000 g. Au bout d'une heure de vie est constatée l'existence d'une détresse
respiratoire qui motive le transfert de l'enfant dans un service de néonatologie.

Une détresse respiratoire néonatale comporte typiquement :


A - Une accélération de la fréquence respiratoire au-dessus de 60/mn
B - Des signes objectifs de lutte respiratoire dits signes de tirage
C - Des apnées
D - Une cyanose à l'air ambiant (FiO2 = 0,21)
E - Aucune des propositions précédentes
Bonne(s) réponse(s) : A B D

A - Polypnée quasiconstante.
B - Rentrant classiquement dans le score de Silvermann.
C - Peut être en rapport avec la prématurité mais indépendamment de la détresse respiratoire.
D - Est due à l'hypoxie.

424
Exclusivement sur DOC - DZ : www.doc-dz.com NADJI 85
RESIDANAT EN POCHE TOME II
Cas Clinique en QCM

Le score de Silverman (ou indice de rétraction) cote de 0 à 2 :


A - Le geignement expiratoire
B - Le battement des ailes du nez
C - Le tirage sus-sternal
D - Le balancement thoraco-abdominal
E - Le creusement xyphoïdien
Bonne(s) réponse(s) : A B D E

C - Il s'agit d'un tirage intercostal. Le tirage sus-sternal est surtout le fait des dyspnées laryngées. Le tirage sous mandibulaire
est le fait des obstructions nasopharyngées.

Dans le cas de ce petit garçon, tous les diagnostics suivants peuvent à priori être évoqués sauf un : lequel ?
A - Maladie des membranes hyalines
B - Détresse respiratoire transitoire (retard de résorption)
C - Inhalation méconiale
D - Infection pulmonaire
E - Pneumothorax
Bonne(s) réponse(s) : C

Lire l'énoncé : il s'agit d'un liquide amniotique clair.

Votre prescription initiale d'examen(s) complémentaire(s), comporte :


A - Une radiographie du thorax de face
B - Des hémocultures
C - Une mesure de pH sanguin
D - Un ECG
E - Une mesure de la TcP02 ( PO 2 cutanée)
Bonne(s) réponse(s) : A B C E

A B - Evidents.
C - Gaz indiqués pour juger de la gravité de l'affection. Si leur mesure doit se répéter fréquemment, on pose un cathéter dans
l'artère ombilicale.
D - Sans rapport.
E - Evident.

Le traitement symptomatique initial comporte :


A - Le réchauffement en incubateur
B - La désobstruction régulière (par aspiration) des voies aériennes et digestives
C - La perfusion de sérum glucosé et de calcium
D - L'oxygénothérapie sous enceinte céphalique
E - L'administration de stimulants respiratoires (théophylline ou caféine)
Bonne(s) réponse(s) : A B C D

A - Il s'agit d'un nouveau né prématuré qui se refroidit rapidement.


B - Un nouveau né ne peut respirer que par le nez ; une obstruction nasale peut être responsable d'une détresse respiratoire.
D - Il s'agit du Hood.
E - Ce nouveau né n'a pas besoin d'être stimulé.

A posteriori. le diagnostic de maladie des membranes hyalines pourra être établi sur :
A- Rapport lécithine/sphingomyéline 2 dans les sécrétions pharyngées prélevées à l'admission
B - Durée d'évolution 3 jours de la détresse respiratoire
C - Positivité des prélèvements bactériologiques initiaux
D - Aspect micro-granité des 2 champs pulmonaires à la radio
E - Absence de toute acidose plasmatique au cours de l'évolution
Bonne(s) réponse(s) : B D

A - Inutile.
B - Classique aggravation pendant 48 heures puis plateau pendant 24 à 48 heures puis amélioration.
D - Aspect radiographique typique.

425
Exclusivement sur DOC - DZ : www.doc-dz.com NADJI 85
RESIDANAT EN POCHE TOME II
Cas Clinique en QCM
Vous êtes conduit à examiner un nouveau-né de 36 heures dont l'histoire est la suivante: Grossesse apparemment normale.
Accouchement à 39 semaines, spontané, par voie basse et en présentation céphalique. Deux éléments à noter: Rupture
prolongée de la poche des eaux de 48 heures et fièvre maternelle à 38,5°C au moment de l'accouchement. A la naissance,
l'Apgar est à 8 à une minute, à 9 à cinq minutes. Il s'agit d'un enfant eutrophique. Son examen est normal. Pendant les
premières 36 heures : rien à signaler. A 36 heures de vie, l'enfant refuse les tétées. On constate l'apparition d'une cyanose
modérée des extrémités. A l'examen le temps de recoloration se situe aux alentours de 3 à 4 secondes. Le pôle inférieur de la
rate déborde légèrement le rebord costal, il en est de même du foie (deux travers de doigt). La température est normale. Le
diagnostic d'infection materno-foetale est suspecté.

Quelles sont les deux localisations les plus fréquentes de l'infection bactérienne materno-foetale ?
A - Respiratoire
B - Cutanée
C - Méningée
D - Péritonéale
E - Urinaire
Bonne(s) réponse(s) : A C

Evident.

Quelles sont les trois constatations biologiques orientant vers une infection bactérienne du nouveau-né ?
A - Leuconeutropénie
B - Hypofibrinémie
C - Accélération de la vitesse de sédimentation
D - Thrombocytopénie
E - Augmentation de la C réactive protéine
Bonne(s) réponse(s) : A D E

B - Hypofibrinémie possible dans les CIVD.


C - Pas chez le nouveau-né.

Quels sont les trois prélèvements bactériologiques qui permettront d'affirmer avec certitude l'infection ?
A - Prélèvement gastrique
B - Prélèvement des selles
C - Hémoculture
D - Prélèvement d'urines
E - Culture du liquide céphalo-rachidien
Bonne(s) réponse(s) : C D E

A B - Après 36 heures, le nouveau-né a été alimenté et a probablement déjà émis son premier Méconium
C E - Sont des prélèvements centraux.
D - Est plus discutable.

Un traitement antibiotique est prescrit en urgence, avant identification du germe. Quels sont les deux
antibiotiques choisis de préférence parmi les suivants ?
A - Ampicilline
B - Streptomycine
C - Colistine
D - Gentamycine
E - Vancomycine
Bonne(s) réponse(s) : A D

Evident.

Quels sont, en matière d'infection materno-foetale, les trois germes les plus fréquemment retrouvés ?
A - Streptocoque B
B - Hémophilus
C - Colibacille
D - Pseudomonas
E - Listeria
Bonne(s) réponse(s) : A C E

Question de cours.
C - Il s'agit de E. Coli K1.

426
Exclusivement sur DOC - DZ : www.doc-dz.com NADJI 85
RESIDANAT EN POCHE TOME II
Cas Clinique en QCM
Noé L. 10 ans, sans antécédent notable. Une semaine auparavant infection respiratoire haute suivie de l'apparition de
douleurs abdominales et d'une éruption purpurique prédominant aux extrémités. A l'examen: bon état général, température
38,2°C, tension artérielle 9/6, pouls 110, temps de recoloration inférieur à 3 secondes, A l'examen cutané: lésions purpuriques
discrètement infiltrées siégeant au niveau de l'extrémité des 4 membres. Pas d'ecchymoses, pas d'hémorragie muqueuse, pas
d'atteinte du tronc et du visage.
Pas d'autre anomalie à l'examen.
Examens complémentaires :
- VS : 5 à la première heure
- GR 4 500 000 avec Hb 15 g, GB 15 500 dont polynucléaires 80 % plaquettes 358 000
- Bilan d'hémostase normal
- Culot urinaire : hématies 63 000/ml, leucocytes 5 000/ml, urines stériles à la culture
Protéinurie 0,30g/24h.
- Ionogramme sanguin normal
- Bilan hépatique normal avec des TGO à 15 et des TGP à 6.

Devant ce tableau purpurique fébrile, quel diagnostic devez-vous éliminer dès l'arrivée à l'hôpital ?
A - Purpura infectieux (méningococcique)
B - Purpura thrombopénique aiguë idiopathique
C - Leucémie aiguë
D - Syndrome de Silvermann
E - Prurigo parasitaire
Bonne(s) réponse(s) : A

Il s'agirait alors d'une extrême urgence.

En fait l'examen clinique et les données hématologiques permettent d'affirmer qu'il s'agit d'un purpura
rhumatoïde qui est compatible avec :
A - Température à 38°2
B - Lésions infiltrées
C - Plaquettes normales
D - VS normale
E - Bilan hépatique normal
Bonne(s) réponse(s) : A B C E

A - Fébricule à 38° classique.


C - Il s'agit d'un purpura vasculaire.
D - Classiquement élevée.

Parmi les complications suivantes, vous redoutez au cours du purpura rhumatoïde :


A - Invagination intestinale aiguë
B - Orchite
C - Convulsion
D - Nécrose hémorragique des surrénales
E - Hyponatrémie
Bonne(s) réponse(s) : A B C

A B - Classique.
C - Signes neurologiques possibles avec risque d'hémorragie intracérébrale ou méningée.
D - Se voit dans les méningococcémies (syndrômes de Waterhouse-Fredericksen).

Vous surveillerez au cours de l'évolution :


A - Plaquettes
B - Hématurie
C - Protéinurie
D - Bilan d'hémostase
E - VS
Bonne(s) réponse(s) : B C

L'atteinte rénale fait tout le pronostic de cette affection.

Le traitement qui vous parait approprié au cours d'un purpura non compliqué comporte :
A - Extencilline
B - Gammaglobulines intraveineuses
C - Corticoïdes
D - Vitamine K
E - Repos
Bonne(s) réponse(s) : E

Surtout en cas d'atteinte testiculaire.


427
Exclusivement sur DOC - DZ : www.doc-dz.com NADJI 85
RESIDANAT EN POCHE TOME II
Cas Clinique en QCM
Un enfant de 2 ans 1/2 présente une dyspnée inspiratoire avec bradypnée évoluant depuis plusieurs heures. Il est enrhumé
depuis plusieurs jours. Les parents vous appellent le soir parce que la dyspnée s'aggrave. A l'examen, vous trouvez un enfant
couché dans son lit, conscient, fébrile à 38°C, présentant un tirage sussternal et intercostal avec cornage. La dyspnée est
inspiratoire, la fréquence respiratoire est de 1 5/mn. De temps en temps, l'enfant présente des accès de toux rauque. Ses
vaccinations sont à jour.

Le type de dyspnée observée traduit un obstacle :


A - Nasal
B - Laryngé
C - Trachéal
D - Pharyngé
E - Bronchiolaire
Bonne(s) réponse(s) : B

A - Tirage sous-mandibulaire associé.


B - Bradypnée inspiratoire avec cornage.
C - Dyspnée aux deux temps.
D - Comme A.
E - Polypnée.

Quel diagnostic pouvez-vous poser avec le plus de probabilité ?


A - Rhino-pharyngite
B - Laryngite aiguë sous glottique
C - Epiglottite
D - Corps étranger dans les voies aériennes
E - Bronchiolite
Bonne(s) réponse(s) : B

Typique.

Votre attitude thérapeutique immédiate comporte :


A - Mise en atmosphère humide
B - Injection intramusculaire de 4 mg de Dexaméthasone
C - Injection intramusculaire d'Ampicilline (25 mg/kg)
D - Injection intrarectale de Diazépam (0,5 mg/kg)
E - Administration d'aspirine (25 mg/kg)
Bonne(s) réponse(s) : A B

Evident.

Par la suite, quelle conduite adoptez-vous ?


A - Hospitalisation immédiate systématique
B - Rester au chevet du malade pendant une 1/2 heure afin d'apprécier l'effet du traitement
C - Si non amélioration dans la 1/2 heure, hospitalisation
D - Si la dyspnée cède totalement : poursuite d'un traitement d'entretien pendant 48 heures
E - Prescription d'un antitussif
Bonne(s) réponse(s) : B C D

- Une laryngite sous glottique peut être très grave et justifie donc la surveillance au chevet du malade pour juger de l'efficacité
du traitement.

Dans le cas où vous auriez suspecté une épiglottite, votre attitude aurait comporté :
A - Hospitalisation immédiate en transportant l'enfant en position assise
B - Faire prévenir le service d'accueil
C - Injection intramusculaire de Lasilix®
D - Un traitement exclusif à domicile par Ampicilline
E - Repos au lit
Bonne(s) réponse(s) : A B

Une épiglottite se traite toujours par une intubation. Si elle est impossible : trachéotomie.

428
Exclusivement sur DOC - DZ : www.doc-dz.com NADJI 85
RESIDANAT EN POCHE TOME II
Cas Clinique en QCM
Fabien 6 ans, n'a jamais été propre la nuit et son lit est mouillé chaque matin. Son développement staturopondéral et
psychomoteur est strictement normal: ses parents sont séparés mais l'équilibre psychologique de l'enfant parait normal.
L'examen clinique est sans particularité, notamment en ce qui concerne les organes génitaux; il n'y a jamais eu d'infection
urinaire.

L'énurésie de l'enfant :
A - Est généralement primaire
B - Est toujours exclusivement nocturne
C - Est une forme clinique d'incontinence urinaire
D - Est le plus souvent associée à une encoprésie
E - Affecte plus souvent les garçons que les filles
Bonne(s) réponse(s) : A E

B - Peut être diurne.


C - Non, car une énurésie exclusivement nocturne exclut l'incontinence urinaire.

Le caractère quotidien de cette énurésie :


A - Impose un bilan uro-radiologique
B - Traduit une forme particulièrement sévère
C - Rend le diagnostic d'énurésie peu probable
D - Oriente vers un trouble de concentration des urines
E - A peu de valeur informative
Bonne(s) réponse(s) : E

Evident.

Le traitement de l'enfant pourra comporter :


A - Une psychothérapie prolongée
B - Le port de couches pour préserver le confort nocturne
C - La séparation du milieu familial
D - Des conseils hygiéno-diététiques
E - Un traitement médicamenteux (imipramine, desmopressine)
Bonne(s) réponse(s) : D E

A - Non, pas d'emblée.


B - Certainement pas.
C - Délirant.
D - Bien sûr, ne pas boire avant de se coucher, réveiller l'enfant pour le faire uriner en début de nuit.

Le pronostic de l'énurésie :
A - Est lié à son éventuel caractère familial
B - Dépend de son caractère primaire ou secondaire
C - Est bon, car elle disparait dans la majorité des cas
D - Dépend de l'uropathie sous-jacente
E - Est à établir en fonction des symptômes associés
Bonne(s) réponse(s) : C

Classique.

Un enfant de 14 ans présente 10 jours après une angine un oedème de la face et des urines anormalement colorées. A
l'examen on note une prise de poids de 1 kg et une T.A. à 130/85 mm Hg. Le diagnostic de glomérulonéphrite aiguë post-
angineuse paraît le plus plausible .

Quelle est la caractéristique symptomatique habituelle de l'angine précédant la glomérulonéphrite aiguë (GNA) ?
A - Erythémateuse
B - Ulcéro-nécrotique
C - Pseudo-membraneuse
D - Vésiculaire
E - Hémorragique
Bonne(s) réponse(s) : A

Puisque streptococcique.

429
Exclusivement sur DOC - DZ : www.doc-dz.com NADJI 85
RESIDANAT EN POCHE TOME II
Cas Clinique en QCM

Quel germe en est habituellement responsable ?


A - Staphylocoque
B - Entérocoque
C - Streptocoque
D - Germes gram négatif
E - Fuso-spirilles
Bonne(s) réponse(s) : C

Evident.

Quelle(s) perturbation (s) sont habituelles au début de la GNA ?


A - Hématurie
B - Hypocomplémentémie
C - Eosinophilie
D - Altération de la fonction rénale
E - Protéinurie
Bonne(s) réponse(s) : A B D E

Syndrome néphritique avec protéinurie modérée quasiconstante. Le complément est abaissé dès le début de la maladie.

Le traitement que vous allez proposer devant ce tableau clinique comporte :


A - Corticoïdes
B - Corticoïdes + immuno-dépresseurs
C - Corticoïdes + pénicilline G
D - Pénicilline G
E - Régime désodé
Bonne(s) réponse(s) : D E

Le régime dessodé est essentiel.


La pénicilline G est classique.
Les corticoïdes ne sont pas indiqués.
Les immunodépresseurs peuvent être éventuellement indiqués dans les formes à croissants diffus.

Il est de mauvais pronostic de constater 3 mois après le début de cette maladie :


A - Taux élevé des antistreptolysines
B - Hypocomplémentémie
C - Hypertension artérielle
D - Fonction rénale altérée
E - Syndrome oedémateux
Bonne(s) réponse(s) : B C D E

A - Pas de valeur pronostique.


B - Doit disparaitre avant la dixième semaine.
C D E - S'ils persistent après quelques semaines, peuvent être signes d'une évolution vers l'insuffisance rénale terminale avec
forme histologique grave (GNA à croissants diffus).

Dans une glomérulonéphrite aiguë bénigne, quelle(s) lésion(s) peut-on observer ?


A - Prolifération extra-capillaire
B - Prolifération endocapillaire
C - Présence de polynucléaires dans les anses capillaires
D - Présence de thrombus dans les anses capillaires
E - Présence de nodules hyalins périphériques
Bonne(s) réponse(s) : B C

A - La prolifération extracapillaire aboutit à la formation de croissants. Ces croissants sont de mauvais pronostics.
B C - Classiques.
D E - Sans rapport.

430
Exclusivement sur DOC - DZ : www.doc-dz.com NADJI 85
RESIDANAT EN POCHE TOME II
Cas Clinique en QCM
Un jeune garçon de 6 ans présente depuis 6 heures, après une contusion minime, de violentes douleurs du genou gauche. A
l'examen: fièvre à 39°C, tachycardie, douleur fémorale métaphysaire inférieure, circulaire, réveillée à la palpation ; mobilisation
douce du genou possible ; ongle incarné infecté du gros orteil gauche. Radiographie du genou de face et de profil : sans
particularité. N.F. : 15 000 globules blancs, dont 80% de neutrophiles.
V.S.: 60/80 C.R.P.: élevée.

Quel diagnostic évoquez-vous ?


A - Arthrite septique du genou
B - Ostéomyélite aiguë
C - Hémarthrose
D - Rhumatisme articulaire aigu
E - Décollement épiphysaire
Bonne(s) réponse(s) : B

La douleur est métaphysaire et la mobilisation douce du genou est possible, ce qui différencie l'ostéomyélite de l'arthrite.

Quel(s) autre(s) examen(s) doit(vent) être réalisé(s) immédiatement ?


A - Hémocultures
B - Scintigraphie
C - Prélèvement bactériologique au niveau du gros orteil
D - Examen tomodensitométrique
E - Ponction articulaire
Bonne(s) réponse(s) : A C

E - Non, ponction métaphysaire. Cependant, la ponction articulaire est en général effectuée avant la ponction métaphysaire
pour éliminer une arthrite associée et pour éviter la contamination de l'articulation si elle est effectuée en second.

Quel traitement appliquez-vous alors ?


A - Anti-inflammatoires non stéroïdiens en attendant le résultat des prélèvements
B - Antibiothérapie orale par un antibiotique antistaphylococcique
C - Antibiothérapie parentérale double associée
D - Abord chirurgical et trépanation osseuse
E - Immobilisation plâtrée
Bonne(s) réponse(s) : C E

Traitement classique.

Sur quel(s) élément(s) fonderez-vous la surveillance de l'efficacité du traitement ?


A - Fièvre
B - Syndrome inflammatoire biologique
C - Radiographie bi-hebdomadaire
D - Scintigraphies répétées
E - Taux des antistaphylolysines
Bonne(s) réponse(s) : A B C

QCM évident.

En cas d'évolution favorable, combien de temps sera maintenu le traitement ?


A - Jusqu'à l'apyrexie
B - Jusqu'à normalisation de la C. R. P.
C - Jusqu'à normalisation des images radiologiques
D - Six semaines
E - Six mois
Bonne(s) réponse(s) : D

C - Non, car les images radiographiques peuvent rester normales.


En général, 2 à 3 semaines en IV et 1 mois per os.

431
Exclusivement sur DOC - DZ : www.doc-dz.com NADJI 85
RESIDANAT EN POCHE TOME II
Cas Clinique en QCM
Vous êtes amené à voir un enfant de 8 ans. Les parents sont inquiets car en dépit d'un traitement par gentamicine
(Gentalline®) (40 mg matin et soir par voie IM prescrit depuis 4 jours), il reste fébrile. En effet, la température que vous
mesurez est à 39,5°C. L'enfant est abattu, se plaint d'une dysphagie douloureuse et de douleurs abdominales. Le poids est de
25 kg. Les amygdales érythémateuses et oedématiées sont recouvertes par un enduit que votre abaisse-langue enlève
aisément. Il existe un purpura du voile du palais. Vous palpez des ganglions sous-maxillaires sensibles. Il n'y a pas de coryza
ni de signes oculaires. Les tympans sont discrètement congestifs, le ventre sensible mais souple. L'auscultation du coeur
montre une tachycardie régulière à 130/mn, un souffle systolique mésocardiaque 1/6 disparaissant en position assise. Vous
évoquez une angine streptococcique.

Quel antibiotique préconisez-vous ?


A - Pénicilline V (Oracilline®)
B - Acide nalidixique (Négram®)
C - Cotrimoxazole (Bactrim®)
D - Doxycycline (Vibramycine®)
E - Cloxacilline (Orbénine®)
Bonne(s) réponse(s) : A

Evident.

Quelle est la cause de l'échec de la gentamicine (Gentalline®) ?


A - Mauvaise indication bactériologique
B - Durée insuffisante du traitement
C - Posologie insuffisante
D - Intervalle trop long entre les doses unitaires
E - Voie d'administration non adaptée
Bonne(s) réponse(s) : A

Les streptocoques résistent aux aminosides.

Quelle est la durée de traitement qui vous parait souhaitable ?


A - 4 jours
B - 7 à 10 jours
C - 12 à 15 jours
D - 15 à 20 jours
E - Un mois
Bonne(s) réponse(s) : B

Classique.

Parmi les examens suivants, lequel proposeriez-vous au moment où vous voyez l'enfant pour affirmer en toute
certitude la nature streptococcique ?
A - Mesure des ASLO
B - Hémoculture
C - Prélèvement de gorge
D - Hémogramme
E - Recherche d'une protéinurie
Bonne(s) réponse(s) : C

Présence d'un streptocoque du groupe A.


A - N'exclue pas une infection antérieure.

Dans le traitement des angines aiguës, la prescription d'amoxicilline est habituellement déconseillée parce que :
A - Un pourcentage croissant de streptocoques du groupe A sont devenus résistants
B - L'amoxicilline ne peut être prescrite en usage pédiatrique
C - L'amoxicilline favorise le développement de résistance par production de pénicillinases
D - L'amoxicilline est moins efficace que la pénicilline V pour éradiquer le streptocoque A dans la gorge
E - L'amoxicilline peut provoquer des éruptions cutanées en présence d'infections virales du pharynx (ex :
mononucléose infectieuse)
Bonne(s) réponse(s) : E

B - Très prescrit en pédiatrie.


C - Le streptocoque ne produit pas de pénicillinase.
E - Classique.

432
Exclusivement sur DOC - DZ : www.doc-dz.com NADJI 85
RESIDANAT EN POCHE TOME II
Cas Clinique en QCM
Un nouveau né naît à terme d'une mère présentant une fièvre à 39 degrés avec frissons au moment de l'accouchement.
L'enfant est totalement asymptomatique.

Quelle attitude adoptez-vous vis à vis de l'enfant ?


A - Traitement antibiotique si la C-Réactive Protéine est supérieure à 30 mg/l
B - Bilan infectieux avec hémocultures, ponction lombaire et antibiothérapie que si les résultats des cultures
bactériologiques sont positifs
C - Prélèvements bactériologiques et mono-antibiothérapie d'emblée
D - Prélèvement bactériologiques et administration de deux antibiotiques en association par voie parentérale
E - Surveillance clinique sans antibiothérapie
Bonne(s) réponse(s) : D

Le traitement ne se discute pas dans ce cas (élimine A et E) et sans attendre les résultats des prélèvements (élimine B).
Il s'agit toujours au moins d'une biantibiothérapie. Classiquement : Ampicilline et Aminéside.

Quels sont les trois germes le plus souvent responsables d'infections materno-foetales en France ?
A - E. Coli, Enterobacter cloacae, proteus mirabilis
B - Streptocoque du groupe A, E. Coli, listeria monocytogenes
C - Streptocoque D, E. Coli, Chlamydia trachomatis
D - Pneumocoque, hemophilus influenzae, méningocoque
E - Streptocoque du groupe B, E. Coli, Listeria monocytogenes
Bonne(s) réponse(s) : E

Classique.

L'interrogatoire apprend que la mère présente une infection urinaire à E. Coli non traitée. L'antibiogramme du
germe n'est pas encore réalisé. Quelle antibiothérapie parentérale vous parait la plus adaptée pour le nouveau-
né si celle-ci doit être débutée avant les résultats des prélèvements bactériologiques de l'enfant ?
A - Ofloxacine - aminoside
B - Ampicilline - aminoside
C - Céfotaxime - aminoside
D - Pénicilline G - aminoside
E - Vancomycine - aminoside
Bonne(s) réponse(s) : C

Très discutable car en fait, il faudrait répondre Céfotaxine-Ampicilline-Aminoside. Dans le but d'atteindre l'E Coli, (Céfotaxine)
et le listeria (Ampicilline). Cependant, les autres items sont absurdes !

Une ponction lombaire est réalisée dans le cadre du bilan bactériologique. Quel(s) résultat(s) est (sont)
pathologique(s) ?
A - Protéinorachie : 0,60 g/l
B - Glycorachie : 0,25 g/l, avec glycémie contemporaine = 0,50 g/l
C - 15 éléments/mm3
D - Glycorachie : 0,25 g/l, avec glycémie contemporaine = 1 g/l
E - Protéinorachie : 1,5 g/l
Bonne(s) réponse(s) : D E

Chez le nouveau-né à terme :


- Protéinorachie normale < ou= 1,2 g/l
- Glychorachie normale > ou = moitié de la glycémie.
- Eléments tolérés jusqu'à 30/mm3.

433
Exclusivement sur DOC - DZ : www.doc-dz.com NADJI 85
RESIDANAT EN POCHE TOME II
Cas Clinique en QCM
Un enfant de 5 ans est amené par sa mère en consultation pour des troubles nocturnes ayant débuté un an auparavant. Les
épisodes se déroulent de la façon suivante : alors que l'enfant s'est couché sans difficulté vers 20 heures, il réveille
brutalement ses parents 3 heures plus tard, par des cris agités, des hurlements. Ceux-ci se rendent au chevet de l'enfant. Il
semble terrifié, ne reconnaissant pas son entourage qui ne peut le rassurer. Il est en sueurs. L'épisode dure environ 5 minutes
puis l'enfant se rendort, abattu, sans aucun souvenir le lendemain matin de ce qui s'est passé. Ces épisodes surviennent
environ 1 fois par mois.

Quel est votre diagnostic ?


A - Epilepsie temporale
B - Cauchemars
C - Terreurs nocturnes
D - Illusions hypnagogiques
E - Pathomimie
Bonne(s) réponse(s) : C

- Tableau typique.
- Dans les cauchemars, il existe rarement une amnésie.

Ces troubles surviennent :


A - Au stade 2 du sommeil lent
B - Au stade 3
C - Au stade 4
D - Lors du sommeil paradoxal
E - Aux stades 3 et 4
Bonne(s) réponse(s) : C

Ondes lentes monomorphes au cours du stade 4 du sommeil lent !

Quelle est la conduite à tenir dans l'immédiat ?


A - Faire un électroencéphalogramme
B - Bilan psychométrique
C - Prescription d'une benzodiazépine
D - Rassurer et éduquer la famille
E - Prescription d'imipramine
Bonne(s) réponse(s) : D

Le caractère peu fréquent des crises rend la réponse D la plus raisonnable.

Quels sont les symptômes qui orientent le diagnostic ?


A - Tachycardie
B - Dilatation pupillaire
C - Illusions hypnopompiques
D - Sueurs
E - Polypnée
Bonne(s) réponse(s) : A D E

C - Il s'agit d'une conduite hallucinatoire.


On peut trouver également une pâleur.

Vous êtes amené à voir à votre consultation un enfant de 8 ans. Les parents sont inquiets : en dépit d'un traitement par
gentamicine (Gentalline®) en intramusculaire prescrit depuis 4 jours par un confrère antérieurement consulté, il reste fébrile.
En effet la température est à 39,5°C, l'enfant est abattu, se plaint d'une dysphagie douloureuse et de douleurs abdominales.
Les amygdales rouges et oedématiées sont recouvertes par un enduit que votre abaisse-langue enlève aisément. Il existe un
purpura du voile du palais. Vous palpez des ganglions sous-maxillaires sensibles. Il n'y a ni coryza ni signes oculaires. Les
tympans sont discrètement congestifs, le ventre est sensible mais souple. Vous évoquez une angine streptococcique.

Les streptocoques du groupe A :


A - Se présentent comme des cocci à Gram positif isolés ou disposés en chaînettes
B - Possèdent une protéine de structure (protéine M) permettant de différencier plus de 50 sérotypes
C - Peuvent produire une toxine érythrogène responsable de l'éruption de la scarlatine
D - Sont habituellement résistants aux antibiotiques de la famille des bêta-lactamines
E - Sont responsables de l'érysipèle
Bonne(s) réponse(s) : A B C E

D - Non, c'est le traitement.

434
Exclusivement sur DOC - DZ : www.doc-dz.com NADJI 85
RESIDANAT EN POCHE TOME II
Cas Clinique en QCM

Devant ce tableau clinique, quel antibiotique préconisez-vous ?


A - Pénicilline G
B - Acide nalidixique (Negram®)
C - Cotrimoxazole (Bactrim®)
D - Doxycycline (Vibramycine®)
E - Cefalotine (Keflin®)
Bonne(s) réponse(s) : A

Evident.

Quelle durée de traitement vous parait souhaitable ?


A - 4-6 jours
B - 8-10 jours
C - 12-15 jours
D - 15-20 jours
E - Plus de 20 jours
Bonne(s) réponse(s) : B

Sans commentaire.

Quelle est la cause de l'échec de la gentamicine (Gentalline®) ?


A - Mauvaise indication bactériologique
B - Durée insuffisante du traitement
C - Résistance bactérienne par mutation chromosomique
D - Résistance bactérienne d'origine plasmidique
E - Voie d'administration non adaptée
Bonne(s) réponse(s) : A

Le streptocoque résiste aux Aminosides.

Parmi les examens suivants, lequel proposeriez-vous au moment où vous voyez l'enfant pour affirmer en toute
certitude la nature streptococcique de I angine ?
A - Dosage des ASLO
B - Hémoculture
C - Prélèvement de gorge
D - NFS
E - Recherche d'une protéinurie
Bonne(s) réponse(s) : C

Evident.

Vous revoyez l'enfant 15 jours plus tard; parmi les examens suivants, lequel proposeriez-vous pour affirmer la
nature streptococcique de cette angine ?
A - Dosage des antistreptolysines O (ASLO)
B - Hémoculture
C - Prélèvement de gorge
D - NFS
E - Recherche d'une protéinurie
Bonne(s) réponse(s) : A

Il faut une ascension des taux des ASLO à 15 jours d'intervalle.


E - A ne pas oublier mais n'affirme pas la nature streptococcique.

435
Exclusivement sur DOC - DZ : www.doc-dz.com NADJI 85
RESIDANAT EN POCHE TOME II
Cas Clinique en QCM
Un garçon de 7 ans est amené par ses parents pour une boiterie douloureuse de la hanche gauche, d'apparition récente.
L'interrogatoire ne retrouve pas d'antécédent particulier. A l'examen, la hanche est douloureuse à la mobilisation ; la flexion,
l'abduction et les rotations sont limitées.
On constate une mayotrophie quadricipitale homolatérale. L'enfant est apyrétique.
Examens biologiques : N.F. : Normale ; V S : 8/17 ; C R P : normale.
Examen radiologique : sur la radiographie du bassin de face : aspect de condensation et d'aplatissement polaire supérieur du
noyau céphalique gauche.

Quel diagnostic évoquez-vous ?


A - Une épiphysiolyse
B - Un "rhume de hanche"
C - Une ostéochondrite de hanche
D - Une coxalgie
E - Une ostéoarthrite
Bonne(s) réponse(s) : C

Tableau typique.
A - Sujet plus âgé.
B - Radiographie normale.
D - Sans rapport.
E - Il y aurait un syndrome inflammatoire.

Quel(s) examen(s) paraclinique(s) est (sont) indispensable(s) pour confirmer le diagnostic ?


A - Scintigraphie au Gallium
B - Scintigraphie au Technétium
C - Imagerie par résonance
D - Arthrographie de hanche
E - Aucun des examens précédents
Bonne(s) réponse(s) : E

La radiographie suffit amplement dans ce cas.

La scintigraphie, si elle est réalisée, mettrait en évidence :


A - Une hyperfixation diffuse de la hanche gauche
B - Une hyperfixation très localisée
C - Une hypofixation
D - Un pincement de l'interligne coxo-fémoral
E - Aucune anomalie
Bonne(s) réponse(s) : C

Il s'agit d'une ischémie donc hypofixation par hypovascularisation.

Quel traitement peut-on proposer d'emblée ?


A - Ostéotomie pelvienne
B - Mise en traction
C - Corticothérapie
D - Ostéotomie fémorale
E - Aucun traitement
Bonne(s) réponse(s) : B

Le but du traitement est de recentrer la tête fémorale dans la cavité pour assurer sa couverture jusqu'à la période de
reconstruction. L'ostéotomie n'est proposée que dans les formes graves.

Quelle sera la durée probable d'évolution des lésions anatomiques en dehors des séquelles ?
A - Une quinzaine de jours
B - Un mois
C - Six mois
D - Deux ans
E - Dix ans
Bonne(s) réponse(s) : D

L'évolution se fait sur 18 mois à 4 ans.

436
Exclusivement sur DOC - DZ : www.doc-dz.com NADJI 85
RESIDANAT EN POCHE TOME II
Cas Clinique en QCM
Un garçon de 5 ans, sans antécédents, présente subitement des vomissements. Le médecin appelé découvre lors de
l'examen une volumineuse masse du flanc et de la fosse iliaque droite, donnant le contact lombaire.
On ne retrouve pas de notion d'hématurie ni d'infection urinaire, ni a fortiori d'infection urinaire fébrile. L'enfant est pâle,
asthénique, grognon. il est immédiatement hospitalisé dans un service de pédiatrie.
Le premier examen réalisé est une échographie abdominale qui confirme le caractère solide de la masse perçue cliniquement.
L'ECBU (cyto bactério urinaire) est normal.

A ce stade, quel(s) diagnostic(s) peut-on déjà évoquer ?


A - Hydronéphrose droite
B - Pyélonéphrite xanthogranulomateuse
C - Tumeur de Wilms (néphroblastome)
D - Sympathoblastome
E - Polykystose de type infantile
Bonne(s) réponse(s) : C D

A - Non, car masse solide à l'échographie.


B - Pas d'antécédent d'infection urinaire.
E - Serait vue à l'échographie.

Quel(s) examen(s) faut-il alors demander ?


A - Radio du thorax
B - Urographie intra-veineuse
C - Scanner abdominal
D - Dosage de catécholamines urinaires
E - Scintigraphie isotopique rénale
Bonne(s) réponse(s) : A B C D E

A - Recherche de méta pulmonaire.


B C - Evident.
D - Pour éliminer un sympathoblastome sécrétant.
E - Discutable, se fait classiquement.

Les examens précédents permettent de confirmer l'origine rénale de la tumeur, quel(s) autre(s) élément(s) faut-
il s'efforcer de préciser ?
A - Intégrité du rein contro-latéral
B - Existence d'adénopathies
C - Intégrité de la vessie
D - Etat de la veine rénale droite
E - Etat de la veine cave inférieure
Bonne(s) réponse(s) : A B D E

A B - Evident.
C - Sans rapport.
D E - Très utile pour le chirurgien.

A priori, quel est le traitement qui sera proposé à cet enfant ?


A - Radiothérapie
B - Chimiothérapie
C - Néphrectomie élargie droite
D - Radiothérapie et néphrectomie élargie
E - Chimiothérapie et néphrectomie élargie
Bonne(s) réponse(s) : E

Question de cours.

Quel est actuellement le pronostic de survie à 5 ans des néphroblastomes sans métastase initiale ?
A - Inférieur à 10%
B - Environ 25 %
C - Environ 50 %
D - Supérieur à 80 %
E - Imprévisible
Bonne(s) réponse(s) : D

65 % si métastase initiale.

437
Exclusivement sur DOC - DZ : www.doc-dz.com NADJI 85
RESIDANAT EN POCHE TOME II
Cas Clinique en QCM
Monsieur M. 35 ans, sans antécédent pathologique, est amené par son épouse en consultation pour un état d'agitation
d'installation brutale..Le malade parle sans cesse ; son discours est intarissable, ses idées s'enchaînent sans lien logique et
les "coq à l'âne" sont nombreux. Il se prétend un génial inventeur promis à un grand avenir scientifique, à la gloire et à la
richesse. Le malade est d'emblée familier avec son examinateur et répond de façon amusée ou ironique à ses questions. Il
nie tout caractère pathologique à son comportement. Son épouse apprend au médecin qu'il ne dort plus depuis trois jours et
qu'il fait de nombreuses avances sur un mode provocant et obscène qui contrastent avec son caractère timide et réservé. Il
n'a pas pris de substances toxiques et on ne note pas d'antécédent éthylique.

Devant un tel épisode d'agitation, quel est le diagnostic qui vous parait le plus probable ?
A - Agitation paranoïde
B - Agitation hystérique
C - Agitation maniaque
D - Agitation psychopathique
E - Agitation schizophrénique
Bonne(s) réponse(s) :

QUESTION ANNULEE

Parmi les symptômes suivants, quel est celui qui est présent dans cette observation ?
A - Fuite des idées
B - Théâtralisme
C - Mythomanie
D - Discordance
E - Maniérisme
Bonne(s) réponse(s) :

QUESTION ANNULEE

Un tel tableau clinique peut faire évoquer aussi une affection organique.
Quelle est parmi les affections suivantes celle qui pourrait entraîner un tel tableau ?
A - Tumeur frontale
B - Maladie d'Alzheimer
C - Insuffisance surrénale
D - Maladie de Creutzfeldt-Jacob
E - Paralysie générale
Bonne(s) réponse(s) :

QUESTION ANNULEE

Quelle est parmi les chimiothérapies suivantes, celle qui doit être utilisée devant un tel état ?
A - Un antidépresseur sédatif par exemple l'Amitriptyline (Laroxyl®)
B - Une benzodiazépine par exemple le Diazepam (Valium®)
C - Un barbiturique par exemple le Phénobarbital (Gardenal®)
D - Une butyrophénome par exemple l'Halopéridol (Haldol®)
E - Un tranquillisant comme le méprobamate (Equanil®)
Bonne(s) réponse(s) :

QUESTION ANNULEE

En cas de refus du malade de se faire soigner, le médecin peut envisager une mesure de placement volontaire.
Parmi les propositions suivantes, quelle(s) est(sont) celle(s) conformes à la loi ?
A - Le certificat du médecin demandant le placement doit être accompagné d'une demande de placement
faite par une personne de l'entourage
B - Le certificat peut être fait par le médecin s'il n'est pas spécialiste
C - Le certificat doit nécessairement comporter le diagnostic des troubles présentés par Monsieur M.
D - La réalisation du placement incombe aux services de police ou de gendarmerie
E - Le placement doit se faire dans un établissement régi par la loi du 30 Juin 1838
Bonne(s) réponse(s) :

QUESTION ANNULEE

438
Exclusivement sur DOC - DZ : www.doc-dz.com NADJI 85
RESIDANAT EN POCHE TOME II
Cas Clinique en QCM
Un enfant de 9 ans présente une crise d'asthme typique qui a débuté 12 h auparavant. C'est la 3ème crise en 15 mois.
Quel est votre avis sur les différentes composantes du traitement proposé ?

Concernant l'administration de bêta 2 stimulant (Salbutamol®), quelle est (quelles sont) la (les) réponse(s)
exacte(s) ?
A - Contre-indiquée chez l'enfant
B - S'utilise de préférence en sirop à cet âge
C - S'utilise en inhalation
D - L'utilisation en inhalation est améliorée par l'usage d'une chambre d'expansion
E - Est actif en quelques minutes
Bonne(s) réponse(s) : C D E

A 9 ans, un enfant peut inhaler (C) et encore plus facilement avec un nebuhaler (D).

Concernant la Théophylline, quelle est ou quelles sont la (les) réponse(s) exacte(s) ?


A - Est contre-indiquée à cet âge
B - S'utilise de préférence en suppositoires
C - S'utilise per os
D - A la dose de charge de 6 mg/kg
E - A la dose d'entretien de 3 mg/kg/jour
Bonne(s) réponse(s) : C D E

B - Non, car absorption aléatoire.


D - Oui, si l'enfant n'est pas déjà sous Théophylline.
E - Non, 12 à 15 mg/kg/j.

Après échec du bêta 2 stimulant et de la Théophylline, l'utilisation de corticoïdes est envisagée :


A - Du fait de leur court délai d'action (moins de 5 minutes)
B - En dose d'attaque de l'ordre de 1 à 3 mg/kg/jour d'équivalent en delta-cortisone (Cortancyl®)
C - Per os ou par voie parentérale
D - Par inhalation exclusive
E - A posologie très dégressive, le traitement devant s'étaler sur une quinzaine de jours
Bonne(s) réponse(s) :

QUESTION ANNULEE.

Ultérieurement les crises d'asthme récidivent de façon fréquente, deux fois par mois. Quel est votre conduite ?
A - Rechercher un terrain allergique
B - Faire un autocontrôle régulier quotidien par mesure du débit de pointe
C - S'assurer qu'il n'y a pas de signe obstructif pendant l'intervalle libre entre les crises
D - Peut justifier un traitement au long cours par la Théophylline retard
E - Rassurer car l'asthme guérit à la puberté
Bonne(s) réponse(s) : A B C D

A C - Sont évidents.
B - Est discutable du fait du mot "quotidien".
D - Il s'agit du traitement.
E - Eviter de rassurer inutilement.

Un nourrisson de 4 mois,jusque-là en bonne santé est malade depuis 24 heures. La mère signale qu'il est apathique, geignard
et a vomi. Pourtant, il n'apparaissait qu'un peu enrhumé. Fait nouveau motivant la consultation, il a convulsé et la maman,
inquiète, a pris la température qui était à 39°C.
Le médecin qui examine l'enfant note une hypotonie, une fontanelle bombante même en dehors du cri, et l'absence de foyer
infectieux évident.

Parmi les 5 examens complémentaires suivants, quel est celui qui est le plus urgent à pratiquer ?
A - Numération formule sanguine
B - Hémoculture
C - Ponction lombaire
D - Examen du fond d'oeil
E - Examen cyto-bactériologique des urines
Bonne(s) réponse(s) : C

Tableau typique de méningite.

439
Exclusivement sur DOC - DZ : www.doc-dz.com NADJI 85
RESIDANAT EN POCHE TOME II
Cas Clinique en QCM

Parmi les germes suivants, lequel(lesquels) est (sont) habituellement trouvé(s) dans cette infection ?
A - Colibacille
B - Hémophilus influenzae
C - Mycoplasma pneumoniae
D - Méningocoque
E - Pseudomonas aeruginosa
Bonne(s) réponse(s) : B D

Le troisième classique est le pneumocoque.

Parmi les 5 familles d'antibiotiques suivantes, laquelle est utilisable pour un traitement en monothérapie par voie
générale à commencer de suite avant le résultat de l'antibiogramme ?
A - Aminosides
B - Ampicillines
C - Macrolides
D - Phénicoles
E - Céphalosporines de 3ème génération
Bonne(s) réponse(s) : E

Classique car actif sur les trois germes, contrairement à B qui n'agit pas sur les haemophilus sécréteur de bêta-lactamase.

La guérison ayant été obtenue facilement, l'enfant est revu en consultation deux mois plus tard. Lequel parmi
les examens suivants vous parait essentiel :
A - Etude de l'audition (auditest)
B - Examen cyto-bactériologique urinaire
C - Numération formule sanguine
D - Mesure du périmètre crânien au ruban-mètre
E - Ponction lombaire
Bonne(s) réponse(s) : A

Le seul discutable est D à la recherche d'une hydrocéphalie séquellaire mais il faut rester classique.

Un enfant de 8 ans, est amené en consultation. A L'examen, on observe un érythème suintant par endroits, aux plis de coudes
et sur le thorax. Les lésions ont débuté à l'âge de 5 mois, elles sont très prurigineuses, gênant souvent le sommeil. Une tante
maternelle a une rhinite saisonnière, la petite soeur a également des lésions cutanées, la mère a une peau très sèche,
formant de petites écailles sur la face antérieure des jambes.

La localisation aux plis des coudes est habituelle dans cette pathologie.
Quelle est l'autre localisation fréquente à cet âge ?
A - Creux poplités
B - Visage
C - Creux axillaires
D - Thorax
E - Fesses
Bonne(s) réponse(s) : A

A 2 ans, inversion des localisations préférentielles aux convexités en faveur des plis de flexion.

La (les) manifestation(s) suivante(s) est (sont) habituelle(s) au cours de cette pathologie :


A - Infection cutanée
B - Fissuration du pli inférieur de l'oreille
C - Dépressions punctiformes des ongles
D - Sécheresse de la peau
E - Chute des cheveux
Bonne(s) réponse(s) : B D

C - Ne pas confondre avec le pouce sucé.

440
Exclusivement sur DOC - DZ : www.doc-dz.com NADJI 85
RESIDANAT EN POCHE TOME II
Cas Clinique en QCM

L'interrogatoire permet souvent de trouver des antécédents familiaux de :


A - Psoriasis
B - Asthme
C - Furoncles
D - Hyperthyroïdie
E - Hypertension artérielle
Bonne(s) réponse(s) : B

Evident.

La mère a une peau sèche avec formation de petites écailles à la partie antérieure des jambes. Cet état cutané
héréditaire est souvent associé à cette pathologie, il s'agit de :
A - Psoriasis
B - Eczéma de contact
C - Parapsoriasis
D - Pityriasis rosé
E - Ichtyose
Bonne(s) réponse(s) : E

Sans commentaire.

François, 9 ans, sans antécédents notables, présente depuis quelques jours une éruption d'allure purpurique avec des
douleurs abdominales diffuses et des douleurs des grosses articulations.
Le médecin traitant l'adresse dans l'hypothèse d'un purpura rhumatoïde.
Durant l'hospitalisation, les douleurs abdominales persistent.
Biologiquement : hémogramme normal, si ce n'est une discrète polynucléose; ionogramme normal ; créatinine à 58 micromol/l
; compte d'Addis normal ; protéinurie de 24 h = 40 mg.

Le diagnostic positif de purpura rhumatoïde repose sur :


A - Les données cliniques
B - L'existence d'une thrombopénie
C - L'élévation possible des IgA sériques
D - L'abaissement du complément (C3, CH50)
E - La présence du facteur rhumatoïde
Bonne(s) réponse(s) : A C

A - Triade : arthralgies, douleurs abdominaux et purpura.


B - Certainement pas.
C - Classique, mais discutable car le diagnostic positif est en général clinique.
D - Le complément est normal.
E - Piège énorme.

L'atteinte articulaire du purpura rhumatoïde :


A - Concerne les grosses articulations
B - Ne laisse jamais de séquelles
C - A souvent la même topographie que les lésions purpuriques
D - N'est que rarement retrouvée
E - Justifie parfois la corticothérapie
Bonne(s) réponse(s) : B C

A - La cheville n'est pas une grosse articulation.


D - Au contraire, très classique.
E - Classiquement non.

Les manifestations abdominales du purpura rhumatoïde :


A - Sont parfois inaugurales
B - Peuvent comporter des hémorragies intestinales
C - Sont en rapport avec de petits hématomes pariétaux de l'intestin
D - Nécessitent parfois la chirurgie
E - Justifient parfois la corticothérapie
Bonne(s) réponse(s) : A B C D

A B C - Classique.
D - Oui, invagination par exemple.
E - Classiquement non.

441
Exclusivement sur DOC - DZ : www.doc-dz.com NADJI 85
RESIDANAT EN POCHE TOME II
Cas Clinique en QCM

L'atteinte rénale du purpura rhumatoïde :


A - Peut comporter un syndrome néphrotique
B - Survient généralement précocement
C - Correspond à une glomérulonéphrite segmentaire et focale
D - Est caractérisée par des dépôts mésangiaux d'lg A
E - Ne laisse pas de séquelles
Bonne(s) réponse(s) : A D

B - Survient n'importe quand, et justifie une surveillance prolongée.


C - Pas uniquement.
E - Faux en cas de prolifération extracapillaire.

Le traitement du purpura rhumatoïde :


A - Peut comporter le repos au lit
B - Justifie un régime du fait du risque d'hypertension artérielle
C - Fait appel à la corticothérapie dans certains cas
D - Nécessite une antibiothérapie de couverture
E - Peut justifier une nutrition parentérale exclusive
Bonne(s) réponse(s) : A C

C - Oui, en cas d'atteinte testiculaire.


B D E - Sans rapport.

Un jeune enfant de 21 mois ayant dans ses antécédents une rougeole à 1 an et des otites à répétition, présente brutalement
un épisode convulsif avec hyperthermie, vomissements et hypertonie. Le fond d'oeil est normal. L'E.E.G. montre une
souffrance cérébrale diffuse à prédominance temporale gauche. La numération formule sanguine objective une
hyperleucocytose à 10 600/mm3 avec 37% de polynucléaires et 50% de Iymphocytes. La V.S. est de 25 mm à la 1ère heure.
L'enfant est hospitalisé pour bilan étiologique et traitement.

Parmi les examens complémentaires suivants, indiquez quel est le plus urgent à réaliser :
A - Hémoculture
B - Scanner cérébral
C - Ponction lombaire
D - Angiographie cérébrale
E - Scintigraphie cérébrale
Bonne(s) réponse(s) :

QUESTION ANNULEE

Le diagnostic de méningo-encéphalite étant évoqué, quel sont les deux microorganismes les plus fréquemment
responsables ?
A - Méningocoque
B - Virus
C - Bacille de Koch
D - Borrelia burgdorferi
E - Escherichia coli
Bonne(s) réponse(s) :

QUESTION ANNULEE

Parmi les virus suivants lequel ou lesquels est ou sont habituellement responsables de telles
méningoencéphalites ?
A - Herpès Simplex
B - Morbillivirus
C - Myxovirus influenzae
D - Herpès zoster varicellae
E - Adénovirus
Bonne(s) réponse(s) :

QUESTION ANNULEE

442
Exclusivement sur DOC - DZ : www.doc-dz.com NADJI 85
RESIDANAT EN POCHE TOME II
Cas Clinique en QCM

Dans l'hypothèse d'une méningoencéphalite herpétique quel est ou quels sont l'(es) examen(s)
complémentaire(s) décisif(s) qui vous apporte(nt) en 48 heures des arguments positifs ?
A - Les données de l'E.E.G
B - Un sérodiagnostic herpétique par ELISA
C - Un scanner cérébral
D - Le dosage d'interféron dans le sang et le L.C.R.
E - Une recherche virale par culture dans le L.C.R.
Bonne(s) réponse(s) :

QUESTION ANNULEE

Quelle est (quelles sont) la ou les attitude(s) thérapeutique(s) indiquée{s) dans cette dernière hypothèse ?
A - Corticothérapie
B - Restriction hydrique modérée
C - Acyclovir (Zovirax)
D - Azidohymédine
E - Immunoglobulines IV
Bonne(s) réponse(s) :

QUESTION ANNULEE

Denis est examiné à 18 mois pour suspicion de surdité : à l'âge de 10 mois sa grand-mère a remarqué son comportement
d'isolement, le fait qu'il ne répond pas à son nom et n'émet que très peu de sons. L'entretien avec les parents et l'examen de
l'enfant apprennent que le langage (pré-langage) est très peu développé. Le père est cadre, la mère technicienne. Denis est
leur premier enfant. La mère a repris le travail quand l'enfant avait 5 mois. Les parents et la nourrice l'ont toujours trouvé un
peu en retard mais aussi "bizarre", et surtout très isolé : c'est un enfant facile, très peu exigeant, qui peut rester longtemps
silencieux dans son coin. Il regarde rarement ; il a des habitudes étranges : jeux avec des objets inhabituels, jeux de main
devant les yeux, ou bien se frotte les oreilles, etc...

L'ensemble du tableau décrit ci-dessus :


A - Peut être dû à une surdité simple
B - Permet d'exclure un déficit sensoriel
C - Evoque un trouble global du développement (autisme..)
D - Evoque un retard intellectuel simple
E - Correspond à un comportement normal pour l'âge
Bonne(s) réponse(s) : C

Sans commentaire.

Parmi les propositions suivantes la ou lesquelles vous parait(ssent) raisonnable(s) ?


A - Un avis spécialisé (pédopsychiatre)
B - Rassurer et tenter de lever l'angoisse de la famille
C - Proposer d'attendre l'entrée à l'école maternelle pour revoir l'enfant avec ses parents
D - Proposer un traitement anti-dépresseur pour l'enfant
E - Proposer une période (quelques semaines) de séparation du milieu familial
Bonne(s) réponse(s) : A E

Sans commentaire.

Parmi les signes énoncés dans le texte quels sont ceux qui peuvent être des arguments en faveur d'un
syndrome autistique ?
A - Comportement d'isolement
B - Reprise du travail de la mère quand l'enfant avait 5 mois
C - Denis est le premier enfant
D - Il regarde rarement
E - Jeux avec des objets inhabituels, jeux de main devant les yeux
Bonne(s) réponse(s) : A D E

Sans commentaire.

443
Exclusivement sur DOC - DZ : www.doc-dz.com NADJI 85
RESIDANAT EN POCHE TOME II
Cas Clinique en QCM

Parmi les signes énoncés dans le texte quels sont ceux qui peuvent faire évoquer un déficit auditif ?
A - Le fait qu'il ne répond pas à son nom
B - Le langage (pré-langage) est très peu développé
C - Il regarde rarement
D - Les parents et la nourrice l'ont toujours trouvé un peu en retard mais aussi "bizarre", et surtout très isolé
E - C'est un enfant facile, très peu exigeant
Bonne(s) réponse(s) : A B D

Sans commentaire.

Un garçon de 2 ans et demi est amené à la consultation du pédiatre car ses parents le trouvent "petit". C'est un enfant sans
antécédents particuliers, né à terme avec un poids de naissance de 2 500 g et une taille de naissance de 49 cm. Cet enfant a
été nourri au lait maternisé dès la naissance et le Stérogyl a été donné régulièrement jusqu'à l'âge de 2 ans.
A l'examen :
poids : 10 kilos
taille : 82 cm
PC : 48 cm.
L'état trophique de l'enfant est bon, les masses musculaires et le pannicule adipeux sont normaux. L'auscultation cardio-
pulmonaire est normale. L'abdomen est souple, sans hépatosplénomégalie. Le développement moteur et intellectuel de
l'enfant est normal pour l'âge.

Parmi les données que l'on peut recueillir sur le carnet de santé de l'enfant, laquelle vous parait la plus utile
pour rechercher la cause de cette "petite taille" ?
A - Courbe du périmètre crânien
B - Courbes de poids et de taille
C - Feuille d'examen du 24ème mois
D - Données de la période néonatale
E - Relevé des vaccinations
Bonne(s) réponse(s) : B

Sans commentaire.

On note en interrogeant la famille que le papa mesure 1m70 pour un poids de 63 kg et que la maman mesure
1m42 pour un poids de 45 kg et qu'à 18 mois l'enfant mesurait 74 cm et pesait 8,100 kg.
Quel diagnostic vous parait le plus probable ?
A - Nanisme psychosocial
B - Hyperthyroïdie congénitale
C - Petite taille idiopathique familiale
D - Intolérance au gluten
E - Déficit en STH
Bonne(s) réponse(s) : C

Sans commentaire.

Parmi les propositions suivantes, laquelle (ou lesquelles) vous parai(ssen)t nécessaire(s) à faire d'emblée ?
A - Faire une étude radiologique de l'âge osseux
B - Faire un dosage d'hormones thyroïdiennes
C - Faire un dosage de STH sous stimulation
D - Faire une électrophorèse des protides
E - Demander une biopsie jéjunale
Bonne(s) réponse(s) :

QUESTION ANNULEE.

Parmi les propositions suivantes se rapportant à cette observation, lesquelles vous paraissent exactes ?
A - Cet enfant vit probablement dans un contexte psychoaffectif perturbé
B - La vitesse de croissance de cet enfant est ralentie
C - A l'âge adulte, la taille de cet enfant sera probablement entre - 2 et - 3 DS
D - La vitesse de croissance de cet enfant est normale
E - Cet enfant fera probablement une puberté tardive
Bonne(s) réponse(s) :

QUESTION ANNULEE.

444
Exclusivement sur DOC - DZ : www.doc-dz.com NADJI 85
RESIDANAT EN POCHE TOME II
Cas Clinique en QCM

Parmi les propositions suivantes concernant la croissance, laquelle (ou lesquelles) est (sont) exacte(s) ?
A - A la naissance, un enfant à terme mesure environ 50 cm, pèse environ 3 kg et a un PC d'environ 38 cm
B - A quatre mois, il a à peu près doublé son poids de naissance
C - A un an, il mesure environ 60 cm
D - A 4 ans, il mesure environ 1 m
E - En dehors de là, avant la puberté, la période de vie où la croissance en taille est la plus rapide est entre 0
et 4 ans
Bonne(s) réponse(s) : B D E

A - Non, PC = 35 cm.
C - Non, 75 cm.

Laquelle (ou lesquelles) des affections suivantes est (sont) susceptible(s) de s'accompagner d'un retard de
croissance staturale ?
A - Syndrome de Turner
B - Sphérocytose héréditaire
C - Maladie de Crohn
D - Maladie de Klinefelter
E - Hypothyroïdie congénitale
Bonne(s) réponse(s) : A B C E

Classique.

Laquelle (ou lesquelles) des propositions suivantes concernant la puberté est (sont) exacte(s) ?
A - Elle débute chez la fille par un développement des seins
B - Elle débute chez la fille par l'apparition des règles
C - Elle débute chez le garçon par l'augmentation de volume des testicules
D - Elle débute chez le garçon par la pilosité pubienne
E - Elle débute chez le garçon par la modification de la voix
Bonne(s) réponse(s) : A C

Sans commentaire.

Un nourrisson âgé de 6 mois présente depuis 12h des crises douloureuses abdominales paroxystiques avec des accès de
pâleur. L'examen clinique, retrouve une infection ORL avec un tympan gauche un peu congestif. La température est à 37°8.
L'enfant présente des selles diarrhéiques lors de votre consultation, l'abdomen est souple, la fosse iliaque droite parait
déshabitée .

Quel(s) diagnostic(s) suspectez-vous ?


A - Une gastro-entérite
B - Une pneumopathie aiguë
C - Une hydronéphrose droite
D - Une infection urinaire
E - Une invagination intestinale aiguë
Bonne(s) réponse(s) : A E

Tableau d'invagination typique mais la présence de selles liquides qui ne doit en rien éliminer l'invagination doit aussi faire
suspecter une gastroentérite.

Quel examen para-clinique demandez-vous en urgence ?


A - Un ECBU (Examen cyto-bactério des urines)
B - U ne copro-culture
C - Un ASP (Abdomen sans préparation)
D - Un cliché thorax
E - Une UIV
Bonne(s) réponse(s) : C

Premier temps du lavement aux hydrosolubles.

445
Exclusivement sur DOC - DZ : www.doc-dz.com NADJI 85
RESIDANAT EN POCHE TOME II
Cas Clinique en QCM

Si l'enfant était vu après plus de 48h d'évolution, avec une deshydratation franche, un état fébrile, des
vomissements bilieux, des rectorragies franches et une altération de l'état général, lequel de ces gestes ou
examens serait contre-indiqué ?
A - L'ASP debout
B - Le lavement opaque
C - L'échographie
D - Le toucher rectal
E - L'aspiration gastrique
Bonne(s) réponse(s) : B

Risque de perforation digestive.

Quelle est l'étiologie responsable le plus souvent de ce tableau clinique chez le nourrisson ?
A - Les infections virales à rotavirus
B - Un diverticule de Meckel
C - Les infections à yersinia
D - Un purpura rhumatoïde
E - Aucune étiologie précise, il s'agit d'une forme primitive chez le nourrisson
Bonne(s) réponse(s) : E

Classique.

Quel traitement proposeriez-vous ?


A - Un traitement médical anti-bactérien
B - Un traitement chirurgical dans tous les cas
C - Un traitement par lavement aux hydrosolubles isolé sans geste chirurgical dans tous les cas
D - Un traitement par lavement aux hydrosolubles avec chirurgie en cas d'échec
E - Un traitement médical anti-infectieux avec nutrition entérale à débit constant
Bonne(s) réponse(s) : D

Il s'agit de l'attitude préconisée actuellement. Les critères de la désinvagination lors du lavement baryté doivent tous être
présents pour qu'on se passe de la chirurgie.

Si on décidait d'opérer cet enfant, quelle serait la principale complication à redouter ?


A - Lithiase biliaire
B - Volvulus sur bride
C - Une éventration
D - Embolie pulmonaire
E - Récidive du tableau initial
Bonne(s) réponse(s) :

QUESTION ANNULEE.

446
Exclusivement sur DOC - DZ : www.doc-dz.com NADJI 85
RESIDANAT EN POCHE TOME II
Cas Clinique en QCM

447
Exclusivement sur DOC - DZ : www.doc-dz.com NADJI 85
RESIDANAT EN POCHE TOME II
Cas Clinique en QCM
Une femme de 36 ans est atteinte d'un rétrécissement mitral pur diagnostiqué à l'âge de 22 ans. En dehors
d'un rhumatisme articulaire à l'âge de 6 ans, elle n'a aucun antécédent particulier. Elle ne fume pas. Elle a
mené à bien deux grossesses à l'âge de 27 et 29 ans. Elle est, depuis, sous contraceptifs oraux.
Jusqu'à présent, elle est en rythme sinusal et le cardiologue qui la suit régulièrement n'a pas jugé utile de
prescrire un traitement.
Cette patiente est admise aux urgences à l'hôpital pour une impotence fonctionnelle du membre inférieur droit,
apparue très rapidement après la violente douleur locale qu'elle a ressentie une heure auparavant.
L'examen clinique note une pression artérielle à 11/7 cmHg, un pouls rapide (100/minute) et irrégulier. Le
coeur est irrégulier à l'auscultation qui retrouve le roulement diastolique. La malade est apyrétique ; le membre
inférieur est livide, glacé, insensible, aréflexique ; le déficit moteur est évident ; les pouls poplité, tibial
postérieur et pédieux ne sont pas perçus du côté droit : tous les autres pouls sont présents.

Parmi les diagnostics suivants, lequel correspond au tableau brutal présenté par la malade ?
A - Ischémie aiguë du membre inférieur
B - Phlébite
C - Phlegmatia caerulea
D - Monoplégie
E - Accident vasculaire cérébral
Bonne(s) réponse(s) : A

Sans commentaire.

Sur laquelle de ces données d'examen vous appuyez-vous pour porter ce diagnostic ?
A - Aréflexie
B - Arythmie complète
C - Abolition des pouls
D - Roulement diastolique
E - Tachycardie
Bonne(s) réponse(s) : C

Sans commentaire.

Quel est le mécanisme de cet accident ?


A - Ischémie cérébrale
B - Embolie artérielle du membre inférieur
C - Hémorragie cérébrale
D - Sciatique paralysante
E - Thrombose carotidienne gauche
Bonne(s) réponse(s) : B

Sans commentaire.

Quel examen complémentaire vous paraît le mieux approprié pour affirmer le diagnostic ?
A - Examen doppler des vaisseaux carotidiens
B - Scanner cérébral
C - Thermographie des membres inférieurs
D - Artériographie des membres inférieurs
E - Artériographie des vaisseaux à destinée cérébrale
Bonne(s) réponse(s) : D

Sans commentaire.

Quel est le seul traitement immédiatement et constamment efficace dans une telle situation ?
A - Endartériectomie carotidienne
B - Héparinothérapie par voie veineuse
C - Embolectomie à la sonde de Fogarty
D - Traitement fibrinolytique
E - Anastomose extra-intra crânienne
Bonne(s) réponse(s) : C

L'ischémie aiguë artérielle est une complication classique du rétrécissement mitral. Son mécanisme est
l'embolie à partir d'un thrombus auriculaire gauche ; thrombus favorisé par la stase sanguine à l'occasion du
passage en arythmie.
Dans ce cas-ci, la conduite à tenir est la désobstruction en urgence : la sonde de Fogarty est idéale sur ce
terrain à artères saines.
Le traitement héparinique à dose efficace est systématiquement prescrit.
Dans un second temps, il faudra réduire le trouble du rythme puis faire le bilan du rétrécissement mitral
(estimation de la surface mitrale à l'écho-dopper continu).

448
Exclusivement sur DOC - DZ : www.doc-dz.com NADJI 85
RESIDANAT EN POCHE TOME II
Cas Clinique en QCM
Mlle X. 17 ans, sans antécédents particuliers. Consulte pour des douleurs thoraciques prolongées, non
déclenchées par l'effort, et pour des palpitations. A l'examen, vous notez :
- un rythme cardiaque à 3 temps, le bruit surajouté étant entendu à l'endapex, en fin de systole
- un petit souffle systolique à la pointe, débutant avec le bruit surajouté ci-dessus et se terminant avec le
deuxième bruit.
Le coeur est de volume normal sur le film thoracique de face. L'ECG montre :
- un rythme sinusal régulier à 70/minute
- un axe de QRS à 60°
- une onde T négative, asymétrique, peu profonde en D3-VF-V5-V6

Le diagnostic le plus probable d'après les données ci-dessus est :


A - Spasme coronarien
B - Souffle cardiaque anorganique
C - Insuffisance mitrale rhumatismale
D - Prolapsus mitral
E - Pleuro-péricardite
Bonne(s) réponse(s) : D

Sans commentaire.

Pour étayer le diagnostic, deux examens non invasifs sont utiles :


A - Vectocardiogramme
B - Echocardiogramme
C - Scintigraphie myocardique au 201 Thallium
D - Phonocardiogramme
E - Phlébogramme jugulaire
Bonne(s) réponse(s) : B D

Sans commentaire.

Les palpitations vous ont amené à faire pratiquer un enregistrement de l'ECG durant 24 heures
(HOLTER). Cet enregistrement a montré des extrasystoles auriculaires sporadiques et de rares
extrasystoles ventriculaires monomorphes. Vous proposez pour traiter Mlle X. :
A - 4 comprimés de Cordarone® par jour, 5 jours par semaine
B - 1 comprimé de .Digoxine® par jour
C - 1 comprimé d'Hydroquinidine® 6 fois par jour
D - L'abstention de tout traitement médicamenteux
E - 1 comprimé de Ténormine® (Aténolol) 3 fois par jour
Bonne(s) réponse(s) : D

Sans commentaire.

En vue de prévenir une greffe bactérienne sur l'endocarde, vous prescrivez 2 g de Clamoxyl® 2
heures et 8 heures après le(les) soin(s) dentaire(s) suivant(s) :
A - Extraction d'une dent dévitalisée avec granulome apical
B - Extraction d'une dent dévitalisée sans granulome apical
C - Obturation d'une carie de l'émail
D - Détartrage
E - Obturation d'une carie de la pulpe
Bonne(s) réponse(s) : A B D E

Sans commentaire.

449
Exclusivement sur DOC - DZ : www.doc-dz.com NADJI 85
RESIDANAT EN POCHE TOME II
Cas Clinique en QCM

Toutes les complications ci-dessous peuvent s'observer, en relation directe avec l'affection dont
souffre Mlle X, sauf une, laquelle ?
A - Fibrillation auriculaire
B - Endocardite bactérienne
C - Embolie artérielle systémique
D - Rupture du pilier postérieur de la valve mitrale
E - Rupture des cordages tendineux de la petite valve mitrale
Bonne(s) réponse(s) : D

Se voit dans l'infarctus du myocarde.


La prolapsus de la valve mitrale est une affection fréquente (4 à 6 % de la population) surtout chez la femme
souvent asymptomatique. Les signes fonctionnels se résument à des précordialgies atypiques, une asthénie,
une dyspnée des palpitations. L'auscultation retrouve typiquement un click mésotélésystolique suivi d'un
souffle, parfois un click isolé, un souffle télésystolique isolé dont la chronologie peut être précisée au
phonogramme. L'ECG peut montrer des anomalies de la repolarisation (T négatives, sous décalage ST)
variables d'un ECG à l'autre.
L'échographie fait le diagnostic avec en bidimensionnel un bombement d'un ou des deux feuillets de la valve
vers l'oreillette en systole des valves épaisses d'aspect myxoïde et en TM un aspect en hamac ou en louche.
Les troubles du rythme ventriculaires et auriculaires sont fréquents.
Le traitement est rarement nécessaire comme dans ce cas-ci. On a décrit cependant des extrasystolies
ventriculaires graves avec morts subites. L'endocardite bactérienne est également peu fréquente ; la
prophylaxie n'est pas systématique mais souhaitable en cas de souffle. Les accidents ischémiques artériels
transitoires ou constitués sont retrouvés dans 3 % des cas ; un mécanisme fibrilla-plaquettaire est incriminé.
Les autres complications sont l'insuffisance mitrale par aggravation du prolapsus ou rupture de cordage. Les
complications en regard de la fréquence du prolapsus sont cependant rares et le prolapsus de la valve mitrale
peut être considéré comme une affection bénigne dans l'immense majorité des cas.

Monsieur B., homme de 75 ans, sans antécédents pathologiques particuliers. T : 1.70 m. P : 78 kg. Consulte à
la suite d'une perte de connaissance avec chute, survenue alors qu'il se promenait. Le sujet s'est
immédiatement relevé et décrit de façon précise les circonstances de sa perte de connaissance. L'examen
clinique, cardiovasculaire et neurologique, est strictement normal.
L'électrocardiogramme à l'état basal montre :
- un rythme sinusal à 70/mn
- un intervalle PR à 0,17 sec
- une durée de QRS égale à 0"12
- un axe de QRS à - 60°,
- un aspect rSr' en V1

La chute est vraisemblablement due à :


A - Une crise comitiale
B - Une drop-attack
C - Un vertige de Ménière
D - Un bloc auriculo-ventriculaire paroxystique
E - Une ivresse aiguë
Bonne(s) réponse(s) : D

Sans commentaire.

L'électrocardiogramme que vous avez enregistré montrait :


A - Un bloc auriculo-ventriculaire du premier degré
B - Un bloc de branche gauche complet
C - Un bloc incomplet de la branche droite
D - Un bloc complet de la branche droite
E - Un hémibloc antérieur gauche
Bonne(s) réponse(s) : D E

QRS > 0,12s ; rSr en V1 ; axe du QRS à - 60°.

Quel traitement proposez-vous ici ?


A - Isuprel® per os 30 mg par jour
B - Acylanide® : 1 comprimé par jour
C - Isoptine® 120 : 3 comprimés par jour
D - Mise en place d'un stimulateur cardiaque "sentinelle"
E - Mise en place d'un stimulateur cardiaque à rythme fixe
Bonne(s) réponse(s) : D

Sans commentaire.

450
Exclusivement sur DOC - DZ : www.doc-dz.com NADJI 85
RESIDANAT EN POCHE TOME II
Cas Clinique en QCM

Le traitement correct (question 53} ayant été assuré, vous pouvez, si Mr B. présente des signes
d'insuffisance cardiaque, utiliser en association deux des cinq médications suivantes :
A - Avlocardyl®
B - Hydroquinidine®
C - Digoxine®
D - Cordarone®
E - Lasilix®
Bonne(s) réponse(s) : C E

Sans commentaire.

Vous pouvez d'autre part, s'il présente des extrasystoles ventriculaires fréquentes, le traiter avec
l'un des cinq médicaments suivants :
A - Ephédrine
B - Diurilix®
C - Adalate®
D - Cordarone®
E - Cédilanide® intraveineuse
Bonne(s) réponse(s) : D

Un interrogatoire précis et soigneux est la première étape du diagnostic étiologique d'une syncope.
L'association ici d'un bloc bifasciculaire (BBD + HBAC) et de syncope à l'emporte-pièce typique oriente vers
un syndrome d'Adams Stockes. L'indication d'un pace maker sentinelle se pose alors, parfois précédée d'une
exploration endocavitaire si le doute persiste quant à la séméiologie de la syncope. On y recherchera : un
dédoublement de H ; un HV supérieur à l'état de base à 70 ms ou une réponse positive à l'ajmaline (HV>100
ms ou apparition d'un BAV II ou III). Une cardiopathie sous jacente doit être recherchée devant des anomalies
de conduction, dans ce cas-ci il s'agit vraisemblablement d'une maladie de Lenègre en raison de l'âge du
patient.

1.2.CAS CLINIOUE N° : 3

Monsieur X.... 70 ans, souffre d'un angor chronique stable, traité depuis 2 ans par Tildiem®. Il y a un mois, il a
été hospitalisé pour une première poussée d'insuffisance ventriculaire gauche à l'occasion d'un paroxysme de
fibrillation auriculaire rapide, on a alors découvert une myocardiopathie dilatée. Après retour spontané en
rythme sinusal, l'ECG montrait un intervalle PR à 0,15 sec, des complexes QRS à 0,13 sec avec un aspect
qRS en D1, V5 et V6 et un aspect rSR' en D3, V1 et V2 ; l'axe de QRS est mesuré à - 60°.
Un traitement associant Cordarone®, Digoxine®, Risordan® et Lasilix® est ajouté au Tildiem®. Le patient est
réhospitalisé pour une bradyarythmie à 45/mn découverte à l'occasion d'un examen de contrôle : il ne se plaint
d'aucun symptôme nouveau, son état hémodynamique est satisfaisant ; I'ECG retrouve le même aspect des
complexes QRS ; le rythme auriculaire est sinusal et régulier à 65/mn. Il existe des ondes P bloquées, non
suivies de QRS, précédées sur 4 cycles d'un allongement progressif de PR. Le bilan biologique est normal.

Sur le premier tracé ECG enregistré il y a un mois, quelle est parmi ces 5 propositions celle qui
correspond à la description des complexes QRS ?
A - Bloc de branche gauche incomplet isolé
B - Bloc de branche gauche complet isolé
C - Bloc de branche droit incomplet isolé
D - Bloc de branche droit complet isolé
E - Bloc de branche droit complet + hémibloc gauche antérieur
Bonne(s) réponse(s) : E

Bloc de branche droit : QRS à 0,13s avec rSr en V1V2.


Hémi-bloc antérieur gauche : axe de QRS à -60° avec QD1 SQ3.

Sur le deuxième tracé ECG, quelle est parmi ces 5 propositions l'anomalie nouvellement apparue
?
A - Bloc sino-auriculaire
B - Bloc auriculo-ventriculaire du 1er degré
C - Bloc auriculo-ventriculaire du 2ème degré de type I
D - Bloc auriculo-ventriculaire du 2ème degré de type II
E - Bloc auriculo-ventriculaire du 3ème degré
Bonne(s) réponse(s) : C

Sans commentaire.

451
Exclusivement sur DOC - DZ : www.doc-dz.com NADJI 85
RESIDANAT EN POCHE TOME II
Cas Clinique en QCM

Quel est le siège le plus probable de ce trouble de conduction nouvellement


apparu ?
A - Intra-auriculaire
B - Sino-auriculaire
C - Noeud d'Aschoff-Tawara
D - Tronc du faisceau de His
E - Fibres de Purkinje
Bonne(s) réponse(s) : C

Sans commentaire.

Parmi les 5 médicaments prescrits chez ce patient,.le(s)quel(s) a(ont) pu favoriser la survenue de


ce nouveau trouble de conduction ?
A - Tildiem®
B - Cordarone®
C - Digoxine®
D - Risordan®
E - Lasilix®
Bonne(s) réponse(s) : A B C

Sans commentaire.

Dans l'immédiat, quelle est parmi ces 5 solutions thérapeutiques, celle qui vous paraît la plus
logique chez ce patient ?
A - Isuprel® en perfusion IV ou per os
B - Interruption des médicaments potentiellement dépresseurs de la conduction
C - Electrostimulation provisoire
D - Perfusion intraveineuse de chlorure de potassium
E - Electrostimulation définitive d'emblée
Bonne(s) réponse(s) : D

Les blocs de branche bifasciculaires ne contre-indiquent pas la Cordarone®, la Digoxine® ni le Tildiem®, mais
l'association des trois doit être évitée.
Le trouble étant ici bien toléré avec échappement haut situé on peut se contenter d'une surveillance sous
scope après arrêt des médicaments bradycardisants.
A plus longue terme, si une association potentiellement bradycardisante est indispensable, un pace maker
définitif pourra être proposé.

Monsieur D. Daniel, âgé de 38 ans, a vu apparaître depuis quelques semaines une dyspnée d'effort
progressive et une augmentation du volume de l'abdomen. L'interrogatoire apprend que le patient a présenté
des crises de RAA à l'âge de 14 et 16 ans, traitées par pénicilline et corticoïdes, une primo-infection
tuberculeuse sévère à l'âge de 20 ans, et enfin un accident de la circulation avec fractures pluri-costales
antérieures et hémothorax gauche à l'âge de 25 ans.
L'auscultation cardiaque révèle une arythmie complète récente et un 3ème bruit proto-diastolique. La TA est à
11/8. Les radiographies thoraciques montrent un coeur de volume normal, un épanchement pleural droit avec
séquelles de la base gauche, des hiles épaissis, des nodules calcifiés du lobe supérieur gauche. Il existe enfin
des calcifications péricardiques nettement visibles sur les incidences transverse et oblique. L'ECG confirme la
fibrillation auriculaire. Le bilan hémodynamique vient confirmer le diagnostic de péricardite chronique
constrictive.

Quelle(s) est(sont) la cause(les causes) possible(s) de la péricardite chronique constrictive chez


Monsieur D. Daniel ?
A - RAA
B - Infection virale
C - Hémopéricarde traumatique
D - Tuberculose
E - Collagénose
Bonne(s) réponse(s) : C D

La tuberculose est l'étiologie la plus fréquente.

452
Exclusivement sur DOC - DZ : www.doc-dz.com NADJI 85
RESIDANAT EN POCHE TOME II
Cas Clinique en QCM

Parmi les signes cliniques suivants, lequel aurait le plus contribué au diagnostic de P.C.C. ?
A - Circulation collatérale abdominale
B - Bruit de galop présystolique
C - Splénomégalie
D - Turgescence des jugulaires
E - OEdèmes des membres inférieurs
Bonne(s) réponse(s) :

Elle permet de différencier ce tableau d'une cirrhose décompensée.

A quel processus hémodynamique est dû le tableau clinique de P.C.C. présenté par ce patient ?
A - Adiastolie du VD
B - Asystolie du VG
C - Adiastolie du VG
D - Asystolie auriculaire
E - Asystolie du VD
Bonne(s) réponse(s) : A

La constriction prédomine sur les cavités droites.

Au plan hémodynamique, la P.C.C. est caractérisée par :


A - Augmentation de la pression veineuse centrale
B - Dépression protodiastolique profonde et brutale de la courbe VD
C - Plateau télédiastolique supérieur au tiers de la pression systolique du VD
D - Pression capillaire pulmonaire à 6 mmHg
E - Egalisation des pressions moyennes dans les cavités droites et la petite circulation
Bonne(s) réponse(s) : A B C E

Le tableau hémodynamique retrouve une égalisation des pressions diastolique de l'oreillette droite à l'artère
pulmonaire, significative si supérieure au tiers de la systolique du VD, avec aspect de dip plateau.

Quel(s) traitement(s) sera(seront) proposé(s) à Monsieur D. Daniel ?


A - Diurétique
B - Corticothérapie
C - Péricardectomie
D - Bêta-bloquant
E - Pénicilline
Bonne(s) réponse(s) : C

En attendant la chirurgie, on mettra en route le traitement de l'insuffisance cardiaque. On recherchera une


tuberculose (crachats, ponction pleurale, examen anatomopathologique du péricarde) et on mettra en route un
traitement aussi tuberculeux.

Une femme de 35 ans est hospitalisée en urgence pour oedème douloureux du membre inférieur gauche
intéressant jambe et cuisse. L'interrogatoire retrouve un épisode identique après le dernier accouchement (par
ailleurs normal), 3 ans auparavant, avec une évolution spontanée alors favorable. Aucun antécédent
pathologique n'est noté. La patiente prend des contraceptifs oraux depuis 2 ans.

Parmi les signes suivants, lequel est le plus évocateur d'une phlébite du membre inférieur gauche
?
A - Diminution du ballottement du mollet gauche
B - Diminution de la température locale
C - Hydarthrose du genou gauche
D - Réseau veineux superficiel mal rempli
E - Hyperthermie à 39 degrés C
Bonne(s) réponse(s) : A

Sans commentaire.

453
Exclusivement sur DOC - DZ : www.doc-dz.com NADJI 85
RESIDANAT EN POCHE TOME II
Cas Clinique en QCM

La phlébographie est indispensable. Quelle technique utiliserez-vous ?


A - Epreuve au fibrinogène marqué
B - Angiographie numérisée par ponction d'une veine du bras
C - Phlébographie unilatérale du membre inférieur gauche
D - Phlébographie bilatérale des membres inférieurs (ponction des veines pédieuses)
complétée par une iliocavographie
E - Iliocavographie seule
Bonne(s) réponse(s) : D

Elle recherche des lésions bilatérales.


Elle permet le bilan d'extension de la thrombose et apprécie le risque emboligène (vacuité de la veine cave).

Le diagnostic de syndrome de Cockett est porté. Il s'agit :


A - D'une phlegmatia coerulea
B - D'une embolie pulmonaire à bas bruit
C - D'une phlébite fémorale bilatérale
D - D'une compression anormale de la veine iliaque primitive gauche, par l'artère iliaque
primitive droite
E - D'une hypoplasie de la veine cave inférieure
Bonne(s) réponse(s) : D

Sans commentaire.

Si vous optez pour un traitement purement médical, quelle durée minimum de l'héparinothérapie
préconiseriez-vous ?
A - 5 jours
B - 15 a 20 jours
C - 3 mois
D - 6 mois
E - 12 mois
Bonne(s) réponse(s) : B

En cas d'atteinte des gros troncs veineux (veine iliaque, veine cave) un traitement fibrinolytique peut être
entrepris en l'absence de contre-indication et si la phlébite est vue suffisamment tôt. Ce traitement ne diminue
pas le risque d'embolie pulmonaire mais semblerait diminuer les séquelles post-phlébitiques.

Un homme de 66 ans est hospitalisé pour oedème aigu du poumon survenu à la suite d'un effort peu
important, alors qu'il arrachait les mauvaises herbes dans son jardin. Dans ses antécédents, on relève une
tuberculose pulmonaire à 20 ans, un ulcère duodénal à 52 ans, une hypertension artérielle modérée depuis 6
ans (20/11 au plus) équilibrée par Clonidine (Catapressan®) et ramenée à 15/9.
Il décrit, depuis deux mois, un angor d'effort caractérisé.
A l'examen clinique, une fois l'épisode d'oedème pulmonaire traité par diurétique intraveineux (furosémide
Lasilix®, 2 ampoules à 40 mg), il persiste encore quelques râles crépitants aux deux bases et on entend un
souffle mésosystolique rude et râpeux 4/6 au deuxième espace intercostal droit. La radioscopie montre un arc
inférieur gauche un peu bombant, arrondi. et l'électrocardiogramme un indice de Sokolow à 52 mm.

Cette sémiologie évoque un obstacle à l'éjection aortique. Quel est, à votre


avis, le type anatomique de cet obstacle ?
A - Retrécissement aortique valvulaire pur
B - Rétrécissement aortique sous-valvulaire fixe
C - Myocardiopathie obstructive
D - Rétrécissement aortique supravalvulaire
E - Coarctation aortique
Bonne(s) réponse(s) : A

Dans la myocardiopathie hypertrophique le souffle est de localisation mésocardiaque.

Pour préciser le diagnostic clinique, vous devrez rechercher :


A - Un souffle diastolique d'accompagnement
B - Une abolition du deuxième bruit
C - Un frémissement systolique
D - Des irradiations du souffle vers le dos
E - Un click protosystolique éjectionnel
Bonne(s) réponse(s) : A B C E

B - Est le témoin du caractère serré du rétrécissement aortique


E - Claquement d'ouverture de l'appareil valvulaire.

454
Exclusivement sur DOC - DZ : www.doc-dz.com NADJI 85
RESIDANAT EN POCHE TOME II
Cas Clinique en QCM

Le cliché radiographique et un examen à l'amplificateur de brillance effectués tout de suite après


l'épisode initial d'oedème pulmonaire pourront montrer :
A - Un aspect flou et pommelé péri-hilaire
B - Un double contour de l'arc inférieur droit
C - Des calcifications situées, de face, au-dessous de la ligne unissant l'angle cardio-phrénique
droit
et le point G

D - Une dilatation débordant à droite de l'aorte ascendante


E - Une opacité scissurale
Bonne(s) réponse(s) : A D E

C - Les calcifications aortiques sont situées au niveau du point G avec un mouvement vertical
D - Lésion de jet du rétrécissement aortique.

Une analyse plus complète de l'ECG pourrait aussi vous montrer :


A - Un sous-décalage de ST et une onde T négative en D1, VL et précordiales gauches
B - Un bloc auriculo-ventriculaire du premier degré
C - Un bloc de branche gauche complet
D - Un aspect QS en V1 V2
E - Une hypertrophie auriculaire droite
Bonne(s) réponse(s) : A B C D

A - Signes d'hypertrophie ventriculaire gauche systolique


B - Les troubles de conduction peuvent se voir par coulée calcaire sur le septum
C.D. - L'hypertrophie ventriculaire gauche peut entraîner un bloc de branche gauche complet avec aspect QS
en V1V2.

Le meilleur élément pour juger du caractère sévère ou non de l'obstacle sera :


A - La valeur de l'indice de Sokolow
B - La valeur de l'index cardio-thoracique
C - Le gradient de pression VG/aorte mesuré par cathétérisme
D - Les constantes ventriculaires gauches mesurées en écho TM
E - Les temps systoliques mesurés sur le carotidogramme
Bonne(s) réponse(s) : C

Le rétrécissement aortique est serré si le gradient pic à pic est supérieur à 70 mmHg
A.B.D. - Apprécient le retentissement sur le ventricule gauche.

Chez ce patient, l'évolution peut se compliquer :


A - D'un rhumatisme cardiaque évolutif
B - D'une endocardite infectieuse
C - D'embolies artérielles, par exemple cérébrales, oculaires
D - D'un trouble de conduction auriculo-ventriculaire sévère
E - D'une thrombose intracardiaque
Bonne(s) réponse(s) : B C D

C - Les embolies sont calcaires, elles sont très rares.

En dehors du Lasilix®, quel traitement utile auriez-vous pu ajouter au moment de la crise


d'oedème pulmonaire ?
A - Un digitalique
B - Une perfusion de Xylocaïne
C - Un dérivé nitré
D - Un bêta-bloquant
E - Un inhibiteur calcique
Bonne(s) réponse(s) : C

Les dérivés nitrés ainsi que les diurétiques doivent être utilisés avec prudence dans le rétrécissement aortique
car ils risquent de provoquer un collapsus par diminution trop importante de la précharge.
Le tableau de ce patient est une indication formelle au remplacement valvulaire : le pronostic d'un
rétrécissement aortique avec insuffisance cardiaque est inférieur à un an. Dans son bilan (outre le bilan
infectieux et préopératoire usuel) le doppler continu permettra de calculer le gradient VG-aorte, d'estimer la
surface aortique, les explorations invasives comprendront un catéthérisme gauche ainsi qu'une
coronarographie d'autant que le patient présente des douleurs angineuses.

455
Exclusivement sur DOC - DZ : www.doc-dz.com NADJI 85
RESIDANAT EN POCHE TOME II
Cas Clinique en QCM
Un homme de 55 ans, chauffeur livreur, vient de faire un infarctus du myocarde à localisation antéro-septale.
La douleur ayant disparu et en l'absence de toute complication, il est traité uniquement par Héparine® en
perfusion (400 mg par 24 heures). Au deuxième jour, on note l'apparition d'un souffle systolique endapexien et
latérosternal gauche, rude, avec frémissement à la palpation. Le rythme cardiaque reste sinusal à 95
battements/minute. Il existe un bruit de galop gauche. Le foie est de volume normal. Il n y a pas d'oedème des
membres inférieurs mais on perçoit des râles fins dans le 1/3 inférieur des deux champs pulmonaires.

Ce souffle peut être causé par :


A - Rupture du septum interventriculaire
B - Insuffisance mitrale fonctionnelle par défaillance du ventricule gauche
C - Insuffisance tricuspidienne par hypertension pulmonaire
D - Rupture d'un pilier de la valve mitrale
E - Fissuration de la paroi du ventricule gauche
Bonne(s) réponse(s) : A D

Sans commentaire.

Quel examen faut-il pratiquer en première intention pour orienter le


diagnostic ?
A - Radiographie pulmonaire standard en lit du malade
B - Phonocardiogramme
C - Echocardiogramme
D - Cathétérisme intra-cardiaque droit
E - Angiographie numérisée
Bonne(s) réponse(s) : C

Examen non invasif, il permet de visualiser en cas d'IM un aspect de prolapsus d'une valve mitrale. Une
épreuve de contraste (injection de microcavitation par une veine brocchiale) permet le diagnostic de rupture
septale (jet négatif dans le VD).
Le doppler pulsé peut mettre en évidence les flux anormaux.

Devant les signes d'aggravation de la défaillance myocardique, la stratégie thérapeutique est


établie sur l'évolution de :
A - Pressions dans l'artère pulmonaire après mise en place d'un cathéter
B - Gaz du sang
C - Diurèse horaire
D - Taux des enzymes : créatine phosphokinase, transaminase, lacticodéhydrogénase
E - Taux de prothrombine
Bonne(s) réponse(s) : A B C

A - La mise en place d'un catéther de Swan-Ganz permet la surveillance de la pression capillaire pulmonaire
et du débit cardiaque et l'adaptation du traitement.

L'aggravation de l'état du malade conduit au diagnostic de choc cardiogénique. Parmi les


paramètres hémodynamiques suivants, un ne correspond pas au tableau de choc cardiogénique
primitif par défaillance VG. Lequel ?
A - Pression ventriculaire gauche télédiastolique abaissée
B - Débit cardiaque effondré
C - Pression aortique moyenne basse
D - Pression pulmonaire moyenne augmentée
E - Résistances artérielles périphériques augmentées
Bonne(s) réponse(s) : A

Elle est élevée


D - L'HTAP est de type post capillaire sans gradient entre la pression artérielle pulmonaire et la pression
capillaire pulmonaire.
E - Elles sont augmentées du fait de la vasoconstriction généralisée (stimulation du système sympathique,
hypoxie).
Ce patient doit être proposé au chirurgien. En attendant l'intervention, une contre pulsion par ballonnet intra-
aortique pourra être proposée.

456
Exclusivement sur DOC - DZ : www.doc-dz.com NADJI 85
RESIDANAT EN POCHE TOME II
Cas Clinique en QCM
Ce tracé ECG a été enregistré chez un homme de 68 ans au décours d'une douleur angineuse spontanée qui
a duré 60 minutes.

Cliquez sur le bouton 'Dessin' pour afficher le schéma.

L'analyse de l'onde P permet de conclure à :

Cliquez sur le bouton 'Dessin' pour afficher le schéma.


A - Normalité
B - Hypertrophie auriculaire droite
C - Hypertrophie auriculaire gauche
D - Hypertrophie bi-auriculaire
E - Bloc intra-auriculaire
Bonne(s) réponse(s) : A

Durée < 0,1 0s.


Amplitude < 2,5 mm.

L'axe moyen de QRS est à :

Cliquez sur le bouton 'Dessin' pour afficher le schéma.


A - - 30 degrés
B - - O degré
C - + 30 degrés
D - + 60 degrés
E - + 90 degrés
Bonne(s) réponse(s) : D

L'axe de QRS est perpendiculaire à la dérivation isoélectrique (aVL) et dans le sens de la dérivation la plus
positive (D2).

L'analyse du complexe QRS permet de conclure à :

Cliquez sur le bouton 'Dessin' pour afficher le schéma.


A - Complexe QRS normal
B - Hypertrophie ventriculaire droite
C - Hypertrophie ventriculaire gauche
D - Infarctus du myocarde inférieur
E - Infarctus du myocarde antéro-latéral
Bonne(s) réponse(s) : C

Indice de Sokolow (SV1 + RV5) > 35 mm.

L'analyse de la repolarisation ventriculaire permet de conclure à l'existence de :

Cliquez sur le bouton 'Dessin' pour afficher le schéma.


A - Ischémie sous-épicardique
B - Lésion sous-épicardique
C - Ischémie-lésion sous-épicardique
D - Ischémie sous-endocardique
E - Lésion sous-endocardique
Bonne(s) réponse(s) : C

Ischémie sous épicardique car T négative.


Lésion sous épicardique car ST sus décalé.

457
Exclusivement sur DOC - DZ : www.doc-dz.com NADJI 85
RESIDANAT EN POCHE TOME II
Cas Clinique en QCM

Quelle est la localisation des anomalies de la repolarisation ventriculaire ?

Cliquez sur le bouton 'Dessin' pour afficher le schéma.


A - Inférieure
B - Postérieure
C - Antéro-latérale
D - Antéro-septale
E - Antérieure étendue
Bonne(s) réponse(s) : C

Sans commentaire.

Un boucher de 53 ans se plaint depuis 5 ans d'épisodes de palpitations à début et fin brusques, sans cause
déclenchante évidente. Son interrogatoire ne retrouve aucun antécédent pathologique notable. En dehors de
ces épisodes de palpitations, il ne se plaint ni de dyspnée, ni d'angor et mène une vie strictement normale. Il
n'y a, à l'auscultation cardiaque, aucun signe en faveur d'une valvulopathie et le cliché thoracique est normal.

Sachant que la fréquence des épisodes est d'environ un par semaine, quel examen vous semble
le plus utile pour en comprendre le mécanisme ?
A - ECG standard au repos
B - ECG d'effort
C - Enregistrement Holter (24 ou 48 h).
D - Etude électrophysiologique endocavitaire
E - Monitoring ECG (hospitalisation)
Bonne(s) réponse(s) : C

Sans commentaire.

L'une de ces crises de palpitations dure plus long temps que les précédentes (2 heures),
permettant l'enregistrement d'un aspect typique de flutter auriculaire, qui cède immédiatement. La
prévention des récidives fait appel en 1ère intention à :
A - Aténolol
B - Nifédipine
C - Flécaïnide
D - Digoxine®
E - Quinidiniques
Bonne(s) réponse(s) : E

C'est une prescription de première intention mais les troubles digestifs fréquents (20 %) obligent souvent au
changement de thérapeutique.
La prescription est précédée d'une dose test avec surveillance ECG quelques heures après la prise d'un
comprimé d'hydroquinidine simple à la recherche d'un allongement de l'espace PR, d'un élargissement du
QRS, d'un allongement de l'espace QT qui contre-indiqueraient la poursuite du traitement.

La prescription est : amiodarone 1 cp, cinq jours par semaine. Quelle est parmi les complications
suivantes, celle qui est à imputer à ce traitement ?
A - Photosensibilisation
B - Rétention urinaire
C - Diarrhée
D - Fourmillement du cuir chevelu
E - Refroidissement des extrémités
Bonne(s) réponse(s) : A

Sans commentaire.

Le malade revient trois mois plus tard, il est amaigri et signale que les épisodes de palpitations,
après une accalmie de quelques semaines, sont à nouveau fréquents, mais plus prolongés et
irréguliers. Quelle complication devez-vous redouter ?
A - Hyperthyroïdie
B - Endocardite bactérienne
C - Pancréatite chronique
D - Rupture de cordage de la valve mitrale
E - Péricardite aiguë
Bonne(s) réponse(s) : A

Sans commentaire.

458
Exclusivement sur DOC - DZ : www.doc-dz.com NADJI 85
RESIDANAT EN POCHE TOME II
Cas Clinique en QCM

Le médecin arrête toute médication ; deux mois plus tard, le malade a repris du poids mais se
plaint d'être un peu essoufflé à l'effort. Le tracé ECG enregistre une fibrillation auriculaire, cette
fois permanente. Quelle attitude envisagez-vous d'adopter ?
A - Réaliser immédiatement un choc électrique externe
B - Réaliser, après anticoagulation, une stimulation auriculaire rapide
C - Prescrire des bêta bloquants isolément
D - Prescrire des quinidiniques isolément
E - Aucune des attitudes précédentes.
Bonne(s) réponse(s) : E

Il faut attendre l'anticoagulation efficace avant d'envisager une réduction.

Madame X. Béatrice, âgée de 24 ans, institutrice, a présenté à l'âge de 12 ans, une crise de rhumatisme
articulaire aigu traitée par les corticoïdes pendant 6 semaines et la pénicilline pendant 3 mois. Elle vient vous
consulter, car lors d'un examen systématique récent de médecine du travail, une anomalie auscultatoire a été
décelée. Elle n'accuse aucun signe fonctionnel même lors de la pratique d'une activité sportive régulière
(gymnastique, tennis). Vous percevez effectivement à l'auscultation un discret souffle protodiastolique le long
du bord gauche du sternum, aucune autre anomalie n'est décelée à l'examen cardiaque. La TA est à 13/8. Les
pouls périphériques sont normaux. Il n'y a aucun souffle artériel, aucun signe d'insuffisance cardiaque. L'ECG
est normal de même que la silhouette cardiovasculaire en radioscopie. Vous posez à juste titre le diagnostic
de valvulopathie post-rhumatismale.

Quelle est la valvulopathie en cause chez cette patiente ?


A - Insuffisance mitrale
B - Insuffisance aortique
C - Rétrécissement aortique
D - Insuffisance tricuspidienne
E - Rétrécissement mitral
Bonne(s) réponse(s) : B

Sans commentaire.

Quel examen paraclinique est le plus utile pour confirmer le diagnostic chez cette patiente ?
A - Phonocardiogramme
B - Cathétérisme droit
C - Cathétérisme gauche
D - Echocardiogramme
E - ECG d'effort
Bonne(s) réponse(s) : D

L'échographie montre les signes indirects de régurgitation (fluttering diastolique de la valve mitrale et du
septum). Elle permet d'apprécier le retentissement sur le ventricule gauche (mesure des diamètres
ventriculaire gauche, calcul de la fraction de raccourcissement).
Le doppler continu et pulsé permet le diagnostic de la régurgitation avec une spécificité et une sensibilité
supérieures à 90 %. Différentes méthodes pour l'appréciation quantitative de la régurgitation sont proposées.

Le risque essentiel de cette valvulopathie chez madame X. est représenté dans les 5 ans qui
viennent, par :
A - Insuffisance ventriculaire gauche
B - Trouble du rythme
C - Embolie systémique
D - Endocardite bactérienne
E - Mort subite
Bonne(s) réponse(s) : D

Sans commentaire.

459
Exclusivement sur DOC - DZ : www.doc-dz.com NADJI 85
RESIDANAT EN POCHE TOME II
Cas Clinique en QCM

Au cours d'une diastole normale, il se produit :


A - Systole auriculaire
B - Contraction iso-volumétrique
C - Remplissage lent
D - Relaxation isométrique
E - Remplissage rapide
Bonne(s) réponse(s) : A C D E

La diastole commence avec la fermeture des sigmoïdes et se termine avec la fermeture des valves auriculo-
ventriculaires. La première phase est la relaxation isovolumétrique (D) la pression ventriculaire chute très
rapidement ; quand elle est inférieure à celle de l'oreillette, les valves auriculo-ventriculaires s'ouvrent. Débute
alors le remplissage rapide (E) puis remplissage lent (C).
La systole auriculaire (A) survient en télédiastole, elle contribue à 20 % du remplissage ventriculaire total.
La fermeture des valves auriculo-ventriculaires survient quand la pression ventriculaire est supérieure à celle
de l'oreillette.

A distance d'une crise de RAA, on peut observer comme séquelle(s) cardiaque(s) :


A - Insuffisance mitrale
B - Péricardite constrictive
C - Rétrécissement tricuspidien
D - Insuffisance aortique
E - Insuffisance tricuspidienne
Bonne(s) réponse(s) : A C D E

La péricardite rhumatismale n'évolue pas vers la constriction. Les conséquence du RAA sont par ordre de
fréquence : le rétrécissement mitral, l'insuffisance aortique, l'insuffisance mitrale, l'insuffisance tricuspidienne,
le rétrécissement aortique, rétrécissement tricuspidien. L'atteinte tricuspidienne est rarement isolée et on
retrouve toujours une atteinte mitrale ou aortique. Il n'y a jamais d'atteinte pulmonaire.

Chez cette patiente, il est impératif de recommander :


A - Surveillance stomatologique régulière
B - Echocardiogramme tous les 3 mois
C - Déconseiller toute activité sportive
D - Déconseiller toute grossesse
E - Déconseiller les traitements oestroprogestatifs
Bonne(s) réponse(s) : A

La prophylaxie de l'endorcardite bactérienne est impérative, la porte d'entrée principale étant stomatologique.
L'éradication des foyers dentaires se fera sous couverture antibiotique.
C et D - Ne sont pas déconseillés devant l'absence d'insuffisance cardiaque. La patiente devra par la suite
être suivie régulièrement par un examen clinique et des examens paracliniques simples (ECG. radiographie
de thorax, échographie, doppler pulse, doppler continu).
L'indication chirurgicale ne doit pas en effet être portée au stade des signes fonctionnels qui sont tardifs. A
côté de la classique augmentation de l'index cardiothoracique au delà de 0,6, la mesure des diamètre du
ventricule gauche à l'échographie et la surveillance des fractions d'éjection à l'angio-scintigraphie ventriculaire
gauche aident à poser l'indication opératoire.

La petite Stéphanie M., est âgée de 4 ans. Lors de l'examen scolaire systématique, on lui a découvert une
hypertension artérielle à 180 mmHg pour la maxima, 80 pour la minima au membre supérieur. Les parents
vous consultent pour élucider ce problème. Il n'y a pas d'antécédent pathologique personnel ni familial.

Quelle(s) est (sont), parmi les signes cliniques suivants, celui(ceux) qui est(sont) en faveur d'une
coarctation aortique ?
A - Hyperpulsation artérielle au niveau du cou
B - Abolition des pouls fémoraux
C - Souffle diastolique de 2ème espace intercostal droit
D - Souffle systolique de l'espace inter-omovertébral gauche
E - Dédoublement constant du 2ème bruit au foyer aortique
Bonne(s) réponse(s) : A B D

Sans commentaire.

460
Exclusivement sur DOC - DZ : www.doc-dz.com NADJI 85
RESIDANAT EN POCHE TOME II
Cas Clinique en QCM

La(les) exploration(s) radiologiques suivantes peuvent contribuer au diagnostic de coarctation de


l'aorte :
A - Radio simple du thorax de face
B - Radio du thorax après imprégnation barytée de l'oesophage
C - Aortographie
D - Artériographie digitalisée
E - Cathétérisme cardiaque par voie veineuse
Bonne(s) réponse(s) : A B C D E

A - Recherche d'érosion costale au bord inférieur des côtes provoquées par la circulation colatérale.
B - Elle montre un refoulement de l'icesophage par la dilatation de l'aorte sus jacente.
C. D. - Elles visualisent la sténose et permettent le bilan : circulation colatérale ; état des segments en amont
et en aval.
E - Elle permet de visualiser la sténose au temps gauche si la sténose n'est pas franchissable par voie
artérielle rétrograde.

Quelles(s) complication(s) l'hypertension artérielle peut-elle induire chez cette enfant ?


A - Des épistaxis à répétition
B - Une paralysie faciale périphérique
C - Une hypotrophie statu-ropondérale
D - Une insuffisance cardiaque
E - Une rétinopathie
Bonne(s) réponse(s) : A C D E

A.E. - Sont les conséquences de toute HTA.


C - Elle prédomine au niveau des membres inférieurs.
D - Elle est tardive

Compte-tenu du fait que cette enfant atteinte d'une coarctation aortique ne mesure que 88 cm,
quel examen, parmi les suivants, vous paraît-il utile de demander ?
A - Test de la sueur avec dosage du chlore
B - Test de stimulation de l'hormone de croissance
C - Recherche d'une entéropathie exsudative
D - Caryotype
E - Exploration du système rénine-angiotensine
Bonne(s) réponse(s) : D

A la recherche d'un syndrome de Turner.

Un homme de 55 ans, présentant une cardiopathie ischémique n'entraînant aucune douleur angineuse, vient
consulter pour des palpitations fréquentes.

L'ECG pratiqué montre l'existence d'une extrasystolie ventriculaire importante, polymorphe avec
quelques doublets, c'est à dire la présence :
A - De complexes QRS fins, mais prématurés et précédés d'une onde P
B - De complexes QRS fins, prématurés et non précédés d'une onde P
C - De complexes QRS élargis et prématurés
D - De manière intermittente d'une paire de complexes QRS élargis et prématurés
E - De complexes QRS élargis mais non prématurés
Bonne(s) réponse(s) : C D

La notion d'extrasystole implique la prématurité, elles sont larges car ventriculaires.

Devant cette extra-systolie importante, quelle est l'attitude à adopter ?


A - Surveiller l'ECG une fois par semaine
B - Réaliser un Holter (enregistrement de L'ECG. sur 24 heures)
C - Doser les enzymes sériques à la recherche d'un infarctus
D - Faire une épreuve d'effort
E - Réaliser une coronarographie
Bonne(s) réponse(s) : B

Il permet de quantifier les extrasystole-ventriculaire et recherche les critères de gravité de cette extra-systolie.
Plusieurs classifications ont été proposées. On retient, comme critères de gravité, des extrasystoles
nombreuses (>30 par heure), polymorphes, répétitives, en salve, la présence de tachycardie ventriculaire
(plus de 3 extrasystoles successives), le caractère lié à l'effort. La signification du phénomène R/T est
actuellement discutée.

461
Exclusivement sur DOC - DZ : www.doc-dz.com NADJI 85
RESIDANAT EN POCHE TOME II
Cas Clinique en QCM

Entre-temps, le malade présente une tachycardie ventriculaire nécessitant son hospitalisation


dans un service spécialisé. L'ECG doit montrer :
A - Une tachycardie très régulière à complexes QRS fins
B - Une tachycardie à complexes QRS élargis
C - Une dissociation des complexes auriculaires et ventriculaires
D - Des phénomènes de capture ou de fusion
E - L'absence d'onde P visible
Bonne(s) réponse(s) : B C D

Ce sont les trois critères indispensables pour parler de tachycardie ventriculaire. L'activité auriculaire est
difficile à repérer et on peut s'aider en faisant un tracé ECG rapide à 50 mm/s ou bien avec une sonde
oesophagienne.
La capture et la fusion sont deux phénomènes pathognomomiques de TV.
La capture est un complexe d'origine supraventriculaire, la fusion a un aspect intermédiaire entre le complexe
supraventriculaire et l'aspect ventriculaire.

Le traitement immédiat de cette tachycardie ventriculaire peut être :


A - La réalisation d'un choc électrique externe
B - L'injection intraveineuse de digitaliques (Cédilanide®)
C - L'injection intraveineuse de xylocaïne (Xylocard®)
D - Une perfusion de trinitrine intraveineuse (Lénitral®)
E - L'injection intraveineuse de furosémide (Lasilix®)
Bonne(s) réponse(s) : A C

L'indication entre les deux se faisant selon la tolérance hémodynamique.

Le retour au rythme sinusal permet de constater la persistance d'une extrasystolie ventriculaire


importante. Quel traitement anti-arythmique faut-il envisager ?
Quelle(s) est(sont) la(les) proposition(s) exacte(s) ?
A - Une digitalisation au long cours
B - La mexilétine (Mexitil®)
C - L'amiodarone (Cordarone®)
D - Le disopyramide (Rythmodan®)
E - Le vérapamil (Isoptine®)
Bonne(s) réponse(s) : B C D

Les bêta-bloquants sont également possibles en l'absence de dysfonction ventriculaire gauche.

Une jeune femme de 30 ans, sans antécédents rhumatismaux évidents, est admise à l'hôpital pour une
hémiparésie droite rapidement régréssive.
L'interrogatoire retrouve un essoufflement modéré à l'effort depuis un an et des accès de palpitation, de durée
habituellement brève, sauf le dernier qui a duré 3 heures, à début assez brusque ; le rythme cardiaque serait
rapide mais irrégulier : il y a une pollakiurie per-critique, la terminaison de l'accès se fait d'une façon
progressive.
L'examen neurologique est normal. Le rythme cardiaque est régulier à 70/mn, l'auscultation entend à la pointe
un éclat de B1, pas de souffle systolique, un claquement d'ouverture, un roulement holodiastolique plus net en
décubitus latérale gauche, constant, que la patiente soit auscultée en décubitus dorsal ou en position debout.
Il y a de plus un éclat net du B2 au foyer pulmonaire.
La radiographie de face montre un coeur peu augmenté de volume, un débord droit, un double contour, une
crosse aortique normale, une saillie à double bosse de l'arc moyen gauche, un arc inférieur gauche non
allongé. Les artères pulmonaires sont un peu augmentées dans les hiles, mais la transparence pulmonaire est
normale. L'ECG montre un rythme sinusal, une hypertrophie auriculaire gauche, un axe de QRS à + 100°, un
bloc incomplet droit. Le bilan biologique est normal.

Quelle cardiopathie vous suggèrent les données cliniques, radiologiques et


électrocardiographiques ?
A - Prolapsus valvulaire mitral
B - Insuffisance mitrale
C - Communication interauriculaire
D - Rétrécissement mitral
E - Myxome de l'oreillette gauche
Bonne(s) réponse(s) : D

Il s'agit d'un rétrécissement mitral avec retentissement sur le coeur droit (axe droit ; bloc incomplet droit) et
HTAP (éclat de B2 pulmonaire, grosses artères pulmonaire à la radio).

462
Exclusivement sur DOC - DZ : www.doc-dz.com NADJI 85
RESIDANAT EN POCHE TOME II
Cas Clinique en QCM

Quel est(sont) le(s) examen(s) paraclinique(s) non invasif(s) (à l'exception de la radiologie


standard et de l'électrocardiogramme de surface à 16 dérivations) immédiatement utile(s) pour
confirmer la cardiopathie mise en évidence par l'examen clinique ?
A - Enregistrement continu de l'électrocardiogramme sur 24 heures
B - Carotidogramme
C - Doppler cervical
D - Echocardiogramme
E - Scintigraphie myocardique au Thallium
Bonne(s) réponse(s) : D

Elle permet de faire le diagnostic du rétrécissement mitral : aspect en genou, mesure de la surface en
bidimensionnel, attraction paradoxale de la petite valve et diminution de l'amplitude la pente EF en TM ;
d'apprécier l'état de l'appareil valvulaire et sous-vavulaire (remaniements, calcifications) ; de calculer la
surface mitrale ; d'apprécier le retentissement (dilatation de l'oreillette gauche; des cavités droites, HTAP).
Le doppler continu permet le calcul du gradient et de la surface mitrale.
Le couplage des deux examens permet de poser d'emblée l'indication chirurgicale.

Quel trouble rythmique paroxystique, en fonction des données de l'interrogatoire, peut être
incriminé à l'origine de l'accident neurologique déficitaire transitoire ?
A - Extrasystoles auriculaires
B - Tachycardie ventriculaire
C - Extrasystoles ventriculaires
D - Fibrillation auriculaire
E - Tachycardie jonctionnelle paroxystique
Bonne(s) réponse(s) : D

Sans commentaire.

Quelle complication induite par le trouble rythmique paroxystique est directement responsable de
l'accident neurologique déficitaire régressif ?
A - Thrombose de l'oreillette gauche,
B - Embolie plaquettaire dans l'artère vertébrale droite
C - Spasme de l'artère sylvienne gauche
D - Chute du débit cardiaque
E - Petite embolie fibrino cruorique sylvienne gauche
Bonne(s) réponse(s) : E

La dilatation de l'oreillette gauche et la fibrillation auriculaire favorisent la stase sanguine et la formation d'un
thrombus auriculaire gauche. La localisation cérébrale est la plus fréquente.

Une exploration hémodynamique est prévue : dans quelle(s) cavité(s) (enregistrement(s)


simultané(s) des pressions) doi(ven)t être située(s) la(les) sonde(s) pour obtenir le diagnostic
hémodynamique de cette cardiopathie ?
A - Oreillette droite
B - Tronc de l'artère pulmonaire
C - Ventricule gauche
D - Racine de l'aorte
E - Capillaire pulmonaire ou oreillette gauche.
Bonne(s) réponse(s) : C E

La mesure de la pression ventriculaire gauche et de la pression capillaire pulmonaire par catéther de Swan-
Ganz ou la mesure de la pression dans l'oreillette gauche par catéthérisme trans-septal permettent de calculer
le gradient de pression oreillette gauche - ventricule gauche ainsi que la surface mitrale.

Une intervention chirurgicale est décidée et sera effectuée dans quatre semaines. Quel(s)
médicament(s) visant à empêcher la récidive de l'accident neurologique, allez-vous
préférentiellement prescrire pendant ces quatre semaines ?
A - Anticoagulants
B - Diurétiques
C - Dérivés quinidiniques
D - Dérivés nitrés à action prolongée
E - Bêta-bloquants
Bonne(s) réponse(s) : A C

Sans commentaire.

463
Exclusivement sur DOC - DZ : www.doc-dz.com NADJI 85
RESIDANAT EN POCHE TOME II
Cas Clinique en QCM
Monsieur D., 59 ans, cafetier, consulte pour des douleurs thoraciques. Elles débutent par les deux bras et
envahissent le thorax antérieur, puis postérieur ; leur tonalité est constrictive : elles augmentent
progressivement en intensité et en diffusion jusqu'à un maximum où elles s'accompagnent souvent d'un
malaise avec sueurs profuses, puis régressent ensuite aussi progressivement qu'elles sont apparues, avec
une durée globale de 20 à 30 minutes.
La trinitrine perlinguale récemment prescrite calme la douleur en 2 minutes. Elles apparaissent exclusivement
la nuit, réveillant le patient entre 2 et 4 heures du matin, leur évolution est périodique avec l'alternance de
phases d'activité où le patient souffre toutes les nuits et de longues phases asymptomatiques. Dans la
journée, Monsieur D., ne se plaint d'aucune gêne. L'interrogatoire apprend par ailleurs que Monsieur D. fume
40 g de tabac par jour, il s'agit d'un homme pléthorique (87 kg pour 1,72 m). L'examen clinique est normal,
hormis une TA à 170/110 mmHg au repos. La radiographie thoracique ne montre pas de cardiomégalie.
L'échocardiogramme est normal, de même que l'ECG de repos : par contre, l'ECG d'effort est possible avec
un sous-décalage ischémique de ST de 2 mm dès le palier de 60 Wans pour une fréquence cardiaque de
135/mn. Le bilan biologique montre essentiellement des triglycérides à 3,50 mmoles/l, le cholestérol total est à
6,2 mmoles/l, le cholestérol HDL est à 1 et le rapport apoprotéines A1/ apoprotéines B à 3, les gamma-GT
sont élevées à 120 UI/ml.

Laquelle(esquelles) parmi ces 5 propositions permet(tent) d'évoquer un mécanisme spasmodique


à l'origine de cet angor ?
A - Crises exclusivement spontanées
B - Crises nocturnes à horaire fixe
C - Crises de longue durée
D - Evolution périodique
E - Epreuve d'effort positive
Bonne(s) réponse(s) : A B D

Sans commentaire.

Laquelle(lesquelles) parmi ces 5 propositions permet(tent) d'évoquer l'existence d'une


athérosclérose coronarienne obstructive associée ?
A - Efficacité de la TNT perlinguale
B - Absence d'angor d'effort
C - Ethylisme chronique
D - ECG de repos normal
E - Epreuve d'effort positive
Bonne(s) réponse(s) : E

A - La trinitrine hormis son effet sur la précharge est aussi anti-spastique.

Parmi ces 5 propositions, laquelle(lesquelles) a(ont) pu favoriser le développement d'une


athérosclérose coronarienne obstructive ?
A - Tabagisme (40 g/j)
B - TA à 170/110 mmHg au repos
C - Cholestérol total à 6,2 mmoles/l
D - HDL-cholestérol à 1 g/l
E - Rapport apo A / apo B = 3
Bonne(s) réponse(s) : A B

D - Le HDL-cholestérol est au contraire "protecteur" et n'est pas athérogène.


Le rapport Apo A/Apo B est inversement corrélé au risque athérogène (l'apoprotéine A est la principale
protéine du HDL-cholestérol, l'apoprotéine B du LDL-cholestérol).

Parmi ces données de l'observation, quelle(s) est (sont) celle(s) dont l'apparition a pu être
favorisée par l'éthylisme ?
A - Hypertension artérielle
B - Triglycérides à 3,50 mmoles/l
C - Cholestérol total à 6,20 mmoles/l
D - Cholestérol-HDL à 1 g/l
E - Gamma-GT à 120 UI/ml
Bonne(s) réponse(s) : A B D E

La consommation modérée d'alcool serait protectrice vis à vis de la coronaropathie ; c'est ainsi que l'on
explique la fréquence relativement faible des maladies cardio vasculaires en France.

464
Exclusivement sur DOC - DZ : www.doc-dz.com NADJI 85
RESIDANAT EN POCHE TOME II
Cas Clinique en QCM

Parmi ces 5 anomalies transitoires de la repolarisation ventriculaire, quelle est celle qui, observée
au cours d'une douleur spontanée, est la plus suggestive d'un spasme coronarien ?
A - Sous-décalage ischémique de ST
B - Sus-décalage ischémique de ST
C - Onde T positive et symétrique
D - onde T négative et symétrique
E - Sous-décalage de ST + onde T négative et symétrique
Bonne(s) réponse(s) : A

Sans commentaire.

Une coronarographie est réalisée. Quelle serait parmi ces 5 propositions celle qui permettrait
d'exclure le diagnostic d'angor spastique sachant que l'exploration a été effectuée sans test
pharmacologique ?
A - Réseau coronarien angiographiquement normal
B - Athérome coronarien diffus sans sténose significative
C - Sténose fixe à 90 % sur l'interventriculaire antérieure
D - Sténoses significatives sur les 3 troncs principaux
E - Aucune des propositions précédentes ne permet d'exclure le spasme
Bonne(s) réponse(s) : E

Le spasme peut être associé aux lésions athéromateuses. Le test au Méthergin® ne sera réalisé qu'en
l'absence de sténose significative. Il peut être réalisé en cours de coronaropaphie ou en milieu d'unité de
soins intensifs cardiologiques avec enregistrement continu de l'ECG.

Si l'hypothèse d'un angor spastique se confirme, quel(s) médicament(s) doit (doivent) à priori être
exclu(s) chez ce patient ?
A - Sectral® (acébutolol)
B - Cordarone® (amiodarone)
C - Adalate® (nifédipine)
D - Risordan® (isosorbide dinitrate)
E - Avlocardyl® (propranolol)
Bonne(s) réponse(s) : A E

La libération de l'effet stimulant sous bêta-bloquant favorise la vasoconstriction.

Mr. Y., 40 ans, sans antécédents pathologiques notables. Aucun trouble fonctionnel de la sphère
cardiovasculaire. Découverte lors d'un examen médical systématique, d'un souffle à l'auscultation cardiaque
dont les caractères sont les suivants :
- foyer maximum : 3ème espace intercostal gauche près du sternum
- chronologie : protodiastolique
- intensité : 4/6.
Il existe à ce même foyer un souffle mésosystolique rude 2/6, sans frémissement, et un dédoublement du
2ème bruit, qui disparaît en inspiration. On note d'autre part, à la pointe du coeur, un murmure présystolique
de faible intensité sans modification de l'intensité du premier bruit. A la fourchette sternale, on entend un bruit
intense, situé 12 centièmes de seconde après le premier bruit. Les chiffres de la TA humérale sont les
suivants : 146/60 mmHg. La radiographie de face du thorax montre une convexité accrue du 3ème arc du bord
gauche, avec un rapport cardiothoracique à 0,55. Sur l'ECG :
- Rythme sinusal régulier à 68/mn
- Axe de QRS à -10°
- Durée de QRS = 0,12 sec
- Absence d'onde q en D 1 -VL-V5-V6
- Aspect en M de l'onde R en D 1 -VL-V5-V6

Monsieur Y. présente :
A - Un rétrécissement mitral pur
B - Une insuffisance pulmonaire
C - Une insuffisance aortique pure
D - Une insuffisance aortique associée à un rétrécissement mitral serré à valves souples
E - Un canal artériel persistant
Bonne(s) réponse(s) : C

Sans commentaire.

465
Exclusivement sur DOC - DZ : www.doc-dz.com NADJI 85
RESIDANAT EN POCHE TOME II
Cas Clinique en QCM

L'auscultation au 3ème espace intercostal gauche et l'ECG suggèrent l'existence :


A- D'une hypertrophie ventriculaire gauche
B - D'un bloc complet de la branche droite du faisceau de Hiss
C - D'un bloc complet de la branche gauche du faisceau de Hiss
D - D'un bloc incomplet de la branche gauche du faisceau de Hiss
E - D'une hypertrophie ventriculaire droite
Bonne(s) réponse(s) : C

Les blocs de branche peuvent en effet donner un dédoublement des bruits du coeur.
L'élargissement du QRS avec grande onde R dans les précordiales gauches donnent le diagnostic de bloc de
branche gauche complet.

4 des 5 étiologies suivantes doivent être discutées à l'origine de l'affection dont souffre Mr. Y.
Lesquelles ?
A - Malformation congénitale
B - Dystrophie du tissu élastique
C - Syphilis
D - Rhumatisme articulaire aigu
E - Athérosclérose coronarienne
Bonne(s) réponse(s) : A B C D

Sans commentaire.

L'évolution de l'affection de Mr. Y. comporte 3 des risques suivants :


A - Insuffisance ventriculaire gauche
B - Constitution d'un symphyse péricardique constritive
C - Syncope de Stokes-Adams
D - Thrombose de l'auricule gauche
E - Mort subite
Bonne(s) réponse(s) : A C E

Les troubles de conduction auriculo-ventriculaires sont rares.

Pour suivre l'évolution de l'affection de M. Y., vous estimez prioritaires les informations apportées
par un examen, de périodicité semestrielle : lequel ?
A - Cathétérisme cardiaque gauche
B - Cathétérisme cardiaque droit
C - Echocardiographie bidimensionnelle
D - Ventriculographie isotopique au repos et à l'effort
E - Etude électrophysiologique intracavitaire
Bonne(s) réponse(s) : C

Elle permet de surveiller les diamètres ventriculaires gauches, la fraction de raccourcissement (qui est
corrélée à la fraction d'éjection) et de réunir les arguments en faveur d'une indication opératoire.

Un homme de 65 ans, en bon état général, vient consulter pour des douleurs à la marche, du mollet droit.
Parmi les antécédents, on retrouve une forte intoxication tabagique.

Quel est le critère clinique le plus évocateur de l'origine artérielle de cette claudication
intermittente ?
A - Apparition de la douleur après une distance de marche variable
B - Cessation de la douleur au repos
C - Crampe nocturne du mollet
D - Atténuation de la douleur lorsque le membre est surélevé
E - Irradiation descendante de la douleur
Bonne(s) réponse(s) : B

Sans commentaire.

466
Exclusivement sur DOC - DZ : www.doc-dz.com NADJI 85
RESIDANAT EN POCHE TOME II
Cas Clinique en QCM

L'examen clinique montre la présence d'un pouls fémoral droit, mais l'absence de pouls sous-
jacents. Au membre inférieur gauche, tous les pouls sont perçus. Quel est le siège probable de
l'oblitération ?
A - Artère iliaque primitive droite
B - Artère iliaque externe droite
C - Artère fémorale superficielle droite
D - Artère tibiale antérieure droite
E - Artères tibiales antérieure et postérieure droites
Bonne(s) réponse(s) : C

Sans commentaire.

Un bilan artériographique est souhaitable chez cet homme jeune. D'après les données de
l'examen clinique, quelle(s) est (sont) la (les) techniques(s) radiologiques(s) envisa-
geable(s) :
A - Angiographie numérisée par ponction d'une veine du bras
B - Artériographie par ponction bifémorale
C - Aorto-artériographie par ponction aortique translombaire
D - Méthode de Seldinger rétrograde par ponction fémorale droite
E - Méthode de Seldinger par ponction axillaire
Bonne(s) réponse(s) : B C

A - Ne permet pas une bonne visualisation des axes vasculaires des membres inférieurs.
D - Le catéthérisme par la méthode de Seldinger n'est pas souhaitable devant la présence de lésion probable
à ce niveau.
B - Ne visualise pas l'aorte.

Le périmètre de marche étant de 300 mètres, vous optez pour un traitement médical, qui
comporte plusieurs mesures. Laquelle est discutable ?
A - Arrêt du tabac
B - Marche à pied quotidienne
C - Hygiène des pieds
D - Anticoagulants oraux
E - Surveillance clinique régulière
Bonne(s) réponse(s) : D

Ils ne diminuent pas la survenue des accidents d'ischémie aiguë et n'agissent pas sur les lésions
athéromateuses et exposent aux accidents des anticoagulants.

Il s'agit d'une femme de 45 ans normotendue, atteinte d'une maladie mitrale connue depuis 20 ans. Elle est
hospitalisée pour oedème aigu du poumon ; à l'auscultation, on entend un souffle systolique d'insuffisance
mitrale et un roulement diastolique de rétrécissement mitral.
Un bilan est réalisé après traitement de l'accident aigu.

Parmi les éléments suivants, révélés par la radio thoracique, quel est celui qui n'est pas
directement en rapport avec la maladie mitrale ?
A - Un double contour de l'arc inférieur droit
B - Un épanchement pleural
C - Une cardiomégalie
D - Des images de sub-oedème pulmonaire
E - Une crosse aortique déroulée
Bonne(s) réponse(s) : E

Sans commentaire.

L'échocardiogramme mettra en évidence les signes contenus dans les propositions suivantes
sauf une, laquelle ?
A - Une grosse oreillette gauche
B - Une ouverture normale des valves aortiques
C - Une dilatation du ventricule droit
D - Une diminution de la pente EF
E - Un fluttering de la grande valve mitrale
Bonne(s) réponse(s) : E

C'est le signe indirect de l'insuffisance aortique à l'échographie TM.

467
Exclusivement sur DOC - DZ : www.doc-dz.com NADJI 85
RESIDANAT EN POCHE TOME II
Cas Clinique en QCM

Quelle est la réponse exacte. Un cathérisme cardiaque droit et gauche montrera :


A - Des pressions normales dans les cavités droites
B - Un débit cardiaque égal à 81/minute
C - Une pression systolique de l'AP égale à 20 mm Hg
D - Une pression moyenne capillaire pulmonaire égale à 12 mm Hg
E - Une grande onde V capillaire
Bonne(s) réponse(s) : E

Elle signe l'insuffisance mitrale.


Les autres résultats attendus seraient une HTAP post-capillaire avec pressions capillaires élevées sans
gradient alvéolo-capillaire avec pressions droites élevées. Le débit cardiaque serait abaissé.

Quel est le signe hémodynamique le plus suggestif de l'insuffisance mitrale ?


A - La grande onde X sur la courbe de pression de l'oreillette droite
B - La grande onde V sur la courbe de pression de l'oreillette gauche
C - Le "dip plateau"
D - L'augmentation des pression dans l'artère pulmonaire
E - L'élévation du débit cardiaque
Bonne(s) réponse(s) : B

C'est une augmentation de la pression auriculaire due au reflux.

Parmi les propositions suivantes, quel est le traitement à éviter ?


A - Tonicardiaques
B - Diurétiques
C - Vasodilatateurs mixtes (artériel et veineux)
D - Association tonicardiaque + diurétique + vasodilatateurs mixtes
E - Bêta-bloquants
Bonne(s) réponse(s) : E

Le seul traitement est chirurgical avec remplacement valvulaire. En attendant, on mettra en route un
traitement médical (intérêt des inhibiteurs de l'enzyme de conversion).

Ce patient âgé de 26 ans est venu au service des urgences car une douleur rétrosternale vive s'est installée
assez rapidement en début de matinée. L'intensité croissante de la douleur persiste au repos. Assis,
légèrement penché en avant, il évite de parler ou d'inspirer trop fortement, car cela augmente sa douleur.
Celle-ci est rétrosternale et irradie en bretelle vers les trapèzes. La tension artérielle est a 13/7, le pouls est
régulier à 75, la température est à 38° C. L'auscultation cardiaque entend un frottement à cheval sur les bruits
du coeur qui sont bien perçus. L'examen pulmonaire, abdominal et général ne montre pas d'autre anomalie. Il
ne fume pas, n'a pas d'antécédent notable, et a fait son service militaire chez les parachutistes.
L'électrocardiogramme que vous réalisez révèle une surélévation du point J de 1 mm en D1, D2, D3 et VF,
avec des ondes T normales, et on observe un sous décalage du segment PQ de 0,5 mm en D2 et VF. Le
cliché thoracique est strictement normal. Les seules anomalies biologiques relevées dans l'immédiat sont une
vitesse de sédimentation à 30 mm à l'heure, des transaminases à 55 U.I. pour TGO et 63 U.I. pour TGP.

L'aspect du segment ST à l'ECG vous amène à porter le diagnostic de :


A - Ischémie sous-épicardique
B - Lésion ischémie sous-épicardique
C - Ischémie sous-endocardique
D - Lésion sous-endocardique
E - Aucune des propositions ci-dessus
Bonne(s) réponse(s) : E

Dans les péricardites le sus décalage du segment ST est concave vers le haut.

Si un échocardiogramme bidimensionnnel était réalisé, que montrerait-il vraisemblablement ?


A - Une épaisseur septale augmentée avec une valve mitrale antérieure touchant le septum en
systole
B - Une akinésie complète de la paroi postérieure du ventricule gauche
C - Un espace clair non échogène en arrière et en avant du coeur témoignant d'une collection
liquidienne antérieure à 500 ml
D - Un anévrisme de la pointe du ventricule gauche
E - Aucune des anomalies ci-dessus
Bonne(s) réponse(s) : E

Un épanchement de 500 ml est un épanchement important qui donnerait un tableau de tamponnade.

468
Exclusivement sur DOC - DZ : www.doc-dz.com NADJI 85
RESIDANAT EN POCHE TOME II
Cas Clinique en QCM

Quelle décision thérapeutique allez-vous prendre dans l'immédiat ?


A - Héparine® sous cutanée, une injection de 0,4 ml toutes les heures
B - Isoptine® 120 mg (vérapamil), 1 gélule 3 fois par jour
C - Indocid® 25 mg (indométacine), 1 gélule 4 fois par jour
D - Risordan® 20 mg (dinitrate d'isosorbide), 1 comprimé 3 fois par jour
E - Solupred® 20 mg (prédnisolone), 1 comprimé 3 fois par jour
Bonne(s) réponse(s) : C

Le traitement comprend le repos au lit, les anti inflammatoires non stéroïdiens.


Les anticoagulants sont contre indiqués en raison du risque théorique d'hémopéricarde et de tamponnade.
Les corticoïdes favoriseraient les rechutes.

Vous le revoyez vers 19 heures. Son épouse est venue le rejoindre. Il souhaite rentrer à son
domicile avec elle : en effet il ne souffre plus aussi intensément et se sent mieux. Votre attitude
comporte :
A - Poursuite de l'hospitalisation en service de cardiologie pendant 8 à 10 j
B - Retour au domicile le soir même avec une lettre pour le médecin de famille
C - Réalisation systématique d'un échocardiogramme bidimensionnel dans les 48 h même si
tout va bien
D - Réalisation systématique d'un électrocardiogramme dans deux semaines, même si tout va
bien
E - Repos physique, à la chambre, pendant une semaine, même si tout va bien
Bonne(s) réponse(s) : B D E

L'évolution des péricardites aiguës bénignes est favorable marquée par des rechutes possibles.
L'ECG montre l'évolution électrique caractéristique avec aplatissement du segment ST puis négativation de T,
enfin normalisation de T au bout de plusieurs semaines.

Si l'électrocardiogramme était réalisé à la deuxième semaine et montrait une inversion de l'onde T


en D1, D2, D3, vous devriez conclure que :
A - La maladie n'a pas fini d'évoluer
B - Le patient doit être hospitalisé
C - Une coronarographie est indispensable
D - Le choix thérapeutique initial n'était pas le bon
E - Un autre électrocardiogramme est a prévoir dans deux semaines
Bonne(s) réponse(s) : A E

Sans commentaire.

Une femme âgée de 50 ans, se plaint d'une dyspnée d'effort. Elle n'a pas d'antécédents connus de RAA.
L'auscultation cardiaque permet d'entendre un roulement diastolique et un dédoublement du B2. Sur
l'électrocardiogramme, il existe des signes d'hypertrophie auriculaire gauche. La radiographie pulmonaire met
en évidence un aspect de "poumon mitral". Il existe un aspect typique sur l'échocardiogramme. Le
cathétérisme met en évidence une élévation de la pression capillaire pulmonaire. Tous ces signes sont
évocateurs d'une sténose mitrale.

Indiquez la proposition inexacte. Les signes auscultatoires chez cette femme atteinte de sténose
mitrale sont dominés par un roulement diastolique dont les caractères sont les suivants :
A - Le roulement siège à la pointe
B - Il est mieux perçu en décubitus latéral gauche
C - Il s'accompagne d'un frémissement cataire
D - Il diminue à l'effort
E - Il diminue lors de l'arythmie complète
Bonne(s) réponse(s) : D

Sans commentaire.

Indiquez la proposition inexacte. Les signes électrocardiographiques consistent :


A - En un allongement de P supérieur à 12/100ème de seconde
B - En une augmentation de l'amplitude de P supérieure à 2,5 mm en D2
C - En un aspect dicrote de P en V1 avec négativité empâtée
D - En une déviation axiale droite
E - En une négativité de T en D3-aVF
Bonne(s) réponse(s) : E

A et C - Indique l'hypertrophie auriculaire gauche.


B - Indique l'hypertrophie auriculaire droite.
C - Se voit en cas d'hypertrophie ventriculaire droite.
469
Exclusivement sur DOC - DZ : www.doc-dz.com NADJI 85
RESIDANAT EN POCHE TOME II
Cas Clinique en QCM

Indiquez la proposition inexacte. Sur l'échocardiogramme, vous constatez :


A - Des mouvements réduits de la grande valve avec aspect en créneau
B - Une absence d'onde A
C - Un épaississement des feuillets
D - Une calcification des feuillets
E - Un mouvement postérieur du feuillet postérieur de la mitrale durant la diastole
Bonne(s) réponse(s) : E

En TM on voit :
- un épaississement des feuillets mitraux
- des calcifications
- une diminution de l'amplitude CE et de la pente EF avec disparition de l'onde a
- une attraction de la petite valve vers la grande valve qui signe la fusion commissurale.

Indiquez la proposition inexacte. Sur la téléradiographie de coeur, on retrouve :


A - Une augmentation de volume des hiles pulmonaires
B - Des lignes de Kerley
C - Une image de double contour des oreillettes au niveau du bord droit du coeur
D - Une élévation du point G au niveau du bord gauche
E - Un comblement de l'espace clair rétrosternal du profil
Bonne(s) réponse(s) : D

Sans commentaire.

Indiquez la proposition inexacte. Du point de vue hémodynamique, cette sténose mitrale peut se
traduire par :
A - Une élévation de la pression artérielle pulmonaire
B - Une augmentation de la pression télédiastolique du ventricule gauche
C - Une ébauche de gradient supérieur ou égal à 15 mmHg entre PAP moyenne et pression
capillaire moyenne
D - Une élévation des pressions dans l'oreillette droite
E - Une élévation de la pression télédiastolique ventriculaire droite
Bonne(s) réponse(s) : B

Il n'y a pas d'insuffisance ventriculaire gauche : les conséquences hémodynamiques se situent en amont au
rétrécissement.

Un homme de 55 ans est hospitalisé pour une symptomatologie dominée par :


- Un essoufflement à l'effort avec une gêne constructive thoracique haute
- Des lipothymies survenant également à l'effort avec impression de "brouillard devant les yeux".
Ces manifestations se sont accentuées depuis un an au point de lui interdire les gestes simples de son métier
de cordonnier. A l'examen clinique, on note un frémissement systolique de base, un souffle systolique râpeux
vers les carotides, maximum au 2ème espace intercostal droit et une forte atténuation du deuxième bruit. Il
n'existe aucun signe d'insuffisance cardiaque et les pouls périphériques sont bien perçus. La radiographie
thoracique objective une hypertrophie ventriculaire gauche : une saillie de l'aorte initiale et un flou hilaire. Le
diagnostic de retrécissement aortique (RAO) est porté.

Parmi les éléments d'auscultation suivants, il est logique de retrouver dans le cadre de cette
cardiopathie :
A - Galop protodiastolique
B - Dédoublement de B1
C - Souffle protodiastolique
D - Renforcement post-extrasystolique du souffle
E - Eclat de B1
Bonne(s) réponse(s) : C D

L'auscultation retrouve un souffle systolique râpeux maximal au deuxième espace intercostal droit, irradiant
dans les vaisseaux du cou, le long du bord gauche du sternum. Son maximum est mésosystolique, il se
renforce après diastole longue (ici après une extrasystole).
Un clic protosystolique peut être entendu ainsi qu'un souffle protodiastolique témoignant d'une insuffisance
aortique associée.
L'abolition du B2 et un maximum télésystolique se retrouvent dans les RAO serrés.

470
Exclusivement sur DOC - DZ : www.doc-dz.com NADJI 85
RESIDANAT EN POCHE TOME II
Cas Clinique en QCM

Parmi les signes électrocardiographiques suivants, quel est celui qui a le moins de chance d'être
rencontré chez ce malade ?
A - Fibrillation auriculaire
B - Surcharge auriculaire gauche
C - Aspect QS en VI et V2
D - Ondes R augmentées d'amplitude en D1,VL,V5 et V6
E - Allongement de PR
Bonne(s) réponse(s) : B

A - Peut se voir.
C - Traduit un bloc de branche gauche.
D - Traduit l'hypertrophie ventriculaire gauche.
E - Les troubles de conduction sont dus à des coulées calcaires sur le septum.

Parmi les étiologies suivantes, vous pouvez avec certitude, éliminer :


A- RAA
B - Hérédo dégénérescence conjonctivo-élastique
C - Malformation congénitale
D - Syphilis
E - Idiopathique
Bonne(s) réponse(s) : B D

C'est l'atteinte idiopathique avec calcification (maladie de Monckeberg) qui est la plus fréquente.
Elle survient plus fréquemment sur des anomalies congénitales (bicuspidie aortique).
L'atteinte rhumatismale constitue 10 % des RAO et est tardive vers 50-60 ans.
B et D - Sont des causes d'insuffisance aortique.

Lors du cathétérisme gauche, la pression aortique systolique est de 120 mm de Hg. Quel est le
chiffre de pression systolique ventriculaire gauche qui vous paraît le plus logique chez ce malade
?
A - 80 mm Hg
B - 100 mm Hg
C - 120 mm Hg
D - 150 mm Hg
E - 200 mm Hg
Bonne(s) réponse(s) : E

Etant donné la présence d'une symptomatologie fonctionnelle, on s'attend à trouver un RAO serré.
Le gradient VG-aorte est significatif à partir de 50 mm Hg et le RAO est considéré comme serré si le gradient
est supérieur à 70 mm Hg (l'index cardiaque étant conservé).

Parmi les complications suivantes, quelle est celle qui a le moins de chance de survenir chez ce
malade avant la mise en place d'une prothèse valvulaire ?
A - Mort subite
B - Embolie cruorique de la grande circulation
C - OEdème aigu du poumon
D - Endocardite bactérienne
E - Infarctus du myocarde.
Bonne(s) réponse(s) : B

Les embolies dans le RAO sont rares et ce sont des embolies calcaires.
C - Le pronostic de survie est alors faible (inférieur à un an) et l'indication chirurgicale est formelle.

471
Exclusivement sur DOC - DZ : www.doc-dz.com NADJI 85
RESIDANAT EN POCHE TOME II
Cas Clinique en QCM
Monsieur A. 42 ans, 1 m 75, 70 kilos. Représentant de commerce. Père mort subitement à 47 ans. Mère en
bonne santé. Lui-même a toujours été en bonne santé jusqu'à il y a 2 mois où il a ressenti pour la première
fois en marchant vite le matin pour aller à son garage situé à 50 mètres de sa maison, au sommet d'une rue
en pente, un serrement médiothoracique accompagné d'une sensation pénible de lourdeur des membres
supérieurs. Il a du s'arrêter avant de reprendre sa marche plus lentement. Depuis lors, la même douleur s'est
reproduite à plusieurs reprises dans les mêmes circonstances.

La scène clinique fait d'abord penser :


A - A une névralgie intercostale
B - A une arthrose cervicale
C - A une angine de poitrine par arthérosclérose coronarienne
D - A une angine de poitrine par spasme coronarien
E - A un spasme du bas oesophage
Bonne(s) réponse(s) : C

Tableau typique d'angor d'effort quant au siège, aux irradiations, au type de la douleur. Les circonstances de
déclenchement (marche en côte rapide), leur reproductivité (seuil ischémique) et l'arrêt à l'effort sont
également typiques.

L'auscultation cardiaque est normale : pouls artériels tous perçus avec souffle systolique
carotidien sous angulo maxillaire gauche 3/6 ; TA = 160/100 mm Hg. Volume cardiaque et
silhouette cardio-aortique normaux. Rapport cardiothoracique = 0,47. L'ECG au repos est
enregistré. Vous retenez comme signe(s) pathologique(s) :
A - Un axe de QRS à + 60 degrés
B - Un rythme sinusal régulier à 68/mn
C - Un intervalle PR à 0"24
D - Un indice SV1 + RV5 = 4,6 mv
E - Une onde T négative asymétrique en D3
Bonne(s) réponse(s) : C D

C - C'est un bloc auriculo-ventriculaire du premier degré.


D - L'indice de Sokolow est supérieur à 35 mm et donc indique une hypertrophie ventriculaire gauche.
E - L'onde T peut être négative en D3 de façon physiologique.

Le bilan complémentaire que vous avez demandé rapporte une valeur anormale pour :
A - Glycémie à jeun : 5,10 mmoles/litre
B - Cholestérolémie : 7,09 mmoles/litres
C - Triglycéridémie : 1,15 mmoles/litre.
D - Uricémie : 450 micromoles /litre.
E - Créatininémie : 95 micromoles/litre.
Bonne(s) réponse(s) : B D

Sans commentaire.

Vous décidez de faire en premier lieu le bilan carotidien. L'examen qui précisera le mieux le siège
et le degré de la sténose est :
A - Electroencéphalogramme avec compression carotidienne droite
B - Carotidogramme
C - Doppler cervical
D - Angiographie digitalisée des artères cervicales
E - Ophtalmodynamométrie
Bonne(s) réponse(s) : D

On commencera le bilan par un doppler carotidien, examen non invasif ; il ne détecte que la sténose
supérieure à 50 %. L'angiographie précise mieux le siège, le degré de sténose, son étendue.

472
Exclusivement sur DOC - DZ : www.doc-dz.com NADJI 85
RESIDANAT EN POCHE TOME II
Cas Clinique en QCM

Le sténose carotidienne étant asymptomatique et de degré modéré, vous décidez l'abstention


thérapeutique à son sujet. Quel est le premier examen complémentaire à faire pour évaluer la
sévérité de l'insuffisance coronarienne ?
A - Scintigraphie myocardique au repos
B - Enregistrement Holter de 24 heures
C - Electrocardiogramme avec hyperpnée
D - Electrocardiogramme d'effort
E - Coronarographie
Bonne(s) réponse(s) : D

L'épreuve d'effort permet de faire le diagnostic d'angor d'effort, d'en apprécier la sévérité (niveau d'effort pour
lequel apparaissent les signes, trouble du rythme ventriculaire...)
Couplée à la scintigraphie myocardique, elle a une sensibilité et une spécificité respective de 85 % et de 90
%. Pour certains, la coronarographie serait systématique chez cet homme jeune.

La restriction alimentaire devra porter surtout sur :


A - Les graisses végétales
B - Les viandes grasses
C - Les féculents
D - Le sel
E - Les produits laitiers non écrémés
Bonne(s) réponse(s) : B C D

Sans commentaire.

Une femme jeune de 33 ans est hospitalisée pour insuffisance cardiaque globale fébrile (température 39
degrés C).
Depuis 15 jours apparition d'une toux et d'une dyspnée progressivement croissante puis spontanée avec
sueurs nocturnes, prise de poids de 7 kg. La température n'a pas été surveillée, elle est à 38 degrés la veille
de l'hospitalisation. Dans les antécédents : syndrome dépressif majeur, 3 tentatives d'autolyse en 2 ans avec
anorexie mentale ; notion de palpitations depuis 2 ans et un examen cardiologique avec échocardiogramme
avait permis de conclure à l'existence d'une insuffisance mitrale probablement rhumatismale de faible
importance. Enfin des soins dentaires ont été pratiqués un mois auparavant. A l'entrée dans le service, 1,55 m
52 kg, mauvais état général avec teint gris, mauvais état dentaire, pression artérielle 120/60 mmHg, fréquence
cardiaque 100/mn, pas d'éruption cutanée, pas de splénomégalie ni d'adénopathies. orthopnée, oedèmes des
membres inférieurs bilatéraux, hépatomégalie avec reflux hépatojugulaire. Les pouls périphériques sont tous
perçus, l'auscultation pulmonaire révèle des crépitants dans les deux champs. L'auscultation cardiaque révèle
un souffle systolique 2/6 apexo-axillaire, un souffle diastolique 2/6 le long du bord gauche du sternum. Un bruit
de galop présystolique et les bruits du coeur sont bien perçus.
La radiographie thoracique montre un oedème alvéolaire, un index cardiothoracique à 0,55.
L'électrocardiogramme montre un rythme sinusal avec un PR à 0,18 secondes, un axe de QRS à + 30 degrés,
une repolarisation et des ventriculogrammes normaux. Numération globulaire : GR 4 180 000/mm3, GB 10
900/mm3 avec 82 % de polynucléaires. La vitesse de sédimentation est à 20/40. Fibrinogène 5,10 g/l. Le
ionogramme sanguin est normal et la créatinine également. Il n'y a pas d'hématurie.

Parmi les examens suivants, l'un d'entre eux, systématiquement réalisé fournira un élément
diagnostic et thérapeutique :
A - La radiographie des sinus
B - Le cathétérisme droit
C - Les hémocultures aérobie et anaérobie
D - Le dosage des antistreptolysines
E - L'électrophorèse des protéines
Bonne(s) réponse(s) : C

"Toute fièvre de plus 20 jours chez un patient porteur d'un souffle cardiaque doit être considérée comme une
endocardite infectieuse". D'autant plus que dans ce cas le tableau clinique est tout à fait évocateur
d'endocardite d'Osler.
On pratiquera, avant toute antibiothérapie, 9 à 12 hémocultures réparties sur 48 heures à l'occasion des pics
fébriles ou des frissons. Elles permettent l'identification des germes et l'antibiogramme.

Dans l'histoire de la maladie de cette patiente un(des) élément(s) important(s) devrai(en)t faire
évoquer le diagnostic :
A - L'anorexie mentale
B - L'apparition du souffle diastolique
C - L'existence de palpitations
D - La perte de poids importante
E - La notion de soins dentaires récents
Bonne(s) réponse(s) : B E

La porte d'entrée dentaire est la plus fréquente, le germe est alors le plus souvent un streptocoque non
groupable. Les soins dentaires doivent se faire sous couverture antibiotique chez les valvulaires.

473
Exclusivement sur DOC - DZ : www.doc-dz.com NADJI 85
RESIDANAT EN POCHE TOME II
Cas Clinique en QCM

L'échocardiogramme peut révéler dans ce cas précis :


A - Un fluttering systolique des valves mitrales
B - Des végétations sur les valves aortiques
C - Un dédoublement des parois aortiques
D - Une akinésie antéroseptoapicale
E - Aucun de ces signes
Bonne(s) réponse(s) : B

L'absence de végétation n'élimine pas le diagnostic. Elles apparaissent comme des échos anormaux ne
limitant pas les mouvements des valves en TM, en bidimensionnel elles sont attenantes aux valves et on peut
apprécier le nombre, la localisation, leur caractère pédiculé.
A - Le fluttering est diastolique.

Quelle est votre hypothèse diagnostique :


A - Péricardite aiguë
B - Rupture du sinus de valsalva
C - Endocardite bactérienne
D - Rechute de RM
E - Dissection aortique
Bonne(s) réponse(s) : C

Sans commentaire.

Quelle(s) thérapeutique(s) immédiate(s) doit-on instituer dans les 12 premières heures ?


A - Remplacement valvulaire aortique
B - Traitement digitalo diurétique
C - Antibiothérapie large spectre par voie orale
D - Pénicilline IV forte dose
E - Traitement vasodilatateur
Bonne(s) réponse(s) : B

Il faut traiter d'urgence l'insuffisance cardiaque, en sachant que, si elle est rebelle au traitement médical, une
indication chirurgicale sera posée. L'antibiothérapie sera entreprise une fois les hémocultures faites, elle est
parentérale par voie veineuse, sur perfusion à l'aiguille, associant deux antibiotiques bactéricides, des
synergiques à visée antistreptococciques (pénicilline G et aminosides) pendant 6 semaines au moins.

Dans le cadre du bilan étiologique l'(les) examen(s) suivant(s) doi(ven)t être pratiqué(s) :
A - Sérodiagnostic de mycoplasme
B - Radiographies des arcades dentaires
C - Urographie intraveineuse
D - Examen ORL
E - Réaction de Bordet-Wassermann
Bonne(s) réponse(s) : B D

Tout foyer infectieux doit être éradiqué.

En dehors du traitement symptomatique, un traitement étiologique par antibiothérapie doit être


institué, la(les) modalité(s) à respecter est(sont) :
A - Injection intramusculaire biquotidienne de pénicilline G
B - Perfusion continue de pénicilline G
C - Association de Pénicilline IV et d'un aminoside IV ou IM
D - La durée habituelle du traitement est de 15 à 22 jours
E - La posologie de la pénicilline G est toujours inférieure à 30 M UI/jour
Bonne(s) réponse(s) : C

Sans commentaire.

474
Exclusivement sur DOC - DZ : www.doc-dz.com NADJI 85
RESIDANAT EN POCHE TOME II
Cas Clinique en QCM
Un homme de 40 ans est hospitalisé en urgence pour une augmentation de volume douloureuse du membre
inférieur gauche. Il a été opéré 10 jours auparavant d'une hernie inguinale droite dont les suites immédiates
avaient été simples. Il n'avait pas été jugé utile de prescrire un traitement anticoagulant préventif. Les
examens complémentaires demandés confirmeront la phlébite ilio-cave suspectée cliniquement.

Parmi les propositions, suivantes, une ou plusieurs sont exactes. Laquelle(lesquelles) ?


A - Les thromboses veineuses sont plus fréquentes au membre inférieur gauche qu'au membre
inférieur droit
B - La thrombo-phlébite est considérée comme le stade initial, généralement inapparent, de la
phlébite
C - Une phlébite iliaque peut s'accompagner d'une disparition des pouls périphériques
D - La gangrène est une complication possible des phlébites
E - Le signe de Homans est particulièrement net dans la phlébite iliaque
Bonne(s) réponse(s) : A C D

A - C'est le syndrome de Cockett, compression de la veine illiaque gauche par l'artère illaque droite.
CD - Concernent la phlématia coeruléa : phlébite avec ischémie artérielle aiguë par compression.
Gamma : caractérise la phlébite surale.
B - La thrombo-phlébite est au contraire le stade constitué avec organisation du caillot et réaction
inflammatoire de la paroi au contraire de la phlébothrombose qui est le stade initial avec caillot partiellement
flottant dans le courant sanguin.

En ce qui concerne la phlébite iliaque, une ou plusieurs des affirmations suivantes sont justes.
Laquelle(lesquelles) ?
A - Elle complique obligatoirement une thrombose surale sous-jacente
B - Elle ne s'accompagne pas d'oedème au stade de thrombose non adhésive
C - Elle s'explore au mieux par une phlébographie rétrograde
D - C'est la localisation habituelle des phlégmatia cerulea
E - La circulation veineuse collatérale sus-pubienne permet son diagnostic rétrospectif
Bonne(s) réponse(s) : B D E

C - La phlébographie se fait à partir d'une veine du dos du pied.


B - Si la thrombose n'est pas adhésive, il n'y a pas de gêne au retour veineux et donc pas d'oedème.

Lorsqu'on suspecte une phlébite iliocave, un ou plusieurs des examens suivants permettent de
confirmer le diagnostic. Lequel(lesquels) ?
A - Doppler
B - Echotomographie abdomino-pelvienne
C - Ilio-portographie
D - Mesure de la pression transcutanée en oxygène
E - Phlébographie au fil de l'eau
Bonne(s) réponse(s) : A B E

E - C'est le meilleur examen. Elle doit être bilatérale avec cavographie : elle fait le diagnostic (non
opacification d'un segment veineux, produit de contraste moulant le caillot), permet le bilan : importance de
l'obstruction, son étendue, présence d'un caillot dans la veine cave. Le doppler non invasif est réalisable au lit
mais les faux négatifs sont fréquents (25 %).

Parmi les affirmations suivantes concernant l'interruption partielle de la veine cave inférieure, une
ou plusieurs sont fausses. Laquelle(lesquelles) ?
A - Elle nécessite toujours un abord abdominal
B - Elle est indiquée en cas de phlébite surale adhésive
C - Elle protège efficacement contre l'embolie pulmonaire massive
D - Elle se place généralement immédiatement au-dessous des veines rénales
E - Elle entraîne obligatoirement un oedème important des deux membres inférieurs
Bonne(s) réponse(s) : A B E

Les indications classiques sont les thrombus flottants des gros troncs veineux : veine fémorale iliaque ou cave
(l'indication est plus ou moins systématique selon les équipes), les contre-indications au traitement
anticoagulant, l'embolie pulmonaire malgré un traitement anticoagulant efficace.
La mise en place de certains filtres peut se faire par voie jugulaire.

475
Exclusivement sur DOC - DZ : www.doc-dz.com NADJI 85
RESIDANAT EN POCHE TOME II
Cas Clinique en QCM
Monsieur R. âgé de 48 ans est hypertendu depuis plusieurs années, hypertension mal équilibrée, le malade
ne respectant pas de façon précise le régime alimentaire et le traitement qui lui ont été prescrits. Monsieur R.
n'est pas connu par ailleurs comme insuffisant coronarien. Il a ressenti brutalement une douleur thoracique
antérieure intense, d'allure déchirante, évoluant par paroxysmes : cette douleur a irradié rapidement vers la
région dorsale, la région lombaire et l'abdomen.
A l'examen clinique : rythme cardiaque à 90/mn, régulier ; l'auscultation montre l'existence d'un léger
frottement péricardique : la pression artérielle est à 160/100 mmHg.
L'examen général montre, par ailleurs, l'abolition des battements artériels au niveau de l'artère fémorale droite
chez ce malade dont la circulation artérielle des membres inférieurs était auparavant normale. A
l'électrocardiogramme, tracé d'hypertrophie ventriculaire gauche déjà connue avec troubles de la
repolarisation ventriculaire : absence de signes de nécrose myocardique. Les enzymes sont demeurées
normales. Le diagnostic évoqué est celui de dissection aortique.

Quel est, parmi les signes suivants présentés par ce patient, celui qui vous paraît le plus
évocateur du diagnostic ?
A - L'absence de souffle cardiaque
B - L'abolition du pouls fémoral droit
C - L'existence d'une hypertension artérielle
D - L'existence de troubles ECG de repolarisation ventriculaire
E - L'absence de signes ECG ou biologiques de nécrose myocardique
Bonne(s) réponse(s) : B

Elle évoque une atteinte de l'artère fémorale droite (dissection type I ou III de De Bakey). On distingue trois
types :
1 - (60 % des cas) Porte d'entrée au dessus des sigmoïdes,.aortiques, atteinte de l'aorte ascendante
horizontale et abdominale jusqu'aux artères iliaques. Les gros vaisseaux du cou et les coronaires peuvent être
touchés.
2 - (10 % des cas). Forme limitée à l'atteinte entre les sigmoïdes et l'artère sous-clavière gauche
3 - Atteinte de l'artère sous-clavière gauche jusqu'àu carrefour iliaque.
Certains distinguent un type IV qui va de l'artère sous-clavière gauche jusqu'aux artères rénales.

L'existence d'un souffle diastolique est un signe classique et précieux pour le diagnostic de
dissection aortique. A ce sujet, il est exact que :
A - Le souffle diastolique est constant au cours de l'évolution
B - Il est le plus souvent dû au prolapsus d'une sigmoïde désinsérée
C - Il peut être dû à la dilatation de l'anneau aortique par hématome pariétal
D - Il peut être lié à une I.A.O. point de départ de la dissection
E - Son absence permet de rejeter le diagnostic
Bonne(s) réponse(s) : B C

Le souffle diastolique n'est retrouvé qu'au cours des types I et II où il existe une dissection de l'anneau
entraînant soit la bascule d'une sigmoïde, soit la dilatation de l'anneau par hématome pariétal.

L'électrocardiogramme de la dissection aortique aiguë :


A - Est le plus souvent strictement normal
B - Peut montrer une onde Q de nécrose par dissection coronarienne
C - Peut montrer une hypertrophie ventriculaire gauche isolée
D - Montre parfois une hypertrophie ventriculaire droite
E - Montre souvent un bloc complet de la branche gauche
Bonne(s) réponse(s) : A B C

B - L'infarctus du myocarde par dissection coronaire se retrouve dans 10 % des cas.


C - L'hypertrophie ventriculaire gauche due aux antécédents d'hypertension est souvent retrouvée.

Lorsqu'une aortographie est pratiquée, elle peut montrer plusieurs aspects angiographiques. Quel
est le plus caractéristique ?
A - Opacification de l'aorte et d'un faux chenal
B - Opacification de l'aorte seule, comprimée par le faux chenal
C - Epaississement de la paroi aortique
D - Opacification du faux chenal seul
E - Constatation d'une insuffisance aortique
Bonne(s) réponse(s) : A

L'aortographie visualise typiquement une image en double manchon avec une ligne claire correspondant à la
zone disséquée.
L'aorte est parfois comprimée par le faux chenal. La densité entre l'aorte et le manchon n'est parfois pas la
même ; ils ne sont parfois pas opacifiés dans le même temps. On retrouve enfin des signes de régurgitation
dans le VG en cas d'IA, une absence de visualisation d'une colatérale.

476
Exclusivement sur DOC - DZ : www.doc-dz.com NADJI 85
RESIDANAT EN POCHE TOME II
Cas Clinique en QCM

Parmi les évolutions défavorables suivantes, quelle est la plus fréquente ?


A - La mort par ischémie viscérale
B - La mort par insuffisance aortique massive et IVG
C - La rupture de l'aorte dans les premières semaines
D - La mort au cours d'une poussée hypertensive
E - La mort par infarctus du myocarde
Bonne(s) réponse(s) : C

En l'absence de traitement :
- 25 % de décès dans les 24 premières heures
- 66 % de décès au cours de la première semaine
- 80 % de décès au bout d'un mois.
(95 % dans les types I et II 35 % dans les types III)

Parmi les affections suivantes, laquelle(lesquelles) prédispose(nt)-elle (s) à la dissection aortique ?


A - Hypertension artérielle
B - Rétrécissement aortique athéromateux
C - Syndrome de Marfan
D - Spondylarthrite ankylosante
E - Elastorrhexie systématisée
Bonne(s) réponse(s) : A C E

L'hypertension artérielle est retrouvée dans 75 % des cas.


C.E - Sont des maladies du tissu élastique.
Les autres formes étiologiques concernent : les cardiopathies congénitales (coarctation de l'aorte, bicuspidie
aortique, CIV) la grossesse, l'aortographie, les canulations artérielles pour circulation extra-corporelle.

Le traitement chirurgical en phase aiguë de la dissection :


A - Ne doit intervenir qu'après contrôle de la pression artérielle
B - S'adresse surtout aux dissections de l'aorte descendante (III)
C - Comporte actuellement une mortalité faible (environ 5 %)
D - Consiste à créer un orifice de réentrée
E - Consiste à fermer l'orifice d'entrée et oblitérer le plan de dissection
Bonne(s) réponse(s) : A E

Les indications chirurgicales principales sont les types I et II. La mortalité opératoire est de 25 à 30 %.
Le traitement consiste en un remplacement de l'aorte disséquée par une prothèse en dacron. On utilise
parfois des colles biologiques.

Femme de 72 ans immobilisée dans un plâtre cruro-pédieux pour fracture de Dupuytren droite depuis 10
jours.
Apparition d'un oedème des orteils, de douleurs sourdes du mollet et d'un état fébrile à 38° avec pouls rapide
(110 par minute). La pression artérielle ne s'est pas modifiée (140/90 aux membres supérieurs). Brusquement
elle se plaint d'une douleur à type de point de côté basithoracique gauche avec sensation d'angoisse. Elle est
polypnéique. Le diagnostic de maladie thrombo-embolique est évoqué.

En faveur de ce diagnostic, vous retenez :


A - La douleur basi-thoracique brutale
B - La tachycardie
C - L'absence de modification de la pression artérielle
D - L'immobilisation plâtrée
E - L'oedème des orteils
Bonne(s) réponse(s) : A B D E

Sans commentaire.

477
Exclusivement sur DOC - DZ : www.doc-dz.com NADJI 85
RESIDANAT EN POCHE TOME II
Cas Clinique en QCM

Pour confirmer ce diagnostic vous pouvez demander :


A - La radiographie pulmonaire
B - L'électrocardiogramme
C - La mesure des gaz du sang
D - La scintigraphie pulmonaire
E - La phlébo-cavographie des membres inférieurs
Bonne(s) réponse(s) : A B C D E

A - Est souvent normal.


On recherche une ascension de la coupole diapharagmatique, une atélectasie en bande, un épanchement
pleural, une amputation d'une branche de l'artère pulmonaire, une hyperclarté.
B - Est anormal dans la plupart des embolies massives. On peut trouver une tachycardie sinusale, des
troubles du rythme supraventriculaire, une rotation axiale droite avec aspect S1Q3, un bloc de branche droite,
des troubles de la repolarisation dans les précordiales droites. Tous ces signes sont fugaces.
C - Montre un effet shunt.
D - Normale, elle élimine l'embolie pulmonaire mais les faux positifs sont nombreux.
E - Elle permet le diagnostic de phlébite, d'apprécier l'étendue de la thrombose, recherche l'existence de
thrombus cave.

Parmi les différents facteurs étiologiques de phlébite, lequel(esquels) est(sont) à retenir ?


A - L'âge
B - L'immobilisation plâtrée
C - Le type de fracture
D - Le côté fracturé
E - La date d'apparition de la symptomatologie
Bonne(s) réponse(s) : B D

Sans commentaire.

Cette complication aurait probablement été évitée par :


A - Calciparinothérapie
B - Traitement antiinflammatoire
C - Traitement vasodilatateur
D - Antivitamines K
E - Antiagrégants plaquettaires
Bonne(s) réponse(s) : A

Elle est systématique à dose iso-coagulante.

La gravité de l'embolie pulmonaire s'évalue avant tout par :


A - La capnie
B - La radiographie pulmonaire
C - L'ECG
D - L'angiographie pulmonaire
E - L'élévation des transaminases
Bonne(s) réponse(s) : D

Elle permet d'apprécier le degré d'obstruction : obstruction < 40 % ; embolie peu importante, obstruction
comprise entre 40 et 59 % ; embolie sévère, obstruction > 60 % ; embolie massive (index de Miller).
L'hypocapnie importante et la présence des signes ECG sont également en faveur d'une embolie pulmonaire
grave.

Parmi les différentes mesures thérapeutiques suivantes, laquelle(lesquelles) est(sont)


indispensable(s) dans l'immédiat ?
A - Héparinothérapie
B - Ablation du plâtre
C - Fibrinolytiques
D - Antibiothérapie
E - Antivitamines K
Bonne(s) réponse(s) : A B

A - Héparinothérapie par voie veineuse à dose efficace 5 mg/kg/j.


C - Se discuterait dans les formes graves avec embolie massive si vues tôt.

478
Exclusivement sur DOC - DZ : www.doc-dz.com NADJI 85
RESIDANAT EN POCHE TOME II
Cas Clinique en QCM

Chez cette patiente, l'indication d'une interruption partielle de la veine cave inférieure par un
dispositif endocave peut être posée sur quel critère ?
A - L'existence d'un caillot iliaque
B - L'état clinique
C - L'étiologie de la phlébite
D - L'âge de la malade
E - Aucune de ces propositions
Bonne(s) réponse(s) : E

Cette indication se discuterait en cas de caillot flottant cave, de contre-indication au traitement anticoagulant,
de récidive d'embolie pulmonaire malgré un traitement anticoagulant à dose efficace.

Une jeune femme de 35 ans, et de 75 kilos pour 1 m 68, mère de quatre enfants, présente trois jours après
son dernier accouchement une phlébite profonde du membre inférieur droit sur un terrain variqueux marqué,
séquelle des grossesses précédentes. Il s'agit de sa première phlébite profonde mais à deux reprises, en
dehors de ses grossesses, dans le passé, la patiente a présenté des phlébites superficielles traitées par des
antiinflammatoires locaux.
Au moment du diagnostic de la phlébite profonde, il n'y a pas de signes d'embolie pulmonaire. Vous décidez
de traiter cette patiente par héparinothérapie.

Avant la mise en route du traitement, quelle investigation biologique allez-vous pratiquer ?


A - Une mesure du temps de coagulation en tube + temps de saignement
B - Une mesure du temps de céphaline activée + temps de Howell
C - Une mesure du temps de Quick + du temps de céphaline activée + numération plaquettaire
D - Un thromboélastogramme
E - Une recherche d'hyperagrégabilité plaquettaire
Bonne(s) réponse(s) : C

Il faut un bilan de coagulation systématique avant tout traitement anticoagulant, avec numération plaquettaire
de référence, si l'héparine est utilisée.

Parmi les modalités d'administration suivante, laquelle allez-vous choisir pour mettre en route le
traitement anticoagulant ?
A - L'héparine IV à raison de 7 500 UI toutes les 6 heures
B - L'héparine IV à en perfusion continue à la seringue électrique à la dose de 150 UI/kg/24 h
C - L'héparine sous cutanée à la dose de 0,2 ml toutes les 8 heures
D - L'héparine IV en perfusion continue à la seringue électrique à la dose de 500 UI/kg/24 h
E - L'héparine sous cutanée à la dose de 0,3 ml toutes les 12 heures
Bonne(s) réponse(s) : D

Sans commentaire.

Parmi les propositions suivantes concernant l'héparine, laquelle(lesquelles) est(sont) juste(s) ?


A - A dose anticoagulante elle inhibe uniquement le facteur Stuart (Xa)
B - Le délai d'action par voie IV est de l'ordre de 30 minutes
C - Elle se fixe sur l'antithrombine III
D - Elle traverse la barrière placentaire
E - Sa demi-vie biologique varie entre 1 et 2 heures
Bonne(s) réponse(s) : C E

L'héparine se fixe sur l'antithrombine III et accélère la neutralisation de la thrombine. Elle neutralise également
le facteur Xa.

La surveillance de l'efficacité biologique du traitement par l'héparine va se faire par :


A - Temps de Quick et héparinémie
B - Temps de Quick et temps de Howell
C - Temps de céphaline activée et héparinémie
D - Dosage du facteur Xa
E - Dosage de l'antithrombine III
Bonne(s) réponse(s) : C

Le temps de céphaline activée apprécie la voie intrinsèque de la coagulation, facteurs XII, XI, IX, VIII ainsi que
le tronc commun : V, X, II et I. C'est le test le plus courant pour la surveillance d'un traitement à l'héparine. Le
test de Howell est un test de coagulation global et le temps de Quick explore la voie intrinsèque facteur VII, X,
V, II, I.

479
Exclusivement sur DOC - DZ : www.doc-dz.com NADJI 85
RESIDANAT EN POCHE TOME II
Cas Clinique en QCM

Parmi les complications suivantes, quelle(s) est(sont) celle(s) qui peut(peuvent) être attribuée(s) à
l'héparinothérapie ?
A - Anémie macrocytaire
B - Thrombocytopénie
C - Fibrinolyse aiguë
D - Insuffisance rénale
E - Cholestase.
Bonne(s) réponse(s) : B

On distingue une thrombopénie précoce (au 3e jour), modérée, sans signe clinique, régressant spontanément
en quelques jours et une thrombopénie plus tardive (10e jour) de mécanisme immunologique avec risque de
thrombose, imposant le passage à l'héparine de bas poids moléculaire ou aux anti vitamines K.

Madame T., à 70 ans, est hospitalisée pour lipothymies et syncopes récidivantes depuis 3 mois. Il n existe pas
d'autres antécédents cardiovasculaires et la patiente ne prend aucun médicament. Un souffle au coeur est
connu depuis environ 10 ans. La première syncope est survenue il y à 5 mois, alors qu'elle montait
rapidement un escalier. Par la suite, à la marche rapide, plusieurs malaises lipothymiques sont survenus. Ce
jour, en courant après un chat, elle perd connaissance et chute lourdement et se fait une plaie de l'arcade
sourcilière. A l'examen, cette patiente est en bon état général, elle pèse 54 kg pour 1,58 m, elle est très active
et vive d'esprit. La pression artérielle est à 110/85 mmHg. La fréquence cardiaque à 70/mn régulière.
L'auscultation cardiaque révèle un souffle mésosystolique râpeux maximum au 2ème espace intercostal droit,
3/6, irradiant vers les 2 carotides. Il existe également un souffle diastolique 1/6 le long du bord gauche du
sternum. Le B1 et le B2 sont abolis au 2ème espace intercostal droit. Il existe un B4. Le reste de l'auscultation
cardiaque est normal. Les artères périphériques sont perçues. L'auscultation pulmonaire est normale. La
radiographie thoracique montre un coeur de volume normal (ICT 0,53) et une transparence parenchymateuse
normale. L'ECG révèle un rythme sinusal régulier avec un espace PR à 0,24 s, un axe de QRS à 0 degré, un
bloc de branche droite complet (largeur du QRS 0,12 s). Il n'existe aucune anomalie biologique. L'ensemble
de ces données permet de conclure à un rétrécissement aortique.

Le caractère serré du rétrécissement aortique peut-être évoqué sur :


A- Le pincement de la différentielle
B - L'irradiation du souffle dans les deux carotides
C - L'abolition du B2
D - L'intensité du souffle systolique
E - Le caractère râpeux du souffle systolique
Bonne(s) réponse(s) : A C

A l'examen clinique, en faveur du caractère serré on retient :


- un B2 aboli à la base ou dédoublé paradoxalement
- un B1 diminué à la pointe
- un maximum télésystolique du souffle
- un pincement de la différentielle de la TA.

Les conditions de survenue de la syncope sont en faveur d'une de ces étiologies. Laquelle ?
A - Bloc auriculoventriculaire paroxystique
B - Accès de fibrillation auriculaire rapide
C - Retrécissement aortique serré
D - Angor vasospastique
E - Torsades de pointe
Bonne(s) réponse(s) : C

La survenue à l'effort de ces syncopes est tout à fait évocatrice.


L'existence d'un obstacle à l'éjection rend difficile l'augmentation physiologique du débit cardiaque à l'effort
d'où apparition des symptômes à l'effort.

Une estimation indirecte de la sévérité de la sténose aortique peut-être envisagée sur


l'échocardiogramme d'après :
A - La dilatation de l'anneau aortique
B - L'hypertrophie concentrique du ventricule gauche
C - Le fluttering de la valve mitrale
D - La fermeture prématurée en systole des valves aortiques
E - La dilatation auriculaire gauche
Bonne(s) réponse(s) : B

L'échographie cardiaque permet de faire le diagnostic de rétrécissement aortique en visualisant des


sigmoïdes épaissies, parfois calcifiées, à l'ouverture diminuée. Elle apprécie le retentissement sur le ventricule
gauche (degré d'hypertrophie).
Le doppler continu permet de calculer le gradient et la surface aortique avec une bonne corrélation à l'étude
hémodynamique.

480
Exclusivement sur DOC - DZ : www.doc-dz.com NADJI 85
RESIDANAT EN POCHE TOME II
Cas Clinique en QCM

L'exploration hémodynamique confirme l'existence d'un gradient ventriculoaortique de 80 mmHg.


Le traitement envisagé doit être :
A - Traitement digitalo diurétique
B - Mise en place d'un pace maker
C - Remplacement valvulaire aortique
D - Traitement vasodilatateur artériel
E - Bêta-bloqueurs
Bonne(s) réponse(s) : C

L'indication chirurgicale doit être portée devant ce rétrécissement aortique serré.


La survenue de syncope est un indice de gravité. Les traitements digitaliques et vasodilatateurs ne sont pas
souhaitables car augmentent le gradient VG-aorte.

Un examen complémentaire est justifié en fonction des données de l'électrocardiogramme.


Lequel ?
A - Phonomécanogramme avec nitrite d'amyle
B - Scintigraphie myocardique
C - Epreuve d'effort
D - Exploration endocavitaire avec enregistrement du faisceau de His
E - Gamma angiocardiographie à l'effort
Bonne(s) réponse(s) : D

En effet la patiente présente des troubles de la conduction avec BAV I et bloc de branche droit. Ces troubles
sont soit dûs à une coulée calcaire sur le septum, soit à un processus dégénératif (maladie de Lenègre) et il
existe un risque de bloc auriculo-ventriculaire postopératoire.

Un homme de 68 ans, retraité mais encore actif, souffrant de claudication intermittente depuis 3 ans, vous
appelle parce que depuis 4 jours son gros orteil gauche est extrêmement douloureux et que la veille est
apparue sur l'extrémité une plaque noirâtre. La claudication intermittente était, jusqu'à présent peu invalidante,
autorisant 600 m à vitesse moyenne. Vous l'avez soigné plusieurs hivers pour des épisodes de bronchite,
largement entretenus par un tabagisme (1 paquet 1/2 de cigarettes par jour). L'hiver précédent, il a fait 2
accidents cérébraux intermittents de quelques minutes avec paralysie du bras droit et aphasie, dont il n'a
gardé aucune séquelle. Lorsque vous arrivez chez lui, il vous montre un gros orteil gauche plus rouge que les
autres et présentant au bord interne une plaque noire de 3 cm. Vous constatez que l'examen des pouls s'est
modifié par rapport à vos constatations antérieures : à gauche, le pouls fémoral que vous perceviez
antérieurement a disparu.
Un bilan biologique effectué 2 mois auparavant montrait que tous les examens étaient normaux : numération
formule sanguine, glycémie, créatinine, cholestérol, triglycérides, uricémie.

On peut affirmer qu'il s'agit d'un début de gangrène et non d'un banal traumatisme ou d'une crise
de goutte en raison de :
A - Début brutal
B - Douleur récente
C - Disparition récente d'un pouls
D - Plaque noire
E - Acide urique normal antérieurement
Bonne(s) réponse(s) : C D

Sans commentaire.

On peut conclure avec certitude à une thrombose totale de :


A - L'Aorte
B - L'iliaque gauche
C - La fémorale superficielle gauche
D - La poplitée gauche
E - Les artères de jambe gauche
Bonne(s) réponse(s) : B

Sans commentaire.

481
Exclusivement sur DOC - DZ : www.doc-dz.com NADJI 85
RESIDANAT EN POCHE TOME II
Cas Clinique en QCM

Parmi les décisions suivantes, laquelle donne les meilleures chances au malade ?
A - Prise de rendez-vous pour bilan Doppler
B - Traitement à domicile par vasodilatateurs IM
C - Traitement à domicile par vasodilatateurs oraux et Calciparine®
D - Hospitalisation rapide en chirurgie vasculaire
E - Hospitalisation pour amputation d'orteil
Bonne(s) réponse(s) : D

L'ischémie aiguë du nombre inférieur est une urgence vasculaire.

Quel est le risque le plus fréquent auquel expose une amputation d'orteil d'emblée ?
A - Décompensation d'un diabète
B - Septicémie
C - Gangrène extensive
D - Infarctus du myocarde
E - Hémiplégie
Bonne(s) réponse(s) : C

Sans commentaire.

En attendant l'hospitalisation que vous n'avez obtenue que dans 3 jours, vous devez prescrire à
domicile :
A - Calciparine® sous cutanée
B - Antivitamines K per os
C - Vasodilatateur oral
D - Antalgiques banals
E - Antibiotiques à large spectre
Bonne(s) réponse(s) : A C D

Sans commentaire.

Vous avez la possibilité d'obtenir un examen Doppler le lendemain. Parmi les renseignements
suivants, lequel sera le plus important pour la décision thérapeutique ?
A - Limites de la thrombose
B - Etat des carotides et vertébrales
C - Perméabilité du réseau artériel en aval de la thrombose
D - Index systolique de cheville
E - Recherche d'un anévrisme
Bonne(s) réponse(s) : B

Il peut constituer une contre-indication à l'anesthésie générale d'autant que le patient a déjà fait des accidents
ischémiques transitoires.

Un patient de 45 ans, porteur de facteurs de risque cardiovasculaire, fait brutalement à l'effort une douleur
thoracique avec chute de la tension artérielle qui l'amène en unité de soins intensifs. Sur l'ECG, il apparait par
la suite une onde Q en D2 D3 VF et de V1 à V3. On note également sur l'ECG une extrasystolie.
Du point de vue biologique, on constate une élévation des CPK-MB.

Indiquer la proposition inexacte. La chute de la tension artérielle chez un tel patient est :
A - Constante
B - Retardée
C - Prédominante sur la diastolique
D - Caractérisée par un pincement de la différentielle
E - Associée à un train fébrile
Bonne(s) réponse(s) : C

La chute tensionnelle est presque constante portant sur la systolique.

482
Exclusivement sur DOC - DZ : www.doc-dz.com NADJI 85
RESIDANAT EN POCHE TOME II
Cas Clinique en QCM

Une seule proposition exacte.


La localisation de la nécrose myocardique est chez ce patient :
A - Inférieure
B - Postérobasale
C - Apicale
D - Antéroseptale
E - Septale profonde
Bonne(s) réponse(s) : E

Le territoire septal profond associe l'atteinte antéro-septale et inférieure.

Indidez la(les) proposition(s) inexacte(s).


L'élévation des CPK observée chez ce patient est caractérisée par une fraction MB :
A - Supérieure à 5 %
B - Existe dès la première heure
C - Persiste le 8ème jour de l'infarctus
D - Est caractéristique d'une nécrose myocardique
E - S'accompagne d'une élévation de la vitesse de sédimentation
Bonne(s) réponse(s) : B C

Les CPK s'élèvent à partir de la 6e heure et se normalisent à partir de 48-72 heures. La fraction MB est
spécifique du myocarde.
L'importance de l'élévation est liée à l'étendue de la nécrose.

Indiquez la proposition inexacte.


L'extrasystolie enregistrée à la phase aiguë de cet infarctus :
A - Peut naître à l'étage auriculaire
B - Naît le plus souvent à l'étage ventriculaire
C - Peut être répétitive (tachycardie ventriculaire)
D - Ne peut pas déclencher de fibrillation ventriculaire
E - Est sensible à la xylocaine
Bonne(s) réponse(s) : D

Les troubles du rythme sont fréquents dans les trois premières heures. Les extrasystoles ventriculaires sont
fréquentes (70 %). Les tachycardies ventriculaires sont vues dans 10 à 30 % des cas. Les troubles du rythme
sont responsables de 30 à 40 % des décès précoces.

Un homme de 58 ans, en bon état général, vient consulter pour une claudication intermittente serrée avec
périmètre de marche inférieur à 100 mètres. Les troubles, apparus depuis plusieurs mois, se sont aggravés
progressivement.
Dans les antécédents, on note un tabagisme à 40 paquets année. A l'examen, le pouls fémoral gauche est
absent, le pouls fémoral droit d'amplitude très diminuée ; il existe un souffle systolique le long des axes
iliaques droits jusqu'au niveau de la fémorale. Les pouls poplités et distaux sont abolis des deux côtés.

Parmi les troubles cliniques suivants, indiquez celui qui est pathognomique de l'existence d'un
syndrome de Leriche :
A - Claudication intermittente serrée des membres inférieurs
B - Troubles trophiques distaux des deux membres inférieurs
C - Impuissance par impossibilité d'érection stable
D - Douleurs de décubitus
E - Claudication intermittente de la fesse
Bonne(s) réponse(s) : C

Le syndrome de Leriche est dû à l'atteinte de la bifurcation aortique. Cliniquement, il existe une claudication
intermittente de fesse ou de cuisse bilatérale, une impuissance. A l'examen, on note l'abolition des deux pouls
fémoraux.

483
Exclusivement sur DOC - DZ : www.doc-dz.com NADJI 85
RESIDANAT EN POCHE TOME II
Cas Clinique en QCM

Le premier examen que vous demandez chez ce malade est la vélocimétrie Doppler des
membres inférieurs. Indiquez, parmi les modifications suivantes des tracés, celle qui apparaît le
plus précocement au cours de l'évolution de l'artériopathie :
A - Diminution de hauteur du pic systolique
B - Elargissement de l'onde systolique
C - Disparition de l'onde de reflux diastolique
D - Aplatissement de l'onde systolique
E - Ressaut sur la portion descendante de la courbe systolique
Bonne(s) réponse(s) : C

En cas de surcharge pariétale, le doppler montre une disparition du ressaut sur la portion descendante de la
courbe.
En cas de sténose, on note une disparition de l'onde négative de reflux diastolique, une accélération de la
vitesse systolique, avec courbe plus empatée. La sténose est significative si on note une démodulation du
signal en aval.

Vous décidez de faire effectuer une opacification artérielle. Parmi les modalités suivantes,
indiquez celle(s) que vous retenez :
A - Aortographie par voie translombaire
B - Aortographie par cathétérisme fémoral rétrograde selon Seldinger
C - Artériographie fémorale bilatérale
D - Anglo numérique par voie intraveineuse
E - Artériographie par injection fémorale à contre-courant
Bonne(s) réponse(s) : A

B - Est à éviter devant la présence de souffle systolique sur les axes iliaques.
D - N'est pas assez performante pour la visualisation des artères périphériques.
A - Il faudra éliminer un anévrysme de l'aorte abdominale.

L'opacification vasculaire a montré une thrombose étendue des axes iliaques gauches et une
surcharge majeure avec sténoses étagées sur les axes iliaques droits. Parmi les conduites
thérapeutiques suivantes, laquelle(lesquelles) retenez-vous ?
A - Pontage aorto-bifémoral
B - Sympathectomie lombaire bilatérale isolée
C - Pontage axillo-bifémoral
D - Angioplastie endoluminale des axes iliaques gauches
E - Pontage aorto-fémoral gauche
Bonne(s) réponse(s) : A

Il s'agit d'une atteinte bilatérale étagée et on retiendra donc le pontage aorto-bifémoral. Il se fera au moyen
d'une prothèse en dacron. La mortalité opératoire est de 2 % et la perméabilité à 5 ans de 80 %.

Une femme de 63 ans, présentant des antécédents lointains de rhumatisme articulaire aigu, a bénéficié
jusque-là d'un état de santé relativement satisfaisant et n'a pas consulté de médecin depuis des années. Elle
a vu apparaître depuis un mois des oedèmes des membres inférieurs et une augmentation de volume de
l'abdomen. L'état général est médiocre avec asthénie et anorexie. L'interrogatoire révèle une dyspnée d'effort
ancienne et des douleurs de l'hypochondre droit. L'examen clinique permet de conclure à une maladie mitrale
et à une insuffisance tricuspide. Vous faites hospitaliser la patiente pour bilan complémentaire et traitement.

L'auscultation cardiaque de cette patiente comporte :


A - Un souffle diastolique le long du bord gauche du sternum
B - Un roulement diastolique de pointe mieux perçu en décubitus latéral gauche
C - Un souffle systolique xiphoïdien se majorant en inspiration
D - Un claquement d'ouverture au foyer tricuspidien
E - Un souffle éjectionnel au 2ème intercostal droit
Bonne(s) réponse(s) : B C

Sans commentaire.

484
Exclusivement sur DOC - DZ : www.doc-dz.com NADJI 85
RESIDANAT EN POCHE TOME II
Cas Clinique en QCM

L'examen clinique recherche des signes périphériques en rapport avec l'atteinte valvulaire et la
défaillance ventriculaire droite.
On peut observer :
A - Une expansion systolique des jugulaires
B - Un élargissement de la tension artérielle différentielle
C - Une hépatomégalie pulsatile
D - Un pouls carotidien petit et retardé
E - Une ascite
Bonne(s) réponse(s) : A C E

A.C. - Se voient dans l'insuffisance tricuspidienne.

Le bilan biologique présente un certain nombre d'anomalies. Cochez celle(s) qui peu(vent) être
directement en rapport avec la défaillance cardiaque et ses conséquences viscérales :
A - Triglycérides à 2,40 mmoles/l
B - Taux de prothrombine à 50 %
C - Transaminases (SGPT) à 140 UI (normale inférieure à 30)
D - Urée à 1,10 g/l
E - Glycémie à jeun à 1,40 g/l
Bonne(s) réponse(s) : B C E

B.C. - Témoignent du retentissement sur le foie avec cytolyse et cholestase puis insuffisance hépato-cellulaire.
D - Traduit une insuffisance vraisemblablement fonctionnelle par hypovolémie efficace.

Un bilan hémodynamique et angiographique est réalisé après quinze jours de traitement digitalo-
diurétique et confirme le diagnostic clinique.
Ce bilan révèle :
A - Index cardiaque à 4 l/mn/m2
B - Aspect de ventricularisation de la courbe auriculaire droite inspiration
C - Gradient de 50 mmHg au retrait ventriculo-aortique
D - Pression capillaire pulmonaire moyenne à 25 mmHg avec télédiastolique ventriculaire
gauche à 10 mmHg
E - Pression télédiastolique ventriculaire droite à 15 mmHg
Bonne(s) réponse(s) : B D E

B - Est un signe d'insuffisance tricuspidienne. L'inspiration majore le retour veineux et augmente donc la fuite
avec égalisation des pressions auriculaires et ventriculaires.
D - On retrouve une HTAP (pression capillaire > 12 mm Hg) de type post-capillaire (absence de gradient entre
l'artère pulmonaire et le capillaire), ainsi qu'un gradient auriculoventriculaire (la pression capillaire étant le
reflet de la pression dans l'oreillette gauche).
E - Traduit l'insuffisance ventriculaire droite.

Monsieur X., âgé de 43 ans, est réveillé à 6 h du matin par une douleur brutale, à type d'étau comprimant le
thorax qui gagne rapidement les épaules et les deux bras. Ce directeur de banque, gros fumeur habituel,
habituellement normotendu, n'a pas d'antécédent particulier et n'avait jamais présenté auparavant la moindre
douleur thoracique. Il était depuis plusieurs semaines en proie à un surmenage et à des stress d'ordre
professionnel.
Le médecin, appelé à 7 h, constate que la TA est à 110/80 et le rythme cardiaque à 62 ; il administre par voie
veineuse 1 cg de morphine et l'hospitalisation immédiate en secteur de soins intensifs est décidée. A l'entrée,
à 8 h, la douleur s'est estompée mais la TA est à 8/6, le rythme cardiaque à 42/mn. Le patient est pâle et
couvert de sueurs, l'interne de garde confirme le diagnostic, qui s'avèrera exact, d'infarctus du myocarde
inaugural, de topographie postéro-inférieure, compliqué d'un état de choc vagotonique.

L'infarctus du myocarde représente la première manifestation d'une insuffisance coronarienne


(infarctus inaugural) :
A - Dans 59 % des cas
B - Dans 10 à 20 % des cas
C - Dans 40 à 60 % des cas
D - Dans 70 à 90 % des cas
E - Dans plus de 90 % des cas
Bonne(s) réponse(s) : C

Sans commentaire.

485
Exclusivement sur DOC - DZ : www.doc-dz.com NADJI 85
RESIDANAT EN POCHE TOME II
Cas Clinique en QCM

Arrivé en unité de soins intensifs à 8 h soit 2 h après le début de la douleur, le bilan biologique
montre :
A - Une élévation significative des CPK
B - Une élévation significative des CPK-MB
C - Une élévation significative des transaminases oxalo-acétiques
D - Une élévation significative de la LDH
E - Un bilan enzymatique normal
Bonne(s) réponse(s) : C

Les premières enzymes à s'élever sont les CPK vers la 6e heure. Certains dosent la myoglobine qui est
élevée dès la 3e heure. C'est un test sensible mais peu spécifique.

Le geste thérapeutique qui s'impose chez ce patient :


A - Le Propranolol (Avlocardyl) 4 cps/jour
B - Le Diltiazem (Tildiem®) 4 cp/jour
C - Une injection IV d'1/2 mg de sulfate d'Atropine®
D - Un remplissage vasculaire
E - Une perfusion IV continue d'Isuprel®
Bonne(s) réponse(s) : C

Le malaise vagal est fréquent dans les infarctus inférieurs, il réagit bien à l'Atropine®.

Les signes directs de l'infarctus postéro-inférieur sont trouvés :


A - Dans les dérivations D2, D3, VF
B - D1 VL, V5, V6
C - V1, V2, V3
D - D2, D3, VF, et V1, V2, V3
E - D2, D3, VF, V5, V6
Bonne(s) réponse(s) : A

B - Infarctus latéral.
C - Infarctus antéro-septal.
D - Infarctus septal profond.
E - Infarctus inféro-latéral.

L'ECG réalisé à la 3ème heure chez ce patient montre le plus probablement dans les dérivations
postéro-inférieures :
A - Un aspect de lésion sous-endocardique isolé
B - Une onde Q de nécrose et une onde T négative d'ischémie sous-épicardique
C - Une onde T négative d'ischémie sous-épicardique isolée
D - Une onde Q de nécrose et un profil d'ischémie lésion sous-épicardique
E - Un sus-décalage isolé du segment ST englobant l'onde T
Bonne(s) réponse(s) : E

L'anomalie la plus précoce est une ischémie sous endocardique : onde T ample, positive symétrique. Puis
apparait la lésion sous-épicardique : sus décalage de ST convexe vers le haut englobant l'onde T. Associée à
l'onde Q de nécrose qui apparait vers la 6e - 12e heuren, elle costitue l'onde en dôme de De Pardee. Ce sus-
décalage disparaît en quelques jours L'onde T se négative à la 48e heure.
L'onde Q de nécrose est le plus souvent définitive, l'onde T évolue de façon variable.

Un état de choc vagotonique s'accompagne généralement :


A - D'une réaction fébrile à 38°5
B - D'un frottement péricardique
C - De troubles digestifs à type de nausées et de vomissements
D - D'une élévation de la pression veineuse centrale
E - D'une extrasystolie ventriculaire
Bonne(s) réponse(s) : C

Il associe bradycardie-hypotension avec baisse de la pression veineuse centrale, nausées vomissements.

486
Exclusivement sur DOC - DZ : www.doc-dz.com NADJI 85
RESIDANAT EN POCHE TOME II
Cas Clinique en QCM
Cette patiente de 22 ans a accouché il y a quatre jours ; il sagissait dun premier enfant. L'accouchement s'est
déroulé sans problème. Vous la voyez pour point de côté gauche, dyspnée, angoisse, température à 38°C.
Le diagnostic retenu est celui d'embolie pulmonaire.

Quels sont parmi les éléments suivants, les 2 signes qui évoquent un infarctus pulmonaire ?
A - Hémoptysie
B - Image dense sous-pleurale
C - Hypoxie
D - VS accélérée
E - Signes électriques de coeur droit
Bonne(s) réponse(s) : A B

Sans commentaire.

Les éléments de gravité chez cette patiente se traduisent par :


A - Galop droit
B - Réaction pleurale à la radiographie
C - Température à 39°
D - Tachypnée à 30
E - Hémoptysie
Bonne(s) réponse(s) : A D

Le galop droit est un signe de décompensation cardiaque droite indiquant alors une embolie pulmonaire grave.
La tachypnée importante traduit un effet shunt important ; une Pa 02 < 60 mmHg est un indice d'embolie
pulmonaire grave.

Quel(s) médicament(s) prescrivez-vous ?


A - Antibiotique
B - Héparine
C - Digitalique
D - Diurétique
E - Vasodilatateurs
Bonne(s) réponse(s) : B

L'héparine est utilisée en perfusion continue intra-veineuse à dose efficace 500 UI/kg/j. Le relais sera pris
ensuite par les antivitamines K. La durée totale du traitement anticoagulant sera de 3 à 6 mois selon les
auteurs.

Malgré votre traitement l'état s'aggrave. Quel(s) examen(s) demandez-vous ?


A - Angiographie pulmonaire
B - Spirométrie
C - Scintigraphie pulmonaire de ventilation et de perfusion
D - Phlébo-cavographie
E - Artériographie bronchique
Bonne(s) réponse(s) : A C D

L'angiographie permet de confirmer le diagnostic et d'apprécier l'importance de l'amputation pulmonaire


(embolie peu importante si < à 40 %, embolie sévère si comprise entre 40 et 59, massive si > à 60 %.
Elle est effectuée dans le même temps que la phlébographie qui permettra de faire le diagnostic de phlébite et
recherchera sur les images de cavographie un thrombus flottant.
La scintigraphie, examen non spécifique, éliminera le diagnostic si elle est normale. Dans ce cas-ci elle n'aide
pas au diagnostic (l'angiographie sera faite de toute façon) mais est un examen de référence qui pourra être
répété par la suite afin de juger de l'évolution sous traitement.

487
Exclusivement sur DOC - DZ : www.doc-dz.com NADJI 85
RESIDANAT EN POCHE TOME II
Cas Clinique en QCM
Madame C. madeleine, 51 ans, a bénéficié il y a 11 ans, d'une comissurotomie mitrale à coeur fermé pour un
rétrécissement mitral pur, serré. Elle en a tiré un réel bénéfice. Il y a deux mois, son cardiologue l'a examinée
et a retrouvé un roulement diastolique 5/10ème et un claquement d'ouverture mitrale à la pointe ; le rythme
était régulier sinusal : la TA était à 12/7 cmHg. Il existait un souffle systolique sur le trajet carotidien gauche.
Le seul traitement prescrit est un comprimé de Digoxine tous les 2 jours.
Brutalement alors qu'elle fait quelques travaux de jardinage, elle ressent des bourdonnements d'oreille, une
instabilité à la marche et une difficulté à se servir de la main gauche. Tout rentre dans l'ordre en 2 heures. Le
médecin appelé note que le rythmecardiaque est irrégulier, le reste de l'examen étant inchangé. L'ECG est le
suivant.

Cliquez sur le bouton "Dessin" pour afficher l'ECG.

Lequel de ces troubles du rythme figure sur le tracé précédent ?

Cliquez sur le bouton 'Dessin' pour afficher le schéma.


A - Tachycardie jonctionnelle
B - Fibrillation auriculaire
C - Flutter auriculaire
D - Extrasystoles auriculaires en salves
E - Rythme jonctionnel
Bonne(s) réponse(s) : B

C'est une fibrillation auriculaire car il n'y a pas d'activité auriculaire organisée et les QRS sont fins et irréguliers.
C - Un flutter serait régulier

Parmi les hypothèses diagnostiques suivantes laquelle vous paraît la plus plausible dans ce cas
précis ?

Cliquez sur le bouton 'Dessin' pour afficher le schéma.


A - Embolie systémique à point de départ cardiaque
B - Hémorragie cérébrale
C - Thrombose de la carotide droite
D - Syndrome de Stokes Adams
E - Embolie systémique à point de départ carotidien
Bonne(s) réponse(s) : A

C'est le diagnostic le plus probable devant le passage récent en arythmie. L'accident évoque une topographie
sylvienne superficielle droite.
E - Si la sténose carotidienne gauche avait été en cause, la topographie de l'accident ischémique aurait été à
droite.

Quel examen complémentaire vous paraît strictement indispensable en première intention pour
étayer votre hypothèse diagnostique ?
Cliquez sur le bouton 'Dessin' pour afficher le schéma.
A - Scanner cérébral
B - Examen doppler des vaisseaux à destinée cérébrale
C - Anériographie carotidienne
D - Angiographie des cavités cardiaques
E - Echocardiographie
Bonne(s) réponse(s) : E

L'échocardiogramme permettra un nouveau bilan : retentissement (recherche d'une dilatation de l'oreillette


gauche, des cavités droites, recherche de signes d'HTAP), mesure de la surface mitrale, appréciation de
l'appareil sous valvulaire, recherche de calcifications.
Le doppler continu permet la mesure du gradient de pression, le calcul de la surface mitrale. Dans certains
cas, l'indication opératoire peut se poser au vu des résultats de l'écho-doppler sans nécessité d'examen
invasif.

Quelle(s) mesure(s) thérapeutique(s) immédiate(s) proposez-vous (sans préjuger des


thérapeutiques pouvant s'avérer nécessaires ultérieurement) ?
Cliquez sur le bouton 'Dessin' pour afficher le schéma.
A - Héparinothérapie
B - Traitement hypotenseur
C - Fibrinolytique
D - Traitement bêta-bloqueur
E - Antivitamines K
Bonne(s) réponse(s) : A

On discutera également le remplacement valvulaire, la réduction du trouble du rythme.

488
Exclusivement sur DOC - DZ : www.doc-dz.com NADJI 85
RESIDANAT EN POCHE TOME II
Cas Clinique en QCM
Une femme de 30 ans, pesant 85 kg, immobilisée 3 semaines pour une entorse du genou présente en 24 h
une douleur du mollet avec oedème, un signe de Homans avec tachycardie.

La phlébographie est souhaitable :


A - Parce que la phlébite peut être plus étendue que son niveau clinique
B - Parce que la malade est obèse
C - Parce que la clinique ne suffit jamais à établir le diagnostic de phlébite surale
D - Parce que les doses d'héparine sont fonction de l'extension réelle de la phlébite
E - Parce que cette phlébite doit être opérée
Bonne(s) réponse(s) : A C

Sans commentaire.

Quel traitement prescrivez-vous ?


A - Héparine® 400 mg, en 4 injections IV discontinues
B - Héparine® 400 mg en perfusion continue
C - Calciparine® 0,20 ml X 3, sous-cutanée
D - Thrombectomie chirurgicale
E - Fibrinolyse in situ
Bonne(s) réponse(s) : B

L'héparinothérapie doit être faite en intraveineuse continue ou discontinue (toutes les deux heures), à la dose
de 500 UI/kg/j, de façon à être efficace (TCK à deux fois le témoin).

24 h après, l'oedème a dépassé le pli inguinal qui est douloureux à la pression. A quel niveau
d'oblitération correspond ce tableau :
A - Fémorale superficielle
B - Fémorale commune
C - Fémorale profonde
D - Iliaque externe
E - Iliaque primitive
Bonne(s) réponse(s) : E

Sans commentaire.

Pour expliquer cette extension, il faut incriminer :


A - L'obésité
B - Les lésions traumatiques du genou
C - Le traitement : Calciparine® 0,2 ml x 3
D - L'absence de traitement préventif durant l'immobilisation
E - L'existence d'un terrain variqueux
Bonne(s) réponse(s) : C

Sans commentaire.

489
Exclusivement sur DOC - DZ : www.doc-dz.com NADJI 85
RESIDANAT EN POCHE TOME II
Cas Clinique en QCM
Monsieur X., est hospitalisé pour bilan étiologique d'une hypertension artérielle.
En 1978, on note une crise de colique néphrétique droite à l'occasion de laquelle, on découvre l'HTA, 220/100
mmHg couché.
Il est traité par l'association Sectral® 200 mg/jour et Aldomet® 500 mg/jour sans résultat.
A l' entrée la PA, après arrêt du traitement, est à 180/100 mmHg aux deux bras, au repos et 190/100 mmHg
debout. La fréquence cardiaque est à 75/mn. L'examen clinique est normal.
L'interrogatoire révèle un tabagisme (20 cig/jour), un éthylisme chronique modéré, et la consommation
quotidienne massive de réglisse "Florent" depuis plus de 10 ans (200 à 250 g/jour) arrêté depuis trois mois.
Le ionogramme sanguin et créatinine en régime normalement salé : Na = 142 mEq/l, K = 3,3 mEq/l, créatinine
à 78 micromol/l.
Ionogramme urinaire : Na = 90 mEq/l ; K = 30 mEq/l pour une diurèse de 1,5l/24 sans protéinurie. L'activité
rénine plasmatique est 0,2 ng/ml/h (NI : 0,2-0,7 ng/ml/h ; l'aldostéronémie = 920 pmol/l (N : 30 à 220 pmol/l).
Un test au Captopril® ( inhibiteur de l'enzyme de conversion de la rénine) est réalisé, 75 mg en une prise :
résultats de la rénine : 0,02 ng/ml/h et de l'aldostéronémie : 1000 pmol/l.
Le dosage de l'acide vanylmandélique et des dérivés méthoxylés sont dans les limites de la normale.

Le tableau clinique et biologique de cette hypertension vous évoque :


A - Une hypertension rénovasculaire
B - Une hypertension essentielle
C - Une hypertension d'origine iatrogène
D - Une hypertension d'origine endocrinienne
E - Aucune de ces étiologies
Bonne(s) réponse(s) : D

A - L'activité rénine plasmatique serait élevée.


C - L'intoxication à la réglisse peut donner une HTA avec hypokaliémie, mais l'aldostérone est normale

La consommation de réglisse peut donner une hypertension artérielle, en faveur de cette étiologie
vous retenez :
A - L'existence d'une hypokaliémie chronique
B - La persistance de l'hypertension malgré l'arrêt de l'absorption de réglisse
C - Diminution de l'activité rénine plasmatique
D - Baisse de l'activité rénine plasmatique sous Captopril®
E - Aucun des éléments précédents
Bonne(s) réponse(s) : A C

Sans commentaire.

L'ensemble du tableau clinique évoque :


A - Une intoxicabon à l'acide glycyrmizinique
B - Un hyperaldostéronisme secondaire
C - Un hyperaldostéronisme primaire
D - Une sténose de l'artère rénale
E - Un phéochromocytome
Bonne(s) réponse(s) : C

C'est le diagnostic le plus probable devant le tableau d'HTA avec hypokaliémi, hyporaldostéronisme, ARP
basse et inefficacité du test au Captopril®.
A - HTA avec hypokaliémie et aldostérone normale.
B - L'ARP et l'aldostérone sont élevés.
D - L'ARP et l'aldostérone sont élevées. Le test au Captopril® est positif.

Dans un but diagnostic, la prescription peut comporter :


A - L'artériographie sélective des artères rénales
B - Le scanner rénal et surrénal
C - L'écho des surrénales
D - Les prélèvements étagés de rénine (VCI et veines rénales)
E - Scintigraphie surrénale au iodocholestérol
Bonne(s) réponse(s) : B E

C - L'écho est rarement positive.


E - Montre une fixation en cas d'adénome, une fixation bilatérale en cas d'hyperplasie. On pourra faire aussi
une phlébographie des surrénales avec dosage d'aldostérone.

490
Exclusivement sur DOC - DZ : www.doc-dz.com NADJI 85
RESIDANAT EN POCHE TOME II
Cas Clinique en QCM

Un traitement par Aldactone® 100 mg/jour est mis en route. Quel(s) résultat(s) attendez-vous ?
A - Inefficacité sur la pression artérielle
B - Normalisation définitive de la pression artérielle
C - Normalisation transitoire de la pression artérielle
D - Normalisation de la kaliémie
E - Majoration des chiffres d'aldostéronémie
Bonne(s) réponse(s) : C D

Ce traitement est mis en route en attendant la chirurgie. Il comportera également un régime désodé et une
recharge potassique.

Une jeune femme de 35 ans est porteuse depuis l'âge de 10 ans d'un souffle cardiaque parfaitement bien
supporté sur le plan fonctionnel. Depuis quelques jours, elle se plaint d'une dyspnée d'effort progressive avec
sensation de palpitations. A l'examen, on note :
- un rythme cardiaque irrégulier
- une séméiologie de retrécissement mitral, roulement diastolique très net en décubitus latéral gauche associé
à un éclat du premier bruit et à un claquement d'ouverture mitrale.
- des râles crépitants aux deux bases
- une tension artérielle à 120/70 mm Hg
- une température à 36°
La radiographie thoracique montre un aspect floconneux du quart inférieur des deux champs pulmonaires, un
arc moyen gauche convexe et un aspect de double contour au niveau de l'arc inférieur droit. A l'ECG, le
rythme est irrégulier à 130/minute avec des complexes QRS fins sans ondes P visibles.

Parmi les signes d'auscultation suivants, quel est celui qui ne peut être retrouvé chez cette
patiente ?
A - Eclat de B2 au foyer pulmonaire
B - Souffle systolique apexien
C - Renforcement présystolique du roulement diastolique
D - Souffle diastolique au 4ème espace intercostal gauche
E - Souffle protodiastolique au foyer pulmonaire
Bonne(s) réponse(s) : C

Le renforcement présystolique correspond à la systole auriculaire, or la patiente est en arythmie par fibrillation
auriculaire donc sans systole auriculaire.
A - Signe d'HTA.
B - Signe d'insuffisance tricuspidienne.
D - Signe d'insuffisance aortienne.
E - Signe d'insuffisance pulmonaire.

Quelle est chez cette patiente la cause la plus probable de la brusque aggravation de son état
hémodynamique ?
A - Rupture spontanée de la valve
B - Endocardite bactérienne
C - Embolie pulmonaire
D - Fibrillation auriculaire
E - Récidive de rhumatisme cardiaque
Bonne(s) réponse(s) : D

La systole auriculaire assure 20 % du volume de remplissage du ventricule gauche.

Quelle(s) complication(s) peuven(t) survenir chez cette patiente ?


A - OEdème aigu du poumon
B - Mort subite
C - Embolie artérielle
D - Tachycardie ventriculaire
E - Syncope
Bonne(s) réponse(s) : A C

A - Est un tournant évolutif de la maladie.


C - Est une complication classique du rétrécissement mitral par embolie à partir d'un thrombus de l'oreillette
gauche, favorisée par la stase sanguine, elle-même favorisée par la fibrillation auriculaire et la dilatation de
l'oreillette gauche. Leur fréquence est estimée à 65 %.

491
Exclusivement sur DOC - DZ : www.doc-dz.com NADJI 85
RESIDANAT EN POCHE TOME II
Cas Clinique en QCM

Quel médicament vous parait le plus indiqué pour ralentir la fréquence cardiaque de cette patiente
?
A - Amiodarone (Cordarone®)
B - Digitalique
C - Verapamil (Isoptine®)
D - Bêta-bloquant
E - Sel de Quinidine
Bonne(s) réponse(s) : B

Sans commentaire.

Quel(s) autre(s) médicament(s) associerez-vous dans l'immédiat ?


A - Antivitamine K
B - Antiagrégant plaquettaire
C - Héparine
D - Diurétique
E - Lidocaïne (Xylocaïne®)
Bonne(s) réponse(s) : C D

La présence d'un trouble du rythme supraventriculaire sur valvulopathie mitrale impose l'anticoagulation.

Un homme de 40 ans consulte pour des douleurs thoraciques nocturnes apparues 8 jours auparavant. Les
crises le réveillent vers 3 heures du matin. La douleur siège dans la région rétrosternale et irradie dans les
bras et le dos : elles durent environ 5 minutes et se répètent 2 à 3 fois dans le courant de la nuit. Le patient a
continué son activité professionnelle, il n'éprouve d'ailleurs aucune gêne dans la journée. Dans les
antécédents, on note un tabagisme à 40 g/j depuis 15 ans. L'examen clinique est normal : la pression
artérielle est à 140/80 mm Hg. L'ECG inter-critique est sans particularités. Le diagnostic d'angor spastique
type Prinzmétal est évoqué.

Parmi les examens complémentaires suivants, quel est celui qui a le plus de chance de confirmer
ce diagnostic ?
A - Enregistrement ECG ambulatoire sur 24 heures
B - ECG d'effort
C - ECG sous stimulation auriculaire
D - Echocardiogramme
E - Scintigraphie myocardique
Bonne(s) réponse(s) : A

Il recherchera les troubles de la repolarisation pendant les crises douloureuses ainsi que les troubles du
rythme et de conduction associés. Il faudra au besoin le répéter.
Dans un deuxième temps, il faudra proposer un test au Néthergin® après avoir éliminé une sténose
significative à la coronarographie.

Au cours d'une crise douloureuse, un ECG est enregistré dans la dérivation D2. Parmi ces
différentes anomalies, quelle(s) est (sont) celle(s) compatible(s) avec le diagnostic évoqué ?
A - Sous décalage descendant de 2 mm de ST
B - Sus-décalage de 5 mm de ST
C - Onde T négative, pointue et symétrique
D - Extrasystoles ventriculaires en salves
E - Bloc auriculo-ventriculaire transitoire
Bonne(s) réponse(s) : B

Sans commentaire.

Le traitement médical peut faire appel à :


A - Propranolol (Avlocardyl®) per os
B - Nifédipine (Adalate®) per os
C - Isosorbide dinitrate (Risordan®) per os ou en perfusion IV
D - Diltiazem (Tildiem®) per os
E - Méthylergométrine (Méthergin®) en perfusion IV lente
Bonne(s) réponse(s) : B C D

L'Avlocadyl® est contre-indiqué du fait de son activité sympathicomimétique intrinsèque.

492
Exclusivement sur DOC - DZ : www.doc-dz.com NADJI 85
RESIDANAT EN POCHE TOME II
Cas Clinique en QCM

Parmi les différentes modalités évolutives suivantes, laquelle(lesquelles) peut-on observer chez
ce patient ?
A - Apparition de syncopes
B - Aggravation de l'angor avec crises subintrantes
C - Infarctus du myocarde
D - Mort subite
E - Rémission durable
Bonne(s) réponse(s) : A B C D E

Toutes ces éventualités peuvent se voir.

Une jeune femme maghrébine âgée de 35 ans, vient consulter parce qu'elle a fait récemment un oedème aigu
du poumon sur un fond de dyspnée d'effort datant de 4 ans. Cette jeune femme a fait un rhumatisme
articulaire aigu à l'âge de 12 ans. L'auscultation permet d'entendre les signes stéthacoustiques d'un
rétrécissement mitral (R.M.). L'auscultation des autres orifices cardiaques est normale. Des examens
complémentaires sont demandés en vue d'une éventuelle opération chirurgicale.

Indiquez parmi les données suivantes fournies par le cathétérisme cardiaque, celle qui confirme le
R.M. :
A - Gradient entre la pression capillaire et la pression diastolique du ventricule gauche
B - Baisse de la pression diastolique du ventricule gauche
C - Gradient entre la pression capillaire et la pression systolique du ventricule gauche
D - Désaturation en oxygène du sang artériel
E - Elargissement de la pression artérielle différentielle
Bonne(s) réponse(s) : A

Il nécessite un catéthérisme droit et gauche.


Le gradient peut être mesuré au doppler continu.

L'examen le plus fiable pour juger de l'importance des lésions sous-valvulaires est :
A - L'auscultation du coeur
B - L'électrocardiogramme
C - Le phonocardiogramme
D - L'échocardiogramme
E - L'angiographie ventriculaire gauche
Bonne(s) réponse(s) : D

Elle apprécie en outre le retentissement (dilatation de l'oreillette gauche, dilatation des cavités droites, HTAP)
elle permet de mesurer la surface mitrale. Le doppler permet la mesure du gradient de pression et
l'appréciation de la surface mitrale.

On doit craindre une thrombose dans l'oreillette gauche lorsqu'existe(nt) :


A - Une fibrillation auriculaire
B - Des lésions sous-valvulaires importantes
C - Une hypertension artérielle pulmonaire
D - Des antécédents d'embolie pulmonaire
E - Des antécédents d'embolie artérielle
Bonne(s) réponse(s) : A E

Sans commentaire.

Parmi les éléments suivants, indiquez celui(ceux) qui contre-indique(nt) une simple
commissurotomie mitrale :
A - Age supérieur à 30 ans
B - Insuffisance tricuspide associée
C - Thrombose intra-auriculaire gauche + embolies systémiques
D - Calcifications sur les valves mitrales
E - Fuite mitrale importante
Bonne(s) réponse(s) : C D E

C - C'est une contre-indication à la commissurotomie à coeur fermé.


D - Nécessite la mise en place d'une prothèse valvulaire.
E - Indication à une plastie ou prothèse valvulaire.
Actuellement, on peut proposer au rétrécissement mitral pur sur valves souples, la réalisation d'une
valvuloplastie avec mise en place du ballon de dilatation par catéthérisme trans-septal.

493
Exclusivement sur DOC - DZ : www.doc-dz.com NADJI 85
RESIDANAT EN POCHE TOME II
Cas Clinique en QCM
Un homme de 65 ans, sans antécédents, consulte pour l'apparition depuis quelques mois de douleurs
thoraciques médiosternales constrictives survenant à l'effort et cédant à l'arrêt et d'autre part de 2 syncopes
brèves isolées survenues lors d'efforts inhabituels.
On constate : frémissement systolique au foyer aortique et au creux sus-sternal. Souffle mésotélésystolique
maximum (4/6) au foyer aortique avec un 2ème maximum (3/6) au foyer mitral sans modifications de ses
caractères stéthoscopiques. Pas de click protosystolique. Existence d'un léger souffle (1/6) parasternal
gauche diastolique accroché à B2. TA à 120/75 mmHg. En radioscopie, coeur de volume normal,
calcifications mobiles projetées de face au dessus la bissectrice de l'angle formé par le bord droit de la
colonne et l'horizontale passant par les coupoles diaphragmatiques.

La sémiologie objective chez ce patient suggère :


A - Un souffle éjectionnel systolique
B - Un souffle de régurgitation systolique
C - Un souffle d'éjection et de régurgitation systolique
D - Des calcifications valvulaires mitrales
E - Des calcifications valvulaires aortiques
Bonne(s) réponse(s) : A E

A - En faveur du caractère éjectionnel : le frémissement systolique, le maximum mésotélésystolique


l'irradiation aux vaisseaux du cou.
D - Elles seraient sous la ligne décrite. Les calcifications sont étudiées en OAD et OAG.

Quel diagnostic parait cliniquement le plus probable ?


A - Rétrécissement aortique valvulaire
B - Rétrécissement aortique sous-valvulaire
C - Insuffisance mitrale
D - Rétrécissement aortique et insuffisance mitrale
E - Myocardiopathie obstructive
Bonne(s) réponse(s) : A

Sans commentaire.

Cocher l'(les) étiologie(s) des syncopes qui peu(ven)t être évoquée(s) à l'interrogatoire :
A - Comitialité
B - Bloc auriculo-ventriculaire paroxystique
C - Syncope vagale
D - Hypertension artérielle pulmonaire sévère
E - Angor syncopal au cours de l'athérosclérose coronarienne
Bonne(s) réponse(s) : B

Les troubles de conduction sont possibles au cours du rétrécissement aortique par coulée calcaire sur le
septum, mais les syncopes ne sont pas liées à l'effort.

Un échocardiogramme TM et bidimensionnel est enregistré chez ce patient. Cet examen :


A - Confirme le siège de l'anomalie cardiaque
B - Permet les mensurations (diamètres, épaisseur pariétale)
C - Permet d'apprécier la fonction du ventricule gauche
D - Permet, par les indices systoliques, d'apprécier la pression du ventricule gauche
E - Permet d'estimer le gradient par le Doppler
Bonne(s) réponse(s) : A B C E

A - Montre des sigmoïdes épaissies, calcifiées à l'ouverture diminuée.


B - Apprécie le retentissement sur le ventricule gauche.
C - La fraction de racourcissement calculée à partir des diamètres mesurés est corrélée à la fraction
d'éjection.
E - Le doppler continu estime le gradient VG-aorte et la surface aortique.

Après ces examens, le patient est perdu de vue. Il consulte à nouveau 6 mois plus tard en raison
de l'apparition de signes d'insuffisance ventriculaire gauche. Le pronostic spontané chez ce
patient est de l'ordre de :
A - 5 ans
B - 3 à 4 ans
C - 1 à 2 ans
D - 6 mois
E - 3 mois
Bonne(s) réponse(s) : C

Sans commentaire.
494
Exclusivement sur DOC - DZ : www.doc-dz.com NADJI 85
RESIDANAT EN POCHE TOME II
Cas Clinique en QCM
Monsieur L. Jean-Luc, âgé de 32 ans, barman dans une boite connue, sans antécédents, cumule les facteurs
de risque malgré les conseils de son médecin. Celui-ci n'est pas outre mesure étonné lorsque le 24 mai à 6
heures, il est appelé en urgence pour ce patient en proie depuis une heure à une violente douleur
rétrosternale constrictive, irradiant vers les mâchoires et les épaules et se majorant en inspiration forcée. A
l'arrivée du médecin 15 minutes plus tard, le patient est angoissé, pâle, couvert de sueurs : la douleur est
toujours très vive à type d'étau. La TA habituellement à 15/8 cmHg est retrouvée à 15/8. Le rythme cardiaque
est à 82/min, régulier, avec toutefois quelques extrasystoles sporadiques. La température est à 38,3 degrés.
Un frottement péricardique est perçu dans la région méso-cardiaque. Le médecin redoute un infarctus du
myocarde et fait hospitaliser d'urgence Monsieur L. mais il reste prudent dans ses conclusions et les
thérapeutiques qu'il administre avant l'arrivée de l'ambulance.

Quel est le diagnostic le plus probable ?


A - Syndrome de menace
B - Infarctus du myocarde
C - Dissection aortique
D - Péricardite aiguë
E - Embolie pulmonaire
Bonne(s) réponse(s) : D

Le diagnostic est évoqué devant le caractère de la douleur augmentant à l'inspiration, l'association avec un
frottement péricardique et une fièvre précoce.

Quel(s) élément(s) pour le diagnostic retenu vous a(ont) été le(s) plus utile(s) ?
A - La notion de facteurs de risque
B - La douleur augmentant en inspiration
C - La température à 38,3 degrés
D - Le frottement péricardique
E - La présence d'extrasystoles
Bonne(s) réponse(s) : B C D

Sans commentaire.

Quel(s) traitement(s) le médecin appelé en urgence a-t-il cru bon à juste titre de prescrire ?
A - De la Streptokinase® par voie veineuse
B - De l'Héparine® par voie veineuse
C - De l'acide acétylcalicylique par voie veineuse
D - Un digitalique d'action rapide par voie veineuse
E - Un diurétique d'action rapide par voie veineuse
Bonne(s) réponse(s) : C

Sans commentaire.

Parmi les facteurs de risque classiquement admis, vous retenez chez ce patient :
A - Une surcharge pondérale à 80 kg pour 1,73 m
B - Un taux d'apoprotéine B bas
C - Un tabagisme à 20 cigarettes/jour
D - Un taux de HDL cholestérol élevé
E - Une TA à 15/8
Bonne(s) réponse(s) : C

A - Le surpoids ne fait pas partie, stricto sensu, des facteurs de risque cardio-vasculaires prouvés. Il doit être
corrigé cependant.
B.D. - L'apoprotéine B est un des constituants des CDL et son élévation est athérogène. Le taux de LDL-
cholestérol élevé est au contraire protecteur.
E - L'HTA est par définition une TA > 160/95 mmHg.

Le frottement péricardique :
A - Est un bruit superficiel
B - Disparait généralement en apnée
C - Est généralement à la foi méso-systolique et méso-diastolique
D - Est souvent discret
E - Est habituellement fugace
Bonne(s) réponse(s) : A C D E

Sans commentaire.

495
Exclusivement sur DOC - DZ : www.doc-dz.com NADJI 85
RESIDANAT EN POCHE TOME II
Cas Clinique en QCM
Monsieur T.V., 41 ans, cadre dans l'industrie, est hospitalisé pour un infarctus du myocarde. Ce patient
tabagique (25 cigarettes par jour depuis 25 ans), dyslipidémique (tpe IIb), obèse (85 kg pour 1,75 m), a un
profil psychologique de type A de Friedmann, il n'a pas d'antécédent cardiovasculaire. Comme seul
antécédent, on ne retient qu'un asthme connu depuis une vingtaine d'années. La veille de l'hospitalisation au
sortir d'un dîner, il avait ressenti une vague gêne épigastrique pendant 15 à 20 minutes qui avait cédé au
coucher. Une reprise de la douleur à 5 heuresdu matin aboutit à la constitution d'un infarctus inférieur typique.
L'évolution immédiate est simple, sans trouble du rythme, sans récidive de douleurs angineuses, ni
insuffisance cardiaque. Au 10ème jour de l'hospitalisation, la sortie est envisagée.

Il s'agit d'un infarctus inférieur non compliqué, d'évolution simple. Parmi ces examens, un est
indispensable pour essayer d'apprécier l'existence ou non de sténose sur la coronaire gauche.
Lequel ?
A - Echocardiogramme
B - Gamma angiocardiographie de repos
C - Epreuve d'effort sur bicyclette ergométrique
D - Scintigraphie myocardique au thallium au repos
E - Gamma tomoscintigraphie
Bonne(s) réponse(s) : C

Elle doit être réalisée avec un matériel de réanimation à proximité. Les contre-indications sont l'angor instable,
la sténose du tronc commun, I'HTA et l'insuffisance cardiaque non contrôlées, le thrombus du VG, la
péricardite, les troubles du rythme et de conductions graves, le rétrécissement aortique et la cardiomyopathie
obstructive. Elle est positive devant une douleur angineuse et/ou un sous décalage de ST horizontal ou
descendant de 1 mm au moins pendant 0,08s.
D - La scintigraphie au repos ne permet pas de détecter l'ischémie. Elle visualise les zones nécrosées.

L'examen réalisé (en réponse à la question n°1) est très fortement positif. Il est utilse de le
compléter par :
A - Une scintigraphie myocardique au pyrophosphate
B - Une coronaroventriculographie
C - Un test à l'Isuprel® avec carotidogramme
D - Un dosage des lactates dans les sinus coronaires
E - Un test au Méthergin® au lit du malade
Bonne(s) réponse(s) : B

Sans commentaire.

Chez ce patient, en cas de récidives de crise d'angor, quelle(s) thérapeutique(s) est(sont) à éviter
?
A - Les dérivés nitrés
B - Les inhibiteurs calciques
C - Le maléate de perhexilline
D - L'amiodarone
E - Les bêta-bloqueurs
Bonne(s) réponse(s) : C E

E - En raison des antécédents d'asthme.


C - En raison des effets secondaires majeurs (neuropathie périphérique, amaigrissement, hypoglycémie,
atteinte hépatique...).

Le bilan complet de la maladie coronaire permet de conclure à une thrombose de la coronaire


droite segment II et une sténose sur l'IVA moyenne, et sur la circonflexe proximale, avec un bon
réseau d'aval et une mauvaise tolérance à l'effort. La thérapeutique que vous choisirez sera :
A - Association inhibiteurs calciques et dérivés nitrés
B - Association nifédipine dérivés nitrés, bêta-bloqueurs
C - Réadaptation et traitement médical
D - Association bêta-bloqueurs et nifédipine
E - Double pontage aorto-coronaire, IVA et circonflexe
Bonne(s) réponse(s) : E

On choisit la chirurgie en raison de l'âge jeune du patient et de l'atteinte bitronculaire.

496
Exclusivement sur DOC - DZ : www.doc-dz.com NADJI 85
RESIDANAT EN POCHE TOME II
Cas Clinique en QCM

Dans le cadre de la prévention de la maladie coronaire, vous instituez d'emblée un des


traitements ou régimes ci-dessous. Lequel ?
A - Régime désodé strict
B - Régime hypocalorique 1200 calories par jour
C - Probucol ou Lurselle® 4 comprimés par jour
D - Régime pauvre en purine
E - Fénofibrate ou Lipanthyl® 3 comprimés par jour
Bonne(s) réponse(s) : B

Sans commentaire.

Sans tenir compte de ce cas particulier, la prévention de la récidive d'infarctus du myocarde et de


la mort subite est efficace si l'on prescrit un des médicaments ci-dessous. Lequel ?
A - Antiarythmiques de classe I

B - Dérivés nitrés au long cours

C - Amiodarone bêta-bloqueurs

E - Inhibiteurs calciques
Bonne(s) réponse(s) : D

Ils diminuent la mortalité après infarctus du myocarde.

Un homme de 70 ans alerte mais tabagique (50 paquets/année) vous consulte pour une visite annuelle.
L'interrogatoire vous apprend que la montée d'une côte entraîne parfois une douleur rétrosternale mais jamais
de douleurs des membres. La tension artérielle est à 19/12 aux deux bras. Le pouls radial bat à 70. Aux
membres inférieurs à droite aucun pouls n'est perçu : à gauche tous les pouls sont bien frappés. La palpation
de l'abdomen vous révèle une masse indolore d'environ 6 cm de large.

Parmi les signes suivants, lequel vous permet d'affirmer que cette masse est un anévrisme de
l'aorte abdominale ?
A - Masse pulsatile
B - Masse expansive
C - Masse médiane
D - Souffle abdominal
E - Abolition de certains pouls
Bonne(s) réponse(s) : B

Le caractère expansif de la masse est fondamental.

Quel exament demanderiez-vous en premier pour confirmer le diagnostic ?


A - Abdomen sans préparation
B - Echotomographie
C - Scanner abdominal
D - Angiographie digitalisée
E - Doppler des membres inférieurs
Bonne(s) réponse(s) : A

Examen le plus simple, la radiographie d'abdomen sans préparation de face et profil montre souvent des
calcifications linéaires diminuant l'anévrisme. Néanmoins, l'échographie est souvent demandée de première
intention.

Quelle artère du membre inférieur droit, est, selon les données cliniques, certainement obstruée ?
A - Iliaque primitive
B - Iliaque externe
C - Fémorale commune
D - Fémorale superficielle
E - Aucune des propositions précédentes ne peut être affirmée avec certitude
Bonne(s) réponse(s) : B

Question annulée.

497
Exclusivement sur DOC - DZ : www.doc-dz.com NADJI 85
RESIDANAT EN POCHE TOME II
Cas Clinique en QCM

Quel élément clinique impose une intervention chirurgicale chez cet homme en bon état général ?
A - Abolition des pouls
B - Taille de l'anévrisme
C - Association d'une hypertension artérielle
D - Age du malade
E - Insuffisance coronarienne
Bonne(s) réponse(s) : B

Il existe une relation de type exponentiel entre la taille de l'anévrisme et le risque de rupture à 5 ans. Le risque
est majeur au delà de 5 à 6 cm de diamètre.

Quel(s) traitements) préconisez-vous ?


A - Sympathectomie
B - Pontage axillo-bifémoral
C - Remplacement aortique par prothèse
D - Embolectomie
E - Résection anévrisme simple
Bonne(s) réponse(s) : C

L'opération consiste en une mise à plat-greffe par prothèse anastomose en termino-terminal.

Un homme de 70 ans, artéritique connu, opéré cinq ans auparavant d'une thrombose termino-aortique par
pontage prothétique aorto-bifémoral, revient consulter pour bilan annuel systématique. Depuis l'intervention,
les possibilités de marche sont redevenues normales. A l'examen, tous les pouls périphériques sont présents
aux différents étages des deux membres inférieurs. Au niveau du Scarpa droit existe une masse de 3 cm de
diamètre, située sur le trajet artériel, battante mais dont on ne peut dire si elle est expansive : elle n'est ni
douloureuse, ni inflammatoire et elle est apparue depuis environ 6 mois. Le reste de l'examen ne révèle
aucune anomalie, excepté une tension artérielle à 160/80 à droite et à 120/60 à gauche.

Parmi les éléments suivants en faveur d'un anévrisme anastomotique, un seul est constant et
justifie de penser systématiquement à ce diagnostic :
A - Présence de battements
B - Indolence
C - Siège sur le trajet de l'axe artériel opéré
D - Apparition récente
E - Absence de retentissement en aval
Bonne(s) réponse(s) : C

Sans commentaire.

Indiquez l'examen complémentaire réalisable en consultation externe, peu couteux et apportant


avec certitude le diagnostic :
A - Radiographie simple
B - Vélocimétrie Doppler
C - Echographie vasculaire
D - Thermographie
E - Angiographie digitalisée
Bonne(s) réponse(s) : C

Sans commentaire.

Un tel anévrysme, petit, récent et asymptomatique peut être suivi de complication(s) grave(s).
Indiquez celle(s) qu'il faut redouter :
A - Compression veineuse fémorale
B - Thrombose de la branche de la prothèse
C - Embolie distale
D - Suppuration de prothèse
E - Rupture
Bonne(s) réponse(s) : B C E

Sans commentaire.

498
Exclusivement sur DOC - DZ : www.doc-dz.com NADJI 85
RESIDANAT EN POCHE TOME II
Cas Clinique en QCM

La différence tensionnelle entre les deux bras évoque une sténose de la sous-clavière gauche.
Indiquez pourquoi celle-ci n'est pas inquiétante :
A - Les sténoses sous-clavières ne retentissent que sur le membre supérieur
B - Les sténoses sous-clavières n'ont jamais de conséquence grave
C - Les 4 points d'écart ne traduisent qu'une sténose peu significative
D - La sténose est actuellement asymptomatique
E - La sous-clavière gauche a un rôle fonctionnel moins important chez le droitier
Bonne(s) réponse(s) : D

Le phénomène de vol vertébral peut avoir une traduction neurologique sévère.

Les examens effectués montrent que le diamètre de l'anévrisme atteint deux fois le diamètre de la
prothèse.Indiquez l'attitude que vous préconisez :
A - Abstention thérapeutique
B - Prescription d'antivitamines K pour prévenir un accident de thrombose
C - Surveillance bisannuelle par examen clinique et échographie
D - Traitement chirurgical de l'anévrysme
E - Embolisation de l'anévrisme
Bonne(s) réponse(s) : D

Le risque de rupture impose l'intervention chirurgicale.

Un homme de 55 ans, sans antécédent particulier, non fumeur, se trouvant asthénique et essoufflé depuis
trois semaines, appelle son médecin car il ressent une douleur rétrosternale constrictive avec sensation de
palpitations. A l'examen est perçu un souffle systolique de type éjectionnel, d'intensité 2/6 maximum au foyer
aortique irradiant dans les vaisseaux du cou et au troisième espace intercostal gauche. Le B1 et le B2 sont
perçus. La fréquence cardiaque est à 140 par minute, régulière.
La pression artérielle est à 90-60 mmHg. Les conjonctives sont pâles et les selles noires (méléna).

La cause la plus probable des douleurs thoraciques est :


A - Un angor fonctionnel sans cardiopathie
B - Un rétrécissement aortique.
C - Une tachysystolie auriculaire
D - Une myocardiopathie hypertrophique et obstructive
E - Une athérosclérose coronarienne
Bonne(s) réponse(s) : A

L'anémie sévère est une cause d'angor fonctionnel, le souffle systolique au foyer aortique n'est pas synonyme
de rétrécissement aortique symptomatique.

En dehors des douleurs thoraciques, quel(s) autre(s) symptôme(s) ou signe(s) tiré(s) de


l'observation concoure(nt) au même diagnostic ?
A - L'essoufflement
B - La tachycardie
C - La pâleur
D - Le souffle systolique
E - L'hypotension
Bonne(s) réponse(s) : A B C D E

L'ensemble du tableau est en faveur d'une anémie aiguë importante et mal tolérée.

Le traitement d'urgence après un bilan simple comporte la prescription :


A - D'un digitalique
B - De vérapamil (Isoptine®)
C - De trinitrine intraveineuse
D - D'un bêta-bloquant
E - D'une transfusion sanguine
Bonne(s) réponse(s) : E

La correction de l'hémorragie et de l'anémie est prioritaire.

499
Exclusivement sur DOC - DZ : www.doc-dz.com NADJI 85
RESIDANAT EN POCHE TOME II
Cas Clinique en QCM

Parmi les examens suivants, vous choisissez de faire en priorité :


A - Un échocardiogramme
B - Un phonomécanocardiogramme
C - Une fibroscopie gastrique
D - Un lavement baryté
E - Un enregistrement Holter
Bonne(s) réponse(s) : C

Sans commentaire.

Un patient âgé de 71 ans est amené aux urgences par le SAMU car il a présenté une syncope. Il n'y a pas
d'antécédent pathologique particulier jusqu'à il y a trois jours où il a présenté au repos une douleur thoracique
latérale gauche à hauteur du mammelon, accompagnée de quelques sueurs. Il a vu son médecin traitant la
veille, qui n' a pas noté d'anomalies, mais lui a prescrit du Risordan® 20 mg 3 fois par jour, et du Sectral® 200
mg 3 fois par jour. Il a commencé le traitement le matin même, avec une prise de 1 comprimé de chaque avec
son petit déjeuner vers 8 h. Il est ensuite sorti pour accompagner son épouse au marché. C'est vers 9 heures
qu'il a commencé à ressentir une sensation de faiblesse, avec des sueurs et une impression de tête vide. Un
quart d'heure plus tard, il a été obligé de s'asseoir. Sa femme a noté sa pâleur, puis l'a vu progressivement
perdre connaissance pendant 2 à 3 minutes. Une fois allongé, il a récupéré progressivement sa conscience et
s'est rassis. A son arrivée, la TA est à 10/6, la fréquence cardiaque à 57/mn et le sujet se sent mieux.
L'examen est entièrement normal pour l'âge, de même que l'ECG, le cliché thoracique, et le bilan biologique
réalisé en urgence (hémogramme, ionogramme, enzymes).

Parmi les mécanismes suivants, lequel(lesquels) retenez-vous comme le(s) plus probable(s) ?
A - Bloc auriculo-ventriculaire paroxystique
B - Hypotension orthostatique
C - Crise comitiale
D - Ictus laryngé
E - Exagération du tonus vagal
Bonne(s) réponse(s) : B E

La présence de prodrome et la bradycardie sont en faveur de ces hypothèses.

Parmi les examens complémentaires suivants, lequel jugez-vous indispensable dans l'immédiat ?
A - Electroencéphalogramme
B - Scanner cérébral avec produit de contraste
C - Etude électrophysiologique cardiaque endocavitaire
D - Ponction lombaire
E - Aucune des propositions précédentes
Bonne(s) réponse(s) : E

Sans commentaire.

Parmi les conduites suivantes, laquelle adoptez-vous dans l'immédiat ?


A - Trinitrine en perfusion
B - Abstention et surveillance
C - Mise en place d'une sonde endocavitaire d'entraînement électrosystolique
D - Dobutrex® en perfusion
E - Gardénal®10 cg/jour
Bonne(s) réponse(s) : B

Sans commentaire.

Vis à vis du traitement en cours, quelle attitude préconisez-vous pour la suite ?


A - Reprise du seul Risordan®
B - Reprise du seul Sectral®
C - Reprise des deux médicaments
D - Arrêt des deux médicaments
E - Réévaluation de l'indication.
Bonne(s) réponse(s) : E

L'épisode douloureux throracique unique et atypique ne justifie pas la mise sous antiangineux avant toute
investigation complémentaire.

500
Exclusivement sur DOC - DZ : www.doc-dz.com NADJI 85
RESIDANAT EN POCHE TOME II
Cas Clinique en QCM
Monsieur B. âgé de 64 ans, retraité en bon état général, est traité pour une HTA modérée (170-100 mmHg)
par Fludex® depuis 1 an. Depuis deux ans, il se plaint de fréquents accès de tachycardie paroxystique à
début brutal, accompagné d'un malaise général avec asthénie intense, sans douleurs thoraciques ; jusqu'à
présent, les crises ont toujours cédé spontanément en quelques dizaines de minutes : il n'existe pas de
polyurie postcritique. Il est hospitalisé cette fois pour une crise qui évolue depuis 8 heures, la PA est 150-80
mmHg ; l'examen clinique ne montre pas de signe d'insuffisance cardiaque ; l'auscultation entend un rythme
régulier à 150 b/mn sans anomalies surajoutées. La radiographie thoracique montre un coeur de volume et de
morphologie normaux. Le bilan biologique est normal. L'ECG suivant est enregistré :

Cliquez sur le bouton "Dessin" pour afficher l'ECG.

Quel est le trouble du rythme cardiaque en cause ?

Cliquez sur le bouton 'Dessin' pour afficher le schéma.


A - Tachycardie sinusale
B - Fibrillation auriculaire
C - Flutter auriculaire
D - Tachycardie jonctionnelle
E - Tachycardie ventriculaire
Bonne(s) réponse(s) : C

L'activité auriculaire régulière et diphasique, à 300/minute, est bien vue dans les dérivations inférieures.

Parmi ces éléments tirés de l'interrogatoire et de l'examen clinique, quel est le seul qui permet
d'évoquer prioritairement ce diagnostic avant même l'enregistrement ECG ?
Cliquez sur le bouton 'Dessin' pour afficher le schéma.
A - Début brutal
B - Rythme régulier à 150 b/mn
C - Bonne tolérance hémodynamique
D - Absence de douleurs thoraciques
E - Absence de polyurie post critique
Bonne(s) réponse(s) : B

Le flutter est une tachycardie supraventriculaire le plus souvent régulière.

Compte-tenu de la nature de l'arythmie et des données cliniques et paracliniques, quelle est


l'étiologie la plus probable ?
Cliquez sur le bouton 'Dessin' pour afficher le schéma.
A - Idiopathique
B - Ischémique
C - Valvulopathie mitrale
D - Myocardiopathie dilatée
E - Métabolique
Bonne(s) réponse(s) : A

Sans commentaire.

Parmi ces 5 propositions thérapeutiques, laquelle vous semble la plus indiquée dans l'immédiat ?

Cliquez sur le bouton 'Dessin' pour afficher le schéma.


A - Manoeuvres vagales
B - Héparinothérapie IV
C - Xylocaïne® IV
D - Héparine® / Xylocaïne;® IV
E - Aucune thérapeutique
Bonne(s) réponse(s) : B

Malgré le caractère modérément emboligène, l'anticoagulation s'impose en cas de flutter auriculaire.

501
Exclusivement sur DOC - DZ : www.doc-dz.com NADJI 85
RESIDANAT EN POCHE TOME II
Cas Clinique en QCM

Après 24 heures d'observation, l'arythmie persiste. Quelle(s) est(sont) l'(les) attitude(s)


thérapeutique(s) la(les) mieux appropriée(s) ?
Cliquez sur le bouton 'Dessin' pour afficher le schéma.
A - Se contenter de poursuivre le même traitement
B - Changer d'antiarythmique
C - Réaliser un choc électrique externe
D - Tenter une réduction par stimulation électrique de l'oreillette
E - Réaliser un choc électrique endocavitaire
Bonne(s) réponse(s) : C D

Ce sont les deux traitements les plus efficaces.

Le rythme sinusal a été restauré. Pour prévenir les récidives, lequel(lesquels) parmi les schémas
thérapeutiques suivants vous semble(nt) correct(s) ?
Cliquez sur le bouton 'Dessin' pour afficher le schéma.
A - Sérécor® (hydroquinidine) : 2 cp/j
B - Sérécor® (hydroquinidine) : 6 cp/j
C - Rythmodan® (dysopyramide) l'action prolongée : 2 cp/j
D - Rythmodan® (dysopyramide) l'action prolongée : 6 cp/j
E - Mexitil® (mexiletine) : 3 gélules/j
Bonne(s) réponse(s) : A C

L'indication de la méxilétine est à l'étage ventriculaire.

Une jeune femme de 30 ans, sans antécédents rhumatismaux évidents, est admise à l'hôpital pour une
hémiparésie droite rapidement régressive. L'interrogatoire retrouve un essoufflement modéré à l'effort depuis
un an et des accès de palpitation, de durée habituellement brève, sauf le dernier qui à duré 3 heures, à début
assez brusque ; le rythme cardiaque serait rapide mais irrégulier ; il y a une pollakiurie per-critique ; la
terminaison de l'accès se fait d'une façon progressive. L'examen neurologique est normal. Le rythme
cardiaque est régulier à 70/mn, l'auscultation entend à la pointe un éclat de B1, pas de souffle systolique, un
claquement d'ouverture, un roulement holodiastolique plus net en décubitus latéral gauche, constant, que la
patiente soit auscultée en décubitus dorsal ou en position debout. Il y a de plus un éclat net du B2 au foyer
pulmonaire. La radiographie de face montre un coeur peu augmenté de volume, un débord droit, un double
contour, une crosse aortique normale, une saillie à double bosse de l'arc moyen gauche, un arc inférieur
gauche non allongé. Les artères pulmonaires sont un peu augmentées dans les hiles, mais la transparence
pulmonaire est normale. L'ECG montre un rythme sinusal, une hypertrophie auriculaire gauche, un axe de
QRS à +100°, un bloc incomplet droit. Le bilan biologique est normal.

Quelle cardiopathie vous suggèrent les données cliniques, radiologiques et


électrocardiographiques ?
A - Prolapsus valvulaire mitral
B - Insuffisance mitrale
C - Communication inter-auriculaire
D - Myxome de l'oreillette gauche
E - Aucune des propositions ci-dessus
Bonne(s) réponse(s) : E

Les données auscultatoires, radiologiques et électrocardiographiques sont en faveur du diagnostic du


rétrécissement mitral.

Quel est l'examen paraclinique non invasif (à l'exception de la radiologie standard)


immédiatement utile pour confirmer la cardiopathie mise en évidence par l'examen clinique ?
A - Electrocardiogramme standard ECG
B - Carotidogramme.
C - Doppler cervical
D - Echocardiogramme
E - Holter
Bonne(s) réponse(s) : D

L'échocardiogramme pourra confirmer ce diagnostic.

502
Exclusivement sur DOC - DZ : www.doc-dz.com NADJI 85
RESIDANAT EN POCHE TOME II
Cas Clinique en QCM

Quel trouble rythmique paroxystique, en fonction des données de l'interrogatoire, peut être
incriminé à l'origine de l'accident neurologique déficitaire transitoire ?
A - Extrasystoles auriculaires
B - Tachysystolie auriculaire : 2/1
C - Flutter auriculaire : 2/1
D - Fibrillation auriculaire
E - Tachycardie jonctionnelle paroxystique
Bonne(s) réponse(s) : D

Le RM se complique volontiers d'accès paroxystique de fibrillation auriculaire avant que celle-ci ne devienne
permanente. Le fait que le rythme cardiaque au cours de l'accès soit irrégulier et que cet épisode se soit
accompagné d'une embolie cérébrale plaide également en faveur de ce diagnostic.

Quelle complication induite par le trouble rythmique paroxystique est vraisemblablement


responsable de l'accident neurologique déficitaire régressif ?
A - Thrombose de l'oreillette gauche
B - Embolie plaquettaire dans l'artère vertébrale droite
C - Spasme de l'artère sylvienne gauche
D - Chute du débit cardiaque
E - Petite embolie fibrino-cruorique sylvienne droite
Bonne(s) réponse(s) : E

La réponse est à l'évidence (E), bien qu'une erreur de l'énoncé ou de l'item existe dans la topographie droite
de l'embolie et non gauche (hémiparésie droite).

Une exploration hémodynamique est prévue : dans quelle(s) cavité(s) (enregistrement(s)


simultané(s) des pressions) doi(ven)t être située(s) la(les) sonde(s) pour obtenir le diagnostic
hémodynamique de cette cardiopathie ?
A - Oreillette droite
B - Tronc de l'artère pulmonaire
C - Ventricule gauche
D - Sinus coronaire
E - Capillaire pulmonaire ou oreillette gauche
Bonne(s) réponse(s) : C E

L'enregistrement simultané des pressions dans le capillaire pulmonaire ou l'OG et le VG permettra d'établir le
gradient de pression entre les deux cavités et d'évaluer la surface mitrale en fonction du débit cardiaque
(formule de Gorlin).

Une intervention chirurgicale est décidée et sera effectuée dans quatre semaines. Quel(s)
médicament(s) visant à empêcher la récidive de l'accident neurologique allez-vous
préférentiellement prescrire pendant ces quatre semaines ?
A - Anticoagulants
B - Diurétiques
C - Dérivés quinidiniques
D - Nifédipine (Adalate®)
E - Bêta-bloquants
Bonne(s) réponse(s) : A C

Sans commentaire.

Un étudiant de 23 ans, sans antécédents particuliers, est hospitalisé pour une douleur thoracique l'ayant
réveillé vers 6 heures du matin et s'accentuant depuis. La température est à 38,2° C, le pouls à 92, la TA à
13/7.

Parmi les caractères de la douleur suivants, quel(s) est (sont) celui (ceux) qui oriente(nt) vers le
diagnostic de péricardite ?
A - Irradiation vers l'épigastre
B - Irradiation vers le cou
C - Accentuation lors des mouvements respiratoires
D - Diminution en décubitus latéral
E - Soulagement par les dérivés nitrés
Bonne(s) réponse(s) : C

La douleur est soulagée lors de la position penchée en avant ou assise, et est aggravée par le décubitus.

503
Exclusivement sur DOC - DZ : www.doc-dz.com NADJI 85
RESIDANAT EN POCHE TOME II
Cas Clinique en QCM

L'ECG est suggestif de péricardite s'il montre :


A - Un sous-décalage de ST en dérivations inférieures
B - Un sus-décalage de ST en dérivations antérieures et inférieures
C - Un sous-décalage du segment PQ
D - Des extrasystoles ventriculaires
E - Un bloc de branche incomplet droit
Bonne(s) réponse(s) : B C

Le tracé ECG présente l'évolution classique en quatre stades :


1- Sus-décalage de ST concave en haut
2- Retour de ST à la ligne isoélectrique avec T aplatie
3- T négative
4- Retour de l'onde T à la ligne isolélectrique en quelques semaines ou quelques mois.

La complication la plus fréquente est :


A - La tamponnade
B - L'hémopéricarde
C - La constriction
D - La récidive
E - Aucune de ces complications
Bonne(s) réponse(s) : D

La tamponnade peut également émailler l'évolution d'une péricardie aiguë bénigne mais reste une
complication plus rare (ce QCM sous-entend que l'étiologie de la péricardite est virale).

La constitution d'une tamponnade sera évoquée sur certain(s) élément(s). Le(s)quel(s) ?


A - Accentuation de la douleur
B - Persistance du frottement péricardique au-delà de 48 heures
C - Pouls de Kussmaul
D - Hépatomégalie douloureuse
E - Microvoltage à l'ECG
Bonne(s) réponse(s) : C D E

Le pouls de Kussmaul correspond à une diminution de l'intensité du pouls lors de l'inspiration, le foie
douloureux s'inscrit dans un tableau d'IVD avec turgescence jugulaire ; enfin, le microvoltage traduit surtout
l'abondance de l'épanchement.

Le diagnostic de péricardite aiguë idiopathique (ou virale) ayant été retenu, quelle thérapeutique
sera préconisée ?
A - Antibiotique à large spectre
B - Antiinflammatoire non stéroïdien
C - Corticoïdes
D - Antibiotique + corticoïdes
E - Aucune de ces thérapeutiques
Bonne(s) réponse(s) : B

Le traitement comportera le repos au lit jusqu'à disparition de la douleur et de la fièvre et la prescription


d'aspirine (2 à 3 g/24 h) ou d'anti inflammatoires non stéroïdiens. Le traitement corticoïde dans cette étiologie
n'est pas indiqué dans un premier temps et favoriserait les rechutes.

Un homme de 55 ans, présentant une cardiopathie ischémique vient consulter pour des palpitations
fréquentes.

L'ECG pratiqué montre l'existence d'une extrasystolie ventriculaire importante, parfois bigéminée,
c'est à dire la présence :
A - De complexes QRS fins mais prématurés et précédés d'une onde P
B - De complexes QRS fins prématurés et non précédés d'une onde P
C - De complexes QRS élargis et prématurés
D - De manière intermittente, d'une alternance de complexes sinusaux et complexes QRS
élargis et prématurés
E - De complexes QRS élargis mais non prématurés
Bonne(s) réponse(s) : D

Le bigéminisme ventriculaire est défini par l'alternance d'un complexe sinusal et d'une ESV.

504
Exclusivement sur DOC - DZ : www.doc-dz.com NADJI 85
RESIDANAT EN POCHE TOME II
Cas Clinique en QCM

Devant cette extra-systolie importante, quel est l'examen complémentaire le plus utile ?
A - Faire un échocardiogramme
B - Réaliser un Holter (enregistrement de l'E.C,G, sur 24 heures)
C - Doser les enzymes sériques à la recherche d'un infarctus
D - Faire une épreuve d'effort
E - Réaliser une coronarographie
Bonne(s) réponse(s) : B

Le Holter de 24 heures permet d'apprécier la sévérité de l'hyperexcitabilité ventriculaire (classification de


Lowa).

Entre-temps le malade présente une tachycardie ventriculaire nécessitant son hospitalisation


dans un service spécialisé, l'ECG peut montrer :
A - Une tachycardie très régulière à complexes QRS fins
B - Une tachycardie à complexes QRS élargis
C - Une dissociation des complexes auriculaires et ventriculaires
D - Une tachycardie à complexes QRS élargis et très irréguliers
E - Une tachycardie à complexes QRS fins et irréguliers
Bonne(s) réponse(s) : B C

Exceptionnellement, une tachycardie ventriculaire peut être irrégulière, spontanément ou sous anti-arythmique.

Le traitement immédiat de cette tachycardie ventriculaire peut être :


A - La réalisation d'un choc électrique externe
B - L'injection intraveineuse de digitaliques (Cédilanide®)
C - L'injection intraveineuse de xylocaïne (Xylocard®)
D - Une perfusion de trinitrine intraveineuse (Lénitral®)
E - La compression des globes oculaires
Bonne(s) réponse(s) : A C

La Xylocaïre® est surtout efficace dans les troubles du rythme ventriculaire à la phase aiguë de l'infarctus.

Après retour en rythme sinusal, le traitement antiarythmique pourra faire appel à un ou plusieurs
des produits suivants, isolément ou en association. Le(s)quel(s) ?
A - Une digitalisation au long cours
B - La mexilétine (Mexitil®)
C - L'amiodarone (Cordarone®)
D - Le disopyramide (Rythmodan®)
E - La flécaïnide (Flécaïne®)
Bonne(s) réponse(s) : B C D E

Le traitement antiarythmique préventif fait appel aux médicaments de la classe I ou à l'amiodarone.

Mademoiselle L. Anne-Marie, âgée de 23 ans, n'a aucun antécédent et s'adonne au hand-ball de façon
régulière sans aucune gêne fonctionnelle. Toutefois, lors des derniers entrainements, elle a ressenti à
plusieurs reprises des accès de palpitations irrégulières et gênantes qui l'ont amenée à consulter.
L'interrogatoire apprend qu'entre les accès, aucun symptôme n'est ressenti. Elle suit une contraception par
oestro-progestatif depuis 4 ans, ne fume pas et n'abuse ni du café ni du thé. L'auscultation cardiaque met en
évidence un souffle systolique de pointe. L'échocardiogramme vient confirmer le diagnostic de prolapsus
mitral qu'avait suggéré l'auscultation. Un examen Holter effectué lors d'une journée d'entraînement sportif
montrera effectivement la survenue, lors des efforts les plus importants, d'extrasystoles ventriculaires. Ces
extrasystoles apparaissent également lors de certaines activités diurnes comme la montée rapide de 3 étages.
L'examen somatique est par ailleurs normal, en dehors d'une carie dentaire.

Au cours d'un prolapsus mitral l'auscultation la plus typique montre :


A - Un souffle proto et mésosystolique de pointe
B - Un souffle holosystolique
C - Un click mésosystolique suivi d'un souffle télésystolique
D - Un click mésosystolique
E - Un souffle protosystolique suivi d'un click mésosystolique
Bonne(s) réponse(s) : C

Le souffle traduit une insuffisance mitrale télésystolique.

505
Exclusivement sur DOC - DZ : www.doc-dz.com NADJI 85
RESIDANAT EN POCHE TOME II
Cas Clinique en QCM

Avant tout ECG, en se basant sur l'interrogatoire, les accès de palpitations pouvaient être dus
chez mademoiselle L. à :
A- Des accès de tachycardie jonctionnelle
B - Des accès d'arythmie complète
C - Des crises de flutter auriculaire
D - Des accès d'extrasystoles auriculaires
E - Des accès d'extrasystoles ventriculaires
Bonne(s) réponse(s) : B D E

La fibrillation auriculaire et les extrasystoles sont les troubles du rythme les plus fréquemment rencontrés dans
le prolapsus mitral. Néanmoins, malgré leur caractère irrégulier, les palpitations sont compatibles avec
n'importe quelle arythmie.

L'échocardiogramme au cours du prolapsus mitral montre :


A - Mouvement paradoxal de la petite valve mitrale au cours de la diastole
B - Une butée de la grande valve mitrale contre le septum lors de l'ouverture
C - Un recul télésystolique de la petite valve mitrale
D - Un mouvement systolique antérieur de la grande valve mitrale
E - Une diminution de la pente EF
Bonne(s) réponse(s) : B C

La proposition (D) s'observe dans la cardiomyopathie obstructive.


A et E - Sont des signes de rétrécissement mitral.

Quelles sont parmi ces 5 propositions thérapeutiques, les deux que l'on peut envisager chez cette
patiente ?
A - Abstention
B - Le Vérapamil (Isoptine 120®)
C - Le Proranolol (Avlocardyl®)
D - L'Amiodarone (Cordarone®)
E - La Mexilétine (Mexitil®)
Bonne(s) réponse(s) : A C

La survenue d'extrasystoles isolées sans phénomène répétitif au caractère répétitif, n'impose pas un
traitement antiarythmique. Sinon, les bêta-bloquants sont les plus efficaces dans cette pathologie.

Le prolapsus mitral se rencontre chez la femme jeune :


A - Dans 1 à 2 % des cas
B - Dans 6 à 10% des cas
C - Dans 20 à 25 % des cas
D - Dans 30 à 40 % des cas
E - Dans 50 % des cas
Bonne(s) réponse(s) : B

La fréquence est estimée de 5 à 20 % selon les études.

Parmi les propositions suivantes concernant le prolapsus mitral, une seule est inexacte, précisez
laquelle ?
A - Il peut n'être qu'une simple découverte échocardiographique
B - Peut se compliquer d'endocardite bactérienne
C - Il peut être à la source d'insuffisance mitrale
D - Il atteint le plus souvent la valve mitrale antérieure
E - Peut être à la source de troubles sévères du rythme cardiaque
Bonne(s) réponse(s) : D

L'atteinte de la petite valve est la plus fréquente.

506
Exclusivement sur DOC - DZ : www.doc-dz.com NADJI 85
RESIDANAT EN POCHE TOME II
Cas Clinique en QCM

Vous conseillez chez cette patiente :


A - L'arrêt du hand-ball en compétition
B - L'arrêt des oestro-progestatifs
C - Soins dentaires sous traitement antibiotique
D - La pose d'un stérilet
E - La poursuite de l'éventuel traitement antiarythmique retenu avec Holter de contrôle 3
semaines plus tard
Bonne(s) réponse(s) : A C E

Une prophylaxie de l'endocardite est nécessaire comme dans toutes les valvulopathies.

Une femme de 73 ans présente brutalement une douleur violente du mollet droit. A l'examen, 6 heures plus
tard le membre inférieur est glacé jusqu'au genou. Les pouls fémoraux sont perçus avec un souffle à leur
niveau et le pouls poplité n'est perçu qu'à gauche. Le coeur est arythmique. Il existait une claudication peu
gênante et ancienne. La paralysie du pied est totale.

Parmi ces 5 éléments, quel(s) est (sont) celui (ceux) en faveur d'une origine embolique ?
A - Une paralysie sensitivo-motrice
B - Une arythmie cardiaque
C - Une claudication antérieure
D - Un début brutal
E - Une évolution très rapide
Bonne(s) réponse(s) : B D E

La paralysie sensitivo-motrice ne permet pas de trancher dans le mécanisme de l'ischémie et l'existence d'une
claudication ancienne plaiderait plutôt pour une origine thrombotique aiguë athéromateuse.

Parmi ces 5 éléments, quel(s) est (sont) celui (ceux) en faveur d'une thrombose aiguë ?
A - Une absence de pouls distaux controlatéraux
B - Une claudication antérieure
C - Des souffles fémoraux
D - Une évolution rapide
E - Un début brutal
Bonne(s) réponse(s) : A B C

Tous ces éléments traduisent l'existence d'une artérite antérieure. L'évolution rapide n'est pas en faveur car
l'existence d'une circulation collatérale rend le caractère de l'ischémie souvent incomplet.

L'artériographie est faite en urgence. Elle montrera le plus probablement à droite :


A - Une interruption de la fémorale superficielle au canal de Hunter avec bonne réinjection des
artères de jambe
B - Une interruption de la fémorale superficielle au Hunter et pas de réinjection sous-jacente
C - Une interruption isolée de la fémorale profonde
D - Une interruption de la poplitée basse
E - Une interruption au 1/3 moyen des artères de jambe
Bonne(s) réponse(s) : B

Sans commentaire.

Si la restauration artérielle n'est pratiquée qu'à la 12ème heure. Quelle(s) complication(s) faut-il
craindre ?
A - Acidose métabolique sévère
B - Hypokaliémie brutale
C - OEdème aigu du poumon à la remise en circuit
D - OEdème de revascularisation de la jambe
E - Syndrome hémorragique par hypofibrinémie
Bonne(s) réponse(s) : A D E

L'acidose s'accompagne souvent d'une hyperkaliémie pouvant être menaçante. La stase vasculaire et la
nécrose musculaire peuvent s'accompagner d'une CIVD.

507
Exclusivement sur DOC - DZ : www.doc-dz.com NADJI 85
RESIDANAT EN POCHE TOME II
Cas Clinique en QCM

Devant ce même tableau vu à la 12ème heure, vous retiendriez en faveur de l'amputation


d'emblée ?
A - L'âge
B - Un taux des CPK > 5 000 UI/l
C - Le diagnostic artériographique de thrombose aiguë
D - Le risque d'un oedème de revascularisation
E - Une créatininémie à 350 micromol/l
Bonne(s) réponse(s) : A C E

Le délai supérieur à 8 heures d'ischémie complète, la précarité du terrain et l'existence d'un lit d'aval de
mauvaise qualité sont autant d'arguments pour faire une amputation d'emblée.

Une femme de 45 ans se plaint d'une douleur de la jambe droite depuis 4 jours. La douleur augmente
régulièrement surtout en position debout et à la marche. Aucun antécédent notable. Vous notez un oedème
rouge de 8 cm de longueur sur 2 cm de largeur sur la face interne de la jambe droite. Pas d'oedème par
ailleurs, pas de signe de Homans, pas d'adénopathie, température à 37,5° C. Les pouls périphériques sont
présents.

Pour établir votre diagnostic, vous devez demander :


A - Une radiographie du tibia
B - Une ponction aspiration, pour bactériologie
C - Une phlébographie en urgence
D - Une biopsie des lésions
E - Aucun de ces examens
Bonne(s) réponse(s) : E

Il s'agit d'une phlébite superficielle. Aucun de ces examens n'est nécessaire, sauf en cas d'extension au
niveau de la crosse de la saphène interne, où certains problèmes diagnostiques peuvent être tranchés par
une phlébographie.

Pour votre traitement, vous prescrivez une ou plusieurs de ces mesures :


A - Héparine® intraveineuse 5000 unités toutes les 4 heures
B - Surélévation du membre pendant 5 jours
C - Continuer la marche normalement
D - Antiinflammatoires locaux et/ou généraux
E - Traitement antibiotique pendant 5 jours
Bonne(s) réponse(s) : C D

Le repos au lit est contre-indiqué. Une contention élastique peut être indiquée.

Deux ans plus tard, la patiente vous consulte à nouveau pour une douleur du membre inférieur
gauche avec oedème de cheville et du mollet gauches (2 cm de plus en circonférence du mollet,
par rapport au coté opposé) depuis 6 jours. Vous demandez un ou plusieurs des examens
suivants :
A - Une pléthysmographie et/ou un doppler veineux
B - Une épreuve au fibrinogène marqué
C - Une phlébographie
D - Une angiographie pulmonaire
E - Une radiographie sans préparation du membre inférieur gauche
Bonne(s) réponse(s) : A B C

Il s'agit d'une thrombose veineuse profonde.


Une épreuve au fibrinogène marqué se fait dans quelques centres spécialisés avec une bonne corrélation
avec les résultats de la phlébographie. En pratique, un doppler veineux et surtout la phlébographie sont
suffisants.

508
Exclusivement sur DOC - DZ : www.doc-dz.com NADJI 85
RESIDANAT EN POCHE TOME II
Cas Clinique en QCM

La patiente, après traitement, reprend son travail et ne revient vous voir qu'après deux ans (sans
problème majeur) pour une appendicite aiguë typique. Vous devez lui proposer un ou plusieurs
des traitements suivants :
A - Prescrire de l'héparine par voie sous-cutanée en pré- et post-opératoire
B - Prescrire un traitement anticoagulant per os
C - Mettre en place un clip cave pendant l'appendicectomie
D - Encourager les mouvements actifs et passifs des mollets pendant et après l'intervention
d'appendicectomie
E - Prescrire un traitement thrombolytique
Bonne(s) réponse(s) : A D

Souvent prises de façon systématiques, ces mesures sont formellement indiquées en cas d'antécédents
veineux.

Un homme de 60 ans a une symptomatologie fonctionnelle récente, mais exclusivement à l'effort, depuis 3
mois : un essoufflement, depuis un mois ; des douleurs angineuses ; il y a deux jours, il a eu une nouvelle
douleur angineuse, avec sensation lipothymique, mais sans perte de connaissance.
A l'auscultation, le rythme cardiaque est régulier à 70/mn , il y a un souffle systolique râpeux, maximum au
foyer aortique, avec à ce même foyer une abolition du premier et du deuxième bruit, et un minuscule souffle
diastolique.
L'examen radiologique objective une nette hypertrophie ventriculaire gauche, montre une crosse aortique un
peu élargie dans son segment initial ; le tracé électrique confirme l'hypertrophie ventriculaire gauche associée
à un bloc de branche complet et à un PR à 0,22 sec. L'examen sous amplificateur de brillance visualise de
grosses calcifications orificielles aortiques, mais pas de calcifications sur le trajet présumé des artères
coronaires. Le diagnostic clinique est donc celui de rétrécissement aortique orificiel calcifié serré, à confirmer
au besoin par les examens complémentaires (phonomécanocardiogramme, échocardiogramme,
éventuellement cathétérisme gauche,etc...).

Quel signe d'auscultation permet de préjuger, avant l'examen sous amplificateur de brillance, que
ce rétrécissement aortique orificiel est probablement fortement calcifié ?
A - Faible intensité du souffle
B - Caractère râpeux du souffle systolique d'éjection
C - Abolition de B2 au foyer aortique
D - Souffle diastolique d'accompagnement
E - Click éjectionnel au foyer aortique
Bonne(s) réponse(s) : C

Sans commentaire.

A l'échocardiogramme, quel est le meilleur signe indirect pour apprécier l'importance du


rétrécissement aortique ?
A - Dilatation de l'oreillette gauche
B - Faible ouverture des valves sigmoïdes
C - Dilatation de l'aorte ascendante
D - Hypertrophie concentrique du ventricule gauche
E - Absence de dilatation de la cavité ventriculaire gauche
Bonne(s) réponse(s) : D

L'hypertrophie concentrique du VG est un bon reflet de la sévérité du RA.

Le cathétérisme gauche met en évidence chez ce patient un gradient ventricule gauche-aorte de


90 mm Hg. Quel traitement allez-vous conseiller à ce patient de 60 ans qui a une
symptomatologie exclusivement d'effort ?
A - Digitaliques
B - Inhibiteur de l'enzyme de conversion
C - Dérivés nitrés à action prolongée
D - Remplacement valvulaire aortique
E - Inhibiteurs calciques
Bonne(s) réponse(s) : D

Devant un gradient VG - AO à 90 mm Hg, a fortiori si le patient est symptomatique, il y a une indication de


remplacement valvulaire.

509
Exclusivement sur DOC - DZ : www.doc-dz.com NADJI 85
RESIDANAT EN POCHE TOME II
Cas Clinique en QCM

Ce malade a un angor exclusivement d'effort. Quel examen complémentaire est justifié pour
éliminer formellement une athérosclérose coronarienne sténosante, éventuellement associée au
rétrécissement aortique orificiel ?
A - ECG d'effort
B - Scintigraphie myocardique au thallium à l'effort
C - Enregistrement continu (méthode de Holter) sur 24 heures
D - Dosage des lipides sanguins
E - Aucune des propositions précédentes n'est exacte
Bonne(s) réponse(s) : E

Dans ce tableau, seule la coronarographie, pratiquée en même temps que le cathétérisme gauche, permettra
d'éliminer formellement une athérosclérose cornarienne.
Les tests d'effort ne sont pas indiqués dans le RAO.

Au phonomécanogramme, quelles seront les deux anomalies les plus suggestives du diagnostic
de rétrécissement aortique ?
A - Caractère rectangulaire du souffle systolique
B - Diminution de l'intensité de B1
C - Allongement du temps de demi-ascension du carotidogramme
D - Onde a ample au jugulogramme
E - Allongement du temps d'éjection corrigé
Bonne(s) réponse(s) : C E

Sans commentaire.

Monsieur C. 55 ans, consulte pour une dyspnée d'effort apparue il y a plusieurs années, mais gênante
(marche à allure rapide en terrain plat) depuis quelques semaines seulement. L'auscultation cardiaque
entend, à la pointe et dans l'aisselle, un souffle holosystolique en jet de vapeur d'intensité 3/6 ; elle
n'augmente pas en inspiration forcée. L'examen clinique ne retrouve pas de signes périphériques
d'insuffisance cardiaque. L'ECG montre une fibrillation auriculaire avec rythme ventriculaire à 100/mn. Sur la
radiographie thoracique, on observe une cardiomégalie (ICT = 0,55), une dilatation de l'oreillette et du
ventricule gauches ainsi qu'une hypervascularisation pulmonaire. Le diagnostic d'insuffisance mitrale pure est
retenu.

Dans quelle classe fonctionnelle (classification internationale NYHA) peut-on situer ce patient ?
A - Classe 0
B - Classe 1
C - Classe 2
D - Classe 3
E - Classe 4
Bonne(s) réponse(s) : D

Classe 3 : dyspnée apparaissant pour des efforts légers, gênant la vie courante.

Quel est le signe d'auscultation fréquent dans cette affection et qui manque dans la description
clinique ?
A - Dédoublement de B1
B - Dédoublement de B2
C - Diminution d'intensité de B2
D - Présence d'un B3
E - Présence d'un B4
Bonne(s) réponse(s) : D

L'existence d'un B3 traduit une régurgitation mitrale assez importante.

Quel est parmi ces 5 signes d'auscultation celui qu'il est impossible d'entendre chez ce patient ?
A - B1 assourdi
B - Eclat de B2 au foyer pulmonaire
C - Présence d'un B3
D - Roulement présystolique
E - Roulement protomésodiastolique
Bonne(s) réponse(s) : D

Le roulement diastolique peut accompagner une fuite mitrale pure. La perte de la systole auriculaire due à la
fibrillation auriculaire fait qu'il ne pourra être en aucun cas présystolique.

510
Exclusivement sur DOC - DZ : www.doc-dz.com NADJI 85
RESIDANAT EN POCHE TOME II
Cas Clinique en QCM

Si on examine ce patient à l'amplificateur de brillance, quelle(s) anomalie(s) peut-on observer ?


A - Calcifications mitrales
B - Expansion systolique de l'oreillette gauche
C - Expansion systolique des artères pulmonaires
D - Mouvement de sonnette cardio-aortique
E - Aucune des propositions précédentes
Bonne(s) réponse(s) : A B

Sans commentaire.

Parmi ces diverses étiologies, laquelle(lesquelles) peu(ven)t être à l'origine d'une insuffisance
mitrale ?
A - Rhumatisme articulaire aigu
B - Infarctus du myocarde
C - Traumatisme thoracique
D - Syphilis
E - Endocardite bactérienne
Bonne(s) réponse(s) : A B C E

Bien que certains auteurs aient pu rapporter des atteintes des muscles papillaires d'origine syphilitique et
pouvant être responsable d'IM, cette étiologie est surtout classique dans l'IA.

Parmi ces diverses lésions anatomiques de l'appareil valvulaire et sous-valvulaire,


laquelle(lesquelles) peu(ven)t être responsables d'une IM pure ?
A - Dilatation de l'anneau
B - Calcifications de l'anneau
C - Symphyse commissurale étendue
D - Ballonisation valvulaire
E - Perforation valvulaire
Bonne(s) réponse(s) : A B D E

Sans commentaire.

Quelles sont parmi ces 5 méthodes d'exploration les deux qui permettent le mieux de préciser le
mécanisme lésionnel dans cette affection ?
A - Phonomécanocardiogramme
B - Echocardiogramme bidimensionnel
C - Cathétérisme cardiaque
D - Angiocardiographie ventriculaire gauche
E - Coronarographie
Bonne(s) réponse(s) : B D

Seuls ces deux examens permettent une étude maphologique précise du mécanisme lésionnel.

Une femme de 55 ans, hypertendue ancienne, est adressée aux urgences pour une douleur thoracique
antérieure, irradiant dans l'espace interscapulovertébral ; l'examen clinique trouve un souffle diastolique au
bord gauche du sternum. La pression artérielle est à 180/100 mm Hg. Tous les pouls sont présents.
L'auscultation pulmonaire est normale. Le diagnostic de dissection aiguë de l'aorte ascendante est envisagé.

Toutes les données de l'énoncé sont en faveur de ce diagnostic, sauf une, laquelle ?
A - Douleur de siège précordial
B - Irradiation dorsale
C - Souffle diastolique
D - Pouls périphériques présents et symétriques
E - Antécédents d'HTA
Bonne(s) réponse(s) : D

Une éventuelle asymétrie des pouls constituerait un argument de plus en faveur d'une dissection.

511
Exclusivement sur DOC - DZ : www.doc-dz.com NADJI 85
RESIDANAT EN POCHE TOME II
Cas Clinique en QCM

Parmi ces examens complémentaires, quel est celui qui n'apportera aucun argument
diagnostique ?
A - Electrocardiogramme
B - Radiographie pulmonaire de face
C - Echocardiogramme
D - Scintigraphie myocardique
E - Scanner
Bonne(s) réponse(s) : D

La scintigraphie myocardique n'a vraiment aucun intérêt dans le diagnostic de la dissection aortique.

En urgence, quel est l'examen qui permet de confirmer de façon formelle ce diagnostic et
d'orienter le choix thérapeutique ?
A - Radiographie pulmonaire de face
B - Angiographie par voie veineuse
C - Coronarographie
D - Aortographie rétrograde
E - Echocardiographie
Bonne(s) réponse(s) : D

Seule l'aortographie rétrograde permettra de confirmer le diagnostic et d'apprécier l'extension exacte de la


dissection.

Quel est le risque évolutif majeur ?


A - Insuffisance aortique aiguë
B - Thrombose aortique aiguë
C - Embolie cérébrale
D - Rupture aortique intra-péricardique
E - Anurie
Bonne(s) réponse(s) : D

Sans commentaire.

Quelle est la mesure thérapeutique absolument nécessaire chez cette malade ?


A - Mise aux anticoagulants
B - Mise en route d'un traitement hypotenseur
C - Remplacement de l'aorte ascendante
D - Remplacement valvulaire aortique
E - Traitement antalgique majeur (morphiniques)
Bonne(s) réponse(s) : B

Etant donné les chiffres tensionnels, cette mesure thérapeutique sera prise immédiatement en attendant
l'intervention chirurgicale puisqu'il s'agit ici d'un type I ou II (aorte ascendante).

Il s'agit d'une femme de 45 ans, normotendue, atteinte d'une maladie mitrale connue depuis 20 ans. Elle est
hospitalisée pour oedème aigu du poumon : à l'auscultation, on entend un souffle systolique d'insuffisance
mitrale et un roulement diastolique de rétrécissement mitral. Un bilan est réalisé après traitement de l'accident
aigu.

Parmi les éléments suivants, révélés par la radiographie thoracique, quel est celui qui n'est pas
directement en rapport avec la maladie mitrale ?
A - Un double contour de l'arc inférieur droit
B - Un épanchement pleural
C - Une cardiomégalie
D - Des images de sub-oedème pulmonaire
E - Une saillie de l'arc supérieur droit
Bonne(s) réponse(s) : C

L'arc supérieur droit répond au tronc veineux brachio-céphalique droit.

512
Exclusivement sur DOC - DZ : www.doc-dz.com NADJI 85
RESIDANAT EN POCHE TOME II
Cas Clinique en QCM

L'échocardiogramme mettra en évidence les signes contenus dans les propositions suivantes
sauf une, laquelle ?
A - Une dilatation du ventricule gauche
B - Une ouverture normale des valves aortiques
C - Une dilatation du ventricule droit
D - Une diminution de la pente EF
E - Un fluttering de la grande valve mitrale
Bonne(s) réponse(s) : E

Le fluttering mitral est un signe d'insuffisance aortique.

Un cathérisme cardiaque droit et gauche pourra montrer :


A - Des pressions normales dans les cavités droites
B - Un débit cardiaque à 8 l/minute
C - Une pression systolique de l'artère pulmonaire égale à 20 mm Hg
D - Une pression moyenne capillaire pulmonaire à 20 mm Hg
E - Une pression télédiastolique ventriculaire gauche normale
Bonne(s) réponse(s) : D

Une maladie mitrale évoluant depuis 20 ans, et en oedème pulmonaire, est incompatible avec des pressions
droites et artérielles pulmonaires normales.

Quel est le signe hémodynamique le plus suggestif de l'insuffisance mitrale ?


A - La grande onde X sur la courbe de pression de l'oreillette droite
B - La grande onde V sur la courbe de pression capillaire pulmonaire
C - Le "dip plateau"
D - L'augmentation des pressions dans l'artère pulmonaire
E - L'élévation du débit cardiaque
Bonne(s) réponse(s) : B

L'onde V capillaire est en général d'autant plus élevée que l'insuffisance mitrale est plus sévère.

Parmi les propositions suivantes, quel est le traitement à éviter ?


A - Tonicardiaques
B - Diurétiques
C - Vasodilatateurs veineux
D - Inhibiteurs de l'enzyme de conversion
E - Bêta-bloquants
Bonne(s) réponse(s) : E

La présence d'insuffisance ventriculaire gauche (OAP récent) contre-indique l'emploi des bêta-bloquants.

Une jeune femme de 35 ans est porteuse depuis l'âge de 25 ans d'une cardiopathie soufflante parfaitement
bien supportée sur le plan fonctionnel.
Depuis quelques jours, elle se plaint d'une dyspnée d'effort progressive avec sensation de palpitations.
A l'examen, on note :
- une séméiologie de retrécissement mitral, roulement diastolique très net en décubitus latéral gauche
- des râles crépitants aux deux bases
- une tension artérielle à 120/70 mm Hg
- une température à 36°9
La radiographie thoracique montre un aspect floconneux du quart inférieur des deux champs pulmonaires, un
arc moyen gauche convexe et un aspect de double contour au niveau de l'arc inférieur droit.
A l'ECG, le rythme est irrégulier à 130/minute avec des complexes QRS fins sans ondes P visibles.

Parmi les signes d'auscultation suivants, quel est celui qui ne peut être trouvé chez cette patiente
?
A - Eclat de B2 au foyer pulmonaire
B - Eclat du premier bruit
C - Renforcement présystolique du roulement diastolique
D - Claquement d'ouverture de la mitrale
E - Souffle systolique xiphoïdien augmentant en inspiration
Bonne(s) réponse(s) : C

Le renforcement présystolique du roulement, généré par la systole auriculaire, fait défaut en cas de fibrillation
auriculaire.

513
Exclusivement sur DOC - DZ : www.doc-dz.com NADJI 85
RESIDANAT EN POCHE TOME II
Cas Clinique en QCM

Quelle est chez cette patiente, la cause la plus probable de la brusque aggravation de son état
hémodynamique ?
A - Rupture de cordages
B - Endocardite bactérienne
C - Embolie pulmonaire
D - Fibrillation auriculaire
E - Pneumopathie aiguë
Bonne(s) réponse(s) : D

La contribution de la contraction auriculaire au remplissage ventriculaire est en effet importante dans le RM.

Quelle(s) complication(s) peu(ven)t survenir chez cette patiente ?


A - OEdème aigu du poumon
B - Mort subite
C - Embolie artérielle
D - Tachycardie ventriculaire
E - Insuffisance cardiaque droite
Bonne(s) réponse(s) : A C E

Les troubles du rythme ventriculaire sont rares dans la sténose mitrale.

Que! médicament vous parait le plus indiqué pour ralentir la fréquence cardiaque de cette
patiente ?
A - Amiodarone (Cordarone®)
B - Digitalique
C - Verapamil (Isoptine®)
D - Bêta-bloquant
E - Sel de Quinidine
Bonne(s) réponse(s) : B

Les digitaliques sont prescrits de première intention.

Quel(s) autre(s) médicament(s) associez-vous dans l'immédiat ?


A - Antivitamine K
B - Antiagrégant plaquettaire
C - Héparine
D - Diurétique
E - Lidocaïne (Xylocaïne®)
Bonne(s) réponse(s) : C D

Une anticoagulation efficace doit être obtenue rapidement d'où l'emploi de l'héparine.

Monsieur H. 25 ans, n'a aucun antécédent pathologique si ce n'est la découverte, à la visite d'incorporation,
d'un petit souffle systolique au foyer aortique. Il y a quinze jours, il a subi deux extractions dentaires. Six jours
plus tard est apparue une fièvre à 39° C sans point d'appel particulier, le médecin traitant a prescrit une
ampicilline (2 g/j per os) pendant 8 jours qui a abaissé la température à 38° C. La nuit dernière, il a été réveillé
par une douleur thoracique rétrosternale constrictive et angoissante, rapidement accompagnée d'une
polypnée intense. A l'admission, on observe un tableau de sub oedème pulmonaire ; le pouls est à 100/mn ; la
TA est à 12/4 cm Hg ; l'auscultation cardiaque entend en bord gauche de sternum un souffle protosystolique
3/10 et un souffle holodiastolique 6/10 ; les bruits sont normaux au foyer aortique ; à la pointe, on entend un
B3 suivi d'un roulement diastolique. Le diagnostic d'endocardite bactérienne est évoqué.

Compte-tenu du tableau clinique actuel, quelle est la nature la plus probable de l'atteinte
valvulaire ?
A - Insuffisance aortique pure ou prédominante
B - Maladie aortique
C - Insuffisance aortique + rétrécissement mitral
D - Maladie aortique + rétrécissement mitral
E - Insuffisance tricuspidienne
Bonne(s) réponse(s) : A

L'insuffisance aortique importante s'accompagne d'un souffle systolique de débit.

514
Exclusivement sur DOC - DZ : www.doc-dz.com NADJI 85
RESIDANAT EN POCHE TOME II
Cas Clinique en QCM

Parmi ces éléments tirés de l'histoire clinique, la sévérité de la ou des lésion(s) valvulaire(s) est
traduite par :
A - Tachycardie à 100/mn
B - TA à 12/4
C - B3.
D - Intensité du souffle diastolique
E - Roulement diastolique à la pointe
Bonne(s) réponse(s) :

QUESTION ANNULEE

Compte-tenu de la porte d'entrée et en attendant les résultats des hémocultures, quel est le
germe le plus probablement en cause ?
A - Staphylocoque doré
B - Staphylocoque epidermidis
C - Streptocoque alpha
D - Entérocoque
E - Pneumocoque
Bonne(s) réponse(s) : C

Le streptoccoque non groupable est à l'origine de 35% des endocardites et est le germe le plus souvent
rencontré devant une porte d'entrée dentaire.

En se fondant sur les mêmes arguments, lequel parmi ces 5 schémas d'antibiothérapie vous
semble le plus logique en première intention ?
A - Ampicilline
B - Erythromicine
C - Pénicilline G + Gentamycine
D - Oxacilline + Gentamycine
E - Cephalosporine + Gentamycine
Bonne(s) réponse(s) : C

Une association de deux antibiotiques bactéricides est nécessaire. Les streptoccoques sont en règle très
sensibles à la pénicilline G (exceptés ceux du groupe D).

Les hémocultures restent négatives. Lequel(lesquels) parmi ces 5 examens complémentaires


serait(ent) susceptible(s) d'affirmer le diagnostic d'endocardite bactérienne ?
A - Dosage des antistreptolysines
B - Dosage du complément et de ses fractions
C - Dosage des complexes immuns circulants
D - Echocardiogramme
E - Cathétérisme cardiaque
Bonne(s) réponse(s) : D

La mise en évidence de végétations valvulaires permet d'affirmer le diagnostic d'endocardite.

Compte-tenu de la localisation des lésions valvulaires, à quelle(s) complication(s) évolutive(s) ce


patient est-il exposé ?
A - Insuffisance ventriculaire gauche aiguë
B - Bloc auriculo-ventriculaire
C - Embolie systémique
D - Shunt gauche droit acquis
E - Anévrisme artériel
Bonne(s) réponse(s) : A B C E

La survenue d'abcès septal peut être à l'origine de trouble conductif.

515
Exclusivement sur DOC - DZ : www.doc-dz.com NADJI 85
RESIDANAT EN POCHE TOME II
Cas Clinique en QCM
Une femme âgée de 50 ans, se plaint d'une dyspnée d'effort. Elle n'a pas d'antécédents connus de RAA
L'auscultation cardiaque permet d'entendre un roulement diastolique et un claquement d'ouverture mitrale. Sur
l'électrocardiogramme, il existe des signes d'hypertrophie auriculaire gauche. La radiographie pulmonaire met
en évidence un aspect de "poumon mitral". Il existe un aspect typique sur l'échocardiogramme. Le
cathétérisme met en évidence une élévation de la pression capillaire pulmonaire. Tous ces signes sont
évocateurs d'une sténose mitrale.

Les signes auscultatoires chez cette femme atteinte de sténose mitrale sont dominés par un
roulement diastolique qui est théoriquement caractérisé par :
A - Siège à la pointe
B - Mieux perçu en décubitus latéral gauche
C - Débute immédiatement après le B2
D - Diminue à l'effort
E - Diminue lors de l'arythmie complète
Bonne(s) réponse(s) : A B E

Il débute après le claquement d'ouverture mitrale, augmente à l'effort. Le renforcement présystolique de ce


roulement disparaît en fibrillation auriculaire.

Les signes électrocardiographiques comportent vraisemblablement :


A - Un allongement de P supérieur à 12/100ème de seconde
B - Une augmentation de l'amplitude de P supérieure à 2,5 mm en D2
C - Un aspect biphasique de P en V1 avec négativité finale empâtée
D - Une déviation axiale droite
E - Une hypertrophie ventriculaire gauche
Bonne(s) réponse(s) : A C D

Dans le RM pur, il n'y a pas d'HVG mais il peut y avoir des signes de retentissement ventriculaire droit. A et C
sont des signes d'hypertrophie auriculaire gauche.
B est un signe d'hypertrophie auriculaire droite.

Sur l'échocardiogramme, vous devez vraisemblablement retrouver tous les signes suivants sauf
un, lequel ?
A - Des mouvements réduits de la grande valve avec aspect en créneau
B - Une absence d'onde A
C - Un épaississement des feuillets
D - Une aire mitrale inférieure à 0.8 cm2
E - Un mouvement postérieur du feuillet postérieur de la mitrale durant la diastole
Bonne(s) réponse(s) : E

En général, il existe un mouvement paradoxal antérieur de la petite valve (feuillet postérieur) qui suit la grande
valve.

Sur la téléradiographie de coeur. on retrouve vraisemblablement tous les signes suivants, sauf
un, lequel ?
A - Une augmentation de volume des hiles pulmonaires
B - Des lignes de Kerley
C - Une image de double contour des oreillettes au niveau du bord droit du coeur
D - Un allongement de l'arc inférieur gauche
E - Un arc moyen gauche en double bosse
Bonne(s) réponse(s) : D

L'allongement de l'arc inférieur gauche traduirait une dilatation du ventricule gauche qui n'existe pas. On peut
voir néanmoins un déplacement de l'arc inférieur gauche par une dilatation du VD avec une pointe sus-
diaphragmatique.

Du point de vue hémodynamique, cette sténose mitrale peut se traduire par tous les signes
suivants, sauf un, lequel ?
A - Une élévation de la pression artérielle pulmonaire
B - Une augmentation de la pression télédiastolique du ventricule gauche
C - Une ébauche de gradient supérieur ou égal à 15 mmHg entre PAP moyenne et pression
capillaire moyenne
D - Une élévation des pressions dans l'oreillette droite
E - Une élévation de la pression télédiastolique ventriculaire droite
Bonne(s) réponse(s) : B

Le ventricule gauche étant normal et la sténose mitrale située en amont, il n'y a aucune raison d'observer une
augmentation de la pression télédiastolique du ventricule gauche.
516
Exclusivement sur DOC - DZ : www.doc-dz.com NADJI 85
RESIDANAT EN POCHE TOME II
Cas Clinique en QCM
Un homme de 52 ans est hospitalisé en urgence pour une violente douleur thoracique. On le trouve assis
dans son lit, penché en avant, pâle et couvert de sueurs.
La respiration est courte et rapide, la douleur de siège rétrosternal est constrictive, elle irradie vers le cou et
les deux épaules. Elle est intense et semble augmenter avec les mouvements respiratoires.
A l'examen, le rythme est régulier à 110/mn, la T.A. est à 120/80 mmHg, les bruits du coeur sont normaux, la
température est de 38,5 degrés, l'auscultation pulmonaire est normale, les pouls sont normalement perçus.

Le ou les signes qui vous font penser que le diagnostic de péricardite aiguë est plus probable que
celui d'infarctus myocardique est ou sont :
A - Le caractère constrictif et la localisation de la douleur
B - L'accentuation de la douleur lors des mouvements respiratoires
C - La position penchée en avant du malade
D - La survenue brutale de la douleur
E - La fièvre à 38°5 degrés
Bonne(s) réponse(s) : B C E

La douleur thoracique de la péricardite peut imiter dans son caractère constrictif une douleur d'origine
coronarienne ; néanmoins, dans ce cas, l'accentuation des douleurs lors de la respiration, la position
antalgique penchée en avant et la fièvre d'emblée oriente vers une origine péricardique.

Tous ces examens peuvent contribuer à faire le diagnostic de péricardite. Lequel envisagez-vous
en dernier recours ?
A - Radiographie du thorax
B - ECG
C - Ponction péricardique
D - Echocardiographie
E - Dosage des enzymes cardiaques
Bonne(s) réponse(s) : C

La ponction péricardique à visée diagnostique est sans aucun doute à réaliser en dernier recours pour
préciser l'étiologie de la péricardite mais en aucun cas pour en faire le diagnostic ; elle est en général réservée
aux formes sub-aiguës d'évolution traînante.
La ponction péricardique à visée décompressive est par contre plus fréquemment réalisée en cas de
tamponnade.

Quelles sont la ou les anomalies électrocardiographiques suggestives d'une péricardite aiguë ?


A - Sous décalage de ST en D2.D3.VF
B - Sous décalage de ST en toutes dérivations
C - Sous décalage de PQ
D - Présence d'extrasystoles ventriculaires
E - Aspect S1 Q3
Bonne(s) réponse(s) : B

Il s'agit classiquement d'un sus-décalage de ST concave vers le haut évoluant en 4 stades avec retour à la
ligne isoélectrique puis renégativation de l'onde T ; ces anomalies siègent en règle dans toutes les dérivations.
On peut également rencontrer un sous-décalage de PQ, des troubles du rythme supraventriculaire (ESA, FA),
une alternance électrique ou un microvoltage en cas d'épanchement abondant.

On retient le diagnostic de péricardite aiguë idiopathique. Quelle est dans cette liste la modalité
évolutive la plus improbable ?
A - Guérison
B - Normalisation de l'ECG en 15 jours
C - Possibilité de rechute
D - Survenue d'un épanchement liquidien péricardique
E - Risque de constriction péricardique
Bonne(s) réponse(s) : E

Le risque d'évolution d'une péricardite aiguë idiopathique vers une péricardite constrictive est très rare.

517
Exclusivement sur DOC - DZ : www.doc-dz.com NADJI 85
RESIDANAT EN POCHE TOME II
Cas Clinique en QCM

Quelle est la thérapeutique la mieux adaptée ?


A - Repos au lit tant qu'il y a de la fièvre
B - Repos au lit et la prescription d'anti-inflammatoires non steroïdiens
C - Corticoïdes
D - Antibiothérapie
E - Corticoïdes + antibiothérapie
Bonne(s) réponse(s) : B

Le traitement classique de la péricardite aiguë idiopathique comprend le repos au lit, la prescription d'anti-
inflammatoires non stéroïdiens ou de salicylés, et éventuellement d'antalgiques.
Les corticoïdes sont à éviter, car ils pourraient favoriser les rechutes.

Monsieur D..., est un homme de 78 ans, polyvasculaire, insuffisant respiratoire chronique sur bronchite
chronique post-tabagique, hospitalisé à de nombreuses reprises en pneumologie dont une fois en réanimation
pour détresse respiratoire aiguë ayant nécessité une ventilation assistée.
Il se présente aux urgences de votre hôpital pour :
- Polypnée brutale
- Râles bronchiques bilatéraux
- Oedèmes des membres inférieurs prédominant à gauche nettement plus inflammatoires de ce côté
- Radiographie pulmonaire inchangée
- PaO2 à 40 mmHg, PaC02 à 39 mmHg (gazométrie artérielle antérieure en état stable : PaO2 60 mmHg,
PaC02 45 mmHg)
Une embolie pulmonaire grave est suspectée.

La gravité de cette embolie pulmonaire est définie par :


A - Tachycardie supérieure à 120/mn
B - PaO2 < 50 mm Hg
C - Signes droits électrocardiographiques
D - Chute de la pression artérielle systolique
E - Syncope
Bonne(s) réponse(s) : B D

La réponse à ce QCM est discutable : la sévérité de l'hypoxémie et les signes de choc avec baisse de la
pression artérielle sont les 2 éléments traduisant le plus la gravité de cette EP ; mais la tachycardie
importante, les signes de coeur pulmonaire aigu à l'ECG et les formes syncopales se voient plus souvent dans
les EP graves.
La réponse ABCDE pourrait donc être également acceptée.

Parmi les propositions suivantes concernant le diagnostic d'embolie pulmonaire, une seule
affirmation est vraie, laquelle ?
A - Une radiographie pulmonaire normale élimine le diagnostic
B - Un électrocardiogramme normal élimine le diagnostic
C - Une scintigraphie pulmonaire normale élimine le diagnostic
D - La P02 n'est jamais supérieure à 80 mm Hg dans l'embolie pulmonaire
E - Devant une embolie pulmonaire, la phlébocavographie retrouve une phlébite dans 90% des
cas
Bonne(s) réponse(s) : C

Il n'y a pas de faux négatifs à la scintigraphie pulmonaire mais uniquement des faux positifs.

Le malade est alors mis à l'héparine à doses efficaces. L'angiographie pulmonaire met en
évidence une obstruction complète de l'artère pulmonaire gauche. L'état du malade s'aggrave. Un
traitement fibrinolytique est alors décidé. Il est exact que ce traitement :
A - Est réservé aux embolies pulmonaires avec signes hémodynamiques
B - Peut être administré par voie veineuse centrale
C - Est formellement contre-indiqué en période post-opératoire
D - Doit être précédé d'une preuve angiographique du diagnostic
E - S'administre exclusivement en continu sur 24 heures
Bonne(s) réponse(s) : A B C D

Les indications des traitements fibrinolytiques concernent les embolies pulmonaires graves sur le plan
hémodynamique (état de choc et signes d'insuffisance ventriculaire droite), prouvées par une angiographie
pulmonaire (montrant une amputation d'au moins 40 %) ; l'administration peut se faire par voie veineuse
centrale ou périphérique ; souvent l'administration en bolus de très fortes doses est également possible.
Exemple de posologie : streptokinase après 100 mg d'hémisuccinate d'hydrocortisone pour éviter les allergies
: 250 000 UI / 30 minutes (dose de charge) puis 100 000 UI / heure pendant 24 heures.

518
Exclusivement sur DOC - DZ : www.doc-dz.com NADJI 85
RESIDANAT EN POCHE TOME II
Cas Clinique en QCM

Sous ce traitement, son état clinique va s'améliorer permettant de pratiquer une


phlébocavographie qui montre un thrombus fémoro-iliaque gauche. Trois jours après l'épisode
initial, survient une hémorragie digestive de grande abondance, alors que l'anticoagulation n'était
pas excessive. Dès lors vous pouvez :
A - Interrompre le traitement anticoagulant
B - Poursuivre de l'héparinothérapie aux mêmes doses
C - Prendre le relais par antivitamines K
D - Interrompre chirurgicalement la veine cave
E - Mettre en place un filtre de Greenfield
Bonne(s) réponse(s) : A D E

L'hémorragie digestive de grande abondance impose l'arrêt du traitement anticoagulant et l'existence d'un
thrombus fémoro-iliaque gauche impose d'interrompre chirurgicalement la veine cave par un clip ou de mettre
en place un filtre de Greenfiels, afin d'éviter d'autres migrations pulmonaires.

L'interruption cave en cas d'embolie pulmonaire est indiquée :


A - Devant un tableau d'embolie pulmonaire récidivante malgré le traitement anti-coagulant
B - Devant un accident des anticoagulants
C - Devant une contre-indication absolue de l'héparinothérapie à doses efficaces
D - Devant un caillot flottant iliaque
E - Devant un accident vasculaire cérébral récent chez un sujet âgé
Bonne(s) réponse(s) : A B D

Une contre indication au traitement anticoagulant au long cours est également une contre-indication mais
dans l'item C, on n'envisage qu'une contre-indication à l'héparine qui pourrait éventuellement faire utiliser les
antivitamines K.
D est discutable : en général l'interruption cave se fait surtout dans les caillots flottants de la veine cave, mais
cette indication peut être étendu aux caillots iliaques.

Une femme de 32 ans présente depuis ses deux grossesses des varices volumineuses et se plaint de
lourdeurs douloureuses des mollets, particulièrement en fin de journée.
A l'examen les varices sont volumineuses, non pulsatiles, strictement unilatérales et ne s'étendent pas à
l'abdomen. En position debout, vous palpez au Scarpa un reflux à la toux. A la manoeuvre de Trendelenbourg,
il n'existe pas de remplissage des veines en position debout tant que le garrot est serré à la cuisse. Lorsqu'on
la fait marcher avec un garrot serré sous le genou, les varices diminuent.

Quelle est l'origine des varices ?


A - Reflux saphène interne
B - Reflux saphène externe
C - Reflux par les perforantes
D - Thrombose iliaque
E - Fistule artérioveineuse
Bonne(s) réponse(s) : A

Le reflux à la toux au niveau du Scarpa traduit une incontinence de la valvule ostiale de la saphène interne.
L'épreuve de Perthes (varices diminuant à la marche avec un garrot) montre que le réseau profond est normal.

Ces varices peuvent se compliquer de :


A - Rupture
B - Thrombose veineuse profonde
C - Ulcère variqueux
D - OEdème
E - Psoriasis localisé
Bonne(s) réponse(s) : A C D

Une autre complication est la thrombose veineuse superficielle. Enfin les thromboses veineuses profondes
peuvent se compliquer de varices mais l'inverse n'est pas vrai.

519
Exclusivement sur DOC - DZ : www.doc-dz.com NADJI 85
RESIDANAT EN POCHE TOME II
Cas Clinique en QCM

Quel traitement choisissez-vous, compte tenu des faits suivants : cette femme travaille, l'état
général est bon, elle est porteuse d'un stérilet :
A - Sclérose
B - Pas à varices
C - Eveinage saphène interne
D - Cure thermale
E - Traitement phlébotonique
Bonne(s) réponse(s) : C

L'importance des varices et le fait qu'il s'agisse d'une incontinence de la valvule ostiale de la saphène interne
plaident pour un traitement chirurgical, d'autant que le réseau profond semble normal (contre-indication à la
chirurgie dans le cas contraire). Les scléroses sont plutôt réservées aux varices tronculaires modérées et aux
varicosités.

Si cette femme était enceinte, quel(s) traitement(s) pourrai(en)t être propose(s) ?


A - Sclérose
B - Bas à varices
C - Eveinage chirurgical
D - Cure thermale
E - Traitement anticoagulant
Bonne(s) réponse(s) : B

La sclérose et le traitement chirurgical sont contre-indiqués au cours de la grossesse dans la mesure où les
varices peuvent partiellement régresser après l'accouchement.

Monsieur R..., 45 ans, gros fumeur (40 Cigs/j), sans antécédent pathologique, notable est admis d'urgence à
l'hôpital car depuis deux heures il ressent une violente douleur thoracique antérieure constrictive, irradiant
dans le bras gauche et la mâchoire inférieure : son médecin traitant lui a administré trois bouffées d'un spray
de trinitrine à quelques minutes d'intervalle sans le moindre effet. A l'admission, le pouls est rapide (90/mn) et
irrégulier avec impression de bigéminisme, l'auscultation cardiaque, en dehors du bigéminisme, est normale, il
n'existe pas de signes d'insuffisance cardiaque congestive. L'E.C.G. ci-dessous est enregistré :

Cliquez sur le bouton "Dessin" pour afficher l'ECG.

Devant ce tableau clinique très évocateur d'insuffisance coronarienne aiguë, quel(s) est(sont)
parmi ces cinq caractères de la douleur celui(ceux) qui d'emblée oriente(nt) vers un diagnostic
d'infarctus du myocarde plutôt qu'une crise angineuse prolongée ?
Cliquez sur le bouton 'Dessin' pour afficher le schéma.
A - Siège de la douleur
B - Irradiations de la douleur
C - Son caractère constrictif
D - Sa durée
E - L'inefficacité des dérivés nitrés perlinguaux
Bonne(s) réponse(s) : D E

A B C - Sont des caractères communs à la crise d'angor et à l'infarctus du myocarde.

Quelle est la localisation E.C.G. de ce probable infarctus ?

Cliquez sur le bouton 'Dessin' pour afficher le schéma.


A - Antéro septal
B - Antéro latéral
C - Antérieur étendu
D - Inférieur
E - Postérieur
Bonne(s) réponse(s) : C

L'aspect d'ischémie sous-endocardique (onde T amples et pointues) et de lésion sous-épicardique est visible
de V2 à V5.
Il s'agit donc probablement d'un infarctus antérieur étendu.

520
Exclusivement sur DOC - DZ : www.doc-dz.com NADJI 85
RESIDANAT EN POCHE TOME II
Cas Clinique en QCM

Quel est parmi ces cinq signes présents sur l'E.C.G., celui qui apparaît le plus précocement dans
l'infarctus transmural ?
Cliquez sur le bouton 'Dessin' pour afficher le schéma.
A - Onde Q de nécrose
B - Ischémie sous-endocardique (onde T ample, positive et symétrique)
C - Ischémie sous-épicardique (onde T négative et symétrique)
D - Sous-décalage ischémique de ST
E - Sus-décalage ischémique de ST
Bonne(s) réponse(s) : B

B est le plus précoce (3 premières heures) et le plus transitoire. Dans les heures qui suivent, apparition de E
puis de A.

A ce stade de l'évolution, lequel parmi ces dosages enzymatiques permet déjà d'affirmer le
diagnostic ?

Cliquez sur le bouton 'Dessin' pour afficher le schéma.


A - Transaminases SGOT
B - CPK totale
C - CPK-MB
D - Alpha HBDH
E - Aucune des propositions précédentes
Bonne(s) réponse(s) : E

Les CPK sont les premiers enzymes à s'élever mais en règle pas avant la 6ème heure.

Quel traitement d'urgence proposez-vous vis-à-vis du trouble du rythme présent sur l'ECG
d'admission ?

Cliquez sur le bouton 'Dessin' pour afficher le schéma.


A - Aucun traiement car cette arytmie est bénigne
B - Sulfate d'atropine 1 mg IVD
C - Cédilanide® : 1 amp. IVD
D - Xylocaïne® : 50 mg IVD lente, puis 2 mg/kg/h IV
E - Cordarone® : per os avec dose de charge
Bonne(s) réponse(s) : D

La xylocaïne est le médicament de choix dans les arythmies ventriculaires à la phase aiguë de l'infarctus du
myocarde.
La réponse E aurait également put se discuter mais l'apparition de taux sériques efficaces après une dose de
charge de cordarone est retardée et donc l'action sera moins immédiate qu'avec la xylocaïne.

Compte-tenu des délais d'admission très précoces, vous décidez d'instituer immédiatement un
traitement visant à limiter la taille de l'infarctus. Parmi ces cinq propositions, Laquelle(lesquelles)
a(ont) une efficacité démontrée dans cette indication ?
Cliquez sur le bouton 'Dessin' pour afficher le schéma.
A - Héparine IV continue
B - Cordarone IV 50 mg/h
C - Nifédipine (Adalate®) 20 mg per os/6h
D - Aténolol (Ténormine®) 5 mg IV puis relais précoce
E - Streptokinase (Streptase®) 1 500 000 U IV/60 mn
Bonne(s) réponse(s) : D E

Les fibrinolytiques administrés précocement (avant la quatrième heure) permettent de limiter la taille de
l'infarctus en reperméabilisant l'artère coronaire thrombosée (étude GISSI par exemple). L'aténolol en IV puis
per os a également fait preuve d'une efficacité dans la prévention de la mortalité précoce et tardive, et la
prévention des récidive d'infarctus (étude ISIS).

521
Exclusivement sur DOC - DZ : www.doc-dz.com NADJI 85
RESIDANAT EN POCHE TOME II
Cas Clinique en QCM

Un patient de 50 ans a présenté trois crises douloureuses caractéristiques d'angine de poitrine d'effort. Un test
ergométrique est réalisé et montre l'existence à la deuxième minute d'un sous-décalage du segment ST
oblique descendant de - 2 mm, sans douleur. La tension artérielle chute de 15/8 à 10/7 cm Hg pendant l'effort.

L'épreuve d'effort est :


A - Négative
B - Positive
C - Ininterprétable
D - Suspecte
E - Normale
Bonne(s) réponse(s) : B

Critère de positivité : sous-décalage de ST > à 1 mm à partir du point J, de durée > à 0,06 sec avec un
segment ST horizontal ou descendant.

L'absence de douleurs est un argument :


A - De positivité
B - De négativité
C - Indifférent
D - Rendant suspecte la valeur de l'épreuve
E - Qui oblige à renouveler le test
Bonne(s) réponse(s) : C

Le seuil "douloureux" peut être nettement supérieur au seuil "ischémique" chez certains sujets.

La chute tensionnelle pendant l'effort peut être un critère de :


A - Faible sévérité de la maladie coronaire
B - Forte sévérité de la maladie coronaire
C - D'improbabilité de maladie coronaire
D - De lésions spathiques
E - De dysfonction ventriculaire gauche
Bonne(s) réponse(s) : C E

La chute tensionnelle à l'effort doit faire arrêter l'épreuve et traduit en l'absence d'infarctus antérieur une
sévérité et une étendue de l'ischémie myocardique à l'effort responsable d'une dysfonction ventriculaire
gauche. Cette chute tensionnelle peut également s'observer en cas de cardiopathie évoluée quelque soit son
origine traduisant alors une inadaptation de la performance ventriculaire gauche à l'effort.

Avec les arguments précédents il faut :


A - Faire une coronarographie
B - Faire une scintigraphie myocardique
C - Faire une stimulation atriale
D - Ne pas faire d'autres examens
E - Donner de l'Aspirine®
Bonne(s) réponse(s) : A

La coronarographie s'impose en raison de la sévérité probable des lésion coronaires.

En cas de survenue de douleurs spontanées chez ce patient, il faut :


A - Faire une scintigraphie d'effort
B - Faire une radiographie thoracique de face
C - Donner un antalgique
D - Faire admettre le patient en unité de soins intensifs
E - Répéter l'épreuve d'effort E.C.G.
Bonne(s) réponse(s) : D

La survenue de douleurs spontanées chez ce patient qui présentait jusque là uniquement un angor d'effort
authentifié par une épreuve d'effort très positive (importance du sous-décalage qui survient pour un faible
palier) doit être considéré comme un angor instable et justifie l'hospitalisation en unité de soins intensifs.

522
Exclusivement sur DOC - DZ : www.doc-dz.com NADJI 85
RESIDANAT EN POCHE TOME II
Cas Clinique en QCM
Monsieur G... âgé de 56 ans, a présenté à plusieurs reprises depuis huit jours, des malaises lipothymiques à
la faveur d'accès de palpitations très rapides et irrégulières à début brutal et fin progressive.
Lorsque vous le voyez, vous constatez, en l'absence de tout traitement, un rythme cardiaque lent et régulier à
36 pulsations/mn. L'auscultation est par ailleurs normale. La TA est à 13/8 cmHg. Il n'y a pas de signes
d'insuffisance cardiaque. L'E.C.G. confirme la bradycardie à 36, chaque onde P étant suivie d'un QRS. La
radio de thorax est normale.

Les accès de palpitations correspondent probablement chez ce patient à :


A - Une tachycardie sinusale
B - Une tachyarythmie complète par fibrillation auriculaire
C - Un flutter auriculaire 2/1
D - Une tachysystolie auriculaire
E - Une tachycardie jonctionnelle
Bonne(s) réponse(s) : B

Le caractère irrégulier, le début brutal et la fin progressive des crises de tachycardie sont en faveur d'une
fibrillation auriculaire.

Compte tenu du contexte clinique, la description de l'E.C.G. est en faveur :


A - D'une bradycardie sinusale physiologique
B - D'un bloc sino-auriculaire
C - D'un bloc auriculo-ventriculaire complet
D - D'un bloc auriculo-ventriculaire avec période de Luciani Wenckebach
E - D'un bloc auriculo-ventriculaire 2/1
Bonne(s) réponse(s) : B

Compte tenu de la survenue de crises de tachycardie paroxystique, l'enregistrement de cette bradycardie


sinusale doit faire suspecter soit un syndrome bradycardie - tachycardie soit une maladie arythmique
auriculaire, avec dysfonction sinusale.

L'examen complémentaire à envisager en priorité est :


A - Un enregistrement holter
B - Un échocardiogramme
C - Une exploration électrophysiologique endocavitaire
D - Une épreuve d'effort sur cyclo ergomètre
E - Un doppler cervico encéphalique
Bonne(s) réponse(s) : A

L'enregistrement de Holter est l'examen à faire en priorité : il permettra d'apprécier la sévérité de la


bradycardie sinusale, sa variation au cours du nycthémère, l'existence de pauses (surtout si > à 2,5 sec), le
nombre d'épisodes éventuels de fibrillation auriculaire avec leur durée et leur fréquence et enfin la succession
de ces différentes phases les unes par rapport aux autres.

Le traitement comportera :
A - Un agent sympatho-mimétique (Isuprel)
B - L'Atropine®
C - Un entraînement électrosystolique endocavitaire
D - Un traitement antiarythmique
E - Un bêta-bloquant cardio sélectif
Bonne(s) réponse(s) : C D

Il faut traiter les accès paroxystiques de FA pour prévenir leur récurrence dans la mesure où ils sont mal
tolérés d'une part et où ils risquent toujours d'occasionner des accidents emboliques d'autre part. Ce
traitement anti-arythmique risquant d'aggraver la dysfonction sinusale, un entraînement électro-systolique
endocavitaire permanent par un stimulateur est indiqué (si la question fait appel à la conduite à tenir
immédiate, pourront se discuter A ou C, bien que cette bradycardie soit bien tolérée).

523
Exclusivement sur DOC - DZ : www.doc-dz.com NADJI 85
RESIDANAT EN POCHE TOME II
Cas Clinique en QCM
Mr. J., 65 ans, est hospitalisé pour une douleur thoracique gauche avec dyspnée importante.
Histoire de la maladie : il y a 3 jours, douleur thoracique gauche d'apparition brutale, avec dyspnée et
angoisse. Pas de fièvre. Pas d'expectoration.
Antécédents : tabagisme à 25 paquets/année, hypertension artérielle modérée, équilibrée à 13-7 avec
Sectral® (bétabloqueur).
Pas d'autres antécédents notables.
Examen : bon état général. température : 37°, tension artérielle à 9-7, fréquence cardiaque à 100,
auscultations pulmonaire et cardiaque normales, pas de signes d'insuffisance cardiaque droite, examen des
membres inférieurs : normal.
Examens complémentaires : radiographie pulmonaire montrant une ascension de la coupole diaphragmatique
gauche, gaz du sang : PaO2 : 57 mmHg (7,6 Kpa) ; PaCO2 : 36 mmHg (4,8 Kpa); pH = 7,4.

Vous pensez à une embolie pulmonaire. Vous pouvez également envisager chez ce malade :
A - Oedème aiguë du poumon chez un hypertendu
B - Décompensation respiratoire aiguë d'une bronchite chronique chez un fumeur
C - Tuberculose pulmonaire
D - Pneumonie à pneumocoques
E - Cancer bronchique
Bonne(s) réponse(s) : E

Aucune de ces propositions n'est exacte.


En effet, l'auscultation pulmonaire est peu en faveur d'un OAP. Les gaz du sang ne reflètent pas une
décompensation respiratoire chez un bronchitique insuffisant respiratoire. L'absence de fièvre et l'aspect
radiologique sont peu évocateurs d'une tuberculose ou d'une pneumonie à pneumocoque.

Au cours d'une embolie pulmonaire, l'électrocardiogramme peut revêtir l'un ou plusieurs des
aspects suivants :
A - Tracé normal
B - Aspect rSr dans les précordiales droites avec onde S profonde en D1 et VL
C - Axe de QRS dévié à gauche
D - Onde P bifide en D2. D3 et AVF
E - Bloc de branche droit incomplet
Bonne(s) réponse(s) : A B E

Un tracé normal est possible.


B traduit un bloc de branche droit. On peut également voir une déviation axiale droite.

En dehors de la surélévation de la coupole diaphragmatique, on peut retrouver chez ce malade à


la radiographie pulmonaire :
A - Epanchement pleural
B - Opacité triangulaire périphérique
C - Atélectasie en bandes
D - Hyperclarté localisée
E - Aspect normal
Bonne(s) réponse(s) : A B C D E

Tous ces aspects peuvent se voir au cours d'une embolie pulmonaire. Plus rarement on peut trouver une
saillie de l'arc moyen gauche traduisant une distension de l'artère pulmonaire.

Pour avoir une confirmation diagnostique de la maladie thrombo embolique. vous pouvez
éventuellement demander :
A - Une scintigraphie pulmonaire, de perfusion aux macroaggrégats marqués
B - Une scintigraphie pulmonaire de ventilation et de perfusion au xénon
C - Une artériographie bronchique
D - Une angiographie pulmonaire
E - Une phlébocavographie
Bonne(s) réponse(s) : A B D E

Tel que le QCM a été modifié (pour étayer le diagnostic de la maladie thromboembolique)
ces 4 items sont à retenir. B permettra d'affiner l'analyse mais sera difficile à obtenir en urgence.

524
Exclusivement sur DOC - DZ : www.doc-dz.com NADJI 85
RESIDANAT EN POCHE TOME II
Cas Clinique en QCM

Dans l'embolie pulmonaire. on retrouve habituellement le ou les signes cliniques suivants :


A - Signes cliniques de thrombose veineuse dans 75 % des cas
B - Tachycardie fréquente
C - Angoisse une fois sur deux
D - Antécédents d'hypertension artérielle
E - Dyspnée dans 80 % des cas
Bonne(s) réponse(s) : B C E

Les signes cliniques de thrombose veineuse sont moins fréquents (ils seraient présents dans environ 50 %
des cas dans une série rapportée de 174 EP). L'angoisse est assez fréquente (50 %) et la dyspnée est le
signe le plus fréquent (80 % des cas environ).

Pour résumer, chez ce malade, on peut affirmer que :


A - L'embolie est certainement peu importante car il n'y a pas de signes cliniques d'insuffisance
cardiaque droite
B - Il n'y a pas de risque de récidive car il n'y a pas de phlébite
C - La prise de Sectral® a certainement favorisé l'embolie
D - Il faut commencer d'urgence un traitement par les antivitamines K
E - Une oxygénotherapie peut être donnée sans risque de décompensation respiratoire
Bonne(s) réponse(s) : E

L'absence de signes d'insuffisance cardiaque droite ne permet pas d'éliminer formellement une E.P.
importante d'autant qu'existe une baisse de la pression artérielle.

Madame G., âgée de 70 ans, est atteinte d'une H.T.A. à 20/11 cmde Hg, ayant entraîné la mise en route d'un
traitement par la clonidine (Catapressan®) et un diurétique d'action progressive et mixte (Aldactazine®).
En raison d'un angor survenant lors des efforts de la vie courante, elle bénéficie par ailleurs d'un traitement
par la nifédipine (Adalate) et l'isosorbide dinitrate (Risordan® 10 : 4 cps/jour). De petits signes d'insuffisance
cardiaque et une tachyarythmie complète par fibrillation auriculaire ont justifié par ailleurs la prescription de
digitaline à raison de 5 gouttes/jour, 4 jours/7. A la suite d'un malaise survenu sur la voie publique, elle est
amenée en urgence à l'hôpital, où vous posez rapidement le diagnostic d'hypotension orthostatique.

Vous avez pu constater en effet lors du passage de la position couchée à la position debout :
A - Une chute de la PA systolique de 10 mmHg sans modification de la PA diastolique
B - Une chute de la PA systolique de 20 mmHg et une chute de la PA diastolique de 10 mmHg
C - Une chute de la PA systolique de 30 mmHg et une chute de la PA diastolique de 20 mmHg
D - Une absence de modification de la PA systolique et une chute de la PA diastolique de 10
mmHg
E - Une chute de la PA systolique de 10 mmHg et une chute de la PA diastolique de 20 mmHg
Bonne(s) réponse(s) :

QUESTION ANNULEE.

Parmi les médications que prenait la patiente, toutes sont susceptibles d'entraîner une
hypotension artériel orthostatique, sauf une. Laquelle ?
A - La digitaline
B - Le Risordan®
C - La nifédipine (Adalate®)
D - La clonidine (Catapressan®)
E - L'Aldactazine®
Bonne(s) réponse(s) : A

Tous les autres médicaments sont des hypotenseurs.

Parmi les facteurs suivants tous sauf un auraient pu encore aggraver la tendance à l'hypotension
orthostatique, lequel ?
A - Des varices des membres inférieurs
B - Un diabète
C - La prise de neuroleptiques
D - La prise de L dopa
E - Une hyperthyroïdie
Bonne(s) réponse(s) : E

Tous les autres facteurs sont susceptibles de donner ou d'aggraver une hypotension orthostatique.

525
Exclusivement sur DOC - DZ : www.doc-dz.com NADJI 85
RESIDANAT EN POCHE TOME II
Cas Clinique en QCM

Le malaise étant survenu lors d'un changement brusque de position, vous auriez pu évoquer les
diagnostics suivants, sauf un. Lequel ?
A - Une arthrose cervicale
B - L'insuffisance vertébro basilaire
C - Une hypersensibilité du sinus carotidien
D - Un myxome de l'oreillette gauche
E - Un BAV paroxystique
Bonne(s) réponse(s) : E

Un BAV paroxystique survient typiquement à l'emporte pièce et n'est pas déclenché ou influencé par un
changement postural. L'arthrose cervicale paraît également difficile à retenir comme cause de malaise.

Parmi les traitements que prenait la patiente, tous ont un effet potentiellement favorable sur
l'insuffisance cardiaque, sauf un. Lequel ?
A - La digitaline
B - L'Aldactazine®
C - La clonidine
D - Le Risordan®
E - La nifédipine
Bonne(s) réponse(s) :

QUESTION ANNULEE.

Un cadre sportif de 50 ans est hospitalisé d'urgence pour une tachypnée sévère. Il a parcouru d'une traite 500
kms en voiture, il y a trois jours.
On note sur la radiographie thoracique une hyperclarté du poumon gauche, sur l'ECG un bloc de branche droit
et un aspect S1Q3.
L'examen note une tachycardie à 120 cycles/mn, l'auscultation pulmonaire est normale, il existe une
turgescence jugulaire en position demi-assise, une cyanose.

Quel est le diagnostic le plus probable ?


A - Embolie pulmonaire
B - Pneumothorax suffocant
C - Emphysème
D - Syndrome de Mac Leod
E - Décompensation respiratoire aiguë d'une bronchopneumopathie chronique obstructive
Bonne(s) réponse(s) : A

L'association tachypnée, tachycardie, signes d'insuffisance ventriculaire droite, et l'aspect ECG et radiologique
sont très en faveur d'une embolie pulmonaire.

Quel est, dans la liste suivante, le résultat des gaz du sang artériel compatible avec le cas de ce
malade ?
A - PaO2 à 80 mmHg et PaCO2 à 30 mmHg
B - PaO2 à 50 mmHg et PaCO2 à 55 mmHg
C - PaO2 à 50 mmHg et PaCO2 à 30 mmHg
D - PaO2 à 80 mmHg et PaCO2 à 40 mmHg
E - PaO2 à 40 mmHg et PaCO2 à 60 mmHg
Bonne(s) réponse(s) : C

L'association hypoxie - hypocapnie traduisant un effet shunt et une hyperventilation, est typiquement présente
au cours d'une embolie pulmonaire.
Il faut néanmoins savoir que les gaz du sang peuvent être normaux (dans environ 10 % des cas).

Dans ce contexte clinique, il est pertinent de demander :


A - Echographie abdominale
B - Cliché en expiration forcée
C - Scintigraphie pulmonaire
D - Angiographie pulmonaire
E - Fibroscopie bronchique
Bonne(s) réponse(s) : D

La réponse à ce QCM est difficile ; en pratique, le diagnostic étant presque certain, il est surtout utile de
demander une angiographie précisant la gravité de l'E.P.
La scintigraphie pulmonaire est surtout intéressante dans les formes sans tableau de gravité, avec
éventuellement renouvellement de l'examen 15 jours plus tard pour confirmer un diagnostic éventuellement
hésitant.

526
Exclusivement sur DOC - DZ : www.doc-dz.com NADJI 85
RESIDANAT EN POCHE TOME II
Cas Clinique en QCM

Comme indice(s) de gravité pour le diagnostic posé, vous retenez :


A - Dyspnée et cyanose
B - Turgescence jugulaire
C - Signes de surcharge ventriculaire gauche à l'ECG
D - Obésité
E - Aspect S1Q3
Bonne(s) réponse(s) : A B E

Bien qu'il n'y ait pas de parallélisme anatomoclinique net, l'existence d'une tachypnée importante d'une
cyanose, d'une hypotension artérielle, d'une syncope, et de signes cliniques et ECG de retentissement
cardiaque droit plaide pour une E.P. grave, surtout s'ils sont réunis.

Quel est le facteur étiologique que l'on peut retenir à l'origine de cette détresse respiratoire ?
A - Exposition professionnelle
B - Voyage prolongé en position assise
C - Age
D - Sexe
E - Aucune des propositions précédentes
Bonne(s) réponse(s) : E

A priori, un voyage de quelques heures en position assise est tout de même bien différent d'un alitement
prolongé, et insuffisant pour favoriser la survenue d'une thrombose veineuse.

L'examen le plus spécifique et sensible pour rechercher une thrombose veineuse iliaque chez ce
patient, est :
A - Doppler
B - Phlébographie
C - Lymphographie
D - Examen au fibrinogène à I 125
E - Aucun de ces examens
Bonne(s) réponse(s) : B

Sans commentaire.

L'état du malade se dégrade rapidement malgré héparine, oxygène et digitaliques. Quelle(s)


thérapeutique(s) peut-on discuter ?
A - Doubler la dose d'héparine
B - Faire un traitement par Urokinase®
C - Lier la veine cave inférieure
D - Poser une ombrelle cave
E - Faire une embolectomie sous CEC
Bonne(s) réponse(s) : B C D E

On est en présence d'une embolie pulmonaire avec des signes cliniques de gravité qui doivent faire discuter
une thrombolyse du caillot par fibrinolytique ou une embolectomie surtout si l'angiographie pulmonaire a
confirmé l'existence d'une amputation supérieure à 40 % du lit vasculaire.
C et D se discuteront éventuellement plus tardivement mais pas dans ce contexte d'urgence, à moins qu'on ait
la preuve de l'existence d'un caillot flottant qui pourrait continuer à emboliser et expliquer la dégradation du
malade ou à moins qu'une embolectomie chirurgicale soit réalisée car ce dernier geste est toujours associé à
une interruption partielle de la veine cave, en raison notamment de l'impossibilité d'anticoaguler ces patients
dans la période post-opératoire immédiate.

527
Exclusivement sur DOC - DZ : www.doc-dz.com NADJI 85
RESIDANAT EN POCHE TOME II
Cas Clinique en QCM
Un homme de 66 ans est hospitalisé pour oedème aigu du poumon survenu à la suite d'un effort peu
important, alors qu'il arrachait les mauvaises herbes dans son jardin. Dans ses antécédents, on relève une
tuberculose pulmonaire à 20 ans, un ulcus duodénal à 52 ans, une hypertension artérielle modérée depuis 6
ans (20/11 au plus) équilibrée par clonidine (Catapressan®) et ramenée à 15/9.
Il décrit, depuis deux mois, un angor d'effort caractérisé.
A l'examen clinique, une fois l'épisode d'oedème pulmonaire traité par diurétiques intraveineux (furosémide
Lasilix®, 2 ampoules à 40 mg) il persiste encore quelques râles crépitants aux deux bases et on entend un
souffle mésosystolique rude et râpeux 4/6 au deuxième espace intercostal droit. La radioscopie montre un arc
inférieur gauche un peu bombant, arrondi. L'électrocardiogramme montre un rythme sinusal à 80/mn, un
intervalle PR à 0,22 sec, des QRS de 0,10 sec avec un indice de Sokolow de 52 mm, des ondes T négatives
et asymétriques dans le précordium gauche.

Cette sémiologie évoque un obstacle à l'éjection ventriculaire. Quel est, à votre avis, le type
anatomique le plus probable de cet obstacle ?
A - Rétrécissement aortique valvulaire pur
B - Rétrécissement aortique sous-valvulaire fixe
C - Myocardiopathie obstructive
D - Rétrécissement aortique supravalvulaire
E - Coarctation de l'aorte
Bonne(s) réponse(s) : A

Par ordre de fréquence, A est à évoquer en premier lieu. Il faudrait avoir quelques renseignements
supplémentaires sur l'auscultation pour étayer ce diagnostic.

Pour préciser le diagnostic clinique, vous devrez rechercher :


A - Un roulement diastolique
B - Une atténuation du deuxième bruit au foyer aortique
C - Un frémissement systolique
D - Des irradiations du souffle vers les vaisseaux du cou
E - Un click protosystolique éjectionnel
Bonne(s) réponse(s) : B C D E

La diminution du 2ème bruit au foyer aortique traduit un remaniement valvulaire important, le frémissement
systolique, l'intensité du souffle, enfin un click proto-systolique traduisent des valves sigmoïdes, qui gardent
encore une certaine mobilité.

Le cliché radiographique et un examen à l'amplificateur de brillance effectués tout de suite après


l'épisode initial d'oedème pulmonaire pourront montrer :
A - Un aspect flou et pommelé péri-hilaire
B - Un double contour de l'arc inférieur droit
C - Des calcifications de l'orifice aortique
D - Une dilatation débordant à droite de l'aorte ascendante
E - Une opacité scissurale
Bonne(s) réponse(s) : A C D E

A et E peuvent traduire des signes d'insuffisance cardiaque gauche.


D est fréquente au cours du rétrécissement aortique.

L'analyse des données électrocardiographiques permet d'envisager :


A - Un bloc auriculo-ventriculaire du premier degré
B - Un bloc de branche gauche complet
C - Une hypertrophie ventriculaire gauche
D - Une surcharge systolique du ventricule gauche
E - Une ischémie sous-épicardique antéro latérale
Bonne(s) réponse(s) : A C D

Le PR à 0,22 sec correspond à un BAV du premier degré.


La durée de QRS inférieure à 0,12 permet d'éliminer un bloc de branche gauche complet, l'aspect
asymétrique de la négativation des onde T traduit plutôt une surcharge systolique du ventricule gauche mais il
est parfois bien difficile de différencier cet aspect d'une véritable ischémie.

528
Exclusivement sur DOC - DZ : www.doc-dz.com NADJI 85
RESIDANAT EN POCHE TOME II
Cas Clinique en QCM

La sévérité de l'obstacle sera évaluée au mieux par le résultat de :


A - La valeur de l'indice de Sokolow
B - La vitesse du flux transorificiel au doppler
C - Le gradient de pression VG/aorte mesure par cathétérisme
D - L'épaisseur de la paroi ventriculaire gauche en écho TM
E - Le temps de demi montée au carotidogramme
Bonne(s) réponse(s) : C

La mesure pratiquement simultanée du gradient de pression et du débit cardiaque lors du cathétérisme


permet une évaluation de la surface aortique plus précise que le doppler continu (formule de Gorlin : surface
aortique = flux aortique systolique VG-aorte)
L'examen doppler dans les mains d'un expérimentateur entraîné permet une évaluation précise du gradient
trans-orificiel mais l'absence de mesure fiable du débit cardiaque limite l'évaluation de la surface.

Chez ce patient l'évolution peut se compliquer :


A - De mort subite
B - D'une endocardite infectieuse
C - D'embolies artérielles, par exemple cérébrales, oculaires
D - D'un trouble de conduction auriculo-ventriculaire sévère
E - D'une thrombose intracardiaque
Bonne(s) réponse(s) : A B C D

Les embolies artérielles sont le plus souvent d'origine calcaire, mais sont rares. L'autre complication est la
défaillance cardiaque gauche, puis globale.

En dehors du Lasilix®, quel traitement utile auriez-vous pu ajouter au moment de la crise


d'oedème pulmonaire ?
A - Un digitalique
B - Une perfusion de Xylocaïne®
C - Un dérive nitré
D - Un bêta-bloquant
E - Du vérapamil (Isoptine®)
Bonne(s) réponse(s) : C

L'emploi des dérivés nitrés (ou d'autres vasodilatateurs agissant surtout sur la précharge) doit se faire
néanmoins avec précaution car une baisse trop importante de la pression télédiastolique du ventricule gauche
peut entraîner une baisse du débit cardiaque.
Le traitement digitalique est déconseillé chez les malades présentant un obstacle important à l'éjection
ventriculaire gauche (rétrécissement aortique serré, cardiomyopathie obstructive).

Une femme de 64 ans consulte pour une dyspnée au moindre effort apparue brutalement depuis une
semaine. L'interrogatoire ne retrouve aucun antécédent notable. La patiente est apyrétique. L'auscultation
cardiaque entend à la pointe un B3, un bref roulement protodiastolique et un souffle holosystolique intense
(4/6), qui irradie largement dans l'aisselle.
La pression artérielle est de 120/80 mmHg. L'ECG est normal : le rythme cardiaque est sinusal. La
radiographie thoracique montre un coeur de volume et de morphologie normaux avec une image d'oedème
pulmonaire interstitiel diffus.

Dans quelle classe fonctionnelle (classification NYHA) peut être située cette patiente ?
A - Classe O
B - Classe 1
C - Classe 2
D - Classe 3
E - Classe 4
Bonne(s) réponse(s) : E

La dyspnée au moindre effort correspond à la classe 4 de la NYHA (pour rappel, classe 3 = gêne pour les
efforts de la vie courante et classe 2 gêne modérée apparaissant pour des efforts inhabituels, classe 1 = pas
de gêne fonctionnelle).

529
Exclusivement sur DOC - DZ : www.doc-dz.com NADJI 85
RESIDANAT EN POCHE TOME II
Cas Clinique en QCM

Le B3 est créé par :


A - L'ouverture de la mitrale
B - Le remplissage ventriculaire rapide
C - La systole auriculaire
D - La fermeture de la mitrale
E - La fermeture des sigmoïdes
Bonne(s) réponse(s) : B

Le B3 résulte des vibrations de l'ensemble du ventricule survenant à la fin de la phase de remplissage rapide
protodiastolique du ventricule.

De quelle cardiopathie est atteinte cette patiente ?


A - Rétrécissement aortique
B - Insuffisance mitrale pure
C - Maladie mitrale
D - Insuffisance tricuspidienne
E - Communication inter ventriculaire
Bonne(s) réponse(s) : B

Le bref roulement diastolique correspond à une sténose fonctionnelle de l'orifice mitral en rapport avec
l'importance de la fuite mitrale et donc du flux protodiastolique de remplissage rapide du ventricule
(hyperdébit).

Quelle est son étiologie la plus probable ?


A - Congénitale
B - Rhumatismale
C - Endocardite bactérienne
D - Ischémique
E - Rupture de cordage mitral
Bonne(s) réponse(s) : E

En l'absence de fièvre ou de signes généraux pouvant évoquer une endocardite, l'hypothèse la plus probable
est une rupture de cordage.
Le syndrome de rupture avec douleur thoracique et oedème pulmonaire est rare. Le volume cardiaque normal
à la radiographie traduit le caractère très récent de cette fuite mitrale.
Les étiologies des ruptures de cordage sont par ordre de fréquence les formes dystrophiques (prolapsus
valvulaire mitral) et les formes dégénératives (sujet âgé) dans environ 50 % des cas, les endocardites
infectieuses dans 25 % des cas, les atteintes rhumatismales dans 20 % des cas, puis viennent l'infarctus du
myocarde et les traumatismes.

Parmi ces 5 explorations, quelle est celle qui permettra le mieux de préciser la lésion en cause et
son étiologie ?
A - Phonomécanocardiographie
B - Echocardiographie-doppler
C - Angioscintigraphie
D - Angiographie VG
E - Coronarographie
Bonne(s) réponse(s) : B

L'aspect échocardiographique associe la présence d'échos anormaux dans l'oreillette gauche en systole (TM),
des vibrations systoliques de la valve mitrale et une grande vitesse d'ouverture (TM). En bidimensionnel, on
notera parfois l'existence d'un prolapsus et on recherchera le passage de l'extrémité distale du feuillet derrière
le niveau de fermeture mitrale en systole.

Si un cathétérisme cardiaque est réalisé, quelle anomalie peut-il montrer ?


A - Un gradient systolique entre le VG et l'aorte
B - Un gradient diastolique entre le VG et le capillaire pulmonaire
C - Un pic systolique élevé sur la courbe capillaire pulmonaire
D - Les propositions B et C (ci-dessus)
E - Un pic systolique sur la courbe auriculaire droite
Bonne(s) réponse(s) : D

Sans commentaire.

530
Exclusivement sur DOC - DZ : www.doc-dz.com NADJI 85
RESIDANAT EN POCHE TOME II
Cas Clinique en QCM
Madame R.... 42 ans, appelle en urgence son médecin pour une impotence fonctionnelle douloureuse du
membre inférieur droit apparue brutalement avec sensation de refroidissement.
Madame R..., qui n'a pas d'antécédents notables précise qu'elle n'a jamais eu jusqu'à présent de douleurs
dans les membres inférieurs mais qu'elle a remarqué depuis une quinzaine de jours la survenue de
palpitations inhabituelles.
A l'auscultation, le coeur est irrégulier et rapide à 120/mn. Un roulement diastolique est bien perçu à la pointe
au cours des diastoles longues. L'ausculation pulmonaire est normale. La pression artérielle est à 130/80
mmHg.
La température est à 37,1°C. L'examen neurologique et vasculaire du membre inférieur gauche est normal.
A droite, le pied est pâle, froid. La sensibilité et la mobilité des orteils est diminuée par rapport au côté sain, le
pouls fémoral est bien perçu, mais les pouls poplité, tibial postérieur et pédieux sont abolis. L'EGC. confirme la
tachyarythmie complète par fibrillation auriculaire.

Quelle(s) est (sont) parmi les données rapportées dans cette observation celle(s) qui vous
apparai(ssen)t plus évocatrice(s) d'une embolie artérielle que d'une thrombose aiguë sur artérite ?
A - La constatation d'une fibrillation auriculaire
B - La suspicion de rétrécissement mitral(roulement diastolique)
C - L'impotence fonctionnelle avec troubles de la sensibilité
D - La présence d'un souffle fémoral droit
E - La brutalité de l'installation du syndrome ischémique en l'absence de claudication
intermittente préexistante
Bonne(s) réponse(s) : A B E

Sans commentaire.

Le diagnostic d'embolie artérielle étant retenu, quelle attitude thérapeutique vous parait la plus
judicieuse ?
A - Traitement thrombolytique
B - Héparinothérapie
C - Traitement vasodilatateur par voie intra veineuse
D - Désobstruction artérielle à la sonde de Fogarty sous héparinothérapie en urgence
E - Désobstruction artérielle à la sonde de Fogarty en cas d'échec du traitement pendant 24
heures
Bonne(s) réponse(s) : D

Sans commentaire

Madame R....est atteinte d'un rétrécissement mitral pur. Hormis le roulement diastolique, quel(s)
autre(s) élément(s) stéthacoustique(s) pouvez-vous entendre à l'auscultation de cette patiente ?
A - Eclat du premier bruit à la pointe
B - Claquement d'ouverture de la mitrale
C - Renforcement présystolique du roulement diastolique
D - Dédoublement fixe du deuxième bruit au foyer pulmonaire
E - Souffle systolique mésocardiaque
Bonne(s) réponse(s) : A B

Le rythme mitral de Durozier associe l'éclat de B1, le claquement d'ouverture mitral et le roulement
diastolique. Le renforcement présystolique du roulement ne peut être entendu chez cette patiente en
fibrillation auriculaire, en raison de la perte de la systole auriculaire.

Laquelle parmi ces anomalies, l'ECG. de cette patiente peut-il vraissemblablement mettre en
évidence, outre la fibrillation auriculaire ?
A - Une hypertrophie auriculaire droite
B - Une hypertrophie auriculaire gauche
C - Un bloc de branche gauche complet
D - Une hypertrophie ventriculaire droite
E - Une hypertrophie ventriculaire gauche
Bonne(s) réponse(s) : D

On peut évidemment observer des signes de surcharge ventriculaire droite (déviation axiale droite de QRS
avec S1 Q3, bloc incomplet droit, inversion du rapport R/S en V1). Les signes d'HAG ne peuvent s'observer
chez cette patiente en fibrillation auriculaire.

531
Exclusivement sur DOC - DZ : www.doc-dz.com NADJI 85
RESIDANAT EN POCHE TOME II
Cas Clinique en QCM

Lequel parmi les examens complémentaires suivants permettra d'apprécier au mieux l'importance
de la sténose de l'orifice mitral ?
A - Radiographie thoracique de face
B - Echocardiogramme-doppler
C - Epreuve d'effort sur cyclo ergomètre
D - Phonocardiogramme
E - Gamma-angiographie ventriculaire
Bonne(s) réponse(s) : B

L'échocardiogramme couplé au doppler permet une mesure assez précise de la surface mitrale : en
bidimensionnel on mesure directement la surface de l'orifice mitral par planimétrie et le doppler permet
également une évaluation de la surface mitrale par la mesure du profil vitesse du flux trans mitral.

Monsieur X..., âgé de 42 ans, sans antécédent particulier, fume 20 cigarettes/jour depuis l'âge de 20 ans. Il
pèse 72 kg pour 1,78 m. Il est normo-tendu à 13/8 et son bilan métabolique est normal.
Lors d'un effort d'amarrage d'un bateau, il ressent une violente douleur rétrosternale constrictive irradiant vers
les épaules, rapidement accompagnée de sueurs profuses, de nausées et d'une sensation lipothymique. La
douleur se prolonge pendant une heure. Lors de son hospitalisation 30 minutes plus tard est constatée une
bradycardie sinusale à 50, alors que la TA est à 9/6. L'ECG objective un sus-décalage du segment ST dans le
territoire inféro latéral. Un bilan enzymatique complet est demandé. L'évolution confirme rapidement le
diagnostic d'infarctus du myocarde. La coronarographie sera normale et un test provocateur du spasme par la
méthylergométrine (Méthergin®) sera positif.

Dans quelles dérivations, aura été enregistré le sus-décalage du segment ST ?


A - V1 V2 V3
B - V4 V5 V6
C - D2 D3 VF V5 V6
D - D1 VL V4
E - D2 D3 VF V1 V2 V3
Bonne(s) réponse(s) : C

Le territoire inférieur correspond aux dérivations D2 - D3 - VF et le territoire latéral correspond à V5-V6.

Le tableau clinique initial évoque :


A - Un choc cardiogénique
B - Un choc hypovolémique
C - Un trouble du rythme
D - Un choc vagotonique
E - Une hypotension artérielle orthostatique
Bonne(s) réponse(s) : D

L'association de la bradycardie, de la chute tensionnelle et des sueurs ainsi que la topographie de l'infarctus
évoquent plus un choc vagotonique.

Quel est le mécanisme initial le plus probable de cet infarctus ?


A - Une embolie coronaire
B - Un spasme coronaire
C - Une thrombose coronaire
D - Une athérosclérose coronaire
E - Aucune de ces réponses n'est exacte
Bonne(s) réponse(s) : B

La positivité du test au méthergin et la normalité de la coronarographie plaident en faveur d'un mécanisme


spastique. Une autre étiologie d'infarctus associée à des artères coronaires normales est représentée par
l'embolie coronaire.
Une coronarographie normale est retrouvée dans environ 4 % des infarctus et dans 16 % des infarctus
survenus chez des patients de moins de 35 ans.

Dans le cas présent, le bilan enzymatique initial aura montré :


A - Des taux normaux des différentes enzymes cardiaques
B - Une augmentation des CPK totales
C - Une augmentation des CPK-MB
D - Une augmentation de la LDH
E - Une augmentation des transaminases oxalo acétiques
Bonne(s) réponse(s) : A

A 1 heure 30 du début de la douleur, le bilan enzymatique sera en général normal.

532
Exclusivement sur DOC - DZ : www.doc-dz.com NADJI 85
RESIDANAT EN POCHE TOME II
Cas Clinique en QCM

Parmi les propositions thérapeutiques suivantes que l'on pourra (ou devra) recommander à la
sortie du patient, une seule est inexacte. Laquelle ?
A - L'arrêt total du tabagisme
B - Le propranolol (Avlocardyl®)
C - Le Risordan® 10 : 4 cps/jour
D - La Molsidomine (Corvasal®)
E - La nifédipine (Adalate®)
Bonne(s) réponse(s) : B

Les bêtabloquants sont à éviter au cours du spasme coronaire car ils sont susceptibles de pouvoir le
déclencher ou l'aggraver.
Le tabagisme peut provoquer des spasmes coronaires.

Un homme de 40 ans, opéré d'une insuffisance aortique il y a deux ans, est porteur d'une prothèse valvulaire.
Il se soumet à une surveillance régulière.
Son état est satisfaisant et l'auscultation (depuis l'intervention) était celle normalement rencontrée chez un
porteur de prothèse aortique.
Depuis une extraction dentaire réalisée 3 semaines auparavant, il présente une fièvre à 38,5 degrés et se sent
fatigué.

Parmi les éventualités suivantes, quel est le diagnostic que vous évoquez en premier ?
A - Embolie pulmonaire
B - Péricardite aiguë
C - Infarctus du myocarde
D - Endocardite infectieuse
E - Dissection aortique
Bonne(s) réponse(s) : D

La notion de l'existence d'une prothèse aortique et d'une fièvre prolongée à la suite d'une extraction dentaire
doit faire évoquer en premier lieu une endocardite sur prothèse.

L'un des éléments suivants, s'il est retrouvé à l'auscultation, possède une grande valeur
d'orientation. Lequel ?
A - Arythmie complète
B - Frottement péricardique
C - Souffle systolique
D - Bruit de galop
E - Souffle diastolique
Bonne(s) réponse(s) : E

L'existence et surtout l'apparition d'un souffle diastolique chez ce patient suspect d'endocardite traduirait une
fuite aortique et donc une éventuelle désinsertion de la prothèse, et serait un argument important pour une
greffe bactérienne sur la prothèse.

Dans ce cas, l'examen clinique ou l'interrogatoire doivent rechercher :


A - Epanchement pleural
B - Purpura
C - Frissons
D - Défense abdominale
E - Splénomégalie
Bonne(s) réponse(s) : B C E

Toute suspicion d'endocardite impose de rechercher des signes cutanés (purpura, faux panari d'Osler) et une
splénomégalie, dont l'existence confortera le diagnostic.
Les frissons peuvent correspondre à des décharges bactériémiques à partir du foyer infectieux.

Parmi les examens biologiques suivants, lequel doit être demandé en priorité ?
A - Numération formule sanguine
B - Hémoculture
C - Amylasémie
D - Enzymes cardiaques
E - Gaz du sang
Bonne(s) réponse(s) : B

Au moins 6 hémocultures, avant toute antibiothérapie, de préférence lors d'une poussée fébrile ou de frissons,
et à ensemencer sur milieux aéro-anaérobies.

533
Exclusivement sur DOC - DZ : www.doc-dz.com NADJI 85
RESIDANAT EN POCHE TOME II
Cas Clinique en QCM

Compte tenu de la porte d'entrée, quel est le germe le plus probablement en cause ?
A - Staphylocoque aureus
B - Staphylocoque blanc
C - Streptocoque
D - Pneumocoque
E - Gram négatif
Bonne(s) réponse(s) : C

Streptocoques non groupables.

Quel traitement antibiotique vous semble le plus logique en première intention ?


A - Amoxicilline
B - Oxacilline + gentamycine
C - Vancomycine
D - Pénicilline + gentamycine
E - Aucun traitement en attendant le résultat des examens
Bonne(s) réponse(s) : D

Une suspicion d'endocardite impose de débuter un traitement antibiotique une fois les prélèvements effectués
: double antibiothérapie, bactéricide, par voie parentérale.

Un homme de 52 ans, chauffeur-routier, est adressé pour le bilan d'une HTA. modérée découverte
fortuitement.
L'interrogatoire retrouve la notion d'un asthme ancien sur un terrain atopique, d'un tabagisme modéré (10 à 15
Cigs/24h) et d'une consommation d'alcool excessive (105 g/24h). Il se plaint d'une discrète dyspnée d'effort.
Le poids est de 85 kgs pour 1,65m. La TA est de 190/105 mmHg, rythme cardiaque est à 95/mn. Le reste de
l'examen clinique est normal.

Parmi ces 5 propositions tirées de l'observation, quelle(s) est(sont) celle(s) qui a(ont) pu
directement favoriser le développement de cette HTA en apparence "primitive" ?
A - Antécédent d'asthme
B - Tabagisme modéré
C - Ethylisme
D - Surpoids
E - Tachycardie de repos
Bonne(s) réponse(s) : C D

La consommation d'alcool et l'obésité sont des facteurs favorisant ou aggravant le développement d'une HTA.
La tachycardie de repos peut traduire une activation du système sympathique, qui peut être un facteur
d'augmentation du débit cardiaque des résistances périphériques et de la pression artérielle mais qui peut
également correspondre à une adaptation pour maintenir un débit cardiaque constant sur une cardiopathie
hypertensive. Cette réponse ne paraît pas devoir être retenue.

Parmi ces 5 examens biologiques, lequel(lesquels) doit (doivent) être demandé(s) en première
intention chez ce patient ?
A - Bilan lipidique
B - Ionogramme sanguin
C - Créatininémie
D - Rénine plasmatique
E - Catécholamines urinaires
Bonne(s) réponse(s) : A B C

Il s'agit du bilan biologique de première intention. A est demandé surtout pour dépister d'autres facteurs de
risque cardiovasculaire associés.

Parmi ces 5 examens complémentaires, le(s) quel(s) est(sont) utile(s) en première intention chez
ce patient ?
A - Electrocardiogramme
B - Radiographie thoracique
C - Examen du fond d'oeil
D - Angiographie numérisée par voie veineuse avec clichés urographiques précoces
E - Tomodensitométrie abdominale pour étude des reins et des surrénales
Bonne(s) réponse(s) : A B C

A et B sont des examens simples pour juger du retentissement de l'HTA sur le coeur et pour rechercher une
HVG.

534
Exclusivement sur DOC - DZ : www.doc-dz.com NADJI 85
RESIDANAT EN POCHE TOME II
Cas Clinique en QCM

Avant tout traitement médicamenteux, et à la recherche d'un effet favorable direct sur la PA, vous
recommandez à ce patient :
A - Régime désodé strict
B - Régime hypocalorique
C - Reclassement professionnel
D - Arrêt de l'alcool
E - Activité physique régulière
Bonne(s) réponse(s) : B D E

Le régime désodé strict est souvent très difficile à suivre, a fortiori chez un chauffeur routier.
Le régime hypocalorique, l'arrêt de l'alcool et une activité physique régulière peuvent permettre un meilleur
contrôle tensionnel, surtout si cela s'accompagne d'une perte de poids.

Si vous décidez de seconde intention d'instituer un traitement médicamenteux complémentaire,


le(s)quel(s) parmi ces 5 médicaments est(sont) contre-indiqué(s) chez ce patient ?
A - Diurétique
B - Bêta-bloquant
C - Alpha-bêta-bloquant type labétalol (Trandate®)
D - Clonidine (Catapressan®)
E - Captopril (Lopril®)
Bonne(s) réponse(s) : B C

Les bêta-bloquants sont contre-indiqués en raison des antécédents d'asthme.


Le labétalol, qui possède à la fois des propriétés alpha et bêta-bloquantes est également contre-indiqué.

Une femme de 60 ans, sans antécédents broncho pulmonaires, est traitée depuis plusieurs années pour une
myocardiopathie non obstructive primitive par une association de Digoxine®, Lasilix® et Aldactone®. Les
signes d'insuffisance cardiaque se sont aggravés depuis un mois, liés à un passage en fibrillation auriculaire ;
à cette occasion, la posologie de Digoxine® a été doublée et on a adjoint de la Cordarone® et un
anticoagulant. Depuis une semaine, la patiente se plaint de nausées et décrit des hallucinations visuelles avec
visions colorées, la dyspnée persiste de manière très importante (stade III) ; l'examen clinique note des
oedèmes des membres inférieurs, un gros foie sensible ; à l'auscultation cardiaque : B3 et souffle
holosystolique 2/6 à l'apex, souffle holosystolique 1/6 à la xyphoïde augmentant en inspiration ; à l'auscultation
pulmonaire : râles sous-crépitants aux deux bases. La PA est à 120/85 mmHg.
L'ECG. retrouve la FA avec une bradyarythmie à 45/mn, un BBD complet et des extrasystoles ventriculaires
parfois bigéminées. Le bilan biologique montre pour l'essentiel : Na+ à 132 mmoles/l, K+ à 4.5 mmoles/l,
créatininémie à 220 micrommoles/l, digoxinémie à 3.5 ng/ml. Un cathétérisme droit réalisé après guérison de
l'intoxication digitalique, mesure une pression artérielle pulmonaire moyenne à 30 mmHg, une pression
capillaire pulmonaire moyenne à 23 mmHg et un index cardiaque à 2.1 l/mn/m2.

Parmi ces cinq signes tirés de l'observation, lequel ou lesquels sont directement attribuables à
une insuffisance ventriculaire droite, dans ce contexte particulier ?
A - Dyspnée
B - B3 à l'apex
C - Souffle holosystolique à la xyphoïde
D - Souffle holosystolique à la pointe
E - Oedèmes des membres inférieurs
Bonne(s) réponse(s) : C E

Le souffle holosystolique augmentant à l'inspiration est un souffle d'insuffisance tricuspide. Comme autre
signe d'IVD, on peut également citer l'hépatomégalie sensible.

Parmi ces cinq signes tirés de l'observation, lequel ou lesquels sont directement attribuables à
une insuffisance ventriculaire droite, dans ce contexte particulier ?
A - Nausées
B - Hallucinations visuelles
C - Bradyarythmie
D - BBD complet
E - Extrasystoles ventriculaires bigéminées
Bonne(s) réponse(s) : D

Les autres signes sont en rapport avec l'intoxication digitalique, qui ne peut provoquer de bloc de branche
dans la mesure où la digitaline déprime la conduction supra-hissienne ou nodale mais jamais la conduction
infra-hissienne.

535
Exclusivement sur DOC - DZ : www.doc-dz.com NADJI 85
RESIDANAT EN POCHE TOME II
Cas Clinique en QCM

Indépendamment de la posologie, lequel parmi ces cinq facteurs a probablement joué le rôle
déterminant dans la survenue de cette intoxication ?
A - Association au Lasilix®
B - Association à l'Aldactone®
C - Association à la Cordarone®
D - Association aux anticoagulants
E - Créatininémie à 220 micromoles/l
Bonne(s) réponse(s) : E

90 % de la digoxine est excrétée sous forme intacte par le rein. En cas d'insuffisance rénale, il convient donc
d'adapter et de diminuer la posologie de digoxine. Classiquement, on disait qu'il fallait mieux utiliser dans ce
cas la digitaline ; cependant la digitaline expose à la production de digoxine issue du catabolisme hépatique,
dont les quantités sont difficilement prévisibles.
Aussi, il faut actuellement préconiser l'emploi de la digoxine en cas d'insuffisance rénale, en diminuant la
posologie.

Comment résumer les données hémodynamiques fournies par le cathétérisme droit ?


A - Hypertension pulmonaire post-capillaire avec débit cardiaque normal
B - Hypertension pulmonaire post-capillaire avec bas débit cardiaque
C - Hypertension pulmonaire pré et post-capillaire avec débit normal
D - Hypertension pulmonaire précapillaire avec débit cardiaque normal
E - Hypertension pulmonaire précapillaire avec bas débit cardiaque
Bonne(s) réponse(s) : B

L'élévation de la pression artérielle pulmonaire moyenne est secondaire à l'élévation de la pression capillaire
pulmonaire (pression capillaire pulmonaire moyenne normal = 12 mm Hg, PAP systolique 30, diastolique 15,
moyenne 20).
L'index cardiaque normal est compris entre 2,5 et 3,5 l/min/m2.

Compte-tenu de ces données hémodynamiques et une fois guérie l'intoxication digitalique, vous
décidez d'adjoindre au traitement en cours un nouveau produit. Lequel ?
A - Dérivé nitré per os
B - Minipress® (prazosine)
C - Inhibiteur de l'enzyme de conversion
D - Isuprel® en glossettes
E - Adalate® (nifédipine)
Bonne(s) réponse(s) : C

Les inhibiteurs de l'enzyme de conversion ont fait la preuve d'une efficacité dans la diminution de la mortalité
au cours de l'insuffisance cardiaque grave (stade 4). Il possède à la fois des propriétés vasodilatatrices sur le
système artériel et veineux.
La réponse A pouvait également se discuter, ainsi que la réponse B. Ce QCM est donc litigieux.

Dans ce cas, parmi les cinq médicaments antérieurement prescrits, lequel doit être supprimé ?
A - Lasilix®
B - Aldactone®
C - Digoxine®
D - Cordarone®
E - Anticoagulant
Bonne(s) réponse(s) : B

Il vaut mieux éviter d'associer un diurétique épargneur de potassium au traitement par les inhibiteurs de
l'enzyme de conversion en raison du risque accru d'hyperkaliémie, d'autant qu'existe une insuffisance rénale.

536
Exclusivement sur DOC - DZ : www.doc-dz.com NADJI 85
RESIDANAT EN POCHE TOME II
Cas Clinique en QCM
Un homme de 64 ans (1,69 m, 65 kgs) souffre depuis un an de douleurs constrictives d'insuffisance
coronarienne, et depuis 3 mois environ d'une dyspnée stade III.
A l'examen, le rythme est irrégulier, la PA est à 180/100 mm Hg. Il n'existe pas de souffle à l'auscultation
cardiaque. L'E.C.G. de repos montre une fibrillation auriculaire (F.A.), avec rythme ventriculaire à 95
cycles/mn, un bloc de branche gauche complet ; l'échocardiographie retrouve une dilatation des cavités
gauches, une hypokinésie globale (fraction d'éjection évaluée à 30 %), et l'absence de valvulopathie.
Le bilan biologique retrouve une glycémie à jeun à 6 mmoI/I, un cholestérol à 8 mmol/l, des triglycérides à 1,5
mmol/l, une natrémie à 135 mmoles/l, une kaliémie à 4 mmoles/l, une créatininémie à 100 micromoles/l. Le
diagnostic retenu est celui d'une myocardiopathie ischémique.

Parmi les règles hygiéno-diététiques suivantes, laquelle(lesquelles) vous paraît(paraissent)


s'imposer ?
A - Régime pauvre en hydrates de carbone
B - Régime pauvre en graisses saturées
C - Régime pauvre en sel
D - Régime hypocalorique
E - Marche prolongée conseillée
Bonne(s) réponse(s) : B C

B en raison de l'hypercholestérolémie, C en raison de l'insuffisance cardiaque. La très discrète élévation des


triglycérides est contingeante à l'hypercholestérolémie et ne nécessite pas de régime pauvre en hydrates de
carbone, d'autant qu'il ne présente pas de surcharge pondérale.

Parmi les données de l'observation, laquelle constitue une contre-indication formelle à l'emploi
des digitaliques ?
A - PA à 180/100 mm Hg
B - Fibrillation auriculaire à 95 cycles/mn
C - Bloc de branche gauche complet
D - Fraction d'éjection à 35 %
E - Aucune des propositions précédentes
Bonne(s) réponse(s) : E

Les contre indications habituelles aux digitaliques sont le bloc auriculo-ventriculaire de haut degré non
appareillé, la tachycardie ventriculaire, les cardiomyopathies obstructives, le tachycardies atriales survenant
dans les syndromes de Wolff, Parkinson, White.

Si on considère le problème thérapeutique de l'HTA, quels sont parmi ces 5 médicaments anti-
hypertenseurs les deux contre-indiqués, en principe, chez ce patient ?
A - Furosémide (Lasilix®)
B - Clonidine (Catapressan®)
C - Acébutolol (Sectral®)
D - Dihydralazine (Népressol®)
E - Enalapril (Rénitec®)
Bonne(s) réponse(s) : C D

Les bêta-bloquants sont en principe contre-indiqués dans l'insuffisance cardiaque sévère, bien que certains
auteurs aient pu les proposer dans cette indication.
L'emploi de l'hydralazine est déconseillé chez le patient coronarien, en l'absence d'association à un bêta-
bloquant, en raison de son effet tachycardisant.

Si on considère le problème thérapeutique de l'angor, quel(s) est(sont) parmi ces 5 médicaments


anti-angineux, celui(ceux) qui est(sont) contre-indiqué(s) en principe chez ce patient ?
A - Propranolol (Avlocardyl®)
B - Amiodarone (Cordarone®)
C - Isosorbide dinitrate (Risordan®)
D - Vérapamil (Isoptine®)
E - Nifédipine (Adalate®)
Bonne(s) réponse(s) : A D

Le propanolol et le vérapamil sont contre-indiqués en raison de leur effet dépresseur sur la contractilité
myocardique, chez ce patient présentant une diminution importante de sa fraction d'éjection.

537
Exclusivement sur DOC - DZ : www.doc-dz.com NADJI 85
RESIDANAT EN POCHE TOME II
Cas Clinique en QCM

Quel(s) effet(s) favorable(s) peut-on espérer de la prescription d'un dérivé nitré retard chez ce
patient ?
A - Amélioration de l'angor
B - Diminution de la dyspnée
C - Ralentissement du rythme cardiaque
D - Retour en rythme sinusal
E - Abaissement du taux de cholestérol plasmatique
Bonne(s) réponse(s) : A B

Les dérivés nitrés vont, en diminuant la précharge, diminuer le travail du myocarde et la consommation du
myocarde en O2 et améliorer la symptomatologie du patient tant sur le plan de l'angor que sur celui de
l'insuffisance cardiaque.

Quel(s) effet(s) favorable(s) peut-on espérer de la prescription d'un inhibiteur de l'enzyme de


conversion chez ce patient ?
A - Diminution de la dyspnée
B - Normalisation de la PA
C - Augmentation de la fraction d'éjection
D - Régression des anomalies métaboliques
E - Retour en rythme sinusal
Bonne(s) réponse(s) : A B C

Les inhibiteurs de l'enzyme de conversion sont des vasodilatateurs mixtes agissant à la fois sur le versant
artériel et veineux. Ils vont en s'opposant aux effets vasoconstricteurs de l'angiotensine II, diminuer la post
charge et la précharge d'où une amélioration de la dyspnée, une diminution de la PA et une augmentation de
la fraction d'éjection.

Parmi ces 5 schémas thérapeutiques, quel est celui qui vous semble le mieux adapté pour ce
patient ?
A - AvlocardyI® + Isoptine® + Risordan®
B - AvlocardyI® + Adalate + Lasilix
C - Digoxine® + Népressol® + Risordan®
D - Digoxine® + Cordarone® + Lopril®
E - Risordan® + Rénitec® + Népressol®
Bonne(s) réponse(s) : D

La fibrillation auriculaire étant probablement récente et sans doute responsable de l'aggravation depuis 3 mois
de son insuffisance cardiaque, il semble licite de tenter une réduction médicamenteuse par l'association
digoxine-cordarone même si cette réduction sera difficile à obtenir. Enfin, l'administration de Lopril® (captopril)
semble licite dans cette myocardiopathie évoluée, dans la mesure où les inhibiteurs de l'enzyme de
conversion ont fait la preuve d'une efficacité dans la réduction de la mortalité chez de tels malades.

Agé de 17 ans, P... Alain, étudiant en terminale D, accuse depuis quelques semaines des céphalées à
prédominance occipitale et une irritabilité qui sont mises sur le compte du travail intense qu'il fournit. Devant la
persistance des symptômes, une consultation médicale est demandée qui révèle une hypertension artérielle à
190/110 mmHg aux deux bras, en décubitus.
Alain n'a aucun antécédent personnel ni familial d'ordre cardiovasculaire. Les chiffres tensionnels antérieurs
étaient, semble t-il, normaux. Les pouls fémoraux n'étant pas perçus, le médecin pose le diagnostic de
coarctation de l'aorte.

En dehors de la coarctation de l'aorte, quelle étiologie doit-on évoquer en priorité devant une HTA
d'apparition récente et brutale chez un adolescent ?
A - Hypertension artérielle essentielle
B - Phéochromocytome
C - Sténose de l'artère rénale
D - Uropathie malformative
E - Hyperaldostéronisme primaire
Bonne(s) réponse(s) : B C D E

La découverte d'une HTA d'apparition brutale et sévère chez l'adolescent doit faire évoquer une HTA
secondaire, avant de parler de premières manifestations d'une HTA essentielle, et afin de retrouver une
étiologie éventuellement curable.

538
Exclusivement sur DOC - DZ : www.doc-dz.com NADJI 85
RESIDANAT EN POCHE TOME II
Cas Clinique en QCM

En dehors de l'hypovascularisation de la moitié inférieure du corps, la coarctation de l'aorte se


traduit par les signes suivants, sauf un, lequel ?
A - Souffle systolique
B - Erosions costales sur le cliché thoracique de face
C - Circulation collatérale de suppléance
D - Augmentation de volume du VG
E - Saillie de l'arc moyen gauche sur le télécoeur de face
Bonne(s) réponse(s) : E

Il s'agit d'une saillie ou d'une modification du bouton aortique (arc supérieur gauche).

Parmi les propositions suivantes, indiquez celle qui correspond au souffle systolique de la
coarctation de l'aorte :
A - Souffle holo-systolique apexo-axillaire
B - Souffle à maximum méso-cardiaque, irradiant dans toutes les directions
C - Souffle méso-systolique prédominant dans le 2ème EID et irradiant vers le haut
D - Souffle maximum à la xiphoïde et augmentant à l'inspiration forcée
E - Souffle méso-systolique retrouvé dans la région inter-omo-vertébrale gauche
Bonne(s) réponse(s) : E

Souffle méso-systolique entendu au 2° et 3° espace intercostal gauche et irradiant dans le dos dans la région
paravertébrale gauche.

Parmi les examens complémentaires suivants, quel est le plus sûr pour confirmer le diagnostic de
coarctation de l'aorte ?
A - Angiographie aortique numérisée, par voie veineuse
B - Scanner
C - Echographie .
D - Angio-scintigraphie
E - Radiographie thoracique
Bonne(s) réponse(s) : A

Sans commentaire.

Quel traitement proposez-vous en priorité à ce jeune patient ?


A - Un diurétique
B - L'association de diurétiques et de bêta-bloquants
C - Un bêta-bloquant
D - Un inhibiteur de l'enzyme de conversion
E - Un traitement chirurgical
Bonne(s) réponse(s) : E

La coarctation de l'aorte représente une cause rare d'HTA mais curable par un traitement chirurgical radical.

Madame X, 70 ans, est hospitalisée pour une chute dans son escalier. Une fracture déplacée du col fémoral
droit est diagnostiquée. L'interrogatoire et l'examen clinique révèlent une surcharge pondérale (75 kg pour
1,65 m), un diabète et une hypertriglycéridémie pour laquelle elle suit un régime. Cette patiente a subit une
hystérectomie pour léiomyome hémorragique à l'âge de 48 ans sans aucune complication.

Parmi les examens du bilan préopératoire, quel est celui(ou quels sont ceux) qui est(sont)
anormal(anormaux) ?
A - TCK : 36 sec (témoin : 33 sec)
B - Temps de Quick : 13 sec (témoin . 12 sec)
C - Hémoglobine : 105 g/I
D - Plaquettes : 310 G/I
E - Temps de saignement : 8 mn (technique de Ivy incision)
Bonne(s) réponse(s) : C

Les tests de coagulation sont normaux : plaquettes 310 g/l signifie 310.10g/l, le temps de saignement par la
méthode d'Ivy (brassard à tension) est normalement inférieur à 10 minutes (méthode de duke, TS < 5
minutes).

539
Exclusivement sur DOC - DZ : www.doc-dz.com NADJI 85
RESIDANAT EN POCHE TOME II
Cas Clinique en QCM

Le chirurgien décide de poser une prothèse de hanche. Une prévention de la maladie


thromboembolique par une héparinothérapie est décidée à la dose de 0,20 ml d'héparine calcique
en sous cutanée toutes les 8 heures. Parmi les propositions de surveillance suivantes, laquelle ou
lesquelles retenez-vous ?
A - Surveillance journalière par TCK
B - Surveillance journalière par le temps de Howell
C - Aucune surveillance de la coagulation plasmatique
D - Surveillance journalière par le temps de Quick
E - Surveillance bi-hebdomadaire de la numération plaquettaire
Bonne(s) réponse(s) :

QUESTION ANNULEE.

Au cinquième jour post-opératoire, Madame X présente une douleur de la jambe gauche, avec
début d'oedème malléolaire. Une phlébite est confirmée par l'examen en rhéopléthysmographie et
doppler, la phlébographie n'est pas réalisée. La décision d'un traitement curatif de cette
thrombose veineuse est prise. Quel traitement choisissez-vous parmi les propositions suivantes ?
A - Héparinate de sodium 400 UI/kg/j en injection intraveineuse continue
B - Héparinate de sodium 40 UI/kg/j en injections intraveineuses discontinues toutes les deux
heures
C - Hémoclar® 50 matin et soir en injections intramusculaires
D - Héparinate de sodium 100 UI/kg/j en injections intraveineuses discontinues toutes les deux
heures
E - Héparinate de sodium 40 UI/kg/j en injection intraveineuse continue
Bonne(s) réponse(s) : A

La posologie habituelle de l'héparine est de 300 à 500 UI/kg /24 heures mais est à adapter à chaque patient
en fonction du TCK (1,5 à 3 fois le témoin) et de l'héparinémie (0,2 à 0,4 UI/ml)

Quel examen demandez-vous pour la surveillance du traitement héparinique ?


A - Taux de prothrombine
B - Temps de céphaline kaolin
C - Temps de coagulation
D - Thromboélastogramme
E - Temps de Reptilase®
Bonne(s) réponse(s) : B

Sans commentaire.

Madame X est sortante, 7 jours après la phlébite qui a bien répondu au traitement. Parmi les
attitudes thérapeutiques suivantes, laquelle vous parait adaptée pour la suite ?
A - Sintrom® pour amener le TP à 30-35 %
B - Poursuite de l'héparinothérapie intraveineuse 2 mois
C - Hémoclar® : 6 gélules/j pendant 2 mois
D - Aspirine® 1 g/j per os pendant 2 mois
E - Arrêt du traitement anticoagulant
Bonne(s) réponse(s) : A

Le relais par les antivitamines K par os peut être entrepris dès la cédation des signes cliniques et sera
prolongé pendant 3 mois au moins.

Un malade de 60 ans, aynt fait l'objet cinq jours auparavant, d'une ostéosynthèse du tibia, présente un point
de côté thoracique avec angoisse et polypnée. Le rythme cardiaque est rapide à 100/mn : la pression artérielle
est de 140/80 mmHg. L'examen ne montre ni cyanose, ni signes veineux évidents au niveau des membres
inférieurs. Le diagnostic d'embolie pulmonaire est évoqué.

Quel(s) examen(s) vous paraissent utiles pour renforcer cette présomption diagnostique ?
A - ECG
B - Dosage des gaz du sang
C - Dosage des L.D.H
D - Tomographie pulmonaires
E - Scintigraphie pulmonaire
Bonne(s) réponse(s) : A B C E

L'ECG à la recherche de signe de coeur pulmonaire aiguë (déviation axiale droite, bloc incomplet droit, trouble
de repolarisation en précordiales droites), les gaz du sang à la recherche d'une hypoxémie-hypocapnie.
On observe habituellement une augmentation des LDH au cours de l'EP mais qui n'est absolument pas
spécifique ; le dosage enzymatique est surtout utile pour éliminer une nécrose myocardique (CPK normales).
540
Exclusivement sur DOC - DZ : www.doc-dz.com NADJI 85
RESIDANAT EN POCHE TOME II
Cas Clinique en QCM

Parmi ces 5 examens, lequel ou lesquels sont utiles pour reconnaître le point de départ de
l'éventuelle embolie ?
A - Doppler veineux du membre inférieur
B - Scanner pelvien
C - Capillaroscopie
D - Phlébographie
E - Pléthysmographie
Bonne(s) réponse(s) : A D E

A et E sont surtout discriminatifs dans les thromboses haut situées. La phlébographie reste la méthode de
choix en permettant la visualisation de la thrombose.

Quel traitement allez-vous proposer immédiatement ?


A - Héparinothérapie IV à doses anticoagulantes
B - Traitement fibrinolytique IV
C -Antivitamine K
D - Antiagrégant plaquettaire
E - Interruption partielle de la veine cave inférieure par filtre endo veineux
Bonne(s) réponse(s) : A

Sans commentaire.

La surveillance biologique à instituer dans les jours qui suivent comporte :


A - Numération des plaquettes
B - Mesure de la vitesse de sédimentation
C - Dosage du taux de prothrombine
D - Trhomboélatrogramme
E - Temps de coagulation sensibilisé (TCK ou TCA)
Bonne(s) réponse(s) : A B E

La numération des plaquettes au cours d'un traitement héparinique doit être réalisée au moins une fois par
semaine afin de dépister précocement une thrombopénie. Le thromboélastogramme est un test global de la
coagulation coûteux et peu reproductible qui n'est plus utilisé en pratique.
La mesure de la vitesse de sédimentation, accélérée initialement permet d'apprécier la régression progressive
du syndrome inflammatoire sous traitement.

Au dixième jour, la surveillance biologique montre un taux de plaquettes à 50 000/mm3. Votre


traitement comporte :
A - Arrêt de l'héparine habituelle
B - Mise en route d'un thrombolyse
C - Passage aux antivitamines K
D - Augmentation de la dose d'héparine
E - Perfusion intraveineuse de plasma frais congelé
Bonne(s) réponse(s) : A C

La survenue d'une thrombopénie au cours d'un traitement héparinique curatif impose l'arrêt immédiat de
l'héparine et la mise en route d'un traitement par antivitamines K à demi-vie brève (Tromexane® par exemple),
lorsque le risque thrombotique est important.

Un homme de 70 ans consulte pour douleur du mollet gauche survenant pour une marche de 100 m. Il
présente dans ses antécédents un déficit de type hémiplégique droit régressif il y a 4 mois et une hypertension
artérielle.
A l'examen clinique il est noté une disparition du pouls fémoral gauche, une tuméfaction abdominale battante
et un discret souffle cervical gauche. La radiographie simple de l'abdomen montre des calcifications de type
artériel évoquant une ectasie aortique.

Quel examen simple permet de faire le diagnostic de thrombose ilio-fémorale ?


A - Prise des pouls distaux
B - Prise des oscillations distales
C - Pléthysmographie
D - Echo doppler
E - Artériographie fémorale gauche
Bonne(s) réponse(s) : D

L'échodoppler est un examen non invasif, facilement reproductible qui précise de façon assez fiable la
topographie et l'importance des lésions athéromateuses, et qui peut dépister une éventuelle thrombose
(interruption du flux).
541
Exclusivement sur DOC - DZ : www.doc-dz.com NADJI 85
RESIDANAT EN POCHE TOME II
Cas Clinique en QCM

Quel examen permet constamment d'affirmer l'anévrisme de l'aorte ?


A - La palpation
B - La radiographie simple
C - L'échotomographie
D - Le Doppler
E - L'angiographie veineuse digitalisée
Bonne(s) réponse(s) : C

L'échotomographie est l'examen le plus simple et le meilleur pour détecter et évaluer la taille d'un anévrysme
de l'aorte abdominale, mais cet examen peut être gêné par une distension gazeuse abdominale importante.
L'angiographie même artérielle est moins précise en raison de la présence de thrombus qui peuvent masquer
l'anévrysme ou minorer artificiellement sa taille. Le scanner abdominal est sans doute aussi performant que
l'échographie mais plus onéreux.

Considérant l'antécédent neurologique, lequel parmi ces 5 examens vous semble le plus utile en
première intention ?
A - Angiographie cervico-encéphalique
B - Doppler et échographie vasculaire, cervicaux
C - Enregistrement holter/24h
D - Electroencéphalogramme
E - Scanner crânien
Bonne(s) réponse(s) : B

Le doppler couplé à l'échographie des troncs cervicaux sera à demander en première intention avant une
éventuelle angiographie cervicoencéphalique, afin de dépister d'éventuelles lésions associées chez ce patient
polyartériel (souffle cervical gauche).

Il y aurait lieu de craindre la fissuration de l'anévrisme devant un ou plusieurs des signes suivants :
A - Douleur abdominale
B - Douleur lombaire
C - Chute tensionnelle brutale
D - Constipation
E - Poussée d'ischémie distale à gauche en dehors de tout collapsus
Bonne(s) réponse(s) : A B C D E

Les douleurs dorsolombaires sont le maître symptôme. Mais il faudra évoquer le diagnostic de fissuration
devant la survenue d'un état de choc, devant une constipation (iléus réflexe ou compression par un
anévrysme ayant récemment augmenté de volume) enfin une poussée d'ischémie distale pouvant être en
rapport avec une anémie sévère.

Le bilan complet révèle un anévrisme sous rénal de 6 cm de diamètre avec thrombose iliaque
gauche et une sténose de la carotide gauche isolée de 80 %. Quelle attitude thérapeutique
proposez-vous ?
A - Restauration carotidienne puis cure de l'anévrisme et de la thrombose iliaque
B - Restauration de l'anévrisme et de l'iliaque, et secondairement de la carotide
C - Restauration seulement de l'anévrysme et de l'iliaque
D - Restauration seulement de la carotide
E - Surveillance sous traitement médical
Bonne(s) réponse(s) :

QUESTION ANNULEE.

542
Exclusivement sur DOC - DZ : www.doc-dz.com NADJI 85
RESIDANAT EN POCHE TOME II
Cas Clinique en QCM
Une malade de 62 ans est hospitalisée pour malaises et pertes de connaissance brutaux et brefs depuis une
huitaine de jours. L'interrogatoire ne retrouve pas d'antécédents particuliers en dehors d'une hypertension
artérielle traitée par diurétiques et d'une constipation chronique pour laquelle la malade prend des laxatifs. A
l'examen, on constate un souffle mitro-aortique d'intensité 1/6ème avec des bruits normaux. La tension
artérielle est à 170/100. L'examen neurologique est normal. L'électrocardiogramme montre un rythme sinusal
à 68/mn avec d'assez nombreuses extrasystoles ventriculaires isolées, polymorphes et à couplage tardif, un
espace PR à 24/100ème de seconde, un axe de QRS à 0 degré, des complexes QRS de 10/100 de seconde,
des ondes T aplaties de façon diffuse avec un QT à 0,62 seconde. La radiographie montre un coeur de
volume normal, on note des calcifications en coup d'ongle du bouton aortique. Le ionogramme objective une
natrémie à 133 mmol/l, une kaliémie à 2,4 mmoI/I et une créatininémie à 85 micromol/l.

Quelle est la cause la plus probable des malaises ?


A - Rétrécissement aortique
B - Bloc sino-auriculaire
C - Bloc auriculo-ventriculaire paroxystique
D - Tachycardie ventriculaire (par torsades de pointe)
E - Poussée hypertensive
Bonne(s) réponse(s) : D

Commentaire : voir question suivante.


Commentaire de la question suivante :
L'association d'une hypokaliémie probablement induite par les diurétiques et les laxatifs, et de signes
d'hypokaliémie sur ECG (allongement important de QT, aplatissement des ondes T, ESV polymorphes à
couplage tardif) doit faire suspecter fortement l'éventualité de torsades de pointe, surtout chez un malade qui
fait des malaises. La description des malaises n'apporte pas d'élément déterminant en faveur de ce diagnostic
et l'examen clinique permet d'éliminer d'autres étiologies mais ne peut faire partie de la réponse compte tenu
de la formulation de la question.

Sur quel(s) argument(s) repose votre diagnostic ?


A - Description des malaises
B - Examen clinique
C - Radiographie
D - Electrocardiogramme
E - Ionogramme
Bonne(s) réponse(s) : D E

L'association d'une hypokaliémie probablement induite par les diurétiques et les laxatifs, et de signes
d'hypokaliémie sur ECG (allongement important de QT, aplatissement des ondes T, ESV polymorphes à
couplage tardif) doit faire suspecter fortement l'éventualité de torsades de pointe, surtout chez un malade qui
fait des malaises. La description des malaises n'apporte pas d'élément déterminant en faveur de ce diagnostic
et l'examen clinique permet d'éliminer d'autres étiologies mais ne peut faire partie de la réponse compte tenu
de la formulation de la question.

Quelle(s) thérapeutique(s) pouvez-vous utiliser en urgence ?


A - Antiarythmique IV
B - Stimulation endocavitaire
C - Administration de chlorure de potassium IV
D - Inhibiteur calcique sublingual
E - Magnésium intraveineux
Bonne(s) réponse(s) : B C E

En cas de torsades de pointe, tout anti-arythmique est à proscrire car il ne pourrait que les aggraver.
Le traitement repose sur la correction de l'hypokaliémie, la stimulation cardiaque rapide 1000 à 120/min,
même en l'absence de bradycardie basale excessive. Enfin, l'administration de magnésium à fortes doses a
souvent un effet spectaculaire même en l'absence d'hypomagnésémie.

Quel(s) antiarythmique(s) oral(oraux) prescrirez-vous ultérieurement à cette malade ?


A - Quinidine (Serecor®, Longacor®)
B - Amiodarone (Cordarone®)
C - Disopyramide (Rythmodan®)
D - Sotalol (Sotalex®)
E - Aucun
Bonne(s) réponse(s) : E

Tous ces médicaments anti-arythmiques susceptibles d'allonger le QT sont à proscrire formellement.

543
Exclusivement sur DOC - DZ : www.doc-dz.com NADJI 85
RESIDANAT EN POCHE TOME II
Cas Clinique en QCM

Quel(s) est(sont) parmi ces médicaments celui(ceux) qui a(ont) également une activité anti-
angineuse ?
A - Quinidine (Serecor®)
B - Amiodarone (Cordarone®)
C - Disopyramide (Rythmodan®)
D - Sotalol (Sotalex®)
E - Aucune des propositions précédentes
Bonne(s) réponse(s) : B D

L'amiodarone et le sotalol, qui est un bêta-bloquant possédant une activité anti-arythmique de la classe III, ont
tous deux une activité anti-angineuse.

Un homme de 53 ans, en bon état général et sans antécédent pathologique notable, est hospitalisé 24 heures
après la survenue d'une douleur thoracique antérieure constrictive, survenue spontanément et qui a duré
environ une heure. A l'admission, l'examen clinique note pour seules anomalies une fébricule à 37,5 degrés et
un bruit de galop présystolique. La radiographie pulmonaire montre un coeur de volume normal sans signes
de stase pulmonaire.

Un tracé ECG est enregistré :

Cliquez sur le bouton "Dessin" pour afficher l'ECG

Comment qualifier ce syndrome coronarien aigu ?

Cliquez sur le bouton 'Dessin' pour afficher le schéma.


A - Angor instable
B - Menace d'infarctus
C - Infarctus rudimentaire
D - Infarctus sous-endocardique circonférentiel
E - Infarctus transmural
Bonne(s) réponse(s) :

QUESTION ANNULEE.

Le bruit de galop présystolique correspond au renforcement pathologique de :

Cliquez sur le bouton 'Dessin' pour afficher le schéma.


A - B1
B - B2
C - B3
D - B4
E - Aucune des propositions précédentes n'est exacte
Bonne(s) réponse(s) :

QUESTION ANNULEE.

Parmi les cinq enzymes dosés chez ce patient, lequel est à ce stade et simultanément le plus
sensible et le plus spécifique pour le diagnostic d'une lyse cellulaire myocardique ?
Cliquez sur le bouton 'Dessin' pour afficher le schéma.
A - CPK totales
B - CPK-MB
C - SGOT
D - LDH totales
E - Alpha HBDH
Bonne(s) réponse(s) :

QUESTION ANNULEE.

544
Exclusivement sur DOC - DZ : www.doc-dz.com NADJI 85
RESIDANAT EN POCHE TOME II
Cas Clinique en QCM

Comment qualifier les anomalies de la repolarisation présentes sur l'ECG ?

Cliquez sur le bouton 'Dessin' pour afficher le schéma.


A - Ischémie sous-épicardique
B - Lésion sous-épicardique
C - Ischémie-lésion sous-épicardique
D - Ischémie sous-endocardique
E - Ischémie-lésion sous-endocardique
Bonne(s) réponse(s) :

QUESTION ANNULEE.

Sur quelle artère coronaire siège probablement l'anomalie causale de l'insuffisance coronarienne
aiguë ?

Cliquez sur le bouton 'Dessin' pour afficher le schéma.


A - Tronc commun de la coronaire gauche
B - Interventriculaire antérieure
C - Diagonale
D - Circonflexe
E - Coronaire droite
Bonne(s) réponse(s) :

QUESTION ANNULEE.

A quelle complication est particulièrement exposé ce patient ?

Cliquez sur le bouton 'Dessin' pour afficher le schéma.


A - Insuffisance ventriculaire gauche
B - Rupture cardiaque
C - Péricardite
D - Accidents thrombo-emboliques
E - Extension d'infarctus
Bonne(s) réponse(s) :

QUESTION ANNULEE.

Quelle attitude pratique vous semble la plus logique ?

Cliquez sur le bouton 'Dessin' pour afficher le schéma.


A - Traitement fibrinolytique immédiat
B - Héparinothérapie seule
C - Héparinothérapie + médicaments antiangineux
D - Coronarographie d'emblée
E - Héparinothérapie + médicaments antiangineux + coronarographie précoce
Bonne(s) réponse(s) :

QUESTION ANNULEE.

Un jeune homme en parfaite santé (20 ans, 60 kg) a été plâtré pour entorse grave de la cheville. Six jours
après il présente des signes d'embolie pulmonaire sans collapsus cardiovasculaire que l'on rattache à une
phlébite fémoropoplitée passée inaperçue. Avant d'entreprendre l'héparinothérapie qui s'impose vous
demandez un bilan d'hémostase : temps de Quick 80 % ; temps de céphaline activée 28 sec (T : 31 sec) ;
plaquettes 35.0000/mm3; fibrinogène 5,5 g/l.

Parmi les éléments de ce bilan d'hémostase lequel aurait dû vous faire prévoir cette maladie
thromboembolique ?
A - Temps de Quick discrètement allongé
B - Temps de céphaline activée discrètement raccourci
C - Plaquettes à la limite supérieure de la normale
D - Discrète hyperfibrinogénémie
E - Aucun des examens ci-dessus
Bonne(s) réponse(s) : E

On pourrait éventuellement observer une augmentation des produits de dégradation du fibrinogène.

545
Exclusivement sur DOC - DZ : www.doc-dz.com NADJI 85
RESIDANAT EN POCHE TOME II
Cas Clinique en QCM

Parmi les schémas de traitement suivants le ou lesquels est(sont) adapté(s) à la situation ?


A - Héparine standard sous cutanée : 5 000 unités toutes les 12 heures
B - Héparine standard intramusculaire : 18 000 unités toutes les 12 heures
C - Héparine standard intra-veineuse : 9 000 unités toutes les 6 heures
D - Héparine standard intraveineuse : 3 000 unités toutes les 2 heures
E - Héparine standard intraveineuse : 36 000 unités par 24 heures administrées en perfusion
continue
Bonne(s) réponse(s) : D E

La posologie initiale avant adaptation aux tests de coagulation est de 400 à 600 UI/kg par 24 heures à
administrer en continue à la seringue électrique ou en injections intraveineuses discontinues toutes les deux
heures.

Le ou lesquels des examens biologiques suivants est(sont) nécessaire(s) pour surveiller


l'efficacité biologique et la tolérance du traitement ?
A - La numération des plaquettes entre le 5-10ème jour du traitement
B - Le temps de saignement
C - Le temps de Quick (taux de prothrombine)
D - Le temps de céphaline activée
E - Le dosage de l'antithrombine III
Bonne(s) réponse(s) : A D

A, à la recherche d'une thrombopénie induite par l'héparine, D pour surveiller l'efficacité biologique.
Le dosage de l'antithrombine III n'est pas à réaliser d'emblée dans la mesure où il existe une cause de
phlébite (période post-opératoire et alitement).

A J5 du traitement vous apprenez que 2 autres membres de la famille ont présentéé des
thromboses veineuses spontanées. L'antithrombine III du patient = 55 %. Quelle est votre
interprétation ?
A - Ce taux est dans les limites de la normale pour un homme jeune de 20 ans
B - L'antithrombine III a été consommée par la maladie thromboembolique
C - Ce résultat doit faire craindre une aggravation de la maladie thromboembolique
D - Vous avez à coup sûr détecté un déficit congénital
E - Ce résultat peut être la conséquence de l'héparinothérapie
Bonne(s) réponse(s) : E

L'héparine n'agit qu'en présence de l'antithrombine III (AT III) qui est son cofacteur, et ce complexe va agir
essentiellement par l'inhibition de la thrombine activée et du facteur Xa ; cela explique la baisse de l'AT III au
cours d'un traitement héparinique par consommation. Une baisse de l'AT III s'observe également au cours des
thromboses mais de façon moins constante.

La protéine C du patient est à 50 %. Quelle est votre interprétation ?


A - Ce taux est normal pour un homme jeune de 20 ans
B - Ce taux reflète une carence en vitamine K
C - Ce résultat est ininterprétable à cause de l'héparinothérapie
D - La protéine C a été consommée par la maladie thromboembolique
E - Forte probabilité de déficit congénital
Bonne(s) réponse(s) : E

Les taux normaux de protéine C sont de 65 à 145 %. Les antivitamines K diminuent de façon importante les
taux de protéine C, mais l'héparine n'interfère pas (protéine vitamine K-dépendante).

Après quinze jours d'héparinothérapie conventionnelle, il est décidé de passer au Sintrom®, 1


cp/j. Combien de temps après la prise du premier comprimé allez-vous évaluer l'efficacité du
traitement pour la première fois ?
A - 6 heures
B - 12 heures
C - 24 heures
D - 48 heures
E - 96 heures
Bonne(s) réponse(s) : D

Le Sintrom® est un dérivé coumarinique dont l'efficacité est obtenue au bout de 48 heures environ.
A noter que l'administration d'antivitamines K chez un patient présentant un déficit en protéines a pu conduire
à la survenue de nécrose cutanée.

546
Exclusivement sur DOC - DZ : www.doc-dz.com NADJI 85
RESIDANAT EN POCHE TOME II
Cas Clinique en QCM

Sur quel examen vous basez-vous pour équilibrer le traitement par antivitamine K ?
A - Le temps de saignement
B - Le dosage du fibrinogène
C - Le temps de thrombine
D - Le dosage de la protéine C
E - Le temps de Quick
Bonne(s) réponse(s) : E

Sans commentaire.

Monsieur P... Paul, 75 ans, a été hospitalisé au bloc de réanimation respiratoire depuis 12 jours en raison d'un
état de choc lié à une septicémie à Gram négatif. Outre les soins de réanimation comprenant une prévention
de la maladie thromboembolique par héparinothérapie IV (1000 UI toutes les 2 heures), il est perfusé par voie
intraveineuse avec du soluté salé physiologique et du sérum glucosé isotonique.
A l'occasion de la surveillance biologique systématique, on relève le bilan suivant, communiqué aussitôt au
service :
- TQ = 10 % TCA = 81"/32" (V = 100 % ; VII + X = 13 % ; II = 16) temps de thrombine = 36"/17" ; temps de
reptilase = 14'"/13" ; fibrinémie = 8 g/l.
- GR = 3,57,10 6/mm3 ; Hb = 11,1 g/dI ; VMG = 90,3 micro3 ; CGM Hb = 34,5 %
- GB = 14.375/mm3 (PNN = 80 % , PE = 2 %, L = 13 % , M = 5 %).

Dans ce cas précis, l'allongement du TCA est consécutif à :


A - Déficit en facteur VII
B - Déficit en facteur II
C - Déficit en facteur IX
D - Hyperfibrinémie
E - Héparinothérapie
Bonne(s) réponse(s) : B C E

Le TCA explore la voie intrinsèque de la coagulation, son allongement peut traduire une diminution des
facteurs XII, XI, prékalliscréine, kininogéne de haut poids molléculaire, VIII, IX, X, V, VII, et fibrinogène, une
anomalie de la fibrinoformation ou la présence d'inhibiteurs (héparine).

Parmi les éléments suivants capables d'allonger le temps de thrombine, indiquez celui qu'il faut
invoquer pour expliquer l'allongement du temps de thrombine dans ce cas particulier :
A - Présence d'héparine
B - Présence de PDF
C - Dysfibrinogénémie
D - Hyperfibrinémie
E - Dysglobulinémie
Bonne(s) réponse(s) : A

La normalité du temps de reptilase (insensible à l'héparine) associée à l'augmentation du temps de thrombine


plaide pour la réponse A.

Dans ce cas, quel diagnostic parmi les suivants peut-on évoquer ?


A - Avitaminose K
B - CIVD
C - Insuffisance hépatique sévère
D - Surdosage en héparine
E - Prélèvement fait par erreur sur Sintrom®
Bonne(s) réponse(s) : A

La normalité du fibrinogène et du facteur V permet d'éliminer B et C. Le surdosage en héparine ne peut


expliquer la baisse du facteur VII.

Peu de temps après la réception des résultats par le service, on décide de replacer un cathéter
sous-clavier. Dans ces conditions indiquez parmi les propositions suivantes celle(s) que vous
retiendrez :
A - Injection IV de 60 ml de PPSB
B - Injection IM de 50 mg de vitamine K1
C - Arrêt de l'héparinothérapie
D - Perfusion de deux poches de plasma frais congelé
E - Perfusion d'un flacon de solution d'AT III
Bonne(s) réponse(s) :

QUESTION ANNULEE.

547
Exclusivement sur DOC - DZ : www.doc-dz.com NADJI 85
RESIDANAT EN POCHE TOME II
Cas Clinique en QCM
Une jeune femme de 22 ans, secrétaire, vous est adressée pour bilan d'une hypertension artérielle. En dehors
de céphalées matinales occasionnelles et d'une certaine asthénie elle est asymptomatique. Son HTA a été
découverte lors d'une visite de médecine du travail, avant embauche il y a quelques mois.
Elle est permanente. Il n'y a pas d'antécédents personnels particuliers et pas de notion d'HTA dans la famille.
A l'examen, il.s'agit, d'une jeune femme morphologiquement normale, avec une pression artérielle à 16/10 cm
de Hg aux 2 bras en position couchée, qui passe à 14,5/9 cm de Hg en position debout. Son pouls est à
70/mn. Le reste de l'examen est normal.
Biologiquement les éléments suivants sont retrouvés :
- Hb : 14,6 g/l ; Ht : 39 % ; protéinurie, glycosurie absentes ; glycémie à jeun et HGPO normales
- Créatininémie : 9 mg/l
- Na+ : 146 mmol/l ; K : 2,9mmol/l ; CI : 100 mmol/l
- Bicarbonates : 34 mmol/l
- Ca : 9 6 mg/I - P : 31 mg/l
- Electrophorèse normale
- Radiographie de thorax normale.

Au sujet des données de cette observation il est exact que :


A - Le même tableau clinique pourrait comporter un souffle lombaire
B - L'hypernatrémie n'élimine pas la prise de diurétique sulfamide
C - Les bicarbonates peuvent être augmentés par la prise importante d'eau de Vichy
D - Les estroprogestatifs pourraient être responsables du tableau clinique et biologique
E - La prise chronique de réglisse est compatible avec ce tableau clinique et biologique
Bonne(s) réponse(s) :

QUESTION ANNULEE.

Chez cette patiente dont la natriurèse est de 100 mmoles/24 h, l'hypokaliémie :


A - A été provoquée par le régime désodé
B - A été provoquée par un régime normalement salé
C - Aurait pu être provoquée par la prise de corticoïdes
D - La chlorémie normale élimine les pertes digestives d'origine haute
E - L'hypokaliémie peut à elle seule être provoquée par l'alcalose métabolique en dehors d'une
fuite rénale de potassium
Bonne(s) réponse(s) : C D

Le régime désodé n'est pas strict (natriurèse 100 mmol/24 H) et ne peut dans ce cas expliquer l'hypokaliémie.
La normalité de la chlorémie permet raisonnablement d'exclure des pertes digestives d'origine haute.
En cas d'alcalose métabolique, l'hypokaliémie est due à une fuite rénale de potassium.

Chez cette patiente hypertendue explorée en régime normosodé, le diagnostic le plus


vraisemblable est :
A - Un syndrome de Cushing
B - Une hyperparathyroïdie
C - Un hyperaldostéronisme primaire
D - Un hyperaldostéronisme dû à une ischémie rénale
E - Une hypertension essentielle
Bonne(s) réponse(s) : C

La constatation d'une hypokaliémie associée à une HTA en l'absence de cause évidente (diurétiques, laxatifs,
réglisse) et en régime normosodé doit faire évoquer en premier lieu un hyperaldostéronisme primaire.

Pour préciser votre diagnostic, vous allez pratiquer en première intention :


A - Ionogrammes sanguin et urinaire sur 24 heures, plusieurs jours de suite, en régime
normosodé
B - Le cycle de la cortisolémie
C - L'aldostéronémie et activité rénine plasmatique
D - Le dosage de l'uricémie
E - Le dosage urinaire des catécholamines
Bonne(s) réponse(s) : A B C

Dans les urines, on trouvera une alcalose urinaire contrastant avec l'alcalose sanguine, une hyperkaliurie avec
natriurèse normale (Na u/K u > 1, phénomène d'échappement K u > 20 mEq/24h)
La mesure de l'activité rénine plasmatique est basse en position couchée et peu stimulable en position
debout. Enfin on note une élévation de l'aldostérone plasmatique et de la tétrahydroaldostéronurie. On
pratiquera un cycle de la cortisolémie pour s'assurer de sa normalité.

548
Exclusivement sur DOC - DZ : www.doc-dz.com NADJI 85
RESIDANAT EN POCHE TOME II
Cas Clinique en QCM

Pour préciser votre diagnostic vous devez savoir que :


A - L'adénome surrénalien de la glomérulée est associé à une conservation du rythme
nycthéméral de l'aldostérone
B - Dans l'adénome surrénalien de la glomérulée, l'aldostéronémie s'élève à l'orthostatisme
C - Dans l'hyperplasie de la glomérulée, le rythme nycthéméral de l'aldostérone a disparu
D - Dans l'hyperplasie de la glomérulée, l'aldosteronémie ne s'élève pas en orthostatisme
E - Au cours du test aux spironolactones, la T.A. et la kaliémie ne sont corrigées que chez les
sujets porteurs d'un adénome de la glomérulée
Bonne(s) réponse(s) : A C

La régulation de la synthèse d'aldostérone se fait surtout par l'ACTH dans l'adénome avec conservation d'un
rythme nycthéméral, alors que la régulation se fait à partir du système rénine-angiotensine dans l'hyperplasie
bilatérale. Donc dans l'adénome : pas de freinage de l'aldostérone par surcharge salée, diminution de
l'aldostérone à l'orthostatisme (fait à 8 h et 12 h, rythme nycthéméral). Dans l'hyperplasie, c'est le contraire.

A propos des effets des spironolactones, il est exact que :


A - Les spironolactones en traitement prolongé peuvent modifier le cycle menstruel chez la
femme
B - Les spironolactones en traitement prolongé peuvent entraîner une impuissance chez
l'homme
C - Les spironolactones en traitement prolongé peuvent entraîner une gynécomastie
D - Les effets secondaires obtenus ne disparaissent pas à l'arrêt des spironolactones
E - Le surdosage en spironolactones peut chez le sujet normal être responsable d'un tableau
biologique évocateur d'insuffisance rénale
Bonne(s) réponse(s) : A B C E

Les effets secondaires sont en général réversibles, cependant dans de rares cas, la gynécomastie a pu
persister à l'arrêt du traitement.
Les spironolactones peuvent donner une acidose hyperchlorémique avec hyperkaliémie et une altération de la
fonction rénale pouvant évoquer un tableau d'insuffisance rénale.

Un homme de 60 ans, obèse et fumeur, consulte pour aggravation récente d'une claudication du mollet droit
avec périmètre de marche à 50 mètres et sensation de pied froid avec paresthésies. Les symptômes d'artérite
ont débuté, il y a 3 ans avec une gêne à la marche dans le mollet gauche, et périmètre de marche de l'ordre
de 300 m ; de plus depuis 6 mois est apparue une gêne à type de fatigabilité à la marche dans tout le membre
inférieur droit, avec engourdissement descendant depuis la hanche jusqu'au pied.
A l'examen, le pouls fémoral est perçu, siège d'un frémissement et d'un souffle râpeux à l'auscultation ; en
aval, aucun pouls n'est retrouvé. A gauche, seul le pouls fémoral est bien perçu, siège d'un souffle. Le Doppler
confirme une perméabilité des axes pédieux et tibiaux postérieurs droits et gauches avec un indice de
pression de 0,60 à droite et 0,80 à gauche.
Avec ces seules données, vous êtes en mesure de préciser le ou les sièges et le ou les types des lésions
artérielles :

A droite :
A - Sténose iliaque externe
B - Thrombose iliaque externe
C - Thrombose fémorale superficielle
D - Thrombose fémorale profonde
E - Thrombose tibio-péronière
Bonne(s) réponse(s) : A C

A s'explique par le souffle et le frémissement fémoral droit. B est exclu car le pouls fémoral est bien perçu. C
est responsable de l'abolition des pouls en aval. E est faux car les axes tibiaux et pédieux sont perméables.

A gauche :
A - Sténose iliaque
B - Thrombose fémorale commune
C - Thrombose fémorale superficielle
D - Thrombose tibio-péronière
E - Thrombose tibiale antérieure
Bonne(s) réponse(s) : A C

Explication identique.

549
Exclusivement sur DOC - DZ : www.doc-dz.com NADJI 85
RESIDANAT EN POCHE TOME II
Cas Clinique en QCM

Avant la décision thérapeutique, il est utile de demander :


A - Pression transcutanée d'O2 (TcPO2)
B - Echographie abdominale
C - Doppler avec épreuve d'effort
D - Angiographie
E - Thermographie
Bonne(s) réponse(s) : B C D

A n'est pas réalisée en pratique couvrante mais permet d'apprécier la foncetion métabolique tissulaire de
l'artériopathe en stade III. B vérifiera l'état de l'aorte abdominale. L'épreuve d'effort permet d'améliorer la
sensibilité du doppler de repos. L'artériographie des membres inférieurs est évidemment indispensable chez
ce patient.

Quelle(s) technique(s) peu(ven)t être utilisée(s) chez ce patient pour le bilan artériographique ?
A - Angiographie digitalisée par voie veineuse
B - Tomodensitométrie avec angioscan
C - Artériographie fémorale rétrograde
D - Aortographie translombaire
E - Artériographie par voie axillaire
Bonne(s) réponse(s) : C D E

L'angiographie digitalisée par voie veineuse est théoriquement faisable mais rarement suffisante pour faire un
bilan complet des lésions artérielles, car elle nécessiterait alors des quantités importantes de produit de
contraste. C et D sont le plus souvent réalisées soit de manière conventionnelle soit par technique numérisée.
Enfin, la voie axillaire est rare mais possible.

Quel sera le geste restaurateur proposé ?


A - Pontage fémoro-poplité droit
B - Endoartérectomie aorto-fémoro-poplitée
C - Pontage aorto-bifémoral
D - Angioplastie transmurale
E - Sympathectomie lombaire isolée
Bonne(s) réponse(s) : C

C sera choisi car les lésions sont étagées et bilatérales.

Monsieur D.., 60 ans sans antécédents connus, est réveillé à 3 h du matin par une violente douleur
précordiale irradiant dans les bras et le cou. Son médecin traitant, arrivant à son chevet lui prescrit une dragée
de Trinitrine par voie sub linguale qui n'entraîne aucune atténuation de la symptomatologie Monsieur D... est
hospitalisé. L'ECG réalisé à 4h30 met en évidence un sus décalage du segment ST de 3 mm dans les
dérivations D2 D3 VF et un sous décalage de 2 mm en dérivations précordiales, le rythme est sinusal et
stable. Ces modifications persistent après une nouvelle administration de Trinitrine. La fréquence cardiaque
est à 50 battements par minute. L'examen ne montre d'anomalie notable si ce n'est une tension artérielle à
90/60 mm Hg.

Quel diagnostic évoquez-vous ?


A - Angor de Prinzmetal
B - Péricardite aiguë
C - Infarctus du myocarde
D - Embolie pulmonaire
E - Dissection aortique
Bonne(s) réponse(s) : C

Les caractères et la durée de la douleur, sa résistance à la trinitrine et les modifications ECG avec image en
miroir sont en faveur d'un infarctus du myocarde.

En plus d'un traitement thrombolytique d'emblée que peut-on envisager ?


A - Bétabloquants intraveineux
B - Adalate (Nifédipine)
C - Aspirine®
D - Héparine en relais du thrombolytique
E - Xylocaïne intraveineuse
Bonne(s) réponse(s) : C D

Les Bêta-bloquants intraveineux à la phase aiguë de l'infarctus du myocarde ont fait la preuve de leur
efficacité dans la baisse de la mortalité (étude ISIS 1) mais la fréquence cardiaque à 50/min est dans ce cas
une contre-indication à leur emploi. L'association de l'aspirine à un traitement thrombolytique permet de
réduire de manière plus importante la mortalité (étude ISIS 2). Par contre, l'administration systématique
d'inhibiteurs calciques ou de Xylocaïne intraveineuse ne semble pas procurer de bénéfice significatif.
550
Exclusivement sur DOC - DZ : www.doc-dz.com NADJI 85
RESIDANAT EN POCHE TOME II
Cas Clinique en QCM

Un traitement thrombolytique serait contre indiqué par :


A - Ulcère gastro duodénal récent
B - Antécédent d'accident vasculaire cérébral de moins de 3 mois
C - Antécédent d'infarctus du myocarde
D - Rétinopathie diabétique
E - Antécédents de chirurgie vasculaire des membres inférieurs de plus de 3 mois
Bonne(s) réponse(s) : A B D

Un AVC de moins de 3 mois est une contre-indication absolue. L'ulcère gastro-duodénal récent ou la
rétinopathie diabétique sont des contre-indications plus relatives, mais représentent un risque de saignement
indiscutable. Enfin, on doit se trouver à plus de 8 à 10 jours d'une intervention et à plus de 3 mois de
l'implantation d'une prothèse vasculaire en dacron.

Sous l'effet du traitement, une revascularisation coronarienne parait obtenue; quel(s) signe(s)
est(sont) en faveur de cette interprétation ?
A - Sédation de la douleur et disparition du sus décalage du segment ST en moins d'une heure
B - Aggravation de la symptomatologie clinique et électrique suivie d'une amélioration rapide
C - Tachycardie ventriculaire brutalement alors que le sus décalage ST est redevenue
isoélectrique
D - Apparition d'une onde Q
E - Elévation majeure de CPK MB dans les 6 premières heures qui suivent le traitement
Bonne(s) réponse(s) : A B C E

La preuve absolue d'une revascularisation ne peut être apportée que par l'opacification coronaire. Néanmoins,
certains critères cliniques de reperfusion existent qu'on appelle "syndrome de revascularisation" : il s'agit des
critères A C E. Néanmoins, ces critères sont peu sensibles et peu spécifiques, et on a aussi décrit des
aggravations paradoxales suivies d'une amélioration (B).

Une coronarographie avec ventriculographie est réalisée après 3 jours d'évolution. Elle montrera
probablement :
A - Une sténose de l'artère coronaire droite
B - Une sténose de l'artère interventriculaire antérieure
C - Une occlusion d'une artère diagonale
D - Une hypokinésie de la paroi inférieure du ventricule gauche
E - Une hypokinésie de la paroi antéro-latérale du ventricule gauche
Bonne(s) réponse(s) : A D

Nous sommes en présence d'un infarctus inférieur (signes ECG en D2D3VF), l'examen angiographique
révèlera donc a priori une sténose de l'artère coronaire droite et une hypokinésie inférieure.

Quelle est la complication évolutive la plus fréquente attendue chez ce patient au cours de
l'évolution hospitalière ?
A - Hémopéricarde
B - Récidive douloureuse précordiale (ischémique)
C - Fibrillation ventriculaire tardive
D - Rupture du coeur
E - Insuffisance ventriculaire gauche
Bonne(s) réponse(s) : B

En effet, la thrombolyse a l'avantage d'avoir reperméabilisé l'artère, qui était occluse par une thrombose.
Cependant, elle laisse persister en général une sténose de l'artère plus ou moins sévère pouvant être
responsable de nouveaux épisodes d'ischémie.

551
Exclusivement sur DOC - DZ : www.doc-dz.com NADJI 85
RESIDANAT EN POCHE TOME II
Cas Clinique en QCM
Une femme de 27 ans, sans antécédent particulier, est hospitalisée pour oedème aigu pulmonaire.
L'interrogatoire retrouve la notion d'une dyspnée d'effort s'aggravant progressivement depuis 1 an 1/2, des
antécédents de palpitations, la notion d'une hémiparésie droite régressive spontanée en quelques heures il y
A 6 mois. Depuis 24 heures, elle se plaint d'une sensation de palpitations persistantes et d'une dyspnée de
repos avec expectorations aérées et rosées.
L'examen clinique montre une patiente fatiguée, la pression artérielle est à 90 mm de mercure pour la
systolique, la fréquence cardiaque est à 130/mn, l'examen neurologique est normal, l'auscultation pulmonaire
montre l'existence de nombreux râles crépitants remontant jusqu'au milieu des deux champs pulmonaire.
L'auscultation cardiaque trouve à la pointe un éclat de B1, un roulement diastolique 3/6, un claquement
d'ouverture de la mitrale, l'absence de souffle systolique, et retrouve au niveau du bord gauche du sternum un
éclat de B2. La radiographie thoracique montre un coeur légèrement augmenté de volume, un débord droit, un
arc moyen gauche en double bosse, un arc inférieur gauche allongé, et des images d'opacités alvéolaires en
aile de papillon au niveau du parenchyme pulmonaire, les artères pulmonaires apparaissent augmentées de
volume au niveau des hiles.
L'ECG montre une arythmie complète par fibrillation auriculaire, un bloc incomplet droit, un axe de QRS à
+120°.
Le bilan biologique est normal.

Le diagnostic de rétrécissement mitral décompensé retenu, quelle est la conduite à tenir


immédiate ?
A - Prescrire des dérivés nitrés par voie perlinguale
B - Instituer un traitement par héparine à doses efficaces
C - Instituer un traitement par diurétiques
D - Instituer un traitement par dobutamine
E - Réduire le trouble du rythme avant toute autre thérapeutique
Bonne(s) réponse(s) : B C

Il faut traiter cet OAP en urgence, tout en essayant de ralentir la fréquence cardiaque par l'emploi de
digitaliques. Il serait vain de vouloir réduire la fibrillation auriculaire avant tout traitement. Chez cette patiente,
l'emploi de dérivés nitrés doit être évitée en raison de l'hypotension artérielle.

Le mécanisme de l'accident neurologique ancien est probablement lié à :


A - Une poussée hypertensive
B - Un accès d'arythmie complète par fibrillation auriculaire
C - Un spasme de l'artère sylvienne gauche
D - Une embolie à partir d'un thrombus du ventricule gauche
E - Une chute du débit cardiaque
Bonne(s) réponse(s) : B

Au cours du RM, les thromboses de l'oreillette gauche sont favorisées par la dilatation de l'OG et la stase
sanguine dans cette cavité, ainsi que par la survenue d'une fibrillation auriculaire.

Le caractère serré du R. M. peut être fortement suspecté sur :


A - L'antécédent d'hémiparésie droite
B - La dyspnée d'effort s'aggravant progressivement
C - L'existence de signes cliniques et radiographiques d'hypertension artérielle pulmonaire
D - L'existence d'une pression artérielle à 90 mm de mercure
E - L'intensité du roulement diastolique
Bonne(s) réponse(s) : B C

A D et E ne sont pas des critères de sévérité du RM.

Parmi ces 5 signes radiologiques, lequel ou lesquels retenez vous en faveur d'une hypertension
artérielle pulmonaire ?
A - Débord droit
B - Arc moyen gauche en double bosse
C - Arc inférieur gauche non allongé
D - Opacités alvéolaires en aile de papillon
E - Artères pulmonaires dilatées
Bonne(s) réponse(s) : A B D E

Ces signes traduisent de manière directe ou indirecte une HTAP au cours du RM. Le débord droit correspond
à une dilatation de l'OD, la partie supérieure de la double bosse de l'arc moyen gauche est formée par le tronc
de l'artère pulmonaire dilatée. D traduit un aspect d'oedème pulmonaire, E est évident.

552
Exclusivement sur DOC - DZ : www.doc-dz.com NADJI 85
RESIDANAT EN POCHE TOME II
Cas Clinique en QCM

Quel examen devez-vous pratiquer pour affirmer de façon formelle, le caractère serré du
rétrécissement mitral ?
A - Un carotidogramme
B - Un cathétérisme droit et gauche
C - Une échocardiographie doppler
D - Une épreuve d'effort
E - Une coronarographie
Bonne(s) réponse(s) : C

L'échocardiographie doppler permet actuellement de quantifier de manière extrêmement fiable le gradient de


pression diastolique entre l'OG et le VG, et la surface fonctionnelle de l'orifice mitral. Cet examen permettra le
plus souvent d'éviter le cathétérisme chez cette patiente jeune.

Quelle(s) autre(s) complication(s) fréquente(s) aurait(ent) pu survenir chez cette patiente ?


A - Hémoptysie
B - Surinfection bronchique
C - Fibrillation ventriculaire
D - Osler
E - Insuffisance ventriculaire gauche
Bonne(s) réponse(s) : A B

Les hémoptysies au cours du RM peuvent survenir à l'occasion d'un infarctus pulmonaire par embolie, à
l'occasion d'un OAP ou de surinfections bronchiques, qui sont fréquentes. Elle témoigne d'une hypertension
veino-capillaire pulmonaire. Par contre, la greffe oslérienne est très rare. Les troubles du rythme sont
auriculaires uniquement et on ne peut parler d'insuffisance ventriculaire gauche.

Après retour spontané en rythme sinusal, le caractère serré du rétrécissement mitral étant affirmé,
une intervention chirurgicale sera pratiquée dans 3 semaines. Quel traitement proposez vous en
attendant cette intervention ?
A - Anticoagulants
B - Nifédipine (Adalate)
C - Anti-agrégants plaquettaires
D - Dérivés quinidiniques
E - Mexilétine (Mexitil®)
Bonne(s) réponse(s) : A D

Compte-tenu du risque potentiel d'un accident embolique lors d'une récidive de la FA, l'indication des
anticoagulants est formelle. Les quinidiniques sont prescrits pour prévenir les récidives de fibrillation
auriculaire.

Mr M. est une homme de 67 ans, un peu obèse et éthylique, qui n'a aucun antécédent pathologique notable,
si ce n'est la constatation ancienne d'un souffle considéré comme banal. Six mois avant son hospitalisation, il
fait un épisode fébrile avec toux, gênes thoraciques, courbatures et douleurs de la gorge qui conduisent à la
prescription d'une antibiothérapie pendant une semaine, le temps que ce tableau s'amende. Les semaines
suivantes, apparaît un discret essoufflement à l'effort et le médecin traitant constate une modification de
l'auscultation : apparition d'un souffle diastolique le long du bord gauche du sternum. Un diurétique est
prescrit. Une nuit, en se levant pour aller uriner, Mr M. perd connaissance pendant quelques secondes, ce qui
motive votre consultation. Vous constatez que la dyspnée reste modérée, s'aggravant peut-être un peu au
cours du temps, il existe en effet un souffle diastolique intense le long du bord gauche et l'ECG objective les
signes d'une hypertrophie ventriculaire gauche.

Quel examen demandez-vous en première intention ?


A - Hémogramme
B - Echocardiographie
C - Angiographie aortique
D - Coronarographie
E - Scannographie thoracique
Bonne(s) réponse(s) : B

Devant l'apparition de ce souffle diastolique à la suite d'un épisode fébrile, l'échocardiographie s'impose d'une
part pour confirmer l'insuffisance aortique, et pour rechercher d'éventuelles végétations.

553
Exclusivement sur DOC - DZ : www.doc-dz.com NADJI 85
RESIDANAT EN POCHE TOME II
Cas Clinique en QCM

En faveur du diagnostic évoqué sur l'ECG, vous retenez :


A - Sokolow à 50 mm
B - Ondes Q fines et profondes en précordiales gauches
C - Sus-décalage de ST en précordiales gauches
D - Ondes T négatives et asymétriques en précordi
E - Elargissement de QRS à 120 msec
Bonne(s) réponse(s) : A B D

Les ondes T négatives et asymétriques en précordiales gauches sont en faveur d'une hypertrophie
ventriculaire gauche plutôt de "type systolique" mais peuvent se rencontrer dans l'IA. Par ailleurs, l'énoncé ne
précise pas HVG "diastolique", la réponse D paraît donc acceptable.

Quelle étiologie vous parait la plus plausible pour expliquer la syncope de ce patient ?
A - Bloc auriculo-ventriculaire paroxystique
B - Insuffisance vertébro-basilaire
C - Torsades de pointe
D - Vaso-vagale
E - Passage en fibrillation auriculaire paroxystique
Bonne(s) réponse(s) : D

Les syncopes mictionnelles nocturnes sont en général d'origine vaso-vagale. Aucun élément de l'énoncé
évoque A B C ou E.

Le bilan effectué confirme une fuite aortique importante avec un ventricule gauche à peine dilaté.
Quelle cause vous parait la plus probable comme étant à l'origine de cette insuffisance aortique ?
A - Post-rhymatismale
B - Maladie annulo-ectasiante
C - Endocardite bactérienne
D - Dissection localisée de l'aorte
E - Maladie de Monckeberg
Bonne(s) réponse(s) : C

Le contexte infectieux et l'apparition brutale du souffle plaide en faveur d'une endocardite. La maladie de
Monckeberg est une des éthologies du RA calcifié dégénératif.

Quatre semaines après votre première consultation, le patient revient et se plaint cette fois d'une
dyspnée plus importante que précédemment, la radiographie thoracique objective une discrète
augmentation du volume cardiaque par rapport au précédent contrôle. Que lui proposez-vous ?
A - Un remplacement vavulaire aortique
B - Une valvuloplastie
C - Un renforcement du traitement diurétique
D - L'adjonction au traitement d'un vasodilatateur
E - Un remplacement de l'aorte ascendante avec remplacement de la valve aortique
Bonne(s) réponse(s) : A

Devant l'aggravation de la symptomatologie, le remplacement valvulaire aortique s'impose. E (opération de


Bentall) est le traitement chirurgical d'une IA par maladie annulo-ectasiante.

Un homme de 39 ans présente une douleur brutale des membres inférieurs.


L'examen met en évidence
- Un aspect livide associé à un refroidissement des deux membres inférieurs
- L'absence des deux pouls fémoraux
Il existe une hypertension artérielle et l'auscultation trouve un souffle diastolique au foyer aortique. Le cliché
thoracique met en évidence un élargissement de la silhouette cardio-aortique ; l'électrocardiogramme est
normal.

Quel élément permet d'établir le diagnostic d'ischémie aiguë sévère ?


A - Aggravation brutale
B - Existence d'une cardiopathie
C - Douleur permanente
D - Déficit sensitivo-moteur
E - Refroidissement
Bonne(s) réponse(s) : D

L'existence d'un déficit sensitivo-moteur au cours d'une ischémie aiguë est un signe de gravité.

554
Exclusivement sur DOC - DZ : www.doc-dz.com NADJI 85
RESIDANAT EN POCHE TOME II
Cas Clinique en QCM

Devant une ischémie aiguë, vous pouvez évoquer une dissection aortique devant :
A - Souffle diastolique au foyer aortique
B - Douleurs précordiales
C - Oedème bilatéral des membres inférieurs
D - Elargissement du médiastin
E - Abolition des réflexes ostéo-tendineux
Bonne(s) réponse(s) : A B D

Un autre signe est une asymétrie des pouls ou de la pression artérielle.

Dans une dissection aortique, le chenal de dissection siège :


A - Dans l'intima
B - Dans la média
C - Entre intima et média
D - Entre média et adventice
E - Indifféremment dans les trois tuniques
Bonne(s) réponse(s) : B

Le clivage se produit au niveau du tiers externe de la media.

La dissection débute au niveau de l'aorte ascendante. Les risques possibles de l'évolution


spontanée comportent :
A - Hémiplégie
B - Oligo-anurie
C - Embolie pulmonaire
D - Paraplégie
E - Tamponnade par hémopéricarde
Bonne(s) réponse(s) : A B D E

Ces complications résultent de la progression de la dissection respectivement vers un des troncs supra-
aortique (A), vers les artères rénales (B) et l'artère d'Adamkiewicz (D). Enfin, la progression rétrograde de la
dissection peut être responsable d'une fissuration dans le sac péricardique (E).

Dès lors il convient de proposer très rapidement.


A - Traitement héparinique
B - Intervention chirurgicale
C - Traitement hypotenseur
D - Epuration extra-rénale
E - Traitement fibrinolytique
Bonne(s) réponse(s) : B C

Le traitement anticoagulant est contre-indiqué. L'indication chirurgicale est formelle dans cette dissection de
type I de De Bakey. Le traitement hypotenseur permet d'éviter les à-coups tensionnels néfastes.

Lors de l'incorporation militaire, il est décelé, à l'auscultation de Monsieur X..., un discret souffle proto-
diastolique le long du bord gauche du sternum, de faible intensité, correspondant, comme le confirme
l'échocardiogramme, à une insuffisance aortique discrète mais certaine. Ce sujet, cultivateur robuste, habitué
aux gros travaux de la ferme n'a pas d'antécédents particuliers, en dehors d'angines à répétition lors de
l'adolescence. Il n'accuse aucun symptôme fonctionnel, même lors de son activité professionnelle et du
cyclisme qu'il pratique régulièrement. En dehors du souffle diastolique, aucune autre anomalie auscultatoire
n'est perçue. La TA est à 130/80 mmHg. Le pouls est régulier à 72, d'amplitude normale. L'examen
stomatologique révèle 2 caries dentaires. L'ECG montre une onde P arrondie positive en D2, D3, aVF et
mesurant 8/100 de seconde. L'axe électrique est à + 300. Le QRS mesure 8/100 de seconde. La
repolarisation est normale. En radioscopie, la silhouette cardio-vasculaire est normale.

Le souffle diastolique d'insuffisance aortique modérée est parfois peu audible. Certaines
manoeuvres ou artifices vous permettent le mieux de le percevoir. Une seule proposition est
inexacte, laquelle ?
A - Le sujet étant debout, légèrement penché en avant
B - Les bras relevés
C - En expiration forcée
D - En décubitus latéral gauche
E - Dans un silence absolu
Bonne(s) réponse(s) : D

Le souffle d'IA est mieux perçu dans la position penchée en avant, bras relevé, en expiration forcée.

555
Exclusivement sur DOC - DZ : www.doc-dz.com NADJI 85
RESIDANAT EN POCHE TOME II
Cas Clinique en QCM

Tous les signes cliniques ou paracliniques suivants vous font penser que la régurgitation est de
faible importance, sauf un, lequel ?
A - La TA à 130/80 mmHg
B - Le pouls régulier à 72 d'amplitude normale
C - L'aspect radiologique normal
D - Le diamètre télédiastolique à 45 mm sur l'échocardiogramme
E - L'axe électrique à + 30°
Bonne(s) réponse(s) : E

Tous les autres critères sont en faveur d'une fuite aortique de faible importance. Le diamètre échographique
télédiastolique normal du VG est compris entre 40 et 55 mm environ chez l'adulte. E n'apporte en fait aucune
information sur l'importance de la fuite mais ne signifie pas que la fuite est importante.

Les caractères de l'onde P correspondent à :


A - Une hypertrophie auriculaire gauche
B - Une onde P normale
C - Une hypertrophie auriculaire droite
D - Une onde P ectopique
E - Une hypertrophie bi-auriculaire
Bonne(s) réponse(s) : B

Sans commentaire.

Quelle étiologie de l'insuffisance aortique vous paraît la plus probable chez ce patient ?
A - Dissection aortique
B - Endocardite bactérienne
C - R.A.A.
D - Congénitale
E - Syphilis
Bonne(s) réponse(s) : C

C'est l'éthiologie la plus probable compte tenu des antécédents d'angines à répétition lors de l'adolescence.

Quelle conduite à tenir adoptez-vous ?


A - Bilan hémodynamique
B - Remplacement valvulaire
C - Surveillance régulière clinique
D - Traitement tonicardiaque et diurétique
E - Interdiction de toute activité sportive
Bonne(s) réponse(s) : C

Nous sommes en présence d'une fuite aortique paraissant "minime à modérée" chez un sujet jeune
asymptomatique. Il convient donc de le surveiller régulièrement cliniquement et échographiquement.

Quel est le signe échocardiographique TM témoignant le mieux de la présence d'une insuffisance


aortique ?
A - Un fluttering diastolique de la grande valve mitrale
B - Un recul télésystolique de la petite valve mitrale
C - Un remaniement avec épaississement des valves sigmoïdes aortiques
D - Des vibrations systoliques de l'appareil valvulaire mitral
E - Des calcifications aortiques
Bonne(s) réponse(s) : A

Le fluttering diastolique de la grande valve mitrale correspond à des vibrations sous l'effet de la régurgitation
aortique.

556
Exclusivement sur DOC - DZ : www.doc-dz.com NADJI 85
RESIDANAT EN POCHE TOME II
Cas Clinique en QCM

Quel conseil donnez-vous pour les 2 caries dentaires observées ?


A - L'abstention et une surveillance régulière
B - De simples avulsions sous antalgiques
C - Des avulsions dentaires précédées de 8 jours d'antibiothérapie par Amoxicilline
D - Des avulsions dentaires précédées et suivies de la prise orale d'Oracilline®
E - Des avulsions dentaires suivies d'une antibiothérapie par pénicilline 20m/j + Streptomycine
1 g/j pendant 3 semaines
Bonne(s) réponse(s) : D

Le schéma actuel d'une prophylaxie antibiotique au cours de soins dentaires chez un patient valvulaire
comporte la prise orale de 2g d'amoxicilline ( ou éventuellement de pénicilline type Oracilline) 1 heure avant
les soins et 8 heures après.

Une patiente de 45 ans, mère de 3 enfants, consulte pour un problème de varices des membres inférieurs.
L'interrogatoire apprend qu'elle a vu apparaître ses varices bilatérales après la seconde grossesse et qu'elle
aurait présenté après le troisième accouchement une phlébite du membre inférieur droit traitée par l'héparine
pendant 3 semaines. Elle n'a jamais soigné ses varices qui entraînent une symptomatologie fonctionnelle
assez typique. L'examen clinique retrouve des varices bilatérales accompagnées d'un ulcère malléolaire droit.
Cet ulcère en place depuis plusieurs années est assurément variqueux. Du côté gauche, un garrot serré
modérément à la racine de la cuisse, sur la patiente couchée, entraîne en position debout la disparition de la
totalité des varices.

Tous ces signes sont en faveur de l'origine variqueuse de l'ulcère, sauf un, lequel ?
A - Douloureux
B - Atone
C - Malléolaire externe
D - Malléolaire interne
E - A bords réguliers
Bonne(s) réponse(s) : C

Les ulcères malléolaires externes orientent plus vers une origine artérielle.

Après l'épreuve du garrot réalisée du coté gauche, vous pouvez dire qu'il s'agit d'une incontinence
valvulaire. De quelle incontinence valvulaire s'agit-il ?
A - De la crosse de la saphène interne
B - De la crosse de la saphène externe
C - Des deux crosses
D - Des perforantes
E - A la fois des crosses et des perforantes
Bonne(s) réponse(s) : A

Le fait que les varices ne réapparaissent pas lors de la station debout avec un garrot signifie qu'elles sont en
rapport avec une incontinence de la crosse de la saphène interne.

Du côté droit. il est indispensable de connaître l'état de la voie veineuse profonde. Quel(s)
manoeuvre(s) ou examen(s) paraclinique(s) permet(tent) d'y répondre ?
A - La marche sous garrot modérément serré à la racine de la cuisse
B - La manoeuvre de Trendelenburg
C - La phlébographie
D - Le doppler veineux
E - Le port d'une bande à contention élastique
Bonne(s) réponse(s) : A C D

La manoeuvre de Perthes-Delbet (A) permet de s'assurer de la perméabilité du réseau profond.

Le traitement des varices des membres inférieurs fait appel à :


A - La chirurgie (éveinage)
B - La sclérothérapie
C - Les vasoconstricteurs
D - Les cures thermales
E - La contention élastique
Bonne(s) réponse(s) : A B C D E

Les phlébotoniques sont des médicaments veinoconstricteurs qui peuvent jouer un rôle de contention
mécanique discrète (notamment les dérivés de l'ergot de seigle). Ces médicaments ont seulement une action
sur les symptômes fonctionnels. L'indication des cures réside surtout dans les troubles trophiques post-
phlébitique.

557
Exclusivement sur DOC - DZ : www.doc-dz.com NADJI 85
RESIDANAT EN POCHE TOME II
Cas Clinique en QCM
Madame X..., âgée de 78 ans, et ayant dans ses antécédents un infarctus du myocarde, a présenté à
plusieurs reprises de brefs étourdissements qui ont été rapportés à une insuffisance circulatoire débutante
alors qu'elle paraît en bon état physique et général. Une syncope récente, survenue brutalement alors qu'elle
lisait son journal, et à l'origine d'un traumatisme frontal lors de la chute qui s'ensuivit, a effrayé l'entourage qui
a constaté l'extrême pâleur lors de la perte de conscience. Celle-ci a été de courte durée (quelques secondes)
avec reprise immédiate de la conscience et rougeur du visage. Le pouls était ralenti à 35 au décours de la
syncope. Lorsque le médecin arrive 30 minutes plus tard, le rythme cardiaque est régulier à 70, un discret
souffle systolique de pointe. L'auscultation carotidienne est normale. La TA est à 160/90 en position couchée
et à 150/100 en position debout. L'ECG objective un rythme de commande sinusale avec un espace PR à
28/100 de seconde, un bloc de branche droit complet et un hémibloc antérieur gauche.

Quelle est la cause la plus probable de la syncope chez cette patiente ?


A - Une hypotension artérielle orthostatique
B - Un malaise vagal
C - Une insuffisance circulatoire cérébrale
D;- Un BAV paroxystique
E - Une sténose aortique
Bonne(s) réponse(s) : D

Le caractère à l'emporte-pièce de la syncope, sa durée, la constatation d'un pouls à 35/min au décours de la


syncope et l'existence de trouble de conduction sur l'ECG basal plaide fortement en faveur d'un BAV
paroxystique.

En faveur de ce diagnostic, vous retenez :


A - Syncope brutale à l'emporte pièce
B - Bradycardie à 35 au décours de la syncope
C - Chute de la PA systolique de 160 à 150 mmHg de la position debout à la position couchée
D - La chute avec traumatisme frontal
E - L'aspect électrocardiographique
Bonne(s) réponse(s) : A B D E

Cf commentaire précédent.
Commentaire de la question précédente :
Le caractère à l'emporte-pièce de la syncope, sa durée, la constatation d'un pouls à 35/min au décours de la
syncope et l'existence de trouble de conduction sur l'ECG basal plaide fortement en faveur d'un BAV
paroxystique.

L'hémibloc antérieur gauche se traduit sur l'ECG par :


A - Un axe à 0°
B - Une hyperdéviation axiale gauche au-delà de -30°
C - Un axe intermédiaire
D - Un axe vertical
E - Une hyperdéviation axiale droite
Bonne(s) réponse(s) : B

L'hémibloc antérieur gauche se traduit par un aspect qR en Dl et rS en D3 avec une déviation axiale gauche
de l'axe de QRS au-delà de -30°.

Le bloc de branche droit complet se traduit en V1 par :


A - Un complexe QR élargi à 12/100 de seconde
B - Un complexe qS élargi à 12/100 de seconde
C - Un complexe rS élargi à 12/100 de seconde
D - Un complexe rSr' à 10/100 de seconde
E - Un complexe qR à 8/100 de seconde
Bonne(s) réponse(s) : A

Le bloc de branche droit peut prendre un aspect rsR' ou rSR'en V1 avec un allongement de la durée de QRS
à 12/100 secondes. Lorsque l'onde r initiale est peu visible, l'aspect rsR' peut ressembler à un aspect qR. D
correspond à un aspect de bloc de branche incomplet droit.

Quelle est votre attitude thérapeutique dans le cas présent ?


A - Abstention thérapeutique et surveillance
B - Mise en place d'un pacemaker
C - Remplacement valvulaire aortique
D - Isuprel per os
E - Atropine sous cutanée
Bonne(s) réponse(s) : B

La mise en place d'un pace-maker est impérative, devant ce BAV paroxystique syncopal.
558
Exclusivement sur DOC - DZ : www.doc-dz.com NADJI 85
RESIDANAT EN POCHE TOME II
Cas Clinique en QCM
Une femme de 30 ans, est admise à l'hôpital pour une hémiparésie droite rapidement régressive.
L'interrogatoire retrouve un essoufflement modéré à l'effort depuis un an et des accès de palpitations, de
durée habituellement brève, sauf le dernier qui a duré 3 heures, à début assez brusque. Le rythme cardiaque
serait rapide et irrégulier.
L'examen neurologique est normal. Le rythme cardiaque est régulier à 70/min. L'auscultation entend à la
pointe un éclat de B1, pas de souffle systolique, un claquement proto-diastolique suivi d'un roulement
diastolique. Le deuxième bruit est claqué au foyer pulmonaire. La radiographie de face montre un coeur peu
augmenté de volume, avec un débord droit, un double contour, une crosse aortique normale, une saillie de
l'arc moyen gauche, un arc inférieur gauche normal, des artères pulmonaires dilatées.
L'électrocardiogramme montre un rythme sinusal, une hypertrophie auriculaire gauche, un axe de QRS à +
100°, un bloc de branche incomplet droit.
Le bilan biologique est normal.

Quelle cardiopathie vous suggèrent les données cliniques, radiologiques et


électrocardiographiques ?
A - Prolapsus valvulaire mitral
B - Insuffisance mitrale
C - Communication inter-auriculaire
D - Rétrécissement mitral
E - Myxome de l'oreillette gauche
Bonne(s) réponse(s) : D

L'AIT chez cette femme jeune associé à une notion d'essoufflement récent et de palpitations irrégulières
évoquait déjà un RM. L'auscultation confirme ce diagnostic, de même que l'aspect électrique et radiologique.

Quel(s) est(sont) le(s) examen(s) paraclinique(s) non invasif(s) à l'exception de la radiographie


pulmonaire standard et de l'électrocardiogramme, immédiatement utile(s) pour confirmer la
cardiopathie ?
A - Enregistrement continu de l'électrocardiogramme sur 24 heures
B - Carotidogramme
C - Doppler cervical
D - Echocardiogramme
E - Scintigraphie myocardique au Thallium
Bonne(s) réponse(s) : D

Seule dans ces examens l'échocardiographie est utile pour confirmer l'existence du RM.

Quel trouble rythmique paroxystique, en fonction des données de l'interrogatoire, peut être
incriminé à l'origine de l'accident neurologique déficitaire transitoire ?
A - Extrasystoles auriculaires
B - Tachycardie ventriculaire
C - Extrasystoles ventriculaires
D - Fibrillation auriculaire
E - Tachycardie jonctionnelle paroxystique
Bonne(s) réponse(s) : D

Le RM se complique volontiers de fibrillation auriculaire, en raison de la dilatation de l'oreillette gauche.


L'aspect paroxystique et irrégulier des palpitations est en faveur de ce diagnostic .

Quelle complication induite par le trouble du rythme paroxystique est directement responsable de
l'accident neurologique déficitaire régressif ?
A - Thrombose de l'oreillette gauche
B - Embolie dans l'artère vertébrale droite
C - Spasme de l'artère sylvienne gauche
D - Chute du débit cardiaque
E - Embolie cruorique sylvienne gauche
Bonne(s) réponse(s) : E

Il s'agit très vraisemblablement d'un embol cruorique à partir d'une thrombose de l'oreillette gauche.

559
Exclusivement sur DOC - DZ : www.doc-dz.com NADJI 85
RESIDANAT EN POCHE TOME II
Cas Clinique en QCM

Une intervention chirurgicale est décidée et sera envisagée dans 4 semaines ; quel(s)
médicament(s) visant à empêcher la récidive de l'accident neurologique allez-vous
préférentiellement prescrire pendant ces 4 semaines ?
A - Anticoagulants
B - Diurétiques
C - Dérivés quinidiniques
D - Dérivés nitrés à action prolongée
E - Bêta-bloquant
Bonne(s) réponse(s) : A C

Le traitement anticoagulant s'impose ainsi que le traitement quinidinique pour prévenir les récidives de
fibrillation auriculaire.

Un homme de 50 ans consulte pour une dyspnée d'effort modérée (deux étages) apparue depuis quelques
mois. L'interrogatoire retrouve la notion d'une scarlatine pendant l'adolescence. L'examen clinique montre une
PA à 160/60 mmHg ; l'auscultation entend en bord gauche de sternum un souffle holodiastolique 4/6, un
souffle mésosystolique 2/6 et à l'apex un roulement mésodiastolique 2/6 ; les bruits sont d'intensité normale à
tous les foyers. L'auscultation pulmonaire est normale. Les pouls périphériques sont normalement perçus et
symétriques. L'ECG montre un rythme sinusal, un espace PR à 0,18 sec, un QRS à 0,10 sec, des ondes Q
fines et profondes en V5, V6, un indice de Sokolow à 50 mm, un discret sous-décalage de ST avec des ondes
T amples et positives dans les dérivations précordiales gauches. La radiographie thoracique mesure un index
cardio-thoracique à 0,60. L'échocardiographie mesure un diamètre télésystolique VG à 60 mm et un diamètre
télédiastolique à 75 mm ; la fraction d'éjection VG est estimée à 45 %.

Quel est le diagnostic ?


A - Insuffisance aortique pure ou prédominante
B - Insuffisance mitrale pure ou prédominante
C - Insuffisance aortique + rétrécissement mitral
D - Maladie aortique
E - Maladie mitrale
Bonne(s) réponse(s) : A

En effet, le souffle mésosystolique au bord gauche a les caractères d'un souffle éjectionnel
d'accompagnement (par hyperdébit) et le roulement mésodiastolique est un roulement de Flint.

Quelle est l'étiologie probable de cette valvulopathie ?


A - Congénitale
B - Post-oslérienne
C - Rhumatismale
D - HTA
E - Ischémique
Bonne(s) réponse(s) : C

La notion d'une scarlatine au cours de l'enfance évoque une origine rhumatismale.

Quel(s) élément(s) témoigne(nt) de l'importance de l'atteinte valvulaire ?


A - Intensité du souffle systolique
B - Intensité du souffle diastolique
C - Roulement mésodiastolique
D - PA à 160/60 mmHg
E - Dyspnée d'effort
Bonne(s) réponse(s) : A B C D E

Tous ces signes témoignent de l'importance de la fuite aortique ; l'intensité du souffle diastolique est moins
bien correlée au volume de la fuite que la durée du souffle dans la diastole. Néanmoins, un souffle diastolique
4/6 témoigne en règle d'une fuite plus importante qu'un souffle 1 à 2/6. Le souffle systolique éjectionnel
d'accompagnement est en général d'intensité modéré mais peut devenir très intense lorsque l'IA est
volumineuse.

560
Exclusivement sur DOC - DZ : www.doc-dz.com NADJI 85
RESIDANAT EN POCHE TOME II
Cas Clinique en QCM

Comment interprétez-vous l'ECG ?


A - Séquelle d'infarctus latéral
B - Bloc de branche gauche incomplet
C - HVG avec surcharge systolique
D - HVG avec surcharge diastolique
E - Ischémie sous-endocardique latérale
Bonne(s) réponse(s) : D

L'HVG de type surcharge diastolique est caractérisée par des ondes T amples et positives dans les
dérivations précordiales gauches, alors que l'HVG de type surcharge systolique se caractérise par des ondes
T négatives et asymétriques.

Comment interprétez-vous les données de l'échocardiographie ?


A - Dilatation VG + fraction d'éjection élevée
B - Dilatation VC + fraction d'éjection abaissée
C - Dilatation VG + fraction d'éjection normale
D - Volumes VG normaux + fraction d'éjection abaissée
E - Fonction VG normale
Bonne(s) réponse(s) : B

Pour mémoire, voici les paramètres échographiques normaux : diamètre télédiastolique VG : 38 à


56 mm ; diamètre TS du VG : 22 à 40 mm ; pourcentage de raccourcissement : 33 % ; fraction d'éjec-
tion : 60 à 70 %.

Les complications évolutives à redouter chez ce patient comportent :


A - Fibrillation auriculaire
B - Endocardite infectieuse
C - Embolies sytémiques
D - Dissection aortique
E - Insuffisance cardiaque gauche
Bonne(s) réponse(s) : B E

L'IA est la valvulopathie la plus exposée à la greffe bactérienne. La dissection peut se voir dans les IA
survenant au cours d'anévrysmes dystrophiques mais pas dans les étiologies rhumatismales.

Quelle attitude thérapeutique proposez-vous ?


A - Aucun traitement, mais surveillance régulière
B - Traitement digitalo-diurétique
C - Remplacement valvulaire mitral
D - Remplacement valvulaire aortique
E - Remplacements valvulaires mitral et aortique
Bonne(s) réponse(s) : D

Il faut réaliser un remplacement valvulaire aortique sans tarder ; ce patient aura néanmoins un pronostic post-
opératoire moins bon que s'il avait été opéré plus tôt dans la mesure où il existe déjà une dilatation
ventriculaire importante et une baisse de la fonction du VG, dont on n'est pas certain de la récupération post-
opératoire.

Une femme de 40 ans, se plaint depuis plusieurs mois de crises de palpitations à début brutal, sans
circonstance déclenchante particulière et qui toutes, jusqu'à présent, ont cédé spontanément et brutalement,
au bout de quelques minutes, immédiatement suivies d'une polyurie. Elle est hospitalisée pour une crise qui
dure depuis une heure et s'accompagne d'un important malaise général avec lipothymie. La PA est de 90/70
mmHg. L'ECG enregistre une tachycardie régulière à 200/mn; les QRS sont fins; les ondes P négatives en D2
et D3 sont vues 200 msec après chaque QRS.

Quelle est la nature du trouble du rythme ?


A - Tachycardie sinusale
B - Tachycardie atriale basse
C - Tachycardie jonctionnelle
D - Flutter auriculaire
E - Tachycardie ventriculaire
Bonne(s) réponse(s) : C

Il s'agit d'une tachycardie jonctionnelle dite tachycardie de Bouveret.

561
Exclusivement sur DOC - DZ : www.doc-dz.com NADJI 85
RESIDANAT EN POCHE TOME II
Cas Clinique en QCM

Parmi ces 5 signes cliniques, lequel permettait déjà d'évoquer le diagnostic avant l'ECG :
A - Début brutal
B - Lipothymie
C - PA à 90/70 mmHg
D - Arrêt brutal
E - Polyurie post-critique
Bonne(s) réponse(s) : E

On sait aujourd'hui que la classique crise polyurique souvent décrite à l'arrêt de ce type de tachycardie est en
rapport avec une libération de facteur atrial natriurétique. Néanmoins, la réponse à cette question est difficile
car d'autres types de tachycardies peuvent entraîner une polyurie et d'autre part le caractère extrêmement
brutal du début et de la fin de la crise est également assez évocateur.

Quel(s) peut(vent) être l'(les) effet(s) des manoeuvres vagales chez cette patiente ?
A - Aucun effet
B - Ralentissement de la tachycardie
C - Accélération de la tachycardie
D - Dissociation auriculo-ventriculaire
E - Interruption de la tachycardie
Bonne(s) réponse(s) : A E

Les manoeuvres vagales peuvent parfois être sans effet lorsque le tonus adrénergique est très élevé. On peut
obtenir un arrêt de la tachycardie, parfois précédé d'un ralentissement. Néanmoins la réponse B n'est pas à
cocher.

Parmi ces 5 drogues injectables par voie IV, quelles sont les 2 qui ont le plus de chance d'obtenir
la réduction de l'arythmie ?
A - Cédilanide®
B - Propranolol (Avlocardyl®)
C - Striadyne®
D - Vérapamil (Isoptine®)
E - Lidocaïne (Xylocaïne®)
Bonne(s) réponse(s) : C D

L'injection rapide de Striadyne (adénosine triphosphate) entraîne une dépression de la conduction du noeud
auriculo-ventriculaire en raison d'un puissant effet vagomimétique. L'injection intra-veineuse lente de
Vérapamil (Isoptine) est également souvent efficace par son effet freinateur sur la conduction nodale. La
Cédilanide peut être efficace mais de manière plus retardée.

Chez cette femme jeune sans cardiopathie organique détectable, quel est le principal risque
évolutif ?
A - Collapsus cardiogénique
B - Infarctus du myocarde
C - Embolies systémiques
D - Mort subite
E - Récidives
Bonne(s) réponse(s) : E

Le principal risque évolutif de ces tachycardies fonctionnelles paroxystiques est leur récidive. Elles ne donnent
pas d'embolies systémiques.

562
Exclusivement sur DOC - DZ : www.doc-dz.com NADJI 85
RESIDANAT EN POCHE TOME II
Cas Clinique en QCM
Un homme de 59 ans est traité depuis 6 mois pour une hypertension artérielle avec angor par l'association
suivante :

- Béta-bloquant : métoprolol (Lopressor® 200 : 2 cp/j)


- Inhibiteur calcique : nifédipine (Adalate® : 6 gel/j )
Un matin au lever, il fait un malaise et tombe. Vous êtes appelé et mettez en
évidence une pression artérielle à 13/6 cm Hg en position couchée et 8/5 cm Hg
en position debout.

Indiquer parmi les propositions suivantes celle(s) qui est (sont) exacte(s) :
A - Le malaise observé est probablement lié à une posologie un peu trop importante
B - L'hypotension orthostatique est indépendante de la prise d'inhibiteur calcique
C - Il suffit ici de réduire la posologie administrée et de conseiller la prudence lors du lever
D - Il est plus logique d'arrêter immédiatement tout traitement antihypertenseur pendant
quelques jours
E - La pression artérielle peut être remontée temporairement par une perfusion de métaraminol
Bonne(s) réponse(s) : A C

Il s'agit d'un malaise par hypotension artérielle orthostatique, en rapport avec une posologie un peu forte
d'hypotenseurs.

Parmi les propositions suivantes concernant l'association métoprolol et nifédipine, indiquer


celle(s) qui est (sont) exacte(s) :
A - Ces médicaments ont tous les deux des propriétés chronotropes négatives
B - Ces médicaments ont tous les deux des propriétés vasodilatatrices
C - Ces médicaments facilitent tous les deux la conduction dans le tissu nodal
D - Ces médicaments sont tous les deux bronchoconstricteurs
E - La tachycardie posturale réflexe ne se voit pas avec cette association
Bonne(s) réponse(s) : E

Seul le métoprolol est chronotrope négatif et favorise la bronchoconstriction. Seule la nifédipine est
vasodilatatrice. Aucun des 2 ne facilite la conduction nodale.

Parmi les propositions suivantes concernant divers antihypertenseurs, indiquer celle(s) qui
est(sont) exacte(s) :
A - Les inhibiteurs calciques sont contre-indiqués en cas d'ulcère gastroduodénal
B - Les inhibiteurs calciques sont contre-indiqués en cas d'adénome prostatique ou de
phénomènes urinaires rétentionnels
C - L'association bêta-bloquant - vérapamil (Isoptine®) a une action inotrope négative marquée
D - L'adjonction d'un bêta-bloquant à un inhibiteur calcique ralentit en général la fréquence
cardiaque
E - Il est fréquent de devoir interrompre un traitement par nifédipine en raison de la survenue de
phénomènes de vasodilatation périphérique
Bonne(s) réponse(s) : C D E

L'association bêta-bloquant Vérapamil a une action inotrope négative, chacun des deux médicaments étant lui-
même inotrope négatif. La nifédipine est souvent responsable de bouffées vasomotrices ou d'oedèmes des
membres inférieurs, surtout en début de traitement.

On décide de modifier le traitement antihypertenseur de ce malade pour utiliser un inhibiteur de


l'enzyme de conversion (captopril) ; indiquer parmi les propositions suivantes concernant ce type
de médicament celle(s) qui est (sont) exacte(s)
A - Le captopril entraîne un hyperaldostéronisme
B - Le captopril tend à entraîner une hyperkaliémie
C - Si un diurétique est associé au captopril, il est préférable d'utiliser un produit hypokaliémiant
D - Un traitement par captopril ne nécessite pas d'ajustement posologique en cas d'insuffisance
rénale
E - L'administration de captopril en cas d'HTA liée à une sténose des deux artères rénales ne
majore pas le risque d'insuffisance rénale
Bonne(s) réponse(s) : B C

En bloquant la synthèse d'angiotensine II, le captopril tend à diminuer l'aldostérone. Par cet effet, il tend plutôt
à augmenter la kaliémie. C'est la raison pour laquelle il est préférable de l'associer avec un diurétique
hypokaliémiant. Il faut diminuer la posologie en cas d'insuffisance rénale. Enfin, le captopril est contre-indiqué
en cas de sténose bilatérale des artères rénales, du fait du risque d'insuffisance rénale aiguë, dans ce cas.

563
Exclusivement sur DOC - DZ : www.doc-dz.com NADJI 85
RESIDANAT EN POCHE TOME II
Cas Clinique en QCM

Au lieu d'utiliser un inhibiteur de l'enzyme de conversion, on utilise un autre traitement


antihypertenseur ; indiquer parmi les propositions suivantes concernant ce type de médicaments
celle(s) qui est (sont) exacte(s) :
A - Lors du début d'un traitement par alpha-bloquant, les phénomènes d'hypotension
orthostatique sont fréquents
B - La dihydralazine (Nepressol®), bien qu'ayant une action vasodilatatrice directe, est contre-
indiquée en cas d'angor
C - La dihydralazine peut être à l'origine d'un lupus
D - L'alpha-méthyl-dopa n'est pas connue comme susceptible d'entraîner des anémies
hémolytiques
E - On sait que l'arrêt brutal d'un traitement bêta-bloquant est dangereux mais l'arrêt brutal d'un
traitement par clonidine n'est pas dangereux
Bonne(s) réponse(s) : A B C

A justifie de débuter la posologie de la prasozine très progressivement, en donnant la première prise au


moment du coucher. La Dihydralazine est contre-indiquée dans l'angor du fait de la tachycardie réflexe qu'elle
induit et qui augmente le travail cardiaque. L'Aldomet peut entraîner des anémies hémolytiques auto-
immunes. Enfin, l'arrêt brutal de la clonidine peut entraîner un rebond hypertensif.

Un homme de 50 ans, fumeur, hypertendu traité par propranolol, est hospitalisé pour une fracture du tibia. Il
est immobilisé par plâtre cruro-pédieux depuis 10 jours. Il a brutalement une douleur basithoracique droite,
intense, associée à une polypnée à 30/mn. L'auscultation cardiaque et pulmonaire est normale. La fréquence
cardiaque est à 110/mn, TA : 150/100 mmHg, température à 37°5. Il n'y a pas de signe périphérique
d'insuffisance cardiaque droite.

Quel diagnostic suspectez-vous ?


A - Infarctus du myocarde
B - Pneumopathie aiguë
C - Crise d'asthme
D - Oedème pulmonaire
E - Embolie pulmonaire
Bonne(s) réponse(s) : E

Le contexte (alitement, plâtre) et la symptomatologie sont très en faveur d'une embolie pulmonaire.

Quel(s) examen(s) pratiquez-vous en premier lieu ?


A - Artériographie bronchique
B - Electrocardiogramme
C - Phlébocavographie
D - Gaz du sang artériel
E - Radiographie de thorax
Bonne(s) réponse(s) : B D E

Ce sont les examens à demander immédiatement devant toute suspicion d'embolie pulmonaire.

L'aspect électrocardiographique peut comporter :


A - Tachycardie sinusale
B - Bloc de branche gauche
C - Ondes P amples en D2
D - Aspect S1 Q3
E - Indice de Sokolow supérieur à 35 mm
Bonne(s) réponse(s) : A D

L'aspect S1Q3 traduit une rotation vers la droite de l'axe de QRS. Des ondes P amples en D2 évoque une
hypertrophie auriculaire droite, qui n'a pas le temps d'apparaître de manière aussi aiguë.

L'aspect radiologique peut révéler :


A - Epanchement pleural
B - Absence d'anomalie
C - Opacité triangulaire à large base d'implantation pleurale
D - Ascension d'une coupole
E - Hyperclarté localisée
Bonne(s) réponse(s) : A B C D E

Sans commentaire.

564
Exclusivement sur DOC - DZ : www.doc-dz.com NADJI 85
RESIDANAT EN POCHE TOME II
Cas Clinique en QCM

La gazométrie artérielle montre - PaO2 : 7,4 kPa (56 mmHg) - PaC02 : 4,3 kPa (33 mmHg) pH :
7,45 - RA : 20 mmol/l. La radiographie du thorax met en évidence des atélectasies en bandes de
la base droite.
Devant ces résultats, quel(s) examen(s) complémentaire(s) demandez-vous ?
A - Fibroscopie bronchique
B - Scintigraphie pulmonaire de perfusion
C - Phlébographie bilatérale
D - Cavographie
E - Angiographie pulmonaire
Bonne(s) réponse(s) : B C D E

Seule l'angiographie pulmonaire permettra de confirmer le diagnostic d'embolie pulmonaire. La


phlébocavographie est très importante pour dépister une éventuelle thrombose veineuse profonde et
l'existence de caillots flottants. B peut ici avoir un intérêt pour suivre l'évolution de la maladie.

Votre traitement comporte :


A - Oxygénothérapie nasale
B - Antivitamines K
C - Arrêt du propranolol
D - Corticothérapie
E - Héparine intraveineuse
Bonne(s) réponse(s) : A E

Le traitement comporte dans l'immédiat une oxygénothérapie compte-tenu de l'hypoxémie et


l'héparinothérapie à doses efficaces.

Un homme de 55 ans est suivi depuis plus de dix ans pour une cardiopathie valvulaire parfaitement tolérée
sur le plan fonctionnel. Il consulte pour l'apparition récente d'une dyspnée invalidante (stade III). L'auscultation
entend des bruits d'intensité normale à la base, un souffle holodiastolique 2/6 de timbre doux en bord gauche
de sternum et une sémiologie apexienne associant un B3, un souffle holosystologique 4/6 qui irradie dans
l'aisselle et un bref roulement mésodiastolique à peine audible (1/6). La PA est de 1 30/80 mmHg. La
radiographie thoracique montre un coeur augmenté de volume (I.C.T=0.55), essentiellement par dilatation de
l'oreillette et du ventricule gauches.

Cliquez sur le bouton "Dessin" pour afficher l'ECG.

Quel est le trouble du rythme présent sur l'ECG ?

Cliquez sur le bouton 'Dessin' pour afficher le schéma.


A - Tachycardie sinusale
B - Extrasystoles auriculaires en salves
C - Tachycardie jonctionnelle
D - Flutter auriculaire
E - Fibrillation auriculaire
Bonne(s) réponse(s) : E

Le rythme ventriculaire est irrégulier et il n'y a pas d'activité auriculaire visible : il s'agit donc d'une fibrillation
auriculaire.

Quel est le diagnostic le plus probable quant à la maladie de fond ?

Cliquez sur le bouton 'Dessin' pour afficher le schéma.


A - Rétrécissement aortique + insuffisance mitrale
B - Insuffisance aortique + insuffisance mitrale
C - Maladie aortique + insuffisance mitrale
D - Insuffisance aortique + rétrécissement mitral
E - Insuffisance aortique + maladie mitrale
Bonne(s) réponse(s) : B

Le souffle holodiastolique au bord gauche du sternum correspond à une insuffisance aortique ; le souffle
holosystolique apexien est un souffle d'insuffisance mitrale ; le bref roulement mésodiastolique à l'apex
correspond à un souffle d'accompagnement d'hyperdébit du à l'insuffisance mitrale, il ne s'agit pas d'une
sténose mitrale autonome.

565
Exclusivement sur DOC - DZ : www.doc-dz.com NADJI 85
RESIDANAT EN POCHE TOME II
Cas Clinique en QCM

Quelle est la lésion fonctionnelle dominante dans cette polyvalvulopathie ?

Cliquez sur le bouton 'Dessin' pour afficher le schéma.


A - Insuffisance mitrale
B - Insuffisance aortique
C - Rétrécissement mitral
D - Rétrécissement aortique
E - Maladie mitrale
Bonne(s) réponse(s) : A

En effet, les signes auscultatoires sont en faveur d'une IM importante, non tant en raison de l'intensité du
souffle qu'en raison de l'existence d'un B3 (galop protodiastolique) et d'un roulement diastolique
d'accompagnement.

Les risques évolutifs liés au trouble du rythme comportent :

Cliquez sur le bouton 'Dessin' pour afficher le schéma.


A - Insuffisance cardiaque gauche ou globale
B - Insuffisance cardiaque droite pure
C - Thrombose auriculaire gauche
D - Embolies systémiques
E - Mort subite
Bonne(s) réponse(s) : A C D

Le passage en fibrillation auriculaire peut être à l'origine d'une poussée d'insuffisance cardiaque en raison de
la perte de la systole auriculaire dans le remplissage ventriculaire. La stase sanguine favorise la formation de
thrombus dans l'oreillette gauche, pouvant donner des embolies systémiques.

Quel examen complémentaire vous semble le plus utile chez ce patient dans l'immédiat ?

Cliquez sur le bouton 'Dessin' pour afficher le schéma.


A - Enregistrement Holter sur 24 h
B - Phonomécanocardiogramme
C - Echo-doppler
D - Angioscintigraphie cardiaque
E - Angio-coronarographie
Bonne(s) réponse(s) : C

L'échodoppler permettra de quantifier la fuite aortique et la fuite mitrale, et d'apprécier le retentissement de


ces fuites sur le ventricule gauche (dilatation des cavités et fonction VG) et l'oreillette gauche ; de plus, cet
examen recherchera la présence d'un éventuel thrombus dans la cavité auriculaire gauche.

Quel(s) traitement(s) proposez-vous dans l'immédiat ?

Cliquez sur le bouton 'Dessin' pour afficher le schéma.


A - Anticoagulant
B - Aspirine®
C - Digitalique
D - ATP (Striadyne) IV
E - Choc électrique externe
Bonne(s) réponse(s) : A C

Le traitement anticoagulant est ici systématique sauf contre-indication. Le traitement digitalique permettra de
ralentir le rythme ventriculaire, voire de réduire le trouble du rythme. Un choc électrique externe ne sera
envisagé qu'ultérieurement après une anticoagulation de plusieurs semaines, et après arrêt des digitaliques.

566
Exclusivement sur DOC - DZ : www.doc-dz.com NADJI 85
RESIDANAT EN POCHE TOME II
Cas Clinique en QCM

En cas de retour spontané en rythme sinusal, quelle(s) est (sont) la(les) classe(s) d'anti-
arythmiques dont la prescription pourrait être envisagée chez ce patient ?
Cliquez sur le bouton 'Dessin' pour afficher le schéma.
A - Classe IA (Quinidiniques - Rythmodan®)
B - Classe IB (Mexitil®)
C - Classe (FIexicaïne®, Rythmol®, Cipralan®)
D - Classe II (Bêta-bloquants)
E - Classe IV (inhibiteurs calciques)
Bonne(s) réponse(s) : A C

Les antiarythmiques de classe IA (quinidiniques-Rythmodan) ou de classe IC (Flécaïne, Rythmol, Cipralan)


sont les médicaments les plus efficaces dans la prévention des récidives de fibrillation auriculaire. Les bêta-
bloquants peuvent être utiles lorsqu'il s'agit de FA d'origine catécholergique, ce qui n'est pas le cas ici.

Monsieur X..., arbitre de football, âgé de 44 ans, est en bon état général et n'accuse aucun symptôme
fonctionnel. Il fume 10 cigarettes par jour, pèse 78 kg pour 1,76m. Il n'est ni diabétique, ni dyslipémique. Son
père est décédé à 52 ans d'infarctus du myocarde. La TA est à 140/80 mmHg.Lors d'un examen systématique
de médecine sportive, l'ECG décèle une lésion sous endocardique en V5 V6 suggestive d'une insuffisance
coronarienne. L'épreuve d'effort s'avère très nettement positive. Une coronarographie mettra en évidence des
sténoses proximales serrées des trois principaux troncs coronariens avec bon lit d'aval .

Quelle entité clinique présente ce patient ?


A - Angor de Prinzmétal
B - Ischémie myocardique silencieuse
C - Dystonie neuro-végétative
D - Angor mixte
E - Angor atypique
Bonne(s) réponse(s) : B

Le patient est asymptomatique bien qu'actif (arbitre de football) et présente des signes d'ischémie électrique
au repos et à l'effort, dont l'origine coronarienne est attestée par la coronarographie : il s'agit donc d'une
ischémie silencieuse.

Comment se présente l'anomalie ECG observée chez ce patient ?


A - Sous-décalage du segment ST de 1 mm, horizontal
B - Sus-décalage du segment ST de 2 mm concave vers le bas
C - Aplatissement de l'onde T
D - Sous-décalage obliquement ascendant du segment ST
E - Onde T négative et symétrique
Bonne(s) réponse(s) : A

La lésion sous-endocardique est un sous-décalage de ST horizontal ou descendant. Un sous-décalage


oblique ascendant de ST est non spécifique d'insuffisance coronaire.

L'épreuve d'effort s'avère très nettement positive c'est à dire que :


A - Le sous-décalage du segment ST est supérieur ou égal à 4 mm
B - Le sous-décalage apparaît très rapidement lors des premiers paliers de l'effort
C - Le sous-décalage apparaît dès une fréquence cardiaque de 120/mn
D - La TA au départ à 140/80 mmHg passe à 200/110 mmHa au maximum de l'effort
E - Le sous-décalage du segment ST persiste à la 1Oème minute de la récupération
Bonne(s) réponse(s) : A B C E

Tous ces critères traduisent la sévérité de l'ischémie : apparition précoce au cours de l'effortet pour une faible
accélération de la féquence cardiaque, amplitude du sous-décalage, persistance tardive lors de la
récupération.

Quel(s) traitement(s) médical(aux) pourrait(ent) être proposé(s) à ce patient dans un premier


temps :
A - Des calcium-bloqueurs
B - Des bêta-bloquants
C - Des dérivés nitrés
D - Des fibrinolytiques
E - Des anti-agrégants plaquettaires
Bonne(s) réponse(s) : A B C E

Tous les anti-angineux A B et C peuvent être proposés. L'adjonction d'anti-agrégants peut être utile pour éviter
la formation d'une thrombose coronaire.

567
Exclusivement sur DOC - DZ : www.doc-dz.com NADJI 85
RESIDANAT EN POCHE TOME II
Cas Clinique en QCM

Quelle serait ultérieurement votre attitude thérapeutique ?


A - Poursuite du traitement mis en route
B - Triple pontage aorto-coronarien
C - Dilatation endoluminale
D - Renforcement du traitement en cas d'aggravation
E - Poursuite de l'abstention avec surveillance renforcée
Bonne(s) réponse(s) : B

Il s'agit d'un patient porteur de lésions coronaires proximales tritronculaires avec bon lit d'aval donc a priori
accessibles à un geste de pontage. Dans ce cas, on préfére le pontage à une dilatation endoluminale, car il y
a 3 troncs coronaires à revasculariser.

Une femme de 35 ans est hospitalisée pour bilan d'une HTA permanente à 180/110 mmHg, apparue depuis 6
mois. Aucun antécédent particulier. Depuis quelques mois, cette femme se plaint de palpitations avec parfois
une gêne thoracique, de céphalées, de sueurs abondantes.
- L'examen clinique confirme l'HTA ainsi qu'une tachycardie à 100/minute. Pas d'autre anomalie.
- Le bilan biologique initial est le suivant :
- cholestérol : 6 mmol/l
- urée sanguine : 5 mmol/l
- créatininémie à 90 micromol/l glycémie : 8, 5 mmol/l à jeun
- NA+ 140 et K+ 3,9 mmol/l
absence de protéinurie à la bandelette dans le sang, compte d'Addis et ex. cyto-bactériologique urinaire
normaux

Il existe un résultat anormal pour :


A - Cholestérol
B - Urée sanguine
C - Glycémie
D - Natrémie
E - Kaliémie
Bonne(s) réponse(s) : C

Sans commentaire.

Quelle étiologie doit-on évoquer en priorité pour cette HTA ?


A - HTA "essentielle"
B - Hyperaldostéronisme primaire
C - Glomérulopathie chronique
D - Phéochromocytome
E - HTA réno-vasculaire
Bonne(s) réponse(s) : D

La symptomatologie (surtout la triade : céphalée, sueurs, palpitations ; plus accessoirement la gêne


thoracique) associée à l'HTA, ainsi que l'augmentation de la glycémie sont en faveur d'un phéochromocytome.

Parmi les cinq examens biologiques suivants, citer les deux susceptibles d'affirmer votre
hypothèse exprimée dans la deuxième question :
A - Dosage de l'activité rénine plasmatique
B - Dosage de l'aldostéronémie
C - Dosage des catécholamines/urines 24 heures
D - Dosage de l'aldostéronurie/24 heures
E - Dosage du bloc métanéphrines-normétanéphrines
Bonne(s) réponse(s) : C E

La mesure de l'excrétion urinaire des métanéphrines sur 24 H a une sensibilité proche de 100 %. Le dosage
des catécholamines urinaires libres reste utile.

568
Exclusivement sur DOC - DZ : www.doc-dz.com NADJI 85
RESIDANAT EN POCHE TOME II
Cas Clinique en QCM

Quel est parmi ces cinq médicaments anti-hypertenseurs celui dont la prescription peut être
maintenue pendant la réalisation de l'ensemble de ce bilan biologique (Cf question précédente)
car il ne devrait pas en perturber significativement les résultats ?
A - Diurétique
B - Béta-bloquant
C - Alpha-méthyldopa (Aldomet®)
D - Inhibiteur de l'enzyme de conversion
E - Inhibiteur calcique
Bonne(s) réponse(s) : E

Parmi ces antihypertenseurs, les inhibiteurs calciques sont les moins susceptibles de perturber les résultats
des tests. Parmi les autres médicaments pouvant perturber les tests, citons : les IMAO, les réserpiniques ; la
guanéthidine, la phentolamine.

Quelle(s) est (sont) la (les) précaution(s) diététique(s) à respecter avant et pendant la réalisation
de ce bilan biologique afin qu'il soit interprétable ?
A - Régime désodé strict
B - Régime normosodé
C - Régime hypersodé
D - Exclusion des bananes
E - Exclusion du café et du thé
Bonne(s) réponse(s) : B D E

La vanille est également contre-indiquée.

Une fois ce bilan réalisé et en attendant ses résultats, lequel parmi ces cinq médicaments anti-
hypertenseurs vous semble le mieux approprié compte tenu de votre hypothèse étiologique ?
A - Diurétique thiazidique
B - Diurétique distal
C - Labetolol (Trandate®)
D - Inhibiteur de l'enzyme de conversion
E - Propranolol (Avlocardyl®)
Bonne(s) réponse(s) : C

Le traitement médical en pré-opératoire repose sur l'association alpha et bêta-bloquant (labetolol).

Madame X, 35 ans, est hospitalisée pour une hypertension artérielle découverte récemment à l'occasion d'une
fatigue générale. Elle a eu une grossesse à l'age de 22 ans sans problème. Il y a un an en raison d'une
élévation de la pression artérielle, son gynécologue lui a ordonné d'arrêter la pilule oestro-progestative qu'elle
prenait depuis une dizaine d'années et a posé un stérilet. Sa pression artérielle est à 190/110 mmHg aux deux
bras au repos, en position allongée et en position debout. Le reste de l'examen clinique est normal. En dehors
de la fatigue Madame X ne se plaint de rien. Elle prend un verre de pastis sans alcool par mois depuis des
années. Les résultats des premiers examens montrent :
- Dans le sang : Sodium 140 mmol/l, Potassium 2,8 mmol/l, Chlore 100 mmol/l, Bicarbonates 29 mmol/l,
Créatinine 80 micromol/l
- Dans les urines : Sodium 140 mmol/24 h, Potassium 40 mmol/24 h, pas de protéinurie.

D'emblée chez cette femme on évoque d'abord :


A - Une hypertension d'origine endocrinienne
B - Une hypertension d'origine iatrogène
C - Une hypertension liée à la prise ancienne de la pilule
D - Une hypertension liée à un phéochromocytome
E - Une hypertension essentielle
Bonne(s) réponse(s) : A

Il s'agit d'une HTA sévère associée à une hypokaliémie importante : on doit évoquer le diagnostic de
syndrome de Conn, d'autant que la prise de pastis sans alcool semble occasionnelle.

569
Exclusivement sur DOC - DZ : www.doc-dz.com NADJI 85
RESIDANAT EN POCHE TOME II
Cas Clinique en QCM

Dans cette observation la natriurèse élevée, supérieure à la kaliurèse permet d'éliminer :


A - Une hyperaldostéronisme primaire
B - Une hyperaldostéronisme secondaire
C - Une tumeur à rénine
D - Une sténose de l'artère rénale
E - Aucun des diagnostics précédents
Bonne(s) réponse(s) : E

Dans les Hyperaldostéronisme secondaires (B, C, D), on a classiquement un rapport Na/K urinaire inférieur à
1 en régime normosodé. L'absence de ce signe ne permet cependant pas d'éliminer formellement ces
diagnostics.

Il existe des causes d'erreur du dosage du potassium. Ainsi le taux de potassium plasmatique
peut-être artificiellement augmenté.
A - En cas d'anémie
B - En cas de thrombocytose
C - Si le prélèvement a été fait avec un garrot serré laissé en place plusieurs minutes
D - Si le prélèvement a été fait après que le patient ait serré d'une façon répétée le poing
E - Si le sang prélevé s'hémolyse secondairement
Bonne(s) réponse(s) : B C D E

Au cours de C et D, l'hyperkaliémie est secondaire à l'acidose. Elle est la conséquence de la lyse cellulaire
dans B.

Une hypokaliémie inférieure à 2,5 mmol/l peut être responsable de manifestations comme :
A - Une diarrhée
B - Une asthénie
C - Une faiblesse musculaire permanente
D - Une polyurie
E - Une aréflexie
Bonne(s) réponse(s) : B C D E

Les hypokaliémies sévères peuvent donner des constipations pouvant aller jusqu'à l'iléus paralytique.

Les signes électrocardiographiques d'une hypokaliémie peuvent comporter :


A - Des ondes U anormalement amples
B - Un élargissement de l'intervalle QT
C - Un sus-décalage du segment ST
D - Des ondes T amples, pointues, symétriques
E - Des troubles du rythme ventriculaire
Bonne(s) réponse(s) : A B E

Les signes ECG d'une hypokaliémie sont l'apparition d'une onde U, l'horizontalisation puis la dépression du
segment ST, l'aplatissement voire la négativation de l'onde T, enfin des troubles du rythme supra-
ventriculaires (ESA, FA) et ventriculaires (ESV, TV, torsades de pointe favorisées par l'allongement du
segment QT).

570
Exclusivement sur DOC - DZ : www.doc-dz.com NADJI 85
RESIDANAT EN POCHE TOME II
Cas Clinique en QCM
Monsieur B... âgé de 64 ans, retraité en bon état général, est traité pour une HTA modérée (170-100 mmHg)
par nifédipine (Adalate® LP 20 mg matin et soir). Depuis deux ans, il se plaint de fréquents accès de
tachycardie paroxystique à début brutal, accompagnée d'un malaise général avec asthénie intense, sans
douleurs thoraciques; jusqu'à présent, les crises ont toujours cédé spontanément en quelques dizaines de
minutes : il n'existe pas de polyurie postcritique.
Il est hospitalisé cette fois pour une crise qui évolue depuis 8 heures ; la PA est 150-80 mmHg ; l'examen
clinique ne montre pas de signe d'insuffisance cardiaque ; l'auscultation entend un rythme régulier à 150 b/mn
sans anomalies surajoutées. La radiographie thoracique montre un coeur de volume et de morphologie
normaux. Le bilan biologique est normal. L'ECG suivant est enregistré :

Cliquez sur le bouton "Dessin" pour afficher l'ECG.

Quel est le trouble du rythme cardiaque en cause ?

Cliquez sur le bouton 'Dessin' pour afficher le schéma.


A - Tachycardie sinusale
B - Fibrillation auriculaire
C - Flutter auriculaire
D - Tachycardie jonctionnelle
E - Tachycardie ventriculaire
Bonne(s) réponse(s) : C

Le flutter auriculaire est une tachycardie régulière à 150/mn, à QRS fins. L'activité auriculaire est représentée
par des ondes F à 300/mn.

Parmi ces éléments tirés de l'interrogatoire et de l'examen clinique, quel est le seul qui permet
d'évoquer prioritairement ce diagnostic avant même l'enregistrement ECG ?
Cliquez sur le bouton 'Dessin' pour afficher le schéma.
A - Début brutal
B - Rythme régulier à 150 b/mn
C - Bonne tolérance hémodynamique
D - Absence de douleurs thoraciques
E - Absence de polyurie post-critique
Bonne(s) réponse(s) : B

Une régularité parfaite à une fréquence de 150/mn est très évocatrice de flutter.

Compte-tenu de la nature de l'arythmie et des données cliniques et paracliniques, quelle est


l'étiologie la plus probable ?
Cliquez sur le bouton 'Dessin' pour afficher le schéma.
A - Idiopathique
B - Ischémique
C - Valvulopathie mitrale
D - Myocardiopathie dilatée
E - Métabolique
Bonne(s) réponse(s) : A

Dans 20 % des cas généralement chez les sujets de sexe masculin, le flutter ne s'accompagne pas de
cardiopathie décelable.

Parmi ces 5 propositions thérapeutiques, laquelle vous semble la plus indiquée dans l'immédiat ?

Cliquez sur le bouton 'Dessin' pour afficher le schéma.


A - Manoeuvres vagales
B - Héparinothérapie IV
C - Xylocaïne IV
D - Héparine / Xylocaïne IV
E - Striadyne IV
Bonne(s) réponse(s) : B

En raison de risque d'embolie artérielle lors de la régularisation. La striadyne permettra de ralentir le rythme
ventriculaire, mais sera inefficace pour réduire le flutter.

571
Exclusivement sur DOC - DZ : www.doc-dz.com NADJI 85
RESIDANAT EN POCHE TOME II
Cas Clinique en QCM

Après 24 heures d'observation, l'arythmie persiste. Quelle est l'attitude la mieux appropriée ?

Cliquez sur le bouton 'Dessin' pour afficher le schéma.


A - Se contenter de poursuivre le même traitement
B - Ajuster le traitement médicamenteux
C - Réaliser un choc électrique externe
D - Tenter une réduction par stimulation électrique de l'oreillette
E - Réaliser un choc électrique endocavitaire
Bonne(s) réponse(s) : D

La stimulation électrique de l'oreillette peut se faire par voie endocavitaire (la sonde étant introduite par voie
veineuse) ou par voie transoesophagienne (la sonde étant introduite par voie nasale). Le rythme sinusal est
obtenu dans 60 à 70 % des cas, soit immédiatement, soit après un passage en FA transitoire.
Principe : capturer l'oreillette par la stimulation de façon à interrompre le circuit de la tachycardie. La
stimulation doit se faire à une fréquence légèrement supérieure à celle de la fréquence de l'oreillette en flutter.
Les formes récentes et sans cardiopathies sous-jacentes sont les formes les plus accessibles à la stimulation.

Monsieur Jacques V..., 56 ans, professeur de lycée, de retour de voyage au Sénégal, ressent à la sortie de
l'aéroport, en portant ses bagages, une gêne épigastrique accompagné de nausées. C'est un diabétique non
insulino-dépendant et un hypertendu léger. Cette gêne persiste jusqu'à son retour à son domicile et varie en
intensité dans les heures qui suivent. Le patient pense à un embarras gastrique. Le lendemain, le médecin
traitant réalise un électrocardiogramme et constate une onde Q et un sus-décalage de 2 mm de ST en D2, D3,
aVF, un sous-décalage de ST en Vl et V2, un aspect normal en V3 R, V4 R. Le patient est adressé au Service
de Cardiologie. A l'admission, TA 95/70, fréquence cardiaque 42/mn, pas de signe de défaillance cardiaque, ni
de marbrures cutanées. A L'E.C.G., on note outre les signes notés par le médecin traitant, un bloc auriculo-
ventriculaire complet à QRS fins.

Avant la réalisation de l'E.C.G., quels sont les signes susceptibles d'orienter vers un accident
coronarien ?
A - Retour d'un voyage en pays d'endémie palustre
B - Diabète
C - HTA
D - Apparition du symptôme à l'effort
E - Siège de la douleur
Bonne(s) réponse(s) : B C D

B et C sont des facteurs de risque cardiovasculaire.

Les signes E.C.G. sont en faveur d'une nécrose :


A - Non transmurale inférieure
B - Transmurale inférieure
C - Antéro-septale
D - Latérale
E - Septale profonde
Bonne(s) réponse(s) : B

- Non transmural : pas d'onde Q.


- Antéroseptal : V1, V2, V3.
- Latéral : D1, VL, V5, V6, haut : D1, VL.
- E : V1, V2, V3, D2, D3, VF antéroseptal et inférieur.

Les troubles de la conduction et l'hypotension artérielle notés chez le patient nécessitent en


première intention :
A - La mise en place d'une sonde de stimulation
B - Atropine l.V.
C - Bêtabloquants
D - Dopamine
E - Remplissage vasculaire
Bonne(s) réponse(s) : B

Les blocs des infarctus inférieurs (généralement en rapport avec une atteinte de la coronaire droite) sont de
siège nodal : en effet, le noeud de Tawara est irrigué par une branche de la coronaire droite chez 90 % des
sujets. Ces lésions sont en rapport avec l'ischémie ou à la période initiale avec une hypertonie vagale. Ils sont
le plus souvent régressifs sous atropine IV.

572
Exclusivement sur DOC - DZ : www.doc-dz.com NADJI 85
RESIDANAT EN POCHE TOME II
Cas Clinique en QCM

Compte tenu du délai entre le début de la douleur et l'admission, quelle est votre attitude ?
A - Angioplastie coronaire d'urgence
B - Thrombolyse intraveineuse
C - Thrombolyse intracoronaire
D - Héparine
E - Bêtabloquant
Bonne(s) réponse(s) : D

Le délai pour la mise en route d'une thrombolyse est dépassé (> 6 heures).
E - Est contre-indiqué en raison du BAV.

Quels sont les signes qui évoqueraient un infarctus du ventricule droit ?


A - Des signes de défaillance ventriculaire droite sans signe de poumon cardiaque
B - Des ondes Q et un sus-décalage de ST en V3R V4R
C - Des ondes Q en V7-V8
D - Ventricule droit dilaté et figé à l'échocardiogramme
E - Un sous-décalage de ST en D3 aVF
Bonne(s) réponse(s) : A B D

- 34 à 50 % des infarctus inféro-postérieurs se compliquent d'infarctus du ventricule droit.


- 5 % des infarctus antérieurs se compliquent d'infarctus du VD.

Monsieur P. est adressé pour insuffisance cardiaque droite. Ce patient de 55 ans, tabagique (30
paquets/année) est bronchitique chronique depuis 10 ans. Il est dyspnéique à l'effort et au décubitus depuis 2
ans et a fait une première décompensation cardiaque droite il y a 6 mois. Depuis 15 jours, sont apparues une
toux et une expectoration muqueuse puis purulente suivie 5 jours plus tard d'une aggravation de la dyspnée
d'effort et d'oedèmes des membres inférieurs. Lors de l'admission, l'examen met en évidence une polypnée
associée à une cyanose et des sueurs. Le faciès est érythrosique ; il existe un hippocratisme digital, une
tachycardie à 120/mn, la tension artérielle est à 125/80 mmHg. On retrouve également des oedèmes des
membres inférieurs, un signe de Harzer, un reflux hépato-jugulaire. L'auscultation révèle des râles
bronchiques diffus, des sous crépitants aux bases et un bruit de galop droit. La température est de 38°. La
radiographie montre une cardiomégalie, des opacités hilifuges bilatérales à prédominance droite. Les gaz du
sang artériel sont les suivants : pH : 7,38 PaC02 : 7,3 kPa (53 mmHg) - PaO2 : 7,5 kPa (55 mmHg) -
Bicarbonates : 30 mmoles/l - SaO2 : 87%.
Le bilan biologique est le suivant : Hématocrite = 57 %, natrémie = 120 mmol/l, protidémie à 50 g/l,
leucocytose = 14 300/mm3.

Ce tableau évoque :
A - Un coeur pulmonaire chronique post-embolique
B - Une décompensation cardiaque droite d'une insuffisance respiratoire chronique
C - Une décompensation cardio-respiratoire secondaire à une tachycardie auriculaire
D - Une décompensation cardiaque globale d'une cardiopathie ischémique
E - Une péricardite
Bonne(s) réponse(s) : B

L'insuffisance respiratoire chronique est suggérée par l'hypercapnie (PCO2 à 55 mmHg) le faciès érythrosique
et l'hippocratisme digital.

Les anomalies électrocardiographiques attendues devraient comporter :


A - Des ondes P amples en D2
B - Un bloc de branche incomplet droit
C - Un bloc auriculo-ventriculaire du 1er degré
D - Un axe de QRS dévié à droite
E - Un indice de Sokolow supérieur à 3,5 millimètres
Bonne(s) réponse(s) : A B D

A - L'onde P a une hauteur supérieure ou égale à 3 mm en D2 ou VF = hypertrophie auriculaire droite.


B D - Signes de surcharge ventriculaire droite.

La cause la plus vraisemblable de la décompensation est :


A - Une pneumopathie bactérienne
B - Un oedème aigu pulmonaire
C - Un cancer bronchique
D - Une embolie pulmonaire
E - Une bronchite aiguë
Bonne(s) réponse(s) : E

Sans commentaire.

573
Exclusivement sur DOC - DZ : www.doc-dz.com NADJI 85
RESIDANAT EN POCHE TOME II
Cas Clinique en QCM

Parmi les traitements suivants le(les)quel(s) vous parai(ssen)t conseillé(s) ?


A - Oxygénothérapie
B - Restriction hydrique
C - Digitaliques
D - Almitrine (Vectarion®)
E - Antibiotiques
Bonne(s) réponse(s) : A B D E

- Oxygénothérapie à faible débit (1 l/mn).


- Restriction hydrique en raison de l'hyponatrémie qui témoigne d'une hyperhydratation globale.

Une femme de 63 ans présentant des antécédents lointains de rhumatisme articulaire aigu, a bénéficié jusque-
là d'un état de santé relativement satisfaisant et n'a pas consulté de médecin depuis des années. Elle a vu
apparaître depuis un mois des oedèmes des membres inférieurs et une augmentation de volume de
l'abdomen. L'état général est médiocre avec asthénie et anorexie. L'interrogatoire révèle une dyspnée d'effort
ancienne et des douleurs de l'hypochondre droit. L'examen clinique permet de conclure à une maladie mitrale
et à une insuffisance tricuspide. Vous faites hospitaliser la patiente pour bilan complémentaire et traitement.

L'auscultation cardiaque de cette patiente comporte vraisemblablement :


A - Un souffle diastolique le long du bord gauche du sternum
B - Un roulement diastolique de pointe mieux perçu en décubitus latéral gauche
C - Un souffle systolique xiphoïdien se majorant en inspiration
D - Un claquement d'ouverture au foyer tricuspidien
E - Un souffle éjectionnel au 2ème espace intercostal droit
Bonne(s) réponse(s) : B C

A - Souffle d'insuffisance aortique.


B - L'auscultation d'un RM comprend classiquement :
- éclat de B1
- B2 - claquement d'ouverture mitrale
- roulement diastolique.
C - Souffle d'insuffisance tricuspide = signe de Carvalho.

L'examen clinique recherche des signes périphériques en rapport avec l'atteinte valvulaire et la
défaillance ventriculaire droite. On peut ainsi observer :
A - Une expansion systolique des jugulaires
B - Un élargissement de la tension artérielle différentielle
C - Une hépatomégalie pulsatile
D - Un pouls carotidien petit et retardé
E - Une ascite
Bonne(s) réponse(s) : A C E

A C - Signes d'insuffisance tricuspide.


B - Insuffisance aortique.
D - Rétrécissement aortique.

Le bilan biologique présente un certain nombre d'anomalies. Cochez celle(s) qui peu(ven)t être
directement en rapport avec la défaillance cardiaque et ses conséquences viscérales :
A - Triglycérides à 2,40 mmoles/l
B - Taux de prothrombine à 50%
C - Transaminases (SGPT) à 140 U.l. (normale inférieure à 30)
D - Urée à 1,10 g/l
E - Glycémie à jeun à 1,40 g/l
Bonne(s) réponse(s) : B C D

Insuffisance cardiaque avec foie cardiaque (BC) et insuffisance rénale fonctionnelle (D).

574
Exclusivement sur DOC - DZ : www.doc-dz.com NADJI 85
RESIDANAT EN POCHE TOME II
Cas Clinique en QCM

Un bilan hémodynamique et angiographique est réalisé après quinze jours de traitement digitalo-
diurétique et confirme le diagnostic clinique. Ce bilan révèle :
A - Index cardiaque à 4 I/mn/m2
B - Aspect de la venticularisation de la courbe auriculaire droite en inspiration
C - Gradient de 50 mmHg au retrait ventriculo-aortique
D - Pression capillaire pulmonaire moyenne à 25 mmHg avec pression télédiastolique
ventriculaire gauche à 10 mmHg
E - Pression télédiastolique ventriculaire droite à 15 mmHg
Bonne(s) réponse(s) : B D

B - Signes d'insuffisance tricuspide.


C - Signe de rétrécissement aortique.
D - La pression capillaire pulmonaire donne indirectement la pression de l'oreillette gauche. Le barrage mitral
crée un gradient ventricule gauche-oreillette gauche en télédiastole (25-10 = 15 mmHg).

Un jeune homme en parfaite santé (20 ans, 60 kg), a été plâtré pour entorse grave de la cheville. Six jours
après, il présente des signes d'embolie pulmonaire sans collapsus cardiovasculaire que l'on rattache à une
phlébite fémoropoplitée passée inaperçue. Avant d'entreprendre l'héparinothérapie qui s'impose, vous
demandez un bilan d'hémostase : Temps de Quick 80% ; temps de Céphaline Activé 28 sec (T:31 sec) ;
plaquettes 350 000 /mm3 ; fibrinogène 5,5 g/l.

Parmi les éléments de ce bilan d'hémostase lequel aurait dû vous faire prévoir cette maladie
thromboembolique ?
A - Temps de Quick discrètement allongé
B - Temps de Céphaline Activé discrètement raccourci
C - Plaquettes à la limite supérieure de la normale
D - Discrète hyperfibrinogénémie
E - Aucun des examens ci-dessus
Bonne(s) réponse(s) : E

Sans commentaire.

Parmi les schémas de traitement suivants le(s)quel(s) est(sont) adapté(s) à la situation ?


A - Héparine standard sous cutanée : 5 000 unités toutes les 12 heures
B - Héparine standard intramusculaire : 18 000 unités toutes les 12 heures
C - Héparine standard intra-veineuse : 9 000 unités toutes les 6 heures
D - Héparine standard intra-veineuse : 3 000 unités toutes les 2 heures
E - Héparine standard intra-veineuse : 36 000 unités par 24 heures administrées en perfusion
continue
Bonne(s) réponse(s) : D E

Héparinothérapie à doses efficaces (300 à 500 voire 600 U/kg/24 h) administrée au mieux en perfusion ou par
voie intra-veineuse toutes les deux heures en doses fonctionnées.

Le ou lesquels des examens biologiques suivants est(sont) nécessaire(s) pour surveiller


l'efficacité biologique et la tolérance du traitement ?
A - La numération des plaquettes entre le 5-10ème jour du traitement
B - Le temps de saignement
C - Le temps de Quick (Taux de prothrombine)
D - Le temps de Céphaline Activé
E - Le dosage de l'antithrombine III
Bonne(s) réponse(s) : A D

Les thrombopénies à l'héparine surviennent entre le 5ème et 15ème jour de traitement anticoagulant. Les
précoces survenant avant le 5ème jour de traitement sont modérées et n'imposent pas l'arrêt du traitement.
Celles survenant après le 5ème jour de traitement, sévères, sont souvent associées à des thromboses
artérielles et/ou veineuses.

A J5 du traitement vous apprenez que 2 autres membres de la famille ont présenté des
thromboses veineuses spontanées. L'antithrombine III du patient est à 55 %. Quelle est votre
interprétation ?
A - Ce taux est dans les limites de la normale pour un homme jeune de 20 ans
B - L'antithrombine III a été consommée par la maladie thrombo-embolique
C - Ce résultat doit faire craindre une aggravation de la maladie thrombo-embolique
D - Vous avez à coup sûr détecté un déficit congénital
E - Ce résultat peut être la conséquence de l'héparinothérapie
Bonne(s) réponse(s) : E

L'antithrombine III diminue lors des traitements par l'héparine, car c'est un cofacteur de l'héparine.
575
Exclusivement sur DOC - DZ : www.doc-dz.com NADJI 85
RESIDANAT EN POCHE TOME II
Cas Clinique en QCM

La protéine C du patient est à 50 %. Quelle est votre interprétation ?


A - Ce taux est normal pour un homme jeune de 20 ans
B - Ce taux reflète une carence en vitamine K
C - Ce résultat est ininterprétable à cause de l'héparinothérapie
D - La protéine C a été consommée par la maladie thrombo-embolique
E - Il existe une forte probabilité de déficit congénital
Bonne(s) réponse(s) : E

La protéine C est un facteur vitamine K dépendant qui diminue lors des traitements par les AVK mais qui est
peu affectée par l'héparine.

Après quinze jours d'héparinothérapie conventionnelle, il est décidé de passer au Sintrom®, 1


cp/j. Combien de temps après la prise du premier comprimé allez-vous évaluer l'efficacité du
traitement pour la première fois ?
A - 6 heures
B - 12 heures
C - 24 heures
D - 48 heures
E - 96 heures
Bonne(s) réponse(s) : D

Sans commentaire.

Sur quel examen vous basez-vous pour équilibrer le traitement antivitamine K ?


A - Le temps de saignement
B - Le dosage de fibrinogène
C - Le temps de thrombine
D - Le temps de Quick exprimé en INR (International Noramlised Ratio)
E - Le temps de céphaline activée
Bonne(s) réponse(s) : D

L'INR est un indice permettant de normaliser le temps de Quick en fonction de la thromboplastine utilisée.

Homme de 46 ans, 170 cm, 58 kg, sans antécédent hormis un asthme nécessitant de temps à autre un
bétamimétique en inhalation. Lors d'une consultation de médecine du travail, on constate une pression
artérielle à 175/105 mm Hg.
Il consulte 15 jours plus tard pour vérification des chiffres de P.A. :
absence de symptomatologie fonctionnelle : examen clinique normal ; P.A. = 170/105 mm Hg ; F.C. = 72/mn ;
ECG : Hypertrophie ventriculaire gauche fond d'oeil : normal ; protéinurie positive.

L'(les) examen(s) complémentaire(s) justifié(s) lors de cette première consultation est(sont) le(s)
suivant(s) :
A - Ionogramme plasmatique
B - Glycémie
C - Créatininémie
D - Activité rénine plasmatique
E - Urographie intraveineuse
Bonne(s) réponse(s) : A B C

D E - Ne sont pas réalisées en première intention.

La constatation 15 jours plus tard d'une P.A. à 175/110 mm Hg peut conduire à la prescription de :
A - Furosémide (Lasilix®)
B - Aténolol (Tenormine®)
C - Hydrochlorothiazide et amiloride (Moduretic®)
D - Nifédipine (Adalate®)
E - Acébutolol (Sectral®)
Bonne(s) réponse(s) : A C D

L'asthme contre-indique l'emploi des bêta-bloquants (Sectral, Ténormine).

576
Exclusivement sur DOC - DZ : www.doc-dz.com NADJI 85
RESIDANAT EN POCHE TOME II
Cas Clinique en QCM

L'absence de modification des chiffres de P.A. sous l'une des thérapeutiques précédentes
prescrites à posologie correcte conduite à la prescription d'Enalapril (Renitec®) 10 mg le matin.
Ce médicament :
A - Bloque la conversion d'angiotensine I en angiotensine II
B - Entraine une augmentation de l'activité rénine plasmatique
C - Entraine une augmentation de l'aldostérone plasmatique
D - Peut être plus efficace s'il est associé à un diurétique thiazidique
E - Peut entrainer uen hypokaliémie s'il est associé à un anti-inflammatoire non stéroïdien
Bonne(s) réponse(s) : A B D

L'Enalapril est un inhibiteur de l'enzyme de conversion. La baisse de l'angiotensine II se traduit pas une baisse
du rétro-contrôle négatif sur la production de rénine.
D - Vrai, car le diurétique augmente l'ARP.
C - Faux : entraîne une baisse de l'aldostérone.

Après 3 jours de traitement, la diminution des chiffres de P.A. (120/70 mm Hg) s'accompagnant
d'une élévation de la créatininémie est plutôt évocatrice d'une HTA :
A - Essentielle
B - Secondaire à une glomérulopathie
C - Secondaire à une phéochromocytome
D - Secondaire à un adénome de Conn
E - Secondaire à une sténose bilatérale de l'artère rénale
Bonne(s) réponse(s) : E

Il s'agit d'une contre-indication à l'utilisation des inhibiteurs de l'enzyme de conversion.

Ceci doit conduire à :


A - Un dosage des catécholamines plasmatiques
B - Un dosage des catécholamines urinaires
C - Un dosage de l'adostéronémie
D - Une angiographie numérisée des artères rénales
E - Une tomodensitométrie abdominale
Bonne(s) réponse(s) : D

Pour affirmer le diagnostic de sténose bilatérale des artères rénales.

Une jeune femme de 33 ans est hospitalisée pour insuffisance cardiaque globale fébrile (température = 39°C)
évoluant depuis une quinzaine de jours. Dans les antécédents on retrouve : syndrome dépréssif majeur ,
notions de palpitations depuis 2 ans et un examen cardiologique avec échocardiogramme avait permis de
conclure à l'existence d'une insuffisance mitrale probablement rhumatismale de faible importance.
Enfin des soins dentaires ont étés pratiqués un mois auparavant.
A l'entrée dans le service 1,55m 52 Kg, mauvais état général, mauvais état dentaire, pression artérielle
120/60 mm Hg, fréquence cardiaque 100/mn, pas d'éruption cutanée, pas de splénomégalie ni
d'adénopathies, orthopnée, oedèmes des membres inférieurs bilatéraux, hépatomégalie avec reflux
hépatojugulaire. Les pouls prériphériques sont tous perçus, l'auscultation cardiaque révèle un souffle
systolique 2/6 apexo axillaire, un souffle diastolique 2/6 le long du bord gauche du sternum. Un bruit de galop
présystolique et les bruits du coeur sont bien perçus.
La radiographie thoracique montre un oedème alvéolaire, un index cardiothoracique à 0,55.
L'électrocardiogramme montre une tachycardie sinusale.

Parmi les examens suivants, quels sont ceux dont l'intérêt est évident ?
A - Sérologie VIH
B - Le cathétérisme droit
C - Les hémocultures aérobie et anaérobie
D - L'échocardiographie
E - Panoramique dentaire
Bonne(s) réponse(s) : C D E

E - A la recherche d'une porte d'entrée qui est ici évidente.

577
Exclusivement sur DOC - DZ : www.doc-dz.com NADJI 85
RESIDANAT EN POCHE TOME II
Cas Clinique en QCM

Dans l'histoire de la maladie de cette patient un ou des éléments importants devrai(en)t faire
évoquer le diagnostic :
A - Syndrome dépressif
B - L'apparition du souffle diastolique
C - L'existence de palpitations
D - La perte de poids importante
E - La notion de soins dentaires récents
Bonne(s) réponse(s) : B D E

[L'apparition du souffle diastolique témoigne d'une insuffisance aortique qui n'existait pas auparavant.
L'existence d'un amaigrissement témoigne de l'atteinte de l'état général de cette endocardite sub-aiguë.

L'échocardiogramme peut révéler dans ce cas précis :


A - Un fluttering diastolique des valves mitrales
B - Des végétations sur les valves aortiques
C - Un dédoublement des parois aortiques
D - Une akinésie antérosptoapicale
E - Une dilatation de l'oreillette gauche
Bonne(s) réponse(s) : A B E

A - Témoin de la régurgitation aortique.


E - En raison de l'insuffisance mitrale ancienne.

Quelle(s) thérapeutique(s) immédiate(s) doit-on instituer dès l'admission ?


A - Remplacement valvulaire aortique
B - Traitement digitalo diurétique
C - Antiobiothérapie large spectre par voie orale
D - Pénicilline I.V. forte dose
E - Oxygénation nasale
Bonne(s) réponse(s) : B D E

A - Peut se discuter dans des délais assez brefs si la tolérance hémodynamique reste médiocre.
D - La péniciline sera associée à un aminoside.

Dans le cadre du bilan étiologique le ou les examens suivants doivent être pratiqués :
A - Sérodiagnostic de mycoplasme
B - Radiographies des arcades dentaires
C - Urographie intraveineuse
D - Examen ORL
E - Réaction de Bordet-Wassermann
Bonne(s) réponse(s) : B D

Les portes d'entrée stomatologiques ou ORL sont les plus fréquentes.

En dehors du traitement symtomatique, un traitement étiologique par antibiothérapie doit être


institué, la modalité à respecter est :
A - Injection intramusculaire biquotidienne de Pénicilline G
B - Perfusion continue de Pénicilline G
C - Association de Pénicilline IV et d'un aminoside IV ou IM
D - Durée habituelle du traitement de 10 jours
E - Posologie de la pénicilline G toujours inférieure à 30 M UI/jour
Bonne(s) réponse(s) : C

Association synergique Pénicilline-aminoside par voie parentérale.

578
Exclusivement sur DOC - DZ : www.doc-dz.com NADJI 85
RESIDANAT EN POCHE TOME II
Cas Clinique en QCM
Madame T. 28 ans a présenté dans l'enfance un rhumatisme articulaire aigu compliqué d'un rétrécissement
mitral bien toléré connu depuis 4 ans.
La veille est apparue brutalement une dyspnée d'éffort contemporaine de palpitations.
Les éléments cliniques et électrocardiographiques recueillis à son entrée, permettent de conclure à une
arythmie complète par fibrillation auriculaire avec fréquence ventriculaire entre 130 et et 180/minute. Le poids
est de 60 kg.

Quel(s) affection(s) peu(ven)t avoir favorisé le déclenchement de cette arythmie ?


A - Le rétrécissement mitral seul
B - Une pneumopathie aiguë
C - Une infection bronchique
D - Une hyperthyroïdie
E - Un traiement oestro-progestatif récemment institué
Bonne(s) réponse(s) : A

Le passage en fibrillation auriculaire au cours d'un rétrécissement mitral marque un tournant évolutif dans
l'évolution de la maladie.
B C D - Sont également des causes déclenchantes de fibrillation auriculaire.

En l'absence de contre-indication, quelle(s) thérapeutique(s) peu(ven)t être préconisée(s) parmi


les suivantes :
A - Héparine 300 mg en perfusion intra-veineuse continue par 24 H
B - Vérapamil (Isoptine®) 1 ampoule de 5 mg x 3/jour en intraveineux lent
C - Amiodarone (Cordarone®) 8 cps en une prise le 1er jour
D - Cédilanide 2 ampoules en intraveineux direct 4 fois/jour
E - Mexiletine (Mexitil®) 1 gélule x 3/jour
Bonne(s) réponse(s) : A C

A - Pour éviter une migration embolique.


C - Dose de charge de Cordarone permettant des taux sériques efficaces au bout de 7 à 10 heures.
D - La Cédilanide peut être utilisée mais à des posologies inférieures (1/2 ampoule 4 fois par jour).
E - Anesthésique local qui ressemble à la xylocaïne. Pas d'action à l'étage supra-ventriculaire.

Au bout de 3 semaines, en l'absence de régularisatio, quelle attitude adoptez-vous parmi les


suivantes en l'absence de contre-indication :
A - Adjonction d'un calcium bloqueur
B - Adjonction d'un bêta-bloquant
C - Arrêt de toute thérapeutique et surveillance
D - Choc électrique externe
E - Stimulation endocavitaire
Bonne(s) réponse(s) : D

La réduction par choc électrique externe a plus de chances de réussir lorsque la fibrillation est récente, à
condition dans ce cas précis que l'oreillette gauche ne soit pas trop dilatée.

Après régulation vous instituez un traitement d'entretien par hydroquinidine (Serecor®). Il s'agit :
A - D'un anti-arythmique de la classe 1 A
B - D'un anti-arythmique de la classe 1 B
C - D'un anti-arythmique de la classe 1 C
D - D'un anti-arythmique à la libération prolongée
E - D'un anti-arythmique spécifique de la fibrillation auriculaire
Bonne(s) réponse(s) : A D

Classification de Vaugham-Williams. Diminue le courant sodique rapide.


D - Permet seulement deux prises par jour.

L'(les) effet(s) indésirable(s) ou accident(s) imposant l'arrêt du traitement par Serecor® sont :
A - Une diarrhée persistante
B - Un rash cutané
C - Une syncope brutale
D - Une hypokaliémie
E - Une torsade de pointes
Bonne(s) réponse(s) : A B C D E

La kaliémie devra être surveillée régulièrement car toute hypokaliémie favorise l'allongement de QT et la
survenue de torsades de pointe et de syncope.

579
Exclusivement sur DOC - DZ : www.doc-dz.com NADJI 85
RESIDANAT EN POCHE TOME II
Cas Clinique en QCM

Le traitement par Serecor® devra être interrompu si vous constatez sur l'ECG l'apparition :
A - D'un allongement du QRS supérieur à 50 %
B - D'un aplatissement de l'onde T
C - D'un segment PR à 0,18 secondes
D - D'un segment QT à 0,30 secondes
E - D'extrasystoles nombreuses et polymorphes
Bonne(s) réponse(s) : A E

L'emploi du Sérécor impose de tester la sensibilité du malade au médicament par la pratique d'un ECG, 24
heures après la première prise (réaction d'idiosyncrasie).
Signes d'alerte :
- allongement de QRS > 25 %
- allongement de QT > 0,04 sec
- allongement de PR > 0,24 sec
- ESV nombreuses et polymorphes.
B - Signe d'imprégnation.

Un homme de 65 ans, sans antécédent pathologique notable, consulte pour des douleurs péri-ombilicales et
lombaires gauches, sans troubles du transit.
Il est apyrétique : l'état général est conservé.
Sa TA est à 180/90.
L'examen retrouve une masse abdominale battante et expansive para-ombilicale gauche.
Numération globulaire, urée et créatinine sanguines sont normales.

Quelle est l'orientation diagnostique ?


A - Tumeur pancréatique
B - Tumeur colique gauche
C - Tumeur du rein gauche
D - Anébrysme de l'aorte abdominale
E - Globe vésical par rétention d'urines
Bonne(s) réponse(s) : D

Une masse abdominale battante dont le caractère expansif permet de la rattacher à un anévrisme.

Vous demandez une radiographie de l'abdomen sans préparation face debout, en faveur du
diagnostic clinique on retiendra :
A - Distention aérique dans l'aire de projection du colon droit
B - Présence d'une ombre para-lombaire gauche
C - Présence de calcifications linéaires dessinant une image en fuseau para-lombaire guahce
D - Présence de calcifications en regard de l'apophyse costiforme gauche du L1 L2
E - Disparition du bord externe du psoas
Bonne(s) réponse(s) : B C

C - Les calcifications se voient dans 60 % des cas.

Le diagnostic clinique sera confirmé par :


A - L'examen doppler artériel
B - L'échographie abdominale
C - L'aortographie
D - L'urographie intra-veineuse
E - Le lavement baryté
Bonne(s) réponse(s) : B

Confirme le diagnostic et permet de mesurer la taille de l'anévrisme.

En cours d'hospitalisation, survient brutalement une douleur distale du membre inférieur droit
avec diminution de la chaleur cutanée locale, déficit sensitivo-moteur, absence des pouls au
niveau de ce membre. Quelle est la complication qui émaille l'évolution de la maladie ?
A - Oblitaération artérielle aiguë par embolie
B - Oblitération artérielle aiguë par thrombose d'une poche anévrysmale
C - Crise fissuraire
D - Phlébite surale
E - Migration d'un calcul urétéral
Bonne(s) réponse(s) : A

Les embolies compliquent 10 % des anévrismes de l'aorte abdominale.

580
Exclusivement sur DOC - DZ : www.doc-dz.com NADJI 85
RESIDANAT EN POCHE TOME II
Cas Clinique en QCM

Quelle conduite thérapeutique préconisez-vous ?


A - Traitement médical symptomatique
B - Ablation d'un calcul urétéral
C - Désobstruction artérielle en urgence et traitement anti-coagulant
D - Duodéno-pancréatectomie
E - Aucune de ces propositions
Bonne(s) réponse(s) : C

Embolectomie par sonde de Fogarty.

Homme de 57 ans, 172 cm, 75 kg, sans antécédents hormis une hypertension connue depuis 6 ans et une
dyspnée d'effort depuis 1 an.
Traitement habituel : Digoxine® 1 comprimé/jour, Moduretic® 1/2 comprimé/j. régime pauvre en sel.
Vous êtes appelé en urgence à son domicile pour l'apparition brutale d'une gêne respiratoire.
A l'examen : F.C. = 135/mn mais rythme cardiaque irrégulier ; PA = 180/115 ; F. R . = 32/mn ; T° = 37° ;
conscience normale ; cyanose des lèvres et des oreilles ; sueurs ; tirage intercostal; souffle systolique de
pointe 3/6 ; galop gauche ; il a "bien" uriné juste avant votre arrivée; l'auscultation pulmonaire montre des
râles humides et sibilants aux deux bases ; RAS par ailleurs.

L'arythmie cardiaque peut correspondre :


A - Une fibrillation auriculaire
B - Une tachycardie sinusale avec extrasystoles auriculaires
C - Une tachycardie sinusale avec extrasystoles ventriculaires
D - Un flutter auriculaire à conduction variable
E - Une tachysystolie à conduction variable
Bonne(s) réponse(s) : A B C D E

Seul l'électrocardiogramme peut faire le diagnostic précis de trouble du rythme, car tous ces diagnostics
s'accompagnent d'une fréquence cardiaque irrégulière.

Le souffle peut correspondre à :


A - Une insuffisance mitrale organique
B - Une insuffisance mitrale fonctionnelle liée à la dilatation du ventricule gauche
C - Un rétrécissement mitral
D - Une anémie
E - Un haut débit cardiaque
Bonne(s) réponse(s) : A B

Le souffle systolique de pointe correspond à une insuffisance mitrale organique ou fonctionnelle.

Les éléments du tableau clinique sont compatibles avec :


A - Une embolie pulmonaire
B - Une crise d'asthme
C - Un choc cardiogénique
D - Une insuffisance cardiaque gauche
E - Une décompensation aiguë d'une insuffisance respiratoire chronique
Bonne(s) réponse(s) : D

Insuffisance cardiaque gauche déclenchée par le passage en fibrillation auriculaire.

La (les) thérapeutique(s) utile(s) est (sont) la (les) suivante(s) :


A - Furosémide : 80 mg IVD
B - Isosorbide dinitrate : 5 mg sublingual
C - Lanatoside C : 1 ampoule de 0,4 mg IVD
D - Furosémide : 1 comprimé per os (40 mg)
E - Verapamil : 1 ampoule de 5 mg IVD
Bonne(s) réponse(s) : A B C

A B C - Association de diurétiques (A), de vasodilatateurs (B) et de digitaliques.


E - Contre-indiqué dans l'insuffisance cardiaque en raison de son effet inotrope négatif (Isoptine®).

581
Exclusivement sur DOC - DZ : www.doc-dz.com NADJI 85
RESIDANAT EN POCHE TOME II
Cas Clinique en QCM

La non amélioration de l'état clinique vous amène 15 minutes plus tard à décider de
l'hospitalisation. La (les) thérapeutique(s) utile(s), pendant le transport par le SAMU est (sont) la
(les) suivantes(s) :
A - 500 CC de sérum physiologique (NaCl à 90 %)
B - Isosorbide dinitrate : en perfusion IV continue
C - Transport en position demi-assise
D - Transport en oxygénothéraphie à 9 l/mn
E - Atropine : 1 ampoule de 1 mg IVD
Bonne(s) réponse(s) : B C D

A - Contre-indiqué.
B - Diminue la précharge (retour veineux) et la pression de remplissage du ventricule gauche.

Un homme de 70 ans, alerte, mais tabagique (50 paquets/année) vous consulte pour une visite annuelle.
L'interrogatoire vous apprend que la montée d'une côte entraîne parfois une douleur rétrosternale mais jamais
de douleurs des membres. La tension artérielle est à 19/12 aux deux bras. Le pouls radial bat à 70. Aux
membres inférieurs, à droite aucun pouls n'est perçu; à gauche tous les pouls sont bien frappés. La palpation
de l'abdomen vous révèle une masse indolore d'environ 6 cm de large.

Parmi les signes suivants, lequel vous permet d'affirmer que cette masse est un anévrysme de
l'aorte abdominale ?
A - Masse pulsatile
B - Masse expansive
C - Masse médiane
D - Souffle abdominal
E - Abolition de certains pouls
Bonne(s) réponse(s) : B

Il s'agit d'un masse battante et expansive, non douloureuse, médiane ou légèrement latéralisée à gauche.

Quel examen demanderiez-vous en premier pour confirmer le diagnostic ?


A - Abdomen sans préparation
B - Echotomographie
C - Scanner abdominal
D - Angiographie digitalisée
E - Doppler des membres inférieurs
Bonne(s) réponse(s) : B

L'échotomographie permet de mesurer le diamètre de l'anévrysme et de rechercher une éventuelle thrombose.

Quelle artère est, selon les données cliniques, certainement obstruée ?


A - lliaque
B - Fémorale commune
C - Fémorale superficielle
D - Poplitée
E - Tibiale antérieure
Bonne(s) réponse(s) : A

L'abolition du pouls fémoral est en faveur d'une thrombose de l'iliaque primitive ou externe.

Quel élément impose une intervention chirurgicale chez cet homme en bon état général ?
A - Abolition des pouls
B - Taille de l'anévrysme
C - Association d'une hypertension artérielle
D - Age du malade
E - Absence de critères d'insuffisance cardiaque
Bonne(s) réponse(s) : B

Le risque de rupture est important dès que l'anévrysme atteint la taille de 6 cm, ce qui impose une correction
chirurgicale.

582
Exclusivement sur DOC - DZ : www.doc-dz.com NADJI 85
RESIDANAT EN POCHE TOME II
Cas Clinique en QCM

Quel traitement préconisez-vous ?


A - Sympathectomie
B - Pontage axillo bifémoral
C - Remplacement aortique par prothèse
D - Embolectomie
E - Résection anévrysme simple
Bonne(s) réponse(s) : C

Sans commentaires.

Un homme de 39 ans est vu en consultation pour une asthénie importante. Depuis un mois cet homme a de
plus des céphalées. Sa tension artérielle a toujours été normale : 130/80 mmHg. Il ne boit pas d'alcool, ne
prend pas d'antésite. Il fume 30 cigarettes par jour depuis l'âge de 18 ans. Il n'existe aucun antécédent familial
d'hypertension artérielle ou de maladie rénale.
A l'examen clinique on note un poids de 66 kg pour 1,75 m, une fréquence respiratoire à 13 par minute, des
réflexes ostéo-tendineux faibles. La pression artérielle est de 235/130 mmHg avec une fréquence cardiaque
de 64 par minute en position couchée et de 250/140 mmHg avec une fréquence cardiaque de 66 par minute
en position debout. Les pouls fémoraux sont bien battants. Le reste de l'examen clinique est normal. Les
examens biologiques sanguins montrent : Glycémie 5,04 mmol/l - Sodium 145 mmol/l - Potassium 2,6 mmol/l -
Chlore 106 mmol/l - Bicarbonate 32 mmol/l - Créatinine 100 micromol/l - Dans les urines de 24 heures (2 900
ml/24 heures soit 2 ml/mn) la concentration en mmol/l est de 80 pour le sodium et de 35 pour le potassium. La
concentration urinaire de créatinine dans ces urines est de 5 000 micromol/l.

Parmi les causes suivantes, éventuellement curables, d'hypertension artérielle, quelles sont les
deux causes que vous retenez ?
A - Phéochromocytome
B - Sténose d'une artère rénale
C - Hyperaldostéronisme primaire par adénome de Conn
D - Hyperglucocorticisme de Cushing
E - Coarctation aortique
Bonne(s) réponse(s) : B C

Ce patient présente une HTA sévère et récente avec hypokaliémie importante. Il faut donc évoquer B et C.

Un électrocardiogramme est fait. Parmi les signes suivants quels sont ceux que l'on peut
s'attendre à trouver sur le tracé obtenu ?
A - Fibrillation ventriculaire
B - Apparition d'une onde U
C - Affaissement du segment ST
D - Ondes T amples pointues symétriques
E - Raccourcissement de l'espace QT
Bonne(s) réponse(s) : B C

On peut également rencontrer des troubles de l'excitabilité ventriculaire à type d'extrasystoles ventriculaires,
une diminution de l'amplitude voire une inversion des ondes T et un allongement de QT.

Deux examens d'orientation diagnostique sont indispensables avant la mise en route de tout
traitement :
A - Dosage de la concentration plasmatique d'aldostérone
B - Dosage de l'activité rénine plasmatique en position couchée et debout
C - Dosage des VMA et des catécholamines urinaires
D - Angiographie des artères rénales
E - Examen tomodensitométrique de l'abdomen centré sur les surrénales
Bonne(s) réponse(s) : A B

En effet, la plupart des médicaments antihypertenseurs risquent d'interférer sur ces dosages et de gêner
l'interprétation des résultats. Dans l'adénome de Conn, l'ARP est très abaissée et peu stimulable par
l'orthostatisme; l'aldostérone plasmatique est le plus souvent élevée.

583
Exclusivement sur DOC - DZ : www.doc-dz.com NADJI 85
RESIDANAT EN POCHE TOME II
Cas Clinique en QCM

On sait que cet homme a un poids stable depuis au moins un mois. Quel est approximativement
son apport sodé alimentaire quotidien ?
A - Environ 160 mmol/jour
B - Environ 240 mmol/jour
C - Environ 100 mmol/jour
D - Environ 80 mmol/jour
E - Il est impossible de répondre à cette question
Bonne(s) réponse(s) : B

Si le patient a un poids stable, les sorties sodées sont égales aux apports sodés soit environ 240 mmol/jour.

Dans cette observation la natriurie élevée :


A - Permet d'éliminer un hyperaldostéronisme primaire
B - Permet d'éliminer un hyperaldostéronisme secondaire
C - Fait suspecter ici un hypominéralo-corticisme
D - Est en rapport avec l'alimentation
E - Est liée à une anomalie tubulaire primitive
Bonne(s) réponse(s) : D

Sans commentaires.

Parmi les traitements suivants, indiquez celui ou ceux qui est (sont) contrindiqué(s) ?
A - Diurétique de l'anse de Henlé (type Lasilix®)
B - Diurétique thiazidique (type Diurilix®)
C - Antihormone minéralo-corticoïde (type Aldactone®)
D - Inhibiteur calcique (type Adalate®)
E - Antihypertenseur d'action centrale (type Aldomet®)
Bonne(s) réponse(s) : A B

A et B en raison de l'hypokaliémie, car ce sont des diurétiques hypokaliémiants.

La clairance de la créatinine (non rapportée à la surface corporelle) de cet homme est de :


A - 120 ml/mn
B - 110 ml/mn
C - 100 ml/mn
D - 90 ml/mn
E - 80 ml/mn
Bonne(s) réponse(s) : C

La clairance de la créatinine se calcule par la formule U*V/P où U et P sont les concentrations urinaire et
plasmatique de créatinine et V le débit urinaire qu'il faut ici rapporter en ml/min.

Un homme de 60 ans hypertendu connu et non traité est hospitalisé d'urgence pour un syndrome douloureux
thoracique.
Ces douleurs ont débuté dans la région pré-sternale où elles étaient très intenses et s'accompagnaient d'une
sensation de malaise. Une heure après, cette douleur pré-sternale devient plus sourde et s'étend dans la
région inter-scapulaire postérieure à type de déchirure. Enfin, quelque temps après sont apparues des
lombalgies gauches et une perte de connaissance pendant deux minutes environ.
A l'examen, le patient est agité et pâle. Le pouls est rapide à 120/mn ; la pression artérielle est à 140/90
mmHg au bras gauche et 90/70 mmHg au bras droit. L'électrocardiogramme retrouve des signes d'ischémie
myocardique dans le territoire postérieur.

Quelle est l'orientation diagnostique ?


A - Embolie pulmonaire
B - Dissection de l'aorte
C - Rupture de l'isthme de l'aorte
D - Infarctus du myocarde
E - Péricardite aiguë
Bonne(s) réponse(s) : B

Le caractère migrateur de la douleur avec irradiation dorsale et lombaire, le terrain hypertendu, et l'asymétrie
tensionnelle évoque fortement le diagnostic de dissection aortique.

584
Exclusivement sur DOC - DZ : www.doc-dz.com NADJI 85
RESIDANAT EN POCHE TOME II
Cas Clinique en QCM

Quels examens complémentaires demandez-vous en urgence ?


A - Radiographie du thorax
B - Angio-pneumographie
C - Aortographie
D - Echocardiogramme
E - Coronarographie
Bonne(s) réponse(s) : A C D

La radiographie thoracique recherchera un élargissement du médiastin. L'échocardiographie recherchera une


image de dissection par voie transthoracique ou mieux par voie transoesophagienne. L'aortographie reste de
l'examen de référence et permettra de préciser le type exact de la dissection et son extension.

En cours d'exploration, la douleur thoracique réapparaît et s'accompagne d'une dyspnée avec


turgescence des jugulaires. Vous commencez immédiatement le traitement qui comporte au
début :
A - Bêtabloquant
B - Héparine 5 mg/kg
C - Thrombolytiques
D - Dérivés nitrés
E - Traitement chirurgical en urgence
Bonne(s) réponse(s) : E

L'apparition d'une dyspnée avec turgescence des jugulaires dans ce contexte traduit un hémopéricarde
compressif ( tamponnade) par fissuration de la dissection dans le sac péricardique .

L'épisode aigu étant maîtrisé, on recherchera comme causes :


A - Une phlébite profonde
B - Un cancer non connu
C - Une anomalie du tissu élastique
D - Une hypertension artérielle mal équilibrée
E - Une virose
Bonne(s) réponse(s) : C D

Les principaux facteurs étiologiques reconnus de dissection sont les maladies du tissu élastique (comme le
syndrome de Marfan) et l'hypertension artérielle.

Le traitement de fond comportera :


A - Maîtrise de la tension artérielle
B - Traitement anti-coagulant
C - Traitement anti-agrégant plaquettaire
D - Dérivés nitrés
E - Surveillance régulière sans aucun de ces traitements
Bonne(s) réponse(s) : A

B est contre-indiqué au cours d'une dissection. C et D n'apportent rien de plus. La maîtrise de la pression
artérielle est fondamentale.

Madame F. 30 ans, non fumeuse, sans antécédents pathologiques, consulte pour la survenue d'un
gonflement bilatéral des doigts, apparu depuis un an.
L'interrogatoire vous apprend qu'elle présente depuis 3 ans, des épisodes intermittents de décoloration
douloureuse, violacée puis rouge. Cette symptomatologie survient particulièrement l'hiver mais aussi l'été en
Bretagne, quand elle se baigne. Enfin, elle vous signale qu'elle supporte moins bien ses lentilles de contact
depuis 6 mois avec une sensation de sécheresse des yeux.

Parmi les propositions suivantes, laquelle ou lesquelles retenez-vous en faveur d'un syndrome de
Raynaud :
A - Début par une décoloration blanche des extrémités
B - Gonflement bilatéral des doigts
C - Déclenchement par le froid
D - Sécheresse oculaire associée
E - Phase douloureuse secondaire
Bonne(s) réponse(s) :

QUESTION ANNULEE

585
Exclusivement sur DOC - DZ : www.doc-dz.com NADJI 85
RESIDANAT EN POCHE TOME II
Cas Clinique en QCM

Parmi les propositions suivantes, laquelle ou lesquelles vous oriente(nt) vers un diagnostic
étiologique de collagénose :
A - Caractère bilatéral du syndrome de Raynaud
B - Absence d'antécédents pathologiques
C - Sémiologie complète du syndrome de Raynaud
D - Sécheresse oculaire
E - Evolution paroxystique des crises
Bonne(s) réponse(s) :

QUESTION ANNULEE

Quel est ou quels sont les examens complémentaires que vous proposez pour confirmer votre
impression clinique de collagénose :
A - Capillaroscopie
B - Artériographie avec clichés positionnels
C - Epreuves fonctionnelles respiratoires
D - Schirmer
E - Biopsie du grêle
Bonne(s) réponse(s) :

QUESTION ANNULEE

Parmi les médicaments suivants, quel(s) est(sont) celui(ceux) contre-indiqués chez cette patiente :
A - Ampicilline
B - Oestrogénothérapie
C - Dihydroergotamine
D - Bêta-bloquants
E - Nifédipine
Bonne(s) réponse(s) :

QUESTION ANNULEE

Une femme de 35 ans, consulte pour céphalées intermittentes.


Elle n'a aucun antécédent pathologique et à arrêté il y a six mois un traitement contraceptif par
oestroprogestatifs pris depuis 6 ans.
Le diagnostic d'hypertension artérielle est retenu.
Le fond d'oeil montre un stade II de rétinopathie hypertensive, l'ECG des signes d'hypertrophie ventriculaire
gauche. Une protéinurie de 0,54 g/24 h est notée, sans anomalie du sédiment urinaire. L'urographie
intraveineuse précise une sécrétion synchrone du produit de contraste, une indentation corticale du contour
rénal au pôle inférieur gauche en regard duquel la cavité calicielle est élargie, le fond caliciel déformé en
massue affleurant le bord extrème du rein.

L'ECG dans l'hypertrophie ventriculaire gauche montre :


A - Une durée de QRS inférieure à 0,10 secondes
B - Axe de T opposé à axe de QRS de 60 degrés à 180 degrés
C - Une durée de PR supérieure à 0, 24 sec.
D - Une négativation de l'onde T en V5, V6
E - Amplitude (SV1 + RV6) < 35 mm
Bonne(s) réponse(s) :

QUESTION ANNULEE

Les anomalies rénales notées à l'urographie IV témoignent de :


A - Cancer du rein
B - Anomalie de la jonction pyelocalicielle
C - Sténose artère rénale
D - Dysplasie segmentaire
E - Nephro-angiosclérose
Bonne(s) réponse(s) :

QUESTION ANNULEE

586
Exclusivement sur DOC - DZ : www.doc-dz.com NADJI 85
RESIDANAT EN POCHE TOME II
Cas Clinique en QCM

La persistance de l'hypertension artérielle après l'arrêt du traitement oestroprogestatif peut


s'expliquer par :
A - Effet prolongé des oestroprogestatifs de synthèse
B - La survenue d'une néphropathie
C - Un tabagisme
D - Les anomalies urographiques
E - Un phéochromocytome
Bonne(s) réponse(s) :

QUESTION ANNULEE

Si vous retenez la responsabilité initiale de l'action hypertensive des oestroprogestatifs, comment


expliquez-vous son mécanisme ?
A - Hyperprolactinémie associée
B - Stimulation de la sécrétion de rénine
C - Augmentation de la synthèse d'angiotensinogène
D - Stimulation adrénergique
E - Stimulation de la synthèse des glucocorticoïdes
Bonne(s) réponse(s) :

QUESTION ANNULEE

Des symptômes suivants, que recherche votre examen clinique, précisez celui (ou ceux) qui
indiquerai(en)t la gravité de l'hypertension artérielle et ferai(en)t suspecter le diagnostic
"d'hypertension artérielle maligne" :
A - Céphalées
B - Paresthésies
C - Acouphènes
D - Bradycardie
E - Amaigrissement
Bonne(s) réponse(s) :

QUESTION ANNULEE

Un homme de 75 ans, en bon état général, artéritique ancien connu (claudication intermittente des mollets
avec périmètre de marche à 500 mètres) ressent brutalement une violente douleur du mollet droit.
A l'examen, 7 H après le début de la symptomatologie douloureuse, le membre inférieur droit est glacé
jusqu'au 1/3 inférieur de la cuisse. Les pouls fémoraux au scarpa sont perçus avec un souffle systolique à leur
niveau ; le pouls poplité n'est perçu qu'à gauche : les pouls tibiaux antérieurs et postérieurs sont absents des
deux côtés.
L'auscultation cardiaque retrouve des bruits du coeur irréguliers. Enfin, il existe une paralysie du pied, mais
pas de nécrose cutanée.

En faveur d'une origine embolique, on retiendra :


A - Une paralysie sensitivo-motrice
B - Des bruits du coeur irréguliers
C - Une claudication intermittente des mollets
D - Un début brutal
E - L'âge du sujet
Bonne(s) réponse(s) : B D

B évoque la survenue d'une fibrillation auriculaire qui a pu être à l'origine d'un accident embolique.
D plaide en faveur d'une embolie le début étant classiquement plus progressif en cas de thrombose aiguë.
C et E plaideraient plutôt pour une thrombose.

En faveur d'une thrombose artérielle in situ, on retiendra :


A - Une absence des pouls distaux controlatéraux
B - Une claudication intermittente
C - Des souffles fémoraux
D - Un membre inférieur glacé
E - Une absence de troubles trophiques
Bonne(s) réponse(s) : A B C

A B C témoignent de la pré-existence d'une artériopathie des membres inférieurs.

587
Exclusivement sur DOC - DZ : www.doc-dz.com NADJI 85
RESIDANAT EN POCHE TOME II
Cas Clinique en QCM

L'artériographie est faite en urgence. En faveur de la thrombose aiguë, on retiendra :


A - La présence d'irrégularités pariétales de l'artère fémorale superficielle gauche
B - L'absence d'opacification isolée de l'artère fémorale profonde droite
C - L'absence d'opacification vasculaire au niveau de la jambe droite (désert vasculaire)
D - Une interruption de l'artère fémorale superficielle droite à son origine avec présence d'une
circulation collatérale sous-jacente
E - Une interruption de l'artère fémorale droite au niveau du canal de Hunter (1/3 inférieur de la
cuisse)
Bonne(s) réponse(s) : D

L'existence d'une circulation collatérale plaide en faveur d'une thrombose artérielle.

La restauration artérielle ne peut être réalisée qu'à la 12ème heure. Dans les suites opératoires, il
faudra craindre l'apparition :
A - D'une acidose métabolique
B - D'une hypokaliémie brutale
C - D'un oedème aigu du poumon
D - D'un oedème du membre inférieur droit
E - D'une oligo-anurie
Bonne(s) réponse(s) : A D E

La réanimation post-opératoire d'une intervention tardive pour ischémie des membres inférieurs surveillera
l'apparition d'une acidose-hyperkaliémie et d'une insuffisance rénale aiguë secondaire à la rhabdomyolyse et
la myoglobinurie. Par ailleurs, l'oedème de revascularisation du membre justifie la pratique d'aponévrotomies
de décharge.

Un homme de 38 ans a depuis quelques semaines une asthénie importante. A l'examen on découvre une
hypertension artérielle à 180/110 mmHg. Les artères fémorales sont bien battantes. L'examen de l'abdomen
est normal.
Cet homme n'a aucun antécédent et la dernière fois que sa pression artérielle a été prise il y a 5 ans celle-ci
était normale. Le fond d'oeil est normal, l'électrocardiogramme montre une onde U. La créatininémie est à 90
micromol/l.
La kaliémie mesurée deux fois est à 3 mmol/l.

Chez cet homme hypertendu ayant une hypokaliémie, l'interrogatoire doit éliminer la prise de
produits hypokaliémants. Ainsi vous devez rechercher la prise de :
A - Réglisse d'une façon régulière
B - Diurétiques thiazidiques type Diurilix®
C - Diurétiques de l'anse type Lasilix®
D - Diurétiques distaux type Aldactone®
E - Analgésiques
Bonne(s) réponse(s) : A B C

A B et C augmentent la kaliurèse. D est un diurétique épargneur de potassium dit anti-aldostérone. E peut être
responsable de néphropathie intersticielle chronique, qui ne s'accompagne pas en règle d'hypokaliémie.

Le ou les diagnostics qu'il convient de discuter est ou sont :


A - Hyperaldostéronisme primaire
B - Sténose d'une artère rénale
C - Phéochromocytome
D - Tumeur sécrétant de la rénine
E - Coarctation de l'aorte
Bonne(s) réponse(s) : A B D E

Toutes ces étiologies peuvent s'accompagner d'un hyperaldostéronisme primaire ou secondaire, pouvant être
responsable d'une hypokaliémie.

L'examen biologique permettant de différencier avec certitude l'hyperaldostéronisme primaire et


hyperaldostéronisme secondaire est le dosage de :
A - L'activité rénine plasmatique
B - L'aldostérone plasmatique
C - La natrémie
D - Les bicarbonates plasmatiques
E - La glycémie
Bonne(s) réponse(s) : A

En cas d'hyperaldostéronisme primaire, l'ARP sera basse alors qu'elle est élevée en cas
d'hyperaldostéronisme secondaire.
588
Exclusivement sur DOC - DZ : www.doc-dz.com NADJI 85
RESIDANAT EN POCHE TOME II
Cas Clinique en QCM

Le ou les examens permettant de différencier hyperplasie surrénalienne bilatérale et adénome de


Conn est ou sont :
A - Dosage de la rénine dans les veines surrénales
B - Dosage de l'aldostérone dans les veines surrénales
C - Test de freination par du soluté salé physiologique
D - Phlébographie surrénalienne
E - Examen tomodensitométrique (scanner)
Bonne(s) réponse(s) :

QUESTION ANNULEE.

Le médicament le plus logique pour traiter l'hypertension artérielle par hyperaldostéronisme


primaire est :
A - Les thiazidiques
B - La spironolactone (Aldactone®)
C - Les béta-bloquants
D - Le furosémide
E - Un inhibiteur de l'enzyme de conversion
Bonne(s) réponse(s) : B

La spironolactone (Aldactone) est un diurétique épargneur de potassium : sa structure de dérivé stéroïdien


proche de l'aldostérone en fait un antagoniste de cette hormone au niveau de son récepteur.

L'onde U vu sur un E.C.G. est en faveur :


A - D'une hypercalcémie
B - D'une intoxication digitalique
C - D'une hyperkaliémie
D - D'une acidose métabolique
E - D'aucun des diagnostics précédents
Bonne(s) réponse(s) : E

L'onde U témoigne de l'hypokaliémie.

Une femme d'une cinquantaine d'années a bénéficié, il y a 11 ans, d'une commissurotomie mitrale à coeur
fermé pour un rétrécissement mitral pur, serré. Elle en a tiré un réel bénéfice. Il y a 2 mois, son cardiologue l'a
examinée et a retrouvé un roulement diastolique 5/10è et un claquement d'ouverture mitrale à la pointe; le
rythme était régulier, sinusal ; la TA était à 120/75 mmHg.
Le seul traitement prescrit est un comprimé de Digoxine tous les 2 jours . Brutalement, alors qu'elle fait
quelques travaux de jardinage, elle ressent des bourdonnements d'oreille, une instabilité à la marche et une
difficulté à se servir de la main gauche.Tout rentre dans l'ordre en 2 heures. Le médecin appelé note que le
rythme cardiaque est irrégulier, le reste de l'examen étant inchangé. L'ECG est le suivant :

Lequel de ces troubles du rythme figure sur le tracé précédent ?


A - Tachycardie jonctionnelle
B - Fibrillation auriculaire
C - Flutter auriculaire
D - Extrasystoles auriculaires en salves
E - Tachysystolie auriculaire
Bonne(s) réponse(s) : B

Ce tracé montre un rythme ventriculaire irrégulier et une fibrillation auriculaire dite à "grosses mailles": les
ondes P d'origine sinusale ont disparu et sont remplacées par des ondes F de fibrillation rapides et irrégulières
dont l'amplitude est importante en raison de la dilatation auriculaire.

Parmi les hypothèses diagnostiques suivantes, laquelle vous parait la plus plausible dans ce cas
précis ?
A - Embolie systémique à point de départ cardiaque
B - Hémorragie cérébrale
C - Thrombose de la carotide droite
D - Tumeur intracrânienne
E - Embolie systémique à point de départ carotidien
Bonne(s) réponse(s) : A

A parait le plus plausible en raison de la coïncidence de cet accident avec le passage récent en fibrillation
auriculaire.

589
Exclusivement sur DOC - DZ : www.doc-dz.com NADJI 85
RESIDANAT EN POCHE TOME II
Cas Clinique en QCM

Quel examen complémentaire vous parait strictement indispensable en première intention pour
étayer votre hypothèse diagnostique ?
A - Scanner cérébral
B - Examen doppler des vaisseaux à destinée cérébrale
C - Artériographie carotidienne
D - Angiographie des cavités cardiaques
E - Echocardiographie
Bonne(s) réponse(s) :

QUESTION ANNULEE.

Quelle(s) mesure(s) thérapeutique(s) immédiate(s) proposez-vous (sans préjuger des


thérapeutiques pouvant s'avérer nécessaires ultérieurement) ?
A - Héparinothérapie
B - Traitement hypotenseur
C - Fibrinolytique
D - Traitement bêta-bloqueur
E - Antivitamines K
Bonne(s) réponse(s) : A

A est indispensable. C est contre-indiqué. E sera donné en relais de A. D est parfois prescrit pour ralentir une
fibrillation auriculaire. Dans ce cas, on préférera tenter une réduction du trouble du rythme par une association
digitaline-cordarone.

Une jeune femme Maghrébine âgée de 35 ans, vient consulter parce qu'elle a fait récemment un oedème aigu
du poumon sur un fond de dyspnée d'effort datant de 4 ans. Cette jeune femme a fait un rhumatisme
articulaire aigu à l'âge de 12 ans.L'auscultation permet d'entendre les signes stéthacoustiques d'un
rétrécissement mitral (R.M.). L'auscultation des autres orifices cardiaques est normale. Des examens
complémentaires sont demandés en vue d'une éventuelle opération chirurgicale.

Indiquez parmi les données suivantes fournies par le cathétérisme cardiaque celle qui confirme le
R. M. :
A - Gradient entre la pression capillaire et la pression diastolique du ventricule gauche
B - Baisse de la pression diastolique du ventricule gauche
C - Gradient entre la pression capillaire et la pression systolique du ventricule gauche
D - Abaissement de l'index cardiaque
E - Hypertension artérielle pulmonaire
Bonne(s) réponse(s) : A

A traduit le barrage entre l'OG et le VG. D et E peuvent être notés mais sont des signes indirects, non
spécifiques du RM.

L'examen le plus fiable pour juger de l'importance des lésions sous-valvulaires est :
A - L'auscultation du coeur
B - L'électrocardiogramme
C - L'évaluation hémodynamique
D - L'échocardiogramme
E - L'angiographie ventriculaire gauche
Bonne(s) réponse(s) : D

L'appréciation de l'importance des lésions sous-valvulaires est primordial dans la décision thérapeutique: si
elles sont peu importantes, on peut proposer une dilatation percutanée par ballonnet ou une comissurotomie
chirurgicale; dans le cas contraire, on préférera le remplacement mitral par une valve prothétique.

On doit craindre une thrombose dans l'oreillette gauche lorsqu'existe(nt) :


A - Une fibrillation auriculaire
B - Des lésions sous-valvulaires importantes
C - Une hypertension artérielle pulmonaire
D - Des antécédents d'embolie pulmonaire
E - Des antécédents d'embolie artérielle
Bonne(s) réponse(s) : A E

A favorise la stagnation du sang au niveau de l'oreillette dilatée et la formation de thrombus. Dans ce


contexte, E évoque un embol d'origine auriculaire gauche.

590
Exclusivement sur DOC - DZ : www.doc-dz.com NADJI 85
RESIDANAT EN POCHE TOME II
Cas Clinique en QCM

Parmi les éléments suivants, indiquez ceux qui contre-indique(nt) une simple commissurotomie
mitrale :
A - Age supérieur à 30 ans
B - Insuffisance tricuspide associée
C - Thrombose intra-auriculaire gauche + embolies systémiques
D - Calcifications sur les valves mitrales
E - Fuite mitrale importante
Bonne(s) réponse(s) :

QUESTION ANNULEE.

Monsieur X..., 62 ans, souffre d'une myocardiopathie non obstructive en apparence primitive. Depuis une
première poussée d'insuffisance cardiaque globale, il y a un an, il reçoit les médicaments suivants : Sintrom®,
Digitaline®, Lasilix® Aldactone® et Cordarone®. Depuis deux mois, sa condition cardiovasculaire se dégrade
à nouveau; il est dyspnéique au moindre effort, se plaint d'une asthénie extrême et perd du poids malgré la
réapparition de volumineux oedèmes des MI. L'examen clinique note une arythmie à 90/mn, une pression
artérielle à 110/90 mmHg. La radiographie thoracique montre une cardiomégalie importante (ICT = 0,65), une
hypervascularisation pulmonaire et un épanchement pleural gauche. L'ECG révèle une fibrillation auriculaire
et un bloc de branche droit complet. Un cathétérisme cardiaque droit confirme l'élévation des pressions de
remplissage ventriculaires (Pression Oreillette Droite à 18 mmHg et Pression Capillaire Pulmonaire à 30
mmHg) et un débit cardiaque très abaissé (IC= 1,5 I/mn/m2). Devant ce tableau d'insuffisance cardiaque de
stade IV réfractaire, le recours à un traitement vasodilatateur apparait souhaitable.

Parmi les signes suivants tirés de l'observation, lequel ou lesquels peuvent être attribués à une
insuffisance ventriculaire droite ?
A - Fibrillation auriculaire
B - Epanchement pleural gauche
C - Oedèmes des membres inférieurs
D - Bloc de branche droit
E - Augmentation de la POD
Bonne(s) réponse(s) :

QUESTION ANNULEE

Si on prescrit un dérivé nitré chez ce patient, quel(s) effet(s) clinique(s)et hémodynamique(s)


favorable(s) peut-on espérer en priorité ?
A - Réduction de la dyspnée
B - Augmentation de la PA systolique
C - Réduction de la POD
D - Réduction de la PCP
E - Augmentation du débit cardiaque
Bonne(s) réponse(s) :

QUESTION ANNULEE

Si on prescrit isolément du Nepressol® (Dihydralazine), quel est l'effet favorable (clinique ou


hémodynamique) que l'on peut espérer en priorité ?
A - Réduction de la dyspnée
B - Augmentation de la PA systolique
C - Réduction de la POD
D - Réduction de la PCP
E - Augmentation du débit cardiaque
Bonne(s) réponse(s) :

QUESTION ANNULEE

Si on prescrit du Minipress® (prazosine), quel(s) effet(s) clinique(s) et hémodynamique(s)


favorable(s) peut-on espérer ?
A - Réduction de la dyspnée
B - Augmentation de la PA systolique
C - Réduction de la POD
D - Réduction de la PCP
E - Augmentation du débit cardiaque
Bonne(s) réponse(s) :

QUESTION ANNULEE

591
Exclusivement sur DOC - DZ : www.doc-dz.com NADJI 85
RESIDANAT EN POCHE TOME II
Cas Clinique en QCM

Si on prescrit du Lopril® (captopril), quel(s) effet(s) favorable(s) clinique(s) ou hémodynamique(s)


peut-on espérer ?
A - Réduction de la dyspnée
B - Augmentation de la PA systolique
C - Réduction de la POD
D - Réduction de la PCP
E - Augmentation du débit cardiaque
Bonne(s) réponse(s) :

QUESTION ANNULEE

En cas de prescription de Lopril®, quel est parmi les 5 médicaments antérieurement prescrits
chez ce patient celui dont le maintien est a priori contre-indiqué ?
A - Sintrom®
B - Digitaline®
C - Lasilix®
D - Aldactone®
E - Cordarone®
Bonne(s) réponse(s) :

QUESTION ANNULEE

Une femme de 50 ans présente en période post-opératoire suivant une hystérectomie pour fibrome, une
douleur du membre inférieur gauche qui fait suspecter une phlébite. La phlébographie confirme le diagnostic
de phlébite et précise sa localisation poplitée gauche. Un traitement par héparine est alors commencé.

L'héparine s'exprime en unités internationales (UI). Quelle est la correspondance de 100 mg


d'héparine en UI ?
A - 500 UI
B - 1 000 UI
C - 5 000 UI
D - 10 000 UI
E - 25 000 UI
Bonne(s) réponse(s) : D

1 mg = 100 UI.

Après 48 heures de traitement le passage à la Calciparine® est décidé.


Sa posologie s'exprime habituellement en ml/kg/24 heures.
Quelle est la correspondance de 1 ml de Calciparine® ?
A - 5 000 UI d'héparine
B - 10 000 UI d'héparine
C - 15 000 UI d'héparine
D - 20 000 UI d'héparine
E - 25 000 UI d'héparine
Bonne(s) réponse(s) : E

Sans commentaire.

Au 6ème jour de traitement héparinique, le relai par les antivitamines K (AVK) est décidé. Parmi
les propositions suivantes concernant les AVK, laquelle (lesquelles) est (sont) exacte(s) ?
A - Les AVK sont commencés le jour de l'arrêt de l'héparine
B - Les AVK sont commencés par une dose de charge de 3 comprimés par jour
C - Le premier contrôle biologique sera fait 6 heures après la première prise
D - L'INR pour être efficace doit être entre 2,5 et 3,5
E - Les AVK seront poursuivis systématiquement pendant un an
Bonne(s) réponse(s) : D

L'INR permet une normalisation de la valeur du temps de Quick quelle que soit la thromboplastine utilisée.

592
Exclusivement sur DOC - DZ : www.doc-dz.com NADJI 85
RESIDANAT EN POCHE TOME II
Cas Clinique en QCM

Parmi les suivants, quel(s) facteurs(s) de la coagulation est (sont) vitamino-K dépendant(s) ?
A - II
B-V
C - VII
D - IX
E-X
Bonne(s) réponse(s) : A C D E

Sans commentaire.

Parmi les suivants, quel(s) est (sont) le ou les facteur(s) qui est (sont) évalué(s) par le temps de
Quick et l'INR ?
A - II
B-V
C - VII
D - IX
E-X
Bonne(s) réponse(s) : A B C E

Le temps de Quick explore la voie tissulaire.

Un homme de 65 ans, chauffeur en retraite artéritique connu, opéré 4 ans auparavant d'une thrombose
termino-aortique par pontage prothétique aorto-bifémoral revient consulter pour bilan annuel systématique.
Depuis l'intervention, les possibilités de marche sont redevenues normales.
A l'examen, tous les pouls périphériques sont présents aux différents étages des deux membres inférieurs. Au
niveau du Scarpa droit existe une masse de 3 cm de diamètre, située sur le trajet artériel, battante mais dont
on ne peut dire si elle est expansive; elle n'est ni douloureuse, ni inflammatoire et elle est apparue depuis
environ 6 mois. Le reste de l'examen ne révèle aucune anomalie. Il pèse 70 kg pour 1,65 m.
Un bilan biologique de routine récemment pratiqué est normal en dehors d'une glycémie à jeun à 1,60 g/l.

Parmi les éléments suivants en faveur d'un anévrysme anastomotique, un seul est constant et
justifie de penser systématiquement à ce diagnostic :
A - Présence de battements
B - Caractère indolore
C - Siège sur le trajet de l'axe artériel opéré
D - Apparition récente
E - Absence de retentissement en aval
Bonne(s) réponse(s) : C

Sans commentaire.

Indiquez l'examen complémentaire réalisable en consultation externe, peu coûteux et apportant


avec certitude le diagnostic :
A - Radiographie simple
B - Vélocimétrie Doppler
C - Echographie vasculaire
D - Thermographie
E - Angiographie numérisée
Bonne(s) réponse(s) : C

Sans commentaire.

Un tel anévrysme, petit, récent et asymtomatique peut être suivi de complication(s) grave(s).
Indiquez celle(s) qu'il faut redouter :
A - Compression veineuse fémorale
B - Thrombose d'une branche de la prothèse
C - Embolie distale
D - Suppuration de prothèse
E - Rupture
Bonne(s) réponse(s) : A B C E

Sans commentaire.

593
Exclusivement sur DOC - DZ : www.doc-dz.com NADJI 85
RESIDANAT EN POCHE TOME II
Cas Clinique en QCM

Parmi les propositions suivantes relatives à l'hyperglycémie constatée, lesquelles sont vraies ?
A - Le diabète ne sera affirmé avec certitude que sur le résultat d'une hyperglycémie provoquée
B - Ce trouble métaboloqie est un des facteurs de risque de l'évolution de l'artériopathie
C - L'existence d'une macroangiopathie impose la mise en route d'une insulinothérapie
immédiate
D - La réduction de la surcharge pondérale est le premier temps du traitement de ce trouble
métabolique
E - Le traitement précoce du diabète aurait probablement évité la consitution de l'anévrysme
anastomotique.
Bonne(s) réponse(s) : B D

A - Une glycémie à jeun suffit ici puisque les chiffres sont nettement supérieurs à ceux de la définition de
l'OMS (> 1,20 g/l).

Vis à vis de l'anévrysme, indiquez l'attitude que vous préconisez :


A - Angioplastie transluminale
B - Prescription d'antivitamines K pour prévenir un accident det hrombose
C - Surveillance bi annuelle par examen clinique et échographie
D - Traitement chirurgical de l'anévrysme
E - Embolisation de l'anévrysme
Bonne(s) réponse(s) : D

En raison du risque de complication.

Monsieur P. , 80 ans veuf, vivant seul est traité depuis un an pour une HTA modérée par l'association de
Sectral® (400 mg/j) et de Moduretic® (Hydrochlorothiazide + Amiloride) ( 1 comprimé/j).
Depuis une semaine, cet homme préalablement actif et en bon état physiologique se plaint d'une
symptomatologie d'effort associant fatigabilité et dyspnée.
Le médecin traitant consulté observe une bradycardie à 35/min et adresse le patient à l'hôpital. La pression
artérielle est de 210/90 mm Hg, la température à 37°C : l'auscultation cardiaque entend un souffle proto-
mésosystolique 3/10 au foyer aortique avec des bruits d'intensité normale ; il existe de discrets signes
d'insuffisance cardiaque congestive. La radiographie thoracique permet de mesurer un index cardio-
thoracique à 0,53 sans autre anomalie.
Un ECG est enregistré : les ondes P sont sinusales et régulières à 70/min ; le rythme ventriculaire est régulier
à 35/min avec des complexes QRS élargis (140 msec) présentant un aspect rSR en V1 et qRS en V6 ; les
ondes P qui précèdent les complexes QRS en sont séparées par un intervalle constant de 180 msec.
La fraction d'éjection ventriculaire gauche estimée par l'échocardiogramme bidimensionnel est de 0,60. Le
ionogramme sanguin et la créatinine sont normaux.

L'ECG permet de conclure à l'existence d'un :


A - Bloc sino-auriculaire
B - Bloc auriculo-ventriculaire du 1 er degré
C - Bloc aurico-ventriculaire du 2ème degré de type Luciani-Wenckebach
D - Bloc auriculo-ventriculaire du 2ème degré de type 2/1
E - Bloc auriculo-ventriculaire du 3ème degré
Bonne(s) réponse(s) : D

Une onde P sur deux est bloquée. Celle qui conduit le fait avec un espace PR constant.

Sachant qu'à l'exploration ECG endo-cavitaire, chaque onde P est suivie d'un potentiel hissien
avec espace AH normal, le siège anatomique du trouble de conduction est probablement :
A - La jonction sino-auriculaire
B - Le noeud d'Aschoff-Tawara (AT)
C - Le noeud d'AT et la branche droite du faisceau de His
D - Le noeud d'AT et la branche gauche du faisceau de His
E - Les deux branches du faisceau de His
Bonne(s) réponse(s) : E

Espace AH = temps de conduction intranodale.


Le trouble de conduction se situe en aval du tronc du faisceau de His puisque l'intervalle AH est normal.

594
Exclusivement sur DOC - DZ : www.doc-dz.com NADJI 85
RESIDANAT EN POCHE TOME II
Cas Clinique en QCM

L'étiologie probable du trouble de conduction est :


A - Un BAV congénital
B - Un rétrécissement aortique calcifié serré
C - Dégénérative (maladie de Lenègre)
D - Une myocardiopathie non obstructive
E - Iatrogène
Bonne(s) réponse(s) : C

Cause la plus fréquente chez les sujets de plus de 60 ans, dû à des lésions dégénératives du tissu de
conduction avec atteinte des branches du faisceau de His.

Quel(s) argument(s) vous incite(nt) à proposer l'implantation d'un stimulateur cardiaque définitif ?
A - La nécessité du traitement par le Sectral
B - Le fait que le patient vive seul
C - La bradycardie à 35/min
D - Le siège probable du trouble de conduction
E - La cause probable du trouble de conduction
Bonne(s) réponse(s) : C D E

A - Le Sectral déprime surtout la conduction nodale qui est ici normale.

Quel(s) effet(s) bénéfique(s) peut-on espérer de l'implantation d'un pace maker, si l'indication est
retenue ?
A - Régression des signes d'insuffisance cardiaque
B - Disparition ou amélioration de la symptomatologie d'effort
C - Disparition du souffle systolique
D - Réduction de la pression artérielle diastolique
E - Amélioration de la fraction d'éjection du ventricule gauche
Bonne(s) réponse(s) : A B C

C pouvait être dû à l'augmentation du volume d'éjection systolique lors de chaque contraction ventriculaire en
raison de la bradycardie.

Un patient de 38 ans a depuis 15 jours une fièvre à 38°C - 38°5C accompagnée de lombalgies et de sueurs.
20 jours auparavant, il avait subi une extraction dentaire sans antibioprophylaxie.
A l'examen, vous notez l'existence d'un souffle systolique intense au foyer aortique, déjà connu du patient qui
avait eu dans l'adolescence un R.A.A.

Parmi les signes suivants, tous sont en faveur d'une endocardite sauf un, lequel ?
A - Purpura pétéchial
B - Hématurie microscopique
C - Pseudo-panaris de la pulpe des doigts
D - Splénomégalie
E - Cyanose des extrémités
Bonne(s) réponse(s) : E

Sans commentaire.

Parmi les germes suivants, lequel est vraisemblablement en cause ?


A - Streptocoque bêta hémolytique A
B - Staphylocoque doré
C - Streptocoque viridans
D - Entérocoque
E - Pneumocoque
Bonne(s) réponse(s) : C

Les streptocoques responsables de la maladie d'Osler n'appartiennent pas au groupe A (responsable


d'infections aiguës de type septicémie, rarement d'endocardite).
C - Deux groupes selon qu'ils appartiennent ou non au groupe D,
a - groupables (groupeD) : entérocoques (streptocoque faecalis), non entérocoques (streptocoque bovis)
b - Non groupables.

595
Exclusivement sur DOC - DZ : www.doc-dz.com NADJI 85
RESIDANAT EN POCHE TOME II
Cas Clinique en QCM

Quels sont les examens qui vous permettront d'étayer solidement le diagnostic d'endocardite ?
A - Hémocultures
B - Echocardiographie
C - ECG
D - Radiographie pulmonaire
E - Recherche de complexes immuns circulants
Bonne(s) réponse(s) : A B

A et B sont évidemment les examens primordiaux à effectuer.


E - Est un stigmate immunologique de l'endocardite, éventuellement utile pour étayer le diagnostic.

Dans L'attente du résultat des examens réclamés, après vous être assuré de l'absence d'allergie
connue aux antibiotiques, quel traitement proposez-vous en première intention ?
A - Erythromycine : 1 g IV x 3
B - Péfloxacine : 400 mg per os x 2
C - Amoxicilline : 1 g per os x 3
D - Pénicilline G : 5.000.000 U en perfusion I.V. x 6 + Nétilmycine 2 mg/kg IM x 2
E - Vancomycine : 1 g IV x 2
Bonne(s) réponse(s) : D

Les streptocoques n'appartenant pas au groupe D sont les plus fréquents. La sensibilité à la pénicilline y est
constante et toujours excellente. L'association à un aminoside permet une action bactéricide synergique.

Quels sont les moyens d'assurer "le bon choix" du traitement antibiotique ?
A - Courbe thermique
B - Auscultation cardiaque et pulmonaire
C - V.S.
D - Echocardiographie
E - Pouvoir bactéricide du sérum
Bonne(s) réponse(s) : A E

Pouvoir bactéricide du sérum : on étudie l'effet bactéricide de différentes dilutions du sérum du malade sur la
souche préalablement isolée, et on détermine ainsi la dilution maximale du sérum capable d'exercer une
bactéricidie (destruction bactérienne).
Il y a une forte corrélation entre l'efficacité thérapeutique et un pouvoir bactéricide du sérum > ou = à la
dilution1/8.
Il est mesuré au moment du pic sérique de l'antibiotique et au moment du taux résiduel.

Quelles sont les complications à redouter en cours d'évolution ?


A - Oedème aigu du poumon
B - Hémorragie cérébro-méningée
C - Ischémie aiguë d'un membre
D - Embolie pulmonaire
E - Ictère
Bonne(s) réponse(s) : A B C

A - Principale complication, conséquence des dégâts valvulaires.


B - Par rupture d'anévrisme mycotique.
C - Les embolies des endocardites du coeur gauche peuvent toucher tous les viscères (embolies septiques de
fragments de végétation).

596
Exclusivement sur DOC - DZ : www.doc-dz.com NADJI 85
RESIDANAT EN POCHE TOME II
Cas Clinique en QCM
Un patient de 40 ans est hospitalisé pour des crises douloureuses constrictives rétrosternales de survenue
nocturne. Ces crises évoluent depuis 8 jours.
Elles apparaissent vers 3 heures du matin et durent 5 min environ. Aucune crise ne survient pendant la
journée. Ce sujet est un gros fumeur habituel à 25 cigarettes par jour. L'examen clinique cardiovasculaire est
normal, la TA est à 140/80 mm Hg. L'ECG est normal.
La possibilité d'un angor spastique est évoquée.

Parmi les examens complémentaires suivants, quel est le plus approprié pour confirmer
l'hypothèse diagnostique ?
A - Enregistrement Holter des 24 H
B - Epreuve d'effort sur cyclo-ergomètre
C - Echo doppler cardiaque
D - Scintigraphie myocardique
E - Coronarographie
Bonne(s) réponse(s) : A

Peut confirmer le diagnostic en montrant un sus-décalage du segment ST contemporain de la douleur compte-


tenu de la fréquence des crises.

Au cours d'une crise douloureuse, un ECG est enregistré. Parmi les anomalies suivantes
enregistrées l'une ne fait pas partie des signes classiques. Laquelle ?
A - Sus-décalage du segment ST de 5 mm
B - Une onde delta de préexcitation intermittente
C - Extrasystolie ventriculaire
D - BAV transitoire
E - Augmentation d'amplitude de l'onde R avec courant de lésion sous-épicardique
Bonne(s) réponse(s) : B

B - Se voit dans les syndrômes de Wolf Parkinson White. A l'acmé de la crise peuvent se voir aussi une
fibrillation ventriculaire, une tachycardie ventriculaire ou fonctionnelle, une dysfonction sinusale.

La coronarographie pourrait montrer les éléments suivants :


A - Coronaires normales
B - Une plaque athéromateuse sur la coronaire droite
C - Une sténose proximale sur la coronaire droite
D - Une sténose serrée sur la coronaire droite et l'IVA
E - Une sténose des trois troncs principaux coronariens
Bonne(s) réponse(s) : A B C D E

Le spasme peut survenir sur des artères coronaires normales ou sur une lésion athéromateuse préexistante.

Le traitement médical peut faire appel à :


A - Le propranolol
B - La nifédipine
C - L'isosorbide dinitrate (Risordan®)
D - La molsidomine (Corvasal®)
E - La méthylergométrine (Methergin®)
Bonne(s) réponse(s) : B C D

A - A éviter car il peut aggraver le spasme.


B C D - Sont des vasodilatateurs coronaires pouvant prévenir ou lever le spasme.
E - Est contre-indiqué car il est utilisé pour déclencher le spasme.

L'évolution peut être marquée par :


A - Une syncope
B - Une aggravation de l'angor
C - Un infarctus du myocarde
D - Une mort subite
E - Une rémission durable
Bonne(s) réponse(s) : A B C D E

A et D par trouble du rythme.

597
Exclusivement sur DOC - DZ : www.doc-dz.com NADJI 85
RESIDANAT EN POCHE TOME II
Cas Clinique en QCM
Un patient de 40 ans, porteur d'une insuffisance aortique connue a subi une extraction dentaire trois semaines
auparavant. Sont apparus depuis quelques jours une asthénie, une fièvre persistante ainsi qu'une majoration
des signes fonctionnels de la valvulopathie. Hospitalisé d'urgence, ce patient a subi des examens biologiques
qui ont retrouvé une hyperleucocytose avec polynucléose, une accélération de la vitesse de sédimentation et
surtout un streptocoque à l'examen des hémocultures. Le diagnostic d'endocardite bactérienne ainsi confirmé,
une polyantibiothérapie a été instaurée qui sera adaptée selon les résultats de l'antibiogramme.

Lorsqu'une porte d'entrée est mise en évidence, quel est le pourcentage d'une étiologie bucco-
dentaire à l'endocardite bactérienne ?
A - 10 %
B - 30 %
C - 50 %
D - 70 %
E - 90 %
Bonne(s) réponse(s) :

QUESTION ANNULEE.

Dans la maladie d'Osler, quel est le mécanisme physiopathologique incriminé dans l'étiologie
bucco-dentaire ?
A - Théorie toxinique
B - Théorie bactériémique
C - Théorie nerveuse
D - Théorie allergique bactérienne
E - Aucune des propositions citées
Bonne(s) réponse(s) : B

La greffe bactérienne sur l'endocarde se fait en règle au cours d'une bactériémie.

Parmi les affections suivantes, quelles sont celles pouvant être en rapport avec une infection
focale dentaire ?
A - Asthme-bronchique
B - Rhumatisme articulaire aigu
C - Uvénites chroniques
D - Hypodermites nodulaires
E - Glomérulopathie chronique
Bonne(s) réponse(s) :

QUESTION ANNULEE.

Quelles sont, parmi les lésions dentaires suivantes, celles qui imposent l'extraction de la dent
causale ?
A - Carie non pénétrante
B - Granulome apico-dentaire
C - Dent en désinclusion
D - Dent mortifiée
E - Alvéolyse prononcée
Bonne(s) réponse(s) : B C D E

Sans commentaire.

Quelles sont les mesures indispensables à prendre chez un patient porteur d'une valvulopathie
avant d'envisager un geste chirurgical bucco-dentaire ?
A - Interrogatoire du patient
B - Radiographie pulmonaire
C - Prescription de bains de bouche
D - Couverture antibiotique
E - Hospitalisation
Bonne(s) réponse(s) : A D

Sans commentaire.

598
Exclusivement sur DOC - DZ : www.doc-dz.com NADJI 85
RESIDANAT EN POCHE TOME II
Cas Clinique en QCM

Quelles affections cardio-vasculaires peuvent-elles être considérées comme ne présentant


aucune contre-indication bactérienne aux extractions dentaires ?
A - Artérite des membres inférieurs
B - Maladie mitrale
C - Insuffisance ventriculaire gauche
D - Cardiomyopathie non obstructive
E - Tétralogie de Fallot
Bonne(s) réponse(s) :

QUESTION ANNULEE

599
Exclusivement sur DOC - DZ : www.doc-dz.com NADJI 85
RESIDANAT EN POCHE TOME II
Cas Clinique en QCM

600
Exclusivement sur DOC - DZ : www.doc-dz.com NADJI 85
RESIDANAT EN POCHE TOME II
Cas Clinique en QCM
Une femme de 62 ans consulte pour les signes suivants, apparus progressivement au cours des 6 derniers mois : asthénie,
apathie, frilosité, crampes musculaires, hypoacousie, et prise de poids de 6 kg. Le visage est arrondi, la peau est froide, les
aisselles sont sèches, le rythme cardiaque régulier à 64/mn ; absence d'oedème malléolaire. La patiente, dont la voix est
rauque et lente, vous apprend qu'elle a été traitée pendant 4 ans par isosorbide dinitrate (Risordan®) et par Cordarone®
(amiodarone) pour une angine de poitrine, mais qu'elle n'a plus eu de crise depuis près d'un an et que ce traitement a été
alors interrompu. Il n'existe pas de goître. Au terme de cet examen, le diagnostic d'hypothyroïdie est hautement probable.

Le tableau clinique et l'anamnèse permettent d'évoquer une hypothyroïdie :


A - Périphérique idiopathique (involution simple de la thyroïde)
B - Centrale (hypothalamo-hypophysaire)
C - Périphérique iatrogénique
D - Par ectopie thyroïdienne
E - Secondaire à une thyroïdite chronique de Hashimoto
Bonne(s) Réponse(s) : C

L'élément évocateur d'une origine iatrogène est la prise de Cordarone® (permet d'évoquer le diagnostic mais non de le
confirmer).

Pour compléter votre information sur cette patiente, vous demandez :


A - Une cholestérolémie
B - Un ECG
C - Une T4 libre plasmatique
D - Une TSH plasmatique
E - Une urographie intraveineuse
Bonne(s) Réponse(s) : A B C D

La cholestérolémie est élevée en cas d'hypothyroïdie. L'ECG sert de référence ; avant le traitement il met souvent en évidence
une bradycardie sinusale, et apprécie le retentissement cardiaque.

Il pourra être utile de compléter ces examens par :


A - Une échocardiographie
B - Une hypoglycémie provoquée par l'insuline
C - Une épreuve de restriction hydrique
D - Un test à la LH-RH
E - Un dosage de la créatine phosphokinase (CPK) sérique
Bonne(s) Réponse(s) : A E

L'échographie recherche une éventuelle péricardite.


CPK : peuvent être élevés en cas d'atteinte musculaire associée à l'hypothyroïdie. Chez cette patiente coronarienne, il peut
être intéressant de demander les CPK-MM (musculaires) et les CPK-MB (myocardiques) en cas de doute.

Le traitement initial de cette patiente va faire appel :


A - A la Thyroxine® par voie IV
B - Au Cynomel® à la dose initiale de 75 microg/jour
C - A la reprise de la Cordarone® associée à 5 gouttes/jour de Thyroxine®
D - Au Lévothyrox® à la dose initiale de 12,5 microg/jour associé à un bêtabloquant en l'absence de contre-
indication de celui-ci
E - Au Lévothyrox® à la dose initiale de 100 microg/jour
Bonne(s) Réponse(s) : D

Le traitement sera débuté, à faible posologie, en milieu hospitalier chez cette patiente coronarienne, en augmentant très
progressivement les doses avec surveillance clinique et ECG.
1 comprimé Levothyrox® = 50 g dose de départ = 1/4 de comprimé.

L'attitude vis-à-vis de cette patiente au cours des 4 semaines suivantes va consister à :


A - Surveiller quotidiennement la fréquence cardiaque
B - Augmenter rapidement l'hormonothérapie supplétive
C - Effectuer des paliers posologiques d'au moins 10 jours de l'hormonothérapie supplétive
D - Pratiquer une surveillance électrocardiographique
E - Doser la TSH, le cholestérol et la CPK sériques après 4 semaines de traitement
Bonne(s) Réponse(s) : A C D E

Sans commentaire.

601
Exclusivement sur DOC - DZ : www.doc-dz.com NADJI 85
RESIDANAT EN POCHE TOME II
Cas Clinique en QCM
Une jeune fille de 24 ans, diabétique insulino-dépendante, est hospitalisée en urgence pour syndrome abdominal aigu.
Quelques jours avant l'hospitalisation, elle s'est plainte de brûlures mictionnelles. Des vomissements, une dyspnée
progressive et des douleurs abdominales sont apparues la veille de l'entrée. Elle n'a pas fait son insuline du soir et n'a pas
mangé. La température est à 39 degrés avec frissons. Le ventre est douloureux dans son ensemble, mais souple. Le pouls
est rapide à 120/min. Le coeur est normal à l'auscultation et la TA est à 80/50 mm de Hg. La malade est stuporeuse et
globalement déshydratée. Sur le plan biologique :
- pH : 7,10 (N = 7,35-7,45)
- bicarbonates : 7 mmol (N = 25 + 5)
- sérum lactescent et triglycérides : 20 g/l (N = 0,75-1,5)
- glycémie : 6 g/l (N = 0,8-1,1 ) ou 33,3 mmol/l
- glycosurie : + + + ; cétonurie : + + +
- ionogramme : Na = 128 mmol/l, Cl = 90 mmol/l
- urée : 1 g/l ou 16,6 mmol/l, créatininémie : 220 micromol/l
- leucocytose : 20 000 dont 87 % de polynucléaires neutrophiles
- culot urinaire : pyurie franche.

Quel(s) diagnostic(s) retenez-vous chez cette malade ?


A - Appendicite aiguë
B - Acido-cétose diabétique
C - Coma hyperosmolaire
D - Pyélonéphrite
E - Cystite
Bonne(s) Réponse(s) : B D

Le diagnostic probable est celui d'une acido-cétose diabétique survenue après une pyélonéphrite.

Parmi les examens suivants, non encore réalisés chez cette malade, quel(s) est(sont) celui(ceux) qui doi(ven)t
être impérativement fait(s) en urgence ?
A - Cholestérol
B - Hémocultures
C - Uricémie
D - Bactériologie des urines
E - Kaliémie
Bonne(s) Réponse(s) : B D E

B - Hémocultures recherchant une septicémie.


D - L'ECBU sera pratiqué avant le début de l'antibiothérapie.
E - La kaliémie permet d'ajuster la rééquilibration hydroélectrolytique, en sachant qu'il peut exister une fausse hyperkaliémie
chez un patient en acidose. L'ECG est donc important pour apprécier la kaliémie.

Le coma diabétique peut comporter un ou plusieurs des signes neuro-psychiques suivants :


A - Coma progressif
B - Agitation psychomotrice
C - Abolition des réflexes ostéo-tendineux
D - Signes d'irritation pyramidale
E - Crises convulsives
Bonne(s) Réponse(s) : A C

La question est mal posée. On ne sait de quel type de coma il s'agit. Vu le cas clinique, on considérera qu'il s'agit d'un coma
acidocétosique.

Chez cette malade, l'hyponatrémie peut être en rapport avec un ou plusieurs des désordres biologiques
suivants :
A - Hypercréatininémie
B - Hypertriglycéridémie
C - Hyperleucocytose
D - Hyperazotémie
E - Hyperglycémie
Bonne(s) Réponse(s) : B E

Ce sont deux causes de fausse hyponatrémie. La natrémie peut sembler basse si le plasma contient une grande quantité de
substances qui prennent la place de l'eau (car les concentrations en Na+ sont exprimées en mmol/litre de plasma).

602
Exclusivement sur DOC - DZ : www.doc-dz.com NADJI 85
RESIDANAT EN POCHE TOME II
Cas Clinique en QCM

La carence en insuline peut avoir les conséquences suivantes, sauf une. Laquelle ?
A - Diminution de la captation cellulaire du glucose
B - Augmentation de la glycogénolyse
C - Diminution de la néoglucogenèse
D - Augmentation de la lipolyse
E - Augmentation du catabolisme azoté
Bonne(s) Réponse(s) : C

Sans commentaire

Dans le traitement de l'acido-cétose diabétique, une ou plusieurs des attitudes thérapeutiques suivantes sont
inadéquates :
A - Administration de sérum-albumine diluée si la TA est basse
B - Administration de vaso-presseurs en cas de collapsus cardiovasculalre
C - Réhydratation par sérum salé isotonique
D - Administration systématique de sérum bicarbonaté isotonique avant tout contrôle biologique
E - Infusion continue d'insuline à action rapide par seringue auto-pulsée à raison de 5 à 10 UI/heure
Bonne(s) Réponse(s) : A B D

Sans commentaire.

Une femme de 55 ans consulte pour asthénie et douleurs rhumatismales diffuses. Dans ses antécédents, on relève 6
grossesses, la dernière survenue à 33 ans. Aménorrhée depuis l'âge de 38 ans, sans bouffées de chaleur. Céphalées traitées
sans succès par diverses médications symptomatiques depuis 3 ans. Intervention bilatérale pour décompression du nerf
médian au canal carpien 2 ans avant la consultation. Douleurs rhumatismales traitées par indométhacine depuis 4 ans,
hyperglycémie détectée un an auparavant, aucun traitement particulier n'ayant été prescrit. A l'examen, malade en surcharge
pondérale (83 kg pour 1m79) avec syndrome dysmorphique associant un prognatisme majeur, une augmentation de taille des
mains et des pieds (l'alliance a été élargie à 2 reprises en 5 ans). On note la présence de nombreuses vergetures non
pigmentées et d'une discrète hypertrichose.
La TA est de 18/10.

Quel(s) élément(s) de l'anamnèse est(sont) en faveur d'une acromégalie ?


A - La grande taille
B - Le syndrome du canal carpien
C - La multiparité
D - L'hyperglycémie
E - Les céphalées
Bonne(s) Réponse(s) : B D E

Sans commentaire.

Quel(s) signe(s) clinique(s) viendra (viendront) renforcer la suspicion d'acromégalie ?


A - Une galactorrhée
B - Une hémianopsie latérale homonyme
C - Une cyphose
D - Des vergetures
E - Une voix bitonale
Bonne(s) Réponse(s) : A C

A - En cas d'hyperprolactinémie de déconnexion hypothalamohypophysaire (voir plus loin).


C - Aspect au maximum en "polichinelle", il faut, bien entendu, préciser le caractère acquis.de ces troubles.

Quel(s) signe(s) radiologique(s) vous permettra(permettront) d'étayer le diagnostic d'acromégalie ?


A - Une dysgénésie épiphysaire
B - Une hypertrophie des sinus frontaux
C - Une déformation de la selle turcique
D - Des calcifications des noyaux gris centraux
E - Tous les signes précédents
Bonne(s) Réponse(s) : B C

La radiographie de la selle turcique mettra en évidence :


- des éléments en faveur de la tumeur de la selle turcique
- des éléments en faveur de l'acromégalie.

603
Exclusivement sur DOC - DZ : www.doc-dz.com NADJI 85
RESIDANAT EN POCHE TOME II
Cas Clinique en QCM

Parmi ces signes biologiques, le(s)quel(s) est(sont) compatible(s) avec le diagnostic d'acromégalie ?
A - Un hyperprolactinisme
B - Une hypophosphorémie
C - Un taux d'hormone somatotrope (GH) s'élevant après injection Intraveineuse de 200 microg de TRH
D - Une baisse d'hormones thyroïdiennes fT3 et fT4 avec TSH élevée
E - Une absence de freinage de la GH au cours de l'hyperglycémie provoquée
Bonne(s) Réponse(s) : A C E

L'hyperprolactinémie peut avoir deux origines :


- par déconnexion hypothalamohypophysaire (en cas d'adénome comprimant la tige pituitaire et empêchant le passage du
PIF), levant donc le tonus inhibiteur normalement exercé par le PIF sur les cellules à prolactine
- par adénome mixte (sécrétion de GH et de prolactine).

Quelle(s) maladie(s) générale(s) peut(peuvent) être responsable(s) de l'apparition d'un syndrome du canal
carpien bilatéral ?
A - Le diabète
B - La maladie d'Addison
C - Le saturnisme
D - La porphyrie aiguë intermittente
E - L'hypothyroïdie
Bonne(s) Réponse(s) : E

Sans commentaire.

La patiente a été opérée d'un macroadénome hypophysaire, et une insuffisance surrénalienne d'origine centrale
se démasque dans les suites de l'intervention. Quelle(s) mesure(s) thérapeutique(s) envisagez-vous ?
A - Un régime désodé
B - Prednisolone, 4 cp à 5 mg par jour
C - Hydrocortisone, 3 cp à 10 mg par jour
D - Fludrocortisone, 1 cp à 50 microg par jour
E - L'association de toutes ces mesures
Bonne(s) Réponse(s) : C

Le régime désodé est formellement contre-indiqué chez les insuffisants surrénaliens. Les minéralocorticoïdes sont
nécessaires en cas d'insuffisance surrénalienne d'origine basse, mais non en cas d'insuffisance corticotrope.

Vous revoyez la patiente 6 mois après l'intervention neurochirurgicale : quel(s) élément(s) du bilan biologique de
contrôle est(sont) en faveur d'une acromégalie toujours évolutive ?
A - Le taux de base de l'hormone somatotrope est normal
B - La phosphorémie est élevée
C - Le taux d hormone somatotrope (GH) s'élève après injection de TRH
D - Le taux d hormone somatotrope (GH) s'élève sous bromocriptine (2 cp à 2,5 mg per os)
E - Le taux de base de la prolactine est normal
Bonne(s) Réponse(s) : B C

Sans commentaire.

Une jeune femme de 24 ans nullipare, sans antécédent pathologique, consulte pour aménorrhée secondaire révélée à l'arrêt
de la prise d'oestroprogestatifs 3 mois auparavant. L'examen clinique est normal, y compris le toucher vaginal. Il n'y a pas de
galactorrhée.

Quel diagnostic peut-on exclure ?


A - Grossesse
B - Ménopause précoce
C - Adénome à prolactine
D - Anovulation fonctionnelle
E - Impubérisme
Bonne(s) Réponse(s) : E

L'impubérisme est exclu, car qu'il s'agit d'une aménorrhée secondaire.

604
Exclusivement sur DOC - DZ : www.doc-dz.com NADJI 85
RESIDANAT EN POCHE TOME II
Cas Clinique en QCM

Quel est l'examen complémentaire à demander en premier lieu ?


A - Dosage de prolactine
B - Test de grossesse
C - Dosage de LH
D - Radiographie du crâne
E - Hystérographie
Bonne(s) réponse(s) : D

Une seule réponse est demandée : test de grossesse. Mais les deux diagnostics à évoquer en premier dans ce contexte sont
la grossesse et l'hyperprolactinémie.

Un dosage de gonadotrophines a été pratiqué. FSH et LH plasmatique sont basses. Ces résultats sont
compatibles avec :
A - Une maladie hypothalamique
B - Un adénome à prolactine
C - Une carence ovarienne
D - Un syndrome des ovaires polykystiques
E - Aucune de ces propositions
Bonne(s) Réponse(s) : A

Sans commentaire.

Si la prolactinémie est élevée, on doit rechercher comme étiologie :


A - Une insuffisance antéhypophysaire
B - Une hypothyroïdie
C - Une prise médicamenteuse
D - Une tumeur de l'hypophyse
E - Un craniopharyngiome
Bonne(s) Réponse(s) :B C D E

Il existe des cas d'insuffisance antéhypohysaire associé à une hyperprolactinémie (ex: macroadénome non sécrétant), mais,
dans ce cas, l'examen clinique ne serait pas normal.

Parmi les médicaments éventuellement responsables d'une hyperprolactinémie, on connait :


A - Le Largactil® (chlorpromazine)
B - Le Primperan® (métoclopramide)
C - Le Parlodel® (bromocriptine)
D - Le Dogmatil® sulpiride)
E - Le Tagamet® (cimétidine)
Bonne(s) Réponse(s) : A B D E

De nombreux médicaments sont responsables d'une hyperprolactinémie, d'où la nécessité de toujours pratiquer un
interrogatoire précis pour s'enquérir des médicaments absorbés. Le Parlodel® est par contre le traitement médical de premier
choix d'une hyperprolactinémie.

Un homme de 35 ans, ouvrier d'usine, consulte pour une sensation de peau cartonnée au niveau des mains, des céphalées,
des sueurs diffuses sans hyperthermie. Il signale qu'il ne peut plus retirer son alliance et qu'il a changé de pointure de
chaussures (pointure supérieure). Il a des troubles visuels.
A l'examen, on note un nez et des traits épais, une tension artérielle à 16-10. La glycémie est 7,8 mmol/l (1,40 g/l), calcémie :
2,55 mmol/l (102 mg/l), phosphorémie : 1,5 mmol/l (52 mg/l), calciurie : 10 mmol/24 h (420 mg/24), cholestérol : 5,7 mmol/l
(2,2 g/l), triglycérides : 2,85 mmol/l (2,5 g/l).

Quelle est l'hormone, anormalement élevée, responsable de ces signes :


A - Prolactine
B - ACTH
C - Hormone de croissance
D - FSH
E - TSH
Bonne(s) Réponse(s) : C

Tableau en faveur d'une acromégalie.

605
Exclusivement sur DOC - DZ : www.doc-dz.com NADJI 85
RESIDANAT EN POCHE TOME II
Cas Clinique en QCM

Citer le test dynamique permettant d'affirmer cette maladie :


A - Dosage de l'hormone le lendemain de la prise de 2 cp de dexaméthasone
B - Dosage après injection IV de 5 mg de Régitine®
C - Dosage après perfusion de 2 litres de sérum physiologique
D - Dosage 3 jours après la prise de 6 comp/j de T3 (Cynomel®)
E - Aucun de ces tests
Bonne(s) Réponse(s) : E

L'absence de freinage de la GH sous HGPO est actuellement le test le plus fiable.

Préciser le paramètre biologique le mieux corrélé au taux de l'hormone augmentée dans cette maladie :
A - Glycémie
B - Calcémie
C - Calciurie
D - Phosphorémie
E - Cholestérolémie
Bonne(s) Réponse(s) : C

Mais la phosphorémie est aussi un bon élément.

Ce malade pourrait éventuellement présenter une modification du champ visuel ; laquelle ?


A - Baisse unilatérale de l'acuité visuelle
B - Hémianopsie unilatérale homonyme
C - Hémianopsie binasale
D - Hémianopsie bitemporale
E - Rétrécissement concentrique bilatéral du champ visuel
Bonne(s) Réponse(s) : D

Ce trouble visuel est le plus classique, lié à une compression du chiasma optique par tumeur, mais de nombreux autres
troubles visuels peuvent se rencontrer.

Les radiographies du crâne vont vraisemblablement montrer :


A - Epaississement de la voûte du crâne
B - Calcifications intra et suprasellaires
C - Atrophie des sinus frontaux
D - Augmentation de volume de la selle turcique
E - Ouverture de l'angle de la mâchoire inférieure
Bonne(s) Réponse(s) : A D E

L'acromégalie est souvent associée à un macroadénome, expliquant ainsi l'augmentation fréquente de volume de la selle
turcique.

Quel est le diagnostic ?


A - Hyperprolactinémie
B - Syndrome de Cushing
C - Acromégalie
D - Syndrome de Sheehan
E - Aucun de ces diagnostics
Bonne(s) Réponse(s) : C

Sans commentaire.

L'hypersécrétion hormonale est vraisemblablement causée par :


A - Adénome hypophysaire
B - Craniopharyngiome
C - Gliome du chiasma optique
D - Anévrisme de la carotide
E - Séquelle de méningite tuberculeuse
Bonne(s) Réponse(s) : A

Sans commentaire.

606
Exclusivement sur DOC - DZ : www.doc-dz.com NADJI 85
RESIDANAT EN POCHE TOME II
Cas Clinique en QCM
Monsieur M. 25 ans, est adressé à l'accueil du centre hospitalier en coma de stade 1. Le mot du médecin précise qu'il est
soigné pour diabète insulinodépendant depuis l'âge de 10 ans et que, depuis quelque temps il a relâché la surveillance de sa
maladie et l'adaptation du traitement. Depuis quelques jours il se disait très fatigué et au cours des dernières 24 heures sont
apparues des douleurs abdominales, des nausées et des vomissements. A l'admission, l'examen clinique met en évidence,
outre le coma stade 1, sans signes neurologiques focalisés, une réaction assez vive à la palpation de l'abdomen, une tension
artérielle à 6 de maxima, une tachycardie à 140/mn, une sécheresse des téguments, un pli cutané persistant, une respiration
ample et rapide (34/minute), une odeur "pomme reinette" de l'haleine. Examens complémentaires : dans le sérum, glycémie
20 mmol/l, Na : 137 mmol/l, K : 4,2 mmol/l, pH : 7,15, C03H : 15 mmol/l, protides : 85 g/l, triglycérides : 4,8 mmol/l, créatinine :
170 micromol/l, dans les urines : sucres et acétones +++.

Indiquez quel(s) est(sont) parmi les notions ci-après celle(s) qui s'inscrit (s'inscrivent) contre l'hypothèse d'un
coma hyperosmolaire. :
A - Présence de cétonurie + + +
B - Tension artérielle basse
C - Existence d'une acidose (sans considérer son degré)
D - Natrémie à 13 mmol/l + glycémie à 20 mmol/l
E - Protidémie élevée
Bonne(s) Réponse(s) : A D

Il peut, dans certains cas de coma hyperosmolaire, exister une petite cétonurie (se limitant à traces ou +).
De même, possibilité d'une petite acidose, mais mineure.
La glycémie est généralement supérieure à 20 mmol/litre.

Les douleurs abdominales, les nausées et la vomissements peuvent avoir pour explication :
A - Une poussée de la pancréatite qui pourrait être responsable du diabète
B - L'hypertriglycéridémie
C - Une hyperhydratation cellulaire
D - Une affection abdominale chirurgicale (appendicite, cholécystite, etc..) responsable de la décompensation
du diabète
E - La céto-acidose diabétique elle-même
Bonne(s) Réponse(s) : D E

La cause la plus probable est cependant l'acidocétose diabétique en elle-même (mais complément multiple, mettre les deux
réponses).

Parmi les signes suivants, lequel vous paraît pouvoir donner la meilleure appréciation quantitative de l'acidose
et de son évolution sous traitement ?
A - La fréquence des vomissements
B - La fréquence respiratoire
C - La fréquence cardiaque
D - La tension artérielle
E - L'odeur de l'haleine
Bonne(s) Réponse(s) : B

La polydipsie est un très bon signe d'acidose dans ce contexte.

Parmi les solutés suivants, quels sont les deux qui vous paraissent le plus adaptés à la situation de ce patient,
comme perfusion initiale ?
A - Solution de mannitol
B - Sérum glucosé hypotonique à 25 % (sans ions)
C - Sérum bicarbonaté isotonique
D - Sérum salé isotonique
E - Sérum salé hypotonique
Bonne(s) Réponse(s) : C D

(car nécessité de deux réponses). L'indication de sérum bicarbonaté isotonique étant limite dans cette indication ; en effet, en
cas d'acidose diabétique ; celui-ci est généralement préconisé pour une acidose sévère avec pH< 7,10 et bicarbonates
effondrés.

607
Exclusivement sur DOC - DZ : www.doc-dz.com NADJI 85
RESIDANAT EN POCHE TOME II
Cas Clinique en QCM

Parmi les mesures thérapeutiques suivantes, laquelle(lesquelles) vous parait (paraissent) adaptée(s) chez ce
patient ?
A - Administration d'insuline retard par voie veineuse toutes les 10 heures
B - L'infusion continue IV d'insuline ordinaire
C - Injection d'insuline ordinaire par voie intramusculaire toutes les heures
D - Injection d'insuline ordinaire par voie intramusculaire toutes les 4 heures
E - Injection sous cutanée d'insuline ordinaire toutes les huit heures
Bonne(s) Réponse(s) : B C

Certains préconisent des injections IM toutes les heures de petites doses d'insuline, mais cette méthode me semble moins
intéressante.

Monsieur M.., âgé de 75 ans, présente un diabète non insulino-dépendant depuis 15 ans, traité par 3 comprimés de Daonil®
par jour. Ce diabète est compliqué d'une insuffisance rénale modérée (créatininémie à 150 micromol/l). Il est hospitalisé en
urgence à 3h30 du matin pour un coma précédé d'un épisode convulsif. Ce tableau s'est constitué vers 3 h du matin. A
l'examen, le malade présente un coma profond avec mydriase bilatérale, Babinski bilatéral, et hémiplégie droite. Il est couvert
de sueurs. La nuque est souple, la tension artérielle est à 160/80, le pouls est à 90 pulsations minute.

Parmi les examens suivants, lequel demandez-vous en priorité ?


A - Une ponction lombaire
B - Une recherche de la glycosurie
C - Une glycémie
D - Un électroencéphalogramme
E - Une radiographie du crâne
Bonne(s) Réponse(s) : C

Faire une glycémie capillaire et prélever glycémie Labo (mais ne pas attendre les résultats de la glycémie Labo pour débuter
le resucrage).

Dans un premier temps, quelle thérapeutique urgente mettez-vous en route chez ce patient ?
A - Une injection de 20 unités d'insuline ordinaire en sous-cutanée
B - Une perfusion de 500 cc de sérum glucosé à 5 % (5)
C - Une injection de 20 cc de sérum glucosé à 30 % en intraveineuse
D - Une injection intramusculaire d'une ampoule de Catapressan®
E - Une héparinothérapie
Bonne(s) Réponse(s) : C

Tout coma chez un diabétique doit être considéré comme un coma hypoglycémique jusqu'à preuve du contraire et doit donc
nécessiter une injection de glucose hypertonique en IV.

Chez ce patient quel est le traitement de fond formellement contre-indiqué ?


A - L'insuline Retard
B - Les Sulfamides Hypoglycémiants
C - Les Biguanides
D - L'insuline ordinaire
E - Aucun de ces traitements
Bonne(s) Réponse(s) : C

Les biguanides sont formellement contre-indiqués en raison de l'âge et de l'insuffisance rénale (risque d'acidose lactique).
Cependant. les sulfamides hypoglycémiants ne sont pas particulièrement recommandés en cas d'insuffisance rénale.

Chez le diabétique présentant une insuffisance rénale, le seuil rénal du glucose :


A - Reste normal
B - S'élève
C - S'abaisse
D - Varie selon l'insulinémie
E - Varie selon les modalités du régime
Bonne(s) Réponse(s) : B

Donc :
- la glycosurie est un mauvais paramètre de surveillance
- apprendre l'autosurveillance par glycémie capillaire.

608
Exclusivement sur DOC - DZ : www.doc-dz.com NADJI 85
RESIDANAT EN POCHE TOME II
Cas Clinique en QCM

Une hypoglycémie isolée pourrait entraîner :


A - Convulsions
B - Coma
C - Babinski bilatéral
D - Hémiplégie
E - Sueurs
Bonne(s) Réponse(s) : A B C D E

L'hyperglycémie peut donner un tableau très polymorphe et notamment se présenter sous un tableau neurologique ou
psychiatrique.

Une femme de 30 ans est hospitalisée en raison de la survenue de malaises répétés associant céphalées avec vertiges, des
douleurs violentes, abdominales et thoraciques, des crampes musculaires. Ces malaises durent quelques minutes,
s'accompagnent de sueurs abondantes, de pâleur généralisée, et s'atténuent progressivement. La mesure de la pression
artérielle au cours d'un malaise retrouve des chiffres à 250/130 alors que la pression est à 130/70 en dehors des malaises. Le
diagnostic de phéochromocytome est évoqué.

Parmi les éléments suivants du tableau clinique indiquez celui(ceux) qui est(sont) particulièrement évocateur(s)
du phéochromocytome ?
A - Jeune âge de la malade
B - Céphalées violentes
C - Accès de faiblesse musculaire des membres inférieurs
D - Phénomènes de sudation abondante et généralisée
E - Accès de vertiges
Bonne(s) Réponse(s) : B D

Ces crises typiques sont présentes dans 30 à 50 % des cas. Le jeune âge du malade est évocateur, mais peut-on dire que
c'est un élément particulièrement évocateur

Un bilan biologique est effectué au décours immédiat d'un malaise. Indiquez parmi les anomalies suivantes
celle(s) dont la découverte est compatible avec le diagnostic :
A - Hypokaliémie
B - Hyperglycémie
C - Acidose métabolique
D - Elévation de l'hématocrite
E - Hyperleucocytose
Bonne(s) Réponse(s) : B D E

Il est responsable d'une hyperleucocytose à polynucléaires.

Parmi les examens suivants indiquez celui qui, au moindre risque, a les plus grandes chances de préciser le
siège exact de la lésion :
A - Urographie avec néphrotomographie
B - Aortographie abdominale et thoracique
C - Scintigraphie à l'iodocholestérol
D - Tomodensitométrie abdominale et thoracique
E - Cathétérisme veineux cave avec dosages étagés de catécholamines
Bonne(s) Réponse(s) : D

Sans commentaire.

Les examens effectués ont précisé qu'il s'agissait d'un phéochromocytome de l'organe de Zuckerkandi. Où se
trouve-t-il situé ?
A - Dans la région pré-coccygienne
B - Au niveau du pédicule rénal
C - Au niveau de la terminaison aortique
D - Dans le médiastin postérieur
E - Au niveau de la bifurcation carotidienne
Bonne(s) Réponse(s) : C

Sans commentaire

609
Exclusivement sur DOC - DZ : www.doc-dz.com NADJI 85
RESIDANAT EN POCHE TOME II
Cas Clinique en QCM

Vous recherchez une atteinte associée. Parmi les suivantes, quelles sont les 2 qui sont fréquemment associées
au phéochromocytome ?
A - Adénome hépatique
B - Adénome antéhypophysaire
C - Adénome thyroïdien
D - Tumeur des ilôts de Langerhans du pancréas
E - Cancer vésiculaire du corps thyroïde
Bonne(s) Réponse(s) : B D

Ces tumeurs dérivent toutes du système APUD.

L'exérèse chirurgicale d'un phéochromocytome peut être compliquée par un ou plusieurs des incidents per-
opératoires suivants ; indiquez le(s)quel(s) :
A - Troubles du rythme cardiaque
B - Phénomènes de bronchospasme
C - Poussées hypertensives
D - Collapsus vasculaire
E - Accidents d'hypoglycémie
Bonne(s) Réponse(s) : A C D

L'intervention sur un phéochromocytome, très délicate, doit être effectuée dans un service spécialisé (préparation
préopératoire - précautions anesthésiques - réanimation postopératoire).

Une femme de 38 ans, mère de 2 enfants consulte pour obésité : 105 kg pour 153 cm et pour un diabète dont le diagnostic a
été porté sur une glycémie à jeûn du plasma veineux à 7,10 mmol/l un jour, 6,90mmol un autre jour. Vous faites une épreuve
d'hyperglycémie provoquée par voie orale (H.G.P.O.) qui donne les chiffres suivants : avant la prise de glucose : 6.85 mmol/l,
après une heure : 12,30 mmol/l, après 2 heures : 11,50 mmol/l. Un examen clinique est normal en dehors d'une surcharge
graisseuse diffuse et de la constatation de quelques vergetures pourpres au niveau de l'abdomen. L'enquête alimentaire
montre une prise quotidienne de 6477 kilojoules (1540 kilocalories) en 2 repas (12 h et 19 h) ; les glucides représentent 48 %
de l'énergie (25 % de l'ensemble sont d'absorption rapide), les lipides 40 % de l'énergie (essentiellement sous forme de
graisses de constitution dans les viandes, les oeufs, le lait et laitages), les protéines 14 % de l'énergie (essentiellement
d'origine animale). Cette
patiente ne prend aucun médicament.

Sur quel(s) argument(s) pouvez-vous affirmer que cette patiente est diabétique ?
A - Tout sujet obèse est diabétique
B - Dans l'obésité il y a toujours un hypo-insulinisme
C - Une glycémie à jeun à 7,10 mmol/l
D - L'ingestion de glucides d'absorption rapide est trop importante
E - Sur aucun des arguments précités
Bonne(s) Réponse(s) : E

Valeur de l'OMS = 8 mmol/litre à jeun.

Quelle(s) est(sont) les conditions à respecter pour qu'une H.G.P O. soit interprétable ?
A - Il faut utiliser 75 g de glucose chez l'adulte en dehors de la femme enceinte
B - Le sujet doit être en position de repos pendant toute la durée de l'épreuve
C - La glucose doit être dilué dans 50 ml d'eau
D - Il est souhaitable de ne piquer le malade qu'une seule fois
E - Il faut que le patient ingère moins de 100 g de glucides dans les 3 jours précédant l'épreuve
Bonne(s) Réponse(s) : A B D

Théoriquement, l'HGPO de la femme enceinte s'effectue avec 100 g de glucose (mais pour d'autres auteurs 75 g sont
suffisants).

Dans l'interprétation de l'H.G.P.O, quel chiffre de glycémie du plasma veineux retenez-vous 2 heures après
l'ingestion de glucose pour affirmer le diagnostic de diabète ?
A - 7,30 mmol/l
B - 9,40 mmol/l
C - 10 mmol/l
D - 11,50 mmol/l
E - Aucune des valeurs précités
Bonne(s) Réponse(s) : D

Valeur de l'OMS est supérieure ou égale à 11 mmol/I.

610
Exclusivement sur DOC - DZ : www.doc-dz.com NADJI 85
RESIDANAT EN POCHE TOME II
Cas Clinique en QCM

Pour quelle(s) raison(s) cette patiente présente-t-elle des vergetures pourpres ?


A - Les séquelles de la grossesse
B - L'insuffisance d'apports en protéines
C - Un hypercortisolisme d'entraînement
D - Un diabète
E - Aucune des propositions précitées
Bonne(s) Réponse(s) : C

Cet hypercorticisme d'entraînement est lié à l'obésité.

Comment peut-on modifier l'alimentation de cette patiente ?


A - En diminuant globalement les calories mais en maintenant le même pourcentage de glucides, lipides et
protides que celui de l'enquête alimentaire
B - En diminuant globalement les calories, les glucides, les lipides et en augmentant la ration protéique (qui
comportera le plus possible de viandes)
C - En supprimant les glucides d'absorption rapide
D - En prescrivant un régime sans sel
E - En fractionnant les apports énergétiques en, au moins, 3 repas
Bonne(s) Réponse(s) : B C E

Sans commentaire.

Quelle(s) médicament(s) pourriez-vous adjoindre aux mesures diététiques sachant que le régime est de 3350
kilojoules (800 kilocalories/jour) ?
A - L'insuline
B - Les sulfamides hypoglycémiants
C - La thyroxine
D - Des oligo-éléments et des vitamines
E - Des dérivés des amphétamines
Bonne(s) Réponse(s) : D

La thyroxine et les dérivés amphétaminiques sont fort nettement contre-indiqués dans ce type de pathologie.

Un homme de 23 ans qui subit un examen médical annuel consulte pour nodule thyroïdien découvert fortuitement il y a
quelques jours. Il ne présente aucun signe de dysfonctionnement thyroïdien et ne s'était jamais plaint de sa thyroïde
auparavant. Il a reçu une irradiation cervicale à l'age de 6 mois pour une hypertrophie thymique et ne présente aucun autre
antécédent. L'examen clinique est normal en dehors d'un nodule dur mesurant 2 x 2 cm à la partie basse du lobe thyroïdien
droit. On retrouve quelques adénopathies cervicales antérieures droites fixées. Le reste de la glande thyroïde est normal. Les
dosages hormonaux thyroïdiens sont normaux, les anticorps anti-microsomes sont à un titre de 1/32. La scintigraphie
thyroïdienne montre un nodule hypofixant coïncidant avec la palpation. L'échographie montre un nodule plein.

Le diagnostic vis-à-vis du nodule de ce patient doit être fortement orienté par les paramètres suivants.
Choisissez le(s)quel(s) :
A - Le sexe de ce sujet
B - La présence d'adénopathies cervicales
C - L'antécédent d'irradiation cervicale
D - Le taux des anticorps anti-microsomes
E - L'apparition récente de ce nodule
Bonne(s) Réponse(s) : B C E

L'antécédent d'irradiation cervicale dans l'enfance est un facteur de risque de cancer thyroïdien.

La scintigraphie est un examen utile pour l'exploration des nodules thyroïdiens parce-que :
A - Un nodule hypofixant signifie à coup sûr un cancer thyroïdien
B - Un nodule hyperfixant (fixant plus que le parenchyme voisin) exclut le plus souvent l'existence d'un nodule
cancéreux
C - La scintigraphie permet une approche exacte de l'histologie
D - La scintigraphie donne une information sur le caractère nodulaire du goître
E - Un nodule faiblement hypofixant est un argument en faveur d'une pathologie différente du cancer
Bonne(s) Réponse(s) : B D

Sans commentaire.

611
Exclusivement sur DOC - DZ : www.doc-dz.com NADJI 85
RESIDANAT EN POCHE TOME II
Cas Clinique en QCM

La meilleure attitude thérapeutique face au nodule thyroïdien de ce patient est :


A - Biopsie thyroïdienne à l'aiguille
B - Traitement par hormones thyroïdiennes
C - Surveillance pendant 6 mois, car le nodule devrait régresser
D - Exploration chirurgicale de la thyroïde
E - Doses pharmacologiques d'iodure.
Bonne(s) Réponse(s) : D

Tout nodule thyroïdien froid doit être exploré chirurgicalement (certaines équipes cependant pratiquent actuellement une
cytoponction à l'aiguille) par des gens entraînés, mais seul un résultat positif (malignité) est significatif. Cependant, l'existence
dans ce contexte d'adénopathies cervicales justifie sans aucun doute la chirurgie.

Si le patient est porteur d'un carcinome papillaire pur sans envahissement ganglionnaire lors du curage, et si
une thyroïdectomie totale a été réalisée, le traitement va être vraisemblablement complété par :
A - Traitement immédiat par l'iode 131
B - Reprise chirurgicale pour un curage ganglionnaire contro-latéral
C - Attente de l'élevation de la TSH plasmatique avant d'administrer une dose
D - Abstention d'emploi d'hormones thyroïdiennes susceptibles de stimuler les éventuelles métastases
E - Destruction du reliquat thyroïdien cervical
Bonne(s) Réponse(s) : C E

Si l'on administre de l'iode 131, il faut attendre l'augmentation de la TSH (mais ce traitement n'est pas systématique,
notamment dépend des écoles).

Quels sont les 3 facteurs pronostiques péjoratifs du cancer papillaire de la thyroïde ?


A - Les antécédents de radiothérapie cervicale
B - L'expansion extracapsulaire de la tumeur
C - L'existence d'un composant vésiculaire peu différencié de cette tumeur
D - L'âge du sujet supérieur à 45 ans
E - La présence d un envahissement ganglionnaire
Bonne(s) Réponse(s) : B C D

Bien que cela puisse sembler surprenant, l'envahissement ganglionnaire n'est pas un élément
pronostic du cancer papillaire.

En cas de thyroïdectomie totale, la surveillance systématique du cancer papillaire du sujet jeune comporte :
A - Palper annuel du cou
B - Scintigraphie à l'I131
C - Radiographie pulmonaire annuelle
D - Dosage de thyroglobuline
E - Dosage de l'antigène carcino-embryonnaire
Bonne(s) Réponse(s) : A B C D

L'indication et la fréquence de la scintigraphie dépendent des équipes. Dans l'item, le rythme


de surveillance n'est pas précisé, il semble cependant justifié de le cocher dans la surveillance.

Une malade de 25 ans, sans antécédents particuliers, présente un tableau clinique de thyrotoxicose avec installation en moins
d'un mois :
- d'un amaigrissement de 6 kg, sans modification de l'appétit
- des palpitations avec dyspnée et tachycardie de repos, régulière à 100
- d'une asthénie avec nervosité excessive, de tremblement fins des extrémités.
Le corps thyroïde est globalement hypertrophié, ferme, homogène, vasculaire avec perception d'un thrill et constatation d'un
souffle parenchymateux L'examen oculaire est normal.
L'index de thyroxine libre est élevé : 7,5 (Normal de 1,5 à 4,5).

Quelle est l'étiologie possible de cet état de thyrotoxicose, compte tenu des données cliniques ?
A - Thyrotoxicose factice
B - Thyroïdite subaiguë
C - Maladie de Basedow
D - Adénome hypophysaire à TSH
E - Goître nodulaire toxique
Bonne(s) Réponse(s) : C

Sans commentaire.

612
Exclusivement sur DOC - DZ : www.doc-dz.com NADJI 85
RESIDANAT EN POCHE TOME II
Cas Clinique en QCM

Les données biologiques sont les suivantes. Laquelle(lesquelles) doi(ven)t faire rechercher un facteur
pathologique associé ?
A - Numération formule leucocytaire : 6 000 GB, 55 % Poly, 45 % Lympho
B - Cholestérol : 3 g/l
C - PBI : 13,5 - /100 ml
D - Pourcentage de fixation thyroïdienne de l'iode radio-actif à la 24ème heure : 86 %
E - Gamma GT : 550 mU (Normal jusqu'à 350 mU)
Bonne(s) Réponse(s) : B

Sans commentaire.

Un traitement par le propanolol (Avlocardyl® : 60 mg/24h) est institué. Dans ce cas, cette thérapeutique aura
une ou plusieurs des conséquences suivantes :
A - Modifiera le taux de T4
B - Diminuera le tremblement
C - Fera prendre un risque d'insuffisance cardiaque
D - Ralentira le rythme cardiaque
E - Réduira le volume du goitre
Bonne(s) Réponse(s) : B D

Le propranolol possède, en plus de son effet béta bloquant, un effet d'inhibition de la conversion périphérique de T4 en T3 au
profit de la RT3 (reverse T3). L'amélioration ressentie les premiers jours chez les hyperthyroïdiens traités, est due au béta- et
non aux antithyroïdiens de synthèse dont le délai d'action est plus long.

La mise en évidence dans le sérum, d'un anticorps thyréostimulant (TSI), évoque un mode de déclenchement
particulier. Lequel ?
A - Stress
B - Facteur traumatique
C - Virus
D - Médicamenteux
E - Aucune des propositions précédentes
Bonne(s) Réponse(s) : E

Une origine immunologie est évoquée dans la pathogénie de la maladie de Basedow.

L'éventualité d'un traitement a l'iode radio-actif étant discutée, l'un ou plusieurs des éléments suivants sont des
contre-indications habituelles :
A - Grossesse débutante
B - Age de la malade : 25 ans
C - Cholécystographie pratiquée après la mise en évidence de l'hyperthyroïdie
D - Lymphocytose relative
E - Volume du goître
Bonne(s) Réponse(s) : A B C

Sans commentaire.

Un traitement au carbimazole (Néomercazole®) étant décidé, le(s)quel(s) des éléments suivants est(sont)
utile(s) et doi(ven)t être indiqué(s) sur l'ordonnance) ?
A - Dosage de T4, 8 jours après le début du traitement
B - Numération-formule urgente en cas de fièvre
C - Contraception efficace
D - Interdiction d'un traitement antibiotique associé
E - Aucun
Bonne(s) Réponse(s) : B C

La NFS doit être pratiquée :


- de façon urgente en cas de fièvre (prévenir le patient)
- toutes les une à deux semaines pendant les trois premiers mois puis tous les mois, en raison du risque grave
d'agranulocytose aiguë
- avant traitement, pour ne pas imputer à tort aux ATS une éventuelle granulopénie antérieure.

613
Exclusivement sur DOC - DZ : www.doc-dz.com NADJI 85
RESIDANAT EN POCHE TOME II
Cas Clinique en QCM

La malade étant au carbimazole (Néomercazole® depuis 2 mois, 30 mg, puis 20 mg par jour), le goitre
augmente de volume. L'état clinique est celui d'une discrète hypothyroïdie. Quelle est la décision logique à
prendre en attendant les résultats du bilan hormonal ?
A - Arrêter le traitement
B - Augmenter le traitement
C - Diminuer le traitement
D - Prescrire du Lugol®
E - Aucune
Bonne(s) Réponse(s) : C

Il semble que la malade soit passée en hypothyroïdie : certains diminuent le traitement. Par la suite, certains adjoignent des
hormones thyroïdiennes, le Néomercazole® ayant peut-être une action immunosuppressive.

Une rechute survenant après l'arrêt du traitement, une thyroïdectomie sub-totale est proposée. En dehors de la
guérison souhaitée sans séquelles, la(les) évolution(s) possible(s) lui sont exposée(s). Laquelle(lesquelles)
est(sont) réelle(s) ?
A - Evolution sur le plan oculaire( exophtalmie)
B - Insuffisance thyroïdienne
C - Hypoparathyroïdie
D - Récidive thyrotoxique
E - Aucune
Bonne(s) Réponse(s) : A B C D

L'évolution la plus classique est l'évolution sur le plan oculaire, l'exophtalmie pouvant d'ailleurs évoluer de façon totalement
indépendante de l'hyperthyroïdie. Toutes les autres évolutions sont cependant possibles bien que l'insuffisance thyroïdienne
et l'hypoparathyroïdie doivent être rares en cas de thyroïdectomie subtotale.

Une patiente de 25 ans a bénéficié d'une thyroïdectomie totale pour épithélioma thyroïdien papillaire. Dans ces conditions, elle
doit suivre un traitement hormonal thyroïdien substitutif et freinateur thyréotrope en raison de l'hormonodépendance de ce
cancer. Après la prise d'iode 131 la malade a bénéficié de la prise de Cynomel® à la dose de 1 comprimé matin, midi et soir.
Cliniquement le résultat est excellent et la tolérance thérapeutique est acceptable.

La surveillance biologique de cette thérapeutique va être :


A - Dosage de la T3 libre
B - Dosage de la T4 et de la T3 libres
C - Dosage de la T3 et de la TSH de base et après TRH
D - Dosage de la T4 totale
E - Dosage de la T4 libre et de la TSH et après TRH
Bonne(s) Réponse(s) : C

Cynomel® = LT3 = hormone pure.

Si on remplace le Cynonel® par de l'Euthyral® (LT4 + LT3) la surveillance comportera :


A - Dosage de T3 T4 libres et TSH (avec test à la TRH)
B - Dosage de la T3 libre
C - Dosage isolé de la TSH avec test a la TRH)
D - Dosage de la T4 libre
E - Dosage de la TBG
Bonne(s) Réponse(s) : A

Euthyrol® = LT3 et LT4.


Sous Euthyrol®, FT3 et FT4 sont en rapport avec l'état fonctionnel.

Le surdosage en hormones thyroïdiennes peut se traduire par :


A - Bloc auriculoventriculaire avec syncopes
B - Frilosité
C - Diarrhée
D - Constipation
E - Adipsie
Bonne(s) Réponse(s) : C

Sans commentaire.

614
Exclusivement sur DOC - DZ : www.doc-dz.com NADJI 85
RESIDANAT EN POCHE TOME II
Cas Clinique en QCM

Compte tenu des 1/2 vies de ces différents produits, l'arrêt du traitement devra être, avant une nouvelle
administration d'iode de :
A - 12 jours pour le Cynonel®
B - 10 jours pour le Lévothyrox®
C - 25 jours pour l'Euthyral®
D - 30 jours pour le Cynonel®
E - 60 jours pour le Lévothyrox®
Bonne(s) Réponse(s) : A C

Le Cynomel® = LT3.
Le Lévothyrox® = LT4.
Thyroxine = D-LT4.

Madame X, 35 ans, mariée, 3 enfants, se plaint d'une altération de l'état général avec perte de 5 kg en 1 mois, anorexie et
asthénie. Il existe de plus une fièvre à 38 degrés en permanence et sur le plan fonctionnel la patiente se plaint d'une douleur
au niveau des mâchoires et des oreilles. La palpation cervicale permet de trouver une thyroïde augmentée de volume surtout
à droite. Elle est modérément douloureuse à la pression et assez ferme. Le coeur est régulier à 120 et il existe hypersudation
et tremblement fin des extrémités. L'examen ORL est normal.

Cet amaigrissement associé à une anorexie permet :


A - D'écarter une hyperthyroïdie
B - D'évoquer une pathologie digestive
C - D'affirmer le caractère simple du goître
D - D'évoquer un processus inflammatoire
E - D'évoquer le caractère psychosomatique des symptômes
Bonne(s) Réponse(s) : B D

Une pathologie digestive ne peut être éliminée, bien que peu évocatrice dans ce contexte. On peut bien entendu évoquer un
processus inflammatoire, bien que ce ne soit pas spécifiquement l'anorexie et l'amaigrissement qui l'évoque mais le reste de
l'examen.

Le ou les diagnostics suivants sont à envisager :


A - Paresthésies pharyngées
B - Thyroïdite subaiguë
C - Oesophagite de la bouche oesophagienne
D - Maladie de Hashimoto
E - Hyperthyroïdie
Bonne(s) Réponse(s) : B D E

Il peut exister une poussée d'hyperthyroïdie dans la maladie de Hashimoto : cependant celle-ci, à long terme, évolue
généralement vers l'hypothyroïdie.

L'établissement du diagnostic rend nécessaire la demande de :


A - Dosage de la TSH
B - Anticorps antithyroïdiens
C - NFS - VS
D - Dosage des hormones T3 T4 libres
E - Dosage de la thyroglobuline
Bonne(s) Réponse(s) : B C D

Pour le diagnostic d'hyperthyroïdie en 85, on utilisait essentiellement T3 et T4 libres, mais actuellement, la TSH ultrasensible
est très intéressante (étant diminuée avant l'augmentation de T3 et T4).

Si le diagnostic de maladie de Hashimoto est retenu, quelle proposition thérapeutique choisiriez-vous dans
l'immédiat ?
A - Thyroïdectomie
B - L Thyroxine
C - Abstention momentanée et surveillance attentive sous traitement symptomatique
D - Immunodépresseurs
E - Iode 131
Bonne(s) Réponse(s) : C

Sans commentaire.

615
Exclusivement sur DOC - DZ : www.doc-dz.com NADJI 85
RESIDANAT EN POCHE TOME II
Cas Clinique en QCM

S'il s'agit d'une thyroïdite subaiguë :


A - Il faut utiliser les corticoïdes pendant 1 mois minimum
B - Cette maladie évolue fréquemment vers le cancer de la thyroïde
C - On peut craindre une hypothyroïdie résiduelle dans 40 % des cas
D - Après un traitement correct il peut y avoir rechute
E - L'hyperthyroïdie initiale nécessite un traitement par anti-thyroïdiens de synthèse
Bonne(s) Réponse(s) : A D

Les rechutes sont généralement liées à un arrêt prématuré de traitement (ce qui est fréquent ! ).

Une jeune femme de 35 ans consulte pour hirsutisme, prise pondérale moyenne, irrégularités menstruelles puis aménorrhée.
A l'examen, elle a un excès pondéral modéré (1 m 60) (70 kg) avec répartition plutôt androïde de l'adiposité, une tension
artérielle à 16/9, un faciès assez arrondi, érythrosique, une pilosité faciale modérée, une peau parsemée de petites
ecchymoses surtout au niveau de la face antérieure des jambes, une discrète atrophie musculaire proximale. La biologie
montre :
- une hématocrite a 48 %
- une natrémie à 140 mmol/l
- une kaliémie à 32 mmol/l
- un C02 à 30
- une chlorémie à 95 mmol/l
- une glycémie à jeun a 7 mmol/l
- la cortisolémie plasmatique est à 800 nmol/l (N 190-600 nmol/l)
On hospitalise la malade pour éclaircir la situation.

Parmi les symptômes présentés par cette femme, vous retenez en faveur de l'hypercortisolisme :
A - Les caractères de l'obésité
B - Atrophie musculaire proximale
C - La valeur de la kaliémie
D - Hématocrite à 48 %
E - Glycémie à 7 mmol/l
Bonne(s) Réponse(s) : A B C E

Sans commentaire.

Le bilan biologique à effectuer d'emblée doit comporter :


A - Un cycle de l'aldostéronémie
B - Une épreuve d'hyperglycémie provoquée
C - Un cycle de la cortisolémie
D - Un cycle de l'ACTHhémie
E - Une épreuve de freinage à la dexaméthazone
Bonne(s) Réponse(s) : C E

La perte du cycle nycthéméral du cortisol est un signe biologique majeur en faveur d'un hypercortisolisme.

La(les) investigation(s) topographique(s) indispensable(s) d'emblée est(sont) :


A - Une bronchoscopie
B - Une phlébographie surrénalienne avec dosages étagés du cortisol
C - Un rétropneumopéritoine
D - Radiotomographie de la selle turcique et/ou scanner hypophysaire
E - Un scanner abdominal
Bonne(s) Réponse(s) : D E

Un examen surrénalien et hypophysaire permettent d'orienter vers une pathologie surrénalienne primitive ou hypophysaire.
NB : C'est une mauvaise question car l'exploration d'un hypercortisolisme se fait en plusieurs étapes :
l) Confirmer l'hypercortisolisme.
2) Et après, préciser son origine, c'est au niveau de cette deuxième étape que se situent ces deux examens.

Si vous choisissez une intervention chirurgicale, vous vous concerterez avec le chirurgien pour :
A - Mettre la malade sous couverture antibiotique
B - Administrer 100 mg d'hémisuccinate d'hydrocortisone toutes les 6 heures des la veille de l'intervention
C - Administrer dès la fin de l'intervention 200 mg toutes les 4 heures
D - Prendre le relai cortisonique per os à raison de 100 mg en 3 prises quotidiennes, puis à doses dégressives
E - Vous vous contenterez d'une simple surveillance
Bonne(s) Réponse(s) : B C D

La préparation pré, per et post-opératoire est indispensable, notamment en raison du risque d'insuffisance surrénalienne
aiguë. Ce protocole cité est peu satisfaisant. La veille notamment, on utilise plutôt de l'acétate de cortisone IM, de même que
le jour de l'intervention (lors de la chirurgie, on peut aussi ajouter de l'hémisuccinate IV).

616
Exclusivement sur DOC - DZ : www.doc-dz.com NADJI 85
RESIDANAT EN POCHE TOME II
Cas Clinique en QCM

Six mois après son équilibration, la malade présente, sans élément déclenchant évident, une température à 40,
des douleurs abdominales aiguës mais diffuses et des vomissements. L'abdomen est souple, la tension
artérielle est à 80/50 mmHg. Vous devez envisager en urgence :
A - Une pyélonéphrite aiguë
B - Une appendicite aiguë
C - Une insuffisance surrénalienne aiguë
D - Une grossesse extra-utérine
E - Une intoxication alimentaire
Bonne(s) Réponse(s) : C

L'insuffisance surrénalienne aiguë doit être évoquée de première intention chez tout insuffisant surrénalien "allant mal". Les
signes digestifs, fréquents, peuvent faire errer le diagnostic vers une pathologie chirurgicale (une telle pathologie peut
d'ailleurs exister et décompenser secondairement une insuffisance surrénalienne).

Une jeune femme de 24 ans vous consulte pour asthénie, diarrhée, palpitations, sueurs. A l'examen, le pouls de repos est à
110/mn, le corps thyroïde est globalement hypertrophié, la tension artérielle est de 180-80 mm Hg. Les réflexes achilléens
sont vifs. On note également une prise de poids de 3 kilos en un mois. Il n'y a pas d'exophtalmie, mais une rétraction de la
paupière supérieure. Les examens complémentaires :
T4 = 8 microg/100 ml ; N = 5,4 - 13,2 ; TBG = 70 microg/ml ; N = 11,2 - 30,2 ; rapport T4 :TGB = 0,40 (N = 0,27 - 0,61) ; T3 =
350 mg/100 ml (N = 70 - 165).
Scintigraphie thyroïdienne : fixation homogène diffuse sur un corps thyroïde globalement augmenté de volume.

Parmi ces signes cliniques, quel est celui qui n'est pas dû à l'hypersécrétion d'hormones thyroïdiennes ?
A - Asthénie
B - Diarrhée
C - Tachycardie
D - Goître
E - Rétraction de la paupière supérieure
Bonne(s) Réponse(s) : D

Le goître n'est pas un élément en faveur d'une thyrotoxicose, mais de son étiologie. Le goître dans ce contexte est évocateur
d'une hyperthyroïdie liée à une maladie de Basedow, mais il n'est pas dû à l'hypersécrétion isolée d'hormones thyroïdiennes.

Quel est l'examen complémentaire qui permet dans ce cas de faire le diagnostic étiologique de maladie de
Basedow ?
A - T4
B - TBG
C - T4/TBG
D - T3
E - Scintigraphie thyroïdienne
Bonne(s) Réponse(s) : E

L'aspect scintigraphique décrit est celui d'une maladie de Basedow. Pour certains, il existe des cas typiques de maladie de
Basedow où l'on peut éventuellement se passer de la scintigraphie.

Quelle est la fréquence de la tachycardie dans l'hyperthyroïdie ?


A - Elle est toujours présente
B - 50 % des cas
C - 30 % des cas
D - 10 % des cas
E - Elle n'est jamais présente
Bonne(s) Réponse(s) : A

A - La tachycardie est un très bon élément de diagnostic, de surveillance. Elle est permanente, persiste au repos, augmente à
l'effort, est généralement sinusale à l'ECG.

Le traitement de cette patiente fait appel à :


A - Chirurgie en urgence
B - Iode radio-actif en urgence
C - Iode radio-actif en préparation à la chirurgie
D - Administration d'antithyroïdiens
E - Aucun de ces traitements
Bonne(s) Réponse(s) : D

D - Les antithyroïdiens de synthèse seront associés aux béta bloquants, aux sédatifs et au repos.

617
Exclusivement sur DOC - DZ : www.doc-dz.com NADJI 85
RESIDANAT EN POCHE TOME II
Cas Clinique en QCM

Pour traiter simultanément l'hyperthyroïdie et l'HTA, vous prescrivez :


A - Alpha méthyl dopa
B - Inhibiteur de l'enzyme de conversion
C - Diurétique thiazidique
D - Alpha-bloquant
E - Bêta-bloquant
Bonne(s) Réponse(s) : E

Sans commentaire.

Une femme de 75 ans, traitée depuis deux ans pour un diabète, est retrouvée chez elle à midi par sa fille qui l'avait quittée à 8
h, avant qu'elle prenne son petit déjeuner, mais après avoir pris ses médicaments habituels et une dragée de Trinitrine®, car
elle ne se sentait pas très bien. Le médecin appelé fait le diagnostic de coma hypoglycémique et pratique une injection
intraveineuse de sérum-glucosé qui réveille aussitôt la malade. Celle-ci continue son traitement habituel mais s'alimente
légèrement.
L'interrogatoire nous apprend que le malaise a dû survenir de façon assez brutale sans aucun prodrome classique habituel :
pas de palpitation, pas de transpiration, mais chute relativement brutale dans sa cuisine.
Le lendemain matin, à 8 h, le médecin est rappelé pour récidive du coma et il adresse la malade à l'hôpital. A l'arrivée, un
bilan d'urgence est pratiqué qui montre une glycémie à 1,6 mmol/l, une urée à 14 mmol/l, une creatininémie à 200 mmol/l. Le
bilan neurologique et vasculaire est strictement normal, en dehors d'une hypertension artérielle à 16/9. La malade est aussitôt
traitée par sérum-glucosé et reprend très rapidement conscience.
Il s'agit d'une femme de 86 kgs pour 1m63 qui présente son ordonnance habituelle et celle-ci comporte :
- un régime hypocalorique hypoglucidique mal suivi.
- Avlocardyl® 40 : 2/jour.
- Daonil® : 1 comprimé le matin, midi et soir, de prescription récente
- Catapressan® : 1 comprimé et demi par jour
- digitaliques : 5 Jours par semaine.
Le diagnostic de son médecin traitant était celui de diabète non insulinodépendant chez une obèse avec hypertension
artérielle relativement importante associée à une coronarite et à une insuffisance cardiaque modérée.

Laquelle(lesquelles) des manifestations suivantes n'est(ne sont) pas habituellement rencontrée(s) dans
l'hypoglycémie ?
A - Transpiration abondante
B - Babinski bilatéral
C - Bradycardie
D - Tremblements
E - Crise comitiale de type "Bravais-Jackson"
Bonne(s) Réponse(s) : C

En cas d'hypoglycémie, il existe des signes liés :


- à la carence cellulaire en glucose, pouvant donner des signes neurologiques ou psychiatriques ; les crises convulsives sont
plus volontiers généralisées, mais une crise localisée n'exclue absolument pas le diagnostic
- à la réaction de système sympathique --> A.D. et bien d'autres signes.

Quel(s) facteur(s) peut-on incriminer dans le déclenchement de l'accident hypoglycémique ?


A - Absence de petit déjeuner
B - Trinitrine
C - Digitaliques
D - Daonil®
E - Catapressan®
Bonne(s) Réponse(s) : A D

Le Daonil® est un sulfamide hypoglycémiant puissant de courte durée d'action, qui n'aurait pas du être prescrit chez cette
patiente obèse et insuffisante rénale.

Pourquoi aucun prodrome annonciateur n'a-t-il été ressenti par le malade ? Une ou plusieurs des réponses
suivantes sont exactes :

A
A - Elévation de l'urée et de la créatinémie
B - Particularité d'action du Daonil®
C - Rôle de l'Avlocardyl®
D - Trinitrine
E - Rôle des digitalines
Bonne(s) Réponse(s) : C

En cas d'hypog!ycémie, il existe une réaction adrénergique compensatrice, masquée ici par la prise de béta- (Avlocardyl®).

618
Exclusivement sur DOC - DZ : www.doc-dz.com NADJI 85
RESIDANAT EN POCHE TOME II
Cas Clinique en QCM

On peut expliquer la rechute par :


A - L'importance de l'activité hypoglycémiante
B - Rôle de l'insuffisance rénale
C - Rôle de l'insuffisance cardiaque
D - Rôle de l'excédent pondéral
E - Rôle des digitaliques
Bonne(s) Réponse(s) : A B

L'insuffisance rénale contre-indique généralement la prise de Daonil® ; éventuellement, on peut diminuer la dose de ce
médicament à demi-vie courte et à élimination rénale.

Quelle est l'attitude thérapeutique à adopter à la suite de cet incident ?


A - Poursuivre le même régime en diminuant les doses de Daonil®
B - Poursuivre le même régime et remplacer le Daonil® par Diamicron®
C - Poursuivre le même régime et remplacer le Daonil® par biguanides
D - Imposer un régime plus restrictif en supprimant totalement les antidiabétiques
E - Mettre en route une insulinothérapie à petites doses répétées.
Bonne(s) Réponse(s) : D

Il faut toujours prescrire en premier un régime hypocalorique assez strict, avant l'utilisation d'antidiabétiques oraux, d'autant
que cette patiente est obèse et que le Daonil® n'est pas une
bonne indication.

Une femme de 40 ans consulte pour une fatigabilité, un ralentissement de l'activité psychomotrice, une frilosité, une infiltration
des téguments, une bradycardie à 50/mn. Ce tableau, qui s'est installe progressivement, évoque l'insuffisance thyroïdienne.
Les examens biologiques confirment ce diagnostic. En effet, 10 ans auparavant, cette malade a subi l'exérèse d'un goître
volumineux, esthétiquement gênant déviant déjà fortement la trachée. Elle était par ailleurs euthyroïdienne à cette époque.
Elle est originaire d'une région d'endémie goîtreuse sa mère et une tante matemelle etaient aussi porteuses d'un goître.
Il n'y a pas, par ailleurs, de signes d'atteinte polyendocrinienne ou hypophysaire.

Parmi les signes suivants, habituels ou caractéristiques au cours de l'insuffisance thyroïdienne périphérique, le
tableau classique de cette affection comporte :
A - La pâleur du visage
B - Des raideurs et des crampes musculaires
C - L'hypertrichose
D - L'hypoacousie
E - Perte de la mémoire
Bonne(s) Réponse(s) : A B D E

Sans commentaire

Parmi les manifestations cardiovasculaires suivantes, laquelle (lesquelles) est(sont) habituelles au cours de
l'insuffisance thyroïdienne chronique ?
A - Augmentation de la vitesse circulatoire
B - Vasodilatation périphérique
C - Microvoltage des complexes à l'ECG
D - Elargissement de la silhouette cardiaque
E - Augmentation du débit cardiaque
Bonne(s) Réponse(s) : C D

L'élargissement de la silhouette cardiaque peut exister en dehors de toute insuffisance cardiaque et être liée au coeur
"myxoedémateux".

Au cas où vos examens cliniques et biologiques ne seraient pas assez probants, vous pratiquez un test à la
TRH. Quelle est la réponse biologique qui vous confirmera, chez cette patiente, l'insuffisance thyroïdienne ?
A - La TSH de base est normale, non stimulée après TRH
B - La TSH de base est basse, elle reste basse après TRH
C - La TSH de base est normale, elle augmente après TRH
D - La TSH de base est élevée, elle s'élève encore après TRH
E - La TSH de base est élevée, elle n'est pas modifiée après TRH
Bonne(s) Réponse(s) : D

Le test au TRH a actuellement de moins en moins d'intérêt depuis l'utilisation de la TSH ultrasensible.

619
Exclusivement sur DOC - DZ : www.doc-dz.com NADJI 85
RESIDANAT EN POCHE TOME II
Cas Clinique en QCM

Une altération corticosurrénalienne est souvent associée à l'insuffisance thyroïdienne primitive. Quelle(s)
est(sont) la(les) proposition(s) exacte(s) ?
A - Cette altération est le plus souvent fonctionnelle, sans lésion organique de la surrénale
B - Le taux des 17 CS et 17 OH stéroïdes urinaires est augmenté du fait de la prise de poids
C - L'hypercholestérolémie du myxoedème est due à cette altération surrénalienne
D - Le retard à l'élimination d'une charge hydrique (opsiurie) est dû uniquement à cette altération surrénalienne
E - Le traitement par la thyroxine suffit à corriger cette altération corticosurrénalienne
Bonne(s) Réponse(s) : A E

L'hypothyroïdie peut en effet entraîner un léger retentissement surrénalien, biologique mais avec cortisol normal. Cependant, il
peut exister une véritable maladie d'Addison associée à une hypothyroïdie dans le cadre d'une pathologie endocrinienne
autoimmune,.mais dans ce cas ce n'est pas une association fréquente et le mot, souvent, de l'item élimine cette possibilité.

Cette malade va être traitée par des doses croissantes de thyroxine. Au bout de trois mois, vous constatez
certain(s) signe(s) évoquant un surdosage thérapeutique qui invitent à réduire les doses de thyroxine.
Le(s)quel(s) ?
A - Accélération du pouls à 70/minute au repos
B - Palpitations et douleurs précordiales de type constrictif
C - Apparition d'une thermophobie et de sudations
D - Gastralgies, par intolérance médicamenteuse
E - Taux de TSH à 4 U/ml (N : 0 - 7)
Bonne(s) Réponse(s) : B C

Une fréquence cardiaque à 70 par minute au repos est une fréquence cardiaque normale.

Après l'intervention d'exérèse du goître, l'évolution spontanée s'est faite vers l'insuffisance thyroïdienne. Celle-ci
était prévisible en raison de l'étendue de la thyroïdectomie, à condition d'avoir fait des contrôles biologiques
adéquats. Il aurait fallu :
A - Ne rien faire tant qu'il n'y a pas de signes cliniques d'hypothyroïdie débutante
B - Prescrire des petites doses de thyroxine pour prévenir cette évolution
C - Prescrire de petites doses d'iode (Lugol®) pour prévenir cette évolution
D - Ne pas opérer ce goître et préférer les antithyroïdiens de synthèse
E - Ne pas opérer ce goître et préférer le réduire par une dose thérapeutique d'iode 131
Bonne(s) Réponse(s) : B

Sans commentaire.

Un homme de 24 ans sans antécédents familiaux ou personnels particuliers vous est adressé pour un amaigrissement de 8 kg
en 3 mois. L'interrogatoire met en évidence :
- un syndrome polyuro-polydipsique installé depuis 3 semaines
- une asthénie physique sans anorexie
- des vomissements alimentaires depuis 48 heures.
L'examen somatique retrouve :
- un poids de 59 kgs pour une taille de 1,77 m
- une TA à 12/7 cm de Hg aux deux bras
- un état d'hydratation correct alors que l'examen systématique se révèle par ailleurs tout à fait normal.
L'enquête alimentaire objective des apports spontanés à 3120 Kcal/jour dont 390 g de glucides, 120 g de lipides et 120 g de
protéines.
Les premiers examens biologiques fournissent les éléments suivants :
- glycosurie au clinitest à + + + +
- glycosurie des 24 heures à 130 g soit 990 mM
- glycémies à jeûn des trois jours précédents : 2,27 g/l soit 12,5 mM/l, 2,94 g/l soit 16,1 mM/l, 1,76 g/l soit 9,7 mM/l
- acétonurie a l'Acétest à + +.

Sur laquelle des propositions suivantes, le diagnostic de diabète sucré est-il affirmé ?
A - Les seules données cliniques
B - L'existence d'une glycosurie
C - L'existence d'une cétonurie
D - La notion d'une hyperglycémie à jeun constatée à 3 reprises
E - Aucune de ces 4 propositions. Il est nécessaire de réaliser une HGPO
Bonne(s) Réponse(s) : D

Les critères OMS permettant de porter le diagnostic de diabète sucré sont :


- glycémie à jeun supérieure ou égale à 7,7 mmol/l (1,4 g/l) ou post prandiale supérieure ou égale 11,1 mol/l (2 g/l) sur plasma
veineux.

620
Exclusivement sur DOC - DZ : www.doc-dz.com NADJI 85
RESIDANAT EN POCHE TOME II
Cas Clinique en QCM

Une des propositions suivantes est en faveur du caractère insulinodépendant de ce diabète : laquelle ?
A - L'absence d'antécédents infectieux récents
B - L'importance des valeurs glycémiques
C - L'importance de la glycosurie
D - L'absence de déshydratation massive
E - La présence de corps cétoniques dans les urines
Bonne(s) Réponse(s) : E

Sans commentaire.

L'amaigrissement des 3 derniers mois est en relation avec une des propositions suivantes :
A - Les vomissements alimentaires
B - La polyurie
C - Un catabolisme protidique et lipidique induit par la carence insulinique
D - Un déséquilibre de l'alimentation au profit des glucides
E - Un déséquilibre de l'alimentation au profit des protides
Bonne(s) Réponse(s) : C

Connaissance.

Le traitement à mettre rapidement en oeuvre repose sur l'une des propositions suivantes, laquelle ?
A - La prescription d'un régime contenant au moins 3500 kcalories/jour
B - La prescription de biguanides
C - La prescription de sulfamides
D - La prescription d'insuline retard
E - La prescription d'insuline rapide
Bonne(s) Réponse(s) : E

Connaissance.

Passée la période aiguë, le régime préconisé visera à maintenir le poids à un niveau normal et contiendra :
A - 20 % proté'ines, 45 % lipides, 35 % de glucides répartis en 3 repas et une collation
B - 15 % protéines, 30 % lipides, 55 % glucides repartis en 2 repas
C - 13 % protéines, 37 % lipides, 50 % glucides répartis en 3 repas et 2 collations
D - 10 % protéines, 20 % lipides, 70 % glucides répartis en 3 repas et 2 collations
E - 10 % protéines, 50 % lipides, 40 % glucides repartis en 2 repas et 1 collation
Bonne(s) Réponse(s) : C

Connaissance.

Un adolescent de 15 ans consulte pour obésité, retard pubertaire et statural. Il pèse 60 kg pour une taille de 1m 45, les
testicules sont en place mais de faible volume, il y a quelques poils pubiens. l'adiposité est diffuse mais à prédominance facio-
tronculaire. Il existe une érythrose cutanée diffuse, la peau des pommettes est fine, on note quelques vergetures rosées sur
l'abdomen, les membres et la région mammaire qui est le siège d'une adipomastie. Les muscles sont grêles. La TA est à 15/9.
Il n'y a pas de mélanodermie.
Bilan hormonal :
- cortisol plasmatique (microg/100 ml) 8 h : 20 ; 16 h : 18; 24 h : 19
- 17 OH stéroïdes urinaires : 10 mg/24 h ; cortisol libre urinaire 110 microg/24 h (N= 20-60)
- test de Nugent cortisol plasmatique à 8 h : 18 (freinage après prise de 1 mg de déxaméthasone la veille à 24 h)
- perfusion de 4 mg de déxaméthasone pendant 4 h, cortisol plasmatique : 17 microg/100 ml
- ACTH plasmatique inférieur à 20 pg/ml 8h, 16 h, 24h (N = 30 à 50 pg/ml à 8h)
- les tests de stimulation de l'ACTH par hypoglycémie, LVP,CRF, sont négatifs
- testotérone plasmatique : 0,6 ng/ml (N = 3 à 4 ng/ml à 15 ans)
- LH et FSH plasmatiques non stimulables par LHRH
- aldostéronurie : 2 microg/ 24 h (N= 3 à 20)
- glycémie à jeûn : 1,10 g/l (6,11 mmol/l).
Radiographie de selle turcique : normale.
Le diagnostic de syndrome de Cushing est porté.

A la lecture du cas clinique, y a-t-il d'autres hypothèses diagnostiques que celle d'un syndrome de Cushing ?
Quelle(s) proposition(s) retenez-vous parmi les propositions suivantes ?
A - Syndrome adiposo-génital de Babinski-Frohlich
B - Syndrome de Conn
C - Hyperandrogénie cortico-surrénalienne
D - Myxoedème
E - Aucune des propositions précédentes
Bonne(s) Réponse(s) : E

Connaissance. 621
Exclusivement sur DOC - DZ : www.doc-dz.com NADJI 85
RESIDANAT EN POCHE TOME II
Cas Clinique en QCM

Dans ce cas clinique rapporté, quelle(s) est(sont) la(les) manifestation(s) clinique(s) compatible(s) avec le
diagnostic d'hypercorticisme glucocorticoïde de l'adolescent ?
A - Petite taille
B - Retard pubertaire
C - Obésité diffuse à prédominance facio-tronculaire
D - Elévation tensionnelle
E - Faible développement musculaire
Bonne(s) Réponse(s) : A B C D E

Le syndrome de Cushing constitue l'une des étiologies de retard pubertaire chez l'enfant.

Parmi les explorations hormonales effectuées, quelle(s) est(sont) celle(s) qui permet(tent) d'orienter le
diagnostic étiologique de ce syndrome de Cushing ?
A - Valeurs stables de la cortisolémie à 8 h, 16 h, 24 h
B - Absence de freinage au test de Nugent
C - Elevation du cortisol libre urinaire
D - Absence de réponse de LH et FSH au LH-RH
E - Valeurs constamment basses de l'ACTH plasmatique
Bonne(s) Réponse(s) : E

Le cycle du cortisol, le C.L.U. des 24 heures avec dosage de la créatininurie, l'absence de freinage au test de Nugent, sont
des éléments pour le diagnostic d'hypercortisolisme, mais non des éléments orientant vers une étiologie.
ACTH bas oriente vers une origine basse.

Compte tenu du bilan hormonal de cet adolescent, quelle est l'exploration morphologique non invasive que vous
demanderez en première intention pour orienter le diagnostic étiologique ?
A - Tomographie de la selle turcique
B - Tomodensitométrie hypophysaire
C - Radiographie de l'abdomen sans préparation
D - Tomodensitométrie surrénalienne
E - Scintigraphie à I'I131 méthyliodobenzylguanidine ( I MBG).
Bonne(s) Réponse(s) : E

Sans commentaire.

Parmi les propositions thérapeutiques suivantes, laquelle vous parait la mieux adaptée au cas présent ?
A - Freinage de l'hypercorticisme par la déxaméthasone
B - Traitement de fond par l'OP'DDD
C - Chirurgie surrénalienne
D - Radiothérapie hypophysaire
E - Chirurgie hypophysaire
Bonne(s) Réponse(s) : C

Les deux causes possibles sont l'adénome surrénalien bénin et le corticosurrénalome. Bien qu'il n'y ait pas d'élément clinique
permettant d'infirmer le corticosurrénalome, celui-ci semble peu probable dans ce contexte. De toute façon, la chirurgie sera
nécessaire dans les deux cas (sauf cas particulier).

Un malade de 35 ans consulte pour un goître nodulaire, non douloureux occupant la presque totalité du lobe droit. Il signale
des antécédents de radiothérapie cervicale dans l'enfance. Ce goître est de consistance très ferme. La VS est normale ainsi
que le bilan hormonal thyroïdien. Le nodule est hypocaptant au technétium 99, le reste du parenchyme thyroïdien fixe
normalement l'isotope. Une ponction du nodule est effectuée. La radiographie cervica!e révèle une déviation trachéale gauche
et une macrocalcification thyroïdienne. Il existe une paralysie récurrentielle droite à l'examen ORL.

Sur les données strictement cartographiques, quelle est l'hypothèse que l'on peut écarter ?
A - Hématocèle thyroïdienne
B - Nodule colloïde
C - Cancer thyroïdien
D - Nodule autonome extinctif
E - Adénome thyroïdien
Bonne(s) Réponse(s) : D

En cas de nodule autonome extinctif, celui-ci serait hyperfixant avec extension au parenchyme adjacent.

622
Exclusivement sur DOC - DZ : www.doc-dz.com NADJI 85
RESIDANAT EN POCHE TOME II
Cas Clinique en QCM

La ponction du nodule ne ramène qu'un demi ml de liquide brun foncé ce qui ne modifie pas son volume. Parmi
les propositions suivantes, quelle(s) est(sont) celle(s) que l'on peut exclure ?
A - Kyste thyroïdien vrai
B - Hématocèle simple isolée
C - Hématocèle localisée au sein d'un adénome
D - Hématocèle localisée au sein d'un épithélioma
E - Thyroïdite nodulaire
Bonne(s) Réponse(s) : A B E

La ponction d'une thyroïde nodulaire est négative (ou blanche).

L'analyse du liquide de ponction met en évidence des cellules thyroïdiennes suspectes de malignité. Parmi les
propositions suivantes tirées de l'observation, quelle(s) est(sont) celle(s) en faveur de la malignité ?
A - Refoulement de la trachée vers la gauche
B - Paralysie du récurrent droit
C - Antécédents de radiothérapie cervicale
D - Présence d'une macrocalcification
E - Absence d'effondrement du nodule après ponction
Bonne(s) Réponse(s) : B C

Le refoulement trachéal est uniquement dû à la taille de la tumeur.

Après examen histologique extemporané per-opératoire, une thyroïdectomie totale est effectuée. Il s'agit d'un
cancer papillaire. Parmi les explorations suivantes, quelle(s) est(sont) celle(s) qui est(sont) utile(s) à pratiquer
en post-opératoire ?
A - Dosage de calcitonine
B - Dosage de l'antigène carcino-embryonnaire
C - Dosage de l'alpha foeto protéine
D - Bilan phosphocalcique
E - Recherche d'un pheochromocytome
Bonne(s) Réponse(s) : D

Sans commentaire.

Après thyroïdectomie totale, il n'y a pas de fixation cervicale de I'I 131. Parmi les propositions suivantes,
quelle(s) est(sont) celle(s) qui est(sont) adaptée(s) à l'état pathologique ?
A - Indication d'un traitement par l'I131
B - Hormonothérapie thyroïdienne bloquant la réactivité de TSH à TRH
C - Dosage régulier de la thyroglobuline sous hormonothérapie thyroïdienne
D - Contrôle régulier de la radiographie de thorax
E - Aucune de ces propositions
Bonne(s) Réponse(s) : B C D

Sans commentaire.

Une malade de 38 ans, mère de 3 enfants, présente une aménorrhée secondaire depuis 6 mois, une prise de poids de 4 kg,
une perte de cheveux, un gonflement des doigts. A l'examen, pas de galactorrhée, TV normal, poids à 64 kg pour 1,58 m,
goiîre diffus induré, indolore.

Quel est l'examen à pratiquer en première intention ?


A - Hystérographie
B - Scanner hypophysaire
C - Echographie ovarienne
D - Mesure de la TSH
E - Un dosage de prolactine
Bonne(s) Réponse(s) : D

Le tableau évoque une hypothyroïdie. La TSH est le paramètre le plus précocement touché
(s'élève en cas d'hypothyroïdie d'origine basse).

623
Exclusivement sur DOC - DZ : www.doc-dz.com NADJI 85
RESIDANAT EN POCHE TOME II
Cas Clinique en QCM

Quel est le diagnostic à envisager ?


A - Acromégalie
B - Prolactinome
C - Maladie de Basedow
D - Thyroïdite d'Hashimoto
E - Thyroïdite de De Quervain
Bonne(s) Réponse(s) : D

Sans commentaire.

Les examens biologiques courants montrent, glycémie, 8 mmol ; cholestérol, 7,2 mmol ; triglycérides, 2,6 mmol ;
créatinine, 90 micromol. Quel traitement choisissez-vous ?
A - Daonil® 2 cp
B - Bactrim® forte 2 cp
C - Glucophage® retard 2 cp
D - Lipavlon® 6 cp
E - Insuline 20 u
Bonne(s) Réponse(s) : C

Mauvaise question, il ne faut jamais prescrire un antidiabétique oral avant d'avoir prescrit un régime hypocalorique
pondéral(car surcharge) adapté.

Sa tension artérielle mesurée à une seule reprise est à 17 - 9,5 cm de Hg. Quelle proposition retenez-vous ?
A - Aldomet® 250 2 cp
B - Catapressan® 3 cp
C - Lopril® 25 2 cp
D - Régime sans sel strict
E - Simple surveillance de la TA
Bonne(s) Réponse(s) : E

On n'institue jamais un traitement antihypertenseur sur un seul chiffre tensionnel non


menaçant.

Quel traitement proposez-vous pour son goitre ?


A - Iode radioactif
B - Thyroïdectomie subtotale
C - Antithyroïdiens de synthèse
D - Iode minérale (Lugol®)
E - Thyroxine
Bonne(s) Réponse(s) : E

Soit :
- Lévothyrox® LT4 (un comprimé = 50 ou 100 µg).
- Thyroxine DLT4.

Un diabétique de 35 ans, insulinodépendant, est mal équilibré. Depuis plusieurs mois, il présente une glycosurie aux examens
semi-quantitatifs quotidiens, fait de nombreux malaises hypoglycémiques, et grossit. Il est traité par une injection quotidienne
matinale de 50 unités d'insuline lente Novo.

L'hypoglycémie :
A - Peut se traduire par des sueurs
B - Peut être à l'origine d'accès de bradycardie
C - Ne peut être affirmée qu'après une épreuve de jeun
D - Entraîne une libération de glucagon
E - Peut se traduire par une crise comitiale
Bonne(s) Réponse(s) : A D E

Sans commentaire.

624
Exclusivement sur DOC - DZ : www.doc-dz.com NADJI 85
RESIDANAT EN POCHE TOME II
Cas Clinique en QCM

Quelle modification thérapeutique envisagez-vous ?


A - Augmentation de la dose d'insuline
B - Diminution de la dose d'insuline
C - Augmentation de la ration glucidique
D - Diminution de la ration glucidique
E - Passage à 2 injections quotidiennes d'insuline, sans modification de la dose totale
Bonne(s) Réponse(s) : B

Le malade fait des malaises hypoglycémiques car il est surdosé en insuline, mais au lieu de diminuer sa dose d'insuline, il
prend du sucre, ce qui de plus le fait grossir.

Si vous deviez prescrire 2 injections quotidiennes d'insuline, que pourriez-vous choisir ?


A - Insuline IPZ®
B - Insuline Actrapid®
C - Insuline NPH®
D - Insuline Rapitard®
E - Durasuline®
Bonne(s) Réponse(s) : C D

Ce sont deux insulines semi-lentes, la Rapitard® contenant en plus une composante ordinaire.

Un contrôle métabolique strict, chez cet homme jeune :


A - Serait dangereux en raison du risque d'accident hypoglycémique
B - Ne peut permettre de prévenir la survenue de complications dégénératives qui sont d'origine génétique
C - Ne pourra être otenu qu'en milieu hospitalier
D - Nécessite l'apprentissage par le patient de la pratique d'auto-glycémies capillaires
E - Nécessité le pancréas artificiel
Bonne(s) Réponse(s) : D

Connaissance.

Quel est le meilleur critère d'un bon contrôle pour une période prolongée ?
A - Hémoglobine glycosylée
B - Absence de cétonurie
C - Chiffre normal de C-peptide urinaire
D - Glycémie normal après le repas de midi
E - Hyperglycémie provoquée normale
Bonne(s) Réponse(s) : A

L'hémoglobine glycosylée A1C reflète l'équilibre glycémique des deux à trois mois précédents. Il est souhaitable d'en pratiquer
le dosage dans le même laboratoire.

Un agriculteur de 54 ans se plaint de raideur et de douleur du genou. La santé de ce sujet a toujours été bonne jusqu'à une
entorse du genou 10 ans auparavant. Depuis, il a une limitation progressive du mouvement de cette articulation avec douleur
et raideur. Il ne présente aucun antécédent important hormis une prise de poids progressive les 20 dernières années. Ses
traits se sont burinés, ce qu'il attribue à son travail à l'extérieur par tous les temps. Sa pointure de chaussure a augmenté
récemment. A l'examen, sa pression artérielle est à 160/90, il présente une hypertrophie thyroïdienne, une hépatomégalie
sans splénomégalie. La limitation de mouvement du genou droit est évidente. Le champ visuel est normal. Les examens
biologiques montrent :
- une hyperglycémie à jeun et une hyperglycémie provoquée de type diabétique
- un taux élevé à jeun de l'hormone de croissance.
- des hormones thyroïdiennes normales.

Chez un tel patient, il est habituel de retrouver la(les) modifications(s) suivante(s) sur la radiographie du crâne :
A - Elargissement des sinus frontaux
B - Elargissement des sinus maxillaires
C - Protubérance occipitale accentuée
D - Elargissement et déformation de la selle turcique
E - Prognathisme
Bonne(s) Réponse(s) : A B C D E

Sans commentaire.

625
Exclusivement sur DOC - DZ : www.doc-dz.com NADJI 85
RESIDANAT EN POCHE TOME II
Cas Clinique en QCM

Au cours de cette maladie, il est habituel que la radiologie montre également :


A - Cardiomégalie
B - Ostéophytose
C - Elargissement des tissus mous (signe du coussinet talonnier)
D - Amincissement de la corticale des os longs
E - Déminéralisation diffuse
Bonne(s) Réponse(s) : A B C E

Sans commentaire.

A propose du cas présenté, il est exacte que :


A - Le diabète est fréquent et peut révéler celle maladie
B - L'insulinémie est basse chez ces patients en début d'évolution
C - L'hypertrophie thyroïdienne entre dans le cadre de la mégasplanchnie
D - Le taux d'hormone de croissance sera habituellement freiné lors de l'hyperglycémie provoquée
E - L'hormone de croissance est stimulable de façon constante par TRH
Bonne(s) Réponse(s) : A C E

La réponse de la GH sous TRH constitue une réponse paradoxale de la GH.

Cliniquement et biologiquement, l'activité de cette maladie se manifeste par :


A - Douleurs articulaires
B - Diabète
C - Hypophosphorémie
D - Sueurs
E - Altérations du champ visuel
Bonne(s) Réponse(s) : A B D E

D E - Est litigieux car ces deux signes ne sont pas présents dans l'énoncé, mais il semble prudent de les mettre.

Quel(s) traitement(s) de fond peu(ven)t être envisagé(s) chez ce patient ?


A - Chirurgie de l'adénome
B - Traitement radiothérapique
C - Prise de Parlodel®
D - Perfusion de somatostatine
E - Aucun de ces traitements
Bonne(s) Réponse(s) : A B

Le Parlodel® marche mal. La somatostatine en cours d'étude donne des résultats encourageants, mais ne constitue pas un
traitement de fond.

Une étudiante de 21 ans, en vacances, est amenée aux urgences inconsciente à 21 heures. Le seul renseignement est qu'elle
jouait au tennis pendant une grande partie de la journée et se plaignait de nausées. Elle a vomi à plusieurs occasions, s'est
plaint de vertiges, puis est devenue confuse avant de perdre conscience. A l'examen, elle est confuse mais consciente, avec
un bronzage important. Sa pression artérielle est à 8,5/4 cm Hg avec une fréquence cardiaque à 130/mn. Sa température est
normale. Il existe un pli cutané. Les muqueuses sont sèches mais les globes oculaires sont souples. La pigmentation est
accrue au niveau des mamelons. Les appareils cardio-vasculaire, pulmonaire, abdominal et génital sont normaux. Aucune
anomalie n'est retrouvée à l'examen neurologique. L'ECG montre une tachycardie sinusale avec grandes ondes T. A la
numération : Hb = 160 g/l, Ht = 50 %, GB = 12 400 avec 50 % de Iymphocytes, 41 % de polynucléaires neutrophiles et 9 %
d'éosinophiles, radio de thorax normale. Glycémie à 2,5 mmol/l, Na = 120 mmol/l, K = 6,1 mmol/l, bicarbonates = 20 mmol/l.
Il n'y a pas d'antécédents pathologiques connus, ni de prise thérapeutique. Le diagnostic d'insuffisance surrénale aiguë est
évoqué. Il est demandé des explorations hormonales :
- la cortisolémie est basse
- le dosage de l'ACTH plasmatique est élevé.

Parmi les éléments suivants, rapportés dans l'observation, quel(s) est(sont) celui(ceux) qui peu(ven)t expliquer
la perte de connaissance ?
A - La fréquence cardiaque
B - Le niveau de la tension artérielle
C - Le taux de la glycémie
D - Le taux de la kaliémie
E - Le taux des bicarbonates
Bonne(s) Réponse(s) : B C

Au cours de l' ISA., une perte de connaissance peut être due à une hypoTA s'aggravant en
orthostatique, à une hypoglycémie, à une hyponatrémie.
La tachycardie est un mécanisme de compensation de l'état de choc.

626
Exclusivement sur DOC - DZ : www.doc-dz.com NADJI 85
RESIDANAT EN POCHE TOME II
Cas Clinique en QCM

Parmi les éléments cliniques et biologiques suivants rapportés dans l'observation, quel(s) est(sont) celui(ceux)
qui est(sont) en faveur du diagnostic ?
A - Les nausées et vomissements
B - Le bronzage important
C - Les signes de déshydratation
D - L'hypotension artérielle
E - L'apyrexie
Bonne(s) Réponse(s) : A B C D

Dans l'ISA, peuvent se voir soit une fièvre soit une hypothermie, soit une température normale. Le bronzage important, en
rapport avec une mélanodermie, est un signe d'hypersécrétion.

Cette jeune fille n'a pas d'antécédent de tuberculose. La cuti-réaction à la teberculose se révèle négative, quel
diagnostic étiologique peut-on envisager ?
A - Une insuffisance corticosurrénale hypôphysaire
B - Une pathologie auto-immune
C - Une maladie infectieuse
D - Un traumatisme
E - Une origine iatrogène
Bonne(s) Réponse(s) : B

A - Une IS d'origine hypophysaire s'accompagne d'un taux d'ACTH bas.


E - La principale origine iatrogène est le sevrage, brutal, total ou relatif, au cours ou au décours d'une corticothérapie d'autant
qu'elle a été plus intense et plus prolongée. Chez elle, on élimine dans le texte une prise médicamenteuse, un acte chirurgical.

Dans l'immédiat, quelle(s) mesure(s) thérapeutique(s) faut-il prescrire ?


A - Perfusion de sérum glucosé hypertonique
B - Perfusion de sérum bicarbonaté
C - Perfusion de sérum salé
D - Apport de KCL dans la perfusion de sérum glucosé
E - Transfusion d'une purée de globules rouges
Bonne(s) Réponse(s) : C E

Il faut apporter du sérum glucosé isotonique, associé à 12 grammes de sel par litre, sans potassium
(hyperkaliémie). Des transfusions de GR ou de macromolécules sont nécessaires en cas de collapsus.

Parmi les médications suivantes, quelle(s) est(sont) celle(s) qui est(sont) conseillée(s) ?
A - Adrénaline
B - Hydrocortisone
C - Cortancyl®
D - Syncortyl®
E - Synacthène®
Bonne(s) Réponse(s) : B D

Le traitement subtitutif doit compenser le déficit en glucocorticoïdes et en minéralcorticoïdes.

627
Exclusivement sur DOC - DZ : www.doc-dz.com NADJI 85
RESIDANAT EN POCHE TOME II
Cas Clinique en QCM
Une femme de 75 ans, traitée depuis deux ans pour un diabète, est retrouvée chez elle à midi par sa fille qui l'avait quittée à 8
h, avant qu'elle prenne son petit déjeuner, mais après avoir pris ses médicaments habituels et une dragée de Trinitrine®, car
elle ne se sentait pas très bien. Le médecin appelé fait le diagnostic de coma hypoglycémique et pratique une injection
intraveineuse de sérum-glucosé qui réveille aussitôt la malade. Celle-ci continue son traitement habituel mais s'alimente
légèrement.
L'interrogatoire nous apprend que le malaise a dû survenir de façon assez brutale sans aucun prodrome classique habituel :
pas de palpitation, pas de transpiration, mais chute relativement brutale dans sa cuisine.
Le lendemain matin, à 8 h, le médecin est rappelé pour récidive du coma et il adresse la malade à l'hôpital. A l'arrivée, un
bilan d'urgence est pratiqué qui montre une glycémie à 1,6 mmol/l, une urée à 14 mmol/l, une creatininémie à 200 mmol/l. Le
bilan neurologique et vasculaire est strictement normal, en dehors d'une hypertension artérielle à 16/9. La malade est aussitôt
traitée par sérum-glucosé et reprend très rapidement conscience.
Il s'agit d'une femme de 86 kgs pour 1m63 qui présente son ordonnance habituelle et celle-ci comporte :
- un régime hypocalorique hypoglucidique mal suivi.
- Avlocardyl® 40 : 2/jour.
- Daonil® : 1 comprimé le matin, midi et soir, de prescription récente
- Catapressan® : 1 comprimé et demi par jour
- digitaliques : 5 Jours par semaine.
Le diagnostic de son médecin traitant était celui de diabète non insulinodépendant chez une obèse avec hypertension
artérielle relativement importante associée à une coronarite et à une insuffisance cardiaque modérée.

Parmi les facteurs suivants, le(s)quel(s) est(sont) susceptible(s) d'avoir joué un rôle dans la montée récente du
diabète ?
A - Facteurs psychologiques liés à la retraite
B - Diminution de l'activité physique
C - Relâchement du régime alimentaire
D - Prise de poids survenue depuis 1 an
E - Traitement hypotenseur
Bonne(s) Réponse(s) : A B C D

Le Catapressan®, ainsi que les autres antihypertenseurs centraux, ne modifie pas l'équilibre glycémique, au contraire des
diurétiques (Lasilix®-thiazidiques). Les béta-bloquants ne déséquilibrent pas un diabète mais doivent être employés avec
prudence (béta-bloquants cardiosélectifs) car ils masquent les signes d'hypoglycémie.

Pour faire le point du diabète de ce malade, il est utile de demander :


A - Un dosage d'insuline et de C-peptide
B - Un fond d'oeil
C - Une recherche de protéinurie
D - Une urographie intraveineuse
E - Un dosage de cortisol et d'ACTH
Bonne(s) Réponse(s) : A B C

B C - Sont effectués dans le cadre de la surveillance des complications du diabète.


A - Dosage apportant peu de renseignements, car il ne présente pas de signes de carence en insuline = réponse litigieuse.

La biologie devrait vraisemblablement montrer chez ce malade :


A - Une hyperuricémie
B - Une hypercalcémie
C - Une hyponatrémie
D - Une hypertriglycéridémie
E - Une hypercholestérolémie
Bonne(s) Réponse(s) : A D E

D - L'hypertriglycéridémie est en rapport avec l'hyperglycémie prolongée.


A E - Existent vraisemblablement chez ce patient pléthorique.

Pour rééquilibrer ce diabète, vous allez conseiller à ce malade :


A - Un régime pratiquement sans glucides
B - Un régime hypocalorique global
C - La reprise progression d'un exercice physique adapté
D - Une augmentation des doses d'hypoglycémiants oraux
E - Un passage à l'insulinothérapie
Bonne(s) Réponse(s) : B C

Le régime et l'exercice physique sont les bases du traitement du DNID.

628
Exclusivement sur DOC - DZ : www.doc-dz.com NADJI 85
RESIDANAT EN POCHE TOME II
Cas Clinique en QCM

Pour surveiller dans l'avenir l'équilibre de son diabète, vous lui conseillez de :
A - Ne rien changer à ce qu'il faisait
B - Lors des contrôles glycémiques, faire en pus une glycémie post-prandiale
C - Doser tous les 3 à 6 mois l'hémoglobine A1C
D - Faire tous les ans une hyperglycémie provoquée par voie orale
E - Faire tous les ans un dosage d'anticorps anti-insuline
Bonne(s) Réponse(s) : B C

Il faut aussi conseiller au patient de surveiller ses glycémies capillaires.

Une jeune femme de 25 ans consulte pour amaigrissement (-4 kg en 2 mois), nervosité, tachycardie à 120, thermophobie,
augmentation de la soif, diarrhée, petit goître diffus prédominant sur le lobe droit, vasculaire. Il existe une asthénie musculaire.
Les yeux sont brillants, les paupières rétractées, il existe une protrusion oculaire bilatérale avec un oedème palpébral
important avec chémosis et une diplopie. Il existe un vitiligo et un myxoedème prétibial. Des explorations sont effectuées :
- T4 : 140 ng/ml (N = 45-115)
- T3 : 2,5 ng/ml (N =0 ,8 à 1,8)
- TSH : 1 microU/ml (N = 0-7) non stimulable par TRH
- hémogramme : 5000 GB (poly neutro 50%) - 4500000 GR - HB : 15 g/1OO ml
- cartographie au 99 TC : captation homogène.
Le diagnostic de maladie de Basedow est porté.

Parmi les manifestations suivantes, présentes chez la malade, les valeurs de T4 et de T3 sont en relation avec :
A - Tachycardie
B - Thermophobie
C - Augmentation de la soif
D - Diarrhée
E - Exophtalmie
Bonne(s) Réponse(s) : A B C D

La question est mal posée, car c'est l'élévation des hormones qui entraîne les manifestations cliniques. L'exophtalmie n'est
pas une conséquence de l'élévation des H thyroïdiennes, mais est en rapport avec les désordres auto-immuns.

Si l'on considère, dans cette observation, uniquement la valeur de la TSH, vous devez envisager parmi les
étiologies suivantes :
A - Maladie de Basedow avec thyrotoxicose
B - Nodule autonome de la thyroïde
C - Hypothyroïdite secondaire à une thyroïdite
D - Hypothyroïdite hypophysaire
E - Hyperthyroïdie par adénome thyréotrope
Bonne(s) Réponse(s) : A B

Dans ce contexte d'hyperthyroïdie, la TSH basse oriente vers deux étiologies de l'hyperthyroïdie primitive.

Parmi les manifestations cliniques de cette malade, vous rattachez à la maladie de Basedow et non à la
thyrotoxicose :
A - Goître
B - Diplopie
C - Vitiligo
D - Myxoedème prétibial
E - Amaigrissement
Bonne(s) Réponse(s) : A B D

Le vitiligo, comme d'autres maladies auto-immunes peut être associé à la maladie de Basedow, mais n'est pas un signe
clinique de la maladie.

Parmi les thérapeutiques suivantes de la thyrotoxicose, vous pouvez prescrire d'emblée chez cette femme :
A - I131
B - Carbimazole (Néomercasole®)
C - Chirurgie
D - Propranolol (Avlocardyl®)
E - Lugol Fort®
Bonne(s) Réponse(s) : B D E

Le Lugol® fort est actuellement peu utilisé. Il peut être utile pour la préparation à un traitement chirurgical.

629
Exclusivement sur DOC - DZ : www.doc-dz.com NADJI 85
RESIDANAT EN POCHE TOME II
Cas Clinique en QCM

Pour traiter l'exophtalmie de cette jeune femme, vous pouvez choisir :


A - Thyroïdectomie sub-totale
B - Corticothérapie
C - Traitement immunodépresseur
D - Décompression orbitaire (intervention d'Ogura)
E - Traitement immédiat par LT3 (Cynomel®)
Bonne(s) Réponse(s) : B C D

Les complications oculaires (exophtalmie oedémateuse) sont susceptibles d'être aggravées par les traitements radicaux,
notamment une thyroïdectomie subtotale.

Une femme de 27 ans consulte pour une aménorrhée d'un an. Elle a accouché 3 ans auparavant dans des conditions tout à
fait normales. Elle a ensuite pris une contraception orale (Gynophase (r)) pendant 2 ans, sans problème particulier, sinon une
hypoménorrhée.
Elle n'a pas été réglée depuis l'arrêt de la pilule. L'examen clinique retrouve :
- un utérus de taille normale rétroversé
- les culs de sacs ne sont pas douloureux mais les annexes ne sont pas nettement perçues
- l'examen des seins met en évidence une galactorrhée bilatérale que la patiente dit effectivement présenter depuis
l'accouchement de son dernier-né sans que cela ne l'ait inquiétée. Il est pratiqué un dosage de prolactinémie objectivant un
taux de 120 ng/ml.

Quelle est la cause la plus probable de cette aménorrhée parmi celles proposées ?
A - Dystrophie ovarienne
B - Contraception oestroprogestative
C - Grossesse débutante
D - Microadénome à prolactine
E - Insuffisance rénale
Bonne(s) Réponse(s) : D

Sans commentaire.

Parmi les médicaments énumérés ci-dessous, quel(s) est(sont) celui(ceux) qui peu(ven)t induire une
aménorrhée hyperprolactinémique ?
A - Sulpiride (Dogmatil®)
B - Clomipramine (Anafranil®)
C - Cimétidine (Tagamet®)
D - Alpha méthyldopa (Aldomet®)
E - Furosémide (Lasilix®)
Bonne(s) Réponse(s) : A B C D

Sans commentaire.

Dans le cadre de cette observation, vous devez demander en priorité :


A - Etude du champ visuel
B - Scanner hypophyso-hypotalamique
C - Dosage de progestérone plasmatique
D - Dosages plasmatiques FSH-LH
E - Test du LHRH
Bonne(s) Réponse(s) : A B

Sans commentaire.

Les examens que vous avez demandés ne montrent aucune anomalie, parmi les thérapeutiques proposées,
laquelle choisissez-vous dans le cadre de cette observation ?
A - Traitement par inducteur de l'ovulation
B - Traitement par bromocriptine (Parlodel®)
C - Traitement neurochirurgical
D - Abstention
E - Cycles artificiels à l'aide d'oestroprogestatifs
Bonne(s) Réponse(s) : B

Sans commentaire.

630
Exclusivement sur DOC - DZ : www.doc-dz.com NADJI 85
RESIDANAT EN POCHE TOME II
Cas Clinique en QCM

Dans ce cas la survenue d'une grossesse :


A - Expose au risque de grossesse multiple
B - Est un facteur d'aggravation de l'hyperprolactinémie
C - Entraîne la guérison définitive
D - Impose l'interruption thérapeutique de la grossesse
E - Autorise la poursuite de la grossesse sous stricte surveillance
Bonne(s) Réponse(s) : B E

Sans commentaire.

Le Gynophase® est un contraceptif oral de type :


A - Combiné minidosé monophasique
B - Combiné minidosé biphasique
C - Combiné faiblement dosé triphasique
D - Combiné normodosé séquentiel
E - Combiné normodosé biphasique
Bonne(s) Réponse(s) : E

Sans commentaire.

Dans l'hypothèse où une aménorrhée post pilule aurait été retenue, quel(s) est(sont) le(les) traitement(s) que
l'on peut proposer pour rétablir l'ovulation après avoir éliminé une cause organique ?
A - FSH
B - Citrate de Clomifène
C - Estradiol 17 bêta (Progynova®)
D - Bromocriptine (Parlodel®)
E - Lynestrénol (Orgamétril®)
Bonne(s) Réponse(s) : B

Le Clomid® (clomifène) a une action d'inhibition compétitive du rétro-contrôle des oestrogènes au niveau hypothalamique, ce
qui entra;ne une élévation de FSH responsable de la maturation folliculaire, qui elle-même favorise l'hypersécrétion
d'oestradiol, et ainsi la survenue d'un pic de LH, déclenchant l'ovulation.

Monsieur R..., 50 ans, ouvrier maçon, se voit découvrir une glycosurie à l'examen de la médecine du travail, il n'a pas
d'antécédent pathologique personnel important. Un de ses oncles est diabétique traité par comprimés. Monsieur R.
consomme 1 litre de vin et 1 ou 2 bocks de bière par jour, il est fatigué, mais a conservé un bon appétit. Il rapporte que depuis
quelque temps il se lève deux fois par nuit pour uriner. A l'examen clinique, on constate un poids (resté stable) à 66 kg pour
1,72 m, un foie modérément augmenté de volume, non dur ; l'absence d'autre anomalie. Le bilan biologique donne les
résultats suivants : glycémie (à jeun sur plasma veineux) à 13,5 mmol/l, cholestérolémie à 5,6 mmol/l, triglycéridémie à 3
mmol/l, créatininémie à 93 microniol/l, transaminases glutamo-oxalacétiques (ASAT) à 28 UI/l, transaminases glutamo-
pyruviques (ALAT) à 25 UI/l, phosphatases alcalines à 100 UI/l, gammaglutamyl-transférases à 160 UI/l, albumialbuminémie à
45g/l, taux de prothrombine à 45 %.
Vous avez à prendre en charge le traitement de ce patlent.

De manière hypothétique, quelle(s) est(sont) la(les) condition(s) qui rendrait (rendraient) d'emblée obligatoire
l'insulinothérapie chez ce patient ?
A - Glycémie régulièrement égale ou supérieure à celle constatée lors du premier bilan
B - Hémoglobine glycosylée supérieure à 10 %
C - Amaigrissement de 3 kilos
D - Présence régulière d'une cétonurie malgré une alimentation normale et l'arrêt de l'alcool
E - Aucune des propositions ci-dessus
Bonne(s) Réponse(s) : C D

Sans commentaire.

Au plan diététique, il vous paraît judicieux de conseiller à ce patient :


A - Arrêter les boissons alcoolisées
B - Diminuer de façon importante les glucides d'assimilation rapide
C - Réduire de façon importante le pain et les fécules
D - Consommer plutôt du pain au son que du pain blanc
E - Préférer, pour la salade, l'huile d'olive à l'huile de tournesol
Bonne(s) Réponse(s) : A B D

Le régime doit apporter une quantité suffisante de glucides d'absorption lente (55 % de la ration calorique globale). Le son
permet de diminuer le pic hyperglycémique post-prandial. Le régime doit apporter des lipides, tels que l'huile de tournesol,
polyinsaturés.

631
Exclusivement sur DOC - DZ : www.doc-dz.com NADJI 85
RESIDANAT EN POCHE TOME II
Cas Clinique en QCM

Vous décidez de ne pas choisir les biguanides comme premier traitement médicamenteux de ce patient,
pourquoi ?
A - Parce que vous n'êtes pas sûr d'obtenir un arrêt ni même une diminution des boissons alcoolisées
B - Parce que le bilan biologique montre chez lui une insuffisance hépatique
C - Parce que le niveau de la créatinémie ne le permet pas
D - Parce que le poids du patient n'y incite guère
E - En raison du risque hypoglycémique
Bonne(s) Réponse(s) : A B D E

Les hypoglycémies ne se rencontrent en principe pas car les biguanides ne sont pas insulino-sécrétants, néanmoins il y a
potentialisation de l'effet lorsque cette médication est associée aux sulfamides et surtout lorsqu'ils sont pris à jeun, associés à
de l'alcool. Il y a en effet dans ce cas là une potentialisation de l'inhibition de la néoglycogénèse hépatique.

Les effets du régime étant insuffisants, vous décidez de prescrire, en ambulatoire, un sulfamide hypoglycémiant.
Quelle(s) proposition(s) vous paraît(paraissent) adaptée(s) pour la prescription et la surveillance de ce
traitement ?
A - Metformine (Glycophage®), 3 comprimés par jour
B - Gliclazide (Diamicron®) ou glibomuride (Glutril®) en débutant par 1/2 comprimé avant les trois repas
principaux
C - Glibenclamide (Daonil® ou Euglucan®), en commençant par un comprimé chacun des trois principaux
repas
D - Autosurveillance glycémique effectuée 5 fois par jour au long cours
E - Hémoglobine glycosylée dosée une fois par semaine
Bonne(s) Réponse(s) : B

Il faut choisir un sulfamide, de métabolisme rénal, de demi-vie courte ou intermédiaire, en débutant le traitement par une
posologie modérée, afin d'éviter les hypoglycémies des sulfamides hypoglycémiants.

Concernant les lipides sanguins, quelle(s) est (sont) la (les) proposition(s) exacte(s) ?
A - Le cholestérol est élevé
B - Les triglycérides sont élevées
C - La consommation d'alcool peut être responsable des troubles observés
D - Le déséquilibre du diabète peut être responsable des troubles observés
E - Un traitement par cholestyramine (Questran®) se trouve indiqué éventuellement en association avec le
traitement médicamenteux du diabète
Bonne(s) Réponse(s) : B C D

Sans commentaire.

Chez un homme de 50 ans, est découvert au cours d'un examen systématique un nodule thyroïdien du lobe droit, ferme,
homogène, mobile à la déglutition, mesurant 2 cm de diamètre. Il n'y a pas de manifestations cliniques de dysfonctionnement
hormonal, de signes inflammatoires locaux ou d'adénopathies cervicales. L'examen général du patient est strictement normal.

Le(s) diagnostic(s) à envisager est(sont) :


A - Adénome toxique
B - Kyste thyroïdien
C - Thyroïdite de De Quervain
D - Adénome non fonctionnel
E - Carcinome thyroïdien
Bonne(s) Réponse(s) : B D E

L'adénome toxique s'accompagne en sus d'hyperthyroïdie.


Lors de la thyroïdite de De Quervain, la thyroïde est typiquement douloureuse, augmentée de volume de façon diffuse, avec
diminution de la mobilité par rapport à la trachée. Parfois il existe une hypertrophie isolée d'un lobe, douloureuse ou un nodule
mal limité, douloureux.

L'(les)examen(s) paraclinique(s) à demander en premier pour vous aider dans votre recherche diagnostique
est(sont) :
A - Radiographie du cou
B - Test à la TRH sur TSH
C - Scintigraphie thyroïdienne
D - Radiographie du thorax
E - Echographie thyroïdienne
Bonne(s) Réponse(s) : C E

Sans commentaire.

632
Exclusivement sur DOC - DZ : www.doc-dz.com NADJI 85
RESIDANAT EN POCHE TOME II
Cas Clinique en QCM

En cas de nodule thyroïdien non kystique et hypofixant, l'attitude thérapeutique sera :


A - Ponction évacuatrice
B - Antithyroïdien de synthèse
C - Radiothérapie locale
D - Solution de Lugol®
E - Intervention chirurgicale
Bonne(s) Réponse(s) : E

Sans commentaire.

Si le nodule est hypoéchogène, le(s) traitement(s) à proposer en première intention sera (seront) :
A - Intervention chirurgicale
B - Freination par L-thyroxine
C - Anti-tyroïdiens de synthèse
D - Ponction évacuatrice du nodule
E - Solution de Lugol
Bonne(s) Réponse(s) : D

Sans commentaire.

Une femme de 55 ans, sans antécédent particulier, consulte pour une prise de poids de 5 kg sans majoration de son appétit.
Elle se plaint de frilosité, de constipation, de gonflement des doigts, de somnolence et de perte de mémoire entravant son
activité. A l'examen, faciès bouffi, peau sèche, perte des poils axillo-pubiens, poids : 68 kg, taille : 166 cm, TA : 12 - 7, coeur
assourdi à 64/mn.

Le diagnostic le plus probable est :


A - Acromégalie
B - Diabète
C - Myxoedème
D - Cushing
E - Troubles post-ménopausiques
Bonne(s) Réponse(s) : C

Sans commentaire.

Parmi les examens suivants, quel est celui que vous demandez pour confirmer le diagnostic ?
A - Scintigraphie thyroïdienne
B - Scanner hypophysaire
C - Dosage de la FSH
D - Dosage de la TSH
E - Dosage de l'Estradiol
Bonne(s) Réponse(s) : D

Sans commentaire.

Quel traitement proposez-vous ?


A - Estrogel® percutané
B - Anorexigènes
C - L-Thyroxine 100 microg/j
D - Cortisone 30 mg/j
E - Bromocriptine 10 mg/j
Bonne(s) Réponse(s) : C

Sans commentaire.

Les examens biologiques montrent : cholestérol : 8 mmol/l, glycémie : 5,6 mmol/l, triglycérides : 1,35 mmol/l.
Quelle mesure thérapeutique ou diététique proposez-vous ?
A - Régime sans graisse
B - Lipanthyl® 3 cp par Jour
C - Questran® 4 sachets par jour
D - Régime sans sucre
E - Aucune des prescriptions ci-dessus
Bonne(s) Réponse(s) : E

Sans commentaire.
633
Exclusivement sur DOC - DZ : www.doc-dz.com NADJI 85
RESIDANAT EN POCHE TOME II
Cas Clinique en QCM
Une femme âgée de 25 ans consulte son médecin parce que depuis plusieurs mois elle présente une fatigue anormale, qui va
croissant au cours de la journée. Au cours des dernières semaines, cette fatigue s'est accentuée, en même temps
qu'apparaissait une anorexie globale, accompagnée parfois d'un état nauséeux. L'entourage de la patiente a remarqué qu'elle
a bronzé beaucoup plus que les étés précédents, et elle s'en étonne dans la mesure où elle n'est pas beaucoup sortie de chez
elle. Elle a maigri et chiffre cet amaigrissement à 4 kg ou 5 kg (52 kg pour 1,65 m actuellement). La survenue
d'étourdissements lorsqu'elle se relève à partir des positions assise ou couchée l'amène à consulter son médecin.
L'interrogatoire ne met en évidence aucun antécédent personnel particulier. L'examen clinique confirme l'amaigrissement et
une hyperpigmentation brune diffuse accentuée aux régions exposées au soleil et aux plis d'extension des doigts. La tension
artérielle est à 9-6. Le médecin évoque en première ligne une insuffisance surrénale et adresse cette patiente à l'hôpital.

Il faut obtenir en urgence :


A - Le cortisol plasmatique
B - La kaliémie
C - La natrémie
D - L'ACTH
E - L'aldostérone
Bonne(s) Réponse(s) : A B C

Sans commentaire.

Devant cette insuffisance surrénale confirmée, quel élément clinique retrouvé dans l'observation et/ou quel
élément biologique ultérieur affirmeront (ou affirmera) la nature primitive ?
A - La valeur de la kaliémie
B - L'asthénie
C - Les nausées
D - L'hypotension orthostatique
E - La mélanodermie
Bonne(s) Réponse(s) : E

La mélanodermie est témoin d'une hypersécrétion d'ACTH, témoin elle-même d'une IS primitive.

Pour mieux cerner l'étiologie de cet hypocorticisme, il faut demander :


A - Un cliché de l'abdomen sans préparation
B - Une intradermo-réaction à la tuberculine
C - L'ACTH plasmatique
D - L'activité rénine
E - Les anticorps anti-surrénaliens
Bonne(s) Réponse(s) : A B E

Les principales étiologies de l'IS primitive sont la tuberculose surrénale et la rétraction


corticale d'origine auto-immune.

La patiente va quitter l'hôpital ; vous devez prescrire :


A - 30 mg/j d'hydrocortisone
B - 75 microgrammes g/j de hydrocortisone
C - 20 mg/j de prednisolone
D - L'adaptation des doses en cas de stress
E - Régime désodé
Bonne(s) Réponse(s) : A B D

Le régime doit être normosodé. En cas de régime hyposodé, il y a risque d'IS, aiguë.

Deux mois plus tard, la surveillance portera sur :


A - Le poids
B - Le dosage du cortisol à jeun
C - L'activité rénine
D - Le dosage du cortisol libre urinaire
E - L'ACTH
Bonne(s) Réponse(s) : A B C

La surveillance a pour but de vérifier l'effet strictement substitutif du traitement.

634
Exclusivement sur DOC - DZ : www.doc-dz.com NADJI 85
RESIDANAT EN POCHE TOME II
Cas Clinique en QCM
Mme P..., mariée depuis 4 ans, sans enfant, consulte pour un ensemble de troubles apparus depuis 6 mois : amaigrissement
de 8 kg, malgré un appétit conservé et nervosité excessive. On note sur le plan gynécologique que les premières règles sont
apparues à l'âge de 14 ans et qu'elles ont toujours été irrégulières avec alternance de cycles courts de 20 jours en moyenne
et de cycles longs d'une quarantaine de jours. Une courbe de température a été réalisée sur une période de plusieurs mois, il
y a un an, et a montré l'absence d'élévation thermique au-dessus de 37°C dans la seconde partie du cycle. L'examen clinique
retient comme signes principaux un rythme cardiaque régulier à 110 par minute, une protrusion des globes oculaires en
dehors de la cavité orbitaire à 24 mm à droite, à 22 mm à gauche, (mesure à l'exophtalmètre de Hertel) avec rétraction
bilatérale de la paupière supérieure. Il existe une moiteur anormale des mains, un tremblement fin des doigts, le foie et la rate
ne sont pas palpables, il n'y a pas de reflux hépato-jugulaire. La glande thyroïde est très modérément augmentée de volume,
régulière et homogène à la palpation avec souffle de siège médian à l'auscultation. L'examen somatique général permet de
retenir l'existence de taches achromiques siégeant au niveau de la peau et évocatrices d'un vitiligo. La pilosité est normale ; il
existe une galactorrhée, les touchers pelviens étant normaux. Les dosages de T3 et T4 libres sont franchement supérieurs à
la normale, la TSH est abaissée .

Parmi les symptômes notés chez cette patiente, quel(s) est (sont) celui (ceux) qui plaide(nt) en faveur d'une
thyrotoxicose ?
A - Le goître
B - L'amaigrissement
C - Le vitiligo
D - La tachycardie
E - La présence d'une galactorrhée
Bonne(s) Réponse(s) : B D

Sans commentaire.

Parmi les symptômes notés chez cette patiente, quel(s) est (sont) celui (ceux) qui plaide(nt) en faveur d'une
maladie de Basedow à l'origine de la thyréotoxicose ?
A - L'exophtalmie
B - Le vitiligo
C - Les troubles des règles
D - La TSH plasmatique basse
E - Le goître diffus
Bonne(s) Réponse(s) : A B E

Les trois signes spécifiques d'une maladie de Basedow sont exophtalmie, goître diffus vasculaire, et myxoedème prétibial
(rare). Le vitiligo est en faveur d'une pathologie auto-immune, comme le Basedow.

Quelle(s) prescription(s) thérapeutique(s) peut-on poser ?


A - Néomercazole®, 1/2 comprimé matin et soir
B - Néomercazole®, 2 comprimés matin, midi et soir
C - Repos au lit strict
D - Avlocardyl®, 1/2 comprimé matin, midi et soir
E - Régime hypercalorique
Bonne(s) Réponse(s) : B C D E

Sans commentaire.

Quel(s) examen(s) complémentaire(s) prescrire à la fin du premier mois de traitement ?


A - Dosage de cholestérolémie
B - Dosage de T4 libre
C - Numération formule globulaire
D - Dosage de T3 libre
E - Electrocardiogramme
Bonne(s) Réponse(s) : B C E

La surveillance du traitement doit comporter un dosage de T4 libre pour évaluer l'effet du traitement, une NFS car il existe un
risque d'agranulocytose aiguë sous .Néomercazole®, et un ECG (dysthyroïdie, prise de béta-bloquant).

Les irrégularités menstruelles et l'aspect de la courbe thermique notés chez cette patiente avant toute
hyperthyroïdie décelable :
A - Peuvent avoir été influencées par la thyrotoxicose
B - Doivent faire pratiquer une enquête sur les prises médicamenteuses antérieures
C - Evoquent une anorexie mentale
D - Sont compatibles avec une hyperprolactinémie
E - Sont liées au vitiligo
Bonne(s) Réponse(s) : B D

Sans commentaire.
635
Exclusivement sur DOC - DZ : www.doc-dz.com NADJI 85
RESIDANAT EN POCHE TOME II
Cas Clinique en QCM
Un homme de 23 ans, qui subit un examen médical, vous consulte pour nodule tyroïdien découvert fortuitement il y a
quelques jours. Il ne présente aucun signe de dysfonctionnement thyroïdien, et ne s'était jamais plaint de sa thyroïde
auparavant. Il a reçu une irradiation cervicale à l'âge de 6 mois pour une hypertrophie thymique, et ne présente aucun autre
antécédent. L'examen clinique est normal, en dehors d'un nodule dur mesurant 2x2 cm à la partie basse du lobe thyroïdien
droit. On retrouve quelques adénopathies cervicales antérieures droites. Le reste de la glande thyroïde est normal. Les
dosages hormonaux thyroïdiens sont normaux, les anticorps anti-thyroglobuline sont à un titre de 1/32, la scintigraphie
thyroïdienne montre un nodule hypofixant coïncidant avec la palpation, l'échographie, un nodule plein.

Votre attitude thérapeutique vis-à-vis du nodule de ce patient doit être influencée par :
A - L'âge de ce sujet
B - La présence d'adénopathies cervicales
C - L'antécédent d'irradiation cervicale
D - Le taux des anticorps anti-thyroglobuline
E - La taille de ce nodule
Bonne(s) Réponse(s) : B

Devant un nodule froid, surtout avec antécédents d'irradiation cervicale, présence d'adénopathies, l'attitude est l'exploration
chirurgicale avec extemporané.

La scintigraphie est un examen utile pour l'exploration des nodules thyroïdiens, parce que :
A - Un nodule hypofixant signifie à coup sur un cancer thyroïdien
B - Un nodule hyperfixant (fixant plus que le parenchyme voisin) exclut virtuellement la possibilité d'un nodule
cancéreux
C - La scintigraphie permet une approche exacte de l'histologie
D - La scintigraphie donne une information sur le caractère fonctionnel du nodule thyroïdien
E - Un nodule faiblement hypofixant est un argument en faveur d'une pathologie différente du cancer
Bonne(s) Réponse(s) : B D

Devant un nodule froid en scintigraphie, l'anatomo-pathologie retrouve 90 % d'adénomes bénins et 10 % de cancers.

La meilleure attitude face au nodule thyroïdien de ce patient est :


A - Biopsie thyroïdienne à l'aiguil!e
B - Doses suppressives d'hormones thyroïdiennes
C - Surveillance pendant 6 mois, car le nodule devrait régresser
D - Exploration chirurgicale de la thyroïde
E - Doses pharmacologiques d'iodure
Bonne(s) Réponse(s) : D

Une biopsie thyroïdienne à l'aiguille négative n'élimine pas le diagnostic.

Le pronostic du cancer papillaire de la thyroïde dépend de :


A - La taille de la tumeur thyroïdienne
B - L'existence de métastases à distance (poumons, os)
C - L'existence d'une composante folliculaire de cette tumeur
D - L'âge du sujet
E - La présence d'un envahissement ganglionnaire
Bonne(s) Réponse(s) : A D E

Lorsqu'il y a association papillo-vésiculaire, le pronostic est celui du cancer papillaire.


Le cancer papillaire dissémine par voie Iymphatique.

Quel(s) est(sont) le(s) élément(s) de surveillance habituel(s) du cancer papillaire du sujet jeune, en cas de
thyroïdectomie totale ?
A - Palper annuel du cou
B - Scintigraphie à l'iode 131
C - Radiographie pulmonaire
D - Dosage de thyroglobuline
E - Dosages d'anticorps anti tyhroglobuline
Bonne(s) Réponse(s) : A C D

Sans commentaire.

636
Exclusivement sur DOC - DZ : www.doc-dz.com NADJI 85
RESIDANAT EN POCHE TOME II
Cas Clinique en QCM

Une femme de 25 ans vient consulter pour une aménorrhée de 10 mois. Elle a eu 2 enfants, après des grossesses normales,
à 19 ans et 22 ans, et a suivi une contraception par les oestroprogestatifs combinés (Stédiril) pendant 2 ans, après la
deuxième grossesse. A l'arrêt de la contraception, les règles ne sont pas réapparues. L'examen clinique trouve, chez cette
jeune femme indemne de tout antécédent pathologique notable, les éléments suivants :
- col sain
- culs de sac libres
- utérus petit, de forme régulière, antéversé
- au niveau des seins, une galactorrhée bilatérale et multi-orificielle est mise en évidence par la pression des seins.
Le reste de l'examen est normal

Parmi les diagnostics évoqués ci-dessous, un seul peut correspondre à un taux plasmatique de prolactine de
100 ng/ml, compte tenu de l'observation clinique :
A - Grossesse en évolution
B - Lactation
C - Insuffisance rénale chronique
D - Contraception oestro-progestatif (par le Stédiril®)
E - Microadénome à prolactine
Bonne(s) Réponse(s) : E

Sans commentaire.

Parmi les médicaments énumérés ci-dessous, le(s)quel(s) peu(ven)t induire une aménorrhée
hyperprolactinémique ?
A - Chlorpromazine (Largactil®)
B - Cimétidine (Tagamet®)
C - Alpha-méthyldopa (Aldomet®)
D - Sulpiride (Dogmatil®)
E - Furosémide (Lasilix®)
Bonne(s) Réponse(s) : A B C D

Les antihypertenseurs (réserpine et méthyldopa).


Les antidépresseurs tricycliques agissent en diminuant le stock ou en inhibant la synthèse des catécholamines.
Les neuroleptiques agissent en bloquant les récepteurs dopaminergiques.

Dans le cadre de l'observation proposée, quels sont, parmi les examens énumérés ci-dessous, les trois qui
doivent être pratiqués par priorité ?

A
A - Radiographies et tomographies de face et de profil de la selle turcique
B - Etude du champ visuel
C - Examen au scanner de la région hypophyso-hypothalamique
D - Test à la LH-RH
E - Dosages plasmatiques radio-immunologiques de FSH et LH
Bonne(s) Réponse(s) : A B C

Le test à pratiquer devant une hyperprolactinémie est le test au TRH : en cas d'hypersécrétion tumorale, la prolactine s'élève
peu ou pas.

Les examens que vous avez demandés n'ayant pas montré d'anomalie, parmi les attitudes thérapeutiques
énumérées ci-dessous, que choisir dans le cadre particulier de cette observation ?
A - Abstention
B - Traitement par les inducteurs de l'ovulation
C - Traitement par la bromocriptine (Parlodel®)
D - Traitement neuro-chirurgical d'emblée
E - Traitement par les cycles artificiels à l'aide d'oestroprogestatifs
Bonne(s) Réponse(s) : C

Sans commentaire.

637
Exclusivement sur DOC - DZ : www.doc-dz.com NADJI 85
RESIDANAT EN POCHE TOME II
Cas Clinique en QCM

La survenue d'une grossesse, après effet de la thérapeutique (ou l'absence de thérapeutique), chez cette
femme hyperprolactinémique peut avoir une ou plusieurs des conséquences exposées ci-dessous :
A - N'a aucune conséquence
B - Peut être un facteur d'aggravation
C - Constitue la manifestation d'une guérison définitive
D - Impose l'interruption thérapeutique de la grossesse
E - Autorise la poursuite de la grossesse sous stricte surveillance
Bonne(s) Réponse(s) : B E

Pendant la grossesse, il y a risque de poussée congestive avec nécrose hémorragique de l'adénome, ainsi qu'une possibilité
de croissance progressive de l'adénome (rôle des oestrogènes).

Madame D... Adèle, 63 ans. vient consulter pour une surdité d'apparition récente. Elle se plaint aussi de crampes musculaires,
de douleurs préthoraciques et d'acroparesthésies. Elle est même réveillée parfois par des crampes et une douleur thoracique
qui disparaît spontanément en 30 mn, douleur décrite comme une striction, Aucune irradiation n'est notée.Ces manifestations
fonctionnelles ne surviennent pas à l'effort. Elle se plaint enfin de sécheresse cutanée et de bouffissure des paupières. Aucun
antécédent remarquable n'est noté. A l'examen : il s'agit d'une patiente s'exprimant lentement. Sa peau est jaunâtre et sèche.
La TA est de 160/95 mmHg. Le pouls est de 47/mn. La température corporelle est de 36°. La thyroïde n'est pas palpable. Le
choc de la pointe du coeur n'est pas perçu. Les bruits du coeur sont lointains. La sensibilité au toucher et à la douleur est
diminuée de façon bilatérale au pouce et à l'index. La flexion forcée du poignet et sa percussion sur sa face antérieure réveille
la douleur dans les 2 index. A l'electrocardiogramme : rythme sinusal, bas voltage, anomalies non spécifiques du segment ST
et de l'onde T.
Examens complémentaires :
- Hb : 14,3 g
- Hte : 37 %
- VGM : 101 micro3
- Na : 130 meq/l, K : 3,9 meq/l
- T4 : 2,3 microg/dl, T3 : 40 ng/dl
- GR : 3 500 000
- GB : 12500, PN : 73
- SGOT : 30 U, SGPT : 24 U.
Radiographie pulmonaire : élargissement de la silhouette cardiaque.

Le(s) diagnostic(s) suivant(s) est(sont) le(s) plus probable(s) :


A - Insuffisance thyroïdienne périphérique
B - Panhypopituitarisme
C - Infarctus du myocarde
D - Epanchement péricardique
E - Anémie macrocytaire
Bonne(s) Réponse(s) : A D

L'ECG et le taux des transaminases permettent d'éliminer un IDM. Lors d'une hypothyroïdie, le myocarde est infiltré et il existe
souvent un épanchement péricardique riche en albumine et en cholestérol.

Parmi les examens suivants, quel est le plus utile ?


A - Test au TRF
B - Test de Querido (stimulation par TSH)
C - Test à la Métopirone
D - Dosage de TSH
E - Dosage de la T4 libre
Bonne(s) Réponse(s) : D

Cet examen permet de diagnostiquer l'origine haute ou basse de l'hypothyroïdie.

Choisissez le traitement le mieux adapté :


A - 200/300 microg de LT4 (à commencer immédiatement)
B - 75 microg de LT3 (à commencer immédiatement)
C - Abstention thérapeutique
D - 20 mg d'hydrocortisone et LT4 (en commençant par 25 microg et en augmentant progressivement jusqu'à
150 microg)
E - 20 mg d'hydrocortisone et 200 microg de LT4
Bonne(s) Réponse(s) : D

Le traitement de l'hypothyroïdie doit être très progressif, avec surveillance journalière de l'ECG, d'autant plus que le sujet est
âgé et qu'il existe une symptomatologie cardiaque (la prise de béta-bloquants est conseillée en cas d'angor).
Une insuffisance surrénalienne fonctionnelle peut se démasquer brutalement en début de traitement, d'autant plus que
l'hypothyroïdie est ancienne.

638
Exclusivement sur DOC - DZ : www.doc-dz.com NADJI 85
RESIDANAT EN POCHE TOME II
Cas Clinique en QCM

Le traitement correct initié, que peut-il se produire ?


A - Diminution de la silhouette cardiaque à la radiographie pulmonaire
B - Amélioration du transit digestif
C - Apparition d'une anti-diurèse
D - Accentuation des crampes musculaires
E - Amélioration de la surdité
Bonne(s) Réponse(s) : A B E

Sans commentaire.

Le traitement peut se compliquer de :


A - Syndrome oedémateux
B - Tamponnade
C - Arythmie complète
D - Bloc auriculo ventriculaire
E - Infarctus du myocarde
Bonne(s) Réponse(s) : C E

L'arythmie complète peut survenir en cas de surdosage.

Madame M, 35 ans, consulte parce qu'elle a pris du poids et parce qu'elle présente depuis quelque temps des céphalées. Elle
n'a pas d'antécédent pathologique particulier. Elle assure une contraception par pilule oestroprogestative normo-dosée. A
l'examen clinique, on note : un poids à 72 kg pour 1,57 m avec des masses musculaires, la surcharge intéressant également
la ceinture scapulaire pelvienne, les cuisses et les mollets ; une érythrose du visage ; quelques fines vergetures blanchâtres,
verticales, aux flancs et à la racine des cuisses ; une tension artérielle à 180- 110 mm de Hg. Les résultats d'un bilan
biologique sont les suivants : cholestérol à 8 mmol/l, triglycérides à 1,7 mmol/l, glycémie à jeun (sur plasma veineux) à 8,1
mmol/l, sodium plasmatique à 142 mmol/l et potassium plasmatique à 4,1 mmol/l, cortisolémie (à 8 h) à 350 microgrammes/l.

Parmi les propositions suivantes, laquelle (lesquelles) vous paraît (paraissent) plausible(s) :
A - La pilule contraceptive peut jouer un rôle dans la prise de poids et l'hypertension
B - L'obésité peut jouer un rôle dans l'hypertension
C - L'augmentation de la cortisolémie peut jouer un rôle dans l'hypertension
D - La pilule peut être responsable directement, indépendamment de l'obésité, de l'augmentation de la
cortisolémie
E - L'hypertension peut être responsable de l'augmentation de la cortisolémie
Bonne(s) Réponse(s) : A B C D

La pilule peut, par stimulation de la synthèse de CBG, augmenter la cortisolémie.

Quel(s) est(sont), parmi les éléments suivants de l'observation celui (ceux) qui peut (peuvent) être significatif(s)
d'un hypercortisolisme ?
A - L'hypertension artérielle
B - Les vergetures
C - Le développement musculaire
D - Le niveau de la glycémie a jeun
E - Le niveau de la natrémie et de la caliémie
Bonne(s) Réponse(s) : A D

Les vergetures de l'hypercortisolisme sont classiquement larges, pourpres, horizontales, occupant la racine des membres,
l'abdomen.
Par ailleurs, il existe une amyotrophie des racines des membres.

Parmi les résultats suivants, lequel rechercheriez-vous comme argument le plus significatif en faveur de
l'étiologie secondaire à l'obésité de l'hypercortisolémie ?
A - Cycle nycthéméral du cortisol aboli
B - Excrétion urinaire des 17 cétostéroïdes normale
C - Test au synacthène positif (augmentation de l'excrétion urinaire des 17 hydroxystéroïdes)
D - Cortisolémie de 8 heures inférieure à 25 microgrammes/l après prise de 2 comprimés de Dexaméthasone
(Décadron®) à 22 heures
E - Cortisol salivaire relativement plus élevé que le cortisol plasmatique
Bonne(s) Réponse(s) : D

L'hypercortisolisme secondaire à l'obésité répond aux freinages rapides et faibles. Le cycle nycthéméral est le plus souvent
non aboli.

639
Exclusivement sur DOC - DZ : www.doc-dz.com NADJI 85
RESIDANAT EN POCHE TOME II
Cas Clinique en QCM

Quels sont, parmi les données suivantes, celle(s) qui s'inscrirai(en)t en faveur d'une maladie de Cushing par
adénome hypophysaire ?
A - Hémianopsie latérale homonyme
B - Image en double fond sur la radiographie de profil de la selle turcique
C - Test à la métopirone négatif (pas d'augmentation de l'ACTHémie et des 17 hydroxystéroïdes urinaires)
D - Fixation unilatérale à l'examen au iodocholestérol
E - ACTH (et/ou bêta LPH) modérément élevées
Bonne(s) Réponse(s) : A B E

La métopirone, par inhibition de la 11 béta hydroxylase, bloque la synthèse de cortisol. La chute de la cortisolémie entraîne
une stimulation de la sécrétion d'ACTH et une élévation des hydroxystéroïdes urinaires et des désoxycorticostéroïdes
sanguins.

Indiquez, parmi les propositions d'ordre thérapeutique suivantes, celle (celles) qui est (sont) fondée(s) :
A - En cas d'adénome surrénalien unilatéral, la chirurgie d'exérèse est la meilleure méthode thérapeutique
B - Dans ce cas, il est inutile de prévoir une suppléance par hydrocortisone après l'intervention
C - En cas de maladie de Cushing le freinage par la dexaméthasone est un moyen thérapeutique
D - La surrénalectomie bilatérale expose en cas de maladie de Cushing au développement ultérieur d'un
adénome hypophysaire
E - Toutes ces propositions sont fondées
Bonne(s) Réponse(s) : A D

Après surrénalectomie locale unilatérale, un traitement supplétif est nécessaire en raison d'une insuffisance voir ed'une
atrophie de la surrénale contralatérale.
L'apparition d'un adénome hypophysaire corticotrope après surrénalectomie bilatérale pour maladie de Cushing réalise le
syndrome de Nelson (ACTH élevée, non freinable, hyperstimulable par le LVP).

Le bilan biologique systématique d'un homme de 55 ans, pesant 91 kg pour 170 cm et dont l'examen clinique est normal, met
en évidence une glycémie à jeun dans le plasma veineux de 7,80 mmol/l (chiffre confirmé par un second examen dans les
mêmes conditions). L'épreuve d'hyperglycémie provoquée par voie orale (HGPO), faite avec 75 g de glucose dilués dans 350
ml d'eau, donne les résultats suivants :
- au temps O = 7,90 mmol à
- à 30 minutes = 13,70 mmol/l
- à 60 minutes = 12,50 mmol/l
- à 90 minutes = 12,30 mmol/l
- à 120 minutes = 11,50 mmol/l.
Entre chaque prélèvement sanguin réalisé avec une nouvelle aiguille, le sujet est allé à son domicile, situé à 300 mètres du
laboratoire. Le Peptide C à jeun est de 1,50 nmol/l et de 2,60 nmol/laprès stimulation par le glucagon (valeurs normales : 0,6
nmoI/I à jeun et 1,50 nmol/l après stimulation).
La cholestérolémie est de 7 mmol/l, la triglycéridémie de 2,50 mmol/l

L'argument (les arguments) suivant(s) permet(tent) d'affirmer le diagnostic de diabète chez ce patient :
A - La glycémie à jeun est à deux reprises à 7,80 mmol/l
B - Il est obèse
C - Il est obèse donc en hypo insulinisme
D - Il est obèse donc il a probablement une pancréatite chronique
E - La glycémie à 30 mn lors de l'HGPO est supérieure à 11 mmol/l
Bonne(s) réponse(s) : A

Sans commentaire.

Il s'agit d'un diabète :


A - Insulino-dépendant
B - Non insulino-dépendant
C - Dont le diagnostic ne peut être affirmé qu'après HGPO
D - Avec insulinorésistance
E - Dont le diagnostic ne peut être affirmé qu'après dosage du peptide C
Bonne(s) réponse(s) : B D

Sans commentaire.

640
Exclusivement sur DOC - DZ : www.doc-dz.com NADJI 85
RESIDANAT EN POCHE TOME II
Cas Clinique en QCM

A partir de quel chiffre de glycémie du plasma veineux dans l'épreuve d'HGPO à 120 minutes peut-on affirmer le
diagnostic de diabète ?
A - 7 mmol/l
B - 8 mmol/l
C - 9 mmol/l
D - 11 mmol/l
E - 13 mmol/l
Bonne(s) réponse(s) : D

Sans commentaire.

La(les) mesure(s) thérapeutique(s) impérative(s) pour le premier mois chez ce malade est (sont) :
A - Sulfamide hypoglycémiant
B - Hypolipidémiant
C - Insuline
D - Régime hypocalorique
E - "Coupe faim"
Bonne(s) réponse(s) : D

Sans commentaire.

Une femme de trente ans, sans antécédent notable, est adressée pour la survenue récente d'une exophtalmie bilatérale.
A l'examen :
- l'acuité visuelle est de 10/10ème à chaque oeil
- l'examen du segment antérieur est normal
- l'examen du fond d'oeil est normal.
- il existe une exophtalmie bilatérale, symétrique, modérée
On retrouve par ailleurs une tachycardie, une moiteur et un tremblement des extrémités, et une notion d'amaigrissement récent
Au total, il s'agit vraisemblablement d'une exophtalmie basedowienne

Quelle(s) donnée(s) de l'examen peut(ven)t évoquer l'origine basedowienne de l'exophtalmie ?


A - Vasodilatation conjonctivale
B - Bilatéralité
C - Rétraction de la paupière supérieure
D - Anesthésie cornéenne
E - Asynergie oculopalpébrale
Bonne(s) réponse(s) : B C E

Jusqu'à 50% des exophtalmies bilatérales seraient d'origine basedowienne. Elle est fréquemment asynergie, parfois
unilatérale en particulier au début. L'asymétrie oculopalpébrale se recherche dans le regard vers le bas.
La rétraction de la paupière supérieure peut s'intégrer dans l'effet sympatomimétique lié à la thyrotoxicose.

Un examen tomodensitométrique du crâne est pratiqué. Quelle(s) image(s) peut-on s'attendre à retrouver ?
A - Exophtalmie
B - Hypertrophie des muscles oculomoteurs
C - Epaississement de la paroi du globe oculaire
D - Augmentation de densité de la paroi orbitaire
E - Masse intraconique
Bonne(s) réponse(s) : A B

L'exophtalmie est classée en trois grades selon la mesure de l'index oculo-orbitaire.


Les muscles oculomoteurs sont épaissis du fait de la réaction inflammatoire et de l'oedème à la phase initiale, dont ils sont le
siège.

Quelle autre cause d'exophtalmie bilatérale pourrait être discutée ?


A - Craniosténose
B - Tumeur hypophysaire
C - Pseudotumeur inflammatoire de l'orbite
D - Syndrome de Claude-Bernard-Horner
E - Pseudo-exophtalmie par myopie forte
Bonne(s) réponse(s) :

QUESTION ANNULEE.

641
Exclusivement sur DOC - DZ : www.doc-dz.com NADJI 85
RESIDANAT EN POCHE TOME II
Cas Clinique en QCM

La prise en charge thérapeutique de cette exophtalmie basedowienne nécessite


A - Une indication chirurgicale (thyroïdectomie) urgente
B - Une corticothérapie intense (environ 1 mg/kg/j)
C - Un antithyroïdien de synthèse
D - Un bêta-bloquant
E - La mise en oeuvre immédiate d'un protocole avec l'iode radioactif
Bonne(s) réponse(s) : C D

Il s'agit d'une exophtalmie modérée sans complication visuelle qui ne nécessite qu'un traitement local par collyre pour éviter le
risque cornéen et une surveillance ophtalmologique régulière. La thyrotoxicose doit être traitée, le traitement médical chez
cette femme est adapté.
Les antithyroïdiens de synthèse ne semblent pas modifier le cours de l'ophtalmopathie.

Une jeune fille de 18 ans se plaint depuis 3 semaines de soif, elle a maigri avec conservation de l'appétit, elle souffre depuis 2
jours de nausées et de douleurs abdominales. Elle est hospitalisée en urgence pour recrudescence des douleurs abdominales
qui prédominent au niveau de la fosse iliaque droite, vomissements et fièvre à 37,8°. L'examen trouve une polypnée, une TA à
100/60 mm de Hg, une déshydratation globale et une grande asthénie. On trouve une douleur nette au point de Mac Burney.
Le reste est sans particularité. On fait en urgence une NFS qui montre : GR à 5,6 millions/mm3, GB à 12 000/mm3 dont 85 %
de polynucléaires neutrophiles. On réalise également un bilan sanguin qui montre : hématocrite à 50. protidémie à 80 g/l, Na+
à 25 mmoles/l. K+ à 5,5 mmoles/l, CI- à 88 mmoles/l. La créatininémie est à 18 mg/l et l'urée sanguine à 0,75 g/l. La glycémie
est à 3 g/l le sérum est lactescent.
Le diagnostic d'appendicite aiguë est néanmoins porté et la malade opérée en urgence. Dès l'induction de l'anesthésie, elle
fait un collapsus cardiovasculaire et un arrêt respiratoire qui nécessitent l'interruption de l'acte chirurgical et la réanimation
médicale d'urgence d'une décompensation acidocétosique d'un diabète sucré méconnu.

Parmi les examens biologiques suivants, non faits à l'entrée, indiquez celui ou ceux qu'il aurait fallu faire
impérativement avant l'intervention :
A - Hémoculture
B - Dosage du CO2 T plasmatique
C - Phosphorémie
D - Uricémie
E - Cétonurie
Bonne(s) réponse(s) : A B E

A. Visée diagnostic, acidose dont témoigne la baisse des bicarbonates, la cétonurie. Il faut aussi rechercher une cause
précipitante, en particulier infection d'autant que l'acidocétose, en soit, peut entraîner une hyperleucocytose.
L'hypothermie étant fréquente dans l'acidocétose, toute fièvre incite à la recherche d'une infection
BE. Sont les examens dont il aurait fallu impérativement les résultats avant l'intervention.

Les manifestations abdominales de cette jeune fille pouvaient correspondre à un syndrome pseudo-
appendiculaire en relation avec la décompensation du diabète sucré. Parmi les signes suivants, quel est celui
ou quels sont ceux qui ne pouvai(en)t pas être en rapport avec l'appendicite aiguë ?
A - Soif antérieure à l'épisode aigu
B - Douleurs abdominales
C - Vomissements
D - Hyperthermie modérée
E - Dyspnée
Bonne(s) réponse(s) : A E

A. Par déshydratation.
D. Provoquée par l'acidose lors de cette décompensation diabétique.

La natrémie de cette jeune fille pouvait être en relation avec une ou plusieurs perturbations métaboliques
connues ou à rechercher. Laquelle ou lesquelles ?
A - Hyperprotidémie
B - Hémoconcentration
C - Hypophosphorémie
D - Hypertriglycéridémie
E - Fuite urinaire du Na+
Bonne(s) réponse(s) : D E

Il existe au cours de l'acidocétose une fuite urinaire de sodium que l'on peut apprécier à 10 meq/kg liée en particulier à
l'élimination des corps cétoniques sous forme de sel de sodium, à la diurèse amniotique. La carence en insuline entraîne un
dysfonctionnement de la lipoprotéine lipase avec défaut du catabolisme des VLDL et des chylomicrons avec possibilité de
type I ou V, sérum lactescent, ceci rajoute une composante de fausse hyponatrémie à cette hyponatrémie avec osmolarité
élevée et donc déshydratation intracellulaire.

642
Exclusivement sur DOC - DZ : www.doc-dz.com NADJI 85
RESIDANAT EN POCHE TOME II
Cas Clinique en QCM

La réanimation médicale de cette jeune fille peut comporter plusieurs attitudes dont certaines sont à proscrire.
Laquelle ou lesquelles ?
A - Perfusion d'un soluté macromoléculaire
B - Utilisation de vas-presseurs
C - Perfusion d'emblée de chlorure de potassium
D - Infusion continue d'insuline par pompe à débit constant à raison de 10 unités/heures
E - Perfusion de sérum salé isotonique
Bonne(s) réponse(s) : B C

Il existe une hypovolémie, une déshydratation globale qui nécessite une expansion volémique. L'apport de potassium d'emblé
est contre-indiqué compte tenu de l'hyperkaliémie, il faudra dans la suite compenser le déficit du stock potassique qui existe
toujours dans l'acidocétose.

Une institutrice de 27 ans, mariée, consulte pour une galactorrhée. Elle allait bien jusqu'il y a un an, où elle a commencé à
noter un écoulement blanchâtre des deux seins et des irrégularités menstruelles. Les 4 derniers mois, elle a présenté une
aménorrhée avec diminution de la libido. Elle se plaint de sueurs excessives avec peau grasse et augmentation de la pilosité
au niveau des bras et du visage. Récemment, elle a ressenti des paresthésies des deux mains. Depuis 4 ou 5 ans.sont
apparues des céphalées rétro-orbitaires bilatérales augmentant récemment d'intensité.
Elle ne prend aucun médicament.
A l'examen, la peau est épaissie, mais l'hyperpilosité minime. Sa pression artérielle est à 14/9 cm de Hg. Le champ visuel au
doigt est normal. Une galactorrhée bilatérale est retrouvée. Le reste de l'examen est normal. Biologiquement :
- glycémie à jeun à 6 mmol/l
- hyperglycémie provoquée à 1 h : 10 mmol/l, à 2 h = 8,5 mmol/l, à 3 h = 5 mmol/l
- calcémie à 2.4 mmol/l, phosphorémie à 1,6 mmol/l
- cortisol à 8 h = 560 mmol/l, à 16 h : 300 mmol/l
- LH à 10 UI/l (N : 3-20), FSH à 5 UI/l (N : 2-15).

Cette patiente a probablement :


A - Un adénome à prolactine
B - Un adénome chromophobe
C - Une acromégalie
D - Un hypopituitarisme
E - Une maladie de Cushing
Bonne(s) réponse(s) : C

Dans un tiers des cas d'acromégalie, il existe une hypersécrétion de prolactine : adénome mixte ou compression de la tige
pituitaire entraînant une libération du frein hypothalamique.

Quelle(s) est (sont) la(les) proposition(s) exacte(s) concernant cette patiente ?


A - Elle aura une réponse normale de son hormone de croissance sous hyperglycémie provoquée
B - Son taux de prolactine sera élevé
C - Elle aura une réponse normale de GH sous L. DOPA (stimulation)
D - Les autres fonctions hypophysaires devront être explorées
E - L'aménorrhée ne trouve pas d'explication
Bonne(s) réponse(s) : B D

La GH n'est pas freinée lors de l'HGPO chez l'acromégale. On note une diminution paradoxale de la GH sous L-dOPA chez
l'acromégale.

Quel médicament freine l'hypersécrétion de GH ?


A - Chlorpromazine
B - Bromoergocryptine
C - Dexaméthasone
D - Médroxyprogestérone
E - Aucun des médicaments ci-dessus
Bonne(s) réponse(s) : B

La 2 bromo alpha-ergocriptine ou bromocriptine freine l'hypersécrétion de GH chez nombre d'acromégales en test aigu. Elle
peut constituer un traitement chez les sujets répondeurs, très rarement en première intention. Elle serait plus active dans les
cas d'adénome mixte GH prolactine.

643
Exclusivement sur DOC - DZ : www.doc-dz.com NADJI 85
RESIDANAT EN POCHE TOME II
Cas Clinique en QCM

Quelle(s) anomalie(s) est(sont) observée(s) en cas d'acromégalie ?


A - Augmentation de la réabsorption tubulaire des phosphates
B - Hypercalciurie
C - Anomalies du système rénine-angiotensine-aldostérone
D - Syndrome du canal carpien
E - Aucune des propositions ci-dessus
Bonne(s) réponse(s) : A B D

Sans commentaire.

Quel traitement proposerez-vous à cette patiente en première intention ?


A - Chirurgie de l'adénome
B - Traitement radiothérapique
C - Perfusion de somatostatine
D - Prise de Parlodel®
E - Aucun de ces traitements
Bonne(s) Réponse(s) : A

La chirurgie de première intention, par voie transphénoïdale est le plus souvent le traitement de l'acromégalie.
Les analogues retard de la somatostatine constitue un traitement prometteur. Ils s'administrent par vois sous-cutanée. Ils sont
parfois emplyoyés en courtes cures préchirurgicales.

Un homme de 68 ans, diabétique non insulino-dépendant, a perdu connaissance pendant qu'il jardinait. On apprend par son
épouse qu'il n'est pas tombé brusquement, mais a eu le temps de s'étendre par terre ; et qu'il est traité par Daonil® (3
comprimés/24 heures) et Glucophage-Retard® (2 comprimés/24 h).
Il est rapidement hospitalisé. On observe à l'admission un coma avec agitation spontanée, des réflexes ostéotendineux
exagérés et symétriques, un Babinski bilatéral. et des sueurs profuses. Il n'existe pas de déficit focalisé. Le reste de l'examen
clinique est normal.

Quel est le diagnostic le plus probable ?


A - Syncope cardiaque
B - Coma acidocétosique
C - Coma hypoglycémique
D - Acidose lactique
E - Accident vasculaire cérébral
Bonne(s) réponse(s) : C

Sans commentaire.

Ce diagnostic peut être rapidement confirmé par :


A - EEG
B - ECG
C - Mesure de la glycémie
D - Recherche d'une glycosurie
E - Gazométrie artérielle
Bonne(s) réponse(s) : C

Il est de bon usage de mesurer la glycémie capillaire, de prélever un tube pour dosage de glycémie au laboratoire puis de
"resucrer".

La survenue de cet épisode a pu être favorisée par :


A - Un accès hypertensif
B - Une activité physique excessive
C - Une infection
D - Une insuffisance coronarienne
E - L'ingestion d'une boisson alcoolisée
Bonne(s) réponse(s) : B C E

L'hypoglycémie peut favoriser A et D.

644
Exclusivement sur DOC - DZ : www.doc-dz.com NADJI 85
RESIDANAT EN POCHE TOME II
Cas Clinique en QCM

Pour explorer le ou les facteur(s) susceptible(s) d'avoir contribué(s) à la survenue de cet épisode, vous
demandez :
A - Un dosage de la créatininémie
B - Un examen du fond d'oeil
C - Une recherche de protéinurie
D - Les médicaments pris (autres que les antidiabétiques oraux)
E - Une enquête sur le comportement alimentaire du patient
Bonne(s) réponse(s) : A D E

La recherche d'une insuffisance rénale hépatique, d'interactions médicamenteuses ou d'erreurs de régime est fondamentale.

Sans attendre le résultat des examens paracliniques, vous prescrivez immédiatement :


A - Une injection IV de 10 U d'Actrapid®
B - Une injection IV de Cédilanide®
C - Une injection IV de 30 ml de sérum glucosé à 30 %
D - Une perfusion IV de sérum bicarbonaté à 1,4 %
E - Une injection IM de 5 mg de Syncortil®
Bonne(s) réponse(s) : C

Il faut ensuite prendre le relai, en hospitalisant, par une perfusion de G 10% et dès que possible par un apport oral de
glucides. Il faut arrêter les hypoglycémiants oraux et apporter 400 g de glucose per os + 400 g de glucose IV par 24 heures.

Madame B. âgée de 64 ans consulte pour asthénie progressive, prise de 5 kg au cours des deux dernières années et frilosité
excessive, récente. L'examen clinique montre une pâleur et une infiltration diffuse des téguments qui sont secs. Le poids est
de 65 kg pour 1m60. La TA est à 14/8. Le corps thyroïde est discrètement augmenté de volume. Le diagnostic de myxoedème
est confirmé par les dosages hormonaux : TSH 48.4 µU/mI (normale 1-4 µU/ml), T3 libre 1.4 pg/ml I(normale 2.6-5.2 pg/ml),
T4 libre 4.2 pg/ml (normal 5.7-15.2 pg/ml). Les anticorps antimicrosomiaux sont positifs au 1/12500e alors que les anticorps
antithyroglobulines sont négatifs. La cholestérolémie est à 10.5 mmol/l(normale 3.8-6.2). La scintigraphie thyroïdienne au
technetium montre un corps thyroïde atrophique hypofixant de façon homogène.

L'étiologie la plus vraisemblable de l'hypothyroïdie chez cette patiente est :


A - Thyroïdite subaiguë de De Quervain
B - Thyroïdite chronique de Ridel
C - Thyroïdite chronique de Hashimoto
D - Trouble congénital de l'hormonosynthèse thyroïdienne à révélation tardive
E - Adénome hypophysaire thyréotrope
Bonne(s) réponse(s) : C

On est en présence d'une hypothyroïdie primaire avec des anticorps antimicrosomes positifs, thyroïdite autoimmune à la
phase involutive. Les anticorps antithyroglobuline sont négatifs dans 10 à 20% des thyroïdites de Hashimoto prouvé
histologiquement.

Avant la mise en route du traitement substitutif un seul des examens suivants est utile. Lequel ?
A - Un scanner hypophysaire
B - Un ECG
C - Une échocardiographie
D - Une radiographie thoracique
E - Un test de Querido (injection de TSH suivie d'une nouvelle scintigraphie
Bonne(s) réponse(s) : B

L'ECG est aussi un examen à pratiquer régulièrement dans la surveillance de l'opothérapie substitutive.

Afin de diminuer le risque d'incidents. il faut :


A - Commencer le traitement hormonal substitutif à petites doses et l'augmenter de façon très progressive
B - Utiliser des extraits thyroïdiens
C - Utiliser de la T4 au lieu de la T3
D - Associer la cholestyramine (Questran®)
E - Prescrire de la prednisone (Cortancyl®) 30 mg/j pendant 1 mois
Bonne(s) réponse(s) : A

Dans les formes importantes d'hypothyroïdie chez le sujet âgé, il ne faut pas hésiter à commencer par une posologie de 12,5
mg de T4 (Lévothyrox® 50 : 1/4 cp) et à faire des paliers longs (4 à 8 semaines).

645
Exclusivement sur DOC - DZ : www.doc-dz.com NADJI 85
RESIDANAT EN POCHE TOME II
Cas Clinique en QCM

Parmi les traitements suivants, le(s)quel(s) apporte(nt) de la T3 ?


A - L-Thyroxine®
B - Lévothyrox®
C - Cynomel®
D - Euthyral®
E - Extraits thyroïdiens
Bonne(s) réponse(s) : C D E

Les extraits thyroïdiens ne doivent plus être employés.


La T3 est réservée à des indications particulières tenant compte de sa 1/2 vie courte de 24 heures.

L'adaptation de la posologie peut être effectuée sur le résultat de :


A - Thyroglobuline
B - Anticorps antimicrosomiaux
C - PBI
D - TSH, un mois après l'installation d'une nouvelle posologie
E - TSH, T3, T4 dès le 3ème jour après l'installation d'une nouvelle posologie
Bonne(s) réponse(s) : D

Sans commentaire.

Vous revoyez la patiente après un an de traitement. Elle va bien et le bilan thyroïdien est normalisé. Vous lui
conseillez :
A - L'arrêt du traitement substitutif thyroïdien
B - Une corticothérapie pendant 6 mois
C - L'exérèse chirurgical de la thyroïde atrophique
D - La poursuite du traitement substitutif thyroïdien avec contrôle annuel des taux de TSH, T3 et T4
E - Aucune de ces propositions citées ci-dessus
Bonne(s) réponse(s) : D

Le traitement substitutif doit être poursuivi à vie.

Un homme de 43 ans consulte pour un goître diffus, ferme, parfaitement limité mobile à la déglutition. Il n'a pas de signes
cliniques de compression, pas d'adénopathies perceptibles. Le sujet se plaint de quelques crampes des membres inférieurs,
mais il n'a pas d'autres signes d'hypométabolisme, pas d'infiltration myxoedémateuse. Les concentrations plasmatiques de T4
et de T3 sont normales, mais la TSH est modérément élevée.
On évoque le diagnostic de thyroïdite d'Hashimoto.
Des explorations complémentaires sont demandées :
- vitesse de sédimentation : 20 mm à la première heure, 40 à la seconde
- electrophorèse : élévation des alpha 2 et des gammaglobulines
- hémogramme : légère anémie
- anticorps antithyroglobuline et antimicrosomiaux : taux extrêmement élevés
- fixation du gallium marqué au niveau de la thyroïde : positive.

Parmi les renseignements notés ci-dessus, le(s)quel(s) constitue(nt) un(des) argument(s) en faveur du
diagnostic évoqué ?
A - L'âge du sujet
B - Le sexe
C - L'absence de signes de compression
D - La consistance du goitre
E - Les valeurs de T3, T4 et TSH
Bonne(s) réponse(s) : A C D E

La thyroïdite lymphocytaire peut survenir à tout âge, elle est plus fréquente dans les 4ème et 5ème décades. Elle affecte
surtout le sexe féminin (9/10). La thyroïdite peut être découverte au stade d'hyperthyroïdie de réserve (T4 T3 normales, TSH
augmentée).

Parmi les résultats biologiques, quel est celui ou quels sont ceux qui est ou sont le( plus caractéristique(s) de
l'affection et la différence(nt) des autres formes de thyroïdite ?
A - La valeur de la vitesse de sédimentation
B - Les caractères de l'électrophorèse
C - Les caractères de l'hémogramme
D - Les taux des anticorps
E - Les caractères de la fixation du gallium marqué
Bonne(s) réponse(s) : D

Il existe un syndrome inflammatoire dans la thyroïdite de De Quervain.


Les anticorps antimicrosomiaux sont les plus spécifiques du Hashimoto.
646
Exclusivement sur DOC - DZ : www.doc-dz.com NADJI 85
RESIDANAT EN POCHE TOME II
Cas Clinique en QCM

Quel(s) procédé(s) permettrai(en)t d'écarter chez ce sujet l'éventualité d'une pathologie thyroïdienne induite par
l'iode ?
A - Enquête anamnestique
B - Echographie thyroïdienne
C - Test de captation de l'iode radioactif
D - Dosages de l'iode sanguin et/ou urinaire
E - Dosage de la thyroglobuline circulante
Bonne(s) réponse(s) : D

Il existe un nombre considérable de médicaments contenant de l'iode. L'enquête anamnestique oriente, mais ne permet pas
d'écarter une pathologie induite par l'iode.
La captation iodée est augmentée, normale ou abaissée dans le Hashimoto. Dans l'hyperthyroïdie induite par l'iode, la
captation reste paradoxalement satisfaisante, du fait d'un défaut d'organification (fixation de l'iodure sur la thyroglobuline de
l'iodure présent en excès dans la thyroïde) et d'une augmentation de la TSH. En effet le taux d'organification est un des
régulateurs de la captation de l'iodure.

Le patient vous interroge sur le pronostic de sa maladie. Si le diagnostic de thyroïdite d'Hashimoto est confirmé,
quelle(s) est(sont) la(les) possibilité(s) évolutive(s) ?
A - Apparition d'une endocrinopathie touchant une autre glande
B - Evolution inéluctable vers une hypothyroïdie caractéristique
C - Cancérisation
D - Envahissement des tissus périphériques
E - Apparition de signes de compression
Bonne(s) réponse(s) : A B

On peut voir apparaître d'autres endocrinopathies auto-immunes et d'autres malades auto-immunes non endocriniennes .
L'hypothyroïdie est en soi semblable aux autres formes d'hypothyroïdie, mais son association au goitre ferme la rend
caractéristique.
On ne voit pas d'évolution vers la cancérisation, le Hashimoto prédispose au lymphome malin thyroïdien.

Parmi les attitudes thérapeutiques suivantes, laquelle(lesquelles) paraî(ssen)t non justifiée(s) dans l'état présent
?
A - Lévothyrox® 100 à 150 mcg/jour
B - Euthyral® 1 comprimé/jour
C - Solupred® 30 mg/jour
D - L-Thyroxine® XX gouttes/jour
E - Basdène® 6 comprimés/jour
Bonne(s) réponse(s) :

QUESTION ANNULEE.

Vous êtes consulté pour le retard de développement d'un adolescent de 15 ans. Il mesure 150 cm (-3 DS) ; le volume
testiculaire est de 6 ml (30 mm), il n'existe pas de pilosité pubienne ou axillaire. L'état général est bon ; ce garçon est un
excellent joueur de football et il souhaite intégrer une section sport-étude. L'étude de la courbe de sa taille montre un
infléchissement très progressif de celle ci : la taille est passée de la moyenne (à l'âge de 5 ans) à - 3 DS à 15 ans; la prise de
taille à été de 3 cm la dernière année : l'âge osseux est de 12 ans. Le père mesure 172 cm, la mère 165, tous deux ont eu un
développement pubertaire normal; la mère à eu ses premières règles à 15 ans.

Quel est parmi les suivants le diagnostic probable ?


A - Syndrome de croissance et puberté retardés idiopathique
B - Syndrome de Klinefelter
C - Hypogonadisme central
D - Hypogonadisme périphérique
E - Insuffisance surrénalienne
Bonne(s) réponse(s) : A

A - Le bon état général, le volume testiculaire, l'infléchissement progressif de la courbe de taille, l'âge osseux et l'âge des
premières règles de la mère sont en faveur de ce diagnostic.
B - Absence de développement des testicules et grande taille.
C, D - Pas de signe d'hypogonadisme.

647
Exclusivement sur DOC - DZ : www.doc-dz.com NADJI 85
RESIDANAT EN POCHE TOME II
Cas Clinique en QCM

Quelle est parmi les suivantes, la donnée qui a étayé votre proposition de diagnostic ?
A - Le volume testiculaire (6 ml) que vous interprétez comme signant un début de puberté
B - L'importance du retard de taille (-3 DS)
C - L'importance du retard d'âge osseux (3 ans)
D - L'absence de pilosité pubienne
E - L'absence de pilosité axillaire
Bonne(s) réponse(s) : A

A - Age osseux en rapport avec la taille des testicules (2 à 3,2 cm à 12 ans).

Quelle proposition s'impose parmi les suivantes :


A - Vérifier le caryotype
B - Doser LH et FSH avant et après LH-RH
C - Doser la testostérone
D - Test à l'ACTH
E - Aucune exploration mais revoir ce jeune homme dans 6 mois
Bonne(s) réponse(s) : E

S'assurer de la progression de la puberté et de la croissance.

Quelle ou quelles sont parmi les suivantes la ou les propositions thérapeutiques qui sera ou seront
probablement recommandées ?
A - Abstention de tout traitement médicamenteux
B - Conseiller de réduire l'activité sportive
C - Traitement substitutif par gonadotrophines chorioniques
D - Traitement substitutif par testostérone
E - Traitement par un anabolisant de synthèse
Bonne(s) réponse(s) : A

C - Proposé dans les cryptorchidies (1000 à 1500 unités par semaine pendant 3 semaines).
D - Entraîne une maturation osseuse et limite la taille finale.

Concernant le diagnostic, on s'attend :


A - A une accélération de la croissance et une apparition des caractères sexuels secondaires dans les mois à
venir, en l'absence de traitement
B - A une puberté normale et une taille adulte satisfaisante au prix d'un traitement adéquat
C - A une taille définitive insuffisante avec ou sans traitement
D - A une taille définitive excessive, à une gynécomastie, quelle que soit l'attitude thérapeutique
E - A une stérilité en l'absence de traitement
Bonne(s) réponse(s) : A

C - Dans la puberté retardée simple la taille finale est normale.


D - Syndrome de Klinefelter (XXY).

Une femme de 55 ans consulte pour asthénie et douleurs rhumatismales diffuses. Dans ses antécédents on relève 6
grossesses. la dernière survenue à 33 ans. Aménorrhée depuis l'âge de 38 ans, sans bouffées de chaleur. Céphalées traitées
sans succès par diverses médications symptomatiques depuis 3 ans. Intervention bilatérale pour décompression du nerf
médian au canal carpien deux ans avant la consultation. Douleurs rhumatismales traitées par indométhacine depuis 4 ans,
hyperglycémie détectée un an auparavant, aucun traitement particulier n'ayant été prescrit. A l'examen, malade en surcharge
pondérale (83 kg pour 1 ,79 m) avec syndrome dysmorphique associant un prognatisme majeur, une augmentation de la taille
des mains et des pieds (l'alliance a été élargie à deux reprises en 5 ans). On note la présence de nombreuses vergetures non
pigmentées et d'une discrète hypertrichose. La tension artérielle est de 18/10.

Quel(s) élément(s) de l'anamnèse est (sont) en faveur d'une acromégalie ?


A - La grande taille
B - Le syndrome du canal carpien
C - La multiparité
D - L'hyperglycémie
E - Les céphalées
Bonne(s) réponse(s) : B D E

A - S'observe dans la giganto-acromégalie en cas d'adénome somatotrope chez l'enfant.


B - Caractère bilatéral très évocateur.
D - Diabète présent dans 1/3 des cas.
E - Syndrome tumoral.

648
Exclusivement sur DOC - DZ : www.doc-dz.com NADJI 85
RESIDANAT EN POCHE TOME II
Cas Clinique en QCM

Quel(s) signe(s) clinique(s) viendra(viendront) renforcer la suspicion d'acromégalie ?


A - Une galactorrhée
B - Une hémianopsie latérale homonyme
C - Une cyphose
D - Des vergetures
E - Une voix bitonale
Bonne(s) réponse(s) : A C

A - Adénome mixte (GH + prolactine) ou compression de la tige pituitaire.


B - Hémianopsie bitemporale dans les tumeurs hypophysaires.
C - Spondylose d'Erdheim.
D - Signe d'hypercorticisme.
E - La voix bitonale se voit dans les compressions du nerf récurrent

Quel(s) signe(s) vous permettra(tront) d'étayer le diagnostic d'acromégalie ?


A - Une dysgénésie épiphysaire
B - Une hypertrophie des sinus frontaux
C - Une déformation de la selle turcique
D - Des calcifications des noyaux gris centraux
E - Aucun des signes précédents
Bonne(s) réponse(s) : B C

B, C - Signes radiologiques classiques d'acromégalie. La selle turcique pouvant présenter un épaississement et une
condensation de la paroi sellaire (bec acromégale) sous l'effet de la GH et/ou un agrandissement voire une destruction du
plancher sous l'effet du volume tumoral.

Parmi ces signes biologiques, le(s)quel(s) est (sont) compatible(s) avec le diagnostic d'acromégalie ?
A - Une hyperprolactinémie
B - Une hyperphosphorémie
C - Un taux d'hormone somatotrope (GH) s'élevant après injection
intraveineuse de 200 microg de TRH

D - Une baisse d'hormones thyroïdiennes fT3 et fT4 avec TSH élevée


E - Une absence de freinage de la GH au cours de l'hyperglycémie provoquée
Bonne(s) réponse(s) : B C E

A - Item discutable : adénome mixte et compression de la tige pituitaire pouvant causer une hyperprolactinémie mais il ne
s'agit plus d'une acromégalie au sens restrictif.
B - Augmentation réabsorption tubulaire du phosphore.
C - Elévation "paradoxale" alors que le test est négatif chez le sujet normal.
E - Très évocateur.

Quelle(s) maladie(s) générale(s) peut(peuvent) être responsable(s) de l'apparition d'un syndrome du canal
carpien bilatéral ?
A - Le diabète
B - La maladie d'Addison
C - Le saturnisme
D - La porphyrie aiguë intermittente
E - L'hypothyroïdie
Bonne(s) réponse(s) : A E

A, E - Classique avec les dépôt d'urate dans la goutte, d'amylose dans le mélome.

La patiente a été opérée d'un macroadénome hypophysaire, et une insuffisance surrénalienne d'origine centrale
se démasque dans les suites de l'intervention. Quelle(s) mesure(s) thérapeutique(s) envisagez-vous ?
A - Un régime désodé
B - Prednisolone, 4 cp à 5mg par jour
C - Hydrocortisone 3 cp à 10 mg par jour
D - Fludrocortisone 1 cp à 50 microg par jour
E - Aucune de ces mesures
Bonne(s) réponse(s) : C

A - Régime normosalé.
B - Inadapté.
C - Est parfois donné systématiquement après l'intervention à titre préventif.
D - Dans les insuffisances corticotrope (central) il n'y a pas de déficit minéralocorticoïde.

649
Exclusivement sur DOC - DZ : www.doc-dz.com NADJI 85
RESIDANAT EN POCHE TOME II
Cas Clinique en QCM

Vous revoyez la patiente 6 mois après l'intervention neurochirurgicale. Quel(s) élément(s) du bilan biologique de
contrôle est (sont) en faveur d'une acromégalie toujours évolutive ?
A - Le taux de base de l'hormone somatotrope est normal
B - La phosphorémie est élevée
C - Le taux d'hormone somatotrope s'élève après injection de TRH
D - Le taux d'hormone somatotrope (GH) s'élève sous bromocriptine (2 cp à 2.5 mg per os)
E - IGF1 (somatomédine) élevée
Bonne(s) réponse(s) : B C E

B - Classique.
C - Signe la persistance de tissus adénomateux échappant aux mécanismes de régulations normaux (voir QCM 9).
D - La bromocriptine baisse la GH et est parfois utilisée dans le traitement de l'acromégalie.
E - Un traitement efficace normalise l'IGF 1 (très bon témoin).

Une jeune femme de 22 ans, en instance de divorce, consulte pour de la nervosité avec crises de larmes, de l'angoisse et des
bouffées de chaleur. Elle est mère de 2 enfants normaux âgés de 3 et 1 ans et soumise depuis son dernier accouchement à
une contraception par oestro-progestatifs toujours en cours. Une maladie de Graves-Basedow est suspectée.

En faveur de ce diagnostic, l'interrogatoire peut retrouver :


A - Anorexie
B - Amaigrissement
C - Thermophobie
D - Polydypsie
E - Constipation
Bonne(s) réponse(s) : B C D

A - Polyphagie.
B, C, D - Classique.
E - Diarrhée.

Dans la maladie de Graves-Basedow le goitre peut être :


A - Absent
B - Diffus
C - Uni-nodulaire
D - Multi-nodulaire
E - Soufflant
Bonne(s) réponse(s) : A B E

A - Le goitre se voit dans 90 à 99% des cas.


B, E - Goitre diffus, homogène, élastique ou ferme et vasculaire.

Dans la maladie de Graves-Basedow les signes oculaires peuvent être :


A - Une protrusion des globes associée à un à une rétraction de la paupière supérieure
B - Réduits à une exophtalmie unilatérale
C - Réduits à une rétraction de la paupière supérieure
D - Une diplopie
E - Un glaucome
Bonne(s) réponse(s) : A B C D

B - Rare, mais possible.


D - Par atteinte oculomotrice.

Dans la maladie de Graves-Basedow, il peut y avoir tachycardie :


A - Permanente
B - Sinusale
C - Irrégulière par fibrillation auriculaire
D - Sensible aux bêta-bloquants
E - Sensible aux digitalines
Bonne(s) Réponse(s) : A B C D

C - Justifiant un traitement anticoagulant.


D, E - Répond très bien aux Bêta-bloquants (traitement de choix) et mal aux digitaliques.

650
Exclusivement sur DOC - DZ : www.doc-dz.com NADJI 85
RESIDANAT EN POCHE TOME II
Cas Clinique en QCM

Si le diagnostic de maladie de Graves-Basedow est confirmé, le choix de l'indication thérapeutique se fera entre
:
A - Administration d'une dose thérapeutique de radio-iode
B - Lobectomie thyroïdienne unilatérale
C - Thyroïdectomie sub-totale
D - Antithyroïdiens de synthèse
E - Tranquilisants et sédatifs
Bonne(s) Réponse(s) : C D

A - Patiente trop jeune.


B - Geste insuffisant.
E - Traitement symptomatique d'appoint.

Au cours du traitement par le Néomercazole, on peut observer :


A - L'apparition d'un goître ou son augmentation s'il existait
B - Un rash
C - Un ictère
D - Une insuffisance rénale aiguë
E - Une agranulocytose
Bonne(s) Réponse(s) : A B C E

A - Signe une hypothyroïdie par surdosage.


B - Allergie médicamenteuse.
C - Hépatite cytolitique, cholestase et stéatose sont rares mais ont été décrites (Réf. : J. Frexinos, Hépato-gastro-entérologie
clinique, Simep).
E - Trouble hématologique dans 0,7% des cas.

Au cours du traitement par le Néomercazole, il faut demander :


A - Numération globulaire et des plaquettes
B - Dosage des transminases
C - Dosage de la créatininémie
D - Dosage de la TSH sanguine
E - Dosage des hormones thyroïdiennes sanguines
Bonne(s) Réponse(s) : A B D E

A - Surveillance des troubles hématologiques.


B - Surveillance hépatite cytolytique (rare).
D, E - Surveillance efficacité et surdosage (hypothyroïdie avec élévation de la TSH et augmentation du goitre).

Une femme de 34 ans se. plaint depuis quelques semaines d'une nervosité inhabituelle, de palpitations, d'une thermophobie
et d'une soif excessive. Son médecin trouve une tachycardie permanente à 108/mn et découvre un nodule arrondi, de 2 cm de
diamètre environ, au niveau du lobe thyroïdien droit. L'examen clinique n'apporte par ailleurs aucun élément pathologique
notable. Une scintigraphie thyroïdienne est demandée. Elle montre une image de nodule hyperfixant de la moitié inférieure du
lobe thyroïdien droit; le reste du parenchyme thyroïdien n'est pas visualisé.

L'adénome thyroïdien toxique :


A - Est une affection auto-immune
B - Survient généralement sans cause déclenchante
C - Est souvent provoqué par une surcharge iodée iatrogénique
D - Se développe parfois en réaction à un panhypopituitarisme
E - Est souvent associé à un phéochromytome
Bonne(s) réponse(s) : B

B - Contrairement à la maladie de Basedow (stress, épisode de la vie génitale...).


C - S'applique parfois à la maladie de Basedow sur goitre pré-existant.
D - C'est l'inverse : adénome hypophysaire à TSH en réponse à une hypothyroïdie périphérique (rare).
E - Syndrome de Gorlin (II B) : phéochomcytome, cancer médullaire de la thyroïde et névromes muqueux.

En cas de nodule chaud thyroïdien, la scintigraphie :


A - Est le principal examen complémentaire permettant d'affirmer le diagnostic
B - Fait courir le risque de dégénérescence cancéreuse du nodule
C - Montre toujours des aspects identiques avant et après stimulation par des injections de TSH
D - Peut dans certains cas objectiver plusieurs nodules chauds
E - Peut toujours être précisée par une échographie thyroïdienne
Bonne(s) réponse(s) : A D E

A - Car apportant le caractère hypofixant du nodule avec extinction du reste du parenchyme.


C - Le parenchyme sain est réactivé par la TSH et devient visible à la scintigraphie (test de Querido).
E - Le terme "toujours" prête à discussion mais l'échographie apporte des renseignements (taille, nature) sur le nodule.
651
Exclusivement sur DOC - DZ : www.doc-dz.com NADJI 85
RESIDANAT EN POCHE TOME II
Cas Clinique en QCM

L'adénome thyroïdien toxique :


A - S'accompagne généralement d'un taux bas de la TSH ultrasensible sérique
B - S'accompagne fréquemment d'un taux élevé de la T3 sérique
C - S'accompagne fréquemment d'un taux élevé de la T4 sérique
D - S'accompagne le plus souvent d'un titre élevé des anticorps anti-récepteurs de TSH
E - S'accompagne fréquemment d'un titre élevé des anticorps anti-microsomes thyroïdiens
Bonne(s) réponse(s) : A B C

A, B, C - Hyperthyroïdie périphérique avec rétrocontrôle négatif de la TSH.


D - Maladie de Basedow.
E - Maladie de Basedow, thyroïdite chronique de Hashimoto.

Le nodule chaud thyroïdien :


A - S'accompagne toujours de signes cliniques de thyrotoxicose
B - Peut être traité par exérèse chirurgicale
C - Se cancérise assez fréquemment
D - Evolue assez souvent, après quelques années, vers une maladie de Basedow
E - Peut se présenter cliniquement comme une cardiothyréose
Bonne(s) réponse(s) : B E

A - Dans les nodules "compensés" où la production d'hormones thyroïdiennes est insuffisante pour entraîner une
hyperthyroïdie.
B - Traitement de choix. Possibilité de traitement complémentaire par Iode 131 après administration d'hormone thyroïdienne
pour préserver le tissu sain.
E - Fréquents troubles du rythme.

L'adénome thyroïdien toxique :


A - Est affirmé par la cyto-ponction thyroïdienne
B - A un aspect pathognomonique sur l'échographie thyroïdienne
C - Impose une confirmation diagnostique par examen tomodensitométrique
D - A généralement un aspect scintigraphique très évocateur
E - Nécessite une scintigraphie corporelle complète à l'iode radioactif à la recherche de métastases
Bonne(s) réponse(s) : D

A - La cytoponction à l'aiguille fine n'a pas sa place dans diagnostic.


B - Difficile à différencier d'une autre tumeur solide.
D - La scintigraphie est l'examen de choix.
E - Pathologie bénigne.

Une femme de 39 ans, sans antécédent, a depuis quelques semaines une asthénie importante. Elle n'a pas de céphalée, pas
de sueur, pas de palpitation. A l'examen on découvre une hypertension artérielle à 180/110 mm Hg. Le fond d'oeil est normal.
L'électrocardiogramme montre une hypertrophie ventriculaire gauche très modérée avec une nette onde U. La clairance de la
créatinine est de 100 ml/mn. La kaliémie mesurée à trois reprises est à 3 mmol/l.

Une hypokaliémie peut être secondaire à la prise :


A - De laxatifs
B - D'antalgiques
C - De diurétiques thiazidiques
D - D'aldactone
E - D'inhibiteur de l'enzyme de conversion de l'angiotensine
Bonne(s) réponse(s) : A C

C - Avec alcalose métabolique dans le tableau complet.


D - Hyperkaliémie avec acidose hyperchlorémique.
E - Hyperkaliémie.

L'examen biologique permettant de différencier hyperaldostéronisme primaire et secondaire est :


A - L'activité rénine plasmatique
B - La glycémie
C - La kaliurèse des 24 heures
D - L'aldostéronémie
E - L'aldostéronurie des 24 heures
Bonne(s) réponse(s) : A

A - Effondrée dans les hyperaldostéronismes primaires.


B - Anomalie de la tolérance aux sucres fréquente dans l'adénome de Conn.
C - Elevée dans les deux cas, mais Na/K urinaire > 1 dans l'hyperaldostéronisme primaire uniquement.
D, E - Elevé dans les deux cas.

652
Exclusivement sur DOC - DZ : www.doc-dz.com NADJI 85
RESIDANAT EN POCHE TOME II
Cas Clinique en QCM

Les examens pouvant permettre de différencier hyperplasie bilatérale et adénome de Conn sont :
A - Le dosage de l'activité rénine dans le sang des veines surrénales
B - Le dosage de l'aldostérone dans le sang des veines surrénales
C - Le test de freination de l'hyperaldostéronisme par du soluté salé isotonique
D - Le dosage de l'hémoglobine glycosylée
E - La scintigraphie surrénalienne au iodo cholestérol
Bonne(s) réponse(s) : B C E

A - Effondrée dans les deux cas.


B - Gradiant d'aldostérone latéralisé du côté de l'adénome.
C - Freination en cas d'hyperplasie uniquement.
D - Hyperfixation latéralisée du côté de l'adénome.

Les examens pouvant permettre de localiser l'adénome de Conn comportent :


A - L'examen tomodensitométrique
B - L'artériographie rénale
C - Les tomographies des loges rénales
D - La radiographie sans préparation de l'abdomen
E - La résonance magnétique nucléaire
Bonne(s) réponse(s) : A E

A, E - Bonne résolution.
B - Phlébographie des veines surrénales.
C, D - Résolution insuffisante.

Le médicament le plus logique pour traiter l'hypertension artérielle secondaire à une hyperplasie bilatérale est :
A - Un diurétique thiazidique
B - Un diurétique de l'anse
C - Un inhibiteur de l'enzyme de conversion de l'angiotensine
D - Un béta-bloquant
E - Une spironolactone
Bonne(s) réponse(s) : E

E - 3 à 5mg/kg/jour, constitue le traitement de première intention, la surrénalectémie bilatérale subtotale ne normalisant le plus
souvent pas la pression artérielle.

653
Exclusivement sur DOC - DZ : www.doc-dz.com NADJI 85
RESIDANAT EN POCHE TOME II
Cas Clinique en QCM
Un homme de 63 ans est hospitalisé pour des douleurs abdominales diffuses avec constipation, anorexie, amaigrissement
progressif de 7 kg depuis neuf mois. Il n'a pas d'autres antécédent pathologique qu'une orchi-épididymite survenue dix ans
auparavant et traitée durant six mois par l'isoniazide. Il s'agit par ailleurs d'un éthylique chronique. L'examen clinique détecte
une hépatomégalie de volume modéré (flèche hépatique 14 cm), globale, ferme, sans signe clinique d'insuffisance hépatique
ou d'hypertension portale. Il existe une hyperpigmentation diffuse, brunâtre, prédominant sur les parties découvertes, les
mamelons, les organes génitaux externes. Les chiffres tensionnels sont de 110/70 mmHg. Le reste de l'examen clinique est
sans particularité.

Voici le résultat des explorations réalisées :


- VS : 36/78 mm
- l'hémogramme est normal
- urée = 0,43 g/l
- Na = 132 mEq/l
- K = 4,8 m Eq/l
- Cl = 98 mEq/l
- cholestérol : 2,9 g/l
- triglycérides : 4,5 g/l
- gamma-GT : 72 UI/l (N = 5 à 36)
- sidérémie : 172 mcg/100 ml
- coefficient de saturation de la sidérophiline : 65 %
- intradermo-réaction à la tuberculine (10 U) : induration de 12 mm
- radiographie sans préparation du thorax, transit gastroduodénal et lavement baryté : normaux. Radiographie de l'abdomen
sans préparation et tomodensitométrie abdominale : on observe deux calcifications de petites taille, en projection de l'aire
surrénalienne gauche.
- cortisolémie :
- 8 h : 6,2 mg/l 00 ml
- 16 h : 3,4 mg/100 ml
- 24 h : 2,2 mg/l 00 ml
- ACTH :
- 8 h : 570 pg/ml (N 10 à 90)
- 16 h : 420 pg/ml
- 24 h : 336 pg/ml
- aldostérone : 12 pg/ml (N 50 à 150)
- activité rénine plasmatique : 13,6 ng/ml/h d'angiotensine 1 (N 0,3 à 1,5)
- 17 OH : 2,2 mg/24 h
- 17 Céto : 6 mg/24 h
- cortisol libre urinaire : 8 mcg/24 h
- aldostéronurie : 2 mcg/24 h (N 3 à 20)
On évoque la maladie d'Addison.

Tous les éléments cliniques suivants notés chez le patient sont en faveur de la maladie d'Addison, à l'exception
d'un seul. Lequel ?
A - Mélanodermie
B - Anorexie
C - Amaigrissement
D - Troubles digestifs
E - Hépatomégalie
Bonne(s) réponse(s) : E

A, B, C, D - Signes cliniques classiques d'insuffisance surrénale basse.

Seul l'un des signes biologiques suivants ne peut être expliqué par un déficit surrénalien. Lequel ?
A - Na : 132 mEq/l
B - Triglycérides : 4,5 g/l
C - 17 céto : 6,7 mg/24 h
D - Aldostéronémie : 12 pg/ml
E - ACTH(8 h) : 570 pg/ml
Bonne(s) réponse(s) : B

A - Hyponatrémie par fuite urinaire de sodium.


C, D - Déficit glucocorticoïde et minéralocorticoïde.
E - Elévation de l'ACTH pour levée du rétrocontrôle négatif par les glucocorticoïdes.

Pour confirmer le diagnostic de maladie d'Addison, vous réalisez l'une des explorations suivantes. Laquelle ?
A - Test de stimulation par la 1.24 corticotrophine (Synacthène®)
B - Test de freinage par la dexaméthasone
C - Test de stimulation de l'activité rénine plasmatique et de l'aldostérone par le furosémide (Lasilix®)
D - Test de restriction hydrique
E - Scintigraphie surrénale à l'iodocholestérol
Bonne(s) réponse(s) : A

A - A réaliser en milieu hospitalier sous surveillance étroite. Confirme l'origine basse du déficit (absence de réponse).
B - Surtout utilisé dans les hypercorticismes.
D - Dans la maladie d'Addison on réalise au contraire une épreuve de diurèse provoquée (test de Robinson).

654
Exclusivement sur DOC - DZ : www.doc-dz.com NADJI 85
RESIDANAT EN POCHE TOME II
Cas Clinique en QCM

Parmi les arguments suivants, lequel(lesquels) observé(s) chez le patient plaide(nt) en faveur d'une maladie
d'Addison d'origine tuberculeuse ?
A - Antécédent d'orchi-épididymite
B - Données de la radiographie thoracique
C - Données de la radiographie d'abdomen
D - Données de la tomodensitométrie surrénale
E - Résultat de l'intradermo-réaction à la tuberculine
Bonne(s) réponse(s) : A C D E

A - L'insuffisance surrénale survient habituellement 10 à 15 jours après l'atteinte tuberculeuse initiale.


C, D - Signes classiques.

Choisissez parmi les thérapeutiques suivantes celle(s) qui s'applique(nt) spécifiquement au sujet :
A - Régime hyposodé
B - Hydrocortisone
C - Prednisone (Cortancyl®)
D - 9-alpha-fluorohydrocortisone
E - Traitement antituberculeux
Bonne(s) réponse(s) : B D E

A - Dangereux, au contraire régime normosalé.


B, D - Déficit gluco et minéralocorticoïde à corriger.
C - Inadapté.
E - Traitement étiologique.

Une jeune femme de 26 ans, d'origine française, est atteinte d'un diabète sucré depuis l'âge de 5 ans. Le diabète a été révélé
par une céto-acidose et depuis elle est insulino-traitée. La malade a été réglée à l'âge de 14 ans et depuis elle conserve des
cycles réguliers. Elle ne se connaît pas d'antécédents familiaux de diabète. Elle poursuit depuis de nombreuses années la
même insulinothérapie. Elle fait chaque matin, avant le petit déjeuner, toujours la même dose d'insuline, soit une injection
sous-cutanée de 40 unités d'une insuline lente au zinc. Elle change chaque jour le lieu d'injection de l'insuline : cuisse,
abdomen et bras
Le matin, la glycémie à jeun est voisine de 2 g/l (11,12 mmol/l). Elle a parfois des hypoglycémies en fin de matinée, qu'elle
perçoit bien et qu'elle corrige par absorption de sucre. Elle a une activité physique variable d'un matin à l'autre. Elle pèse 54
kg pour une taille de 1 m 65. Elle a réduit son apport glucidique à 130 g/j pour éviter d'être glycosurique. Elle prend
uniquement du fromage lors de ses collations ; midi et soir, l'apport est riche en viande et en oeufs. A l'examen, la TA est à
16/10 en position debout et couchée, les pouls périphériques sont bien perçus, les réflexes achilléens sont absents, il n'y a
pas de trouble du rythme cardiaque, le transit digestif est normal. Au fond d'oeil il existe quelques microanévrysmes et des
exsudats ponctués ; l'acuité visuelle est normale. Vue récemment par un diabétologue, il a été demandé les explorations
biologiques suivantes :
- présence d'une microalbuminurie à 150 mg/l
- taux d'hémoglobine glycosée HbAl à 13 % (N : 5 à 8)
- créatinine plasmatique à 10 mg/l
- cholestérol à 2,80 g/l
- triglycérides à 2,50 g/l
Cette jeune femme est adressée en hospitalisation car, mariée depuis peu, elle désire une grossesse. Elle n'est pas inscrite à
une association de diabétiques et n'a jamais mesuré sa glycémie sur sang capillaire au moyen d'une bandelette hémoréactive.
Elle n'a pas actuellement de moyen contraceptif.

Après lecture de cette observation, quelle est l'étiologie probable de ce diabète qui a débuté dans l'enfance ?
Une seule des propositions suivantes est exacte. Laquelle ?
A - Diabète gestationnel
B - Diabète de type II, non insulino-dépendant, à début précoce (type M.O.D.Y.), devenu rapidement insulino-
requérant
C - Diabète de type I, juvénile, insulinoprive et insulino-dépendant
D - Diabète nutritionnel, par malnutrition au cours de la première enfance
E - Diabète endocrinien, d'origine hypophysaire ou surrénalienne
Bonne(s) réponse(s) : C

A - Le diabète gestationnel survient chez la mère durant la grossesse.


B - Le diabète "M.O.D.Y." survient plus tard et n'est pas insulino-dépendant d'emblée.
C - Tableau typique.
D - Ne donne pas un diabète d'emblée insulino-dépendant.
E - Habituellement non insulino-dépendant et tableau différent.

655
Exclusivement sur DOC - DZ : www.doc-dz.com NADJI 85
RESIDANAT EN POCHE TOME II
Cas Clinique en QCM

Que pensez-vous de l'équilibre du diabète, de sa thérapeutique et de sa diététique ? Une seule des propositions
suivantes est exacte. Laquelle ?
A - L'alimentation est hyperlipidique
B - La ration glucidique est suffisante
C - Le diabète est équilibré
D - Le type d'insuline utilisé est adapté aux besoins du diabète
E - Le schéma d'injection de l'insuline est valable
Bonne(s) réponse(s) : A

A, B - La patiente n'étant pas en surcharge pondérale, doit suivre un régime normo-calorique (1600 à 2000 cal/jour). La ration
glucidique est donc insuffisante au profit des graisses (viande, oeuf).
C - Non, glycémie à jeun : 2g/l et HbA1 = 13%.
D, E - Mauvais équilibre glycémique justifiant une insulinothérapie reposant sur 2 voire 3 injections par jour d'une insuline
semi-lente et/ou rapide.

Que pensez-vous de ce diabète et de son évolution ? Une seule des propositions suivantes est exacte. Laquelle
?
A - Il s'agit d'un vrai diabète instable
B - Il existe une neuropathie végétative
C - Il existe une rétinopathie proliférante
D - Il existe une glomérulopathie diabétique de type Kimmelstiel-Wilson
E - Il existe une hypertension artérielle
Bonne(s) réponse(s) : E

A, B - Pas d'arguments dans l'observation.


C - Le fond d'oeil montre quelques anomalies pouvant au plus correspondre à une rétinopathie non-proliférante.
D - Glomérulopathie à forme nodulaire s'observant à un stade de lésions diffuses avancées.
E - HTA à traiter impérativement chez une diabétique présentant un début de rétinopathie et néphropathie.

En cours d'hospitalisation, laquelle ou lesquelles des attitudes suivantes allez-vous retenir ?


A - Réduction de l'apport en protéines animales
B - Association d'une sulfonylurée à l'insuline
C - Conservation du même type d'insuline mais augmentation de la posologie pour corriger la glycémie du
matin
D - Education à l'autosurveillance glycémique
E - Adaptation d'un nouveau schéma thérapeutique, p.ex. la pompe à infusion sous-cutanée ou l'association
matin et soir d'insulines ordinaire et semi-lente
Bonne(s) réponse(s) : A D E

A - Voir question [246].


B - Inefficace dans le diabète de type I où l'insulinotsécrétion endogène est nulle.
C - Inefficace voire dangereux (hypoglycémie).
D - Indispensable pour une bonne prise en charge.
E - Un équilibre glycémique optimum doit être obtenu avant le début de la grossesse.

A propos de la grossesse souhaitée, quelle est votre position au moment de l'hospitalisation de la malade ?
Quelle est ou quelles sont, parmi les propositions suivantes, celle ou celles que vous retenez ?
A - Il y a des risques maternels à la survenue d'une grossesse
B - Il n'y a aucun risque pour l'enfant si la conception à déjà eu lieu, pourvu que l'on assure ensuite l'équilibre
pendant la grossesse
C - Compte tenu des antécédents familiaux, l'enfant n'a aucun risque de devenir diabétique
D - La ration glucidique actuelle peut convenir en cas de grossesse
E - On ne peut pas obtenir un bon équilibre du diabète en cours de grossesse avec une seule injection de
l'insuline utilisée par la malade
Bonne(s) réponse(s) : A E

A - Aggravation des complications chroniques du diabète par déséquilibre glycémique, hypertension artérielle...
B, C, E - Evident.
D - Insuffisante.

656
Exclusivement sur DOC - DZ : www.doc-dz.com NADJI 85
RESIDANAT EN POCHE TOME II
Cas Clinique en QCM
Une femme de 30 ans, pesant 45 kg pour 1,68 m, vient consulter pour oedèmes. Ceux-ci auraient été constatés pour la
première fois il y a plus de trois ans et seraient maintenant permanents. Elle a, de plus, une constipation chronique. L'examen
clinique ne retient aucune anomalie physique, à l'exception d'une discrète sensibilité du cadre colique. Enfin, la patiente se
montre fort anxieuse d'une hypokaliémie trouvée à 3,4 mmol/l il y a un mois et à 2,8 mmol/l il y a 24 heures.

Une hypokaliémie sévère peut entraîner les complications suivantes, à l'exception d'une, laquelle ?
A - Paralysie périphérique
B - Trouble du rythme cardiaque
C - Constipation
D - Hypoglycémie
E - Rhabdomyolyse
Bonne(s) réponse(s) : D

A - Paralysie flasque touchant initialement les jambes.


B - Pouvant entraîner une mort subite.
C - Attente de la musculature lisse, iléus paralytique.
D - Dans l'hyperminéralocorticisme intolérance aux hydrates de carbone voire diabète dans plus de la moitié des cas.
E - Exceptionnel.

Le diagnostic d'oedèmes cycliques est possible. Indiquez parmi les anomalies biologiques suivantes celle qui
vous orienterait vers un abus de diurétiques :
A - Hypoprotidémie
B - Alcalose métabolique
C - Acidose métabolique
D - Hypernatrémie
E - Hyperchlorémie
Bonne(s) réponse(s) : B

A - Au contraire, hyperprotidémie par déshydratation.


B - Avec les diurétiques de l'anse et les thiazidiques.
C, E - Avec les antialdostérones qui sont hyperkalémiants.
D - Au contraire, hyponatrémie.

Et celle qui orienterait vers un abus de laxatifs :


A - Hypoprotidémie
B - Alcalose métabolique
C - Acidose métabolique
D - Hypochlorémie
E - Hyperchlorémie
Bonne(s) réponse(s) : C

C - Par perte extrarénale de bicarbonate.


A - Item litigieux car une hypoprotidémie pourrait se voir en cas de malabsorption secondaire à un abus prolongé de laxatif.

Indiquez le seul paramètre biologique dont la variation vous permet de faire la distinction entre hypokaliémie
d'origine digestive et hypokaliémie induite par les diurétiques thiazidiques ou de l'anse :
A - Kaliurèse
B - Chlorurie
C - Protéinurie > 0, 5 g/24 h
D - Aldostéronurie
E - Amino-acidurie
Bonne(s) réponse(s) : A

A - Kaliurèse élevée si perte rénale (diurétique) et basse si perte extra-rénale (digestive).


B - Item litigieux car les diurétiques thiazidiques et de l'anse inhibent la réabsorption du chlore et augmentent la chlorurie.

Entretenue pendant plusieurs années, une kypokaliémie chronique peut induire une néphropathie organique.
Des anomalies ci-dessous, indiquez celle(s) qui peu(ven)t s'observer en cas de néphropathie kaliopénique :
A - Protéinurie < 1 g/24 h
B - Protéinurie tubulaire
C - Hypocalciurie
D - Perte du pouvoir de concentration de l'urine
E - Augmentation de la filtration glomérulaire
Bonne(s) réponse(s) : A B C D

A, B - Protéinurie modérée possible.


D - Présente précocement avec, à un stade avancé, perte du pouvoir de dilution des urines.
Réf. : P. Godeau, Traité de Médecine, Flammarion.
657
Exclusivement sur DOC - DZ : www.doc-dz.com NADJI 85
RESIDANAT EN POCHE TOME II
Cas Clinique en QCM

Des propositions thérapeutiques suivantes destinées au traitement d'une hypokaliémie à 2,8 mmol/l, sans
signes cliniques d'accompagnement, comme dans le cas présent, laquelle retenez-vous ?
A - Perfusion de chlorure de K
B - Perfusion de chlorure de Na
C - Régime riche en sodium
D - Régime riche en fruits et légumes
E - Diurétiques épargnant le potassium
Bonne(s) réponse(s) : D

A - Inutile, compte-tenu de l'ancienneté des troubles et de la bonne tolérance de l'hypokaliémie.


D - Bananes, abricots secs, lentilles, levure, pommes...

Une femme de 62 ans consulte pour les signes suivants apparus progressivement au cours des six derniers mois : asthénie,
apathie, frilosité, crampes musculaires et prise de poids de 6 kg. Le visage est arrondi, la peau est froide, les aisselles sont
sèches, le rythme cardiaque à 64/mn, il n'y a pas d'oedème malléolaire. La patiente, dont la voix est rauque et lente, vous
apprend qu'elle a été traitée pendant 4 ans par isosorbide dinitrate (Risordan®) et par Cordarone® (amiodarone) pour une
angine de poitrine, mais qu'elle n'a plus eu de crise depuis près d'un an et que ce traitement a été alors interrompu. Il n'existe
pas de goitre. Au terme de cet examen, le diagnostic d'hypothyroïdie est hautement probable.

Le tableau clinique et l'anamnèse permettent d'évoquer une hypothyroïdie :


A - Périphérique idiopathique (involution simple de la thyroïde)
B - Centrale (hypothalamo-hypophysaire)
C - Périphérique iatrogénique
D - Par ectopie thyroïdienne
E - Secondaire à une thyroïdite chronique de Hashimoto
Bonne(s) réponse(s) : C

A - Signes apparus trop rapidement (6 mois).


B - Déficit thyréotrope rarement aussi complet et isolé, sans atteinte des autres fonctions antihypophysaires.
C - La rapidité d'installation des troubles et la prise de cordarone sont en faveur d'une hypothèse iatrogénique chez cette
patiente sans goitre.
D - Age trop tardif.
E - Absence de goitre.

Pour compléter votre information sur cette patiente. vous demandez :


A - Une cholestérolémie
B - Un ECG
C - Une T4 libre plasmatique
D - Une TSH plasmatique
E - Une scintigraphie thyroïdienne
Bonne(s) réponse(s) : A B C D

A - Hypercholestérolémie.
B - Signes d'hypothyroïdie (bradycardie sinusale, bloc auriculo-ventriculaire, bloc de branche, troubles de la repolarisation
diffus avec microvoltage) et signes d'insuffisance coronarienne.
C - Effondrée.
D - Elevée par levée du rétrocontrôle négatif par les hormones thyroïdiennes.
E - Montrerait une scintigraphie blanche ou une diminution globule de la fixation de l'iode, mais examen non indispensable.

Il pourra être utile de compléter ces examens par :


A - Une échocardiographie
B - Une hypoglycémie provoquée par l'insuline
C - Une épreuve de restriction hydrique
D - Un test à la LH - RH
E - Aucun des examens précédents
Bonne(s) réponse(s) : A

Dans ce contexte, et compte-tenu des antécédents d'angor, on recherchera une insuffisance cardiaque avec des zones
hypokinétiques (ischémie) et un épanchement péricardique (hypothyroïdie). Un bilan cardiaque soigneux est indispensable
avant toute opothérapie substitutive.

658
Exclusivement sur DOC - DZ : www.doc-dz.com NADJI 85
RESIDANAT EN POCHE TOME II
Cas Clinique en QCM

Le traitement initial de cette patiente va faire appel :


A - A la Thyroxine® par voie IV
B - Au Cynomel, (tri-iodothyronine) à la dose initiale de 75 microg/jour
C - A la reprise de la Cordarone® associée à 5 gouttes/jour de Thyroxine®
D - Au Levothyrox® (Lévothyroxine) à la dose initiale de 25 microg/jour associé secondairement à un bêta-
bloquant en l'absence de contre-indication de celui-ci
E - Au Levothyrox® à la dose initiale de 100 microg/jour
Bonne(s) réponse(s) : D

A DEFINIR PUISQUE NON PRECISE


A, B - Inadapté et posologie trop élevée.
C - Absurde.
D - On peut débuter le traitement par LT4 à dose plus élevée compte-tenu des antécédents d'angor et de l'âge de la patiente.
On augmentera ensuite par palier de 10 jours minimum sous stricte surveillance cardiologique.

L'attitude vis-à-vis de cette patiente au cours des quatre semaines suivantes va consister à :
A - Surveiller quotidiennement la fréquence cardiaque
B - Augmenter rapidement l'hormonothérapie supplétive
C - Effectuer des paliers posologiques d'au moins dix jours de l'hormonothérapie supplétive
D - Pratiquer une surveillance électrocardiographique
E - Doser la TSH, le cholestérol et la CPK sérique après quatre semaines de traitement
Bonne(s) réponse(s) : A C D E

A, D - Surveillance de la tolérance cardiaque, la survenue de signes d'angor devant éventuellement faire interrompre le
traitement substitutif.
E - Abaissement puis normalisation de la TSH, du cholestérol et des CPK (musculaires).

Une jeune femme de 35 ans, sans antécédents particuliers sinon un petit accident de la circulation avec simple contusion
crânienne, consulte pour une polyurie.

Quel(s) est (sont) I'(les) examen(s) en faveur d'un diabète insipide ?


A - Densité urinaire inférieure à 1005
B - Glycosurie : 200 g/24h
C - Calcémie 130 mg - Calciurie très élevée
D - Natriurèse inférieure à 20 mEq/24h
E - Diurèse des 24 heures supérieure à 3 litres
Bonne(s) réponse(s) : A E

A, E - Clairance de l'eau libre positive.


B - Diabète sucré.
C - Néphropathie calcique.
D - Natriurèse fonction des apports.

Quel(s) examen(s) urinaire(s) pratiquer dans ce cas ?


A - La densité
B - Le rapport Na/K
C - L'A.M.P. cyclique
D - L'osmolarité
E - La clairance de l'eau libre
Bonne(s) réponse(s) : A D E

A - Densité < 1005


C - Utile pour les pathologies parathyroïdiennes (PTH).
D - Osmolarité urinaire < 300mom/l ; osmolarité plasmatique le plus souvent normale.
E - Fortement positive (perte du pouvoir de concentration des urines).

Quel(s) résultat(s) attendre de l'épreuve de restriction hydrique dans le cas d'un diabète insipide ?
A - A la 8ème heure : le poids a diminué de 200 g et la TA est passée de 13 à 12
B - A la 6ème heure la densité urinaire passe de 1001 à 1002
C - Pas de modification du poids et de la TA après l'épreuve
D - Hyperthermie et polyurie pendant la restriction
E - Pas d'intérêt à réaliser cette épreuve
Bonne(s) réponse(s) : B D

A - Perte de poids plus importante (polyurie).


B - Absence de concentration des urines.
D - Déshydratation par polyurie d'où hyperthermie. Surveillance hospitalière indispensable.

659
Exclusivement sur DOC - DZ : www.doc-dz.com NADJI 85
RESIDANAT EN POCHE TOME II
Cas Clinique en QCM

Quelle est l'épreuve qui permet d'apprécier l'origine néphrogénique du diabète insipide ?
A - Test de la restriction hydrique
B - Test à la nicotine
C - Test à l'éthanol
D - Epreuve de surcharge salée
E - Test à la post hypophyse ou LVP
Bonne(s) réponse(s) : E

A - Ne permet pas de différencier un diabète insipide central et néphrogénique.


B - Stimule l'ADH (test de Cortes et Ganod) et réduit la diurèse (diagnostic différentiel avec la potomanie).
C - Inhibe l'ADH : utilisé dans le syndrome de sécrétion inapproprié d'ADH.
D - Augmente l'osmolarité plasmatique et donc l'osmolarité urinaire chez le sujet normal.
E - Négativation de la clairance de l'eau libre dans le diabète insipide central, pas d'effet dans le diabète insipide
néphrogénique.

Les étiologies possibles d'un diabète insipide central comportent :


A - Lupus
B - Sarcoïdose
C - Histiocytose
D - Syndrome de Kallmann de Morsier
E - Cranio-pharyngiome
Bonne(s) réponse(s) : B C E

B, C, E - Connaissances.
D - Hypogonadisme hypognadotrope avec anosmie par agénésie du diencéphale.

Une femme âgée de 25 ans consulte pour aménorrhée secondaire. L'interrogatoire permet de préciser qu'elle a eu ses
premières règles à l'âge de 13 ans, les cycles étant réguliers de 28 jours. Cette patiente s'est mariée à l'âge de 19 ans et a
alors pris des oestroprogestatifs (Stédiril® puis Adépal®) jusqu'à l'âge de 25 ans. Désirant une grossesse, elle a arrêté la
prise d'Adépal : elle consulte pour une aménorrhée apparue depuis 3 mois à l'arrêt de la contraception orale. La patiente ne
se plaint d'aucun signe fonctionnel. L'examen clinique note une galactorrhée provoquée bilatérale ignorée de la patiente, le
volume utérin est difficile à apprécier en raison d'une obésité.

Quel(s) diagnostic(s) peut-on évoquer ?


A - Synéchie utérine
B - Grossesse
C - Adénome à prolactine.
D - Syndrome des ovaires polykystiques
E - Hyperprolactinémie iatrogène
Bonne(s) réponse(s) : B C D E

A - Pas de galactorrhée (pas d'hyperprolactinémie).


B - Premier diagnostic à évoquer.
C - Les oestrogènes contenus dans la pilule contraceptive sont alors incriminés.
D - S'accompagnant dans 25% des cas d'une hyperprolactinémie mais rarement d'une galactorrhée.
E - Neuroleptiques, tricycliques, alpha-métyl-dopa, cimétidine... donnent une hypoprolactinémie en règle modéré (< 200mg/ml).

Quel(s) examen(s) complémentaire(s) demander dans un premier temps dans le cadre du bilan étiologique ?
A - Echographie gynécologique
B - HCG Béta
C - Dosage de prolactine
D - Radio de crâne standard
E - Courbe de température
Bonne(s) réponse(s) : A B C D

A - Ovaire polykystique.
B - Grossesse, indispensable à doser avant de réaliser la radio de crâne.
E - Etudie l'ovulation et la qualité du corps jaune.

660
Exclusivement sur DOC - DZ : www.doc-dz.com NADJI 85
RESIDANAT EN POCHE TOME II
Cas Clinique en QCM

Le premier bilan paraclinique réalisé permet de retrouver une hyperprolactinémie à 75 ng/ml. Quel(s) est (sont)
le(s) diagnostic(s) possible(s) au terme de cette exploration ?
A - Hyperprolactinémie iatrogène
B - Hyperprolactinémie idiopathique
C - Selle turcique vide
D - Adénome à prolactine
E - Grossesse
Bonne(s) réponse(s) : A B C D E

A - En règle modérée et répondant amplement au TRH. Représente la majorité des hyperprolactinémies.


C - Rare.
D - Prolactinémie environ 100mg/ml dans les microadénomes et > 200mg/ml dans les macroadénomes.
Réponse nulle ou diminuée au TRH.

Dans le cadre de la recherche étiologique quel(s) examen(s) complémentaire(s) doit-on réaliser en dehors de la
grossesse ?
A - Echographie cérébrale
B - Résonance magnétique nucléaire
C - Etude du champ visuel
D - Scanner hypophysaire
E - Test au TRH
Bonne(s) réponse(s) : C D E

A - Pas d'indication devant une hyperprolactinémie chez l'adulte.


B - Pas de première intention pour l'instant.
C, D - Recherche d'un processus tumoral.
E - La réponse normale est une stimulation de la prolactine. Cette réponse est ample dans les étiologie iatrogènes et
l'hyperprolactinémie fonctionnelle, elle est diminuée dans les adénomes.

Le bilan permet de retrouver sur les radios de crâne un bombement localisé de la paroi antéro-inférieure de la
selle turcique, avec amincissement de la corticale. Quel diagnostic retenez-vous ?
A - Hyper prolactinémie idiopathique
B - Macro adénome hypophysaire
C - Micro adénome hypophysaire
D - Arachnoïdocèle (selle turcique vide)
E - Méningiome
Bonne(s) réponse(s) : C

A - Pas d'anomalie radiologique.


B - Item litigieux, car une image similaire pourrait être observée avec un macroadénome (diamètre > 10mm) peu important.
D, E - Atteinte de l'ensemble de la selle turcique.

Quel(s) traitement(s) peut-on proposer à la patiente ?


A - Surveillance simple en raison du caractère modéré de l'hyperprolactinémie
B - Traitement médical anti PRL agoniste de la Dopamine (type Bromocriptine)
C - Oestroprogestatifs minidosés
D - Radiothérapie
E - Traitement chirurgical puis radiothérapie
Bonne(s) réponse(s) : B

A - Non, car risque de croissance tumorale et désir de grossesse.


B - Sera alors poursuivi ou arrêté (selon les écoles) lors de la grossesse.
D, E - La radiothérapie, avec le risque majeur d'insuffisance antéhypohysaire qu'elle comporte, risque de compromettre une
grossesse ultérieure.

661
Exclusivement sur DOC - DZ : www.doc-dz.com NADJI 85
RESIDANAT EN POCHE TOME II
Cas Clinique en QCM

Mr C. âgé de 50 ans, alcoolique pèse 80 kg pour 1m66. A la suite d'un excès important de boissons alcoolisés il s'endort par
terre écrasant son membre inférieur droit. On le retrouve 20 heures plus tard dans sa chambre avec un membre inférieur droit
oedématié. A l'entrée à l'hôpital il est anurique, la pression artérielle est normale, il n'a aucun antécédent urinaire.
Le prélèvement de sang veineux montre les résultats suivants :
- Sodium 13 mmol/l - Potassium 7,8 mmol/l - CO2 total plasmatique 18 mmol/l
- Chlore 104 mmol/l - Calcium 1,9 mmol/l - Phosphore 2,5 mmol/l
- Créatinine 500 micromol/l - Acide urique 100 micromol/l. - Urée 20 mmol/l.

Quelle est chez ce malade l'anomalie biologique immédiatement la plus menaçante ?


A - Hyperuricémie
B - Hyperkaliémie
C - Hypocalcémie
D - Hyperphosphorémie
E - Baisse du C02 total
Bonne(s) réponse(s) : B

Une kaliémie > 7mM/l représente un risque majeur de trouble du rythme ou de la conduction cardiaque pouvant entraîner à
tout moment une mort subite.

Quelle est la cause la plus vraisemblable de cette insuffisance rénale aiguë, avec anurie ?
A - Etat de choc septique
B - Pancréatite aiguë
C - Rhabdomyolyse aiguë
D - Hyperuricémie majeure avec néphropathie interstitielle goutteuse
E - Hyperuricémie avec lithiase urique bilatérale
Bonne(s) réponse(s) : C

Secondaire à la compression du membre inférieur droit avec lyse musculaire.

Quel(s) dosage(s) plasmatique(s) faut-il demander pour confirmer la cause suspectée de l'insuffisance rénale
aiguë ?
A - Nouveau dosage de l'acide urique
B - Amylase
C - Phosphatases alcalines
D - Aldolase
E - Créatine phosphokinase
Bonne(s) réponse(s) : D E

D, E - Libérées par la lyse musculaire.

Quelle est la meilleure mesure thérapeutique à envisager immédiatement ?


A - Gluconate de calcium intraveineux
B - Zyloric® per os (allopurinol)
C - Kayexalate® en lavements
D - Perfusion intra-veineuse de Dopamine
E - Séance d'hémodialyse
Bonne(s) réponse(s) : E

A - Traitement d'urgence des hyperkaliémies menaçantes, moins efficace que l'hémodialyse cependant.
C - Traitement de l'hyperkaliémie, mais insuffisant ici.
E - Moyen le plus efficace et le plus rapide de corriger l'ensemble des troubles associés à l'insuffisance rénale aiguë.

Après la période anurique, à la reprise de la diurèse, quelle complication risque d'apparaître ?


A - Hypercalcémie
B - Hyperuricémie
C - Hyperphosphorémie
D - Hyperkaliémie
E - Hyperglycémie
Bonne(s) réponse(s) : A

Par déshydratation secondaire à la polyurie survenant lors de la reprise de la diurèse et pouvant être aggravée par une
compensation excessive des pertes urinaires.

662
Exclusivement sur DOC - DZ : www.doc-dz.com NADJI 85
RESIDANAT EN POCHE TOME II
Cas Clinique en QCM
Une femme de 70 ans est hospitalisée pour une insuffisance rénale découverte 72 heures après une cholécystectomie pour
cholécyste lithiasique. Avant l'intervention, la créatinémie était à 90 micromoles/l, la pression artérielle à 140/80 mmHg.
Pendant l'intervention elle a présenté un collapsus cardio-vasculaire pendant une heure, traité par remplissage avec une
aspiration gastrique. L'examen révèle des oedèmes des lombes prenant le godet. Les muqueuses de cavité buccale sont
humides et on note une empreinte des dents sur la langue. La pression artérielle est de 170/110 mmHg, la fréquence
cardiaque est de 100 par minute. Il existe un bruit du galop à l'auscultation du coeur. La diurèse est de 100 ml par 24 heures.
Les examens sanguins montrent :
- Urée 35 mmol/l - Créatinine 600 micromol/l - Sodium 110 mmol/l - Potassium 3,1 mmol/l
- Bicarbonates 33 mmol/l - pH 7,5 - Protides 51 g/l - Hémoglobine 9,10 g/100 ml - Calcium 1,9 mmol/l.

Chez cette malade un ou plusieurs signes témoignent d'une hyperhydratation intracellulaire :


A - Les oedèmes des lombes
B - La tachycardie
C - L'empreinte des dents sur la langue
D - Le bruit de galop cardiaque
E - L'hypertension artérielle
Bonne(s) réponse(s) : C

C - On peut aussi observer des troubles digestifs (dégoût de l'eau, anorexie, nausée, vomissement), des troubles nerveux
(céphalées, crampes, convulsion voire coma), une asthénie et parfois une hypothermie.

Chez cette malade un ou plusieurs signes témoignent d'une hyperhydratation extra-cellulaire :


A - Les oedèmes des lombes
B - L'humidité des muqueuses de la cavité buccale
C - L'empreinte de dents sur la langue
D - Le bruit du galop cardiaque
E - L'hypertension artérielle
Bonne(s) réponse(s) : A D E

A - Oedèmes généralisés, blancs, mous, indolores, déclives, prenant le godet.

Il y a une hyperhydratation intracellulaire puisqu'il existe :


A - Une hypocalcémie
B - Une anémie
C - Une hyponatrémie
D - Une hypoprotidémie
E - Une alcalose
Bonne(s) réponse(s) : C

B, D - Peuvent être observées dans les hémodilutions importantes (hyperhydratation extracellulaire).


C - La natrémie reflète l'hydratation intracellulaire.

L'hyponatrémie peut expliquer :


A - Les nausées
B - Les céphalées
C - Les oedèmes
D - Le bruit de galop
E - La désorientation
Bonne(s) réponse(s) : A B E

Signes cliniques d'hyperhydratation intracellulaire.

Chez cette malade, l'hypocalcémie est surtout liée à :


A - L'alcalose métabolique
B - L'anémie
C - L'hypoprotidémie
D - L'hyponatrémie
E - L'anémie
Bonne(s) réponse(s) : C

40% du calcium est lié aux protéines plasmatiques (albumine) et l'interprétation de la calcémie doit toujours tenir compte de la
protidémie.
B, E - Répétition.

663
Exclusivement sur DOC - DZ : www.doc-dz.com NADJI 85
RESIDANAT EN POCHE TOME II
Cas Clinique en QCM

L'alcalose métabolique de cette malade est secondaire à :


A - L'insuffisance rénale
B - L'aspiration gastrique
C - L'anémie
D - L'hypoprotidémie
E - L'hypocalcémie
Bonne(s) réponse(s) : B

A - Acidose métabolique.
B - Par perte d'ion H+.
E - Les hypercalcémies peuvent s'accompagner d'une alcalose métabolique (cancer, vitamine D, Sarcoïdose)
Réf. : P. Godeau, Traité de Médecine, Flammarion.

L'hypokaliémie de cette malade peut être liée à un ou plusieurs facteurs :


A - L'insuffisance rénale
B - L'aspiration gastrique
C - L'alcalose métabolique
D - L'hypoprotidémie
E - L'anémie
Bonne(s) réponse(s) : B C

A - Hyperkaliémie.
B - S'observe dans les aspirations prolongées mal compensées.
C - C'est le mécanisme principal par transfert intracellulaire du K+.

Pour corriger l'hyponatrémie de cette malade il faut :


A - Perfuser en trois heures un litre de soluté salé isotonique
B - Perfuser en trois heures 500 ml de plasma
C - Injecter en intraveineux 8 g de chlorure de sodium
D - Donner un diurétique thiazidique
E - Faire une séance de rein artificiel
Bonne(s) réponse(s) : E

A - Aggravation de l'hyperhydratation globale.


B, C - Aggravation de l'hyperhydratation extra-cellulaire.
D - Inadapté.
E - Corrigera rapidement l'ensemble des troubles hydroélectriques associés.

Monsieur B . âgé de 57 ans, agent de bureau, présente un diabète. A l'âge de 48 ans une glycosurie avait été découverte à la
médecine du travail. La glycémie à jeun avait été dosé à 7,15 mmol/l (1,30 g/l). Une épreuve d'hyperglycémie provoquée par
voie orale avait été pratiquée (75 g de glucose dosages sur plasma veineux) donnant : au temps 0mn : 7,10 mmol/l ; à 30mn :
9,80 mmol/l; à 60mn : 14,20 mmol/l ; à 90mn : 13,20 mmol/l ; à 120mn : 12,3 mmol/l. A l'âge de 52 ans les glycémies à jeun
ont été constatées autour de 10 à 12 mmol/l. Un régime a été conseillé, mais non suivi. Puis monsieur B. a été traité par
comprimés, qu'il a secondairement abandonné. Depuis trois mois il présente une polyurie et a maigri de 4 kgs. Dans les
antécédents familiaux on relève : un diabète chez la grand mère paternelle et un fils. Monsieur B. consomme 3840
kcalories/jour. L'examen montre une hépatomégalie. Le poids est à 86 kg pour 1,68 m. Les examens biologiques donnent :
glycémie à 15 mmol/l ; hémoglobine glycosylée à 10,2 %, réserve alcaline à 27 mmol/l ; glycosurie + + ; pas de cétonurie ;
créatinine à 182 micromol/l ; protéinurie à 0,30 g/24 h ; transaminases GO (Asat) à 60 unités/l et GP (Alat) à 40 unités/l ;
protides totaux à 65 g/l avec bloc bêta-gamma et uricémie à 3/50 micromol/l .

A l'âge de 48 ans monsieur B. avait :


A - Une diminution de tolérance au glucose parce que la glycémie à 120mn était supérieure à 11 mmol/l
B - Une diminution de tolérance au glucose parce que la glycémie la plus élevée était supérieure à 11 mmol/l
C - Un diabète parce que la glycémie à jeun dépassait 7 mmol/l et la glycémie à 120mn 11 mmol/l
D - Un diabète parce que la glycémie à 120mn dépassait 11 mmol/l et que des glycémies intermédiaires
dépassaient aussi 11 mmol/l
E - Un diabète nécessitant seulement, pour le diagnostic, une seconde glycémie à jeun
Bonne(s) réponse(s) : D

Les critères de diabètes retenus sur une HGPO (75g de glucose, dosage glycémie sur plasma veineux) ont :
- deux glycémies à jeun > 7,7mM/l.
- et/ou glycémie à 120 minutes ³ 11,1 mM/l deux fois ou avec une valeur intermédiaire ³ 11,1mM/l (OMS, 1979).

664
Exclusivement sur DOC - DZ : www.doc-dz.com NADJI 85
RESIDANAT EN POCHE TOME II
Cas Clinique en QCM

Vous décidez de mettre ce patient au régime. Indiquez le régime qui vous parait adéquat :
A - 1200 calories avec 80 grammes de glucides
B - 1200 calories avec 250 grammes de glucides
C - 1600 calories avec 100 grammes de lipides
D - 2000 calories avec 270 grammes de glucides
E - 2000 calories avec 150 grammes de glucides
Bonne(s) réponse(s) : D

Dans un premier temps, on conseille un régime apportant les 2/3 des apports antérieurs. La moitié de cette ration doit être
composée de glucides.

Le régime (peut-être mal suivi) n'ayant pas permis d'obtenir un équilibre satisfaisant du diabète, vous proposez
un traitement médicamenteux. Vous récusez les biguanides parce que :
A - C'est un diabète héréditaire
B - C'est un diabète avec surcharge pondérale
C - Il existe une hépathopathie. Monsieur B. continue à boire du vin en quantité importante
D - La créatininémie constitue une contre-indication
E - L'uricémie constitue une contre-indication
Bonne(s) réponse(s) : C D

B - Au contraire, c'est l'indication de choix.


C - Contre-indication relative en l'absence d'insuffisance hépatique mais association à l'alcool déconseillée.
D - Contre-indication absolue car risque d'acidose classique.

Ce faisant vous écartez un des médicaments suivants. Lequel ?


A - Glucidoral®
B - Daonil®
C - Glibénèse®
D - Glucophage®
E - Diamicron®
Bonne(s) réponse(s) : D

A, B, C, E - Sulfamides hypoglycémiants.
D - Biguanides.

Vous récusez aussi les sulfamides et vous prescrivez de l'insuline parce que :
A - Le surpoids est une contre-indication absolue aux sulfamides
B - La créatininémie, à ce niveau, constitue une contre-indication relative
C - L'uricémie constitue une contre-indication
D - La perte de poids spontanée indique un diabète insulino-nécessitant
E - Une glycémie proche de la normale est souhaitable en raison des complications rénales
Bonne(s) réponse(s) : B E

A - Non, cependant ils peuvent aggraver l'hyperinsulinisme et on préfère les biguanides chez le diabétique avec surpoids.
B - Contrairement aux biguanides où l'insuffisance rénale même modérée est une contre-indication absolue, avec les
sulfamides, seule l'insuffisance rénale sévère est une contre-indication.
D - A priori non, tant que le patient est en surpoids majeur, mais il serait utile d'évaluer l'insulinothérapie.
E - Comme pour la rétinopathie.

M. P... 45 ans, consulte pour une asthénie, une polyurie, une soif excessive qui sont apparues depuis 3 mois environ ; le poids
et l'appétit sont conservés. L'examen clinique est normal. Mais le poids est de 90 kgs pour 1 m 75. Il existe une glycosurie + +
+ sans cétonurie. Deux glycémies à jeun sont respectivement à 9,1 mmol/l et 10 mmol/l (1,80 g/l. Le sérum est opalescent à
jeun. Cholestérol total = 5, 2 mmol/l (2,9 g/l), triglycérides = 4, 6 mmol/l (4g/l)

Quel(s) diagnostic(s) portez-vous ?


A - Trouble de la tolérance glucidique
B - Hyperlipoprotéinémie de type IV
C - Diabète non insulinodépendant
D - Hyperlipoprotéinémie de type IIa
E - Hyperlipoprotéinémie de type V
Bonne(s) réponse(s) : B C

B - Hypertriglycéridémie endogène souvent associée au diabète (glucido-dépendante).


C - Oui, car deux glycémies à jeun > 7,7mM/l (critère OMS, 1979).

665
Exclusivement sur DOC - DZ : www.doc-dz.com NADJI 85
RESIDANAT EN POCHE TOME II
Cas Clinique en QCM

Quel(s) examen(s) complémentaire(s) demandez-vous ?


A - ECG
B - Créatininémie
C - Fond d'oeil
D - Protéinurie des 24 heures
E - Epreuve de charge orale en glucose
Bonne(s) réponse(s) : A B C D

A, B, C, D - Font partie du bilan du diabète.


E - Inutile, le diagnostic de diabète étant déjà porté. Aucun intérêt dans la surveillance d'un diabète.

Quelle sera votre prescription dans l'immédiat ?


A - Insuline Rapitard® 10 U. matin et soir
B - Metformine
C - Daonil®
D - Régime hypocalorique sans sucre d'absorption rapide
E - Glucidoral®
Bonne(s) réponse(s) : D

D - Traitement de première intention du diabète de type II et de la dyslipidémie de ce patient. Si échec, on introduira des
hypoglycémiants oraux (biguanides) dans un deuxième temps.

Sur quel(s) élément(s) jugez-vous l'efficacité thérapeutique ?


A - Pesée
B - Hémoglobine glycosylée
C - Triglycéridémie
D - Insulinémie
E - Glycémie à jeun et post-prandiale
Bonne(s) réponse(s) : A B C E

A - Une perte de poids est indispensable.


B - Reflet de la glycémie de 6 à 8 semaines précédentes.
C - Doit se normaliser sous régime bien suivi.
D - Aucun intérêt ici (hyperinsulinisme).
E - Doivent être inférieurs à 7,7mM/l.

Une patiente de 38 ans, est hospitalisée pour des douleurs dorso-lombaires intenses, surtout à la marche. La taille actuelle est
inférieure de 5 cm à la taille antérieurement connue. A l'examen clinique on trouve une prédominance de la graisse à la partie
supérieure du corps, une tension artérielle à 170-110 mmHg. La patiente est en aménorrhée depuis 6 mois. Les radiographies
osseuses montrent une ostéoporose diffuse avec des séquelles de fracture de côte sans notion de traumatisme. La vitesse de
sédimentation est à 10 mm à la première heure. La glycémie à jeun est à 1,60g/l (8,90 mmol/l). La glycémie post-prandiale est
à 2,60g/l (14,45 mmol/l).

Quelle est l'hypersécrétion hormonale responsable ?


A - Aldostérone
B - Testostérone®
C - Noradrénaline
D - Déhydroépiandrostérone
E - Cortisol
Bonne(s) réponse(s) : E

Tableau clinique très évocateur d'un hypercorticisme.


A - Ne pourrait expliquer que l'HTA et donne un intolérance aux hydrates de carbone plus qu'un diabète franc.

Dans cette affection on observe :


A - Augmentation de l'épaisseur de la peau
B - Vitiligo
C - Ecchymoses
D - Peau moite
E - Erythrose
Bonne(s) réponse(s) : C E

A - Au contraire, peau mince et fragile résistant mal aux traumatismes.


B - Se voit dans certaines insuffisances surrénales lentes.
D - Hyperthyroïdie.

666
Exclusivement sur DOC - DZ : www.doc-dz.com NADJI 85
RESIDANAT EN POCHE TOME II
Cas Clinique en QCM

Dans cette affection la glycémie est :


A - Toujours basse
B - Parfois basse
C - Toujours élevée
D - Parfois élevée
E - Toujours normale
Bonne(s) réponse(s) : D

Fréquente intolérance aux hydrates de carbone voire diabète franc. Ces troubles sont liés à une stimulation de la
néoglucogenèse et une insulinorésistance périphérique avec hyperinsulinisme induit par les glucocorticoïdes.

Quel(s) dosage(s) urinaire(s) oriente(nt) le diagnostic de cette hypersécrétion hormonale ?


A - 17 hydroxycorticostéroïdes
B - THA tétrahydroaldostérone
C - Cathécholamines
D - Déhydroépiandrostérone
E - Testostérone
Bonne(s) réponse(s) : A

En règle générale, les 17 OH-stéroïdes urinaires sont élevés (> 8mg/24 heures) mais ne sont pas fiables. On préfère le
dosage du Cortisol libre urinaire qui est le critère le plus fiable et est retrouvé élevé sur les urines de 24 heures (> 100mg/24
heures).

L'absence de réponse à l'un de ces tests permet d'affirmer le diagnostic, lequel ?


A - 3 mg/jour de dexaméthasone pendant 5 jours
B - 5 mg IV de Phentholamine (Régitine)
C - Perfusion de 2 litres de sérum physiologique en 2 heures
D - 8 mg/jour de Dexaméthasone per os
E - Aucun de ces tests
Bonne(s) réponse(s) : A

A - Freinage faible qui doit abaisser les 17 OH-stéroïdes urinaires et le cortisol libre urinaire chez le sujet normal (test positif)
et pas dans les véritables hypercorticismes (test négatif).
B - Test de freination utilisé dans le phéochromocytome.
C - Diagnostic des hyperaldortéronismes.
D - Freinage fort permettant de différencier une maladie de Cushing (freinage positif) d'une sécrétion ectopique (freinage
négatif).

Le traitement médical de cette affection repose sur :


A - Un anticortisolique de synthèse (OP'DDD...)
B - Un antialdostérone (Spironolactone)
C - Un alpha bloquant (Régitine)
D - La Dexaméthasone per os
E - Aucun de ces médicaments
Bonne(s) réponse(s) : A

A - Associé à l'hydrocortisone (20mg/jour) pour prévenir une insuffisance surrénale.

Un sujet de 60 ans, atteint de diabète, traité par l'insuline avec contrôles irréguliers depuis 30 ans vient consulter pour
l'apparition de douleurs vives des membres inférieurs, d'apparition récente, survenant surtout la nuit, à type de paresthésie et
de dysesthésie entraînant l'insomnie. On découvre à l'examen un mal perforant plantaire.

Si on évoque une neuropathie diabétique, l'examen clinique pourra apporter des signes d'orientation,
le(s)quel(s) ?
A - Pouls mal perçus
B - Signe de Lasègue
C - Des mouvements de la hanche limités
D - Une perte de la sensibilité du diapason
E - Des réflexes achilléens nuls
Bonne(s) réponse(s) : D E

D, E - Traduisent une atteinte des fibres myélinisées de gros diamètre, témoin classique de la polynévrite diabétique.

667
Exclusivement sur DOC - DZ : www.doc-dz.com NADJI 85
RESIDANAT EN POCHE TOME II
Cas Clinique en QCM

Il est indispensable de pratiquer :


A - Electromyogramme
B - Etude des vitesses de conduction motrice et sensitive
C - Radiographie du bassin
D - Doppler artériel des membres inférieurs
E - Radiographie de la colonne lombaire
Bonne(s) réponse(s) : A B D

A, B - Le terme "indispensable" de l'énoncer peut prêter à discussion, si l'examen clinique est complet il peut suffire pour le
diagnostic de neuropathie diabétique.
D - L'association d'une macroangiopathie avec artérite des membres inférieurs et d'une neuropathie périphérique et végétative
est classique dans le mal perforant plantaire.

A propos du pronostic de la neuropathie diabétique :


A - Il est bon, avec récupération rapide
B - Il existe une possibilité d'association de troubles moteurs
C - Il existe une persistance des douleurs
D - Il n'y a pas de thérapeutique efficace
E - L'équilibre du diabète est le meilleur traitement de la neuropathie diabétique
Bonne(s) réponse(s) : B C D E

B - La polynéphrite est avant tout sensitive mais une atteinte motrice est possible, notamment dans la forme cachectique avec
amyotrophie proximale (cuisses).
C - Les douleurs disparaissent souvent après un certain temps d'évolution de la neuropathie.
D - On ne dispose que de traitements symptomatiques mais non curatifs.
E - Permettant de la stabiliser ou de ralentir la dégradation des paramètres cliniques et électrophysiologiques.

La neuropathie sensitive peut être associée à une neuropathie neuro-végétative. Ainsi il faut rechercher :
A - Toux opiniâtre
B - Impuissance
C - Hypotension orthostatique
D - Diarrhée motrice
E - Douleurs diffuses
Bonne(s) réponse(s) : B C D

B - D'origine souvent mixte (vasculaire et nerveuse) chez le diabétique.


C - Témoin de l'atteinte de l'ensemble du système cardiovasculaire.
D - Mise en évidence par un test au carmin. On observe aussi : gastroparésie, anhydrose, troubles de la motricité pupillaire...

Devant le mal perforant, la décision thérapeutique comportera :


A - Laisser évoluer sans traitement
B - Désinfecter
C - Conseiller le port de chaussures avec semelles orthopédiques
D - Conseiller le port de chaussures étroites à semelles de caoutchouc
E - Conseiller le port de bottes en caoutchouc
Bonne(s) réponse(s) : B C

D, E - Port de chaussures larges ne comprimant pas les pieds et n'entraînant pas de macération.

668
Exclusivement sur DOC - DZ : www.doc-dz.com NADJI 85
RESIDANAT EN POCHE TOME II
Cas Clinique en QCM
Une enfant de 13 ans est adressée pour l'exploration d'une hypercholestérolémie de 10,26 mmol/l, dépistée lors d'un bilan
systématique. Le grand-père de cette fillette est décédé brutalement à 46 ans, I'un de ses oncles à fait une hémiplégie à 58
ans et son père âgé de 38 ans est traité par du Fénofibrate (Lipanthyl). L'examen clinique est normal, l'état général est bon, la
pression artérielle est de 120/80 mmHg, le poids est stable à 43 kgs pour 153 cm. Les examens biologiques sanguins donnent
les résultats suivants :

Glucose : 5,30 mmol/l


Cholestérol : 11,05 mmol/l
Triglycérides : 0,92 mmol/l
Albumine : 40 g/l

Le ionogramme sanguin est normal. L'enquête alimentaire montre une prise énergétique de 2100 kcal/j (8732 kilojoules), 45
p.cent de lipides (3/4 sous forme de graisses animales), 43 p.cent de glucides (25 p.cent de sucres simples) et 12 p.cent de
protéines (3/4 sous forme de protéines animales).

La première étiologie à envisager devant cette hypercholestérolémie est la suivante :


A - Syndrome néphrotique
B - Diabète
C - Hypercholestérolémie familiale
D - Hypothyroïdie
E - Consommation excessive de graisses
Bonne(s) réponse(s) : C

A - Aucun élément clinique ni paraclinique.


B - La glycémie normale, l'absence d'hypertriglycéridémie associée sont contre ce diagnostic.
C - Diagnostic le plus probable, compte-tenu des antécédents familiaux et du chiffre très élevé de la cholestérolémie chez un
sujet jeune.
D - Pas de signe clinique, donne des hypercholestérolémies moins importantes.
E - Cholestérolémie trop élevée.

L'aspect du sérum de cette jeune patiente après décantation serait :


A - Lactescent
B - Opalescent
C - Clair avec anneau lactescent
D - Xanthochromique
E - Clair
Bonne(s) réponse(s) : E

A - Hypertriglycéridémie (I, IV) : surcharge en VLDL ou chylomicron.


B - Dyslipidémie mixte avec surcharge en VLDL (IIb, III).
C - Chylomicron.
E - Surcharge pure en LDL (IIa).

Les apolipoprotéines correspondant à cette hypercholestérolémie sont essentiellement :


A - De type A
B - De type B
C - De type C
D - De type E
E - De type H
Bonne(s) réponse(s) : B

A - L'apo A est la fraction soluble des HDL et des chylomicrons.


B - L'apo B est la fraction soluble des VLDL et LDL, elle est élevée (> 0,9g/l) dans les hypercholestérolémies de type IIa et IIb.

Le(s) risque(s) lié(s) à l'hyperlipoprotéinémie à plus ou moins long terme en l'absence de traitement est (sont) :
A - Thrombose coronarienne
B - Diabète
C - Obésité
D - Pancréatite aiguë
E - Xanthomes tendineux
Bonne(s) réponse(s) : A E

A - Atteinte précoce faisant la gravité du pronostic. L'artérite des membres inférieurs est plus tardive.
D - Dans les hypertriglycéridémie importantes (> 20g/l).
E - Siège le plus souvent aux tendons d'Achille (90%).

669
Exclusivement sur DOC - DZ : www.doc-dz.com NADJI 85
RESIDANAT EN POCHE TOME II
Cas Clinique en QCM

La (les) mesure(s) diététique(s) à envisager contre cette hyperlipoprotéinémie est (sont) :


A - Diminution des graisses totales
B - Diminution des apports de poissons
C - Diminution de la ration énergétique totale
D - Diminution du pourcentage des sucres simples
E - Augmentation des sources d'acides gras polyinsaturés
Bonne(s) réponse(s) : A E

A - La ration lipidique doit représenter 1/3 de la ration calorique.


B, E - Régime pauvre en cholestérol (oeuf, abats...) et riche en graisses polyinsaturées (poisson, huile végétale...).
C - Régime normocalorique.
D - Apport de 50% de la ration calorique sous forme de glucide dont 60 à 80% de sucres d'absorption lente.

Le médicament à utiliser en première intention après quelques mois de suivi du régime en cas de résultats
insuffisants :
A - Cholestyramine (Questran)
B - Bezafibrate (Befizal)
C - Fenofibrate (Lipanthyl)
D - Probucol (Lurselle)
E - Gemfibrozil (Lipur)
Bonne(s) réponse(s) : A

A - Traitement de première intention des hypercholestérolémies. Abaisse la cholestérolémie de 20 à 25% par inhibition du
cycle entéro-hépatique des acides biliaires. Réduction de 19% des accidents coronariens chez les sujets traités.
B, C, E - Traitement de l'hypertriglycéridémie. Abaissent de 25 à 50% la triglycéridémie.
D - Entraîne une réduction indésirable de 20 à 30% du HDL Cholestérol.
Réf. : Ph. Doroz, Guide Pratique des Médicaments, Maloine.

Un homme de 52 ans, hypertendu traité par 2 comprimés par jour de Catapressan vient vous consulter parce que, lors d'un
examen systématique, une prise de sang a donné les résultats suivants : glycémie 7,5 mmol/l (1,35 g/l), cholestérol 8 mmol/l
(3,09 g/l), triglycérides 4 mmol/l (3, 54 g/l). Il pèse 82 kgs pour 1,70 m ; la TA est à 17/9 ; l'examen est par ailleurs négatif.

Pour préciser le diagnostic des troubles métaboliques de ce patient, vous demandez dans un premier temps :
A - Un 2è dosage de glycémie, cholestérol et triglycérides
B - Un dosage d'insuline et de peptide-C à jeun
C - Une charge glucosée avec dosage de la glycémie avant et après charge
D - Une hyperlipémie provoquée
E - Un dosage des acides gras libres
Bonne(s) réponse(s) : A C

C - Il faut deux glycémies à jeun > ou = 7,7 mmol/l pour affirmer le diabète. Chez ce patient dont la glycémie à jeun n'est pas
normale, l'HGPO est nécessaire pour pouvoir affirmer le diagnostic.

Parmi les facteurs suivants, lequel (lesquels) peuvent jouer a priori un rôle dans l'élévation des triglycérides d'un
tel patient ?
A - L'excès pondéral
B - L'hyperglycémie si elle se confirme
C - L'hypertension artérielle
D - La médication hypotensive
E - Une consommation d'alcool trop importante
Bonne(s) réponse(s) : A B E

Dyslipidémie liée à la consommation d'alcool, de sucre et la ration calorique.

Quelle(s) autre(s) anomalie(s) biologique(s) convient-il de rechercher chez un tel patient ?


A - Une hyperuricémie
B - Une hypercalcémie
C - Une hyperkaliémie
D - Une élévation des gamma-GT
E - Une élévation de GH
Bonne(s) réponse(s) : A D

A - Triade métabolique : hypertriglycéridémie, diabète, hyperuricémie.


E - En l'absence d'anomalie clinique évocatrice d'acromégalie autre que l'HTA et une intolérance aux hydrates de carbone, il
n'est pas recommandé de doser le GH.

670
Exclusivement sur DOC - DZ : www.doc-dz.com NADJI 85
RESIDANAT EN POCHE TOME II
Cas Clinique en QCM

Le patient est menacé à plus ou moins longue échéance par le(s) risque(s) suivant(s) :
A - La survenue rapide d'une acido-cétose
B - Des accidents vasculaires cérébraux et coronariens
C - Le développement d'un Cushing
D - Une lithiase rénale calcique
E - La survenue d'un phéochromocytome
Bonne(s) réponse(s) : B

A - Pas dans un DNID.


D - La lithiase unique est possible sur ce terrain.

Comme prescription(s) diététique(s), vous allez conseiller dans un premier temps :


A - Un régime pauvre en glucides (moins de 100 g) pour faire baisser sa glycémie
B - Un régime de restriction hydrique pour faire baisser sa TA
C - Un régime pauvre en graisses pour faire baisser cholestérol et triglycérides
D - La suppression des boissons alcoolisées
E - Un régime hypocalorique pour le faire maigrir
Bonne(s) réponse(s) : D E

A - Le régime de l'intolérance aux hydrates de carbone ou d'un diabète et de l'hypertriglycéridémie doit écarter les glucides
simples mais les glucides complexes doivent représenter 50 % de la ration calorique.
C - Inefficace dans ce type de dyslipidémie qui dépendent du sucre et de l'alcool.

En même temps que le régime, vous allez prescrire d'emblée :


A - Du Lipanthyl®
B - Du Questran®
C - Un bêta-bloquant
D - Un sulfamide hypoglycémiant
E - Aucune médication
Bonne(s) réponse(s) : E

Pas de médicament hypolipémiants ni antidiabétiques oraux en première intention.

Mademoiselle B. consulte pour que lui soit prescrit un traitement oestroprogestatif anticonceptionnel. Elle fume 15 cigarettes
par jour. Un diabète non-insulino-dépendant a été découvert chez sa mère à 40 ans. Un bilan biologique est fait à Melle B. ,
après 12 heures de jeûne, qui montre : cholestérolémie 5 mmol/l (1.9 g/l),triglycéridémie 4mmol/l (3.5 g/l); glycémie à jeun 5,2
mmol/l, glycémie post prandiale 6 mmol/l. Taux plasmatique d'apolipoprotéine B : 1,2 g/l.
A l'examen clinique, Melle B. pèse 64 Kg pour 1 m70. L'examen clinique est normal.

Une électrophorèse des lipoprotéines est réalisée après 12 heures de jeûne. Quel en est l'aspect le plus
probable ?
A - Tracé normal
B - Hyper bêta et hyper prébêta lipoprotéinémie
C - Hyper prébêta lipoprotéinémie
D - Bloc bêta et prébêta
E - Hyper alpha lipoprotéinémie
Bonne(s) réponse(s) : C

B - Type IIB
C - Type IV
D - Type III (Broad béta)

De quelle dyslipoprotéinémie vraisemblablement s'agit-il selon la classification de Fredrickson ?


A - Type I
B - Type IIa
C - Type IIb
D - Type III
E - Type IV
Bonne(s) réponse(s) : E

Hypertriglycéridémie pure.

671
Exclusivement sur DOC - DZ : www.doc-dz.com NADJI 85
RESIDANAT EN POCHE TOME II
Cas Clinique en QCM

Le médecin décide de faire faire à la malade un ou plusieurs régimes d'inductions alimentaires. Quel en est le
but ?
A - Déterminer le nutriment provoquant l'augmentation du cholestérol
B - Déterminer le nutriment provoquant l'augmentation des triglycérides
C - Abaisser le taux des triglycérides
D - Abaisser le taux de cholestérol
E - Abaisser les taux de cholestérol et de triglycérides
Bonne(s) réponse(s) : B

Sans commentaire.

Parmi les médicaments suivants, quel est celui qui induit une baisse du taux des triglycérides plasmatiques ?
A - Cholestyramine (Questran®)
B - Fénofibrate (Lipanthyl®)
C - Probucol (Lurselle®)
D - Prednisone (Cortancyl®)
E - Piracetam (Nootropyl®)
Bonne(s) réponse(s) : B

A C - Agissent sur le cholestérol.


E - Rien à voir, il s'agit d'un psychostimulant.

Parmi les conseils suivants, quel est celui ou quels sont ceux que vous donneriez à Melle B. ?
A - Ne pas prendre de traitement oestroprogestifs
B - Traitement oestroprogestatif + Questran®
C - De ne pas grossir
D - Traitement oestroprogestatif plus Lipanthyl®
E - De prévenir ses enfants et collatéraux d'un risque de pathologie métabolique accrue
Bonne(s) réponse(s) : A C E

Sans commentaire.

Un homme de 54 ans est hospitalisé à l'occasion d'un épisode grippal avec fièvre à 38,5°C, expectoration muco-purulente,
râles bronchiques diffus sans foyer pulmonaire localisé ni à l'examen clinique ni à la radiographie.
Il s'agit d'un obèse (101 kgs pour 1,71 m) gros mangeur, grand buveur, grand fumeur (2 paquets par jour). Depuis plus d'un an
il était essoufflé à l'effort et toussait souvent. Le coeur est régulier à 90, la TA à 18/10, avec un souffle carotidien bilatéral ; les
pouls fémoraux sont perçus mais les pouls tibiaux très faibles. L'examen est par ailleurs négatif.
Les premiers examens biologiques donnent les résultats suivants :
Hémogramme : GR 5 600 000 /mm3 Hte : 16 g/dl
GB 11 000 dont polyneutro 68 %
Glycémie : 10,5 mmol/l (1,89 g/l). Créatinine 100 micromol/l
Cholestérol : 9 mmol/l (3,48 g/l) : triglycérides 4 mmol/l (3,54 g/l).
Un deuxième examen de sang confirme ces premiers résultats.

Parmi les facteurs suivants, lequel (lesquels) pouvai(en)t chez ce patient jouer un rôle dans la dyspnée d'effort
qui préexistait à l'épisode infectieux récent ?
A - Le tabagisme
B - L'obésité
C - Un diabète
D - L'éthylisme
E - L'hypertension
Bonne(s) réponse(s) : A B C D E

C - Cardiopathie ischémique liée à la macroangiopathie.


D - Myocardiopathie.

Chez ce patient, vous portez le(s) diagnostic(s) de :


A - Intolérance au glucose
B - Diabète sucré
C - Hypertriglycéridémie exogène (type I)
D - Hyperlipoprotéinémie mixte (type II b)
E - Hypertriglycéridémie mixte
Bonne(s) réponse(s) : B D

B - On dispose de deux glycémies à jeun > ou = 7,7 mmom/l, ce qui suffit au diagnostic de diabète sucré.
D - Elévation du cholestérol et des triglycérides avec CT/GT < 2,5.

672
Exclusivement sur DOC - DZ : www.doc-dz.com NADJI 85
RESIDANAT EN POCHE TOME II
Cas Clinique en QCM

Dans l'élévation des triglycérides de ce patient, on peut évoquer la responsabilité de :


A - L'obésité
B - Le diabète
C - L'éthylisme
D - L'hypertension artérielle
E - L'épisode infectieux récent
Bonne(s) réponse(s) : A B C

Sans commentaire.

Comme première(s) prescription(s) diététique(s), vous lui conseillez de :


A - Réduire l'apport de lipides pour faire baisser cholestérol et triglycérides
B - Restreindre les apports glucidiques à moins de 100 g pour faire baisser sa glycémie
C - Boire au moins deux litres par jour pour faire baisser sa créatinine
D - Supprimer les boissons alcoolisées
E - Suivre un régime hypocalorique pour perdre du poids
Bonne(s) réponse(s) : D E

B - 200 g de glucide sont conseillés en écartant les sucres simples.

En même temps et d'emblée vous allez conseiller la prise de :


A - Un sulfamide hypoglycémiant
B - Un biguanide
C - Un diurétique
D - Un hypercholestérolémiant
E - Aucune médication
Bonne(s) réponse(s) : E

Un hypolipémiant ou un antidiabétique ne sera préconisé qu'après échec du régime bien suivi.

Si, après guérison de l'épisode infectieux, ce patient conserve une insuffisance respiratoire importante, quelle(s)
médication(s) est (sont) à priori contre-indiquée(s) ?
A - Sulfamide hypoglycémiant
B - Biguanide
C - Fibrate
D - Cholestyramine
E - Diurétiques
Bonne(s) réponse(s) : B

Les biguanides sont contre-indiqués en cas d'insuffisance cardiaque, respiratoire, hépatique ou rénale.

Melle B..., âgée de 18 ans, consulte en raison d'une aménorrhée primaire.


Elle ne présente aucun antécédent particulier et a été vaccinée par le B.C.G. Sa soeur aînée a eu ses premières règles à 18
ans.
Sa taille est 1,50 m et les caractères sexuels secondaires sont incomplètement développées (Stade 3 de TANNER).
L'examen gynécologique montre une vulve et un vagin normaux, une absence de glaire cervicale, un petit utérus hypoplasique
au TR.
Différents examens complémentaires sont pratiqués dont les résultats sont les suivants :
- Taux de base en F.S.H. et L.H. faibles, avec réponse en F.S.H. supérieure à celle en L.H. au test au LH.RH
- OEstradiol : 35 pg/ml (normale pour l'âge phase folliculaire : 30 à 200 pg/ml)
- Prolactine 5 ng/ml (normale pour l'âge 5 à 20 ng/ml)
- Delta 4 androsténedione 1,2 ng/ml (normale pour l'âge 1,2 à 2,4 ng/ml)
- Testostérone 0,35 ng/ml (normale pour l'âge 0,3 à 0,9 ng/ml).

De quel type d'aménorrhée primaire s'agit-il ?


A - D'origine surrénalienne
B - D'origine hypophysaire
C - D'origine ovarienne
D - D'origine hypothalamique
E - D'origine utérine
Bonne(s) réponse(s) : D

FSH et LH faible répondent au LHRH.

673
Exclusivement sur DOC - DZ : www.doc-dz.com NADJI 85
RESIDANAT EN POCHE TOME II
Cas Clinique en QCM

Quel examen complémentaire convient-il de pratiquer pour compléter le bilan ?


A - Frottis cyto-hormonaux
B - Radiographie du poignet
C - Coelioscopie
D - Biopsie d'ovaire
E - Hystérographie
Bonne(s) réponse(s) : B

A la recherche d'un retard de l'âge osseux.

Parmi ces 5 propositions diagnostiques laquelle retenez-vous ?


A - Syndrome de Stein-Levental
B - Syndrome de Rokitansky
C - Retard pubertaire simple
D - Syndrome de Turner
E - Syndrome de Morris
Bonne(s) réponse(s) : C

Tableau évocateur avec antécédent familial probable de retard pubertaire simple, tableau clinique d'impubérisme, FSH et LH
basse avec réponse préférentielle de la FSH au LHRH.
Référence : Médecin de la reproduction par Maurais-Javis chez Flammarion.

La réponse au test au LH.RH est de quel type ?


A - Prépubertaire
B - Adulte
C - Dystrophie ovarienne scléro-kystique
D - Dysgénésie gonadique
E - Insuffisance anté-hypophysaire
Bonne(s) réponse(s) : A

Sans commentaire.

Quelle conduite proposez-vous ?


A - Résection cunéiforme des ovaires
B - Inducteur d'ovulation
C - Abstention thérapeutique
D - Traitement oestro-progestatif séquentiel
E - Oestrogénothérapie
Bonne(s) réponse(s) : C

D E - Seraient nuisibles en bloquant la croissance.

Une femme âgée de 25 ans consulte son médecin pour asthénie depuis plusieurs mois, asthénie croissante au cours de la
journée. Au cours des dernières semaines, cette asthénie s'est accentuée, associée à une anorexie et, parfois à un état
nauséeux. L'amaigrissement est chiffré à 5 kgs et l'entourage a noté que la patiente avait bronzé beaucoup plus que les étés
précédents et s'en étonne puisqu'elle n'est pas beaucoup sortie de chez elle.

Devant cette suspicion d'insuffisance surrénale, quel(s) signe(s) va (vont) vous orienter pour confirmer le
diagnostic ?
A - Cicatrices pigmentées
B - Episodes d'hypoglycémie à jeun
C - Hypotension artérielle
D - Douleurs osseuses
E - Tendance à la constipation
Bonne(s) réponse(s) : A B C E

La mélanodermie touche aussi les zones découvertes, points de frottement (coudes, genoux), les plis de flexion palmaires.
C - Orthostatique initialement qui aussi observée en décubitus par la suite.
E - Observée chez 1/5 des patients.

674
Exclusivement sur DOC - DZ : www.doc-dz.com NADJI 85
RESIDANAT EN POCHE TOME II
Cas Clinique en QCM

Quel(s) est (sont) l'(les) examen(s) dont il faut obtenir d'urgence le résultats ?
A - Cortisol plasmatique
B - Kaliémie
C - Natrémie
D - ACTH plasmatique
E - Aldostérone
Bonne(s) réponse(s) : B C

Sans commentaire.

L'insuffisance surrénale étant confirmée, quel élément clinique ou biologique affirmera l'origine primitivement
surrénalienne ?
A - Valeur de la kaliémie
B - Asthénie
C - Nausées
D - Mélanodermie
E - Hypotension orthostatique
Bonne(s) réponse(s) : D

A - Une élévation franche de la kaliémie serait cependant en faveur d'un déficit primitif car le déficit minéralocorticoïde est
absent ou modéré dans les insuffisances secondaires.
D - La mélanodermie est liée à l'augmentation de synthèse de l'ACTH et ne s'observe pas dans les déficits surrénaliens
secondaires.

Pour mieux préciser l'étiologie de cet hypocorticisme primitif, quel(s) examen(s) faut-il demander ?
A - Cliché d'abdomen sans préparation
B - Intradermo réaction à la tuberculine
C - Dosage d'ACTH
D - Mesure de l'activité rénine plasmatique
E - Anticorps anti-surrénaliens
Bonne(s) réponse(s) : A B E

A - Calcifications surrénaliennes bilatérales dans la tuberculose.


E - En faveur d'une rétraction corticale des surrénales si taux élevés.

La patiente va quitter l'hôpital. Quel traitement prescrivez-vous ?


A - 30 mg/j d'hydrocortisone en trois prises
B - 75 mg/j de fludrocortisone en une prise
C - 20 mg/j de prédnisolone (Cortancyl®)
D - Adaptation des doses en cas de stress (doubler l'hydrocortisone)
E - Régime désodé
Bonne(s) réponse(s) : A D

B - La posologie habituelle est de 25 à 100 microgrammes par jour et la dose préconisée ici est donc beaucoup trop élevée.
C - Corticothérapie inadaptée.
D - Rôle fondamental de l'éducation.
E - Régime impérativement normosalé.

Trois mois plus tard, la surveillance du traitement portera sur :


A - Le poids
B - Le dosage du cortisol à jeun
C - L'ACTH plasmatique à 8 heures
D - La pression artérielle
E - Le ionogramme sanguin
Bonne(s) réponse(s) : A D E

Les éléments de surveillance sont le poids, la pression artérielle et les inogrammes sanguins et urinaires. La persistance d'un
amaigrissement, de l'hypotension artérielle et de l'asthénie s'observe en cas de sous dosage. Le surdosage entraîne une prise
de poids trop rapide, une élévation de la tension artérielle et une hypokaliémie.
Références : Abrégé d'Endocrinologie par J. Hazard chez Masson.

675
Exclusivement sur DOC - DZ : www.doc-dz.com NADJI 85
RESIDANAT EN POCHE TOME II
Cas Clinique en QCM
Un homme de 41 ans, vient consulter sur les conseils du médecin du travail : son frère est décédé tout récemment d'un
infarctus du myocarde, son père est décédé il y a quelques années d'un accident vasculaire cérébral; lui-même sait depuis
plusieurs années qu'il a une tension artérielle un peu élevée (170/90 mm Hg) mais ne s'en est pas occupé. Il vient d'avoir une
prise de sang qui a montré un cholestérol à 9 mmol/l (3,48 g/l) et des triglycérides à 2,2 mmol/l (1,95 g/l). A l'examen : poids
81 kg - taille 1,71 m - TA 170/95 mm Hg.
L'examen est par ailleurs négatif.

Pour confirmer et préciser le diagnostic d'hyperlipoprotéinémie, vous demandez dans un premier temps :
A - Un 2ème dosage de cholestérol et triglycérides en notant l'aspect du sérum
B - Un 2ème dosage de cholestérol et triglycérides après 8 jours de régime sans graisses
C - Une immuno-électrophorèse des lipoprotéines
D - Un dosage radio-immunologique des chylomicrons et VLDL
E - Un dosage des diverses apoprotéines
Bonne(s) réponse(s) : A

Le dosage du cholestérol total et des triglycérides associé à un aspect du sérum permet en général de classer la plupart des
dyslipoprotéinémie.
C, D, E - Les examens seront à réaliser dans un deuxième temps.

Pour aider à apprécier le risque athérogène que comporte cette hyperlipoprotéinémie, il est utile de demander :
A - Un dosage du cholestérol HDL et un calcul du LDL
B - Un dosage du cholestérol VLDL et LDL
C - Un dosage des chylomicrons
D - Un dosage des apoprotéines A1 et B
E - Un dosage des apoprotéines C et E
Bonne(s) réponse(s) : A D

L'augmentation du LDL cholestérol (dont la fraction soluble est l'apo B) est athérogène. Les VLDL sont peu athérogènes.

Le risque vasculaire d'une telle hyperlipoprotéinémie est accru lorsqu'il s'y associe :
A - Hypertension artérielle
B - Tabagisme
C - Sédentarité
D - Carence en vitamine A
E - Carence en folates
Bonne(s) réponse(s) : A B C

Sans commentaire.

Il est également indiqué en présence d'un tel patient de :


A - Procéder à une enquête alimentaire
B - Rechercher d'autres cas d'hyperlipoprotéinémie dans sa famille
C - Demander une hyperglycémie provoquée
D - Pratiquer une urographie intra-veineuse
E - Doser l'uricémie
Bonne(s) réponse(s) : A B C E

C, E - Trouble de la tolérance glucidique et hyperuricémie sont souvent présent sur ce terrain.

Dans un premier temps vos prescriptions diététiques seront :


A - Régime hypocalorique pour le faire maigrir
B - Réduire l'ensemble des graisses de l'alimentation
C - Remplacer des graisses animales au profit de graisses végétales insaturées
D - Remplacer les protéines animales par des protéines végétales
E - Supprimer l'alcool
Bonne(s) réponse(s) : A C E

B - Ne peut pas être préconisé en l'absence d'une enquête alimentaire précisant la part des lipides dans la ration calorique
globale et le niveau de cette dernière.

676
Exclusivement sur DOC - DZ : www.doc-dz.com NADJI 85
RESIDANAT EN POCHE TOME II
Cas Clinique en QCM

Votre thérapeutique de 1ère intention comportera en outre la (les) mesure(s) suivantes :


A - Traiter l'hypertension artérielle
B - Prescrire un anorexigène
C - Prescrire de l'acide nicotinique
D - Supprimer le tabac
E - Conseiller un exercice physique progressif et adapté
Bonne(s) réponse(s) : A D E

B - Surtout pas.
D - Le tabac est un facteur de risque majeur de l'athérome. Se méfier cependant d'une prise de poids à l'arrêt du tabac.

Un homme de 40 ans consulte pour asthénie ; l'examen clinique montre un poids stable de 51 kg pour 162 cm ; la pression
artérielle est de 130/80 mmHg. Le foie est hypertrophié avec une flèche hépatique de 13 cm sur la ligne médioclaviculaire. Le
reste de l'examen est normal. L'interrogatoire retrouve la notion d'une consommation journalière de 2,0 l de vin rouge à 10
degrés.
L'enquête alimentaire note la prise de 2200 kilocalories/jour (sans l'alcool) dont 13 % de protéines, 44 % de lipides et 43 % de
glucides (12 % de saccharose). Les examens sanguins donnent les résultats suivants : glucose : 7 mmol/l, sodium : 129
mmol/l, triglycérides : 70 mmol/l, cholestérol : 17 mmol/l, gamma GT : 1491 U/l (N inférieure à 40), ASAT : 92 U/l (N inférieure
à 40), ALAT : 66 U/l ( N inférieure à 50).

La consommation d'alcool de ce patient correspond en grammes d'alcool pur à :


A - 80
B - 100
C - 120
D - 160
E - 180
Bonne(s) réponse(s) : D

Un litre de vin rouge à 10° contient 80 grammes d'alcool.

Selon la classification internationale, l'hyperlipoprotéinémie de ce patient est probablement de type :


A-I
B - IIb
C- III
D - IV
E-V
Bonne(s) réponse(s) : D

Dyslipidémie avec élévation franche des triglycérides et moindre du cholestérol. TG/CT supérieur à 2,5 mais inférieur à 10
(sinon on aurait pu envisager un type V). Le contexte éthylique est habituellement retrouvé dans ce type de dyslipidémie.

L'hyponatrémie de ce patient est liée à :


A - Des apports sodés insuffisants
B - Une rétention hydrique
C - Une rétention hydrosodée
D - L'hypertriglycéridémie
E - Des pertes sodées digestives
Bonne(s) réponse(s) : D

"Fausse" hyponatrémie due à la présence en grande quantité de triglycérides. Ceci s'observe aussi dans les grandes
hyperglycémies, les perfusions de mannitol, le myélome à forte concentration de protides plasmatiques.

La correction de l'hypertriglycéridémie de ce patient peut être obtenue par :


A - La suppression de l'alcool
B - La suppression des glucides d'absorption lente
C - La suppression des graisses saturées
D - L'apport de graisses polyinsaturées
E - La prescription d'un médicament hypolipémiant
Bonne(s) réponse(s) : A E

A - Mesure essentielle chez ce patient.


B - Ce sont les glucides simples qu'il faut supprimer dans la dyslipidémie type IV.
E - Les dérivés du clofibrate.
C, D - La réduction des apports lipidiques est peu efficace dans ce type de dyslipidémie. L'apport lipidique doit cependant être
équilibré.

677
Exclusivement sur DOC - DZ : www.doc-dz.com NADJI 85
RESIDANAT EN POCHE TOME II
Cas Clinique en QCM

Le risque (les risques) que peut (peuvent) faire courir à ce patient la consommation excessive d'alcool est (sont)
:
A - Cancer de l'oesophage
B - Cancer des voies aero-digestives supérieures
C - Cancer de la vessie
D - Pancréatite
E - Cardiomyopathie
Bonne(s) réponse(s) : A B D E

Sans commentaire.

Une femme de 61 ans vient consulter pour asthénie et se plaint en outre de constipation et de frilosité. Elle est plus lente, plus
maladroite.
Sa peau est sèche, un peu infiltrée, le coeur est régulier à 60, la TA à 15/8. L'examen clinique est par ailleurs négatif. Un tel
ensemble évoque fortement une hypothyroïdie.

Parmi les propositions suivantes concernant l'hypothyroïdie de l'adulte, laquelle (lesquelles) est (sont) exacte(s)
?
A - Elle s'accompagne souvent d'une réaction myotonique avec lenteur de la décontraction
B - Elle peut se révéler par une anémie
C - L'athérome, surtout coronarien, est fréquent
D - L'hypothyroïdie peut aboutir au coma
E - L'étiologie auto-immune est fréquente
Bonne(s) réponse(s) : A B C D E

A - Explique l'allongement du temps de déconcentration sur le réflexogramme achilléen.


E - Thyroïdite de Hashimoto ou traitement par iode ou chirurgie d'une maladie de Basedow.

Parmi les propositions suivantes concernant le diagnostic de l'hypothyroïdie primitive, lesquelles sont exactes ?
A - L'effondrement de la fixation thyroïdienne suffit à affirmer le diagnostic
B - Un taux plasmatique de T3 abaissé suffit à affirmer le diagnostic
C - Il n'y a pas d'hypothyroïdie primitive sans élévation de la TSH
D - La prolactine peut être élevée
E - La FSH est basse et s'élève sous métopirone
Bonne(s) réponse(s) : D

Peut s'observer dans de nombreuses pathologies (syndrome de base T3) en l'absence d'hypothyroïde véritable. De plus un
taux abaissé de T3 peut se voir dans les hypothyroïdes centrales comme périphériques.
C - L'élévation de la TSH est très précoce mais peut manquer en cas de dysfonctionnement hypothalamo-hypophysaire
associé.
D - Effet stimulant du TRH sur la sécrétion de prolactine.

Parmi les situations suivantes, laquelle (lesquelles) est (sont) une cause classique d'hypothyroïdie ?
A - Thyroïdite chronique auto-immune (maladie de Hashimoto)
B - Thyroïdite sub-aiguë type de Quervain
C - Maladie de Basedow traitée par iode radio-actif
D - Traitement par amiodarone (Cordarone)
E - Epithélioma thyroïdien à stroma amyloïde
Bonne(s) réponse(s) : A B C D

Sans commentaire.

Pour commencer le traitement d'une telle patiente, quel choix faites-vous parmi les propositions suivantes ?
A - De la L-Thyroxine par voie IV
B - De la L-Thyroxine per os à la dose de 12,5 micro-g/jour
C - De la L-Thyroxine per os à la dose de 100 micro-g/jour
D - De la T3 à la dose de 75 micro-g/jour
E - Du Lugol à la dose de 30 gouttes par jour
Bonne(s) réponse(s) : B

Compte-tenu de l'âge de la patiente et du risque cardiaque que représente le traitement substitutif on débutera à une faible
posologie à augmenter progressivement par palier de 10 à 15 jours.

678
Exclusivement sur DOC - DZ : www.doc-dz.com NADJI 85
RESIDANAT EN POCHE TOME II
Cas Clinique en QCM

Au cours des quatre semaines suivantes, il faut :


A - Surveiller deux fois par jour la fréquence cardiaque et la TA
B - Augmenter tous les 2 ou 3 jours la dose thérapeutique
C - Augmenter la dose thérapeutique par paliers d'au moins 8 à 10 jours
D - Prévoir systématiquement un électrocardiogramme
E - Doser la TSH et le cholestérol tous les 15 jours
Bonne(s) réponse(s) : A

B - La persistance d'une TSH élevée signe un sous-dosage.

Après un certain temps, pour dire que la thérapeutique supplétive est bien adaptée, il faut obtenir :
A - Une normalisation des taux plasmatiques de T3, T4, TSH
B - Une normalisation des taux plasmatiques de T3 et T4, la TSH restant élevée
C - Un effondrement du taux plasmatique de thyroglobuline
D - Le retour à une fixation thyroïdienne normale de l'iode radio-actif
E - Une image scintigraphique normale
Bonne(s) réponse(s) : A

B - La persistance d'une TSH élevée signe un sous-dosage.

Une femme âgée de 75 ans est amenée au service d'urgence de l'hôpital, dans un état de coma avec agitation.
Elle-même présente depuis cinq jours une hyperthermie importante.
Elle est diabétique depuis quelque années et est traitée par une association de sulfamide hypoglycémiant et de biguanide.
La peau et la bouche sont sèches, le globe oculaire facilement dépressible.
Le poids est de 48 kg.
Ionogramme sanguin : Na = 165 mmol/l, K = 5,2 mmol/l, CI = 118 mmol/l, bicarbonates = 35 mmol/l, la glycémie est à 1.45 g/l
(normale 0,9) la créatininémie à 220 micromol/l. Il n'est pas trouvé de corps cétoniques.

Vous placez un cathéter pour mesurer la pression veineuse centrale (PVC) et la pression artérielle (PA).
Pensez-vous que les chiffres montreront :
A - PA et PVC élevées
B - PA abaissée et PVC non élevée
C - PA abaissée, PVC élevée
D - PA élevée, PVC abaissée
E - PA et PVC normales
Bonne(s) réponse(s) : B

Tableau clinique et biologique de coma hyperosmolaire ou déshydratation globale.

Vous allez administrer immédiatement :


A - 120 unités d'insuline intraveineux
B - 500 ml de sérum bicarbonaté hypertonique
C - 500 ml de mannitol
D - 2 litres de soluté glucosé isotonique ou hypotonique
E - 1 litre de mélange de 500 ml de sérum isotonique (CINa) avec 500 ml de sérum bicarbonaté hypertonique
Bonne(s) réponse(s) : D

Le traitement repose sur la réhydratation qui doit être faite initialement avec des solutés hypotoniques (glucosé à 25 g/l puis
glucosé à 50 g/l).

Quel est selon vous le déficit hydrique calculé avec les notions chiffrées fournies :
A - 15 litres
B - 12 litres
C - 7 litres
D - 3 litres
E - 2 litres
Bonne(s) réponse(s) : C

C - Calculé à partir de la formule :


V = 0,6 x poids x (f(Na;140) - 1), le déficit hydrique (V) est ici de 5,4 l. Il se situe donc entre 3 et 7 litre mais la réponse la plus
proche est 7 litres.

679
Exclusivement sur DOC - DZ : www.doc-dz.com NADJI 85
RESIDANAT EN POCHE TOME II
Cas Clinique en QCM

Parmi les signes suivants, la déshydratation globale peut être traduite par :
A - Irritabilité, hallucinations
B - Somnolence
C - Hyperthermie
D - Hématome sous-dural
E - Oedème rétinien
Bonne(s) réponse(s) : A C D

B - Signe d'hyperhydratation cellulaire.


D - Surtout chez le nourrisson.

Un homme de 65 ans est hospitalisé pour dyspnée récente et troubles confusionnels. Sont également cliniquement notés :
- des antécédents de bronchite chronique sur un tagagisme ancien,
- une HTA évoluant depuis 5 ans et traitée par Modurétic® et Catapressan®,
- la T.A. est à 130/85, la T° à 37 degrés,
Le reste de l'examen clinique est normal.
Biologiquement :
Hémoglobine : 11,8 g/100 ml - Hématocrite 37 % - G.B. : 9800
Dans les urines : Prot. : 0 - Glycosurie : 0 - densité : 1020 -
Osmolalité : 496 mOsm/kg
Dans le sang : Urée : 0,20 g/l - Créat. : 10 mg/l - Uricémie : 30 mg/l -
Protidémie : 70 g/l - Cholestérol : 2,6 g/l -
Natrémie : 115 mEq/l - Chlore : 80 mEq/l - Kaliémie : 4,5 mEq/l
Osmolalite plasmatique : 230 mOsm/kg -
Cortisolémie normale - bicar. : 28 mEq/l.
La radiographie du thorax montre une atélectasie lobaire supérieure gauche avec épanchement pleural du même côté.

Dans quelle(s) circonstance(s) parmi les suivantes peut on rencontrer une hyponatrémie ?
A - Hyperlipidémie
B - Syndrome de Conn
C - Glycémie élevée
D - Hypercalcémie
E - Traitement par thiazides
Bonne(s) réponse(s) : A C E

A - Fausse hyponatrémie liée à la présence en excès d'une substance occupant un grand volume plasmatique.
C - Fausse hyponatrémie liée à la présence en excès d'une substance osmotiquement active.
E - Hyponatrémie de déplétion.

Quel est le diagnostic le plus vraisemblable chez ce patient ?


A - Syndrome de sécrétion inappropriée d'A.D.H.
B - Insuffisance surrénale aiguë
C - Néphropathie avec perte de sel
D - Hyperhydratation par insuffisance cardiaque
E - Cirrhose hépatique
Bonne(s) réponse(s) : A

Tableau très évocateur avec hypo-osmolarité plasmatique et osmolarité urinaire élevée. Pathologie pulmonaire.

Dans un syndrome de Schwartz-Bartter (hypersécrétion inappropriée d'ADH) (SIADH) on note


typiquement :
A - Une hypervolémie
B - Une perte potassique urinaire
C - Un hyperaldostéronisme
D - Une osmolalité urinaire basse
E - Une hypoglycémie
Bonne(s) réponse(s) : A

C - Aldostérone et rénine ne sont pas modifiées.


D - Au contraire, il est retrouvé une hyperosmolarité urinaire avec clearance de l'eau libre négative.
Référence : Traité de Médecine par P. Godeau chez Flammarion.

680
Exclusivement sur DOC - DZ : www.doc-dz.com NADJI 85
RESIDANAT EN POCHE TOME II
Cas Clinique en QCM

L'osmolalité urinaire chez ce patient vous paraît ?


A - Adaptée à son hyponatrémie
B - Anormalement élevée
C - Affirmer un mécanisme de diurèse osmotique
D - Concordante avec le diagnostic de SIADH(syndrome de sécrétion inappropriée d'ADH)
E - Aucune des propositions précédentes n'est exacte
Bonne(s) réponse(s) : B D

Sans commentaire.

Parmi ces traitements lequel vous parait nécessaire dans tous les cas ?
A - Furosémide
B - Thiazidiques
C - Restriction hydrique
D - Perfusion de sérum salé hypertonique
E - Lithium
Bonne(s) réponse(s) : C

E - Peut être parfois proposé pour son action inhibitrice de l'ADH sur le tube contourné distal.

Un homme de 28 ans est amené à l'hôpital pour parésie des membres inférieurs.
L'examen neurologique est normal en dehors d'un certain degré d'hypotonie musculaire. Le reste de l'examen clinique est
normal. Examens biologiques : sang : urée = 6 mmol/l, créatinine = 130 micromol/l, ionogramme (mmol/l) : Na = 137, K = 1,9,
CI = 116, bicarbonates = 14. Protides totaux : 80 g/l, hématocrite : 52 %, Osmolarité = 295 mosmol/l. Gaz du sang artériel : pH
= 7,23 PC02 = 36 mmHg P02 = 85 mmHg. Glycémie = 5 mmol/l, Corps cétoniques = 0.
Urines (sur échantillon) : Na = 100 mmol/l, K = 55 mmol/l, CI = 70 mmol/l, pH (sous huile) = 7,2 osmolarité = 900 mosmol/l.

Ce malade est atteint d'un désordre de l'équilibre acido-basique, lequel ?


A - Une acidose métabolique compensée
B - Une alcalose respiratoire pure
C - Une acidose métabolique non compensée
D - Une alcalose mixte, métabolique et respiratoire
E - Une acidose métabolique surcompensée par une alcalose respiratoire
Bonne(s) réponse(s) : C

Sans commentaire.

La confrontation des examens sanguins et urinaires montre que la réponse rénale est inappropriée en ce qui
concerne :
A - Le potassium
B - Le chlore
C - Le pH
D - L'osmolarité
E - Aucune des propositions ci-dessus
Bonne(s) réponse(s) : A B C

A - La kaliurie devrait être nulle.

La baisse des bicarbonates sanguins observée chez ce patient ne peut être due qu'à un mécanisme. Lequel ?
A - Une diarrhée abondante
B - La compensation rénale d'une hyperventilation
C - Une acidose tubulaire proximale
D - Une acidose tubulaire distale
E - Une acidose par insuffisance rénale globale
Bonne(s) réponse(s) : D

Acidose métabolique sans trou urinaire en faveur d'une fuite de bicarbonate tubulaire. Dans l'acidose tubulaire proximale, le
pH urinaire devient inférieur à 5,8 dès que les bicarbonates plasmatiques sont inférieurs au seuil de réabsorption tubulaire. Ce
patient garde un pH urinaire élevé malgré des bicarbonates plasmatiques effondrés à 14 mmom/l, ce qui est en faveur d'un
acidose tubulaire distale.

681
Exclusivement sur DOC - DZ : www.doc-dz.com NADJI 85
RESIDANAT EN POCHE TOME II
Cas Clinique en QCM

Compte tenu des résultats du ionogramme sanguin il peut être dangereux de prescrire :
A - Soluté glucosé à 50 g par litre
B - KCl IV 2 g en 8 heures
C - Soluté de CI Na iso ou hypertonique exclusivement
D - Régime normo sodé
E - Apport hydrique libre
Bonne(s) réponse(s) : B C

L'apport de chlorure de potassium est urgent. La dose de 2 g en 8 h (réponse B) est insuffisante et la réponse C exclu le
potassium. Il est possible de perfuser jusqu'à 20 mmol/h (1,5 g) de chlorure de potassium à la concentration de 75 mmol/l (5,7
g).
Référence : Guide pratique des médicaments par Pb. Doroz chez Maloine.

Une femme de 58 ans, ménopausée depuis 4 ans, vient vous consulter.


Elle est frileuse, a pris 5 ou 6 kgs depuis quelques mois.

Parmi les autres symptômes suivants, dont elle se plaint, lequel (lesquels) fait (font) évoquer la possibilité d'une
insuffisance thyroïdienne ?
A - Lenteur des gestes
B - Fourmillements et quelques crampes
C - Asthénie
D - Céphalées
E - Tremblement
Bonne(s) réponse(s) : A B C

B - Si topographie évocatrice rechercher un symtôme de canal carpien bilatéral.

Parmi les constatations suivantes faites à l'examen clinique, laquelle (lesquelles) fait (font) évoquer le diagnostic
d'insuffisance thyroïdienne ?
A - Peau sèche
B - Infiltration cutanée
C - Réflexes tendineux très vifs
D - Bradycardie
E - Hypertension artérielle à 19/10
Bonne(s) réponse(s) : A B D

Sans commentaire.

Quel(s) examen(s) demandez-vous dans un premier temps pour affirmer le diagnostic d'hypothyroïdie primitive ?
A - Dosage de T3 - T4 - TSH
B - Dosage de thyroglobuline
C - Test au TRH
D - Test à la métopyrone
E - Recherche d'anticorps antithyroïdiens
Bonne(s) réponse(s) : A

C - Le test au TRH est interressant dans les déficits thyréotropes.

Parmi les résultats biologiques ci-dessous de cette patiente, lequel (lesquels) est (sont) inattendus et doit
(doivent) faire rechercher une pathologie associée ?
A - Anémie légèrement macrocytaire avec 11 g d'hémoglobine/dl
B - Calcémie à 2,9 mmol/l ( 116 mg/l)
C - Cholestérol 8 mmol/l (3,09 g/l)
D - FSH supérieure à 30 microg/l
E - Prolactine 30 ng/ml
Bonne(s) réponse(s) : A B

A - L'anémie est habituellement micro ou macrocytaire. Son caractère macrocytaire doit faire rechercher une maladie de
Biermer associée.
B - L'hypercalcémie s'observe dans l'hyperthyroïdie mais pas l'hypothyroïdie.
D - Taux correspondant à la ménopause.
E - Le TRH est élevé dans l'hypothyroïdie basse et son action stimulante de la sécrétion de prolactine peut entraîner une
hyperprolactinémie modérée (normale chez la femme < ou = 20 ng/ml).

682
Exclusivement sur DOC - DZ : www.doc-dz.com NADJI 85
RESIDANAT EN POCHE TOME II
Cas Clinique en QCM

Pour traiter cette hypothyroïdie, vous allez prescrire :


A - Thyroxine
B - Lugol
C - Néomercazole
D - Lithium
E - Aldostérone
Bonne(s) réponse(s) : A

Sans commentaire.

Vous apprenez que cette malade a été traitée il y a 8 ans et pendant 18 mois par Néomercazole pour une
maladie de Basedow avec goitre et exophtalmie. Cette notion oriente vers quelle étiologie de l'hypothyroïdie
actuelle ?
A - Une agénésie thyroïdienne
B - Un effet prolongé de l'antithyroïdien de synthèse
C - Une ectopie thyroïdienne
D - Une thyroïdite subaiguë type de Quervain
E - Une maladie de Hashimoto
Bonne(s) réponse(s) : E

Basedow et thyroïdite de Hashimoto sont deux pathologies auto-immunes parfois difficiles à différencier.

683
Exclusivement sur DOC - DZ : www.doc-dz.com NADJI 85
RESIDANAT EN POCHE TOME II
Cas Clinique en QCM

684
Exclusivement sur DOC - DZ : www.doc-dz.com NADJI 85
RESIDANAT EN POCHE TOME II
Cas Clinique en QCM

685
Exclusivement sur DOC - DZ : www.doc-dz.com NADJI 85
RESIDANAT EN POCHE TOME II
Cas Clinique en QCM
Une femme de 34 ans, d'origine corse par son père, consulte pour asthénie et anémie. A l'examen, pâleur, pas d'autre
anomalie, en particulier pas de splénomégalie. A l'interrogatoire, pas de prise médicamenteuse, deux grossesses avec des
enfants de 6 ans et 3 ans bien portants. Dans la famille, deux soeurs bien portantes, père et mère bien portants.
Hémogramme :
- Hémoglobine : 9 g/dl
- VGM : 77 microns 3
- CCMH : 33 %
- Globules rouges : 4 200 000/mm3
- Réticulocytes : 1%
- Globules blancs normaux
- Plaquettes : 285 000
- VS : 20-40

Parmi les descriptions suivantes, quelle est celle qui convient pour l'anomalie de l'hémogramme présentée par
cette femme ?
A - Anémie normochrome normocytaire arégenérative
B - Anémie microcytaire normochrome arégénérative
C - Anémie microcytaire hypochrome arégénérative
D - Anémie microcytaire normochrome régénérative
E - Anémie macrocytaire normochrome arégenérative
Bonne(s) réponse(s) : B

VGM = Ht/Gr = 77 micron3.


CCMH = Hb/Ht = 0,33.
Réticulocytes = 42 000 < 120 000 donc absence de régénération.

Quel(s) est (sont) le(s) mécanisme(s) possible(s) de cette anémie ?


A - Défaut de synthèse de l'hème
B - Défaut de synthèse de l'ADN
C - Défaut de synthèse de la globine
D - Erythroblastopénie
E - Hémorragie aiguë
Bonne(s) réponse(s) : A C

A - Microcytose par ralentissement de production de l'Hb et CCMH atteint une mitose plus tard (permet la libération de
l'hématie).
B - Macrocytose.
C - Microcytose idem A.
DE - Ecartés devant l'absence de réticulocytose.

Quel est parmi les examens suivants, celui que vous choisissez de réaliser en premier pour trancher entre les
hypothèses physiopathologiques que vous avez retenues ?
A - Recherche biochimique de sang dans les selles
B - Dosage du fer sérique
C - Electrophorèse de l'hémoglobine
D - Myélogramme
E - T.O.G.D.
Bonne(s) réponse(s) : B

La carence en fer entraîne un défaut de synthèse de l'hémoglobine.

Quelle est, d'après les données de l'observation, l'étiologie qui vous paraît la plus vraisemblable parmi les
suivantes :
A - Drépanocytose hétérozygote (HBAS)
B - Carence en acide folique
C - Carence martiale
D - Autoimmunisation
E - Inflammation
Bonne(s) réponse(s) : C

A - Anémie normo ou macrocytaire.


B - Anémie macrocytaire.
D - Anémie normo ou macrocytaire (régénération).
E - VS élevée normo puis macrocytaire.

686
Exclusivement sur DOC - DZ : www.doc-dz.com NADJI 85
RESIDANAT EN POCHE TOME II
Cas Clinique en QCM

En attendant les résultats des examens complémentaires, quelle thérapeutique d'urgence vous paraît justifiée
par l'état de cette patiente ?
A - Transfusion de deux culots globulaires
B - Fer intramusculaire, 100 mg/jour
C - Acide folique 5 mg par jour
D - Chélateur du fer (Desferal® 100mg IM un jour sur deux)
E - Aucune
Bonne(s) réponse(s) : E

Il n'y a pas de signe de mauvaise tolérance de l'anémie, le traitement martial per os n'est pas
urgent et sera démarré après enquête étiologique complète.

Un homme de 64 ans, sans antécédent particulier, vient consulter pour une asthénie avec dyspnée d'effort. L'examen clinique
est normal. L'hémogramme donne :
- Hb : 8.9 g/dl
- CCMH : 33%
- VGM : 108 fl
- GB : 2 x 109/l
- Plaquettes : 130 x 109/l
- Formule leucocytaire : polyneutrophile 50 % ; lymphocytes 40 % ; monocytes 10 % ; réticulocytes : 24 x 109/l
- VS : 35-60.

Parmi les propositions suivantes concernant cet hémogramme, laquelle est la plus exacte ?
A - Anémie normochrome normocytaire régénérative, neutropénie, thrombopénie
B - Anémie normochrome macrocytaire arégénérative, neutropénie, thrombopénie
C - Anémie hypochrome macrocytaire arégénérative, neutropénie, thrombopénie
D - Anémie normochrome microcytaire arégénérative, neutropénie
E - Anémie normochrome macrocytaire arégénérative, thrombopénie
Bonne(s) réponse(s) : B

- VGM = Ht/Gr = 108 µ3.


- CCMH = Hb/Ht = 0,33..
- PN = 1000 donc neutropénie.

Quel est le mécanisme le plus probable de l'anémie ?


A - Défaut de synthèse de l'hème
B - Défaut de synthèse de l'ADN
C - Défaut de synthèse de la globine
D - Défaut de synthèse de la spectrine
E - Défaut de synthèse de la G 6 P D
Bonne(s) réponse(s) : B

La macrocytose est la conséquence d'un asynchronisme de synthèse hémoglobine/ADN.

Parmi les examens suivants, lequel doit être exécuté en premier pour orienter le diagnostic ?
A - Myélogramme
B - Dosage du fer sérique
C - Electrophorèse de l'hémoglobine
D - Dosage de la bilirubine
E - Dosage de vitamine B12 et de l'acide folique
Bonne(s) réponse(s) : A

Permet d'apprécier la richesse médullaire et la qualité des constituants.

Parmi les maladies suivantes la(lesquelles) est (sont) compatible(s) avec ce tableau clinique et biologique ?
A - Anémie inflammatoire
B - Anémie réfractaire (dysmyélopoïèse primitive)
C - Anémie par carence martiale
D - Anémie hémolytique constitutionnelle
E - Anémie par carence en vitamine B12
Bonne(s) réponse(s) : B E

A - Anémie normo ou macrocytaire.


C - Anémie microcytaire.
D - Anémie normo ou macrocytaire sans neutropénie.

687
Exclusivement sur DOC - DZ : www.doc-dz.com NADJI 85
RESIDANAT EN POCHE TOME II
Cas Clinique en QCM
Un homme de 45 ans, dont le poids normal est de 60 kg, présente des adénopathies cervicales bilatérales, un
amaigrissement de 7 kg en 6 mois, l'absence de fièvre, pas de sueurs nocturnes. L'examen permet de percevoir une rate
débordant de 4 cm le rebord costal. Pas d'hépatomégalie, pas d'adénopathie dans les autres territoires accessibles à
l'examen clinique. Le bilan objective : VS supérieure à 100 à la première heure, fibrinémie à 9 g/l, leucopénie à 3.000 globules
blancs, pas de cellule anormale dans le sang, radiographie pulmonaire normale.

Parmi les diagnostics suivants, le plus probable est :


A - Leucose lymphoïde chronique aleucémique
B - Maladie de Hodgkin
C - Lymphome non Hodgkinien
D - Leucose myéloïde chronique
E - Aucune des affections précédentes
Bonne(s) réponse(s) : B

A - Peu évocateur en raison de l'âge, de l'importance de l'altération de l'état général, de l'absence de Iymphocytose.
C - Probable, découverte le plus souvent devant des adénopathies profondes thoraciques ou abdominales, ou une localisation
ORL.
D - Pas d'adénopathies périphériques (sauf si acutisation), hémogramme peu évocateur.

Biopsie ganglionnaire exceptée, l'examen le plus important non cité jusqu'alors est :
A - Lymphographie
B - Tomographies du médiastin
C - Médullogramme
D - Ponction-Biopsie du foie
E - Scintigraphie splénique
Bonne(s) réponse(s) : A

A - Permet de stadifier la maladie sur le plan topographique et de suivre l évolution de façon simple (cliché de contrôle).
B - Inutile si radiographie de thorax normale.
C - Permet d'apprécier l'envahissement médullaire et le mode d'extension topographique.
D - Uniquement si hépatomégalie ou anomalies du bilan hépatique.
E - Sans intérêt.

La laparotomie exploratrice n'est pas justifiée si :


A - Le médiastin est indemne
B - La biopsie de crête iliaque est positive
C - La scintigraphie splénique est négative
D - On prévoyait une irradiation couvrant le volume splénique
E - La biologie hépatique est perturbée
Bonne(s) réponse(s) : B

L'envahissement médullaire impose une chimiothérapie.

La lymphographie était négative, et la laparotomie a objectivé une rate de 400 grammes, des ganglions du hile
splénique et du pédicule hépatique envahis, un foie et une moelle osseuse histologiquement normaux. Le stade
pathologique (PS) de ce patient est :
A - PS IIE
B - PS IIB
C - PS IIIA
D - PS IIIB
E - PS IVB
Bonne(s) réponse(s) : D

Atteinte ganglionnaire de part et d'autre du diaphragme et présence de signes d'évolutivité


clinique (amaigrissement de 10 % du poids en 6 mois).

688
Exclusivement sur DOC - DZ : www.doc-dz.com NADJI 85
RESIDANAT EN POCHE TOME II
Cas Clinique en QCM
Une femme de 35 ans, sans antécédent particulier, présente un purpura des membres inférieurs apparu brutalement depuis
une semaine, sans fièvre. A l'examen, les lésions ne s'effacent pas à la vitropression ; certains éléments sont palpables,
papuleux ; d'autres sont bulleux ; d'autres enfin nécrotiques, et on retrouve à distance, sur les membres supérieurs et
l'abdomen, quelques rares éléments purpuriques de petite taille.

Parmi ces éléments cliniques, le(s) quel(s) est(sont) en faveur de l'origine vascularitique de ce purpura ?
A - L'absence d'effacement à la vitropression
B - La présence d'éléments papuleux
C - La présence d'éléments bulleux
D - L'existence d'éléments à distance
E - L'absence de fièvre
Bonne(s) réponse(s) : B C

A - Définition du purpura.
D.E - En faveur d'une autre cause (par exemple : thrombopénie).

Parmi ces examens complémentaires, le(s) quel(s) par leur modification peut(peuvent) vous apporter un
élément diagnostique en faveur du purpura vascularitique ?
A - Bilan de coagulation
B - Taux de plaquettes
C - NFS
D - Biopsie cutanée
E - Test du lacet
Bonne(s) réponse(s) : D

A - Doit être normal en ce qui concerne l'hémostase primaire.


B - Normal dans les purpuras vasculaires.
D - Permet la caractérisation histologique des lésions des petits vaisseaux.

Quel(s) autre(s) examen(s) complémentaire(s) vous parai(ssen)t important(s) à pratiquer chez cette jeune
femme ?
A - Radiographie pulmonaire
B - Bilan hépatique
C - Bilan rénal
D - Test de transformation lymphoblasique à la PHA
E - Dosage des IgE
Bonne(s) réponse(s) : C

Facteur pronostique de la plupart des vascularites. Le degré d'atteinte est apprécié à l'histologie (PBR).

Quel(s) diagnostic(s) peut-on évoquer devant le tableau présenté par cette patiente :
A - Purpura thrombopénique idiopathique
B - Cryoglobulinémie
C - Toxidermie
D - Insuffisance veineuse
E - Hémopathie
Bonne(s) réponse(s) : B C

Les autres causes ne sont pas des vascularites.

689
Exclusivement sur DOC - DZ : www.doc-dz.com NADJI 85
RESIDANAT EN POCHE TOME II
Cas Clinique en QCM
Un homme de 68 ans, sans antécédent particulier, consulte pour une asthénie modérée. L'examen clinique révèle l'existence
d'une splénomégalie non douloureuse, débordant le rebord costal de 4 cm en inspiration profonde, et d'adénopathies
cervicales, axillaires et inguinales, symétriques, faites de ganglions d'environ 2 cm de diamètre, sans périadénite. Il n'existe ni
signes inflammatoires, ni compression. La température est normale et le reste de l'examen est normal.
L'hémogramme donne les résultats suivants :
- Hématies : 4,4 x 1012/l
- Hémoglobine : 13 g/dl
- Hématocrite : 0, 38 l/l
- Réticulocytes : 1 %
- Leucocytes : 50 x 10 exposant 9/l : polynucléaires neutrophiles : 8 %
Iymphocytes : 91 %
monocytes : 1 %
- Plaquettes : 235 x 10 exposant 9/ l
- Vitesse de sédimentation : 1ère heure 6 mm, 2ème heure 10 mm.

L'analyse de l'hémogramme reconnaît :


A - Une anémie
B - Une hyperleucocytose
C - Une neutropénie
D - Une hyperlymphocytose
E - Une monocytose
Bonne(s) réponse(s) : B D

Sans commentaire, évident.

Parmi les maladies suivantes laquelle évoquez-vous ?


A - Maladie de Hodgkin
B - Mononucléose infectieuse
C - Leucémie aiguë
D - Leucémie lymphoïde chronique
E - Agranulocytose
Bonne(s) réponse(s) : D

L'âge du patient, l'existence d'une altération modérée de l'état général, d'une splénomégalie et de polyadénopathies orientent
vers ce diagnostic.

Quelle anomalie peut-on s'attendre à observer au myélogramme chez ce patient ?


A - Une érythroblastopénie
B - Une Iymphocytose
C - Une plasmocytose
D - Un blocage de maturation myélocytaire
E - Des cellules anormales
Bonne(s) réponse(s) : B

A - Pas au stade de début.


C - Myélome.
D - Leucémie myélocytaire.
E - Métastases médullaires, leucémies aiguës.

Parmi les examens complémentaires suivants, quel(s) est(sont) celui(ceux) qui doi(ven)t être effectué(s) de
façon systématique ?
A - Réaction de Paul et Bunnel
B - Dosage des protéines sériques et électrophorèse
C - Temps de céphaline kaolin
D - Test de Coombs direct
E - Biopsie ganglionnaire
Bonne(s) réponse(s) : B D

A - Sans rapport.
B - Recherche d'une gammapathie monoclonale ou hypogammoglobulinémie.
C - Sans rapport.
D - L'anémie hémolytique auto-immune est une complication classique.
E - Sans intérêt.

690
Exclusivement sur DOC - DZ : www.doc-dz.com NADJI 85
RESIDANAT EN POCHE TOME II
Cas Clinique en QCM

Parmi les affections suivantes, quelle(s) est(sont) celle(s) que l'on rencontre plus particulièrement au cours de
l'évolution ?
A - Hémorragie cérébro-méningée
B - Insuffisance myéloïde aiguë
C - Zona
D - Paraplégie spasmodique
E - Anémie hémolytique auto-immune
Bonne(s) réponse(s) : C E

Caractérisent toutes deux la dysimmunité cellulaire et humorale.

Une femme de 25 ans consulte pour maladie de Hodgkin. Le diagnostic a été porté sur l'examen histologique d'un ganglion
sus-claviculaire droit apparu 3 mois plus tôt :
"Disparition de l'architecture normale du ganglion. Population cellulaire en majorité lymphocytaire, avec présence de
polynucléaires neutrophiles et éosinophiles, de plasmocytes, d'histiocytes et d'assez nombreuses cellules de Sternberg.
Absence de fibrose".
On apprend qu'elle a des sueurs nocturnes abondantes, elle a maigri de 2 kg en 1 mois. L'examen clinique montre des
ganglions sus-claviculaires bilatéraux et une splénomégalie débordant de 5 cm. Sur ces données, la maladie est classée au
stade anatomoclinique III B.

Quel est le type histologique de cette maladie de Hodgkin ?


A - A prédominance lymphocytaire
B - Cellulaire mixte
C - Sclérose nodulaire
D - Réflexion lymphocytaire
E - Inclassable
Bonne(s) réponse(s) : B

A - Cellules de Steinberg rares (type I).


B - Lymphocytes et cellules de Steinberg en quantité importante, ainsi que des polynucléaires neutrophiles et éosinophiles et
des histiocytes et plasmocytes (Type III).
C - Type II = nodules + travées de collagène.
D - Type IV, de type fibreux ou réticulaire.

Parmi les examens de laboratoire suivants, quel est celui dont le résultat peut éventuellement modifier le
classement en stade anatomoclinique ?
A - Hémogramme
B - Biopsie ostéomédullaire
C - Vitesse de sédimentation
D - Taux de fibrinogène
E - Electrophorèse du sérum
Bonne(s) réponse(s) : B

Fait partie du bilan d'extension. Sa positivité implique un stade IV (topographique). Les autres examens précisent l'évolutivité
biologique.

Si ces divers examens n'ont pas modifié le stade anatomoclinique, quelles sont les chances de mise en
rémission complète par un traitement adéquat ?
A - 10 %
B - 25 %
C - 50 %
D - 75 %
E - 95 %
Bonne(s) réponse(s) : D

Le traitement repose sur l'alternance chimio et radiothérapie.

Une polychimiothérapie selon le protocole MOPP (Moutarde azotée, Vincristine, Procarbazine. Prednisone) est
prévue. Cette association peut entraîner comme complication(s) :
A - Insuffisance médullaire
B - Neuropathie périphérique
C - Insuffisance cardiaque
D - Stérilité
E - Hépatite chronique
Bonne(s) réponse(s) : A B D

C - Anthracyclines (Adriamycine®).
E - Methrotrexate®, Purinéthol®.
691
Exclusivement sur DOC - DZ : www.doc-dz.com NADJI 85
RESIDANAT EN POCHE TOME II
Cas Clinique en QCM
Un homme de 60 ans présente des adénopathies jugulo-carotidiennes, d'environ 1,5 cm de grand axe, bilatérales et
grossièrement symétriques, sans caractère inflammatoire, fermes et mobiles, des adénopathies axillaires de 2 cm de grand
axe présentant les mêmes caractéristiques. La rate déborde le rebord costal de 4 cm. Le reste de l'examen est entièrement
négatif et en particulier il n'existe aucun contexte infectieux clinique. L'hémogramme montre :
- hémoglobine : 14 g/100 ml
- globules rouges : 4,5 x 10 exposant 12/l
- hématocrite : 46 %
- réticulocytes : 0,2 %
- leucocytes : 45 x 10 exposant 9/l dont 7 % de polynucléaires neutrophiles, 1 % de monocytes et 92 % de Iymphocytes sans
anomalie morphologique.
L'uricémie est normale. L'électrophorèse des protéines du sérum montre une hypogammaglobulinémie. La vitesse de
sédimentation est normale.

Parmi les hypothèses diagnostiques suivantes, quelle est celle qui vous paraît la plus plausible ?
A - Une agranulocytose aiguë immuno-allergique
B - Une infection virale aiguë
C - Une leucémie à tricholeucocytes
D - Une leucémie lymphocytaire chronique
E - Une leucémie aiguë lymphoblatique
Bonne(s) réponse(s) : D

Diagnostic évoqué sur l'âge, le caractère des polyadénopathies, la splénomégalie, la


conservation de l'état général, les données de l'hémogramme.

Parmi les examens complémentaires suivants, quel est le seul utile pour confirmer le diagnostic ?
A - La lymphographie par voie pédieuse
B - Une biopsie ganglionnaire
C - Un examen de la moelle osseuse
D - Une sérologie virale
E - Une échographie splénique
Bonne(s) réponse(s) : C

Qui montre un envahissement lymphocytaire de la moelle. La biopsie du ganglion est non


spécifique.

Le diagnostic étant posé, quel est parmi les éléments suivants celui qui serait indiscutablement de mauvais
pronostic ?
A - L'apparition d'une hyperuricémie
B - L'apparition d'une anémie par insuffisance médullaire
C - Une splénomégalie plus importante
D - Une augmentation symétrique du volume des adénopathies
E - Une augmentation de la vitesse de sédimentation
Bonne(s) réponse(s) : B

A - Conséquence du renouvellement cellulaire rapide.


B - Fait passer la classification au stade C. Médiane de survie 20 mois (stade le plus évolué
dans classification de Binet).
C.D.E - Montrent une évolutivité, moins préoccupante que B.

Parmi les attitudes thérapeutiques suivantes, quelle est celle qui vous parait la plus indiquée ?
A - Abstention thérapeutique
B - Traitement antibiotique d'épreuve
C - Chimiothérapie
D - Splénectomie
E - Irradiation en mantelet des adénopathies sus diaphragmatiques
Bonne(s) réponse(s) : C

Les attitudes possibles sont :


Abstention (stade A) et surveillance ; monochimiothérapie (chloraminophène) stade B. ; protocole thérapeutique plus lourd
(stade C).

692
Exclusivement sur DOC - DZ : www.doc-dz.com NADJI 85
RESIDANAT EN POCHE TOME II
Cas Clinique en QCM
Une femme de 40 ans, d'origine française, consulte pour asthénie. Elle n'accuse aucun antécédent particulier. Elle n'a pas
d'enfant et n'est pas enceinte. L'examen clinique retrouve un syndrome anémique isolé. Vous faites pratiquer un
hémogramme qui vous donne les résultats suivants :
- Erythrocytes : 4,6.10 exposant 12/l
- Hémoglobine : 8,7 g/dl
- Hématocrite : 27 %
- Leucocytes : 9.10 exposant 9/l, dont : Polynucléaires neutrophiles : 64 %
Polynucléaires éosinophiles : 6 %
Lymphocytes : 24 %
Monocytes : 6 %
- Plaquettes : 540.10 exposant 9/l
La VS est à 12/25. Le fer sérique est dosé ainsi que la capacité totale de fixation (CTE) et le coefficient de saturation de la
sidérophiline (CSS).
L'on aboutit ainsi au diagnostic d'anémie ferriprive.

Au vu de l'hémogramme, comment définissez-vous l'anémie ?


A - Normochrome macrocytaire
B - Hypochrome normocytaire
C - Microcytaire normochrome
D - Normochrome normocytaire
E - Aucune de ces propositions n'est exacte
Bonne(s) réponse(s) :C

VGM = Ht/Gr = 58 µ3.


CCMH = Hb/Ht est normale.

Quel(s) résultat(s) du dosage du fer sérique et de la capacité totale de fixation pourrai(en)t correspondre au
diagnostic porté chez cette malade ?
(les unités de mesure sont en microgr/100 ml)
A - Fer sérique : 270, CTF : 300
B - Fer sérique : 25, CTF : 410
C - Fer sérique : 100, CTF : 300
D - Fer sérique : 50, CTF : 250
E - Fer sérique : 10, CTF : 500
Bonne(s) réponse(s) : B E

Taux de fer sérique = 50 à 150 µg pour 100 ml.


CTS = 150 à 450.
CTS élevée si anémie ferriprive, basse en cas d'inflammation (catabolisme).

Quel(s) autre(s) résultat(s) anorma(ux)l de l'hémogramme sont concordants avec une carence en fer ?
A - Nombre des globules blancs
B - Nombre des plaquettes
C - Nombre des monocytes
D - Nombre des éosinophiles
E - Nombre des Iymphocytes
Bonne(s) réponse(s) : B

La carence martiale est une étiologie classique des thrombocytoses.

En dehors de la carence en fer, quelle(s) autre(s) affection(s) pourrai(en)t induire une anémie du type de celle
présentée par la malade ?
A - Trouble de synthèse de l'hème
B - Carence en acide folique
C - Erythroblastopénie chronique
D - Envahissement médullaire par des cellules néoplasiques
E - Microsphérocytose héréditaire
Bonne(s) réponse(s) : A

A - Dans le cadre d'une anémie sidéroblastique congénitale microcytaire, transmission récessive liée au sexe.
B - Macrocytose.
C - Normocytose.
D - Donne une insuffisance médullaire avec normocytose et atteinte des autres lignées.
E - Anémie normochrome normocytaire ou macrocytaire (hémolyse chronique ou en poussée).

693
Exclusivement sur DOC - DZ : www.doc-dz.com NADJI 85
RESIDANAT EN POCHE TOME II
Cas Clinique en QCM

Quelle(s) étiologie(s) peu(ven)t être responsable(s) exclusifs de l'anémie ferriprive chez cette malade ?
A - Carence d'apport en fer
B - Hémorragies digestives
C - Don du sang trisannuel
D - Hémorragie abondante et brutale 15 jours auparavant
E - Ménorragies chroniques
Bonne(s) réponse(s) : B E

A - Pratiquement jamais en cause.


B - Cause fréquente (100 ml = 50 mg de perte de fer).
C - Peut aggraver mais non expliquer.
D - Serait régénérative avec restauration du chiffre d'Hb.
E - Cause classique à rechercher systématiquement chez la femme.

Vous allez traiter cette malade par l'administration d'un sel ferreux. Quelle(s) est (sont) la (les) proposition(s)
fausse(s) ?
A - Les sels ferreux par voie orale sont mieux absorbés quand la prise se situe au milieu des repas
B - La voie intraveineuse doit être préférée a la voie orale
C - L'administration de sels ferreux par voie orale colore les selles en noir
D - La dose optimale par voie orale est de 100 à 200 mg de fer élément par jour
E - Un mois de traitement est suffisant pour restaurer les réserves en fer
Bonne(s) réponse(s) : A B E

A - Mieux tolérés mais moins bien absorbés.


B - Non, sauf malabsorption ou troubles digestifs majeurs.
C - Ce dont il faut prévenir le patient.
D - 100 mg à 250 mg par jour dont 20 % seulement sont absorbés.
E - Non, au minimum 3 mois sont nécessaires.

Un homme de 60 ans, gros fumeur, hypertendu, est hospitalisé pour malaise. Dans les antécédents, on retrouve des
céphalées. L'examen met en évidence une hypertension artérielle à 18-12, une rate débordant de 5 cm sous le rebord costal,
une distension thoracique avec une langue dépapillée, un foie de 13 cm de hauteur ferme, une érythrose des extrémités.
L'examen neurologique est normal .
L'hémogramme montre :
- GR : 6 500 000/mm3
- Hématocrite : 60 %
- Leucocytes : 16 000/mm3 dont 60 % de PN - 0 % de PB - 30 % de lymphocytes - 10 % de monocytes
- Plaquettes : 600 000/mm3
- VS : 1-2

Quelle(s) est(sont) la(les) manifestation(s) clinique(s) expliquée(s) par les anomalies de l'hémogramme ?
A - HTA
B - Céphalées
C - Erythrose
D - Langue dépapillée
E - Distension thoracique
Bonne(s) réponse(s) : B C

B - Fait partie du syndrome d'hyperviscosité sanguine.


C - Conséquence de la polyglobulie.

Quel est, parmi ces examens, celui que vous allez pratiquer pour affirmer le diagnostic de polyglobulie ?
A - Mesure du volume globulaire moyen
B - Mesure du taux d hémoglobine
C - Mesure du volume plasmatique à l'albumine
D - Mesure du pool potassique
E - Mesure du volume globulaire au Chrome 51
Bonne(s) réponse(s) : E

Seul cet examen permet d'affirmer la polyglobulie vraie, définie par : VG > 36 ml/kg chez l'homme, VG > 32 ml/kg chez la
femme.

694
Exclusivement sur DOC - DZ : www.doc-dz.com NADJI 85
RESIDANAT EN POCHE TOME II
Cas Clinique en QCM

Ce patient a une polyglobulie vraie. Quel(s) est(sont) l'(es) examen(s) que vous allez pratiquer pour exclure le
diagnostic de polyglobulie secondaire ?
A - Scintigraphie pulmonaire
B - Gaz du sang en air ambiant
C - Urographie intra-veineuse
D - Uricémie
E - Fond d'oeil
Bonne(s) réponse(s) : B C

Permettent d'éliminer les polyglobulies secondaires :


B - A une désaturation du sang artériel en oxygène (causes respiratoires ou cardiaques).
C - A une cause tumorale rénale (cancer ou kyste du rein).
Il faut en outre rechercher une hémoglobinopathie, un hémangioblastome du cervelet, un hépatome, un fibrome utérin et un
cancer ovarien (chez la femme), un syndrome de Cushing.

Quel est le diagnostic le plus vraisemblable que vous évoquez en fonction du contexte clinique et biologique ?
A - Une insuffisance respiratoire de type restrictif
B - Une conséquence directe du tabagisme
C - Un syndrome myéloprolifératif
D - Un cancer du rein
E - Un hépatome
Bonne(s) réponse(s) : C

En raison de l'existence d'une splénomégalie, et de signes d'atteinte des trois lignées sanguines
(hyperleucocytose, thrombocytose).

Madame S amène son fils de 18 mois pour une tuméfaction frontale gauche déclenchée par une chute. quelques heures
auparavant. Proéminente (près de 3 cm), volumineuse (diamètre 7 cm), cette tuméfaction est très douloureuse et difficile à
examiner. Le bilan d'hémostase réalisé en urgence. donne les résultats suivants :
- plaquettes : 320 000/mm3
- temps de Quick : 85 %
- temps de céphaline activé : 92 sec (témoin : 33 sec)
- temps de thrombine : 18 sec (témoin : 18 sec)
- fibrinémie : 2, 20 g/l
- temps de saignement (Ivy 3 points) : 3 min 30 sec
L'allongement du temps de céphaline activé est corrigé in vitro par l'apport de plasma témoin normal.

Parmi les hypothèses diagnostics suivantes, une seule est possible. Laquelle ?
A - Maladie de Willebrand typique
B - Thrombopathie constitutionnelle
C - Hémophilie fruste
D - Anticoagulant circulant
E - Hémophilie sévère
Bonne(s) réponse(s) : E

A - Entraîne un allongement du temps de saignement.


B - Idem.
C - N'est révélée que lors d'interventions chirurgicales.
D - Seulement avec un temps de Quick allongé, un temps de thrombine normal et un taux de facteur II, V, VII, X normal.

Parmi les facteurs de coagulation suivants, indiquez celui dont le déficit constitutionnel est le plus
vraisemblablement, du fait de sa fréquence, responsable de ce tableau :
A - VIII
B - IX
C-X
D - XI
E - XII
Bonne(s) réponse(s) : A

L'hémophilie B (déficit en facteur IX) est six fois moins fréquente.


Les déficits en facteurs XI et XII sont rares et le plus souvent sans traduction clinique.

695
Exclusivement sur DOC - DZ : www.doc-dz.com NADJI 85
RESIDANAT EN POCHE TOME II
Cas Clinique en QCM

Quel geste thérapeutique vous semble convenir à cet état ?


A - Perfusion de sang frais
B - Perfusion de PPSB et ponction évacuatrice
C - Perfusion de cryoprécipités et ponction évacuatrice
D - Vitamine K IM
E - Abstention thérapeutique
Bonne(s) réponse(s) : C

Le cryoprécipité apporte le facteur VIII manquant et du fibrinogène. Doses : 20 à 40 U/kg


toutes les 8 heures.
La ponction évacuatrice est nécessaire ainsi que l'immobilisation et la rééducation pour
prévenir les complications rhumatologiques.

Compte-tenu de vos réponses aux questions 1 et 2, quelle est, pour la mère, la probabilité de transmettre
l'anomalie à ses garçons ?
A-0%
B - 25 %
C - 50 %
D - 75 %
E - 100 %
Bonne(s) réponse(s) : C

Maladie héréditaire avec transmission récessive liée au sexe (prouvée par l'X).

Indiquez, parmi les manifestations cliniques suivantes, celle(s) dont il faut prévenir la famille de l'apparition
probable :
A - Hématomes musculaires compressifs
B - Hématuries
C - Purpura pétéchial
D - Hémorragies de section
E - Hémarthroses spontanées
Bonne(s) réponse(s) : A B D E

C - Trouble de l'hémostase primaire non concernée par l'hémophilie A.

Chez cet enfant, en l'absence de traitement spécifique du trouble de l'hémostase, il est absolument interdit de
pratiquer :
A - Injection intramusculaire
B - Vaccination anti-hépatite B
C - Adénoïdectomie
D - Injection intraveineuse d'un médicament
E - Prescription d'aspirine à titre d'antalgique
Bonne(s) réponse(s) : A C E

B - Au contraire, en raison des risque transfusionnels.


D - Sans risque hémorragique.

Un homme de 61 ans consulte pour des céphalées, des bourdonnements d'oreilles, un prurit à l'eau. Il présente depuis
plusieurs mois une érythrose. L'examen clinique montre, en outre, un débord splénique de 4 cm. Il présente l'hémogramme
suivant :
- G.R. : 7200 000/mm3
- Hte. : 0,69
- Hb : 22 g/100 ml
- G.B : 16000/mm3 dont PN : 79 %, PE : l %, PB : 1%, L : 16 %, M : 3 %
- Plaquettes : 550 000/mm3.

Parmi les éléments suivants, le(s)quel(s) plaide(nt) pour la nature primitive de cette polyglobulie ?
A - Le fait qu'il s'agisse d'un homme
B - L'érythrose de ce sujet
C - La palpation d'une splénomégalie
D - L'hyperleucocytose
E - L'hyperplaquettose
Bonne(s) réponse(s) : C D E

A - L'âge est davantage un argument d'orientation.


B - Non spécifique.
C - Argument clinique majeur, mais inconstant.
DE - En relation avec le syndrome myéloprolifératif.
696
Exclusivement sur DOC - DZ : www.doc-dz.com NADJI 85
RESIDANAT EN POCHE TOME II
Cas Clinique en QCM

Parmi les examens suivants, indiquer celui qui est nécessaire pour affirmer le diagnostic de polyglobulie de
Vaquez chez ce malade :
A - Le score des phosphatases alcalines leucocytaires
B - Le caryotype médullaire
C - Le myélogramme
D - La scintigraphie médullaire
E - Aucun des précédents
Bonne(s) réponse(s) : E

Permet seule le diagnostic de syndrome myéloprolifératif et au contraire du myélogramme permet d'apprécier l'architecture de
la moelle.

Parmi les thérapeutiques suivantes, Iaquelle (lesquelles) est (sont) utilisable(s) dans la maladie de Vaquez ?
A - Chimiothérapie par voie orale
B - Saignées répétées
C - Phosphore radioactif
D - Chrome radioactif
E - Polychimiothérapie séquentielle
Bonne(s) réponse(s) : A B C

Le traitement dépend de l'âge du patient, du taux de plaquettes et du degré d'urgence


(hyperviscosité imposant les saignées).

Sachant que ce malade n'est pas disposé à se plier à une surveillance régulière, quelle est la thérapeutique qui
vous semble la plus adaptée pour réduire durablement sa polyglobulie ?
A - Corticoïdes par voie orale
B - Saignées répétées
C - Phosphore radioactif
D - Chrome radioactif
E - Polychimiothérapie séquentielle
Bonne(s) réponse(s) : C

Les saignées répétées nécessitent une surveillance régulière de l'hématocrite et des plaquettes.

Un homme de 55 ans se plaint de vertiges et de céphalées. A l'examen, érythrose faciale nette ; front, pommettes, nez sont
rouges. La rate est palpée 2 cm sous le rebord costal.
- Coeur : éclat de B2 ; T A : 19-10 cm de Hg
- NFS : GR ; 7,8 x 1012/l ; Hb, 22 g/100 ml ; HT, 60 % ; G B, 12 x 10 exposant 9/l dont 65 % de PN, 4 % de mono, 31 % de
lympho
- Plaquettes : 390 x 10 exposant 9/l
- Gaz du sang artériel : p02 : 85 mm de Hg, SaO2 : 95%, pC02 : 39 mm de Hg
- Fonction rénale : normale
- U I V : normale
- Mesure de la masse sanguine : en attente
- Uricémie : normale

Des ce stade, vous pouvez écarter certaines causes de polyglobulie secondaire, en fonction de certaine(s)
donnée(s) de l'observation. Laquelle(lesquelles) ?
A - Gaz du sang artériel
B - Fonction rénale normale
C - UIV normale
D - Le chiffre de plaquettes
E - L'uricémie normale
Bonne(s) réponse(s) : A C

Permettent d'éliminer les polyglobulies secondaires à une hypoxie tissulaire (A) ou à une hypersécrétion d'érythropoïétine (B =
cancer du rein, kyste rénal, sténose de l'artère rénale).

697
Exclusivement sur DOC - DZ : www.doc-dz.com NADJI 85
RESIDANAT EN POCHE TOME II
Cas Clinique en QCM

Quel(s) élément(s) de l'observation retenez vous en faveur du diagnostic de maladie de Vaquez ?


A - Erythrose du visage
B - TA : 19-10
C - Hématocrite : 60%
D - Plaquettes : 390 x 10 exposanrt 9/l
E - Rate palpable
Bonne(s) réponse(s) : E

ABC - Signes de polyglobulie non spécifiques.


D - Plutôt en défaveur.
E - Argument majeur mais inconstant.

La mesure du volume globulaire par la technique des hématies marquées au chrome 51 :


A - Permet de différencier polyglobulies primitives et secondaires
B - Permet de dlfférencier polyglobulie vraie et hémoconcentration
C - Est un traitement de la maladie de Vaquez
D - Permet d'identifier les thalassémies mineures
E - Montre dans les polyglobulies un allongement de la durée de vie des GR
Bonne(s) réponse(s) : B

Seul test diagnostic permettant d'affirmer la polyglobulie vraie et d'éliminer hémoconcentration et pseudo-polyglobulie.

Vous êtes interne de garde en médecine et vous êtes appelé auprès de ce patient pour une phlébite surale
typique. Vous n'avez pas d'autre renseignement sur le diagnostic. Quel(s) traitement(s) prescrivez-vous ?
A - Phosphore 32 : 0,l millicurie/kg
B - Misulban : 3 comprimés/jour pendant 15 jours
C - Saignées : 350 ml
D - Héparinothérapie IV à la pompe : 20000 unités/jour
E - Abstention de tout traitement
Bonne(s) réponse(s) : C D

La saignée entraîne une hyperplaquettose, source elle-même de thrombose. Elle n'est dont pratiquée que si les plaquettes
sont inférieures à 800 000 par mm3. Pas de prescription de chimiothérapie sans diagnostic de certitude.

Fille âgée de six ans, sans antécédent. Epistaxis abondantes, gingivorragies et purpura depuis 48 h.
A l'examen : pétéchies, ecchymoses, pâleur cutanéo muqueuse importante. Rate débordant de 4 cm le rebord costal.
Polyadénopathies modérées, cervicales et axillaires. Apyrétique.
Hémogramme :
- Globules rouges : 3 200 000/mm3
- Hématocrite : 2 9 %
- Hémoglobine : 9,6 g/dl
- Réticulocytes : 0,8 %
- Globules blancs : 2 600/mm3, Polynucléaires : 17 %
Polynucléaires éosinophiles : 1%
Lymphocytes : 72%
Lymphoblastes : 10 %
- Plaquettes : 10 000
Le myélogramme montre une moelle riche avec 90 % de lymphoblastes, 5 % d'érythroblastes, 5 % de granuleux, absence de
mégacaryocytes.

Comment caractérisez-vous cette anémie ?


A - Normocytaire non régénérative
B - Macrocytaire régénérative
C - Hypochrome non régénérative
D - Microcytaire non régénérative
E - Macrocytaire non régénérative
Bonne(s) réponse(s) : A

VGM = Ht/GR = 90 µ3 = normocytaire.


CCMH = Hb/Ht = 0,33 = normochrome.
Réticulocytes < 120000 par mm3 = arégénérative.

698
Exclusivement sur DOC - DZ : www.doc-dz.com NADJI 85
RESIDANAT EN POCHE TOME II
Cas Clinique en QCM

Avant de débuter le traitement, quel(s) est (sont) le(s) examen(s) qui vous parai(ssen)t utile(s) ?
A - Uricémie
B - Temps de saignement
C - MNI-test
D - Electromyogramme
E - Fer sérique
Bonne(s) réponse(s) : A

Il s'agit d'une leucémie aiguë Iymphoblastique. L'uricémie augmente en raison de l'hypercatabolisme des nucléoprotéines
cellulaires secondaires à la maladie et au traitement.

Parmi les traitements symptomatiques suivants, quel(s) est(sont) celui(ceux) qui vous semble(nt) justifié(s) ?
A - Boissons alcalines abondantes
B - Allopurinol (Zyloric®)
C - Antibiotiques à large spectre par voie intramusculaire
D - Transfusion de sang frais
E - Transfusion de globules blancs
Bonne(s) réponse(s) : A B

Permettent de prévenir les lithiases uriques.


C - Pas d'indication, pas de voie IM (thrombopénie).
D.E - Pas d'indication (cf: hémogramme).

Le traitement spécifique comporte en particulier de la vincristine (Oncovin®). Parmi les complications suivantes
la(les) quelle(s) peut(vent) être attribuée(s) à ce médicament ?
A - Surdité
B - Constipation
C - Neuropathie périphérique
D - Alopécie
E - Cardiomyopathie
Bonne(s) réponse(s) : B C D

A - Concerne le Cisplatyl®.
B - Risque de syndrome pseudo-occlusif parfois prévenu par l'adjonction de Prostigmine®.
C - Complication classique
D - Transitoire
E - Concerne les anthracyclines (Adriamycine®).

Quel est la durée prévisible du traitement chimiothérapique classique en l'absence de rechute ?


A - 2-4 mois
B - 8-12 mois
C - 2-5 mois
D - 6-8 mois
E - A vie
Bonne(s) réponse(s) : C

Sans commentaire.

699
Exclusivement sur DOC - DZ : www.doc-dz.com NADJI 85
RESIDANAT EN POCHE TOME II
Cas Clinique en QCM
Une femme de 42 ans présente un état anémique. L'interrogatoire révèle que depuis deux ans il existe des malaises (2 ou 3)
à type de lipothymies avec sueurs et impression de lourdeur abdominale. Depuis 6 mois, les troubles deviennent permanents
et s'accentuent : asthénie, dyspnée d'effort et palpitation. La pâleur apparaît évidente. On ne retrouve pas d'autres troubles
digestifs. Les règles sont régulières (durée 4 jours, sans caillot), l'évaluation de leur volume est normal. L'examen clinique est
normal.
Hémogramme :
- globules rouges : 3,5 T/l
- hémoglobine : 7,5 g/dl
- hématocrite : 26 %
- VGM : 74 fl
- CHM : 22 pg
- CCHM : 28 %
- plaquettes : 520 G/l
- globules blancs : 12,8 G/l, polynucléaires neutrophiles : 0,80 %
polynucléaires éosinophiles : 0,05 %
polynucléaires basophiles : 0
Iymphocytes : 0,12 %
monocytes : 0,03 %
- Vitesse de sédimentation, 1ère heure : 35
- Fer sérique : 4,2 micromol/l
Le diagnostic d'anémie ferriprive est retenu

Parmi les caractères suivants, lequel(lesquels) peu(ven)t être expliqué(s) par la carence martiale ?
A - Le taux des éosinophiles
B - Le chiffre de plaquettes
C - Le VGM
D - Le taux de polynucléaires neutrophiles
E - Le taux de Iymphocytes
Bonne(s) réponse(s) : B C

La carence martiale entraîne une microcytose, définie par un VGM < 85 µ3 et est fréquemment associée à une
thrombocytose. Les leucocytes sont en nombre normal.

Quel examen choisissez-vous pour confirmer la carence martiale ?


A - Capacité totale de fixation de la sidérophiline
B - Dosage de l'haptoglobine
C - Dosage de la céruloplasmine
D - Hémosidérinurie
E - Myélogramme avec coloration de Perls
Bonne(s) réponse(s) : A

A - Dosage indirect de fer sérique (lié à la transferrine ou sidérophilline) lors de la constitution la déplétion se fait aux dépens
de la ferritine puis on observe une augmentation réactionnelle du taux de sidérophilline.
B - Utilisé dans le diagnostic des anémies hémolytiques (alors effondrée).
C - Protéine porteuse du cuivre, non concernée.
D - Présente dans les hémolyses intravasculaires.
E - La coloration de Perls met en évidence la présence d'hémosidérine (sidéroblastes), fer de réserve insoluble lentement
disponible. Cet examen est inutile au diagnostic.

La carence martiale étant affirmée, citez les deux examens qui vous paraissent d'emblée les plus adaptés pour
découvrir l'étiologie :
A - Coloscopie
B - Fibroscopie gastrique
C - Hystérographie
D - Urographie intra-veineuse
E - Scanner abdominal
Bonne(s) réponse(s) : A B

Orientation sur les données cliniques. Les autres étiologies sont gynécologiques, liées à l'absorption (gastrectomie, pica,
maladie coeliaque), ou à une majoration des besoins (nourrisson, croissance, grossesse).

Si l'étiologie n'est pas rapidement trouvée, quelle attitude vous parait la plus adaptée à la situation ?
A - Transit du grêle
B - Recherche d'une hématurie microscopique
C - Etude de l'absorption du fer
D - Etude de l'incorporation du fer 59
E - Scintigraphie abdominale avec globules rouges marqués
Bonne(s) réponse(s) : A

En l'absence de pertes de fer digestives ou gynécologiques, il faut rechercher un trouble de l'absorption, évoqué ici sur
l'association à des lipothymies avec sueurs pouvant être liées à une hypoglycémie. Il n'y a pas d'éléments du dossier en
faveur d'une majoration des besoins en fer.

700
Exclusivement sur DOC - DZ : www.doc-dz.com NADJI 85
RESIDANAT EN POCHE TOME II
Cas Clinique en QCM

Quelle que soit l'étiologie, il est nécessaire de corriger la carence martiale .Quelle prescription journalière faites-
vous ?
A - 10mg de fer métal per os
B - 100mg de fer métal per os
C - 100mg de fer métal par voie intramusculaire
D - 1g de fer métal per os
E - 1g de fer métal par voie intramusculaire
Bonne(s) réponse(s) : B

La correction se fait par voie orale, sauf dans des circonstances exceptionnelles (voie per os impossible, intolérance absolue).
La dose quotidienne nécessaire est de 50 mg. 20 % seulement sont absorbes. Il faut donc 250 mg par jour en 2 ou 3 prises,
mieux tolérés au moment des repas.

Surveillance. Ce traitement sera poursuivi :


A - 1 mois
B - Jusqu'à normalisation du taux d hémoglobine
C - Jusqu'à normalisation du VGM
D - Jusqu'à normalisation du fer sérique
E - Jusqu'à normalisation de la capacité totale de saturation de la sidérophiline
Bonne(s) réponse(s) : E

Durée de correction des paramètres :


B - 2 mois.
C - Quelques semaines.
D - Quelques semaines.
E - Plusieurs mois (dernier élément à se normaliser).

Un homme de 63 ans consulte pour une gêne post-prandiale au niveau de l'hypochondre gauche. Il n a pas d'antécédent
médical ou chirurgical particulier. Chez ce patient en bon état général, apyrétique, l'examen objective une splénomégalie
régulière, indolore, débordant le gril costal de 10 cm. On ne trouve ni hépatomégalie, ni circulation veineuse collatérale. On
note une pâleur modérée des conjonctives. Il n'y a pas d'adénopathies palpables et le reste de l'examen est normal.
Hémogramme :
- Hématies : 3 x 10 exposant 12/l
- Hémoglobine : 10 g/dl
- Hématocrite : 0,28 l/l
- Volume globulaire moyen : 93,3 fl
- CCHM : 35,7 %
- Réticulocytes : 31 x 10 exposant 9/l
- Leucocytes : 33 x 10 exposant 9/l
- Formule leucocytaire : polynucléaires neutrophiles : 65 %
polynucléaires éosinophiles : 1 %
polynucléaires basophiles : 1 %
Iymphocytes : 10 %
monocytes : 2 %
promyélocytes : 2 %
myélocytes : 7 %
métamyélocytes : 12 %
- Plaquettes : 700 x 10 exposant 9/l
- Vitesse de sédimentation : 15 mm à la première heure
- Fibrinogène : 2,5 g/l

L'analyse de cet hémogramme révèle :


A - Anémie normochrome normocytaire
B - Anémie régénérative
C - Polynucléose neutrophile
D - Myélémie
E - Thrombocytose
Bonne(s) réponse(s) : A C D E

Par définition.

Indépendamment de la clinique et du reste de l'hémogramme, l'anomalie présentée au niveau des plaquettes


peut se rencontrer au cours :
A - D'un cancer
B - D'une polyarthrite rhumatoïde
C - D'une carence martiale
D - D'un syndrome myéloprolifératif
E - Après une splénectomie
Bonne(s) réponse(s) : A B C D E

Ainsi qu'au cours des régénérations médullaires après hémorragie ou hémolyse, des stress, de l'exercice ou de la grossesse.

701
Exclusivement sur DOC - DZ : www.doc-dz.com NADJI 85
RESIDANAT EN POCHE TOME II
Cas Clinique en QCM

Compte-tenu des données cliniques et biologiques dont vous disposez, deux diagnostics sont possibles parmi
les suivants :
A - Anémie ferriprive
B - Leucémie myéloïde chronique
C - Leucémie aiguë myéloblastique
D - Maladie de Waldenstrِm
E - Splénomégalie myéloïde
Bonne(s) réponse(s) : B E

L'association d'une splénomégalie avec myélose, thrombocytose et anémie oriente vers un syndrome myéloprolifératif dont la
nature exacte sera précisée par la biopsie osseuse.

Pour distinguer ces deux maladies, il est utile de disposer de :


A - Dosage du fer sérique
B - P.A.L. (phosphatase alcaline leucocytaire)
C - Caryotype médullaire
D - Immuno-électrophorèse des protides
E - Biopsie ostéomédullaire
Bonne(s) réponse(s) : B C E

Dans la LMC, les PAL sont effondrées (sauf en cas d'acutisation), le chromosome Philadelphie est présent, la moelle est
hyperplasique (lignée granuleuse surtout) et équilibrée. La fibrose ne survient qu'en fin d'évolution. Dans la splénomégalie
myéloïde, les PAL ne sont pas effondrées, il n'existe pas de chromosome Philadelphie et l'architecture de la moelle est
modifiée (fibrose classée en 3 stades).

Une femme de 28 ans, victime 1/2 heure plus tôt d'un accident d'automobile, arrive à l'hôpital conduite par le SAMU. Elle est
pâle, couverte de sueurs, se plaint d'avoir soif et décrit une douleur sous-costale gauche irradiant en bretelle.
Il y a une certaine défense à la palpation de l'hypocondre gauche, le toucher rectal est modérément douloureux. La patiente
n'a pas d'antécédent pathologique notable, elle n'a jamais été opérée ; elle a même trois grossesses à terme. Ses trois
enfants sont en bonne santé. Une carte de groupe sanguin établie à la fin de sa première grossesse porte les mentions :
"groupe A1. Rhésus positif - 1ère détermination".

On évoque le diagnostic d'hémopéritoine par rupture de la rate. En dehors de l'examen local, l'appréciation de
l'importance et du retentissement général de cette hémorragie se base à l'instant de l'arrivée sur :
A - Les caractères du pouls
B - Les chiffres tensionnels
C - L'hématocrite
D - Le taux d'hémoglobine
E - Le chiffre des réticulocytes
Bonne(s) réponse(s) : A B

En cas d'anémie d'installation rapide, on note un pouls rapide et une hypotension qui peut aller jusqu'au choc (TA < 80 mm
Hg). L'hémogramme est un mauvais reflet dans les premières heures, l'anémie étant initialement sous estimée (perte de
globules et de plasma) puis surestimée (hémodilution par afflux de liquide extracellulaire). L'anémie n'est régénérative
qu'autour de 48 heures.

Dès l'arrivée, on prélève du sang en vue de divers examens biologiques. Parmi ces examens, il faut
nécessairement inclure :
A - Une mesure isotopique de la volémie
B - Une mesure de la vitesse de sédimentation globulaire
C - Une deuxième détermination du groupe ABO et rhésus
D - Une recherche d'agglutinines irrégulières
E - Un test de Coombs direct
Bonne(s) réponse(s) : C D

D - N'est à pratiquer qu'en cas de stimulations antigéniques nombreuses (grossesses, transfusions multiples) et se fait en
dehors de l'urgence.

702
Exclusivement sur DOC - DZ : www.doc-dz.com NADJI 85
RESIDANAT EN POCHE TOME II
Cas Clinique en QCM

La malade est transfusée en même temps qu'on pratique la splénectomie. Parmi les sangs des groupes
suivants, quel est le meilleur choix pour la transfuser en l'absence de sang A1 rhésus positif ?
A - O Rhésus positif
B - A2 Rhésus positif
C - A 1 Rhésus négatif
D - A1 B Rhésus positif
E - A2 B Rhésus positif
Bonne(s) réponse(s) : B

A - Le groupe O renferme les "donneurs universels dangeureux", comportant des sujets avec hémolysines immunes, anti A.
Ils sont donc réservés aux sujets O sauf extrême nécessité.
B - Réponse la plus adaptée
C - Non, patiente rhésus +.
D - La patiente possède des anticorps anti-B.
E - La patiente possède des anticorps anti-B.

Aucun incident notable n'est survenu pendant l'intervention. L'opérée entre en salle de réveil avec une tension
artérielle à 14/8. Un dernier sac de sang a été branché à ce moment. Une demi-heure plus tard, surviennent de
violents frissons et une élévation thermique à 40,2°C. La tension artérielle est à 15/9. Il ne vient pas de sang par
les drains. L'hypothèse étiologique la plus probable est celle :
A - D'une incompatibilité ABO
B - De !a présence chez la patiente d'agglutinines irrégulières anti-érythrocytes
C - De la présence d anticorps anti-HLA
D - D'une septicémie à pneumocoques
E - D'une souillure bactérienne du sac
Bonne(s) réponse(s) : C

Survenue d'un syndrome frisson-hyperthermie (sans choc) évocateur d'immunisation anti-HLA.

Devant cet incident, il faut :


A - Arrêter la transfusion
B - Recontrôler les groupes du sac et de l'opérée
C - Injecter un antibiotique
D - Injecter du chlorure de potassium
E - Injecter de l'héparine
Bonne(s) réponse(s) : A B

A.B - Sont systématiques devant tout accident transfusionnel, ainsi que la pratique d'un test de Coombs direct, la recherche
d'une contamination bactérienne (culot, hémoculture), de signes de CIVD.

Une infirmière de 23 ans est hospitalisée en urgence pour un syndrome hémorragique. A l'examen : ecchymoses spontanées
multiples, hématurie macroscopique, pâleur, tachycardie.
Le reste de l'examen est normal.
Hémogramme :
- hémoglobine : 12 g
- globules rouges normochromes normocytaires, globules blancs et plaquettes normaux
- VS : 10 à la 1ère heure
- hémostase : temps de Quick : 5 %
temps de céphaline activée : 80" - témoin 30" dosage des éléments du complexe prothrombique en attente
temps de thrombine à 17" - témoin à 16"
- fibrinogène : 4 g/l.
Radiographie pulmonaire normale. Antécédents : néant.
N'a pris aucun médicament depuis plusieurs mois, sauf une pilule minidosée.

Les résultats de ce bilan d'hémostase préliminaire sont en théorie compatibles avec :


A - CIVD par septicémie à perfringens
B - Maladie de Willebrand
C - Intoxication par l'aspirine
D - Insuffisance cellulaire hépatique
E - Déficit isolé en facteur V
Bonne(s) réponse(s) : E

A - Fibrinogène normal (abaissé dans CIVD).


B - TP abaissé (normal dans le Willebrand).
C - Pas de modification aussi importante du TP et du TCA.
D - Fibrinogène normal (abaissé dans l'insuffisance hépatique majeure avec TP à 5 %) et TT normal.
E - Allonge le TP et le TCA.

703
Exclusivement sur DOC - DZ : www.doc-dz.com NADJI 85
RESIDANAT EN POCHE TOME II
Cas Clinique en QCM

Les résultats des dosages en attente sont : facteur V 100 % ; facteur VII+X 5 % ; facteur II 5 %.
Quel diagnostic affirmez-vous ?
A - Insuffisance cellulaire hépatique
B - Hypovitaminose K
C - Amylose primitive
D - Déficit en protéine C
E - CIVD
Bonne(s) réponse(s) : B

A - Non, facteur V serait effondré.


B - Les facteurs Il, VII, IX et X sont vitaminoK-dépendants.
C - Donne des déficits en facteur X.
D - Donne des thromboses.
E - Non, le facteur V serait très bas.

Quel traitement d'urgence est indiqué ?


A - Injection de cryoprécipité congelé
B - Injection de plasma frais congelé
C - Injection de PPSB chauffé
D - Injection de reptilase
E - Transfusion de culots globulaires
Bonne(s) réponse(s) : C

A - Contient du facteur VIII.


B - Uniquement en l'absence de PPSB.
C - Apporte tous les facteurs vitamino K dépendant.
D - Hémostatique d'indication rare, agit sur tous les temps de l'hémostase.
E - Pas de chute du taux d'hémoglobine.

En l'absence de toute étiologie classique retrouvée chez cette patiente, il faut rechercher la prise (occultée ou
criminelle) de :
A - Aspirine
B - Héparine
C - Antivitamine K
D - Tagamet®
E - Alphaméthyldopa
Bonne(s) réponse(s) : C

A - Troubles de l'hémostase primaire, allongement du temps de saignement.


B - Allongerait le temps de thrombine ici normal.
C - Tableau éloquent.
D - Pas de troubles de l'hémostase.
E - Pas de troubles de l'hémostase.

Une jeune femme de 28 ans consulte pour asthénie. Elle a 3 enfants de 6, 4 et 2 ans. Elle est donneuse de sang depuis l'âge
de 18 ans. Ses règles sont peu abondantes, sans caillots, et durent trois jours. Les selles ne sont pas noires. L'examen trouve
une langue dépapillée, une koïlonychie débutante, une pâleur, une tachycardie. L'hémogramme est le suivant :
- hématies : 4 000.000/mm3,
- hématocrite : 21 %,
- hémoglobine : 7.5 g/dl.
- Leucocytes : 6 500/mm3 dont 67 % de polynucléaires neutrophiles, 30 % de Iymphocytes, 5 % de monocytes.
- 560.000 plaquettes/mm3.
Le dosage du fer sérique montre une sidérémie à 3 micromol/l, une capacité totale à 110 micromol/l.

On peut retenir comme diagnostic plausible ?


A - Anémie hypochrome microcytaire hyposidérémique
B - Anémie normochrome microcytaire hyposidérémique
C - Anémie normochrome normocytaire hyposidérémique
D - Anémie normochrome macrocytaire hyposidérémique
E - Hyposidérémie sans anémie
Bonne(s) réponse(s) : B

VGM = Ht/nbGr = 52,5 µ3 (N = 85-100).


CCMH = Hb/Ht = 0,35 (N = 30-36%).
Fer sérique normal (N = 12-24 µmol/litre).

704
Exclusivement sur DOC - DZ : www.doc-dz.com NADJI 85
RESIDANAT EN POCHE TOME II
Cas Clinique en QCM

Parmi les examens suivants, à la recherche d'une étiologie, quel est celui qui vous parait utile dans ce cas ?
A - Electrophorèse de l'hémoglobine
B - Dosage de la ferritine plasmatique
C - Etude de la cinétique du Fer 59
D - Médullogramme
E - Fibroscopie oeso-gastro-duodénale
Bonne(s) réponse(s) : E

Parmi les origines des pertes sanguines, il faut rechercher une cause digestive en priorité chez l'homme, gynécologique en
priorité chez la femme. Ces deux étiologies sont les plus fréquentes. On pensera ensuite aux troubles de l'absorption puis aux
augmentations des besoins.

Si ce premier examen est négatif, quel est celui parmi les examens suivants que vous effectuez dans un
deuxième temps ?
A - Urographie intraveineuse
B - Coloscopie
C - Etude de la durée de vie des hématies marquées au Chrome 51
D - Artériographie coeliaque
E - Recherche de sang dans les selles
Bonne(s) réponse(s) : B

Démarche diagnostique = fibroscopie, puis coloscopie, puis transit du grêle (si les examens précédents n'apportent pas la
réponse).

Parmi les symptômes de la patiente, certains sont la conséquence directe de l'hyposidérémie. Lequel(lesquels) ?
A - Tachycardie
B - Koïlonychie
C - Asthénie
D - Langue dépapillée
E - 550 000 plaquettes/mm3
Bonne(s) réponse(s) : B C D E

A - Conséquence de l'anémie.
B - Troubles des phanères liés à la carence martiale.
C - Retentit sur tous les métabolismes de l'organisme.
D - Atrophie des muqueuse digestives liées à la carence martiale.
E - Une des plus fréquentes cause de thrombocytose.

La quantité totale de fer élément à prescrire per os chez cette patiente pour corriger son hémogramme et
restaurer un stock martial convenable est approximativement (dose cumulative pour la durée du traitement) :
A - Cent milligrammes
B - Un gramme
C - Vingt grammes
D - Cent grammes
E - Deux cent grammes
Bonne(s) réponse(s) : C

Soit 200 mg de fer par jour pendant deux mois puis deux mois, à demi-dose pour reconstituer les réserves.

L'association, à ces symptômes, d'une carence en folates peut faire orienter le diagnostic vers une ou plusieurs
des étiologies suivantes :
A - Anémie de Biermer
B - Insuffisance pancréatique externe
C - Maladie coeliaque
D - Maladie des laxatifs
E - Ulcère duodénal
Bonne(s) réponse(s) : C

Donne un tableau multicarentiel associant entre autres, carence martiale et en folates.

705
Exclusivement sur DOC - DZ : www.doc-dz.com NADJI 85
RESIDANAT EN POCHE TOME II
Cas Clinique en QCM
Un homme de 70 ans est hospitalisé pour syndrome anémique à lnstallation progressive. Traité par gastrectomie des 2/3 vingt
ans plus tôt pour ulcère, il n'a jamais plus souffert depuis. Le transit digestif est normal. Les selles de coloration normale
Depuis six mois, son alimentation comporte essentiellement des pommes de terre, des pâtes et des conserves de viande et
de poisson. L'examen montre un embonpoint conservé, un teint pâle. Hémogramme :
- hémoglobine : 7 g/dl
- hématocrite : 0.21 l/l
- érythrocytes : 1,8 x 10 exposant 12/l
- réticulocytes : 20 10 exposant 9/l
- leucocytes : 3 x 10 exposant 9/l (neutrophiles : 40%, éosinophiles : 2%, lymphocytes : 50 %, monocytes : 8%),
- plaquettes : 110.10 exposant 9/l
- Sur le frottis, présence de polynucléaires hypersegmentés. Bilirubine non conjuguée : 30 mcmol/l, bilirubine conjuguée : 5
mcmol/l.
- Sidérémie : 25 mcmol/l, coefficient de saturation de la sidérophiline : 0,50. Absence d'autoanticorps antiérythrocytes.

L'anémie peut être décrite comme :


A - Microcytaire hypochrome arégénérative
B - Microcytaire hypochrome régénérative
C - Normocytaire normochrome arégénérative
D - Macrocytaire normochrome régénérative
E - Macrocytaire normochrome arégénérative
Bonne(s) réponse(s) : E

VGM = Ht/Gr, CCMH = Hb/Ht.


VGM = 117 µ3 = macrocytose, CCMH = 0,33 normale.
Réticulocytes < 150 000/mm3 donc arégénérative.

Quel mécanisme est principalement à l'origine de l'anémie ?


A - Trouble de synthèse de l'hémoglobine
B - Trouble de synthèse de l'ADN
C - Raréfaction des cellules-souches hématopoïétiques
D - Destruction excessive des érythrocytes circulants
E - Pertes excessives par hémorragie
Bonne(s) réponse(s) : B

Cette anomalie entraîne, en l'absence d'anomalie de synthèse de l'Hb associée, un asynchronisme entre la maturation
nucléocytoplasmique et une macrocytose. Il existe une insuffisance médullaire qualitative, car une grande partie des cellules
souches mégaloblastiques n'arrive pas à terme (hémolyse intramédullaire). Toutes les cellules à renouvellement rapide sont
touchées par les défauts de maturation.

Parmi ces examens pouvant aider au diagnostic, lequel (lesquels) doit(doivent), pour être correctement
interprété(s), être effectué(s) avant tout traitement ?
A - Tubage gastrique
B - Myélogramme
C - Gastroscopie avec biopsie
D - Dosages de vitamine B12 et de folates dans le sang
E - Recherche d'anticorps anti-facteur intrinsèque
Bonne(s) réponse(s) : B D

Les deux étiologies les plus fréquentes sont les carences en vitamine B12 et en folates. Les dosages doivent être effectués
avant tout traitement, ainsi que le myélogramme, la mégaloblastose médullaire disparaissant en 24 à 48 heures après apport
de vitamine B12.

On apprend que la moelle est mégaloblastique et que le test de Schilling est normal. Quels sont les deux
facteurs qui, chez ce patient, peuvent contribuer à l'installation d'une anémie mégaloblastique ?
A - Carence d'apport en vitamine B12
B - Trouble d'absorption de la vitamine B12
C - Carence d'apport en folates
D - Trouble d'absorption des folates
E - Trouble d'utilisation des folates
Bonne(s) réponse(s) : C D

Le régime pratiqué par le patient est riche en B12 (viande, poisson) mais pauvre en folates (absence de légumes verts, de
fruits). La normalité du test de Schilling élimine un trouble d'absorption de la vitamine B12. Les troubles d'utilisation des folates
sont extrêmement rares.

706
Exclusivement sur DOC - DZ : www.doc-dz.com NADJI 85
RESIDANAT EN POCHE TOME II
Cas Clinique en QCM

Le traitement substitutif doit comporter :


A - Vitamine B12, intramusculaire, 1000 gammas par mois à vie
B - Vitamine B12, 100 gammas par jour, per os jusqu'à guérison de l'anémie
C - Acide folinique, intramusculaire, 5 mg par jour jusqu'à guérison de l'anémie
D - Acide folique, per os, 5 mg par jour jusqu'à guérison de l'anémie
E - Acide folique, per os, 50 mg par jour jusqu'à guérison de l'anémie
Bonne(s) réponse(s) : D

Les doses exactes sont de 3 x 5 mg (3 comprimés de Foldine®) par jour jusqu'à disparition totale des troubles
hématologiques et tant que la cause persiste. Il n'y a pas d'effets secondaires. La voie parentérale est réservée aux
traitements par médicaments antifoliques et aux grandes malabsorptions.

Avec un traitement substitutif correct, et en dehors de toute transfusion, on peut s'attendre à la normalisation
(12-13 g/dl) du taux d'hémoglobine aux :
A - 5ème jour
B - 10ème jour
C - 20ème jour
D - 50ème jour
E - 100ème jour
Bonne(s) réponse(s) : D

Durée = environ 2 mois.

Un homme de 50 ans est adressé pour polyglobulie, à la suite d'un hémogramme systématique. Il n'accuse aucun trouble
fonctionnel. A l'examen clinique : surpoids, érythrose discrète ; il n'y a pas de splénomégalie ni d'autre viscéromégalie.
Auscultation du coeur et des poumons normale ; tension artérielle 18/10. Il n'a pas d'antécédents pathologiques ; il boit 1/4 de
litre de vin et fume 20 cigarettes par jour.
A l'hémogramme :
- hémoglobine : 18,5 g/dl
- hématocrite : 0,55
- érythrocytes : 6,3 x 10 exposant 12/l
- leucocytes : 12 x 10 exposant 9/l (polynucléaires neutrophiles : 70 %, éosinophiles : 39 %, Iymphocytes : 17 %, monocytes :
10 %)
Vitesse de sédimentation : 2/4
- plaquettes : 350.10 exposant 9/l

Lequel des examens suivants doit être pratique en premier lieu ?


A - Myélogramme
B - Biopsie ostéomédullaire
C - Mesure isotopique du volume globulaire
D - Mesure de la Sa02
E - Dosage de l'oxycarbonémie
Bonne(s) réponse(s) : C

La définition de la polyglobulie est donnée par le volume globulaire > 36 ml/g chez les hommes et > 32 ml/kg chez les
femmes. Permet d'éliminer l'hémoconcentration.

Les examens pratiqués ont montré : myélogramme de densité et de compositions normales. Biopsie : moelle
modérément hyperplasique, sans fibrose. Volume globulaire : 38 ml/kg (limite acceptée : 36 ml/kg), Sa02 : 93
%, oxycarbonémie 1 %.
Quel(s) diagnostic(s) parmi les suivants peut-on exclure à ce stade de l'exploration ?
A - Thalassémie hétérozygote
B - Hémoconcentration chronique
C - Polyglobulie d'insuffisance respiratoire
D - Polyglobulie par cancer du rein
E - Maladie de Vaquez
Bonne(s) réponse(s) : A B C

A - Hb > 13 g/100 ml, absence de microcytose, VGM = Ht/Gr = 87 µ3.


B - Il s'agit d'une polyglobulie vraie (VG > 36 ml/kg).
C - Non, car SaO2 est normale.
D - L'UIV est l'examen indispensable pour diagnostiquer une polyglobulie secondaire (+ écho, scanner) par sécrétion
inappropriée d'érythropoïétine. Les gaz du sang sont de même pratiqués de façon systématique.
E - Diagnostic le plus probable.

707
Exclusivement sur DOC - DZ : www.doc-dz.com NADJI 85
RESIDANAT EN POCHE TOME II
Cas Clinique en QCM

Parmi les données suivantes, laquelle (lesquelles) est (sont) plutôt en faveur d'une maladie de Vaquez que
d'une polyglobulie secondaire ?
A - Valeur du volume globulaire total
B - Données de l'étude morphologique de la moelle
C - Le résultat de la biopsie médullaire
D - Taux des leucocytes
E - Valeur de la vitesse de sédimentation
Bonne(s) réponse(s) : D

Il s'agit d'un syndrome myéloprolifératif avec atteinte de toutes les lignées (hyperleucocytose, thrombocytose). La valeur du
volume globulaire total définit la polyglobulie vraie mais ne donne aucune orientation étiologique. La VS est basse, mais ce
n'est pas un signe spécifique.

Le patient ayant arrêté de fumer, et l'hématocrite restant inchangé, quelle(s) mesure(s) adopter ?
A - Injection de phosphore 32
B - Traitement par hydroxyurée (Hydrea®)
C - Traitement par Busulfan (Misulban®)
D - Traltement par Desferrioxamine (Desferal®)
E - Surveillance de l'hémogramme
Bonne(s) réponse(s) : E

A - Uniquement chez les sujet âgés avec facteurs de risque vasculaire. Risque leucémigène.
B - Associée aux saignées chez le sujet jeune.
C - Sujet âgé, facteurs de risque vasculaire.
D - Chélateur du fer (thalassémie).
E - Est un des signes d'hyperviscosité.

Un homme de 60 ans consulte pour dyspnée d'effort. Il existe une pâleur modérée ; pas d'anomalie à l'examen du coeur et
des poumons, pas d'adénopathie ou de splénomégalie.
A l'hémogramme :
- hémoglobine : 9 g/dl
- VGM : 90 fl
- leucocytes : 7.10 exposant 9/l (PN : 65 - PE : 2 - L : 30 - M : 3)
- plaquettes : 255.10 exposant 9/l
- Vitesse de sédimentation : 105 - 140
- fibrinogène : 3 g/l
- fer sérique : 20 micromol/l
- Protéinémie : 95 g/l dont 55 g/l de gammaglobulines en pic à base étroite.
L'immunoélectrophorèse montre une importante augmentation monoclonale d'lgG kappa. On retient, comme probable, le
diagnostic de myélome multiple.

Le(s) argument(s) contre le diagnostic d'anémie inflammatoire est (sont) ?


A - Hémoglobine : 9 g/dl
B - Leucocytes : 7,9.10 exposant 9/l
C - Fibrinogène : 3 g/l
D - Gammaglobulines : 35 g/l
E - Fer sérique : 20 micromol/l
Bonne(s) réponse(s) : C E

Le dosage de la ferritinémie et du coefficient de saturation de la sidérophilline permettent de différencier anémies


inflammatoires et ferriprives, où le fer sérique est chaque fois bas.
Le fibrinogène est normal et non augmenté comme lors de syndromes inflammatoires.

L'(s) argument(s) contre le diagnostic d'immunoglobuline monoclonale bénigne est (sont) :


A - Hémoglobine : 9 g/dl
B - Vitesse de sédimentation : 105 - 140
C - Fibrinogène : 3,7 g/l
D - Gammaglobulines : 55 g/l
E - Composant monoclonal IgG kappa
Bonne(s) réponse(s) : A D

Chez un sujet âgé bien portant, les arguments en faveur d'une Ig monoclonale bénigne sont une VS peu accélérée, un pic <
20 g/litre, l'absence de lésions osseuses, une plasmocytose médullaire < 5 %, la conservation des autres Ig, l'absence
d'anémie et d'hypercalcémie, l'absence de P.B.J et surtout l'évolution (surveillance+++).

708
Exclusivement sur DOC - DZ : www.doc-dz.com NADJI 85
RESIDANAT EN POCHE TOME II
Cas Clinique en QCM

Le diagnostic de maladie de Waldenstrِm n'est pas retenu en raison de :


A - Hémoglobine : 9 g/dl
B - Vitesse de sédimentation : 105 - 140
C - Fibrinogène : 3,7 g/l
D - Gammaglobulines : 55 g/l
E - Composant monoclonal IgG kappa
Bonne(s) réponse(s) : A B C D

Ne font pas partie des arguments de diagnostic différentiel (appartiennent à tous les tableaux de dysglobulinémie
monoclonale).
E - Il s'agit d'une immunoglobuline monoclonale de type IgM.

Chez ce malade, le diagnostic de myélome multiple sera confirmé au vu d'un ou de plusieurs des résultats
suivants, lequel(lesquels) ?
A - Présence de lésions ostéolytiques sur les radiographies du squelette
B - Hyperuricémie
C - Hypocalcémie
D - Protéinurie : 0,50 g/24 h, non sélective à l'électrophorèse
E - Plasmocytose médullaire supérieure à 20 %
Bonne(s) réponse(s) : A E

Les critères diagnostiques sont les suivants :


- radiologiques (présence de géodes)
- protidiques (Ig monoclonale, baisse des autres Ig, PBJ)
- médullaire (plasmocytes > 15 %).
B - Non spécifique.
C - Non, hypercalcémie (n'est pas un critère diagnostique, mais pronostique).
D - La PBJ est formée de chaînes légères à l'électrophorèse.

Le diagnostic de myélome multiple a été confirmé. Un traitement associant melphalan (Alkeran®) et prednisone
est instauré. Ce traitement expose à une ou plusieurs des complications suivantes. La(les)quelle(s) ?
A - Insuffisance médullaire
B - Neuropathie périphérique
C - Insuffisance cardiaque
D - Hyperglycémie
E - Leucémie aiguë secondaire
Bonne(s) réponse(s) : A D E

A - Toxicité médullaire réversible en 10 à 20 jours.


B - Concerne la vincristine.
C - Concerne les anthracyclines (Adriamycine®).
D - Complication de la corticothérapie.
E - Complication des alkylants (mélphalan).

Un homme de 35 ans consulte pour pesanteurs de l'hypochondre gauche. L'examen décèle une splénomégalie débordant de
5 cm, un foie ferme mesurant 14 cm sur la ligne médioclaviculaire. Il n'y a pas d'adénopathie, pas d'ictère., pas d'oedème, pas
d'ascite. Le patient n'est pas fébrile et n'a pas séjourné outre-mer. L'hémogramme montre : 3.500 000 GR ; hémoglobine 10,5
g/dl ; hématocrite 35 % ; GB 3.500 dont 65 % PN - 25 %, L - 10 % Mo ; 120.000 plaquettes.
La VS est à 10 mm.

Le tableau clinique est compatible avec un ou plusieurs des états suivants :


A - Hypertension portale
B - Leucémie myéloïde chronique
C - Syndrome Iymphoprolifératif
D - Hémolyse chronique
E - Maladie de surcharge
Bonne(s) réponse(s) : A C D E

B - Il existerait plutôt une hyperleucocytose avec myélémie (sans hiatus entre les formes jeunes) avec un pourcentage modéré
de blastes et une thrombocytose.

709
Exclusivement sur DOC - DZ : www.doc-dz.com NADJI 85
RESIDANAT EN POCHE TOME II
Cas Clinique en QCM

Dans les examens suivants, le(les)quel(s) vous sembl(en)t utile(s) pour éliminer l'hypothèse d'une hémolyse ?
A - Fer sérique
B - Numération des réticulocytes
C - Médullogramme
D - Dosage de la bilirubine non conjuguée
E - Scintigraphie splénique
Bonne(s) réponse(s) : B D

B - Permet de définir si l'anémie est régénérative si réticulocytes > 150 000 par mm3 (Ccomme dans l'hyperhémolyse).
D - Le troisième dosage utilisé est celui de l'haptoglobine effondrée dans l'hyperhémolyse.

L'hypothèse d'une hémolyse est écartée. Vous cherchez à étayer celle d'une hypertension portale : par quel(s)
examen(s) ?
A - Scintigraphie splénique
B - Scintigraphie hépatique
C - Fibroscopie gastrique
D - Tomodensitométrie abdominale
E - Mesure de l'élimination de la BSP
Bonne(s) réponse(s) : C

La fibroscopie permet de mettre en évidence les varices oesophagiennes ou cardiotubérositaires, signant l'hypertension
portale.

L'hypothèse d'une hypertension portale est exclue. Il vous reste alors à envisager quelle(s) hypothèse(s) parmi
les suivante(s) :
A - Leucémie myéloïde chronique
B - Leucémie à tricholeucocytes
C - Purpura thrombopénique idiopathique
D - Maladie de surcharge
E - Lymphome splénique
Bonne(s) réponse(s) : B D E

A - Voir question [100].


B - Syndrome lymphoprolifératif constitué de cellules à cytoplasme chevelu.
C - Argument en faveur d'une thrombopénie secondaire.
D - Type amylose, Nieman Pick, Gaucher.

Quel(s) examen(s) vous parai(ssen)t utile(s) pour étayer cette (ces) hypothèse(s) ?
A - Etude de la durée de vie des plaquettes
B - Etude de la durée de vie des hématies
C - Biopsie médullaire
D - Biopsie hépatique
E - Caryotype médullaire
Bonne(s) réponse(s) : C D

A.B - Si suspicion de destruction ou de séquestration.


C - Diagnostic de syndrome lymphoprolifératif.
D - Diagnostic de maladie de surcharge.
E - Recherche du chromosome Philadelphia (LMC).

Le patient, lassé par ces explorations, décide d'accepter une splénectomie proposée par un de vos confrères et
réalisée malgré votre avis défavorable. Les modifications hématologiques post-opératoires peuvent comporter :
A - Une thrombocytose à 800.000 plaquettes/mm3
B - La présence de corps de Jolly dans les hématies
C - Une Iymphopénie à 300/mm3
D - Une hyperéosinophilie à 1200/mm3
E - Une hyperleucocytose à 1 .000/mm3
Bonne(s) réponse(s) : A B E

A - Apparait au décours (48 heures), maximum 10 jours puis régression (environ 600 OOO/mm3).
B - Témoins d'une asthénie fonctionnelle.
C - Sans rapport.
D - Sans rapport.
E - Dans les suites immédiates.

710
Exclusivement sur DOC - DZ : www.doc-dz.com NADJI 85
RESIDANAT EN POCHE TOME II
Cas Clinique en QCM
Une jeune fille de 18 ans arrive en urgence avec un purpura des membres inférieurs (pétéchies et ecchymoses) ; à l'examen,
vous ne retrouvez pas d'hémorragie buccale, rien au fond d'oeil. Pas de grosse rate, ni d'adénopathie, ni de gros foie. Elle
n'absorbe aucun médicament. Il n'y a pas d'altération de l'état général, la température est normale. Elle a présenté une
rhinopharingite il y a 15 jours.
L'hémogramme est le suivant :
- globules rouges : 4 100 000/mm3
- hémoglobine : 12,9 g/dl
- hématocrite : 38,4 %
- globules blancs : 7 300/mm3 (formule normale)
- plaquettes : 5 000/mm3
- vitesse de sédimentation : 7 mm à la 1ère heure - 15 mm à la 2ème heure
La fibrinémie est à 3,3 g/l
Le diagnostic de purpura thrombopénique idiopathique (PTI) est évoqué.

Pour étayer le diagnostic de PTI, quel est l'examen le plus utile à faire ?
A - Anticorps anti-HLA
B - Recherche d'anticorps antiplaquettes
C - Recherche d'une coagulopathie de consommation
D - Myélogramme sternal
E - Temps de saignement
Bonne(s) réponse(s) : D

Premier geste permettant de différencier atteintes centrales et périphériques, mais ne permet pas de faire la différence entre
les diverses étiologies d'atteinte périphérique. C'est l'examen le plus utile, mais il est insuffisant.

Quelle(s) étiologie(s) convient-il de rechercher, à l'aide de tests complémentaires, avant d'affirmer qu'il s'agit
bien d'un PTI ?
A - Lupus érythémateux aigu
B - Purpura par oestro-progestatif
C - Mononucléose infectieuse
D - Rubéole
E - Déficit de l'agrégation des plaquettes
Bonne(s) réponse(s) : A C D

A - Thrombopénie auto immune secondaire (syndrome d'Evans).


B - Pas d'effet secondaire de ce type décrit.
C - Thrombopénie virale classique.
D - Thrombopénie virale classique.
E - S'accompagne parfois d'une thrombopénie modérée.

Le diagnostic de PTI ayant été confirmé, quel est le choix thérapeutique de première intention ?
A - Prednisone
B - Gammaglobulines IV : 0,4 g/kg/j
C - Splénectomie
D - Transfusions de plaquettes : 6 unités plaquettaires
E - Plasmaphérèse
Bonne(s) réponse(s) : A

A - A la dose d'1mg/kg/jour.
B - Efficacité transitoire.
C - Si échec de A.
D - Si risque hémorragique aigu.
E - Pas d'utilisation classique.

L'évolution peut se faire vers :


A - Guérison définitive en 3 semaines
B - Rechute immédiate à l'arrêt du traitement
C - Réapparition de la maladie après une longue phase de guérison apparente
D - Apparition d'une leucémie aiguë Iymphoblastique
E - Apparition d'une aplasie médullaire
Bonne(s) réponse(s) : A B C

PTI corticosensible ou dépendant. Les rechutes à distance sont rares mais possibles.

711
Exclusivement sur DOC - DZ : www.doc-dz.com NADJI 85
RESIDANAT EN POCHE TOME II
Cas Clinique en QCM
Monsieur P. Léon, 71 ans, vient consulter en raison de l'apparition, à plusieurs reprises, d'ecchymoses larges provoquées par
des traumatismes minimes, des gingivorragies intermittentes, des hématomes aux points d'injections intramusculaires
d'antibiotiques, prescrits pour un fébricule chronique rebelle. Sur le plan clinique, on note, outre une pollakiurie, une
augmentation de volume de la prostate, un amaigrissement de 3 kg, une asthénie. Le diagnostic de cancer de la prostate a
été affirmé. Le bilan biologique que vous avez demandé donne les résultats suivants :
- temps de saignement (Ivy) : 15 min
- temps de Quick : 40 %
- temps de céphaline activé : 52 sec (témoin : 33 sec)
- V : 35 %
- VII + X : 75 %
- II : 40 %
- fibrinémie : 0,30 g/l
- PDF : 80 mcg/ml
- test à l'éthanol : positif
- vitesse de sédimentation : 60 mm 1ère heure
- hémoglobine : 11,5 g/dl
- globules blancs : 11 000/mm3 dont 80 % de polynucléaires
- Plaquettes : 80 000/mm3

Parmi les diagnostics suivants, indiquez celui qui est le plus probable pour expliquer toutes les anomalies de
l'hémostase :
A - Thrombopénie par aplasie médullaire
B - Coagulopathie de consommation
C - Insuffisance hépatique
D - Hypofibrinémie constitutionnelle
E - Carence sévère en vitamine K
Bonne(s) réponse(s) : B

A - Pas de pancytopénie ; n'expliquerait pas le tableau.


B - Correspond au tableau, contexte évocateur de néo-prostatique.
C - Dans l'insuffisance hépatique, le test à l'éthanol est négatif, la thrombopénie est modérée, les facteurs VII et X sont
diminués de façon prédominante par rapport aux facteurs II et V.
D - Pas de thrombopénie, pas de complexes solubles ni de PDF.
E - Pas de thrombopénie, fibrine normale, pas de PDF ni de complexes solubles.

Quel est le mécanisme le plus probable susceptible d'expliquer l'anomalie de la coagulation présentée par ce
malade ?
A - Altération diffuse de l'endothélium vasculaire
B - Transformation du fibrinogène en fibrine sans formation de thrombine
C - Déficit acquis en antithrombine III
D - Hyperactivité fibrinolytique
E - Activation directe de la coagulation par des cellules cancéreuses
Bonne(s) réponse(s) : E

Activation de la voie exogène de la coagulation par les thromboplastines tissulaires libérées par destruction des cellules par la
tumeur.

Avant un prélèvement biopsique à visée diagnostique, quelle prescription de substitut faites-vous pour préparer
le geste ?
A - Plasma frais congelé + fibrinogène
B - PPSB + fibrinogène
C - Concentrés d'antithrombine III
D - Transfusion de plaquettes
E - Facteur VIII concentré
Bonne(s) réponse(s) : A

Le PFC apporte tous les facteurs de la coagulation sauf les plaquettes. Le PPSB est formellement contre-indiqué (apportant
des facteurs activés entretenant la CIVD). Doit être complété par apport de plaquettes (> 50 OOO/mm3).

En dehors du traitement substitutif, ce malade est justifiable d'un traitement par :


A - Acide epsilon aminocaproïque (Hémocaprol®)
B - Corticothérapie (1 mg/kg/jour)
C - Antivitamine K
D - Vitamine KIV (20 mg matin et soir)
E - Oestrogénothérapie
Bonne(s) réponse(s) : E

Traitement palliatif d'un cancer de la prostate probablement disséminé (LIND).

712
Exclusivement sur DOC - DZ : www.doc-dz.com NADJI 85
RESIDANAT EN POCHE TOME II
Cas Clinique en QCM
Un jeune homme âgé de 20 ans, sans antécédents personnels pathologiques, a été opéré d'une appendicectomie banale. Il
présente, 4 jours plus tard, des signes cliniques et paracliniques d'embolie pulmonaire sans collapsus cardiovascu!aire. La
phlébographie découvre une thrombose veineuse fémoro-poplitée à droite. Un traitement anticoagulant par l'héparine est
entrepris ; il est nécessaire d'administrer 700 U/kg/24 h pour équilibrer le traitement. L'interrogatoire approfondi vous apprend
l'existence d'un contexte familial de thrombose. Du côté maternel, la grand-mère est décédée d'une embolie pulmonaire, 8
jours après la mise en place d'une prothèse totale de hanche ; du côté paternel, une soeur (tante du propositus) a présenté à
deux reprises une phlébite après accouchement et l'un de ses deux enfants, un garçon de 8 ans, a présenté une phlébite à
l'occasion d'une immobilisation plâtrée.

Parmi les résultats biologiques suivants, recueillis 48 h après l'acte opératoire, le(les)quel(s) aurai(en)t pu vous
faire prévoir cette maladie thrombo-embolique ?
A - Temps de céphaline active : 28 sec (T = 31 sec)
B - Plaquettes : 380 000/mm3
C - Fibrinogène : 4, 5 g /l
D - Produits de dégradation du fibrinogène : 20 mcg/ml
E - Aucun des examens ci-dessus
Bonne(s) réponse(s) : E

Les quatre premières données sont normales ou subnormales.

Cinq jours après avoir commencé l'héparinothérapie, vous demandez un dosage d'antithrombine III. Le résultat
est à 60 %. Le(les)quel(s) des commentaire(s) suivant(s) est (sont) juste(s) ?
A - Le taux est dans les limites de la normale pour un homme de 20 ans
B - Un déficit congénital peut être suspecté
C - Ce déficit peut être en rapport avec le traitement par l'héparine
D - Ce déficit est normal en post-opératoire
E - Ce déficit traduit une hypovitaminose K
Bonne(s) réponse(s) : B C

Ce déficit peut être congénital ou acquis (héparine, oestroprogestatif, syndrome néphrotique l'asparaginase, CIVD,
insuffisance hépatique).

En faveur d'un déficit congénital en antithrombine III, vous retenez :


A - Existence d'un déficit en antithrombine III chez les ascendants et collatéraux des propositus
B - Persistance du déficit sous antivitamine K
C - Persistance de déficit après sevrage du traitement par l'héparine
D - Persistance du déficit après injection IV de vitamine K
E - Persistance de déficit après injection de 3 unités de plasma frais congelé
Bonne(s) réponse(s) : A B C

Transmission autosomique dominante. Correction du déficit par injection de PFC ou d'AT III concentré.

Le déficit congénital en antithrombine III et de l'ensemble des éléments biologiques en votre possession ont
finalement pu être écarté : compte-tenu du contexte familial, quelle(s) autre(s) pathologie(s) de l'hémostase
devez-vous encore rechercher ?
A - Déficit congénital en facteur XII
B - Anticoagulant circulant type lupus
C - Déficit congénital en protéine C
D - Une augmentation de l'agrégation des plaquettes
E - Une augmentation du facteur Willebrand
Bonne(s) réponse(s) : C

Etiologie des maladie thromboemboliques familiales :


- déficit en protéine C
- déficit en protéine S
- déficit en antithrombine III.

713
Exclusivement sur DOC - DZ : www.doc-dz.com NADJI 85
RESIDANAT EN POCHE TOME II
Cas Clinique en QCM
Madame X. 27 ans, est admise d'urgence pour une fièvre à 40°C avec frissons ayant débuté 48 heures plus tôt. Un
hémogramme réalisé le jour même montre :
- hématies : 3,9 x 10 exposant 12/l
- hémoglobine : 11,8 g/dl
- VGM : 94 microns3
- leucocytes : 1 000/mm3, polynucléaires neutrophiles : 1 %
Iymphocytes : 95 %
monocytes : 4 %
- plaquettes : 140 x 10 exposant 9/l
En dehors d'une angine ulcéronécrotique, l'examen clinique est négatif et ne retrouve ni hépato, ni splénomégalie, ni
adénopathie superficielle. Devant ces résultats, on a été amené à évoquer le diagnostic d'agranulocytose probablement
médicamenteuse et l'interrogatoire orienté à révélé :
- la prise depuis 5 ans d'un contraceptif oral minidosé
- la prescription d'un antalgique contenant de la noramidopyrine pour troubles des règles il y a 6 mois, interrompu
définitivement 3 semaines plus tôt
- la prise, il y a 4 jours, à l'occasion d'un syndrome grippal avec fièvre résolutive en 48 heures, d'un traitement non précisé
(cachets)
- la réapparition de la fièvre 48 heures plus tard, à 40°C, qui a conduit le médecin traitant à prescrire un traitement de
pénicilline par voie intramusculaire, d'aspirine vitamine C par voie orale, et un hémogramme réalisé le matin même.

Concernant la réalisation éventuelle du myélogramme, laquelle(lesquelles) des propositions suivantes vous


parait (paraissent) justifiée(s) ?
A - Est inutile
B - Ne doit être pratiqué qu'en cas d'agranulocytose persistante
C - Est contre-indiqué en raison du risque infectieux local
D - Est indispensable
E - Doit être remplacé par une biopsie médullaire
Bonne(s) réponse(s) : D

Permet de mettre en évidence soit une absence totale de granulocytes, soit un début de régénération médullaire.

Parmi les médicaments pris par la malade, lequel est probablement en cause ?
A - Le médicament contenant de la noramidopyrine pris il y a 3 semaines
B - La pénicilline par voie intramusculaire la veille de la découverte de la granulocytose
C - Le médicament de nature non précisée
D - Le contraceptif oral
E - L'aspirine-vitamine C
Bonne(s) réponse(s) : C

A - La prise remonte à 3 semaines (trop ancienne).


B - Anémie immunoallergique.
C - Interrogatoire policier nécessaire.
E.D. - Ne provoquent pas d'agranulocytose.

Quelle modalité d'antibiothérapie choisissez-vous pour cette patiente ?


A - Cycline par voie intraveineuse
B - Bêta-lactamine + aminoside
C - Pénicilline + Colimycine®
D - Chloramphénicol + macrolide
E - Bactrim® ou Eusaprim® intraveineux
Bonne(s) réponse(s) : B

Antibiothérapie par voie parentérale à large spectre après prélèvement (hémoculture).

Cette prescription doit être éxécutée :


A - Dès l'arrivée de la patiente
B - Après hémocultures
C - Après résultat des hémocultures
D - Après obtention d'un antibiogramme
E - En cas d'inefficacité des transfusions de globules blancs
Bonne(s) réponse(s) : B

Permettant d'isoler éventuellement le germe en cause, mais ne doivent pas retarder le traitement (ne pas attendre le résultat).

714
Exclusivement sur DOC - DZ : www.doc-dz.com NADJI 85
RESIDANAT EN POCHE TOME II
Cas Clinique en QCM

Après interruption du(des) médicament(s) présumé(s) responsable(s), pensez-vous qu'une agranulocytose :


A - Régresse spontanément habituellement en 1 à 2 mois
B - Peut évoluer vers une leucémie aiguë
C - Régresse spontanément habituellement en moins de 15 jours
D - Régresse plus rapidement grâce à un traitement spécifique
E - Peut persister sous forme d'une neutropénie chronique définitive
Bonne(s) réponse(s) : C

Le risque majeur est l'infection pendant la période d'agranulocytose.

La confirmation de la responsabilité du médicament peut être obtenue par :


A - Test de transformation lymphoblasique
B - Test de dégranulation des basophiles
C - Test d'inhibition de migration des macrophages
D - Dosage des réagines spécifiques
E - Aucun des examens précédents
Bonne(s) réponse(s) : E

La réintroduction est bien sûr formellement contre-indiquée.

Une patiente de 35 ans, mère de 4 enfants dont le dernier à 6 ans, est soumise à une contraception par stérilet depuis sa
dernière grossesse. Elle est donneuse de sang régulière, deux fois par an.
Elle utilise volontiers des laxatifs huileux pour vaincre une constipation fonctionnelle. Elle est pâle, fatiguée et se plaint d'une
difficulté de déglutition avec sensation de striction pharyngée. La langue est dépapillée, les ongles fins et fragiles.
L'hémogramme montre :
- hématies : 4.000.000/mm3
- hémoglobine : 7 g/100 ml
- hématocrite : 25 %
- plaquettes : 600.000/mm3
- leucocytes : 6 500/mm3 dont 70 % de neutrophiles, 20 % de Iymphocytes, 10 % de monocytes.
La sidérémie est à 4 micromol/l, la capacité totale de transferrine à 100 micromol/l, les réticulocytes à 0,1 %.

Parmi les symptômes suivants, l'hyposidérémie peut être mise en relation directe avec :
A - Langue dépapillée
B - Lithiase oxalique
C - Constipation
D - Ongles fragiles
E - Thrombocytose
Bonne(s) réponse(s) : A D E

A - Signe d'atrophie des muqueuses digestives (glossite, dysphagie, brûlures oesophagiennes ou épigastriques).
B - Sans rapport.
C - L'hyposidérémie peut donner les troubles digestifs cités à l'item A, mais est sans relation avec la constipation.
D - Les ongles sont mous, cassants, concaves (koïnolychie).
E - Fréquemment associée.

Pour préciser l'étiologie de cette anémie, il est nécessaire de demander :


A - Hématies leucocytes/minute
B - Médullogramme
C - Electrophorèse de l'hémoglobine
D - Etude de la cinétique du 59 fer
E - Aucun des examens précédents
Bonne(s) réponse(s) : E

Les étiologies des anémies ferriprives sont :


- perte de fer: d'origine digestive ou gynécologique (90 % des cas) essentiellement
- plus rarement: prélèvements de sang répétés, saignements provoqués, le contexte implique des examens mettant en
évidence une perte de fer en particulier examen gynécologique
- troubles de l'absorption: résection digestive, maladie coeliaque, pica
- majorations des besoins : nourrisson, croissance, règles abondantes, grossesses répétées et
rapprochées.

715
Exclusivement sur DOC - DZ : www.doc-dz.com NADJI 85
RESIDANAT EN POCHE TOME II
Cas Clinique en QCM

Parmi les données anamnésiques, quel(s) facteur(s) intervien(nen)t dans la genèse de l'anémie ?
A - 4 grossesses
B - Contraception par stérilet
C - Dons du sang
D - Prise de laxatifs
E - Constipation
Bonne(s) réponse(s) : A C D

En période d'activité génitale, environ 40 % des femmes ont un bilan de fer négatif et 8 à 10 % une anémie ferriprive. Les
grossesses surtout répétées, accentuent le risque de carence. Les hémorragies liées au stérilet et les dons de sang
participent à la genèse de l'anémie.
D.E - Sans rapport.

Le déficit en fer chez cette patiente peut être estimé à :


A - 2 mg
B - 20 mg
C - 200 mg
D - 2 000 mg
E - 20 000 mg
Bonne(s) réponse(s) : D

Un adulte de 70 kg a environ 4 gr de fer dans l'organisme, répartis en :


- fer héminique : hémoglobine (75 % du total), myoglobine (5 %), enzyme (0,5)
- fer non héminique : de réserve (25 %) foie, rate, moelle, hémosidérine, ferritine
- fer plasmatique (0,1 %).

La dose totale de fer (exprimée en fer métal et administrée per os) pour corriger ce déficit devrait être de :
A - 2 mg
B - 20 mg
C - 200 mg
D - 2 000 mg
E - 20 000 mg
Bonne(s) réponse(s) : E

Soit 100 à 250 mg par jour pendant deux mois, puis deux mois à demi dose pour reconstituer
les réserves.

La dose usuelle quotidienne de fer per os, exprimée en fer métal, est :
A - 1 mg
B - 5 mg
C - 10 mg
D - 50 mg
E - 100 mg
Bonne(s) réponse(s) : E

100 mg par jour sont nécessaires pour corriger le déficit, certains préconisent même des doses plus fortes pour la raison
suivante : 50 mg est la dose nécessaire pour obtenir une réponse médullaire optimale, mais comme 20 % du fer per os au
plus est absorbé, 250 mg/jour est la dose à prescrire par jour.

Ce traitement poursuivi pendant 3 mois peut entraîner :


A - Coloration noire des selles
B - Coloration rouille des urines
C - Constitution d'une hémosidérose splénique
D - Pigmentation cutanée d'aspect métallique
E - Dépôts de fer cornéens
Bonne(s) réponse(s) : A

Il faut prévenir le patient des effets secondaires :


- coloration des selles en noir, constipation, diarrhée, nausées, brûlures abdominales
- céphalées, vertiges
La pigmentation cutanée au point de piqûre peut survenir en cas de traitement parentéral, d'indication exceptionnelle
(inefficacité du fer per os dans les grandes malabsorptions).

716
Exclusivement sur DOC - DZ : www.doc-dz.com NADJI 85
RESIDANAT EN POCHE TOME II
Cas Clinique en QCM
Une femme de 45 ans présente une anémie ferriprive secondaire à des ménorragies compliquant l'évolution d'un fibrome
utérin. Elle reçoit le soir de son hospitalisation une transfusion d'un concentré globulaire. L'examen préopératoire fait par
l'anesthésiste le lendemain matin constate l'existence d'un subictère. Le reste de l'examen clinique est normal. La tension est
de 12/7, la diurèse normale, la température de 36,8°C.
L'examen biologique donne les résultats suivants :
- Hématies : 32 x 10 exposant 12/l
- hémoglobine : 65 grs/l
- hématocrite : 28 %
- Groupe sanguin : A-rhésus positif
- Billirubine totale : 22 micromoles
- urée sanguine : 7 mmoles
- créatinine sérique : 120 micromoles
Le contrôle du test prétransfusionnel de comptabilité fait au lit du malade prouve que le sang transfusé était bien isogroupe.

Lequel des diagnostics suivants est le plus probable ?


A - Ictère infectieux
B - Ictère rétentionnel
C - lctère hémolytique par hémolyse intravasculaire transfusionnelle
D - Ictère hémolytique par hémolyse intratissulaire transfusionnelle
E - Ictère hémolytique auto-immun
Bonne(s) réponse(s) : D

Il s'agit très probablement d'un accident transfusionnel minime, soit dans le système rhésus (anticorps acquis lors de
transfusion antérieures ou de grossesses), soit dans d'autres systèmes (Kell, P, Duffy...). L'hémolyse est intratissulaire, plus
rarement intravasculaire.

Indiquez celui(ceux) des examens suivants qui vous parai(ssen)t nécessaire compte tenu du fait que cette
malade doit subir une hystérectomie :
A - Recherche d'anticorps anti-HLA
B - Détermination des phénotypes rhésus et Kell
C - Recherche et identification des anticorps sériques
D - Test de Coombs direct
E - Recherche d'anticorps anti-plaquettes
Bonne(s) réponse(s) : B C D

Ces déterminations permettent d'éviter les accidents transfusionnels ultérieurs en sélectionnant les sangs compatibles.
Le test de Coombs direct permet de rechercher les anticorps fixés sur les globules rouges du sujet. Le test indirect permet de
mettre en évidence des anticorps dans un sérum à étudier. La recherche d'anticorps anti-HLA et d'anticorps anti-plaquettes se
fait en cas de nécessité de transfusions de leucocytes ou de plaquettes.

En cas de nouvelle transfusion en cours d'intervention, quel produit sanguin conseillez-vous ?


A - Sang total standard isogroupe
B - Concentré érythrocytaire déleucocyté
C - Concentré érythrocytaire choisi après test de comptabilité fait au laboratoire (Cross-match)
D - Concentré érythrocytaire décongelé
E - Albumine humaine
Bonne(s) réponse(s) : C

Il s'agit de culots érythrocytaires phénotypés compatibles avec le sang de la patiente


permettant d'éviter les accidents d'hémolyse mineurs.

En cas de négativité des tests immunologiques, que conseillez-vous ?


A - Recherche des antigènes et anticorps de l'hépatite B
B - Hémoculture
C - Dosage des bilirubines libres et conjuguées
D - Répétition des examens immuno-hématologiques 10 jours plus tard
E - Remise de l'intervention chirurgicale
Bonne(s) réponse(s) : E

A - Eliminé en raison de la durée d'incubation.


B - Non, absence de fièvre ou de signes infectieux.
C - N'apportant pas d'éléments d'orientation thérapeutique.
D - Elle est indispensable, le taux des anticorps étant parfois trop faible pour une détection précoce.
E - Il s'agit d'une intervention à risque hémorragique chez une patiente anémique. Il est donc indispensable de déterminer la
cause de l'hémolyse afin de transfuser des culots compatibles , phénotypés (risque d'accidents plus graves dans le cas
contraire). L'intervention doit donc être, repoussée.
Un délai de 34 jours est suffisant pour la positivation des tests immunologiques.
Commentaires :
Question litigieuse, E et D sont indispensables, mais par ordre pratique, E plus que D.

717
Exclusivement sur DOC - DZ : www.doc-dz.com NADJI 85
RESIDANAT EN POCHE TOME II
Cas Clinique en QCM

Malgré ces incidents, la malade a été opérée sans complication immédiate, quel traitement ultérieur de l'anémie
proposez-vous ?
A - Abstention thérapeutique
B - Sel ferreux par voie orale
C - Sel ferreux et acide folique per os
D - Fer injectable
E - Transfusions de sang total
Bonne(s) réponse(s) : B

Il faut recharger en fer la patiente, per os, bien sûr.

Un homme de 60 ans, gros fumeur, hypertendu, est hospitalisé pour malaise.


Dans les antécédents, on retrouve des céphalées depuis quelques mois L'examen met en évidence une hypertension
artérielle à 18-12, une rate débordant de 5 cm sous le rebord costal, une distension thoracique, un foie de 13 cm de hauteur
ferme, une érythrose des extrémités, une langue dépapillée.
L'examen neurologique est normal.
L'hémogramme montre :
- GR : 6 500 000/mm3
- hématocrite : 60 %
- leucocytes : 16 000/mm3 dont 60 % de PN ; 0 % de PE ; 0 % de PB ; 30 % de L ; 10 % de M,
- plaquettes : 600 000/mm3
- VS : 1-2.

Quelle(s) est(sont) la(les) manifestation(s) clinique(s) expliquée(s) par les anomalies de l'hémogramme ?
A - Aucune
B - Céphalées
C - Erythrose
D - Langue dépapillée
E - Distension thoracique
Bonne(s) réponse(s) : B C

B - En relation avec l'hyperviscosité.


C - Signe de polyglobulie.
D - Sans rapport.
E - Sans rapport.

Quel est, parmi ces examens, celui que vous allez pratiquer pour affirmer le diagnostic de polyglobulie ?
A - Mesure du volume globulaire moyen
B - Mesure du taux d'hémoglobine
C - Mesure du volume plasmatique à l'albumine
D - Myélogramme
E - Mesure du volume globulaire au Chrome 51
Bonne(s) réponse(s) : E

La mesure de la masse globulaire totale par isotopes (CRS l) fait seule le diagnostic de polyglobulie vraie.

Ce patient a une polyglobulie vraie. Compte-tenu de l'examen clinique, quel(s) est(sont) le(les) examen(s) que
vous allez pratiquer pour exclure les étiologies de polyglobulie secondaire ?
A - Scintigraphie pulmonaire
B - Gaz du sang en air ambiant
C - Urographie intra-veineuse
D - Uricémie
E - Tomodensitométrie cérébrale
Bonne(s) réponse(s) : B C

A - N'est pas un examen suffisant pour faire le diagnostic de polyglobulie secondaire à une cause respiratoire.
B - Examen indispensable devant toute polyglobulie (diagnostic des causes respiratoires).
C - Idem B. Causes urologiques.
D - L'hyperuricémie fait partie du tableau de maladie de Vaquez.
E - L'hypertrophie splénique constatée dans la maladie de Vaquez est un argument important mais non constant.

718
Exclusivement sur DOC - DZ : www.doc-dz.com NADJI 85
RESIDANAT EN POCHE TOME II
Cas Clinique en QCM

Quel est le diagnostic le plus vraisemblable en fonction des données disponibles ?


A - Une insuffisance respiratoire de type restrictif
B - Une conséquence directe du tabagisme
C - Un syndrome myéloprolifératif
D - Un cancer du rein
E - Un hépatome
Bonne(s) réponse(s) : C

A - N'explique pas la splénomégalie.


B - Idem item A.
C - En raison de la splénomégalie, de l'hyperleucocytose, de la thrombocytose.
D - N'explique pas la splénomégalie.
E - Idem item D.

Un homme de 32 ans consulte pour une discrète baisse de l'état général et une pesanteur de l'hypochondre gauche.
L'examen met en évidence une splénomégalie débordant de 4 centimètres le rebord costal gauche. Il n'y a aucune autre
anomalie clinique. La numération formule sanguine est la suivante :
- G.R. : 4 700 000/mm3
- Hb : 13,5 g/dl
- Ht : 44 %
- G B : 87 000/mm3, PN : 38 %, PE : 2 %, PB : 13 %, L : 5 %
- Myéloblastes : 2 %
- Promyélocytes : 5 %
- Myélocytes : 19 %
- Métamyélocytes : 16 %
- Plaquettes : 485 000/mm3.

Quel est le diagnostic vraisemblable chez ce patient ?


A - Leucémie lymphoïde chronique
B - Leucémie myéloïde chronique
C - Leucémie aiguë Iymphoblastique
D - Leucémie aiguë myéloblastique
E - Leucémie à tricholeucocytes
Bonne(s) réponse(s) : B

A - Non, ce n'est ni le terrain (plutôt homme de la soixantaine), ni la présentation clinique (polyadénopathie +).
B - Tableau caractéristique : altération de l'état général, splénomégalie, hyperleucocytose avec myélémie, thrombocytose.
C - Non, pourcentage de blastes < 30 %.
D - Non, idem item C.
E - Non, il s'agit d'une splénomégalie avec prolifération sanguine de tricholeucocytes.

Votre diagnostic sera confirmé par le résultat de :


A - L'examen cytologique de la moelle
B - Le groupage HLA
C - L'immunoélectrophorèse
D - Le caryotype médullaire
E - L'examen tomodensitométrique de la rate
Bonne(s) réponse(s) : D

A - Montre une moelle riche constituée à 80-90 % de cellules de la lignée granuleuse à tous les stades de la maturation.
Permet l'étude cytogénétique.
B - Sans intérêt diagnostic.
D - Met en évidence le chromosome Philadelphie, anomalie acquise, spécifique, présente dès le début de la maladie
(translocation réciproque chromosomes 22/9) et à tous les stades, dans 90 % des cas, permet un diagnostic précoce.
E - Sans intérêt diagnostic.

Le diagnostic de leucémie myéloïde chronique est retenu, quelle est la durée médiane de survie dans cette
affection ?
A - Identique à la population saine du même âge
B - 3 à 6 mois
C - 1 à 2 ans
D - 2 à 4 ans
E - 5 à 10 ans
Bonne(s) réponse(s) :

Médiane de survie : 3 ans (peut aller jusqu'à 10 ans).

719
Exclusivement sur DOC - DZ : www.doc-dz.com NADJI 85
RESIDANAT EN POCHE TOME II
Cas Clinique en QCM

Parmi les complications suivantes, on pourra observer chez ce malade :


A - Lithiase rénale
B - Priapisme
C - Prurit
D - Polynévrite des membres inférieurs
E - Infarctus splénique
Bonne(s) réponse(s) : A B C E

A - Par hyperuricémie.
B - Par thrombose veineuse.
C - Parfois annonciateur d'acutisation.
D - Ne fait pas partie des complications.
E - Par thrombose artérielle. Les autres complications sont les hémorragies, de rares infections, l'insuffisance médullaire
celles du traitement, la myélofibrose. Les deux principales causes de décès sont les thromboses ou les hémorragies, en
dehors de l'acutisation.

Parmi les évolutions suivantes, laquelle(lesquelles) est(sont) susceptible(s) de survenir chez ce patient ?
A - Transformation en leucémie aiguë lymphoblastique
B - Transformation en leucémie aiguë myéloblastique
C - Transformation en maladie de Hodgkin
D - Myélofibrose
E - Transformation en leucémie lymphoïde chronique
Bonne(s) réponse(s) : A B D

A.B. - Acutisation suspectée devant une altération de l'état général une fièvre isolée, des sueurs, un prurit, une augmentation
rapide de la rate et du foie, l'apparition de foyers blastiques ; confirmée par la présence de plus de 30 % de blastes dans le
sang ou la moelle. Le type est précisé par l'étude cytologique et cytochimique.
C - Sans rapport.
D - Complication classique.
E - Sans rapport.

Cette maladie ouvre droit à une déclaration de maladie professionnelle pour :


A - Mineur de charbon
B - Manipulateur en électroradiologie
C - Peintre au pistolet
D - Travailleur de l'aniline
E - Soudeur à l'arc
Bonne(s) réponse(s) : B C

Les sujets exposés sont ceux qui sont confrontés au benzène et aux radiations.
A - Risque de silicose et d'anthracose.
B - Exposition aux radiations.
C - Exposition au plomb, mais utilisation parfois de diluants benzéniques.
D - Exposition aux colorants aromatiques.
E - Exposition au plomb.

Un homme de 21 ans a appelé, il y a 2 jours, son médecin traitant pour fièvre à 39°C et des douleurs pharyngées apparues la
veille. Ce malade consomme depuis 3 ans différentes spécialités contenant des antalgiques, en raison de maux dentaires. La
numération formule sanguine (NFS) montre :
- G R : 33 800 000 par mm3
- Globules Blancs : 1.100 par mm3 PN : 19 %, PE : 0 %, PB : 0 %, L : 90 %
- plaquettes : 250.000 par mm3
Le tableau s'aggrave avec altération de l'état général et douleurs abdominales et le malade est hospitalisé. Il existe une
angine ulcéronécrotique et un abdomen sensible. La NFS faite en urgence confirme les résultats précédents.
Le diagnostic d'agranulocytose aiguë médicamenteuse est suspecté.

Parmi les substances antalgiques suivantes quelle(s) est (sont) celle(s) qui est(sont) connue(s) pour donner une
agranulocytose aiguë ?
A - Acide acétylsalicylique
B - Amidopyrine
C - Acide glafénique
D - Paracétamol
E - Clométacine
Bonne(s) réponse(s) : B

A - Antiagrégant plaquettaire.
B - Etiologie classique (Avafortan®, Baralgine®, Salgydall®, Optalidon®...).
C - Néphrotoxicité.
D - Hépatotoxicité à forte dose.
E - Hépato et néphrotoxicité.

720
Exclusivement sur DOC - DZ : www.doc-dz.com NADJI 85
RESIDANAT EN POCHE TOME II
Cas Clinique en QCM

Devant les données de l'énoncé introductif quel(s) est(sont) le(s) diagnostic(s) différentiels(s) possible(s) ?
A - Diphtérie maligne
B - Mononucléose infectieuse
C - Toxoplasmose
D - Leucémie aiguë myéloblastique
E - Déficit immunitaire T
Bonne(s) réponse(s) : D

A - N'entraîne pas d'agranulocytose.


B - Donnerait plutôt une hyperleucocytose modérée avec inversion de la formule, neutropénie absolue modérée et
lymphocytes normaux cotoyant de grands mononucléaires bleutés.
C - Ne donne pas d'angine.
D - Diagnostic à confirmer ou écarter par myélogramme (si oui, présence de plus de 30 % de blastes).
E - Ne touche pas la lignée granuleuse.

Un myélogramme est pratiqué. Parmi les résultats suivants, lequel est le plus probable sachant que le
médicament responsable a été arrêté il y a 5 jours ?
A - Moelle normale
B - Moelle avec absence complète de la lignée granuleuse
C - Moelle aplasique
D - Moelle avec blocage au stade promyélocytaire
E - Moelle normale avec réaction lymphoplasmocytaire
Bonne(s) réponse(s) : E

En cas d'agranulocytose toxique, après arrêt du médicament responsable, la guérison est obtenue en 5 à 15 jours sous
traitement symptomatique, parfois trois semaines. Le myélogramme initial peut montrer soit une absence totale de précurseur
granuleux, soit un aspect de "pseudo blocage de maturation". La guérison est marquée, au niveau médullaire, par
l'enrichissement en petits lymphocytes. La plasmocytose médullaire est fréquente.

Parmi les mécanismes physiopathologiques suivants, quel est celui qui rend compte de l'accident
d'agranulocytose aiguë médicamenteuse ?
A - Iso-immunisation anti-HLA
B - Réaction anaphylactique
C - Hypersensibilité retardée
D - Autoimmunisation anti-leucocytes
E - Aucun des mécanismes ci-dessus
Bonne(s) réponse(s) : E

Les deux mécanismes en cause sont la toxicité (cytotoxicité directe sur la lignée granuleuse) et l'immuno-allergie (fixation du
complexe antigène-médicament sur la surface des cellules de la lignée granuleuse et destruction).

Quand peut-on espérer le retour à la normale de la numération formule sanguine à partir de la date d'arrêt du
médicament responsable ?
A - Date impossible à préciser
B - Plusieurs mois
C - 3 semaines à 2 mois
D - 6 à 15 jours
E - 24 à 48 heures
Bonne(s) réponse(s) : D

Le traitement symptomatique est indispensable avec isolement et précautions d'asepsie majeure, antibiothérapie à large
spectre en cas de fièvre, après les prélèvements d'usage. Les décès sont le plus souvent liés à des complications
infectieuses. Lorsque celles-ci sont soigneusement prévenues, la guérison s'obtient en 6 à 15 jours, parfois 3 semaines.

721
Exclusivement sur DOC - DZ : www.doc-dz.com NADJI 85
RESIDANAT EN POCHE TOME II
Cas Clinique en QCM
Un patient de 46 ans est vu en consultation pour une splénomégalie. L'étude des antécédents apprend qu'un oncle paternel a
été splénectomisé et que le père du patient a un sub-ictère conjonctival chronique, qu'une des filles du patient a été
cholécystectomisée à l'âge de 12 ans pour lithiase et qu'il n'y a pas d'antécédent maternel connu. L'examen clinique montre
une discrète splénomégalie, un subictère conjonctival.
Hémogramme :
- 4 500 000 hématies
- hématocrite à 40 %
- hémoglobine à 13 g/100 ml
- réticulocytose à 16 %
- présence de microsphérocytes sur le frottis
- leucocytes et plaquettes sont normaux
Billirubine totale : 34 micromol/l dont 32 micromol/l de bilirubine non conjuguée
Le test de Coombs direct est négatif. La résistance osmotique est diminuée.

Selon les données ci-dessus, il est vraisemblable qu'il existe :


A - Hémolyse extra-corpusculaire
B - Hémolyse corpusculaire
C - Hémolyse congénitale
D - Hémolyse acquise
E - Trouble familial de la glycuroconjugaison
Bonne(s) réponse(s) :B C

Le tableau associant une anémie hémolytique (subictère conjonctival, splénomégalie, anémie régénérative, bilirubine non
conjuguée augmentée) avec notion d'hérédité (atteinte familiale), test de Coombs négatif, résistance osmotique diminuée et
présence de microsphérocytes oriente vers la plus fréquente des anémies hémolytiques constitutionnelles, la sphérocytose
héréditaire.

Pour confirmer l'hypothèse d'une sphérocytose héréditaire, vous prescrivez chez ce patient :
A - Test de Ham-Dacie
B - Electrophorèse de l'hémoglobine
C - Autohémolyse in vitro
D - Dosage du fer sérique
E - Dosage des enzymes érythrocytaires
Bonne(s) réponse(s) : C

A - Concerne l'hémoglobinurie paroxystique nocturne (maladie de Marchiafava-Micheli).


B - Tableau clinique et biologique non évocateur d'hémoglobinopathie. Doit toutefois être faite en complément.
C - Elle est augmentée à 37° avec correction par le glucose, n'est pas spécifique (existe dans les anémies hémolytiques auto-
immunes).
D - N'est pas un argument diagnostique d'orientation.
E - A faire pour éliminer une anomalie enzymatique.

Le diagnostic de sphérocytose héréditaire étant confirmé, laquelle, parmi les complications suivantes, peut
survenir chez ce patient ?
A - Anurie aiguë
B - Lithiase rénale
C - Hypertension portale
D - Ictère nucléaire
E - Crise de déglobulisation
Bonne(s) réponse(s) : E

Les complications sont :


- aiguës : crises de déglobulisation périphérique ou centrale, complication de la lithiase (calcul du cholédoque)
- chroniques : retard de croissance saturo pondérale dans les formes sévères à début précoce, ulcère de jambes
(exceptionnels).
Carence en folates par hyperérythropoïèse.

Le traitement chez ce patient peut utiliser un des éléments suivants :


A - Corticothérapie
B - Splénectomie
C - Immunosupresseurs
D - Echanges plasmatiques
E - Exsanguinotransfusions
Bonne(s) réponse(s) : B

C'est la seule thérapeutique efficace, faisable après 5 ans d'âge, avec vaccination antipneumococcique, vérification
manométrique des voies biliaires per opératoire et surveillance des plaquettes au décours de l'intervention.

722
Exclusivement sur DOC - DZ : www.doc-dz.com NADJI 85
RESIDANAT EN POCHE TOME II
Cas Clinique en QCM

Les caractéristiques de la lithiase biliaire de cette maladie sont toutes vraies, sauf une. Laquelle ?
A - Les calculs sont constitués de bilirubinate de calcium
B - Cette lithiase est parfois radio-transparente
C - C'est la principale cause de lithiase biliaire chez l'enfant
D - Le traitement par l'acide ursodésoxycholique peut éviter ces complications
E - Le risque de lithiase est considérablement réduit par la splénectomie
Bonne(s) réponse(s) : D

Il s'agit de calculs pigmentaires qui ne sont pas dissous par l'acide ursodésoxycholique (efficace sur les lithiases
cholestéroliques).

Chez ce patient, la probabilité de transmettre cette affection à un descendant direct est :


A - Egale à 1
B - Egale à 0,5
C - Egale à 0,25
D - Nulle
E - Fonction du sexe du descendant
Bonne(s) réponse(s) : B

C'est une maladie à transmission autosomique dominante (avec cas sporadiques fréquents).

Une femme de 70 ans est hospitalisée pour douleurs angineuses. Antécédents : infarctus du myocarde il y a trois ans. Pas de
récidive des douleurs depuis cette date. Hystérectomie pour fibrome il y a 20 ans. A l'examen : pâleur, tachycardie 110/minute
régulière, tension artérielle 13-7 cmHg, ECG signes de nécrose postéro-latérale ancienne, pas de changement par rapport
aux ECG antérieurs, transaminases normales.
NFS : GR = 1.380.000/mm3, Hb = 6 g/dl, Ht = 18,1 %, réticulocytes = 3 %, GB = 3600/mm3 dont 30 % PN, 5 % PE, 60 % L, 5
%M
- plaquettes : 110.000/mm3
- VS : 35-60

Quelle(s) est(sont) I'(les) anomalie(s) de cet hémogramme ?


A - Anémie hypochrome
B - Neutropénie
C - Eosinophilie
D - Lymphocytose
E - Thrombopénie
Bonne(s) réponse(s) : B E

A - CCMH = Hb/Ht = 0,33 (N = 0,30 à 0,36) soit normochromie.


B - PN = 1080/mm3 (N = 1800 à 7000).
C - PE = 160/mm3 (N = 50 à 500).
D - Lymphocytes = 2160 (N = 1000 à 4000).
E - Normale = 150 à 400 000/mm3.

En dehors d'une anémie mégaloblastique, quelle(s) est (sont) I'(les) étiologie(s) qui peut (peuvent) expliquer
l'ensemble de l'hémogramme ?
A - Inflammation
B - Leucémie aiguë myéloblastique
C - Anémie réfractaire
D - Myxoedème
E - Erythroblastopénie
Bonne(s) réponse(s) : B C

A - Non, donne une anémie normochrome normo ou microcytaire avec thrombocytose.


B - A évoquer devant les signes d'insuffisance médullaire, donnerait plutôt une anémie normocytaire ; diagnostic :
myélogramme.
C - Anémie normo ou macrocytaire, arégénérative avec parfois leucopénie, thrombopénie.
D - Donne des anémies macrocytaires ou normocytaires normochromes isolées.
E - Il s'agit d'un déficit quantitatif isolé en érythroblastes.

723
Exclusivement sur DOC - DZ : www.doc-dz.com NADJI 85
RESIDANAT EN POCHE TOME II
Cas Clinique en QCM

Le myélogramme montre une anémie mégaloblastique, quels sont, parmi les médicaments suivants, celui
(ceux) qui peu(ven)t être responsable(s) d'une anémie mégaloblastique par carence en acide folique et dont il
faut rechercher la prise à l'interrogatoire chez cette patiente ?
A - Méthotréxate®
B - Bactrim®
C - Tériam®
D - Rimifon
E - Furosémide®
Bonne(s) réponse(s) : A B C

Ainsi que les barbituriques, les hydantoïnes, les contraceptifs oraux.

On ne retrouve pas de prise médicamenteuse particulière, le dosage de l'acide folique montre un taux normal et
le taux de vitamine B12 est très abaissé. La fibroscopie gastrique montre une muqueuse atrophique, atrophie
confirmée par la biopsie. Parmi les examens supplémentaires suivants, lequel vous parait nécessaire en vue
d'affirmer le diagnostic de maladie de Biermer ?
A - Test de Schilling avec et sans facteur intrinsèque
B - Dosage de la méthylmalonylurie
C - Test thérapeutique à la vitamine B12
D - Test au Xylose
E - Aucun
Bonne(s) réponse(s) : A

Permet l'étude de l'absorption de la vitamine B12 marquée. L'adjonction de facteur intrinsèque, normalisant l'absorption dans
la maladie de Biermer confirme l'existence du déficit de cette substance. Ce test est à faire après prelèvement pour dosage
vitaminique.

Quelle thérapeutique d'urgence vous parait justifiée a l'arrivée de cette patiente ?


A - Transfusion de 2 flacons de sang frais
B - Transfusion de 2 culots globulaires
C - Plasmaphérèse
D - Injection intraveineuse de 5.000 gammas de vitamine B12
E - Traitement digitalique
Bonne(s) réponse(s) : B

L'indication d'une transfusion en urgence est posée devant l'existence de douleurs angineuses chez une patiente aux
antécédents d'infarctus du myocarde.

Le diagnostic de Biermer retenu, la patiente traitée, on la revoit 3 mois plus tard avec un hémogramme normal.
Quel examen faut-il demander régulièrement ?
A - Réticulocytes
B - Anticorps anti-facteur intrinsèque
C - Taux du facteur intrinsèque
D - Fibroscopie gastrique
E - Vitesse de conduction nerveuse
Bonne(s) réponse(s) : B

La gastrite biermérienne est un terrain à haut risque carcinologique et nécessite une surveillance fibroscopique annuelle.

724
Exclusivement sur DOC - DZ : www.doc-dz.com NADJI 85
RESIDANAT EN POCHE TOME II
Cas Clinique en QCM
Une jeune femme de 24 ans, sans antécédent particulier, présente brutalement au décours d'un épisode infectieux
rhinopharyngé, rapidement résolutif sans traitement, un purpura pétéchial étendu. A l'examen, pas de syndrome tumoral. On
retrouve un purpura pétéchial sur les 2 membres inférieurs et sur la partie supérieure du tronc. Quelques ecchymoses sont
présentes. Il existe des bulles hémorragiques buccales. La malade présente quelques céphalées. En outre, les règles
apparues depuis 8 jours, à leur date normale, sont toujours présentes. Il n'y a pas d'autre symptomatologie fonctionnelle. Le
reste de l'examen clinique ne montre aucun signe pathologique. L'hémogramme fait en urgence révèle une thrombopénie
majeure à 10 000 plaquettes/mm3, le reste de la numération formule sanguine est dans les limites de la normale.

Dans le cadre de cette thrombopénie, outre le syndrome hémorragique présent, quel(s) autre(s) signe(s)
hémorragique(s) pouvez-vous rencontrer ?
A - Un hématome du psoas
B - Une hémorragie cérébro-méningée
C - Une hématurie
D - Une hémarthrose spontanée du genou
E - Une épistaxis
Bonne(s) réponse(s) : A B E

Le purpura thrombopénique est diffus, cutanéomuqueux, pétéchial et/ou ecchymotique.


A - Oui, il s'agit d'un syndrome hémorragique diffus.
B - La surveillance du fond d'oeil permet d'évaluer le risque hémorragique cérébroméningé.
C - Hémorragie muqueuse (atteinte de l'arbre urinaire).
D - Emaille l'évolution de l'hémophilie.
E - Survenue fréquente ainsi que les gingivorragies.

Quel(s) examen(s) allez-vous demander en urgence ?


A- Groupe sanguin et recherche d'agglutinines irrégulières
B - Examen du fond d'oeil
C - Ponction lombaire avec étude cytologique du LCR
D - Echographie splénique
E - Hystérographie
Bonne(s) réponse(s) : A B

A - Systématique devant tout syndrome hémorragique.


B - Indispensable, les hémorragies rétiniennes peuvent être annonciatrices d'hémorragies cérébrales ou méningées.
C - Pas en urgence, surtout si thrombopénie profonde mais suspecter l'hémorragie méningée, si signes hémorragiques au
FO, en raison des céphalées et des signes de gravité (profondeur de la thrombopénie, présence de bulles hémorragiques), la
PL est contre-indiquée en raison de la thrombopénie.
D - Sans intérêt diagnostique ou pronostique en urgence.
E - Sans intérêt. Le diagnostic de purpura thrombopénique idiopathique est un diagnostic d'élimination. Il nécessite les
éléments suivants :
- Absence de prise médicamenteuse, de transfusion récente
- Biologie négative: FAN, MNI test, sérologie HbS, HIV, CMV, rubéole....
- Coagulation normale (pas de ClVD)
- Coombs globulaire négatif.

Pour faire le diagnostic étiologique vous demandez :


A - Echographie splénique
B - Etude du myélogramme
C - Sérologie HIV
D - Recherche de facteurs antinucléaires
E - Mesure de la masse sanguine
Bonne(s) réponse(s) : B D

B - Est obligatoire.
D - Recherchera des anticorps anti-ADN.

Le diagnostic finalement retenu est celui de purpura thrombopénique auto-immun. Dès lors vous prescrivez :
A - Transfusion de plasma frais
B - Injection intramusculaire de vitamine
C - Prednisone 1,5 mg/kilo/jour
D - Transfusion de plaquettes
E - Plasmaphérèses
Bonne(s) réponse(s) : C

A - N'apporte pas de plaquettes.


B - IM formellement contre indiquées.
C - Traitement symptomatique permettant de passer un cap (peu efficace) hémorragique inutile en dehors de l'urgence.
E - Pas en première intention, efficacité transitoire.

725
Exclusivement sur DOC - DZ : www.doc-dz.com NADJI 85
RESIDANAT EN POCHE TOME II
Cas Clinique en QCM

L'évolution se fait vers la chronicité et la thrombopénie entre 10 et 30.000/mm3 persiste; avant d'envisager une
splénectomie, vous demandez :
A - Vaccination antipneumococcique
B - Transfusions de plaquettes
C - Essai thérapeutique par immunosuppresseurs
D - Etude de l'agrégabilité-plaquettaire
E - Echographie splénique
Bonne(s) réponse(s) : A

A - Nécessaire, pour prévenir la sensibilité des splénectomisés au pneumocoque.


B - Non, mais à discuter en pré opératoire immédiat, efficacité extrêmement transitoire (pour assurer une hémostase très
temporaire).
C - Effectués en troisième intention, après échec de la splénectomie et de la corticothérapie.
D - Sans intérêt.
E - Seul le marquage isotopique permet la localisation du lieu de séquestration.

La splénectomie est réalisée, et entraîne une rémission du purpura thrombopénique. On peut voir en relation
avec la splénectomie :
A - Présence de corps de Jolly intra-érythrocytaire
B - Excès de basophiles circulants
C - Thrombocytose à 600 000 plaquettes/mm3
D - Méningococcémie
E - Lymphopénie
Bonne(s) réponse(s) : A C D

A - Ordinairement détruits par la rate, leur présence témoigne d'une atrophie splénique ou d'une asthénie.
B - Sans rapport ; étiologies: syndromes myéloprolifératifs, grands hypercipémies, hypothyroïdie.
C - Entraîne une augmentation immédiate des plaquettes qui régresse en quelques mois.
D - Les infections méningococciques foudroyantes pouvant se déclarer chez le sujet splénectomisé (idem sensibilité des
splénectomisés au pneumocoque).
E - Non, une splénomégalie peut entraîner en revanche une lymphopénie par hémodilution.

Un malade de 65 ans consulte en raison de vertiges et d'une dyspnée d'effort. L'examen clinique constate une érythrose
faciale, une splénomégalie modérée.
L'examen cardiovasculaire et l'examen physique du thorax sont normaux. La numération formule sanguine donne les résultats
suivants :
- globules rouges : 7,37 x 10 exposant 12 par litre
- hémoglobine : 18 g/dl
- hématocrite : 57 %
- VGM : 77 microncube
- CCMH : 31,5 %
- globules blancs : 13,5 x 10 exposant 9/l dont 80 % de polynucléaires neutrophiles
- plaquettes : 825 x 10 exposant 9 par litre

Vous évoquez l'éventualité d'une polyglobulie. Parmi les examens suivants, indiquer celui nécessaire à
l'affirmation de ce diagnostic :
A - Myélogramme
B - Biopsie ostéomédullaire
C - Mesure isotopique du volume globulaire total
D - Mesure isotopique de la durée de vie des globules rouges
E - Mesure de la viscosité sanguine
Bonne(s) réponse(s) : C

Seule la mesure isotopique du volume globulaire total peut affirmer la polyglobulie, définie pour VGT > 32 ml/kg chez la
femme, > 36 ml/kg chez l'homme.

Parmi les affections suivantes, indiquer celle(s) qui est(sont) susceptible(s) de provoquer une polyglobulie
secondaire :
A - Tumeur post-hypophysaire
B - Tumeur rénale
C - Tumeur du cervelet
D - Tumeur testiculaire
E - Hépatome malin
Bonne(s) réponse(s) : B C E

Les causes tumorales susceptibles de donner une polyglobulie secondaire sont les cancers ou kyste du rein, les hémangio-
blastomes du cervelet, les hépatomes, fibromes utérins, cancers ovariens, syndrome de Cushing.

726
Exclusivement sur DOC - DZ : www.doc-dz.com NADJI 85
RESIDANAT EN POCHE TOME II
Cas Clinique en QCM

Vous avez posé chez ce malade le diagnostic de polyglobulie primitive de Vaquez. L'anomalie de taille des
globules rouges s'explique très probablement par :
A - L'hypoxie liée à l'hyperviscosité sanguine
B - Un trouble acquis de synthèse des chaînes de globine
C - Un saignement digestif occulte
D - Une anomalie du métabolisme de la vitamine B12
E - Un éthylisme chronique
Bonne(s) réponse(s) : C

L'étiologie la plus probable est une carence martiale en relation avec une perte de sang chronique occulte.

Parmi les anomalies suivantes, présentes chez ce malade, quelle(s) est (sont) celle(s) qui permet(tent) de
suspecter que cette polyglobulie est une maladie de Vaquez ?
A - Splénomégalie
B - Globules rouges supérieurs à 6,5 x 10 exposant 12 par litre
C - Globules blancs supérieurs à 12 x 10 exposant 9 par litre
D - Plaquettes supérieures à 600 x 10 exposant 9/l
E - Erythrose faciale
Bonne(s) réponse(s) : A C D

A - Elément fondamental du diagnostic, mais inconstant.


B - Ne permet ni d'affirmer la polyglobulie, ni d'orienter vers une cause primitive ou secondaire.
C - Il s'agit d'un syndrome myéloprolifératif.
D - Idem item C.
E - Signe clinique de polyglobulie sans valeur d'orientation.

Une femme de 30 ans est vue en urgence pour hématémèse : elle a rejeté, une heure plus tôt, environ un demi-litre de sang.
Elle ne signale pas d'antécédents pathologiques, en particulier digestifs ou hémorragiques. Elle prenait depuis quelques jours
de l'aspirine pour des odontalgies. Elle a eu deux enfants. A l'arrivée, patiente pâle, agitée, qui a soif ; le pouls est petit, à 100
par minute, la tension artérielle à 8/6. La patiente, qui est donneuse régulière de sang, a une carte portant les mentions de
groupe A1, Rhésus positif CCee, Kell positif, absence d'anticorps irréguliers. Le dernier don remonte à un mois.
L'hémogramme en urgence donne :
hématocrite à 0 40, hémoglobine à 13 g/l
Gh = 4,4.10 exposant 12/l
GB = 12.10 exposant 9/l
plaquettes = 250.10 exposant 9/l.

Une endoscopie faite en urgence montre une ulcération de l'antre en coup d'ongle.

A ce stade, l'indication de transfuser ou de ne pas transfuser immédiatement la malade se fonde sur :


A - L'estimation directe de la quantité de sang perdue
B - Les données de l'examen clinique
C - Le taux de l'hématocrite
D - Le taux d'hémoglobine
E - Le nombre des érythrocytes
Bonne(s) réponse(s) : B

L'hémogramme estime mal le degré de spoliation sanguine dans les premières heures, par perte parallèle d'hématies et de
plasma (sous-estimation de l'anémie), puis par hémodilution (afflux de liquide extracellulaire avec surestimation de l'anémie).

Une nouvelle hématémèse survient, on décide de transfuser. Parmi les poches de sang phénotypé
immédiatement disponibles, laquelle(lesquelles) doit-on préférentiellement utiliser ?
A - O Rh positif CcEe, Kell positif
B - O Rh positif CCee, Kell négatif
C - A1 Rh négatif ccee, Kell positif
D - A1 Rh positif ccEE, Kell positif
E - A2 Rh positif CCee, Kell positif
Bonne(s) réponse(s) : E

Les accidents transfusionnels majeurs sont la conséquence de la destruction des globules rouges du donneur par les
anticorps du receveur. Le sang phénotypé doit respecter la compatibilité des systèmes les plus immunogènes (D, Kell, E, C,
Kidd, Duffy, S) responsables des auto-immunisations tranfusionnelles.

727
Exclusivement sur DOC - DZ : www.doc-dz.com NADJI 85
RESIDANAT EN POCHE TOME II
Cas Clinique en QCM

On transfuse une première poche de sang sans incident, puis une deuxième. La malade est prise de frissons, la
TA est à 11/6 puis la température s'élève à 39 degrés. Cet incident est probablement dû à :
A - Un choc hémolytique par incompatibilité de groupe érythrocytaire
B - L'administration de sang contaminé par des bactéries
C - La transfusion de sang d'un donneur impaludé
D - Des anticorps anti-HLA
E - Un déficit en IgM
Bonne(s) réponse(s) : D

Il s'agit d'un syndrome frissons-hyperthermie consécutif à une auto-immunisation par incompatibilité leucoplaquettaire (HLA)
des cellules résiduelles du culot avec celles du receveur. Il se combat par antihistaminiques et corticoïdes IV, et se prévient
par l'utilisation de culots déleucocytés-déplaquettés.

7 jours plus tard, l'Hb = 9 g/dl, VGM et CCMH normaux et réticulocytes = 120 x 10 exposant 9/l. Dès lors vous
prescrivez :
A - Nouvelle transfusion de deux concentrés érythrocytaires
B - Recherche d'auto-anticorps anti-érythrocytes
C - Sels ferreux, dose équivalente à 100 mg de fer métal par jour, pendant deux mois
D - Eviction de l'aspirine
E - Aucune de ces mesures
Bonne(s) réponse(s) : C D

Une fois le cap de l'urgence passé (ici présence de signes cliniques de gravité), l'apport martial est nécessaire et il n'y a plus
d'indication à la transfusion. L'éviction de l'aspirine va de soi.

Un homme de 50 ans, éthylique notoire, consulte pour une asthénie d'aggravation progressive. L'examen clinique retrouve
une discrète pâleur cutanéo-muqueuse, un ictère conjonctival, une hépatosplénomégalie (la rate débordant de 3 cm le rebord
costal), une lame d'ascite, des oedèmes et quelques ecchymoses des membres inférieurs. L'hémogramme est le suivant :
- Erythrocytes : 2,7 x 10 exposant 12/l
- Hémoglobine : 10,5 g/dl
- Hématocrite : 30 %
- Réticulocytes : 15.x 10 exposant 9/l
- Leucocytes : 3,2 x 10 exposant 9/l dont polynucléaires neutrophiles : 40 %, polynucléaires éosinophiles : 8 %, lymphocytes :
45 %, monocytes : 7 %
- Plaquettes : 120 x 10 exposant 9/l
La VS est à 60 mm à la première heure, 110 mm à la deuxième heure.
Le myélogramme montre une richesse médullaire normale : 35 % d'érythroblastes avec tendance mégaloblastique ; la lignée
granulocytaire, de composition normale, comporte des myélocytes et métamyélocytes de grande taille ; la lignée
mégacaryocytaire est indemne. Le temps de Quick est à 55 %. Il existe une hyperbilirubinémie à 35 micromol/l de formule
mixte, avec élévation des transaminases et de la gamma GT sérique.
La sidérémie est à 210 microg/100 ml avec un coefficient de saturation de la transferrine à 80 %. Il existe à l'électrophorèse
des protéines sériques une hypergammaglobulinémie.

L'analyse de l'hémogramme révèle :


A - Anémie normocytaire
B - Réticulocytopénie
C - Neutropénie
D - Thrombopénie
E - Eosinophilie
Bonne(s) réponse(s) : B C D

A - VGM = Ht/nbGR = 111µ3.


B - Réticulocytes < 150 000/mm3.
C - PN = 1280/mm3 (N = 1800-7000/mm3).
D - Plaquettes :120 000/mm3 (N = 150 à 400 000/mm3).
E - PE = 256/mm3 (N = 50 à 5OO/mm3).

Parmi les mécanismes pouvant intervenir dans la genèse de l'anémie chez ce malade, vous retenez :
A - Hémolyse auto-immune
B - Carence en vitamine B12
C - Hémosidérose
D - Carence en folates
E - Hypersplénisme
Bonne(s) réponse(s) : B D E

A - Il existe des hémolyses au cours de l'alcoolisme, mais non auto-immunes (par hypersplénisme, modifications
corpusculaires acquises).
B - Item discutable. L'anémie mégaloblastique de l'éthylique est due à une carence en folates avec défaut d'apport et troubles
d'utilisation. L'éthylisme n'est pas une étiologie des carences en vitamines B12 (carence d'apport exceptionnelle), mais
certains citent la possibilité de troubles de l'absorption entraînant des carences multi-vitaminiques, dont en B12.

728
Exclusivement sur DOC - DZ : www.doc-dz.com NADJI 85
RESIDANAT EN POCHE TOME II
Cas Clinique en QCM

Indiquez, parmi les médicaments suivants, celui (ceux) qui risque(nt) d'aggraver l'anémie de ce malade, voire
d'entraîner une grande pancytopénie :
A - Spironolactone (Aldactone®)
B - Triamtérène (Tériam®)
C - Furosémide (Lasilix®)
D - Cotrimoxazole (Bactrim®)
E - Gentamicine (Gentalline®)
Bonne(s) réponse(s) : B D

B - Donne des anémies macrocytaires par carence en acide folique.


D - Association :
1) triméthoprime: donnes des leucopénies, thrombopénies, anémies-mégaloblastiques.
2) sulfamide : agranulocytose, pancytopénie toxique, leucopénie, anémie hémolytique.

L'allongement du temps de Quick, observé chez ce malade, peut s'expliquer par un déficit en :
A - Facteur antihémophilique B (facteur IX)
B - Prothrombine (facteur II)
C - Proaccélérine (facteur VI)
D - Proconvertine (facteur VII)
E - Facteur Stuart (facteur X)
Bonne(s) réponse(s) : B C D E

Les modifications de l'hémostase chez les insuffisants hépatiques sont dues au défaut de production des facteurs synthétisés
dans le foie: II, VII, IX, X, l'fibrinogène, protéine C, antithrombine III, antiplasmines.
BCDE.- Influencent le TQ.

Cet allongement du temps de Quick dû à l'insuffisance cellulaire hépatique, peut être corrigé par :
A - Etamsylate (Dicynone®)
B - Transfusion de concentrés plaquettaires
C - Transfusions de plasma frais
D - Vitamine K1
E - Transfusion de facteur VIII
Bonne(s) réponse(s) : C

A - Sans rapport, diminue la perméabilité vasculaire.


B - Sans influence sur le TQ.
C - Apporte tous les facteurs de la coagulation, présents à un taux supérieur à 70.
D - Inefficace (diagnostic différentiel avec les carences en vitamine K).
E - N'influence pas le TP.

L'hypergammaglobulinémie présentée par ce sujet vous parait :


A - Pouvoir expliquer en partie l'accélération de la VS
B - Correspondre vraisemblablement à une hypergammaglobulinémie polyclonale avec bloc béta gamma
C - Correspondre vraisemblablement à une gammapathie monoclonale
D - Pouvoir s'accompagner d'une plasmocytose médullaire comprise entre 5 et 10 %
E - Avoir toutes chances de prédominer sur les IgA
Bonne(s) réponse(s) : A B E

Sans commentaire.

Vous êtes témoin d'un accident : au bas d'une descente une jeune femme est tombée de son cyclomoteur en s'empalant sur
la poignée du frein.
L'examen succint révèle à l'évidence une plaie artérielle au niveau du triangle de Scarpa.

Devant ce tableau, quel geste devez-vous faire ?


A - Confectionner un bandage
B - Poser un garrot
C - Comprimer la plaie
D - Injecter un tonicardiaque
E - Faire appeler un spécialiste
Bonne(s) réponse(s) : C

Sans commentaire.

729
Exclusivement sur DOC - DZ : www.doc-dz.com NADJI 85
RESIDANAT EN POCHE TOME II
Cas Clinique en QCM

Le SAMU prend en charge cette patiente pour son transport en milieu hospitalier. Indiquez parmi les gestes
suivants celui(ceux) qui vous semble(nt) approprié(s) à cette situation :
A - Surveillance du pouls
B - Surveillance de la TA
C - Prélèvement de sang pour groupe sanguin
D - Perfusion de solutés macromoléculaires
E - Transfusion de deux poches de globules O Rh+
Bonne(s) réponse(s) : A B C D

Pas de transfusion de culots globulaires avant détermination du groupe sanguin. On utilise, en attendant pour le remplissage,
des solutés macromoléculaires.

A son arrivée en milieu hospitalier, 30 minutes plus tard, l'appréciation de la perte sanguine reposera sur :
A - La numération globulaire
B - Le volume d'urines émises
C - La détermination du volume globulaire moyen
D - La surveillance du pouls
E - La mesure de la TA
Bonne(s) réponse(s) : B D E

A - La mesure de la NFS apprécie très mal la perte sanguine en raison de la perte initiale parallèle de sang et de plasma.
B - Reflet direct de l'hémodynamique.
C - Idem item A.
D - Le choc hypovolémique se traduit par un pouls petit et rapide, une tension basse et imprenable.
E - Idem Item D.

Indiquez parmi les propositions suivantes, quelle devra être l'attitude la plus utile à adopter dès son arrivée en
milieu hospitalier ?
A - Mise en place d'une PVC
B - Mise en place d'une sonde vésicale
C - Mise en place d'un cerceau chauffant
D - Oxygénothérapie
E - Hémostase chirurgicale
Bonne(s) réponse(s) : E

Le traitement étiologique est bien sûr le seul capable d'interrompre l'hémorragie.

Un mois après son séjour hospitalier, il serait judicieux de contrôler :


A - Le dosage de vitamine B12
B - Le dosage des folates
C - La présence d'antigène HBs
D - L'absence d'anticorps irréguliers
E - La glycémie
Bonne(s) réponse(s) :

Supprimée.

730
Exclusivement sur DOC - DZ : www.doc-dz.com NADJI 85
RESIDANAT EN POCHE TOME II
Cas Clinique en QCM
Monsieur K., 69 ans, consulte pour une anémie et des troubles de la marche. Ses troubles, à type d'asthénie et de dyspnée
d'effort ont commencé il y a six mois. Depuis 15 jours, le patient est gêné pour marcher et s'est donc inquiété. Il est pâle. L'état
général est assez bien conservé ; le patient est sub-ictérique. Il n'y a pas de purpura et on ne palpe ni foie, ni rate, ni
adénopathie. La langue est dépapillée.
L'hémogramme montre les résultats suivants :

GR 1.500 g/l GB. 4.5 T/l (4.500/mm3)


Hb 7 g/dl dont : PN. 75 %
HT 21 % L 32 %
VGM 140 micron3 M 3%
Plaquettes 110.000/mm3
L'examen neurologique montre une diminution des réflexes rotuliens et achilléens, une perte du sens de la position des orteils
des deux cotés.
Le diagnostic final est celui d'anémie de Biermer compliquée d'un syndrome neuro-anémique.

Parmi les anomalies biologiques suivantes obtenues avant tout traitement, après des prescriptions
complémentaires, indiquez celle(s) qui vous semble(nt) compatible(s) avec ce diagnostic ?
A - Taux élevé des LDH plasmatiques
B - Réticulocytose 30.000/mm3
C - Bilirubine non conjuguée 30 µmol/l
D - Hypersegmentation des polynucléaires neutrophiles
E - Taux bas de thiamine dans le sang
Bonne(s) réponse(s) : A B C D

L'augmentation des LDL et une bilirubine libre modérément augmentée font partie du tableau d'hémolyse intramédullaire
toujours présent.
L'hypersegmentation des polynucléaires neutrophiles en attachant de l'importance au fait que l'on retrouve plus de 3 % des
polynucléaires avec 5 lobes ou plus est classique. C'est un élément très évocateur dans les maladies de Biermer à expression
hématologique atténuée.
L'anémie est classiquement normochrome macrocytaire avec des réticulocytes normaux ou même légèrement augmentés en
pourcentage mais inférieurs à 100 000/mm3 en valeur absolue.

Parmi les signes neurologiques suivants, lequel ou lesquels pourrai(en)t à s'expliquer par le syndrome neuro-
anémique ?
A - Paralysie oculomotrice
B - Anesthésie des membres inférieurs
C - Douleur des membres inférieurs
D - Déficit des releveurs du pied
E - Nystagmus
Bonne(s) réponse(s) : B C D

Il s'agit d'un tableau de sclérose combinée de la moelle avec atteinte de fibres longues avec douleurs, paresthésies, troubles
de la marche et signe de Romberg, perte du sens de positionnement des orteils et anomalie au test du Diapason et d'autre
part un syndrome pyramidal pouvant se limiter à un signe de Babinski mais avec potentiellement une diminution de la force
musculaire.

Parmi les résultats suivants, lequel ou lesquels conforte(nt) le diagnostic de maladie de Biermer ?
A - Taux d'acide folique sanguin élevé
B - Absence d'anticorps antifacteur intrinsèque (FI)
C - Test de Shilling abaissé et corrigé par l'adjonction de facteur intrinsèque
D - Achlorhydrie histaminorésistante
E - Taux normal du Fl dans le liquide gastrique
Bonne(s) réponse(s) : C D

Le diagnostic de maladie de Biermer repose sur :


- abaissement du taux sérique de vit B12
- maladie de l'estomac avec achlorhydrie histamino-résistante au tubage gastrique
- absence de facteur intrinsèque par dosage direct du FI dans le liquide gastrique ou preuve indirecte par réalisation d'un test
de Shilling avec et sans FI
- présence d'ac anti FI semblant spécifiques de la maladie mais absents dans au moins 30 % des cas.

731
Exclusivement sur DOC - DZ : www.doc-dz.com NADJI 85
RESIDANAT EN POCHE TOME II
Cas Clinique en QCM

Parmi les symptômes suivants indiquez celui(ceux) qu'un traitement antérieur par la vitamine B12 aurai(en)t pu
modifier ?
A - Mégaloblatose médullaire
B - L'aspect de la langue
C - L'atrophie gastrique
D - La malabsorption de la vitamine B12
E - Le chiffre des plaquettes
Bonne(s) réponse(s) : A B E

La mégaloblastose, la glossite de Hunter, la thrombopénie fréquente et souvent aux alentours de 100 000/m3 sont toutes des
conséquences de la carence en B12 et seront très rapidement corrigées en quelques jours.
La malabsorption de la vitamine B12 va persister car elle est liée à un déficit en FI qui n'est pas corrigée de même que
l'achlorhydrie histamino-résistante.

Une femme de 45 ans, mère de 3 enfants, sans antécédent pathologique particulier, consulte pour une asthénie marquée
évoluant depuis plusieurs mois.
L'état général reste satisfaisant, l'examen clinique est négatif en dehors d'une pâleur des téguments et des muqueuses et d'un
subictère conjonctival. L'hémogramme montre :
- Hémoglobine : 9,5 g/dl. VGM = 110 micro3. CCMH = 33 %
- Leucocytes : 4,5 x 10 exposant 9/l
- Formule leucocytaire :
- polynucléaires neutrophiles : 60 %
- polynucléaires éosinophiles : 2 %
- lymphocytes : 33 %
- monocytes : 5 %
- plaquettes : 160 x 10 exposant 9/l
- réticulocytes : 300 x 10 exposant 9/l
- VS : 60 mn à la première heure

Cet hémogramme révèle :


A - Une anémie normocytaire
B - Une anémie mégaloblastique
C - Une anémie macrocytaire
D - Une pancytopénie
E - Une leucopénie et une anémie
Bonne(s) réponse(s) : C

QCM évident de connaissances générales.

Indiquez, parmi les examens suivants, celui qui vous paraît à ce stade le plus utile pour préciser le diagnostic :
A - Estimation des phosphatases alcalines leucocytaires
B - Dosage de la bilirubine totale et non conjuguée
C - Temps de Quick
D - Myélogramme
E - Dosage du fer sérique
Bonne(s) réponse(s) : B

On a un diagnostic d'anémie macrocytaire régénérative. Dans les examens proposés, le premier à réaliser est un dosage de
bilirubine totale et libre qui peut orienter vers un mécanisme d'hémolyse.

Parmi les diagnostics suivants, quels sont chez cette malade les deux les plus probables, sachant que la
réaction de Coombs directe est positive (de type mixte IgG + C) ?
A - Anémie de Biermer
B - Myélome multiple
C - Lupus érythémateux disséminé
D - Syndrome de Moschcowitz
E - Anémie hémolytique auto-immune
Bonne(s) réponse(s) : C E

On rappellera quand même :


- que l'anémie de Biermer est classiquement macrocytaire et arégénératire
- que le syndrome de Mosckowitz entre dans le cadre des microangiopathies thrombotiques et donne un tableau d'anémie
hémolytique de type microangiopathique avec schizocytose et Coombs négatif direct.

732
Exclusivement sur DOC - DZ : www.doc-dz.com NADJI 85
RESIDANAT EN POCHE TOME II
Cas Clinique en QCM

Le traitement à proposer en première intention est :


A - Vitamine B12 par voie intramusculaire (1000 microgr/jour)
B - Sel ferreux per os (soit 100 mg de fer/jour pendant 3 mois)
C - Prednisone (1 à 1,5 mg/kg/jour)
D - Acide folinique par voie intramusculaire (25 mg/jour)
E - Transfusions de concentrés érythrocytaires (2 unités par semaine)
Bonne(s) réponse(s) : C

Il s'agit d'une anémie hémolytique auto immune-probable qui doit être traitée par corticothérapie.

En cas d'échec de ce premier traitement, quel procédé thérapeutique doit être alors proposé ?
A - Azathioprime (Imurel®)
B - Splénectomie
C - Androgènes à fortes doses
D - Procarbazine (Natulan®)
E - Sérum antilymphocytaire
Bonne(s) réponse(s) : B

Une fois le diagnostic porté d'AHAI à auto ac chauds, une corticothérapie doit être débutée. En cas d'échec ou d'intolérance
aux corticoïdes, la splénectomie doit être réalisée. Le recours aux immunosuppresseurs n'est classiquement retenu qu'après
échec ou rechute après splénectomie.

Un patient de 50 ans dont l'hémogramme était normal un an auparavant, a l'hémogramme suivant :


- hématocrite : 60 %
- globules rouges : 6 500 000/mm3
- hémoglobine : 20 g/dl
- globules blancs : 11 900 mm3
- polynucléaires neutrophiles : 75 %
- lymphocytes : 15 %
- monocytes : 10 %
- plaquettes : 1 000 000/mm
- VS : 1-2
Le volume globulaire isotopique est à 50 ml/kg.
L'examen clinique en particulier neurologique est normal.

Quels sont dans l'observation le ou les élément(s) en faveur du diagnostic de maladie de Vaquez ?
A - Le chiffre des plaquettes
B - Le taux de l'hématocrite
C - Le chiffre des globules blancs
D - La vitesse de sédimentation
E - L'importance de l'hypervolémie
Bonne(s) réponse(s) : A

Une thrombocytose > 400 000/mm3 est évocatrice du diagnostic de maladie de Vaquez.
L'hyperleucocytose > 120 000/mm3 (en l'absence d'infection) est également un des critères retenus par le groupe d'étude des
polyglobulies primitives.
Quelque soit l'étiologie d'une polyglobulie vraie, la VS est diminuée.
Ni l'importance de l'hypovolémie ni l'augmentation importante de l'hématocrite ne sont des éléments orientant vers le
diagnostic de la maladie de Vaquez. On retiendra cependant que la moyenne du VGT dans la maladie de Vaquez est de 49
ml/kg.

Parmi les autres signes et symptômes suivants, quel est celui(ceux) qui est(sont) compatible(s) avec le
diagnostic de maladie de Vaquez ?
A - Détection d'une splénomégalie par échographie
B - SaO2 92 %
C - Présence d'une myélofibrose réticulinique à la biopsie médullaire
D - Taux abaissé d'érythropoïétine plasmatique
E - Caryotype médullaire normal
Bonne(s) réponse(s) : A C D E

A) Evident (clinique, scintigraphique ou échographique),


- la saturation artérielle en O2 doit être strictement > 92 % pour affiner le diagnostic de maladie de Vaquez.
- La myélofibrose réticulinique à la BM est compatible, en général absente aux stades précoces de la maladie.
- La baisse du taux d'EP plasmatique par dosage radio-immunologique n'est pas de pratique courante mais pourrait être très
sensible pour différencier polyglobulies primitives et secondaires.
- Le caryotype médullaire peut être normal. Il existe cependant des anomalies fréquentes non spécifiques récemment décrites.

733
Exclusivement sur DOC - DZ : www.doc-dz.com NADJI 85
RESIDANAT EN POCHE TOME II
Cas Clinique en QCM

Parmi les examens suivants, lequel est utile dans le cas de ce patient, compte tenu des résultats déjà en votre
possession, pour confirmer le diagnostic de maladie de Vaquez ?
A - Dosage de la vitamine B12
B - Dosage des phosphatases alcalines leucocytaires
C - Dosage de l'uricémie
D - Myélogramme
E - Aucun de ces examens
Bonne(s) réponse(s) :

Attention : il est impossible de répondre à ce QCM avec les données en notre possession car : si on retient les données du
QCM n°20 et notamment une saturation à 72 % (il y a un gribouillage), il faut alors chez ce patient avoir comme critères pour
retenir le diagnostic de polyglobulie primitive :
un VGMO> 30ml/g (critère présent ici), une splénomégalie (critère présent ici) et 2 critères parmi :
- thrombocytose > 400 000mm3 (présent ici)
- score des PAL > 100
- vitamine B12 sérique > 900pg/ml ou capacité de liaison de la vitamine B12 > 2 200 pq/ml.
Aucun de ces deux critères n'est prépondérant sur l'autre et on ne peut donc répondre par un QCM simple.

Le diagnostic de maladie de Vaquez est affirmé. Quel traitement de fond doit être proposé chez ce patient en
tenant compte des données cliniques et biologiques ?
A - Saignées seulement
B - Saignées + hydroxyurée
C - Saignées + phosphore 32
D - Aspirine à petites doses
E - Surveillance et traitement en cas d'aggravation
Bonne(s) réponse(s) : B

Les saignées sont le traitement d'urgence chez ce patient de 50 ans avec augmentation très importante du VGT et des
risques hémorragiques, thrombotiques et cardiovasculaires non négligeables.
L'hydréa est le traitement de choix (agent non radiomimétique non mutagène) et sera préféré à cet âge au P 32. Il doit être
associé d'emblée, du fait du risque de thrombocytose, potentialisée par la carence martiale induite par les saignées seules.

Une femme de 60 ans a une anémie à 8 g/dl, VGM 101 micron3, CCMH 33 %, réticulocytes 350 000/mm3, VS 72-120,
globules blancs et plaquettes normaux, bilirubine totale 30 micromoles/l, bilirubine non conjuguée 28 micromoles/l,
haptoglobine effondrée, LDH 300 UI (N 290). Pas de fièvre, adénopathies axillaires bilatérales sensiblement symétriques de 2
cm de diamètre, splénomégalie de 2 cm sous le rebord costal.

Quel est le diagnostic le plus précis que l'on puisse affirmer à ce stade ?
A - Hémolyse extra corpusculaire
B - Hémolyse périphérique à prédominance intra tissulaire
C - Hémolyse périphérique à prédominance intra vasculaire
D - Hémolyse intra médullaire
E - Déficit de la glycuro conjugaison
Bonne(s) réponse(s) : B

La réponse est évidente.


L'augmentation des LDL permet de retenir comme très probable une hémolyse à prédominance intra vasculaire. Les LDL
érythrocytaires sont augmentées car libérées dans le plasma lors de l'éclatement intravasculaire du GR.

Le test de Coombs direct est positif (IgG et complément), quel est le diagnostic le plus précis que vous pouvez
affirmer ?
A - Anémie hémolytique immunologique
B - Anémie hémolytique auto-immune
C - Anémie hémolytique immuno-allergique
D - Isoimmunisation anti-érythrocytaire
E - Hémolyse virale
Bonne(s) réponse(s) : A

Les hémolyses immunologiques ont en commun la fixation d'un anticorps sur le globule rouge entraînant sa destruction et se
répartissant en catégories qui en l'absence d'anamnèse supplémentaire ne peuvent être différenciées : hémolyse iso immune,
auto-immune, immuno-allergique.

734
Exclusivement sur DOC - DZ : www.doc-dz.com NADJI 85
RESIDANAT EN POCHE TOME II
Cas Clinique en QCM

L'élution est positive. L'anticorps IgG est dirigé contre un antigène du système rhésus. Quelle étiologie faut-il
rechercher ?
A - Cancer épithélial profond
B - Myélome
C - Prise d'Aldomet®
D - Lupus érythémateux disséminé
E - Pneumonie atypique
Bonne(s) réponse(s) : D

Les hémolyses à l'Aldomet® sont liées à une IgM chaude non agglutinante fixant le complément. Un test de Coombs positif de
type IgG antirhésus peut être observé dchez 10 % des consommateurs d'Aldomet® mais en l'absence d'hémolyse.
Certaines observations de cancers épithéliaux profonds ont été rapportées en association avec des AHAI mais sont rarissimes.
Les AHAI liées à une infection aiguë sont le plus souvent de type IgM.
Le myélome peut rarement être associé à une AHAI à IgM.
Le diagnostic le plus probable est celui de LEAD.

La biopsie ganglionnaire met en évidence une infiltration de petits lymphocytes mûrs. L'étude des marqueurs de
membrane montre qu'il s'agit d'une population B micron kappa. Quel est votre diagnostic ?
A - Myélome multiple
B - Lymphome malin non hodgkinien
C - Adénopathie réactionnelle
D - Maladie de Hodgkin
E - Mononucléose infectieuse
Bonne(s) réponse(s) : B

Sans commentaire.

Quel est le traitement de l'anémie à prescrire en première intention ?


A - Gammaglobulines à fortes doses
B - Corticothérapie
C - Plasmaphérèses
D - Chimiothérapie anti mitotique
E - Splénectomie
Bonne(s) réponse(s) : B

La corticothérapie est le traitement d'attaque à instituer en cas d'AHAI à Ac chauds.

Une femme de 25 ans est hospitalisée en raison de l'apparition d'un purpura pétéchial et ecchymotique, cutanéo-muqueux
avec épistaxis et gingivorragies. Elle n'a pas d'antécédent pathologique.
L'examen ne décèle ni adénopathie ni splénomégalie. La température est à 37 ; le fond d'oeil est normal.
L'hémogramme montre les résultats suivants :
- hématies : 4 000 000/mm3, hémoglobine : 12 g/100 ml, hématocrite : 36 %
- plaquettes : 5 000/mm3
- leucocytes : 5 000/mm3 dont 60 % neutrophiles, 30 % lymphocytes, 10 % monocytes.

Parmi les cinq hypothèses diagnostiques suivantes, quelle(s) est(sont) celle(s) que vous pouvez exclure sur les
arguments cliniques et biologiques dont vous disposez ?
A - Leucémie aiguë
B - Purpura rhumatoïde
C - Purpura thrombopénique idiopathique
D - Insuffisance médullaire
E - Maladie de Willebrand
Bonne(s) réponse(s) :

QUESTION ANNULEE.

Parmi les examens suivants, il est indispensable de réaliser sans délai :


A - Temps de saignement (Ivy)
B - Etude de la coagulation
C - Etude de l'agrégation plaquettaire en présence d'ADP
D - Mesure de la durée de vie des plaquettes marquées au 51Cr
E - Myélogramme
Bonne(s) réponse(s) : B E

Le myélogramme est indispensable pour préciser le mécanisme périphérique ou central de la thrombopénie ici très profonde.
L'étude de la coagulation est également à réaliser en urgence avec recherche de signes de CIVD en demandant TP, TCK,
temps de thrombine, fibrinogène, facteur V, complexes solubles, temps de lyse des euglobulines, PDF.

735
Exclusivement sur DOC - DZ : www.doc-dz.com NADJI 85
RESIDANAT EN POCHE TOME II
Cas Clinique en QCM

Les résultats des examens précédents sont normaux. Vous cherchez alors une cause infectieuse latente ou
passée inaperçue. Parmi les examens utiles vous prescrivez un ou plusieurs des suivants :
A - Intradermoréaction tuberculinique 10 unités
B - Ponction lombaire
C - Sérodiagnostic de toxoplasmose
D - Anticorps anti HIV
E - Sérodiagnostic de Wright
Bonne(s) réponse(s) : D

La séropositivité HIV est une cause reconnue de purpura thrombopénique immunologique.


Aucune des autres infections citées n'est responsable de thrombopénie.

Vous complétez en outre l'enquête étiologique par un ou plusieurs des examens suivants :
A - Biopsie médullaire
B - Anticorps antinucléaires
C - Recherche d'agglutinines irrégulières
D - Recherche d'anticorps antileucoplaquettes (anti HLA)
E - Immunoélectrophorèse du sérum
Bonne(s) réponse(s) : B

Il faut réaliser un bilan immunologique minimum et notamment rechercher une connectivite (LEAD etc...). Les autres examens
n'on aucun intérêt.

Les résultats des examens précédents sont normaux ou négatifs. Vous n'avez cependant pas attendu pour
prescrire un traitement. Celui-ci comporte, en première intention, un des choix suivants :
A - Corticoïde per os 0,1 mg/kg/jour
B - Corticoïde intramusculaire 0,1 mg/kg/jour
C - Corticoïde per os 1 mg/kg/jour
D - Corticoïde intramusculaire 1 mg/kg/jour
E - Corticoïde per os 1 mg/kg/jour et Synacthène® immédiat 0,25 mg intramusculaire
Bonne(s) réponse(s) : C

Le premier traitement est la corticothérapie per os à dose minimale initiale de 1 mg/kg/jour pour une durée de 3 semaines à 1
mois pour juger de l'efficacité.
Chez cette patiente, étant donné les risques hémorragiques graves, le seul traitement d'urgence qui aurait pu être proposé,
permettant une augmentation rapide du taux de plaquette est un traitement par a globulines : 40 mg/kg/jour ou 1 g/kg/jour x 2
jours avec une efficacité en 48 à 72 heures.

Huit jours plus tard, la patiente à 20 000 plaquettes/mm3. Le syndrome hémorragique ne s'est pas majoré. Vous
décidez alors une des attitudes suivantes :
A - Augmentation des corticoïdes à 5 mg/kg/j
B - Transfusions plaquettaires
C - Splénectomie
D - Adjonction d'un immunosupresseur
E - Ne pas modifier le traitement à ce stade
Bonne(s) réponse(s) : E

Il faut attendre 3 semaines à 1 mois de corticothérapie à 1-2 mg/kg/jour pour juger de l'efficacité de traitement.

Trois mois plus tard, la patiente a 300 000 plaquettes/mm3,lors d'une consultation de surveillance. Le traitement
est arrêté depuis quinze jours. Vous lui donnez un ou plusieurs des conseils suivants :
A - Interdiction définitive de toute grossesse
B - Contre-indication absolue aux oestroprogestatifs
C - Surveillance des plaquettes par numérations trimestrielles en raison du risque de rechute
D - Contre-indication définitive aux injections intramusculaires
E - Nécessité d'une enquête familiale par hémogramme + plaquettes chez les ascendants et collatéraux
Bonne(s) réponse(s) : C

Une surveillance simple espacée est indispensable pour détecter les rechutes qui sont fréquentes.
La guérison sous corticoïdes, sans rechute, est estimée à 20-30 % des cas traités.

736
Exclusivement sur DOC - DZ : www.doc-dz.com NADJI 85
RESIDANAT EN POCHE TOME II
Cas Clinique en QCM

Cinq ans plus tard, la guérison semble acquise ; cependant ce cas constitue une contre-indication permanente
à:
A - Antiagrégants plaquettaires
B - Héparinothérapie
C - Antivitaminiques K
D - Thrombolytiques
E - Aucune des thérapeutiques précédentes
Bonne(s) réponse(s) : E

Evident.

Une femme de 32 ans se plaint depuis quelques mois de fatigabilité excessive et d'une discrète dyspnée d'effort, qui ne
l'empêchent pas cependant de mener toutes ses activités, L'examen clinique ne décèle qu'une discrète pâleur. Il n'y a pas
d'antécédent pathologique.
Hémogramme :
- hémoglobine. : 8,5 g/dl
- hématocrite : 0,28
- érythrocytes : 4,0.10 exposant 12/l
- réticulocytes : 30.10 exposant 9/l
- leucocytes : 6,3 . 10 exposanr 9/l (PN 63 PE 2 L 30)
- plaquettes : 400.10 exposant 9/l
- sidérémie : 6 mcmol (0,34 mg/l)
- capacité totale de fixation du fer mcmol (3,92 mg/l)

Il existe une anémie :


A - Normochrome normocytaire régénérative
B - Normochrome macrocytaire arégénérative
C - Hypochrome normocytaire arégénérative
D - Hypochrome microcytaire arégénérative
E - Hypochrome microcytaire régénérative
Bonne(s) réponse(s) : D

Le VGM et la CCMH sont définis par les supports suivants : VGM = Ht /nb de GR = 70m3 ; CCMH = Hb/Ht = 0,30.
Les réticulocytes sont à 30 000/mm3. Il s'agit donc d'une anémie hypochrome microcytaire arégénérative.

Cette anémie est liée à :


A - Un trouble de synthèse des acides nucléiques
B - Un trouble de synthèse de l'hème
C - Un trouble de synthèse de la globine
D - Une réduction du nombre des cellules souches myéloïdes
E - Une destruction excessive des érythrocytes circulants
Bonne(s) réponse(s) : C

La microcytose signe une anomalie de la synthèse de l'hémoglobine.

L'étiologie la plus probable, chez cette patiente, est :


A - Une intoxication chronique méconnue
B - Des hémorragies digestives occultes
C - Une malabsorption digestive
D - Des hémorragies génitales excessives
E - Un état inflammatoire
Bonne(s) réponse(s) : B

C'est la cause de carence martiale la plus fréquente chez l'homme adulte.

Indépendamment d'un traitement étiologique éventuel, cette anémie est justiciable :


A - De transfusions de concentrés érythrocytaires
B - D'un traitement par la vitamine B12
C - D'un traitement par l'acide folique
D - D'un traitement par le fer
E - D'un traitement par corticoïde
Bonne(s) réponse(s) : D

Seul le traitement par fer est nécessaire.


Le taux d'Hb étant > 8g % et le patient présentant une dyspnée modérée bien tolérée, il n'y a aucune indication
transfusionnelle.

737
Exclusivement sur DOC - DZ : www.doc-dz.com NADJI 85
RESIDANAT EN POCHE TOME II
Cas Clinique en QCM
Une femme de 25 ans, consulte à l'occasion de vertiges d'apparition récente. Secrétaire, en bonne santé avant son mariage,
elle est mère de trois enfants de 4, 2 et 1 ans. Ses grossesses ont été normales, mais une hémorragie de la délivrance est
survenue après le deuxième accouchement, sans toutefois nécessiter de transfusion sanguine. Une période de fatigue s'est
prolongée plusieurs mois après les deux dernières naissances. Elle ne présente aucun autre antécédent. Les seuls
traitements reçus comprenaient vitamine B12 et acide folique.
Elle est porteuse d'un stérilet depuis 6 mois.
L'hémogramme donne les résultats suivants :
- hématies 3.8 x 10 exposant 12/l
- hémoglobine 9,5 g/dl
- hématocrite 0,28, volume globulaire 73,3 fl
- teneur corpusculaire en hémoglobine 25 pg
- leucocytes 3.600 x 10 exposant 9/l
- polynucléaires neutrophiles 55 %
- polynucléaires éosinophiles 1 %
- lymphocytes 45 %
- monocytes 4 %
Le dosage du fer sérique donne les résultats suivants : 6 micro mol/l capacité totale de fixation 80 micro mol/l.
Le diagnostic d'anémie microcytaire ferriprive est porté.

Parmi les facteurs étiologiques possibles, vous retenez dans cette observation :
A - L'âge de la malade
B - La prise de vitamine B12 et d'acide folique
C - Des grossesses rapprochées
D - Une hémorragie de la délivrance non compensée
E - Le port d'un stérilet
Bonne(s) réponse(s) : A C D

Sans commentaire.

Le traitement à proposer est :


A - Des transfusions de globules rouges déplasmatisés
B - Des sels ferreux per os à la dose de 50 mg de fer métal par jour
C - Des sels ferreux per os à la dose de 100 mg de fer métal par jour
D - Des sels ferreux per os à la dose de 400 mg de fer métal par jour
E - Des injections parentérales de sels ferreux
Bonne(s) réponse(s) : C

Le traitement repose bien sur la prescription de fer par voie orale sous forme de sels divers dont les avantages respectifs sont
peu évidents. L'important est de prescrire une dose suffisante de 100 à 200 mg/jour de fer métal.

Parmi les tests de surveillance biologiques suivants, celui dont le retour à la normale permet d'affirmer la
compensation de la carence est :
A - Numération globulaire
B - Détermination du taux de l'hémoglobine
C - Détermination du volume globulaire moyen
D - Dosage du fer sérique
E - Détermination de la capacité totale de fixation du fer
Bonne(s) réponse(s) : E

Le seul élément simple du bilan biologique qui permet d'affirmer que la compensation en fer de la carence est suffisante est la
détermination de la capacité totale de fixation de fer. Toutes les autres propositions ici sont inutiles. Au bout de trois mois, si la
capacité totale de saturation reste élevée, il faut poursuivre le traitement un mois de plus.

Parmi les signes suivants quel est celui ou quels sont ceux qui peuvent apparaître en cours de traitement ?
A - Urines rouges
B - Dyspepsie
C - Troubles du transit
D - Coloration noire des selles
E - Troubles neurologiques
Bonne(s) réponse(s) : B C D

Le traitement par fer oral entraîne une coloration noire non malodorante des selles et des troubles digestifs variés (nausées,
constipation, douleurs épigastriques, diarrhées) qui peuvent être diminués en cas de prise des comprimés au moment des
repas.

738
Exclusivement sur DOC - DZ : www.doc-dz.com NADJI 85
RESIDANAT EN POCHE TOME II
Cas Clinique en QCM
Un homme de 42 ans, représentant de commerce, consulte pour une baisse de l'état général : asthénie et amaigrissement. Il
à tendance à abuser de boissons alcoolisées depuis de nombreuses années. A l'examen clinique, on relève une
splénomégalie mesurée à 8 cm sous le rebord costal et une hépatomégalie à bord dur, régulier, débordant de 5 cm le rebord
costal. Pas d'adénopathie. Le teint est pâle sans ictère. La température est normale. Aucune autre particularité clinique n'est
relevée. L'hémogramme montre : G.R. : 2,2 T/l Hb : 82 g/l Hte : 27 % VGM 123 fl

CCHM : 33 %
Plaquettes 70 G/l

Réticulocytes : 15 G/l
G.B. : 3,2 G/l
PNN : 36 %
Eosino : 1 %
Baso : 0
Lympho : 57 %
Mono : 6 %

Fibrinogène : 1, 5 g/l
V.S. : 70 (1 ère heure)

Parmi les examens suivants, lequel ou lesquels vous parait ou vous paraissent utile(s) pour expliquer l'état
hématologique ?
A - Dosage du fer sérique
B - Dosage des folates sériques
C - Biopsie médullaire
D - Myélogramme
E - Durée de vie des G. R. au 51 Cr
Bonne(s) réponse(s) : B D

- Pancytonénie d'hypersplénisme sur cirrhose alcoolique probable avec carence en folates expliquant la macrocytose.
- Le myélogramme, non systématique, montrerait un tableau de mégaloblastose médullaire (en pratique ne se fait jamais).

Dans le contexte clinique, quelle est l'étiologie qui vous paraît la plus probable pour expliquer l'anémie ?
A - Hypervolémie plasmatique
B - Carence en folate
C - Syndrome inflammatoire
D - Aplasie médullaire
E - Hypersplénisme
Bonne(s) réponse(s) : B

Aggravant l'anémie d'hypersplénisme et lui donnant son caractère macrocytaire.

Dans le contexte clinique, quels sont les deux mécanismes qui vous paraissent pouvoir expliquer le nombre de
plaquettes et de PNN ?
A - Hypervolémie plasmatique
B - Carence en folate
C - Syndrome inflammatoire
D - Aplasie médullaire
E - Hypersplénisme
Bonne(s) réponse(s) : B E

B - La carence en folates freine également la production granuleuse et plaquettaire au niveau de la moelle osseuse.
E - Séquestration splénique des trois lignées sanguines.

Dans le contexte clinique, quelle est l'explication la plus probable de l'élévation de la V. S. ?


A - Infection bactérienne
B - Cryoglobulinémie
C - Hypergammaglobulinémie polyclonale
D - Dysglobulinémie monoclonale
E - Thrombopénie
Bonne(s) réponse(s) : C

Fréquente chez le cirrhotique avec bloc Bêta et Gamma.

739
Exclusivement sur DOC - DZ : www.doc-dz.com NADJI 85
RESIDANAT EN POCHE TOME II
Cas Clinique en QCM

Quelle cause vous apparait la plus probable pour expliquer la splénomégalie ?


A - Lymphome splénique
B - Hypertension portale
C - Hémolyse
D - Splénomégalie myéloïde
E - Insuffisance hépato-cellulaire
Bonne(s) réponse(s) : B

Sans commentaire.

Un garçon de 7 ans présente une asthénie importante, une fébricule à 38°. Deux mois auparavant, il a présenté un ictère
pendant une semaine. A l'examen on trouve une pâleur de la peau et des muqueuses, un subictère conjonctival une douleur
au point de Murphy, une splénomégalie à 2 travers de doigt.
Biologie :
Hématies : 3 090 000/mm3
Hémoglobine : 9,7 g/100 ml
Hématocrite : 27 %
V.G.M. : 90 fl
Leucocytes :8 200/mm3
Poly neutro : 38 %
Eosino : 2 %
Lympho : 58 %
Mono : 2 %
Plaquettes : 210 000/mm3
Réticulocytes : 12 %
Absence de corps de Heinz dans les hématies.
Bilirubine totale : 28 micromoles/l
Bilirubine directe : 5 micromoles/l
Bilirubine indirecte : 23 micromoles/l
Transaminases : GOT : 12 U.I.
GPT : 14 U.I.

Parmi les hypothèses diagnostiques suivantes, laquelle est la plus plausible ?


A - Anémie par carence martiale
B - Anémie pré-aplasique compliquant une hépatite virale
C - Anémie hémolytique corpusculaire
D - Maladie coeliaque
E - Maladie de Fanconi
Bonne(s) réponse(s) : C

Il s'agit d'une anémie régénérative avec hyperbilirubinémie libre stigmates d'une hyperhémolyse : probable microsphérocytose
héréditaire ou maladie de Minkowsky-Chauffard.

L'argument principal en faveur de ce diagnostic est :


A - La pâleur
B - La douleur de l'hypochondre droit
C - L'antécédent d'hépatite virale
D - La splénomégalie
E - Le sexe
Bonne(s) réponse(s) : D

Témoignant d'une hémolyse chronique.

Parmi les données biologiques, le principal argument en faveur de ce diagnostic est :


A - La valeur des transaminases
B - La numération des hématies
C - Le nombre des Iymphocytes
D - Le taux des réticulocytes
E - La valeur de la bilirubine indirecte
Bonne(s) réponse(s) : E

Reflet de l'hypercatabolisme de l'hémoglobine.

740
Exclusivement sur DOC - DZ : www.doc-dz.com NADJI 85
RESIDANAT EN POCHE TOME II
Cas Clinique en QCM

Pour confirmer votre hypothèse diagnostique, vous demandez un examen. Lequel ?


A - Un myélogramme
B - Une électrophorèse des protides
C - Un caryotype
D - Un dosage du fer sérique
E - Une étude de la résistance osmotique des hématies
Bonne(s) réponse(s) : E

- Diminution de la résistance à l'hypotonie in vitro traduisant la microsphérocytose.


- Le meilleur test diagnostic de la maladie de Minkowsky-Chauffard est l'étude de l'auto-hémolyse in vitro qui est augmentée,
corrigée par le glucose mais non par l'ATP.

En l'absence de traitement, on devrait craindre la survenue :


A - D'une anurie aiguë par hémolyse intra-vasculaire
B - D'un ictère nucléaire
C - D'une leucémie aiguë
D - D'une hépatite chronique
E - D'une angiocholite
Bonne(s) réponse(s) : E

(Une seule réponse est correcte dans cette question à compléments multiples).
E - Migration d'une lithiase pigmentaire.

La thérapeutique la plus appropriée chez cet enfant comporte :


A - La prescription d'acide folique
B - La greffe de moelle
C - Un traitement martial
D - La splénectomie
E - L'éviction à vie des quinidines et des sulfamides
Bonne(s) réponse(s) : A D

A - Systématique dans ces états d'hyper-réactivité médullaire peuvent entraîner une carence.
D - Indiquée à partir de 5 ans donc traitement de choix chez ce garçon.

Une jeune femme de 20 ans découvre une adénopathie sus-claviculaire droite. Elle n'a pas d'antécédent connu, pas de
symptômes particuliers : apyrexie, poids stable. Le reste de l'examen est normal. Les radiographies thoraciques montrent un
élargissement du médiastin supérieur et antérieur. L'hémogramme est normal, la vitesse de sédimentation est à 6 mm à la
première heure. Un adénogramme est pratiqué et montre une prédominance lymphocytaire avec quelques rares cellules
volumineuses, à gros noyau bourgeonnant et nucléolé, ayant les caractères de la cellule de Sternberg.

Parmi les explorations suivantes, la ou lesquelles vous semble(nt) indispensable(s) à ce stade des explorations
?
A - Médullogramme
B - Médiastinoscopie
C - Biopsie ganglionnaire sus claviculaire
D - Laparotomie exploratrice avec splénectomie
E - Laparotomie exploratrice sans splénectomie
Bonne(s) réponse(s) : C

Permettra d'affirmer le diagnostic de maladie de Hodgkin et d'en préciser le type.

Le bilan d'extension anatomique doit en principe comporter un ou plusieurs des examens suivants : le ou
lesquels ?
A - Lymphographie bipédieuse
B - Urographie Intraveineuse
C - Fibroscopie gastrique
D - Scintigraphie osseuse
E - Ponction lombaire
Bonne(s) réponse(s) : A D

A - Systématique à la recherche d'une atteinte abdominale.


D - Non systématique. Seulement en cas de stade > II ou de signes ethniques ou biologiques d'évolutivité.

741
Exclusivement sur DOC - DZ : www.doc-dz.com NADJI 85
RESIDANAT EN POCHE TOME II
Cas Clinique en QCM

Le ou les examens précédents n'ont pas montré d'anomalie. Vous pouvez en fonction des données dont vous
disposez, classer cette maladie dans l'un des stades suivants. Lequel ?
A - IAa
B - IIAa
C - IIAb
D - IIBb
E - IIIAa
Bonne(s) réponse(s) : B

Deux sites ganglionnaires atteints d'un même côté du diaphragme sans signes cliniques ou biologiques d'évolutivité.

Un traitement polychimiothérapique associant Méchloréthamine, Oncovin, Procarbazine et Prednisone


(M.O.P.P.) est entrepris. Les complications de ce traitement peuvent comporter :
A - Neuropathie périphérique
B - Vomissements
C - Insuffisance cardiaque
D - Hépatite médicamenteuse
E - Alopécie
Bonne(s) réponse(s) : A B E

A - Liée à l'Oncovin.

Le traitement chimiothérapique est suivi d'une irradiation en mantelet. Celle-ci doit délivrer :
A - 20 grays en 2 semaines
B - 40 grays en 4 semaines
C - 60 grays en 6 semaines
D - 40 grays en une semaine
E - 40 grays en une seule séance
Bonne(s) réponse(s) : B

40 Gy est la dose tumoricide nécessaire et suffisante, 2 Gy par séance à raison de 5 séances par semaine.

Le traitement optimal étant effectué, l'espérance de cette jeune patiente d'être en rémission persistante au bout
de 5 ans est de :
A - 10 %
B - 30 %
C - 50 %
D - 70 %
E - 90 %
Bonne(s) réponse(s) : E

Sans commentaire.

A quelle(s) complication(s) tardive(s) la patiente reste-t-elle exposée du fait du traitement ?


A - Hypothyroïdie
B - Péricardite
C - Zona
D - Leucémie aiguë
E - Diabète
Bonne(s) réponse(s) : A B C D

A - Secondaire à la radiothérapie en mantelet.


B - Secondaire à la radiothérapie en mantelet.
C - Secondaire à l'immuno-suppression résiduelle diminuant avec le temps.
D - Effets leucémogènes des agents alkylants (Méthochlormétine et Procarbozine) associés à la radiothérapie.

742
Exclusivement sur DOC - DZ : www.doc-dz.com NADJI 85
RESIDANAT EN POCHE TOME II
Cas Clinique en QCM
Un garçon de 11 ans, sans antécédent pathologique, vous est adressé pour syndrome hémorragique et hyperthermie. La
température est entre 39 et 40°C depuis 48 heures et l'examen retrouve un purpura pétéchial et ecchymotique au niveau des
membres inférieurs, des gingivorragies et des bulles hémorragiques endobuccales. Il existe une splénomégalie de 2 travers
de doigt, une pâleur cutanéo-muqueuse, un souffle systolique de pointe à l'auscultation cardiaque. Le fond d'oeil montre une
petite hémorragie près de la papille de l'oeil droit. Le cliché thoracique est normal. L'hémogramme révèle :
- Hémoglobine : 6 g/100ml
- Leucocytes : 0,5.10 exposant 9/l dont :
- polynucléaires neutrophiles : 20 %
- lymphocytes : 80%
- Plaquettes : 10.10 exposant 9/l.
Le T.P., le T.C.A., le dosage du fibrinogène sont normaux. Le myélogramme retrouve, dans une moelle riche, 85 % de cellules
blastiques peu différenciées, à rapport nucléocytoplasmique élevé, sans grains cytoplasmiques.

Le diagnostic retenu est celui de leucémie aiguë lymphoblastique (L.A.L.). Quel autre diagnostic pouvait être
discuté au vu de l'hémogramme et du myélogramme, devant un tel tableau clinique ?
A - Une aplasie médullaire
B - Une mononucléose infectieuse
C - Un état d'hypersplénisme
D - Une leucémie lymphoïde chronique
E - Aucune de ces affections
Bonne(s) réponse(s) : E

Sans commentaire.

Il est urgent de mettre en oeuvre dans ce cas :


A - Une série de 6 à 8 hémocultures sur 24 h puis mise en route d'une antibiothérapie par voie veineuse
B - Même antibiothérapie, mais mise en route en moins de 3 heures, pendant lesquelles on réalise seulement
2 ou 3 hémocultures
C - Transfusion de concentrés plaquettaires
D - Transfusion de plasma frais
E - Transfusion de fractions coagulantes PPSB
Bonne(s) réponse(s) : B C

Stratégie systématique devant une neutropénie fébrile.

Au cours d'une L.A. L. de l'enfant en première poussée, l'on retrouve aussi fréquemment l'une des
manifestations suivantes :
A - Des localisations cutanées spécifiques
B - Des douleurs osseuses
C - Une hyperplasie gingivale
D - Une positivité de la réaction cytochimique des myéloperoxydases
E - La présence de corps d'Auer dans certains blastes
Bonne(s) réponse(s) : B

Sans commentaire.

Chez ce patient, vous retenez comme facteur(s) de "bon pronostic" :


A - L'absence d'hyperleucocytose
B - La présence d'une splénomégalie
C - L'absence de tuméfaction médiastinale
D - Le sexe
E - L'absence d'hépatomégalie et d'adénopathies superficielles
Bonne(s) réponse(s) : A C E

- L'hyperleucocytose et le syndrome tumoral sont des facteurs de mauvais pronostic.


- L'existence d'une tuméfaction médiastinale est liée au type T de la prolifération et ne retentit sur le pronostic que par ce
caractère immunologique et non par sa présence en elle-même.

743
Exclusivement sur DOC - DZ : www.doc-dz.com NADJI 85
RESIDANAT EN POCHE TOME II
Cas Clinique en QCM

Le protocole de chimiothérapie d'induction entrepris chez ce patient associera vincristine, prednisone et une
anthracycline (daunorubicine). Une rémission complète est obtenue. Les autres mesures essentielles dès
l'obtention de la rémission complète comporteront :
A - Une irradiation en mantelet
B - Si la PL est normale une prévention systématique des rechutes neuroméningées
C - Un traitement d'entretien au long cours par le purinéthol (6 micropropuline) et le Méthotrexate
D - Des cures de réinduction régulières par les médications ayant permis d'obtenir l'état de rémission complète
E - Une irradiation testiculaire systématique
Bonne(s) réponse(s) : B C D

Sans commentaire.

Certaine(s) complication(s) iatrogène(s) est (sont) à redouter à plus ou moins long terme, compte tenu des
principales drogues administrées en induction chez ce patient :
A - Polynévrite
B - Fibrose pulmonaire
C - Insuffisance rénale
D - Myocardiopathie
E - Surdité
Bonne(s) réponse(s) : A D

A - Liée à la Vincristine.
D - Liée aux anthracyclines.

Madame Th..., 65 ans, sans antécédent pathologique remarquable, est venue consulter en raison de l'installation progressive
d'une fatigabilité inhabituelle et d'une dyspnée d'effort. L'existence d'une pâleur à conduit à la réalisation d'un hémogramme
qui apporte les résultats suivants :
- Globules rouges 1,97.10 exposant 12/l, hémoglobine 8,4 g/dl, hématocrite 25 % volume globulaire moyen 129f/l ; TCMH 42,8
p/g.
- Globules blancs 3,2.10 exposant 9/l 3 200 mm3 dont 54 % de polynucléaires neutrophiles, 37 % de Iymphocytes, 9 % de
monocytes.
Plaquettes : 120.10 exposant 9/l 120 000/mm3.
Réticulocytes : 17 000/mm3.
- L'examen clinique montre l'existence d'un subictère conjonctival ; il n'y a pas d'hépatosplénomégalie palpable. L'examen
neurologique est en tout point normal. Le coeur est régulier, la tension artérielle à 15-8. L'électrocardiogramme sans anomalie.
Un complément d'examens biologiques est réalisé :
- Bilirubine totale à 30 micromoles/l, dont 22 micromoles de bilirubine libre ; myélogramme riche avec population
érythroblastique majoritaire, comportant des éléments mégaloblastiques ; dosage de la vitamine B12 inférieur à 100ng/l
(normale : 150 à 620) dosage des folates normal.

L'hypovitaminémie Bl2 est directement ou indirectement à l'origine d'une ou plusieurs des modifications
suivantes :
A - Nombre des polynucléaires neutrophiles
B - Nombre des Iymphocytes
C - Nombre des plaquettes
D - Anomalies érythroblastiques médullaires
E - Elévation de la bilirubine libre
Bonne(s) réponse(s) : A C D E

E - Témoin d'un certain degré d'hémolyse et de l'avortement médullaire érythoblastique.

Quel(s) examen(s) vous apparaît ou apparaissent nécessaire(s) au diagnostic étiologique de cette carence ?
A - Radiographies d'estomac et des premières anses grêles
B - Dosage du fer sérique
C - Etude de la cinétique des plaquettes marquées
D - Test de Schilling
E - Tubage gastrique avec stimulation à la Pentagastrine
Bonne(s) réponse(s) : D E

D - Apporte la preuve d'une malabsoption de vitamines B12 par déficit en facteur intrinsèque.
E - Recherche d'une achlorhydrie.

744
Exclusivement sur DOC - DZ : www.doc-dz.com NADJI 85
RESIDANAT EN POCHE TOME II
Cas Clinique en QCM

Les causes théoriquement possibles d'une carence en vitamine B12 comportent :


A - Ethylisme chronique
B - Surconsommation au cours d'une hémolyse aiguë
C - Iléite terminale
D - Absence de facteur intrinsèque
E - Traitement au long cours par les hydantoïnes
Bonne(s) réponse(s) : C D

C - L'Iléon distal est le siège de l'absorption de la vitamine B12.


D - Maladie de Biermer avec atrophie gastrique à l'origine du défaut de sécrétion de FI.

Une maladie de Biermer a pu être affirmée chez cette patiente. L'évolution de cette maladie par ailleurs
correctement substituée en vitamine B12 peut se compliquer de :
A - Ulcère gastrique
B - Cancer gastrique
C - Epilepsie
D - Apparition d'un syndrome cordonal postérieur
E - Thyroïdite auto-immune
Bonne(s) réponse(s) : B E

B - 5 % des cas imposant une surveillance stricte endoscopique.


E - Association fréquente du Biermer à d'autres maladies auto-immunes.

Au bout de 6 mois de traitement substitutif, quel signe peut encore être retrouvé ?
A - Macrocytose érythrocytaire
B - Mégaloblastose médullaire
C - Baisse de l'hémoglobine
D - Achlorhydrie gastrique
E - Elévation de la bilirubine libre
Bonne(s) réponse(s) : D

L'atrophie gastrique persiste.

Un homme de 25 ans, agriculteur, marié, sans enfant, doit être opéré d'une amygdalectomie. Dans ses antécédents on note
plusieurs épistaxis dans l'enfance et une hémorragie buccale importante, non explorée, il y a 5 ans, après une avulsion
dentaire. Son bilan d'hémostase pré-opératoire est le suivant :
- plaquettes : 275.10 exposant 9/l
- taux de prothrombine 78 %
- temps de céphaline avec activateur TCA - témoin = 30 sec. - malade = 50 sec.
- fibrinogène : 2,8 g/l

Ces résultats sont compatibles avec :


A - Une hémophilie A
B - Une hémophilie B
C - Une thrombopathie
D - Un traitement antivitamine K
E - Un anticoagulant circulant anti-facteur Vlllc.
Bonne(s) réponse(s) : A B

Sans commentaire.

Un examen autre d'hémostase aurait dû être demandé d'emblée dans le bilan pré-opératoire. Lequel ?
A - Temps de saignement
B - Agrégation plaquettaire à l'A.D.P.
C - Test d'adhésivité plaquettaire au verre
D - Test de Von Kaulla
E - Dosage des facteurs du complexe prothrombinique
Bonne(s) réponse(s) : A

Sans commentaire.

745
Exclusivement sur DOC - DZ : www.doc-dz.com NADJI 85
RESIDANAT EN POCHE TOME II
Cas Clinique en QCM

Cet examen (demandé à la question précédente) est pratiqué chez le patient et son résultat est anormal.
Compte tenu de cette nouvelle information et du bilan initial, un diagnostic parmi la liste suivante doit être
évoqué en premier. Lequel ?
A - Hémophilie A
B - Hémophilie B
C - Maladie de Willebrand
D - Anticorps anti-prothrombinase
E - Insuffisance hépatocellulaire
Bonne(s) réponse(s) : C

L'allongement concomitant du TS témoin d'une anomalie de l'hémostase primaire, et du TCA est très évocateur d'une maladie
de Willebrand.

Parmi les examens suivants, lequel (lesquels) demandez-vous pour confirmer ce diagnostic ?
A - Dosage du facteur Vlllc
B - Dosage du facteur V
C - Epreuve de Koller à la vitamine K
D - Dosage du cofacteur de la ristocétine
E - Recherche d'une insuffisance hépato-cellulaire
Bonne(s) réponse(s) : A D

D - Mesure l'activité du facteur Willebrand.

Pour préparer l'intervention chirurgicale, quelle (s) thérapeutique (s) proposez-vous ?


A - Vitamine K intraveineuse
B - P.P.S.B. intraveineux
C - Préparation de facteur VIII
D - Transfusion de plaquettes
E - Injection de reptilase
Bonne(s) réponse(s) : C

Sans commentaire.

Un diagnostic de myélome est porté chez un homme de 65 ans. Les examens pratiqués montrent :
- V.S. = 120 à la lère heure ;
- Electrophorèse du sérum : bande étroite identifiée comme IgG Kappa à l'immunoélectrophorèse. L'évaluation de la
dysglobulinémie est de 42 g/l les autres Ig sont à : IgA 0,20 g/l et IgM 0,15 g/l ;
- Dans les urines, on trouve une protéinurie de Bence- Jones : excrétion de chaîne Kappa à 4,3 g/24 heures ;
- Sur l'hémogramme on relève : hémoglobine = 81 g/l; VGM = 88 fl; réticulocytes 12.10 exposant 9/l; leucocytes et plaquettes
normaux ;
- Par ailleurs :
- Créatinémie : 180 micromol/l
- Uricémie : 660 micromol/l
- Calcémie : 2,3 micromol/l
Le myélogramme montre une moelle riche avec une plasmocytose à 34 %.

Parmi les examens suivants lequel ou lesquels vous paraît (paraissent) nécessaire(s) pour compléter les
explorations ?
A - Scintigraphie osseuse
B - Radiographie du squelette
C - Lymphographie
D - Echographie abdominale
E - Ponction biopsie hépatique
Bonne(s) réponse(s) : B

B - A la recherche de lacunes osseuses ou de fractures pathologiques (tassements vertébraux).

Quel examen supplémentaire vous paraît le plus important pour expliquer l'anémie ?
A - Test de Coombs
B - Durée de vie des globules rouges
C - Mesure isotopique du volume globulaire total
D - Dosage du fer sérique
E - Recherche d'agglutinine irrégulière
Bonne(s) réponse(s) : C

Anémie non régénérative centrale avec hémodilution par la globuline monoclonale abaissant artificiellement le chiffre d'Hb.

746
Exclusivement sur DOC - DZ : www.doc-dz.com NADJI 85
RESIDANAT EN POCHE TOME II
Cas Clinique en QCM

Parmi les critères suivants, lequel ou lesquels a(ont) une valeur pronostique ?
A - VS = 120
B - Uricémie à 660 micromol/l
C - Hémoglobine : 81 g/l
D - Créatinémie : 180 micromol/l
E - Protéinurie de Bence-Jones : 4,3 g/24 h.
Bonne(s) réponse(s) : C D E

Sans commentaire

Un traitement associant Alkeran et Prednisone est donné en cures mensuelles de 4 jours. Parmi les critères
suivants lequel vous paraît le plus valable pour juger de l'efficacité du traitement ?
A - Mesure de l'immunoglobuline monoclonale
B - Recalcification osseuse
C - Créatinémie
D - Dosage de l'hémoglobine sur l'hémogramme
E - Taux de plasmocytes dans la moelle osseuse
Bonne(s) réponse(s) : A

A - Reflet le plus fidèle de la masse tumorale.

Parmi les affirmations suivantes concernant l'évolution sous ce traitement proposé, laquelle ou lesquelles est
(sont) exacte(s) ?
A - La guérison ne peut être attendue
B - Une réponse objective est attendue dans 2/3 des cas
C - Le traitement fait courir un risque de leucémie aiguë
D - La durée de survie médiane probable est de 10 ans
E - Le traitement augmente le risque d'amylose
Bonne(s) réponse(s) : A B C

Sans commentaire.

Une femme âgée de 63 ans (60 kgs), est connue depuis longtemps pour avoir une lithiase vésiculaire asymptomatique. Elle
présente depuis une dizaine de jours, un ictère rapidement progressif dont la séméiologie clinique et biologique est celle d'un
ictère par rétention. L'indication opératoire (cholécystectomie) est portée. A noter, dans ses antécédents, une phlébite avec
embolie pulmonaire survenue après hystérectomie pour fibrome il y a 10 ans. L'un de ses deux fils à subi une intervention
chirurgicale de la veine cave pour maladie thromboembolique sévère. Le bilan d'hémostase préopératoire très complet que
vous avez demandé est le suivant : taux de prothrombine : 45 % ; T. Céphaline activé : 38 sec (30 sec) ; T. Saignement Ivy
vrai : 8mn 30sec ; Plaquettes : 180 000/mm3 ; Fibrinémie : 2,85 g/l ; V : 85 %, VII + X : 35 %; II : 4,5 % : antithrombine III : 110
%; Protéine C : 30 %.

Quel argument permettra d'écarter le diagnostic d'insuffisance hépatocellulaire sévère ?


A - Le taux de facteur II
B - Le taux de facteur V
C - La numération des plaquettes
D - Le taux de protéine C
E - Le taux de fibrinogène
Bonne(s) réponse(s) : B

Sans commentaire.

Quel(s) argument(s) permette(nt) d'évoquer le diagnostic d'hypovitaminose K ?


A - La notion d'ictère par rétention
B - L'association d'un T. de Quick et de Céphaline allongés
C - L'existence d'un taux de facteur V normal tandis que le taux des facteurs VII + X et II est diminué
D - Un taux bas de protéine C
E - L'association d'un Ivy à 8,30 min et d'une numération plaquettaire à 180 000/mm3
Bonne(s) réponse(s) : A B C

Sans commentaire.

747
Exclusivement sur DOC - DZ : www.doc-dz.com NADJI 85
RESIDANAT EN POCHE TOME II
Cas Clinique en QCM

L'intervention chirurgicale est programmée dans 3 jours. Quelle est l'attitude la plus logique pour préparer cette
patiente à l'intervention ?
A - Perfuser du PPSB 36 heures avant l'intervention
B - Perfuser du PPSB juste avant l'induction anesthésique
C - Donner de la vitamine K per os matin et soir jusqu'à l'intervention
D - Donner de la vitamine K par voie parentérale immédiatement
E - Perfuser du plasma frais congelé en per-opératoire
Bonne(s) réponse(s) : D

D - En trois jours la vitamine K a le temps d'agir par la synthèse des facteurs vitamines K dépendants.

Moyennant le traitement adéquat, vous avez corrigé les anomalies du temps de Quick et du temps de céphaline
et l'intervention s'est bien passée. Compte-tenu des antécédents, vous soumettez cette patiente à une
héparinothérapie préventive : 5 000 U d'héparine standard (calciparine) 2 fois par jour. Quel est l'examen de
laboratoire le plus démonstratif de l'efficacité du traitement ?
A - Un temps de saignement
B - Un temps de Quick
C - Un temps de céphaline activé
D - Une numération des plaquettes
E - Aucun des examens ci-dessus
Bonne(s) réponse(s) : C

Sans commentaire.

Au dixième jour post-opératoire, la malade, toujours sous héparine à la même dose, allait sortir, lorsqu'elle
présente des signes cliniques d'une embolie pulmonaire, qui est confirmée. Le bilan d'hémostase que vous
demandez est le suivant : T. Quick : 90% ; T. Céphaline Activé : 32 sec (30 sec) ; Fibrinogène : 3,80 g/l ;
Antithrombine III : 75 % ; Protéine C : 50 %. Comment interprétez-vous ces résultats ?
A - Le taux d'AT III discrètement perturbé est en rapport avec l'héparinothérapie
B - Il pourrait s'agir d'un déficit constitutionnel en AT III
C - Le taux bas de protéine C est lié à l'héparinothérapie
D - Le taux bas de protéine C est la conséquence d'une hypovitaminose K résiduelle
E - Il pourrait s'agir d'un déficit constitutionnel en protéine C
Bonne(s) réponse(s) : A E

A - L'héparine interfère avec l'AT III et diminue son taux circulant.


D - Non, car le TQ s'est normalisé (la protéine C se corrige le plus précocement). La protéine C malgré un traitement par
vitamine K et la levée de l'ictère reste basse. Ceci confronté aux antécédents familiaux et personnels est très en faveur d'un
déficit constitutionnel.

Dans l'immédiat, quelle solution thérapeutique choisissez-vous ?


A - Maintenir le traitement par voie SC en augmentant la dose à 5 000 U x 3/jour
B - Arrêter la calciparine et prescrire une héparine de bas poids moléculaire : 1 injection SC/24h
C - Arrêter la calciparine et prescrire 8 000 U d'héparine par voie IV toutes les 6 heures
D - Administrer de l'héparine standard par voie IV à la pompe, à la dose de 32 000 U/24h
E - Arrêter d'urgence toute héparinothérapie au profit d'un traitement par les antivitamines K
Bonne(s) réponse(s) : D

Sans commentaire.

Aucun membre de la famille n'est joignable pour effectuer une enquête familiale. Dix jours après avoir
commencé le traitement, vous redemandez un dosage d'antithrombine III : 65 % et de protéine C : 45 %. La
malade souhaite rentrer chez elle. Quel traitement de consolidation lui proposez-vous pour les 6 prochains mois
?
A - Aspirine - Dipyridamole per os
B - Héparine sous-cutanée : 5 000 U x 2/jour
C - Hémoclar (glossettes)
D - Héparine de bas poids moléculaire : 1 mg SC/24h
E - Antivitamines K en cherchant à obtenir un INR à 3
Bonne(s) réponse(s) : E

Les AVK sont le traitement de choix des anticoagulations au long cours à titre préventif.

748
Exclusivement sur DOC - DZ : www.doc-dz.com NADJI 85
RESIDANAT EN POCHE TOME II
Cas Clinique en QCM
Une jeune fille de 18 ans (57 kg pour 168 cm) arrive en urgence avec un purpura des membres inférieurs (pétéchies et
ecchymoses). Elle n'a jusqu'à ce jour jamais été malade. Elle n'absorbe qu'un oestro-progestatif depuis 6 mois et n'est pas en
contact avec des produits toxiques. L'état général est bon ; l'examen clinique ne montre ni adénopathies superficielles, ni gros
foie, ni splénomégalie.

L'hémogramme montre :
GR : 4,1 x 10 exposant 12/l
Hb : 12,9 g/dl
Hte : 38,4 %
VGM : 93 fl
CCMH : 33,5 %

Plaquettes : 5.10 exposant 9/l


GB : 7,3.109/l
PN : 61 ; Eo : 2 ; L : 26
Mono : 11
VS : 2-6

Outre l'étude de la coagulation, quel est ou quels sont le ou les examens à réaliser en urgence ?
A - Temps de saignement
B - Fond d'oeil
C - Myélogramme
D - Auto-anticorps antiplaquettes
E - Anticorps anti-plaquettes
Bonne(s) réponse(s) : B C

B - A la recherche d'hémorragies rétiniennes.


C - Dans le but de préciser la nature centrale ou périphérique de la thrombopénie.

S'il s'agit d'un purpura thrombopénique idiopathique, l'étude de l'hémostase doit montrer :
A - TP normal
B - TCA normal
C - Fibrinogène normal
D - Test à l'éthanol positif
E - Temps de thrombine allongé
Bonne(s) réponse(s) : A B C

La coagulation n'est pas perturbée. Seule l'hémostase primaire sera altérée.

Quel(s) traitement(s) décidez-vous ?


A - Abstention thérapeutique
B - Prednisolone (Solupred®) = 60mg/j
C - Solumedrol® IM = 60mg/j
D - Immunosuppresseur non stéroïdien
E - Transfusion de plaquettes = 6 concentrés
Bonne(s) réponse(s) : B

C'est le traitement de première intervention de choix.

Quelle(s) étiologie(s) convient-il malgré tout de rechercher de parti-pris ?


A - Lupus érythémateux aigu
B - Purpura par oestro-progestatif
C - Infection par virus Epstein-Barr
D - Infection par VIH
E - Hyperoestrogénisme
Bonne(s) réponse(s) : A C D

A - Les PTI sont parfois révélateurs de maladies auto-immunes et notamment LEAD.


C - Dans le cadre d'une MNI.
D - La première cause actuelle des PTI est devenue l'infection VIH.

749
Exclusivement sur DOC - DZ : www.doc-dz.com NADJI 85
RESIDANAT EN POCHE TOME II
Cas Clinique en QCM

Parmi les médicaments reconnus comme responsables d'un purpura de même type (moelle riche) vous retenez :
A - Misulban®
B - Quinidine
C - Aminosides
D - Digitoxine
E - Corticoïdes
Bonne(s) réponse(s) : B D

Sans commentaire.

Le 1er traitement mis en route se révèle être un échec; la thrombopénie persistant six mois plus tard, quel est le
traitement de deuxième intention ?
A - Immunoglobulines IV
B - Vinblastine (Velbe®) en perfusion
C - Cyclophosphamide (Endoxan®) per os
D - Azothioprine (Imurel®)
E - Splénectomie
Bonne(s) réponse(s) : E

E - Après s'être éventuellement garantie d'une séquestration splénique prédominante par plaquettes marquées au C51.

Une femme de 42 ans présente un état anémique, L'interrogatoire révèle que depuis deux ans il existe des malaises (2 ou 3)
à type de lipothymies avec sueurs et impression de lourdeur abdominale. Depuis 6 mois, les troubles deviennent permanents
et s'accentuent : asthénie, dyspnée d'effort et palpitation. La pâleur apparaît évidente. On ne retrouve pas d'autres troubles
digestifs. Les règles sont régulières (durée 4 jours, sans caillot), l'évaluation de leur volume est normal. L'examen clinique est
normal.
Hémogramme :
- globules rouges 3, 5 T/l
- hémoglobine : 7, 5 g/dl
- hématocrite : 26%
- VGM : 74 fl
- CHM : 22 pg
- CCHM : 28 %
- plaquettes : 520 g/l
- globules blancs : 12,8 g/l
polynucléaires neutrophiles 0,80%
polynucléaires éosinophiles 0,05%
polynucléaires basophiles 0
Iymphocytes 0,12%
monocytes 0,03%
Vitesse de sédimentation, 1ère heure = 35
Fer sérique : 4,2 micromol/l
Le diagnostic d'anémie ferriprive est retenu.

Parmi les caractères suivants lequel (lesquels) peuvent être expliqués par la carence martiale ?
A - Le taux des éosinophiles
B - Le chiffre des plaquettes
C - Le VGM
D - Le taux de polynucléaires neutrophiles
E - Le taux de Iymphocytes
Bonne(s) réponse(s) : B C

Sans commentaire.

Quel examen choisissez vous pour confirmer la carence martiale ?


A - Capacité totale de fixation de la sidérophiline
B - Dosage de l'haptoglobine
C - Dosage de la céruloplasmine
D - Hémosidérinurie
E - Myélogramme avec coloration de Perls
Bonne(s) réponse(s) : A

Sans commentaire.

750
Exclusivement sur DOC - DZ : www.doc-dz.com NADJI 85
RESIDANAT EN POCHE TOME II
Cas Clinique en QCM

La carence martiale étant affirmée, citez les deux examens qui vous paraissent d'emblée les plus adaptés pour
découvrir l'étiologie :
A - Coloscopie
B - Fibroscopie gastrique
C - Hystérographie
D - Urographie intraveineuse
E - Scanner abdominal
Bonne(s) réponse(s) : A B

Il n'y a pas de point d'appel gynécologique, la première cause à rechercher est un saignement d'origine digestive.

Si l'étiologie n'est pas rapidement trouvée, quelle attitude vous paraît la plus adaptée à la situation ?
A - Transit du grêle
B - Recherche d'une hématurie microscopique
C - Etude de l'absorption du fer
D - Etude de l'incorporation du fer 59
E - Scintigraphie abdominale avec globules rouges marqués
Bonne(s) réponse(s) : A

Sans commentaire.

Quelle que soit l'étiologie il est nécessaire de corriger la carence martiale. Quelle prescription journalière faites-
vous ?
A - 10 mg de fer métal per os
B - 100 mg de fer métal per os
C - 100 mg de fer métal par voie intramusculaire
D - 1 g de fer métal per os
E - 1 g de fer métal par voie intramusculaire
Bonne(s) réponse(s) : B

Sans commentaire.

Surveillance. Ce traitement sera poursuivi :


A - 1 mois
B - Jusqu'à normalisation du taux d'hémoglobine
C - Jusqu'à normalisation du V.G.M
D - Jusqu'à normalisation du fer sérique
E - Jusqu'à normalisation de la capacité totale de saturation de la sidérophiline
Bonne(s) réponse(s) : E

E - Meilleur reflet de l'état des réserves qui est en fait mieux apprécié par la Ferritine.

Un homme de 65 ans, sans antécédents notables présente un syndrome infectieux assez banal, d'allure grippale, traité de
façon symptomatique. Quelques jours plus tard, le tableau s'aggrave brutalement avec majoration de la fièvre qui devient
oscillante, frissons, baisse tensionnelle. L'examen clinique retrouve quelques petites lésions d'allure ulcéro-nécrotique au
niveau de la cavité buccale. Il n'existe ni adénopathie, ni splénomégalie.
Un hémogramme réalisé en urgence donne les résultats suivants :
- Erythrocytes : 4,5 10 exposant 12/l
- Hémoglobine : 13 g/dl
- Hématocrite : 40 %
- Leucocytes : 1,5. 10 exposant 9/l dont :
Polynucléaires neutrophiles :5 %
Polynucléaires éosinophiles : 1 %
Lymphocytes : 86 %
Monocytes : 8 %
- Plaquettes : 160.10 exposant 9/l
La VS est à 80 mm à la première heure.

Cet hémogramme met en évidence comme anomalie(s) :


A - Anémie normochrome normocytaire
B - Réaction mononucléosique
C - Agranulocytose
D - Hyperlymphocytose
E - Thrombopénie
Bonne(s) réponse(s) : C

Sans commentaire.
751
Exclusivement sur DOC - DZ : www.doc-dz.com NADJI 85
RESIDANAT EN POCHE TOME II
Cas Clinique en QCM

Les anomalies hématologiques constatées peuvent résulter de la prise de : (tous ces médicaments étant
retrouvés chez le malade).
A - Vitamine C
B - Aspirine
C - Phénylbutazone
D - Acide folique
E - Noramidopyrine
Bonne(s) réponse(s) : C E

Sans commentaire.

Dans cette situation, quels gestes d'urgence allez-vous mettre en oeuvre ?


A - Transfusion de concentrés granulocytaires
B - Commencer immédiatement une série d'hémoculture de 8 à 10 hémocultures sur 48 heures, à l'acmé des
pics fébriles
C - Androgénothérapie à fortes doses (Nilevar : 1 mg/kg/j)
D - Corticothérapie à fortes doses (Prednisone : 2 mg/kg/j)
E - Polyantibiothérapie intraveineuse dans les six heures qui suivent l'admission
Bonne(s) réponse(s) : B E

On attend ni les résultats ni le délai de 48 heures pour débuter l'antibiothérapie.

Sous réserve d'un traitement d'urgence adapté, et après retour à la normale de l'hémogramme, quelle est
l'évolution prévisible ?
A - Guérison sans séquelles
B - Guérison avec persistance d'une sensibilité accrue aux infections bactériennes
C - Evolution secondaire vers une aplasie médullaire
D - Rechute si reprise du toxique
E - Guérison avec risque différé d'éclosion d'un syndrome myélodysplasique
Bonne(s) réponse(s) : A D

Sans commentaire.

Pour identifier avec certitude le médicament en cause il faut avoir recours à :


A - La réintroduction d'une dose test du médicament suspect
B - Un test de transformation Iymphoblastique en présence du médicament
C - Un test de dégranulation des basophiles en présence du médicament
D - Un dosage des IgE
E - Aucun de ces tests
Bonne(s) réponse(s) : E

Les mécanismes des agranulocytoses ne sont pas univoques :


- Toxicité directe
- Mécanisme immuno-allergique.

Une femme de 37 ans, sans antécédent pathologique personnel ou familial, accuse un fléchissement de l'état général
d'installation progressive, avec dyspnée d'effort, céphalées et palpitations. Son médecin, constatant une pâleur, un subictère
et une splénomégalie palpable, prescrit des examens de laboratoire dont les résultats sont les suivants :
- Hémogramme :
Globules rouges = 2 000 000/mm3 Hémoglobine = 7 g/dl Hématocrite = 21 % Réticulocytes = 18 %
Globules blancs = 10 500/mm3 avec pourcentages : 79 % polynucléaires neutrophiles 2 % éosinophiles 14 % Iymphocytes 5
% monocytes
- Fer sérique = 25 micromol/l
- Bilirubine = 40 micromol/l (dont 37 de bilirubine non conjuguée).

En faveur du diagnostic d'anémie hémolytique régénérative vous retenez dans cet énoncé :
A - Le degré de l'anémie
B - La splénomégalie
C - La réticulocytose
D - Le taux de fer sérique
E - L'ictère à bilirubine non conjugué
Bonne(s) réponse(s) : B C E

Sans commentaire.

752
Exclusivement sur DOC - DZ : www.doc-dz.com NADJI 85
RESIDANAT EN POCHE TOME II
Cas Clinique en QCM

Parmi les explorations biologiques suivantes indiquez celle qu'il faut demander en premier lieu :
A - Durée de vie des hématies
B - Myélogramme
C - Test de Coombs direct
D - Dosage des enzymes érythrocytaires
E - Electrophorèse de l'hémoglobine
Bonne(s) réponse(s) : C

Sans commentaire.

A l'issue du bilan clinique et biologique, c'est le diagnostic d'anémie hémolytique auto-immune qui est retenu :
forme apparemment idiopathique, à autoanticorps IgG. Parmi les attitudes thérapeutiques suivantes quelle est
celle qui doit être envisagée en première intention ?
A - Transfusion d'au moins deux concentrés érythrocytaires
B - Vitaminothérapie associant acide folique et vit. Bl2
C - Corticothérapie
D - Azathioprine
E - Splénectomie
Bonne(s) réponse(s) : C

Sans commentaire.

On découvre assez souvent un état pathologique associé à ce type d'anémie. Indiquez dans la liste ci-dessous
celui (ou ceux) qui doit (doivent) être recherché(s) comme cause(s) d'anémie hémolytique auto-immune :
A - Kyste de l'ovaire
B - Ulcère gastro-duodénal
C - Hémopathie Iymphoïde maligne
D - Lupus érythémateux disséminé
E - Kyste du rein
Bonne(s) réponse(s) : A C D

Sans commentaire.

Les autoanticorps ont une spécificité de groupe érythrocytaire qui varie suivant la classe d'immunoglobulines à
laquelle ils appartiennent. Dans le cas présent cette spécificité intéresserait le plus probablement les antigènes
appartenant à un des systèmes suivants. Indiquez lequel ?
A - ABO
B - Rh
C - Li
D-P
E - Kell
Bonne(s) réponse(s) : B

Sans commentaire.

Madame X..., 60 ans, est hospitalisée en urgence pour hyperthermie, frissons. Chez cette patiente, le diagnostic de myélome
à chaîne légère Kappa de stade III à été posé trois mois auparavant. Trois cures de chimiothérapie associant vincristine,
melphalan, cyclophosphamide et prednisone ont été réalisées dont la dernière dix jours avant l'hospitalisation actuelle. Un
hémogramme réalisé deux jours avant l'hospitalisation objective les données suivantes :
- Hémoglobine = 105 g/l
- Globules blancs = 0,9.10 exposant 9/l
- Plaquettes = 75 . 10 exposant 9/l
- Polynucléaires neutrophiles = 30 %
- Lymphocytes = 70 %
A l'entrée, la température est à 39 °C, la tension artérielle à 90/60 mmHg, le pouls à 120/min. L'auscultation pulmonaire met
en évidence un foyer de crépitants de la partie-moyenne du champ pulmonaire droit. La gorge est propre. Il n'y a pas de
symptomatologie fonctionnelle urinaire. On ne palpe ni adénopathie, ni splénomégalie.

Le diagnostic de myélome à chaînes légères résulte des données de :


A- Myélogramme
B - Biopsie ganglionnaire
C - Scintigraphie osseuse
D - Electrophorèse des protides urinaires
E - Immuno-électrophorèse des protides urinaires
Bonne(s) réponse(s) : A D E

Sans commentaire.
753
Exclusivement sur DOC - DZ : www.doc-dz.com NADJI 85
RESIDANAT EN POCHE TOME II
Cas Clinique en QCM

De plus il est indiqué de demander pour cette patiente :


A - Hémocultures
B - Radiographie pulmonaire
C - Myélogramme
D - Scanner thoracique
E - Dosage de la créatinine plasmatique
Bonne(s) réponse(s) : A B E

Sans commentaire.

La neutropénie peut-être attribué à :


A - Myélome
B - Corticothérapie
C - Melphalan
D - Cyclophosphamide
E - Hypersplénisme
Bonne(s) réponse(s) : A C D

Sans commentaire.

Chez cette patiente, vous décidez :


A - La mise en place d'une voie d'abord veineuse
B - Aucune antibiothérapie avant les résultats des prélèvements
C - Une antibiothérapie à large spectre d'emblée
D - Une hydratation par voie veineuse avec alcalinisation
E - Des concentrés de globules blancs
Bonne(s) réponse(s) : A C D

Sans commentaire.

La survenue ultérieure chez cette patiente d'une insuffisance rénale pourrait être attribuée à :
A - Précipités de chaînes légères
B - Hyperhydratation
C - Existence d'une hypercalcémie
D - Septicémie à Gram négatif
E - Traitement éventuel par aminosides
Bonne(s) réponse(s) : A C D E

Sans commentaire.

La chimiothérapie prescrite chez cette patiente peut se compliquer de :


A - Polyglobulie
B - Insuffisance médullaire
C - Neuropathie
D - Cystite
E - Fibrose médullaire
Bonne(s) réponse(s) : CD

B - Toxicité hématologique de la chimiothérapie au long cours.


C - Liée à la Vincristine.
D - Liée à l'endoxan.

754
Exclusivement sur DOC - DZ : www.doc-dz.com NADJI 85
RESIDANAT EN POCHE TOME II
Cas Clinique en QCM
Monsieur P. Paul, 75 ans, a été hospitalisé en réanimation respiratoire depuis 12 jours en raison d'un état de choc lié à une
septicémie à Gram négatif. Outre les soins de réanimation comprenant une prévention de la maladie thromboembolique par
héparinothérapie IV (1000 UI toutes les 2 heures), il est perfusé par voie intraveineuse avec du soluté salé physiologique et du
sérum glucosé isotomique. A l'occasion de la surveillance biologique systématique, on relève le bilan suivant, communiqué
aussitôt au service :
Taux de prothrombine : 10 % ; TCA = 81''/32''; (V = 100 %; VII + X = 13 % ; II = 16 %) ; temps de thrombine = 36''/17" ; temps
de reptilase = 14"/13''; fibrinémie= 8 g/l.
- GR = 3,57.10 exposant 6/mm3; Hb = 11,1 g/dl; VGM = 90,3 micro3, CGM Hb = 34,5%.
- GB = 1 4.375/mm3 (PNN = 80 %, PE = 2 %, L = 13 %, Mo = 5 %).

Dans ce cas, quel diagnostic parmi les suivants peut-on invoquer ?


A - Avitaminose K
B - CIVD
C - Insuffisance hépatique sévère
D - Surdosage en héparine
E - Prélèvement fait par erreur sur Sintrom®
Bonne(s) réponse(s) : A

Tous les signes du diagnostic biologique d'une avitaminose K sont réunis :


- temps de Quick allongé
- TCK allongé
- facteur II, VII, X et IX diminués
- facteur V normal
- fibrinogène normal.
On aurait aussi : plaquettes normales.

Dans ce cas précis, l'allongement du TCA est consécutif à :


A - Déficit en facteur VII
B - Déficit en facteur II
C - Déficit en facteur IX
D - Hyperfibrinémie
E - Héparinothérapie
Bonne(s) réponse(s) : C E

L'allongement du TCA est lié à l'avitaminose K par le biais du déficit en F IX qui est un facteur vitamine K dépendant
synthétisé au niveau du foie.

Parmi les éléments suivants capables d'allonger le temps de thrombine, indiquez celui qu'il faut invoquer en
premier pour expliquer l'allongement du temps de thrombine dans ce cas particulier :
A - Présence d'héparine
B - Présence de PDF
C - Dysfibrinogénémie
D - Hyperfibrinémie
E - Dysglobulinémie
Bonne(s) réponse(s) : A

Sans commentaire.

Peu de temps après la réception des résultats par le service on décide de replacer un cathéter sous-clavier.
Dans ces conditions indiquez parmi les propositions suivantes celle(s) que vous retiendrez pour corriger
rapidement l'anomalie de l'hémostase :
A - Injection IV de 60 ml de PPSB
B - Injection IM de 50 mg de vitamine KI
C - Arrêt de l'héparinothérapie
D - Perfusion de deux poches de plasma frais congelé
E - Perfusion d'un flacon de solution d'ATT III
Bonne(s) réponse(s) : B C

La vitamine K sera injectée idéalement en perfusion intraveineuse courte de 100 à 150 mg à répéter 24 heures après.
L'injection intraveineuse sous forme de Bolus est à prescrire car elle peut s'associer à des réactions de type choc
anaphylactique.

755
Exclusivement sur DOC - DZ : www.doc-dz.com NADJI 85
RESIDANAT EN POCHE TOME II
Cas Clinique en QCM
Monsieur K., 69 ans, consulte pour une anémie et des troubles de la marche. Ses troubles, à type d'asthénie et de dyspnée
d'effort ont commencé il y a six mois. Depuis 15 jours, le patient est gêné pour marcher et s'est donc inquiété. Il est pâle. L'état
général est assez bien conservé; le patient est subictérique. Il n'y a pas de purpura et on ne palpe ni foie, ni rate, ni
adénopathie. La langue est dépapillée. L'hémogramme montre les résultats suivants :

G.R. 1 500 G/l G.B.


4.5 T/l (4 500/mm3)
Hb 7 g/dl dont :P.N. :5 %, L : 52%,M : 3%, Plaquettes : 110 000mm3
Ht 21 %
VGM 140 Micron-cube C G M N 33.3 %
L'examen neurologique montre une diminution des réflexes rotuliens et achilléens, une perte du sens de la position des orteils
des deux côtés. Le diagnostic final est celui d'anémie de Biermer compliquée d'un syndrome neuro-anémique.

Parmi les anomalies biologiques suivantes observées avant tout traitement, indiquez celle(s) qui sont
expliquées par ce diagnostic :
A - Hyperuricémie (600 mmol/l)
B - Réticulocytose 30 000/mm3
C - Bilirubine non conjuguée 30 micromol/l
D - Hypersegmentation des polynucléaires neutrophiles
E - Augmentation des transaminases (1,5 fois la normale)
Bonne(s) réponse(s) :A B C D

L'hyperuricémie est expliquée par les signes d'hémolyse intramédullaire. On pourrait aussi observer une augmentation des
LDH et du fer sérique.
Il s'agit d'une anémie macrocytaire arégénérative.
Les granulocytes sont de grande taille avec des noyaux hypersegmentés et cette anomalie est à la fois la plus précoce et la
dernière à disparaître.

Parmi les signes neurologiques suivants, lequel ou lesquels pourrai(en)t s'expliquer par le syndrome neuro-
anémique ?
A - Signe de Babinski
B - Anesthésie des membres inférieurs
C - Douleur des membres inférieurs
D - Aréflexie achilléenne
E - Nystagmus
Bonne(s) réponse(s) : A B C D

Sans commentaire.

Parmi les résultats suivants, quels sont les éléments du diagnostic de maladie de Biermer ?
A - Taux d'acide folique sanguin élevé
B - Absence d'anticorps anti-FI
C - Test de Shilling anormal, corrigé par l'adjonction de facteur intrinsèque
D - Achlorhydrie histaminorésistante
E - Taux normal du Fl dans le liquide gastrique
Bonne(s) réponse(s) : C D

Sans commentaire.

Parmi les symptômes suivants indiquez celui(ceux) qu'un traitement antérieur par la vitamine B12 aurai(en)t pu
modifier :
A - Mégaloblatose médullaire
B - L'aspect de la langue
C - L'atrophie gastrique
D - La malabsorption de la vitamine B12
E - Le chiffre des plaquettes
Bonne(s) réponse(s) : A B E

Sans commentaire.

756
Exclusivement sur DOC - DZ : www.doc-dz.com NADJI 85
RESIDANAT EN POCHE TOME II
Cas Clinique en QCM
Une jeune femme de 24 ans, sans antécédent particulier, présente brutalement au décours d'un épisode infectieux
rhinopharyngé, rapidement résolutif sans traitement, un purpura pétéchial étendu. A l'examen, pas de syndrome tumoral. On
retrouve un purpura pétéchial sur les 2 membres inférieurs et sur la partie supérieure du tronc. Quelques ecchymoses sont
présentes. Il existe des bulles hémorragiques buccales. La malade présente quelques céphalées. En outre, les règles
apparues depuis 8 jours, à leur date normale, sont toujours présentes. Il n'y a pas d'autre symptomatologie fonctionnelle. Le
reste de l'examen clinique, ne montre aucun signe pathologique. L'hémogramme fait en urgence révèle une thrombopénie
majeure à 10 000 plaquettes/mm3, le reste de la numération formule sanguine est dans les limites de la normale.

Dans le cadre de cette thrombopénie, outre le syndrome hémorragique présent, quelle(s) autre(s)
manifestation(s) hémorragique(s) devez-vous redouter ?
A - Un hématome du psoas
B - Une hémorragie cérébro-méningée
C - Une hématurie
D - Une hémarthrose spontanée du genou
E - Une épistaxis
Bonne(s) réponse(s) : D E

A, C et D sont des complications observées classiquement à l'occasion des pathologies constitutionnelles de la coagulation.

Quel(s) examen(s) allez-vous demander en urgence ?


A - Groupe sanguin et recherche d'agglutinines irrégulières
B - Examen du fond d'oeil
C - Ponction lombaire avec étude cytologique du LCR
D - Echographie splénique
E - Hystérographie
Bonne(s) réponse(s) : A B

Sans commentaire.

Pour faire le diagnostic étiologique vous demandez :


A - Echographie splénique
B - Etude du myélogramme
C - Sérologie HIV
D - Recherche de facteurs antinucléaires
E - Mesure de la masse sanguine
Bonne(s) réponse(s) : B C D

Sans commentaire.

Le diagnostic finalement retenu est celui de purpura thrombopénique auto-immun. Dès lors vous prescrivez :
A - Transfusion de plasma frais
B - Injection intramusculaire de vitamine K1
C - Prednisone 1,5 mg/kilo/jour
D - Transfusion de plaquettes
E - Plasmaphérèses
Bonne(s) réponse(s) : C

Il s'agit d'une forme grave. Il serait logique d'associer à la corticothérapie d'emblée un traitement pur gammaglobulines
polyvalentes intraveineuse à la dose de 400 mg/kg/jour, 4 jours de suite.

La splénectomie est réalisée, et entraîne une rémission du purpura thrombopénique. On peut voir en relation
avec la splénectomie :
A - Présence de corps de Jolly intra-érythrocytaire
B - Excès de basophiles circulants
C - Thrombocytose à 600 000 plaquettes/mm3
D - Augmentation des réticulocytes
E - Lymphopénie
Bonne(s) réponse(s) : A C

La splénectomie s'accompagne d'hyperlymphocytose.

757
Exclusivement sur DOC - DZ : www.doc-dz.com NADJI 85
RESIDANAT EN POCHE TOME II
Cas Clinique en QCM
A l'occasion de ses premières règles une jeune fille de 13 ans, présente des ménorragies profuses que diverses
thérapeutiques "coagulantes" (Reptilase, vitamine K1, Hémocaprol) n'ont pu arrêter. Elle est hospitalisée avec un état
d'anémie sévère ; on observe : dyspnée, vertiges, lipothymies.
L'examen clinique montre une pâleur impressionnante, une tachycardie régulière, sans organomégalie. L'auscultation
pulmonaire est normale.
Le bilan sanguin demandé en urgence apporte les éléments suivants :
Globules blancs : 13 000/mm3
Globules rouges : 1 820 000mm3
Hémoglobine : 5 g/100 ml
Hématocrite : 17 %
Volume globulaire moyen : 94 µ3
Hémoglobine glob. moyenne : 30 microg
Conc. hémo. glob. moyenne : 30 %
Plaquettes : 319 000/mm3
Myélocytes 6
Métamyélocytes 6
Erythroblastes : quelques.
Réticulocytes = 233 x 10 exposant 9/l
Coombs négatif
T.S. 12 minutes (normal inférieur ou égal à 5 minutes)
TCK 65 sec (T = 34 sec )
TP 100 %
Fibrinogène 4, 69/l.
Le bilan d'hémostase objective :
Facteur XII 92 %
Facteur XI 87 %
Facteur IX 112 %
Facteur VIII 26 %.
L'interrogatoire apprend que le père a du être abondamment transfusé à l'occasion d'une intervention chirurgicale mineure ;
trois autres enfants et la mère sont en parfaite santé.

Avant d'instituer un traitement, un autre examen complémentaire est indispensable pour lequel le prélèvement
doit être fait de suite sous peine de voir les résultats modifiés par la thérapeutique. Lequel ?
A - Dosage de l'antithrombine
B - Immuno-électrophorèse
C - Myélogramme
D - Dosage du facteur VII
E - Dosage de l'antigène Willebrand
Bonne(s) réponse(s) : E

Il s'agit probablement d'une maladie de Willebrand étant donné le tableau d'hémostase avec baisse modérée du F VIII
coagulant, allongement du TCK avec un temps de Quick normal, allongement du TS en ivy. La transmission autosomique
dominante probable va aussi en faveur de ce diagnostic. On rappelle qu'il existe cependant des formes autosomiques
récessives de la maladie.

Vous décidez de transfuser pour corriger l'anémie ; il s'agit d'une première transfusion : quelle(s) est (sont) la ou
les détermination(s) que vous demanderez au préalable ?
A - Groupage simple ABO et rhésus standard
B - Phénotype érythrocytaire complet
C - Groupage HLA
D - Recherche d'anticorps irréguliers anti-érythrocytaires
E - Recherche d'anticorps anti-leucocyto-plaquettaires
Bonne(s) réponse(s) : B D

Sans commentaire.

Par quel produit thérapeutique traiterez-vous le syndrome hémorragique ?


A - Concentré plaquettaires standard
B - Fraction P.P.S.B.
C - Fraction anti-hémophilique A
D - Plasma frais congelé dépourvu de cryoprécipité
E - Plasma lyophilisé
Bonne(s) réponse(s) : C

Il s'agit de cryoprécipité lyophilisé qui apporte le facteur Willehand à raison d'environ 900 UI de VIII : AG. L'action sur le temps
de saignement étant brève, il faudra renouveler les injections toutes les huit heures. Elle est obtenue avec 30 U/kg au moins.

758
Exclusivement sur DOC - DZ : www.doc-dz.com NADJI 85
RESIDANAT EN POCHE TOME II
Cas Clinique en QCM

Un hémogramme comparable à celui qui accompagne cette observation peut s'observer dans une ou plusieurs
des situations ci-dessous ; laquelle ou lesquelles ?
A - Leucémie myéloïde chronique
B - Hémolyse aiguë
C - Leucémie aiguë
D - Aplasie médullaire
E - Infection bactérienne aiguë
Bonne(s) réponse(s) : B E

Il s'agit des causes étiologiques d'hyperleucocytose avec myélémie. La LMC n'est pas à retenir ici car le tableau typique n'a
aucune raison de comporter une anémie régénérative.

Dans le cas présent, quelle(s) proposition(s) ferez-vous pour l'avenir ?


A - Corticothérapie au long cours
B - Hormonothérapie (progestatifs)
C - Traitement substitutif régulier chaque semaine
D - Vaccination contre l'hépatite B
E - Interruption systématique des grossesses ultérieures éventuelles
Bonne(s) réponse(s) :

QUESTION ANNULEE.

Un traitement permet dans certains cas d'accroître le taux plasmatique de la protéine déficitaire au cours de
cette maladie ; quel est-il ?
A - Indométacine
B - Propranolol
C - D-amino D-Arginine Vasopressine (DDAVP)
D - Acide folinique
E - Vitamine B6
Bonne(s) réponse(s) : C

Sans commentaire.

Une femme de 42 ans présente un état anémique.


L'interrogatoire révèle que depuis deux ans il existe des malaises (2 ou 3) à type de lipothymies avec sueurs et impression de
lourdeur abdominale.
Depuis 6 mois, les troubles deviennent permanents et s'accentuent : asthénie, dyspnée d'effort et palpitation. La pâleur
apparaît évidente. On ne retrouve pas d'autres troubles digestifs. Les règles sont régulières (durée 4 jours, sans caillot),
l'évaluation de leur volume est normal.
L'examen clinique est normal.
Hémogramme :
- globules rouges 3 500 000/mm3
- hémoglobine = 7,5 g/dl
- hématocrite = 26 %
- VGM = 74 fl
- TCMH = 22 pg
- CCMH = 28 %
- plaquettes = 520 G/l
- globules blancs = 12,8 G/l
¥ polynucléaires neutrophiles 80 %
¥ polynucléaires éosinophiles 5 %
¥ polynucléaires basophiles 0 %
¥ lymphocytes 12 %
¥ monocytes 3%
Vitesse de sédimentation, 1ère heure = 35
Fer sérique : 4,2 micromol/l
Le diagnostic d'anémie ferriprive est retenu.

Parmi les caractères suivants lequel (lesquels) peu(ven)t être expliqué(s) par la carence martiale ?
A - Le taux des éosinophiles
B - Le chiffre de plaquettes
C - Le VGM
D - Le taux de polynucléaires neutrophiles
E - Le taux de Iymphocytes
Bonne(s) réponse(s) : B C

On peut observer lors des carences martiales une tendance à la leuconeutropénie modérée. Il peut y avoir également soit
thrombopénie, soit plus souvent thrombocytose.

759
Exclusivement sur DOC - DZ : www.doc-dz.com NADJI 85
RESIDANAT EN POCHE TOME II
Cas Clinique en QCM

Quel examen choisissez vous pour confirmer la carence martiale ?


A - Capacité totale de fixation de la sidérophiline
B - Dosage de l'haptoglobine
C - Dosage de la ceruloplasmine
D - Hémosidérrurie
E - Myélogramme avec coloration de Perls
Bonne(s) réponse(s) : A

Sans commentaire.

La carence martiale étant affirmée, citez les deux examens qui vous paraissent d'emblée les plus adaptés pour
découvrir l'étiologie :
A - Coloscopie
B - Fibroscopie gastrique
C - Hystérographie
D - Urographie intra-veineuse
E - Scanner abdominal
Bonne(s) réponse(s) : A B

A cet âge, la première cause de carence martiale est gynécologique. Cependant, en l'absence d'un interrogatoire précis
orientant vers une cause gynécologique et en présence d'un examen gynécologique normal, il faut envisager ensuite les
causes digestives qui seront au mieux initialement explorées par la fibroscopie gastrique et la coloscopie. Les causes rares
seront recherchées ensuite seulement.

Si l'étiologie n'est pas rapidement trouvée, quelle attitude vous paraît la plus adaptée à la situation ?
A - Transit du grêle
B - Recherche d'une hématurie microscopique
C - Etude de l'absorption du fer
D - Etude de l'incorporation du fer 59
E - Scintigraphie abdominale avec globules rouges marqués
Bonne(s) réponse(s) : A

Sans commentaire.

Quelle que soit l'étiologie il est nécessaire de corriger la carence martiale. Quelle prescription journalière faites-
vous ?
A - 10 mg de fer métal per os
B - 100 mg de fer métal per os
C - 100 mg de fer métal par voie intramusculaire
D - 1 g de fer métal per os
E - 1 g de fer métal par voie intramusculaire
Bonne(s) réponse(s) : B

Sans commentaire.

Surveillance. Ce traitement sera poursuivi :


A - 1 mois
B - Jusqu'à normalisation du taux d'hémoglobine
C - Jusqu'à normalisation du V.G M
D - Jusqu'à normalisation du fer sérique
E - Jusqu'à normalisation de la capacité totale de saturation de la sidérophiline
Bonne(s) réponse(s) : E

Sans commentaire.

760
Exclusivement sur DOC - DZ : www.doc-dz.com NADJI 85
RESIDANAT EN POCHE TOME II
Cas Clinique en QCM
Un homme de 32 ans consulte pour une discrète baisse de l'état général et une pesanteur de l'hypochondre gauche.
L'examen met en évidence une splénomégalie débordant de 4 centimètres le rebord costal gauche. Il n'y a aucune autre
anomalie clinique.
La numération formule sanguine est la suivante :
G R = 4 700 000/mm3 Hb = 13,5 g/dl Hte : 44 %
G.B. = 87 000 PN = 38 % PE = 2 % PB = 13 % L = 5 %
Myéloblastes = 2 % Promyélocytes = 5 %
Myélocytes = 19 % Métamyélocytes = 16 %
Plaquettes = 485 000/mm3.

Quel est (sont) le (les) diagnostic(s) compatible(s) avec ces données ?


A - Leucémie lymphoïde chronique
B - Leucémie myéloïde chronique
C - Leucémie aiguë Iymphoblastique
D - Leucémie aiguë myéloblastique
E - Leucémie à tricholeucocytes
Bonne(s) réponse(s) : B

Sans commentaire.

Vous évoquez un ou plusieurs diagnostic et cherchez à le confirmer par :


A - L'examen cytologique de la moelle
B - Le groupage HLA
C - L'immunoélectrophorèse
D - Le caryotype médullaire
E - L'examen tomodensitométrique de la rate
Bonne(s) réponse(s) : D

Sans commentaire.

Le diagnostic de leucémie myéloïde chronique est retenu, quelle est la durée médiane de survie dans cette
affection ?
A - Identique à la population saine du même âge
B - 3 à 6 mois
C - 1 à 2 ans
D - 2 à 4 ans
E - 5 à 10 ans
Bonne(s) réponse(s) : D

Sans commentaire.

Parmi les complications suivantes, on pourra observer chez ce malade :


A - Goutte
B - Priapisme
C - Erythème noueux
D - Polynévrite des membres inférieurs
E - Infarctus splénique
Bonne(s) réponse(s) : A B E

Sans commentaire.

Parmi les évolutions suivantes, laquelle (lesquelles) est (sont) susceptible(s) de survenir chez ce patient ?
A - Transformation en leucémie aiguë Iymphoblastique
B - Transformation en leucémie aiguë myéloblastique
C - Transformation en maladie de Hodgkin
D - Myélofibrose
E - Transformation en leucémie Iymphoïde chronique
Bonne(s) réponse(s) : A B D

La myélofibrose se voit dans les formes évoluées souvent en association à la transformation aigue.
La transformation se fait essentiellement sur le mode myéloblastique 2 à 3 ans après le diagnostic de LMC. Ces formes sont
chimiorésistantes et de très mauvais pronostic.

761
Exclusivement sur DOC - DZ : www.doc-dz.com NADJI 85
RESIDANAT EN POCHE TOME II
Cas Clinique en QCM

Cette maladie ouvre droit à une déclaration de maladie professionnelle pour :


A - Mineur de charbon
B - Manipulateur en échographie
C - Peintre au pistolet
D - Travailleur de l'aniline
E - Soudeur à l'arc
Bonne(s) réponse(s) :

QUESTION ANNULEE.

Un homme de 60 ans consulte pour dyspnée d'effort. Il existe une pâleur modérée, pas d'anomalie à l'examen du coeur et des
poumons, pas d'adénopathie ou de splénomégalie. A l'hémogramme : hémoglobine 9 g/dl, VGM 90 fl, leucocytes 7.10 9/l, (PN
65 - PE 2 - L 30 - M 3), plaquettes 255.10 exposant 9/l.
Vitesse de sédimentation 105 - 140; fibrinogène 3 g/l ; fer sérique : 20 micromol/l. Protéinémie 95 g/l dont 55 g/l de
gammaglobulines en pic à base étroite ; l'immunoélectrophorèse montre une importante augmentation monoclonale d'IgG
Kappa.
On retient comme probable, le diagnostic de myélome multiple.

L'(es) argument(s) contre le diagnostic d'anémie inflammatoire est (sont) ?


A - Hémoglobine 9 g/dl
B - Leucocytes 7 10 exposant 9/l
C - Fibrinogène 3 g/l
D - Gammaglobulines 35 g/l
E - Fer sérique 20 micromol/l
Bonne(s) réponse(s) : C E

Sans commentaire.

L'(es) argument(s) contre le diagnostic d'immunoglobuline monoclonale bénigne est (sont) :


A - Hémoglobine 9 g/dl
B - Vitesse de sédimentation 105 - 140
C - Fibrinogène 3 g/l
D - Gammaglobulines 55 g/l
E - Composant monoclonal IgG Kappa
Bonne(s) réponse(s) :

QUESTION ANNULEE.

Le diagnostic de maladie de Waldenstrِm n'est pas retenu en raison de :


A - Hémoglobine 9 g/dl
B - Vitesse de sédimentation 105 - 140
C - Fibrinogène 3,7 g/l
D - Gammaglobulines 55 g/l
E - Composant monoclonal IgG Kappa
Bonne(s) réponse(s) : E

La maladie de Waldenstrِm est une affection caractérisée par la présence d'une immunoglobuline M monoclonale à un taux
supérieur à 5 g/l et d'une infiltration lymphoïde médullaire le plus souvent polymorphe lymphoplasmocytaire.

Chez ce malade, le diagnostic de myélome multiple sera confirmé au vu d'un ou de plusieurs des résultats
suivants : lequel ou lesquels ?
A - Présence de lésions ostéolytiques sur les radiographies du squelette
B - Hyperuricémie
C - Hypocalcémie
D - Protéinurie 0,50 g/24 h, non sélective à l'électrophorèse
E - Plasmocytose médullaire supérieure à 20 %
Bonne(s) réponse(s) : A E

Sans commentaire.

762
Exclusivement sur DOC - DZ : www.doc-dz.com NADJI 85
RESIDANAT EN POCHE TOME II
Cas Clinique en QCM

Le diagnostic de myélome multiple a été confirmé. Un traitement associant Melphalan (Alkeran®) et


Prednisone® est instauré. Ce traitement expose à une ou plusieurs des complications suivantes, la ou
lesquelles ?
A - Insuffisance médullaire
B - Neuropathie périphérique
C - Insuffisance cardiaque
D - Hyperglycémie
E - Leucémie aiguë secondaire
Bonne(s) réponse(s) : A D E

Les complications du Melphalan sont :


- hématologiques touchant toutes les lignées
- gastro-intestinales à type de nausées, vomissements, mucites, diarrhées
- alopécie
- rénales rares à type d'augmentation de l'urée
- pulmonaires avec des cas de fibrose pulmonaire extensive
- des réactions d'hypersensibilité au Melphalan IV sont décrites
- risque de leucémies aigues secondaires.

Monsieur Z. , 71 ans, consulte pour une angine fébrile avec dysphagie importante. Il est en assez bon état général. Il déclare
avoir maigri de 4 à 5 kg depuis un an. A l'examen, existent une hypertrophie bilatérale des amygdales, de multiples
adénopathies cervicales et sus-claviculaires mobiles et indolores ; la palpation abdominale révèle une splénomégalie
débordant de 6 cm le rebord costal. Les téguments sont discrètement décolorés. l'hémogramme donne les résultats suivants :
GR = 3,6.1012/l, GB = 86.10 exposant 9 g/l, Hb=118 g/l, Ht = 0,34, Plaq = 112.10 exposant 9/l.
Formule leucocytaire PNn 0,18 PNeo 0,02 Lc 0,78 Mo 0,02.
La VS est augmentée à 56 mm à la première heure; la radiographie de thorax ne montre pas d'image anormale médiastinale
ou parenchymateuse.

Quel diagnostic évoquez-vous ?


A - Mononucléose infectieuse
B - Leucémie aiguë
C - Maladie de Hodgkin
D - Leucémie lymphoïde chronique
E - Leucémie myéloïde chronique
Bonne(s) réponse(s) : D

Il existe une très importante hyperleucocytose à 86000/mm3 faite de lymphocytes. La thrombopénie et l'anémie modérée
entrant bien dans le tableau de LLC. Il s'agirait d'un stade C de la classification de Binet.

Indiquez les deux examens qui vous paraissent les plus importants pour compléter votre bilan hématologique :
A - Vitamine B12 sérique
B - Examen tomodensitométrique du thorax
C - Myélogramme
D - Electrophorèse des protéines sériques
E - Dosage du lysozyme urinaire
Bonne(s) réponse(s) : C D

- Le myélogramme va mettre en évidence une infiltration par des petits lymphocytes. La biopsie médullaire sera cependant
réalisée, confirmant le diagnostic mais aussi apportant un élément pronostic en montrant le caractère local ou diffus de
l'infiltration lymphocytaire. Ce dernier caractère étant associé à un pronostic plus défavorable.
- L'électrophorèse des protéines sériques est indispensable devant un tableau de LCC pour apprécier le déficit immunitaire
humoral souvent associé à type le plus souvent d'hypogammaglobulinémie. On pourra parfois observer cependant un petit pic
monoclonal.

Quelles sont les complications classiques que vous pouvez redouter ?


A - Syndrome hémorragique
B - Transformation en leucose aiguë myéloblastique
C - Anémie hémolytique auto-immune
D - Infections sévères
E - Thromboses vasculaires
Bonne(s) réponse(s) : A C D

- Syndrome hémorragique en cas de stade C très évolutif avec thrombopénie profonde ou thrombopénie d'autre cause (Cf
commentaire de la question suivante).
- Anémies hémolytiques à test de Coombs positif le plus souvent IgG + complément.

763
Exclusivement sur DOC - DZ : www.doc-dz.com NADJI 85
RESIDANAT EN POCHE TOME II
Cas Clinique en QCM

Si la thrombopénie se majore, quelles sont les hypothèses qu'il vous paraît raisonnable d'évoquer ?
A - Hypersplénisme
B - Insuffisance médullaire
C - Coagulation intravasculaire latente
D - Thrombopénie de mécanisme immunologique
E - Carence en fer
Bonne(s) réponse(s) : A B D

On pourrait rajouter dans les causes de thrombopénies au cours de la LLC les thrombopénies iatrogènes liées aux traitements
chimiothérapiques.

Si au cours de l'évolution, vous constatez la conjonction des signes suivants : altération de l'état général, fièvre,
augmentation rapide de volume d'une des adénopathies, à quoi penserez-vous en priorité ?
A - Adénite bacillaire
B - Evolution lymphomateuse
C - Adénite suppurée
D - Toxoplasmose
E - Intolérance médicamenteuse
Bonne(s) réponse(s) : B

Sans commentaire.

Une femme de 45 ans, mère de 3 enfants, sans antécédent pathologique particulier, consulte pour une asthénie marquée
évoluant depuis plusieurs mois.
L'état général reste satisfaisant, l'examen clinique est négatif en dehors d'une pâleur des téguments et des muqueuses et d'un
subictère conjonctival. L'hémogramme montre :
- Hémoglobine : 9,5 g/dl . VGM = 105 micron-cube - CCMM = 33 %
- Leucocytes : 4, 5 x 10 exposant 9/l
- Formule leucocytaire :
* Polynucléaires neutrophiles : 60 %
* Polynucléaires éosinophiles : 2 %
* Lymphocytes : 33 %
* Monocytes : 5 %
- Plaquettes : 160 x 10 exposant 9/l
- Réticulocytes : 300 x 10 exposant 9/l
- VS : 60 mm à la première heure

Cet hémogramme révèle :


A - Une anémie normocytaire
B - Une anémie mégaloblastique
C - Une anémie macrocytaire
D - Une pancytopénie
E - Une leucopénie et une anémie
Bonne(s) réponse(s) : C

Sans commentaire.

Indiquez, parmi les examens suivants, celui qui vous parait à ce stade le plus utile pour en préciser le diagnostic
:
A - Estimation des phosphatases alcalines leucocytaires
B - Dosage de la bilirubine totale et non conjuguée
C - Temps de Quick
D - Myélogramme
E - Dosage du fer sérique
Bonne(s) réponse(s) : B

Il s'agit d'une anémie macrocytaire régénérative (taux de méticulocytes à 300.000/mm3). Le premier diagnostic à évoquer est
une origine hémolytique.

764
Exclusivement sur DOC - DZ : www.doc-dz.com NADJI 85
RESIDANAT EN POCHE TOME II
Cas Clinique en QCM

Parmi les diagnostics suivants, quels sont chez cette malade les deux les plus probables sachant que la
réaction de Coombs directe est positive (de type mixte IgG + C) :
A - Anémie de Biermer
B - Myélome multiple
C - Lupus érythémateux disséminé
D - Syndrome de Moschowitz
E - Anémie hémolytique auto-immune idiopathique
Bonne(s) réponse(s) : C E

Sans commentaire.

Le traitement à proposer en première intention est :


A - Vitamine B12 par voie intramusculaire (1000 microgr/jour)
B - Sel ferreux per os (soit 100 mg de fer/jour pendant 3 mois)
C - Prednisone ( 1 à 1, 5 mg/kg/jour)
D - Acide folinique par voie intramusculaire (25 mg/jour)
E - Transfusions de concentrés érythrocytaires (2 unités par semaine)
Bonne(s) réponse(s) : C

Sans commentaire.

En cas d'échec de ce premier traitement, quelle option thérapeutique doit être alors proposée ?
A - Azathioprine (Imurel)
B - Splénectomie
C - Androgènes à fortes doses
D - Procarbazine (Natulan)
E - Sérum anti-lymphocytaire
Bonne(s) réponse(s) : B

Comme autre possibilité thérapeutique d'une anémie hémolytique auto-immune aurait pu figurer un essai de traitement par
gammaglobulines polyvalentes intraveineuses à forte dose.

Un homme de 70 ans est hospitalisé pour syndrome anémique d'installation progressive.Traité par vagotomie-pyloroplastie
vingt ans plus tôt pour ulcère il n'a jamais plus souffert depuis. Le transit digestif est normal, les selles de coloration normale.
Depuis six mois, son alimentation comporte essentiellement des pommes de terre, des pâtes et des conserves de viande et
de poisson .
L'examen montre un embonpoint conservé, un teint pâle. Hémogramme : hémoglobine 7 g/dl, hématocrite G.R. =
1800000/mm3 ; réticulocytes 20000 par mcl. Leucocytes 3100/mm3 = 21 % (neutrophiles 40%, éosinophiles 2%, Iymphocytes
50 %, monocytes 8 %), plaquettes 110000/mm3.
Sur le frottis, présence de polynucléaires hypersegmentés. Bilirubine non conjuguée 30 mcmol/l, bilirubine conjuguée 5
mcmol/l. Sidérémie 25 mcmol/l coefficient de saturation de la sidérophiline 0,50. Absence d'auto-anticorps antiérythrocytes.

L'anémie peut être décrite comme :


A - Microcytaire hypochrome arégénérative
B - Microcytaire hypochrome régénérative
C - Normocytaire normochrome arégénérative
D - Macrocytaire normochrome régénérative
E - Macrocytaire normochrome arégénérative
Bonne(s) réponse(s) :

Quel mécanisme est principalement à l'origine de l'anémie ?


A - Trouble de synthèse de l'hémoglobine
B - Trouble de synthèse de l'ADN
C - Raréfaction des cellules-souches hématopoiétiques
D - Destruction excessive des érythrocytes circulants
E - Pertes excessives par hémorragie
Bonne(s) réponse(s) :

765
Exclusivement sur DOC - DZ : www.doc-dz.com NADJI 85
RESIDANAT EN POCHE TOME II
Cas Clinique en QCM

Parmi ces examens pouvant aider au diagnostic, lequel (lesquels) doit (doivent), pour être correctement
interprété(s), être effectué(s) avant tout traitement ?
A - Tubage gastrique
B - Myélogramme
C - Gastroscopie avec biopsie
D - Dosages de vitamine B12 et de folates dans le sang
E - Recherche d'anticorps anti-facteur intrinsèque
Bonne(s) réponse(s) :

On apprend que la moelle est mégaloblastique et que le test de Schilling est normal. Quels sont les deux
facteurs qui, chez ce patient, peuvent contribuer à l'installation d'une anémie mégaloblastique ?
A - Carence d'apport en vitamine B12
B - Trouble d'absorption de la vitamine B12
C - Carence d'apport en folates
D - Trouble d'absorption des folates
E - Trouble d'utilisation des folates
Bonne(s) réponse(s) :

Le traitement substitutif doit comporter :


A - Vitamine B12 intramusculaire 1000 gammas par mois à vie
B - Vitamine B12, 100 gammas par jour per os jusqu'à guérison de l'anémie
C - Acide folinique intramusculaire 5 mg par jour jusqu'à guérison de l'anémie
D - Acide folique per os, 5 mg par jour jusqu'à guérison de l'anémie
E - Fer per os 100 mg métal par jour jusqu'à guérison de l'anémie
Bonne(s) réponse(s) :

Un garçon de 11 ans, sans antécédents pathologiques, vous est adressé pour syndrome hémorragique et hyperthermie. La
température est entre 39 et 40° C depuis 48 heures et l'examen retrouve un purpura pétéchial et ecchymotique au niveau des
membres, des gingivorragies et des bulles hémorragiques endobuccales. Il existe une splénomégalie de 2 travers de doigt,
une pâleur cutanéomuqueuse, un souffle systolique de pointe à l'auscultation cardiaque.
L'étude du fond d'oeil montre une petite hémorragie près de la papille de l'oeil droit.
L'hémogramme révèle :
- Hémoglobine : 6 g/dl
- Leucocytes : 2,5 . 10 exposant 9/l dont :
Polynucléaires neutrophiles : 20 %
Lymphocytes : 75 %
Monocytes : 5 %
- Plaquettes : 10 . 10 exposant 9/l
Le taux de prothrombine, le temps de céphaline activé, le dosage du fibrinogène sont normaux. Les explorations biologiques
usuelles hépatiques, rénales et métaboliques sont normales Le myélogramme révèle, dans une moelle riche, 85 % de cellules
peu différenciées, à rapport nucléo-cytoplasmique élevé, sans granulations cytoplasmiques.

Le diagnostic retenu est celui de leucémie aiguë Iymphoblastique (LAL). Un autre diagnostic pourrait-il être
évoqué, au vu du tableau clinique, de l'hémogramme et du myélogramme ?
A - Aplasie médullaire
B - Mononucléose infectieuse
C - Etat d'hypersplénisme
D - Leucémie Iymphoïde chronique
E - Aucune des propositions ci-dessus
Bonne(s) réponse(s) : E

Sans commentaire.

766
Exclusivement sur DOC - DZ : www.doc-dz.com NADJI 85
RESIDANAT EN POCHE TOME II
Cas Clinique en QCM

Certain(s) geste(s) s'impose(nt) chez ce malade :


A - Réalisation de 6 à 8 hémocultures sur 24 heures puis mise en route d'une antibiothérapie double par voie
veineuse
B - Même antibiothérapie, mais mise en route en moins de 3 heures pendant lesquelles on réalise 2 ou 3
hémocultures
C - Transfusion de concentrés plaquettaires
D - Transfusion de plasma frais
E - Transfusion de concentrés érythrocytaires
Bonne(s) réponse(s) : B C E

- L'antibiothérapie en cas de neutrogénie fébrile, quelque soit son étiologie, sera immédiate (à débuter dans les 3 heures au
maximum), à large spectre et adaptée si besoin (systématiquement une céphalosporine de 3ème génération associé à un
aminocide avec, suivant le terrain ou la gravité initiale, un traitement anti-staphylococique par vancomycine), intraveineuse.
- Il n'y a pas d'indication évidente d'emblée aux transfusions de plasma frais en l'absence de signes de choc ou de CIVD
associé avec baisse du fibrinogène.
- Les transfusions de concentrés plaquettaires doivent être immédiats, s'agissant idéalement de CUP (Concentrés Unitaires
de Plaquettes) en situation de malade hématologique mais le plus souvent de plaquettes de Pool.

Au cours d'une LAL commune de l'enfant, en première poussée, l'on retrouve aussi fréquemment :
A - Des localisations cutanées spécifiques
B - Des douleurs osseuses, avec aspect radiologique en bandes claires métaphysaires
C - Une hyperplasie gingivale
D - Une positivité de la réaction cytochimique des myélopéroxydases
E - La présence de corps d'Auer dans quelques blastes
Bonne(s) réponse(s) : B

Toutes les autres propositions caractérisent ici les leucémies aiguës myéloblastiques. A et C s'observent essentiellement dans
les formes M4 et M5 des LAM, dans la classification cytologique FAb (Franco-Américano-britanique).

Chez cet enfant, le relativement "bon" pronostic à terme s'appuie sur :


A - L'absence d'hyperleucocytose
B - La présence d'une splénomégalie
C - L'absence de tuméfaction médiastinale
D - L'absence de localisation neuroméningée
E - L'absence d'hépatomégalie et d'adénopathies superficielles
Bonne(s) réponse(s) : A C D E

Rappelons les facteurs de mauvais pronostic des LAL de l'enfant au diagnostic :


- l'âge inférieur à 1 an et supérieur à 10 ans,
- la race noire,
- le sexe masculin,
- l'hyperleuccocytose supérieure à 10.000/m3 et très défavorable au dessus de 100.000/m3,
- le syndrome tumoral avec grosse rate, hépatomégalie, adénopathies volumineuses, masse médiastinale,
- un taux d'hémoglobine supérieur à 10g/100 est de mauvais pronostic,
- une atteinte du SNC initiale,
- la cytologie appréciée par la classification FAb avec surtout le type L3 ou leucémie de type burkitt de mauvais pronostic,
- l'immunologie avec un pronostic bon pour le formes pré-pré B (calla positives, CD10 positives), des formes T et pré B de
moins bon pronostic,
- la cytogénétique avec dans l'ordre allant dans le sens d'un pronostic, de plus en plus défavorable : les hyperdiploidies
supérieures à 56 de très bon pronostic, hyperdiploidies entre 47 et 50, caryotypes normal, les déletions et translocations de
très mauvaix pronostic notamment les
t (9, 22) ; t (8, 14) ; t (8, 22).

Le protocole de chimiothérapie d'induction associera chez cet enfant : Prednisone, Vincristine et Daunorubicine.
Une rémission complète est obtenue. Les autres mesures essentielles et d'intérêt démontré après l'obtention de
la rémission comporteront :
A - Une irradiation en mantelet
B - Arrêt de toute chimiothérapie
C - Une prévention systématique des localisations neuroméningées
D - Une chimiothérapie d'entretien
E - Immunothérapie
Bonne(s) réponse(s) : C D

Sans commentaire.

767
Exclusivement sur DOC - DZ : www.doc-dz.com NADJI 85
RESIDANAT EN POCHE TOME II
Cas Clinique en QCM

Certaine(s) complication(s) iatrogène(s) est (sont) possibles compte tenu des médicaments utilisés en induction
chez ce malade :
A - Polynévrite
B - Fibrose pulmonaire
C - Insuffisance rénale
D - Myocardiopathie
E - Surdité
Bonne(s) réponse(s) : A D

- La vincristine est un alcaloïde de la pervenche qui bloque les mitoses avec arrêt en métaphase. Les complications en sont :
hématologiques avec leucopénie et thrombogénie ; neurologiques avec dysesthésies et paresthésies, perte des réflexes,
fatigue musculaire, céphalées, ptosis, diplopie ; digestives avec constipation sévère pouvant aller jusqu'à l'iléus réflexe ;
également nausées, alopécie, fièvre, fatigue, polyurie, douleurs diffuses, thrombophlébite au point d'injection, nécrose cutanée
grave en cas d'extravasation.
- La daunorubicine est une anthracycline. La toxicité principale est cardiaque allant des troubles ECG transitoires à la
cardiomyopathie grave en passant par les myocardites aiguës avec péricardite. Les anomalies apparaissent dès la dose
cumulative de 180 mg/m2 obtenus mais le risque est majeur à 600 mg/m2. Les autres effets secondaires sont : alopécie,
troubles digestifs, mucite et myélosuppression.

Quels sont !es trois modes habituels de rechute de LAL de l'enfant ?


A - Médullaires
B - Neuroméningées
C - Testiculaires
D - Gastriques
E - Pulmonaires
Bonne(s) réponse(s) : A B C

Sans commentaire.

Pierre D..., âgé de dix huit mois, est hospitalisé en pédiatrie pour un volumineux hématome frontal consécutif à une chute.
C'est un deuxième enfant, il a une soeur âgée de trois ans . Il est né à terme, sans problème obstétrical ni néonatal, et son
développement staturo-pondéral et psychomoteur a été normal . On a cependant noté chez lui, très tôt, l'apparition fréquente
d'ecchymoses ou d'hématomes provoqués par des traumatismes souvent minimes, et cette tendance s'aggrave depuis les
premiers pas. L'enquête ne permet de découvrir aucune prédisposition familiale hémorragique.
Le premier bilan d'hémostase a donné les résultats suivants :
- Temps de saignement (IVY = incision) : 6 minutes.
- Plaquettes : 380.10 exposant 9/l (380.000/mm3
- Temps de Quick = 88 %
- Temps de céphaline kaolin = 98 secondes pour un témoin à 35 secondes

Ce premier bilan d'hémostase peut être complété et il faudra faire en priorité :


A - Un temps de coagulation
B - Un temps de Howell
C - Un thrombo-élastogramme
D - Une étude de la rétraction du caillot
E - Aucun des examens ci-dessus
Bonne(s) réponse(s) : E

Sans commentaire.

Compte-tenu des données biologiques et du tableau clinique, quel(s) déficit(s) de l'hémostase est-il possible
d'écarter d'emblée ?
A - Prothrombine (II)
B - Proaccélérine (V)
C - Proconvertine (VII)
D - Facteur Stuart (X)
E - Aucun des déficits ci-dessus
Bonne(s) réponse(s) : A B C D

A, B, C et D s'accompagnent d'un allongement du temps de Quick.

768
Exclusivement sur DOC - DZ : www.doc-dz.com NADJI 85
RESIDANAT EN POCHE TOME II
Cas Clinique en QCM

Le déficit constitutionnel sévère en facteur Hageman (XII) a une expression biologique de base analogue à celle
de cette observation. Lequel des arguments ci-dessous peut-il être retenu contre ce diagnostic ? Le déficit en
facteur Hageman (XII) :
A - Est inexistant en Europe
B - Ne se révèle qu'à l'âge adulte
C - Ne s'exprime que par un purpura pétéchial spontané
D - N'entraîne d'hémorragies qu'en cas d'intervention chirurgicale
E - Ne s'accompagne d'aucune manifestation hémorragique
Bonne(s) réponse(s) : E

Les déficits en facteur XII comme les déficits en prékallicrénine et en kininogène de haut poids moléculaire sont des
découvertes fortuites par la constatation d'un allongement important du temps de coagulation, lors d'un examen systématique.
Le diagnostic précis repose sur le dosage spécifique du facteur. Il n'y a aucun signe hémorragique et ces déficits ne
nécessitent pas de traitement, même en situation chirurgicale.

Une hémophilie A peut expliquer l'ensemble de ce tableau. Quelle(s) donnée(s), déjà recueillies ou restant à
recueillir est (sont) en faveur de ce diagnostic plutôt que de celui de la maladie de Willebrand ?
A - Temps de saignement normal
B - Absence de thrombopénie
C - Résistance capillaire normale
D - Activité coagulante du facteur VIII (VIII c) à 1%
E - Normalité du taux de facteur Willebrand
Bonne(s) réponse(s) : A B D E

- Le temps de saignement dans la Maladie de Willebrand est augmenté en raison de la diminution du taux du facteur
Willebrand ou à des anomalies qualitatives de ce facteur.
- Il existe des formes de maladie de Willebrand avec thrombopénie notamment dans les formes II B de la maladie,
caractérisées par une augmentation de l'aggrégation plaquettaire induite à la ristocétine.
- Le taux du F VIII est diminué dans la maladie de Willebrand mais en général en 10 et 40%.
- Le taux du Facteur Willebrand est normal dans l'hémophilie A. Cependant, il peut être également normal dans les formes II
A, II B et II C de la maladie de Willebrand ou l'anomalie est qualitative.

Parmi les règles suivantes, indiquer celle(s) qu'il conviendra d'appliquer chez ce sujet hémophile :
A - Interdiction de toute activité sportive, même réduite
B - Interdiction des antalgiques salicylés
C - Interdiction des injections intramusculaires
D - Contre-indication de toute vaccination
E - Surveillance clinique articulaire et musculaire à intervalles réguliers
Bonne(s) réponse(s) : A B C E

Sans commentaire.

Une femme de 70 ans est hospitalisée pour douleurs angineuses.


Antécédents : infarctus du myocarde il y a trois ans. Pas de récidive des douleurs depuis cette date. Hystérectomie pour
fibrome il y a 20 ans.
A l'examen : pâleur, tachycardie 110/minute régulière, tension artérielle 13-7 cmHg, ECG signes de nécrose postéro-latérale
ancienne, pas de changement par rapport aux ECG antérieurs, transaminases normales.
NFS : 1.380.000/mm3 GR, Hb 6 g/dl, Hte 18,1 %, réticulocytes 3%, GB 3.600/mm3, PN 30 %, PE 5 %, Iymphocytes 60 %,
monocytes 5 %, plaquettes 110.000/mm3, VS 35-60/mm.

Quelle(s) est (sont) I'(les) anomalie(s) de cet hémogramme ?


A - Anémie hypochrome
B - Neutropénie
C - Eosinophilie
D - Lymphocytose
E - Thrombopénie
Bonne(s) réponse(s) : B E

Sans commentaire.

769
Exclusivement sur DOC - DZ : www.doc-dz.com NADJI 85
RESIDANAT EN POCHE TOME II
Cas Clinique en QCM

Quelle(s) est (sont) I'(les)étiologie(s) qui peut (peuvent) expliquer l'ensemble de l'hémogramme ?
A - Inflammation
B - Anémie mégaloblastique
C - Anémie réfractaire
D - Myxoedème
E - Erythroblastopénie
Bonne(s) réponse(s) : B C

- L'anémie du myxoedème est normochrome normocytaire ou macrocytaire avec une réticulocytose normale ou abaissée et
les globules blancs et les plaquettes sont en nombre normal.
- L'érythroblastopénie ne touche par définition que la lignée rouge.
- L'inflammation entranerait essentiellement une thrombocytose et une hyperleucocytose.

Quels sont les médicaments qui peuvent donner un tel tableau et dont il faut rechercher la prise à l'interrogatoire
chez cette patiente ?
A - Méthotréxate®
B - Bactrim®
C - Tériam®
D - Rimifon®
E - Furosemide®
Bonne(s) réponse(s) : A B C

A, B et C sont des médicaments à action antifolique qui pourraient être responsables d'un tableau de pancytopénie avec
anémie mégaloblastique.

On ne retrouve pas de prise médicamenteuse particulière, le dosage de l'acide folique montre un taux normal et
le taux de vitamine B12 est très abaissé. La fibroscopie gastrique montre une muqueuse atrophique, atrophie
confirmée par la biopsie. Parmi les examens supplémentaires suivants, lequel ou lesquels vous paraît
(paraissent) nécessaire(s) en vue de confirmer le diagnostic de maladie de Biermer ?
A - Test de Schilling avec et sans facteur intrinsèque
B - Dosage de la méthylmalonylurie
C - Transit du grêle
D - Test au Xylose
E - Recherche d'une achlorhydrie gastrique
Bonne(s) réponse(s) : A E

C et D ne font pas partie des examens permettant de confirmer le diagnostic de Maladie de Biermer. Ils entrent dans les
examens à pratiquer devant une anémie mégaloblastique par carence en vitamine B12 à la recherche d'une cause autre que
le Biermer.

Quelle thérapeutique d'urgence vous parait justifiée à l'arrivée de cette patiente ?


A - Transfusion de 2 flacons de sang frais
B - Transfusion de 2 culots globulaires
C - Plasmaphérèse
D - Injection intraveineuse de 5.000 gammas de vitamine B12
E - Traitement digitalique
Bonne(s) réponse(s) : B

Sans commentaire.

Le diagnostic de Biermer retenu, la patiente traitée, on la revoit 3 mois plus tard avec un hémogramme normal.
Que faut-il surveiller régulièrement par la suite ?
A - Réticulocytes
B - Anticorps anti-facteur intrinsèque
C - Taux du facteur intrinsèque
D - Fibroscopie gastrique
E - Vitesse de conduction nerveuse
Bonne(s) réponse(s) : D

Sans commentaire.

770
Exclusivement sur DOC - DZ : www.doc-dz.com NADJI 85
RESIDANAT EN POCHE TOME II
Cas Clinique en QCM
Un homme de 40 ans est fébrile depuis deux semaines environ, avec des sueurs abondantes. Le foie est augmenté de
volume, la rate est palpable. On obtient la notion d'un abus habituel de boissons alcoolisées, on note l'érythrose des
pommettes avec des varicosités, des trémulations des extrémités. L'hypothèse d'une brucellose est envisagée.

Laquelle ou lesquelles des professions suivantes constitueraient un argument en faveur de la brucellose ?


A - Cultivateur
B - Ouvrier d'abattoir
C - Eleveur de poulets
D - Vétérinaire
E - Infirmier
Bonne(s) réponse(s) : A B D

En France, 80% de transmissions par ovins et caprins, 15% par bovins, quelques % par porcins, exceptionnellement par
cheval, chien, lièvre, chat, pigeon.

Parmi les premiers résultats d'examens complémentaires, lequel ou lesquels peuvent être considérés comme
étayant l'hypothèse de la brucellose ?
A - Hémogramme avec 7000 Leucocytes et 80 % de polynucléaires
B - Séro-diagnostic de Wright positif au 1/400è
C - Hémoculture positive à un germe gram négatif non encore identifié
D - V.S. = 30 à la 1ère heure
E - Transaminases TGP 160 UI, TGO 220 UI
Bonne(s) réponse(s) : B C

Brucella est un petit bacille gram négatif.


Le sérodiagnostic de WRIGHT met en évidence la présence d'anticorps de type Igm. II n'est positif entre le 10e et le 15e jour.
En pratique, la réaction d'agglutination au rose bengale permet un dépistage précoce et très rapide, avec une très bonne
sensibilité.

Parmi les signes suivants, lequel ou lesquels peuvent être retenus en faveur de l'hypothèse de la brucellose ?
A - Foyer pulmonaire lobaire
B - Orchite
C - Sueurs nocturnes
D - Erythème palmaire
E - Sciatalgie récente
Bonne(s) réponse(s) : B C E

Sans commentaire.

Parmi les antibiotiques suivants, lequel ou lesquels ont une action clinique reconnue dans le traitement de la
brucellose ?
A - Ampicilline (Totapen®)
B - Colistine (Colimycine®)
C - Céfalotine (Keflin®)
D - Minocycline (Mynocine®)
E - Rifampicine
Bonne(s) réponse(s) : D E

Brucella est un germe intracellulaire obligatoire. Les antibiotiques utilisés doivent avoir une très bonne pénétration
intracellulaire soit :
- cycline
- rifampicine
- fluroquinolons, type pefloxacine.

771
Exclusivement sur DOC - DZ : www.doc-dz.com NADJI 85
RESIDANAT EN POCHE TOME II
Cas Clinique en QCM
Monsieur H. 25 ans, n'a aucun antécédent pathologique si ce n'est la découverte à la visite d'incorporation d'un petit souffle
systolique au foyer aortique.
Il y a quinze jours, il a subi deux extractions dentaires. Six jours plus tard est apparue une fièvre à 39 degré C sans point
d'appel particulier ; le médecin traitant a prescrit une ampicilline (2 g/j per os) pendant 8 jours qui a abaissé la température à
38 degré C.
La nuit dernière, il a été réveillé par une douleur thoracique rétrosternale constrictive et angoissante, rapidement
accompagnée d'une polypnée intense.
A l'admission, on observe un tableau de suboedème pulmonaire ; le pouls est à 110/mn ; la TA est à 12/4 ; l'auscultation
cardiaque entend en bord gauche de sternum un souffle protosystolique 3/10 et un souffle holodiastolique 6/10 ; les bruits sont
normaux au foyer aortique; à la pointe, on entend un B3 suivi d'un roulement diastolique Le diagnostic d'endocardite
bactérienne est évoqué.

Compte-tenu du tableau clinique actuel, quelle est la nature la plus probable de l'atteinte valvulaire ?
A - Insuffisance aortique pure ou prédominante
B - Maladie aortique
C - Insuffisance aortique + rétrécissement mitral
D - Maladie aortique + rétrécissement mitral
E - Maladie mitrale
Bonne(s) réponse(s) : A

Sans commentaire.

Parmi ces éléments tirés de l'histoire clinique, la sévérité de la ou des lésion(s) valvulaire(s) est traduite par :
A - Tachycardie à 110/mn
B - TA à 12/4
C - Intensité du souffle systolique
D - Intensité du souffle diastolique
E - Roulement diastolique à la pointe
Bonne(s) réponse(s) : A B E

L'existence d'une insuffisance cardiaque est le principal signe de gravité.

Compte-tenu de la porte d'entrée et en attendant les résultats des hémocultures, quel est le germe le plus
probablement en cause ?
A - Staphylocoque doré
B - Staphylocoque epidermidis
C - Steptocoque alpha
D - Entérocoque
E - Gram (-)
Bonne(s) réponse(s) : C

50% des cas d'endocardites sur valves natives.


Le streptocoque alpha (ou viridans plutôt) est un germe commensal de la cavité buccale et la dissémination est souvent
secondaire à des soins dentaires.

En se fondant sur les mêmes arguments, lequel parmi ces 5 schémas d'antibiothérapie vous semble le plus
logique en première intention ?
A - Ampicilline
B - Vancomycine
C - Pénicilline G + Gentamycine
D - Oxacilline + Gentamycine
E - Céphalosporine + Gentamycine
Bonne(s) réponse(s) : C

Ces deux antibiotiques sont les plus actifs sur le streptocoque, bactéricides et synergiques.

Les hémocultures restent négatives Lequel ou lesquels parmi ces 5 examens complémentaires serai(en)t
susceptible(s) d'affirmer le diagnostic d'endocardite bactérienne ?
A - Dosage des antistreptolysines
B - Dosage du complément et de ses fractions
C - Prélèvement pharyngé
D - échocardiogramme
E - Cathétérisme cardiaque
Bonne(s) réponse(s) : D

Seul examen pouvant affirmer le diagnostic d'endocardite, l'échographie uni et bidimensionnelle visualise des végétations
supérieur ou égal à 3 mm dans 60% des cas.

772
Exclusivement sur DOC - DZ : www.doc-dz.com NADJI 85
RESIDANAT EN POCHE TOME II
Cas Clinique en QCM

Compte-tenu de la localisation des lésions valvulaires. à quelle(s) complication(s) évolutive(s) ce patient est-il
exposé ?
A - Insuffisance ventriculaire gauche aiguë
B - Bloc auriculo-ventriculaire
C - Embolie systémique
D - Communication inter ventriculaire
E - Anévrisme artériel
Bonne(s) réponse(s) : A B C D E

Les abcès septaux atteignant le faisceau de Hiss sont exceptionnels dans les endocardites à streptocoque viridans. C'est
plutôt le cas des endocardites staphylococciques. Les anévrismes sont plus fréquents.

Parmi ces 5 modalités évolutives, laquelle ou lesquelles inciteront à envisager une intervention chirurgicale
précoce en phase infectieuse ?
A - Insuffisance ventriculaire gauche évolutive et résistant au traitement médical
B - Apparition précoce de signes (radio. écho ) de dilatation ventriculaire gauche
C - Augmentation d'intensité des souffles
D - Persistance de la fièvre après 1 semaine de traitement antibiotique
E - Intervention systématique toute endocardite bactérienne doit être opérée précocement
Bonne(s) réponse(s) : A

La chirurgie est évitée au maximum en phase infectieuse. Elle peut être réalisée après 3 semaines de traitement si nécessaire.

L'enfant Z. 3 ans, fréquente la crèche depuis l'âge de 3 mois. Depuis 2 jours elle semble fatiguée. Lors d'un changement de
vêtement, une puéricultrice signale des adénopathies cervicales postérieures.
Malgré la mise en garde de la mère, celle-ci ramène son enfant le lendemain son visage est cramoisi et en la déshabillant la
puéricultrice constate une éruption maculeuse sur le tronc et les membres supérieurs. La température est à 38°. L'enfant a été
en contact avec une femme enceinte de huit semaines, deux jours avant l'éruption

La description clinique évoque :


A - Une rougeole
B - Une allergie médicamenteuse
C - Une varicelle
D - Une scarlatine
E - Aucune des propositions n'est acceptable
Bonne(s) réponse(s) : E

Description séméologique d'une rubéole.

Relevez la proposition fausse concernant les durées d'incubation des maladies suivantes :
A - Rubéole 14-18 jours
B - Rougeole 10 à 12 jours
C - Oreillons 18 à 21 jours
D - Scarlatine 10 à 15 jours
E - Varicelle 12 à 16 jours
Bonne(s) réponse(s) : D

Incubation de 3 à 5 jours.

Les adénopathies dans la rubéole présentent les caractéristiques suivantes sauf une :
A - Précédent l'éruption d'une semaine
B - Peuvent persister plusieurs semaines après l'éruption
C - Prédominent dans les aires : sous occipitale, cervicale, postérieure et épitrochléennes
D - Leur diamètre est toujours supérieur à 1 cm
E - Peuvent être douloureuses lors de l'éruption
Bonne(s) réponse(s) : D

Souvent microadénopathies.

773
Exclusivement sur DOC - DZ : www.doc-dz.com NADJI 85
RESIDANAT EN POCHE TOME II
Cas Clinique en QCM

Les formes cliniques de la rubéole sont variées ; relevez la plus fréquente :


A - Forme inapparente
B - Forme avec polyarthrite des extrémités
C - Forme avec purpura thrombopénique
D - Forme avec méningo-encéphalite
E - Forme avec fièvre supérieure à 39°
Bonne(s) réponse(s) : D

Comme la majorité des infections virales, une sérologie systématique est donc nécessaire chez toute femme enceinte pour
connaître son immunité face à la maladie.

Quelle attitude adopter vis à vis de cette femme enceinte dont l'immunité anti rubéoleuse n'est pas connue ?
A - Faire un premier prélèvement sérique et le renouveler 10 j après
B - Faire un premier prélèvement sérique et le renouveler 3 à 4 semaines après
C - Faire une injection intramusculaire de gamma globuline standard
D - Faire immédiatement un prélèvement sérique à la recherche d'IgM spécifiques
E - Envisager un avortement
Bonne(s) réponse(s) : C

L'immunisation passive est efficace jusqu'à 5 jours après le contage.


Le délai d'apparition des anticorps est d'environ 16 jours après le contage, mais très variable d'un sujet à l'autre.
Le titre maximal peut être présent 3 jours à 3 semaines après cette date.

Devant la rubéole de cette enfant vous proposez comme traitement parmi les propositions suivantes :
A - Injection intramusculaire de gamma globuline spécifique
B - L'abstention thérapeutique
C - Une antibiothérapie par macrolide à titre prophylactique des complications Infectieuses
D - Des anti-inflammatoires non stéroïdiens
E - Des anti-histaminiques
Bonne(s) réponse(s) : B

C'est une maladie bénigne.


Les complications à type d'encéphalite ou purpura thrombopénique sont rares.

Mr J Y. 25 ans, est employé aux abattoirs depuis deux mois. Il vient vous consulter pour un état fébrile traînant accompagné
de sueurs et de douleurs musculaires et articulaires diffuses et d'une douleur de la fesse Le début, imprécis, remonterait à une
dizaine de jours. La température mesurée à votre cabinet est à 38°5, le pouls en rapport. Mr J Y n'a pas maigri, est
modérément asthénique. Vous palpez une splénomégalie, une hépatomégalie, des micro adénopathies cervicales. La NFS
montre 4 000 globules blancs par mm3 ; 40 % de ploynucléaires neutrophiles. Vous soupçonnez à juste titre une brucellose
aiguë qui est confirmée par la séro-agglutination de Wright positive au 1/1280°, et ultérieurement par la positivité de
l'hémoculture.

Parmi les traitements suivants lequel choisissez vous :


A - Vaccinothérapie antimélitococcique
B - Ampicilline + streptomycine
C - Vibramycine + rifampicine
D - Rifampicine + cotrimoxazole
E - Colimycine + Gentamicine
Bonne(s) réponse(s) : C

Vibramycine 100 mg x 2/j.


Rifampicine 450 mg x 2/j.

Quelle durée de traitement envisagez-vous ?


A - 1 à 2 semaines
B - 4 à 8 semaines
C - 10 semaines
D - 12 semaines
E - 18 semaines
Bonne(s) réponse(s) : B

En pratique, bithérapie pendant 4 semaines et 2 semaines de cycline seule.

774
Exclusivement sur DOC - DZ : www.doc-dz.com NADJI 85
RESIDANAT EN POCHE TOME II
Cas Clinique en QCM

Outre l'efficacité bactériologique in vitro quelle est la motivation principale de votre choix :
A - Elimination hépato-biliaire
B - Pénétration intra-cellulaire
C - Concentration lymphatique
D - Concentration sérique élevée
E - Aucun autre critère d'efficacité in vitro
Bonne(s) réponse(s) : B

Brucella est une bactérie intracellulaire obligatoire comme les rickettsies.

Sur quel argument arrêtez-vous votre traitement ?


A - Apyrexie stricte depuis 1 semaine
B - Négativation du séro-diagnostic de Wright
C - Non positivation de l'IDR à la Mélitine
D - Négativation de l'hémoculture
E - Aucune de ces propositions
Bonne(s) réponse(s) : E

Il y a rarement stérilisation complète des foyers et les rechutes sont fréquentes. Il n'existe aucun argument objectif formel de la
guérison d'une brucella.

Retenez la ou les proposition(s) exacte(s) en matière de prophylaxie de la brucellose :


A - Abattage des animaux atteints
B - Education sanitaire et surveillance des professions exposées
C - Vaccination des professionnels exposés
D - Antibioprévention par les tétracyclines
E - Vaccination des génisses de certains troupeaux
Bonne(s) réponse(s) : A B C E

Les mesures sanitaires prises par rapport au bétail contaminé sont primordiales.
La vaccination humaine est efficace pendant 2 ans après 1 injection de fraction antigénique.

Un homme de 28 ans, employé de bureau dans la région parisienne, est admis à l'hôpital pour un état fébrile accompagné de
toux. Ces symptômes ont débuté 10 jours auparavant et la toux est devenue de plus en plus fréquente et gênante. Il s'agit
d'une toux sèche non productive. La température à l'entrée est à 38°2. L'état général est bien conservé. L'interrogatoire
retrouve la notion d'un éthylisme modéré. L'examen clinique met en évidence quelques râles sous-crépitants et râles
bronchiques aux deux bases pulmonaires. La radiographie pulmonaire de face et de profil n'objective aucun foyer
parenchymateux systématisé. On note seulement une opacité très floue et mal limitée dans le lobe inférieur gauche et une
"scissurit" à droite. Le reste de l'examen clinique est négatif. L'hémogramme montre une leucocytose à 8 000/mm3 avec 50 %
de polynucléaires neutrophiles. Les hémocultures resteront stériles. La recherche d'agglutinines froides est positive.

Le tableau clinique décrit évoque principalement :


A - Une pneumonie franche lobaire aiguë
B - Une pneumonie interstitielle
C - Une bronchopneumonie
D - Un cancer bronchique
E - Une bronchite chronique
Bonne(s) réponse(s) : B

La toux sèche et non productive fait éliminer le diagnostic de bronchopneumonie.

L'agent étiologique le plus probable est :


A - Bordetella pertussis
B - Pneumocoque
C - Virus grippal
D - Mycoplasma pneumoniae
E - Klebsiella pneumoniae
Bonne(s) réponse(s) : B

Le plus souvent responsable d'un tableau de pneumopathie atypique. Une élévation significative des agglutinations froides est
retrouvée dans 50% des cas.

775
Exclusivement sur DOC - DZ : www.doc-dz.com NADJI 85
RESIDANAT EN POCHE TOME II
Cas Clinique en QCM

L'examen biologique le plus utile pour établir le diagnostic étiologique est :


A - Hémoculture
B - Sérologie
C - Lavage bronchiolo alvéolaire
D - Examen bactériologique de l'expectoration
E - Ponction transtrachéale
Bonne(s) réponse(s) : B

Mycoplasma pneumoniae peut être retrouvé à l'examen bactériologique du LBA ou de l'expectoration. Sa culture est lente sur
milieux spéciaux.

Le traitement le plus approprié est :


A - Pénicilline G
B - Tétracycline
C - Gamma-globulines standard
D - Amantadine
E - Colbatothérapie
Bonne(s) réponse(s) : B

La maladie évolue favorablement même sans traitement. Le traitement antibiotique à visée intracellulaire de préférence
(germe intracellulaire facultatif) hâte la guérison. Cyclines ou macrolides sont très actifs.

Un adolescent de 17 ans, quelques jours après avoir développé une parotidite ourlienne, se plaint de douleurs scrotales
violentes. A l'examen, les bourses sont tuméfiées et rouges et le testicule droit est augmenté de volume et très sensible.
Après quelques jours, la température réapparaît en même temps que des céphalées violentes et des vomissements. Les
données d'examens sont en faveur d'une réaction méningée, ce que confirme la ponction lombaire.

Parmi les propositions suivantes, laquelle(lesquelles) caractérise(nt) l'orchite ourlienne ?


A - S'observe rarement avant la puberté
B - Il s'agit en fait d'une orchi-épididymite
C - Peut s'accompagner d'urétrite
D - L'atrophie testiculaire est une séquelle possible
E - La fonction endocrine du testicule est parfois atteinte
Bonne(s) réponse(s) : A

Localisation la plus fréquente après les glandes salivaires.


La stérilité et l'atrophie testiculaire sont absolument exceptionnelles.

Parmi les atteintes glandulaires suivantes, deux ne sont jamais observées au cours ou au décours des oreillons.
Lesquelles ?
A - Pancréatite
B - Ovarite
C - Prostatite
D - Thyroïdite
E - Hypophysite
Bonne(s) réponse(s) : C E

L'ovarite et la mammite sont plus rares.

Outre la méningite, les oreillons peuvent être responsables d'autres complications. Laquelle ou lesquelles ?
A - Encéphalite
B - Rhumatisme polyarticulaire
C - Sciatique
D - Polynévrites
E - Surdité de perception
Bonne(s) réponse(s) : A B E

Localisation très rares ainsi qu'oculaires, endocardiques, hépatiques ou rénales.

776
Exclusivement sur DOC - DZ : www.doc-dz.com NADJI 85
RESIDANAT EN POCHE TOME II
Cas Clinique en QCM

Dans la liste suivante, quel(s) examen(s) peut(vent) orienter en quelques heures vers le diagnostic d'oreillons
au début ?
A - Dosage des amylases sanguines
B - Sérologie par fixation du complément
C - Isolement du virus dans la salive
D - Hémogramme
E - Aucune de ces propositions
Bonne(s) réponse(s) : A

Exceptionnellement pratiqué;
Le diagnostic est généralement clinique;

Le traitement de l'orchite ourlienne doit comporter les propositions suivantes, sauf deux. Lesquelles ?
A - Anti-inflammatoires non stéroïdiens
B - Repos strict au lit
C - Chimiothérapie antivirale par acyclovir
D - Androgènes
E - Port d'un suspensoir
Bonne(s) réponse(s) : C D

L'acyclovir est inactif sur les virus à ARN.

Quelle(s) proposition(s) s'applique(nt) à la vaccination anti-ourlienne ?


A - Virus vivant atténué
B - Nécessite 3 injections à 1 mois d'intervalle
C - Nécessite un rappel tous les 3 ans
D - Est indiquée chez les hommes non immunisés
E - Est déconseillée chez l'enfant de moins de 1 ans
Bonne(s) réponse(s) : A D E

Il existe 3 types de vaccins : 2 vivants atténués et 1 vaccin inactivé. Très employé aux Etats-Unis, la vaccination reste discutée
en France. Elle peut être administrée chez l'enfant en association avec la rougeole et la rubéole (R.O.R) après 1 an.

Vous êtes appelé le 13 Mars 19.. dans la matinée par la mère d'un enfant de 4 ans. Il présente un coryza important oculaire
(yeux rouges, gonflés), nasal (rhinite aqueuse) et trachéo-bronchique (toux sèche et douloureuse). La température est à 39°,
l'enfant est abattu. A l'examen : pharyngite, microadénopathies cervicales. A l'interrogatoire contact avec un camarade
rougeoleux en classe le 1er Mars précédent. Vous suspectez justement une rougeole.

Où recherchez-vous le signe de Kِplick qui affirme la rougeole ?


A - Sur les piliers des amygdales
B - Sur le voile du palais
C - Sur la paroi antérieure du pharynx
D - A la face inférieure de la langue
E - A la face interne des joues
Bonne(s) réponse(s) :E

C'est un énanthème buccal.


Classiquement tâches ardoisées en regard des prémolaires.

Ou recherchez-vous l'éruption débutante ?


A - A la paume des mains
B - Aux plis de flexion
C - A la racine du cou
D - Derrière les oreilles
E - Aux joues
Bonne(s) réponse(s) : D

Sans commentaire.

777
Exclusivement sur DOC - DZ : www.doc-dz.com NADJI 85
RESIDANAT EN POCHE TOME II
Cas Clinique en QCM

Au 5ème jour après la sortie de l'éruption l'enfant demeure fébrile. Vous soupçonnez une complication de
surinfection : la ou lesquelles recherchez-vous en priorité ?
A - Angine streptococcique
B - Otite aiguë
C - Méningite lymphocytaire
D - Bronchopneumonie
E - Adénophlegmon cervical
Bonne(s) réponse(s) : B D

D - Dans les pays en voie de développement les complications de la rougeole sont une des principales causes de mortalité
des enfants de 1 à 4 ans.
La mortalité est 100 fois plus élevée que dans les pays développés = 10% des cas.
Les kératites de surinfections sont redoutables également.
C - L'encéphalite postéruptive apparait 1 à 15 jours après l'exanthème.

Quel(s) germe(s) est(sont) habituellement en cause au cours de ces complications de surinfection ?


A - Escherichia Coli
B - Listeria monocytogènes
C - Haemophilus influenzae
D - Streptococcus pneumoniae
E - Klebsiella pneumoniae
Bonne(s) réponse(s) : C D

Germes responsables de la majorité des infections des voies aériennes chez le nourrisson et l'enfant.

Vous décidez d'une antibiothérapie. Parmi les antibiotiques suivants, le ou lesquels vous paraissent les mieux
adaptés à cette surinfection ?
A - Colistine (Colimycine®)
B - Ampicilline (Totapen®)
C - Rifampicine (Rimactan®)
D - Cefalexine (Keforal®)
E - Gentamicine (Gentalline®)
Bonne(s) réponse(s) : B

Réponse discutée car actuellement il existe 10 à 20% des souches d'hémophiles produisant une bêta lactamase donc
ampicilline - R -. On pourrait proposer l'association Ampicilline + acide clavulanique (Augmentin®) ou alfatil (Cefaclor®).

Si vous aviez vu l'enfant le 4 Mars quelle mesure prophylactique prise à ce moment aurait permis d'éviter la
survenue de cette rougeole ?
A - Isolement
B - Vaccination
C - Gamma-globuline standard
D - Vaccination + gamma-globuline standard
E - Prescription d'acyclovir (Zovirax®)
Bonne(s) réponse(s) : C

L'injection de 30 à 50 mg/kg de gammaglobulines standards réalise une séroprévention si elle est faite avant le 6e jour. Le 6e
ou 7e jour, elle n'induira qu'une séroatténuation.

Mademoiselle X.., 17 ans, est hospitalisée pour une angine à fausses membranes. On ne lui connait pas d'antécédent
particulier. Elle a bénéficié des vaccinations obligatoires au cours de son enfance.
A l'examen, la fièvre est à 38° ; le pouls à 110/mn. On note une pâleur. Il existe quelques adénopathies locorégionales.
L'ensemble du pharynx est érythémateux, il existe des fausses membranes adhérentes et cohérentes au niveau des
amygdales. On suspecte une diphtérie.

Retenez le ou les principaux germes responsables d'angines à fausses membranes en dehors de la diphtérie :
A - Mycoplasma pneumoniae
B - Virus d'Epstein Barr
C - Pasteurella multocida
D - Virus Coxsakie A
E - Treponema pallidum
Bonne(s) réponse(s) : B

Principal diagnostic différentiel de la diphtérie, plus fréquent. La fausse membrane correspond à un exsudat inflammatoire
important.

778
Exclusivement sur DOC - DZ : www.doc-dz.com NADJI 85
RESIDANAT EN POCHE TOME II
Cas Clinique en QCM

Retenez le ou les examen(s) d'orientation diagnostique obtenu(s) en urgence qui vous permettront d'adopter
une conduite à tenir :
A - Contre Immuno Electrophorèse vis à vis de l'antigène diphtérique
B - MNI test
C - Hémoculture
D - Numération formule sanguine
E - Prélèvement de gorge
Bonne(s) réponse(s) : B E

Nécessaires et suffisants, le MNI test ou recherche d'anticorps hétérophiles par agglutination est rapide,facile. Il y a quelques
faux positifs;
Pour le prélèvement de gorge, préciser la recherche de corynebactériae diphtériae pour ensemencement sur milieux
spéciaux, et IF directe sur lame.

Retenez le ou les éléments cliniques à retenir en faveur d'une angine diphtérique commune :
A - Existence d'une splénomégalie constante
B - Existence d'une polyadénopathie touchant les différentes aires ganglionnaires
C - Existence d'une pâleur et d'une tachycardie
D - Coryza avec jetage unilatéral et érosion narinaires
E - Tâches purpuriques vélo-palatines
Bonne(s) réponse(s) : C D

A.B.E - Sont en faveur de la MNI (A - 50% des cas).


La diphtérie correspond à une multiplication locale pharynco-laryngée et à une dissémination toxinique secondaire.

Retenez la ou les mesures que vous appliquerez à Melle X... et à son entourage :
A - Antibiothérapie de Mlle X... par métronidazole (Flagyl®)
B - Sérothérapie spécifique pour Mlle X...
C - Sérothérapie spécifique pour l'entourage
D - Vaccination de Mlle X...
E - Vaccination (ou rappel le cas échéant) de l'entourage
Bonne(s) réponse(s) : B E

La sérothérapie par antitoxine n'est efficace qu'administrée précocement avant les premiers signes de diffusion toxinique : 40
à 100 000 UI dans les cas graves.
La vaccination est obligatoire et les rappels doivent être faits tous les 5 ans.

Indiquez la ou les proposition(s) exacte(s) concernant la vaccination antidiphtérique, en France :


A - N'est pas obligatoire mais recommandée
B - Est le plus souvent associée
C - Est une anatoxine
D - Ne présente aucune contre-indication
E - Représente une protection absolue contre la survenue d'une diphtérie
Bonne(s) réponse(s) : B C

Elle n'immunise pas complètement mais diminue les formes graves.


Elle permet une diminution de la prévalence de l'infection.

Cet enfant de 5 ans, émigré, n'ayant reçu aucune vaccination, présente une toux quinteuse, diurne et nocturne, depuis une
huitaine de jours. Les quintes déclenchent parfois des vomissements. Il existe des douleurs abdominales et la température est
normale. Comme l'examen montre quelques râles au niveau des bases, le médecin a prescrit un sirop antitussif et des
injections journalières de Pénicilline + Colimycine, sans résultat au bout de 4 jours. Les parents vous montrent alors l'enfant

Contre le diagnostic de coqueluche. vous retenez :


A - Toux quinteuse
B - Toux émétisante
C - Douleurs abdominales
D - Caractère normal de la température
E - Inefficacité du traitement antibiotique
Bonne(s) réponse(s) : C

Sans commentaire.

779
Exclusivement sur DOC - DZ : www.doc-dz.com NADJI 85
RESIDANAT EN POCHE TOME II
Cas Clinique en QCM

Parmi les agents infectieux suivants, quel est celui responsable de la coqueluche ?
A - Streptocoque pyogène du groupe A
B - Listeria monocytogènes
C - Bordetella pertussis
D - Salmonella enteritidis
E - Adénovirus de type 7
Bonne(s) réponse(s) : C

Bacille gram négatif capsulé, pathogène strict de l'homme, retrouvé dans les voies respiratoires supérieures.

Si le diagnostic de coqueluche vous paraît possible, quel(s) examen(s) demandez-vous pour le confirmer ?
A - Hémogramme
B - Prélèvement de gorge
C - Sérologie
D - Prélèvement naso-pharyngé, pour immunofluorescence
E - Intradermo-réaction
Bonne(s) réponse(s) : A D

A - Leucocytose pouvant aller jusqu'à 50 g/l.


D - Après étalement du prélèvement sur lame, des anticorps marqués à la fluorescéine révèle la présence de bordetella
pertussis. Technique rapide mais de nombreux faux +.
Le prélèvement est mis en culture également sur milieu de Bordet-Gengou.

Dans l'hypothèse où il s'agit bien d'une coqueluche, quelle(s) complication(s) pouvez-vous redouter ?
A - Pneumonie
B - Otite
C - Atélectasie
D - Hépatite
E - Pancréatite
Bonne(s) réponse(s) : A B C

La pneumonie peut être une surinfection bactérienne ou une pneumocoqueluche alvéolaire moins grave.

Le traitement qui a été préalablement prescrit s'avérant inefficace, que proposez-vous ?


A - Antibiothérapie par Chloramphénicol
B - Gamma-globulines anti-coquelucheuses
C - Antibiothérapie par Streptomycine
D - Antibiothérapie par Ampicilline
E - Aucune des thérapeutiques précédentes
Bonne(s) réponse(s) : D

Le but du traitement antibiotique est la prévention des surinfections respiratoires fréquentes : les germes habituellement
responsables sont les streptocoques et haemophilus.

Il existe dans l'entourage un jeune frère de 3 mois, non vacciné lui aussi. Quelle(s) mesure(s) envisagez-vous à
son égard dans l'immédiat ?
A - Antibiothérapie prophylactique
B - Gammaglobulines anti-coquelucheuses
C - Isolement
D - Vaccination
E - Aucune des mesures précédentes
Bonne(s) réponse(s) : B C D

B - Gamma - globulines humaines spécifiques : 200 mg en IM. Son efficacité est discutée
C - L'éviction scolaire est de 20 jours après le début des quintes pour les frères et soeurs
D - La vaccination le plus fréquemment associée à diphtérie + tétanos + polio, recommandée dès l'âge de 3 mois, 3 injections
+ rappels 1 an et 5 ans après.

780
Exclusivement sur DOC - DZ : www.doc-dz.com NADJI 85
RESIDANAT EN POCHE TOME II
Cas Clinique en QCM
Mr X, âgé de 40 ans, est de retour depuis 3 semaines d'un séjour touristique de deux mois au Sénégal, et il vous consulte en
septembre pour une diarrhée 4 selles par jour, selles pâteuses non sanglantes.
L'interrogatoire vous apprend :
1°) qu'il avait pris conseil auprès de son médecin-traitant et avait donc été vacciné contre la fièvre jaune. Avait subi un rappel
anti-poliomyélitique, antitétanique et antityphoparatyphique, et pris durant tout son séjour un comprimé de Nivaquine® par jour
et l'a arrêté dès son retour.
2°) qu'il présente également :
quelques signes généraux : courbatures, douleurs lombaires avec température entre 38° et 39° depuis 4 jours quelques
signes digestifs : anorexie, épigastralgies, 1 ou 2 vomissements, langue pâteuse.
A l'examen clinique :
- hépatomégalie à 2 travers de doigts.
- rate non perçue urines claires.
L'auscultation cardio-pulmonaire est normale.
L'examen neurologique est normal.
L'hémogramme pratiqué est normal.
La radiographie pulmonaire est normale.

Quel(s) diagnostic(s) devez-vous évoquer devant une diarrhée au retour d'un pays tropical parmi ceux proposés
?
A - Salmonellose
B - Shigellose
C - Hépatite virale
D - Verminose intestinale
E - Bilharziose
Bonne(s) réponse(s) : A B D E

Sans commentaire.

Quel est le diagnostic le plus probable dans le cas précis de Mr X ?


A - Fièvre typhoïde
B - Paludisme
C - Hépatite virale
D - Amibiase
E - Ascaridiose
Bonne(s) réponse(s) : B

L'arrêt de la prophylaxie antipaludéenne au retour de France permet d'évoquer le diagnostic.


On aurait pu discuter une amibiase hépatique mais elle n'est pas contemporaine d'amibiase digestive et donc de diarrhée.

Quel est l'examen à demander pour préciser ce diagnostic ?


A - Coproculture
B - Frottis sanguin et goutte épaisse
C - Bilan hépatique
D - Sérologies parasitaires
E - Examen parasitologique des selles
Bonne(s) réponse(s) : B

Réalisés aux pics fébriles, ils recherchent la présence de plasmodium au sein des hématies : la lecture du frottis après
coloration au M.G.G est immédiate, rapide et décèle une parasitémie de l'ordre de 250 parasites/microl.
La goutte épaisse contient 12 à 244 parasites/microl. C'est une technique d'enrichissement : elle détecte une parasitémie plus
faible de 50/microl.

Dans cette région Mr X . encourait également d'être exposé à :


A - Myase furonculeuse (ver de Cayor)
B - Ankylostomose
C - Amibiase
D - Bilharziose
E - Filariose à filaire de Médine
Bonne(s) réponse(s) : A B C D

Le vede CAYOR est répandu en Afrique. Il vit sur le sol des cases ou dans le linge. Il pénètre activement la peau pour former
une tuméfaction furonculoïde.

781
Exclusivement sur DOC - DZ : www.doc-dz.com NADJI 85
RESIDANAT EN POCHE TOME II
Cas Clinique en QCM

Pour prévenir cette (ces) dernière(s) affection(s) qu'auriez-vous pu proposer comme précautions ?
A - Eviter les bains en eau douce
B - Port de chaussures
C - Eviter de boire l'eau des puits non filtrée
D - Eviter les crudités
E - Repasser le linge de corps avec un fer très chaud
Bonne(s) réponse(s) : A B C D E

A - Evite les bilharsioses.


B - Prévention de l'ankylostomose.
C - Prévention de l'amibiase, la filariose de Medine.
D - Prévention de l'amibiase.
E - Prévention des myases.

Un enfant âgé de 7 ans présente subitement après 24 heures de fièvre isolée à 38 degrés C des lésions cutanées de type
vésiculaire non indurée à base souple et à contour clair au niveau thoracique.

Devant ce tableau évocateur d'une varicelle retenez la ou les mesure(s) à prendre éventuellement :
A - Traitement anti prurigineux
B - Eviction scolaire pendant 40 jours
C - Traitement antibiotique par un macrolide
D - Application d'une solution antiseptique sur les lésions les plus inflammatoires
E - Immunoglobulines à l'entourage
Bonne(s) réponse(s) : A D

La principale complication est une surinfection des lésions par grattage.


La gamma-thérapie préventive est indiquée pour les femmes enceintes, les immunodéprimées et les nouveaux-nés.

Parmi les sujets suivants, retenez celui ou ceux qui vous paraisse(nt) les plus exposés à la survenue d'une
varicelle grave :
A - Enfant de moins de 5 ans
B - Enfant de race noire
C - Rhumatisant traité par antiinflammatoire non stéroïdien
D - Sujet immunodéprimé
E - Femme enceinte
Bonne(s) réponse(s) : D E

D - Forme maligne de l'immunodéprimé : varicelle bulleuse ou hémorragique avec dissémination à tous les organes et
mortalité de 8 à 15% des cas.
E - Jusqu'au 6e mois de grossesse, risque d'embryopathie foetopathie ; forme grave du nouveau- né si varicelle perpartum.

Retenez la ou les proposition(s) exacte(s) concernant une femme enceinte, en cas de contage varicelleux :
A - Injecter des Immunoglobulines spécifiques VZ à la dose de 1 ml/kg IV
B - Injecter 0,3 mI/kg IM d'immunoglobulines spécifiques VZ si elle n'est pas immunisée vis à vis de la varicelle
C - Vaccin anti varicelleux vivant
D - Hospitaliser cette patiente
E - Mettre en route un traitement antibiotique par le Clamoxyl® 4 g/j
Bonne(s) réponse(s) : B

90% des femmes sont immunisées à l'âge de procréer


Les immunoglobulines sont des IgG de convalescent de zona (ZIG) et doivent être injectées moins de 72 heures après le
contage. Protection pendant 4 semaines.

Les complications possibles de la varicelles sont la ou les suivante(s) :


A - Cérébellite aiguë
B - Pneumonie avec hypoxie sévère
C - Syndrome hémorragique
D - Embryopathie
E - Algies post varicelleuse
Bonne(s) réponse(s) : A B C D

A - Complication la plus fréquente en dehors de la forme de l'immunodéprimé.


C - Purpura trombopénique bénin le plus souvent.

782
Exclusivement sur DOC - DZ : www.doc-dz.com NADJI 85
RESIDANAT EN POCHE TOME II
Cas Clinique en QCM

Devant toute éruption vésiculeuse retenez le ou les diagnostics que l'on peut évoquer en dehors de la varicelle :
A - Une atteinte cutanée par le virus du SIDA (LAV)
B - Un zona
C - Une brûlure
D - Un syndrome de Lyell
E - Un herpès
Bonne(s) réponse(s) : B E

B - Le zona correspond à une résurgence du virus varicelle - zona (VZV)


C D - Réalisent des bulles.

Un homme de 60 ans, jardinier de son état, présente brutalement des frissons, un malaise général et une douleur de la jambe
gauche.
L'interrogatoire ne retrouve pas d'épisode antérieur mais met en évidence une intoxication alcoolo tabagique. Il s'est blessé le
pied en marchant sur un clou il y a 5 jours.
A l'examen, il existe une fièvre à 39°C, une déshydratation et une légère obnubilation. Localement, on retrouve un érythème
intense et une augmentation de la chaleur locale depuis le dos du pied jusqu'au tiers de la jambe.
La peau est tendue, luisante et présente par endroit un décollement bulleux limité. On retrouve également une adénopathie
inguinale douloureuse, des râles bronchiques diffus, un discret souffle protosystolique de pointe et un intertrigo bilatéral entre
les 4ème et 5ème orteils. Ainsi qu'un mauvais état dentaire.
La numération formule montre une polynucléose neutrophile et une vitesse de sédimentation à 60 à la première heure.

Parmi les diagnostics suivants, vous pouvez évoquer :


A - Erysipèle
B - Lymphangite
C - Pemphigus
D - Impétigo
E - Pemphigoïde bulleuse
Bonne(s) réponse(s) : A B

Il faut toujours penser à la responsabilité du streptocoque devant ce tableau clinique, généralement du groupe A.

Quel antibiotique choisissez-vous pour le traitement ?


A - Pénicilline G
B - Ampicilline
C - Amoxilline
D - Cefotaxime
E - Aucun des produits précédents
Bonne(s) réponse(s) : A

On pourrait discuter l'emploi d'une pénicilline M ou d'une synergistine en raison de l'association fréquente, peut être
secondaire, du staphylocoque.

En cas d'allergie aux bêtalactamines quel est l'antibiotique de choix ?


A - Streptomycine
B - Erythromycine
C - Gentamycine
D - Tobramycine
E - Cotrimoxazol
Bonne(s) réponse(s) : B

Les macrolides ont la meilleure activité sur le streptocoque après les Blactamines.

Parmi les traitements adjuvants suivants, il n'est pas indiqué ou/et il est contrindiqué de prescrire :
A - Héparine
B - Méprobamate
C - Corticoïdes locaux
D - Sérothérapie antittanique
E - Amphotéricine B
Bonne(s) réponse(s) : C E

C - Evident lors d'une infection cutanée.


E - Inutile, antifungique.
A - L'héparinate de calcium a également un effet antiinflammatoire.

783
Exclusivement sur DOC - DZ : www.doc-dz.com NADJI 85
RESIDANAT EN POCHE TOME II
Cas Clinique en QCM

La ou les portes d'entrées possibles sont :


A - Foyer infectieux dentaire
B - Infection bronchique chronique
C - Pied d'athlète
D - Embol septique
E - Plaie d'origine professionnelle
Bonne(s) réponse(s) : C E

Effraction de la barrière cutanée, qu'elle que soit sa taille, sa profondeur, ou son origine.

La prévention des récidives comporte :


A - Arrêt du tabac
B - Vasodilatateurs
C - Antibiothérapie continue
D - Traitement de l'intertrigo
E - Vaccination antitétanique
Bonne(s) réponse(s) : D

C - L'antibiothérapie continue préventive n'a pas d'indication.


Les récidives sont fréquentes (40 à 50%). Le traitement de la première poussée par la pénicilline G intraveineuse, 10 millions
UI, diminuerait les récidives à 20% des cas.

Parmi les traitements suivants, lequel ou lesquels est (sont) inutiles dans le traitement du pied d'athlète qui est
ici trichophytique ?
A - Amphotéricine B
B - Ketoconazole
C - Grisefuline
D - Mycostatine
E - Clotrimazole
Bonne(s) réponse(s) : A D

B E- Sont anti candidosiques et anti-trychophytiques.


C - Est seulement anti-trichophytique.

Au 2ème jour des règles, une femme de 29 ans présente brutalement une syncope, des frissons, une diarrhée aqueuse, des
vomissements et des myalgies : elle est hospitalisée..
A l'examen gynécologique, l'utérus est de taille normale et les annexes libres.
A I examen au spéculum on constate la présence de pus dans le vagin, la patiente utilise des tampons vaginaux qu'elle
change régulièrement.
Les autres manifestations cliniques du syndrome de choc toxique Staphylococcique (toxic shock syndrome) sont observées
au cours de l'hospitalisation.
La N.F.S. montre une polynucléose à neutrophiles.
La réponse au traitement antibiotique antistaphylococcique est rapidement favorable (apyrexie au 3ème jour).

Parmi les espèces suivantes, laquelle vous paraît la plus vraisemblablement imputable dans l'étiologie du
syndrome ?
A - Staphylococcus Epidermidis
B - Staphylococcus Saprophyticus
C - Staphylococcus Aureus
D - Staphylococcus Intermedius
E - Staphylococcus Hominis
Bonne(s) réponse(s) : C

Les staphylocoques non aureus, DNase et coagulase ne peuvent être pathogènes que lors d'infections nocosomiales. Seuls
les staphylocoques saprophyticus seraient responsables d'infections urinaires de la femme jeune.

Parmi les manifestations cliniques ci après, laquelle ou lesquelles caractérisent le T.S.S. ?


A - Syndrome méningé
B - hypotension artérielle
C - Rash scarlatiniforme
D - Pyrexie (T° 39°C)
E - Arrêt des matières et des gaz
Bonne(s) réponse(s) : B C D

Il y a souvent céphalées et vomissements.

784
Exclusivement sur DOC - DZ : www.doc-dz.com NADJI 85
RESIDANAT EN POCHE TOME II
Cas Clinique en QCM

Le T.S.S. est dû à :
A - La dissémination métastatique du staphylocoque à partir du foyer initial
B - La diffusion d une toxine staphylococcique à partir du foyer initial
C - Un état d'allergie du malade vis à vis de staphylocoque
D - La décompensation d'une tare préexistante lors d'une infection banale staphylococcique
E - Une déficience du système immunitaire du malade
Bonne(s) réponse(s) : B

Exotoxine découverte en 1981 appelée TSST. Sa production serait transmise par un bactériophage et varie selon les souches.

Sachant que le staphylocoque isolé possède une résistance hétérogène, choisissez la proposition qui vous
semble la plus judicieuse. :
A - Pénicilline G - gentamicine
B - Oxacilline - gentamicine
C - Céfalotine - gentamicine
D - Méthicilline - gentamicine
E - Aucune proposition n'est valable
Bonne(s) réponse(s) : E

B - Actuellement environ 40% des souches sont résistantes à la méticilline. Le traitement de 1ère intention comprend toujours
la vancomycine, antibiotique actif sur 100% des staphylocoques dorés. Dans ces cas-là, il sera associé parfois à un autre
antistaphylococcique majeur : fosfomycine, rifampicine ou pefloxacine.

Un de vos malades vient de rentrer d'un voyage de 6 mois dans le nord de l'Inde, durant lequel il n'avait pris aucune
chimioprophylaxie anti-parasitaire. Il présente brutalement une fièvre à 40°C, des troubles gastriques (nausées,
vomissements), des troubles de la conscience avec légère confusion. L'examen clinique montre une hépatomégalie discrète.
Ce sujet est pâle, vraisemblablement anémique.

Vous évoquez en priorité un accès pernicieux de paludisme. Quel examen demandez-vous en urgence pour
confirmer votre diagnostic ?
A - Ponction sternale
B - Examen parasitologique de selles
C - Ponction lombaire
D - Frottis sanguin à la recherche d'hématozoaires
E - Sérodiagnostic de paludisme
Bonne(s) réponse(s) : D

Frottis goutte-épaisse.

Votre malade n'ayant jamais quitté la France avant ce voyage en Inde, quel(s) diagnostic(s) pouviez-vous
éliminer d'emblée sur des critères géographiques ?
A - Maladie du sommeil (Trypanosomiase)
B - Abcès amibien du foie
C - Leishmaniose viscérale débutante
D - Bilharziose à Schistosoma mansoni en phase "toxémique"
E - Paludisme à Plasmodium ovale
Bonne(s) réponse(s) : A C D

Sans commentaire.

Le laboratoire vous ayant répondu : "Présence de nombreux trophozoïtes de Plasmodium falciparum", quel est
le traitement que vous devez prescrire ?
A - Métronidazole (Flagyl®) en perfusion intra-veineuse
B - Chloroquine (Nivaquine®) per os
C - Glucantine en intramusculaire
D - Déhydroémétine en sous cutané
E - Quinine en perfusion intra-veineuse
Bonne(s) réponse(s) : E

Du fait du tableau clinique.

785
Exclusivement sur DOC - DZ : www.doc-dz.com NADJI 85
RESIDANAT EN POCHE TOME II
Cas Clinique en QCM

Parmi les régions suivantes, il n'a jamais été rapporté de cas de résistance du Plasmodium falciparum à la
nivaquine en provenance de :
A - Antilles Françaises
B - Afrique de l'Est (Tanzanie)
C - La Réunion
D - Cambodge
E - Guyane Française
Bonne(s) réponse(s) : A C E

Sans commentaire.

Au cours d'un accès pernicieux à Plasmodium falciparum, vous devez redouter la survenue de :
A - Convulsions
B - Ictère hémolytique
C - Insuffisance rénale aiguë
D - Résistance à la quinine pour les patients revenant de Thaïlande
E - Coma carus et une issue fatale en 24 heures
Bonne(s) réponse(s) : A B C E

Piège ; résistance alachi-mioprophylaxie par nivaquine.

Une femme de 25 ans, enceinte de 3 mois, vous consulte pour une suspicion d'herpès génital. Elle a depuis 2 jours trois
lésions vésiculeuses, actuellement ulcérées, de 2 mm de diamètre chacune, sur la face externe de la petite lèvre droite. C'est
la troisième fois depuis 1 an qu'elle souffre de pareilles lésions à cet endroit.

Vous pourrez confirmer avec quasi certitude le diagnostic d'herpès par :


A - Recherche d'herpès simplex virus (HSV) dans les lésions par isolement en culture de cellules in vitro
B - Recherche d'antigènes HSV par cytodiagnostic en immunofluorescence sur des cellules prélevées à cette
patiente
C - Dosage des anticorps HSV par neutralisation ou fixation du complément dans un sérum prélevé de la
patiente
D - Dosage des anticorps par neutralisation ou fixation du complément dans 2 sérums de la patiente, à J1 et
J21
E - Dosage des IgM spécifiques de l'HSV type II dans un sérum de la patiente 5 à 10 jours après la
consultation
Bonne(s) réponse(s) : A B D

A - Réponse en 24 à 48 h.
B - En théorie possible par IF avec anticorps monoclonaux, en pratique uniquement pour diagnostic urgent. On observe des
faux négatifs.
C - Peu sensible, ne fait pas la distinction entre HSV 1/HSV 2 ni IgM/IgG.
D - Comme toute virose.
E - En cas de réactivation, ils peuvent manquer. Il s'agit en principe d'IgG dont le titre varie. On peut parfois détecter des IgM
persistants (ELISA ou IFI).

Le diagnostic d'herpès génital récurrent étant acquis, quelle attitude raisonnable conseillez-vous à cette femme ?
A - De réfléchir à l'éventualité d'une IVG
B - De prendre de l'Acyclovir® par voie orale jusqu'à l'accouchement qui pourra se faire alors par voie basse
C - D'accoucher systématiquement par césarienne
D - De rechercher une excrétion asymptomatique D'HSV dans les sécrétions génitales par inoculation à des
cultures de cellules in vitro
E - Aucune des propositions précédentes
Bonne(s) réponse(s) : C

La contamination se fait en per-partum.


L'aciclovir diminue seulement l'excrétion virale, ne prévient pas les récidives. Il y a d'autre part un risque de foetotoxicité et de
mutagénèse.
D - Si pratiquée juste avant l'accouchement, sa négativité pourrait permettre un accouchement par voie basse.

786
Exclusivement sur DOC - DZ : www.doc-dz.com NADJI 85
RESIDANAT EN POCHE TOME II
Cas Clinique en QCM

Cette femme ne revient vous voir qu'au moment du travail, à 8 mois 1/2.
Depuis la dernière visite, elle n'a pas eu de récidives manifestes d'herpès et l'examen gynécologique ne montre
pas de lésions. La poche des eaux n'est pas rompue. Si cette femme accouche par voie basse, le risque
d'herpès pour le nouveau-né peut être estimé à :
A - 85 %
B - 50 %
C - 20 %
D-5%
E - Moins de 1/1000
Bonne(s) réponse(s) : E

Egal à 1/1000
- Une primo infection à l'accouchement donne 75% de risques. Une récurrence génitale seulement 5%.

Cette femme ayant décidé d'accoucher par voie basse, quelle(s) mesure(s) allez-vous préconiser ?
A - Le dosage des AC anti HSV chez l'enfant
B - Le traitement systématique du nouveau-né par acyclovir oral pendant 10 jours
C - La séparation de l'enfant et de la mère
D - L'interdiction d'allaiter l'enfant au sein
E - Aucune des propositions ci-dessus
Bonne(s) réponse(s) : E

Pas d'électrode sur le scalp.


Bétadine® dans le vagin et soins du nouveau-né.
Recherche d'isolement viral au niveau des voies génitales maternelles pendant le travail. Si examen positif, discussion du
traitement de l'enfant par aciclovir (600 mg/m2/jour en 4 prises pendant 10 jours).
A - Valeur discutée.
C - Infection exceptionnelle à partir de gingivostomatite plutôt que d'herpès labial.

Un homme de 75 ans qui vient de subir une hémicolectomie gauche est placé dans une unité de soins intensifs ; il reçoit un
traitement par Ampicilline® 4 g/j par I.V., les suites sont d'abord simples : crise des gaz au 4ème jour, absence de fièvre,
diurèse bien contrôlée par une sonde vésicale la plaie opératoire n'est ni rouge, ni tuméfiée, il est perfusé par l'intermédiaire
d'un cathéter brachial mis en place le jour de l'intervention. Il existe une discrète rougeur du siège avec excoriation cutanée.
Au 5ème jour post opératoire, soudain, une fièvre apparait (39,5°C) avec des frissons répétés ; douze heures plus tard, le
malade signale une douleur thoracique droite et on découvre un foyer de râles alvéolaires, il n'y a pas de crachats
hémoptoïques ; le cliché du thorax révèle une image vaguement arrondie, non systématisée, relativement dense, située dans
le lobe inférieur droit et trois images rondes éparses sur le champ pulmonaire gauche.
Sur le plan biologique on note : hémogramme avec 21 000 leucocytes (82 % de polynucléaires neutrophiles), 350 000
plaquettes par mm3.
Urée sanguine : 15 mmoI/l, créatinine : 180 micromol/l. Quatre hémocultures effectuées lors du début de la fièvre sont
signalées positives dès le lendemain : il s'agit d'un staphylocoque.

Parmi les suivantes, quelle est la porte d'entrée la plus vraisemblable chez ce patient ?
A - Appareil respiratoire (laryngite post-intubation)
B - Urines
C - Cathéter veineux
D - Plaie opératoire
E - Téguments du siège
Bonne(s) réponse(s) : C

A - Surtout si intubation + ventilation.


B - Possible.
D - Possible mais l'absence de suppuration est précise dans le texte.
E - Possible mais pas de surinfection cutanée notée.

Les images pulmonaires sont probablement des métastases septiques staphylococciques. Quel(s) en est(sont)
le(s) risque(s) ?
A - Syndrome de détresse respiratoire aiguë (SDRA) par oedème pulmonaire lésionnel
B - Pleurésie purulente
C - Pyopneumothorax
D - Bronchospasme diffus
E - Vomique purulente
Bonne(s) réponse(s) : B C

Par rupture d'un abcès dans la plèvre.


E - Rarement à ce stade.

787
Exclusivement sur DOC - DZ : www.doc-dz.com NADJI 85
RESIDANAT EN POCHE TOME II
Cas Clinique en QCM

Quel(s) est(sont) parmi les suivants le (les) métastases infectieuses que l'on peut observer en pareille
circonstance ?
A - Méningite purulente
B - Epidurite suppurée
C - Ostéo-arthrite de la hanche
D - Pyélonéphrite bilatérale
E - Prostatite
Bonne(s) réponse(s) : B C D E

A - On observe plutôt des abcès cérébraux ; une méningite secondaire à la rupture d'un abcès éventuellement pourrait être
envisageable.
D - Secondaire à l'existence d'abcès intrarénaux.

Parmi les suivantes quelle(s) antibiothérapie(s) proposez-vous en première intention ?


A - Pénicilline G (15 M d'unités/jour)
B - Pénicilline M (Oxacilline®) 6 g/j et Métronidazole (Flagyl®) 1,5 g/j
C - Céphalosporine de 3ème génération (Claforan®) 4 g/j
D - Pénicilline M (Oxacilline®) 6 g/j et Amikacine (Amiklin®) 1 g/j
E - Ampicilline (Totapen®) à doses fortes (12 g/j)
Bonne(s) réponse(s) : D

Bien que Q.C.M. à réactualiser.


A, E - N'ont d'activité que sur les staphylocoques ne produisant pas de pénicillinase = exceptionnels en milieu hospitalier.
B - Le Métronidazole n'a pas d'activité anti-staphylococcique.
C - Activité médiocre sur staphylocoque phase +.
D - 50% des souches hospitalières de staphylocoques sont résistantes à la méticilline, 40% sont résistantes aux aminosides.
En 1987, en première intention, on utilise toujours la Vancomycine en I.V. Dans ce cas précis (insuffisance rénale modérée)
500 mg X 2/j à adapter selon les dosages sériques au pic et en résiduel.

Parmi les antibiothérapies proposées et compte tenu de l'énoncé, quelle(s) est(sont) celle(s) qui demande(nt)
une surveillance particulière ?
A - Pénicilline G 15 M d'unités/jour
B - Pénicilline M + Métronidazole
C - Céphalosporine de 3ème génération
D - Pénicilline M + Amikacine
E - Ampicilline à fortes doses (12 g/jour)
Bonne(s) réponse(s) : D

Seule utilisable dans le cas clinique.


Surveillance de la fonction rénale et du dosage sérique de l'amikacine = risque de néphrotoxicité et d'accumulation secondaire
à l'insuffisance rénale modérée du patient.

Si la réponse du laboratoire de bactériologie avait été "présence de bacille Gram négatif aéro-anaérobie" dans
les hémocultures, de quel(s) germe(s) pouvait-il s'agir dans le cas de ce patient ?
A - Klebsiella pneumoniae
B - Bacteroïdes fragilis
C - Salmonella typhi murium
D - Clostridium perfringens
E - Pseudomonas aeruginosa
Bonne(s) réponse(s) : A

B D - Anaérobies stricts.
E - Aérobie strict.
C A - Sont des entérobactéries aéro-anaérobies mais on ne retrouve C que dans un contexte d'infection ou de portage digestif.

788
Exclusivement sur DOC - DZ : www.doc-dz.com NADJI 85
RESIDANAT EN POCHE TOME II
Cas Clinique en QCM
Un agriculteur du centre de la France, sans antécédent notable, devient soudain fébrile à 39,5°C, fièvre accompagnée de
frissons, de toux sèche, d'une dyspnée modeste et d'un point de côté discret de la base thoracique droite. A l'auscultation on
perçoit à ce niveau un foyer de râles fins et un souffle rude inspiratoire. L'examen clinique est par ailleurs normal.
L'hémogramme montre 13 100 globules blancs par mm3, dont 78 % de polynucléaires neutrophiles. La VS est à 70/104 mm.
L'examen radiologique montre une opacité mal limitée, homogène, sans bronchogramme aérique ni rétraction. Située dans la
pyramide basale droite. On pense à une pneumopathie à pneumocoques.

Parmi les données suivantes, indiquez celle(s) qui est (sont) évocatrice(s) de cette étiologie :
A - Brutalité du début
B - Données de l'auscultation
C - Données radiologiques
D - Leucocytose avec polynucléose
E - Existence d'une toux
Bonne(s) réponse(s) : A B D

C - Il existe souvent un bronchogramme aérique.

Parmi les moyens biologiques ci-dessus, l'hypothèse pneumococcique peut être confortée par :
A - Résultat de l'hémoculture
B - Résultat de la contre immuno-électrophorèse des urines
C - C. réactive protéine élevée
D - Fibrinogène 7 g/l
E - Gamma GT : 60 mU (normale 25 mU)
Bonne(s) réponse(s) : A B C D

A - 30% d'hémocultures positives, argument pronostique.


B - Positif dans 75% des cas.
C - La C réactive protéine est par définition une protéine précipitant avec le polysaccharide C de paroi du pneumocoque et
dont l'augmentation a été rapportée dans les cas de pneumococcies.
D - syndrome inflammatoire.

Parmi les traitements suivants, indiquez celui ou ceux qui est(sont) le(s) plus sûrs :
A - Erythromycine orale
B - Pénicilline G injectable
C - Doxycycline intra-veineuse
D - Nétilmycine
E - Amoxicilline
Bonne(s) réponse(s) : B E

A - Non, 20% des souches résistantes.


B - Posologie discutée 4 à 12 millions d'UI/j (70 à 200 000 UI/kg/j) mais reste le meilleur traitement de la pneumococcie.
C - Non, plus de 40% des souches R.
D - Naturellement résistant.

Après 3 jours de traitement, la température est normalisée, mais l'image radiologique est inchangée, ainsi que
les signes d'auscultation. Parmi les propositions suivantes, indiquez celle qui vous paraît la plus logique :
A - Cette pneumopathie est venue surcharger une néoplasie bronchique
B - L'antibiothérapie est partiellement efficace
C - A ce stade, cette évolution est normale
D - Il existe probablement une pleurésie purule
E - Il existe une obstruction bronchique par un matériel muco-purulent nécessitant une fibroscopie pour
drainage
Bonne(s) réponse(s) : C

La guérison radiologique est toujours différée.

En dehors du pneumocoque, quel(s) autre(s) germe(s) aurai(en)t pu être en cause chez ce patient ?
A - Pseudomonas aéruginosae
B - Haemophilus influenzae
C - Klebsiella pneumoniae
D - Coxiella burneti
E - Brucella abortus
Bonne(s) réponse(s) : B

A - Principalement infection nosocomiale.


C - Surtout chez l'éthylique ou sur terrain débilité.
D - Donne un tableau de pneumonie atypique.

789
Exclusivement sur DOC - DZ : www.doc-dz.com NADJI 85
RESIDANAT EN POCHE TOME II
Cas Clinique en QCM
Une jeune femme présente brutalement : fièvre à 39°C, angine rouge, frissons, toux sèche, céphalées, conjonctivite,
adénopathies cervicales fermes et sensibles, râles bronchiques à l'auscultation.

Quel est votre diagnostic ?


A - PFLA
B - Légionellose
C - Tuberculose miliaire
D - Mononucléose infectieuse
E - Pneumopathie virale
Bonne(s) réponse(s) : E

Sans commentaire.

A quel agent penserez-vous en premier devant ce tableau clinique très évocateur ?


A - Mycoplasme d'Eaton
B - Myxovirus grippal
C - Entérovirus
D - Virus Cocksackie
E - Adénovirus
Bonne(s) réponse(s) : E

En faveur = association pharyngo-conjonctivite et bronchite aiguë.


A - Pneumopathie atypique.
B - Pas d'adénopathie, d'hyperthémie conjonctivale.
C D - Idem C comprend les cocksackies, les échovirus, les poliovirus et les entérovirus 68-71, surtout infections infantiles avec
peu de conjonctivite.

Quelle est l'image radiographique la plus évocatrice de votre diagnostic ?


A - Opacité arrondie basale
B - Condensation lobaire
C - Image hilifuge de type Glanzmann
D - Opacité de type pleural
E - Miliaire
Bonne(s) réponse(s) : C

Séméiologie radiologique.

Quel examen allez-vous demander pour confirmer votre diagnostic ?


A - Examen bactériologique des crachats
B - Tomographies
C - Ponction transtrachéale
D - Hémoculture
E - Deux sérologies virales à 15 jours d'intervalle
Bonne(s) réponse(s) : E

Par réaction de fixation du complément couramment. Les anticorps augmentent à partir du 6e jour, maximum en 15 à 20 jours,
et restent élevés 2 mois. Les titres résiduels sont faibles d'où un titre supérieur à 1/64e est en faveur d'une infection récente.
Positivité d'isolement du virus sur prélèvement de gorge, conjonctival et diagnostic rapide par ME, IF, ELISA ; fait uniquement
dans les cas graves ou douteux.

Quel traitement allez-vous mettre en route ?


A - Macrolide
B - Cytosine - arabinoside
C - Pénicilline
D - Rifampicine
E - Synergistine
Bonne(s) réponse(s) : C

On ne peut être sur que l'angine ne soit dûe aussi à une participation de streptocoque bêta, qu'il faut traiter pour prévenir le
risque de RAA.

790
Exclusivement sur DOC - DZ : www.doc-dz.com NADJI 85
RESIDANAT EN POCHE TOME II
Cas Clinique en QCM

Quelle(s) complication(s) pouvez-vous éventuellement constater au cours de l'évolution de cette maladie ?


A - Pleurésie purulente
B - Péricardite bénigne
C - Abcès du poumon
D - Méningite lymphocytaire
E - Anémie hémolytique à auto-anticorps froids
Bonne(s) réponse(s) : D

Sans commentaire.

Quelle(s) modification(s) de l'hémogramme pouvez-vous trouver chez cette malade ?


A - Polynucléose neutrophile
B - Leucopénie
C - Eosinophilie
D - Leucocytose
E - Lympho-monocytose
Bonne(s) réponse(s) : B E

Sans commentaire.

Un homme âgé de 45 ans, développe brutalement un état fébrile à 39°0 en plateau, accompagné de céphalées intenses, de
myalgies, d'arthralgies et d'une asthénie importante et sur lequel le traitement antithermique n'a pas d'effet.
En 4 jours, apparaît un érythème qui amène le patient à la consultation.
A l'examen, on peut noter en outre l'existence d'adénopathies cervico-axillaires et d'une splénomégalie. L'interrogatoire retient
uniquement la notion d'un bref séjour dans la campagne provençale 8 jours avant le début. La possibilité d'une fièvre
boutonneuse méditerranéenne est évoquée.

Quel est l'agent de cette maladie ?


A - Coxiella burnetti
B - Chlamydia psittaci
C - Rickettsia conori
D - Rickettsia akari
E - Haverillia moniliformis
Bonne(s) réponse(s) : C

A - Agent de la fièvre Q.
D - Agent de la fièvre vésiculeuse ou varicelliforme.
E - Infection secondaire à la morsure de rat associant fièvre + éruption morbilliforme + arthrites des grosses articulations +
pharyngo-laryngite.

Parmi les propositions suivantes, quelle(s) est(sont) celle(s) qui caractérise(nt) ce germe ?
A - Parasitisme intra-cellulaire obligatoire
B - Tropisme respiratoire
C - Production d'une exotoxine neurotrope
D - Tropisme endothélial vasculaire
E - Tropisme ganglionnaire
Bonne(s) réponse(s) : A D

A - Comme Brucella
D - Provoquent une vascularite oblitérante au niveau de petits vaisseaux : peau, SNC, myocarde surtout.

Quelle(s) proposition(s) peut (peuvent) s'appliquer à l'épidémiologie de cette maladie ?


A - Transmission par voie aérienne
B - Transmission manuportée
C - Transmission par morsure de tique
D - Transmission par une morsure de pou
E - Réservoir strictement humain
Bonne(s) réponse(s) : C

Rhipicéphalus sanguineux (tique du chien), réservoir autonome de rickettsie. Contamination par la salive de la tique au cours
de la piqûre ou par l'intermédiaire des doigts souillés mis en contact avec la conjonctive oculaire.

791
Exclusivement sur DOC - DZ : www.doc-dz.com NADJI 85
RESIDANAT EN POCHE TOME II
Cas Clinique en QCM

Quel(s) élément(s) caractérise(nt) l'éruption de cette maladie ?


A - Faite de macules rosées généralisées
B - Evolue en une seule poussée
C - Maculo-nodulaire
D - S'intensifie aux plis de flexion
E - Débute toujours sur le tronc
Bonne(s) réponse(s) : A B C

Macules, papules, nodules rosés parfois pupuriques.


Débute aux membres inférieurs et aux avant-bras.
L'atteinte des paumes et plantes est évocatrice.

L'examen de ce patient a omis de rechercher un signe clinique fondamental. Lequel ?


A - Pneumopathie lobaire
B - Chancre d'inoculation escharrotique
C - Purpura nécrotique des extrémités
D - Angine ulcéro-nécrotique
E - Hémorragies sous unguéales
Bonne(s) réponse(s) : B

Tache noire ulcérée avec zone inflammatoire, située aux membres inférieurs avec adénopathie satellite = témoin de la piqûre
de la tique.

Quel(s) examen(s) biologique(s) permet(ent) un diagnostic de certitude ?


A - Positivité des hémocultures anaérobies
B - Examen direct des sécrétions pharyngées
C - Electrosynérèse du sérum
D - Coproculture
E - Sérologie par immunofluorescence indirecte
Bonne(s) réponse(s) : E

A - Ne pousse pas en milieu d'hémoculture, nécessite des cultures spéciales sur oeuf embryonné
E - Plus banalement par réaction de fixation du complément, moins spécifique. L'I.F. indirecte est spécifique de R.Conori.

Quelle(s) complication(s) peu(ven)t apparaître en l'absence de traitement ?


A - Suppuration ganglionnaire
B - Hépatite rétentionnelle
C - Arthrites suppurées
D - Phlébites
E - Méningo-encéphalite
Bonne(s) réponse(s) : A

Sans commentaire.

Quel antibiotique doit être utilisé devant cette affection ?


A - Pénicilline G
B - Aminosides
C - Doxycycline
D - Sulfamides
E - Métronidazole
Bonne(s) réponse(s) : C

25 à 50 mg/10 g/j de doxycycline.


Il faut un ATB à visée intracellulaire d'où cyclines ou phénicolés.
Remarque : maladie à déclaration obligatoire.

792
Exclusivement sur DOC - DZ : www.doc-dz.com NADJI 85
RESIDANAT EN POCHE TOME II
Cas Clinique en QCM
Mr P., antillais de 30 ans, est professeur d'anglais dans un C E S. Il est atteint d'une malformation rénale congénitale. Il a subi
3 ans auparavant une fracture du crâne au cours d'un accident de la circulation c'est un éthylique avéré.
Une nuit il est pris de vives céphalées qui le réveillent. Il frissonne, prend sa température. II a 39° Au matin, toujours
céphalalgique il veut prendre son petit déjeuner mais le vomit peu après. Il vous fait appeler. Vous constatez une raideur de la
nuque, du rachis, il est photophobique, ses réflexes tendineux sont vifs.
La température est à 40°C. Le foie est un peu augmenté de volume. Le pouls est à 110 par minute. La pression artérielle à
13/8 cm Hg.
Vous évoquez le diagnostic de méningite à pneumocoque et vous hospitalisez le malade.

Parmi les éléments ci-après, un seul est sans relation avec le diagnostic envisagé :
A - La malformation rénale congénitale
B - La fracture du crâne
C - L'hyperréflectivité tendineuse
D - Les frissons
E - Les vomissements
Bonne(s) réponse(s) : A

Sans commentaire.

L'étiologie pneumococcique s'appuie à priori sur :


A - La brutalité d'installation du syndrome
B - L'absence d'Herpés
C - L'absence de purpura
D - L'absence d'arthralgies
E - Les antécédents de fracture du crâne
Bonne(s) réponse(s) : E

A B - Peuvent s'observer aussi bien dans les pneumococcies que dans les méningoccies.
C - Le purpura est un signe de ménigococcémie fulminante.

En l'absence de germe dans le LCR, l'identification de l'étiologie bactérienne peut être apportée par :
A - Les hémocultures
B - La recherche d'antigène pneumococcique dans les urines
C - Une broncho-aspiration
D - La recherche d'antigène pneumococcique dans le LCR
E - Une intradermo-réaction
Bonne(s) réponse(s) : A B C D

C - Parfois pneumopathie primitive ou non.


A - Phase septicémique fréquente.

Le médecin de l'hôpital qui a reçu le patient propose de le traiter sans attendre par 20 millions d'unités de
Pénicilline IV et 240 mg de Gentamicine IM. Parmi les propositions suivantes quel(quelles) est(sont) la(les)
proposition(s) acceptable(s) ?
A - L'association synergique est judicieuse
B - La gentamicine est inutile parce qu'elle ne traverse pas la barrière méningée
C - La gentamicine devrait être injectée par voie intrathécale
D - La gentamicine est contre-indiquée parce qu'elle est inactive sur le pneumocoque
E - La posologie proposée pour la gentamicine est inadaptée
Bonne(s) réponse(s) : B

La Pénicilline G à fortes doses est suffisante car grande sensibilité du pneumocoque (C.M.I. = 0,03 micro g/ml). Seuls 0,5%
des souches seraient tolérantes ou résistantes, c'est-à-dire que leur C.M.I. serait plus élevée.
Dans ce cas on utiliserait du céfotaxime.

793
Exclusivement sur DOC - DZ : www.doc-dz.com NADJI 85
RESIDANAT EN POCHE TOME II
Cas Clinique en QCM

L'hémogramme a montré : 4 900 000 G.R./mm3, 15 000 G.B./mm : 3, PN 76 % PE 10 % L 12 %, M 2 %.


A quel élément de l'observation rattachez-vous l'éosinophilie ?
A - A l'éthylisme
B - A l'origine antillaise (parasitose)
C - A la malformation rénale
D - A la fracture du crâne
E - A la méningite à pneumocoque
Bonne(s) réponse(s) : B

Les parasitoses existant dans les Antilles et donnant une hyperéosinophilie sont :
- anguilullose
- ankylostomose
- bilharziose digestive
- ascaridiose

Parmi les propositions suivantes, laquelle est correcte ?


A - Faire une déclaration de cette méningite à la DDASS
B - Faire donner un sulfamide à tous les élèves de sa classe
C - Faire faire un prélèvement de gorge à la femme, aux enfants et aux élèves
D - Faire fermer le C.E.S.
E - Faire une radiographie du crâne au malade
Bonne(s) réponse(s) : E

A - La recherche d'une fracture de l'étage antérieur de la base de crâne :


La communication entre le L.C.R. et les fosses sinusiennes explique la survenue de méningites récidivantes à pneumocoque.

Après 8 jours de traitement, Mr P est apparemment guéri. Il ne se plaint plus. Sa température est normale. Son
examen est normal. Au 10è jour la fièvre réapparait. Le patient a quelques démangeaisons. Mais l'examen
clinique est normal, l liquide céphalo-rachidien est clair, il contient 50 Iymphocytes/mm3. La chimie est normale.
Quelle est la cause la plus probable de cette rechute fébrile ?
A - Une rechute de la méningite
B - Un abcès du cerveau
C - Une ventriculite
D - Une allergie aux antibiotiques prescrits
E - Une hydrocéphalie
Bonne(s) réponse(s) : D

Devant une réponse fébrile, il faut ne penser qu'en premier au cloisonnement méningé. La pénicilline peut donner de
nombreuses manifestations d'allergie c'est-à-dire :
anaphylaxie, oedème de Quincke, éruption cutanée, fièvre, hyperleucocytose neutrophile.

La période des vacances est volontiers le mois des toxi-infections alimentaires. Ce jour d'août vous avez affaire à une
épidémie familiale :
L'épisode a frappé les convives d'un restaurant au bord de l'eau où l'on passe de table en table, en attendant le plat du jour,
des crudités, des charcuteries et un plat de poisson en conserve baigné dans une sauce épicée.
Les malades ont tous pris de ce poisson et ont ressenti les premiers symptômes 12 à 48 heures plus tard. Au moment de la
consultation, les malades présentaient vomissement, diarrhée incessante. Ils étaient fébriles de 38 à 38,5° degrés,
déshydratés, adynamiques.

La symptomatologie évoque-t-elle préférentiellement une toxi-infection à :


A - Salmonelle
B - Vibrio para-haemoliticus
C - Staphylococcus aureus
D - Vibrio cholerae
E - Clostridium perfringens
Bonne(s) réponse(s) : A

Les diarrhées des toxi-infections alimentaires à salmonelles sont fébriles et ont une incubation de 12 à 48 heures. La
charcuterie pourrait être responsable.
Les toxi-infections à clostridium ont une incubation de 8 à 12 heures ; le tableau comprend diarrhée et douleurs abdominales ;
les vomissements et la fièvre sont rares (plats en sauce, plats cuisinés à l'avance).
L'incubation des toxi-infections à staphylocoques est de 2 à 4 heures ; l'affection n'est pas fébrile. Le tableau associé diarrhée
et vomissements

794
Exclusivement sur DOC - DZ : www.doc-dz.com NADJI 85
RESIDANAT EN POCHE TOME II
Cas Clinique en QCM

Vous calculez le taux d'attaque de l'infection en utilisant la formule :


A - Nombre de nouveaux cas/population étudiée ( à un temps donné )
B - Nombre de cas existants/population étudiée ( pour une période donnée (mois, an) )
C - Nombre de cas existants/population étudiée ( pour une intervalle de temps couvrant l'événement )
D - Nombre de nouveaux cas/population étudiée ( pour une intervalle de temps couvrant l'événement )
E - Nombre de nouveaux cas/cas antérieurs ( pour une période donnée (mois an) )
Bonne(s) réponse(s) : C

Sans commentaire.

Quelle(s) mesure(s) doi(ven)t être prise(s) au plan général :


A - Fermeture du restaurant
B - Visite des locaux
C - Consultation médicale du personnel de cuisine
D - Prélèvements et contrôle des produits alimentaires
E - Prélèvements à faire aux malades
Bonne(s) réponse(s) : B C D E

Enquête alimentaire et déclaration sont obligatoires La visite des locaux parait importante pour vérifier les conditions de
conservation , de stockage des aliments. Les salmonelles doivent être recherchées dans les aliments, chez le personnel du
restaurant et chez les malades.Le personnel de cuisine ne pourra reprendre son travail qu'après deux coprocultures négatives
à une semaine d'intervalle.

Quel est l'essentiel du traitement que vous prescrivez ?


A - Symptomatique
B - Antibiothérapie
C - Sulfamidothérapie
D - Antiseptique intestinal léger
E - Antihistaminiques
Bonne(s) réponse(s) : A

Le traitement antiobiotique (amoxicilline) est réservé aux formes graves, ou chez les sujets débilités.

Un homme de 40 ans, marié, consulte pour une érosion du gland, indolore, à base ferme, unique et accompagnée d'une
adénopathie homolatérale ferme non inflammatoire apparue depuis 4 jours. Il admet avoir eu des rapports extra-conjugaux à
deux reprises, il y a un an avec une prostituée et il y a 8 jours avec une collègue de bureau. Il dit avoir utilisé à plusieurs
reprises il y a 3 ans des toilettes publiques fréquentées par des homosexuels, mais il nie absolument avoir eu des rapports à
cette occasion.

Quel est le diagnostic le plus probable ?


A- Chancre mou
B - Herpès
C - Chancre syphilitique
D - Aphte
E - Ulcération gonococcique
Bonne(s) réponse(s) : C

L'érosion est indolore à base ferme accompagnée d'une adénopathie non inflammatoire. Le chancre mou s'accompagne
d'adénopathies inflammatoires douloureuses ayant tendance à se fertiliser ; il n'est pas induré à la base.
Ce diagnostic doit être de toutes façons confirmé par des prélèvements bactériologiques et une sérologie. La recherche de
chancre mou doit être systématique (direct + mise en culture).

Si l'interrogatoire est fiable, et la durée d'incubation standard, qui peut l'avoir contaminé ?
A - La collègue de bureau
B - L'épouse
C - La prostituée
D - Les sièges de toilettes
E - Aucune des propositions précédentes
Bonne(s) réponse(s) : B

L'incubation de la syphilis primaire est de 3 semaines environ.

795
Exclusivement sur DOC - DZ : www.doc-dz.com NADJI 85
RESIDANAT EN POCHE TOME II
Cas Clinique en QCM

En l'absence de traitement immédiat de ce patient, quel(s) est(sont) le(s) risque(s) évolutif(s) ?


A - Epididymite
B - Alopécie
C - Réaction allergique à un traitement ultérieur
D - Lésions linguales
E - Polyadénopathies
Bonne(s) réponse(s) : B D E

La polyadénopathie, l'alopécie, les lésions linguales (lésions érosives appelées plaques fauchées) appartiennent au tableau
de syphilis secondaire.
Les orchiépididymites syphilitiques sont exceptionnelles.

Si vous instituez le traitement efficace en dose unique, quand le risque de contamination disparait-il ?
A - Immédiatement
B - En 3 jours
C - En 1 mois
D - En 6 mois
E - En 1 an
Bonne(s) réponse(s) : B

Les tréponèmes disparaissent des lésions en 24 à 48 heures.

Quelle(s) raison(s) peut-il y avoir d'instituer une surveillance clinique étiologique de ce patient ?
A - Efficacité inconstante du traitement
B - Risque de rechute même après traitement efficace
C - Déterminer la fin de contagiosité
D - Dépister une recontamination
E - L'empêcher de se réexposer au risque vénérien
Bonne(s) réponse(s) : D

Il n'y a pas de résistance du tréponème à la pénicilline si le traitement est bien conduit.

Un cuisinier, célibataire, de 34 ans, consulte pour une éruption fébrile. Il a été traité jusqu'à l'adolescence par l'Extencilline® à
la suite d'un rhumatisme articulaire aigu de l'enfance dont il ne garde aucune séquelle. Depuis trois jours, il présente une
éruption étendue sur le tronc, les membres et le visage, atteignant la paume des mains et la plante des pieds. Chaque
élément est papuleux, ferme, infiltré, non prurigineux, parfois recouvert de squames.
Il est fébrile depuis 24 heures, sa température est à 38,2 degrés. Il n'a pas de frissons, pas de céphalées, mais quelques
arthralgies.
A l'examen, on palpe des micro-adénopathies cervicales et inguinales. Le foie et la rate ne sont pas palpables. Il n'a pas
d'alopécie. Sa cavité buccale et sa gorge sont normales.
Il est homosexuel, averti des problèmes vénériens. Il a eu de nombreux partenaires occasionnels durant les derniers mois. Il
ne se souvient pas avoir eu de lésion génitale ou anale depuis un an, date à laquelle il a été traité pour une urétrite aiguë par
la spectinomycine (Trobicine®). Ses réactions sérologiques étaient alors négatives. Il n'a aucun autre antécédent notable.
L'examen des organes génitaux de la muqueuse anale par anuscopie est normal. L'hémogramme est normal, il n'a pas de
protéinurie. Il existe un syndrome inflammatoire modéré. Les réactions sérologiques sont positives : VDRL 16 U, TPHA 1/2560

Quel(s) élément(s) clinique(s) font partie de sa maladie ?


A - Eruption du tronc
B - Fièvre
C - Poly-adénopathies
D - Antécédent d'urétrite
E - Antécédent de RAA
Bonne(s) réponse(s) : A

Ces manifestations font partie du tableau de syphilis secondaire.

796
Exclusivement sur DOC - DZ : www.doc-dz.com NADJI 85
RESIDANAT EN POCHE TOME II
Cas Clinique en QCM

Quel(s) est(sont) le(s) traitement(s) efficace(s) ?


A - Pénicilline® un million d'unités/jour en IM pendant 8 j
B - Biclinocilline® (bénéthamine péni) un million d'unités/jour en IM pendant 15 j
C - Extencilline® (benzathine péni) 2,4 millions d'unités, injection unique
D - Doxycycline 400 mg per os en une seule prise
E - Erythromycine 1 g/j per os pendant 10 jours
Bonne(s) réponse(s) : B C

Les traitements par érythromycine et tétracyclines doivent être poursuivis pendant 15 jours.
La demie-vie de la pénicilline G est trop courte ; il faudrait l'administrer toutes les 6 heures pour obtenir des taux sériques
tréponémicides.
On choisit des pénicillines retard (Extencilline®) au semi-retard (Biclinocilline®, Bipénicilline®).
Le traitement préconisé actuellement pour les syphilis primaire et secondaire est l'Extencilline®, à la dose de 2,4 millions
d'unités.

Dans quelle(s) circonstance(s) le traiterez-vous par érythromycine 2g/j pendant 20 jours ?


A - Inefficacité du traitement pénicillinique
B - Allergie à la Pénicilline
C - Inefficacité du traitement par érythromycine 1 g/j pendant 10 jours
D - Intolérance à la doxycycline
E - N'importe quand, comme traitement de première intention
Bonne(s) réponse(s) : B

En cas d'allergie à la pénicilline, le choix se porte sur l'érythromycine ou sur les tétracyclines.

Six mois plus tard, le patient est en bonne santé, les réactions sérologiques sont maintenant VDRL 4 U, TPHA
1/1280. Que faites-vous ?
A - Nouveau traitement pénicillinique
B - Une réaction de Nelson
C - Nouveau traitement autre que pénicilline
D - Abstention
E - Une vérification du VDRL et du TPHA par un autre laboratoire
Bonne(s) réponse(s) : D

Il est normal que les réactions utilisant des antigènes tréponèmiques restent positives plus longtemps surtout lorsque le
traitement a été tardif.
Il peut d'ailleurs persister une cicatrice sérologique si le traitement n'a pas été précoce.

Après le traitement de ce patient, quelles sont parmi les éventualités suivantes, celle(s) qui vous parai(ssen)t
possible(s) ?
A - Négativation en deux ans du VDRL
B - FTA reste positif deux ans plus tard
C - VDRL reste positif deux ans plus tard
D - Négativation en deux mois du FTA
E - TPHA reste positif deux ans plus tard
Bonne(s) réponse(s) : A B C E

Les réactions utilisant les antigènes cardiolipidiques (Kline, VDRL) se négativent plus précocement que les autres ; le VDRL
peut rester faiblement positif (une croix) pendant très longtemps.

Lors du traitement de ce patient, quels sont les éléments d'une réaction d'Herxheimer que vous vous attendez à
observer ?
A - Hyperthermie
B - Urticaire immédiate
C - Bronchoconstriction
D - Exacerbation des signes cutanés
E - Chute tensionnelle brutale
Bonne(s) réponse(s) : A D

Cette réaction suit de quelques heures la première dose thérapeutique ; elle est en rapport avec une substance pyrogène
libérée lors de la lyse des tréponèmes. Elle régresse en 48 heures. Elle peut survenir lors du traitement des syphilis
secondaires (où elle est bénigne) et tertiaires.

797
Exclusivement sur DOC - DZ : www.doc-dz.com NADJI 85
RESIDANAT EN POCHE TOME II
Cas Clinique en QCM
Une jeune femme de 24 ans, 55 Kg, a des problèmes de santé personnels ou familiaux rapprochés. Conseillez-la. (Dans les
questions sur des médicaments, il ne vous est pas demandé de préciser la dose).

Son fils, 6 ans, a la scarlatine. Quelle(s) prophylaxie(s) conseillerez-vous pour sa nièce, 4 ans, qui vit à son
foyer ?
A - Oracilline® 500 000 unités par jour pendant huit jours
B - Oracilline® 500 000 unités par jour pendant huit jours et éviction scolaire obligatoire
C - Troléandomycine® 8 jours
D - Mynocine®, 50 mg 2 fois/jour pendant huit jours
E - Mynocine®, 50 mg 2fois/jour et éviction scolaire obligatoire
Bonne(s) réponse(s) :

Question annulée.

Elle-même a été mordue par un chien qui a disparu. Quelle(s) prophylaxie(s) conseillez-vous ?
A - Vérification du rappel antitétanique
B - Vaccin antirabique préventif
C - Vaccin antirabique curatif
D - Prophylaxie de la pasteurellose par les cyclines
E - Gamma-globulines standard
Bonne(s) réponse(s) : A B

Le chien ne pouvant être surveillé, la vaccination préventive s'impose.


D - Ne se fait pas ; la pasteurellose est une maladie très rare.

Son gynécologue trouve une salpingite ; la sérologie des chlamydiae est positive. Elle désire une nouvelle
grossesse. Quel traitement anti-infectieux lui donnera les chances maximum ?
A - Ampicille 10 millions/jour pendant trois semaines
B - Vibramycine® pendant trois semaines
C - Vibraveineuse® deux semaines. puis huit jours per os
D - Flagyl® pendant trois semaines
E - Bactrim® pendant trois semaines
Bonne(s) réponse(s) : B

Les antiobiotiques électifs des chlamydiae sont les tétracyclines et les macrolides.

Son mari contracte une méningite à méningocoque B. Quelle(s) prophylaxie(s) conseillerez-vous pour
l'entourage immédiat ?
A - Bactrim® pendant 5 jours
B - Oracilline® pendant 5 jours
C - Rovamycine® pendant 5 jours
D - Baypen® pendant 5 jours
E - Totapen® pendant 5 jours
Bonne(s) réponse(s) : C

L'avantage de la Rovamycine® est sa bonne pénétration dans les amygdales et les glandes salivaires.

Un homme de 65 ans, cultivateur n'ayant reçu aucun vaccin, est atteint d'un trismus d'apparition rapidement progressive, sans
fièvre. Son examen révèle un panaris torpide secondaire à une blessure superficielle remontant à une quinzaine de jours et
survenue lors de la réparation d'une barrière. Vous suspectez le diagnostic de tétanos.

L'agent responsable du tétanos possède un ou plusieurs des caractères suivants :


A - Germe tellurique
B - Germe aérobie strict
C - Entérobactérie Gram négatif
D - Germe sporulé
E - Germe saprophyte du pharynx
Bonne(s) réponse(s) : A D

Bacilles Gram positifs longs et fins anaérobies.


Les spores sont rondes terminales (renflements terminaux).

798
Exclusivement sur DOC - DZ : www.doc-dz.com NADJI 85
RESIDANAT EN POCHE TOME II
Cas Clinique en QCM

L'action du germe, cause du tétanos, se fait selon une ou plusieurs modalités suivantes. Laquelle ou lesquelles ?
A - Diffusion septicémique
B - Diffusion d'une exotoxine à tropisme nerveux
C - Diffusion d'une endotoxine libérée par lyse bactérienne
D - Diffusion d'une toxine agissant sur les muscles
E - Aucune d'entre elles
Bonne(s) réponse(s) : B

L'exotine circule endocytée dans les cellules axonales de façon centripète jusqu'à la moelle. Il peut y avoir une toxinémie mais
pas de septicémie.

Parmi les caractères des contractures du tétanos, quel(s) est(sont) celui(ceux) qui est(sont) exact(s) ?
A - Irréductibles
B - Indolores
C - Localisées uniquement à la face
D - Fébriles
E - Provoquées par certains gestes d'examens ou de traitement
Bonne(s) réponse(s) : A E

Le tétanos n'est pas une affection fébrile. Les contractures intenses douloureuses permanentes sont exagérées au moment
des paroxysmes spontanés ou déclenchés par la moindre excitation.

Le trismus est habituellement le premier signe du tétanos. On peut l'observer dans d'autres maladies. Laquelle
ou lesquelles ?
A - Paralysie faciale
B - Maladie sérique
C - Accident de la dent de sagesse
D - Méningite cérébrospinale
E - Phlegmon de l'amygdale
Bonne(s) réponse(s) : B C E

Dans les incidents des dents de sagesse, les phlegmons de l'amygdale, le trismus est unilatéral, le patient est fébrile.
L'arthrite temporo-mandibulaire post-sérique survient 9 jours après l'injection de sérum et est associée aux autres signes de la
maladie sérique.

Parmi les examens suivants, quel(s) est(sont) celui(ceux) nécessaire(s) pour poser le diagnostic de tétanos ?
A - Hémoculture
B - Prélèvement au niveau de la plaie
C - Sérodiagnostic
D - Intradermo-réaction
E - Aucun d'entre eux
Bonne(s) réponse(s) : E

Sans commentaire.

Si vous aviez vu ce patient au moment de sa blessure digitale, quel(s) geste(s) préventif(s) auriez-vous fait(s)
pour prévenir le tétanos ?
A - Injection de sérum antitétanique
B - Antibiothérapie
C - Injection de gamma-globulines standard
D - Vaccination anti-tétanique
E - Aucune des mesures précédentes
Bonne(s) réponse(s) : A D

Evident.

799
Exclusivement sur DOC - DZ : www.doc-dz.com NADJI 85
RESIDANAT EN POCHE TOME II
Cas Clinique en QCM

Devant ce patient suspect de tétanos, quelle(s) attitude(s) adoptez-vous en tant que médecin généraliste en
cabinet rural ?
A - Injection de sérum antitétanique
B - Antibiothérapie
C - Prescription d'une thérapeutique décontracturante
D - Hospitalisation d'urgence
E - Prophylaxie de l'entourage par spiramycine (Rovamycine®)
Bonne(s) réponse(s) : D

Sans commentaire.

Un homme de 28 ans vous consulte car il a été mordu à la jambe droite par un chien inconnu. Les plaies sont superficielles et
non pénétrantes. L'accident s'est produit dans le Loiret, deux heures auparavant. Cet homme circulait à vélomoteur et a été
attaqué par ce chien qui a disparu. L'animal ne présentait apparemment aucun trouble clinique. Dans les antécédents de cet
ouvrier agricole, on note une épilepsie traitée par Gardénal®, un diabète non insulino-dépendant non équilibré et une
vaccination antitétanique lors de son service militaire, il y a 8 ans, sans rappel depuis cette date.

Outre le tétanos et la rage, indiquez parmi les infections ou agents infectieux suivants, celle(s) qui peut
(peuvent) compliquer une morsure de chien :
A - Tularémie (Francisella tularensis)
B - Pasteurella multocida
C - Staphylocoque
D - Herpès
E - Chlamydia psittaci
Bonne(s) réponse(s) : B C

La tularémie se contracte dans 95 % des cas en France lors d'un contact direct avec le lièvre.
Toute plaie peut se surinfecter avec un staphylocoque.

Quelle prévention antitétanique faites-vous dans l'immédiat ?


A - Immunoglobulines antitétaniques et rappel de vaccin antitétanique
B - Immunoglobulines antitétaniques et vaccination complète antitétanique
C - Rappel de vaccin antitétanique seul
D - Immunoglobulines antitétaniques seules
E - Aucune
Bonne(s) réponse(s) : C

Le sujet a été vacciné moins de 10 ans auparavant.

Parmi les propositions concernant la vaccination antitétanique, indiquez celle(s) qui est(sont) exacte(s) :
A - La vaccination antitétanique est contre-indiquée chez le diabétique
B - Le vaccin antitétanique est composé de bactéries tuées
C - Chez l'adulte on peut faire une vaccination raccourcie avec le vaccin adsorbé : 2 injections à un mois
d'intervalle, le 1er rappel un an plus tard
D - La vaccination antitétanique n'est plus indiquée après l'âge de 65 ans
E - La vaccination antitétanique est contre-indiquée chez un sujet épileptique
Bonne(s) réponse(s) : C

Le vaccin antitétanique est une anatoxine. Le tétanos est actuellement une maladie touchant des sujets âgés qui n'ont jamais
été vaccinés ; il est donc très important de le faire. Les visites médicales (service militaire, médecine du travail) assurent le
plus souvent la protection des jeunes et des sujets d'âge moyen.

Parmi les propositions suivantes concernant la prévention de la rage chez un homme victime d'une morsure
récente par un chien inconnu, indiquez celle(s) qui est(sont) exacte(s) :
A - Lavage de la morsure avec de l'eau savonneuse
B - Traitement vaccinal antirabique
C - Aciclovir (Zovirax®) : 5 comprimés par jour pendant 7 jours
D - Application sur les plaies de gentamicine qui est rabicide
E - Injection d'immunoglobulines standard (0,3 ml/kg)
Bonne(s) réponse(s) : A B

Le rinçage de la plaie est très important visant à éliminer le virus (rinçage abondant) ; on applique ensuite un antiseptique. Il
ne faut pas suturer la plaie. Les plaies importantes peuvent être infiltrées avec du sérum antirabique.

800
Exclusivement sur DOC - DZ : www.doc-dz.com NADJI 85
RESIDANAT EN POCHE TOME II
Cas Clinique en QCM

Parmi les propositions concernant l'évaluation du risque rabique, indiquer celle(s) qui est(sont) importante(s) :
A - La morsure directe ou à travers un vêtement
B - La situation géographique où a eu lieu l'accident
C - L'existence d'un diabète chez la victime
D - L'espèce du chien
E - Le siège de la morsure
Bonne(s) réponse(s) : A B E

Les vêtements non déchirés constituent une barrière.


Le début de l'infection est d'autant plus rapide que la porte d'entrée est proche du SNC (face) ou située sur une zone
richement innervée (extrémités).
Le risque varie suivant les régions (nord-est de la France).

Parmi les propositions concernant la rage, indiquez celle(s) qui est(sont) exacte(s) :
A - Elle peut être transmise à l'homme par la salive d'un animal apparemment sain
B - L'incubation chez l'homme est, en moyenne, inférieure à une semaine
C - La rage réalise une encephalomyélite mortelle chez l'homme
D - L'enzootie actuelle en France a pour principal réservoir le chien
E - La vaccination antirabique curative ne peut être effectuée qu'en centre antirabique spécialisé
Bonne(s) réponse(s) : A C E

L'incubation de la rage chez l'homme est en moyenne de 40 jours (de une à deux semaines à un an).
Le principal réservoir en France est le renard, qui contamine l'homme par l'intermédiaire du chien et du chat (plus rarement
directement).
L'animal est infectant dans les jours précédant l'apparition de la maladie ; il peut donc paraître normal. Il doit être surveillé
pendant 14 jours (certificats vétérinaires à j1, j7 et j14).

Vous avez vu cette jeune femme de 25 ans il y a trois jours avec une angine érythémato-pultacée. Vous lui avez prescrit de la
Pénicilline G®.
La fièvre est tombée au deuxième jour.
Au troisième jour, surviennent :
- une réascension thermique à 38,5 degrés
- un érythème en plaques très prurigineuses sur le visage et le thorax
- des arthralgies au niveau de toutes les articulations.

Quel est le diagnostic le plus probable ?


A - Mononucléose infectieuse
B - Scarlatine
C - Allergie à la pénicilline
D - Lupus érythémateux disséminé
E - Rhumatisme articulaire aigu
Bonne(s) réponse(s) : C

Le rash de la scarlatine survient 48 heures après l'angine ; il débute à la base du thorax, de l'abdomen, à la racine des
cuisses, pas à la face. Il se généralise en 48 heures atteignant en dernier lieu la face. Il s'agit d'un exanthème diffus, sans
intervalles de peau saine, rouge écarlate, avec sur fond érythémateux diffus un semis de petites granulations (sensation de
granité) il prédomine aux plis de flexion, il n'est pas prurigineux.

Quel mécanisme physiopathologique commun peut expliquer l'ensemble des 3 symptômes observés au
troisième jour ?
A - Résistance bactérienne
B - Réaction d'Erxheimer
C - Réinfection par des germes de sortie
D - Auto-immunité
E - Complexes immuns circulants
Bonne(s) réponse(s) : E

Sans commentaire.

Quelle mesure thérapeutique prenez-vous dans l'immédiat ?


A - Remplacement de la pénicilline par un macrolide
B - Remplacement de la pénicilline par une céphalosporine
C - Ajout d'une tétracycline à la pénicilline
D - Ajout d'une céphalosporine à la pénicilline
E - Ajout de prednisone à la pénicilline
Bonne(s) réponse(s) : A

Il faut arrêter la pénicilline. Les macrolides sont une bonne alternative thérapeutique sur les angines streptococciques.

801
Exclusivement sur DOC - DZ : www.doc-dz.com NADJI 85
RESIDANAT EN POCHE TOME II
Cas Clinique en QCM

Quel examen de laboratoire est nécessaire pour établir le diagnostic ?


A - RAST-pénicilline
B - Test de transformation lymphoblastique à la Pénicilline G®
C - Dosage de complexes immuns circulants
D - Antibiogramme
E - Aucun des examens précédents
Bonne(s) réponse(s) : E

Sans commentaire.

Une femme de 30 ans, non vaccinée contre l'hépatite B, se plaint depuis 10 jours d'asthénie, d'anorexie intense et
d'arthralgies. L'examen clinique révèle un subictère et des traces d'injections intraveineuses en rapport avec une toxicomanie.
Vous évoquez une hépatite virale aiguë.

L'examen le plus utile pour confirmer ce diagnostic est :


A - Le dosage des transaminases sériques
B - La détermination du taux de prothrombine
C - L'électrophorèse des protéines plasmatiques
D - Le dosage dans le sang de la bilirubine libre et conjuguée
E - La numération formule sanguine
Bonne(s) réponse(s) : A

Il n'existe pas d'hépatite virale aiguë sans cytolyse. La mise en évidence d'une élévation marquée (plus de 10 fois la normale)
des transaminases sériques est indispensable au diagnostic.

Vis-à-vis de quel(s) virus, les drogués partageant leurs seringues, constituent-ils un groupe à risque accru
d'infection ?
A - Le virus de l'hépatite A
B - Le virus de l'hépatite B
C - L'agent Delta
D - Le virus VIH (virus de l'immunodéficience humaine)
E - Le virus de l'Herpès simplex type 2
Bonne(s) réponse(s) : B C D

Lors de l'hépatite virale A, la virémie est très brève ; le risque de transmission par voie sanguine peut être estimé nul.

Le diagnostic d'hépatite aiguë étant porté chez cette femme, la recherche d'antigène HBs dans le sang est
négative. Quelle recherche doit être positive pour rapporter au virus HB, l'hépatite aiguë observée ?
A - La recherche d'anticorps anti-Delta
B - La recherche d'anticorps anti-HBs
C - La recherche d'antigène HBe
D - La recherche d'anticorps anti-HBe
E - La recherche d'anticorps anti-HBc IgM
Bonne(s) réponse(s) : E

S'assurer de l'absence de facteur rhumatoïde qui peut être à l'origine de faux positif. La non mise en évidence de l'antigène
HbS s'explique de différentes manières :
- titre très faible de celui-ci
- l'antigène peut ne pas être reconnu par les immusérums ou les anticorps monoclonaux utilisés dans sa détection
- l'anticorps Hbs peut être sous forme complexée avec l'anticorps anti-Hbs (situation exceptionnelle lors de l'hépatite aiguë).

Après avoir porté le diagnostic d'hépatite aiguë à virus HB, vous estimez que le risque pour cette patiente de
passer à l'hépatite chronique est environ de :
A - 1/1000
B - 1/500
C - 1/10
D - 50%
E - 95 %
Bonne(s) réponse(s) : B

Le chiffre est plus proche de la réalité, si on exclut le portage chronique simple du virus B.

802
Exclusivement sur DOC - DZ : www.doc-dz.com NADJI 85
RESIDANAT EN POCHE TOME II
Cas Clinique en QCM

Pour parler de porteur chronique du virus HB, il faut ou faudra observer dans le sang au delà de 6 mois
d'évolution :
A - La présence d'antigène HBs
B - La présence d'antigène HBe
C - La présence d'antigène HBc
D - La présence d'anticorps HBs
E - L'absence d'anticorps HBc
Bonne(s) réponse(s) : A

Il faut exclure la proposition B, du fait de la possibilité d'association de l'antigène Hbe à l'antigène Hbs, (même si ce dernier
ne définit pas le portage chronique).

Le mari de cette femme, exempt jusqu'alors d'infection HB, a été vacciné contre l'hépatite B lorsque sa femme a
développé son hépatite aiguë. Que peut-on s'attendre à trouver dans le sang du mari quatre mois après la
vaccination s'il est, entre temps, resté indemne de contamination par le virus HB ?
A - Anticorps anti-HBs
B - Anticorps anti-HBe
C - Anticorps anti-HBc
D - Anticorps anti-HBc IgM
E - Anticorps anti-Delta
Bonne(s) réponse(s) : A

Certains vaccins de la première génération sont riches en antigène Hbe.

Un toxicomane à l'héroïne, âgé de 22 ans, est hospitalisé pour un ictère d'apparition récente, précédé de quelques jours de
fièvre et de malaise général avec nausées, selles décolorées, urines foncées. A l'examen à l'entrée, le foie est modérément
augmenté de volume, lisse, régulier. La rate est à peine palpable. Le bilan biologique pratiqué montre une bilirubine sanguine
à 60 mg/ml. Les ALAT (TGP) sont à 1 200 unités/l les ASAT (TGO) à 650 unités/l, le taux de prothrombine à 80 %,
l'hémogramme
4 900 000/mm3 globules rouges, 14,5 g/dl d'hémoglobine, 4 500/mm3 globules blancs dont 45 % polynucléaires neutrophiles.
Le diagnostic d'hépatite virale est porté.

Ce malade doit être traité par :


A - Extrait hépatique intramusculaire matin et soir
B - Prednisone 30 mg par jour
C - Vidarabine intraveineuse pendant 3 semaines
D - Interféron intraveineux pendant 15 jours
E - Aucun de ces médicaments
Bonne(s) réponse(s) : E

Il n'existe pas de traitement de l'hépatite aiguë virale non compliquée. Il faut simplement conseiller : le repos, l'abstention de
toute consommation d'alcool et l'éviction de tout médicament potentiellement hépatotoxique.

La surveillance habituelle de ce sujet comporte :


A - Etat de conscience
B - Ammoniémie
C - Taux de prothrombine
D - E.E.G
E - Courbe de température toutes les 3 heures
Bonne(s) réponse(s) : A C E

La complication précoce la plus redoutable (mais rare : 1/1000) de l'hépatite aiguë virale est l'évolution vers une forme
fulminante annoncée par une chute du taux de prothrombine. Elle se caractérise cliniquement par l'apparition d'un astérixis,
d'un foetor hépaticus, de signes neurologiques extra-pyramidaux, d'une désorientation temporo-spatiale, de troubles du
comportement, puis par l'évolution vers un coma et la mort dans 80 % des cas. La température habituellement redevenue
normale lors de l'apparition de l'ictère est en revanche élevée voire parfois diminuée dans les formes fulminantes. Un
syndrome hémorragique diffus est habituel.

803
Exclusivement sur DOC - DZ : www.doc-dz.com NADJI 85
RESIDANAT EN POCHE TOME II
Cas Clinique en QCM

La sérologie virale est positive pour l'antigène HBs chez ce malade. Quels marqueurs sériques permettront
d'affirmer qu'il s'agit d'une hépatite B aiguë ?
A - Présence d'anticorps anti-HBs
B - Présence d'anticorps anti-HBc
C - Présence d'anticorps IgM anti-HBc
D - Présence d'antigènes E
E - Présence d'anticorps E
Bonne(s) réponse(s) : C

En l'absence de facteur rhumatoïde, à l'origine de faux positifs, seule la présence d'anticorps anti Hbc permet le diagnostic
d'hépatite B aiguë.

Cette hépatite, à plus ou moins long terme chez ce malade, peut se compliquer de :
A - Hépatite chronique persistante
B - Cirrhose
C - Portage chronique du virus B
D - Hépatocarcinome
E - Hépatite chronique active
Bonne(s) réponse(s) : A B C D E

L'évolution de l'hépatite B aiguë est favorable dans 90 % des cas. Cependant chez certains malades la possibilité d'évolution
à plus ou moins long terme vers une hépatite chronique active, une cirrhose ou carcinome hépatocellulaire, en fait toute la
gravité.

La vaccination de sa concubine contre l'hépatite B va entraîner dans les semaines suivantes, l'apparition de :
A - Anticorps anti-HBs
B - Anticorps anti-HBc
C - Anticorps anti-HBe
D - Anticorps IgM anti-HBc
E - Toutes les réponses sont exactes
Bonne(s) réponse(s) : A

A - Certains vaccins de première génération contiennent l'antigène Hbe.

En cas de piqûre accidentelle d'une infirmière par le sang de ce malade porteur du virus B, et si l'anticorps anti-
HBs est absent du sérum de l'infirmière, il faut :
A - Administrer des gamma-globulines standards
B - Administrer des gamma-globulines anti-HBs
C - Commencer la vaccination
D - Associer vaccination et gamma-globulines anti-HBs
E - Associer vaccination et gamma-globulines standard
Bonne(s) réponse(s) : D

Il s'agit d'une immunoprophylaxie passive active. Les gammaglobulines anti Hbs assurent la protection passive du sujet-
contact en attendant la constitution d'une immunité active développée par le patient même.

Un homme de 35 ans est adressé pour un ictère apparu en trois jours, précédé d'arthralgies. Le malade est apyrétique : il
n'existe pas de prurit, ni de douleurs abdominales. Le diagnostic d'hépatite virale est porté.

Il est habituel d'observer au cours d'une hépatite aiguë bénigne :


A - ALAT sériques à 50 fois la normale
B - ASAT sériques à 45 fois la normale
C - Phosphatases alcalines à 2 fois la normale
D - Temps de Quick à 30 % avec un facteur V à 40 %
E - Albuminémie à 23 g/l
Bonne(s) réponse(s) : A B C

Le diagnostic d'hépatite virale aiguë est évoqué devant une cytolyse marquée (élévation des ASAT/ALAT à plus de 10 fois la
normale ; l'élévation des ALAT étant supérieure à celle des ASAT).
Si le TP est < 30 % et l'albuminémie < 30g/l, il s'agit déjà d'une hépatite grave et non bénigne.

804
Exclusivement sur DOC - DZ : www.doc-dz.com NADJI 85
RESIDANAT EN POCHE TOME II
Cas Clinique en QCM

Parmi les examens biologiques suivants, citez celui qui évoque une hépatite virale A aiguë :
A - Anticorps HBc type IgG positif
B - Anticorps HBs positif
C - Anticorps HA type IgG positif
D - Anticorps HA type IgM positif
E - Anticorps anti-CMV positif
Bonne(s) réponse(s) : D

sans commentaire.

Vous devez redouter l'apparition d'une hépatite fulminante devant :


A - Ictère marqué
B - Prurit
C - Diarrhée
D - Syndrome confusionnel
E - Hématomes spontanés
Bonne(s) réponse(s) : C D E

A Un ictère marqué peut, en effet, annoncer une forme fulminante ; mais très souvent il s'agit de forme choléstatique marquée
d'une hépatite aiguë bénigne.
Idem pour (C) : la diarrhée n'est pas un signe de l'hépatite fulminante.

Parmi les propositions thérapeutiques suivantes ; citez celle qui est utile au traitement de cette hépatite aiguë
bénigne :
A - Amoxicilline per os
B - Rifadine per os
C - Gammaglobulines spécifiques
D - Extraits hépatiques per os
E - Aucune de ces propositions
Bonne(s) réponse(s) : E

Sans commentaire.

Après 4 mois d'évolution, les transaminases restent élevées à 5 fois la normale, le diagnostic d'hépatite A étant
confirmé, quelle forme clinique retenez-vous chez ce patient ?
A - Hépatite chronique active
B - Hépatite chronique persistante
C - Hépatite prolongée simple
D - Cirrhose peu active
E - Aucune de ces propositions
Bonne(s) réponse(s) : C

Pour parler d'hépatite chronique, il faut une persistance des anomalies biologiques (cytolyse notamment) au-delà du sixième
mois.
La cirrhose fait habituellement suite à une hépatite chronique active. Elle ne peut donc se constituer qu'une à plusieurs
années après l'épisode aigu.

Une femme âgée de 28 ans, d'origine marocaine, en France depuis 4 ans, mère de deux enfants âgés de 8 et 6 ans,
scolarisés, consulte pour douleurs thoraciques, fièvre à 39°C, et amaigrissement progressif depuis six mois. Elle a quelques
douleurs des articulations : poignets, chevilles. l'examen clinique et l'examen radiographique permettent d'affirmer une
pleurésie gauche. La ponction pleurale retire un liquide citrin, poisseux. L'examen cardiaque est normal, pas de signes de
phlébite ; pas d'hépatomégalie ni d'ascite.
L'examen gynécologique est normal. L'intradermo-réaction tuberculinique est positive à 18 mm. Cette jeune femme n'a pas
été vaccinée par le BCG. Elle n'a pas d'antécédent chirurgical.

Parmi les signes radiologiques suivants, quel est celui qui sera le plus probablement constaté dans ce cas ?
A - Opacité dense basale avec niveau liquide horizontal
B - Opacité basale avec déplacement du médiastin du côté de l'opacité
C - Opacité systématisée avec bronchogramme aérien
D - Opacité basale dense, homogène, avec limite supérieure concave en haut et dedans
E - Opacité basale qui n'efface pas la coupole diaphragmatique
Bonne(s) réponse(s) : D

Opacité de la base avec une courbe de Damoiseau (limite supérieure concave remontant vers la périphérie sur le cliché de
face) ; elle se prolonge parfois par une ligne bordante. L'opacité se confond avec le diaphragme dont elle efface la limite
supérieure.

805
Exclusivement sur DOC - DZ : www.doc-dz.com NADJI 85
RESIDANAT EN POCHE TOME II
Cas Clinique en QCM

Pour orienter la recherche étiologique de la pleurésie vous retenez comme examen(s) :


A - Vitesse de sédimentation
B - Biopsies pleurales
C - Formule des éléments blancs du liquide pleural
D - Echocardiographie
E - Scanner thoracique
Bonne(s) réponse(s) : B C

Sans commentaire.

Dans ce cas vous retenez comme étiologie(s) possible(s) :


A - Tuberculose pleurale
B - Mésothéliome
C - Amibiase pleurale pulmonaire
D - Infection bactérienne
E - Lupus érythémateux disséminé
Bonne(s) réponse(s) : A E

L'évolution prolongée du tableau depuis 6 mois va contre C et D.


Aucun argument épidémiologique pour une mésothéliome.
La transplantation, l'altération de l'état général depuis 6 mois, la notion d'une IDR positive malgré l'absence de vaccination
sont en faveur d'une tuberculose. Mais un lupus est une autre hypothèse diagnostique, que l'on ne peut rejeter sur les
éléments donnés.

La preuve de l'étiologie tuberculeuse de la pleurésie peut être apportée par :


A - Intradermo-réaction tuberculinique
B - Formule des éléments blancs du liquide pleural
C - Recherche de BK aux cultures sur tubages gastriques
D - Biopsie pleurale
E - Numération formule sanguine
Bonne(s) réponse(s) : C D

A et B sont des éléments en faveur d'une tuberculose mais n'en sont pas une preuve.
L'examen des tubages au direct est souvent négatif, la culture des BK demande 4 à 8 semaines ; le meilleur examen pour
affirmer la nature tuberculeuse d'une pleurésie est la biopsie pleurale.

Si le diagnostic de tuberculose est confirmé, il est licite de prescrire à cette malade :


A - Intervention chirurgicale
B - Evacuation du liquide pleural
C - Triple antibiothérapie antituberculeuse
D - Traitement par isoniazide en monothérapie
E - Kinésithérapie respiratoire
Bonne(s) réponse(s) : B C E

L'évacuation du liquide pleural et la kinésithérapie précoce visent à éviter les séquelles (enkystement, fibrose).

Melle Dup. A., stagiaire puéricultrice âgée de 23 ans, consulte pour dysurie, brûlures mictionnelles, pollakiurie, température à
37°C. Le diagnostic d'infection de l'appareil urinaire (I.A.U.) est évoqué.

Parmi les bactéries à Gram négatif suivantes, quelle est la plus fréquemment responsable d'infection du tractus
urinaire dans cette situation clinique ?
A- Serratia marcescens
B - Proteus mirabilis
C - Escherichia coli
D - Yersinia enterocolitica
E - Klebsiella pneumoniae
Bonne(s) réponse(s) : C

sans commentaire.

806
Exclusivement sur DOC - DZ : www.doc-dz.com NADJI 85
RESIDANAT EN POCHE TOME II
Cas Clinique en QCM

Parmi les bactéries à Gram positif suivantes, quelle est !a plus fréquemment responsable d'IAU dans cette
situation clinique ?
A - Streptococcus pyogène (groupe A)
B - Streptococcus faecalis (groupe B)
C - Staphylococcus saprophyticus
D - Listeria monocytogènes
E - Lactobacillus
Bonne(s) réponse(s) : B

Sans commentaire.

Un examen cytobactériologique des urines est demandé avant de commencer un traitement. Le respect des
conditions correctes de prélèvement impose de :
A - Faire une toilette locale
B - Réaliser le prélèvement par sondage chez la femme
C - Utiliser un flacon stérile
D - Recueillir les urines du premier jet
E - Acheminer rapidement le prélèvement au laboratoire
Bonne(s) réponse(s) : A C E

Il faut éviter le sondage ; on y recourt uniquement lorsque le recueil des urines est impossible (incontinence).
Le premier jet ne doit pas être recueilli afin d'éviter au maximum l'isolement de germes présents sur les muqueuses génitales.

Vous recevez les premiers résultats de l'examen cytobactériologique des urines :


Leucocyturie : 5 x 10 exposant 5/ml (500/mm3)
Bactériurie : 10 exposant 6/mI
Examen direct : bacille Gram négatif
Quelle est l'interprétation possible ?
A - Infection guérie
B - Un nouveau prélèvement est nécessaire
C - Urine normale
D - Contamination due au prélèvement avec mise en culture tardive
E - Infection typique
Bonne(s) réponse(s) : E

Il existe donc une leucocyturie et une bactériurie > 10 exposant 5 germes/ml. Le diagnostic ne fait aucun doute.

Le laboratoire vient justement de vous faire parvenir la réponse : culture pure de E. coli sensible à tous les
antibactériens. Devant ce résultat quel(s) antibiotique(s) pouvez-vous utiliser ?
A - Colimycine (Colistine®)
B - Gentamycine (Gentalline®)
C - Triméthoprime + sulfaméthoxazole (Bactrim®)
D - Méticilline (Flabelline®, Pénistaph®)
E - Pénicilline G
Bonne(s) réponse(s) : A B C

Question mal posée.


Les antibiotiques A B C sont efficaces sur les bactéries Gram négatif, donc théoriquement utilisables. Il n'est pas question de
traiter une cystite par colistine parentérale ou gentamicine parentérale (la colistine per os n'est pas absorbée).
Le Bactrim® est une possibilité thérapeutique, mais il peut induire des effets indésirables sévères.
Les pénicillines A (ampicilline, amoxicilline...) sont efficaces sur les E. coli non sécréteurs de bêta-lactamase.
Dans le cas contraire, on peut prescrire de l'Augmentin® (amoxicilline + inhibiteur des bêta-lactamases) ; les quinolones de
deuxième génération (Noroxine® par exemple) sont une bonne alternative.

807
Exclusivement sur DOC - DZ : www.doc-dz.com NADJI 85
RESIDANAT EN POCHE TOME II
Cas Clinique en QCM
Monsieur Rat. Pierre, âgé de 39 ans, revient en France après un séjour de 2 ans en Guyanne avec une chimioprophylaxie à la
Nivaquine® poursuivie 6 semaines après son retour.
8 semaines après son arrivée, au cours d'une sortie de ski de fond, il est pris de malaises avec céphalées et vomissements.
Rentré à son domicile, sa température est à 40,2 degrés, avec frissons intenses.
Une automédication à l'aspirine, puis à la Nivaquine® (50 mg/jour) pendant 3 jours est sans effet sur cette fièvre qui persiste
entre 38°7 et 39°8 durant les jours suivants. L'examen clinique est alors strictement normal, sauf une légère splénomégalie
avec rate ferme accrochée en fin d'inspiration.

Parmi les éléments suivants du dossier, vous pourriez éliminer le diagnostic de paludisme devant :
A - Chimioprophylaxie correcte
B - Délai d'apparition de 8 semaines après son retour en France
C - Fièvre atypique, non rythmée
D - Inefficacité de la Nivaquine®
E - Aucun de ces éléments
Bonne(s) réponse(s) : E

La Guyane est une zone de résistance du paludisme aux amino-4-quinoléines.

La recherche systématique des plasmodiums sur frottis mince, met en évidence de nombreux trophozoïtes
annulaires de Plasmodium falciparum. Quelle attitude thérapeutique proposez-vous ?
A - Augmentation des doses de Nivaquine®
B - Changer pour la Flavoquine® per os
C - Changer pour le Lariam (Méfloquine) per os
D - Surveillance simple de l'évolution clinique
E - Perfusion de Quinoforme®
Bonne(s) réponse(s) : C

Son plasmodium est visiblement résistant aux amino-4-quinoléines (Nivaquine®, Flavoquine®) compte tenu de l'échec de la
prophylaxie correctement suivie 50 mg par jour est une dose trop faible pour traiter un accès palustre simple (on donne 500
mg par jour pendant 5 jours).
Il faut traiter son paludisme par la Méfloquine® (4-quinoléine méthanol) efficace contre les souches résistantes, par la quinine,
ou encore par l'association d'un sulfamide et de pyriméthamine (Fansidar®).
La Méfloquine® est le traitement le plus simple (1,5 g en 3 prises quotidiennes) le traitement parentéral n'est pas nécessaire.

Dans quel(s) pays peut-on retrouver des résistances aux animo-4-quinoléines ? Cochez la ou les propositions
exactes.
A - Brésil
B - Ile de la réunion
C - Gabon
D - Thaïlande
E - Rhodésie
Bonne(s) réponse(s) : A C D

Sans commentaire.

Si votre diagnostic est celui d'un paludisme, à quel tableau palustre correspond celui présenté par ce sujet ?
A - Accès pernicieux
B - Accès rémittent
C - Accès de reviviscence
D - Fièvre bilieuse hémoglobinurique
E - Accès de primo-invasion
Bonne(s) réponse(s) : E

L'incubation étant habituellement de 7 à 12 jours, il peut s'agir d'un accès de primo-invasion tardif en raison du rôle
partiellement protecteur possible de la Nivaquine®. C'est d'ailleurs le tableau clinique.

Dans la classification OMS des chloroquino-résistances observées dans le paludisme, à quelle définition
correspond la résistance R2 ?
A - Négativation de la parasitémie sans recrudescence précoce
B - Réduction partielle avec persistance des accès
C - Négativation de la parasitémie mais recrudescence précoce dans les 30 jours suivant le traitement
D - Pas de diminution de la parasitémie
E - Pas de décroissance thermique et aggravation vers une atteinte polyviscérale chronique
Bonne(s) réponse(s) : B

Question très "pointue"


C - Correspond à la résistance R1.
D - Correspond à la résistance R3.
808
Exclusivement sur DOC - DZ : www.doc-dz.com NADJI 85
RESIDANAT EN POCHE TOME II
Cas Clinique en QCM
Un homme de 35 ans, sans antécédent pathologique particulier, fumeur, présente la symptomatologie suivante :
- douleur thoracique droite brutale
- fièvre à 40°C avec frissons
- toux, expectoration adhérente au crachoir, rouillée
Examen clinique : syndrome de condensation pulmonaire de la base droite, herpès naso-labial.
Radiographie : opacité alvéolaire systématisée du lobe inférieur droit, petite réaction pleurale du même côté.
Vous pensez à une pneumonie infectieuse.

En fonction du tableau clinique et radiologique, quel germe vous paraît le plus probablement en cause ?
A - Mycoplasme
B - Bacille de Koch
C - Pneumocoque
D - Streptocoque anaérobie
E - Staphylocoque
Bonne(s) réponse(s) : C

Les symptômes cliniques sont caractéristiques d'une pneumonie franche lobaire aiguë : le pneumocoque est le germe le plus
souvent isolé dans ce type de tableau.

Le syndrome alvéolaire constaté sur la radiographie peut se caractériser par un ou plusieurs des éléments
suivants, éventuellement associés. Lequel (lesquels) ?
A - Opacité homogène non rétractile
B - Bronchogramme aérien
C - Miliaire
D - Images interstitielles
E - Hyperclarté
Bonne(s) réponse(s) : A B

L'image radiologique est effectivement une opacité dense, homogène, limitée par les scissures, de type alvéolaire avec un
bronchogramme aérien.
L'opacité n'est pas rétractile (par opposition aux images en rapport avec une atélectasie).

Le syndrome clinique de condensation parenchymateuse peut se caractériser par un ou plusieurs éléments


suivants. Lequel(lesquels) ?
A - Wheezing
B - Matité
C - Abolition du murmure vésiculaire
D - Souffle expiratoire voilé
E - Râles secs de fin d'inspiration
Bonne(s) réponse(s) : B C E

Sans commentaire.

Si vous voulez avoir une confirmation bactériologique, quel(s) examen(s) pouvez-vous demander ?
A - Hémoculture
B - Sérodiagnostic par séroagglutination
C - Tubages gastriques 3 jours de suite
D - Uroculture
E - Tests cutanés aux extraits microbiens
Bonne(s) réponse(s) : A

L'examen cytobactériologique des crachats peut apporter un argument lorsqu'un germe prédomine de façon très importante.
Les hémocultures sont positives dans environ 30 % des cas.

Quel antibiotique est-il justifié de donner en première intention chez ce malade ?


A - Pénicilline G
B - Erythromycine
C - Gentamicine
D - Thiamphénicol
E - Céfotaxime
Bonne(s) réponse(s) : A

Le traitement de première intention est la pénicilline G. En cas d'allergie on prescrit de l'érythromycine (20 % des
pneumocoques lui sont résistants).

809
Exclusivement sur DOC - DZ : www.doc-dz.com NADJI 85
RESIDANAT EN POCHE TOME II
Cas Clinique en QCM

Avec un traitement correct, l'évolution habituelle de cette pneumonie se fait selon une ou plusieurs des
modalités suivantes. Laquelle ou lesquelles ?
A - Défervescence thermique au plus tard à la 48ème heure
B - Nettoyage radiologique en 5 jours
C - Adénopathies calcifiées quelques mois plus tard
D - Images pulmonaires bulleuses séquellaires
E - Persistance de la toux pendant 15 jours
Bonne(s) réponse(s) : A

L'apyrexie doit être rapide, premier critère d'efficacité du traitement.


les images radiologiques peuvent mettre un mois à disparaître ; il peut persister une séquelle radiologique stable.
La toux ne persiste pas après 2 semaines de traitement efficace.

Un homme de 30 ans, homosexuel, est hospitalisé pour un état fébrile autour de 38° depuis 2 mois accompagné de sueurs
nocturnes. Il signale une diarrhée qui évolue en même temps. Il existe un amaigrissement de 10 kg. (Poids normal 75 kg, taille
1m77). On retrouve une hypertrophie des chaînes ganglionnaires cervicales postérieures et des adénopathies axillaires
bilatérales. Chaque ganglion mesurant environ 1 cm de diamètre. Ces adénopathies sont fermes.

Parmi les éléments cliniques présentés par cet homosexuel, vous retenez en faveur d'une pathologie liée au
HIV ?
A - Amaigrissement
B - Fièvre
C - Sueurs nocturnes
D - Adénopathies persistantes
E - Diarrhée
Bonne(s) réponse(s) : A B C D E

Tous ces signes se voient dans l'infection par le VIH.

Vous recevez l'hémogramme suivant : GR 35 000 000 ; Hb 13 g/dI ; GB 3 200/mm3 avec 30 % de Iymphocytes
; plaquettes 90 000. L'étude des surpopulations lymphocytaires montre :
A - Rapport T4/T8 inversé
B - Nombre absolu de T4 normal
C - Nombre absolu de T8 normal
D - Nombre absolu de T4 diminué
E - Nombre absolu de T8 diminué
Bonne(s) réponse(s) :

Question annulée.

Vous pratiquez une sérologie HIV par la méthode Western Blot qui montre la présence d'anticorps anti-HlV
dirigés contre les protéines, GPMO, GP41, P40, P25. Dès lors vous concluez au terme de ce bilan clinique et
paraclinique à :
A - Sérologie positive
B - Sérologie négative
C - Le sujet a une infection à HIV stade 4A
D - Le sujet a une infection à HIV stade 3
E - Le sujet a une infection à HIV stade 4C
Bonne(s) réponse(s) : A C

La présence de deux bandes d'anticorps dirigés contre deux antigènes d'enveloppe suffit à définir la positivité du Western
Blot. Gp 110 et Gp 41 sont des glycoprotéines d'enveloppe. P25 est la protéine de structure majeure du crps.
Le stade IV définit le stade symptomatique ; il est divisé en sous classes A, B, C, D, E.
A correspond à la présence de signes généraux : fièvre depuis plus d'un mois, amaigrissement de plus de 10 %, diarrhée de
plus d'un mois.

810
Exclusivement sur DOC - DZ : www.doc-dz.com NADJI 85
RESIDANAT EN POCHE TOME II
Cas Clinique en QCM

Vous êtes inquiet devant la diarrhée persistante. Les diarrhées au cours du SIDA peuvent habituellement être
dues à :
A - Cryptosporidium
B - Mycobactérium avium
C - Isospora belli
D - Candida
E - Cytomégalovirus
Bonne(s) réponse(s) : A B C E

Les cryptosporidies et l'isosporose sont deux affections fréquentes et redoutables par l'absence de traitement, la dénutrition
qu'elles peuvent entraîner.
Mycobacterium avium peut donner au cours du SIDA, des formes disséminées avec atteinte digestive et présence de
mycobactéries dans les selles. Les candida souvent retrouvés dans les selles ne sont pas responsables de diarrhée.

Deux mois plus tard, après une amélioration notable due à vos soins, le malade revient dans le service. Il a une
obnubilation progressive. Le scanner cérébral montre des images hypodenses arrondies. Parmi les atteintes
cérébrales suivantes, dire laquelle est probablement la cause de ce tableau :
A - Lymphome
B - Cryptococcose
C - Toxoplasmose
D - Abcès à candida
E - Cytomégalovirose
Bonne(s) réponse(s) : C

La cryptococcose est plutôt responsable de méningites, le CMV d'encéphalites.


D - N'est pas rencontré au cours du SIDA.
La réponse est C par argument de fréquence et par l'aspect scanographique : images hypodenses prenant le contraste en
couronne après injection. Mais les lymphomes cérébraux peuvent prendre cet aspect et c'est l'évolution sous traitement
antitoxoplasmique qui confirmera le diagnostic.

Quel traitement proposez-vous alors en urgence avant même la poursuite des investigations ?
A - Spiramycine
B - Pyriméthamine + sulfadoxine
C - Aciclovir
D - Amphotéricine B
E - Corticothérapie
Bonne(s) réponse(s) : B

Le traitement comprend l'adiazine® (sulfadiazine) et le Malocid® (pyriméthamine) : traitement d'attaque pendant 3 semaines
puis, traitement d'entretien souvent mal toléré sur le plan hématologique.

Un sénégalais résidant à Marseille est revenu depuis deux jours d'un pèlerinage à la Mecque. Il présente brusquement un
syndrome diarrhéique sévère fait de selles glairo-sanglantes très fréquentes (30 en quelques heures), des douleurs intenses
le long du cadre colique et des vomissements. A son arrivée à l'hôpital, la fièvre est à 39°C, les yeux sont excaves, la langue
est rôtie et la peau garde le pli. Il se plaint de crampes dans les membres inférieurs.
Son pouls est à 150/mn, la TA maxima à 7 cmHg.
Vous envisagez les hypothèses diagnostiques suivantes : choléra, dysenterie amibienne, amibiase colique maligne,
dysenterie bacillaire, salmonellose.

Quel(s) est(sont) le(s) élément(s) en faveur du diagnostic de choléra ?


A - Retour de la Mecque il y a deux jours
B - Déshydratation
C - Selles glairo-sanglantes incessantes
D - Fièvre à 39°C
E - Crampes dans les membres inférieurs
Bonne(s) réponse(s) : A B

La diarrhée du choléra est massive "eau de riz" sans syndrome infectieux avec déshydratation rapide.
E - Témoigne de l'hypokaliémie en rapport avec une diarrhée importante.

811
Exclusivement sur DOC - DZ : www.doc-dz.com NADJI 85
RESIDANAT EN POCHE TOME II
Cas Clinique en QCM

En faveur du diagnostic de dysenterie amibienne, vous retenez :


A - Retour de la Mecque il y a deux jours
B - Selles glairo-sanglantes
C - Fièvre à 39°C
D - Déshydratation
E - Crampes des membres inférieurs
Bonne(s) réponse(s) : A B

L'amibiase colique se traduit par un syndrome dysentérique avec selles glairo-sanglantes. Il n'y a pas de fièvre.

Quel(s) est(sont) le(s) élément(s) qui est(sont) en faveur de l'amibiase colique maligne ?
A - Retour de la Mecque
B - Selles glairo-sanglantes
C - Température a 39°C
D - Déshydratation
E - TA maxima à 7 cmHg
Bonne(s) réponse(s) : A B C D E

Ce tableau rare associe diarrhée glairo-sanglante, fièvre élevée, collapsus, possibilité de colectasie avec perforations
digestives et péritonites.

En faveur de la dysenterie bacillaire vous retenez :


A - Retour de la Mecque
B - Selles glairo-sanglantes incessantes
C - Température a 39°C
D - Déshydratation
E - TA maxima à 7 cmHg
Bonne(s) réponse(s) : A B C D E

Les shigelloses réalisent un syndrome dysentérique avec diarrhée importante, selles fécales glairo-sanglantes purulentes,
responsables d'une altération importante de l'état général avec déshydratation ; la température est souvent très élevée.

S'il s'agit d'une dysenterie bacillaire, quelle espèce de shigella pourrait être plutôt responsable du tableau de ce
malade ?
A - Shigella flexneri
B - Shigella bondii
C - Shigella sonnei
D - Shigella dysenteriae
E - Association Shigella flexneri et Shigella bondii
Bonne(s) réponse(s) : A

Sans commentaire.

Parmi les éléments suivants, lequel(lesquels) peut(peuvent) s'observer dans une toxi-infection à salmonelle ?
A - Selles glairo-sanglantes
B - Température à 39°C
C - Douleurs abdominales
D - Vomissements
E - Déshydratation
Bonne(s) réponse(s) : B C D

L'incubation des toxi-infections à salmonelles est de 12 à 24 heures. Le tableau comporte une fièvre, des vomissements, des
douleurs abdominales, une diarrhée liquide fétide.

Au terme de votre réflexion, vous retenez comme hypothèse(s) diagnostique(s) :


A - Choléra
B - Dysenterie amibienne banale
C - Intolérance alimentaire
D - Dysenterie bacillaire
E - Toxi-infection à salmonelle
Bonne(s) réponse(s) : D

Sans commentaire.

812
Exclusivement sur DOC - DZ : www.doc-dz.com NADJI 85
RESIDANAT EN POCHE TOME II
Cas Clinique en QCM
Un ingénieur agronome de 50 ans est hospitalisé en urgence pour un état fébrile à 39°C, d'installation brutale, accompagné
de céphalées intenses avec nausées et diarrhées modérées. Ce malade signale un séjour de deux mois en Thaïlande 15
jours auparavant, durant lesquels il affirme avoir pris régulièrement de la Nivaquine®, qu'il poursuit actuellement à titre
prophylactique. L'examen révèle une légère obnubilation sans désorientation temporo-spatiale ni raideur de nuque. Il n'y a pas
de subictère ni d'hépato-splénomégalie.
L'examen neurologique et cardio-pulmonaire est normal.
Le résultat des examens biologiques pratiqués dès l'admission est le suivant :
- hémoglobine : 86 g/litre
- érythrocytes : 3 T/litre
- leucocytes : 12 G/litre
polynucléaires neutrophiles : 75 %
polynucléaires éosinophiles : 1 %
lymphocytes : 18 %
monocytes : 2 %
- thrombocytes : 60 G/litre
- taux de prothrombine : 45 %
- fibrinogène : 0,8 g/litre
- nombreux érythrocytes parasites par des trophozoïtes de Plasmodium
- parasitémie = 6 %

D'après les données épidémiologiques, cliniques et biologiques, quelle est l'espèce plasmodiale responsable de
cet accès palustre ?
A - Plasmodium falciparum
B - Plasmodium vivax
C - Plasmodium malariae
D - Plasmodium ovale
E - Plasmodium cynomolgi
Bonne(s) réponse(s) : A

La notion d'une obnubilation, de désordres biologiques évocateurs d'une CIVD et d'une hyperleucocytose sont en faveur d'un
accès pernicieux donc à Plasmodium falciparum.

Vu l'espèce plasmodiale suspectée et la présence d'une légère obnubilation, quelle complication du paludisme
évoquez-vous ?
A - Fièvre tierce
B - Neuropaludisme
C - Paludisme viscéral évolutif
D - Fièvre bilieuse hémoglobinurique
E - Méningite parasitaire
Bonne(s) réponse(s) : B

On redoute un accès pernicieux.

Les examens biologiques :


A - Sont normaux
B - Font évoquer une fibrinolyse
C - Font évoquer une coagulation intravasculaire disséminée débutante
D - Font évoquer une hémolyse intravasculaire
E - Ne montrent qu'une lymphocytose aspécifique
Bonne(s) réponse(s) : C

L'accès pernicieux palustre est une cause de CIVD ; la thrombopénie, la fibripénie, la chute du TP sont très évocateurs.

La survenue dans ce contexte d'un accès palustre vous fait penser que :
A - Le malade a mal pris sa prophylaxie par la Nivaquine®
B - Le malade fait un accès de reviviscence à plasmodium malariae
C - Le malade fait un accès de primo-invasion à Plasmodium falciparum
D - Le malade fait une fièvre bilieuse hémoglobinurique
E - Le malade présente un paludisme viscéral évolutif
Bonne(s) réponse(s) : C

En Thaïlande, il existe des souches de Plasmodium résistantes aux amino-4-quinoléines. La Nivaquine® n'est pas une
prophylaxie adaptée.

813
Exclusivement sur DOC - DZ : www.doc-dz.com NADJI 85
RESIDANAT EN POCHE TOME II
Cas Clinique en QCM

Parmi les médicaments suivants, lequel prescrivez-vous pour traiter en urgence ce malade ?
A - La Nivaquine® (Chloroquine)
B - La Flavoquine® (Amodiaquine)
C - Le Flagyl® (Métronidazole)
D - Le Lariam® (Mefloquine)
E - La Primaquine
Bonne(s) réponse(s) : D

A et B sont des amino-4-quinoléines.


La Primaquine® est une amino-8-quinoléine possédant une action sur les gamétocytes intervenant dans la transmission à
l'anophèle ; elle est très toxique (hémolyse, méthémoglobinémie).
Le traitement de l'accès pernicieux repose sur la quinine IV.
Les rares souches de plasmodium résistantes isolées en Thaïlande et au Cambodge sont plutôt des diminutions de sensibilité
que des résistances vraies.
C'est peut être la raison pour laquelle elle n'est pas proposée ici.
La Méfloquine® est une 4-quinoléine méthanol ; elle est active sur Plasmodium falciparum résistant
aux amino-4-quinoléines.

Monsieur RAT. Pierre âgé de 39 ans, revient en France après un séjour de 2 ans en Guyane avec chimioprophylaxie à la
Nivaquine poursuivie 6 semaines après son retour. 8 semaines après son arrivée, au cours d'une sortie de ski de fond, il est
pris de malaises avec céphalées, et vomissements. Rentré à son domicile sa température est de 40,2° C avec frissons
intenses. Une automédication à l'aspirine, puis à la Nivaquine (500 mg/jour) pendant trois jours est sans effet sur cette fièvre
qui persiste entre 38,7° et 39,8° durant les jours suivants. L'examen clinique est alors strictement normal sauf une légère
splénomégalie avec rate ferme accrochée en fin d'inspiration.

Parmi les éléments suivants du dossier, quel élément permet d'éliminer le diagnostic de paludisme ?
A - Chimioprophylaxie apparemment correcte
B - Délai d'apparition de 8 semaines après son retour en France
C - Fièvre atypique, non rythmée
D - Inefficacité de la Nivaquine
E - Aucun de ces éléments ci-dessus
Bonne(s) réponse(s) : E

On est surpris du délai de 8 semaines, mais un délai aussi long peut s'observer avec certaines souches résistantes à la
chloroquine.

Si la recherche systématique de Plasmodium sur frottis mince met en évidence de nombreux trophozoïtes
annulaires de Plasmodium Falciparum, quelles sont les 2 attitudes possibles ?
A - Augmentation des doses de Nivaquine®
B - Changer pour la Flavoquine® per os
C - Changer pour le Lariam® ou l'Halofantrine per os
D - Surveillance simple de l'évolution clinique
E - Perfusion de Quinine®
Bonne(s) réponse(s) : C E

B - l'amodiaquine n'est plus qu'exceptionnellement utilisée en raison de ses effets indésirables.

Dans quel(s) pays peut-on retrouver des résistances aux amino-4-quinoléines ?


A - Brésil
B - Sénégal
C - Gabon
D - Thaïlande
E - Rhodésie
Bonne(s) réponse(s) : A B C D E

Sans commentaire.

Si votre diagnostic est celui d'un paludisme, à quel tableau palustre correspond celui présenté par ce sujet ?
A - Accès pernicieux
B - Paludisme viscéral évolutif
C - Accès de reviviscence
D - Fièvre bilieuse hémoglobinurique
E - Accès de primo-invasion
Bonne(s) réponse(s) : E

A - C'est une forme maligne avec des signes neurologiques.


B - Il existe une importante altération de l'état général.
C - On n'a pas la notion d'accès antérieurs.
D - S'accompagne d'état de choc et survient au décours d'une prise de quinine.
814
Exclusivement sur DOC - DZ : www.doc-dz.com NADJI 85
RESIDANAT EN POCHE TOME II
Cas Clinique en QCM

Dans la classification des chloroquino-résistances observées dans le paludisme à quelle définition correspond la
résistance R2 ?
A - Négativation de la parasitémie sans recrudescence précoce
B - Diminution de la parasitémie sans disparition complète
C - Négativation de la parasitémie mais recrudescence précoce dans les 30 jours suivant le traitement
D - Pas de diminution de la parasitémie
E - Pas de décroissance thermique et aggravation vers une atteinte polyviscéral -chronique
Bonne(s) réponse(s) : B

R1 - Disparition des parasites du sang périphérique mais réapparition à J14 du traitement.


R2 - Diminution de la parasitémie à moins de 25 % du taux avant traitement, sans disparition complète.
R3 - Faible ou absence de diminution de la parasitémie.

Un homme, âgé de 23 ans, développe dans les derniers jours de ses vacances passées en Corse un état fébrile à 39° avec
céphalées, myalgies et arthralgies, asthénie importante. Le seul traitement antithermique n'a pas d'effet. Dès son retour, soit 6
jours après les premiers symptômes, on peut constater la présence d'une éruption accompagnée d'adénopathies cervico-
axillaires et d'une splénomégalie. Un tel tableau évoque la possibilité d'une fièvre boutonneuse méditerranéenne.

Quel est l'agent de cette maladie ?


A - Chlamydia psittaci
B - Rickettsia conori
C - Salmonella typhi
D - Rickettsia rickettsi
E - Borrelia burgdorferi
Bonne(s) réponse(s) :B

A - Agent de la psittacose.
D - Agent de la fièvre pourprée des montagnes rocheuses.
E - Agent de la maladie de lyme.

Quelle(s) proposition(s) peut (peuvent) s'appliquer à l'épidémiologie de cette maladie ?


A - Inhalation de poussières souillées
B - Absorption d'eau contaminée
C - Morsure de tique
D - Porte d'entrée conjonctivale possible
E - En France, s'observe surtout dans les régions méditerranéennes
Bonne(s) réponse(s) : C D E

Le réservoir de virus est constitué par les tiques qui le transmettent à leur descendance, et par les rongeurs sauvages.

Quel(s) élément(s) caractérise(nt) l'éruption de la fièvre boutonneuse méditerranéenne ?


A - Eruption bulleuse
B - Evolution descendante
C - Entraîne une fine desquamation
D - Eruption maculo-papuleuse
E - Début derrière les oreilles
Bonne(s) réponse(s) : C D

L'éruption débute sur le tronc ou sur les membres.

L'examen de ce patient n'a pas été complet. Quel signe clinique fondamental aurait pu être mis en évidence ?
A - Aphtose buccale
B - Angine pseudo-membraneuse
C - Adénophlegmon inguinal
D - Arthrite fluxionnaire
E - Point d'inoculation escharrotique
Bonne(s) réponse(s) : E

La période d'état est caractérisée par la triade suivante : fièvre, éruption, escarre au point d'inoculation.

815
Exclusivement sur DOC - DZ : www.doc-dz.com NADJI 85
RESIDANAT EN POCHE TOME II
Cas Clinique en QCM

Quel(s) examen(s) biologique(s) permet(ent) un diagnostic de certitude ?


A - Hémoculture anaérobie
B - Prélèvement pharyngé
C - Sérologie par immunofluorescence indirecte
D - Coproculture
E - Recherche du germe dans l'expectoration
Bonne(s) réponse(s) : C

C'est le seul examen à visée diagnostique réalisé en pratique.

Parmi les antibiotiques suivants, lequel(s) peut (peuvent) être utilisé(s) ?


A - Doxycycline (Vibramycine®)
B - Céphalotine (Keflin®)
C - Ampicilline (Totapen®)
D - Gentamicine (Gentalline®)
E - Cotrimoxazole (Bactrim®, Eusaprim®)
Bonne(s) réponse(s) : A

On peut utiliser pour le traitement : les phénicolés ; les cyclines ; les macrolides ; la rifampicine ; les fluoroquinolones, qui ont
une bonne diffusion intracellulaire.

Mr. M. revient d'un voyage professionnel de 15 jours en République Centre Africaine. Il présente dès son retour un épisode
fébrile à 40°C, un état stuporeux, un myosis bilatéral, un ictère, une diarrhée, une dyspnée de Cheynes-Stockes
T.A. : 15/8 cm Hg
3 800.GB dont 82% de PNN/mm3
91 000 plaquettes/mm3

Quelle est l'infection que vous devez d'urgence rechercher ?


A - Dengue
B - Bilharziose à S mansoni
C - Paludisme
D - Amibiase colique
E - Trypanosomiase africaine
Bonne(s) réponse(s) : C

Sans commentaire.

Quel examen demanderez-vous dans ce but ?


A - Examen parasitologique des selles
B - Examen parasitologique sanguin par triple centrifugation
C - Recherche d'anticorps spécifiques
D - Frottis sanguin et goutte épaisse
E - Etude du liquide céphalo-rachidien
Bonne(s) réponse(s) : D

Le frottis peut être lu en quelques minutes. La goutte épaisse nécessite 24 heures.

Les éléments de mauvais pronostic comportent :


A - Troubles de la conscience
B - Aréflexie tendineuse
C - Splénomégalie
D - Hépatomégalie
E - Fièvre à 40°
Bonne(s) réponse(s) : A B D

Les signes de gravité d'un accès palustre sont :


- troubles neurologiques, fièvre > 40°C
- hépatomégalie
- hémoglobine < 7 g/dl
- bilirubine totale > 50 micromol/l
- créatininémie > 260 micromol/l
- hyperleucocytose > 12.000
- parasitémie > 2 %.

816
Exclusivement sur DOC - DZ : www.doc-dz.com NADJI 85
RESIDANAT EN POCHE TOME II
Cas Clinique en QCM

Parmi les examens suivants, le(s) quel(s) est (sont) utilisé(s) pour suivre l'évolution ?
A - Examen parasitologique du sang par triple centrifugation
B - Frottis sanguin + goutte épaisse
C - Sérologie spécifique
D - Glycémie
E - NFS
Bonne(s) réponse(s) : A B D E

A - Cette technique permet de mettre en évidence une faible parasitémie.


B - Technique classique pour suivre la parasitémie.
D - L'accès palustre et un traitement par la quinine peuvent être à l'origine d'hypoglycémie.
E - Pour suivre le taux d'hémoglobine.

Le traitement de ce patient comportera vraisemblablement :


A - Injection d'immunoglobulines spécifiques
B - Exsanguino transfusion
C - Quinine en perfusion lente
D - Administration d'oxygène
E - Corticothérapie par voie parentérale
Bonne(s) réponse(s) : C D

A E - N'ont jamais d'indication au cours d'un traitement anti-paludéen.


B - Se discute en extrême urgence au cours des formes graves avec hémolyse intense.

Depuis plusieurs mois, une femme de ménage de 42 ans se plaint de lésions inflammatoires du pourtour unguéal des 3
premiers doigts de la main droite. Les régions péri-unguéales sont douloureuses, rouges et oedématiées. La pression en fait
sourdre une gouttelette de pus jaunâtre. Sur les bords latéraux de chaque ongle atteint, existe une tache brun jaunâtre qui
tend à décoller l'ongle. Cette femme est obèse (70 kg pour 1,52m) et fume beaucoup, ce dont témoigne la coloration jaune de
certains de ses doigts. Elle prend les médicaments suivants : estroprogestatifs à visée contraceptive, calcium pour une
"tétanie", sulfamide hypoglycémiant pour un diabète "gras", furosémide (Lasilix®) pour une HTA. Elle refuse tout régime
hypocalorique mais ingurgite irrégulièrement de la "levure de bière" pour maigrir.
L'examen clinique permet de porter le diagnostic d'onyxis avec périonyxis dont l'origine mycosique est fortement soupçonnée.

Quelles sont les 3 causes de péri-onyxis avec écoulement purulent ?


A - Psoriasis vulgaire
B - Infection par Pseudomonas aeruginosa
C - Infection par Trichophyton interdigitale
D - Infection par Candida albicans
E - Infection par Staphylococcus aureus
Bonne(s) réponse(s) : B D E

Sans commentaire.

Parmi les facteurs suivants lequel ou lesquels peut(vent) favoriser une candidose unguéale ?
A - L'immersion fréquente des mains dans l'eau
B - Les soins excessifs de manucurie
C - Le contact avec des animaux familiers
D - Le tabagisme
E - Une candidose vaginale
Bonne(s) réponse(s) : A B E

La macération et les traumatismes répétés favorisent l'infection à Candida.

Chez cette femme, quel(s) facteur(s) général(aux) peu(ven)t favoriser une candidose unguéale ?
A - Un diabète
B - La prise de calcium au long cours
C - Les sulfamides hypoglycémiants
D - Le furosémide
E - L'ingestion de "levure de bière"
Bonne(s) réponse(s) : A

Les autres facteurs favorisant une candidose sont :


- antibiothérapie, corticothérapie, immunosuppression, contraception oestroprogestative
- diabète
- grossesse
- humidité, macération, acidité.

817
Exclusivement sur DOC - DZ : www.doc-dz.com NADJI 85
RESIDANAT EN POCHE TOME II
Cas Clinique en QCM

Vous devez rechercher d'autre(s) localisation(s) d'une candidose sous la forme de :


A - Glossite
B - Perlèche
C - Intertrigo sous-mammaire
D - Teigne
E - Anite
Bonne(s) réponse(s) : A B C E

Les teignes sont dues à des dermatophytes.

Vous décidez de traiter cette malade par un antifongique per os. Quel médicament prescrivez-vous ?
A - Nystatine® en comprimés
B - Amphotéricine B® en suspension
C - Kétoconazole® en comprimés
D - Métronidazole® en comprimés
E - Griséofulvine® en comprimés
Bonne(s) réponse(s) : C

A - La mycostatine n'est pas absorbée.


B - L'amphotéricine B possède trop d'effets indésirables pour être prescrite dans cette indication.
D - Le métronidazole n'a pas d'effet sur les champignons.
E - La griséofulvine n'est active que sur les dermatophytes.

Pour le médicament que vous avez prescrit, quel paramètre biologique devez-vous surveiller régulièrement ?
A - L'uricémie
B - La triglycéridémie
C - La sérologie anti-candidosique
D - Les transaminases (Asat, Alat)
E - La glycémie post-prandiale
Bonne(s) réponse(s) : D

Le kétoconazole peut provoquer : des troubles digestifs, une hépatite, une éruption, une diminution de la libido, une
gynécomastie.

Dans l'hypothèse où vous utilisez un traitement local, quel(s) est (sont) celui (ceux) qui peu(ven)t être efficace(s)
?
A - Nystatine en pommade
B - Amphotéricine B en crème
C - Dérivés imidazolés en crème
D - Métronidazole en crème
E - Griséofulvine en pommade
Bonne(s) réponse(s) : A B C

Sans commentaire.

818
Exclusivement sur DOC - DZ : www.doc-dz.com NADJI 85
RESIDANAT EN POCHE TOME II
Cas Clinique en QCM
Un ingénieur agronome de 50 ans est hospitalisé en urgence pour un état fébrile à 39°, d'installation brutale, accompagné de
céphalées intenses avec nausées et diarrhées modérées. Ce malade signale un séjour de deux mois en Thaïlande 15 jours
auparavant, durant lesquels il affirme avoir pris régulièrement de la Nivaquine, qu'il poursuit actuellement à titre
prophylactique. L'examen révèle une légère obnubilation sans désorientation temporo-spaciale ni raideur de nuque. Il n'y a
pas de de subictère ni d'hépato-splénomégalie. L'examen neurologique et cardio-pulmonaire est normal.

Le résultat des examens biologiques pratiqués dès l'admission sont les suivants :
- hémoglobine : 86 g/litre
- érythrocytes : 3 T/litre
- leucocytes : 12 G/litre
polynucléaires neutrophiles : 75%
polynucléaires éosinophiles : 1%
Iymphocytes : 18%
monocytes : 2%
- Taux de prothrombine : 45%
- Fibrinogène : 0,8 g/litre
- nombreux érythrocytes parasités par des trophozoïtes de Plasmodium. Parasitémie = 6%.

D'après les données épidémiologiques, cliniques et biologiques, quelle est l'espèce plasmodiale responsable de
cet accès palustre ?
A - Plasmodium falciparum
B - Plasmodium vivax
C - Plasmodium malariae
D - Plasmodium ovale
E - Plasmodium cynomolgi
Bonne(s) réponse(s) : A

L'existence de signes neurologiques (obnubilation) et la survenue de l'accès au cours d'un traitement prophylactique par
Nivaquine, plaident en faveur de P. falciparum.

Vu l'espèce plasmodiale suspectée et la présence d'une légère obnubilation, quelle complication du paludisme
évoquez-vous ?
A - Fièvre tierce
B - Neuropaludisme
C - Paludisme viscéral évolutif
D - Fièvre bilieuse hémoglobinurique
E - Méningite parasitaire
Bonne(s) réponse(s) : B

L'obnubilation est un signe neurologique et témoigne donc de la gravité de l'accès palustre.

Les examens biologiques :


A - Sont normaux
B - Font évoquer une fibrinolyse
C - Font évoquer une coagulation intra-vasculaire disséminée débutante
D - Font évoquer une hémolyse intra-vasculaire
E - Ne montrent qu'une Iymphocytose aspécifique
Bonne(s) réponse(s) : C

La CIVD doit être évoquée devant la chute du TP et du fibrinogène.

La survenue dans ce contexte d'un accès palustre vous fait penser que :
A - Le malade a mal pris sa prophylaxie par la Nivaquine®
B - Le malade fait un accès de reviviscence à Plasmodium malariae
C - Le malade fait un accès de primo-invasion à Plasmodium falciparum résistant à la Nivaquine®
D - Le malade fait une fièvre bilieuse hémoglobinurique
E - Le malade présente un paludisme viscéral évolutif
Bonne(s) réponse(s) : C

Sans commentaire.

819
Exclusivement sur DOC - DZ : www.doc-dz.com NADJI 85
RESIDANAT EN POCHE TOME II
Cas Clinique en QCM

Parmi les médicaments suivants, lequel prescririez-vous pour traiter en urgence ce malade ?
A - La Nivaquine®
B - La Flavoquine®
C - Le Flagyl®
D - Le Lariam®
E - La Primaquine®
Bonne(s) réponse(s) : D

Compte-tenu de l'existence d'une obnubilation, il serait plus prudent d'utiliser la quinine par voie veineuse dont l'action est
beaucoup plus rapide que celle du Lariam.

Une jeune femme est atteinte de fièvre à 39° C d'apparition brutale, avec frissons, toux sèche, céphalées, pharyngite et
conjonctivite. On découvre quelques râles sous-crépitants.

Le diagnostic le plus vraisemblable est :


A - Pneumonie franche lobaire aiguë
B - Légionellose
C - Pneumopathie virale
D - Mononucléose infectieuse
E - Tuberculose
Bonne(s) réponse(s) : C

A et B - Ce sont des pneumopathies lobaires.


C - L'atteinte pharyngée et conjonctivale sont évocatrices de l'étiologie virale.
D - Au cours d'une MNI les signes extrapulmonaires sont plus riches.
E - Le début est progressif.

A quel agent penserez-vous en premier lieu devant ce tableau clinique ?


A - Mycoplasma pneumoniae
B - Myxovirus
C - Pneumocoque
D - Legionella pneumophila
E - BK
Bonne(s) réponse(s) : B

Les myxovirus et le virus respiratoire syncitial sont responsables d'épidémies dans les collectivités.
A - Le début est progressif.

Quelle est l'image radiographique qui serait la plus évocatrice de votre diagnostic ?
A - Opacité arrondie basale
B - Condensation lobaire
C - Image hilifuge
D - Opacité de type pleural
E - Miliaire
Bonne(s) réponse(s) : C

C'est l'image radiologique des pneumopathies virales et atypiques.

Quel examen pouvez-vous demander pour confirmer votre diagnostic ?


A - Examen bactériologique des crachats
B - Tomographies pulmonaires
C - Ponction transtrachéale
D - Hémocultures
E - Deux sérologies à 15 jours d'intervalle
Bonne(s) réponse(s) : E

La confirmation sérologique n'est pas indispensable devant un tel tableau si l'évolution est favorable.

820
Exclusivement sur DOC - DZ : www.doc-dz.com NADJI 85
RESIDANAT EN POCHE TOME II
Cas Clinique en QCM

Quel traitement allez-vous mettre en route ?


A - Gentamicine®
B - Zovirax® (Acyclovir)
C - Pénicilline G®
D - Rifampicine®
E - Aspirine®
Bonne(s) réponse(s) : E

Les antibiotiques n'ont pas d'indications en dehors d'une surinfection bactérienne. Le Zovirax est efficace sur les
pneumopathies varicelleuses.

Quelle complication pouvez-vous éventuellement constater au cours de l'évolution de cette maladie ?


A - Endocardite
B - Surinfection bactérienne
C - Diarrhée
D - Ostéite
E - Cellulite
Bonne(s) réponse(s) : B

Cette éventualité doit être dépistée par un examen clinique répété qui décèlerait : réascension thermique, crachats purulents,
signes de condensation.

Quelle est la modification habituelle de l'hémogramme dans cette pathologie ?


A - Thrombopénie
B - Leucopénie
C - Hyperéosinophilie
D - Anémie
E - Hyperleucocytose
Bonne(s) réponse(s) : B

Leucopénie avec hyperlymphocytose relative.

M. P., 30 ans, éthylique, professeur dans un C.E.S.. a une malformation rénale congénitale. 3 ans auparavant il a eu une
fracture du crâne (accident de la circulation). Une nuit, il est pris de vives céphalées qui le réveillent, il frissonne. Au matin,
toujours céphalalgique, il prend son petit déjeuner mais le vomit peu après. Il est photophobique. Les réflexes ostéo-tendineux
sont vifs. La température est à 40° C. Le foie est augmenté de volume (flèche hépatique = 15 cm). La fréquence cardiaque est
à 110 par minute. La pression artérielle à 130/80 mmHg. Vous évoquez le diagnostic de méningite à pneumocoque.

Le(s) élément(s) suivant(s) est (sont) en relation avec le diagnostic de méningite :


A - La malformation rénale congénitale
B - Les céphalées
C - La photophobie
D - L'hépatomégalie
E - Les vomissements
Bonne(s) réponse(s) : B C E

B E - Témoignent de l'hypertension intracrânienne.


C - Traduit l'irritation des méninges.

L'étiologie pneumococcique s'appuie à priori sur :


A - L'intensité de la fièvre
B - L'absence d'herpès
C - L'absence de purpura
D - L'absence d'arthralgies
E - Les antécédents de fracture du crâne
Bonne(s) réponse(s) : E

Hôte normal de l'oropharynx, le pneumocoque pénètre dans le système nerveux central par les brèches ostéo-méningées.

821
Exclusivement sur DOC - DZ : www.doc-dz.com NADJI 85
RESIDANAT EN POCHE TOME II
Cas Clinique en QCM

En l'absence de germe dans le LCR, l'origine bactérienne de la méningite pourrait être apportée par :
A - Hémocultures
B - Urocultures
C - Broncho-aspiration
D - Electroencéphalogramme
E - Intradermo-réaction spécifique
Bonne(s) réponse(s) : A

Sans commentaire.

Quel(s) traitement(s) proposez-vous en première intention ?


A - 30 millions d'unités de Pénicilline G® par jour
B - 12 g d'Amoxicilline® par jour
C - 4 g de Tétracycline® par jour
D - 2 g de Claforan® par jour
E - 240 mg de Gentamicine® par jour
Bonne(s) réponse(s) : A B

E - Se discute en association à A ou B, pendant quelques jours, pour augmenter la vitesse de bactéricidie. En cas d'allergie
on prescrira une céphalosporine de 3ème génération ou un phénicolé.

Parmi les propositions suivantes, laquelle est vraie ?


A - Faire une déclaration de cette méningite à la DDASS
B - Donner un sulfamide à tous les élèves de sa classe
C - Faire un prélèvement de gorge aux membres de sa famille
D - Laisser ouvert le C.E.S.
E - Vacciner sa famille contre le pneumocoque
Bonne(s) réponse(s) : D

Seules les méningites à méningocoques doivent être déclarées.

Après 8 jours de traitement, M. P. est apparemment guéri. Sa température est normale. Au 10ème jour, la fièvre
réapparait. Le patient se plaint de prurit, mais l'examen clinique reste normal. Le liquide rachidien est clair et
contient 50 lymphocytes/mm3, la chimie du LCR normale. La cause la plus probable de la rechute fébrile est :
A - Une rechute de la méningite
B - Un abcès de cerveau
C - Une ventriculite
D - Une allergie aux antibiotiques
E - Une hydrocéphalie
Bonne(s) réponse(s) : D

L'absence d'élévation de la protéinorachie et l'apparition du prurit dix jours après le début du traitement concordent pour
retenir le diagnostic d'allergie aux antibiotiques.

Mr. X., âgé de 37 ans, ancien toxicomane, séropositif connu vis-à-vis du virus de l'immunodéficience humaine (test Elisa et
Western Blot) depuis 4 ans environ, consulte pour une toux persistante non productive évoluant depuis 15 jours. Depuis 2
jours, il se sent dyspnéique tandis que sa température atteint 39°5 . Le reste de l'examen clinique est normal.

Quel(s) est (sont) le(s) diagnostic(s) le(s) plus probable(s) chez ce patient ?
A - Pneumonie à.Pneumocoque
B - Pneumonie à Cryptosporidilum
C - Pneumonie à Pneumocystis carinii
D - Pneumonie varicelleuse
E - Pneumonie à Cytomégalovirus
Bonne(s) réponse(s) : C

A - Tableau plus brutal, avec crachats et symptomatologie pulmonaire plus riche.


B - Les cryptosporidies sont à l'origine de diarrhées chroniques.
D - La pneumopathie est exceptionnellement isolée.
E - Moins fréquente que la pneumopathie à P. carinii, elle peut s'y associer. Le tableau est plus souvent celui d'une infection
disséminée à CMV, associant : encéphalite, colite, pneumopathie, insuffisance surrénale, rétinite. Le diagnostic est
histologique.

822
Exclusivement sur DOC - DZ : www.doc-dz.com NADJI 85
RESIDANAT EN POCHE TOME II
Cas Clinique en QCM

Relevez le(s) examen(s) complémentaire(s) qui devrai(en)t permettre d'affirmer le diagnostic étiologique :
A - Recherche d'une antigénémie circulante
B - Hémocultures
C - Examen direct après coloration sur le produit de lavage broncho-alvéolaire
D - Examen tomodensitométrique thoracique
E - Myéloculture
Bonne(s) réponse(s) : C

Les pneumocystis sont mis en évidence par la coloration de Grocott ou de Gram-Weigert.

Relevez le(s) traitement(s) anti-infectieux qui paraît(ssent) adapté(s) au diagnostic :


A - Céfotaxime (Clarofan®)
B - Triméthoprime-Sulfaméthoxazole (Bactrim®)
C - DHPG (Ganciclovir®) par voie sous-cutanée
D - Acyclovir (Zovirax®) par voie intraveineuses
E - Méfloquine (Lariam®) par voie orale
Bonne(s) réponse(s) : B

La pentamidine (lomidine) est également efficace par voie veineuse ou en aérosol.

Dès le lendemain de la prescription, le patient présente des signes d'intolérance au traitement. Retenez la
proposition thérapeutique de remplacement :
A - Sulfate de quinine (Quinoforme®) par voie intraveineuse
B - Ofloxacine (Oflocet®) par voie orale
C - Pentamidine (Lomidine®) par voie intraveineuse
D - Dapsone (Disulone®) par voie intraveineuse
E - Erythromycine (Erythrocine®) par voie intraveineuse
Bonne(s) réponse(s) : C

Sans commentaire.

En présence d'une évolution satisfaisante, la durée de traitement habituellement préconisée est de l'ordre de :
A - 7 jours
B - 15 jours
C - 21 jours
D - 30 jours
E - Entre 2 et 3 mois
Bonne(s) réponse(s) : C

Des rechutes sont fréquentes en l'absence de traitement d'entretien (pentamidine en aérosol).

Une femme de 28 ans consulte pour des accès fébriles survenant de façon spontanée accompagnés de céphalées avec
nausées, cédant à la prise de Nivaquine. L'interrogatoire révèle que ces épisodes sont apparus après un séjour de 1 mois au
Cameroun effectué deux ans auparavant. Durant ce séjour, la personne avait présenté un épisode fébrile étiqueté "accès
palustre", malgré une prise régulière de Nivaquine® à titre prophylactique. Depuis son retour, elle à présenté trois accès
identiques dont un deux jours après son premier accouchement. Dès les prodromes, elle prend immédiatement 500 mg de
Nivaquine pendant 5 jours et devient apyrétique en moins de 24 heures. Malgré l'efficacité de cette auto-médication, la
répétition des accès l'inquiète. L'examen clinique est strictement normal ainsi que l'hémogramme.

Ce tableau clinique vous évoque prioritairement :


A - Un paludisme viscéral évolutif
B - Un accès de réviviscence à plasmodium falciparum
C - Un accès de réviviscence à plasmodium ovale
D - Un paludisme à plasmodium falciparum résistant à la Nivaquine
E - Un accès palustre pernicieux
Bonne(s) réponse(s) : C

Un accès palustre survenant plus d'un an après le séjour en zone d'endémie ne peut être du à P. falciparum, ce qui élimine A
B D E. La longévité de P. falciparum est en moyenne de 2 mois et atteint exceptionnellement un an ; celle de P. vivax et P.
ovale est de 4 à 5 ans et celle de P. malariae atteint 20 ans.

823
Exclusivement sur DOC - DZ : www.doc-dz.com NADJI 85
RESIDANAT EN POCHE TOME II
Cas Clinique en QCM

La malade étant apyrétique au moment de la consultation, quel est l'examen le plus approprié pour confirmer le
diagnostic ?
A - Frottis sanguin
B - Goutte épaisse
C - Sérologie palustre
D - Echographie de la rate
E - Electrophorèse de l'hémoglobine
Bonne(s) réponse(s) : C

Sans commentaire.

Selon vous, la répétition des accès signifie :


A - Que la malade a en fait mal pris sa chimioprophylaxie
B - Que la souche plasmodiale est résistante à la Nivaquine
C - Qu'il existe dans le sang des gamétocytes de plasmodium responsables des rechutes
D - Qu'il existe des formes plasmodiales intra-viscérales sur lesquelles la Nivaquine est sans effet
E - Que la malade a été obligatoirement réinfestée
Bonne(s) réponse(s) : D

La prophylaxie n'empêche pas l'impaludation. Elle est inactive sur les formes intra-hépatocytaires à l'origine des accès de
revinscence.

Le fait d'avoir de tels accès fébriles :


A - Contre-indique formellement une deuxième grossesse
B - Impose une contraception pendant au moins 4 ans
C - Impose la prise quotidienne de Nivaquine pendant 2 ans
D - Impose un traitement radical par le Lariam
E - Impose la prise de Nivaquine dès le début d'un accès
Bonne(s) réponse(s) : (B) E

A : pas de façon définitive


B : se discute compte-tenu du risque materno-foetal encouru
D : la méfloquine n'est pas active sur les formes intra-hépatiques

Si la patiente ne se traite pas dès le début de l'accès, comment évoluera la courbe thermique ?
A - Sous forme de fièvre tierce
B - Sous forme de fièvre en plateau
C - Sous forme de fièvre désarticulée
D - Sous forme d'un pic thermique durant 24 heures
E - De façon imprévisible
Bonne(s) réponse(s) : A

S'il s'agit de P. ovale.

La malade vous demande pendant combien de temps de tels accès peuvent survenir, si elle ne retourne pas en
zone d'endémie :
A - Environ 3 mois
B - Environ 2 ans
C - Environ 20 ans
D - Toute la vie
E - Impossible de répondre sans le diagnostic d'espèce plasmodiale
Bonne(s) réponse(s) : E

Voir question [251].

824
Exclusivement sur DOC - DZ : www.doc-dz.com NADJI 85
RESIDANAT EN POCHE TOME II
Cas Clinique en QCM
Un de vos patients âgé de 45 ans, tabagique, a séjourné de longues années en Afrique Noire et n'y est pas retourné depuis 8
ans. Il vous appelle pour une fièvre élevée à 40°C, évoluant depuis 8 jours, et un point de côté de la base pulmonaire droite,
bloquant l'inspiration. L'examen clinique montre une discrète hypoventilation de la base pulmonaire droite, et retrouve une
hépatomégalie franche, douloureuse à la palpation et à l'ébranlement.

Quel diagnostic évoquez-vous ?


A - Pneumonie atypique
B - Pleurésie sérofibrineuse
C - Pneumonie franche lobaire aiguë
D - Abcès amibien du foie
E - Hépatite virale aiguë
Bonne(s) réponse(s) : D

L'amibiase hépatique survient souvent sans manifestation digestive et peut se déclarer plusieurs années après le séjour en
zones d'endémie. L'hépatomégalie est isolée (sans splénomégalie, sans ictère).

Votre diagnostic pourrait être étayé par le résultat de :


A - Radiographie de l'abdomen sans préparation
B - Echographie hépato-vésiculaire
C - Numération formule sanguine
D - Recherche d'urobilirubinurie
E - Dosage de la bilirubinémie
Bonne(s) réponse(s) : A B C

A - Surélévation de l'hémicoupole diaphragmatique droite avec comblement du cul de sac pleural.


B - Mise en évidence d'une collection liquidienne (la phase présuppurative est exceptionnelle).
C - Hyperleucocytose à polynucléaires neutrophiles.

Quel traitement proposez-vous ?


A - Dihydro-Emétine intraveineuse
B - Métronidazole per os
C - Mezlocilline intraveineux
D - Anti-inflammatoire
E - Ceftriaxone intraveineuse
Bonne(s) réponse(s) : B

Le Flagyl est le traitement de choix (1,5 à 2 g/24 H pendant 10 jours per os). La voie IV est recommandée dans les formes
avec nausées et vomissements. La dihydro-émétine s'emploie en cas d'échec du Flagyl, per os, par voie I.M. ou S.C.

Quel est l'examen biologique qui apportera la certitude étiologique ?


A - Sérologie spécifique
B - Hémocultures
C - Coprocultures
D - Examen parasitologique des selles
E - Intradermo-réaction
Bonne(s) réponse(s) : A

La technique d'immunofluorescence indirecte et celle d'hémagglutination passive sont très sensibles et très spécifiques. Elles
sont parfois négatives au début de la maladie.

Un homme de 60 ans, éthylique, pesant 60 kgs, vient consulter pour une altération marquée de l'état général, associée à une
fièvre évoluant depuis une quinzaine de jours La radiographie pulmonaire aussitôt pratiquée, met en évidence une opacité de
type alvéolaire, excavée bilatérale. L'expectoration est riche en bacilles acido-alcoolo-résistants.

Parmi les traitements suivants, un ou plusieurs schémas thérapeutiques peuvent être proposés, le(s)quel(s) ?
A - Rifampicine 600 mg, I.N.H. 300 mg, Ethambutol 1 200 mg,Pyrazinamide 1 800 mg
B - Rifampicine 300 mg, I.N.H. 600 mg, Ethambutol 2 000 mg, Pyrazinamide 500 mg
C - Rifampicine 300 mg, I.N.H. 600 mg, Ethambutol 2 000 mg, Pyrazinamide 2 500 mg
D - Rifampicine 600 mg, I.N.H. 300 mg, Ethambutol 1 200 mg
E - Rifampicine 600 mg, Streptomycine 1 gramme, Ethambutol 1 200 mg
Bonne(s) réponse(s) : A D

INH : 5 mg/kg
Rifampicine : 10 mg/kg
Ethambutol : 20 mg/kg
Pyrazinamide : 30 mg/kg

825
Exclusivement sur DOC - DZ : www.doc-dz.com NADJI 85
RESIDANAT EN POCHE TOME II
Cas Clinique en QCM

Le patient est hospitalisé. Progressivement s'installe une torpeur sans signes neurologiques de localisation.
Quel(s) examen(s) devez-vous pratiquer ?
A - Electroencéphalogramme
B - Ponction lombaire
C - Gaz du sang
D - Ammoniémie
E - Scanner cérébral
Bonne(s) réponse(s) : A B C D E

A et D - Ils sont à la recherche d'arguments pour une encéphalopathie hépatique.


B - Une méningite associée est toujours possible.
C - Pour argumenter une décompensation ventilatoire.
E - Penser à l'hématome sous-dural chez ce patient éthylique.

La surveillance du bilan biologique hépatique après 15 jours de traitement permet de noter l'apparition d'une
cytolyse (TGO : 260 - TGP : 280 - Nle < 30). Quelle(s) attitude(s) adoptez-vous ?
A - Surveillance du bilan hépatique
B - Arrêt de la Rifampicine
C - Arrêt de l'I.N.H.
D - Arrêt de la Rifampicine et de l'I.N.H.
E - Réduction de posologie de l'I.N.H.
Bonne(s) réponse(s) : A E

Un test d'acétylation de l'isoniazide aurait pu déceler un "acétyleur lent" pour lequel 3 mg/kg d'INH sont suffisants.

A la troisième semaine il se plaint de douleurs articulaires au niveau des petites articulations. Le taux d'acide
urique est à 90 mg/l (normale < 70 mg/l). Quel médicament peut être responsable ?
A - Rifampicine
B - Isoniazide
C - Pyrazinamide
D - Ethambutol
E - Streptomycine
Bonne(s) réponse(s) : C

Les effets secondaires du pyrazinamide sont : intolérance digestive ; toxicité hépatique ; hyperuricémie ; l'éthambutol peut
également être à l'origine d'une hyperuricémie ; l'hyperuricémie due à ces médicaments est le plus souvent asymptomatique.

Un jeune Antillais de 12 ans, arrivé récemment en métropole, non vacciné par le B.C.G., présente : fièvre à 38°, asthénie,
anorexie, amaigrissement, sueurs nocturnes et toux.

Quel diagnostic évoquez-vous ?


A - Mononucléose infectieuse
B - Sarcoïdose
C - Hémopathie
D - Primo-infection tuberculeuse
E - Fièvre de Malte
Bonne(s) réponse(s) : D

C'est le premier diagnostic à évoquer de principe compte-tenu du tableau. La P.I.T. n'est symptomatique que dans 5 à 10 %
des cas.

Quel(s) examen(s) allez-vous demander en premier pour conforter votre diagnostic ?


A - Prélèvement de gorge
B - Numération formule
C - Radio thoracique
D - Vitesse de sédimentation
E - Tests tuberculiniques cutanés
Bonne(s) réponse(s) : C E

C - A la recherche d'adénopathies médiastinales, du chancre d'inoculation, de troubles de la ventilation, d'un épanchement


pleural.
E - Si ils sont négatifs, les tests tuberculiniques doivent être répétés. L'allergie n'apparaît qu'un à trois mois après le contage.

826
Exclusivement sur DOC - DZ : www.doc-dz.com NADJI 85
RESIDANAT EN POCHE TOME II
Cas Clinique en QCM

Les aspects radiologiques thoraciques compatibles avec ce diagnostic comportent :


A - Epanchement pleural
B - Adénopathies hilaires bilatérales
C - Adénopathies hilaires unilatérales
D - Infiltrat d'un lobe supérieur
E - Adénopathies hilaires accompagnant un chancre homolatéral (complexe primaire)
Bonne(s) réponse(s) : A B C E

D - Témoigne d'une tuberculose pulmonaire commune (tuberculose pulmonaire de réactivation).

Si la RP est normale vous recommandez :


A - Surveillance
B - INH seul pendant 6 mois
C - INH seul pendant 12 mois
D - Traitement double pendant 6 mois (INH + Rifampicine)
E - Traitement quadruple pendant 2 mois puis double pendant 4 mois
Bonne(s) réponse(s) : E

Le traitement de la P.I.T. patente est identique à celui de la tuberculose pulmonaire commune.

Vous examinez un garçon de 8 ans, chez qui le diagnostic de rhumatisme articulaire aigu est suspecté. Les circonstances
d'apparition des premiers symptômes 15 jours après une angine sont en effet évocatrices. Vous devez confirmer ce
diagnostic, surveiller la survenue possible de complications, et choisir les indications thérapeutiques.

Parmi les signes suivants, quel est celui ou quels sont ceux qui est (sont) observés au cours d'une crise de
rhumatisme articulaire aigu ?
A - Fièvre
B - Arthralgies
C - Splénomégalie
D - Stomatite
E - Exanthème
Bonne(s) réponse(s) : A B E

A B - polyarthrite aiguë fébrile des grosses articulations.


E - l'érythème marginé et les nodosités de Meynet sont rares.

Parmi les signes biologiques suivants, quel est celui ou quels sont ceux qui confirmera(ront) le diagnostic de
rhumatisme articulaire aigu ?
A - Elévation du taux des ASLO
B - Hémoculture positive à streptocoque du groupe A
C - Accélération de la V.S.
D - Augmentation de la C.R.P.
E - Hypogammaglobulinémie
Bonne(s) réponse(s) : A C D

Le syndrome inflammatoire est majeur et constant (augmentation de : vitesse de sédimentation, alpha 2 et gamma globulines,
fibrine, haptoglobine, (réactive protéine). Les ASLO s'élèvent de façon importante au cours des dosages répétés. Le
streptocoque A n'est habituellement pas retrouvé.

Parmi les signes cliniques et électriques suivants, quel est celui ou quels sont ceux qui traduit(sent) l'existence
d'une péricardite rhumatismale ?
A - Assourdissement des bruits du coeur
B - Bruit de frottement au niveau de l'aire précordiale
C - Souffle perçu au 2ème espace intercostal gauche
D - Allongement de l'espace P.R à L'E.C.G.
E - Sous dénivellation de S.T. et inversion de T à L'E.C.G.
Bonne(s) réponse(s) : A B E

A - Traduit un épanchement important.


D - Traduit une myocardite superficielle.

827
Exclusivement sur DOC - DZ : www.doc-dz.com NADJI 85
RESIDANAT EN POCHE TOME II
Cas Clinique en QCM

Le traitement le plus efficace de la crise de rhumatisme articulaire aigu est la corticothérapie par la prednisone.
Pour cet enfant de cet âge, quelle est la posologie utilisée pour le traitement d'attaque ?
A - 0,5 mg.kg/24 h
B - 1 mg/kg/24 h
C - 2 mg/kg/24 h
D - 3 mg/kg/24 h
E - 4 mg/kg/24 h
Bonne(s) réponse(s) : C

Le traitement de la poussée aiguë associe :


- repos au lit pendant un mois, en cas de cardite rhumatismale le repos doit se prolonger 4 à 6 mois
- prednisone 2mg/kg/jour jusqu'à normalisation de la VS puis diminution progressive
- pénicillinothérapie d'attaque (2 millions d'unités/jour pendant 10 jours) puis d'entretien (extencilline).

Parmi les antibiotiques suivants, quel est celui ou quels sont ceux qui pourra (pourront) être utilisés(s) pour le
traitement prophylactique ultérieur ?
A - Pénicilline V per os
B - Pénicilline retard intramusculaire
C - Tétracycline
D - Macrolides
E - Chloramphénicol
Bonne(s) réponse(s) : A B D

L'extencilline IM est habituellement utilisée (1 200 000 unités/2 semaines ; 600 000 unités avant 6 ans). En cas d'allergie, on
peut utiliser un macrolide per os en prise quotidienne.

Un homme de 24 ans consulte pour des douleurs pharyngées bilatérales, avec dysphagie et otalgies apparues depuis 24 à 48
heures dans un contexte d'anorexie et d'asthénie marquées. A l'examen, on note une muqueuse pharyngée rouge dans son
ensemble avec, au niveau amygdalien, l'existence dans les cryptes d'un enduit jaunâtre bilatéral s'étendant à l'amygdale sans
la dépasser, facilement détachable au porte-coton. Il existe par ailleurs plusieurs adénopathies jugulo-carotidiennes bilatérales
de 3 à 4 cm de diamètre, régulières, fermes, lisses, légèrement douloureuses, ainsi que plusieurs adénopathies spinales et
occipitales, bilatérales, plus petites mais de mêmes caractères à la palpation. On note par ailleurs une rate palpable et
quelques adénopathies axillaires et inguinales. Le reste de l'examen clinique est sans particularités hormis une température
modérément élevée à 38°.

Parmi les arguments cliniques suivants, quels sont celui ou ceux qui oriente(nt) plus particulièrement vers une
mononucléose infectieuse ?
A - Asthénie marquée
B - Otalgies
C - Température
D - Existence de polyadénopathies cervicales
E - Existence d'une splénomégalie
Bonne(s) réponse(s) : A D E

La forme typique de mononucléose infectieuse associe : asthénie profonde, fièvre, adénopathies, angine. On peut également
noter : une splénonégalie (50 %), un exanthème fugace.

Parmi les examens complémentaires suivants, lequel (ou lesquels) apporte(nt) à ce stade un argument positif
au diagnostic ?
A - NFS
B - VS
C - MNI test
D - Dye test
E - Prélèvement de gorge
Bonne(s) réponse(s) : A C

A - Syndrome mononucléosique
C - Met en évidence des IgM sériques capables d'agglutiner des hématies hétérologues
E - Dans les pays occidentaux 20 % des adultes sont excréteurs de virus EBV.

828
Exclusivement sur DOC - DZ : www.doc-dz.com NADJI 85
RESIDANAT EN POCHE TOME II
Cas Clinique en QCM

La diphtérie est responsable d'une angine à fausses membranes dont les caractères sont différents de ceux de
la mononucléose infectieuse. Parmi les caractères suivants, quel est (ou quels sont) celui (ceux) qui
concerne(nt) l'angine diphtérique ?
A - Fausses membranes grisâtres
B - Confluentes
C - Extensives
D - Non adhérentes
E - Non hémorragiques
Bonne(s) réponse(s) : B C

Les fausses membranes de l'angine diphtérique peuvent être blanches ou grisâtres ; elles sont très envahissantes et
fortement adhérentes. Les fausses membranes de l'angine de la M.N.I. respectent la luette.

L'une des thérapeutiques suivantes est à proscrire si vous soupçonnez une mononucléose infectieuse. Laquelle
?
A - Aérosols antiseptiques et antalgiques
B - Corticothérapie par voie générale
C - Pénicillinothérapie par voie générale
D - Traitement par ampicilline par voie générale
E - Traitement par rovamycine per os
Bonne(s) réponse(s) : D

Un traitement par ampicilline au cours d'une M.N.I. provoque un rash dans 90 % des cas.

Un sujet de 50 ans sans antécédent particulier, non fumeur, présente à la suite d'un bain accidentel en eau froide au mois de
Décembre un point de côté brutal avec frissons et hyperthermie à 40° .
Vous êtes appelé une demi journée plus tard : les frissons ont disparu, la fièvre est à 40°, le point de côté persiste, et la toux
ramène une expectoration rouillée. La percussion thoracique est normale, les vibrations vocales ne sont pas modifiées;
L'auscultation met en évidence un foyer de râles crépitants à mi-hauteur de l'hémothorax gauche. La radiographie du thorax
ne montre rien de particulier. L'hémogramme est le suivant : 4 300 000 GR/mm3 15 000 GB/mm3 PN 78 % Créatininémie =
180 micromoles/l. Vous faites des hémocultures, et débutez le traitement.

Parmi les antibiotiques suivants, deux sont plus particulièrement indiqués pour traiter ce sujet. Lesquels ?
A - Pénicilline G®
B - Céphalosporine®
C - Gentamicine®
D - Tétracyclines®
E - Macrolides®
Bonne(s) réponse(s) : A E

La prépondérance du pneumocoque fait prescrire souvent la pénicilline G en première intention. Les macrolides ont l'avantage
d'être actif sur les légionelles et les mycoplasmes mais dans certaines régions 20 % des pneumocoques sont résistants aux
macrolides. Le tableau est ici très évocateur de pneumopathie à pneumocoque : hyperthermie brutale, douleur thoracique,
frisson solennel, crachats rouillés.

Si vous utilisez la pénicilline, quelle dose prescrivez-vous ?


A - 50 000 u/kg/j
B - 100 000 u/kg/j
C - 150 000 u/kg/j
D - 200 000 u/kg/j
E - 400 000 u/kg/j
Bonne(s) réponse(s) : B

Pendant 10 à 15 jours.

Si vous utilisiez la gentamicine, quel(s) accident(s) redouteriez-vous ?


A - Agranulocytose
B - Surdité
C - Atteinte hépatique
D - Accident rénal
E - Troubles de l'équilibre
Bonne(s) réponse(s) : B D E

Sans commentaire.

829
Exclusivement sur DOC - DZ : www.doc-dz.com NADJI 85
RESIDANAT EN POCHE TOME II
Cas Clinique en QCM

Si vous utilisiez une céphalosporine, quelle(s) est (sont) l'(les) affirmation(s) vraie(s) ?
A - Il est inutile de diminuer les doses à cause de la créatininémie
B - Elles donnent parfois un test de Coombs positif
C - Elles sont hépato-toxiques
D - Elles peuvent entraîner des réactions allergiques
E - Elles sont susceptibles de maintenir l'hyperthermie
Bonne(s) réponse(s) : B D E

A - une insuffisance rénale majeure doit conduire à réduire la posologie des céphalosporines de 3ème génération.

Si vous utilisez la doxycycline, quelle dose prescrivez-vous ?


A - 200 mg 1 fois par jour
B - 200 mg 2 fois par jour
C - 200 mg 3 fois par jour
D - 200 mg 4 fois par jour
E - 2 g par jour
Bonne(s) réponse(s) : A

La doxycycline doit être ingérée au cours d'un repas sans laitage et à distance des pansements gastriques.

Si vous utilisez un macrolide, vous savez que certains d'entre eux peuvent entraîner un surdosage de certains
médicaments qui leur sont associés. Lequel (lesquels) parmi les suivants ?
A - Carbamazépine
B - Antivitamine K
C - Théophylline
D - Béta-bloquants
E - Dérivés de l'ergot de seigle
Bonne(s) réponse(s) : A C E

Sans commentaire.

Un patient originaire d'Afrique Centrale, vivant en France depuis 2 ans, présente une altération de l'état général avec
amaigrissement de 3 kg, fièvre à 38,5° C, sueurs nocturnes et asthénies. Il existe une toux et une expectoration abondante.
La sérologie VIH est connue positive depuis son arrivée en France. L'intradermo-réaction à la tuberculine est positive à 10 mm
à la 72ème heure. Une tuberculose est évoquée.

Plaid(ent) en faveur d'une tuberculose :


A - La notion de transplantation
B - La sérologie HIV positive
C - L'IDR positive
D - L'amaigrissement récent
E - La fièvre
Bonne(s) réponse(s) : A B C D E

Sans commentaire.

La confirmation diagnostique d'une maladie tuberculeuse repose sur :


A - La découverte de BAAR à l'examen direct des crachats
B - La mise en évidence de mycobactéries en culture
C - L'identification de Mycobacterium tuberculosis sur la culture
D - La découverte de BAAR sur une pièce histologique
E - La positivité de l'intradermo-réaction à la tuberculine
Bonne(s) réponse(s) : C

A B D - Ne permettent pas de différencier mycobactérium tuberculosis d'une mycobactérie atypique.

830
Exclusivement sur DOC - DZ : www.doc-dz.com NADJI 85
RESIDANAT EN POCHE TOME II
Cas Clinique en QCM

Le bilan pré-thérapeutique de cette tuberculose doit comprendre si l'on engage une quadrithérapie :
A - Dosage des transaminases
B - Bilirubinémie
C - Numération des globules rouges
D - Créatininémie
E - Uricémie
Bonne(s) réponse(s) : A D E

A - Pour surveiller la toxicité hépatique de l'isoniazide et du pyrazinamide.


D - L'éthambutol est rarement responsable d'une insuffisance rénale.
E - Hyperuricémie possible due au pyrazinamide ou à l'éthambutol.

La phase initiale du traitement (avant diminution du nombre d'antibiotiques) durera :


A - Jusqu'à l'apyrexie
B - 1 mois
C - 2 mois
D - 6 mois
E - 9 mois
Bonne(s) réponse(s) : C

Le pyrazinamide et l'éthambutol sont arrêtés à la fin du 2ème mois. L'isoniazide et la rifampicine sont poursuivis jusqu'à la fin
du 6ème mois.

Après 1 mois de traitement il existe toujours des BAAR à l'examen direct. Vous pouvez conclure :
A - Qu'il s'agit d'un échec du traitement
B - Qu'il s'agit d'une rechute
C - Que le malade n'a surement pas pris son traitement
D - Qu'il s'agit d'une constatation banale après seulement 1 mois de traitement
E - Que le BK est probablement résistant aux antituberculeux
Bonne(s) réponse(s) : D

Il s'agit vraisemblablement de BK tués par le traitement (intérêt de la culture).

Quelle est l'influence probable de la sérologie VIH (+) chez ce sujet ?


A - Elle a probablement favorisé l'éclosion de la tuberculose
B - Elle va interdire d'utiliser les antituberculeux usuels
C - Elle va entraîner une négativation très lente de la bactériologie
D - Elle va favoriser les rechutes et invite à prolonger le traitement
E - Elle n'a aucune influence
Bonne(s) réponse(s) : A D

M. tuberculosis est maintenant souvent considéré comme un germe opportuniste.

Un enfant de 12 ans se plaint brusquement de céphalées et de fièvre (39° 5). Le médecin constate une raideur méningée et
quelques éléments purpuriques sur les jambes. Il n'y a aucun signe de choc. La pression artérielle est normale. Le liquide
céphalo-rachidien est purulent avec 900 polynucléaires/mm3, protéinorachie 1 g/l, glycorachie 2 mmol/l (glycémie : 4,7
mmol/l). Il y a dans la famille 2 autres enfants de 8 et 10 ans fréquentant la même école qui n'a pas d'internat.

Parmi les germes suivants, quel est celui qui est le plus fréquemment en cause à cet âge et en l'absence de
traumatisme ?
A - Méningocoque
B - Staphylocoque
C - Haemophilus
D - Pneumocoque
E - Listeria
Bonne(s) réponse(s) : A

Sans commentaire.

831
Exclusivement sur DOC - DZ : www.doc-dz.com NADJI 85
RESIDANAT EN POCHE TOME II
Cas Clinique en QCM

Le germe n'étant pas identifié, vous désirez commencer le traitement. Quel sera votre choix ?
A - Céfazoline (Cefacidal®) 100 mg/kg/24 h l.V.
B - Amoxicilline (Clamoxyl®) 200 mg/kg/24 h l.V.
C - Gentamicine (Gentalline®) 3 mg/kg/24 h l.V.
D - Peni V (Oracilline®) 2 millions d'U/24 h I.V.
E - Erythromycine 100 mg/kg/24 h per os
Bonne(s) réponse(s) : B

On peut y associer pendant quelques jours un aminoside pour augmenter la vitesse de bactéricidie.

A l'examen direct du L.C.R. si l'on vous répond présence de cocci à Gram négatif, le(s)quel(s) pourrai(en)t être
en cause parmi les germes suivants ?
A - Méningocoque
B - Listeria
C - Pneumocoque
D - Haemophilus
E - Escherichia coli
Bonne(s) réponse(s) : A

B - Bacille gram positif


C - Coque gram positif
D - Bacille gram négatif
E : Bacille gram négatif, responsable de méningites néo-natales.

Si c'est une méningite à méningocoque B, quelle(s) est(sont) la(les) mesure(s) d'hygiène à prendre
obligatoirement ?
A - Désinfecter l'habitation
B - Faire une déclaration de maladie contagieuse
C - Fermer l'école
D - Faire un prélèvement de gorge à tous les élèves de la même école
E - Vaccination anti-méningococcique aux élève de sa classe
Bonne(s) réponse(s) : B

Arrêté du 3 mai 1989


Pour la méningite à méningocoque : éviction scolaire du patient ; pas d'éviction des sujets contacts ; prophylaxie
médicamenteuse de l'entourage ; vaccination des sujets contact en cas de méningite du groupe A ou C.

Si c'est une méningite à méningocoque, vous choisissez de faire une chimioprophylaxie de l'entourage par la
spiramycine (Rovamycine®) : citer parmi les propositions suivantes, celle(s) qui est(sont) exacte(s) :
A - Elle s'applique aux frères et soeurs vivant sous le même toit
B - Elle n'est faite que si le prélèvement de gorge est positif pour le méningocoque chez les frères et soeurs
C - Cette mesure est à étendre à toute l'école
D - Cet antibiotique a été choisi parce qu'il passe bien dans le liquide rachidien
E - La dose usuelle est de 50 mg/kg/jour durant 5 jours
Bonne(s) réponse(s) : A E

C - Non, mais certainement les camarades de classe, les camarades de jeux, les voisins de dortoir, éventuellement toute la
classe.
D - Cet antibiotique a été choisi en raison de sa mauvaise diffusion dans le LCR (pour ne pas masquer une méningite en
évolution) et de sa bonne concentration salivaire (pour éradiquer les germes du pharynx des porteurs asymptomatiques).

Un malade ayant subi une intervention chirurgicale sur les voies urinaires présente dans les suites opératoires un tableau
septicémique.
Trois hémocultures sont positives en 48 heures à Enterobacter cloacae. Un traitement antibiotique est adapté selon la
sensibilité du germe à l'antibiogramme et associe cefotaxime (Claforan®) et amikacine (Amiklin®).

Enterobacter cloacae est :


A - Une corynébactérie commensale
B - Un cocci à Gram positif
C - Un bacille à Gram négatif appartenant à la famille des entérobactéries
D - Un bacille à Gram négatif appartenant à la famille des Pseudomonadaceae
E - Une mycobactérie
Bonne(s) réponse(s) : C

C'est une entérobactérie opportuniste.

832
Exclusivement sur DOC - DZ : www.doc-dz.com NADJI 85
RESIDANAT EN POCHE TOME II
Cas Clinique en QCM

Parmi les bactéries suivantes, lesquelles sont responsables d'infections du tractus urinaire après manoeuvre
instrumentale ?
A - Serratia marcescens
B - Streptoccus pneumoniae
C - Pseudomonas aeruginosa
D - Acinetobacter
E - Brucella
Bonne(s) réponse(s) : A C

A - 2 % des infections urinaires nosocomiales.


C - 11 %.
D - Souvent isolés dans les urines des patients sondés mais rarement responsables de véritables infections.

Le cefotaxime (Claforan®) est :


A - Une ampicilline
B - Une uréido-pénicilline
C - Une céphaslosporine de 1ère génération
D - Une céphalosporine de 3ème génération
E - Un macrolide
Bonne(s) réponse(s) : D

Son métabolite, le désacétylcéfotaxime est également actif.

L'amikacine (Amiklin®) est :


A - Une béta-lactamine
B - Une aminoside
C - Une cycline
D - Un macrolide
E - Une quinolone
Bonne(s) réponse(s) : B

Sans commentaire.

La concentration minimale inhibitrice (CMI) :


A - Est la plus forte concentration d'antibiotique capable d'inhiber toute culture visible en milieu liquide
B - Est la plus faible concentration d'antibiotique capable d'inhiber toute culture visible en milieu liquide
C - Est la concentration d'antibiotique qui laisse subsister moins de 0,01 % de survivants
D - Peut être déterminée par la méthode des disques en milieu gélosé (antibiogramme)
E - Est la plus faible concentration d'antibiotique assurant une bonne tolérance
Bonne(s) réponse(s) : B D

C - La CMB est la plus faible concentration d'antibiotique détruisant après 18 h de contact à 37°C, 99,99% d'une population
bactérienne.
D - Il existe une relation entre le diamètre d'inhibition de la croissance bactérienne et la valeur de la CMI.

Un pouvoir bactéricide des antibiotiques et associations donne les résultats suivants : amikacine : 0,01 % de
survivants, céfotaxime : 1 % de survivants, céfotaxime + amikacine : 0,01 % de survivants : Vous pouvez
conclure :
A - Que l'amikacine utilisée seule est bactéricide
B - Que le céfotaxime seul est bactéricide
C - Que l'association est bactéricide
D - Que l'association est bactériostatique
E - Que le céfotaxime est totalement inefficace
Bonne(s) réponse(s) : A C

E - L'efficacité antibiotique dépend également du site de l'infection et de la distribution de l'antibiotique.

833
Exclusivement sur DOC - DZ : www.doc-dz.com NADJI 85
RESIDANAT EN POCHE TOME II
Cas Clinique en QCM
Monsieur A. D. jeune Syrien vivant en France depuis un an, est hospitalisé pour fièvre s'étant élevée progressivement jusqu'à
40°C.
En outre, il souffre de céphalées intenses, de douleurs dans l'hypochondre droit, de nausées. Il est constipé. Les symptômes
sont apparus 15 jours après le retour du patient de vacances passées en Syrie. A l'examen, on note une angine
érythémateuse avec ulcérations des piliers antérieurs du voile du palais, l'abdomen est souple, il n'y a pas
d'hépatosplénomégalie, ni d'adénopathies.
La numération formule sanguine montre 4 600 leucocytes par mm3 avec 76 % de polynucléaires.
Six hémocultures sont pratiquées sur 48 heures d'hospitalisation. Les 6 flacons donnent une culture positive en 24 heures à
48 heures.
L'identification montre qu'il s'agit de Salmonella TYPHI.
Le malade est traité par du Chloramphénicol. La fièvre disparait en 4 jours, les signes cliniques rétrocèdent rapidement.
Un séro-diagnostic de Widal-Félix est pratiqué au douzième jour de la maladie et donne les résultats suivants :
TO = 1/400, TH = 1/800, AH = 0, BO = 1/200, BH = 0.
Le malade sort de l'hôpital quand les coprocultures de contrôle se sont révélées négatives.

Salmonella typhi :
A - S'appelle encore "bacille d'Eberth"
B - Est gram positif
C - Fait partie des entérobactéries
D - Possède toujours des antigènes somatiques de type O
E - Possède souvent un antigène Vi
Bonne(s) réponse(s) :A C D E

B - C'est un BGN.
D - L'antigène O correspond au lipopolysaccharide.

Outre le chloramphénicol qui reste un traitement classique, on peut utiliser dans le traitement de la fièvre
typhoïde :
A - La pénicilline G
B - La ceftriaxone
C - L'érythromycine
D - La colimycine
E - Le thiamphénicol
Bonne(s) réponse(s) : B E

On peut également prescrire : cotrimoxazole, ampicilline, fluoroquinolones.


C - Les macrolides sont inefficaces sur les entérobactéries.

Le séro-diagnostic de Widal-Félix est une réaction :


A - De fixation du complément
B - D'agglutination conditionnée (indirecte)
C - De précipitation
D - D'immunofluorescence
E - D'agglutination directe
Bonne(s) réponse(s) : E

Le principe de ce séro-diagnostic est basé sur la capacité des anticorps sériques du patient d'agglutiner des bactéries tuées
appartenant aux différents sérotypes responsables des fièvres typhoïdes.

Le séro-diagnostic de Widal-Félix distingue les anticorps d'une part contre les antigènes O, d'autre part contre
les antigènes H :
A - Les anticorps anti-Osont de nature gamma-globulinique
B - Les anticorps anti-H sont de nature alpha-globulinique
C - Les antigènes H sont de nature protéique flagellaire
D - Les antigènes O sont liés au corps bactérien
E - Les antigènes H constituent l'endotoxine
Bonne(s) réponse(s) : A C D

D - Ils correspondent au lipopolysaccharide (endotoxine).

834
Exclusivement sur DOC - DZ : www.doc-dz.com NADJI 85
RESIDANAT EN POCHE TOME II
Cas Clinique en QCM

Sur le plan physiopathologique et épidémiologique :


A - Il n'existe pas de réservoir animal de S. typhi
B - La contamination est digestive
C - Les bactéries traversent la muqueuse intestinale
D - Les bactéries libèrent l'endotoxine
E - Les porteurs sains ne jouent aucun rôle dans la dissémination de la maladie
Bonne(s) réponse(s) : A B C D

A B - Le réservoir de germes est strictement humain. La contamination peut être directe, ou plus souvent indirecte par
ingestion d'eau ou d'aliments souillés.
C D - Après passage de la muqueuse intestinale, les germes gagnent les ganglions lymphatiques mésentériques où ils se
multiplient.
E - Ils en sont les principaux vecteurs.

Vous avez un patient qui est porteur de sonde urinaire depuis 3 jours du fait d'une intervention chirurgicale.

Les infections urinaires nosocomiales peuvent être le fait des germes suivants, quel est le plus fréquent ?
A - E. coli
B - Entérocoque
C - Protéus
D - Pseudomonas
E - Klebsiella pneumoniae
Bonne(s) réponse(s) : A

Les infections urinaires nasocomiales sont dues à :


- E. Coli 32 %
- Enterococcus sp. 13 %
- Pseudomonas aeruginosa 10 %
- Klebsiella sp. 10 %
- Proteus-Providencia 9 %.

Parmi les facteurs de risque infectieux chez ce malade, relevez celui qui est le plus important :
A - L'âge
B - La longueur de l'hospitalisation avant l'intervention chirurgicale
C - La colonisation bactérienne de la poche de drainage
D - La prescription d'une antibiothérapie
E - La maladie sous-jacente
Bonne(s) réponse(s) : C

Sans commentaire.

Le CDC (Center of Diseases Control) d'Atlanta attache la valeur la plus grande aux mesures suivantes pour
prévenir l'infection urinaire chez les patients porteurs d'une sonde, sauf une, laquelle ?
A - Formation du personnel
B - Ne sonder que si nécessaire
C - Se laver les mains avant de placer une sonde
D - Utiliser une sonde la plus petite possible
E - Changer fréquemment la sonde
Bonne(s) réponse(s) : D

Sans commentaire.

Parmi les affirmations suivantes, quelle(s) est (sont) celle(s) qui affirme(nt) l'infection chez le patient sondé ?
A - Leucocyturie > 10 GB/champ
B - Bactériurie 10 exposant 5/ml
C - Bactériurie 10 exposant 3/ml
D - La fièvre
E - Les signes cliniques locaux
Bonne(s) réponse(s) : B

A - Fréquent chez les patients sondés, même en l'absence d'infection.


D E - Ne sont pas spécifiques d'une infection.

835
Exclusivement sur DOC - DZ : www.doc-dz.com NADJI 85
RESIDANAT EN POCHE TOME II
Cas Clinique en QCM

Parmi les moyens de réduire l'infection urinaire chez les porteurs de sonde, quel est celui qui vous paraît le plus
important ?
A - Le nettoyage quotidien du méat urétral par la Bétadine
B - L'injection vésicale d'antibiotique ou d'antiseptique
C - L'antibioprophylaxie (un comprimé le soir)
D - La stérilisation des urines de la poche par un antiseptique
E - La connection permanente de la sonde et de la poche
Bonne(s) réponse(s) : A

Cf. commentaire question 308


C - Risque de sélection d'une bactérie résistante.

Quand et comment traitez-vous une infection urinaire sur sonde ?


A - Même si elle est asymptomatique
B - Quand il y a une bactériurie isolé qui persiste 24 h après le retrait de la sonde
C - Par une dose unique d'antibiotique
D - Par une antibiothérapie prolongée au moins 7 jours
E - Quand la sonde urétrale peut être retirée
Bonne(s) réponse(s) : D

L'indication thérapeutique est fonction de la symptomatologie et de l'atteinte du haut appareil urinaire.


E - La sonde doit alors être retirée si possible.

Ahmed, âgé de 12 ans, est rentré depuis deux jours d'Afrique du Nord où il était en vacances avec ses parents. A la fin de son
séjour, il a eu un peu de diarrhée. Depuis son retour, il est fébrile (39°C), prostré et se plaint de la tête. L'examen montre
l'absence de raideur méningée. L'abdomen est ballonné, un peu gargouillant à la palpation de la fosse iliaque droite. Il n'y a
pas de splénomégalie. L'inspection des téguments révèle la présence de quelques macules rosées de la paroi abdominale et
de la base du thorax.
L'enfant est hospitalisé.

Sachant que vous intervenez au 5ème jour de la maladie, quel est parmi les examens suivants, celui qui
présente le plus grand intérêt ?
A - Vitesse de sédimentation
B - Hémogramme
C - Hémoculture
D - Coproculture
E - Sérodiagnostic de Widal et de Wright
Bonne(s) réponse(s) : C

L'hémoculture est un élément diagnostic de certitude, positive au cours du premier septennaire de la maladie, avant
l'antibiothérapie. La coproculture n'est positive que dans 30 % des cas.
E - Positif en 0 au 8ème jour.

Quelques heures après l'admission de l'enfant, vous prenez connaissance des examens biologiques. La V.S.
est de 15 mm, l'hémogramme montre une leucopénie à 3 500 globules blancs par mm3 dont 40 % de
polynucléaires neutrophiles.
Parmi les maladies suivantes, quelles sont celles qui comportent habituellement une telle leuco-neutropénie ?
A - Brucellose
B - Septicémie à Staphylocoques
C - Endocardite lente
D - Typhoïde
E - Hépatite virale
Bonne(s) réponse(s) : A D E

Sans commentaire.

Dans le cas présent et compte-tenu des données de l'anamnèse, de l'examen clinique et de l'hémogramme,
quel diagnostic retenez-vous ?
A - Brucellose
B - Septicémie à Staphylocoques
C - Endocardite lente
D - Typhoïde
E - Hépatite virale
Bonne(s) réponse(s) : D

Les tâches rosées lenticulaires sont un signe quasi-pathognomonique de la typhoïde.

836
Exclusivement sur DOC - DZ : www.doc-dz.com NADJI 85
RESIDANAT EN POCHE TOME II
Cas Clinique en QCM

Dans la liste suivante, quels sont les antibiotiques habituellement efficaces dans cette maladie ?
A - Chloramphénicol
B - Fluoroquinolone
C - Erythromycine
D - Cotrimoxazole
E - Céftriaxone
Bonne(s) réponse(s) : A B D E

C - Les entérobactéries sont naturellement résistantes aux macrolides.

Dans cette liste, quel est l'antibiotique utilisé uniquement par voie veineuse ?
A - Chloramphénicol
B - Ampicilline
C - Erythromycine
D - Cotrimoxazole
E - Ceftriaxone
Bonne(s) réponse(s) : E

La voie IM est également utilisable pour cet antibiotique.

Un enfant âgé de 7 ans, sans antécédent pathologique particulier, après 24 heures de fièvre isolée à 38°C, développe
subitement des lésions cutanées sur la paroi thoracique, de type vésiculaire, non indurées, à base souple et à contenu clair.

Devant ce tableau évocateur d'une varicelle, indiquez la ou les mesure(s) à prendre :


A - Traitement par aciclovir (Zovirax®)
B - Isolement pendant 40 jours
C - Traitement antibiotique par un macrolide
D - Application d'une solution antiseptique sur les vésicules rompues
E - Injection d'immunoglobulines à l'entourage
Bonne(s) réponse(s) : D

A - Ne s'applique qu'aux patients immunodéprimés présentant une varicelle.


E - Uniquement aux immunodéprimés de l'entourage.

Parmi les sujets suivants, indiquez celui ou ceux pour qui le développement d'une varicelle peut avoir des
conséquences graves :
A - Enfant de moins de 10 ans
B - Enfant de race noire
C - Sujet traité par antiinflammatoire non stéroïdien
D - Sujet immunodéprimé
E - Femme enceinte
Bonne(s) réponse(s) : D E

A - La majorité des varicelles.


E - Les varicelles de la femme enceinte sont plus fréquemment responsables d'une pneumopathie grave. Elles exposent
également au risque de varicelle congénitale.

Un sujet, exposé au risque de faire une varicelle grave, a été en contact avec un varicelleux. Vous décidez une
prophylaxie de la varicelle. Quelle(s) méthode(s) pouvez-vous utiliser si le contage date de moins de 72 heures ?
A - Injecter des immunoglobulines polyvalentes dites standard : 5 ml
B - Injecter des immunoglobulines spécifiques anti-varicelle-zona (0,3 ml/kg en IM)
C - Entreprendre un traitement par aciclovir (Zovirax®) per os
D - Hospitaliser le sujet
E - Mettre en route un traitement antibiotique par Clamoxyl® 4 g/j
Bonne(s) réponse(s) : B C

Sans commentaire.

837
Exclusivement sur DOC - DZ : www.doc-dz.com NADJI 85
RESIDANAT EN POCHE TOME II
Cas Clinique en QCM

Les complications possibles de la varicelle sont la ou les suivante(s) :


A - Cérébellite aiguë
B - Pneumonie interstitielle avec hypoxémie sévère
C - Surinfections cutanées
D - Parotidite
E - Algies post-varicelleuses
Bonne(s) réponse(s) : A B C

A - Ataxie aiguë cérebelleuse (2 à 3 pour mille).

Devant toute éruption cutanée vésiculeuse, retenez le ou les diagnostics que l'on peut évoquer en dehors de la
varicelle :
A - Une atteinte cutanée par le virus VIH
B - Un zona
C - Une rougeole
D - Un syndrome main-pied-bouche (virus Coxsackie A 16)
E - Une rubéole
Bonne(s) réponse(s) : B D

A - Le rash cutané de la primo-infection au VIH est morbilliforme ou urticarien.

Monsieur M. , âgé de 40 ans, ingénieur agricole, part pour 4 ans au Burkina Faso, en zone pré-sahélienne pour implanter des
rizières. Il est accompagné de sa femme et de son fils Arnaud âgé de 4 ans. Leur départ aura lieu dans 6 mois. Il vous rend
visite le 05.02.1989.

S'agissant d'Arnaud, vous conseillez :


A - Le vaccin antiméningococcique A
B - Le vaccin anti-amarile
C - Le vaccin TAB
D - Le vaccin Hévac B
E - Le vaccin ROR
Bonne(s) réponse(s) :

QUESTION ANNULEE.

Madame M. T., âgée de 34 ans, n'a gardé aucun souvenir et aucune trace de ses vaccinations antérieures.
Vous prévoyez :
A - Un vaccin Typhim Vi
B - Le vaccin anti-amarile
C - Le vaccin Hevac B
D - Le BCG
E - Le vaccin anti-méningococcique A
Bonne(s) réponse(s) : A B C E

Sans commentaire.

Elle désire un autre enfant :


A - Vous lui faites faire une sérologie de la rubéole
B - Vous contrôlez son état immunitaire à l'égard de la toxoplasmose
C - Vous la vaccinez contre la toxoplasmose
D - Vous lui dites qu'étant donné le pays où elle sera, il n'y a aucun risque, qu'elle peut ignorer son état
immunitaire concernant la toxoplasmose
E - Vous conseillez d'arrêter la Nivaquine prophylactique en cas de grossesse car elle est abortive
Bonne(s) réponse(s) : B

A - Contrôler d'abord sa sérologie prénuptiale ou de sa première grossesse.


B - Sur ses examens prénuptiaux ou de sa première grossesse.
E - Surtout pas. La grossesse est une indication majeure à la prophylaxie antipalustre.

838
Exclusivement sur DOC - DZ : www.doc-dz.com NADJI 85
RESIDANAT EN POCHE TOME II
Cas Clinique en QCM

Monsieur M., a déjà travaillé sous les tropiques. Son vaccin DT TAB et sa vaccination anti-amarile datent de
janvier 1977 :
A - Son vaccin anti-amarile est encore valide
B - Son vaccin anti-amarile n'est plus valide
C - La revaccination anti-amarile est valide le jour même
D - Vous effectuez un rappel TAB DT
E - Vous effectuez un rappel PT (polio, tétanos)
Bonne(s) réponse(s) : B C D E

A - Validité de 10 ans.
C - Mais à compter du 10ème jour qui suit une primo-vaccination.

On a récemment rapporté un cas de paludisme chloroquino résistant contracté au Burkina Faso :


A - Vous déconseillez fortement la chimio-prophylaxie du paludisme par la chloroquine
B - Vous conseillez une chimioprophylaxie par méfloquine
C - Vous préconisez pour Arnaud : 10 mg/kg/sem de chloroquine ; pour les parents 100 mg/j
D - Vous prescrivez la chloroquine en la faisant commencer 15 jours avant le départ
E - Vous prescrivez la chloroquine à partir du jour du départ et vous insistez sur sa poursuite pendant 4
semaines après le retour
Bonne(s) réponse(s) : C E

A B C - Dans un pays du groupe II, la prophylaxie doit être réalisée par la chloroquine, en ayant à sa disposition de la
méfloquine ou de l'halofantrine à visée curative.
D - A débuter le jour du départ.

En cas de diarrhée aiguë chez Arnaud, vous prescrivez :


A - L'hospitalisation immédiate
B - La surveillance de l'état d'hydratation
C - Pour cela le peser 4 fois par jour
D - Se contenter de surveiller le pli cutané
E - Introduire l'eau de riz et la gelée de coing dans son régime et lui supprimer le lait
Bonne(s) réponse(s) : B C E

Sans commentaire.

Une femme de 30 ans se présente aux urgences d'un C.H.U., pour une fièvre à 40°2 avec des frissons et des sueurs intenses
depuis 4 jours. Elle revient d'un séjour en Afrique 6 jours plus tôt où elle a pris sa Nivaquine® régulièrement, elle n'a pas de
diarrhée. Le médecin de ville a prescrit une goutte épaisse qui est négative. L'examen clinique est normal au niveau cardiaque
pulmonaire, abdominal et génital. La TA est à 12/8 et le pouls à 140/mn. Elle se plaint d'une discrète dysurie, et se lève de
façon inhabituelle 2 fois par nuit pour uriner. Elle n'a pas de douleur lombaire et les reins ne sont pas augmentés de volume à
la palpation. La NFS montre 15 600 globules blancs dont 90 % de polynucléaires neutrophiles.
Vous avez d'emblée demandé, hémocultures, coprocultures et ECBU.
Après 24 heures, le laboratoire vous informe que l'hémoculture est positive révélant un bacille Gram négatif. Vous vous
orientez vers le diagnostic d'infection à bacille Gram négatif.

La thyphoïde est possible mais, parmi les critères suivants, lequel (lesquels) rend (rendent) ce diagnostic peu
probable ?
A - Les coprocultures sont négatives
B - Les données de la numération
C - L'absence de diarrhée
D - La tachycardie
E - L'absence de splénomégalie
Bonne(s) réponse(s) : B C D

A - Seulement 30 % des typhoïdes s'accompagnent d'une coproculture positive.


B - Leuconeutropénie habituellement au cours de la typhoïde.
C - Présente dans 70 % des typhoïdes.
D - Dissociation du pouls au cours de la typhoïde.

839
Exclusivement sur DOC - DZ : www.doc-dz.com NADJI 85
RESIDANAT EN POCHE TOME II
Cas Clinique en QCM

La brucellose est possible mais parmi les éléments suivants, lequel ou lesquels rend(ent) ce diagnostic très peu
probable ?
A - L'hémoculture a poussé en 24 heures
B - Il n'y a pas de Brucellose en Afrique
C - Les données de la numération
D - Frissons intenses depuis 4 jours
E - L'identification d'un bacille gram négatif à l'hémoculture
Bonne(s) réponse(s) : A C D

A - Au cours de la brucellose, les hémocultures poussent en 5 à 8 jours.


B - Maladie connue dans le monde entier.
C - Leuconeutropénie au cours de la brucellose.
D - Les frissons ne sont pas classiques au cours d'une brucellose.

L'ECBU révèle une infection urinaire : pyurie significative, avec bactériurie franche : 10 exposant 8 germes/ml.
Le germe est un bacille Gram négatif. Vous vous rangez au diagnostic de pyélonéphrite. Parmi les germes
suivants, lequel vous parait le plus probable chez cette jeune femme, qui n'a aucun antécédent urinaire :
A - Entérocoques
B - E. coli
C - Serratia
D - Proteus
E - Pseudomonas
Bonne(s) réponse(s) : B

Sans commentaire.

Parmi les antibiotiques suivants, lequel ou lesquels pourriez-vous utiliser d' emblée sans précision de dose ou
de mode d'administration en première intention avant l'antibiogramme :
A - Co-trimoxazole
B - Quinolone
C - Aminoside
D - Macrolide
E - Métronidazole
Bonne(s) réponse(s) : A B C

C - En association.
D - Inactif sur les entérobactéries et mauvaise concentration urinaire.

Un jeune homme de 25 ans est employé aux abattoirs depuis deux mois. Il vient vous consulter pour un état fébrile traînant
accompagné de sueurs et de douleurs musculaires et articulaires diffuses et d'une douleur de la fesse. Le début, imprécis,
remonterait à une dizaine de jours. La température mesurée à votre cabinet est à 38°5, le pouls en rapport. Ce jeune homme
n'a pas maigri, est modérément asthénique. Vous palpez une splénomégalie, une hépatomégalie, des micro-adénopathies
cervicales. La NFS montre 4 000 globules blancs par mm3 40 % de polynucléaires neutrophiles. Vous soupçonnez à juste titre
une brucellose aiguë qui est confirmée par la séro-agglutination de Wright positive au 1/1280°, et ultérieurement par la
positivité de l'hémoculture.

Parmi les traitements suivants lequel aura la meilleure efficacité ?


A - Vaccinothérapie antimélitococcique
B - Ampicilline + streptomycine
C - Vibramycine + rifampicine
D - Rifampicine + cotrimoxazole
E - Colimycine + Gentamicine
Bonne(s) réponse(s) : C

Rifampicine 900 mg le matin.


Vibramycine 200 mg le soir.

Quelle durée de traitement envisagez-vous pour traiter cette septicémie brucellienne dans sa phase aiguë ?
A - 8 jours
B - 15 jours
C - 30 jours
D - 45 jours
E - 80 jours
Bonne(s) réponse(s) : D

La prolongation du traitement au-delà de 6 semaines ne réduit pas le risque de rechute qui est de 5 %.

840
Exclusivement sur DOC - DZ : www.doc-dz.com NADJI 85
RESIDANAT EN POCHE TOME II
Cas Clinique en QCM

Outre l'efficacité bactériologique in vitro, quelle est la motivation principale de votre choix :
A - Elimination hépato-biliaire
B - Pénétration intracellulaire
C - Concentration Iymphatique
D - Concentration sérique élevée
E - Aucun autre critère d'efficacité in vitro
Bonne(s) réponse(s) : B

Sans commentaire.

Sur quel argument arrêtez-vous votre traitement ?


A - Apyrexie stricte depuis 1 semaine
B - Négativation du séro-diagnostic de Wright
C - Non positivation de l'IDR à la Mélitine
D - Négativation de l'hémoculture
E - Aucune de ces propositions
Bonne(s) réponse(s) : E

L'expérience clinique collective fixe la durée de traitement à 6 semaines.

Retenez la ou les proposition(s) exacte(s) en matière de prophylaxie de la brucellose :


A - Abattage des animaux atteints
B - Education sanitaire et surveillance des professions exposées
C - Vaccination des professionnels exposés
D - Antibioprévention par les tétracyclines
E - Déclaration obligatoire
Bonne(s) réponse(s) : A B C E

Sans commentaire.

Un homme de 25 ans, jusque là en bonne santé, consulte pour une asthénie avec fièvre à 38°C et douleurs à la déglutition. A
l'examen, on trouve une angine et de petites adénopathies cervicales. Le médecin prescrit un traitement par sulfamides, anti-
inflammatoires non stéroïdiens et paracétamol.
Deux jours plus tard, devant l'absence d'amélioration, le patient consulte un autre médecin qui arrête le traitement précédent
et prescrit de l'Ampicilline et de l'acide acétylsalicylique. Cinq jours plus tard, vous êtes amené à examiner ce patient.
L'énanthème a disparu mais on note l'existence d'un exanthème maculo-papuleux de type morbiliforme, rouge orangé,
prurigineux disséminé sur la presque totalité du tégument.

Parmi ces diagnostics, vous devez évoquer :


A - Erythrodermie
B - Scarlatine
C - Toxidermie
D - Rougeole
E - Mononucléose infectieuse
Bonne(s) réponse(s) : A C E

A - L'érythème disséminé signe l'érythrodermie.


B - On observerait un érythème scarlatiniforme associé à l'énanthème.
C - Toxidermie érythrodermique.

La recherche de la cause justifie :


A - Numération formule sanguine
B - Fond d'oeil
C - Electrophorèse des protides
D - M.N.I test
E - IgE
Bonne(s) réponse(s) : A D

Pour argumenter une MNI.


- A. Syndrome mononucléosique.

841
Exclusivement sur DOC - DZ : www.doc-dz.com NADJI 85
RESIDANAT EN POCHE TOME II
Cas Clinique en QCM

Au cas où vous évoqueriez une toxidermie, quels sont parmi les médicaments suivants, les 3 que vous
suspecteriez le plus ?
A - Sulfamides
B - Anti-inflammatoires non stéroïdiens
C - Ampicilline
D - Aspirine
E - Paracétamol
Bonne(s) réponse(s) : A B C

E - Plus souvent à l'origine d'urticaire exceptionnellement à l'origine de toxidermies.


A, B et C - Le sont fréquemment.

Parmi les examens complémentaires suivants, vous demanderiez pour affirmer le diagnostic de toxidermie ?
A - Numération des éosinophiles
B - IgE spécifiques
C - Intradermoréaction au médicament
D - Test de transformation Iymphoblastique aux médicaments
E - Aucun de ces examens
Bonne(s) réponse(s) : E

Aucun examen n'apporte d'éléments diagnostiques formels.

Dans l'hypothèse d'une mononucléose infectieuse, et à propos de l'éruption cutanée, il est vrai que :
A - On peut observer une telle éruption après prise d'ampicilline en dehors de ce contexte
B - On peut l'observer au cours de la mononucléose infectieuse en dehors de la prise d'ampicilline
C - Au cours de la MNI, son apparition est favorisée par la prise d'ampicilline
D - Elle n'interdit pas la prise ultérieure d'une autre bêtalactamine
E - L'ampicilline peut être poursuivie à condition d'y associer une corticothérapie générale
Bonne(s) réponse(s) : A B C D

B - L'éruption est alors plus fugace, moins intense, sur le tronc et la racine des membres, non prurigineuse.
C - La fréquence du rash de la MNI atteint 90 % en cas de prise d'ampicilline.
E - Au cours de la MNI la corticothérapie est indiquée dans les formes avec : oedème pharyngé, complications
hématologiques, neurologiques ou cardiaques.

Monsieur A, âgé de 17 ans, algérien, est hospitalisé pour fièvre à 40° C évoluant depuis 48 heures. Les symptômes sont
apparus après son retour d'Algérie.
Il se plaint de céphalées intenses, de douleurs de l'hypochondre droit, de nausées. Il est constipé.
L'abdomen est souple; il y a une splénomégalie sans hépatomégalie, ni adénopathie, ni signe méningé. L'hémogramme
montre 4 600 leucocytes/mm3 (76 % de polynucléaires).
Trois hémocultures effectuées pendant les 24 premières heures d'hospitalisation donnent une culture positive à Salmonella
typhi. Le diagnostic est donc celui de fièvre typhoïde.

Salmonella typhi :
A - Est un germe pathogène opportuniste
B - Est un bacille à Gram positif
C - Est une entérobactérie
D - Possède un antigène Vi
E - Cultive facilement sur milieux ordinaires
Bonne(s) réponse(s) : C D E

A, B, E - Entérobactérie, donc bacille à Gram négatif, pathogène spécifique, strictement humaine.


D - Cet antigène capsulaire serait responsable de la résistance à l'action bactéricide des macrophages et est à l'origine des
vaccins actuels.

A propos de la physiopathologie de la fièvre typhoïde, il est exact que :


A - Salmonella typhi se transmet toujours par contamination interhumaine
B - La contamination est digestive
C - Les bactéries traversent la muqueuse intestinale
D - La fièvre typhoïde est une septicémie d'origine Iymphatique
E - Les porteurs sains ne jouent aucun rôle dans la dissémination de la maladie
Bonne(s) réponse(s) : A B C D

A, B - Le réservoir de germes est strictement humain. La contamination peut être directe, ou plus souvent indirecte par
ingestion d'eau ou d'aliments souillés.
C, D - Après passage de la muqueuse intestinale, les germes gagnent les ganglions lymphatiques mésentériques où ils se
multiplient.
E - Au contraire, les porteurs sains sont des vecteurs importants de la maladie.

842
Exclusivement sur DOC - DZ : www.doc-dz.com NADJI 85
RESIDANAT EN POCHE TOME II
Cas Clinique en QCM

La coproculture consiste à rechercher la présence de bactéries entéropathogènes dans les selles. Outre
Salmonella, elle peut révéler la présence de :
A - Shigella
B - Campylobacter
C - Pneumocoque
D - Rotavirus
E - Yersinia enterocolitica
Bonne(s) réponse(s) : A B E

C - Le pneumocoque n'a pas de tropisme digestif.


D - Mis en évidence par des techniques immunologiques ou virologiques.

Les complications possibles de la fièvre typhoïde sont la ou les suivantes :


A - Les hémorragies intestinales
B - Les perforations intestinales
C - Le collapsus cardio-vasculaire
D - Une encéphalopathie typhique
E - Une orchite
Bonne(s) réponse(s) : A B C D E

A - 5 % des cas.
B - 3 % des cas.
C - Origine endotoxinique.
D - Tuphos grave, comateux parfois.
E - Exceptionnelles.

Quel(s) est (sont) le(s) traitement(s) possible(s) ?


A - La pénicilline G
B - L'ofloxacine (Oflocet®)
C - La ceftriaxone (Rocéphine®)
D - Le chloramphénicol
E - Le cotrimoxazole (Bactrim®)
Bonne(s) réponse(s) : B C D E

B - L'ofloxacine permet une meilleure bactéricidie intracellulaire et une meilleure éradication des germes chez les porteurs
sains.

Le vaccin typhoïdique (TAB) est contre-indiqué en cas de :


A - Forte réaction après une injection antérieure
B - Néphropathie évolutive
C - Grossesse
D - Enfant de moins de 2 ans
E - Personne âgée de plus de 35 ans
Bonne(s) réponse(s) : A B C D

Le vaccin tend à être supplanté par un vaccin injectable polysaccharidique, élaboré à partir de l'antigène Vi de Salmonella
typhi.

Monsieur X..., âgé de 63 ans, a été opéré d'un carcinome épidermoïde du larynx et est porteur d'une trachéotomie depuis trois
ans. Il y a deux ans, il a été opéré d'un Iymphome pulmonaire et a été traité par du cyclophosphamide ( 100 mg/jour). A la
suite d'une rechute, il est hospitalisé les 28 et 29 novembre pour une cure de chimiothérapie comportant des doses totales de
90 mg d'adriamycine et 1110 mg de cyclophosphamide, ainsi que de la prednisone (70 mg pendant cinq jours).
Le 8 décembre, il est réhospitalisé pour une fièvre à 40°C accompagnant une leucopénie sévère : 200 globules blancs par
mm3, 30 % de polynucléaires neutrophiles, 10.000 plaquettes par mm3. Il pèse 60 kg. Malgré un traitement par céfotaxime (3
g/jour) et amikacine (1 g/jour), il reste fébrile. Une pneumopathie du lobe inférieur droit est constatée cliniquement le 15
décembre, et radiologiquement des infiltrats bilatéraux sont visibles le 19 décembre. L'électrocardiogramme ne montre pas
d'anomalie notable. A l'examen clinique il n'existe pas d'autre point d'appel infectieux en dehors du poumon.

Parmi les diagnostics suivants quel(s) est (sont) celui (ou ceux) que vous éliminez d'emblée ?
A - Extension du Iymphome
B - Embolie pulmonaire
C - Pneumonie à pneumocoques
D - Pneumopathie à germes anaérobies
E - Aucun de ces diagnostics
Bonne(s) réponse(s) : E

Secondairement, le diagnostic de pneumonie à pneumocoques peut être récusé compte tenu de l'antibiothérapie prescrite.

843
Exclusivement sur DOC - DZ : www.doc-dz.com NADJI 85
RESIDANAT EN POCHE TOME II
Cas Clinique en QCM

Parmi les signes et circonstances suivantes quels sont ceux qui sont en faveur d'une complication bactérienne ?
A - Fièvre élevée
B - Neutropénie
C - Images radiologiques
D - Corticothérapie
E - Absence d'atteinte pleurale
Bonne(s) réponse(s) : A B C D

QUESTION ANNULEE.

Vous discutez un lavage broncho-alvéolaire chez ce malade. Parmi les propositions suivantes concernant le
lavage broncho-alvéolaire, indiquez celles qui sont justes :
A - Il est formellement contre-indiqué en cas de neutropénie
B - Il permet de mettre en évidence les micro-organismes intracellulaires
C - Il permet de rechercher les Pneumocystis carinii
D - L'existence d'une hémorragie alvéolaire signe l'étiologie virale
E - L'absence de bactéries visibles après coloration de Gram et de Ziehl exclut l'étiologie bactérienne
Bonne(s) réponse(s) : B C

A - Il est primordial d'identifier un germe responsable d'un état septique.


B - Un pourcentage de leucocytes contenant des bactéries supérieurs à 5 % est un excellent critère d'infection pulmonaire.
C - Colorations de Gram-Weigert, de Grocott ou technique d'immunofluorescence.

Vous suspectez en priorité comme agent responsable :


A - Les bactéries
B - Les myxovirus respiratoires
C - Le cytomégalovirus
D - L'aspergillus
E - Le Pneumocystis carinii
Bonne(s) réponse(s) : A

Les infections bactériennes sont les plus fréquentes chez les patients neutropéniques. Le début brutal, l'existence d'un foyer
clinique sont d'autres arguments pour une étiologie bactérienne.

Parmi les bactéries suivantes, indiquez celles qui pourraient être responsables de cette pneumopathie
extensive, malgré la première antibiothérapie :
A - Streptococcus pneumoniae
B - Staphylococcus aureus
C - Haemophilus influenzae
D - Legionella pneumophila
E - Pseudomonas aeruginosa
Bonne(s) réponse(s) : B D E

A, C - Sont toujours sensibles aux céphalosporines de 3ème génération.

Un homme de 29 ans, vient de passer 2 ans en République Centre Africaine comme mécanicien automobile. Il a suivi une
chimioprophylaxie correcte antipalustre par la Nivaquine, chimioprophylaxie qu'il a poursuivie pendant 6 semaines après son
retour. L'interrogatoire permet de retrouver une notion d'impaludation probable, avec un seul accès typique de paludisme, au
milieu du séjour, il y a environ 10 mois, et un contexte hygiéno-diététique déplorable (nombreux bains de rivière, alimentation
improvisée, eau de boisson non contrôlée). Trois mois après son retour, il vient consulter pour une diarrhée modérée,
constituée de 4 - 5 selles par jour sans glaires ni sang. L'examen clinique est normal; on découvre une hyperéosinophilie à 32
% sur 16 100 GB, sans anémie, ni déficit en hémoglobine; urée, glycémie, créatinine, transaminases, phosphatases alcalines
et radiographie pulmonaire sont normales.

Vous devez suspecter en priorité devant cette diarrhée avec hyperéosinophilie :


A - Ascaridiose
B - Anguillulose
C - Ankylostomose
D - Bilharziose intestinale
E - Filariose
Bonne(s) réponse(s) : A B C D

E - N'occasionne pas de diarrhées. En fait, l'importance de l'hyperéosinophilie et l'intervalle de 3 mois orientent fortement vers
une anguillulose.

844
Exclusivement sur DOC - DZ : www.doc-dz.com NADJI 85
RESIDANAT EN POCHE TOME II
Cas Clinique en QCM

Actuellement il existe un diagnostic sérologique fiable pour :


A - Ascaridiose
B - Anguillulose
C - Ankylostomose
D - Bilharziose intestinale
E - Filariose
Bonne(s) réponse(s) : D E

A, B, C - Les sérologies des nématodoses sont d'interprétation difficile en raison de nombreuses réactions croisées.

Dans le cadre de cette observation, quels sont les deux prélèvements les plus utiles pour vous aider au
diagnostic étiologique ?
A - Gorge
B - Selles
C - Urines
D - Crachats
E - Sang
Bonne(s) réponse(s) : B E

B - Pour un examen parasitologique et une coproculture.


E - Pour la sérologie de bilharziose.

Le médicament contre l'ascaridiose est :


A - Combantrin®
B - Solaskil®
C - Notézine®
D - Mintézol®
E - Biltricide®
Bonne(s) réponse(s) : B

B - L'efficacité du Solaskil® est excellente dans l'ascaridiose. Sont également efficaces Combantrin® et Fluvermal®.

Le médicament efficace contre l'Anguillulose est :


A - Combantrin®
B - Solaskil®
C - Notézine®
D - Mintézol®
E - Biltricide®
Bonne(s) réponse(s) : C

25 mg/kg à mâcher le soir 3 à 4 jours de suite, une 2ème cure doit être entreprise 3 semaines plus tard.

Le médicament efficace contre l'Ankylostomose est :


A - Combantrin®
B - Solaskil®
C - Notézine®
D - Mintézol®
E - Biltricide®
Bonne(s) réponse(s) : A

Le Fluvermal® est également efficace.

Le médicament efficace contre la bilharziose intestinale est :


A - Combantrin®
B - Solaskil®
C - Notézine®
D - Mintézol®
E - Biltricide®
Bonne(s) réponse(s) : E

Le Biltricide® est efficace sur tous les schistosomes.

845
Exclusivement sur DOC - DZ : www.doc-dz.com NADJI 85
RESIDANAT EN POCHE TOME II
Cas Clinique en QCM

Parmi les médicaments suivants quel est celui qui n'est efficace que dans la Bilharziose intestinale à
Schistosoma mansoni ?
A - Ambilhar®
B - Biltricide®
C - Oltipraz®
D - Zentel®
E - Vansil®
Bonne(s) réponse(s) : E

Le Biltricide est actif sur les 4 espèces.

Monsieur E.D., âgé de 21 ans, libanais vivant en France depuis un an, est hospitalisé pour fièvre s'étant élevée
progressivement jusqu'à 40°C.
En outre, il souffre de céphalées intenses, de douleurs dans l'hypochondre droit, de nausées. Il est constipé. Les symptômes
sont apparus 15 jours après le retour du patient de vacances passées au Moyen-Orient.
A l'examen, on note une angine érythémateuse avec ulcérations des piliers antérieurs du voile du palais, l'abdomen est
souple, il n'y a pas d'hépatosplénomégalie, ni d'adénopathies. La numération formule sanguine montre 4,6.109 leucocytes par
litre avec 76 % de polynucléaires.
Six hémocultures sont pratiquées sur 48 heures d'hospitalisation. Les 6 flacons donnent une culture positive en 24 heures à
48 heures. L'identification monte qu'il s'agit de Salmonella typhi.
Le malade est traité par du Chloramphénicol. La fièvre disparait en 4 jours, les signes cliniques rétrocédent rapidement.
Un séro diagnostic de Widal-Félix est pratiqué au douzième jour de la maladie et donne les résultats suivants :
TO = 1 :400, TH = 1/800, AO = 0, AH = 0, BO = 1/200, BH = O.
Le malade sort de l'hôpital après que les coprocultures de contrôle se soient révélées négatives.

Salmonella typhi :
A - S'appelle encore "bacille d'Eberth"
B - Est Gram positif
C - Fait partie des enterobacteriacae
D - Possède toujours des antigènes somatiques de type O
E - Possède souvent des antigènes Vi
Bonne(s) réponse(s) : A C D E

B - C'est un BGN.
D - L'antigène O correspond au lipopolysaccharide.

Outre le chloramphénicol qui reste le traitement classique, on peut utiliser dans le traitement de la fièvre
typhoïde :
A - La pénicilline
B - La ceftriaxone
C - L'érythromycine
D - La colimycine
E - Le thiamphénicol
Bonne(s) réponse(s) : B E

On peut également utiliser le cotrimoxazole, l'ampicilline, les fluoroquinolones.

Le séro-diagnostic de Widal-Félix est une réaction :


A - De fixation du complément
B - D'agglutination conditionnée (indirecte)
C - De précipitation
D - D'immuno-fluorescence
E - D'agglutination directe
Bonne(s) réponse(s) : E

Le principe du séro-diagnostic est basé sur la capacité des anticorps sériques du patient d'agglutiner des bactéries tuées
appartenant aux différents sérotypes responsables des fièvres typhoïdes.

846
Exclusivement sur DOC - DZ : www.doc-dz.com NADJI 85
RESIDANAT EN POCHE TOME II
Cas Clinique en QCM

Le séro diagnostic de Widal-Félix distingue les anticorps dirigés d'une part contre les antigènes O, d'autres part
contre les antigènes H :
A - Les anticorps anti-O apparaissent avant les anticorps anti-H
B - Les anticorps anti-H apparaissent avant les anticorps anti-O
C - Les antigènes H sont de nature protéique
D - Les antigènes O sont liés au corps bactérien
E - Les antigènes H constituent l'endotoxine
Bonne(s) réponse(s) : A C D

A - Les anti-O apparaissent vers le 8ème jour et les anti-H vers le 10ème-12ème jour.
C - Ce sont les antigènes flagellaires.
D - Ils correspondent au lipopolysaccharide (endotoxine).

Sur le plan physiopathologique et épidémiologique :


A - Il n'existe pas de réservoir animal de S. typhi
B - La contamination est digestive
C - Les bactéries traversent la muqueuse intestinale
D - Les bactéries peuvent se Iyser en libérant l'endotoxine
E - Les porteurs sains ne jouent aucun rôle dans la dissémination de la maladie
Bonne(s) réponse(s) : A B C D

Sans commentaire.

Un homme de 60 ans, jardinier a brutalement des frissons, un malaise général et une douleur de la jambe gauche. Il n'y a pas
eu d'épisode antérieur mais une intoxication alcoolo tabagique.Il s'est blessé le pied en marchant sur un clou il y a 5 jours.
Il existe une fièvre à 39°C, une déshydratation et une légère obnubilation. Localement, un érythème intense et une
augmentation de la chaleur locale depuis le dos du pied jusqu'au tiers de la jambe.
La peau est tendue, luisante avec par endroit un décollement bulleux limité. Il existe une adénopathie inguinale douloureuse,
des râles bronchiques diffus, un discret souffle protosystolique de pointe et un intertrigo bilatéral entre les 4ème et 5ème
orteils, ainsi qu'un mauvais état dentaire.
La numération formule montre une polynucléose neutrophile et une vitesse de sédimentation à 60 à la première heure.

Parmi les diagnostics suivants, vous pouvez évoquer :


A - Erysipèle
B - Lymphangite
C - Pemphigus
D - Impétigo
E - Pemphigoïde bulleuse
Bonne(s) réponse(s) : A B

A - Traduit la réaction selon un mode inflammatoire particulier à un foyer infectieux à Streptocoque.


B - Traduit la dissémination lymphatique de l'infection.
C, E - L'érysipèle peut prendre un aspect bulleux.
D - Les signes locaux et généraux sont trop importants.

Quel antibiotique choisissez-vous pour le traitement ?


A - Pénicilline G
B - Ampicilline
C - Amoxicilline
D - Cefotaxime
E - Aucun des produits précédents
Bonne(s) réponse(s) : A

La pénicilline G est l'antibiotique de choix pour le traitement des érysipèles.

En cas d'allergie aux bêtalactamines quel est l'anbiotique de choix ?


A - Streptomycine
B - Erythromycine
C - Gentamycine
D - Tobramycine
E - Cotrimoxazole
Bonne(s) réponse(s) : B

A, B, D - Les aminosides sont inactifs en monothérapie sur les streptocoques.

847
Exclusivement sur DOC - DZ : www.doc-dz.com NADJI 85
RESIDANAT EN POCHE TOME II
Cas Clinique en QCM

La ou les portes d'entrée vraisemblables sont :


A - Foyer infectieux dentaire
B - Infection bronchique chronique
C - Intertrigo des orteils
D - Embol septique
E - Plaie d'origine professionnelle
Bonne(s) réponse(s) : C E

L'érysipèle est une réaction inflammatoire à un foyer streptococcique de voisinage.

La prévention des récidives chez ce malade sera faite par :


A - Arrêt du tabac
B - Vasodilatateurs
C - Antibiothérapie continue
D - Traitement de l'intertrigo
E - Vaccination antitétanique
Bonne(s) réponse(s) : D

QUESTION ANNULEE.

Un enfant revient d'un séjour en Algérie et présente une diarrhée persistante depuis une dizaine de jours. A l'entrée dans le
service, on note un état de dénutrition assez avancé, une diarrhée et une fièvre à 38°5. L'hypothèse d'une fièvre typhoïde est
avancée.

Parmi les 5 examens complémentaires suivants, quels sont les 3 examens les plus urgents à pratiquer ?
A - Numération formule sanguine
B - Vitesse de sédimentation
C - Hémoculture
D - Coproculture
E - Dosage de la C réactive protéine
Bonne(s) réponse(s) : A C D

QUESTION ANNULEE.

Parmi les examens d'identification suivants, quels sont les deux examens qui permettront de confirmer le
diagnostic d'infection à Salmonella typhi?
A - Recherche de l'antigène O : 4,5
B - Recherche de l'antigène Vi
C - Recherche de l'antigène H : d
D - Recherche de l'antigène O : 9,12
E - Recherche de l'antigène H : b
Bonne(s) réponse(s) : C D

QUESTION ANNULEE.

Parmi les 5 antibiotiques suivants, quels sont ceux qui sont normalement efficaces en monothérapie et prescrits
avant le résultat de l'antibiogramme ?
A - Ampicilline
B - Rifampicine
C - Acide nalidixique
D - Péfloxacine
E - Thiamphénicol
Bonne(s) réponse(s) : A D E

Les fluoroquinolones ont une meilleure bactéricidie et sont plus efficaces pour éradiquer les germes et éviter le passage au
stade de porteur chronique.

848
Exclusivement sur DOC - DZ : www.doc-dz.com NADJI 85
RESIDANAT EN POCHE TOME II
Cas Clinique en QCM

Quelles sont les deux propriétés essentielles que doit présenter un antibiotique pour être efficace dans le
traitement de la fièvre thyphoïde ?
A - Se concentrer dans le système nerveux splanchnique
B - Se concentrer dans les ganglions Iymphatiques et dans la Iymphe
C - Se concentrer dans le plancher du IVème ventricule cérébral
D - S'éliminer sous forme active dans la bile
E - S'éliminer dans les urines
Bonne(s) réponse(s) : B D

La fièvre typhoïde est une septicémie d'origine lymphatique. La bile est un gîte fréquemment à l'origine du portage chronique
de salmonelles.

Quel est l'incident le plus fréquemment rencontré au cours d'un traitement par une Bêta-lactamine par voie orale
?
A - Toxicité labyrinthique
B - Choc anaphylactique
C - Modification de la flore intestinale
D - Accident par induction enzymatique au niveau des microsomes hépatiques
E - Toxicité cochléaire
Bonne(s) réponse(s) : C

Cette modification peut se traduire par une diarrhée et l'apparition d'une candidose.

Un homme de 23 ans, rentré depuis 48 heures d'un voyage en Thaïlande, consulte pour un écoulement uréthral purulent
apparu depuis son retour. Son dernier rapport sexuel remonte à 8 jours. L'examen clinique montre un prépuce oedématié, une
rougeur du méat qui laisse sourdre une goutte purulente.
Vous envisagez une uréthrite à gonocoque.

Indiquez la ou les propositions exactes concernant les gonocoques :


A - Ce sont des diplocoques gram négatif
B - Ils se cultivent facilement sur les milieux usuels
C - Ils sont pathogènes pour l'animal
D - Les gonocoques virulents sont porteurs de pili
E - Ils peuvent déterminer des septicémies avec éventuellement arthrites
Bonne(s) réponse(s) : A D E

B - Il faut une gélose enrichie avec du sang cuit.


C - Ils sont pathogènes exclusivement pour l'homme qui constitue le seul réservoir naturel de germes.
D - Les pili jouent un rôle capital dans l'attachement des bactéries aux cellules hôtes.

Vous instituez un traitement minute comportant une bêtalactamine : quelle est celle que vous choisirez ?
A - Oxacilline (Bristopen®)
B - Lincomycine (Lincocine®)
C - Ampicilline + Probénécide (Prototapen®)
D - Spiramycine (Rovamycine®)
E - Netilmicine (Nétromycine®)
Bonne(s) réponse(s) : C

Le Prototapen® comprend 3,5 gr d'ampicilline et 1 gr de probénécide qui retarde l'élimination urinaire de l'ampicilline.

Le laboratoire vient justement de vous faire parvenir la réponse : "présence de Neisseria gonorrhoeae. Cette
souche est productrice de bêtalactamase".
Devant ce résultat quel est(sont) le (les) antibiotique(s) indiqués ?
A - Spectinomycine (Trobicine®)
B - Cefotaxime (Claforan®)
C - Thiamphénicol (Thiophénicol®)
D - Amoxicilline (Clamoxyl®)
E - Doxycycline (Vibramycine®)
Bonne(s) réponse(s) : A B C E

A - 2 à 4 gr IM.
B - 1 gr IM.
C - 2,5 gr P.O.
D - La bêta-lactamase. plasmidique inactive la péni G et l'ampicilline.
E - 300 mg P.O.

849
Exclusivement sur DOC - DZ : www.doc-dz.com NADJI 85
RESIDANAT EN POCHE TOME II
Cas Clinique en QCM

En réalité, l'échec thérapeutique pourrait également s'expliquer par une infection mixte dans laquelle le ou les
micro-organismes suivants peuvent être impliqués :
A - Chlamydia trachomatis
B - Mycoplasme
C - Streptocoques
D - Legionella
E - Erysipelothrix insidiosa
Bonne(s) réponse(s) : A B

A - Sérogroupes D à K.
B - Uréaplasma uréalyticum.
E - Agent du Rouget du Porc.

Un homme de 30 ans, exerçant la profession de routier, consulte pour une érosion superficielle, minime, non douloureuse,
siègeant au niveau du sillon balano-préputial et s'accompagnant d'une adénopathie satellite. Cette lésion est apparue depuis
48 heures et inquiète le malade. Vous envisagez la possibilité d'un chancre syphilitique.

Parmi les propositions suivantes, quelle(s) est(sont) celle(s) que vous retenez ?
A - La confirmation biologique est inutile
B - Il faut faire un prélèvement sur un écouvillon que vous envoyez au laboratoire
C - Il faut faire pratiquer par le laboratoire un examen de la sérosité du chancre au microscope à fond noir
D - Il faut faire un prélèvement sanguin pour examen sérologique
E - Il faut faire un frottis sur une lame pour mise en évidence du tréponème
Bonne(s) réponse(s) : C D E

A - Elle doit être obtenue à chaque fois que c'est possible.


B - Inutile, les prélèvements doivent être examinés le plus vite possible.
D - TPHA et VDRL vers le 10ème jour du chancre et répéter au 21ème jour.
E - Anticorps fluorescents ou coloration à l'argent.

Quelles sont la ou les propositions exactes concernant l'agent de la syphilis, Treponema Pallidum :
A - Est une bactérie à Gram positif
B - Est une bactérie hélicoïdale
C - Sa culture est impossible
D - Est parasite strict de l'homme
E - Donne un effet cytopathogène caractéristique après inoculation aux cultures cellulaires (cellules de reins
de singe)
Bonne(s) réponse(s) : B D

A - Gram négatif.
C - Elle est possible in vivo (orchite de lapins).
D - Les mammifères peuvent être injfectés expérimentalement.

Parmi les groupes de réactions sérologiques ci-dessous, un seul ne concerne que des réactions mettant en jeu
des antigènes tréponémiques spécifiques :
A - TPHA - VDRL - Nelson
B - FTA - TPHA - Kline
C - TPHA - FTA - Nelson
D - Kline - Nelson - VDRL
E - TPHA - FTA - VDRL
Bonne(s) réponse(s) : C

Kline et VDRL sont des réactions non spécifiques.

Quelle(s) est(sont) la (les) proposition(s) exacte(s) concernant le diagnostic indirect de la syphilis ?


A - Les anticorps antitréponémiques apparaissent dans le sérum au cours de la phase primaire 20 à 40 jours
après la contamination
B - Les réactions sérologiques se négativent en 3 mois sans traitement
C - La positivité de la réaction du FTA est celle qui apparaît en premier
D - La positivité de la réaction de Nelson est celle qui apparaît en premier
E - Une réaction de VDRL positive est possible en l'absence de syphilis authentique
Bonne(s) réponse(s) : A C E

A C D - FTA positif au 8ème jour. TPHA vers le 12ème jour. VDRL et Kline vers le 20ème jour. Nelson à la fin de la 6ème
semaine après l'apparition du chancre.
E - LEAD, MNI...

850
Exclusivement sur DOC - DZ : www.doc-dz.com NADJI 85
RESIDANAT EN POCHE TOME II
Cas Clinique en QCM

Quelle(s) est(sont) la(les) proposition(s) de réponses que vous retenez concernant le traitement de la syphilis ?
A - La pénicilline est l'antibiotique de choix
B - Une seule injection d'Extencilline est suffisante dans le traitement du chancre
C - En cas de traitement précoce le test de Nelson peut rester négatif
D - Le tréponème est résistant à la tétracycline
E - En cas d'allergie à la pénicilline, on utilisera l'érythromycine
Bonne(s) réponse(s) : A B C

B - La prudence fait pratiquer l'injection à une semaine d'intervalle.


C - Et toutes les sérologies peuvent se négativer.
D - Les cyclines sont un traitement efficace de la syphillis.
E - On utilisera les cyclines, sauf chez la femme enceinte (érythromycine).

Un agriculteur de 40 ans est hospitalisé pour rechute d'une fièvre au décours d'un syndrome grippal. 4 hémocultures en flacon
de Castaneda aérobie sont pratiquées ainsi qu'un bilan sérologique de brucellose. Les réponses du laboratoire sont les
suivantes :
- Epreuves à l'antigène tamponné (EAT) : positif
- Séro-agglutination de Wright : positif au 1/640
- Immunofluorescence : positif au 1/640
- Les hémocultures sont négatives après 40 heures d'incubation.

Ces données :
A - Peuvent correspondre à une brucellose
B - Peuvent correspondre à une brucellose aiguë
C - Permettent d'exclure une brucellose
D - Permettent d'exclure une brucellose aiguë
E - Sont caractéristiques d'une brucellose à la phase chronique
Bonne(s) réponse(s) : A B

Les sérologies positives à des taux élevées évoquent une brucellose aiguë. Les hémocultures poussent en 5 à 8 jours au
cours d'une brucellose.

Parmi les mesures suivantes quelle est celle que l'on doit entreprendre en priorité ?
A - Attendre les résultats d'un antibiogramme
B - Pratiquer une IDR à la mélitine
C - Entreprendre une désensibilisation
D - Débuter le traitement antibiotique
E - Rechercher une interférence sérologique
Bonne(s) réponse(s) : D

B - Intéressant pour porter le diagnostic de brucellose chronique.


C - Traitement de la brucellose chronique.

Quels sont le ou les antibiotiques pouvant être utilisés dans le traitement de la brucellose ?
A - Ampicilline
B - Rifampicine
C - Colimycine
D - Tétracycline
E - Erythromycine
Bonne(s) réponse(s) : B D

A - Inefficaces en pratique clinique malgré une bonne activité in vitro.


En pratique : association doxycycline-rifampicine.

La brucellose :
A - Est provoquée par Brucella melitensis, B abortus suis ou B abortus bovis
B - Est une maladie strictement humaine
C - Est aussi appelée fièvre ondulante ou fièvre sudoroalgique
D - Est une maladie professionnelle
E - Est de diagnostic facile sur la clinique
Bonne(s) réponse(s) : A C D

B - L'infection des ovins, coprins, porcins et bovidés est à l'origine des infections humaines.
E - Les examens biologiques contribuent amplement à porter le diagnostic.

851
Exclusivement sur DOC - DZ : www.doc-dz.com NADJI 85
RESIDANAT EN POCHE TOME II
Cas Clinique en QCM

Les Brucella :
A - Sont des bactéries à gram négatif
B - Ont des exigences particulières de culture
C - Cultivent rapidement en moins de 24 heures
D - Présentent des antigènes communs avec d'autres bactéries en particulier Vibrio cholerae et Francisella
tularensis
E - Induisent une réaction d'hypersensibilité retardée
Bonne(s) réponse(s) : A B D E

B - Gélose au foie de boeuf.


C - En 5 à 8 jours.
D - A l'origine de réactions sérologiques croisées.
E - Testées par l'IDR à la mélitine.

Un jeune enfant de 21 mois ayant dans ses antécédents une rougeole à 1 an, un eczéma récurrent, et des otites à répétition,
présente brutalement un épisode convulsif avec hyperthermie, vomissements et hypertonie. Le F.O. est normal. L'E.E.G.
montre une souffrance cérébrale diffuse à prédominance temporale gauche. La numération formule sanguine objective une
hyperleucocytose à 10600/mm3 avec 37 % de polynucléaires et 50 % de Iymphocytes. La V.S. est de 25 mm à la 1ère heure.
L'interrogatoire apprend que la maman a présenté un herpès labial 3 semaines plus tôt. L'enfant est hospitalisé pour bilan
étiologique et traitement.

Parmi les examens complémentaires suivants indiquez lequel est le plus urgent à réaliser :
A - Hémoculture
B - Scanner cérébral
C - Ponction lombaire
D - Examen ORL
E - Bilan immunologique
Bonne(s) réponse(s) : C

Tout syndrome méningé fébrile impose une ponction lombaire en urgence.

Le diagnostic de méningo-encéphalite étant évoqué, quel est ou quels sont le ou les microorganismes pouvant
être responsables ?
A - Neisseria meningitidis
B - Virus
C - Mycobacterium tuberculosis
D - Borrelia burgdorferi
E - Escherichia coli
Bonne(s) réponse(s) : B D

A - Responsable de méningite purulente.


B - Herpès simplex, herpès zoster, CMV, VIH, adénovirus, entérovirus, myxovirus, rage, arbovirus.
C - Tableau de pie-mérite basillaire.
E - Méningite purulente néo-natale.
QCM difficile car toute méningite peut s'accompagner du tableau clinique présenté.

Parmi les virus suivants, lequel ou lesquels est ou sont habituellement responsables de telles
méningoencéphalites ?
A - Herpès Simplex
B - Morbillivirus
C - Myxovirus influenzae
D - Herpès zooster varicellae
E - Adénovirus
Bonne(s) réponse(s) :

QUESTION ANNULEE.

852
Exclusivement sur DOC - DZ : www.doc-dz.com NADJI 85
RESIDANAT EN POCHE TOME II
Cas Clinique en QCM

Dans l'hypothèse d'une méningoencéphalite herpétique quel est ou quels sont le ou les examens
complémentaires qui vous apporteront dans la journée des arguments positifs ?
A - La recherche d'antigènes herpétiques dans le L.C.R.
B - Un sérodiagnostic herpétique par ELISA
C - La recherche d'anticorps dans le L.C.R.
D - Le dosage d'interféron alpha dans le sang et le L.C.R.
E - Une recherche virale par culture dans L.C.R.
Bonne(s) réponse(s) : A C D

A - Test immunoenzymatique.
D - Témoigne d'un processus infectieux viral non spécifique.
E - Effet cytopathogène en 2 à 3 jours.

Quelle est l'attitude thérapeutique indiquée dans cette dernière hypothèse ?


A - Corticothérapie
B - Sérum salé hypertonique
C - Acyclovir (Zovirax®)
D - Foscarnet
E - Abstention
Bonne(s) réponse(s) : C

Par voie veineuse : 10 mg/kg toutes les 8 heures.


D - Anti-CMV.

Une femme de 25 ans consulte pour des lésions cutanées siègeant aux membres inférieurs, apparues depuis 8 jours. Ces
lésions, au nombre de 6, sont érythémateuses, infiltrées, douloureuses, de la taille d'une pièce de 5 francs.
Une biopsie effectuée constate une dermo-hypodermite non spécifique.

Parmi les diagnostics suivants, quel est le plus probable ?


A - Erythème noueux
B - Panniculite
C - Phlébites superficielles
D - Psoriasis
E - Erythème polymorphe
Bonne(s) réponse(s) : A

B - Les lésions histologiques sont hypodermiques.

Parmi les examens suivants, quel(s) est (sont) celui (ceux) intéressant(s) à pratiquer pour le diagnostic
étiologique ?
A - Radiographie pulmonaire
B - Intradermo-réaction à la tuberculine
C - Examen ORL
D - Recherche d'anticorps anti-nucléaires
E - Recherche d'un HLA B5
Bonne(s) réponse(s) : A B C

QUESTION ANNULEE.

Parmi les causes suivantes, quelle(s) est (sont) celle(s) qui peuvent être évoquée(s) ?
A - Tuberculose
B - Sarcoïdose
C - Maladie de Behcet
D - Yersiniose
E - Streptococcie
Bonne(s) réponse(s) : A B C D E

Par ordre de fréquence, les étiologies des érythèmes noueux sont en France :
- Sarcoïdose.
- Origine indéterminée.
- Streptococcique.
- Yersiniose.

853
Exclusivement sur DOC - DZ : www.doc-dz.com NADJI 85
RESIDANAT EN POCHE TOME II
Cas Clinique en QCM

La radio pulmonaire révèle des adénopathies hilaires bilatérales, symétriques. Quel diagnostic est le plus
probable ?
A - Tuberculose
B - Sarcoïdose
C - Streptococcie
D - Maladie de Behcet
E - Lupus érythémateux systémique
Bonne(s) réponse(s) : B

A - Adénopathies non symétriques.


B - Adénopathies interbronchiques bilatérales symmétriques.

Parmi les thérapeutiques suivantes, quelle(s) est (sont) celle(s) à proposer ?


A - Corticothérapie générale
B - Antipaludéen de synthèse
C - Immunosuppresseur
D - Antibiothérapie par voie générale
E - Abstention thérapeutique
Bonne(s) réponse(s) : D E

La pénicillinothérapie de tout érythème noueux n'ayant pas fait la preuve de son origine est de règle. Dans l'hypothèse d'une
sarcoïdose, l'évolution des lésions est spontanément favorable.

Un malade ayant subi une intervention chirurgicale sur les voies urinaires présente dans les suites opératoires un tableau
septicémique.
Trois hémocultures sont positives en 48 heures à Enterobacter cloacae.
Un traitement antibiotique est adapté selon la sensibilité du germe à l'antibiogramme et associe cefotaxime (Claforan®) et
amikacine (Amiklin®).

Enterobacter cloacae est :


A - Une corynébactérie commensale
B - Un cocci à Gram positif
C - Un bacille à Gram négatif appartenant à la famille des entérobactéries
D - Un bacille à Gram négatif appartenant à la famille des Pseudomonadaceae
E - Une mycobactérie
Bonne(s) réponse(s) : C

C'est une entérobactérie opportuniste.

Parmi les bactéries suivantes, quelles sont celles qui sont responsables d'infections du tractus urinaire après
manoeuvre instrumentale ?
A - Serratia marcescens
B - Streptococcus pneumoniae
C - Pseudomonas aeruginosa
D - Acinetobacter .
E - Brucella
Bonne(s) réponse(s) : A C D

Le germe le plus souvent responsable des infections urinaires nosocomiales est E. Coli.

Le cefotaxime (Claforan®) est :


A - Une ampicilline
B - Une uréido-pénicilline
C - Une céphalosporine de lre génération
D - Une céphalosporine de 3è génération
E - Un macrolide
Bonne(s) réponse(s) : D

Son métabolite, le désacétylcéfotascime.

854
Exclusivement sur DOC - DZ : www.doc-dz.com NADJI 85
RESIDANAT EN POCHE TOME II
Cas Clinique en QCM

L'amikacine (Amiklin®) est :


A - Une béta-lactamine
B - Un aminoside
C - Une cycline
D - Un macrolide
E - Une quinolone
Bonne(s) réponse(s) : B

Sans commentaire.

La concentration minimale inhibitrice (CMI) :


A - Est la plus forte concentration d'antibiotique capable d'inhiber toute culture visible en milieu liquide
B - Est la plus faible concentration d'antibiotique capable d'inhiber toute culture visible en milieu liquide
C - Est la concentration d'antibiotique qui laisse subsister moins de 0,01 % de survivants
D - Peut être déterminée par la méthode des disques en milieu gélosé (antibiogramme)
E - Est généralement inférieure à la concentration minimale bactéricide (CMB)
Bonne(s) réponse(s) : B D E

C - La CMB est la plus faible concentration d'antibiotique détruisant après 18 h de contact à 37°C,
99,99 % d'une population bactérienne.
D - Une relation est établie entre le diamètre d'inhibition de croissance et la valeur de la CMI.
E - Si la CMB est proche de la CMI, l'antibiotique est bactéricide pour la souche étudiée.

Un pouvoir bactéricide des antibiotiques et associations donne les résultats suivants :


amikacine : 0,01 % de survivants, céfotaxime : 1 % de survivants, céfotaxime + amikacine : 0,01 % de
survivants.
Vous pouvez conclure :
A - Que l'amikacine seule est bactéricide
B - Que le céfotaxime seul est bactéricide
C - Que l'association est bactéricide
D - Que l'association est bactériostatique
E - Que l'association est antagoniste
Bonne(s) réponse(s) : A C

Sans commentaire.

855
Exclusivement sur DOC - DZ : www.doc-dz.com NADJI 85
RESIDANAT EN POCHE TOME II
Cas Clinique en QCM

856
Exclusivement sur DOC - DZ : www.doc-dz.com NADJI 85
RESIDANAT EN POCHE TOME II
Cas Clinique en QCM
Une femme de 59 ans présente brutalement, alors qu'elle était au téléphone, une céphalée postérieure non accompagnée de
signes digestifs, qui va diminuer d'intésité dans les jours suivants, sans disparaître Cette patiente est une migraineuse depuis
30 ans, traitée par Dihydroergotamine au long cours Cinq jours après l'épisode initial, la malade présente en pleine nu un
malaise avec perte de connaissance, avec mouvements toniques des quatre membres durant 2 minutes environ Le médecin
traitant constate une raideur de nuque et un ptosis La malade est alors hospitalisée La PA est à 1 10/60mmHg, la température
est à 38,6 degrés, la patiente est très sommolente ; le syndrome méningé est franc et l'examen neurologique ne retrouve
qu'un ptosis droit associé à une dilatation pupillaire et une abolition du réflexe photomoteur droit. L'examen de l'oculomotricité
est rendu difficile par l'état de vigilance. L'examen tomodensitométrique pratiqué le même jour montre une opacité hématique
au niveau de la citerne sus-optique et des deux vallées sylviennes L'évolution clinique après l'hospitalisation est marquée par
une amélioration spontanée de la conscience, mais 10 jours après l'hospitalisation, la patiente sera retrouvée dans le coma
avec un déficit hémicorporel gauche, déviation de la tête et des yeux à droite.

Les éléments séméiologiques notés au niveau de l'oeil droit sont compatibles avec :
A - Une atteinte du nerf moteur oculaire commun droit
B - Une lésion du système symphatique péri-carotidien droit
C - Une lésion de la bandelette longitudinale postérieure
D - Une lésion protubérantielle gauche
E - Aucune de ces propositions n'est exacte
Bonne(s) réponse(s) : A

B - Donne un myosis.
C - Paralysie internucléaire.
D - Donne un VI gauche.

Le malaise nocturne ayant motivé l'hospitalisation évoque :


A - Une syncope vaso-vagale
B - Une manifestation de spasmophilie
C - Une crise d'épilepsie d'allure généralisée d'emblée
D - Des crises toniques postérieures témoignant d'un engagement des amygdales cérébelleuses
E - Une phase prodromique de migraine accompagnée
Bonne(s) réponse(s) : C

Sans commentaire.

Quel examen complémentaire devait être réalisé devant le tableau initial, au cours de l'hospitalisation ?
A - Une artériographie carotidienne droite
B - Une artériographie tétrapédiculaire
C - Une ventriculographie
D - Une angiographie digitalisée par voie veineuse.
E - Des études bactériologiques et sérologiques du LCR
Bonne(s) réponse(s) : B

Une artériographie des 4 axes doit être pratiquée à la recherche d'une malformation susceptible de saigner ou de resaigner.

Parmi les propositions diagnostiques suivantes, laquelle vous parait la plus probable en fonction des données
cliniques et paracliniques ?
A - Hémorragie méningée
B - Accident vasculaire cérébral ischémique
C - Encéphalite herpétique
D - Syphilis du système nerveux central
E - Tumeur de la région hypothalamique
Bonne(s) réponse(s) : A

Sans commentaire.

Un homme de 40 ans est amené pour une perte de connaissance brutale sur la voie publique.

En faveur d'une crise d'épilepsie grand mal vous retiendrez :


A - La précession par une sensation de malaise avec flou visuel
B - Une blessure liée à la chute
C - La morsure de la langue
D - L'amnésie totale de l'épisode critique
E - L'observation par l'entourage d'une phase tonique puis d'une phase clonique
Bonne(s) réponse(s) : B C D E

B - Est compatible avec une crise d'épilepsie mais n'oriente pas nécessairement vers ce diagnostic.

857
Exclusivement sur DOC - DZ : www.doc-dz.com NADJI 85
RESIDANAT EN POCHE TOME II
Cas Clinique en QCM

Parmi les éléments cités ci-dessous vous retenez en faveur d'une épilepsie généralisée primaire :
A - Existence à distance de la crise d'un signe de Babinski bilatéral
B - L'association à des absences petit mal
C - Existence d'absence dans la fratrie
D - Découverte de pointes ondes à 3 Hz généralisées a l'E.E G
E - Le fait qu'une crise grand mal traduit forcément une épilepsie généralisée
Bonne(s) réponse(s) : B C D

Sans commentaire.

Parmi les éléments ci-dessous vous retenez en faveur d'une épilepsie lésionnelle :
A - La notion d'un accouchement difficile
B - L'association avec des crises partielles motrices
C - L'existence de pointes ondes et de polypointes bilatérales à l'E.E.G
D - Une antécédent de traumatisme crânien sévère
E - L'existence d'une obnubilation post-critique
Bonne(s) réponse(s) : A B D

Sans commentaire.

Chez ce patient vous avez finalement conclu à une épilepsie généralisée primaire non traitée présentant tous
les arguments de la question Numéro 2, en faveur de ce diagnostic. Votre attitude thérapeutique comporte :
A - Une simple surveillance en contrôlant tous les deux mois l'E.E.G
B - 0,10 gramme de Phénobarbital quotidien per os en une prise
C - 1 gramme de Valproate de Sodium (Dépakine) quotidien en deux prises
D - 0, 5 gramme d'Ethosuximide en deux prises quotidiennes
E - Des principes d'hygiène de vie avec temps de sommeil suffisant et tempérance
Bonne(s) réponse(s) : C E

Le choix porte sur la Dépakine® devant l'association à des crises type petit mal.
D Le Zarontin® n'est pas efficace sur les crises type grand mal.

Monsieur LUC...Jean, 64 ans, est hospitalisé pour une paraparésie spasmodique d'installation progressive (le début des
troubles de la marche, qui se sont traduit par une fatigabilité, remontent environ à six mois). A l'examen vous constatez une
amyotrophie et des fasciculations de la main droite et de l'avant-bras gauches. Le reflexe tricipital gauche est aboli et le
réflexe palmaire gauche est vif. Tous les autres réflexes du membre supérieur sont normaux. Aux membres inférieurs. on note
uniquement des réflexes rotuliens et achilléens vifs et un signe de Babinski bilatéral. La sensibilité est normale, à tous les
modes, superficiels ou profonds. Avant de commencer à ressentir une fatigue à la marche, il y a six mois, Mr LUC... a souffert
du membre supérieur gauche (la douleur débutait au niveau de l'épine, de l'omoplate et irradiait à la face dorsale du bras et de
l'avant bras. Cette douleur était majorée par les mouvements du cou qui étaient très limités à l'époque. L'examen général est
normal, de même que le reste de l'examen neurologique.

Compte-tenu de ces renseignements, quel diagnostic retenez-vous ?


A - Névralgie cervico-brachiale discarthrosique
B - Myélopathie cervicarthrosique
C - Compression du tronc secondaire postérieur du plexus brachial
D - Névralgie amyotrophique (syndrome de Parsonage et Turner)
E - Syndrome du défilé costo-scalénique
Bonne(s) réponse(s) : B

Il existe une atteinte centrale médullaire devant le syndrome pyramidal des 2 membres inférieurs.

La topographie ressentie par la patiente avant les troubles de la marche est très évocatrice d'un territoire
radiculaire Lequel ?
A-C5
B-C6
C-C7
D-C8
E-D1
Bonne(s) réponse(s) : C

Abolition du tricipital gauche.

858
Exclusivement sur DOC - DZ : www.doc-dz.com NADJI 85
RESIDANAT EN POCHE TOME II
Cas Clinique en QCM

Parmi les examens complémentaires suivants. indiquez les deux qui sont nécessaires au diagnostic étiologique.
A - Tomographies de profil du rachis cervical.
B - Electromyogramme
C - Myélographie cervicale
D - Tomodensitométrie
E - Ponction lombaire
Bonne(s) réponse(s) : C D

A - Ne donne pas le diagnostic étiologique.


B - Confirme l'atteinte périphérique.
E - Pourrait montrer une dissociation albumino cytologique sans valeur étiologique.

Les données cliniques, l'âge du sujet et les donnés des examens complémentaires ont pour but principal
d'éliminer une maladie neurologique dont les signes d'examen sont comparables. Laquelle ?
A - Syringomyélie
B - Compression médullaire
C - Sclérose en plaques
D - Sclérose latérale amyotrophique
E - Séquelles de poliomyélite antérieure aigüe.
Bonne(s) réponse(s) : D

Fortement évoqué devant l'association d'un syndrome pyramidal et d'un syndrome neurogène, sans troubles sensitifs.

Monsieur S... âgé de 66 ans. a été hospitalisé il y a une dizaine de jours pour faiblesse généralisée. En fait depuis un an
environ il présente des difficultés de plus en plus grandes à se déplacer à effectuer les actes de la vie quotidienne à se
retourner dans son lit à se laver sans aide et est devenu totalement dépendant de son entourage. A l'examen la marche se fait
lentement à petits pas, le buste penché en avant les bras collés au corps et on note une hypertonie importante des quatre
membres avec signe de la roue dentée. Il n'y a pas de déficit moteur pas d'anomalie des réflexes. pas d'atteinte des
sensibilités : bon état général : pas d'atteinte des fonctions supérieures.

L'association d'akinésie et d'hypertonie fait évoquer une maladie de Parkinson. Cependant ce malade ne
présente pas de tremblement : quelle est la proposition exacte ?
A - Il ne s'agit en aucun cas d'une maladie de Parkison
B - Il s'agit d'une forme akinéto-hypertonique de maladie de Parkinson
C - Il faut évoquer un syndrome parkinsonien des neuroleptiques
D - Il faut évoquer un syndrome pseudo-bulbaire
E - Aucune proposition n'est exacte
Bonne(s) réponse(s) : B

Sans commentaire.

L'hypertonie de la maladie de Parkinson est qualifiée de plastique ceci veut dire :


A - Une augmentation du tonus de repos disparaissant au mouvement
B - Une augmentation du tonus se renforçant avec la répétition du mouvement
C - Une augmentation du tonus se renforçant avec l'amplitude du mouvement
D - Une augmentation du tonus constante pendant la durée du mouvement
E - Aucune proposition n'est exacte
Bonne(s) réponse(s) : D

Sans commentaire.

Parmi les propositions suivantes concernant le tremblement de la maladie de Parkinson, laquelle (ou lesquelles)
est (sont) exacte(s) ?
A - Le tremblement augmente au froid et à l'émotion
B - Le tremblement est un tremblement de repos
C - Le tremblement est souvent à prédominance unilatérale
D - Le tremblement n'est presque jamais isolé
E - Le tremblement disparaît pendant le sommeil
Bonne(s) réponse(s) : A B C D E

Sans commentaire.

859
Exclusivement sur DOC - DZ : www.doc-dz.com NADJI 85
RESIDANAT EN POCHE TOME II
Cas Clinique en QCM

Comment sont les réflexes ostéotendineux dans la maladie de Parkinson ?


A - Normaux
B - Augmentés
C - Diminués
D - Abolis
E - Pendulaires
Bonne(s) réponse(s) : A

Peuvent être augmentés sans signification précise.

Parmi ces situations, laquelle (lesquelles) justifie(nt) une contre indication ou de précautions particulières
préalables à la Dopa-thérapie ?
A - Antécédents psychiatriques
B - Insuffisance respiratoire
C - Insuffisance cardiaque
D - Sujet âgé de plus de 70 ans
E - Troubles urinaires (prostate)
Bonne(s) réponse(s) : A C

A - Peut réactiver un délire.


C - L'énoncé permet de choisir aussi cette réponse, il faut prendre des précautions en présence d'une insuffisance cardiaque :
la contrôler !

Parmi les situations suivantes indiquez celle(s) qui contre-indique(nt) l'utilisation des anti-cholinergiques.
A - Antécédents digestifs
B - Insuffisance cardiaque
C - Sujet âgé de plus de 70 ans
D - Troubles urinaires (prostate)
E - Glaucome
Bonne(s) réponse(s) : D E

Deux des contre-indications classiques des anticholinergiques.

Une femme de 55 ans se réveille un matin avec un torticolis La douleur descend à la face antéro-externe du moignon de
l'épaule, du bras et de l'avant bras, atteint le pouce et l'index sur leur face palmaire. Elle a déjà eu des épisodes semblables,
régressant en quelques jours. Son seul antécédent pathologique est l'existence de varices des membres inférieurs

Le siège de la douleur permet un diagnostic topographique Lequel ?


A - Nerf médian
B - Racine C5
C - Racine C6
D - Racine C7
E - Racine C8
Bonne(s) réponse(s) : C

Sans commentaire.

L'examen va plus particulièrement rechercher un déficit moteur au niveau :


A - Du biceps
B - Du long supinateur
C - Des fléchisseurs des 2 premiers doigts
D - Des fléchisseurs des 2 derniers doigts
E - Des interosseux
Bonne(s) réponse(s) : B

A - C5 C6
C - C7 C8 D1
D - C8 D1

860
Exclusivement sur DOC - DZ : www.doc-dz.com NADJI 85
RESIDANAT EN POCHE TOME II
Cas Clinique en QCM

On recherchera aussi une modification (diminution ou abolition) du :


A - Réflexe bicipital
B - Réflexe stylo-radial
C - Réflexe cubito-promateur
D - Réflexe tricipital
E - Réflexe palmo-mentonnier
Bonne(s) réponse(s) : A B

C.C8
D.C7

En fait il n'y a ni déficit moteur ni modification des réflexes dans ce cas particulier. Les radiographies simples du
rachis cervical montrent des images de cervicarthrose banales à cet âge. Quel diagnostic vous paraît le plus
probable :
A - Syndrome du canal carpien
B - Névralgie cervico-brachiale vertébrale commune
C - Hernie discale
D - Neurinome cervical
E - Syndrome du défilé costo-claviculaire
Bonne(s) réponse(s) : B

Sans commentaire.

Vous êtes appelé en urgence auprès d'un homme âgé de 60 ans, quelques heures auparavant, il a présenté un malaise brutal
associant rapidement un vertige intense, des vomissements et un déséquilibre, il existe, de plus, lorsque vous l'examinez, une
dysarthrie et des troubles importants de la déglutition, L'examen du malade permet de constater : une tachycardie modérée,
une tension artérielle à 15/1 OcmHg, une hypoesthésie de l'hémiface droite avec quelques douleurs à ce niveau, une
paralysie du voile à droite, un syndrome de Claude Bernard Horner à droite un discret syndrome cérébelleux droit ; du côté du
corps, vous notez l'existence d'une hémianesthésie dissociée respectant la face.

L'étiologie de cette séméiologie vous paraît être plutôt :


A - Tumorale bénigne
B - Tumorale maligne
C - Vasculaire ischémique
D - Vasculaire hémorragique
E - Infectieuse
Bonne(s) réponse(s) : C

La seméologie oriente vers un syndrome commun dont l'étiologie est plutôt C.

Parmi les antécédents suivants. quel(s) est (sont) celui (ceux) dont l'existence pourrait avoir dans ce cas une
signification étiologique ?
A - Prise régulière d'une benzodiazépine le soir pour faciliter le sommeil
B - Existence de cas de varicelle dans l'entourage du malade
C - Notion d'une tumeur maligne primitive du cerveau à l'origine du décès du père du malade
D - Manipulation du rachis cervical les jours précédents
E - Aucun de ces antécédents
Bonne(s) réponse(s) : D

On peut supposer que la manipulation du rachis cervical a entraîné le départ d'un embol par une plaque athéromateuse d'une
artère vertébrale.

Parmi les signes suivants, quel(s) est (sont) celui (ceux) que vous pourriez encore recueillir à l'examen
neurologique ?
A - Paralysie de la verticalité du regard
B - Nystagmus
C - Hémiplégie droite
D - Aphasie mixte
E - Aucun de ces signes
Bonne(s) réponse(s) : B

Le nystagmus fait partie du syndrome vestibulaire qui appartient au syndrome.

861
Exclusivement sur DOC - DZ : www.doc-dz.com NADJI 85
RESIDANAT EN POCHE TOME II
Cas Clinique en QCM

Vous décidez d'hospitaliser le malade. Parmi les examens suivants. quel (s) est (sont) celui (ceux) susceptible
(s) d'apporter une confirmation de la localisation lésionnelle ?
A - Ponction lombaire
B - Potentiels évoqués visuels
C - Electromyogramme des membres inférieurs avec mesure des vitesses de conduction sensivité
D - Examen du fond de l'oeil
E - Aucun de ces examens
Bonne(s) réponse(s) : E

Sans commentaires.

Une tomodensitométre est pratiquée. On peut s'attendre à observer :


A - Une hypodensité pariétale droite
B - Une hypodensité thalamique droite
C - Une hyperdensité spontanée de la région temporo-pariéto-occipitale gauche
D - Une hyperdensité spontanée protubérantielle
E - Aucune de ces anomalies
Bonne(s) réponse(s) : E

Syndrome de Wallenberg : ramollissement de la région latérale rétro-olivaire du bulbe.

Parmi les mesures thérapeutiques suivantes. Iaquelle (lesquelles) vous paraît (paraissent) souhaitable (s) ?
A - Préscription d'un traitement anti-épileptique
B - Mise en place d'une sonde gastrique
C - Préscription d'un traitement à visée anti-oedémateuse par le mannitol
D - Intervention neurochirurgicale en urgence
E - Aucune de ces mesures
Bonne(s) réponse(s) : B

Devant les troubles de la déglutition il paraît souhaitable de choisir cette réponse.

Parmi les faits suivants, quel est celui qui vous semble devoir caractériser
l'évolution la plus habituelle de cette affection. Toutes les mesures thérapeutiques souhaitables ayant été prises
?
A - Décès dans les semaines suivantes
B - Guérison totale sans séquelle
C - Amélioration avec persistance d'une hémiplégie résiduelle
D - Amélioration avec persistance du trouble sensitif de l'hémiface gauche
E - Aucun de ces faits
Bonne(s) réponse(s) : E

L'évolution la plus habituelle est la régression des troubles avec persistance du syndrome de CBH et du syndrome sensitif
alterne.

Un homme de 64 ans droitier est adressé en consultation à la suite d'un


déficit moteur du membre supérieur droit apparu brutalement il y a trois
jours, et ayant régressé totalement 6 heures après, sans séquelle.

A l'interrogatoire vous allez probablement retrouver :


A - Le déficit moteur intéressait également l'autre membre supérieur
B - Pendant cet épisode, l'entourage a noté une déviation de la bouche
C - Il existait imultanément une difficulté d'élocution
D - Le patient a présenté des céphalées très violentes et des vomissements immédiatement avant
l'installation de son déficit
E - Le patient a eu une diplopie transitoire
Bonne(s) réponse(s) : B C

C'est un accident sylvien gauche superficiel donc hémiplégie brachiofaciale avec aphasie transistoire car siégeant à gauche
chez un droitier.

862
Exclusivement sur DOC - DZ : www.doc-dz.com NADJI 85
RESIDANAT EN POCHE TOME II
Cas Clinique en QCM

Une hypothèse diagnostique vous paraît s'imposer. Laquelle ?


A - Sclérose en plaque
B - Tumeur rolandique gauche
C - Accident ischémique transitoire Sylvien
D - Hémorragie cérébro-méningée
E - Insuffisance vertébro-basilaire
Bonne(s) réponse(s) : C

Sans commentaire.

Cette hypothèse diagnostique vous amène à rechercher par l'interrogatoire certain(s) éléments
complémentaire(s) lequel ou lesquels ?
A - Hypoacousie gauche
B - Amaurose transitoire de l'oeil gauche
C - Vertiges positionnels fréquents
D - Céphalées matinales calmées par les vomissements
E - Autre épisode identique dans les mois précédents
Bonne(s) réponse(s) : B E

Amaurose ou CMT évocateurs d'une embolie d'origine carotidienne d'où naît la sylvienne.

L'examen peut montrer :


A - Un signe de Babinski bilatéral
B - Une raideur de nuque
C - Un souffle carotidien gauche
D - Un nystagmus
E - L'absence de déficit neurologique
Bonne(s) réponse(s) : C E

Le déficit étant transitoire l'examen peut être normal. L'origine probable est carotidienne donc sténose pouvant se traduire
cliniquement par un souffle.

Le bilan para-clinique comporte deux examens essentiels : lesquels ?


A - Ponction lombaire
B - Doppler des vaisseaux du cou
C - Scanner
D - Artériographie
E - Electromyogramme
Bonne(s) réponse(s) : B C

En fonction de ces deux examens une éventuelle intervention peut être décidée ; une artériographie est alors proposée ou
discutée.

Un homme de 70 ans, viticulteur, est adressé dans le service de neurochirurgie pour l'apparition depuis une dizaine de jours -
d'une confusion mentale avec difficultés à la marche. A l'examen, on retouve un homme apathique, somnolent, répondant
avec lenteur aux questions, présentant une désorientation temporo-spatiale. Sa démarche est raide, un peu hésitant avec
tendance aux rétropulsions. Il a un signe de la main creuse à la manoeuvre des bras tendus, au membre supérieur gauche
Les réflexes ostéotendineux sont vifs et diffusés aux quatre membres. Le reflexe cutané plantaire est Indifférent à droite et en
extension à gauche. Le fond d oeil est normal. L'interrogatoire de la famille n'apprend pas d' antécédents importants, sinon
que c'est -un bon vivant-, c'est normal, vu son métier et que de temps en temps il est un peu -éméché- et se cogne dans la
porte basse de sa cave, sans autre conséquence qu'une bosse sur le front.

Parmi les propositions suivantes, laquelle (lesquelles) rencontre-t-on dans un tableau d'hématome sous dural
chronique :
A - Il s'agit de sujets âgés ( 50 ans)
B - Le tableau clinique se constitue progressivement
C - On retrouve un antécédent traumatique violent
D - Il existe un tableau d'hypertension intracrânienne sévère
E - Il existe volontiers des troubles de coagulation
Bonne(s) réponse(s) : A B E

C - Il s'agit de petits traumatismes répétés.


D - S'il existe une HIC elle est le plus souvent modérée se limitant à des céphalées.

863
Exclusivement sur DOC - DZ : www.doc-dz.com NADJI 85
RESIDANAT EN POCHE TOME II
Cas Clinique en QCM

En dehors de l'hématome sous dural chronique, citez le (ou les) autre (s) diagnostic(s) qui pourrai(en)t être
envisagé(s) ?
A - Tumeur frontale
B - Tumeur temporale gauche
C - Hydrocéphalie à pression normale
D - Démence sénile
E - Hématome extra-dural
Bonne(s) réponse(s) : A C

B - Examen clinique non compatible = A.


E - Signes de localisation plus nets, évolution plus aiguë.

En dehors du scanner quel (s) autre(s) examen(s) complémentaire (s) permet (tent) de faire le diagnostic positif
d'hématome sous dural chez un adulte ?
A - La cisternographie isotopique
B - L'angiographie cérébrale
C - La gamma-anglo-encéphalographie Isotopique
D - L'échographie B
E - Les radiographies simples du crâne
Bonne(s) réponse(s) : B C

B - Image de refoulement des vaisseaux corticaux.


C - Zone d'hyperfixation au niveau de l'hématome.
D - Ne permet pas de diagnostic étiologique.

En fonction de l'ancienneté de l'hématome, le SCANNER sans injection de produit de contraste intraveineux


peut montrer :
A - Déplacement des structures médianes isolée
B - Hyperdensité périphérique entre la voûte et le parenchyme
C - Hypodensité parenchymateuse
D - Hyperdensité biconvexe périphérique en regard d'un trait de fracture
E - Hyperdensité intra-parenchymateuse
Bonne(s) réponse(s) : A B

B - Exclut C et D
D - Evoque un HED

Quelle est la complication la plus fréquente de la cure chirurgicale d'un hématome sous dural chronique ?
A - Hémiplégie
B - Ramollissement
C - Hydrocéphalie
D - Récidive de l'hématome
E - Abcès
Bonne(s) réponse(s) : D

Sans commentaire.

Une femme de 60 ans, diabétique connue, présente depuis six mois des douleurs de l'hémiface gauche, de grande intensité,
déclenchées par l'attouchement de la région nasogénienne sous forme de décharges intéressant la région du maxillaire
inférieur. L'examen met en évidence une hypoacousie gauche que la malade avait notée depuis quelques mois, mais qui ne
l'avait pas inquiétée. L'examen neurologique se limite à la constation d'une hypoesthésie cornéenne gauche.

La surdité est probablement en rapport avec :


A - Une atteinte de l'oreille moyenne
B - Une atteinte de l'oreille interne
C - Une atteinte du nerf auditif
D - Une atteinte du tronc cérébral
E - Une atteinte du lobe temporal droit
Bonne(s) réponse(s) : C

L'association d'une surdité et d'une névralgie secondaire homolatérale du V évoque un syndrome de l'angle pontocérébelleux
et donc une atteinte du VIII.

864
Exclusivement sur DOC - DZ : www.doc-dz.com NADJI 85
RESIDANAT EN POCHE TOME II
Cas Clinique en QCM

A un stade plus évolué, l'affection en cause peut comporter :


A - Un syndrome de Claude Bernard Horner
B - Une hémianopsie latérale homonyme
C - Un syndrome cérébelleux gauche
D - Une hypertension intra-crânienne
E - Une paralysie faciale de type central
Bonne(s) réponse(s) : C D

Le syndrome de l'angle ponto cérébelleux associe de façon variable une atteinte du VIII du VII (paralysie faciale de type
périphérique) et du V. Il peut s'y ajouter une atteinte du IV du V et du VI.

La douleur faciale peut être soulagée par :


A - La carbamazépine
B - Les barbituriques
C - La diphénylhydantoïne
D - L'indométacine
E - La dihydroergotamine
Bonne(s) réponse(s) : A C

Sans commentaire.

Pour contribuer au diagnostic.il est utile de demander :


A - Potentiels évoqués visuels
B - Potentiels évoqués auditifs
C - Doppler
D - Electronystagmogramme
E - Electromyogramme
Bonne(s) réponse(s) : B D

B - Localise l'atteinte sur le VII cochléaire.


D - Décrit l'atteinte du VIII vestibulaire.

L'étiologie la plus probable est :


A - Vasculaire
B - Inflammatoire
C - Tumorale
D - Dégénérative
E - En rapport avec le diabète
Bonne(s) réponse(s) : C

Très probablement neurinome du VIII.

Madame B, 59 ans, est hospitalisée à la demande de sa famille parce que depuis environ un an, elle ''perd la mémoire" et
s'est perdue à plusieurs reprises en sortant seule dans son quartier. L'examen confirme la présence d'une amnesie de fixation
massive. d'une amnésie des faits anciens.d'une désorientation spatiale. Il existe par ailleurs un manque du mot, une apraxie
idéomotrice et constructive.des troubles de l'écriture et de la lecture. L'humeur est légèrement dépressive. On retrouve
quelques idées de persécution : "On me vole mes affaires". Le reste de l'examen neurologique est normal. Le scanner
objective une "atrophie bilatérale avec dilatation globale du système ventriculaire prédominant sur les carrefours et les zones
occipitales"

Quel diagnostic peut-on évoquer ?


A - Syndrome de Korsakoff
B - Encéphalopathie de Gayet Wernicke
C - Maladie d'Alzheimer
D - Délire chronique paranoïaque
E - Maladie de Pick
Bonne(s) réponse(s) : C

Le syndrome aphaso-apraxique décrit, correspond à une maladie d'Alzheimer. Les autres diagnostics présentent d'autres
tableaux.

865
Exclusivement sur DOC - DZ : www.doc-dz.com NADJI 85
RESIDANAT EN POCHE TOME II
Cas Clinique en QCM

Parmi les symptômes suivants le (ou lesquels) est (sont) des arguments permettant d'évoquer en priorité ce
diagnostic ?
A - Humeur dépressive
B - Apraxie
C - Manque du mot
D - Idées délirantes de persécution
E -- Résultats du scanner
Bonne(s) réponse(s) : B C E

La maladie d'Alzheimer constitue typiquement un syndrome aphaso apraxo agnosique avec au scanner une dilatation
ventriculaire et une atrophie corticale à prédominance postérieure.

La preuve formelle du diagnostic pourrait être apportée par :


A - Les résultats du scanner
B - L'élèctroencephalogramme
C - L'examen psychométrique
D - L'aspect histologique des lésions cérébrales
E - L'étude anamnestique
Bonne(s) réponse(s) : D

Atrophie cérébrale, dégénérescence neurofibrillaire et vacuolaire. Plaques séniles, gliose fibrillaire.

L'amnésie de fixation peut se rencontrer dans plusieurs maladies.


Le ou lesquels parmi ceux ci ?
A - Démence sénile
B - Syndrome de Korsakoff
C - Démence artériopathique
D - Amnésie psychogène hystérique
E - Ictus amnésique
Bonne(s) réponse(s) : A B C

Sans commentaire.

Parmi les mesures suivantes, la ou lesquelles vous paraît (paraissent) être adoptées) pour cette patiente ?
A - Antidépresseurs.
B - Hospitalisation en Placement d'Office
C - Mesure légale de protection des bien (Tutelle)
D - Sauvegarde de Justice
E - Maintien dans la famille s'il existe la possibilité d'une surveillance permanente
Bonne(s) réponse(s) : C E

A - Non indiqué ici


B - Non indiqué : mesure prise par le préfet envers une personne présentant un danger pour la société ou pour elle-même.
C - Régime de protection. Le sujet est représenté civilement par un interne.
D - Mesure trop légère pour cette patiente

Le pronostic de cet état vous paraît :


A - Favorable. Possibilité de récupération totale ou partielle après traitement approprié
B - Imprévisible
C - Lié à la précocité du traitement
D - Médiocre. Chronicisation des troubles qui peuvent évoluer sur vingt ans
E - Défavorable. Evolution mortelle inéluctable dans les dix ans au plus
Bonne(s) réponse(s) : E

Sans commentaires

866
Exclusivement sur DOC - DZ : www.doc-dz.com NADJI 85
RESIDANAT EN POCHE TOME II
Cas Clinique en QCM
Un patient de 35 ans présente de survenue progressive, des fourmillements des quatre extrémités suivis au bout de 4 jours
d'un déficit moteur progressif dominant à la racine des membres. On note une abolition des réponses tendineuses aux quatre
membres. Ce patient est alité en permanence.

La sémiologie correspond à une atteinte :


A - Centrale
B - Périphérique
C - Diffuse
D - Localisée
E - Intéressant le tronc cérébral
Bonne(s) réponse(s) : B C

Périphériques car les réflexes sont abolis


Diffusés aux quatres membres

Le résultat de la ponction lombaire au 10ème jour est probablement :


A - Un liquide normal
B - Une pléiocytose
C - Une pléiocytose + une hyperalbuminorachie à 2 g
D - Une hypralbuminorachie à 2 g et un compte de cellules normal
E - Une hypoglycorachie
Bonne(s) réponse(s) : D

Classique dissociation albuminocytologique observée une semaine après le début mais inconstante.

Au cours de l'évolution, il est possible de voir apparaître :


A - Une paralysie oculomotrice unilatérale
B - Une paralysie du VII bilatérale
C - Un signe de Babinski bilatéral
D - Des troubles de la respiration
E - Une hémianopsie latérale homonyme
Bonne(s) réponse(s) : B D

La diplégie faciale est évocatrice.


La surveillance clinique permet de poser l'indication de l'intubation et ventilation assistée
(examen clinique + spirométrie).

L'électromyogramme met en évidence :


A - Des signes neurogènes
B - Des signes myogènes
C - Un retard de la réponse distale
D - Une réaction myotonique
E - Un ralentissement des vitesses de conduction
Bonne(s) réponse(s) : A C E

A - Evident (recrutement temporel)


C E - Car c'est maladie démyélinisante du système nerveux périphérique.

Il est vraisemblable de trouver dans les antécédents :


A - Un épisode infectieux
B - Un traumatisme
C - Une vaccination
D - Une intoxication à l'arsenic
E - Une hypertension artérielle
Bonne(s) réponse(s) : A C

2 fois sur 3 on retrouve l'une ou l'autre.

867
Exclusivement sur DOC - DZ : www.doc-dz.com NADJI 85
RESIDANAT EN POCHE TOME II
Cas Clinique en QCM

L'évolution la plus habituelle se fera :


A - Vers l'aggravation et le décès
B - Vers la guérison complète en trois mois
C - Vers la persistance de séquelles motrices importantes
D - Vers la chronicité
E - La guérison complète en un an
Bonne(s) réponse(s) : B

Une.aréflexie sequellaire est fréquente.

Le traitement doit comporter obligatoirement d'emblée :


A - Des corticoïdes
B - Une prise en charge kinésithérapique active
C - Un traitement antibiotique massif
D - Un traitement anticoagulant
E - Un traitement immunosuppresseur
Bonne(s) réponse(s) : B D

Il s'agit de prévenir les deux complications de l'alitement : thromboemboliques et de décubitus, donc anticoagulants nursing et
kinésithérapie.

Une femme de 52 ans. droitière, sans autre antécédent que des céphalées remontant à deux ans, attribuées par elle à une
sinusite. est hospitalisée à la suite d'une crise comitiale généralisée typique. L'origine tumorale de cette comitialité est
rapidement affirmée il existe au F O un oedème papillaire bilatéral les radiographies du crâne montrent une condensation de
la voûte, coronale droite : à la tomodensitométrie, processus expansif frontal externe droit, spontanément hyperdense et bien
limite. en regard de l'épaississement de la voûte : effet de masse important avec déviation de la partie antérieure du système
ventriculaire. de 6 mm vers la gauche, après l'injection, prise de contraste massive et homogène de la tumeur dont l'axe est de
60 mm.

Lequel des examen ci-dessous pourrait être utile à l'identification de ce processus expansif ?
A - Radiographies des trous optiques
B - Scintigraphie cérébrale
C - Recherche de cellules malignes dans le LCR
D - Relevé du champ visuel
E - Tomographies de la selle turcique
Bonne(s) réponse(s) : B

La tumeur la plus probable est un méningiome (tumeur bénigne). Montrerait une zone d'hyperfixation.

Indiquez parmi les symptômes suivants celui ou ceux qui vous paraissent compatibles avec cette localisation
tumorale :
A - Indifférence, apathie
B - Manque du mot à l'épreuve de dénomination des objets
C - Incontinence urinaire
D - Hémianospsie latérale droite
E - Apraxie constructive
Bonne(s) réponse(s) : A C

Les signes les plus fréquents sont de type psychiatriques (A) et à type de deshinibition, incontinence urinaire (c).

Quelle est la nature histologique la plus probable de cette lésion ?


A - Médulloblastome
B - Méningiome
C - Cholestéatome
D - Métastase d'un cancer cliniquement non dépisté
E - Adénome hypophysaire géant
Bonne(s) réponse(s) : B

Evidents : tous les critères de localisation d'aspect de la prise de contraste au cancer.

868
Exclusivement sur DOC - DZ : www.doc-dz.com NADJI 85
RESIDANAT EN POCHE TOME II
Cas Clinique en QCM

Quelle conduite thérapeutique adoptez-vous ?


A - Dérivation ventriculo-atriale
B - Biopsie en condition stéréotaxique
C - Abord direct de la lésion en vue d'exérèse
D - Radiothérapie de première intention
E - Traitement anti-oedémateux et surveillance TDM dans un mois
Bonne(s) réponse(s) : C

Exérèse totale y compris l'insertion au niveau de la dure mère (récidives).

Monsieur B..., 40 ans éthylique chronique est représentant de commerce. Dans les antécédents on retrouve une première
crise d'épilepsie généralisée il y a 6 ans : E E.G. et scintigraphie cérébrale s'étaient avérés normaux à l'époque. Un traitement
par Orténal (15 cg/jour) est depuis irrégulièrement suivi. Quatre nouvelles crises généralisées sont rapportées depuis. La
consultation actuelle est motivée par les faits suivants : depuis 8 mois le malade a présenté de courts épisodes d'environ 1
minute pendant lesquels le patient ressent comme une boule dans la gorge, il se met à mâchonner, interrompt la conversation
puis la reprend après l'épisode. Il présente des céphalées. A l'examen neurologique on retrouve une hémianopsie latérale
homonyme gauche en quadrant supérieur et un oedème papillaire au fond d'oeil.

L'épilepsie généralisée tardive est :


A - Toujours essentielle
B - Souvent secondaire à une lésion cérébrale acquise
C - Caractéristique du petit mal
D - Parfois associée à des crises focales
E - Justiciable d'un scanner
Bonne(s) réponse(s) : B D E

B - 25% des cas d'epilepsie de l'adulte ont pour étiologie l'alcoolisme


D - Toute crise focale peut secondairement se généraliser
E- - Evident dans ce contexte

Les crises présentées par le malade depuis 8 mois ont leur origine dans :
A - Le lobe frontal gauche
B - Le lobe pariétal droit
C - Le lobe frontal droit
D - Le lobe temporal droit
E - Le lobe occipital droit
Bonne(s) réponse(s) : D

Le caractère des crises et l'existence d'une quadranopsie latérale homonyme supérieure gauche oriente vers cette localisation.

Sur l'ensemble des données cliniques quel est le diagnostic le plus probable ?
A - Anévrysme sylvien droit
B - Métastase temporale droite
C - Astrocytome temporal droit
D - Hématome sous-dural chronique hémisphérique droit
E - Angiome temporal droit
Bonne(s) réponse(s) : C

A B E - Auraient été vus à la scintigraphie 6 ans avant


D - N'explique pas la HLH
C - Est choisi sur l'évolution, la négativité de la scintigraphie au début, le fait que les astrocytomes soient très épileptogènes et
la compatibilité de la localisation avec les signes cliniques.

Dans un but diagnostic quel est le premier examen complémentaire à mettre en oeuvre ?
A - Scanner sans injection
B - Scanner sans et avec injection
C - Etude du L.C.R
D - Scintigraphie cérébrale
E - Artériographie carotidienne droite
Bonne(s) réponse(s) : B

Sans commentaire.

869
Exclusivement sur DOC - DZ : www.doc-dz.com NADJI 85
RESIDANAT EN POCHE TOME II
Cas Clinique en QCM
L'histoire clinique de cette femme de 45 ans débute brutalement, alors qu'elle jardinait, par la survenue de céphalées
occipitales violentes accompagnées de vomissements et d'un malaise général sans cependant perte de connaissance.
L'interrogatoire ne retrouve aucun antécédent pathologique notable en dehors de fréquentes céphalées qu'elle appelle ses
migraines, et pour lesquelles plusieurs traitements ont été institué sans résultat. L'examen de cette femme en bon état
général. montre : - la persistance de céphalées importantes accompagnées d' un état nauséeux et d'une photophobie.
- un syndrome méninge net avec signes de Kernig et de Brudzinski
- une diplopie dans le regard direct avec un discret ptosis de la paupière droite et une dilatation relative de la pupille de ce côté
La TA. est à 19/10, le pouls à 80/mn et la température à 38°2.

Parmi les hypothèses diagnostiques suivantes quelle(s) est (sont) celle(s) que vous retenez :
A - Migraine ophtalmoplégique
B - Hypertension intracrânienne d'origine tumorale
C - Hémorragie méningée
D - Méningite virale
E - Encéphalopathie hypertensive
Bonne(s) réponse(s) : C

B - Tableau trop brutal. Pas de signes méningés


D - Donne rarement des signes déficitaires
E - Pas de signes méningés

Quel(s) est (sont) l'(les) examen(s) complémentaire(s) suivant(s) indispensable(s) pour affirmer votre diagnostic :
A - Scanner
B - Electroencéphalogramme
C - Ponction lombaire
D - Fond d'oeil
E - Doppler des vaisseaux cervicaux
Bonne(s) réponse(s) : A C

Sans commentaire.

Les signes ophtalmologiques présentés par cette malade sont en rapport avec :
A - Une atteinte du III extrinsèque
B - Une atteinte du III extrinsèque et intrinsèque
C - Une atteinte du Vl
D - Un syndrome de Claude Bernard-Horner
E - Une atteinte du IV
Bonne(s) réponse(s) : B

Diplopie et ptosis : atteinte extrinsèque


Mydriase : atteinte intrinsèque.

En fonction de votre diagnostic initial, quelle attitude proposez-vous ?


A - Mise en route d'un traitement antimigraineux
B - Mise en route d'une antibiothérapie
C - Attentisme sous surveillance
D - Bilan d'une hypertension artérielle
E - Poursuite des explorations neuroradiologiques
Bonne(s) réponse(s) : E

L'attitude pratique est de contacter le neurochirurgienet de prescrire une artériographie, une intervention peut être alors
proposée.

870
Exclusivement sur DOC - DZ : www.doc-dz.com NADJI 85
RESIDANAT EN POCHE TOME II
Cas Clinique en QCM
Un homme de 45 ans pesant 70 kg sans aucun antécédent débute sa maladie de façon brutale par la fièvre (40°) avec
frissons et une toux évacuant une expectoration Jaunâtre. L'antibiothérapie prescrite se limite à la Gentamycine (Gentalline) à
raison de 3 injections intra musculaires de 70 mg 48 heures plus tard là température est de 39° et il apparait une obnubilation
Examen clinique syndrome méningé franc syndrome de condensation de la base pulmonaire droite avec un épanchement
pleural minime température 40° pli cutané pâteux Tension artérielle 100- 70 mmHg Examens complémentaires : Cliché
thoracique : syndrome alvéolaire de la base droite Liquide céphalo-rachidien trouble Sang : Na 140 mmol/l Cl 100 mmol/l C02
total 2,7 mmol/l. Urée 8 mmol/l Potassium 4,7 mmol/l Protides 79 g/l.

Le diagnostic bactériologique peut être affirmé de façon formelle part l'examen direct et la culture
A - Du liquide céphalo-rachidien
B - De l'expectoration
C - Du prélèvement recueilli par ponction transtrachéale
D - Du liquide pleural
E - Des urines
Bonne(s) réponse(s) : A C

Sans commentaire.

L'état d'hydratation est :


A - Une déshydratation extra cellulaire pure
B - Une déshydratation intra cellulaire pure
C - Une déshydratation mixte
D - Une déshydratation extra cellulaire avec hyper hydratation intra cellulaire
E - Une hyperhydratation extra cellulaire
Bonne(s) réponse(s) : A

Pli cutané, protides augmentés, natrémie normale.

Le liquide céphalo-rachidien contient à I examen direct des cocci à gram positif en diplocoques. Il s'agit
probablement :
A - D'un pneumocoque
B - D'un méningocoque
C - D'un bactéroïde Fragilis
D - D'un Escheria coli
E - D'un pseudomonas aeruginosa
Bonne(s) réponse(s) : A

Sans commentaire.

L'aggravation sous gentamycine est dûe :


A - A une posologie insuffisante
B - A la résistance habituelle du germe en cause
C - A la présence d'un gîte bactérien extra pulmonaire et extra méninge
D - A la nécessité d'une bithérapie dans ce type d'infection
E - A toutes les propositions ci-dessus
Bonne(s) réponse(s) : B

Sans commentaire.

Le traitement doit comporter en première intention :


A - La colistine (Colimycine®)
B - La Pénicilline G
C - La minocycline (Mynocine®)
D - L'amikacine (Amiklin®)
E - La carbénicilline (Pyopen®)
Bonne(s) réponse(s) : B

Sans commentaire.

871
Exclusivement sur DOC - DZ : www.doc-dz.com NADJI 85
RESIDANAT EN POCHE TOME II
Cas Clinique en QCM
Femme de 25 ans. célibataire, sans antécédent, ayant présenté il y a un mois un vertige rotatoire aigu avec vomissements,
impossibilité de tenir debout. Pas de bourdonnement d'oreille Amélioration au bout de 3 jours. Depuis elle accuse un
déséquilibre interminent avec parfois quelques paroxysmes rotatoires de quelques secondes, pas particulièrement liés aux
changements de position. De plus, elle accuse une sensation de cartonnement dans l'hémicrâne gauche et des
fourmillements intermittents brusques dans les 2 membres inférieurs lors de la flexion de la nuque Enfin,la vision de son oeil
droit semble floue depuis un mois. L'examen neurologique objective une marche normale mais avec un léger déséquilibre au
demi-tour. La manoeuvre de Romberg objective une tendance constante à la chute vers l'arrière La marche aveugle, difficile à
réaliser, est très perturbée, sans prépondérance directionnelle. L'épreuve des index,bras tendus, la malade étant assise au
fauteuil, objective une divergence progressive Les réflexes sont vifs aux 2 membres supérieurs, mais polycinétiques aux 2
membres inférieurs avec un clonus de la rotule du coté gauche et un Babinski gauche. Le membre inférieur gauche chute
légèrement et oscille a la manoeuvre de Mingazzini La pallesthésie, la sensibilité discriminative et les sens des positions
segmentaires sont altérées au membre inférieur gauche. Par contre, la sensibilité à la piqûre est diminuée sur tout le membre
inférieur droit et sur l'hémi-abdomen droit jusqu'au niveau ombilical L'examen des paires crâniennes objective une
hypoacousie droite avec Weber latéralisé à droite et Rinne négatif, enfin un nystagmus multidirectionnel. Le reste de l'examen
clinique est normal.

La distribution des troubles sensitifs aux membres inférieurs et sur l'abdomen est compatible avec :
A - Un syndrome de la corne antérieure
B - Un syndrome cordonnal postérieur bilatéral
C - Un syndrome syringomyélique
D - Un syndrome de Brown Sequard
E - Un syndrome alterne du tronc
Bonne(s) réponse(s) : D

Le syndrome pyramidal avec altération de la sensibilité profonde à gauche, et de la sensibilité thermoalgique à droite :
hemisection de la moelle.

Quel type de nystagmus est toujours d'origine centrale ?


A - Horizontal
B - Horizontal rotatoire
C - Pendulaire
D - Rotatoire
E - Vertical
Bonne(s) réponse(s) : E

Tous les types de nystagmus peuvent être d'origine centrale.


Le nystagmus vertical n'est jamais d'origine périphérique.

L'argument ou les arguments qui plaide(nt) en faveur de la nature centrale du syndrome vestibulaire dans cette
observation est ou sont :
A - L'intensité du vertige initial rotatoire aigu
B - La constatation d'un nystagmus multidirectionnel
C - L'absence de déviations "harmonieuses"
D - L'atteinte de voies longues constatée à l'examen
E - L'hypoacousie droite
Bonne(s) réponse(s) : B C D

A - Les atteintes centrales donnent habituellement des vertiges peu intenses.


B - Serait plutôt en faveur d'une atteinte périphérique.

Le meilleur argument en faveur d'un diagnostic de sclérose en plaques dans cette observation est :
A - Le signe de Romberg
B - Le signe de Lhermine
C - L'âge de la patiente
D - La caractère multifocal des lésions
E - Le nystagmus multidirectionnel
Bonne(s) réponse(s) : D

Sans commentaire.

872
Exclusivement sur DOC - DZ : www.doc-dz.com NADJI 85
RESIDANAT EN POCHE TOME II
Cas Clinique en QCM

Les résultats des examens complémentaires suivants sont compatibles avec le diagnostic de sclérose en
plaques sauf un lequel ?
A - Cytochimie du L C.R normale
B - Latences augmentées aux potentiels évoqués visuels
C - Hypodensité paraventriculaire prenant le contraste au scanner
D - 15 lymphocytes : 0.50 9/1 de proteinorachie dans le L.C.R
E - 2 éléments blancs 2 g/l de protéinorachie dans le L.C.R
Bonne(s) réponse(s) : E

A - Le L.C.R. peut être normal


B - Permet un diagnostic infraclinique
C - Aspect d'une "plaque" au scanner
D - Compatible.

Une femme de 25 ans consulte pour des troubles de la marche et des fourmillements des jambes apparus progressivement
depuis deux mois. De plus. elle a remarqué que lorsqu'elle baissait la tête, elle ressentait une impression de courant
électrique descendant le long de la colonne jusqu'aux membres inférieurs. A L'examen les anomalies suivantes sont
constatées :
- Une exagération des réflexes osteo-tendineux (ROT) aux membres inférieurs avec signe de Babinski bilatéral
- Des troubles du sens de position des orteils et une hypoesthésie tactile remontant jusqu'à l'ombilic
- Une limitation de l'adduction des yeux dans les deux regards latéraux avec nystagmus de l'oeil en abduction
En reprenant l'interroqatoire, on apprend qu'un an auparavant, la patiente avait eu un important flou visuel de l'oeil gauche,
installé en 24 heures et qui avait régressé en 6 semaines sans laisser de séquelle. L'ophtalmologiste consulte quelques jours
après le début du trouble visuel avait constaté :
- Une acuité visuelle (A.V ) à 1/10 à gauche. 10/10 à droite
- Un scotome central de l'oeil gauche
- Un fond d'oeil normal, ainsi que le reste de l'examen ophtalmologique.

Les sensations de décharge lors des mouvements du cou correspondent :


A - A une atteinte spino-thalamique
B - A une atteinte radiculaire
C - A une atteinte des cordons postérieurs
D - A une atteinte des cornes postérieures
E - A une atteinte du lemniscus médian
Bonne(s) réponse(s) : C

Signe de Jean Lhermitte très évocateur de SEP.

La lésion responsable de la perte de la vue survenue un an auparavant correspond à une atteinte :


A - De la rétine
B - De la papille optique
C - Du nerf optique extra-oculaire
D - De la bandelette droite
E - De la région occipitale
Bonne(s) réponse(s) : C

L'épisode correspond à une NORB dans le cadre d'une SEP

Quel est l'examen qui a le plus de chances de vous apporter une information diagnostique sur l'épisode visuel
survenu 1 an auparavant ?
A - Un électro-rétinogramme
B - Une angiographie rétinienne
C - Des potentiels évoques visuels
D - Une tonométrie
E - Une goniométrie
Bonne(s) réponse(s) : C

Sans commentaire.

Le trouble de l'oculo-motricité correspond :


A - A une paralysie partielle des 111
B - A une atteinte nucléaire
C - A une atteinte des voies intémucléaires
D - A une atteinte des noyaux vestibulaires
E - A une atteinte cérébelleuse
Bonne(s) réponse(s) : C

Ophtalmoplégie inter nucléaire due à l'atteinte de la bandelette longitudinale postérieure très évocatrice de SEP.
873
Exclusivement sur DOC - DZ : www.doc-dz.com NADJI 85
RESIDANAT EN POCHE TOME II
Cas Clinique en QCM

Si l'altération biologique est caractéristique du diagnostic évoqué sur les données cliniques, on retrouvera dans
le L.C.R. :
A - Une élévation de l'albunime
B - Une élévation des Alpha globulines
C - Une élévation des Béta globulines
D - Une élévation des Gamma globulines
E - Une répartition oligoclonale des gamma globulines
Bonne(s) réponse(s) : D E

D E - Evocateurs La répartition oligoclonale à l'électrophorèse peut se voir dans la syphilis, la trypanosomiase, la Loase.

Une femme de 65 ans, modérément obèse, hypertendue bien controlée par le traitement médical vient consulter car elle
présente depuis plusieurs jours des phénomènes de cécité monoculaire transitoire droite ; chaque épisode dure quelques
secondes puis la vision redevient normale; ces phénomènes se sont produits deux à trois fois par jour depuis quatre à cinq
jours. L'ophtalmologiste n'a retrouvé aucune anomalie au fond d'oeil et a fait une échotomographie des axes artériels
cervicaux. Cet examen montre l'existence d'une sténose de l'origine de l'artère carotide interne droite

Parmi les pathologies suivantes indiquez celle qui est probablement responsable des troubles présentées par la
malade :
A - Hypotension orthostatique
B - Thrombose de l'artère centrale de la rétine
C - Embols plaquenaires rétiniens
D - Polyglobulie
E - Crises de migraine
Bonne(s) réponse(s) : C

Ce sont des emboles provenant d'une plaque d'athérome de la carotide interne.

La vascularisation rétinienne est assurée par l'artère ophtalmique qui est une branche de :
A - L'artère cérébrale antérieure
B - L'artère communicante antérieure
C - Le siphon carotidien
D - L'artère maxillaire interne
E - Le tronc basilaire
Bonne(s) réponse(s) : C

Sans commentaire.

Dès lors vous demandez :


A - Rendez-vous d'ophtalmodynamométrie
B - Rendez-vous d'électroencéphalogramme
C - Rendez-vous de tomodensitométrie cervicale
D - Bilan biologique lipidique
E - Aucun des examens ci-dessus
Bonne(s) réponse(s) : E

L'examen à demander est une artériographie qui précisera l'état des artères à destinée céphalique et permettra de poser
l'indication d'une intervention chirurgicale.

Parmi les traitements suivants, indiquez celui que vous conseillez :


A - Héparinothérapie par voie veineuse
B - Calciparinothérapie par voie sous-cutanée
C - Antivitamines K
D - Anti-agrégants plaquenaires
E - Endartériectomie carotidienne d'urgence
Bonne(s) réponse(s) : E

Le mot urgence gène un peu. C'est vrai qu'il est urgent de faire un bilan angiographique et de proposer une endartériectomie
pour éviter la constitution d'une cécité monoculaire ou un autre accident vasculaire.

874
Exclusivement sur DOC - DZ : www.doc-dz.com NADJI 85
RESIDANAT EN POCHE TOME II
Cas Clinique en QCM

Le développement de la lésion carotidienne peut s'expliquer par :


A - Tabagisme antérieur
B - Obésité
C - Elévation du cholestérol HDL
D - Hyperuricémie
E - Hypertension artérielle
Bonne(s) réponse(s) : A E

Deux facteurs de risque significatifs beaucoup plus pour l'HTA que pour le tabac.

Un homme de 53 ans consulte pour l'apparition récente de sensations vertigineuses avec tendance au déséquilibre vers la
droite. Il ne s'agit pas d'un sujet hypertendu, il rapporte que depuis au moins 3 ans, peut-être plus, il entend de sifflements
dans son oreille droite. et que depuis très longtemps il a pris l'habitude d'écouter le téléphone avec son oreille gauche Il ne
s'est jamais plaint de la moindre céphalée. L'examen met en évidence une surdité totale de l'oreille droite, une déviation des
index vers la droite, une déviation à la marche yeux fermés vers la droite une hypoesthésie cornéenne droite. Le revêtement
cutané présente de nombreux "molluscum pendulum" et des tâches café au lait éparsés. dont deux très grandes (supérieures
à 10 cm2) sur le dos.

Où se situe la lésion responsable de la symptomatologie ?


A - Dans l'angle ponto-cérébelleux droit
B - Au niveau du conduit auditif interne droit
C - Au niveau de l'hémisphère cérébelleux droit
D - Au niveau du tronc cérébral
E - Au niveau du tronc basilaire
Bonne(s) réponse(s) : A

Sans commentaire.

Quel (s) examen(s) est (sont) utile(s) au diagnostic topographique et étiologique ?


A - EEG
B - Potentiels auditifs évoqués du tronc cérébral
C - Cisternographie computérisée
D - Imagerie par résonnance magnétique
E - Fond d'oeil
Bonne(s) réponse(s) : B C D

A E - N'ont pas d'intérêt


B - Précise l'atteinte responsable de la surdité
C D - Font le diagnostic et précisent le volume

Les examens cliniques et paracliniques permettent de penser à un neurinome chez un malade porteur d une
phacomatose. Comment expliquez-vous l'hypoesthésie cornéenne ?
A - La tumeur comprime le nerf facial dans le conduit auditif
B - La tumeur comprime le tronc cérébral
C - La tumeur comprime le nerf trijumeau vers le haut
D - La tumeur se développe aux dépens du ganglion de Gasser
E - Le volume tumoral entraîne une diminution du reflexe photo-moteur
Bonne(s) réponse(s) : C

Le V est classiquement le premier nerf atteint avec le VIII.

Le volume tumoral calculé est de (2x2x2) cm3. Quelle(s) complication(s) post-opératoire(s) doit-on redouter ?
A - Syndrome cérébelleux gauche
B - Hémiplégie droite
C - Paralysie facial droite
D - Paralysie du trapèze droit
E - Kératite droite
Bonne(s) réponse(s) : C

C - Par atteinte du VII


E - Par atteinte de V

875
Exclusivement sur DOC - DZ : www.doc-dz.com NADJI 85
RESIDANAT EN POCHE TOME II
Cas Clinique en QCM

Quelle(s) autre(s) tumeur(s) est (sont) susceptible(s) de survenir chez ce malade avec une fréquence
significative ?
A - Un neurinome du Vlll gauche
B - Un neurinome rachidien
C - Un méningiome intracrânien
D - Un médulloblastome
E - Un glioblastome
Bonne(s) réponse(s) : A B C E

Dans les phacomatoses et notamment la maladie de Recklinghausen les méningiomes, tumeurs gliales, altérations osseuses,
voire tumeurs endocriniennes (phéochromocytome) surviennent assez fréquemment.

Un garçon de 10 ans présente depuis la veille au soir une fièvre élevée, des céphalées et des vomissements. Le seul
traitement donné est de l'aspirine qui ne calme ni la fièvre ni les douleurs. Il est amené en urgence à l'hôpital où on retrouve
un syndrome méningé et quelques tâches purpuriques sur le thorax.
La tension artérielle est à 10/6, la gorge est rouge. La ponction lombaire faite immédiatement montre 2 500 éléments avec
90% de PN une albuminorachie 3 0 90g/l, une glycorache à 1 1 mmoles/l. Il n'y a pas de germes au gram. Le ionogramme
montre un sodium à 136, un chlore à 103, un potasslum à 4 1, une réserve alcaline à 17. La glycémie est a 52 mmoles/l.
la numération formule sanguine montre 4 500 000 GR, 23 000 leucocytes avec 75 de PN. Les plaquenes sont à 2 50 000.

Quel est le diagnostic le plus probable ?


A - Méningite à hémophilus
B - Méningite tuberculeuse
C - Méningite cérébro-spinale
D - Méningite à pneumocoque
E - Méningite purulente décapitée
Bonne(s) réponse(s) : C

Le terrain, les taches purpuriques, les résultats du LCR permettent d'évoquer ce diagnostic.

Quel(s) examen(s) peut(vent) aider à la progression du diagnostic :


A - Recherche des antigènes solubles dans les urines
B - Culture du LCR
C - Dosage de la CRP
D - Prélèvement de gorge
E - Sérodiagnostic
Bonne(s) réponse(s) : A B D

C Ne montre qu'un syndrome inflammatoire


D Le germe peut être retrouvé dans les prélèvements

Dans les heures et jours suivants vous devez surveiller l'évolution de :


A - La température
B - Le nombre de pétéchies
C - La tension artérielle
D - La leucocytose
E - Le ionogramme sanguin
Bonne(s) réponse(s) : A B C E

B C - Par crainte du purpura fulminans


E - Risque d'hyponatrémie par sécrétion inappropriée d'ADH.

Le traitement initial comporte :


A - Perfusion 3 litres/m2
B - Un traitement antibiotique IV
C - Une injection d hydrocortisone
D - Un traitement héparinique
E - Un traitement par le valium
Bonne(s) réponse(s) : B

A - Quantité trop importante dans ce cas. Aucune précision sur la nature du liquide
C D - Non indiqués dans ce cas éventuellement en cas, de purpura fulminans
E - Enfant trop âgé pour une prévention systématique des crises convulsives.

876
Exclusivement sur DOC - DZ : www.doc-dz.com NADJI 85
RESIDANAT EN POCHE TOME II
Cas Clinique en QCM

Le traitement antibiotique initial est composé de :


A - Ampicilline 200 mg/kg/24h et Gentamycine 3mg/kg/24h
B - Céfotaxime 200 mg/kg/24h et Amikacine 15 mg/kg/24h
C - Céfotaxime 200 mg/kg/24h
D - Ampicilline 200 mg/kg/24h
E - Chloramphénicol injectable 200 mg/kg/24h
Bonne(s) réponse(s) : D

Une monothérapie suffit sur la notion d'âge et la forte suspiscion du méningocoque. En cas d'allergie, le chloramphénicol peut
être utilisé mais à la dose de 50 à 75 mg/kg/24 h

Monsieur X âgé de 60 ans consulte pour une faiblesse diffuse associée à des crampes nocturnes des mollets. L'examen
neurologique met en évidence :
- Un déficit musculaire global plus marqué aux extrémités et aux membres supérieurs
- Une atrophie des muscles surtout marquée aux mains
- Des réflexes ostéo-tendineux vifs polycinétiques diffusés des réflèxes cutanés plantaires en flexion des réflexes abdominaux
abolis
- Aucun trouble sensitif tant superficiel que profond
- A l'étage céphalique quelques troubles de déglutition une voix nasonnée avec un voile hypotone il n existe pas de rire et
pleurer spasmodiques, la langue est irrégulière creusée, la protraction est difficile, enfin le reflexe massétérin est vif
- Le patient signale des tressautements musculaires non retrouvés lors de l'examen.

La ou les structure(s) nerveuse(s) en cause est (sont) :


A - Faisceau pyramidal
B - Cervelet
C - Corne antérieure
D - Nerfs rachidiens
E - Jonction neuro-musculaire
Bonne(s) réponse(s) : A C

A - ROT vifs polycinétiques diffusés, - réflexes abdominaux abolis


C - Amyotrophie, fasciculations, aspect de la langue (amyotrophie), déficit musculaire.

En faveur d'un syndrome neurogène périphérique, vous retenez :


A - Déficit moteur non dissocié, touchant l'ensemble des motilités
B - Réflexes cutanés abdominaux abolis
C - Atrophie musculaire
D - Absence de troubles sensitifs
E - Réflexes ostéo-tendineux vifs et diffusés
Bonne(s) réponse(s) : A C

Sans commentaire.

Une atteinte de corne antérieure serait éliminée par la présence de :


A - Troubles sensitifs
B - Amyotrophie musculaire
C - Crampes musculaires
D - Existence de fasciculations
E - Vitesses de conduction motrices ralenties
Bonne(s) réponse(s) : A E

Troubles sensitifs et vitesses de conduction ralenties font remettre en cause le diagnostic

Parmi les examens complémentaires, lequel et (ou lesquels) (sont) utile(s) au diagnostic ?
A - Scanner cérébral
B - Dosage vitamine B 12
C - Etude du LCR
D - Electromyographie et vitesse de conduction
E - Potentiels évoqués
Bonne(s) réponse(s) : D

D est utile au diagnostic : atteinte neurogène périphérique avec vitesses de conduction normales. Les autres examens sont
pratiqués au moindre doute pour remettre en cause le diagnostic et retrouver une étiologie curable.

877
Exclusivement sur DOC - DZ : www.doc-dz.com NADJI 85
RESIDANAT EN POCHE TOME II
Cas Clinique en QCM

Parmi les maladies suivantes, quel est le diagnostic que vous envisagez ?
A - Compression médullaire cervicale haute
B - Syringomyélie
C - Maladie de Charcot
D - Sclérose en plaques
E - Neuropathie diabètique
Bonne(s) réponse(s) : C

Envisagé d'emblée devant l'association d'un syndrome pyramidal et d'un syndrome neuro périphérique sans troubles sensitifs.

Un homme de 35 ans est hospitalisé pour syndrome méningé avec fièvre. La ponction lombaire ramène un liquide un peu
trouble contenant des polynucléaires altérés. Ce malade ne présente pas d'antécédent en dehors d'un traumatisme cranio-
facial datant de 4 ans et ayant nécessite une hospitalisation de 7 jours. Le syndrome méningé a été précédé de céphalées
frontales avec obstruction nasale 8 jours avant. Les radiographies de sinus montrent une opacité ethmoïdale, maxillaire et
frontale droites.

L'antécédent de traumatisme fait soupçonner une méningite par brèche dure-mérienne. Quelle ou quelles
localisation(s) anatomique(s) de cette brèche doit-on rechercher ?
A - Rocher
B - Lame criblée
C - Toit ethmoïdal
D - Sinus maxillaire
E - Sinus frontal
Bonne(s) réponse(s) : A B C E

La fracture peut intéresser :


- l'étage antérieur du crâne, paroi postérieure du sinus frontal ou lame criblée de l'éthmoïde
- le rocher
- les parois du sinus sphénoïdal.

S'il s'agit bien d'une méningite d'origine ORL, quel germe sera le plus fréquemment retrouvé ?
A - Staphylocoque
B - Streptocoque
C- Pneumocoque
D - Hemophilus
E - Klebsielle
Bonne(s) réponse(s) : C

Sans commentaire.

Si le bilan radiologique avait été pratique lors du traumatisme quel ou quels signes ont une valeur d'orientation
pour une brèche dure-mérienne ?
A - Opacité ethmoïdale unilatérale
B - Fracture du malaire
C - Fracture des os propres du nez
D - Fracture du toit éthmoïdal
E - Pneumatocèle
Bonne(s) réponse(s) : A D E

Autre signes radiologiques :


- fractures frontales irradiant vers la base
- fracture de la voûte vers la région temporale
- opacité sinus frontal.

Sur quel ou quels signes radiologiques affirme-t-on la brèche dure-mérienne ?


A - Opacité éthmoïdale unilatérale
B - Fracture du malaire
C - Fracture des os propres du nez
D - Fracture du toit ethmoïdal
E - Pneumatocèle
Bonne(s) réponse(s) : E

Outre, le scanner et le transit isotopique du LCR une image aérique spontanée au niveau de l'espace sous arachnoïdien
dessinant les citernes de la base et parfois les cavités ventriculaires, affirme le diagnostic.

878
Exclusivement sur DOC - DZ : www.doc-dz.com NADJI 85
RESIDANAT EN POCHE TOME II
Cas Clinique en QCM

Quel ou quels sont les caractères d'une rhinorrhée cérébrospinale post traumatique ?
A - Apériodique
B - Unilatérale
C - Aqueuse
D - Muqueuse
E - Purulente
Bonne(s) réponse(s) : A B C

La rhinorrhée unilatérale est faite de LCR et survient lors de certaines positions de la tête.

Quel ou quels sont les éléments qui permettront de suspecter fortement l'origine cérébrospinale de la rhinorrhée
?
A - Présence de mucus
B - Présence de germes
C - Présence d'électrolytes
D - Présence de glucose
E - Présence d'urée
Bonne(s) réponse(s) : D

Sans commentaire.

Le traitement médical a guéri la méningite. Quel ou quels autre(s) traitement (s) doit-on proposer au malade ?
A - Surveillance clinique
B - Ponctions lombaires décompressives
C - Traitement médical de la sinusite
D - Fermeture de la brèche dure-mérienne
E - Antibiothérapie au long cours
Bonne(s) réponse(s) : A C D

D - Seul traitement permettant d'éviter les récidives.

Mme X... 30 ans consulte pour la répétition 2 à 4 fois par mois de crises de céphalées apparues dans l'enfance. Elles ont
augmenté récemment de fréquence, depuis la fin d'une grossesse normale, pendant laquelle elles avaient disparu. Les crises
durent 4 à 12 heures - intéressant la région fronto - temporo orbitaire.
Presque toutes les crises se terminent par des vomissements.

Le diagnostic qui doit être évoqué sur ce récit est :


A - Migraine commune
B - Migraine ophtalmique
C - Maladie de Horton
D - Algie vasculaire de la face
E - Méningiome de l'étage antérieur
Bonne(s) réponse(s) : A

En sa faveur, le terrain, le type de céphalée, sa durée, l'amélioration pendant la grossesse.

L'examen clinique que vous allez pratiquer montrera probablement :


A - L'absence d'anomalie
B - Une hémianopsie latérale hononyme
C - Une stase papillaire au fond d'oeil
D - Un syndrome pyramidal bilatéral
E - Quelques éléments d'un syndrome frontal
Bonne(s) réponse(s) : A

Sans commentaire.

879
Exclusivement sur DOC - DZ : www.doc-dz.com NADJI 85
RESIDANAT EN POCHE TOME II
Cas Clinique en QCM

Il est indispensable de pratiquer :


A - Un E E G
B - Une artériographie carotidienne
C - Un examen tomodensitométrique cérébral
D - Des radiographies du crâne
E - Aucun des examens ci-dessus
Bonne(s) réponse(s) : E

Dans les migraines typiques où l'examen clinique est normal (par définition) il est inutile de pratiquer des examens
complémentaires. En cas d'atypies l'examen le plus indicatif est le scanner.

Quelle est la fréquence, dans la population, de l'affection dont souffre cette patiente ?
A - 0,1 %
B-1%
C - 10 %
D - 35 %
E - 55 %
Bonne(s) réponse(s) : C

A prédominance féminine.

Le traitement de la crise fait appel en première intention à :


A - La carbamazépine
B - La dihydroergotamine
C - Le méthysérgide (Désernil®)
D - La chlorpromazine (Largactil®)
E - Certains Bêta-bloquants (Avlocardyl®)
Bonne(s) réponse(s) : B

C'est le traitement le plus efficace de la céphalée migraineuse.


On peut utiliser également l'aspirine et les AINS.

Le traitement de fond de l'affection peut faire appel à :


A - La carbamazépine
B - La dihydroergotamine
C - Le méthysergide (Désernil®)
D - La chlorpromazine (Largactil®)
E - Certains Beta-bloquants (Avlocardyl®)
Bonne(s) réponse(s) : B C E

Dans les formes peu sévères : dihydroergotamine, pizotifène oxetérone


Dans les formes plus graves : propanolol
Lorsqu'il existe un syndrome anxiodépressif : amitryptiline
Après de nombreux essais sans résultat : méthysergide

Mr J... 32 ans a souffert depuis plusieurs années de céphalées, évoluant en crises durant environ une demi journée, parfois
suivies de nausées ou de vomissements. Il consulte pour la survenue de secousses musculaires au niveau de l'épaule droite
se propageant au bras. à l'avant bras et à la main. Ces secousses durent 1 à 2 minutes et sont suivies d'une gène
fonctionnelle pendant environ une demi-heure. Il n'y a jamais eu de perte de connaissance. L'examen clinique est
parfaitement normal.

A quoi correspond cet épisode neurologique :


A - Une ischémie cérébrale transitoire
B - Une crise de migraine accompagnée
C - Une crise d'épilepsie partielle
D - Une crise d'épilepsie partielle, secondairement généralisée
E - Une hypertension intracrânienne
Bonne(s) réponse(s) : C

C'est une cure partielle à symptomatologie élémentaire de type bravais jackonsienne.

880
Exclusivement sur DOC - DZ : www.doc-dz.com NADJI 85
RESIDANAT EN POCHE TOME II
Cas Clinique en QCM

Quel argument permettrait de progresser vers le diagnostic ?


A - Un doppler des vaisseaux du cou montrant une élévation bilatérale des index de résistance
B - Un E.C.G. normal
C - Un E.E.G. montrant des pointes-ondes rolandiques gauches
D - Une rétinopathie hypertensive stade I au fond d'oeil.
E - Une radiographie du crâne normale
Bonne(s) réponse(s) : C

L'EEG trouve ici sa meilleure indication.


Il a une grande valeur, s'il montre des pointes ondes dans la région corticale correspondant au territoire intéressé.

Quels sont les 2 diagnostics étiologiques les plus probables :


A - Une ischémie sylvienne gauche par embolie
B - Un hématome intra-hémisphérique gauche
C - Une tumeur intra-hémisphérique gauche
D - Une hémorragie méningée
E - Un angiome hémisphérique gauche
Bonne(s) réponse(s) : C E

Les tumeurs représentent 10% des étiologies des épilepsies et 30% des étiologies des cures partielles.
Un angiome se manifeste 1 fois sur 3 par une crise épileptique.
A D - Examen clinique normal
B - Terrain, absence d'autres signes.

La confirmation de ces hypothèses étiologiques fait appel à deux examens paracliniques parmi les suivants :
A - Une ponction lombaire
B - Un scanner cérébral
C - Une angiographie digitalisée
D - Une angiographie carotidienne gauche
E - Une angiographie vertébrale gauche
Bonne(s) réponse(s) : B D

Sans commentaire.

Une femme de 38 ans consulte pour des douleurs hemi-crâniennes très intenses de survenue récente. Elle n'a pas d
antécédent notable en particulier pas de maladie migraineuse familiale ou personnelle : cependant la patiente a déjà présenté
des accès douloureux identiques pendant une quinzaine de jours il y a six mois, accès ayant spontanément disparu. Cette
patiente ne prend habituellement aucun traitement et elle n'a pas été soulagée de ses douleurs par plusieurs prises d'Aspirine.
Les douleurs évoluent par accès quotidiens, qui durent une à deux heures avec un horaire nocturne assez régulier (vers 22 h).
Elles siègent toujours du coté gauche et, pendant la crise.la patiente présente un oeil rouge qui pleure et une rhinorrhée.
L'intensité de la douleur et l'insomnie qu'elle entraîne perturbent l'activité professionnelle de la patiente qui souhaite à tout prix
un traitement. L'examen neurologique est strictement normal : il n'y a pas d'altération de l'état général. Vous pouvez le
diagnostic d'algies vasculaires de la face sans facteur déclanchant reconnu. Le traitement de la crise repose habituellement
sur le tanrate d'ergotamine (Gynergène®) administré par voie orale, voire sous-cutanée dans les formes les plus sévères.

Parmi les situations suivantes, la (ou les)quelle(s) constitue(nt) une contre indication à ce traitement de
Gynergène® ?
A - Maladie asthmatique
B - Insuffisance coronarienne
C - Hypotension orthostatique
D - Syndrome dépressif
E - Grossesse
Bonne(s) réponse(s) : B E

Contre-indications :
- Grossesse et allaitement
- Enfant de moins de 10 ans
- Artérites, syndrome de Raynaud
- Insuffisance coronarienne, HTA
- Etats infecteux sévères
- Hyperthyoïdie
- Atteintes rénales ou hépatiques
- Associations avec macrolides et alcool.

881
Exclusivement sur DOC - DZ : www.doc-dz.com NADJI 85
RESIDANAT EN POCHE TOME II
Cas Clinique en QCM

Quelle est la complication principale à redouter d'un traitement par le Gynergène® ?


A- Hypoglycémie
B - Bronchospasme
C - Ischémie aigüe des extrémités
D - Rétention aigüe d'urines
E - Crise de glaucome aigu
Bonne(s) réponse(s) : C

Sans commentaires

Votre patiente utilise habituellement de l'Optalidon®, antalgique pyrazolé contenant de la noramidopyrine. Quel
est l'effet indésirable grave auquel expose ce médicament ?
A - Hépatite grave
B - Agranulocytose
C - Coagulation intravasculaire disséminée
D - Insuffisance rénale aigüe
E - Insuffisance coronarienne aigüe
Bonne(s) réponse(s) : B

Agranulocytoses immuno-allergiques.

Vous envisagez un traitement de fond par la dihydroergotamine (Ikaran®, Séglor®) : quel risque particulier
provoque la prescription simultanée de macrolides (TAO®, Erythromycine®) ?
A - Hépatite aigüe
B - Bronchospasme
C - Accident allergique aigu
D - Ischémie aigüe des membres
E - Néphropathie extra-membraneuse
Bonne(s) réponse(s) : D

Sans commentaire.

Le méthysérgide (Désernil®) est un antagoniste de la sérotonine qui peut être utilisé dans les formes rebelles
d'algies vasculaires de la face : quel est le risque grave auquel expose la prise chronique de ce médicament ?
A - Hépatite aigüe
B - Agranulocytose
C - Méthémoglobinémie
D - Néphropathie interstitielle
E - Fibrose rétropéritonéale
Bonne(s) réponse(s) : E

Parfois fibroses pleuropulmonaires et cardiaques pouvant régresser à l'arrêt du traitement.


Fenêtres thérapeutiques d'un mois tous les semestres ou trimestres (en fonction des écoles) UIV annuelle.

Devant une gène à la mastication, s'accentuant au cours des repas. on a porté chez Mr R 28 ans le diagnostic de myasthénie
Malgré une thymectomie les troubles musculaires persistent et le patient va être traité médicalement et régulièrement surveillé.

Dans la myasthénie l'atteinte musculaire la plus évocatrice siège habituellement au niveau :


A - Des sphincters striés
B - De l'oesophage
C - Des petits muscles thénariens
D - De la musculature oculaire extrinsèque
E - De la musculature oculaire intrinsèque
Bonne(s) réponse(s) : D

La musculature intrinsèque est respectée, il peut exister une faiblesse sphinctérienne mais de traduction clinique
exceptionnelle.

882
Exclusivement sur DOC - DZ : www.doc-dz.com NADJI 85
RESIDANAT EN POCHE TOME II
Cas Clinique en QCM

L'atteinte musculaire est caractérisée par :


A - L'existence d'une amyotrophie rapidement observable
B - L'existence d'une induration douloureuse des muscles
C - La présence d'une myotonie au geste volontaire
D - La variabilité du déficit musculaire
E - L amélioration du déficit par le repos
Bonne(s) réponse(s) : D E

Variabilité avec augmentation à l'effort et en fin de journée et amélioration par le repos.

Le traitement médicamenteux de la myasthénie peut faire appel à :


A - Des anticholinestérasiques
B - Des corticoïdes
C - Des immunosuppresseurs
D - Des vasodilatateurs artériolaires
E - Des anabolisants protidiques
Bonne(s) réponse(s) : A B C

Plus la thymectomie et après échec de toutes ces mesures, dans les formes graves les plasmaphérèses.

Le traitement médical adapté permet de prévenir la survenue éventuelle de :


A - Rétraction musculo-tendineuse
B - Cataracte précoce
C - Détérioration intellectuelle
D - Insuffisance respiratoire aigüe
E - Cardiomyopathie
Bonne(s) réponse(s) : D

Le traitement médical fait appel ici aux anticholinestérasiques. Le risque de surdosage impose une surveillance pour prévenir
les complications respiratoires (ainsi que la prise de médicaments contre-indiqués).

Une femme de 65 ans, droitière homogène présente brutalement et simultanément une amaurose totale de l'oeil gauche, des
paresthésies de la main et de l'hémiface droites. des difficultés du langage.
En 20 minutes, tout rentre dans l'ordre. L'examen neurologique, 2 heures plus tard, est normal.

L'ischémie transitoire a intéressé :


A - Le territoire sylvien droit
B - Le territoire carotidien gauche
C - Le territoire carotidien droit
D - Le territoire cérébral postérieur gauche
E - Le territoire cérébral postérieur droit
Bonne(s) réponse(s) : B

L'association amaurose oeil gauche et paresthésies brachiofaciales droites trouble du langage, signe le diagnostic
topographique.

Quelle est l'étiologie qui doit être évoquée en premier lieu ?


A - Artérite de Horton
B - Dyplasie fibro-musculaire de artères cervicales
C - Arythmie complète par fibrillation auriculaire
D - Dissection carotidienne
E - Plaque athéromateuse carotidienne
Bonne(s) réponse(s) : E

QCM difficile car complément simple. L'ACFA est évoqué en premier car peut bénéficier immédiatement et de façon
incontestable des anticoagulants. Mais E est la cause la plus fréquente des troubles "simultanés" de ce genre.

883
Exclusivement sur DOC - DZ : www.doc-dz.com NADJI 85
RESIDANAT EN POCHE TOME II
Cas Clinique en QCM

Quels sont les 2 gestes cliniques indispensables au diagnostic étiologique ?


A - L'auscultation cardiaque
B - L'auscultation de la boite crânienne
C - L'auscultation des vaisseaux du cou
D - L'auscultation pulmonaire
E - La palpation du creux axillaire
Bonne(s) réponse(s) : A C

A - Recherche une arythmie


C - Recherche un souffle vasculaire

Quels sont les 2 examens complémentaires non invasifs indispensables au diagnostic ?


A - E.C.G
B - Scanner cérébral
C - Examen Doppler
D - Fond d'oeil
E - Capillaroscopie
Bonne(s) réponse(s) : A C

A - Recherche une arythmie


B - Sera très probablement normal (AIT)
C - Recherche d'un ralentissement du flux carotidien

48 heures plus tard, un scanner cérébral sans injection de produit de contraste est réalisé. Le ou les aspects
suivants peuvent être observés :
A - Une zone hyperdense thalamo sous-corticale droite
B - Examen normal
C - Une zone hypodense pariétale droite
D - Une zone hypodense pariétale gauche
E - Un déplacement des éléments médians
Bonne(s) réponse(s) : B

48 h après un AIT ayant duré 20 minutes, le scanner est normal.

Un malade de 35 ans présente depuis trois semaines une gêne de la marche, qui comporte une sensation de fatigue
progressive l'obligeant à s'arrêter au bout d'une centaine de mètres Depuis la même époque, il ressent des besoins impérieux
et fréquents d'uriner, il présente des sensations de picotements initialement dans les pieds qui sont remontés progressivement
jusqu'aux genoux. L'examen clinique confirme le trouble de la marche qui est difficile, manque de souplesse. Il existe au
niveau des membres inférieurs une hyperréflectivité diffusante. un signe de Babinski bilatéral, un trouble de la reconnaissance
des positions segmentaires des orteils et de la perception du diapason Le reste de l'examen général et neurologique est
normal.

Parmi les signes énumérés dans le texte, seraient(ait) compatible(s) avec le diagnostic de sclérose latérale
amyotrophique :
A - Fatigabilité
B - Paresthésies des membres inférieurs
C - Troubles sphinctériens
D - Hyperréflectivité des membres Inférieurs
E - Troubles de la perception du diapason
Bonne(s) réponse(s) : D

Car syndrome pyramidal.

Ajoutés à ceux de la présentation, quel(s) élément(s) peu(ven)t plaider fortement en faveur du diagnostic de
sclérose en plaques
A - Notion d'un trouble visuel unilatéral régressif deux ans plus tôt
B - Abolition des réflexes du membre supérieur
C - Anesthésie en selle
D - Crise d épilepsie généralisée il y a quatre ans
E - Surdité progressive unilatérale de transmission
Bonne(s) réponse(s) : A

A - Très évocateur
B - Exceptionnel dans une SEP
C - Atteinte de la queue de cheval
D - 3 à 5 % d'épilepsie dans une SEP
E - Mécanisme non central.

884
Exclusivement sur DOC - DZ : www.doc-dz.com NADJI 85
RESIDANAT EN POCHE TOME II
Cas Clinique en QCM

Dans l'hypothèse où la scène clinique initialement décrite correspond à une sclérose en plaques, ce diagnostic
pourrait être conforté par :
A - Radiographie de la colonne dorsale
B - Potentiels évoqués visuels
C - Scanner cérébral
D - Electromyogramme
E - Electrophorèse des protéines sanguines
Bonne(s) réponse(s) : B C

A - Ne montre pas de signe en rapport avec une SEP.


B - Peut montrer une atteinte infraclinique très évocatrice
C _ En montrant une "plaque de démyélisnisation" dans la substance blanche = hypodensité rehaussée par le produit de
contraste si récente.
D - Inutile si suspiscion de SEP
E - Electrophorèse des protéines du LCR.

Lequel ou lesquels des résultats suivants de l'examen du L C R est (sont) compatible(s) avec le diagnostic de
SEP ?
A - Eléments blancs : 7/ml; protéines 0,20 g/l
B - Rapport immunoglobulines/protéines (pourcentage des gammaglobulines) : 7%
C - Rapport immunoglobulines/protéines (pourcentage des gammaglobulines) : 15%
D - Eléments blancs 1 0/ml ; protéinorachie : 2,6 g/l, IgG/protéines : 24%
E - Eléments blancs : 150/ml, protéines : 150 g/l : laG : 22%
Bonne(s) réponse(s) : A B C

Nombre de cellules normal : 1/3


Elévation modérée de 2 à 10/ml des lymphocites : 2/3
Protéinorachie normale : 2/5
modéremment augmentée 0,6 à 0,8 g/l : 3/5
Gammaglobulines du LCR normal : 1/4
augmentées (20 à 30%) : 3/4 avec pic oligoclonal

La sémiologie indiquée dans le texte initial comporte :


A - Une atteinte pyramidale
B - Une atteinte cérébelleuse
C - Une atteinte des voies lemniscales
D - Une atteinte de la voie spinothalamique
E - Des manifestations compatibles avec une lésion focale
Bonne(s) réponse(s) : A C E

Sans commentaire.

La sémiologie est compatible avec :


A - Une atteinte de la queue de cheval
B - Une polyradiculonévrite
C - Une atteinte de la moelle dorsale
D - Une lésion hémisphérique unilatérale
E - Une atteinte cérébelleuse
Bonne(s) réponse(s) : A

Sans commentaire.

Quelle est la proposition la plus évocatrice d'une compression médullaire :


A - Protéines : 0,37 g/l : Eléments blancs inférieurs à 2 g/l : IgG : 15%
B - Protéines : 1,20 g/l : de 2 éléments blancs/ml : IgG :15%
C - Protéines : 0,40 g/l : 10 éléments blancs/ml : IgG 10%
D - Protéines : 0,80 g/l : 22 éléments blancs/ml : IgG : 15%
E - Protéines : 0,20 g/l : de 2 éléments blancs/ml : IgG : 7%
Bonne(s) réponse(s) : B

Sans commentaire.

885
Exclusivement sur DOC - DZ : www.doc-dz.com NADJI 85
RESIDANAT EN POCHE TOME II
Cas Clinique en QCM
Une femme âgée de 53 ans vient consulter pour des douleurs affectant les
membres inférieurs et évoluant depuis plus d'un an. Il s'agit de sensations
douloureuses, parfois à type de brulures, parfois à type de tiraillements,
particulièrement vives au niveau de la plante des pieds mais affectant aussi
les jambes et les cuisses. Ces sensations douloureuses, bien que supportables, sont très génantes, d'autant qu'elles
s'exagèrent lors de la marche. Au début la malade se souvient que le coté gauche était plus particulièrement intéressé mais
progressivement les troubles se sont bilatéralisés. La malade signale aussi qu'elle tombe souvent, en particulier si le terrain
est un peu accidenté ou si elle est bousculée. Elle redoute tout particulièrement de marcher dans la nature, en terrain
caillouteux. Elle éprouve une grande fatigabilité des membres inférieurs et ne peut plus faire de marches prolongées.
La patiente est mariée et a eu 4 enfants agés respectivement de 30 ans, 25 ans, 23 ans et 18 ans. Elle n'exerce pas d'activité
professionnelle
régulière, mais continue d'exercer à domicile de façon interminente son ancien métier de couturière Elle précise qu'elle n'est
pas génée dans cette activité.
Dans ses antécédents. on note une cholécystectomie pour lithiase biliaire, il y a 8 ans. Elle est ménopausée depuis 2 ans.
En fin d'examen la malade signale que depuis 2 ans environ elle éprouve
quelques difficultés pour retenir ses urines. Ce symptôme génant a d'ailleurs déjà donné lieu à une consultation qui aurait
abouti à retenir
les antécédents obstétricaux dans la génèse des troubles urinaires.
Depuis le début de ses douleurs. la patiente a absorbé sans succès divers antalgiques. Elle se souvient d'un traitement par du
Glifanan (Glafénine®) pendant quelques semaines. Elle est actuellement traitée par du Témesta (Lorazepam®) à la dose de 1
comprimé à 1 mg 3 fois par jour.

Parmi les antécédents neurologiques suivants, vous devez particulièrement rechercher :


A - Accidents ischémiques transitoires
B - Troubles visuels régressifs
C - Troubles de l'équilibre régressifs
D - Céphalées migraineuses
E - Convulsions dans l'enfance
Bonne(s) réponse(s) : A B C

Sans commentaire.

Parmi les signes d'examen suivants, vous devez particulièrement mettre en évidence :
A - Hémianopsie latérale homonyme
B - Signe de Babinski bilatéral
C - Troubles sensitifs objectifs des membres inférieurs I
D - Pieds creux
E - Hypertonie musculaire au niveau des membres inférieurs
Bonne(s) réponse(s) : B C E

Il s'agit d'une compression medullaire lente. Il faut rechercher des signes pyramidaux et sensitifs profonds et superficiels.

Si une scannographie cérébrale est pratiquée on peut s'attendre à observer :


A - Une atrophie cérébrale diffuse
B - Une dilatation des ventricules
C - Un aspect d hématome sous-dural bilatéral
D - Une hyperdensité frontale droite
E - L'absence d'anomalie
Bonne(s) réponse(s) : E

Dans le cadre d'une compression médullaire, la TDM est normale.

Devant une telle séméiologie il est licite de :


A - Prescrire un traitement antalgique et surveiller l'évolution pendant 3 mois
B - Prescrire une corticothérapie brève
C - Orienter vers une consultation urologique
D - Pratiquer des examens radiologiques de façon rapide
E - Pratiquer certains examens biologiques
Bonne(s) réponse(s) : D

La myélographie après radiographies du rachis est à prescrire de façon rapide.

886
Exclusivement sur DOC - DZ : www.doc-dz.com NADJI 85
RESIDANAT EN POCHE TOME II
Cas Clinique en QCM

Parmi les étiologies suivantes laquelle vous semble plausible ?


A - Affection dysmétabolique
B - Myasthénie
C - Affection tumorale
D - Affection dégénérative
E - Infection
Bonne(s) réponse(s) : C

Les douleurs existent au repos et s'accentuent à la marche. L'atteinte médullaire existe. Une étiologie tumorale et plausible.

Parmi les examens suivants lequel vous semble le plus pertinent ?


A - Tomodensitométrie cérébrale
B - Angiographie vertébrale
C - Tomodensitométrie médiastinale
D - Myélographie
E - Electromyogramme
Bonne(s) réponse(s) : D

Sans commentaires

L'existence de troubles urinaires vous semble plutôt :


A - Réléver d'une pathologie indépendante
B - Témoigner d'une participation du système nerveux périphérique
C - Témoigner d'une atteinte hémisphérique cérébrale bilatérale
D - Devoir être rapportée au même ensemble séméiologique que les troubles moteurs et sensitifs
E - Etre spécifique de la femme dans le cadre de l'affection suspectée
Bonne(s) réponse(s) : D

Sans commentaires

Le risque évolutif spontané vous semble être :


A - L'extension du déficit à la musculature respiratoire
B - L'apparition de troubles mentaux
C - L'apparition de crises épileptiques
D - L'aggravation du déficit moteur et sensitif
E - L'apparition de signes d'atteinte bulbaire
Bonne(s) réponse(s) : D

La myélographie est urgente à la recherche d'une cause "curable" c'est-à-dire pouvant à défaut de faire disparaître les
troubles, arrêter leur aggravation.

Un homme de 30 ans est découvert dans un coma de stade 1. Les circonstances et les antécédents ne sont pas connus : il
présente les traces d'un traumatisme cranio-facial. Il y a une raideur de nuque, le liquide cephalorachidien est hémorragique. Il
reprend conscience quelques heures plus tard en se plaignant de fortes céphalées : il n'y a pas de déficit neurologique. Les
radiographies de crâne sont normales.

A ce stade quel est ou quels sont les diagnostics étiologiques vraiment plausibles ?
A - Hématome extra-dural
B - Hémorragie méningée post traumatique
C - Coma barbiturique
D - Hémorragie méningée suivie d'une chute
E - Hémorragie cérébro-méningée
Bonne(s) réponse(s) : B D E

L'hémorragie peut être secondaire à sa chute ou il peut s'agir de la rupture d'un anévrysme avec chute secondaire.

887
Exclusivement sur DOC - DZ : www.doc-dz.com NADJI 85
RESIDANAT EN POCHE TOME II
Cas Clinique en QCM

Pour établir le diagnostic étiologique vous allez demander :


A - Scanographie encéphalique
B - Bilan artériographique cérébral complet
C - Angiographie des vaisseaux du cou
D - E.E.G
E - Encéphalographie gazeuse fractionnée
Bonne(s) réponse(s) : A B

A - Est actuellement systématique et de première intention.


B - L'artériographie est pratiquée en commençant classiquement du côté de la céphalée, avec avis neurochirurgical.

En scanographie, une hémorragie sous arachnoïdienne se traduit par :


A - Une prise de contraste cisternale après injection d'iode intra-veineuse
B - Un effet de -Masse-
C - Une hypodensité spontanée des espaces sous arachnoïdiens
D - Une hyperdensité spontanée des espaces sous arachnoïdiens
E - Aucun des signes précédents
Bonne(s) réponse(s) : D

Sans commentaire.

Dans l'hypothèse d'une hémorragie méningée anévrismale, le pronostic des 15 jours à venir est grévé par :
A - Hydrocéphalie à pression normale
B - Méningite
C - Resaignement
D - Spasme artériel cérebral
E - CIVD (coagulation intravasculaire disséminé)
Bonne(s) réponse(s) : C D

Classiquement le resaignement est le risque majeur ce qui impose une intervention "rapide" après bilan artériographique. Le
risque de l'intervention n'est pas négligeable dans les premiers jours du fait du spasme artériel. L'intervention est idéalement
pratiquée au 6e, 7e jour (risque lié au spasme plus faible risque de resaignement de plus en plus important).

Une femme de 45 ans sans antécédents ni facteurs connus de risque vasculaire est hospitalisée à la suite d'une hémiparésie
droite survenue brutalement la veille, précédée d'une céphalée brutale, mais supportable. L'examen d'entrée objective un
déficit modéré sensitivo-moteur à prédominance facio-brachiale de l'hémicorps droit avec une aphasie marquée par un
manque du, mot, une fluence réduite et des difficultés articulatoires, La vigilance est tout à fait normale. La tension artérielle
est à 17/10 aux 2 bras. L'auscultation cardiaque est normale Il n'existe pas de souffle cervical. Biologiquement, on retient
l'absence de signes inflammatoires, bilan lipidique normal, B,W. négatif.
Le scanner crânien réalisé le premier jour d'hospitalisation est normal (avant et après injection iodée).

Quelle(s) hypothèse(s) diagnostique(s) envisagez-vous ?


A - Embolie d'origine cardiaque
B - Hémorragie méningée pure
C - Lacune de l'hypertendue
D - Hématome sous-dural
E - Thrombose artérielle
Bonne(s) réponse(s) : A E

La normalité du scanner affirme le diagnostic d'accident ischémique. La lacune de l'hypertendue est doublement écartée
- sur le terrain : âge jeune
- sur la présence d'une aphasie.

Dans l'hypothèse d'une ischémie cérébrale, quel territoire vasculaire peut être incriminé à l'origine du déficit ?
A - Sylvien profond
B - Cérébral antérieur
C - Sylvien total
D - Sylvien superficiel
E - Cérébrale postérieure
Bonne(s) réponse(s) : D

L'accident sylvien profond ou total donne une hémiplégie proportionnelle ; l'accident cérébral antérieur donne une hémiplégie
à prédominance crural.
Il n'y a pas d'hémiplégie dans les cérébrales postérieures.

888
Exclusivement sur DOC - DZ : www.doc-dz.com NADJI 85
RESIDANAT EN POCHE TOME II
Cas Clinique en QCM

La normalité du scanner
A - Elimine l'hypothèse d'un ramollissement cérébral
B - Elimine l'hypothèse d'une hémorragie cérébrale
C - Elimine l'hypothèse d'un hématome sous-dural
D - Autorise la prescription d'anticoagulants
E - Elimine l'hypothèse d'une tumeur cérébrale
Bonne(s) réponse(s) : B C D

La normalité du TDM dans ce cas autorise la mise sous anticoagulant (d'intérêt discuté dans cet accident constitué).
L'hypothèse d'une tumeur cérébrale est écartée par le bon sens, dans ce cas, et non par le scanner ; exceptionnellement des
astrocytomes de bas grades, de petite taille, isodenses peuvent échapper au TDM.

Quel est le facteur de risque indiscutable favorisant ce type d'accident


vasculaire cérébral :
A - Sédentarité
B - Obésité
C -.Hypercholestérolémie
D - Hypertension artérielle
E- Tabagisme
Bonne(s) réponse(s) : D

En effet l'HTA est, avec l'âge, le facteur de risque le plus important, et de loin, d'accident vasculaire.

Madame S,.., 54 ans, c'est adressée en neurologie pour bilan de sensation de faiblesse, associée à des troubles sensitifs
subjectifs du membre inférieur droit.
On retrouve l'existence de troubles ayant débuté il y a 1 an par une sensation de faiblesse du membre inférieur droit avec
impression de jambe lourde, disparaissant au bout de deux mois. 5 ans auparavant cette malade avait présenté une baisse
d'acuité visuelle à droite régressive en quelques semaines.

Actuellement : - Faiblesse du membre Inférieur droit après un effort et augmentée le soir, associée de façon inconstante à des
douleurs à type de crampes du mollet droit,pouvant durer 2 à 3 heures malgré le repos, ne la réveillant pas la nuit : - Parfois,
quelques épisodes de sensation de "jambe de bois" avec perte de la sensibilité du membre inférieur droit ; - Depuis 2 mois,
des troubles sphinctériens se sont surajoutés, à type de besoins impérieux et 2 à 3 épisodes d'incontinence urinaire. A
l'examen on note un discret déficit moteur du membre inférieur droit avec chute de la jambe droite, après quelques secondes,
à la manoeuvre de Mingazzini. Des réflexes ostéo-tendineux très vifs aux 4 membres, un signe de Babinski bilatéral, une
abolition des réflexes cutanés abdominaux, une déviation latérale droite à la manoeuvre de Romberg.
Il n'y a pas de déficit objectif des divers types de sensibilité, mais on note quelques erreurs à la graphesthésie au niveau du
membre inférieur droit.

Ce tableau clinique correspond à :


A - une myopathie
B- Une polynévrite
C - Une polyradiculonévrite
D - Une myélopathie
E - Une myasthénie
Bonne(s) réponse(s) : D

Mais la déviation latérale droite à la manoeuvre de Romberg renvoie à une lésion plus haut située au niveau du tronc.

L'aggravation de la symptomatologie à l'effort chez cette malade est :


A - En faveur d'une myasthénie
B - En faveur d'une atteinte périphérique
C - En faveur d'un facteur vasculaire
D - Non spécifique d'une étiopathogénie
E - En faveur d'un facteur compressif
Bonne(s) réponse(s) : D

Se voit aussi bien dans une compression médullaire qu'une sclérose en plaques.

889
Exclusivement sur DOC - DZ : www.doc-dz.com NADJI 85
RESIDANAT EN POCHE TOME II
Cas Clinique en QCM

Parmi les diagnostics étiologiques évoqués. lequel vous parait le plus plausible ?
A - Compression médullaire
B - Sclérose en plaques
C - Syringomyélie
D - Maladie de Charcot
E - Accident vasculaire de la moelle
Bonne(s) réponse(s) : B

Malgré l'âge un peu tardif (49 ans), le diagnostic de SEP est très hautement probable : sur la notion de signes cliniques
évocateurs évoluant par poussées (2 dans le cas présent), ne pouvant être expliqués par une topographie unique (ici 3
topographies différentes le nerf optique, le tronc cérébral et la moelle).
Donc dissémination dans le temps et dans l'espace.

Parmi les examens suivants. lequel vous paraît le plus utile ?


A - Artériographie médullaire
B - Artériographie des vaisseaux cervicaux
C - Examen tomodensitométrique cérébral
D - Examen tomodensitométrique rachidien
E - Myélographie cervicale à l'Amipaque®
Bonne(s) réponse(s) : E

La présence d'une augmentation des gammaglobulines avec un profil oligoclonal permet dans ce cas de confirmer le
diagnostic.
Moins spécifique est la réaction cellulaire absente ou très modérée (5 à 10 éléments/mm3)
La protéinorachie est normale ou discrètement augmentée (<1g/l).

Monsieur S... âgé de 66 ans, a été hospitalisé il y a une dizaine de jours pour faiblesse généralisée. En fait depuis un an
environ il présente des difficultés de plus en plus grandes à se déplacer, à effectuer les actes de la vie quotidienne, à se
retourner dans son lit, à se laver sans aide et est
devenu totalement dépendant de son entourage.
A l'examen la marche se fait lentement, à petits pas, le buste penche en avant les bras collés au corps et on note une
hypertonie importante des quatre membres avec signe de la roue dentée. Il n'y a pas de déficit moteur, pas d'anomalie des
réflexes, pas d'atteinte des sensibilités ; bon état général ; pas d'atteinte des fonctions supérieures.

L'association d'akinésie et d'hypertonie fait évoquer une maladie de Parkinson


Cependant ce malade ne présente pas de tremblement ; vous en conclue que :
A - Il ne s agit en aucun cas d'une maladie de Parkison.
B - Il s'agit d'une forme akinéto-hypertonique de maladie de Parkinson.
C - Il s'agit d'un syndrome pyramidal
D - Il faut évoquer un syndrome pseudo-bulbaire
E - Aucune proposition n'est exacte.
Bonne(s) réponse(s) : B

Un syndrome pseudobulbaire (dans le cadre d'un état lacunaire de l'hypertendu) pourrait simuler un Parkinson akinéto-
hypertonique mais il existe alors un syndrome pyramidal : or dans ce cas, les réflexes sont notés normaux.

L'hypertonie de la maladie de Parkinson est qualifiée de plastique. Ceci veut dire :


A - Une augmentation du tonus de repos disparaissant au mouvement.

B - Une augmentation du tonus constante pendant la durée du mouvement

C - Une augmentation du tonus se renforçant avec l'amplitude du mouvement D - Une augmentation du tonus
se renforçant avec la répétition du mouvement

D - Une augmentation du tonus se renforçant avec la vitesse de mouvement passif

E - Une augmentation du tonus se renforçant avec la vitesse de mouvement passif


Bonne(s) réponse(s) : B

L'hypertonie spastique (pyramidale) au contraire se renforce avec l'amplitude du mouvement. L'hypertonie oppositionnelle
(frontale) se renforce avec la vitesse du mouvement et disparaît au repos.

890
Exclusivement sur DOC - DZ : www.doc-dz.com NADJI 85
RESIDANAT EN POCHE TOME II
Cas Clinique en QCM

Parmi les propositions suivantes concernant le tremblement de la maladie de Parkinson, laquelle (ou lesquelles)
est (sont) exacte(s) ?
A - Le tremblement augmente à l'émotion.
B - Le tremblement est un tremblement de repos.
C - Le tremblement est souvent à prédominance unilatérale,
D - Le tremblement n'est pas augmente par le calcul mental
E - Le tremblement disparaît pendant le sommeil.
Bonne(s) réponse(s) : A B C E

Le tremblement parkinsonien est le type même du tremblement de repos ; il est augmenté par l'émotion, la concentration
(calcul mental), disparaît pendant le sommeil. La maladie de Parkinson est presque toujours asymétrique, rarement unilatérale.

Comment sont les réflexes ostéotendineux dans la maladie de Parkinson ?


A- Normaux.
B - Augmentés.
C - Diminués.
D - Abolis.
E - Pendulaires.
Bonne(s) réponse(s) : A

Il sont augmentés en cas de syndrome pyramidal, diminués ou abolis en cas d'atteinte périphérique, pendulaires dans
l'hypotonie du syndrome cérébelleux.

Parmi ces situations. Iaquelle (lesquelles) justifient une contre-indication ou de précautions particulières
préalables à la Dopa-thérapie ?
A - Antécédents de délire hallucinatoire
B - Insuffisance respiratoire.
C - Insuffisance cardiaque grave
D - Sujet âgé de plus de 70 ans.
E - Troubles urinaires (prostate),
Bonne(s) réponse(s) : A C D

Les contre-indications principales de la L-dopa sont d'ordre mental (délire hallucinatoire, détérioration intellectuelle) et
cardiaque (en cas d'insuffisance cardiaque ou coronaire en évolution). Pour ces deux raisons, la L-dopa doit être maniée avec
prudence chez le sujet âgé.

Parmi les situations suivantes indiquez celle(s) qui contre-indique(nt) l'utilisation des anti-cholinergiques.
A - Détérioration amnésique
B - Insuffisance cardiaque.
C - Antécédents d'ulcère gastrique
D - Troubles urinaires (prostate).
E - Glaucome.
Bonne(s) réponse(s) : A D E

Le glaucome aigu et hypertrophie prostatique sont les contre-indications classiques ; il faut ajouter la détérioration
intellectuelle et les antécédents de délire chez le parkinsonien. La dénervation cholinergique a été montrée corrélée avec la
démence : la prescription d'anticholinergique sur le terrain ne fait qu'aggraver le dépôt pharmacologique, on explique ainsi la
folie atropinique du parkinsonien détérioré.

Une jeune femme de 24 ans, consulte pour des paresthésies localisées à la région fronto-orbitaire, associées à des
décharges fulgurantes, volontiers déclenchées par le contact des téguments de la région douloureuse, entrecoupées de
périodes de calme complet durant lesquelles la douleur ne peut être provoquée. L'examen ne décèle aucun signe local pas
plus que de signes neurologiques à distance. Par contre l'interrogatoire apprend que la patiente a présenté 18 mois
auparavant une baisse subite de l'acuité visuelle de son oeil droit totalement régressive en quelque semaines. L'examen du
fond d'oeil droit objective un aspect blanc nacré de la papille.

Comment qualifier ces crises douloureuses :


A - Migraine ophtalmique
B - Algie sinusienne
C - Algie vasculaire de la face
D - Névralgie du trijumeau de type essentiel
E - Névralgie du trijumeau de type symptomatique
Bonne(s) réponse(s) : E

Des douleurs dans le territoire du V, fulgurantes, avec trigger-zone, survenant par crises évoquent une hévralgie du V.
Malgré absence de signes neurologiques intercritiques et la notion d'une trigger-zone (plus fréquente dans la névralgie
essentielle), deux éléments doivent faire retenir le diagnostic de névralgie symptomatique : le jeune âge, et surtout
l'antécédent de névrite optique font rapporter l'ensemble à une S.E.P.
891
Exclusivement sur DOC - DZ : www.doc-dz.com NADJI 85
RESIDANAT EN POCHE TOME II
Cas Clinique en QCM

Que signifie l'antécédent visuel ?


A - Névrite optique
B - Glaucome aigu
C - Compression chiasmatique
D - Trouble de l'accommodation
E - Accompagnement migraineux
Bonne(s) réponse(s) : A

Sans commentaire.

Cette situation clinique justifie la demande de :


A - Radiographies du crâne
B - Tomodensitométrie cérébrale
C - Examen du liquide Céphalo-Rachidien
D - Mesure du temps de latence des potentiels évoqués visuels
E - Artériographie carotidienne gauche
Bonne(s) réponse(s) : C D

l'examen du LCR est indispensable.


L'étude des PEV devrait être complétée par PEA, PES. Elle peut être utile pour confirmer le diagnostic de NORB (retard de
l'ordre PIOU) et rechercher des signes infracliniques sur l'autre oeil.
Plus intéressant serait un examen IRM.

Parmi les diagnostics suivants, lequel vous parait le plus approprié


A - Adénome hypophysaire
B - Thrombophlébite du sinus caverneux
C - Sclérose en plaques
D - Sténose de la carotide interne
E - Céphalée vasculaire
Bonne(s) réponse(s) : C

La SEP est révélée dans 30 % des cas par une NORB et dans 1 % des cas par une névralgie du V.

Madame X, 30 ans, mariée, mère d'un enfant, vient consulter pour l'apparition depuis 48 heures et l'aggravation progressive
d'un déficit du membre inférieur droit. La pointe de son pied a tendance à accrocher les marches des escaliers.
A l'examen, les réflexes ostéo-tendineux sont tous vifs et diffusés. Le réflexe rotulien droit est polycinétique. Il existe un signe
de Babinski à droite. La malade signale des paresthésies au niveau de la jambe droite. L'examen des sensibilités
élémentaires montre une diminution de la sensibilité à la douleur au niveau de la jambe droite. Il existe un crochetage à
l'épreuve doigt-nez, yeux ouverts, à gauche.A l'entretien, on apprend qu'elle a présenté il y a 3 ans, une cécité de l'oeil gauche
régressive en 3 semaines.

Le tableau clinique actuel permet de localiser les lésions au niveau :


A - Du faisceau pyramidal croise partant du cortex gauche
B - Du nerf sciatique poplité externe droit
C - Du cordon postérieur droit de la moelle
D - De la voie cérébelleuse
E - Du faisceau spino-thalamique
Bonne(s) réponse(s) :

QUESTION ANNULEE.

Vous concluez que l'ensemble des lésions neurologiques présentées par cette patiente est :
A - Unifocal évoluant en une poussée
B - Multifocal évoluant en une poussée
C - Unifocal évoluant en plusieurs poussées
D - Multifocal évoluant en plusieurs poussées
E - Diffus et progressif
Bonne(s) réponse(s) : D

Il y a deux poussées et deux localisations au minimum différentes (nerf optique et tronc cérébral ou tronc central + cérébral).

892
Exclusivement sur DOC - DZ : www.doc-dz.com NADJI 85
RESIDANAT EN POCHE TOME II
Cas Clinique en QCM

Parmi les explorations suivantes, quelle(s) est (sont) celle(s) qui aurai(ent) une forte probabilité d'être anormales
?
A - Electro-encéphalogramme
B - Electromyogramme
C - Vitesse de conduction motrice périphérique
D - Vitesse de conduction sensitive périphérique
E - Potentiels évoqués visuels
Bonne(s) réponse(s) : E

Le PEV gauche sera très certainement altéré (retard de l'ordre P 100).

Le diagnostic correct est compatible avec un examen du L.C.R. indiquant :


A - Normalité
B - Hyper-lymphocytorachie à 10/mm3
C - Transsudat non inflammatoire
D - Hyper-gamma globulinorachie oligo-clonale
E - Hyper-protéinorachie 1,5 g/I
Bonne(s) réponse(s) : A B C D

La protéinorachie est normale ou discrètement augmentée (exceptionnellement supérieur à 1g/l).


Le LCR peut être normal (A-C compatible, B-D très évocateur).

Quelle(s) thérapeutique(s) pouvez-vous envisager ?


A - Corticoïdes
B - Immuno-dépresseurs
C - Oxygène hyperbare
D - Aspirine
E - Antibiotiques
Bonne(s) réponse(s) : A

Devant une poussée, les corticoïdes sont toujours indiqués.


L'inefficacité totale de l'O2 hyperbare est démontrée ; quant aux immunosuppresseurs leur effet est discuté, ils ne sont jamais
indiqués pour traiter une poussée, mais pour tenter de prévenir l'aggravation des poussées évolutives.

Une femme de 50 ans est amenée aux urgences car elle a fait une perte de connaissance qui d'après son mari aurait duré
une dizaine de minutes et pendant laquelle elle aurait eu la tête et les yeux tournés vers la gauche A l'interrogatoire, on
apprend :
1 - Qu'elle se plaint depuis 3 ou 4 ans de céphalées d'abord modérées, volontiers le matin, qui se sont aggravées petit à petit
au point d'être pratiquement permanentes depuis 6 mois.
2 - Que depuis environ 1 an son caractère et son comportement ont change avec des phases de tristesse et de repli sur soi
alternant avec des phases ou elle.avait paru à son entourage un peu "excitée", entreprenant de nombreuses choses sans les
mener à bien, faisant quelques achats inconsidérés
3 -. Qu'elle a eu un malaise analogue six mois auparavant, et que les deux fois qu'elle s'est rendu compte que sa tête et ses
yeux tournaient malgré elle vers la gauche avant de perdre conscience.
A l'examen, la patiente, bien que se plaignant de céphalées parait un peu joviale et agitée. Elle.parle beaucoup et passe
volontiers "du coq à l'âne". Elle a du mal à fixer son attention sans cependant qu'il y ait de troubles de la mémoire ou de
l'orientation temporo-spatiale.
On note par ailleurs des ROT exagérés à gauche et un signe de Babinski gauche.

Quelle est la cause de la perte de conscience ?


A - Hémorragie intratumorale à répétition
B - Hypertension intracrânienne paroxystique
C - Embolie d'origine cardiaque
D - Crise d'épilepsie généralisée primaire
E - Crise d'épilepsie partielle secondairement généralisée
Bonne(s) réponse(s) : E

Il s'agit d'une crise adversive caractéristique d'une décharge au niveau de l'aire motrice supplémentaire (région prémotrice
dans le lobe frontal).

893
Exclusivement sur DOC - DZ : www.doc-dz.com NADJI 85
RESIDANAT EN POCHE TOME II
Cas Clinique en QCM

Quelle est la localisation du processus pathologique ?


A - Temporal droit
B - Frontal gauche
C - Occipital gauche
D - Frontal droit
E - Thalamus droit
Bonne(s) Réponse(s) : D

Le caractère droite ou gauche de la crise adversive ne permet pas de prédire le côté de la lésion.
Seul le syndrome pyramidal gauche permet donc d'affirmer que la lésion est droite.

Parmi les lésions suivantes, quelle(s) est(sont) celle(s) qui vous parai(ssen)t la(les) plus vraisemblable(s) en
fonction de sa(leur) fréquence ?
A - Métastase
B - Astrocytome de bas grade
C - Astroscytome de bas grade
D - Méningiome
E - Lymphome
Bonne(s) Réponse(s) : B D

La lenteur de l'évolution (4 ans) élimine à priori un processus expansif à croissance rapide (métastase, lymphome, abcès...)

Parmi les traitements suivants, quel est celui qu'il y a lieu de prévoir après l'intervention en tout état de cause ?
A - Radiothérapie
B - Chimiothérapie anti-tumorale
C - Traitement anticonvulsivant
D - Corticothérapie
E - Abstention thérapeutique
Bonne(s) Réponse(s) : C

Indiqués après toute intervention neurochirurgicale touchant le cortex, et d'autant plus que le patient a présenté 2 crises
comitiales.

Une femme de 65 ans sans antécédents particuliers est suivie depuis 4 mois pour une gonalgie droite ; malgré le traitement
instauré elle a de plus en plus de mal à marcher et elle consulte un rhumatologue qui, à l'examen, outre les signes locaux
d'arthrose du genou, trouve des réflexes ostéotendineux plus vifs au MI droit avec un signe de Babinski. Par ailleurs existent
des troubles sensitifs : à droite des troubles de la sensibilité profonde au niveau du MI et à gauche, remontant jusqu'en D10
métamère, des troubles de la sensibilité superficielle. La démarche est spatique à droite. Il s'agit donc vraisemblablement
d'une compression médullaire.

A quel niveau vertébral siège le plus probablement cette compression médullaire ?


A - D8
B - D9
C - D10
D - D11
E - D12
Bonne(s) réponse(s) : A

Il y a un décalage de 2 entre niveau vertébral et niveau métamérique dans la région dorsale.


En outre la limite supérieure des troubles sensitifs ne rejoint pas toujours le niveau lésionnel : la lésion pourrait donc être au-
dessus de D8, elle ne peut pas être en dessous de D8.

Par rapport à la moelle, la compression médullaire est :


A - Antérieure
B - Latérale droite
C - Latérale gauche
D - Postérieure
E - Intramédullaire
Bonne(s) réponse(s) : B

Il s'agit d'un syndrome de Brow-Sequard : syndrome pyramidal et cordonnal postérieur homolatéral et "syndrome"
spinothalamique latéral postérieur à la lésion.

894
Exclusivement sur DOC - DZ : www.doc-dz.com NADJI 85
RESIDANAT EN POCHE TOME II
Cas Clinique en QCM

Etant donné le contexte clinique, vous pensez que la compression médullaire est due à :
A - Une épidurite métastatique
B - Une épidurite infectieuse
C - Un méningiome dorsal
D - Une hernie discale
E - Un canal lombaire étroit
Bonne(s) réponse(s) : C

C'est la cause la plus fréquente de compression médullaire dorsale à cet âge.

Pour préciser le diagnostic étiologique et topographique vous allez demander :


A- IRM médullaire
B - Electromyogramme
C - Potentiels évoqués sensitifs
D - Scanner dorsal
E - Myélographie dorsale
Bonne(s) réponse(s) : A

Question litigieuse :
L'IRM est l'examen de choix dans une telle situation, nécessaire et suffisant pour le neurologue et le neurochirurgien
la myélographie dorsale éventuellement couplée au scanner dorsal s'impose si l'on ne peut obtenir d'IRM.

Un homme de 53 ans consulte pour l'apparition récente de sensations vertigineuses avec tendance au déséquilibre vers la
droite. Il ne s'agit pas d'un sujet hypertendu. Il rapporte que depuis au moins 3 ans, peut-être plus, il entend des sifflements
dans son oreille droite et que depuis très longtemps il a pris l'habitude d'écouter le téléphone avec son oreille gauche. Il ne
s'est jamais plaint de la moindre céphalée.
L'examen met en évidence une surdité totale de l'oreille droite, une déviation des index vers la droite, une déviation, à la
marche yeux fermés, vers la droite, une hypoesthésie cornéenne droite.
Le revêtement cutané comporte de nombreux "molluscum pendulum" et des taches café au lait éparses, dont deux très
grandes (supérieures à 10 cm2) sur le dos.

Où se situe la lésion responsable de la symptomatologie ?


A - Dans l'angle ponto-cérébelleux droit
B - Au niveau du conduit auditif interne gauche
C - Au niveau de l'hémisphère cérébelleux droit
D - Au niveau du tronc cérébral
E - Au niveau du tronc basilaire
Bonne(s) réponse(s) : A

Un syndrome cochléovestibulaire droit avec atteinte du V droit revient à l'angle ponto-cérébelleux.


Au niveau du CAI, l'atteinte du VIII aurait été isolée.

Les examens cliniques et paracliniques permettent de penser à un neurinome chez un malade porteur d'une
phacomatose. Comment expliquez-vous l'hypoesthésie cornéenne ?
A - La tumeur comprime le nerf facial dans le conduit auditif
B - La tumeur comprime le tronc cérébral
C - La tumeur comprime le nerf trijumeau vers le haut
D - La tumeur se développe aux dépens du ganglion de Gasser
E - Le volume tumoral entraîne une paralysie du III
Bonne(s) réponse(s) : C

Sans commentaire.

Le volume tumoral calculé est de (2 x 2 x 2) cm3. Quelle(s) complication(s) postopératoire(s) doit-on redouter ?
A - Syndrome cérébelleux gauche
B - Hémiplégie droite
C - Paralysie faciale droite
D - Paralysie du trapèze droit
E - Kératite droite
Bonne(s) réponse(s) : C E

La paralysie faciale droite (la kératite n'est qu'une conséquence l'inocclusion palbébrale) est une complication fréquente de la
chirurgie du neurinome du VIII dans l'angle (le VII et le VIII sont très proches à cet endroit).

895
Exclusivement sur DOC - DZ : www.doc-dz.com NADJI 85
RESIDANAT EN POCHE TOME II
Cas Clinique en QCM

Quelle(s) autre(s) tumeur(s) est(sont) susceptible(s) de survenir chez ce malade, avec une fréquence
significative ?
A - Un neurinome du VIII gauche
B - Un neurinome rachidien
C - Un méningiome intracrânien
D - Un médulloblastome
E - Un épendymome
Bonne(s) réponse(s) : A B C

Il s'agit d'une neurofibromatose (de type central) : le tableau le plus caractéristique est le double neurinome du VIII, on
rencontre aussi fréquemment des neurinomes rachidiens, méningiomes multiples, gliome du nerf optique.

Monsieur X., âgé de 42 ans, sans antécédent pathologique notable, est admis pour vertige. Brutalement au réveil, encore
couche, il présente un grand vertige rotatoire horaire avec manifestations végétatives marquées et est hospitalisé quelques
heures plus tard. A son admission, il n'y a pas de trouble de la conscience : l'examen neurologique montre un syndrome
vestibulaire avec nystagmus horizontal gauche et forte latérodéviation droite à la manoeuvre des bras tendus et à celle de
Romberg, une anesthésie thermo-douloureuse de l'hémicorps gauche respectant la face, une hypoesthésie faciale droite, une
paralysie de l'hémivoile droit et de la corde vocale droite, une hypermétrie et une adiadococinésie du membre supérieur droit,
un signe de Stewart-Holmes et un signe de Claude-Bernard-Horner droit. l'examens somatique et cardiovasculaire sont
normaux.

Quel est le niveau lésionnel ?


A - Capsule interne
B - Pied du pédoncule
C - Thalamique
D - Médioprotubérantiel
E - Latérobulbaire
Bonne(s) réponse(s) : E

Un tel tableau est pathognomonique d'infarctus bulbaire rétro-olivaire (ou de la fossette latérale du bulbe, ou syndrome de
Wallenberg). Il est parfois associé à un ramollissement de la PICA.

L'histoire clinique est plutôt en faveur :


A - D'un processus expansif malin
B - D'un processus expansif bénin
C - D'une sclérose en plaques
D - D'un accident vasculaire de nature ischémique
E - D'un accident vasculaire de nature hémorragique
Bonne(s) réponse(s) : D

Sans commentaire.

Vous concluez à :
A - Syndrome de Weber
B - Syndrome de Wallenberg
C - Syndrome de Milliard Gubler
D - Syndrome de Fossile
E - Syndrome de Parinaud
Bonne(s) réponse(s) : B

Sans commentaire.

Quel examen vous paraît indispensable lors des deux premiers jours ?
A - Tomodensitométrie
B - Artériographie carotidienne
C - Examen du LCR
D - Doppler des vaisseaux cervicaux
E - Un examen cochléovestibulaire
Bonne(s) réponse(s) : D

Le caractère ischémique étant pratiquement assuré devant un tel tableau, le TDM n'a pas d'autre intérêt que de rechercher un
ramollissement du cervelet associé.
Le doppler avecécho à la recherche de lésions vertébrales (dysplasie dissection) s'impose chez ce patient jeune, sans
facteurs de risque vasculaire.

896
Exclusivement sur DOC - DZ : www.doc-dz.com NADJI 85
RESIDANAT EN POCHE TOME II
Cas Clinique en QCM

Parmi les signes suivants, le(s)quel(s) ne fait(font) pas partie du syndrome cérébelleux ?
A - Signe de Stewart-Holmes
B - Signe de Claude-Bernard-Horner
C - Adiadococinésie
D - Paralysie de la corde vocale
E - Hypermétrie
Bonne(s) réponse(s) : B D

Sans commentaire.

Une femme de 44 ans consulte pour l'apparition récente et progressive de céphalées. Celles-ci sont sévères, quasi
permanentes, insomniantes, plus marquées le matin. Vous craignez une hypertension intracrânienne.

Vous poursuivez l'interrogatoire. Quel(s) signe(s) fonctionnel(s) pourrai(en)t renforcer cette conviction ?
A - Coliques abdominales
B - Vomissements répétés
C - Diarrhée profuse
D - Diplopie intermittente
E - Tendance à la somnolence diurne
Bonne(s) réponse(s) : B D E

Céphalée, vomissements, et troubles visuels (diplopie et oedème papillaire) sont les signes principaux d'HIC.
Les troubles de la conscience sont un signe de gravité.

L'examen clinique peut orienter vers ce diagnostic. Par quel(s) signe(s) ?


A - Météorisme abdominal
B - Signe de Murphy
C - Paralysie du VI
D - Bradycardie
E - Aréflexie pupillaire à la stimulation lumineuse
Bonne(s) réponse(s) : C D E

La diplopie est en général due à une atteinte du VI plus rarement du III, une aréflexie traduit dans ce contexte une atteinte du
III.
La tachycardie est un signe tardif et inconstant d'HIC.
Ppour mémoire : le signe de Murphy se rapporte à la colique hépatique....

L'examen du fond d'oeil reste le temps essentiel du diagnostic. Quels sont les signes retrouvés au cours d'une
hypertension intracrânienne d'installation récente ?
A - Aspect blanchâtre de la papille
B - Hémorragies de la région maculaire
C - Effacement des bords de la papille
D - Exsudats péripapillaires
E - Rétrécissements segmentaires des artères rétiniennes
Bonne(s) réponse(s) : B C D

L'aspect blanchâtre est un signe plus tardif en général et s'accentue au fur et à mesure de l'évolution. Il ne paraît pas donc
devoir figurer dans les signes précoces.

Dans la liste ci-dessous, quels sont les médicaments utiles au traitement de l'hypertension intracrânienne ?
A - Antalgiques morphiniques
B - Corticostéroïdes
C - Solutés hyperosmolaires
D - Anti-inflammatoires non stéroïdiens
E - Anticalciques
Bonne(s) réponse(s) : B C

Corticoïdes à fortes doses et soluté hyperosmolaires type Mannitol® sont les principaux traitements d'urgence de l'HIC.

897
Exclusivement sur DOC - DZ : www.doc-dz.com NADJI 85
RESIDANAT EN POCHE TOME II
Cas Clinique en QCM

La présence d'un syndrome cérébello-vestibulaire central attire l'attention sur une tumeur de la fosse
postérieure. Parmi les signes ci-dessous quels sont ceux qui ont permis d'authentifier ce syndrome ?
A - Abolition de la sensibilité vibratoire des membres inférieurs
B - Ataxie axiale
C - Nystagmus vertical
D - Nystagmus des regards latéraux
E - Dysmétrie des membres supérieurs
Bonne(s) réponse(s) : B C D E

Dysmétrie des membres supérieurs et ataxie axiale traduisent un syndrome cérébelleux. Le nystagmus horizontal traduit en
général une lésion protubérantielle, et le nystagmus vertical une lésion pédonculaire.

Un homme de 43 ans consulte pour une paralysie faciale droite remarquée ce matin au réveil et précédée depuis 2 jours
d'otalgies droites.
Dans ses antécédents, on note :
- une otite bilatérale de l'enfance, ayant guéri sans séquelles apparentes
- un traumatisme crânien il y a 8 ans, avec coma de quelques heures et fracture occipitale. Il a été suivi pendant 3 à 4 ans
pour des céphalées et vertiges résiduels.
L'examen clinique montre une altération de l'état général avec températures à 38.5.
Le visage est asymétrique au repos ; le sillon nasogénien droit et les rides frontales droites sont effacés. Cette asymétrie
s'accentue à la mimique. Lorsqu'on lui demande de fermer les yeux, l'oeil droit reste ouvert et le globe oculaire se porte en
haut et en dehors. L'examen de l'oreille droite montre une rougeur et des croûtelles jaunâtres de la conque ; le conduit auditif
externe est inflammatoire ; le tympan est hyperkémique dans son ensemble ; ses reliefs sont conservés; il n'est pas bombant.

Quel(s) est(sont) parmi les signes suivants, celui(ceux) qui est(sont) caractéristique(s) de la topographie
périphérique de cette paralysie ?
A - Effacement du sillon nasogénien
B - Effacement des rides du front
C - A la tentative de fermeture des yeux, l'oeil se porte en haut et en dehors
D - L'oeil ne se ferme pas
E - Le visage est asymétrique
Bonne(s) réponse(s) : B C D

Question un peu ambiguë = la PF périphérique touche également le facial supérieur et le facial inférieur contrairement à la PF
centrale qui prédomine sur le facial inférieur. Dans ce dernier cas l'effacement des rides du front est très discret, l'oeil se ferme
en général avec un signe des cils de Souque.
En fait, le signe réellement pathognomonique d'une atteinte périphérique est le signe de Charles Bell : inocclusion palpébrale
+ déviation de la pupille en haut et en dehors.

Quelle(s) est(sont) parmi les altérations suivantes celle(s) que vous allez rechercher pour affirmer le siège
intrapétreux de cette paralysie ?
A - Trouble du goût
B - Trouble de la sécrétion lacrymale
C - Trouble de la sécrétion salivaire parotidienne
D - Atteinte de la sensibilité de l'hémiface
E - Altérations à l'électromyographie de la face
Bonne(s) réponse(s) : A B

Le VII bis ou nerf intermédiaire de Wrisberg accompagne le VII et émet plusieurs branches dans le rocher : la corde du
tympan est la plus importante et véhicule les sensations gustatives des 2/3 antérieurs de la langue ; un contingent
somesthésique innerve la zone de Ramsay Hunt (tympan, CAE, conque), un contingent viscéromoteur assure l'innervation
lacrymale et salivaire à l'exception de la parotide (IX) ; la sensibilité de la face est assurée par le V.
L'EMG a un intérêt évolutif et pronostic.

Quelle(s) est(sont) parmi les propositions suivantes concernant les éléments de cette observation, celle(s) qui
va(vont) vous permettre de porter un jugement sur la gravité de cette paralysie et le pronostic fonctionnel ?
A - Otalgie
B - Température à 38.5
C - Nature périphérique de la paralysie
D - Siège intrapétreux de la paralysie
E - Aucun de ces éléments
Bonne(s) réponse(s) : E

Sans commentaire.

898
Exclusivement sur DOC - DZ : www.doc-dz.com NADJI 85
RESIDANAT EN POCHE TOME II
Cas Clinique en QCM

Quel diagnostic étiologique retenez-vous à l'origine de cette paralysie faciale ?


A - Paralysie a frigore (ou de Bell)
B - Otite moyenne aiguë
C - Cholestéatome de l'oreille
D - Complication traumatique tardive
E - Zona auriculaire
Bonne(s) réponse(s) :

QUESTION ANNULEE.

Quel a été l'élément, parmi les suivants, qui vous a permis ce diagnostic ?
A - Otalgies
B - Tympan hyperhémique
C - Croûtelles du pavillon
D - Antécédents traumatiques
E - Antécédents auriculaires
Bonne(s) réponse(s) :

QUESTION ANNULEE.

Monsieur C..., chauffeur de taxi, 92 kg pour 1,70 m. Le 7 janvier, il présente, au milieu de la matinée, au volant de sa voiture,
une "vision trouble" de l'oeil gauche, et un engourdissement de la main droite. Il ne peut désserrer son frein à main. Cela n'a
dure que quelques secondes, et a totalement régressé. Le lendemain, en montant les escaliers, il a eu de nouveau un trouble
de son oeil gauche, puis il est tombé : sa jambe droite ne le portait plus, sa main droite était inerte et sa bouche de travers. Il
dit n'avoir ressenti aucune insensibilité, ni aucune paresthésie et que son champ visuel ne lui a pas paru amputé. Arrivé chez
lui, à sa femme qui s'inquiétait, il a répondu en bafouillant des mots mal compréhensibles.
A l'arrivée à l'accueil, l'examen neurologique était normal. L'examen cardiovasculaire est normal. La T.A. est à 15/9, le pouls à
70. La palpation des carotides est normale. Le F.O. montre des signes nets d'athérosclérose, avec engainement des artères
et signe du croisement. L'auscultation des vaisseaux du cou est normale. L'électrocardiogramme montre un rythme sinusal et
l'absence de troubles de la repolarisation.

Au cours de ces deux épisodes, une atteinte lésionnelle est probable au niveau de :
A - La radiation optique ou scissure calcarine
B - Le nerf optique et rétine gauche
C - La capsule interne gauche
D - Le thalamus gauche
E - La racine C8-D1 droite
Bonne(s) réponse(s) : B C

Il s'agit d'un syndrome cortico-pyramidal : association d'une ischémie de l'artère ophtalmique et l'artère sylvienne, toutes deux
branches de la carotide interne. L'existence d'un déficit proportionnel renvoie à une ischémie de la capsule interne dans le
territoire sylvien profond.

Quel diagnostic évoquez-vous ?


A - Migraine
B - Sclérose en plaques
C - Epilepsie
D - Accident ischémique transitoire
E - Hyperglycémie
Bonne(s) réponse(s) : D

C'est à dire régressif complètement en moins de 24 heures.

Quel mécanisme physiopathologique suspectez-vous chez ce patient ?


A - Embolie d'origine cardiaque
B - Sténose athéromateuse du siphon carotidien gauche
C - Sténose athéromateuse de la bifurcation carotidienne gauche au cou
D - Angiome cérébral
E - Spasme vasculaire
Bonne(s) réponse(s) : C

L'artère ophtalmique naît en amont du siphon carotidien : un syndrome opticopyramidal ne peut donc pas être expliqué par
une sténose du sylvien. Un embole d'origine cardiaque est très improbable devant 2 AIT aussi stéréotypés.

899
Exclusivement sur DOC - DZ : www.doc-dz.com NADJI 85
RESIDANAT EN POCHE TOME II
Cas Clinique en QCM

Quels sont les examens que vous pouvez envisager pour en apporter la preuve directe ?
A - Examen ultra-sonique des vaisseaux du cou
B - Electro-encéphalogramme
C - Angiographie des vaisseaux du cou
D - Scanner
E - Enregistrement pendant 24 heures de l'électrocardiogramme
Bonne(s) réponse(s) : A C

Sans commentaire.

Le scanner X étant normal, le traitement peut faire appel à :


A - Aspirine
B - Héparine
C - Endartériectomie
D - Alcaloïde de l'ergot
E - Spasmolytiques
Bonne(s) réponse(s) : A B C

A discuter...en fonction du degré de sténose essentiellement.

Mme B...52 ans est admise en neurologie à la demande de son généraliste pour l'installation en quelques minutes, de
"troubles du langage".
A l'examen d'entrée, 3 heures après le début, elle ne comprend pas les ordres simples et son langage spontané est un jargon
incompréhensible. Dans la soirée, 3 heures plus tard, alors que son langage est presque normal (quelques paraphasies) on
peut constater une hémianopsie latérale homonyme droite qui aura disparu totalement le lendemain. Deux épisodes
analogues, de troubles du langage - durant moins d'une heure, se sont produits 10 et 15 jours plus tôt.
L'examen clinique ne montre par ailleurs qu'une obésité (84 kg pour 1m,60) et une HTA à 180/120

Les troubles du langage observés chez cette patiente pendant l'épisode actuel réalisent :
A - Un mutisme
B - Une dysarthrie
C - Une aphasie de Broca
D - Une aphasie de Wernicke
E - Une confusion mentale
Bonne(s) réponse(s) : D

Caractérisée par un jargon fluent, et des troubles de la compréhension (contrairement au Broca qui est une aphasie
d'expression non lfuente, avec peu ou pas de trouble de la compréhension).

Devant ce tableau clinique quelle est votre proposition diagnostique ?


A - Lacune de la capsule interne droite
B - Hémorragie cérébrale à minima
C - Accident ischémique sylvien droit
D - Accident ischémique du territoire carotidien gauche
E - Crise épileptique
Bonne(s) Réponse(s) : D

Accident ischémique du territoire postérieur de l'artère sylvienne gauche, pariétotemporale, intéressant la zone de Wernicke et
les radiations optiques d'où l'HLH droite.

Pour éclairer spécifiquement le mécanisme physiopathologique de ces accidents neurologiques on peut utiliser :
A - Scintigraphie cérébrale
B - Electro-encéphalogramme
C - Electrocardiogramme
D - Radiographie pulmonaire
E - Vélocimétrie doppler
Bonne(s) réponse(s) : C E

Devant ces accidents ischémiques transitoires on recherche en premier lieu un point de départ cardiaque ou carotidien.

900
Exclusivement sur DOC - DZ : www.doc-dz.com NADJI 85
RESIDANAT EN POCHE TOME II
Cas Clinique en QCM

L'examen tomodensitométrique réalisé en urgence est normal. Avant de poursuivre des examens plus détaillés,
quel(s) traitement(s) utile(s) pouvez-vous prescrire ?
A - Anti-hypertenseurs
B - Anti agrégants
C - Antivitamines K
D - Vasodilatateurs
E - Fibrinolytiques
Bonne(s) réponse(s) : A B

Les anticoagulants se discutent sous forme d'héparine après contrôle de la tension artérielle.

Un homme de 40 ans sans aucun antécédent particulier présente brutalement une céphalée généralisée. Il vomit plusieurs
fois. Il est adressé à l'hôpital. Une heure plus tard la céphalée persiste et à l'examen on retrouve un syndrome méningé, un
ptosis de la paupière supérieure droite et une mydriase droite, seule l'excursion en dehors du globe oculaire droit est possible.
La température est à 37,8°C, le pouls à 75, la pression artérielle à 130/80.
Le diagnostic évoqué est celui d'hémorragie méningée.

La ponction lombaire :
A - Peut confirmer le diagnostic
B - Est indispensable
C - Peut être normale
D - Sera faite si le scanner ne montre rien d'anormal
E - Peut montrer une simple réaction lymphocytaire
Bonne(s) réponse(s) : A D

La PL n'est plus systématique, le scanner cérébral étant l'examen à pratiquer en première intention. En revanche si le scanner
est normal (20 % des cas), la PL est indiquée.
La PL est toujours hémorragique précocement puis survient une réaction lympho-monocytaire avant normalisation du liquide.

Le scanner pratiqué en urgence :


A - Peut être normal
B - Evite de pratiquer l'angiographie
C - Peut montrer une hémorragie méningée au niveau du polygone de Willis
D - Peut montrer une hémorragie méningée localisée au voisinage de l'apophyse clinoïde antérieure droite
E - Doit être refait 48 heures plus tard, s'il est normal
Bonne(s) réponse(s) : A C D

Le scanner est normal dans 20 % des cas. Il impose dans ce cas de pratiquer une PL, et non de refaire un scanner.
L'artériographie est le meilleur examen pour mettre en évidence l'anévrysme, la présence de sang est à rechercher au niveau
des citernes de la base, la région du polygone, les vallées sylviennes, l'échancrure interhémisphérique.

Au plan sémiologique l'existence d'un ptosis de la paupière supérieure droite, d'une mydriase droite, d'une
paralysie du globe oculaire droit en haut en dedans et en bas, évoque :
A - Une atteinte du II droit
B - Une atteinte du chiasma
C - Une atteinte du pédoncule cérébral gauche
D - Une atteinte du III droit
E - Une atteinte du VI droit
Bonne(s) réponse(s) : D

Le III innerve le droit supérieur, le droit inférieur, le droit interne, le petit oblique, le releveur de la paupière supérieure, l'iris, et
le muscle ciliaire.

Vous évoquez a priori une rupture d'anévrisme intracrânien. Où se situe-t-il ?


A - Artère communicante antérieure
B - Artère cérébrale antérieure droite
C - Artère sylvienne droite à la jonction de ses deux premiers segments
D - Terminaison carotidienne droite au voisinage de la communicante postérieure
E - Artère vertébrale droite
Bonne(s) réponse(s) : D

L'atteinte du III évoque une rupture d'anévrysme carotidien supraclinoïdien à développement postérieur.

901
Exclusivement sur DOC - DZ : www.doc-dz.com NADJI 85
RESIDANAT EN POCHE TOME II
Cas Clinique en QCM

L'angiographie cérébrale doit :


A - Explorer seulement le territoire carotidien droit
B - Explorer seulement le territoire vertébrobasilaire
C - Explorer seulement le territoire carotidien gauche
D - Explorer tous les axes artériels cérébraux
E - Etre réalisée en urgence
Bonne(s) réponse(s) : D E

Il faut explorer tous les axes à la recherche d'anévrysmes multiples.


En dessous de 60 ans, l'hémorragie méningée constitue une urgence neurochirurgicale : l'angiographie est l'examen
préopératoire indispensable ; la date de l'intervention est discutée.

Quelles sont la ou les complication(s) qui menace(nt) dans la première semaine le malade s'il n'est pas traité ?
A - Infection méningée
B - Resaignement
C - Ischémie cérébrale par spasme vasculaire
D - Coagulation intravasculaire disséminée
E - Hydrocéphalie basse pression
Bonne(s) réponse(s) : B C

Le risque de resaignement est maximum au 9° jour, le risque de spasme est maximum dans les 10 premiers jours,
l'hydrocéphalie à basse pression constitue une complication plus tardive.
En période aiguë peut survenir une hydrocéphalie aiguë à haute pression.

Une femme de 59 ans présente brutalement, alors qu'elle était au téléphone une céphalée postérieure non accompagnée de
signes digestifs, qui va diminuer d'intensité dans les jours suivants, sans disparaître. Cette patiente est une migraineuse
depuis 30 ans, traitée par dihydroergotamine au long cours.
Cinq jours après l'épisode initial, la malade présente en pleine nuit un mal aise avec perte de connaissance, avec
mouvements cloniques des quatres membres durant 2 minutes environ.
Le médecin traitant constate une raideur de nuque et un ptosis. La malade est alors hospitalisée. La TA est à 110/60 mmHg,
la température est à 38,6 degrés, la patiente est très somnolente ; le syndrome méningé est franc et l'examen neurologique ne
retrouve qu'un ptosis droit associé à un strabisme externe et avec abolition du reflex photomoteur droit.
L'examen tomodensitométrique pratiqué le même jour montre une opacité hématique au niveau de la citerne sus-optique et
des deux vallées sylviennes.
L'évolution clinique après l'hospitalisation est marquée par une amélioration spontanée de la conscience, mais 10 jours après
l'hospitalisation, la patiente sera retrouvée dans le coma avec un déficit hémicorporel gauche, déviation de la tête et des yeux
à droite.

Les signes oculaires sont en faveur de :


A - Une atteinte du nerf moteur oculaire commun droit
B - Une lésion du sympathique péricarotidien droit
C - Une lésion du moteur oculaire externe droit
D - Une lésion protubérantielle gauche
E - Aucune de ces propositions n'est exacte
Bonne(s) réponse(s) : A

Sans commentaire.

Le malaise nocturne ayant motivé l'hospitalisation évoque :


A - Une syncope vasovagale
B - Une manifestation de spasmophilie
C - Une crise d'épilépsie d'allure généralisée d'emblée
D - Des crises toniques postérieures témoignant d'un engagement des amygdales cérébelleuses
E - Une phase prodromique de migraine accompagnée
Bonne(s) réponse(s) : C

Sans commentaire.

902
Exclusivement sur DOC - DZ : www.doc-dz.com NADJI 85
RESIDANAT EN POCHE TOME II
Cas Clinique en QCM

Quel examen complémentaire devait être réalisé devant le tableau initial, au cours de l'hospitalisation ?
A - Une artériographie carotidienne droite
B - Une artériographie carotidienne et vertébrale bilatérale
C - Une ventriculographie
D - Une angiographie digitalisée par voie veineuse
E - Des études bactériologiques et sérologiques du LCR
Bonne(s) réponse(s) : B

Le diagnostic d'hémorragie méningée, probable cliniquement, est confirmé par le scanner. L'association à un III droit évoque
en premier lieu un anévrysme carotidien supraclinoïdien droit, ou de la terminaison du tronc basilaire, d'où la nécessité
d'étudier à la fois le système carotidien et le système vertébrobasilaire ; d'autre part, une exploration des 4 axes à la recherche
d'anévrysmes multiples est en général indiquée.

Parmi les propositions diagnostiques suivantes, laquelle vous paraît la plus probable en fonction des données
cliniques et paracliniques ?
A - Hémorragie méningée
B - Accident vasculaire cérébral ischémique
C - Méningite à méningocoque
D - Syphilis du système nerveux central
E - Tumeur de la région hypothalamique
Bonne(s) réponse(s) : A

Sans commentaire.

Un patient de 48 ans vient consulter pour des douleurs faciales. Celles-ci ont débuté il y a quelques jours. Il s'agit de douleurs
siégeant au niveau de la région orbitaire, très intenses continues lors de l'épisode douloureux. Chaque crise douloureuse
débute assez brutalement et dure une heure environ. La crise se répète deux à trois fois dans la journée. Ce patient est
indemne de passé pathologique et n'a jamais présenté de douleur de ce type. Vous évoquez une possible algie vasculaire de
la face.

Quel(s) signe(s) associé(s) à la douleur devez-vous rechercher lors de l'interrogatoire ?


A - La présence d'une zone gâchette
B - Un larmoiement oculaire du côté de la douleur
C - Un scotome sibillant
D - Un écoulement nasal du côté de la douleur
E - Des vomissements
Bonne(s) réponse(s) : B D

Les signes neurovégétatifs homolatéraux sont très fréquents dans l'algie vasculaire (larmoiements 80 % ; écoulement nasal
30 %).
Scotomes scintillants et vomissements orienteraient vers une migraine.
La présence d'une zone gâchette se voit dans la névralgie du V.

La présence d'un signe de Claude-Bernard-Horner au cours de la crise signifie :


A - La présence d'une lésion intracrânienne
B - Une ischémie associée du III
C - L'irritation sympathique associée à l'algie
D - Une élévation réactionnelle de la pression intra-oculaire
E - Aucune des propositions précédentes
Bonne(s) réponse(s) :

QUESTION ANNULEE.

Quel(s) signe(s) clinique(s) peut-on mettre en évidence au cours de l'accès douloureux ?


A - Hypoesthésie dans le territoire du VI
B - Abolition du reflex cornéen
C - Parésie de l'occlusion palpébrale
D - Souffle rétro-orbitaire
E - Réaction vasomotrice cutanée et conjonctivale
Bonne(s) réponse(s) : E

Sans commentaire.

903
Exclusivement sur DOC - DZ : www.doc-dz.com NADJI 85
RESIDANAT EN POCHE TOME II
Cas Clinique en QCM

L'évolution habituelle d'une algie vasculaire de la face est marquée par :


A - Aggravation par apparition d'une douleur continue
B - Répétition des accès douloureux pendant quelques semaines
C - Evolution vers une migraine typique
D - Répétition de périodes de crises douloureuses séparées par des intervalles de plusieurs mois
E - Une crise douloureuse tous les 2 ou 3 mois
Bonne(s) réponse(s) : B D

Sans commentaire.

Quel(s) moyen(s) thérapeutique(s) peut-on proposer ?


A - Prescription de propranolol (Avlocardyl®)
B - Prescription de carbamazépine (Tégrétol®)
C - Prescription de tartrate d'érgotamine au début de la crise
D - Thermocoagulation du ganglion de Gasser
E - Prescription de méthysergine (Désernil®)
Bonne(s) réponse(s) : A C E

Le seul traitement de la crise est le tartrate d'ergotamine, mais celui-ci n'est généralement actif qu'en parentéral.
Les traitement de fond sont les mêmes que dans la migraine ; le plus actif étant l'Avlocardyl® et le Désernyl®. Plus
récemment, les corticoïdes et le lithium ont été essayés.

Un homme de 37 ans, droitier, sans antécédent, a présenté en 2 mois trois épisodes paroxystiques stéréotypés. Subitement, il
perçoit simultanément trois fragments musicaux : l'un des trois lui est familier mais il ne parvient pas à l'identifier ; les autres lui
paraissent étranges malgré les efforts angoissants qu'il effectue pour les reconnaître.
Parallèlement il éprouve un sentiment d'étrangeté, ne perçoit pas les propos de son entourage et ne peut s'exprimer
verbalement. Son regard est "vague" et il effectue de sa main droite des mouvements de grattage. Le "malaise" s'estompe en
4 minutes et il n'en conserve qu'un souvenir flou.
Entre les 2ème et 3ème épisodes, voulant parler de "politique", il a prononcé le mot "république" et évoquant le chant des
"cigales" il a parlé du chant des "fourmis".
Les traits de son visage se contractent mieux du côté gauche que du côté droit mais l'examen est par ailleurs normal.

Les trois épisodes paroxystiques correspondent à :


A - Un ictus amnésique
B - Une crise Bravais Jacksonienne
C - Une absence petit mal
D - Une crise psychomotrice (crise partielle complexe)
E - Une syncope vagale
Bonne(s) réponse(s) : D

On entend par crise partielle complexe une crise partielle s'accompagnant d'une trouble de la conscience.

Les troubles intervenus entre les 2ème et 3ème épisodes traduisent :


A - Une obnubilation
B - Une confusion postcritique
C - Une crise uncinée
D - Une dysarthrie
E - Une aphasie transitoire
Bonne(s) réponse(s) : E

Sans commentaire.

La localisation lésionnelle est :


A - Le bulbe
B - La protubérance
C - Le lobe frontal gauche
D - Le lobe temporal gauche
E - Le lobe occipital gauche
Bonne(s) réponse(s) : D

Sans commentaire.

904
Exclusivement sur DOC - DZ : www.doc-dz.com NADJI 85
RESIDANAT EN POCHE TOME II
Cas Clinique en QCM

Quel peut être le processus en cause ?


A - Hématome sous dural aigu
B - Infarctus sylvien par embolie
C - Tumeur cérébrale
D - Maladie d'Alzheimer
E - Encéphalite
Bonne(s) réponse(s) : C

La répétition de crises comitiales surtout partielles, avec signes neurologiques (asymétrie faciale) déficitaires à l'examen,
renvoyant vers une même localisation (temporo-rolandique) évoque un processus expansif.

Une femme de 55 ans, droitière, est vue en consultation pour un déficit fonctionnel du bras et de la main droits qui a
totalement disparu en 5 à 6 heures sans laisser la moindre gène. On apprend en l'interrogeant que le début du trouble avait
été brutal.

A l'interrogatoire vous allez probablement retrouver que :


A - Le déficit moteur intéressait également l'autre membre supérieur
B - Pendant cet épisode, l'entourage avait noté une déviation de la bouche
C - Il existait simultanément une difficulté d'élocution
D - La patiente avait présenté des céphalées très violentes et des vomissements immédiatement avant
l'installation de son déficit
E - La patiente avait eu une diplopie transitoire
Bonne(s) réponse(s) : B C

Le diagnostic le plus probable étant l'AIT sylvien gauche (superficiel a priori puisque le déficit du MIG n'a pas marqué la
malade), on recherche une PF centrale et une aphasie associée chez cette droitière.

Une hypothèse diagnostique paraît s'imposer, laquelle ?


A - Sclérose en plaques
B - Tumeur rolandique gauche
C - Accident ischémique transitoire sylvien
D - Hémorragie cérébroméningée
E - Insuffisance vertébrobasilaire
Bonne(s) réponse(s) : C

Cf supra.

Cette hypothèse diagnostique vous amène à rechercher par l'interrogatoire certain(s) éléments
complémentaire(s) le(s)quel(s) ?
A - Hypoacousie gauche
B - Amaurose transitoire de l'oeil gauche
C - Vertiges positionnels fréquents
D - Céphalées matinales calmées par les vomissements
E - Autre épisode identique dans les mois précédents
Bonne(s) réponse(s) : B E

Le syndrome opticopyramidal (ischémie de l'artère ophtalmique et de l'artère sylvienne) est très évocateur d'embole carotidien.
L'existence de nombreux d'épisodes antérieurs a un intérêt diagnostic sur la lésion en cause (les AIT étaient-ils tous du même
côté : plutôt en faveur d'un embol carotidien, ou dans des territoires différents : plutôt en faveur d'un embol cardiaque, et
pronostic sur le risque de récidive.

L'examen peut montrer :


A - Un signe de Babinski bilatéral
B - Une raideur de nuque
C - Un souffle carotidien gauche
D - Un nystagmus
E - L'absence de déficit neurologique
Bonne(s) réponse(s) : A C E

Question embarrassante.
L'examen pourrait montrer des signes témoignant d'accidents vasculaires antérieurs (on ne vous dit rien des ATCD) ; en
particulier un signe de Babinski uni ou bilatéral.

905
Exclusivement sur DOC - DZ : www.doc-dz.com NADJI 85
RESIDANAT EN POCHE TOME II
Cas Clinique en QCM

Le bilan paraclinique comporte des examens essentiels. Parmi les suivants, à ce titre quel(s) est(sont)
celui(ceux) qui vous parai(ssen)t utile(s) ?
A - Ponction lombaire
B - Doppler des vaisseaux du cou
C - Scanner
D - Artériographie
E - Electromyogramme
Bonne(s) réponse(s) :

QUESTION ANNULEE.

Une femme de 65 ans consulte pour des algies faciales gauches :


- celles-ci se produisent depuis trois mois environ
- elles surviennent par crises très aiguës, de très courte durée (quelques secondes)
- ont un caractère très aigu, d'intensité insupportable
- se produisent plusieurs fois par jour et en particulier dès qu'un aliment entre en contact avec la région gingivale supérieure
gauche
- elles affectaient initialement de manière exclusive le domaine du nerf orbitaire gauche mais intéressent maintenant
également le territoire mandibulaire.
Vous évoquez le diagnostic de névralgie du trijumeau.

Une névralgie faciale peut être révélatrice de :


A - Cancer du sinus maxillaire
B - Sclérose en plaques
C - Neurinome du nerf facial
D - Cancer du cavum
E - Tumeur parotidienne
Bonne(s) réponse(s) : A B C D

Toute lésion au contact du noyau du V (intra-axiale) comme dans la SEP, ou au contact de ses branches (neurinome du nerf
facial, cancer du cavum, cancer du sinus maxillaire venant au contact du nerf sous-orbitaire) peut déclencher une névralgie du
V symptomatique. La parotide n'est pas au contact d'une branche du V.

Parmi les signes suivants le(s)quel(s) accompagne(nt) la névralgie essentielle du trijumeau ?


A - Anesthésie cornéenne
B - Anesthésie sous-orbitaire
C - Trismus
D - Hydrorrhée nasale
E - Aucun de ces signes
Bonne(s) réponse(s) : E

Un peu discutable : la névralgie faciale peut s'accompagner d'une congestion de l'oeil ou de la muqueuse nasale, mais
classiquement l'absence de ces signes l'oppose à l'algie vasculaire de la face.
Il n'y a pas de trismus mais une contraction de l'hémiface qui a fait surnommer cette affection tic douloureux de la face.
Par définition l'examen neurologique est normal dans la névralgie essentielle.

L'efficacité d'un traitement par le ou les médicaments suivants aura valeur d'orientation ou de confirmation pour
le diagnostic :
A - Aspirine
B - Morphine
C - Dihydroergotamine
D - Carbamazépine
E - Paracétamol
Bonne(s) réponse(s) : D

L'efficacité Tégrétol® (carbamazépine) apparaît très spécifique de la névralgie du V surtout essentielle ou dans le cadre d'une
SEP.

906
Exclusivement sur DOC - DZ : www.doc-dz.com NADJI 85
RESIDANAT EN POCHE TOME II
Cas Clinique en QCM

Si le traitement médical ne donne pas un résultat suffisamment durable, on peut envisager :


A - Une infiltration par xylocaïne du nerf sous-orbitaire
B - Une alcoolisation du ganglionn de Gasser
C - Une infiltration par la xylocaïne du nerf dentaire inférieur
D - Une section du nerf sous-orbitaire et du nerf dentaire inférieur
E - Une thermocoagulation élective de fibres du trijumeau
Bonne(s) Réponse(s) : E

La thermocoagulation percutanée bloque la conduction des fibres nociceptives et épargne relativement les fibres de gros
calibres. Il s'agit de l'introduction à travers la joue d'une électrode jusque dans le trous ovale. C'est actuellement le traitement
chirurgical le plus utilisé.

Un sujet de 75 ans est traité pour une maladie de parkinson par l'association Levodopa benserazide (Modopar® 250, 1 gélule
à 7 heures, 12 heures et 20 heures) depuis quelques années. Il consulte, car chaque jour vers 11 heures et vers 19 heures,
les signes de la maladie réapparaissent progressivement conduisant à une incapacité fonctionnelle qui dure respectivement
jusqu'à 13 heures et 21 heures environ.

Quelle est la meilleure définition de la fluctuation d'efficacité réalisée dans cette observation ?
A - Akinésie paradoxale
B - Akinésie paroxystique
C - Effet"On-Off"
D - Akinésie circadienne
E - Akinésie de fin de dose
Bonne(s) réponse(s) : E

- L'akinésie de fin de dose apparaît de plus en plus précocement au fur et à mesure de l'évolution de la maladie.
- L'effet On-Off correspond à l'apparation brusque et inatendue de l'akinésie (apparaît après quelques années d'évolution) en
rapport avec un taux de L. DOPA trop bas.
- L'akinésie paradoxale survient en milieu de dose, alors que le taux de L. DOPA est élevé.
- L'akinésie paroxystique survient indépendamment des prises de L. DOPA comme l'akinésie circadienne qui survient toujours
au même moment de la journée.

En première intention, vous considérez comme logique la ou les attitude(s) suivante(s) :


A - Diminution de la dose de dopa
B - Fragmentation de la dose actuelle en 6 prises
C - Adjonction d'un comprimé de 15 mg de trihexyphénidyle (Artane 5®) matin et soir
D - Arrêt du traitement pendant 8 jours
E - Prescription d'un IMAO de type A
Bonne(s) Réponse(s) : B

La fragmentation des doses devrait permettre de maintenir un taux de L. DOPA plus constant, et d'éviter le "sous-dosage" en
fin de dose.

Secondairement si la symptomatologie persiste, on peut envisager utilement :


A - L'adjonction d'agonistes dopaminergiques directs
B - Un traitement antidépresseur à doses modérées par la clomipramine (Anafranil®)
C - Une thalamotomie stéréotaxique
D - L'adjonction de Deprenyl
E - La prescription de L dopa à libération prolongée
Bonne(s) réponse(s) : A C E

Le Deprenyl est un inhibiteur de mono amine oxydose (IMAO), agoniste dopaminergique.

La survenue de fluctuations d'action dans la maladie de Parkinson se situe en moyenne par rapport au début du
traitement au bout de :
A - Quelques mois
B - 2 ans
C - 5 ans
D - 8 ans
E -10 ans
Bonne(s) réponse(s) : C

Sans commentaire.

907
Exclusivement sur DOC - DZ : www.doc-dz.com NADJI 85
RESIDANAT EN POCHE TOME II
Cas Clinique en QCM
Une jeune femme consulte pour une baisse d'acuité visuelle droite, survenue de façon rapidement progressive en deux jours,
accompagnée de douleurs à l'élévation du regard. Dans les antécédents, on note des épisodes de dysesthésies des membres
inférieurs, avec troubles mictionnels, l'absence d'hypertension artérielle et de diabète. L'acuité visuelle droite est de 1/20,
l'acuité visuelle gauche est de 10/10. La mobilité oculaire n'est pas limitée, mais un nystagmus lors du regard latéral extrême
droit est présent. Le fond d'oeil est normal des deux côtés, ainsi que le tonus oculaire. L'hypothèse la plus probable est celle
d'une névrite optique rétrobulbaire droite.

Le terme de névrite optique rétrobulbaire désigne classiquement :


A - Une atteinte inflammatoire du nerf optique quelle que soit la topographie
B - Une atteinte du nerf optique sans anomalies visibles du fond d'oeil
C - Toute neuropathie optique
D - L'atteinte totale du nerf optique dans sa portion orbitaire
E - Une neuropapillite
Bonne(s) réponse(s) : B

Le F.O. est normal (initialement au moins) car le nerf optique est touché dans sa partie retrobulbaire.

Avant l'examen du fond d'oeil la symptomatologie subjective pourrait faire évoquer :


A - L'oblitération de l'artère centrale de la rétine
B - Une papillite
C - Un oedème papillaire de stase
D - Un glaucome aigu
E - Une cataracte
Bonne(s) réponse(s) : B

Une papillite traduit l'atteinte de la partie la plus antérieure du nerf optique (intraorbitaire) : les signes subjectifs sont les
mêmes : seul le F.O. montre un aspect d'oedème de la pupille. En fait 20 % des névrites optiques de la S.E.P. sont en fait des
papillites (névrites optiques antérieures) : il s'agit en fait de la même maladie : l'inflammation du nerf optique, quelque soit le
segment atteint.

Sur quel(s) examen(s) complémentaire(s), en dehors du relevé du champ visuel, va s'étayer le diagnostic
symptomatique de névrite optique rétrobulbaire ?
A - L'angiofluorographie
B - La radiographie
C - L'électrorétinographie
D - L'échographie
E - Les potentiels évoqués visuels
Bonne(s) réponse(s) : E

Les PEV vont montrer :


- une diminution d'amplitude.
- surtout un retard de l'onde P100.

L'étude du champ visuel est réalisée à la recherche :


A - D'une quadranopsie
B - D'une hémianopsie
C - D'un élargissement de la tache aveugle
D - D'un scotome central
E - D'un déficit arciforme
Bonne(s) réponse(s) : D

Ce sont les fibres maculaires qui sont touchées en premier.

Sur quel(s) argument(s) essentiel(s) le diagnostic de sclérose en plaques peut-il être étayé ?
A - La radiographie du canal optique
B - Les antécédents familiaux
C - Les atteintes neurologiques de poussées antérieures ou concomitantes
D - L'hyperpression du liquide céphalo-rachidien
E - L'hypergammaglobulinorachie
Bonne(s) réponse(s) : C E

Sans commentaire.

908
Exclusivement sur DOC - DZ : www.doc-dz.com NADJI 85
RESIDANAT EN POCHE TOME II
Cas Clinique en QCM

La sclérose en plaques n'est pas la seule étiologie à évoquer Quel(s) est(sont) le(s) diagnostic(s) étiologique(s)
compatible(s) avec le tableau présenté ?
A - L'intoxication alcoolo-tabagique
B - Une neuropathie carentielle
C - Une neuropathie optique de Leber
D - Une neuropathie optique type Devic
E - Une arachnoïdite opto-chiasmatique
Bonne(s) réponse(s) : D

Question discutable : le diagnostique de SEP est quasi certain devant ce tableau.


- Le syndrome de Devic (neuromyélite aiguë) est une double névrite optique avec atteinte médullaire sévère : l'étiologie la plus
fréquente en est la sclérose en plaque, et, de plus, le tableau présenté par la patiente est peu évocateur de ce diagnostic.
- La maladie de Leber est une neuropathie optique familiale d'origine mitochondriale se manifestant par une baisse de l'acuité
visuelle. Elle est est bilatérale et progressive en quelques mois.

Quelle(s) attitude(s) thérapeutique(s) adopterez-vous dans un premier temps ?


A - Antibiothérapie
B - La polyvitaminothérapie B
C - Les anti-inflammatoires non stéroïdiens
D - La corticothérapie générale
E - Les immunosuppresseurs
Bonne(s) réponse(s) : D

C'est le traitement des poussées. Les immunosuppresseurs sont données comme traitement de fond dans les formes
sévères. Leur bénéfice est discuté.

Vous êtes amené à voir un homme de 40 ans quelques heures après une crise comitiale généralisée. A l'interrogatoire de ce
patient très céphalalgique vous apprenez que cinq ou six ans auparavant il avait déjà fait une crise d'épilepsie. A cette époque
il avait été pratiqué seulement un électro-encéphalogramme. Aucun traitement n'avait été instauré. A l'examen neurologique
on retrouve une hémianopsie latérale homonyme gauche au doigt. La nuque est raide. L'interrogatoire ne retrouve aucune
notion d'anomalie du champ visuel antérieur : le malade est dessinateur. Il n'y a aucune autre anomalie à l'examen
neurologique ou somatique.

La lésion neurologique peut se situer au niveau de deux des localisations suivantes :


A - Chiasma optique
B - Nerf optique gauche
C - Radiations optiques droites
D - Radiations optiques gauches
E - Lobe occipital droit
Bonne(s) réponse(s) : C E

Sans commentaire.

La lésion en cause peut être :


A - Une tumeur
B - Un infarctus cérébral
C - Une malformation vasculaire
D - Une méningite
E - Une sclérose en plaques
Bonne(s) réponse(s) : A C D

- Crise comitiale + signe de localisation + syndrome méningé : évoque, parmi les diagnostics proposés, en premier une
malformation vasculaire (ce d'autant qu'il existe un antécédent comitial).
- Le reste est plus litigieux : une méningite peut présenter des signes de localisation (tuberculeuse en particulier, par le biais
d'une artérite).
Une tumeur doit aussi être évoquée de principe, mais n'explique pas la raideur de la nuque...

Si vous êtes uniciste et que la crise comitiale d'il y a 5 ans fait partie de l'histoire, quel est parmi les suivants le
diagnostic que l'on peut évoquer en priorité ?
A - Métastase
B - SEP
C - Angiome
D - Embolie
E - Méningiome
Bonne(s) réponse(s) : C

Sans commentaire.

909
Exclusivement sur DOC - DZ : www.doc-dz.com NADJI 85
RESIDANAT EN POCHE TOME II
Cas Clinique en QCM

Parmi les examens suivants, quel est celui qui peut vous apporter le plus d'informations :
A - Scintigraphie
B - Doppler des vaisseaux du cou
C - Scanner sans injection de produit de contraste
D - Scanner avec et sans injection de produit de contraste
E - Electroencéphalogramme
Bonne(s) réponse(s) : D

Sans commentaire.

Monsieur X., âgé de 66 ans, est adressé pour vertiges. En fait il s'agit de troubles de l'équilibre installés progressivement
depuis une quinzaine de jours, à type de maladresse, d'instabilité et d'insécurité à la marche ; depuis la même époque, il
souffre de céphalées postérieures survenant essentiellement à l'effort. En dehors de la notion d'un amaigrissement récent de
4 kg, il n'y a pas d'antécédent pathologique. A l'examen la marche est festonnante, malhabile, avec des embardées ; à la
manoeuvre de Romberg, on note une rétropulsion et un élargissement marqué du polygone de sustentation; par ailleurs, il
existe une discrète maladresse aux manoeuvres de coordination de l'hémicorps droit et un signe de Babinski droit. Le reste de
l'examen neurologique est normal.

On conclut à un syndrome cérébelleux. Celui-ci est plutôt en faveur :


A - D'une atteinte du vermis cérébelleux
B - D'une atteinte de l'hémisphère cérébelleux
C - D'une atteinte du pédoncule cérébelleux supérieur
D - D'une atteinte diffuse du cervelet
E - D'une atteinte du pédoncule cérébelleux moyen
Bonne(s) réponse(s) : A

En effet, les troubles prédominent nettement sur la statique (les troubles de la coordination de l'hémicorps droit sont discrets).

Parmi les 5 signes suivants, lequel ne fait pas partie du syndrome cérébelleux ?
A - Hypotonie
B - Dysmétrie
C - Adiadococinésie
D - Asynergie
E - Signe de Babinski
Bonne(s) réponse(s) : E

Sans commentaire.

Dans le contexte clinique et compte-tenu de sa fréquence, à laquelle de ces étiologies peut-on rattacher
probablement le syndrome cérébelleux ?
A - Hémangioblastome
B - Métastase
C - Atrophie olivo-ponto-cérébelleuse
D - Sclérose en plaques
E - Syndrome hérédo-dégénératif
Bonne(s) réponse(s) : B

Les métastases sont les tumeurs intracrâniennes les plus fréquentes de l'adulte aussi bien sustentorielle que de la fosse
postérieure.

En cas de syndrome cérébelleux aigu toutes les étiologies suivantes peuvent être évoquées sauf une, laquelle ?
A - Cérébelleux post-infectieuse
B - Infarctus
C - Hémorragie
D - Intoxication médicamenteuse
E - Syndrome de Guillain et Barré
Bonne(s) réponse(s) : E

Dans le Guillain-Barré, on peut classiquement observer une ataxie en rapport avec des troubles de la sensibilité profonde,
mais pas de syndrome cérébelleux (cependant dans une forme particulière de Guillain Barré, le syndrome de Miller Ficher,
existe une ataxie propioceptive et cérébelleuse et des troubles oculomoteurs).

910
Exclusivement sur DOC - DZ : www.doc-dz.com NADJI 85
RESIDANAT EN POCHE TOME II
Cas Clinique en QCM
Un homme de 41 ans est hospitalisé à l'occasion d'un état d'agitation anxieuse et de confusion.
L'examen clinique initial révèle l'existence de sueurs profuses, d'une trémulation permanente des extrêmités, d'une
tachycardie régulière à 120/min. La pression artérielle est de 120/80mmHg. Le foie est palpable, à bord inférieur ferme.
L'examen neurologique est normal ; les réflexes ostéotendineux sont vifs mais non diffusés. Les premiers examens
biologiques donnent :
glycémie : 5.8 mmol/l, protidémie : 53 9/1, ALAT (transaminases glutamiques pyruviques) : 255 UI/l (normale = moins de 20
UI/l), ASAT (transaminases glutamiques oxalo-acétiques) : 315 UI/l (normale inférieure à 20 UI/l), Gamma GT 252 UI/l
(normale inférieure à 21 UI/l), natrémie : 129 mmol/l, chlorémie : 85 mmol/l, cholestérolémie totale : 15 mmol/l (5,80 9/1),
triglycéridémie : 80 mmol/l (70 9/1), NFS 4,5.1012/1, VGM = 1 07 fI.

Le diagnostic suggéré par les données cliniques est :


A - Crise aiguë d'hyperthyroïdie
B - Début de délirium tremens
C - Pancréatite aiguë
D - Encéphalopathie de Gayet-Wernicke
E - Hépatite fulminante
Bonne(s) réponse(s) : B

- Il s'agit d'un alcoolique chronique (VGM ; Gamma GT) agité, trémulant, confus.
- Il n'y a pas de signes (en particulier oculomoteurs) orientant vers un Gayet-Wernicke.
- L'absence de douleurs abdominales, une glycémie normale (l'amylase et la calcémie ne sont pas données) rendent le
diagnostic de pancréatite peu probable.

L'hyperlipoprotéinémie de ce malade :
A - Peut s'accompagner de xanthomes tendineux
B - Peut s'accompagner de xanthomes éruptifs
C - Est fortement athérogène
D - Peut s'accompagner d'une pancréatite aiguë
E - Est en rapport avec une production excessive de VLDL
Bonne(s) réponse(s) : D E

A, B, C caractérisent les hypercholestérolémies importantes ; ici le risque est lié à l'augmentation des triglycérides (qui
constituent 3/4 des VLDL) et donc celui d'une pancréatite aiguë.

Les données biologiques suggèrent :


A - Une hyperlipoprotéinémie de type V
B - Un éthylisme chronique
C - Une pancréatite chronique en poussée
D - Une déplétion sodée importante
E - Une hépatite alcoolique aiguë
Bonne(s) réponse(s) : B E

- Il s'agit très probablement d'une hyperlipoprotéinémie de type IV (endogène) secondaire à l'éthylisme.


E? - Le diagnostic d'hépatite alcoolique est fortement suggéré chez cet alcoolique probablement cirrhotique (hépatomégalie
ferme) avec augmentation des SGDT > SGPT mais il faut souligner que des augmentations des transaminases supérieures à
10 fois la normale sont exceptionnelles dans les hépatites alcooliques.

Le(s) traitement(s) d'urgence est(sont) :


A - Injections intramusculaires d'Equanil®
B - Réhydratation
C - Administration de clofibrate (Lipavlon®)
D - Aspiration gastrique
E - Association Pénicilline-Colimycine
Bonne(s) réponse(s) : A B

C'est la prévention du D.T. (il faudrait aussi prévenir la pancréatite par une diète).

911
Exclusivement sur DOC - DZ : www.doc-dz.com NADJI 85
RESIDANAT EN POCHE TOME II
Cas Clinique en QCM
Un homme de 35 ans est hospitalisé pour syndrome méningé avec fièvre. La ponction lombaire ramène un liquide un peu
trouble contenant des polynucléaires altérés. Ce malade ne présente pas d'antécédent en dehors d'un traumatisme crânio-
facial datant de 4 ans et ayant nécessité une hospitalisation de 7 jours. Le syndrome méningé a été précédé de céphalées
frontales avec obstruction nasale 8 jours auparavant. Les radiographies de sinus montrent des opacités éthmoïdale, maxillaire
et frontale droites.

L'antécédent de traumatisme fait soupçonner une méningite par brèche dure-mérienne. Quelle(s) localisation(s)
anatomique(s) de cette brèche doit-on rechercher ?
A - Rocher
B - Lame criblée de l'éthmoïde
C - Sinus sphénoïdal
D - Sinus maxillaire
E - Sinus frontal
Bonne(s) réponse(s) : A B C E

Le sinus maxillaire n'est pas à proximité des méninges.

S'il s'agit bien d'une méningite d'origine ORL, quel germe sera le plus fréquemment retrouvé ?
A - Branhamella catarrhalis
B - Streptocoque
C - Pneumocoque
D - Hémophilus
E - Entérocoque
Bonne(s) réponse(s) : E

Sans commentaire.

Si le bilan radiologique avait été pratiqué lors du traumatisme quel ou quels signes ont une valeur d'orientation
pour une brèche dure-mérienne ?
A - Opacité ethmoïdale unilatérale
B - Fracture du malaire
C - Fracture des os propres du nez
D - Fracture du toit éthmoïdal
E - Pneumatocèle
Bonne(s) réponse(s) : D E

Un pneumatocèle (passage d'air dans la boîte crânienne) signe la brèche osseuse et dure-mèrienne.

Quel est ou quels sont les caractères d'une rhinorrhée cérébrospinale post traumatique ?
A - Intermittente
B - Augmentée à l'effort physique
C - Aqueuse
D - Muqueuse
E - Purulente
Bonne(s) réponse(s) : A B C

L'effort physique augmente la pression du LCR.

Quel est ou quels sont le(s) éléments qui permettront de suspecter fortement l'origine cérébrospinale de la
rhinorrhée ?
A - Présence de mucus
B - Présence de germes
C - Présence d'électrolytes
D - Présence de glucose
E - Présence d'urée
Bonne(s) réponse(s) : D

Sans commentaire.

912
Exclusivement sur DOC - DZ : www.doc-dz.com NADJI 85
RESIDANAT EN POCHE TOME II
Cas Clinique en QCM

Le traitement médical a guéri la méningite. Quel ou quels autre(s) traitement(s) doit-on proposer au malade ?
A - Surveillance clinique
B - Drainage lombaire
C - Traitement médical de la sinusite
D - Fermeture de la brèche dure-mérienne
E - Antibiothérapie au long cours
Bonne(s) réponse(s) : D

Sans commentaire.

Mr L. 65 ans, est gêné depuis 6 à 8 mois pour les gestes précis de la main droite. Son écriture est petite et se rétrécit encore
davantage au cours d'une même ligne d'écriture. Il met beaucoup plus de temps à se raser. Quand il marche, son bras droit
ne balance pas, mais parfois sa main se met alors à trembler. Il a remarqué que ce tremblement se produit également lorsqu'il
regarde la télévision avec attention, le moindre mouvement faisant disparaître ce tremblement.

Parmi les arguments cliniques énumérés ci-dessus, trois viennent appuyer la suspicion d'une maladie de
parkinson. Lesquels ?
A - L'unilatéralité des symptômes
B - L'exagération des réflexes osteo-tendineux
C - La perte des mouvements automatiques
D - La présence de troubles de l'équilibre
E - Le ralentissement des gestes
Bonne(s) réponse(s) : A C E

La maladie de Parkinson débute presque toujours de façon unilatérale et demeure asymétrique. En revanche, un début par
des troubles de l'équilibre (non mentionnés ici) doit faire récuser ce diagnostic, les chutes peuvent cependant apparaître tard
dans l'évolution.

Le tremblement décrit par le malade correspond à un tremblement :


A - Intentionnel
B - Cérébelleux
C - De repos
D - D'attitude
E - D'action
Bonne(s) réponse(s) : C

Le tremblement parkinsonnien est un tremblement de repos, 4-7 cycles/seconde à type "d'émiettement" de la main.

L'examen clinique ne montre qu'une très discrète hypertonie droite. Une manoeuvre est capable de la rendre
évidente :
A - Le calcul mental
B - Le fait de serrer volontairement le poing gauche
C - La position allongée
D - La position debout
E - Distraire le sujet en le faisant parler
Bonne(s) réponse(s) : B

C'est la manoeuvre de Froment.

Dans l'observation ainsi décrite, à quel anti-parkinsonien donnez-vous la priorité ?


A - Artane®
B - Disipal®
C - Eldéprine®
D - Modopar®
E - Trivastal®
Bonne(s) réponse(s) : D

L'akinésie apparaît comme le symptôme majeur dans cette forme (micrographie) ; la L.DOPA est sans conteste à donner en
première intention. L'artane® et le Trivastal® sont intéressant dans les formes tremblantes "pures".

913
Exclusivement sur DOC - DZ : www.doc-dz.com NADJI 85
RESIDANAT EN POCHE TOME II
Cas Clinique en QCM

L'un des antécédents énoncés ci-dessous aurait un intérêt majeur pour la prescription thérapeutique et la
surveillance. Lequel ?
A - Artérite des membres inférieurs
B - Notion d'épisodes confusionnels nocturnes
C - Hypertension artérielle
D - Ulcère gastrique cicatrisé
E - Bronchite chronique
Bonne(s) réponse(s) : B

La seule véritable contre-indication de la L.DOPA est d'ordre psychiatrique. La notion d'épisodes confusionnels nocturnes
justifie une surveillance accrue. A noter : le risque est encore plus important avec les anticholinergiques.

Une femme de 30 ans, présente des crises migraineuses typiques dont la recrudescence réente l'amène à prendre 2
comprimés par jour de 2 mg de tartrate d'ergotamine (Gynergène cafeine®) depuis 15 jours. Depuis une semaine, les
céphalées sont permanentes et intenses. Elles s'accompagnent d'une asthénie, d'un état nauséeux permanent et d'un
refroidissement avec paresthésies des extrémités.

Quelle(s) proposition(s) vous paraî(ssen)t exacte(s) lors de cette phase aiguë ?


A - Il s'agit d'un état de mal migraineux
B - L'hospitalisation est souhaitable
C - Le Gynergene caféiné joue un rôle dans le tableau actuel
D - Il faut prescrire de la dihydroergotamine injectable
E - Il faut instituer un traitement au propranolol (Avlocardyl®) per os
Bonne(s) réponse(s) : A B C

L'état de mal migraineux est presque toujours consécutif à un abus d'ergotamine. Il faut cesser ce traitement dangereux
(risque d'ergotisme = ischémies artérielles : cérébrale, cardiaque des membres inférieurs), hospitaliser, soulager par
antalgiques, anxiolytiques. Un traitement de fond ne sera entrepris que secondairement.

Si vous aviez vu auparavant cette malade, vous auriez autorisé comme dose maximum par semaine de
Gynergene Cafeine® :
A - 2 mg
B - 3 mg
C - 8 mg
D - 24 mg
E - 30 mg
Bonne(s) réponse(s) : C

Sans commentaire.

Le tartrate d'ergotamine (Gynergène Caféine®) :


A - Peut provoquer des nausées
B - Peut favoriser des crises d'angine de poitrine
C - Est autorisé pendant la grossesse
D - Ne doit pas être associé aux antibiotiques de type macrolide
E - Est recommandé dans les crises de migraine accompagnée
Bonne(s) réponse(s) : A B D E

Sans commentaire.

A distance de cette phase aiguë, vous souhaitez entreprendre un traitement de fond ayant fait la preuve de son
efficacité. Le(s)quel(s) réponde(nt) à cette définition ?
A - Tartrate d'ergotamine, caféine et cyclizine (Migwell®)
B - Propranolol (Avlocardyl®)
C - Pizotifène (Sanmigran®)
D - Méthysergide (Desernil®)
E - Amitriptiline (Laroxyl®)
Bonne(s) réponse(s) : B C D E

Le tartrate d'ergotamine est un traitement de la crise (la dihydroergotamine est un des traitements de fond qu'on peut proposer
: ne pas les confondre).

914
Exclusivement sur DOC - DZ : www.doc-dz.com NADJI 85
RESIDANAT EN POCHE TOME II
Cas Clinique en QCM
Un homme de 54 ans est hospitalisé pour un syndrome méningé évoluant depuis dix jours, avec céphalées diffuses,
vomissements, diplopie par paralysie du VI droit, fièvre à 38,5°C. La conscience est normale ainsi que le reste de l'examen
clinique. Il n'a reçu aucun antibiotique.
Ses antécédents :
- primo-infection tuberculeuse à l'âge de 8 ans, non traitée
- à 52 ans, maladie de Hodgkin stade II (jugulo-carotidienne droite et sus-claviculaire gauche) traitée par chimiothérapie puis
radiothérapie. En surveillance depuis.
Sa biologie actuelle :
- hémogramme : hémoglobine : 135 g/l, leucocytes : 12 800 /mm3 (80 % de polynucléaires neutrophiles), plaquettes : 240
000/mm3
- VS : 40-68 mm
- Ponction lombaire : éléments : 410/mm3 (70 % de polynucléaires plus ou moins altérés, 30 % de cellules mononucléées),
protéinorachie : 1,90 9/1, glycorachie : 1,1 mmol/l, glycémie : 6,3 mmol/l. Présence de quelques bacilles à gram positif à
l'examen direct.

En ne tenant compte que des éléments cliniques et anamnestiques et avant tout résultat concernant le LCR,
vous pouvez écarter définitivement :
A - Une méningite tuberculeuse
B - Une méningite carcinomateuse
C - Une méningite à Neisseria méningitidis
D - Une méningite à Listeria monocytogénès
E - Un abcès du cerveau
Bonne(s) réponse(s) : C

L'évolution torpide (depuis 10 jours, avec fièvre modérée, l'absence de syndrome méningé franc) élimine une méningite à
méningocoque, d'installation beaucoup plus bruyante.

En tenant compte du contexte clinique et du résultat de la ponction lombaire, vous devez porter, en toute
logique, le diagnostic de :
A - Méningite à Listeria monocytogénès
B - Méningite à Haemophilus influenzae
C - Méningite à Streptococcus pneumoniae
D - Méningite tuberculeuse
E - Méningite à Cryptococcus néoformans
Bonne(s) réponse(s) : A

La méningite à Listeria est un diagnostics différentiel de la méningite tuberculeuse.


- Survient plus souvent chez les immuno-déprimés.
- Volontiers subaiguë.
- Paralysie fréquente des nerfs crâniens.
- Petit bacille gram+.

Dès que vous êtes en possession des résultats de la ponction lombaire, et avant confirmation de la nature du
germe causal, quel schéma antibiotique vous paraît approprié à cette situation clinique ?
A - Cefotaxime (Claforan®) + Amikacine (Amiklin®)
B - Pefloxacine (Péflacine®) + Amikacine (Amiklin®)
C - Ampicilline + Gentamicine (Gentalline®)
D - Vancomycine (Vancocine®) + Cefotaxime (Claforan®)
E - Oxacilline (Bristopen®) + Amikacine (Amiklin®)
Bonne(s) réponse(s) : C

Sans commentaire.

Concernant le taux de glycorachie de ce malade, il est exact que :


A - Il s'agit d'un taux normal
B - Il s'agit d'un taux incompatible avec le diagnostic de méningite tuberculeuse
C - Ce taux évoque très fortement la présence d'une bactérie dans le LCR
D - Ce taux est compatible avec le diagnostic de méningite virale
E - Ce taux est compatible avec le diagnostic de méningite à Listeria monocytogénès
Bonne(s) réponse(s) : C E

Une hypoglycorachie (inférieure à glycémie/2) se voit dans les méningites bactériennes, mycotiques, carcinomateuses. La
glycorachie est normale dans les méningites virales.

915
Exclusivement sur DOC - DZ : www.doc-dz.com NADJI 85
RESIDANAT EN POCHE TOME II
Cas Clinique en QCM
Mme F. 45 ans, est réveillée depuis 3 mois chaque nuit à 2 ou 3 heures d'intervalle par des dysesthésies des mains, surtout à
droite. Depuis une à deux semaines elle est gênée pour effectuer les mouvements fins (prendre une épingle par exemple).
L'examen clinique.ne montre ni déficit moteur ni amyotrophie de la main, mais permet de constater un déficit de la sensibilité
discriminative de la pulpe de l'index et du médius. Un syndrome du canal carpien est suspecté.

Quel signe clinique est évocateur du diagnostic ?


A - Accentuation des douleurs par élévation du bras
B - Dysesthésies reproduites par la flexion ou l'extension forcée du poignet
C - Aggravation par le froid
D - Cyanose des doigts
E - Abolition du pouls radial
Bonne(s) réponse(s) : B

- L'élévation du bras calme les douleurs, la compression du médian au canal carpien (flexion/extension poignet) les aggrave.
- C et D évoquent un syndrome de Raynaud.
- E évoque un syndrome du défilé costo claviculaire (ou défilé des scalènes).

L'examen neurophysiologique peut confirmer ce diagnostic en montrant :


A - Une dénervation complète
B - Un allongement de la latence distale de la réponse motrice
C - Un bloc neuro-musculaire
D - Un ralentissement de la V.C.N. sensitive
E - Une dénervation débutante
Bonne(s) réponse(s) : B E

Au maximum il existe un bloc de conduction, mais non un bloc neuro-musculaire !

Parmi ces étiologies générales, deux doivent être recherchées. :


A - Hypercorticisme
B - Polyarthrite rhumatoïde
C - Carence vitaminique
D - Myxoedème
E - Lupus érythémateux dissémine
Bonne(s) réponse(s) : B D

Les autres causes du syndrome canal carpien sont : l'amylose, la goutte, l'acromégalie, les traumatismes (fractures),
microtraumatismes (flexion - extension du poignet). Le plus souvent aucune cause n'est retrouvé.

Le syndrome du canal carpien étant confirmé, vous proposez :


A - Une libération d'emblée du nerf médian
B - Une rééducation active
C - Un traitement anti-inflammatoire par voie générale
D - Une infiltration locale de corticoïdes suivie de surveillance
E - Une immobilisation du poignet
Bonne(s) réponse(s) : D

Il n'existe pas de critères formels (cliniques ou électriques) pour décider de la conduite à tenir :
- Troubles purement sensitifs : soit mesures symptomatiques (arrêt de l'activité manuelle incriminée, immobilisation du
poignet, antalgiques, antiinflammatoire), soit plutôt infiltrations locales de corticoïdes (= D plutôt que E).
- En cas de déficit moteur, de dénervation importante à l'EMG, de symptômes résistants aux traitements précédents, la
chirurgie sera indiquée.

Si un déficit moteur s'installait ultérieurement, au niveau de quel(s) muscle(s) faudrait-il le rechercher ?


A - 1er interosseux dorsal
B - Opposant du pouce
C - Adducteur du pouce
D - Abducteur du Vème doigt
E - Court fléchisseur du pouce
Bonne(s) réponse(s) : B E

A, C, D sont innervés par le cubital.

916
Exclusivement sur DOC - DZ : www.doc-dz.com NADJI 85
RESIDANAT EN POCHE TOME II
Cas Clinique en QCM

L'apparition d'un déficit moteur avec amyotrophie dans le territoire du nerf médian, indique :
A - La répétition des injections locales de corticoïdes
B - Le recours à une biopsie synoviale
C - Une décompression chirurgicale sans délai du canal carpien
D - Une surveillance neurophysiologique accrue
E - Une corticothérapie générale à forte dose
Bonne(s) réponse(s) : C

Sans commentaire.

Une jeune fille âgée de 18 ans vient consulter pour des troubles évoluant depuis environ 4 mois et apparus de façon
insidieuses. Elle est très fatiguée. Cette fatigue, essentiellement physique, envahit toute la journée mais est surtout
insupportable en fin de journée. Elle s'aggrave aussi considérablement à l'effort, limitant l'activité de la malade. Depuis 2 mois
la malade signale l'existence d'épisodes de diplopie horizontale intermittente, déclenchés ou aggraves par la fixation
prolongée du regard. La mère de la malade signale aussi l'existence, généralement en fin de journée, d'une chute de la
paupière gauche, quelques fois des deux paupières. Enfin, et cela est plus récent puisque datant de seulement 2 ou 3
semaines, la malade présente d'une part des difficultés de parole et sa voix est affaiblie et d'autre part se plaint d'avoir du mal
à mastiquer les aliments un peu durs et parfois des difficultés de déglutition. Il n'existe pas d'antécédents notables. Lors de
l'examen, qui a eu lieu à 10h30, on ne relève aucune anomalie. L'examen neurologique somatique est normal.

Quel diagnostic évoquez-vous ?


A - Sclérose en plaques
B - Syndrome neuro-anémique
C - Méningite tuberculeuse
D - Myasthénie
E - Maladie de Charcot
Bonne(s) réponse(s) : D

Sans commentaire.

Parmi les épreuves suivantes laquelle vous semble pertinente pour argumenter le diagnostic ?
A - Calcul mental
B - Epreuve du doigt au nez
C - Apprentissage d'une série de mots
D - Lecture à haute voix d'un texte
E - Copier une figure complexe
Bonne(s) réponse(s) : D

Afin de mettre en évidence à l'effort une diminution du volume vocale, un trouble de la phonation (nasonnée), éventuellement
une dypnée.

Quel examen électrophysiologique vous semble utile pour argumenter le diagnostic ?


A - Electroencéphalogramme
B - Potentiels évoqués visuels
C - Electrocardiogramme
D - Mesure de la vitesse de conduction motrice des nerfs
E - Epuisement de la réponse à la stimulation répétitive
Bonne(s) réponse(s) : D

E - C'est la recherche d'un bloc myasthénique : diminution d'amplitude des potentiels moteurs lors de la stimulation répétée.

Quel examen radiologique vous semble utile ?


A - Scanner X cérébral
B - Scanner X thoracique
C - Examen par résonance magnétique cérébrale
D - Angiographie cérébrale
E - Aucun de ces examens
Bonne(s) réponse(s) : B

A la recherche d'un thymome (fréquent chez des myasthéniques), ou d'une hyperplasie thymique, dont l'exerèse constitue un
volet du traitement de la myasthénie.

917
Exclusivement sur DOC - DZ : www.doc-dz.com NADJI 85
RESIDANAT EN POCHE TOME II
Cas Clinique en QCM

Parmi les médicaments suivants lequel vous semble indésirable car risquant d'aggraver la malade ?
A - Valium (Diazépam)
B - Vitamine B6
C - Aspirine
D - Aucun de ces médicaments
E - Tous ces médicaments
Bonne(s) réponse(s) : A

Car myorelaxant.

Comment interprétez-vous les signes oculaires ?


A - Atteinte tronculaire du nerf oculaire externe
B - Atteinte tronculaire du nerf moteur oculaire commun
C - Syndrome de Claude-Bernard-Horner
D - Atteinte hémisphérique cérébrale
E - Aucun de ces faits n'est exact
Bonne(s) réponse(s) : E

L'interprétation se fait en termes de muscles et non de nerfs.

Quel est selon vous le risque évolutif majeur ?


A - Complications thrombo-emboliques
B - Paraplégie
C - Complications oculaires
D - Troubles respiratoires
E - Hydrocéphalie
Bonne(s) réponse(s) : D

Sans commentaire.

Parmi les signes cliniques quels sont celui ou ceux qui vous semblent de ce point de vue inquiétant (menace
d'évolution grave) ?
A - Aggravation de la fatigue a l'effort
B - Existence d'une diplopie
C - Existence d'un ptosis
D - Existence de troubles de la parole
E - Existence de troubles de la déglutition
Bonne(s) réponse(s) : E

- La présence de troubles de la déglutition fait ici toute la gravité de la maladie.


- La réponse D.E peut être discutée dans la mesure où les troubles de la phonation, 2 mois après le début de la maladie,
témoignent déjà d'une évolutivité qui doit faire craindre l'aggravation.

Un homme de 70 ans consulte pour l'apparition récente de mouvements anormaux. Il s'agit de mouvements de grande
amplitude, intermittents, du tronc, de la face, des membres supérieurs, gênant l'activité volontaire et survenant surtout en
milieu de matinée et d'après-midi. La maladie a débuté il y a 10 ans par un tremblement de repos du membre supérieur droit
traité par Artane®. Puis, il y a 8 ans, il a été gêné à la marche qui se faisait à petits pas. Le Modopar® a été introduit il y a 6
ans avec une bonne efficacité mais son médecin a dû progressivement augmenter les doses. Il prend actuellement 250 mg de
Modopar® 3 fois par jour.

Les mouvements anormaux récemment apparus sont :


A - Une myotonie
B - Un tremblement intentionnel
C - Une aggravation du tremblement de repos
D - Une adiadococinésie
E - Des dyskinésies
Bonne(s) réponse(s) : E

Sans commentaire.

918
Exclusivement sur DOC - DZ : www.doc-dz.com NADJI 85
RESIDANAT EN POCHE TOME II
Cas Clinique en QCM

Le traitement par la L. DOPA peut donner :


A - Des bouffées confusionnelles
B - Des troubles de l'accomodation
C - Une hypotension orthostatique
D - Une rétention aiguë d'urine
E - Une majoration d'un broncho-spasme
Bonne(s) réponse(s) : A C

Sans commentaire.

Devant ces mouvements anormaux, vous pouvez proposer :


A - La diminution de la dose de L. DOPA
B - L'arrêt de la L. DOPA
C - L'adjonction d'anticholinergiques
D - Une fragmentation de la dose totale de L. DOPA
E - L'adjonction d'Halopéridol
Bonne(s) réponse(s) : A D

Il s'agit de dyskinésie de milieu de dose probablement en rapport avec un surdosage.

Ces mouvements anormaux sont dus :


A - A une hypersensibilité de dénervation des récepteurs dopaminergiques
B - A une activité cérébelleuse anormale
C - A une perte neuronale dans le noyau rouge
D - A une atteinte de la voie pyramidale
E - A aucun de ces mécanismes
Bonne(s) réponse(s) : A

Sans commentaire.

Madame B..., 45 ans, vient consulter pour des douleurs du membre supérieur gauche évoluant depuis plusieurs semaines.
Elle ne présente aucun antécédent particulier. La douleur est de forte intensité, parcourant l'ensemble du bras jusqu'au
majeur. Il existe des paresthésies permanentes de ce doigt. A l'examen, vous retrouvez du côté gauche, une faiblesse lors de
l'extension de l'avant-bras sur le bras, de l'extension des premières phalanges des quatre derniers doigts sur la main alors
que l'extension des deux dernières phalanges est normale. Le réflexe tricipital est aboli, les autres réflexes du membre
supérieur gauche sont normaux. Il existe une hypoesthésie de la face palmaire et dorsale du majeur gauche. La colonne
cervicale présente une mobilité normale, vous ne retrouvez aucun point douloureux.

L'examen clinique met donc en évidence un déficit, du ou des :


A - Sus épineux
B - Long supinateur
C - Extenseur commun des doigts
D - Lombricaux
E - Triceps
Bonne(s) réponse(s) : C E

Les lombricaux (C8-D1) sont responsables de la flexion de la métacarpo-phalangienne et l'extension des deux dernières
phalanges.

La lésion se situe au niveau :


A - Racine C6
B - Nerf radial
C - Racine C7
D - Tronc primaire inférieur du plexus brachial
E - Nerf cubital
Bonne(s) réponse(s) : C

Sans commentaire.

919
Exclusivement sur DOC - DZ : www.doc-dz.com NADJI 85
RESIDANAT EN POCHE TOME II
Cas Clinique en QCM

Les arguments que vous pouvez retrouver à l'interrogatoire ou à l'examen évoquant une compression
médullaire sont :
A - Un réflexe palmo-mentonnier présent à gauche
B - Un Babinski bilatéral
C - Une hypoesthésie des membres inférieurs à limite supérieure nette
D - Des fasciculations
E - Une abolition du réflexe crémastérien droit
Bonne(s) réponse(s) : B C

Il faut rechercher des signes d'atteinte sous-lésionnelle.

Devant ce tableau, il est licite de demander :


A - Un électroencéphalogramme
B - Un scanner crânien
C - Un scanner rachidien
D - Une radiographie du crâne en incidence de Blondeau
E - Une électromyographie
Bonne(s) réponse(s) : C

Sans commentaire.

L'absence d'anomalie à l'examen clinique de la colonne cervicale :


A - Permet d'éliminer une pathologie osseuse
B - Permet d'éliminer une lésion radiculaire
C - Permet d'éliminer un processus tumoral neurologique
D - Permet d'éliminer une compression médullaire
E - N'a aucune valeur pronostique ou diagnostique
Bonne(s) réponse(s) : E

Sans commentaire.

Un homme âgé de 42 ans, est hospitalisé pour le bilan de crises d'épilepsie ayant débuté il y a 25 mois, d'une manière
spontanée, chez un patient droitier jusqu'alors sans aucun antécédent pathologique. Il s'agit de crises épileptiques
généralisées qui se sont répétées 3 fois et qui sont précédées de paresthésies intéressant la main gauche qui s'étendent dans
le bras homolatéral, avec la survenue de mouvements tonico-cloniques avant la perte de connaissance. L'interrogatoire
retrouve aussi la survenue de céphalées bifrontales d'apparition récente, cédant mal aux antalgiques périphériques. L'examen
neurologique est strictement normal : il ne retrouve ni déficit sensitivomoteur, ni atteinte des nerfs crâniens, ni syndrome
méningé. Par contre, le fond d'oeil met en évidence l'existence d'un flou papillaire bilatéral.

Ce tableau clinique doit faire évoquer :


A - Un anévrysme intra-crânien
B - Une lésion expansive intra-crânienne
C - Une migraine accompagnée
D - Une malformation artérioveineuse cérébrale
E - Une hydrocéphalie
Bonne(s) réponse(s) : B

HIC (céphalées + oedème papillaire) d'apparition progressive récente et crises camitiales doivent faire rechercher un
processus sparsif.

Cette sémiologie pourrait faire évoquer une lésion intra-crânienne siégeant :


A - Dans le cervelet
B - Dans l'hémisphère cérébral dominant
C - A proximité du cortex pariétal
D - Dans le troisième ventricule
E - Dans la région sellaire
Bonne(s) réponse(s) : C

C - Car début par des paresthésies : probablement au niveau de l'aire somesthérique primaire.

920
Exclusivement sur DOC - DZ : www.doc-dz.com NADJI 85
RESIDANAT EN POCHE TOME II
Cas Clinique en QCM

Parmi les explorations complémentaires suivantes, laquelle est justifiée ?


A - Une scintigraphie cérébrale
B - Un scanner cérébral avec et sans injection de produit de contraste
C - Une artériographie vertébrale
D - Une cisternographie isotopique
E - Une ponction lombaire
Bonne(s) réponse(s) : B

Sans commentaire.

Que peut mettre en évidence un scanner cérébral, en relation avec la clinique ?


A - Une dilatation des ventricules cérébraux
B - Une hypodensité prenant le produit de contraste
C - Un effet de masse
D - Une atrophie corticale diffuse
E - Une hémorragie sous arachnoïdienne
Bonne(s) réponse(s) : B C

Sans commentaire.

Quelle est l'hypothèse diagnostique la plus vraisemblable ?


A - Une hydrocéphalie à pression normale
B - Un méningiome de la voûte
C - Un angiome cortical
D - Un glioblastome pariétal
E - Un médulloblastome cérébelleux
Bonne(s) réponse(s) : D

D - Astrocystome grade IV (glioblastome) par dégénérescence d'un astrocytome de bas grade (I-II) pré-existant (expliquant les
crises camitiales depuis 25 mois). En effet, la durée entre les premiers signes et la consultation est incompatible avec un
glioblastome apparu de Novo.

Monsieur A..., 28 ans, est hospitalisé en urgence du fait de la survenue d'une violente céphalée temporale gauche. Celle-ci
s'est installée brutalement alors qu'il jouait au tennis et est devenue rapidement intolérable. Parallèlement, sont survenus des
vomissements et une discrète somnolence. L'examen neurologique montre un syndrome méningé franc. Il existe un ptosis de
l'oeil droit et un strabisme divergent. Le fond d'oeil est normal. L'examen général est par ailleurs sans anomalie. Vous
évoquez une hémorragie sous-arachnoïdienne.

Parmi les examens complémentaires suivants, quels sont ceux qui permettent de faire le diagnostic
d'hémorragie sous-arachnoïdienne ?
A - EEG
B - Scanner cérébral
C - Ponction lombaire
D - Electromyographie
E - Myélographie
Bonne(s) réponse(s) : B C

Sans commentaire.

Quel examen complémentaire permet d'en affirmer le mécanisme ?


A - Scanner cérébral
B - EEG
C - Ponction lombaire
D - Artériographie
E - Myélographie
Bonne(s) réponse(s) : D

Sans commentaire.

921
Exclusivement sur DOC - DZ : www.doc-dz.com NADJI 85
RESIDANAT EN POCHE TOME II
Cas Clinique en QCM

Quelle est la signification du ptosis et du strabisme divergent ?


A - Hypertension intracrânienne
B - Trouble de la vigilance
C - Paralysie du III
D - Paralysie du VI
E - Ophtalmoplégie internucléaire
Bonne(s) réponse(s) : C

Sans commentaire.

Quelle étiologie doit-on évoquer en priorité ?


A - Angiome pariéto-occipital droit
B - Anévrysme de la communicante antérieure
C - Anévrysme de l'artère cérébrale antérieure
D - Anévrysme de la communicante postérieure
E - Anévrysme du tronc basilaire
Bonne(s) réponse(s) : D

C'est-à-dire anévrysme de l'artère carotide interne à l'origine de la communicante postérieure, comprimant le III dans le sinus
caverneux. L'autre localisation d'anévrysme pouvant comprimer le III est la terminaison du tronc basilaire (moins fréquente).

Citez, parmi les complications suivantes, celles qui peuvent survenir en phase précoce:
A - Récidive hémorragique
B - Hydrocéphalie à pression normale
C - Vasospasme
D - Hydrocéphalie aiguë
E - Anomalies électrocardiographiques
Bonne(s) réponse(s) : A C D E

Les anomalies électrocardiolytiques peuvent stimuler une ischémie myocardique. Mais les véritables complications sont A, C
et D.

Cette patiente de 54 ans est admise pour douleurs rachidiennes et impotence motrice des membres inférieurs. C'est environ
depuis un an et demi que sont apparues des douleurs nocturnes au niveau du mamelon gauche. Ces douleurs sont
impulsives à la toux et à la défécation et surviennent essentiellement 'pendant la deuxième moitié de la nuit. Dans le même
temps sont apparues des difficultés à la marche, plus nettes dans l'obscurité, avec talonnement et sensation électrique au
niveau des deux pieds. La patiente a également noté des difficultés urinaires à type de mictions impérieuses.
L'examen neurologique met en évidence un déficit moteur du membre inférieur gauche. Les ROT sont vifs, polycinétiques aux
deux membres inférieurs avec signe de Babinski bilatéral.
L'examen sensitif des membres inférieurs montre :
- un trouble de la sensibilité profonde bilatéral
- une hypoesthésie tactile et algique bilatérale remontant jusqu'à la région sous-mamelonnaire.
Une ponction lombaire est réalisée : elle met en évidence un blocage incomplet avec dissociation albumino-cytologique. Une
myélographie par voie lombaire met en évidence une tumeur intra-durale extra-médullaire de niveau D6-D7.

Quel(s) est (sont) le(s) signe(s) en relation avec le syndrome lésionnel ?


A - Babinski bilatéral
B - Douleur mamelonnaire
C - Troubles sphinctériens
D - Déficit moteur du membre inférieur
E - Trouble de la sensibilité profonde
Bonne(s) réponse(s) : B

A C D E - Sont en rapport avec le syndrome sous-lésionnel.

Quel(s) est (sont) le(s) signe(s) compatible(s) avec le syndrome sous-lésionnel ?


A - Babinski bilatéral
B - ROT polycinétiques des membres inférieurs
C - Douleur en hémiceinture
D - Trouble de la sensibilité profonde des membres inférieurs
E - Difficultés mictionnelles
Bonne(s) réponse(s) : A B D E

La douleur en hémiceinture est une douleur radiculaire de niveau lésionnel.

922
Exclusivement sur DOC - DZ : www.doc-dz.com NADJI 85
RESIDANAT EN POCHE TOME II
Cas Clinique en QCM

Parmi les anomalies suivantes retenir celle qui vous paraît compatible avec ce cas clinique :
A - Hyperpression du LCR
B - Ascension de la pression du LCR lombaire lors de la compression jugulaire
C - Protéinorachie à 3,55 g/l
D - Glycorachie à 3,77 mmol/l
E - Présence de 15 % de plasmocytes à l'étude cytologique du LCR
Bonne(s) réponse(s) : C

Augmentation isolée de la protéinorachie, en rapport avec un blocage du LCR.


On devrait trouver dans ce cas l'absence d'augmentation de pression du LCR lors de la compression jugulaire (épreuve de
Queckenstedt-Stookey).

Parmi les tumeurs suivantes, indiquer celles qui sont les tumeurs intra-durales extra-médullaires ?
A - Méningiome
B - Neurinome
C - Métastase épidurale
D - Ependymome
E - Astrocytome
Bonne(s) réponse(s) : A B

C - La métastase épidurale est en fait extra-durale.


D E - Ependymome et astrocytome sont intramédullaires.

La malade est opérée de cette tumeur intra-durale extra-médullaire qui refoulait la moelle en avant et vers la
droite. L'anatomie-pathologique confirme histologiquement l'un des diagnostics précédents compatibles avec le
siège de la tumeur. Quel est le traitement complémentaire le plus adapté ?
A - Corticothérapie
B - Chimiothérapie
C - Radiothérapie
D - Stimulation électrique péridurale
E - Rééducation fonctionnelle
Bonne(s) réponse(s) : E

Méningiomes et neurinomes sont des tumeurs bénignes : le traitement repose sur la chirurgie seule.

Un homme de 18 ans est hospitalisé pour un traumatisme crânien avec perte de connaissance, impact frontal et facial. Il n'y a
pas de fracture du crâne sur les radiographies simples du crâne face/profil. Fracture du nez.
Le lendemain matin en se penchant en avant, quelques gouttes de liquide clair s'écoulent par le nez. Il n'y a pas de
céphalées. L'examen neurologique est normal. Température : 37, 2 °C. T.A. : 14/9 cm Hg.

Dans ce cas-là :
A - Le tarissement de l'écoulement peut survenir spontanément
B - La mise en évidence de glucose dans le liquide oriente vers un écoulement de LCR
C - La mise en évidence d'albumine oriente vers un écoulement de LCR
D - Le méchage des fosses nasales doit être prescrit
E - L'olfaction peut être altérée
Bonne(s) réponse(s) : A B E

E - Une fracture de la lame criblée de l'ethmoïde peut détruire les filets nerveux de l'olfaction.

Quels examens sont utiles pour localiser la brèche ?


A - Cisternographie isotopique
B - Tomodensitométrie de la base du crâne
C - Cisternographie computérisée au métrizamide
D - Artériographie cérébrale
E - IRM avec injection de Gadolinium
Bonne(s) réponse(s) : A B

- Le scanner avec fenêtres osseuses pour mettre au mieux en évidence la fracture (supérieur à l'IRM).
- La cisternographie permet de mettre en évidence le passage de LCR (qui contient un radio isotope).

923
Exclusivement sur DOC - DZ : www.doc-dz.com NADJI 85
RESIDANAT EN POCHE TOME II
Cas Clinique en QCM

La lésion osseuse peut siéger :


A - Sur le sphénoïde
B - Sur le frontal
C - Sur le pariétal
D - Sur le malaire
E - Sur l'éthmoïde
Bonne(s) réponse(s) : A B E

Sans commentaire.

Quelles complications directement liées à l'écoulement de LCR, peuvent survenir ?


A - Méningite purulente
B - Hypotension intracrânienne
C - Pneumatocèle intracrânien
D - Méningite virale
E - Epilepsie
Bonne(s) réponse(s) : A B C

- Méningite purulente : le plus souvent à pneumocoque.


- Pneumatocèle intracrânien = passage d'air des cavités nasales dans l'espace méningé.

Un homme de 30 ans consulte pour une diplopie apparue il y a 6 mois et s'aggravant progressivement. Au début, il s'agissait
d'un phénomène de quelques minutes, n'apparaissant qu'épisodiquement. Actuellement, il s'agit de manifestations
quotidiennes. Le matin, le patient est asymptomatique. Ce n'est qu'en fin de journée qu'il voit double, ce qui le gêne beaucoup
pour regagner son domicile le soir en voiture. Depuis une semaine, il présente épisodiquement dans la journée des difficultés
d'élocution.
A l'examen, vous ne constatez pas de troubles de l'oculomotricité. Les pupilles sont symétriques, de taille normale, réactives à
la lumière.
L'examen des dernières paires crâniennes est sans anomalie. Il n'y a pas de déficit moteur. La sensibilité à tous les modes est
normale. Les réflexes cutanés plantaires sont en flexion. Le F.O. est normal.

Devant ce tableau clinique, vous serez conforté dans votre diagnostic en retrouvant à l'interrogatoire :
A - La notion de troubles transitoires de la sensibilité
B - La notion d'amaurose transitoire
C - Des antécédents de faiblesse transitoire des membres inférieurs
D - L'existence d'une dysphagie transitoire
E - Un épisode de rétention aiguë d'urines
Bonne(s) réponse(s) : C D

Sans commentaire.

Contribura(ont) également au diagnostic :


A - L'électromyographie
B - Le scanner cérébral
C - L'artériographie cérébrale
D - Un test à la Prostigmine
E - Un test à la Mégimide
Bonne(s) réponse(s) : A D

A - Recherche d'un bloc myasthénique.

Votre diagnostic affirmé, vous demandez :


A - Un scanner abdominal
B - Un scanner thoracique
C - Une échographie abdominale
D - Une IRM cérébrale
E - Une étude des potentiels évoqués visuels
Bonne(s) réponse(s) : B

A la recherche d'une hyperplasie thymique ou d'un thymome.

924
Exclusivement sur DOC - DZ : www.doc-dz.com NADJI 85
RESIDANAT EN POCHE TOME II
Cas Clinique en QCM

Vous proposez d'emblée un traitement par :


A - Aspirine
B - Mégimide
C - Prostigmine
D - Atropine
E - Anxiolytiques
Bonne(s) réponse(s) : C

Sans commentaire.

Un homme de 40 ans, agriculteur, est admis pour l'installation brutale alors qu'il soulevait des poids, d'une hémiplégie droite
avec aphasie. Ceci s'est produit peu après le repas de midi. A l'examen, 2 heures après le début des troubles, vous constatez
une hémiplégie droite totale et proportionnelle. La conscience est normale mais il existe une aphasie totale d'expression et de
compréhension. La nuque est souple. Le patient n'a pas vomi. L'auscultation cardiaque est normale, il n'y a pas de souffle
carotidien. La tension artérielle est à 16/9, le pouls à 80/mn. L'ECG enregistre u n rythme sinusal. Le bilan biologique usuel est
normal.
A l'interrogatoire de son épouse, vous apprenez que le patient était jusque là en très bonne santé. Le dernier contrôle de la TA
remonte à un an et était de 13/8. Il ne prend aucun médicament, fume environ 5 cigarettes par jour depuis 15 ans. Son poids
est de 70 kilos pour 1,75m. Sa consommation d'alcool est en moyenne de 1 litre de vin par jour.
Deux jours auparavant, ce patient a présenté une violente douleur irradiant dans la tempe gauche, qui a cédé spontanément
en 24 heures.
Le scanner est normal.
Le doppler cervical montre un très important ralentissement circulatoire sur la carotide interne gauche.
La carotide interne droite est normale.

Cette observation fait évoquer :


A - Un infarctus sylvien gauche total
B - Un infarctus du territoire de la cérébrale antérieure gauche
C - Un hématome temporal gauche
D - Un hématome protubérantiel
E - Une lacune de la capsule interne gauche
Bonne(s) réponse(s) : A

A - Car il existe une hémiplégie proportionnelle (donc atteinte du territoire profond) et une aphasie totale
(donc probablement atteinte corticale étendue).

Les données du Doppler carotidien et de la clinique orientent vers :


A - Une occlusion athéromateuse de la carotide interne gauche
B - Une malformation artérioveineuse
C - Une dissection de la carotide interne gauche
D - Un anévrysme artériel de la bifurcation sylvienne gauche
E - Une embolie d'origine cardiaque
Bonne(s) Réponse(s) : C

C - Car homme jeune, sans FdR vasculaire, surtout douleur violente précédent les troubles. Le doppler montre un
ralentissement, sans occlusion.
Remarque : l'examen clinique aurait dû rechercher un Claude-Bernard-Horner à gauche, très évocateur de dissection.

La cause de ce tableau clinique va être établie au moyen d'un des examens suivants :
A - Etude du LCR
B - Echocardiogramme
C - Un marqueur biologique
D - Angiographie carotidienne
E - Une mesure du Débit Sanguin Cérébral (DSC)
Bonne(s) réponse(s) : D

L'angiographie va rechercher un amincissement "en queue de radis" de la carotide interne gauche.

Après discussion des contre-indications, le traitement instauré sera probablement :


A - Une héparinothérapie
B - La prescription de tartrate d'ergotamine
C - Une corticothérapie
D - La prescription de dihydroergotamine
E - La prescription d'inhibiteurs calciques par voie veineuse
Bonne(s) réponse(s) : A

La dissection de la carotide est très emboligène et constitue une indication aux anticoagulants.

925
Exclusivement sur DOC - DZ : www.doc-dz.com NADJI 85
RESIDANAT EN POCHE TOME II
Cas Clinique en QCM
Une femme de 60 ans se plaint depuis 2 ans d'une douleur latéro-vertébrale gauche qui cède complètement au repos. Depuis
6 mois,cette douleur est devenue permanente et irradie sur la paroi latérale du thorax jusqu'à la région sous-mammelonnaire
gauche La douleur, de type lancinant, est toujours restée unilatérale. Elle n'est pas améliorée par les salicylés.
Le motif de la consultation est en fait la survenue récente, depuis quelques semaines, d'une fatigabilité à la marche qu'elle
ressent tout particulièrement pour monter les trois étages de son domicile. La gêne fonctionnelle s'aggrave régulièrement, ce
qui inquiète d'autant plus la patiente qu'à l'âge de 24 ans elle avait été victime d'une fracture-tassement de L2 responsable
d'une paraparésie transitoire sans aucune symptomatologie neurologique résiduelle.
Il faut enfin noter, dans ses antécédents, une intervention chirurgicale suivie de radiothérapie pour un cancer du sein droit à
l'âge de 48 ans.

Une telle sémiologie doit faire évoquer :


A - Une syringomyélie
B - Un syndrome de la queue de cheval
C - Une compression médullaire lente
D - Une malformation vasculaire médullaire
E - Une myélopathie dégénérative
Bonne(s) réponse(s) : C

Il existe en effet un syndrome lésionnel radiculaire (douleur thoracique en ceinture) précédant le syndrome sous-lésionnel
(déficit des MI).

Quels sont le ou les signes que l'on doit logiquement mettre en évidence :
A - Une dissociation thermo-algésique suspendue
B - Une amyotrophie thénarienne gauche
C - Une aréflexie achilléenne gauche
D - Un syndrome pyramidal
E - Un syndrome méningé
Bonne(s) réponse(s) : D

L'histoire clinique sur 2 ans évoque un processus lent, et rend peu probable une compression épidurale métastatique du
cancer du sein (mais celle-ci doit être recherchée). Le diagnostic le plus probable est celui de méningiome dorsal (compte-
tenu de l'âge de la malade, et que l'association cancer du sein-méningiome est plus fréquente que ne le veut le hasard). Un
neurinome est également possible.
Le traitement sera bien évidemment chirurgical.
En revanche, une épidurite métastatique confirmée relève plus de la radiothérapie en urgence).

Quels sont les examens complémentaires à effectuer ?


A - Un scanner rachidien centré sur D1-D2
B - Un scanner rachidien centré sur D4-D5
C - Un scanner rachidien centré sur Ll-L2
D - Une radiographie de la colonne vertébrale
E - Une scintigraphie osseuse
Bonne(s) réponse(s) : B D E

Sans commentaire.

Devant un tel syndrome clinique, il est justifié de :


A - Prescrire une physiothérapie et des anti-inflammatoires
B - Surveiller la malade et temporiser
C - Demander une EMG des membres inférieurs
D - Pratiquer une saccoradiculographie
E - Réaliser rapidement un traitement chirurgical
Bonne(s) réponse(s) : E

Sans commentaire.

926
Exclusivement sur DOC - DZ : www.doc-dz.com NADJI 85
RESIDANAT EN POCHE TOME II
Cas Clinique en QCM
Un bûcheron de 45 ans souffre de lombalgies depuis plusieurs années. Récemment il a présenté une douleur intense irradiant
dans la fesse gauche, la face antéro-externe de la jambe, le dessus du pied jusqu'au gros orteil. En quelques jours, la douleur
a diminué en même temps qu'il trouvait qu'il ne pouvait plus relever les orteils de son pied gauche et qu'il éprouvait des
difficultés à uriner.

Quel(s) type(s) de sciatique présente ce patient ?


A - Tronculaire
B - Radiculaire L5
C - Radiculaire S1
D - Paralysante
E - Elément d'un syndrome de la queue de cheval
Bonne(s) réponse(s) : B

Sans commentaire.

Quel(s) examen(s) nécessaire(s) au diagnostic étiologique, peu(ven)t être proposé(s) ?


A - Scanner lombaire
B - Radiculographie
C - Scanner de la cuisse gauche
D - Réflexogramme achilléen
E - Myélographie lipiodolée
Bonne(s) réponse(s) : A B

Sans commentaire.

Quel est le diagnostic étiologique le plus probable ?


A - Hernie discale foraminale L4 L5 gauche
B - Hernie discale médiane L5 S1
C - Hernie discale médiane L4 L5
D - Hernie discale foraminale L5 Sl gauche
E - Neurinome de la queue de cheval
Bonne(s) réponse(s) : C

La présence de troubles sphinctériens indique une souffrance des racines sacrées (ce qui ne serait pas le cas dans une
hernie foraminale).

Quelle(s) est (sont) la (les) thérapeutique(s) appropriée(s) ?


A - Chimio-nucléolyse
B - Repos strict au lit pendant 15 jours
C - Infiltration de corticoïdes
D - Tractions isolées suivies du port d'un lombostat
E - Intervention chirurgicale en urgence
Bonne(s) réponse(s) : E

Il s'agit d'un syndrome de compression de la queue de cheval (car troubles sphinctériens) relevant de la chirurgie en urgence.
Remarque : Le caractère paralysant de la sciatique, classiquement indication à la chirurgie en urgence, est actuellement une
indication plus discutable, d'après des études récentes.

Une jeune femme de 28 ans, sans antécédent remarquable (TA : 12/7 à un récent contrôle de la médecine du travail), suivant
une contraception par oestroprogestatif, éprouve subitement un vertige rotatoire intense, accompagné de vomissements, de
paresthésies de la moitié droite du visage et d'une dysphonie sans acouphène. Depuis 4 jours, suite à une chute accidentelle
sans gravité au cours de laquelle elle s'était "tordue le cou", elle se plaignait d'une douleur de la nuque et de l'occiput du côté
droit. L'examen clinique permet les constatations suivantes: nystagmus horizontal (gauche) et rotatoire (horaire), hypoesthésie
de la moitié droite du visage, myosis droit, dysmétrie des membres droits. Hypoesthésie thermoalgique des membres gauches
et de la moitié gauche du tronc.
Vous concluez à un syndrome de Wallenberg.

Quels signes d'examen manquent dans cette description pour que le syndrome de Wallenberg soit complet ?
A - Babinski gauche
B - "Bobbing" oculaire
C - Paralysie hémivoile droit
D - Paralysie hémipharynx droit
E - Paralyse hémilangue droite
Bonne(s) réponse(s) : C D

E - Le XII est médian par rapport au IX et au X : il n'est donc pas touché dans un infarctus latéral du bulbe (Wallenberg).
B - Le "Bobbing" oculaire est constitué par des saccades vers le bas, irrégulières, et traduit généralement des lésions
protubérantielles étendues.
927
Exclusivement sur DOC - DZ : www.doc-dz.com NADJI 85
RESIDANAT EN POCHE TOME II
Cas Clinique en QCM

Ou se situe la lésion responsable ?


A - Pédonculaire
B - Pariétal
C - Pédoncule cérébelleux moyen
D - Protubérance
E - Fossette latérale du bulbe
Bonne(s) réponse(s) : E

Sans commentaire.

Le scanner montre un infarctus cérébelleux. Quel est le mécanisme le plus probable de cet infarctus ?
A - Athérosclérose
B - Embolie cardiaque
C - Dissection artérielle
D - Accident de la pilule
E - Migraine compliquée
Bonne(s) réponse(s) : C

C - La dissection artérielle ici post-traumatique est aussi plus fréquente chez les femmes qui prennent la pilule.

Quelle est la séquelle la plus fréquente du syndrome de Wallenberg ?


A - Troubles vertigineux
B - Troubles de la déglutition
C - Troubles de la phonation
D - Troubles sensitifs
E - Troubles cérébelleux
Bonne(s) réponse(s) : D

A type d'hyperpathie.

Madame P., 30 ans, est hospitalisée en raison de la survenue rapidement progressive sur 24 heures d'une chute de l'acuité
visuelle de l'oeil droit.
Dans ses antécédents, on retient un épisode identique 2 ans auparavant au niveau de l'oeil gauche et une période de troubles
de l'équilibre, régressive en 15 jours environ, il y a un an.
L'examen neurologique met en évidence :
- un syndrome pyramidal des 4 membres
- un nystagmus multidirectionnel
- de discrets troubles de la sensibilité profonde aux membres inférieurs
Le diagnostic de sclérose en plaques est alors évoqué.

Dans ce contexte, la chute rapide de l'acuité visuelle évoque une névrite optique rétrobulbaire (NORB). Parmi
les signes suivants, quel(s) est (sont) celui (ceux) que l'on peut observer dans une NORB :
A - Dyschromatopsie
B - Diplopie
C - Abolition du réflexe cornéen
D - Scotome central
E - Hémianopsie bitemporale
Bonne(s) réponse(s) : A D

Sans commentaire.

Le diagnostic de sclérose en plaques est généralement étayé par les examens complémentaires suivants :
A - Mesure du débit sanguin cérébral
B - Etude du liquide céphalo-rachidien
C - Angiographie
D - Imagerie par résonance magnétique (IRM)
E - Electromyographie
Bonne(s) réponse(s) : B D

B - Le LCR peut montrer une protéinorachie normale ou discrètement augmentée (< 1 g/l), une discrète réaction cellulaire (<
50 éléments), une augmentation oligoclonale des gamma-globulines.
D - L'IRM montre généralement des hypersignaux de la substance blanche en T2.

928
Exclusivement sur DOC - DZ : www.doc-dz.com NADJI 85
RESIDANAT EN POCHE TOME II
Cas Clinique en QCM

Une ponction lombaire est alors réalisée. Citez les éléments biologiques qui seraient compatibles avec le
diagnostic de sclérose en plaques :
A - 8 Lymphocytes par mm3
B - Protéinorachie : 3 g/l
C - Protéinorachie : 0, 70 g/l
D - Glycorachie : 0, 60 mmoles/l
E - Pic oligoclonal des immunoglobulines
Bonne(s) réponse(s) : A C E

Sans commentaire.

Concernant l'imagerie par résonance magnétique (IRM), dans la sclérose en plaques, citez la ou les réponses
justes :
A - Il existe une étroite relation entre le siège des plaques et la symptomatologie clinique
B - Le nombre de plaques de sclérose a une valeur pronostique
C - L'IRM permet à elle seule de faire le diagnostic de sclérose en plaques
D - Des images radiologiques identiques à celles observées dans la sclérose en plaques peuvent révéler
d'autres affections
E - Ces images siègent n'importe où au niveau de la substance blanche du système nerveux central
Bonne(s) réponse(s) : E

Sans commentaire.

Le traitement de la poussée de sclérose en plaques fait appel à :


A - Corticothérapie au long cours
B - D-Pénicillamine
C - Cyclophosphamide (Endoxan®)
D - Aziathioprine (Imurel®)
E - Corticothérapie pendant la poussée
Bonne(s) réponse(s) : E

Sans commentaire.

Une jeune fille de 22 ans, présente une baisse brutale unilatérale de l'acuité visuelle de l'oeil droit. L'interrogatoire retrouve la
notion d'un épisode de paresthésie du membre inférieur droit il y a 3 mois. Auparavant au cours d'une visite de routine, la
vision était de 10/10ème au niveau des 2 yeux.
L'examen retrouve, au niveau de l'oeil droit, une acuité visuelle réduite à une perception lumineuse, alors qu'au niveau de l'oeil
gauche, elle est de 10/10ème. L'examen des fonds d'oeil est sans particularité. D'emblée le diagnostic de sclérose en plaques
est évoqué.

Quel(s) autre(s) diagnostic(s) peut on évoquer devant un tel tableau ?


A - Simulation
B - Adénome hypophysaire
C - Intoxication à l'héroïne
D - Botulisme
E - Glaucome chronique à angle ouvert
Bonne(s) réponse(s) : A

B - La nécrose d'un adénome hypophysaire donne une atteinte chiasmatique (hémianopsie bitemporale) avec céphalées.

S'il s'agit d'une sclérose en plaques par quel mécanisme peut on expliquer la baisse d'acuité visuelle unilatérale
droite ?
A - Atteinte du chiasma
B - Atteinte de la bandelette optique gauche
C - Atteinte du cortex occipital gauche
D - Atteinte du nerf moteur oculaire externe droit
E - Aucune des réponses précédentes n'est exacte
Bonne(s) réponse(s) : E

Atteinte du nerf optique.

929
Exclusivement sur DOC - DZ : www.doc-dz.com NADJI 85
RESIDANAT EN POCHE TOME II
Cas Clinique en QCM

S'il s'agit d'une sclérose en plaques le champ visuel réalisé chez cette patiente, montrera de façon très probable
:
A - Un scotome central unilatéral au niveau du champ visuel de l'oeil droit
B - Une hémianopsie bitemporale
C - Une hémianopsie binasale
D - Une hémianopsie latérale homonyme droite
E - Une hémianopsie latérale homonyme gauche
Bonne(s) réponse(s) : A

Sans commentaire.

Parmi les examens suivants, quel est celui ou quels sont ceux qui seront perturbés s'il s'agit d'une sclérose en
plaques ?
A - Potentiels évoqués visuels
B - Electrorétinogramme
C - Angiographie fluorescéinique
D - Echographie oculaire
E - Aucune des propositions précédentes n'est exacte
Bonne(s) réponse(s) : A

Sans commentaire.

Hormis la baisse de l'acuité visuelle unilatérale, on peut rencontrer au cours de l'évolution d'une sclérose en
plaques :
A - Une cataracte
B - Une baisse d'acuité visuelle bilatérale
C - Un nystagmus
D - Une diplopie
E - Une conjonctivite
Bonne(s) réponse(s) : B C D

B - Oui, mais en général un oeil après l'autre.

L'évolution de cette poussée peut se faire :


A - Vers la récupération fonctionnelle totale
B - Vers une récupération fonctionnelle partielle
C - Vers une perte fonctionnelle totale
D - Vers une atrophie optique
E - Aucune des propositions précédentes n'est exacte
Bonne(s) réponse(s) : A B C D

C - Rare mais possible : en revanche les BAV progressifs dans la SEP sont souvent irréversibles.

Un homme de 38 ans, technicien réparateur en appareils électroménagers, est hospitalisé pour discussion d'une
chimionucléolyse. Lombalgique ancien, il souffre depuis trois semaines d'une lombosciatique droite qui, en dépit d'un signe
inhabituel, est classée L5. Le patient est en arrêt de travail depuis une semaine. La douleur est mécanique, impulsive à la
toux. Le poids est 67 kg la taille 172 cm, l'examen montre un signe de Lasègue droit à 45° et l'absence de signe neurologique.
Dans les antécédents, on note un asthme traité par corticothérapie intermittente depuis 8 ans, une hypercholestérolémie et
une hyperuricémie modérées et asymptomatiques. Le dossier radiographique comporte des clichés standards, une
radiculographie et une scanographie lombaire. Ces examens montrent une petite scoliose structurale lombaire gauche une
spondylolyse de L5, une hernie discale paramédiane droite L4 - L5, une hernie discale foraminale droite L3 - L4 et des kystes
radiculaires des deux racines S1. La nucléolyse ayant été récusée, une injection épidurale de prednisolone est effectuée: la
sciatique disparait et, moyennant quelques prescriptions supplémentaires, le patient est autorisé à reprendre son activité
professionnelle après trois semaines.

Le trajet douloureux étant défini par les 5 éléments suivants, lequel a fait considérer le trajet L5 comme
"inhabituel" ?
A - Région inguinale
B - Face interne de la cuisse
C - Face externe de la jambe
D - Dos du pied
E - Paresthésies gros orteil
Bonne(s) réponse(s) : B

Le trajet L5 est : fesse, face postérieure de la cuisse, face extérieure de la jambe, dos du pied, gros orteil. Mais la douleur
inguinale est elle aussi atypique...Question litigieuse.

930
Exclusivement sur DOC - DZ : www.doc-dz.com NADJI 85
RESIDANAT EN POCHE TOME II
Cas Clinique en QCM

Parmi les lésions constatées, laquelle peut expliquer cette sciatique droite ?
A - La scoliose

B - La spondylolyse de L5

C - La hernie L4 - L5 paramédiane droite

D - La hernie foraminale L3 - L4 droite


E - Les kystes radiculaires de S1
Bonne(s) Réponse(s) : C

Sans commentaire.

Parmi les antécédents du patient, lequel (ou lesquels) a pu (ou ont pu) favoriser la survenue de cette sciatique ?
A - L'asthme

B - La corticothérapie

C - L'hypercholestérolémie

D - L'hyperuricémie
E - La profession exercée
Bonne(s) Réponse(s) : E

Sans commentaire.

Parmi ces mêmes antécédents (et parmi d'autres arguments) l'un a été pris en compte pour récuser la
chimionucléolyse. Lequel ?
A - L'asthme

B - La corticothérapie

C - L'hypercholestérolémie

D - L'hyperuricémie
E - La profession exercée
Bonne(s) Réponse(s) : A

La chimionucléolyse (par chymopropaïne) peut provoquer des accidents allergiques (donc à éviter sur terrains atopiques) ; de
toute façon, ses résultats sont inférieurs à ceux de la chirurgie.

Toutes les lésions citées ont été mises en évidence par une ou plusieurs des techniques radiologiques utilisées
; l'une d'entre elles ne pouvait être mise en évidence que par la scanographie. Laquelle ?
A - La scoliose

B - La spondylolyse

C - La hernie paramédiane L4-L5

D - La hernie foraminale L3-L4


E - Les kystes radiculaires
Bonne(s) Réponse(s) : B

A - Mise en évidence uniquement sur les radiographies


B - Spondylolyse = défaut osseux dans la partie interarticulaire de la vertèbre. Peut être mis en évidence par incidences
radiologiques obliques, mais probablement mieux vue par le TDM.

931
Exclusivement sur DOC - DZ : www.doc-dz.com NADJI 85
RESIDANAT EN POCHE TOME II
Cas Clinique en QCM
Une jeune femme de 30 ans consulte pour des épisodes de diplopie transitoire associée à une asthénie. L'interrogatoire ne
trouve aucun antécédent notable, aucune prise médicamenteuse. Les troubles remontent à 6 mois environ, soit 2 mois après
sa première grossesse, et ont débuté de façon insidieuse. La patiente allègue des troubles visuels en fin de journée, avec une
chute des paupières. Elle est gênée dans ses activités ménagères quotidiennes, en particulier pour soulever des poids,
l'ensemble des signes se majorant en fin de semaine. On relève enfin une perte de poids de 3 kgs en 3 mois.
L'examen clinique standard, réalisé en fin de matinée, ne retrouve qu'une diplopie dans les regards extrêmes, et une très
discrète diplégie faciale. Le reste de l'examen neurologique et général est normal.
Vous évoquez une myasthénie.

Concernant les mécanismes physiopathologiques de la myasthénia gravis, il s'agit :


A - D'une dystrophie musculaire
B - D'une maladie autoimmune
C - D'un blocage des récepteurs cholinergiques présynaptiques de la jonction neuromusculaire
D - D'une affection héréditaire
E - D'un blocage des récepteurs cholinergiques post-synaptiques de la jonction neuromusculaire
Bonne(s) réponse(s) : B E

C - C'est le cas du syndrome de Lambert-Eaton.

Citez, parmi les examens complémentaires suivants, ceux qui permettent d'affirmer le diagnostic de myasthénie
:
A - Electromyographie
B - Potentiels évoqués sensitifs
C - Test à la prostigmine
D - Scanner cérébral
E - Dosage des anticorps anti-récepteurs à l'acétylcholine
Bonne(s) réponse(s) : A C E

Sans commentaire.

La découverte d'une myasthénie impose :


A - Un scanner cérébral
B - Un scanner thoracique
C - La recherche d'une affection auto-immune associée
D - Un arbre généalogique
E - Des potentiels évoqués visuels
Bonne(s) réponse(s) : B C

A la recherche d'une tumeur thymique, fréquente, qui impose son exérèse.

Parmi les traitements suivants, quel est celui qui est utilisé dans le traitement symptomatique d'une myasthénie
?
A - Corticoïdes au long cours
B - Plasmaphérèses
C - Anticholinestérasiques
D - Thymectomie
E - Immunosuppresseurs
Bonne(s) réponse(s) : C

A B D E - Font tous partie du traitement de fond de la maladie.

Citez, parmi les signes suivants, ceux qui peuvent être secondaires à un surdosage en anticholinestérasiques :
A - Constipation
B - Myosis
C - Diarrhée
D - Crampes
E - Tachycardie
Bonne(s) réponse(s) : B C D

Effet muscarinique = hyperactivité parasympathique.

932
Exclusivement sur DOC - DZ : www.doc-dz.com NADJI 85
RESIDANAT EN POCHE TOME II
Cas Clinique en QCM
Un homme de 45 ans, exerçant un métier d'effort puisqu'il est mineur se plaint depuis plus de 8 mois d'une douleur du
membre supérieur droit qui, progressivement, a diffusé jusqu'à l'extrémité distale.
Maintenant le patient ressent parfaitement des paresthésies électriques dans les premiers et 2ème doigts de la main droite,
notamment lorsqu'il penche la tête en avant ou qu'il fait un effort. Depuis quelques semaines, ce patient droitier accuse une
diminution nette et évolutive de la force musculaire de la main droite, notamment à la préhension, qui l'a obligé à interrompre
son travail depuis 8 jours.
Dans ses antécédents, on note des épisodes fréquents et anciens de cervicalgie s'accompagnant parfois de blocage cervical
à type de torticolis. Sa colonne cervicale a été l'objet de traumatisme et de surmenage d'autant plus important qu'en dehors de
son métier il a joué au rugby pendant plus de 15 ans en première ligne. Il se plaint d'ailleurs aussi de lombalgies récidivantes
irradiant dans les deux fesses avec une prédominance droite. Enfin, il a une maladie professionnelle pulmonaire silicotique.

La sémiologie évoque avant tout :


A - Un syndrome du canal carpien
B - Une atteinte du cubital au coude
C - Une compression du plexus brachial
D - Une radiculalgie cervicale
E - Une souffrance de la corne antérieure médullaire
Bonne(s) réponse(s) : D

Sans commentaire.

L'examen clinique peut logiquement mettre en évidence :


A - Une aréflexie tricipitale
B - Une amyotrophie hypothénarienne
C - Une aréflexie bicipitale droite
D - Une griffe cubitale
E - Une hypoesthésie des 1er et 2ème doigts
Bonne(s) réponse(s) : E

Mais on peut retrouver d'autres signes témoignant de l'atteinte d'autres racines que C6 : en particulier, il semble exister un
déficit moteur de type C8 et/ou D1, et on pourrait retrouver une amyotrophie hypothénarienne. C'est donc littigieux.

Quelles sont les explorations complémentaires à demander en priorité ?


A - Un scanner vertébral cervical
B - Des radiographies cervicales de 3/4
C - Un EMG du membre supérieur
D - Une scintigraphie osseuse vertébrale
E - Une artériographie vertébrale
Bonne(s) réponse(s) : A B

Sans commentaire.

Quelle est l'hypothèse diagnostique la plus probable ?


A - Une tumeur de l'apex pulmonaire droit
B - Une hernie discale cervicale
C - Un syndrome canalaire du membre supérieur droit
D - Une dégénérescence des facettes articulaires cervicales
E - Un canal vertébral étroit
Bonne(s) réponse(s) : E

Très littigieux : canal étroit avec des lésions discales étayées (en particulier névralgie cervico-brachiale C6).
Si on avait répondu B, on aurait été gêné pour expliquer la topographie des troubles moteurs (préhension) de type C8-D1). Le
canal cervical vertébral est étroit, souvent associé en outre à un canal lombaire étroit (probablement à l'origine des
lombalgies).

Quelles sont les attitudes thérapeutiques licites ?


A - Une manipulation vertébrale cervicale
B - Une association antalgique-anti-inflammatoire décontractant et physiothérapique sous surveillance
médicale
C - Une radiothérapie anti-inflammatoire
D - Une immobilisation par minerve
E - Une chirurgie décompressive
Bonne(s) réponse(s) : A D E

Sans commentaire.

933
Exclusivement sur DOC - DZ : www.doc-dz.com NADJI 85
RESIDANAT EN POCHE TOME II
Cas Clinique en QCM
Une jeune femme de 38 ans est hospitalisée pour des céphalées brutales diffuses et une paralysie oculomotrice: on constate
à l'examen clinique de l'oeil droit un ptosis, une mydriase, un strabisme divergent avec diplopie. Le F.O. est normal. La
conscience est normale. Il n'y a pas d'antécédent notable. La T.A et à 13/9. L'auscultation du coeur et des vaisseaux est
normale. Le scanner révèle une hémorragie sous arachnoïdienne diffuse de faible abondance. L'artériographie cérébrale met
en évidence un anévrysme intracrânien.

S'il s'agit d'une paralysie complète du nerf moteur oculaire commun, on doit constater les anomalies suivantes :
A - Paralysie de l'élévation du globe oculaire
B - Paralysie de l'abaissement du globe oculaire
C - Paralysie de l'adduction du globe oculaire
D - Paralysie de l'abduction du globe oculaire
E - Paralysie pupillaire
Bonne(s) réponse(s) : A B C E

D - Abduction = moteur oculaire externe = VI.

Quelle est la localisation la plus probable de cet anévrysme ?


A - Artère communicante antérieure
B - Artère cérébrale moyenne
C - Terminaison de l'artère carotide interne
D - Artère vertébrale
E - Artère cérébrale postérieure
Bonne(s) réponse(s) : C

Anévrysme supraclinoïdien de la carotide interne.


L'autre localisation possible (plus rare) étant la terminaison du tronc basilaire, à l'émergence du III.

Quels sont les risques d'évolution spontanée ?


A - Hydrocéphalie aiguë
B - Récidive hémorragique
C - Atrophie optique
D - Spasme artériel
E - Fistule artérioveineuse
Bonne(s) réponse(s) : A B D

Sans commentaire.

Dans l'attente d'une intervention chirurgicale, quelles mesures thérapeutiques sont nécessaires ?
A - Antalgiques non salicylés
B - Antiépileptique
C - Repos en décubitus
D - Inhibiteur calcique
E - Fibrinolytiques
Bonne(s) réponse(s) : A B C D

Sans commentaire.

934
Exclusivement sur DOC - DZ : www.doc-dz.com NADJI 85
RESIDANAT EN POCHE TOME II
Cas Clinique en QCM

935
Exclusivement sur DOC - DZ : www.doc-dz.com NADJI 85
RESIDANAT EN POCHE TOME II
Cas Clinique en QCM
Un homme de 54 ans est hospitalisé pour un syndrome fébrile, ayant débuté brutalement par des frissons, des céphalées, une
ascension thermique à 40°2. La famille vous signale un éthylisme chronique certain, une allergie connue aux bêta-lactamines.
Vous observez une toux grasse, ramenant une expectoration rouillée, et une dyspnée peu importante avec tachypnée. La
température est à 39°9 degrés. A l'examen existent une légère cyanose. une sub-matité de la base droite au sein de laquelle
est perçu un souffle tubaire typique, entouré d'une couronne de râles sous-crépitants fins. Un premier bilan biologique en
urgence donne :
Numération formule sanguine : pas d'anémie, 13.800 leucocytes/mm3 avec 84 % de polynucléaires neutrophiles Gaz du sang
artériel : PaO2 62 mmHg (8.4 kpa), PaC02 35 mmHg (4.8 kpa), pH 7,48. Dans les heures qui suivent apparaissent deux
nouveaux éléments cliniques :
un herpès péri-buccal et des troubles neuro-psychiques : tremblements, Iogorrhée, agressivité, délire.

La radiographie pulmonaire devrait mettre a priori en évidence :


A - Des opacités floues disséminées dans les deux champs
B - Une atélectasie du lobe moyen
C - Une image hydroaérique du sommet du lobe inférieur droit
D - Une opacité systématisée non rétractile du lobe inférieur droit, dense et homogène
E - Une image miliaire
Bonne(s) réponse(s) : D

Le tableau clinique est celui d'une pneumonie franche lobaire aiguë. La radiologie retrouve une opacité dense et homogène
bien limitée, systématisée, de type alvéolaire, sans signes d'atétectasie.

Les désordres neuro-psychiques observés sont dûs à :


A - Une insuffisance respiratoire aiguë
B - Un délirium tremens
C - Une méningite
D - Une septicémie
E - La fièvre élevée
Bonne(s) réponse(s) : B

A - Faux : hypoxie modérée.


B - Vraisemblable devant le terrain et les troubles neuropsychiques.
C - En cas de doute, la PL s'impose.

Vous prescrivez immédiatement :


A - Pénicilline G
B - Macrolide
C - Céphalosporine
D - Aminoside
E - Cycline
Bonne(s) réponse(s) : A

La pénicilline est le traitement de choix du pneumocoque.

Un traitement complémentaire doit être institué, qui comprendra :


A - Vitamine A
B - Une réhydratation par voie veineuse
C - De l'Equanil® injectable
D - Une oxygénothérapie nasale
E - Antivitamine K
Bonne(s) réponse(s) : B C

Connaissance.

936
Exclusivement sur DOC - DZ : www.doc-dz.com NADJI 85
RESIDANAT EN POCHE TOME II
Cas Clinique en QCM
Monsieur L. 55 ans, est admis à l'hôpital pour un syndrome fébrile à 39°5 d'apparition brutale, avec frissons et toux, ramenant
une expectoration purulente.
Ses antécédents sont représentés par une HTA traitée par furosémide et une tuberculose, bactériologiquement prouvée, à
l'âge de 18 ans, non traitée et dont il garde des séquelles au niveau de l'apex droit. Il a fumé deux paquets de cigarettes par
jour pendant 25 ans.
De l'interrogatoire, on retient une altération de l'état général récente avec un amaigrissement de 5 kg en 3 mois, et deux
épisodes hémoptoïques minimes. L'examen permet de retrouver un herpès labial et une matité de la base droite.
Radiologiquement, il existe une opacité systématisée du lobe inférieur droit accompagnée d'un épanchement pleural
homolatéral et d'adénopathies médiastinales.
Il existe une hyperleucocytose à 20 000 GB/mm3 à prédominance de polynucléaires (85 %), sans anémie. La vitesse de
sédimentation est accélérées (70 mm à la 1 ère heure). La fonction rénale et le bilan biologique hépatique sont normaux. La
natrémie est à 125 mmoles/l sans autre anomalie métabolique.

La consommation tabagique peut être chiffrée en paquets année. Dans cette observation, quelle est la valeur
obtenue ?
A - 12, 5 paquets-année
B - 25 "
C - 37,5 "
D - 50 "
E - Aucune de ces propositions
Bonne(s) réponse(s) : D

Pour résumer la consommation tabagique, il est pratique de compter la quantité totale de paquets fumés au cours de
l'existence. Un paquet par jour pendant un an (ou 20 cigarettes par jour pendant un an) est qualifié de 1 paquet-année.

Il faut évoquer comme diagnostic :


A - Tuberculose
B - Embolie pulmonaire
C - Cancer bronchique
D - Pneumopathie aiguë
E - Pleurésie
Bonne(s) réponse(s) : A C D E

A D - On retient le diagnostic de pneumonie franche lobaire aiguë.


Le diagnostic de pneumopathie à pneumocoque est évoqué devant le début brutal, l'expectoration purulente, la présence d'un
herpès labial, l'hyperleucocytose à polynucléaires et l'image radiologique.
De principe, on effectuera les recherches bactériologiques pour une mycobactérie en raison des antécédents, de l'absence
initiale de traitement antituberculeux.
C - Les pneumopathies aiguës infectieuses sont une circonstance de découverte des cancers bronchiques. On retiendra ce
diagnostic devant l'altération de l'état général, les hémoptysies, les adénopathies médiastinales avec épanchement pleural
(Item E).
E - Le syndrome pleural peut être dû à une pleurésie réactionnelle inflammatoire, une pleurésie purulente ou un
envahissement néoplasique.

Vous proposez initialement :


A - Numération formule sanguine
B - Hémoculture
C - Ponction pleurale
D - Epreuves fonctionnelles respiratoires
E - Endoscopie trachéo-bronchique
Bonne(s) réponse(s) : A B C

Les examens effectués d'emblée permettront un diagnostic bactériologie : hémocultures, ponction pleurale exploratrice, voire
prélèvements bactériologiques au-décours de l'endoscopie bronchique.
Secondairement une endoscopie bronchique permettra de découvrir un obstacle (par compression extrinsèque ou obstacle
endobronchique) et de faire des prélèvements biopsiques.

Un syndrome de compression cave supérieure apparaît. Celui-ci peut comporter :


A - Une circulation collatérale thoracique
B - Une paralysie récurrentielle
C - Un oedème en pèlerine
D - Des télangiectasies
E - Une paralysie phrénique droite
Bonne(s) réponse(s) : A C D

Le syndrome cave supérieur comporte des céphalées, un oedème en pélerine, une cyanose faciale, une turgescence
veineuse sous-cutanée avec varicosités et lacis veineux (thoracique supérieur et sublingual).
Une paralysie phrénique droite (paralysie diaphragmatique, hoquet) ou récurrentielle (dysphonie) sont des signes de
compression médiastinale sans rapport direct avec une compression de la veine cave supérieure.

937
Exclusivement sur DOC - DZ : www.doc-dz.com NADJI 85
RESIDANAT EN POCHE TOME II
Cas Clinique en QCM

Sachant que le diagnostic retenu sera celui de cancer à petites cellules, le bilan d'extension doit comporter :
A - Echographie hépatique
B - Scintigraphie osseuse
C - Calcémie
D - Scanner cérébral
E - Biologie hépatique
Bonne(s) réponse(s) : A B C D E

Le bilan d'extension permet ici d'apprécier l'étendue du cancer avant chimiothérapie :


recherche de métastases hépatiques (cholestase biologique, échographie), osseuses (hypercalcémie lytique, scintigraphie
osseuse), surrénaliennes (échographie) et système nerveux central (scanner cérébral).
NB : une hypercalcémie peut être d'origine néoplasique.

L'enfant T., 8 ans, vous est adressé pour virage récent de l'intradermoréaction tuberculinique à 10 U. Il n'a jamais été vacciné
par le BCG. Il ne présente aucun symptôme La radiographie thoracique révèle une volumineuse adénopathie hilaire droite.
Les recherches de BK se révèlent négatives à 3 reprises. Le bilan biologique ne montre qu'une discrète élévation de la VS (25
mm à la 1 ère heure). Cet enfant est le quatrième d'une famille de 6 enfants. Le père est manoeuvre et a eu récemment une
radiographie thoracique considérée comme normale par la médecine du travail. La mère et la grand-mère paraissent en
bonne santé et n'ont pas d'activité professionnelle. Quelques jours après, cet enfant présente une dyspnée aiguë et une
expectoration qui justifient une hospitalisation. La radiographie montre l'apparition d'une opacité rétractile du lobe supérieur
droit.

Il est nécessaire chez les frères et soeurs de pratiquer :


A - Une radiographie pulmonaire
B - Des réactions tuberculiniques
C - Une recherche de BK
D - Un contrôle radiographique quelques mois plus tard si le premier cliché est normal
E - Un traitement de 6 mois par Isoniazide
Bonne(s) réponse(s) : A B

A B - Evident non commenté.


E - Une chimioprophylaxie n'est concevable qu'en l'absence de tout signe clinique et/ou radiologique de tuberculose évolutive
(INH® pendant 6 mois).

Le traitement, avant l'épisode de dyspnée, aurait dû être :


A - Isoniazide seul pendant 6 mois
B - Isoniazide + rifampicine + éthambutol
C - Abstention thérapeutique
D - Exérèse chirurgicale du ganglion médiastinal
E - Aucune des propositions n'est exacte
Bonne(s) réponse(s) : B

Une primo-infection tuberculeuse patente impose un traitement anti-tuberculeux classique (trithérapie pendant 9 mois ou
quadrithérapie pendant 6 mois). A noter que chez l'enfant la dose d'INH® est de 10 mg/kg/24 h en l'absence de taux sérique
d'izoniazidémie.

L'épisode aigu récent :


A - Correspond à une fistule ganglio-bronchique
B - Impose une endoscopie bronchique
C - Justifie une corticothérapie
D - Est une indication chirurgicale immédiate
E - Nécessite de renouveler la recherche de BK
Bonne(s) réponse(s) : B D E

C - Faux, la corticothérapie est contre-indiquée dans les adénopathies latéro-trachéales ou juxta bronchiques menaçant de se
fistuliser.
Une fistule gangliobronchique sera recherchée (item B) et les recherches de BK renouvelées (item E) mais il peut s'agir d'un
trouble de ventilation isolé. La menace de fistulisation est une indication à une chirurgie d'exérèse (item D).

938
Exclusivement sur DOC - DZ : www.doc-dz.com NADJI 85
RESIDANAT EN POCHE TOME II
Cas Clinique en QCM

Il faut craindre, malgré le traitement, la survenue de :


A - Greffe aspergillaire
B - Sténose bronchique
C - Localisation méningée
D - Miliaire
E - Hémoptysie
Bonne(s) réponse(s) : B D

D - Dans 10 à 20 % des cas évolution de la primo-infection tuberculeuse vers une tuberculose évolutive.
B - Des complications loco-régionales sont possibles.
E - Une hémoptysie serait la conséquence lointaine de dystrophies bronchiques localisées ou de broncholithiase.

Cet homme de 58 ans vient vous consulter car il tousse la nuit et n'arrive plus à dormir. Cela dure depuis une dizaine de jours
et va en augmentant. Il a commencé à être asthénique il y a un mois et demi ; il avait un peu de fièvre et un confrère lui avait
prescrit du Clamoxil® (amoxicilline) pour 8 jours. Pendant une quinzaine de jours il a noté une stabilisation, mais
progressivement l'asthénie a progressé et la fièvre est réapparue. Il a un peu maigri (1,5 kg). De plus il est intrigué par
l'apparition d'une petite tuméfaction à l'extrémité du 2ème orteil gauche, douloureuse, depuis le matin même.
L'examen vous montre un sujet un peu pâle, avec des vaisseaux du cou nettement pulsatiles, la TA est à 14/5. La tuméfaction
de l'orteil est localisée, centrée par un point blanc. La peau autour est normale. L'examen pulmonaire montre uniquement des
râles humides et fins aux deux bases. L'auscultation cardiaque montre un souffle qui n'était pas connu : il est systolique 3/6,
au 4ème espace intercostal gauche, irradiant franchement vers la base, et jusque dans les vaisseaux du cou. Il s'accompagne
d'un frémissement. Il est suivi d'un deuxième bruit un peu atténué, et d'un souffle diastolique 2/6 qui est maximum au 4ème
espace intercostal droit. En décubitus latéral gauche, vous percevez un bruit de galop. Il n'y a pas d'adénopathie ou de
splénomégalie.
Les urines montrent une protéinurie très discrète et une hématurie microscopique.

Quel examen complémentaire vous parait devoir être réalisé en priorité ?


A - Electrocardiogramme
B - Echocardiogramme
C - Hémocultures
D - Hémogramme et vitesse de sédimentation
E - Cliché pulmonaire
Bonne(s) réponse(s) : C

L'endocardite infectieuse est à évoquer en priorité donc C.

Quelle(s) mesure(s) thérapeutique(s) vous parai(ssen)t adaptée(s) dans l'immédiat ?


A - Arrêt de travail et repos à domicile
B - Hospitalisation immédiate
C - Reprise du Clamoxil®
D - Lasilix® 40 mg, 1 comprimé
E - Kenarcort retard® (triamicinolone), 1 ampoule intramusculaire
Bonne(s) réponse(s) : B D

Evident, Lasilix® contre l'insuffisance cardiaque.


E - incongru.

Vu le contexte clinique, vous allez vraisemblablement constater :


A - ECG : rythme sinusal et bloc de branche gauche
B - Echocardiogramme TM et bidimensionnel : surface mitrale inférieure à 2 cm2, oreillette gauche
dilatée ; pas d'autre anomalie retrouvée

C - Hémocultures : au 3ème jour pas encore de germe qui pousse


D - Leucocytose à 14.000 GB dont 85 % de polynucléaires et vitesse de sédimentation à 80 à l'heure
E - Cliché pulmonaire : diminution de la transparence pulmonaire aux deux bases, présence de lignes de
Kerley et cardiomégalie
Bonne(s) réponse(s) : A C D E

Tout est possible ici sauf (B) qui signe un rétrécissement mitral.

939
Exclusivement sur DOC - DZ : www.doc-dz.com NADJI 85
RESIDANAT EN POCHE TOME II
Cas Clinique en QCM

En fonction des données de divers examens complémentaires, vous avez fixé un traitement. Au bout d'une
semaine, la température s'est normalisée, mais survient brusquement une aphasie, heureusement régressive.
Vous en concluez :
A - Que le traitement est inefficace
B - Que le traitement n'est pas nécessairement inefficace et peut être conservé sans changement
C - Qu'il faut refaire des examens complémentaires après 48 heures d'arrêt du traitement en cours
D - Que le traitement doit être radicalement changé
E - Qu'une ponction lombaire est nécessaire pour ajuster la suite du traitement
Bonne(s) réponse(s) : B

Une embolie ne signifie pas que l'infection n'est pas maîtrisée.

Au bout de deux semaines, la température reste normalisée mais l'état clinique ne s'améliore pas. Sur
l'échocardiogramme une valvule mitrale se fermant au début de l'onde P de l'ECG, votre attitude comporte :
A - Refaire des examens complémentaires après 48 heures d'arrêt du traitement en cours
B - Changer de traitement médical et se donner une semaine pour juger du résultat
C - Faire un cathétérisme cardiaque
D - Exiger le repos au lit strict
E - Demander au chirurgien d'intervenir
Bonne(s) réponse(s) : C D E

C'est l'heure de la chirurgie !

Monsieur P., 55 ans, fumeur à 30 paquets année, a exercé de 1953 à 1973 la profession de mineur de fonds dans le bassin
houiller de Lorraine. Il est employé de banque depuis 1973. Il est hospitalisé le 8 décembre 1983 pour hémoptysies et bilan
d'une dyspnée d'effort apparue 5 ans auparavant et d'intensité progressivement croissante. L'examen clinique permet de noter
un hippocratisme digital et des râles bronchiques diffus. Le cliché thoracique montre des opacités nodulaires diffuses et des
masses pseudo-tumorales au niveau des deux lobes supérieurs. Les résultats des explorations fonctionnelles respiratoires et
de la gazométrie artérielle sont les suivantes :

Valeurs théoriques Valeurs observées


VEMS. (ml) 2 800 1 200
CV. (ml) 3 750 2 400
VEMS. /C. V. x 100 75 50
CT. (ml) 5 300 4 800
Pa02 (torr) 63
PaC02 (torr) 32

Les EFR et la gazométrie objectivent :


A - Un trouble ventilatoire obstructif pur
B - Un trouble ventilatoire restrictif pur
C - Un trouble ventilatoire mixte à prédominance obstructive
D - Une hypoventilation alvéolaire
E - Une hyperventilation alvéolaire
Bonne(s) réponse(s) : A E

A B C - VEMS/CV est inférieur à 75 % définissant un trouble ventilatoire obstructif. La CT est à 90° de la théorique donc il n'y
a pas de trouble ventilatoire restrictif associé.
D - Est définie par l'association hypoxie-hypercapnie.

Quelle(s) affection(s) I'hippocratisme digital doit-il faire rechercher ?


A - Bronchite chronique
B - Cancer bronchique
C - Mycose pulmonaire
D - Dilatation des bronches
E - Pneumopathie bactérienne
Bonne(s) réponse(s) : C D

A - En principe, pas dans la bronchite chronique et doit faire rechercher un cancer bronchique, une dilatation des bronches ou
une fibrose.
D - Fréquent dans les formes diffuses de l'adulte.

940
Exclusivement sur DOC - DZ : www.doc-dz.com NADJI 85
RESIDANAT EN POCHE TOME II
Cas Clinique en QCM

Chez ce patient fumeur et silicotique, quel(s) diagnostic(s) les hémoptysies doivent elles faire évoquer ?
A - Infection bronchique
B - Tuberculose pulmonaire
C - Cancer bronchique
D - Aspergillome
E - Vidange d'une masse pseudo-tumorale
Bonne(s) réponse(s) : B C E

C - De principe chez ce patient de 55 ans tabagique.


B - Les surinfections tuberculeuses sont fréquentes mais souvent torpides et méconnues. La rareté de l'isolement du BK
autorise des traitements spécifiques sans preuve bactériologique. Il peut s'agir de mycobactéries atypiques.
Le diagnostic sera évoqué devant un fébricule, une hémoptysie, une accélération de la VS, des modifications radiologiques
rapides.
E - Rare.

Pour que ce patient puisse être indemnisé, qui doit faire la déclaration de maladie professionnelle ?
A - Le médecin conseil de la sécurité sociale
B - Le patient lui-même
C - Le médecin expert en pneumoconiose
D - Le médecin du travail
E - Le médecin traitant
Bonne(s) réponse(s) : C

Non commenté, connaissance.

Une jeune femme de trente ans, qui a présenté dans l'enfance une primo-infection tuberculeuse grave correctement traitée
par les antibiotiques antituberculeux, présente depuis plusieurs années des épisodes infectieux à répétition, avec
bronchorrhée purulente. Lors de ces épisodes, l'examen révèle une submatité et des râles bronchiques localisés dans la
région antérolatérale droite, à hauteur des 5 et 6e espaces intercostaux. Le cliché radiologique objective une opacité
triangulaire basale droite, à base médiastinale, correspondant sur le cliché de profil, à la zone de projection du lobe moyen.
Sur les tomographies de face, on distingue une opacité ganglionnaire hilaire droite.

Parmi les investigations suivantes, quelle(s) est (sont) celle(s) que vous mettrez en oeuvre pour préciser le
diagnostic ?
A - Fibroscopie endobronchique
B - Bronchographie droite
C - Scanner thoracique
D - Echographie thoracique
E - Lavage bronchoalvéolaire dans le territoire moyen
Bonne(s) réponse(s) : A B

Le diagnostic suspecté est celui de dilatation des bronches devant des épisodes infectieux à répétition avec bronchorrhée et
trouble de ventilation du lobe moyen.
A - L'endoscopie bronchique est le premier temps des examens à visée diagnostique. Elle permet de préciser la localisation
des bronches inflammatoires, de rechercher une cause locale, d'effectuer des prélèvements bactériologiques et de préparer la
bronchographie.
B - La bronchographie permet l'analyse topographique et morphologique des dilatations des bronches.
C - L'évaluation de l'intérêt du scanner thoracique par rapport à la bronchographie est en cours.
D E - Non intéressants dans cette pathologie.

Les examens pratiqués confirment l'existence d'un syndrome du lobe moyen secondaire à une primo-infection
tuberculeuse de l'enfance. Parmi les éléments cliniques et paracliniques suivants, quel(s) est(sont) celui(ceux)
qui évoque(nt) cette affection ?
A - Présence de bacilles tuberculeux à l'examen de l'expectoration
B - Sténose partielle de la bronche lobaire moyenne
C - Présence de bronchectasies lobaires moyennes
D - Surinfection aspergillaire
E - Hémoptysies
Bonne(s) réponse(s) : B C

B C - Un syndrome du lobe moyen est un collapsus lobaire moyen par obstruction bronchique proximale extrinsèque. Cette
sténose est responsable d'une rétention puis d'ectasies bronchiques.
A D E - Non évocateurs d'une localisation lobaire moyenne.

941
Exclusivement sur DOC - DZ : www.doc-dz.com NADJI 85
RESIDANAT EN POCHE TOME II
Cas Clinique en QCM

Ce syndrome du lobe moyen est une complication lointaine de la primo-infection tuberculeuse de l'enfance.
Parmi les manifestations cliniques suivantes quelle(s) est (sont) celle(s) qui peu(ven)t également survenir à titre
de complication de sa primo-infection tuberculeuse ganglionnaire médiastinale ?
A - Dysphonie
B - Dilatation bronchique diffuse des deux lobes inférieurs
C - Syndrome de Claude Bernard Horner
D - Broncholithiase
E - Aucune de ces manifestations
Bonne(s) réponse(s) : D

D - La broncholithiase résulte d'une calcification ganglionnaire post-tuberculeuse juxta-bronchique qui a perforé la paroi. La
perforation ou la migration entraînent hémoptysies, toux incoercible parfois suivie d'un rejet de concrétions.

En dehors de la primo-infection tuberculeuse, quelle(s) est(sont) la(les) autre(s) étiologie(s) possible(s) de


syndrome du lobe moyen ?
A - Mucoviscidose
B - Tumeur "bénigne" de la lobaire moyenne
C - Adénopathie hilaire sarcoïdosique
D - Coqueluche
E - Aucune de ces étiologies
Bonne(s) réponse(s) : E

A - Responsable de dilatations des bronches acquises diffuses.


B - Une lésion endobronchique avec collapsus du lobe moyen n'est pas classée dans les syndromes du lobe moyen (voir
question[24]).
C - Classiquement adénopathies non compressives.
D - Responsable d'un syndrome de Mac Leod avec altérations bronchiques et vasculaires.

Parmi les tactiques thérapeutiques suivantes, quelle est celle que vous mettrez en oeuvre chez cette patiente ?
A - Traitement antibiotique antituberculeux pendant 6 à 9 mois
B - Traitement par laser endobronchique
C - Exérèse du lobe moyen
D - Abstention thérapeutique
E - Instillation répétées d'antibiotiques dans la bronche lobaire moyenne

Bonne(s) réponse(s) : C

A - Le traitement anti-tuberculeux a été déjà correctement conduit. Il s'agit ici de séquelles tuberculeuses.
B - Non car la compression est extrinsèque.
C - Le traitement chirurgical est indiqué : dilatation des bronches localisée unilatérale symptomatique malgré le traitement
médical bien conduit.
D E - Faux, évident.

Monsieur X, 56 ans manoeuvre dans une entreprise de terassement est hospitalisé le 9/10/1984 en urgence pour un
syndrome hyperthermique (40°C) avec malaise général, frissons, céphalées, myalgies. Il est très polypnéique ( 40 cycles
respiratoires par minute), la toux ramène une expectoration peu abondante de caractère muco-purulent. L'examen clinique
objective un syndrome de condensation lobaire supérieur gauche. Il s'y associe des signes gastro-intestinaux avec diarrhée,
douleurs abdominales, des signes neurologiques à type d'obnubilation, avec bouffées délirantes. Le bilan biologique réalisé
en urgence objective une hyperleucocytose modérée, avec une hyponatrémie (Na : 130 mM/l) et une hypophosphorémie. Les
gaz du sang retrouvent une hypoxémie à 52 mm Hg avec hypocapnie (PaCO2 : 30 mmHg).
Après 72 heures de traitement par pénicilline (10. M unités/24 heures), l'état du malade tend à s'aggraver avec extension de
l'image radiologique au poumon controlatéral, amenant à réviser l'hypothèse daignostique et la thérapeutique.

Compte tenu du tableau clinique décrit ci-dessus, devant un syndrome de comblement alvéolaire, une
hypothèse diagnostique mérite d'être évoquée, laquelle ?
A - Pneumopathie aiguë bactérienne
B - Tuberculeuse militaire
C - Lymphome à localisation pulmonaire
D - Cancer bronchio-alvéolaire
E - Carcinome anaplasique à petites cellules
Bonne(s) Réponse(s) : A

Non commenté : évident.

942
Exclusivement sur DOC - DZ : www.doc-dz.com NADJI 85
RESIDANAT EN POCHE TOME II
Cas Clinique en QCM

Chez notre patient un certian nombre d'arguments cliniques et biologiques orientent le diagnostic en faveur
d'une infection à legionella pneumophila. Le(s)quel(s) ?
A - Coexistence de signes pulmonaires et digestifs
B - Syndrome d'hypoxie - hypercapnie
C - Hyperthermie grave
D - Résistance à la pénicilline
E - Myalgies
Bonne(s) Réponse(s) : A D

Une légionnellose doit être évoquée devant une pneumopathie aiguë quant il existe des signes digestifs
(diarrhée, douleur abdominales) et neurologiques (prostration, syndrome confusionnel avec PL normale).
Biologiquement sont évocateurs une atteinte hépatique, une hyponatrémie, une élévation des CPK.
La résistance après 48 h de traitement par bêta-lactamines est un argument supplémentaire et impose l'emploi des
tétracyclines.

Devant une suspicion d'infection à la légionella pneumophila. Quel est le moyen diagnostique utilisé en pratique
courante ?
A - Isolement du germe (dans les sécrétions bronchiques ou le sang)
B - La recherche de legionella par immunofluorescence sur produits pathologiques
C - La recherche d'anticorps spécifiques dans le sérum
D - La biopsie pulmonaire transbronchique
E - La ponction transtrachéale
Bonne(s) Réponse(s) : B

L'immunofluorescence permet d'obtenir un diagnostic rapide et sensible (sensibilité 80 %, peu de faux positifs).

Quelle(s) est(sont) l'(les) antibiotique(s) le(s) mieux adapté(s) au traitement de l'infection par legionella ?
A - Bétalactamines
B - Aminosides
C - Macrolides
D - Bactrim®
E - Amphotéricine
Bonne(s) Réponse(s) : C

Sont utilisés les macrolides, la rifampicine et la pefloxacine

Un homme de 69 ans est opéré d'un adénome de la prostate le 12 janvier. Ce malade, gros fumeur, est atteint d'une
bronchopathie chronique obstructive responsable d'une dyspnée pour la montée d'un étage. L'intervention pratiquée sous
anesthésie péridurale s'est déroulée sans incident. Au 4ème jour post-opératoire le malade est fébrile à 38,2° C. Il présente
une toux sèche. Au cours d'un effort de toux, il élimine un crachat sanglant. A l'auscultation, il existe quelques ronchus dans
les deux champs pulmonaires. La radiographie montre une ascension de la coupole droite, un flou de la base droite et un petit
comblement du cul de sac costodiaphragmatique gauche.

Parmi les données initiales suivantes, laquelle(lesquelles) plaide(nt) en faveur d'une pneumopathie et contre
l'embolie pulmonaire :
A - Ronchus à I auscultation pulmonaire
B - Hémoptysie de sang rouge
C - Précession de la fièvre sur les autres troubles
D - Hyperleucocytose ( 14 000 GB/mm3)
E - Effet shunt aux gaz du sang
Bonne(s) réponse(s) :

Réponse impossible

Parmi les données initiales suivantes laquelle ou lesquelles plaide (nt) en faveur d'une embolie pulmonaire et
contre une pneumopathie :
A - Ronchus à l'auscultation pulmonaire
B - Hémoptysie de sang rouge
C - Précession de la fièvre sur les autres troubles
D - Hyperleucocytose ( 14 000 GB/mm3)
E - Effet shunt aux gaz du sang
Bonne(s) réponse(s) : B

Commentaire groupés (n°24 et 25).


A-Traduit un encombrement bronchique banal chez un bronchopathe chronique.

943
Exclusivement sur DOC - DZ : www.doc-dz.com NADJI 85
RESIDANAT EN POCHE TOME II
Cas Clinique en QCM

En attendant de confirmer l'une ou l'autre hypothèse, votre traitement peut comprendre :


A - Antivitamine K
B - Héparine : 5 mg/kg/24 heures
C - Urokinase : 2 000 U/kg/heure
D - Ampicilline : 2 g/24 heures
E - Pristinamycine : 2 g/24 heures.
Bonne(s) réponse(s) : B D

A - Les AVK n'ont pas d'utilité ici en raison de leur délai d'action prolongé et de leur peu de maniabilité.
B - En intraveineux continu ou discontinu, toutes les deux heures. Posologie à adapter selon les tests de coagulation (TH,
TCK).
C - Réservé aux embolies pulmonaires graves (obstruction quantifiée par l'angiographie pulmonaire).
D - Couvre les germes de surinfection trouvés chez les bronchopathes chroniques. Posologie un peu faible (3 à 4 g/24 h).

Chez ce malade pour confirmer l'hypothèse d'embolie pulmonaire vous prescrivez :


A - De nouveaux gaz du sang
B - Un dosage des LDH
C - Une scintigraphie de perfusion
D - Une angiographie
E - Une étude de la coagulation
Bonne(s) réponse(s) : D

A E - Sans intérêt pour le diagnostic.


B - Non spécifique.
C - Inutilisable en raison des anomalies scintigraphiques observées dans les bronchopathies chroniques obstructives. Même
avec une scintigraphie de référence, les anomalies radiologiques actuelles ne permettraient pas d'interprétation
D - C'est l'examen de référence. Il permet de visualiser des signes directs ou indirects d'amputation du lit vasculaire.

La suspicion d'embolie pulmonaire est confirmée. Après 10 jours d'hospitalisation, l'évolution est favorable. Il
reçoit alors des antivitamines K.
Quand pourra-t-on arrêter les anticoagulants chez ce patient après contrôle de l'état veineux ?
A - 10 à 15 jours
B - 1 à 2 mois
C - 3 à 9 mois
D - 2 ans à 5 ans
E - Jamais
Bonne(s) réponse(s) :

Impossible.
Les anticoagulants sont poursuivis de 6 semaines à 3 mois en fonction de l'état veineux et donc du risque de récidive. Item B
et C justes.

Par une chaude nuit de mai, Monsieur X, jeune citadin de 18 ans en vacances à la campagne, présente, pour la première fois
de son existence, une crise dyspneïque avec sensation d'étouffement qui le réveille et l'oblige à appeler le médecin de la ville
voisine. Ce dernier porte le diagnostic d'asthme.

Parmi les éléments suivants, lequel n'appartient pas au tableau classique de l'asthme de l'adulte jeune ?
A - Crise dyspnéique nocturne
B - Sibilance dans les deux champs pulmonaires
C - Tachycardie
D - Apyrexie
E - OEdèmes des membres inférieurs
Bonne(s) réponse(s) : E

Les oedèmes des membres inférieurs ne sont pas un élément clinique du tableau d'une crise d'asthme paroxystique chez un
sujet sans autre pathologie.

944
Exclusivement sur DOC - DZ : www.doc-dz.com NADJI 85
RESIDANAT EN POCHE TOME II
Cas Clinique en QCM

Il faut rechercher pour éliminer une forme grave ?


A - Des sueurs
B - Un blocage thoracique
C - Une tachycardie importante
D - Une pâleur des conjonctives
E - Une voix bitonale
Bonne(s) réponse(s) : A B C

A - Les sueurs sont un signe d'hypercapnie. L'existence d'une normo ou d'une hypercapnie lors d'une crise d'asthme est un
signe de gravité (état de mal gazométrique).
B - Un blocage thoracique avec une abolition du murmure vésiculaire à l'auscultation est un signe de bronchospasme sévère.
C - Une tachycardie sinusale supérieure à 120/m est un signe de gravité en l'absence de fièvre.

De ces 5 médicaments, lequel utilisez-vous en premier ?


A - Théophylline (suppositoires)
B - Anti-histaminique (injectable)
C - Anti-cholinergique (spray)
D - Béta-sympatomimétique (spray)
E - Cromoglycate (inhalation)
Bonne(s) réponse(s) : D

Non commentée.

Grâce à votre traitement, cette première crise cède rapidement. Vous revoyez votre jeune malade le lendemain
et vous diagnostiquez une atopie. Parmi ces éléments, le(s) quel(s) vous parai(ssen)t orienter dans ce sens ?
A - La mère de Monsieur X. est asthmatique
B - Le père de Monsieur X. est un grand fumeur
C - Monsieur X. a du rhume des foins
D - Monsieur X. est allergique à la pénicilline
E - Monsieur X. a été traité, un an auparavant, pour une parasitose intestinale
Bonne(s) réponse(s) : A C D

L'atopie comprend, outre l'asthme (A) des équivalents asthmatiques : rhinite (C), trachéobronchite spasmodique, conjonctivite,
eczéma, urticaire, oedème de Quincke, allergies digestives ou médicamenteuses (pénicilline D) ou aspirine principalement).

Vous allez mettre en oeuvre pour orienter vers la nature allergique de cette première crise d'asthme ?
A - Spirographie simple
B - Numération Formule Sanguine
C - Dosage des IgE sériques
D - Radiographie pulmonaire
E - ECG
Bonne(s) réponse(s) : B C

Item B - La NFS recherche une hyperéosinophilie.


Item C - On peut doser les IgE totales (Prist-test) ou les IgE spécifiques à un antigène dormé (Prast-test). Ici, pour orienter
vers la nature allergique, on utilise le Prist-test.

Vers quel type d'allergène allez-vous orienter vos recherches chez ce patient ?
A - Poussières de maison
B - Poils d'animaux
C - Pollens
D - Acariens
E - Moisissures
Bonne(s) réponse(s) : C

La période de survenue de la crise (mois de mai) et le lieu de la crise (campagne) oriente ver une asthme extrinsèque par
allergie aux pollens.

945
Exclusivement sur DOC - DZ : www.doc-dz.com NADJI 85
RESIDANAT EN POCHE TOME II
Cas Clinique en QCM

Vous revoyez 15 jours après Monsieur X., qui n'a pas, grâce à votre traitement, présenté de nouvelles crises.
Quel(s) examens(s) vous parai(ssen)t utile(s) pour le bilan de cet asthme ?
A - Spirographie
B - Scintigraphie pulmonaire
C - Gazométrie
D - Fibroscopie bronchique
E - Angiographie pulmonaire
Bonne(s) réponse(s) : A

Non commenté, évident.

Devant cette crise isolée, que conseillez-vous à Monsieur X. au plan thérapeutique ?


A - Sympathomimétiques en cas de crise
B - Théphylline au long cours
C - Corticothérapie prolongée
D - Antibiothérapie de 10 jours
E - Cures climatiques
Bonne(s) réponse(s) : A

B C - Traitement de l'asthme à dyspnée continue ou d'un asthme mal contrôlé malgré un traitement par sympathomimétiques
en spray bien conduit.
D - Pas d'arguments.
E - A visée psychologique.

Un homme de 46 ans, fumeur, est hospitalisé pour douleur thoracique gauche et dyspnée . Les troubles se sont installés
progressivement en 3 mois, au décours d'un épisode infectieux d'allure virale.

Vous évoquez cliniquement une pleurésie sur :


A - Augmentation des vibrations vocales
B - Pincement des espaces intercostaux
C - Toux hémoptoïque
D - Matité
E - Abolition du murmure vésiculaire
Bonne(s) réponse(s) : D E

A - Faux, les vibrations vocales sont diminuées (ce qui distingue une matité d'origine pleurale d'une matité d'origine
parenchymateuse).
B - Faux, au contraire dans les épanchements de grande abondance,.il peut exister sur la radio de thorax une dilatation d'un
hemithorax.
C - Evident.
D E - Vrai, avec parfois un souffle pleurétique (doux lointain, voilé, expiratoire) ou des frottements pleuraux (rythmés par la
respiration et disparaissant en apnée).

La radiographie confirme le diagnostic, en montrant, pour une pleurésie de moyenne abondance de la grande
cavité, une opacité :
A - Dense et inhomogène
B - Dense et homogène
C - Mal limitée
D - Latéro-thoracique épargnant le cul-de-sac
E - De tout le champ pulmonaire, refoulant le médiastin
Bonne(s) réponse(s) : B

C'est une opacité dense et homogène (B vrai, A faux) dont la limite supérieure est floue (C faux), concave en haut et en
dedans prolongée vers le haut et en dehors par une ligne bordante axillaire.
E - Possible dans les épanchements de grande abondance.

Le premier examen complémentaire à demander est :


A - Une gazométrie
B - Une ponction pleurale
C - Une fibroscopie bronchique
D - Des tomographies de profil gauche
E - Un scanner thoraco-abdominal
Bonne(s) réponse(s) : A

Evident.

946
Exclusivement sur DOC - DZ : www.doc-dz.com NADJI 85
RESIDANAT EN POCHE TOME II
Cas Clinique en QCM

Dans le cours du bilan, vous avez effectué une ponction pleurale. Vous pouvez affirmer la nature néoplasique
de cet épanchement sur :
A - Liquide sérohématique
B - Présence de Iymphocytes 100 %
C - Taux de glucose effondré
D - Taux de protide à 56 grammes/litre
E - Aucune des propositions précédentes
Bonne(s) réponse(s) : E

Seule la présence de cellules néoplasiques permettrait d'affirmer la nature de l'épanchement.

L'anatomie pathologique vous oriente vers une métastase pleurale d'une cancer épidermoïde. Au niveau de
quel(s) organe(s) devez-vous rechercher le primitif ?
A - Prostate
B - Bronche
C - OEsophage
D - Côlon
E - Rein
Bonne(s) réponse(s) : B C

Non commenté, connaissance.


La métastase reproduit le type histologique du cancer d'origine.

En raison du caractère récidivant de l'épanchement, malgré les ponctions et la corticothérapie, vous cherchez à
obtenir une symphyse pleurale, quelle méthode utilisez-vous ?
A - Injections de bléomycine dans la cavité pleurale
B - Décortication chirurgicale
C - Talcage par thoracoscopie
D - Injection de streptokinase dans la plèvre
E - Lavages pleuraux avec du sérum glucosé hypertonique
Bonne(s) réponse(s) : C

Non commenté.

Mr C, 28 ans, malien, vit en France, en foyer d'adultes depuis 3 ans.


Antécédents : bilharziose urinaire traitée.
Histoire de la maladie : depuis 2 mois, altération modérée de l'état général.
Amaigrissement : 60 à 58 kg. Toux incessante mais peu productive.
A l'examen clinique, on note :
- température à 38°C
- quelques sueurs
- auscultations pulmonaire et cardiaque normales
- pas d'adénopathie périphérique
- rate, foie non palpés
- abdomen souple
- sphère ORL normale.
Les premiers examens complémentaires montrent :
- un infiltrat du lobe supérieur droit avec, en son sein, une cavité de 2 cm de diamètre
- 6 000 GB/mm3
- VS : 45/mm à la 1ère heure
- IDR à 10 unités à la tuberculine : 20 mm phlycténulaire
- 1 à 9 bacilles alcoolo-résistants par champ au deuxième tubage gastrique alors que le premier et le troisième sont négatifs.

Quel est l'argument décisif qui confirme la tuberculose chez ce patient et le fait mettre sous traitement ?
A - L'IDR à la tuberculine est positive à 20 mm phlycténulaire
B - L'association infiltrat-cavité à la radiographie du thorax
C - La notion de sujet transplanté
D - Les signes généraux et le tableau infectieux
E - L'existence de quelques bacilles alcoolo-résistants à l'examen direct
Bonne(s) réponse(s) : E

Dans un contexte évocateur (terrain transplanté, altération de l'état général, infiltrat excavé du lobe supérieur droit), l'examen
de certitude est l'obtention de BK. La découverte de bacilles alcoolo-résistants à l'examen direct dans ce contexte est
l'argument décisif qui permet de porter le diagnostic et de mettre le patient sous traitement.

947
Exclusivement sur DOC - DZ : www.doc-dz.com NADJI 85
RESIDANAT EN POCHE TOME II
Cas Clinique en QCM

Avant la mise sous traitement, il faut pratiquer :


A - Dosage des transaminases
B - Dosage de la glycémie
C - ECG
D - Dosage de la créatinine
E - Réflexogramme achilléen
Bonne(s) réponse(s) : A D

Le bilan préthérapeutique comporte un bilan hépatique, un bilan rénal, un bilan ophtalmologique et éventuellement un
audiogramme à moduler en fonction des produits utilisés.

Vous décidez d'appliquer à ce malade le traitement antituberculeux standard de 9 mois, utilisé en France. Quels
sont les trois anti-tuberculeux utilisés à la phase initiale du traitement ?
A - Streptomycine
B - Isoniazide
C - Rifampicine
D - Ethambutol
E - Ethionamide
Bonne(s) réponse(s) : B C D

Non commentée, connaissance.

Vous conseillez au malade :


A - De prendre tous ces médicaments en une fois par jour
B - De prendre ces médicaments au milieu des repas
C - D'interrompre son traitement si ses urines deviennent rouges-violacées
D - De signaler un éventuel prurit ou des lésions cutanées
E - De ne pas prendre en quantité importante de boissons alcoolisées
Bonne(s) réponse(s) : A D E

Le traitement anti-tuberculeux s'administre le matin à jeûn (B faux) en une prise par jour. La couleur orangée des urines est
due à la rifampicine et n'est pas un signe de toxicité.
Il est déconseillé d'ingérer des boissons alcoolisées (toxicité hépatique et nerveuse). Des accidents cutanés
immunoallergiques dues à la rifampicine sont possibles (D).

Quelle(s) mesure(s) s'impose(nt) ?


A - Déclaration à la D.D.A.S.S.
B - Radiographie thoracique des sujets vivants dans le même foyer
C - Placement sanatorial du malade durant 9 mois
D - Réactions cutanées tuberculiniques des sujets contacts
E - Chimioprophylaxie des sujets contacts
Bonne(s) réponse(s) : A B D

C - Faux.
E - La chimioprophylaxie des sujets contacts concerne avant tous les jeunes enfants, les vieillards et les immunodéprimés.
A - La tuberculose est une maladie à déclaration obligatoire (tableau n° 27).

L'antibiogramme vous montrera que les bacilles sont sensibles à tous les antibiotiques. Quel(s) est(sont) le(les)
anti-tuberculeux que vous maintiendrez jusqu'au 9ème mois ?
A - Streptomycine
B - Isoniazide
C - Rifampicine
D - Ethambutol
E - Ethionamide
Bonne(s) réponse(s) : B C

Non commentée, connaissance.

948
Exclusivement sur DOC - DZ : www.doc-dz.com NADJI 85
RESIDANAT EN POCHE TOME II
Cas Clinique en QCM

Quel(s) examen(s) vous aide(nt) au 9ème mois à affirmer la guérison ?


A - Dosage des transminases
B - Recherche de BK
C - I.D.R. 10 unités à la tuberculine
D - Radiographie du thorax
E - Epreuves fonctionnelles respiratoires
Bonne(s) réponse(s) : B

Non commentée, connaissance.

Monsieur H., 28 ans, vous appelle pour une gêne respiratoire sifflante, nocturne, n'ayant pas cédé à la prise de théophylline et
de salbutamol. De petites gênes respiratoires sifflantes sont survenues les jours précédents. A l'examen : sujet dyspnéique
non cyanosé. A l'auscultation : sibilances disséminées, murmure vésiculaire bien audible, bruits du coeur modérément
accélérés à 90/mn, P.A. à 160/100 mmHg. Il est boulanger depuis l'âge de 14 ans. Des accès dyspnéiques et une rhinorrhée
aqueuse sont apparues depuis 6 ans, disparaissant durant les périodes de vacances, récidivant dans les 3 premiers jours de
la reprise du travail.
Examens récents : radiographie pulmonaire normale, test à l'acétylcholine positif (pour 100 microgrammes, chute de 30 % du
VEMS), tests aux pneumallergènes usuels négatifs, tests cutanés à la farine de blé positifs, test nasal (solution de farine de
blé) positif. IgE totales à 44 UI/ml. (N à 100 UI/ml). Après l'injection de terbutaline, de soludécadron IV et la prise de 200 mg
de théophylline retard, amélioration de la dyspnée et de l'auscultation. A signaler l'apparition de tremblements fins des
extrémités.

Le diagnostic d'asthme professionnel est étayé par :


A - La disparition de l'asthme en vacances
B - La récidive de l'asthme dans les 3 premiers jours de la reprise du travail
C - Le taux des IgE totales
D - Le test de provocation bronchique avec la substance incriminée
E - Le test à l'acétylcholine
Bonne(s) réponse(s) : A B D

C - Le taux des IgE totales confirme l'atopie mais n'étaye pas le diagnostic d'asthme professionnel.
E - Le test à l'acétylcholine déclenche pharmacologiquement un bronchospasme.

Les facteurs déclenchants de l'asthme peuvent être multiples, même en présence d'un sujet allergique. Parmi
les facteurs ci-dessous cités, indiquez les étiologies qui peuvent être vérifiées par des tests effectués en milieu
hospitalier :
A - Prise d'aspirine
B - Effort
C - Conflit psychologique
D - Variation climatique
E - Aucun des éléments précédents
Bonne(s) réponse(s) : A B

Non commentée, connaissance.

La mesure thérapeutique la plus efficace chez ce patient serait :


A - Corticoïdes à dose minimale efficace
B - Béta-adrénergiques oraux
C - Eviction
D - Cromoglycate disodique
E - Aérosol de corticoïdes
Bonne(s) réponse(s) : C

L'éviction de l'allergène est ici le traitement de fond idéal.

949
Exclusivement sur DOC - DZ : www.doc-dz.com NADJI 85
RESIDANAT EN POCHE TOME II
Cas Clinique en QCM

Dans un état de mal asthmatique :


A - Le murmure vésiculaire est souvent diminué
B - Des signes cardiaques droits sont observés
C - La surinfection peut être une cause déclenchante
D - L'hypoxie est constamment présente
E - L'hospitalisation s'impose
Bonne(s) réponse(s) : A B C D E

C'est une urgence thérapeutique (E), des facteurs déclenchants sont souvent retrouvés : infection bronchopulmonaire (C),
prise de médicaments allergisants, désensibilisation ou vaccination mal conduite, sevrage des corticoïdes, choc affectif etc...
L'hypoxémie est marquée (D), la capnie est variable suivant la gravité. Les signes de gravité cliniques sont : l'encéphalopathie
respiratoire, la diminution ou la disparition du murmure vésiculaire, des signes d'insuffisance ventriculaire droits aiguë, une
tachycardie supérieure à 120/mn en l'absence de fièvre, un collapsus cardiocirculatoire, une cyanose et des sueurs, un tirage
des muscles inspiratoires accessoires.

Quelle est parmi les thérapeutiques citées, celle qui aura le plus de chance d'être responsable de tremblements
?
A - Aérosols de bétastimulants (4 bouffées par jour)
B - Théophylline per os (200 mg)
C - Cromoglycate disodique (4 capsules par jour)
D - Corticoïdes (Soludecadron® 1 amp)
E - Terbulaline (1 injection sous-cutanée à 0,5 mg)
Bonne(s) réponse(s) : E

Les sympathomimétiques et la théophylline sont responsables de tremblements. Les items A et B sont possibles, l'item E est
le plus probable en raison de la voie d'administration.

Un homme de 55 ans vous consulte pour une insuffisance respiratoire chronique grave, 3 mois après une décompensation
aiguë. Ce malade, fumeur de 30 cigarettes/24 heures, est atteint depuis 15 ans d'une bronchopathie chronique obstructive. La
dyspnée s'aggrave progressivement. Le malade ne peut marcher plus de 50 mètres sans reprendre son souffle. Il y a 3 mois,
à l'occasion d'une pneumopathie, le malade a été mis sous ventilation assistée durant 10 jours. Après un séjour en maison de
repos, il revient vous consulter à ce moment, Ies gaz du sang montre : Pa02 = 53 mm Hg - PaC02 = 48 mm Hg.

Quel(s) traitement(s) médicamenteux est (sont) utile(s) à ce malade en traitement continu :


A - Théophylline : 10 mg/kg/24 heures
B - Béta mimétiques (salbutamol : 5 bouffées 3 fois / jour)
C - Antibiothérapie (amoxicilline : 2 g/24 heures)
D - Almitrine : 2 cps/24 heures
E - Digoxine : 1 cp/24 heures
Bonne(s) réponse(s) : A

B - L'énoncé ne précise pas s'il y a un facteur spastique. De plus, les doses utilisées sont trop importantes (usuellement deux
bouffées 3 fois par jour).
C - L'antibiothérapie est utilisée en cas de surinfection bronchique.
D - Contreversé. Vérifier la gazométrie après le début du traitement si l'almitrine est utilisée.
E - Dangereux car hypoxie.

Chez ce malade, quel est le signe qui conduit à l'oxygénothérapie de longue durée :
A - Hypercapnie à 48 mmHg
B - Hypoxie à 53 mmHg
C - VEMS à 27% de la normale
D - Taux d'hémoglobine à 17 g/100 ml
E - Hypertrophie auriculaire droite
Bonne(s) réponse(s) : B

L'indication de l'oxygénothérapie est une hypoxémie (PaO2 inférieure ou égale à 60 mm de mercure à deux reprises et dans
des conditions basales) avec ou sans manifestations de coeur pulmonaire chronique.

L'oxygénothérapie de longue durée ne pourra pas être délivrée chez ce patient si :


A - Le malade continue à fumer
B - Le malade habite un immeuble de grande hauteur
C - Le malade reste hypercapnique
D - Le malade ne semble pas vouloir coopérer au traitement
E - Le malade ne bénéficie pas d'assurance sociale
Bonne(s) réponse(s) : A

A - Risque d'explosion avec l'oxygène. L'oxygénothérapie nécessite par ailleurs une coopération du patient (D).
C - Ce n'est pas une contre-indication. L'oxygénothérapie est réalisée à faible débit (inférieur à 1 litre par minute).

950
Exclusivement sur DOC - DZ : www.doc-dz.com NADJI 85
RESIDANAT EN POCHE TOME II
Cas Clinique en QCM

A quel débit prescrivez-vous l'oxygène de longue durée au domicile ?


A - Au débit maximum ne donnant pas d'hypercapnie sévère
B - Au débit maximum toléré par voie nasale
C - A un débit donnant une saturation en oxygène du sang artériel supérieur à 0,92 sans somnolence
D - Au débit maximum faisant disparaître la dyspnée
E - A 2 litres par minute de principe
Bonne(s) réponse(s) : C

Le but de l'oxygénothérapie est d'obtenir une oxygénation correcte sans diminuer le stimulus hypoxémique respiratoire qui
aggraverait l'hypercapnie (dont la somnolence est un des signes). On contrôle lors d'une hospitalisation les gaz du sang afin
d'apprécier sur le pH et la capnie la tolérance du traitement et le débit à adopter (entre 0,5 et 2 litres/mn afin d'obtenir une
saturation de l'hémoglobine supérieure ou égale à 90%).

Un homme de 55 ans vous consulte pour une insuffisance respiratoire chronique grave, 3 mois après une décompensation
aiguë. Ce malade, fumeur de 30 cigarettes/24 heures, est atteint depuis 15 ans d'une bronchopathie chronique obstructive. La
dyspnée s'aggrave progressivement. Le malade ne peut marcher plus de 50 mètres sans reprendre son souffle. Il y a 3 mois,
à l'occasion d'une pneumopathie, le malade a été mis sous ventilation assistée durant 10 jours. Après un séjour en maison de
repos, il revient vous consulter à ce moment, Ies gaz du sang montre : Pa02 = 53 mm Hg - PaC02 = 48 mm Hg.

L(es) effet(s) attendu(s) de l'oxygénothérapie à long terme chez ce malade :


A - Amélioration importante et rapide du confort respiratoire
B - Allongement de la survie
C - Diminution de la polyglobulie
D - Perte de poids
E - Stabilisation de l'hypertension artérielle pulmonaire.
Bonne(s) réponse(s) : B C E

L'amélioration de l'oxygénation sanguine améliore la polyglobulie.


Elle stabilise l'HTAP. Elle allonge la durée de la survie dans des études contrôlées.

Un homme de 35 ans, sans antécédents pathologiques particuliers, fumeur


(15 années-tabac), présente la symptomatologie suivante :
douleur thoracique droite brutale. Fièvre à 40°C avec frissons, toux, expectoration purulente striée de sang.
Examen clinique : syndrome de condensation pulmonaire de la base droite. Herpès naso-labial.
Radiographie : opacité alvéolaire systématisée du lobe inférieur droit. Petite réaction pleurale du même côté.
Vous pensez à une pneumonie infectieuse.

En fonction du tableau clinique et radiologique, quel germe vous parait


le plus probablement en cause ?
A - Mycoplasme
B - Bacille de Koch
C - Pneumocoque
D - Streptocoque anaérobie
E - Staphylocoque
Bonne(s) réponse(s) : C

Description typique du pneumocoque.

Le syndrome alvéolaire constaté sur la radiographie peut se caractériser par un ou plusieurs éléments suivants,
éventuellement associés les uns aux autres ; le(s)quel(s) ?
A - Opacité homogène non rétractile
B - Bronchogramme aérien
C - Alvéologramme
D - Images en rayon de miel
E - Lignes de Kerley
Bonne(s) réponse(s) : A B C

Car D et E sont des signes de syndrome interstitiel.

951
Exclusivement sur DOC - DZ : www.doc-dz.com NADJI 85
RESIDANAT EN POCHE TOME II
Cas Clinique en QCM

Le syndrome clinique de condensation parenchymateuse peut se caractériser par un ou plusieurs éléments


suivants; le(s)quel(s) ?
A - Wheezing
B - Matité
C - Abolition du murmure vésiculaire
D - Souffle expiratoire voilé
E - Râles secs de fin d'inspiration
Bonne(s) réponse(s) : B C E

Par définition.

Si vous voulez avoir une confirmation bactériologique, quel(s) examen(s) fiable(s) pouvez-vous demander ?
A - Hémocultures
B - Examen d'expectoration
C - Tubages gastriques 3 jours de suite
D - Urocultures
E - Tests cutanés aux extraits microbiens
Bonne(s) réponse(s) : A

C'est le seul fiable.

Quel(s) antibiotique(s) pouvez-vous donner en première intention chez ce malade ?


A - Pénicilline G
B - Erythromycine
C - Gentamicine
D - Thiamphénicol
E - Ticarcilline (céphalosporine de 3ème génération)
Bonne(s) réponse(s) : A B

Premier item évident, érythromycine si allergie.

Avec un traitement correct, l'évolution habituelle de cette pneumonie se fait selon laquelle (lesquelles) des
modalités suivantes ?
A - Défervescence thermique au plus tard à la 48ème heure
B - Nettoyage radiologique en 5 jours
C - Adénopathies calcifiées quelques mois plus tard
D - Images pulmonaires bulleuses séquellaires
E - Aucune des affirmations précédentes n'est exacte
Bonne(s) réponse(s) : A

En général en 1 à 2 ou 3 jours.

Un homme de 60 ans, fumeur, est hospitalisé pour des douleurs thoraciques variables avec la respiration, associée à une
dyspnée d'effort d'apparition récente spontanément régressive.
Dans les antécédents, on note : tuberculose pulmonaire traitée il y a 10 ans par INH®, rifampicine, éthambutol pendant 9
mois. Il persiste des séquelles bi apicales à type d'infiltrat. A l'examen : fréquence respiratoire à 20/mn, T° = 38°, TA : 15/9,
pouls à 100/mn. Auscultation cardiaque normale, mauvais état veineux des membres inférieurs. ECG : ischémie sous
épicardique en V4, V5, rythme sinusal. Radiographie de thorax : infiltrat biapical ; opacité arrondie de 1 cm de diamètre
adossée à la plèvre à droite, à gauche atélectasie en bande sus-diaphragmatique. Gaz du sang ; PaO2 = 60 mmHg ; PaCO2
= 30 mmHg : pH = 7,42.

On peut mettre en doute le diagnostic d'embolie pulmonaire devant :


A - L'absence d'hémoptysie
B - Le caractère arrondi de l'opacité radiologique
C - La caractère régressif de la dyspnée
D - La fièvre à 38°C
E - Aucun de ces éléments
Bonne(s) réponse(s) : E

L'embolie pulmonaire peut revêtir tous ces aspects.

952
Exclusivement sur DOC - DZ : www.doc-dz.com NADJI 85
RESIDANAT EN POCHE TOME II
Cas Clinique en QCM

Pour affirmer le diagnostic d'embolie pulmonaire :


A - Les examens déjà réalisés sont suffisants
B - Il faut pratiquer une scintigraphie de perfusion
C - Il faut pratiquer une angiographie pulmonaire
D - Il faut pratiquer une échographie
E - Il faut pratiquer un cathétérisme droit
Bonne(s) réponse(s) : C

Fiable à plus de 80 %.

Vous instituez dès l'arrivée du malade :


A - Oxygénothérapie
B - Anticoagulant (héparine)
C - Dobutamine
D - Thrombolytiques
E - Antibiotiques par pénicilline - Gentalline® + Flagyl®
Bonne(s) réponse(s) : A B

Evident.

Le diagnostic d'embolie pulmonaire étant confirmé et le traitement initialement équilibré, la surveillance de


l'hypocoagulabilité doit être effectuée les premiers jours du traitement :
A - Toutes les deux heures
B - Toutes les 24 heures
C - Tous les 3 jours
D - Une fois par semaine
E - Tous les 16 jours
Bonne(s) réponse(s) : B

Connaissance.

Après un traitement de 3 semaines par Héparine® ou Calciparine®, qu'envisagez-vous ?


A - Poursuite pendant 6 mois à un an de la Calciparine®
B - Relais par antivitamines K pendant 6 mois à un an
C - Traitement par antiagrégant plaquettaire
D - Arrêt des anticoagulants et port de bas de contention
E - Aucun de ces traitements n'est adapté
Bonne(s) réponse(s) : B

En général, héparine IV 1 à 2 semaines puis Calciparine® 1 à 2 mois et AC per os.

Lors d'une radiographie pulmonaire systématique, on découvre, chez un homme 49 ans, une opacité ronde de 4 cm, dans le
lobe supérieur droit. Il est originaire de Tunisie et travaille comme O.S. dans une usine de câbles métalliques. Il a fumé 40
paquets/années. Il n'a aucun antécédent connu. L'examen clinique est normal. Les examens biologiques sont les suivants :
NFS : G.R. = 4 300 000/mm3 - GB = 8 000/mm3, PN : 66 % - L : 20 % - E : 4 % - M : 10 % - V.S. : 32 à la 1ère heure.

Indiquez, parmi les examens biologiques suivants, celui(ceux) qui peu(ven)t être utile(s) :
A - Recherche de BK par tubage
B - Sérologie d'hydatidose
C - Gazométrie sanguine
D - Sérologie d'amibiase
E - I.D.R. 10 unités tuberculiniques
Bonne(s) réponse(s) : B D

A E - Ne sont pas des examens biologiques. La recherche de BK par tubage est rarement positive dans un tuberculome en
raison du caractère fermé des lésions (ceci même après aspiration endoscopique dirigée vers la lésion).
C - Aucun intérêt pour le diagnostic étiologique.
B - A faire chez ce sujet originaire d'Afrique du Nord.
La découverte d'un kyste hydatique pulmonaire peut être faite lors d'une radiographie pulmonaire systématique : opacité
arrondie ou ovalaire, régulière, homogène et dense, bien limitée. Intérêt du contexte épidémiologique, d'une hyperéosinophilie
(mais elle n'est très élevée qu'en cas de fissuration du kyste) et de la sérologie (moyennement fiable).
Le diagnostic peut n'être fait qu'après exérèse chirurgicale.
D - Le contexte clinique est ici peu évocateur mais la sérologie sera faite par esprit de système en raison de l'origine
géographique du sujet.
Le diagnostic d'abcès amibien est évoqué par l'atteinte préférentielle de la base droite, un hyperleucocytose à polynucléaire
neutrophiles, une atteinte hépathique associée, et seule pathognomonique, une sérologie positive.

953
Exclusivement sur DOC - DZ : www.doc-dz.com NADJI 85
RESIDANAT EN POCHE TOME II
Cas Clinique en QCM

Parmi les explorations suivantes, indiquez celle(s) qui est(sont) justifiée(s) pour leur intérêt diagnostique :
A - La bronchoscopie
B - La médiastinoscopie
C - La bronchographie
D - La tomodensitométrie thoracique
E - La scintigraphie pulmonaire de ventilation/perfusion
Bonne(s) réponse(s) : A D

A -L'endoscopie permet d'éliminer une atteinte bronchique et de faire des prélèvements bactériologiques.
Dans ce contexte (kyste hydatique possible), on n'effectuera pas de prélèvements biopsiques (risque de rupture du kyste et de
diffusion intra-thoracique).
D -Le scanner thoracique précise la densité de l'opacité, l'existence des lésions parenchymateuses ou extra-
parenchymateuses associées, les rapports de l'opacité avec les structures de voisinage

Si les examens précédents n'apportent pas d'éléments en faveur d'un diagnostic particulier, indiquer la(les)
hypothèse(s) que vous devez envisager :
A - Abcès du poumon
B - Kyste hydatique
C - Cancer bronchique primitif
D - Tuberculose
E - Bronchectasies localisées
Bonne(s) réponse(s) : B C D

A - Non, évident car il n'y a pas de contexte infectieux.


E - Non, l'image radiologique et l'histoire clinique ne sont pas compatibles.
Il faut retenir 3 hypothèses diagnostiques :
- cancer bronchique primitif à forme périphérique (adénocarcinome principalement) : on retient l'âge du sujet et le tabagisme
- un tuberculome
- un kyste hydatique.

Indiquez, parmi les gestes suivants, celui que vous conseillez pour parvenir au diagnostic :
A - Ponction transbronchique
B - Ponction transpariétale
C - La biopsie préscalénique de Daniels
D - Thoracotomie exploratrice
E - Pleuroscopie
Bonne(s) réponse(s) : D

A B - Contre-indiqués (risque de dissémination dans l'hypothèse d'un kyste hydatique)


C - La biopsie du tissus cellulograisseux prescalénique s'efforce de reconnaître et recueillir des microadénopathies : aucun
intérêt ici.
E - La pleuroscopie visualise la plèvre (aucun intérêt ici). Les biopsies qui pourraient être effectuées par cette voie sont aussi
contre-indiquées.

Mme Z., 35 ans, non fumeuse, employée de bureau, consulte pour des crises de dyspnée intermittentes.
Histoire de la maladie : depuis 18 mois, épisodes de gène respiratoire initialement nocturnes, accompagnés de sifflements
intrathoraciques et d'une toux pénible. Ces crises surviennent vers 4 heures du matin, et cèdent au bout de 2 heures lorsque
la malade expectore de petits crachats blanchâtres et épais. Elle fait également des crises après un effort important. Durant
l'hiver, la fréquence des crises a augmenté (plusieurs fois par semaine), surtout à l'occasion d'épisodes bronchitiques. Ses
expectorations étaient alors purulentes. Son état respiratoire s'est amélioré au printemps et durant l'été.
Antécédents : sinusite maxillaire 2 ans auparavant, rhinite spasmodique isolée dans l'enfance, survenant au printemps et à
l'été ; asthme chez sa mère ; coryza spasmodique chez un frère ; eczéma constitutionnel chez sa fille.
Examens : quelques sibilances des bases en expiration forcée. Radiographie pulmonaire normale. Examen cardiovasculaire:
normal.

Parmi les éléments cliniques suivants, le(s)quel(s) vous permet(tent) d'évoquer le diagnostic d'asthme ?
A - Crises de dyspnée paroxystiques
B - Sinusite
C - Prédominance nocturne des crises
D - Sibilances auscultatoires
E - Toux récidivante
Bonne(s) réponse(s) : A C D E

B - Un foyer infectieux stomatologique ou ORL peut être un facteur déclenchant d'une crise d'asthme ou un facteur d'entretien
de la maladie asthmatique. A ne pas confondre avec la rhinite (rhinite annuelle ou rhume des foins) qui est un équivalent
asthmatique.

954
Exclusivement sur DOC - DZ : www.doc-dz.com NADJI 85
RESIDANAT EN POCHE TOME II
Cas Clinique en QCM

Vous faites faire une spirométrie qui montre les valeurs suivantes :
capacité vitale = 3000 ml (valeur théorique = 3900 ml)
V.E.M.S. = 1850 ml (val. théo. = 3000 ml)
volume résiduel = 2480 ml (val. théo. = 1560 ml).
Ces résultats vous permettent de porter la (les) conclusion(s) suivante(s) :
A - Syndrome mixte, obstructif et restrictif
B - Syndrome ostructif
C - Syndrome restrictif avec distension
D - Trouble de diffusion
E - Emphysème
Bonne(s) réponse(s) : B

La mesure de la capacité vitale (CV), du volume résiduel (VR) et du volume expiratoire maximal seconde (VEMS) permet de
décrire deux types de syndrome ventilatoire élémentaire : le syndrome obstructif et le syndrome restrictif.
La capacité pulmonaire totale (CT) n'est pas indiquée ici.
Dans le syndrome obstructif, le VEMS est abaissé par rapport à la capacité vitale (VEMS/CV inférieur à 80 % chez un sujet
jeune, 65 % chez un sujet de plus de 60 ans) et le VR est augmenté par rapport à la CT (VR/CT supérieur à 25 % chez un
sujet jeune, 30 % chez un sujet de plus de 60 ans).
Le syndrome restrictif est caractérisé par une diminution de la CV avec conservation du VEMS. La CT est diminuée, ce qui
définit le syndrome restrictif.
Les données indiquées permettent de calculer :
- une CV à 77 % de la théorique (limite inférieure de la normale)
- un VEMS/CV à 60 % pour une théorique à 78 %
- un VR à 150 % de la théorique.
La réponse est donc B.

Pour rechercher la réversibilité de ces anomalies spirométriques, vous pouvez pratiquer la(les) épreuve(s)
suivante(s) :
A - Inhalation de salbutamol (sympathomimétique)
B - Inhalation d'acétylcholine
C - Inhalation d'un parasympatholytique
D - Inhalation de cromoglycate disodique
E - Inhalation d'oxygène
Bonne(s) réponse(s) : A C

Non commentée, connaissance.

Parmi les éléments suivants, le(s)quel(s) peu(ven)t orienter vers une composante allergique ?
A - Recrudescences au moment des infections bronchiques
B - Terrain atopique
C - Absence d'hyperleucocytose à l'hémogramme
D - Déclenchement des crises à l'effort
E - Absence d'anomalies sur le cliché pulmonaire
Bonne(s) réponse(s) : B

La recherche d'une atopie est effectuée chez le patient et son entourage familial. Elle comprend, outre l'asthme, des
équivalents asthmatiques :
- rhinite (rhinite annuelle ou rhume des foins) et sinusite allergique
- oedème de Quincke
- trachéobronchite spasmodique
- urticaire, eczéma
- allergies digestives ou médicamenteuses (pénicilline et aspirine surtout).

Les tests cutanés montrent une positivité avec réaction immédiate aux allergènes suivants : poussière de
maison, plumes, acariens, pollens de graminées, extraits microbiens, poils de chien, moisissures.
Vous concluez que :
A - L'asthme est directement en rapport avec une allergie pollinique
B - Il existe une allergie microbienne qui doit être traitée
C - Il existe peut être une composante allergique aux acariens
D - L'asthme est d'origine allergique pure
E - Il faudra désensibiliser à tous ces allergènes
Bonne(s) réponse(s) : C

A - Les tests cutanés n'ont qu'une valeur d'orientation vis à vis d'un antigène qui ne peut être rendu responsable que quand
cette opération cutanée s'accompagne d'une réaction syndromique, équivalent mineur d'une crise. Par ailleurs ce sujet
présente une polysensibilisation.
B - L'allergie microbienne ne se traite pas.
D - N'élimine pas une composante intrinsèque.
E - La désensibilisation spécifique a une efficacité quasi nulle en cas de polysensibilisation.

955
Exclusivement sur DOC - DZ : www.doc-dz.com NADJI 85
RESIDANAT EN POCHE TOME II
Cas Clinique en QCM

Parmi les thérapeutiques suivantes, que pouvez-vous prescrire pour faire céder rapidement la crise ?
A - Deux bouffées de salbutamol (sympathomimétique)
B - Une capsule de Lomudal® (cromoglycate)
C - Une capsule de Zaditen® (kétotifène)
D - Deux bouffées de Bécotide® (corticoïde)
E - Deux comprimés de Polaramine® (antihistaminique)
Bonne(s) réponse(s) : A

B C - Sont utilisés dans le traitement préventif des asthmes extrinsèques. Ils ne présentent aucune efficacité sur la crise et
doivent donc être débutés en période d'accalmie.
D - Les corticoïdes en inhalation sont utilisés dans un asthme sévère (asthme corticodépendant, asthme à dyspnée
paroxystique mal équilibré par les autres thérapeutiques) en traitement d'entretien.
E - Les antihistarniniques ont un faible effet thérapeutique.
A - Thérapeutique de la crise d'asthme car les sympathomimétiques sont très rapidement efficaces (quelques dizaines de
secondes).

Pour prévenir les crises déclenchées par l'effort, vous pouvez prescrire :
A - Théophylline
B - Cromogylycate (Lomudal®)
C - Corticoïdes
D - Sympathomimétiques bétastimulants
E - Polaramine® (antihistaminique)
Bonne(s) réponse(s) : A B D

Certains médicaments ont une action élective sur l'asthme d'effort : on a insisté sur la valeur protectrice du cromoglycate
disodique.
D - Les sympathomimétiques pris en inhalation avant le début de l'exercice ont une efficacité quasi-constante.
A - Une telle action n'est retrouvée que 3 fois sur 4 avec les théophyllines et 1 fois sur 3 avec les atropiniques.
C E - Inefficaces.

Un malade de 60 ans est hospitalisé dans le service de Pneumologie pour une insuffisance respiratoire aiguë. Ce malade est
connu du service comme un insuffisant respiratoire chronique. Après quelques heures d'oxygénothérapie, la situation
s'améliore. Dans ses antécédents, on note il y a plusieurs années, une fracture de jambe immobilisée par plâtre. Au cours de
cette immobilisation, il s'était plaint d'une douleur thoracique brève qui n'avait pas entraîné d'investigation complémentaire.
Depuis cet accident, il a développé de volumineuses varices de ce membre. Ces varices donnent fréquemment lieu à des
phlébites superficielles qu'il traite uniquement par des anti inflammatoires locaux. Le diagnostic d'embolie récidivante est
retenu.

Devant ce tableau clinique, quel(s) autre(s) diagnostic(s) doi(ven)t être évoqué(s) ?


A - Décompensation d'une sclérose pulmonaire emphysémateuse
B - Pneumothorax aigu
C - Cancer bronchique secondaire
D - Epanchement pleural
E - Péricardite
Bonne(s) réponse(s) : A B C D

A part la péricardite tout est possible mais cas annulé et pour cause !

Pour étayer le diagnostic vous demandez :


A - Radiographie pulmonaire
B - ECG
C - Bronchoscopie
D - Angiographie de l'artère pulmonaire
E - Dosage des gaz sanguins
Bonne(s) réponse(s) : A B D E

A - En premier lieu.
B - Normal dans 20 %.
D - Examen le plus spécifique.
E - Systématique peut montrer une hypo O2.

956
Exclusivement sur DOC - DZ : www.doc-dz.com NADJI 85
RESIDANAT EN POCHE TOME II
Cas Clinique en QCM

Parmi les investigations suivantes, laquelle(lesquelles) peu(ven)t venir étayer l'étiologie ?


A - Artériographie des membres inférieurs
B - Doppler des membres inférieurs
C - Phlébocavographie
D - Rhéopléthysmographie
E - Thermographie des membres inférieurs
Bonne(s) réponse(s) : C

En même temps que l'angiographie, retrouve une étiologie dans 50% des cas.

Quel(s) traitement(s) doi(ven)t être envisagé(s) ?


A - Broncho-aspiration
B - Traitement fibrinolytique
C - Interruption partielle de la veine cave par filtre-ombrelle intracave
D - Interruption partielle de la veine cave par clip péricave
E - Intervention directe de décaillotage de l'artère pulmonaire sous CEC
Bonne(s) réponse(s) : B C

B - Discuté si amputation > à 50 % du lit vasculaire.


C - En cas de caillot flottant.

Quel(s) traitement(s) ultérieur(s) vous parai(ssen)t primordia(l)ux :


A - Eveinage des varices du membre inférieur
B - Contention élastique au long cours
C - Traitement anticoagulant au long cours
D - Crénothérapie
E - Traitement vasoactif et toniques veineux
Bonne(s) réponse(s) : B C

A - Contre-indiqué car phlébite probable.


D E - Pas de preuves d'une quelconque efficacité.

Vous êtes appelé d'urgence auprès d'un homme de 35 ans, manoeuvre dans une entreprise de travaux publics pour
température à 40° C, point de côté droit, dyspnée. L'interrogatoire révèle un éthylisme chronique. Un tabagisme habituel,
aucun problème pulmonaire jusqu'à ce jour. L'examen met en évidence des frissons et un syndrome de condensation
pulmonaire apical droit.

Quel diagnostic évoquez-vous ?


A - Pleurésie purulente
B - Abcès du poumon
C - Infarctus pulmonaire
D - Pneumonie lobaire aiguë
E - Pneumothorax
Bonne(s) réponse(s) : D

La découverte d'un syndrome de condensation pulmonaire élimine A B E.


L'infarctus pulmonaire est peu probable en raison d'un contexte infectieux brutal sans problèmes respiratoires antérieurs.

Qu'allez-vous trouver a priori à l'examen physique du thorax ?


A - Matité
B - Crépitants
C - Souffle tubaire
D - Hypersonorité
E - Souffle cavitaire
Bonne(s) réponse(s) : A B C

A B C - Syndrome pneumonique : exagération des vibrations vocales, matité ou submatité de type parenchymateux (non
déclive et ne tournant pas dans l'aisselle), souffle tubaire entouré d'une couronne de crépitants.
D - Pneumothorax.
E - Dans les abcès pulmonaires étendus.

957
Exclusivement sur DOC - DZ : www.doc-dz.com NADJI 85
RESIDANAT EN POCHE TOME II
Cas Clinique en QCM

Vous rechercherez à la radiographie :


A - Opacité systématisée non rétractile
B - Opacité déclive à concavité supérieure
C - Hyperclarté diffuse
D - Image cavitaire
E - Opacité non systématisée
Bonne(s) réponse(s) : A

B - Epanchement pleural liquidien.


C - Raréfaction parenchymateuse de type emphyzème.
D - Suppuration pulmonaire primitive (abcès du poumon) ou secondaire.
E - Etiologies multiples.

Quel examen complémentaire est indispensable ?


A - Examen des crachats
B - Hémocultures
C - Urocultures
D - Numération formule
E - Vitesse de sédimentation
Bonne(s) réponse(s) : B

A - Peu rentable et non spécifique. Intérêt de la ponction transtrachéale.


B - Ne sont positives que dans 30 % des cas, leur positivité indique une forme sévère (puisque septicémique).

Vous devez craindre chez ce patient :


A - Pleurésie purulente
B - Abcès du poumon
C - Délirium
D - Insuffisance respiratoire aiguë
E - Pneumothorax
Bonne(s) réponse(s) : A B C

A - Dans les formes septicémiques : extension à la plèvre, au péricarde, aux méninges, à l'endocarde, aux articulations (rare).
La persistance de la fièvre sous traitement doit faire évoquer entre autres un foyer infectieux à distance.
B - Les pneumopathies à pneumocoque s'excavent rarement.
C - La pneumonie est une circonstance déclenchante classique d'un syndrome confusionnel chez l'éthylique. A l'opposé, toute
infection pulmonaire sévère chez un alcoolique justifie un traitement préventif du délirium trémens (réhydratation, sédation).
D - Non, en l'absence d'insuffisance respiratoire chronique sous jacente
E - Non, la pneumonie à pneumocoque n'est pas une pneumopathie dissécante ni bulleuse.

Parmi les antibiotiques suivants, lequel vous parait le plus adapté ?


A - Aminoside
B - Chloramphenicol
C - Cépha losporine
D - Tétracycline
E - Pénicilline
Bonne(s) réponse(s) : E

Connaissance.

Quel sera la durée de votre traitement ?


A - 1 semaine
B - 2 à 3 semaines
C - 1 mois
D - Jusqu'à apyrexie
E - 2 mois
Bonne(s) réponse(s) : B

L'antibiothérapie est poursuivie jusqu'au retour à la normale du cliché thoracique et après au moins 8 jours d'apyrexie, soit un
minimum de 10 jours, en moyenne 2 ou 3 semaines.

958
Exclusivement sur DOC - DZ : www.doc-dz.com NADJI 85
RESIDANAT EN POCHE TOME II
Cas Clinique en QCM
Vous êtes appelé d'urgence auprès d'un homme de 60 ans, agriculteur, tabagique (40 paquets/année) aux habitudes
éthyliques certaines, qui présente depuis 24 heures un syndrome respiratoire aigu. Celui-ci a débuté par un point de côté
brutal au niveau de la base droite, la fièvre s'est rapidement élevée à 39° C. L'expectoration est minime mais muco-purulente.
A l'examen clinique, le sujet est en bon état général : l'examen thoracique révèle un syndrome de condensation de la base
droite.

Vous évoquez comme diagnostic(s) :


A - Une maladie des éleveurs d'oiseaux
B - Un infarctus pulmonaire
C - Un oedème aigu du poumon
D - Une pneumonie à pneumocoque
E - Un pneumothorax spontané
Bonne(s) réponse(s) : D

Le début brutal, l'association d'une fièvre avec expectoration mucopurulente, d'une douleur thoracique et d'un syndrome de
condensation pulmonaire doit faire évoquer en premier lieu l'item D. L'infarctus pulmonaire survient après une embolie
pulmonaire et est responsable à distance d'une douleur thoracique, d'une expectoration hémoptoïque et d'une fièvre
(anamnèse).

Vous vous attendez à trouver à l'examen de ce patient, tous les signes cliniques énumérés ci-dessous sauf un.
Lequel ?
A - Herpès Labial
B - Adénopathie sus-claviculaire droite
C - Râles crépitants
D - Souffle tubaire
E - Submatité de la base droite
Bonne(s) réponse(s) : B

L'examen peut mettre en évidence : à l'inspection, une rougeur de la pommette du côté atteint, un herpès nasolabial (A).
L'examen physique retrouve un syndrome pneumonique : exagération des vibrations vocales, matité ou submatité de type
parenchymateux (non déclive et ne tournant pas dans l'aisselle (E), souffle tubaire (D) entouré d'une couronne de râles
crépitants (C). Un frottement pleural peut être noté. Il existe aussi une langue saburrale et une oligurie.
Une adénopathie sus-claviculaire droite (B) doit faire rechercher un cancer bronchopulmonaire.

Vous pouvez éventuellement observer tous les signes radiologiques thoraciques, sauf un. Lequel ?
A - Comblement du cul-de-sac pleural droit
B - Opacité dense triangulaire à sommet hilaire, de la base droite avec image de bronchogramme aérien
C - Opacités bilatérales denses polysegmentaires
D - Adénopathies hilaires bilatérales
E - Opacité dense triangulaire avec image d'excavation d'un centimètre de diamètre en son sein
Bonne(s) réponse(s) : D

La radiographie confirme la condensation parenchymateuse : opacité dense et homogène de type alvéolaire (bronchogramme
aérien) habituellement bien limitée avec une topographie systématisée (B).
Une excavation (E) doit faire évoquer trois germes : un staphylocoque, une klebsielle et des anacrobies. Cette abcédation est
rare dans les pneumopathies à pneumocoques.
Une pleurésie (A) doit être ponctionnée afin de distinguer une réaction pleurale au contact d'un foyer d'une pleurésie purulente.
Des opacités polysegmentaires (C) traduisent une pneumopathie extensive et doit faire rechercher d'autres germes que le
pneumocoque ou d'autres diagnostics (embolies pulmonaires à répétition).

L'antibiotique dont la prescription est totalement inadaptée chez ce patient est :


A - Penicilline G
B - Ampicilline
C - Gentamicine
D - Erythromycine
E - Céphalosporine
Bonne(s) réponse(s) : C

Les aminosides ont une mauvaise diffusion tissulaire pulmonaire et sont inactifs sur le pneumocoque (non pénétration de la
paroi).
La pénicilline G est l'antibiothérapie de choix dans les pneumopathies à pneumocoque, les ampicillines et les céphalosporines
sont moins intéressantes (20 % des pneumocoques sont résistants aux macrolides).

959
Exclusivement sur DOC - DZ : www.doc-dz.com NADJI 85
RESIDANAT EN POCHE TOME II
Cas Clinique en QCM

Quelle est l'évolution clinique observée usuellement après mise en route rapide d'un traitement antibiotique
adapté chez un tel patient ?
A - Apparition d'un pyopneumothorax
B - Nettoyage radiologique et défervescence thermique en 24 heures
C - Défervescence thermique en 2 à 3 jours et nettoyage radiologique en 15 jours - 3 semaines
D - Apparition d'une miliaire signant un oedème aigu lésionnel
E - Apparition d'une obnubilation avec méningoencéphalite
Bonne(s) réponse(s) : C

La défervescence thermique doit être obtenue en 24 à 48 h. Les signes physiques et surtout radiologiques sont plus longs à
s'amender (RP toutes les semaines).

Après 4 semaines, la radiographie du thorax montre la persistance d'une image trabéculaire de la base droite
avec une opacité à la corne inférieure du hile droit. Parmi les examens ci-dessous, quel est celui qu'il faut
demander en priorité ?
A - Ponction transtrachéale pour recherche bactériologique
B - Scanner thoracique
C - Fibroscopie bronchique
D - Scintigraphie pulmonaire
E - Recherches répétées de BK dans l'expectoration ou le liquide de tubage gastrique
Bonne(s) réponse(s) : C

La fibroscopie bronchique est systématique s'il s'agit d'un sujet fumeur de plus de 40 ans, d'un tabagique dont la
consommation est supérieure à 20 paquets - année, à fortiori s'il persiste des anomalies radiologiques.

Une femme de 23 ans consulte pour un asthme datant de l'enfance. Les crises étaient peu fréquentes jusqu'à l'année dernière
: elles survenaient soit après un effort, soit entre Mars et Juin, période pendant laquelle existait également une rhinite
spasmodique. Les crises depuis 1 an sont devenues plus fréquentes et importantes (pluri-hebdomadaires), à la suite d'une
bronchite. Il persiste entre les crises une gêne respiratoire le plus souvent minime (le matin au réveil surtout). Depuis 5 jours,
elle a des crises tous les jours avec une gêne respiratoire persistante (dyspnée à 1 étage environ). A l'examen, il existe des
sibilants dans les 2 poumons, l'expectoration est abondante, mousseuse ; la patiente n'est pas cyanosée ; la température est
à 37,5° C. Il faut signaler enfin un retard des règles de 2 mois.

Le traitement que vous proposez comporte :


A - Synacthène retard
B - Théophylline
C - Corticoïde retard
D - Corticoïde par voie orale
E - Sympatho-mimétique
Bonne(s) réponse(s) : B D E

Il s'agit d'une attaque d'asthme avec crises pluriquotidiennes depuis 5 jours et dyspnée continue entre les crises: une
hospitalisation est justifiée.
Le traitement associera une théophylline (en urgence, voie intraveineuse lente puis perfusion sur 24 h ), des
sympathomimétiques (voie sous-cutanée ou intraveineuse en cas de crise sévère) et une corticothérapie (voie orale, voire
voie intraveineuse, les corticoïdes retards sont contre-indiqués).

Quel traitement antibiotique proposez-vous ?


A - Macrolides
B - Pénicilline injectable
C - Aminoside
D - Cyclines
E - Aucun antibiotique
Bonne(s) réponse(s) : A E

E - Pour certains auteurs, le traitement antibiotique n'est pas systématique en l'absence d'élément infectieux.
A - Pour d'autres, une antibiothérapie sera prescrite (péni A ou macrolide).Il faut se méfier des interactions entre la
théophylline et les macrolides (risque de surdosage en théophylline), du risque allergique des § lactamines. Enfin à noter que
les cyclines sont contre-indiquées chez la femme enceinte.

960
Exclusivement sur DOC - DZ : www.doc-dz.com NADJI 85
RESIDANAT EN POCHE TOME II
Cas Clinique en QCM

Les circonstances anamnestiques et cliniques permettent d'énoncer la(les) proposition(s) suivante(s) :


A - Il faut faire une étude bactériologique de l'expectoration pour montrer la nature infectieuse de cet asthme
B - Il faut faire un bilan allergologique avant tout traitement
C - Il est dangereux de faire un bilan allergologique en période de crise
D - Il faut commencer immédiatement une désensibilisation
E - Aucune de ces propositions n'est vraie
Bonne(s) réponse(s) : C

A - Faux, il n'y a pas d'élément infectieux déclenchant cette attaque d'asthme.


B - Inutile, l'interrogatoire suffit. De plus une désensibilisation risquerait d'aggraver la symptomatologie en période de crise.
C - Vrai, voir item B.
D - Faux, voir item B.

En faveur d'une origine allergique, vous retenez :


A - Le retard des règles
B - L'expectoration
C - La rhinite spasmodique
D - Le déclenchement des crises à l'effort
E - La périodicité des crises
Bonne(s) réponse(s) : C E

B - La rhinite spasmodique est un équivalent asthmatique indiquant un terrain atopique. Les autres équivalents sont : une
trachéo-bronchite spasmodique, un eczéma, une urticaire, un oedème de Quincke, une allergie digestive ou médicamenteuse.
E - La découverte d'une unité de temps, de lieu et de contact lors des crises oriente vers un asthme extrinsèque.
D - Faux, l'asthme d'effort n'est pas un asthme allergique. Néanmoins, l'atopie favorise ce type d'asthme (sans doute par
abaissement du seuil de sensibilité des récepteurs à l'irritation).

Quel(s) examen(s“ complémentaire(s) proposez-vous pour confirmer l'hypothèse allergique ?


A - Hémogramme
B - Test de provocation au carbachol ou à l'acétyl choline
C - Tests cutanés
D - Dosage des IgE
E - Epreuve d'effort
Bonne(s) réponse(s) : C D

A - Une hyperéosinophilie (supérieure à 400 éosinophiles par mm3 en l'absence d'une parasitose) constitue un argument en
faveur de l'origine allergique de l'asthme. Cependant, cette hyperéosinophilie est inconstante et non spécifique.
B - Tests mettant en évidence une hyperréactivité bronchique (asthme extrinsèque ou intrinsèque).
C D - Les test cutanés ont une valeur d'orientation vers un antigène, cet antigène ne peut être rendu responsable d'une crise
qu'en cas de réaction syndromique, les dosages sériques d'IgE peuvent intéresser les IgE totales ou les IgE spécifiques à un
antigène donné.

Quel type d'allergie peut être a priori évoqué ?


A - Poil de chat
B - Poussière de maison
C - Pollen de oramine
D - Allergie bactérienne
E - Allergie alimentaire
Bonne(s) réponse(s) : C

Une pollinose est évoquée devant la périodicité des crises. L'association d'une pollinose à une rhinite allergique est fréquente.

961
Exclusivement sur DOC - DZ : www.doc-dz.com NADJI 85
RESIDANAT EN POCHE TOME II
Cas Clinique en QCM
Un homme de 46 ans est hospitalisé pour oedème en pèlerine avec :
- turgescence des jugulaires
- céphalées matinales
- fièvre à 38° C.
Cet ouvrier est fumeur (20 cigarettes/24 heures). Il n'a pas d'antécédents particuliers. Depuis 3 mois, il présente une altération
progressive de l'état général et une toux.

La radiographie du thorax pulmonaire montrera le plus probablement :


A - Une opacité de l'apex droit avec Iyse de la première côte
B - Une altération de tout le poumon droit avec attraction du médiastin
C - Des opacités biapicales excavées
D - Un épanchement pleural bilatéral avec cardiomégalie
E - Une opacité latéro-médiastinale droit refoulant l'axe trachéal
Bonne(s) réponse(s) : E

L'association de céphalées, d'un oedème en pèlerine, d'une turgescence veineuse réalise un syndrome de compression
médiastinale de la veine cave supérieure.
L'association de ce syndrome à une altération de l'état général fébrile doit faire suspecter en premier lieu des métastases
ganglionnaires de cancer broncho-pulmonaire.
Le cancer anaplasique à petits cellules se développe dans la grande majorité des cas en coulées exobronchiques avec de
nombreuses adénopathies. Ceci explique l'aspect radiologique (élargissement du médiastin) et la fréquence du syndrome
cave supérieur.
NB item A : cancer de l'apex qui peut se traduire par un syndrome de Pancoast Tobias (douleurs radiculaires inférieures du
plexus brachial, atteinte du système sympathique avec syndrome de Claude Bernard Horner, cancer de l'apex avec lyse de la
première côte).

Chez ce malade sans antécédent, vous pensez avant tout à :


A - Un cancer anaplasique à petites cellules
B - Une maladie de Hodgkin
C - Un thymome malin
D - Un dysembryome
E - Un neuroblastome
Bonne(s) réponse(s) : A

Voir question [116].

Pour obtenir le diagnostic étiologique, vous demandez :


A - Une angiopneumographie
B - Un scanner thoracique
C - Une lymphographie
D - Une fibroscopie
E - Une ponction trans-trachéale
Bonne(s) réponse(s) : D

La fibroscopie montre généralement un aspect de compression extrinsèque et d'infiltration inflammatoire de la muqueuse,


cependant les biopsies pratiquées seront souvent positives.
En cas de négativité et en seconde intention, on pourrait alors discuter d'autres modes de prélèvements : ponction
transpariétale, médiastinoscopie voir thoracotomie exploratrice.
La recherche d'une métastase sera systématique d'autant plus qu'elle peut permettre des prélèvements histologiques.

On peut envisager comme traitement :


A - Héparinothérapie
B - Radiothérapie
C - Chimiothérapie
D - Corticothérapie
E - Chirurgie thoracique
Bonne(s) réponse(s) : A B C D

Le cancer anaplasique à petites cellules a une très grande chimiosensibilité. Dans cette indication, la chimiothérapie est
systématique d'autant plus que l'extension médiastinale ou métastatique quasi constante contre-indique la chirurgie. La
radiothérapie médiastinale peut être utilisée en association avec la chimiothérapie ou en urgence dans le traitement d'un
syndrome cave supérieur.
Enfin une anticoagulation efficace est urgente en raison du syndrome cave supérieure avec une corticothérapie parentérale.

962
Exclusivement sur DOC - DZ : www.doc-dz.com NADJI 85
RESIDANAT EN POCHE TOME II
Cas Clinique en QCM
Un homme de 55 ans (poids 100 kg pour 170 cm) présente les perturbations biologiques suivantes : glycémie à jeûn (8
heures) 2,00 g/l, glycémie post prandiale (9 h 30) 2,30 g/l, glycémie à 12h 1,05 g/l, et glycémie à 18h 1,30 g/l. L'hémoglobine
glycosylée totale est à 8 %. Le taux du cholestérol est à 2 g/l et les triglycérides à 3,5 g/l. L'interrogatoire révèle des crampes
nocturnes des mollets depuis plusieurs mois. A l'examen, on ne note rien de particulier. Si ce n'est une hypertension artérielle
modérée à 160/100 mm de Hg. La créatininémie est normale 11 mg/l, et l'examen des urines ne montre pas de protéinurie.

Vous pouvez conclure que :


A - Ce sujet a un diabète de type I
B - Ce sujet a un diabète de type II
C - Ce sujet a une simple intolérance au glucose
D - Ce sujet n'a ni diabète ni intolérance au glucose
E - Ce sujet a une hypertriglycéridémie endogène modérée (type IV)
Bonne(s) réponse(s) : B E

Ce sujet présente un diabète patent.


E - Car TG augmentés beaucoup plus que l'augmentation du cholestérol

Parmi les mesures thérapeutiques suivantes, vous mettez en oeuvre :


A - Régime hypocalorique
B - Régime de restriction sodée
C - Régime normocalorique sans sucres simples
D - Régime normocalorique et pauvre en graisses
E - Régime normocalorique mais hypoglucidique
Bonne(s) réponse(s) : A B

A - Evident.
B - Pour réduire l'HTA.

Supposez qu'au bout de quelques semaines ou mois les mesures thérapeutiques s'avèrent insuffisantes sur les
perturbations glycémiques. Parmi les traitements complémentaires suivants lequel allez-vous choisir ?
A - Prescription de sulfamides hypoglycémiants
B - Insulinothérapie
C - Prescriptions de biguanides
D - Prescriptions combinée de biguanides et de sulfamides
E - Prescription combinée d'insuline et de biguanides
Bonne(s) réponse(s) : C

Antidiabétiques de l'obèse.

Supposez qu'au bout de quelques semaines ou mois les nouvelles mesures thérapeutiques que vous avez
envisagées s'avèrent insuffisantes sur les perturbations lipidiques. Parmi les traitements suivants, vous pouvez
envisager :
A - Prescription d'un fibrate
B - Prescription de cholestyramine
C - Prescription de néomycine
D - Prescription d'un dérivé de la thyroxine
E - Modifications du régime avec prescription d'un régime enrichi en triglycérides à chaîne moyenne
Bonne(s) réponse(s) : A

Le fénofibrate après échec du régime peut faire diminuer les TG.

963
Exclusivement sur DOC - DZ : www.doc-dz.com NADJI 85
RESIDANAT EN POCHE TOME II
Cas Clinique en QCM
Un employé de bureau de 52 ans est hospitalisé pour aggravation récente de sa dyspnée et somnolence diurne. Dans ses
antécédents, on note une toux grasse chronique avec expectoration peu abondante, muqueuse et un tabagisme à 40
cigarettes/j, depuis l'âge de 18 ans. Durant les trois dernières années, il a noté une dyspnée d'effort s'aggravant
régulièrement. A l'examen, on observe une fréquence respiratoire de 20/mn, une cyanose, une PA à 160/90 mmHg, un rythme
cardiaque à 130/mn une température à 38° C et des oedèmes des membres inférieurs. L'expectoration est mucopurulente.
L'auscultation révèle des râles ronflants.
A l'entrée, les principaux résultats biologiques sont les suivants :
- VS à 32 à la première heure, kaliémie 4,9 mmol/l
- GR 5 980 000/mm3, GB 13 800/mm3 (90 % de polynucléaires neutrophiles), Hb 180 g/l
- Gazométrie artérielle (sans apport d'oxygène) : pH 7,30, PaO2 45 mmHg, (5.98 KPa), PaC02 55 mmHg (7,3 1 KPa), C03H
35 mmol/l. L'électrocardiogramme montre une tachycardie sinusale avec 16 extrasystoles ventriculaires par minute. La
radiographie pulmonaire révèle une distension thoracique, sans foyer parenchymateux.
Un pneumocoque est isolé de l'expectoration avant toute antibiothérapie.
A la sortie, une exploration fonctionnelle respiratoire donnera les résultats suivants (les valeurs normales théoriques sont
indiquées entre parenthèses) :
- Capacité vitale 4,1l (4,1l) - V.E.M.S .: 1,3l (2,86l) - Volume résiduel : 3,9l (2,3 l) - Capacité pulmonaire totale : 8l (6,4 l).
La gazométrie artérielle s'est stabilisée à pH 7,38 - PaO2 55 mmHg (7,33 KPa), PaC02 50 mmHg (6.67 KPa).

L'analyse des gaz du sang artériels indiqués à l'entrée du malade met en évidence :
A - Une acidose respiratoire aiguë
B - Une acidose respiratoire chronique
C - Une acidose mixte compensée
D - Une alcalose respiratoire
E - Une alcalose métabolique
Bonne(s) réponse(s) : A

Il existe une hypoventilation alvéolaire (définie par l'association hypoxie-hypercapnie). L'hypercapnie entraîne une
augmentation des bicarbonates plasmatiques (régulation d'origine rénale) mais cette augmentation est insuffisante pour
compenser l'acidose respiratoire due à l'hypercapnie :
pH = pK + Log (bicarbonates/PaCO2).

Quelles sont les 2 mesures thérapeutiques que vous proposez en priorité ?


A - Digitaliques
B - Perfusion alcaline
C - Oxygénothérapie nasale à 1 l/minute
D - Codéïne
E - Kinésithérapie
Bonne(s) réponse(s) : C E

A - Contre-indiqué en raison de l'hypoxie et de l'acidose qui accroient leurs risques de toxicité. De plus, lors des insuffisances
respiratoires aiguës chez des insuffisants respiratoires chroniques, et en l'absence de pathologie associée, le débit cardiaque
est normal ou élevé
B - L'alcalinisation ou l'emploi de sédatifs comportent un risque majeur de dépression des centres respiratoires. De plus, la
codéïne est un antitussif et la toux est nécessaire chez ces patients afin de réduire l'encombrement.
C E - Oxygénothérapie à faible débit (risque de dépression des centres respiratoires) et kinésithérapie sont fondamentales.

Quelle conclusion tirer de l'examen bactériologique chez ce patient ?


A - Le pneumocoque isolé impose une pénicillinothérapie
B - Le pneumocoque isolé impose un antibiogramme
C - L'examen de l'expectoration doit être répété
D - L'examen de l'expectoration n'a aucune valeur
E - Le pneumocoque isolé n'est pas pathogène
Bonne(s) réponse(s) : D

L'examen cytobactériologique des crachats est contaminé par la flore oropharyngée or le pneumocoque est un saprophyte de
cette flore. L'examen n'a donc pas de valeur.

Quelle conclusion tirer de l'exploration fonctionnelle respiratoire ?


A - Syndrome mixte à prédominance obstructive
B - Syndrome obstructif
C - Syndrome restrictif
D - Résultats ininterprétables
E - Syndrome mixte à prédominance restrictive
Bonne(s) réponse(s) : B

Un syndrome restrictif est défini par une baisse des volumes en particulier de la capacité pulmonaire totale (CT). Ici
l'augmentation de la CT élimine un syndrome restrictif et traduit une distention thoracique. Le VEMS est diminué avec un
rapport de Tiffeneau (VEMS/CV) diminué, ce qui caractérise un syndrome obstructif.

964
Exclusivement sur DOC - DZ : www.doc-dz.com NADJI 85
RESIDANAT EN POCHE TOME II
Cas Clinique en QCM

Sur quel argument indiquez-vous une oxygénothérapie au long cours ?


A - La PaO2 à la sortie du malade
B - La PaC02 à la sortie du malade
C - Valeur de l'exploration fonctionnelle respiratoire
D - Persistance de l'hypoxémie inférieure à 60 mmHg
E - Persistance d'une PaC02 supérieure 3,50 mmHg sur 2 mois
Bonne(s) réponse(s) : D

Connaissance. L'oxygénothérapie continue est indiquée dans les bronchopneumopaties chroniques obstructives qui
présentent une hypoxie inférieure à 60 mm de Hg en état stable (deux gaz du sang artériel) à plus de trois mois d'un épisode
de décompensation aiguë et ceci malgré un traitement médical (désencombrement bronchique, bronchodilatateurs,
rééducation respiratoire) correct.

Madame L. 38 ans, est hospitalisée en raison de l'association :


- d'une fièvre à 40° C
- de frissons généralisés
- d'une toux non productive.
Ces symptômes sont apparus brutalement et sont mal tolérés. A l'examen clinique, il existe un syndrome de condensation
basithoracique, antérieur droit, correspondant radiologiquement à une opacité hétérogène mal limitée, se projetant en regard
du lobe moyen. Parmi les examens biologiques obtenus, on retient l'existence d'une hyperleucocytose (25 000 GB/mm3) à
prédominance de polynucléaires neutrophiles (80 %). L'examen bactériologique des expectorations met en évidence la
présence de nombreux bacilles et cocci gram positifs.
Madame L... est alors traitée par Pénicylline G. Après une amélioration transitoire, la fièvre réapparait et survient un ictère
conjonctival dans un contexte d'altération de l'état général. L'image radiologique se modifie avec une extension des lésions
qui se bilatéralisent, restant grossièrement hilifuges. L'hyperleucocytose persiste mais le compte des hématies est impossible
(Coombs positif, présence d'hémagglutinines froides).

Le diagnostic de pneumonie bactérienne gram + est cohérent avec :


A - Fièvre
B - Frissons
C - Opacité radiologique non systématisée
D - Présence d'agglutinines froides
E - Hyperleucocytose
Bonne(s) réponse(s) : A B E

C - Peut se voir au décours de pneumopathies bactériennes à bacilles Gram négatif, de pneumopathies virales ou à germes
apparentés.
D - Est évocateur d'une pneumopathie à mycoplasme.

En cas de pneumonie bactérienne, la preuve bactériologique peut être apportée par:


A - L'examen d'expectoration
B - Les hémocultures
C - Le brossage bronchique protégé
D - La ponction trachéale
E - Aucune de ces propositions
Bonne(s) réponse(s) : B D E

A - L'examen cytobactériologique des crachats est difficile à interpréter (souillure par la flore oropharyngée), il peut être
complété par une numération de germes (non précisée ici).

A la lecture de cette observation, quel est l'agent microbien que vous devez incriminer en priorité ?
A - Streptococcus pneumoniae
B - Hémophilus influenzae
C - Mycoplasma pneumoniae
D - Staphylococcus aureus
E - Bactéroïdes fragilis
Bonne(s) réponse(s) : C

C - Sont en faveur du diagnostic : le début rapide, la non systématisation du foyer initial moins dense que dans un pneumonie
franche lobaire aiguë, le caractère secondairement bilatéral hilifuge des opacités, l'ictère (à biliruline libre) associé à une
anémie hémolytique auto-immune par agglutinines froides (IgM anti I).
A - Tableau initial est compatible mais l'évolution sous pénicilline doit faire rediscuter un autre agent étiologique.
B - Plus souvent responsable de surinfections bronchiques ou de bronchites aiguës. Les pneumopathies s'observent chez les
sujets ayant un tabagisme simple ou une bronchopathie chronique. Difficulté du diagnostic (le germe étant fréquemment
hébergé dans les voies respiratoires) : la positivité des hémocultures et la présence de souches encapsulées typables sont
des éléments du diagnostic.
D - Réalisant une pneumopathie bactérienne suppurative : début rapide ou progressif, terrain particulier, expectoration
purulente parfois hémoptoïque tendance à l'excavation du ou des foyers.
E - Bacille anaérobie responsable d'abcès pulmonaire.

965
Exclusivement sur DOC - DZ : www.doc-dz.com NADJI 85
RESIDANAT EN POCHE TOME II
Cas Clinique en QCM

Les pneumopathies aiguës de l'adulte non immunodéprimé justifient le recours en première intention :
A - Aux aminosides
B - Aux macrolides
C - Aux cyclines
D - Aux céphalosporines
E - A aucune de ces propositions
Bonne(s) réponse(s) : B

Macrolides 3 grammes/jour. Pour certaines pénicillines du groupe A - Par voie orale type amoxicilline et révision de la
thérapeutique si échec à 48 h.

Homme de 60 ans, adressé pour dyspnée d'effort rapidement croissante depuis plusieurs mois. Fumeur (15 cigarettes par jour
depuis 40 ans), il ne mentionne pas d'antécédent notable. Il a exercé diverses professions sédentaires, sans exposition à des
toxiques. Les radiographies de médecine du travail n'ont pas montré d'anomalies, mais le dernier examen remonte à plus de
deux ans. La dyspnée est apparue insidieusement, sans autre phénomène associé, pas de douleurs thoraciques, pas de toux,
ni crachats. Après une aggravation lente, le handicap est majeur depuis quelques mois : pas de dyspnée au repos, mais elle
apparaît après 20 ou 30 mn de marche à plat et pour 5 marches d'escalier.
L'examen met en évidence une tachypnée au repos à 25 par minute. L'ampliation thoracique est faible. A l'auscultation, râles
crépitants des deux bases. L'auscultation cardiaque montre un éclat de B1 au foyer pulmonaire, accompagné d'une discrète
hépatalgie et de petits oedèmes malléolaires. Présence d'un hippocratisme digital, avec cyanose unguéale. Amaigrissement
de 10 kg en quelques mois. Examen général normal. Pas de fièvre. Radiographie pulmonaire : image interstitielle, de type
réticulonodulaire, diffuse aux deux champs pulmonaires, avec ascension des deux coupoles. Médiastin et plèvre sont
normaux. Majoration des ombres vasculaires pulmonaires. avec augmentation du volume des branches d'origine des artères
pulmonaires. Examens complémentaires :
- spirométrie : CV 2,800 l - valeur théorique = 4 l.
VEMS 2,500 l - valeur théorique = 2.8 l
VEMS/CV = 80 % - valeur théorique = 70 %
- gaz du sang artériel :
(Air repos) (Air effort) (Sous 02 : 100 %)
- Pa02 (mm Hg) 55 40 500
- Sa02 % 87 82 99
- PaC02 (mm Hg) 34 33 40
- pH 7,46 7,37 7,41
- C02 total (mmol/l) 25 26 26

L'analyse de la spirométrie permet de conclure à:


A - Un syndrome obstructif
B - Un syndrome restrictif
C - Un syndrome de distension
D - Un syndrome obstructif avec distension
E - Un syndrome restrictif avec distension
Bonne(s) réponse(s) : B

Le rapport de Tiffeneau est normal (supérieur à 75 %) ce qui élimine un syndrome obstructif (A et E faux).
La distention ne peut être affirmée que sur la capacité pulmonaire totale (CT). Il existe une diminution des volumes (capacité
vitale) avec diminution des débits correspondants. Ceci suggère un syndrome restrictif qui serait affirmé sur la diminution de la
CT.

Chez ce malade, l'hypoxie peut s'expliquer par :


A - Une inadéquation du rapport ventilation-perfusion
B - Un effet espace mort
C - Une hypoventilation alvéolaire
D - Un trouble de diffusion des gaz
E - Une diminution du temps de contact air/sang
Bonne(s) réponse(s) : A B D E

Il existe chez ce malade une hyperventilation en réponse au stimulus hypoxique (C faux).

L'ensemble des données cliniques et paracliniques évoque :


A - Une cardiopathie gauche
B - Une bronchopneupathie chronique obstructive
C - Une fibrose pulmonaire primitive
D - Une fibrose pulmonaire secondaire à une collagénose
E - Une miliaire tuberculeuse
Bonne(s) réponse(s) : C

Le tableau clinique radiologique et spirométrique est celui d'une pneumopathie : cliniquement, dyspnée d'effort, crépitants aux
deux bases, signes d'hypertension artérielle pulmonaire ; radiologiquement, syndrome interstitiel, rétraction parenchymateuse
(ascension des coupoles) et hypertension artérielle pulmonaire ; gazométriquement, hypoxie majeure par l'effort avec alcalose
respiratoire traduisant un bloc alvéolocapillaire corrigé par inhalation d'oxygène pur.
En faveur de la fibrose idiopathique : l'existence d'un hippocratisme digital (80 % des fibroses idiopathiques), l'absence de
signes cliniques de collagénose, l'absence d'étiologies toxiques.

966
Exclusivement sur DOC - DZ : www.doc-dz.com NADJI 85
RESIDANAT EN POCHE TOME II
Cas Clinique en QCM

On peut retenir comme élément péjoratif :


A - L'hippocratisme digital
B - La présence de râles crépitants
C - L'abaissement du transfert du CO
D - La rapidité de l'évolution récente
E - L'aggravation de l'hypoxie à l'effort
Bonne(s) réponse(s) : C D

C - L'abaissement du transfert du CO traduit l'importance du bloc alvéolo capillaire.


D- Evident.

La confirmation du diagnostic pourrait être apportée par :


A - Un cathétérisme cardiaque droit
B - Une biopsie pulmonaire transbronchique
C - La baisse de la compliance pulmonaire
D - La recherche d'anticorps antipoumon
E - La disparition des signes cliniques et radiologiques sous corticoïdes
Bonne(s) réponse(s) : C

La biopsie pulmonaire à pince bloquée n'oriente pas vers un diagnostic étiologique de la fibrose qui est spécifique mais
permet d'écarter d'autres hypotèses.
La réponse à la corticothérapie est inconstante (20 à 40 % des cas).

Le traitement de la dyspnée repose sur :


A - Les analeptiques respiratoires
B - L'oxygénothérapie,continue (repos)
C - Les digitaliques
D - L'oxygénothérapie à l'effort
E - La prise régulière d'un diurétique
Bonne(s) réponse(s) : B

Les analeptiques respiratoires n'ont aucune place (l'hypoxie exerçant déjà un stimulus des centres respiratoires).
Les digitaliques sont contre indiquées du fait de l'hypoxie. La prise régulière d'un diurétique n'a pas de raison en première
intention car la symptomatologie de l'insuffisance ventriculaire droite est secondaire à l'hypertension artérielle pulmonaire
hypoxique (de plus l'énoncé précise qu'il s'agit du traitement de la dyspnée).
Le traitement est donc l'oxygénothérapie : au repos (car PaO2<60 mm de Hg) et à l'effort. Les autres traitements sont la
prescription large et précoce des antibiotiques (fréquence et gravité des surinfections, traitement d'un coeur pulmonaire
chronique éventuel).

Monsieur X., 62 ans, souffre d'une myocardiopathie non obstructive en apparence primitive. Depuis une première poussée
d'insuffisance cardiaque globale, il y a un an, il reçoit les médicaments suivants : Sintrom®, Digitaline®, Lasilix®, Aldactone®
et Cordarone®. Depuis deux mois, sa condition cardiovasculaire se dégrade à nouveau : il est dyspnéique au moindre effort,
se plaint d'une asthénie extrême et perd du poids malgré la réapparition de volumineux oedèmes des membres inférieurs.
L'examen clinique note une arythmie à 90/mn, une pression artérielle à 110/90 mmHg. La radiographie thoracique montre une
cardiomégalie importante (ICT = 0,65), une hypervascularisation pulmonaire et un épanchement pleural gauche. L'ECG révèle
une fibrillation auriculaire et un bloc de branche droit complet. Un cathétérisme cardiaque droit confirmé l'élévation des
pressions de remplissage ventriculaires (POD à 18 mmHg et PCP à 30 mmHg) et un débit cardiaque très abaissé (IC = 1,5
I/mn/m2).
Devant ce tableau d'insuffisance cardiaque de stade IV réfractaire, le recours à un traitement vasodilatateur apparaît
souhaitable.

Parmi les signes suivants tirés de l'observation, lequel ou lesquels peuvent être attribués à une insuffisance
ventriculaire droite ?
A - Dyspnée
B - Epanchement pleural gauche
C - OEdèmes des membres inférieurs
D - Bloc de branche droit
E - Augmentation de la POD
Bonne(s) réponse(s) : C D E

A B - Sont plutôt des signes gauches.

967
Exclusivement sur DOC - DZ : www.doc-dz.com NADJI 85
RESIDANAT EN POCHE TOME II
Cas Clinique en QCM

Si on prescrit un dérivé nitré chez ce patient, quel(s) effet(s) clinique(s) et hémodynamique(s) favorable(s) peut-
on espérer en priorité ?
A - Réduction de la dyspnée
B - Augmentation de la PA systolique
C - Réduction de la POD
D - Réduction de la PCP
E - Augmentation du débit cardiaque
Bonne(s) réponse(s) : A C D

Par diminution de la précharge.

Si on prescrit isolément du Nepressol® (hydralazine) quel est l'effet favorable (clinique ou hémodynamique) que
l'on peut espérer en priorité ?
A - Réduction de la dyspnée
B - Diminution de la PA systolique
C - Réduction de la POD
D - Réduction de la PCP
E - Augmentation du débit cardiaque
Bonne(s) réponse(s) : A E

Sans commentaire.

Si on prescrit du Minipress® (prazosine), quel(s) effet(s) clinique(s) et hémodynamique(s) favorables peut-on


espérer ?
A - Réduction de la dyspnée
B - Diminution de la PA systolique
C - Réduction de la POD
D - Réduction de la PCP
E - Augmentation du débit cardiaque
Bonne(s) réponse(s) : A C D

Il s'agit d'un vasodilatateur et bloquant mixte.

Si on prescrit du Lopril® (captopril), quel(s) effet(s) favorable(s) clinique(s) ou hémodynamique(s) peut-on


espérer ?
A - Réduction de la dyspnée
B - Diminution de la PA systolique
C - Réduction de la POD
D - Réduction de la PCP
E - Augmentation du débit cardiaque
Bonne(s) réponse(s) : A C D

Diminution de la pré et post charge.

En cas de prescription de Lopril®, quel est parmi les 5 médicaments antérieurement prescrits chez ce patient
celui dont le maintien est a priori contre indiqué ?
A - Sintrom®
B - Digitaline®
C - Lasilix®
D - Aldactone®
E - Cordarone®
Bonne(s) réponse(s) : D

Car risque d'hyperkaliémie.

968
Exclusivement sur DOC - DZ : www.doc-dz.com NADJI 85
RESIDANAT EN POCHE TOME II
Cas Clinique en QCM
Un homme de 48 ans, maçon, consulte pour hémoptysie. Dans ses antécédents, on note une tuberculose pulmonaire traitée
10 ans auparavant par une association rifampicine, isoniazide, éthambutol pendant 2 mois, puis rifampicine, isoniazide
pendant 10 mois supplémentaires. Hygiène de vie : alcool environ 1 litre de vin par jour, tabac 15 à 20 cigarettes par jour
depuis 30 ans. Etat général relativement conservé. A l'auscultation : submatité au niveau du sommet droit, quelques
sibilances aux deux temps respiratoires. Examen radiologique : opacité systématisée et rétractile du lobe supérieur droit.
Quelques opacités trabéculo-nodulaires dans la région rétroclaviculaire gauche. Pas de cavité visible.

Chez ce malade 3 diagnostics pouvaient être évoqués compte tenu des antécédents et des signes actuels :
cancer, rechute de tuberculose, greffe aspergillaire. Parmi les propositions suivantes, quelle(s) est(sont) celle(s)
qui rende(nt) le diagnostic de cancer le plus probable ?
A - Le traitement de la tuberculose a été correct
B - Habitudes tabagiques
C - Occupations professionnelles
D - Absence de cavités résiduelles de la tuberculose
E - Aspect radiologique du lobe supérieur droit
Bonne(s) réponse(s) : A B D E

A - Exact - Une triple association anti tuberculeuse (Rifampicine Isoniazide Ethambuthol) pendant 12 mois entraîne une
guérison de la tuberculose dans près de 100 % de cas.
B - Tabagisme chiffré ici à environ 30 paquets années.
B - La tuberculose évolutive se caractérise par deux lésions élémentaires : les nodules (dont la confluence réalise des infiltrats
) et les cavernes
C - Risque professionnel d'exposition à la silice (qui n'est pas un facteur favorisant des cancers bronchiques).

Devant la constatation d'un bourgeon endo-bronchique en endoscopie, quel est le diagnostic le plus probable ?
A - Cylindrome
B - Tumeur carcinoïde
C - Cancer épidermoïde
D - Cancer anaplasique à petites cellules
E - Cancer bronchiolo-alvéolaire
Bonne(s) réponse(s) : C

A et B - Tumeurs bronchiques "à malignité réduites", tumeurs primitives épithéliales rares (2 à 3 % des néoplasies
bronchiques).
C - C'est le type histologique le plus fréquent (40 % des néoplasies bronchiques). Touchant presque exclusivement les
grosses bronches, il est visualisé en endoscopie sous la forme d'un bourgeon plus que de végétations. D - Il se développe en
coulées exobronchiques avec nombreuses adénopathies d'où l'aspect endoscopique de compressions extrinsèques et
d'infiltrats inflammatoires de la muqueuse.
E - Aspect normal de l'arbre bronchique ou aspect d'infiltration cartonnée.

Le prélèvement biopsique du bourgeon endobronchique montre histologiquement des amas de cellules


acidophiles, à noyau hyperchromatique parfois en mitose et à cytoplasme assez abondant, formant des globes
parakératosiques. Quel diagnostic doit-on poser ?
A - Granulome nodulaire épithélioïde
B - Cancer épidermoïde
C - Cancer anaplasique à petites cellules
D - Cancer bronchiolo-alvéolaire
E - Tumeur carcinoïde bronchique
Bonne(s) réponse(s) : B

Bien que provenant d'un revêtement cylindrique glandulaire, le cancer épidermoïde reproduit les structures des différentes
couches épithéliales cutanées (basale, stratifiée, cornée). Les globes parakératosiques signent la maturation cornée.

Le traitement chirurgical du cancer bronchique est souhaitable quand il n'existe pas de contre-indications. Parmi
les circonstances suivantes, indiquez celle(s) qui n'est (sont) pas une (des) contre-indication(s) opératoire(s) :
A - Paralysie récurrentielle gauche
B - Abaissement du VEMS au dessous de 800 ml
C - Epanchement pleural liquidien de nature inflammatoire
D - Epanchement pleural liquidien contenant des cellules cancéreuses
E - Découverte du cancer à l'occasion d'un pneumothorax spontané
Bonne(s) réponse(s) : C E

Connaissance.

969
Exclusivement sur DOC - DZ : www.doc-dz.com NADJI 85
RESIDANAT EN POCHE TOME II
Cas Clinique en QCM
Un homme de 65 ans, fumeur ( 30 paquets / année), et porteur d'une bronchite ancienne avec syndrome spirographique
abstructif, est hospitalisé pour une aggravation de son état déterminée par une surinfection bronchique saisonnière. Il
présente une toux fréquente avec expectoration muco-purulente. Il est dyspnéïque. La température est à 38.2 degrés. A
l'examen : cyanose modérée, râles ronflants diffus sans foyer de condensation. Coeur rapide. Le foie est légèrement
hypertrophié et sensible. Il n'y a pas d'oedème des membres inférieurs. La radiographie pulmonaire met en évidence une
distension et une surcharge des bases, sans foyer.
Electrocardiogramme : tacycardie sinusale à 96/minute, conduction auriculoventriculaire normale, onde P ample et pointue
d'amplitude supérieure à 2 mm en D2, D3, AVF, axe de QRS à +130°, persistance d'ondes S en V5, V6.
Gaz du sang artériel : PaO2 50 mmHg (6,7 kpa), PaCO2 58 mmHg (7,9 kpa), HCO3 36 millimoles/litre, pH 7,40.
Numération formule sanguine : 6.060.000 globules rouges par mm3, 180 g par litre d'hémoglobine, 59% d'hématocrite, 12000
leucocytes par mm3 avec 78 % de polynucléaires neutrophiles.

Lors d'une épreuve fonctionnelle respiratoire qui sera pratiquée chez ce patient, vous pouvez affirmer le
syndrome obstructif sur :
A - La diminution de la capacité vitale (CV)
B - La diminution du volume expiratoire maximal seconde (VEMS)
C - La diminution du rapport volume expiratoire maximal seconde sur capacité vitale (VEMS/CV)
D - L'augmentation du rapport volume résiduel sur capacité total (VR/CPT)
E - L'élévation de la capacité résiduelle fonctionnelle (CRF)
Bonne(s) Réponse(s) : C

Dans un syndrome obstructif le VEMS est abaissé par rapport à la capacité vitale (VEMS/CV inférieur à
75 % ou 65 % chez le sujet de plus de 60 ans).
Un syndrome obstructif débutant est apprécié sur le débit maximal à 50 % de la capacité vitale car le rapport de Tiffeneau
peut être normal

Les gaz du sang artériel sont compatibles avec le diagnostic :


A - D'acidose mixte
B - D'hypoxie avec acidose ventilatoire compensée
C - D'alcalose ventilatoire
D - D'hypoxie avec acidose métabolique
E - Aucun de ces diagnostics
Bonne(s) Réponse(s) : B

Il existe une hypoxie (PaO2 inférieure à 80 mmHg) et une hypercapnie (PaCO2 supérieure à 40 mmHg). L'acidose respiratoire
due à l'hypercapnie est compensée (pH à 7,40) par une augmentation des bicarbonates plasmatiques.

Le tracé électrocardiographique est compatible avec le diagnostic de :


A - Tachycardie paroxystique
B - Hypertrophie ventriculaire gauche
C - Hypertrophie ventriculaire droite
D - Ischémie myocardique
E - Hypertrophie auriculaire droite
Bonne(s) Réponse(s) : C E

C - Déviation axiale droite de l'axe de QRS (au delà de + 110°) et ondes S persistantes dans les dérivations précordiales
gauches.
E - Onde P pointue en D2 D3 VF (mais de durée normale) et dont l'amplitude excède 2 mm (théoriquement 2,5 mm) en D2.

Quel est le témoin le plus fiable de l'ancienneté de l'insuffisance respiratoire ?


A - Pa O2 = 6,7 kPa (50 mm Hg)
B - Pa CO2 = 7,9 kPa (58 mm Hg)
C - HCO3 = 36 millimoles/litre
D - Hémoglobine = 180 g/l
E - Globules Blancs = 12.000/mm3
Bonne(s) Réponse(s) : C D

Il existe deux signes biologiques de chronicité de l'insuffisance respiratoire - une polyglobulie définie par l'augmentation de
l'ématocrite (plus fiable que l'augmentation du taux d'hémoglobuline, item D) : elle témoigne d'une tentative de compensation
de la baisse de la PaO2 artérielle par une augmentation du contenu artériel en oxygène. Une élévation des bicarbonates
plasmatiques (mécanisme compensateur rénal d'une acidose respiratoire).

970
Exclusivement sur DOC - DZ : www.doc-dz.com NADJI 85
RESIDANAT EN POCHE TOME II
Cas Clinique en QCM

Quelle(s) mesure(s) thérapeutique(s) faut-il conseiller ?


A - Une saignée lente de 400 ml
B - Une oxygénothérapie par sonde nasale à faible débit
C - La prescription de codéïne
D - La prescription d'une antibiothérapie type Ampicilline
E - L'assistance au drainage bronchique par un kinésithérapeute
Bonne(s) Réponse(s) : C

La codéïne est un alcaloïde de l'opium qui a une action anti-tussive et qui déprime les centres respiratoire. La prescription des
sédatifs et de tous médicaments déprimant les centres respiratoires est prescrite.

Monsieur M., manoeuvre de son métier, fumeur de 20 cigarettes par jour, est adressé par son médecin dans un service de
Pneumologie à la suite de l'apparition d'un oedème de la face et des creux sus-claviculaires, avec distension des veines
jugulaires et circulation collatérale thoracique antérieure. L'aspect est celui d'un syndrome de compression de la veine cave
supérieure. Le cliché de thorax révèle une image en bande d'environ 1 cm d'épaisseur, s'étendant horizontalement de la partie
supérieure du hile à la région axillaire sur le cliché de face, et à la paroi thoracique antérieure sur le cliché de profil. Le
médiastin supérieur est élargi et l'on note des opacités polylobées évoquant des adénopathies au niveau du hile droit. Le reste
de l'examen clinique est sans particularité.

Quelle interprétation donnez-vous à l'image en bande, visible au niveau du champ pulmonaire droit ?
A - Scissurité
B - Epanchement pleural enkysté dans la petite scissure
C - Atélectasie du segment ventral du lobe supérieur droit
D - Atélectasie du lobe moyen
E - Aucune de ces hypothèses
Bonne(s) réponse(s) : C

Non commentée, connaissance.

La fibroscopie révèle une infiltration avec sténose partielle d'un orifice bronchique en rapport avec un cancer
anaplasique à petites cellules (diagnostiqué à la biopsie). Quelle(s) mesure(s) thérapeutique(s) adopteriez-vous
?
A - Abord chirurgical avec exérèse pulmonaire et curage ganglionnaire pour lever la compression cave
supérieur
B - Association radiothérapie-chimiothérapie
C - Abstention thérapeutique
D - Mise en oeuvre d'un traitement anticoagulant
E - Mise en oeuvre d'un traitement fibrinolytique
Bonne(s) réponse(s) : B D

Pas de chirurgie mais association chimio-radiothérapie et anticoagulant du fait de la stase veineuse (VCS).

En dehors du cancer anaplasique à petites cellules, quelle(s) autre(s) formation(s) pathologique(s), peu(ven)t
entrainer le développement d'un syndrome de compression cave supérieur ?
A - Adénopathies sarcoïdiennes
B - Goître plongeant
C.- Thymome Iympho-épithélial
D - Kyste pleuro-péricardique
E - Neurinome
Bonne(s) réponse(s) : B C

Les adénopathies sarcoïdiennes ne sont pas compressives. Le kyste pleuropéricardique est dans le médiastin postérieur.

Les cancers bronchiques anaplasiques sont caractérisés par :


A - Leur faible pouvoir métastatique
B - La fréquence des syndromes associés paranéoplasiques
C - Le caractère tardif de l'envahissement ganglionnaire médiastinal
D - L'absence de granules de sécrétion dans les cellules néoplasiques en microscopie électronique
E - Leur évolution plus rapide que les cancers épidermoïdes ou glandulaires
Bonne(s) réponse(s) : B D E

Connaissance.

971
Exclusivement sur DOC - DZ : www.doc-dz.com NADJI 85
RESIDANAT EN POCHE TOME II
Cas Clinique en QCM
Monsieur M., manoeuvre de son métier, fumeur de 20 cigarettes par jour, est adressé par son médecin dans un service de
Pneumologie à la suite de l'apparition d'un oedème de la face et des creux sus-claviculaires, avec distension des veines
jugulaires et circulation collatérale thoracique antérieure. L'aspect est celui d'un syndrome de compression de la veine cave
supérieure. Le cliché de thorax révèle une image en bande d'environ 1 cm d'épaisseur, s'étendant horizontalement de la partie
supérieure du hile à la région axillaire sur le cliché de face, et à la paroi thoracique antérieure sur le cliché de profil. Le
médiastin supérieur est élargi et l'on note des opacités polylobées évoquant des adénopathies au niveau du hile droit. Le reste
de l'examen clinique est sans particularité.

Parmi les chimiothérapies suivantes, utilisées chez ce malade, quelle est celle qui est susceptible d'entrainer le
développement d'une insuffisance cardiaque ?
A - Adriamycine
B - Cyclophosphamide
C - Bléomycine
D - 5 fluoro-uracile
E - Oncovin
Bonne(s) réponse(s) : A

Connaissance.

Une patiente de 35 ans, au chômage, sans antécédent, est hospitalisée pour suspicion de tuberculose devant un décalage
thermique à 38° C, des sueurs nocturnes, une altération de l'état général évoluant depuis 2 mois. Les jours qui ont précédé
l'hospitalisation, elle a expectoré du sang à 3 reprises.

Pour étayer le diagnostic de tuberculose, dès l'admission, vous demandez :


A - NFS, VS
B - Radiographie pulmonaire
C - Recherche de B.K. dans les expectorations
D - Intradermo-réaction à la tuberculine
E - Fibroscopie bronchique
Bonne(s) réponse(s) : B C D

Il est probable que la radiographie pulmonaire montrera des infiltrats avec cavernes caractéristiques et que la recherche de
BK sera positive dès l'examen direct, devant ce tableau clinique.

Le diagnostic confirmé, vous avez le choix entre deux protocoles thérapeutiques : lesquels ?
A - Isoniazide - rifampicine - pyrazinamide - éthambutol
B - Isoniazide - rifampicine - éthambutol
C - Isoniazide - rifampicine - pyrazinamide
D - Isoniazide - rifampicine - streptomycine
E - Rifampicine - éthambutol
Bonne(s) réponse(s) : A B

Deux protocoles sont employés :


- une quadrithérapie de 6 mois : INH-ETB-RF-PZA pendant 2 mois puis INH-RF pendant les 4 mois suivants
- ou bien une trithérapie de 9 mois : INH-RF-ETB pendant 3 mois puis INH-RF les 6 mois suivants.

Sous traitement, elle se plaint de phénomènes douloureux de l'orteil gauche, rapportés à une crise de goutte en
raison d'une uricémie très élevée. Quel médicament suspectez-vous ?
A - Rifampicine
B - Isoniazide
C - Pyrazinamide
D - Ethambutol
E - Streptomycine
Bonne(s) réponse(s) : C

La pyrazinamide augmente le taux d'uricémie, la goutte est alors possible mais rare et impose l'arrêt du médicament.

Sous traitement, les transaminases augmentent dans une proportion de 7 fois les valeurs normales. Quel
médicament suspectez-vous ?
A - Rifampicine
B - Isoniazide
C - Pyrazinamide
D - Ethambutol
E - Steptomycine
Bonne(s) réponse(s) : B

Les métabolites de l'INH® sont hépatotoxiques. Mais c'est le rifampicine qui induit la formation de ces métabolites au niveau
hépatique.
972
Exclusivement sur DOC - DZ : www.doc-dz.com NADJI 85
RESIDANAT EN POCHE TOME II
Cas Clinique en QCM

Et que conseillez-vous vis-à-vis de ce médicament ?


A - Arrêt définitif
B - Arrêt puis reprise à des posologies progressives
C - Arrêt puis reprise à des posologies identiques
D - Réduction progressive jusqu'à l'arrêt
E - Poursuite à la même dose
Bonne(s) réponse(s) : B

Il faut arrêter l'INH® et le reprendre à posologie progressive après normalisation du taux des transaminases. La reprise du
traitement s'effectue en surveillant de très près les transaminases et en s'arrêtant à demi-doses d'INH®.

Le cas échéant, une interférence métabolique avec la rifampicine se manifesterait en cas de prise simultanée
de :
A - Hypoglycémiants
B - Antivitamines K
C - Héparine sous cutanée
D - Oestroprogestatifs
E - Aminosides
Bonne(s) réponse(s) : B D

La rifampicine diminue la concentration de ces 2 groupes de médicaments et donc leur efficacité thérapeutique.

Un homme de 44 ans, ingénieur, fumeur depuis l'âge de 16 ans (10 à 30 cigarettes brunes sans filtre par jour), sans autre
antécédent notable, consulte, car au cours d'une visite d'embauche après changement d'employeur, on a trouvé à la radio
pulmonaire systématique une opacité au niveau du hile droit.

Le(s) facteur(s) de risque de cancer bronchique que vous recherchez est(sont) :


A - Quantité de tabac fumé
B - Quantité de tabac chiqué
C - Exposition à l'amiante
D - Alcoolisme chronique
E - Antécédent familial de cancer bronchique
Bonne(s) réponse(s) : A C E

Le tabac chiqué favorise les cancers ORL. On admet actuellement comme très classiques les facteurs génétiques et
environnement (tabac, amiante) comme facteurs de risque de cancers.

La suspicion d'un cancer bronchique doit faire rechercher :


A - Crachats hémoptoïques
B - Paralysie récurrentielle droite
C - Dyspnée d'effort
D - Douleurs thoraciques
E - Adénopathie sus-claviculaire
Bonne(s) réponse(s) : A C D E

Les hémoptysies sont un signe à forte orientation diagnostique. La dyspnée fait partie du bilan d'opérabilité, les douleurs
thoraciques et adénopathies du bilan d'extension.

Quel est l'examen parmi les suivants qui permet le plus souvent le diagnostic positif ?
A - Tomodensitométrie thoracique
B - Médiastinoscopie
C - Endoscopie bronchique
D - Dosage de l'antigène carcino-embryonnaire
E - Ponction transpariétale
Bonne(s) réponse(s) : C

L'endoscopie bronchique réalisée grâce à un bronchoscope souple utilisant les fibres de verre (fibroscope) a révolutionné le
diagnostic de cancer bronchique.

973
Exclusivement sur DOC - DZ : www.doc-dz.com NADJI 85
RESIDANAT EN POCHE TOME II
Cas Clinique en QCM

Si l'examen anatomopathologique révèle un carcinome épidermoïde et si le bilan ne montre pas d'extension


médiastinale à distance, quel est le traitement qui sera envisagé en premier lieu ?
A - Chimiothérapie
B - Chirurgie
C - Radiothérapie
D - Association chimio et radiothérapie
E - Immunothérapie
Bonne(s) réponse(s) : B

La chirurgie d'exérèse complète reste le seul traitement qui est susceptible de guérir un cancer bronchique.

Sur 100 malades atteints de carcinome bronchique, tous stades, tous âges, tous types histologiques confondus,
combien sont en vie 5 ans après le diagnostic ?
A-0à1
B - 5 à 10
C - 15 à 20
D - 25 à 30
E - 35 à 40
Bonne(s) réponse(s) : B

Un quart des cancers du poumon diagnostiqués sont opérables. Parmi ces malades, 1/4 seulement sont survivants à 5 ans.
Ceci fait environ 5 à 10%.

Vous êtes appelé d'urgence auprès d'un homme de 35 ans, éthylique connu, qui présente depuis 48 heures un syndrome
respiratoire aigu, fébrile et tussigène. Il a débuté brutalement par un point de côté violent, basi-thoracique droit. La
température s'est rapidement élevée à 40 avec sensation de malaise et apparition de grands frissons. Il n'a pas d'antécédents
de tuberculose, il n'a pas de diabète. L'étude des gaz du sang artériel : PaO2 = 60 mmHg (8 kPa), PaC02 = 35 mmHg (4,7
kPa) pH = 7,50, bicarbonates = 27 mEq/l. Les hémocultures sont positives pour streptococcus pneumoniae. La radiographie
thoracique permet de constater une opacité systématisée du lobe supérieur droit sans atteinte pleurale.

Parmi les constatations cliniques suivantes, quel(s) signe(s) physique(s) devriez-vous observer dans le creux
axillaire droit ?
A - Matité et abolition des vibrations vocales
B - Tympanisme
C - Souffle tubaire et râles crépitants
D - Souffle amphorique
E - Râles sibilants
Bonne(s) réponse(s) : C

Le souffle tubaire et les râles crépitants sont caractéristiques de pneumopathie et sont habituellement associés à une matité et
une augmentation des vibrations vocales. Le creux axillaire droit correspond plus à la projection du segment postéro-externe
du lobe moyen qu'au lobe supérieur.

Parmi les propositions suivantes, quelle est celle qui permet d'expliquer les résultats des gaz du sang artériel ?
A - Hypoventilation alvéolaire
B - Troubles du transfert des gaz
C - Effet shunt localisé
D - Troubles diffus de la ventilation et perfusion pulmonaire
E - Effet espace mort
Bonne(s) réponse(s) : C

L'hypoxie et l'hypocapnie légère constatées ici réalisent un effet shunt.

Parmi les examens suivants, citez le(s) examens(s) utilisé(s) pour préciser la cause éventuelle de cette
pneumonie à streptococcus pneumoniae :
A - Numération formule sanguine
B - Exploration fonctionnelle respiratoire
C - Examen radiographique des dents
D - Radiographie des sinus maxillaires
E - Endoscopie bronchique
Bonne(s) réponse(s) : C D E

La cause la plus fréquente des pneumopathies à pneumocoques est une infection de la sphère ORL ou dentaire. La
fibroscopie bronchique permet de rechercher un obstacle endobronchique (type sténose néoplasique) surtout s'il s'agit d'un
gros fumeur même, si cet homme n'a que 35 ans).

974
Exclusivement sur DOC - DZ : www.doc-dz.com NADJI 85
RESIDANAT EN POCHE TOME II
Cas Clinique en QCM

Parmi les complications des pneumopathies à redouter, quelle(s) est (sont) celle(s) qui doive(nt) être
recherchée(s) dans ce cas ?
A - Anémie hémolytique
B - Pleurésie purulente
C - Choc septique
D - Méningite purulente
E - Syndrome de Stevens Johnson
Bonne(s) réponse(s) : B D

Pleurésie et méningite purulentes sont fréquentes dans l'évolution d'une pneumonie à pneumocoques. L'anémie hémolytique
est une complication classique des pneumopathies à mycoplasma pneumoniae (agglutinines froides). Le choc septique se voit
plus souvent dans les infections à Gram.

Parmi les antibiotiques suivants, quel(s) est (sont) celui (ceux) que l'on peut prescrire chez ce patient ?
A - Colimycine
B - Erythromycine
C - Pénicilline G
D - Acide nalidixique
E - Nétromycine
Bonne(s) réponse(s) : B C

La Pénicilline G reste le traitement classique. Le pneumocoque n'est presque jamais résistant à la Pénicilline G. La posologie
doit être de 3 millions, 4 fois par 24 heures, en perfusion intraveineuse régulièrement réparties.

Un homme âgé, traité pour cancer prostatique par oestrogènes, présente une fièvre à 38° C, une dyspnée importante, une
atteinte de l'état général.

Vous suspectez une pleurésie abondante devant :


A - Matité hémithoracique
B - Abolition des vibrations vocales
C - Frottements pleuraux
D - Souffle pleurétique
E - Hémithorax muet
Bonne(s) réponse(s) : A E

Souffle et frottement disparaissent si épanchement important.

Chez ce malade, l'existence d'un épanchement pleural droit abondant est confirmé, sur la radio pulmonaire par :
A - Hémithorax opaque
B - Pincement des côtes
C - Attraction du médiastin à droite
D - Refoulement du médiastin à gauche
E - Abaissement de la coupole diaphragmatique droite
Bonne(s) réponse(s) : A D E

Evident.

Quelle étiologie suspectez-vous ?


A - Pleurésie tuberculeuse
B - Mésothéliome pleural malin
C - Pleurésie métapneumonique
D - Pleurésie secondaire à cancer bronchique
E - Pleurésie métastatique d'origine prostatique
Bonne(s) réponse(s) : E

D'après l'énoncé.

975
Exclusivement sur DOC - DZ : www.doc-dz.com NADJI 85
RESIDANAT EN POCHE TOME II
Cas Clinique en QCM

En faveur d'un épanchement exsudatif, l'analyse biochimique du liquide pleural peut montrer :
A - Taux de protides supérieur à 35 g/l
B - Rivalta positif
C - Taux élevé d'amylases
D - Taux élevé d'acide hyaluronique
E - Présence de phosphatases acides
Bonne(s) réponse(s) : A B E

Par définition.

Pour confirmer votre diagnostic vous pouvez demander :


A - Cytologie pleurale
B - Scanner
C - Fibroscopie bronchique
D - Biopsie pleurale à l'aiguille
E - Thoracoscopie
Bonne(s) réponse(s) : A B D

C - Point de départ extra bronchique.


E - Il existe une AEG.

Parmi les traitements proposés, quel est celui que vous retenez ?
A - Corticothérapie générale
B - Polychimiothérapie
C - Chimiothérapie intrapleurale
D - Isotopes intrapleuraux
E - Symphyse pleurale par talcage
Bonne(s) réponse(s) : C

Sans commentaire.

Un homme de 63 ans consulte pour l'aggravation rapide d'une dyspnée d'effort et l'apparition de crachats hémoptoïques. Il se
plaint de fatigue, de céphalées et d'endormissement facile. C'est un bronchitique chronique de longue date, fumeur avéré (40
paquets-année) et éthylique qui a travaillé 20 ans comme mineur de fer, puis dans une minoterie. Il a maigri de plus de 10 kg.
L'examen montre des creux sus-claviculaires comblés, des jugulaires discrètement turgescentes, une ébauche de circulation
collatérale à la partie supérieure du thorax. On ne trouve pas d'adénopathie superficielle palpable. Le foie est normal. La R.P.
met en évidence une opacité dense, polylobée, latéro-trachéale droite : le parenchyme pulmonaire paraît normal, sauf au
niveau de la pointe du coeur où existent des travées opaques allongées et tubulées. Le reste de l'examen est normal.

Quel est le diagnostic que vous pouvez d'emblée évoquer :


A - Sarcoïdose
B - Cancer bronchique
C - Lymphome malin non Hodgkinien
D - Thymome malin
E - Maladie de Hodgkin
Bonne(s) réponse(s) : B

L'existence d'hémoptysies élimine déjà thymome et sarcoïdose. Sur ce terrain, plus qu'un Iymphome hodgkinien ou non
hodgkinien, on doit évoquer en premier lieu un cancer bronchique.

Compte tenu de votre diagnostic probable, Ies antécédents qui peuvent être liés à la pathologie actuelle
comportent :
A - Mineur de fer
B - Exposition à des poussières siliceuses
C - Tabagisme
D - Ethylisme
E - Ouvrier dans une minoterie
Bonne(s) réponse(s) : A B C

Outre le tabagisme, la sidérose professionnelle des mineurs de fer est un facteur de risque de cancer. D'autre part, tout
mineur est exposé aux poussières de silice : les poussières de silice sont actuellement considérées comme facteurs de risque
de cancer bien que ceci soit très controversé.

976
Exclusivement sur DOC - DZ : www.doc-dz.com NADJI 85
RESIDANAT EN POCHE TOME II
Cas Clinique en QCM

Les images basales gauches paraissent sans rapport avec l'affection qui a motivé la consultation. Quel
diagnostic pouvez-vous faire les concernant ?
A - Tuberculose pulmonaire
B - Pneumopathie virale
C - Sidérose
D - Lymphangite métastatique
E - Bronchectasies
Bonne(s) réponse(s) : C

La thésaurismose, liée au fer, s'appelle la sidérose. Elle explique l'image radiologique interstitielle.

A quoi rattachez-vous, dans cette liste, les anomalies cliniques mises en évidence ?
A - IVD
B - Cirrhose hépatique
C - OEdème de Quincke allergique
D - Syndrome cave supérieur débutant
E - Emphysème sous cutané
Bonne(s) réponse(s) : D

Ce patient présente un syndrome cave supérieure complet.


Ce syndrome cave n'est pas débutant mais établi.

En définitive, quel diagnostic allez-vous porter chez ce malade, au vu du bilan biologique qui montre une
natrémie à 115 mmol/l ; une chlorémie à 120 mmol/l ; une kaliémie à 4 mmol/l ; un hématocrite à 40 % ; une Hb
à 110 g/l.
A - Cancer bronchique épidermoïde
B - Cancer microcellulaire avec S. de Schwartz-Barner
C - Lymphome malin
D - Sarcoïdose pluriviscérale
E - Broncho-pneumopathie chronique obstructive
Bonne(s) réponse(s) : B

L'hyponatrémie à 115 oriente d'emblée vers une hypersécrétion inappropriée d'ADH (Schwartz-Bartter).
Ce syndrome paranéoplasique est particulièrement fréquent dans les carcinomes bronchiques anaplasiques à petites cellules.

A quoi sont dues les anomalies biologiques constatées ?


A- Hémodilution
B - Insuffisance respiratoire chronique.
C - Insuffisance rénale chronique
D - Insuffisance surrénalienne
E - Etat inflammatoire chronique
Bonne(s) réponse(s) : A

L'hémodilution est liée à l'antidiurèse secondaire à la sécrétion d'ADH.

Les céphalées du malade pourraient faire penser également à la possibilité de métastases cérébrales. Quel est
l'examen le plus fiable pour les mettre en évidence ?
A - EEG
B - Scintigraphie cérébrale
C - Scanner cérébral
D - Artériographie cérébrale
E - Radios du crâne
Bonne(s) réponse(s) : C

Les métastases cérébrales, même asymptomatiques, existent dans 20 % des carcinomes anaplasiques bronchiques. De ce
fait, le scanner cérébral est systématique quand le diagnostic est fait car les métastases cérébrales, si elles existent,
nécessitent une radiothérapie.

977
Exclusivement sur DOC - DZ : www.doc-dz.com NADJI 85
RESIDANAT EN POCHE TOME II
Cas Clinique en QCM
M. P., 58 ans. agriculteur, vient consulter pour une toux récidivante avec altération récente de l'état général.
Antécédents :
artérite des membres inférieurs, stade II. Bronchite chronique, sans dyspnée depuis 15 ans. Fumeur (40 paquets/année).
Alcool : 1/2 litre/jour. Asthme depuis l'enfance, avec 2 à 3 crises par an l'été.
Histoire de la maladie :
il y a 6 mois, épisode d'éruption cutanée des membres inférieurs, mal étiqueté (le diagnostic d'érythème noueux aurait été
évoqué). Asthénie depuis 2 mois et perte de poids de 6 kg associées à des arthralgies et à une dysphonie. Expectoration
purulente, striée de sang depuis 8 jours.
A l'examen :
température à 38°5 C. Toux, expectoration purulente, crachats hémoptoïques. Auscultation :
râles sibilants diffus des bases. Hippocratisme digital, articulations des doigts légèrement gonflées.
La radiographie pulmonaire montre :
opacité dense, à projection hilaire gauche, avec comblement de la fenêtre aortico-pulmonaire et atélectasie du lobe supérieur
gauche. L'ensemble de ce tableau vous évoque une pathologie maligne.

Parmi les éléments suivants. le(s)quel(s) constitue(nt) un(des) facteur(s) de risque de cancer bronchique ?
A - Tabagisme
B - Artérite des membres inférieurs
C - Asthme
D - Travail exposé à la poussière de foin
E - La prise de boissons alcoolisées
Bonne(s) réponse(s) : A

Seul le tabagisme est un facteur de risque de carcinome bronchique parmi les réponses proposées ici.

Chez ce malade, quel(s) signe(s) clinique(s) doi(ven)t particulièrement faire craindre une pathologie maligne ?
A - Erythème noueux
B - Hippocratisme digital
C - Crachats hémoptoïques
D - Dysphonie
E - Râles sibilants des bases chez un bronchitique chronique
Bonne(s) réponse(s) : C D

Les hémoptysies orientent vers une lésion endobronchique. La dysphonie évoque une paralysie récurentielle gauche,
d'origine néoplasique.

L'hippocratisme digital que vous avez constaté chez ce malade peut se rencontrer dans la (les) affection(s)
suivante(s) :
A - Cirrhose
B - Fibrose interstitielle diffuse
C - Bronchectasies
D - Asthme paroxystique ancien
E - Bronchite chronique
Bonne(s) réponse(s) : A B C E

Quatre sortes d'étiologie doivent être recherchées devant un hippocratisme digital :


- l'insuffisance respiratoire chronique hypoxique quelle que soit sa cause
- les hépatopathies chroniques
- les cardiopathies malformatives cyanogènes
- les suppurations chroniques en particulier respiratoires.

La fibroscopie bronchique montre une tumeur bourgeonnante obstruant la bronche lobaire supérieure gauche.
Parmi les propositions suivantes, quelle est la forme de cancer bronchique la plus probable chez ce malade ?
A - Cancer épidermoïde
B - Cancer anaplasique à petites cellules
C - Adénocarcinome
D - Cancer indifférencié à grandes cellules
E - Cancer bronchiolo-alvéolaire
Bonne(s) réponse(s) : A

Le tableau clinique associant hippocratisme digital, arthrite des mains, évoque une ostéoarthropathie hypertrophiante
pneumonique ou syndrome de Pierre Marie. Il s'agit d'un syndrome paranéoplasique accompagnant les cancers bronchiques
essentiellement épidermoïde.

978
Exclusivement sur DOC - DZ : www.doc-dz.com NADJI 85
RESIDANAT EN POCHE TOME II
Cas Clinique en QCM

Parmi les symptômes suivants, témoigne(nt) d'une extension importante du cancer :


A - Hémoptysies répétées
B - Dysphonie
C - Hippocratisme digital
D - Fièvre persistante malgré une antibiothérapie
E - Toux rebelle résistant aux antitussifs
Bonne(s) réponse(s) : B

La dysphonie, témoigne d'une extension médiastinale, et est considérée par la plupart des chirurgiens thoraciques comme
une contre-indication opératoire.

M. J., 65 ans, est hospitalisé pour une douleur thoracique gauche d'apparition brutale, avec dyspnée importante.
Histoire de la maladie :
il y a 3 jours, douleur thoracique gauche d'apparition brutale, avec dyspnée et angoisse. Pas de fièvre. Pas d'expectoration.
Antécédents :
Tabagisme : 25 paquets-année. Hypertension artérielle modérée, équilibrée à 13/8 avec Sectral® (bétabloqueur). Pas d'autres
antécédents notables.
Examen :
bon état général. Température : 37,2°C. Pression artérielle : 10/7. Fréquence cardiaque: 100/mn. Auscultation pulmonaire et
cardiaque : normale. Pas de signes cliniques d'insuffisance cardiaque droite. Examen des membres : normal.
Examens complémentaires :
radiographie pulmonaire: ascension de la coupole diaphragmatique gauche. ECG : tachycardie sinusale à 100, bloc incomplet
droit. Gaz du sang : Pa O2 = 57 mmHg (7,6 Kpa), Pa CO2 = 36 mmHg (4,8 Kpa), pH = 7,45.

Vous pensez à une embolie pulmonaire. Quel(s) autre(s) diagnostic(s) peut (peuvent) être envisagé(s) chez ce
malade ?
A - Oedème aigu du poumon chez un hypertendu
B - Décompensation respiratoire aiguë d'une bronchite chronique chez un fumeur
C - Cancer bronchique
D - Pneumonie à pneumocoques
E - Crise d'asthme
Bonne(s) réponse(s) :

- L'OAP dans le cadre d'une hypertension artérielle et l'asthme ne sont jamais cause directe de douleur.
- Le tableau clinique est trop aigu pour un cancer bronchopulmonaire.
- La pneumonie à pneumocoques aurait été fébrile.
- L'IRA d'un BPCO serait accompagnée d'une hypercapnie.

Dans l'embolie pulmonaire, l'électrocardiogramme peut présenter l'un ou plusieurs des caractères suivants :
lequel(lesquels) ?
A - Tracé rigoureusement normal
B - Aspect rSr' dans les précordiales droites, avec onde S profonde en D1 et VL
C - Axe de QRS dévié à gauche
D - Onde P bifide en D2, D3 et VF
E - Ondes T négatives en VI, V2, V3
Bonne(s) réponse(s) : A B

Les modifications ECG sont inconstantes et fugaces au cours des embolies pulmonaires. Un ECG.normal n'élimine pas le
diagnostic. Les signes rencontrés sont ceux d'un coeur pulmonaire aigu : axe droit, S1, Q3, bloc de branche droit.

En dehors de la surélévation d'une coupole diaphragmatique, la radiographie pulmonaire d'un malade atteint
d'une embolie pulmonaire récente peut montrer un ou plusieurs des aspects suivants : le(les)quel(s) ?
A - Epanchement pleural
B - Opacité ronde intra-parenchymateuse
C - Atélectasies en bandes
D - Hyperclarté localisée
E - Aspect normal
Bonne(s) réponse(s) : A B C D E

Toutes les propositions sont vraies. La radiographie pulmonaire normale n'élimine pas le diagnostic.

979
Exclusivement sur DOC - DZ : www.doc-dz.com NADJI 85
RESIDANAT EN POCHE TOME II
Cas Clinique en QCM

Chez ce malade, les résultats de la mesure des gaz du sang sont compatibles avec le diagnostic d'embolie
pulmonaire. De quelle(s) façon(s) peut-on expliquer les anomalies rencontrées ?
A - Effet shunt
B - Hypoventilation alvéolaire
C - Revascularisation systémique du foyer embolisé
D - Bronchoconstruction réflexe débordant le territoire embolisé
E - Augmentation de l'espace mort avec ventilation inefficace
Bonne(s) réponse(s) : A

L'hypoxie-hypocapnie témoigne d'un effet shunt par définition.

Quel(s) examen(s) pouvez-vous demander pour confirmer le diagnostic d'embolie pulmonaire ?


A - Fibroscopie bronchique
B - Angiographie pulmonaire
C - Artériographie bronchique
D - Phlébocavographie
E - Scintigraphie de ventilation/perfusion
Bonne(s) réponse(s) : B E

L'angiographie pulmonaire est l'examen de référence qu'il faut pratiquer lorsqu'on en a les moyens. La scintigraphie de
ventilation/perfusion (6 incidences) peut apporter des éléments diagnostiques s'il existe une ventilation conservée, une
perfusion abolie dans le territoire de la douleur. Mais c'est rarement le cas lorsqu'il existe une surélévation de la coupole.

Quel(s) signe(s) retrouve-t-on dans l'embolie pulmonaire ?


A - Signes cliniques de thrombose veineuse dans 80 % des cas
B - Tachycardie fréquente
C - Angoisse une fois sur deux
D - Dyspnée fréquente
E - Antécédents d'hypertension artérielle
Bonne(s) réponse(s) : B C D

Il ne faut s'attendre à trouver de signe clinique de thrombose veineuse que dans 50 % des cas d'embolie pulmonaire. C'est la
phlébographie des membres inférieurs qui est positive dans 80 % des cas.

Concernant ce malade, indiquez quelle(s) affirmation(s) est (sont) exacte(s) :


A - L'embolie est certainement peu importante car il n'y a pas d'aspect S1 Q3
B - La prise de Sectral® à certainement favorisé l'embolie pulmonaire
C - Il faut commencer d'urgence un traitement par les antivitamine K
D - Une oxygénothérapie peut être donnée sans risque de décompensation respiratoire
E - Un traitement diurétique permettra d'éviter les récidives ultérieures
Bonne(s) réponse(s) : D

L'oxygénothérapie fait partie des mesures thérapeutiques d'urgence associée à l'héparinothérapie dès que le diagnostic
d'embolie pulmonaire est suspecté.

Un homme de 50 ans présente une altération modérée de l'état général avec fièvre à 38,5° C, douleur basithoracique droite,
toux et expectoration muco-purulente. Il s'agit d'un tabagique (30 paquets-année).
L'examen clinique est pauvre : l'auscultation révèle quelques râles crépitants à la base droite. La radiographie pulmonaire de
face et de profil met en évidence une opacité sous-hilaire droite. On note par ailleurs un trouble de ventilation basal droit de
face et postéro-basal sur le cliché de profil. La VS. est à 90 à la première heure et il existe à l'hémogramme une leucocytose à
12000/mm3.

Vous allez immédiatement :


A - Faire un examen tomographique du thorax
B - Pratiquer une recherche de BK par tubage
C - Pratiquer des hémocultures
D - Programmer une fibroscopie
E - Administrer un traitement antibiotique non spécifique
Bonne(s) réponse(s) : C D E

Le traitement antibiotique type amoxicilline sera débuté après les hémocultures.

980
Exclusivement sur DOC - DZ : www.doc-dz.com NADJI 85
RESIDANAT EN POCHE TOME II
Cas Clinique en QCM

Secondairement, une fibroscopie révèle l'existence d'un bourgeon néoplasique qui obstrue totalement une
bronche. Quelle est cette bronche ?
A - Lobaire supérieure droite
B - Lobaire moyenne
C - Nelson droite
D - Tronc intermédiaire
E - Tronc de la pyramide basale
Bonne(s) réponse(s) : E

L'opacité radiologique correspond à un trouble de ventilation d'un ou plusieurs segments de la pyramide basale droite.

Si le résultat de l'examen anatomopathologique de la biopsie per-fibroscopie est négatif, quel(s) est (sont) le(s)
examen(s) qui peu(ven)t permettre le diagnostic de cancer bronchique ?
A - Cytologie de brossage bronchique
B - Biopsie transpariétale
C - Biopsie pré-scalénique
D - Dosage de l'antigène carcino-embryonnaire
E - Nouvelles biopsies per-fibroscopie
Bonne(s) réponse(s) : A E

La fibroscopie bronchique doit être répétée et devra comporter de nouvelles biopsies, un brossage à visée cytologique en
sachant que l'on porte exceptionnellement le diagnostic sur la seule cytologie.

Quel(s) examen(s) allez-vous demander dans le cadre du bilan général de ce malade ?


A - Echographie cardiaque
B - Scanner thoracique
C - Scintigraphie thyroïdienne
D - Echographie hépatique
E - Transit oesogastroduadénal
Bonne(s) réponse(s) : B D

Le scanner thoracique va préciser l'extension médiastinale et pariétale; l'échographie hépatique recherche des métastases.
Ces 2 examens font partie du bilan d'opérabilité.

Le bilan fonctionnel respiratoire étant satisfaisant, quelle(s) intervention(s) chirurgicale(s) peut-on proposer ?
A - Segmentectomie
B - Pneumonectomie droite
C - Lobectomie inférieure droite
D - Résection atypique
E - Bilobectomie inférieure et moyenne
Bonne(s) réponse(s) : C

La discussion sera nuancée à thorax ouvert : l'extension inattendue ou une anomalie constitutionnelle des scissures pourra
conduire à une bilobectomie par exemple.

Un homme de 75 ans, assureur en retraite, traité depuis 2 ans par oestroprogestatifs pour un cancer de la prostate, consulte
en raison de l'apparition progressive d'une dyspnée d'effort, puis de repos. S'y associent une fièvre à 38° C et une altération
de l'état général.
De ses antécédents, on retient uniquement une tuberculose pulmonaire gauche traitée en 1940 par pneumothorax, ayant
laissé pour séquelle une minime calcification du sommet.

Sur quel(s) élément(s) clinique(s) évoquez-vous chez ce patient l'existence d'un épanchement pleural liquidien
gauche abondant ?
A - Matité de l'hémithorax gauche
B - Souffle pleurétique gauche
C - Abolition des vibrations vocales à gauche
D - Frottement pleural à gauche
E - Déplacement à droite de la pointe du coeur
Bonne(s) réponse(s) : A B C E

Le syndrome pleural associe : une diminution de l'ampliation thoracique, matité, abolition des vibrations vocales et du
murmure vésiculaire. Le souffle pleurétique ne s'entend que quand l'épanchement est minime ou à la limite supérieure de cet
épanchement.
Le choc de pointe s'est effectivement déplacé sur la droite dans ce cas-là et non la pointe du coeur comme ceci est mal
exprimé à l'énoncé.

981
Exclusivement sur DOC - DZ : www.doc-dz.com NADJI 85
RESIDANAT EN POCHE TOME II
Cas Clinique en QCM

Chez ce patient, sur quel(s) signe(s) radiologique(s) retiendrez-vous un épanchement pleural liquidien abondant
droit ?
A - Attraction du médiastin à gauche
B - Refoulement du médiastin à droite
C - Hémithorax gauche opaque
D - Pincement intercostal gauche
E - Abaissement de la coupole diaphragmatique gauche
Bonne(s) réponse(s) :

Il existe manifestement une erreur dans l'énoncé :


- s'il s'agit d'un épanchement droit, aucune proposition n'est vraie
- s'il agit d'un épanchement gauche (ce qui semble probable), les réponses sont B, C, E.

Quelle(s) étiologie(s) évoquez-vous de prime abord :


A - Pleurésie métastatique
B - Pleurésie pancréatique
C - Pleurésie tuberculeuse
D - Pleurésie à pneumocoques
E - Mésothéliome malin
Bonne(s) réponse(s) : A C

Le contexte fait évoquer soit une rechute tuberculeuse soit une pleurésie métastatique.

Sur le liquide pleural, macroscopiquement citrin et clair, recueilli par ponction, quel(s) examen(s) ne vous
parai(ssen)t pas indispensable(s) ?
A - Dosage des protides
B - Recherche de cellules malignes
C - Recherche de BK
D - Dosage de l'amylase pleurale
E - Dosage de l'acide hyaluronique
Bonne(s) réponse(s) : D E

La réponse est assez mal formulée car il est habituel d'enseigner que l'amylopleurie et le dosage d'acide hyaluronique dans le
liquide pleural doivent être systématiquement prélevés devant un exsudat.

Quel(s) examen(s) est (sont) susceptible(s) de donner un diagnostic de certitude ?


A - Analyse des protéines du liquide
B - Recherche de B. K.
C - Cytologie pleurale
D - Biopsie pleurale à l'aiguille
E - Biopsie pleurale sous thoracoscopie
Bonne(s) réponse(s) : B D E

Seul un examen histologique permettra le diagnostic de certitude rapide (granulome épithéliogiganto-cellulaire dans la
tuberculose, prolifération maligne en cas de métastase). En cas de tuberculose, la recherche de BK est pratiquement toujours
négative au direct. Elle est positive dans 60 % des cas aux cultures alors que l'histologie est positive dans 100 % des cas si
les biopsies à l'aiguille sont répétées jusqu'à 3 fois.
Dans ce cas précis, chez un ancien tuberculeux avec séquelles de pneumothorax thérapeutique, il arrive que les pleurésies
tuberculeuses soient purulentes avec BK au direct. Elles témoignent alors d'une fistule bronchopleurale à partir de lésions
pulmonaires évolutives.

En cas d'étiologie maligne, quelle(s) conduite(s) thérapeutique(s) peut-on proposer pour cet apanchement
récidivant ?
A - Abstention
B - Corticothérapie
C - Castration chirurgicale ou chimique
D - Ponctions répétées (évacuatrices)
E - Symphyse pleurale (par cyclines ou talcage)
Bonne(s) réponse(s) : E

La pleurésie métastatique justifie une tentative de symphyse après drainage ou par talcage en pleuroscopie.
Les possibilité thérapeutiques générales d'un cancer prostatique métastasé à la plèvre après échec de l'hormothérapie sont
très discutées. Il faut noter une erreur d'énonçé dans le cas clinique au début en ce qui concerne l'hormonothérapie : il s'agit
d'oestrogènes et non d'oestroprogestatifs. Dans ce cas clinique, l'énonçé du cas et des questions sont globalement imprécis.

982
Exclusivement sur DOC - DZ : www.doc-dz.com NADJI 85
RESIDANAT EN POCHE TOME II
Cas Clinique en QCM
Une femme de 45 ans, ayant eu la coqueluche dans l'enfance, se plaint d'une expectoration matinale abondante et purulente
ainsi que d'une dyspnée d'effort grade III. A l'examen, on note la présence de râles bulleux consonnants des bases et un
hippocratisme digital.

Quel est votre diagnostic ?


A - Bronchite chronique
B - Asthme vieilli
C - Emphysème
D - Bronchectasies
E - Bronchites aiguës à répétition
Bonne(s) réponse(s) : D

C'est la définition (notion de coqueluche soulignée).

En faveur du diagnostic, vous retenez :


A - Antécédents de coqueluche
B - Dyspnée
C - Hippocratisme digital
D - Expectoration matinale
E - Râles consonnants
Bonne(s) réponse(s) : A C D E

La dyspnée est plutôt en faveur d'une insuffisance cardiaque.

Pour confirmer le diagnostic, il est utile de demander :


A - Radiographie pulmonaire
B - Fibroscopie bronchique
C - Tomodensitométrie
D - Bronchographie lipiodolée
E - pH métrie oesophagienne
Bonne(s) réponse(s) : A C

Seul examen fiable mais contre-indiqué du fait de l'insuffisance cardiaque.

Les aspects radiologiques compatibles avec le diagnostic de bronchectasies comportent :


A - Hyperclarté des bases
B - Images lobulées de deux bases
C - Lobe inférieur rétracté
D - Images polykystiques
E - Adénopathies hilaires
Bonne(s) réponse(s) : B D

Connaissance.

L'évolution de cette malade peut se compliquer de :


A - Hémoptysies
B - Abcédation
C - Réaction pleurale
D - Insuffisance respiratoire chronique
E - Pneumothorax
Bonne(s) réponse(s) : A B C D E

Tout peut se voir.

Les gaz du sang donnant chez elle les résultats suivants :


PaCO2 à 35 mmHg (4,7 kPa) et pH à 7,41 ; sur quel(s) signe(s) vous baserez-vous pour parler chez elle
d'insuffisance respiratoire chronique ?
A - Antécédents
B - Trouble ventilatoire mixte à prédominance obstructive non réversible sous béta-mimétique
C - PaCO2
D - Dyspnée d'effort grade III
E - Expectoration
Bonne(s) réponse(s) : A

Par définition l'IRC = toux et crachats plus de 3 mois par an, 2 années consécutives.
983
Exclusivement sur DOC - DZ : www.doc-dz.com NADJI 85
RESIDANAT EN POCHE TOME II
Cas Clinique en QCM

A quel(s) élément(s) thérapeutique(s) allez-vous avoir recours chez cette malade, sachant que les
bronchectasies sont bilatérales ?
A - Drainage bronchique déclive
B - Drainage d'une sinusite éventuelle
C - Antibiothérapie préventive systématique
D - Extractions de chicots dentaires
E - Chirurgie
Bonne(s) réponse(s) : B C D

Pour prévenir les complications.


A - Incongru.
E - Trop tard.

Un homme de 35 ans, manutentionnaire vient consulter pour baisse de l'état général et température à 38°C depuis 10 jours. Il
est marié, père de 2 enfants (6 et 4 ans). Il dit boire un litre de vin et fumer un paquet de cigarettes par jour. Il tousse et
crache. La radiographie pulmonaire montre une image excavée du segment apical du lobe inférieur droit. L'expectoration est
riche en bacilles alcoolo-acido-résistants à l'examen direct.

Quelle décision doit être prise ?


A - Hospitalisation immédiate
B - Traitement à domicile
C - Traitement sanatorial de 9 mois
D - Arrêt de travail de 8 jours
E - Arrêt de travail de 9 mois
Bonne(s) réponse(s) : A

Le sujet bacillaire doit être hospitalisé pour éviter la contamination de son entourage, en particulier de ses 2 enfants.

Quel(s) examen(s) devra-t-on demander d'emblée pour orienter la thérapeutique ?


A - Numération-formule
B - Antibiogramme des bacilles
C - Electrophorèse des protéines
D - Etude de la vision des couleurs
E - Bilan hépatique
Bonne(s) réponse(s) : B D E

Vision des couleurs et bilan hépatique permettent d'écarter d'emblée une contre-indication respectivement à l'éthambutol et à
l'INH® Ils serviront de base pour la surveillance.
L'antibiogramme du BK demandé systématiquement, permettra d'adapter secondairement les antituberculeux. Le terme de
"orienter" est flou et il serait avantageusement remplacé par le terme "adapter".

En l'absence de contre-indication précise, vers quel(s) antibiotique(s) se portera votre choix ?


A - Streptomycine
B - Rifampicine
C - Isoniazide
D - Ethambutol
E - Pyrazinamide
Bonne(s) réponse(s) : B C D E

Deux régimes thérapeutiques sont possibles :


- l) INH-ETB-RF-PZA : pendant 2 mois puis lNH-RF pendant les 4 mois suivants
- 2) INH-ETB-RF : 3 mois puis INH-RF les 6 mois suivants.

L'initiation de l'antibiothérapie nécessite de vérifier :


A - L'examen ophtalmologique
B - Les réflexes
C - Le bilan biologique hépatique
D - L'audiogramme
E - Le bilan biologique rénal
Bonne(s) réponse(s) : A B C

L'éthambutol expose au risque de névrite optique rétro-bulbaire, surtout chez l'éthylique. L'INH® expose au risque de
polynévrite et de cytolyse hépatique surtout chez les éthyliques dénutris.

984
Exclusivement sur DOC - DZ : www.doc-dz.com NADJI 85
RESIDANAT EN POCHE TOME II
Cas Clinique en QCM

En l'absence de test d'acétylation, quelle dose d'isoniazide doit-on donner chez ce patient ?
A - 1 mg/kg/j
B - 3 mg/kg/j
C - 5 mg/kg/j
D - 10 mg/kg/j
E - 15 mg/kg/j
Bonne(s) réponse(s) : C

5 mg/kg/j est la posologie classique de l'INH mais on s'achemine de plus en plus vers 3 mg/kg/j.

Par quel(s) examen(s) surveillerez-vous ce traitement ?


A - Etude des réflexes
B - Audiogramme
C - Numération formule
D - Examen ophtalmologique
E - Dosage des transaminases
Bonne(s) réponse(s) : A E

La surveillance neurologique doit être poursuivie pendant tout le traitement (recherche de paresthésie, abolition des réflexes).
Le dosage des transaminases doit être contrôlée surtout pendant les 2 premiers mois. Quant à l'examen ophtalmologique, il
doit être répété tous les 2 mois pendant la période du traitement par éthambutol.

Que ferez-vous vis à vis des enfants ?


A - Test tuberculinique au moment du diagnostic et non répété
B - Radiographie pulmonaire au moment du diagnostic et 2 mois après
C - Test tuberculinique au moment du diagnostic et 2 mois après
D - Radiographie au moment du diagnostic et non répété
E - Placement sanatorial
Bonne(s) réponse(s) : B C

Malgré la vaccination par le BCG que les enfants ont certainement reçu, il est indispensable de contrôler la radiographie
pulmonaire et l'intradermo réaction au moment du diagnostic et après un certain temps, car ils pouvaient être en incubation.

Un patient âgé de 68 ans, 70 kg, 170 cm, consulte pour dyspnée d'effort, toux, douleur thoracique, amaigrissement de 6 kg.
Ces signes se sont installés progressivement en deux mois. On note l'absence d'antécédents tuberculeux, un tabagisme
modéré, un diabète tardif traité et équilibré par un régime à 200 g d'hydrates de carbone et metformine (Glucophage retard® :
2 comprimés). Ce diabète est accompagné d'une artérite traitée par anticoagulants par voie orale : fluindione (Préviscan®) 1
comp. ; le taux de prothrombine récent est à 35 %. L'examen clinique, radiographique et la ponction pleurale faite après arrêt
des anticoagulants, affirment une pleurésie sérofibrineuse gauche abondante à type d'exsudat avec formule Iymphocytaire.
Les examens complémentaires affirmeront la nature tuberculeuse de l'épanchement.

Parmi ces signes physiques, vous devez retrouver à l'examen :


A - Matité avec exagération des vibrations vocales
B - Abolition du murmure vésiculaire
C - Souffle amphorique
D - Abolition des vibrations vocales
E - Immobilité de l'hémithorax gauche
Bonne(s) réponse(s) : B D E

Le syndrome pleural typique associe : immobilité de l'hémithorax inconstante, matité hydrique et franche, abolition des
vibrations vocales et du murmure vésiculaire, souffle pleurétique inconstant.

Vous allez demander chez ce patient pour instituer le traitement :


A - Dosage de créatinine sanguine
B - Contrôle ophtalmologique de la vision des couleurs
C - Numération formule sanguine
D - EEG
E - Transaminases
Bonne(s) réponse(s) : A B E

La créatininémie permet d'adapter la dose d'éthambutol et de rifampicine chez ce patient âgé diabétique et porteur d'artérite.

985
Exclusivement sur DOC - DZ : www.doc-dz.com NADJI 85
RESIDANAT EN POCHE TOME II
Cas Clinique en QCM

Les anticoagulants par voie orale doivent être pris en compte dans la mise en route du traitement
antituberculeux. Parmi les propositions suivantes, quelle est(sont) celle(s) qui vous semble(nt)
exacte(s) :
A - L'INH® interfère avec le métabolisme des antivitamines K
B - L'utilisation de la rifampicine oblige le plus souvent à augmenter les doses du traitement anticoagulant
C - Le traitement anticoagulant doit être interrompu
D - Il faut baisser la dose du traitement anticoagulant
E - Le taux de prothrombine sera contrôlé toutes les semaines pendant les 2 premiers mois du traitement
Bonne(s) réponse(s) : B E

La rifampicine entraîne une induction enzymatique responsable d'un catabolisme accrue des antivitamines K d'où la nécessité
:
- d'augmenter les doses
- de surveiller étroitement le TP pendant les 1ères semaines de traitement.

L'introduction de la pyrazinamide dans l'association initiale :


A - Permet de réduire la durée totale du traitement
B - A une action sur les BK intra-macrophagiques
C - Permet de réduire la dose d'INH®
D - Autorise à ne pas utiliser la rifampicine
E - Est susceptible de provoquer une toxicité rénale
Bonne(s) réponse(s) : A B

L'introduction de la pyrazinamide a permis de passer d'un traitement anti-tuberculeux de 9 mois à un traitement


antituberculeux de 6 mois équivalent, pour lequel il n'y a pas plus de rechute. En ce qui concerne la réponse B, la
pyrazinamide a une action bactéricide sur les BK intracellulaires donc intramacrophagique.

Le traitement médical de cette pleurésie tuberculeuse par une association initiale comprenant rifampicine,
isoniazide, éthambutol et pyrazinamide, puis rifampicine et isoniazide sera poursuivi :
A - Jusqu'à disparition des signes cliniques de pleurésie
B - Jusqu'à normalisation de la radiographie
C - 6 mois
D - 9 mois
E - 12 mois
Bonne(s) réponse(s) : C

La quadrithérapie antituberculeuse ne se conçoit que dans un schéma thérapeutique de 6 mois comportant : INH-ETB-PZA
pendant 2 mois puis INH-RF pendant les 4 mois suivants.

Le traitement de cette pleurésie tuberculeuse comporte également une kinésithérapie ; celle-ci :


A - Doit être prescrite dès la première ponction exploratrice
B - Comporte une rééducation diaphragmatique
C - A pour but un drainage des sécrétions bronchiques
D - A pour but de réduire la symphyse pleurale
E - Sera commencée après évacuation complète de l'épanchement
Bonne(s) réponse(s) : B D E

La kinésithérapie a pour but de réduire les séquelles pleurales qui peuvent être majeures en cas de pleurésie tuberculeuse.

986
Exclusivement sur DOC - DZ : www.doc-dz.com NADJI 85
RESIDANAT EN POCHE TOME II
Cas Clinique en QCM
Il s'agit d'un homme de 61 ans ayant fumé pendant 20 ans et qui présente depuis un mois une toux qui va en augmentant. Les
cultures de crachats révèlent une flore normale très importante quantitativement et 3 cytologies dans les crachats se révèlent
négatives. L'ensemble du bilan biologique, notamment les tests hépatiques, sont strictement normaux.
- La radio pulmonaIre montre une lésIon dans le lobe moyen droit.
- Le reste de l'examen clinique est sensiblement normal.
- Une bronchoscopie et une médiastinoscopie sont faites.
- Les biopsies de la lésion lombaire et des ganglions médiastinaux permettent de porter le diagnostic de cancer à petites
cellules du poumon.

Par quel examen doit-on compléter le bilan ?


A - Ponction biopsie hépatique
B - Ponction lombaire
C - Myélogramme, biopsie médullaire
D - Scintigraphie osseuse
E - Echocardiogramme
Bonne(s) réponse(s) : C

La biopsie ostéo-médullaire dans le carcinome anaplasique à petites cellules bronchiques permet de voir s'il existe des
métastases médullaires ou pas. Elle permet donc, dans l'évolution, de mieux comprendre les épisodes d'hypoplasie :
intolérance aux cytostatiques ou envahissement métastatique évolué pour lesquels la conduite à tenir ne sera pas la même.

Que recommandez-vous comme traitement ?


A - Résection de la tumeur
B - Association d'une chimiothérapie et d'une radiothérapie
C - Radiothérapie de la lésion lombaire
D - Polychimiothérapie
E - Pneumonectomie droite si la capacité pulmonaire ventilatoire du poumon gauche le permet
Bonne(s) réponse(s) : D

La polychimiothérapie dans le cancer anaplasique bronchique permet d'obtenir 60 % de réponses (partielles ou complètes)
dans les cancers localisés et 40% dans les formes disséminées.

Le pronostic de ce patient est essentiellement lié à :


A - L'âge
B - Le sexe
C - Le type histologique de la tumeur
D - L'existence d'un envahissement des ganglions sus-claviculaires droits
E - L'existence d'un épanchement pleural homolatéral
Bonne(s) réponse(s) : C

Le cancer anaplasique est le cancer le plus rapidement évolutif et donc Ie plus chimiosensible. Le pronostic dépend du type
histologique, I'existence d'adénopathies homolatérales ne permet pas de les classer dans les formes disséminées.

L'irradiation de principe de l'encéphale au cours des carcinomes à petites cellules :


A - Diminue le risque de récidives cérébrales
B - Nécessite une dose minimale de 30 Gy
C - Augmente la survie des malades
D - Evite la dissémination dans d'autres organes
E - Doit toujours être associée à une chimiothérapie
Bonne(s) réponse(s) : A B E

La radiothérapie de principe de l'encéphale au cours des cancers anaplasiques, diminue l'incidence des métastases
cérébrales, mais en diminue les possibilités de thérapeutique si elles apparaissent. Il faut également signaler des troubles des
fonctions intellectuelles à long terme chez les gens ayant eu une irradiation cérébrale.

987
Exclusivement sur DOC - DZ : www.doc-dz.com NADJI 85
RESIDANAT EN POCHE TOME II
Cas Clinique en QCM
Melle D., 24 ans, vous est adressée par le médecin du travail à la suite de la découverte d'une image thoracique anormale,
sur le cliché systématique. Histoire de la maladie :
il y a deux mois, épisode fébrile avec arthralgies des genoux, éruption au niveau de la face antérieure de jambes avec
quelques éléments nodulaires indurés dermo-épidermiques, ayant regressé spontanément en quinze jours.
Antécédents :
pas d'antécedents pathologiques particuliers : non fumeuse, pas de prise médicamenteuse habituelle. vaccinée par le BCG ;
IDR à 10 unité de tuberculine positive à 10 mm un an auparavant.
Examen clinique :
normal; pas d'adénopathies périphériques ; pas de splénomégalie ; auscultation pulmonaire normale ; pas de fièvre au
moment de l'examen.
Examens complémentaires :
VS : 15/25, ionogramme sanguin et numération formule sanguine : normaux ; IDR à 10 unités : négative radiographie
pulmonaire : adénopathies hilaires et médiastinales bilatérales et symétriques, bien
limitées ; syndrome interstitiel pulmonaire diffus, prédominant dans les régions axillaires.

Vous pensez à une sarcoïdose avec syndrome de Lofgren ; parmi les autres affections pouvant débuter par un
érythème noueux, vous retenez :
A- Primo-infection tuberculeuse
B - Infection streptococcique
C - Alvéolite allergique
D - Cancer bronchique anaplasique à petites cellules
E - Asthme allergique
Bonne(s) réponse(s) : A B

La primo-infection tuberculeuse et l'infection streptococcique sont, avec la sarcoïdose, les 3 étiologies principales des
érythèmes noueux.

Le tableau clinique et radiologique vous permet d'éliminer ces autres affections. Les éléments caractéristiques
entrant dans la définition du syndrome de Lِfgren comprennent :
A - Adénopathies hilaires bilatérales
B - Anergie tuberculinique
C - Hyperlymphocytose sanguine
D - Erythème noueux
E - Hyper alpha2-globulinémie
Bonne(s) réponse(s) : A D

Le syndrome de Lِfgren est défini par l'association d'adénopathies hilaires bilatérales, d'érythème noueux à des arthralgies,
voire de la fièvre.

Vous pouvez obtenir la preuve diagnostique de cette maladie par un ou plusieurs des examens suivants.
Le(s)quel(s) ?
A - Bronchofibroscopie avec biopsies trans-bronchiques
B - Cytodiagnostic de l'expectoration
C - Biopsie d'une glande salivaire accessoire
D - Biopsies sur les points d'éruption cutanée
E - Ponction-biopsie hépatique
Bonne(s) réponse(s) : A C D E

Le granulome epithélio-giganto-cellulaire sans caseum retrouvé par ces différents procédés, est indispensable au diagnostic.

L'activité de la maladie chez cette patiente peut être évaluée par un ou plusieurs des examens suivants ;
le(s)quel(s) ?
A - Vitesse de sédimentation
B - Lavage broncho-alvéolaire
C - Dosage de l'enzyme de conversion de l'angiotensine
D - Hémogramme
E - Scintigraphie au technetium
Bonne(s) réponse(s) : B C

Enzyme de conversion de l'angiotensine, Iymphocytose au lavage alvéolaire, fixation à la scintigraphie pulmonaire au gallium
sont des critères d'activités de la maladie et non des critères pronostiques de l'évolution.

988
Exclusivement sur DOC - DZ : www.doc-dz.com NADJI 85
RESIDANAT EN POCHE TOME II
Cas Clinique en QCM

Quelle est l'anomalie retrouvée au lavage broncho-alvéolaire, dans une sarcoldose active au début ?
A - Hyperlymphocytose
B - Polynucléose neutrophile
C - Hyperéosinophilie
D - Pourcentage de macrophages augmenté
E - Aucune anomalie significative
Bonne(s) réponse(s) : A

L'alvéolite Iymphocytaire caractérise la sarcoïdose débutante et peut être découverte même en l'absence de signes
radiologiques médiastinaux-pulmonaires.

Melle D., 24 ans, vous est adressée par le médecin du travail à la suite de la découverte d'une image thoracique anormale,
sur le cliché systématique. Histoire de la maladie :
il y a deux mois, épisode fébrile avec arthralgies des genoux, éruption au niveau de la face antérieure de jambes avec
quelques éléments nodulaires indurés dermo-épidermiques, ayant regressé spontanément en quinze jours.
Antécédents :
pas d'antécedents pathologiques particuliers : non fumeuse, pas de prise médicamenteuse habituelle. vaccinée par le BCG ;
IDR à 10 unité de tuberculine positive à 10 mm un an auparavant.
Examen clinique :
normal; pas d'adénopathies périphériques ; pas de splénomégalie ; auscultation pulmonaire normale ; pas de fièvre au
moment de l'examen.
Examens complémentaires :
VS : 15/25, ionogramme sanguin et numération formule sanguine : normaux ; IDR à 10 unités : négative radiographie
pulmonaire : adénopathies hilaires et médiastinales bilatérales et symétriques, bien
limitées ; syndrome interstitiel pulmonaire diffus, prédominant dans les régions axillaires.

En rapport avec la sarcoïdose, l'investigation biologique pourrait éventuellement mettre en évidence chez cette
patiente :
A - Présence de complexes immuns circulants
B - Hyperkaliémie
C - Hypercalcémie
D - Hypocholestérolémie
E - Hyperosmolarité plasmatique
Bonne(s) réponse(s) : A C

Les immuns complexes circulants peuvent être trouvés dans 30 à 60 % des sarcoïdoses à la phase active initiale,
I'hypercalcémie est secondaire à l'hypersensibilité à la 1-25-dihydrovitamine D. Il existerait, en effet, au niveau du granulome
sarcoïdosique, une conversion de 25 en 1-25 vitamine D3, métabolite actif.

Un marocain de 32 ans, en France depuis quelques jours, est adressé en consultation pour amaigrissement de 6 kg en 2 mois
et asthénie. Il n'a pas d'antécédent particulier, fume 1 paquet et demi par jour et ne boit pas d'alcool.
A l'interrogatoire le patient signale l'apparition de douleurs sous-orbitraires gauches intermittentes mais s'aggravant depuis 1
mois. Il dit être souvent enrhumé et gêné pour respirer ; il tousse et crache un expectoration mucopurulente de façon
quotidienne.
L'examen révèle une adénopathie spinale haute gauche de 2 x 3 cm de diamètre indolore, ferme, non fixée. Le reste de
l'examen clinique est normal.

Quels sont les deux diagnostics à évoquer devant ce tableau ?


A - Tuberculose pulmonaire évolutive
B - Tumeur gastrique
C - Cancer du cavum
D - Cancer primitif bronchique
E - Amyloïdose primitive
Bonne(s) réponse(s) : A C

En faveur de la tuberculose : le terrain, l'altération de l'état général, l'expectoration purulente, l'adénopathie cervicale.
En faveur d'un carcinome ORL, particulièrement du cavum : l'adénopathie cervicale postéro-supérieure et les douleurs sous-
orbitaires témoignant d'une obstruction sinusienne.

Parmi les éléments cliniques suivants quel(s) est(sont) celui(ceux) qui va(vont) guider le diagnostic ?
A - L'auscultation cervicale
B - L'examen ophtalmologique
C - La recherche d'un syndrome méningé
D - L'otoscopie
E - Les rhinoscopies antérieure et postérieure
Bonne(s) réponse(s) : D E

On recherchera par otoscopie, une otite séreuse par obstruction tubaire. La rhinoscopie antérieure et postérieure permettra de
rechercher une lésion ORL susceptible d'expliquer l'adénopathie spinale postérieure haute.

989
Exclusivement sur DOC - DZ : www.doc-dz.com NADJI 85
RESIDANAT EN POCHE TOME II
Cas Clinique en QCM

Quel(s) est(sont) l'(les) examen(s) paraclinique(s) suivant(s) qui n'est (ne sont) pas utile(s) au diagnostic ?
A - IDR tuberculinique
B - Scanner du rhinopharynx
C - Scintigraphie thyroïdienne
D - Impédancemétrie
E - Radiographies pulmonaires
Bonne(s) réponse(s) : C

L'impédancemétrie mettra aussi en évidence des signes d'obstruction tubaire.

Parmi les autres examens qui vous semblent indispensables pour étayer diagnostic, vous retenez :
A - Panendoscopie + examen du cavum
B - Exploration fonctionnelle respiratoire
C - Cervicotomie exploratrice avec examen histologique extemporané du ganglion
D - Biopsies systématiques du cavum
E - EEG
Bonne(s) réponse(s) : A C

Cette question paraît litigieuse. En effet, que signifient biopsies systématiques.


On connaît l'aspect macroscopiquement trompeur de la muqueuse du cavum, en particulier quand il existe une infection.

Cet homme de 62 ans est hospitalisé en urgence pour des crachats de sang rouge survenus 2 heures auparavant. Il présente
dans ses antécédents : une tuberculose cavitaire des deux sommets 13 ans auparavant, ayant été traitée durant un an 1/2
(traitement dont il a oublié les modalités). Il s'agit de plus d'un gros fumeur invétéré ayant fumé 50 paquets/année.
A la radiographie vous notez des lésions importantes des deux sommets avec calcifications, rétraction et attraction des hiles
vers le haut, images bulleuses,

Parmi les étiologies proposées pour ces hémoptysies, vous retenez dans le cas particulier :
A - Récidive de tuberculose
B - Cancer bronchique
C - Aspergillome intracavitaire
D - Fibrose pulmonaire
E - Pneumonie franche lobaire aiguë
Bonne(s) réponse(s) : A B C

L'aspergillome intracavitaire peut être évoqué même en l'absence d'images en grelot évocatrices. La fibrose et la pneumonie
ne donnent jamais d'hémoptysie à elles seules.

Comme les lésions sont bilatérales et qu'on ne sait quel est le côté qui saigne, vous pouvez prescrire pour
localiser l'hémoptysie :
A - Angiopneumographie
B - Scintigraphie pulmonaire
C - Fibroscopie bronchique
D - Artériographie bronchique
E - Bronchographie successivement bilatérale
Bonne(s) réponse(s) :

QUESTION ANNULEE

Les hémoptysies ont repris, devenant alarmantes, avec risque asphyxique et anémique. Dans ce cas il est
logique de prescrire en grande urgence :
A - Oxygénothérapie à la sonde nasale
B - Substances vasopressives
C - Artériographie bronchique avec embolisation en urgence
D - Tentative d'obturation bronchique provisoire avec une sonde de Fogarty sous fibroscopie
E - Perfusion de corticoïdes
Bonne(s) réponse(s) : A B C

Devant des hémoptysies abondantes, le risque est plus l'asphyxie par noyade que le choc hémorragique. Le traitement
nécessitera :
- la position latérale de sécurité du côté du saignement s'il est connu
- une oxygénothérapie
- l'administration de substances vasopressives
- l'embolisation s'impose d'urgence si les drogue vaso-actives sont inefficaces. L'embolisation sera indiquée plus à distance si
les drogues vaso-actives ont été efficaces afin d'éviter la récidive hémorragique. L'embolisation sera d'autant plus efficace que
les substances vasopressives ont pu être arrêtées plusieurs heures avant l'artériographie.

990
Exclusivement sur DOC - DZ : www.doc-dz.com NADJI 85
RESIDANAT EN POCHE TOME II
Cas Clinique en QCM

Un diagnostic d'aspergillome pourrait être établi sur :


A - Une opacité excavée avec image en grelot
B - La présence d'Aspergillus fumigatus au prélèvement de gorge
C - Des IgE spécifiques fortement positives pour les extraits d'aspergillus
D - La présence de précipitines antiaspergillaires
E - La mise en évidence de la truffe aspergillaire lors de la fibroscopie bronchique
Bonne(s) réponse(s) : A D

La présence d'Aspergillus au prélèvement de gorge n'a aucune valeur diagnostique. Les IgE spécifiques anti-aspergillaires
augmentent uniquement dans l'aspergillose bronchopulmonaire allergique. Enfin la truffe aspergillaire ne se voit qu'en
peropératoire dans une cavité parenchymateuse et non dans les bronches.

Toutes les recherches étiologiques sont restées négatives. Bien que le patient soit ambulatoire et qu'il ne
présente pas de phlébite clinique, vous cherchez à éliminer une éventuelle embolie pulmonaire. Quel(s)
examen(s) peu(ven)t vous aider ?
A - Scintigraphie pulmonaire
B - Angiopneumographie
C - Pléthysmographie des membres inférieurs
D - Phlébographie des membres inférieurs
E - Artériographie bronchique
Bonne(s) réponse(s) : B

Seule l'angiographie peut être contributive chez ce malade qui présente des séquelles des deux sommets et qui aura
forcément une scintigraphie pulmonaire de perfusion anormale.

Un sujet de 35 ans, splénectomisé il y a 5 ans pour rupture traumatique de la rate, consulte pour des signes respiratoires
apparus depuis 3 jours. La fièvre est à 38°6 et le malade se plaint d'une toux fréquente ramenant une expectoration
muqueuse, L'examen clinique pulmonaire met en évidence quelques râles crépitants aux 2 bases. Un cliché du thorax de face
révèle la présence d'un aspect réticulaire avec de petits nodules disséminés aux 2 bases et une réaction pleurale modérée à
droite. L'hémogramme montre une leucocytose à 16.000/mm3. Les germes bactériens le plus souvent responsables de
broncho-pneumopathies aiguës étant Streptococcus pneumoniae et Mycoplasma pneumoniae, vous discutez le rôle de ces
germes dans la genèse du tableau de ce malade.

En faveur d'une infection par Streptococcus pneumoniae, vous retenez :


A - Absence de point de côté
B - Leucocytose à 16.000/mm3
C - Expectoration muqueuse
D - Antécédents de splénectomie
E - La réaction pleurale
Bonne(s) réponse(s) : B D E

La splénectomie favorise les infections pneumococciques graves et justifie, sur certains terrains (par exemple lymphome), une
prévention par Oracilline® systématique.

En faveur d'une infection par Mycoplasma pneumoniae, vous retenez :


A - Le début progressif
B - La température à 38°6
C - La pauvreté de l'examen clinique
D - La leucocytose à 16.000/mm3
E - La réaction pleurale
Bonne(s) réponse(s) : A B C

Le mycoplasme a un début plus progressif que le pneumocoque. Il s'accompagne rarement de douleurs thoraciques ou de
fièvre à 40°. Il s'accompagne plus rarement de pleurésie.

Les signes radiologiques permettent d'évoquer :


A - Une pneumonie alvéolaire
B - Une pneumonie atypique
C - Une pneumonie lobulaire
D - Une bronchopneumonie
E - Une pneumonie interstitielle
Bonne(s) réponse(s) : B D E

Ces trois dénominations sont possibles mais font appel à des définitions différentes :
- pneumonie atypique signifie à germe apparenté ou viral
- bronchopneumonie signifie l'association d'une atteinte anatomiquement bronchique et parenchymateuse
- pneumopathie interstitielle a une définition essentiellement radiologique.

991
Exclusivement sur DOC - DZ : www.doc-dz.com NADJI 85
RESIDANAT EN POCHE TOME II
Cas Clinique en QCM

Parmi les germes suivants, quels sont ceux qui provoquent un tableau pulmonaire comparable à celui de
l'infection par Mycoplasma pneumonie ?
A - Haemophilus influenzae
B - Staphylococcus aureus
C - Chlamydia psittaci
D - Klebsiella pneumoniae
E - Coxiella burnetti
Bonne(s) réponse(s) : C E

Mycoplasme, Chlamydia et Rickettsi sont 3 germes habituellement responsables de pneumopathie de l'adulte commun à
début progressif, unie ou bilatérale, d'évolution le plus souvent favorable.

Dans l'hypothèse où Streptococcus pneumoniae serait la cause de l'infection, il serait utile de demander :
A - Bactériologie des crachats
B - Coloration de Gram sur l'expectoration
C - Hémoculture
D - Ponction transtrachéale
E - Contre-immuno-électrophorèse du sérum
Bonne(s) réponse(s) :

QUESTION ANNULEE

Dans l'hypothèse où Mycoplasma serait la cause de l'infection, il serait utile de demander :


A - Bactériologie des crachats
B - Agglutinines froides
C - Hémoculture
D - Ponction transtrachéale
E - Sérologie spécifique
Bonne(s) réponse(s) : B E

Le mycoplasme est un germe fragile dont la culture, quelque soit le prélèvement, n'est pas réalisée de routine. Le diagnostic
de certitude est obtenu par séroconversion sur 2 prélèvements sanguins faits à 10 jours d'intervalle. La présence
d'agglutinines froides est très évocatrice du diagnostic quand elle est positive, ce qui est inconstant.

Un homme de 40 ans consulte pour une bronchite trainante depuis un mois. L'expectoration reste purulente malgré les divers
sirops administrés. Le malade qui mesure 178 cm et pèse 70 kg a maigri de 5 kg.
Antécédents : tabagisme supérieur à 20 paquets-année, a été monteur en chauffage central durant 20 ans.
Examen clinique à l'entrée : râles de la base droite, pas d'hépatomégalie ni d'adénopathies.
Radiographie thoracique : opacité triangulaire postérobasale droite.
Biologie : GR = 4000 x 10 exposant 3 mm3, GB = 10 x 10 exposant 3 mm3, VS = 60/80 mm.

Que faites-vous en priorité lors de cette première consultation ?


A - Une intradermoréaction à la tuberculine
B - Une recherche de BK dans les crachats
C - Une analyse bactériologique standard des crachats
D - Un traitement antibiotique
E - Une demande de scanner thoracique
Bonne(s) réponse(s) : D

L'association expectoration purulente, syndrome inflammatoire avec hyperleucocytose et opacité pulmonaire incite à penser
qu'il s'agit d'une infection, même s'il existe une pathologie néoplasique sous-jacente. Il faut donc assurer une désinfection par
antibiothérapie et un drainage bronchique.

Le malade est revu 3 jours plus tard. Il n'est plus fébrile, l'expectoration a disparu mais non la toux. Quelle est
votre attitude ?
A - Vous considérez le malade guéri
B - Vous refaites une radiographie de face du thorax
C - Vous demandez des tomographie du médiastin
D - Vous demandez une échographie thoracique
E - Vous continuez le traitement antibiotique
Bonne(s) réponse(s) : B E

Les nouvelles radiographies permettent de savoir si l'opacité persiste, bien qu'à 3 jours du début du traitement il soit un peu
prématuré de réaliser de nouvelles radiographies. L'antibiothérapie doit être poursuivie 8 à 10 jours en tout.

992
Exclusivement sur DOC - DZ : www.doc-dz.com NADJI 85
RESIDANAT EN POCHE TOME II
Cas Clinique en QCM

En fait le fébricule persiste après 10 jours d'antibiothérapie ainsi que l'expectoration mucopurulente. Une
fibroscopie est demandée, elle révèle un bourgeon hémorragique obstruant totalement une bronche. De quelle
bronche s'agit-il le plus probablement ?
A - Bronche lobaire supérieure
B - Bronche lobaire moyenne
C - Tronc intermédiaire
D - Tronc de la pyramide basale droite
E - Bronche apicale du lobe inférieur droit
Bonne(s) réponse(s) : D

L'opacité siège en effet dans le territoire de la pyramide basale droite.

L'examen anatomopathologique révélera qu'il s'agit d'un cancer épidermoïde. Vous demanderez alors :
A - Un lavement baryté
B - Une échographie cardiaque
C - Une échographie hépatique
D - Une urographie intraveineuse
E - Une fibroscopie oesophagienne
Bonne(s) réponse(s) : C

La lésion observée est un carcinome épidermoïde qui doit être considéré comme un primitif bronchique dans ce contexte.
Dans le cadre du bilan d'extension, une échographie hépatique est utile à la recherche de localisation métastatique.

Les épreuves fonctionnelles respiratoires montrent un VEMS à 800 ml. Quelle exérèse chirurgicale proposez-
vous a priori chez ce patient ?
A - Une résection atypique
B - Une segmentectomie
C - Une lobectomie
D - Une pneumonectomie
E - Toute intervention est contre-indiquée
Bonne(s) réponse(s) : E

Un VEMS de base inférieur à 1 litre contre-indique toute intervention du fait de l'insuffisance respiratoire qui en résulterait en
per, postopératoire immédiat et tardif.

Un homme de 55 ans, mineur de charbon pendant 30 ans, fumeur habituel, a quitté le service depuis 5 ans. A ce moment, le
cliché radiologique révélait une silicose de type 3 m, avec bronchopathie chronique obstructive justifiant une I.P.P. de 15 %.
Chaque année, un nouveau cliché radiologique a été pratiqué, l'image s'avérant stable jusqu'au dernier examen pratiqué, qui
est à l'origine de la consultation : on note en effet l'apparition d'une opacité lobaire supérieure droite, aux limites bien
dessinées, d'un diamètre d'environ quatre centimètres sur un fond pneumoconiotique inchangé. L'examen clinique de
l'appareil respiratoire est sans particularité, mais on note un hippocratisme digital.
L'examen scannographique révèle que cette formation est située dans le segment apical du lobe supérieur droit, qu'elle est en
voie de nécrose avec petite perte de substance centrale. On note la présence de petites adénopathies calcifiées au niveau du
hile droit.
La fibroscopie endobronchique ne montre pas d'anomalies notamment au niveau des branches segmentaires de la lobaire
supérieure droite, l'apicale apparaissant cependant refoulée vers l'avant. L'examen cytologique des sécrétions provenant de
cette bronche révèle la présence de cellules nécrotiques et de quelques petits placards cellulaires évoquant l'existence d'un
cancer de type épidermoïde.
Le bilan fonctionnel objective une amputation de 20 % de la capacité vitale et de 40 % du VEMS, réversible en partie sous
salbutamol, la PaO2 est à 65 mmHg. le PaC02 à 36 pour un pH à 7,42. L'examen clinique général est sans particularité en
dehors d'une obésité (90 kg pour 1 m 67). le bilan biologique est normal.

Des pseudotumeurs pneumoconiotiques auraient pu se discuter chez ce malade. Généralement ces


pseudotumeurs :
A - Sont habituellement bilatérales
B - Peuvent se développer après la fin de l'exposition au risque
C - Peuvent se nécroser en dehors de complication tuberculeuse
D - Peuvent se compliquer de tuberculose
E - Peuvent se fistuliser dans la plèvre
Bonne(s) réponse(s) : B C D

La réponse est litigieuse. En effet, il existe des pneumothorax par traction dans les silicoses surtout avec masse
pseudotumorale. Le terme "fistuliser" peut prêter à confusion.

993
Exclusivement sur DOC - DZ : www.doc-dz.com NADJI 85
RESIDANAT EN POCHE TOME II
Cas Clinique en QCM

En dehors du cancer bronchopulmonaire, l'hippocratisme digital peut évoquer :


A - Le mésothéliome pleural
B - Les dilatations bronchiques étendues
C - La rectocolite hémorragique
D - La fibrose interstitielle diffuse
E - La pneumoconiose des mineurs de charbon
Bonne(s) réponse(s) : A B C D E

Toutes ces affections peuvent s'accompagner d'un hippocratisme digital mais avec une variable. La survenue d'un
hippocratisme récent, alors que la maladie chronique évolue depuis longtemps, doit faire craindre un cancer bronchique
surajouté.

La survenue d'un cancer bronchopulmonaire est considérée comme une complication ouvrant droit à réparation
médico-légale en cours de :
A - Silicose
B - Asbestose
C - Talcose
D - Sidéro-silicose des mineurs de fer
E - Pneumoconiose des mineurs de charbon
Bonne(s) réponse(s) : B D

Ce sont les deux seules maladies citées qui peuvent bénéficier d'une réparation médico-légale si elles se compliquent de
carcinome bronchique primitif avec l'exposition au chrome.

Quelle sera votre orientation thérapeutique chez ce patient ?


A - Chimiothérapie seule
B - Radiothérapie seule
C - Radiothérapie suivie de chimiothérapie
D - Exérèse lobaire supérieure droite
E - Abstention thérapeutique
Bonne(s) réponse(s) : D

L'exérèse lobaire supérieure droite n'est pas contre-indiquée ni par le bilan d'extension locorégionale, ni par le bilan
fonctionnel respiratoire. Elle permettra :
- le diagnostic histologique indispensable malgré la cytologie évocatrice
- la guérison, s'il se confirme que l'exérèse est possible et qu'il n'y a pas d'adénopathie envahie.
Cependant un complément de bilan d'extension carcinologique s'impose afin de ne pas opérer un cancer déjà métastasé.

Parmi les manifestations pathologiques suivantes, quelle(s) est(sont) celle(s) qui ouvre(nt) droit à réparation
chez le mineur de charbon pneumoconiotique ?
A - Pneumothorax spontané
B - Cancer bronchique
C - Mésothéliome pleural
D - Infection à mycobactéries atypiques
E - Aspergillose intracavitaire
Bonne(s) réponse(s) : A D E

Sans commentaire.

994
Exclusivement sur DOC - DZ : www.doc-dz.com NADJI 85
RESIDANAT EN POCHE TOME II
Cas Clinique en QCM
Un homme de 58 ans, fumeur invétéré souffrant de bronchite chronique depuis plus de 10 ans, est admis en milieu hospitalier
à l'occasion d'une surinfection bronchique avec signes de décompensation respiratoire débutante. A son entrée, le malade est
conscient, bien orienté. Il signale présenter depuis trois jours une expectoration purulente, d'abord facile à expectorer,
devenue moins abondante au cours des 24 dernières heures.
A l'examen, il s'agit d'un sujet robuste, présentant une érythrocyanose de la face, polypnéique. L'auscultation révèle des râles
ronflants dans les deux champs pulmonaires, le coeur est rapide mais régulier, sans anomalie à l'auscultation, le pouls est
bien frappé, la TA à 17/10. Il n'existe pas d'oedème des membres inférieurs, pas d'hépatomégalie ni de reflux hépatojugulaire.
Le cliché radiologique objective une distension des artères pulmonaires et de ses branches, sans anomalie parenchymateuse.
Un examen des gaz du sang est pratiquée : la PaO2 est à 55 mmHg (7,3 Kpa), la PaCO2 à 52 mmHg (7 Kpa), le pH du sang
artériel à 7,32.

Quelle(s) conduite(s) pratique(s) tenir devant un tel malade ?


A - Mise en oeuvre d'un traitement par amoxycilline
B - Mise en oeuvre d'un traitement par un aminoside
C - Intubation immédiate et mise sous assistance ventilatoire
D - Oxygénothérapie au débit de 5 l/minute
E - Kinésithérapie respiratoire
Bonne(s) réponse(s) : A E

Le diagnostic de surinfection impose antibiotique et kinésithérapie. Le meilleur antibiotique sur ce terrain est représenté par
les bêta-lactamines car les germes le plus souvent en cause sont Hémophilus Influenzae et Pneumocoque. La kinésithérapie
est également indispensable afin d'assurer un désencombrement chez ce malade qui ne peut cracher seul. L'état du sujet
n'indique pas une ventilation assistée qui pourra certainement être évitée. Quant à l'oxygénothérapie, elle ne doit être
prescrite qu'à faible débit (< 1 litre/mn) en cas d'hypercapnie).

Une apparition de l'hypercapnie peut se traduire chez ce patient par :


A - Sueurs profuses
B - Cyanose des extrémités
C - Lividité sur les membres inférieurs avec peau froide
D - Effondrement tensionnel
E - Flapping tremor
Bonne(s) réponse(s) : A E

Ce sont deux signes d'hypercapnie qui s'associent en cas d'aggravation à des céphalées puis à une encéphalopathie avec
troubles du comportement voire coma.

Quelles sont les trois espèces de germes le plus souvent en cause dans les poussées infectieuses chez un tel
bronchitique chronique ?
A - Klebsiella pneumoniae
B - Hémophilus pertussis
C - Streptococcus pneumoniae
D - Mycoplasma pneumoniae
E - Hémophilus influenzae
Bonne(s) réponse(s) : C E

La fréquence de ces deux germes justifie une antibiothérapie de première intention par §-lactamine.

Un trouble ventilatoire obstructif pur se traduira chez ce patient par :


A - Diminution du volume expiratoire maximum seconde
B - Augmentation de la résistance des voies aériennes
C - Rapport de Tiffeneau supérieur à 70
D - Diminution du volume résiduel
E - Diminution de la capacité de diffusion du CO
Bonne(s) réponse(s) : A B C

La question paraît litigieuse car tout trouble obstructif peut s'accompagner d'une altération de la diffusion ; par contre, si on
s'intéresse à la KCO : DLCO /VA (VA = volume alvéolaire réellement ventilé), il n'y a pas de trouble de diffusion.

995
Exclusivement sur DOC - DZ : www.doc-dz.com NADJI 85
RESIDANAT EN POCHE TOME II
Cas Clinique en QCM
Monsieur V. Paul, 75 ans, est hospitalisé pour fièvre et troubles du comportement récemment majorés. C'est un ouvrier du
bâtiment retraité qui semble vivre assez misérablement. Il se plaint de maux de tête, de sueurs nocturnes, il aurait beaucoup
maigri. L'examen des différents appareils relève peu de signes, sauf des râles dans les deux poumons. L'examen radiologique
montre une miliaire diffuse des deux champs pulmonaires.

Le diagnostic de tuberculose pulmonaire étant fortement évoqué, sa confirmation sera apportée par la ou les
demande(s) suivante(s) :
A - Recherche de B.K. dans l'expectoration
B - Numération globulaire et formule sanguine
C - Tubages gastriques si la recherche est négative sur l'expectoration
D - Hémoculture
E - Tubages gastriques demandés d'emblée
Bonne(s) réponse(s) : A B D

La miliaire tuberculeuse s'accompagne rarement d'expectoration mais celle-ci doit-être analysée si elle existe.
La miliaire donne exceptionnellement des BAAR positifs dans l'expectoration ou les tubages, il faudra attendre les cultures
dans la plupart des cas.
Des hémocultures pour mycobactéries ont été récemment mises au point ; elles nécessitent des flacons de milieux spéciaux
qui ne sont pas disponibles partout.
Elles nécessitent également un long délai de culture. Elles trouvent leur meilleure indication dans les miliaires tuberculeuses
puisque celles-ci paraissent relever d'un mécanisme hématogène.
Dans cette QCM on regrette que la ponction lombaire ne soit pas évoquée. Elle apporterait en effet des arguments immédiats
en faveur de la miliaire tuberculeuse si elle montrait une méningite lymphocytaire à liquide clair avec hypoglycorachie.

La culture du B.K. à partir des crachats se fait :


A - Sur milieu sélectifs ne permettant que la culture du B.K.
B - Sur gélose au sang
C - Avec des crachats décontaminés par action de la soude, ensemencés ensuite sur milieux appropriés
D - Avec des crachats décontaminés par le formol
E - Dans aucune des conditions ci-dessus
Bonne(s) réponse(s) : A

Le milieu de Lِwenstein est spécifique des mycobactéries (en particulier BK mais aussi mycobactéries atypiques). Donc la
question prête à confusion.

La culture est positive à Mycobacterium tuberculosis. Une telle positivité est obtenue en :
A - 24 heures
B - 48 heures
C - Une semaine
D - De 2 à 4 semaines
E - Les cultures de B.K., comme celles du bacille de la lèpre, sont toujours négatives
Bonne(s) réponse(s) : D

Le temps de multiplication des BK est long (24 heures environ), ce qui explique l'apparition si lente des colonies en cultures.

Parmi les bactéries suivantes laquelle(lesquelles) peu(ven)t être responsable(s) de la miliaire humaine ?
A - Mycobacterium leprae
B - Mycobacterium kansasii
C - Mycobacterium chelonei
D - Mycobacterium africanum
E - Mycobacterium tuberculosis
Bonne(s) réponse(s) : E

Cette question est également difficile à interpréter : il faudrait certainement dans l'énoncé préciser de quel terrain il s'agit. En
effet, les mycobactéries atypiques sur terrain non immunodéprimé sont peu virulentes et ne donnent pas de miliaire. Mais bien
sûr quelques cas exceptionnels ont été publiés, essentiellement chez l'immunodéprimé ou chez les enfants très dénutris.

Parmi les propositions suivantes concernant le BCG, laquelle(lesquelles) est(sont) exacte(s) ?


A - A été obtenu à partir d'une souche de Mycobacterium bovis
B - Est un bacille qui a perdu son acido-alcoolo-résistance
C - Est une souche vaccinale inactivée par le formol
D - Est utilisé uniquement par voie buccale
E - Est une souche non létale pour le cobaye
Bonne(s) réponse(s) : A C

Sans commentaire.

996
Exclusivement sur DOC - DZ : www.doc-dz.com NADJI 85
RESIDANAT EN POCHE TOME II
Cas Clinique en QCM
Une jeune femme d'origine africaine, en France depuis seulement quelques mois, non vaccinée par le BCG, est hospitalisée
pour fièvre, amaigrissement, toux et expectoration purulente.
La radiographie pulmonaire met en évidence une opacité excavée à parois épaisses, au sein d'un infiltrat nodulaire du lobe
supérieur droit.
La V.S. est à 50 mm à la première heure.

Quel(s) diagnostic(s) peut-on envisager ?


A - Pneumonie à pneumocoque
B - Pneumonie virale
C - Abcès du poumon
D - Embolie pulmonaire
E - Tuberculose pulmonaire
Bonne(s) réponse(s) : E

Le diagnostic de tuberculose est seul très vraisemblable devant :


- l'infiltrat nodulaire et excavé
- la localisation dans le lobe supérieur.
L'existence de nodules adjacents et l'absence de niveau hydro-aérique sont des arguments contre un abcès.

Quel examen vous paraît déterminant pour conforter votre diagnostic ?


A - Tomographies
B - Tests tuberculiniques cutanés
C - Recherche de BK dans l'expectoration ou par tubage
D - Hémogramme
E - Culot urinaire
Bonne(s) réponse(s) : C

Le diagnostic formel sera apporté par la présence de BAAR (bacille acido-alcoolo-résistants) dans l'expectoration ou les
tubages. Ces examens devront être répétés au moins 3 fois.

Quel intérêt y a-t-il à associer la pyrazinamide au traitement antituberculeux triple classique (isoniazide,
rifampicine, éthambutol) ?
A - Amélioration de la diffusion des antituberculeux
B - Régression plus rapide des lésions
C - Possibilité de réduire le traitement à 6 mois
D - Stérilisation plus rapide de l'expectoration
E - Diminution des effets secondaires du traitement
Bonne(s) réponse(s) : B C

Le pyrazinamide a une activité bactéricide à pH acide. Il n'agit donc que sur les BK intracellulaires (macrophages) à
croissance lente. Ceci permet la régression plus rapide des lésions et la réduction du traitement à 6 mois mais il ne permet
pas de stériliser plus vite l'expectoration, car il n'agit pas sur les BK extracellulaires.

L'(es) effet(s) secondaire(s) possible(s) de l'isoniazide à rechercher chez cette patiente est(sont) :
A - Trouble de la vision et des couleurs
B - Hépatite cytotoxique
C - Surdité
D - Polynévrite sensitive motrice
E - Néphropathie tubulaire
Bonne(s) réponse(s) : B D

Les métabolites de l'INH ont une cytotoxicité hépatique et neurologique. Cette toxicité se majore en cas d'hépatopathie,
d'intoxication éthylique et de dénutrition.

Que faut-il redouter, chez cette patiente qui utilise un oestroprogestatif ?


A - La survenue d'une thrombophlébite
B - Une stérilité
C - Une grossesse
D - Une toxicité hépatique accrue de la rifampicine
E - Une dysménorrhée
Bonne(s) réponse(s) : C

La rifampicine est responsable d'une induction enzymatique qui permet un catabolisme plus rapide des oestroprogestatifs.
Ces derniers risquent donc d'être inefficaces en association avec le traitement anti-tuberculeux.

997
Exclusivement sur DOC - DZ : www.doc-dz.com NADJI 85
RESIDANAT EN POCHE TOME II
Cas Clinique en QCM
Un homme de 45 ans, employé de bureau, fumeur (40 paquets-année), consulte pour une dyspnée d'effort d'évolution
progressive, évoluant depuis plusieurs années.
Histoire de la maladie : Dyspnée d'effort progressivement croissante évoluant depuis au moins 5 à 6 ans. Cette dyspnée
survient actuellement à la marche rapide sur terrain plat. Il n'y a jamais eu d'expectoration.
Antécédents : aucun antécédent pathologique notable.
Examen : distension thoracique à l'inspection. Hypersonorité diffuse à la palpation.
Auscultation : diminution du murmure vésiculaire. Pas de râles. Vous faites pratiquer une radiographie pulmonaire et des
explorations fonctionnelles respiratoires.
L'ensemble des examens clinique, radiologique et fonctionnel vous oriente vers le diagnostic d'emphysème pulmonaire primitif
(ou emphysème panlobulaire).

Vous envisagez le diagnostic d'emphysème pulmonaire primitif. Quelle(s) est(sont) la(les) valeur(s)
gazométrique(s) et/ou acidobasique(s) compatible(s) avec ce diagnostic (en l'absence de pathologie associée) ?
A.- PaO2 de repos = 80 mmHg (10,7 KPa)
B - PaCO2 de repos = 55 mmHg (7,33 KPa)
C - PaO2 à l'effort = 55 mmHg (7,33 KPa)
D - pH de repos = 7,40
E - HCO3 de repos 40 mmol/l
Bonne(s) réponse(s) : A C D

Le diagnostic d'emphysème pulmonaire primitif est compatible avec une gazométrie artérielle de repos longtemps normale
mais elle s'accompagne d'une désaturation en O2 à l'effort.

Ces constatations gazométriques peuvent être expliquées par :


A - Trouble de la distribution du rapport ventilation/perfusion
B - Distension et destruction des alvéoles
C - Destruction des capillaires pulmonaires
D - Epaississement des cloisons alvéolo-capillaires
E - Diminution du débit cardiaque
Bonne(s) réponse(s) : A B C

L'emphysème pulmonaire primitif ou panlobulaire est caractérisé par une destruction alvéolaire parallèle à la destruction
capillaire. L'existence d'un syndrome obstructif peut, d'autre part, rendre compte d'une altération des rapports ventilation-
perfusion.

Chez ce malade que montrera la spirographie et la boucle débit/volume ?


A - Syndrome obstructif
B - Augmentation de la capacité pulmonaire totale
C - Augmentation du volume résiduel
D - Augmentation de la capacité vitale
E - Diminution du débit moyen entre 25 et 75 % de la capacité vitale
Bonne(s) réponse(s) : A B C E

Ces anomalies témoignent d'une obstruction sur les voies aériennes centrales et périphériques associées à une hyperinflation
(augmentation des volumes).

Les autres examens fonctionnels montrent :


A - Diminution de la capacité de transfert du CO
B - Diminution de la compliance pulmonaire statique
C - Diminution de l'espace mort
D - Zones d'hypofixation à la scintigraphie pulmonaire de perfusion
E - Diminution de la résistance des voies aériennes
Bonne(s) réponse(s) : A D

La diffusion du CO est diminuée quand il existe une obstruction bronchique.


La scintigraphie de perfusion montre des défects dans toutes les pathologies où les vaisseaux sont atteints mais aussi dans
les syndromes obstructifs.
Quant à la compliance statique et à la résistance des voies aériennes, elle augmente dans ce cas précis.

998
Exclusivement sur DOC - DZ : www.doc-dz.com NADJI 85
RESIDANAT EN POCHE TOME II
Cas Clinique en QCM

Les anomalies fonctionnelles observées résultent d'un ou plusieurs des mécanismes physiopathologiques
suivants. Lequel ou lesquels ?
A - Augmentation de la pression de rétraction élastique du parenchyme pulmonaire
B - Hypersécrétion bronchique des gros troncs
C - Réduction de la surface d'échange des gaz
D - Collapsus expiratoire des voies aériennes
E - Inflammation muqueuse au niveau des bronches lobulaires
Bonne(s) réponse(s) : C D

Il existe une destruction des cloisons alvéolaires et des capillaires donc une diminution de la surface d'échange.
la proposition D est litigieuse car il existe obligatoirement un collapsus expiratoire des petites voies aériennes périphériques
du fait de la diminution de la force de rétraction pulmonaire. Cependant, il ne s'agit pas du même collapsus (visible
endoscopiquement sur les grosses voies aériennes) que dans les autres BPCO où on constate parfois une véritable
bronchomalacie.

Dans ce cas d'emphysème panlobulaire primitif, que montre la radiographie pulmonaire ?


A - Abaissement des coupoles diaphragmatiques
B - Hyperclarté des bases
C - Dilatation des artères pulmonaires
D - Cardiomégalie
E - Distension thoracique
Bonne(s) réponse(s) : A B C E

La distension pulmonaire est le signe essentiel sur la radiographie de thorax, elle rend compte de l'aplatissement des
coupoles et de l'hyperclarté des bases qui ne s'obscurcissent pas en expiration.
La dilatation des artères pulmonaires est tardive mais il est difficile de ne pas cocher cette proposition puisque les valeurs
fonctionnelles, en particulier les gaz du sang, ne sont pas mentionnés dans l'énoncé.

Un malade de 60 ans, fumeur, pesant 80 kg pour 1.65 m, ayant l'occasion plusieurs fois par an de faire des repas d'affaires et
à cette occasion, d'abuser de boissons alcoolisées, se plaint d'une toux apparue assez soudainement, il y a dix mois. Diverses
médications ont été employées sans succès. La persistance de la toux l'inquiète. Il se culpabilise : "Ah, ce tabac auquel je ne
puis pas renoncer". Il en perd l'appétit et a maigri.

Les signes fonctionnels que l'on aurait pu rechercher en plus de ceux qui ont été signalés et qui auraient eu un
intérêt diagnostique et thérapeutique comportent :
A - Douleurs thoraciques
B - Dyspnée
C - Hémoptysies
D - Douleur abdominale aiguë
E - Crampes des mollets
Bonne(s) réponse(s) : A B C

A - Recherche d'une atteinte coronaire chez ce fumeur obèse.


B - A la recherche d'une atteinte pulmonaire (néo) ou d'une insuffisance ventriculaire gauche.
C - Signe d'appel majeur sur ce terrain d'un cancer broncho-pulmonaire.
D E - Ne font parties que d'un tableau d'éthylisme chronique.

Parmi les signes constatés à l'examen, le diagnostic peut être orienté par :
A - Examen bucco-dentaire
B - Découverte d'un ganglion axillaire droit
C - Matité de la base gauche
D - Recherche de signes d'insuffisance ventriculaire gauche
E - Toucher rectal
Bonne(s) réponse(s) : A B C D E

Evident. La recherche d'un cancer de la prostate est systématique.

Certain(s) examen(s) complémentaire(s) s'impose(nt). Le(s)quel(s) ?


A - Radiographie thoracique de face et de profil, en inspiration
B - Fibroscopie bronchique
C - Examen de l'expectoration à la recherche des germes banals et mise en culture
D - Exploration fonctionnelle respiratoire
E - Electrocardiogramme
Bonne(s) réponse(s) : A B D E

A B D - Font partie du bilan respiratoire. Le terrain et les signes d'appel imposent la fibroscopie.
C - Non, en tout cas pas à ce stade. Aucun élément oriente vers une étiologie infectieuse.
E - Ou l'interrogatoire ou l'examen aient retrouvé ou non des signes en faveur d'une coronaropathie.
999
Exclusivement sur DOC - DZ : www.doc-dz.com NADJI 85
RESIDANAT EN POCHE TOME II
Cas Clinique en QCM

Les diagnostics auxquels il est légitime de penser incluent :


A - Bronchite chronique
B - Corps étranger endobronchique
C - Cancer bronchique
D - Insuffisance ventriculaire gauche
E - Dilatation bronchique
Bonne(s) réponse(s) : B C D

A - Le tableau, et notamment la survenue brutale et relativement récente de la toux, exclu ce diagnostic.


B - Le début brutal est compatible.
C - Le terrain et l'altération de l'état général plaide en faveur.
D - Famille par coronaropathie.

Le traitement qui s'impose chez ce malade, dès le premier jour, comporte :


A - Un antitussif opiacé ou analogue
B - L'arrêt du tabac immédiat et définitif
C - Une médication théophyllinique
D - Une médication corticoïde
E - La kinésithérapie
Bonne(s) réponse(s) : B C

Dès le premier jour.


A - N'est pas à envisager d'oublie.
B C - Devraient permettre une amélioration rapide et s'imposent dans tous les cas.
D - Jamais d'oubli.
E - Inutile devant l'absence d'expectoration.

Au cours d'une belle soirée d'été, un jeune homme, vivant habituellement en milieu urbain, âgé de 18 ans, en vacances à la
campagne, présente pour la première fois de son existence une crise dyspnéique avec sensation d'étouffement qui le réveille
et l'oblige à appeler le médecin de la ville voisine. Ce dernier porte le diagnostic d'asthme.

Le tableau classique de l'asthme paroxystique de l'adulte jeune comporte :


A - Crise dyspnéique nocturne
B - Sibilance dans les deux champs pulmonaires
C - Bradycardie
D - Fièvre
E - Oedèmes des membres inférieurs
Bonne(s) réponse(s) : A B

Les autres propositions témoignent uniquement de complication.

Il faut rechercher, pour éliminer une forme grave, chez ce patient :


A - Des sueurs
B - Un blocage thoracique
C - Une disparition du murmure vésiculaire
D - Une pâleur des conjonctives
E - Une modification du timbre de la voix
Bonne(s) réponse(s) :

QUESTION ANNULEE

Le premier médicament à utiliser chez ce patient est :


A - Théophylline à action prolongée
B - Anti-histaminique injectable
C - Anticholinergique en inhalation
D - Bêta-sympathomimétique en inhalation
E - Cromoglycate (inhalation)
Bonne(s) réponse(s) : D

Le meilleur traitement des crises d'asthme est l'administration de §-2-mimétiques :


- en commençant par les inhalations
- en surveillant les modalités d'administration.

1000
Exclusivement sur DOC - DZ : www.doc-dz.com NADJI 85
RESIDANAT EN POCHE TOME II
Cas Clinique en QCM

Grâce à votre traitement, cette première crise cède rapidement. Vous revoyez votre jeune malade le lendemain.
Vous suspectez une atopie. Sur quel(s) élément(s) ?
A - Sa mère est asthmatique
B - Son père est un grand fumeur
C - Le patient a des antécédents de rhume des foins
D - Il est épileptique
E - Il a été traité un an auparavant pour une parasitose intestinale
Bonne(s) réponse(s) : A C

Les arguments en faveur d'une atopie à rechercher à l'interrogatoire sont les parentés morbides et équivalents respiratoires de
l'asthme chez le malade et dans sa famille :
- coryza spasmodique (rhume des foins)
- conjonctivites allergiques
- eczéma, urticaire
- toux spasmodique
- oedème de Quincke...

La nature allergique de cette première crise d'asthme peut être évoquée sur le résultat de :
A - Spirographie
B - Numération formule sanguine
C - Dosage des IgE sériques
D - Réaction tuberculinique
E - Immunoélectrophorèse des protides sanguins
Bonne(s) réponse(s) : B C

L'hyperéosinophilie à la NFS et un taux élevé d'IgE totales peuvent se voir dans l'asthme allergique mais ni l'un ni l'autre ne
sont spécifiques
Cette question paraît donc litigieuse.

Au vu des données précédentes, vers quel type d'allergènes allez-vous orienter vos recherches ?
A - Poussières de maison
B - Poils d'animaux
C - Pollens
D - Acariens
E - Moisissures
Bonne(s) réponse(s) :

QUESTION ANNULEE

Après 15 jours de traitement, le patient n'a pas présenté de nouvelle crise. Quel(s) examen(s) demandez-vous ?
A - Spirographie
B - Scintigraphie pulmonaire
C - Boucle débit-volume
D - Fibroscopie bronchique
E - Angiographie pulmonaire
Bonne(s) réponse(s) : A

Seule la spirométrie peut avoir un intérêt à distance de la crise. En effet, le diagnostic d'asthme est porté cliniquement dans
cette forme typique. A distance, s'il n'y a pas de gêne intercritique, les EFR peuvent être normales mais elles serviront de
toute façon de référence pour l'évolution.

Au cours d'une belle soirée d'été, un jeune homme, vivant habituellement en milieu urbain, âgé de 18 ans, en vacances à la
campagne, présente pour la première fois de son existence une crise dyspnéique avec sensation d'étouffement qui le réveille
et l'oblige à appeler le médecin de la ville voisine. Ce dernier porte le diagnostic d'asthme.

Devant cette crise isolée, que conseillez-vous à Monsieur X..., au plan thérapeutique ?
A - Sympathomimétiques en spray en cas de crise
B - Théophylline au long cours
C - Corticothérapie prolongée
D - Antibiothérapie de 10 jours
E - Cures climatiques
Bonne(s) réponse(s) : A

C'est le seul traitement nécessaire devant un asthme intermittent peu sévère du sujet jeune.

1001
Exclusivement sur DOC - DZ : www.doc-dz.com NADJI 85
RESIDANAT EN POCHE TOME II
Cas Clinique en QCM
Un homme de 61 ans, consulte pour dyspnée d'effort apparue progressivement depuis trois ans et actuellement invalidante
pour la montée des escaliers des deux étages de son appartement.
L'interrogatoire retient un tabagisme habituel à un paquet de cigarettes par jour depuis 35 ans, une toux avec expectoration
matinale depuis de nombreuses années. A l'examen, le thorax n'est pas distendu, il n'y a pas de cyanose, ni de signe
d'insuffisance ventriculaire droite. A l'auscultation, on perçoit des râles d'encombrement bronchique et le murmure vésiculaire
est bien audible.
L'expectoration est actuellement non purulente et facile à émettre.

Quel est votre diagnostic ?


A - Fibrose pulmonaire
B - Bronchite chronique obstructive
C - Emphysème primitif
D - Asthme
E - Oedème pulmonaire
Bonne(s) réponse(s) : B

La bronchite chronique répond à une définition clinique : il s'agit d'une toux et d'une expectoration quotidienne pendant 3 mois
par an, deux années consécutives. Le patient décrit ici répond à cette définition.

Vous devez retrouver à l'exploration fonctionnelle respiratoire ?


A - Baisse du VEMS
B - Baisse du rapport VEMS/CV
C - Augmentation du volume résiduel (VR)
D - Baisse de la compliance
E - Baisse de la capacité pulmonaire totale
Bonne(s) réponse(s) : A B C

Les EFR sont certainement perturbées puisque le malade se plaint d'une dyspnée d'effort. On doit
retrouver :
- un syndrome obstructif : baisse des débits en particulier du VEMS
- avec distension thoracique avec augmentation des volumes (VR et CPT)

Le bilan doit comporter :


A - Radio pulmonaire
B - E.C.G.
C - Gaz du sang
D - Ionogramme sanguin
E - Tomographies pulmonaires
Bonne(s) réponse(s) : A B C D

Il faut évaluer la fonction cardio-respiratoire ce qui justifie radio, ECG, gaz du sang ; s'assurer qu'il n'y a pas de complication
en particulier de cancer bronchique (radio, voire fibro) le ionogramme sanguin est utile pour s'assurer de sa normalité avant
toute thérapeutique.

La prescription thérapeutique doit comporter :


A - Fluidifiants bronchiques
B - Broncho-dilatateurs
C - Arrêt du tabac
D - Kinésithérapie respiratoire
E - Tonicardiaque
Bonne(s) réponse(s) : B C D

- Les bronchodilatateurs peuvent améliorer un syndrome obstructif en partie réversible


- Il est impératif d'arrêter le tabagisme pour éviter que la fonction respiratoire continue à se dégrader
- Une kinésithérapie de drainage peut aussi améliorer la dyspnée, en partie liée à l'encombrement

Dans quel(s) cas, devez vous craindre une aggravation de l'état de ce malade ?
A - Usage d'antitussifs
B - Usage de tranquillisants
C - Usage de Béta 2 mimétiques
D - Infection broncho-pulmonaire
E - Phlébite
Bonne(s) réponse(s) : A B D E

En ce qui concerne les médicaments :


- Les antitussifs contiennent, pour la plupart, des opiacés. Ils sont formellement contre-indiqués s'il existe une hypercapnie.
- Tous les sédatifs sont contre-indiqués dans le même contexte.
L'infection bronchopulmonaire, de même qu'un accident thrombo-embolique sont des causes habituelles de décompensation
aiguë d'insuffisance respiratoire chronique.
1002
Exclusivement sur DOC - DZ : www.doc-dz.com NADJI 85
RESIDANAT EN POCHE TOME II
Cas Clinique en QCM

Une aggravation éventuelle sera évoquée devant :


A - Céphalées
B - Insomnie
C - Cyanose
D - Sueurs
E - Tachycardie
Bonne(s) réponse(s) : A B C D E

- Une aggravation de l'hypercapnie est évoquée devant : les céphalées, l'inversion du rythme nycthéméral (insomnie nocturne,
somnolence diurne), sueurs.
- Une hypoxémie devant l'apparition d'une cyanose et l'existence d'une tachycardie.

Devant l'apparition de ces signes, une gazométrie artérielle montre . Pa 02 = 45 mm Hg (5,9 KPa ) Pa C02 = 58
mm Hg ( 7,7 KPa ) Ph = 7,37. Donner les deux mesures à prendre immédiatement :
A - Oxygénothérapie
B - Hospitalisation
C - Digitaliques
D - Antibiothérapie
E - Fluidifiants bronchiques
Bonne(s) réponse(s) : A B

L'hospitalisation s'impose pour :


- Diagnostiquer la cause de la décompensation et la traiter
- Mettre en route une oxygénothérapie à faible débit
- Surveiller l'évolution, qui si elle est défavorable peut conduire à la ventilation assistée

Un homme de 56 ans est hospitalisé pour une importante dyspnée d'aggravation progressive depuis 3/4 heure. Il est assis
dans son lit, cyanosé, couvert de sueurs: son rythme respiratoire est ralenti, avec allongement du temps inspiratoire.
L'interrogatoire révèle l'existence d'une dysphonie progressive, installée depuis plusieurs mois pour aboutir à une voix
actuellement pratiquement éteinte Dans les jours précédents, était apparue une gêne respiratoire à l'effort. On note par
ailleurs une intoxication tabagique de l'ordre de deux paquets de cigarettes par jour depuis l'âge de 20 ans et une
imprégnation éthylique notable (1,5 litre de vin par jour). Il n'y a pas d'otalgie, pas de dysphagie, pas d'expectoration
sanglante. L'auscultation pulmonaire est normale ; la palpation cervicale ne montre pas d'anomalie, non plus que l'examen
général somatique.

Devant ce type de dyspnée, quel est le diagnostic à évoquer en premier lieu parmi les suivants ?
A - Asthme
B - Oedème aigu du poumon
C - Bronchite chronique
D - Epanchement pleural
E - Dyspnée laryngée
Bonne(s) réponse(s) : E

L'association d'une dyspnée inspiratoire à une dysphonie récente signe la localisation laryngée de l'atteinte.

Quel(s) est(sont) les éléments cliniques qui, s'ils existent, vont conforter le diagnostic topographique de cette
dyspnée ?
A - Tirage
B - Cornage
C - Wheezzing
D - Modifications de la sonorité à la percussion thoracique
E - Râles bronchiques
Bonne(s) réponse(s) : A B

Tirage et cornage sont les deux signes qui peuvent accompagner une dyspnée laryngée.

1003
Exclusivement sur DOC - DZ : www.doc-dz.com NADJI 85
RESIDANAT EN POCHE TOME II
Cas Clinique en QCM

Le contexte clinique, le terrain et l'évolution évoquent essentiellement l'une des étiologies suivantes :
A - Polype des cordes vocales
B - Cancer du poumon
C - Cancer glottique
D - Paralysie des dilatateurs de la glotte
E - Epanchement pleural
Bonne(s) réponse(s) : C

Les arguments en faveur de ce diagnostic sont :


- L'âge 56 ans
- Le tabagisme à 70 paquets/an, l'intoxication éthylique
- L'existence d'une dysphonie lentement progressive
- Et l'existence d'une dyspnée rapidement progressive avec aggravation brutale évoquant une obstruction sur un processus
expansif

Quel examen allez-vous faire pratiquer pour confirmer votre diagnostic ?


A - ECG
B - Radiographie du thorax
C - Gazométrie artérielle
D - Laryngoscopie
E - Bronchoscopie
Bonne(s) réponse(s) : D

La laryngoscopie doit être pratiquée en urgence pour conforter le diagnostic et réaliser une trachéotomie si l'état respiratoire
ne s'améliore pas médicalement.

Parmi les modalités thérapeutiques suivantes, laquelle(lesquelles) allez-vous immédiatement mettre en oeuvre ?
A - Pulvérisation locale d'un sympathomimétique
B - Corticothérapie par voie parentérale
C - Traitement diurétique par lasilix IV
D - Atropine 1/2 mg par voie sous-cutanée
E - Oxygénothérapie
Bonne(s) réponse(s) : B E

Le premier geste doit consister en une corticothérapie par voie générale afin de faire régresser la part d'oedème éventuel.

En cas d'échec de la(des) méthode(s) utilisée(s) ci-dessus, quels sont les deux gestes que l'on peut proposer ?
A - Intubation
B - Trachéotomie
C - Pelage des cordes vocales sous endoscopie laryngée
D - Drainage pleural
E - Saignée
Bonne(s) réponse(s) : B

Devant un obstacle tumoral laryngé l'intubation est impossible. L'amélioration de la ventilation ne peut passer que par la
trachéotomie.

1004
Exclusivement sur DOC - DZ : www.doc-dz.com NADJI 85
RESIDANAT EN POCHE TOME II
Cas Clinique en QCM
Un homme de 45 ans, présente à l'occasion d'un épisode infectieux apparemment banal, une hémoptysie de moyenne
abondance.
Il a été traité, 7 ans auparavant, pour une tuberculose étendue bilatérale, avec volumineuse cavité lobaire supérieure droite et
opacités infiltratives diffuses intéressant le segment apical du lobe supérieur droit. Un traitement par rifampicine, isoniazide et
éthambutol, a rapidement assuré la disparition des bacilles tuberculeux de l'expectoration, puis une régression des anomalies
radiologiques. Après 9 mois de traitement persistait cependant une cavité aux parois fines, d'un diamètre d'environ 2 cm, au
niveau d'un lobe supérieur droit rétracté et siège d'opacités linéaires d'aspect cicatriciel. Une exérèse de ces lésions
résiduelles a été proposée au patient, qui l'a refusée. Le patient a alors repris son activité professionnelle, sous surveillance
radiologique périodique.
L'examen radiologique réalisé à l'occasion de l'hémoptysie récente révèle une modification nette des anomalies lobaires
supérieures droites par rapport au dernier cliché pratiqué, deux années plus tôt : la cavité s'est légèrement rétractée, sa paroi
s'est épaissie et elle contient une opacité arrondie séparée de la paroi supérieure par un croisement gazeux. On note en outre
un épaississement pleural de l'apex. L'aspect est évocateur de greffe aspergillaire dans une cavité tuberculeuse détergée.

Parmi les examens biologiques suivants, quel est celui qui apporte un argument décisif en faveur du diagnostic
d'aspergillome intracavitaire ?
A - Présence d'Aspergillus fumigatus à la culture de l'expectoration
B - Présence de 6 arcs de précipitation après immunoélectrophorèse de sérum en présence de l'antigène
Aspergillus fumigatus
C - RAST positif, vis-à-vis d'Aspergillus fumigatus
D - Réaction cutanée semi-retardée positive vis-à-vis d'Aspergillus fumigatus
E - Réaction cutanée retardée positive vis-à-vis d'Aspergillus fumigatus
Bonne(s) réponse(s) : B

La présence de 6 arcs de précipitation à la sérologie aspergillaire est un résultat franchement positif qui témoigne de la
colonisation du parenchyme pulmonaire par Aspergillus fumigatus.
La présence d'Aspergillus fumigatus dans l'expectoration ne témoigne que de la colonisation bronchique. Quant aux autres
examens, ils sont intéressants quand on suspecte une réaction d'hypersensibilité à l'Aspergillus fumigatus.

Quel est le mécanisme physiopathologique de l'hémoptysie présentée par ce malade ?


A - Anévrysme de Rasmussen
B - Fistule broncho-artérielle pulmonaire
C - Hypervascularisation péri-cavitaire d'origine systémique
D - Thrombose d'une branche de l'artère pulmonaire suivie d'infarcissement hémorragique
E - Hémorragie alvéolaire diffuse au voisinage de la cavité parasitée
Bonne(s) réponse(s) : C

Sur les séquelles de tuberculose, particulièrement au pourtour de cavités résiduelles apparaît une hypervascularisation
bronchique d'origine systémique. Ces néovaisseaux des tissus de cicatrisation sont fragiles et peuvent saigner en particulier
lorsqu'il y a effraction par une colonisation aspergillaire.

Parmi les substances antifongiques suivantes, quelle(s) est(sont) celle(s) qui, absorbée(s) par voie orale, sont
susceptibles de diffuser dans l'organisme ?
A - Amphotéricine B (Fungizone®)
B - Flucytosine (Ancotil®)
C - Kétoconazole (Nizoral®)
D - Nystatine (Mycostatine®)
E - Aucune de ces substances
Bonne(s) réponse(s) : C

Le Kétoconazole est le seul antifongique cité susceptible de traverser la barrière intestinale en restant actif.

En dehors des aspergillomes intra-cavitaires, quelle(s) affection(s) respiratoire(s) peut(peuvent) être


provoquée(s) par Aspergillus fumigatus ?
A - Asthme allergique IgE dépendant
B - Infiltrats éosinophiles pulmonaires
C - Infection diffuse du tissu pulmonaire
D - Emphysème centrolobulaire
E - Fibrose interstitielle diffuse
Bonne(s) réponse(s) : A B C

C : Aspergillose pulmonaire invasive : affection qui se voit chez les immunodéprimés, responsable d'atteinte vasculaire
conduisant à des zones de nécrose ischémique. Tableau gravissime.
A B : L'asthme aspergillaire et les infiltrats à éosinophiles se voient le plus souvent sur des asthmes vieillis ; ils entrent dans le
cadre de l'aspergillose bronchopulmonaire allergique ou maladie de Hinson-Peppys.

1005
Exclusivement sur DOC - DZ : www.doc-dz.com NADJI 85
RESIDANAT EN POCHE TOME II
Cas Clinique en QCM
Un agriculteur de 50 ans, non fumeur, sans antécédent particulier, est hospitalisé pour le bilan d'une dyspnée d'installation
progressive depuis 2 mois avec amaigrissement de 5 kgs, syndrome sub-fébrile, apparus fin Mars.

La radiographie pulmonaire montre un aspect en "verre dépoli" des deux bases pulmonaires. Il n'existe pas de cardiomégalie.
Vous évoquez la possibilité d'une maladie du poumon de fermier.

Qu'entendez-vous à l'auscultation pulmonaire ?


A - Sibilances diffuses dans les 2 poumons
B - Silence respiratoire
C - Râles crépitants bilatéraux
D - Auscultation normale
E - Ronchus diffus dans les deux champs
Bonne(s) réponse(s) : C

La maladie des poumons de fermier est une pneumopathie interstitielle avec "alvéolite". Elle se manifeste le plus souvent par
des râles crépitants à l'auscultation.

Quelle(s) perturbation(s) doit ou doivent être observée(s) sur l'EFR


A - Augmentation du volume résiduel
B - Trouble ventilatoire restrictif pur
C - Abaissement de la compliance pulmonaire statique
D - Désaturation en oxygène artériel de repos et resaturation à l'exercice
E - Baisse de la capacité de diffusion
Bonne(s) réponse(s) : B C D E

Comme toutes pneumopathies interstitielles, les altérations des EFR sont :


- au début : un trouble de diffusion avec hypoxie d'effort, voire de repos
- a un stade plus évolué : un syndrome restrictif avec baisse de la compliance pulmonaire statique

Un lavage broncho-alvéolaire avec numération formule cytologique est réalisé. Que peut-on observer ?
A - Une numération formule cytologique normale
B - Une Iymphocytose supérieure à 50%
C - Une hypercellularité franche
D - Une élévation franche du pourcentage des mastocytes
E - Une élévation du pourcentage des lymphocytes et des polynucléaires neutrophiles
Bonne(s) réponse(s) : B C E

La maladie du poumon de fermier s'accompagne de modifications cytologiques des alvéoles :


- augmentation de la cellularité globale
- lymphocytose soit seule, soit accompagnée de polynucléose neutrophile au tout début, ou au contraire à un stade évolué de
fibrose

Le traitement repose sur :


A - Streptomycine
B - Cyclines
C - Macrolides
D - Arrêt de l'exposition professionnelle
E - Théophylline
Bonne(s) réponse(s) : D

Le meilleur traitement est bien sûr l'éviction des allergènes inhalés.


Il s'agit d'une maladie immunologique et non infectieuse.

1006
Exclusivement sur DOC - DZ : www.doc-dz.com NADJI 85
RESIDANAT EN POCHE TOME II
Cas Clinique en QCM
Un homme de 35 ans consulte pour hémoptysie. Il s'agit d'un patient tabagique (15 paquets/année), sans antécédent familial
notable, et qui à l'âge de 5 ans aurait fait une infection respiratoire qualifiée par sa famille de sévère. Au décours de cet
épisode, il a développé une bronchorrhée matinale, quasi quotidienne, à recrudescence hivernale, qu'il a négligée; ses
activités professionnelles ne l'exposent pas à un aérocontaminant particulier et il ne se plaint pas de dyspnée. L'hémoptysie
est apparue le matin même de la consultation, au cours d'une quinte de toux; son volume total est évalué à 50 ml. A l'examen,
il s'agit d'un sujet longiligne pesant 63 kg pour 1m73, apyrétique, et sans cyanose; les muqueuses sont normalement
colorées, la fréquence respiratoire est à 20/mn, la fréquence cardiaque à 90/mn, la T.A.à 130/80 mmHg. L'examen cutané est
normal, mais il existe un hippocratisme digital très ancien. L'auscultation cardiaque est normale. L'examen des cavités rhino et
oropharyngées ne permet pas de retrouver de trace de saignement. A l'auscultation du thorax, on entend, outre le murmure
vésiculaire, des râles bronchiques qui prédominent au niveau de la base pulmonaire gauche. Le reste de l'examen clinique est
normal. La Numération Formule Sanguine (plaquettes incluses), le TCK et le TP sont normaux. L'analyse des clichés de
thorax (face et profil) permet d'observer, au sein des deux lobes inférieurs, des cavités multiples, arrondies, tassées, aux
limites mal définies. Le lobe gauche, le poumon droit et le médiastin, paraissent radiologiquement normaux.

L'affection chronique dont souffre ce patient est :


A - Une dilatation des bronches
B - Une bronchite chronique
C - Un asthme à dyspnée continue
D - Une emphysème pan-lobulaire
E - Une insuffisance ventriculaire gauche
Bonne(s) réponse(s) : A

Les arguments en faveur de ce diagnostic sont :


- l'antécédent respiratoire de l'enfance
- la bronchorrhée chronique
- les hémoptysies
- le syndrome bronchique
- les cavités multiples des deux lobes inférieurs sur la radio du thorax

Quelles sont parmi les agents infectants de l'enfance, la ou les possible(s) cause(s) de l'affection chronique dont
souffre ce patient ?
A - Mycobacterium tuberculosis
B - Virus respiratoire syncitial
C - Borderella pertussis
D - Streptococcus pneumoniae
E - Legionella pneumophila
Bonne(s) réponse(s) : B C

Les viroses graves de l'enfance peuvent être responsables de bronchectasies, par exemple le VRS mais aussi la rougeole.
La coqueluche est une cause de bronchectasie.
La tuberculose donne aussi des bronchectasies mais le plus souvent localisées aux apex. De plus, chez un patient de 35 ans
ce diagnostic étiologique est peu probable.

Quelle(s) investigation(s) paraclinique(s) est(sont) susceptible(s) de préciser l'origine anatomique du


saignement ?
A - Tomodensitométrie thoracique
B - Fibroscopie bronchique
C - Artériographie bronchique en période hémorragique
D - Bronchographie
E - Angiographie pulmonaire
Bonne(s) réponse(s) : B

Seule la fibroscopie permet de localiser l'origine du saignement dans un territoire anatomique bronchique précis.
L'artériographie bronchique permet de mettre en évidence une hypervascularisation bronchique systémique mais si cette
dernière est diffuse, elle ne précise pas le territoire qui saigne.

Quel(s) réseau(x) vasculaire(s) est(sont) à la source des hémoptysies dans le cas présent ?
A - Artères pulmonaires
B - Artères systémiques bronchiques
C - Capillaires pulmonaires
D - Veines pulmonaires
E - Aucun des réseaux ci-dessus mentionnés
Bonne(s) réponse(s) : B

Les bronchectasies sont à l'origine d'hémoptysies du fait de l'existence à leur périphérie d'un réseau hypervascularisé fragile
qui naît des artères bronchiques. Il s'agit de sang d'origine artérielle ce qui fait toute leur gravité.

1007
Exclusivement sur DOC - DZ : www.doc-dz.com NADJI 85
RESIDANAT EN POCHE TOME II
Cas Clinique en QCM

Vous prescrivez en première intention pour juguler l'hémoptysie de ce patient ?


A - Embolisation artérielle bronchique sélective
B - Perfusion de lysyl-vasopressine
C - Intubation sélective par sonde de Carlens
D - Bronchoconstricteur (Carbamyl-choline)
E - Transfusion de facteurs de coagulation
Bonne(s) réponse(s) : B

Le premier traitement à envisager si l'hémoptysie persiste, est un agent vasoconstricteur actif sur les artères systémiques, tel
la lysyl-vasopressine.

L'hémoptysie étant tarie, quel(s) examen(s) prescrivez-vous pour réaliser le bilan d'extension des lésions
bronchiques ?
A - R.M.N. thoracique
B - Tomodensitométrie thoracique en coupes fines
C - Bronchographie bilatérale
D - Angiographie pulmonaire
E - Artériographie bronchique
Bonne(s) réponse(s) : B

La tomodensitométrie est l'examen de choix pour porter le diagnostic et faire le bilan de l'étendue des DDB de façon non
agressive.
La bronchographie est de plus en plus abandonnée dans cette indication. L'artériographie bronchique n'est pas indispensable
si les hémoptysies sont taries et qu'une embolisation n'est pas nécessaire.

Mademoiselle S. 22 ans, étudiante, est asthmatique de longue date et présente 2 à 3 crises par mois. Elle ne prend aucun
traitement de fond. Elle arrive d'urgence pour une crise dont le début remonte à quelques heures et qui fait suite à une
bronchite aiguë. La crise n'a pas cédé après six bouffées de Salbutamol. L'examen clinique est typique de crise d'asthme avec
freinage expiratoire, distension thoracique, sibilants diffus mais rares. Malgré les thérapeutiques d'urgence, la crise ne cède
pas totalement. La patiente est admise en hospitalisation, traitée et surveillée.

Parmi les propositions suivantes, cocher celle(s) qui vous paraît ou paraissent faire partie du traitement de
première intention :
A - Bécotide 2 bouffées en surveillant les modalités d'administration
B - Théophylline 1 ampoule en intraveineux direct
C - Soludécadron 1 ampoule en intraveineux direct
D - Bricanyl 1/2 ampoule par voie sous cutanée
E - Mise en place d'un suppositoire de Théophylline
Bonne(s) réponse(s) : D

Chez tout asthmatique qui présente une crise "résistante au Salbutamol en spray" il faudra :
- Vérifier les modalités d'administration du spray. S'il reste inefficace :
- Administrer 1/2 ampoule de Bricanyl sous cutané
- Continuer la surveillance

Au bout de 24 heures, son état s'aggrave. Parmi les critères suivants, le(s)quel(s) vous paraît(paraissent) être
des critères de gravité d'une crise d'asthme ?
A - L'absence de sédation sous ventoline correctement administrée
B - L'existence d'un silence auscultatoire
C - Une hypercapnie
D - La notion d'état de mal dans les antécédents
E - Un peak-flow entre 400 et 500 ml/mn
Bonne(s) réponse(s) : A B C D

L'existence de silence auscultatoire traduit une obstruction majeure de même que l'hypercapnie.
L'absence de sédation sous traitement "habituel" fait suspecter une crise plus grave.
Enfin, tout malade ayant déjà fait un état de mal est susceptible d'en refaire un d'apparition brutale.

1008
Exclusivement sur DOC - DZ : www.doc-dz.com NADJI 85
RESIDANAT EN POCHE TOME II
Cas Clinique en QCM

Finalement la patiente est admise en réanimation. Parmi les propositions suivantes concernant l'état de mal
asthmatique, quelle est ou quelles sont celle(s) qui vous paraît ou paraissent exacte(s) ?
A - L'oxygénothérapie à faible débit est contre-indiquée
B - Les béta 2 mimétiques intraveineux constituent le meilleur traitement
C - Les corticoïdes ont un délai d'action inférieur à 4 heures
D - La théophylline intra-veineuse est strictement indispensable
E - Un antitussif est nécessaire
Bonne(s) réponse(s) : B

Les béta 2 mimétiques constituent le meilleur traitement de l'état de mal. Leur posologie doit être adaptée en augmentant les
doses jusqu'à amélioration.
La théophylline intraveineuse peut être utilisée en continu à la seringue électrique, mais son efficacité est contestée par
beaucoup, elle n'est pas indispensable.
En revanche, les antitussifs sont contre-indiqués chez ces malades qui ont une hypersécrétion visqueuse à évacuer
impérativement pour éviter les bouchons endobronchiques.

Après 48 heures en soins intensifs, la crise cède et la patiente est transférée en salle. Parmi les éléments
suivants, le(s)quel(s) fait(font) partie du bilan étiologique de l'asthme de cette malade ?
A - La numération formule sanguine
B - Les IgE totales
C - Un examen ORL
D - Des tests allergologiques cutanés orientés par l'interrogatoire
E - Des EFR avec test à l'acétylcholine
Bonne(s) réponse(s) : A C D

L'Acétylcholine est formellement contre-indiquée chez tout asthmatique diagnostiqué et à plus forte raison s'il a fait un état de
mal.
La NFS recherche une hyperéosinophilie.
L'examen ORL recherche des foyers infectieux et des arguments en faveur d'une allergie (oedème de la muqueuse).
Les tests allergologiques n'ont de valeur que s'il existe des allergènes déclenchants retrouvés à l'interrogatoire.

L'enquête étiologique s'est révélée négative et elle sort au quinzième jour. Chez cette patiente, quel traitement
de sortie proposeriez-vous ?
A - Aucun traitement en dehors du Salbutamol en cas de crise
B - Corticothérapie au long cours
C - Théophylline retard seule
D - Théophylline retard avec béta 2 mimétiques en spray systématiques
E - Théophylline retard avec béta 2 mimétiques en spray en cas de crise
Bonne(s) réponse(s) : D

Chez un malade asthmatique aux antécédents d'état de mal, il est indispensable de donner un traitement de fond qui doit
comporter :
- Obligatoirement un béta 2 mimétique en spray systématique (2 bouffées x 4/jour)
- Pour la plupart des équipes une théophylline retard en 2 prises par 24 heures, posologie adaptée au poids
- Si une corticothérapie est prescrite, elle doit être progressivement dégressive et interrompue et non "au long cours" du fait de
ses effets secondaires majeurs.

Un malade de 65 ans, ancien gros fumeur (70 paquets/année) consulte pour des crachats hémoptoïques et une baisse
importante de l'état général. Il présente un hippocratisme digital. Sur la radio pulmonaire on constate un gros hile avec
atélectasie du lobe supérieur droit. La capacité vitale et la gazométrie artérielle sont normales le Vems est subnormal (83%
de la valeur théorique).

Quel diagnostic évoquez-vous d'emblée ?


A - Tuberculose pulmonaire
B - Abcès
C - Bronchectasies
D - Cancer bronchique
E - Tumeur bronchique bénigne
Bonne(s) réponse(s) : D

L'association de :
- hémoptysie
- altération de l'état général
- tabagisme
- et atéléctasie avec hypertrophie hilaire
doit faire penser en premier lieu à un cancer bronchique primitif.

1009
Exclusivement sur DOC - DZ : www.doc-dz.com NADJI 85
RESIDANAT EN POCHE TOME II
Cas Clinique en QCM

A la recherche d'agents cancérigènes, vous allez rechercher chez ce malade :


A - Tabac
B - Silice
C - Amiante
D - Goudrons
E - Chrome
Bonne(s) réponse(s) : A C D E

L'exposition à l'amiante multiplie le risuqe d'avoir un cancer bronchique chez un fumeur. Les goudrons (en particulier ceux
contenus dans la fumée de tabac), sont cancérigènes. Mais il ne sont pas reconnus comme facteur de maladie
professionnelle.

Parmi les examens suivants, quel est celui qui permet d'affirmer le diagnostic ?
A - Fibroscopie bronchique avec biopsie ou aspiration
B - Scanner corps entier
C - Echographie hépatique
D - Scintigraphie osseuse
E - Scintigraphie pulmonaire de perfusion
Bonne(s) réponse(s) : A

Seule la fibroscopie bronchique permet, parmi les examens cités d'apporter une certitude histologique, grâce aux biopsies
bronchiques.

Quel est dans cette liste le type histologique de cancer bronchique le plus fréquemment rencontré ?
A - Cancer anaplasique à petites cellules
B - Adénocarcinome
C - Bronchiolo-alvéolaire
D - Epidermoïde
E - Cancer à grandes cellules
Bonne(s) réponse(s) : D

Fréquence des différents types histologiques :


- Epidermoïdes environ 60%
- Adénocarcinome environ 20%
- Anaplasiques à petites cellules environ 20%
Les bronchiolo-alvéolaires sont rares et rangés dans les adénocarcinomes.
Le cancer à grandes cellules est également rare et bénéficie du même traitement que les épidermoïdes.

Chez ce malade, le bilan d'extension s'avérant satisfaisant et s'agissant d'un carcinome épidermoïde, quelle
décision thérapeutique allez-vous prendre ?
A - Abstention avec simple surveillance
B - Radiothérapie
C - Polychimiothérapie
D - Exérèse chirurgicale
E - Immunothérapie
Bonne(s) réponse(s) : D

L'exérèse chirurgicale si elle est complète (carcinologiquement satisfaisante) est le seul traitement curatif du cancer
bronchique épidermoïde. La radio et la chimiothérapie ne sont que des méthodes palliatives qui n'entraînent pas de guérison
dans l'immense majorité des cas, mais de rares rémissions.

Quel est le pourcentage de survie à 5 ans chez ce patient, s'il est opéré ?
A - Moins de 5 %
B - 5 à 10%
C - 10 à 25 %
D - 25 à 35 %
E - 50 %
Bonne(s) réponse(s) : C

La guérison est actuellement définie par une survie à 5 ans sans récidive. Elle est de 10 à 25% pour le cancer épidermoïde
opéré, tous stades de dissémination ganglionnaire confondus. Il est évident que les N0 ont un meilleur pronostic que les N2
médiastinaux.

1010
Exclusivement sur DOC - DZ : www.doc-dz.com NADJI 85
RESIDANAT EN POCHE TOME II
Cas Clinique en QCM
Une femme de 56 ans, européenne, sans profession, vient consulter pour une toux sèche, isolée, durant depuis trois mois. La
toux est principalement nocturne mais aussi diurne. Parmi les antécédents, on note l'existence d'une hypertension artérielle
essentielle traitée depuis un an; la malade n'a jamais fumé. L'examen clinique n'est pas significatif.

Quelle(s) hypothèse(s) étiologique(s) est(sont) plausible(s) devant ce tableau clinique ?


A - Dilatation des bronches
B - Pleurésie
C - Reflux gastro-oesophagien
D - Tumeur des voies aériennes
E - Fibrose pulmonaire
Bonne(s) réponse(s) : B C D E

Mauvaise formulation de la question, car les renseignements sont trop vagues.


Une toux sèche nocturne peut faire évoquer :
- Un reflux gastro-oesophagien
- Une tumeur des voies aériennes qui se manifesterait de par sa localisation surtout en décubitus dorsal
- La toux pleurale et la toux des fibroses peuvent être à recrudescence nocturne

Parmi les médicaments (noms commerciaux) que la patiente pourrait prendre pour son hypertension artérielle,
lequel est susceptible de provoquer une toux chronique ?
A - Captolane®
B - Catapressan®
C - Aldomet®
D - Minipress®
E - Aucun
Bonne(s) réponse(s) : A

Les inhibiteurs de l'enzyme de conversion de l'angiotensine, particulièrement le Captopril et l'Enalapril peuvent être
responsables de toux chroniques.

Sur la radiographie thoracique, y a-t-il un signe qui dans ce contexte clinique serait évocateur d'un diagnostic
étiologique précis ?
A - Absence de poche à air gastrique
B - Aorte déroulée
C - Complexe primaire calcifié
D - Lobe azygos
E - Côte cervicale
Bonne(s) réponse(s) : A

L'absence de poche à air gastrique témoigne le plus souvent de l'existence d'une hernie hiatale.
Par conséquent, ce signe évoque une toux secondaire à un reflux gastro-oesophagien.

Chez cette patiente, l'asthme peut être éliminé devant :


A - L'absence de râles sibilants au moment de l'examen
B - L'absence de dyspnée au moment de l'examen
C - L'absence de rhinite associée
D - L'absence d'éosinophilie sanguine
E - Aucune des propositions précédentes.
Bonne(s) réponse(s) : E

L'asthme ne peut être éliminé en phase intercritique sur des arguments cliniques. Il nécessite une exploration fonctionnelle
respiratoire avec test de provocation à l'acétylcholine pour mettre en évidence ou éliminer une hyper-réactivité bronchique.

Le diagnostic d'asthme pourrait être affirmé devant :


A - Un taux élevé d'IgE totales
B - Des tests cutanés positifs à la poussière de maison
C - Un test de provocation bronchique positif à l'acétylcholine ou à l'histamine
D - Un aspect de distension pulmonaire radiologique
E - Aucune des propositions suivantes
Bonne(s) réponse(s) : C

Commentaire : voir plus haut


Le test à l'acétyl choline est dit positif s'il entraîne une chute du VEMS de 20% pour des doses d'acétyl choline inférieures à
1500 µg.

1011
Exclusivement sur DOC - DZ : www.doc-dz.com NADJI 85
RESIDANAT EN POCHE TOME II
Cas Clinique en QCM

La spirométrie met en évidence, sur la courbe débit-volume, une diminution isolée du débit expiratoire à 50% de
la CV. Cette anomalie peut-elle être interprétée comme la traduction :
A - D'une obstruction des voies aériennes périphériques
B - D'une augmentation de la pression de recul élastique du poumon
C - D'une obstruction laryngée
D - D'une diminution de la force des muscles expiratoires
E - Aucune des propositions précédentes
Bonne(s) réponse(s) : E

- Une obstruction des voies aériennes périphériques entraîne une diminution des débits pour toute la partie terminale de la
courbe débit-volume, de même une augmentation de la pression de recul.
- Une obstruction laryngée modifie la courbe inspiratoire.
- Une diminution de la force expiratoire modifie la partie initiale de l'expiration sur la courbe.

Une femme de 45 ans, ayant eu la coqueluche dans l'enfance, vient consulter pour une expectoration matinale, régulière,
abondante, parfois purulente, parfois hémoptoïque. On lui trouve un hippocratisme digital à l'examen clinique.

Quel diagnostic allez-vous suspecter ?


A - Asthme surinfecté
B - Bronchite chronique
C - Emphysème
D - Bronchites aiguës à répétition
E - Bronchectasies
Bonne(s) réponse(s) : E

Les arguments diagnostiques sont l'association :


- D'antécédent de coqueluche
- La bronchorrhée chronique parfois purulente
- Les hémoptysies
- L'hipppocratisme digital

Le ou les argument(s) qui a(ont) servi à asseoir votre diagnostic est (sont) :
A - Antécédents de coqueluche
B - Crachats hémoptoïques
C - Hippocratisme digital
D - Expectoration matinale
E - Sexe de la malade
Bonne(s) réponse(s) : A B C D

Commentaire : voir plus haut.

Parmi ces examens complémentaires, quel est celui qu'il faut maintenant demander en premier lieu ?
A - Artériographie bronchique
B - Fibroscopie bronchique
C - Tomodensitométrie thoracique en haute résolution
D - Bronchographie
E - Scintigraphie pulmonaire de perfusion
Bonne(s) réponse(s) : C

La tonodensitométrie permet actuellement de porter le diagnostic de bronchectasies à condition qu'elle soit réalisée en coupes
fines, éventuellement en orientant les coupes dans l'axe des bronches (en oblique). Elle remplace dans ces conditions la
bronchographie.

Les aspects radiographiques les plus compatibles avec le diagnostic envisagé, comprennent :
A - Atélectasie en bande
B - Opacités en doigts de gant
C - Atélectasie lobaire inférieure
D - Images polykystiques
E - Adénopathies hilaires
Bonne(s) réponse(s) : C D

Les images polykystiques sont la traduction directe des dilatations bronchiques. L'atélectasie lobaire inférieure est fréquente
dans les bronchectasies diffuses et témoignent d'une destruction du parenchyme adjacent.
Les opacités en doigt de gant peuvent se voir, mais traduisent un bronchocèle associé.
Les atélectasies en bandes doivent faire évoquer en premier lieu d'autres diagnostics (en particulier l'embolie pulmonaire).
Les adénopathies sont fréquentes dans les DDB mais le plus souvent invisibles sur la simple radio standard.

1012
Exclusivement sur DOC - DZ : www.doc-dz.com NADJI 85
RESIDANAT EN POCHE TOME II
Cas Clinique en QCM
Une femme de 50 ans, est hospitalisée pour exploration d'une insuffisance respiratoire sévère.
Elle n'a pas d'antécédent pulmonaire ou cardiaque, elle a noté depuis 10 ans l'apparition d'une dyspnée d'effort progressive,
(dyspnée à un étage).
Plusieurs années après le début de cette dyspnée, la malade a présenté des poussées de surinfections bronchiques
hivernales 2 ou 3 fois par an, associant toux et expectoration purulente, ces poussées cédant à des traitements antibiotiques
variés.
Elle pèse 51 kg pour 1,60m. Il n'y a pas de cyanose ni d'hippocratisme digital ni de dyspnée au repos.
A l'auscultation pulmonaire, le murmure vésiculaire est particulièrement diminué aux bases. Le coeur est régulier, rapide à
100/mn. Il n'y a aucun signe d'insuffisance cardiaque.
L'exploration fonctionnelle respiratoire objective une insuffisance respiratoire obstructive majeure avec un VEMS à 425 ml
(théorique 2020 ml), un coefficient de Tiffeneau à 28 %. Cette insuffisance respiratoire obstructive n'est pas modifiée par les
broncho-dilatateurs. La gazométrie artérielle montre PaO2 à 7,6 kPa (55 mmHg), PaC02 à 5,1 kPa (37 mmHg). Les
bicarbonates plasmatiques sont normaux à 27 mmoles/l. L'hématocrite est à 52 %.

Parmi les diagnostics suivants, lequel retenez-vous ?


A - Bronchite chronique
B - Asthme à dyspnée continue
C - Bronchite chronique avec emphysème centrolobulaire
D - Emphysème panlobulaire
E - Cancer bronchique
Bonne(s) réponse(s) : D

Le diagnostic de bronchite chronique simple est éliminé du fait de l'altération des EFR, polyglobulie, dyspnée majeure.
L'asthme n'est pas retenu car l'obstruction bronchique est irréversible.
Chronologie = dyspnée puis surinfection ainsi que l'absence d'hypercapnie suggère plus un emphysème panlobulaire. Il n'y a
pas d'argument en faveur d'un cancer.
Commentaire : attention, un asthme à dyspnée continue ne peut être éliminé qu'après un test aux corticoïdes qui affirmerait
l'absence totale de réversibilité sous bronchodilatateurs aux EFR : formulation ambiguë.

La radiographie thoracique peut mettre en évidence :


A - Une redistribution de la perfusion vers les lobes supérieurs
B - Une hyperclarté prédominant aux bases
C - Une augmentation de l'espace clair retrosternal
D - De grosses artères pulmonaires
E - Des opacités micro-nodulaires diffuses
Bonne(s) réponse(s) : B C D

Les images observées sont celles d'une distension thoracique liée à l'emphysème.

Quel(s) autre(s) examen(s) pouvez-vous demander ?


A - Exploration de la mécanique ventilatoire (compliance)
B - Mesure du volume résiduel
C - Test de broncho-motricité à l'acétylcholine
D - Radiographie des sinus de la face
E - Electrocardiogramme
Bonne(s) réponse(s) : A B D E

Commentaire : formulation à revoir, on peut toujours demander de nombreux examens, mais mieux vaut savoir lesquels sont
nécessaires !
Seul le test à l'acétylcholine est formellement contre-indiqué puisqu'il ne ferait qu'aggraver l'obstruction s'il était positif.

En dehors des périodes de surinfection, la kinésithérapie respiratoire vous parait indiquée: vous préciserez sur
votre ordonnance au kinésithérapeute la nécessité de :
A - Drainage bronchique postural
B - Rééducation de la ventilation diaphragmatique
C - Augmentation de la ventilation thoracique
D - Apprentissage de la ventilation à basse fréquence
E - Rééducation des muscles abdominaux
Bonne(s) réponse(s) : B C

Il est important de réentraîner les muscles respiratoires : diaphragme et muscles accessoires.


Les abdominaux, quant à eux, sont le plus souvent très développés du fait d'une respiration abdominale.

1013
Exclusivement sur DOC - DZ : www.doc-dz.com NADJI 85
RESIDANAT EN POCHE TOME II
Cas Clinique en QCM

En dehors de la kinésithérapie, le traitement nécessite la prescription de :


A - Oxygénothérapie de longue durée
B - Antibiothérapie lors des surinfections
C - Bêta-2-mimétiques
D - Vaccination anti-grippale annuelle
E - Vaccination anti-pneumoccocique annuelle
Bonne(s) réponse(s) : A B D

Attention : les §2 peuvent être indiqués si le test aux corticoïdes met en évidence une réversibilité partielle de l'obstruction aux
EFR.

Madame P. , âgée de 55 ans est hospitalisée pour dyspnée. Dans ses antécédents on retrouve une phlébite surale droite il y a
3 ans, deux phlébites fémoro-poplitées gauches il y a 18 et 12 mois. Le traitement anti-coagulant a été interrompu il y a 9
mois. Madame P. n'a pas de passé respiratoire.
Depuis deux mois, cette patiente se plaint d'une dyspnée d'effort d'intensité croissante gênant actuellement même la toilette. A
L'examen clinique, le poids est de 80 kg pour 168 cm, la tension artérielle est de 120/80 mm Hg, la température est normale.
On note une cyanose des extrémités avec hippocratisme digital. Il existe des oedèmes des membres inférieurs indolores une
hépatomégalie douloureuse avec reflux hépato-jugulaire et ascite.
L'auscultation pulmonaire est normale. L'auscultation cardiaque indique un bruit de galop droit, un éclat du B2 au foyer
pulmonaire et un souffle systolique au foyer tricuspidien. Vous portez le diagnostic d'insuffisance ventriculaire droite.

Quel(s) signe(s) ne fait (font) pas partie du tableau d'insuffisance ventriculaire droite ?
A - Dyspnée d'effort avec orthopnée
B - Hépatalgies d'effort
C - Souffle systolique tricuspidien
D - Eclat du B2 au 2ème espace inter-costal droit
E - Signe de Harzer
Bonne(s) réponse(s) : A

La dyspnée d'effort avec orthopnée est un signe d'insuffisance ventriculaire gauche.

Quelle est la cause la plus probable du tableau présenté par Madame P. ?


A - Insuffisance tricuspidienne
B - Rétrécissement tricuspidien
C - Coeur pulmonaire chronique post-embolique
D - Hypertension artérielle pulmonaire primitive
E - Insuffisance respiratoire restrictive due à l'obésité
Bonne(s) réponse(s) : C

Les antécédents multiples de phlébite plaident fortement en faveur de cette hypothèse.

Quel(s) examen(s) vous parait (paraissent) utile(s) ?


A - Electrocardiogramme et radiographie du thorax
B - Echographie cardiaque avec doppler pulsé
C - Phlébo-cavographie
D - Scintigraphie pulmonaire
E - Echographie abdomino-pelvienne
Bonne(s) réponse(s) : A B C D E

La radio, l'ECG et l'échocardiographie-doppler permettront d'évaluer la fonction cardiaque droite et le degré d' HTAP.
La phlébocavographie recherche des thrombus récents à traiter.
La scintigraphie confirmera le diagnostic en faveur d'embolies en montrant des défects multiples.
Enfin, l'échographie abdominopelvienne recherchera un obstacle sur la circulation cave inférieure.

Lors d'une étude hémodynamique par cathétérisme droit, les résultats confortant votre hypothèse sont :
A - Pression artérielle pulmonaire systolique : 80 mm Hg
B - Pression systolique ventriculaire droite : 20 mm Hg
C - Pression capillaire : 40 mm Hg
D - Pression dans l'oreillette droite : 0 mm Hg
E - Pression télédiastolique du ventricule droit à 15 mm Hg
Bonne(s) réponse(s) : A E

Sans commentaire.

1014
Exclusivement sur DOC - DZ : www.doc-dz.com NADJI 85
RESIDANAT EN POCHE TOME II
Cas Clinique en QCM

Concernant le traitement, vous pouvez employer dans ce cas :


A - Digitoxine 4 c/jour
B - Héparine 400 mg/jour en intraveineux
C - Furosémide (Lasilix®) 1 cp/jour
D - Un calcium bloqueur
E - Oxygénothérapie par sonde nasale à 10 litres/minute d'emblée
Bonne(s) réponse(s) : B C E

1) Il faut traiter la cause, à savoir l'embolie et surtout prévenir les récidives par Héparine.
2) Il faut traiter symptomatiquement l'hypoxie et l'HTAP par oxygénothérapie, la défaillance cardiaque droite avec des
diurétiques.

Après la phase aiguë quelle(s) attitude(s) pourrez-vous retenir ?


A - Annuloplastie tricuspidienne
B - Digitoxine au long cours
C - Traitement continu par antivitamines K
D - Traitement antiagrégant plaquettaire
E - Oxygénothérapie à domicile
Bonne(s) réponse(s) : C E

Les antivitamines K semblent nécessaires puisque la malade présente des accidents thrombo-emboliques à répétition à l'arrêt
du traitement.
L'oxygène est justifié s'il persiste une hypoxie chronique.

Cette femme de 72 ans, non fumeuse, jusque là bien portante, sans passé respiratoire, présente depuis 8 à 10 jours, une
altération de l'état général avec fièvre à 38°5-39°C, une toux ramenant une expectoration purulente numulaire non fétide.
- Examen clinique : foyer de sous-crépitants sous-claviculaire droit ;
- Radio pulmonaire : opacité alvéolaire lobaire supérieure droite avec image hydroaérique, réalisant un niveau liquide
- Biologie : VS 130, GB 14500/mm3 dont 85% PNN, Hb 9 g/dl.

Le diagnostic d'abcès pulmonaire étant retenu, l'enquête microbiologique doit s'appuyer sur :
A - Hémoculture
B - Sérologie virale
C - Bactériologie des crachats
D - Prélèvements protégés endobronchiques
E - Ecouvillonage pharyngé
Bonne(s) réponse(s) : A C D

Le choix entre bactériologie des crachats et prélèvements endobronchiques protégés dépend des moyens locaux. Certes, le
prélèvement protégé est plus spécifique que l'ECBC, cependant si la malade n'a aucun antécédent respiratoire et se met à
cracher du pus franc, l'existence d'un seul germe dans l'expectoration peut être pris en compte.

Le diagnostic d'abcès pulmonaire impose chez cette malade la recherche de causes favorisantes, lesquelles ?
A - Diabète
B - Mauvais état bucco-dentaire
C - Trouble de déglutition
D - Cancer bronchique à petites cellules
E - Maladie bronchectasique
Bonne(s) réponse(s) : A B C D E

Les causes à rechercher sont de 3 ordres :


- locales = tumeur endobronchique, bronchectasies, corps étranger inhalé,
- locorégionales = porte d'entrée ORL ou dentaire, trouble de déglutition,
- générales = diabète, corticothérapie, foyer infectieux à distance (endocardite droite)...
La proposition D prête à confusion, il faut rechercher un cancer bronchique mais pas forcément "à petites cellules".

1015
Exclusivement sur DOC - DZ : www.doc-dz.com NADJI 85
RESIDANAT EN POCHE TOME II
Cas Clinique en QCM

Quelle prescription antibiotique jugez-vous opportune en cas de négativité de l'enquête infectieuse ?


A - Traitement anti-tuberculeux d'épreuve
B - Cyclines par voie orale
C - Macrolides
D - Pénicilline G injectable en monothérapie
E - Augmentin® + Aminoside
Bonne(s) réponse(s) : E

L'association Augmentin-aminoside couvre la grande majorité des germes responsables d'abcès pulmonaires :
- pneumocoques
- bacilles à Gram négatif
- germes anaérobies
Cependant, certains germes plus rarement en cause y sont résistants. Se méfier du staphylocoque si le traitement est
inefficace.

Après guérison, persiste une image cavitaire lobaire supérieure droite, à paroi fine ; quelle est la principale
complication à craindre ?
A - Surinfection aspergillaire
B - Pneumothorax spontané
C - Surinfection à pyogènes
D - Cancérisation
E - Surinfection tuberculeuse
Bonne(s) réponse(s) : A

Une cavité d'abcès détergée peut faire le lit d'une colonisation aspergillaire aboutissant à la formation d'un aspergillome,
pourvoyeur d'hémoptysies. Si bien qu'il est de règle de surveiller par des radiographies systématiques, périodiques, ces
cavités.

Vous êtes appelé d'urgence auprès d'un homme de 60 ans, agriculteur, tabagique (40 paquets/année),qui a depuis 24 heures
une infection respiratoire aiguë. Celle-ci a débuté par un point de côté brutal de la base thoracique droite, la fièvre s'est
rapidement élevée à 39°C. L'expectoration est minime, muco-purulente. A l'examen clinique, le sujet est en bon état général et
l'examen thoracique montre un syndrome de condensation de la base droite.

Vous évoquez en première hypothèse le diagnostic de pneumonie à :


A - Pneumocoque
B - Escherichia coli
C - Haemophilus influenzae
D - Mycoplasma pneumoniae
E - Staphylocoque
Bonne(s) réponse(s) : A

Il s'agit d'un tableau typique de pneumonie franche lobaire aiguë à pneumocoque.

Parmi les signes cliniques suivants, vous vous attendez à trouver à l'examen de ce patient :
A - Un herpès labial
B - Une adénopathie sus-claviculaire droite
C - Des râles crépitants
D - Un souffle tubaire
E - Une matité de la base droite
Bonne(s) réponse(s) : A C D E

L'herpès est fréquent bien qu'inconstant et non spécifique. Il peut avoir une localisation autre que nasolabial (périorificiel en
général).

Parmi les signes radiologiques thoraciques suivants, vous pouvez observer :


A - Une radiographie strictement normale
B - Une opacité de type alvéolaire triangulaire à sommet hilaire de la base droite
C - Des opacités bilatérales polysegmentaires
D - Des adénopathies hilaires bilatérales
E - Des opacités de type interstitiel plurisegmentaires
Bonne(s) réponse(s) : A B

- La radiographie pulmonaire peut être normale si elle est réalisée précocement (moins de 24 heures après la douleur).
- L'image la plus typique et de loin la plus fréquente est une opacité alvéolaire systématisée non rétractile, intéressant un ou
plusieurs lobes.

1016
Exclusivement sur DOC - DZ : www.doc-dz.com NADJI 85
RESIDANAT EN POCHE TOME II
Cas Clinique en QCM

L'antibiotique dont la prescription est totalement inadaptée chez ce patient est :


A - Pénicilline G
B - Amoxicilline
C - Gentamicine
D - Erythromycine
E - Norfloxacine
Bonne(s) réponse(s) :

QUESTION ANNULEE.

Quelle est l'évolution clinique observée usuellement après mise en route rapide d'un traitement antibiotique
adapté chez un tel patient ?
A - Apparition d'un pyopneumothorax
B - Nettoyage radiologique et défervescence thermique en 24 heures
C - Défervescence thermique en 2 à 3 jours et nettoyage radiologique en 15 jours - 3 semaines
D - Apparition d'une miliaire signant un oedème aigu lésionnel
E - Apparition d'une obnubilation avec méningoencéphalite
Bonne(s) réponse(s) : C

Le pneumocoque est très sensible aux antibiotiques tels que les pénicillines et les macrolides. Si l'apyrexie est obtenue en 24
à 48 heures, le nettoyage radiologique nécessite 2 à 3 semaines. Les complications locales et générales sont exceptionnelles
si le traitement adapté est entrepris d'urgence.

Après 4 semaines, la radiographie du thorax montre la persistance d'une image trabéculaire de la base droite
avec une opacité du hile droit. Parmi les examens ci-dessous, lequel faut-il demander en priorité ?
A - Une ponction transtrachéale pour examen bactériologique
B - Un scanner thoracique
C - Une fibroscopie bronchique
D - Une scintigraphie pulmonaire
E - Des recherches répétées de BK dans le liquide de tubage gastrique
Bonne(s) réponse(s) : C

Chez cet homme de 60 ans, fumeur, l'absence de nettoyage complet de la radiographie doit faire craindre un cancer
bronchique en premier lieu.

Cette femme de 82 ans, à l'état général bien conservé, non tabagique, originaire du sud-algérien qu'elle a quitté 30 ans
auparavant sans plus y retourner, opérée d'un adéno-cancer colique en 1978, subit en juillet 1986 un contrôle radiologique
pulmonaire motivé par un syndrome infectieux fébrile pulmonaire avec toux sèche non productive, ni hémoptoïque.
L'on découvre à la fois une opacité dense, grossièrement arrondie à contour polybé, de 6x 7 cm de la pyramide basale gauche
et un volumineux goitre cervico-thoracique mobile à la déglutition, connu depuis 50 ans, discrètement compressif (dysphagie
depuis 6 mois).
La TA est à 16/9, l'ECG normal avec rythme sinusal. Des examens complémentaires n'est à retenir qu'une VS à 35 mm à la
première heure. L'ACE est augmenté.

Quel diagnostic vous paraît le plus vraisemblable devant cette opacité arrondie ?
A - Kyste bronchogénique
B - Tumeur maligne primitive
C - Tumeur d'origine métastatique
D - Kyste hydatique
E - Aspergillome
Bonne(s) réponse(s) : C

Du fait de l'aspect radiologique irrégulier, de l'antécédent de cancer colique, de l'augmentation de l'ACE.

Quels examens vous paraissent pertinents pour établir le diagnostic ?


A - Fibroscopie bronchique
B - Ponction-biopsie thyroïdienne
C - Cyto diagnostic des sécrétions bronchiques
D - Médiastinoscopie
E - Ponction biopsie transthoracique
Bonne(s) réponse(s) : A E

Les deux examens qui pourront apporter une histologie, donc un diagnostic formel, sont la fibroscopie et la biopsie
transpariétale.
La cytologie même si elle est suspecte n'apportera pas le diagnostic formel.

1017
Exclusivement sur DOC - DZ : www.doc-dz.com NADJI 85
RESIDANAT EN POCHE TOME II
Cas Clinique en QCM

Dans l'hypothèse d'échec de ces examens à visée diagnostique, que proposez-vous comme conduite
thérapeutique si le reste de l'examen le permet ?
A - Chimiothérapie antimitotique
B - Radiothérapie
C - Association radio + chimiothérapie
D - Thoracotomie
E - Abstention thérapeutique
Bonne(s) réponse(s) : D

La thoracotomie permettra le diagnostic et un geste d'exérèse thérapeutique si la métastase est unique. Dans le même temps
pourra être réalisée une exérèse du goître symptomatique.

Si l'indication d'une intervention chirurgicale devait être discutée, quelles investigations préalables vous
paraîtraient indispensables ?
A - Exploration Fonctionnelle Respiratoire (EFR)
B - Scintigraphie pulmonaire de perfusion
C - Recherche d'autres métastases (foie, squelette...)
D - Colonoscopie
E - Cathétérisme cardiaque
Bonne(s) réponse(s) : A C D

Il convient en préopératoire d'évaluer :


1) le pronostic de la maladie : néoplasique = recherche de récidive locale par coloscopie, de métastases à distance,
2) d'évaluer la fonction respiratoire pour s'assurer qu'une exérèse est possible.

Un insuffisant respiratoire, 60 ans, est. adressé en urgence pour décompensation respiratoire aiguë. Antécédents : BPCO
(Broncho Pneumopathie Chronique Obstructive).
L'histoire de la maladie débute trois jours avant l'hospitalisation par une dyspnée qui s'est majorée brutalement, une angoisse
avec douleur basithoracique gauche aux mouvements respiratoires forcés, disparue spontanément.
A l'examen clinique :
- Une toux avec expectoration muco-purulente de faible abondance
- Un syndrome fébrile à 38 degrés C
- Une cyanose des extrémités
L'auscultation objective des ronchus disséminés, une tachycardie. On retrouve une hépatomégalie avec reflux hépato-
jugulaire et oedème des membres inférieurs.
En urgence vous disposez :
- ECG: signes de coeur pulmonaire
- Hb = 18,9 g/dl et hyperleucocytose
- lonogramme sanguin : Na 130 mEq/l, Chlore 96 mEq/l
- lonogramme urinaire : Na 56 mEq/l, Chlore 27 mEq/l
- Gaz du sang :
pH 7,28
PaCO2 = 3 5 mmHg
Pa O2 = 45 mmHg
Bicarbonates 17 mEq/l
C02 total 18, 5 mEq/l
Base excès - 10 mEq/l
SaO2 75 %.

Les gaz du sang objectivent :


A - Une acidose respiratoire décompensée
B - Une acidose métabolique décompensée
C - Un effet shunt
D - Un effet espace mort
E - Une acidose mixte
Bonne(s) réponse(s) : B C

L'effet shunt est défini par l'association d'hypoxie et d'hypocapnie. L'acidose métabolique est authentifiée par la chute des
bicarbonates et l'hypocapnie. Elle est décompensée puisque le pH est à 7,28.

Quel est (ou quels sont) le (ou les) mécanisme(s) physiopathologique(s) qui peut (peuvent) avoir entraîné de
telles perturbations de PaO2 et CO2 ?
A - Un effet espace mort par encombrement
B - Un effet espace mort par pneumothorax
C - Un effet shunt par pneumonie
D - Un effet shunt en relation avec une embolie pulmonaire
E - Une intoxication au CO
Bonne(s) réponse(s) : C D

L'effet shunt peut être lié à une pneumonie ou à une embolie pulmonaire chez ce patient fébrile, avec expectoration purulente,
qui a présenté une douleur brutale, angoissante, quelques jours avant.

1018
Exclusivement sur DOC - DZ : www.doc-dz.com NADJI 85
RESIDANAT EN POCHE TOME II
Cas Clinique en QCM

Avant de disposer de la radiographie thoracique, quel(s) diagnostic(s) peuvent être évoqués ?


A - Embolie pulmonaire
B - Pneumothorax
C - Hypoventilation par encombrement
D - Infarctus du myocarde sans insuffisance cardiaque
E - Pneumonie
Bonne(s) réponse(s) : A E

L'existence d'une douleur doit faire rechercher une pneumonie, une embolie. Le pneumothorax n'est pas vraisemblable car il
aurait plutôt entraîné une hypercapnie. Quant à l'infarctus, 3 jours après la douleur, l'ECG serait parlant. De plus, il ne peut
entraîner de décompensation respiratoire sans passer par une décompensation cardiaque gauche.

La radiographie de thorax ne met en évidence que des images de BPCO avec emphysème. Quels examens
vous paraissent les plus importants dans l'immédiat ?
A - Epreuves fonctionnelles respiratoires
B - Fibroscopie bronchique avec biopsies
C - Phlébocavographie ou écho doppler des membres inférieurs
D - Angiographie pulmonaire
E - Hémocultures
Bonne(s) réponse(s) : C D

Si la radiographie pulmonaire est normale, le diagnostic de pneumonie est exclu ; il ne reste plus que celui d'embolie
pulmonaire qu'il faut affirmer par :
- une angiographie,
- voire une phlébocavographie car l'angiographie peut être d'interprétation difficile, surtout chez un bronchopathe chronique
emphysémateux.

Ces examens confirment votre diagnostic, mais objectivent en plus un risque imminent d'aggravation de l'état
cardio-respiratoire. Quelle(s) thérapeutique (s) vous paraît (paraissent) la (les) mieux adaptée(s) ?
A - Calciparine sous-cutanée
B - Héparine en continu à la pompe
C - Association de deux antibiotiques
D - Oxygénothérapie par sonde nasale
E - Digoxine
Bonne(s) réponse(s) : B D

Le traitement de l'embolie pulmonaire récente est l'héparinothérapie en continu à la seringue électrique à dose efficace,
adapté au TCK. C'est le traitement préventif des récidives en situation aiguë. L'hypoxie impose une oxygénothérapie et contre-
indique la digoxine. Un antibiotique serait à conseiller mais pas une association de principe.

Vous êtes appelé d'urgence auprès d'un homme de 35 ans, éthylique connu, qui présente depuis 48 heures un syndrome
respiratoire aigu, fébrile et tussigène.
Celui-ci a débuté brutalement par un point de côté violent, basi-thoracique droit. La température s'est rapidement élevée à
40°C, avec sensation de malaise et apparition de grands frissons. Ce patient n'a pas d'antécédents de tuberculose, il n'a pas
de diabète. L'étude des gaz du sang artériel permet de constater: PaO2 = 60 mmHg, PaC02 = 35 mmHg, pH = 7,50,
bicarbonates 27 mEq/l.
Les hémocultures sont positives pour Streptococcus pneumoniae. La radiographie thoracique permet de constater une
opacité systématisée du lobe supérieur droit sans atteinte pleurale.

Dans le creux axillaire droit vous devez rechercher :


A - Abolition des vibrations vocales
B - Tympanisme
C - Râles crépitants
D - Souffle tubaire
E - Râles sibilants
Bonne(s) réponse(s) : C D

Le syndrome de condensation alvéolaire comporte, quand il est complet :


- abolition du murmure vésiculaire,
- matité,
- augmentation des vibrations vocales,
- râles crépitants et souffle tubaire.

1019
Exclusivement sur DOC - DZ : www.doc-dz.com NADJI 85
RESIDANAT EN POCHE TOME II
Cas Clinique en QCM

Parmi les propositions suivantes, quelle est celle qui permet d'expliquer les résultats des gaz du sang artériel ?
A - Hypoventilation alvéolaire globale
B - Troubles du transfert des gaz
C - Effet shunt
D - Hypoperfusion lobaire
E - Effet espace mort
Bonne(s) réponse(s) : B C

Seul un trouble de la diffusion alvéolocapillaire ou un effet shunt permet d'expliquer l'hypoxie associée à l'hypocapnie. Les
autres propositions s'accompagnerait d'hypercapnie.

Cette infection à Streptococcus pneumoniae a actuellement en France plus de 95 % de chance d'être sensible à
:
A - Amoxicilline
B - Erythromycine
C - Pénicilline G
D - Gentamycine
E - Doxycycline
Bonne(s) réponse(s) : A B C

La résistance du pneumocoque aux pénicillines est inférieure à 1 % et la résistance aux macrolides bien que plus élevée est
inférieure à 5 % (4 % en France - Registre 1985).

Parmi les complications possibles de cette pneumonie à pneumocoque, vous devez rechercher ici :
A - Anémie hémolytique
B - Pleurésie purulente
C - Sinusite
D - Méningite purulente
E - Dermite pluriorificielle de Stevens Johnson
Bonne(s) réponse(s) : C D

Chez ce patient éthylique chronique, il faudrait se méfier particulièrement d'une dissémination hématogène du pneumocoque
et en premier lieu d'une atteinte méningée. La sinusite est à rechercher de principe comme porte d'entrée à une
pneumococcie.

Au 14ème jour de traitement, le malade est apyrétique. La radio montre toujours un foyer du lobe supérieur. Dès
lors il faut :
A - Donner l'antibiotique par voie injectable pour être certain de la prise
B - Ajouter un deuxième antibiotique
C - Changer l'antibiothérapie
D - Poursuivre les mêmes antibiotiques pour 15 jours supplémentaires
E - Arrêter les antibiotiques
Bonne(s) réponse(s) : E

Si la malade est apyrétique après 2 semaines, le traitement peut être interrompu, même si la radiographie n'est pas totalement
nettoyée. Mais une surveillance s'impose.
- Si la fièvre réapparait, il faut en rechercher la cause.
- Si la radiographie ne se nettoie pas, il faut réaliser une fibroscopie.

1020
Exclusivement sur DOC - DZ : www.doc-dz.com NADJI 85
RESIDANAT EN POCHE TOME II
Cas Clinique en QCM

1021
Exclusivement sur DOC - DZ : www.doc-dz.com NADJI 85
RESIDANAT EN POCHE TOME II
Cas Clinique en QCM
Un homme de 54 ans est hospitalisé pour un syndrome fébrile ayant débuté brutalement par des frissons, des céphalées, une
ascension thermique à 40, 2°C. La famille signale un éthylisme chronique certain, une allergie connue aux bêta-lactamines.
Nous observons une toux grasse, ramenant une expectoration rouillée, une dyspnée peu importante avec tachypnée. La
température est à 39,9°C.
L'examen retrouve une légère cyanose, une sub-matité de la base droite bien perçue en arrière au sein de laquelle est perçu
un souffle tubaire typique entouré d'une couronne de râles sous-crépitants fins.
Un premier bilan biologique en urgence donne :
Numération formule hémo-leucocytaire : pas d'anémie, 13.800 leucocytes/mm3 avec 84% de polynucléaires neutrophiles.
Gaz du sang artériel, Pa02 68 mmHg, PaC02 35 mmHg, pH 7,48 SaO2 90 %.
Dans les heures qui suivent apparaissent deux nouveaux éléments cliniques: un herpès péri-buccal et des troubles neuro-
psychiques : tremblements logorrhée, agressivité, délire.

Le cliché pulmonaire met en évidence une de ces anomalies :


A - Opacités floues disséminées dans les deux champs pulmonaires
B - Atélectasie du lobe moyen
C - Image hydroaérique du segment apical du lobe inférieur droit
D - Opacité systématisée non rétractile du lobe inférieur droit
E - Une miliaire
Bonne(s) réponse(s) : D

Tableau de pneumonie franche lobaire aiguë, faisant rechercher une opacité en foyer systématisé, non rétractile de
topographie LID (sur les signes physiques).

Les désordres neuro-psychiques observés sont dûs à une de ces causes, laquelle ?
A - Insuffisance respiratoire aiguë
B - Encéphalopathie alcoolique aiguë de type Delirium tremens
C - Méningite
D - Septicémie
E - Importance du décalage thermique
Bonne(s) réponse(s) : B

Le tableau neurologique d'agitation délirante est caractéristique de Delirium Tremens chez cet éthylique chronique en sevrage.

Une antibiothérapie.doit être immédiatement prescrite, avec quel antibiotique ?


A - Péni G
B - Macrolide
C - Céphalosporine
D - Aminoside
E - Cycline
Bonne(s) réponse(s) : B

Les macrolides sont efficaces dans plus de 95 % des cas sur le pneumocoque. Les pénicillines (et les céphalosporines) sont à
exclure du fait d'une allergie connue aux bêta-lactamines.

Le traitement complémentaire comprendra :


A - Corticothérapie
B - Réhydratation par voie veineuse
C - Equanil injectable
D - Oxygénothérapie nasale
E - Aucune de ces mesures
Bonne(s) réponse(s) : B C

Ces mesures visent à traiter le Delirium Tremens. L'oxygénothérapie n'est pas indispensable car une PaO2 à 68 % ne
représente pas une hypoxie importante, mais elle peut être prescrite.

1022
Exclusivement sur DOC - DZ : www.doc-dz.com NADJI 85
RESIDANAT EN POCHE TOME II
Cas Clinique en QCM

1023
Exclusivement sur DOC - DZ : www.doc-dz.com NADJI 85
RESIDANAT EN POCHE TOME II
Cas Clinique en QCM
Une femme de 40 ans vient consulter pour une hypoacousie bilatérale progressivement croissante. Elle est mère de trois
enfants. Elle n'a jamais présenté d'antécédents de surdité, ni d'otite, et n'a jamais eu de pathologie médicale particulière (en
particulier, pas de prise d'ototoxique). La malade allègue des bourdonnements graves, intermittents, de plus en plus fréquents
au niveau des deux oreilles.
A l'examen, les deux tympans sont strictement normaux. L'épreuve de Valsalva est positive des deux côtés. L'épreuve de
Weber montre une absence de latéralisation. Le Rinne est négatif des deux côtés.
Le diagnostic le plus probable est celui d'otospongiose ou d'ankylose stapédovestibulaire.

L'audiométrie tonale réalisée montre de chaque côté interrogé :


A - Un accolement des courbes aériennes et osseuses
B - Une perte en décibels sur la conduction aérienne, plus importante qu'en conduction osseuse
C - Une chute de l'audition uniquement sur les fréquences aiguës
D - Une perte en décibels sur la conduction osseuse, plus importante qu'en conduction aérienne
E - Une encoche sur la fréquence 2000 Hz
Bonne(s) réponse(s) : B E

B - L'otospongiose est une surdité de transmission à tympan normal.


E - C'est la classique encoche de Carhart.

Parmi les examens suivants, lequel est inutile au diagnostic d'otospongiose ?


A - Impédancemétrie
B - Audiométrie tonale
C - Audiométrie vocale
D - Etude des réflexes stapédiens
E - Etude des potentiels évoqués auditifs
Bonne(s) réponse(s) : E

L'otospongiose est liée à une dystrophie osseuse localisée fixant la platine de l'étrier dans la fenêtre ovale. Les P.E.A.
trouvent leur intérêt pour le diagnostic des atteintes perceptives.

Devant ces données cliniques, on peut retenir comme diagnostic différentiel plausible :
A - Une otite séreuse bilatérale
B - Une presbyacousie précoce
C - Une maladie de Ménière bilatérale
D - Une tumeur du tronc cérébral
E - Une tympanosclérose
Bonne(s) réponse(s) : E

A - Dans l'otite séreuse on constate une rétraction du tympan, et chez l'adulte cette otite doit faire rechercher une tumeur du
cavum.
B C D - Sont des surdités de perception cochléaire et rétrocochléaire.
E - Processus irréversible, aboutissement d'otite chronique qui entraîne une métaplasie de la muqueuse de l'oreille moyenne
avec sclérose et granulome à cholestérine. Le tympan est rétracté, de couleur pourpre.

Pour affirmer le blocage de l'étrier dans la fosse ovale, quel est l'examen le plus fiable ?
A - Audiométrie tonale
B - Audiométrie vocale
C - Recherche du réflexe stapédien
D - Sonomanométrie
E - Epreuve de Rinne
Bonne(s) réponse(s) : C

Ce blocage empêche le réflexe stapédien.

Le traitement de l'otospongiose dans ce cas est :


A - Médical par les vasodilatateurs
B - Prothétique
C - Médical par le calcium
D - Médical par le magnésium
E - Chirurgical
Bonne(s) réponse(s) : E

Le traitement est bien sur chirurgical. Au cours de l'intervention on remplace l'étrier et sa


platine par une prothèse entre enclume et fenêtre ovale, généralement un piston synthétique.
B - Ce terme désigne généralement la prothèse auditive.

1024
Exclusivement sur DOC - DZ : www.doc-dz.com NADJI 85
RESIDANAT EN POCHE TOME II
Cas Clinique en QCM
Un homme de 45 ans d'origine maghrébine, consulte pour l'apparition récente d'une tuméfaction bilatérale cervicale haute
apparue depuis quelques semaines. L'état général du patient est excellent. Depuis 10 jours, il présente une plénitude
auriculaire droite avec une sensation d'hypoacousie droite. La rhinoscopie antérieure montre des fosses nasales libres

Quel diagnostic vous parait le plus probable sur ce récit ?


A - Otite aiguë
B - Kyste congénital du cou
C - Adénopathie inflammatoire de cause générale
D - Tumeur du cavum
E - Aucune des propositions ci-dessus
Bonne(s) réponse(s) : D

Les arguments devant y faire penser sont :


- d'abord l'origine maghrébine, car l'Afrique du nord constitue une zone à risque de cancer du cavum.
- l'existence d'adénopathie cervicale haute, le cancer du cavum étant très lymphophile, le drainage se faisant vers la partie
supérieure des chaînes jugulaires et spinales
- Enfin, devant l'existence de l'otite séreuse chez l'adulte.

En faveur de ce diagnostic vous retenez essentiellement :


A - Le bilan ionique
B - Le tabagisme
C - La présence du virus Epstein Barr
D - L'hémogramme
E - L'ensemble de ces éléments
Bonne(s) réponse(s) : C

Il y a association très étroite entre carcinome du cavum et virus Epstein-Barr, à tel point qu'une sérologie remontant après
traitement est en faveur d'une récidive.

Parmi ces examens, deux sont susceptibles d'orienter le diagnostic de la lésion primitive. Lesquels ?
A - Scintigraphie salivaire
B - Tomographies du massif facial en incidence de Hirtz
C - Cliché dentaire panoramique
D - Echographie
E - Impédancemétrie
Bonne(s) réponse(s) : B E

B - Montre l'obstruction tumorale du cavum et une éventuelle lyse osseuse de la base du


crâne.
E - L'impédancémétrie montrant une courbe décalée dans les pressions négatives, traduit une obstruction de la troupe
d'Eustache par la tumeur.

La thérapeutique curatrice la plus adéquate comporte :


A - Chirurgie
B - Hormonothérapie
C - Chimiothérapie
D - Corticothérapie
E - Radiothérapie externe
Bonne(s) réponse(s) : C E

Le cancer du cavum est radiosensible et repose donc sur la radiothérapie externe. Mais les progrès de la chimiothérapie en
font un traitement efficace. La chirurgie n'a sa place que pour l'exérèse de reliquats ganglionnaires persistant 3 mois après la
fin de la radiothérapie.

1025
Exclusivement sur DOC - DZ : www.doc-dz.com NADJI 85
RESIDANAT EN POCHE TOME II
Cas Clinique en QCM
Un patient de 50 ans, marchand forain, fumeur modéré, présente depuis trois mois une dysphonie quasi-permanente, non
régressive, avec voix rauque et douleurs pharyngo-laryngées en fin de journée. Il existe d'une façon concomitante une toux,
mais pas de dysphagie, ni d'otalgie. Ce patient n'a jamais présenté auparavant de dysphonie.
La palpation cervicale est normale. La radiographie pulmonaire, la numération formule sanguine, la vitesse de sédimentation
sont normales.

Devant cette dysphonie, il faut évoquer en premier lieu :


A - Une atteinte des cordes vocales
B - Une lésion épiglottique
C - Une lésion de la margelle laryngée
D - Une sténose trachéale
E - Une compression récurrentielle par une tumeur thyroïdienne
Bonne(s) réponse(s) : A

A - La dysphonie est le signe d'appel du carcinome des cordes vocales dont le tabac est le facteur de risque principal.
B - Ne donne pas toujours une dysphonie, parfois une dysphagie.
C - Le terrain est plutôt alcoolo-tabagique. Le début se fait souvent par des douleurs pharyngées, laryngées.
D - La dyspnée serait au premier plan.
E - La voix est bitonale.

Le moyen le plus simple pour objectiver la lésion causale est représenté par :
A - La fibroscopie bronchique
B - Une laryngoscopie directe en suspension
C - Une laryngoscopie indirecte au miroir laryngé
D - Un examen stroboscopique pour visualiser le mouvement cordal
E - Une laryngographie lipiodolée
Bonne(s) réponse(s) : C

C'est le moyen le plus simple nécessitant parfois une anesthésie locale en raison des réflexes souvent importants chez le
fumeur. La laryngoscopie directe en suspension est l'étape suivante indispensable, permettant la biopsie. Elle se pratique
sous anesthésie générale ou neuroleptanalgésie.

Parmi les affections suivantes, 2 doivent être évoquées d'abord. Lesquelles ?


A - Laryngite aiguë
B - Tuberculose laryngée
C - Paralysie récurrentielle gauche
D - Cancer laryngé
E - Dysplasie laryngée
Bonne(s) réponse(s) : D E

La dysphonie persistante chez un fumeur doit évoquer d'abord un cancer laryngé.


La dysplasie laryngée fait le lit du cancer ; l'arrêt du tabac peut faire régresser cette lésion.

Quel est l'élément essentiel qui permettra le diagnostic exact de cette affection ?
A - Tomographies du larynx de face
B - Biopsie laryngée
C - Recherche de bacille de Koch dans les crachats
D - Xérographie de profil du larynx
E - Cytologie du larynx
Bonne(s) réponse(s) : B

Pouvant seule affirmer le caractère malin de la lésion. Elle est indispensable avant toute thérapeutique.

Si l'on réalise une biopsie exérèse de cette lésion et que celle-ci soit histologiquement bénigne, que conseillez-
vous au malade ?
A - De moins surmener le larynx
B - D'arrêter le tabac
C - Une surveillance laryngée très régulière
D - Des corticoïdes par voie générale
E - Une intervention chirurgicale
Bonne(s) réponse(s) : A B C

A - Le surmenage vocal peut être à l'origine de nodules des cordes vocales.


B - L'arrêt des facteurs irritatifs permet de faire régresser la dysplasie.
C - Au moins deux fois par an tant que les lésions persistent.

1026
Exclusivement sur DOC - DZ : www.doc-dz.com NADJI 85
RESIDANAT EN POCHE TOME II
Cas Clinique en QCM

Si au contraire, la biopsie répond carcinome spinocellulaire infiltrant, alors que la lésion est unilatérale mesure
un demi-centimètre carré et que le larynx est parfaitement mobile, on peut proposer :
A - Une surveillance au miroir laryngé tous les mois
B - Une laryngoscopie en suspension 3 mois plus tard pour biopsie
C - Une séance de laser
D - Une chirurgie partielle conservatrice de la voix
E - Une laryngectomie totale
Bonne(s) réponse(s) : D

A.B - Le diagnostic du carcinome fait, il ne faut pas attendre pour traîter, car l'évolution locale de la tumeur pourrait contre-
indiquer une chirurgie partielle, et assombrir le pronostic.
D - Cordectomie simple si la lésion est à distance de la commissure antérieure ou du processus vocal de l'aryténoïde.
E - La laryngectomie totale est totalement disproportionnée pour une lésion cordale avec mobilité laryngée conservée.

Un homme de 50 ans, grand fumeur, présente une discrète gêne pharyngée depuis quelques mois. Il a constaté récemment
l'apparition d'une tuméfaction cervicale sous digastrique, haute, dure, ligneuse, mobile. L'examen laryngoscopique montre une
lésion ulcéro-bourgeonnante occupant le sinus piriforme avec fixité de l'hémilarynx.

S'il s'agit d'un cancer, quel est le type histologique le plus fréquemment rencontré ?
A - Un épithélioma malpighien spinocellulaire
B - Epithélioma glandulaire
C - Un fibrosarcome
D - Un lymphome malin non hodgkinien
E - Une tuberculose
Bonne(s) réponse(s) : A

Evident.

Le bilan d'extension local de cette lésion comporte :


A - Une radiographie des poumons face
B - Un transit pharyngo-oesophagien
C - Une radiographie de profil et des tomographies de larynx
D - Une fibroscopie bronchique
E - Une hypopharyngo-laryngoscopie directe
Bonne(s) réponse(s) : B C E

A D - Le bilan local exclut la fibroscopie bronchique et la radiographie pulmonaire.


B - Le transit pharyngo-oesophagien peut être utile pour visualiser notamment une extension vers la bouche oesophagienne.
C - Ce bilan radiologique local pouvant montrer une extension vers l'endolarynx.
E - L'hypopharyngo-laryngoscopie directe est bien sur indispensable.

Quel(s) geste(s) thérapeutique(s) doi(ven)t être retenu(s) chez ce malade en bon état général ?
A - Chimiothérapie seule
B - Radiothérapie exclusive
C - Laryngectomie partielle
D - Pharyngolaryngectomie totale avec curage et radiothérapie postopératoire
E - Curage ganglionnaire et radiothérapie
Bonne(s) réponse(s) : D

L'envahissement ganglionnaire est particulièrement fréquent dans cette localisation et justifie le curage ganglionnaire de
principe. La fixité de l'hémilarynx contre-indique toute chirurgie partielle.
La radiothérapie post-opératoire améliore les taux de survie.
La radiothérapie exclusive est moins bonne.
La place de la chimiothérapie reste à définir.

Si le traitement de ce patient comporte une irradiation, quel(s) soin(s) doi(ven)t être impérativement pratiqué(s) ?
A - Avulsion des dents cariées ou traitées
B - Confection de gouttières fluorées en cas de dents saines
C - Mise en place d'une sonde naso-oesophagienne
D - Gastrostomie
E - Corticothérapie
Bonne(s) réponse(s) : A B

A - Risque d'ostéoradionécrose.
B - Les gouttières fluorées sont prophylactiques des caries.
C D - La reprise de l'alimentation per os en l'absence de complication se fait vers le 10-12ème jour post-opératoire.

1027
Exclusivement sur DOC - DZ : www.doc-dz.com NADJI 85
RESIDANAT EN POCHE TOME II
Cas Clinique en QCM

Si un traitement chirurgical est retenu, que faut-il conseiller au malade ?


A - Rééducation orthophonique
B - Massage et rééducation de l'épaule
C - Soins de canule quotidiens
D - Surveillance spécialisée régulière
E - Suppression de toute boisson alcoolisée
Bonne(s) réponse(s) : A C D E

A - Le larynx étant sacrifié, cette rééducation doit permettre au malade d'utiliser une voix
oesophagienne.
C - Ceci doit être appris au malade, qui doit se prendre en charge dès la période d'hospitalisation.
D - Très importante, pour surveiller l'absence de récidive ou de deuxième localisation.
E - Evident, la poursuite de l'intoxication expose à d'autres localisations.
Enfin la réponse à l'item B est plus difficile et discutable. En effet si l'évidement ganglionnaire homolatéral complet en
monobloc avec la tumeur est systématique, il peut être fait sur un mode radical ou conservateur. Dans l'énoncé il est précisé
que l'adénopathie est mobile. L'évidement complet sur un mode conservateur, respectant le nerf spinal, est possible. Les
séquelles de l'épaule ne sont pas certaines.

Un menuisier de 64 ans présente depuis plus de 6 mois une obstruction nasale gauche, progressive, survenue au décours
d'un rhume. Il présente de plus une anosmie, sauf pour les odeurs très fortes. Il présente de temps à autre des céphalées
frontales en barre, calmées par de l'aspirine, et mouche de temps à autre du pus strié de sang.
Ce patient n'a pas de terrain allergique connu. Il fume 1 paquet de cigarettes par jour. Il avait été traité 5 ans auparavant pour
une sinusite maxillaire gauche.

Quels sont les deux éléments cliniques qui doivent particulièrement attirer votre attention ?
A - L'âge du patient
B - Les maux de tête
C - L'obstruction nasale unilatérale
D - Les antécédents de sinusite maxillaire
E - La rhinorrhée purulente striée de sang
Bonne(s) réponse(s) : C E

B - Ces céphalées, par leur intermittence et leur sensibilité aux antalgiques habituels n'ont pas de caractère spécifique.
D - Les antécédents de sinusite maxillaire ne sont pas à retenir. Le terrain (travailleur du bois) non proposé ici est un élément
très évocateur.

Quel examen faut-il réaliser en tout premier lieu ?


A - Radiographie en incidence de Blondeau
B - Une olfactométrie
C - Une rhinoscopie antérieure
D - Une rhinomanométrie (mesure des pressions des fosses nasales)
E - Un bilan allergologique
Bonne(s) réponse(s) : C

Comme toujours c'est l'examen clinique qui doit être fait en premier lieu. La rhinoscopie antérieure met en évidence la tumeur
dans les fosses nasales.

Les éléments dont vous disposez vous permettent d'évoquer le diagnostic de cette affection. Quel est-il parmi
les propositions suivantes ?
A - Polypose nasale
B - Fibrome nasopharyngien
C - Cancer éthmoïdo-maxillaire
D - Rhinite chronique congestive
E - Sinusite maxillaire gauche
Bonne(s) réponse(s) : C

L'exposition (professionnelle) aux poussières de bois constitue le facteur de risque d'adénocarcinome de l'ethmoïde. Le
fibrome nasopharyngien concerne le garçon dans la période péri-pubertaire.

Avec quel examen confirmer le diagnostic ?


A - Biopsie à la pince mors-cuvette
B - Cytologie des sécrétions nasales
C - Tomographies de face et des sinus
D - Scannographie
E - Sinusoscopie
Bonne(s) réponse(s) : A

Le diagnostic est anatomo-pathologique.

1028
Exclusivement sur DOC - DZ : www.doc-dz.com NADJI 85
RESIDANAT EN POCHE TOME II
Cas Clinique en QCM

Sur quel(s) signe(s) clinique(s) évoquez-vous la gravité de cette affection ?


A - L'anosmie
B - Les céphalées
C - La rhinorrhée
D - L'obstruction nasale
E - L'âge du patient
Bonne(s) réponse(s) : A

A - Fait craindre un envahissement de la base du crâne au niveau de la lame criblée de l'ethmoïde où passent les fibres du
nerf olfactif.
B - Les céphalées, lorsqu'elles sont importantes, résistantes aux antalgiques usuels, plus ou moins permanentes, évoquent
une extension aux méninges. Ici elles semblent modérées, pouvant s'expliquer par le syndrome obstructif nasal.

Quel est actuellement l'examen qui permet de préjuger de l'extension de l'affection ?


A - Fond d'oeil
B - Rhinoscopie postérieure
C - Scanner
D - Tomographies des sinus de face
E - Palpation des aires ganglionnaires cervicales
Bonne(s) réponse(s) : C

Il permet de visualiser la tumeur osseuse, les lyses osseuses des parois orbitaires notamment, un éventuel envahissement
cérébral.

Quelle est l'histologie la plus fréquente de cette affection ?


A - Carcinome spinocellulaire très différencié
B - Carcinome glandulaire
C - Lymphome malin non hodgkinien
D - Adénome pléomorphe
E - Chémodectome
Bonne(s) réponse(s) : B

A - Les carcinomes des autres sinus sont le plus souvent des carcinomes spinocellulaires.
D - Le plus souvent localisé à la parotide.
E - Le glomus carotidien se situe à la face interne de la bifurcation carotidienne et peut donne lieu à une tumeur, le
chémodectome ou paragangliome non chromaffine.

Quelle est ou quelles sont la ou les proposition(s) qui s'applique(nt) à cette affection ?
A - Maladie liée au sexe
B - Maladie favorisée par les sinusites chroniques
C - Maladie des travailleurs du bois
D - Infection reconnue au tableau des maladies professionnelles
E - Maladie favorisée par le tabagisme chronique
Bonne(s) réponse(s) : C D

Le rôle du tabagisme associé n'est pas prouvé.

Un patient de 60 ans vient consulter pour une otalgie gauche, intermittente, volontiers rythmée par la déglutition, et dont le
malade évalue l'ancienneté à trois/quatre mois.
Ce patient est un ancien pilote d'aviation militaire. Il n'a jamais fumé et ne présente pas d'intoxication alcoolique particulière.
Il a l'impression de moins entendre de l'oreille gauche, et accuse quelques acouphènes graves, également intermittents. Par
ailleurs, le malade n'allègue aucun autre symptôme clinique O.R.L.
L'examen montre un tympan gauche rétracté. L'oropharynx et le pharyngolarynx bien visibles sont également normaux.
L'épreuve de Valsalva est négative du côté gauche. L'épreuve de Weber montre une latéralisation à gauche, l'audiométrie
tonale une discrète surdité de transmission.

Sur quel(s) critère(s) faut-il impérativement poursuivre les investigations cliniques ?


A - Otalgie
B - Acouphènes
C - Valsalva négatif à gauche
D - Une surdité de transmission
E - Le métier du patient
Bonne(s) réponse(s) : A C D

Chez l'adulte cette surdité de transmission de principe doit faire pratiquer un examen du cavum à la recherche d'une tumeur,
d'autant qu'il existe une otalgie.
C - Traduit un dysfonctionnement tubaire, précise le mécanisme de l'atteinte
transmissionnelle.
B - Les acouphènes sont ici symptomatiques de l'obstruction tubaire.1029
Exclusivement sur DOC - DZ : www.doc-dz.com NADJI 85
RESIDANAT EN POCHE TOME II
Cas Clinique en QCM

Quelle région faut-il examiner minutieusement ?


A - La margelle laryngée
B - Le sillon amygdaloglosse
C - Le rhinopharynx
D - Le voile du palais
E - Le sinus maxillaire gauche
Bonne(s) réponse(s) : C

Cf réponse précédente.

Les éléments dont vous disposez permettent dès lors d'évoquer le diagnostic de cette affection. Quel est-il ?
A - Cancer de la margelle laryngée
B - Cancer du sillon amygdaloglosse
C - Fibrome nasopharyngien
D - Cancer du naso-pharynx
E - Polypose nasale
Bonne(s) réponse(s) : D

A B - Peuvent entraîner des otalgies par voie réflexe,mais n'expliquent pas la surdité de
transmission.
C - Tumeur histologiquement bénigne du garçon.

Parmi les histologies suivantes, laquelle s'adapte le plus souvent à cette affection ?
A - Adénome pléiomorphe
B - Carcinome glandulaire
C - Lymphome malin non hodgkinien
D - Carcinome spinocellulaire plus ou moins différencié
E - Schwanome
Bonne(s) réponse(s) : D

Le Iymphone malin est plus rare dans cette localisation que les carcinomes spino-cellulaires.

Le diagnostic d'extension impose :


A - Palpation cervicale des aires ganglionnaires
B - Scanner
C - Dosage de l'antigène carcino-embryonnaire
D - Dosage de l'alpha-foeto-protéine
E - Ponction lombaire
Bonne(s) réponse(s) : A B

A - Cancer lymphophile; le drainage se faisant vers la partie supérieure des chaînes jugulaires et spinales.
B - Permet d'étudier une extension à la base du crâne, voire intra-crânienne.
C D - Sur le plan biologique, c'est la sérologie Epstein-Barr qui est de loin la plus étroitement correlée au carcinome
épidermoïde du cavum.

L'impédancemétrie permet de caractériser ce type de surdité de transmission. Que doit-elle montrer ?


A - Le pic est centré sur la pression zéro
B - Le pic est centré sur la pression + 200
C - Présence de deux pics
D - Courbe décalée vers les pressions négatives
E - Courbe à compliance très élevée (en tour Eiffel)
Bonne(s) réponse(s) : D

Il existe une dépression dans la caisse du tympan dont l'aération est compromise.
E - Evoque une rupture de la chaîne des osselets.

1030
Exclusivement sur DOC - DZ : www.doc-dz.com NADJI 85
RESIDANAT EN POCHE TOME II
Cas Clinique en QCM
Une femme de 30 ans présente une surdité progressive dont le début remonte à quelques mois. Il s'agit d'une surdité
bilatérale prédominant du coté droit et de type transmissionnel.

Parmi les signes suivants, on peut observer :


A - Rinne négatif
B - Weber latéralisé du côté le plus sourd
C - La surdité prédomine sur les aigus
D - La malade présente un recrutement
E - La malade présente une fatigabilité au téléphone
Bonne(s) réponse(s) : A B

C D E - Caractérisent des surdités de perception.

Parmi les étiologies suivantes, quelle(s) est(sont) celle(s) qui entraîne(nt) une surdité de transmission pure ?
A - Bouchon de cérumen
B - Otite muqueuse chronique à tympan fermé
C - Neurinome de l'acoustique
D - Surdité professionnelle
E - Surdité toxique
Bonne(s) réponse(s) : A B

C - Surdité de perception rétrocochléaire (absence de recrutement).


D E - Surdité de perception endocochléaire.

Parmi les signes suivants, quel(s) est(sont) celui ceux) d'une otospongiose ?
A - Tympan normal
B - Présence d'acouphènes
C - Mobilité normale du tympan
D - Lyse de la branche descendante de l'enclume
E - Lyse des branches de l'étrier
Bonne(s) réponse(s) : A B C

A - Surdité de transmission à tympan normal.


B - Les acouphènes sont permanents, évolutifs.
C - La mobilité normale du tympan signe un fonctionnement normal de la trompe d'Eustache
et du système tympano-ossiculaire dans l'otospongiose mais une ostéodystrophie localisée bloquae la platine de l'étrier dans
la fenêtre ovale.

L'otospongiose présente un ou plusieurs caractère(s) parmi ceux-ci. Le(s)quel(s) ?


A - Facteur héréditaire
B - Classique aggravation avec les grossesses
C - Prédominance masculine
D - Débute dans l'enfance
E - Evolue vers le blocage de la trompe d'Eustache
Bonne(s) réponse(s) : A B

A - Autosomique dominant à expressivité variable.


C - Prédominance féminine.
E - Cf réponse précédente.

S'il s'agit d'une otospongiose, parmi les traitements suivants, lequel proposez-vous à cette patiente ?
A - Traitement chirurgical
B - Traitement médical par le calcium
C - Appareillage auditif stéréophonique
D - Stimulation de la cochlée par électrode
E - Traitement médical par le magnésium
Bonne(s) réponse(s) : A

Mise en place d'une prothèse entre enclume et fenêtre ovale.

1031
Exclusivement sur DOC - DZ : www.doc-dz.com NADJI 85
RESIDANAT EN POCHE TOME II
Cas Clinique en QCM
Enfant de 5 ans, sans antécédent autre que plusieurs rhinopharyngites dont trois se sont compliquées d'otites purulentes
ayant nécessité paracentèses et antibiotiques.
A été hospitalisé deux mois avant votre examen pour ce qui est décrit par la famille comme ayant été une méningite.
L'hospitalisation a duré 15 jours et à la sortie l'enfant était considéré comme cliniquement guéri. Il semble bien, d'après les
renseignements fournis par la famille que les signes méningés aient été précédés pendant 48 h par un épisode rhinopharyngé
aiguë avec otalgie bilatérale.
L'enfant vous est conduit parce que depuis son retour à domicile, il a changé de caractère ; il est devenu triste, renfermé, tend
à s'isoler et ne prend plus volontiers part aux conversations de la famille et aux jeux de ses frères et soeurs.
Il a précédemment subi un examen neuropédiatrique qui écarte toute séquelle endocrânienne de la méningite. On soupçonne
une hypoacousie d'être à l'origine de ce changement de caractère. Cette hypoacousie peut être due :
- soit à une otite séreuse bilatérale résiduelle (l'épisode a débuté par un épisode rhino-otitique)
- soit à une atteinte des 2 nerfs auditifs.

L'otoscopie est le premier examen à effectuer ; dans l'hypothèse d'une otite séreuse résiduelle elle montrera :
A - Des tympans normaux
B - Des tympans rouges, violacés, bombants
C - Des tympans ternes, sans tache lumineuse
D - Une otorrhée bilatérale
E - Des squames blanchâtres évoquant un cholestéatome
Bonne(s) réponse(s) : C

B - Aspect d'otite suppurée.

Dans l'hypothèse d'une otite séreuse bilatérale l'audiométrie tonale montrera :


A - Une courbe osseuse meilleure que la courbe aérienne
B - Une courbe osseuse moins bonne que la courbe aérienne
C - Une courbe osseuse égale à la courbe aérienne
D - Un tympanogramme plat
E - Un tympanogramme normal
Bonne(s) réponse(s) : A D

La compliance du tympan est diminuée.

Dans l'hypothèse d'une atteinte bilatérale du VIII, l'audiométrie montrera :


A - Une courbe osseuse meilleure que la courbe aérienne
B - Une courbe osseuse moins bonne que la courbe aérienne
C - Une courbe osseuse égale à la courbe aérienne
D - Un tympanogramme plat
E - Un tympanogramme normal
Bonne(s) réponse(s) : C E

Il s'agit alors d'une surdité de perception.

Il convient de demander au service qui a soigné l'enfant des informations sur le germe trouvé dans le LCR, car
ce germe a pu, compte tenu de l'histoire, venir de l'oreille. Ce dernier nécessite investigations et éventuellement
chirurgie auriculaire, s'il s'agit :
A - De méningocoque
B - De Pfeiffer
C - De pneumocoque
D - De staphylocoque doré
E - De streptocoque bêta-hémolytique
Bonne(s) réponse(s) : C

A cet âge l'haemophilus est moins souvent en cause que le pneumocoque.

Il convient également de demander des informations sur le traitement reçu qui peut avoir une responsabilité
dans la surdité si celle-ci a un caractère de perception. Cette responsabilité peut être évoquée si le traitement a
comporté :
A - Ampicilline
B - Pénicilline
C - Streptomycine
D - Gentamycine
E - Kanamycine
Bonne(s) réponse(s) : C D E

Il s'agit de 3 aminosides, famille d'antibiotiques connue pour son ototoxicité.

1032
Exclusivement sur DOC - DZ : www.doc-dz.com NADJI 85
RESIDANAT EN POCHE TOME II
Cas Clinique en QCM
Un homme de 30 ans fait très souvent des angines. A la suite d'une nouvelle infection amygdalienne, il a pris pendant quatre
jours un comprimé par jour d'Oracilline® à 1 million d'unités, puis s'est arrêté en raison de la persistance de la dysphagie.
Actuellement, il a 38°C, son faciès est vultueux, et sa dysphagie est totale, avec douleur irradiée vers l'oreille droite.
L'examen endobuccal, gêné par un trismus serré, permet néanmoins de voir un bombement du pilier antérieur de l'amygdale
droite. Il existe une adénopathie cervicale droite mal limitée et douloureuse.
Le diagnostic de phlegmon péri-amygdalien est probable.

Le phlegmon péri-amygdalien correspond à :


A - Une nécrose amygdalienne
B - Un abcès péri-amygdalien
C - Une lymphangite cervicale
D - Un adénophlegmon
E - Une myosite suppurée
Bonne(s) réponse(s) : B

L'infection s'étend du parenchyme amygdalien vers les tissus de voisinage à travers la capsule. Son extension se trouve
ensuite limitée par les muscles constricteurs du voile.

Un seul des signes suivants ne se voit pas dans son tableau clinique. Lequel ?
A - Parésie vélopalatine
B - Bombement du pilier antérieur
C - Bombement du pilier postérieur
D - Oedème de la luette
E - Fluctuation jugale
Bonne(s) réponse(s) : E

A - Est responsable de la voix nasonnée.


B - Dans le cas d'un abcès péri-amygdalien qui est le plus fréquent.
D - Classique.

La présence d'une collection suppurée est affirmée par :


A - Le scanner
B - La formule et la numération sanguine
C - La rougeur du voile du palais
D - Une ponction à travers le pilier antérieur
E - Une chute de la température
Bonne(s) réponse(s) : D

B C - N'ont rien de spécifique.


D - Affirme la collection en ramenant du pus. De plus permet un prélèvement à visée bactériologique avant antibiothérapie.
L'ouverture large pratiquée ensuite assure le drainage et soulage le malade.
E - Le syndrome infectieux est net en cas de phlegmon péri-amygdalien.

Le trismus est dû à :
A - Une contracture des muscles ptérygoïdiens
B - Une atteinte de l'articulation temporo-mandibulaire
C - Une irritation des branches du trijumeau
D - Une augmentation de la loge amygdalienne
E - Une stase veineuse péri-amygdalienne
Bonne(s) réponse(s) : A

Le trismus résulte de la contracture des muscles masticateurs, ici ptérygoidiens interne et externe, les autres étant le temporal
est le masséter.

Devant ce tableau, un geste s'impose rapidement. Lequel ?


A - Injection d'antibiotiques
B - Injection de corticoïdes
C - Incision de la collection suppurée
D - Administration d'anti-inflammatoires non stéroïdiens
E - Bains de bouche très chauds
Bonne(s) réponse(s) : C

Cf : question [47].

1033
Exclusivement sur DOC - DZ : www.doc-dz.com NADJI 85
RESIDANAT EN POCHE TOME II
Cas Clinique en QCM

Une fois la guérison obtenue, il faudra prévoir dans les mois suivants :
A - Une cure thermale
B - Une amygdalectomie
C - Une antibiothérapie séquentielle
D - Une cryothérapie d'amygdales
E - Un traitement immunothérapique
Bonne(s) réponse(s) : B

C'est une indication formelle d'amygdalectomie en raison du risque de récidive et de dissémination septique.

Une jeune femme de 25 ans consulte pour une gêne auditive remarquée après l'accouchement de son deuxième enfant, il y a
6 mois.
Elle prédomine à droite, où sont récemment apparus des bourdonnements d'oreille de timbre grave, intermittents, surtout
perçus le soir. Elle n'a ni vertiges, ni troubles de l'équilibre, aucun antécédent otologique.
Une tante maternelle a également des problèmes auditifs et a eu une intervention chirurgicale auriculaire il y a 10 ans. Cette
patiente est par ailleurs en parfaite santé et l'interrogatoire ne révèle aucun antécédent toxique ou traumatique.
L'otoscopie montre des conduits auditifs externes et des tympans normaux. Le reste de l'examen ORL clinique est sans
particularité. Lors des épreuves acoumétriques, le Weber est perçu du côté droit, le Rinne est négatif des deux côtés, le
Schwabach normal. Le tympanogramme est normal, et les réflexes stapédiens abolis des deux côtés. Une radiographie en
incidence de Schuller est normale.

A quel niveau se situe la lésion responsable de cette surdité ?


A - Oreille externe
B - Oreille moyenne
C - Oreille interne
D - Nerf auditif
E - Voies auditives centrales
Bonne(s) réponse(s) : B

La formule acoumétrique avec Rinne négatif, traduit l'existence d'une surdité de transmission bilatérale, le Weber étant
latéralisé du côté le plus sourd. La normalité du conduit élimine un obstacle à ce niveau ou une otite externe. L'épreuve de
Schwabach compare la conduction osseuse du patient avec celle de l'examinateur (n'est plus utilisée).

L'audiométrie montrera typiquement à ce stade initial de la maladie :


A - Une courbe de conduction osseuse peu modifiée
B - Une courbe de conduction osseuse abaissée
C - Une courbe de conduction aérienne normale
D - Une courbe de conduction aérienne abaissée
E - Des signes de recrutement cochléaire
Bonne(s) réponse(s) : A D

Il est précisé qu'il s'agit d'un stade initial, d'une maladie évoquant en premier lieu
l'otospongiose où la transmission est pure dans 80 % des cas. L'évolution spontanée se fait vers une surdité mixte.

Quelle est l'étiologie de cette surdité ?


A - Cholestéatome de l'oreille
B - Otite séromuqueuse à tympan fermé
C - Malformation ossiculaire
D - Otospongiose
E - Neurinome du VIII
Bonne(s) réponse(s) : D

Les antécédents familiaux, le sexe, l'aggravation à l'occasion de la grossesse, l'absence d passé otologique, la surdité de
transmission à tympan normal, les acouphènes de timbre sont autant d'argument en faveur d'otospongiose.
A - Otite chronique.
B - Courbe vers les pressions négatives, tympan rétracté.
C - Mode de révélation différent.
E - Neurinome du VIII, surdité de perception rétrocochléaire.

Quel est le traitement adéquat ?


A - Vasodilatateurs au long cours
B - Mise en place d'aérateurs transtympaniques
C - Chirurgie de l'étrier
D - Chirurgie au niveau du VIII
E - Prothèse auditive
Bonne(s) réponse(s) : C

On pratique une stapédectomie avec remplacement par un piston fixé à l'enclume et en contact avec la fenêtre ovale.

1034
Exclusivement sur DOC - DZ : www.doc-dz.com NADJI 85
RESIDANAT EN POCHE TOME II
Cas Clinique en QCM

Quel est le pronostic fonctionnel de cette surdité si la thérapeutique adéquate a été appliquée ?
A - Restitution d'une audition normale
B - Amélioration partielle de l'audition
C - Stabilisation de la surdité
D - Aggravation progressive de la surdité
E - Imprévisible
Bonne(s) réponse(s) : A

A ce stade de la maladie, où la courbe osseuse n'est pas abaissée, il est tout à fait raisonnable d'envisager une restitution
d'audition normale. En effet l'intervention en faisant remonter la courbe aérienne "ferme" le Rinne.

Vous êtes appelé d'urgence au chevet d'un enfant de 7 ans qui présente depuis 2 jours un syndrome infectieux avec
température à 39°, et un catarrhe rhinopharyngé dans un contexte épidémique viral.
Depuis quelques heures, se sont installées progressivement des difficultés respiratoires, surtout lors de l'inspiration, qui est
bruyante. Toute tentative d'alimentation est très difficile et douloureuse, même pour les liquides. Vous le trouvez dans son lit,
en position demi assise, anxieux, respirant lentement et difficilement. Sa voix est étouffée. A chaque inspiration, les creux sus-
claviculaires et sus-sternaux se dépriment.
L'examen de la cavité buccale montre une rougeur pharyngée diffuse, sans autres éléments. Le nez est encombré de
sécrétions muqueuses. A l'auscultation thoracique, le murmure vésiculaire est perceptible à droite comme à gauche,
symétrique. On entend quelques ronchus. Le pouls est à 110, le faciès coloré, les lèvres et les extrémités digitales un peu
cyanosées.

Parmi les éléments de ce dossier, l'origine laryngée de cette dyspnée est évoquée par :
A - Respiration lente
B - Prédominance inspiratoire
C - Dépression des creux
D - Dysphagie douloureuse
E - Voix étouffée
Bonne(s) réponse(s) : A B C E

C'est une bradypnée inspiratoire avec à ce stade un tirage. La voix étouffée est également en faveur d'une atteinte laryngée.

Le degré de gravité de cette dyspnée est apprécié sur :


A - Installation depuis plusieurs heures
B - Cyanose discrète des lèvres et des extrêmités
C - Température à 39°
D - Rythme respiratoire lent
E - Prédominance inspiratoire
Bonne(s) réponse(s) : A B

L'installation depuis plusieurs heures fait craindre une décompensation rapide. La cyanose est un signe d'hypoxie et de
mauvaise tolérance, avec perturbation gazométrique.
C - Signe infectieux non spécifique mais évocateur quant à l'étiologie.
D E - Caractérise le type de dyspnée et non pas la gravité.

Les éléments dont vous disposez vous permettent sûrement de porter le diagnostic précis de cette affection.
Quel est-il parmi les propositions suivantes ?
A - Papillomatose laryngée
B - Corps étranger laryngé
C - Epiglottite
D - Laryngite striduleuse (faux croup)
E - Croup
Bonne(s) réponse(s) : C

En sa faveur on retient le contexte infectieux marqué, suivant une infection vocale, la


dysphagie, le refus de s'allonger, la voix étouffée, l'âge.
C'est une urgence thérapeutique imposant le transfert par S.A.M.U. en milieu spécialisé. Le décubitus est à proscrire, risquant
de décompenser l'état respiratoire.

1035
Exclusivement sur DOC - DZ : www.doc-dz.com NADJI 85
RESIDANAT EN POCHE TOME II
Cas Clinique en QCM

Quel est l'élément essentiel de cette observation qui vous a permis le diagnostic exact de cette dyspnée
laryngée ?
A - Dysphagie douloureuse
B - Hyperthermie
C - Anxiété
D - Position demi-assise
E - Cyanose des extrêmités
Bonne(s) réponse(s) : A

La tuméfaction de l'épiglotte est responsable de cette dysphagie qui situe donc le niveau de l'atteinte.

Un homme de 60 ans, consulte pour une dysphagie apparue deux mois plus tôt, sans régurgitation, avec douleur rapportée à
l'oreille gauche. Il est un peu dysphonique, et accuse un amaigrissement de 4 kg. L'examen de la cavité buccale, de la langue
et des amygdales, est normal. On note la présence d'une adénopathie sous-digastrique gauche, de 4 cm de diamètre, dure,
fixée. Au terme de la consultation, le diagnostic de cancer du sinus piriforme est évoqué.

Le sinus piriforme se situe au niveau :


A - De l'hypopharynx
B - De la bouche de l'oesophage
C - De la sous-glotte
D - Du ventricule laryngé
E - De la fossette glosso-épiglottique
Bonne(s) réponse(s) : A

Cf anatomie.

L'interrogatoire pourra retenir comme élément déterminant en faveur d'un cancer du sinus piriforme :
A - Un facteur racial
B - Un alcoolo-tabagisme
C - Les poussières industrielles
D - Certaines habitudes alimentaires
E - Une sinusite chronique
Bonne(s) réponse(s) : B

Evident.

Quel examen doit-on pratiquer en premier lieu pour préciser le diagnostic ?


A - Transit pharyngo-oesophagien
B - Oesophagoscopie
C - Pharyngographie lipiodolée
D - Laryngoscopie
E - Radiographie du crâne
Bonne(s) réponse(s) : D

L'examen est d'abord clinique, c'est la laryngoscopie indirecte, éventuellement avec anesthésie locale (étant donné les
réflexes importants sur ce terrain). Il doit être complété par une panendoscopie permettant les biopsies.

Le diagnostic de certitude sera donné par :


A - Un examen cytologique des crachats
B - Le dosage de l'antigène carcino-embryonnaire
C - La biopsie de la lésion
D - Un scanner cervical
E - La présence de sang dans les selles
Bonne(s) réponse(s) : C

Le diagnostic anatomo-pathologique doit précéder tout traitement (médico-légal).

1036
Exclusivement sur DOC - DZ : www.doc-dz.com NADJI 85
RESIDANAT EN POCHE TOME II
Cas Clinique en QCM

Le cancer du sinus piriforme est presque toujours :


A - Un adénocarcinome
B - Un lymphome malin
C - Un épithélio-lymphome
D - Un épithélioma spinocellulaire
E - Un sarcome
Bonne(s) réponse(s) : D

Evident.

Le traitement généralement admis de ce cancer fait appel à :


A - La chimiothérapie isolée
B - L'endocuriethérapie
C - La radiothérapie
D - L'immunothérapie
E - La chirurgie
Bonne(s) réponse(s) : C E

Le traitement classique est radiochirurgical :


- pharyngo-laryngectomie totale, avec curage ganglionnaire de type radical étant donné le caractère fixé (N3) de l'adénopathie
- suivie de cobaltothérapie post-opératoire, après soins dentaires, délivrant en général 70 Gy sur le site tumoral et les aires
ganglionnaires.
La place de la chimiothérapie reste à définir.

L'évolution peut se faire vers :


A - Métastase ganglionnaire
B - Métastase pulmonaire
C - Récidive locale
D - Deuxième cancer digestif
E - Métastases osseuses
Bonne(s) réponse(s) : A B C D E

Le pronostic est sévère (moins 40 % de survie à 5 ans).

Un homme de 30 ans fait très souvent des angines. A la suite d'une nouvelle infection amygdalienne, il a pris pendant quatre
jours un comprimé par jour d'Oracilline® dosé à 1 million d'unités., puis s'est arrêté en raison de la persistance de la
dysphagie.
Actuellement, il a 38°, son faciès est vultueux et sa dysphagie est totale avec douleur irradiée vers l'oreille droite. L'examen
endobuccal, gêné par un trismus serré, permet néanmoins de voir un bombement du pilier antérieur de l'amygdale droite. Il
existe une adénopathie cervicale droite, mal limitée et douloureuse. Le diagnostic de phlegmon de l'amygdale est probable.

Le phlegmon de l'amygdale correspond à :


A - Une nécrose amygdalienne
B - Un abcès péri-amygdalien
C - Une lymphangite cervicale
D - Un adénophlegmon
E - Une myosite suppurée
Bonne(s) réponse(s) : B

L'infection s'étend du parenchyme amygdalien vers les tissus voisins à travers la capsule de l'amygdale. Les muscles
constricteurs du pharynx constituent une barrière à une extension.

Un seul des signes suivants ne se voit pas dans son tableau clinique. Lequel ?
A - Trismus
B - Bombement du pilier antérieur
C - Bombement du pilier postérieur
D - Oedème de la luette
E - Fluctuation jugale
Bonne(s) réponse(s) : E

Le bombement du pilier antérieur est le plus fréquent (phlegmon antéro-supérieur).


L'oedème de la luette est classique. Le trismus manque exceptionnellement à ce stade.

1037
Exclusivement sur DOC - DZ : www.doc-dz.com NADJI 85
RESIDANAT EN POCHE TOME II
Cas Clinique en QCM

La présence d'une collection suppurée est affirmée par :


A - Le scanner
B - La formule et la numération sanguine
C - La rougeur du voile du palais
D - Une ponction à travers le pilier antérieur
E - Une chute de la température
Bonne(s) réponse(s) : D

La ponction en ramenant le pus confirme le stade de collection et permet le prélèvement à visée bactériologique.
Elle est suivie de l'incision du voile qui assure le drainage et soulage le malade.

Le trismus est dû à :
A - Une contracture des muscles ptérygoïdiens
B - Une atteinte de l'articulation temporo-mandibulaire
C - Une irritation des branches du trijumeau
D - Une augmentation du volume de la loge amygdalienne
E - Une stase veineuse péri-amygdalienne
Bonne(s) réponse(s) : A

Limitation temporaire de l'ouverture buccale, elle est liée à une atteinte des muscles
masticateurs (masseter temporal, ptérygoïdiens).

Devant ce tableau, un geste s'impose rapidement. lequel ?


A - Injection d'antibiotiques
B - Injection de corticoïdes
C - Incision de la collection suppurée
D - Administration d'anti-inflammatoires non stéroïdiens
E - Bains de bouche très chauds
Bonne(s) réponse(s) : A

Le traitement médical est l'antibiothérapie par voie parentale à haute dose.

Une fois la guérison obtenue, il faudra prévoir ultérieurement :


A - Une cure thermale
B - Une amygdalectomie
C - Une antibiothérapie séquentielle
D - Une cryothérapie des amygdales
E - Un traitement immunothérapique
Bonne(s) réponse(s) : B

Indication formelle d'amygdalectomie, en raison du risque de récidive et des risques septiques. Est pratiquée à froid.

Un enfant de 2 ans présente depuis 24 heures une otalgie gauche, il existe une fièvre à 38°. C'est le premier épisode de ce
type, et dans la semaine qui précédait, l'enfant avait été enrhumé. L'otoscopie faite le soir découvre une membrane
tympanique congestive et bombée avec disparition des reliefs et du triangle lumineux. Le reste de l'examen est strictement
normal. Le diagnostic d'otite moyenne purulente gauche est posé.

Quels sont les trois germes les plus fréquemment rencontrés dans l'otite moyenne aiguë purulente à cet âge ?
A - Haemophilus influenzae
B - Staphylococcus aureus
C - Streptococcus pneumoniae
D - Proteus mirabilis
E - Mycoplasma pneumoniae
Bonne(s) réponse(s) : A B C

Evident.

1038
Exclusivement sur DOC - DZ : www.doc-dz.com NADJI 85
RESIDANAT EN POCHE TOME II
Cas Clinique en QCM

Quels sont, parmi les éléments cliniques de cette observation, les 2 qui ont permis de retenir le diagnostic d'otite
moyenne aiguë purulente ?
A - Tympan rouge
B - Otalgie
C - Tympan bombant
D - Disparition du triangle lumineux
E - Effacement des reliefs
Bonne(s) réponse(s) : C E

Le bombement est postéro-inférieur, car c'est le quadrant le plus déclive où doit se faire la paracentèse.
A B D - Ne sont pas spécifiques.

La paracentèse s'effectue classiquement en un point de la membrane tympanique, situé :


A - En plein centre de la membrane tympanique
B - Dans le quadrant antéro-supérieur de la pars tensa
C - Dans le quadrant postéro-supérieur de la pars tensa
D - Dans un des quadrants inférieurs de la pars tensa
E - Dans la pars flaccida
Bonne(s) réponse(s) : D

Elle ne doit jamais être faite en région postéro-supérieure (osselets). Elle permet un prélèvement pour bactériologie. Doit être
associée à une antibiothérapie par voie générale.

Quel est parmi les cinq antibiotiques suivants, celui dont le spectre d'activité concerne le plus grand nombre de
germes responsables de l'otite moyenne aiguë purulente ?
A - Pénicilline v
B - Amoxicilline
C - Rifampicine
D - Tétracycline
E - Gentamicine
Bonne(s) réponse(s) : B

En raison de la fréquence des germes rencontrés qui y sont sensibles habituellement :


- pneumocoque
- streptocoque
- haemophilus (résistances croissantes).

Un homme de 50 ans consulte pour une dysphagie apparue depuis quelques semaines sans altération de l'état général.Il se
plaint d'une otalgie droite. Il fume un paquet de cigarettes par jour depuis l'âge de 18 ans en inhalant la fumée. La
consommation de vin est de un litre par jour depuis 20 ans.

Le diagnostic de tumeur maligne des voies aérodigestives supérieures est alors évoqué, quel(s) éléments(s) de
l'examen peu(ven)t orienter vers ce diagnostic ?
A - Prélèvement de gorge à visée bactériologique
B - Palpation des aires ganglionnaires
C - Laryngoscopie indirecte
D - Palpation des structures buccopharyngées
E - Rhinoscopie antérieure
Bonne(s) réponse(s) : B C D

B - En dehors du cancer de la corde vocale, les cancers des V.A.D.S. sont lymphophiles.
C D - L'examen clinique reste le premier temps des explorations. Il doit comprendre un toucher buccopharyngé.

L'examen clinique ORL découvre une lésion ulcéro bourgeonnante de la base de la langue. Quel(s) examen(s)
complémentaire(s) doi(ven)t être demandé(s) ?
A - Endoscopie des voies aérodigestives supérieures
B - Biopsie de la lésion
C - Radiographie pulmonaire
D - Dosage des anticorps anti-EBV
E - Radiographie panoramique dentaire
Bonne(s) réponse(s) : A B C E

A - La panendoscopie permet de faire le bilan d'extension locale, de pratiquer les biopsies indispensables au diagnostic et la
recherche d'une deuxième localisation.
C - Recherche une métastase ou un carcinome bronchique associé (même facteurs de risque).
D - Pratiqué dans les carcinomes du cavum, et constituant un marqueur tumoral.
E - L'état dentaire de ces malades est souvent déplorable est les soins dentaires sont à prévoir en vue d'une radiothérapie.

1039
Exclusivement sur DOC - DZ : www.doc-dz.com NADJI 85
RESIDANAT EN POCHE TOME II
Cas Clinique en QCM

Les examens complémentaires effectués ont confirmé le diagnostic de tumeur maligne de la base de langue.
Parmi les résultats histologiques suivants, quel(s) est(sont) celui(ceux) à envisager en premier lieu pour cette
localisation ?
A - Carcinome épidermoïde peu différencié
B - Lymphome malin non Hodgkinien
C - Liposarcome
D - Chondrosarcome
E - Cystadénolymphome
Bonne(s) réponse(s) : A

Evident.

A l'issue de ce bilan il est conclu à une lésion de l'hémibase de langue droite isolée N0 M0. Quel choix
thérapeutique proposerez-vous au patient ?
A - Chimiothérapie exclusive
B - Association radio chirurgicale
C - Exérèse basilinguale seule
D - Association chimio-immunothérapie
E - Immunostimulation
Bonne(s) réponse(s) : B

A D E - N'ont pas fait leurs preuves.


La latéralité de la lésion est en faveur d'un traitement chirurgical suivi de radiothérapie, surtout s'il s'agit d'une tumeur ulcérée
ou infiltrante.

Un menuisier de 50 ans, pléthorique, vous est amené en consultation parce qu'il présente, depuis une demi-heure, une
hémorragie spontanée, abondante de sang rouge, extériorisée par la fosse nasale droite, avec écoulement pharyngé
postérieur.
Il s'agit du premier épisode de ce type ; il n'y a pas eu de syndrome infectieux dans les jours précédents, pas de traitement
médicamenteux en cours ou récent, pas d'antécédents rhinologiques : pas d'obstruction nasale, ni de rhinorrhée, ni d'anosmie.
On relève encore, à l'interrogatoire, que ce malade fume un paquet de cigarettes par jour; il boit peu dit-il. Depuis plusieurs
mois, il présente des acouphènes bilatéraux, des céphalées intermittentes associées à des vertiges de brève durée.
A la rhinoscopie, il n'est pas possible de voir exactement l'endroit qui saigne ; l'hémorragie paraît plutôt provenir de la région
postérieure de la fosse nasale droite.
La pression artérielle est à 200/110, le pouls bien frappé, à 110 ; les conjonctives ne sont pas décolorées.

Quel est le diagnostic étiologique de cette épistaxis ?


A - Epistaxis essentielle
B - Hypertension artérielle
C - Troubles de la crase sanguine
D - Polypose naso-sinusienne
E - Sinusite maxillaire aiguë
Bonne(s) réponse(s) : B

A - Diagnostic d'élimination.
B - En faveur on retient l'état pléthorique, les céphalées, les vertiges, les chiffres tensionnels.
D E - Pas de signes visibles en rhinoscopie et de toute façon une épistaxis abondante doit faire rechercher une autre cause.

Quelle est la maladie que le contexte professionnel impose d'éliminer formellement ?


A - Maladie de Rendu-Osler
B - Sinusite maxillaire chronique
C - Epithélioma du cavum
D - Epithélioma de l'ethmoïde
E - Allergie à la poussière de bois
Bonne(s) réponse(s) : D

Cancer des travailleurs du bois, pris en charge comme maladie professionnelle.

Parmi les diagnostics suivants, quel est celui qui peut être éliminé d'emblée du fait de l'âge du malade ?
A - Néphrite chronique
B - Fibrome nasopharyngien
C - Rhinite vasomotrice saisonnière
D - Cancer du sinus maxillaire
E - Rhinite médicamenteuse
Bonne(s) réponse(s) : B

C'est une tumeur histologiquement bénigne du garçon aux alentours de la puberté.

1040
Exclusivement sur DOC - DZ : www.doc-dz.com NADJI 85
RESIDANAT EN POCHE TOME II
Cas Clinique en QCM

Parmi les modalités thérapeutiques suivantes, laquelle(lesquelles) allez-vous utiliser dans un premier temps ?
A - Tamponnement antérieur
B - Tamponnement postérieur
C - Traitement antihypertenseur par voie générale
D - Ligature de l'artère maxillaire interne
E - Embolisation de l'artère maxillaire interne
Bonne(s) réponse(s) : A C

Un tamponnement antérieur peut suffir à arrêter une épistaxis qui semble postérieure, surtout s'il existe une H.T.A. sur
laquelle on va agir médicalement. Le tamponnement postérieur est exceptionnellement pratiqué d'emblée, car il s'agit d'un
geste choquant. La ligature et l'embolisation de l'artère maxillaire interne constituent le dernier recours à des épistaxis rebelles
abondantes.

Un homme de 50 ans en bon état général, sans antécédents vient vous consulter pour, une dysphonie et une dysphagie
douloureuse évoluant depuis 6 mois et s'aggravant progressivement. Vous pensez à un cancer de l'endolarynx.

Parmi les facteurs étiologiques suivants, le(s)quel(s) doit ou doivent être recherché(s) ?
A - Alcoolisme
B - Tabagisme
C - Travail du bois
D - Infection à virus Epstein-Barr
E - Travail du cuir
Bonne(s) réponse(s) : A B

A - Le tabagisme est le facteur de risque au premier plan des cancers de l'endolarynx ; l'alcool a une action de synergie et
peut expliquer l'existence d'une autre localisation.
C - Adénocarcinome de l'ethmoïde.
D - Dans le carcinome du cavum (U.C N.T.).
E - Expose aux cancers éthmoïdo-maxillaires.

Parmi les examens cliniques et paracliniques suivants, quel(s) est(sont) celui(ceux) qui vous parai(ssent)
indispensables pour le bilan d'extension ?
A - Examen laryngoscopique
B - Radiographie et tomographie de larynx
C - Pan endoscopie avec biopsie
D - Scintigraphie thyroïdienne
E - Radiographie du thorax
Bonne(s) réponse(s) : A C E

Réponse difficile en ce qui concerne l'item B qui ne nous parait pas indispensable. Le scanner laryngé est plus intéressant,
notamment en ce qui concerne l'envahissement de la loge H.T.E.

Le plus souvent l'examen histologique de la tumeur va vous montrer la présence :


A - D'un fibrosarcome
B - D'un épithélioma glandulaire
C - D'un épithélioma malpighien
D - D'un lymphome non hodgkinien
E - D'un cylindrome
Bonne(s) réponse(s) : C

Evident.

Après traitement radiochirurgical, la surveillance ambulatoire comportera :


A - Radiographie pulmonaire tous les 6 mois
B - Suppression tabac et alcool
C - Surveillance dentaire régulière
D - Massages et rééducation de l'épaule
E - Examen ORL régulier
Bonne(s) réponse(s) : A B C E

La surveillance post-thérapeutique est fondamentale :


- pour dépister la survenue de récidive loco-régionale ou de deuxième localisation, surveiller le trachéostome
- pour s'assurer de l'abstention du tabac et de l'alcool. La surveillance pulmonaire est nécessaire pour dépister une métastase,
un autre cancer, une tuberculose, tous les six mois pendant 3 ans. La surveillance dentaire est habituelle.
D - Fait allusion aux séquelles liées au sacrifice du nerf spinal. Lors d'un curage ganglionnaire de type radical (emportant le
muscle sterno-cleido-mastoidien la jugulaire interne les chaînes ganglionnaires), le sacrifice du spinal peut être rendu
nécessaire en raison de métastases ganglionnaires à son contact.

1041
Exclusivement sur DOC - DZ : www.doc-dz.com NADJI 85
RESIDANAT EN POCHE TOME II
Cas Clinique en QCM
Un enfant de 12 ans présente depuis 5 années une tuméfaction cervicale antérieure, médiane, juxta-hyoïdienne, de volume
variable (une noisette), indolore, mobile à la déglutition, adhérente à l'os hyoïde.
Il est adressé en consultation ORL car, depuis 4 jours, cette tuméfaction a pris un aspect inflammatoire : elle est douloureuse
spontanément et à la palpation, et vient de se fistuliser à la peau dans sa partie centrale, laissant s'écouler du pus franc. Les
signes généraux sont discrets et l'état général est peu atteint, avec température à 38°C.
Le reste de l'examen ORL est normal, ainsi que l'examen des dents.

Quel diagnostic allez-vous porter ?


A - Adénite sus-thyroïdienne
B - Adénite sous-thyroïdienne
C - Thyroïdite
D - Sous-maxillite lithiasique
E - Kyste du tractus thyréoglosse
Bonne(s) réponse(s) : E

Il s'agit d'une tuméfaction médiane se développant entre l'os hyoïde et le cartilage thyroïde, lié à la persistance du canal de
Bochdalek.

Parmi les signes suivants, vous avez retenu pour porter ce diagnostic :
A - La tuméfaction est mobile
B - Elle est douloureuse
C - Les signes généraux sont peu intenses
D - La tuméfaction adhère à l'os hyoïde
E - La tuméfaction s'élève pendant la déglutition
Bonne(s) réponse(s) : D E

A - C'est la mobilité à la déglutition qui est évocatrice, la mobilité de la tuméfaction elle-même est de peu d'intérêt.
B C - Traduit une surinfection, circonstance de découverte.

Quel est l'examen indispensable à pratiquer avant toute décision thérapeutique ?


A - Une radiographie du cou face et profil
B - Une radiographie de la déglutition
C - Une endoscopie oesophagienne
D - Une cartographie thyroïdienne
E - Une angiographie
Bonne(s) réponse(s) : D

L'existence du kyste du tractus thyréoglosse doit faire vérifier que la migration de la glande thyroïde s'est faite jusqu'au cou et
qu'il ne s'agit pas d'une glande ectopique.

Quelle est, parmi les propositions thérapeutiques suivantes, celle qu'il convient de retenir ?
A - La ponction de la collection
B - L'incision de la collection
C - La radiothérapie
D - L'ablation de la masse
E - L'exérèse chirurgicale incluant l'ablation du corps de l'os hyoïde
Bonne(s) réponse(s) : E

A B D - Sont insuffisants et exposent aux récidives.


L'ablation du corps de l'os hyoïde est rendue nécessaire du fait de l'adhérence du canal à la
face antérieure de l'os hyoïde.

1042
Exclusivement sur DOC - DZ : www.doc-dz.com NADJI 85
RESIDANAT EN POCHE TOME II
Cas Clinique en QCM
Un homme de 40 ans vient consulter pour une tuméfaction sous-auriculaire droite. Il avait déjà remarqué une discrète
déformation à ce niveau, depuis plusieurs années, qui ne l'avait pas inquiété. Mais elle a augmenté rapidement de volume
depuis quelques semaines, parfaitement indolore.

Chez ce sujet en bonne santé, et sans antécédents pathologiques notables, l'examen montre effectivement un nodule
d'environ 3 cm de diamètre, de consistance pierreuse, soulevant le lobule de l'oreille, indolore à la palpation sans adhérence à
la peau. Le reste de l'examen ORL et cervical est normal. Il n'y a pas de paralysie faciale. La denture est en bon état. La
salive, claire, s'écoule normalement à l'orifice du canal de Sténon, à la pression de la glande parotide.

Quelle(s) hypothèse(s) diagnostique(s) pouvez-vous formuler à ce stade de votre examen ?


A - Une sarcoïdose
B - Une lithiase salivaire
C - Une adénopathie
D - Une tumeur parotidienne
E - Un kyste sébacé
Bonne(s) réponse(s) : D

A - La sarcoïdose atteint rarement la parotide, et cette atteinte n'est pas souvent révélatrice. Il existe des signes généraux
associés. L'atteinte est en règle bilatérale, la consistance est inhomogène, mal limitée.
B - Rare au niveau de la parotide. La symptomatologie est rythmée par les repas.
C - Il s'agit d'un diagnostic différentiel et la consistance pierreuse n'est pas en faveur d'une adénopathie.
E - Tuméfaction molle, avec souvent des antécédents de surinfection.

Parmi les propositions suivantes, laquelle(lesquelles) retiendrez-vous comme argument en faveur de la


bénignité de l'affection ?
A - Indolence de la tuméfaction
B - Absence d'adhérence à la peau
C - Absence de paralysie faciale
D - Consistance de la tuméfaction
E - Aucun de ces arguments
Bonne(s) réponse(s) : E

La formulation de la question et des items est très ambiguë.


A - La douleur ferait évoquer un cylindrome mais son absence est un argument trop faible en faveur de la bénignité.
B C - Leur existence serait en faveur de la malignité, mais leur absence ne signifie pas la bénignité.
D - La consistance pierreuse est plutôt en faveur de la malignité.

Quels examens, parmi les suivants, allez-vous demander pour préciser votre(vos) hypothèse(s) diagnostique(s)
?
A - Un test de Kweim
B - Une angiographie
C - Une sialographie
D - Une échographie
E - Une drill biopsie
Bonne(s) réponse(s) : D E

Là encore la réponse est difficile, l'échographie étant préconisée par certains auteurs.
B - L'angiographie est un examen d'exception pour l'exploration parotidienne. Indiquée
lorsque l'on suspecte une tumeur vasculaire.
C - A un intérêt quand on a une orientation clinique vers une lithiase, une affection
inflammatoire ou une sialose.
D - Intéressante pour la nature kystique ou non, l'existence de nodule, les limites. De plus en plus pratiquée.
E - Doit précéder de peu l'intervention. Apporte des renseignements précis dans les mains d'un cytologiste entraîné (mais il
existe des faux négatifs et des faux positifs).

Les examens demandés n'apportent pas d'élément formel pour le diagnostic. Vous retenez sur l'argument de
fréquence, l'hypothèse d'une tumeur mixte de la glande parotide (adénome pléiomorphe) et posez une
indication opératoire. Si cette hypothèse se vérifiait à l'examen histologique extemporané, quelle attitude
thérapeutique choisirez-vous parmi les suivantes ?
A - Parotidectomie totale ou partielle avec conservation du nerf facial
B - Parotidectomie totale avec sacrifice du nerf facial
C - Parotidectomie suivie d'une radiothérapie
D - Parotidectomie suivie d'une chimiothérapie
E - Enucléation simple de la tumeur
Bonne(s) réponse(s) : A

Le nerf facial doit être conservé en raison du caractère bénin de l'affection.


Elle est souvent non capsulée et donc nécessite un geste large pour éviter la récidive (E).

1043
Exclusivement sur DOC - DZ : www.doc-dz.com NADJI 85
RESIDANAT EN POCHE TOME II
Cas Clinique en QCM

L'examen histologique extemporané montre qu'il s'agit en réalité d'un cylindrome de la glande parotide Une
parotidectomie totale avec conservation du nerf facial est pratiquée. Parmi les propositions suivantes,
laquelle(lesquelles) retiendrez-vous ?
A - Vous portez un diagnostic de bénignité et considérez que ce malade est guéri
B - Vous assurez simplement une surveillance clinique régulière
C - Vous faites pratiquer une radiothérapie complémentaire
D - Vous demandez régulièrement une radiographie des poumons
E - Vous faites pratiquer un bilan osseux avec scintigraphie du squelette
Bonne(s) réponse(s) : C D

La réponse est très discutable en ce qui concerne C.


A - Faux, il s'agit d'une tumeur maligne, de pronostic grave.
C - Il semble que ce soit la tendance actuelle sur un nodule de taille déjà conséquente (3 cm) étant donné la sévérité de
l'affection, et malgré la faible radiosensibilité du cylindrome.
B - Est à exclure si on répond C.
D - Les métastases à distance se font essentiellement aux poumons justifiant la surveillance
pulmonaire régulière. Elles sont en principe tardives mais dans un certain nombre de cas sont retrouvées lors du premier
examen.
E - Ne s'impose pas. Les métastases osseuses sont beaucoup plus rares.

Un enfant de 4 ans est fébrile depuis quelques jours. Le médecin a diagnostiqué une rhinopharyngite et prescrit un traitement
par paracétamol, aspirine et instillations nasales antibiotiques et vasoconstrictrices.
Cet enfant chétif et hypotrophique est fréquemment sujet à de telles poussées de rhinopharyngites. Trois jours plus tard, le
médecin est appelé à nouveau car depuis quelques heures, la température est réapparue à 39°C et l'enfant se plaint de
douleurs au niveau de l'oreille droite, entraînant une insomnie la nuit précédente. L'otoscopie montre à droite un tympan
rouge, bombant, pulsatile, ayant perdu ses reliefs. A gauche, le tympan est discrètement inflammatoire, rosé, hyperhémique.
La palpation de la région mastoïdienne est indolore des deux côtés. Les fosses nasales sont encombrées d'une sécrétion
mucopurulente. L'examen du pharynx montre une trainée mucopurulente sur la paroi postérieure et une importante
hypertrophie des végétations adénoïdes.

A l'origine de cette otite moyenne aiguë les 4 germes les plus fréquemment rencontrés sont :
A - Streptocoque hémolytique A
B - Staphylocoque
C - Pneumocoque
D - Hémophilus
E - Proteus
Bonne(s) réponse(s) : A B C D

L'otite aiguë est habituellement liée à un seul germe.

Un traitement antibiotique est prescrit. Quels sont les deux antibiotiques que vous choisirez de préférence parmi
les suivants ?
A - Pénicilline G
B - Ampicilline
C - Macrolide
D - Aminoside
E - Tétracycline
Bonne(s) réponse(s) : B C

Ont un spectre d'activité sur les cocci Gram + et sur des hémophilus (partielle pour les
macrolides, avec quelques résistances pour les ampicillines).
Les aminosides seuls ne sont pas actifs sur les Gram et de plus sont otoxiques.

Une paracentèse est indiquée. Elle sera pratiquée au niveau de la zone suivante du tympan :
A - Quadrant postéro-supérieur
B - Partie supérieure
C - Quadrant antéro-supérieur
D - Quadrant antéro ou postéro-inférieur
E - Au centre
Bonne(s) réponse(s) : D

D - Zone de déclivité non dangereuse.


Jamais, car risque de lésion ossiculaire.

1044
Exclusivement sur DOC - DZ : www.doc-dz.com NADJI 85
RESIDANAT EN POCHE TOME II
Cas Clinique en QCM

Quelle est la complication qui peut survenir le plus fréquemment si le traitement n'est pas efficace ?
A - Cholestéatome de l'oreille
B - Mastoïdite
C - Dyspepsie grave
D - Surdité totale par labyrinthite
E - Paralysie faciale
Bonne(s) réponse(s) : B

Evident. Propagation de l'infection vers le système cellulaire pneumatique de l'os temporal entraînant une ostéite.

Quelle est la meilleure conduite prophylactique à proposer pour éviter la répétition d'une telle otite ?
A - Antibiothérapie au long cours par voie générale
B - Corticothérapie
C - Instillations auriculaires régulières d'antibiotique
D - Adénoïdectomie
E - Mastoïdectomie
Bonne(s) réponse(s) : D

L'hypertrophie et l'infection chronique exposent une obstruction tubaire source d'otite séreuse se surinfectant.

Une femme de 28 ans est hospitalisée en urgence pour des vertiges. Ceux-ci sont apparus de façon brutale, la veille, et
persistent accompagnés de vomissements. Dans les antécédents on retrouve la notion d'otites fréquentes dans l'enfance avec
écoulement purulent intermittent de l'oreille gauche depuis 6 mois. L'examen clinique retrouve un syndrome vestibulaire de
type périphérique avec un écoulement purulent de l'oreille gauche. Après nettoyage, on observe une perforation tympanique
gauche, marginale, avec présence de débris blanchâtres dans la caisse en arrière de la perforation. L'oreille contro latérale
est normale ainsi que le reste de l'examen clinique ORL. L'audiogramme montre une surdité mixte du côté gauche avec une
composante transmissionnelle de l'ordre de 30 décibels sur toutes les fréquences, l'audition est normale à droite.

Quel est le type de surdité donné par une rupture de chaîne ossiculaire ?
A - Surdité de transmission à 15 décibels
B - Surdité de transmission à 30 décibels
C - Surdité de transmission supérieure à 30 décibels
D - Surdité mixte
E - Surdité de perception
Bonne(s) réponse(s) : C

Il s'agit d'une surdité de transmission importante, de 30 à 60 dB.

Parmi ces éléments séméiologiques, lequel n'est pas de nature véstibulaire ?


A - Déviation des index
B - Romberg
C - Déviation à la marche aveugle
D - Adiadococinésie
E - Nystagmus
Bonne(s) réponse(s) : D

C'est un signe cérébelleux.

Parmi ces éléments sémiologiques, lequel ou lesquels sont en faveur d'un syndrome vestibulaire périphérique ?
A - Brutalité et intensité du syndrome vestibulaire
B - Existence de signes neurovégétatifs associés
C - Syndrome vestibulaire harmonieux
D - Atteinte auditive associée
E - Acouphènes associés
Bonne(s) réponse(s) : A B C D E

Evident.

1045
Exclusivement sur DOC - DZ : www.doc-dz.com NADJI 85
RESIDANAT EN POCHE TOME II
Cas Clinique en QCM

L'histoire clinique fait évoquer en premier le diagnostic de :


A - Otite moyenne aiguë
B - Otite séro-muqueuse
C - Otite chronique simple
D - Otorrhée tubaire
E - Otite cholestéatomateuse
Bonne(s) réponse(s) : E

En sa faveur on retient le long passé d'otite chronique, la survenue d'une complication avec fistule labyrinthique traduisant une
otite agressive, l'aspect marginal de la perforation très invocateur, et enfin la présence de débris blanchâtres dans la caisse.

Parmi les complications des otites cholestéatomateuses, laquelle(lesquelles) nécessite(nt) un traitement


chirurgical rapide ?
A - Surdité
B - Otorrhée
C - Paralysie faciale
D - Vertiges
E - Abcès cérébral
Bonne(s) réponse(s) : C D E

C - Pour essayer de sauvegarder le nerf.


D - Pouvant traduire une fistule labyrinthique.
E - Présentant un risque vital.

Quelle est la nature histologique du cholestéatome ?


A - Epithélium cylindrique
B - Epithélium glandulaire
C - Epithélium malpighien
D - Infiltrat lymphoplasmocytaire
E - Granulome à cholestérine
Bonne(s) réponse(s) : E

Evident.

Dans une otite chronique, quelle est l'élément de la chaîne ossiculaire le plus fréquemment atteint ?
A - Marteau
B - Corps de l'enclume
C - Branche descendante de l'enclume
D - Jonction enclume-étrier
E - Etrier
Bonne(s) réponse(s) : C

C'est la lyse de la branche descendante de l'enclume que l'on retrouve le plus souvent dans une otite cholestéatomateuse.

Mme M., 72 ans, est hospitalisée d'urgence pour une épistaxis grave.
Son hématocrite est à 0,24. Son hémoglobine est à 6 g/100ml. Sa tension artérielle, habituellement à 15, est descendue à 10.
Elle saigne régulièrement depuis plusieurs années. Ces épistaxis se rapprochent au fur et à mesure qu'elle avance en âge.
L'interrogatoire apprend que son père présentait les mêmes symptômes.

Quel diagnostic évoquent ces épistaxis ?


A - Hémophilie
B - Hémogénie
C - Cirrhose
D - Maladie de Rendu-Osler
E - Hypertension artérielle
Bonne(s) réponse(s) : D

Il s'agit d'une dysplasie vasculaire généralisée héréditaire de révélation semi-tardive. La maladie de Rendu-Osler est le plus
souvent révélée par des épistaxis, et la notion familiale aide au diagnostic. Les lésions télangiectasiques touchant surtout les
muqueuses.
L'hémophilie est une maladie du garçon.
Dans l'hypothèse d'une cirrhose, l'évolution depuis quelques années aurait vraisemblablement été émaillée de complications,
ictère, ascite, hémorragie digestive.
Quand aux cirrhoses entrant dans le cadre de maladies héréditaires elles sont de révélations plus précoces.
On ne peut conclure à une hypertension sur les seuls chiffres donnés ici, s'agit-il d'ailleurs de la maxima ou de la minima (?).

1046
Exclusivement sur DOC - DZ : www.doc-dz.com NADJI 85
RESIDANAT EN POCHE TOME II
Cas Clinique en QCM

Sur quel(s) élément(s) peut-on affirmer le diagnostic ?


A - Dosages biologiques
B - Biopsie de la muqueuse nasale
C - Examen de la peau du visage et des oreilles
D - Examen des doigts
E - Examen de la langue et des lèvres
Bonne(s) réponse(s) : C D E

Le diagnostic repose sur la notion d'antécédents familiaux, la survenue d'épistaxis et l'existence de télangiectasies. En effet
les lésions cutanéo-muqueuses sont quasi-constantes, d'apparition semi-tardive. Les télangiectasies cutanées prédominent
aux extrémités : la face, la pulpe des doigts, la région peri-unguéales, paumes et plantes.
Les télangiectasies muqueuses sont habituellement nasales et bucco-labiales. Ces télangiectasies disparaissent à la
vitropression.
Les dosages biologiques n'ont pas d'apport sur le plan diagnostique, notamment les tests d'hémostase qui sont normaux. De
même l'histologie n'apporte pas d'élément diagnostique.

Quel traitement général immédiat faut-il instituer ?


A - Transfusion de sang total
B - Perfusion de concentré plaquettaire
C - Corticoïde
D - Coagulants
E - Aucun traitement
Bonne(s) réponse(s) : E

Aucun des traitements proposés ici, et non pas aucun traitement. Il s'agit d'une femme âgée et la chute tensionnelle est
significative. Le taux d'hémoglobine est bas et expose à des risques d'hypoxie des organes centraux. Il faut transfuser des
culots globulaires.
La transfusion de sang total ne se pratique plus.
Corticoïdes, coagulants, concentrés plaquettaires seraient inutiles.

Quel traitement local immédiat faut-il instituer ?


A - Tamponnement nasal
B - Pommade nasale hémostatique
C - Cautérisation chimique
D - Cautérisation au bistouri électrique
E - Aucun traitement
Bonne(s) réponse(s) : A

Le tamponnement constitué de traitement local immédiat.


Les cautérisations chimiques ou électriques en phase hémorragiques sont peu efficaces.
Les pommades hémostatiques n'auront aucun effet sur le saignement.

Le traitement ultérieur à envisager repose sur :


A - Exérèse de la muqueuse nasale et greffe de peau
B - Radiothérapie externe
C - Oestrogénothérapie
D - Cryothérapie
E - Ligatures vasculaires
Bonne(s) réponse(s) : A C

La dermoplastie est le procédé chirurgical le plus classique.


L'oestrogénothérapie a été proposée par certains auteurs.

1047
Exclusivement sur DOC - DZ : www.doc-dz.com NADJI 85
RESIDANAT EN POCHE TOME II
Cas Clinique en QCM
Une femme de 40 ans vient consulter pour une hypoacousie bilatérale progressivement croissante. Elle est mère de trois
enfants, elle n'a jamais présentée d'antécédents de surdité ni d'otite et n'a jamais eu de pathologie médicale particulière (pas
de prise de médicaments ototoxiques). La malade allègue des bourdonnements d'oreille graves, intermittents, de plus en plus
fréquents au niveau des deux oreilles.
A l'examen les deux tympans sont strictement normaux et mobiles lors de l'épreuve de Valsalva. Le diagnostic le plus
probable est celui d'une otospongiose ou d'ankylose stapédo-vestibulaire.

Devant ces données cliniques, on peut retenir comme diagnostic différentiel plausible :
A - Une otite séreuse bilatérale
B - Une presbyacousie précoce
C - Une maladie de Ménière bilatérale
D - Une tumeur du tronc cérébral
E - Une otite fibro-adhésive
Bonne(s) réponse(s) : B

Dans l'otite fibroadhésive on retrouve un long passé d'otite, le tympan est remanié et non mobile à l'épreuve de Vasalva.
Il n'y a aucun épisode vestibulaire pouvant évoquer un Ménière.
Dans l'otite séreuse le tympan n'est pas mobile en Vasalva.
Il n'y a aucun signe neurologique, d'atteinte d'autre paire crânienne en faveur d'une tumeur du tronc cérébral.

L'audiométrie tonale réalisée montre de chaque côté interrogé :


A - Un accolement des courbes aériennes et osseuses
B - Une perte en décibels sur la conduction aérienne plus importante qu'en conduction osseuse
C - Une chute de l'audition uniquement sur les fréquences aiguës
D - Une perte en décibels sur la conduction osseuse plus importante qu'en conduction aérienne
E - Une encoche sur la fréquence 4 000 Hz
Bonne(s) réponse(s) : B

L'otospongiose réalise, du moins initialement, une hypoacousie transmissionnelle pure, touchant toutes les fréquences mais
prédominant sur les graves.
L'encoche bilatérale sur le 4000 Hz est plutôt évocatrice de traumatisme sonore chronique.
La proposition E tente d'introduire une confusion avec l'encoche de Carhart qui est une encoche sur le courbe osseuse,
typiquement située sur la fréquence 2000 Hz. Cette encoche peut se rencontrer de maniére non exceptionnelle au cours des
otospongioses (elle peut se situer sur la fréquence 4000 Hz mais elle est alors moins profonde et moins caractéristique).

Parmi les examens suivants lequel est inutile au diagnostic d'otospongiose ?


A - Impédancemétrie
B - Audiométrie tonale
C - Audiométrie vocale
D - Etude des réflexes stapédiens
E - Potentiels évoques auditifs
Bonne(s) réponse(s) : E

Le diagnostic de l'otospongiose est formulé sur le vu des données cliniques, de l'audiogramme, de l'impédancemétrie. Il est
confirmé lors de l'intervention chirurgicale.
Chez l'adulte les PEA visent à différencier les surdités cochléaires des surdités rétrocochléaires.

Pour affirmer le blocage de l'étrier dans la fosse ovale, quel est l'examen le plus fiable ?
A - Audiométrie vocale
B - Audiométrie tonale
C - Recherche du réflexe stapédien
D - Epreuve de Weber
E - Epreuve de Rinne
Bonne(s) réponse(s) : C

Le réflexe stapédien est quasiment toujours absent dans l'otospongiose même en cas d'ankylose débutante cependant dans
un certain nombre de cas le réflexe stapédien est retrouvé sous la forme d'un effet "on-off" (double déflexion négative en
début et en fin de stimulation).

Quel est le premier traitement à envisager ?


A - Médical par vasodilatateurs
B - Prothétique
C - Médical par le calcium
D - Médical par le magnésium
E - Chirurgical
Bonne(s) réponse(s) : E

L'intervention consiste à remplacer l'étrier bloqué par un piston qui repose sur la platine ouverte, par l'intermédiaire d'un
greffon d'aponévrose.
1048
Exclusivement sur DOC - DZ : www.doc-dz.com NADJI 85
RESIDANAT EN POCHE TOME II
Cas Clinique en QCM
Un homme de 43 ans sans antécédent particulier vient consulter pour des douleurs sourdes sous-orbitaires droites, évoluant
depuis 3 semaines, sans rythme précis, et s'accompagnant d'un mouchage purulent unilatéral du même côté. Il n'y a pas de
fièvre, ni d'altération de l'état général.
L'examen physique permet de constater qu'il n'existe ni oedème, ni rougeur au niveau de la pommette droite, et que le cul-de-
sac vestibulaire supérieur droit est souple et indolore.
Par ailleurs, l'examen endobuccal note un écoulement mucopurulent sur la paroi pharyngée postérieure ainsi que la présence
au niveau de la première molaire supérieure droite d'une couronne mise en place 6 mois plus tôt. Le reste de l'examen endo
et exobuccal est normal.

Quel diagnostic vous semble le plus probable parmi ces affections d'origine dentaire ?
A - Sinusite maxillaire chronique unilatérale
B - Sinusite allergique
C - Cellulite jugale
D - Empyème sinusien unilatéral
E - Pulpite aiguë
Bonne(s) réponse(s) : A

L'absence de fièvre, la conservation de l'état général, les données de l'examen ne permettent pas de retenir le diagnostic
d'empyème sinusien ou de cellulite jugale.
La pulpite aiguë est caractérisée par une douleur dentaire.

Parmi ces incidences radiologiques, quelle est la première à demander pour confirmer le diagnostic ?
A - Incidence de Worms - Bretton
B - Incidence occlusale antérieure
C - Incidence de Blondeau
D - Incidence de Schuller
E - Incidence dite "maxillaire défilé"
Bonne(s) réponse(s) : C

L'incidence de Blondeau explore le sinus maxillaire, un éventuel niveau, une opacité plus ou moins totale.
C'est le panoramique dentaire qui permet d'apprécier les rapports de la denture avec le sinus maxillaire.

En admettant que la première molaire supérieure droite soit à l'origine des symptômes constatés, quelle(s)
peu(ven)t être la(les) lésion(s) initiale(s) ?
A - Un kyste radiculo-dentaire
B - Un kyste corono-dentaire
C - Un granulome apical
D - Monoarthrite apicale
E - Un mauvais scellement de la couronne
Bonne(s) réponse(s) : A C D

Les lésions de l'apex dentaire peuvent évoluer vers la sinusite maxillaire.

En admettant au contraire que la première molaire supérieure droite ne soit pas à l'origine des symptômes
constatés et compte tenu de ceux-ci, quel est alors le diagnostic envisageable ?
A - Dent de sagesse supérieure droite incluse
B - Canine supérieure droite incluse
C - Cancer du sinus maxillaire
D - Parodontopathie
E - Aucune des propositions ci-dessus
Bonne(s) réponse(s) :

QUESTION ANNULEE

1049
Exclusivement sur DOC - DZ : www.doc-dz.com NADJI 85
RESIDANAT EN POCHE TOME II
Cas Clinique en QCM
Un homme de 45 ans d'origine maghrébine, consulte pour l'apparition récente d'une tuméfaction bilatérale cervicale haute
apparue depuis quelques semaines. Son état général est excellent.
Depuis 10 jours, il présente une plénitude auriculaire droite avec une sensation d'hypoacousie droite. La rhinoscopie
antérieure montre des fosses nasales libres.

Quel(s) diagnostic(s) doivent être évoqués en premier ?


A - Otite séromuqueuse
B - Toxoplasmose
C - Cancer de l'éthmoïde
D - Tumeur du cavum
E - Lymphome malin non hodgkinien
Bonne(s) réponse(s) : D E

L'otite séromuqueuse n'est pas le diagnostic de la pathologie initiale, elle en est la conséquence. Elle n'explique pas la
tuméfaction bilatérale haute.
Si dans la toxoplasmose il existe également des adénopathies il existe également des adénopathies cervicales hautes,
postérieures, multiples, mobiles, on n'explique pas l'existence d'une otite séromuqueuse.
Le cancer de l'éthmoïde s'accompagne peu de métastase ganglionnaire (au stade de découverte) on retrouve en général une
symptomatologie chronique.
UNCT et LMNH peuvent se présenter sur un tableau tout à fait voisin, l'UNCT venant cependant en tête par ordre de
fréquence. L'obstruction ou l'infiltration de l'orifice de la trompe d'Eustache par la tumeur explique la réaction séreuse au
niveau de l'oreille moyenne.

L'existence d'un sérodiagnostic positif au virus d'Epstein-Barr permet de retenir un seul diagnostic. Lequel ?
A - Otite séromuqueuse
B - Toxoplasmose
C - Cancer de l'éthmoïde
D - Cancer du cavum
E - Lymphome malin non hodgkinien
Bonne(s) réponse(s) : D

La spécificité de la sérologie EBV n'est bien sûr pas absolue et toutes les associations peuvent se voir. Cependant un taux
élevé d'anticorps spécifiques d'EBV est très souvent associé au cancer du cavum, en particulier les IgG /VCA (virion capside
antigen) et les IgG /EA (early - antigen).
Les titres en anticorps constituent un véritable marqueur tumoral car ils doivent diminuer dans l'année du traitement en cas de
bonne réponse thérapeutique. Une remontée du taux des anticorps précède la rechute.

Quel(s) examen(s) permettra(ont) d'affirmer le diagnostic ?


A - Examen audiovestibulaire
B - Scanner
C - Rhinoscopie postérieure
D - Echographie cervicale
E - Impédancemétrie
Bonne(s) réponse(s) : C

L'examen audiovestibulaire, l'impédancemétrie précisent le type d'atteinte auditive, mais le problème n'est pas là.
C'est la rhinoscopie postérieure, au besoin sous anesthésie générale, qui permet d'affirmer le diagnostic par les biopsies qui
sont aussi réalisées.
L'échographie cervicale précisera seulement les rapports des tuméfactions cervicales, leur échostructure.
Le scanner est maintenant indispensable au bilan d'extension tumorale ; sur le plan thérapeutique il permet également au
radiothérapeute de déterminer le volume-cible. Il est utile également à la surveillance thérapeutique. Le diagnostic étant
anatomopathologique le scanner ne peut affirmer le diagnostic.

La thérapeutique curatrice la plus adéquate comporte :


A - Chirurgie
B - Hormonothérapie
C - Chimiothérapie
D - Corticothérapie
E - Radiothérapie externe
Bonne(s) réponse(s) : C E

Il s'agit en effet d'un cancer chimio et radiosensible.


La chirurgie sur les reliquats ganglionnaires est en fait peu pratiquée. Le traitement du cancer du cavum n'est pas chirurgical.

1050
Exclusivement sur DOC - DZ : www.doc-dz.com NADJI 85
RESIDANAT EN POCHE TOME II
Cas Clinique en QCM
Un homme de 27 ans, maghrébin d'origine, en France depuis 6 mois, présente depuis 4 mois une hypoacousie gauche. Il
consulte pour une adénopathie spinale haute et gauche, de 3 cm de diamètre, fixée aux plans profonds.
L'état général est conservé ; l'examen clinique est normal par ailleurs. Il n'y a pas d'intoxication alcoolo-tabagique.

L'hypo-acousie gauche est liée à :


A - La compression du conduit auditif interne
B - L'atteinte du VIII
C - L'envahissement ou l'atteinte de la trompe d'Eustache
D - La destruction de l'oreille moyenne
E - La destruction de l'oreille interne
Bonne(s) réponse(s) : C

L'orifice tubaire est obstrué ou infiltré par l'extension tumorale responsable de l'otite séromuqueuse se traduisant par l'atteinte
auditive transmissionnelle.

Le premier diagnostic évoqué est :


A - Un kyste amygdaloïde
B - Une tumeur de la parotide, avec adénopathie métastatique
C - Une mastoïdite compliquant une otite chronique avec adénopathie
D - Un cancer du cavum avec adénopathie métastatique
E - Un carcinome de la base de langue avec adénopathie métastatique
Bonne(s) réponse(s) : D

Le cancer du cavum est lymphophile. Les métastases ganglionnaires sont hautes et postérieures. L'Afrique du nord constitue
une zone à risque intermédiaire d'UCNT et atteint dans 20 % des cas des sujets de moins de 20 ans.
Ce n'est pas le terrain d'un carcinome de la base de langue.
Les affections de l'oreille moyenne entraîne peu de réaction ganglionnaire, de plus on ne note ici aucun passé otologique.
L'examen clinique ne mentionne pas de tuméfaction parotidienne.

Pour étayer ce diagnostic, un ou plusieurs des examens suivants sont utiles :


A - Une biopsie de la base de langue
B - Une épipharyngoscopie ou rhinoscopie postérieure avec biopsie
C - Un dosage des anticorps dirigés contre le virus d'Epstein-Barr
D - Une sialographie
E - Une otoscopie
Bonne(s) réponse(s) : B C

C'est la rhinoscopie postérieure avec biopsies qui permet le diagnostic anatomo-pathologique.


Dans des formes sous-muqueuses il est nécessaire de faire des biopsies profondes.
La sérologie du virus Epstein Barr est étroitement correlée au carcinome du rhinopharynx
(cf question [125]).

Une telle adénopathie a plus de chance d'être contrôlée par radiothérapie seule s'il s'agit de la métastase :
A - D'un carcinome épidermoïde de la base de langue
B - D'un carcinome de la parotide
C - D'un carcinome indifférencié du cavum
D - D'un carcinome papillaire de la thyroïde
E - D'un mélanome du cuir chevelu
Bonne(s) réponse(s) : C

Les métastases ganglionnaires des carcinomes indifférenciés du cavum sont plus radio-sensibles que celles des autres
cancers proposés ici.

Un homme de 27 ans, maghrébin d'origine, en France depuis 6 mois, présente depuis 4 mois une hypoacousie gauche.
Il consulte pour une adénopathie spinale haute et gauche, de 3 cm de diamètre, fixée aux plans profonds. L'état général est
conservé; l'examen clinique est normal par ailleurs. Il n'y a pas d'intoxication alcoolo-tabagique. La ponction ganglionnaire
révèle la présence de cellules malignes.

L'hypo-acousie gauche est liée à :


A - La compression du conduit auditif interne
B - L'atteinte du VIII
C - L'envahissement ou l'atteinte de la trompe d'Eustache
D - La destruction de l'oreille moyenne
E - La destruction de l'oreille interne
Bonne(s) réponse(s) : C

Réponse évidente.

1051
Exclusivement sur DOC - DZ : www.doc-dz.com NADJI 85
RESIDANAT EN POCHE TOME II
Cas Clinique en QCM

Le premier diagnostic évoqué est :


A - Un cancer bronchopulmonaire avec adénopathie métastatique
B - Une tumeur de la parotide, avec adénopathie métastatique
C - Un choléstéatome
D - Un cancer du cavum avec adénopathie métastatique
E - Un carcinome de la base de langue avec adénopathie métastatique
Bonne(s) réponse(s) : D

L'hypo-acousie unilatérale chez un sujet originaire d'une région à risque de cancer du cavum doit motiver d'emblée un
examen du cavum.

Pour étayer ce diagnostic, un ou plusieurs des examens suivants sont utiles :


A - Une biopsie de l'amygdale
B - Une épipharyngoscopie, ou rhinoscopie postérieure avec biopsie
C - Un dosage des anticorps dirigés contre le virus d'Epstein-Barr
D - Une sialographie
E - Un audiogramme
Bonne(s) réponse(s) : B C

C - Constitue un marqueur tumoral.


E - L'audiogramme n'apporte pas d'élément diagnostique spécifique de l'affection en cause.

Une telle adénopathie a plus de chance d'être contrôlée par radiothérapie seule s'il s'agit de la métastase :
A - D'un carcinome épidermoïde de la base de langue
B - D'un carcinome de la parotide
C - D'un carcinome indifférencié du cavum
D - D'un carcinome papillaire de la thyroïde
E - D'un mélanome du cuir chevelu
Bonne(s) réponse(s) : C

A, B, D, E - L'acte chirurgical y est l'acte initial dans l'association thérapeutique, alors que l'on intervient sur les métastases de
carcinome du cavum, que sur les reliquats de la radiothérapie.

Une femme de 28 ans est hospitalisée en urgence pour des vertiges. Ceux-ci sont apparus de façon brutale, la veille, et
persistent accompagnés de vomissements. Dans les antécédents, on retrouve la notion d'otites fréquentes dans l'enfance
avec écoulement purulent intermittent de l'oreille gauche depuis 6 mois. L'examen clinique retrouve un syndrome vestibulaire
de type périphérique avec un écoulement purulent de l'oreille gauche. Après nettoyage, on observe une perforation
tympanique gauche, marginale, avec présence de débris blanchâtres dans la caisse en arrière de la perforation. L'oreille
controlatérale est normale ainsi que le reste de l'examen clinique ORL. L'audiogramme montre une surdité mixte du côté
gauche avec une composante transmissionnelle de l'ordre de 30 décibels sur toutes les fréquences : l'audition est normale à
droite.

Quel est le type de surdité donné par une interruption de la chaîne ossiculaire ?
A - Cophose
B - Surdité de transmission à 20 décibels
C - Surdité de transmission supérieure à 30 décibels
D - Surdité mixte
E - Surdité de perception
Bonne(s) réponse(s) : C

La surdité est de type transmissionnel avec un Rinne de -50 à -60 dB.

Parmi ces éléments séméiologiques, le(s)quel(s) est(sont) de nature vestibulaire ?


A - Déviation des index
B - Déviation à l'épreuve de Romberg
C - Déviation à la marche aveugle
D - Adiadococinésie
E - Nystagmus
Bonne(s) réponse(s) : A B C E

D - C'est un signe cérébelleux.


Dans les vertiges vestibulaires périphériques, la déviation spontanée des index, yeux fermés, se fait dans le sens opposé du
nystagmus, défini cliniquement par la secousse rapide. De même, la manoeuvre de Romberg, étudiant la statique debout yeux
fermés, montre une tendance à la chute du côté opposé au nystagmus. L'ensemble définit le syndrome harmonieux. La
marche aveugle sensibilise l'épreuve de Romberg. Le nystagmus des lésions vestibulaires périphériques est unidirectionnel,
horizontal et rotatoire, supprimé ou diminué par la fixation oculaire volontaire.

1052
Exclusivement sur DOC - DZ : www.doc-dz.com NADJI 85
RESIDANAT EN POCHE TOME II
Cas Clinique en QCM

Parmi ces éléments sémiologiques, le(s)quel(s) est(sont) en faveur d'un syndrome vestibulaire périphérique ?
A - Nystagmus spontané de forme rotatoire pure
B - Existence de signe neurovégétatifs associés
C - Syndrome vestibulaire harmonieux
D - Atteinte auditive associée
E - Acouphènes associés
Bonne(s) réponse(s) : B C D E

A - Lésion bulbaire.

L'histoire clinique fait évoquer en premier le diagnostic de :


A - Otite moyenne aiguë
B - Otite séro-muqueuse
C - Otite chronique simple
D - Cancer de l'oreille moyenne
E - Otite cholestéatomateuse
Bonne(s) réponse(s) : E

La perforation est marginale et doit faire craindre un cholestéatome qui est quasi certain devant la présence de débris
blanchâtres épidermiques.

Parmi les complications des otites cholestéatomateuses, laquelle(lesquelles) nécessite(nt) un traitement


chirurgical rapide ?
A - Surdité
B - Otorrhée
C - Paralysie faciale
D - Vertiges
E - Syndrome méningé
Bonne(s) réponse(s) : A C D E

- Une surdité, des vertiges doivent faire redouter une effraction de l'appareil cochléo-vestibulaire.
- Le syndrome méningé s'explique par une érosion du tegmen ; la paralysie faciale peut être liée à une lyse du canal facial.

Quelle est la nature histologique du cholestéatome ?


A - Epithélium cylindrique
B - Epithélium glandulaire
C - Epithélium malpighien
D - Infiltrat lymphoplasmocytaire
E - Granulome à cholestérine
Bonne(s) réponse(s) : C

Les granulomes à cholestorine sont des granulations jaunâtres souvent associées au cholestéatome.

1053
Exclusivement sur DOC - DZ : www.doc-dz.com NADJI 85
RESIDANAT EN POCHE TOME II
Cas Clinique en QCM
Monsieur CAM. , 55 ans, ouvrier agricole, consulte pour une masse latéro-cervicale droite apparue il y a trois mois. Il n'a pas
de douleur, mais signale une anorexie et une asthénie. Il fume plus d'un paquet de cigarettes par jour et avoue boire 2 litres
de vin par jour. Il se plaint de dysphagie sans troubles respiratoires. Antécédents : bronchite chronique non traitée. L'examen
montre :
- une pharyngite érythémateuse, un état buccodentaire très altéré, et une stase salivaire qui empêche la visualisation de
l'hypopharynx. Par contre l'examen du larynx révèle une diminution de la mobilité de l'hémilarynx droit
- larynx et cordes vocales rouges
- une masse de 4 cm de diamètre indolore au bord antérieur du sterno-cléido-mastoïdien, près de son insertion supérieure,
non battante, sans caractère inflammatoire, bien mobile, et sans modification de la peau en regard
- apyrexie
- le foie déborde de 4 cm
L'examen complémentaire :
- Numération Formule : pas d'anémie
- Hyperleucocytose : 15 000 blancs, 70 % de polynucléaires
- Vitesse de sédimentation : lère heure : 60 mm 2e heure : 120 mm
- macrocytose à 120 microns3.
Vous évoquez le diagnostic d'un carcinome des voies aérodigestives supérieures.

Le ou les signes suivants va ou vont vous orienter directement vers le diagnostic :


A - L'accroissement de la vitesse de sédimentation
B - La pharyngolaryngite
C - La stase salivaire
D - L'immobilité laryngée
E - La dysphagie
Bonne(s) réponse(s) : C D E

La stase salivaire masquant l'hypopharynx est classiquement évocatrice d'une lésion hypopharyngée.

Sur les seuls arguments cliniques quel premier diagnostic évoquez-vous ?


A - Un cancer thyroïdien
B - Un cancer de la base de langue
C - Un cancer de l'hypopharynx
D - Un cancer de l'amygdale palatine
E - Une autre localisation cancéreuse
Bonne(s) réponse(s) : C

Terrain alcoolo-tabagique, présence d'une adénopathie cervicale, dysphagie, diminution de mobilité de l'hémilarynx, sont
autant d'éléments en faveur de cette localisation.

Parmi les raisons suivantes, quelle(s) est(sont) celle(s) qui pourrai(en)t vous interdire la chirurgie ?
A - L'altération trop importante de l'état général
B - Une tumeur de plus d'1 cm de diamètre
C - Une métastase pleuro-pulmonaire
D - Une adénopathie bilatérale
E - Une adénopathie de plus de 5 cm de diamètre
Bonne(s) réponse(s) : A C

C - L'atteinte de la plèvre est extrêmement péjorative. Une métastase pulmonaire unique peut être encore compatible avec un
traitement chirurgical.
D E - Le traitement chirurgical des aires ganglionnaires cervicales reste possible.

Parmi les explorations ci-dessous, quelle est celle qui permet d'affirmer le diagnostic ?
A - Transit pharyngo-laryngé lipiodolé
B - Endoscopie et biopsie des voies aérodigestives supérieures
C - Tomographies du larynx
D - Tomodensitométrie cervicale
E - Scintigraphie thyroïdienne
Bonne(s) réponse(s) : B

Le diagnostic est anatomo-pathologique.

1054
Exclusivement sur DOC - DZ : www.doc-dz.com NADJI 85
RESIDANAT EN POCHE TOME II
Cas Clinique en QCM
Un jeune homme de 22 ans est hospitalisé en urgence dans un état comateux à la suite d'un accident de la voie publique. Il
émerge du coma au bout de 2 jours l'examen montre alors une paralysie complète de l'hémiface droite avec impossibilité
d'occlusion oculaire du côté droit. Il se plaint par ailleurs de vertiges et d'une diminution de l'audition du côté droit. L'examen
local retrouve du sang dans le conduit auditif externe avec un hémotympan. Les radiographies mettent en évidence un trait de
fracture sur le rocher droit.

L'origine périphérique de la paralysie faciale est affirmée sur :


A - Le caractère total de la paralysie
B - L'impossibilité d'occlusion oculaire
C - La découverte secondaire de la paralysie
D - L'existence d'un trismus
E - L'anesthésie de l'hémiface
Bonne(s) réponse(s) : A B

A - Atteinte du facial supérieur et inférieur.


B - Alors que la paralysie faciale centrale prédomine sur le facial inférieur.
E - Voir trijumeau.

Quels examens complémentaires doivent être demandés rapidement ?


A - Tomodensitométrie du rocher
B - Tomographie du rocher
C - Electromyographie faciale
D - Audiométrie tonale
E - Potentiels évoqués somesthésiques
Bonne(s) réponse(s) : A C D

A - Recherche un trait de fracture du rocher expliquant l'atteinte faciale et auditive.


B - A discuter. Se fera de moins en moins à l'avenir.
C - Apprécie les signes de dénervation. Valeur pronostique.
D - Précise l'atteinte auditive, transmission ou perception.

Quelle(s) complication(s) peut(peuvent) entraîner les fractures du rocher ?


A - Cophose
B - Aréflexie vestibulaire
C - Névralgie faciale
D - Liquorrhée cérébrospinale
E - Luxation ossiculaire
Bonne(s) réponse(s) : A B D E

A B - Atteinte de l'appareil cochléovestibulaire.


D - Par fracture de la corticale méningée avec brèche durale.
E - Atteinte auditive transmissionnelle.

L'apparition secondaire de la paralysie faciale est en faveur d'un des types de lésion suivants, lequel ?
A - Perte de substance
B - Section incomplète
C - Hématome
D - Section complète
E - Aucune des propositions ci-dessus
Bonne(s) réponse(s) : E

Ces atteintes secondaires sont expliquées par le développement d'un oedème et justifie un traitement anti-inflammatoire.

1055
Exclusivement sur DOC - DZ : www.doc-dz.com NADJI 85
RESIDANAT EN POCHE TOME II
Cas Clinique en QCM
Un homme de 50 ans, maçon, consulte pour la première fois en raison d'une otalgie gauche, apparemment isolée, évoluant
depuis trois semaines.
A l'anamnèse, on note :
- une hernie hiatale traitée médicalement 4 ans auparavant
- plusieurs accidents du travail, bénins, sans traumatisme récent du crâne
- l'absence de passé pathologique auriculaire.
Ce patient est en bon état général, apyrétique. Il boit 4 à 5 canettes de bière par jour, un litre de vin et fume 25 cigarettes par
jour.
L'examen clinique ne montre pas d'anomalies des pavillons, ni des conduits auditifs externes; les tympans sont normaux.
L'examen du pharynx ne peut être pratiqué dans de bonnes conditions, du fait de réflexes nauséeux très importants. La région
cervicale est souple; il n'y a pas d'adénopathie palpable.
Vous demandez à ce malade de revenir, à jeun avec une préparation sédative, afin de compléter votre examen sous
anesthésie locale.

A ce stade de l'anamnèse et de l'examen clinique, quel(s) est(sont) le(s) diagnostic(s) que l'on peut évoquer ?
A - Une otospongiose
B - Une otomastoïdite
C - Un zona auriculaire
D - Une algie symptomatique d'origine extra-auriculaire
E - Une névralgie essentielle du nerf glossopharyngien
Bonne(s) réponse(s) : D

A - Indolore.
B - Les anomalies tympaniques ne sont pas retrouvées dans cette observation.
C - Pas de lésion vésiculeuse décrite.
D - Douleur de la base de la langue irradiant vers l'oreille évoluant par crise paroxystique d'une minute avec début et fin
brusque ; une zone gachette est souvent retrouvée.

Parmi les éléments suivants de cette observation, quel(s) est(sont) celui(ceux) qui vous ont permis de retenir à
ce stade ce(s) diagnostic(s) d'orientation ?
A - Intoxication alcoolo-tabagique
B - Sexe
C - Caractère isolé de cette otalgie
D - Absence d'antécédents pathologiques
E - Antécédents digestifs
Bonne(s) réponse(s) : A C

B - Si le cancer des voies aérodigestives est plus fréquent chez l'homme, toute otalgie à tympan normal n'est pas synonyme
de cancer.
C - L'otalgie à tympan normal doit faire rechercher une cause pharyngo-laryngée.

L'examen buccal pratiqué 2 jours plus tard montre une lésion ulcéro-bourgeonnante de l'amygdale gauche, de 2
cm de diamètre, indurée et hémorragique à la palpation. Le reste de l'examen ORL est normal. Compte tenu de
ces éléments complémentaires, quel diagnostic vous paraît maintenant le plus probable ?
A - Amygdalite chronique cryptique
B - Chancre syphilitique
C - Angine de Vincent
D - Cancer de l'amygdale
E - Phlegmon d'origine dentaire
Bonne(s) réponse(s) : D

Réponse évidente.

Parmi les affections suivantes, laquelle(lesquelles) ne s'accompagne(nt) pas d'otalgie ?


A - Angine virale
B - Cancer de la corde vocale
C - Cancer de l'hypopharynx
D - Otite moyenne aiguë
E - Pathologie de l'articulation temporomaxillaire
Bonne(s) réponse(s) : B

A - L'irradiation de la douleur aux oreilles est classique.

1056
Exclusivement sur DOC - DZ : www.doc-dz.com NADJI 85
RESIDANAT EN POCHE TOME II
Cas Clinique en QCM

Quel sont parmi les examens complémentaires suivants, ceux que vous demanderez en priorité chez ce malade
?
A - Frottis de gorge et examen bactériologique
B - Biopsie
C - Transit pharyngo oesophagien
D - Scintigraphie osseuse
E - Endoscopie de la sphère ORL
Bonne(s) réponse(s) : B E

B - Permet de faire le diagnostic anatomo-pathologique.


E - A la recherche d'une autre localisation.

Marion B. , âgée de 10 mois (poids 8 kg, taille 70 cm), a été bien portante jusqu'à l'âge de 3 mois. De 3 à 9 mois, elle a eu de
nombreuses infections rhinopharyngées fébriles traitées par antibiotiques.
L'épisode actuel se déroule ainsi :
Le 15 janvier, Marion a une rhinorrhée purulente, le pharynx rouge, encombré de mucopus, une fièvre à 38°5. Elle reçoit des
antithermiques et un traitement local (instillation nasale de Soframycine simple).
Le 19 janvier la persistance de la fièvre conduit à faire une numération formule sanguine (GR : 4 800 000 ; BG : 11 000, PN :
60 %, L : 40 %) et un examen des urines (qui sont stériles). La radiographie du thorax est également normale. Le même
traitement est poursuivi.
Le 22 janvier, survenue de vomissements, fièvre à 39° ; plusieurs fois Marion s'est réveillée dans la nuit en criant.
Consultation du 23 janvier :
Fièvre à 39°. Une selle liquide durant l'examen.
Pharyngite persistante. Examen cardio-pulmonaire normal.
Abdomen, orifices herniaires normaux. Ni éruption, ni purpura. Aucun signe méningé. Présence de quelques ganglions sous-
maxillaires.

Considérant que cet enfant n'a ni syndrome méningé, ni infection urinaire, ni signe respiratoire, quel examen est
indispensable pour porter le diagnostic ?
A - Hémocultures
B - Dosage de la C réactive protéine
C - Sédimentation globulaire
D - Coproculture
E - Aucun des examens ci-dessus
Bonne(s) réponse(s) : E

Il faut examiner les tympans.

Quel diagnostic vous parait le plus probable ?


A - Septicémie
B - Otite
C - Sinusite maxillaire
D - Ethmoïdite
E - Sinusite frontale
Bonne(s) réponse(s) : B

Hyperthermie, douleur, signes digestifs, au cours d'une rhinopharyngite traînante en sont évocateurs.

Au cours d'une otite, l'aspect du tympan peut être :


A - Rosé
B - Rouge et oedématié
C - Bombant avec disparition des reliefs osseux
D - Phlycténulaire
E - Aucun de ces aspects n'est observé
Bonne(s) réponse(s) : A B C D

Sans commentaire.

1057
Exclusivement sur DOC - DZ : www.doc-dz.com NADJI 85
RESIDANAT EN POCHE TOME II
Cas Clinique en QCM

Laquelle (ou lesquelles) des propositions suivantes est (sont) exacte(s) ?


A - Des pharyngites répétées peuvent se compliquer d'une otite séreuse
B - Au cours d'une otite séreuse, la ponction à travers le tympan permet de retirer un liquide aseptique
C - Une adénoïdite ne peut être responsable d'otites répétées
D - L'otite séreuse peut se compliquer de mastoïdite
E - Une otite peut être la cause d'une paralysie faciale périphérique
Bonne(s) réponse(s) : B D E

A - Ce sont plutôt des rhinopharyngites qui peuvent entraîner ce genre d'otite.


C - Une adénoïdite est au contraire souvent en cause.
D - Oui, au cours d'une surinfection.

Tous les éléments suivants font partie du tableau d'otite séreuse sauf un. Lequel ?
A - Diminution de la mobilité tympanique
B - Aplatissement du tympanogramme
C - Surdité de perception
D - Hypoventilation tubaire
E - Otalgie
Bonne(s) réponse(s) : C

C'est bien sûr une hypoacousie de transmission.

Quelle est (ou quelles sont) l'(les) indication(s) justifiant une paracentèse en cas d'otite aiguë suppurée ?
A - Fièvre élevée
B - Angine associée
C - Otalgie intense ne répondant pas au traitement
D - Bronchopneumopathie associée
E - Fièvre ne répondant pas au traitement
Bonne(s) réponse(s) : C E

A - Il faut d'abord essayer de faire baisser la fièvre médicalement.

Un enfant de 10 ans présente depuis 5 années une tuméfaction cervicale antérieure, médiane, juxtahyoïdienne, de volume
variable (une noisette), indolore, mobile à la déglutition, adhérente à l'os hyoïde.
Il est adressé en consultation ORL car, depuis 4 jours, cette tuméfaction a pris un aspect inflammatoire; elle est douloureuse,
spontanément et à la palpation et vient de se fistuliser à la peau dans sa partie centrale, laissant s'écouler du pus franc. Les
signes généraux sont discrets et l'état général est peu atteint, avec température à 38°C.
Le reste de l'examen ORL est normal, ainsi que l'examen des dents.

Quel diagnostic allez-vous porter ?


A - Laryngocèle
B - Adénite soushyoïdienne
C - Thyroïdite
D - Kyste du sinus cervical
E - Kyste du tractus thyréoglosse
Bonne(s) réponse(s) : E

D - Kyste congénital latéro-cervical.


E - Kyste congénital, médian, mobile à la déglutition, pouvant se fistuliser à la peau lors d'une surinfection.

Parmi les signes suivants, vous avez retenu pour porter ce diagnostic ?
A - La tuméfaction est médiane
B - L'écoulement est purulent
C - Les signes généraux sont peu intenses
D - La tuméfaction adhère à l'os hyoïde
E - La tuméfaction s'élève pendant la déglutition
Bonne(s) réponse(s) : A D E

D - S'explique par la migration embryologique de la thyroïde.

1058
Exclusivement sur DOC - DZ : www.doc-dz.com NADJI 85
RESIDANAT EN POCHE TOME II
Cas Clinique en QCM

Quel est l'examen indispensable à pratiquer avant toute décision thérapeutique ?


A - Une radiographie du cou face et profil
B - Une radiographie de la déglutition
C - Une endoscopie oesophagienne
D - Une scintigraphie thyroïdienne
E - Un scanner cervical
Bonne(s) réponse(s) : D

Il faut en effet s'assurer qu'il ne s'agit pas d'une thyroïde ectopique et qu'il existe une glande thyroïde normale.

Quelle est, parmi les propositions thérapeutiques suivantes, celle qu'il convient de retenir ?
A - La ponction de la collection
B - L'incision de la collection
C - La corticothérapie
D - L'ablation de la masse
E - L'exérèse chirurgicale incluant l'ablation du corps de l'os hyoïde
Bonne(s) réponse(s) : E

E - L'ablation en monobloc du trajet fistuleux, du kyste et du corps de l'os lyoïde permet d'éviter des récidives qui sont
inéluctables si l'on se contente d'enlever seulement la masse kystique.

Une jeune femme de 26 ans consulte pour une surdité bilatérale prédominant à droite, d'aggravation progressive, dont le
début remonte à 3 ans, au cours de sa première grossesse et des acouphènes de timbre grave apparus depuis quelques
mois, intermittents et peu gênants.
Elle n'a aucun autre problème de santé, et notamment pas de passé pathologique ORL, traumatique ou toxique. Sa mère et
une tante maternelle ont également des difficultés ; l'une d'elles a été opérée de l'oreille, avec un bon résultat fonctionnel .
Cette malade répond à vos questions sans difficultés; elle n'élève pas la voix.
L'examen clinique ORL est normal : les conduits auditifs sont perméables et les tympans normaux ; le nez et le cavum sont
libres et propres ; il n'a pas d'atteinte des nerfs crâniens.
L'acoumétrie aux diapasons montre un Weber localisé à droite, un Rinne négatif des deux côtés.
Vous évoquez une surdité de transmission due à une otospongiose.

Parmi les éléments suivants de cette observation, lequel vous a permis de soupçonner la nature
transmissionnelle de cette surdité ?
A - Surdité bilatérale
B - Surdité asymétrique
C - Acouphènes de timbre grave
D - Eléments familiaux de surdité
E - Pas de passé auriculaire
Bonne(s) réponse(s) : C

Ce n'est cependant pas l'élément de l'observation le plus évocateur de surdité transmissionnelle.

Parmi les éléments de ce dossier, lequel (ou lesquels) est ( ou sont) caractéristique (s) de cette surdité
transmissionnelle ?
A - Tympans normaux
B - Rinne négatif des deux côtés
C - Weber latéralisé à droite
D - Nez et cavum libres et propres
E - Pas d'atteinte des nerfs crâniens
Bonne(s) réponse(s) : A

L'otospongiose est une surdité de transmission à tympan normal.

Parmi les éléments suivants du dossier, lequel (ou lesquels) a (ou ont) orienté votre diagnostic étiologique ?
A - L'âge
B - La bilatéralité de la surdité
C - Répond à vos questions sans difficulté
D - Le début lors d'une grossesse
E - Les éléments de l'anamnèse familiale
Bonne(s) réponse(s) : A D E

- Chez la femme, l'aggravation parallèlement aux épisodes de la vie génitale est évocatrice d'otospongiose, de même que les
antécédents familiaux.
- C'est l'âge de révélation de l'otospongiose, la plupart des cas étant notés entre 20 et 30 ans.

1059
Exclusivement sur DOC - DZ : www.doc-dz.com NADJI 85
RESIDANAT EN POCHE TOME II
Cas Clinique en QCM

Parmi les examens complémentaires suivants que vous pouvez demander, lequel (ou lesquels) amènera (ou
amèneront) le (ou les) argument(s) décisif(s) pour porter ce diagnostic d'otospongiose ?
A - Audiométrie tonale liminaire
B - Tympanométrie et étude des réflexes stapédiens
C - Potentiels évoqués auditifs
D - Andrométrie vocale
E - Examen radiologique de l'oreille
Bonne(s) réponse(s) : A B

A - Met en évidence une surdité transmissionnelle pure ou mixte. Une encoche à 2000Hz sur la courbe de conduction
osseuse (encoche de Carhart) est évocatrice.
B - Le tympanogramme est normal, avec souvent un pic peu élevé. Le réflexe stapédien est en règle générale absent par
fixation de la platine.
D - Confirme l'audiométrie tonale.

Laquelle des modalités thérapeutiques suivantes peut-on proposer à cette malade ?


A - Tympanoplastie
B - Remplacement de l'étrier
C - Prothèse auditive
D - Remplacement de l'enclume
E - Aérateur transtympanique
Bonne(s) réponse(s) : B

Stapédectomie, ouverture de la platine ; remplacement de l'étrier par un piston maintenu à l'enclume, entrant en contact à son
autre extrémité avec l'oreille interne soit directement (piston transplatinaire), soit par l'intermédiaire d'un greffon.

Quelle sera en principe l'évolution de cette surdité, lorsque le traitement aura été réalisé ?
A - Restitution d'une audition socialement normale
B - Stabilisation de la surdité
C - Aggravation progressive de la surdité
D - Surdité profonde d'évolution rapide
E - Evolution imprévisible
Bonne(s) réponse(s) : A

On obtient normalement une fermeture du Rinne audiométrique.

Un homme de 70 ans, se plaint depuis plusieurs semaines de douleurs de la joue droite apparues, dit-il après un
refroidissement. Son nez reste bouché et coule à droite depuis lors ; le mouchage, modéré, est mucopurulent et quelquefois
strié de sang.
Ces douleurs irradient vers la narine et la mâchoire supérieure, survenant par paroxysme de quelques minutes, sans cause
apparente, et dans l'intervalle de ces crises, multiquotidiennes, les régions jugales et gingivales droites restent comme
endolories.
Ce patient n'a pas d'antécédents notables, sauf un écoulement nasal bilatéral séreux depuis quelques années, n'ayant réagi à
aucun traitement ; il n'a pas de passé allergique.
A l'examen, la face est symétrique, hormis un léger empâtement de la joue droite, remarqué uniquement à la palpation, qui est
d'ailleurs assez douloureuse au niveau de la région sous-orbitaire. On note de plus une hypoesthésie des téguments de la
joue droite. La rhinoscopie montre un discret bombement du cornet moyen droit obstruant en partie la fosse nasale ; la
muqueuse est par ailleurs d'aspect normal. Il est édenté complet de la mâchoire supérieure. L'examen général général est
normal, la TA est de 16/11 mmHg.

Quelle est la nature de cette douleur ?


A - Névralgie du V1
B - Névralgie du V2
C - Névralgie du V3
D - Névralgie du glossopharyngien
E - Algie faciale de type neurovasculaire
Bonne(s) réponse(s) : B

V2 : Territoire du maxillaire supérieur = partie supérieure de la joure, partie latérale du nez, lêvre supérieure, gencive
supérieure et muqueuse.

1060
Exclusivement sur DOC - DZ : www.doc-dz.com NADJI 85
RESIDANAT EN POCHE TOME II
Cas Clinique en QCM

Quelles sont, parmi les propositions suivantes, celles qui vous permettent de retenir la nature symptomatique de
cette douleur ?
A - Fond douloureux permanent
B - Allure paroxystique
C - Hypoesthésie cutanée
D - Irradiation vers la mâchoire
E - Pas de facteur déclenchant
Bonne(s) réponse(s) : C

A - Dans les névralgies essentielles, il existe des intervalles libres.


L'hypoesthésie cutanée dans ce territoire doit faire suspecter une atteinte organique ou différencie ainsi les névralgies
essentielles des névralgies symptomatiques d'une affection.

Quels sont, parmi les éléments cliniques suivants, ceux qui peuvent orienter votre diagnostic étiologique ?
A - Empâtement de la joue
B - Rhinorrhée séreuse de longue date
C - Chicots dentaires
D - Mouchage unilatéral purulosanglant
E - Obstruction nasale
Bonne(s) réponse(s) : A D E

B - En faveur d'une rhinite banale.


C - Il s'agit d'un édenté complet de la mâchoire supérieure.

Parmi les examens suivants, lequel (ou lesquels) demanderez-vous pour préciser le diagnostic ?
A - Imagerie sinusienne
B - Frottis nasal pour examen bactériologique
C - Sinusoscopie avec biopsie
D - Electromyographie du masseter
E - Tests allergologiques
Bonne(s) réponse(s) : A C

A - Radiographie des sinus, scanner des sinus.


C - Les biopsies sont indispensables pour le diagnostic anatomo-pathologique.

Quel est le diagnostic le plus probable de cette affection ?


A - Sinusite maxillaire chronique
B - Polypose nasosinusienne
C - Cancer du sinus maxillaire
D - Cancer de l'éthmoïde
E - Kyste radiculodentaire sur racine restante
Bonne(s) réponse(s) : C

A - Le caractère douloureux n'est pas en faveur d'une sinusite maxillaire chronique.


D - Les douleurs sont plutôt situées dans le territoire du nerf ophtalmique. Les téguments infiltrés sont ceux de la région
orbitraire, rebords orbitraires inférieur et interne.

Vous êtes appelé en consultation auprès d'un enfant de 5 ans atteint de fièvre à 40 degrés avec toux évoluant depuis 48
heures. A l'examen, vous découvrez un écoulement nasal purulent bilatéral, un pharynx rouge, sans hypertrophie
amygdalienne et du pus dans le cavum. Les tympans sont congestifs. L'examen pulmonaire est normal. Il n'y a pas de signes
méningés et le reste de l'examen clinique n'est pas perturbé.
A l'interrogatoire vous apprenez que cet enfant a fait depuis 2 ans de nombreuses rhinopharyngites compliquées d'otites
purulentes.

Vous traitez cet enfant par :


A - Acide acétyl salicylique 50 à 75 mg/kg/jour en 4 à 6 prises
B - Macrolides
C - Gentalline IM 3 mg/kg/jour en 2 injections
D - Corticoïdes 2 mg/kg/jour pendant 15 jours
E - Sirop de codéine
Bonne(s) réponse(s) : A B

- Aminosides, corticoïdes à cette dose ne sont pas adaptés au traitement de cette infection.
- La toux est liée aux sécrétions rhinopharyngées et ne doit pas être empêchée.

1061
Exclusivement sur DOC - DZ : www.doc-dz.com NADJI 85
RESIDANAT EN POCHE TOME II
Cas Clinique en QCM

Parmi les circonstances favorisantes de ces infections ORL répétées, vous recherchez :
A - Une vie en collectivité
B - Un tabagisme familial
C - Un terrain atopique
D - Un déficit immunitaire
E - Une anémie par carence en fer
Bonne(s) réponse(s) :

QUESTION ANNULEE

A titre de prévention, vous proposez pour cet enfant :


A - L'adénoïdectomie
B - Une corticothérapie
C - Une recherche de reflux gastro-oesophagien
D - Une antibiothérapie au long cours
E - L'abstention de tout examen et de tout traitement
Bonne(s) réponse(s) : A

L'otite séro-muqueuse chronique avec surinfections fréquentes est une indication à l'adénoïdectomie.

L'évolution de cette rhinopharyngite peut être marquée par toutes les complications suivantes, sauf une.
Laquelle ?
A - Adénite cervicale
B - Bronchopneumophatie aiguë
C - Otite
D - Syndrome de Grisel
E - Pyélonéphrite
Bonne(s) réponse(s) : E

D - Torticolis atloïdo-axoïdien lié à une inflammation du rhino-pharynx.

Un sujet de 30 ans polytraumatisé est hospitalisé pendant plusieurs semaines, d'abord en réanimation, puis dans un service
de soins intensifs orthopédiques.
Au cours de son séjour, à la suite d'une ostéomyélite sur ostéosynthèse, il reçoit plusieurs variétés d'aminosides.
Depuis sa sortie, il a remarqué une détérioration de son audition et une instabilité importante à la marche, sans aucun vertige.

Quel est le type de surdité le plus évocateur d'une atteinte traumatique ?


A - Une surdité de perception élective à la fréquence 500
B - Une surdité de perception élective à la fréquence 1000
C - Une surdité de perception élective à la fréquence 2000
D - Une surdité de perception élective à la fréquence 4000
E - Une surdité de perception élective à la fréquence 8000
Bonne(s) réponse(s) : D

Encoche sur le 4000 Hz évocatrice de traumatisme sonore.

La surdité de ce patient est bilatérale et symétrique. Elle atteint surtout les fréquences aiguës.
Quelles peuvent être les causes de cette surdité ?
A - Un état antérieur à l'accident
B - Une contusion du tronc cérébral
C - Une contusion cochléaire bilatérale
D - Une fracture bilatérale du rocher
E - Une atteinte par les aminosides
Bonne(s) réponse(s) :

QUESTION ANNULEE.

1062
Exclusivement sur DOC - DZ : www.doc-dz.com NADJI 85
RESIDANAT EN POCHE TOME II
Cas Clinique en QCM

Que pensez-vous que l'examen vestibulaire va mettre en évidence ?


A - Aucun signe vestibulaire spontané
B - Un nystagmus horizontal rotatoire unilatéral
C - Une déviation segmentaire unilatérale
D - Une déviation axiale unilatérale
E - Une aréflexie calorique bilatérale
Bonne(s) réponse(s) : A E

La symptomatologie vestibulaire y est très discrète du fait de l'atteinte bilatérale et symétrique avec un développement
progressif de la compensation centrale. Tout se réduit le plus souvent à des troubles de l'équilibre non systématisés ou à des
troubles visuels à type d'oscilloscopie.

Parmi les manifestations suivantes, quelles sont celles qui peuvent traduire une lésion de l'oreille interne ?
A - Un nystagmus
B - Une déviation lente des index
C - Une déviation à la manoeuvre de Romberg
D - Des vomissements
E - Une perte de connaissance
Bonne(s) réponse(s) : A B C D

E - Les pertes de connaissances y sont très rares.

Chez ce patient, quelles sont les mesures qui vous paraissent appropriées ?
A - Repos au lit
B - Antihistaminiques
C - Benzodiazépines
D - Reprendre l'activité physique dès que possible
E - Rééducation vestibulaire
Bonne(s) réponse(s) : D E

La rééducation vestibulaire peut être utile au début de la reprise de l'activité physique, même chez le sujet jeune.

Une femme de 40 ans vient consulter pour une hypoacousie bilatérale.


Elle est mère de trois enfants. Elle n'a jamais présenté d'antécédents de surdité ni d'otite et n'a jamais eu de pathologie
médicale particulière (pas de prise de médicaments ototoxiques). La malade allègue des bourdonnements d'oreille graves,
intermittents, de plus en plus fréquents des deux oreilles.
A l'examen les deux tympans sont strictement normaux.
- L'épreuve de Valsalva est positive des deux côtés
- L'épreuve de Weber montre une absence de latéralisation
- Le Rinne est négatif des deux côtés
- Le diagnostic le plus probable est celui d'une otospongiose ou d'ankylose stapédo-vestibulaire.

Devant ces données cliniques, on peut retenir comme diagnostic différentiel plausible :
A - Une malformation ossiculaire
B - Une presbyacousie précoce
C - Une maladie de Ménière bilatérale
D - Une tumeur du tronc cérébral
E - Une tympanosclérose
Bonne(s) réponse(s) : A E

Il s'agit en effet d'atteintes transmissionnelles. La malformation ossiculaire est responsable d'une hypoacousie existant dès la
naissance, non évolutive. La tympanosclérose est une séquelle d'otite chronique simple pouvant entraîner un blocage
ossiculaire. Cependant, le tympan est exceptionnellement normal, il présente des plaques blanches plus ou moins diffuses.

L'audiométrie tonale réalisée doit montrer de chaque côté interrogé :


A - Un accolement des courbes aériennes et osseuses
B - Une perte de décibels sur la conduction aérienne plus importante qu'en conduction osseuse
C - Une chute de l'audition uniquement sur les fréquences aiguës
D - Une perte en décibels sur la conduction osseuse plus importante qu'en conduction aérienne
E - Une cophose
Bonne(s) réponse(s) : B

C'est la caractéristique audiométrique des hypoacousies de transmission.

1063
Exclusivement sur DOC - DZ : www.doc-dz.com NADJI 85
RESIDANAT EN POCHE TOME II
Cas Clinique en QCM

Parmi les examens suivants lesquels sont inutiles au diagnostic d'otospongiose :


A - Impédancemétrie
B - Audiométrie tonale
C - Audiométrie par réflexes d'orientation conditionnée (ROC)
D - Etude des réflexes stapédiens
E - Potentiels évoqués auditifs
Bonne(s) réponse(s) : C E

C - Technique audiométrique concernant le nourrisson.

Pour affirmer le blocage de l'étrier dans la fosse ovale, quel est l'examen le plus fiable :
A - Audiométrie vocale
B - Audiométrie tonale
C - Recherche du réflexe stapédien
D - Epreuve de Weber
E - Epreuve de Rinne
Bonne(s) réponse(s) : C

Cf. question [168]

Le traitement de l'otospongiose dans ce cas est :


A - Médical par vaso-dilatateurs
B - Prothétique
C - Médical par le calcium
D - Médical par le magnésium
E - Chirurgical
Bonne(s) réponse(s) : E

Cf. question [169].

1064
Exclusivement sur DOC - DZ : www.doc-dz.com NADJI 85
RESIDANAT EN POCHE TOME II
Cas Clinique en QCM

1065
Exclusivement sur DOC - DZ : www.doc-dz.com NADJI 85
RESIDANAT EN POCHE TOME II
Cas Clinique en QCM
Ce sujet de 64 ans présente brutalement des douleurs oculaires droites intenses, accompagnées d'une baisse visuelle
considérable, d'une rougeur de l'oeil droit et des nausées. La cornée droite apparaît trouble à l'examen, et à la palpation. Le
globe oculaire est dur comme une bille de verre. Il s'agit vraisemblablement d'une crise de glaucome aigu.

Parmi les 5 propositions suivantes concernant les étiologies des baisses visuelles brutales, quelle est celle qui
s'accompagne habituellement d'une rougeur oculaire et peut donc être envisagée chez ce patient ?
A - Une iridocyclite (uvéite antérieure)
B - Une hémorragie du vitré
C - Une oblitération de l'artère centrale de la rétine
D - Une thrombose de la veine centrale de la rétine
E - Une névrite optique aiguë
Bonne(s) réponse(s) : A

Il s'agit ici d'associer baisse d'acuité visuelle et rougeur oculaire : il faut évoquer le glaucome aigu par fermeture de l'angle,
l'uvéite antérieure et la kératite superficielle.

Parmi les 5 propositions suivantes concernant le mécanisme physiopathologique à l'origine de cette crise de
glaucome aigu, quelle est celle que vous retenez ?
A - Une hypersécrétion d'humeur aqueuse
B - Une hypoexcrétion d'humeur aqueuse par obstruction des veines aqueuses
C - Une hypoexcrétion d'humeur aqueuse par sclérose du trabéculum scléral
D - Une hypoexcrétion d'humeur aqueuse par fermeture de l'angle iridocornéen
E - Une hypoexcrétion d'humeur aqueuse par augmentation de la pression veineuse
Bonne(s) réponse(s) : D

L'humeur aqueuse est sécrétée normalement, mais l'excrétion est diminuée en raison d'une fermeture de l'angle iridocornéen,
qui survient sur une prédisposition anatomique : l'angle iridocornéen étroit avec chambre antérieure peu profonde.

Parmi les 5 propositions suivantes concernant les signes de l'examen ophtalmologique de ce patient avant
traitement, quel est le signe qui ne s'observe pas habituellement ?
A - Un cercle périkératique
B - Un oedème cornéen
C - Un myosis
D - Une augmentation considérable de la pression oculaire
E - Une aréflexie pupillaire
Bonne(s) réponse(s) : C

Au cours de la crise de glaucome aigu par fermeture de l'angle, la pupille est en demi-mydriase aréflexive.

Parmi les 5 thérapeutiques suivantes concernant le traitement de ce patient, quelle est celle qui est
formellement contre-indiquée ?
A - La prescription d'inhibiteurs de l'anhydrase carbonique
B - La perfusion de mannitol
C - L'instillation d'atropine
D - L'instillation de Pilocarpine®
E - La prescription de glycérine par voie buccale
Bonne(s) réponse(s) : C

Le traitement local repose sur la prescription de myotiques (par exemple pilocarpine) dès que la pression intraoculaire
commence à baisser, de façon à favoriser la réouverture de l'angle iridocornéen.

L'interrogatoire du patient va rechercher des médicaments à l'origine possible du déclenchement de cette crise.
Parmi ces thérapeutiques, laquelle ne peut être incriminée ?
A - Les inhibiteurs de la monoamine-oxydase
B - Les antiparkinsonniens
C - Les dérivés nitrés
D - L'éthambutol
E - Les antidépresseurs tricycliques
Bonne(s) réponse(s) : D

Si le lien de causalité entre la prise de dérivés nitrés et le déclenchement d'une crise de glaucome aigu est difficile à établir,
l'éthambutol n'intervient pas du tout sur la dilatation pupillaire, il peut par contre être à l'origine d'une neuropathie optique
toxique.

1066
Exclusivement sur DOC - DZ : www.doc-dz.com NADJI 85
RESIDANAT EN POCHE TOME II
Cas Clinique en QCM
Un sujet de sexe masculin, âgé de 65 ans, porteur de lunettes, accuse une diminution spontanée de l'acuité visuelle de l'oeil
gauche à 2/10. Le sujet signale qu'il a présenté quelques jours auparavant des petits signes cliniques prémonitoires qui n'ont
pas attiré son attention. L'interrogatoire retrouvant des antécédents de décollement de rétine dans la famille, fait penser à la
possibilité d'un décollement de la rétine de l'oeil gauche, idiopathique.

Parmi les signes suivants, le(s)quel(s) peu(ven)t faire évoquer un diagnostic de décollement de rétine ?
A - Myodésopsies
B - Céphalées
C - Phosphènes
D - Photophobie
E - Halos colorés
Bonne(s) réponse(s) : A C

Les céphalées et les halos colorés évoquent le diagnostic d'hypertonie oculaire ; la photophobie, signe non spécifique, peut
évoquer les diagnostics de kératite, d'uvéite.

Parmi les signes suivants, le(s)quel(s) appartien(nen)t au décollement de la rétine, idiopathique ?


A - Amputation du champ visuel
B - Aréflexie pupillaire
C - Rougeur oculaire
D - Hypertonie oculaire
E - Douleurs oculaires
Bonne(s) réponse(s) : A

Attention, la question concerne les signes appartenant au décollement de la rétine ; l'aréflexie pupillaire, la rougeur oculaire,
l'hypertonie et les douleurs oculaires évoquent le diagnostic de crise de glaucome aigu par fermeture de l'angle.

Parmi les amétropies suivantes, laquelle,(lesquelles) expose(nt) le plus au décollement de la rétine ?


A - Myopie
B - Hypermétropie
C - Aphakie
D - Astigmatisme
E - Presbytie
Bonne(s) réponse(s) : A C

La myopie et l'aphakie s'accompagnent de modifications de la périphérie rétinienne et du vitré qui peuvent être à l'origine d'un
décollement de la rétine.

Parmi les lésions rétiniennes suivantes retrouvées sur l'oeil adelphe laquelle(lesquelles) nécessite(nt) un
traitement préventif par laser ?
A - Dégénérescence palissadique
B - Dégénérescence givrée
C - Dégénérescence pavimenteuse
D - Exsudats cotonneux
E - Trou rond
Bonne(s) réponse(s) : A B E

Le laser permet de faire un barrage autour des zones de rétine fragile qui prédisposent à la survenue d'un décollement de la
rétine. La dégénérescence pavimenteuse et les exsudats cotonneux ne constituent pas des zones de rétine fragile et donc ne
nécessitent aucun traitement particulier.

Parmi les traitements suivants, le(s)quel(s) peu(ven)t être utilisé(s) en présence d'un décollement avéré de la
rétine ?
A - Cryoapplication
B - Indentation
C - Vitrectomie
D - Laser
E - Injection intra-oculaire de gaz
Bonne(s) réponse(s) : A B C E

Le barrage au laser d'un décollement de rétine est insuffisant, car la cicatrice solide du laser apparaissant en 10 à 15 jours
minimum, le décollement peut pendant ce délai continuer à s'étendre et dépasser le barrage.

1067
Exclusivement sur DOC - DZ : www.doc-dz.com NADJI 85
RESIDANAT EN POCHE TOME II
Cas Clinique en QCM
Madame M, âgée de 25 ans, consulte pour une baisse d'acuité visuelle droite, survenue de façon rapidement progressive en
deux jours, accompagnée de douleurs à l'élévation du regard. Dans les antécédents. on note des épisodes de dysesthésies
des membres inférieurs, avec troubles mictionnels, l'absence d'hypertension artérielle et de diabète. L'acuité visuelle droite est
de 1/20, l'acuité visuelle gauche est de 10/10. La mobilité oculaire n'est pas limitée, mais un nystagmus lors du regard latéral
extrême droit est présent. Le fond d'oeil est normal des deux côtés. ainsi que le tonus oculaire. L'hypothèse la plus probable
est celle d'une névrite optique rétrobulbaire droite.

Le terme de névrite optique rétrobulbaire désigne classiquement :


A - Une atteinte inflammatoire du nerf optique quelle que soit la topographie
B - Une atteinte du faisceau papillo-maculaire quelle qu'en soit l'étiologie
C - Toute neuropathie optique
D - L'atteinte totale du nerf optique dans sa portion orbitaire
E - Une neuropapillite
Bonne(s) réponse(s) : B

Le terme de névrite optique rétrobulbaire désignait classiquement une atteinte du tronc du nerf optique (où se trouve le
faisceau papillo-maculaire) localisée ou totale, dans sa portion orbitaire, c'est-à-dire en arrière de la lame criblée (en avant de
la lame criblée se trouve la tête du nerf optique, ou pupille).

La symptomatologie fonctionnelle doit faire retenir un diagnostic différentiel plausible. Lequel ?


A - L'oblitération de l'artère centrale de la rétine
B - La thrombose de la veine centrale de la rétine
C - Un oedème papillaire de stase
D - Un glaucome aigu
E - Une cataracte
Bonne(s) réponse(s) : B

L'oblitération de l'artère centrale de la rétine et l'oedème papillaire de stase s'accompagnent d'une modification du fond d'oeil.
Le glaucome aigu s'accompagne d'une augmentation du tonus oculaire. La cataracte ne survient pas aussi rapidement, en
dehors d'un contexte traumatique.

Sur quel(s) examen(s) complémentaire(s), en dehors du relevé du champ visuel, va s'étayer le diagnostic
symptomatique de névrite optique rétrobulbaire ?
A - L'angiofluorographie
B - La radiographie
C - L'électrorétinographie
D - L'échographie
E - Les potentiels évoqués visuels
Bonne(s) réponse(s) : E

En l'absence de troubles des milieux (cataracte, hémorragie intra-vitréenne) et d'anomalies du fond d'oeil (rétine, pupille),
seuls les potentiels évoqués visuels permettent d'affirmer l'existence d'une pathologie sur les voies optiques.

L'étude du champ visuel est réalisée à la recherche :


A - D'une quadranopsie
B - D'une hémianopsie
C - D'un élargissement de la tache aveugle
D - D'un scotome centro-coecal
E - D'un déficit arciforme
Bonne(s) réponse(s) : D

Les déficits en secteur (quadranopsie et hémianopsie) font rechercher un pathologie au niveau du chiasma ou
rétrochiasmasmatique. L'élargissement de la tache aveugle évoque un oedème papillaire. Le déficit arciforme évoque le
retentissement sur les fibres optiques de la papille d'un glaucome chronique.

Sur quel(s) argument(s) essentiel(s) le diagnostic de sclérose en plaque peut-il être étayé ?
A - La radiographie du canal optique
B - Les antécédents familiaux
C - Les atteintes neurologiques de poussées antérieures ou concomitantes
D - L'hyperpression du liquide céphalorachidien
E - L'hypergammaglobulinorachie
Bonne(s) réponse(s) : C E

L'examen neurologique recherche les séquelles de poussées antérieures, ou les signes d'une atteinte multiloculaire
concomitante. L'hypergammaglobulinorachie oligoclonale à électrophérèse des protéines du liquide céphalorachidien est
évocatrice du diagnostic.

1068
Exclusivement sur DOC - DZ : www.doc-dz.com NADJI 85
RESIDANAT EN POCHE TOME II
Cas Clinique en QCM

La sclérose en plaques n'est pas la seule étiologie à évoquer. Quel(s) est(sont) le(s) diagnostic(s) étiologique(s)
compatible(s) avec le tableau présenté ?
A - L'intoxication alcoolo-tabagique
B - Une neuropathie carentielle
C - Une neuropathie optique de Leber
D - Une névrite optique inflammatoire secondaire à un foyer infectieux régional
E - Une arachnoïdite opto-chiasmatique
Bonne(s) réponse(s) : D

L'intoxication alcoolo-tabagique et la neuropathie carentielle entraînent une neuropathie optique bilatérale. La neuropathie
optique de Leber peut être unilatérale au début, mais sans antécédents neurologiques. L'arachnoïdite optochiasmatique
survient après une méningite, un traumatisme crânien grave ou une intervention neurochirurgicale.

Quelle(s) attitude(s) thérapeutique(s) adopterez-vous dans un premier temps ?


A - Antibiothérapie
B - La polyvitaminothérapie
C - Les anti-inflammatoires non stéroïdiens
D - La corticothérapie générale
E - Les immunosuppresseurs
Bonne(s) réponse(s) : B D

Ce traitement permet d'espérer (sans aucune certitude) diminuer la durée de la poussée inflammatoire.

Une femme de 50 ans, comptable, se plaint brutalement, en fin de journée, de douleurs importantes au niveau de l'oeil et de
l'orbite gauches. Elle signale en outre une diminution de l'acuité visuelle de ce côté. A l'examen, la constatation d'un oeil rouge
sans sécrétion permet d'évoquer une crise de glaucome aigu. L'interrogatoire révèle que cette patiente est traitée depuis
plusieurs mois pour dépression nerveuse, ménopause précoce et rhumatisme chronique.

Le glaucome par fermeture de l'angle survient le plus souvent chez les sujets porteurs d'une amétropie.
Laquelle ?
A - Myopie
B - Astigmatisme
C - Hypermétropie
D - Anisométropie
E - Aphakie
Bonne(s) réponse(s) : C

L'hypermétropie peut s'accompagner d'une chambre antérieure peu profonde, d'un angle iridocornéen étroit, qui prédisposent
à la survenue d'un glaucome aigu par fermeture de l'angle.

Parmi les signes suivants, le(s)quel(s) est(sont) présent(s) au cours d'une crise de glaucome aigu ?
A - Oedème cornéen
B - Semi-mydriase
C - Chambre antérieure profonde
D - Aréflexie pupillaire
E - Cercle périkératique
Bonne(s) réponse(s) : A B D E

Au contraire, la chambre est très peu profonde.

Parmi les médicaments suivants prescrits à la patiente, le(s)quel(s) est(sont) susceptible(s) de déclencher une
crise de glaucome par fermeture de l'angle ?
A - Antipaludéens de synthèse
B - Antispasmodiques digestifs
C - Oestrogènes
D - Antiinflammatoire non stéroïdiens
E - Antidépresseurs
Bonne(s) réponse(s) : B E

Les antispasmodiques et les antidépresseurs peuvent posséder un effet anticholinergique, à l'origine d'une demi-mydriase et
donc d'une crise de glaucome aigu.

1069
Exclusivement sur DOC - DZ : www.doc-dz.com NADJI 85
RESIDANAT EN POCHE TOME II
Cas Clinique en QCM

En sachant que cette patiente présente une légère insuffisance rénale, parmi les produits suivants le(s)quel(s)
proposez-vous pour réduire la crise de glaucome aigu ?
A - Pilocarpine® collyre 2%
B - Adrénaline® collyre 1%
C - Acétazolamide® 500 mg en intraveineux
D - Mannitol® 20% (2 g par kg de poids) en intraveineux
E - Acéclidine® ou Glaucostat® collyre
Bonne(s) réponse(s) : A E

La pilocarpine et le glaucostat sont des myotiques qui permettent de lever le blocage pupillaire, et de rouvrir l'angle
iridocornéen. L'acétazolamide et le mannitol doivent être employés avec précaution, à posologie réduite, en raison de la
légère insuffisance rénale.

Une jeune femme, âgée de 32 ans, myope depuis l'âge de 10 ans, présente brusquement un matin une sensation de voile
noir devant son oeil droit, qu'elle localise dans le champ visuel inférieur gauche. Elle n'a aucun antécédent oculaire particulier.
Elle n'a aucun antécédent général. Elle a deux enfants en bonne santé. Elle consulte un ophtalmologiste, qui lui précise
qu'elle a un décollement de la rétine de l'oeil droit.

Parmi les signes fonctionnels suivants recherchés à l'interrogatoire, quel est celui ou quels sont ceux qui ont pu
orienter le diagnostic du spécialiste vers le décollement de la rétine ?
A - Scotome négatif central
B - Myodésopsies
C - Douleur du globe oculaire
D - Phosphènes
E - Rougeur conjonctivale
Bonne(s) réponse(s) : B D

Le scotome négatif central peut traduire une pathologie de la macula. Les douleurs du globe oculaire et la rougeur
conjontivale évoquent en premier le diagnostic de glaucome aigu par fermeture de l'angle.

Parmi les appareils suivants, le(s)quel(s) le spécialiste a pu utiliser pour faire le diagnostic de décollement de la
rétine au cours d'un examen courant ?
A - Ophtalmoscope
B - Verre à gonioscopie
C - Verre à 3 miroirs
D - Appareil de Javal
E - Ophtalmodynamomètre
Bonne(s) réponse(s) : A C

Le verre à gonioscopie permet d'examiner l'angle iridocornéen.


L'appareil de Javel permet de mesurer l'astigmatisme de la face antérieure de la cornée. L'ophtalmodynamomètre est utilisé
pour mesurer la pression de l'artère ophtalmique.

L'ophtalmologiste a examiné l'oeil gauche. La(les) lésions qu'il recherche en périphérie rétinienne et qui
justifi(en)t un traitement prophylactique est(sont) :
A - Dégénérescence palissadique
B - Dégénérescence myélinique
C - Déchirure à lambeau
D - Dégénérescence pavimenteuse
E - Choroldose myopique
Bonne(s) réponse(s) : A C

La dégénérescence myélinique, la dégénérescence pavimenteuse et la choroïdose myopique ne sont pas des zones de rétine
fragile, et ne justifient donc pas un traitement prophylactique du décollement de rétine par laser.

Dix jours après la première consultation, la jeune femme présente une baisse d'acuité visuelle très importante
de l'oeil gauche. L'ophtalmologiste constate une hémorragie très abondante du vitré. Quel appareil peut-il
utiliser pour affirmer qu'elle présente un décollement de la rétine bilatéral ?
A - Ophtalmoscope direct
B - Echographe
C - Esthésiomètre
D - Angiographe en fluorescence
E - Verre à 3 miroirs
Bonne(s) réponse(s) : B

Pour affirmer qu'il existe un décollement de rétine de l'oeil gauche, l'échographie bidimensionnelle permet de préciser la
position de la rétine, alors que l'ophtalmoscope est impossible en raison de l'hémorragie intravitréenne.

1070
Exclusivement sur DOC - DZ : www.doc-dz.com NADJI 85
RESIDANAT EN POCHE TOME II
Cas Clinique en QCM

Lors de la première consultation, l'ophtalmologiste avait constaté des lésions de la périphérie rétinienne. Il avait
proposé un traitement prophylactique que la patiente a refusé et qui aurait pu éviter l'accident au dixième jour.
Quel est l'appareil actuellement utilisé pour ce type de traitement ?
A - Laser à argon
B - Laser YAG
C - Diathermie trans-sclérale
D - Laser à rubis
E - Diathermie choroïdienne ab externo
Bonne(s) réponse(s) : A

Le laser à argon est utilisé pour réaliser un barrage autour des zones de rétine fragile, et les isoler du reste de la rétine.

Madame M., âgée de 25 ans, consulte pour une baisse d'acuité visuelle droite survenue de façon rapidement progressive en
deux jours, accompagnée de douleurs à l'élévation du regard. Dans les antécédents on note des épisodes de dysesthésie des
membres inférieurs avec troubles mictionnels, l'absence d'hypertension artérielle et de diabète. L'acuité visuelle droite est de
1/20, l'acuité visuelle gauche est de 10/10. La mobilité oculaire n'est pas limitée, mais un nystagmus lors du regard latéral
extrême droit est présent. Le fond d'oeil est normal des deux côtés ainsi que le tonus oculaire. L'hypothèse la plus probable
est celle d'une névrite optique rétrobulbaire droite.

Le terme de névrite optique rétrobulbaire axiale désigne classiquement :


A - Une atteinte inflammatoire du nerf optique quelle que soit la topographie
B - Une atteinte du faisceau papillo-maculaire quelle qu'en soit l'étiologie
C - Toute neuropathie optique
D - L'atteinte totale du nerf optique dans sa portion rétrobulbaire
E - Une neuropapillite
Bonne(s) réponse(s) : B

Ce cas clinique a été posé de façon parfaitement identique en 1985 dans la région Ile-de-France partie 2 cas clinique n°3!!

La symptomatologie fonctionnelle doit faire retenir un diagnostic différentiel plausible :


A - L'oblitération de l'artère centrale de la rétine
B - La thrombose de la veine centrale de la rétine
C - Un oedème papillaire de stase
D - Un glaucome aigu
E - Une cataracte
Bonne(s) réponse(s) : B

Sans commentaires.

En dehors du relevé du champ visuel le diagnostic symptomatique de névrite optique rétrobulbaire fait appel à :
A - L'angiofluorographie
B - La radiographie
C - L'électrorétinographie
D - L'examen de la vision des couleurs
E - Les potentiels évoqués visuels
Bonne(s) réponse(s) : D E

L'examen de la vision des couleurs, comme l'acuité visuelle ou les potentiels évoqués visuels permet d'explorer le nerf optique
rétrobulbaire.

L'étude du champ visuel est réalisée à la recherche :


A - D'une quadranopsie
B - D'une hémianopsie
C - D'un élargissement de la tache aveugle
D - D'un scotome centro-coecal
E - D'un déficit arciforme
Bonne(s) réponse(s) : D

Sans commentaires.

1071
Exclusivement sur DOC - DZ : www.doc-dz.com NADJI 85
RESIDANAT EN POCHE TOME II
Cas Clinique en QCM

La suspicion de sclérose en plaques pourra s'étayer sur quel(s) argument(s) essentiel(s) ?


A - La radiographie du canal optique normal
B - Les antécédents familiaux
C - Les atteintes neurologiques de poussées antérieures ou concomitantes
D - L'hyperpression du liquide céphalorachidien
E - L'hypergammaglobulinorachie
Bonne(s) réponse(s) : C E

Sans commentaire.

Quelle(s) attitude(s) thérapeutique(s) adopterez-vous dans un premier temps, si vous retenez le diagnostic de
sclérose en plaques ?
A - L'expectative
B - La polyvitaminothérapie
C - Les anti-inflammatoires non stéroïdiens
D - La corticothérapie générale
E - Les immunosuppresseurs
Bonne(s) réponse(s) : B D

Sans commentaire.

Une jeune fille de 22 ans présente une baisse brutale unilatérale de l'acuité visuelle de l'oeil droit. L'interrogatoire retrouve la
notion d'un épisode de paresthésies du membre inférieur il y a 3 mois. Auparavant, au cours d'une visite de routine, la vision
était de 10/10 au niveau des 2 yeux. L'examen retrouve au niveau de l'oeil droit, une acuité visuelle réduite à une perception
lumineuse, alors qu'au niveau de l'oeil gauche, elle est de 10/10, L'examen des fonds d'oeil est sans particularité. D'emblée le
diagnostic de sclérose en plaques est évoqué.

Quel(s) autre(s) diagnostic(s) peut on évoquer devant un tel tableau ?


A - Simulation
B - Adénome hypophysaire
C - Intoxication à l'héroïne
D - Botulisme
E - Glaucome chronique à angle ouvert
Bonne(s) réponse(s) : A

L'adénome hypophysaire entraîne une hémianopsie bitemporale ; l'intoxication à l'héroïne entraîne un myosis ; le botulisme
entraîne des paralysies oculomotrices. Le glaucome chronique à angle ouvert entraîne une baisse progressive de l'acuité
visuelle.

S'il s'agit d'une sclérose en plaques, par quel mécanisme peut-on expliquer la baisse d'acuité visuelle
unilatérale droite ?
A - Atteinte du chiasma
B - Atteinte de la bandelette optique gauche
C - Atteinte du cortex occipital gauche
D - Atteinte du nerf moteur oculaire externe droit
E - Aucune des réponses précédentes n'est exacte
Bonne(s) réponse(s) : E

La baisse d'acuité visuelle est expliquée par une névrite optique rétrobulbaire (entre la lame criblée et le chiasma), et en
particulier du faisceau pupille maculaire.

S'il s'agit d'une sclérose en plaques, le champ visuel réalisé chez cette patiente montrera de façon très probable
:
A - Un scotome central unilatéral au niveau du champ visuel de l'oeil droit
B - Une hémianopsie bitemporale
C - Une hémianopsie binasale
D - Une hémianopsie latérale homonyme droite
E - Une hemianopsie latérale homonyme gauche
Bonne(s) réponse(s) : A

L'hémianopsie bitemporale ou binasale doit faire rechercher une lésion chiasmatique.


L'hémianopsie latérale homonyme doit faire rechercher une lésion rétrochiasmatique.

1072
Exclusivement sur DOC - DZ : www.doc-dz.com NADJI 85
RESIDANAT EN POCHE TOME II
Cas Clinique en QCM

Parmi les examens suivants, quel est celui ou quels sont ceux qui seront perturbés s'il s'agit d'une sclérose en
plaques ?
A - Potentiels évoqués visuels
B - Electrorétinogramme
C - Angiographie fluorescéinique
D - Echographie oculaire
E - Aucune des propositions précédentes n'est exacte
Bonne(s) réponse(s) : A

Il n'y a pas d'atteinte anatomique (échographie) ou fonctionnelle de la rétine (électrorétinogramme, angiographie


fluoréscéinique) au cours de la sclérose en plaques.

Hormis la baisse de l'acuité visuelle unilatérale, on rencontre au cours de l'évolution d'une sclérose en plaques :
A - Une cataracte
B - Une baisse d'acuité visuelle bilatérale
C - Un nystagmus
D - Une diplopie
E - Une conjonctivite
Bonne(s) réponse(s) : B C D

La baisse d'acuité visuelle peut être bilatérale en cas d'atteinte du nerf optique controlatéral au cours d'une poussée
ultérieure. Le nystagmus et la diplopie traduisent une atteinte des nerfs crâniens ou des voies supra ou internucléaires.

L'évolution de cette poussée peut se faire :


A - Vers la récupération fonctionnelle
B - Vers une récupération fonctionnelle partielle
C - Vers une perte fonctionnelle totale
D - Vers une atrophie optique
E - Aucune des propositions précédentes n'est exacte
Bonne(s) réponse(s) : A B C D

L'évolution peut se faire vers la récupération fonctionnelle totale sans séquelle, ou bien vers l'atrophie opaque, avec tous les
intermédiaires possibles.

Un homme se présente après avoir reçu de l'eau de javel dans un oeil.

Le danger consiste dans le fait que :


A - L'eau de javel est acide et va entraîner une ulcération cornéenne
B - Il risque d'y avoir des corps étrangers sous la paupière supérieure
C - L'eau de javel est une base forte entraînant une brûlure cornéoconjonctivale grave
D - Les voies lacrymales risquent d'être obstruées
E - L'eau de javel contient des colorants toxiques
Bonne(s) réponse(s) : C

Mais la proposition D est aussi exacte, les voies lacrymales risquant d'être obstruées secondairement.

Le médecin généraliste, devant un tel cas, doit :


A - Mettre le blessé sous antibiotiques par voie générale
B - S'assurer que le blessé est vacciné contre le tétanos
C - Laver l'oeil atteint à grande eau, même non stérile
D - Se contenter de faire une occlusion de l'oeil blessé
E - Envoyer le blessé dans un centre spécialisé
Bonne(s) réponse(s) : C E

Le lavage doit être immédiat, et prolongé plusieurs minutes, pour favoriser l'élimination du produit, et en limiter la pénétration
dans le globe oculaire. Ensuite le patient doit être envoyé dans un centre spécialisé.

1073
Exclusivement sur DOC - DZ : www.doc-dz.com NADJI 85
RESIDANAT EN POCHE TOME II
Cas Clinique en QCM

Dans ce cas et immédiatement les atteintes physiques résiduelles peuvent être :


A - Une brûlure du point lacrymal inférieur
B - Une ischémie conjonctivale du secteur nasal
C - Une ulcération épithéliale cornéenne
D - Un oedème stromal
E - Un tyndall de chambre antérieure
Bonne(s) réponse(s) : A B C D E

L'importance des lésions varie selon la concentration en base du liquide.

Quel(s) est(sont) le(s) geste(s) thérapeutique(s) spécialisé(s) indiqué(s) en urgence ?


A - Un nettoyage des culs de sac palpébraux
B - Un lavage des voies lacrymales
C - Une instillation d'atropine
D - Une injection sous-conjonctivale de corticoïde et d'antibiotiques
E - Une injection sous-conjonctivale de vasodilatateurs
Bonne(s) réponse(s) : A B C D E

Tous ces gestes doivent être pratiqués dès l'arrivée en service spécialisé.

Quel(s) est(sont) le(s) risque(s) oculaire(s) entraîné(s) par cet accident ?


A - Une sténose des voies lacrymales dans les semaines suivantes
B - Un symblépharon
C - Une uvéite antérieure
D - La persistance de synéchies iridocristalliniennes
E - Une cataracte
Bonne(s) réponse(s) : A B C D E

Les autres complications possibles sont le trichiasis, l'ectropion, la kératite chronique et la taie cornéenne avec ou sans
néovascularisation cornéenne.

Une femme de 50 ans présente le soir, brutalement, une douleur orbitaire irradiant à l'hémicrâne, s'accompagnant de
vomissements et de baisse unilatérale de l'acuité visuelle.

Quel diagnostic évoquez-vous ?


A - Une névrite optique aiguë
B - Une crise de glaucome aigu
C - Une thrombose de l'artère centrale de la rétine
D - Une fistule carotido-caverneuse
E - Une maladie de Honon
Bonne(s) réponse(s) : B

L'association de douleur, d'une baisse d'acuité visuelle et de vomissements est évocatrice d'une crise de glaucome aigu par
fermeture de l'angle.

Quel(s) argument(s) recherché(s) à l'interrogatoire orienteront votre diagnostic ?


A - La prise de neuroleptiques
B - Des conjonctivites à répétition
C - La vision de halos colorés autour des points lumineux
D - Une hypermétropie importante
E - Des crises douloureuses oculaires le soir avec impression de baisse d'acuité visuelle
Bonne(s) réponse(s) : A C D E

La prise de neuroleptique peut déclencher la crise aiguë sur un terrain prédisposant (hypermétropie), éventuellement
précédée de crises à minima spontanément régressives (perception de halos colorés et douleurs oculaires nocturnes).

1074
Exclusivement sur DOC - DZ : www.doc-dz.com NADJI 85
RESIDANAT EN POCHE TOME II
Cas Clinique en QCM

Quel(s) signe(s) physique(s) va(vont) accompagner obligatoirement ce tableau ?


A - Un chémosis palpébral
B - Une hyperhémie conjonctivale avec cercle périkératique
C - Un oedème cornéen épithélial
D - Un tyndall de la chambre antérieure
E - Une chambre antérieure étroite
Bonne(s) réponse(s) : B C E

Le chemosis (oedème de la conjonctive) est évocateur d'une conjonctivite ou d'une kératite. Le tyndall de la chambre
antérieure est évocateur d'une uvéite antérieure aiguë.

Quelle(s) manoeuvre(s) instrumentale(s) suffira(ont) à votre diagnostic ?


A - Une prise de la tension oculaire au schiotz
B - La prise de la tension de l'artère centrale de la rétine
C - L'étude des ductions et des versions
D - Une tonométrie en urgence
E - Une prise de la tension oculaire à l'aplanation
Bonne(s) réponse(s) : D E

La prise de la tension oculaire (tonométrie) à l'aplanation a remplacé la mesure de la tension oculaire au Schiotz (tonométrie à
indentation).

Quelle(s) thérapeutique(s) doi(ven)t être instituée(s) en urgence ?


A - Un inhibiteur de l'anhydrase carbonique par voie générale type acétazolamide
B - Un collyre à l'atropine
C - Un collyre bêtabloquant (type Timoptol®)
D - Un collyre à la pilocarpine
E - Une solution osmotique par voie générale
Bonne(s) réponse(s) : A D E

L'atropine est formellement contre-indiquée. Le collyre bêta-bloquant (type Trimoptol®) ne présente pas d'intérêt au cours
d'une crise aiguë, en raison de son délai d'action de plusieurs heures.

Quel geste chirurgical doit être indiqué dans ce cas ?


A - Une trabéculectomie
B - Une iridectomie périphérique
C - Une trabéculotomie
D - Une phakoexérèse
E - Une injection rétrobulbaire de xylocaïne
Bonne(s) réponse(s) : B

L'iridectomie périphérique chirurgicale est réalisée après normalisation de la tension oculaire.


On pourrait aussi pratiquer une iridectomie périphérique au laser.

Une femme de 65 ans, aphaque, se plaint après avoir vu un éclair fixe, une amputation progressive du champ visuel de cet
oeil.

Quel diagnostic ceci vous évoque-t-il ?


A - Un glaucome chronique
B - Une thrombose d'une branche de la veine centrale de la rétine
C - Un décollement de rétine
D - Une ischémie de la tête du nerf optique
E - Une névrite optique rétrobulbaire
Bonne(s) réponse(s) : C

L'éclair fixe (phosphène) associé à une amputation du champ visuel unilatéral chez une aphake (cataracte opérée) de 65 ans
doit faire rechercher un décollement de la rétine.

1075
Exclusivement sur DOC - DZ : www.doc-dz.com NADJI 85
RESIDANAT EN POCHE TOME II
Cas Clinique en QCM

Quel(s) argument(s) recherché(s) à l'interrogatoire sera(ient) en faveur de ce diagnostic ?


A - Antécédents familiaux de glaucome
B - Myopie forte
C - Chute accidentelle dans un escalier quelques jours auparavant
D - Phakoexérèse de cet oeil quatre mois auparavant
E - Antécédents familiaux de diabète
Bonne(s) réponse(s) : B C D E

Les antécédents personnels ou familiaux de glaucome n'ont aucune incidence sur le risque de survenue d'un décollement de
la rétine.

Quel(s) est(sont) le(s) signe(s) retrouvé(s) à l'examen biomicroscopique constamment dans ce cas ?
A - Une hyperhémie conjonctivale
B - Un Tyndall de chambre antérieure
C - Une anisocorie
D - Une légère hypotonie
E - Un vitré filamenteux
Bonne(s) réponse(s) : D E

Le décollement de rétine s'accompagne d'une baisse du tonus oculaire, et de modifications du vitré liées à l'aphakie, qui
apparait filamenteux.

Le diagnostic sera affirmé par :


A - Une tonométrie
B - Une mesure de la tension de l'artère centrale de la rétine
C - Une opthalmoscopie indirecte à l'appareil de Sheppens
D - Une gonioscopie
E - Ophtalmoscopie au verre à trois miroirs
Bonne(s) réponse(s) : C E

Seule l'ophtalmoscope permet, lorsque les milieux sont clairs (cornée, cristallin et vitré),
d'affirmer le diagnostic de décollement de la rétine.

Un patient de 50 ans présente brutalement à un oeil, des photopsies et des corps flottants vitréens abondants. Quelques jours
plus tard, sa vision baisse fortement, ce qui nécessite son hospitalisation pour intervention chirurgicale.

Quel diagnostic évoque cette symptomatologie ?


A - Cataracte
B - Déchirure rétinienne sans décollement de rétine
C - Déchirure rétinienne avec décollement de rétine
D - Choroïdite aiguë
E - Glaucome aigu
Bonne(s) réponse(s) : C

Les phosphènes (traduisant les tractions du vitré sur la rétine) et les myodesopsies (par décollement postérieur du vitré ou par
hémorragie intravitréenne) doivent faire rechercher une déchirure avant que n'apparaisse un décollement de la rétine.

Quel(s) état(s) anatomique(s) oculaire(s) prédispose(nt) à cette pathologie oculaire ?


A - Astigmatisme
B - Hypermétropie
C - Aphaquie
D - Myopie
E - Angle iridocornéen étroit
Bonne(s) réponse(s) : C D

Les facteurs de risque du décollement de rétine sont la myopie, l'aphaquie, l'âge, les antécédents personnels ou familiaux de
décollement de la rétine, les traumatismes du globe.

1076
Exclusivement sur DOC - DZ : www.doc-dz.com NADJI 85
RESIDANAT EN POCHE TOME II
Cas Clinique en QCM

Que!(s) examen(s) opthalmologique(s) permet(tent) de faire un diagnostic précis ?


A - Ophtalmoscopie
B - Biomicrospie avec verre de contact
C - Gonioscopie
D - Angiographie à la fluorescéine
E - Echographie
Bonne(s) réponse(s) : A B

L'échographie est indiquée en cas de troubles des milieux normalement transparents (cornée, cristallin, vitré).

Si cette symptomatologie ne s'accompagnait pas de baisse de vision, quel diagnostic devrait être évoqué ?
A - Déchirure rétinienne avec décollement de rétine
B - Déchirure rétinienne sans décollement de rétine
C - Cataracte
D - Rétinopathie diabétique non proliférante
E - Choroïdite extramaculaire
Bonne(s) réponse(s) : B

La baisse d'acuité visuelle est liée au décollement de la macula. En l'absence de décollement de la rétine, un traitement par
laser doit être entrepris en urgence pour prévenir l'apparition d'un décollement de la rétine.

Quelle intervention chirurgicale doit être effectuée chez ce patient ?


A - Trabéculectomie
B - Extraction intracapsulaire
C - Extraction extracapsulaire
D - Indentation sclérale avec cryocoagulation
E - Photocoagulation au laser
Bonne(s) réponse(s) : D

Le laser n'est indiqué que pour prévenir l'apparition du décollement. La trabéculectomie est le traitement chirurgical du
glaucome chronique à angle ouvert.

Un homme de 57 ans, ouvrier agricole, constate brutalement, alors qu'il est au travail, un "trouble" de la vision de l'oeil
gauche. En même temps, il ne sent plus sa jambe droite, il a l'impression qu'elle ne le porte plus et son bras droit devient
inerte. Il veut appeler, mais n'émet que des sons inintelligibles.

Parmi les hypothèses concernant la nature ce cet épisode, laquelle discutez-vous en priorité ?
A - Migraine accompagnée
B - Accident ischémique cérébral
C - Syndrome de Raynaud
D - Epilepsie partielle
E - Aucune des hypothèses précédentes
Bonne(s) réponse(s) : B

L'accident ischémique cérébral dans le territoire sylvien superficiel entraîne un déficit sensitif et moteur à prédominance
brachiale, une aphasie, une hémianopsie latérale homonyme (par atteinte des radiations optiques).

Parmi les hypothèses concernant la lésion responsable, laquelle vous paraît à retenir en premier pour décider
des examens nécessaires ?
A - Embolie d'origine cardiaque
B - Anévrysme artérioveineux
C - Athérome de la carotide interne
D - Insuffisance circulatoire vertébro-basilaire
E - Aucune des hypothèses précédentes
Bonne(s) réponse(s) : A

Si l'athérome de la carotide interne est la cause la plus fréquemment retrouvée, l'embolie d'origine cardiaque doit être
recherchée en premier en raison des conséquences thérapeutiques pour prévenir les récidives à court terme.

1077
Exclusivement sur DOC - DZ : www.doc-dz.com NADJI 85
RESIDANAT EN POCHE TOME II
Cas Clinique en QCM

Quel examen demanderez-vous en priorité ?


A - Tomodensitométrie craniocérébrale
B - Artériographie numérisée par voie veineuse
C - Exploration par ultrasons des vaisseaux du cou
D - Electroencéphalogramme
E - Echocardiogramme
Bonne(s) réponse(s) : E

La question est directement liée à la précédente : l'échocardiogramme ainsi que l'électrocardiogramme permettent de
rechercher une pathologie emboligène (trouble du rythme, infarctus avec thrombose murale) curable.

Que décidez-vous ?
A - Investigation nécessaire dans les heures qui viennent
B - Investigation nécessaire dans les mois qui viennent
C - Investigation nécessaire dans les semaines qui viennent
D - Exploration dans les jours qui viennent
E - Aucune des propositions précédentes
Bonne(s) réponse(s) : A

Evident.

Un patient présente brutalement une diplopie disparaissant à l'occlusion alternée d'un des deux yeux.

A quelle étiologie doit-on penser ?


A - Cataracte unilatérale
B - Glaucome aigu unilatéral
C - Paralysie oculomotrice
D - Corps étranger cornéen
E - Occlusion de l'artère centrale de la rétine
Bonne(s) réponse(s) : C

Seule la paralysie oculomotrice entraîne une diplopie en vision binoculaire, disparaissant en vision monoculaire. La cataracte
peut entraîner une diplopie qui persiste en vision monoculaire.

Parmi les autres signes fonctionnels associés vous devez retrouver :


A - Injection périkératique
B - Attitude vicieuse de la tête
C - Douleur oculaire
D - Occlusion d'un oeil
E - Baisse d'acuité visuelle unilatérale
Bonne(s) réponse(s) : B D

L'attitude vicieuse de la tête (tournée du côté du muscle paralysé) permet de diminuer la diplopie ; l'occlusion d'un oeil par un
ptosis peut survenir par paralysie du releveur de la paupière supérieure. L'injection périkératique, la douleur et la baisse
d'acuité visuelle évoquent le diagnostic de glaucome aigu.

Par quel(s) examen(s) paraclinique(s) étudie-t-on cette diplopie ?


A - Ophtalmoscopie
B - Verre rouge
C - Biomicroscopie
D - Test de Hess Lancaster
E - Angiographie à la fluorescéine
Bonne(s) réponse(s) : B D

L'examen au verre rouge et le test de Hess Lancaster étudient l'oculomotricité ; ils sont utiles pour explorer une paralysie
oculomotrice.

1078
Exclusivement sur DOC - DZ : www.doc-dz.com NADJI 85
RESIDANAT EN POCHE TOME II
Cas Clinique en QCM

Quel(s) type(s) de diplopie peu(ven)t être mis en évidence ?


A - Monoculaire
B - Croisée
C - Homonyme
D - Verticale
E - Aucune des propositions précédentes
Bonne(s) réponse(s) : B C D

La diplopie peut être croisée (en cas de paralysie d'un muscle droit interne), homonyme (en cas de paralysie d'un muscle droit
externe), ou verticale (en cas de paralysie d'un muscle droit supérieur ou inférieur).

Quel traitement peut permettre la suppression de cette diplopie ?


A - Anticoagulants
B - Extraction de la cataracte
C - Iridotomie au laser
D - Ablation du corps étranger
E - Verres prismatiques
Bonne(s) réponse(s) : E

Les verres prismatiques permettent de supprimer la diplopie car le prisme entraîne un déplacement de l'image du point de
fixation en regard de la position de déviation (la puissance du prisme mesure l'angle de la déviation).

Un patient de 60 ans, diabétique non insulino-dépendant depuis plus de 10 ans, non fumeur, hypertendu artériel, présente une
baisse d'acuité visuelle progressive bilatérale. Les milieux oculaires sont transparents, la tension oculaire est normale.

A quel diagnostic doit-on penser ?


A - Cataracte
B - Hémorragie du vitré
C - Oedème maculaire
D - Glaucome primitif chronique
E - Neuropathie optique toxique
Bonne(s) réponse(s) : C

Les milieux oculaires sont transparents (éliminant la cataracte et l'hémorragie du vitré) et la tension oculaire est normale
(éliminant un glaucome chronique primitif). En l'absence de toute prise médicamenteuse pouvant entraîner une neuropathie
optique toxique, il s'agit ici d'une rétinopathie diabétique débutante avec oedème maculaire bilatéral.

Selon l'observation ce diagnostic est étayé par :


A - Diabétique non insulino-dépendant depuis plus de 10 ans
B - Hypertendu artériel
C - Cristallin transparent
D - Pression oculaire normale
E - Baisse d'acuité visuelle progressive bilatérale
Bonne(s) réponse(s) : A B C E

L'hypertension artérielle peut entraîner une rétinopathie hypertensive ou une artériosclérose rétinienne, mais elle n'intervient
pas directement dans la constitution de l'oedème maculaire.

Quels sont les 2 examens paracliniques qui permettent d'affirmer le diagnostic ?


A - Biomicroscopie ou ophtalmoscopie
B - Vision des couleurs
C - Angiographie fluorescéinique
D - Potentiels évoqués visuels
E - Champ visuel
Bonne(s) réponse(s) : A C

La vision des couleurs, le champ visuel et les potentiels évoqués visuels sont utilisés pour explorer une pathologie siégeant
sur les voies optiques rétrobulbaires.

1079
Exclusivement sur DOC - DZ : www.doc-dz.com NADJI 85
RESIDANAT EN POCHE TOME II
Cas Clinique en QCM

Parmi ces 2 examens, quel est le signe caractéristique qui affirme le diagnostic ?
A - Hémorragie rétinienne
B - Pas de fluorescence maculaire
C - Fluorescence maculaire
D - Fluorescence papillaire
E - Exsudats secs
Bonne(s) réponse(s) : C

Au cours de l'angiographie à la fluorescéine apparaît une hyperfluorescence maculaire, traduisant la fuite du colorant à travers
les parois vasculaires dans l'oedème maculaire.

Quelle est l'orientation thérapeutique essentielle en dehors de la photocoagulation au laser Argon ?


A - Equilibration du diabète
B - Antiagrégants plaquettaires
C - Hémostatiques
D - Vitrectomie
E - Hypophysiolyse
Bonne(s) réponse(s) : A

Seule l'équilibration du diabète (régimes, hypoglycémiants oraux ou insuline) peut éviter l'aggravation de la rétinopathie
diabétique, et en particulier la constitution d'un oedème maculaire cystoïde avec baisse d'acuité visuelle.

Un homme de 35 ans, travailleur manuel, sans antécédent pathologique connu, remarque depuis quelques mois une baisse
progressive de la vision de l'oeil droit. Lorsqu'il consulte, l'examen attentif et comparatif des yeux révèle une mydriase et une
leucorie droites. L'existence d'une cataracte est l'hypothèse diagnostique la plus probable.

Parmi les étiologies suivantes, quelle est celle qu'il vous paraît logique d'envisager ici a priori ?
A - Sénile
B - Endocrinienne
C - Congénitale
D - Traumatique
E - Post-uvéitique
Bonne(s) réponse(s) : D

L'existence d'une cataracte strictement unilatérale, chez un travailleur manuel (donc exposé au risque de projections de corps
étrangers) jeune (35 ans) élimine les causes séniles (trop jeune) endocrinienne (atteinte bilatérale), congénitale (existant dès
la naissance) ; de même une cataracte post-uvéitique est précédée de poussées d'uvéite antérieure (le patient n'a aucun
antécédent pathologique connu).

Quel(s) signe(s) l'examen à la lampe à fente est-il susceptible d'objectiver pour confirmer cette présomption ?
A - Rubéose irienne
B - Secclusion pupillaire
C - Hétérochromie irienne
D - Taie cornéenne, séquelle d'une plaie perforante punctiforme
E - Pannus cornéen
Bonne(s) réponse(s) : C D

La rubéole irienne évoque une rétinopathie ischémique (par exemple diabétique) avec néovascularisation. La sécclusion
pupillaire évoque des antécédents d'uvéite antérieure ; ici la pupille est en mydriase. Le pannus cornéen est un stade évolué
du trachome oculaire.

Parmi les examens suivants, indiquez celui que vous prescrivez pour étayer votre diagnostic étiologique :
A - Radiographie standard de l'orbite
B - Exploration du cycle glycémique
C - Bilan phosphocalcique
D - Etablissement du caryotype
E - Bilan immunitaire
Bonne(s) réponse(s) : A

Les radiographies standard de l'orbite sont nécessaires et suffisantes pour mettre en évidence un corps étranger intraoculaire
métallique (donc radio-opaque).

1080
Exclusivement sur DOC - DZ : www.doc-dz.com NADJI 85
RESIDANAT EN POCHE TOME II
Cas Clinique en QCM

La mydriase vous paraît la conséquence :


A - D'une atrophie optique
B - D'un blocage du sphincter de l'iris par des synéchies postérieures
C - De l'existence d'un corps étranger intra-oculaire métallique méconnu
D - D'une agénésie du sphincter de l'iris
E - D'une rupture contusive du sphincter de l'iris
Bonne(s) réponse(s) : C

Quelques mois après la pénétration d'un corps étranger métallique dans l'oeil apparait une altération des tissus oculaires par
oxydation et diffusion du fer : c'est la sidérose oculaire qui comprend une atrophie (avec pigmentation, hétérochromie) de l'iris,
une cataracte, une hypertonie, des anomalies à l'électrorétinogramme.

Le pronostic sensoriel dépend :


A - Du niveau de l'acuité visuelle
B - De l'état du champ visuel
C - De la densité de la cataracte
D - Du degré d'altération de l'électrorétinogramme
E - De la durée de l'évolution
Bonne(s) réponse(s) : B D E

Le niveau de l'acuité visuelle actuel peut être limité par la cataracte, et donc améliorable par la chirurgie. Mais la durée de
l'évolution conditionne l'importance de la sidérose oculaire (altération du champ visuel liée à l'hypertonie et à la toxicité
rétinienne, visible à l'électrorétinogramme).

Quelle est l'attitude thérapeutique la plus appropriée ?


A - Abstention
B - Extraction de la cataracte
C - Extraction de la cataracte et du corps étranger intra-oculaire méconnu
D - Traitement endocrinien
E - Traitement anti-inflammatoire
Bonne(s) réponse(s) : C

L'intervention consiste à extraire le corps étranger (éventuellement à l'électro-aimant), puis à extraire la cataracte, après avoir
prévenu le patient de la récupération probablement modeste après l'opération.

Un patient se plaint d'une douleur vive, unilatérale après un traumatisme oculaire par coup d'ongle. L'interrogatoire apprend
qu'il a ressenti, à plusieurs reprises, cette douleur le matin au réveil. Il ouvre difficilement les paupières, larmoie, ne supporte
pas la forte lumière, l'oeil est rouge. L'examen met en évidence une ulcération cornéenne, celle-ci est traitée par collyres
antibiotiques, et cicatrise très rapidement.

La symptomatologie fonctionnelle associée à la douleur comporte vraisemblablement :


A - Phosphènes
B - Sécrétion
C - Larmoiement
D - Blépharospasme
E - Métamorphopsie
Bonne(s) réponse(s) : B C D

Les phosphènes et les métamorphopsies sont évocateurs d'un décollement de rétine. Les sécrétions, le larmoiement et le
blépharospasme (contraction réflexe des paupières par irritation des nerfs ciliaires, branches du trijumeau), lorsqu'ils sont
associés à des douleurs, sont très évocateurs du diagnostic de kératite antérieure aiguë, sans préjuger de la cause.

Où siège la rayure oculaire ?


A - A la cornée
B - A la conjonctive
C - Dans le canthus interne
D - Autour de la cornée
E - Sur le bord libre de la paupière supérieure
Bonne(s) réponse(s) : D

C'est le cercle périkératique, rencontré lors d'une kératite, d'une uvéite antérieure aiguë ou d'un glaucome aigu par fermeture
de l'angle.

1081
Exclusivement sur DOC - DZ : www.doc-dz.com NADJI 85
RESIDANAT EN POCHE TOME II
Cas Clinique en QCM

Quel examen permet de mettre en évidence l'ulcération cornéenne ?


A - Instillation de fluorescéine
B - Instillation de Rose Bengale
C - Test de Schirmer
D - Angiographie à la fluorescéine
E - Tonométrie
Bonne(s) réponse(s) : A

La fluorescéine s'accumule dans la zone désépithélialisée. Le rose bengale est utilisé pour le diagnostic d'un syndrome sec (il
se fixe sur les cellules épithéliales kératinisées, normalement absentes de la surface de la cornée).

Quelle est l'étiologie de cette ulcération ?


A - Conjonctivite virale
B - Sclérite rhumatismale
C - Kératalgie post-traumatique
D - Kératite herpétique
E - Hyposécrétion lacrymale
Bonne(s) réponse(s) : C

Les propositions de réponses semblent peu cohérentes avec la question et l'observation, puisque le contexte traumatique est
évident.

Un sujet de sexe masculin, âgé de 55 ans, ouvrier spécialisé en mécanique, se plaint d'un oeil droit douloureux et rouge
depuis 48 h. A l'examen, on retrouve au niveau de l'oeil droit des signes d'iridocyclite. La vision est égale à 8/10 oeil droit,
10/10 oeil gauche après correction d'une hypermétropie de + 1,5 dioptrie. Le tonus oculaire est égal à 28 mmHg oeil droit et à
16 mmHg oeil gauche. L'examen gonioscopique met en évidence un angle irido-cornéen ouvert à droite comme à gauche. Le
fond d'oeil est normal des deux côtés. Dans les antécédents du patient, on retrouve une symptomatologie évoquant une
spondylarthrite ankylosante et la notion de projections métalliques au cours du travail.

Parmi les signes cliniques suivants, le(s)quel(s) vous permet(tent) de poser le diagnostic d'iridocyclite ?
A - Précipités à la face postérieure de la cornée
B - Myosis
C - Sécrétions mucopurulentes
D - Synéchies pupillaires
E - Halos colorés
Bonne(s) réponse(s) : A B D

Les sécrétions mucopurulentes évoquent le diagnostic de conjonctivite bactérienne. Les halos colorés évoquent une
hypertonie oculaire. Au cours d'une iridocyclite (ou uvéite antérieure), il existe un tyndall dans la chambre antérieure, avec des
précipités à la face postérieure de la cornée ; la pupille est en myosis, exposant au risque de synéchies pupillaires (synéchies
irido-cristalliniennes).

Quel(s) est(sont) le(s) caractère(s) de la rougeur au cours d'une iridocyclite ?


A - Rougeur diffuse à toute la conjonctive
B - Rougeur localisée au niveau de la conjonctive palpébrale
C - Rougeur localisée au niveau du cul-de-sac conjonctival
D - Rougeur localisée autour de la cornée
E - Rougeur persistant à l'instillation d'un collyre à la néosynéphrine 5 %
Bonne(s) réponse(s) : D E

C'est une rougeur ciliaire, localisée autour de la cornée (cercle périkératique), qui persiste après installation d'un collyre à la
néosynéphrine (vasoconstricteur qui fait disparaître une rougeur par hyperhémie conjonctivale).

Parmi les signes fonctionnels suivants, le(s)quel(s) accompagne(nt) une iridocyclite ?


A - Photophobie
B - Larmoiement
C - Trouble de la vision colorée
D - Héméralopie
E - Blépharospasme
Bonne(s) réponse(s) : A B E

Les troubles de la vision des couleurs peuvent accompagner une neuropathie optique ou une atteinte rétinienne. L'éméralopie
(gêne en vision crépusculaire et lors du passage de lumière à l'obscurité) se rencontre dans la myopie, la rétinopathie
pigmentaire.

1082
Exclusivement sur DOC - DZ : www.doc-dz.com NADJI 85
RESIDANAT EN POCHE TOME II
Cas Clinique en QCM

L'augmentation de la pression intra-oculaire constatée sur l'oeil droit peut être rattachée à :
A - L'hypermétropie (+ 1,50)
B - L'intumescence du cristallin
C - Un glaucome chronique à angle ouvert
D - L'inflammation intra-oculaire
E - Des synéchies pupillaires étendues
Bonne(s) réponse(s) : C D E

L'hypermétropie peut constituer un facteur anatomique prédisposant à la survenue d'un glaucome aigu par fermeture de
l'angle, de même qu'une intumescence (par plaie) du cristallin. Mais l'inflammation intra-oculaire peut entraîner des troubles
de résorption de l'humeur aqueuse ; les synéchies irido-cristalliniennes sur 360° peuvent entraîner un blocage pupillaire avec
hypertonie, enfin, le patient peut être atteint d'un glaucome chronique à angle ouvert préexistant à la poussée d'iridocyclite.

Le traitement de l'iridocyclite comportant l'instillation répétée de collyre à base de cortisone, quelle(s)


complication(s) peut-on voir survenir ?
A - Cataracte
B - Hypertonie
C - Dystrophie cornéenne
D - Sclérose des canalicules lacrymaux
E - Conjonctivite papillaire
Bonne(s) réponse(s) : A B

La cataracte apparait après un an de traitement environ ; l'hypertonie apparait après quelques mois de traitement. Les
complications contre-indiquent donc l'emploi de collyre à base de cortisone de façon prolongée.

Etant donné le métier du malade, il convient d'éliminer une éventuelle sidérose. Parmi les signes suivants,
le(s)quel(s) se rencontre(nt) au cours de la sidérose ?
A - Hypertonie
B - Cataracte
C - Altération de l'électrorétinogramme
D - Hétérochromie irienne
E - Mydriase
Bonne(s) réponse(s) : A B C D E

Tous ces éléments se rencontrent au cours de la sidérose oculaire ; il faut alors pratiquer des radiographies de l'orbite, à la
recherche d'un corps étranger intraoculaire radio-opaque.

Un sujet âgé de 70 ans, hypertendu (pression artérielle 21/11 cmHg), présente à la suite d'un accident vasculaire cérébral,
une diplopie par paralysie complète du III droit, associée à une hémiplégie totale du côté gauche, avec paralysie faciale
gauche. La conscience est normale.

Parmi les muscles suivants, le(s)quel(s) est(sont) paralysé(s) dans l'atteinte complète du III ?
A - Droit interne
B - Droit externe
C - Orbiculaire des paupières
D - Petit oblique
E - Grand oblique
Bonne(s) réponse(s) : A D

Le droit externe est innervé par le nerf moteur oculaire externe (VI), le grand oblique est innervé par le nerf pathétique (IV) ; le
muscle orbiculaire des paupières est innervé par le nerf facial (VII).

Comment est la pupille dans la paralysie complète du III ?


A - De taille normale
B - En mydriase
C - En myosis
D - Réactive à l'excitation lumineuse
E - Non réactive à la convergence
Bonne(s) réponse(s) : B E

Dans la paralysie complète du III, la pupille en mydriase, non réactive à l'excitation lumineuse (directe ou consensuelle), et
non réactive à la convergence.

1083
Exclusivement sur DOC - DZ : www.doc-dz.com NADJI 85
RESIDANAT EN POCHE TOME II
Cas Clinique en QCM

L'innervation parasympathique de la pupille suit le trajet du :


A - Nerf du droit supérieur
B - Nerf du droit inférieur
C - Nerf du petit oblique
D - Nerf du grand oblique
E - Nerf du droit interne
Bonne(s) réponse(s) : C

L'innervation parasympathique de la pupille suit le trajet du III, puis du rameau destiné au petit oblique, qu'il quitte pour se
terminer dans le ganglion ciliaire ; ensuite les nerfs ciliaires courts donnent naissance aux filets nerveux pour le corps ciliaire
et l'iris.

Comment est la diplopie dans la paralysie complète du III droit ?


A - Maximum dans plusieurs directions du regard
B - Homonyme
C - Augmentant dans le regard vers la droite
D - Augmentant dans le regard vers le haut
E - Disparaissant à l'occlusion de l'oeil gauche
Bonne(s) réponse(s) : A D E

Dans la paralysie complète du III droit, la diplopie est maximale dans plusieurs directions du regard (en bas, à gauche) ; elle
est croisée, augmente dans le regard vers le haut (par paralysie du droit supérieur), diminue dans le regard vers la droite, et
disparaît à l'occlusion d'un oeil (ce n'est pas une diplopie monoculaire).

Quel(s) examen(s) ophtalmologique(s) demander pour apprécier le déficit oculomoteur paralytique ?


A - Un test de Hess Lancaster
B - Echographie orbitaire
C - Angiographie fluorescéinique
D - Potentiels évoqués visuels
E - Test de duction forcée
Bonne(s) réponse(s) : A

Le test de Hess Lancaster permet l'étude des mouvements oculaires, en particulier lors des paralysies oculomotrices. Le test
de duction forcée, réalisé sous anesthésie locale, recherche un mauvais relâchement oculomoteur, par exemple par
incarcération d'un muscle dans une fracture du plancher de l'orbite au cours d'un traumatisme facial.

L'association paralysie du III droit et hémiplégie gauche évoque un syndrome topographique. Lequel ?
A - Millard-Gubler
B - Weber
C - Pannaud
D - Foville
E - Benedikt
Bonne(s) réponse(s) : B

Le syndrome de Weber associe une paralysie du côté de la lésion et une hémiplégie controlatérale. Le syndrome de Millard-
Gubler associe une paralysie faciale périphérique du côté de la lésion et une hémiplégie controlatérale. Le syndrome de
Parinaud associe une paralysie de la verticalité du regard et de la convergence avec des anomalies de la réflexivité pupillaire.
Le syndrome de Foville associe une paralysie du VI et une paralysie de la latéralité du regard du côté de la lésion. Le
syndrome de Bénédikt associe une paralysie du III du côté de la lésion, et, du côté opposé, des tremblements, des
mouvements chroréo-athétosiques.

L'association paralysie du III droit et hémiplégie gauche évoque une topographie lésionnelle bien précise.
Laquelle ?
A - Pied du pédoncule
B - Calotte pédonculaire
C - Commissure blanche postérieure
D - Pied de la protubérance
E - Calotte protubérantielle
Bonne(s) réponse(s) : A

Les hémorragies du pied du pédoncule sont marquées par un syndrome de Weber.

1084
Exclusivement sur DOC - DZ : www.doc-dz.com NADJI 85
RESIDANAT EN POCHE TOME II
Cas Clinique en QCM
Un enfant de 6 ans est vu en consultation parce qu'il présente une médiocre vision découverte par la médecine scolaire.
L'interrogatoire précise qu'il présente une photophobie. La vision n'est pas améliorable à plus de 3/10ème de chaque oeil de
loin et de près. L'hospitalisation est décidée parce qu'il suit mal sa scolarité.

Quel diagnostic doit être envisagé ?


A - Glaucome congénital
B - Cataracte congénitale
C - Tumeur chiasmatique
D - Décollement de rétine
E - Rétinoblastome
Bonne(s) réponse(s) : B

Le glaucome congénital entraîne une photophobie et une baisse d'acuité visuelle plus précoces.

Quelle(s) étiologie(s) peu(ven)t être envisagée(s) ?


A - Hérédo-familiale
B - Myopique
C - Rubéole
D - Traumatique
E - Médicamenteuse (chez la gestante)
Bonne(s) réponse(s) : A C E

L'hérédité, les viroses (rubéole, oreillon) et les médicaments pris par la mère (cytotoxiques, thalidomide) peuvent être à
l'origine d'une cataracte congénitale.

Quel(s) examen(s) clinique(s) permet(tent) d'affirmer le diagnostic ?


A - Biomicroscopie
B - Gonioscopie
C - Ophtalmoscopie
D - Angiographie fluorescéinique
E - Champ visuel
Bonne(s) réponse(s) : A B

L'examen du cristallin à la lampe à fente, et l'examen de la lueur pupillaire, en ophtalmoscopie permettent d'affirmer le
diagnostic.

Vous devez attendre l'absence d'anomalie à :


A - Echographie
B - Electrorétinogramme
C - Potentiels évoqués visuels
D - Radiographie d'orbite
E - Vision de couleurs
Bonne(s) réponse(s) : A B C D

La vision des couleurs peut être perturbée par les opacités cristalliniennes.

L'intervention proposée est :


A - Enucléation
B - Trabéculotomie
C - Extraction intracapsulaire
D - Extraction extracapsulaire
E - Intervention neurochirurgicale
Bonne(s) réponse(s) : D

La récupération peut être médiocre en raison d'une amblyopie.

1085
Exclusivement sur DOC - DZ : www.doc-dz.com NADJI 85
RESIDANAT EN POCHE TOME II
Cas Clinique en QCM
Un homme de 51 ans, diabétique insulinodépendant depuis 10 ans mais mal équilibré est hospitalisé pour une diplopie
apparue brutalement depuis 2 jours accompagnée de douleurs orbitaires. En fait le patient arrive à voir simple dans le regard
vers la gauche. Sa diplopie débute à partir de la position primaire et le décalage s'accentue dans le regard vers la droite.
Il se présente d'ailleurs avec une position de la tête tournée vers la droite pour favoriser le regard plus agréable vers la
gauche. C'est le premier épisode. Il n'y a pas de céphalées, pas de fièvre, pas de déficit du champ visuel, pas de déficit
neurologique en dehors d'une limitation manifeste d'abduction de l'oeil droit. La TA est chiffrée à 18/10 cm Hg.
On pense à une paralysie oculomotrice du nerf abducteur droit (VIème paire crânienne) complication du diabète.

Quel est le territoire du nerf abducteur droit (VIème paire crânienne) ?


A - Le muscle droit externe droit et le muscle droit interne gauche
B - Les muscles droit externe, petit oblique, droit supérieur, droit inférieur de l'oeil droit
C - Le muscle droit interne droit
D - Le muscle droit externe droit
E - Le muscle droit externe droit et la motricité de la pupille
Bonne(s) réponse(s) : D

C'est le seul muscle innervé par le VI.

Parmi les autres mécanismes de diplopie mono ou binoculaire vous retenez :


A - Des opacités cristalliniennes
B - Une hémianopsie latérale homonyme droite
C - Une hétérophorie décompensée
D - Une occlusion de la veine centrale de la rétine
E - Une anisocorie
Bonne(s) réponse(s) : A C

L'hétérophorie est une insuffisance de convergence qui peut se manifester par une diplopie lors qu'elle est décompensée.

Quel(s) examen(s) réalise-t-on pour caractériser le déficit oculomoteur ?


A - Electrorétinogramme
B - Test de Lancaster
C - Etude des champs d'action des muscles oculomoteurs
D - Examen du champ visuel
E - Skiascopie
Bonne(s) réponse(s) : B C

Le test le Lancaster permet l'étude des champs d'action des muscles oculomoteurs.

La paralysie oculomotrice dans le cadre du diabète peut être due à :


A - Une hémorragie du vitré
B - Un accident vasculaire cérébral
C - L'exophtalmie
D - Une hypoglycémie
E - La neuropathie par microangiopathie
Bonne(s) réponse(s) : B D E

L'hémorragie du vitré n'entraîne pas de paralysie oculomotrice. L'exophtalmie n'est pas une complication du diabète.

Citez une autre manifestation ophtalmologique du diabète :


A - Glaucome chronique à angle ouvert
B - Glaucome par fermeture de l'angle
C - Glaucome néovasculaire
D - Uvéite antérieure
E - Aucune de ces possibilités
Bonne(s) réponse(s) : C

Le glaucome néovasculaire est la complication ultime de la rétinopathie diabétique ischémique avec prolifération de néo-
vaisseaux et rubéose irienne.

1086
Exclusivement sur DOC - DZ : www.doc-dz.com NADJI 85
RESIDANAT EN POCHE TOME II
Cas Clinique en QCM
Un homme de 50 ans, ingénieur, myope fort, porteur de lunettes, consulte pour une baisse de l'acuité visuelle de l'oeil droit
sans douleur, rapidement progressive avec amputation du champ visuel inférieur, déformation des images. L'interrogatoire
précise que les troubles remontent à 8 jours.
Tout a commencé après un choc modéré sur l'oeil droit par la perception dans le champ nasal d'éclairs lumineux déclenchés
par les mouvements brusques de l'oeil et particulièrement nets dans l'obscurité..Ces éclairs se reproduisant toujours dans les
mêmes circonstances se sont ensuite accompagnés de perception de points noirs, de filaments, et c'est devant l'apparition
des déformations des images et de l'amputation progressive du champ visuel de la périphérie vers le centre que le patient
vient consulter. L'examen ophtalmologique précise la baisse d'acuité visuelle du côté droit à 1/20, il existe une discrète
hypotonie oculaire. Le segment antérieur est normal. Après dilatation pupillaire l'examen du fond d'oeil précise l'existence d'un
soulèvement rétinien supérieur prédominant dans le secteur temporal ayant atteint la macula. Ce décollement est en relation
avec une vaste déchirure à lambeau située sur le méridien de 10 heures. L'examen de l'oeil du côté opposé permet de
retrouver des lésions dégénératives situées dans le quadrant temporal supérieur.
Le reste de l'examen clinique est entièrement normal chez cet adulte en bonne santé jusque-là.

Parmi les signes fonctionnels décrits, vous évoquez le diagnostic de décollement rétinien devant :
A - Phosphènes
B - Baisse d'acuité visuelle
C - Hypotonie
D - Métamorphopsies
E - Amputation rapide et progressive du champ visuel de la périphérie vers le centre
Bonne(s) réponse(s) : B D E

Les phosphènes évoquent des tractions du vitré sur la rétine, mais pas obligatoirement un décollement.
L'hypotonie n'est pas un signe fonctionnel, mais un signe physique. Les métamorphopsies et la baisse d'acuité visuelle
traduisent le soulèvement de la macula.

Dans notre observation vous retenez comme facteur(s) favorisant(s) de décollement de rétine :
A - L'âge
B - Le myopie
C - Le métier
D - Le traumatisme
E - Le sexe
Bonne(s) réponse(s) : A B D

Il n'est pas précisé dans l'observation si le choc est survenu au travail ou pas.

Quelle est la sanction thérapeutique de ce décollement de rétine constitué ?


A - Abstention
B - Photocoagulation
C - Traitement médical
D - Chirurgie
E - Rééducation oculaire
Bonne(s) réponse(s) : D

Seule la chirurgie peut espérer guérir un décollement de rétine constitué.

Le traitement préventif par photocoagulation au laser des lésions dégénératives de l'oeil du côté opposé est
impératif en présence de :
A - Déchirure de la rétine
B - Décollement postérieur du vitre
C - Atrophie choroïdienne périphérique
D - Hémorragies rétiniennes
E - Exsudats
Bonne(s) réponse(s) : A

Une déchirure de la rétine expose à un risque majeur de décollement de la rétine de l'oeil adelphe.

1087
Exclusivement sur DOC - DZ : www.doc-dz.com NADJI 85
RESIDANAT EN POCHE TOME II
Cas Clinique en QCM
Une femme de 50 ans, comptable, se plaint brutalement, en fin de journée, de douleurs importantes au niveau de l'oeil et de
l'orbite gauches. Elle signale en outre une diminution de l'acuité visuelle de ce côté.
A l'examen, la constatation d'un oeil rouge sans sécrétion permet d'évoquer une crise de glaucome aigu.
L'interrogatoire révèle que cette patiente est traitée depuis plusieurs mois pour dépression nerveuse, ménopause précoce et
rhumatisme chronique.

Le glaucome par fermeture de l'angle survient le plus souvent chez les sujets porteurs d'une amétropie.
Laquelle ?
A - Myopie
B - Astigmatisme
C - Hypermétropie
D - Anisométropie
E - Aphakie
Bonne(s) réponse(s) : C

Par diminution de la profondeur de la chambre antérieure.

Parmi les signes suivants, le(s)quel(s) est sont présent(s) au cours d'une crise de glaucome aigu ?
A - Oedème cornéen
B - Semi mydriase
C - Hypertonie oculaire
D - Aréflexie pupillaire
E - Cercle périkératique
Bonne(s) réponse(s) : A B C D E

Tableau typique de glaucome aigu par fermeture de l'angle.

Parmi les médicaments suivants prescrits à la patiente, le(s)quel(s) est(sont) susceptible(s) de déclencher une
crise de glaucome par fermeture de l'angle ?
A - Antipaludéen de synthèse
B - Antispasmodique digestif
C - Oestrogène
D - Anti-inflammatoire non stéroïdien
E - Antidépresseur
Bonne(s) réponse(s) : B E

Les antipaludéens de synthèse peuvent entraîner l'apparition d'une rétinopathie.

Parmi les produits suivants le(s)quel(s) proposez-vous pour réduire la crise ?


A - Pilocarpine collyre 2 %
B - Adrénaline collyre 1 %
C - Acétazolamide 500 mg en intraveineux
D - Mannitol 20 % (2 g par kg de poids) en intraveineux
E - Acéclidine ou Glaucostat® collyre
Bonne(s) réponse(s) : A C D E

La pression intra-oculaire est diminuée par l'injection de Diamox® et de mannitol, puis la pupille est resserrée par l'instillation
de pilocarpine ou de Glaucostat® (fallait-il choisir une seule des deux propositions ?).

Une jeune femme est amenée chez son généraliste après avoir reçu de l'ammoniaque dans les deux yeux. Elle souffre, elle a
un blépharospasme, elle est photophobe. A l'ouverture forcée des paupières, on constate une hyperhémie conjonctivale et
une cornée opalescente.

Quel est le geste primordial à effectuer en urgence par le généraliste ?


A - Pansement occlusif sur les 2 yeux
B - Installation d'un collyre antibiotique
C - Lavage abondant au sérum, des 2 yeux
D - Installation d'un collyre myotique
E - Prélèvement des sécrétions afin de déterminer le pH de la solution pour préparer une solution tampon
Bonne(s) réponse(s) : C

Le lavage abondant à l'eau est le seul geste à pratiquer, en extrême urgence, et pendant 10 à 15 minutes, avant le transfert
en milieu spécialisé.

1088
Exclusivement sur DOC - DZ : www.doc-dz.com NADJI 85
RESIDANAT EN POCHE TOME II
Cas Clinique en QCM

Par quel mécanisme physico-chimique l'ammoniaque constitue un danger pour les yeux ?
A - Nécrose et coagulation des protéines de surface
B - Diffusion intratissulaire de l'agent chimique
C - Atteinte mécanique de l'épithélium cornéen
D - Ischémie des vaisseaux ciliaires antérieurs
E - Dénaturation de la qualité des larmes
Bonne(s) réponse(s) : B

La diffusion de l'ammoniaque à travers la cornée dans le segment antérieur représente le danger principal (risque de
cataracte, glaucome...).

Les lésions immédiates entraînées par l'ammoniaque et objectivées par l'examen ophtalmologique comportent :
A - Brûlure tarsale
B - Brûlure conjonctivale avec zone d'ischémie
C - Cataracte
D - Brûlure des points lacrymaux
E - Brûlure du stroma cornéen
Bonne(s) réponse(s) : A B D E

La cataracte peut apparaître dans les heures ou les jours qui suivent une brûlure par une base forte.

Chez le spécialiste, en urgence, dans les cas graves, sont envisagés :


A - Lavage des voies lacrymales
B - Tonométrie avec prise du coefficient d'évacuation de l'humeur aqueuse
C - Instillation d'une solution tampon
D - Lavage des culs de sac conjonctivaux à la seringue
E - Potentiels évoqués visuels
Bonne(s) réponse(s) : A D

Il faut éliminer tout reste d'agent chimique pouvant entretenir le processus de brûlure.

L'examen ophtalmologique met en évidence des zones d'ischémie conjonctivale importante, un oedème
cornéen étendu et une nécrose des points lacrymaux. Quel(s) geste(s) de chirurgie sont à envisager ?
A - Suture des 2 paupières (blépharoraphie)
B - Péritomie limbique
C - Ponction lavage de chambre antérieure
D - Sous-conjonctivale d'une solution tampon
E - Phakoexérèse
Bonne(s) réponse(s) : C

La ponction lavage de la chambre antérieure permet d'éliminer le toxique qui a déjà pénétré dans le segment antérieur.

Dans les suites immédiates d'une brûlure par base, il faut instiller :
A - Collyre à l'atropine
B - Collyre à l'épinéphrine
C - Collyre à la pilocarpine
D - Collyre cortisone
E - Collyre antibiotique
Bonne(s) réponse(s) : A E

L'atropine produit une mydriase qui prévient les synéchies irido-cristaliniennes. Les antibiotiques préviennent une surinfection
bactérienne.

1089
Exclusivement sur DOC - DZ : www.doc-dz.com NADJI 85
RESIDANAT EN POCHE TOME II
Cas Clinique en QCM
Ce sujet de 64 ans, présente brutalement des douleurs oculaires droites intenses, accompagnées d'une baisse visuelle
considérable, d'une rougeur de l'oeil droit et des nausées. La cornée droite apparaît trouble à l'examen. A la palpation, le
globe oculaire est dur comme une "bille de verre". Il s'agit vraisemblablement d'une crise de glaucome aigu.

Parmi les 5 propositions suivantes concernant les étiologies des baisses visuelles brutales. Il faut envisager
chez ce patient :
A - Une iridocyclite (uvéite antérieure)
B - Une kératite virale
C - Une oblitération de l'artère centrale de la rétine
D - Une thrombose de la veine centrale de la rétine
E - Une névrite optique aiguë
Bonne(s) réponse(s) :

QUESTION ANNULEE

Parmi les 5 propositions suivantes concernant le mécanisme physiopathologique à l'origine de cette crise de
glaucome aigu, vous retenez comme vraisemblable(s) :
A - Une hypersécrétion d'humeur aqueuse
B - Une hyposécrétion d'humeur aqueuse par obstruction des veines aqueuses
C - Une hyposécrétion d'humeur aqueuse par sclérose du trabéculum scléral
D - Une hypoexcretion d'humeur aqueuse par fermeture de l'angle irido-cornéen
E - Un blocage pupillaire
Bonne(s) réponse(s) : D E

Le blocage pupillaire entraîne la fermeture de l'angle iridocornéen, et l'hypertonie oculaire aiguë.

Parmi les 5 propositions suivantes concernant les signes de l'examen ophtalmologique de ce patient avant
traitement, quel est le signe qui ne s'observe pas habituellement ?
A - Un cercle périkératique
B - Un oedème cornéen
C - Un myosis
D - Une augmentation considérable de la pression oculaire
E - Une aréflexie pupillaire
Bonne(s) réponse(s) : C

La pupille est en demi mydriase.

Parmi les 5 thérapeutiques suivantes concernant le traitement de ce patient, quelle est celle qui est
formellement contre-indiquée ?
A - La prescription d'inhibiteurs de l'anhydrase carbonique
B - La perfusion de mannitol
C - L'instillation d'atropine
D - L'instillation de pilocarpine
E - La prescription de glycérine par voie buccale
Bonne(s) réponse(s) : C

L'atropine entretient la mydriase ; elle est donc formellement contre-indiquée.

L'interrogatoire du patient va rechercher des médicaments à l'origine possible du déclenchement de cette crise.
Parmi ces thérapeutiques, laquelle ne peut être incriminée ?
A - Les inhibiteurs de la monoamine-oxydase
B - Les antiparkinsoniens
C - Les dérives nitrés
D - L'éthambutol
E - Les antidépresseurs tricycliques
Bonne(s) réponse(s) : D

L'éthambutol peut entraîner une neuropathie optique.

1090
Exclusivement sur DOC - DZ : www.doc-dz.com NADJI 85
RESIDANAT EN POCHE TOME II
Cas Clinique en QCM
Un homme de 60 ans, consulte en urgence pour une baisse brutale de l'acuité visuelle de l'oeil gauche.
Il est discrètement hypertendu, diabétique et son cholestérol est à la limite supérieure de la normale.
L'examen ophtalmologique vous révèle une acuité visuelle réduite à 1/10è à gauche, 10/10è à droite. Il existe une amputation
inférieure du champ visuel gauche avec une limite horizontale nette passant par le point de fixation. L'examen du fond d'oeil
confirme le diagnostic en montrant un oedème rétinien ischémique de l'hémirétine supérieure avec macula rouge cerise.

Dans une occlusion de la branche temporale supérieure de l'artère centrale de la rétine, l'amputation du champ
visuel est :
A - Nasale supérieure
B - Nasale inférieure
C - Temporale supérieure
D - Temporale inférieure
E - Totale
Bonne(s) réponse(s) : B

Le territoire rétinien temporal supérieur correspond à la zone nasale inférieure du champ visuel.

Dans une occlusion de l'artère centrale de la rétine, l'examen du fond d'oeil montre :
A - Un oedème ischémique de la rétine
B - Un oedème maculaire cystoïde
C - Un oedème papillaire
D - Des hémorragies diffuses
E - Des hémorragies le long des vaisseaux
Bonne(s) réponse(s) : A

C'est l'occlusion de la veine centrale de la rétine qui entraîne un oedème papillaire, des hémorragies diffuses et des
hémorragies le long des vaisseaux. La forme oedémateuse de l'OVCR peut se compliquer d'oedème maculaire cystoïde.

Dans un intérêt diagnostique, il est utile de prescrire :


A - Champ visuel Goldmann
B - Test de Lancaster
C - Echographie
D - Angiographie
E - Electrorétinographie
Bonne(s) réponse(s) : A D

Le champ visuel confirme l'amputation bien limitée du champ visuel. L'angiographie rétinienne en fluorescence permet de
localiser précisément le site de l'occlusion artérielle.

Quel ou quels examen(s) complémentaire(s) a (ont) un intérêt étiologique ?


A - Vitesse de sédimentation
B - Clearance de la créatinine
C - Calcémie
D - Phosphorémie
E - Groupage tissulaire H.L.A.
Bonne(s) réponse(s) : A

La vitesse de sédimentation, si elle est très élevée, oriente vers le diagnostic de maladie de Horton.

Les occlusions de l'artère centrale de la rétine peuvent résulter de :


A - Thrombose
B - Hyperviscosité sanguine
C - Embole de cholestérol
D - Embole de calcaire
E - Embole de plaquettes
Bonne(s) réponse(s) : A B C D E

L'oblitération de l'artère centrale de la rétine peut résulter d'un thrombose locale, d'une hyperviscosité sanguine, ou d'un embol
de nature variable (cholestérol, calcaire, plaquette...).

1091
Exclusivement sur DOC - DZ : www.doc-dz.com NADJI 85
RESIDANAT EN POCHE TOME II
Cas Clinique en QCM

Parmi ces thérapeutiques, toutes sont utiles en urgence, sauf une. Laquelle ?
A - Anticoagulants
B - Anti-agrégants plaquettaires
C - Vasodilatateurs
D - Macromolécules
E - Thrombolytiques
Bonne(s) réponse(s) : B

Question ambiguë, les fibrinolytiques n'ayant pas fait la preuve de leur intérêt dans l'occlusion de l'artère centrale de la rétine.

Un homme se présente après avoir reçu de l'eau de javel concentrée dans un oeil :

Le danger consiste dans le fait que :


A - L'eau de javel est acide et va entraîner une ulcération cornéenne
B - Il risque d'y avoir des corps étrangers sous la paupière supérieure
C - L'eau de javel est une base forte entraînant une brûlure cornéo-conjonctivale grave
D - Les voies lacrymales risquent d'être obstruées
E - L'eau de javel contient des colorants toxiques
Bonne(s) réponse(s) : C

Les bases provoquent des brûlures prolongées et profondes des tissus oculaires, donc très graves.

Le médecin généraliste devant un tel cas doit :


A - Mettre le blessé sous antibiotiques par voie générale
B - S'assurer que le blessé est vacciné contre le tétanos
C - Laver l'oeil atteint à grande eau, même non stérile
D - Se contenter de faire une occlusion de l'oeil blessé
E - Envoyer le blessé dans un centre spécialisé
Bonne(s) réponse(s) : C E

Le lavage doit être immédiat (+++) et très abondant (+++), avant tout transfert en centre spécialisé. Ceci conditionne de façon
déterminante le pronostic à moyen terme.

Quel(s) est (sont) le(s) geste(s) thérapeutique(s) spécialisé(s) indiqué(s) en urgence ?


A - Un nettoyage des culs de sac palpébraux
B - Un lavage des voies lacrymales
C - Une instillation d'Atropine
D - Une injection sous-conjonctivale de vaso-dilatateur
E - Une désinsertion conjonctivale en cas de chémosis ischémique
Bonne(s) réponse(s) : A B C D E

Les gestes sont réalisés à l'arrivée en centre spécialisé, après avoir renouvelé le lavage abondant et prolongé à l'eau.

Quel(s) est (sont) le(s) risque(s) oculaire(s) entraîné(s) par cet accident ?
A - Une sténose des voies lacrymales dans les semaines suivantes
B - Un symblépharon
C - Une uvéite antérieure
D - Un glaucome
E - Une opacification de la cornée
Bonne(s) réponse(s) : A B C D E

La brûlure des voies lacrymales entraîne des sténoses cicatricielles. La brûlure conjonctivale entraîne la formation de
cicatrices entre la conjonctive bulbaire et palpébrale (symblépharon). La pénétration intra-oculaire du produit entraîne des
lésions de l'endothélium cornéen (oedème), de l'angle irido-cornéen (glaucome) et du cristallin (cataracte).

1092
Exclusivement sur DOC - DZ : www.doc-dz.com NADJI 85
RESIDANAT EN POCHE TOME II
Cas Clinique en QCM
Un blessé, à la suite d'un choc sur le côté droit de la face, présente un oedème de la pommette et un hématome conjonctival.

A l'examen clinique, vous recherchez systématiquement la présence :


A - D'une anesthésie de l'aile du nez
B - D'une diplopie
C - D'une épistaxis contemporaine de l'accident
D - D'un emphysème sous-cutané
E - D'une limitation de l'ouverture buccale
Bonne(s) réponse(s) : A B C D E

Les signes sont évocateurs d'une fracture de la paroi interne ou du plancher de l'orbite, qui sont le plus souvent atteints dans
les contusions faciales.

L'investigation radiologique élective comporte :


A - Un profil
B - Un Worms
C - Un Blondeau (nez, menton, plaque)
D - Un face basse (nez, front, plaque)
E - Un Hirtz (incidence axiale)
Bonne(s) réponse(s) : C E

Il faut explorer l'orbite et les sinus voisins (opacité par hemosinus secondaire à une fracture).

Quel(s) examen(s) complémentaire(s) peu(ven)t être utile(s) s'il existe une suspicion clinique de diplopie :
A - Un électromyogramme
B - Un test de Schirmer
C - Un test de duction forcée
D - Un test de Lancaster
E - Un examen tomodensitométrique de la région orbitaire avec coupes frontales
Bonne(s) réponse(s) : C D E

La diplopie peut révéler une incarcération musculaire dans le foyer de fracture. La limitation des mouvements est confirmée
par le test de Lancaster (vision dans le regard latéral) et par le test de duction forcée (résistance à la traction active du
muscle). L'incarcération est mise en évidence avec les coupes frontales d'un examen tomodensitométrique.

Dans l'éventualité où les clichés radiologiques et les examens cliniques complémentaires confirment l'existence
d'une fracture maxillo-malaire et d'une diplopie, le traitement fera appel :
A - A une simple réduction de la fracture par voie percutanée au crochet
B - A une simple réfection du plancher orbitaire
C - Une libération du canal optique
D - Une canthopexie interne
E - Un blocage inter-maxillaire associé
Bonne(s) réponse(s) : B

Il faut simplement désincarcérer le muscle, et reformer le plancher orbitaire.

Dans l'éventualité précédente, un tel traumatisme pourra entraîner à titre de séquelle(s) :


A - Une diplopie persistante
B - Une énophtalmie
C - Une névralgie du nerf sous orbitaire
D - Une limitation de l'ouverture buccale
E - Une anesthésie de l'aile du nez
Bonne(s) réponse(s) : A B C E

Il peut persister une diplopie par cicatrice musculaire fibreuse. Il peut exister une énophtalmie en cas d'incarcération de la
graisse orbitraire dans le foyer de fracture. Les lésions nerveuses provoquées par la fracture peuvent entraîner une
névralgie.du nerf sous orbitaire et une anesthésie de l'aile du nez par lésion d'une branche du facial.

1093
Exclusivement sur DOC - DZ : www.doc-dz.com NADJI 85
RESIDANAT EN POCHE TOME II
Cas Clinique en QCM
Un homme de 55 ans, se plaint d'un oeil droit douloureux et rouge depuis 48 heures. A l'examen, on retrouve au niveau de
l'oeil droit des signes d'iridocyclite. La vision est égale à 8/10 oeil droit, 10/10 oeil gauche après correction d'une
hypermétropie de + 1,5 dioptrie. Le tonus oculaire est égal à 28 mmHg oeil droit et à 16 mmHg oeil gauche. L'examen
gonioscopique met en évidence un angle irido-cornéen ouvert à droite comme à gauche. Le fond d'oeil est normal des deux
cotés Dans les antécédents du patient, on retrouve une symptomatologie évoquant une spondylarthrite ankylosante et la
notion de projections métalliques au cours du travail .

Parmi les signes cliniques suivants, le(s)quel(s) vous permet(tent) de poser le diagnostic d'iridocyclite ?
A - Précipités à la face postérieure de la cornée
B - Myosis
C - Sécrétions muco-purulentes
D - Synéchies pupillaires
E - Halos colorés
Bonne(s) réponse(s) : A B D

Les sécrétions muco-purulentes font partie du tableau de conjonctivite. Les halos colorés sont perçus lorsque la cornée est
oedémateuse (glaucome aigü par fermeture de l'angle).

Quel(s) est (sont) le ou les caractère(s) de la rougeur au cours d'une iridocyclite ?


A - Rougeur diffuse à toute la conjonctive
B - Rougeur localisée au niveau de la conjonctive palpébrale
C - Rougeur localisée au niveau du cul-de-sac conjonctival
D - Rougeur périkératique
E - Rougeur par hémorragie sous conjonctivale
Bonne(s) réponse(s) : D

C'est une rougeur ciliaire, diffuse mais prédominant autour de la cornée : c'est le cercle périkératique.

Parmi les signes fonctionnels suivants, le(s)quel(s) accompagne(nt) une iridocyclite ?


A - Photophobie
B - Larmoiement
C - Trouble de la vision colorée
D - Héméralopie
E - Blépharospasme
Bonne(s) réponse(s) : A B E

Les troubles de la vision des couleurs où de la vision nocturne (héméralopie) traduisent une atteinte rétinienne ou du nerf
optique, sans rapport avec l'iridocyclite.

L'augmentation de la pression intra-oculaire constatée sur l'oeil droit peut être rattachée à :
A - L'hypermétropie (+ 1,50)
B - L'intumescence du cristallin
C - Un glaucome chronique à angle ouvert
D - L'inflammation intra-oculaire
E - Des synéchies pupillaires étendues
Bonne(s) réponse(s) : D E

L'inflammation intra-oculaire entraîne un dépôt dans l'angle iridocornéen, qui ralentit l'élimination de l'humeur aqueuse. Elle
peut aussi entraîner des synéchies iridocristalliniennes étendues, pouvant au maximum entraîner un blocage pupillaire.

Le traitement de l'iridocyclite comportant l'instillation répétée de collyre à base de cortisone, quelle(s)


complication(s) peut-on voir survenir à long terme ?
A - Cataracte
B - Hypertonie
C - Dystrophie cornéenne
D - Sclérose des canalicules lacrymaux
E - Conjonctivite papillaire
Bonne(s) réponse(s) : A B

La corticothérapie prolongée par voie locale peut entraîner une cataracte et un glaucome unilatéraux (++).

1094
Exclusivement sur DOC - DZ : www.doc-dz.com NADJI 85
RESIDANAT EN POCHE TOME II
Cas Clinique en QCM
Une femme de 60 ans consulte en urgence pour un oeil droit douloureux depuis 24 heures. Cette symptomatologie est
apparue quelques heures après un examen du fond d'oeil pratiqué sous mydriase. La douleur est intense, permanente,
émétisante. Elle s'accompagne à l'examen d'une rougeur périkératique. La pression intra-oculaire atteint 70 mm Hg. Ce
tableau clinique est suffisamment typique pour que le diagnostic de glaucome aigu soit porté sans hésitation.

Quels sont les 3 signes physiques que l'examen oculaire attentif et comparatif peut mettre en évidence du côté
droit ?
A - Un hyphéma
B - Un trouble cornéen
C - Une baisse de l'acuité visuelle
D - Un iridodonésis
E - Un effacement de la chambre antérieure
Bonne(s) réponse(s) : B C E

L'hyphéma apparaît après un traumatisme. L'iridodonésis (mouvement de l'iris) apparaît après subluxation post-traumatique
du cristallin.

Quels sont les deux mécanismes physio-pathologiques par lesquels il est possible d'expliquer la brutale
ascension de la pression intra-oculaire ?
A - Par hypersécrétion de l'humeur aqueuse
B - Par obstruction du trabéculum, l'angle iridocornéen étant ouvert
C - Par blocage pupillaire secondaire à la mydriase
D - Par fermeture de l'angle irido-cornéen secondaire au blocage pupillaire
E - Par luxation antérieure du cristallin
Bonne(s) réponse(s) : C D

La mydriase entraîne le blocage pupillaire, qui lui-même provoque une augmentation de pression dans la chambre
postérieure, qui ferme l'angle irido-cornéen.

D'autres circonstances déclenchantes que la mydriase médicamenteuse sont susceptibles de provoquer un


glaucome aigu. Lesquelles ?
A - Le choc affectif
B - Une intervention chirurgicale
C - L'instillation de Pilocarpine
D - Le séjour dans l'obscurité
E - L'instillation d'un collyre corticoïde
Bonne(s) réponse(s) : A B D

La pilocarpine prévient la survenue d'un glaucome aigu. Le collyre corticoïde peut entraîner un glaucome chronique à angle
ouvert.

Citez les mesures thérapeutiques à mettre en oeuvre d'urgence :


A - Instiller un collyre myotique
B - Injecter par voie intraveineuse un inhibiteur de l'anhydrase carboniue
C - Administrer un antalgique
D - Faire ingérer un déshydratant hyperosmotique
E - Combattre la rougeur ciliaire par un collyre sympathicomimétique
Bonne(s) réponse(s) : A B C

Il faut abaisser la pression intra-oculaire, et lever le blocage pupillaire en mettant la pupille en myosis.

L'examen ophtalmologique ayant révélé que l'oeil gauche présente une prédisposition anatomique au glaucome
aigu, quelles sont les options thérapeutiques qui doivent être proposées à cette patiente ?
A - Surveillance régulière de la pression intra-oculaire
B - Instillation définitive d'un collyre bétabloquant
C - Instillation définitive d'un collyre parasympathicomimétique
D - Iridectomie périphérique chirurgicale
E - Iridotomie au laser
Bonne(s) réponse(s) : C D E

La surveillance régulière de la pression intra-oculaire et le traitement par collyre bêta-bloquant concerne le glaucome
chronique à angle ouvert.

1095
Exclusivement sur DOC - DZ : www.doc-dz.com NADJI 85
RESIDANAT EN POCHE TOME II
Cas Clinique en QCM
Un homme se présente après avoir reçu de l'eau de Javel dans un oeil.

Le danger consiste dans le fait que :


A - L'eau de Javel est acide et va entraîner une ulcération cornéenne
B - Il risque d'y avoir des corps étrangers sous la paupière supérieure
C - L'eau de Javel est une base forte entraînant une brûlure cornéoconjonctivale grave
D - L'eau de Javel peut entraîner une uvéite réactionnelle
E - L'eau de Javel contient des colorants toxiques
Bonne(s) réponse(s) : C D

L'importance de la réaction dépend de la dilution de l'eau de Javel.

Le médecin généraliste devant un tel cas doit :


A - Mettre le blessé sous antibiotiques par voie générale
B - S'assurer que le blessé est vacciné contre le tétanos
C - Laver l'oeil atteint à grande eau, même non stérile
D - Se contenter de faire une occlusion de l'oeil blessé
E - Envoyer le blessé dans un centre spécialisé
Bonne(s) réponse(s) : C E

Le lavage immédiat et prolongé est capital avant le transfert en service spécialisé.

Quelle(s) est (sont) la(les) conséquence(s) immédiate(s) de cette brûlure par eau de Javel ?
A - Une brûlure du point lacrymal inférieur
B - Une ischémie conjonctivale du secteur nasal
C - Une ulcération épithéliale cornéenne
D - Un oedème stromal
E - Un tyndall de la chambre antérieure
Bonne(s) réponse(s) : A B C D E

Les dégâts sont variables selon la durée d'exposition, et la concentration du produit.

Quel(s) est (sont) le(s) geste(s) thérapeutique(s) spécialisé(s) indiqué(s) en urgence ?


A - Un nettoyage des culs de sac palpébraux
B - Un lavage des voies lacrymales
C - Une instillation d'Atropine
D - Une antibiothérapie locale
E - Une injection sous-conjonctivale de vasodilatateurs
Bonne(s) réponse(s) : A B C D E

Il faut lutter contre l'ischémie et la nécrose tissulaire en lavant à nouveau abondamment, puis en traîtant par vaso-dilatateurs,
antibiotiques et Atropine.

Un patient de 50 ans présente brutalement dans un oeil des phosphènes et des corps flottants vitréens abondants. Quelques
jours plus tard, sa vision baisse fortement, ce qui nécessite son hospitalisation pour intervention chirurgicale. Vous évoquez le
diagnostic d'un décollement de rétine par déchirure.

Quel(s) état(s) anatomique(s) prédispose(nt) à cette pathologie oculaire ?


A - Astigmatisme
B - Hypermétropie
C - Aphaquie
D - Myopie
E - Angle iridocornéen étroit
Bonne(s) réponse(s) : C D

L'aphaquie, la myopie, l'âge et les antécédents de traumatisme constituent les facteurs prédisposants du décollement.

1096
Exclusivement sur DOC - DZ : www.doc-dz.com NADJI 85
RESIDANAT EN POCHE TOME II
Cas Clinique en QCM

Quel examen ophtalmologique permet de faire un diagnostic précis lorsque le vitré est totalement opaque ?
A - Ophtalmoscopie
B - Bimicroscopie avec verre de contact
C - Gonioscopie
D - Angiographie à la fluorescéïne
E - Echographie
Bonne(s) réponse(s) : E

L'échographie permet d'examiner la rétine à travers un vitré opaque (hémorragie intravitréenne).

La baisse visuelle accompagnant le décollement de la rétine est due à :


A - Un soulèvement de la macula
B - Une hémorragie intra-vitréenne
C - Une cataracte
D - Une hypotonie
E - Un décollement postérieur du vitré
Bonne(s) réponse(s) : A B

C'est surtout le soulèvement de la macula qui est responsable de la baisse d'acuité visuelle.

Quelle intervention chirurgicale doit être effectuée chez ce patient ?


A - Trabéculectomie
B - Extraction Intra-capsulaire
C - Extraction Extra-capsulaire
D - Indentation sclérale avec cryocoagulation
E - Photocoagulation au laser
Bonne(s) réponse(s) : D

L'indentation et la cryo-application sont les deux gestes fréquemment associés pour traiter un décollement de rétine simple.

Un homme de 35 ans vous est adressé à la suite d'un accident de travail. Il a reçu un éclat métallique dans l'oeil gauche, en
frappant avec un marteau sur un burin. Il a une photophobie et un larmoiement modérés, l'acuité visuelle est de 1/10ème, non
améliorable.

Dans ce contexte, la présence d'un corps étranger intra-oculaire est suspectée devant :
A - Une hémorragie sous-conjonctivale
B - Une plaie cornéenne
C - Une plaie conjonctivale
D - Une baisse d'acuité visuelle
E - Une hypotonie
Bonne(s) réponse(s) : A B C E

La baisse d'acuité visuelle n'est pas particulièrement évocatrice de ce diagnostic.

Dans ce contexte est-il exact que ?


A - L'examen de fond d'oeil après dilatation s'impose en cas de suspicion de corps étranger vitréo-rétinien
B - L'hémorragie du vitré masque un corps étranger vitréo-rétinien
C - L'électrorétinogramme est un élément important du diagnostic en urgence
D - L'I.R.M. risque de mobiliser un corps étranger intra-oculaire et d'aggraver les lésions initiales
E - Les radiographies du crâne et le scanner permettent souvent la localisation des corps étrangers vitréo-
rétiniens
Bonne(s) réponse(s) : A B D E

L'électrorétinogramme est important pour rechercher une toxicité rétinienne (sidérose), à distance du traumatisme initial.

L'examen du fond d'oeil fait suspecter un éclat métallique intra-vitréen. Ceci est confirmé par les examens
paracliniques. Le traitement en urgence comporte :
A - Une antibiothérapie locale
B - Une extraction du corps étranger
C - Une greffe de cornée
D - Une mise à jour de la vaccination anti-tétanique
E - Une antibiothérapie générale
Bonne(s) réponse(s) : A B D E

Il faut aussi fermer la porte d'entrée par des points de suture.


1097
Exclusivement sur DOC - DZ : www.doc-dz.com NADJI 85
RESIDANAT EN POCHE TOME II
Cas Clinique en QCM

Après extraction du corps étranger, l'évolution peut se compliquer de :


A - Une baisse d'acuité visuelle
B - Un décollement de la rétine
C - Une infection
D - Une cataracte
E - Une exophtalmie
Bonne(s) réponse(s) : A B C D

La baisse d'acuité visuelle peut être secondaire à un décollement de rétine, à une cataracte ou à une infection (endophtalmie).

Un homme de 60 ans, consulte en urgence pour une baisse brutale de l'acuité visuelle de l'oeil gauche. Il est discrètement
hypertendu, diabétique et son cholestérol est à la limite supérieure de la normale.
L'examen ophtalmologique vous révèle une acuité visuelle réduite à 1/10è à gauche. 10/1Oè à droite. Il existe une amputation
inférieure du champ visuel gauche avec une limite horizontale nette passant par le point de fixation.
L'examen du fond d'oeil confirme le diagnostic en montrant un oedème rétinien ischémique de l'hémirétine supérieure avec
macula rouge cerise.

Dans une occlusion de la branche temporale supérieure de l'artère centrale de la rétine, l'amputation du champ
visuel est :
A - Nasale supérieure
B - Nasale inférieure
C - Temporale supérieure
D - Temporale inférieure
E - Totale
Bonne(s) réponse(s) : B

Le champ visuel nasal inférieur correspond à la rétine temporale supérieure.

Dans une occlusion de l'artère centrale de la rétine, l'examen du fond d'oeil montre :
A - Un oedème ischémique de la rétine
B - Un oedème maculaire cystoïde
C - Un oedème papillaire
D - Des hémorragies diffuses
E - Des hémorragies le long des vaisseaux
Bonne(s) réponse(s) : A

C'est l'occlusion de la veine centrale de la rétine qui peut se compliquer d'oedème papillaire, d'oedème maculaire cystoïde,
d'hémorragies diffuses et le long des vaisseaux.

Quels sont les examens complémentaires ophtamologiques qui peuvent avoir un intérêt diagnostique ?
A - Champ visuel Goldmann
B - Test de Lancaster
C - Echographie
D - Angiographie
E - Electrorétinographie
Bonne(s) réponse(s) : D

L'angiographie rétinienne en fluorescence permet de confirmer le diagnostic.

Quel ou quels examen(s) complémentaire(s) a (ont) un intérêt étiologique ?


A - Vitesse de sédimentation
B - Clearance de la créatinine
C - Calcémie
D - Phosphorémie
E - Groupage tissulaire H.L.A.
Bonne(s) réponse(s) : A

La vitesse de sédimentation, si elle est élevée, oriente vers une origine inflammatoire (maladie de Horton).

1098
Exclusivement sur DOC - DZ : www.doc-dz.com NADJI 85
RESIDANAT EN POCHE TOME II
Cas Clinique en QCM

Quel est ou quels sont les mécanismes pathogéniques des occlusions de l'artère centrale de la rétine ?
A - Thrombose
B - Hyperviscosité sanguine
C - Embol de cholestérol
D - Embol de calcaire
E - Embol de plaquettes
Bonne(s) réponse(s) : A B C D E

Tous ces mécanismes peuvent être impliqués, séparément ou en association, dans la survenue d'une occlusion de l'artère
centrale de la rétine.

Parmi ces thérapeutiques, toutes peuvent être utiles en urgence, sauf une, laquelle ?
A - Anticoagulants
B - Anti-agrégants plaquettaires
C - Vasodilatateurs
D - Macromolécules
E - Thrombolytiques
Bonne(s) réponse(s) : D

Question ambiguë, car seuls les anticoagulants et les corticoïdes ont un intérêt en urgence. Ni les vasodilatateurs, ni les
fibrinolytiques n'ont fait la preuve de leur efficacité.

Un homme de 51 ans, diabétique insulinodépendant depuis 10 ans mais mal équilibré est hospitalisé pour une diplopie
apparue brutalement depuis 2 jours accompagnée de douleurs orbitaires. En fait le patient arrive à voir simple dans le regard
vers la gauche. Sa diplopie débute à partir de la position primaire et le décalage s'accentue dans le regard vers la droite. Il se
présente d'ailleurs avec une position de la tête tournée vers la droite pour favoriser le regard vers la gauche. C'est le premier
épisode.
Il n'y a pas de céphalées, pas de fièvre, pas de déficit du champ visuel, pas de déficit-neurologique en dehors d'une limitation
manifeste d'abduction de l'oeil droit. La TA est chiffrée à 180/100 mm Hg.
On pense à une paralysie oculomotrice du nerf abducteur droit (VIème paire crânienne) complication du diabète.

Quel est le territoire d'innervation du nerf abducteur droit (VIème paire crânienne) ?
A - Le muscle droit externe droit et le muscle droit interne gauche
B - Les muscles droit externe, petit oblique, droit supérieur, droit inférieur de l'oeil droit
C - Le muscle droit interne droit
D - Le muscle droit externe droit
E - Le muscle droit externe droit et la motricité de la pupille
Bonne(s) réponse(s) : D

Par définition.

Parmi les autres mécanismes de diplopie mono ou binoculaire vous retenez :


A - Des opacités cristalliniennes
B - Une hémianopsie latérale homonyme droite
C - Une hétérophorie décompensée
D - Une occlusion de la veine centrale de la rétine
E - Une anisocorie
Bonne(s) réponse(s) : A C E

L'hétérophorie (insuffisance de convergence) décompensée peut entraîner une diplopie.

Quel(s) examen(s) réalise-t-on pour caractériser le déficit oculomoteur ?


A - Electrorétinogramme
B - Test de Lancaster
C - Etude des champs d'action des muscles oculomoteurs
D - Examen du champ visuel
E - Skiascopie
Bonne(s) réponse(s) : B C

Le test de Lancaster étudie le champ d'action des muscles oculo-moteurs.

1099
Exclusivement sur DOC - DZ : www.doc-dz.com NADJI 85
RESIDANAT EN POCHE TOME II
Cas Clinique en QCM

La paralysie oculomotrice dans le cadre du diabète peut être due à :


A - Une hémorragie du vitré
B - Un accident vasculaire cérébral
C - L'exophtalmie
D - Un déséquilibre du diabète
E - La neuropathie par microangiopathie
Bonne(s) réponse(s) : B E

Sans commentaire.

Citez une autre manifestation ophtalmologique du diabète ?


A - Glaucome chronique à angle ouvert
B - Glaucome par fermeture de l'angle
C - Microangiopathie rétinienne
D - Uvéite antérieure
E - Aucune de ces possibilités
Bonne(s) réponse(s) : C

La microangiopathie est la manifestation rétinienne du diabète.

Un homme de 35 ans, travailleur manuel, sans antécédent pathologique connu, remarque depuis quelques mois, une baisse
progressive de la vision de l'oeil droit, à 2/10 - 3/10.
Lorsqu'il consulte, l'examen, attentif et comparatif des yeux révèle une mydriase et une leucocorie droites, sans signes
inflammatoires de la chambre antérieure. L'existence d'une cataracte avec corps étranger métallique intra oculaire est
l'hypothèse diagnostique la plus probable.

Quel(s) signe(s) l'examen à la lampe à fente est-il susceptible d'objectiver pour confirmer la présence d'un corps
étranger métallique intra-oculaire ?
A - Rubéose irienne
B - Sécclusion pupillaire
C - Hétérochromie irienne
D - Taie cornéenne, séquelle d'une plaie perforante punctiforme
E - Pannus cornéen
Bonne(s) réponse(s) : C E

L'hétérochromie de l'iris peut traduire une surcharge en fer oxydé de l'iris. La séquelle de plaie perforante est très évocatrice
du diagnostic de corps étranger intra-oculaire.

Parmi les examens suivants, indiquez ceux que vous prescrivez pour étayer votre diagnostic étiologique :
A - Radiographie standard de l'orbite
B - Echographie oculaire
C - Bilan phospho-calcique
D - Examen du champ visuel
E - Examen de la vision colorée
Bonne(s) réponse(s) : A B

La radiographie et l'échographie peuvent permettre de visualiser le corps étranger. Le dosage du fer dans l'humeur acqueuse
et l'électrorétinogramme peuvent aussi aider à poser le diagnostic de sidérose oculaire.

La mydriase unilatérale vous paraît être la conséquence :


A - De la diminution de la vision
B - D'un blocage de sphincter de l'iris par des synéchies postérieures
C - De l'existence d'un corps étranger intra-oculaire métallique méconnu
D - D'une agénésie du sphincter de l'iris
E - D'une rupture contusive du sphincter de l'iris
Bonne(s) réponse(s) : C

La sidérose entraîne une mydriase aréflexique. A ce stade, l'ablation du corps étranger est souvent impuissante à stopper
l'évolution.

1100
Exclusivement sur DOC - DZ : www.doc-dz.com NADJI 85
RESIDANAT EN POCHE TOME II
Cas Clinique en QCM

Le pronostic sensoriel dépend :


A - Du niveau de l'acuité visuelle
B - De l'état du champ visuel
C - De la densité de la cataracte
D - Du degré d'altération de l'électrorétinogramme
E - De la durée de l'évolution
Bonne(s) réponse(s) : A D E

Le séjour prolongé d'un corps étranger dans l'oeil entraîne une sidérose, avec baisse d'acuité visuelle et altération de l'ERG
irréversible.

Quelle est l'attitude thérapeutique la plus appropriée ?


A - Abstention
B - Extraction de la cataracte
C - Extraction de la cataracte et du corps étranger intra-oculaire méconnu
D - Traitement endocrinien
E - Traitement anti-inflammatoire
Bonne(s) réponse(s) : C

L'acuité visuelle est encore non négligeable. Il est donc tout-à-fait justifié d'intervenir pour retirer la cataracte et le corps
étranger intra-oculaire.

Monsieur B..., âgé de 70 ans, atteint de bronchopathie chronique, et de cataracte de l'oeil droit, présente depuis 24 heures un
oeil gauche douloureux avec cornée trouble et mydriase, céphalées homolatérales et nausées.
Dans ses antécédents, on note des épisodes de brouillard visuel, à l'oeil droit aussi (avant que la vision ne soit très diminuée
par la cataracte).
Monsieur B... porte des lunettes depuis l'enfance avec une correction de + 4 dioptries à chaque oeil (il avait dit-il un strabisme
convergent qui a été guéri par le port de lunettes).
Le cristallin de l'oeil gauche paraît transparent, mais les détails du fond d'oeil ne sont pas visibles (cornée trouble).

De quelle affection ophtalmologique s'agit-il ?


A - Cataracte au début
B - Récidive de son strabisme
C - Oblitération de l'artère centrale de la rétine
D - Glaucome chronique à angle ouvert
E - Glaucome par fermeture de l'angle
Bonne(s) réponse(s) :

QUESTION ANNULEE

Quel(s) est (sont) le(s) élément(s) qui pouvai(ent) faire redouter, chez ce patient, la survenue d'une telle
affection ?
A - Valeur de la réfraction
B - Antécédents de strabisme guéri par lunettes
C - Episodes de brouillard visuel
D - Existence d'une cataracte à l'oeil droit
E - Aucun de ces éléments
Bonne(s) réponse(s) :

QUESTION ANNULEE

Quel(s) est (sont) le(s) élément(s) pouvant faire redouter la survenue de la même affection sur l'oeil droit ?
A - Valeur de la réfraction
B - L'âge du patient
C - L'existence de cette affection sur un des deux yeux
D - La bronchopathie chronique
E - Aucun de ces éléments, car l'affection est toujours strictement unilatérale
Bonne(s) réponse(s) :

QUESTION ANNULEE

1101
Exclusivement sur DOC - DZ : www.doc-dz.com NADJI 85
RESIDANAT EN POCHE TOME II
Cas Clinique en QCM

Quel(s) traitement(s) proposer chez un tel patient ?


A - Uniquement la surveillance de l'oeil gauche
B - La prise d'antalgiques mineurs, sans autre traitement
C - De l'acétazolamide per os
D - De l'acétazolamide en intraveineux
E - Un traitement par laser
Bonne(s) réponse(s) :

QUESTION ANNULEE

Quelle attitude thérapeutique préventive faut-il proposer pour l'oeil droit ?


A - Abstention
B - Collyre myotique
C - Collyre mydriatique
D - Iridectomie
E - Trabeculectomie
Bonne(s) réponse(s) :

QUESTION ANNULEE

1102
Exclusivement sur DOC - DZ : www.doc-dz.com NADJI 85
RESIDANAT EN POCHE TOME II
Cas Clinique en QCM

1103
Exclusivement sur DOC - DZ : www.doc-dz.com NADJI 85
RESIDANAT EN POCHE TOME II
Cas Clinique en QCM
Une femme de 72 ans est hospitalisée pour un ulcère de la jambe droite, évoluant depuis six mois. Dans ses antécédents, on
retrouve : une fracture de cette même jambe à l'âge de 40 ans, ayant nécessité une immobilisation plâtrée pendant trois mois,
un diabète depuis dix ans, traité par régime hypocalorique et un biguanide ; une hypertension artérielle, traitée par bêta-
bloquants.
La patiente a l'habitude de traiter son ulcère seule : elle le nettoie à l'éther et applique dessus un pansement commercialisé
imprégné de Néomycine®, de Polymyxine® et de Triamcinolone.
A l'examen, il s'agit d'une patiente en bon état général, pesant 70 kg pour 1,60 m. L'ulcère siège au-dessus de la malléole
interne droite, mesure 4 x 3 cm, ses bords sont réguliers, le fond est sale, sans tendance au bourgeonnement.
La peau périulcéreuse est rouge, suintante, et prurigineuse. Il existe quelques plaques de dermite ocre, et des dilatations
variqueuses. Les pouls pédieux et tibial postérieur droits ne sont pas perçus. La patiente ressent des douleurs vives au
moment des soins. Le reste de l'examen clinique ne révèle aucune anomalie. La tension artérielle est à 16/9, la température
est normale.
Parmi les examens biologiques usuels, les seules anomalies sont une augmentation de la créatininémie à 210
micromoles/litre et une glycémie à jeun à 10 millimoles/litre.

Cet ulcère de jambe est probablement d'étiologie mixte. Quels sont le(les) élément(s) en faveur d'une
participation veineuse ?
A - Le siège sus-malléolaire
B - L'aspect de l'ulcère
C - La taille de l'ulcère
D - L'existence d'un diabète
E - Les données de l'examen clinique des vaisseaux du membre
Bonne(s) réponse(s) : A E

Le siège sus-malléolaire interne oriente vers une origine veineuse, de même que les plaques de dermite ocre et les quelques
dilatations variqueuses.

Quels sont le(les) élément(s) en faveur d'une participation artérielle ?


A - Le siège sus-malléolaire
B - L'aspect de l'ulcère
C - Les données de l'examen clinique des vaisseaux du membre
D - Le caractère des douleurs
E - Les antécédents d'immobilisation
Bonne(s) réponse(s) : C

L'abolition des pouls tibiaux postérieurs et pédieux est un signe d'artérite.

Quels(s) examen(s) complémentaire(s) est(sont) utile(s) en première intention ?


A - Doppler artériel et veineux
B - Aortographie
C - Phlébographie
D - Thermographie
E - Hyperglycémie provoquée orale
Bonne(s) réponse(s) : A

Le doppler artério-veineux est un examen simple, non invasif, permettant de mettre en évidence les freins circulatoires
artériels et les incontinences ostioles.

Quel(les) mesure(s) peu(ven)t être utile(s) ?


A - Antibiotiques généraux
B - Bains antiseptiques
C - Corticothérapie générale
D - Détersion de l'ulcère
E - Vaccination antitétanique
Bonne(s) réponse(s) : B D E

En plus de la vaccination antitétanique, il faut mettre en route la première phase thérapeutique (détersion) qui sera suivie des
deux suivantes : culture de bourgeon charnu et épidermisation.

L'aspect des téguments autour de l'ulcère doit faire évoquer une complication. Laquelle ?
A - Erysipèle
B - Eczéma de contact
C - Phlébite
D - Impétigo
E - Etat pré-épithéliomateux
Bonne(s) réponse(s) : B

L'aspect est typiquement celui d'un eczéma périulcéreux qu'il faut assécher par une pâte à l'eau, puis dans un second temps,
en cas de non guérison, appliquer une corticothérapie locale, périulcéreuse, la plus prudente possible sur ce mauvais terrain.
1104
Exclusivement sur DOC - DZ : www.doc-dz.com NADJI 85
RESIDANAT EN POCHE TOME II
Cas Clinique en QCM
Monsieur B. agriculteur, vous amène son fils de 7 ans qui présente depuis 3 semaines une teigne. Il s'agit d'une lésion
ovalaire bien circonscrite, de 4 cm de diamètre siégeant sur le cuir chevelu postérieur, inflammatoire et tuméfiée, réalisant un
aspect de macaron surélevé par rapport au plan du cuir chevelu. A l'inspection, les orifices folliculaires sont dilatés et
purulents et il existe une chute des cheveux à ce niveau.

L'examen clinique doit vraisemblablement retrouver :


A - La présence d'adénopathies
B - L'existence d'un état fébrile
C - L'existence d'un prurit
D - L'existence d'une douleur spontanée et provoquée au niveau de la lésion
E - Aucune des propositions est exacte
Bonne(s) réponse(s) : E

L'aspect clinique est très évocateur de teignes inflammatoires = kérion du cuir chevelu. Il n'existe aucun signe général, aucune
douleur et aucune réaction ganglionnaire.

Quel(s) est(sont) l'(les) agent(s) étiologique(s) responsable(s) possible(s) de cette teigne ?


A - Microsporum canis
B - Trichophyton violacéum
C - Trichophyton schonleïni
D - Trichophyton ochraceum
E - Trichophyton mentagrophytes
Bonne(s) réponse(s) : D E

Sans commentaire.

Quel est(sont) le(les) examen(s) qui vous permettra(tront) d'apporter la certitude diagnostique ?
A - Examen du pus dans une goutte de bleu lactique
B - Culture pour recherche bactériologique
C - Culture sur milieu de Sabouraud
D - Examen en lumière de Wood
E - Toutes ces propositions sont exactes
Bonne(s) réponse(s) : A B C

L'examen direct des poils parasités montrerait :


- soit un parasitisme de type microïde = correspondant à trichophyton mentagrophyte
- soit un parasitisme de type mégaspore : trichophyton pchraceum.

Concernant cette teigne :


A - Il existe une fluorescence rouge en lumière de Wood
B - La suppuration est liée à la présence d'une abondante matière favique
C - La surinfection staphylococcique est très fréquente
D - La guérison s'accompagne d'une repousse de cheveux
E - La contamination est essentiellement interhumaine
Bonne(s) réponse(s) : C D

L'examen en lumière de Wood est négatif.


Le réservoir animal est pour T. mentagrophytes : les équidés, les souris blanches et les cobayes.
Ochraceum = bovidés (d'où contamination des éleveurs et des vétérinaires).

Quelle(s) conduite(s) thérapeutique(s) adoptez-vous ?


A - Grisefuline® 10 mg/kg/j pendant 2 semaines
B - Grisefuline® 10 mg/kg/j pendant 6 semaines
C - Pyostacine®150 mg/kg/j pendant 3 semaines
D - Eviction scolaire
E - Pévaryl® lotion, 2 applications par jour
Bonne(s) réponse(s) : B C E

La teigne est une maladie à déclaration obligatoire, justiciable d'éviction scolaire. Mais les teignes à dermatophytes zoophiles
n'ont pas de contagiosité interhumaine, les mesures d'éviction scolaire sont abusives.
Il importe de traiter l'animal responsable.

1105
Exclusivement sur DOC - DZ : www.doc-dz.com NADJI 85
RESIDANAT EN POCHE TOME II
Cas Clinique en QCM
Une infirmière présente à l'index de la main gauche un ongle épaissi, strié, friable sur sa moitié proximale, le bourrelet
périunguéal est oedematié, suintant, mais non suppuré.

Quel diagnostic peut-on évoquer ?


A - Onyxis à levures
B - Onyxis psoriasique
C - Onyxis dermatophytique
D - Panaris staphylococcique
E - Autre étiologie
Bonne(s) réponse(s) : A

Le périonyxis subaiguë inflammatoire évoque en premier lieu une étiologie candidosique.

Quel est le examen biologique à demander ?


A - Examen bactériologique d'un prélèvement
B - Examen mycologique avec culture
C - Dosage des protéines sériques
D - Hémogramme et vitesse de sédimentation
E - Aucun de ces examens.
Bonne(s) réponse(s) : B

Le prélèvement est indispensable pour confirmer l'origine fongique.

Quel(s) traitement(s) envisager ?


A - Meulage de l'ongle et traitement local spécifique
B - Traitement par voie générale pendant 3 mois
C - Traitement spécifique par voie générale pendant 6 mois
D - Traitement par voie générale pendant 3 mois accompagné d'un traitement local et du meulage de l'ongle
atteint
E - Aucun traitement, attendre la repousse d'un ongle sain
Bonne(s) réponse(s) : D

Le traitement général est le Nizoral® (kétoconazole).

Une infirmière présente à l'index de la main gauche un ongle épaissi, strié, friable sur sa moitié proximale, le bourrelet
périunguéal est oedematié, suintant, mais non suppuré.

Quelle(s) prophylaxie(s) conseillez-vous ?


A - Port de gants de protection
B - Faire un traitement local à titre préventif tous les mois
C - Proscrire le vernis à ongles
D - Couper les ongles très court
E - Faire un traitement par voie générale à titre prophylactique 2 fois/an
Bonne(s) réponse(s) : A

Le port de gant de protection est impératif afin de protéger l'index des détergents et savons.

Un malien de 25 ans, en France depuis 5 ans, consulte pour une uréthrite persistante. L'interrogatoire vous apprend qu'il a vu
un premier médecin 10 jours plus tôt pour un écoulement uréthral, jaune, épais, tachant le linge, apparu 3 à 4 jours après un
rapport occasionnel à Paris. Il n'y a pas eu de prélèvement uréthral fait à cette occasion, mais seulement une sérologie
syphilitique qui reviendra avec les résultats suivants : Kline 1/1-TPHA 1/80. Son médecin lui a fait une injection
intramusculaire unique. Cette injection a paru améliorer les signes, mais la persistance de l'écoulement justifie une nouvelle
consultation. A l'examen, la pression du méat visualise une goutte de sérosité un peu louche. Il n'y a pas d'érosion génitale.
L'examen des aires ganglionnaires inguinales montre des ganglions de petite taille non inflammatoires.

Quelle est votre première hypothèse diagnostique concernant la symptomatologie ?


A - Uréthrite syphilitique
B - Uréthrite à chlamydia
C - Uréthrite à gonocoques
D - Uréthrite candidosique
E - Uréthrite à trichomonas
Bonne(s) réponse(s) : B

Le premier épisode évoquait une uréthrite gonococcique améliorée par le traitement minute.
La persistante d'un écoulement doit évoquer une infection mixte, en premier lieu :
gonoccoque - chlamydia.

1106
Exclusivement sur DOC - DZ : www.doc-dz.com NADJI 85
RESIDANAT EN POCHE TOME II
Cas Clinique en QCM

Tous les examens ci-dessous peuvent apparaître justifiés dans le cas présent, sauf un. Lequel ?
A - Appréciation quantitative du nombre des leucocytes à l'examen direct de l'écoulement
B - Examen cytobactériologique des urines
C - Examen de l'écoulement à l'ultramicroscope à fond noir
D - Culture de l'écoulement sur milieu de Sabouraud
E - Examen direct de l'écoulement après coloration au bleu de méthylène
Bonne(s) réponse(s) : C

L'ultramicroscopie à fond noir met en évidence des tréponèmes. L'uréthrite syphilitique est très rare.

Certaine(s) des interprétations ci-dessous de la sérologie-tréponémique de votre patient sont possibles.


Laquelle(lesquelles) ?
A - Syphilis congénitale méconnue
B - Syphilis secondaire évolutive
C - Fausse sérologie syphilitique
D - Séquelle sérologique d'une tréponématose endémique
E - Syphilis ancienne
Bonne(s) réponse(s) : A D E

Cette formule est celle d'une tréponématose ancienne (syphilitique ou endémique). Les réactions non tréponémiques
(cardiolipidiques) tendent à se négativer. Les tests tréponémique persistent.

En attendant les résultats des examens complémentaires que vous avez demandés, quel traitement allez-vous
lui proposer ?
A - Soins locaux
B - Une injection unique d'Extencilline® : 2,4 millions d'unités
C - Un traitement de 10 jours par le Bactrim® : 4 comprimés par jour
D - Un traitement de 8 jours par le Totapen® : 2 g par jour
E - Un traitement de 10 jours par la Vibramycine® : 200 mg par jour
Bonne(s) réponse(s) : E

Les tétracyclines constituent le traitement de choix d'une infection à chlamydia. Les pénicillines sont inactives sur les
chlamydia.

Une jeune fille de 20 ans, sans antécédents particuliers, présente depuis 6 mois un eczéma localisé au lobe des oreilles, à la
face antérieure du poignet gauche, et à l'abdomen. Il n'y a pas d'antécédent familial allergique.

L'étiologie la plus probable est :


A - Eczéma atopique
B - Eczéma de contact au chrome
C - Eczéma de contact au nickel
D - Eczéma de contact au caoutchouc
E - Mycose
Bonne(s) réponse(s) : C

La localisation est typique d'une allergie aux bijoux de fantaisie contenant du nickel auquel sont allergiques 10% environ des
femmes occidentales.

L'examen complémentaire le mieux approprié pour confirmer le diagnostic est :


A - La biopsie cutanée
B - Les patch-tests
C - Dosage des IgE totales
D - Numération-formule sanguine
E - Un test de transformation lymphoblasique
Bonne(s) réponse(s) : B

Le patch-test est un examen simple et peu coûteux.

1107
Exclusivement sur DOC - DZ : www.doc-dz.com NADJI 85
RESIDANAT EN POCHE TOME II
Cas Clinique en QCM

Quelle(s) mesure(s) thérapeutique(s) proposez-vous ?


A - Corticothérapie générale
B - Corticothérapie locale
C - Eviction de l'allergène suspecté
D - Désensibilisation spécifique
E - Application d'un topique réducteur
Bonne(s) réponse(s) : B C

L'éviction de l'allergène est impératif, quand il est possible.


La désensibilisation des eczémas de contact n'existe pas.

Le mécanisme en cause est :


A - Une allergie réaginique de type I
B - Une maladie par immun complexe
C - Un phénomène d'hypersensibilité retardée de type IV
D - Une action toxique directe
E - Un mécanisme photo-allergique
Bonne(s) réponse(s) : C

L'eczéma de contact est une hypersensibilité retardée à médiation cellulaire (type IV de Gell et Coombs).
2 étapes sont envisagées :
l) l'étape de sensibilisation, correspondant à la migration des cellules sensibilisées de l'épiderme au relais ganglionnaire
2) l'étape de révélation avec migration des cellules réactives vis-à-vis de l'antigène, du ganglion lymphatique à l'épiderme.

Les cellules impliquées dans ce phénomène sont :


A - Les polynucléaires neutrophiles
B - Les mastocytes
C - Les Iymphocytes T
D - Les Iymphocytes B
E - Les mélanocytes
Bonne(s) réponse(s) : C

Il faut y ajouter la cellule de Langerhans qui présente l'antigène au lymphocyte immunocompétent et produit l'interleukine 1.

Une malade de 78 ans, aux antécédents d'hypertension artérielle et de diabète, ayant été traitée il y a deux mois pour une
pneumopathie bactérienne, présente depuis 15 jours une éruption cutanée prurigineuse, polymorphe associant les lésions
suivantes : bulles tendues reposant sur une base érythémateuse, placards pseudo-urticariens, érosions post-bulleuses. Les
lésions sont généralisées mais il n'y a pas d'atteinte muqueuse. Le frottement appuyé du pouce sur la peau cliniquement
saine de la patiente n'induit pas de décollement. L'état général est altéré. La numération-formule sanguine montre une
hyperéosinophilie. Vous portez le diagnostic de pemphigoïde bulleuse.

Parmi les éléments cliniques suivants, quel(s) est (sont) celui (ceux) qui évoque(nt) le diagnostic de
pemphigoïde bulleuse :
A - Antécédents de diabète
B - Episode récent d'infection pulmonaire
C - Caractère polymorphe de l'éruption
D - Existence d'une altération de l'état général
E - Existence de bulles tendues
Bonne(s) réponse(s) : C E

Le prurit, le polymorphisme des lésions, la solidité des bulles et l'absence (ou la rareté) des lésions muqueuses sont de bons
éléments en faveur de la pemphigoïde bulleuse.

Vous avez pratiqué une biopsie de peau pour examen anatomo-pathologique. Parmi les réponses suivantes,
quelle est celle qui confirme votre diagnostic clinique :
A - Bulle intraépidermique superficielle (sous la couche cornée)
B - Bulle intraépidermique profonde
C - Bulle à la jonction dermo-épidermique
D - Bulle intradermique superficielle
E - Bulle intradermique profonde
Bonne(s) réponse(s) : C

Evident (voir cours).

1108
Exclusivement sur DOC - DZ : www.doc-dz.com NADJI 85
RESIDANAT EN POCHE TOME II
Cas Clinique en QCM

Quels sont les deux examens que vous allez demander pour confirmer votre diagnostic :
A - Ponction du liquide de bulle
B - Examen en immunofluorescence d'une biopsie cutanée
C - Recherche d'anticorps circulants antiépiderme
D - Groupage HLA
E - Biopsie jéjunale
Bonne(s) réponse(s) : B C

En IF : les dépôts sur la membrane basale ont un aspect homogène et linéaire. Il s'agit le plus souvent d'IgG et de C3.
Dans le pemphigus, les dépôts d'anticorps, antisubstance intercellulaire, sont en maille dans l'épiderme. Ceux-ci sont présents
dans le sérum, leurs taux sont un indicateur pronostique.

Parmi les résultats biologiques suivants, lequel(s) est (sont) évocateur(s) du diagnostic de pemphigoïde
bulleuse :
A - Présence d'un staphylocoque doré
B - Présence d'un dépôt linéaire d'immunoglobulines G et de complément à la jonction dermo-épidermique
C - Présence d'une hyperéosinophilie sanguine
D - Groupe HLA B8
E - Présence d'un anticorps circulant dirigé contre la susbstance intercellulaire de l'épiderme
Bonne(s) réponse(s) : B C

Voir question précédente; l'éssinophilie est très fréquente au cours de la pemphigoïde bulleuse.
HLA B8 est retrouvé chez 77 à 87% des malades atteints de dermatite herpétiforme.

Dans une école, une épidémie de phtiriase du cuir chevelu a été dépistée. Un des enfants de cette collectivité est trouvé
porteur dans le cuir chevelu de lésions croûteuses, jaunâtres, qui, aux dires de sa mère, se sont étendues rapidement. Il a des
adénopathies cervicales postérieures.

Quel diagnostic convient-il de soulever ?


A - Zona
B - Folliculite
C - Varicelle
D - Impétiginisation
E - Teigne suppurée
Bonne(s) réponse(s) : D

Il est extrêmement commun que les phytiriases du cuir chevelu se compliquent d'impétiginisation au niveau des excoriations
de grattage, siégeant surtout dans la région occipitale.

Au cours des infections cutanées, les deux germes bactériens le plus souvent en cause sont :
A - Staphylocoque doré
B - Streptocoque
C - Entérocoque
D - Colibacilles
E - Staphylocoque blanc
Bonne(s) réponse(s) : A B

Evident.

En absence de traitement, citer la complication à rechercher en priorité .


A - Endocardite
B - Septicémie
C - Hépatite
D - Glomérulonéphrite
E - Ostéite
Bonne(s) réponse(s) : D

Les impétigos se compliqueraient dans 5% des cas de glomérulonéphnite aiguë.

1109
Exclusivement sur DOC - DZ : www.doc-dz.com NADJI 85
RESIDANAT EN POCHE TOME II
Cas Clinique en QCM

Quelle(s) mesure(s) thérapeutique(s) est (sont) nécessaire(s) ?


A - Lotion aux pyréthrines
B - Corticothérapie locale
C - Antibiothérapie générale
D - Antihistaminiques per os
E - Soins locaux antiseptiques.
Bonne(s) réponse(s) : A C E

Il faut associer un traitement antiparasitaire à un traitement antibactérien (antisepsie et antibiothérapie).

Une femme de 25 ans consulte pour une éruption apparue 48 h auparavant composée de macules érythémateuses rose pâle,
ovalaires bien limitées, séparées par des intervalles de peau saine. Les macules prédominent au tronc, respectent la face, et
ne sont pas prurigineuses. L'examen général retrouve des microadénopathies occipitales, cervicales, épitrochléennes et des
plaques fauchées muqueuses.
L'interrogatoire note l'absence d'altération de l'état général, l'absence de fièvre, un retard de régles de 1 mois et demi, un
traitement par Tranxène®, l'existence d'un terrain atopique avec allergie aux bêta lactamines ; enfin, la notion d'un rapport
extra-conjugal cinq semaines auparavant.
Vous portez le diagnostic clinique de roséole syphilitique chez une femme enceinte.

Le (s) élément(s) qui fait (font) partie de la description clinique d'un érythème roséoliforme est (sont) :
A - Macules rose pâle
B - Aspect ovalaire mal limité
C - Intervalles de peau saine
D - Absence de prurit
E - Respect du visage
Bonne(s) réponse(s) : A C

Une éruption roséoliforme est une éruption de macules rondes ou ovulaires de taille variable de teinte rose pâle ne faisant
aucune saillie et ménageant des intervalles de peau saine.

Sur la base de l'examen clinique, quels sont les deux diagnostics différentiels principaux qui doivent être
éliminés devant cette éruption roséoliforme ?
A - Scarlatine
B - Toxidermie médicamenteuse
C - Rubéole
D - Lucite
E - Eczéma atopique
Bonne(s) réponse(s) : B C

Il faut de principe devant toute éruption suspecte chez une femme enceinte, envisager une rubéole et pratiquer une sérologie.

Le(s) élément(s) en faveur d'une origine syphilitique est(sont) :


A - Les plaques muqueuses
B - La présence d'intervalles de peau saine
C - Les microadénopathies
D - Un retard des règles
E - Absence de prurit
Bonne(s) réponse(s) : A C E

Toute roséole doit faire systématiquement rechercher :


- une cicatrice de chancre et son adénopathie satellite
- des microadénopathies, évocatrices si occipitales ou épitrochléennes
- et surtout les plaques muqueuses = fourmillant de tréponèmes, très contagieuses.
L'absence de prurit est un argument pour l'origine syphilitique et contre l'origine médicamenteuse.

Le(s) mode(s) évolutifs de cette roséole après traitement est (sont) :


A - Desquamation importante
B - Evolution contusiforme
C - Disparition rapide de l'érythème après traitement efficace
D - Persistance de l'érythème pendant quelques semaines en l'absence de traitement
E - Possibilité de séquelles dyschromiques
Bonne(s) réponse(s) : C D E

Les séquelles dyschromiques sont la leucomélanodermie du cou (collier de Vénus) = taches blanches arrondies,
irrégulièrement réparties sur un fond bistre.

1110
Exclusivement sur DOC - DZ : www.doc-dz.com NADJI 85
RESIDANAT EN POCHE TOME II
Cas Clinique en QCM

Chez cette patiente, quels sont les deux examens les plus utiles que vous demanderez ?
A - Sérologie de la rubéole
B - Sérologie de la syphilis
C - Test de transformation lymphocytaire (TTL) au Tranxène®
D - Dosage des IgE globales
E - Hémocultures
Bonne(s) réponse(s) : A B

Les deux sérologies s'imposent devant une éruption roséoliforme de la femme enceinte.

Dans les semaines suivantes, en l'absence de traitement, on peut observer :


A - Un tabes
B - Une alopécie
C - Une glomérulonéphrite aiguë
D - Des papules infiltrées
E - Un signe d'Argyll Robertson
Bonne(s) réponse(s) : B C D

Le tabès et le signe d'Argyll Robertson font partie de la syphilis neurologique tertiaire.

Entre le 3e et le 6e mois après le chancre peuvent survenir deux types d'alopécie :


1) alopécie en clairière : aires alopéciques de la région temporo-occipitale
2) alopécie diffuse : plus rarement rencontrée
. Les papules inflitrées correspondent aux syphilides papuleuses de la deuxièmes floraison de la syphilis secondaire.

Quelle(s) mesure(s) thérapeutique(s) peut-on proposer à cette patiente allergique aux béta lactamines ?
A - Pénicilline sous couvert d'adrénaline
B - Dose progressive de pénicilline
C - Pénicilline après vérification de l'allergie par intradermoréaction à la pénicilline
D - Macrolides
E - Aminosides
Bonne(s) réponse(s) : D

La pénicilline est fortement contre-indiquée, les aminosides inefficaces. La seule solution thérapeutique : les macrolides.

Une femme de 32 ans présente depuis 4 ans et demi des lésions de psoriasis de 6 cm2 chacune environ : une au niveau de
chacun des coudes et chacun des genoux, une dans la région sacrée. S'y associent des lésions des ongles et du cuir chevelu,
une atteinte du pli interfessier.

Ces lésions, ayant l'aspect typique des lésions psoriasiques, sont :


A - Mal limitées
B - A contour arrondis
C - Erythémateuses
D - Croûteuses
E - Non suintantes
Bonne(s) réponse(s) : B C E

La plaque psoriasique est typiquement une plaque arrondie à bord net, érythémateuse, rouge vif, surélevée et squameuse.

Ces lésions dermatologiques sont nettement en faveur du psoriasis car :


A - Elles siègent aux faces antérieures des genoux
B - Elles siègent aux faces antérieures des coudes
C - Elles sont source de plaques alopéciques
D - Elles s'accompagnent de ponctuations unguéales
E - Elles s'accompagnent d'intertrigo interfessier
Bonne(s) réponse(s) : A D E

Les localisations les plus typiques sont les genoux, les coudes et la région sacrée.
Les ongles doivent être systématiquement inspectés dans l'examen d'un psoriasis.
L'atteinte du pli interfessier fait partie du psoriasis inversé.

1111
Exclusivement sur DOC - DZ : www.doc-dz.com NADJI 85
RESIDANAT EN POCHE TOME II
Cas Clinique en QCM

Comme chez tout psoriasique, on peut redouter la survenue chez cette patiente :
A - D'un syndrome de Kaposi-Juliusberg
B - D'un rhumatisme central
C - D'un psoriasis vésiculeux
D - D'une érythrodermie
E - De surinfections bactériennes
Bonne(s) réponse(s) : B D

Le syndrome de Kaposi-Juliusberg est la surinfection par le virus de l'herpès ou de la vaccine d'une dermatite atopique. Les 3
grandes complications du psoriasis sont le psoriasis pustuleux, l'érythodermike et le rhumatisme psoriasique (central ou
périphérique)

Laquelle de ces thérapeutiques vous paraît indiquée chez cette patiente ?


A - Photochimiothérapie
B - Corticothérapie générale
C - Vaseline salicylée locale
D - Etrétinate (Tigason®)
E - Méthotrexate per os
Bonne(s) réponse(s) : C

La vaseline salicylée locale suffit à traiter ces quelques plaques de psoriasis. Les traitements généraux sont inutiles. La
corticothérapie générale est interdite ; c'est à son arrêt que sont observées les complications graves du psoriasis :
érythrodermie et psoriasis pustuleux.

Une femme de 30 ans consulte en raison de l'apparition récente de lésions inflammatoires érythémateuses et chaudes de 3 à
4 cm de diamètre, au nombre de 5 éléments situés sur les 2 jambes. Ces éléments sont fermes et profonds et leur palpation
est douloureuse. La température est à 38 degrés C. La patiente se plaint d'arthralgies diffuses et d'asthénie. L'interrogatoire
retrouve la notion d'une diarrhée et de douleurs abdominales récentes. L'hémogramme est normal. L'intradermo-réaction à la
tuberculine est positive. La VS est à 25 mm à la 1ère heure. Les ASLO sont à 100 unités. La radiographie thoracique est
normale. Trois hémocultures sont stériles. Vous portez le diagnostic d'érythème noueux.

Quel(s) est(sont) le(s) élément(s) qui font partie du tableau d'érythème noueux ?
A - Eléments profonds et fermes
B - Caractère inflammatoire
C - Douleurs abdominales
D - Localisation aux 2 jambes
E - Association d'une fièvre à 38 degrés et d'arthralgies
Bonne(s) réponse(s) : A B D E

Les nodules d'érythème noueux siègent symétriquement aux membres inférieurs. Ils sont peu nombreux de 3 à 10. Ils sont
chauds, dououreux, hypodermiques. L'évolution est caractéristique vers la régression (en passant par les différents stades de
la biligénie)

Quel(s) mode(s) évolutif(s) peut(vent) prendre cet érythème noueux ?


A - Suppuration
B - Cicatrice résiduelle
C - Evolution contusiforme
D - Guérison spontanée après quelques poussées
E - Persistance des lésions inchangées pendant plusieurs mois
Bonne(s) réponse(s) : C D

Voir réponse précédente.

Devant ce tableau clinique présenté par cette patiente, quel(s) est(sont) le(s) diagnostic(s) étiologique(s) à
envisager en priorité ?
A - Maladie de Crohn
B - Recto-colite hémorragique
C - Sarcoïdose
D - Tuberculose
E - Tumeur carcinoïde du grêle
Bonne(s) réponse(s) : A B

Les entéropathies inflammatoires doivent être recerchées devant tout érythème noueux.

1112
Exclusivement sur DOC - DZ : www.doc-dz.com NADJI 85
RESIDANAT EN POCHE TOME II
Cas Clinique en QCM

Certain(s) élément(s) du tableau clinique est (sont) en faveur d'une yersiniose, lequel(lesquels) ?
A - Douleurs abdominales
B - Diarrhée
C - Intradermo-réaction à la tuberculose positive
D - Survenue chez une femme de 30 ans
E - Bilatéralité des lésions
Bonne(s) réponse(s) : A B

Les autres manifestations à rechercher sont les adénopathies cervicales sous-angulo-maxilaires et les polyarthrites aiguës.

Quel(s) examen(s) permet(tent) d'affirmer chez cette patiente le diagnostic de yersiniose ?


A - Intradermo-réaction à la tuberculose
B - Sérologie des yersinioses
C - Coproculture
D - Biopsie d'un élément d'érythème noueux
E - Mise en culture d'une biopsie d'un élément d'érythème noueux
Bonne(s) réponse(s) : B C

Positivité au ascension des anticorps à 3 semaines d'invervalle.

Quel(s) traitement(s) proposez-vous à cette patiente ?


A - Repos au lit
B - Antibiothérapie anti-yersinia
C - Anticoagulants
D - Corticothérapie
E - Thérapeutiques immunostimulantes
Bonne(s) réponse(s) : A B

Une antibiothérapie (monothérapie) de 15 jours à 3 semaines est indiquée.


On utilisera : aminosides, phénicolés, cyclines, cotrimoxazole, quinolones...

Un adulte jeune vous consulte pour un écoulement uréthral apparu la veille. Dans ses antécédents, on retrouve une syphilis
primaire traitée deux ans auparavant par une injection unique de 2,4 millions de benzathine pénicilline (Extencilline®) intra-
musculaire. Depuis, il n'y a pas eu de surveillance sérologique. Le patient raconte un rapport 3 jours auparavant avec une
amie rencontrée dans un bar. Il existe un prurit méatique et des brûlures intenses à la miction. L'écoulement uréthral est
abondant, jaune-verdâtre bien lié.

Devant une uréthrite antérieure aiguë, le germe le plus souvent rencontré est neisseria gonorrhoease, mais il
est possible d'incriminer également :
A - Hemophilus ducreyi
B - Chlamydia trachomatis
C - Chlarnydia psittaci
D - Trichomonas vaginalis
E - Neisseria meningitidis
Bonne(s) réponse(s) : B D

Chlamydia trachomatis serait responsable de 30 à 50 % des uréthrites non gonococciques.

En dehors d'une atteinte uréthrale, vous devez rechercher, également chez cet adulte, une atteinte primitive :
A - Pharyngée
B - Oculaire
C - Pulmonaire
D - Nasale
E - Anale
Bonne(s) réponse(s) : A E

La gonococcie pharyngée est en général asymptomatique et de guérison spontanée en 10 à 12 semaines.

1113
Exclusivement sur DOC - DZ : www.doc-dz.com NADJI 85
RESIDANAT EN POCHE TOME II
Cas Clinique en QCM

Devant cette uréthrite aiguë, vous devez rechercher systématiquement une (plusieurs) complication(s) :
A - Prostatique
B - Epididymaire
C - Pyélo-urétérale
D - Articulaire
E - Osseuse
Bonne(s) réponse(s) : A B D

Les complications loco-régionales sont la prostatite, l'épididymite (plus fréquentes toutefois avec chlamydia trachomatis). Les
septicémies à gonoccoque peuvent provoquer une polyarthrite, accompagnée de ténosynovites et de lésions
maculopapuleuses à centre nécrotique ou pustuleux.

Neisseria gonorrhoeae est un germe :


A - Diplocoque
B - Gram +
C - Intracellulaire
D - Extracellulaire
E - Immobile
Bonne(s) réponse(s) : A C D E

Sans commentaire.

Chez la partenaire de notre patient vous devez retenir en faveur d'une atteinte uro-génitale basse gonococcique
:
A - Des pertes blanches adhérentes, peu abondantes et prurigineuses
B - Des pertes jaunes-verdâtres abondantes et mousseuses
C - Des ulcérations vulvo-vaginales multiples
D - Une exocervicite muco-purulente
E - Des micro-adénopathies
Bonne(s) réponse(s) : B D

L'infection chez la femme peut revêtir plusieurs formes : cervicite, uréthrite, skénite bartholinite, salpingite.

Dans le cas d'une uréthrite gonococcique, parmi les protocoles thérapeutiques suivants, le(s)quel(s) est (sont)
habituellement efficace(s) :
A - Spectinomycine (Trobicine®), 2 g en une seule injection IM
B - Ampicilline, 3,5 g per os plus 1 g Benemide®, en une seule prise
C - Métronidazol (Flagyl®), 2 g en une seule prise, per os
D - Erythromycine, 2 g en une seule injection IM
E - Doxycycline, 200 mg per os en une seule prise
Bonne(s) réponse(s) : A B

On peut ajouter à ces deux traitements :


(1) procaïne-pénicilline 4,8 M U1 IM + probénédice 1g per os ou
(2) amoxicilline 3g + probénicide 1g per os.

Un homme de 42 ans consulte pour un psoriasis évoluant depuis 6 mois. Le début a été brutal succédant à un accident de la
voie publique. L'extension a été progressive avec une rémission durant les vacances au bord de la mer. A l'examen, il existe
des plaques érythémato-squameuses bien limitées, épaisses, des coudes, genoux, plaque sacrée, cuisses et avant-bras. Sur
le tronc, les lésions sont émiettées, asymétriques. Des nappes érythémateuses rouge vif sont notées dans les plis inguinaux
et des plaques squameuses au cuir chevelu. Les ongles sont épaissis et ponctués de dépressions et il existe une
kératodermie palmo-plantaire. Son fils est également atteint de psoriasis.

Dans l'histoire et l'examen clinique quel(s) est(sont) le(les) élément(s) en faveur du diagnostic :
A - Début après un accident
B - La régression l'été
C - L'asymétrie des lésions du tronc
D - L'antécédent familial
E - Le début à 40 ans
Bonne(s) réponse(s) : A B D

Un stress est retrouvé dans 50 % des cas, au cours du mois précédant la première poussée de psoriasis. La régression
estivale témoigne de l'effet bénéfique des UVA. Des antécédents familiaux de psoriasis sont retrouvés 1 fois sur 3.

1114
Exclusivement sur DOC - DZ : www.doc-dz.com NADJI 85
RESIDANAT EN POCHE TOME II
Cas Clinique en QCM

Quel sont le(les) aspect(s) clinique(s) compatible(s) avec le diagnostic de psoriasis :


A - Nappe érythémateuse rouge vif des plis
B - Le respect du visage
C - Ongles ponctués de dépression
D - L'alopécie des golfes temporaux
E - La kératodermie plantaire
Bonne(s) réponse(s) : A B C E

"A" témoigne d'un psoriasis inversé. Le psoriasis respecte le visage, en revanche l'oreille est un siège fréquent de psoriasis
(replis du pavillon, conduit auditif externe). Le psoriasis atteint le cuir chevelu, sans atteindre le cheveu (pas d'alopécie). La
kératodermie palmoplantaire est sèche et fissuraire.

Quel examen biologique peut être utile pour le diagnostic ?


A - VS
B - NFS
C - Uricémie
D - Electrophorèse des protides
E - Aucun des examens précédents
Bonne(s) réponse(s) : E

Le diagnostic de psoriasis est fait cliniquement. Une biopsie cutanée peut être utile dans certaines formes.
On a décrit des associations au diabète, à l'hyperuricémie, à l'hypertriglycéridémie.

L'amélioration des lésions l'été est un élément favorable pour la mise à un traitement. Lequel ?
A - Caryolysine® locale
B - PUVA
C - Méthotrexate
D - Corticothérapie locale
E - Dioxyanthranol
Bonne(s) réponse(s) : B

Le soleil réalise une UVA-thérapie naturelle.

Un homme de 28 ans consulte pour une ulcération du sillon balano-préputial. L'interrogatoire apprend que cet homme est
homosexuel, à partenaires multiples. Il a eu deux ans auparavant une syphilis qui a été traitée par pénicilline, la sérologie
syphilitique s'est négativée à la suite de ce traitement, d'après le patient. Il a présenté à plusieurs reprises des uréthrites
aiguës gonococciques, toujours soignées avec succès par antibiotiques. Il n'a pas d'autre antécédent notable. La lésion
actuelle a débuté huit jours avant la consultation et s'étend malgré l'application d'antibiotiques locaux. Il s'agit d'une petite
ulcération arrondie, rose, indolore, dure à la palpation. On note la présence de plusieurs ganglions inguinaux non
inflammatoires.

Quel est le diagnostic le plus probable ?


A - Syphilis
B - Chancre mou
C - Herpès
D - Gonococcie
E - Aphte
Bonne(s) réponse(s) : A

Le caractère induré, indolore du chancre, les adénopathies non inflammatoires sont très évocateurs de syphilis.

L'examen au microscope à fond noir ne retrouve pas de tréponème et la recherche de bacille de Ducreyi est
négative. Quelle conclusion pouvez-vous en tirer ?
A - Il ne s'agit ni d'une syphilis, ni d'un chancre mou
B - Ces examens peuvent être faussement négatifs
C - Il est inutile de faire des prélèvements directs sur les muqueuses génitales
D - En cas de chancre syphilitique, le tréponème ne peut être isolé qu'après huit jours d'évolution
E - Aucune des propositions ci-dessus n'est exacte
Bonne(s) réponse(s) : B

L'absence d'isolement de tréponèmes au microscope à fond noir n'a aucune signification, car c'est un examen difficile, à
répéter 3 jours de suite. La sérologie ne commence à se positiver qu'au delà du 10e jour après le chancre.

1115
Exclusivement sur DOC - DZ : www.doc-dz.com NADJI 85
RESIDANAT EN POCHE TOME II
Cas Clinique en QCM

Vous demandez une sérologie syphilitique. Elle donne les résultats suivants :
- TPHA positif au 1 /640e
- VDRL positif, titre 16
Quelle(s) conclusion(s) pouvez-vous en tirer ?
A - Il peut s'agir de la séquelle de sa syphilis antérieure
B - Il faut faire un test de Nelson qui seul affirmera la syphilis
C - Il peut s'agir d'une syphilis récente
D - Il faut refaire une sérologie dans 15 jours, afin de voir si les taux se modifient
E - Aucune des propositions ci-dessus n'est exacte
Bonne(s) réponse(s) : A C D

Le test de Nelson est inutile, le TPHA est positif affirmant la tréponématose. Il importe de pratiquer un VDRL quantitatif, dont
on surveillera la décroissance ou la négativation (si traitement précoce avant 6 semaines). Ce test n'a néanmoins pas de
valeur absolue, il existe des négativations spontanées du VDRL.

Parmi les éléments rapportés dans l'observation, vous retenez en faveur du diagnostic de chancre mou :
A - Homosexualité
B - Antécédent de syphilis correctement traitée
C - Ulcération indolore
D - Présence d'adénopathies non inflammatoires
E - Aucune des propositions ci-dessus
Bonne(s) réponse(s) : E

L'homosexualité est un facteur de risque pour toutes les MST.

Dans l'observation de ce patient, existe(nt)t-il(s) un (des) élément(s) en faveur du diagnostic de syphilis primaire
?
A - Homosexualité
B - Antécédent de syphilis correctement traitée
C - Ulcération indolore
D - Présence d'adénopathies non inflammatoires
E - Aucune des propositions ci-dessus `
Bonne(s) réponse(s) : C D

Le chancre syphilitique est indolore. Les adénopathies ne sont pas inflammatoires.

Un homme de 24 ans consulte pour la première fois pour une dermatose érythémato-squameuse, non prurigineuse, faite de
plaques bien limitées de 5 à 20 cm2 de surface, rouges, recouvertes de squames blanches qui se détachent facilement.
L'interrogatoire apprend que l'éruption a commencé plusieurs mois ou années auparavant mais qu'elle se limitait à des
plaques de même type au niveau des faces d'extension des membres. Le patient n'avait appliqué jusqu'alors que des
topiques locaux obtenus sans ordonnance et dont il ne se rappele plus le nom. Les résultats étaient d'ailleurs nuls ou
médiocres. La poussée actuelle paraît avoir été secondaire à une situation professionnelle récemment conflictuelle.
Le reste de l'examen clinique cutané retrouve un état desquamatif du cuir chevelu et une hyperkératose distale sous-unguéale
au niveau de quelques doigts. L'examen somatique est par ailleurs normal. Il ne semble pas y avoir de signes articulaires.
L'interrogatoire ne retrouve pas d'antécédent familial de même type. L'état général est bien conservé. Le patient a eu 3
poussées d'aphtes buccaux au cours de l'année écoulée. Il a été opéré d'une fissure anale il y a 2 ans. Sa mère est
asthmatique.

Quel(s) examen(s) complémentaire(s) confirme(nt) le diagnostic ?


A - Biopsie cutanée
B - Radio de thorax
C - Recherche d'anticorps anti-épiderme
D - Cytodiagnostic
E - Immunofluorescence directe
Bonne(s) réponse(s) : A

Le diagnostic clinique de psoriasis est évident, rendant inutile tout examen complémentaire diagnostique. Une biopsie cutanée
montrerait : Uune hyperkératose parakératosique, une hyperacanthose, une couche granuleuse absente, des microabcès de
Munro-Sabouraud et des pustules spongiformes multiloculaires de Kogoj-Lapière.

1116
Exclusivement sur DOC - DZ : www.doc-dz.com NADJI 85
RESIDANAT EN POCHE TOME II
Cas Clinique en QCM

Quel(s) est (sont) le(s) élément(s) qui, dans l'observation, est (sont) en faveur du diagnostic ?
A - Aphtes buccaux
B - Hyperkératose sous-unguéale
C - Mère asthmatique
D - Desquamation du cuir chevelu
E - Plaques aux faces d'extension des membres
Bonne(s) réponse(s) : B D E

Les lésions buccales du psoriasis sont la langue scrotale et la glossite exfoliatrice marginée (ou langue géographique). Les
aphtes n'appartiennent pas au psoriasis. Les atteintes unguéales sont : dépression en "dé à coudre", onycholyse,
hyperkératose sous-unguéale avec pachyonychle. Le psoriasis respecte le cheveu. L'atopie n'est pas associée au psoriasis.

Parmi ces questions posées par votre patient, vous pouvez répondre affirmativement à :
A - Est-ce contagieux
B - Mes enfants risquent-ils d'avoir la même chose
C - Faut-il faire un régime alimentaire
D - Faut-il éviter le soleil
E - L'évolution est-elle prévisible
Bonne(s) réponse(s) : B

Le psoriasis est familial dans 1/3 des cas. Les HLA plus fréquemment rencontrés sont HLA B13 - B17 - CW 6 - DRW 7.
Le soleil améliore la dermatose dans la majorité des cas (efficacité des UVA dans la PUVA).
Toutefois, chez certains sujets, l'exposition solaire aggrave la dermatose. Lors d'une exposition trop intense, un psoriasis peut
apparaître sur les zones insolées, il est alors qualifié d'actinique. L'évolution est imprévisible ; trois grandes complications
dominent : l'érythrodermie, le psoriasis pustuleux et le rhumatisme psoriasique.

Parmi les traitements suivants, quel(s) est(sont) celui(ceux) que vous n'utiliserez sûrement pas chez ce patient ?
A - Dioxyanthranol
B - Corticothérapie locale
C - Méthotrexate
D - Puva®
E - Caryolysine locale
Bonne(s) réponse(s) : C

Le méthotréxate ne doit être employé que pour les psoriasis sévères, en raison de sa toxicité médullaire, hépatique et son
oncogénicité chez ce sujet jeune.

Un homme de 40 ans, brun à peau mate, a, sur l'épaule droite, une tumeur pigmentée. Celle-ci est survenue sur un "grain de
beauté" datant de l'enfance et a connu une extension depuis 2 ans environ. Aucun antécédent particulier personnel n'est noté
à l'interrogatoire mais le père du patient est décédé d'un mélanone malin. A l'examen, on note un nodule pigmenté de 5 mm
de diamètre reposant sur une plaque elle-même pigmentée de 2,5 cm x 1 cm, à bords irréguliers, d'aspect polychrome. La
chemise du patient est tachée par un discret saignement. Il n' y a pas de prurit. L'examen ne révèle pas d'adénopathie, en
particulier axillaire. Cliniquement, vous évoquez en premier le diagnostic de nodule sur SSM.

Quel(s) élément(s) clinique(s) fait (font) évoquer ce diagnostic de mélanome malin :


A - La polychromie
B - L'absence de prurit
C - L'absence d'adénopathie
D - Le caractère nodulaire infiltré
E - Le saignement
Bonne(s) réponse(s) : A D E

Le caractère nodulaire infiltré et le saignement sont malheureusement des signes tardifs On peut ajouter à ces critères : bords
polycycliques, taille supérieure à 1cm, contours encochés, surface irrégulière.

L'examen histopathologique a confirmé le diagnostic de SSM et a montré qu'il était survenu sur un naevus
préexistant. Quel(s) caractère(s) clinique(s) aurai(en)t permis de suspecter la transformation maligne de ce
naevus ?
A - Une extension de la lésion en surface
B - Un saignement
C - Un halo inflammatoire périnaevique
D - Une modification de la couleur
E - Un halo achromique périnaevique
Bonne(s) réponse(s) : A B C D

Un halo achromique périnaevique se voit au cours des halo-naevus de Jutton. Un halo achromique peut aussi entourer un
mélanome malin témoignant d'une réaction immunologique" du patient.

1117
Exclusivement sur DOC - DZ : www.doc-dz.com NADJI 85
RESIDANAT EN POCHE TOME II
Cas Clinique en QCM

L'examen anatomo-pathologique a donc confirmé le diagnostic et montré une tumeur de grade III (selon Clarke)
de 1,30 mm d'épaisseur. Quel est le meilleur critère histologique au plan pronostic ?
A - Epaisseur
B - Grade selon Clarke
C - Nombre de mitoses
D - Intensité de la réaction inflammatoire
E - Ulcération épidermique
Bonne(s) réponse(s) : A

L'épaisseur est l'indice de Breslow. Une épaisseur inférieure à 0,76 mm permet d'espérer une survie à 5 ans de 96%.

Au cours de la surveillance postopératoire, quel est le site le plus fréquent de la première métastase ?
A - Poumons
B - Foie
C - Os
D - Ganglion
E - Cerveau
Bonne(s) réponse(s) : D

La palpation ganglionnaire est un geste fondamental dans la surveillance d'un mélanome malin. La présence d'adénopathies
fait tomber l'espérance de survie à 5 ans à 25%.

Madame R. 25 ans, de race blanche, enceinte de six mois, vous consulte en septembre 1985 pour des taches achromiques
bien limitées, de taille variable, irrégulièrement réparties sur le thorax, non prurigineuses, au retour d'un séjour de 3 mois au
Gabon.

Quel est le diagnostic le plus probable ?


A - Pityriasis rosé de Gibert
B - Syphilis secondaire
C - Lèpre indéterminée
D - Eczématides achromiantes
E - Pityriasis versicolor
Bonne(s) réponse(s) : E

Le pityriasis versicolor est dû à malassezia furfur. La peau moite ou des affections chroniques débilitantes paraissent jouer un
rôle favorisant.
Les lésions sont des macules pigmentées très finement squameuses. Sur peau foncée, les macules semblent dépigmentées.
Les lésions sont vertes en lumière de Wood.
Le pityriasis rosé de Gilbert commence par un médaillon unique, suivi de multiples éléments érythémato-squameux du tronc.
Les macules de lèpre indéterminée sont hypoesthésiques.
Au moindre doute avec une syphilis secondaire, une sérologie est impérative.

Quel est (quels sont) l(es) autre(s )signe(s) cliniques que l'on peut trouver à l'examen ?
A - Localisation aux paumes et aux plantes
B - Existence de taches pigmentées
C - Halo fin hyperpigmenté entourant les taches achromiques
D - Aspect furfuracé et mat des taches observées
E - Hypoesthésie dans le territoire des taches
Bonne(s) réponse(s) : B D

Les paumes et les plantes sont toujours indemmes.


A. Est évocateur de syphilis.
C. De vitiligo.
E. De lèpre.

Comment affirmer le diagnostic suspecte ?


A - Examen en lumière de Wood
B - Papier cellophane adhésif
C - Recherche d'un signe du copeau avec un abaisse-langue
D - Pratique d'un TPHA ou d'un FTA
E - Culture sur milieu de Sabouraud
Bonne(s) réponse(s) : A C D

Les taches sont vertes en lumière de Wood.


Le scotch-test montre des levures arrondies, groupées en amas, en grappes de raisin.
Le signe du copeau est le détachement à la curette d'une fine squame non adhérente.

1118
Exclusivement sur DOC - DZ : www.doc-dz.com NADJI 85
RESIDANAT EN POCHE TOME II
Cas Clinique en QCM

Cette affection cutanée est caractérisée par :


A - Risque d'hypopigmentation résiduelle
B - Contagiosité faible
C - Favorisée par la grossesse et l'hypercorticisme
D - Favorisée par la chaleur et l'hypersécrétion sébacée
E - Rechutes fréquentes
Bonne(s) réponse(s) : A C D E

L'affection paraît peu contagieuse (les cas conjugaux sont rares).

Le traitement à envisager en première intention :


A - Est strictement local
B - Est d'une durée de 3 semaines minimum
C - Comporte une détersion cutanée et l'utilisation d'un antiseptique pendant 24 heures
D - Nécessite une désinfection soigneuse du linge de corps
E - Doit faire supprimer toute corticothérapie non indispensable
Bonne(s) réponse(s) : A B D E

Le traitement doit :
- être appliqué à l'ensemble du revêtement cutané
- être précédé d'une détersion cutanée
- utiliser des antifongiques actifs sur le pytirosporum orbiculare.

On peut utiliser :
A - Pevaryl® (éconazole) lotion, 2 applications/jour
B - Selsun® (sulfure de sélénium), en application 2 fois/semaine avec rinçage immédiat
C - Un ammonium quaternaire en application 2 fois/jour (ex. : Hexomédine® transcutanée)
D - Une pommade antifongique cortisonée 1 fois/jour (ex : Lococartel Vioforme®)
E - Mycostatine® (nystatine) 12 dragées/jour pendant 20 jours
Bonne(s) réponse(s) : A

- Noter que, contrairement à l'énoncé de la question C, l'Héxomédine® n'est pas un ammonium quaternaire.
- Le Selsun® doit être laissé sur le tégument pendant vingt minutes.
- Les imidazolés (Daktarin®, Pévaryl®, Fazol®) en applications biquotidiennes pendant 3 semaines sont actifs et bien tolérés.
- Le Nizoral® (200mg/5 pendant 5 à 10 jours) ne doit s'adresser qu'aux formes inaccessibles aux traitements locaux.

Le malade est un maçon d'une trentaine d'années, fumeur habituel (1 paquet/jour/10 ans), gros mangeur de viande, habitué à
travailler torse nu en plein air. Porteur depuis de longues années d'un noevus pigmentaire de la région sous-épineuse qui lui
paraissait s'étendre, il a demandé l'exérèse. Il s'agit d'un mélanome malin de type nodulaire, atteignant en profondeur le
derme réticulaire et mesurant 2 mm d'épaisseur selon Breslow : les mitoses et les atypies cellulaires sont assez fréquentes.
La lésion est ulcérée en surface. Il n'y a, à l'examen clinique, ni nodule cutané satellite, ni adénopathie.

Parmi les circonstances étiologiques signalées laquelle est connue pour sa corrélation épidémiologique avec le
mélanome malin ?
A - Maçon
B - Trente ans
C - Tabagisme (10 paquets/année)
D - Consommation de viande
E - Exposition au soleil
Bonne(s) réponse(s) : E

En particulier si sujets blonds...

Quel est le niveau d'extension de ce mélanome selon la classification de Clark ?


A-I
B - II
C- III
D - IV
E-V
Bonne(s) réponse(s) : D

A - Strictement épidermique.
B - Effraction de la membrane basale.
C - Envahissement de tout le derme papillaire.
D - Atteinte réticulaire.
E - Métastases à distance.

1119
Exclusivement sur DOC - DZ : www.doc-dz.com NADJI 85
RESIDANAT EN POCHE TOME II
Cas Clinique en QCM

L'observation comporte des éléments cliniques qui ont une portée pour le pronostic. Quel est, parmi ceux qui
suivent, celui qui n'a aucune valeur pronostique ?
A - Homme
B - Mélanome du tronc
C - Naevus ancien
D - Absence de nodule cutané satellite
E - Absence d'adénopathie
Bonne(s) réponse(s) : C

L'ancienneté ne joue aucun rôle, en revanche le pronostic est meilleur chez les femmes et certaines localisations sont plus
péjoratives : cuir chevelu, plante du pied ou région médiothoracique.

Parmi les données anatomo-pathologiques, quelle est celle qui n'a pas d'incidence sur le pronostic ?
A - Atteinte du derme réticulaire
B - Epaisseur de 2 mm
C - Mélanome nodulaire.
D - Atypies et mitoses fréquentes
E - Ulcération de la lésion
Bonne(s) réponse(s) : E

Sans commentaire.

Un homme de 25 ans, jusque là en bonne santé, consulte pour une asthénie avec fièvre à 38° C et douleurs à la déglutition. A
l'examen, on retrouve une angine et de petites adénopathies cervicales. Le médecin prescrit un traitement par sulfamides,
antiinflammatoires non stéroïdiens et paracétamol. Deux jours plus tard, devant l'absence d'amélioration, le patient consulte
un autre médecin qui arrête le traitement précédent et prescrit de l'ampicilline et de l'acide acétylsalicylique. Cinq jours plus
tard, vous êtes amenés à examiner ce patient. L'énanthème a disparu mais on note l'existence d'un exanthème maculo-
papuleux de type morbiliforme, rouge orangé, prurigineux, disséminé sur la presque totalité du tégument.

Parmi ces diagnostics, vous devez évoquer :


A - Erythrodermie
B - Scarlatine
C - Toxidermie
D - Rougeole
E - Mononucléose infectieuse
Bonne(s) réponse(s) : C D E

Ce tableau clinique associant syndrome général, angine, adénopathies et rash à l'ampicilline est très évocateur de MNI. Les
manifestations cutanées de la MNI sont constituées par des exanthèmes rubéoliformes, morbilliformes, voire scarlatiniformes.
Ils siègent sur le tronc et la racine des membres et d'évolution fugace.
Ce tableau clinique même très évocateur de MNI ne saurait éliminer une toxidermie et une rougeole.

La recherche du diagnostic étiologique nécessite de demander :


A - Numération formule sanguine
B - Fond d'oeil
C - Electrophorèse des protides
D - MNI test
E - IgE
Bonne(s) réponse(s) : A D

Sans commentaire (évident).

Au cas où vous évoqueriez une toxidermie, quels sont parmi les médicaments suivants. les 3 que vous
suspecteriez le plus ?
A - Sulfamides
B - Antiinflammatoires non stéroïdiens
C - Ampicilline
D - Aspirine
E - Paracétamol
Bonne(s) réponse(s) : A B C

Noter la fréquence des manifestations cutanées survenant quand l'ampicilline est associée à une autre pathologie = MNI,
infection à CMV, LLC, hyperuricémie traitée par allopurinol.

1120
Exclusivement sur DOC - DZ : www.doc-dz.com NADJI 85
RESIDANAT EN POCHE TOME II
Cas Clinique en QCM

Parmi les examens complémentaires suivants, vous demanderiez pour affirmer le diagnostic de toxidermie ?
A - Numération des éosinophiles
B - IgE spécifiques
C - IDR au médicament
D - TTL aux médicaments
E - Aucun de ces examens
Bonne(s) réponse(s) : E

Le seul diagnostic est clinique. La réintroduction serait dangereuse.

Dans l'hypothèse d'une mononucléose infectieuse, et à propos de l'éruption cutanée, il est vrai que :
A - On peut observer une telle éruption après prise d'ampicilline en dehors de ce contexte
B - On peut l'observer au cours de la mononucléose infectieuse en dehors de la prise d'ampicilline
C - Au cours de la MNI, son apparition est favorisée par la prise d'ampicilline
D - Elle n'interdit pas la prise ultérieure d'une autre bêta-lactamine
E - L'ampicilline peut être reprise à condition d'y associer une corticothérapie générale
Bonne(s) réponse(s) : A B C D

La prise d'ampicilline augmente considérablement la fréquence des rash dans la MNI, qui s'observent alors dans plus de 90%
des cas.

Un enfant de 8 ans est amené en consultation. A l'examen on observe un érythème suintant par endroits, aux plis de coudes
et sur le thorax. Les lésions ont débuté à l'âge de 5 mois, elles sont très prurigineuses, gênant souvent le sommeil. Une tante
maternelle a une rhinite saisonnière, la petite soeur a également des lésions cutanées, la mère a une peau très sèche formant
de petites écailles sur la face antérieure des jambes. Vous portez le diagnostic de dermatite atopique.

La localisation aux plis des coudes est habituelle dans la dermatite atopique de l'enfant. Parmi les localisations
suivantes, laquelle est inhabituelle ?
A - Creux poplités
B - Visage
C - Creux axillaires
D - Thorax
E - Fesses
Bonne(s) réponse(s) : C

La dermatite atopique du grand enfant touche les grands plis de flexion, les plis du coude, les creux poplités et la face latérale
du cou.
Thorax et fesses sont aussi des locations inhabituelles : ceci rend la question difficile.

La (les) manifestation(s) suivante(s) est (sont) habituelle(s) au cours de la dermatite atopique :


A - Infection cutanée
B - Fissuration du pli inférieur de l'oreille
C - Dépressions punctiformes des ongles
D - Sécheresse de la peau
E - Chute de cheveux
Bonne(s) réponse(s) : A B D

Les dépressions ponctiformes sont un signe de psoriasis.


Les critères mineurs de la dermatite atopique sont :
l) terrain atopique personnel ou familial
2) positivité immédiate des tests cutanés à divers allergènes
3) dermographisme blanc, blancheur cholinergique retardée
4) xérose
5) ichtyose vulgaire, kératose pilaire, accentuation des plis palmaires
6) pâleur faciale, aspect sombre des régions sous orbitaires
7) repli sous palpébral inférieur
8) cataracte antérieure sous corpsculaire
9) kératocone
10) infections cutanées récidivantes
11) tendance aux dermatoses non spécifiques des mains
12) elévation des IgE sériques totales

1121
Exclusivement sur DOC - DZ : www.doc-dz.com NADJI 85
RESIDANAT EN POCHE TOME II
Cas Clinique en QCM

L'interrogatoire permet souvent de trouver des antécédents familiaux de :


A - Psoriasis
B - Asthme
C - Furoncles
D - Hyperthyroïdie
E - Hypertension artérielle
Bonne(s) réponse(s) : B

Terrain atopique : rhinite, conjonctivite allergique, asthme.

La mère a une peau sèche avec formation de petites écailles à la partie antérieure des jambes. Cet état cutané
héréditaire est souvent associé à la dermatite atopique, il s'agit de :
A - Psoriasis
B - Eczéma de contact
C - Parapsoriasis
D - Pityriasis rosé
E - Ichtyose
Bonne(s) réponse(s) : E

L'ichtyose autosomale dominante dite vulgaire s'associe dans 1/3 des cas à une dermatite atopique, un asthme ou une rhinite
allergique.

Un homme de 65 ans, insuffisant cardiaque, hypertendu non diabétique, est hospitalisé pour un bilan d'une dermatose
bulleuse apparue depuis 1 mois. Les lésions initiales ont été buccales, caractérisées par des érosions étiquetées "aphtes" et
traitées sans succès par des antiseptiques locaux. Les bulles cutanées sont apparues en peau saine et forment rapidement
des érosions à bord décollés. Le signe de Nikolsky est positif. On note dans les antécédents un long passé de colite. Il existe
une fébricule à 38° C. L'hémogramme montre une hyperleucocytose à 11 000 GB/mm3 avec polynucléose.

Sur quel(s) argument(s) repose le diagnostic ?


A - Lésions muqueuses
B - Bulles en peau saine
C - Signe de Nikolsky positif
D - Hyperleucocytose à polynucléaires
E - Antécédent digestif
Bonne(s) réponse(s) : A B C

Les lésions muqueuses, les bulles en peau saine et le signe de Nikolsky évoquent les pemphigus.
Dans la pemphigoïde bulleuse : les lésions muqueuses sont plus rares, les bulles surviennent en peau érythémateuse, il n'y a
pas de signe de Nikolsky. Des antécédents digestifs (maladie coeliaque ?) orienterait vers une dermatite herpétiforme.

Que montrera en règle l'immunofluorescence directe ?


A - Fluorescence inter-cellulaire dans l'épiderme
B - Dépôt linéaire d'lgG à la jonction dermo-épidermique
C - Dépôt granuleux d'lgA au sommet des papilles
D - Dépôt linéaires d'lgA à la jonction dermo-épidermique
E - Absence de dépôt
Bonne(s) réponse(s) : A

Evident.

Que montrera la biopsie de peau ?


A - Une acantholyse
B - Une bulle intra-épidermique
C - Une bulle sous-épidermique
D - Une nécrose de l'épiderme
E - Des micro-abcès
Bonne(s) réponse(s) : A B

Le prélèvement doit porter sur une bulle récente. La biopsie montre un clivage intraépidermique profond au dessus de l'assise
basale.
Le décollement se fait par acantholyse.

1122
Exclusivement sur DOC - DZ : www.doc-dz.com NADJI 85
RESIDANAT EN POCHE TOME II
Cas Clinique en QCM

Vous traitez ce patient par corticothérapie générale à 2 mg/kg/jour. La surveillance de ce traitement comportera :
A - Méthémoglobinémie
B - Immunofluorescence directe
C - Glycémie
D - Immunofluorescence indirecte
E - Tension artérielle
Bonne(s) réponse(s) : C D E

Surveillance :
l) de la corticothérapie générale
2) du taux des anticorps anti-substance intercellulaire, corrélé à la sévérité et l'extension du pemphigus.

Mademoiselle F. 23 ans, est hospitalisée le 20 mai 1986 pour "oedème douloureux et fébrile de l'hémiface gauche".
Antécédents :
- père diabétique insulino dépendant
- poussées urticariennes depuis plusieurs années, sans étiologie formelle retrouvée, notion mal documentée d'intolérance
médicamenteuse.
Anammèse :
Le 13 mai 1986 ; lésions cutanées de la partie gauche de la lèvre supérieure, faites de petites vésicules groupées en bouquet
et s'accompagnant de "cuissons" (des poussées analogues avaient été déjà observées). Sur les conseils d'une amie, elle
applique localement diprosone Néomycine®, crème (dipropionate de bêtaméthasone).
Le 18 mai, les vésicules deviennent croûteuses, laissant sourdre un suintement louche ; un érythème discrètement infiltré
entoure les croûtelles. Des céphalées entraînent la prise d'Aspirine®.
Le 20 mai, l'érythème s'est étendu, tuméfiant la lèvre supérieure, la température est à 38,3° C.
Examen :
Le 20 mai à 18 h, on constate un érythème oedémateux déformant, l'hémiface gauche, gonflant les paupières. La face latérale
gauche du cou indemne le matin, commence à être atteinte par un bourrelet inflammatoire à extension centrifuge. La lésion
croûtelleuse de la lèvre persiste, entourée de deux zones de desquamation.
La température est à 40.2° C, la patiente frissonne et est abattue. L'hémogramme, fait à l'entrée, montre : GR 4,08 T/l - GB
18,2 G/l - P neutro 0,98.

Quel est le diagnostic présumé au 13 mai 1986 ?


A - Infection herpétique
B - Allergie au rouge à lèvre
C - Impétigo
D - Aphte muqueux
E - Autre diagnostic
Bonne(s) réponse(s) : A

Evident.

Quel est le diagnostic une semaine plus tard?


A - OEdème de Quincke
B - Eczéma aigu
C - Staphylococcie maligne de la face
D - Erysipèle
E - Syndrome de Kaposi Juliusberg
Bonne(s) réponse(s) : C

Il s'agit d'une cellulite diffuse avec phlébite extensive.

Pour confirmer ce diagnostic et apprécier la gravité, vous réalisez en urgence :


A - Cycle glycémique
B - Hémocultures
C - Test de dégranulation des basophiles à l'aspirine
D - Prélèvements bactériologiques au niveau des croûtelles
E - Sérologie herpès virus type I
Bonne(s) réponse(s) : A B D E

Les hémocultures sont très souvent positives.

1123
Exclusivement sur DOC - DZ : www.doc-dz.com NADJI 85
RESIDANAT EN POCHE TOME II
Cas Clinique en QCM

Le traitement à envisager en urgence le 20 mai 1986 est :


A - Solumédrol® IV® 80 mg
B - Héparinothérapie
C - Pénicilline G® injectable 20 M unités/jour
D - Pyostacine® (prinistinamycine) 2 g/jour
E - Josacine® (Josamycine) 2 g/jour
Bonne(s) réponse(s) : D

L'antibiothérapie est dont précoce adaptée au staphylocoque.

Une femme de 55 ans consulte pour une lésion pigmentée de 1,5 cm, présente depuis plusieurs années au niveau de la crête
tibiale. La lésion est de coloration irrégulière brun noir, et présente une périphérie plane et une composante centrale en relief.
L'examen clinique retrouve plusieurs autres lésions pigmentaires planes d'allure bénigne au niveau du dos sans modification
récente. L'examen clinique est par ailleurs normal, en particulier, ne montre pas d'adénopathie ni de splénomégalie. Par
ailleurs, l'interrogatoire révèle qu'il s'agit d'une femme nullipare, sportive (cyclisme + tennis) fumant un demi-paquet de
cigarettes par jour.

Devant une tumeur pigmentée, il faut évoquer systématiquement :


A - Naevus naevocellulaire
B - Angiome thrombose
C - Verrue séborrhéïque
D - Mélanome malin
E - Epithélioma baso-cellulaire pigmenté
Bonne(s) réponse(s) : A B C D E

Sans commentaire.

Quelle est la conduite à tenir vis-à vis de la lésion de cette patiente ?


A - Electrocoagulation
B - Biopsie de la partie la plus en relief
C - Exérèse de l'ensemble de la lésion
D - Simple surveillance en l'absence de signes fonctionnels
E - Biopsie à la jonction peau saine peau pigmentée
Bonne(s) réponse(s) : C

Les caractères suspects de la tumeur imposent ici une histologie, ce qui exclut les items A et D.
La biopsie partielle ne permet pas une étude satisfaisante (infiltration en profondeur, morphologie cellulaire variable d'une
zone à l'autre).

Le pronostic du mélanome malin est lié en premier lieu à :


A - La localisation
B - La présence d'ulcération
C - L'épaisseur de la tumeur mesurée en millimètres
D - Le nombre de mitoses
E - L'intensité de la pigmentation
Bonne(s) réponse(s) : C

L'épaisseur tumorale est mesurée précisément au micromètre oculaire (au l/l00mm). Pour l'épaisseur inférieure à 0,76 mm,
les chances de guérison après exérèse élargies sont proches de 100 %.

Parmi les facteurs de risque du mélanome malin vous retenez :


A - Le tabagisme
B - L'exposition solaire
C - L'exercice physique
D - Une peau claire
E - La nulliparité
Bonne(s) réponse(s) : B D

L'effet néfaste des radiations solaires est à l'origine du siège fréquent des mélanomes sur les zones découvertes, de
l'augmentation de l'incidence chez les sujets à carnation claire. La grossesse peut entraîner une poussée évolutive du
mélanome.

1124
Exclusivement sur DOC - DZ : www.doc-dz.com NADJI 85
RESIDANAT EN POCHE TOME II
Cas Clinique en QCM
Ses parents amènent Pierre, âgé de 4 ans, dernier enfant d'une fratrie de trois enfants, pour des lésions d'eczéma rebelles de
la saignée des coudes et des creux poplités. Cet eczéma aurait débuté vers l'âge de 3 mois au niveau du visage. Plus
disséminées au début, les lésions se sont localisées aux endroits actuels depuis environ un an, avec de temps à autre des
poussées plus diffuses. Il va plutôt mieux en hiver.
Actuellement, l'examen retrouve en outre, chez ce garçon aux yeux clairs, visage un peu squameux et pâle, un intertrigo
fissuraire sous auriculaire bilatéral. Pierre a fait entre 1 et 2 ans d'assez nombreuses bronchites subaiguës. Sa mère présente
un eczéma chronique du dos des mains et des doigts depuis l'âge de 29 ans. Différents bilans biologiques ont déjà été
pratiqués dont on retient une VS accélérée à certains moments, une tendance à la Iymphocytose, à deux examens une
éosinophilie à 600 éléments, une élévation des Ig E. Enfin, un bilan "immunologique" a révélé un test de dégranulation des
basophiles positif vis à vis des acariens.

Quel est ou quels sont les arguments anamnestiques qui sont en faveur d'une atopie ?
A - L'existence d'un eczéma des mains de la mère
B - L'absence d'atteinte cutanée dans la fratrie
C - Le début des lésions cutanées vers 3 mois chez l'intéressé
D - L'existence d'épisodes de bronchite asthmatiforme chez l'enfant
E - L'amélioration des lésions en hiver
Bonne(s) réponse(s) : C D

L'eczéma des mains d'apparition tardive chez la mère peut témoigner d'une sensibilisation de contact. L'âge moyen du début
de la dermatite atopique est 8 à 9 mois mais les lésions caractéristiques sont déjà notées chez 30% des patients avant l'âge
de 3 mois. La bronchite asthmatiforme récidivante d'un nourrisson est considérée comme un équivalent d'asthme.

Quel est ou quels sont les arguments cliniques qui sont en faveur d'une atopie ?
A - L'existence d'un intertrigo fissuraire sous-auriculaire
B - La couleur claire des yeux
C - L'atteinte de la saignée des coudes
D - L'aspect furfuracé et pale du visage
E - Le respect des pieds
Bonne(s) réponse(s) : A C D

A. C. Appartiennent aux localisations classiques de la dermatite atopique.


D. Est un des nombreux signes mineurs caractérisant l'atopique (au même titre que le 2e pli sous palpébral, la kératose
pilaire, l'hyperlinéarité palmaire...).

Quel(s) est(sont) le(s) élément(s) biologique(s) en faveur d'une atopie ?


A - Une VS accélérée
B - Une éosinophilie à 600/ml
C - Un test de dégranulation des basophiles positif vis-à-vis des acariens
D - Des Ig E à 650 KU/ml
E - Une Iymphocytose relative
Bonne(s) réponse(s) : B C D

Sans commentaire.

Parmi les traitements suivants lequel(lesquels) est(sont) contre-indiqué(s) chez cet enfant ?
A - Une crème vaselinée
B - Une pommade à l'hydrocortisone
C - Une corticothérapie générale
D - Une pommade au Phénergan
E - Des antihistamiques per os
Bonne(s) réponse(s) : C D

La corticothérapie expose au risque de corticodépendance.


Les topiques antihistaminiques induisent des allergies de contact.

Interrogé par les parents sur le devenir de l'atopie de cet enfant, quelle(s) réponse(s) pouvez-vous leur donner ?
A - Guérison certaine à la puberté au plus tard
B - Possibilité de guérison vers l'âge de 7 à 8 ans
C - Possibilité d'apparition d'un asthme
D - Aucun risque d'affection identique chez un enfant ultérieur
E - Evolution capricieuse
Bonne(s) réponse(s) : B C E

10 % des dermatites atopiques persistent chez l'adulte.


L'association à l'asthme est d'autant plus à craindre que la dermatite atopique est sévère.

1125
Exclusivement sur DOC - DZ : www.doc-dz.com NADJI 85
RESIDANAT EN POCHE TOME II
Cas Clinique en QCM
Une femme de 18 ans consulte pour des lésions érythémateuses et prurigineuses des lobules des oreilles, non calmées par
les antihistaminiques. A l'examen, la surface des plaques est couverte de vésicules, certaines d'entre elles sont rompues et
donnent lieu à un suintement clair. Il n'existe pas d'adénopathies pré-auriculaires. Cette dermite auriculaire est apparue
quelques jours après le port de nouvelles boucles d'oreilles métalliques. La malade avait d'ailleurs constaté des poussées
identiques après le port de divers accessoires vestimentaires (boucles d'oreilles, bracelet de montre). Dans les antécédents, il
faut noter une rhinite saisonnière chez la patiente et un asthme chez son frère. Le diagnostic de dermite de contact est posé
et les métaux sont soupçonnés comme allergènes.

En faveur du diagnostic clinique d'eczéma, vous retenez :


A - Le prurit
B - L'absence d'adénopathies
C - L'âge de la malade
D - La présence de vésicules
E - L'existence d'un suintement
Bonne(s) réponse(s) : A D E

Les propositions B et C n'orientent pas spécifiquement vers le diagnostic d'eczéma.

Quelle(s) est (sont) le(s) argument(s) en faveur d'une dermite de contact ?


A - L'antécédent d'asthme chez son frère
B - L'antécédent personnel de rhinite
C - La localisation au site du contact
D - La chronologie des récidives
E - L'inefficacité des antihistaminiques
Bonne(s) réponse(s) : C D

L'anamnèse est un élément capital du diagnostic.


Les items A et B se rapportent au terrain atopique qui ne constitue pas un facteur de risque d'eczéma de contact.

Quelle(s) complication(s) doit-on redouter ?


A - La contagiosité
B - La diffusion de l'eczéma
C - Un érythème noueux
D - La surinfection par une bactérie gram +
E - Le dégénérescence épithéliomateuse
Bonne(s) réponse(s) : B D

Si le siège initial correspond toujours à la zone de contact, les lésions d'eczéma peuvent secondairement s'étendre à distance.

Parmi les propositions suivantes concernant la physiopathologie de l'eczéma de contact, il est exact que :
A - L'allergie se manifeste dans les 3 heures qui suivent le premier contact de l'allergène avec la peau
B - L'allergène est reconnu par les cellules de Langerhans
C - Les polynucléaires éosinophiles sont porteurs d'une "mémoire" spécifique pour l'allergène
D - La fixation d'IgE sur les Iymphocytes "sensibilisé
E - Lors du contact déclenchant apparait un afflux de lymphocytes dermiques
Bonne(s) réponse(s) : B

Les cellules de Langerhans porteuses des antigènes HLA-DR captent l'allergène puis le présentent aux lymphocytes T. La
phase de sensibilisation dure environ une semaine. En cas de nouveau contact, les lymphocytes sensibilisés affluent dans le
derme dans les 48 heures, responsables d'une réaction d'allergie retardée.

Chez cette malade, quel est le métal responsable de l'eczéma de contact ?


A - Le chrome
B - Le cobalt
C - Le nickel
D - Le cuivre
E - Le fer
Bonne(s) réponse(s) : C

Le nickel est contenu dans les bijoux fantaisies, pièces métalliques de certains vêtements, matériel de cuisine, certaines
lessives, amalgames dentaires.

1126
Exclusivement sur DOC - DZ : www.doc-dz.com NADJI 85
RESIDANAT EN POCHE TOME II
Cas Clinique en QCM

Quelle(s) est(sont) le(s) test(s) que vous utilisez pour démontrer la responsabilité du métal incriminé ?
A - L'élévation des IgE sériques spécifiques
B - L'existence d'une éosinophilie
C - Les tests intradermiques aux particules métalliques
D - Les données de l'examen histologique
E - Les résultats des test épicutanés ("patch-tests")
Bonne(s) réponse(s) : E

Il s'agit de la seule méthode admise pour prouver la responsabilité d'une substance dans l'eczéma de contact. L'interprétation
est cependant délicate et les discordances avec la clinique ne sont pas rares.

L'allergène métallique que vous avez incriminé est présent dans :


A - Les bijoux "fantaisie"
B - La colle Araldite®
C -Les pièces de monnaie
D - Le ciment
E - Les boutons de "jeans"
Bonne(s) réponse(s) : A C E

Les pièces de monnaie de 1 franc et 50 centimes contiennent du nickel et peuvent induire une sensibilisation à travers la
poche du pantalon !

Chez cette patiente, la(les) mesure(s) thérapeutique(s) essentielle(s) est(sont) :


A - L'application d'une crème anti-hismatique (Phénergan crème®)
B - L'utilisation de dermocorticoïdes
C - La prescription d'une antibiothérapie générale par bêta-lactames (pénicilline V)
D - La désensibilisation spécifique
E - La remise d'une liste des produits contenant l'allergène
Bonne(s) réponse(s) : B E

L'éviction du produit sensibilisant est indispensable à la guérison.


Les dermocorticoïdes pendant quelques jours permettront d'accélérer la guérison de la poussée actuelle.
Les antihistaminiques locaux sont à éviter car allergisants.
La désensibilisation spécifique est rarement utilisée dans les eczémas de contact mais obtient parfois des résultats
satisfaisants. C'est le cas des patients traités par Caryolysine® locale qui développent une allergie de contact dans un tiers
des cas.

Depuis plusieurs mois, une femme de ménage de 42 ans se plaint de lésions inflammatoires du pourtour unguéal des 3
premiers doigts de la main droite. Les régions péri-unguéales sont douleureuses, rouges et oedématiées. La pression en fait
sourdre une gouttelette de pus jaunâtre. Sur les bords latéraux de chaque ongle atteint, existe une tâche brun jaunâtre, qui
tend à décoller l'ongle. Cette femme est obèse ( 70kg pour 1,52m) et fume beaucoup, ce dont témoigne la coloration jaune de
certains de ses doigts. Elle prend les médicaments suivants : oestroprogestatifs à visée contraceptive, calcium pour une
"tétanie", sulfamide hypoglycémiant pour un diabète "gras", furosémide (Lasilix) pour une HTA. Elle refuse tout régime
hypocalorique mais ingurgite irrégulièrement de la "levure de bière" pour maigrir. L'examen clinique permet de porter le
diagnosticd'onyxis avec périonyxis dont l'origine mycosique est fortement soupçonnée.

Quelles sont les 3 causes de périonyxis avec écoulement purulent ?


A - Psoriasis vulgaire
B - Infection par pseudomonas aéruginosa
C - Infection par trichophyton interdigitale
D - Infection par candida albicans
E - Infection par staphylococcus aureus
Bonne(s) réponse(s) : B D E

Le périonyxis est absent dans les infections à dermatophytes.

Chez cette femme, quel(s) facteur(s) local(ux) peu(ven)t favoriser une candidosese unguéale ?
A - L'immersion fréquente des mains dans l'eau
B - Les soins excessifs de manucurie
C - Le contact avec des animaux familiers
D - Le tabagisme
E - Une candidose vaginale
Bonne(s) réponse(s) : A B E

La candidose vaginale peut être le point de départ d'une localisation secondaire unguéale par autoinoculation.

1127
Exclusivement sur DOC - DZ : www.doc-dz.com NADJI 85
RESIDANAT EN POCHE TOME II
Cas Clinique en QCM

Chez cette femme, quel(s) facteur(s) général(ux) peu(ven)t favoriser une candidose unguéale ?
A - Un diabète
B - La prise de calcium au long cours
C - Les sulfamides hypoglycémiants
D - Le furosémide
E - L'ingestion de "levure de bière"
Bonne(s) réponse(s) : A

Les facteurs généraux favorisant les candidoses sont : le diabète, l'immunodépression (essentiellement déficit de l'immunité
cellulaire) et la corticothérapie.

Vous devez rechercher d'autre(s) localisation(s) d'une candidose sous la forme de :


A - Glossite
B - Perlèche
C - Intertrigo sous-mammaire
D - Teigne
E - Anite
Bonne(s) réponse(s) : A B C E

La glossite peut revêtir plusieurs aspects : érythémateuse, langue noire villeuse, glossite losangique médiane.

Vous décidez de traiter cette malade par un antifongique per os.


Quel médicament prescrivez-vous ?
A - Nystatine en comprimés
B - Amphotéricine B en suspension
C - Kétoconazole en comprimés
D - Métronidazole en comprimés
E - Griséofulvine en comprimés
Bonne(s) réponse(s) : C

La localisation unguéale impose l'utilisation d'un antifongique à diffusion systémique ; le métronidazole et la griséofulvine sont
inactifs sur le candida albicans.

Pour le médicament que vous avez prescrit, quel paramètre biologique devez-vous surveiller régulièrement ?
A - L'uricémie
B - La triglycéridémie
C - La sérologie anti-candidosique
D - Les transaminases (ASAT, ALAT)
E - La glycémie post-prandiale
Bonne(s) réponse(s) : D

Les transaminases sont dosées au 15e et 30e jour de traitement puis mensuellement. L'arrêt du médicament s'impose si leur
taux augmente au delà de deux fois la normale.

Dans l'hypothèse où vous utilisez un traitement local, quel(s) est(sont) celui(ceux) qui peu(ven)t être efficace(s) ?
A - Nystatine en pommade
B - Amphotéricine B en crème
C - Dérivés imidazolés en crème
D - Métronidazole en crème
E - Griséofulvine en pommade
Bonne(s) réponse(s) : A B C

Sans commentaire.

1128
Exclusivement sur DOC - DZ : www.doc-dz.com NADJI 85
RESIDANAT EN POCHE TOME II
Cas Clinique en QCM
Monsieur B. 37 ans, comptable, 2 enfants, vous consulte pour une ulcération située dans le sillon balano-préputial, apparue 8
jours auparavant et qui l'inquiète légitimement. C'est le premier épisode du genre. Tout a commencé par une érosion bien
limitée d'environ 1 cm sur 0,5 cm, ovalaire, située à côté du frein du prépuce. L'érosion s'est progressivement creusée mais
reste relativement superficielle. Elle est indolore, un peu suintante, non indurée. Monsieur B. n'a reçu aucun traitement local
ou général. L'examen des organes génitaux externes est par ailleurs normal ainsi que le toucher rectal. On trouve une
adénopathie inguinale droite de 3 cm de diamètre, indolore, mobile, non inflammatoire. La température est à 37,2°C. Le reste
de l'examen clinique est normal. Enfin, il n'existe aucun antécédent pathologique en particulier vénérien. Depuis environ 3
mois Monsieur B. a une "amie" Melle X. qu'il voit tous les après-midi. Elle semble bien portante. Monsieur B. a par contre très
peu de relations sexuelles avec sa femme, la dernière remonte à environ 1 mois. Vous évoquez le diagnostic de chancre
syphilitique.

Vous pratiquez un examen direct à la recherche de tréponéma pallidum sur le chancre. Parmi ces 5
propositions, une seule est exacte, laquelle ?
A - Examen au microscope optique d'un frottis de la sérosité sur une lame colorée au Giemsa
B - Examen au microscope optique d'un frottis de la sérosité sur une lame colorée au Gram
C - Examen au microscope à fond noir d'un frottis de la sérosité sur une lame colorée au Giemsa
D - Examen au microscope à fond noir d'un frottis de la sérosité sur une lame colorée au Gram
E - Examen au microscope à fond noir d'une goutte de sérosité diluée sans coloration
Bonne(s) réponse(s) : E

L'examen direct au microscope à fond noir, sans coloration, permet de visualiser le tréponème : spiralé, de 10 mm de long
environ, à mobilité "lente et majestueuse".

Afin d'éliminer absolument un chancre mou, vous recherchez également des bacilles de ducreyi. Quelle(s)
méthode(s) adoptez-vous pour ce faire ?
A - Examen au microscope optique d'un frottis de la sérosité sur une lame colorée au Giemsa
B - Examen au microscope optique d'un frottis de la sérosité sur une lame colorée au Gram
C - Examen au microscope à fond noir d'un frottis de la sérosité sur une lame colorée au Gram
D - Examen au microscope à fond noir d'une goutte de sérosité diluée dans du sérum physiologique entre
lame et lamelle sans coloration
E - Culture sur milieu d'isolement d'hemophilus ducreyi (sérum de veau foetal)
Bonne(s) réponse(s) : A B

Le bacille de Ducreyi est difficile à visualiser au direct et l'examen au Giemsa est plus sensible que le Gram.
La culture sur milieu spécial permet d'observer un coccobacille à coloration bipolaire, disposé en chaîne de bicyclette.

Vous avez affirmé le diagnostic de syphilis primaire et éliminé un chancre d'autre origine. Choisissez les deux
modalités thérapeutiques les plus judicieuses chez ce malade :
A - Pénicilline G IM 1 million par jour pendant 15 jours
B - Benzathine pénicilline G IM 1 million par jour pendant 15 jours
C - Procaïne pénicilline G IM 1 million par jour pendant 15 jours
D - Benzathine pénicilline G IM 2,4 millions à J0, dose répétée à J8
E - Spectinomycine IM 2 g par jour pendant 15 jours
Bonne(s) réponse(s) : C D

La benzathine pénicilline est une pénicilline retard dont les injections quotidiennes sont inutiles.
Les injections IM de pénicilline étant douloureuses, l'association procaïne-pénicilline G est préférable pour un traitement de 15
jours.
La spectinomycine est inactive sur le tréponème.

M. B. vous demande quel sera son avenir médical à moyen terme ? Vous lui expliquez :
A - Que le risque de tabès persiste même si le traitement est correct, ce qui nécessite une surveillance
annuelle
B - Que la guérison est certaine sous traitement
C - Que le traitement doit être poursuivi jusqu'à négativation du TPHA
D - Que des cures de consolidation seront probablement nécessaires dans un an
E - Que sa sérologie réaginique restera définitivement positive
Bonne(s) réponse(s) : B

Une pénicillinothérapie bien conduite assure une guérion certaine qui se traduira par la négativation ou la diminution du titre
du VDRL alors que le TPHA reste positif de façon prolongée.

1129
Exclusivement sur DOC - DZ : www.doc-dz.com NADJI 85
RESIDANAT EN POCHE TOME II
Cas Clinique en QCM
Une femme de 25 ans, enceinte de 3 mois, vous consulte pour une suspicion d'herpès génital. Elle a depuis 2 jours trois
lésions vésiculeuses, actuellement ulcérées, de 2 mm de diamètre chacune, sur la face externe de la petite lèvre droite. C'est
la troisième fois depuis 1 an qu'elle souffre de pareilles lésions à cet endroit.

Vous pourrez confirmer le diagnostic d'herpès par :


A - La recherche d'herpès simplex virus (HVS) dans les lésions par isolement en culture de cellules in vitro
B - La recherche d'antigènes HSV par cytodiagnostic en immunofluorescence sur des cellules prélevées à la
base des ulcérations
C - Le dosage des anticorps HSV par neutralisation ou fixation du complément dans un sérum prélevé à cette
patiente
D - Le dosage des anticorps par neutralisation ou fixation du complément dans 2 sérums prélevés à cette
patiente, le 1er et le 10ème jour de la consultation
E - Le dosage des IgM spécifiques de l'HSV type II dans un sérum prélevé à cette patiente 5 à 10 jours après
la consultation
Bonne(s) réponse(s) : A B

Au cours des récurrences herpétiques, le titre des anticorps n'augmente pas ou peu et la recherche d'IgM spécifiques est
généralement négative si bien que la sérologie n'est ici d'aucun intérêt.

Le diagnostic d'herpès génital récurrent étant acquis, quelle attitude raisonnable conseillez-vous à cette femme ?
A - De réfléchir à l'éventualité d'une IVG
B - De prendre de l'aciclovir par voie orale jusqu'à l'accouchement qui pourra se faire alors par voie basse
C - D'accoucher systématiquement par césarienne
D - De débuter la recherche hebdomadaire d'une excrétion asymptomatique d'HSV dans les sécrétions
génitales par inoculation à des cultures de cellules
E - Aucune des propositions précédentes
Bonne(s) réponse(s) : E

Chez cette patiente aux antécédents d'herpès génital on proposera une surveillance clinique virologique (immunolluorescence
directe) toutes les deux semaines à partir de la 30e semaines d'aménorrhée puis hebdomadaire à partir de la 37e semaine.

Cette femme ne revient vous voir qu'au moment du travail, à 8 mois 1/2. Depuis la dernière visite, elle n'a pas
eu de récidives manifestes d'herpès et l'examen gynécologique ne montre pas de lésions. La poche des eaux
n'est pas rompue. Si cette femme accouche par voie basse, le risque d'herpès pour le nouveau-né peut être
estimé à :
A - 85 %
B - 50 %
C - 20 %
D-5%
E - 0,1 %
Bonne(s) réponse(s) : E

En cas de récurrence herpétique génitale symptomatique du prépartum.


Le risque d'herpès néonatal est de 2 à 5%. Chez les patientes asymptomatique à terme mais présentant des antécédents
d'herpès génital, le risque n'est que de 0,1 %.

Cette femme ayant décidé d'accoucher par voie basse, quelle(s) mesure(s) allez-vous préconiser ?
A - Le monitorage du foetus par électrodes de scalp
B - Le traitement systématique du nouveau-né par aciclovir oral pendant 10 jours
C - Le prélèvement des sécrétions oculaires et pharyngées de l'enfant pour recherche d'HSV
D - La séparation de l'enfant et de la mère
E - L'interdiction d'allaiter l'enfant au sein
Bonne(s) réponse(s) : C

L'énoncé ne précise pas les résultats de la virologie maternelle au moment de l'accouchement or les récurrences prouvées
virologiquement sont asymptomatiques dans un tiers des cas.

1130
Exclusivement sur DOC - DZ : www.doc-dz.com NADJI 85
RESIDANAT EN POCHE TOME II
Cas Clinique en QCM
Monsieur R. 80 ans, sans antécédent particulier, est hospitalisé en dermatologie pour une éruption bulleuse évoluant depuis
trois semaines. L'examen cutané montre de nombreuses bulles (30 environ), diffuses, prédominant à la racine des membres
et sur le tronc. Ces bulles sont assez volumineuses (plus de 1 cm de diamètre), tendues, à liquide clair et reposent sur des
plaques érythémateuses ou urticariennes. Il existe un prurit intense qui a précédé l'apparition de l'éruption. Les bulles évoluent
vers des érosions arrondies qui cicatrisent lentement. Il n'y a pas de lésion muqueuse, pas de signe de Nikolsky. L'examen
clinique ne montre rien de particulier par ailleurs et l'état général est bien conservé, sans fièvre. La biopsie cutanée d'une
lésion bulleuse récente montre une volumineuse bulle sous-épidermique, le toit épidermique est intact ; le derme est le siège
d'un infiltrat inflammatoire contenant des polynucléaires éosinophiles.

Quel(s) diagnostic(s) peut-on éliminer formellement d'après le siège sous-épidermique du décollement ?


A - Dermatite herpétiforme
B - Erythème polymorphe
C - Pemphigus vulgaire
D - Pemphigus superficiel
E - Porphyrie cutanée tardive
Bonne(s) réponse(s) : C D

Les pemphigus entraînent un décollement intra-épidermique : suprabasal dans le pemphigus vulgaire, sous-corné dans le
pemphigus superficiel.

Devant ce tableau de dermatose bulleuse, vous évoquez le diagnostic de pemphigoïde bulleuse devant ?
A - Age
B - Bulles tendues
C - Plaques urticariennes
D - Prurit intense pré-éruptif
E - Erosions
Bonne(s) réponse(s) : A B C D

Les érosions font suite à toute dermatose vésiculo-bulleuse sans préjuger de son étiologie.

Parmi les examens complémentaires suivants, quel est celui qui est indispensable au diagnostic de cette
maladie bulleuse ?
A - Hémogramme
B - Cytodiagnostic du liquide de bulle
C - Biopsie cutanée avec étude en microscopie électronique
D - Immunofluorescence directe cutanée
E - Groupage tissulaire HLA
Bonne(s) réponse(s) : D

L'immunofluorescence directe cutanée montre des dépôts linéaires le long de la jonction dermo-épidermique, en IgG et C3. La
microscopie électronique permet de localiser le clivage dans la portion supérieure de la lamina lucida, mais n'est pas
nécessaire au diagnostic.

Quel(s) type(s) d'anticorps circulants est (sont) retrouvé(s) dans la majorité des cas de pemphigoïde bulleuse ?
A - Antinucléaire
B - Anti-substance intercellulaire épidermique
C - Anti-membrane basale
D - Anti-couche cornée
E - Anti-muscle lisse
Bonne(s) réponse(s) : D

Des IgG circulantes anti-membrane basale sont observées dans 70% des pemphigoïdes bulleuses, sans incidence
pronostique.

Parmi les mesures suivantes, laquelle envisagez-vous DE PREMIERE INTENTION pour le traitement de cette
pemphigoïde bulleuse ?
A - Etretinate® (1 mg/kg/jour)
B - Pénicilline IV® ( 10 millions par jour)
C - Prednisone® ( 1 à 1 5 mg/kg/jour)
D - Endoxan® (50 à 100 mg/jour)
E - Echanges plasmatiques (plasmaphérèses)
Bonne(s) réponse(s) : C

En première intention, les corticoïdes constituent le traitement de choix.


Les immunosuppresseurs ne sont pas utilisés en monothérapie car leur délai d'action est trop long.
Ils sont en revanche associés avec intérêt aux corticoïdes dans les formes très sévères de pemphigoïde bulleuse et chez les
sujets présentant une contre-indication à l'emploi de fortes doses de corticoïdes.

1131
Exclusivement sur DOC - DZ : www.doc-dz.com NADJI 85
RESIDANAT EN POCHE TOME II
Cas Clinique en QCM
Un jeune homme de 21 ans, est admis en urgence pour une éruption cutanée et muqueuse d'apparition brutale,
accompagnée d'une fièvre à 38,5 °C. L'éruption cutanée, non prurigineuse, est faite d'éléments papulo-oedémateux, rouges,
arrondis, d'environ 1 cm de diamètre, isolés ou confluents centrés par une zone plus claire ou cyanotique et, pour certains, par
une bulle réalisant un aspect en cocarde. Les éléments cutanés sont répartis de façon diffuse, grossièrement symétrique et
prédominent aux extrémités des membres. Les lèvres sont oedématiées et recouvertes d'érosion et de croûtes
hémorragiques, empêchant le malade de s'alimenter. Il existe quelques érosions douloureuses endobuccales. La muqueuse
du gland est également le siège de lésions érosives. Il est noté une conjonctivite congestive bilatérale.
L'interrogatoire apprend que le malade a présenté 10 jours auparavant une
lésion de la lèvre supérieure, formée d'une tache rouge, cuisante, surmontée
d'un bouquet de petites vésicules, qui a guéri spontanément en 8 jours. Des
lésions identiques, toujours de même siège labial, était déjà survenues à
plusieurs reprises au cours des derniers mois.

Parmi les diagnostics dermatologiques suivants, quel est le plus probable pour l'éruption diffuse cutanéo-
muqueuse de ce malade ?
A - Pemphigoïde bulleuse
B - Dermatite herpétiforme
C - Urticaire généralisée
D - Erythème polymorphe
E - Pemphigus vulgaire
Bonne(s) réponse(s) : D

Description typique d'érythème polymorphe.

Quel diagnostic rétrospectif évoquez-vous pour l'éruption de la lèvre apparue 10 jours plus tôt ?
A - Impétigo
B - Herpès
C - Zona
D - Epidermomycose
E - Eczéma
Bonne(s) réponse(s) : B

Il s'agissait d'une récurrence herpétique.


L'herpès est la cause la plus fréquente des érythèmes polymorphes de l'adulte jeune et les séries récentes montrent
l'efficacité du Zovirax® dans la prévention des récidives d'érythème polymorphe post herpétique.

Quel est (quels sont) le (les) mode(s) évolutif(s) vraisemblable(s) chez un tel malade ?
A - Atteinte cornéenne
B - Guérison en trois semaines
C - Récidive ultérieure
D - Cicatrisation cutanés atrophique indélébile
E - Surinfection des lésions muqueuses
Bonne(s) réponse(s) : A B C E

Les lésions régressent habituellement sans séquelle au niveau cutané. L'atteinte oculaire constitue par contre un facteur
pronostique important.

Parmi les mesures suivantes, laquelle (lesquelles) a (ont) un intérêt pour ce malade ?
A - Antisepsie locale
B - Corticothérapie locale
C - Antibiothérapie générale à large spectre
D - Immunodépresseurs
E - Surveillance ophtalmologique
Bonne(s) réponse(s) : A E

L'antibiothérapie est réservée aux infections septicémiques documentées (hémocultures, cartographie bactérologique
cutanée) et adaptée aux germes isolés aux différents prélèvements. Aucun traitement ne permet de réduire la durée de la
poussée d'érythème polymorphe.

1132
Exclusivement sur DOC - DZ : www.doc-dz.com NADJI 85
RESIDANAT EN POCHE TOME II
Cas Clinique en QCM
Un homme de 40 ans, représentant de commerce, présente une éruption légèrement infiltrée, formée de petits éléments rosés
de 5 à 10 mm de diamètre, squameux au niveau du tronc et des membres supérieurs. L'examen clinique montre par ailleurs
des adénopathies de petite taille, au niveau des aires cervicales. L'examen de la muqueuse est normal, il n'y a pas d'hépato-
splénomégalie, pas de fièvre, l'état général est bon. Six semaines auparavant, le patient a présenté un écoulement purulent
uréthral pour lequel un traitement par cycline à la dose d'1 g par jour a été prescrit, mais n'a été suivi que 2 jours par le
patient, en raison d'une amélioration rapide. Une semaine avant l'éruption, le patient a repris ce traitement de son propre chef
pendant 2 jours, en raison d'un épisode grippal.

Devant l'aspect dermatologique de l'éruption quel sont les deux diagnostics que vous évoquez ?
A - Psoriasis en goutte
B - Toxidermie
C - Erythosma
D - Syphilis secondaire
E - Varicelle
Bonne(s) réponse(s) : A D

Le psoriasis en goutte s'observe habituellement chez l'enfant ou adulte jeune, souvent précédé d'une infection ORL, il réalise
une éruption de papules érythémato-squameuses lentriculaires bien limitées respectant le visage à la différence de la syphilis
secondaire qui atteint électivement les régions péribuccale, sillon nasogénien paumes et plantes.

Quel examen paraclinique est susceptible de préciser le diagnostic étiologique vraisemblablement en cause
dans ce cas ?
A - Biopsie cutanée
B - Sérologie des chlamydia
C - VDRL
D - Uricémie
E - TTL aux cyclines
Bonne(s) réponse(s) : C

Au stade de syphilis secondaire, toutes les réactions sérologiques sont positives.

Afin de compléter le bilan, il est indispensable de demander :


A - Coloration de gram sur frottis urétral
B - Sérologie HIV
C - TPHA
D - Immunofluorescence directe antichlamydia sur frottis urétral
E - Aucun des examens précédents
Bonne(s) réponse(s) : E

Devant une éruption évocatrice et un VDRL fortement positif, la confirmation par TPHA n'est pas indispensable. Pour
diagnostiquer une infection à chlamydia associée, la proposition (D) risque d'être peu contributive.

Parmi les antibiotiques suivants, lequel n'a aucune efficacité dans le traitement de la maladie responsable de
l'éruption ?
A - Pénicilline G
B - Tétracyclines
C - Spectinomycine
D - Erythromycine
E - Benzathine-pénicilline
Bonne(s) réponse(s) : C

La spectinomycine est prescrite en traitement minute des uréthrites gonococciques. La pénicilline est l'antibiotique de choix
pour le traitement de la syphilis. En cas d'allergie, on préfèrera les cyclines aux macrolides.

Parmi les affirmations suivantes, concernant l'infection à chlamydia, laquelle est vraie ?
A - Elle n'est jamais associée à une infection à gonocoque
B - Elle est sensible à la spectinomycine
C - Elle est l'indication d'un traitement minute
D - Peut se compliquer de salpingite chez la femme
E - Ne peut être mise en évidence que par inoculation sur culture cellulaire
Bonne(s) réponse(s) : D

Les chlamydiae sont responsables de 80% des stérilités tubaires et 40% des stérilités féminines globales.
Les chlamydiae peuvent être diagnostiqués rapidement par la mise en évidence des antigènes par immunofluorescence
directe ou Elisa.
Mais ces techniques sont rarement utilisées.

1133
Exclusivement sur DOC - DZ : www.doc-dz.com NADJI 85
RESIDANAT EN POCHE TOME II
Cas Clinique en QCM

Parmi les antibiotiques suivants, lequel(lesquels) est(sont) utile(s) dans le traitement des infections à chlamydia
?
A - Gentamicine
B - Pénicilline
C - Minocycline
D - Tétracycline
E - Erythromycine
Bonne(s) réponse(s) : C D E

Sont également actives les fluoroquinolones.

Un enfant de 6 ans est amené à la consultation pour examen de son cuir chevelu. Depuis 3 semaines environ des cheveux
sont tombés et une plaque poudreuse de 3 cm de diamètre non prurigineuse est apparue. A l'examen, le pourtour est
desquamatif avec des vésicules de petite taille en bordure. Certains cheveux sont cassés, ou enrobés d'une poudre grisâtre.
A l'interrogatoire, nous apprenons que cet enfant vit a la campagne dans un centre d'élevage de volailles, il y a d'autres
animaux domestiques à la maison : chiens, chats.

Pour établir le diagnostic :


A - On fait un prélèvement
B - On coupe une mèche de cheveux
C - On prélève au centre de la plaque d'alopécie des squames à l'aide d'une curette
D - On arrache au pourtour de la plaque des cheveux et on prélève des squames (à la curette)
E - Il est inutile de faire un prélèvement, le diagnostic est évident
Bonne(s) réponse(s) : A D

Un prélèvement à visée mycologique est ici nécessaire. L'évolution centrifuge des dermatophyties justifie de prélever sur le
pourtour des lésions.

Il s'agit probablement :
A - D'une pelade
B - D'une teigne à trichophyton schoënlenii
C - D'une teigne à trichophyton violaceum
D - D'une teigne à epidermophyton floccosum
E - D'une teigne à microscporum canis
Bonne(s) réponse(s) : E

Le cuir chevelu est normal dans la pelade


Les teignes trichophytiques, de transmission interhumaine, réalisent de multiples plaques de petite taille ; les cheveux cassés
très courts sont en général non visibles
L'épidermophyton ne parasite jamais phanères

Le traitement :
A - Est inutile
B - Doit comporter: griseofuline (voie orale) pendant 3 semaines + rasage des cheveux parasités et traitement
local
C - Doit comporter la recherche et le traitement des animaux atteints
D - Doit être administré aux membres de la famille atteints
E - Du patient suffit
Bonne(s) réponse(s) : B C

La teigne à microsporum canis ne se transmet pas d'homme à homme mais l'extension des lésions par auto-inoculation est
toujours possible chez le patient et fera rechercher systématiquement d'autres localisations cutanées (herpès circiné...).

La prophylaxie :
A - Consiste en un traitement de tous les animaux du voisinage
B - Consiste à éloigner l'enfant
C - Nécessite l'isolement du malade
D - Est inutile, le traitement de l'enfant étant suffisant
E - Consiste à rechercher les animaux atteints et à la traiter
Bonne(s) réponse(s) : E

Sans commentaire.

1134
Exclusivement sur DOC - DZ : www.doc-dz.com NADJI 85
RESIDANAT EN POCHE TOME II
Cas Clinique en QCM
Un homme de 45 ans vient vous consulter pour une lésion du gland évoluant depuis 10 jours. Cet homme français d'origine,
marié, cadre technico-commercial, voyage beaucoup à l'étranger et en France. Au cours de ses voyages, il a déjà contracté
une syphilis il y a 2 ans ; le traitement a consisté en 2 injections simultanées d'Extencilline®. A l'époque, la sérologie était
positive : il n'y a pas eu de nouvelle sérologie depuis. Il n'a pas d'autres antécédents de maladies sexuellement
transmissibles. Il est par ailleurs en bonne santé ; il fume 20 cigarettes et boit l'équivalent d'un litre de vin par jour,
principalement sous forme de whisky et de cocktails. Les lésions ont débuté il y a 10 jours ; elles étaient légèrement sensibles.
Il a vu un autre médecin il y a 5 jours qui lui a fait faire un examen bactériologique d'une ulcération génitale avec culture et
examen au microscope à fond noir et une sérologie de la syphilis. Il lui a également prescrit une injection unique
d'Extencilline® (2 400 000 U.). Le patient a appliqué un antiseptique de sa propre initiative.
5 jours après l'injection d'Extencilline®, il n'a observé aucune amélioration et c'est ce qui l'amène à consulter. Entre-temps, il a
eu les résultats des examens complémentaires demandés par le premier médecin :
- examen bactériologique présence de pus ; pas de diplocoques gram - ; quelques cocci Gram + ; cultures ; staphylocoque
blanc coagulase
- examen au microscope à fond noir : négatif
- sérologie syphilitique : VDRL et TPHA + 1/80ème.
A l'examen de la région génitale, vous découvrez 3 lésions réparties sur le gland et sur la zone balano-préputiale à droite. Ce
sont des ulcérations de petite taille à contours irréguliers. Leur base n'est pas indurée. Elles sont sensibles à la palpation, leur
fond est jaunâtre, légèrement purulent. On trouve dans le creux inguinal droit deux ganglions de 3 à 4 cm de diamètre,
douloureux à la pression. L'un d'eux est ramolli en son centre. Le reste de l'examen est normal ; aucune lésion muqueuse (à
part une mauvaise hygiène bucco-dentaire) ou sur le reste du tégument, pas d'hépatosplénomégalie. En reprenant
l'interrogatoire, vous apprenez :
- qu' il n'a jamais de rapport homosexuel
- qu'il a eu il y a 15 jours un rapport avec une prostituée dans un hôtel à Douala au Cameroun
- qu'il n'a jamais eu de lésion similaire du gland
- qu'il a parfois des ulcérations dans la cavité buccale (langue, gencives, joues), qualifiées d'aphtes par son médecin traitant.

L'analyse de l'ensemble des éléments cliniques et paracliniques vous permet de retenir trois hypothèses
diagnostiques plausibles :
A - Un herpès génital de primo-infection
B - Un chancre mou
C - Un chancre syphilitique
D - Une maladie de Nicolas-Favre
E - Une infection due à une association fuso-spirillaire
Bonne(s) réponse(s) : A B C

La maladie de Nicolas-Favre provoque un microchancre unique inconstant et fugace.


L'incubation est de l'ordre de 20 jours.
L'association fuso-spirillaire peut entraîner des ulcérations génitales mais le diagnostic se fait au microscope à fond noir.
Il est difficile d'écarter totalement le chancre syphilique : en effet, la clinique n'est pas toujours typique; par ailleurs, il n'est pas
précisé si le patient a appliqué l'antiseptique avant ou après les prélèvements bactériologiques. Enfin, le recul de 5 jours ne
suffit pas pour conclure à l'inefficacité de la pénicillinothérapie.

L'analyse de l'ensemble des éléments cliniques et paracliniques vous permet de retenir comme hypothèse
diagnostique principale :
A - Un herpès génital de primo-infection
B - Un chancre mou
C - Un chancre syphilitique
D - Une maladie de Nicolas-Favre
E - Une infection due à une association fuso-spirillaire
Bonne(s) réponse(s) : B

La courte incubation (5 jours) et l'aspect des lésions évoquent fortement ce diagnostic.

Parmi les données de l'interrogatoire, vous retenez en faveur de votre hypothèse diagnostique principale :
A - Les antécédents de syphilis
B - L'intoxication alcoolo-tabagique chronique
C - Les rapports avec une prostituée au Cameroun
D - Le caractère douloureux des lésions
E - L'inefficacité d'une injection unique d'Extencilline®
Bonne(s) réponse(s) : C D E

L'hémophilus ducreyi résiste à l'Extencilline® en raison de la production d'une bêta-lactamase.

Parmi les données de l'examen physique, vous retenez en faveur de votre hypothèse diagnostique principale :
A - La multiplicité des lésions génitales
B - Le caractère inflammatoire des adénopathies inguinales
C - L'unilatéralité des adénopathies
D - Le caractère douloureux des lésions génitales
E - Le caractère irrégulier des bords des ulcérations génitales
Bonne(s) réponse(s) : A B D E

Sans commentaire.
1135
Exclusivement sur DOC - DZ : www.doc-dz.com NADJI 85
RESIDANAT EN POCHE TOME II
Cas Clinique en QCM

Parmi les premiers examens complémentaires, vous retenez comme compatibles avec votre hypothèse
diagnostique principale :
A - La négativité de l'examen au microscope à fond noir
B - La présence de pus à l'examen direct (polynucléaires altérés)
C - La présence de staphylocoques blancs coagulase négative
D - La réaction du VDRL négative
E - La réaction du TPHA positive à 1/80 ème
Bonne(s) réponse(s) : A B C D E

Les staphylocoques blancs peuvent contaminer tout prélèvement cutané.


La faible positivité du TPHA est probablement en rapport avec une cicatrice sérologique de la syphilis contractée deux ans
auparavant. Mais ne permet pas d'éliminer une recontamination récente et une contrôle sérologique s'impose.

Pour confirmer votre diagnostic principal, vous allez demander :


A - Un séro-diagnostic IgM de l'herpès
B - Une culture pour recherche de bacilles de Ducreyi
C - Un test de Nelson (TPI)
D - Un séro-diagnostic des chlamydiae
E - Une ponction ganglionnaire pour un examen bactériologique
Bonne(s) réponse(s) : B

Le bacille de ducreyi est extrêmement difficile à visualiser à l'examen direct au Gram. Le diagnostic repose sur la culture sur
milieu spécifique.

Un patient âgé de 40 ans se plaint depuis 6 mois d'ulcérations buccales aphtoïdes. Depuis une dizaine de jours, il présente en
outre des bulles, des ulcérations postbulleuses et des croûtelles sur la région thoracique et dorsale médiane. L'état général
est conservé ; il n'y a aucun antécédent particulier. Au terme du premier examen, l'hypothèse d'un pemphigus vulgaire semble
la plus probable.

Cependant l'analyse des données ci-dessus peut faire discuter :


A - Un impétigo staphylococcique
B - Une toxidermie médicamenteuse
C - Une porphyrie cutanée
D - Un zona
E - Un eczéma constitutionnel
Bonne(s) réponse(s) : B

Le staphylocoque doré peut induire un impétigo bulleux compatible avec les lésions thoraciques. Il faudrait admettre alors la
coexistence de deux dermatoses indépendantes : une staphylococcie cutanée aiguë et une éruption buccale chronique.

Dans l'hypothèse du pemphigus vulgaire, l'éruption bulleuse du tronc devrait :


A - Survenir en peau saine
B - Etre déclenchée préférentiellement par la lumière
C - Etre d'aspect assez monomorphe
D - Etre prurigineuse
E - S'accompagner d'une Iymphocytose sanguine
Bonne(s) réponse(s) : A

La photosensibilité n'est pas un caractère habituel dans le pemphigus. Le prurit manque le plus souvent. Le polymorphisme
est lié à la coexistence d'éléments d'âge différent.

L'enquête étiologique de ce pemphigus vulgaire doit faire rechercher :


A - Une infection bactérienne intercurrente
B - Une entéropathie jéjunale
C - Un cancer viscéral
D - La prise de certains médicaments
E - Une maladie sexuellement transmissible
Bonne(s) réponse(s) : D

Les principales associations pathologiques notées dans le pemphigus vulgaire sont :


- le traitements médicamenteux (D-pénicillamine surtout)
- la myasthénie et le thymome.
Des observations d'association à des néoplasies profondes sont décrites mais exceptionnelles et isolées.

1136
Exclusivement sur DOC - DZ : www.doc-dz.com NADJI 85
RESIDANAT EN POCHE TOME II
Cas Clinique en QCM

Pour confirmer le diagnostic de pemphigus vulgaire, la biopsie d'une bulle cutanée est importante. L'aspect
caractéristique est :
A - Une bulle sous-épidermique
B - Une bulle intra-épidermique avec ballonisation des kératinocytes
C - Une bulle Intra-épidermique multiloculaire avec nécrose de l'épiderme
D - Une bulle Intra-épidermique avec acantholyse
E - Une bulle par nécrose de l'assise basale
Bonne(s) réponse(s) : D

Le décollement bulleux, dans le pemphigus, est dû à l'acantholyse (perte de cohésion entre les kératinocytes), elle-même liée
au dépôt local d'IgG anti-substance intercellulaire.

Dans le pemphigus vulgaire, il existe des anticorps dirigés contre :


A - La zone de la membrane basale
B - La substance intercellulaire de l'épiderme
C - Le cytoplasme des cellules de l'assise basale
D - Les desmosomes des kératinocytes
E - Les noyaux des kératinocytes
Bonne(s) réponse(s) : B

Ces anticorps sont décelés dans la peau par immunofluorescence directe et dans le sérum par immunofluorescence indirecte.

Chez ce patient le diagnostic de pemphigus a été confirmé. Il s'agit d'une forme évolutive et un traitement est
indispensable.
Parmi les thérapeutiques suivantes, quelle est celle que vous envisagez :
A - Mycostatine® per os 3 millions par jour
B - Corticothérapie générale 1,5 mg/kg/jour
C - Régime sans gluten
D - Griséofulvine 1 g par jour
E - Isoniazide 450 mg par jour
Bonne(s) réponse(s) : B

Le pemphigus évolutif justifie le recours à de fortes doses de corticoïdes en traitement d'attaque.


Le régime sans gluten n'est indiqué que dans la dermatite herpétiforme.

Une malade âgée de 72 ans, sans antécédent particulier, est hospitalisée pour l'apparition récente d'une dermatose bulleuse.
Il s'agit de bulles apparues sur un fond de lésions urticariennes. Elles s'accompagnent d'un prurit et sont de grande
dimension. Il existe une altération de l'état général avec déshydratation extracellulaire. On retrouve des bulles dans la cavité
buccale. La malade ne prenait aucun médicament avant l'apparition des lésions.

Parmi les éléments cliniques suivants, certain(s) est (sont) des critères de gravité :
A - Sexe féminin
B - Caractère prurigineux
C - Déshydratation
D - Survenue sur peau urticarienne
E - Atteinte de la muqueuse buccale
Bonne(s) réponse(s) : C E

La déshydratation reflète l'étendue des lésions cutanées suintantes ; l'atteinte buccale entrave une réhydratation et un apport
protido-calorique satisfaisants.

Quel est sur de simples arguments cliniques, le diagnostic le plus probable ?


A - Pemphigus
B - Herpès gestationnis
C - Pemphigoïde bulleuse
D - Dermite des prés
E - Dermatite herpétiforme
Bonne(s) réponse(s) : C

Le pemphigus est beaucoup plus rare que la pemphigoïde bulleuse, les bulles y siègent typiquement en peau saine, le prurit
est absent. Les bulles fragiles se rompent rapidement et l'on observe surtout des érosions post-bulleuses.
La présence de lésions buccales est notée dans 10 à 40% des séries de pemphigoïdes bulleuses.
L'âge de la patiente, la grande dimension des bulles, l'étendue des lésions sont des arguments contre la dermatite
herpétiforme.

1137
Exclusivement sur DOC - DZ : www.doc-dz.com NADJI 85
RESIDANAT EN POCHE TOME II
Cas Clinique en QCM

Parmi les signes anatomo-pathologiques suivants, un seul peut être retenu comme argument en faveur du
diagnostic évoqué :
A - Acantholyse
B - Bulle sous-épidermique
C - Bulle intra-épidermique
D - Infiltrat dermique à polynucléaires neutrophiles
E - Nécrose épidermique
Bonne(s) réponse(s) : B

L'histologie typique de pemphigoïde bulleuse révèle une bulle sous épidermique riche en éosinophiles, dont le toit est
constitué par un épiderme non acantholytique et non nécrosé.
L'infiltrat dermique sous jacent est composé en majorité de polynucléaires éosinophiles.

Parmi les signes immunologiques rencontrés dans cette affection, vous retenez :
A - Anticorps fixés anti-substance intercellulaire
B - Anticorps fixés anti-membrane basale de type IgG
C - Anticorps fixés anti-membrane basale de type IgA
D - Anticorps anti-substance intercellulaire circulants
E - Immunofluorescence directe négative
Bonne(s) réponse(s) : B

L'immunofluorescence directe retrouve des dépôts linéaires d'IgG le long de la jonction dermo-épidermique. Cet aspect n'est
pas spécifique mais est indispensable au diagnostic de la pemphigoïde bulleuse.
Les propositions (A) et (D) se rapportent au pemphigus.
C - S'observe dans la dermatite herpétiforme.

Parmi les propositions thérapeutiques suivantes, 2 peuvent être discutées chez cette malade :
A - Corticoïdes généraux seuls puis immunosuppresseurs
B - Corticoïdes généraux et immunosuppresseurs d'emblée
C - Corticoïdes locaux seuls
D - Antipaludéens de synthèse
E - Antibiotiques généraux seuls
Bonne(s) réponse(s) : A B

Les corticoïdes représentent le traitement de référence de la pemphigoïde bulleuse.


L'utilisation des immunosuppresseurs tend à se répandre car elle permet de réduire les doses de corticoïdes et donc d'en
minimiser les effets secondaires.

Quel sera le meilleur élément de surveillance de l'efficacité du traitement ?


A - Le taux des anticorps anti-peau
B - Le nombre de bulles nouvelles apparaissant chaque jour
C - Le poids de la malade
D - La disparition du prurit
E - L'éosinophilie
Bonne(s) réponse(s) : B

Les titres d'anticorps anti-membrane basale n'ont aucun intérêt évolutif

Une femme de 32 ans présente depuis 2 jours une éruption des genoux et des faces dorsales des pieds et des mains,
érythémateuse, papuleuse, non prurigineuse, faite d'éléments de 1 cm environ à centre fripé ou bulleux réalisant un aspect en
cocarde. Il existe une conjonctivite et des érosions des lèvres. Le reste de l'examen est normal. On retrouve à l'interrogatoire
plusieurs poussées identiques au retour des sports d'hiver.

Votre diagnostic est :


A - Urticaire
B - Psoriasis annulaire
C - Pemphigoïde bulleuse
D - Erythème polymorphe
E - Erythème pigmenté fixe
Bonne(s) réponse(s) : D

Eruption caractéristique par son aspect en cocardes, sa topographie symétrique, l'atteinte muqueuse.

1138
Exclusivement sur DOC - DZ : www.doc-dz.com NADJI 85
RESIDANAT EN POCHE TOME II
Cas Clinique en QCM

Le caractère récidivant du syndrome évoque deux étiologies. Lesquelles ?


A - Médicamenteuse
B - Vitaminique
C - Paranéoplasique
D - Herpétique
E - Parasitaire
Bonne(s) réponse(s) :

QUESTION ANNULEE

Quelle étiologie est la plus probable dans ce cas ?


A - Prise médicamenteuse
B - Herpès
C - Cancer profond
D - Pédiculose
E - Déficit en vitamine B12
Bonne(s) réponse(s) : B

L'herpès est une cause fréquente d'érythème polymorphe. Certains patients font un érythème polymorphe à chaque
récurrence herpétique.
C'est le cas de cette femme qui fait une poussée d'herpès labial avec le soleil des cîmes, compliquée d'érythème polymorphe.
Dans 15 % des érythèmes polymorphes récidivants on retrouve une poussée d'herpès 3 à 10 jours plus tôt.

Ce syndrome peut s'accompagner de :


A - Fièvre
B - Amylose
C - Kératite
D - Syndrome néphrotique
E - Rhumatisme articulaire aigu
Bonne(s) réponse(s) : A C

La fièvre est habituelle dans cette affection parfois très élevée (40).
Les atteintes oculaires sévères s'accompagnent de kératite soit par érosions cornéennes soit par diminution de la sécrétion
lacrymale. Ces érosions conjonctivo-palpébrales voire cornéennes peuvent laisser des séquelles sévères.

Quel examen complémentaire est indispensable ?


A - Test de dégranulation des basophiles
B - Sérologie herpétique
C - TOGD
D - Dosage de la vitamine B12
E - Aucun de ces examens
Bonne(s) réponse(s) : E

Aucun n'est indispensable.


La sérologie herpétique est de peu d'intérêt dans les récurrences ; il est vrai qu'en pratique on la fait quand même.

1139
Exclusivement sur DOC - DZ : www.doc-dz.com NADJI 85
RESIDANAT EN POCHE TOME II
Cas Clinique en QCM
Un enfant de 10 ans est amené en consultation pour des lésions du cuir chevelu apparues il y a 3 semaines. Il est l'ainé de 3
enfants : une soeur de 7 ans, un frère de 5 ans. Les lésions sont apparues au retour de vacances qu'il avait passées dans la
ferme de ses grand-parents où il a recueilli un petit chat qu'il a ramené chez ses parents.
L'examen montre 2 plaques bien limitées, prurigineuses de 2 à 3 cm de diamètre finement squameuses non cicatricielles. Les
squames sont grisâtres et se détachent facilement. Quelques cheveux sont cassés 2 à 5 mm au-dessus de l'orifice folliculaire,
d'autres viennent à la traction. Depuis 2 ans, il est traité par une pommade au goudron pour un psoriasis des coudes et des
genoux.
Une corticothérapie locale au niveau du cuir chevelu n'a pas entraîné d'amélioration. La mère est venue seule avec lui, les
autres enfants n'auraient rien. Elle-même a un examen dermatologique normal mais elle signale chez son mari la survenue
depuis une semaine d'une lésion de l'avant bras ayant tendance à s'étendre. Vous évoquez le diagnostic de teigne
microsporique.

En faveur de ce diagnostic, l'observation met en évidence :


A - Enfant
B - Alopécie non cicatricielle
C - Cheveux cassés courts
D - Squames fines
E - Lésion cutanée du père
Bonne(s) réponse(s) : A B C D E

Les plaques dues à des teignes microscopiques sont le plus souvent de grande taille (4 à 7 cm), peu nombreuses (1 à 4), à
surface recouverte de squames grisâtres hérissées de cheveux cassés courts.
L'alopécie n'est pas cicatricielle, les cheveux repoussent après le traitement.
Ces teignes sont l'apanage de l'enfant, elles guérissent spontanément à la puberté.
Microsporum canis (le plus vraisemblablement en cause ici) peut être responsable d'herpès circiné (lésion du père).

Chez ce patient quel élément permet d'éliminer formellement un psoriasis du cuir chevelu ?
A - Caractère grisâtre des squames
B - Prurit
C - Alopécie
D - Squames fines
E - Lésions bien limitées
Bonne(s) réponse(s) : C

Le psoriasis ne donne jamais d'alopécie, par contre, toute lésion squameuse du cuir chevelu agace la patiente, qui touche sa
lésion ; un prurit peut en résulter.

Quels sont le ou les examens utiles(s) soit pour confirmer le diagnostic, soit pour des raisons épidémiologiques
ou thérapeutiques ?
A - La numération formule sanguine
B - L'examen en lumière de Wood
C - Le prélèvement mycologique direct
D - La biopsie du cuir chevelu
E - La culture sur Sabouraud
Bonne(s) réponse(s) : B C E

La lumière de wood met le plus souvent en évidence une fluorescence intense des lésions.
L'examen mycologique direct montre des filaments mycéliens ; la culture sur milieu de Sabouraud permet d'identifier le
dermatophyte en 10 à 30 jours.
La biopsie est inutile.
La numération fait partie de la surveillance d'un traitement par la griséfuline.

Parmi les traitements suivants, quel est celui ou ceux que l'on peut proposer ?
A - Amphotéricine B en suspension 3 cuillères à café/jour durant 3 semaines
B - Griséofulvine 500 mg/jour durant 6 semaines
C - Mycostatine® pommade 3 applications/jour jusqu'à guérison
D - Miconazole crème 2 applications/jour jusqu'à guérison
E - Solution de Milian®, 2 applications/jour jusqu'à guérison
Bonne(s) réponse(s) : B D

Le traitement d'une teigne est double : par voie générale et locale.


L'amphotéricine B et la Mycostatine® n'ont pas d'activité sur les dermatophytes.
La solution de Milian® est un antiseptique coloré (vert) de toute façon non adapté pour le cuir chevelu ; les lésions ne sont pas
surinfectées, c'est inutile.

1140
Exclusivement sur DOC - DZ : www.doc-dz.com NADJI 85
RESIDANAT EN POCHE TOME II
Cas Clinique en QCM

Une ou plusieurs mesures complémentaires doivent être prises :


A - Faire examiner et traiter le chat
B - Examiner tous les membres de la famille
C - Faire un prélèvement mycologique à la fin du traitement
D - Eviction scolaire durant 3 mois
E - Déclaration obligatoire
Bonne(s) réponse(s) : A B C

L'enquête épidémiologique est indispensable (A et B).


Il est habituel de contrôler la guérison par un examen mycologique.
L'éviction scolaire est théoriquement la règle jusqu'à négativation de l'examen direct ; en pratique elle se discute cas par cas.

Mademoiselle H... 35 ans est hospitalisée pour la deuxième fois. Elle est tantôt abattue, tantôt excitée, déambulant, riant par
intermittence. Au moment de l'entretien, elle se dresse sur son lit et crie : "qu'on me fiche la paix, je ne suis pas une fille
publique, je ne veux pas aller au palais de justice". Elle parle avec volubilité, mais les mêmes phrases reviennent maintes fois
au cours de ses déclarations souvent débitées comme des litanies.
Elle dit qu'elle est la fille de Napoléon : on l'emprisonne, on la viole, on lui lance des poudres, nuit et jour on l'injurie à travers
les murs.
Dans les antécédents on note un accès analogue il y a 7 ans. Entre temps le fonctionnement psychique a été normal. Son
frère aîné a été traité pour un état mélancolique. L'examen neurologique est normal.

L'analyse de l'énoncé du cas vous permet d'identifier la ou les notions de :


A - Idées délirantes
B - Troubles de l'humeur
C - Excitation psychomotrice
D - Hallucinations
E - Anxiété massive
Bonne(s) réponse(s) : A B C D

Sans commentaire.

Quel diagnostic retenez-vous ?


A - Psychose hallucinatoire chronique
B - Manie délirante
C - Schizophrénie paranoïde
D - Bouffée délirante polymorphe
E - Mélancolie à forme psychotique
Bonne(s) réponse(s) : D

Sans commentaire.

En faveur du diagnostic, vous retenez :


A - Systématisation du délire
B - Caractère dominant des troubles de l'humeur
C - Antécédents de mélancolie du frère
D - Multiplicité des thèmes délirants
E - Antécédents d'accès identique
Bonne(s) réponse(s) : D E

Sans commentaire.

Le traitement de l'accès comporte :


A - Antidépresseur tricyclique
B - Neuroleptique incisif
C - Psychothérapie institutionnelle
D - Carbamazépine (Tégrétol®)
E - Neuroleptique sédatif
Bonne(s) réponse(s) :

QUESTION ANNULEE.

1141
Exclusivement sur DOC - DZ : www.doc-dz.com NADJI 85
RESIDANAT EN POCHE TOME II
Cas Clinique en QCM

L'évolution pourra être marquée par :


A - Guérison sans récidive
B - Récidive identique
C - Schizophrénie paranoïde
D - Evolution maniaco-dépressive
E - Délire chronique interprétatif
Bonne(s) réponse(s) : A B C

Sans commentaire.

Un homme de 18 ans consulte pour des lésions cutanées prurigineuses. La maladie a commencé 1 mois auparavant par un
prurit qu'il essaie de traiter par de la crème Parfenac®, anti-inflammatoire non corticoïde conseillé par son pharmacien.
Devant l'aggravation du prurit, il consulte son médecin traitant qui pose le diagnostic "d'allergie". La crème Diprosone®,
dermocorticoïde prescrite par le médecin, atténue légèrement les lésions cutanées.
Lorsque nous l'examinons, il présente des lésions de grattage non spécifiques disséminées sur les membres et la face
antérieure du tronc. Un examen minutieux des faces latérales des doigts permet de voir des sillons grisâtres. Sur le gland il
existe une lésion papuleuse excoriée. Le visage est indemne.
L'examen des zones pileuses est normal. Il présente une langue géographique. Le traitement suivant est prescrit : deux
applications d'Ascabiol® à 15 minutes d'intervalle sur tout le tégument. Le lendemain le prurit n'a pas disparu.

Chez ce patient qui présente un prurit, vous retenez en faveur d'une gale ?
A - L'existence d'une langue géographique
B - Le respect du visage
C - L'aggravation du prurit après application de Parfenac®
D - L'atténuation du prurit après application du Diprosone®
E - La présence d'une papule excoriée du gland
Bonne(s) réponse(s) : B E

L'affection respecte le cuir chevelu, la face, le dos ; elle peut atteindre le cuir chevelu chez l'enfant.
Le chancre scabieux correspond à cette papule excoriée, siègeant sur le pénis ou le scrotum.
Le Parfénac® peut être responsable d'une eczématisation.

Chez ce patient atteint de gale, les caractères du prurit n'ont pas été précisés. Habituellement, il :
A - Peut prédominer au cuir chevelu
B - S'accompagne de dermographisme
C - Est fréquemment familial
D - Peut entraîner une insomnie
E - Disparaît 30 mn après application d'Ascabiol®
Bonne(s) réponse(s) : C D

Le dermographisme est l'apanage de l'urticaire.


La contagion et la recrudescence nocturne du prurit sont deux caractères fondamentaux. Après application d'Ascorbiol® le
prurit est rapidement interrompu (24 h environ) ; il peut revenir secondairement sans pour autant signifier un échec
thérapeutique ; il faut prévenir les patients afin d'éviter les applications itératives irritantes de scabicide.

Les données de l'examen clinique sont incomplètes chez ce patient atteint de gale. On aurait pu voir :
A - Des vésicules perlées dans les espaces interdigitaux
B - Une impétiginisation
C - Des sarcoptes le long de poils
D - Des lésions de grattage sur les fesses
E - Des papules de prurigo
Bonne(s) réponse(s) : A B D E

Le sarcopte femelle vit dans la couche cornée ; après fécondation, il reste en surface mais ne s'accroche pas aux poils.
D et E sont des lésions induites par le grattage donc non spécifiques. La vésicule perlée contient la femelle qui pond ses
oeufs.
La gale se surinfecte fréquemment chez les gens à hygiène médiocre.

Si le prurit n'a pas disparu 24 heures après les deux applications d'Ascabiol® il faut :
A - Récuser le diagnostic de gale
B - L'attribuer à une résistance du sarcopte vis à vis de l'Ascabiol®
C - Continuer les applications d'Ascabiol® jusqu'à disparition du prurit
D - S'abstenir, car il peut s'agir d'une évolution normale
E - Adjoindre un antiparasitaire per os
Bonne(s) réponse(s) : D

Le prurit cède en 24 à 48 heures ; il n'y a pas de traitement par voie orale de la gale.

1142
Exclusivement sur DOC - DZ : www.doc-dz.com NADJI 85
RESIDANAT EN POCHE TOME II
Cas Clinique en QCM

La traitement prescrit doit être complété par :


A - Raser les poils pubiens
B - Traiter simultanément l'entourage
C - Désinfecter les draps
D - Déclaration à la DDASS
E - Faire examiner les animaux domestiques par un vétérinaire
Bonne(s) réponse(s) : B C

Traiter l'entourage est indispensable de même que la désinfection de la literie, des vêtements, des gants, des chaussures
(poudre Aphtiria)®.
La gale est une parasitose humaine, le parasite a un cycle cutané humain complet.

Une femme de 25 ans consulte pour des lésions douloureuses des jambes apparues il y a 8 jours. L'affection a commencé par
des douleurs articulaires aux chevilles et aux genoux, d'horaire inflammatoire. A l'examen on constate sur les jambes des
nodules fermes rouges, chauds sensibles à la pression, de 3 à 4 cm de diamètre, siègeant surtout en regard des crêtes
tibiales, débordant en bas sur la cheville, mais s'arrêtant en haut au niveau de la rotule. Les nodules sont apparus en
plusieurs poussées. Le plus ancien est violacé avec une auréole jaune. L'articulation de la cheville droite est augmentée de
volume.
Il existe par ailleurs une fièvre à 38,5°C, avec des variations de 1 degré dans la journée. Le reste de l'examen clinique est
normal.
Vous portez le diagnostic d'érythème noueux et en reprenant l'interrogatoire, vous apprenez que cette patiente n'a aucun
antécédent d'affection systémique, aucun antécédent d'affection pulmonaire ou digestive chronique. Vous apprenez
également qu'il y a eu un discret épisode diarrhéique 5 jours avant le début de cette poussée d'érythème noueux.

Dans l'énoncé, vous pouvez rattacher au syndrome d'érythème noueux :


A - Nodules dermo-hypodermiques
B - Arthralgies
C - Localisations à la face antéro-externe de jambe
D - Fièvre
E - Diarrhée
Bonne(s) réponse(s) : A B C D

L'éruption de l'érythème noueux est accompagnée voire précédée de quelques jours par des arthralgies et une fièvre ; une
rhinopharyngite, des douleurs abdominales et malaise peuvent également se voir. La diarrhée n'appartient pas au syndrome
mais peut être une étiologie.

Devant cet érythème noueux, un ou plusieurs signes ont une valeur d'orientation étiologique :
A - Bilatéralité des lésions
B - Femme jeune
C - Association à des arthralgies
D - Association à une fièvre
E - Episode diarrhéique
Bonne(s) réponse(s) : E

La femme jeune est le terrain électif sur lequel survient l'érythème noueux.

Quelle que soit votre hypothèse diagnostique, il faut, au cours d'un érythème noueux, envisager
systématiquement des étiologies infectieuses. Quelles sont les trois plus fréquentes ?
A - Infection staphylococcique
B - Infection streptococcique
C - Infection à Yersinia pseudotuberculosis
D - Infection à Mycobactérium tuberculosis
E - Infection à Treponema pallidum
Bonne(s) réponse(s) : B C D

Le staphylocoque et le tréponème ne sont pas responsables d'érythème noueux.


C'est lors de la primo-infection tuberculeuse que l'on voit l'érythème noueux.

1143
Exclusivement sur DOC - DZ : www.doc-dz.com NADJI 85
RESIDANAT EN POCHE TOME II
Cas Clinique en QCM

On doit de même envisager systématiquement parmi les hypothèses non infectieuses :


A - Maladie de Crohn
B - Rectocolite hémorragique
C - P.A.N.
D - Cancer du pancréas
E - Sarcoïdose
Bonne(s) réponse(s) : A B E

La sarcoïdose dans le cadre du syndrome de Lِfgren (érythème noueux, arthralgies, adénopathies hilaires bilatérales) est la
première cause.
Les maladies néoplasiques connues pour donner des érythèmes noueux sont les hémopathies (LMC, LLC, LAM). La PAN
peut donner des nodules cutanés siégeant sur les trajets artériels des membres inférieurs faits d'éléments d'âges différents,
douloureux avec tendance à la nécrose et à l'ulcération.

L'enquête étiologique devant un tableau d'érythème noueux comporte systématiquement :


A - Biopsie de l'érythème noueux
B - Radiographie thoracique
C - VS
D - Intradermoréaction à la tuberculine
E - Recherche de protéinurie
Bonne(s) réponse(s) : B C D

La biopsie montre des infiltrats inflammatoires péri-vasculaires dermo-hypodermiques n'apportant aucun élément de
présomption histologique. La biopsie est justifiée en cas de doute diagnostic.

Chez cette patiente en particulier, quel est le 1er examen complémentaire à demander pour le diagnostic ?
A - Hémoculture
B - Lavement baryté
C - Colonoscopie
D - Sérodiagnostic de Yersinia enterocolitica
E - Examen parasitologique des selles
Bonne(s) réponse(s) : D

Compte tenu des antécédents de diarrhée il est nécessaire de rechercher une infection à Yersinia enterocolitica (responsable
de diarrhée fébrile avec douleurs abdominales).
Il faut faire une coproculture et une sérologie.

Si l'étiologie précédemment évoquée n'est finalement pas retenue on devra discuter, en raison de sa fréquence,
le syndrome de Lِfgren. Il est caractérisé par l'association à l'érythème noueux de :
A - Uvéite
B - Adénopathies hilobronchiques bilatérales
C - Paralysie faciale périphérique
D - Sérologie de mycoplasme positive
E - Adénopathies épitrochléennes bilatérales
Bonne(s) réponse(s) : B

Sans commentaire.

Si aucune étiologie n'a finalement été retrouvée au terme de vos explorations complémentaires, quelle sera
votre attitude thérapeutique ?
A - Corticothérapie générale
B - Traitement par tétracyclines
C - Corticothérapie locale
D - Traitement antituberculeux
E - Repos allongé
Bonne(s) réponse(s) : E

Très souvent aucune étiologie n'est retrouvée ; il n'y a donc pas de traitement étiologique. Le traitement de l'érythème noueux
associe le repos et les antalgiques ; on peut être amené à prescrire des anti-inflammatoires non stéroïdiens.

1144
Exclusivement sur DOC - DZ : www.doc-dz.com NADJI 85
RESIDANAT EN POCHE TOME II
Cas Clinique en QCM
Une fillette de 5 ans est adressée en septembre au retour des vacances annuelles passées au bord de la mer, pour une
dermatose extensive évoluant depuis 5 semaines, ayant résisté à des traitements locaux associant corticoïdes et Nystatine®
pommade. L'examen de cette fillette en bon état général, sans antécédents médico-chirurgicaux, révèle une dermatose
polymorphe prédominante à la région faciale péri narinaire, s'étendant aux jambes, faites de médaillons érythémateux et
squamo croûteux à extension centrifuge et d'autres plus récents bulleux et pustuleux, fugaces, évoluant vers le stade
croûteux. Son petit frère de 3 ans aurait présenté à minima les mêmes phénomènes il y a 6 semaines, résolutifs sous simple
traitement antiseptique local.

Quel(s) diagnostic(s) évoquer ?


A - Trichophytie cutanée
B - Toxidermie
C - Impétigo
D - Psoriasis
E - Candidose cutanée
Bonne(s) réponse(s) : C

Le diagnostic est évident sur :


- notion de contamination par le frère
- localisation périnarinaire des lésions avec auto-inoculation sur les membres inférieurs
- aspect bulleux puis croûteux des lésions
- résistance évidente à des traitements non adaptés.

Quel(s) examen(s) complémentaire(s) pratiquer ?


A - Hémocultures
B - Examen mycologique cutané
C - Examen bactériologique cutané
D - Biopsie d'une bulle
E - Examen parasitaire cutané
Bonne(s) réponse(s) : C

Il retrouve du streptocoque (environ 30 % des cas) ou du staphylocoque (environ 75 % des cas), ou une infection mixte
(environ 15-20 % des cas).

Quel(s) traitement(s) proposer ?


A - Griséofulvine per os
B - Erythromycine per os
C - Antisepsie locale
D - Nystatine per os
E - Corticoïdes locaux
Bonne(s) réponse(s) : B C

Le traitement de l'impétigo associe un traitement local et général par des antibiotiques actifs sur le streptocoque et le
staphylocoque.

Quel(s) examen(s) de contrôle pratiquer après la guérison ?


A - Albuminurie
B - Dosage des transaminases
C - Vitesse de sédimentation
D - Electrocardiogramme
E - Examen cytobactériologique des urines
Bonne(s) réponse(s) : A

Certains impétigos peuvent se compliquer de glomérulonéphrite aiguë post-streptococcique ; par contre il n'y a pas de
rhumatisme articulaire aigu dans les suites.

Quel(s) caractère(s) appartien(en)t à l'affection en cause ?


A - Transmission interhumaine
B - Contagiosité importante
C - Transmission héréditaire autosomique dominante
D - Affection dysimmunitaire
E - Peut être primitive ou secondaire à une autre affection
Bonne(s) réponse(s) : A B E

L'affection est contagieuse ; le patient peut s'auto-inoculer à distance des lésions faciales initiales et peut transmettre
l'affection à d'autres enfants.
Nombreuses dermatoses peuvent s'impétiginiser ; tout impétigo de l'adulte dans une localisation inhabituelle fera chercher
une autre dermatose pré-éxistante (gale, eczéma, varicelle, herpès...).

1145
Exclusivement sur DOC - DZ : www.doc-dz.com NADJI 85
RESIDANAT EN POCHE TOME II
Cas Clinique en QCM
Une femme de 25 ans vous consulte en automne pour une lésion pigmentée de la jambe qu'elle a remarquée depuis moins
d'un an. La lésion est indolore mais la patiente s'inquiète de cette tache qui lui parait avoir grossi depuis ses dernières
vacances d'été sur la côte d'azur.
Vous notez une tache brunâtre de 8 mm de diamètre, sans relief, sur la face externe du mollet.
A l'examen attentif la coloration n'apparaît pas tout à fait homogène, avec des zones brun clair et brun foncé, les limites sont
nettes mais irrégulières.

Quel est le premier diagnostic auquel vous devez penser en fonction de ses implications pronostiques ?
A - Tache "café au lait"
B - Verrue séborrhéïque
C - Mélanome malin superficiel extensif
D - Naevus pigmentaire
E - Epithélioma basocellulaire tatoué
Bonne(s) réponse(s) : C

Sans commentaire.

Parmi les caractères suivants, vous retrouvez en faveur de cette hypothèse diagnostique ?
A - L'indolence
B - La polychromie
C - L'irrégularité des bords
D - L'augmentation de taille récente
E - L'absence d'adénopathie
Bonne(s) réponse(s) : A B C D

Tous sont des caractéristiques des SSM.


A ce stade, il est extrêmement probable s'il s'agisse d'un SSM et que les aires ganglionnaires soient libres.

Parmi les caractères suivants, le(s)quel(s) va(vont) à l'encontre d'un diagnostic de lésion maligne ?
A - L'absence de douleur
B - L'absence de saignement
C - L'absence de relief
D - L'absence de naevus préexistant
E - Aucun des caractères précités
Bonne(s) réponse(s) : E

- La plupart des mélanomes malins ne se développent pas sur naevus pré-existant.


- Lors de la phase d'extension superficielle, le mélanome est une lésion plane.
- Le saignement est un signe observé sur des mélanomes présentant un nodule, plus on moins ulcéré saignotant, ce qui est
tardif.

Parmi les attitudes suivantes, laquelle(lesquelles) vous paraît(paraissent) justifiée(s), en fonction des seules
données cliniques ?
A - Abstention et surveillance
B - Détruire la totalité de la lésion par électrocoagulation
C - Faire une ponction à l'aiguille pour étude cytologique
D - Faire enlever d'emblée la totalité de la lésion
E - Faire une biopsie en bordure de la lésion
Bonne(s) réponse(s) : D

La lésion est de petite taille : il est plus simple de l'ôter en totalité.


D'autre part, une biopsie se ferait au centre de la lésion.
Ce geste biopsique n'est pas contre-indiqué en cas de suspicion de mélanome ; on le réserve aux lésions douteuses de
grande taille.

L'étude anatomopathologique montre qu'il s'agit d'un naevus jonctionnel. Parmi les propositions suivantes
concernant le naevus jonctionnel, choisissez la ou les proposition(s) vraie(s) :
A - Tumeur bénigne du système pigmentaire
B - Lésion susceptible d'évoluer vers un mélanome malin
C - Tumeur à malignité purement locale
D - Tumeur maligne grave
E - Lésion comportant des thèques intra-épidermiques au contact de la basale
Bonne(s) réponse(s) : A B E

Le naevus jonctionnel se caractérise par des thèques mélanocytaires intra-épidermiques faisant à la jonction dermo-
épidermique saillie dans le derme papillaire.
La présence de cellules naeviques isolées dans la moitié supérieure de l'épiderme doit être considérée comme un signe de
transformation.
1146
Exclusivement sur DOC - DZ : www.doc-dz.com NADJI 85
RESIDANAT EN POCHE TOME II
Cas Clinique en QCM
Une femme de 35 ans, employée de bureau, vous consulte pour une dermatose non prurigineuse évoluant par poussées
depuis l'âge de 32 ans. D'après la patiente la poussée actuelle serait liée à des ennuis familiaux récents.
A l'examen, vous notez la présence de plaques érythémato-squameuses multiples, siégeant aux jambes, dans la région
lombaire, le tronc et au cuir chevelu sans entraîner d'alopécie. La patiente ne présente pas de signes généraux.

Quel est votre diagnostic ?


A - Eczéma
B - Lichen
C - Pityriasis rosé de Gibert
D - Psoriasis
E - Pityriasis versicolor
Bonne(s) réponse(s) : D

Le tableau est caractéristique de psoriasis.

Dans la pathologie évoquée il n'est pas exceptionnel d'observer :


A - Un oedème des membres inférieurs
B - Une atteinte des grands plis
C - Une atteinte conjonctivale
D - Des déformations articulaires périphériques
E - Un hippocratisme digital
Bonne(s) réponse(s) : B D

Le psoriasis peut atteindre les plis, se manifestant par un intertrigo chronique avec surtout atteinte des grands plis (inguinaux,
sous mammaires, espace interfessier). La muqueuse conjonctivale n'est pas atteinte, contrairement aux muqueuses buccales
et génitales qui peuvent présenter des lésions (plaques érythémateuses du gland, langue géographique, aspect lichénoïde
des muqueuses jugales et gingivales).
D peut se voir dans les arthropathies psoriasiques, plus fréquentes chez l'homme.

Quel(s) examen(s) complémentaire(s) demanderez-vous ?


A - Un prélèvement bactériologique cutané
B - Un prélèvement viral cutané
C - Un examen cutané en immunofluorescence directe
D - Un prélèvement mycologique cutané
E - Aucune des propositions ci-dessus
Bonne(s) réponse(s) : E

L'examen qui confirmerait un diagnostic cliniquement évident est l'examen histologique d'une biopsie. Il est inutile devant ce
tableau.

Votre traitement comportera :


A - Pénicilline par voie orale
B - PUVA
C - Anti-inflammatoires non stéroïdiens
D - Antimycosiques locaux
E - Dérivés synthétiques de la vitamine A per os
Bonne(s) réponse(s) : B

Il s'agit d'un psoriasis étendu ; la puvathérapie est tout à fait adapté.

Une femme de 25 ans consulte pour des lésions douloureuses des jambes apparues il y a 8 jours. L'affection à commencé par
des douleurs articulaires aux chevilles et aux genoux, d'horaire inflammatoire. A l'examen, on constate sur les jambes des
nodules fermes rouges, chauds sensibles à la pression, de 3 à 4 cm de diamètre siègeant surtout en regard des crêtes
tibiales, débordant en bas la cheville, mais s'arrêtant en haut au niveau de la rotule. Les nodules sont apparus en plusieurs
poussées.Le plus ancien est violacée avec une auréole jaune. L'articulation de la cheville droite est augmentée de volume. Il
existe par ailleurs une fièvre à 38,5°C, avec des variations de 1 degré dans la journée. Le reste de l'examen clinique est
normal. Vous portez le diagnostic d'érythème noueux et en reprenant l'interrogatoire, vous apprenez que cette patiente n'a
aucun antécédent d'affection systémique, pulmonaire ou digestive chronique. Vous apprenez également qu'il y a eu un discret
épisode diarrhéique 5 jours avant le début de cette poussée d'érythème noueux.

A partir des données l'érythème noueux se traduit par :


A - Nodules dermo-hypodermiques
B - Arthralgies
C - Localisations à la face antéro-externe de jambe
D - Fièvre
E - Diarrhée
Bonne(s) réponse(s) : A B C D

1147
Les localisations habituelles sont les crêtes tibiales, les faces antéro-internes et antéro-externes de jambe.
Exclusivement sur DOC - DZ : www.doc-dz.com NADJI 85
RESIDANAT EN POCHE TOME II
Cas Clinique en QCM

Devant cet érythème noueux, un ou plusieurs signes ont une valeur d'orientation étiologique :
A - Bilatéralité des lésions
B - Femme jeune
C - Association à des arthralgies
D - Association à une fièvre
E - Episode diarrhéique
Bonne(s) réponse(s) : E

Le terrain, (femme jeune), le caractère bilatéral, l'association à un fébricule et à des arthralgies sont des caractères habituels
de tout érythème noueux quelque soit la cause.

Quelles sont les trois étiologies infectieuses les plus fréquentes à l'origine d'un érythème noueux ?
A - Infection staphylococcique
B - Infection streptococcique
C - Infection à yersinia pseudo-tuberculosis
D - Infection à myco-bactérium tuberculosis
E - Infection à treponema pallidum
Bonne(s) réponse(s) : B C D

L'étiologie infectieuse la plus courante est le streptocoque. Les yersinioses (yersinia enterolitica, yersinia pseudo-tuberculosis)
sont une étiologie rare. Seule la primo-infection tuberculeuse peut s'accompagner d'érythème noueux et elle concerne
maintenant surtout les immigrés.

On doit de même envisager systématiquement parmi les hypothèses non infectieuses :


A - Maladie de Crohn
B - Recto-colite hemorragique
C - P.A.N.
D - Cancer du pancréas
E - Sarcoïdose
Bonne(s) réponse(s) : A B E

Le syndrome de Lِfgren est l'étiologie la plus fréquente d'érythème noueux. Les entéropathies chroniques (RCH, Crohn)
peuvent s'accompagner d'érythème noueux (dans 1 à 10 % des cas). Dans la P.A.N. on peut observer des nodules cutanés
siégeant sur les trajets artériels des membres inférieurs avec tendance à la nécrose et à l'ulcération ; il ne s'agit pas d'un
érythème noueux.

L'enquête étiologique devant un tableau d'érythème noueux comporte systématiquement :


A - Biopsie de l'érythème noueux
B - Radiographie thoracique
C - V.S
D - Intradermo-réaction à la tuberculine
E - Recherche de protéinurie
Bonne(s) réponse(s) : B D

L'histologie cutanée est la plupart du temps non spécifique, montrant des infiltrats inflammatoires péri-vasculaires dermo-
hypodermiques. La VS fait partie du bilan mais n'est pas spécifique évidemment d'une étiologie particulière.

Chez cette patiente particulière quel est le 1er examen complémentaire à demander pour le diagnostic ?
A - Hémoculture
B - Lavement baryté
C - Colonoscopie
D - Sérodiagnostic de yersinia enterocolitica
E - Examen parasitologique des selles
Bonne(s) réponse(s) : D

Il faut demander une coproculture et un sérodiagnostic des yersinioses.

1148
Exclusivement sur DOC - DZ : www.doc-dz.com NADJI 85
RESIDANAT EN POCHE TOME II
Cas Clinique en QCM

Si l'étiologie précédemment évoquée n'est finalement pas retenue on devra discuter en raison de sa fréquence
le syndrome de Lِfgren. Il est caractérisé par l'association à l'érythème noueux de :
A - Uvéite
B - Adénopathies hilobronchiques bilatérales
C - Paralysie faciale périphérique
D - Sérologie de mycoplasme positive
E - Adénopathies épitrochléennes bilatérales
Bonne(s) réponse(s) : B

La sarcoïdose peut s'accompagner d'uvéite, de paralysie faciale, d'adénopathies périphériques mais ces manifestations
n'appartiennent pas au syndrome de Lِfgren.

Si aucune étiologie n'a finalement été retrouvée au terme de vos explorations complémentaires, quelle sera
votre attitude thérapeutique ?
A - Corticothérapie générale
B - Traitement par tétracyclines
C - Corticothérapie locale
D - Traitement anti-tuberculeux
E - Repos allongé
Bonne(s) réponse(s) : E

Le seul traitement de l'érythème noueux (hormis un éventuel traitement étiologique) est le repos au lit et les antalgiques si
nécessaire.

Un homme de 40 ans, marié, consulte pour une érosion du gland indolore, à base ferme, unique et accompagnée d'une
adénopathie homolatérale ferme non inflammatoire apparue depuis 4 jours. Il admet avoir eu des rapports extraconjugaux à
deux reprises, il y a un an avec une prostituée et il y a 8 jours avec une collègue de bureau. Il se dit inquiet d'avoir utilisé à
plusieurs reprises, il y a 3 ans des toilettes publiques fréquentées par des homosexuels, mais il nie absolument avoir eu des
rapports à cette occasion.

Quel est le diagnostic le plus probable ?


A - Chancre mou
B - Herpès
C - Chancre syphilitique
D - Aphte
E - Ulcération gonococcique
Bonne(s) réponse(s) : C

Le caractère de la lésion : exulcération (caractère superficiel), à base ferme, indolore, accompagnée d'une adénopathie non
inflammatoire évoque avant tout un chancre syphilitique. Le gonocoque ne donne pas de chancre.

Si l'interrogatoire est fiable, et la durée d'incubation standard, qui peut l'avoir contaminé ?
A - La collègue de bureau
B - L'épouse
C - La prostituée
D - Les sièges de toilettes
E - Aucune des propositions précédentes
Bonne(s) réponse(s) : B

L'incubation est de 3 semaines.

En l'absence de traitement immédiat de ce patient, on pourrait observer à l'évolution :


A - Epididymite
B - Alopécie
C - Réaction allergique à un traitement ultérieur
D - Lésions linguales
E - Polyadénopathie
Bonne(s) réponse(s) : B D E

L'absence de traitement conduit à un tableau de syphilis secondaire 6 à 8 semaines après le chancre.

1149
Exclusivement sur DOC - DZ : www.doc-dz.com NADJI 85
RESIDANAT EN POCHE TOME II
Cas Clinique en QCM

Si vous instituez le traitement efficace en dose unique, dans quel délai le risque de contamination disparait-il ?
A - Immédiatement
B - En 3 jours
C - En 1 mois
D - En 6 mois
E - En 1 an
Bonne(s) réponse(s) : B

C'est le temps qu'il faut pour que les tréponèmes disparaissent du chancre.

Quelle(s) raison(s) peut-il y avoir d'instituer une surveillance de ce patient ?


A - Efficacité inconstante du traitement
B - Risque de rechute même après traitement efficace
C - Déterminer la fin de contagiosité
D - Dépister une recontamination
E - L'empêcher de se réexposer au risque vénérien
Bonne(s) réponse(s) : D E

Le tréponème ne résiste pas à la pénicilline correctement administrée. "Empêcher" est un bien grand mot ! au moins l'informer
et lui donner des conseils de protection.

Une femme de 55 ans consulte pour une lésion pigmentée de 1,5 cm présente depuis plusieurs années, au niveau de la crête
tibiale. La lésion est de coloration irrégulière brun noir, avec une périphérie plane et une composante centrale en relief.
L'examen clinique retrouve plusieurs autres lésions pigmentaires planes d'allure bénigne au niveau du dos sans modification
récente.
L'examen clinique est par ailleurs normal, en particulier ne montre pas d'adénopathie ni de splénomégalie. Par ailleurs,
l'interrogatoire révèle qu'il s'agit d'une femme nullipare, sportive (cyclisme + tennis) fumant un demi paquet de cigarettes par
jour.

Devant une tumeur pigmentée il faut évoquer systématiquement :


A - Naevus naevocellulaire
B - Angiome thrombosé
C - Verrue séborrhéïque
D - Mélanome malin
E - Epithélioma baso-cellulaire pigmenté
Bonne(s) réponse(s) : A B C D E

On peut rajouter certaines kératoses actiniques, les histiocytofibromes pigmentés.

Quelle est la conduite à tenir vis-à-vis de la lésion de cette patiente ?


A - Electrocoagulation
B - Biopsie de la partie la plus en relief
C - Exérèse de l'ensemble de la lésion
D - Simple surveillance en l'absence de signes fonctionnels
E - Biopsie à la jonction peau saine peau pigmentée
Bonne(s) réponse(s) : C

La très forte suspicion clinique de mélanone malin autorise à une éxérèse d'emblée totale.

Le pronostic du mélanome malin en premier lieu est lié à :


A - La localisation
B - La présence d'ulcération
C - L'épaisseur de la tumeur mesurée en millimètres
D - Le nombre de mitoses
E - l'intensité de la pigmentation
Bonne(s) réponse(s) : C

C'est le critère pronostic le plus important avec le chiffre de 0,75 mm en-dessous duquel les métastases sont très rares. Plus
l'épaisseur est importante, plus le pronostic est défavorable.
A, B, D - Sont également des facteurs pronostic mais de moindre importance.

1150
Exclusivement sur DOC - DZ : www.doc-dz.com NADJI 85
RESIDANAT EN POCHE TOME II
Cas Clinique en QCM

Parmi les facteurs de risque du mélanome malin vous retenez :


A - Le tabagisme
B - L'exposition solaire
C - L'exercice physique
D - Une peau claire
E - La nulliparité
Bonne(s) réponse(s) : B D

L'exposition solaire intense semble être un facteur de risque, et non l'accumulation d'expositions répétées (comme c'est le cas
pour les épithéliomas). Le type de peau est également un facteur important.

Un homme de 32 ans, pâtissier, consulte pour un furoncle de l'avant-bras gauche, La lésion forme une tuméfaction nodulaire
rouge violacée, douloureuse à la pression, dont le sommet est marqué par une dépression nécrotique d'où sort une goutte de
pus, Cette lésion est la cinquième d'une série de lésions identiques ayant débuté trois mois auparavant et dont les cicatrices
déprimées sont encore visibles sur le thorax (2), sur la base du cou (1), sur la région temporo-maxillaire (1). L'état général est
conservé, il n'y a pas de fièvre. Le patient pèse 78 kg pour 1.72 m. Il n'y a pas d'adénopathie axillaire et l'examen somatique
est sans particularité.

Anatomiquement, le furoncle est une lésion qui touche :


A - La glande sudorale apocrine
B - Le follicule pilo-sébacé
C - La glande sudorale eccrine
D - Les muqueuses
E - N'importe lequel de ces gîtes
Bonne(s) réponse(s) : B

Le furoncle est une infection staphylococcique d'un follicule pilo-sébacé.

L'agent microbien du furoncle est :


A - Le streptocoque bêta hémolytique
B - Un bacille à gram négatif
C - Le staphylococcus epidermidis
D - Le staphylococcus aureus
E - Propionibacterium acnès
Bonne(s) réponse(s) : D

Evident.

Les furoncles peuvent révéler une anomalie systémique. Laquelle ?


A - Hypertension artérielle
B - Insuffisance rénale
C - Hyposidérémie
D - Diabète
E - Lupus érythémateux aigu disséminé
Bonne(s) réponse(s) : D

Les infections cutanées, dont les furoncles, sont plus fréquentes et volontiers récidivantes chez les sujets diabétiques. Il est
important de vérifier la glycémie des patients ayant des furoncles à répétition.

Cocher dans cette liste le(s) moyen(s) thérapeutique(s) envisageable(s) pour le furoncle du bras présenté dans
le cas clinique :
A - Anti-inflammatoire non stéroïdien par voie systémique
B - Pommade antibiotique
C - Pommade corticoïde
D - Pansement humide avec antiseptique
E - Excision chirurgicale
Bonne(s) réponse(s) : B D

Il ne faut jamais donner d'anti-inflammatoires non stéroïdiens dans une infection cutanée pour ne pas risquer une extension
de l'infection avec une symptomatologie abâtardie. Une antibiothérapie générale n'est pas indiquée (pas d'adénopathie, pas
de fièvre, pas de terrain particulier et le furoncle n'est pas sur la face).

1151
Exclusivement sur DOC - DZ : www.doc-dz.com NADJI 85
RESIDANAT EN POCHE TOME II
Cas Clinique en QCM

Quelle mesure prophylactique et hygiéno-diététique faut-il proposer à ce malade :


A - Repos au lit
B - Régime sans sel
C - Cure thermale
D - Arrêt de travail
E - Déclaration obligatoire de la maladie
Bonne(s) réponse(s) : D

Un arrêt de travail est indispensable compte-tenu de la profession du patient.

Pour éviter la récidive des furoncles on peut proposer :


A - Antibiothérapie générale adaptée au germe
B - Vaccination spécifique
C - Traitement antiseptique et/ou antibiotique des gîtes microbiens
D - Injection de gammaglobulines tous les 15 jours pendant 3 mois
E - Traitement de restauration immunitaire
Bonne(s) réponse(s) : C

Les principaux gîtes à traiter sont : les narines, les conduits auditifs, l'ombilic, le périné, l'anus, les cicatrices d'anciens
furoncles.

Un homme de 40 ans, représentant de commerce présente une éruption légèrement infiltrée, formée de petits éléments rosés
de 5 à 10 mm de diamètre, squameux au niveau du tronc et des membres supérieurs. L'examen clinique montre par ailleurs
des adénopathies de petite taille, au niveau des aires cervicales. L'examen de la muqueuse est normal, il n'y a pas d'hépato-
splénomégalie, pas de fièvre, l'état général est bon. Six semaines auparavant, le patient a eu un écoulement purulent uréthral
pour lequel un traitement par cycline à la dose d'1 g par jour a été prescrit, mais n'a été suivi que 2 jours par le patient, en
raison d'une amélioration rapide. Une semaine avant l'éruption, le patient a repris ce traitement de son propre chef pendant 2
jours, en raison d'un épisode grippal.

Devant l'aspect dermatologique de l'éruption quel sont les deux diagnostics que vous évoquez ?
A - Psoriasis en goutte
B - Toxidermie
C - Erythrasma
D - Syphilis secondaire
E - Varicelle
Bonne(s) réponse(s) : A D

Les toxidermies ne donnent habituellement pas ce type de lésions maculo-papuleuses squameuses éparses non confluantes.
L'érythrasma est un intertrigo inguinal de couleur chamois. La varicelle donne une éruption vésiculeuse.

Quel examen paraclinique est susceptible de préciser le diagnostic étiologique vraisemblablement en cause
dans ce cas ?
A - Biopsie cutanée
B - Sérologie Chlamydia
C - VDRL
D - Uricémie
E - Test de transformation lymphoblastiques aux cyclines
Bonne(s) réponse(s) : C

Il faut bien sur évoquer une syphilis secondaire et la séologie est le premier examen à demander.

Les examens complémentaires suivants, peuvent avoir un intérêt chez ce patient, sauf un. Lequel ?
A - Coloration de Gram sur frottis urétral
B - Sérologie HIV
C - TPHA
D - Immunofluorescence directe antichlamydia sur frottis urétral
E - Test de Nelson
Bonne(s) réponse(s) : E

Il peut être intéressant de vérifier la persistance de l'uréthrite (leucocytes sur le gram et peut-être micro-organismes comme
gonocoque, trichomonas...) et de retrouver le germe responsable ; (traitement non suivi). Il faut vérifier la sérologie HIV
(terrain). Le test de Nelson, complexe, coûteux, (test d'immobilisation des tréponèmes) n'a quasiment plus d'indications.

1152
Exclusivement sur DOC - DZ : www.doc-dz.com NADJI 85
RESIDANAT EN POCHE TOME II
Cas Clinique en QCM

Parmi les antibiotiques suivants, lequel n'a aucune efficacité dans le traitement de la maladie responsable de
l'éruption ?
A - Pénicilline G
B - Tétracyclines
C - Spectinomycine
D - Erythromycine
E - Benzathine-Pénicilline
Bonne(s) réponse(s) : C

La trobicine (spectinomycine) donnée en traitement des uréthrites gonococciques ne peut décapiter une syphilis contactée en
même temps.

Parmi les affirmations suivantes, concernant l'infection à Chlamydia, laquelle est vraie ?
A - Elle n'est jamais associée à une infection à gonocoque
B - Elle est sensible à la Spectinomycine
C - Elle est l'indication d'un traitement minute
D - Elle peut se compliquer de salpingite chez la femme
E - Elle ne peut être mise en évidence que par inoculation sur culture cellulaire
Bonne(s) réponse(s) : D

Au contraire, elle est assez souvent associée à une gonococcie. La spectinomycine n'a aucune action sur les chlamydias et
les mycoplasmes. Le traitement (qui tend actuellement à être raccourci) est d'au moins 10-12 jours chez l'homme et 15 jours-3
semaines chez la femme. On peut parfois mettre le micro-organisme en évidence par immuno fluorescence directe, sur les
sécrétions génitales.

Parmi les antibiotiques suivants, lequel ou lesquels sont utiles dans le traitement des infections à Chlamydia ?
A - Gentamycine
B - Pénicilline
C - Minocycline
D - Tétracycline
E - Erythromycine
Bonne(s) réponse(s) : C D E

Les macrolides, les tétracyclines et plus récemment les quinolones de nouvelle génération (fluoroquinolones) sont les
antibiotiques actifs sur les chlamydias.

Un homme de 18 ans consulte pour des lésions cutanées prurigineuses. La maladie a commencé 1 mois auparavant par un
prurit qu'il essaie de traiter par de la crème Parfenac®, anti-inflammatoire non corticoïde conseillée par son pharmacien.
Devant l'aggravation du prurit, il consulte son médecin traitant qui fait le diagnostic d'"allergie". La crème Diprosone®,
dermocorticoïde prescrite par le médecin, atténue légèrement les lésions cutanées. Lorsque nous l'examinons, il présente des
lésions de grattage non spécifiques disséminées sur les membres et la face antérieure du tronc. Un examen minutieux des
faces latérales des doigts permet de voir des sillons grisâtres. Sur le gland il existe une lésion papuleuse excoriée. Le visage
est indemne. L'examen des zones pileuses est normal. Il présente une langue géographique. Le traitement suivant est prescrit
: deux applications d'Ascabiol® à 15 minutes d'intervalle sur tout le tégument. Le lendemain le prurit n'a pas disparu.

Chez ce patient qui présente un prurit, vous retenez en faveur d'une gale ?
A - L'existence d'une langue géographique
B - Le respect du visage
C - L'aggravation du prurit après application de Parfenac®
D - L'atténuation du prurit après application du Diprosone®
E - La présence d'une papule excoriée du gland
Bonne(s) réponse(s) : B E

Le prurit de la gale respecte la tête, (cuir chevelu, visage), souvent le dos. Les localisations génitales sont fréquentes chez
l'homme adulte. Une aggravation du prurit par le parfenac peut être en rapport avec un eczéma de contact. L'atténuation du
prurit après application d'un corticoïde peut s'expliquer par l'action anti-inflammatoire et n'est spécifique d'aucune pathologie.

Chez ce patient atteint de gale, les caractères du prurit n'ont pas été précisés; habituellement, il :
A - Peut prédominer au cuir chevelu
B - S'accompagne de dermographisme
C - Est fréquemment familial
D - Peut entraîner une insomnie
E - Disparait en quelques minutes après application d'Asacabiol®
Bonne(s) réponse(s) : C D

Le prurit de la gale à recrudescence vespérale et nocturne respecte la tête (visage, cuir chevelu) souvent le dos, est
fréquemment conjugual ou familial. Il faut quelques jours pour qu'il disparaisse après
traitement ; (parfois plus).

1153
Exclusivement sur DOC - DZ : www.doc-dz.com NADJI 85
RESIDANAT EN POCHE TOME II
Cas Clinique en QCM

Les données de l'examen clinique sont incomplètes chez ce patient atteint de gale, on aurait pu voir :
A - Des vésicules perlées dans les espaces interdigitaux
B - Une impétiginisation
C - Des sarcoptes le long de poils
D - Des lésions de grattage sur les fesses
E - Des papules de prurigo
Bonne(s) réponse(s) : A B D E

Les sillons et vésicules perlées ont une topographie élective au niveau des espaces interdigitaux et des faces antérieures des
poignets. Un impétigo chez l'adulte doit faire rechercher une dermatose purigineuse sous-jacente, en particulier la gale. Les
sarcoptes siègent dans la couche cornée de l'épiderme.

Si le prurit n'a pas disparu 24 heures après les deux applications d'Ascabiol® il faut :
A - Récuser le diagnostic de gale
B - L'attribuer à une résistance du sarcopte vis à vis de l'Ascabiol®
C - Continuer les applications d'Ascabiol® jusqu'à disparition du prurit
D - S'abstenir, car il peut s'agir d'une évolution normale
E - Adjoindre un antiparasitaire per os
Bonne(s) réponse(s) : D

Il n'y a pas d'anti-parasitaire par voie orale actif sur le sarcopte. Il ne faut pas répéter les applications d'ascabiol, produit
pouvant être responsable de dermite irritative.

Le traitement prescrit doit être complété par :


A - Raser les poils pubiens
B - Traiter simultanément l'entourage
C - Désinfecter les draps
D - Déclaration à la DDASS
E - Faire examiner les animaux domestiques par un vétérinaire
Bonne(s) réponse(s) : B C

Il faut traiter l'entourage pour éviter les recontaminations. Les vêtements seront enfermés dans un sac avec de la poudre
aphtiria pour une durée de 24 heures. Les draps peuvent être lavés à 60°.

Un nourisson de deux mois est amené à la consultation pour une éruption érythématovésiculeuse, suintante et croûteuse
atteignant les joues, le front et la région rétro-auriculaire, évoluant depuis 15 jours dans un contexte d'état général bien
conservé. Le reste de l'examen du tégument est normal, notamment au niveau du cuir chevelu, en dehors d'une dermite du
siège au niveau de laquelle sera mis en évidence du candida Albicans. L'enfant est apyrétique et ne présente aucun autre
symptôme. L'interrogatoire des parents révèle l'existence d'un prurit très intense, l'absence de lésion à la naissance et
l'absence de tout antécédent dermatologique chez un frère âgé de 4 ans. Le bilan biologique pratiqué chez cet enfant est
normal, en dehors d'une discrète élévation des IgE à 150 UI/l ( normale inférieure à 100).

Quel(s) est(sont) I'(les) éléments clinique(s) qui orientera(ront) chez cet enfant vers le diagnostic de dermatite
atopique et non vers celui de dermite séborrhéique du nourrisson ?
A - La dermite du siège à Candida albicans
B - L'absence de fièvre
C - Le caractère vésiculeux et suintant des lésions
D - La topographie des lésions du visage
E - L'âge de l'enfant
Bonne(s) réponse(s) : C D E

La dermatite séborrhéique atteint le très jeune nourrisson (dès le premier mois) et siège initialement au cuir chevelu (squames
grasses). L'eczéma du nourrisson, un peu plus tardif (2-3 mois) se caractérise par des lésions vésiculeuses puis suintantes et
croûteuses qui sur le visage atteignent les convexités. L'éruption est très purigineuse. N'importe quel érythème fessier peut
être colonisé par le candida albicans.

En fonction du contexte clinique présenté par cet enfant, quel(s) examen(s) paraclinique(s) vous paraît(sent)
utile(s) avant de traiter cet enfant ?
A - Cytodiagnostic des lésions cutanées
B - Sérologie Herpès-virus
C - Dosage du IgE plasmatiques
D - Biopsie cutanée
E - Aucun de ces examens
Bonne(s) réponse(s) : E

Le diagnostic est clinique. Aucun examen n'est nécessaire.

1154
Exclusivement sur DOC - DZ : www.doc-dz.com NADJI 85
RESIDANAT EN POCHE TOME II
Cas Clinique en QCM

Quelle(s) vaccination(s) pourrait(ent) être pratiquée(s) sans danger chez cet enfant ?
A - Vaccination anti-poliomyélitique
B - Vaccination anti-diphtérique
C - Vaccination anti-rougeoleuse
D - Vaccination anti-tétanique
E - Vaccination anti-tuberculeuse
Bonne(s) réponse(s) : A B C D E

Aucune de celles-ci n'est contre-indiquée chez l'atopique.

Quelle(s) est(sont) la(les) proposition(s) exacte(s) concernant le pronostic à moyen terme de cette éruption ?
A - Le faible taux d'lgE circulantes rend probable la disparition de cette affection à l'âge de 2 ans
B - L'absence de symptomatologie chez le frère est un critère de bon pronostic chez cet enfant
C - Aucun élément ne permet de prévoir l'évolution de cette dermatose à moyen terme
D - La détermination du groupe HLA pourrait permettre d'apprécier un pronostic évolutif
E - Le début par l'extrémité céphalique est de mauvais pronostic
Bonne(s) réponse(s) : C

Une proportion importante d'eczémas du nourrisson disparaît spontanément vers 2 ans. Certains critères auraient une valeur
péjorative (début tardif, topographie atypique en fonction de l'âge).

Face au tableau présenté par cet enfant, laquelle ou lesquelles de ces mesures thérapeutiques vous
paraît(ssent) utilisables ?
A - Imidazolé local au niveau du siège
B - Application d'hexamidine en solution aqueuse
C - Prescription de méquitazine par voie générale
D - Corticothérapie locale au niveau des lésions du visage
E - Prescription d'un macrolide par voie générale
Bonne(s) réponse(s) : A B D

Localement il faut assurer des changes fréquents avec toilette à l'eau, bains de siège avec antiseptique dilué, et appliquer un
imidazolé antimycosique. L'eczéma du visage sera traité par une corticothérapie locale, prudente, très faible (hydrocortisone,
classe la plus faible) et de très courte durée en association avec une bonne antiseptie locale. Pas d'antibiothérapie en
l'absence de surinfection.

Une femme de 25 ans, vous consulte en automne pour une lésion pigmentée de la jambe qu'elle a remarquée depuis moins
d'un an. La lésion est indolore mais la patiente s'inquiète de cette tache qui lui parait avoir grossi depuis ses dernières
vacances d'été sur la côte d'azur. Vous notez une tâche brunâtre de 8 mm de diamètre, sans relief, sur la face externe du
mollet. A l'examen attentif la coloration n'apparait pas tout à fait homogène, avec des zones brun clair et brun foncé; les limites
sont nettes mais irrégulières.

Quel est le premier diagnostic auquel vous devez penser en fonction de ses implications pronostiques ?
A - Tache "café au lait"
B - Verrue séborrhéique
C - Mélanome malin superficiel extensif
D - Naevus pigmentaire
E - Epithélioma basocellulaire tatoué
Bonne(s) réponse(s) : C

Evident.

Parmi les caractères suivants vous retrouvez en faveur de cette hypothèse diagnostique ?
A - L'indolence
B - La polychromie
C - L'irrégularité des bords
D - L'augmentation de taille récente
E - L'absence d'adénopathie
Bonne(s) réponse(s) : B C D

On peut citer d'autres caractères inquiétants : perte de la striation cutanée normale, ulcération de la lésion, saignement,
nodule.

1155
Exclusivement sur DOC - DZ : www.doc-dz.com NADJI 85
RESIDANAT EN POCHE TOME II
Cas Clinique en QCM

Parmi les caractères suivants, lequel(lesquels) va(vont) à l'encontre d'un diagnostic de lésion maligne ?
A - L'absence de douleur
B - L'absence de saignement
C - L'absence de relief
D - L'absence de naevus préexistant
E - Aucun des caractères précités
Bonne(s) réponse(s) : E

La majorité des mélanomes surviennent en peau saine et non sur naevus préexistant.

Parmi les attitudes suivantes laquelle vous paraît justifiée, en fonction des seules données cliniques ?
A - Abstention et surveillance
B - Détruire la totalité de la lésion par électrocoagulation
C - Faire une ponction à l'aiguille pour étude cytologique
D - Faire enlever d'emblée la totalité de la lésion
E - Faire une biopsie en bordure de la lésion
Bonne(s) réponse(s) : D

Le diagnostic de mélanone étant évoqué, la lésion étant de petite taille, il est préférable de l'ôter en totalité d'emblée plutôt que
de faire une biopsie qui de toutes façons n'a aucune raison de se faire en bordure ! L'examen histologique est évidemment
indispensable.

L'étude anatomo-pathologique montre qu'il s'agit d'un naevus jonctionnel. Parmi les propositions suivantes
concernant le naevus jonctionnel, choisissez la ou les proposition(s) vraie(s) :
A - Tumeur bénigne du système pigmentaire
B - Lésion susceptible d'évoluer vers un mélanome malin
C - Tumeur à malignité purement locale
D - Tumeur maligne grave
E - Lésion comportant des thèques intra-épidermiques au contact de la basale
Bonne(s) réponse(s) : A B E

Il s'agit d'un naevus caractérisé par la présence de thèques naeviques dans les couches basales de l'épiderme faisant saillie
dans le derme papillaire. Cette activité jonctionnelle témoigne d'une éventuelle activité de croissance du naevus ; (cas de la
quasi totalité des naevis des enfants).

Une femme de 32 ans a depuis 2 jours une éruption des genoux et des faces dorsales des pieds et des mains,
érythémateuse, papuleuse, non prurigineuse, faite d'éléments de 1 cm environ à centre fripé ou bulleux réalisant un aspect en
cocarde. Il existe une conjonctivite et des érosions des lèvres. Le reste de l'examen est normal. On retrouve à l'interrogatoire
plusieurs poussées identiques au retour des sports d'hiver.

Votre diagnostic est :


A - Urticaire
B - Psoriasis annulaire
C - Pemphigoïde bulleuse
D - Erythème polymorphe
E - Erythème pigmenté fixe
Bonne(s) réponse(s) : D

L'aspect de lésions en cocardes avec bulle centrale, siégeant de façon symétrique sur les zones d'extension des membres
accompagnées de lésions muqueuses, évoque avant tout l'érythème polymorphe.

Le caractère récidivant du syndrome évoque en général deux étiologies :


A - Médicamenteuse
B - Vitaminique
C - Paranéoplasique
D - Herpétique
E - Parasitaire
Bonne(s) réponse(s) : A D

Les étiologies infectieuses (avant tout le virus de l'herpès, le mycoplasme) et médicamenteuses (pyrazolés, sulfamides, AINS,
pénicillines, barbituriques, hydantoïnes, phénothiazine), sont les principales causes. Certaines maladies générales (LEAD
Behcet, Crohn, .... certains cancers), ont été incriminées plus rarement.

1156
Exclusivement sur DOC - DZ : www.doc-dz.com NADJI 85
RESIDANAT EN POCHE TOME II
Cas Clinique en QCM

Quelle étiologie est la plus probable dans ce cas précis ?


A - Prise médicamenteuse
B - Herpès
C - Cancer profond
D - Pédiculose
E - Déficit en vitamine Bl2
Bonne(s) réponse(s) : B

La récidive au moment des expositions solaires évoque avant tout des récurrences herpétiques déclenchées par les rayons
ultra violets.

Ce syndrome peut s'accompagner de :


A - Fièvre
B - Amylose
C - Kératite
D - Syndrome néphrotique
E - Rhumatisme articulaire aigu
Bonne(s) réponse(s) : A C

Les érythèmes polymorphes s'accompagnent souvent de signes généraux : malaise, fièvre, arthralgies. L'atteinte muqueuse
touche les lèvres, la cavité buccale, les muqueuses génitales. Les érosions conjonctivo-palpébrales peuvent s'accompagner
d'érosions cornéennes, imposant un avis ophtalmo urgent.

Quel examen complémentaire est indispensable ?


A - Test de dégranulation des basophiles
B - Sérologie herpétique
C - TOGD
D - Dosage de la vitamine B12
E - Aucun de ces examens
Bonne(s) réponse(s) : E

Le diagnostic se fait sur l'interrogatoire et l'aspect clinique.

Une fillette de 5 ans est adressée en septembre au retour des vacances annuelles passées au bord de la mer, pour une
dermatose extensive évoluant depuis 5 semaines, ayant résisté à des traitements locaux associant corticoïdes et Nystatine
pommade. L'examen de cette fillette en bon état général, sans antécédents médico-chirurgicaux révèle une dermatose
polymorphe prédominante à la région faciale péri narinaire s'étendant aux jambes faites de médaillons érythémateux et
squamo croûteux à extension centrifuge et d'autres plus récents bulleux et pustuleux, fugaces évoluant vers le stade croûteux.
Son petit frère de 3 ans aurait présenté à minima les mêmes phénomènes il y a 6 semaines, résolutifs sous simple traitement
antiseptique local.

Quel diagnostic envisager ?


A - Trichophytie cutanée
B - Toxidermie
C - Impétigo
D - Psoriasis
E - Candidose cutanée
Bonne(s) réponse(s) : C

La sémiologie des lésions, (transitoirement bulleuses puis croûteuses), la localisation périnarinaire, l'auto-inoculation (jambes)
et la contagiosité inter-humaine (frère atteint) évoquent l'impétigo.

Quel examen complémentaire pratiquer ?


A - Hémocultures
B - Examen mycologique cutané
C - Examen bactériologique cutané
D - Biopsie d'une bulle
E - Examen parasitaire cutané
Bonne(s) réponse(s) : C

L'impétigo est streptococcique et /ou staphylococcique.

1157
Exclusivement sur DOC - DZ : www.doc-dz.com NADJI 85
RESIDANAT EN POCHE TOME II
Cas Clinique en QCM

Quel(s) traitement(s) proposer ?


A - Griséofulvine per os
B - Erythromycine per os
C - Antisepsie locale.
D - Nystatine per os
E - Corticoïdes locaux
Bonne(s) réponse(s) : B C

Les corticoïdes sont évidemment contre-indiqués. La possibilité d'une infection streptococcique impose un traitement par voie
générale pour éviter une éventuelle glomérulonéphrite post-streptococcique.

Quel(s) examen(s) de contrôle pratiquer après la guérison ?


A - Albuminurie
B - Dosage des transaminases
C - Vitesse de sédimentation
D - Electrocardiogramme
E - Examen cytobactériologique des urines
Bonne(s) réponse(s) : A

A la recherche d'une atteinte rénale post-streptococcique.

Quel(s) caractère(s) appartien(en)t à l'affection en cause ?


A - Transmission interhumaine
B - Contagiosité importante
C - Transmission héréditaire autosomique dominante
D - Affection dysimmunitaire
E - Caractère primitif ou secondaire à une autre affection
Bonne(s) réponse(s) : E

Chez l'enfant, l'impétigo est le plus souvent primitif. Chez l'adulte il est souvent secondaire à une dermatose prurigineuse
(gale, eczéma...) qu'il faut rechercher systématiquement.

Un enfant de 8 ans sans antécédent particulier présente brusquement au retour d'une promenade un purpura des membres
inférieurs fait de petites taches ne s'effaçant pas à la vitro-pression, certaines étant un peu palpables (papuleuses). Il souffre
dans les genoux et les chevilles et a eu un peu mal au ventre. Il n'y a pas d'hémorragie muqueuse.

Quels sont les trois éléments cliniques en faveur de l'origine vasculaire de ce purpura ?
A - L'absence d'effacement à la vitro-pression
B - La présence d'éléments papuleux
C - Les douleurs articulaires
D - L'absence de fièvre
E - L'absence d'hémorragie muqueuse
Bonne(s) réponse(s) : B C E

A est la définition du purpura. Un purpura vasculaire est fait d'éléments papuleux, plus ou moins infiltrés.

Lequel des examens suivants vous parait utile ?


A - Radio pulmonaire
B - Bilan rénal
C - Dosage des IgE
D - Test de transformation Iymphoblastique à la Phytohémagglutinine
E - Bilan hépatique
Bonne(s) réponse(s) : B

L'atteinte rénale (hématurie, protéinurie, syndrome néphrotique par glomérulonéphrite segmentaire et focale à dépôts d'Ig A)
conditionne le pronostic du purpura rhumatoïde.

1158
Exclusivement sur DOC - DZ : www.doc-dz.com NADJI 85
RESIDANAT EN POCHE TOME II
Cas Clinique en QCM

Lequel des diagnostics suivants vous semble à retenir ?


A - Purpura thrombopénique idiopathique
B - Cryoglobulinémie
C - Toxidermie
D - Purpura rhumatoïde
E - Hémopathie
Bonne(s) réponse(s) : D

L'âge, le purpura vasculaire, la symptomatologie abdominale évoquent avant tout le diagnostic de purpura rhumatoïde.

Lequel des traitements suivants est à envisager ?


A - Corticothérapie (Prednisone 1 mg/jour)
B - Traitement antibiotique (Oracéfal® )
C - Traitement antihistaminique (Polaramine®)
D - Traitement anti-inflammatoire (Indocid®)
E - Aucun
Bonne(s) réponse(s) : E

La corticothérapie est indiquée dans certaines complications (rénales après PBR, testiculaires...) Il faut proposer un repos au
lit.

Un homme de 50 ans, coffreur dans le bâtiment, sans antécédents cutanés personnels mais ayant un cas d'eczéma
constitutionnel dans sa famille, à depuis 6 ans un eczéma lichénifié peu intense ayant débuté sur le dos des pieds puis apparu
aux faces antérieures des avant-bras et au dos des mains. La peau est épaissie, quadrillée, avec de petites croûtelles, et de
temps en temps des petites vésicules très prurigineuses. L'application de dermocorticoïdes donne une amélioration partielle
mais provisoire.

Les caractères de l'éruption évoquent principalement un des diagnostics suivants :


A - Eczéma constitutionnel d'apparition tardive
B - Eczéma professionnel
C - Psoriasis lichénifié
D - Dermite caustique (orthoergique)
E - Porphyrie cutanée
Bonne(s) réponse(s) : B

Le type de profession, l'âge et la localisation des lésions évoque une origine professionnelle.

Pour prouver ou écarter une origine professionnelle vous préférez les 2 moyens suivants :
A - Biopsie cutanée
B - Arrêt de travail et reprise après guérison clinique
C - Tests épicutanés
D - Tests intradermiques
E - Tests de transformation Iymphoblastique (TTL)
Bonne(s) réponse(s) : B C

L'évolution de la dermatose en fonction des périodes de travail et de vacances est un très bon argument. Les tests épicutanés
fait avec les allergènes manipulés lors du travail sont évidemment décisifs.

Quel sera le test qui sera probablement positif ?


A - Test au bois de coffrage
B - Test au cobalt
C - Test au bichromate de K
D - Test au Nickel
E - Test au zinc
Bonne(s) réponse(s) : C

Les chromates contenus dans le ciment, le cuir font partie des allergènes les plus courant.

1159
Exclusivement sur DOC - DZ : www.doc-dz.com NADJI 85
RESIDANAT EN POCHE TOME II
Cas Clinique en QCM

Une éventuelle déclaration de maladie professionnelle est à faire ?


A - Même en cas de doute (par ex si les tests sont négatifs)
B - Seulement si le test est positif
C - Seulement par le médecin du travail
D - Seulement à la demande de la sécurité sociale
E - Seulement par un allergologue diplôme
Bonne(s) réponse(s) : A

Le médecin traitant peut très bien déclarer la maladie. Il peut être impossible de tester tous les allergènes manipulés lors du
travail ; une forte suspicion conduit à déclarer la maladie.

Une jeune infirmière de 28 ans se plaint de prurit cutané localisé essentiellement aux mains, elle l'attribue au début à des
détergents dont elle se sert couramment. Malgré l'arrêt de ces détergents, le prurit persiste et commence à s'étendre à
d'autres parties du corps, les coudes, le dos, la face antérieure du thorax. L'augmentation du prurit l'amène à consulter. A
l'examen, la peau, au niveau des mains et des bras présente des lésions de grattage et quelques lésions montrent des petites
vésicules.

Quel est votre diagnostic ?


A - Gale
B - Larva migrans cutanée
C - Leishmaniose cutanée
D - Piqûre d'insectes
E - Lésions allergiques
Bonne(s) réponse(s) : A

L'absence de lésions au niveau de certaines zones prurigineuses (thorax, dos), le type de lésions des mains et des bras
peuvent faire évoquer la gale et non un eczéma de contact à des produits manipulés.

Comment allez-vous en faire le diagnostic ?


A - Par recherche du parasite dans le sang
B - Par biopsie cutanée au niveau d'une lésion
C - Par examen direct dans les squames prélevées par grattage énergique
D - Par examen direct des squames prélevées en superficie
E - Après mise en culture des squames sur le milieu de Sabouraud
Bonne(s) réponse(s) : D

Les sarcoptes se localisent dans la couche cornée. Il est préférable de gratter la peau au niveau des sillons et vésicules.

Comment pensez-vous que l'affection puisse se contracter ?


A - Par piqûre de moustique
B - Par une intoxication alimentaire
C - Par utilisation d'un savon contaminé
D - Par contact avec un chien atteint d'une dermite érosive
E - Par un contact étroit ou prolongé avec un malade
Bonne(s) réponse(s) : E

La gale se contracte soit par contacts intimes avec une personne atteinte (rapports sexuels) ou encore par contacts étroits et
prolongés (cas du personnel soignant) ou exceptionnellement par du linge contaminé (draps).

Le traitement a administrer est :


A - De l'érythromycine
B - Du pevaryl
C - Du mintezol
D - De la diphétarsone
E - De l'ascabiol
Bonne(s) réponse(s) : E

Evident.

1160
Exclusivement sur DOC - DZ : www.doc-dz.com NADJI 85
RESIDANAT EN POCHE TOME II
Cas Clinique en QCM
Un enfant de 10 ans est amené en consultation pour des lésions du cuir chevelu apparues il y a 3 semaines. Il est l'ainé de 3
enfants : une soeur de 7 ans,un frère de 5 ans. Les lésions sont apparues au retour de vacances qu'il avait passées dans la
ferme de ses grand-parents où il a recueilli un petit chat qu'il a ramené chez ses parents. L'examen montre 2 plaques bien
limitées, prurigineuses de 2 à 3 cm de diamètre finement squameuse non cicatricielles. Les squames sont grisâtres et se
détachent facilement. Quelques cheveux sont cassés 2 à 5 mm au dessus de l'orifice folliculaire. D'autres viennent à la
traction. Depuis, 2 ans, il est traité par une pommade au goudron pour un psoriasis des coudes et des genoux. Une
corticothérapie locale sur le cuir chevelu n'a pas entraîné d'amélioration. La mère est venue seule avec lui, les autres enfants
n'auraient rien. Elle même a un examen dermatologique normal, mais elle signale chez son mari la survenue depuis une
semaine d'une lésion de l'avant bras ayant tendance à s'étendre. Vous évoquez le diagnostic de teigne microsporique.

En faveur de ce diagnostic, on retient :


A - Enfant
B - Alopécie non cicatricielle
C - Cheveux cassés courts
D - Squames fines
E - Lésion cutanée du père
Bonne(s) réponse(s) : A B C D E

Les teignes tondantes sèches microsporiques et trichophytiques atteignent les enfants de 4 à 12 ans. Les teignes
microsporiques donnent des plaques peu nombreuses de grande taille (4-7 cm) recouvertes de squames grisâtres avec
cheveux cassés courts ; (par opposition aux teignes trichophytiques qui donnent de nombreuses petites plaques avec
cheveux ras englués dans des squames). La lésion cutané du père est un herpès circiné. Les lésions guérissent sous
traitement ou spontanément à la puberté sans alopécie séquellaire, ce qui n'est pas le cas des teignes faviques et suppurées.

Chez ce patient quel élément permet d'éliminer formellement un psoriasis du cuir chevelu ?
A - Caractère grisâtre des squames
B - Prurit
C - Alopécie
D - Squames fines
E - Lésions bien limitées
Bonne(s) réponse(s) : C

Le psoriasis du cuir chevelu n'est jamais responsable d'alopécie.

Quels sont le ou les examens utiles soit pour confirmer le diagnostic, soit pour des raisons épidémiologiques ou
thérapeutiques ?
A - La numération formule sanguine
B - L'examen en lumière de Wood
C - Le prélèvement mycologique direct
D - La biopsie du cuir chevelu
E - La culture sur milieu de Sabouraud
Bonne(s) réponse(s) : B C E

Les teignes microsporiques donnent en lumière de wood une fluorescence verte intense comme les teignes faviques ; les
teignes trichophytiques et les teignes suppurées ne donnent pas de fluorescence. L'examen direct peut permettre un
diagnostic rapide ; les cultures demandent environ 3 semaines.

Parmi les traitements suivants, quels sont les deux que l'on peut proposer ?
A - Amphotéricine B en suspension 3 cuillères à café/jour durant 3 semaines
B - Griséofulvine 500 mg/jour durant 6 semaines
C - Mycostatine pommade 3 applications/jour jusqu'à guérison
D - Miconazole crème 2 applications/jour jusqu'à guérison
E - Solution de Milian, 2 applications/jour jusqu'à guérison
Bonne(s) réponse(s) : B D

Le traitement des teignes associe un traitement général par Griséofulvine ou Kétokonazole (Nizoral) et un traitement local par
un imidazolé. Le traitement doit être poursuivi 6 à 8 semaines.

Une ou plusieurs mesures complémentaires doivent être prises ; la(es)quelle(s) ?


A - Faire examiner et traiter le chat
B - Examiner tous les membres de la famille
C - Faire un prélèvement mycologique à la fin du traitement
D - Eviction scolaire durant 3 mois
E - Déclaration obligatoire
Bonne(s) réponse(s) : A B C E

L'éviction solaire se poursuit théoriquement jusqu'à négativation de l'examen direct et de l'examen en lumière de wood.

1161
Exclusivement sur DOC - DZ : www.doc-dz.com NADJI 85
RESIDANAT EN POCHE TOME II
Cas Clinique en QCM
Un enfant de 12 ans vous est amené par sa maman car il présente des lésions croûteuses de la face. Les lésions sont de
diamètre variable (4 à 8 mm) constituées d'une croûte jaunâtre ; il existe des lésions plus récentes faites d'une bulle flasque à
contenu trouble reposant sur une peau érythémateuse. Les lésions prédominent dans la région péri-buccale et péri-narinaire.

Quel diagnostic évoquez-vous ?


A - Ecthyma
B - Pyodermites végétantes
C - Pemphigus vulgaire
D - Impétigo contagiosum
E - Dermatite herpétiforme
Bonne(s) réponse(s) : B

Le tableau clinique est typique d'impétigo : lésions bulleuses éphémères remplacées par des croûtes jaunâtres de topographie
péri-buccale et péri-narinaire.

Quel(s) est (sont) le(s) germe(s) en cause dans cette affection ?


A - Bacille pyocyanique
B - Streptocoque bêta hémolytique
C - Coxsackie B6
D - Staphylocoque aureus
E - Staphylocoque blanc
Bonne(s) réponse(s) : B D

L'affection est assez souvent mixte streptococcique et staphylococcique.

Un traitement associant soins locaux, antibiotiques cutanés et prescription d'une antibiothérapie par voie orale
est nécessaire :
A - Pour prévenir les récidives
B - Pour prévenir les complications rénales
C - Car, sur ce terrain, le risque de cicatrices inesthétiques est majeur
D - Pour diminuer le risque de contagiosité vis à vis de l'entourage
E - Pour éviter une extension des lésions
Bonne(s) réponse(s) : B D E

On associe un traitement par voie générale (érythromycine ou pénicilline V) et un traitement antiseptique local.

Quel(s) est (sont) le(s) examen(s) de surveillance chez cet enfant au décours de cet épisode dermatologique ?
A - Prise de la tension artérielle
B - Prélèvements bactériens des orifices naturels
C - Hémocultures
D - Recherche d'une protéinurie
E - Recherche d'un diabète sucré
Bonne(s) réponse(s) : D

Il faut vérifier 3 semaines à 1 mois plus tard, l'absence d'atteinte rénale.

L'enfant a été traité par ampicilline 50 mg/Kg/j. 8 jours plus tard, il présente de grands décollements très
superficiels avec signe de Nikolski positif et des érosions muqueuses. Quel diagnostic évoquez-vous ?
A - Récidive d'impétigo
B - Pemphigus foliacé
C - Syndrome de Lyell
D - Erythème polymorphe grave
E - Epydermolyse bulleuse congénitale à révélation tardive
Bonne(s) réponse(s) : C

Le contexte (impétigo), le caractère très superficiel des décollements évoque un syndrome de Lyell staphylococcique.

1162
Exclusivement sur DOC - DZ : www.doc-dz.com NADJI 85
RESIDANAT EN POCHE TOME II
Cas Clinique en QCM
Un patient âgé de 40 ans se plaint depuis 6 mois d'ulcérations buccales aphtoïdes. Depuis une dizaine de jours il a en outre
des bulles, des ulcérations postbulleuses et des croûtelles sur la région thoracique et dorsale médiane. L'état général est
conservé ; il n'y a aucun antécédent particulier.

Quel est le diagnostic le plus probable ?


A - Un impétigo staphylococcique
B - Une toxidermie médicamenteuse
C - Une porphyrie cutanée
D - Un zona
E - Un pemphigus vulgaire
Bonne(s) réponse(s) : E

Ce tableau n'est pas celui d'un impétigo qui donne des bulles éphémères vite remplacées par des croûtes jaunâtres, de
topographie péri-buccale et péri-narinaire et qui touche l'enfant. Les bulles de porphyrie touchent les zones photo-exposées et
surtout les dos des mains et avant-bras. La topographie et l'évolution ne vont absolument pas avec le diagnostic de zona. Le
début de la maladie par des érosions buccales, suivi de bulles cutanées chez un patient de 40 ans évoque un pemphigus ;
certains d'entre eux sont induits par des prises médicamenteuses.

L'éruption bulleuse du tronc dans la maladie dont vous avez fait le diagnostic peut :
A - Survenir en peau saine
B - Etre déclenchée préférentiellement par la lumière
C - Etre d'aspect assez monomorphe
D - Etre prurigineuse
E - S'accompagner d'une polyadénopathie
Bonne(s) réponse(s) : A C

Les bulles de pemphigus surviennent en peau saine, ne sont pas prurigineuses ; l'éruption est monomorphe. Ceci par
opposition à la pemphigoïde bulleuse où les bulles siègent sur des placards érythémateux ou urticariens et prurigineux.

L'enquête étiologique de cette maladie doit faire rechercher :


A - Une infection bactérienne intercurrente
B - Une entéropathie jéjunale
C - Un cancer viscéral
D - La prise de certains médicaments
E - Une maladie sexuellement transmissible
Bonne(s) réponse(s) : D

Certains médicaments peuvent induire un pemphigus : D pénicillamine, pénicillines, lopril, thiopronine, pyritinol, rifampicine,
phénylbutazone, piroxicam... Des associations de pemphigues et de néoplasie ont été rapportées, elles sont très rares et
possiblement fortuites.

Pour confirmer le diagnostic attendu la biopsie d'une bulle cutanée est importante. L'aspect caractéristique est :
A - Une bulle sous épidermique
B - Une bulle intraépidermique avec ballonisation des kératinocytes
C - Une bulle intraépidermique multiloculaire avec nécrose de l'épiderme
D - Une bulle intraépidermique avec acantholyse
E - Une bulle par nécrose de l'assise basale
Bonne(s) réponse(s) : D

Le clivage est intraépidermique avec cellules acantholytiques (cellules épidermiques détachées, libres dans la cavité de la
bulle). Un examen cytologique du produit de grattage du fond d'une érosion montre les cellules acantholytiques.

Un enfant de 4 ans est adressé pour la présence de lésions cutanées localisées au visage, au cuir chevelu, aux membres
supérieurs, évoluant depuis 2 semaines. Au début, il s'agissait de bulles flasques, puis secondairement ces lésions sont
devenues croûteuses, jaunâtres. Par ailleurs il existe des lésions érythémateuses, squameuses, sèches, très prurigineuses,
au niveau du pli du coude et du creux poplité ainsi qu'une fissure rétro-auriculaire et une sécheresse cutanée généralisée qui
existent depuis plus d'un an. Le père du patient est asthmatique et un frère présente une rhinite pollinique, sa mère présente
un psoriasis. Le reste de l'examen clinique est normal en dehors d'adénopathies axillaires et cervicales.

Quels sont les deux diagnostics que vous évoquez chez ce patient ?
A - Scabiose
B - Impétigo
C - Erysipèle
D - Dermatite atopique
E - Pemphigoïde bulleuse
Bonne(s) réponse(s) : B D

La topographie des lésions prurigineuses, leur non contagiosité depuis 1 an excluent une gale. Les diagnostics de dermatite
atopique et d'impétigo sont évidents. 1163
Exclusivement sur DOC - DZ : www.doc-dz.com NADJI 85
RESIDANAT EN POCHE TOME II
Cas Clinique en QCM

L'affection cutanée d'apparition récente peut se compliquer de :


A - Glomérulonéphrite aiguë
B - Rhumatisme articulaire aigu
C - Dermatophytie
D - Adénopathies
E - Candidose
Bonne(s) réponse(s) : A D

Les streptococcies cutanées ne se compliquent jamais de RAA.

Le ou les germe(s) en cause peuvent être :


A - Streptocoque
B - Staphylocoque
C - Propionibacterium acnes
D - Pneumocoque
E - Pseudomonas aeruginosa
Bonne(s) réponse(s) : A B

L'affection est streptococcique et / ou staphylococcique.

Concernant l'affection récente, la ou les thérapeutique(s) suivante(s) peut ou peuvent être indiquée(s) :
A - Pénicilline V
B - Erythromycine
C - Pristinamycine
D - Chlorhexidine
E - Désensibilisation aux antigènes microbiens
Bonne(s) réponse(s) : A B C D

Le traitement associe une antibiothérapie générale active sur le streptocoque surtout, le staphylocoque et une antiseptie
locale. Le traitement de choix est l'érythromycine.

En faveur du diagnostic de l'affection présente depuis un an, vous retenez :


A - Fissure rétro-auriculaire
B - Atteinte des creux poplités et des plis du coude
C - Antécédents familiaux de rhinite
D - Antécédents familiaux de psoriasis
E - Adénopathies cervicales
Bonne(s) réponse(s) : A B C

Il existe un terrain atopique familial en faveur du diagnostic ; la topographie des lésions aux plis est caractéristique de même
que la fissure rétro-auriculaire fréquente chez les atopiques.

Pour confirmer le diagnostic de l'affection présente depuis un an il est indispensable de disposer du résultat de :
A - Recherche d'une éosinophilie sanguine
B - Recherche de parasites dans les selles
C - Tests cutanés allergologiques
D - Dosage d'IgE
E - Aucun de ces examens
Bonne(s) réponse(s) : E

Le diagnostic de dermatite atopique se fait sur l'interrogatoire et l'examen clinique.

1164
Exclusivement sur DOC - DZ : www.doc-dz.com NADJI 85
RESIDANAT EN POCHE TOME II
Cas Clinique en QCM
Une femme de 35 ans, employée de bureau, vous consulte pour une dermatose non prurigineuse` évoluant par poussées
depuis l'âge de 32 ans. D'après la patiente, la poussée actuelle serait liée à des ennuis familiaux récents. A l'examen, vous
notez la présence de plaques érythémato-squameuses multiples siégeant aux jambes, dans la région lombaire, le tronc et au
cuir chevelu sans entraîner d'alopécie. La patiente ne présente pas de signes généraux.

Quel est votre diagnostic ?


A - Eczéma
B - Lichen
C - Pityriasis rosé de Gibert
D - Psoriasis
E - Pityriasis versicolor
Bonne(s) réponse(s) : D

Le diagnostic de psoriasis est évident : il s'agit d'une dermatose érythémato-squameuse évoluant par poussées, rythmée par
les épisodes de stress ; les localisations au niveau lombaire et au niveau du cuir chevelu sont habituelles.

Dans la pathologie évoquée il n'est pas exceptionnel d'observer :


A - Un oedème des membres inférieurs
B - Une atteinte des grands plis
C - Une atteinte conjonctivale
D - Des atteintes articulaires périphériques
E - Un hippocratisme digital
Bonne(s) réponse(s) : E

- L'atteinte des plis (inguinaux, inter-fessiers, région génitale, ombilic, plis sous-mammaires) peut être la seule manifestation,
rendant le diagnostic difficile.
- 20 % des psoriasiques souffrent d'une atteinte articulaire (périphérique avec mono, oligo ou polyarthrites ou centrale, avec
spondylarthrite de tableau voisin de la SPA).

Quel(s) examen(s) complémentaire(s) demanderez-vous ?


A - Un prélèvement bactériologique cutané
B - Un prélèvement viral cutané
C - Un examen cutané en immunofluorescence directe
D - Un prélèvement mycologique cutané
E - Aucune des propositions ci-dessus
Bonne(s) réponse(s) : E

Le diagnostic de ce psoriasis est aisé sur l'histoire, l'examen clinique.

Votre traitement comportera :


A - Pénicilline par voie orale
B - PUVA
C - Anti-inflammatoire non stéroïdiens
D - Anti-mycosiques locaux
E - Dérivés synthétiques de la vitamine A per os
Bonne(s) réponse(s) : B

- L'atteinte diffuse du tégument (plaques multiples) est une bonne indication à la puvathérapie.
- E n'est pas dans cette forme un traitement de 1ère intention.

1165
Exclusivement sur DOC - DZ : www.doc-dz.com NADJI 85
RESIDANAT EN POCHE TOME II
Cas Clinique en QCM
Un homme de 30 ans consulte pour une ulcération du sillon balano-préputial. L'interrogatoire apprend que cet homme est
homosexuel, à partenaires multiples. Il a eu deux ans auparavant une syphilis qui a été traitée par pénicilline ; la sérologie
syphilitique s'est négativée à la suite de ce traitement, d'après le patient. Il a eu à plusieurs reprises des uréthrites aiguës
gonococciques, toujours soignées avec succès par antibiotiques. Il n'a pas d'autre antécédent notable. La lésion actuelle a
débuté huit jours avant la consultation et s'étend malgré l'application d'antibiotiques locaux. Il s'agit d'une petite ulcération
arrondie, rose, indolore, dure à la palpation. On note la présence de plusieurs ganglions inguinaux non inflammatoires.

Quel est le diagnostic le plus probable ?


A - Syphilis
B - Chancre mou
C - Herpès
D - Gonococcie
E - Aphte
Bonne(s) réponse(s) : A

- Le diagnostic de syphilis est très probable : terrain, caractères de la lésion (fond propre, indolore, indurée, adénopathies non
inflammatoires, non douloureuses) la différencient du chancre mou.
- D ne donne pas ce type de lésion.
- C donne des érosions rondes groupées en bouquets, douloureuses.
- Les aphtes sont très douloureux.

L'examen au microscope à fond noir ne retrouve pas de tréponème et la recherche de bacille de Ducrey est
négative. Quelle conclusion pouvez-vous en tirer ?
A - Il ne s'agit ni d'une syphilis, ni d'un chancre mou
B - Ces examens peuvent être faussement négatifs
C - Il est inutile de faire des prélèvements directs sur les muqueuses génitales.
D - En cas de chancre syphilitique, le tréponème ne peut être isolé qu'après huit jours d'évolution
E - Aucune des propositions ci-dessus n'est exacte
Bonne(s) réponse(s) : B

Ces examens peuvent être négatifs surtout si le patient a appliqué des antiseptiques.

Vous demandez une sérologie syphilitique. Elle donne les résultats suivants :
- TPHA positif au 1/640ème
- VDRL positif, titre 1/16
Quelle(s) conclusion(s) pouvez-vous en tirer ?
A - Il peut sagir de la séquelle de sa syphilis antérieure
B - Il faut faire un test de Nelson qui seul affirmera la syphilis
C - Il peut sagir d'une syphilis récente
D - Il faut faire un F.T.A.
E - Aucune des propositions ci-dessus n'est exacte
Bonne(s) réponse(s) : C

Une séquelle sérologique se manifeste par une positivité des réactions tréponémiques (TPHA, FTA), les réactions
cardiolipidiques étant négatives (VDRL). Le TPHA se positive dans une syphilis récente vers le 8-10ème jour, le VDRL vers le
15ème jour, le FTA vers le 5-10ème jour.

Parmi les éléments rapportés dans l'observation, vous retenez en faveur du diagnostic de chancre mou :
A - Homosexualité
B - Antécédent de syphilis correctement traitée
C - Ulcération indolore
D - Présence d'adénopathies non inflammatoires
E - Aucune des propositions ci-dessus
Bonne(s) réponse(s) : E

L'ulcération du chancre mou est douloureuse de même que les adénopathies qui évoluent souvent vers la fistulisation.

En faveur du diagnostic de syphilis vous retenez :


A - Homosexualité
B - Antécédent de syphilis correctement traitée
C - Ulcération indolore
D - Présence d'adénopathies non inflammatoires
E - Aucune des propositions ci-dessus
Bonne(s) réponse(s) : C D

A et B définissent un "terrain à risque" pour la survenue d'une autre M.S.T.

1166
Exclusivement sur DOC - DZ : www.doc-dz.com NADJI 85
RESIDANAT EN POCHE TOME II
Cas Clinique en QCM
Une femme de 62 ans est hospitalisée pour bilan d'un diabète récemment découvert. L'interrogatoire retrouve comme
antécédents pathologiques : une phlébite puerpérale à l'âge de 25 ans, une pleurésie tuberculeuse à l'âge de 32 ans. Trois
mois avant l'hospitalisation actuelle, la patiente a eu une bronchite aiguë, traitée par Tétracycline.
Après 15 jours de traitement, est survenu une diarrhée aiguë, qui a duré 4 jours et a guéri après traitement symptomatique et
arrêt des antibiotiques. A cette occasion, on pratique entre autres examens une glycémie à jeun, qui est à 8 mmol/l (taux
retrouvé à 3 reprises). Il existe une glycosurie à 15 mmol/24h. L'examen montre une femme en bon état général, pesant 82kg
pour 1,64 m. L'attention est attirée par des lésions cutanées, localisées dans la région périnéale (pli inter-fessier, plis
inguinaux) et sous les seins. Ces lésions, présentes depuis quelques semaines, sont modérément prurigineuses. Il s'agit de
plaques érythémateuses luisantes, prédominant au fond des plis à partir desquels elles s'étendent de façon progressive avec
une collerette desquamative. De nombreuses petites pustules blanchâtres sont visibles sur ces plaques notamment à leur
périphérie. Un prélèvement mycologique et bactériologique cutané sur un pli sous-mammaire montre la présence de
nombreux staphylocoques dorés, de corynébactéries et de nombreux candida albicans.

Dans l'histoire clinique de cette patiente, quel(s) élément(s) est (sont) facteur(s) favorisant(s) de son intertrigo ?
A - Diabète
B - Excès pondéral
C - Antécédent de bronchopathie
D - Antécédent de phlébite
E - Antécédent de pleurésie tuberculeuse.
Bonne(s) réponse(s) : A B

Le diabète favorise les infections; l'obésité favorise la macération, surtout dans les plis.

Parmi les interprétations suivantes de l'examen myco-bactériologique, vous retenez 3 des propositions
suivantes :
A - Tous ces germes sont des saprophytes de la peau
B - Il peut s'agir d'un intertrigo à dermatophyte
C - Il peut s'agir d'un intertrigo à candida
D - Il peut s'agir d'un intertrigo microbien
E - Il peut s'agir d'un psoriasis des plis
Bonne(s) réponse(s) : C D E

- Candida Albicans est un saprophyte du tube digestif.


- Un intertrigo à dermatophytes a une bordure active, érythémateuse, vésiculeuse, à contours plus ou moins polycycliques, et
avec une tendance à la guérison centrale (centre prenant une couleur bistre). Il se localise surtout au niveau des plis
inguinaux et des espaces inter-orteils.
- Le psoriasis inversé (psoriasis des plis) atteint les plis inguinaux, interfessiers, la région génitale, l'ombilic, les plis sous
mammaires ; l'intertrigo est fait de plaques érythémateuses lisses peu squameuses à bordure bien limitée ; un intertrigo
psoriasique peut se surinfecter (bactéries, mycoses) et avoir alors un aspect modifié.

Parmi les caractères cliniques de cet intertrigo, trois sont en faveur d'une étiologie candidosique :
A - Localisation péri-anale
B - Caractère modéré du prurit
C - Présence de pustules
D - Caractère érythémateux
E - Présence d'une collerette desquamative
Bonne(s) réponse(s) : A C E

Les intertrigos candidosiques se localisent préférentiellement dans les plis interfessiers, inguinaux, sous-mammaires,
axillaires et interdigitaux. Ils forment des placards rouges, à bords bien limités, avec collerette desquamative et pustules un
peu à distance ; le fond des plis peut être recouvert d'un enduit blanchâtre.

Parmi les affections suivantes, laquelle ou lesquelles sont des cause(s) de dermite des plis ?
A - Candidose
B - Dermatophytie
C - Psoriasis
D - Erythrasma
E - Dermatite herpétiforme
Bonne(s) réponse(s) : A B C D

E - Est une dermatose bulleuse auto-immune atteignant préférentiellement les faces d'extension des membres.

1167
Exclusivement sur DOC - DZ : www.doc-dz.com NADJI 85
RESIDANAT EN POCHE TOME II
Cas Clinique en QCM

Parmi les médicaments suivants, lequel ou lesquels sont actifs sur le candida albicans ?
A - Nystatine
B - Griséofulvine
C - Miconazole
D - 5 fluoro-cytosine
E - Ketoconazole
Bonne(s) réponse(s) : A C D E

La griséfuline n'est active que sur les dermatophytes.

Quel est le meilleur traitement de l'intertrigo de cette patiente ?


A - Equilibrer le diabète sans autre mesure complémentaire
B - Application locale d'un anti-candidosique
C - Application locale d'un anti-candidosique et prise de nystatine per os
D - Application locale d'un anti-candidosique et prise de griséofulvine per os
E - Prise de nystatine per os, sans traitement local
Bonne(s) réponse(s) : C

Il faut en plus équilibrer le diabète. Le traitement per os vise à stériliser le foyer digestif éventuel.

Un enfant de 6 ans a des lésions cutanées du visage, du cuir chevelu et des mains, évoluant depuis une semaine. Il s'agit de
lésions croûteuses, jaunâtres, localisées principalement autour de l'orifice buccal et narinaire mais aussi sur les mains et les
doigts. Des croûtes sont présentes aussi sur le cuir chevelu et il existe des lésions érythématosquameuses, infiltrées,
lichénifiées très prurigineuses du pli du coude et du creux poplité, ainsi qu'une fissure rétro auriculaire et une sécheresse
cutanée généralisée. Le père de l'enfant est asthmatique et un frère est affligé d'une rhinite saisonnière. Sa mère a un
psoriasis. Le reste de l'examen clinique est normal en dehors d'adénopathies axillaires et cervicales.

Quel(s) diagnostic(s) évoque-vous ?


A - Scabiose
B - Impétigo
C - Pédiculose
D - Dermatite atopique
E - Ecthyma
Bonne(s) réponse(s) : B D

Le diagnostic d'impétigo est évident sur l'existence de croûtes jaunâtres de topographie péri-buccale et péri-narinaire, avec
auto-inoculation à distance (lésions des mains et des doigts). Les adénopathies sont en rapport.
L'eczéma atopique est diagnostiqué sur le terrain atopique et les plaques lichénifiées des plis.

Le ou les germe(s) en cause dans les lésions croûteuses du visage peut(vent) être :
A - Streptocoque
B - Staphylocoque doré
C - Propionibacter acnei
D - Microcoques
E - Staphylocoque épidermidis
Bonne(s) réponse(s) : A B

L'impétigo est streptococcique et/ou staphylococcique.

La ou les thérapeutique(s) suivante(s) peut(vent) être indiquée(s) :


A - Pénicilline V
B - Erythromycine
C - Désensibilisation aux antigènes microbiens
D - Chlorhexidine
E - Pristinamycine
Bonne(s) réponse(s) : A B D E

Le traitement associe une antibiothérapie par voie générale dirigée contre le streptocoque et accessoirement le staphylocoque
et une antiseptie locale. Le traitement général vise avant tout à prévenir les complications post-streptococciques (GNA).

1168
Exclusivement sur DOC - DZ : www.doc-dz.com NADJI 85
RESIDANAT EN POCHE TOME II
Cas Clinique en QCM

Parmi les éléments suivants, lesquels sont en faveur d'une atopie chez cet enfant ?
A - Fissure rétro-auriculaire
B - Atteinte des creux poplités et des plis du coude
C - Le psoriasis de la mère
D - L'asthme du père
E - La rhinite saisonnière du frère
Bonne(s) réponse(s) : A B D E

- D et E prouvent l'existence d'un terrain atopique familial.


- L'atteinte des plis est l'atteinte élective de l'eczéma atopique chez l'enfant après 2-3 ans.
- La fissure sous et derrière le lobule de l'oreille est un marqueur mineur d'atopie.

Quel(s) examen(s) complémentaire(s) est (sont) indispensable(s) pour confirmer ce diagnostic ?


A - Recherche d'une éosinophilie sanguine
B - Tests cutanés allergologiques
C - RAST IgE polliniques
D - Dosage des IgE totaux
E - Aucun de ces examens
Bonne(s) réponse(s) : E

Le diagnostic est clinique, orienté par le contexte familial d'atopie. Les IgE sont fréquemment augmentées.

Un homme de 25 ans, a vu s'installer une éruption prurigineuse, faite de placards urticariens et de petites bulles en bouquets
sur la face d'extension de ses bras, sa nuque, son dos, la face postérieure de ses cuisses et sur ses fesses. Il n'y a pas de
lésion muqueuse, pas de signe de Nikolsky. La biopsie cutanée standard montre une bulle sous épidermique contenant des
polynucléaires neutrophiles et éosinophiles. En immunofluorescence directe il existe des dépots granuleux au sommet des
papilles dermiques.

Quel diagnostic évoquez-vous ?


A - Pemphigoïde bulleuse
B - Pemphigus
C - Dermatite herpétiforme
D - Dermatite à IgA linéaire
E - Erythème polymorphe
Bonne(s) réponse(s) : C

Le tableau clinique (placards urticariens, bulles groupées en bouquets, âge jeune), l'histologie (bulle sous épidermique) et l'IF
(dépôts granuleux au sommet des papilles dermiques) font porter ce diagnostic.

Quel type de dépôt observe-t-on en immunofluorescence directe dans cette pathologie ?


A - Dépôt d'IgA
B - Dépôt d'IgG
C - Dépôt d'IgG et de complément
D - Dépôt d'IgM
E - Dépôt d'IgA et de complément
Bonne(s) réponse(s) : A

Il s'agit d'IgA et de C3.

Quelle autre atteinte faut-il rechercher ?


A - Une atteinte gastrique
B - Une atteinte hépatique
C - Une atteinte rectale
D - Une atteinte intestinale
E - Aucune autre atteinte
Bonne(s) réponse(s) : D

Une entéropathie au gluten est constamment associée. Même en l'absence de stigmates cliniques et biologiques de
malabsorption, la biopsie intestinale montre une atrophie villositaire de degré variable.

1169
Exclusivement sur DOC - DZ : www.doc-dz.com NADJI 85
RESIDANAT EN POCHE TOME II
Cas Clinique en QCM

Quels traitements sont reconnus efficaces dans cette maladie ?


A - Le régime sans gluten
B - Le disulone (Dapsone®)
C - La corticothérapie générale
D - L'Imurel® (azathioprine)
E - Les échanges plasmatiques
Bonne(s) réponse(s) : A B

Les sulfones (disulone) sont le traitement de choix ; le résultat rapide peut être immédiat.

Un jeune homme homosexuel, héroïnomane a depuis 2 mois un prurit à recrudescence nocturne. Son affection a été
initialement étiquetée allergie et traitée par corticothérapie générale sans résultats ; puis par application locale d'antiseptiques.
Les lésions qui se distribuent sur les mains, les aisselles, l'abdomen, les fesses, les organes génitaux externes et les racines
des cuisses sont de plusieurs types : excoriations, lésions suintantes purulentes, lésions en placards érythémato-vésiculeux,
squames. Le dos, la face et le cuir chevelu sont respectés. L'examen met en évidence une micropolyadénopathie diffuse.
Dans l'entourage du patient plusieurs personnes présentent une dermatose purigineuse. Il existe une hématurie
microscopique et une protéinurie. La recherche d'antigène HBS est positive ainsi que le TPHA.
Il existe un déficit de l'immunité cellulaire. Vous portez le diagnostic de gale sarcoptique.

La contamination de ce sujet peut être secondaire à :


A - Un contact par la literie
B - Un rapport sexuel
C - Une origine alimentaire
D - Un contact avec un chien
E - Une injection intra-veineuse
Bonne(s) réponse(s) : AB

Si les contacts cutanés étroits et prolongés avec une personne contaminée représentent le mode de contamination habituel
(rapports sexuels), la gale peut se contracter par de la literie souillée (environ 5 % des cas).

Quelle(s) est (sont) la (les) complications(s) que ce patient semble avoir :


A - Glomérulonéphrite post-infectieuse
B - Eczématisation
C - Impétiginisation
D - Urticaire
E - Immunodépression induite par le sarcopte
Bonne(s) réponse(s) : ABC

Un impetigo chez l'adulte doit toujours faire rechercher une dermatose prurigineuse sous-jacente et en particulier la gale.
Les placards érythémateux et vésiculeux évoquent une eczématisation.
Le sarcopte n'induit pas d'immunodépression, par contre celle-ci favorise le développement des sarcoptes : possibilité de gale
norvégienne.

Quel (s) est (sont) le(s) moyen(s) qui peut(vent) vous permettre d'établir le diagnostic de gale sarcoptique ?
A - Sérologie spécifique
B - Cultures sur milieu spécifique
C - Examen microscopique du produit de grattage d'une lésion
D - Examen en immunofluorescence sur coupe congelée de peau lésionnelle
E - Test thérapeutique avec une solution de benzoate de benzyle (Ascabiol®)
Bonne(s) réponse(s) : CE

On réalise le grattage au mieux au niveau d'une élevure perlée ou d'un sillon. Il n'y a pas de sérologie (parasitisme
épidermique !).

Quelle(s) peut(vent) être la (les) modalité(s) évolutive(s) en l'absence de traitement efficace chez ce patient
immunodéprimé ?
A - Septicémie sarcoptique
B - Eczéma atopique
C - Persistance chronique de la gale
D - Parasitisme intestinal
E - Développement d'une gale norvégienne
Bonne(s) réponse(s) : CE

La gale norvégienne, très croûteuse, très contagieuse (multitude de sarcoptes), souvent peu prurigineuse touche
l'immunodéprimé, le sujet âgé.

1170
Exclusivement sur DOC - DZ : www.doc-dz.com NADJI 85
RESIDANAT EN POCHE TOME II
Cas Clinique en QCM

Comment allez-vous conduire votre traitement :


A - Application unique de Selsun
B - Deux badigeons d'Ascabiol à 24 h. d'intervalle
C - Badigeons d'Ascabiol quotidiens pendant 10 jours
D - Corticothérapie générale
E - Antibiothérapie générale
Bonne(s) réponse(s) : BE

Une antibiothérapie et des soins antiseptiques locaux sont indispensables vu la surinfection.


D est aberrant ! L'ascabiol est irritant et ne doit pas être réappliqué inutilement ; les gales norvégiennes nécessitent des
applications répétées, et des kératolytiques.

Quelle(s) peut(vent) être la (les) modalitée(s) évolutive(s) après traitement efficace ?


A - Persistance du prurit pendant quelques jours
B - Recontamination par l'entourage
C - Guérison
D - Aggravation de l'eczématisation par le traitement
E - Persistance de nodules prurigineux pendant plusieurs semaines
Bonne(s) réponse(s) : ABCDE

Surtout dans les gales anciennes, eczématisées, le prurit peut en effet persister plusieurs jours après le traitement (ou
diminuer puis récidiver), ce qui ne doit pas conduire à des réapplications itératives d'ascabiol. L'ascabiol, irritant, peut
aggraver certaines lésions présentes (eczéma en phase érythémato-vésiculeuse).
Les nodules scabieux peuvent être longs à disparaître (ils représentent une réaction inflammatoire cutanée aux antigènes
parasitaires et ne contiennent pas de sarcoptes).
A distance, certains peuvent nécessiter une corticothérapie locale.

Quelle(s) mesure(s) complémentaire(s) vous parai(ssen)t nécessaires ?


A - Traitement de l'entourage, même en l'absence de prurit
B - Désinfection des vêtements et sous-vêtements
C - Vaccination antisarcoptique
D - Traitement désensibilisant
E - Traitement des animaux familiers
Bonne(s) réponse(s) : AB

Sarcoptes Scabiei est l'agent de la gale, parasite strictement humain. Les vêtements, la literie seront enfermés 48 heures
dans un sac avec de la poudre Aphtiria ou lavés à 60°.

Un patient de 60 ans, éthylique, tabagique consulte pour une plaque blanche située sur le bord gauche de la langue. La lésion
évolue depuis six mois environ. L'examen stomatologique révèle un mauvais état dentaire. Vous faites le diagnostic de
leucokératose.

Quel(s) est (sont) le(s) facteur(s) favorisant l'apparition d'une leucokératose ?


A - Traumatisme local chronique
B - Agression médicamenteuse
C - Infection virale
D - Intoxication tabagique chronique
E - Etat dépressif
Bonne(s) réponse(s) : AD

Les piments, l'alcool sont également des facteurs irritants.

Quel(s) signe(s) serait(aient) en faveur de la transformation maligne de cette leucokératose ?


A - Couleur nacrée
B - Base indurée
C - Augmentation de sa superficie
D - Adénopathie satellite
E - Hémorragie au contact
Bonne(s) réponse(s) : BDE

Une ulcération est également évocatrice de transformation.

1171
Exclusivement sur DOC - DZ : www.doc-dz.com NADJI 85
RESIDANAT EN POCHE TOME II
Cas Clinique en QCM

Le lichen plan est un diagnostic différentiel. Quel est son siège le plus fréquent ?
A - Crête gingivale
B - Face interne des joues
C - Piliers amygdaliens
D - Voile mobile
E - Lèvre rouge
Bonne(s) réponse(s) : B

Le lichen plan buccal a une prédilection pour la langue et la partie postéro-inférieure des joues.

Quel(s) est (sont) le(s) élément(s) thérapeutique(s) habituellement utilisé(s) dans le traitement d'une
leucokératose linguale ?
A - Endocurithérapie
B - Corticoïdes locaux
C - Surveillance régulière
D - Mise en état de la denture
E - Anti inflammatoires non stéroïdiens
Bonne(s) réponse(s) : CD

La destruction de la lésion est impérative (cryothérapie, voire chirurgie selon les cas).
A est trop agressif pour ce type de lésion.
Une surveillance régulière est indispensable (possibilité de récidives, autres lésions). Il faut supprimer les facteurs de risque
(traumatisme, tabac...).

La leucokératose appartient au groupe des lésions muqueuses pré-cancéreuses. Quel(s) est (sont) celle(s) dont
la transformation maligne est inéluctable à plus ou moins long terme ?
A - Leucokératose
B - Lichen plan
C - Erythroplasie de Queyrat
D - Papillomatose orale Floride
E - Leucoplasie
Bonne(s) réponse(s) : C D

La leucokératose est employée pour une leucoplasie hyperkératosique ; leurs étiologies sont diverses (tabagisme, irritation) et
leur transformation possible n'est pas inéluctable. Le lichen buccal (surtout ancien, érosif, atrophique) peut se transformer en
lésion maligne.

Une femme de 37 ans consulte pour un naevus sur le mollet gauche apparu récemment et qui a rapidement augmenté de
volume. A l'examen, vous notez une discrète surélévation sans infiltration cutanée, avec un noircissement localisé du naevus
qui mesure 7 mm x 5 mm. Vous suspectez un mélanome malin et faites réaliser une biopsie exérèse.

En dehors du revêtement cutané, quelles sont les autres localisations primitives possibles du mélanome malin ?
A - Poumon
B - Choroïde
C - Foie
D - Ovaire
E - Muqueuse digestive
Bonne(s) réponse(s) : B E

Le mélanome malin existe également sur les muqueuses buccale, ORL, génitale.

Quel(s) est (sont) parmi cette liste les états précancéreux des mélanomes malins ?
A - Naevus bleu
B - Mélanose de Dubreuilh
C - Naevus jonctionnel
D - Naevomatose gigantocellulaire
E - Naevus de Sutton
Bonne(s) réponse(s) : B

Les naevi jonctionnels (thèques naeviques épidermiques faisant saillie dans le derme superficiel) peuvent évoluer vers un
mélanome (éventualité rare), mais ne peuvent être considérés comme lésions pré-cancéreuses.
B est en fait déjà un mélanome intra-épidermique et devrait être appelé mélanome.

1172
Exclusivement sur DOC - DZ : www.doc-dz.com NADJI 85
RESIDANAT EN POCHE TOME II
Cas Clinique en QCM

La lésion est classée niveau II dans la classification de Clark et MIHM. A quoi cela correspond-il ?
A - Paramètre tenant compte du niveau d'invasion épidermique et dermique
B - Paramètre tenant compte de l'épaisseur tumorale maximale
C - Tumeur entre 0,7 et 2 mm d'épaisseur
D - Effraction de la membrane basale et pénétration du derme papillaire
E - Tumeur de plus de 2 mm d'épaisseur
Bonne(s) réponse(s) : AD

B est l'indice de Breslow.


La classification de Clark tient compte de l'envahissement en profondeur.
Un niveau II signifie que la membrane basale a été franchie et que le derme papillaire est envahi de façon discontinue.

La lésion est stade I de la classification de Breslow qui prend en compte :


A - Le degré de différenciation
B - L'index mitotique
C - L'extension intradermique
D - L'épaisseur maximale
E - L'intensité des embols vasculaires
Bonne(s) réponse(s) : D

L'indice de Breslow est constitué par l'épaisseur de la tumeur ; en dessous de 0,75 mm, les métastases sont très rares.
B et E sont des facteurs de mauvais pronostic.

Un homme de 24 ans consulte pour la première fois pour une dermatose érythémato-squameuse non prurigineuse, faite de
plaques bien limitées de 5 à 20 cm2 de surface, rouges, recouvertes de squames blanches qui se détachent facilement.
L'interrogatoire apprend que l'éruption a commencé plusieurs mois ou années auparavant mais qu'elle se limitait à des
plaques de même type au niveau des faces d'extension des membres. Le patient n'avait appliqué jusqu'alors que des
topiques locaux obtenus sans ordonnance et dont il ne se rappelle plus le nom. Les résultats étaient d'ailleurs nuls ou
médiocres. La poussée actuelle parait avoir été secondaire à une situation professionnelle récemment conflictuelle. On note
par ailleurs un état desquamatif du cuir chevelu et une hyperkératose distale sous unguéale au niveau de quelques doigts.
L'examen somatique est par ailleurs normal. Il ne semble pas y avoir de signes articulaires. L'interrogatoire ne retrouve pas
d'antécédent familial de même type. L'état général est bien conservé. Le patient a eu 3 poussées d'aphtes buccaux au cours
de l'année écoulée. Il a été opéré d'une fissure anale il y a 2 ans. Sa mère est asthmatique.

Quel(s) examen(s) complémentaire(s) confirme(nt) le diagnostic ?


A - Biopsie cutanée
B - Radio de thorax
C - Recherche d'anticorps anti-épiderme
D - Cytodiagnostic
E - Immunofluorescence directe
Bonne(s) réponse(s) : A

Aucun autre examen cité ne peut faire porter le diagnostic de psoriasis. Ceci dit, l'histoire de la maladie et l'examen clinique
très évocateurs ne rendent pas indispensables la biopsie pour porter le diagnostic.

Quel(s) est (sont) le(s) élément(s) qui, dans l'observation, est (sont) en faveur du diagnostic que vous avez
retenu ?
A - Aphtes buccaux
B - Hyperkératose sous unguéale
C - Mère asthmatique
D - Desquamation du cuir chevelu
E - Plaques aux faces d'extension des membres
Bonne(s) réponse(s) : BDE

B D E sont des localisations classiques du psoriasis.

Parmi ces questions posées par votre patient, vous pouvez répondre affirmativement à :
A - Est-ce contagieux
B - Mes enfants risquent-ils d'avoir la même chose
C - Faut-il faire un régime alimentaire
D - Faut-il éviter le soleil
E - L'évolution est-elle prévisible
Bonne(s) réponse(s) : B

Les UVA améliorent les lésions (principe du traitement par la puvathérapie).


Il existe une prédisposition familiale pour le psoriasis. L'évolution chronique est peu prévisible, rythmée souvent par les
épisodes de stress qui déclenchent des poussées.

1173
Exclusivement sur DOC - DZ : www.doc-dz.com NADJI 85
RESIDANAT EN POCHE TOME II
Cas Clinique en QCM

Parmi les traitements suivants, quels sont ceux (ou celui) que vous n'utiliserez surement pas chez ce patient ?
A - Dioxyanthranol
B - Corticothérapie locale
C - Méthotrexate
D - P.U.V.A.
E - Caryolysine locale
Bonne(s) réponse(s) : C E

Il n'est pas question d'utiliser des agents antimitotiques chez ce patient jeune n'ayant pas un psoriasis sévère résistant aux
traitements classiques.
A, B, D sont des traitements possibles ; la décision dépendra de l'extension des lésions, et des possibilités du patient
(disponibilité pour la Puva, niveau de compréhension pour le dioxyanthranol qui est irritant si mal utilisé).
B et D peuvent être associés (par exemple corticothérapie locale et kératolytiques si lésions très squameuses, puis relai par
Puva).

1174
Exclusivement sur DOC - DZ : www.doc-dz.com NADJI 85
RESIDANAT EN POCHE TOME II
Cas Clinique en QCM

1175
Exclusivement sur DOC - DZ : www.doc-dz.com NADJI 85
RESIDANAT EN POCHE TOME II
Cas Clinique en QCM
Monsieur Y. 73 ans, ingénieur en retraite, est hospitalisé à la demande de son généraliste parce que depuis quelques
semaines, il ne fait plus rien ou presque, ne s'intéresse plus à rien et semble par moments "perdre la tête" il prétend avoir une
réunion importante pour le lendemain, ou se croit en déplacement pour son travail. Les disputes avec sa femme sont
fréquentes, pour les motifs les plus mineurs. Il erre la nuit dans son appartement. A l'entretien, Monsieur Y ne sort de son
mutisme que pour lancer quelques phrases inachevées : "ses impôts ne pourront être payés" , "le monde est fichu". Dans les
antécédents, on retrouve : un épisode dépressif sévère il y a 15 ans, et un infarctus du myocarde il y a 9 ans, le patient s'étant
parfaitement rétabli après l'un et l'autre. Le médecin avait prescrit 15 jours auparavant 50 gouttes de Laroxyl®, le soir.

Le(s) diagnostic(s) le(s) plus probable(s) pouvant être évoqué(s) est(sont) celui(ceux) de :
A - Névrose d'angoisse
B - Syndrome dépressif
C - Syndrome confusionnel
D - Simulation
E - Pathologie de couple
Bonne(s) réponse(s) : B C

B - Est évident dans la 2ème partie de l'énoncé.


- thème de ruine
- fin du monde
- antécédent de syndrome dépressif sévère.
Peuvent évoquer une mélancolie.
C - Dans la lère partie de l'énoncé.
A savoir que chez les personnes âgées, les syndromes dépressifs masquent souvent un processus démentiel débutant et
comporte quelquefois une désorientation temporo-spatiale.

Tous les examens complémentaires suivants sont à effectuer sauf un. Lequel ?
A - Bilan hydroélectrolytique
B - E.E.G.
C - P.L. avec étude manométrique
D - Fond d'oeil
E - Doppler artériel encéphalique
Bonne(s) réponse(s) : B

Est ici l'examen qui a le moins d'intérêt mais (C) ne se fait pas sans signes neurologiques.
Q.C.M. difficile.

Le traitement à visée psychotrope essentielle à envisager à court terme comportera :


A - Tranquillisant benzodiazépinique
B - Hypnotique non barbiturique
C - Antidépresseur
D - Neuroleptique desinhibiteur
E - Neuroleptique sédatif
Bonne(s) réponse(s) : C E

A - N'a pas d'intérêt et d'autant que les benzodiazépines sont confusogènes


B - Est avantageusement remplacé par (E) chez les sujets âgés.
L'autre problème des benzodiazépines chez les gens âgés en est leur propriété myorel exante myorelaxante qui les gène
beaucoup.
C - Permet de lever la composante dépressive et améliore le tableau de toute façon.
E - Permet de diminuer les troubles du comportement.

Les deux contre-incidactions majeures du Laroxyl® sont :


A - Glaucome à angle fermé
B - Ulcère gastroduodénal
C - Psoriasis
D - Troubles de la conduction auriculo-ventriculaire
E - Hypoglycémie
Bonne(s) réponse(s) : A D

A - Effet mydriatique parasympathyquolylique des anti-dépresseurs tricycliques.


D - Par toxicité : prudence, prudence...

1176
Exclusivement sur DOC - DZ : www.doc-dz.com NADJI 85
RESIDANAT EN POCHE TOME II
Cas Clinique en QCM

Après amélioration ou guérison de l'épisode actuel, un examen psychométrique permettrait éventuellement :


A - De mettre en évidence une détérioration pathologique
B - De comprendre pourquoi Monsieur Y. fait des dépressions
C - De préciser l'impact de la mise à la retraite dans le vécu du sujet
D - De guider la posologie du traitement
E - D'apprécier l'organisation temporo spatiale
Bonne(s) réponse(s) : A

Q.C.M. difficile.
A - Est évident et intéressant d'autant plus si on possède un test antérieur. Il faut refaire le test pour suivre l'évolution de la
détérioration.
C - Particulièrement énigmatique.
E - Ne veut rien dire.

Sylvie, célibataire âgée de 25 ans est amenée au service de porte de l'hôpital en proie à une angoisse intense. Elle demande
de l'aide ; soumise à l'influence de la télépathie, des êtres mystérieux la torturent par les rayons qu'ils lui envoient, ils lui volent
sa pensée, on la transforme en sorcière, des voix inconnues lui hurlent des injures et lui imposent des actes ou des attitudes
absurdes. Cet état dure depuis plusieurs années, depuis le jour où une voix lui a fait comprendre en se révélant à elle la cause
du malaise intérieur qu'elle éprouvait obscurément, elle fait du spiritisme et revient d'un pèlerinage à Lourdes. L examen
clinique est négatif, cependant il existe une grossesse d'environ 18 semaines

Quel est le nom du syndrome évoqué ci-dessus ?


A - Syndrome obsessionnel
B - Idées délirantes
C - Automatisme psychologique
D - Automatisme mental
E - Syndrome de conversion hystérique
Bonne(s) réponse(s) : D

Q.C.M. vicieux.
A - Ne veut rien dire ici.
B - Bien sûr, mais soyons plus précis.
C - Jeu de mots : mental et non psychologique.
E - Complètement à côté.

Parmi les suivants, quel diagnostic peut être retenu ?


A - Schizophrénie
B - Bouffée délirante polymorphe
C - Episode psychotique gravidique
D - Pharmaco psychose
E - Psychose hystérique
Bonne(s) réponse(s) : A

N'est affirmé que sur l'évolution du syndrome, de l'âge, sans la notion de troubles sur plusieurs années.
BCD - Eléments possibles.

Quelle est la conduite à tenir ?


A - Adresser la malade au dispensaire de son secteur
B - Adresser la malade à son médecin traitant
C - Hospitalisation
D - Prise de mesures médico-légales
E - Lui conseiller de consulter une assistante sociale
Bonne(s) réponse(s) : C

Ne se discute pas. Il faut :


- instaurer un traitement
- calmer l'angoisse
- surveiller la grossesse.

Compte tenu de la sémiologie présentée et de la gravidité, quelle est la classe médicamenteuse à utiliser si l'on
prescrit un traitement ?
A - Antidépresseur
B - Bêtabloquant
C - Neuroleptique
D - Normothymique
E - Thymérétique
Bonne(s) réponse(s) : C

On emploiera de préférence le Largactil®. La grossesse est supérieure à 3 mois donc n'est plus une contre-indication.
1177
Exclusivement sur DOC - DZ : www.doc-dz.com NADJI 85
RESIDANAT EN POCHE TOME II
Cas Clinique en QCM

Quelle(s( attitude(s) adopter vis-à-avis de la grossesse ?


A - Suivi obstétrical
B - Interruption thérapeutique de grossesse
C - Evolution spontanée de la grossesse
D - Suivi psychiatrique
E - Liaison avec le service de PMI
Bonne(s) réponse(s) : A C D E

Sans commentaire.

Agée de 41 ans, Marina, mère de deux enfants de 8 et 10 ans est conduite à l'hôpital sur les conseils du médecin de famille :
sa préoccupation excessive du ménage l'amène à négliger le reste de ses activités domestiques, son mari l'appelle la "mère
ménage" ; la vie familiale est paralysée par les violentes colères suscitées par le va-et-vient quotidien et les accusations de
porter atteinte à la propreté de l'appartement si soigneusement entretenu. Elle ne peut plus faire ses courses car dès qu'elle
sort, elle se sent obligée de faire aussi le ménage de la rue, ramassant sans fin les papiers gras et les détritus qui jonchent
inévitablement le sol au point qu'on est obligé de la ramener chez elle. Elle a conscience du caractère insolite de sa conduite
"mais c'est plus fort que moi, je ne puis pas m'en empêcher" dit-elle, Son mari menace de divorcer car il ne la croit pas
malade, point de vue que soutiennent ses parents.

Dans quel type d'affection ranger ce comportement ?


A - Névrose phobique
B - Délire chronique
C - Névrose obsessionnelle
D - Psychopathie
E - Débilité mentale
Bonne(s) réponse(s) : C

On retrouve :
- les rituels
- les obsersions
- le sentiment de morbidité.
A - Eliminer par absence d'objet phobogène et conduite contraphobique.
B - Aucun élément délirant.
DE - Rien à voir.

Quel est le symptôme caractéristique ?


A - Phobie d'impulsion
B - Obsession idéique
C - Obsession impulsion
D - Compulsion
E - Rituel conjuratoire
Bonne(s) réponse(s) : E

Q.C.M. difficile.
CD - Sont vrais mais le symptôme caractéristique en est le rituel.

Sur quel(s) élément(s) clinique(s) caractéristique(s) se fonde(nt) le diagnostic positif ?


A - Détérioration du climat familial
B - Amaigrissement récent
C - Sentiment de contrainte intérieure
D - Réactions thymiques
E - Conscience du caractère morbide
Bonne(s) réponse(s) : C E

C - Décrit dans la névrose obsessionnelle soulagée par le rituel.


E - Dans toute névrose.

Parmi les différentes modalités de traitement vous proposez initialement :


A - Consultations ambulatoires
B - Hospitalisation complète
C - Hospitalisation partielle
D - Séjour en maison de repos
E - Hospitalisation à domicile
Bonne(s) réponse(s) : B

"Initialement" permet de cocher cet item.


ACD - Seront possibles ensuite, dans un second temps.
B - Permet la mise en place d'un traitement.

1178
Exclusivement sur DOC - DZ : www.doc-dz.com NADJI 85
RESIDANAT EN POCHE TOME II
Cas Clinique en QCM

Parmi les traitements proposés, quel est celui qui doit être prescrit dans l'immédiat ?
A - Cure type psychanalytique
B - Traitement psychochirurgical
C - Traitement antidépresseur tricyclique
D - Traitement électrothérapique
E - Traitement ergothérapique
Bonne(s) réponse(s) : C

Seul traitement ayant fait preuve d'une efficacité immédiate.


Car un syndrome dépressif est souvent contemporain, majorant les syndromes névrotiques.

Monsieur P. a 44 ans Depuis l'âge de 18 ans il se plaint de pensées impérieuses qui assaillent son esprit dans un contexte
d'anxiété et d'irritation. Dans les rues il ne peut éviter de compter le nombre de maison qui brodent les trottoirs. Il se pose
d'incessantes questions sur la nature des matériaux de divers objets courants (fer à repasser, casserole, meubles etc). Quand
il signe un chèque il s'interroge pendant un quart d'heure pour savoir Si sa signature est authentique. Il a cessé toute activité
professionnelle à l'âge de 22 ans et vit d'une rente allouée par ses parents, propriétaires d'un hôtel Au cours de l'entretien, le
contact est aisé. Il parle facilement, il vient d'effectuer sans changement un essai de psychothérapie analytique pendant 8
mois. Le récit de son existence est clair, cohérent, mais exprimé avec froideur, sans émotion. L'humeur est syntone.

Quel diagnostic psychiatrique évoquez-vous ?


A - Névrose obsessionnelle
B - Névrose phobique
C - Schizophrénie simple
D - Dépression névrotique
E - Névrose hystérique
Bonne(s) réponse(s) : A

Névrose devant anxiété et l'irritation obsessionnelle :


- rituels
- doute permanent.
B - Pas d'objet phobogène.
C - Pas de trouble du contact ni de délire même à minima.
DE - Sans objet.

Compte tenu de l'âge de début des troubles, quel est le principal diagnostic différentiel ?
A - Névrose d'angoisse
B - Névrose traumatique
C - Schizophrénie pseudonévrotique
D - Etat limite
E - Psychopathie dysharmonique
Bonne(s) réponse(s) : C

Mais l'évolution est très différente et à l'âge du patient -44 ans- il n'existe aucun signe de la lignée schizophrénique. Mais il
faut toujours y penser chez le sujet jeune.

La(les) mécanisme(s) de défense inconscient(s) mis en jeu dans ce type de troubles est ou sont :
A - Indentification projective
B - Déni
C - Isolation
D - Refoulement
E - Dénégation
Bonne(s) réponse(s) : C D

Il faut le savoir !

Dans le cadre des théories psychanalytiques classiques cette observation témoigne d'une fixation à un stade du
développement. Lequel ?
A - Période pré-oedipienne
B - Stade schizoparanoïde
C - Stade oral
D - Période dépressive
E - Stade anal
Bonne(s) réponse(s) : E

Il faut également le savoir !

1179
Exclusivement sur DOC - DZ : www.doc-dz.com NADJI 85
RESIDANAT EN POCHE TOME II
Cas Clinique en QCM

La résistance au changement psychothérapique étant notoire, parmi les types de médicaments psychotropes on
peut attendre une efficacité des :
A - Neuroléptiques dérivés des butyrophénones
B - Benzodiazépines à demi-vie courte
C - Neuroleptiquesretard
D - Antidépresseurs tricycliques
E - Bêta-bloquants
Bonne(s) réponse(s) : B D

Mais (B) n'est qu'un traitement d'appoint qui permet de soulager l'angoisse de manière ponctuelle.
D - Ont fait preuve d'efficacité car le syndrome dépressif est souvent contemporain et la symptomatologie névrotique.

Un adolescent de 17 ans est vu en consultation pour des troubles apparus progressivement depuis 8 mois : désintérêt pour
ses études, moins grande communication au sein de sa famille, tendance à s'isoler, colères impulsives sans cause
déclenchante, accès de rires immotivés. Ses parents signalent de longues périodes de contemplation devant une glace et des
plaintes concernant son apparence physique. Lors de l'entretien, il exprime le sentiment que les gens et les choses lui
paraissent transformés et bizarres, qu'il a l'impression que les os de son visage s'allongent et le rendent désagréable à
regarder. On lui envoie de l'extérieur des pensées qui ne sont pas les siennes. Quand il lit, une voix intérieure répète sa
lecture. Il a quelque fois envie de disparaître pour en finir avec cette souffrance qui le submerge.

Parmi les symptômes suivants, quel est celui(ceux) qui est(sont) présent(s) dans cette observation ?
A - Discordance
B - Déréalisation
C - Automatisme mental
D - Hallucination psychiques
E - Phobies d'impulsion
Bonne(s) réponse(s) : A B C D

A - Fait partie du syndrome dissociatif.


B - Evident.
C - "Une voix répète sa lecture".
D - Idem que (C).
E - Non, il y a ici des impulsions clastiques.

La répétition par une voix intérieure de la lecture évoque spécifiquement l'un des symptômes suivants :
A - Interprétation délirante
B - Hallucinations psycho-sensonelle
C - Idées obsédantes
D - Automatisme mental
E - Déréalisation
Bonne(s) réponse(s) : D

C'est la définition. On retrouve associé le commentaire de la pensée et des actes.

Parmi les états pathologiques suivants, quel est celui que cette observation évoque en premier lieu :
A - Crise d'originalité juvénile
B - Dépression névrotique
C - Schizophrénie.
D - Névrose phobique
E - Névrose d'angoisse
Bonne(s) réponse(s) : C

Devant le tableau décrit et la persistance de celui-ci à 8 mois.

Parmi les traitements suivants, le(s)quel(s) vous paraît(paraissent) le(s) plus approprié(s) au cas de ce patient ?
A - Anafranil® (clomipramine)
B - Largactil® (chlorpromazine)
C - Haldol® (halopéridol)
D - Valium® (diazépam)
E - Marsilid® (I.M.A.O.)
Bonne(s) réponse(s) : B C

L'association d'un neuroleptique sédatif et d'un anti-délirant à dose majeure, le plus souvent en parentéral (intramusculaire)
est ici d'une bonne indication.
Le syndrome dépressif décrit dans l'énoncé diminuera avec les symptômes délirants et n'est pas l'indication d'antidépresseurs
qui risquent au départ de réactiver le délire.

1180
Exclusivement sur DOC - DZ : www.doc-dz.com NADJI 85
RESIDANAT EN POCHE TOME II
Cas Clinique en QCM

Parmi les propositions suivantes concernant l'évolution, laquelle ou lesquelles est ou sont exacte(s) ?
A - Guérison par séparation de la famille
B - Alternance de phases de rémission et d aggravation
C - Survenue d'épisodes processuels
D - Impulsions suicidaires
E - Evolution chronique lit
Bonne(s) réponse(s) : B C D E

B - Les plages de rémission seront de plus en plus courtes.


C - Bien sûr, puisque c'est une maladie chronique évoluant par phase donc, (E).
D - Très fréquente chez les schizophrènes, lors d'épisode délirant ou d'angoisse psychotique intense.

Madame X. a 35 ans. Elle a été hospitalisée dans l'unité de réanimation de l'hôpital après une tentative de suicide par
ingestion médicamenteuse.
Il s'agissait d'Anafranil® à 25mg et de Tranxène® 50.
Le mari, avisé par sa fille âgée de 10 ans, a dit que ces médicaments avaient été prescrits, il y a deux ans, par un généraliste
parce que sa femme était fatiguée nerveuse et insomniaque.
La cause de la tentative de suicide réside dans le fait que le mari de Madame X vient d'avoir une aventure : celle-ci a été
découverte et il a été question de divorce. La patiente exprime des idées de culpabilité pour avoir tenté de se tuer devant sa
fille qu'elle estime être gravement menacée de ce fait. Elle pense qu'elle n'est pas digne d'être appelée mère et que son mari
a bien fait de la tromper.

Un des diagnostics énumérés ci-dessous est juste. Le cocher :


A - Hystérie
B - Schizophrénie
C - Bouffée délirante
D - Accès dépressif
E - Idées obsédantes
Bonne(s) réponse(s) : D

Cela ne préjuge en rien de la structure sous-jacente.

La prescription de comprimés de Tranxène® 50 comporte un danger. Le cocher :


A - Epilepsie
B - Insuffisance rénale
C - Assuétude
D - Hypersomnie
E - Lipothymie
Bonne(s) réponse(s) : C

Comme toutes les benzodiazépines avec risque de sevrage à l'arrêt.

Tous les éléments suivants orientent le diagnostic, sauf un. Lequel ?


A - L'âge de la patiente
B - Le fait que son mari l'a trompée
C - La tentative de suicide
D - Le fait que son médecin lui avait prescrit du Tranxène®
E - Ses idées de culpabilité et d'indignité
Bonne(s) réponse(s) : D

Ne veut rien dire. La prescription d'un anxiolytique n'oriente aucun diagnostic.


C - Fait partie du processus dépressif.
B - Elément réactionnel.
E - Syndrome dépressif.

Le diagnostic actuel à porter est finalement :


A - Chantage au suicide
B - Etat dépressif périodique
C - Etat dépressif masqué
D - Tentative de suicide dû à une dépression
E - Idée délirante de jalousie
Bonne(s) réponse(s) : D

Question de bon sens.


A - Ne veut rien dire.

1181
Exclusivement sur DOC - DZ : www.doc-dz.com NADJI 85
RESIDANAT EN POCHE TOME II
Cas Clinique en QCM

Quelle est ou quelles sont, parmi les mesures suivantes, celle ou celles qui doivent être adoptées ?
A - Sortie de l'hôpital
B - Reprise du traitement antérieur
C - Surveillance par le généraliste
D - Consultation en psychiatrie
E - Transfert dans un service de psychiatrie en placement volontaire
Bonne(s) réponse(s) : D

A - Peut être envisagé après (D) en fonction du risque.


B - On ne connait pas les doses prescrites.
C - Pourrait être coché mais un soutien psychiatrique est nécessaire vu le geste.
E - Semble exagéré. A peser en fonction du risque et de la patiente.

Cette femme de 44 ans récemment installée dans son immeuble, au cours d'un entretien avec l'interne de garde, dit : "tout a
commencé par un sifflement, c'est là que j'ai compris qu'ils avaient installé des fils électriques dans le grenier pour
m'espionner, le voisin en fait partie ; ils me poussent à faire ceci ou cela ; quand je pense, ils répètent aux autres mes
pensées ; c'est elle que j'entends quand ils parlent ; hier toute la soirée ils ont parlé de moi".
L'attitude est calme pendant l'entretien, comme elle était calme quand cette femme est allé trouver le voisin pour lui annoncer
que s'il continuait ainsi, elle aurait à se défendre. L'interne propose une hospitalisation que la patiente refuse.

A quel ou quels syndromes psychopathologiques correspond ce cas clinique ?


A - Dramatisation chez une hystérique
B - Onirisme
C - Obsession
D - Automatisme mental
E - Syndrome d'influence
Bonne(s) réponse(s) : D E

Ces deux syndromes sont souvent liés l'un à l'autre. Ici, on retrouve :
- écho de la pensée
- commentaires idéiques
- pensées et actes imposes.
B - Non, car l'onirisme -dans un autre contexte- est constitué d'hallucinations sensorielles souvent visuelles.

Quel diagnostic retenez-vous ?


A - Délire paranoïaque
B - Schizophrénie
C - Délire onirique
D - Confusion mentale
E - Psychose hallucinatoire chronique
Bonne(s) réponse(s) : E

Le tableau est typique :


- Terrain : femme de 45 ans, récemment emménagée
- Hallucinations auditives
- Syndrome d'influence et automatisme mental.
Seul (A) peut être discuté mais le tableau hallucinatoire l'élimine.

Si un placement volontaire est indiqué, quelle est ou quelles sont les formalités nécessaires ?
A - Il faut un certificat du maire ou du commissaire de police
B - Il faut un certificat d'un médecin assermenté
C - Il faut une demande écrite de l'entourage
D - Il faut un certificat d'un médecin extérieur à l'hôpital
E - Il faut un arrêté préfectoral
Bonne(s) réponse(s) : C D

A - Concerne le placement d'office ainsi que (E).


B - Tout docteur ou médecin peut le rédiger s'il est extérieur à l'hôpital de placement,.et non parent jusqu'au 2ème degré avec
le demandeur et le directeur de l'établissement.

1182
Exclusivement sur DOC - DZ : www.doc-dz.com NADJI 85
RESIDANAT EN POCHE TOME II
Cas Clinique en QCM

Quelle chimiothérapie est adaptée au traitement immédiat ?


A - Sels de lithium
B - Neuroleptiques incisifs
C - Cure de sommeil
D - Neuroleptiques retard
E : - Antidépresseurs à doses réduites
Bonne(s) réponse(s) : B

E - Est le seul discutable car il existe toujours une tonalité dépressive mais "doses réduites" l'élimine.
D - Pas dans l'immédiat.

Quel médicament peut être utile dans le cadre d'un suivi au long cours ?
A - Carbonate de lithium (Téralithe®)
B - Modecate®
C - Valium®
D - Anafranil®
E - Laroxyl®
Bonne(s) réponse(s) : B

Neuroleptique retard incisif avec peu d'effet sédlatif.


A - Sans objet.
DE - Uniquement si persiste une tonalité dépressive.

Une jeune femme de 24 ans, parlant volontiers, détaillant ses troubles, vient consulter pour des sensations de vertige et
d'instabilité à la marche survenues il y a 3 semaines, qui la gênent surtout lorsqu'elle est a l'extérieur de chez elle. L'examen
somatique ne permet pas de déceler de troubles neurologiques ni circulatoires.
Elle nous apprend, par ailleurs, qu'elle a été hospitalisée à trois reprises ces dernières années, pour des douleurs digestives
ayant abouti à une appendicectomie.
La suite de l'entretien permet de se rendre compte qu'elle vient de prendre un emploi, il y a un mois, dans une entreprise
nouvelle et qu'elle a dû quitter le domicile de ses parents pour prendre une chambre en ville.

Quel diagnostic retenez-vous ?


A - Névrose d'angoisse
B - Hystérie de conversion
C - Névrose phobique
D - Dépression névrotique
E - Hypocondrie délirante
Bonne(s) réponse(s) : B

C'est la classique astasie-abasie, on retrouve :


- une personnalité hystérique
- aucune atteinte lésionnelle objectivable (définition de la conversion)
- le facteur déclenchant.
E - Non, car il y a ici des troubles, ce qui n'existe pas dans l'hypocondrie.

Quel(s) autre(s) symptôme(s) peu(ven)t être observé(s) ?


A - Délire hypocondriaque
B - Confusion mentale
C - Théâtralisme
D - Crises névropathiques
E - Discordance
Bonne(s) réponse(s) : C D

C - Fait partie de la personnalité hystérique.


D - Sont des troubles des caractères "crise de nerf" retrouvée souvent chez les hystériques.

Le(s) trouble(s) sexuel(s) le(s) plus fréquent(s) dans un tel cas est (sont) :
A - Frigidité
B - Exhibitionnisme
C - Aménorrhée
D - Homosexualité
E - Dyspareunie
Bonne(s) réponse(s) : A E

Troubles fréquents dans cette pathologie.


B - Concerne surtout les obsessionnels (compulsion).
C - Non.
D - C'est un autre problème.

1183
Exclusivement sur DOC - DZ : www.doc-dz.com NADJI 85
RESIDANAT EN POCHE TOME II
Cas Clinique en QCM

Parmi les mécanismes psychodynamiques susceptibles d'expliquer de tels symptômes, lequel ou lesquels
retenez-vous ?
A - Refoulement
B - Déplacement
C - Projection
D - Délire
E - Sublimation
Bonne(s) réponse(s) : A B

A - Bien sûr.
B - Le déplacement sur le corps du conflit inconscient.
E - Concerne les personnes non névrosées.

Comme traitement vous proposez :


A - Neuroleptique incisif
B - Thérapie comportementale
C - Tranquillisant
D - Entretiens psychothérapiques
E - Enquête sociale en vue d'une action de soutien
Bonne(s) réponse(s) : C D

B - Est de peu d'intérêt ici.


D - Entretien d'inspiration analytique, voire psychanalyse.
C - Permet de diminuer les tensions si elles existent, il ne faut pas oublier "la belle indifférence".

Une jeune fille lycéenne, est arrêtée dans la ville alors qu'elle s'est volontairement installée sur la chaussée, perturbant la
circulation. Elle apparait très excitée, très exaltée. Ses propos sont mal ordonnés, pas toujours compréhensibles.
Tout ce qu'elle a fait dit-elle, était destiné à lutter et à conjurer un sort qu'elle sent sur elle depuis un certain temps et dont elle
perçoit les effets par des idées étrangères imposées, des transformations dans la forme de son corps, mais l'origine de cela
lui apparaît inconnue. Elle ne peut en indiquer le début les auteurs, les motifs.
La famille fait savoir que la malade avait disparu depuis 3 - 4 jours, que son comportement jusqu'ici avait toujours été normal.
Les résultats scolaires sont satisfaisants et au lycée, la malade paraît bien intégrée.

Cette symptomatologie doit faire discuter :


A - Un état maniaque
B - Une schizophrénie
C - Une bouffée délirante aiguë
D - Une crise d'originalité juvénile
E - Une pharmacopsychose
Bonne(s) réponse(s) : A C E

Peuvent être discuté.


A - On retrouve l'exaltation de l'humeur avec excitation.
Le délire, syndrome d'influence-transformation corporelle, peut s'intégrer à une manie délirante.
C - Bien évidemment, c'est ici le diagnostic.
E - Certaines toxiques, hallucinogènes en particulier, peuvent donner ce tableau.
B - Ne sera pas discuté dans l'immédiat mais suivant l'évolution.

L'evolution à long terme est habituellement :


A - La guérison totale sans séquelle
B - Une psychose maniaco-dépressive
C - L'amélioration mais avec des séquelles durables portant sur le fonctionnement intellectuel
D - La guérison avec des rechutes de même type
E - Toutes les évolutions sont possibles
Bonne(s) réponse(s) : A

E - Bien sûr on peut avoir :


- une bouffée délirante polymorphe ex abrupto
- une évolution schyzophrénique
- une évolution dysthynique.

1184
Exclusivement sur DOC - DZ : www.doc-dz.com NADJI 85
RESIDANAT EN POCHE TOME II
Cas Clinique en QCM

Parmi les décisions thérapeutiques suivantes, laquelle ou lesquelles sont indiquées ?


A - Hospitalisation en milieu psychiatrique
B - Neuroleptiques
C - Psychothérapie ambulatoire
D - Tranquillisants
E - Hospitalisation en milieu médical pour bilan
Bonne(s) réponse(s) : A B

Evident.

La sémiologie du tableau décrit comporte essentiellement 2 éléments ; Lesquels ?


A - Des troubles du cours de la pensée
B - Un automatisme mental
C - Des obsessions
D - Des troubles de l'humeur
E - Des paramnésies
Bonne(s) réponse(s) : A D

On ne retrouve pas d'automatisme mental ici mais, un syndrome d'influence. Les propos désordonnés, incohérents signent les
troubles du cours de la pensée.

Parmi les médicaments envisagés pour le traitement, lequel choisissez-vous ?


A - Naloxone
B - Phénothiazines
C - Benzodiazépine
D - Lithium
E - Dépamide®
Bonne(s) réponse(s) : B

Neuroleptiques sédatifs.

Robert, 50 ans, est hospitalisé pour une tentative de suicide médicamenteuse grave.
Antécédents : Hospitalisé il y a dix ans pour dépression, Un grand-père décédé de suicide.
Sa fille aînée s'est mariée il y a 8 mois. Travaille dans le commerce. Depuis un mois, il se plaint de fatigue, de céphalées,
d'insomnie marquée le matin, d'un manque de goût pour ses activités habituelles. Le visage est soucieux, la parole monotone
et les phrases courtes, parfois laconiques. Inactif depuis deux semaines, il confie "qu'il doit payer"', "qu'il a assez fait souffrir"
les siens, que "ses intestins sont bouchés". Il se croit incurable, s'inquiète de son commerce. Son médecin lui a prescrit des
fortifiants et des benzodiazépines. Examen normal, hormis une constipation sévère. Examens paracliniques normaux. Le
lendemain, prostré, on n'obtient de lui que cette parole "ils m'ont condamné".

En vous fondant sur le discours de ce patient (expressions comprises entre les guillemets, exclusivement,
quel(s) symptôme(s) psychiatrique(s) releveriez-vous parmi les suivants ?
A - Idées d'indignité
B - Idées de culpabilité
C - Thèmes de revendication
D - Plaintes hypocondriaques
E - Thèmes de persécution
Bonne(s) réponse(s) : A B D

E - Non, car ce ne sont pas des thèmes de persécutions, "ils m'ont condamné" est une culpabilité délirante.

Parmi les symptômes suivants, relevés chez ce patient, le syndrome d'inhibition psychomotrice (ou
ralentissement psychomoteur) est directement responsable de :
A - Céphalées
B - Constipation
C - Parole monotone
D - Prostration
E - Perte du goût pour les activités habituelles
Bonne(s) réponse(s) : B C D E

B - En fait classiquement partie.


E - "Asthénie vitale".

1185
Exclusivement sur DOC - DZ : www.doc-dz.com NADJI 85
RESIDANAT EN POCHE TOME II
Cas Clinique en QCM

En fonction des éléments en votre possession, quel diagnostic porteriez-vous ?


A - Dépression réactionnelle
B - Dépression secondaire à une affection psychotique organique
C - Dépression mélancolique
D - Dépression masquée
E - Dépression secondaire aux benzodiazépines
Bonne(s) réponse(s) : C

La sévérité du tableau dépressif avec la note délirante et les antécédents permettent le diagnostic de mélancolie.

Parmi les propositions suivantes concernant la décision à prendre après réanimation, laquelle ou lesquelles sont
correctes ?
A - Retour au domicile sous la surveillance du médecin
B - Transfert dans le service de psychiatrie de l'hôpital général
C - Placement volontaire en hôpital psychiatrique sous le régime de la loi du 30.6.38
D - Consultation au Centre de Santé Mentale de son secteur
E - Hospitalisation à domicile
Bonne(s) réponse(s) : B C

Si (B) n'est pas souhaité par le patient, il faut recourir à (C) vu le risque suicidaire majeur.

Parmi les propositions suivantes concernant le traitement médicamenteux, laquelle est exacte ?
A - Neuroleptique sédatif
B - Antidépresseurs tricycliques et neurolytiques sédatifs
C - Poursuite du traitement tranquillisant
D - Traitement psychostimulant
E - Neuroleptique désinhibiteur
Bonne(s) réponse(s) : B

Les antidépresseurs tricycliques sont évidemment indispensables ici. Les neuroleptiques sédatifs permettront de calmer
l'angoisse et de diminuer le risque de levée d'inbibition anxieuse.

Votre attitude thérapeutique pratique, à plus long terme, comportera :


A - Simple poursuite du traitement antérieur
B - Prescription ambulatoire de tricycliques
C - Envoi en maison de repos
D - Post-cure dans le cadre du service de psychiatrie de secteur
E - Psychothérapie ambulatoire
Bonne(s) réponse(s) : B D E

(C est discutable).
B - Pendant 6 à 12 mois en diminuant les dose après 6 mois de traitement.
DE - Sont évidents.

Madame X. 54 ans, selon les dires de ses voisins qui ont appelé le médecin, est en quelques années devenue totalement
incapable de vivre seule.
Elle se nourrit de manière incohérente, elle est incontinente, ne reconnaît plus ses proches, ne sait plus s'habiller seule Elle
laisse ses casseroles sur le feu et représente un danger pour son entourage.
A l'examen, elle apparaît négligée, passive et indifférente, elle est incapable de répondre aux questions les plus simples
concernant son nom, son âge, son lieu d'habitation.

La symptomatologie prédominante dans ce tableau est représentée par :


A - La détérioration des fonctions intellectuelles
B - Les troubles de l'humeur
C - Les troubles des fonctions symboliques
D - L'autisme et le retrait social
E - Les troubles somatiques
Bonne(s) réponse(s) : A C

A - Troubles de la mémoire rencontrés ici - Agnosie.


C - Apraxie - incontinence.

1186
Exclusivement sur DOC - DZ : www.doc-dz.com NADJI 85
RESIDANAT EN POCHE TOME II
Cas Clinique en QCM

Le diagnostic le plus probable est :


A - Une hydrocéphalie à pression normale
B - Une démence artériopathique
C - Une maladie d'Alzheimer
D - Une mélancolie d'involution
E - Une démence sénile
Bonne(s) réponse(s) : C

A - Ne peut être évoqué à l'énoncé et (B) non plus.


D - On ne retrouve pas ici de syndrome dépressif quoiqu'il soit surement présent.
E - L'âge de la patiente l'élimine.
ici on retrouve la célèbre apraxo-agnosie de (C), l'âge est un argument supplémentaire.

Une aide au diagnostic clinique peut être apportée par :


A - Des tests psychométriques
B - Des tests projectifs
C - La tomodensitométrie
D - Le transit isotopique du LCR
E - Bilan artériographique cérébral
Bonne(s) réponse(s) : A C

A - Objectivera la détérioration.
C - Montre une atrophie corticale pariéto temporale avec dilatation ventriculaire.
D - N'est intéressant qu'avec des arguments cliniques en faveur d'une hypertension intracrânienne à pression normale.
E - Idem pour la démence arthériopathique (pas d'argument clinique).

Au pain des mesures d'assistance il est justifié : d'envisager :


A - Hospitalisation à domicile
B - Attribution d'une allocation adulte handicapé
C - Curatelle psychiatrie de secteur
D - Tutelle
E - Sauvegarde de justice
Bonne(s) réponse(s) : D

Q.C.M. difficile.
Il faut hospitaliser cette patiente en milieu médico psychiatrique donc (A) est faux.
E - Sera mis en place, mais uniquement en attendant le jugement de tutelle.

On peut attendre une efficacité thérapeutique de :


A - Rééducation du langage
B - Antidépresseur
C - Vasodilatateur cérébraux
D - Anticholinergiques
E - Ergothérapie
Bonne(s) réponse(s) : E

Aucun médicament n'est efficace.


E - Permet une légère résocialisation.

Divorcé depuis de nombreuses années, Mr X. 60 ans, a de gros problèmes financiers dont il rend ses enfants responsables
(ceux-ci cherchent à le déposséder de ses biens). Il fait un exposé de ses malheurs :sa mère pour se débarrasser de sa
femme, aurait fait interner arbitrairement celle-ci avec la complicité d'un médecin. Ce dernier continue à lui nuire, dressant ses
enfants contre lui, en essayant de le faire interner. Il soupçonne également sa bru de n'être pas étrangère à la mort
accidentelle de son fils. Ne s'était -elle pas fait constituer une assurance vie à son profit quelques temps avant ?.
Ces allégations sont sans fondements, Mr X est décidé à intéresser la justice à cette affaire, sinon il fera justice lui-même.

La ou les caractéristique(s) de ce délire est ou sont :


A - La systématisation
B - Les mécanismes Hallucinatoires
C - La structure paranoïde
D - Les mécanismes interprétatifs
E - Le caractère passionnel
Bonne(s) réponse(s) : A D

A - Evident, c'est un des caractères du délire paranoïaque.


BC - Faux.
D - Evident, interprétation délirante.
E - Non, c'est ici un délire d'interprétation.
1187
Exclusivement sur DOC - DZ : www.doc-dz.com NADJI 85
RESIDANAT EN POCHE TOME II
Cas Clinique en QCM

L'évolution spontanée de ces délires est caractérisée par :


A - L'aggravation sur un mode déficitaire
B - L'importance du risque suicidaire
C - Le risque d'actes médico-légaux
D - La mise à distance du délire et son enkystement sans véritable critique
E - La chronicité avec conservation du caractère actif du délire mais sans déficit intellectuel
Bonne(s) réponse(s) : C D E

A - Le propre de la schizophrénie.
B - Est possible, mais ce n'est pas un risque important (en fréquence), plutôt (C) (acte hétéro-agressif).
D - On a, en général, une certaine cicatrisation du délire.

La personnalité préexistante à ce type de délire est caractérisée par :


A - L'immaturité affective et la suggestibilité
B - La méticulosité
C - L'orgueil
D - L'hypertrophie du Moi
E - La méfiance
Bonne(s) réponse(s) : C D E

Personnalité paranoïaque, on y ajoute la rigidité et la fausseté du jugement.

Le(s) mécanisme(s) psychopathologique(s) invoqué(s) dans ce type de délire est ou sont :


A - Le déplacement d'une anxiété d'origine inconsciente
B - La projection
C - L'isolation
D - L'absence de limite du Moi
E - La conversion
Bonne(s) réponse(s) : B

E - Il faut le savoir.

Les neuroleptiques :
A - Sont le traitement de choix de ces états, qu'ils corrigent totalement
B - Permettent une mise à distance du délire et une diminution des réactions affectives sans entamer la
conviction délirante
C - Ont peu d'action sur ce type de délire
D - Doivent être poursuivis indéfiniment
E - Doivent être associés à des antidépresseurs tricycliques
Bonne(s) réponse(s) : B C D

A - Est faux, les neuroleptiques atteignent difficilement ce type de délire.


B.C. - Sont vrais.

Le cadre nosographique auquel est rattaché ce type de délire est :


A - Les psychoses hallucinatoires chroniques
B - Les psychoses paranoïdes
C - Les psychoses paranoïaques
D - Les psychoses réactionnelles
E - Les névroses obsessionnelles
Bonne(s) réponse(s) : C

Sans commentaire.

1188
Exclusivement sur DOC - DZ : www.doc-dz.com NADJI 85
RESIDANAT EN POCHE TOME II
Cas Clinique en QCM
Un homme de 25 ans se présente au service d'urgence amené par des amis pour un tableau associant des douleurs
musculaires, des frissons, des vomissements, de la diarrhée, une asthénie intense et des tremblements. A l'examen il
présente un foie un peu augmenté de volume, une température à 38 degrés, des polyadénopathies, un souffle systolique et
des multiples traces d'injections aux plis des coudes. Il demande à être hospitalisé et à recevoir des soins de façon anonyme,
mais ne bénéficie pas de couverture sociale. Il déclare prendre de façon occasionnelle du haschich, du LDS et de la cocaïne
et de manière quasi quotidienne, en alternance de l'Imménoctol® avec de l'alcool et de l'héroïne.
Ses dernières prises remontent à une vingtaine d'heures et il demande qu'on lui donne de manière dégressive du Palfium®
pour parvenir à s'en passer.

Quel(s) diagnostic(s) évoquez-vous devant le tableau présente ?


A - Un prédélirium tremens
B - Un sevrage aux opiacés
C - Une conversion hystérique
D - Un syndrome de surdosage
E - Une endocardite infectieuse
Bonne(s) réponse(s) : B E

A - Il manque agitation et sueur.


B - Le tableau l'évoque: douleurs musculaires, crampes, diarrhée.
D - Non, car somnolence, donc...
E - Bien sûr, d'autant plus qu'existe un souffle systolique et un foie augmente de volume.

Quel(s) est(sont) le(s) produit(s) parmi ceux utilises par, ce garçon, qui est(sont) susceptible(s) d'occasionner
une dépendance physique ?
A - Le haschich
B - Le L.S.D.
C - La cocaïne
D - L'Immenoctol®
E - L'héroïne
Bonne(s) réponse(s) : D E

ABC - La dépendance est psychique.


DE - Donnent un syndrome de sevrage à l'arrêt brutal.

Parmi ces produits et leur voie d'administration, quel(s) est (sont) celui (ceux) que vous utiliseriez ?
A - Palfium® I. M.
B - Tranxene®
C - Tercian®
D - Catapressan®
E - Visceralgine® per os
Bonne(s) réponse(s) : B C D E

BC - Permettent d'obtenir une bonne sédation et de diminuer les symptômes du manque.


D - Dans le sevrage, en dehors de l'urgence.
E - Diminue les crampes du sevrage.

Parmi les- propositions suivantes concernant le traitement de l'intoxication aiguë à la cocaïne, laquelle ou
lesquelles est ou sont correctes ?
A - Neuroleptiques sédatifs
B - Antidépresseurs tricycliques
C - Antidépresseurs I.M.A 0.
D - Normothymiques
E - Sels de lithium
Bonne(s) réponse(s) : A

Evident.

La loi du 31 décembre 1970 prévoit :


A - Le gratuité des soins
B - L'obligation de se présenter devant un magistrat après la cure
C - L'anonymat
D - Le placement volontaire
E - Déclaration obligatoire nominale à la DDASS par le médecin traitant
Bonne(s) réponse(s) : A C

C'est la loi.

1189
Exclusivement sur DOC - DZ : www.doc-dz.com NADJI 85
RESIDANAT EN POCHE TOME II
Cas Clinique en QCM
Une femme de 40 ans présente depuis plusieurs jours une insomnie presque complète avec une hyperactivité incessante un
peu désordonnée. Elle se sent toute puissante et ce sentiment l'amène à multiplier les opérations d'achats et de ventes dans
son commerce d'immobilier. Elle fait des ventes à perte et des achats inconsidérés sans supporter la moindre critique.
Amenée devant un médecin par son mari inquiet de son état, elle se montre bavarde, sautant du coq à l'âne, ironique et
agressive, Elle n'est pas désorientée, Elle avait fait, il y a 5 ans, une grave dépression avec une tentative de suicide.
Depuis, elle n'avait eu aucun trouble.

Parmi la liste de manifestations psychopathologiques suivantes, quelle est celle, que l'on retrouve dans cette
observation ?
A - Confusion mentale
B - Idées délirantes de persécution
C - Détérioration intellectuelle
D - Etat d'excitation psychomotrice
E - Dissociation psychique
Bonne(s) réponse(s) : D

Parmi la liste des maladies ou affections syndromiques suivantes, quelle est celle qui correspond le mieux au
cas décrit ?
A - Intoxication aux amphétamines
B - Bouffée délirante aiguë
C - Accès maniaque
D - Délire paranoïde aigu
E - Démence
Bonne(s) réponse(s) : C

On retrouve ici :
- exaltation de l'humeur
- excitation psychomotrice
- sentiment de toute puissance
- logorrhée - coq à l'âne.
Les antécédents de syndromes dépressifs graves font évoquer une maladie maniaco-dépressive bipolaire.

Parmi la liste des classes médicamenteuses suivantes, quelle est ou quelles sont celle(s) que l'on pourrait
donner dans ce cas, soit isolément, soit associées ?
A - Anticomitiaux
B - Neuroleptiques sédatifs
C - Bêtabloquants
D - Neuroleptiques incisifs
E - Antidépresseurs sédatifs
Bonne(s) réponse(s) : B D

Le plus souvent en association (Haldol®, Nozinan®) à forte dose et en parentéral (faute de mieux).

Parmi les mesures de protection susceptibles d'être adoptées, laquelle doit-on choisir ?
A - Rédaction d'un certificat médical simple
B - Rédaction d'un certificat de mise sous sauvegarde de justice par l'assistante sociale de son quartier
C - Rédaction d'un certificat de mise sous sauvegarde de justice avec désignation d'un mandataire spécial
D - Absence de toute mesure de protection
E - Rédaction d'un certificat médical prescrivant une Assistance en économie familiale et domestique
Bonne(s) réponse(s) : C

C'est une faute de ne pas le faire. La sauvegarde prend acte immédiatement et permet de préserver les biens du patient.
Valable 2 mois, renouvelable 6 mois.

1190
Exclusivement sur DOC - DZ : www.doc-dz.com NADJI 85
RESIDANAT EN POCHE TOME II
Cas Clinique en QCM
Madame Z., âgée de 46 ans, est hospitalisée en urgence. En effet, depuis quelques jours elle refuse de sortir de son lit,
restant prostrée tout au long de la journée, immobile, le faciès figé, les yeux fixes. Les troubles ont débuté progressivement
trois semaines auparavant par des crampes épigastriques et abdominales.
Madame Z., s'est alors cru atteinte d'un cancer digestif (sa mère est décédée d'une telle affection dix ans auparavant),. Elle dit
à ce propos, ne s'exprimant que si elle est fortement sollicitée, que "tout est foutu " "que tout est pourri à l'intérieur ".
"Personne ne peut plus rien faire pour elle..., "tout ceci est une juste punition, car je n'ai pas su soigner ma mère". Le ton de la
voix est faible et monotone.
Puis, progressivement Madame Z... néglige ses occupations, ne sort plus, ni pour faire ses courses ni pour voir ses amies.
Elle déclare "ne plus avoir de mémoire..., ne plus pouvoir regarder la télévision" car il lui faudrait faire de trop grands efforts
pour suivre le programme.
Le diagnostic de dépression mélancolique est posé.

Quel(s) éléments(s) clinique(s) permet(tent) d'affirmer qu'il s'agit d'un épisode dépressif ?
A - Les idées de mort
B - L'insomnie
C - Plaintes hypocondriaques
D - L'inhibition psychomotrice
E - La notion d'age
Bonne(s) réponse(s) : A D

A - "Tout est foutu"; les idées de mort font partie de la plupart des états dépressifs.
B - Existe souvent : insomnie matinale mais n'est pas décrite dans l'énoncé.
C - Ne fait pas partie intégrante du syndrome dépressif.
D - Bien sûr : un des points cardinaux du syndrome.

Quel(s) élément(s) clinique(s) permet(tent) d'affirmer qu'il s'agit d'un épisode mélancolique ?
A - L'importance de l'inhibition
B - L'absence d'élément déclenchant
C - L'auto-accusation
D - Les troubles digestifs
E - Absence d'accès dysthymique antérieur
Bonne(s) réponse(s) : A B C

A - Evident "l'importance".
B - Est un des éléments de la mélancolie.
C - Surtout délirante.
D - Existe dans toute dépression.

Si la malade refuse l'hospitalisation quelle(s) mesure(s) est (sont) indispensable(s) ?


A - Hospitalisation en médecine interne
B - Mesure de placement d'office
C - Maison de repos
D - Mesure de placement volontaire
E - Aucune réponse n'est bonne
Bonne(s) réponse(s) : D

Evident, vu le grand risque suicidaire.

Quel(s) élément(s) est(sont) en faveur d'une psychose maniaco-dépressive ?


A - Hérédité mélancolique
B - Biotype leptosome
C - Origine réactionnelle à un traumatisme affectif grave
D - Episode antérieur mélancolique ou maniaque
E - Anomalie du taux de base de prolactine
Bonne(s) réponse(s) : A D

Q.C.M. difficile car l'énoncé ne permet aucune conclusion.

1191
Exclusivement sur DOC - DZ : www.doc-dz.com NADJI 85
RESIDANAT EN POCHE TOME II
Cas Clinique en QCM

Quel(s) traitement(s) pouvez-vous prescrire immédiatement, dès l'hospitalisation ?


A - Cure de Sakel
B - Antidépresseur en perfusion
C - Neuroleptique d'action prolongée
D - Sels de lithium
E - Sismothérapie
Bonne(s) réponse(s) : B E

Le lithium n'est pas curatif de la mélancolie. Les sismothérapies agissent plus vite que les anti-dépresseurs tricycliques et
peuvent être employés en association.
Les antidépresseurs tricycliques en perfusion sont employés à dose progressive sur 34 jours en tenant compte des contre-
indications.

Monsieur C., 45 ans, sans antécédents psychiatriques, est hospitalisé pour des troubles du comportement rapidement
installés depuis 36 heures. Il a du mal à tenir en place, parle beaucoup, dort peu. Il semble par moments très perplexe,
comprenant mal les propos du médecin : il se trompe dans les dates, se croit à son travail, oublie au fur et à mesure ce qu'on
lui dit paraît légèrement somnoler par instants. à d'autres, semble très inquiet, "où suis-je ?. Qui êtes vous ?", demande-t il aux
personnes présentes. Brusquement, il jette un objet dans un coin de la pièce comme pour se défendre d'une vision terrifiante.
L'examen clinique est normal,mis à part une fièvre à 38,5 degrés et des signes de déshydratation (soif, langue sèche,
ébauche de pli cutané).
L'examen neurologique est normal

Parmi la liste de symptômes suivants, quel est celui ou quels sont ceux qu'on retrouve dans cette observation ?
A - Discordance.
B - Obnubilation
C - Perplexité
D - Automatisme mental
E - Désorientation temporo-spatiale
Bonne(s) réponse(s) : C E

Evident.

Quel diagnostic peut-on évoquer en priorité ?


A - Confusion mentale
B - Délire paranoïaque
C - Schizophrénie paranoïde
D - Accès maniaque
E - Manie atypique
Bonne(s) réponse(s) : A

Le syndrome confusionnel ne fait pas parité de l'accès maniaque.


E - Etait possible, mais il existe une perplexité.

Parmi ces étiologies, la ou lesquelles peut ou peuvent être évoquée(s) à propos du syndrome aigu présenté par
ce patient ?
A - Toxicomanie aux barbituriques
B - Ivresse aiguë
C - Névrose hystérique
D - Hématome sous-dural
E - Hydrocéphalie à pression normale
Bonne(s) réponse(s) : A

A - En particulier le sevrage des barbituriques.


B - Eliminer sur la durée : 36 heures.
D.E. - Donnent des signes neurologiques absents ici.

Quel est la ou quelles sont le mesures thérapeutiques qui s'imposent dans l'immédiat ?
A - Hospitalisation
B - Contention
C - Réhydratation
D - Prescription de benzodiazépines
E - Espéral
Bonne(s) réponse(s) : A D

D - Est faux. On prescrira soit une réintroduction des barbituriques puis un sevrage très progressif, soit des neuroleptiques,
voire de l'Equanil®, en cas de suspicion de délirium tremens.

1192
Exclusivement sur DOC - DZ : www.doc-dz.com NADJI 85
RESIDANAT EN POCHE TOME II
Cas Clinique en QCM

Quel est ou quels sont les signes paracliniques évocateurs d'une étiologie alcoolique ?
A - Séquelles d'une fracture de côte
B - Macrocytose
C - Augmentation des triglycérides
D - Polyglobulie
E - Hyperazotémie
Bonne(s) réponse(s) : D

Ainsi qu'une augmentation des gamma GT et des perturbations des paramètres hépatiques.

Une femme de 52 ans est amenée en consultation par son mari qui la trouve changée depuis quelques semaines. La malade
est habillée de couleurs sombres, ne répond aux questions que de manière laconique, participant à l'entretien à contre-coeur.
Elle affirme qu'il est inutile de la soigner, qu'elle ne mérite pas qu'on s'occupe d'elle et que, de toute manière, étant ruinée, elle
ne pourra pas payer les médicaments. Elle se reproche de n'avoir plus aucun sentiment, de n'être attachée à personne et de
n'être capable de rien. Son mari confirme que depuis ces derniers temps elle reste inactive, négligeant les tâches les plus
élémentaires, de plus, elle dort très peu.

Quel diagnostic proposez-vous ?


A - Dépression d'involution
B - Délire de persécution
C - Dépression mélancolique
D - Début de démence
E - Paraphrénie
Bonne(s) réponse(s) : C

On retrouve en effet :
- tristesse apparente, insomnie
- inhibition psychomotrice
- thème d'incurabilité, de culpabilité délirante chez une femme de 52 ans.
Le tableau est typique.

L'évolution à court terme se fera vers l'apparition :


A - De réactions agressives
B - D'une démence
C - D'un état délirant sévère
D - D'un risque suicidaire
E - De fugues avec divagation
Bonne(s) réponse(s) : D

Bien sûr, c'est une urgence psychiatrique qui impose l'hospitalisation.

La conduite thérapeutique doit comporter :


A - Hospitalisation
B - Traitement à domicile avec psychothérapie
C - Antidépresseur majeur
D - Benzodiazépine
E - Neuroleptique incisif
Bonne(s) réponse(s) : A C

Antidépresseur tricyclique à doses majeures en dehors des contre-indication par voie parentérale.
E - Est un piège : neuroleptique sédatif.
D - Ne suffit pas devant un tel tableau.

Ce traitement peut se compliquer par :


A - Episode délirant
B - Confusion mentale
C - Passage à la manie
D - Détérioration mentale
E - Risque de suicide au début
Bonne(s) réponse(s) : A B C E

A - Toujours possible dans cette pathologie. Il faut de plus savoir que les anti-dépresseurs tricycliques (surtout les
désinhibiteurs) peuvent provoquer des activations délirantes.
B - Les antidépresseurs tricycliques sont confusogènes.
C - Par le syndrome d'inversion de l'humeur (à surveiller).
D - Non.
E - Par levée de l'inhibition anxieuse, ce qui oblige à la prescription de neuroleptiques sédatif en association avec les
antidépresseurs tricycliques.

1193
Exclusivement sur DOC - DZ : www.doc-dz.com NADJI 85
RESIDANAT EN POCHE TOME II
Cas Clinique en QCM

En cas d'inefficacité thérapeutique après trois semaines de recul vous proposerez :


A - Association d'un I.M.A.O.
B - Changement d'antidépresseur
C - Sismothérapie
D - Thérapie comportementale
E - Psychothérapie analytique
Bonne(s) réponse(s) : B C

A - Les IMAO ne peuvent pas être associés à un antidépresseur tricycliques (cf : règle de prescription).
C - De plus en plus envisagé même en association avec les anti-dépresseurs tricycliques.
C - méthode à l'avantage de la rapidité d'efficacité.

Un employé de banque de 30 ans consulte sur incitation de son épouse pour les troubles suivants :
il ne peut s'endormir que s'il a disposé ses vêtements d'une certaine manière sur une chaise. Il doit, dans son travail, contrôler
5 fois ses opérations et ses livres, de telle sorte qu'il passe deux fois plus de temps que nécessaire à son bureau. Avant de
sortir de chez lui, il remonte 2 fois s'assurer que la porte est fermée à clef. Dans son enfance, il a présenté une angoisse très
grande lors de la fréquentation scolaire.

La symptomatologie actuelle comporte-t-elle :


A - Claustrophobie
B - Ritualisation
C - Obsessions idéatoires
D - Crises d'angoisse
E - Compulsions
Bonne(s) réponse(s) : B E

A - Il n'y en a pas ici.


B - C'est une contrainte intellectuelle : pensée magique qui soulage l'angoisse.
C - Non décrit dans l'observation.
D - Dans ces antécédents mais pas actuellement.
E - Acte qui soulage une contrainte intérieure "besoin de remonter voir si la porte est fermée".

La personnalité est de type :


A - Hystérique
B - Paranoïaque
C - Obsessionnelle
D - Cyclothymique
E - Psychopathique
Bonne(s) réponse(s) : C

Q.C.M. difficile dans le sens où la personnalité peut être dissociée de la névrose.

Le diagnostic est celui de :


A - Névrose obsessionnelle
B - Névrose d'angoisse
C - Névrose phobique
D - Dépression anxieuse
E - Aucune de ces propositions
Bonne(s) réponse(s) : A

Rituels affirmant le diagnostic.


B - Non liées aux rituels.
C - Pas d'objet phobogène.
D - Pas de signe dépressif.

La thérapeutique chimique comportera :


A - Imipraminiques
B - Lorazepam
C - Carbamazépine
D - Disulfiram
E - Abstention
Bonne(s) réponse(s) : A

Seuls médicaments (antidépresseur tricyclique) ayant fait preuve d'efficacité dans cette pathologie
B - Les anxiolytiques sont décevants.

1194
Exclusivement sur DOC - DZ : www.doc-dz.com NADJI 85
RESIDANAT EN POCHE TOME II
Cas Clinique en QCM

L'évolution se fera vers :


A - Délire chronique
B - Déficit démentiel
C - Amélioration progressive
D - Apparition d'une névrose structurée
E - Etat maniaque
Bonne(s) réponse(s) : C D

C - L'amélioration est quelquefois possible, avec les thérapeutiques


D - Bien sûr, mais n'est-elle pas déjà structurée ?
Q.C.M. difficile car (A) est toujours possible.

Mlle A., âgée de 20 ans, est hospitalisée à la suite d'une fugue pour un état d'agitation apparu 24 heures auparavant. Elle est
très exaltée, logorrhéique, plutôt euphorique, familière avec le médecin ; son discours est émaillé de jeux de mots, de coqs à
l'âne. Il n'existe pas de néologismes, le contact est superficiel mais toujours possible. L'entretien avec la famille ne permet pas
de retrouver d'antécédents personnels médico-chirurgicaux et notamment psychiatriques : par contre deux tantes sont
hospitalisées de temps en temps pour des épisodes dépressifs, et le grand-père paternel s'est pendu il y a 10 ans.

Quel est le diagnostic le plus probable ?


A - Agitation épileptique
B - Manifestation paroxystique d'une psychopathie
C - Accès maniaque
D - Bouffée délirante aiguë
E - Schizophrénie
Bonne(s) réponse(s) : C

On peut hésiter avec (D) mais ici, il y a prévalence des troubles thymiques et la symptomatologie est celle d'un accès
maniaque vus également les antécédents.

Quelle est la modalité évolutive à redouter en premier ?


A - Constitution progressive d'un tableau schizophrénique
B - Apparition d'un état de mal comitial
C - Evolution vers la démence
D - Survenue épisodique de crises maniaques et/ou mélancoliques
E - Organisation d'un délire chronique
Bonne(s) réponse(s) : D

Vus les antécédents familiaux, vu l'accès maniaque inaugural.

Parmi les médicaments psychotropes suivants, indiquez la classe la plus appropriée dans le traitement de la
crise :
A - Antidépresseurs
B - Anticomitiaux
C - Tranquillisants (benzodiazépines)
D - Neuroleptiques
E - Hypnotiques
Bonne(s) réponse(s) : D

Neuroleptiques sédatifsn, soit en monothérapie, soit en bithérapie, si il existe une symptomatologie délirante associée.
Le traitement à forte dose au départ sera administré suivant la clinique pendant 3 mois.

Devant les conduites médico-légales au cours de la fugue quelle(s) mesure(s) doit(doivent) être envisagée(s) ?
A - Sauvegarde de justice
B - Sauvegarde de justice avec désignation d'un mandataire
C - Demande d'ouverture immédiate d'une tutelle
D - Rédaction d'un certificat attestant que la malade est irresponsable
E - Alerter la famille
Bonne(s) réponse(s) : B E

B - Est demandée par le médecin.


C - Procédure longue pour les maladies au long cours (démence).
D- !!!.
E - Un peu de bon sens !

1195
Exclusivement sur DOC - DZ : www.doc-dz.com NADJI 85
RESIDANAT EN POCHE TOME II
Cas Clinique en QCM

Le traitement préventif par les sels de lithium peut être indiqué en cas de :
A - Etat d'anxiété
B - Rechute identique dans les deux ans
C - Grossesse
D - Apparition d'idées délirantes
E - Répétition des fugues
Bonne(s) réponse(s) : B

Deux accès maniaques en 2 ans : indication de litothérapie.


C - C'est une contre-indication.
E - Ne veut rien dire.

Mme X., 25 ans, est hospitalisée pour un tableau apparu 15 jours après l'accouchement de son deuxième enfant.
Elle se présente, très angoissée, pleurant à certains moments, alors qu'à d'autres elle est totalement absorbée par l'écoute de
voix qui lui disent qu'on va tuer son enfant ou qu'on le lui a changé à la maternité. Elle est totalement insomniaque, ne peut
rester en place et ne sait pas la date du jour ni ou elle est. Elle est très inquiète da son corps et a le sentiment que son ventre
à éclate. Elle est le plus souvent totalement indifférente à son enfant ; parfois, cependant, elle se rue sur lui en craignant qu'on
ne lui prenne.
Le mari paraît bien dépassé par la situation mais la soeur de la patiente apparait comme un point de repère plus solide, bien
que pas très disponible car elle travaille.
Mme X..., n'avait présente aucun trouble lors de sa première grossesse et de son premier accouchement deux ans
auparavant.

Parmi les propositions suivantes concernant le diagnostic à évoquer, quelle est ou quelles sont celle ou celles
que vous évoquez ?
A - Un état mélancolique du post-partum
B - Un délirium tremens
C - Un accès maniaque du post-partum
D - Un état confusodélirant du post-partum
E - Un post-partum blue
Bonne(s) réponse(s) : A D

D - Est évident puisqu'il existe un délire, une désorientation temporo-spatiale.


A - Car on retrouve des thèmes mélancoliques délirants - pertes d'objets....
B - Aucun élément ne peut permettre ce diagnostic.
C - Il n'existe pas d'exhaltation de l'humeur.
E - Rien à voir : dépression réactionnelle du 5ème jour du post-partum sans mesure ici.

Parmi les propositions suivantes quelle est ou quelles sont celles qui doivent être adoptées ?
A - Maintien dans le service de suites de couches
B - Transfert dans le service de pathologie obstétricale
C - Transfert dans le service de psychiatrie de l'hôpital général
D - Transfert dans le service libre de l'hôpital psychiatrique
E - Sortie et mise en place d'une hospitalisation à domicile
Bonne(s) réponse(s) : C D

Il faut un transfert en milieu spécialisé si la malade est d'accord, (D) est possible.
P.S. - Attention au risque de suicide et d'infanticide.

Parmi les propositions suivantes concernant l'enfant, quelle est ou quelles sont celles qui doit ou qui doivent
être adoptées ?
A - Il est hospitalisé avec la mère dans la même chambre
B - Les rencontres mère-enfant sont interdites jusqu'à guérison de la mère
C - Il est hospitalisé dans un service de pédiatrie voisin et des rencontres mère-enfant sont organisées
quotidiennement
D - Il est gardé par la soeur qui arrête momentanément son travail et qui amène l'enfant à sa mère tous les
jours
E - Il est plaçé pour 6 mois par le juge des enfants dans une famille d'accueil
Bonne(s) réponse(s) : C D

A - Hors de question : risque d'infanticide.


B - Il faut éviter de rompre les relations mère-enfant.
C - Bonne solution (cf : unité conjointe d'hospitalisation mère-enfant).
D - Pourquoi pas, si l'état de l'enfant ne justifie pas son hospitalisation.
E - Hors de question !

1196
Exclusivement sur DOC - DZ : www.doc-dz.com NADJI 85
RESIDANAT EN POCHE TOME II
Cas Clinique en QCM
Mme C., 48 ans, est hospitalisée pour la première fois dans un état de dénutrition s'accompagnant d'un laisser aller personnel,
d'incurie et de troubles psychiques majeurs. L'entretien et la reconstruction de l'anamnèse mettent en évidence les troubles
suivants. Depuis le décès de son père il y a un an, elle a réduit son alimentation. Elle pense qu'on veut l'empoisonner.
Elle a cessé ses fonctions d'assistante à la faculté il y a 15 ans. Depuis cette date, par périodes elle se réfugie chez elle car
dès qu'elle traverse le hall de l'immeuble qu'elle habite, elle entend des paroles désobligeantes et insupportables. Ces paroles
disent qu'elle a des pensées obscènes et honteuses. Parfois lors du journal télévisé certaines allusions tendraient à lui
imposer de nombreux actes de repentir. Depuis que des déménageurs sont venus lui apporter un frigidaire il y a 3 ans, elle ne
peut se débarrasser d'une odeur de saumure dans la cuisine. Lors de l'entretien, Mme C... est syntone, le contact est
excellent. l'enchaînement des pensées est cohérent.

Quelle(s) prescription(s) médicamenteuse(s) pouvez-vous retenir comme adaptée(s) à ce tableau à l'entrée ?


A - Sismothérapie
B - Neuroleptique à action prolongée
C - Neuroleptique sédatif + neuroleptique incisif
D - Benzodiazépines seules
E - Antidépresseur
Bonne(s) réponse(s) : A C

A - La sismothérapie, avec en général un support chimiothérapique permet une résolution rapide des symptômes.
A retenir que les sismothérapies sont à nouveaux reconsidérées à raison.
B - Non.
C - Bien sûr, évident devant un tel tableau.
D - Pas seules.
E - Pas assez précis, uniquement associé au neuroleptique.

Quelle(s) évolution(s) est(sont) possible(s) ?


A - Guérison
B - Chronicisation des troubles psychotiques
C - Récidive lors d'une grossesse ultérieure
D - Absence de trouble lors d'une grossesse ultérieure
E - Rechute de troubles psychotiques quelques mois après l'épisode actuel
Bonne(s) réponse(s) : A B C D E

A - Le plus souvent : 78 % des cas, mais tout est possible.

Parmi les facteurs de risque d'un tel épisode pathologique, quel est celui ou ceux que vous retenez ?
A - Primiparité
B - Age compris entre 15 et 30 ans
C - Antécédents d'I.V.G.
D - Alcoolisme de la mère
E - Notion d'un traumatisme pendant la grossesse
Bonne(s) réponse(s) : A

Seul A est vrai dans la liste proposée ; d'autres facteurs tels que antécédents psychiatriques familiaux ou personnels se
dégagent de certaines études.
Q.C.M. difficile.

Les troubles présentés depuis un an peuvent correspondre à un diagnostic particulier parmi les suivants. Lequel
?
A - Dépression réactionnelle
B - Dépression de deuil
C - Etat névrotique réactionnel
D - Deuil compliqué
E - Psychose hallucinatoire chronique
Bonne(s) réponse(s) : E

On retrouve en effet :
- l'âge de début des troubles (environ 35 ans ) (48-15)
- femme célibataire intellectuelle
- syndrome d'influence
- hallucination visuelle et olfactive
- thème persécutif.
Pour les autres propositions, on notera certes l'aggravation des symptômes après le deuil, mais leur début est très antérieur.

1197
Exclusivement sur DOC - DZ : www.doc-dz.com NADJI 85
RESIDANAT EN POCHE TOME II
Cas Clinique en QCM

Les troubles antérieurs au décès du père permettent d'évoquer :


A - Rites conjuratoires obsessionnels
B - Délire d'action extérieure
C - Honte mélancolique
D - Hallucinations acoustico-verbales
E - Hallucinations olfactives
Bonne(s) réponse(s) : B D E

B - Syndrome d'influence.
D - Evidente.
E - Evidente.
AC - Sans objet.

La mécanisme des principaux troubles psychiatriques répond 3 un des types suivants. Lequel ?
A - Délire imaginatif
B - Délire mélancolique de culpabilité
C - Délire hallucino-interprétatif
D - Délire interprétatif systématisé
E - Délire paraphrénique confabulatoire
Bonne(s) réponse(s) : C

En effet, le délire n'est pas interprétatif donc (D) éliminé.


AE - S'élimine évidemment.
B - Il n'y a pas de syndrome dépressif intense.

Parmi les médicaments suivants quel est celui qui doit s'avérer le plus utile ?
A - Diazépam (Valium®)
B - Viloxazine (Vivalan®))
C - Chlorpromazine (Largactil®)
D - Imipramine (Tofranil®)
E - Chlorazepate (Tranxene®)
Bonne(s) réponse(s) : C

A défaut d'autre réponse possible (Q.C.M. simple).


En fait, l'idéal était l'association d'un antidépresseur tricyclique et d'un neuroleptique hallucinolytique (type Haldol®).
Le (B) est peu efficace (jamais seul).
E - N'a aucun intérêt ici.

Dans la mesure où la stratégie thérapeutique comporte une chimiothérapie neuroleptique de long cours, quel
est ou quels sont les médicaments adéquats parmi les suivants ?
A - Amitriptyline (Laroxil®)
B - Décanoate d'halopéridol (Haldol Decanoas®)
C - Palmitate de pipotiazine (Piportil L4®)
D - Halopéridol (Haldol®)
E - Thioridazine (Melleril®)
Bonne(s) réponse(s) : B C D

B.C. - Car se sont des neuroleptiques retards qui permettent :


- de diminuer les doses cumulées
- une vie plus facile
- meilleure observance du traitement
- facilitation de la surveillance.
E - Agit surtout sur l'anxiété donc peu d'intérêt ici.
A - N'est pas un neuroleptique mais un antidépresseur tricyclique.

1198
Exclusivement sur DOC - DZ : www.doc-dz.com NADJI 85
RESIDANAT EN POCHE TOME II
Cas Clinique en QCM
Monsieur P. a 44 ans Depuis l'âge de 18 ans il se plaint de pensées impérieuses qui assaillent son esprit dans un contexte
d'anxiété et d'irritation. Dans les rues il ne peut éviter de compter le nombre de maisons qui bordent les trottoirs. Il se pose
d'incessantes questions sur la nature des matériaux, de divers objets courants.
Quand il signe un chèque il s'interroge longuement sur la conformité de sa signature avec le modèle dépose. Il a cessé toute
activité professionnelle à l'âge de 22 ans et vit d'une rente allouée par ses parents.
Au cours de l'entretien, le contact est aisé. Il parle facilement. Le récit de son existence est clair, cohérent et précis, mais
exprimé avec froideur, sans émotion. L'humeur est syntone.

Quel diagnostic psychiatrique évoquez-vous ?


A - Névrose obessionnelle
B - Névrose phobique
C - Schizophrénie simple
D - Dépression névrotique
E - Névrose hystérique
Bonne(s) réponse(s) : A

On retrouve une symptomatologie obsessionnelle :


- doute permanent
- compulsions
- rituels.
B - Pas d'objet phobogène.
C - Evolution non déficitaire, pas d'élément schyzophrénique.
D - Pas de symptômes dépressifs.
E - Idiot !

A l'âge de 18 ans, quel diagnostic différentiel auriez-vous évoque ?


A - Névrose d'angoisse
B - Névrose traumatique
C - Schizophrénie pseudonévrotique
D - Etat limite
E - Psychopathie dysharmonique
Bonne(s) réponse(s) : C

Le diagnostic différentiel à évoquer à 18 ans mais ici :


- pas de symptômes de la lignée schyzophrénique
- l'évolution va à l'encontre.

Les deux mécanismes de défense inconscients mis en jeu dans ce type de troubles sont :
A - Identification projectrice
B - Déni
C - Isolation
D - Refoulement
E - Cuvage
Bonne(s) réponse(s) : C D

Pas de commentaire - réviser les éléments psychologiques des névroses.

Selon les théories psychanalytiques, à quel stade du développement libidinal se situe la fixation à l'origine de
ces troubles ?
A - Période oedipienne
B - Stade schizoparanoïde
C - Stade oral
D - Période depressive
E - Stade sadique anal
Bonne(s) réponse(s) : E

Par définition (à retenir).

Le type de médicaments psychotropes susceptibles d'être efficace est :


A - Neuroleptiques dérivé des butyrophénones
B - Barbiturique
C - Neuroleptiqueretard
D - Antidépresseurs tricyclique
E - Bêta-bloquant
Bonne(s) réponse(s) : D

Seuls psychotropes ayant fait preuve de leur efficacité dans ce type de pathologie. Les bêta- sont employés dans les fonds
anxieux.

1199
Exclusivement sur DOC - DZ : www.doc-dz.com NADJI 85
RESIDANAT EN POCHE TOME II
Cas Clinique en QCM
Madame X. a 33 ans. Elle est enseignante, mariée et mère de deux enfants Récemment, elle a éprouvé une crise d'angoisse
dans un grand magasin.
Depuis elle ne peut plus aller dans les grandes surfaces et évite la foule.
L'idée d'aller seule au lycée où elle enseigne, est très inquiétant. Elle ne peut s'y rendre que si son mari l'accompagne, Lors
de son dernier voyage à Paris, elle s'est promenée en taxi, l'utilisation du métro étant trop inquiétant et de fait impossible.
L'entretien spontané est riche en événements à résonance émotionnelle : attachement ambivalent à sa mère, deuil difficile
lors du décès du père il y a 2 ans idéalisation du frère aîné, brillant avocat. L'humeur est syntone.
Le contact est chaleureux. Il n'y a aucun autre trouble psychopathologique notamment : absence de pensées
compulsionnelles, de ralentissement psychomoteur, d'insomnie, d'alcoolisme secondaire.

Quel diagnostic retenez-vous ?


A - Névrose hypocondriaque
B - Dépression névrotique
C - Névrose obsessionnelle
D - Névrose phobique
E - Névrose hystérique
Bonne(s) réponse(s) : D

Evident.
Situation phobogène : agoraphobie/claustrophobie.
Objet contraphobique : le mari.
Absence d'autre trouble.

Sur quel(s) élément(s) sémiologique(s) peut-on fonder le diagnostic ?


A - Absence d'idées délirantes
B - Conduite d'évitement
C - Absence d'alcoolisme
D - Conduites de réassurance
E - Absence de ralentissement
Bonne(s) réponse(s) : A B D E

Q.C.M. difficile.
En fait B.D. font le diagnostic.
A - Est important à éliminer et explorer.
E - Elimine le syndrome dépressif.

Indiquez parmi les traitements suivants celui ou ceux qui sont appropriés à ce type de troubles :
A - Neuroleptiques
B - Sismothérapie
C - Psychothérapie analytique
D - Thérapie comportementale
E - Antidépresseurs tricycliques
Bonne(s) réponse(s) : C D E

D - Thérapie de déconditionnement (désensibilisation).


C - Efficace dans les névroses phobiques mais plus long.
E- Démontrée.

Au cas où une cure psychanalytique serait entreprise, ce type de psychothérapie est caractérisé par :
A - La libre association est la règle fondamentale
B - Le rythme des séances varie d'une semaine à l'autre
C - Le transfert n'est pas interprété
D - Les effets sont exclusivement d'ordre symptomatique
E - La cure peut durer plusieurs années
Bonne(s) réponse(s) : A E

B - Est bien sûr faux : 2-3 séances par semaine de manière régulière.
C - Le transfert est analysé par principe.
D - Non, efficacité sur la structure.

1200
Exclusivement sur DOC - DZ : www.doc-dz.com NADJI 85
RESIDANAT EN POCHE TOME II
Cas Clinique en QCM
Monsieur C., 38 ans, attribue à une diarrhée post-antibiothérapie survenue il y a 4 ans le fait qu'actuellement, préoccupé par la
crainte de microbes et de la contagion, il se lave les mains plusieurs dizaines de fois par jour. Il se désinfecte après tout
contact. Depuis cette période, il lui est impossible de s'assoir sans avoir désinfecté le siège, de prononcer certains mots
comme "diarrhée" sans effectuer un geste (croiser les doigts des deux mains et les diriger vers le sol). Très angoissé,
Monsieur C..., évoque en pleurant son, incapacité "à se dominer" et le caractère de plus en plus envahissant de "ses manies".
Célibataire, il a une vie sentimentale très pauvre.

Le lavage répété des mains chez ce patient peut être considéré comme :
A - Une conduite d'évitement
B - Une échopraxie
C - Un rituel compulsif
D - Un tic
E - Un symptôme phobique
Bonne(s) réponse(s) : C

C'est le type même du rituel compulsif qui permet, par définition, de soulager une tension intérieure.

Les préoccupations de ce patient concernant la contagion et les microbes peuvent être considérées comme :
A - Une phobie d'impulsion
B - Une interprétation délirante
C - Des hallucinations psychiques
D - Une idée obsédante
E - Une idée fixe post-onirique
Bonne(s) réponse(s) : D

B - Fait partie du délire paranoïaque sans objet ici.


C - Sans objet.

Parmi les maladies suivantes, laquelle vous parait le mieux correspondre au cas de ce patient ?
A - Névrose phobique
B - Schizophrénie à début tardif
C - Délire hypocondriaque
D - Névrose obsessionnelle
E - Névrose traumatique
Bonne(s) réponse(s) : D

Evident d'après la clinique. Seule (B) peut se discuter mais :


1 - Rare
2 - Age supérieur à 35 ans
3 - Aucun élément de la lignée schyzophrénique.

Parmi les classes de médicaments suivants, laquelle ou lesquelles vous paraît(paraissent) la ou les plus
susceptible(s) d'atténuer les craintes et les troubles du comportement de ce patient ?
A - Neuroleptiques retard
B - Sels de lithium
C - Tranquillisants du groupe des benzodiazépines
D - Antidépresseurs imipraminiques
E - Barbituriques
Bonne(s) réponse(s) : C D

C - Permet de diminuer les phénomènes d'anxiété donc enraye les processus d'aggravation des rituels.
D - Ont montre leur efficacité dans ces pathologies en particulier en diminuant la tonalité dépressive souvent associée ou
sous-jacente.

Parmi les méthodes psychothérapiques suivantes, la ou lesquelles vous paraît(paraissent la ou les plus
susceptible(s) d'améliorer les troubles de ce patient ?
A - Psychothérapie de déconditionnement
B - Suggestion sous hypnose
C - Psychothérapie inspiration psychanalytique
D - Psychothérapie familiale
E - Psychodrame
Bonne(s) réponse(s) : A C

A - Est d'inspiration comportementaliste et n'agit que sur le symptôme et non sur la structure.
C - Efficace mais long et doit émaner d'une demande du patient.
B - Dans l'hystérie.
DE - Concerne les psychoses le plus souvent.

1201
Exclusivement sur DOC - DZ : www.doc-dz.com NADJI 85
RESIDANAT EN POCHE TOME II
Cas Clinique en QCM
Une femme de 52 ans est hospitalisée pour la 12ème fois en 18 ans pour un tableau clinique fait d'excitation psychique,
d'agitation, de tachypsychie, de fuite des idées. Elle n'est pas confuse, mais fait état de projets grandioses, de désirs de
réforme sociale. Insomniaque, elle a conservé tout son appétit Elle multiplie les activités sur un mode brouillon et se livre à de
nombreux achats ne correspondant pas à ses besoins. Elle se montre agressive envers l'entourage et très caustique.
Sur le plan clinique, la malade est obèse. discrètement hypertendue.
Elle a par ailleurs une hyperthyroïdie connue et traitée par les antithyroïdiens de synthèse.

Ce tableau clinique correspond :


A - A une B.D.P. (bouffée délirante polymorphe)
B - A un accès maniaque
C - A l'évolution dysthymique tardive d'une schizophrénie
D - A l'entrée dans un délire chronique
E - A l'entrée dans un état démentiel du présénium
Bonne(s) réponse(s) : B

C'est typique :
- antécédents
- symptomatologie.
Il s'intègre certainement dans une psychose maniaco dépressive. La réponse (C) ne peut être retenue, il n'y a aucun
symptôme de cette lignée.

Cette malade hospitalisée demande sa sortie contre avis médical.


Vous envisagez son placement volontaire sous le régime de la loi de 1838. Pour cela il faut :
A - Un certificat médical par un médecin extérieur à l'hôpital
B - Un arrêté du Préfet
C - Une demande de placement par un membre de la famille
D - Un arrêté du Procureur de la République
E - Une décision du Juge des Tutelles
Bonne(s) réponse(s) : A C

C'est la loi du 30 juin 1838


Il faut :
- une demande écrite par l'entourage
- un certificat médical de placement demande par un docteur en médecine extérieur famille (jusqu'au 2ème degré) CI à
l'hôpital de placement.
- une pièce d'identité du patient.

Parmi les mesures thérapeutiques suivantes, laquelle instaurerez-vous en premier lieu ?


A - Réhydratation
B - Benzodiazépines injectables
C - Tricycliques en perfusion
D - Sismothérapie
E - Neuroleptique sédatif injectable
Bonne(s) réponse(s) : E

Utilisé per os si possible, en général, on a recours à la voie intramusculaire.


Neuroleptique sédatifs, majeur (Nozinan® ou Tercian®) à forte dose, souvent associé.
Ne pas oublier :
- les correcteurs (ponalide intramusculaire)
- la surveillance médicale.

Dans un but thérapeutique il est indispensable de demander :


A - Frottis vaginaux
B - Créatinémie
C - E.CG
D - Tomodensitométrie crânienne
E - Bilan thyroïdien
Bonne(s) réponse(s) : B C E

Il faut avoir l'arrière-pensée de la litothérapie donc :


- examen clinique complet
- ionogramme (Na+IK+) glycémie, réaction immunologique de grossesse
- NFS, créatinine
- T3 T4 TSH - ECG - EEG.

1202
Exclusivement sur DOC - DZ : www.doc-dz.com NADJI 85
RESIDANAT EN POCHE TOME II
Cas Clinique en QCM

Une telle affection peut évoluer vers :


A - Un délire chronique
B - Un état démentiel
C - Récidive sur le même mode
D - Apparition d'un état dépressif de type mélancolique
E - Décès par suicide
Bonne(s) réponse(s) : C D E

C - C'est le cas sans doute ici.


D - Syndrome d'inversion de l'humeur.
E - Par inversion mélancolique.
Il faut admettre que tout accès maniaque s'intègre dans une psychose maniaco dépressive ou bipolaire.

Quel(s) traitement(s) prophylactique(s) peu(ven)t être envisagé(s) entre les crises ?


A - Antidépresseur tricyclique
B - Neuroleptique incisif
C - Benzodiazépine au long cours
D - Sels de lithium
E - Carbamazépine (Tégrétol®)
Bonne(s) réponse(s) : D E

Ce sont les deux thymorégulateurs qui font preuve d'efficacité, chacun dans des indications précises.
ABC - Ne sont en aucun cas prophylactiques.

Quelle mesure de protection immédiate devez-vous envisager chez cette patiente ?


A - Demande de placement d'office
B - Sauvegarde de justice
C - Curatelle
D - Tutelle
E - Placement volontaire
Bonne(s) réponse(s) : B

On demande une mesure de protection donc cela élimine (A) et (E).


CD - Découlent d'un jugement et ne sont pas immédiat. Donc :
B - Est valable 2 mois, renouvelable 6 mois. Demandée en urgence, elle prend acte immédiatement.

Madame X. a 33 ans. Elle est enseignante, mariée et mère de deux enfants. La dernière fois qu'elle est rentrée dans un grand
magasin, elle a éprouvé une crise d'angoisse très pénible l'obligeant à rentrer précipitamment chez elle.
Depuis elle ne peut plus aller dans les grandes surfaces et évite la foule. L' idée d'aller seule au lycée où elle enseigne est très
inquiétante. Elle ne peut s'y rendre que si son mari l'accompagne. Lors de son dernier voyage à Paris, elle s'est promenée en
taxi, l'utilisation du métro étant trop angoissante et de fait impossible.
L'entretien spontané est riche en éléments à résonance émotionnelle : attachement ambivalent à sa mère, deuil difficile lors
du décès du père il y a 2 ans, idéalisation du frère aîné, brillant avocat. L'humeur est syntone. Le contact est chaleureux. Il n'y
a ni pensées compulsives ni ralentissement psychomoteur, ni insomnie ou alcoolisme secondaire.

Quel diagnostic retenez-vous parmi les suivants ?


A - Névrose hypocondriaque
B - Dépression névrotique
C - Névrose obsessionnelle
D - Névrose hystérique
E - Névrose phobique
Bonne(s) réponse(s) : E

On retrouve :
- situation Phobogène (agoraphobie, claustrophobie)
- objet contraphobique : mari
- personnalité névrotique.
Les autres propositions sont sans objet.

Quel est ou quels sont les éléments spécifiques du diagnostic parmi les suivants ?
A - Attachement ambivalent à sa mère
B - Evitement des grands magasins
C - Absence d'alcoolisme
D - Conduite de réassurance
E - Absence de ralentissement
Bonne(s) réponse(s) : B D

Font partie des conduites contraphobiques et sont donc spécifiques.


Le reste, A.C.E., n'a rien de spécifique.
1203
Exclusivement sur DOC - DZ : www.doc-dz.com NADJI 85
RESIDANAT EN POCHE TOME II
Cas Clinique en QCM

Indiquez parmi les traitements suivants celui ou ceux qui sont appropriés à ce type de trouble :
A - Cure chimiothérapique par halopéridol
B - Hypnothérapie
C - Psychothérapie analytique
D - Thérapie comportementale par immersion
E - Thérapie comportementale de désensibilisation
Bonne(s) réponse(s) : B C D E
.
B - Utilisé par certains avec des résultats plus ou moins bons.
C - Bien sûr.
DE - Font partie des thérapies comportementales. Il faut savoir que la thérapie non chimique la plus efficace est (E).

Au cas où le traitement serait axe sur une chimiothérapie psychotrope, indiquez celle ou celles susceptibles
d'être efficaces :
A - Antidépresseur tricyclique
B - Neuroleptique phénothiazinique
C - Barbiturique à doses filées
D - Anxiolytique benzodiazépinique
E - Sels de lithium
Bonne(s) réponse(s) : A D

A - Ont prouvé leur efficacité au long cours d'autant que toute tonalité dépressive aggrave les névroses phobiques.
D - Bien sûr, permettant de diminuer les phénomènes anxieux.

Au cas où une cure psychanalytique serait envisagée, indiquez la ou les propositions qui caractérisent ce type
de psychothérapie :
A - Elle repose sur la règle fondamentale d'association libre d'idées
B - Elle suppose la mise en jeu d'une névrose de transfert
C - Elle utilise le matériel onirique.
D - Elle ne vise qu'à réduire les symptômes
E - Elle ne prend en compte que le matériel conscient
Bonne(s) réponse(s) : A B C

Par définition et principe.


D - Non, elle vise à être un traitement étiologique.
E - Bien sûr que non !

Madame X. 72 ans, est hospitalisée pour tentative de suicide médicamenteuses. A son réveil, elle est extrêmement
angoissée, cherche à tomber de son lit, pour dit-elle, se tuer, car elle désire mourir, puisqu'elle n'est plus bonne à rien et
qu'elle a honte.
Elle dit qu'il est inutile qu'on la soigne car elle est incurable.
Son mari explique que cet épisode est le premier de ce type et qu'il est apparu brutalement.
Elle avait eu peur deux mois auparavant d'avoir un cancer de l'intestin, mais la négativité des examens l'avait rassurée.
Son mari la décrit comme une femme secrète et cherchant à éviter les contacts ; depuis qu'il la connaît, elle se lave très
fréquemment les mains, cette habitude s'aggravant au fil des années.
Parfaite ménagère, elle parvenait moins bien ces derniers temps à s'occuper de la maison.
Elle n'a jamais été suivie par un psychiatre et avait pour seul traitement un hypnotique le soir.

Quel est le diagnostic à retenir à l'admission ?


A - Une bouffée délirante aiguë
B - Une manie délirante
C - Une dépression réactionnelle
D - Une démence sénile
E - Une mélancolie
Bonne(s) réponse(s) : E

C'est évident.
Syndrome dépressif majeur avec idée d'incurabilité survenant chez une personnalité obsessionnelle.
A - Pas d'objet (à noter que les B.D.P. chez les gens âgés existent mais sont rares).
C - Non, syndrome dépressif ici intense et avec idées délirantes.
D - Sans objet: pas de syndrome confusionnel.

1204
Exclusivement sur DOC - DZ : www.doc-dz.com NADJI 85
RESIDANAT EN POCHE TOME II
Cas Clinique en QCM

Comment appelle-t-on, cette peur du cancer en terme séméiologique ?


A - Une nosophobie
B - Une idée obsédante
C - Un symptôme de conversion
D - Une somatisation
E - Un symptôme psychosomatique
Bonne(s) réponse(s) : A

C'est la peur des maladies que l'on rencontre surtout dans les névroses phobiques ou obsessionnelle.

Comment s'appelle pour cette malade la nécessité de se laver très fréquemment les mains ?
A - Une idée délirante
B - Une idée obsedante
C - Un rituel obsessionnel
D - Une phobie
E - Un tic
Bonne(s) réponse(s) : C

A.B. - Non, car ici il y a acte.


D - Est certainement à l'origine de (C).

La personnalité de cette malade est de type :


A - Hystérique
B - Paranoïaque
C - Obsessionnel
D - Phobique
E - Schizoïde
Bonne(s) réponse(s) : C

Par les rites de lavage, souci de propreté (ménage).


D - Est éliminé par la présence de rituels et l'absence de comportement contraphobique.
A noter la fréquence des décompensations mélancoliques chez les obsessionnels.

Quel(s) traitement(s) proposez-vous d'emblée ?


A - Sels de lithium
B - Vasodilatateurs cérébraux
C - Sismothérapie
D - Antidépresseur
E - Psychothérapie comportementale
Bonne(s) réponse(s) : C D

A - N'est pas curateur des mélancolies.


BE - Sans objet.
C - C'est ci une indication :
- forme majeure, anxieuse
- chez une personne âgée.
Elle permet l'amélioration rapide.
D - En dehors des contre-indications, doit être associé à C.

1205
Exclusivement sur DOC - DZ : www.doc-dz.com NADJI 85
RESIDANAT EN POCHE TOME II
Cas Clinique en QCM
Un médecin est appelé en consultation auprès d'un homme de 40 ans, consultation demandée par l'épouse car celui-ci se dit
en parfaite santé et refuse tout soin. Depuis quelques jours, il est agité, parle sans cesse, ses idées s'enchaînent sans lien
logique et il accumule les jeux de mots. Il rit beaucoup, annonce qu'il va se lancer dans de brillantes affaires. Il s'est rendu le
matin même à la salle des ventes et dit avoir acheté 3 villas. Il reçoit le médecin sur un ton amusé et familier et se moque de
son allure qu'il trouve "vieillotte". Sa femme apprend au médecin qu'il ne dort plus depuis 3 jours et qu'il lui fait des avances
obscènes, ce qui contraste avec son caractère timide et réservé habituel. Il n'a pas pris de substance toxique ni d'alcool.

Devant un tel épisode d'agitation, quel est le diagnostic qui vous parait le plus probable ?
A - Agitation paranoïaque
B - Agitation hystérique
C - Agitation maniaque
D - Agitation psychopathique
E - Agitation schizophrénique
Bonne(s) réponse(s) : C

C'est typique, on retrouve :


- agitation, exaltation thymique
- demi des troubles
- logorrhée, tachypsuchie
- jeux de mots, jovialité, familiarité
- dépenses d'argent, accès de toute puissance
- augmentation de la libido.

Quelles sont, parmi les symptômes suivants, celui ou ceux présents dans cette observation ?
A - Fuite des idées
B - Théâtralisme
C - Mythomanie
D - Discordance
E - Exaltation thymique
Bonne(s) réponse(s) : A E

Cf : réponse antérieure.

Un tel tableau clinique peut faire évoquer aussi une affection organique.
Quelles sont, parmi les affections suivantes, celle ou celles qui pourraient entraîner un tel tableau ?
A - Tumeur frontale
B - Maladie d'Alzheimer
C - Insuffisance surrénale
D - Hydrocéphalie à pression normale
E - Maladie de Kreutzfeld-Jakob
Bonne(s) réponse(s) : A

Q.C.M. difficile car en fait :


A - Présente une moria (niaiserie avec apathie).
Il n'y a pas de syntonie et il existe un syndrome déficitaire et neurologique associé.
C - Ne donne pas de tableau maniaque mais plutôt dépressif.
BD - Donnent des agitations mais avec des signes démentiels.
C'est donc par élimination qu'il faut procéder ici.

Quelles sont parmi les chimiothérapies suivantes celle ou celles qui pourraient être utilisées devant un tel état ?
A - Un antidépresseur sédatif par exemple l'Amitriptyline®
B - Un benzodiazépine par exemple le Valium®
C - Un barbiturique par exemple le Gardénal®
D - Une butyrophénone par exemple l'Halopéridol®
E - Une phénothiazine sédative par exemple la Clorpromazine®
Bonne(s) réponse(s) : D E

En association possible, c'est surtout (E) qui est efficace.

1206
Exclusivement sur DOC - DZ : www.doc-dz.com NADJI 85
RESIDANAT EN POCHE TOME II
Cas Clinique en QCM

En cas de refus du malade de se faire soigner, le médecin peut envisager une mesure de placement volontaire.
Parmi les propositions suivantes, quelles sont celle ou celles conformes à la loi ?
A - Le certificat du médecin demandant le placement doit être accompagné d'une demande de placement
faite par une personne de l'entourage (exemple épouse)
B - Le certificat peut être fait par le médecin même s'il n'est pas spécialiste
C - Ce certificat doit nécessairement comporter le diagnostic des troubles présentés par Monsieur X
D - La réalisation du placement incombe aux services de police ou de gendarmerie
E - Le placement doit se faire dans un établissement régit par la loi du 30 juin 1838
Bonne(s) réponse(s) : A B E

C - Pas de diagnostic.
D - Pas pour le placement volontaire mais pour le placement d'office.
A - Le médecin ne doit pas être apparenté (jusqu'au 2ème degré) au demandeur du placement volontaire et au directeur de
l'hôpital.
Un certificat peut être fait par tout docteur en médecine, donc (B).
E - Par définition.

Madame X., Françoise, 34 ans, mariée, mère de deux enfants, avait présenté dans les suites de son premier accouchement
une épisode confuso-onirique aigu qui avait nécessité l'hospitalisation pendant une quinzaine de jours et un traitement
neuroleptique.
Sa mère et sa grand-mère maternelle ont présente chacune deux épisodes dépressifs graves.
Elle a présenté à 4 reprises, à 28 ans, 29 ans, 31 ans, et 33 ans, des épisodes d'agitation avec euphorie, hyperactivité,
dépenses excessives, perte du contrôle émotionnel.
Elle vient consulter à la demande de sa famille pour une certaine excitation, des activités désordonnées et une insomnie.
L'examen physique ne met en évidence aucune anomalie. L'humeur est euphorique avec une subexcitation psychique mais
sans trouble majeur du comportement, le débit verbal est accéléré.

Dans quel cadre clinique se situe un tel état ?


A - Hystérie
B - Schizophrénie
C - Psychosemaniaco-dépressive
D - Bouffée délirante
E - Déséquilibre psychique
Bonne(s) réponse(s) : C

L'épisode actuel est un accès maniaque typique.


D'autre part, il existe :
- des antécédents personnels d'accès maniaque
- des antécédents familiaux de syndromes dépressifs graves
On peut donc affirmer la P.M.D.

Parmi les psychotropes suivants quelle(s) catégorie(s) de médicaments vous paraissent les plus indiquées pour
le traitement ou la prévention ?
A - Benzodiazépines
B - Neuroleptiques
C - Antidépresseurs
D - Antithyroïdiens de synthèse
E - Sels de lithium
Bonne(s) réponse(s) : B E

B - Traitement de l'accès à doses majeures, souvent deux neuroleptiques associés.


E - C'est un traitement curatif et prophylactique de l'accès maniaque.

Dans la perspective de l'institution de ce traitement, quels sont parmi les 5 examens suivants ceux qui sont
indispensables à pratiquer ?
A - E.E G.
B - Bilan hépatique
C - Bilan thyroïdien
D - Bilan rénal
E - Frottis vaginaux
Bonne(s) réponse(s) : C D

A - Est discutable, il est souvent pratiqué mais il n'est pas exigé chez les gens jeunes, donc...
CD - Bien sûr.
Manquent les :
- la réaction immunologique de grossesse
- le bilan cardiaque
- NFS - ionogramme sanguin.

1207
Exclusivement sur DOC - DZ : www.doc-dz.com NADJI 85
RESIDANAT EN POCHE TOME II
Cas Clinique en QCM

Parmi les cinq chiffres suivants, quel est le chiffre de lithiémie efficace à obtenir pour un traitement
prophylactique ?
A - 0,2 mEq/l
B - 0,4 mEq/l
C - 0,8 mEq/l
D - 1,3 mEq/l
E - 1,7 mEq/l
Bonne(s) réponse(s) : C

En fait, 0,6 à 1 mEg/l.

A partir de quel chiffre de lithiémie sanguine y a t-il un risque de toxicité ?


A - 0,4 mEq/l
B - O,8 mEq/l
C - 0,9 mEq/l
D - 1,3 mEq/l
E - 1,6 mEq/l
Bonne(s) réponse(s) : D

La dose toxique est donc très proche de la fourchette prophylactique.

Compte tenu des risques liés à un tel traitement, une précaution est souhaitable. Laquelle ?
A - Tonicardiaque
B - Diurétique
C - Mise en oeuvre d'une contraception
D - Vitaminothérapie
E - Extraits thyroïdiens
Bonne(s) réponse(s) : C

La lithothérapie est contre-indiquée dans la grossesse, surtout dans les 3 premiers mois donc :
il faut interrompre le lithium avec une couverture contraceptive de 3 mois, s'il existe un désir de grossesse.

Un jeune homme de 17 ans est amené à votre consultation par sa mère. En effet, depuis quelques jours, il refuse de sortir de
sa chambre, se refuse à la moindre activité, ne va plus en classe, a abandonné toutes ses activités de loisir. Il ne voit plus
personne, hormis sa famille. Il tient des propos étranges, parfois incompréhensibles, prétendant que des extra-terrestres ont
décidé de l'enlever en raison de ses extraordinaires pouvoirs télépathiques et de son intelligence supérieure. Il pense avoir pu,
jusqu'à présent, leur échapper, mais ceux-ci, dépités croit-il, lui envoient dans le corps des courants électriques et veulent lui
arracher les nerfs. Le patient est angoissé et parle de s'enfuir dans le désert, seul lieu où il peut être vraiment en sécurité.
Depuis un an et demi, il ne fait plus rien en classe, redouble sa seconde.

Quel est le diagnostic le plus probable ?


A - Bouffée délirante aiguë
B - Mélancolie
C - Dépression névrotique
D - Schizophrénie
E - Crise de l'adolescence
Bonne(s) réponse(s) :D

La sémiologie nous donne (A) ou (D), le reste est une notion de durée.
Il existe un fléchissement scolaire depuis 1 an et demi, donc c'est au delà de la bouffée délirante polymorphe.

Quel(s) symptôme(s) relevez-vous ?


A - Délire
B - Apragmatisme
C - Impulsion
D - Dysthymie
E - Excitation maniaque
Bonne(s) réponse(s) : A B

A - Est évident, délire paranoïde, mécanisme et thème multiples.


B - Baisse du rendement scolaire, repli sur soi (autisme), refus d'activité.
D - Non, car on ne note ni exaltation, ni dépression vraies.

1208
Exclusivement sur DOC - DZ : www.doc-dz.com NADJI 85
RESIDANAT EN POCHE TOME II
Cas Clinique en QCM

Quel traitement chimiothérapique instituerez-vous ?


A - Sels de lithium
B - Antidépresseurs
C - Neuroleptiques
D - Vitaminothérapie
E - Hypnotiques
Bonne(s) réponse(s) : C

Association anti-délirants et sédatifs (car anxiolytiques).

Quelle est la modalité de prise en charge la plus indiquée ?


A - Cure analytique
B - Hospitalisation
C - Consultations ambulatoires
D - Hospitalisation à domicile
E - Maison de repos
Bonne(s) réponse(s) : B

Pour mettre en place le traitement.

Un homme de 42 ans vient consulter car depuis son mariage, qui remonte à quelques mois, il a tendance à se laver les mains
de façon répétitive. Il lui arrive de plus en plus souvent de toucher certains objets selon des rituels complexes extrêmement
contraignants notamment à l'heure du coucher.
Célibataire il était d'un caractère sérieux, économe, obstiné dans son travail et dénué de toute fantaisie. Tourmenté par des
scrupules religieux, il fut tenté par le sacerdoce mais devint finalement un fonctionnaire très consciencieux.
Au moment de l'examen, il est anxieux et incapable de prendre une décision.

Cette symptomatologie évoque :


A - Une schizophrénie de type catatonique
B - Une névrose obsessionnelle
C - Une névrose phobique
D - Une personnalité psychopathique
E - Une mélancolie délirante
Bonne(s) réponse(s) : B

En effet, sont décrits :


- des conduites compulsives
- des rituels
- une personnalité obsessionnelle.
Qui associent :
- sérieux
- scrupulosité
- économie (avarice)
- doute permanent.

Les traits de caractère prémorbides de ce malade, renvoient, selon la théorie psychanalytique, à un stade de
développement libidinal. Lequel ?
A - Oral
B - Anal
C - Phallique
D - Génital
E - Oedipien
Bonne(s) réponse(s) : B

Stade dit "sadique anal".

Ce type de personnalité est caractérisé par :


A - La méticulosité
B - L'orgueil
C - L'avarice
D - Le théâtralisme
E - L'impulsivité
Bonne(s) réponse(s) : A C

B - Personnalité paranoïaque.
D - Personnalité hystérique.
E - Personnalité psychopathique.

1209
Exclusivement sur DOC - DZ : www.doc-dz.com NADJI 85
RESIDANAT EN POCHE TOME II
Cas Clinique en QCM

Si l'on tente d'empêcher ce malade de faire ses actes répétitifs, on observe :


A - Une accentuation considérable de ces conduites
B - Une décompensation aiguë sur un mode délirant
C - Une augmentation de l'anxiété
D - Un virage maniaque
E - Aucune de ces éventualités
Bonne(s) réponse(s) : C

Les rituels sont des mécanismes de défense du Moi qui permettent de diminuer la tension anxieuse.
La décompensation -en général dépression- survient quand les rituels deviennent inopérant.

L'évolution habituelle de ces états est :


A - La guérison
B - L'apparition d'une détérioration intellectuelle
C - Une évolution cyclique
D - D'autant plus mauvaise que le début est plus précoce
E - Aucune des propositions n'est exacte
Bonne(s) réponse(s) : D

Nous rappellerons que la névrose obsessionnelle est la névrose qui a le plus mauvais pronostic.
Les symptômes deviennent permanents et les rituels de plus en plus complexes et inopérants.
Il est à noter une forme torpide chez le sujet jeune, qui est une forme maligne.

Un jeune homme de 18 ans 1/2, lycéen, quitte sa famille brusquement. Il est retrouvé, après 3 jours d'errance, dans un port, et
déclare à la Police qu'il est venu "s'embarquer pour la dernière croisade", puis s'enferme dans le mutisme. Bon élève
jusqu'alors, il négligeait depuis 8 mois son travail, s'enfermait dans sa chambre, déambulait la nuit, dialoguait avec des
Interlocuteurs imaginaires. Tous ses camarades le trouvaient bizarre, mais ne pouvaient exactement expliquer pourquoi.

La symptomatologie actuelle comporte :


A - Idées délirantes
B - Excitation intellectuelle
C - Discordance
D - Humeur dépressive
E - Troubles psychosensoriels
Bonne(s) réponse(s) : A C

A - Déclaration "qu'il est venu...".


C - Bizarrerie.
E - On ne peut pas affirmer l'hallucination ici.

Le diagnostic évoqué est :


A - Bouffée délirante polymorphe
B - Schizophrénie simple
C - Schizophrénie paranoïde
D - Délire paranoïaque interprétatif
E - Névrose de destinée
Bonne(s) réponse(s) : C

En fait, on peut hésiter avec (B), mais ici, c'est le délire qui prévaut et non pas les autres pôles.
A - Est éliminé sur la durée (8 mois).

Le pronostic comporte :
A - Rémission totale
B - Evolution périodique sans symptômes intercurrents
C - Détérioration de l'adaptation sociale
D - Evolution d'un seul tenant
E - Evolution démentielle
Bonne(s) réponse(s) : B C D

A - Ne peut pas être coché. L'évolution déficitaire est constante (C).


B - Est possible en particulier dans certaines formes dysthymiques de bon pronostic.
D - C'est, hélas ! le cas le plus fréquent.

1210
Exclusivement sur DOC - DZ : www.doc-dz.com NADJI 85
RESIDANAT EN POCHE TOME II
Cas Clinique en QCM

Devant un tel tableau, il est particulièrement intéressant de pratiquer :


A - Un E.E.G.
B - Un scanner
C - Un test de niveau
D - Un test projectif
E - Une narcoanalyse
Bonne(s) réponse(s) : C D

C - Permet de suivre l'évolution de la détérioration.


D - Est intéressant sur un plan diagnostic.

Il est indiqué de prescrire :


A - Psychanalyse
B - Neuroleptiques
C - Psychothérapie de psychose
D - I.M.A.O.
E - Psychothérapie familiale
Bonne(s) réponse(s) : B C E

B - C'est la base du traitement.


C - Surtout des entretiens.
E - C'est utile au malade et à sa famille.

Madame D. 37 ans, arrive très inquiète aux urgences de l'hôpital. Elle a présenté il y a deux heures un épisode neurologique
marqué par une protraction incoercible de la langue, accompagnée d'une contracture du cou avec torsion de la tête vers la
droite.
Au cours de l'examen, les mouvements de protrusion de la langue réapparaissent ainsi qu'un plafonnement du regard vers le
haut. L'attitude rassurante du médecin semble favoriser leur atténuation. L'examen neurologique est, par ailleurs, strictement
normal. Madame D., anxieuse depuis quelques temps et dormant mal, prenait depuis 3 jours le traitement suivant :
- Tranxene® : dix mg : deux comprimés par jour
- Mogadon® : un comprimé le soir
- Haldol® : faible : 25 gouttes le soir.

Quel diagnostic peut-on évoquer ?


A - Conversion hystérique
B - Crise dyskinétique aiguë
C - Crise d'angoisse aiguë
D - Simulation
E - Méningite lymphocytaire
Bonne(s) réponse(s) : B

C'est une description classique :


- protraction de la langue
- plafonnement du regard
- contracture et rotation du cou chez une personne sous neuroleptiques (Haldol®) à faible dose.

Parmi les propositions suivantes, laquelle vous parait exacte ?


A - Le traitement pris depuis 3 jours n'est pas en cause
B - Le traitement a pu favoriser mais non provoquer les symptômes
C - L'association de Tranxene® et .Haldol® est responsable des symptômes
D - L'Haldol® est seul responsable des symptômes présentés
E - L'origine psychogène des troubles est vraisemblable
Bonne(s) réponse(s) : D

Bien sûr.
Effet secondaire extrapyramidal de l'Haldol® (neuroleptiques incisifs). C'est très fréquent.

Quelle attitude vous paraît la plus appropriée dans l'immédiat ?


A - Hospitalisation pour bilan
B - Scanner en urgence
C - Prescription d'un correcteur antiparkinsonien par voie intramusculaire (Ponalide® IM un ampoule)
D - Réassurance et mise en place d'une prise en charge psychothérapique
E - Benzodiazépine par voie intramusculaire (Valium® dix mg un ampoule en IM)
Bonne(s) réponse(s) : C

L'effet est quasi immédiat. Le Ponalide® pourra se prescrire ici per os, voire en intraveineuse.

1211
Exclusivement sur DOC - DZ : www.doc-dz.com NADJI 85
RESIDANAT EN POCHE TOME II
Cas Clinique en QCM

L'atténuation des symptômes présentés, par la réassurance :


A - Fait évoquer à coup sur le diagnostic d'anxiété aiguë
B - Fait évoquer à coup sur un accident de conversion
C - Fait douter de la responsabilité du traitement prescrit dans les symptômes présentés
D - Fait évoquer la simulation
E - N'est pas incompatible avec le diagnostic d'accident iatrogène
Bonne(s) réponse(s) : E

La réassurance diminue l'angoisse qui majore les symptômes, mais elle est moins efficace que le Ponalide®.

La prise prolongée de benzodiazépines peut provoquer :


A - Constipation
B - Syndrome de sevrage
C - Diminution de la libido
D - Aménorrhée galactorrhée
E - Troubles de la mémoire
Bonne(s) réponse(s) : B E

C - Est discutable, fréquent mais non classique.


Rappelons que les benzodiazépines sont :
- anxiolytiques
- hypnotiques
- anticonvulsivantes
- amnésiantes
- myorelaxantes.

Une jeune fille de 18 ans est examinée à la suite d'un amaigrissement progressivement installé depuis un an. Elle pèse 38 kg
pour 1m64 (poids antérieur 56 kg). Elle est pâle, affaiblie, peu anxieuse, peu préoccupée de sa maigreur qu'elle ne cherche
pas à dissimuler. Elle prétend s'alimenter normalement ce qu'infirme sa famille, disant que ce qu'elle mange "ne lui profite
pas". Elle dit ne pas vomir mais sa mère prétend qu'elle se lève souvent au milieu des repas pour se rendre aux toilettes.
Cette perte de poids est survenue, dans les suites d'un régime entrepris parce qu'elle se trouvait "boulotte". Elle n'a plus ses
règles depuis 6 mois, ce qui ne paraît guère la préoccuper. C'est une étudiante brillante qui poursuit assidûment une classe
préparatoire à une grande école. Elle ne semble pas déprimée. Le contact est facile mais superficiel.

Parmi les symptômes suivants, le ou lesquels est ou sont présent(s) dans cette observation ?
A - Aménorrhée primaire
B - Anorexie
C - Perturbation de l'image du corps
D - Hyperintellectualisation
E - Aménorrhée secondaire
Bonne(s) réponse(s) : B D E

A - Non, aménorrhée secondaire.


B - Bien sûr : anorexie mentale ici.
C - Non, le corps est perçu comme maigre, l'image n'est pas perturbée, c'est la conscience de la morbidité qui est perturbée.
D - Elle existe dans les anorexies mentales.
E - Toujours présente, avant l'anorexie et persistante après.

Parmi les diagnostics suivants, un seul répond au tableau présenté par cette patiente. Lequel ?
A - Refus délirant de s'alimenter
B - Anorexie dépressive
C - Schizophrénie débutante avec discordance
D - Anorexie mentale essentielle
E - Hystérie grave avec troubles des conduites alimentaires
Bonne(s) réponse(s) : D

Evident.

Parmi les moyens thérapeutiques suivants, lequel vous parait le plus approprié ?
A - Vitaminothérapie
B - Hormonothérapie
C - Diététique
D - Prise en charge psychiatrique
E - Induction d'un cycle menstruel artificiel
Bonne(s) réponse(s) : D

C'est évident.

1212
Exclusivement sur DOC - DZ : www.doc-dz.com NADJI 85
RESIDANAT EN POCHE TOME II
Cas Clinique en QCM

Parmi les attitudes thérapeutiques suivantes, laquelle vous paraît la plus appropriée dans l'immédiat :
A - Placement d office et gavage
B - Psychothérapie analytique en ambulatoire
C - Thérapie familiale en ambulatoire
D - Tranquillisants et psychothérapie de soutien en ambulatoire
E - Hospitalisation avec contrat thérapeutique concernant la reprise du poids
Bonne(s) réponse(s) : E

C'est actuellement le seul moyen thérapeutique qui fait preuve de résultat.


De toute façon, devant la gravité du tableau ici, il faut une hospitalisation.

Un homme de 5 7 ans forain est amené en pleine nuit aux urgences par la police à qui il a demandé protection. Il est
légèrement obnubilé, désorienté dans l'espace, perplexe et, au travers d'un discours bredouillant, se plaint de céphalées et
surtout d'entendre hurler son chien pour le prévenir des voleurs. Il voit dans les ombres du rideau la silhouette de ses voleurs,
se retourne en sursaut car il sent leurs mains frôler son cou.
Son frère qui l'accompagne signale que le patient a toujours été en parfaite santé jusqu'à ce jour, solide moralement quoique
volontiers querelleur, qu'il ne consomme jamais plus d'un demi-litre de vin par jour et qu'on a retrouvé dans la caravane où il
dormait son chien mort au milieu de vomissements.

Le ou les mécanismes délirants présentés par ce patient sont :


A - Imagination
B - Illusion
C - Hallucination
D - Intuition
E - Interprétation
Bonne(s) réponse(s) : C

C'est évident.

La symptomatologie peut être regroupée dans le(s) cadre(s) suivant(s) :


A - Confusion mentale
B - Bouffée délirante
C - Délire interprétatif
D - Syndrome confuso-onirique
E - Psychose hallucinatoire
Bonne(s) réponse(s) : A D E

A - Bien sûr, il existe ici une D.T.S. avec obnubilation, un syndrome hallucinatoire qui fait souvent partie du syndrome confuso
onirique, donc (D).
E - La psychose hallucinatoire ne peut pas être éliminée.
Une psychopharmacose ou une ivresse aiguë pouvant donner de tels tableaux de manière aiguë.

Si le placement volontaire s'impose le dossier d'admission comprendra :


A - Une demande manuscrite du frère
B - Un procès-verbal de police
C - Un certificat médical daté de moins de 15 jours
D - Une pièce établissant l'identité du patient
E - Un arrêté préfectoral
Bonne(s) réponse(s) : A C D

B.E. - Concerne le placement d'office.

Plusieurs hypothèses étiologiques sont soulevées. Laquelle vous paraît la plus probable ?
A - Sevrage d'alcool
B - Intoxication à l'oxyde de carbone
C - Intoxication alimentaire
D - Epilepsie temporale
E - Hémorragie méningée
Bonne(s) réponse(s) : B

Devant le terrain: forrain vivant avec des poêles.


Le tableau : syndrome confuso-onirique avec hallucinations et céphalées.
Le chien : mort (le CO est plus lourd que l'air) ; mort dans ses vomissements (le C.O. est émétisant).

1213
Exclusivement sur DOC - DZ : www.doc-dz.com NADJI 85
RESIDANAT EN POCHE TOME II
Cas Clinique en QCM

Si malgré les dires du frère on suspecte un alcoolisme, ce diagnostic pourra être confirmé par :
A - Alcoolémie
B - Créatinine
C - Augmentation du VGM
D - Taux de prothrombine
E - Gamma G.T.
Bonne(s) réponse(s) : C E

L'alcoolémie élevée ne montre qu'une prise récente d'alcool donc, en aucun cas, un alcoolisme chronique.
Seule la concordante augmentation des gammaGT et du VGM permettra une forte suspicion d'alcoolisme.

Monsieur B C., 65 ans, consulte pour des troubles de mémoire, installés progressivement depuis quelques mois, intéressant
essentiellement les faits récents et à un moindre degré les faits anciens. Son épouse qui l'accompagne signale, depuis sa
retraite, un désintérêt progressif, des modifications du caractère, une indifférence affective et des troubles du sommeil Il aurait
abandonné complètement la lecture, autrefois son passe-temps favori, et passerait l'essentiel de ses journées inactif dans son
fauteuil, regardant parfois la télévision, mais n'émettant jamais aucun commentaire sur les émissions.
L'examen neurologique est normal de même que l'examen cardiovasculaire.
La tension artérielle est à 14/9.
Vous étudiez cliniquement les fonctions supérieures de ce patient et envisagez un examen neuropsychologique.

Parmi ces diagnostics, quel est celui qui se discute le plus souvent ?
A - Hématome sous-dural bilatéral
B - Interaction vieillissement/état dépressif
C - Paralysie générale
D - Carence vitaminique
E - Démence vasculaire
Bonne(s) réponse(s) : B

Tout le problème est là, il faut savoir que de toute façon, tout processus démentiel au début est amélioré par les
antidépresseurs car comportent souvent des éléments dépressifs.

Vous retenez en faveur d'une confusion mentale :


A - Le début progressif sur plusieurs mois
B - La désorientation temporo-spatiale
C - L'obnubilation globale de la conscience
D - Une altération isolée du langage
E - L existence d'un onirisme
Bonne(s) réponse(s) : B C E

A - Bien sûr que non : par définition, la confusion est un phénomène aigu.
B - Bien sûr.
C - Bien sûr.
E - Oui, fréquemment associé : syndrome confuso-onirique.

Compte tenu des troubles de mémoire, vous chercher à éliminer un syndrome amnésique de type Korsakovien
Parmi les données ci-dessous, retenez celle ou celles qui ne s'observe(nt) que dans la démence et pas dans le
syndrome amnésique :
A - Amnésie antérograde
B - Amnésie rétrograde
C - Troubles du langage
D - Apraxie idéomotrice
E - Détérioration intellectuelle
Bonne(s) réponse(s) : D E

A - Existe dans le syndrome de Korsakoff et dans la démence.


B - Ni dans l'une, ni dans l'autre.
C - Non, ni dans l'une, ni dans l'autre.
D - Dans un cas de démence, se souvenir du syndrome aphaso-apraxo, agnosique.
E - Bien sûr, dans la démence

1214
Exclusivement sur DOC - DZ : www.doc-dz.com NADJI 85
RESIDANAT EN POCHE TOME II
Cas Clinique en QCM

Vous demandez un examen tomodensitométrique. Les performances de cet examen permettent parfois
d'affirmer le ou les diagnostic(s) étiologique(s) suivant(s) :
A - Paralysie générale
B - Maladie d Alzheimer
C - Tumeur frontale
D - Hématome sous-dural
E - "Multiinfarct démentia"
Bonne(s) réponse(s) : B C D E

A - Non, il n'existe pas d'image au scanner.


B - Dilatation ventriculaire avec atrophie pariéto-temporale.
C - Bien sûr.
D - Bien sûr.
E - Images lacunaires multiples.

Les propositions suivantes concernent le diagnostic étiologique d'un syndrome démentiel. Cochez celle ou
celles qui est (sont) exacte(s) :
A - La dysarthrie intermittente est un signe clinique de la paralysie générale
B - La maladie de Creutzfeld-Jacob est une dégénérescence abiotrophique d'origine inconnue
C - La "multiinfarct démentia" peut être d'origine embolique cardiaque
D - Le terme de "pseudobulbaire" qualifie un état clinique consécutif à des lésions supranucléaires bilatérales
E - En l'absence de signes neurologiques de localisation, l'étiologie vasculaire d'un syndrome démentiel doit
être évoquée en priorité
Bonne(s) réponse(s) : A C D

A - Est vrai. (à rechercher : le 33 de la rue Ledru-Rolin).


B - Non, l'étiologie en est connue (origine virale).
C - Oui, origine cardiaque gauche : micro-embolies multiples et répétées.
D - Oui, (cf : cours).
E - Non, toujours penser aux "démences curables" :
- hypertension à pression normale
- hématomes sous duraux... .
L'étiologie vasculaire est évoquée en dernier.

Cochez la ou les affirmation(s) exacte(s) concernant la maladie d'Alzheimer :


A - Il existe un déficit des voies cholinergiques projetant du noyau de Meynert vers le cortex
B - Le caractère familial est retrouvé dans 10 % des cas environ
C - La spongiose est une lésion histopathologique typique
D - Le traitement par la physostigmine est inefficace cliniquement
E - Les lésions histopathologiques sont qualitativement différentes dans la forme présénile et la forme sénile
Bonne(s) réponse(s) : A B D

A - Est vrai, hypothèse expérimentale.


B - Est vrai, 8,8% selon certaines séries.
C - Ce n'est pas une spongiose, on retrouve :
- dégénérescence neuro-fibrillaire
- plaques séniles
- dégénérescence granulo-vacuolaire.
D - Oui.
E - Est faux.

Peut ou peuvent plaider en faveur du diagnostic de maladie d Alzheimer :


A - La constatation de troubles gestuels
B - Les antécédents d'accidents ischémiques transitoires à répétition
C - L'existence d'une note aphasique avec paraphasies
D - La normalité de l'examen neurologique objectif (en dehors des troubles neuropsychologiques)
E - La désorientation dans les lieux familiers
Bonne(s) réponse(s) : A C D E

A - Oui, l'apraxie ferait partie de ce syndrome démentiel.


B - Bien sûr que non.
C - Oui, syndrome aphasique.
E - Oui, c'est un gros problème.

1215
Exclusivement sur DOC - DZ : www.doc-dz.com NADJI 85
RESIDANAT EN POCHE TOME II
Cas Clinique en QCM
Mme P. est amenée par le SAMU car elle présente depuis 2 à 3 jours des troubles majeurs du comportement. Elle met à fond
son poste de radio pour ne pas entendre les voix humaines qui lui,prédisent qu'on va l'enlever si elle sort et si elle revient dans
un magasin où elle a l'habitude d'aller. Ces voix l'accusent d'avoir dérobé des boites de chocolats, d'avoir des pensées
obcènes et lui imposent en particulier de se masturber. Ces voix sont tantôt à l'intérieur de l'oreille, tantôt localisées dans la
rue, sans qu'elle puisse préciser ni leur tonalité ni leur provenance, notamment d'une personne connue. Elle se sont
amplifiées récemment mais elles avaient commencé il y a deux ans, depuis qu'un marchand de meubles lui a livré un buffet et
déposé simultanément un flacon dégageant une odeur nauséabonde. Elle a eu beaucoup de mal par la suite pour faire
disparaître cette odeur de son appartement.
A l'arrivée à l'hôpital elle s'est apaisée et a exprime sur un ton neutre ces sensations anormales. Elle n'est ni exaltée ni
abattue et ne comprend pas pourquoi ces phénomènes se produisent. Le contact est bon. Il n'existe aucun autre élément
psychopathologique.

Quel est ou quels sont les éléments sémiologiques ou diagnostiques ?


A - Hallucinations audito-verbales
B - Idées délirantes d'influence
C - Hallucinations olfactives
D - Pensées obsessionnelles
E - Fuite des idées
Bonne(s) réponse(s) : A B C

A - Evident : il faut lire le texte.


B - Evident : imposition d'acte.
D - ???.
E - Non.

Quel diagnostic retenez-vous parmi les suivants ?


A - Délire paranoïaque systématisé
B - Mélancolie délirante
C - Schizophrénie paranoïde
D - Psychose hallucinatoire chronique
E - Etat maniaque
Bonne(s) réponse(s) : D

Il est difficile de répondre sans l'âge de la patiente, vu que les seuls éléments sont regroupés dans le délire.
B.E. - Non, il n'existe pas d'éléments dysthymiques.

Quelle est la structure mentale qui sous-tend souvent ce type de troubles ?


A - Personnalité hystérique
B - Personnalité paranoïaque passionnelle
C - Personnalité schizophrénique
D - Personnalité sensitive
E - Personnalité phobique
Bonne(s) réponse(s) : D

Il ne faut le savoir.
Ce n'est pas toujours le cas, mais souvent.

Quel est le moyen thérapeutique adéquat à court terme ?


A - Thérapie comportementale
B - Cure analytique
C - Chimiothérapie neuroleptique
D - Antidépresseur I.M A O.
E - Sels de lithium
Bonne(s) réponse(s) : C

C'est évident.
En général, neuroleptique d'action prolongée par la suite.

Quel sera ou quels seront les médicaments utilisés pour le traitement au long cours ?
A - Téralithe® (carbonate de lithium)
B - Piportil® L4 (palmitate de pipotiazine)
C - Haldol® (halopéridol)
D - Largactyl® (Chlorpromazine)
E - Tegretol® (carbamazépine)
Bonne(s) réponse(s) : B C D

Sont tous les trois possibles.


Piportil® et Haldol®, en forme retard ont souvent la préférence.

1216
Exclusivement sur DOC - DZ : www.doc-dz.com NADJI 85
RESIDANAT EN POCHE TOME II
Cas Clinique en QCM
Monsieur X. est amené à l'hôpital après avoir tenté d'étrangler sa femme.
Celle-ci raconte que son mari est extrêmement jaloux d'elle et la suspecte constamment de le tromper. Il a acquis depuis
quelques jours la conviction que l'un de leurs voisins était son amant : il en voit la preuve dans le fait que ce voisin "l'évite
depuis quelque temps" et que "la femme de celui-ci à l'air triste. C est en voulant faire "avouer" sa femme qu'il a essayé de
l'étrangler.
Madame X. décrit son mari comme un homme autoritaire, sur de lui, ayant des idées arrêtées qu'il entend bien imposer à sa
famille.
Monsieur X. est à fait persuadé que sa femme le trompe et trouve que les faits expliquent parfaitement son geste. Il est assez
méfiant et parle peu de lui-même. Ses propos par ailleurs sont bien adaptés.

Quel diagnostic retenez-vous ?


A - Episode dépressif masqué
B - Délire schizophrénique
C - Délire de jalousie
D - Hystérie
E - Névrose obsessionnelle
Bonne(s) réponse(s) : C

Description typique :
- délire paranoïaque, mécanisme : interprétatif, thème : jalousie.
C'est un délire passionnel en secteur.
- personnalité paranoïaque rigide, autoritaire....

Au plan sémiologique ce diagnostic est caractérisé par :


A - Le délire passionnel
B - Le geste hétéroagressif
C - Le mécanisme interprétatif du délire
D - La psychorigidité
E - Fabulation
Bonne(s) réponse(s) : A B C D

Cf : Q.CM. précédent.
B - Est discutable, il ne caractérise pas le diagnostic mais en fait souvent partie.

Parmi les éléments suivants, lequel (ou lesquels) caractérise(nt) la personnalité de ce patient ?
A - La suggestibilité
B - L'indécision
C - La tendance dépressive
D - La méfiance
E - La surestimation de Soi
Bonne(s) réponse(s) : D E

Avec la rigidité, la fausseté du jugement, sont les traits de la personnalité paranoïaque.

Quel traitement proposez-vous à ce patient ?


A - Cure psychanalytique
B - Neuroleptiques
C - Relaxation
D - Electrochocs
E - Thérapie par immersion
Bonne(s) réponse(s) : B

Evident, peu efficace.

Parmi le ou les éléments suivants, indiquez celui ou ceux représentant des facteurs de dangerosité :
A - Réaction asthéno-dépressive
B - Absence d'alcoolisme
C - Désignation nominale du rival présumé
D - Impulsion hétéro-agressive
E - Possession d'une arme
Bonne(s) réponse(s) : C D E

E - Est évident.
CD - Sont des critères de dangerosité classiques.

1217
Exclusivement sur DOC - DZ : www.doc-dz.com NADJI 85
RESIDANAT EN POCHE TOME II
Cas Clinique en QCM
Mme B., 32 ans arrive dans un service d'urgences dans un état proche de la stupeur immobile, mutique, visage inexpressif et
figé Depuis 8 jours, elle refuse toute alimentation. Cloîtrée chez elle, elle est, aux dires de son mari, persuadée qu'on va venir
la punir pour les crimes dont elle s'accuse et les catastrophes qu'elle croit occasionner à travers le monde. "Ma nourriture est
empoisonnée par mon mari, il collabore avec ceux qui veulent me punir".
Elle pense que, vu ses fautes, "elle l'a bien mérité". Au petit matin, elle a tenté de se pendre.
On apprendra qu'elle ne dormait plus depuis 15 jours, et que, dans ses antécédents, elle a déjà fait deux tentatives de suicide.

Parmi les symptômes suivants, le ou lesquels est ou sont présent(s) dans cette observation ?
A - Automatisme mental
B - Délire de persécution
C - Auto-accusation délirante
D - Anorexie mentale
E - Ralentissement psychomoteur
Bonne(s) réponse(s) : C E

B - Ce n'est pas à proprement dire un délire de persécution mais bien plutôt "une auto accusation délirante" donc (C).
D - Bien sûr que non, c'est une anorexie..
E - Oui, état stuporeux.

Quel diagnostic évoquez-vous ?


A - Dépression masquée
B - Délire paranoïaque
C - Confusion mentale aiguë
D - Bouffée délirante
E - Mélancolie délirante
Bonne(s) réponse(s) : E

Typique et classique.

Les tentatives de suicide antérieures vous paraissent ici témoigner :


A - D'états pathologiques antérieurs du même type
B - D'une tendance à la manipulation de l'entourage
C - D'une fragilité de la personnalité névrotique
D - D'un désir purement hétéro-agressif
E - De conduites impulsives chez une personnalité psychopathique
Bonne(s) réponse(s) : A

Il faut évoquer donc une P.M.D. et rechercher des accès maniaques.

Le traitement le plus approprié en urgence vous paraît être :


A - Sels de lithium isolément
B - Sels de lithium + tranquillisants
C - Sismothérapie
D - Neuroleptiques isolément par voie IM
E - Benzodiazépines par voie IM
Bonne(s) réponse(s) : C

L'alternative était antidépresseur en perfusion mais cela n'est pas propose ici.

Une jeune fille de 21 ans est hospitalisée pour des troubles du comportement et des propos bizarres. Elle ne s'alimente plus
depuis 3 jours, reste enfermée dans sa chambre, volets clos "pour arrêter les ondes de sectorisation". Elle est anxieuse,
agitée, se dit envoûtée par un "mage", qui lui impose des pensées et des actes. Depuis l'âge de 18 ans, elle a été hospitalisée
à 3 reprises en milieu psychiatrique pour des manifestations de ce type. Entre ces épisodes, elle vit avec sa mère, a
abandonné ses études, n'a aucune activité.
Elle se dit "autodidacte en sciences mathématiques" et passe son temps à chercher "l'équation universelle".

Parmi les diagnostics suivants lequel(lesquels) est(sont) le(s) plus probable(s) ?


A - Crise d'originalité juvénile
B - Manies délirantes
C - Bouffée délirante
D - Schizophrénie paranoïde
E - Anorexie mentale
Bonne(s) réponse(s) : D

En effet :
A.E. - Sont évidemment éliminés.
B - Est éliminé devant la bizarrerie qui existe entre les "crises".
C - Est éliminé sur la durée : 3 rechutes en 3 ans. Bizarrerie et désinvestissement (retrait).
1218
Exclusivement sur DOC - DZ : www.doc-dz.com NADJI 85
RESIDANAT EN POCHE TOME II
Cas Clinique en QCM

Quels sont les trois éléments cliniques les plus évocateurs de ce diagnostic ?
A - L'absence d'éléments déclenchants
B - L'age de début
C - L'importance de l'anxiété
D - La durée d'évolution
E - La présence d'un syndrome d'automatisme mental
Bonne(s) réponse(s) : A B D

Q.C.M. difficile.
B.D. - Ne posent pas de problème.
C - Est vrai : anxiété psychotique.
E - Est souvent présent.
Mais (C) et (E) ne sont pas les plus caractéristiques.

Parmi les renseignements suivants quel ou quels sont ceux qui ont le plus de valeur pour orienter le diagnostic ?
A - La mère de la malade est anxieuse et hyperprotectrice
B - Les parents sont divorcés
C - Un grand oncle maternel de 80 ans est traité pour un affaiblissement intellectuel
D - Un frère de 24 ans est hospitalié depuis 4 ans pour une psychose chronique
E - Un frère est décédé accidentellement il y a une dizaine d'années
Bonne(s) réponse(s) : D

C'est le seul item satisfaisant.


Récurrence familiale de certaines formes de schizophrénie.

Quel(s) examen(s) complémentaire(s) pourrait(ent) être utile(s) au diagnostic ?


A - Dosage de la sérotonine dans le L C R
B - Recherches de toxiques dans les urines
C - Tomodensitométrie cérébrale
D - Test de Rorschach
E - Test à la déxaméthasone
Bonne(s) réponse(s) : D

C'est un test de personnalité dit, test projectif : utile au diagnostic.


B - Est utile lors d'un premier accès pour le diagnostic différentiel avec les psycho-pharmacoses.

L'évolution habituelle de ce type d'affection est :


A - Evolution périodique intercritique sans aucune manifestation pathologique
B - Guérison sans récidive après traitement
C - Evolution chronique avec désorganisation progressive de la personnalité et des capacités d'adaptation
sociale
D - Enkystement du délire avec possibilité d'un fonctionnement social et professionnel satisfaisant
E - Evolution rapide vers une détérioration intellectuelle et physique avec état grabataire
Bonne(s) réponse(s) : C

C'est hélas ! le cas le plus fréquent.

Un jeune homme de 22 ans se présente en proie à une vive agitation anxieuse. Il se plaint de douleurs précordiales, d'une
sensation de "manque d'air", de fourmillements "insupportables" dans les membres. Il exprime sa "peur de mourir ou de
devenir fou". Cette crise aiguë est survenue brusquement il y a environ une heure, alors qu'il était, depuis 15 jours environ,
inquiet en permanence, incapable de s'endormir le soir. Un peu rassure par la présence du médecin, il vit depuis 15 jours
avec la crainte permanente et obsédante d'être atteint du SIDA. Il a entendu dire que les prostituées, qu'il fréquente, sont
souvent porteuses de cette maladie. Il ne peut, depuis, se défendre de cette peur, s'examine en permanence, incapable de se
concentrer sur son travail, il passe son temps à lire des ouvrages sur le SIDA qui ne parviennent qu'à l'inquiéter davantage.
Aucun antécédent pathologique, sinon, à l'âge de 16 ans, une peur injustifiée d'avoir attrapé la syphilis.

La crainte d'être atteint du SIDA, correspond, chez ce patient, à l'un des diagnostics suivants. Lequel ?
A - Conversion hystérique
B - Hypocondrie délirante
C - Nosophobie
D - Phobie d'impulsion
E - Maladie psychosomatique
Bonne(s) réponse(s) : C

Nosophobie typique ici : de plus en plus fréquente de nos jours.


B - Non, car il ne dit pas "j'ai le SIDA...".

1219
Exclusivement sur DOC - DZ : www.doc-dz.com NADJI 85
RESIDANAT EN POCHE TOME II
Cas Clinique en QCM

L'état dans lequel il se présente aux urgences peut être qualifié de l'un des termes suivants. Lequel ?
A - Anxiété chronique généralisée
B - Bouffée délirante
C - Etat confusionnel
D - Crise d'angoisse de type -panique
E - Dépression névrotique
Bonne(s) réponse(s) : D

C'est très à la mode.


La description est typique ici.

Le traitement le plus approprié en urgence vous paraît être, parmi les suivants :
A - Tranxene® IM un ampoule de 50 mg
B - Temesta® un mg : un comprimé
C - Gardenal® : 0,20 g par voie IM
D - Anafranil® 25 MG : un ampoule par voie IM
E - Droleptan® : un ampoule par voie IM
Bonne(s) réponse(s) : A

C'est la moins mauvaise des solutions proposées, mais ce n'est pas la meilleure actuellement.

La crainte éprouvée à 16 ans, d'avoir contracté la syphilis vous paraît être :


A - Un symptome du même ordre que les symptômes actuels
B - La première manifestation d'une hypochondrie chronique
C - Le premier symptôme d'une névrose obsessionnelle
D - Sans relation aucune avec la pathologie actuelle
E - La première manifestation d'une névrose hystérique grave
Bonne(s) réponse(s) : A

C - Non, 16 ans : trop jeune pour une structuration névrotique : c'est la période de latence.

M C., prêtre retraité de 82 ans, voit dans une maison de retraite. Son langage intentionel est un jargon incompréhensible, bien
qu'il paraisse s'adapter, par le rythme et l'intonation, à une conversation. Il somnole toute la journée et erre dans les couloirs. Il
est alors agressif. L'évocation de sa vie sacerdotale le fait pleurer. Il doit être aidé pour se raser et s'habiller. Cet état s'est
progressivement aggravé depuis 3 ans . Une hospitalisation est demandée à cause des troubles du comportement ; l'Haldol ®
(5 mg/j) et le Temesta ® (2,5 mg/j), prescrits depuis 2 mois n'ont aucun effet.

Quel diagnostic peut être évoqué ?


A - Etat dépressif réactionnel
B - Mélancolie d'involution
C - Troubles caractériels isolés de la sénescence
D - Démence sénilz
E - Confusion mentale iatrogène
Bonne(s) réponse(s) : D

Devant le tableau et l'âge.

Dans les 48 heures d'hospitalisation, quel traitement choisirez-vous ?


A - Neuroleptiques sédatifs
B - Benzodiazépines
C - Antidépresseurs tricycliques
D - Hypnotiques barbituriques
E - Bêta-bloqueur
Bonne(s) réponse(s) : A

Evident, on demande dans les 48 premières heures.

1220
Exclusivement sur DOC - DZ : www.doc-dz.com NADJI 85
RESIDANAT EN POCHE TOME II
Cas Clinique en QCM

Parmi les explorations complémentaires suivantes, laquelle peut être utile ?


A - Examen tomodensitométrique
B - Radios du crâne sans préparation
C - Ponction lombaire
D - Test à la dexamethasone
E - Examen psychométrique
Bonne(s) réponse(s) : A

QCM difficile car A.B.E. sont utiles.


Mais, on recherche toujours, guidé par la clinique, un processus tumoral.

Parmi ces cinq mesures d'assistance, laquelle peut-on proposer dans l'immédiat ?
A -Tutelle
B - Tutelle allégée
C - Placement volontaire
D - Curatelle
E - Signalement à la DDASS
Bonne(s) réponse(s) : C

A.B.D - Ne servent à rien dans l'immédiat. Il faut hospitaliser ( donc C, si refus) et demander une sauvegarde de justice.
E - Sans objet.

Madame X. est amenée en consultation par sa famille en raison de comportements inquiétants. Depuis quelques jours, elle
est quasi mutique, ne parlant que pour s'accuser d'une faute grave et pour demander la mort. Elle ne s'alimente plus, ne dort
plus, reste allongée toute la journée sur son lit. Elle fait preuve d'incurie. Elle a maigri de deux Kg en 10 jours, Madame X a
été hospitalisé à plusieurs reprises pour des épisodes semblables.

Quel est le diagnostic le plus probable ?


A - Syndrome dépressif névrotique
B - Mélancolie
C - Schizophrénie
D - Psychopathie
E - Névrose hystérique
Bonne(s) réponse(s) : B

C'est évident

Quelle est la modalité de traitement à instituer ?


A - Consultation ambulatoire
B - Hospitalisation à domicile
C - Hospitalisation
D - Maison de repos
E - Cure thermale
Bonne(s) réponse(s) : C

Sans commentaire.

Quelles sont la(les) thérapeutique(s) pouvant être instituée(s) ?


A - Sismothérapie
B - Antidépresseurs
C - Oxygénateurs cérébraux
D - Lithiothérapie
E - Amphétamines
Bonne(s) réponse(s) : A B

D - Sera peut-être à envisager par la suite.


A - Peut de nouveau être envisagé de première intention.

1221
Exclusivement sur DOC - DZ : www.doc-dz.com NADJI 85
RESIDANAT EN POCHE TOME II
Cas Clinique en QCM

Quel est le traitement préventif à envisager ?


A - Vitaminothérapie
B - Relaxation
C - Neuroleptiques
D - Lithium
E - Amphétamines
Bonne(s) réponse(s) : D

Devant :
- un syndrome mélancolique
- des antécédents de mélancolie.

Une femme de 30 ans est hospitalisée dans les suites d'une tentative de suicide par absorption médicamenteuse.
Cette intoxication volontaire survient au lendemain de la rupture d'une aventure sentimentale nouée il y a quelques semaines.
La malade souffre beaucoup, parait triste, considère que sa vie est finie et s'exprime d'une voix triste et monotone. Elle a
présenté il y a quelques mols des crises étiquetées "spasmophilie" et des douleurs pelviennes pour lesquelles aucune atteinte
organique n'a pu être mise en évidence.

Cette symptomatologie correspond à :


A - Un état dépressif réactionnel
B - Un état dépressif d'involution
C - Un état dépressif névrotique
D - Un état anxieux
E - Un état dépressif mélancolique
Bonne(s) réponse(s) : A C

En effet, chez cette patiente (A) est évident. Mais, la personnalité sous-jacente de type hystérique doit faire évoquer (C).

Cet épisode aigu peut-il évoluer vers :


A - La chronicité
B - Une rapide amélioration
C - Un délire de jalousie chronique
D - Une récidive suicidaire dans les mois ou années qui viennent
E - Une schizophrénie
Bonne(s) réponse(s) : A B D

A - C'est quelquefois le cas sur de telles structures de personnalité.

Vous pensez que cette patiente a une personnalité :


A - Schizoïde
B - Psychopathique
C - Hystérique
D - Sensitive
E - Obsessionnelle
Bonne(s) réponse(s) : C

Evident :
- spasmophilie
- douleurs pelviennes sans origine organique
- intolérance à la frustration
- personnalité immature.

Vous proposerez comme traitement :


A - Une psychothérapie
B - Des sels de lithium
C - Des antidépresseurs
D - Des neuroleptiques
E - Des anxiolytiques
Bonne(s) réponse(s) : A C E

C'est évident :
- le syndrome dépressif existe donc (C) et (E).
A - Est utile chez de telles personnes.

1222
Exclusivement sur DOC - DZ : www.doc-dz.com NADJI 85
RESIDANAT EN POCHE TOME II
Cas Clinique en QCM
Mr V. 46 ans, boucher, vient d'être opéré le 20 juillet en urgence d'une hernie inguino-scrotale étranglée. Ce patient
pléthorique, au faciès érythrosique, présente 4 jours après l'intervention les troubles suivants : il se croit aux abattoirs et attend
impatiemment les fêtes de Noël. Il est excité, se lève sans cesse. Dans la nuit il a des gestes automatiques d'aiguisage de
couteaux, il appelle sa femme à haute voix pour lui rappeler les commandes de viande à effectuer. Il cherche des frigidaires
dans sa chambre .Il dit à l'infirmière qu'il veut s'en aller car le leader du comité local du syndicat veut le faire licencier. Sa
température est de 38 degrés. L'état abdominal et digestif est normal.
L'examen neurologique ne note qu'un tremblement rapide.

Quel est le diagnostic psychiatrique ?


A - Bouffée délirante aiguë
B - Mélancolie délirante
C - Délire paranoïaque
D - Etat confuso-onirique
E - Etat maniaque
Bonne(s) réponse(s) : D

On retrouve :
- une D.T.S.
- un onirisme à thème professionnel.
Avec le faciès érytrosique.
Après une opération chirurgicale (donc un sevrage) et des signes cliniques (température, tremblements) donc, sûrement :
délirium trémens.

Les éléments sémiologiques du diagnostic comportent :


A - Idées délirantes mélancoliques
B - Désorientation spatiale
C - Onirisme
D - Idée délirante de persécution
E - Tremblement
Bonne(s) réponse(s) : B C D E

Evident.

Quel est le facteur de causalité le plus probable ?


A - Sevrage aux benzodiazépines
B - Abus de L S D 25
C - Encéphalopathie porto-cave
D - Sevrage alcoolique
E - Complication infectieuse
Bonne(s) réponse(s) : D

(Cf : question [220]).

Quel est ou quels sont le(s) médicament(s) utile(s) au traitement immédiat ?


A - Clomipramine (Anafranil®)
B - Meprobamate (Equanil®)
C - Sécobarbital (Immenoctal®)
D - Vitamine B 1
E - Halopéridol (Haldol®)
Bonne(s) réponse(s) : B D E

A noter que l'équanil est de moins en moins employé. Il est pourtant utile.

Quel est l'élément sémiologique susceptible de compliquer le plus l'évolution à long terme ?
A - La désorientation temporelle
B - La fièvre
C - L'onirisme
D - L'idée délirante de persécution
E - L'excitation psychomotrice
Bonne(s) réponse(s) : D

Souvent, chez les alcooliques, on retrouve un délire passionnel (paranoïaque) : de jalousie ou de persécution.

1223
Exclusivement sur DOC - DZ : www.doc-dz.com NADJI 85
RESIDANAT EN POCHE TOME II
Cas Clinique en QCM

L'évolution peut se faire vers :


A - Guérison totale
B - Syndrome de Korsakoff
C - Encéphalopathie de Gayet-Wernicke
D - Délire post onirique
E - Schizophrénie
Bonne(s) réponse(s) : A B C D

En général (A).
BC - Sont prévenus par la vitamine B1.
D - N'est pas rare.

Mr Y., 22 ans, est amené par ses copains à la consultation. Son regard est lointain et égaré : il est indifférent. Ses amis disent
que, depuis quelques jours il entend l'écho de sa voix au loin. Par moments il a peur car il pense qu'on lui impose certains
actes, qu'on devine ses pensées. Pendant que ses amis évoquent ses troubles, il apparaît étonné ou indifférent.
Les troubles ont commencé il y a une dizaine de jours, une à deux heures après l'absorption de 30 microgrammes de LSD 25,
comme cela se produit deux fois par mois. Il a dit peu de temps après la prise qu'il avait des impressions de lévitation du corps
et voyait avec netteté des figures spiralées et colorées de manière répétitive et accélérée. Le lendemain il a dit à ses amis
qu'ils étaient tous surveillés, que le LSD 25 n'était pas pur mais mélangé à un pesticide.
L'examen somatique est normal. L'orientation temporo-spatiale est normale.

Quel est le diagnostic psychiatrique ?


A - Schizophrénie paranoïde
B - Psychose hallucinatoire chronique
C - Bouffée délirante polymorphe
D - Pharmacopsychose aiguë
E - Confusion mentale
Bonne(s) réponse(s) : D

Q.C.M. difficile car (C) est toujours possible. L'apparition de perplexité anxieuse, de syndrome d'influence/automatisme
mental, d'hallucinations, de délire de persécution après la prise de LSD signent le diagnostic.

Quel ou quels sont les éléments sémiologiques du diagnostic ?


A - Hallucinations visuelles
B - Idées délirantes d'influence
C - Confusion mentale
D - Automatisme mental
E - Pensées compulsionnelles
Bonne(s) réponse(s) : A B D

Evident, tout est dans le texte.

Quelle est ou quelles sont les substances toxiques susceptibles d'induire ce type de troubles ?
A - Amphétamines
B - Ether
C - Barbituriques
D - Cocaïne
E - Héroïne
Bonne(s) réponse(s) : A C D E

Il faut le savoir. L'éther donne des états comateux.

Quel est ou quels sont les médicaments efficaces pour le traitement ?


A - Chlorpromazine (Largactil®)
B - Clomipramine (Anafranil®)
C - Clonidine (Catapressan®)
D - Halopéridol (Haldol®)
E - Narcan®
Bonne(s) réponse(s) : A D

C - Sert pour les sevrages aux opiacés


E - Sert pour les overdoses aux opiacés.

1224
Exclusivement sur DOC - DZ : www.doc-dz.com NADJI 85
RESIDANAT EN POCHE TOME II
Cas Clinique en QCM

Quelle est ou quelles sont la ou les modalité(s) évolutive(s) possible(s) au cours des deux années suivantes ?
A - Guérison définitive
B - Délire chronique paranoïaque
C - Pharmaco psychose chronique
D - Schizophrénie
E - Psychose hallucinatoire chronique
Bonne(s) réponse(s) : A C

Q.C.M. difficile car (B) à déjà été décrit.

Quelle est ou quelles sont la(les) modalité(s) nécessaire(s) au traitement au long cours ?
A - Arrêt de la prise de LSD 25
B - Chimiothérapie neuroleptique
C - Relation psychothérapique d'appui
D - Bêta-bloqueur
E - Clonidine (Catapressan®)
Bonne(s) réponse(s) : A C

Au long cours.
A - Evident.
C - Evident.
B - N'est pas au long cours.
E - Non, à court terme : uniquement pour les opiacés.

Mr M, 42 ans, se présente au commissariat pour se plaindre de son voisinage. Il dit au commissaire qu'il est l'objet de
surveillance et d'un complot politique. Il y a un an, il n'a pas obtenu la promotion qu'il attendait. Il pense que les propos
malveillants qu'a tenus son collègue de bureau y sont pour quelque chose.
Il y a 8 jours il a trouvé un pneu de sa voiture crevé à l'aide d'un couteau.
Ce n'est pas le fait du hasard. Depuis il comprend tout, sauf le but de ce complot. Mais il pense qu'il ne mérite pas d'être
l'objet de telles manoeuvres. Il se montre anxieux, plutôt abattu par tout cela. Il se proclame honnête, serviable et dévoué.
Aucun autre trouble n'est présent. Il n'avait jamais eu de trouble psychique jusque-là.

Quel diagnostic retenez-vous parmi les suivants ?


A - Mélancolie
B - Schizophrénie paranoïde
C - Délire paranoïaque
D - Névrose obsessionnelle
E - Hystérie
Bonne(s) réponse(s) : C

Tableau typique mais il n'existe pas de sthénicité. Ce qui est souvent le cas.
On a :
1 - Délire systématise
2 - Mécanisme interprétatif
3 - Thème persécutif
Préjudice.

Quel est le mécanisme constitutif des principaux troubles psychiques ?


A - Imaginatif
B - Hallucinatoire
C - Conversion hystérique
D - Interprétatif
E - Confabulatoire
Bonne(s) réponse(s) : D

Bien sûr. Cf : question précédente

Quel est le diagnostic différentiel à discuter en premier, du fait des implications thérapeutiques ?
A - Psychose hystérique
B - Psychose hallucinatoire chronique
C - Mélancolie délirante
D - Délire imaginatif
E - Hébéphrénie
Bonne(s) réponse(s) : C

En effet, on retrouve quelquefois des thèmes qui peuvent donner le change avec "des persécutions" dans les mélancolies.
D'autre part, ici, le patient est "anxieux et abattu", ce qui est rare dans les personnalités paranaïaques.

1225
Exclusivement sur DOC - DZ : www.doc-dz.com NADJI 85
RESIDANAT EN POCHE TOME II
Cas Clinique en QCM

Parmi les traitements suivants, quel est celui susceptible d'être utile ?
A - Neuroleptiques
B - Sels de lithium
C - Antidépresseurs I.M.A.O.
D - Cure de sommeil
E - Sismothérapie
Bonne(s) réponse(s) : A

C'est évident. Ils sont en général peu efficaces.

Dans le cas où il n'accepterait pas l'hospitalisation jugée nécessaire, quelle est la mesure administrative
adéquate ?
A - Tutelle
B - Signalement à la DDASS
C - Internement d'office
D - Curatelle
E - Placement volontaire
Bonne(s) réponse(s) : C

On peut hésiter avec (E) mais, dans la paranoïa, il convient d'être prudent... risque de persécution....

Vers l'âge de 25 ans, une jeune femme ressent des malaises caractérisés par des sensations de striction du thorax et du cou
avec une peur de mourir.
Ces malaises surviennent sans cause déclenchante évidente, dans des circonstances diverses. Ils durent 1/2 heure environ.
Les examens cliniques et paracliniques sont normaux aussi bien au cours des malaises qu'en dehors de ceux-ci.
Après un an d'évolution la symptomatologie s'enrichit de la crainte de la survenue d'un malaise dans un lieu où la patiente ne
pourrait pas être secourue. Elle réduit dès lors ses activités, évitant les endroits isolés mais aussi ceux où il y a une foule et
plus généralement toute sortie dans la rue
Elle ne peut plus sortir qu'accompagnée. Quand elle est dans un lieu public, elle est prise d'une peur intense, inexpliquée,
avec l'impression qu'elle va perdre connaissance.

Parmi les symptômes suivants, cette observation met en évidence :


A - Angoisse psychotique
B - Anxiété aiguë
C - Agoraphobie
D - Nosophobie
E - Phobie d'impulsion
Bonne(s) réponse(s) : B C

Le tableau est celui d'attaque de panique (crise d'angoisse aiguë), agoraphobie.


ADE- Sans objet.

Parmi les syndromes névrotiques suivants, quel est celui ou quels sont ceux que l'on peut diagnostiquer ici ?
A - Névrose obsessionnelle
B - Névrose hystérique
C - Névrose traumatique
D - Névrose d'angoisse
E - Névrose phobique
Bonne(s) réponse(s) : D E

A - Non, ce sont ici des phobies typiques, sans notion d'aucun trait obsessionnel.
B.C. - Sans objet.
D - Evident : névrose non structurée.
E - Sur l'agoraphobie et la phobie de la foule, les conduites d'évitement, les "objets" contraphobiques (accompagnant),
l'anxiété anticipatoire.

Le fait de ne plus sortir seule de chez elle peut être qualifié par un seul des termes suivants. Lequel ?
A - Rite conjuratoire
B - Conduite d'évitement
C - Clinophilie
D - Claustrophobie
E - Anxiété anticipatoire
Bonne(s) réponse(s) : B

Sans commentaire.

1226
Exclusivement sur DOC - DZ : www.doc-dz.com NADJI 85
RESIDANAT EN POCHE TOME II
Cas Clinique en QCM

Parmi les traitements suivants, le ou lesquels vous semble(nt) approprié(s) à cet état ?
A - Cure type psychanalytique seule
B - Psychothérapie d'inspiration analytique associée à des antidépresseurs
C - Psychothérapie comportementale associée à des antidépresseurs et à des anxiolytiques
D - Anxiolytiques de type benzodiazépines seuls
E - Neuroleptiques incisifs
Bonne(s) réponse(s) : C

Sans hésitation.
Les B.Z.D. sont peu efficaces seuls.
L'analyse donne ici peu de résultats.

Une jeune fille de 17 ans est amenée en consultation par sa mère sur les conseils du médecin traitant.
Elle est très maigre et a perdu 16 kg en un an. Elle a progressivement réduit son alimentation alléguant des douleurs
d'estomac, quand on la force à manger, elle vomit presque régulièrement.
Par ailleurs, le comportement de cette jeune fille s'est progressivement modifié : en famille elle se montre tyrannique, en
particulier elle contrôle de façon étroite l'alimentation des autres membres de la famille et ne fréquente pratiquement plus ses
camarades de classe. Ses résultats scolaires restent excellents.
Cette jeune fille ne se plaint de rien, affirmant au contraire qu'à part ses douleurs d'estomac elle ne s'est jamais mieux portée.
L'examen clinique est négatif. Le transit gastroduodénal ne montre rien d'anormal. A l'interrogatoire on note une aménorrhée
depuis 10 mois.

Quel diagnostic retenez-vous ?


A - Etat dépressif chronique
B - Installation d'une psychose dissociative
C - Anorexie mentale
D - Crise sévère d'adolescence
E - Névrose phobique
Bonne(s) réponse(s) :C

Tableau typique. Le fait qu'elle ne voit plus ses camarades n'est pas un signe de retrait autistique. D'autre part, "ses résultats
scolaires restent excellents", ce qui élimine (B).

Parmi les symptômes suivants quel est celui dont l'existence est indispensable au diagnostic ?
A - Les vomissements
B - Les troubles du caractère
C - L'aménorrhée
D - Les douleurs d'estomac
E - L'inhibition psychomotrice
Bonne(s) réponse(s) : C

Symptôme qui s'installe rapidement et qui disparaît en dernier.

On peut rencontrer des conduites de restriction alimentaire au cours de :


A - Anorexie mentale
B - Délire de persécution
C - Dépression névrotique
D - Hernie hiatale
E - Névrose obsessionnelle
Bonne(s) réponse(s) : A B C

Q.C.M. difficile.
A - Est vrai.
B - Par peut de l'empoisonnement.
C - L'anorexie avec perte de l'appétit fait partie des dépressions.

A l'origine d'une hypokaliémie lors de l'évolution de la maladie de cette adolescente, on peut trouver :
A - Réduction de l'apport alimentaire
B - Vomissements répétés
C - Utilisation des diurétiques
D - Usage de laxatifs
E - Hyperactivité motrice
Bonne(s) réponse(s) : A B C D

Les diurétiques sont peu employés, mais les laxatifs fréquemment.

1227
Exclusivement sur DOC - DZ : www.doc-dz.com NADJI 85
RESIDANAT EN POCHE TOME II
Cas Clinique en QCM

Le projet thérapeutique proposé à cette malade comporte :


A - Hospitalisation avec séparation de la famille
B - Hormonothérapie
C - Psychothérapie individuelle
D - Réalimentation par sonde gastrique
E - Traitement antidépresseur
Bonne(s) réponse(s) : A C

A - Est souvent très profitable avec mise en place d'un contrat....

On doit craindre une évolution péjorative à long terme devant :


A - Importance de l'amaigrissement
B - Début précoce des troubles
C - Importance de la composante dépressive
D - Présence de vomissements
E - Prise en charge tardive
Bonne(s) réponse(s) : A E

A - Oui, car il peut mettre en jeu le pronostic vital.


B - Non, plutôt début tardif.
C - Ce n'est pas le cas ici.
E - Bien évidemment.

Frédéric E. 22 ans, est hospitalisé après une tentative de suicide médicamenteuse.


Il ne dort plus la nuit : des voix le menacent d'emprisonnement, lui promettent la mort. Il pense être suivi par des policiers en
civil ; les vols d'oiseaux lui prédisent sa fin.
Cela a commencé il y a un mois, peu après la lecture du journal qui relatait une noyade dont il s'est senti responsable.
Ses traits sont figés, ses gestes lents. Il parle d'une voix monocorde.
L'hospitalisation lui parait totalement inutile : seul un châtiment le soulagerait en lui permettant d'expier ses crimes.
Il n'a jamais présenté de troubles psychiques auparavant.

Classiquement la présence d'un délire élimine le diagnostic de :


A - Paranoïa
B - Mélancolie
C - Psychose hallucinatoire chronique
D - Schizophrénie
E - Etat névrotique aigu
Bonne(s) réponse(s) : E

Sans commentaire.

La symptomatologie présente dans cette observation met en évidence :


A - Des obsessions
B - Des phobies
C - Des idées délirantes
D - Des hallucinations
E - Une inhibition psychomotrice
Bonne(s) réponse(s) : C D E

Sans commentaire.

Quel(s) diagnostic(s) discuter en priorité ?


A - Un début de schizophrénie
B - Une psychose hallucinatoire chronique
C - Un accès mélancolique
D - Une névrose phobo-obsessionnelle
E - Une affection neurologique
Bonne(s) réponse(s) : A C

Le diagnostic est (C) très probablement. Mais, vu l'âge, il faut discuter (A).

1228
Exclusivement sur DOC - DZ : www.doc-dz.com NADJI 85
RESIDANAT EN POCHE TOME II
Cas Clinique en QCM

Au cours de l'évolution d'ici quelques semaines, vous devez craindre la survenue de :


A - Une altération des fonctions intellectuelles
B - L'installation d'une pathologie plus discordante
C - L'apparition d'un syndrome de Korsakoff
D - Un suicide
E - Un accès maniaque
Bonne(s) réponse(s) : B D E

Q.C.M. difficile.
Par ordre de probabilité :
D - Risque de tout syndrome mélancolique.
E - Par inversion de l'humeur.
B - Vu l'âge.

Marianne G. 20 ans, s'exprime avec indifférence, hausse les épaules quand on lui parle, ne semble répondre qu'avec regret.
Elle prétend qu'un "déblocage" s'est produit en elle : le "bloc" qu'elle avait dans le cerveau gauche a disparu. remplacé par "un
souffle palpable" qui lui dit ce qu'il faut faire : boire. manger ou non. Son côté gauche a "spirituellement disparu", ses vertèbres
s'effacent. Elle s'arrête parfois en milieu de phrase et après un long silence change de sujet.
Marianne refuse de s'alimenter.
Le bilan clinique est normal, hormis un amaigrissement modéré. Il n'y a pas d'aménorrhée.
Depuis l'âge de 16 ans, elle souffre de troubles semblables avec des rémissions incomplètes.
Voici un an qu'elle vit chez une grand-mère, lisant la nuit, dormant le jour, sans autres activités.

La symptomatologie de l'observation comporte :


A - Hallucination cénesthésiques
B - Sitiophobie
C - Parakinésies
D - Hallucinations psychiques
E - Hypersomnie
Bonne(s) réponse(s) : D

Sans commentaire.

Quel diagnostic retenir :


A - Arriération mentale
B - Dépression névrotique
C - Hypochondrie
D - Schizophrénie
E - Délire d'imagination
Bonne(s) réponse(s) : D

Schizophrénie paranoïde :
- terrain âge et évolution sur 4 ans
- automatisme mental/syndrome d'influence
- transformation corporelle
- etrangeté - troubles du comportement
- barrage.

Le traitement de première intention comporte :


A - Un neuroleptique d'action prolongée
B - Un neuroleptique sédatif
C - Un tranquillisant
D - Un neuroleptique incisif
E - Un antidépresseur
Bonne(s) réponse(s) : B D

Souvent en association.
A - Pas en première intention.

1229
Exclusivement sur DOC - DZ : www.doc-dz.com NADJI 85
RESIDANAT EN POCHE TOME II
Cas Clinique en QCM

La surveillance d'un tel traitement porte essentiellement sur :


A - La tension artérielle
B - La vigilance
C - La symptomatologie neurologique
D - La symptomatologie thyroïdienne
E - L'état cutané
Bonne(s) réponse(s) : A B C E

A - Risque d'hypotension orthostatique.


B - Trouble de la vigilance. Syndrome confusionnel.
C - Dystonie aiguë.
E - Photosensibilisation, allergie (en particulier, pour le Largactyl®).

Monsieur M. 35 ans, sans antécédent pathologique, est amené par son épouse en consultation pour un état d'agitation
d'installation brutale. Le malade parle sans cesse, son discours est intarissable, ses idées s'enchaînent sans lien logique,
changent constamment, les "coq à l'âne" sont nombreux.
Il se prétend un génial inventeur, promis à un grand avenir scientifique a la gloire et à la richesse. Le malade est d'emblée
familier avec son examinateur et répond de façon amusée ou ironique à ses questions. Il nie tout caractère pathologique à son
comportement. Son épouse apprend au médecin qu'il ne dort plus depuis trois jours et qu'il lui fait de nombreuses avances sur
un mode provocant et obscène qui contraste avec son caractère timide et réservé.Il n'a pas pris de substances toxiques et on
ne note pas d'antécédent éthylique.

Devant un tel épisode d'agitation, quel est le diagnostic qui vous parait le plus probable ?
A - Agitation paranoïaque
B - Agitation hystérique
C - Agitation maniaque
D - Agitation psychopathique
E - Agitation schizophrénique
Bonne(s) réponse(s) : C

Evident.
Installation brutale.
Logorrhée.
Tachypsychie - "coq à l'âne".
Idées de grandeur, de richesse.
Familiarité.
Anosognosie.
Hypersexualité.

Parmi les symptômes suivants, quel est celui qui est présent dans cette observation ?
A - Fuite des idées
B - Théâtralisme
C - Mythomanie
D - Discordance
E - Maniérisme
Bonne(s) réponse(s) : C

Sans commentaire.

Un tel tableau clinique peut évoquer aussi une affection organique. Quelle est, parmi les affections suivantes,
celle qui pourrait entraîner un tel tableau ?
A - Tumeur frontale
B - Maladie d'Alzheimer
C - Insuffisance surrénale
D - Maladie de Creutzfeldt-Jacob
E - Insuffisance vertébro-basilaire
Bonne(s) réponse(s) : A

Mais en fait le tableau est différent et il existe une niaiserie.

Quelle est parmi les chimiothérapies suivantes. celle qui doit être utilisée devant un tel état ?
A - Un antidépresseur sédatif
B - Une benzodiazépine
C - Un barbiturique
D - Une butyrophénone
E - Un tranquillisant
Bonne(s) réponse(s) : D

En fait, un neuroleptique. C'est d'ailleurs rarement une butyrophénone et souvent une phénotiazine sédative.
1230
Exclusivement sur DOC - DZ : www.doc-dz.com NADJI 85
RESIDANAT EN POCHE TOME II
Cas Clinique en QCM

En cas de refus du malade de se faire soigner, le médecin peut envisager une mesure de placement volontaire.
Parmi les propositions suivantes, quelle(s) est(sont) celle(s) qui est(sont) conforme(s) à la loi ?
A - Le certificat du médecin demandant le placement doit être accompagné d'une demande de placement
faite par une personne de l'entourage (ex : épouse)
B - Le certificat peut être fait par le médecin même s'il n'est pas spécialiste
C - Le certificat doit nécessairement comporter le diagnostic des troubles présentés par Monsieur M.
D - La réalisation du placement incombe aux services de police ou de gendarmerie
E - Le placement doit se faire dans un établissement régi par la loi du 30 juin 1838
Bonne(s) réponse(s) : A B E

Simple ! relire la question.

Mr B., âge de 43 ans est admis en psychiatrie pour insomnie. Dès l'entrée, on constate chez lui une excitation psychomotrice
avec logorrhée. Ses propos sont difficilement compréhensibles en raison d'une fuite des idées. néanmoins, on peut repérer
des idées de grandeur et de toute puissance. Il est agressif avec les autres et son contact est facile mais fait de causticité. La
vigilance est normale, sans désorientation temporo-spatiale.
Son épouse signale que Monsieur B. comme sa mère, a déjà été hospitalisé en milieu psychiatrique à plusieurs reprises
depuis l'âge de 25 ans. L'examen somatique est sans anomalie.

Quel est le diagnostic probable ?


A - Bouffée délirante
B - Accès maniaque
C - Schizophrénie
D - Délire paranoïaque
E - Ivresse pathologique
Bonne(s) réponse(s) : B

Insomnie.
Logorrhée, excitation psychomotrice.
Tachypsychie.
Idées de grandeur.
Familiarité.
Antécédents personnels et familiaux.

Quel est le traitement que vous instaurez d'emblée ?


A - Antidépresseurs tricycliques en perfusion
B - Diazépam (Valium®) par voie intraveineuse
C - Halopéridol (Haldol®) par voie intramusculaire
D - Méprobamate (Equanil®) par voie intramusculaire
E - Phénobarbital (Gardenal®) par voie intramusculaire
Bonne(s) réponse(s) : C

Evident. En général, en association avec une phénotiazine sédative.

Cet état peut évoluer vers :


A - Récidive sur un mode identique
B - Apparition d'un état dépressif
C - Apparition d'un état mixte
D - Transformation en délire hallucinatoire chronique
E - Apparition d'un état démentiel
Bonne(s) réponse(s) : B

C'est un devoir de le faire. Elle prend effet immédiatement.

Quelle est la mesure administrative à prendre dans l'immédiat ?


A - Placement d'office
B - Sauvegarde de justice
C - Curatelle
D - Tutelle
E - Contrôle judiciaire
Bonne(s) réponse(s) : C

Suivant les antécédents.

1231
Exclusivement sur DOC - DZ : www.doc-dz.com NADJI 85
RESIDANAT EN POCHE TOME II
Cas Clinique en QCM

Quel est le traitement prophylactique à envisager au long cours chez ce patient ?


A - Benzodiazépine
B - Neuroleptique sédatif
C - Sels de lithium
D - Antidépresseur sédatif
E - Neuroleptique incisif
Bonne(s) réponse(s) : A B C

Sans commentaire.

Une femme de 45 ans, mariée, mère de 2 filles de 19 et 14 ans est hospitalisée pour des céphalées.
L'examen clinique, l'EEG, le scanner sont normaux. La malade est triste figée, son visage est peu expressif, elle parle
lentement et à voix basse.
Elle se plaint d'être fatiguée, de ne pouvoir dormir. Elle a perdu l'appétit et a maigri de 6 kg en 3 mois. Elle est convaincue
d'avoir une tumeur cérébrale et pense que les médecins lui cachent la vérité parce qu'elle est condamnée et qu'ils ne peuvent
rien faire. L'entourage signale que cet état s'est installé à la suite du mariage de sa fille il y a 6 mois.
Il n'y a aucun antécédent médical ou psychiatrique chez cette malade dont l'activité et le comportement étaient tout à fait
normaux jusqu'ici. On ne retrouve pas non plus d'antécédents psychiatriques dans la famille.

La conviction qu'a la malade d'avoir une tumeur cérébrale peut être qualifiée de :
A - Trouble psychosomatique
B - Phénomène de conversion
C - Idée délirante hypocondriaque
D - Obsessions
E - Phobie du cancer
Bonne(s) réponse(s) : C

A. Non, le trouble psychosomatique est le résultat somatique d'un problème psychique.


B. Non, c'est la traduction fonctionnelle d'un conflit infra psychique, se voit dans la névrose hystérique.
D. Rien à voir.
E. Fait partie des nosophobies.
C. Conviction délirante d'être atteint d'une maladie.

Les 2 diagnostics à évoquer sont :


A - Dépression névrotique
B - Dépression réactionnelle
C - Dépression mélancolique
D - Psychasthénie
E - Délire de relation
Bonne(s) réponse(s) : B C

QCM difficile.
Quelle est la différence entre A et B ? Elle n'est pas catégorielle en tout cas.

L'insomnie dont se plaint la malade est d'autant plus évocatrice du diagnostic porté qu'il s'agit :
A - D'une insomnie d'endormissement
B - D'une insomnie totale
C - D'une insomnie matinale
D - De réveils fréquents avec cauchemar
E - De somnambulisme
Bonne(s) réponse(s) : A C D

QCM difficile.
A et C sont classiques dans la mélancolie.
D est fréquent dans les syndromes dépressifs non mélancolique.
B est extrêmement rare et se voit souvent dans les syndromes maniaques.

Dans un tel cas, vous prescrivez :


A - Hospitalisation en milieu psychiatrique
B - Psychothérapie ambulatoire
C - Placement volontaire si refus de soins
D - Mise sous sauvegarde de justice
E - Hospitalisation domicile
Bonne(s) réponse(s) : A C

C. Devant la gravité du tableau :


- idées délirantes
- anorexie importante
- troubles du sommeil importants.
1232
Exclusivement sur DOC - DZ : www.doc-dz.com NADJI 85
RESIDANAT EN POCHE TOME II
Cas Clinique en QCM

Parmi les thérapeutiques suivantes laquelle(lesquelles) est(sont) indiquée(s) chez cette malade ?
A - Benzodiazépine
B - Butyrophénone
C - Imipramine
D - Phénothiazine
E - Sismothérapie
Bonne(s) réponse(s) : A C D

QCMdifficile.
A est indiqué si il existe une tonalité ancienne au syndrome dépressif, ou en association avec des antidépresseurs
B. Non car il n'existe pas d'éléments hallucinatoires, et les butyrophénones sont dépressogènes donc à éviter ici.
D. Les phénotiazines sont utiles ici par leurs propriétés sédatives, antidélirantes, anxiolytiques.
E. Le tableau n'est pas une indication classique des ECT, mais cette réponse est très discutable.
C. Bien évidemment, c'est une indication majeure.

Un sujet de 50 ans va se plaindre au Procureur de la République et lui demande d'intervenir parce que la Gendarmerie auprès
de laquelle il habite, utilise son antenne et son poste de radio pour surveiller ses faits et gestes. Il le sait parce qu'il capte
directement les voix de gendarmes qui répètent ce qu'il fait, répètent sa pensée, et aussi font des commentaires de ce type :
"on l'aura". Ils le traitent de "salaud, de pédéraste". La population capte aussi ces émissions. Le malade est très inquiet, pense
que l'on veut le "liquider". Il perçoit la nuit des "odeurs de poisson" qui passent sous sa porte et des "champs magnétiques"
qui agissent sur ses organes génitaux. Toutes ces actions ont débuté il y a une dizaine d'années et vont en s'amplifiant.

Le diagnostic le plus vraisemblable est celui de :


A - Schizophrénie paranoïde
B - Bouffée délirante
C - Psychose hallucinatoire chronique
D - Délire d'interprétation
E - Personnalité paranoïaque
Bonne(s) réponse(s) : C

Tableau classique avec :


- délire hallucinatoire auditif, olfactil de persécution
- automatisme mental
- chez une personne de 40 - 50 ans.
A. Non : pas de dissociation, trop âgé.
B. Non : pas de tonalité thymique, trop âgé.
D. Non : dans le tableau décrit, on note une grande participation hallucinatoire.
E. Non, bien sûr, il existe ici des éléments délirants.

Ce cas met en jeu le ou les mécanismes psychologiques suivants :


A - Projectif
B - Imaginatif
C - Interprétatif
D - Hallucinatoire
E - Intuitif
Bonne(s) réponse(s) : A D

A. C'est le mécanisme psychodynamique à proprement parler de la P.H.C.


D. C'est évident.
C. Non.

Le médicament qui a la plus grande probabilité d'être actif dans ce cas est :
A - Phénothiazines (Nozinan®)
B - Butyrophénones (Halopéridol®)
C - Benzodiazépines (Valium®)
D - Psychostimulants
E - Barbituriques (Nembutal®)
Bonne(s) réponse(s) : B

C'est le neuroleptique qui a le plus d'activité hallucinolytique.

1233
Exclusivement sur DOC - DZ : www.doc-dz.com NADJI 85
RESIDANAT EN POCHE TOME II
Cas Clinique en QCM

Dans le cas décrit, les réactions comportementales habituelles peuvent se traduire par :
A - Fugue
B - Claustration
C - Suicide
D - Activités procédurières
E - Agressivité
Bonne(s) réponse(s) : A B C E

A. Souvent par angoisse ou pour fuir le ou les persécuteurs.


B. Classique, c'est la réaction la plus courante.
C. Toujours à craindre dans les moments de dépression.
D. Non habituel
E. Possible.

Monsieur C..., 29 ans, est admis en urgence dans un service de psychiatrie pour des troubles du comportement apparus
depuis 24 heures. A l'examen, le malade est perplexe, modérément agité. Le contact est difficile mais le malade arrive (avec
difficulté) à faire part d'une expérience d'étrangeté et de transformation du monde ambiant. Angoissé, il affirme que son corps
se transforme en s'amincissant et que son cerveau va se "liquéfier". Il a par moment des attitudes d'écoute.
Sa mère confirme que son comportement général s'est modifié depuis 15 jours avec tendance à l'isolement et au repli sur soi.
Monsieur C... n'a par ailleurs aucun antécédent psychiatrique ou médical.

Quel(s) symptôme(s) repérez-vous dans ce cas clinique ?


A - Dépersonnalisation
B - Illusion
C - Conversion
D - Automatisme mental
E - Hallucination auditive
Bonne(s) réponse(s) : A E

A. Est évident
E. Déduit des attitudes d'écoute.

Quel est le diagnostic le plus plausible (en première intention) ?


A - Accès confuso-onirique
B - Début de schizophrénie
C - Bouffée délirante polymorphe
D - Mode d'entrée dans un délire paraphrénique
E - Mode d'entrée dans un délire paranoïaque
Bonne(s) réponse(s) : C

A. Ne comporte pas d'hallucinations auditives.


B. Rien ne permet de l'affirmer d'autant qu'il est précisé l'absence d'antécédent.
D. Rien à voir (jeu de mot : paraphrénie - schizophrénie).
E. Rien à voir.

Quel(s) médicament(s) prescrivez-vous ?


A - Tranquillisant (benzodiazépine)
B - Neuroleptique incisif (type Haldol®)
C - Tranquillisant type carbamate
D - Neuroleptique sédatif (type Nozinan®)
E - Antidépresseur sédatif
Bonne(s) réponse(s) : B D

A. N'a pas ou peu d'activité sur l'angoisse psychotique.


B. D. De préférence monothérapie mais l'association est fréquente.

Quelle(s) évolution(s), à long terme, est(sont) possible(s) ?


A - Guérison
B - Récidive d'épisodes identiques
C - Psychose maniaco-dépressive
D - Evolution vers la chronicité
E - Evolution démentielle
Bonne(s) réponse(s) : A B C D

En fait tout est possible.


E. Est hors de propos.

1234
Exclusivement sur DOC - DZ : www.doc-dz.com NADJI 85
RESIDANAT EN POCHE TOME II
Cas Clinique en QCM
Mr C., 48 ans, consulte pour des malaises paroxystiques, durant quelques heures, qui associent : oppression thoracique,
vertiges, palpitations, peur envahissante de mourir, de perdre la raison, de s'effondrer, impression que "tout le menace", qu'il
ne pourra rentrer chez lui. Ils surviennent quand il est seul dans la rue, dans les ascenseurs, sous les tunnels, sur les ponts.
Il balise ses déplacements en notant la situation des pharmacies car la simple vue des officines arrête ses malaises. La liste
des médecins de garde l'apaise pour le week-end.
L'aggravation actuelle fait suite à son divorce et à un nouveau licenciement pour absentéisme malgré une excellente
qualification (expert-comptable). Il se décrit comme un inquiet dont la vie est centrée par des stratagèmes car, en cas de
malaise, il fuit son travail et reste chez lui d'où ses difficultés professionnelles.

Pendant les malaises, on observe :


A - Une désorientation temporo-spatiale
B - Un délire de persécution
C - Un sentiment de dépersonnalisation
D - Un raptus impulsif
E - Un divinement de la pensée
Bonne(s) réponse(s) : C

QCM très vicieux.


A. Non : c'est possible mais aucun élément ne permet de le soupçonner ici. Le fait "qu'il ait peur de ne pas pouvoir rentrer
chez lui "ne témoigne pas d'une désorientation.
C. Il est dit "sentiment " et non "idée délirante".

Comment qualifier au mieux ces épisodes ?


A - Crise d'angoisse
B - Obsession idéative
C - Automatisme mental
D - Accès maniaque
E - Décompensation délirante
Bonne(s) réponse(s) : A

Bien sûr : typiquement une attaque de panique.

Son absentéisme doit évoquer :


A - Un dédoublement de personnalité
B - Un état somnambulique
C - Un évitement
D - Un syndrome d'influence
E - Une fuite des idées
Bonne(s) réponse(s) : C

Bien sûr, le reste est sans objet.


Attaque de panique, construction phobique secondaire/évitement.

Le rôle de l'emplacement des pharmacies et de la liste des médecins est à comprendre chez ce patient comme
celui :
A - D'une épreuve de réalité
B - D'un effet placebo
C - D'une interprétation délirante en réseau
D - D'un objet contraphobique
E - D'un rituel compulsif
Bonne(s) réponse(s) : D

Cf. QCM précédente et la construction phobique secondaire.

Quel diagnostic doit être évoqué ?


A - Bouffée délirante
B - Névrose obsessionnelle
C - Névrose phobique
D - Névrose d'angoisse
E - Hystérie
Bonne(s) réponse(s) : D

C. Non car les crises d'angoisse existent avant les phobies qui ne sont que secondaires.

1235
Exclusivement sur DOC - DZ : www.doc-dz.com NADJI 85
RESIDANAT EN POCHE TOME II
Cas Clinique en QCM

Les thérapeutiques possibles comportent :


A - Thérapie comportementale
B - Psychothérapie d'inspiration psychanalytique
C - Anxiolytiques
D - Relaxation
E - Tricycliques
Bonne(s) réponse(s) : A C D E

Aucun résultat des psychothérapies analytiques dans cette indication.

Un homme de 40 ans est conduit aux urgences par sa femme pour un état d'agitation qui dure depuis quelques jours. Il dort
deux heures par nuit, marche toute la journée, range et dérange ses affaires. Il a fait récemment beaucoup d'achats et même
des chèques sans provisions. Il est débraillé, logorrhéique, ayant ces jours derniers agressé ses voisins en riant. Il a
également, aux dires de sa femme, récemment écrit des lettres au Président de la République et à des ministres pour leur
proposer des solutions. Dans ses antécédents on note l'existence de deux épisodes dépressifs ayant cédé sous électrochocs
au cours d'hospitalisations. Il nie toute pathologie, prétendant ne s'être jamais senti aussi bien ni aussi intelligent. Il refuse
l'hospitalisation, que sa femme par contre souhaite très nettement. L'examen clinique note une hypertension artérielle traitée
par salidiurétiques.

Parmi les symptômes suivants, vous retrouvez dans cette observation :


A - Trouble de l'humeur
B - Tasikinésie
C - Discordance
D - Idées délirantes mégalomaniaques
E - Diffluence de la pensée
Bonne(s) réponse(s) : A B D E

A. Evidemment, exaltation de l'humeur. Nous sommes, ici, devant un accès maniaque typique.
B. Bien sûr : il marche toute la journée, ne tient pas en place...
E. Bien entendu. Autrement dit, sa pensée va dans tous les sens (différent de la discordance).

Parmi les propositions suivantes concernant le délire présenté par ce patient, la ou lesquelles est ou sont
exacte(s) ?
A - Il s'agit d'idées délirantes congruentes à l'humeur
B - Les thèmes délirants principaux sont persécutifs
C - Le délire est principalement sous-tendu par des hallucinations
D - Il s'agit d'un délire de type paranoïaque
E - Il s'agit d'un délire interprétatif
Bonne(s) réponse(s) : A

C'est évident.
D. Oui, ce sont des idées délirantes congruentes à l'humeur.

Quel(s) diagnostic(s) vous parai(ssen)t pouvoir être retenu(s) ?


A - Bouffée délirante polymorphe
B - Schizophrénie dysthymique
C - Manie délirante
D - Confusion mentale aiguë
E - Psychose maniaco-dépressive
Bonne(s) réponse(s) : C E

A. Non, le délire n'est pas paranoïde, l'âge est trop grand et les antécédents orientent ailleurs.
B. Sans objet.
C. Oui, mais le terme délirant peut quelque fois être gênant.
D. Non
E. Pathologies de fond ici :
- 2 syndromes dépressifs (graves ?)
- 1 accès maniaque
donc PMD bipolaire.

1236
Exclusivement sur DOC - DZ : www.doc-dz.com NADJI 85
RESIDANAT EN POCHE TOME II
Cas Clinique en QCM

Quelle(s) mesure(s) doit-on prendre pour ce patient dans l'immédiat ?


A - Placement volontaire
B - Sauvegarde de justice
C - Placement d'office
D - Instauration d'une tutelle
E - Prise en charge ambulatoire avec instauration d'un traitement neuroleptique
Bonne(s) réponse(s) : A B

A. Il refuse des soins nécessaires et sa femme comprend la nécessité d'hospitalisation.


B. Bien sûr, toujours y penser dans ce cas. Mesure de protection, simple, valable 2 mois renouvelable pour 6 mois.
C . Non cf. A.
D. Non car le trouble maniaque n'est pas un trouble chronique.
E. Sans objet.

Quelle(s) médicament(s) prescrit-on pour le traitement de ce trouble ?


A - Halopéridol (Haldol®) 3 mg par jour
B - Halopéridol (Haldol®) 30 mg par jour
C - Halopéridol (Haldol®) 300 mg par jour
D - Sultopride (Barnetil®) 40 mg par jour
E - Sultopride (Barnetil®) 1,8 g par jour
Bonne(s) réponse(s) : A E

Pour les autres propositions, les posologies sont incorrectes.

Quel(s) traitement(s) au long cours peut-on proposer chez ce malade ?


A - Carbonate de lithium (Téralithe®)
B - Neuroleptique
C - Benzodiazépine
D - Carbamazépine (Tégretol®)
E - Lithium (Oligosol®)
Bonne(s) réponse(s) : A D

Mais ici, c'est l'indication typique de A.


B. Pas au long cours.
C. Aucun intérêt
E. L'homéopathie n'est pas au programme !!!

Mademoiselle H... 35 ans est hospitalisée pour la deuxième fois. Elle est tantôt abattue, tantôt excitée, déambulant, riant par
intermittence. Au moment de l'entretien, elle se dresse sur son lit et crie : "qu'on me fiche la paix, je ne suis pas une fille
publique, je ne veux pas aller au palais de justice"... Elle parle avec volubilité, mais les mêmes phrases reviennent maintes
fois au cours de ses déclarations, souvent débitées comme des litanies.
Elle dit qu'elle est la fille de Napoléon, on l'emprisonne, on la viole, on lui lance des poudres, nuit et jour on l'injurie à travers
les murs.
Dans les antécédents on note un accès analogue il y a 7 ans. Entre temps le fonctionnement psychique a été normal. Son
frère aîné a été traité pour un état mélancolique. L'examen neurologique est normal.

L'analyse de l'énoncé du cas vous permet d'identifier la ou les notions de :


A - Idées délirantes
B - Troubles de l'humeur
C - Excitation psychomotrice
D - Hallucinations
E - Anxiété massive
Bonne(s) réponse(s) : A B C D E

Réponses évidentes.
E est discutable, on s'attend à en retrouver mais elle n'est pas décrite dans l'énoncé, elle est suggérée dans le syndrome
décrit.

1237
Exclusivement sur DOC - DZ : www.doc-dz.com NADJI 85
RESIDANAT EN POCHE TOME II
Cas Clinique en QCM

Quel diagnostic retenez-vous ?


A - Psychose hallucinatoire chronique
B - Manie délirante
C - Schizophrénie paranoïde
D - Bouffée délirante polymorphe
E - Mélancolie à forme psychotique
Bonne(s) réponse(s) : D

A. Ce n'est pas le tableau.


B. Non, les troubles de l'humeur (mixtes d'ailleurs) sont au second plan.
C. Non, on ne peut pas l'affirmer car entre les deux accès, le fonctionnement était normal.
D. Deuxième épisode.
E. Non, le tableau est différent.
(Une mélancolie est toujours psychotique, il faut dire délirante ici).

En faveur du diagnostic, vous retenez :


A - Systématisation du délire
B - Caractère dominant des troubles de l'humeur
C - Antécédents de mélancolie du frère
D - Multiplicité des thèmes délirants
E - Antécédents d'accès identique
Bonne(s) réponse(s) : D

A. Non, ici le délire est paranoïde.


B. Non, ils sont au second plan.
C. Difficile de répondre. La comorbidité familiale PMD-schizophrénie étant démontrée mais cela me semble un peu pointu
pour le concours de l'internat, donc non.
D. Oui.
E. Non, un antécédent de BDP n'oriente en rien le diagnostic d'un épisode suivant. Puisque l'on démontre aussi une
augmentation de la fréquence des antécédents de BDP dans les PMD.

Le traitement de l'accès comporte :


A - Antidépresseur tricyclique
B - Neuroleptique incisif
C - Psychothérapie institutionnelle
D - Carbamazépine (Tégrétol®)
E - Neuroleptique sédatif
Bonne(s) réponse(s) : B E

A. Noter qu'on peut se contenter de B qui rompt la chaîne délire / anxiété / agitation.

L'évolution pourra être marquée par :


A - Guérison sans récidive
B - Récidive identique
C - Schizophrénie paranoïde
D - Evolution maniaco-dépressive
E - Délire chronique interprétatif
Bonne(s) réponse(s) : A B C D

E. Est trop rare pour en tenir compte.

1238
Exclusivement sur DOC - DZ : www.doc-dz.com NADJI 85
RESIDANAT EN POCHE TOME II
Cas Clinique en QCM
Un patient de 56 ans présente depuis 6 mois au moins une insomnie partielle avec des réveils matinaux précoces
accompagnés d'anxiété. Il est effondré à l'idée qu'une nouvelle journée arrive, pressentant une catastrophe.
Il se plaint de ne pouvoir rien faire : il est réduit à néant et devient insensible à ses proches. Semi-mutique, son entourage dit
qu'il ressasse de vieilles idées telles que celle d'avoir volé des chiffres dans son entreprise.
Rien ne fait pour le détourner de sa douleur.
Cet état contraste avec le caractère jovial et bon vivant qu'il présente habituellement.
A l'interrogatoire, on ne décèle aucun antécédent ni aucun facteur déclenchant.
L'examen somatique est normal en dehors d'un état saburrale.

Quel est le diagnostic le plus probable ?


A - Névrose post-traumatique
B - Psychasthénie
C - Paranoïa
D - Dépression réactionnelle
E - Dépression mélancolique
Bonne(s) réponse(s) : E

Classique. A et D sont là pour faire douter le candidat.


L'auto-accusation décrite fait partie d'un syndrome de culpabilité délirante très fréquent dans la mélancolie.

Quel(s) est(sont) le(s) symptôme(s) retrouvé(s) dans cette observation ?


A - Anesthésie affective
B - Boulimie
C - Fabulation
D - Aboulie
E - Persécution
Bonne(s) réponse(s) : A D

Sémiologie dure.
D. Est retrouvée dans quasiment tous les syndromes dépressifs.

Parmi les propositions suivantes, il est exact que :


A - L'affection est au stade chronique et ne nécessite qu'un traitement ambulatoire
B - Le risque suicidaire justifie l'hospitalisation
C - Le traitement repose sur la stimulation du malade
D - L'hospitalisation s'impose même en cas de refus du malade
E - Le recours à la loi de 1838 (placement volontaire) peut être nécessaire
Bonne(s) réponse(s) : B D E

Evident, question de bon sens.

Les contre-indications éventuelles du traitement à envisager comportent :


A - Syndrome extrapyramidal
B - Glaucome à angle fermé
C - Insuffisance hépatocellulaire
D - Troubles du rythme cardiaque
E - Adénome prostatique
Bonne(s) réponse(s) : A B D E

Seule A est discutable mais dans ce tableau à anxiété massive, on a souvent recours aux neuroleptiques donc A est vrai.

Quelle est la principale indication thérapeutique ?


A - Neuroleptique deshinibiteur
B - Neuroleptique sédatif
C - Antidépresseur
D - Abstention thérapeutique
E - Médicament tranquillisant
Bonne(s) réponse(s) : C

Evident. Accessoirement B, cf. question précédente.

1239
Exclusivement sur DOC - DZ : www.doc-dz.com NADJI 85
RESIDANAT EN POCHE TOME II
Cas Clinique en QCM
Une femme de 42 ans a perdu 6 kg en deux mois. Son dernier fils s'est marié et s'est installé dans une autre région. Elle a
consulté plusieurs généralistes et spécialistes car elle craint d'avoir un cancer : elle est généralement triste mais surtout
inquiète, sur le qui-vive, observant le moindre malaise, le moindre changement dans le fonctionnement de ses organes. Elle
n'a pas d'antécédents particuliers. Elle a toujours été active et rarement malade. En l'interrogeant on apprend que grâce à
plusieurs hypnotiques elle s'endort facilement le soir mais se réveille dès 4 heures du matin et ne peut plus se rendormir ; elle
se sent un peu plus détendue le soir mais toute la journée "traîne" dans sa maison sans savoir que faire de son temps et sans
contacter un ami ou un parent.

Parmi les diagnostics suivants, lequel retenez-vous ?


A - Névrose hystérique
B - Schizophrénie dysthymique
C - Etat dépressif
D - Névrose d'angoisse
E - Hypocondrie délirante
Bonne(s) réponse(s) : C

A. Non la conversion est un autre symptôme, il s'agit d'un dysfonctionnement sans substratum anatomique.
B. Non rien à voir.
C. Oui, c'est un syndrome dépressif atypique avec préoccupations hypochondriaques.
D. Non car il manque les attaques de panique mais c'est le diagnostic différentiel ici
E. Pas de délire.

Parmi les examens complémentaires suivants quels sont les deux prioritaires avant tout traitement médical ?
A - Radiographie du crâne
B - Test de frénation à la dexamétasone
C - Electromyogramme
D - Electrocardiogramme
E - NFS-VS
Bonne(s) réponse(s) : D E

QCM impensable ! devait être éliminé si D est évident pour mettre le traitement en route.
A. Non aucun intérêt sauf si signes de localisation neurologique. .
B. Aucun intérêt ici.
C. Idem.
E. A la limite, systématique dans tout bilan d'entrée.

Chez cette malade on peut considérer que le risque de suicide est :


A - Négligeable
B - Majoré quand on évoque cette éventualité avec la patiente
C - Lié à la volonté consciente de faire pression sur l'entourage
D - Plus grand s'il y a déjà eu des tentatives antérieures
E - Suffisant pour justifier une hospitalisation
Bonne(s) réponse(s) : D E

A. Non, possibilité à ne jamais négliger.


B. Ne veut rien dire.
C. Réponse de "comptoir".
D. Prouvé épidémiologiquement.
E. Bien sûr, même attitude devant tout syndrome dépressif avéré.

Parmi les conduites thérapeutiques suivantes, vous retenez :


A - Prescription d'antidépresseurs par voie parentérale en milieu hospitalier
B - Psychothérapie de soutien en ambulatoire
C - Prescription du sels de lithium
D - Electrochocs
E - Prescription de benzodiazépines
Bonne(s) réponse(s) : A E

A. E. Evidents d'autant qu'il existe une grande tonalité anxieuse.


B. Ne suffit pas, le tableau est grave.
C. Sans objet.
D. Ce n'est pas l'indication type.

1240
Exclusivement sur DOC - DZ : www.doc-dz.com NADJI 85
RESIDANAT EN POCHE TOME II
Cas Clinique en QCM

Parmi les modalités évolutives suivantes, trois sont plus probables que les autres. Indiquez lesquelles :
A - Guérison totale et définitive
B - Evolution vers un délire chronique
C - Evolution vers une schizophrénie
D - Rechute après guérison
E - Survenue d'épisodes maniaques
Bonne(s) réponse(s) : A D E

B. Non car on retrouve un élément déclenchant et cette évolution est rare.


C.Sans objet
E.Toujours une possibilité qu'un syndrome dépressif soit une voie d'entrée dans la maladie bipôlaire.

Madame G.... 25 ans, mère de deux enfants, éprouve depuis longtemps une anxiété diffuse, dramatisant les conséquences de
nombreux événements. Depuis quelques mois elle est très angoissée à l'idée de prendre le métro pour aller travailler. Elle a
peur d'y avoir un malaise. Elle demande le plus souvent à son mari de l'accompagner à son travail.
Il n'y a pas de troubles de l'humeur ni du tonus psychomoteur.

Cette observation met en évidence :


A - Idée obsédante
B - Anxiété diffuse
C - Etat dépressif
D - Agoraphobie
E - Phobie d'impulsion
Bonne(s) réponse(s) : B D

B. Est évident.
D. En effet la phobie du métro peut-être considérée comme une agoraphobie.
Ici, on nous présente une construction phobique secondaire.

Quel(s) diagnostic(s) retenez-vous ?


A - Dépression névrotique
B - Névrose obsessionnelle
C - Névrose d'angoisse
D - Névrose hypocondriaque
E - Névrose phobique
Bonne(s) réponse(s) : C

QCM difficile.
Nous pensons qu'il faut éliminer E car la personnalité décrite n'est pas de type phobique et parce que le noyau de la
pathologie est l'anxiété diffuse, flottante, appelée anxiété généralisée.

Quel(s) type(s) de médicaments peu(ven)t être prescrit(s) dans ce cas ?


A - Neuroleptique sédatif
B - Antidépresseur tricyclique
C - Antidépresseur I.M.A.O.
D - Benzodiazépine
E - Association antidépresseur et benzodiazépine
Bonne(s) réponse(s) : B C D E

QCM difficile.
On nous dit : "peuvent être prescrit".
A. Non, ce n'est pas une indication.
B. D. E. Sont classiques quoique E est discuté.
C. Est d'une très bonne efficacité ici quoique peu prescrit en France.

Il est également utile de proposer :


A - Relaxation
B - Sismothérapie
C - Psychothérapie individuelle
D - Thérapie de déconditionnement
E - Thérapie familiale
Bonne(s) réponse(s) : A C D

A. Est évident.
C. Oui puisqu'il n'est pas précisé le type de psychothérapie.
D. Oui, au moins pour se débarrasser des phobies parasitantes.
B. E. Sans objet ici.

1241
Exclusivement sur DOC - DZ : www.doc-dz.com NADJI 85
RESIDANAT EN POCHE TOME II
Cas Clinique en QCM

Il est utile de compléter ce traitement en conseillant :


A - Au moins de refuser l'accompagnement
B - Un congé de longue durée
C - D'avoir un autre enfant
D - L'éventualité d'une psychothérapie
E - Une consultation cardiologique
Bonne(s) réponse(s) : D

QCM particulièrement insensé.


A. Ne soyons pas sadique !
B. Non, car on l'enferme alors dans sa pathologie.
E. Sans objet.

Mademoiselle J... Lucette, âgée de 28 ans. vient de faire une tentative de suicide par ingestion médicamenteuse
(Valium®,Temesta®, Halcion®). Le psychiatre du service de réanimation où elle avait été admise a recommandé une
hospitalisation en psychiatrie.
A l'examen, Mademoiselle J... parle lentement avec une expression de tristesse. Elle se plaint d'une grande lassitude. d'un
abattement extrême d'une perte du goût de vivre et d'un désintérêt total. Elle est anxieuse.
Elle évoque en pleurant une rupture sentimentale survenue il y a deux mois, raison pour laquelle "elle voulait en finir avec la
vie". Il s'agit en fait d'une troisième tentative, les deux premières après un échec sentimental et professionnel. Abordant sa
biographie, Melle J... se plaint de ne rencontrer que des échecs. Elle apparaît souvent en état de sujétion à l'égard des autres
et en recherche perpétuelle de réassurance affective.
Elle a maigri récemment de 5 kg, a perdu l'appétit et ne dort presque plus.

Quel est le diagnostic qui vous semble le plus plausible ?


A - Dépression endogène
B - Dépression nevrotico-réactionnelle
C - Déséquilibre psychique
D - Névrose d'angoisse
E - Névrose phobique
Bonne(s) réponse(s) : B

QCM évident.

En fonction des éléments biographiques dont vous avez connaissance. à quel type de personnalité correspond
le tableau clinique décrit ?
A - Sensitive
B - Obsessionnel
C - Hystérique
D - Schizoïde
E - Psychopathique
Bonne(s) réponse(s) : C

Tableau typique :
. femme jeune
. echecs sentimentaux fréquents avec dramatisation de l'existence
. suggestionabilité.

Quel(s) traitement(s) médicamenteux prescrivez-vous chez cette malade pendant son hospitalisation ?
A - Largactil®
B - Tranxène®
C - Laroxyl®
D - Equanil®
E - Piportil®
Bonne(s) réponse(s) : B C

Tout en sachant que B peut suffire car le Laroxyl® est un antidépresseur sédatif.qui n'est pas anxiogène.
B. Malgré tout, car elle présente une grande composante anxieuse.
A. D. E. Sont sans objet ici.

1242
Exclusivement sur DOC - DZ : www.doc-dz.com NADJI 85
RESIDANAT EN POCHE TOME II
Cas Clinique en QCM

Cet épisode peut évoluer vers :


A - La chronicité
B - Une amélioration rapide
C - Un délire chronique
D - Une schizophrénie
E - Une récidive suicidaire
Bonne(s) réponse(s) : A B E

A. C'est le problème : évolution vers une dépréssion chronique résistante.


B. E. vEidents.
C. D. A. Priori pas plus de risque que la population générale.

A l'issue de son hospitalisation, quelle(s) mesure(s) thérapeutique(s) allez-vous conseiller ?


A - Psychothérapie institutionnelle
B - Psychothérapie d'inspiration psychanalytique
C - Thérapie comportementale
D - Sels de lithium
E - Relaxation
Bonne(s) réponse(s) : B

QCM difficile.
A. Sans objet.
C. Peu d'efficacité ici.
E. Est toujours possible mais ce n'est pas son indication.
B. A quelque efficacité devant ce tableau...

Monsieur A... 19 ans consulte, amené par ses parents. Depuis 8 mois ceux-ci en déplorent l'inefficience scolaire. Il passe de
nombreuses heures dans sa chambre. Il parle de moins en moins, devient indifférent aux problèmes de la famille. Selon lui,
tout va bien : il ne se sent pas malade et juge sa consultation inutile. Il explique son changement depuis plusieurs mois par
son besoin de découvrir les fondements biologiques de la foi religieuse. Il a accumulé un grand nombre de livres sur les
multiples communautés religieuses.
Le contact est détaché et lointain, interrompu par des sourires inexpliqués et surprenants. Le sommeil est irrégulier.

Quel est le diagnostic le plus probable ?


A - Crise d'originalité juvénile
B - Etat dépressif
C - Invasion schizophrénique
D - Psychasthénie
E - Etat névrotique anxieux
Bonne(s) réponse(s) : C

Tableau classique (cf question sur la schizophrénie)

Quel est ou quels sont les symptômes évocateurs du diagnostic ?


A - Sommeil irrégulier
B - Détachement affectif
C - Affects inappropriés
D - Pensées discordante
E - Dénégation de l'état pathologique
Bonne(s) réponse(s) : B C D E

Evident.
A. Evoque toute pathologie, et n'est donc pas plus évocateur de la schizophrénie que d'autres diagnostics.

Quel est le traitement médicamenteux le plus adéquat ?


A - Benzodiazépine
B - Neuroleptique incisif
C - Psychostimulant
D - Antidépresseur et neuroleptique
E - Antidépresseur
Bonne(s) réponse(s) : B

Evident, de préférence en monothérapie.

1243
Exclusivement sur DOC - DZ : www.doc-dz.com NADJI 85
RESIDANAT EN POCHE TOME II
Cas Clinique en QCM

Il est également utile de prescrire :


A - Psychothérapie familiale
B - Psychothérapie de soutien
C - Relaxation
D - Psychothérapie institutionnelle
E - Thérapie de déconditionnement
Bonne(s) réponse(s) : A D

QCM difficile au niveau de l'internat.


A. Sont très efficaces (cf thérapies systémiques).
B. Ne sont pas efficaces dans les psychoses dissociatives et le terme est très imprécis.
C. E. Aucun intérêt.
D. Ont fait preuve d'efficacité.

Dans la mesure où l'hospitalisation plein temps n'est pas indispensable quelle(s) modalité(s) de prise en charge
peu(ven)t être utile(s) ?
A - Consultations régulières
B - Hôpital de jour
C - C.A.T.
D - Maison de repos
E - Visites à domicile
Bonne(s) réponse(s) : A B C

Mais de toute façon l'hospitalisation s'impose :


- pour faire le bilan clinique
- pour mettre le traitement en place.

Monsieur Y... 43 ans, ingénieur est amené en urgence dans le service de psychiatrie par le SAMU. Il vient de tenter
d'étrangler sa femme. Il justifie son acte en disant que leur voisin expliquera mieux que quiconque la situation. Sa femme
décrit les troubles suivants. Depuis deux ans son mari manifeste une jalousie morbide, la questionnant avec véhémence sur
ses sorties quotidiennes. Depuis quelques semaines elle pense qu'il la fait suivre, il note ses heures de sorties, téléphone à
l'improviste pour vérifier sa présence à la maison, il est encore plus méfiant, susceptible., vindicatif et tyrannique, convaincu
de connaître la vérité.

Quel est le diagnostic psychiatrique ?


A - Dépression à forme psychotique
B - Schizophrénie paranoïde
C - Paranoïa sensitive
D - Délire passionnel
E - Délire d'interprétation
Bonne(s) réponse(s) : D

Délire paranoïaque passionnel. Thème : jalousie.


Tableau typique.

Ce type de personnalité est caractérisé par :


A - Méfiance
B - Doute obsédant
C - Surestimation de soi
D - Fuite passive
E - Psychorigidité
Bonne(s) réponse(s) : A C E

Evident.
B. Personnalité obsessionnelle.
D. Personnalité phobique.

Parmi les éléments cliniques évocateurs de dangerosité (actuelle et future) vous retenez :
A - Hétéro-agressivité
B - Réticence
C - Tendance dépressive
D - Désignation nominale du rival
E - Absence d'alcoolisme
Bonne(s) réponse(s) : A D

Mais c'est surtout D qui est "le" critère de dangerosité.

1244
Exclusivement sur DOC - DZ : www.doc-dz.com NADJI 85
RESIDANAT EN POCHE TOME II
Cas Clinique en QCM

Le traitement à court terme comporte :


A - Hospitalisation en service ouvert
B - Neuroleptiques
C - Antidépresseur tricyclique
D - Carbamazépine (Tegrétol®)
E - Déconditionnement
Bonne(s) réponse(s) : B

Mais ils sont peu efficaces ici.


A. Sûrement pas, il faut souvent avoir recours au PO.

Quelle est la mesure médico-légale appropriée que pourrait nécessiter ce cas ?


A - Sauvegarde de justice
B - Demande de curatelle
C - Placement volontaire
D - Demande de tutelle
E - Placement d'office
Bonne(s) réponse(s) : E

C. Non car on risque d'induire un autre délire centré sur le signataire du PV

Les modalités thérapeutiques utiles à long terme comportent :


A - Sorties d'essai
B - Neuroleptique à action prolongée
C - Sels de lithium
D - Cure psychanalytique
E - Thérapie du couple
Bonne(s) réponse(s) : A B

A. Bien sûr mais à long terme.


B. Seule possibilité d'observance du traitement dans cette pathologie.

Mme B., 32 ans, se décide à consulter. Sa tenue est stricte avec sa voix monocorde et peu d'émotion elle fait le récit de ses
troubles. Depuis la mort de son père il y a 3 ans elle manifeste une crainte des animaux, des frôlements des passants. Sa
crainte de la saleté est pratiquement permanente, au point qu'elle se sente contrainte de se dévêtir intégralement pour faire le
ménage, prenant plusieurs douches aussitôt après. Elle se contorsionne pour pénétrer dans ses vêtements de façon à ne pas
toucher l'extérieur. Elle lave tout, les interrupteurs, le téléphone.
Ses craintes permanentes sont en réalité apparues il y a plusieurs années après la naissance de sa fille. Au début elle lavait
sans arrêt ses jouets. D'ailleurs elle pense qu'elle n'aime pas assez sa fille.

Les symptômes décrits correspondent à :


A - Attaques de panique
B - Idées obsédantes
C - Phobies d'impulsion
D - Obsessions compulsives
E - Etat dépressif
Bonne(s) réponse(s) : B D E

A. Non.
B. Evident.
C. Non, il n'est pas décrit ici.
D. Bien sûr : les lavages...
E. Oui, voire monotone, baisse de l'émotion exprimée.
Auto-accusation : "je n'aime pas ma fille".

Quel diagnostic retenez-vous ?


A - Dépression névrotique
B - Dépression du post-partum
C - Névrose phobique
D - Névrose d'angoisse
E - Névrose obsessionnelle
Bonne(s) réponse(s) : E

Question piège car A et B sont vraies mais le noyau pathologique est E.


C. Non, car ici il existe des obsessions, des compulsions et des rituels, et pas uniquement des phobies.

1245
Exclusivement sur DOC - DZ : www.doc-dz.com NADJI 85
RESIDANAT EN POCHE TOME II
Cas Clinique en QCM

Vous pouvez attendre une efficacité thérapeutique sur les symptômes décrits en prescrivant :
A - Antidépresseur tricyclique
B - Neuroleptique butyrophénone
C - Antidépresseur tricyclique et anxiolytique
D - Cure type psychanalytique
E - Psychothérapie
Bonne(s) réponse(s) : A C D E

QCM mal posé.


A. C. Evidents.
D. A une certaine efficacité.
B. Rien à voir.
E. Bien sûr mais de quel type ?

Les modalités évolutives habituelles comportent :


A - Atténuation des troubles
B - Etat inchangé
C - Accès dépressif
D - Apparition de symptômes nouveaux
E - Evolution schizophrénique
Bonne(s) réponse(s) : A B C D E

Seul E me semble difficile à admettre vu l'âge de la patiente. Mais cette évolution est très fréquente dans les N.O. plus jeune.
Donc oui.

Monsieur T..., 45 ans, est amené ou service d'urgence après avoir été trouvé dans la rue, en train d'errer, la tenue négligée,
tenant des propos incohérents.
Il est en proie à des mouvements désordonnés et croit reconnaître certains de ses proches parmi les membres du personnel ;
par moments, il a l'impression d'être chez lui, puis s'aperçoit de son erreur et finalement s'égare et s'embrouille : il cherche ses
affaires, s'emporte, pleure, puis retombe dans un état de perplexité et d'hébétude.
Cet état évolue par étapes successives de sursauts et d'accalmies. Il refuse de manger et n'a pas absorbé de liquides depuis
plusieurs heures.
Il revient d'un séjour de plusieurs mois dans un pays tropical et prenait un traitement antimalarique. Le bilan biologique
systématique est négatif. La température est à 37,8 degrés.

Dans la symptomatologie présente dans cette observation, vous identifiez :


A - Désorientation temporelle et spatiale
B - Fausses reconnaissances
C - Délire paranoïde
D - Amnésie antérograde
E - Fugue
Bonne(s) réponse(s) : A B E

Question de sémiologie.
A. B. Evidents.
D. Non : ce sont des oublis à mesure.
E. Errements ou fugue ? réponse difficile.

Quel(s) diagnostic(s) doi(ven)t être porté(s) ?


A - Délire aigu
B - Bouffée délirante aiguë
C - Confusion mentale
D - Maladie de Pick
E - Psychose hallucinatoire aiguë
Bonne(s) réponse(s) : C

A. B. Sans objet.
E. D. Le tableau est différent.

Parmi les étiologies susceptibles d'être évoquées, laquelle doit-on retenir en premier lieu ?
A - Neurologique
B - Métabolique
C - Toxique
D - Infectieuse
E - Endogène
Bonne(s) réponse(s) : D

Première étiologie devant une confusion chez un homme de 45 ans revenant d'un pays d'endémie palustre.

1246
Exclusivement sur DOC - DZ : www.doc-dz.com NADJI 85
RESIDANAT EN POCHE TOME II
Cas Clinique en QCM

Quelle(s) est (sont) la (les) mesure(s) thérapeutique(s) la(les) plus appropriée(s) ?


A -.Electroconvulsivothérapie
B - Neuroleptiques sédatifs
C - Surveillance sans traitement
D - Réhydratation
E - Vitaminothérapie
Bonne(s) réponse(s) : B D E

A. Sans objet.
B. Utile mais surveillance stricte de la vigilance, permet de diminuer l'agitation anxieuse.
D. Bien sûr.
E. Toujours adjoindre vit B1 B6 souvent car l'étiologie alcoolique doit être suspectée.

Quel est le pronostic à court terme ?


A - Evolution vers un délire chronique
B - Guérison sans séquelles
C - Encéphalopathie
D -.Risque suicidaire
E - Syndrome secondaire malin
Bonne(s) réponse(s) : B

C'est tout de même l'évolution la plus fréquente.

Salvador B..., 52 ans, est hospitalisé pour la première fois en service de psychiatrie après une tentative de suicide
médicamenteuse (10 comprimés de Tranxène 10), sa femme le menaçait d'un divorce, prétextant notamment les abus
d'alcool et les violences de son époux. Il boit plus d'un litre de vin par jour depuis bientôt 20 ans. Ses tentatives de sevrage ont
toutes échoué. Il minimise le rôle de l'alcool et pense que sa femme veut se débarrasser de lui pour toucher une pension
alimentaire. Salvador se plaint d'asthénie, tremblements et vomissements matinaux, de cauchemars sur des thèmes
professionnels. Pour cela, il est actuellement en arrêt de travail depuis 6 mois. Couvert de sueurs, les traits figés, il semble
inquiet. Tantôt, il paraît absent de l'entretien, tantôt il cherche réassurance auprès du personnel.

Le tableau clinique décrit évoque :


A - Une intoxication alcoolique aiguë
B - Une décompensation paranoïaque
C - Un syndrome de Korsakoff
D - Des symptômes de sevrage
E - Une bouffée délirante
Bonne(s) réponse(s) : D

Tableau de prédélirium tremens avec troubles physiques (sueurs, tremblements) et psychiatriques (troubles de la vigilance,
angoisse).

Que redoutez-vous à court terme ?


A - Suicide
B - Bloc auriculo-ventriculaire
C - Délirium tremens
D - Encéphalopathie de Gayet-Wernicke
E - Une crise comitiale
Bonne(s) réponse(s) : A C D E

A - Compte-tenu de son antécédent récent de tentative de suicide.


C - Evident.
D - Souvent associé à un coma hyperthermique.
E - Chez l'éthylique chronique dénutri, sujet à des troubles digestifs.

A long terme, on peut redouter :


A - Myopathie
B - Reprise de l'intoxication alcoolique
C - Apparition d'une discordance
D - Polynévrite
E - Délire de jalousie
Bonne(s) réponse(s) : B D E

C - Signe de schizophrénie.
E - Délire paranoïaque passionnel systématisé en secteur.

1247
Exclusivement sur DOC - DZ : www.doc-dz.com NADJI 85
RESIDANAT EN POCHE TOME II
Cas Clinique en QCM

Devant, un tel tableau clinique, quel(s) médicament(s) pourra-t-on utiliser dans les premières heures ?
A - Carbamates
B - Vitamines B 1
C - Disulfiram (Espéral)
D - Réhydratation
E - Marsilid
Bonne(s) réponse(s) : A B D

A - Carbamates ou benzodiazépines ou neuroleptiques.


B - Afin de prévenir syndrome de Korsakoff et encéphalopathie de Gayet-Werwicke.
D - Elément majeur du traitement.
C - Peut avoir un intérêt, associé à des techniques comportementales, dans le maintien de l'abstinence.
E - IMAO, contre-indiqué.

Bernard B..., 40 ans, élude les questions ou répond à côté en évoquant ses préoccupations habituelles : "On le prend pour un
proxénète, les gens lui envoient des rayons en le regardant et arrêtent le cours de sa pensée; on parle de lui à la télévision et
sur les affiches publicitaires, etc..."
Son expression est animée de grimaces, de sourires complices, de brusques interruptions inexpliquées ; son discours est
émaillé de termes grossiers ou d'allusions sans rapport avec le sujet. Les premiers troubles datent de l'adolescence : alors à
type de réactions caractérielles mégalomaniaques, ils laissent place à l'incohérence et à l'hermétisme au cours d'une
aggravation progressive. Evitant tout traitement régulier, il a perdu sa fortune dans la gestion déplorable de son héritage
paternel.

Quel(s) symptôme(s) relevez-vous dans cette observation ?


A - Syndrome d'influence
B - Troubles du cours de la pensée
C - Délire paranoïde
D - Discordance
E - Ralentissement psychomoteur
Bonne(s) réponse(s) : A B C D

A - Mr B. a l'impression d'être commandé par une force extérieure : "les gens en le regardant arrêtent le cours de sa pensée".
B - Interruptions inexpliquées et allusions sans rapport dans son discours.
C - On note de nombreuses idées délirantes non systématisées.
D - Expression des émotions et mimiques inadaptées.

Quel diagnostic peut-on évoquer ?


A - Psychose hallucinatoire chronique
B - Schizophrénie paranoïde
C - Paranoïa
D - Accès maniaque délirant
E - Syndrome frontal
Bonne(s) réponse(s) : B

A - Début plus tardif entre 30 et 50 ans, plus souvent chez la femme.


C - Dans ce cas, le délire est systématisé et à une cohérence interne.
D - Seul diagnostic avec lequel on peut hésiter mais l'anamnèse ne va pas dans ce sens.

Le bilan spécifique en vue de la prescription médicamenteuse comprendra :


A - Examen neurologique
B - Bilan thyroïdien
C - Ionogramme sanguin et urinaire
D - E.C.G.
E - Test à la dexaméthasone
Bonne(s) réponse(s) : A D

Les neuroleptiques ne sont pas des médicaments anodins ; ils nécessitent de réaliser un examen neurologique préalable et
un ECG (tropisme cardiaque). La surveillance doit être rigoureuse.

Devant ce type de trouble, quel(s) type(s) de médicament(s) est(sont) indiqué(s) ?


A - Benzodiazépines
B - I.M.A.O.
C - Antidépresseurs tricycliques
D - Butyrophénones
E - Sels de lithium
Bonne(s) réponse(s) : D

D - Classe de neuroleptique dont fait partie l'Haldol®.

1248
Exclusivement sur DOC - DZ : www.doc-dz.com NADJI 85
RESIDANAT EN POCHE TOME II
Cas Clinique en QCM

Si vous décidez d'un traitement par neuroleptique d'action prolongée, un ou plusieurs de ces médicaments en
présente une forme, lequel ou lesquels ?
A - Piportil (Pipotiazine)
B - Largactil (Chlorpromazine)
C - Moditen (Fluphénazine)
D - Nozinan (Lévomépromazine)
E - Haldol (Halopéridol)
Bonne(s) réponse(s) : A C E

A - Piportil® L4, 25-100 mg/4 semaines en IM.


C - Moditen® Retard ou Modecate® IM 25-100 mg/3 semaines.
E - Haldol® Decanoas IM.

Dans l'état actuel, parmi ces mesures, laquelle(lesquelles) peu(ven)t être envisagée(s) ?
A - Placement d'office
B - Demande d'allocation d'adulte handicapé (AAH)
C - Mise sous tutelle
D - Admission dans un centre d'aide par le travail (CAT)
E - Expertise médico-légale
Bonne(s) réponse(s) : B C

Ruiné et sans ressource, les réponses B et C semblent indispensables.

Monsieur T.., 48 ans, célibataire, paraplégique depuis un accident de la voie publique, vit dans un centre de rééducation
fonctionnelle depuis plusieurs mois se dit accusé "d'entendre des voix".
Il éprouve depuis quelques mois un sentiment d'étrangeté, l'impression bizarre de ne plus être tout à fait lui-même; "on"
entrave sa pensée par des commentaires, des souvenirs qu'il qualifie "d'incongrus". De temps en temps des voix disent "qu'il
est valide" ou critiquent ses actes. C'est surtout la voix du directeur du centre qui se moque de lui : " une nuit il a fait une
intrusion dans ma chambre pour tenter de me faire sortir". Il ne l'a pas vu mais a affirmé avoir senti des mains qui le
touchaient et lui imposaient des mouvements.
Malgré ses préoccupations, Monsieur T... adopte une conduite quotidienne adaptée, coopère avec sérieux aux soins
médicaux et n'a aucun problème relationnel avec les autres patients.

La symptomatologie de cette observation comporte :


A - Hallucinations psychiques
B - Confabulation
C - Onirisme
D - Hallucinations cénesthésiques
E - Délire imaginatif
Bonne(s) réponse(s) : A D

A, D - Evident.
B - On appelle confabulation des faux-souvenirs venant combler par suppléance une période de lacune mnésique.
C - Faux, car il n'existe pas d'hallucinations visuelles.

Les troubles idéiques présentés par ce malade rentrent dans le cadre syndromique :
A - D'un état maniaque
B - De troubles cognitifs post-traumatiques
C - D'un automatisme mental
D - D'une névrose obsessionnelle
E - D'un délire fantastique
Bonne(s) réponse(s) : C

Comprend plusieurs variétés d'hallucinations psychiques diversement associées. Le sujet peut éprouver le sentiment qu'il a
perdu son autonomie de penser. Il a des pensées qui lui sont étrangères, qui peuvent l'influencer et le déposséder de sa
propre volonté (syndrome d'influence). Il peut y avoir commentaire des actes, écho de la pensée.

Parmi les thèmes délirants exprimés, on retient :


A - L'érotomanie
B - La jalousie
C - La persécution
D - L'indignité
E - L'influence
Bonne(s) réponse(s) : C E

A - Sans aucun rapport.


C - Thème délirant au premier plan.
E - "Affirme avoir senti des mains... qui lui imposaient des mouvements".

1249
Exclusivement sur DOC - DZ : www.doc-dz.com NADJI 85
RESIDANAT EN POCHE TOME II
Cas Clinique en QCM

Parmi les éléments cliniques suivants, le(s)quel(s) est(sont) déterminant(s) dans la démarche diagnostique ?
A - Antécédents de polytraumatisme
B - Age de survenue des troubles
C - Absence de dissociation
D - Bonne adaptation au milieu
E - Sexe masculin
Bonne(s) réponse(s) : B C D

C - Pour certains (auteurs anglo-saxons en particulier), la P.H.C. n'est qu'une forme de schizophrémie tardive. Or la
dissocation est un signe majeur de la schizophrénie.
E - La P.H.C. est, au contraire, plus fréquente chez la femme.

Quel diagnostic retenez-vous ?


A - Schizophrénie tardive
B - Paranoïa passionnelle
C - Psychose hallucinatoire chronique
D - Bouffée délirante aiguë
E - Schizophrénie paranoïde
Bonne(s) réponse(s) : C

Tableau clinique typique de P.H.C.

Les modalités évolutives possibles sous traitement adapté, comportent :


A - Extension en réseau du délire de persécution
B - Enkystement du délire
C - Disparition du délire
D - Evolution démentielle
E - Rechutes périodiques avec troubles de l'humeur
Bonne(s) réponse(s) : B C

Sous traitement, on obtient généralement une extinction assez complète des phénomènes hallucinatoires avecs mise à
distance, "enkystement" voire parfois disparition du délire.

Quel traitement vous semble le plus adapté ?


A - Valium : 30 à 60 mg/j
B - Piportil : 20 à 30 mg/j
C - Carbonate de lithium
D - Sismothérapie
E - Laroxyl 50 à 100 mg/j
Bonne(s) réponse(s) : B

Le traitement adapté est bien sûr un traitement neuroleptique, tel le Piportil.

M.P..., 20 ans, après de brillantes études secondaires, échoue à trois reprises successives au baccalauréat ; ses parents
l'amènent en consultation parce qu'ils le trouvent inquiétant : il ne leur parle pratiquement p!us, prend des crises de fou rire qui
contrastent avec son isolement et son allure renfermée habituelle, s'habille de façon excentrique, quitte parfois le domicile
familial sans explications ou laisse échapper des propos ou des grimaces incompréhensibles.

Quel(s) symptôme(s) peut-on relever dans cette observation ?


A - Mutisme
B - Discordance
C - Autisme
D - Catalepsie
E - Hallucinations psychosensorielles
Bonne(s) réponse(s) : A B C

A - "Il ne leur parle pratiquement plus".


B - Crise de fou rire sans motif apparent.
C - Allure renfermée habituelle et isolement.

1250
Exclusivement sur DOC - DZ : www.doc-dz.com NADJI 85
RESIDANAT EN POCHE TOME II
Cas Clinique en QCM

Quel diagnostic doit faire évoquer ce tableau clinique ?


A - Bouffée délirante aiguë polymorphe
B - Hébéphrénie
C - Schizophrénie paranoïde
D - Toxicomanie
E - Déséquilibre psychique
Bonne(s) réponse(s) : B

Apparition précoce entre 15 et 20 ans de façon insidieuse avec une symptômatologie dominée par la dissociation psychique,
l'autisme et la pauvreté des manifestations délirantes.

Quel(s) examen(s) paraclinique(s) a(ont) un intérêt pour le diagnostic ?


A - Test de Wechsler Bellevue
B - Test de Rorschach
C - Scanner X
D - E.E.G
E - Examen ophtalmologique
Bonne(s) réponse(s) : B

A - Est un test d'efficience intellectuelle qui n'a pas un intérêt diagnostique mais qui peut permettre d'apprécier l'éventuelle
"détérioration intellectuelle". Celle-ci est probablement en rapport avec un déficit fonctionnel témoignant d'un défaut d'attention
et d'un manque de motivation.

Parmi les éléments du projet thérapeutique, l'on doit envisager :


A - Une chimiothérapie neuroleptique
B - Une forme adaptée de psychothérapie
C - Une forme adaptée d'ergothérapie
D - Une forme adaptée de sociothérapie
E - Une sismothérapie
Bonne(s) réponse(s) :

Sans commentaire.

Parmi les médicaments susceptibles d'être indiqués, l'on retiendra (en première intention) :
A - Piportil 10 mg - 1 cp matin et soir
B - Nozinan 25 mg - 2 cp (le soir)
C - Catapressan 3 cp/jour
D - Anafranil 75 mg 1 cp/jour
E - Valium 10. 3 cp/jour
Bonne(s) réponse(s) : A B

A - Pour son effet désinhibiteur (ou antidéficitaire).


B - Pour lutter contre l'angoisse psychotique qui risque d'être réactivé par A.

Parmi les mesures complémentaires immédiates, on discutera avant tout :


A - L'indication d'une cure psychanalytique type
B - D'une psychothérapie familiale systémique
C - D'une forme appropriée de collaboration avec la famille
D - Un placement en centre d'aide par le travail
E - L'ouverture d'un dossier en vue de l'obtention d'une allocation d'handicapé adulte
Bonne(s) réponse(s) : C

C - Est primordiale dans le devenir évolutif des schizophrénies.

1251
Exclusivement sur DOC - DZ : www.doc-dz.com NADJI 85
RESIDANAT EN POCHE TOME II
Cas Clinique en QCM
Un homme de 32 ans est arrêté par la police après une rixe dans un café. Le maintien de la garde à vue s'avère impossible du
fait de l'agitation extrême de ce sujet que l'on n'arrive pas à maîtriser. Il blesse plusieurs agents, détériore une bonne partie du
mobilier du Commissariat avant d'arriver au service des urgences. Ligoté sur une civière, il vocifère, hurle des propos
incohérents, semble dialoguer avec des personnages imaginaires. Tout dialogue est impossible, encore moins un examen
somatique qui se limite au constat d'une haleine chargée d'alcool. Cependant dans les minutes qui suivent, sans aucun
traitement, ce sujet sombre rapidement dans le coma : il ne réagit plus au pincement, ronfle bruyamment mais l'examen
neurologique ne montre pas d'anomalie particulière en dehors d'une hypotonie généralisée. Le lendemain matin, après un
réveil laborieux, il déclare ne se souvenir de rien, et reste perplexe quand on lui fait le récit de ses exploits.

Cette observation évoque un diagnostic précis. Indiquez lequel parmi les propositions suivantes :
A - Crise de manie
B - Syndrome de Korsakoff
C - Ivresse pathologique
D - Troubles du caractère chez un alcoolique chronique
E - Ivresse banale chez un psychopathe
Bonne(s) réponse(s) : C

Crise d'agitation excito-motrice associée à un état confusionnel hallucinatoire et délirant, se terminant par un sommeil
comateux avec amnésie.

Parmi les symptômes suivants, trois caractérisent particulièrement cet épisode pathologique :
A - Etat confusionnel
B - Phobie d'impulsion
C - Agitation clastique
D - Automatisme mental
E - Amnésie lacunaire
Bonne(s) réponse(s) : A C E

Voir question précédente.

Les diagnostics susceptibles d'être masqués par l'intoxication alcoolique comportent :


A - Crise d'hystérie
B - Hématome intracrânien post-traumatique
C - Coma acidocétosique
D - Poly-intoxication associée à l'alcool
E - Personnalité pathologique sous-jacente
Bonne(s) réponse(s) : B D E

B - Toujours y penser devant des troubles du comportement chez un alcoolique chronique (hématome sous-dural). De même,
évoquer l'hypoglycémie fréquente chez l'alcoolique dénutri.

Devant un tel malade, à l'admission, il est urgent d'organiser :


A - L'admission en milieu psychiatrique
B - Le retour au commissariat pour garde à vue après administration d'un traitement tranquillisant
C - La mise en placement d'office immédiate
D - La surveillance régulière jusqu'au réveil des constantes physiologiques (tension, température, pouls)
E - Retour à domicile avec traitement neuroleptique ambulatoire
Bonne(s) réponse(s) : A D

Le risque de coma existe et nécessite alors une réanimation appropriée d'où A et D.

Les risques majeurs liés aux troubles présentés par ce malade comportent :
A - Risque d'un geste homicide au cours de la crise excito-motrice
B - Risque de se blesser soi-même au cours de cette crise
C - Risque de faire des dépenses inconsidérées
D - Risque d'obstruction bronchique après vomissement au cours du coma
E - Risque d'un accident en l'absence de traitement
Bonne(s) réponse(s) : A B D E

C - Se voit dans les accès maniaques.

1252
Exclusivement sur DOC - DZ : www.doc-dz.com NADJI 85
RESIDANAT EN POCHE TOME II
Cas Clinique en QCM
Mme C.. 38 ans, consulte en urgence car elle a une impression étrange, a peur de perdre la tête et de ne plus être elle-même.
Depuis quelques jours elle se sentait tendue, ayant peur du moindre bruit, s'inquiétant fortement du moindre retard de ses
enfants, redoutant diverses catastrophes pour sa famille; elle était aussi oppressée, sujette aux sueurs.
Dans ses antécédents on note depuis une dizaine d'années la peur d'avoir un malaise dans les grands magasins et au volant
de sa voiture. Elle ne pouvait plus sortir sans être accompagnée de son mari, ni se promener seule, si elle n'avait pas sur elle
une liste de médecins de garde.

Quel est le diagnostic des troubles actuels et critiques ?


A - Idées obsédantes
B - Hystérie
C - Agoraphobie
D - Paroxysme anxieux
E - Hypocondrie
Bonne(s) réponse(s) : D

C - Trouble existant chez la patiente depuis une dizaine d'années. Bien lire l'énoncé de la question.

La sémiologie comporte :
A - Anxiété diffuse aiguë
B - Idées délirantes
C - Angoisse somatiques
D - Angoisse de dépersonnalisation
E - Délire de persécution
Bonne(s) réponse(s) : A C D

A - Evidente.
C - "Sentiment d'oppression, sujette aux sueurs".
D - "Impression étrange, peur de perdre la tête et de ne plus être elle-même".

Quel est le diagnostic des troubles antérieurs ?


A - Hypocondrie
B - Obsession
C - Hystérie
D - Névrose d'angoisse
E - Agoraphobie
Bonne(s) réponse(s) : E

E - Peur de sortir dans les lieux publics, comme les rues, magasins et moyens de transport. On note des idées de
catastrophes, des conduites d'évitement et la présence d'objet ou de personne ayant un rôle contra-phobique.

Les troubles antérieurs auraient pu motiver la prescription de :


A - Neuroleptique
B - Tranquillisant
C - Antidépresseur sédatif
D - Psychothérapie de relaxation
E - Sismothérapie
Bonne(s) réponse(s) : B D

C - Les antidépresseurs, indiqués dans l'agraphobie avec attaque de panique, sont les dérivés imipraminiques
(antidépresseurs "stimulants").

1253
Exclusivement sur DOC - DZ : www.doc-dz.com NADJI 85
RESIDANAT EN POCHE TOME II
Cas Clinique en QCM
Monsieur D., 40 ans, sans antécédent pathologique, est conduit aux urgences par son épouse pour un état d'agitation
d'installation brutale. Son discours est intarissable, émaillé de jeux de mots, les idées s'enchaînant sans lien logique.
D'emblée familier avec l'examinateur, il répond de façon ironique ou agressive aux questions. Il se prétend un inventeur de
génie, dit avoir trouvé le "mouvement perpétuel", se croit promis à la gloire. Il nie tout caractère pathologique à son
comportement. Son épouse nous apprend qu'il ne dort plus depuis trois nuits, consomme de l'alcool de façon un peu
excessive et inhabituelle, lui fait des avances sexuelles provoquantes et obscènes ce qui contraste avec son caractère
habituel plutôt réservé. Il n'a pas pris de substances toxiques. Dans ses antécédents, on retrouve la notion d'un état dépressif
à l'âge de 30 ans, traité en ambulatoire. Le bilan biologique à l'entrée est normal (alcoolémie en moyenne à 0,40 g/l).

Dans cette observation on retrouve :


A - Fuite des idées
B - Discordance
C - Idées délirantes mégalomaniaques
D - Logorrhée
E - Troubles du cours de la pensée
Bonne(s) réponse(s) : A C D

Ces symptômes s'intègrent dans l'accélération considérable des processus intellectuels.


B et E - Sont des syndromes de schizophrénie.

Quel diagnostic retenez-vous ?


A - Agitation catatonique
B - Manie délirante
C - Agitation psychopathique
D - Confusion mentale
E - Ivresse pathologique
Bonne(s) réponse(s) : B

Tableau ne posant pas de problème de réponse.

Un tel tableau clinique pourrait aussi faire évoquer une maladie organique. Parmi les affections suivantes
laquelle pourrait entraîner un tel tableau ?
A - Maladie d'Alzheimer
B - Tumeur frontale
C - Encéphalopathie de Gayet-Wernicke
D - Hématome sous dural chronique temporo-pariétal
E - Hydrocéphalie à pression normale
Bonne(s) réponse(s) : B

Les troubles psychiques sont particulièrement fréquents au cours des tumeurs frontales, souvent au premier plan surtout
lorsque la localisation tumorale est antérieure. Ils peuvent réaliser un tableau d'allure maniaque comme la moria.

Parmi les traitements chimiothérapiques suivants, le ou lesquels peu(ven)t être prescrit(s) d'emblée avec utilité ?
A - Benzodiazépines
B - Phénothiazines
C - Neuroleptiques de type butyrophénones
D - Barbituriques
E - Disulfiram
Bonne(s) réponse(s) : B C

Sans commentaire.

Ce patient refuse l'hospitalisation proposée, malgré l'insistance de son épouse et du médecin. Quelle attitude
préconisez-vous ?
A - Prise en charge ambulatoire avec chimiothérapie
B - Surveillance en hospitalisation libre
C - Placement d'office
D - Signalement au service de secteur
E - Placement volontaire
Bonne(s) réponse(s) : E

Le patient présente une agitation nécessitant une hospitalisation immédiate à laquelle adhère son épouse. La solution du
placement volontaire reste la plus logique.

1254
Exclusivement sur DOC - DZ : www.doc-dz.com NADJI 85
RESIDANAT EN POCHE TOME II
Cas Clinique en QCM
Mr F..., 48 ans, consulte le service de psychiatrie, amené par sa famille. Il a le teint bouffi, les cheveux clairsemés. Depuis
quelques jours, il se trompe de trajet en allant à son magasin. Il fait des erreurs sur les dates. Il est très souvent réveillé la nuit
par des frayeurs ; il se lève et crie, demandant à sa femme d'appeler la police pour chasser les loups qui rôdent dans le jardin.
Sa femme signale qu'il a beaucoup de sueurs la nuit avec des tremblements rapides des doigts. L'examen note une
hépatomégalie, avec une hyporéflexie archilléenne. l'absence de traumatisme et de syndrome infectieux. Il n'y a pas
d'antécédents psychiatriques.

Quel est le diagnostic psychiatrique ?


A - Bouffée délirante aiguë
B - Délire paranoïaque
C - Etat confuso-onirique
D.- Mélancolie délirante
E - Etat maniaque
Bonne(s) réponse(s) : C

Présentation du sujet, désorientation temporo-spatiale, délire onirique avec hallucinations visuelles de nature zoopsique et
adhésion du sujet, existence de symptômes neurovégétatifs sont en faveur du diagnostic.

La sémiologie de ce malade comporte :


A - Désorientation spatiale
B - Délire de persécution
C - Hallucinations visuelles
D - Agitation anxieuse
E - Automatisme mental
Bonne(s) réponse(s) : A C D

A - "Depuis quelques jours, il se trompe de trajet en allant à son magasin".


C - "... chasser les loups qui rôdent dans le jardin".
D - Evident.

Quelle est la cause probable des troubles ?


A - Hypothyroïdie
B - Traumatisme affectif
C - Sevrage alcoolique
D - Intoxication aux opiacés
E - Brucellose
Bonne(s) réponse(s) : C

Tableau de délirium tremens. Urgence médicale.

Le traitement comporte :
A - Maintien à domicile
B - Halopéridol (Haldol)
C - Benzodiazépine
D - Alcool intraveineux
E - Réhydratation
Bonne(s) réponse(s) : B C E

A - Nécessité d'une hospitalisation immédiate.


B, C - Soit un neuroleptique plutôt sédatif, soit une benzodiazépine, soit un carbamate.
E - Réhydratation par voie buccale (4-6 litres/24 heures) ou intra-veineuse. Elément majeur du traitement.

Quelle est la mesure éventuellement nécessaire ?


A - Placement d'office
B - Tutelle
C - Curatelle
D - Sauvegarde de justice
E - Invalidité
Bonne(s) réponse(s) :

QUESTION SUPPRIMEE

1255
Exclusivement sur DOC - DZ : www.doc-dz.com NADJI 85
RESIDANAT EN POCHE TOME II
Cas Clinique en QCM
Un généraliste adresse l'une de ses patientes dans un service de long séjour avec la lettre suivante : "Je vous adresse Mme
ROB.. Lucienne âgée de 81 ans pour atteinte de l'état général, avec adynamie et lombalgies; elle semble actuellement très
dépressive, elle pleure souvent. Elle est en arythmie et a un glaucome".
Son traitement actuel associe Risordan® et Aldactazine®. Lorsque l'interne du service qui l'accueille lui demande ce quelle
souhaite, elle déclare : "Donnez moi une pilule pour rejoindre mon mari et mon fils, j'ai trop mal au dos, je suis une bouche
inutile pour la société." On apprend qu'ils sont décédés, il y a quelques années.
Après un entretien d'une demi-heure, une relation médecin-malade de bonne qualité s'est établie. L'examen somatique est
normal hormis l'arythmie. Les examens complémentaires faits en ville sont normaux.

Quel(s) est(sont) le(s) symptôme(s) présent(s) dans cette observation ?


A - Des idées de suicide
B - Une auto-accusation
C - Un sentiment de dévalorisation
D - Des plaintes hypochondriaques
E - Un taedium vitae
Bonne(s) réponse(s) : A C E

A - "Donnez-moi une pilule pour rejoindre mon mari et mon fils..."


C - "Je suis une bouche inutile pour la société".
D- Chez cette patiente, il s'agit de plaintes somatiques ayant une équivalence dépressive et non des plaintes
hypocondriaques.
E - Est synonyme d'ennui de vivre.

Quel(s) est(sont) le(s) risque(s) évolutif(s) possible(s) en l'absence de traitement ?


A - Un état grabataire
B - Des automutilations
C - Un accès maniaque
D - Un suicide
E - Une attaque de panique
Bonne(s) réponse(s) : A D

A - Est l'aboutissement de l'altération de l'état général et d'une propension au laisser-aller (équivalent suicidaire).

Quel est le psychotrope que vous proposez en monothérapie ?


A - Amitriptyline (Laroxyl®)
B - Chlorpromazine (Largactil®)
C - Viloxazine (Vivalan®)
D - Thioridazine (Melleril®)
E - Clomipramine (Anafranil®)
Bonne(s) réponse(s) : C

A E - Antidépresseurs tricycliques contre-indiqués en raison de l'existence d'un glaucome et de l'arythmie.


B D - Neuroleptiques sans intérêt dans l'observation.
C - Antidépresseur non tricyclique, plutôt de type stimulant, dénué de cardrotoxicité et pouvant être prescrit en cas de
glaucome. Effets secondaires digestifs.

Quelle est la durée de traitement au bout de laquelle vous pourrez juger de l'efficacité ou de l'inefficacité du
produit à priori adéquat ?
A - 24 à 72 heures
B - 5 à 8 jours
C - 2 à 3 semaines
D - 4 à 6 semaines
E - 2 à 6 mois
Bonne(s) réponse(s) : C

Connaissance.

1256
Exclusivement sur DOC - DZ : www.doc-dz.com NADJI 85
RESIDANAT EN POCHE TOME II
Cas Clinique en QCM
Mr J . 25 ans, est amené aux urgences, à la suite d'une défenestration du premier étage ; il présente une fracture du
calcanéum droit. Les semaines précédentes, il avait fait plusieurs fugues et présentait une hyperactivité diurne et nocturne
contrastant avec sa personnalité habituellement intravertie. Au cours de l'entretien qu'il n'accepte qu'avec réticence, il proteste
contre son hospitalisation, il se porterait bien mais finit par confier qu'il a sauté par la fenêtre pour échapper aux ondes qu'on
lui envoie et qui influencent son cerveau ; ainsi, il ne se sent plus lui même : on lui a changé sa personnalité. Il entend des
voix qui l'insultent. Dans ses antécédents, il a été hospitalisé trois fois en psychiatrie, mais il est toujours refusé des soins
ambulatoires

Ce résumé d'observation permet d'identifier :


A - Phobie d'impulsion
B - Dépersonnalisation
C - Discordance
D - Sentiment d'influence
E - Désorientation spatiale
Bonne(s) réponse(s) : B C D

B - "Il ne se sent plus lui-même : on lui a changé sa personnalité".


C - Le patient vient de réaliser une T.S. et dit bien se porter...
D - Ondes qui influencent son cerveau.

Quel(s) mécanisme(s) peut-on relever à l'origine de ces symptômes ?


A - Imagination
B - Intuition
C - Interprétation
D - Hallucination
E - Fabulation
Bonne(s) réponse(s) : D

A - Mécanisme prépondérant dans la paraphrénie.


B - Mécanisme fréquemment rencontré dans les schizophrénies, qui n'apparaît pas dans l'observation.
C - Plutôt retrouvé dans les délires chroniques systématissés parnoïaques.

Quel diagnostic envisager ?


A - Délire paranoïaque
B - Schizophrénie paranoïde
C - Bouffée délirante
D - Accès confusionnel
E - Psychopathie
Bonne(s) réponse(s) : B

Sans commentaire.

Quel(s) examen(s) paraclinique(s) peu(ven)t compléter ce diagnostic ?


A - Ponction lombaire
B - Scanner cérébral
C - Test de Wechsler-Bellevue
D - Test de Rorschach
E - Test de Benton
Bonne(s) réponse(s) : C D E

C - Epreuve psychométrique testant efficience intellectuelle, intéressant sur un angle qualitatif.


D - Test projectif des taches d'encre, dont certains signes sont considérés comme pathognomoniques.
E - Epreuve perceptivo-motice à analyser sur un plan qualitatif. On peut retrouver rationalisme et géométrisme morbide et un
temps d'exécution anormalement élévé.

Quelle décision proposez-vous ?


A - Maintien en service d'orthopédie avec traitement psychiatrique
B - Transfert en service de psychiatrie, en placement libre
C - Transfert en service de psychiatrie, en placement volontaire
D - Retour au domicile avec traitement ambulatoire
E - Transfert en service de psychiatrie, en placement d'office
Bonne(s) réponse(s) : C

Du fait des nombreuses fugues et de son refus d'hospitalisation, un transfert dans un service de psychiatrie en P.V. s'avère
souhaitable.

1257
Exclusivement sur DOC - DZ : www.doc-dz.com NADJI 85
RESIDANAT EN POCHE TOME II
Cas Clinique en QCM

Quel traitement proposez-vous ?


A - Antidépresseurs tricycliques
B - Neuroleptiques incisifs
C - Neuroleptiques sédatifs
D - Benzodiazépines
E - Bêta-bloqueurs
Bonne(s) réponse(s) : B C

La mise en route d'un traitement neuroleptique apparaît indispensable afin d'agir sur l'excitation psycho-motrice et l'activité
délirante. Par exemple, prescription d'une association d'Haldol® et de Nozinan®.

Madame C, 52 ans, veuve depuis 10 ans, sans profession, vivant seule, est amenée en consultation par sa soeur, inquiète de
son isolement progressif depuis 1 an. Dernièrement, elle ne sort plus que pour quelques courses, ne reçoit plus ses amis, et,
interrogée par sa soeur, elle à répondu qu'elle n'en pouvait plus qu'elle pensait se suicider "si ça ne s'arrêtait pas". Malgré une
certaine réticence, elle confie qu'elle est victime de personnes malfaisantes qui depuis quelques années tentent de
l'empoisonner au moyen de gaz toxiques, envoyés par les conduites d'aération. Elle en sent l'odeur nauséabonde et leur
attribue le décès récent de son chien. Elle entend également des voix qui l'insultent et la traitent de "folle" et de vieille toquée.
Elle à l'impression que l'on devine et répète ses pensées. Plus récemment, ces malfaisants ont entrepris de la torturer à
distance au moyen d'un "scanner radioactif" qui lui provoque la nuit lorsqu'elle est couchée des picotements désagréables
dans les jambes et des vibrations au niveau du vagin. Madame C. ne connait pas l'identité de ses tortionnaires mais à
supposé qu'ils pouvaient faire partie des services secrets, ou bien de la franc-maçonnerie, sans qu'elle s'explique les raisons
qui l'auraient fait prendre pour victime.

Quel est le diagnostic le plus probable ?


A - Délire paranoïaque
B - Mélancolie d'involution
C - Hystérie
D - Psychose hallucinatoire chronique
E - Etat démentiel débutant
Bonne(s) réponse(s) : D

Tableau classique : âge de début, sexe féminin, automatisme mental, hallucinations auditives acoustico-verbales, olfactives,
coenesthésiques, délire de type persécutif vague, élaboration délirante pauvre sur un mode dépressif.

Parmi les manifestations suivantes laquelle (lesquelles) serai(en)t également compatible(s) avec ce tableau ?
A - Hallucinations visuelles
B - Désorientation temporo-spatiale
C - Automatisme mental
D - Tachypsychie
E - Troubles du sommeil
Bonne(s) réponse(s) : A C E

A - Les hallucinations visuelles, olfactives et gustatives sont plus rares que les hallucinations psychiques, auditives et
coenesthésiques.
C - Existe déjà dans l'observation.

Devant un tel tableau, on prescrit en première intention :


A - Des antidépresseurs tricycliques type Anafranil
B - Des anxiolytiques type benzodiazépines
C - Des neuroleptiques
D - Un IMAO
E - Des barbituriques
Bonne(s) réponse(s) : C

Réponse logique, en première intention compte-tenu de la symptomatologie et de la bonne réponse thérapeutique habituelle.

L'évolution sous traitement la plus vraisemblable sera :


A - La critique rapide et complète des phénomènes délirants
B - Un virage de l'humeur après trois semaines
C - Un enkystement du délire avec persistance d'un certain isolement
D - Une démence sénile
E - Des passages à l'acte médico-légaux à l'encontre des persécuteurs
Bonne(s) réponse(s) : C

On observe souvent une extinction assez complète des phénomènes hallucinatoires et une mise à distance du délire sans
véritable disparition.

1258
Exclusivement sur DOC - DZ : www.doc-dz.com NADJI 85
RESIDANAT EN POCHE TOME II
Cas Clinique en QCM

Sous traitement approprié, on pourra observer des effets secondaires à type(s) de :


A - Akinésie
B - Tremblement
C - Hypotension orthostatique
D - Diarrhée
E - Recrudescence de l' insomnie
Bonne(s) réponse(s) : A B C

Le traitement approprié correspond au traitement neuroleptique dont les effets secondaires sont d'ordre neuroleptique
(dyskinésies aiguës ou tardives, syndrome extra-pyramidal), neuro-végétatif (cardio-vasculaire, régulation thermique),
allergiques et toxiques, digestifs, métaboliques et endocriniens et de nature psychique.

Une femme de 72 ans, est hospitalisée depuis 15 jours dans un service de convalescence au décours d'un accident de voiture
survenu 3 mois plus tôt. Son mari à été tué sur le coup ; elle à présenté une fracture bimalléolaire de la jambe gauche.
Son ralentissement complique beaucoup la rééducation. De plus, elle pleure beaucoup et se reproche de ne plus être bonne à
rien. Elle se plaint de cauchemars, se réveille et appelle son mari qu'elle croit, alors, encore vivant. La patiente est traitée
depuis 5 ans pour une maladie de Parkinson par Sinemet®. Son traitement actuel comprend, en outre 1 mg de Rohypnol®; et
elle dit trembler davantage.
Après la consultation, un traitement par Tofranil® à faible dose (20 mg/j) est instauré. Au bout de 3 jours, la patiente présente
une agitation nocturne avec propos incohérents. Dans la journée, elle est désorientée, opposante, elle refuse de s'alimenter et
se sent persécutée par l'entourage. Elle présente des fausses reconnaissances. Il n'existe pas de signes neurologiques de
localisation.

Quels sont parmi les symptômes suivants, celui(ceux) présent(s) dans cette observation avant la prescription du
Tofranil® ?
A - Rationalisme morbide
B - Inhibition psychomotrice
C - Fuite des idées
D - Auto-accusation
E - Mythomanie
Bonne(s) réponse(s) : B D

A - Pensée soumise au raisonnement qui se déroule sans objet, indépendant et indifférent à la réalité, et au concret.
Caractérise la pensée de nombreuses schizophrènes.
C - Rencontrée dans les états maniaques.
D - Chez les personnalités hystériques, les déséquilibrés ou les pervers.

Devant le tableau clinique ayant suscité la consultation du début, quel(s) est(sont) le(s) diagnostic(s) à évoquer ?
A - Aggravation brutale de la maladie de parkinson
B - Etat crépusculaire hystérique
C - Syndrome dépressif
D - Maladie de Pick
E - Névrose obsessionnelle
Bonne(s) réponse(s) : A C

A, C - D'autant que la survenue d'un état dépressif au cours d'une maladie de Parkinson n'est pas rare et qu'une aggravation
de la symptomatologie neurologique par des évènements de vie stressants est possible.

Devant l'évolution clinique au 3ème jour, après prescription du Tofranil, quel(s) est(sont) le(s) diagnostic(s) à
évoquer ?
A - Levée d'inhibition sous Tofranil®
B - Mélancolie délirante
C - Virage maniaque
D - Syndrome de Korsakoff post-traumatique
E - Syndrome confusionnel aigu
Bonne(s) réponse(s) : E

La confusion mentale est plus fréquente chez les personnes agées et est directement reliée à l'action anticholinergique des
tricycliques.

1259
Exclusivement sur DOC - DZ : www.doc-dz.com NADJI 85
RESIDANAT EN POCHE TOME II
Cas Clinique en QCM

Certaine(s) mesure(s) thérapeutique(s) s'impose(nt)-elle(s) dans l'immédiat ?


A - Arrêt du Tofranil®
B - Arrêt du Rohypnol®
C - Arrêt du Sinemet®
D - Prescription d'un anti-comitial
E - Prescription d'Artane®
Bonne(s) réponse(s) : A

A - Le syndrome confusionnel apparaît consécutif à l'introduction 3 jours plus tôt du Tofranil®. Il est donc nécessaire de
l'interrompre. Ne pas arrêter brutalement le Rohypnol® ni le Simenet®.
E - L'artane® aggraverait la symptomatologie confusionnelle.

Voici six mois, à table, Manuel P..., 46 ans, à éprouvé le sentiment que ses enfants ne lui ressemblaient pas et donc n'étaient
pas les siens. Depuis il n'a de cesse de démontrer que son épouse est adultère : taches sur les draps, fleurs à la fenêtre,
regards appuyés en direction d'autres hommes. Celle-ci dément, se plaint d'être tyrannisée par les investigations de son mari.
Les enfants confirment le comportement de leur père qui oscille entre la violence et les menaces, ou l'isolement et la
prostration ; il a caché un fusil dans sa chambre et pense connaître son rival dont il ne révèle pas le nom; sa femme vient vous
exprimer son inquiétude au centre médicopsychologique du secteur (CMP).

Parmi ces propositions la(lesquelles) caractérise(nt) ce délire ?


A - Un syndrome d'influence
B - Un syndrome de discordance
C - Un délire passionnel
D - Un délire systématisé
E - Un délire mélancolique
Bonne(s) réponse(s) : C D

A,B - sont retrouvés dans les schizophrénies.


E - Thèmes d'indignité, incurabilité, damnation, ruine..., non retrouvés ici.

Les mécanismes générateurs du délire incluent dans ce cas :


A - L'intuition
B - L'imagination
C - L'interprétation
D - L'illusion
E - L'hallucination
Bonne(s) réponse(s) : A C

Voir question suivante.

Le diagnostic à porter est celui de :


A - Délire de relation des sensitifs
B - Délire paranoïaque
C - Psychose hallucinatoire chronique
D - Schizophrénie paranoïde
E - Psychose dysthymique
Bonne(s) réponse(s) : B

Il s'agit d'un délire paranoïaque passionel de jalousie. Il est typiquement en secteur et peut aboutir à des actes médico-légaux.
Outre l'interprétation et l'intuition, on peut retrouver des illusions.

Parmi ces mesures, laquelle est formellement à exclure ?


A - Convoquer le malade au CMP par lettre
B - Se rendre à son domicile pour l'examiner
C - Remettre à sa femme un certificat d'internement
D - Prévenir l'assistante sociale
E - Prévenir le commissariat de police
Bonne(s) réponse(s) : C

Du fait de la dangerosité des paranoïaques et du risque de passage à l'acte agressif envers sa femme.

1260
Exclusivement sur DOC - DZ : www.doc-dz.com NADJI 85
RESIDANAT EN POCHE TOME II
Cas Clinique en QCM

Si le malade acceptait de se traiter, l'on indiquerait :


A - Un antidépresseur sédatif
B - Un neuroleptique incisif
C - Un neuroleptique sédatif
D - Un tranquillisant
E - Un thymorégulateur
Bonne(s) réponse(s) : B C

Association d'un neuroleptique sédatif à un neuroleptique incisif, sans oublier l'importance de la relation psychothérapique.

Pierre, 40 ans, est persuadé que son père le manipule. Selon lui, il a imaginé des combinaisons machiavéliques pour qu'il se
marie; il le fait surveiller par un inspecteur privé, pour savoir s'il rencontre d'autres femmes, En allant au travail, il pense avoir
rencontré et reconnu ce policier à son allure. Depuis quelques jours, P... refuse de sortir de sa chambre. La nuit il injurie son
père à travers la cloison. Il menace sa mère à haute voix car elle en est devenue la complice. Les parents, épuisés par le
comportement de P..., demandent un internement à leur médecin. A l'arrivée au service de psychiatrie, il refuse de parler car
tout le monde connait son histoire" ses parents doivent être traités". Il n'a présenté aucun trouble psychique jusqu'à ces
derniers mois.

Quel est le type de trouble relationnel observé à l'arrivée du patient en psychiatrie ?


A - Mutisme
B - Négativisme
C - Réticence
D - Aphasie
E - Catalepsie
Bonne(s) réponse(s) : C

Dans le contexte décrit, le refus de parler est un signe de réticence. Il ne s'agit pas de mutisme car le sujet explique que tout
le monde connaît son histoire et que "ses parents doivent être traités".

Quelle est la nature du délire ?


A - Confabulatoire
B - Imaginatif
C - Systématisé
D - Hallucinatoire
E - Oniroïde
Bonne(s) réponse(s) : C

A - Se voit dans la paraphrénie.

Quel est le diagnostic à retenir ?


A - Mélancolie délirante
B - Schizophrénie paranoïde
C - Délire paranoïaque
D - Psychose hallucinatoire chronique
E - Hystérie
Bonne(s) réponse(s) : C

Sans commentaire.

Les modalités nécessaires à la demande de placement volontaire comportent :


A - Demande écrite des parents
B - Accord du Directeur de l'hôpital psychiatrique
C - Arrêté préfectoral
D - Certificat du médecin traitant
E - Nécessité d'une expertise médico-légale
Bonne(s) réponse(s) : A D

Il s'agit des deux documents nécessaires au placement volontaire.

1261
Exclusivement sur DOC - DZ : www.doc-dz.com NADJI 85
RESIDANAT EN POCHE TOME II
Cas Clinique en QCM

Quel(s) traitement(s) médicamenteux peut-on utiliser dans l'immédiat ?


A - Droleptan®
B - Haldol®
C - Anafranil®
D - Valium®
E - Largactil®
Bonne(s) réponse(s) : A B E

On peut discuter d'utiliser le Droleptan® dont l'usage est réservé aux états d'agitation aigus et les effets secondaires intenses.

Quels sont les éléments du traitement au long cours ?


A - Traitement antidépresseur seul
B - Cure psychanalytique
C - Neuroleptique à action prolongée
D - Aide thérapeutique de la famille
E - Consultation thérapeutique au dispensaire du secteur
Bonne(s) réponse(s) : C D E

Un neuroleptique retard associé à un soutien psychologique du patient et de la famille dans le cardre d'une consultation
sectorisée semble être la meilleure solution.

Les difficultés de Mr.E., 43 ans, ne sont pas, selon lui, d'ordre médical. Leur régression, quand il prend son traitement, est
pure coïncidence, car il est en fait victime du diable qui le harcèle de questions religieuses spécieuses ou se joue de lui ; croit-
il faire sa prière, il comprend soudainement qu'il l'a détournée à son profit. Fréquemment isolé dans le noir, il parle comme à
un interlocuteur qu'il lui arrive d'injurier. Il est ainsi malmené en permanence depuis trois ans, à la suite d'une relation sexuelle
au moment des règles de sa femme, ce qu'interdit sa religion musulmane. Peu après, il avait impulsivement tenté de se
pendre. Le dialogue est possible si l'on ne met pas en doute ses propos, ou s'il ne concerne pas la religion. On notera une
hypertension artérielle ancienne, actuellement bien équilibrée par le régime.

Ces idées délirantes :


A - Sont systématisées
B - Sont produites par un mécanisme intuitif
C - Sont centrées sur un thème passionnel
D - Sont développées en réseau
E - Sont associées à un syndrome de discordance
Bonne(s) réponse(s) : A B

A - Le délire possède une logique et cohérence interne.


B - "Il comprend et soudainement..."

Quel diagnostic évoquer ?


A - Psychose hystérique
B - Psychose dysthymique
C - Bouffée délirante
D - Psychose hallucinatoire chronique
E - Mélancolie délirante
Bonne(s) réponse(s) : D

Tableau classique.

Que retenir en faveur d'un trouble thymique ?


A - Idées de culpabilité
B - Réticence
C - Tentative de suicide
D - Refus du traitement
E - Tonalité pessimiste du délire
Bonne(s) réponse(s) : A C E

La participation thymique au délire est souvent constatée dans les psychoses hallucinatoires chroniques.

1262
Exclusivement sur DOC - DZ : www.doc-dz.com NADJI 85
RESIDANAT EN POCHE TOME II
Cas Clinique en QCM

L'indication thérapeutique, doit envisager :


A - Des antidépresseurs tricycliques associés à des neuroleptiques
B - Des neuroleptiques d'action prolongée
C - Des sels de lithium
D - Des neuroleptiques sédatifs seuls
E - Des neuroleptiques incisifs seuls
Bonne(s) réponse(s) : A

On peut envisager d'associer au neuroleptique, un antidépresseur compte tenu de la note thymique.

Parmi ces médicaments, lequel est contre-indiqué, compte-tenu de l'hypertension artérielle ?


A - Phénothiazines
B - Antidépresseurs tricycliques
C - Sels de lithium
D - Benzodiazépines
Bonne(s) réponse(s) : E

Sans commentaire.

Florent vient consulter pour des difficultés scolaires. A 8 ans il suit péniblement un C.E.1 après avoir redoublé un C.P. Il ne lit
pas couramment. La maîtresse signale sa tendance à rester silencieux et à l'écart des autres enfants. Quand il est en
confiance, il parle avec elle et montre qu'il n'est pas coupé de la réalité. Il ne s'intéresse à rien. Sa mère pense qu'il est
handicapé, elle s'étonne à d'autres moments de la pertinence de ses remarques. Son père, lui, le trouve intelligent. Une
grande rivalité existe avec son frère qui travaille très bien à l'école. Au cours de la consultation, après un silence prolongé le
contact peut s'établir et il parle des exigences scolaires de sa mère de son impossibilité à les satisfaire. Son agressivité à son
égard paraît évidente mais non reconnue. Dès le retour de sa mère, il redevient silencieux et fermé. Son développement n'a
posé aucun problème.

Devant ce tableau, quel(s) type(s) d'organisation peut-on évoquer ?


A - Dépression de l'enfant
B - Déficience intellectuelle
C - Psychose
D - Organisation névrotique
E - Aucune organisation pathologique
Bonne(s) réponse(s) : A B D E

Peut-on envisager une psychose ?

Parmi les éléments suivants, quel(s) est (sont) celui (ceux) qui fait (font) penser à une dépression ?
A - Mauvais résultats scolaires
B - Absence de contact avec les autres
C - Désintérêt pour ce qui l'entoure
D - Un développement normal
E - Relation avec la réalité
Bonne(s) réponse(s) : A B C

A B C - Diagnostic positif de dépression.


E - Signe contre le diagnostic de psychose.

Parmi les symptômes suivants, quel est celui qui se retrouve dans cette observation ?
A - Phobie
B - Dyslexie
C - Inhibition psychique
D - Rupture avec la réalité
E - Mutisme
Bonne(s) réponse(s) : B C

Sans commentaire.

1263
Exclusivement sur DOC - DZ : www.doc-dz.com NADJI 85
RESIDANAT EN POCHE TOME II
Cas Clinique en QCM

Quelle(s) attitude(s) thérapeutique(s) est (sont) envisageable(s) ?


A - Attendre et voir l'évolution
B - Conseiller une rééducation orthophonique
C - Prévoir un placement en l.M.P. (Institut Médico-Pédagogique)
D - Prévoir un placement en hôpital de jour
E - Envisager une psychothérapie
Bonne(s) réponse(s) : E

Basée sur la relation mère-enfant.

Cette malade de 39 ans est amenée en consultation par son mari pour ses difficultés à sortir de chez elle. Elle s'occupe de
façon normale mais redoute de sortir seule. Si sa fille l'accompagne, elle arrive à aller dans la rue mais en restant sur le
trottoir qui entoure le bloc d'immeubles où elle habite et elle refuse de traverser la rue. Avec sa mère, elle accepte de circuler
dans son quartier. Ce n'est qu'avec son mari qu'elle peut aller en voiture et se rendre plus loin. Quand on lui demande ce qui
l'empêche de sortir, elle dit : "C'est dangereux toutes ces voitures... Je sais que je suis peureuse... ça m'angoisse trop... Je
pourrais avoir un malaise et personne ne me porterait secours"...

Parmi les symptômes suivants, quel est celui ou quels sont ceux que présente(nt) cette malade ?
A - Phobie
B - Obsession-impulsion
C - Obsession phobique
D - Compulsion
E - Comportement d'évitement
Bonne(s) réponse(s) : A E

Agoraphobie : peur d'être seule, d'être dans un endroit où le sujet ne pourrait s'échapper sans difficulté ou être secouru.

Comment désigne-t-on la nécessité, pour elle, de sortir accompagnée ?


A - Comportement de dépendance
B - Moyen contraphobique
C - Mécanisme fusionnel
D - Comportement de manipulation
E - Agoraphobie
Bonne(s) réponse(s) : B

Ou conduite de réassurance.

Parmi les diagnostics suivants, quel est celui ou quels sont ceux qui peuvent être envisagés ici ?
A - Névrose obsessionnelle
B - Déséquilibre psychique
C - Névrose phobique
D - Délire d'interprétation
E - Névrose d'angoisse
Bonne(s) réponse(s) : C

A - Absence de symptomatologie obsessionnelle (crainte en dehors de l'objet).


E - Angoisse flottante avec fixation transitoire.

Parmi les traitements suivants, quels sont ceux qui sont justifiés dans un tel cas ?
A - Isolement du milieu familial
B - Neuroleptiques
C - Psychothérapie d'inspiration psychanalytique
D - Thérapie comportementale
E - Lithium
Bonne(s) réponse(s) : C D

Désensibilisation et immersion.

1264
Exclusivement sur DOC - DZ : www.doc-dz.com NADJI 85
RESIDANAT EN POCHE TOME II
Cas Clinique en QCM
Monsieur N... Marc, âgé de 31 ans, est hospitalisé en psychiatrie après avoir fait des démarches auprès du commissariat de
police dans l'intention de porter plainte "contre X" pour "coups et blessures" ; en effet, il prétend qu'un médecin lui veut du mal
et qu'à cette fin, "il lui a planté des aiguilles dans le crâne", ce qui lui occasionne des maux de tête intenses. Monsieur N... a
une présentation négligée ; son faciès est rigide. Il aura, au cours de l'entretien, de nombreux gestes stéréotypés. Il fait état
avec difficulté d'une persécution de longue date de ce médecin qui cherche à le détruire sans que le motif soit clairement
exprimé. Son persécuteur peut agir à distance sur le corps du malade, provoquant des douleurs multiples et aussi sur son
cerveau afin d'en faire "un robot". Monsieur N... avoue aussi se méfier de son interlocuteur présent qui pourrait participer au
"complot des médecins", destiné à lui nuire, mais les médecins peuvent à la fois "guérir et tuer".

Comment désigner les sensations corporelles éprouvées par le patient ?


A - Illusions
B - Interprétations
C - Pseudo-hallucinations corporelles
D - Hallucinations cénesthésiques
E - Etat de dépersonnalisation
Bonne(s) réponse(s) : D

Hallucinations (fausses perceptions) psychosensorielles intéressant la sensibilité interne.


Pseudohallucinations : représentations vives, spontanées, se rapprochant des hallucinations sans créer comme elles
l'apparence d'une réalité extérieure.

Quel diagnostic vous apparaît le plus vraisemblable ?


A - Psychose délirante aiguë
B - Schizophrénie paranoïde
C - Délire paranoïaque de persécution
D - Délire de relation
E - Schizophrénie hébéphrénique
Bonne(s) réponse(s) : B

- Délire de persécution flou, mécanisme hallucinatoire. Illogisme.


- En faveur de la schizophrénie : la notion d'une certaine chronicité des troubles ("longue date"), les bizarreries du
comportement (gestes stéréotypés), propos diffluents ("guérir et tuer").

Quelle fourchette journalière de posologie de l'halopéridol vous paraît la plus appropriée dans le cas présent ?
A-2à4g
B - 80 mg à 1,2 g
C - 30 à 60 mg
D - 5 à 25 mg
E - 0,5 à 1 mg
Bonne(s) réponse(s) : D

Sans commentaire.

Dans les premières 24 heures du traitement par l'halopéridol, quelle est la complication susceptible de survenir ?
A - Dyskinésie
B - Syndrome akinéto-hypertonique
C - Akathisie
D - Galactorrhée
E - Hypotension orthostatique
Bonne(s) réponse(s) : A

Les dyskinésies aigues apparaissent habituellement dans les premières 24 heures. Les autres signes (B C D) sont
d'apparition plus tardive.
L'halopéridol® donne peu d'hypotension orthostatique.

1265
Exclusivement sur DOC - DZ : www.doc-dz.com NADJI 85
RESIDANAT EN POCHE TOME II
Cas Clinique en QCM
A la suite d'un accident de voiture, un homme de 42 ans, se présente aux urgences, l'examen somatique est normal,
l'alcoolémie est inférieure à la valeur limite mais le patient est volubile, expose des projets grandioses, fait preuve optimisme
surprenant dans ces circonstances : son accident sera une bonne occasion de changer sa voiture. Il interpelle le médecin
familièrement, et refuse toute proposition de traitement : son médecin l'attend, il n'a pas sommeil bien qu'il n'ait pas dormi
depuis 48 heures.
Sa femme avertie arrive pour confirmer que son activité récemment décuplée épuise son entourage et que, si elle a préféré,
au début, ce changement d'état qui a succédé à la morosité entraînée par la mort de son père trois mois avant, le seuil de
tolérance est dépassé.

Quel est le diagnostic le plus probable ?


A - Entrée dans la schizophrénie
B - Mythomanie
C - Accès maniaque
D - Psychose aiguë
E - Délire paranoïaque
Bonne(s) réponse(s) : C

- Reproduction du temps de sommeil.


- Plus grande communcabilité.
- Fuite des idées.
- Hyperactivité, projets grandioses.
- Exaltation euphorique de l'humeur.
- Probable virage de l'humeur.

La sémiologie est marquée par :


A - La dissociation
B - L'exaltation de l'humeur
C - La scrupulosité
D - L'excitation psychique
E - Le négativisme
Bonne(s) réponse(s) : A E

Signes de schizophrénie.

Devant le refus répété du patient de rester à l'hôpital, vous devez :


A - Obtenir son placement à la demande d'un tiers en milieu spécialisé
B - Lui proposer une consultation au dispensaire d'hygiène mentale
C - Lui faire signer une "sortie contre avis médical"
D - Le laisser libre de sa décision, sans autre formalité
E - Faire une déclaration de sauvegarde de justice
Bonne(s) réponse(s) : A

Sans commentaire.

Parmi les mesures suivantes, laquelle ou lesquelles, vous parai(ssen)t souhaitable(s) ?


A - Déclaration à la DASS
B - Sauvegarde de Justice
C - Curatelle
D - Tutelle
E - Désignation d'un mandataire
Bonne(s) réponse(s) : B

Mesure simple, souple et rapidement comprise.


Valable pendant deux mois à partir de la date d'inscription sur le registre du Parquet, renouvelable.
Mesure transitoire, d'urgence, protectrice et conservatrice.

Le traitement médicamenteux indiqué comporte :


A - Avlocardyl®
B - Antidépresseurs
C - Benzodiazipide
D - Neuroleptiques
E - Lithium
Bonne(s) réponse(s) : D E

Sédation de l'accès méningé.

1266
Exclusivement sur DOC - DZ : www.doc-dz.com NADJI 85
RESIDANAT EN POCHE TOME II
Cas Clinique en QCM
Une jeune fille de 17 ans, est amenée en consultation par sa mère sur les conseils du médecin traitant.
Elle est très maigre et a perdu 16 kg en un an. Elle a progressivement réduit son alimentation alléguant des douleurs
d'estomac : quand on la force à manger elle vomit presque régulièrement.
Par ailleurs, le comportement de cette jeune fille s'est progressivement modifié : en famille elle se montre tyrannique; en
particulier elle contrôle de façon étroite l'alimentation des autres membres de la famille et ne fréquente pratiquement plus ses
camarades de classe. Ses résultats restent excellents .
Cette jeune fille ne se plaint de rien, affirmant au contraire qu'à part ses douleurs d'estomac elle ne s'est jamais mieux portée.
L'examen clinique est négatif. Le transit gastro-duodénal ne montre rien d'anormal. A l'interrogatoire on note une aménorrhée
depuis 10 mois.

Quel diagnostic retenez-vous ?


A - Etat dépressif chronique
B - Installation d'une psychose dissociative
C - Anorexie mentale
D - Crise sévère d'adolescence
E - Névrose phobique
Bonne(s) réponse(s) : C

Diagnostic d'anorexie mentale :


- refus de maintenir un poids corporel au dessus d'un poids minimum normal pour l'âge
- peur de prendre du poids
- perturbation de l'estimation de son propre poids
- aménorrhée supérieure à trois cycles.

Parmi les symptômes suivants quel est celui dont l'existence est indispensable au diagnostic ?
A - Les vomissements
B - Les troubles du caractère
C - L'aménorrhée
D - Les douleurs d'estomac
E - L'inhibition psychomotrice
Bonne(s) réponse(s) : C

Sans commentaire.

On peut rencontrer des conduites de restriction alimentaire au cours de :


A - Anorexie mentale
B - Délire de persécution
C - Dépression névrotique
D - Hernie hiatale
E - Névrose obsessionnelle
Bonne(s) réponse(s) : A C D E

La restriction alimentaire est rapportée à une diminution ou une perte d'appétit, à un désir de maigrir ou obéit à des règles
diététiques personnelles. Le refus alimentaire a des motivations pathologiques et souvent délirantes.

A l'origine d'une hypokaliémie lors de l'évolution de la maladie de cette adolescente, on peut trouver :
A - Réduction de l'apport alimentaire
B - Vomissements répétés
C - Utilisation des diurétiques
D - Usage de laxatifs
E - Hyperactivité motrice
Bonne(s) réponse(s) : A B C D

- La carence d'apports isolée donne exceptionnellement une hypokaliémie.


- La prise de laxatifs et de diurétiques est à rechercher.

Le projet thérapeutique proposé à cette malade comporte :


A - Hospitalisation avec séparation de la famille
B - Hormonothérapie
C - Psychothérapie individuelle
D - Réalimentation par sonde gastrique
E - Traitement antidépresseur
Bonne(s) réponse(s) : A C D

L'hospitalisation avec isolement du milieu familial est considérée par la plupart des auteurs comme une mesure indispensable.

1267
Exclusivement sur DOC - DZ : www.doc-dz.com NADJI 85
RESIDANAT EN POCHE TOME II
Cas Clinique en QCM

On doit craindre une évolution péjorative à long terme devant :


A - Importance de l'amaigrissement
B - Début précoce des troubles
C - Importance de la composante dépressive
D - Présence de vomissements
E - Prise en charge tardive
Bonne(s) réponse(s) : A D E

- Le début précoce des troubles est considéré comme un facteur de bon pronostic (sauf en cas d'anorexie pré-pubère).
- Est aussi considéré comme de mauvais pronostic l'existence d'épisodes de boulimie. (mauvais ?)

Mme R. , 32 ans, consulte depuis plusieurs années pour des troubles qui ont changé d'allure.
Initialement il y a 8 ans, elle souffrait de pensées parasites qu'elle ne pouvait chasser de son esprit, l'empêchant à de
nombreux moments de la journée de réaliser ses projets. Ces pensées étaient les suivantes : elle se représentait son fils
amputé d'un pied; si elle ne marchait pas sur les sillons tracés sur les trottoirs, elle était envahie par l'idée qu'il allait lui arriver
malheur ; quand, dans son travail, elle faisait des comptes, elle se sentait poussée à les refaire plusieurs fois, doutant chaque
fois du résultat.
Depuis 2 ans, éventuellement sous l'effet du traitement, les troubles ont changé : elle n'est plus tourmentée par les pensées
morbides passées ; par contre elle ne peut plus conduire sa voiture seule par crainte de ne pas pouvoir démarrer aux feux
rouges; elle ne va dans les grands magasins qu'accompagnée par son mari; seule elle aurait trop peur d'y avoir un malaise et
que personne ne l'aide. L'humeur est syntone.

Quel est le diagnostic des troubles observés il y a 8 ans ?


A - Pensées délirantes de persécution
B - Délire mélancolique
C - Phobie sociale
D - Obsessions idéatives
E - Agoraphobie
Bonne(s) réponse(s) : D

Obsession : idée s'imposant à la patiente et dont elle reconnait le caractère morbide.


Les obsessions idéatives sont des obsessions de caractère interrogatif et ruminatif.

Quel est le traitement le plus adapté des troubles anciens ?


A - Neuroleptique type phénothiazine
B - Benzodiazépine
C - Neuroleptique type butyrophénone
D - Antidépresseur tricyclique
E - Relaxation
Bonne(s) réponse(s) : D

- Efficacité de la domipromine (Anafronil) à fortes doses jusqu'à 300 mg/jour.


- Efficacité d'autres antidépresseur type sérotoninergique (exemple : fluoxétine Prozac).

Quel est le diagnostic des troubles observés depuis 2 ans ?


A - Pensées délirantes de persécution
B - Délire mélancolique
C - Phobie sociale
D - Obsessions idéatives
E - Agoraphobie
Bonne(s) réponse(s) : E

Peur d'être seule, d'être dans un endroit d'où la personne ne pourrait s'échapper sans difficulté si elle venait à ressentir une
incapacité. Comportement d'évitement et besoin d'être accompagnée.

Quelles sont les thérapeutiques des troubles apparus depuis 2 ans ?


A - Antidépresseur tricyclique
B - Thérapie comportementale
C - Neuroleptique sédatif
D - Psychothérapie verbale
E - Sismothérapies
Bonne(s) réponse(s) : A B D

Les antidépresseurs diminuent la symptomatologie anxieuse (traitement à long terme). Thérapie comportementale :
désensibilisation systématique et immersion.
Les psychothérapies verbales peuvent être envisagées, souvent dans un second temps.

1268
Exclusivement sur DOC - DZ : www.doc-dz.com NADJI 85
RESIDANAT EN POCHE TOME II
Cas Clinique en QCM

Les 2 types de troubles exposent aux modalités évolutives suivantes :


A - Accès dépressif
B - Psychose hallucinatoire chronique
C - Paroxysme anxieux
D - Délire paranoïaque passionnel
E - Schizophrénie
Bonne(s) réponse(s) : A C

- Troubles anxieux : pas de risque évolutif vers une psychose.


- Les autres complications sont abus d'alcool et/ou de tranquillisants, perte majeure d'autonomie.

Une jeune femme de 22 ans est arrêtée par la police car elle s'est installée à un carrefour et cherche à régler la circulation.
Amenée à l'hôpital, non sans difficulté (violence à agent) car son comportement exalté a inquiété les policiers, elle présente
une excitation psychomotrice, parle fort, change de sujet, fait des jeux de mots, mais elle semble désorientée, dit ne plus
savoir qui elle est car elle reçoit des ordres qui influencent ses actes.
Grâce à sa carte d'identité, on peut contacter la famille : celle-ci indique qu'elle a fugué depuis deux jours mais qu'elle ne
présentait auparavant ni trouble relationnel ni difficulté universitaire.

Quel(s) diagnostic(s) évoquez-vous ?


A - Un état maniaque
B - Une bouffée délirante aiguë
C - Une forme d'entrée dans la schizophrénie
D - Une crise d'originalité juvénile
E - Etat dépressif
Bonne(s) réponse(s) : B

On pourrait évoquer une manie atypique mais absence de syndrome d'influence dans la manie.
Il faut six mois d'évolution des troubles pour pouvoir évoquer l'entrée dans la schizophrénie.

Quel(s) type(s) de traitement instaurez-vous ?


A - Neuroleptique sédatif
B - Tranquillisant injectable
C - Neuroleptique incisif
D - Association de neuroleptiques
E - Béta bloquant
Bonne(s) réponse(s) : A D

But : diminution de l'agitation psychomotrice et du délire, donc utilisation de neuroleptiques.


D - Type Haldol, Largactil. Cependant, beaucoup d'auteurs recommandent maintenant une monothérapie.

De quel(s) effet(s) indésirable(s) faut-il surveiller l'apparition dans l'immédiat ?


A - Dyskinésie aiguë
B - Somnolence
C - Polydipsie
D - Hypotension artérielle
E - Agranulocytose
Bonne(s) réponse(s) : B D

A - Après 48 heures de traitement.


C E - Effets indésirables plus tardifs.

Cette malade refuse les soins et cherche à partir de l'hôpital. Que faites-vous ?
A - Vous la laissez partir
B - Vous demandez un placement volontaire
C - Vous proposez un placement d'office
D - Vous passez outre et imposez le traitement
E - Vous la mettez sous curatelle
Bonne(s) réponse(s) : B C

- Etat de dangerosité pour elle-même ou pour autrui. On décide une hospitalisation à la demande d'un tiers (anciennement
placement à la demande d'un tiers). La famille est présente.
- La curatelle (conseil et contrôle de l'administration des biens) ne peut concerner qu'une pathologie chronique.
C - Si le placement volontaire ne peut être réalisé. Menace pour l'ordre public.

1269
Exclusivement sur DOC - DZ : www.doc-dz.com NADJI 85
RESIDANAT EN POCHE TOME II
Cas Clinique en QCM
Monsieur B. Charles, âgé de 55 ans, ouvrier serrurier, vient de faire une tentative de suicide par pendaison.
Au cours de l'entretien, il apparaît profondément triste et des crises de larmes interrompent fréquemment son discours. Il
exprime spontanément des idées de mort car il a trop honte de la faute démesurée qu'il a commise il y a quelques années :
l'emprunt, quelques jours, d'un fer à souder de l'atelier pour effectuer à son domicile des travaux.
On apprend de son entourage que les premiers symptômes (tristesse et isolement) ont fait leur apparition quelques semaines
après le déplacement de Monsieur B. de son poste de travail.

Comment qualifier les idées de Monsieur B. concernant sa "faute" ?


A - Désintérêt
B - Insatisfaction pessimiste
C - Auto-dépréciation
D - Auto-accusation
E - Auto-apitoiement
Bonne(s) réponse(s) : D

Idée délirante de culpabilité, portant sur une faute vénielle aux conséquences grossies jusqu'à l'absurde.

Quels sont les symptômes somatiques dont la présence viendrait conforter le diagnostic ?
A - Céphalées
B - Hypotension artérielle
C - Constipation
D - Algies disséminées
E - Insomnie
Bonne(s) réponse(s) : C E

C - Troubles digestifs : anorexie, constipation.


E - Surtout terminale, le sujet se réveillant au petit matin avec l'angoisse de la journée à vivre.

Quel diagnostic évoquez-vous ?


A - Délire de relation
B - Délire de préjudice
C - Dépression névrotique
D - Dépression mélancolique
E - Dépression psychogène
Bonne(s) réponse(s) : D

Episode dépressif : tristesse de l'humeur, douleur morale, idées de mort, idées suicidaires, repli sur soi, idées de culpabilité et
d'indignité.
De type mélancolique : perte d'intérêt massive, pas de réactivité, dépression plus marquée le matin, anorexie, idées délirantes
de culpabilité.

En cas de refus d'hospitalisation du patient, quelle mesure légale est la plus appropriée pour imposer des soins
au patient ?
A - Mesure d'injonction thérapeutique
B - Placement volontaire
C - Mise sous tutelle
D - Mise sous sauvegarde de justice
E - Placement d'office
Bonne(s) réponse(s) : B

B - Depuis juillet 1990 : hospitalisation à la demande d'un tiers (pas d'office car pas de menace pour l'ordre public).

1270
Exclusivement sur DOC - DZ : www.doc-dz.com NADJI 85
RESIDANAT EN POCHE TOME II
Cas Clinique en QCM
Monsieur M. 42 ans, se présente à la consultation du service de psychiatrie. Il est très anxieux et un peu abattu car "ça
recommence". Il y a un an il avait été hospitalisé pour un état hyperanxieux, presque affolé car il pensait être victime d'un
complot. Il avait compris le manège. Un de ses collègues ne le saluait plus depuis que le chef de bureau lui avait proposé une
promotion.
Il se demande si son subalterne n'est pas de connivence avec le chef de bureau, car celui-ci lui a imposé de partager le même
vestiaire, sûrement pour surveiller ses actes et ses fréquentations.
Il souhaite être hospitalisé car il se sent à nouveau menacé. Il pense même que cette surveillance n'a pas cessé depuis un an.
Lors de l'entretien, on ne note aucun autre trouble psychique. Le ton affectif est celui de la souffrance avec un aspect sombre
de la mimique, une humeur douloureuse. Il parle aussi des intentions de divorce de son épouse. Il en souffre mais n'exprime
aucune rancune. Il pense même lui laisser la garde des enfants.

Quels sont les symptômes présents dans l'observation ?


A - Dysthymie
B - Idées délirantes de jalousie
C - Angoisse hypocondriaque
D - Idées délirantes de persécution
E - Idées délirantes de mégalomanie
Bonne(s) réponse(s) : D

Délire : de type interprétatif, thème persécutif, bien construit.

Quel(s) est (sont) le(s) diagnostic(s) psychiatrique(s) possible(s) ?


A - Attaque de panique
B - Délire paranoïaque de jalousie
C - Mélancolie délirante
D - Délire de persécution
E - Délire chronique imaginatif
Bonne(s) réponse(s) : C D

B - Conviction délirante d'être trompé.


C - Tristesse de l'humeur, souffrance morale.
E - Le mécanisme dans l'observation est interprétatif.

Quelles sont les modalités évolutives à craindre principalement ?


A - Evolution démentielle
B - Chronicité délirante
C - Evolution dépressive
D - Tentative de suicide
E - Toxicomanie
Bonne(s) réponse(s) : B C D

B - Pas de mise à distance du délire.

Quelle est la modalité thérapeutique à mettre en place en 1ère intention ?


A - Anxiolytique seul
B - Halopéridol (Haldol®) seul
C - Chlorpromazine (Largactil®) seul
D - Sismothérapie
E - Association d'halopéridol et d'antidépresseur
Bonne(s) réponse(s) :

Sans commentaire.

1271
Exclusivement sur DOC - DZ : www.doc-dz.com NADJI 85
RESIDANAT EN POCHE TOME II
Cas Clinique en QCM
Un jeune homme de 17 ans est amené à votre consultation par sa mère. En effet, depuis quelques jours, il refuse de sortir de
sa chambre, se refuse à la moindre activité, ne va plus en classe, a abandonné toutes ses activités de loisir. Il ne voit plus
personne, hormis sa famille. Il tient des propos étranges, parfois incompréhensibles, prétendant que des extra-terrestres ont
décidé de l'enlever en raison de ses extraordinaires pouvoir télépathiques et de son intelligence supérieure. Il pense avoir pu,
jusqu'à présent, leur échapper, mais ceux-ci, dépités croit-il, lui envoient dans le corps des courants électriques et veulent lui
arracher les nerfs.Le patient est angoissé et parle de s'enfuir dans le désert, le seul lieu où il peut être vraiment en sécurité.
Depuis un an et demi, il ne fait plus rien en classe et redouble sa seconde.

Quel est le diagnostic le plus probable ?


A - Bouffée délirante aiguë
B - Mélancolie
C - Dépression névrotique
D - Schizophrénie
E - Crise de l'adolescence
Bonne(s) réponse(s) :D

A - Non, car évolution supérieure à 6 mois.


D - Age du patient, délire paranoïaque, fléchissement des activités, bizarreries.

Quel(s) symptôme(s) relevez-vous ?


A - Délire
B - Apragmatisme
C - Impulsion
D - Dysthymie
E - Excitation maniaque
Bonne(s) réponse(s) : A B

B - Activité inadaptée et inefficace, avec perte d'intérêt.

Quel traitement chimiothérapique instituerez-vous ?


A - Sels de lithium
B - Antidépresseurs
C - Neuroleptiques
D - Vitaminothérapie
E - Hypnotiques
Bonne(s) réponse(s) : C

C - Traitement antipsychotique.

Quelle est la modalité de prise en charge la plus indiquée ?


A - Cure analytique
B - Hospitalisation
C - Consultations ambulatoires
D - Hospitalisation à domicile
E - Maison de repos
Bonne(s) réponse(s) : B

Sans commentaire.

Madame X... Françoise, 34 ans, mariée, mère de deux enfants, avait présenté dans les suites de son premier accouchement
un épisode confuso-onirique aigu qui avait nécessité une hospitalisation pendant une quinzaine de jours et un traitement
neuroleptique. Sa mère et sa grand-mère maternelles ont présenté chacune deux épisodes dépressifs graves. Françoise a
présenté à 4 reprises à 28 ans, 29 ans, 31 et 33 ans, des épisodes d'agitation avec euphorie, hyper-activité, dépenses
excessives, perte du contrôle émotionnel et pulsionnel .
Madame X... vient consulter à la demande de sa famille pour une excitation modérée, des activités désordonnées et une
insomnie. L'examen physique met en évidence un petit goitre. L'humeur est euphorique mais sans trouble majeur du
comportement; le débit verbal est accéléré.

Quel est le diagnostic nosographique ?


A - Hystérie
B - Schizophrénie
C - Psychose maniaco-dépressive unipolaire
D - Psychose maniaco-dépressive bi-polaire
E - Déséquilibre psychique
Bonne(s) réponse(s) : C

Quatrième accès maniaque. Absence d'antécédent d'épisode dépressif.

1272
Exclusivement sur DOC - DZ : www.doc-dz.com NADJI 85
RESIDANAT EN POCHE TOME II
Cas Clinique en QCM

Quelle(s) mesure(s) médicolégale(s) est(sont) susceptible(s) d'être prise(s)


dans ce cas ?
A - Placement volontaire
B - Placement d'office
C - Curatelle simple
D - Sauvegarde de Justice
E - Tutelle/Curatelle
Bonne(s) réponse(s) : A D

A - Ou maintenant hospitalisation par demande d'un tiers.


D - Décision temporaire (durée de deux mois). Protection contre tout côté susceptible de lui nuire (conserve droits civils).

Parmi les psychotropes suivants, quelle catégorie de médicaments vous paraît la plus indiquée comme
thérapeutique au long cours de cette pathologie ?
A - Benzodiazépines
B - Neuroleptiques
C - Antidépresseurs
D - Anti-thyroïdiens de synthèse
E - Sels de Lithium
Bonne(s) réponse(s) : E

Le traitement au long cours est basé sur les thymorégulateurs.

Quels sont les deux examens indispensables à faire chez cette malade avant l'institution de ce
traitement ?
A - E.E.G.
B - E.C.G.
C - Bilan thyroïdien
D - Bilan rénal
E - Frottis vaginaux
Bonne(s) réponse(s) : B D

Contre-indications absolues :
- insuffisance rénale et hépatique
- hyponatrémie
- grossesse
- pathologie thyroïdienne
- cardiopathie sévère (troubles du rythme).

Un traitement complémentaire doit être instauré. Lequel ?


A - Tonicardiaque
B - Diurétique
C - Mise en oeuvre d'une contraception
D - Antiparkinsoniens de synthèse
E - Extraits thyroïdiens
Bonne(s) réponse(s) : C

Action thératogène du lithium au premier trimestre de grossesse.

1273
Exclusivement sur DOC - DZ : www.doc-dz.com NADJI 85
RESIDANAT EN POCHE TOME II
Cas Clinique en QCM
Un adolescent de 17 ans est vu en consultation pour des troubles apparus progressivement depuis 8 mois : désintérêt pour
ses études, moins grande communication au sein de sa famille, tendance à s'isoler, colères impulsives sans cause
déclenchante, accès de rires immotivés. Ses parents signalent de longues périodes de contemplation devant une glace et des
plaintes concernant son apparence physique. Lors de l'entretien, il exprime le sentiment que les gens et les choses lui
paraissent transformés et bizarres, qu'il a l'impression que les os de son visage s'allongent et le rendent désagréable à
regarder.
On lui envoie de l'extérieur des pensées qui ne sont pas les siennes. Quand il lit, une voix intérieure répète sa lecture. Il a
quelques fois envie de disparaître pour en finir avec cette souffrance qui le submerge.

Parmi les symptômes suivants, quel est celui (ceux) qui est (sont) présent(s) dans cette observation :
A - Discordance
B - Déréalisation
C - Syndrome d'influence
D - Hallucination psychique
E - Phobies d'impulsion
Bonne(s) réponse(s) : A B C D

A - Rires imotivés.
B - Perte de contact avec la réalité (gens perçus comme bizarres).
C - Pensées imposées par l'extérieur.
D - Voix intérieure.

La répétition par une voix intérieure de la lecture évoque spécifiquement l'un des symptômes suivants :
A - Interprétation délirante
B - Echolalie
C - Idées obsédantes
D - Automatisme mental
E - Déréalisation
Bonne(s) réponse(s) : D

Automatisme mental : dédoublement de la pensée (écho de la lecture).

Parmi les états pathologiques suivants, quel est celui que cette observation évoque en premier lieu ?
A - Crise d'originalité juvénile
B - Dépression névrotique
C - Schizophrénie
D - Névrose phobique
E - Névrose d'angoisse
Bonne(s) réponse(s) : C

- Début des troubles supérieur à 8 mois.


- Syndrome dissociatif.
- Syndrome délirant.

Parmi les traitements suivants, le ou lesquels vous paraît(ssent) le (les) plus approprié(s) au cas de ce patient ?
A - Anafranil® (Clomipramine)
B - Largactil® (Chlorpromazine)
C - Haldol® (Halopéridol)
D - Valium® (Diazépam)
E - Marsilid® (I.M.A.O.)
Bonne(s) réponse(s) : B C

Neuroleptiques sédatifs en monothérapie ou en association.

Parmi les propositions suivantes concernant l'évolution, laquelle ou lesquelles est ou sont possible(s) ?
A - Guérison par séparation de la famille
B - Alternance de phases de rémission et d'aggravation
C - Survenue d'épisodes processuels
D - Impulsions suicidaires
E - Evolution chronique
Bonne(s) réponse(s) : B C D E

E - Rarement évolution discontinue, surtout depuis l'utilisation des neuroleptiques.

1274
Exclusivement sur DOC - DZ : www.doc-dz.com NADJI 85
RESIDANAT EN POCHE TOME II
Cas Clinique en QCM
Monsieur N... 26 ans est amené en urgence par le SAMU. Il s'était agité chez lui mais l'agitation a rapidement cédé avec
l'injection d'un neuroleptique.
Il a l'impression qu'un scénario étrange se déroule devant ses yeux. Il ne sait s'il vit un rêve ou la réalité. Il a peur qu'on le tue.
Il se sent poursuivi par des voitures, notamment les voitures de couleur rouge.
A certains moments, il prend son visage à deux mains, palpe ses joues, puis regarde ses mains avec étonnement et dit "à qui
est ce visage ?".
En quelques heures, il s'est montré tantôt rêveur, tantôt abattu et triste, tantôt jovial. Il interroge ses interlocuteurs, en leur
demandant s'il est schizophrène .
L'entretien avec son épouse révèle qu'il n'est pas bien depuis 10 jours, n'a aucun antécédent personnel ni familial. Il a eu une
promotion professionnelle il y a 6 mois. Récemment il appris que sa soeur cadette souffrait d'un cancer.

Quels sont les symptômes observés ?


A - Conscience oniroïde
B - Hallucinations auditives
C - Dépersonnalisation
D - Etat d'étrangeté
E - Idées délirantes de persécution
Bonne(s) réponse(s) : A C D E

A - Perplexité, impression de rêve.


C - Perte de l'intégrité corporelle.

Quel est le diagnostic psychiatrique ?


A - Mélancolie délirante
B - Schizophrénie paranoïde
C - Hystérie grave
D - Etat délirant et oniroïde aigu
E - Paranoïa aiguë
Bonne(s) réponse(s) : D

Ou bouffée délirante aiguë.

Quel est le principal diagnostic différentiel avec une affection organique ?


A - Hydrocéphalie à pression normale
B - Tumeur frontale
C - Cranio-pharyngiome
D - Epilepsie temporale
E - Hypothyroïdie
Bonne(s) réponse(s) : D

Epilepsie temporale : allure paroxystique, courte durée, répétition stéréotypée, contenu amnésique.

Quels sont les éléments thérapeutiques des premiers jours ?


A - Traitement antidépresseur
B - Benzodiazépine seule
C - Haldol® (haloperidol)
D - Psychothérapie de soutien
E - Neuroleptique à action prolongée d'emblée
Bonne(s) réponse(s) : C

Sans commentaire.

Indiquez les modalités évolutives possibles dans les cinq années à venir :
A - Absence de troubles
B - Schizophrénie
C - Troubles dépressifs
D - Récidive identique
E - Anxiété hyponcondriaque
Bonne(s) réponse(s) : A B C D

C - Surtout dans le cadre d'une psychose maniaco-dépressive.

1275
Exclusivement sur DOC - DZ : www.doc-dz.com NADJI 85
RESIDANAT EN POCHE TOME II
Cas Clinique en QCM
Monsieur Z..., 54 ans, est amené à la consultation par sa femme.
Il apparait d'abord prostré sur sa chaise, puis vous agrippe le bras en disant : "La place X... (la principale place de la ville) a
brûlé. Il n'y a que les maisons des juifs qui sont debout. C'est une punition, je suis un misérable, mieux vaut être mort". Depuis
quelques mois, vous dit sa femme, il n'était plus le même homme, ne s'intéressant plus à la lecture comme auparavant, fuyant
les contacts avec les amis de la famille, semblant triste et pleurant parfois tout seul. Il a maigri de 6 kg en 4 mois. Il dort mal la
nuit, mais somnole pendant la journée. Il n'aurait jamais présenté de troubles psychiatriques auparavant.

Quel diagnostic évoquez- vous ?


A - Mélancolie délirante
B - Psychose paranoïaque
C - Psychose schizophrénique
D - Schizophrénie dysthymique
E - Etat démentiel
Bonne(s) réponse(s) : A

Episode dépressif type mélancolique (tristesse de humeur, anorexie, insomnie, désintérêt, repli sur soi), autodévalorisation,
idées de mort avec culpabilité délirante (punition).

Parmi les traitements suivants le(s) quel(s) vous semble(nt) indiqué(s) ?


A - Anafranil 150 mg/j + Haldol 1 0 mg/j
B - Survector 200 mg/j + Temesta 10 mg/j
C - Tercian 150 mg/j
D - Sismothérapie
E - Thérapie comportementale
Bonne(s) réponse(s) : A D

- Traitement antidépresseur majeur


- Si résistance au traitement : sismothérapie.

Ce patient a été traité par Laroxyl 200 mg/j, Largactil 250 mg/j, Artane 10 mg/j. Sa symptomatologie s'est
amendée, et a laissé la place à un tableau associant : insomnie, propos incohérents, agitation avec
déambulation. Le patient ignore la date, le lieu. L'examen neurologique est normal. Quel diagnostic portez-vous
?
A - Rechute du tableau initial
B - Démence type Alzheimer
C - Accident vasculaire ischémique
D - Confusion iatrogène
E - Dépersonnalisation hystérique
Bonne(s) réponse(s) : D

Désorientation temporo-spatiale, perplexité.

Dans le processus physiopathologique de cette nouvelle symptomatologie, vous mettez plutôt en cause ?
A - L'acétylcholine
B - La Dopamine
C - La Noradrénaline
D - La sérotonine
E - L'Histamine
Bonne(s) réponse(s) : A

Potentialisation des effets anticholinergiques par les phénothiazines et les tricycliques.

1276
Exclusivement sur DOC - DZ : www.doc-dz.com NADJI 85
RESIDANAT EN POCHE TOME II
Cas Clinique en QCM
A la suite d'une défenestration sans conséquence vous consulte Hélène, étudiante en 2è année de DEUG d'Espagnol.
Amaigrie, le teint pâle, elle refuse de vous serrer la main, de s'asseoir, exige de repartir chez elle.
Rêveuse, elle vous dit : "à son regard, j'ai compris son amour, sa voix m'a imposé de sauter par la fenêtre, des ondes
maléfiques me brûlaient le ventre". En souriant, elle vous annonce qu'un accident mortel va vous arriver ou que son ami va
vous tuer. Hélène verse quelques larmes, semble tendre l'oreille puis quitte le bureau.
Sa camarade signale qu'elle demeurait solitaire depuis la rentrée universitaire il y a 8 mois. Elle tient des propos étranges
depuis 3 mois. Deux ans auparavant, elle a été hospitalisée dans une clinique pour un épisode baptisé "dépression". Sa mère,
serait soignée pour une maladie maniaco-dépressive.

Quels sont les symptômes observés ?


A - Hallucinations auditives
B - Délire systématisé de persécution
C - Délire d'influence paranoïde
D - Agoraphobie
E - Discordance affective
Bonne(s) réponse(s) : A C E

A - Attitude d'écoute.
C - Délire non systématisé, pensées imposées (hallucinations psychiques).
E - Sourires, pleurs immotivés.

Quel est le diagnostic nosographique ?


A - Psychose hallucinatoire chronique
B - Délire chronique paraphrénique
C - Délire paranoïaque
D - Mélancolie délirante
E - Schizophrénie paranoïde
Bonne(s) réponse(s) : E

- Evolution depuis plus de six mois.


- Syndrome dissociatif.
- Syndrome délirant paranoïde.

Quel est le symptôme qui a contribué le plus à provoquer le passage à l'acte suicidaire ?
A - Repliement
B - Hallucinations cénesthésiques
C - Réticence
D - Tendances dépressives
E - Hallucinations acoustico-verbales
Bonne(s) réponse(s) : E

Hallucinations impérieuses.

En cas de refus d'hospitalisation, quelle est la mesure médico-légale adéquate ?


A - Sauvegarde de justice
B - Curatelle
C - Placement d'office
D - Placement volontaire
E - Tutelle
Bonne(s) réponse(s) : D

C - Pas de dangerosité pour l'ordre public.


D - Hospitalisation à la demande d'un tiers, nécessité de deux certificats médicaux.

Quels sont les trois médicaments adéquats susceptibles d'être prescrits initialement ?
A - Diazepam (Valium®)
B - Fluphenazine (Moditen®)
C - Clomipramine (Anafranil®)
D - Chlorpromazine (Largactil®)
E - Haloperidol (Haldol®)
Bonne(s) réponse(s) : B D E

Traitement par neuroleptiques (action recherchée sur les psychoses d'agitation).

1277
Exclusivement sur DOC - DZ : www.doc-dz.com NADJI 85
RESIDANAT EN POCHE TOME II
Cas Clinique en QCM

Compte tenu de la grande réticence de la patiente à se traiter, quel est le traitement le plus adéquat au long
cours ?
A - Chlorazepate (Tranxène®)
B - Haloperidol (Haldol®)
C - Amitriptyline (Laroxyl®)
D - Chlorpromazine (Largactil®)
E - Décanoate de fluphenazine (Modecate®)
Bonne(s) réponse(s) : E

- Neuroleptiques-Retard : 1 injection toutes les deux à quatre semaines.


- Autres neuroleptiques retard : Haldol® Decinoas, Fluanxol retard®, Piportil®, Moditen Retard®.

1278
Exclusivement sur DOC - DZ : www.doc-dz.com NADJI 85
RESIDANAT EN POCHE TOME II
Cas Clinique en QCM

1279
Exclusivement sur DOC - DZ : www.doc-dz.com NADJI 85
RESIDANAT EN POCHE TOME II
Cas Clinique en QCM

Ce viticulteur de 75 ans a toujours bu environ 1 l de vin rouge par jour. Il a traité ses vignes, il y a fort longtemps, avec de
l'arsenic. C'est un sujet blond aux yeux bleus. Il présente depuis quelques semaines une petite lésion légèrement saillante et
irrégulière du pavillon de l'oreille. La surface de cette lésion est irrégulière, recouverte de squames kératosiques dont l'ablation
découvre une ulcération superficielle indurée, fixée au cartilage sous-jacent. La biopsie de cette lésion a révélée un carcinome
(épithélioma) spino-cellulaire. En examinant de malade on découvre ce multiples lésions dyskératosiques du visage (front,
pommettes) et du dos des mains, ainsi qu'une petite adénopathie pré-tragienne homolatérale dure de 1 cm de diamètre. La
radiographie pulmonaire est normale. La lésion est classée T1 N1 M0. Le malade est en excellent état général.

Laquelle(lesquelles) des propositions suivantes est(sont) correcte(s) ? Les lésions dyskératosiques et des
mains de ce malade :
A - Sont sans aucun rapport avec l'adénopathie pré-tragienne
B - Sont fréquemment associées aux épithéliomas spino-cellulaires
C - Siègent comme les épithéliomas spino-cellulaires dans les zones exposées au soleil
D - Traduisent une dissémination à distance de l'épithélioma
E - Sont souvent des lésions pré-épithéliomateuses
Bonne(s) réponse(s) : B C E

A - Il s'agit d'une adénopathie satellite des lésions de l'oreille.

Parmi les éléments suivants de l'observation, quel est celui qui n'est pas épidémiologiquement corrélé avec
l'épithélioma spino-cellulaire ?
A - Arsenisme.
B - Blond aux yeux bleus
C - Ethylisme modéré (1 l/jour)
D - Soixante quinze ans
E - Viticulteur
Bonne(s) réponse(s) : C

Ils surviennent surtout chez les sujets de race blanche, les blonds à peau claire, porteurs de taches de rousseur (notamment
dans les pays à forte insolation : Australie...).
Les ultraviolets constituent le principal facteur de risque (par exemple chez les patients travaillant en plein air).

Quelle signification sémiologique accorder à l'adénopathie ?


A - Adénopathie satellite des lésions dyskératosiques
B - Réaction immunitaire à l'épithélioma spino-cellulaire
C - Métastase ganglionnaire
D - Ganglion banal, l'épithélioma spino-cellulaire ne donnant pas de métastase
E - Adénopathie satellite d'une surinfection microbienne de l'ulcération
Bonne(s) réponse(s) : C

D - 10 % des cancers spinocellulaires environ ont une extension lymphatique.

Quelle est parmi les propositions suivantes, celle qui convient pour exprimer la classification T1 dans le système
TNM ?
A - Diamètre de moins de 2 cm
B - Epaisseur de moins de 0,75 mm
C - Envahissement limite au derme superficiel
D - Lésion fixée au cartilage sous-jacent
E - Caractère superficiel de l'ulcération
Bonne(s) réponse(s) : A

Connaissance.

Quelle indication thérapeutique prévoir dans un tel cas ?


A - Chimiothérapie locale par une pommade au fluoro-uracile
B - Chimiothérapie intra-artérielle
C - Chimiothérapie systématique première suivie d'irradiation
D - Exérèse chirurgicale de la lésion et irradiation ganglionnaire prétragienne
E - Irradiation externe exclusive de la lésion et des aires ganglionnaires
Bonne(s) réponse(s) : D

L'exérèse la plus précoce et la plus complète de la tumeur doit être réalisée sous anesthésie générale ou locale.
Une exérèse large est nécessaire.

1280
Exclusivement sur DOC - DZ : www.doc-dz.com NADJI 85
RESIDANAT EN POCHE TOME II
Cas Clinique en QCM
Au cours d'une visite systématique de médecine du travail, sont mises en évidence chez un maçon salarié de 30 ans des
lésions cutanées prurigineuses à type d'érythème et de vésicules au dos des mains et de la partie distale des avant bras.
Dans le dossier médical, au cours des consultations antérieures, était notée la notion de lésions cutanées de même
localisation à type de fissures et d'érosion. Le diagnostic de dermite eczématiforme est posé.

Cette maladie, sous réserve du respect des conditions d'ouverture des droits sera considérée par la Sécurité
Sociale, régime général, comme :
A - Une "maladie à caractère professionnel"
B - Un "accident de travail"
C - Une ''maladie de longue durée"
D - Une "maladie professionnelle indemnisable"
E - Une "longue maladie"
Bonne(s) réponse(s) : D

Evident.

En cas de "maladie à caractère professionnel" qui doit faire la déclaration à l'inspection du travail ?
A - Le médecin traitant
B - L'employeur
C - Le médecin du travail
D - Le dermatologue
E - Le salarié
Bonne(s) réponse(s) : A C D

Tout médecin peut et doit faire cette déclaration. Cette déclaration est effectuée lorsque les conditions définies par les
tableaux de maladies professionnelles sont remplies.

En cas de Maladie professionnelle indemnisable qui doit faire la déclaration à la caisse primaire d'assurance
maladie ?
A - Le médecin traitant
B - L'employeur
C - Le médecin du travail
D - Le dermatologue
E - Le salarié
Bonne(s) réponse(s) : E

Le salarié doit effectuer la déclaration dans les 15 jours qui suivent la cessation du travail.

En cas de maladie professionnelle indemnisable, les conditions de prise en charge par la Sécurité Sociale
régime général dans ce cas clinique nécessitent de :
A - Produire un certificat d'un médecin inspecteur du travail
B - Remplir les conditions requises par un tableau officiel
C - Etre salarié
D - Etre exposé au risque depuis plus de 10 ans
E - Ne pas être en arrêt de travail
Bonne(s) réponse(s) : B C

Evident et déjà commenté.

Une dermatose professionnelle orthoergique peut être causée par :


A - Des produits chimiques
B - Des parasites
C - Des agents physiques
D - Une hypersensibilité individuelle
E - Des agents microbiens
Bonne(s) réponse(s) : A B C E

Contrairement aux dermatoses allergiques, les dermatoses orthoergiques sont susceptibles de survenir chez tous les sujets
exposés.

1281
Exclusivement sur DOC - DZ : www.doc-dz.com NADJI 85
RESIDANAT EN POCHE TOME II
Cas Clinique en QCM

Le médecin du travail face à ce cas clinique doit :


A - Prescrire un traitement
B - Effectuer au cours de ta consultation des tests épicutanés
C - Envoyer ce patient à son médecin traitant
D - Signaler à l'employeur l'affection dont est victime son salarié
E - Envisager une action préventive au poste de travail
Bonne(s) réponse(s) : C E

AB - Ne sont pas du ressort du médecin du travail.


D - Est une violation évidente du secret professionnel.

Un sujet, sans antécédent notable ni facteur de risque, conducteur de poids lourds en transport routier internationaux, se
fracture les deux os de l'avant bras en descendant de son camion. Il est alors plâtré après réduction.
La survenue d'une phlébite sous plâtre dans les jours suivants conduit à la prescription d'un traitement anticoagulant.
Sous ce traitement apparaissent des hémorragies diffusés liées à un surdosage, qui nécessitent un arrêt brutal de la
thérapeutique et la prescription de P.P.S.B.
Il présente alors un infarctus myocardique postéro-diaphragmatique.

Pourquoi l'infarctus du myocarde peut-il être pris en charge en tant qu'accident du travail ?
A - Parce qu'il y a filiation entre l'accident et les complications
B - Par présomption d'imputabilité
C - Par l'absence d'antécédent et de facteur de risque (état antérieur)
D - Par arrêt des anticoagulants
E - Parce qu'il n'y a pas eu de reprise du travail entre temps
Bonne(s) réponse(s) : A B C

Sans commentaire.

Le sujet va bénéficier au titre de la législation sur les accidents du travail d'un ou plusieurs avantages ;
le(s)quel(s) ?
A - Prise en charge totale des frais médicaux et pharmaceutiques
B - Prise en charge des frais d'hospitalisation sauf ticket modérateur
C - Droit à une indemnité journalière
D - Droit à une indemnité de changement d'emploi s'il ne peut pas reprendre son métier
E - Droit éventuel à une rente d'incapacité permanente partielle
Bonne(s) réponse(s) : A C E

B - Le patient n'a pas à faire l'avance du ticket modérateur.


E - Après consolidation.

Ce sujet conserve, au troisième moisn, des manifestations à type de douleur angineuse pour des efforts
modérés, des investigations complémentaires sont nécessaires pour le pronostic (exemple : coronarographie)
Ils sont pris en charge par :
A - L'employeur
B - Sa mutuelle
C - Le fond de garantie automobile
D - Le régime accident du travail
E - Le régime de l'invalidité
Bonne(s) réponse(s) : D

C'est la seule réponse logique, si le rapport entre l'accident et l'infarctus du myocarde est retenu.

Ce sujet, qui bénéficie d'un pontage aorto-coronarien., est consolidé au sixième mois avec un traitement
anticoagulant. Qui va alors décider de la reprise du travail ?
A - Le médecin du travail
B - Le médecin traitant
C - Le médecin conseil
D - Un médecin expert
E - Le médecin inspecteur régional du travail
Bonne(s) réponse(s) : B

Le salarié subira une visite médicale après chaque arrêt de travail :


- pour accident de travail ou maladie professionnelle, quelle que soit sa durée
- pour maladie ayant nécessité plus de 3 semaines d'arrêt ou en cas d'absences répétées pour raison de santé
- au retour d'un congé de maternité
Si le médecin du travail estime qu'il n'y a pas de problème d'aptitude, le salarié reprend son activité antérieure.
Cet examen doit avoir lieu le jour de la reprise.

1282
Exclusivement sur DOC - DZ : www.doc-dz.com NADJI 85
RESIDANAT EN POCHE TOME II
Cas Clinique en QCM

Cette reprise du travail décidée, qui va formuler l'avis d'aptitude ?


A - Le médecin traitant
B - Le médecin du travail
C - Le médecin de la commission du permis de conduire
D - Le médecin conseil de la caisse de Sécurité Sociale
E - Le chef du personnel de l'entreprise
Bonne(s) réponse(s) : B

Le médecin du travail n'est pas lié par l'avis du médecin traitant et/ou du médecin conseil de la Sécurité Sociale quant à
l'opportunité de la reprise.
L'avis d'aptitude doit être formulé par écrit à l'employeur.

Ce sujet désire reprendre son travail antérieur : pourquoi ne le peut-il pas ?


A - Absence prolongée de plus de six mois
B - Poste de sécurité
C - Traitement anticoagulant
D - Nécessité d'un régime diététique particulier
E - Avis défavorable du médecin traitant
Bonne(s) réponse(s) : B

Le poste ne doit pas présenter de danger pour la sécurité du salarié ou de ses collègues de travail. On peut considérer que
l'insuffisance coronaire implique un risque de mort subite qui contre-indique ce poste.

Monsieur P. F., né en 1920, à la retraite depuis 1980, vient consulter en 1985 dans votre cabinet pour une toux persistante
depuis six mois, une dyspnée discrète d'apparition progressive, une baisse de l'état général, une fébricule.
A l'examen clinique. Monsieur P. F., est amaigri ; l'état général est moyen.
L'auscultation cardiaque est normale, ainsi que l'auscultation pulmonaire. Vous notez une hépatomégalie avec un discret
reflux hépatojugulaire et un oedème des membres inférieurs.
Monsieur P. F. n'a jamais fumé. L'étude de son curriculum laboris nous apprend qu'il a travaillé comme tailleur de pierre de
l'âge de 20 ans à l'âge de 45 ans.

L'histoire de la maladie est très évocatrice d'une silicose. Quel est l'argument principal ?
A - L'apparition progressive des signes cliniques
B - La toux
C - L'insuffisance cardiaque droite
D - La pauvreté de l'auscultation
E - Son métier de tailleur de pierre
Bonne(s) réponse(s) : E

Le diagnostic de silicose repose sur l'existence de signes radiologiques évocateurs chez un sujet ayant été exposé à
l'inhalation de poussières de silice.

Pour asseoir ce diagnostic vous vous entourez d'un certain nombre d'examens complémentaires. Quel est
l'examen indispensable pour confirmer le diagnostic de silicose ?
A - L'échographie cardiaque
B - La radiographie pulmonaire standard
C - Les épreuves spirométriques
D - La mesure de la diffusion du CO
E - La recherche de silice libre dans le liquide de lavage broncho-alvéolaire
Bonne(s) réponse(s) : B

C'est l'étape capitale du diagnostic.


La réparation de l'affection repose sur l'association : exposition professionnelle, images radiographiques compatibles.

1283
Exclusivement sur DOC - DZ : www.doc-dz.com NADJI 85
RESIDANAT EN POCHE TOME II
Cas Clinique en QCM

La baisse de l'état général, l'amaigrissement, la fébricule vous inquiètent. Quelle(s) complication(s) de la


silicose allez-vous rechercher ?
A - Une tuberculose pleuro-pulmonaire
B - Une aspergillose intra-cavitaire
C - Une sarcoïdose
D - Un cancer de la plèvre
E - Une plaque pleurale
Bonne(s) réponse(s) : A B

Les complications habituelles de la silicose sont :


- hémoptysies
- pneumothorax par rupture de bulle d'emphysème
- suppuration bronchopulmonaire
- aspergillome
- coeur pulmonaire chronique.

La réparation de la silicose est subordonnée aux conditions qui figurent dans le tableau n°25 du Régime
Général de la Sécurité Sociale Dans le cas présent, le diagnostic positif de la silicose étant établi., Monsieur P.
F., pourra-t-il prétendre à l'indemnisation ?
A - Oui, parce que Monsieur P. F., remplit les deux conditions d'exposition au risque et de délai de prise en
charge non dépassé
B - Non, parce que le délai de prise en charge était dépassé lorsque vous avez constaté la maladie
professionnelle
C - Non, parce que la profession qu'il a exercée n'est pas nommément désignée dans le tableau n°25 et que
la liste des travaux est limitative
D - Non, parce que Mr. P. F., ne remplit pas la condition d'exposition professionnelle au risque et que le délai
de prise en charge était dépasse
E - Oui, parce qu'il existe des dispositions réglementaires particulières concernant la réparation de la silicose
Bonne(s) réponse(s) : E

Le délai de prise en charge de 15 ans est dépassé (arrêt du travail en 65, consultation en 85), mais l'avis du collège des 3
médecins peut être demandé.
Le délai d'exposition de 5 ans est respecté.

Vous recevez en consultation un patient de 48 ans pour une dyspnée accompagnée de douleurs basithoraciques.
L'interrogatoire vous apprend que cette dyspnée s'est constituée progressivement en quelques années, aggravée par des
épisodes bronchitiques, et qu'il existe une intoxication tabagique à 10 cigarettes par jour. Il a travaillé au fond, dans une mine
de charbon du Pas de calais, pendant 17 ans et, depuis 2 ans, il est conducteur d'engins dans une entreprise de travaux
publics.
A l'examen, vous retrouvez des ronchus et des sibilants diffus, ainsi qu'une diminution du murmure vésiculaire aux bases. Les
bruits du coeur sont normaux. La radiographie objective des opacités réticulo-micronodulaires et nodulaires confluentes, des
opacités denses ovalaires de 4 a 5 cm dans les régions sous-claviculaires, des hyperclartés aux
bases et une distorsion de l'arbre bronchique. L'ombre cardiaque est normale et les diaphragmes aplatis.
Vous évoquez alors le diagnostic de silicose.

Quel(s) argument(s) permet(tent) ce diagnostic ?


A - La densité des images radiologiques
B - La répartition des images réticulo-micronodulaires
C - Le caractère confluent de ces images
D - La localisation sous claviculaire des opacités ovulaires
E - L'absence de lésion pleurale radiologiquement marquée
Bonne(s) réponse(s) : B C D

Un épaississement pleural est un bon argument diagnostic.


Les opacités pseudotumorales sont habituellement localisées aux lobes supérieurs et bilatérales.

Quel(s) autre(s) examen(s) complémentaire(s) va (vont) permettre d'apprécier le handicap ?


A - Les épreuves fonctionnelles respiratoires
B - La fibroscopie bronchique
C - La biopsie transbronchique
D - La pleuroscopie
E - Les tomographies pulmonaires
Bonne(s) réponse(s) : A

La reconnaissance de la silicose comme maladie professionnelle permet :


- une indemnité de changement d'emploi s'il existe uniquement des troubles radiologiques sans troubles fonctionnels
respiratoires
- une rente d'I.P.P. s'il existe des troubles fonctionnels respiratoires.

1284
Exclusivement sur DOC - DZ : www.doc-dz.com NADJI 85
RESIDANAT EN POCHE TOME II
Cas Clinique en QCM

Quelle(s) complication(s) évolutive(s) légalement reconnue(s) risque(nt) de survenir en liaison avec la silicose ?
A - Cancer bronchique
B - Tuberculose pulmonaire
C - Aspergillose pulmonaire
D - Mésothéliome pleural
E - Pneumothorax
Bonne(s) réponse(s) : B C E

Les complications sont :


- insuffisance ventriculaire droite
- tuberculose ou autre mycobactériose
- nécrose cavitaire aseptique
- aspergillose intracavitaire
- pneumothorax spontané
- suppuration bronchopulmonaire
- insuffisance respiratoire aiguë.

Les arguments complémentaires ayant confirmé votre diagnostic, certaines démarches sont nécessaires pour
bénéficier du régime des maladies professionnelles. Laquelle (lesquelles) ?
A - Vous allez faire la déclaration de maladie professionnelle
B - Vous allez faire un certificat médical en triple exemplaire
C - Le patient va être examiné par un médecin expert en pneumoconiose
D - Le patient va être examiné par le collège des 3 médecins
E - Vous allez déclarer la maladie à la D.D.A.S.S.
Bonne(s) réponse(s) : B C

AB - Le médecin, qui a constaté l'affection, remet un certificat au malade qui fait lui-même la déclaration à la Caisse de
Sécurité Sociale.
C- Un médecin agréé en pneumoconiose est désigné, il vérifie le diagnostic.
D - Si les conditions habituelles (délai d'exposition au risque de 5 ans, délai de prise en charge de 15 ans) ne sont pas
réalisées, on a recours au collège des 3 médecins.

Quelle attitude allez-vous adopter quant à sa profession actuelle ?


A - Vous allez prévenir son employeur
B - Vous allez faire un certificat médical pour son employeur
C - Vous allez donner une lettre destinée au médecin du travail pour l'informer
D - Il est inapte à son emploi actuel et doit bénéficier d'une mutation
E - Son employeur actuel doit le garder car il s'agit d'une maladie professionnelle
Bonne(s) réponse(s) : C

Pas de commentaire.

La silicose est une pneumoconiose fibrogène. Certaines mesures ont permis d'en diminuer l'importance et la
fréquence. Laquelle (lesquelles) ?
A - L'automatisation de certains chantiers
B - L'interdiction de fumer sur les chantiers
C - La ventilation et l'apport d'air frais
D - La projection d'eau sur la roche et à travers certains outils
E - L'utilisation de la dynamite pour les tirs de mine
Bonne(s) réponse(s) : A D

Certaines mesures basées sur l'humidification permettent de diminuer l'empoussiérage.

1285
Exclusivement sur DOC - DZ : www.doc-dz.com NADJI 85
RESIDANAT EN POCHE TOME II
Cas Clinique en QCM
Un gardien de parking souterrain est vu en consultation de neurologie, pour des céphalées (la consommation tabagique est
de 10 cigarettes/jour) qui surviennent le soir, depuis quelques mois. Elles sont intenses, en casque, et ne sont pas pulsatiles.
L'examen ORL est strictement normal, mais le patient signale des nausées et des sensations vertigineuses les fins de
semaines. L'exploration hépatique et vésiculaire ne vous apporte aucun élément particulier.
Vous suspectez une origine professionnelle devant la disparition totale des symptômes les week- ends et aux congés.

Outre l'intoxication oxycarbonée, quelle(s) étiologie(s) devez-vous évoquer ?


A - Un diabète
B - Une tumeur intracrânienne
C - Une migraine accompagnée
D - Une céphalée psychogène
E - Une hypertension artérielle
Bonne(s) Réponse(s) : B D E

Evident.

Outre l'oxycarbonémie, parmi les examens suivants, le(s)quel(s) est(sont) nécessaire(s) au diagnostic
différentiel ?
A - Examen tomodensitométrique de l'encéphale
B - Radio des sinus
C - Examen du fond d'oeil
D - Ionogramme
E - Transaminases
Bonne(s) Réponse(s) : A B C

Le T.D.M. permet d'éliminer une pathologie tumorale intracrânienne.

Le bilan vous révèle une oxycarbonémie à 2 ml pour 100 ml, outre l'exposition professionnelle, quelle(s) en
est(sont) la(les) cause(s) ?
A - Un pré-diabète
B - Un tabagisme à 40
C - Une intoxication par chlorure de méthylène
D - Une insuffisance respiratoire chronique obstructive
E - Une polynévrite alcoolique
Bonne(s) Réponse(s) : B C D

L'oxycarbonémie est fonction du nombre de cigarettes fumées quotidiennement. On l'estime pour 20 cigarettes à 0,6 ml de
CO pour 100 ml de sang et pour 40 cigarettes à 1,2 ml de CO pour 100 ml de sang.
L'insuffisance respiratoire, l'hémolyse peuvent entraîner un taux d'oxycarbonémie jusqu'à 2 ml de CO pour 100 ml de sang.
- pour 40 cigarettes à 1,2 ml de CO pour 100 ml de sang :
L'exposition professionnelle au chlorure de méthylène peut entraîner une augmentation de la carboxyhémoglobinémie de 9%.
L'insuffisance respiratoire, l'hémolyse peuvent entraîner un taux d'oxycarbonémie jusqu'à 2 ml de CO pour 100 ml de sang.

Dans le but de faire cesser les céphalées, quelle(s) attitude(s) pouvez-vous préconiser ?
A - Prescrire des antalgiques types glafénine
B - Prescrire des vasoconstricteurs dérivés de l'ergot de seigle
C - Prescrire un arrêt temporaire de travail
D - Prescrire un arrêt du tabac
E - Prévenir le médecin du Travail pour obtenir une ventilation du local
Bonne(s) Réponse(s) : C D E

L'éviction de l'atmosphère toxique et le contrôle de la source d'intoxication sont indispensables.

Vous constatez, au vu du tableau n° 64 des maladies professionnelles indemnisables, que votre patient peut
bénéficier de cette réglementation. Vous allez :
A - Lui interdire définitivement de reprendre le travail
B - Faire une déclaration de maladie professionnelle
C - L'envoyer à son médecin du Travail
D - Faire un certificat médical descriptif en triple exemplaire
E - L'envoyer au médecin conseil de la Sécurité Sociale
Bonne(s) Réponse(s) : C D

B - C'est au salarié de faire la déclaration.

1286
Exclusivement sur DOC - DZ : www.doc-dz.com NADJI 85
RESIDANAT EN POCHE TOME II
Cas Clinique en QCM
Vous êtes appelé en urgence en novembre à 8 heures du matin, dans un appartement parisien, vous découvrez :
une femme de 25 ans, enceinte de 3 mois, comateuse. L'examen clinique immédiat met en évidence un coma profond avec
une hypertonie généralisée provoquée, un réflexe cutané plantaire droit en extension. La pupille droite est en position
intermédiaire non réactive à la lumière. La ventilation est normale. On note une écchymose sur la pommette droite et une
perte d'urines.
Le reste de l'examen est normal, en particulier, il n'y a ni encombrement bronchique, ni cyanose. Un cendrier plein de mégots
se trouve sur la table de nuit.
Le mari de la victime est conscient, il a vomi et se sent asthénique, l'examen clinique est normal.
Leur enfant de 5 ans présente des céphalées isolées (examen clinique normal). L'interrogatoire révèle que toute la famille
s'est couchée la veille à 21 heures et qu'aucun incident n'a été noté durant la nuit. C'est le mari qui vous a appelé.
Le diagnostic porté est celui d'intoxication aiguë au monoxyde de carbone (CO).

Ce diagnostic est spécialement fondé sur un ou plusieurs éléments, le(s)quel(s) ?


A - Le caractère collectif de l'intoxication
B - Le signe de Babinski droit
C - L'absence de convulsions chez les trois victimes
D - L'anomalie pupillaire chez la femme
E - L'absence de cyanose chez les trois victimes
Bonne(s) Réponse(s) : A

Sans commentaire

Connaissant les causes les plus fréquentes des intoxications au CO en France, pour confirmer votre diagnostic,
vous cherchez dans l'appartement une ou plusieurs des anomalies suivantes :
A - Un chauffe-eau à gaz vétuste
B - Une fuite sur l'installation du gaz de ville
C - Une fuite sur le robinet d'une bouteille de butane
D - Un calfeutrage des portes et fenêtres
E - Une fissure dans le conduit de cheminée
Bonne(s) Réponse(s) : A D E

Les principales sources d'oxyde de carbone sont :


- les appareils de chauffage défectueux
- les fissures dans les conduits de fumée
- la combustion d'un chauffe-eau en atmosphère confinée ou saturée de vapeur d'eau
- gaz d'échappement de moteurs à essence
- fumées dégagées par les incendies

Si vous accompagnez cette malade durant son transport par une ambulance médicalisée, quel(s) geste(s)
thérapeutique(s) assurerez-vous ?
A - Position latérale de sécurité
B - Pose d'une sonde gastrique
C - Administration d'oxygène au masque
D - Une perfusion de lactate de soude molaire
E - Perfusion de deux ampoules d'Isuprel® (isoprotérénol) dans 250 ml de soluté glucosé à 5 %
Bonne(s) réponse(s) : A C

A - Eviter l'inhalation bronchique chez cette patiente dans le coma.


C - L'apport massif d'oxygène permet la formation d'oxyhémoglobine.
E - N'est utile qu'en cas de collapsus cardiovasculaire.

A l'égard des autres membres de la famille, quelle est votre attitude ?


A - Hospitaliser le mari pour passage au caisson hyperbare et l'enfant pour mise en observation simple
B - Hospitaliser le mari et l'enfant pour oxygénothérapie
C - Hospitaliser le mari pour mise en observation simple
D - Hospitaliser ni le mari, ni l'enfant
E - Ne passer au caisson hyperbare le mari et l'enfant que s'ils présentent (à la prise de sang) un taux de
carboxyhémoglobine supérieur à 5 %
Bonne(s) réponse(s) : B

Les indications de l'oxygène hyperbare sont :


- les patients ayant un trouble de conscience à l'admission
- les patients ayant une anomalie objective à l'examen neurologique
- les patients ayant perdus connaissance
- les femmes enceintes.

1287
Exclusivement sur DOC - DZ : www.doc-dz.com NADJI 85
RESIDANAT EN POCHE TOME II
Cas Clinique en QCM

La mesure du pourcentage de carboxyhémoglobine, à l'arrivée à l'hôpital, dans le sang de la malade donnera la


certitude d'une intoxication à l'oxyde de carbone, si on trouve :
A-1%
B-4%
C-6%
D-8%
E - Plus de 15 %
Bonne(s) réponse(s) : E

On parle d'intoxication aiguë récente au CO pour un taux de 3 ml de CO/100 ml de sang (soit 15 % d'Hb-CO). Un taux
inférieur est insuffisant chez cette fumeuse.

On vous appelle sur un chantier voisin de votre cabinet pour donner des soins à un maçon qui s'est blesse en tombant de sa
hauteur. A l'examen vous trouvez quelques égratignures du visage qui ne nécessitent qu'une désinfection et un simple
pansement, mais vous constatez une odeur alcoolique de l'haleine, des trémulations des mains, des conjonctives injectées et
subictériques. L'interrogatoire vous apprend une consommation de l'ordre de deux litres de vin par jour, et un tabagisme à 25
cigarettes par jour.
Le sujet est épileptique, traité par 5cg de Gardenal® le matin et 10cg de Gardenal® le soir, mais il a arrêté depuis deux jours,
et la chute est survenue dans un contexte de crise tonico-clonique.

Cet accident peut-il être considéré comme un accident du travail ?


A - Oui car survenu sur les lieux du travail pendant le temps de travail
B - Oui car il y a eu intervention médicale
C - Non car le sujet est manifestement en état d'imprégnation éthylique
D - Non car le sujet est épileptique et qu'il ne suit pas son traitement
E - Non car il est alcoolique et ne suit pas son traitement, ce qui a conduit à la chute
Bonne(s) réponse(s) : A

"Est considéré comme accident du travail, quelle qu'en soit la cause, l'accident survenu par le fait ou à l'occasion du travail...".
Mais théoriquement l'employeur n'aurait pas dû accepter la présence sur le chantier de ce patient en état d'ivresse.

S'il y a eu déclaration d'accident de travail, le caractère professionnel de l'accident peut être contesté par :
A - Le médecin du travail
B - Le médecin inspecteur de la santé
C - Le médecin inspecteur du travail
D - L'employeur
E - L'inspecteur du travail
Bonne(s) réponse(s) : D

Connaissance.

Dans ce cas particulier, le chef de chantier a les documents administratifs nécessaires. Quel est votre rôle au
plan administratif ?
A - Vous devez remplir une feuille de Sécurité Sociale en plus des formulaires d'accident du travail (A.T.)
B - Vous devez prescrire un arrêt de travail pour que le régime de l'AT soit accordé
C - Vous gardez un volet du feuillet A.T. pour vos honoraires
D - Vous devez, après examen, rédiger un certificat médical décrivant les lésions observées
E - Vous vous faites payer par l'employeur
Bonne(s) réponse(s) : C D

Le praticien rédigera sur un imprimé spécial prévu à cet effet le certificat médical initial. L'un des exemplaires de ce certificat
doit être envoyé directement à la Sécurité Sociale dans les 48 heures, l'autre est remis à la victime.
Le médecin remplit les volets 1 et 2 remis par l'employeur à l'accidenté. Le volet 1 est remis à la victime.
Le volet 2 est conservé par le praticien pour se faire régler les honoraires directement par les Caisses.
Tous les soins sont gratuits pour la victime.

En raison du traumatisme crânien et d'une perte de connaissance initiale, vous avez jugé prudent de faire
hospitaliser le blessé. Il vient vous revoir à sa sortie pour sa reprise de travail.
Vous avez encore des formulaires à remplir. Le(s)quel(s) ?
A - Un certificat médical de reprise de travail
B - Un certificat d'aptitude
C - Un certificat final descriptif
D - Une lettre pour son employeur
E - Renvoyer à la caisse, le volet du triptyque concernant vos honoraires
Bonne(s) réponse(s) : C E

A - Sera effectué par le médecin du travail.


C - Lorsque les soins sont terminés et l'état du sujet considéré comme guéri ou consolidé par le praticien.

1288
Exclusivement sur DOC - DZ : www.doc-dz.com NADJI 85
RESIDANAT EN POCHE TOME II
Cas Clinique en QCM
Un homme de 73 ans est hospitalisé pour une altération de l'état général. L'examen découvre un épanchement pleural droit.
Après évacuation d'un liquide séro-hémorragique, on découvre une masse mamelonnée attenante à la paroi thoracique.

Quelle est l'hypothèse diagnostique ?


A - Masse pseudotumorale
B - Pleurésie tuberculeuse
C - Cancer bronchique
D - Mésothéliome pleural
E - Aucun des diagnostics ci dessus
Bonne(s) réponse(s) : D

Evident.

Quel(s) examen(s) va (vont) affirmer le diagnostic ?


A - Dosage de l'acide hyaluronique pleural
B - Dosage de l'acide hippurique pleural
C - Biopsie sous pleuroscopie
D - Fibroscopie bronchique
E - Bronchoscopie
Bonne(s) réponse(s) : A C

La teneur élevée de l'acide hyaluronique contrastant avec un taux normal d'antigène carcinoembryonnaire est un argument
important en faveur du diagnostic (il est constamment élevé dans certaines séries).
La pleuroscopie permet de différencier le mésothéliome d'une tumeur métastatique. Elle montre un groupement de nodules ou
des coulées tumorales diffuses au niveau de la plèvre pariétale.
On peut voir également des plaques hyalines ou des calcifications sur la plèvre pariétale antérieure dues à l'amiante. Elle
permet en dernier lieu d'effectuer des prélèvements histologiques (positifs dans 90 à 95 % à des cas).

Quel élément étiologique va-t-on rechercher dans l'anamnèse en regard du diagnostic ?


A - Recherche d'un cancer primitif
B - Consommation tabagique
C - Exposition au radon
D - Exposition à l'amiante
E - Antécédent de tuberculose
Bonne(s) réponse(s) : D

Un interrogatoire soigneux permet de retrouver une exposition à l'amiante dans 75 % des cas avec un délai moyen depuis le
début de l'exposition de 35 ans.

Quelle est la profession exposant au radon ?


A - Electroradiologie
B - Profession du bâtiment
C - Exploitation des carrières de calcaire
D - Exploitation des mines d'uranium
E - Exploitation des mines de bauxite
Bonne(s) réponse(s) : D

Les sujets travaillant dans les exploitations d'uranium sont exposés au radon (risque de cancer du poumon).

L'exposition à l'amiante caractérise la(les) profession(s) suivante(s) :


A - Chantiers de construction navale
B - Calorifugeage
C - Fabrication de garniture de freins
D - Chantiers de démolition de centrales thermiques
E - Electroradiologie
Bonne(s) réponse(s) : A B C D

Classique.

1289
Exclusivement sur DOC - DZ : www.doc-dz.com NADJI 85
RESIDANAT EN POCHE TOME II
Cas Clinique en QCM

Dans le cas où on retient pour ce patient une étiologie professionnelle, et pour que le malade soit indemnisé il
est nécessaire que :
A - Le délai de prise en charge soit respecté
B - La préexistence de lésions parenchymateuses soit établie
C - Le délai d'exposition au risque soit accepté
D - La profession exacte apparaisse dans la liste des travaux du tableau
E - Toutes les conditions précédentes soient réunies
Bonne(s) réponse(s) : A C

D - La liste des travaux est proposée à titre indicatif et non limitatif dans ce tableau.
Pour ce tableau un critère biologique est maintenant exigé : nombre de corps asbestosiques par gramme de tissus
pulmonaires lors d'un prélèvement anatomopathologique ou nombre de corps asbestosiques par ml de liquide au lavage
alvéolaire.

Au cours d'une visite systématique de médecine du travail, sont mises en évidence chez un maçon salarié de 30 ans des
lésions cutanées prurigineuses à type d'érythème et de vésicules au dos des mains et de la partie distale des avant-bras.
Dans le dossier médical, au cours des consultations antérieures, était notée la notion de lésions cutanées de même
localisation à type de fissures et d'érosion. Le diagnostic de dermite eczématiforme est posé.

Cette maladie, sous réserve du respect des conditions d'ouverture des droits, sera considérée par la Sécurité
Sociale, régime général, comme :
A - Une "maladie à caractère professionnel"
B - Un "accident de travail"
C - Une "maladie de longue durée"
D - Une "maladie professionnelle indemnisable"
E - Une "longue maladie"
Bonne(s) réponse(s) : D

Evident.

En cas de "maladie à caractère professionnel" qui doit faire la déclaration à l'inspection du travail ?
A - Le médecin traitant
B - L'employeur
C - Le médecin du travail
D - Le dermatologue
E - Le salarié
Bonne(s) réponse(s) : A C

Selon l'article L 500 du code de la Sécurité Sociale, tout médecin quel qu'il soit, doit déclarer à l'inspecteur du Travail, toute
maladie ayant un caractère professionnel dont il peut connaître l'existence.

En cas de "maladie professionnelle indemnisable" qui doit faire la déclaration à la caisse primaire d'assurance
maladie?
A - Le médecin traitant
B - L'employeur
C - Le médecin du travail
D - Le dermatologue
E - Le salarié
Bonne(s) réponse(s) : E

La victime établit une déclaration en triple exemplaire.


Cette déclaration doit mentionner les produits nocifs incriminés. Cette déclaration doit être déposée à la Caisse Primaire
d'Assurance Maladie et intervenir dans les 15 jours suivant la première constatation médicale.

En cas de maladie professionnelle indemnisable, les conditions de prise en charge par la Sécurité Sociale
Régime Général dans ce cas clinique nécessitent de :
A - Produire un certificat d'un médecin inspecteur du travail
B - Remplir les conditions requises par un tableau officiel
C - Etre salarié
D - Etre exposé au risque depuis plus de 10 ans
E - Ne pas être en arrêt de travail
Bonne(s) réponse(s) : B C

La durée d'exposition dépend de chaque tableau,


L'affection en cause doit être inscrite sur une liste officielle.
Chaque tableau est établi par décret en Conseil d'état sur rapport ministériel et après avis de la Commission d'hygiène
industrielle.

1290
Exclusivement sur DOC - DZ : www.doc-dz.com NADJI 85
RESIDANAT EN POCHE TOME II
Cas Clinique en QCM

Le médecin du travail face à ce cas clinique doit :


A - Prescrire un traitement
B - Signaler à l'employeur l'affection professionnelle
C - Envoyer ce patient à son médecin traitant
D - Signaler à l'employeur le risque incriminé
E - Envisager une action préventive au poste de travail
Bonne(s) réponse(s) : B C D E

Pas de commentaire.

Un mineur de fer, âgé de 60 ans, gros fumeur, présente des crachats hémoptoïques répétés. On lui trouve un hippocratisme
digital. On note sur la radiographie pulmonaire une miliaire fine et une opacité arrondie du lobe supérieur droit.

Le diagnostic le plus vraisemblable est :


A - Tuberculome
B - Abcès
C - Cancer
D - Aspergillome
E - Hamartome
Bonne(s) réponse(s) : C

Pas de commentaire.

En faveur de votre diagnostic, vous retenez :


A - Tabagisme
B - Hippocratisme digital
C - Facteur professionnel
D - Hémoptysies
E - Miliaire
Bonne(s) réponse(s) : A C D

L'incidence du cancer bronchopulmonaire est 3 à 5 fois celle de la population générale mais pour certains la responsabilité du
fer est incertaine.

Son bilan fonctionnel est le suivant : CV à 2,41, VEMS à 1,21 Tiffeneau à 50 %, CPT à 4,91 (théorique à 5,61),
paC02 à 32 mmHg (4,3 kPa), paO2 à 63 mmHg (8,4 kPa). Vous pouvez conclure à :
A - Syndrome restrictif pur
B - Syndrome obstructif pur
C - Syndrome ventilatoire mixte
D - Hyperventilation alvéolaire
E - Hypoventilation alvéolaire
Bonne(s) réponse(s) : C D

Il existe un trouble obstructif :


- avec diminution du V.E M.S. et du Tiffeneau et un trouble restrictif
- avec diminution de la C.P.T., de la C.V..
La diminution de la PaC02 évoque une hyperventilation.

Parmi les pneumoconioses dont le cancer bronchique représente une complication légale vous retenez:
A - Silicose
B - Sidérose
C - Stanniose
D - Bérylliose
E - Asbestose
Bonne(s) réponse(s) : A B E

C - Le stannose (et non stanniose) est due à l'étain. Il s'agit d'une pneumoconiose de surcharge non sclérogène.
D - La bérylliose est due à une exposition au silicate d'aluminium et de glucinium, et provoque des pneumopathies aiguës
toxiques oedémateuses.

1291
Exclusivement sur DOC - DZ : www.doc-dz.com NADJI 85
RESIDANAT EN POCHE TOME II
Cas Clinique en QCM

Les médecins qui peuvent effectuer la déclaration de maladie professionnelle sont :


A - Médecins traitants
B - Pneumologues consultants
C - Médecins experts (MAP)
D - Médecins du travail
E - Médecins conseil de la Caisse Minière
Bonne(s) réponse(s) : A B D

C - Une fois la déclaration de pneumoconiose effectuée, la Caisse de Sécurité Sociale transmet le dossier à l'inspecteur du
Travail qui procède à une enquête sur les conditions d'exposition au risque et saisit un médecin agréé, en matière de
pneumoconiose. Celui-ci informe ou confirme le diagnostic de pneumoconiose.

Monsieur DUPONT, 35 ans, marié, 2 enfants, grutier sur un chantier de montagne isolé, est victime d'un accident du travail et
présente une fracture du fémur imposant l'appel, par ses camarades de travail d'une ambulance privée, qui le conduit en
urgence au centre hospitalier le plus proche en service de chirurgie.
La fracture de Monsieur DUPONT a été traitée par un enclouage centro-médullaire. L'hospitalisation en traumatologie a duré 3
semaines et a été suivie d'un séjour de 4 mois en service de Rééducation Fonctionnelle.

Qui doit faire la déclaration d'accident du travail à la caisse de Sécurité Sociale ?


A - L'employeur
B - La victime
C - Le médecin du travail
D - L'infirmière du chantier
E - Le médecin traitant
Bonne(s) réponse(s) : A

Cette déclaration doit être effectuée dans les 48 heures.

L'ambulance privée qui a assuré le transport réclame le paiement des frais.


Qui doit prendre en charge ce règlement ?
A - L'employeur
B - Le Service social de l'entreprise
C - Le blessé lui-même
D - Une assurance privée
E - La caisse de Sécurité Sociale
Bonne(s) réponse(s) : C

Selon le principe du tiers payant, la caisse règle directement les honoraires, factures et frais à l'exception de ceux liés aux
déplacements dont le blessé doit s'acquitter et se faire rembourser ultérieurement.

Un certificat descriptif appelé certificat initial doit être rédigé pour garantir les droits de Monsieur DUPONT. Qui
doit rédiger ce certificat ?
A - Le médecin traitant habituel
B - Le médecin du travail
C - Un médecin expert
D - Le médecin conseil
E - Le médecin hospitalier
Bonne(s) réponse(s) : A

Evident.

A partir de quelle date Monsieur DUPONT perçoit-il les prestations en espèces versées par la caisse de
Sécurité Sociale ?
A - Du jour de l'accident
B - Du lendemain de l'accident
C - Du jour de l'intervention chirurgicale
D - Du jour de transfert en service de Rééducation Fonctionnelle
E - Du jour de sortie du service de Rééducation Fonctionnelle
Bonne(s) réponse(s) : B

La journée au cours de laquelle l'accident s'est produit est à la charge de l'employeur.

1292
Exclusivement sur DOC - DZ : www.doc-dz.com NADJI 85
RESIDANAT EN POCHE TOME II
Cas Clinique en QCM

Monsieur DUPONT, après sa sortie du service de Rééducation Fonctionnelle poursuit des soins physiques chez
un kinésithérapeute proche de son domicile. Son état de santé s'améliore. Qui pourra proposer une date de
consolidation ?
A - Le kinésithérapeute
B - Le médecin traitant
C - Le médecin du travail
D - Le médecin inspecteur régional du travail
E - Le médecin conseil
Bonne(s) réponse(s) : B

C'est au médecin traitant qu'appartient l'initiative de rédiger le certificat final descriptif

Monsieur P. F., né en 1920, à la retraite depuis 1980, vient consulter en 1985 dans votre cabinet pour une toux persistante
depuis six mois, une dyspnée discrète d'apparition progressive, une baisse de l'état général, une fébricule.
A l'examen clinique, Monsieur P. F., est amaigri, l'état général est moyen.
L'auscultation cardiaque est normale, ainsi que l'auscultation pulmonaire. Vous notez une hépatomégalie avec un discret
reflux hépatojugulaire et un oedème des membres inférieurs.
Monsieur P. F., n'a jamais fumé. L'étude de son curriculum laboris nous apprend qu'il a travaillé comme tailleur de pierre de
l'âge de 20 ans à l'âge de 45 ans.

L'histoire de la maladie est très évocatrice d'une silicose. Quel est l'argument principal ?
A - L'apparition progressive des signes cliniques
B - La toux
C - L'insuffisance cardiaque droite
D - La pauvreté de l'auscultation
E - Son métier de tailleur de pierre
Bonne(s) réponse(s) : E

Le diagnostic repose sur la radiographie pulmonaire et la notion d'exposition au risque.


Les signes fonctionnels et physiques respiratoires sont très souvent tardifs, et non spécifiques (en dehors de l'expectoration
noirâtre).

Pour asseoir ce diagnostic, vous vous entourez d'un certain nombre d'examens complémentaires. Quel est
l'examen indispensable pour confirmer le diagnostic de silicose ?
A - La tomographie pulmonaire
B - La radiographie pulmonaire standard
C - Les épreuves spirométriques
D - La mesure de la diffusion du CO
E - La recherche de silice libre dans le liquide de lavage broncho-alvéolaire
Bonne(s) réponse(s) : B

C'est l'étape capitale du diagnostic.

La baisse de l'état général, l'amaigrissement, la fébricule vous inquiètent. Quelle(s) complication(s) de la


silicose allez-vous rechercher ?
A - Une tuberculose pleuro-pulmonaire
B - Une aspergillose intra-cavitaire
C - Une sarcoïdose
D - Un cancer de la plèvre
E - Une plaque pleurale
Bonne(s) réponse(s) : A B

La silicose favorise la greffe de bacilles tuberculeux et de mycobactéries atypiques.


Devant ce tableau, on aurait pu également évoquer une surinfection bronchopulmonaire.

1293
Exclusivement sur DOC - DZ : www.doc-dz.com NADJI 85
RESIDANAT EN POCHE TOME II
Cas Clinique en QCM

La réparation de la silicose est subordonnée aux conditions qui figurent dans le tableau n°25 du Régime
Général de la Sécurité Sociale. Dans le cas présent, le diagnostic positif de la silicose étant établi, Monsieur P.
F., pourra-t-il prétendre à l'indemnisation ?
A - Oui, parce que Monsieur P. F., remplit les deux conditions d'exposition au risque et de délai de prise en
charge non dépassé
B - Non, parce que le délai de prise en charge était dépassé lorsque vous avez constaté la maladie
professionnelle
C - Non, parce que la profession qu'il a exercée n'est pas nommément désignée dans le tableau n° 25 et que
la liste des travaux est limitative
D - Non, parce que Mr. P. F., ne remplit pas la condition d'exposition professionnelle au risque et que le délai
de prise en charge était dépassé
E - Oui, parce qu'il existe des dispositions réglementaires particulières concernant la réparation de la silicose
Bonne(s) réponse(s) : E

Le délai d'exposition est de 5 ans en une ou plusieurs expositions séparées. Il est bien respecté dans cette observation.
Le délai de prise en charge (15 ans) est dépassé.
Dans ce cas, le collège des 3 médecins doit prendre en charge le dossier (disposition particulière concernant les
pneumoconioses).

Vous recevez en consultation un patient de 48 ans pour une dyspnée accompagnée de douleurs basithoraciques.
L'interrogatoire vous apprend que cette dyspnée s'est constituée progressivement en quelques années, aggravée par des
épisodes bronchitiques, et qu'il existe une intoxication tabagique à 10 cigarettes par jour. Il a travaillé au fond, dans une mine
de charbon du Pas-de-Calais, pendant 17 ans et, depuis 2 ans, il est conducteur d'engins dans une entreprise de travaux
publics. A l'examen, vous retrouvez des ronchus et des sibillants diffus, ainsi qu'une diminution du murmure vésiculaire aux
bases. Les bruits du coeur sont normaux.
La radiographie objective des opacités réticulo-micronodulaires et nodulaires confluentes, des opacités denses ovulaires de 4
à 5 cm dans les régions sous-claviculaires, des hyperclartés aux bases et une distorsion de l'arbre bronchique. L'ombre
cardiaque est normale et les diaphragmes aplatis.
Vous évoquez alors le diagnostic de silicose.

Quel(s) autre(s) examen(s) complémentaire(s) va (vont) permettre d'apprécier le handicap ?


A - La spirométrie
B - La fibroscopie bronchique
C - La biopsie transbronchique
D - Les gaz du sang
E - Les tomographies pulmonaires
Bonne(s) réponse(s) : A D

Les gaz du sang doivent être effectués au repos et à l'effort. L'exercice entraîne parfois une hypoxémie en cas de
pneumopathie interstitielle.

Quelle(s) complication(s) évolutive(s) légalement reconnue(s) risque(nt) de survenir en liaison avec la silicose ?
A - Cancer bronchique
B - Tuberculose pulmonaire
C - Aspergillose pulmonaire
D - Mésothéliome pleural
E - Pneumothorax
Bonne(s) réponse(s) : B C E

Q.C.M. extrêmement classique (et de bon goût).

Les différents arguments ayant confirmé votre diagnostic, certaines démarches sont nécessaires pour bénéficier
du régime des Maladies Professionnelles. Laquelle (lesquelles) ?
A - Vous allez faire la déclaration de maladie professionnelle
B - Vous allez faire un certificat médical en triple exemplaire
C - Le patient va être examiné par un médecin expert en pneumoconiose
D - Le patient va être examiné par le collège des 3 médecins
E - Vous allez déclarer la maladie à la D.D.A.S.S.
Bonne(s) réponse(s) : A B C

A - C'est le patient lui-même qui fait la déclaration.


B - Ce certificat indique la nature de la maladie, les manifestations observées (en particulier celles mentionnées sur les
tableaux) et les suites probables.
C - L'inspecteur du travail saisit un médecin agréé en matière de pneumoconiose qui infirme ou confirme le diagnostic.

1294
Exclusivement sur DOC - DZ : www.doc-dz.com NADJI 85
RESIDANAT EN POCHE TOME II
Cas Clinique en QCM

Quelle attitude allez-vous adopter quant à sa profession actuelle ?


A - Vous allez prévenir son employeur
B - Vous allez faire un certificat médical pour son employeur
C - Vous allez donner une lettre destinée au médecin du travail pour l'informer
D - Vous le déclarez inapte à son emploi actuel pour bénéficier d'une mutation
E - Vous faites un certificat contre-indiquant les efforts physiques
Bonne(s) réponse(s) : C

DE - Ne sont pas du ressort du médecin traitant.


AB - Rupture du secret professionnel.

La silicose est une pneumoconiose fibrogène. Certaines mesures ont permis d'en diminuer l'importance et la
fréquence. Laquelle (lesquelles) ?
A - L'automatisation de certains chantiers
B - L'interdiction de fumer sur les chantiers
C - La ventilation et l'apport d'air frais
D - La projection d'eau sur la roche et à travers certains outils
E - L'utilisation de la dynamite pour les tirs de mine
Bonne(s) réponse(s) : A C D

La lutte contre les poussières comprend en particulier :


- aspiration et ventilation
- port de masque
- humidification.

Monsieur X. a travaillé dans une fonderie de 1950 à 1957. Depuis cette date. il n'est plus exposé à des aérocontaminants
professionnels. Il fume un paquet de cigarettes par jour depuis 1940. En 1980, surviennent des hémoptysies. La radiographie
révèle des opacités micronodulaires dans les deux champs pulmonaires.

Quel(s) diagnostic(s) pouvez-vous évoquer :


A - Silicose
B - Tuberculose
C - Cancer bronchique
D - Sarcoïdose
E - Mésothéliome
Bonne(s) réponse(s) : A B C

Dans la métallurgie de nombreux travailleurs sont exposés (par exemple lors de la fabrication des moules de fonderie).
Les hémoptysies sont très rares et doivent faire rechercher une tuberculose ++ et bien sûr un cancer bronchopulmonaire chez
ce tabagique.

Quel(s) serait(ent) le (les) signe(s) radiologique(s) à rechercher en faveur d'une silicose ?


A - Opacités micronodulaires
B - Plaques pleurales calcifiées
C - Ganglions hilaires calcifiés
D - Pleurésie
E - Calcifications diaphragmatiques
Bonne(s) réponse(s) : A C

BDE - Evoquent une asbestose.

Quelle(s) complication(s) est (sont) souvent associée(s) à la silicose ?


A - Cancer bronchique
B - Tuberculose
C - Infection à mycobactérie atypique
D - Pneumothorax
E - Insuffisance cardiaque gauche
Bonne(s) réponse(s) : B C D

Connaissance.

1295
Exclusivement sur DOC - DZ : www.doc-dz.com NADJI 85
RESIDANAT EN POCHE TOME II
Cas Clinique en QCM

En cas de silicose, la déclaration au titre des maladies professionnelles indemnisables :


A - Est possible car la silicose figure dans la liste des affections professionnelles donnant lieu à un tableau
dans le régime de la Sécurité Sociale
B - Doit être faite par le salarié
C - Est adressée à l'inspection du travail
D - Donne lieu immédiatement au versement d'une somme forfaitaire
E - Permet la prise en charge totale des frais médicaux engagés
Bonne(s) réponse(s) : A B E

C - Elle est adressée à la C.P.A.M..


Celle-ci transmet un exemplaire de la déclaration et un exemplaire du certificat médical initial à l'inspecteur du Travail.

Une femme de trente ans, exerçant depuis 8 ans la profession d'infirmière en dermatologie, consulte pour une dermatose.
Elle n'a aucun antécédent notable en dehors d'une appendicectomie mais son père est diabétique.
L'interrogatoire permet de préciser qu'au début les lésions étaient localisées à certains doigts et aux espaces interdigitaux et
qu'elles étaient au départ vésiculeuses et suintantes L'affection a une évolution fluctuante avec dans l'année des périodes de
rémissions et de poussées. A l'examen il s'agit d'une éruption érythémateuse et prurigineuse, elle est discrètement
kératosique et l'on note quelques zones fissuraires au niveau des plis. Les ongles sont indemnes.

Quel diagnostic peut-on poser ?


A - Psoriasis
B - Mycose (dermatophytes)
C - Urticaire
D - Eczéma
E - Gale (scabiose)
Bonne(s) réponse(s) : D

Le diagnostic positif repose sur la reconnaissance de la lésion érythémato-vésiculeuse.

Quel(s) élément(s) peu(ven)t orienter vers une origine professionnelle ?


A - La manipulation récente d'un caustique
B - La profession réputée "exposée"
C - Le caractère prurigineux
D - L'existence de poussées à l'occasion du travail
E - La régression durant les congés annuels
Bonne(s) réponse(s) : A B C D E

A - Peut orienter vers une dermatose orthoergique.

Quelle(s) étiologie(s) peut on suspecter ?


A - L'alcool éthylique dénaturé
B - La pénicilline
C - Le permanganate de potassium
D - Les céphalosporines
E - Des colorants (fluorescéine-éosine)
Bonne(s) réponse(s) : B D

Commentaires :
Les professions médicales, en particulier les infirmières sont touchées par les eczémas professionnels : allergie au formol, au
glutaraldéhyde, aux gants de caoutchouc, aux antibiotiques (pénicillines, streptomycine, néomycine, kanamycine...), aux
antiseptiques (ammoniums quaternaires...), aux phénothiazines.

Après confirmation du diagnostic et de l'étiologie, une déclaration de maladie professionnelle est possible si :
A - Le diagnostic a été confirmé par un spécialiste
B - La maladie fait l'objet d'un tableau de maladie professionnelle
C - L'agent incriminé est repris dans le même tableau
D - Le médecin du travail a informé l'employeur du risque
E - La patiente manipule habituellement le produit responsable
Bonne(s) réponse(s) : B C

E - Est discutable, le patient doit être exposé au toxique, une déclaration est possible même si le patient ne manipule
qu'occasionnellement le produit responsable.

1296
Exclusivement sur DOC - DZ : www.doc-dz.com NADJI 85
RESIDANAT EN POCHE TOME II
Cas Clinique en QCM

Parmi les mesures préventives suivantes, vous devez préconiser ?


A - Une désensibilisation
B - La prise d'une douche après le travail
C - Le port de gants lors de la manipulation de certains produits
D - Une mutation dans un autre service hospitalier
E - Une dispense médicale pour la manipulation de produit en cause
Bonne(s) réponse(s) : C

L'éviction de l'allergène est l'élément primordial du traitement préventif.

Une jeune femme de 24 ans, mariée, vient consulter pour un psoriasis qui évolue depuis 10 mois. A l'interrogatoire, on
apprend qu'elle est au chômage depuis 1 an. Elle a une fille âgée de 3 ans, qui est en bonne santé.
Les lésions érythémato-squameuses, non prurigineuses, siègent d'une façon symétrique aux coudes et aux genoux. Le tronc
et la région sacrée sont indemnes. On note un aspect de dermite séborrhéique dans les sillons naso-géniens. Le cuir chevelu
est indemne mais elle se plaint d'avoir souvent des "péllicules" et les cheveux gras. Certains ongles présentent de petites
dépressions punctiformes. Pour son psoriasis, elle n'a eu qu'une seule injection intramusculaire de Kenacort® au tout début
de l'éruption. Enfin, elle prend parfois un anxiolytique le soir avant de se coucher.

En faveur d'un psoriasis la symptomatologie physique comporte :


A - Le siège des lésions aux coudes
B - Le respect de la région sacrée
C - Les pustules plantaires
D - Les lésions unguéales
E - La séborrhée du cuir chevelu
Bonne(s) réponse(s) : A C D

Des pustules plantaires peuvent s'observer dans le psoriasis en association avec des lésions de psoriasis vulgaires.

Chez cette jeune femme, à la place des plaques érythémato-squameuses, le psoriasis aurait pu se présenter
sous la forme :
A - D'un intertrigo sous-mammaire
B - D'arthropathies
C - D'une pustulose généralisée
D - D'une érythrodermie
E - D'une alopécie cicatricielle
Bonne(s) réponse(s) : A B C D

Le psoriasis du cuir chevelu n'est pas alopéciant. Le psoriasis inversé se manifeste par des intertrigos des grands plis où les
lésions perdent souvent leur caractère squameux en raison de la macération.

Chez cette jeune femme, le diagnostic de psoriasis est cliniquement évident. Quel(s) examen(s)
complémentaire(s) permet(tent) de le confirmer ?
A - Le groupe HLA
B - Une biopsie cutanée
C - Une recherche d'anticorps anti-épiderme
D - Un dosage de l'uricémie
E - Une capillaroscopie
Bonne(s) réponse(s) : B

L'histologie n'est pas indispensable devant un psoriasis cliniquement évident. Elle montrerait typiquement :
- une parakératose feuilletée
- une hyperacanthose
- une papillomatose
- des microabcès de Munro Saboureau
- une augmentation de l'index mitotique épidermique.

Pour traiter le psoriasis de cette jeune femme, quel(s) est(sont) le(s) traitement(s) que vous utilisez ?
A - Des kératolytiques
B - Un dérivé de l'acide rétinoïque per os (Tigason®)
C - Du Cortancyl® per os (0,1 mg/kg/j)
D - De la puvathérapie corps entier
E - Des dermocorticoïdes
Bonne(s) réponse(s) : A E

Devant une atteinte si localisée (BD) est envisageable. La corticothérapie générale doit être évitée dans le psoriasis cutané en
raison du risque de corticodépendance et de la transformation en psoriasis pustuleux.

1297
Exclusivement sur DOC - DZ : www.doc-dz.com NADJI 85
RESIDANAT EN POCHE TOME II
Cas Clinique en QCM

En définitive, le psoriasis est une maladie :


A - Atteignant environ 2 % de la population
B - Souvent héréditaire
C - Souvent congénitale
D - Respectant les muqueuses génitales
E - Parfois grave et invalidante
Bonne(s) réponse(s) : A B E

Des antécédents familiaux sont notés chez 30 % des malades. La proposition E se rapporte aux arthropathies, aux
érythrodermies et aux psoriasis pustuleux.

Un mineur de fer, âgé de 60 ans, gros fumeur, présente des crachats hémoptoïdes répétés. On lui trouve un hippocratisme
digital. On note sur la radiographie pulmonaire une miliaire fine et une opacité arrondie du lobe supérieur droit.

Votre diagnostic le plus vraisemblable pour expliquer l'opacité ronde intrapulmonaire est :
A - Tuberculome
B - Abcès
C - Cancer bronchique distal
D - Aspergillose
E - Hamartome
Bonne(s) réponse(s) : C

Par argument de fréquence chez un tabagique de 60 ans, mineur de fer.

Dans le contexte de ce malade, quelle étiologie attribuez-vous ?


A - Miliaire silicotique
B - Carcinomatose miliaire métastatique
C - Tuberculose miliaire
D - Miliaire sidérotique
E - Miliaire sarcoïdosique
Bonne(s) réponse(s) :

QUESTION ANNULEE
car plusieurs réponses auraient été possibles : D puis B C.

En faveur de votre diagnostic, vous retenez :


A - Tabagisme
B - Hippocratisme digital
C - Facteur professionnel
D - Hémoptysies
E - Miliaire
Bonne(s) réponse(s) : A C D

Puisque 42 annulée en faveur du diagnostic de cancer bronchique distal : tabagisme, profession, hémoptysies.

Le cancer bronchique représente une complication légale de :


A - Silicose
B - Sidérose
C - Stanniose
D - Bérylliose
E - Asbestose
Bonne(s) réponse(s) : B E

Légale c'est à dire reconnue par un tableau de Maladie Professionnelle.

1298
Exclusivement sur DOC - DZ : www.doc-dz.com NADJI 85
RESIDANAT EN POCHE TOME II
Cas Clinique en QCM

Les médecins qui peuvent effectuer la déclaration de maladie professionnelle sont :


A - Médecins traitants
B - Pneumologues consultants
C - Médecins experts (MAP)
D - Médecins du travail
E - Médecins conseils de la Caisse Minière
Bonne(s) réponse(s) :

QUESTION ANNULEE.
Ce n'est pas au médecin d'effectuer la déclaration d'une maladie professionnelle indemnisable mais au patient. Le médecin
effectue un certificat médical.

Mr G., 43 ans, est employé de mairie. Il présente depuis 3 mois de petites épistaxis de la fosse nasale gauche accompagnées
d'une obstruction nasale gauche. L'examen de la fosse nasale objective un aspect bourgeonnant au niveau du méat moyen
gauche. Il n'y a pas d'adénopathie. La radiographie pulmonaire est normale. Le diagnostic de tumeur de l'éthmoïde est
envisagé.

Quelle est la profession exposée que l'interrogatoire doit rechercher dans les antécédents du malade ?
A - Industrie du chrome
B - Industrie du chlore
C - Industrie de l'amiante
D - Industrie du bois
E - Exposition au charbon
Bonne(s) réponse(s) : D

Sans commentaire.

Si cette profession est bien en cause, quelle sera la réponse histologique de la biopsie ?
A - Epithélioma spinocellulaire
B - Epithélioma glandulaire
C - Tumeur nerveuse maligne des fosses nasales
D - Lymphome malin non hodgkinien
E - Mélanome malin
Bonne(s) réponse(s) : B

Sans commentaire.

Quel traitement doit être envisagé ?


A - Exérèse chirurgicale seule
B - Destruction au laser
C - Chimiothérapie par infusions intra-artérielles
D - Irradiation seule
E - Exérèse chirurgicale et irradiation
Bonne(s) réponse(s) : D

L'adénocarcinome de l'ethmoïde est typique des travailleurs du bois. Il existe une réparation en maladie professionnelle.

Quelle(s) extension(s) doit(vent) être recherchée(s) avant la mise en oeuvre du traitement ?


A - Sinus maxillaire
B - Etage antérieur et lobes frontaux
C - Orbite
D - Fosse ptérygo-maxillaire
E - Palais osseux
Bonne(s) réponse(s) :

QUESTION ANNULEE.

1299
Exclusivement sur DOC - DZ : www.doc-dz.com NADJI 85
RESIDANAT EN POCHE TOME II
Cas Clinique en QCM
Vous suivez en tant que médecin traitant, Mme X., 40 ans, épileptique. Les crises généralisées tonicocloniques sont peu
fréquentes (6 en 18 ans) et le plus souvent nocturnes. L'EEG ne montre aucune anomalie, l'étiologie est inconnue. Mme X. est
traitée actuellement par 1 Gardenal 10 cg et 2 Dépakine par jour. Elle travaille comme secrétaire dans une petite entreprise
proche de votre cabinet. A la fin d'une journée de travail, inhabituellement stressante, une crise a lieu dans le secrétariat où
travaillent d'autres personnes.

Le directeur de l'entreprise, ayant trouvé dans les papiers de l'intéressée votre nom et votre adresse, vous à fait
venir et vous demande des renseignements concernant Mme X. Quelle est ou quelles sont, la ou les
affirmations correctes ?
A - Vous lui faites part du diagnostic et de l'histoire de la maladie
B - Vous vous retranchez derrière le secret professionnel
C - Vous écrivez au médecin du travail pour qu'il modifie le traitement
D - Vous rédigez un certificat d'inaptitude temporaire
E - Vous donnez des soins à Mme X
Bonne(s) réponse(s) : B E

A - En aucun cas. Cf. réponse B


C - Non : le médecin du travail n'a pas de rôle curatif sauf urgence.
D - Non : le médecin du travail établit un certificat d'aptitude/inaptitude. Le médecin traitant peut faire un arrêt de travail (ce
que ne fait pas un médecin du travail).
E - En premier lieu.

Parmi les affirmations suivantes, laquelle ou lesquelles est (sont) exacte(s) ?


A - Les épileptiques ont un absentéisme plus important que les autres salariés
B - En milieu ordinaire de travail, les épileptiques ont deux fois plus d'accidents que les autres salariés
C - Plus de la moitié des épileptiques ont un poste de travail aménagé
D - L'épilepsie est légalement incompatible avec la conduite d'un poids lourd
E - Il peut légalement exister une compatibilité éventuelle entre une épilepsie et la conduite d'un véhicule léger
Bonne(s) réponse(s) : D E

D, E : L'arrêté du 24 mars 1981 fixant la liste des affections incompatibles avec la délivrance ou le maintien du permis de
conduire par groupes de véhicules :
- poids lourd : incompatibilité
- véhicule léger : compatibilité temporaire éventuelle fonction de la clinique, de la surveillance, du traitement et des résultats
du traitement.

Le médecin du travail ayant été informé par l'intéressée du diagnostic d'épilepsie :


A - Devra la déclarer inapte à tout poste de travail
B - Devra prendre l'avis du médecin inspecteur du travail
C - Devra suivre les directives que vous lui donnerez
D - Pourra vous communiquer ses conclusions médicales
E - Devra exercer vis-à-vis d'elle une surveillance médicale particulière
Bonne(s) réponse(s) : D E

A - Faux : surtout pas


Il faut confronter les données du poste et l'état de santé du salarié.
Certains postes dits de sécurité ou travail en hauteur sont à éviter.
Le médecin du travail doit étudier chaque situation au cas par cas, et faire des propositions pour
une éventuelle adaptation de poste.
Dans le cas clinique présenté, sont des éléments favorables :
- rareté des crises
- crises le plus souvent nocturnes
- traitement suivi
- poste occupé : secrétariat
. régularité des horaires
. absence de traumatismes sensoriels importants (bruit, luminosité intermittente...)
. durée de l'activité (et du sommeil).
De façon générale un épileptique a d'autant moins de risques de présenter une crise que son
attention est plus éveillée et qu'il est mieux protégé par le travail que par le repos.
B - Faux
Le médecin du travail peut prendre avis, sans aucune obligation. Pas de hiérarchisation
professionnelle entre les deux.
C - Faux
Pour les mêmes raisons que la réponse B : indépendance professionnelle du médecin du travail.
D - Vrai
Le médecin du travail peut communiquer ses conclusions médicales après accord du salarié.
E - Vrai/Faux
- Mais piège de l'intitulé sur l'obligation et la définition d'une surveillance médicale particulière.
- Ce terme en médecine du travail est codifié (code du travail) et s'applique aux salariés affectés
à certains travaux (liste fixée par arrêté du 11/07/1977) et/ou à certaines catégories de salariés
(femmes enceintes, handicapés...) dans lesquelles ne figure pas l'épilepsie.
- Cependant, à sa diligence, le médecin du travail peut décider d'un suivi médical particulier de
salariés présentant une pathologie.

1300
Exclusivement sur DOC - DZ : www.doc-dz.com NADJI 85
RESIDANAT EN POCHE TOME II
Cas Clinique en QCM

Lorsqu'un médecin du travail donne un avis d'inaptitude médicale à un poste de travail donné concernant un
salarié cet avis doit :
A - Etre motivé médicalement par écrit
B - Etre donné à l'employeur
C - Etre obligatoirement suivi d'un licenciement de la part de l'employeur
D - Etre donné après une étude du poste de travail
E - Etre adressé au médecin inspecteur de la D.R.A.S.S
Bonne(s) réponse(s) : B D

A - Faux
L'avis d'inaptitude doit être notifié par écrit et transmis à l'employeur sans aucune indication médicale.
Par ailleurs, cet avis concernant l'inaptitude à un poste précis devra indiquer les postes de travail compatibles avec l'état de
santé du salarié (qu'on ne précise pas) afin qu'une solution interne à l'établissement, toujours préférable, soit adoptée.
B - Vrai
C - Faux (cf. réponse A)
L'inaptitude à un poste précis ne signifie pas inaptitude à tous les autres postes de l'entreprise. Il convient de rechercher une
solution interne et de préconiser l'affectation à un poste ayant des caractéristiques compatibles avec l'état de santé du salarié.
D - Vrai
L'aptitude médicale d'un salarié à un poste (et l'inaptitude) naît de la confrontation des données fournies par l'étude du poste
concerné et l'examen du salarié. Déterminer l'aptitude c'est rechercher la meilleure adéquation individu - poste de travail donc
nécessité de connaître ces deux composantes.

Un salarié d'un garage d'automobiles, spécialisé dans la mise au point des moteurs au banc d'essais, se plaint de céphalées
tenaces rebelles aux antalgiques habituels, d'asthénie diffuse, de vertiges, de troubles mnésiques et de nausées. Cette
symptomatologie paraît régresser partiellement en période de congés que le sujet passe habituellement à la campagne.
L'interrogatoire met en évidence un tabagisme à 20 c/jour et une consommation habituelle de 1 litre de vin à 11 degrés par
jour, plus quelques apéritifs.

Pour poser le diagnostic d'oxycarbonisme chronique professionnel donnant droit à réparation, les données ci-
dessus :
A - Sont suffisantes et probantes
B - Doivent être complétées par un examen toxicologique
C - Doivent être complétées par deux examens toxicologiques
D - N'ont aucun rapport avec une intoxication oxycarbonée chronique
E - N'ont aucun rapport avec une intoxication professionnelle
Bonne(s) réponse(s) : C

Réparation tableau n°64 R6 - 40 RA

- Les signes cliniques sont évocateurs d'oxycarbonisme chronique.


- Le diagnostic d'oxycarbonisme chronique professionnel donnant droit à réparation repose sur (cf. tableau des maladies
professionnelles) :
. dosage du CO sanguin > 1,5 ml/100 ml de sang
. teneur en CO du local à hauteur des voies respiratoires > 50 cm3/m3.

En dehors de la source professionnelle éventuelle, quelle(s) cause(s) de dégagement d'oxyde de carbone est
(sont) à retenir parmi les suivantes :
A - Veilleuse de chauffe-eau à gaz allumé
B - Fuite de gaz de ville sans combustion
C - Poêle à mazout en fonctionnement
D - Fuite de bouteille de gaz butane sans combustion
E - Consommation de tabac
Bonne(s) réponse(s) : C E

B - Non : Co dégagé si combustion incomplète.


C - Si poêle à mazout défectueux.

Quel(s) examen(s) biologique(s) est (sont) indispensable(s) pour établir un diagnostic d'oxycarbonisme
chronique chez ce salarié :
A - Hémoglobinémie
B - Dosage de nicotinurie
C - Phosphatases alcalines leucocytaires
D - Oxycarbonémie
E - Aucun de ces examens
Bonne(s) réponse(s) : D

Sans commentaire.

1301
Exclusivement sur DOC - DZ : www.doc-dz.com NADJI 85
RESIDANAT EN POCHE TOME II
Cas Clinique en QCM

Quel(s) examen(s) est (sont) obligatoire(s) pour établir le diagnostic d'oxycarbonisme chronique professionnel ?
A - Oxycarbonémie
B - Gaz du sang
C - Dosages du CO dans l'atmosphère de travail
D - Dosages du CO au domicile du sujet
E - Aucun de ces examens
Bonne(s) réponse(s) : A C

A - A faire dans les 6 à 8 heures après sevrage du tabac.

Cf. réponses à la question [5]


C - Ces deux examens figurent dans le tableau des maladies professionnelles.

Vous voulez traiter spécifiquement cette intoxication, vous proposez :


A - Oxygénothérapie hyperbare
B - Vitaminothérapie B12 à haute dose
C - Eviction temporaire du poste de travail
D - Suppression du tabagisme
E - Toutes les propositions sont fausses
Bonne(s) réponse(s) : E

A - Non : c'est le traitement de l'intoxication aiguë.


D - Supprimera une des causes.
C - Peut être vrai pour traiter spécifiquement le salarié en ne l'exposant plus. Mais pour traiter l'intoxication, il faut supprimer la
cause c'est-à-dire l'émanation de CO dans le local associés aux moyens de prévention individuelle (masque). Si c'est
considéré comme vrai, le risque persiste avec la possibilité d'une nouvelle intoxication à la reprise.

Un ouvrier ébéniste de 40 ans est employé dans une fabrique de meubles d'art depuis l'âge de 15 ans. Il manipule des colles
contact, des vernis, des traitements du bois (xyloprotecteurs). Les bois sont aussi bien du chêne, du merisier, du cèdre rouge,
des acajous. Il vous consulte (en tant que médecin traitant) pour l'apparition d'une rhinorrhée séreuse récidivante depuis 6
mois, avec gêne respiratoire. Elle semble aggravée par l'exposition professionnelle. A l'interrogatoire, il
signale qu'il a présenté un rhume des foins dans l'enfance durant 2 printemps (vers l'âge de 10 ans). A l'examen clinique:
obstruction nasale sans déviation apparente de la cloison, présence de sibilances en fin d'expiration.

Parmi les produits manipulés par cet ouvrier. lequel(s) est (sont) susceptible(s) de donner les troubles clinique
allégués par le salarié ?
A - Solvants chlorés
B - Colles à bois vinyliques
C.- Poussière de chêne.
D - Poussière de bois exotique
E - Vernis polyuréthanes
Bonne(s) réponse(s) : C D E

C, D - La rhinite invoquée par le salarié apparaît dans le tableau 47 R6 : affections professionnelles provoquées par les bois.
E - Utilisation de vernis polyuréthanes responsables de la rhinite récidivante (n°62 R6 - 43 RA).

Le ponçage de panneaux d'acajou entraîne l'émission de fines poussières jugées responsables de :


A - Dermatose irritative des mains
B - Pneumoconiose
C - Cancer de l'ethmoïde
D - Alvéolite allergique extrinsèque
E - Asthme
Bonne(s) réponse(s) : A C D E

N°47 R6 - 36 RA

1302
Exclusivement sur DOC - DZ : www.doc-dz.com NADJI 85
RESIDANAT EN POCHE TOME II
Cas Clinique en QCM

Parmi les examens complémentaires suivants, lequel ou lesquels vous permet(tent) d'affirmer le caractère
professionnel de la rhinite ?
A - Rhinomanométrie antérieure et postérieure, avant et après exposition
B - Rhino-pléthysmographie occlusive
C - Test de provocation nasal réalisée
D - Capillaroscopie à la recherche d'une hyperhémie des cornets
E - Radiographie incidence Hirtz et Blondeau
Bonne(s) réponse(s) : A C

Sans commentaire.

Sur quel(s) critère(s) pourriez-vous envisager la responsabilité professionnelle des signes d'asthme ?
A - Crises survenant 15 minutes après le début du contact avec les produits incriminés
B - Crises survenant 6 heures après le début de l'exposition
C - Présence d'une hyperréactivité bronchique non spécifique lors d'un test avec un dérivé cholinergique
D - Test de suppression réadmission clinique positif
E - Chute de 5 % du VEMS entre le début et la fin de la journée de travail
Bonne(s) réponse(s) : B D

B - Dyspnée en fin de journée.


C - Fait la preuve de l'asthme mais pas de l'origine professionnelle. Les meilleurs tests pour affirmer l'origine professionnelle
sont les épreuves fonctionnelles de provocation (en milieu hospitalier) avec l'allergène suspect.
D - L'asthme professionnel ne se distingue pas de l'asthme ordinaire du point de vue sémiologique. C'est la chronologie des
accès qui en fait sa spécificité. Il disparaît si arrêt de travail (week-end, vacances) pour réapparaître à la reprise.
E - Non : il faut une chute de 20% du VEMS.

L'apparition d'un asthme aux poussières de bois à l'âge de 40 ans est favorisé par :
A - Père et grand père porteur d'un eczéma au ciment
B - Antécédents personnels de rhume des foins dans l'enfance
C - Consommation de 20 cigarettes par jour depuis 20 ans
D - Utilisation de cires et encaustiques
E - Utilisation d'anti-histaminique pour sa rhinite
Bonne(s) réponse(s) : B

Sans commentaire.

Véronique âgée de 20 ans, coiffeuse, consulte son médecin traitant pour un eczéma intéressant les faces dorsales et latérales
des doigts des deux mains. Un bilan allergologique pratiqué par un dermatologue de ville met en évidence une sensibilisation
au nickel, à la paraphénylènediamine ainsi qu'à l'acide thioglycolique (composant des liquides de permanentes). L'affection
est déclarée dans le cadre du tableau n° 15 des maladies professionnelles indemnisables (amines aromatiques et leurs
dérivés) et reconnue. Avant que n'intervienne cette reconnaissance, Véronique aura été déclarée inapte à tous les travaux de
la coiffure par son médecin du travail et licenciée par son employeur qui ne pouvait lui proposer aucun autre poste de travail.

Pour laquelle ou lesquelles des raisons suivantes, les dermatologues et les médecins du travail sont-ils d'accord
pour ne pas pratiquer de tests épicutanés chez les jeunes avant orientation professionnelle ?
A - Négativité des tests chez environ 70 % des sujets jeunes
B - Risque de sensibilisation induite
C - Possibilité d'un terrain atopique méconnu
D - Pas de caractère prédictif absolu
E - Fréquence des accidents locaux et généraux
Bonne(s) réponse(s) : B D

Sans commentaire.

1303
Exclusivement sur DOC - DZ : www.doc-dz.com NADJI 85
RESIDANAT EN POCHE TOME II
Cas Clinique en QCM

Parmi les règles suivantes laquelle ou lesquelles s'applique(nt) à la mise en oeuvre et à l'interprétation des
patch-tests ?
A - Doivent être pratiqués pendant les périodes de rémission
B - Ne peuvent être pratiqués qu'en milieu hospitalier
C - Doivent être lus 48 et 72 h après application
D - Sont positifs même lorsque n'existe qu'un des éléments caractéristiques de l'eczéma
E - Une intra-dermo doit être pratiquée en cas de patch-tests douteux
Bonne(s) réponse(s) : A C

A - Vrai
Le patch-test (dépôt sur la peau du réactogène supposé pour réaliser un eczéma en miniature) doit se pratiquer en dehors
d'une poussée aiguë de la maladie.
B - Faux
L'interprétation d'un patch-test nécessite une grande expérience de la part du testeur. Il est souvent pratiqué par les
dermatologues et pas nécessairement en milieu hospitalier.
C - Vrai
La durée d'application d'un patch-test est de 48 heures.
D - Faux
Un patch-test positif se caractérise par un petit placard prurigineux fait d'un érythème oedémateux se couvrant de fines
vésicules ou par une réaction érythémato-oedémateuse sans vésicules, avec prurit.
Une réaction purement érythémateuse sans prurit signale un test douteux.
E - Faux
Une intra-dermo peut être dangereuse avec certains réactogènes.

Laquelle ou lesquelles de ces caractéristiques peut ou peuvent s'appliquer aux eczémas allergiques
professionnels ?
A - Restent strictement localisés aux points de contact
B - Rôle du terrain individuel prépondérant
C - Peu fréquents
D - Reconnaissent toujours une étiologie chimique
E - Tendance à la régression lors des arrêts de travail
Bonne(s) réponse(s) : B E

A - Non : fréquence des lésions à distance appelées lésions secondes.


B - C'est ce qui différencie les dermatoses allergiques (dont l'eczéma) des dermatoses orthoergiques.
C - Non : c'est le contraire.
D - Non (origine végétale, fruits, arbres...).
E - C'est l'une des caractéristiques de l'origine professionnelle.

Dans le cas de la maladie professionnelle présentée par Véronique lequel ou lesquels des médecins suivants
ont-ils pu légalement rédiger le certificat initial ?
A - Son médecin traitant
B - Le dermatologue qui à effectué et interprété les tests
C - Son médecin du travail
D - Le médecin-conseil de la caisse de Sécurité Sociale
E - Un médecin expert désigné conjointement par le médecin traitant et le médecin-conseil
Bonne(s) réponse(s) : A B C

Sans commentaire.

Dans le cas de Véronique laquelle des personnes suivantes a-t-elle établi la déclaration qui doit accompagner le
certificat initial constatant la maladie professionnelle ?
A - Son médecin traitant
B - Le dermatologue qui à effectué et interprété les tests
C - Son employeur
D - Véronique elle-même
E - Son médecin du travail
Bonne(s) réponse(s) : D

C'est toujours la victime qui fait la déclaration d'une maladie professionnelle.

1304
Exclusivement sur DOC - DZ : www.doc-dz.com NADJI 85
RESIDANAT EN POCHE TOME II
Cas Clinique en QCM

Parmi les personnalités ou organismes suivants, laquelle ou lesquels peut (peuvent) légalement contester le
caractère professionnel ou apporter la preuve que l'affection présentée par Véronique n'était pas une maladie
professionnelle ?
A - Son employeur
B - Son médecin du travail
C - L'inspection du travail
D - Le médecin contrôleur par son employeur
E - Les services administratifs de la caisse de Sécurité Sociale
Bonne(s) réponse(s) : A E

Sans commentaire.

Sa maladie professionnelle ayant été reconnue, les prestations auxquelles Véronique pourrait avoir droit
incluent :
A - Prestations en nature
B - Indemnités journalières
C - Rente d'incapacité permanente
D - Indemnité de changement d'emploi
E - Droit au reclassement professionnel
Bonne(s) réponse(s) : A B C E

A, B, C - Comme pour toute maladie professionnelle reconnue.


E - Loi du 07/01/1981 : quand l'inaptitude fait suite à une maladie professionnelle ou un accident de travail (trajet exclu), le
reclassement est une obligation pour l'employeur. Si impossibilité, le salarié licencié bénéficie d'une protection légale plus
renforcée (indemnités supplémentaires).

Mme X. âgée de 40 ans, exerçant à domicile la profession de colleuse de talons et semelles pour
une entreprise de chaussures est suivie par son médecin traitant depuis quelques mois pour des
douleurs de la loge antéroexterne des deux jambes gênant la marche. Elle présente par ailleurs
des nausées et une grande lassitude. A l'examen clinique sont notés une hypoesthésie distale au
niveau des quatre membres avec troubles de la marche et une hyporéflectivité ostéotendineuse.
A l'électromyogramme: discrètes perturbations de l'excitabilité avec vitesse de conduction motrice
au niveau des deux nerfs sciatiques poplités externes, inférieure à la normale. Le diagnostic de
polynévrite est posé. L'analyse chimique de la colle utilisée révèle la présence d'hexane.

Tableau N° 59 du régime général


_________________________________________________________________
| Maladies engendrées | Délai de prise en charge | Liste indicative des |
| par l'hexane | | principaux travaux |
| | | susceptibles de |
| | | provoquer ces maladies |
|___________________|_______________________|______________________|
| Polynévrites, avec | 30 jours | Travaux de collage, |
| troubles des | | notamment sur cuir ou |
| réactions électriques | | matière plastique, des |
| | | produits contenant de |
| | | l'hexane |
|___________________|_______________________|______________________|

L'employeur doit légalement ou (réglementairement) :


A - Déclarer la maladie de Mme X à la caisse primaire de Sécurité Sociale
B - Avoir déclaré à la Sécurité Sociale l'usage par ses salariés de la colle incriminée
C - Avoir fourni au médecin du travail des renseignements concernant la colle utilisée
D - Avoir adressé en consultation hospitalière, Mme X au moment de son embauche
E - Adresser Mme X au médecin du travail au moment de la reprise de travail
Bonne(s) réponse(s) : B C E

A - Non : c'est la victime qui fait la déclaration de maladie professionnelle à la C.P.A.M.


B - Code de la Sécurité Sociale : obligation pour les employeurs de déclarer à la C.P.A.M. et à l'Inspection du Travail les
procédés de travail susceptibles d'entraîner une maladie professionnelle.
E - Une visite de reprise est obligatoire après une maladie professionnelle quelle que soit la durée de l'arrêt de travail (ainsi
qu'après un accident de travail).
Si l'arrêt de travail n'est pas lié à un accident du travail ou à une maladie professionnelle, une visite de reprise sera obligatoire
après un arrêt de travail de plus de 21 jours.

1305
Exclusivement sur DOC - DZ : www.doc-dz.com NADJI 85
RESIDANAT EN POCHE TOME II
Cas Clinique en QCM

Une consultation médicale de médecine du travail a pour but(s) :


A - De donner à l'employeur un avis concernant l'aptitude médicale au poste de travail
B - De contrôler le bien fondé d'un arrêt de travail
C - De rechercher une éventuelle pathologie d'origine professionnelle
D - De renseigner l'employeur sur l'état de santé général de son salarié
E - De répertorier les arrêts de travail pour maladie ou accident
Bonne(s) Réponse(s) : A C

A - Poste occupé par le salarié


B - En aucun cas : rôle du médecin de contrôle
D - En aucun cas : secret professionnel
E - Libellé "piège" : répertorier les arrêts de travail n'est pas un but en soi, mais analyser les causes de maladie ou d'accident
afin de mettre en oeuvre des moyens de prévention fait partie des missions d'un médecin du travail.

Compte tenu du délai de prise en charge du tableau n°59 :


A - La salariée devait avoir été exposée au risque pendant 30 jours
B - La maladie a du être déclarée à la Sécurité Sociale dans les 30 jours suivant l'arrêt de travail
C - La maladie a du être constatée dans les 30 jours qui ont suivi l'arrêt de l'exposition au tosique
D - La maladie a du être examinée par le médecin-conseil dans les 30 jours suivant la déclaration
E - Les indemnités journalières n'ont été versées qu'au trentième jour de la maladie
Bonne(s) Réponse(s) : C

C'est la définition du délai de prise en charge qui apparaît dans la colonne du milieu de tous les tableaux de maladie
professionnelle : délai maximum entre la fin de l'exposition au risque et l'apparition de l'affection.
Passé ce délai, l'imputabilité de la maladie au risque n'est plus admise.

L'(les) examen(s) complémentaire(s) au diagnostic différentiel est(sont) :


A - Recherche de l'activité de la gamma-glutamyl transférase
B - Dosage de la créatinémie
C - Détermination du volume globulaire moyen
D - Dosage de la glycémie à jeun
E - Détermination de la glucose-6 phosphate-déshydrogénase
Bonne(s) Réponse(s) : A C D

A - Témoin non spécifique de l'alcoolisme qui entraîne souvent une polynévrite des membres inférieurs
C - Cf. réponse A
D - Rercherche de diabète à l'origine d'une polynévrite des membres inférieurs.

Une rente de 10 % correspond à un taux d'incapacité permanente partielle de :


A - 10 %
B - 50 %
C - 20 %
D - 40 %
E-5%
Bonne(s) Réponse(s) : C

La rente est calculée par application au salaire de référecne du taux d'incapacité permanente partielle, corrigé
administrativement.
Cette correction consiste à diminuer de moitié la fraction du taux située au dessous de 50 % (et à augmenter d'autant celle
située au dessus).

L'intéressée ayant contesté le taux d'I.P.P. qui lui a été alloué, le litige est réglé en première instance par :
A - Le médecin-conseil de Sécurité Sociale
B - Un médecin expert en Sécurité Sociale
C - La commission régionale d'incapacité permanente
D - Le comité des rentes
E - La commission départementale de réforme
Bonne(s) Réponse(s) : C

Sans commentaire.

1306
Exclusivement sur DOC - DZ : www.doc-dz.com NADJI 85
RESIDANAT EN POCHE TOME II
Cas Clinique en QCM

Monsieur DURAND, ouvrier du bâtiment de 29 ans, marié. père de 2 enfants. membre du syndicat, est victime d'un accident
du travail le 30 août 1982 à 8 heures du matin sur le chantier. Le Chef de chantier informe l'employeur et déclenche les
secours. La victime est hospitalisée en urgence, opérée pour fracture du 1/3 moyen de jambe, par un orthopédiste qui
pratique une ostéosynthèse. Après un séjour de 3 semaines en milieu hospitalier, il est transféré en Centre de Rééducation où
il reste deux mois; puis il rentre à son domicile avec poursuite des soins. Depuis son embauche, il y a 5 ans, Monsieur
DURAND adhère à une Mutuelle complémentaire maladie et accident, pour lui et sa famille.

Qui doit déclarer dans les 48 heures l'accident du travail à la Caisse Primaire d'Assurance-Maladie ?
A - La victime et ayants-droits
B - Le médecin du travail
C - Le chirurgien orthopédiste
D - L'employeur
E - Le chef de chantier
Bonne(s) Réponse(s) : D

Obligation pour tout employeur de déclarer un accident du travail à la C.P.A.M. dans les 48 heures.

Le chirurgien orthopédiste rédige le certificat initial d'accident du travail. Ce certificat doit obligatoirement
comporter :
A - Le nom de la victime
B - Le nom de l'employeur
C - Le nom du médecin du travail
D - Le nom et la signature du chirurgien-rédacteur
E - La date de l'accident
Bonne(s) Réponse(s) : A B D E

B - Au sens large : raison sociale de l'établissement

Par qui est payée la journée du 30 août, jour de l'accident ?


A - Comité d'entreprise
B - Les oeuvres sociales de l'entreprise
C - La Caisse Primaire d'Assurance Maladie
D - L'employeur
E - Son syndicat
Bonne(s) Réponse(s) : D

C'est toujours l'employeur qui prend en charge le jour de l'accident du travail.

Quel organisme prend en charge les frais du Centre de Rééducation ?


A - L'employeur
B - La mutuelle complémentaire
C - La Caisse Primaire d'Assurance Maladie
D - La Caisse Régionale d'Assurance Maladie
E - Les oeuvres sociales de la ville de Monsieur DURAND
Bonne(s) Réponse(s) : C

Les soins de rééducation après accident du travail font partie des prestations en nature prises en charge par la C.P.A.M.

Le 10 janvier 1983, Monsieur DURAND se présente à l'entreprise pour reprendre son poste, avec un certificat
de reprise de travail de son médecin-traitant. Quelle condition supplémentaire doit être remplie ?
A - Accord de l'employeur
B - Accord du chef du personnel
C - Accord verbal du médecin du travail
D - Avis d'aptitude au poste délivré par le médecin du travail
E - Aucune condition supplémentaire
Bonne(s) Réponse(s) : D

Avis d'aptitude émis lors de la visite de reprise obligatoire après un accident du travail quelle que soit la durée de l'arrêt.

1307
Exclusivement sur DOC - DZ : www.doc-dz.com NADJI 85
RESIDANAT EN POCHE TOME II
Cas Clinique en QCM

1308
Exclusivement sur DOC - DZ : www.doc-dz.com NADJI 85
RESIDANAT EN POCHE TOME II
Cas Clinique en QCM

1309
Exclusivement sur DOC - DZ : www.doc-dz.com NADJI 85
RESIDANAT EN POCHE TOME II
Cas Clinique en QCM
Mademoiselle Z... 17 ans, mariée, travaille comme dactylographe dans une usine à 20 kilomètres de chez elle.
Elle vient consulter car elle est enceinte de 11 semaines. Sa tension artérielle est à 16-9.
Le reste de l'examen somatique est satisfaisant.

Si elle vous demande une interruption volontaire de grossesse (I V G), quelle est votre attitude ?
A - Vous lui demandez une autorisation d'un de ses parents
B - Vous demandez l'autorisation de son mari
C - Vous l'adressez à un service social pour un entretien
D - Vous lui indiquez que les délais sont dépassés
E - Vous lui faites confirmer sa demande par écrit
Bonne(s) réponse(s) : D

L'intitulé du cas clinique est litigieux puisque le terme n'est pas précis : 11 semaines de gestation ou 11 semaines
d'aménorrhée. "L'IVG n'est possible qu'avant la fin de la 10ème semaine de gestation", article
L 162-l du code de la Santé Publique.

Si elle souhaite poursuivre sa grossesse, à quel(s) organisme(s) doit-elle en faire la déclaration ?


A - La mairie
B - La D.D.A.S.S
C - La médecine du travail
D - La caisse d'allocation familiale
E - Son entreprise
Bonne(s) réponse(s) : D

La déclaration de grossesse est adressée à la caisse d'allocation familiale.

Avant quel délai doit-elle officiellement déclarer sa grossesse pour bénéficier des prestations ?
A - 12 semaines
B - 15 "
C - 18 "
D - 21 "
E - 24 "
Bonne(s) réponse(s) : B

La déclaration de grossesse doit s'effectuer avant la fin du 3ème mois, soit 13 semaines de grossesse ou 15 semaines
d'aménorhée.
C'est la date de début présumé de la grossesse qui est mentionnée sur cette déclaration.

Quel(s) élément(s) de son dossier est(ou) sont facteur(s) d'accouchement prématuré ?


A - Son âge
B - La tension artérielle
C - Le trajet jusqu'au lieu de travail
D - Le fait de travailler
E - Le type de travail
Bonne(s) réponse(s) : A B C

Les facteurs de risque généraux d'accouchement prématuré sont :


- tabac
- transport long
- certaines professions avec travail physique pénible ou station debout prolongée
- âge < 19 ans.

De quelle(s) prestation(s) pourra-t-elle bénéficier ?


A - Allocations familiales de grossesse
B - Indemnités journalières au cours du congé pré et postnatal
C - Surveillance gratuite de sa grossesse en P.M.I.
D - Gratuité des transports publics
E - Allocation du fond national de solidarité
Bonne(s) réponse(s) : B C

L'assurance maternité comporte :


1 - des prestations en nature
- examens de surveillance obligatoires
- un forfait lors de l'accouchement
- examen médical postnatal dans les 8 semaines suivant la naissance de l'enfant
2 - des prestations en espèces après déclaration de la grossesse
- indemnités journalières au cours du congé pré et postnatal
- allocations prénatales.

1310
Exclusivement sur DOC - DZ : www.doc-dz.com NADJI 85
RESIDANAT EN POCHE TOME II
Cas Clinique en QCM
Vous venez de diagnostiquer une grossesse chez une jeune femme immigrée portugaise de 17 ans, séparée de sa famille.

Un ou plusieurs des éléments du dossier interdit (sent) légalement l'interruption volontaire de grossesse. Lequel
(lesquels) ?
A - Elle réside en France depuis 2 ans
B - Son âge
C - Elle n'a pas d'enfant
D - Elle a fait il y a moins de 18 mois une fausse couche provoquée
E - Elle est enceinte de 12 semaines
Bonne(s) réponse(s) : E

L'IVG légale ne peut-être pratiquée que jusqu'à la 10ème semaine de grossesse.(ici item litigieux car il n'est pas précisé s'il
s'agit de semaines de grossesse ou de semaines d'aménorrhée ; dans ce dernier cas, l'IVG serait encore possible).
Les femmes étrangères bénéficient également de l'IVG légale si elles résident depuis plus de 3 mois en France (sauf pour les
réfugiées politiques pour lesquelles aucun délai n'est prévu). Les mineures doivent avoir le consentement de l'un des parents,
outre leur consentement donné en absence d'une tierce personne.

Vous lui indiquez les examens prénataux auxquels elle doit obligatoirement se soumettre. Ils doivent être
réalisés :
A - Avant 3 mois, 6 mois, 9 mois
B - Avant 3 mois, 8 mois, 9 mois
C - Avant 3 mois, 6 mois, 7 mois, 8 mois
D - Avant 3 mois, 4 mois, 6 mois. 8 mois
E - Avant 3 mois. 6 mois, 8 mois, 9 mois
Bonne(s) réponse(s) : E

Sans commentaire.

Vous lui expliquez que le versement des allocations prénatales est soumis à certaines conditions Laquelle (ou
lesquelles) des propositions ci-dessous fait (font) partie de ces conditions :
A - La future mère doit travailler
B - La future mère doit se soumettre aux examens prénataux obligatoires
C - La future mère doit respecter le congé prénatal
D - Ses ressources doivent être inférieures à un certain plafond de revenu
E - Elle doit être de nationalité française
Bonne(s) réponse(s) : B

B - Les 4 examens de surveillance sont obligatoires pour faire valoir les droits aux allocations prénatales.
C - Le congé prénatal est un droit et non une obligation.

Vous l'adressez pour le suivi de la grossesse à l'équipe de protection maternelle et infantile du quartier. Depuis
la loi de décentralisation, ce service est sous la responsabilité :
A - Du département par l'intermédiaire de la direction départementale des affaires sanitaires et sociales
B - Des services extérieurs du ministère de la santé par l'intermédiaire de la direction départementale des
affaires sanitaires et sociales
C - Des services de pédiatrie du C.H U
D - De la direction régionale des affaires sanitaires et sociales
E - Des caisses d'allocations familiales
Bonne(s) réponse(s) : A

Sans commentaire.

1311
Exclusivement sur DOC - DZ : www.doc-dz.com NADJI 85
RESIDANAT EN POCHE TOME II
Cas Clinique en QCM
Un homme de 48 ans est hospitalisé pour des crampes nocturnes et une atteinte de l'état général : amaigrissement de 4 kilos
en trois mois, anorexie importante, fatigabilité.
Cette personne reconnait fumer 2 paquets de cigarettes par jour et consomme quotidiennement 2 litres de vin à 12°, 1,5 litre
de bière à 6°.
Ce malade prenait pour seul traitement de l'Equanil® (2 comprimés par jour) en raison de troubles du caractère (irritabilité
familiale) et d'insomnie.

Quelle quantité journalière d'alcool pur (en g) absorbe ce malade ?


A - 125 g
B - 156 g
C - 264 g
D - 276 g
E - 300 g
Bonne(s) réponse(s) : C

Le degré alcoolique est le pourcentage d'alcool en volume contenu dans un liquide : 1 litre à 10° contient 100 ml d'alcool pur.
La densité de l'alcool est de 0,8 g/ml.
Donc : 2 litres à 12° contiennent 240 ml d'alcool pur, 1,5 litre à 6° contiennent 90 ml d'alcool pur.
Il y a donc 330 ml x 0,8 g/ml = 264 g.

Chez ce malade, quel est l'apport calorique quotidien lié à la consommation d'alcool ?
A - 792 Kcal
B - 1320 Kcal
C - 1584 Kcal
D - 1848 Kcal
E - 2310 Kcal
Bonne(s) réponse(s) : D

1 g d'alcool pur représente 7 calories.

Il se dit incapable de diminuer cette consommation malgré les tentatives d'arrêt. Il allègue une perte de l'appétit,
également une sensation de malaise général et un tremblement des mains présents surtout le matin et
s'estompant après ingestion d'alcool. Mis à part un visage congestionné et des dilatations capillaires localisées
aux pommettes et aux ailes du nez, l'examen clinique ne montre pas d'anomalie
Sur quel(s) élément(s) pouvez-vous affirmer que ce malade est physiquement dépendant à l'alcool ?
A - Consommation excessive d'alcool (supérieure à 120g par jour)
B - Incapacité de diminuer sa consommation d'alcool
C - Visage congestionné et dilatations capillaires localisées aux pommettes et aux ailes du nez
D - Malaise et/ou tremblement des mains le matin s'estompant après ingestion d 'alcool
E - Crampes nocturnes
Bonne(s) réponse(s) : D

D - La dépendance physique est l'apparition d'un syndrome physique d'abstinence lorsque cesse la consommation.
A - L'importance de la consommation n'est pas corrélée à la dépendance physique.
B - Dépendance psychique.
C - Signes d'imprégnation.
E - Signes évocateurs de polynévrite alcoolique.

Le bilan hépatique montre.les anomalies suivantes : transaminases 65 Ul (normale = 50) gamma GT 275 Ul
(normale = 45).
Quel(s) est (sont) I'(les) élément(s) qui pourrait(ent) contribuer à l'élévation de l'activité plasmatique de la
gamma GT observée chez malade ?
A - Une hépatopathie alcoolique
B - Une alcoolisation chronique
C - Une pathologie osseuse
D - Une stéatose
E - La prise d'Equanil®
Bonne(s) réponse(s) : A B D

Toute pathologie hépatique est susceptible de provoquer une élévation des gamma GT ; il s'agit donc d'un paramètre
biologique peu spécifique. L'Equanil® n'est pas hépatotoxique.

1312
Exclusivement sur DOC - DZ : www.doc-dz.com NADJI 85
RESIDANAT EN POCHE TOME II
Cas Clinique en QCM

L'ensemble des symptômes de ce malade peut très bien s'expliquer par la consommation excessive d'alcool.
Néanmoins ce sujet est à haut risque de certaines affections devant être systématiquement recherchées.
Laquelle (lesquelles) ?
A Cancer du larynx

B - Cancer de l'oesophage
C - Bronchopneumopathie chronique
D - Cancer du poumon
E - Cancer de la vessie
Bonne(s) réponse(s) : A B C D

La consommation excessive d'alcool est un facteur favorisant des cancers des voies aérodigestives supérieures.
L'intoxication tabagique augmente le risque de cancer du poumon, de bronchopneumopathie chronique, de cancer du larynx
et de cancer de vessie. Dans ce dernier cas, le dépistage systématique n'est pas pratiqué car le risque reste malgré tout faible.

Parmi les troubles subjectifs suivants, accusés par le patient, l'alcoolisme chronique peut-être responsable de :
A Crampes nocturnes
B - Fatigabilité
C - Anorexie
D - Troubles du caractère à type d'irritabilité
E - Insomnie
Bonne(s) réponse(s) : A B C D E

Ce sont des signes d'imprégnation alcoolique.

Après la sortie de l'hôpital où il à suivi une cure de sevrage, quelle(s) est (sont) la (les) structure(s)
spécialisée(s) pouvant contribuer à sa prise en charge ?
A - Centres d'hygiène alimentaire
B - Foyers de post-cure
C - Services de moyen séjour
D - Mouvements d'anciens buveurs
E - Services de soins à domicile
Bonne(s) réponse(s) : A B D

Des résultats positifs sont variables allant de 25 à 55% des cas.

Médecin hospitalier, vous avez suivi depuis plusieurs années monsieur X... pour une hydronéphrose du rein droit objectivée
par plusieurs urographies intraveineuses. Vous lui conseillez de se faire opérer par le Docteur Z.... chirurgien à l'Hôpital. Vous
assistez à l'intervention chirurgicale. Quelques jours après, le chirurgien vous téléphone car il vient de s'apercevoir qu'il a
enlevé le rein gauche sain, selon le compte-rendu du laboratoire d'anatomie-pathologique.

Le chirurgien et vous-même, devez prévenir :


A - Le directeur de l'hôpital
B - Le procureur de la République
C - Le Président du conseil de l'Ordre
D - Le président du tribunal administratif
E - Ou devez-vous garder le secret
Bonne(s) réponse(s) : A

La responsabilité de l'administration d'un hôpital est engagée du fait de l'action de ses agents publics. Mais quelque soit la
responsabilité de l'hôpital, le médecin hospitalier est toujours soumis à sa responsabilité pénale éventuelle qui est personnelle.

Votre assurance personnelle prévenue par vos soins, vous demande un résumé d'observation. Cochez la(les)
information(s) que vous pouvez transmettre à votre assurance :
A - Les examens radiologiques
B - L'examen anatomo-pathologique du rein gauche
C - Les antécédents du malade, autres que ceux intéressant le motif de l'intervention
D - La profession du malade
E - Aucune information
Bonne(s) réponse(s) : E

Le secret médical est absolu vis à vis de l'assurance.

1313
Exclusivement sur DOC - DZ : www.doc-dz.com NADJI 85
RESIDANAT EN POCHE TOME II
Cas Clinique en QCM

La malade, prévenu de l'erreur, porte plainte au pénal


Vous recevez d'un juge d'instruction une convocation à titre de témoin :
A - Vous devez répondre à cette convocation
B - Vous pouvez,.dans le cadre de votre défense, faire état de faits médicaux
C - La présence de votre avocat est obligatoire
D - Un membre de l'Ordre des médecins doit assiter à cet interrogatoire
E - Vous pouvez vous faire accompagner d'un confrère
Bonne(s) réponse(s) : A

Le médecin témoin doit toujours se rendre aux convocations des magistrats, il n'est pas délié du secret professionnel vis à vis
du magistrat. Etant témoin, il n'a pas à se défendre. Le secret de l'instruction s'oppose aux items D.E.

Le juge d'instruction estime nécessaire une perquisition dans votre service pour saisir votre dossier Vous
pouvez vous opposer à cette perquisition si elle est effectuée :
A - Par le greffier du tribunal
B - Par un huissier
C - Par le juge accompagné d'un membre du conseil de l'Ordre
D - Par le directeur de l'hôpital
E - Aucune de ces propositions n'est exacte
Bonne(s) réponse(s) : C

La saisie de documents médicaux exige une décision du procureur de la République ou du juge d'instruction. Elle doit être
effectuée en présence d'un membre du conseil départemental de l'Ordre des Médecins. Donner un dossier médical en dehors
d'une procédure légale de saisie est une violation condamnable du secret professionnel.

Interné au pavillon d'urgence, un homme, monsieur Z... blessé par balles, se présente à vous. Il déclare avoir été victime d'un
jeune voisin, âgé de 15 ans, qui a tiré sur lui, au cours d'une dispute. La balle a traversé la cuisse, sans léser les vaisseaux,
mais il existe des troubles sensitifs et moteurs, qui vous font craindre une atteinte du nerf sciatique. Vous hospitalisez ce
malade.

La loi vous oblige à :


A - Garder le secret
B - Prévenir la police
C - Prévenir la gendarmerie
D - Prévenir le procureur de la République
E - Prévenir le directeur de l'hôpital
Bonne(s) réponse(s) : A

Le secret professionnel est, sauf dérogation, légal et absolu. Les dérogations du secret comportent les déclarations de faits
tombant sous le coup de l'article 63 du code pénal : "sera puni d'un emprisonnement ou d'une amende celui qui, ayant
connaissance d'un crime déjà tenté ou consommé, n'aura pas averti les autorités administratives ou judiciaires". Dans le cas
clinique, on ne rentre pas dans ce cadre.

Quelques heures après, un officier de police se présente au pavillon d'urgence pour demander si un blessé par
balles a été hospitalisé.
Qui peut répondre à cet officier de police, pour lui donner l'identité du blessé ?
A - L'interne
B - Le médecin de garde
C - Le directeur de l'hôpital
D - Le chef de service, ou son représentant dûment mandaté
E - Personne ne peut donner l'information
Bonne(s) réponse(s) : E

De même pour cette question, le secret professionnel est absolu.

Personne n'ayant voulu prendre la responsabilité de répondre, l'officier de police revient le lendemain. porteur
d'une réquisition, en vue d'examiner monsieur Z, de décrire ses blessures et de fixer la durée d'ITT.
Cette réquisition doit s'adresser :
A - Au directeur de l'hôpital.
B - A l'interne
C - Au chirurgien qui a opéré le blessé.
D - A un médecin de l'hôpital qui n'a pas eu à soigner le blessé
E - A un médecin inscrit sur la liste des experts près la cour d'appel
Bonne(s) réponse(s) : D E

Tout docteur en médecine est tenu de déférer aux réquisitions de l'autorité publique ; le médecin requis agit alors en qualité
d'expert-auxiliaire de la justice. Il est tenu de déférer sous peine d'amende, aux réquisitions et commissions. Seuls les
médecins ayant donné des soins aux malades, peuvent refuser la réquisition.
1314
Exclusivement sur DOC - DZ : www.doc-dz.com NADJI 85
RESIDANAT EN POCHE TOME II
Cas Clinique en QCM

Dans les jours suivants, le blessé vous demande d'adresser un résumé d'observation
Quelle est, ou quelles(sont) la(les) personne(s) susceptibles de recevoir un tel résumé d'observation ?
A - Son employeur
B - Le docteur S... que le blessé vous désigne comme son médecin-traitant
C - Son avocat
D - Le médecin-conseil de sa compagnie d'assurance
E - Le juge des enfants
Bonne(s) réponse(s) : B

Article 28 de la loi du 31.12.70 : "les établissements d'hospitalisation publics sont tenus de communiquer le dossier des
malades hospitalisés, ou reçus en consultation externe, au médecin appelé à donner des soins à ces malades". Le médecin
traitant est délié du secret professionnel vis à vis des autres médecins traitants du malade.

Freddy, 29 ans, sans domicile fixe et travaillant occasionnellement, vient à votre cabinet : il souffre à la fois de fatigue, d'un
manque d'élan, d'irritabilité, d'énervement, mais aussi de tremblements, de myalgies, d'insomnie. Le pouls et la tension sont
normaux, pas de fièvre, une perte de poids de 7 kg .une mydriase sans signe de Babinski, un hypocondre droit douloureux, un
subictère conjonctival et de nombreux points d'injection.Votre patient avoue alors être toxicomane, avoir fait de la prison pour
vol avec effraction de pharmacie il y a deux ans, et être actuellement recherché par la police pour petit trafic. Il consomme
surtout des opiacés, mais aussi des barbituriques (sécobarbital) Dans ses antécédents de nombreuses tentatives de suicide,
instabilité socio-professionnelle et sentimentale, toxoplasmose il y a 4 mois. Freddy voudrait arrêter la drogue, mais demande
"quelque chose" pour ses douleurs et un certificat médical destiné à éviter les poursuites judiciaires.

En fonction des éléments en votre possession, vous vous orientez peut-être vers l'idée d'un épisode somatique
intercurrent chez ce patient.
Lequel (lesquels) ?
A - Syndrome de sevrage aux opiacés
B - Hépatite virale
C - Délirium trémens
D - Affection digestive chirurgicale
E - Simulation
Bonne(s) réponse(s) : A B

Le tableau clinique décrit d'une part un syndrome de sevrage aux opaciés :


"asthénie, irritabilité, énervement, myalgies, insomnie, mydriase, tremblement", ainsi qu'un tableau évocateur d'hépatite virale :
"traces de points d'injection, amaigrissement, asthénie, subictère conjonctival, hypochondre droit douloureux, myalgies".

En fonction des éléments en votre possession, quel diagnostic psychopathologique vous paraît le plus probable
?
A - Névrose hystérique de conversion
B - Psychose schizophrénique
C - Déséquilibre psychique (ou psychopathie)
D - Dépression d'allure mélancolique
E - Hypocondrie névrotique
Bonne(s) réponse(s) : C

Il s'agit d'un tableau de psychopathe qui se caractérise par une instabilité, une impulsivité et une inadaptation gênant
l'insertion socio-professionnelle.

Freddy vous a demandé quelque chose pour ses douleurs.


Que lui donneriez-vous ?
A - Un opiacé du type antalgique majeur
B - Un antalgique
C - Un anti spasmodique
D - Un psychostimulant
E - Un sédatif
Bonne(s) réponse(s) : B C E

Le traitement d'un syndrome de sevrage aux opaciés comporte :


- anxiolytiques ou sédatifs
- antalgiques à type d'anti-spasmodiques
- catapressan®.

1315
Exclusivement sur DOC - DZ : www.doc-dz.com NADJI 85
RESIDANAT EN POCHE TOME II
Cas Clinique en QCM

En fonction du contexte défini plus haut, quelle vous parait-être la meilleure attitude possible parmi les suivantes
?
A - Hospitaliser Freddy en hôpital psychiatrique
B - Hospitalisation pour bilan somatique et psycho social
C - Sevrage dégressif des opiacés à votre cabinet
D - Psychothérapie de soutien
E - Déconditionnement par thérapie comportementale
Bonne(s) réponse(s) : B

Le traitement du sevrage nécessite une hospitalisation en milieu médical (voire éventuellement en milieu psychiatrique à
condition de traiter également les problèmes somatiques), de même pour le bilan de l'hépatite virale, on n'utilise jamais de
sevrage dégressif, ni de surveillance en ambulatoire. La simple psychothérapie est insuffisante.
L'item E n'est pas adapté à ce problème.

En fonction des éléments dont vous disposez, comment pourrait s'appliquer la loi de 1970 sur les toxicomanies
chez Freddy ?
A - Vous pouvez signaler Freddy à la D.D.A.S.S. qui le prendra en charge
B - Vous rédigez un certificat demandant la suspension des poursuites judiciaires
C - Votre patient sera automatiquement protégé contre les poursuites pour trafic
D - Votre patient, sous certaines conditions, pourrait éviter les poursuites pour usage personnel de stupéfiants
E - Votre patient peut être hospitalisé en gardant l'anonymat
Bonne(s) réponse(s) : A D E

A - Toute personne ayant fait usage de stupéfiants est placée sous surveillance de l'autorité sanitaire.
B.C.D - L'utilisation de drogue et le trafic de drogue sont des délits entraînant des poursuites judiciaires.
En ce qui concerne les usagés, les poursuites seront suspendues par le procureur de la République, si ces personnes se sont
conformées au traitement médical prescrit et l'auront poursuivi jusqu'à son terme. En cas de récidive, les personnes sont
également passibles de peines mais là encore le procureur de la République pourra apprécier s'il convient ou non d'exercer
cette action.
E - Si le patient se présente spontanément au service sanitaire, aucun contrôle ne lui est imposé, aucune poursuite n'est
ordonnée et l'anonymat lui est assuré.

Parmi les propositions suivantes concernant le traitement du syndrome de sevrage aux opiacés,
laquelle(lesquelles) est(sont) exacte(s) ?
A - Prescription de doses dégressives
B - Suppression immédiate et totale
C - Prescription d'antidotes type Nalorphine®
D - Administration de neuroleptiques sédatifs
E - Administration de tranquillisants
Bonne(s) réponse(s) : B E

B - La suppression de la drogue est toujours complète et brutale.


On n'utilise jamais de sevrage dégressif.
C.D.E - Voir question [27].

Monsieur Durand, ouvrier du bâtiment de 29 ans, marié, père de 2 enfants. membre du syndicat WBC, est victime d'un
accident du travail le 30 août 1982, à 8 heures du matin, sur le chantier. Le chef de chantier informe l'employeur et déclenche
les secours. La victime est hospitalisée en urgence, opérée pour fracture du 1/3 moyen de jambe, par un orthopédiste qui
pratique une ostéosynthèse. Après un séjour de 3 semaines en milieu hospitalier, il est transféré en centre de rééducation ou
il reste deux mois, puis il rentre à son domicile avec poursuite des soins. Depuis son embauche, il y a 5 ans, monsieur Durand
adhère à une mutuelle complémentaire maladie et accident pour lui et sa famille.

Qui doit déclarer dans les 48 heures l'accident du travail à la caisse primaire d'assurance-maladie ?
A - La victime et ayants-droits
B - Le médecin du travail
C - Le chirurgien orthopédiste
D - L'employeur
E - Le chef de chantier
Bonne(s) réponse(s) : D

L'employeur doit déclarer l'AT dans les 48 heures à la CPAM.

1316
Exclusivement sur DOC - DZ : www.doc-dz.com NADJI 85
RESIDANAT EN POCHE TOME II
Cas Clinique en QCM

Le chirurgien orthopédiste rédige le certificat initial d'accident du travail. Ce certificat doit obligatoirement
comporter :
A - Le nom de la victime
B - Le nom de l'employeur
C - Le nom du médecin du travail
D - Le nom et la signature du chirurgien-rédacteur
E - La date de l'accident
Bonne(s) réponse(s) : A D E

Ce certificat doit comporter :


- l'identification du médecin de la victime
- la date de l'accident
- la description soigneuse des blessures
- l'appréciation des suites éventuelles ainsi que la durée probable de l'incapacité permanente
- la prescription éventuelle d'un arrêt de travail.

Par qui est payée la journée du 30 août jour de l'accident ?


A - Comité d'entreprise
B - Les oeuvres sociales de l'entreprise
C - La caisse primaire d'assurance maladie
D - L'employeur
E - Le syndicat WBC
Bonne(s) réponse(s) : D

Le jour de l'accident du travail est dû par l'employeur.


Les indemnités sont accordées dès le premier jour qui suit l'arrêt du travail sans distinction de jours ouvrables.

Quel organisme prend en charge les frais du centre de rééducation ?


A - L'employeur
B - La mutuelle complémentaire
C - La caisse primaire d'assurance maladie
D - La Caisse Régionale d'Assurance Maladie
E - Les oeuvres sociales de la ville de Monsieur DURAND
Bonne(s) réponse(s) : C

Il s'agit d'une prestation en nature. Toutes ces prestations sont gratuites.

Le 10 janvier 1983, monsieur Durand se présente à l'entreprise pour reprendre son poste. avec un certificat de
reprise de travail de son médecin-traitant.
Quelle condition supplémentaire doit être remplie ?
A - Accord de l'employeur
B - Accord du chef du personnel
C - Accord verbal du médecin du travail
D - Avis d'aptitude au poste délivré par le médecin du travail
E - Aucune condition supplémentaire
Bonne(s) réponse(s) : D

Cette visite de reprise doit avoir lieu le jour même de la reprise. L'avis d'aptitude doit être formulé par écrit à l'employeur ou à
son préposé (le chef du personnel).

Six mois après l'accident, monsieur Durand est consolidé avec son accord et celui du médecin-traitant.
Il conteste par contre le taux d'incapacité permanente partielle qui lui est proposé.
A qui va être soumis ce différend ?
A - Médecin-conseil de la caisse primaire d'assurance maladie
B - Chirurgien orthopédiste qui a opéré monsieur Durand
C - Commission technique régionale
D - Commission grâcieuse préalable
E - Commission de première instance de la sécurité sociale
Bonne(s) réponse(s) : C

D - Lors de la contestation d'une décision administrative, la victime peut réclamer l'avis de la commission gracieuse préalable
qui est un bureau de conciliation formé d'administrateurs.
La commission de première instance est présidée par un magistrat, assisté d'un représentant ouvrier et d'un représentant
patronal.

1317
Exclusivement sur DOC - DZ : www.doc-dz.com NADJI 85
RESIDANAT EN POCHE TOME II
Cas Clinique en QCM
Un jeune homme de 25 ans se présente au service d'urgence amené par des amis pour un tableau associant des douleurs
musculaires, des frissons, des vomissements, de la diarrhée, une asthénie intense et des tremblements.
A l'examen, il présente un foie un peu augmenté de volume, une température à 38°2, des polyadénopathies, un souffle à
l'auscultation cardiaque et de multiples traces d'injections aux plis des coudes. Il demande à être hospitalisé et à recevoir des
soins de façon anonyme, mais ne bénéficie pas de couverture sociale. Il déclare prendre de façon occasionnelle du haschich,
du LSD et des amphétamines et de manière quasi quotidienne, en alternance de l'Imménoctal® avec de l'alcool et de
l'héroïne. Ses dernières prises remontent à une vingtaine d'heures et il demande qu'on lui donne de manière dégressive du
Palfium® pour parvenir à s'en passer.

Quel(s) diagnostic(s) évoquez-vous devant le tableau présenté ?


A - Un pré-délirium trémens
B - Un syndrome de sèvrage aux drogues toxiques
C - Une conversion hystérique
D - Un syndrome de surdosage
E - Une endocardite infectieuse
Bonne(s) réponse(s) : B E

Le syndrome de sevrage aux opiacés débute quelques heures après la dernière prise, ainsi que le syndrome de sevrage aux
barbituritiques rapides.
Le tableau clinique est évocateur. (Le tableau clinique n'évoque pas un pré-délirium trémens qui comporte une confusion
mentale avec onirisme). On doit systématiquement suspecter une endocardite infectieuse chez un sujet présentant un souffle
cardiaque et un train fébrile, ce d'autant qu'il est toxicomane.

Quel(s) est (sont) le(s) produit(s) parmi ceux utilisés par cet homme qui est (sont) susceptible(s) d'occasionner
une dépendance physique ?
A - Le haschich
B - Le LSD
C - Les amphétamines
D - L'Immenoctal®
E - L'héroïne
Bonne(s) réponse(s) : A C D E

Le LSD ne détermine pas de dépendance physique.


Le haschich et les amphétamines sont susceptibles d'entrainer une dépendance physique modérée.
Les barbituriques et surtout les opiacés entrainent une dépendance physique majeure.

Quel(s) est (sont) la (les) loi(s) qui permet(tent) d'assurer les soins à ce patient anonymement en l'absence de
toute couverture sociale ?
A - La loi du 30 juin 1838
B - La loi du 15 avril 1954
C - La loi du 31 décembre 1970
D - La loi du 3 janvier 1968
E - La loi du 17 janvier 1975
Bonne(s) réponse(s) : C

La loi du 31 décembre 1970 tente de faire une différence entre trafiquants et utilisateurs de drogues, alourdissant les peines
des premiers et prévoyant une alternative de type thérapeutique pour les seconds. En particulier, si l'intoxiqué se présente
spontanément pour son traitement, l'anonymat lui est assuré. La prise en charge de la cure et de la postcure est gratuite.

Parmi ces produits et leur voie d'administration, quel(s) est (sont) celui
(ceux) que vous utiliseriez ?
A - Palfium I M, à doses dégressives
B - Tranxène® per os
C - Bêta-bloquant
D - Gardénal® per os à doses dégressives
E - Viscéralgine® per os
Bonne(s) réponse(s) : B D E

Le traitement du sevrage des opiacés comporte la prescription d'anxiolytiques, d'antalgiques, antispasmodiques, du


Catapressan®.
Le sevrage est complet et brutal.
En cas d'intoxication aux barbituriques, le sevrage est progressif en utilisant le phénobarbital en association au Valium®.

1318
Exclusivement sur DOC - DZ : www.doc-dz.com NADJI 85
RESIDANAT EN POCHE TOME II
Cas Clinique en QCM

Parmi les propositions suivantes concernant le syndrome déficitaire de la période de sevrage,


laquelle(lesquelles) est (sont) correcte(s) ?
A - Inhibition psychomotrice
B - Apragmatisme
C - Syndrome dépressif
D - Fugue
E - Aucune des propositions ci-dessus
Bonne(s) réponse(s) : A B C

Le syndrome déficitaire est une conséquence commune de toutes les intoxications marquées et qui est particulièrement
ressenti par le sujet après l'arrêt de l'intoxication.
Il associe un ralentissement idéatoire, un flou de la pensée, une asthénie, un apragmatisme ainsi qu'un syndrome dépressif.

Parmi les propositions suivantes concernant le traitement de l'intoxication aiguë aux amphétamines.
Laquelle(lesquelles) est(sont) correcte(s) ?
A - Neuroleptiques sédatifs
B - Antidépresseurs tricycliques
C - Antidépresseurs I.M.A.O.
D - Normothymiques
E - Sels de lithium
Bonne(s) réponse(s) : A

Dans les intoxications aiguës aux amphétamines, on traite les différentes complications de façon symptomatique :
- les troubles psychiatriques sont traités par des neuroleptiques par voie parentérale
- en cas de convulsions, on utilise du diazépam
- en cas de troubles cardiovasculaires, on utilise de la phentolamine, de l'alphaméthyldopa, des anti-arythmiques.

Parmi les propositions suivantes concernant le traitement du syndrome déficitaire de la période de sevrage aux
drogues sédatives.
Laquelle(lesquelles) est(sont) correcte(s) ?
A - Anticomitiaux
B - Antalgiques
C - Antidépresseurs tricycliques
D - Myorelaxants
E - Antitrémulants
Bonne(s) réponse(s) : C

Le syndrome déficitaire du sevrage doit être traité par des antidépresseurs ou des neuroleptiques désinhibiteurs type
Dogmatil® ou Prazinil®, car il est cause de rechutes.

Un homme de 38 ans consulte pour asthénie : l'examen clinique montre un poids stable de 51 kg pour 162 cm : la pression
artérielle est de 130/80 mmHg, le foie est hypertrophie avec une flèche hépatique de 13 cm sur la ligne médio-claviculaire. Le
reste de l'examen est normal. L'interrogatoire retrouve la notion d'une consommation journalière de 1,5 l de vin rouge à 10
degrés. L'enquête alimentaire note la prise de 2200 kilocalories/jour (sans l'alcool) dont 13 p.cent de protéines, 44 p.cent de
lipides et 43 p.cent de glucides (12 p.cent de saccharose). Les examens sanguins donnent les résultats suivants : glucose = 7
mmol/l, sodium = 129 mmol/l, triglycérides = 50 mmol/l, cholestérol = 17 mmol/l, gamma GT = 1491 U/I (N inférieure à 40),
ASAT = 92 U/l (N inférieure à 40), ALAT = 66 U/l ( N inférieure à 50).

La consommation d'alcool de ce patient correspond en grammes d'alcool pur à :


A - 80
B - 100
C - 120
D - 150
E - 180
Bonne(s) réponse(s) : C

Le degré d'alcoolisation d'un liquide est le pourcentage d'alcool pur qu'il contient :
1 litre à 10° contient 10 cl ou 100ml d'alcool pur.
La densité de l'alcool est de 0,8 g/ml.

1319
Exclusivement sur DOC - DZ : www.doc-dz.com NADJI 85
RESIDANAT EN POCHE TOME II
Cas Clinique en QCM

Selon la classification internationale, l'hyperlipoprotéinémie de ce patient est probablement de type :


A-I
B - IIb
C - III
D - IV
E-V
Bonne(s) réponse(s) : E

On pourrait également discuter une hyperlipidémie de type IV mais l'exogénose nous oriente vers le type V.

L'hyponatrémie de ce patient est liée à :


A - Des apports sodés insuffisants
B - Une rétention hydrique
C - Une rétention hydrosodée
D - L'hypertriglycéridémie
E - Des pertes sodées digestives
Bonne(s) réponse(s) : D

Il s'agit d'une pseudohyponatrémie. Le volume de plasma occupé par les lipides est anormalement élevé et la part d'eau
plasmatique est réduite. Le sodium n'est présent que dans la phase aqueuse. La concentration en sodium par litre d'eau
plasmatique reste inchangée et l'osmolalité n'est pas modifiée, par contre la natrémie est diminuée. Le diagnostic est évoqué
sur l'aspect lactescent du sérum et les anomalies biologiques associées (taux de lipides supérieurs à 30 g/l). On le confirme
par mesure directe de l'osmolalité plasmatique par cryoscopie.

La correction de l'hypertriglycéridémie de ce patient peut être obtenue par :


A - La suppression de l'alcool
B - La suppression des glucides d'absorption lente
C - La suppression des graisses saturées
D - L'apport de graisses polyinsaturées
E - La prescription d'un médicament hypolipémiant
Bonne(s) réponse(s) : A

E - Ne dispense pas du régime qui reste l'élément primordial du traitement.

Le(s) risque(s) que peut(peuvent) faire courir à ce patient la consommation excessive d'alcool est (sont) :
A - Cancer de l'oesophage
B - Cancer des voies aéro-digestives supérieures
C - Cancer de l'estomac
D - Pancréatite
E - Cardiomyopathie
Bonne(s) réponse(s) : A B D E

L'alcool et le tabac n'ont pas un rôle pathogénique formellement établi.


E - Il s'agit de cardiomyopathies non obstructives.

Vous suivez Mr X... pour une épilepsie essentielle traitée par les hydantoïnes. Depuis quelques mois, il ne se sent pas en
forme et a constaté une baisse d'efficience professionnelle et une recrudescence de ses crises d'épilepsie.
L'interrogatoire et l'examen clinique vous font évoquer une alcoolisation chronique.

M. X... vous révèle une consommation d'un litre de vin à 10 degrés par jour.
A combien estimez-vous sa consommation d'alcool pur par jour ?
A - Plus de 120 grammes
B - 110 à 120 grammes
C - 90 à 100 grammes
D - 70 à 80 grammes
E - 50 à 70 grammes
Bonne(s) réponse(s) : D

Le degré d'une boisson est le pourcentage (en volume) d'alcool pur contenu dans celle-ci : 1 litre à 10° contient 10% d'alcool
pur soit 100 ml d'alcool pur. La densité de l'alcool est de 0,8 g/ml.

1320
Exclusivement sur DOC - DZ : www.doc-dz.com NADJI 85
RESIDANAT EN POCHE TOME II
Cas Clinique en QCM

Si vous utilisez la grille de Le Go, quel(s) signe(s) clinique(s) présente(s) par M. X... allez-vous y faire figurer ?
A - Angiomes stellaires
B - Trémulation des doigts
C - Insomnie
D - Varicosités du visage
E - Hypertension artérielle
Bonne(s) réponse(s) : C D E

La grille de Le Go comporte les signes cliniques suivants :


- aspect : visage, conjonctives, langue
- troubles subjectifs: nerveux, digestifs, moteurs
- tremblements : bouche, extrémités
- autres données : foie, poids, tension artérielle.

M. X... vous montre un bilan biologique récent.


Quel est(sont) le(s) signe(s) évocateur(s) d'une alcoolisation chronique ?
A - Cholestérol total : 1,8 g/l
B - Volume globulaire moyen : 102 microns cubes
C - Uricémie : 40 mg/l
D - Augmentation des gamma globulines sériques
E - Crétinine : 12 mq/l
Bonne(s) réponse(s) : B

Les stigmates biologiques évocateurs d'alcoolisme chronique sont une macrocytose, l'élévation des gamma-glutamyl
transférases, une hyperlipidémie de type IV.

Vous conseillez un sevrage à Mr X... et vous lui prescrivez des vitamines B et une benzodiazépine à faible
dose. Quinze jours plus tard, la gamma glutamyl transférase (GGT) est toujours élevée.
Quelle(s) hypothèse(s) évoquer ?
A - La prise d'hydantoïnes
B - La persistance de l'alcoolisation
C - L'hypertention artérielle
D - Une atteinte hépatique
E - La prise de Vitamine B
Bonne(s) réponse(s) : A B D

L'élévation des gamma.GT est retrouvée dans 70 à 90% des cas d'alcoolisme chronique. Ce taux se normalise après 2 à 4
semaines de sevrage.
Cependant cette élévation n'est pas spécifique :
- toute atteinte hépatique peut s'accompagner d'une telle élévation
- prise médicamenteuse tels que barbituriques, hydantoïnes, etc...

Médecin de garde, vous êtes appelé au domicile d'un enfant de 2 ans, qui présente une température à 40 degrés.
Cet enfant est habituellement suivi par un de vos confrères. Vous diagnostiquez une angine, mais au cours de l'examen
général. vous êtes surpris de découvrir des hématomes sur le revêtement cutané. De plus, cet enfant parle peu et vous
semble craintif. Interrogés sur l'origine de ces hématomes, les parents accusent la turbulence de l'enfant, très intrépide
d'après eux et toujours couvert de plaies et de bosses. Les hématomes siègent sur le dos, à la pommette droite, sur le bord
interne des deux avant-bras.

Vous souhaitez hospitaliser l'enfant pour des examens complémentaires. Les parents refusent cette
hospitalisation.
A qui pouvez-vous faire appel, pour imposer cette mesure :
A - Assistante sociale du quartier
B - Directeur de la DDASS
C - Juge des enfants
D - Président du conseil de l'Ordre
E - Procureur de la République
Bonne(s) réponse(s) : C

Ce tableau clinique fait suspecter des sévices à enfant. Seul le juge des enfants peut retirer provisoirement ou définitivement
l'enfant de son milieu familial.

1321
Exclusivement sur DOC - DZ : www.doc-dz.com NADJI 85
RESIDANAT EN POCHE TOME II
Cas Clinique en QCM

L'hospitalisation ayant été obtenue, la loi vous autorise à faire état de vos soupçons :
A - Au médecin-inspecteur de la D.D.A.S.S.
B - Au confrère, médecin-traitant.
C - A l'assistante sociale du secteur
D - Au médecin chef de la caisse régionale de sécurité sociale
E - Aux grands-parents
Bonne(s) réponse(s) : A B C

Le médecin est délié du secret professionnel lorsqu'il est confronté à une suspicion de sévices à enfant (loi du 15.6.71), il
signale l'enfant à l'autorité sanitaire (A,C).
D'autre part, les médecins traitants du même patient sont déliés du secret médical entre eux (B).

Lors de l'hospitalisation, les sévices sont confirmés. Les parents sont poursuivis et le magistrat chargé de
l'affaire, souhaite vous entendre ; il vous convoque à son cabinet :
A - Vous devez vous rendre à cette convocation
B - Vous pouvez faire état de vos constatations médicales
C - Les parents doivent donner leur accord à votre audition
D - Un membre du conseil de l'Ordre doit vous accompagner
E - Aucune de ces propositions n'est exacte
Bonne(s) réponse(s) : A B

Toute personne convoquée comme témoin est tenue de se rendre à la convocation du magistrat. Le médecin bénéficie d'une
dérogation facultative au secret médical en cas de sévices à enfants.

Un médecin expert a été nommé par le juge pour examiner l'enfant, confirmer les sévices et préciser s'il persiste
des séquelles. Il vous téléphone pour obtenir des renseignements.
A - Vous devez exiger une demande écrite de l'expert
B - Vous devez lui adresser un résumé d'observation
C - Vous devez exiger une autorisation écrite des parents
D - Vous devez exiger une autorisation du conseil de l'Ordre
E - Vous êtes autorisé à garder le secret
Bonne(s) réponse(s) : E

Le médecin est seul juge de sa conduite vis à vis du secret professionnel qu'il détient, en effet la dérogation est facultative en
cas de sévices à enfant.

Un homme de 25 ans est retrouvé comateux à son domicile. Près de lui on retrouve une arme à feu et une lettre dans laquelle
il a écrit qu'il va se suicider et qu'il désire que ses organes soient prélevés à fin thérapeutique. Il est hospitalisé en urgence. Le
diagnostic porté est celui de plaie crânio-cérébrale. Quelques heures plus tard, il est considéré en coma dépassé.

Parmi les signes suivants, vous retenez pour affirmer un coma dépassé :
A - Rigidité cadavérique
B - Trace E.E.G, nul aréactif
C - Respiration spontanée
D - Arrêt circulatoire total
E - Hypotonie des globes oculaires
Bonne(s) réponse(s) : B

Les quatres signes fondamentaux du coma dépassé sont :


- la perte totale de la conscience et de toute activité spontanée
- abolition de toute réactivité dans le domaine des nerfs crâniens
- abolition de la respiration spontanée
- tracé EEG nul.
Les autres signes cliniques sont contingents :
- hypotonie musculaire, abolition des réflexes ostéotendineux, perte du tonus sphinctérien, absence de réaction à toute
stimulation nociceptive
- rythme cardiaque sinusal à fréquence lente, non modifié par les manoeuvres vagales et l'atropine
- température centrale abaissée entre 32 et 34° C.
Par ailleurs, les signes fondamentaux doivent être permanents pendant un délai suffisant.

1322
Exclusivement sur DOC - DZ : www.doc-dz.com NADJI 85
RESIDANAT EN POCHE TOME II
Cas Clinique en QCM

Qui à l'hôpital, dans un centre de réanimation, doit constater le décès pour qu'un prélèvement d'organe puisse
être effectué ?
A - Deux médecins dont un chef de service
B - Le médecin d'état-civil
C - Un médecin expert
D - Le médecin traitant
E - Le médecin qui va procéder à la greffe de l'organe prélevé
Bonne(s) réponse(s) : A

Le constat du décès d'un sujet soumis à une réanimation prolongée sera établi après consultation de deux médecins dont l'un
sera obligatoirement un chef de service hospitalier, secondés chaque fois qu'il est désirable par un spécialiste d'électro-
encéphalographie.

Le décès constaté, le prélèvement d'organe à fin thérapeutique sera possible dans le cas présent :
A - Seulement si le défunt a laissé un accord écrit
B - Sans aucune condition
C - Seulement dans un établissement hospitalier autorisé
D - Sauf opposition de la famille
E - Seulement avec l'accord du procureur de la République
Bonne(s) réponse(s) : C E

Les prélèvements d'organe sont réglementés par la loi du 22-12-76 : ils peuvent être effectués sur le cadavre d'un sujet s'il n'a
pas fait connaître de son vivant son refus d'un tel prélèvement ou si sa famille n'a pas indiqué sur un registre que le défunt
était opposé à une telle opération.
En cas de décès de cause suspecte, l'accord préalable du procureur de la République est indispensable.
Le suicide est une mort violente posant un problème médico-légal.

Dans le cas présent, le prélèvement étant réalisé, des soins de conservation du cadavre peuvent être effectués :
A - Sans aucune condition
B - Uniquement en cas de demande écrite de la personne ayant qualité pour organiser les obsèques
C - Uniquement après un délai de 48 heures
D - Uniquement avec l'accord du procureur de la République
E - Uniquement par un thanatopracteur agréé par la Préfecture
Bonne(s) réponse(s) : D

Les soins de conservation nécessitent un certificat d'un médecin délégué à l'état-civil attestant que la mort ne pose pas de
problème médico-légal.
Dans ce cas, ils sont pratiqués en cas de demande écrite de la personne qui s'occupe des funérailles.
En cas de mort posant un problème médico-légal, ces soins peuvent être effectués uniquement avec l'accord du procureur de
la République.

Dans cette affaire médico-légale, une incinération peut être effectuée :


A - Uniquement avec l'accord du procureur de la République
B - Uniquement plus de 48 heures après le décès
C - Sauf opposition de la famille
D - Sans conditions particulières
E - Uniquement après l'accord du médecin chef du service dans lequel est décédé le patient
Bonne(s) réponse(s) : A

Une incinération d'un cadavre ne peut être effectuée que si un certificat médical précise l'absence de problème médico-légal.
Dans le cas contraire, elle n'est possible qu'avec l'accord du procureur de la République. Elle n'est réalisée que 24 h après et
6 jours au plus après le décès.

Un matin d'automne, des chasseurs découvrent dans un bois le corps froid et raide d'un homme porteur de plusieurs plaies au
visage et au cuir chevelu. Les vêtements sont déchirés et ensanglantés. Les gendarmes alertés examinent les lieux et le
corps : ils ne trouvent aucun papier d'identité, ils évoquent la possibilité d'une lutte.

D'après les circonstances décrites, la mort peut être qualifiée de :


A - Apparente
B - Suspecte
C - Violente
D - Naturelle
E - Subite
Bonne(s) réponse(s) : B C

A - Mort apparente : disparition momentanée des critères grossiers de la vie (respiration, pouls, battements cardiaques,
tension artérielle) chez un sujet encore vivant mais qui prend l'apparence de la mort.
D - Mort naturelle : mort spontanée ne posent pas de problème médico-légal.
E - Mort subite : décès brutal, inattendu et de cause naturelle. 1323
Exclusivement sur DOC - DZ : www.doc-dz.com NADJI 85
RESIDANAT EN POCHE TOME II
Cas Clinique en QCM

Un médecin généraliste est requis pour examiner le corps.


Quel(s) sera (seront) l'(es) objectif(s) de sa mission ?
A - Affirmer la réalité de la mort
B - Décrire les blessures
C - Estimer le moment de la mort
D - Procéder à l'autopsie
E - Identifier le corps
Bonne(s) réponse(s) : A B C

Tout docteur en médecine peut être requis, par exemple en cas de violence (examen d'un blessé), mort suspecte
(constatations immédiates en cas de découvertes d'un cadavre, ou autopsie), accident avec suspicion d'intoxication alcoolique
(prise de sang, examen clinique).

Pour apprécier l'heure de la mort, vous devez apprécier :


A - Les lividités cadavériques
B - La perte de transparence de la cornée
C - La présence de larves de mouches
D - La tache verte abdominale
E - Des traces de vomissements
Bonne(s) réponse(s) : A B C D

A B - Sont des signes de l'évolution cadavérique, de même que la rigidité cadavérique et le refroidissement du corps.
C D - Sont des signes tardifs de mort avec putréfaction.

Le médecin généraliste avise les autorités judiciaires qu'il n'est pas compétent pour procéder à l'autopsie.
Le procureur désignera un expert de médecine légale sur la liste de la cour d'Appel.
Les modalités de l'autopsie judiciaire comporteront :
A - Elle doit avoir lieu obligatoirement 24 h après le décès
B - L'accord de la famille est nécessaire
C - L'autorisation du maire de la commune
D - La présence du médecin d'Etat Civil
E - La réquisition du Procureur est indispensable
Bonne(s) réponse(s) : E

L'autopsie est faite à la demande de l'autorité judiciaire (procureur de la République ou juge d'instruction). Il n'y a pas, dans ce
cas, d'opposition possible.

Une femme, traitée pour hypertension artérielle depuis plusieurs années, débute une grossesse.

Pour qu'une indication d'interruption volontaire de grossesse pour motif thérapeutique soit posée, il faut :
A - Impérativement que la vie de la femme soit en danger
B - Que la santé de la femme soit en péril grave
C - Que la grossesse soit inférieure à 12 semaines d'aménorrhée
D - Que la femme soit âgée de plus de 40 ans
E - Que son conjoint ait donné son accord
Bonne(s) réponse(s) : B

Article L 162-12 du code de la Santé publique :


-"l'IVG peut être pratiquée à toute époque si deux médecins attestent, après examen et discussion, que la poursuite de la
grossesse met en péril grave la santé de la femme ou qu'il existe une forte probabilité que l'enfant à naître soit atteint d'une
affection d'une particulière gravité, reconnue comme incurable au moment du diagnostic".

Combien de médecins doivent examiner et signer le certificat d'I.V.G. pour motif thérapeutique :
A-1
B-2
C-3
D-4
E-5
Bonne(s) réponse(s) : B

Article L 162-12 du code de la Santé publique :


-"l'IVG peut être pratiquée à toute époque si deux médecins attestent, après examen et discussion, que la poursuite de la
grossesse met en péril grave la santé de la femme ou qu'il existe une forte probabilité que l'enfant à naître soit atteint d'une
affection d'une particulière gravité, reconnue comme incurable au moment du diagnostic".

1324
Exclusivement sur DOC - DZ : www.doc-dz.com NADJI 85
RESIDANAT EN POCHE TOME II
Cas Clinique en QCM

Le certificat préalable à cette I V.G. pour motif thérapeutique doit être signé par :
A - Un médecin assermenté
B - Un Président du tribunal de Grande Instance
C - Un médecin exerçant son activité en milieu hospitalier public ou privé
D - Un médecin expert près de la cour d'Appel
E - Le directeur de l'hôpital
Bonne(s) réponse(s) : C D

Article 162-12 du code de la Santé publique : "L'un des deux médecins doit exercer son activité dans un établissement public
ou dans un établissement privé satisfaisant aux conditions de l'article L 176 et l'autre doit être inscrit sur la liste d'expert près
de la cour de Cassation ou près d'une cour d'Appel."

Cette I.V.G pour motif thérapeutique pourra être réalisée :


A - Uniquement dans un centre hospitalier universitaire
B - Uniquement en centre hospitalier
C - En établissement d'hospitalisation public ou privé (agréé selon l'art L 172 du Code de la Santé)
D - En cabinet médical
E - En centre médico-social
Bonne(s) réponse(s) : C

Article 162-13 du code de la Santé publique : " L'IVG thérapeutique ne peut être pratiquée que par un médecin. Elle ne peut
être avoir lieu que dans un établissement d'hospitalisation public ou dans un établissement d'hospitalisation privé satisfaisant
aux dispositions de l'article L 176 (et non L 172 comme c'est indiqué dans la proposition).

Le bulletin de déclaration de cette I.V.G pour motif thérapeutique doit être adressé :
A - Au médecin inspecteur régional de la Santé
B - Au directeur de la D.D.A.S.S
C - Au président du conseil régional de l'Ordre
D - Au procureur de la République
E - Au commissaire de la République
Bonne(s) réponse(s) : C

Le bulletin de déclaration de l'IVG thérapeutique indiquant la décision prise avec explication et discussion est adressé au
président du conseil départemental de l'Ordre, sous pli recommandé, sans indication du nom de la patiente.

Un homme de 48 ans est hospitalisé pour des crampes nocturnes et une atteinte de l'état général : amaigrissement de 4 kilos
en trois mois, anorexie importante, fatigabilité. Cette personne reconnaît fumer 2 paquets de cigarettes par jour depuis l'âge
de 18 ans et consomme quotidiennement 3 litres de vin à 12°, 2 litres de bière à 6°.

Quelle quantité journalière d'alcool pur (en g) absorbe ce malade ?


A - 125 g
B - 156 g
C - 336 g
D - 288 g
E - 384 g
Bonne(s) réponse(s) : E

- Titre en degré = volume d'alcool pour cent ml.


- Densité de l'alcool = 0,8.
- Consommation = Somme (titre x volume (ml) x densité de l'alcool /100) = 12 X 3000 X 0,8 /100 + 6 X 2000 X 0,8/100 = 384
gr.

Chez ce malade, quel est l'apport calorique quotidien lié à la consommation d'alcool ?
A - 792 Kcal
B - 1320 Kcal
C - 1584 Kcal
D - 2688 Kcal
E - 2324 Kcal
Bonne(s) réponse(s) : D

Un gramme d'alcool apporte 7 Kcal et 2688 = 384 x 7.

1325
Exclusivement sur DOC - DZ : www.doc-dz.com NADJI 85
RESIDANAT EN POCHE TOME II
Cas Clinique en QCM

Il se dit incapable de diminuer cette consommation malgré les tentatives d'arrêt. Il allègue une perte de l'appétit,
également une sensation de malaise général et un tremblement des mains présent surtout le matin et
s'estompant après ingestion d'alcool. Mis à part un visage congestionné et des dilatations capillaires localisées
aux pommettes et aux ailes du nez, l'examen clinique ne montre pas d'anomalie. Sur quel(s) élément(s) pouvez-
vous affirmer que ce malade est dépendant de l'alcool ?
A - Consommation excessive d'alcool (supérieure à 120 g par jour)
B - Incapacité de diminuer sa consommation d'alcool
C- Visage congestionné et dilatations capillaires localisées aux pommettes et aux ailes du nez
D - Malaise et/ou tremblement des mains le matin s'estompant après ingestion d'alcool
E - Crampes nocturnes
Bonne(s) réponse(s) : B D

La dépendance n'est pas fonction de la dose mais de l'apparition des symptômes lors du sevrage.

Ce sujet est à haut risque de certaines affections. Laquelle ou lesquelles ?


A - Cancer du larynx
B - Cancer de l'oesophage
C - Pancréatite chronique
D - Cancer de la thyroïde
E - Cancer de la vessie
Bonne(s) réponse(s) : A B D E

L'association alcool-tabac est un des facteurs de cancérogenèse les plus répandus.

Parmi les troubles suivants, dont se plaint le patient, l'alcoolisme chronique peut être responsable de :
A - Crampes nocturnes
B - Fatigabilité
C - Anorexie
D - Troubles du caractère à type d'irritabilité
E - Insomnie
Bonne(s) réponse(s) : A B C D E

Sans commentaire.

Après la sortie de l'hôpital où il a suivi une cure de sevrage, quelle(s) est(sont) la(les) structure(s) spécialisée(s)
pouvant contribuer ; à sa prise en charge ?
A - Centres d'Hygiène Alimentaire
B - Foyers de post-cure
C - Services de moyen séjour
D - Mouvements d'anciens buveurs
E - Services de soins à domicile
Bonne(s) réponse(s) : A B D

Sans commentaire.

Un jeune homme de 16 ans est transporté dans un état gravissime au Centre Hospitalier, car il a été victime d'un accident de
moto. Son état ne permet pas d'envisager d'intervention chirurgicale car il est en coma profond nécessitant une ventilation
artificielle dans un service de réanimation. Son état s'aggrave rapidement et il n'existe plus aucun espoir de survie. L'équipe
de chirurgie digestive souhaite si cela est possible pouvoir disposer d'un prélèvement de foie sur ce patient en vue de
transplantation.

Les possibilités de prélèvement sur le coma dépassé nécessitent :


A - Analyse méthodique des circonstances de l'accident
B - Caractère entièrement artificiel de la respiration
C - Abolition totale de tout réflexe, hypotonie complète, mydriase
D - Tracé nul à l'électroencéphalogramme pendant une durée suffisante en dehors de toute hypothermie ou
drogue sédative
E - Tracé électrocardiographique plat
Bonne(s) réponse(s) :

QUESTION ANNULEE
L''item C ne peut être retenu car parfois subsistes des réflexes médullaires.

1326
Exclusivement sur DOC - DZ : www.doc-dz.com NADJI 85
RESIDANAT EN POCHE TOME II
Cas Clinique en QCM

De plus, en vue d'un éventuel prélèvement :


A - Il est nécessaire d'obtenir le consentement oral du représentant légal
B - Il est nécessaire d'obtenir le consentement écrit du représentant légal
C - Le défunt doit avoir donné l'autorisation de prélèvement de son vivant
D - La règle est l'autorisation de prélèvement sur le cadavre si le défunt n'a pas exprimé le refus de son vivant
E - Il est impossible de prélever chez le mineur
Bonne(s) réponse(s) : B

Article 2 : Loi 76-1181 du 22.12.1976.


Article 11 : Loi 78-501 du 31.03.1978.

Le certificat de décès qui sera rédigé :


A - Est obligatoirement établi par un médecin
B - Comprend une partie précisant l'identité du défunt et destinée à la mairie
C - Comprend une partie anonyme précisant les causes du décès et destinée à l'institut de médecine légale
D - Doit être produit pour que l'officier d'état civil établisse le permis d'inhumer
E - Comporte une question quant à l'existence ou non d'un obstacle médico-légal à l'inhumation
Bonne(s) réponse(s) :

QUESTION ANNULEE

A propos des possibilités de prélèvements à l'hôpital :


A - Tous les établissements hospitaliers ne sont pas habilités à faire des prélèvements d'organes à visée
scientifique
B - Ils doivent avoir obtenu une autorisation de la DDASS
C - Ils doivent être inscrits sur une liste d'établissements autorisés établie par le Ministre de la Santé Public
D - L'existence de preuves concordantes du décès doit être établie par deux médecins de l'établissement dont
l'un est chef de service
E - Il est interdit que participent au constat de décès les médecins appartenant à l'équipe effectuant le
prélèvement
Bonne(s) réponse(s) : A C D E

C'est le ministère qui autorise un établissement à pratiquer tel ou tel type de prélèvements (article 14-15 de la loi 78-501).
Les items D et E sont explicités dans l'article 20 de la loi 78-501.

Un enfant de 8 ans est victime d'un accident de la voie publique. Il est amené par ses parents, appartenant à une secte, aux
urgence d'un hôpital privé. L'état de l'enfant est gravissime, une transfusion est nécessaire en urgence. Les parents refusent.

Quelle est votre première attitude ?


A - Avant tout geste thérapeutique, vous essayez de raisonner et de convaincre les parents de l'utilité de ce
geste
B - Vous pratiquez sans délai les gestes thérapeutiques indispensables
C - Avant tout acte thérapeutique, vous essayez de joindre le juge des enfants au tribunal
D - Vous faites avertir sans délai le Directeur de l'établissement du problème en cours
E - Vous faites avertir sans délai le directeur de la DDASS
Bonne(s) réponse(s) : C

Réponse C, mais en pratique B.


Décret du 14 janvier 1974.
Réponse discutable car il joue aussi l'article 63.2 du Code Pénal et art. 4 Code de Déantologie cas de "non, assistance à
personne en danger".
A - Consentement des parents s'impose de façon formelle dès que la maladie est mineur.

Si vous aviez respecté la décision des parents et que l'enfant était décédé, quel motif aurait pu motiver votre
inculpation ?
A - Violation du secret professionnel médical
B - Faute grave détachable du service
C - Non respect de la législation sur la protection de l'enfance
D - Non assistance à personne en danger
E - Non dénonciation d'un crime
Bonne(s) réponse(s) : D

Voir question [1].

1327
Exclusivement sur DOC - DZ : www.doc-dz.com NADJI 85
RESIDANAT EN POCHE TOME II
Cas Clinique en QCM

Dans l'hypothèse où malgré vos tentatives, les parents ramènent l'enfant à leur domicile. Il y décède. Pendant
ce temps, vous avez averti les autorités afin que des mesures d'urgence soient prisent. Les parents portent
plainte pour violation du secret médical. Quelle est la proposition exacte ?
A - Cette plainte est recevable car il n'y a pas de secret médical vis-à-vis d'un magistrat
B - Cette plainte n'est pas recevable, car vous avez respecté la législation de protection des mineurs
C - Cette plainte est recevable car en cas d'urgence, il n'y a pas de secret médical vis-à-vis des services de
police
D - Cette plainte est recevable car il y a dérogation du secret médical en cas de sévices à enfants ou
équivalent
E - Cette plainte est recevable dans tous les cas
Bonne(s) réponse(s) : B

Il n'y a pas eu violation du secret professionnel.


Art. 375 Code Civil.

Supposons que vos hésitations aient retardé la mise en place du traitement et que l'enfant décède. Les parents
portent plainte. En cas de condamnation à réparation, qui paye cette réparation ?
A - L'assurance responsabilité civile des parents
B - L'assurance responsabilité civile de l'hôpital
C - Vous même dans tous les cas
D - Votre assurance de responsabilité civile
E - La sécurité sociale
Bonne(s) réponse(s) : D

Sans commentaire.

Finalement, l'enfant ne décède pas, mais en raison du retard de la thérapeutique il subit un gros handicap
permanent. Il décide à l'âge de 35 ans de porter plainte contre vous afin d'obtenir réparation :
A - Cette plainte n'est pas recevable en raison de la prescription
B - Cette plainte est recevable compte tenu du délai de prescription applicable dans ce cas
C - Cette plainte devra se faire au niveau de la juridiction administrative
D - Une enquête et une expertise médicale seront alors ordonnées par la juridiction compétente
E - Cette plainte pourra se faire devant le conseil régional de l'ordre des médecins
Bonne(s) réponse(s) : B D

A - Le délai de prescription est de 30 ans en cas de responsabilité civile, elle est de 4 ans si l'activité est exercée au sein d'un
établissement public et la plainte est posée auprès des juridictions administratives. En cas de responsabilité pénale le délai
est de 3 ans.
C - Non il s'agit d'un hôpital privé.
E - Elle se fera devant les juridictions civiles.

Monsieur Mr Jean, âgé de 2 6 ans, toxicomane, vous consulte pour : fatigue, irritabilité, nervosisme. Les signes fonctionnels
sont : tremblements, myalgies, insomnie. A l'examen, le pouls et la tension sont normaux, pas de fièvre, une perte de poids de
7 kilos, une mydriase sans signe de Babinski, un hypocondre droit douloureux, un sub-ictère conjonctival. Il consomme surtout
des opiacés, mais aussi des barbituriques (Secobarbital®). Dans ses antécédents, il allègue de nombreuses tentatives de
suicide, instabilité socio-professionnelle et sentimentale.

En fonction des éléments en votre possession, vous pensez à un épisode somatique intercurrent chez ce
patient. Lequel ou lesquels ?
A - Syndrome de sevrage aux opiacés
B - Hépatite virale
C - Délirium tremens
D - Affection digestive chirurgicale
E - Simulation
Bonne(s) réponse(s) : A B

A - Tremblements, myalgies, mydriase.


B - Trypochonche droit douloureux, subictère.

1328
Exclusivement sur DOC - DZ : www.doc-dz.com NADJI 85
RESIDANAT EN POCHE TOME II
Cas Clinique en QCM

Le patient vous a demandé un antalgique pour ses myalgies et les douleurs abdominales. Vous pouvez
envisager de prescrire :
A - Un antalgique du tableau B
B - Un antidépresseur tricyclique
C - Un antispasmodique
D - Un psychostimulant
E - Un tranquillisant
Bonne(s) réponse(s) : C E

L'autalgique le plus efficace est l'aspirine, après vérification crase sanguine.

Lors de la prescription d'un antalgique du tableau B, votre ordonnance comporte la date de prescription, la
posologie du médicament, le nom du malade ; cette ordonnance doit également comporter :
A - L'adresse du malade
B - Le diagnostic de la maladie
C - La durée de prescription
D - Votre signature
E - Le nom du pharmacien
Bonne(s) réponse(s) : A C D

Livre V, Titre III, articles L.626 à L.630 au Code de la Santé Publique (première partie) et R. 5149 et 5299 (deuxième partie).

Quelle est la durée maximum de prescription sans renouvellement d'un médicament inscrit au Tableau B
(Tableau B' excepté) ?
A - 3 jours
B - 7 jours
C - 30 jours
D - 60 jours
E - Durée illimitée
Bonne(s) réponse(s) : B

Tableau B' : 60 jours.

En fonction des éléments dont vous disposez, comment pourrait s'appliquer la Loi de 1970 sur les toxicomanies
?
A - Vous pouvez signaler ce patient à la DDASS qui le prendra en charge
B - Vous rédigez un certificat demandant la suspension des poursuites judiciaires
C - Votre patient sera automatiquement protégé contre des poursuites pour trafic
D - Votre patient, sous certaines conditions. pourrait éviter les poursuites pour usage personnel de stupéfiants
E - Votre patient peut être hospitalisé en gardant l'anonymat
Bonne(s) réponse(s) : A D E

A - Déclaration non obligatoire.


E - Sur sa demande expresse art. L 355-21.
Loi n° 70.1320 du 31.12.1970 modifiée par la loi 86.76 du 17 janvier 1986.

Une jeune femme étrangère de 17 ans résidant depuis 5 mois et demi en France, mariée avec un homme Portugais résidant
en France, vous consulte en tant que Médecin Généraliste pour une aménorrhée de 9 semaines et demies
Elle est sans travail et a interrompu sa scolarité depuis un an
Elle présente une hypertension artérielle (18/10) traitée par bêta-bloquants. Elle vous dit que si elle est enceinte, elle désire
une IVG.

Vous lui répondez :


A - Si vous ne souhaitez pas pratiquer l'IVG vous devez l'en avertir immédiatement
B - Faire pratiquer un examen clinique, gynécologique
C - Faire pratiquer un examen biologique de grossesse
D - Vous devez la prévenir des risques pour sa santé et les maternités futures de l'IVG
E - Vous l'adressez immédiatement à un confrère pratiquant cette intervention
Bonne(s) réponse(s) : A B C D

A - Article L 162.8 : "doit informer au plus tard lors de la visite de l'intéressée de son refus" c'est la clause de conscience.
B C - Si le médecin fait jouer ou non la clause de conscience c'est obligé de : "s'assurer de la réalité de la grossesse".
E - Le médecin doit communiquer "la liste et les adresses des établissements ou sont effectués les IVG".

1329
Exclusivement sur DOC - DZ : www.doc-dz.com NADJI 85
RESIDANAT EN POCHE TOME II
Cas Clinique en QCM

Le test de grossesse est positif, vous ne pratiquez pas vous-même l'IVG, que devez vous faire ?
A - Lui remettre un dossier guide des formalités et des mesures sociales
B - Recueillir le consentement d'un des deux parents puisqu'elle est mineure
C - Recueillir la demande de la patiente par écrit
D - Obtenir une attestation prouvant qu'elle réside en France depuis plus de trois mois
E - Obtenir l'entente préalable de la sécurité sociale
Bonne(s) réponse(s) : A C D

B - Ordre représentant légal si célibataire moins de 18 ans.


C - Valable sans exclusion. Pour les mineurs célibataires, la demande d'IVG doit être donnée en l'absence des parents ou du
représentant légal.
D - Art. 1 décret 75.334 du 13 mai 1975.
E - Pas nécessaire si elle procure un document justifiant qu'elle réside régulièrement en France depuis 3 mois.

Que doit faire la patiente ?


A - Se présenter dans les délais les plus brefs à une consultation spécialisée d'un établissement public ou
privé pratiquant l'IVG
B - Consulter un Conseiller conjugal au planning familial
C - Respecter un délai de 7 jours entre cette consultation et l'IVG
D - Ce délai peut être ramené à 2 jours, si le terme des 10 semaines risque d'être dépassé
E - La patiente peut obtenir l'IVG dans un but thérapeutique puisqu'elle est hypertendue, sans formalités
complémentaires
Bonne(s) réponse(s) : A B C D

A - Avec un certificat du 1er médecin justifiant qu'il s'est conformé aux dispositions de l'article L 126.3 du Code de la Santé.
D - Art. 6 loi 6 décembre 1979.
E - IVG thérapeutique peut être "pratiquée à tout moment de la grossesse si deux médecins attestent après examen et
discussion que la poursuite de la grossesse met en péril grave la santé de la femme...", ce qui n'est pas le cas ici.

Seul l'établissement hospitalier public pratique les IVG dans le département :


A - Le chef de service concerné de l'hôpital peut refuser de pratiquer lui même l'IVG pour clause de
conscience
B - Le chef de service ne peut pas refuser de laisser pratiquer l'IVG dans son service
C - Le nombre d'IVG ne peut pas être supérieur au quart des actes chirurgicaux et obstétricaux du service
D - Les l.V.G. seront déclarées nominativement à la DDASS
E - En cas de désaccord des parents, il faudra obtenir l'autorisation du juge des enfants
Bonne(s) réponse(s) : A B

A - Comme tout médecin.


B - La clause de conscience art. L 162.8 est personnelle.
C - Au quart des actes de l'établissement.
D - Déclaration obligatoire "ne fait aucune mention de l'identité de la femme" art. 162.10.

Une fois l'I.V.G. réalisée :


A - Les frais seront pris en charge par la sécurité sociale
B - Il faudra donner une information sur la contraception
C - Il faudra prescrire un contraceptif
D - Il faudra conserver trois ans la demande écrite de la jeune femme
E - Il faudra transmettre la demande écrite de la jeune femme au conseil de l'ordre des médecins
Bonne(s) réponse(s) : A B

A - Loi n° 82.1172 du 31.12.1982 (sauf pour les étrangères ne résidant pas dans des situations régulières).
D - Délai de 1 an, décret du 12 mai 1985. Le Directeur de l'établissement doit conserver :
- Le certificat de la première visite médicale.
- L'attestation de l'entretien psychosocial.
- La demande écrite de la femme.
- Le consentement des parents si mineure non mariée.

1330
Exclusivement sur DOC - DZ : www.doc-dz.com NADJI 85
RESIDANAT EN POCHE TOME II
Cas Clinique en QCM
Un petit garçon de 3 ans joue, un après-midi, à la maison. Vers 18 heures, pour préparer le repas, sa mère le laisse une
quinzaine de minutes sans surveillance.
Elle le retrouve alors dans la salle de bain au milieu de la pharmacie familiale. L'enfant a plusieurs comprimés dans la main.
Affolée, la mère vous appelle au téléphone. Vous arrivez au domicile quelques minutes plus tard.

Dans l'immédiat, quel(s) geste(s) est(sont) indiqué(s) pour faire correctement face à la situation ?
A - Examiner cliniquement l'enfant
B - Dresser la liste des produits contenus dans la pharmacie
C - Téléphoner au centre anti-poison pour en connaître les dangers
D - Faire boire un verre de lait à l'enfant
E - Faire vomir l'enfant à l'aide d'un abaisse-langue
Bonne(s) réponse(s) : A B C

D E - Vous n'êtes pas sur des produits ingênés. Les ingestions de solvants ou caustiques contredisent les vomissements.

La pharmacie contenait 5 médicaments différents. Quel est, parmi ces 5 produits, le plus dangereux pour
l'enfant ?
A - Rhinathiol (carbocystéine)
B - Tofranil (imipramine)
C - Urbanyl (clobazam)
D - Stédiril (norgestrel- et éthinylestradiol)
E - Alvityl (polyvitamines)
Bonne(s) réponse(s) : B

Sans commentaire.

Votre rapide enquête montre que l'enfant est susceptible d'avoir ingéré une dose dangereuse de ce produit.
Comme vous exercez à distance d'un centre hospitalier, vous décidez d'effectuer une évacuation gastrique sur
place. Quelle(s) donnée(s) contre-indiquerai(en)t a priori ce geste ?
A - Un délai de plus de 3 heures après l'ingestion toxique
B - L'ingestion concomitante de White spirit
C - Une altération de la conscience
D - La présentation en sirop du toxique absorbé
E - Le caractère dépresseur du toxique absorbé
Bonne(s) réponse(s) : B C

Le lavage gastrique est actif immédiatement, peut être réalisé chez des sujets inconscients après intubation, est contre
indiqué en cas d'ingestion de corrosifs puissants et d'hydrocarbures (toxicité pulmonaires en cas de fausse route).

Pour provoquer des vomissements à domicile, vous pouvez recourir à :


A - Ingestion de sirop d'érable
B - Ingestion de sirop d'ipéca
C - Injection intramusculaire d'ipéca
D - Injection intramusculaire de nalorphine
E - Injection intramusculaire d'apomorphine
Bonne(s) réponse(s) : B E

Ces vomissements ne doivent pas être induits en cas de troubles de la conscience ou de convulsion et en cas d'ingestion
d'hydrocarbures ou de corrosifs.
B - Action retardée.
E - Action immédiate.

Une fois cette évacuation terminée, vous faites hospitaliser l'enfant. Quel risque, lié au médicament le plus
toxique, menace particulièrement l'enfant dans les heures qui suivent son admission ?
A - Insuffisance hépatique
B - Insuffisance rénale
C - Trouble du rythme cardiaque
D - Acidose métabolique
E - Hypoglycémie
Bonne(s) réponse(s) : C

Sans commentaire.

1331
Exclusivement sur DOC - DZ : www.doc-dz.com NADJI 85
RESIDANAT EN POCHE TOME II
Cas Clinique en QCM

Dès l'admission vous allez prescrire :


A - Evacuation gastrique complémentaire par lavage d'estomac
B - Surveillance continue par cardio-moniteur
C - Diurèse forcée alcaline
D - Administration d'hyposulfite de sodium
E - Injection de bleu de méthylène
Bonne(s) réponse(s) : A B

Sans commentaire.

Un automobiliste (homme de 70 kgs) s'arrête pour prendre son déjeuner au cours duquel il consomme 1/2 litre de vin à 12° et
25 ml de digestif à 40°.

Quelle est son alcoolémie approximativement quand il reprend la route à la fin du repas ?
A - Inférieure à 0,40 g/l
B - 0, 60 g/l
C - 0, 80 g/l
D - 1 g/l
E - Supérieure à 1 g/l
Bonne(s) réponse(s) : E

Alcoolémie (pour mille) = (Quantité d'alcool absorbé) / (poids du corps en Kg x ,7 mâle)

Il subit un éthylotest en contrôle systématique quelques minutes après avoir repris la route. Quel poids d'alcool
ce test indique-t-il ? (exprimé par litre d'air expiré).
A - De l'ordre de 0,40 mg
B - De l'ordre de 0,40 g
C - Supérieur à 0,40 mg
D - Supérieur à 0,40 g
E - Supérieur à 4 g
Bonne(s) réponse(s) : C

Concentration alvéolaire alcoolique = Alcoolémie/2 000

Au cas éventuel où l'éthylotest révélerait chez cet automobiliste un taux d'alcool délictuel, les forces de l'ordre
seraient autorisées à :
A - Retenir son permis de conduire immédiatement
B - Retenir son permis de conduire pour une durée maxima de 72 heures
C - Retenir son permis de conduire seulement après les résultats d'une alcoolémie
D - Décréter la suspension du permis de conduire immédiatement, sans vérification de l'alcoolémie
E - Immobiliser son véhicule si personne ne peut le conduire légalement
Bonne(s) réponse(s) : A B E

Article 18.1 Code de la route ajouté par la loi du 17 janvier 1976.

Concernant les accidents de la circulation, il est exact que :


A - Le trafic de nuit ne représente que 20 % de la circulation routière globale
B - Le trafic de nuit est responsable de 20 % des accidents mortels
C - La gravité des blessures est supérieure la nuit que pendant le trafic diurne
D - La vision nocturne physiologique humaine est de mauvaise qualité
E - La luminance apportée par les phares standards ne permet qu'une acuité visuelle de l'ordre de 3/10
Bonne(s) réponse(s) : A C D E

Sans commentaire.

1332
Exclusivement sur DOC - DZ : www.doc-dz.com NADJI 85
RESIDANAT EN POCHE TOME II
Cas Clinique en QCM
Vous suivez M. X... pour une épilepsie essentielle traitée par les hydantoïnes. Depuis quelques mois il ne se sent pas en
forme et a constaté une baisse d'efficience professionnelle et une recrudescence de ses crises d'épilepsie. L'interrogatoire et
l'examen clinique vous font évoquer une alcoolisation chronique.

M. X... vous révèle une consommation quotidienne d'un litre et demi de vin à 10 degrés. A combien estimez-
vous approximativement sa consommation d'alcool pur par jour ?
A - 160 grammes
B - 140 grammes
C - 120 grammes
D - 100 grammes
E - 80 grammes
Bonne(s) réponse(s) : C

1 litre à 10° = 80 g d'alcool.

Si vous utilisez la grille de Le Go, quel(s) signe(s) clinique(s) présenté(s) par M. X. allez vous y faire figurer ?
A - Angiomes stellaires
B - Trémulation des doigts
C - Insomnie
D - Varicosités du visage
E - Hypertension artérielle
Bonne(s) réponse(s) : A B D

Sans commentaire.

M. X.. vous montre un bilan biologique sanguin récent. Quel est ou quels sont le(s) signe(s) évocateur(s) d'une
alcoolisation chronique ?
A - Triglycéridémie = 2,4 mmol/l
B - Volume globulaire moyen : 102 microns cubes
C - Uricémie : 40 mg/l
D - Cholestérol total = 7,5 mmol/l
E - Albumine = 47 g/l
Bonne(s) réponse(s) : B

Sans commentaire.

Vous conseillez un sevrage à M. X.. et vous lui prescrivez des vitamines B et une benzodiazépine à faible dose.
Quinze jours plus tard, la gamma glutamyl transférase (G.G.T.) est toujours élevée. Quelle(s) hypothèse(s)
évoquer ?
A - La prise d'hydantoïnes
B - La persistance de l'alcoolisation
C - L'hypertension artérielle
D - Une atteinte hépatique
E - La prise de Vitamine B
Bonne(s) réponse(s) : A B D

Sans commentaire.

Mademoiselle Z..., française, 16 ans et demi, célibataire, travaille comme manutentionnaire dans une usine à 24 kilomètres de
chez elle. Elle vient consulter car elle est enceinte de 9 semaines. Sa tension artérielle est à 160-90 mmHg. Le reste de
l'examen somatique est satisfaisant.

Si elle vous demande une interruption volontaire de grossesse (I.V.G.), quelle est votre attitude ?
A - Vous devez l'adresser au préalable à un obstétricien
B - Vous demandez l'autorisation d'un de ses parents
C - Vous devez l'adressez au service social de l'entreprise pour entretien
D - Vous lui faites confirmer sa demande par écrit dès la première consultation
E - Vous refusez car les délais sont dépassés
Bonne(s) réponse(s) : B D

Loi du 17 janvier 1975.


B - Ou du représentant légal.
D - Donnée en dehors de la présence des parents ou du représentant légal.
E - Délai de 10 semaines.

1333
Exclusivement sur DOC - DZ : www.doc-dz.com NADJI 85
RESIDANAT EN POCHE TOME II
Cas Clinique en QCM

Si elle souhaite poursuivre sa grossesse, à quel organisme doit-elle en faire la déclaration ?


A - La mairie
B - La D.D.A.S.S.
C - La Médecine du travail
D - La Caisse d'Allocations Familiales
E - La D.R.A.S.S.
Bonne(s) réponse(s) : C

Ainsi qu'à la caisse de Sécurité Sociale.

Combien de facteur(s) de risque d'accouchement prématuré présente-t-telle ?


A - Aucun
B - 1 facteur
C - 2 facteurs
D - De 2 à 4 facteurs
E - 5 facteurs
Bonne(s) réponse(s) : D

- Jeune âge.
- Manutentionnaire dans l'usine.
- Durée de trajet long.

Mademoiselle Z... a déclaré sa grossesse. De quelle(s) prestation(s) pourra-t-elle bénéficier ?


A - Allocations pour jeune enfant
B - Indemnités journalières au cours du congé pré et post natal
C - Allocation du fond national de solidarité
D - Hébergement en maison maternelle
E - Surveillance gratuite de sa grossesse en P. M. I.
Bonne(s) réponse(s) : B E

Sans commentaire.

1334
Exclusivement sur DOC - DZ : www.doc-dz.com NADJI 85
RESIDANAT EN POCHE TOME II
Cas Clinique en QCM

1335
Exclusivement sur DOC - DZ : www.doc-dz.com NADJI 85
RESIDANAT EN POCHE TOME II
Cas Clinique en QCM

1336
Exclusivement sur DOC - DZ : www.doc-dz.com NADJI 85
RESIDANAT EN POCHE TOME II
Cas Clinique en QCM
En septembre 1981, un médecin d'une commune de 15 000 habitants, signale à la DDASS plusieurs cas de gastroentérite. Un
contrôle de l'eau d'adduction publique est aussitôt pratiqué, et révèle l'existence d'une pollution bactériologique (Escherichia
Coli).
Une enquête épidémiologique (enquête de cohorte rétrospective) est alors réalisée dans un quartier bien différencié de cette
commune, afin de mesurer la fréquence de la diarrhée
- chez les habitants qui ont consommé en septembre l'eau du robinet
- chez les habitants qui n'en ont pas consommé durant cette période (puits personnel, consommation d'eau minérale...).

Cliquez sur le bouton "Dessin" pour afficher le tableau.

Quel est le risque relatif entre les buveurs d'eau du robinet et les non-buveurs de cette eau ?

Cliquez sur le bouton "dessin" pour afficher le tableau.


A - Environ 2
B - Environ 4
C - Environ 6
D - Environ 8
E - Environ 10
Bonne(s) réponse(s) : E

Le risque est le rapport entre la fréquence des diarrhées chez les buveurs d'eau du robinet (400/530) et les fréquences chez
les non buveurs (6/80).
Soit R.R. = (400/530) / ( 6/80 ) = 32000 / 3180 = environ 10

Ce résultat signifie :

Cliquez sur le bouton "dessin" pour afficher le tableau.


A - Que l'effectif de la population est trop faible pour conclure
B - Que la fréquence de la diarrhée est à peu près équivalente dans le groupe exposé et le groupe non exposé
C - Que le risque de contracter une diarrhée était fortement augmenté dans le groupe exposé, par rapport au
groupe non exposé
D - Qu'il existe une présomption de relation causale entre la consommation d'eau du robinet et les
manifestations diarrhéiques
E - Que la présence d'Escherichia Coli dans l'eau du robinet est la cause des manifestations diarrhéiques
Bonne(s) réponse(s) : C D

Le risque relatif est augmenté très fortement (C), ce qui laisse présumer une relation causale (D) que l'enquête ne peut
affirmer (E faux).

Les enquêtes de cohorte font partie :

Cliquez sur le bouton "dessin" pour afficher le tableau.


A - De l'épidémiologie descriptive
B - De l'épidémiologie analytique
C - De l'épidémiologie expérimentale
D - Des enquêtes cas-témoins
E - Des enquêtes de prévalence
Bonne(s) réponse(s) : B

L'épidémiologie descripive vise à connaître l'état sanitaire du pays. L'épidémiologie analytique recherche les causes d'une
maladie en mettant en évidence une liaison entre facteur d'exposition et maladie.

L'analyse de l'eau a révélé :

Cliquez sur le bouton "dessin" pour afficher le tableau.


A - Le taux élevé des nitrates est un facteur causal de pollution bactériologique
B - La pollution bactériologique est un facteur causal d'augmentation des nitrates
C - Il n'y a pas de relation causale entre la pollution bactériologique et le taux élevé de nitrates
D - Le taux élevé de nitrates est une explication possible de l'épidémie de diarrhée
E - Le taux élevé de nitrates ne doit pas être incriminé dans la survenue de cette épidémie
Bonne(s) réponse(s) : C E

En théorie, il serait possible de se poser des questions sur le rôle de ces nitrates augmentées.
En pratique, leur taux est très peu élevé, et il n'y a pas de relation connue entre nitrates et pollution bactériologique.
L'association de ces 2 éléments, sans autre hypothèse sous-jacente ne suffit pas à mettre en cause les nitrates, encore moins
à en faire une explication (D).

1337
Exclusivement sur DOC - DZ : www.doc-dz.com NADJI 85
RESIDANAT EN POCHE TOME II
Cas Clinique en QCM

A la suite des conclusions de l'enquête, la mairie de cette commune a pris une série de mesures, Parmi les
mesures suivantes, laquelle(lesquelles) vous parai(ssen)t cohérente(s) ?
Cliquez sur le bouton "dessin" pour afficher le tableau.
A - Protection des captages (élimination d'une décharge sauvage d'ordures ménagères)
B - Stérilisation de l'eau d'adduction publique par addition d'une quantité suffisante de chlore
C - Désinfection de l'eau d'adduction publique par addition d'une quantité suffisante de chlore
D - Contrôle des normes d'étanchéité d'une station d'épuration située en amont du champ captant
E - Création d'un emploi de technicien des eaux
Bonne(s) réponse(s) : A C D E

Il est cohérent de prendre toutes ces mesures. Sauf la stérilisation.

Parmi les maladies suivantes, laquelle(lesquelles) peut(peuvent) être transmise(s) par une eau polluée (eau
d'adduction publique, ou eau de puits) ?
Cliquez sur le bouton "dessin" pour afficher le tableau.
A - Hépatite à virus A
B - Méthémoglobinémie
C - Saturnisme
D - Salmonellose
E - Hépatite à virus B
Bonne(s) réponse(s) : A B C D

Une eau polluée transporte des germes pathogènes tels que salmonelles et virus A pour l'hépatite. Le virus B transmet, lui,
essentiellement de façon parentérale et sexuelle. La méthémoglobinémie est une conséquence des nitrates chez le
nourrisson, le saturnisme, conséquence de l'excès de plomb.

Le schéma ci-dessous représente l'évolution des cas de méningite cérébro-spinale à méningocoques dans un département de
la région, qui comporte 100 000 habitants.

Cliquez sur le bouton "Dessin" pour afficher le tableau.

Le taux de mortalité spécifique de la méningite cérébrospinale dans ce département pour 1981 est de :

Cliquez sur le bouton "dessin" pour afficher le tableau.


A - 10/100 000
B - 8/1 00 000
C - 8/10
D - 2/10
E - 2/100 000
Bonne(s) réponse(s) : E

Il est bien évident que D signifie décès et non début. On constate pendant l'année 1981, 2 décès sur une population de 100
000 habitants. D'où un taux de mortalité spécifique de 2/100 000.

Le taux de létalité de la méningite cérébro-spinale pour 1981 est-il de :

Cliquez sur le bouton "dessin" pour afficher le tableau.


A - 10/100.000
B - 8/100.000
C - 8/10
D - 2/10
E - 2/100.000
Bonne(s) réponse(s) : D

La léthalité mesure la proportion de décès chez les sujets malades, soit 2 décès pour 10 malades en 1981.

1338
Exclusivement sur DOC - DZ : www.doc-dz.com NADJI 85
RESIDANAT EN POCHE TOME II
Cas Clinique en QCM

Devant la recrudescence des cas déclarés, le maire vous demande les mesures à prendre dans le cas où une
méningite surviendrait dans le groupe scolaire de la commune, Laquelle(lesquelles) des affirmations ci-dessous
est(sont) exacte(s) ?

Cliquez sur le bouton "dessin" pour afficher le tableau.


A - Fermeture immédiate de l'école
B - Eviction scolaire des enfants de la même classe
C - Vaccination de tous les enfants
D - Chimioprophylaxie des proches
E - Chimioprophylaxie chez tous les habitants du village
Bonne(s) réponse(s) : D

On protège uniquement les sujets ayant des contacts proches et répétés avec la victime. On utilise le Rovamycine® 2 g/j
pendant 5 jours.

La déclaration du cas éventuel :

Cliquez sur le bouton "dessin" pour afficher le tableau.


A - Sera obligatoire
B - Sera facultative
C - Au médecin conseil de la Sécurité Sociale
D - A la Direction Départementale des Affaires Sanitaires et Sociales
E - Entraînera systématiquement une analyse et une chloration de l'eau d'alimentation de la commune
Bonne(s) réponse(s) : A D

E. La transmission est uniquement interhumaine par les gouttelettes de salive.

Entre 1966 et 1967, huit jeunes femmes sont hospitalisées aux Etats-Unis pour adénocarcinome à cellules claires du vagin .
Cette tumeur étant jusque là considérée comme rarissime, une enquête épidémiologique a été menée à la recherche des
facteurs explicatifs dans les antécédents de ces jeunes cancéreuses comparées à 32 ans, femmes saines de même âge et de
même lieu de naissance.

Cliquez sur le bouton "Dessin" pour afficher le tableau.

De quel(s) type(s) d'enquête s'agit ?

Cliquez sur le bouton "dessin" pour afficher le tableau.


A - Enquête descriptive
B - Enquête prospective
C - Enquête rétrospective
D - Enquête cas témoins
E - Enquête de cohorte
Bonne(s) réponse(s) : C D

Cette enquête est une enquête cas -émoin (D) car elle compare 8 malades et 32 témoins en recherchant leur antécédents,
c'est donc une démarche rétrospective (C).
C'est aussi une enquête analytique et non descriptive (A) qui vise à mettre en évidence une liaison entre un facteur
d'exposition et une maladie.
Les enquêtes de cohorte (E) parlent de 2 populations, exposées-non exposées, suivies de façon prospective (B) pour suivre
l'apparition de la maladie dans les 2 groupes.

Le choix d'un groupe comparatif de même âge et de même lieu de naissance correspond à une méthode
appelée en épidémiologie :
Cliquez sur le bouton "dessin" pour afficher le tableau.
A - Une standardisation
B - Un appariement
C - Une pyramide des âges
D - Une étude en double insu
E - Une randomisation
Bonne(s) réponse(s) : B

Le but est dans cet appariement de rendre les groupes aussi comparables que possible.

1339
Exclusivement sur DOC - DZ : www.doc-dz.com NADJI 85
RESIDANAT EN POCHE TOME II
Cas Clinique en QCM

(Diéthylstilbestrol = D.E.S.OE.)
On peut dire, au vu du tableau de la page précédente, que :
Cliquez sur le bouton "dessin" pour afficher le tableau.
A - Le D.E.S.OE est un facteur de risque de l'adénocarcinome du vagin
B - Le D.E.S.OE est la cause de l'adénocarcinome du vagin
C - Toutes les femmes ayant pris du D.E.S.OE pendant leur grossesse ont fait ou feront un adénocarcinome
du vagin
D - Le rôle du D.E.S.OE est statistiquement démontré
E - Le nombre de cas est trop faible pour conclure
Bonne(s) réponse(s) : A D

Le rôle comme facteur de risque (A) du D.E.S.O.E. est statistiquement démontré (D) bien que les effectifs soient faibles (E). Il
n'est bien sûr pas possible d'en faire une relation causale (B) et donc d'établir avec certitude un pronostic pour les femmes
exposées (C).

Ce type d'enquête :

Cliquez sur le bouton "dessin" pour afficher le tableau.


A - Nécessite un diagnostic parfaitement défini
B - Est possible même quand la maladie est rare
C - Peut être biaisé par les perdus de vue
D - Demande plusieurs années de travail continu
E - Facilite la sélection du groupe témoin
Bonne(s) réponse(s) : A B

Une enquête rétrospective nécessite de parfaitement définir les 2 populations, malades (A)-témoins (leur sélection est
particulièrement difficile E faux). Ce type d'enquête est le seul possible pour les malades rares comme celle étudiée.
Les items C et D caractériseraient les enquêtes prospectives.

Le laboratoire F vous confie un essai thérapeutique. Il s'agit de déterminer l'efficacité d'un nouvel anti agrégant plaquettaire, le
.i.Magicor;®. Le laboratoire affirme que les essais faits à l'étranger montrent qu'associé avec les traitements hypotenseurs
classiques, il est très actif en prévention des conséquences morbides de l'hypertension artérielle. L'essai aura une durée de 5
ans au moins et portera sur une population homogène d'hypertendus sous bêta-bloquants.

Le protocole en double aveugle que vous proposez comportera-t-il un ouplusieurs des critères ci-dessous :
A - Deux groupes d'hypertendus, chaque groupe étant constitué des malades sous bêta bloquants d'une des
deux unités d'hospitalisation de votre CHR
B - Deux groupes d'hypertendus établis par randomisation
C - Un groupe traité par le .i.Magicor;®, l'autre non
D - Un groupe recevant le .i.Magicor;®, l'autre groupe recevant un médicament de référence (.i.Aspégic;® par
exemple)
E - Vous devrez ignorer lequel des groupes recoit le nouveau médicament
Bonne(s) réponse(s) : B D E

La séparation des 2 groupes doit se faire par randomisation. Le fait que le médecin ignore quel groupe reçoit Magicor® et que
le malade ne puisse identifier quel traitement il prend, c'est la définition du double-aveugle.
La réponse D n'est bonne qu'à cette condition.
La réponse C est fausse si l'autre groupe ne reçoit rien, il faut de toute façon un traitement placébo.

Dans ce cas l'utilisation d'une méthode en double aveugle :


A - Ne se justifie pas puisque les malades sont répartis en deux groupes
B - Est impossible parce que l'expérimentateur ignore quel groupe recoit le .i.Magicor;®
C - A pour but de supprimer les biais liés à la subjectivité du malade
D - A pour but de supprimer les biais liés à la subjectivité de l'expérimentateur
E - A pour but d'égaliser les caractéristiques des sujets des deux groupes
Bonne(s) réponse(s) : C D

Le but par définition du double aveugle qui l'impose est contenu dans ces 2 réponses.

1340
Exclusivement sur DOC - DZ : www.doc-dz.com NADJI 85
RESIDANAT EN POCHE TOME II
Cas Clinique en QCM

Dans ce cas, la randomisation :


A - Ne se justifie pas puisque les malades sont répartis en deux groupes
B - Est impossible parce que l'expérimentateur ignore quel groupe recoit le .i.Magicor;®
C - A pour but, de supprimer les biais liés à la subjectivité de l'expérimentateur
D - A pour but de supprimer les biais liés à la subjectivité du malade
E - A pour but d'égaliser les caractéristiques des sujets des deux groupes
Bonne(s) réponse(s) : E

A B. Sont toujours faux.


C D. Sont les deux buts de l'utilisation d'une méthode en double aveugle.
E. Est la seule justification de la randomisation.

Quel indicateur vous paraît-il le plus adapté pour vérifier les dires du laboratoire pharmaceutique à la fin de
l'essai ?
A - L'incidence de l'hypertension artérielle
B - L'évolution des chiffres tensionnels
C - Le risque relatif d'accident vasculaire cérébral dans les deux groupes
D - Le taux proportionnel de mortalité par cardiopathies ischémiques dans les deux groupes
E - La létalité des accidents vasculaires cérébraux dans les deux groupes
Bonne(s) réponse(s) : C

Le but annoncé de médicament n'est pas de jouer sur l'HTA mais de prévenir ses conséquences morbides, ce n'est pas non
plus de savoir si leur gravité est plus ou moins grande. Le risque relatif d'A.V.C. permet de comparer la fréquence d'apparition
de cette complication dans les 2 groupes.

Quel est le biais le plus à craindre dans cet essai tel qu'il est proposé ?
A - L'hétérogénéité des deux groupes
B - Un biais de mesure
C - Le nombre des perdus de vue
D - Un biais de mémorisation
E - Une variation temporelle de l'incidence de l'HTA
Bonne(s) réponse(s) : C

Sur une durée de 5 ans, il risque d'y avoir un certain nombre de perdus de vue. Le problème est qu'ils ne soient pas
également répartis dans les deux groupes (par exemple à cause d'une surmortalité), ce qui rend le biais évident.

En France en 1980, les décès foeto-infantiles se décomposaient de la façon suivante pour 1000 naissances (enfants vivants
ou non) et 50 millions d'habitants :

- morts nés 10
- décès de 0 à 7 jours 6
- décès de 7 à 28 jours 2
- décès de 28 jours à 1 an 4

Le taux de mortalité infantile est de :


A - 22/50.000.000
B - 2 2/1.000
C - 12/50.000.000
D - 12/1.000
E - 12/990
Bonne(s) réponse(s) : E

Le taux de mortalité infantile est le nombre de décès au cours de la première année (6 + 2 + 4 =12) rapporté à 1000
naissances vivantes. Les chiffres donnés étant rapportés au nombre de naissances totales, il faut déduire les 10 morts-nés
soit 12/990.
A C. Les taux sont toujours ramenés à 1000.
B. Est le taux de mortalité foeto infantile.

Le taux de mortalité périnatale est-il de :


A - 16/1.000
B - 18/1.000
C - 8/1.000
D - 6/1.000
E - 10/1.000
Bonne(s) réponse(s) : A

A. Taux de mortalité périnatale: nombre de morts-nés + nombre de décès au cours des 7 premiers jours de vie rapportée à
1000 naissances totales : soit 10 + 6 = 16/1000.
E. Taux de mortinatalité ou mortalité foetale tardive.
1341
Exclusivement sur DOC - DZ : www.doc-dz.com NADJI 85
RESIDANAT EN POCHE TOME II
Cas Clinique en QCM

Pour comparer entre différents pays les causes exogènes de mortalité : foeto-infantile, quelle est la donnée la
plus fiable ?
A - Le nombre de décès de 0 à 7 jours
B - Le taux de mortalité périnatale
C - Le nombre de décès de 0 à 28 jours
D - Le taux de mortalité néonatale précoce
E - Le taux de mortalité post-néonatale
Bonne(s) réponse(s) : E

Les causes exogènes se différencient le plus des causes endogènes pour la période la plus tardive, à savoir ici la période
post-néonatale (décès entre 29 jours et 365e jour de vie rapporté à 1000 naissances vivantes).

Parmi les propositions ci-dessous laquelle(lesquelles) caractérise(nt) la mortinatalité ?


A - C'est le nombre de mort-nés à partir de la 28ème semaine de gestation
B - C'est le nombre de décès de la 28 ème semaine de gestation au 7ème jour après la naissance
C - C'est le nombre de décès de 0 à 6 Jours
D - C'est l'indicateur le moins sensible aux actions de protection maternelle et infantile
E - C'est un mauvais indicateur des causes endogènes de mortalité foeto infantile
Bonne(s) réponse(s) : A D

La mortalité est le nombre de morts-nés (28 semaines révolues d'aménorrhée) rapporté à 1000 naissances totales (A). C'est
un bon indicateur des causes endogènes de mortalité foeto-infantile (E faux), et par la même, moins sensible aux notions de
P.M.I. qui agissent surtout sur les facteurs exogènes.
B - Caractérise la mortalité périnatale.
C - Caractérise la mortalité néo-natale précoce.

Médecin, vous suivez depuis plusieurs années un couple de personnes âgées Mr et Mme N...
Lui, 82 ans, diabétique, présente des séquelles d'hémiplégie et se déplace difficilement à l'intérieur du domicile. Il a besoin
d'aide pour tous les actes d'hygiène. Elle, 78 ans, valide, est traitée périodiquement pour un état dépressif Ils n'ont pas
d'enfants et vivent dans un petit logement vétuste, au 2ème étage sans ascenseur, ils ne bénéficient que de très faibles
ressources.

Parmi les services ci-dessous, lequel(lesquels) vous paraît(paraissent) adapté(s) et peut(peuvent) les aider
dans leur choix de rester à leur domicile ?
A - Soins à domicile
B - Hospitalisation à domicile
C - Aide ménagère
D - Garde malade
E - Foyer restaurant
Bonne(s) réponse(s) : A C

A. Apporte, sur prescription et sous contrôle du médecin généraliste, des soins infirmiers, de nursing et d'hygiène ou des soins
plus techniques. Le forfait journalier est pris en charge en totalité par la Caisse d'Assurance Maladie.
B. Nécessite une hospitalisation préalable.
C. Apporte une aide matérielle et psychologique aux personnes âgés isolées ou dépendantes. Prise en charge dans ce cas
par l'aide sociale.
D. Non, ce couple n'a pas de ressources.
E. Les foyers restaurants permettent une alimentation équilibrée mais ne peuvent résoudre les problèmes d'hygiène corporelle.

Mme N... doit être hospitalisée pour un nouvel état dépressif. Que conseillez-vous à Mr N... comme solution
temporaire parmi les équipements et services ci-dessous existant à proximité ?
A - Un placement en foyer logement
B - Un placement en maison de retraite avec section de cure médicalisée
C - Un placement en hospice
D - Une hospitalisation
E - Une hospitalisation à domicile
Bonne(s) réponse(s) : B

A. L'autonomie de Mr N ne permet pas son placement en foyer logement.


B. Ces sections de cure médicale permettent d'éviter le placement en institutions sanitaires en assurant la surveillance
médicale des personnes âgées la nécessitant.
C. Les hospices ont disparu.
D. Semble abusif.
E. Nécessité auparavant une hospitalisation.

1342
Exclusivement sur DOC - DZ : www.doc-dz.com NADJI 85
RESIDANAT EN POCHE TOME II
Cas Clinique en QCM

Quelques mois plus tard le maintien à domicile de Mr N... devient impossible (décompensation de son diabète,
quasi-cécité) d'autant que Mme N... reste très dépressive. Près de leur domicile existe un établissement de long
séjour médicalisé, ils s'inquiètent des conditions financières. Vous leur répondez :
A - Que les frais sont effectivement à la charge du patient
B - Que les frais sont entièrement à la charge de la Sécurité Sociale
C - Qu'une partie des frais, correspondante aux soins, est à la charge de la Sécurité Sociale
D - Qu'ils bénéficieront d'une prise en charge par l'aide sociale des frais d'hébergement
E - Qu'ils pourront bénéficier d'une allocation logement en déduction des frais de séjour
Bonne(s) réponse(s) : C D

Le prix de journée est divisé :


- en forfait hébergement : qui est à la charge du malade, de sa famille ou de l'aide sociale
- en forfait soin, pris en charge par la Sécurité Sociale.

Laquelle(lesquelles) des propositions ci-dessous justifie(nt) elle(s) cette décision d'admission en long séjour
médicalisé ?
A - L'existence d'une pathologie chronique
B - L'insalubrité du logememt
C - L'absence d'environnement socio-affectif
D - La dépendance physique
E - Le faible niveau de ressources
Bonne(s) réponse(s) : A D

Les services de long séjour accueillent les personnes âgés atteintes d'une affection chronique, nécessitant un environnement
médical permanent.

Des médecins généralistes, faisant partie d'une association locale de promotion de santé, souhaitent pour guider une action
de prévention, effectuer une enquête épidémiologique sur les accidents de l'enfant d'âge scolaire dans leur quartier. Pendant
trois mois, tous les accidents survenant dans ce quartier de dix mille habitants sont colligés.

Quelle(s) est(sont) ou quelles sont la(les) ou les caractéristiques de cette enquête ?


A - Exhaustive
B - Sur échantillon représentatif
C - Descriptive
D - Analytique
E - Expérimentale
Bonne(s) réponse(s) : A C

Cette enquête est exhaustive (A) car on enregistre tous les accidents de tous les enfants. Il n'y a pas eu d'échantillonnage (B).
Nous sommes dans une optique descriptive (C) et non analytigue (D) car il n'est pas possible avec ce simple recueil d'étudier
l'association entre un facteur de risque et les accidents. Nous ne sommes pas non plus en situation expérimentale car il n'y a
pas eu intervention (préventive par exemple).

15 accidents sont survenus au cours de la période d'étude. Ces résultats permettent d'évaluer sur le quartier :
A - Le taux d'incidence annuelle qui est de 60 pour mille
B - Le taux d'incidence annuelle qui est de 15 pour mille
C - Le taux de prévalence annuelle qui est de 60 pour mille
D - Le taux de prévalence
E - Aucun de ces paramètres
Bonne(s) réponse(s) : E

Le nombre de nouveaux cas par an serait de 15 X 4 = 60. Mais la population exposée est de 10 000 habitants et non 1000.

60 % des accidents sont survenus en milieu défavorisé. Que conclure du rôle des conditions socio-
économiques sur la survenue des accidents ?
A - Il y a relation de cause à effet
B - Le milieu défavorisé est facteur de risque de la survenue d'accidents
C - On ne peut conclure car l'enquête n'est pas effectuée sur échantillon représentatif
D - On ne peut conclure sans connaître les conditions socio-économiques du quartier
E - On ne peut conclure car il s'agit d'une enquête d'observation
Bonne(s) réponse(s) : D

Une éventuelle relation entre conditions socio-économiques et accidents n'est possible à établir que si on connait ce facteur
dans la population générale. Par exemple si 80 % de ce quartier vit en milieu défavorisé, on concluera à un rôle protecteur de
la C.S.F, faible. Si 60 % du quartier vit en milieu défavorisé, il n'y a pas de liaison entre C.S.E. - accidents. Si 40 % du quartier
est défavorisé, le pourcentage étant plus élevé chez les accidentés, on concluera à une liaison entre milieu défavorisé et
accident.

1343
Exclusivement sur DOC - DZ : www.doc-dz.com NADJI 85
RESIDANAT EN POCHE TOME II
Cas Clinique en QCM

Sur les 15 accidents, 5 ont été mortels. Quel est le taux de létalité des accidents de l'enfant ?
A - 33 pour 1000
B - 5 pour 10.000
C - 20 pour 10.000
D - 13,2 pour 10.000
E - 13 pour 100
Bonne(s) réponse(s) :

Impossible. Ce taux de létalité est le rapport du nombre d'accidents mortels au nombre d'accidents totaux, soit 5/15 = 33 %. C.
Est le taux de mortalité spécifique par accident 20 morts/an.

Si ce quartier se comporte comme l'ensemble de la population française, quelle est la première cause de décès
par accident de l'enfant à laquelle on peut s'attendre ?
A - Noyades
B - Accidents de circulation
C - Intoxications médicamenteuses
D - Chutes
E - Brûlures
Bonne(s) réponse(s) : B

De 5 à 14 ans, la première cause de mortalité : accidents de circulation (vélo, vélomoteur), puis les chutes et les noyades.

Pour évaluer l'efficacité du vaccin coquelucheux dans la prévention de la coqueluche les données suivantes ont été collectées
en 1983 dans la fratrie des enfants qui avaient la maladie :

Cliquez sur le bouton "Dessin" pour afficher le tableau.

Quelle est l'incidence de la coqueluche chez les sujets contacts vaccinés contre cette maladie ?

Cliquez sur le bouton "dessin" pour afficher le tableau.


A-0%
B - 10 %
C - 25 %
D - 45 %
E - 75 %
Bonne(s) réponse(s) : B

Nombre de cas apparus de coqueluche, 400, sur nombre de sujets exposés (contacts vaccinés) 4 000.
400 / 4.000 = 10%.

Quel est le risque relatif de contracter la coqueluche chez les enfants non vaccinés par rapport aux enfants
vaccinés ?
Cliquez sur le bouton "dessin" pour afficher le tableau.
A - 0,25
B - 0,50
C - 1,0
D - 2,0
E - 4,0
Bonne(s) réponse(s) : E

Le risque relatif est le rapport entre l'incidence de la coqueluche chez les contacts non vaccinés et l'incidence chez les contats
vaccinés soit : (400/1000) / (400/4000) = 4,0.

Le vaccin coquelucheux est un vaccin :

Cliquez sur le bouton "dessin" pour afficher le tableau.


A - Vivant atténué
B - Bactérien entier inactivé
C - Synthétique
D - Polysaccharidique
E - Anatoxinique
Bonne(s) réponse(s) : B

Il s'agit d'un vaccin complet, détoxifié par la chaleur.

1344
Exclusivement sur DOC - DZ : www.doc-dz.com NADJI 85
RESIDANAT EN POCHE TOME II
Cas Clinique en QCM

Laquelle ou lesquelles des propositions ci-dessous constituent une ou des contre-indication(s) du vaccin
coquelucheux ?
Cliquez sur le bouton "dessin" pour afficher le tableau.
A - Déficit immunitaire
B - Diabète
C - Albuminurie
D - Antécédents de convulsions fébriles
E - Trisomie 21
Bonne(s) réponse(s) : D

On signale des réactions neurologique (1/10 000 vaccinations) avec convulsions hyperthermiques et parfois des
encéphalopathies.

Le meilleur test diagnostique d'anémie est le dosage de l'hémoglobine. En prenant comme borne de normalité, la valeur de 12
g/dl, on retrouve sur une population de 100 personnes 19 anémiques bien classés et 10 sujets normaux considérés comme
anémiques à tort (faux positifs). Par des méthodes plus complexes que le dosage de l'Hb, on a identifié avec certitude 20 cas
d'anémie parmi ces 100 personnes.
Pour tenter d'améliorer le score, on déplace la borne à 10 g. La sensibilité passe à 75 % et 2 sujets normaux sont toujours mal
classés.

Quelle est la sensibilité du premier test (Hb = 12g) ?


A - 60 %
B - 70 %
C - 75 %
D - 80 %
E - 95 %
Bonne(s) réponse(s) : E

On peut reconstituer pour le premier test (Hb = 12 g) le tableau suivant :

Malades Non malades Total


Test positif 19 10 29
Test négatif 1 70 71
Total 20 80 100

La sensibilité est la proportion de vrais positifs chez les malades soit :


19/20 = 95 %.

Quelle est la spécificité du premier test (Hb = 12 g) ?


A - 67,5 %
B - 70 %
C - 80 %
D - 87,5 %
E - 92,5 %
Bonne(s) réponse(s) : D

La spécificité est la proportion de vrais négatifs chez les non-malades soit :


70/80 = 87,5 %

Quelle est la spécificité du 2ème test (Hb = 10 g) ?


A - 62,5 %
B - 67,5 %
C-75%
D - 87,5 %
E - 87,5 %
Bonne(s) réponse(s) : E

Il est possible de reconstituer le tableau du 2ème test (Hb = 10 g). En effet, il y a toujours 20 malades sur 100 personnes, si la
sensibilité devient 75 %, cela signifie que le nombre de vrais positifs passe à 15 15/20 (= 75 %). Par ailleurs, il reste 2 faux
positifs.

Malade Malades Malades Non Malades Tolal


Test positif 15 2 17
Test négatif 5 78 83
Total 20 80 100

La spécificité de ce test est de 78/80 = 97,5 %

1345
Exclusivement sur DOC - DZ : www.doc-dz.com NADJI 85
RESIDANAT EN POCHE TOME II
Cas Clinique en QCM

Combien de sujets anémiques ont échappé au diagnostic avec la borne à 10g d'hémoglobine ?
A-1
B-3
C-5
D-6
E - Aucun des chiffres ci-dessus
Bonne(s) réponse(s) : C

Le nombre des sujets ayant échappé au diagnostic de ce 2ème test correspond au nombre de faux-négatifs, à savoir 5. Il y a
20 anémiques avec une sensibilité de 75 %, il y en a 15 avec un test positif et 5 avec un test négatif.

Un essai thérapeutique comparatif vise à apprécier l'efficacité d'un nouvel antihypertenseur, qui est contre-indiqué chez les
insuffisants rénaux. Le groupe témoin recevra un traitement de référence classique, d'efficacité démontrée.

Pour que l'essai soit pratiqué en double aveugle, il est impératif de respecter :
A - Méconnaissance par le patient du traitement utilisé
B - Méconnaissance par le médecin du traitement prescrit
C - Médicament à tester et médicament de référence d'apparence aussi proche que possible
D - Utilisation d'un placébo dans le groupe témoin
E - Utilisation du médicament à tester, à doses différentes dans les 2 groupes
Bonne(s) réponse(s) : A B C

A B. Sont les 2 aspects du double aveugle.


C. En est une conséquence.évidente.
D. Est faux puisque c'est un essai comparatif fait à un traitement de référence.
E . Il n'est pas question d'étude dose-effet mais de comparaison avec un traitement classique.

Certains patients sont sous tonicardiaques pour traiter une insuffisance cardiaque secondaire à leur
hypertension.
Quelle(s) attitude(s) peut-on adopter vis-à-vis de ces patients ?
A - Arrêter les tonicardiaques pendant l'étude
B - Les laisser dans l'étude et tenir compte du traitement associé lors de l'analyse des résultats
C - Les exclure de l'étude
D - N'arrêter les tonicardiaques que dans le groupe recevant le nouveau traitement
E - Les laisser dans l'étude sans en tenir compte lors de l'analyse
Bonne(s) réponse(s) : B C

B. La présence d'un traitement associé ne perturbe pas l'essai à la seule condition qu'il soit utilisé de façon comparable dans
les 2 groupes ce qui est garanti par le tirage au sort. Ceci est l'attitude pragmatique.
C. L'attitude explicative serait pour ne pas se heurter à l'hypothèse d'une interaction entre traitement étudié et traitement
associé, différente de l'interaction avec le traitement de référence qui serait source de biais de ne pas inclure ces malades
dans l'étude.

Quelle attitude adopter vis-à-vis d'un patient insuffisant rénal ?


A - L'exclure de l'étude d'emblée
B - Ne l'exclure que si le tirage au sort le répartit dans le groupe du nouveau traitement
C - Le maintenir dans l'étude sans tenir compte de l'insuffisance rénale
D - Le répartir d'office dans le groupe témoin
E - Le changer de groupe si le tirage au sort le répartit dans le groupe qui recevra le nouveau traitement
Bonne(s) réponse(s) : A

Il n'est pas éthiquement possible de faire participer un tel malade, (pour lequel le traitement étudié est contre-indiqué), à
l'étude. Il faut l'exclure d'emblée (avant tirage au sort). Toute autre attitude est source de biais.

A l'issue de l'étude, les calculs statistiques ne font pas apparaître de différence significative entre les résultats
dans les deux groupes. Comment formuler ces résultats ?
A - Le nouveau traitement est inefficace
B - Le nouveau traitement n'est pas plus efficace que le traitement de référence
C - Le traitement de référence est meilleur
D - Dans les conditions de l'essai, il n'a pas été montré de différence significative d'action entre les deux
traitements
E - L'étude ne permet pas de conclusions
Bonne(s) réponse(s) : D

D. Est la seule formulation possible.

1346
Exclusivement sur DOC - DZ : www.doc-dz.com NADJI 85
RESIDANAT EN POCHE TOME II
Cas Clinique en QCM
Le graphique ci-dessous représente de façon simplifiée les 60 cas de tuberculose survenus dans dans un camp de 1000
réfugiés du sud est asiatique en France.

Cliquez sur le bouton "Dessin" pour afficher le tableau.

Le taux de prévalence pour le mois de janvier est de :

Cliquez sur le bouton "dessin" pour afficher le tableau.


A - 20/60
B - 30/60
C - 20/1000
D - 30/1000
E - 60/1000
Bonne(s) réponse(s) : E

La prévalence pour le mois de janvier reste égale au nombre de cas de tuberculose recensés pendant ce mois rapportés à la
population exposée soit 60 cas pour 1000 réfugiés.

Le taux d'incidence durant le mois de janvier est de :

Cliquez sur le bouton "dessin" pour afficher le tableau.


A - 20/60
B - 30/60
C - 20/1000
D - 30/1000
E - 60/1000
Bonne(s) réponse(s) : D

L'incidence est le nombre de nouveaux cas ayant débuté pendant ce mois rapporté à la population de référence soit 30
nouveaux cas pour 1000 sujets (lignes 2,4,5 du graphique).

Le taux de létalité durant le mois de janvier est de :

Cliquez sur le bouton "dessin" pour afficher le tableau.


A - 20/60
B - 30/60
C - 20/1000
D - 30/1000
E - 60/1000
Bonne(s) réponse(s) : A

Le taux de létalité est le nombre de décès sur le nombre de malades.

Selon les règles utilisées en France, quelle(s) est(sont) la(les) mesure{s) nécessaire(s) vis-à-vis de malades
tuberculeux bacillifères dépistés ?
Cliquez sur le bouton "dessin" pour afficher le tableau.
A - Isolement
B - Vaccination BCG immédiate
C - Traitement par 3 antibiotiques antituberculeux pendant au moins 9 mois
D - Traitement par 2 antibiotiques antituberculeux pendant 6 mois
E - Déclaration obligatoire
Bonne(s) réponse(s) : A C E

Le patient sera hospitalisé 15 jours s'il est bacillifère.


On dépistera également les sujets contacts par un cliché du thorax initial, à renouveller au cours de 3ème mois, de même que
les tests tuberculiniques.

1347
Exclusivement sur DOC - DZ : www.doc-dz.com NADJI 85
RESIDANAT EN POCHE TOME II
Cas Clinique en QCM

Parmi les mesures ci-dessous, laquelle(lesquelles) est(sont) justifiée(s) pour les enfants du camp dont l'examen
radiologique et clinique est normal et dont l'intradermoréaction à 10 unités détermine une induration
inflammatoire de 15 mm, incapables de préciser leurs vaccinations antérieures ?
Cliquez sur le bouton "dessin" pour afficher le tableau.
A - Isolement
B - Vaccination BCG immédiate
C - 3 antibiotiques antituberculeux pendant au moins 9 mois
D - 2 antibiotiques antituberculeux pendant 6 mois
E - Simple surveillance
Bonne(s) réponse(s) : D

Les primo -infections tuberculeuses latentes sont traitées par I.N.H®. pendant 6 mois et rifampicine pendant 3 mois.

On a évalué l'hémocult comme méthode de dépistage des cancers rectocoliques dans une région rurale de Grande Bretagne.
La totalité des hommes de 35 à 55 ans (3 000 personnes) a subi un hémocult. Pour certains d'entre eux le diagnostic de
cancer rectocolique était connu. Les autres ont subi un examen clinique et endoscopique.
Les résultats sont les suivants : 30 cancers au total ont été recensés dont 25 avaient un hémocult positif. Parmi les individus
indemnes, 250 avaient un hémocult positif.

Dans cette étude, le taux d'incidence du cancer recto-colique chez les hommes de 35 à 55 ans est de :
A - 10 pour 1 000
B - 25 pour 1 000
C - 25 pour 250
D - un autre nombre
E - on ne peut pas le calculer
Bonne(s) réponse(s) : E

C'est une question piège, si A,B,C sont de toute évidence faux, répondre 30/3000 = 10/1000 revient à parler de prévalence.
L'incidence concerne les nouveaux cas, et nous n'avons aucune information sur eux.

Dans cette étude, le taux de prévalence du cancer rectocolique chez les hommes de 35 à 55 ans est de :
A - 10 pour 1000
B - 25 pour 1000
C - 2 5 pour 250
D - Un autre nombre
E - On ne peut pas le calculer
Bonne(s) réponse(s) : A

Le résultat dévoilé dans la question précédente est bien celui recherché ici :
nombre de cas présents/nombre de sujets exposés = 30/3 000 = 10/1000.

Dans cette étude, la sensibilité du test de dépistage (hémocult) est de :


A - 2720 sur 2970
B - 25 sur 275
C - 25 sur 30
D - Une autre valeur
E - On ne peut pas le calculer
Bonne(s) réponse(s) : C

Reconstituons le tableau de contingence avec la répartition des hémocults positifs et négatifs chez les sujets présentant un
cancer rectocolique et les autres.
Cancer rectocolique Sujets indemnes Total
Hemoccult + 25 250 275
Hemoccult- 5 2720 2725
Total 30 2970 3000

Nous avons souligné les effectifs donnés dans l'énoncé et à partir desquels on peut faire tout reconstruire.
Sensibilité : proportion de vrais positifs (25) chez les sujets cancereux : 25/30.

Dans cette étude, la spécificité du test de dépistage (hémocult) est de :


A - 2720 sur 2970
B - 25 sur 275
C - 25 sur 30
D - Une autre valeur
E - On ne peut pas le calculer
Bonne(s) réponse(s) : A

Spécificité : proportion de vrais négatifs (2720) chez les non cancéreux (2970).
1348
Exclusivement sur DOC - DZ : www.doc-dz.com NADJI 85
RESIDANAT EN POCHE TOME II
Cas Clinique en QCM

On améliore la technique de l'hémocult dans le sens d'une réduction des faux positifs sans que varie le nombre
des faux négatifs On peut dire :
A - Que la sensibilité s'améliore
B - Que la spécificité s'améliore
C - Que la valeur prédictive positive s'améliore
D - Que la valeur prédictive négative s'améliore
E - Que ce test a une meilleure valeur diagnostique
Bonne(s) réponse(s) : B C D E

S'il y a 30 sujets cancereux, il y a 2 970 sujets indemnes, la spécificité s'améliore.


C. La valeur prédictive positive est la proportion de malades chez les sujets avec hémoccult positif. Ce dernier chiffre
diminuant, la V.P.P s'améliore.
D. La valeur prédictive négative est la proportion de non malades chez les sujets avec hémocult négatif (V.N/V.N. + F.N.),
puisque V.N. augmente la V.P.N. s'améliore.
E. Quel que soit l'indice que l'on met derrière ce terme général de valeur diagnostique, sur les 4 indices précédemment
calculés, 3 s'améliorent, 1 est inchangé, on peut donc affirmer que ce test a une meilleure valeur diagnostique.

Dans une petite ville de Grande Bretagne (50 000 habitants) existe depuis plusieurs années un registre des cancers, qui
recense tous les cas de cancesr diagnostiqués.
On se propose d'étudier en 1982 le rôle du tabagisme : notion d'exposition âge de début, importance de la consommation,
dans deux groupes :
- un groupe constitué par tous les malades atteints de cancer bronchique figurant dans le registre en 1981
- un groupe de même taille de sujets indemnes de cette affection.

Cette enquête épidémiologique est une étude :


A - Prospective
B - Rétrospective
C - Exposés - non exposés
D - Cas-témoins
E - Expérimentale
Bonne(s) réponse(s) : B D

A C. Une enquête prospective s'intéresse au devenir par exemple de 2 cohortes de sujets exposés ou non exposés par
rapport à un facteur de risque, quant à l'apparition d'une maladie.
B. Une enquête rétrospective s'intéresse de façon rétroactive à des différences d'exposition (ici le tabagisme) dans deux
populations (cancers bronchiques - témoins), afin d'étudier le rôle de ce facteur d'exposition sur la maladie. C'est une enquête
cas-témoin.
E. Une enquête expérimentale vise à évaluer l'effet d'une intervention en contrôlant l'effet de cette intervention dans différents
groupes. Cette intervention peut être aussi bien un traitement (essai thérapeutique) qu'une mesure de santé publique.

Le groupe de sujets indemnes est constitué de façon à ce que pour chaque malade, on choisisse un individu
sain ayant les mêmes caractéristiques d'âge, de sexe, socio-professionnelles, d'ethnie...
Cette méthode constitue en épidémiologie :
A - Un sondage
B - Un échantillonnage représentatif
C - Un appariement
D - Une standardisation
E - Une étude en double aveugle
Bonne(s) réponse(s) : C

L'appariement consiste à constituer un groupe témoin qui, dans l'idéal, ne différerait du groupe étudié, que par la présence de
la maladie chez les uns et pas chez les autres. On essaie donc pour toutes les caractéristiques que l'on pense pouvoir influer
sur la maladie ou sur le tabagisme, de les neutraliser en ayant la même répartition dans les 2 groupes.

Pour cette ville de 50 000 habitants, 50 personnes atteintes de cancer bronchique figurent dans le registre de
l'année 1981. 10 personnes sont décédées de cette maladie durant l'année. Le taux de 20 pour 100 000
correspond pour cette ville en 1981 :
A - Au taux de mortalité proportionnelle par cancer bronchique
B - Au taux de mortalité spécifique par cancer bronchique
C - Au taux de prévalence du cancer bronchique
D - Au taux de létalité du cancer bronchique
E - Au taux brut de mortalité dans la population
Bonne(s) réponse(s) : B

A. Est le rapport entre le taux de morlalité spécifique par cancer bronchique sur le taux brut de mortalité dans la population E
(c'est-à-dire toutes causes confondues, que l'on ne connait pas)
B. Taux de mortalité par cancer bronchique est le nombre de morts par cancer bronchique : 10 rapporté à la population
exposée, 50 000 habitants. Ce taux de 10 sur 50 000 est ramené à une échelle classique de 20 sur 100 000.
C. Le taux de prévalence est de 50/50 000 soit 100/100 000.
D. Le taux de létalité est de 10/50 = 0,2.

1349
Exclusivement sur DOC - DZ : www.doc-dz.com NADJI 85
RESIDANAT EN POCHE TOME II
Cas Clinique en QCM

Quelle donnée manque pour que l'ensemble des cinq taux proposés dans la question précédente puissent être
calculés pour 1981 dans cette ville ?
A - Nombre total de décès dans cette ville en 1981
B - Nombre de nouveaux cas de cancers bronchiques apparus en 1981
C - Structure par âge de la population
D - Une autre donnée que les trois précédentes
E - Il ne manque aucune donnée
Bonne(s) réponse(s) : A

Nous n'avons que les 2 taux non calculables à la question précédente : taux brut de mortalité et taux de mortalité
proportionnelle imposent d'avoir le nombre de décès toutes causes confondues dans cette population pendant la période de
référence.

Pour mieux étudier s'il y a relation entre la contraception par voie orale (oestroprogestatifs) et le cancer du col, des auteurs ont
suivi de décembre 1976 à janvier 1980 un groupe de 15 000 femmes de 18 à 58 ans indemnes de cancer du col à l'entrée
dans l'étude. La surveillance consistait essentiellement en un frottis annuel, et parallèlement la prise de contraceptif était
recherchée par questionnaire. Les résultats de l'enquête en fonction de la durée de la prise de contraceptifs sont les suivants :

Cliquez sur le bouton "Dessin" pour afficher le tableau.

De quel(s) type(s) d'enquête s'agit-il ?

Cliquez sur le bouton "dessin" pour afficher le tableau.


A - Enquête longitudinale
B - Enquête prospective
C - Enquête de cohorte
D - Enquête épidémiologique analytique
E - Enquête cas-témoins
Bonne(s) réponse(s) : A B C D

A. Cette enquête s'étale sur 5 ans et permet ainsi de mesurer l'incidence de la maladie car c'est une enquête longitudinale
(s'opposant à transversale).
C. C'est une étude de cohorte puisque l'on suit une population de sujet indemne pendant un certain temps.
B. Cette enquête est prospective car on surveille l'apparition d'un évènement (le cancer du col).
D. L'épidémiologie analytique recherche les causes d'une maladie en mettant en évidence des liaisons entre la maladie et des
facteurs d'exposition (ici la contraception).
E. La maladie sert à différencier deux groupes ce qui n'est pas du tout le cas ici.

Avez-vous suffisamment d'éléments pour compléter le tableau et calculer le taux d'incidence du cancer du col
chez les femmes ayant utilisé un contraceptif pendant plus de 4 ans ?
Cliquez sur le bouton "dessin" pour afficher le tableau.
A - Non, il faudrait connaître le nombre de femmes qui ont été exposées pendant plus de 4 ans
B - Non, car on ne connaît pas l'incidence chez les non exposées
C - Non, il faudrait connaître l'incidence du cancer du col dans la population générale
D - Oui : incidence = 160/100.000
E - Oui, car le risque relatif est un rapport d'incidence
Bonne(s) réponse(s) : D E

E. L'incidence chez les sujets non exposés est connue, c'est 32. L'incidence chez les femmes ayant utilisé un contraceptif oral
depuis plus de 4 ans est le produit de l'incidence du cancer chez les sujets non exposés (32) par le risque relatif (5) :
5 X 32/100 000 = 160/100 000.
En effet le risque relatif est bien le rapport entre l'incidence de la maladie chez les femmes exposées à l'incidence chez les
femmes non exposées.

Le fait que le risque relatif augmente de 1 à 5 en fonction de l'exposition aux risques :

Cliquez sur le bouton "dessin" pour afficher le tableau.


A - Confirme l'hypothèse selon laquelle la pilule est un facteur de risque du col
B - Prouve que la pilule est la cause du cancer du col
C - Est un argument en faveur de la causalité
D - N'a aucune signification étiologique dans ce type d'enquête
E - N'a rien à voir avec la relation de causalité
Bonne(s) réponse(s) : A C

A. Le risque relatif augmentant avec la prise de pilule, la pilule est un facteur de risque.
B. On ne parle de la "cause" ne serait-ce que l'incidence n'est pas nulle chez les sujets non exposées.
C. Une augmentation du risque relatif en même temps que l'importance de l'exposition est certainement un argument en
faveur de la causalité (E) et de même signification étiologique (D).

1350
Exclusivement sur DOC - DZ : www.doc-dz.com NADJI 85
RESIDANAT EN POCHE TOME II
Cas Clinique en QCM

Comment, à votre avis, a-t-on procédé pour chiffrer le risque relatif ?

Cliquez sur le bouton "dessin" pour afficher le tableau.


A - On a utilisé une méthode d'approximation
B - On a calculé directement le risque relatif par le rapport de 2 incidences
C - On a comparé la prise de contraceptifs chez les femmes atteintes de cancer du col et des femmes
indemnes
D - On a comparé la fréquence de survenue du cancer du col chez les utilisatrices de contraceptif
E - On a fait une étude cas témoin
Bonne(s) réponse(s) : B D

B D. Le risque relatif est le rapport de 2 incidences, la fréquence de survenue du cancer chez les utilisatrices de contraceptif
et la fréquence chez les non-utilisatrices.
A C E. Dans les enquêtes cas-témoins ,on n'a qu'une estimation du risque relatif, car ayant un groupe de malades et un
groupe de témoins on ne peut mesurer l'incidence de la maladie.

Dans un département français comptant un million d'habitants avec un sex-ratio de un, on a dénombré en 1982, 20 080
naissances dont 80 morts-nés. Sur l'année on a relevé 12 000 décès dont 320 avant l'âge d'un an, 500 entre 1 et 4 ans, 500
entre 5 et 14 ans. Dans ce département. il existe un registre du cancer sur lequel ont été enregistrés, en 1982, 500 décès par
cancer du poumon.

Le taux de mortalité infantile en 1982 peut être estimé à :


A - 16 pour mille, donc inférieur à la moyenne nationale
B - 16 pour mille, donc supérieur à la moyenne nationale
C - 132 pour mille, donc inférieur à la moyenne nationale
D - 132 pour mille, donc supérieur à la moyenne nationale
E - Ne peut être estimé avec les données de l'énoncé
Bonne(s) réponse(s) : B

Le taux de mortalité infantile est le nombre de décès au cours de la première année de la vie rapporté à 1000 naissances
vivantes, soit :
320 / (20080 - 80) = (320 / 20 000) = (16 / 1000)
Il est supérieur à la moyenne nationale de 8/1000.
. 32/1000 est une prise en compte de tous les décès jusqu'à 14 ans.

Le taux de mortalité spécifique par cancer du poumon peut être évalué à :


A - 42 pour mille
B - 50 pour mille
C - 55 pour mille
D - 0,50 pour mille
E - 5 pour mille
Bonne(s) réponse(s) : D

Le taux de mortalité spécifique par cancer du poumon est le nombre de décès par cette cause rapportée à la population
exposée soit 500/1 000 000 = 0,5 pour mille.

Mille cas de cancer de l'utérus dont 240 nouveaux cas en 1982 sont enregistrés dans le registre. Le taux
d'incidence pour 1982 du cancer de l'utérus peut être évalué à :
A - 0,48 pour mille
B - 0,24 pour mille
C - 0,96 pour mille
D - 2 pour mille
E - 4,8 pour mille
Bonne(s) réponse(s) : A

Le sex-ratio est de un : il y a donc 500 000 femmes. Attention ici au fait que seule la population des femmes est exposée au
cancer de l'utérus.
L'incidence est donc de : 240 / 500 000 soit 0,48 pour 1000.
La prévalence est, elle, de 2 pour I 000.

1351
Exclusivement sur DOC - DZ : www.doc-dz.com NADJI 85
RESIDANAT EN POCHE TOME II
Cas Clinique en QCM

Le taux (national) de mortalité par maladie cardiovasculaire est de 37 %. Evaluer le nombre de décès d'origine
cardiovasculaire dans ce département :
A - Moins de 1000
B - 1000 à 1500
C - 2000 à 3000
D - 3500 à 4500
E - 5000 à 5500
Bonne(s) réponse(s) : D

Le taux proportionnel de mortalilé pour une cause est Ie rapport entre le nombre de décès pour cette cause et le nombre de
décès totaux ou plus exactement entre le taux de mortalité spécifique pour cette cause et le taux brut de mortalité. Nombre de
décès d'origine cardiovasculaire : 12 000 X 0,37 = 4 440.

Le 1er décembre, 1983 .dans une petite bourgade d'Indre et Loire de 2000 habitants le Dr M... est appelé auprès de plusieurs
jeunes enfants atteints d'une diarrhée aiguë. Il s'agit dans tous les cas d'une diarrhée modérément fébrile, sans gravité. Mais
le caractère épidémique de ces manifestations diarrhéiques amène le Dr M. a soupçonné une éventuelle pollution de l'eau de
distribution publique, eau traitée par chloration).

Pour connaître immédiatement les résultats des dernières analyses de cette eau, le Dr M... a la possibilité de
téléphoner :
A - Au service des eaux de la D.D.A.S.S. d'Indre et Loire
B - Au bureau municipal d'hygiène de cette commune
C - A la D.R.A.S.S.
D - Au comité de bassin
E - A l'agence financière de bassin
Bonne(s) réponse(s) : A

Pas de commentaire.

La dernière analyse. effectuée le 15 novembre 1983, a révélé :


- la présence de deux Escherichia Coli dans 100 ml d'eau
- un taux de nitrates à 75 mg/l.
Comment interprétez-vous ces résultats ?
A - Bactériologiquement, cette eau n'est pas potable
B - Bactériologiquement, cette eau est potable
C - Le taux de nitrates dépasse la limite maxima admissible qui est de 1 mg/l
D - Le taux de nitrates dépasse la limite maxima admissible qui est de 50 mg/l
E - Ces résultats suffisent pour expliquer l'épidémie de diarrhée
Bonne(s) réponse(s) : A D

Selon les normes définies par les pouvoirs publics, l'eau ne doit pas contenir de germes pathogènes (escherichia coli,
streptocoque fécal, clostridium). Le taux de nitrites doit être inférieur à 50 mg/l. Au delà, il existe un risque de
méthémoglobinémie chez le nourrisson.

Pour diminuer la teneur croissante des nitrates dans les eaux de consommation, la recommandation suivante
peut être proposée :
A - Diminuer les apports d'engrais chimiques
B - Eviter l'utilisation du DDT
C - Eviter les rejets de détergents dans les eaux de surface
D - Eviter les rejets d'hydrocarbures dans les eaux de surface
E - Diminuer certaines cultures (carottes, épinards)
Bonne(s) réponse(s) : A

Les nitrites proviennent des engrais agricoles, des eaux usées domestiques, et de certaines industries.

L'évolution de la diarrhée se révèle rapidement favorable pour tous les enfants. Toutes les coprocultures
demandées au début de chaque épisode diarrhéique, reviennent négatives. Parmi les étiologies suivantes,
laquelle est compatible avec cette épidémie ?
A - Dysenteries bacillaires
B - Infections à rotavirus
C - Salmonelloses
D - Excès de nitrates dans l'eau de distribution publique
E - Infections à klebsielles
Bonne(s) réponse(s) : B

A C. La coproculture permet habituellement de retrouver le germe en cause.

1352
Exclusivement sur DOC - DZ : www.doc-dz.com NADJI 85
RESIDANAT EN POCHE TOME II
Cas Clinique en QCM
Le schéma ci-dessous représente l'évolution d'une épidémie de tuberculose dans un foyer de 100 travailleurs immigrés
célibataires :

Cliquez sur le bouton "Dessin" pour afficher le tableau.

Dans le tableau, les ronds vides correspondent au début, le ronds pleins à la fin.

Le taux de prévalence pour le mois de janvier est de :

Cliquez sur le bouton "dessin" pour afficher le tableau.


A - 21/6
B - 2/100
C - 3/100
D - 6/100
E - Un autre chiffre
Bonne(s) réponse(s) : D

Le taux de prévalence est le nombre de cas enregistrés pendant la période de référence rapportée à la population exposée
soit: 6/100.

Le taux d'incidence pour le mois de janvier est de :

Cliquez sur le bouton "dessin" pour afficher le tableau.


A - 21/6
B - 2/100
C - 3/100
D - 6/100
E - Un autre chiffre
Bonne(s) réponse(s) : C

Le taux d'incidence est le nombre de nouveaux cas enregistrés pendant la période de référence rapportée au nombre de
sujets exposés. Ici 3/100.

Si tous les cas survenus se terminent par un décès, le taux de létalité pour le mois de janvier est de :

Cliquez sur le bouton "dessin" pour afficher le tableau.


A - 21/6
B - 2/100
C - 3/100
D - 6/100
E - Un autre chiffre
Bonne(s) réponse(s) : A

La létalité est le rapport entre le nombre de décès et le nombre de malades soit : 2/6. Il n'y a en effet que 2 évolutions se
terminant en janvier.
Signalons que cette question est très mal formulée et que la létalité est le plus souvent le rapport entre taux de mortalité et
incidence de la maladie, et non taux de mortalité sur
prévalence comme ici.

Quelle(s) est(sont) la(les) mesure(s) nécessaire(s) vis-à-vis des malades tuberculeux bacillifères dépistés dans
ce foyer ?

Cliquez sur le bouton "dessin" pour afficher le tableau.


A - Isolement
B - Vaccination BCG immédiate
C - Traitement par antibiotiques antituberculeux pendant au moins 9 mois
D - Traitement par antibiotiques antituberculeux pendant 4 mois
E - Déclaration à la Direction Départementale des Affaires Sanitaires et Sociales
Bonne(s) réponse(s) : A C E

On conseille un traitement triple (I.N.H., rifampicine, éthambutol) pendant 3 mois.

1353
Exclusivement sur DOC - DZ : www.doc-dz.com NADJI 85
RESIDANAT EN POCHE TOME II
Cas Clinique en QCM

Quelle(s) est(sont) la(les) mesure(s) nécessaire(s) vis-à-vis des pensionnaires du foyer dont l'examen
radiologique est normal et dont vous considérez l'intradermoréaction à 10 unités comme négative ?
Cliquez sur le bouton "dessin" pour afficher le tableau.
A - Vaccination BCG immédiate
B - Traitement par 2 antibiotiques antituberculeux pendant 6 mois
C - Traitement par 3 antibiotiques antituberculeux pendant 3 mois
D - Nouveau contrôle de l'intradermoréaction dans un mois
E - Isolement
Bonne(s) réponse(s) : D

Il faut conseiller également un nouveau cliché du thorax dans 3 mois.

Un essai thérapeutique randomise, en double aveugle. a pour but d'apprécier l'efficacité de l'aspirine. 500mg/jour, dans la
prévention des récidives d'accidents ischémiques cérébraux liés à l'athérosclérose. A l'époque où l'essai a été réalisé aucune
autre médicament n'avait démontré son efficacité dans cette indication.

Que prescrire au groupe témoin ?


A - Un autre antiagrégant plaquettaire
B - Un placebo
C - Rien
D - L'aspirine à faible dose (50mg)
E - L'association aspirine 50mg - autre antiagrégant plaquettaire
Bonne(s) réponse(s) : B

Donner un traitement placébo est la seule façon de préserver l'essai en double aveugle. Ce médicament sans principe actif
permet de séparer les deux composantes de l'action médicamenteuse (l'effet de suggestion sur le malade et son entourage)
qui sera le même quelque soit le médicament, et le principe pharmacologique actif qui n'existera que dans le groupe étudié.

Quelle attitude adopter vis-à-vis d'un patient présentant un ulcère gastrique ?


A - L'inclure dans l'étude sans tenir compte de l'ulcère
B - L'inclure dans le groupe témoin avant la randomisation
C - L'exclure de l'étude avant la randomisation
D - L'exclure de l'étude si la randomisation le repartit dans le groupe le groupe témoin
E - Le changer de groupe si la randomisation le répartit dans le groupe aspirine
Bonne(s) réponse(s) : C

L'ulcère est une contre-indicalion à la prise d'aspirine, il n'est donc pas éthiquement concevable d'en donner (A). Il faut
l'exclure de l'étude avant la randomisation. Toute modification après tirage au sort est source de biais et perte du double
aveugle (D.E.). Toute affectation arbitraire à un groupe va contre le principe de tirage au sort et d'attribution aléatoire du
traitement.

Dans les deux groupes, quelle attitude adopter vis-à-vis des traitements antihypertenseurs suivis par de
nombreux patients ?
A - L'arrêter pour tous les patients
B - Le maintenir uniquement dans le groupe aspirine 500 mg
C - Le maintenir uniquement dans le groupe témoin
D - Le maintenir dans !es deux groupes sans le mentionner
E - Le maintienir dans les deux groupes et en tenir compte dans l'analyse
Bonne(s) réponse(s) : E

Il faut poursuivre ces traitements car ils sont nécessaires. Par leur maintien dans les groupes c'est la seule garantie de
comparabilité des groupes. Dans tous les cas, il faut les mentionner pour pouvoir en tenir compte dans l'analyse. L'attitude vis-
a-vis de tels traitements associés doit être parfaitement précise dans le protocole et en particulier dans les critères d'inclusion
des sujets dans l'étude.

1354
Exclusivement sur DOC - DZ : www.doc-dz.com NADJI 85
RESIDANAT EN POCHE TOME II
Cas Clinique en QCM

100 patients ont été inclus dans chaque groupe, 20 patients étaient hypertendus dans le groupe aspirine
500mg, 10 patients dans le groupe témoin. Dans ce cas le risque relatif de faire un accident vasculaire si on est
hypertendu est :
A - Egal à 2
B - Egal à 0,5
C - Egal à 3
D - Calculable si le taux de létalité est commun
E - Incalculable avec le protocole de l'étude
Bonne(s) réponse(s) : E

Pour calculer le risque relatif, qui est le rapport de la fréquence des accidents vasculaires chez les hypertendus et de cette
même fréquence chez les sujets non exposés, il faudrait faire une enquête appropriée d'épidémiologie analytique et non un
essai thérapeutique (épidémiologie expérimentale). Les rapports existants dans cet exemple, entre le nombre d'hypertendus
dans les 2 groupes, ne répondent en rien à cette question (A,B).

On envisage le dépistage d'une maladie M grâce à un test biologique. La maladie M est d'autant plus probable que la valeur
obtenue au test est élevée.

On hésite entre deux seuils de normalité : A = 30 unités internationales, B = 20 unités internationales. On peut
prévoir avec certitude que :
A - Le test A sera plus sensible que le test B
B - Le test A sera plus spécifique que le test B
C - Le nombre de faux positifs sera plus grand avec le test A
D - Le nombre de faux négatifs sera plus grand avec le test A
E - Le test A dépistera plus de malades que le test B
Bonne(s) réponse(s) : B D

Test négatif Test positif


V.N. Vrais négatifs Faux positifs Sujets sains
F.N. Faux négatifs Vrais négatifs Sujets malades

L'augmentation de la norme fait que le nombre de faux positifs est plus faible avec le test A (C) et le nombre de faux négatifs
plus grand (D). Le nombre de malades et de non malades restent le même, la sensibilité, vrais positifs chez les malades, est
plus faible pour A (A), par contre la spécificité (vrais négatifs chez les sujets sains est plus grande pour A. Le test A dépiste
moins de malades (le nombre de vrais positifs est inférieur).

Si l'on choisit une valeur seuil telle que le test a une spécificité de 100%. Cela implique que :
A - Sa valeur prédictive positive est aussi de 100%
B - Devant un test positif, on peut affirmer que le sujet est malade
C - On prend le risque de laisser échapper beaucoup de malades au dépistage
D - La valeur prédictive négative est de 100
E - Il n'y a pas de faux négatifs
Bonne(s) réponse(s) : A B C

Une spécificité de 100% signale qu'il n'y a aucun faux-positif, donc que devant un test positif on peut affirmer la maladie (B) ou
ce qui revient au même que la valeur prédictive positive :
V.P./V.P. + F.P. = 1 = 100 % (A).
Par contre le nombre de faux négatifs augmente (E) donc on laisse échapper des malades au dépistages (C). La valeur
prédictive négative V.N/V.N + F.N diminue et n'est pas égale à 100 % (D).

On choisit finalement une valeur seuil telle que la sensibilité est de 95% et
la spécificité de 90 %. Votre première campagne de dépistage s'applique à une population ou laprévalence de
la maladie est de 2 %.
En une semaine, vous examinez 1000 personnes. Le nombre attendu :
A - De sujets positifs est de 117
B - De vrais positifs est de 19
C - De vrais négatifs est de 980
D - De faux négatifs est de 98
E - On ne peut prédire sur les seules données de l'énoncé le nombre de vrais positifs
Bonne(s) réponse(s) : A B

Voici le tableau de contingence reconstitué :


Malades Non malades Total
Test + 19 98 117
TEST- 1 882 883
Total 20 980 1000

Procédons pas à pas :


Prévalence de la maladie 2 %, sur 1000 sujets il y a 20 malades. La sensibitité est de 95 % donc 95 % de ces malades sont
des vrais positifs soit 19 (B) 1 seul faux négatif (D) sur les 980 (1 000 - 20) non malades. 90 % (spécificité) sont des vrais
négatifs soit 882 vrais négatifs (C) et 98 faux positifs. Le nombre de sujets positifs est égal à la somme des vrais positifs et
des faux positifs 19 + 98 = 117 (A).
1355
Exclusivement sur DOC - DZ : www.doc-dz.com NADJI 85
RESIDANAT EN POCHE TOME II
Cas Clinique en QCM

Votre deuxième campagne de dépistage s'applique à une population où la prévalence de la maladie est plus
élevée :
A - La valeur prédictive positive du test sera meilleure que dans le cas
précédent

B - La valeur prédictive négative du test sera meilleure que dans le cas précédent
C - La sensibilité du test sera meilleure que dans le cas précédent
D - La spécificité du test sera meilleure que dans le cas précédent
E - Le rendement du dépistage sera meilleur que dans le cas précédent
Bonne(s) réponse(s) : C D

C D. La sensibilité et la spécificité du test ne changent pas si la prévalence augmente le nombre de malades augmente ainsi
que le nombre de vrais positifs, par contre le nombre de non malades baisse ainsi que le nombre de vrais négatifs.

Chez des enfants considérés à haut risque de rechute de crise convulsive hyperpyrétique, un essai thérapeutique a été
entrepris pour évaluer l'efficacité d'un traitement intermittent par le nitrazépan lors des poussées fébriles.
Après avoir donné des explications détaillées aux parents sur la nature relativement bénigne des convulsions fébriles et sur
les traitements possibles, un traitement prophylactique intermittent par le nitrazépan a été suggéré. 31 parents ont accepté de
donner du nitrazépan, 24 ont préféré l'abstention thérapeutique.

Cliquez sur le bouton "Dessin" pour afficher le tableau.

Il s'agit d'un essai :

Cliquez sur le bouton "dessin" pour afficher le tableau.


A - Randomisé
B - Comparatif
C - Séquentiel
D - En double aveugle
E - Croisé (cross over)
Bonne(s) réponse(s) : B

B. Cet essai est comparatif : on compare la prise de Nitrazépan® et l'abstention thérapeutique.


Il n'y a pas de randomisation, l'attribution des traitements ne s'est pas fait par tirage au sort, mais par choix des parents.
D. Il n'y a pas de double aveugle. Médecin et malades connaissent le traitement.
C E. S'appliquent à des comparaisons successives dans le temps soit en conséquence simple, soit en conséquence croisée
chez les groupes comparés (cross over design).

La randomisation, d'un façon générale :

Cliquez sur le bouton "dessin" pour afficher le tableau.


A - Est contre-indiquée chez l'enfant
B - N'est pas applicable au traitement par une benzodiazépine
C - Peut favoriser la survenue de rechutes
D - Permet d'éviter un biais lors du recueil de données
E - Permet l'attribution aléatoire du traitement étudie
Bonne(s) réponse(s) : E

L'attribution aléatoire du traitement (E) permet d'éviter un biais de recrutement et non de recueil de données (D). Pour le tirage
au sort, âge du sujet et type de médicament (A,B) n'ont aucune importance. Seul compte le fait que les sujets puissent
indifféremment recevoir les 2 traitements à comparer. La randomisation n'a par ailleurs bien sûr aucun rapport avec l'évolution
de la maladie (C).

Dans cet essai thérapeutique, tel qu'il est conçu, p < 0.05 signifie que :

Cliquez sur le bouton "dessin" pour afficher le tableau.


A - Le nitrazépan est efficace dans la prévention des rechutes
B - Le nitrazépan n'est pas efficace dans la prévention des rechutes
C - Le risque de rechutes est significativement diminué dans le groupe traité
D - Les rechutes sont fréquentes dans le groupe non traité
E - Les rechutes sont rares dans le groupe traité
Bonne(s) réponse(s) : C

P < 0,05 signifie que la diminution du nombre de rechutes (46% chez les non traités, 19 % chez les traités) est significative
dans cette population.

1356
Exclusivement sur DOC - DZ : www.doc-dz.com NADJI 85
RESIDANAT EN POCHE TOME II
Cas Clinique en QCM

P < 0.05 peut avoir pour explication(s) ?

Cliquez sur le bouton "dessin" pour afficher le tableau.


A - L'existence d'un biais de sélection
B - L'intervention du nitrazépan, si elle est réelle
C - L'intervention du hasard
D - Le jeune âge
E - Une erreur dans le calcul du khi-deux
Bonne(s) réponse(s) : A B C

La question n'est pas bien formulée, mais on peut dire que cette différence significative peut avoir pour origine un biais de
sélection puisqu'il n'y a pas eu randomisation (A), elle peut être due au traitement, qui serait efficace, elle peut être due au
hasard (5 chances sur 100). Mais elle ne peut pas être due à l'âge ou à une erreur de calcul.

La validité d'un nouveau test de dépistage du cancer du côlon a été évalué dans un service de gastro-entérologie d'un CHU.
Ce test a été étudié chez 100 malades venus consulter pour suspicion de cancer. Parmi ces 100 consultants, 50 avaient un
cancer du côlon prouvé à la biopsie. Les résultats d'étude de validité sont décrits dans le tableau 1 :

Cliquez sur le bouton "Dessin" pour afficher le tableau 1.

Quelle est la sensibilité du test en C.H.U. ?

Cliquez sur le bouton "dessin" pour afficher le tableau.


A - 50/100
B - 36/50
C - 14/40
D - 10/50
E - 40/50
Bonne(s) réponse(s) : B

La sensibilité qui est la proportion de test + chez les malades atteints de cancer du colon est de 36/50 ou 72 %.

Quelle est la spécificité du test en C.H.U. ?

Cliquez sur le bouton "dessin" pour afficher le tableau.


A - 50/100
B - 36/50
C - 14/40
D - 10/50
E - 40/50
Bonne(s) réponse(s) : E

La spécificité est la proportion de test négatif chez les non malades soit 40/50 = 80%.

La valeur prédictive positive mesure :

Cliquez sur le bouton "dessin" pour afficher le tableau.


A - La proportion des tests positifs chez les sujets malades
B - La proportion de tests négatifs chez les sujets indemnes de maladie
C - La proportion de sujets malades parmi ceux soumis au tests
D - La proportion de malades parmi les sujets positifs au test
E - La proportion de non malades parmi les sujets positifs au test
Bonne(s) réponse(s) : D

A. Est la sensibilité.
B. Est la spécificité.
D. Valeur prédictive positive est égale à la proportion de malade parmi les sujets positifs au test.
E. N'est pas la valeur prédictive négative, elle le serait si c'était la proportion de non malades parmi les sujets négatifs au test.

1357
Exclusivement sur DOC - DZ : www.doc-dz.com NADJI 85
RESIDANAT EN POCHE TOME II
Cas Clinique en QCM

Si le même test avait été effectué en médecine générale, on observerait par rapport au CHU des variations de :

Cliquez sur le bouton "dessin" pour afficher le tableau.


A - la sensibilité du test
B - la spécificité du test
C - la valeur prédictive positive du test
D - la valeur prédictive négative du test
E - la prévalence de la maladie dans la clientèle d'un médecin généraliste
Bonne(s) réponse(s) : C D E

En médecine générale, la prévalence de la maladie est bien plus faible (E), ceci ne change en rien la sensibilité et la
spécificité du test (A,B) mais la valeur prédictive positive augmente (C) et la valeur prédictive négative diminue (D).

Mademoiselle Z..., d'origine française, 16 ans et demi, célibataire, travaille comme manutentionnaire dans une usine à 24
kilomètres de chez elle. Elle vient consulter car elle est enceinte de 9 semaines. Sa tension artérielle est à 160/90 mmHg. Le
reste de l'examen somatique est satisfaisant.

Si elle vous demande une interruption volontaire de grossesse (I.V.G.) quelle est votre attitude ?
A - Vous devez l'adresser au préalable à un obstétricien
B - Vous demandez l'autorisation d'un de ses parents
C - Vous devez l'adressez au service social de l'entreprise pour entretien
D - Vous lui faites confirmer sa demande par écrit
E - Vous refusez car les délais sont dépassés
Bonne(s) réponse(s) :B D

B. Est obligatoire chez cette mineure.


Le consentement de la mineure célibataire doit également être obtenu, il doit être recueilli hors de la présence des parents.

Si elle souhaite poursuivre sa grossesse, à quel organisme doit-elle en faire la déclaration ?


A - La mairie
B - La D.D.A.S.S.
C - La médecine du travail
D - La Caisse d'Allocation Familiale
E - La D.R.A.S.S.
Bonne(s) réponse(s) : D

Evident.

Combien de facteur(s) de risque d'accouchement prématuré présente-t-telle ?


A - Aucun
B - 1 facteur
C - 2 facteurs
D - De 2 à 4 facteurs
E - 5 facteurs
Bonne(s) réponse(s) : D

Il existe plusieurs facteurs de risques :


- moins de 18 ans
- célibataire
- travail difficile (manutentionnaire)
- HTA
- longs trajets quotidiens.

Mademoiselle Z... a déclaré sa grossesse. De quelle(s) prestation(s) pourra-t-elle bénéficier ?


A - Allocations au jeune enfant
B - Indemnités journalières au cours du congé pré et post natal
C - Allocation du fond national de solidarité
D - Hébergement en maison maternelle
E - Surveillance gratuite de sa grossesse en P.M.I.
Bonne(s) réponse(s) : A B D E

Pas de commentaire.

1358
Exclusivement sur DOC - DZ : www.doc-dz.com NADJI 85
RESIDANAT EN POCHE TOME II
Cas Clinique en QCM
Dans une petite ville de Grande Bretagne (50 000 habitants) existe depuis plusieurs années un registre des cancers qui
recense tous les cas de cancers diagnostiqués. On se propose d'étudier en 1982 le rôle du tabagisme en recherchant dans
deux groupes :
- un premier constitué par tous les malades atteints de cancer bronchique figurant dans le registre en 1981
- un second de même taille de sujets indemnes de cette affection
- la notion d'exposition au tabagisme
- l'âge de début
- l'ancienneté et l'importance de la consommation de tabac.

Cette enquête épidémiologique est une étude :


A - Prospective
B - Exposés, non exposés
C - Transversale
D - Cas-témoins
E - Expérimentale
Bonne(s) réponse(s) : D

Cette enquête épidémiologique est une étude cas témoin (D, et non B). C'est une étude rétrospective (non A) visant à étudier
l'existence de l'exposition au tabac dans un groupe de malades (cancereux) et de témoins.

Le groupe de sujets indemnes est constitué de façon à ce que pour chaque malade on choisisse un individu
sain ayant les mêmes caractéristiques d'âge, de sexe., socioprofessionnelles, d'ethnie. Cette méthode constitue
en épidémiologie :
A - Un ajustement
B - Un échantillonnage représentatif
C - Un appariement
D - Une standardisation
E - Une étude en double aveugle
Bonne(s) réponse(s) : C

C. Les groupes sont appariés, ce qui assure leur comparabilité.


A D. Ajustement et standardisation ne sont pas des procédures de choix de population témoin mais des techniques d'analyse.
B. Un échantillon représentatif est tiré au sort dans la population de référence.
E. Une étude en double aveugle rentre dans le cadre de l'épidémiologie expérimentale (essais thérapeutiques).

Durant l'année 1981, le registre des cancers a permis de recenser dans cette ville de 50 000 habitants un
cancer bronchique chez 50 personnes. 10 personnes sont décédées de cette maladie durant l'année. Le taux
de 20 pour 100 correspond pour cette ville en 1981 :
A - Au taux de mortalité proportionnelle par cancer bronchique
B - Au taux de mortalité spécifique par cancer bronchique
C - Au taux de prévalence du cancer bronchique
D - Au taux de létalité du cancer bronchique
E - Au taux brut de mortalité dans la population
Bonne(s) réponse(s) : D

Pour le cancer bronchique. Le taux de létalité est égal au nombre de personnes sur le nombre de malades 10/50 = 20 pour
100. Le taux brut de mortalité nous étant inconnu, il nous est impossible de calculer le taux de mortalité proportionnelle. Le
taux de mortalité spécifique est de 10/50 000. Le taux de prévalence est de 50/50 000 = 1 pour 1000.

Quel est le taux de prévalence de cancer bronchique dans cette ville en 1981 ?
A - 10 pour 10.000
B - 2 pour 10.000
C - 2000 pour 10.000
D - 50 pour 10.000
E - On ne peut calculer le taux de prévalence
Bonne(s) réponse(s) : A

Le taux de prévalence a déjà été calculé à la question précédente. C'est le rapport entre le nombre de cas de cancers
bronchiques recensés en 1981 et la population exposée.

1359
Exclusivement sur DOC - DZ : www.doc-dz.com NADJI 85
RESIDANAT EN POCHE TOME II
Cas Clinique en QCM
Cette cliente d'une grande ville, cardiaque, traitée par tonicardiaque et diurétique vient vous consulter pour une cystite
colibacillaire. Elle met en cause l'eau potable distribuée dans son immeuble, et qui depuis peu a un goût de moisi. Nous
sommes en juillet : il fait chaud et sec.

Elle veut faire examiner l'eau. Quel est l'organisme réglementairement chargé de cet examen ?
A - Le laboratoire de chimie analytique de la ville
B - Le laboratoire national de santé publique
C - Le laboratoire d'hygiène
D - Un des laboratoires d'analyses médicales de la ville
E - Le laboratoire régional de l'agence de bassin
Bonne(s) réponse(s) : C

C'est le bureau municipal d'hygiène qui surveille les eaux d'alimentation.

Que dites-vous à cette cliente ?


A - Il y a eu développement de germes anaérobies dans les canalisations expliquant le goût de moisi
B - Cette eau est dangereuse : ne pas la boire
C - Cette eau contient un excès de nitrates dû à la concentration des minéraux dans l'eau par temps chaud et
sec
D - Le goût de moisi est habituellement dû au développement de champignons microscopiques
E - Le goût de moisi est dû aux canalisations en plomb
Bonne(s) réponse(s) : D

Selon la législation, cependant, cette eau n'est pas potable. Une eau potable ne doit en effet présenter ni odeur, ni saveur
désagréable.

Le syndic de l'immeuble collectif où habite cette cliente vient de décider de mettre un adoucisseur pour l'eau de
la ville à l'entrée de l'immeuble. Cette mesure :
A - Permettra d'éviter les inconvénients d'une eau trop calcaire pour les machines à laver le linge et la
vaisselle
B - Est contraire à la réglementation sur la distribution de l'eau potable aux usagers des immeubles collectifs
C - Ne devrait être prise que pour l'eau chaude
D - Enlève tout le calcium de l'eau ce qui expose à une ostéomalacie
E - Diminue le risque de maladies cardiovasculaires
Bonne(s) réponse(s) : A

Une eau riche en sels calcaires et magnésiums est dite eau dure. Celle-ci présente des inconvénients pour l'usage ménager
(difficultés de savonnage, entartrage des canalisations,...). L'appauvrissement excessif en minéraux des eaux peut-être nocif ?
L'eau douce semble en particulier augmenter la fréquence des maladies cardiovasculaires.

L'analyse de l'eau donne les résultats suivants :


- Escherichia Coli : 1 dans 100 ml d' eau
- nitrates : 30 mg/litre.
Vous concluez que :
A - L'eau est potable
B - La consommation de cette eau peut être à l'origine d'une méthémoglobinurie
C - L'eau n'est pas potable
D - Les résultats sont insuffisants pour porter un avis sur la potabilité
E - Le goût de moisi est expliqué par les résultats de l'analyse
Bonne(s) réponse(s) : C

Pour juger de la potabilité d'une eau, on effectue une analyse quantitative des :
- escherichia coli
- streptocoques fécaux
- clostridium sulfino réducteurs.
La présence d'E. Coli signe obligatoirement une contamination fécale récente, c'est un témoin très sensible et très spécifique.
B. Les nitrates présentent un danger pour un taux supérieur à 60 ml/l.

1360
Exclusivement sur DOC - DZ : www.doc-dz.com NADJI 85
RESIDANAT EN POCHE TOME II
Cas Clinique en QCM
Dans un quartier défavorise. en 1980. les décès foeto-infantiles se décomposaient de la façon suivante pour 1000 naissances
(enfants vivants ou non) et 20.000 habitants.
- morts nés 20
- décès de O a 7 jours 12
- décès de 7 a 28 jours 4
- décès de 28 jours a 1 an 8

Le taux de mortalité infantile est de :


A - 44/20.000
B - 44/1000
C - 24/20.000
D - 24/1000
E - 24/980
Bonne(s) réponse(s) : E

Le taux de mortalité infantile est le nombre de décès survenus pendant la première année de vie (12 + 4 + 8) rapporté à 1000
naissances vivantes (1000 - 20), soit 24/980.
C. Est le taux de mortalité au cours de la première année.
B. Est le taux de mortalité foeto-infantile : ensemble de mortalité foetale tardive (morts né)et infantile calculée sur 1000
naissances totales.
20 + 24/980 + 20 = 44/1 000.

Le taux de mortalité périnatale est de :


A - 32/1000
B - 36/1000
C - 1 6/1000
D - 12/1000
E - 3 2/980
Bonne(s) réponse(s) : A

Le taux de mortalité périnatale est le nombre de morts nés plus le nombre de décès au cours des 7 premiers jours de vie
rapporté à 1000 naissances totales soit 20 + 12 = 32 pour 1000
E - Est faux car cette fois, on rapporte au nombre de naissances totales

Pour comparer à la moyenne nationale les causes exogènes de mortalité foeto-infantile dans ce quartier, quelle
est la donnée la plus fiable ?
A - Le nombre de décès de 0 à 7 jours
B - Le taux de mortalité périnatale
C - Le nombre de décès de 0 à 28 jours
D - Le taux de mortalité néonatale précoce
E - Le taux de mortalité post néonatale
Bonne(s) réponse(s) :E

Les causes exogènes (infections, accidents) de mortalité infantile sont logiquement croissantes en fonction de l'âge et
augmentent au fur et à mesure que les causes endogènes diminuent en proportion (facteurs héréditaires, malformations). Les
données les plus fiables correspondent aux âges les plus tardifs, c'est-à-dire la mortalité post néo-natale qui mesure le
nombre de décès du 29ème jour au 365ème jour de vie rapporté à 1000 naissances vivantes. Cette période est la plus tardive
enregistrée parmi ces taux :
B. Est le nombre de morts-nés + nombre de décès au cours des 7 premiers jours rapporté à 1000 naissances totales.
D. Est le nombre de décès au cours des 7 premiers jours rapporte à 1000 naissances vivantes.

Parmi les propositions ci-dessous laquelle(lesquelles) caractérise(nt) la mortinatalité ?


A - C'est le nombre de mort-nés à partir de la 28ème semaine de gestation
B - C'est le nombre de décès de la 28ème semaine de gestation au 7ème jour après la naissance
C - C'est le nombre de décès de 0 à 6 jours
D - C'est un indicateur sensible aux actions de lutte contre les maladies infectieuses
E - C'est un bon indicateur des causes endogènes de mortalité foeto infantile
Bonne(s) réponse(s) : A E

La mortinatalilé ou mortalité foetale tardive ou mortalité intrautérine est le nombre de décès en cours de grossesse après 28
semaines d'aménorrhée (morts-nés) rapporté à 1000 naissances totales.
B. Caractérise la mortalité périnatale.
C. La mortalité néo-natale précoce.
Le taux reflète essentiellemenl l'incompatibilité biologique de base au début et la santé mère enfant vers la fin de la
grossesse. Il n'est pas sensible aux causes exogènes telles que les infections mais donne une indication sur les causes
endogènes (le meilleur indicateur étant le taux de mortalité néo-natale).

1361
Exclusivement sur DOC - DZ : www.doc-dz.com NADJI 85
RESIDANAT EN POCHE TOME II
Cas Clinique en QCM
Une enquête cas-témoins a donné les résultats suivants (tableau de contingence ci-dessous) :

Cliquez sur le bouton "Dessin" pour afficher le tableau.

Dans cette enquête le taux de prévalence de l'infarctus du myocarde est :

Cliquez sur le bouton "dessin" pour afficher le tableau.


A - Egal à 50
B - Compris entre 40 et 50 %
C - Compris entre 25 et 40 %
D - Compris entre 50 et 60 %
E - Compris entre 60 et 75 %
Bonne(s) réponse(s) : A

La prévalence dans une étude cas-témoin est impossible à calculer, elle ne reflète pas la prévalence de la maladie et ne
reflète que l'importance relative attribué par le protocole de l'enquête aux groupes de malades et de témoins. Ici nombre de
sujets malades :100 (71 + 29), nombre de sujets totaux 200.

p < 0,01 signifie que :

Cliquez sur le bouton "dessin" pour afficher le tableau.


A - La fréquence de l'éthylisme est significativement plus élevée dans le groupe de malades atteints d'un
infarctus du myocarde (p < 0.01)
B - Il existe une association entre l'infarctus du myocarde et l'éthylisme chronique (degré de signification p <
0.01 )
C - Il existe un biais de mesure (degré de signification p < 0.01)
D -Il existe un biais de sélection (degré de signification p < 0.01)
E - Il existe un facteur de confusion (degré de signification p < 0.01)
Bonne(s) réponse(s) : A B

L'association entre infarctus et éthylisme, qui est ici significative, indique une fréquence plus grande de l'éthylisme chez les
maladies (71 %) que chez les non-malades (52 %). Si l'échantillon était représentatif, c'est-à-dire que la prévalence est
exacte, on pourrait dire aussi que la fréquence de l'infarctus (71/123 = 58 %) est plus importante chez les éthyliques que chez
les sujets non exposés (29/77 = 38 %). Si C,D,E sont possibles, ils ne sont pas contenus dans le constat P<0.01.

A propos des résultats de cette enquête cas-témoins, vous êtes en droit de vous poser la (les) question(s)
suivante(s) :

Cliquez sur le bouton "dessin" pour afficher le tableau.


A - Existe-t-il un biais de sélection ?
B - Existe-t-il un biais de mesure ?
C - Existe-t-il un facteur de confusion ?
D - Y a-t-il des perdus de vue ?
E - Les groupes ont-ils été constitués par tirage au sort ?
Bonne(s) réponse(s) : A B C

A. Il est légitime devant toute enquête cas -émoin de s'interroger sur un biais de sélection (le taux de 52 % d'éthyliques chez
les témoins rend probable cette hypothèse).
B. Le biais de mesure signifierait que l'exposition éthylique n'a pas été enregistré de la même façon chez les malades et chez
les témoins.
C. La présence d'un facteur de confusion lié à la maladie et au facteur d'exposition étudié est à rechercher systématiquement.
D. Est valable pour les enquêtes de cohorte.
E - Pour les enquêtes épidémiologiques expérimentales.

Dans cette enquête cas-témoins, le tabagisme s'est révélé être associé à la fois à l'éthylisme chronique et à
l'infarctus du myocarde. Cela signifie que le tabagisme peut intervenir comme :
Cliquez sur le bouton "dessin" pour afficher le tableau.
A - Facteur causal de l'infarctus du myocarde
B - Facteur causal de l'éthylisme chronique
C - Biais de sélection
D - Biais de mesure
E - Facteur de confusion
Bonne(s) réponse(s) : E

Le tabagisme interviendrait comme facteur de confusion qui peut aller jusqu'à expliquer toute association observée entre
éthylisme et infarctus. Un facteur de confusion se caractérise par son association à la fois à la maladie étudiée et au facteur
d'exposition incriminé.

1362
Exclusivement sur DOC - DZ : www.doc-dz.com NADJI 85
RESIDANAT EN POCHE TOME II
Cas Clinique en QCM
Le graphique ci-dessous représente de façon simplifiée les 60 cas de tuberculose survenus dans dans un camp de 1000
réfugiés du sud-est asiatique en France.

Cliquez sur le bouton "Dessin" pour afficher le tableau.

Le taux de prévalence pour le mois de janvier est de :

Cliquez sur le bouton "dessin" pour afficher le tableau.


A - 20/60
B - 30/60
C - 20/1000
D - 30/1000
E - 60/1000
Bonne(s) réponse(s) : E

La prévalence est égale au nombre total de cas existant pendant la période considérée rapportée à l'effectif de la populalion
exposée, soit 60 pour 1 000.

Le taux d'incidence durant le mois de janvier est de :

Cliquez sur le bouton "dessin" pour afficher le tableau.


A - 20/60
B - 30/60
C - 20/1000
D - 30/1000
E - 60/1000
Bonne(s) réponse(s) : D

L'incidence est le nombre de nouveaux cas apparus pendant le mois de janvier rapporté à la population concernée. Il y a eu
en janvier 30 nouveaux cas.

Le taux de mortalité spécifique durant le mois de janvier est de :

Cliquez sur le bouton "dessin" pour afficher le tableau.


A - 20/60
B - 30/60
C - 20/1000
D - 30/1000
E - 60/1000
Bonne(s) réponse(s) : C

Le taux de mortalité spécifique est le taux de mortalité pour une cause (ici la tuberculose) ; c'est le nombre de décès en janvier
sur l'effectif total de la population. Soit taux de mortalité spécifique : 20 décès pour 1000.

Selon les règles utilisées en France, quelle(s) est(sont) la(les) mesure(s) nécessaire(s) vis à vis de malades
tuberculeux bacillifères dépistes ?
Cliquez sur le bouton "dessin" pour afficher le tableau.
A - Isolement
B - Séroprévention
C - Traitement par 3 antibiotiques antituberculeux pendant au mois 9 mois
D - Traitement par 2 antibiotiques antituberculeux pendant 6 mois
E - Déclaration obligatoire
Bonne(s) réponse(s) : A C E

C E. S'adressent à tout malade atteint de tuberculose.

1363
Exclusivement sur DOC - DZ : www.doc-dz.com NADJI 85
RESIDANAT EN POCHE TOME II
Cas Clinique en QCM

Parmi les mesures ci-dessous laquelle(lesquelles) est(sont) justifiée(s) pour les enfants du camp dont l'examen
radiologique et clinique est normal et dont l'intra dermoréaction à 10 unités détermine une induration
inflammatoire de 15 mm, incapables de préciser leurs vaccinations antérieures ?
Cliquez sur le bouton "dessin" pour afficher le tableau.
A - Isolement
B - Vaccination BCG immédiate
C - 3 antibiotiques antituberculeux pendant au moins 9 mois
D - 2 antibiotiques antituberculeux pendant 6 mois
E - Séroprévention
Bonne(s) réponse(s) : D

Pas de commentaire.

Le schéma ci-dessous réprésente l'évolution des cas de méningite cérébrospinale à méningocoque B dans un département
français métropolitain, qui comporte 100 000 habitants.

Cliquez sur le bouton "Dessin" pour afficher le tableau.

Le taux de mortalité spécifique de la méningite cérébrospinale dans ce département pour 1981 est de :

Cliquez sur le bouton "dessin" pour afficher le tableau.


A - 10/100.000
B - 8/100.000
C - 8/10
D - 2/10
E - 2/100.000
Bonne(s) réponse(s) : E

Le taux de mortalité spécifique de la méningite cérébrospinale est un taux de mortalité pour une cause particulière. Il y a eu 2
décès en 1981 pour cette cause de mortalité. Le taux recherché est 2/100 000.

Le taux de létalité de la méningite cérébrospinale pour 1981 est de :

Cliquez sur le bouton "dessin" pour afficher le tableau.


A - 10/100.000
B - 8/100.000
C - 8/10
D - 2/10
E - 2/100.000
Bonne(s) réponse(s) : D

Le taux de létalité est le nombre de décès dû à la maladie rapporté au nombre de malades soit pour 1981, 2 décès pour 10
malades.

Devant la recrudescence des cas déclarés, le maire vous demande les mesures a prendre dans le cas ou une
méningite surviendrait dans le groupe scolaire de la commune Laquelle (ou lesquelles) des affirmations ci-
dessous est(sont) exacte(s) ?
Cliquez sur le bouton "dessin" pour afficher le tableau.
A - Fermeture immédiate de l'école
B - Eviction scolaire des enfants de la même classe
C - Vaccination de tous les enfants
D - Chimioprophylaxie des proches
E - Chimioprophylaxie chez tous les habitants du village
Bonne(s) réponse(s) : D

Mesures à prendre en cas de méningite :


A. Pas de fermeture des écoles.
B. Pas d'éviction des autres enfants. Eviction du malade jusqu'à sa guérison.
D. La vaccination n'est pas utilisée en pratique.
D. C'est la seule de ces mesures qui est utilisée bien qu'elle serve essentiellement à rassurer l'entourage.
E. La chimioprophylaxie ne diminue pas le nombre de porteurs dans l'entourage, on ne l'empêche pas au delà de l'entourage
immediat.

1364
Exclusivement sur DOC - DZ : www.doc-dz.com NADJI 85
RESIDANAT EN POCHE TOME II
Cas Clinique en QCM

La déclaration d'un cas éventuel :

Cliquez sur le bouton "dessin" pour afficher le tableau.


A - Serait obligatoire
B - Serait adressée au médecin conseil de la sécurité sociale
C - Serait adressée à la direction départementale des affaires sanitaires et sociales
D - Entraînera systématiquement une analyse et une chloration de l'eau d'alimentation de la commune
E - Rend nécessaire une décontamination des locaux scolaires
Bonne(s) réponse(s) : C

A. La méningite fait toujours partie (décret du 10 juin 1986) des maladies à déclaration obligatoire.
B C. La déclaration se fait à la D.D.A.S.S..
D,E - La présence d'une méningite ne nécessite pas de désinfection, ni de locaux, ni de l'eau, ni d'alimentation (il n'y a pas de
survie du germe en dehors de l'hôte : l'homme).

Monsieur X... vient consulter pour altération de l'état général. Vous retenez le diagnostic d'alcoolisation chronique.

Parmi les signes que révèle l'interrogatoire ou l'examen clinique, quel(s) est(sont) celui ou ceux qui est(sont)
attribuable(s) à cette alcoolisation chronique ?
A - Erreur de saisie
B - Erreur de saisie
C - Erreur de saisie
D - Erreur de saisie
E - Erreur de saisie
Bonne(s) réponse(s) : A B D

Discutable.

Le bilan biologique vous révèle une hypertriglycéridémie. Quelle mesure prenez-vous ?


A - Prescription d'un régime pauvre en graisses
B - Prescription d'un régime pauvre en hydrates de carbone
C - Prescription d'un hypolipémiant
D - Contrôle du bilan lipidique à distance du sevrage
E - Contrôle du bilan thyroïdien
Bonne(s) réponse(s) : D

Seule mesure immédiatement logique afin de faire la part de l'alcoolisation dans la perturbation lipidique.

Dès le premier examen, la tension artérielle (TA) est un peu élevée. Quelle mesure prenez-vous ?
A - Prescription d'un antihypertenseur
B - Prescription d'un régime désodé
C - Attendre 6 mois après le sevrage
D - Contrôle de la TA 15 jours après le sevrage
E - Prescription d'un diurétique
Bonne(s) réponse(s) : D

Evident.

Si vous utilisez la grille de Le GO, qu'espérez-vous apprécier ?


A - L'état de dépendance vis-à-vis de l'alcool
B - Le retentissement clinique de l'alcoolisation chronique
C - La consommation quotidienne en boissons alcoolisées
D - Typer le type d'alcoolisation
E - Le retentissement fonctionnel d'une éventuelle polynévrite
Bonne(s) réponse(s) :

Connaissance.

1365
Exclusivement sur DOC - DZ : www.doc-dz.com NADJI 85
RESIDANAT EN POCHE TOME II
Cas Clinique en QCM
En 1983, la consommation médicale finale atteignait en France 318 milliards de francs contre 205 milliards de francs en 1980.

Lequel ou lesquels des éléments suivants sont inclus dans ces sommes ?
A - Dépenses de soins médicaux restant à la charge des ménages
B - Ensemble des dépenses des caisses d'assurance maladie
C - Ensemble des dépenses d'hospitalisation
D - Ensemble des dépenses de prévention (PMI, médecine du travail)
E - Ensemble des dépenses de recherche médicale
Bonne(s) réponse(s) : A B C

La consommation médicale finale est un sous-ensemble de la consommation finale des ménages. Elle regroupe les soins
d'hospitalisation, soins ambulatoires et assimilés, et les biens médicaux. La recherche médicale et les dépenses de prévention
font partie des dépenses de santé mais pas de la C.M.F..

Les dépenses d'hospitalisation se situaient, en 1983 dans la tranche (en milliards de F) :


A - 95 à 110
B - 110 à 125
C - 125 à 140
D - 140 à 155
E - 155 à 200
Bonne(s) réponse(s) : E

En 1984, l'hospitalisation représentait 49,4% des dépenses soit 194,8 milliards de F. Les dépenses de santé en F par
Français :
1980 3845
1981 4526
1982 5300
1983 5847
1984 6462
1985 7130

L'augmentation entre 1980 et 1983 des dépenses d'hospitalisation :


A - A été principalement supportée par les caisses d'assurance maladie
B - A été principalement supportée par les ménages
C - A été principalement supportée par l'état
D - Est plus importante que l'augmentation de la consommation finale des ménages
E - Est plus importante que l'accroissement du produit intérieur brut
Bonne(s) réponse(s) : A D E

A. La part de la Sécurité Sociale était de 72% en 1983, cette part est croissante.
La part de la Sécurite Sociale dans le financement des soins hospitaliers est encore plus élevé : 88 %.
D. La C.M.F. est le poste de consommation des ménages dont la croissance est la plus forte.
E. La C.M.F. est passée de 3 % du P.I.B. en 1950 à 5,7 % en 1970, 8,2 % en 1984.

Entre 1970 et 1983, la croissance de la consommation médicale finale a été de 16 % par an. Lequel(lesquels)
des différents postes a(ont) eu un accroissement inférieur à cette moyenne ?
A - Dépenses d'hospitalisation
B - Dépenses consacrées aux visites et consultations de médecins généralistes
C - Dépenses consacrées aux examens biologiques et radiologiques
D - Dépenses consacrées aux soins dentaires
E - Aucun des quatre énumérés ci-dessus
Bonne(s) réponse(s) : E

Seul les biens médicaux ont eu un accroissement inférieur à cette moyenne.

1366
Exclusivement sur DOC - DZ : www.doc-dz.com NADJI 85
RESIDANAT EN POCHE TOME II
Cas Clinique en QCM
Le meilleur test diagnostique d'anémie est le dosage de l'hémoglobine. En prenant comme borne de normalité la valeur de 12
g/dl, on retrouve sur une population de 100 personnes 18 anémiques bien classés et 12 sujets normaux considérés comme
anémiques à tort (faux positifs). Par des méthodes plus complexes que le dosage de l'Hb. on a identifié avec certitude 20 cas
d'anémie parmi ces 100 personnes. Pour tenter d'améliorer le score, on déplace la borne à 10 g/dl. La sensibilité passe à 80%
et 4 sujets normaux sont toujours mal classés.

Quelle est la sensibilité du premier test (Hb = 12 g) ?


A - 90 %
B - 85 %
C - 80 %
D - 75 %
E - 70 %
Bonne(s) réponse(s) : A

Ces données de l'énoncé nous permettent de reconstruire le tableau du 1er test.


Malades Non malades Total
Test + 18 12 30
Test- 2 68 70
Total 20 80 100

La sensibilité est la proportion de vrais positifs chez les malades soit 18/20 = 90 %.

Quelle est la spécificité du premier test (Hb = 12 g) ?


A - 90 %
B - 85 %
C - 80 %
D - 75 %
E - 70 %
Bonne(s) réponse(s) : B

La spécificité est la proportion de vrais négatifs chez les non malades. Sur 100 personnes, il y a 20 anémiques et 80 non
malades. Parmi eux, 12 sont faux positifs, 68 sont donc vrais négatifs Spécificité : 68/80 = 85 %.

Quelle est la spécificité du 2ème test (Hb = 10 g) ?


A - 75 %
B - 80 %
C - 85 %
D - 90 %
E - 95 %
Bonne(s) réponse(s) : E

Avec cette nouvelle borne, la répartition des sujets devient :

Malades Non Malades Total


16 4 20
4 76 80
20 80 100

Il est logique que la sensibilité du test ayant diminué, la spécificité augmente. 4 sujets non anémiques le sont déclarés à tort,
les 76 autres sont bien classés.
Spécificité : 76/80 = 95 %.

Combien de sujets anémiques ont-ils échappé au diagnostic avec la borne à 10 g d'hémoglobine ?


A-1
B-3
C-5
D-6
E - Aucun des chiffres ci-dessus
Bonne(s) réponse(s) : E

Avec cette borne à 10 g, sur les 20 sujets anémiques, 80 % sont bien classés (sensibilité) soit 16 et 4 sont mal classés.

1367
Exclusivement sur DOC - DZ : www.doc-dz.com NADJI 85
RESIDANAT EN POCHE TOME II
Cas Clinique en QCM
En septembre 1981, un médecin d'une commune de 15 000 habitants, signale à la D.D.A.S.S. plusieurs cas d'hépatite virale
A. Un contrôle de l'eau d'adduction publique est aussitôt pratiqué et révèle l'existence d'une pollution bactériologique (70 E.
coli dans 100 ml d'eau). Une enquête épidémiologique (enquête de cohorte rétrospective) est alors réalisée dans un quartier
bien différencié de cette commune, afin de mesurer la fréquence de l'hépatite virale :
- chez les habitants qui ont consommé en septembre l'eau du robinet
- chez les habitants qui n'en ont pas consommé durant cette période (puits personnel consommation d'eau minérale...).

Cliquez sur le bouton "Dessin" pour afficher le tableau.

Quel est le risque relatif entre les buveurs d'eau du robinet et les non-buveurs de cette eau ?

Cliquez sur le bouton "dessin" pour afficher le tableau.


A - Environ 2
B - Environ 4
C - Environ 6
D - Environ 8
E - Environ 10
Bonne(s) réponse(s) : A

Le risque relatif est le rapport entre la fréquence des malades chez les buveurs et les non buveurs d'eau.
R.R. = (40/530) / (3/80) = (3200 / 1590) = environ 2

Le résultat du risque relatif signifie que :

Cliquez sur le bouton "dessin" pour afficher le tableau.


A - L'effectif de la population est trop faible pour conclure
B - La fréquence de l'hépatite A est à peu près équivalente dans le groupe exposé et le groupe non expose
C - Le risque de contracter une hépatite A était augmenté dans le groupe exposé, par rapport au groupe non
exposé
D - Il existe une relation causale entre la consommation d'eau du robinet et l'hépatite A
E - Il existe un facteur de confusion
Bonne(s) réponse(s) : C

L'augmentation du risque dans le groupe exposé reflète une association entre l'hépatite et le fait de boire de l'eau du robinet. Il
n'est pas possible d'en extrapoler une relation causale.

Les enquêtes de cohorte font partie :

Cliquez sur le bouton "dessin" pour afficher le tableau.


A - De l'épidémiologie descriptive
B - De l'épidémiologie analytique
C - De l'épidémiologie expérimentale
D - Des enquêtes cas-témoins
E - Des enquêtes de prévalence
Bonne(s) réponse(s) : B

Les enquêtes de cohorte comme celle-ci font partie de l'épidémiologie analytique. Elles s'attachent à étudier le rôle de facteurs
de risque dans le déterminisme d'un problème de santé.

A la suite des conclusions de l'enquête, la mairie de cette commune a pris une série de mesures. Parmi les
mesures suivantes, laquelle (lesquelles) vous parai(ssen)t cohérente(s) ?
Cliquez sur le bouton "dessin" pour afficher le tableau.
A - Protection des captages (élimination d'une décharge sauvage d'ordures ménagères)
B - Stérilisation de l'eau d'adduction publique par addition d'une quantité suffisante de chlore
C - Vaccination contre l'hépatite A des sujets apparemment sains
D - Contrôle des normes d'étanchéité d'une station d'épuration située en amont du champ captant
E - Injection de gammaglobulines spécifiques chez les sujets apparemment sains
Bonne(s) réponse(s) : A B D

Pas de commentaires.

1368
Exclusivement sur DOC - DZ : www.doc-dz.com NADJI 85
RESIDANAT EN POCHE TOME II
Cas Clinique en QCM

Parmi les maladies suivantes. Iaquelle(lesquelles) peut(peuvent) être transmise(s) par une eau polluée (eau
d'adduction publique, ou eau de puits) ?
Cliquez sur le bouton "dessin" pour afficher le tableau.
A - Shigellose
B - Brucellose
C - Saturnisme
D - Salmonellose
E - Hépatite à virus B
Bonne(s) réponse(s) : A C D

L'intoxication peut provenir de l'eau captée elle-même ou des tuyaux de plomb (corrosion des canalisations).

Le laboratoire F... vous confie un essai thérapeutique.


Il s'agit de déterminer l'efficacité d'un nouvel anti agrégant plaquettaire, l'.i.Agregal;®. Le laboratoire affirme que les essais
faits à l'étranger montrent qu'associé avec les traitements hypotenseurs classiques, il est très actif en prévention des
conséquences morbides de l'hypertension artérielle. L'essai aura une durée de 7 ans au moins et portera sur une population
homogène d'hypertendus sous bêta-bloquants.

Le protocole en double aveugle que vous proposez devra comporter :


A - Deux groupes d'hypertendus, chaque groupe étant constitue des malades sous bêta bloquants d'un des
deux CHR participant à l'essai
B - Deux groupes d'hypertendus établis par randomisation
C - Un groupe traité par l'.i.Agregal;® seul, l'autre restant sous bêta bloquants
D - Un groupe recevant l'.i.Agrégal;® + bêta-bloquants. l'autre recevant un médicament de référence
(.i.Aspégic;® par exemple) + bêta bloquants
E - Vous devrez ignorer lequel des groupes reçoit le nouveau médicament
Bonne(s) réponse(s) : B D E

L'essai thérapeutique suivant nécessite :


- randomisation : c'est-à-dire répartition aléaloire des sujets dans les 2 groupes (B) et non A.
- double aveugle, on ignore le groupe qui reçoit le nouveau traitement (E).
Les traitements par bêta-bloquants ne doivent pas être arrêtés, mais associés dans un groupe à l'Agrégal®, dans l'autre à un
placebo ou un traitement de référence comme l'Aspégic® (D), et non C.

Dans ce cas l'utilisation d'une méthode en double aveugle


A - Ne se justifie pas puisque les malades sont repartis en deux groupes
B - Implique nécessairement l'utilisation d'un placebo
C - A pour but de supprimer les biais liés à la subjectivité du malade
D - A pour but de supprimer les biais liés à la subjectivité de l'expérimentateur
E - A pour but d'égaliser les caractéristiques des sujets des deux groupes
Bonne(s) réponse(s) : C D

Le double aveugle a cette double fonction de supprimer les biais liés à la subjectivité du malade et de son entourage (C) mais
aussi de l'expérimentateur.
E. Est assuré par la randomisation.

Dans ce cas, la randomisation :


A - Ne se justifie pas puisque les malades sont répartis en deux groupes
B - Est impossible parce que l'expérimentateur ignore quel groupe reçoit l'.i.Agrégal;®
C - A pour but de supprimer les biais liés à la subjectivité de l'expérimentateur
D - A pour but de supprimer les biais liés à la subjectivité du malade
E - A pour but d'égaliser les caractéristiques des sujets des deux groupes
Bonne(s) réponse(s) : E

A. Est une simple caractéristique d'un essai comparatif.


B C D. Sont des items en rapport avec le double aveugle.
E. La randomisation vise à rendre les groupes aussi comparables que possible.

1369
Exclusivement sur DOC - DZ : www.doc-dz.com NADJI 85
RESIDANAT EN POCHE TOME II
Cas Clinique en QCM

Quel indicateur vous paraît-il le plus pertinent à la fin de l'essai pour vérifier l'efficacité éventuelle du produit ?
A - La prévalence de l'hypertension artérielle
B - Les effets secondaires dans les deux groupes
C - Le risque relatif d'accident vasculaire cérébral dans les deux groupes
D - Le taux proportionnel de mortalité par cardiopathies ischémiques dans les deux groupes
E - La létalité des accidents vasculaires cérébraux dans les deux groupes
Bonne(s) réponse(s) : C

L'effet annoncé du traitement est la prévention des conséquences morbides de l'HTA. C'est la fréquence relative des
accidents vasculaires cérébraux qui est donc la plus appropriée pour répondre à cette question, et en particulier le risque
relatif qui en est le rapport.

Quel est le biais le plus à craindre dans cet essai tel qu'il est proposé ?
A - L'hétérogénéité des deux groupes
B - Un biais de mesure
C - Le nombre des perdus de vue
D - Un biais de mémorisation
E - Une variation temporelle de l'incidence de l'HTA
Bonne(s) réponse(s) : C

Dans une enquête de cette durée, où les patients sont suivis sur 7 ans, le principale source de biais est le nombre de perdus
de vue qui peut avoir un lien avec le traitement et l'effet mesuré (mort par AVC par exemple).

Avant de lancer un programme de prévention vis-à-vis d'une maladie, on est généralement amené à réaliser une enquête
épidémiologique destinée à mieux connaître les conditions particulières de développement de cette maladie da la population-
cible :
- fréquence d'apparition des cas
- proportion de sujets contagieux
- modalités de transmission
- groupes à risque.
C'est dans ces conditions qu'une enquête a été menée sur la tuberculose dans une région africaine. Un échantillon de 10.500
personnes indemnes de tuberculose a été identifié au début de l'enquête. Ces personnes ont été réexaminées chaque année
pendant trois ans. 500 d'entre elles vivaient avec un tuberculeux contagieux à leur domicile. Le tableau suivant montre le
nombre de tuberculoses actives identifiées dans l'échantillon pendant les 3 années :

Présence d'un tuberculeux OUI 500 12


contagieux au domicile NON 10 000 12

Quel est le taux annuel d'incidence de la tuberculose chez les sujets avec contact familial ?

Cliquez sur le bouton "dessin" pour afficher le tableau.


A - 0,004
B - 0,008
C - 0,024
D - 0,072
E - On ne peut pas répondre
Bonne(s) réponse(s) : B

Sans commentaire.

Quel est le taux annuel d'incidence chez les sujets sans contact familial ?

Cliquez sur le bouton "dessin" pour afficher le tableau.


A - 0,004
B - 0,008
C - 0,024
D - 0,072
E - On ne peut pas répondre
Bonne(s) réponse(s) : A

Sans commentaire.

1370
Exclusivement sur DOC - DZ : www.doc-dz.com NADJI 85
RESIDANAT EN POCHE TOME II
Cas Clinique en QCM

On estime que la maladie est active pendant en moyenne 10 ans. Quel est le nombre moyen de ces actifs que
l'on peut espérer identifier à tout moment parmi les 500 personnes vivant dans l'entourage d'un contagieux ?

Cliquez sur le bouton "dessin" pour afficher le tableau.


A - 10
B - 20
C - 30
D - 40
E - On ne peut pas répondre
Bonne(s) réponse(s) : D

Sans commentaire.

Quel est le risque relatif de contracter une tuberculose, si l'on a un tuberculeux vivant à son domicile ?

Cliquez sur le bouton "dessin" pour afficher le tableau.


A - 0,1
B - 0,5
C - 10
D - 20
E - On ne peut pas répondre
Bonne(s) réponse(s) : D

Sans commentaire.

Une jeune femme portugaise de 17 ans résidant légalement depuis cinq mois et demi en France, dont le mari est salarié, vous
consulte en tant que médecin pour une aménorrhée de neuf semaines et demi.
Elle est sans travail et a interrompu sa scolarité depuis un an.
Elle présente une hypertension artérielle (18/10) traitée par bêtabloquants.
Elle vous dit que si elle est enceinte, elle désire une IVG.

Que lui dites-vous, et qu'elle est la conduite à tenir ?


A - Si vous ne souhaitez pas pratiquer l'IVG vous devez l'en avertir immédiatement
B - Vous assurer de la réalité de la grossesse
C - Vous prenez contact avec le représentant de son pays
D - Vous devez la prévenir des risques pour de l'IVG sa santé et les maternités futures
E - Si vous pensez pratiquer une IVG vous demandez l'accord du mari
Bonne(s) réponse(s) : A B D

Sans commentaire.

Le test de grossesse est positif, vous n'acceptez pas de pratiquer l'IVG, que devez-vous faire ?
A - Lui remettre un dossier guide des formalités et des mesures sociales
B - Recueillir le consentement des parents puisqu'elle est mineure
C - Lors de la deuxième consultation. obtenir la confirmation de la demande de la patiente par écrit
D - Obtenir une attestation prouvant qu'elle réside en France depuis plus de trois mois
E - Vous n'avez aucune obligation
Bonne(s) réponse(s) : A C D

Sans commentaire.

Que doit faire la patiente ?


A - Se présenter dans les plus brefs délais à une consultation d'un établissement public ou privé pratiquant
les IVG
B - Se rendre dans un centre de consultation médico-social agréé
C - Respecter un délai de huit jours entre cette consultation et l'IVG
D - Ce délai peut être ramené à deux jours, si le terme des dix semaines risque d'être dépassé
E - Obtenir l'IVG dans un but thérapeutique puisqu'elle est hypertendue sans plus amples formalités
Bonne(s) réponse(s) : B

Sans commentaire.

1371
Exclusivement sur DOC - DZ : www.doc-dz.com NADJI 85
RESIDANAT EN POCHE TOME II
Cas Clinique en QCM

Seul l'établissement hospitalier public pratique les IVG dans le département :


A - Le chef de service concerné de l'hôpital peut refuser de pratiquer les IVG pour cause de conscience
B - Le chef de service ne peut pas refuser de pratiquer l'IVG
C - Le nombre d'IVG ne peut être supérieur au quart des actes chirurgicaux obstétricaux du service
D - Les IVG seront déclarées nominativement à la DDASS
E - En cas de désaccord des parents, il faudra obtenir l'autorisation du Juge des Enfants
Bonne(s) réponse(s) :

QUESTION ANNULEE

Une fois l'IVG réalisée :


A - Les frais seront pris en charge par la Sécurité Sociale
B - Il faudra donner une information sur la contraception
C - Il faudra prescrire un contraceptif
D - Il faudra conserver un an le consentement écrit de la jeune femme
E - Il faudra conserver cinq ans le consentement écrit de la jeune femme
Bonne(s) réponse(s) : A B D

Sans commentaire.

En France. en 1980, les décès foeto-infantiles se décomposaient de la façon suivante,.pour 1000 naissances et 50 millions
d'habitants :
- morts nés 10
- décès de 0 à 7 jours 6
- décès de 7 à 28 jours 2
- décès de 28 jours à 1 an 4

Le taux de mortalité infantile est de :


A - 22/5.000.000
B - 22/ 1.000
C - 12/5.000.000
D - 12/1.000
E - 12/990
Bonne(s) réponse(s) :

QUESTION ANNULEE

Le taux de mortalité périnatale est de :


A - 16/1.000
B - 18/1.000
C - 8/1.000
D - 6/1.000
E - 10/1.000
Bonne(s) réponse(s) : A

C'est le nombre de décès de la 28ème semaine d'aménorrhée au 7ème jour de vie.

Des données ci-dessous, laquelle permet d'évaluer le mieux l'impact des causes exogènes de mortalité foeto-
infantile ?
A - Le nombre de décès de 0 à 7 jours
B - Le taux de mortalité périnatale
C - Le nombre de décès de 28 jours à 1 an
D - Le taux de mortalité néonatale précoce
E - Le taux de mortalité néonatale tardive
Bonne(s) réponse(s) : C

Sans commentaire.

1372
Exclusivement sur DOC - DZ : www.doc-dz.com NADJI 85
RESIDANAT EN POCHE TOME II
Cas Clinique en QCM

La mortinatalité est :
A - Le nombre de mort in utéro à partir de la 28ème semaine de gestation
B - Le nombre de décès de la 28ème semaine de gestation au 7ème jour après la naissance
C - Le nombre de décès de 0 à 6 jours
D - L'indicateur le moins sensible aux actions de protection maternelle et infantile
E - Un mauvais indicateur des causes endogènes de mortalité foeto-infantile
Bonne(s) réponse(s) : A D

Sans commentaire.

Entre 1966 et 1967, huit jeunes femmes sont hospitalisées aux Etats-Unis pour adénocarcinome à cellules claires du vagin.
Cette tumeur étant jusque-là considérée comme rarissime, une enquête épidémiologique a été menée à la recherche des
facteurs explicatifs dans les antécédents de ces jeunes cancéreuses comparées à 32 femmes saines de même âge et de
même lieu de naissance.

Cliquez sur le bouton "Dessin" pour afficher le tableau.

De quel(s) type(s) d'enquête s'agit-il ?

Cliquez sur le bouton "dessin" pour afficher le tableau.


A - Enquête de type exposés - non exposés
B - Enquête analytique
C - Enquête rétrospective
D - Enquête cas/témoins
E - Enquête de cohorte
Bonne(s) réponse(s) : B C D

Sans commentaire.

Le choix d'un groupe comparatif de même âge et de même lieu de naissance correspond à une méthode
appelée en épidémiologie :
Cliquez sur le bouton "dessin" pour afficher le tableau.
A - Une standardisation
B - Un appariement
C - Un ajustement
D - Une étude en double insu
E - Une randomisation
Bonne(s) réponse(s) : B

Sans commentaire.

Peut-on dire, au vu du tableau de la page précédente, que :


(Diéthylstilbestrol = D.E.S.OE).
Cliquez sur le bouton "dessin" pour afficher le tableau.
A - Le D.E.S.OE est un facteur de risque de l'adénocarcinome du vagin
B - Le D.E.S.OE est la cause de l'adénocarcinome du vagin
C - Toutes les femmes ayant pris du D.E.S.OE pendant leur grossesse ont fait ou feront un adénocarcinome
du vagin
D - Le rôle du D.E.S.OE n'est pas statistiquement démontré
E - Toutes les femmes prenant du D;E.S.OE sont atteintes d'adénocarcinome du vagin
Bonne(s) réponse(s) : A

Sans commentaire.

Ce type d'enquête :

Cliquez sur le bouton "dessin" pour afficher le tableau.


A - Nécessite un diagnostic parfaitement défini
B - Est possible même quand la maladie est rare
C - Peut-être biaisé par les perdus de vue
D - Demande habituellement plusieurs années de travail continu
E - Nécessite beaucoup de soins dans la constitution du groupe témoin
Bonne(s) réponse(s) : A B E

Sans commentaire.

1373
Exclusivement sur DOC - DZ : www.doc-dz.com NADJI 85
RESIDANAT EN POCHE TOME II
Cas Clinique en QCM
Pour mieux étudier s'il y a une relation entre la contraception par voie orale (oestro-progestatifs) et le cancer du col utérin, un
groupe de 15 000 femmes de 18 à 58 ans, indemnes de cancer du col, a été suivi de décembre 1976 à janvier 1980
La surveillance a consisté essentiellement en un frottis annuel ; parallèlement la prise de contraceptif était recherchée par
questionnaire.

Il s'agit d'une enquête :


A - Longitudinale
B - Prospective
C - De cohorte
D - D'épidémiologie analytique
E - Cas-témoins
Bonne(s) réponse(s) : A B C D

Sans commentaire.

Sur le tableau suivant, figurent les résultats de l'enquête en fonction de la durée de la prise de contraceptifs :

Cliquez sur le bouton "dessin" pour afficher le tableau.

Avez-vous suffisamment d'éléments pour compléter le tableau et calculer et le taux d'incidence du cancer du col
chez les femmes ayant utilisé un contraceptif pendant plus de 4 ans ?
A - Non, il faudrait connaître le nombre de femmes qui ont été exposées pendant plus de 4 ans
B - Non, car on ne connait pas l'incidence chez les non exposées
C - Non, il faudrait connaître l'incidence du cancer du col dans la population générale
D - Oui, 1 = 160/100 000
E - Non, il faudrait connaître le nombre de fumeuses qui n'ont pas été exposées
Bonne(s) réponse(s) : D

Sans commentaire

Le fait que le risque relatif augmente de 1 à 5 en fonction de l'exposition au risque :


A - Confirme l'hypothèse selon laquelle la pilule est un facteur de risque de cancer du col
B - Prouve que la pilule est la cause du cancer du col
C - Est un argument en faveur de la causalité
D - N'a aucune signification étiologique dans ce type d'enquête
E - N'a rien à voir avec la relation de causalité
Bonne(s) réponse(s) : A C

Sans commentaire.

Comment, à votre avis, a-t-on procédé pour chiffrer le risque relatif ?


A - On a calculé directement le risque relatif par le rapport de 2 incidences
B - On a comparé la prise de contraceptifs chez les femmes atteintes de cancer du col et chez les femmes
indemnes
C - On a comparé la fréquence de survenue du cancer du col chez les utilisatrices
D - On a calculé le chi-deux
E - On a utilisé une méthode d'approximation (odds ratio)
Bonne(s) réponse(s) : A

Risque relatif = taux d'incidence des sujets exposés/taux d'incidence des sujets non exposés.

Une enquête cas-témoins a donné les résultats suivants (tableau de contingence ci-dessous) :

Cliquez sur le bouton "Dessin" pour afficher le tableau.

Pour obtenir ce résultat de chi-deux, il a fallu :

Cliquez sur le bouton "dessin" pour afficher le tableau.


A - Calculer p
B - Prendre en compte les effectifs observés
C - Prendre en compte les effectifs théoriques (ou calculés)
D - Prendre en compte les pourcentages observés
E - Prendre en compte les pourcentages théoriques (ou calculés)
Bonne(s) réponse(s) :

QUESTION ANNULEE.
1374
Exclusivement sur DOC - DZ : www.doc-dz.com NADJI 85
RESIDANAT EN POCHE TOME II
Cas Clinique en QCM

Dans le cas considéré, p < 0,01 signifie que :

Cliquez sur le bouton "dessin" pour afficher le tableau.


A - La fréquence de l'éthylisme est plus élevée dans le groupe de malades atteints d'un infarctus du myocarde
(degré de signification p < 0.01)
B - Il existe une association entre l'infarctus du myocarde et l'éthylisme chronique (degré de signification p <
0,01)
C - Il existe un biais de mesure (degré de signification p < 0,01)
D - Il existe un biAis de sélection (degré de signification p < 0,01)
E - Il existe un facteur de confusion (degré de signification p < 0,01 )
Bonne(s) réponse(s) :

QUESTION ANNULEE.

A propos des résultats de cette enquête cas-témoins, il est impératif de s'assurer de :

Cliquez sur le bouton "dessin" pour afficher le tableau.


A - L'absence d'un biais dE sélection
B - L'absence d'un biais de mesure
C - Des critères du diagnostic d'éthylisme chronique
D - L'identité des enquêteurs
E - Que les groupes ont été constitués par tirage au sort
Bonne(s) réponse(s) : A B C

Sans commentaire.

Dans cette enquête cas-témoins, le tabagisme s'est révélé être associé à la fois à l'éthylisme chronique et à
l'infarctus du myocarde. Cela signifie que le tabagisme peut intervenir comme :
Cliquez sur le bouton "dessin" pour afficher le tableau.
A - Facteur causal de l'infarctus du myocarde
B - Facteur causal de l'éthylisme chronique
C - Biais de sélection
D - Biais de mesure
E - Facteur de confusion
Bonne(s) réponse(s) : C E

Sans commentaire.

Le graphique ci-dessous représente de façon simplifiée les 60 cas de tuberculose survenus dans un camp de 1 000 réfugiés
du sud-est asiatique en France

Cliquez sur le bouton "Dessin" pour afficher le graphique.

Le taux de prévalence pour l'année 1986 est de :

Cliquez sur le bouton "Dessin" pour afficher le graphique.


A - 20/60
B - 3/60
C - 20/1.000
D - 30/1.000
E - 60/1.000
Bonne(s) réponse(s) : E

Prévalence = nb cas/effectif de la population = 60/1000.

Le taux d'incidence durant l'année 1986 est de :

Cliquez sur le bouton "Dessin" pour afficher le graphique.


A - 20/60
B - 30/60
C - 20/1.000
D - 30/1.000
E - 60/1.000
Bonne(s) réponse(s) : D

Incidence = nb de nouveaux cas/effectif de la population = 30/1000.

1375
Exclusivement sur DOC - DZ : www.doc-dz.com NADJI 85
RESIDANAT EN POCHE TOME II
Cas Clinique en QCM

Le taux de létalité durant l'année 1986 est de :

Cliquez sur le bouton "Dessin" pour afficher le graphique.


A - 20/60
B - 30/60
C - 20/1.000
D - 30/1.000
E - 60/1.000
Bonne(s) réponse(s) : C

Taux de létalité = nb de décès par une maladie/nb de cas de la maladie = 20/1000.

Selon les règles utilisées en France, quelle(s) est(sont) la(les) mesure(s) nécessaire(s) vis-à-vis des malades
tuberculeux bacillifères dépistés ?
Cliquez sur le bouton "Dessin" pour afficher le graphique.
A - Intradermoréaction à la tuberculine
B - Vaccination BCG immédiate
C - Traitement par 3 antibiotiques antituberculeux pendant au moins 9 mois
D - Traitement par 2 antibiotiques antituberculeux pendant 6 mois
E - Déclaration obligatoire
Bonne(s) réponse(s) : E

A B. Aucun intérêt dès lors que le diagnostic est établi.


C D. Non, il faut traiter par tri ou quadrithérapie pendant 2 mois puis bithérapie.

Le diagnostic de thyrotoxicose pose le problème du choix de tests les plus performants. On compare une population de 200
hyperthyroïdiens (preuve apportée par l'évolution) à une population témoin identique en nombre.
Le dosage de la thyroxine (T4) donne 160 résultats corrects chez les malades et 180 corrects chez les non malades.
Sur les deux groupes, le dosage de la triiodothyronine (T3) donne 30 faux positifs et 10 faux négatifs.
En couplant les deux tests, on classe positifs les sujets ayant les deux dosages positifs, et négatifs tous les autres cas : on
obtient 160 malades et 195 normaux bien classés.

Quelle est la sensibilité du dosage de la thyroxine (T4) ?


A - 60 %
B - 70 %
C - 80 %
D - 85 %
E - 95 %
Bonne(s) réponse(s) :

QUESTION ANNULEE.

Quelle est la spécificité du dosage de la thyroxine (T4) ?


A - 60 %
B - 70 %
C - 80 %
D - 90 %
E - 95 %
Bonne(s) réponse(s) :

QUESTION ANNULEE.

Quelle est la sensibilité du dosage de la triiodothyronine (T3) ?


A - 60 %
B - 70 %
C - 80 %
D - 85 %
E - 95 %
Bonne(s) réponse(s) :

QUESTION ANNULEE.

1376
Exclusivement sur DOC - DZ : www.doc-dz.com NADJI 85
RESIDANAT EN POCHE TOME II
Cas Clinique en QCM

Quelle est la spécificité du dosage de la triiodothyronine (T3) ?


A - 75 %
B - 80 %
C - 85 %
D - 90 %
E - 95 %
Bonne(s) réponse(s) :

QUESTION ANNULEE.

Quelle est la sensibilité du test couplé ?


A - 60 %
B - 70 %
C - 80 %
D - 85 %
E - 95 %
Bonne(s) réponse(s) :

QUESTION ANNULEE.

Quelle est la spécificité du test couplé ?


A - 87,5 %
B - 90 %
C - 92,5 %
D - 95 %
E - 97,5 %
Bonne(s) réponse(s) :

QUESTION ANNULEE.

Quel est le meilleur test pour éliminer le diagnostic de thyrotoxicose, quand le résultat est négatif ?
A - Le dosage de la thyroxine (T4)
B - Le dosage de la triiodothyronine (T3)
C - Le test couplé
D - L'évolution clinique
E - Aucune des propositions ci-dessus
Bonne(s) réponse(s) :

QUESTION ANNULEE.

Quel est le meilleur test pour poser le diagnostic de thyrotoxicose quand le résultat est positif ?
A - Le dosage de la thyroxine (T4)
B - Le dosage de la triiodothyronine (T3)
C - Le test couplé
D - L'évolution clinique
E - Aucune des propositions ci-dessus
Bonne(s) réponse(s) :

QUESTION ANNULEE.

1377
Exclusivement sur DOC - DZ : www.doc-dz.com NADJI 85
RESIDANAT EN POCHE TOME II
Cas Clinique en QCM
Un nouveau test vient d'être proposé pour le dépistage de la polyarthrite rhumatoïde. On procède à une évaluation de ce test
et on obtient les données suivant :

Cliquez sur le bouton "Dessin" pour afficher le tableau.

Quelle est la prévalence de la polyarthrite rhumatoïde dans cette population ?

Cliquez sur le bouton "dessin" pour afficher le tableau.


A - 0,02
B - 0,06
C - 0,10
D - 0,14
E - 0,18
Bonne(s) réponse(s) : C

Prévalence = nb de cas/effectif de la population = 986/9863 = 10 %.

Quelle est la spécificité du test ?

Cliquez sur le bouton "dessin" pour afficher le tableau.


A - 0,50
B - 0,60
C - 0,70
D - 0,80
E - 0,90
Bonne(s) réponse(s) : E

Spécificité = VN/(VN+FP) = 7989/(7989+888) = 0,90.

Quelle est la sensibilité du test ?

Cliquez sur le bouton "dessin" pour afficher le tableau.


A - 0,50
B - 0,60
C - 0,70
D - 0,80
E - 0,90
Bonne(s) réponse(s) : E

Sensibilité = VP/(VP+FN) = 887/(887+99) = 0,90.

Quelle est la valeur prédictive positive du test ?

Cliquez sur le bouton "dessin" pour afficher le tableau.


A - 0,50
B - 0,60
C - 0,70
D - 0,80
E - 0,90
Bonne(s) réponse(s) : A

Valeur prédictif d'un test positif = VP/(VP+FP) = 887/887+888 = 0,50.

Si on évalue ce test dans une population où la prévalence est deux fois plus élevée :

Cliquez sur le bouton "dessin" pour afficher le tableau.


A - La sensibilité du test sera augmentée
B - La spécificité du test sera diminuée
C - La valeur prédictive positive sera augmentée
D - La valeur prédictive négative sera diminuée
E - Sensibilité et spécificité seront augmentées
Bonne(s) réponse(s) : A C

Sans commentaire.

1378
Exclusivement sur DOC - DZ : www.doc-dz.com NADJI 85
RESIDANAT EN POCHE TOME II
Cas Clinique en QCM
Une enquête épidémiologique à porté sur 60 000 femmes âgées de 35 à 59 ans, et étudié la relation entre l'ingestion d'alcool
et la survenue d'un accident cardiaque ou vasculaire cérébral. Il était demandé aux femmes leur consommation moyenne
quotidienne, traduite en grammes d'alcool pur; l'événement retenu était la survenue dans les 10 ans après inclusion, d'un
infarctus ou d'un accident vasculaire cérébral.
Les résultats sont les suivants : parmi les 30 000 femmes ayant une consommation nulle, 450 ont fait un infarctus ; parmi les
20 000 femmes consommant moins de 15 g d'alcool par jour, 210 ont fait un infarctus, et parmi les 10 000 femmes restantes
(= ou > 15 g/j), 75 ont fait un infarctus.

Il s'agit :
A - D'une enquête transversale
B - D'une enquête prospective avec contrôle du facteur de risque
C - D'une enquête cas-témoin
D - D'une enquête prospective sans contrôle du facteur de risque
E - D'une expérimentation
Bonne(s) réponse(s) : D

Choix au départ de deux groupes présentant des caractéristiques communes sur tous les plans, sauf pour le facteur étudié.
A - Etude à un moment donné de plusieurs cohortes.
C - Rétrospective ou transversale, compare des sujets malades à des sujets sains, regarde l'exposition à un facteur de risque.

Si l'on prend comme référence les femmes ne buvant pas d'alcool, quel est le risque relatif de faire un infarctus
pour les femmes buvant moins de 15 g d'alcool/jour ?
A - (450/210) / (30.000/20.000) = 1,4
B - (450 x 19.790) / (29.550 x 210) = 1,4
C - (210 x 30.000) / (20.000 x 450) = 0,7
D - (210/75) / (20.000/60.000) = 0,9
E - On ne peut le calculer
Bonne(s) réponse(s) : C

Risque relatif : (Taux Incidence du Pb dans la population exposée au FDR / Taux Incidence du Pb dans la population non
exposée au FDR)

Taux incidence : (Nombre de Cas / Effectif population) pendant une période donnée

Ici : (I < 15 g ) / ( I pas alcool) = (210 / 20 000) x (30 000 / 450)

Même question pour les femmes buvant plus de 15 g d'alcool par jour.
A - (450/75) / (30.000/10.000) = 2
B - (450 x 9.925) / (29.500 x 75) = 2
C - (75 x 30.000) / (10.000 x 450) = 0,5
D - (75/735) / (10.000/60.000) = 0,6
E - On ne peut le calculer
Bonne(s) réponse(s) : C

(I > 15 g ) / ( I pas alcool) = (75 / 10 000) x (30 000 / 450)

Les résultats rapportés permettent de conclure que :


A - L'alcool est une cause d'infarctus du myocarde chez la femme
B - L'alcool à des propriétés protectrices contre l'infarctus du myocarde chez la femme
C - Dans cette étude, le risque d'infarctus du myocarde est plus élevé chez les femmes qui consomment de
l'alcool
D - Dans cette étude, le risque d'infarctus du myocarde est plus faible chez les femmes qui consomment de
l'alcool
E - Il serait utile de prendre en compte d'autres facteurs de risque d'infarctus du myocarde avant de conclure
sur le rôle éventuel de l'alcool
Bonne(s) réponse(s) : D E

Sans commentaire.

1379
Exclusivement sur DOC - DZ : www.doc-dz.com NADJI 85
RESIDANAT EN POCHE TOME II
Cas Clinique en QCM

En ce qui concerne les accidents vasculaires cérébraux (AVC), les résultats vont dans le sens inverse de ceux
de l'infarctus du myocarde : 8 AVC, 6 AVC et 4 AVC respectivement dans les 3 groupes prédéfinis (Alcool : 0,15
g = ou > 15 g) Quelle(s) conclusion(s) peut-on en tirer ?
A - L'alcool protège contre le risque d'AVC
B - Les résultats sont contradictoires, ce qui suppose un biais dans l'étude
C - Il semble exister une relation entre alcool et AVC
D - L'alcool est une cause d'AVC chez la femme
E - Ces résultats méritent d'être vérifiés sur un plus grand nombre de sujets
Bonne(s) réponse(s) : C E

E - Les trois groupes avec AVC sont trop restreints pour pouvoir effectuer une étude comparative. Des études en utilisant des
facteurs de correction peuvent être employées comme le test de chi-deux de Yates.

Un registre de l'infarctus du myocarde a été mis en place dans trois départements français pour une durée de 10 ans. Durant
l'année 1987, les résultats suivants ont été relevés dans deux départements :
Département 1 :
Population 1 million d'habitants :
- taux brut de mortalité 30 000/1 000 000
- taux spécifique de mortalité par infarctus du myocarde : 500/100 000.
Département 2 :
Population 1 million d'habitants :
- taux brut de mortalité 25 000/1 000 000
- taux spécifique de mortalité par infarctus du myocarde : 500/100 000.

Au terme des 10 années, quel indicateur traduira le mieux l'efficacité des actions d'éducation pour la santé
conduites dans les trois départements pour lutter contre le tabagisme, les déséquilibres alimentaires et leurs
influence sur la survenue d'infarctus du myocarde ?
A - Taux de prévalence
B - Taux d'incidence
C - Taux de létalité
D - taux de mortalité spécifique
E - Taux de mortalité proportionnelle
Bonne(s) réponse(s) : B

A - (Nombre de cas existant à T1 + Nombre de nouveaux cas apparus entre T1 et T2 / Effectif de la population T1 + T2 / 2)

B - (Nombre nouveaux cas pendant une période donnée / Effectif population)

Le taux d'incidence mesure la vitesse de propagation de la maladie dans une population.

C - (Nombre de décès pour une maladie / Nombre total de cas de cette maladie)

Au terme des 10 années, quel indicateur traduira le mieux les progrès thérapeutiques réalisés ?
A - Taux de prévalence
B - Taux d'incidence
C - Taux de mortalité spécifique
D - Taux de létalité
E - Taux de mortalité proportionnelle
Bonne(s) réponse(s) : D

Voir question précédente.

A propos des résultats rapportés dans l'énoncé, quelle(s) est(sont) la(les) proposition(s) exactes ?
A - Le risque pour un individu de mourir de cette maladie est le même dans les deux populations
B - On ne peut calculer le taux proportionnel de mortalité par infarctus
C - Le taux proportionnel de mortalité par infarctus est plus élevé dans le département 2
D - Le taux de létalité de l'infarctus du myocarde est plus élevé dans le département 2
E - On ne peut calculer le taux de létalité par infarctus
Bonne(s) réponse(s) : A C E

A - Taux bruts de mortalité identiques.


E - Voir commentaire .

1380
Exclusivement sur DOC - DZ : www.doc-dz.com NADJI 85
RESIDANAT EN POCHE TOME II
Cas Clinique en QCM

Après standardisation sur l'âge, on obtient un taux brut de mortalité identique dans les deux populations, on
peut conclure que :
A - Les taux de mortalité proportionnelle par infarctus deviennent identiques dans les deux populations
B - La population 1 est plus vieille que la population 2
C - On ne peut rien conclure quant à la distribution par âge dans ces deux populations
D - Ces taux standardisés ne sont pas des taux réels directement observés sur les deux populations
E - La standardisation pour l'âge n'a aucun intérêt pour étudier l'épidémiologie de l'infarctus du myocarde
Bonne(s) réponse(s) : B D

Sans commentaire.

Durant l'année 1987 une sévère épidémie de grippe affecte le département 1, entraînant 10% de décès de plus
que l'année précédente. La population est restée stable par rapport à l'année 1987, ainsi que les taux
d'incidence, de prévalence et le nombre de décès par infarctus.
Quel(s) résultat(s) a-t-on enregistré en 1984 par rapport à 1983 ?
A - Une baisse du taux de létalité de l'infarctus du myocarde
B - Une baisse du taux de mortalité proportionnelle par infarctus du myocarde
C - Un taux de létalité de l'infarctus du myocarde identique
D - Un taux de mortalité proportionnelle par infarctus du myocarde identique
E - Un taux de mortalité spécifique par infarctus du myocarde identique
Bonne(s) réponse(s) : B C E

Voir commentaire [494]

Le diagnostic de thyrotoxicose pose le problème du choix de tests les plus performants. On compare une population de 200
hyperthyroïdiens (preuve apportée par l'évolution) à une population témoin identique
en nombre.
Le dosage de la thyroxine (T4) donne 160 résultats corrects chez les malades et 180 corrects chez les non malades.
Sur les deux groupes, le dosage de la triiodothyronine (T3) donne 30 faux positifs et 10 faux négatifs.
En couplant les deux tests, on classe positifs les sujets ayant les deux dosages positifs, et négatifs tous les autres cas : on
obtient 160 malades et 195 normaux bien classés.

Quelle est la sensibilité du dosage de la thyroxine (T4) ?


A - 60 %
B - 70 %
C - 80 %
D - 85 %
E - 95 %
Bonne(s) réponse(s) : C
SUJETS MALADES SUJETS SAINS TOTAL
Test+ Vrais positifs : a Faux positifs : b a+b
Test- Faux négatifs : c Vrais négatifs : d c+d
TOTAL a+c b+d

Sensibilité : Vrais positifs / ( Vrais positifs + Faux négatifs) = a / (a + c)

Quelle est la spécificité du dosage de la thyroxine (T4) ?


A - 60 %
B - 70 %
C - 80 %
D - 90 %
E - 95 %
Bonne(s) réponse(s) : D

Spécificité : (Vrais négatifs) / (Vrais négatifs + Faux positifs) = d / ( b + d)

Quelle est la sensibilité du dosage de la triiodothyronine (T3) ?


A - 60 %
B - 70 %
C - 80 %
D - 85 %
E - 95 %
Bonne(s) réponse(s) : E

Voir commentaire [11].

1381
Exclusivement sur DOC - DZ : www.doc-dz.com NADJI 85
RESIDANAT EN POCHE TOME II
Cas Clinique en QCM

Quelle est la spécificité du dosage de la triiodothyronine (T3) ?


A - 75 %
B - 80 %
C - 85 %
D - 90 %
E - 95 %
Bonne(s) réponse(s) : C

Voir commentaire [12].

Quelle est la sensibilité du test couplé ?


A - 60 %
B - 70 %
C - 80 %
D - 85 %
E - 95 %
Bonne(s) réponse(s) : C

Sans commentaire.

Quelle est la spécificité du test couplé ?


A - 87,5 %
B - 90 %
C - 92,5 %
D - 95 %
E - 97,5 %
Bonne(s) réponse(s) : A

Sans commentaire.

Quel est le meilleur test pour éliminer le diagnostic de thyrotoxicose, quand le résultat est négatif ?
A - Le dosage de la thyroxine (T4)
B - Le dosage de la triiodothyronine (T3)
C - Le test couplé
D - L'évolution clinique
E - Aucune des propositions ci-dessus
Bonne(s) réponse(s) : B

Sensibilité de T3 est la plus forte.

Quel est le meilleur test pour poser le diagnostic de thyrotoxicose quand le résultat est positif ?
A - Le dosage de la thyroxine (T4)
B - Le dosage de la triiodothyronine (T3)
C - Le test couplé
D - L'évolution clinique
E - Aucune des propositions ci-dessus
Bonne(s) réponse(s) : A

Spécificité T4 la plus forte.

1382
Exclusivement sur DOC - DZ : www.doc-dz.com NADJI 85
RESIDANAT EN POCHE TOME II
Cas Clinique en QCM
Entre 1966 et 1967, huit jeunes femmes sont hospitalisées aux Etats-Unis pour adénocarcinome à cellules claires du vagin.
Cette tumeur étant jusque là considérée comme rarissime, une enquête épidémiologique a été menée à la recherche des
facteurs explicatifs dans les antécédents de ces jeunes cancéreuses comparées à 32 femmes saines de même âge et de
même lieu de naissance.

Cliquez sur le bouton "Dessin" pour afficher le graphique.

De quel(s) type(s) d'enquête s'agit-il ?

Cliquez sur le bouton "dessin" pour afficher le tableau.


A - Enquête descriptive
B - Enquête prospective
C - Enquête rétrospective
D - Enquête cas/témoins
E - Enquête de cohorte
Bonne(s) réponse(s) : C D

C = D ; but : rechercher des hypothèses de facteurs étiologiques.

Le choix d'un groupe comparatif de même âge et de même lieu de naissance correspond à une méthode
appelée en épidémiologie :
Cliquez sur le bouton "dessin" pour afficher le tableau.
A - Une standardisation
B - Un appariement
C - Une pyramide des âges
D - Une étude en double insu
E - Une randomisation
Bonne(s) réponse(s) : B

Sans commentaire.

Peut-on dire, au vu du tableau ci-dessus, que : (Diéthylstilbestrol = D.E.S.OE) :

Cliquez sur le bouton "dessin" pour afficher le tableau.


A - Le D.E.S.OE est un facteur de risque de l'adénocarcinome du vagin
B - Le D.E.S.OE est la cause de l'adénocarcinome du vagin
C - Toutes les femmes ayant pris du D.E.S.OE pendant leur grossesse ont fait ou feront un adénocarcinome
du vagin
D - Le lien entre le D.E.S.OE et l'adénocarcinome du vagin est statistiquement démontré
E - Le nombre de cas est trop faible pour conclure
Bonne(s) réponse(s) : A D

A - Hypothèse de facteur étiologique.


D - p < 0,00001.
B - Il faut pourvoir effectuer des tests en laboratoire avant de l'affirmer.

Pour confirmer les résultats de l'étude, des auteurs proposent un essai thérapeutique dont le critère de
jugement sera l'apparition d'adénocarcinome chez la descendance féminine des femmes enceintes. Quelle est
la ou les propositions exactes ?
Cliquez sur le bouton "dessin" pour afficher le tableau.
A - On ne peut jamais faire d'essai thérapeutique chez la femme enceinte
B - On ne peut jamais faire d'essai thérapeutique impliquant les enfants
C - Parmi les critères d'exclusion de l'essai, il est raisonnable de ne pas retenir les femmes ayant bénéficié
d'une fécondation in vitro
D - Parmi les critères d'exclusion de l'essai, il est raisonnable de ne pas retenir les femmes de moins de 29
ans
E - Des raisons éthiques interdisent de mener une telle étude
Bonne(s) réponse(s) : E

A, B - Variable en fonction des produits utilisés.

1383
Exclusivement sur DOC - DZ : www.doc-dz.com NADJI 85
RESIDANAT EN POCHE TOME II
Cas Clinique en QCM
On envisage le dépistage d'une maladie M grâce à un test biologique quantitatif.

On hésite entre deux seuils de "normalité" (limites supérieures) : A = 30 unités internationales B = 20 unités
internationales On peut prévoir avec certitude que :
A - Le test A sera plus sensible que le test B
B - Le test A sera plus spécifique que le test B
C - Le nombre de faux + sera plus grand avec le test A
D - Le nombre de faux + sera plus grand avec le test B
E - Le test A dépistera plus de malades que le test B
Bonne(s) réponse(s) : A D

Sensibilité : proportion de malades diagnostiqués par rapport à l'intégrité des malades.


Spécificité : proportion de sujets sains diagnostiqués. LEn augmentant le seuil de normalité, on réduit le nombre de patients
en dehors de cette norme, la sensibilité diminue et la spécificité augmente.

Si l'on choisit une valeur seuil telle que le test à une spécificité de 100 % cela implique que :
A - Sa valeur prédictive positive est aussi de 100 %
B - Devant un test positif, on peut affirmer que le sujet est malade
C - On prend le risque de laisser échapper beaucoup de malades au dépistage
D - La valeur prédictive négative est de 100 %
E - Il n'y a pas de faux négatifs
Bonne(s) réponse(s) : A B
M+ M-
+ a b a+b
- c d c+d
a+c b+d
Vp test+ = a / ( a + b) sensibilité = a / ( a + c) spécificité = d / (b + d)
Vp test- = d / (c + d)
A - spécificité : (100 / 100) = (d / b + d) , b = 0
Si b = 0 Vp test+ = 100 % CQFD !
B - Par définition de la valeur prédictive du test positif.
C - Dépend de la sensibilité.

On choisit finalement une valeur seuil telle que la sensibilité est de 9 5 % et la spécificité de 90 %. Votre
première campagne de dépistage s'applique à une population où la prévalence de la maladie est de 2 %. En
une semaine, vous examinez 1000 personnes. Le nombre attendu :
A - De sujets positifs est 117
B - De vrais positifs est 19
C - De vrais négatifs est 980
D - De faux négatifs est 98
E - On ne peut prédire sur les seules données de l'énoncé le nombre de vrais et faux négatifs
Bonne(s) réponse(s) : A B
Malades Non malades TOTAL
Test+ a b a+b
Test c d c+d
Total a+c b+d 1000

Prévalence : (Nombre de cas malades / population) = a + c = 20 ; donc b + d = 980


Sensibilité : a / ( a + c) = 95 /100
Spécificité : d / (b + d) = 90 /100
a + c = 20 et a / ( a + c) = 95 / 100 , d = 882
d'ou b + d = 117

Votre deuxième campagne de dépistage s'applique à une population où la prévalence de la maladie est plus
élevée.
A - La valeur prédictive positive du test sera meilleure que dans le cas précédent
B - La validité interne du test sera meilleure que dans le cas précédent
C - La sensibilité du test sera meilleure que dans le cas précédent
D - La spécification du test sera meilleure que dans le cas précédent
E - Le rendement du dépistage sera meilleur que dans le cas précédent
Bonne(s) réponse(s) : A E

Si prévalence augmente, a augmente et donc Vp+ = a / (a + b) est meilleure.

1384
Exclusivement sur DOC - DZ : www.doc-dz.com NADJI 85
RESIDANAT EN POCHE TOME II
Cas Clinique en QCM
70 % de la population normale a une glycémie inférieure au taux C, et 85 % des diabétiques ont une glycémie supérieure à C.
Si l'on choisit le point C comme point de partage entre les deux groupes (considérant comme normales les glycémies
inférieures à C, et comme diabétiques les glycémies supérieures à C)

Quelle est la sensibilité du test ?


A - 0,70
B - 0,30
C - 0,85
D - 0,15
E - 0,50
Bonne(s) réponse(s) : C

Diabétique Non diabétique TOTAL


Glycémie>c 85 30 115
Glycémie<c 15 70 85
TOTAL 100 100 200

Sensibilité : 85 / 100 = 0,85

Quelle est sa spécificité ?


A - 0,70
B - 0,30
C - 0,85
D - 0,15
E - 0,50
Bonne(s) réponse(s) : A

Spécificité : 70 / 100 = 0,70

Si la prévalence P du diabète sucré est égale à 5 %, quelle est la valeur prédictive positive, c'est à dire la
probabilité du diabète sucré pour un patient dont la glycémie à jeun est égale à C ? (seules 2 décimales sont
conservées).
A - 0,13
B - 0,15
C - 0,30
D - 0,50
E - 0,85
Bonne(s) réponse(s) :

Réponse : 0,06
Vp+ = a / ( a + b) = 0,06
P=a+c=5 a =4,25 c = 0,75 b + d = 95 b = 66,5 d = 28,5

Et dans ces mêmes conditions (P = 0,05), quelle est la valeur prédictive négative, c'est-à-dire la probabilité pour
que ce patient dont la glycémie = C ne soit pas diabétique ?
A - 0,15
B - 0,30
C - 0,50
D - 0,87
E - 0,59
Bonne(s) réponse(s) :

Réponse : 0,97
Vp- = d / (c + d) = (28,5 / 29,25) = 0,97

On veut étudier l'action hypouricémiante d'un nouveau produit M. La justification de cet essai tient au fait qu'une élévation du
taux de l'uricémie peut entraîner l'apparition de crises de goutte et de lithiases urinaires.

Les sujets obéissant aux critères d'inclusion seront :


A - Le tout venant d'un service hospitalier
B - Les malades ayant déjà eu au moins une crise de goutte typique
C - Les malades dont l'uricémie est supérieure à la normale ( 80 mg/l)
D - Des sujets indemnes de toute maladie sélectionnés par tirage au sort
E - Autre réponse
Bonne(s) réponse(s) : C

C - En effet, dans l'énoncé on considère l'hypermicémie a priori comme délétère, et on teste les capacités d'un nouveau
médicament à pouvoir la réduire.
1385
Exclusivement sur DOC - DZ : www.doc-dz.com NADJI 85
RESIDANAT EN POCHE TOME II
Cas Clinique en QCM

Le régime alimentaire sera le même :


A - Uniquement pour les sujets du groupe traité par le nouveau médicament M
B - Uniquement pour un échantillon représentatif du groupe traité par le nouveau médicament M
C - Uniquement pour les sujets du groupe témoin
D - Uniquement pour un échantillon représentatif du groupe témoin
E - Pour l'ensemble des sujets des deux groupes
Bonne(s) réponse(s) : E

E - Evident.

le groupe témoin doit recevoir:


A - Un régime alimentaire et un traitement anti-inflammatoire (Aspirine)
B - Un régime, un placebo et un traitement anti-inflammatoire (Aspirine)
C - Un régime et un placebo
D - Un régime seul
E - Autre réponse
Bonne(s) réponse(s) : C

C - Principe du double aveugle.

Le critère de jugement sera :


A - Le taux d'uricémie final
B - La baisse du taux d'uricémie par rapport au taux initial avant traitement
C - La disparition des crises de goutte
D - Le retour à la normale du taux de l'uricémie
E - Autre réponse
Bonne(s) réponse(s) : B

B - Evident.

Le graphique ci-dessous représente de façon simplifiée les 60 cas de toxi-infection alimentaire survenus dans un camp de 1
000 réfugiés du sud-est asiatique.

Cliquez sur le bouton "Dessin" pour afficher le graphique.

Le taux de prévalence pour le mois de janvier est de :

Cliquez sur le bouton "dessin" pour afficher le tableau.


A - 20/60
B - 30/60
C - 20/1.000
D - 30/1.000
E - 60/1.000
Bonne(s) réponse(s) : E

Prévalence = (Nb tot. de cas existant à T1 + Nb de nvx cas survenus entre T1 et T2) / (Effectif de la population à T1 + T2 / 2)

Le taux d'incidence durant le mois de janvier est de :

Cliquez sur le bouton "dessin" pour afficher le tableau.


A - 20/60
B - 30/60
C - 20/1.000
D - 30/1.000
E - 60/1.000
Bonne(s) réponse(s) : D

Incidence :
Nombre de cas survenant au sein d'une population pendant 1 temps / Effectif de la population

1386
Exclusivement sur DOC - DZ : www.doc-dz.com NADJI 85
RESIDANAT EN POCHE TOME II
Cas Clinique en QCM

Le taux de létalité durant le mois de Janvier est de :

Cliquez sur le bouton "dessin" pour afficher le tableau.


A - 20/60
B - 30/60
C - 20/1.000
D - 30/1.000
E - 60/1.000
Bonne(s) réponse(s) : A

Totalité : (Nombre de décès pour une maladie / Nombre total de cas de cette maladie) pour une période donnée

D'une manière générale, la prévalence d'une maladie M peut être influencé par :

Cliquez sur le bouton "dessin" pour afficher le tableau.


A - L'incidence de cette maladie
B - La durée moyenne de cette maladie
C - La qualité du recueil des données
D - L'existence d'un éventuel biais de mesure
E - L'existence d'un éventuel biais de sélection
Bonne(s) réponse(s) : A B C D E

La prévalence mesure l'importance de la propagation d'une maladie dans une population.


AB - P=Ixd I = incidence d = durée de la maladie.

En Septembre 1981, un médecin d'une commune de 15 000 habitants, signale à la D.D.A.S.S. plusieurs cas de gastro-
entérite. Un contrôle de l'eau d'adduction publique est aussitôt pratiqué et révèle l'existence d'une pollution bactériologique (E.
COLI). Une enquête épidémiologique (enquête rétrospective) est alors réalisée dans un quartier bien différencié de cette
commune, afin de mesurer la fréquence de la diarrhée.
Chez les habitants qui ont consommé en Septembre l'eau du robinet.
Chez les habitants qui n'en n'ont pas consommé durant cette période (puits personnel, consommation d'eau minérale...).

Cliquez sur le bouton "Dessin" pour afficher le tableau.

Quel est le risque relatif entre les buveurs d'eau du robinet et les non-buveurs de cette eau ?

Cliquez sur le bouton "dessin" pour afficher le tableau.


A - Environ 2
B - Environ 4
C - Environ 6
D - Environ 8
E - Environ 10
Bonne(s) réponse(s) : E

Risque relatif :
(Taux d'incidence du Pb dans la population exposée / Taux d'incidence du Pb dans la population non exposée)
population exposée : 400 / 530 = 0,75
population non exposée : (6 / 80) = 0,075

Ce résultat signifie :

Cliquez sur le bouton "dessin" pour afficher le tableau.


A - Que l'effectif de la population est trop faible pour conclure
B - Que la fréquence de la diarrhée est à peu près équivalente dans le groupe exposé et le groupe non exposé
C - Que le risque de contracter une diarrhée était fortement augmenté dans le groupe exposé, par rapport au
groupe non exposé
D - Qu'il existe une présomption de relation causale entre la consommation d'eau du robinet et les
manifestations diarrhéiques
E - Que la présence d'E.Coli dans l'eau du robinet est la cause des manifestations diarrhéiques
Bonne(s) réponse(s) : C D

Sans commentaire.

1387
Exclusivement sur DOC - DZ : www.doc-dz.com NADJI 85
RESIDANAT EN POCHE TOME II
Cas Clinique en QCM

L'analyse de l'eau a révélé :


- la présence de 50 E.Coli dans 100 ml d'eau
- un taux de nitrates à 60 mg/l
Il est exact que :

Cliquez sur le bouton "dessin" pour afficher le tableau.


A - Le taux élevé des nitrates est un facteur causal de pollution bactériologique
B - La pollution bactériologique est un facteur causal d'augmentation des nitrates
C - Il n'y a pas de relation causale entre la pollution bactériologique et le taux élevé de nitrates
D - Le taux élevé de nitrates est une explication possible de l'épidémie de diarrhée
E - Le taux élevé de nitrates ne doit pas être incriminé dans la survenue de cette épidémie
Bonne(s) réponse(s) : C E

Sans commentaire.

D'une manière générale, parmi les maladies suivantes, laquelle (lesquelles) peut (peuvent) être transmise(s) par
une eau de boisson polluée ?

Cliquez sur le bouton "dessin" pour afficher le tableau.


A - Hépatite à virus A
B - Botulisme
C - Saturnisme
D - Salmonellose
E - Hépatite à virus B
Bonne(s) réponse(s) : A C D

Evident !

Le schéma ci-dessous représente l'évolution d'une épidémie de tuberculose gravidique dans un foyer de 100 femmes
immigrées célibataires :

Cliquez sur le bouton "Dessin" pour afficher le schéma.

Le taux de prévalence au 1er février est de :

Cliquez sur le bouton "dessin" pour afficher le tableau.


A - 4/6
B - 2/100
C - 3/100
D - 4/100
E - 6/100
Bonne(s) réponse(s) : D

Prévalence :
(Nbre de pers. atteintes par la maladie / Nbre de pers. de la population) à un temps donné.

Le taux d'incidence pour le mois de janvier est de :

Cliquez sur le bouton "dessin" pour afficher le tableau.


A - 3/6
B - 2/100
C - 3/100
D - 4/100
E - 6/100
Bonne(s) réponse(s) : C

Incidence :
( Nbre de pers. nouvellement atteintes / Nbre de pers. de la population) dans une période déterminée.

1388
Exclusivement sur DOC - DZ : www.doc-dz.com NADJI 85
RESIDANAT EN POCHE TOME II
Cas Clinique en QCM

Quelle est la durée moyenne de la maladie ?

Cliquez sur le bouton "dessin" pour afficher le tableau.


A - Environ 1 semaine
B - Environ 1 mois
C - Environ 2 mois
D - Environ 3 mois
E - Impossible à estimer
Bonne(s) réponse(s) : E

Sans commentaire.

Quelle(s) est (sont) la (les) mesure(s) nécessaire(s) vis-à-vis des malades tuberculeux bacillifères dépistés dans
ce foyer ?
Cliquez sur le bouton "dessin" pour afficher le tableau.
A - Isolement
B - Vaccination BCG immédiate
C - Traitement par antibiotiques antituberculeux pendant au moins 9 mois
D - Traitement par antibiotiques antituberculeux pendant 4 mois
E - Déclaration à la Direction Départementale des Affaires Sanitaires et Sociales
Bonne(s) réponse(s) : A C E

Sans commentaire.

Quelle(s) est (sont) la (les) mesure(s) nécessaire(s) vis-à-vis des pensionnaires du foyer dont l'examen
radiologique est normal et dont vous considérez l'intra-dermo-réaction à 10 unités comme négative :
Cliquez sur le bouton "dessin" pour afficher le tableau.
A - Vaccination BCG immédiate
B - Traitement par 2 antibiotiques antituberculeux pendant 6 mois
C - Traitement par 3 antibiotiques antituberculeux pendant 3 mois
D - Nouveau contrôle de l'intra-dermo-réaction dans un mois
E - Isolement
Bonne(s) réponse(s) : A B

B - Ou deux antituberculeux pendant 2 mois puis un antituberculeux pendant 2 mois.

Pour mieux étudier s'il y a une relation entre la contraception par voie orale (oestro-progestatifs) et le cancer du col utérin, un
groupe de 15 000 femmes de 18 à 58 ans, indemnes de cancer du col, a été suivi de décembre 1976 à janvier 1980.
La surveillance a consisté essentiellement en un frottis annuel ; parallèlement la prise de contraceptifs était recherchée par
questionnaire.

Il s'agit d'une enquête :


A - De prévalence
B - Prospective
C - De cohorte
D - D'épidémiologie analytique
E - Cas-témoins
Bonne(s) réponse(s) : B C

Sans commentaire.

1389
Exclusivement sur DOC - DZ : www.doc-dz.com NADJI 85
RESIDANAT EN POCHE TOME II
Cas Clinique en QCM

Sur le tableau suivant, figurent les résultats de l'enquête en fonction de la durée de la prise de contraceptifs :

Cliquez sur le bouton "dessin" pour afficher le tableau.

Avez-vous suffisamment d'éléments pour compléter le tableau et calculer le taux d'incidence du cancer du col
chez les femmes ayant utilisé un contraceptif pendant plus de 4 ans :
A - Non, il faudrait connaître le nombre de femmes qui ont été exposées pendant plus de 4 ans
B - Non, car on ne connaît pas l'incidence chez les non-exposées
C - Non, il faudrait connaître l'incidence du cancer du col dans la population générale
D - Oui : I = 160/100 000
E - Non, il faudrait connaître le nombre de femmes qui n'ont pas été exposées
Bonne(s) réponse(s) : D

Inc. pop. non exposée x Risque relatif pop. exposée = Inc. pop. exposée.
Inexp. = 32/100 000
R1 = 5
I exp = 32 x 5/100 000

Le fait que le risque relatif augmente de 1 à 5 en fonction de l'exposition au risque :


A - Confirme l'hypothèse selon laquelle la pilule est un facteur de risque de cancer du col
B - Prouve que la pilule est la cause du cancer du col
C - Est un argument en faveur de la causalité
D - N'a aucune signification étiologique dans ce type d'enquête
E - N'a rien à voir avec la relation de causalité
Bonne(s) réponse(s) : A C

Sans commentaire.

Comment, à votre avis, a-t-on procédé pour chiffrer le risque relatif ?


A - On a calculé directement le risque relatif par le rapport de 2 incidences
B - On a comparé la prise de contraceptifs chez les femmes atteintes de cancer du col, et des femmes
indemnes
C - On a comparé la fréquence de survenue du cancer du col chez les utilisatrices
D - On a calculé le chi-deux
E - On a utilisé une méthode d'approximation (odds ratio)
Bonne(s) réponse(s) : A

B - Comparaison de la survenue de cancer de col dans les deux groupes (CO et sans CO).

On envisage le dépistage d'une maladie M grâce à un test biologique quantitatif.


La maladie M est d'autant plus probable que la valeur obtenue au test est élevée.

On hésite entre deux seuils au delà desquels le résultat est anormal


A = 30 unités internationales
B = 20 unités internationales
On peut prévoir avec certitude que :
A - Le test A sera plus sensible que le test B
B - Le test A sera plus spécifique que le test B
C - Le nombre de faux positif sera plus grand avec le test A
D - Le nombre de faux négatif sera plus grand avec le test A
E - Le test A dépistera plus de malades que le test B
Bonne(s) réponse(s) : B

Sensibilité : (vrais+) / (vrais+ + faux-).


Spécificité : (vrais-) / ( faux+ + vrais-).
C D - Plus faible

Si l'on choisit une valeur seuil telle que le test a une spécificité de 100%. Cela implique que :
A - Sa valeur prédictive positive est aussi de 100 %
B - Devant un test positif, on peut affirmer que le sujet est malade
C - On prend le risque de laisser échapper beaucoup de malades au dépistage
D - La valeur prédictive négative est de 100%
E - Il n'y a pas de faux négatifs
Bonne(s) réponse(s) : A B C E

Vpp = (vrais positifs) / (vrais positifs + faux positifs).


VpM = (vrais négatifs) / (vrais négatifs + faux négatifs).
Spécificité = (V-) / ( F + FV-)
1390
Exclusivement sur DOC - DZ : www.doc-dz.com NADJI 85
RESIDANAT EN POCHE TOME II
Cas Clinique en QCM

On choisit finalement une valeur seuil telle que la sensibilité est de 95 % et la spécificité de 90%.
Votre première campagne de dépistage s'applique à une population où la prévalence de la maladie est de 2 %.
En une semaine, vous examinez 1 000 personnes. Le(s) nombre(s) attendu(s) :
A - De sujets positifs est de 117
B - De vrais positifs est de 19
C - De vrais négatifs est de 980
D - De faux négatifs est de 98
E - On ne peut prédire sur les seules données de l'énoncé le nombre de vrais et faux négatifs
Bonne(s) réponse(s) : A B

Sensibilité = a / (a + c) = 95 / 100
Spécificité = d / (b + d) = 90 / 100
Et P = 2/1000.
Et P = (nbre de cas / population).

M+ M-
test + a 19 b 98 117
test - c 1 d 882 883
20 980 1000

Votre deuxième campagne de dépistage s'applique à une population où la prévalence de la maladie est plus
élevée :
A - La valeur prédictive positive du test sera meilleure que dans le cas précédent
B - La valeur prédictive négative du test sera meilleure que dans le cas précédent
C - La sensibilité du test sera meilleure que dans le cas précédent
D - La spécificité du test sera meilleure que dans le cas précédent
E - Le rendement du dépistage sera meilleur que dans le cas précédent
Bonne(s) réponse(s) : A E

Si prévalence augmente, a augmente et donc Vp+ = a / (a + b) est meilleure.

Un essai thérapeutique randomisé, en double aveugle, a pour but d'apprécier l'efficacité de l'aspirine, 500 mg/jour, dans la
prévention des récidives d'accidents ischémiques cérébraux liés à l'athérosclérose.
A l'époque où l'essai a été réalisé aucun autre médicament n'avait démontré son efficacité dans cette indication.

Que prescrire au groupe témoin :


A - Un autre antiaggrégant plaquettaire
B - Un placebo
C - Rien
D - L'aspirine à faible dose (50 mg)
E - L'association aspirine 50 mg - autre antiaggrégant plaquettaire
Bonne(s) réponse(s) : B

Sans commentaire.

Quelle attitude adopter vis-à-vis d'un patient présentant un ulcère gastrique ?


A - L'inclure dans l'étude sans tenir compte de l'ulcère
B - L'inclure dans le groupe témoin avant la randomisation
C - L'exclure de l'étude avant la randomisation
D - L'exclure de l'étude si la randomisation le repartit dans le groupe le groupe témoin
E - Le changer de groupe si la randomisation le répartit dans le groupe aspirine
Bonne(s) réponse(s) : A

Le critère ulcère gastrique n'ayant pas été retenu comme critère d'exclusion dans l'énoncé.

100 patients ont été inclus dans chaque groupe. 30 patients étaient hypertendus dans le groupe aspirine 500
mg, 10 patients dans le groupe témoin.
Dans ce cas le risque relatif de faire un accident vasculaire si on est hypertendu est :
A - Egal à 2
B - Egal à 0, 5
C - Egal à 3
D - Calculable si le taux de létalité est commun
E - Incalculable avec le protocole de l'étude
Bonne(s) réponse(s) : E

Le risque relatif est obtenu par le rapport :


taux d'incidence de la pathologie dans la population exposée / taux d'incidence de la pathologie dans la population non
exposée.
1391
Exclusivement sur DOC - DZ : www.doc-dz.com NADJI 85
RESIDANAT EN POCHE TOME II
Cas Clinique en QCM

Dans les deux groupes, quelle attitude adopter vis-à-vis des traitements antihypertenseurs suivis par de
nombreux patients :
A - L'arrêter pour tous les patients
B - Le maintenir uniquement dans le groupe aspirine 500 mg
C - Le maintenir uniquement dans le groupe témoin
D - Le maintenir dans les deux groupes sans le mentionner
E - Le maintenir dans les deux groupes et en tenir compte dans l'analyse
Bonne(s) réponse(s) : E

Sans commentaire.

Chez des enfants considérés à haut risque de rechute de crise convulsive hyperpyrétique, un essai thérapeutique a été
entrepris pour évaluer l'efficacité d'un traitement intermittent par le Nitrazepam, lors des poussées fébriles.
Après avoir donné des explications détaillées aux parents sur la nature relativement bénigne des convulsions fébriles et sur
les traitements possibles, un traitement prophylactique intermittent par le Nitrazepam a été suggéré.
31 parents ont accepté de donner du Nitrazepam ; 24 ont préféré l'abstention thérapeutique.

Cliquez sur le bouton "Dessin" pour afficher le graphique.

Il s'agit d'un essai :

Cliquez sur le bouton "dessin" pour afficher le tableau.


A - Randomisé
B - Comparatif
C - Séquentiel
D - En double aveugle
E - Croisé ("cross over")
Bonne(s) réponse(s) : B

A - Triage au sort des deux groupes.


C - Chaque groupe est son propre témoin.
D - Ni le malade ni le prescripteur ne connaissent la nature de la prescription.

La randomisation, d'un façon générale :

Cliquez sur le bouton "dessin" pour afficher le tableau.


A - Est contre-indiquée chez l'enfant
B - N'est pas applicable au traitement par une benzodiazépine
C - Peut favoriser la survenue de rechutes
D - Permet d'éviter un biais lors du recueil de données
E - Permet l'attribution aléatoire du traitement étudié à chaque sujet
Bonne(s) réponse(s) : E

Par définition.

Dans cet essai thérapeutique, tel qu'il est conçu, p < 0,05 signifie que :

Cliquez sur le bouton "dessin" pour afficher le tableau.


A - Le Nitrazepam est efficace dans la prévention des rechutes
B - Le Nitrazepam n'est pas efficace dans la prévention des rechutes
C - Le risque de rechutes est significativement diminué dans le groupe traité
D - Les rechutes sont fréquentes dans le groupe non traité
E - Les rechutes sont rares dans le groupe traité
Bonne(s) réponse(s) : A

Sans commentaire.

p < 0,05 peut avoir pour explication(s) ?

Cliquez sur le bouton "dessin" pour afficher le tableau.


A - L'existence d'un biais de sélection
B - L'activité du Nitrazepam
C - L'intervention du hasard
D - Le jeune âge
E - Une erreur dans le calcul du chi-deux
Bonne(s) réponse(s) : A B E

C - Le test du X ² élimine le hasard.

1392
Exclusivement sur DOC - DZ : www.doc-dz.com NADJI 85
RESIDANAT EN POCHE TOME II
Cas Clinique en QCM
Vous examinez une femme enceinte (premier examen prénatal à 2 mois de grossesse).

Dans le cadre de la prévention de la prématurité, il est licite de rechercher dans les antécédents de cette femme
:
A - Un accouchement prématuré
B - Une mort périnatale
C - Un avortement tardif
D - Une fausse couche curetée
E - Une césarienne
Bonne(s) réponse(s) : D E

Les causes de prématurité sont :


- le jeune âge
- les facteurs socio-économiques
- multiparité, gémellité, malformations pelviennes et les complications chirurgico-médicales pelviennes.

Cette femme vous demande des renseignements sur la réglementation de la protection prénatale. A propos de
l'allocation du jeune enfant, vous lui répondez qu'elle est versée actuellement sous forme :
A - De mensualités à partir du 4ème mois de grossesse jusqu'au 3ème mois révolu de la vie de l'enfant
B - De mensualités à partir du 6ème mois de grossesse jusqu'au 6ème mois révolu de l'enfant
C - D'allocations prénatales exclusivement
D - D'allocations postnatales exclusivement
E - D'allocations familiales
Bonne(s) réponse(s) : A

E - Peut être étendue au-delà. A partir du 4ème mois de l'enfant et jusqu'à ses 3 ans (en fonction des ressources) : 861
F/mois par enfant (nov.1990).

Cette femme est déjà mère de deux enfants. A propos du congé post-natal, vous lui répondez qu'il ouvre droit à
une indemnité journalière, et que sa durée sera de :
A - 8 semaines
B - 10 semaines
C - 12 semaines
D - 16 semaines
E - 18 semaines
Bonne(s) réponse(s) : E

Sans commentaire.

A propos du remboursement des frais de soins, vous lui répondez que les examens prénataux sont pris en
charge par l'assurance maternité, et que tous les frais de soins en rapport ou non avec la grossesse sont pris en
charge à 100 % par la Sécurité Sociale à partir du :
A - 3ème mois
B - 4ème mois
C - 5ème mois
D - 6ème mois
E - 8ème mois
Bonne(s) réponse(s) : B

Sans commentaire.

Le 1er décembre 1989, dans une petite bourgade d'Indre et Loire de 2000 habitants le Dr M. est appelé auprès de plusieurs
jeunes enfants atteints d'une diarrhée aiguë.
Il s'agit dans tous les cas d'une diarrhée, modérément fébrile, sans gravité.
Mais le caractère épidémique de ces manifestations diarrhéiques amène le Dr M. à soupçonné une éventuelle pollution de
l'eau de distribution publique, (eau traitée par chloration).

Pour connaître immédiatement les résultats des dernières analyses de cette eau, le Dr M. a la possibilité de
téléphoner :
A - Au service des eaux de la D.D.A.S.S. d'Indre et Loire
B - Au bureau municipal d'hygiène de cette commune
C - A la D.R.A.S.S.
D - Au comité de bassin
E - A l'agence financière de bassin
Bonne(s) réponse(s) : B

Sans commentaire.
1393
Exclusivement sur DOC - DZ : www.doc-dz.com NADJI 85
RESIDANAT EN POCHE TOME II
Cas Clinique en QCM

La dernière analyse, effectuée le 15 novembre 1989, a révélé : - la présence de 2 E. Coli dans 100 ml d'eau -
un taux de nitrates à 75 mg/l. Comment interprétez-vous ces résultats ?
A - Bactériologiquement, cette eau n'est pas potable
B - Bactériologiquement, cette eau est potable
C - Le taux de nitrates dépasse la limite maxima admissible qui est 1 mg/l
D - Le taux de nitrates dépasse la limite maxima admissible qui est 50 mg/l
E - Ces résultats suffisent pour expliquer l'épidémie de diarrhée
Bonne(s) réponse(s) : A E

Sans commentaire.

Pour diminuer la teneur croissante des nitrates dans les eaux de consommation, la recommandation suivante
peut être proposée :
A - Diminuer les apports d'engrais chimiques
B - Eviter l'utilisation du DDT
C - Eviter les rejets de détergents dans les eaux de surface
D - Eviter les rejets d'hydrocarbures dans les eaux de surface
E - Diminuer certaines cultures (carottes, épinards)
Bonne(s) réponse(s) : A E

Sans commentaire.

L'évolution de la diarrhée se révèle rapidement favorable pour tous les enfants. Toutes les coprocultures
demandées au début de chaque épisode diarrhéique, reviennent négatives. Parmi les causes suivantes,
laquelle est compatible avec cette épidémie ?
A - Dysenterie bacillaire
B - Infection à rotavirus
C - Salmonellose
D - Excès de nitrates dans l'eau de distribution publique
E - Infection à Klebsielles
Bonne(s) réponse(s) : D

Sans commentaire.

Un nouveau test vient d'être proposé pour le dépistage de la maladie de Lyme.


On procède à une évaluation de ce test et on obtient les données suivantes :

Cliquez sur le bouton "Dessin" pour afficher le graphique.

Quelle est la prévalence de la maladie de Lyme dans cette population ?

Cliquez sur le bouton "dessin" pour afficher le tableau.


A - 0,02
B - 0,06
C - 0,10
D - 0,14
E - 0,18
Bonne(s) réponse(s) : C

P = (Nbre de personnes atteintes / Nbre de personnes de la population).


P = 986 / 9863 = 0,0996, soit environ 0,1.

Quelle est la spécificité du test ?

Cliquez sur le bouton "dessin" pour afficher le tableau.


A - 0,50
B - 0,60
C - 0,70
D - 0,80
E - 0,90
Bonne(s) réponse(s) : E

Spécificité = (V-) / (V- + F+).


Sp = (7989 / 8877).
Sp environ 0,9.

1394
Exclusivement sur DOC - DZ : www.doc-dz.com NADJI 85
RESIDANAT EN POCHE TOME II
Cas Clinique en QCM

Quelle est la sensibilité du test ?

Cliquez sur le bouton "dessin" pour afficher le tableau.


A - 0,50
B - 0,60
C - 0,70
D - 0,80
E - 0,90
Bonne(s) réponse(s) : E

Sensibilité = (V+) / (V+ + F-).


Sens. ( 887 / 986 ) = 0,9.

Quelle est la valeur prédictive positive du test ?

Cliquez sur le bouton "dessin" pour afficher le tableau.


A - 0,50
B - 0,60
C - 0,70
D - 0,80
E - 0,90
Bonne(s) réponse(s) : A

Vp+ = ( V+ ) / ( V+ + F+).
Vp+ = ( 887 / 1775 ) = environ 0,5.

Si on évalue ce test dans une population où la prévalence est deux fois plus élevée :

Cliquez sur le bouton "dessin" pour afficher le tableau.


A - La sensibilité du test sera augmentée
B - La spécificité du test sera diminuée
C - La valeur prédictive positive sera augmentée
D - La valeur prédictive négative sera diminuée
E - Le nombre de faux positif sera inchangé
Bonne(s) réponse(s) : D E

A - Inchangée.
B - Inchangée.
C - Diminuée.

Vous êtes médecin responsable médical d'une pouponnière. Un enfant âgé de 2 ans, hébergé dans cet établissement,
présente une méningite que vous suspectez à méningocoques.

Quelle(s) mesure(s) obligatoire(s) prendrez-vous ou ferez-vous prendre immédiatement ?


A - Hospitalisation de l'enfant
B - Désinfection de tout objet ou linge avec lequel il a été en contact
C - Déclaration obligatoire dès la suspicion de l'infection
D - Surveillance médicale de tous les enfants et de tout le personnel de l'établissement
E - Désinfection des locaux où l'enfant a séjourné
Bonne(s) réponse(s) : C D

QUESTION ANNULEE.
D - Chimioprophylaxie obligatoire de 5 jours.

Le laboratoire confirme que l'agent pathogène est un méningocoque. Quelle(s) est (sont) la (les) mesure(s) à
prendre dans la pouponnière ?
A - Rechercher les porteurs de méningocoques par prélèvements pharyngés chez les enfants et le personnel
B - Désinfection rhino-pharyngée des porteurs de germes
C - Antibioprophylaxie chez tous les enfants et le personnel de l'établissement
D - Séroprophylaxie spécifique associée à une antibioprophylaxie chez les enfants de moins de 6 mois
E - Interdiction d'admission de nouveaux enfants
Bonne(s) réponse(s) : C E

QUESTION ANNULEE.
E - Mesure préventive raisonnable mais non obligatoire.

1395
Exclusivement sur DOC - DZ : www.doc-dz.com NADJI 85
RESIDANAT EN POCHE TOME II
Cas Clinique en QCM

Le méningocoque en cause est identifié comme étant un sérotype C. Vous entreprenez une prophylaxie par
vaccination. A quel(s) groupe(s) allez-vous appliquer cette vaccination ?
A - Tout le personnel de l'établissement
B - Tous les enfants de l'établissement
C - Aux enfants âgés d'au moins 1 an
D - Aux enfants âgés d'au moins 3 mois
E - Tous les enfants sauf ceux présentant un déficit immunitaire
Bonne(s) réponse(s) : A B

"Pour tous les sujets au contact"...Décret du 10 juin 1986.

A quel moment, l'enfant qui a été hospitalisé, pourra-t-il réintégrer la pouponnière ?


A - 20 jours après la guérison
B - Dès la guérison
C - 30 jours après la guérison
D - Dès que les prélèvements pharyngés sont négatifs
E - Après guérison suivie d'une antibioprophylaxie de portage
Bonne(s) réponse(s) : B

Sans commentaire.

Quelle est la proportion approximative du sérotype C parmi les méningocoques isolés actuellement en France
(1989) ?
A - 50 %
B - 80 %
C - 10 %
D - 30 %
E - 20 %
Bonne(s) réponse(s) : E

E - 18 %.

1396
Exclusivement sur DOC - DZ : www.doc-dz.com NADJI 85
RESIDANAT EN POCHE TOME II
Cas Clinique en QCM

1397
Exclusivement sur DOC - DZ : www.doc-dz.com NADJI 85
RESIDANAT EN POCHE TOME II
Cas Clinique en QCM
Une femme de 40 ans, non ménopausée, présente une tuméfaction du sein gauche, connue depuis plusieurs mois. A
l'examen clinique, cette tuméfaction fait 4 cm de diamètre, elle n'est adhérente ni à la peau, ni aux plans profonds. La
palpation des aires ganglionnaires retrouve deux ganglions axillaires gauches mobiles, non inflammatoires, indolores,
suspects. Enfin, le bilan paraclinique d'extension s'avère négatif. Une drill-biopsie confirme qu'il s'agit d'un épithélioma
glandulaire.

Comment classez-vous cette tumeur du sein dans la classification T.N.M. ?


A - T2 N1b M0
B - T3 N2 M0
C - T4 N1b M0
D - T2 N1b M1
E - T4 N2 M0
Bonne(s) réponse(s) :A

Classification TNM :
- T1 : < 2 cm, T2 : 2 à 5 cm, T3 : 5-10 cm, T4 : > 10 cm ou envahissement cutané.
- N0 : pas de ganglion palpable.
- N1 : ganglion homolatéral mobile.
a : non suspect
b : suspect
- N2 : ganglion homolatéral fixé.
- N3 : ganglion sus ou retro claviculaire homolatéral, ou oedème du bras.

Parmi les éléments suivants, on associe habituellement au risque de cancer du sein :


A - Tabagisme
B - Un seul enfant à 32 ans
C - Accouchement difficile
D - Allaitement
E - Mère ayant été traitée pour un cancer du sein
Bonne(s) réponse(s) : B E

Voir les facteurs de risques de cancer du sein.

La malade est traitée par chirurgie (mammectomie totale avec curage axillaire). L'analyse histologique et
biochimique de la pièce opératoire fournit des éléments parmi lesquels vous retenez comme facteur(s) de
mauvais pronostic :
A - Envahissement métastatique de 4 ganglions sur 15
B - Tumeur peu différenciée, de grade histologique Scarf et Bloom "3"
C - Pas d'envahissement mammelonnaire
D - Absence de récepteurs hormonaux
E - Fibroadénome associé
Bonne(s) réponse(s) : A B D

Plus de trois ganglions envahis est considéré comme de mauvais pronostic - La classification histopronostique de Scarff et
Bloom comporte 3 grades :
I: Bien différencié.
II : Moyennement différencié.
III : Indifférencié, de mauvais pronostic.
L'absence de récepteurs hormonaux rend peu probable la réponse à l'hormonothérapie.
Autres facteurs de mauvais pronostics :
- T > 5 cm
- sein inflammatoire
- métastase.

Compte-tenu des résultats histologiques et biochimiques de la pièce opératoire, quel schéma thérapeutique
logique vous paraît le mieux adapté parmi ceux proposé après la mammectomie :
A - Surveillance régulière seule
B - Irradiation loco-régionale seule
C - Hormonothérapie adjuvante seule
D - Irradiation loco-régionale + hormonothérapie adjuvante
E - Irradiation loco-régionale + chimiothérapie adjuvante
Bonne(s) réponse(s) : E

L'irradiation s'impose après chirurgie car elle assure un meilleur contrôle loco-régional que la chirurgie seule (récidives locales
moins nombreuses). Les ganglions étant envahis, la radiothérapie devra concerner, outre la paroi thoracique régionale du
sein, les aires ganglionnaires axillaires, sus-claviculaires et mammaires internes homolatérales.
La chimiothérapie vise à traiter les métastases infracliniques probables dans ce contexte (N+, SBR III).

1398
Exclusivement sur DOC - DZ : www.doc-dz.com NADJI 85
RESIDANAT EN POCHE TOME II
Cas Clinique en QCM

Au cours de la surveillance, pré, per et post thérapeutique de cet épithélioma glandulaire du sein, quel marqueur
biologique peut être régulièrement demandé pour suivre l'évolution tumorale, parmi la liste suivante ?
A - Alpha foeto-protéine
B - ACE (antigène carcino embryonnaire)
C - Bêta HCG
D - FSH
E - Phosphatases acides
Bonne(s) réponse(s) : B

S'il était élevé initialement, surtout.

Un malade de 78 ans consulte son médecin de façon systématique tous les 6 mois ; il a en effet été opéré 8 ans auparavant
d'une lobectomie inférieure droite pour cancer épidermoïde du poumon ; les bilans cliniques ne montrent aucun signe de
récidive. Les clichés pulmonaires montrent une image thoracique droite parfaitement stable. Numération et VS sont normales.
Par contre, l'examen découvre une masse abdominale para-ombilicale droite, bien limitée vers le haut (n'atteignant pas
l'auvent costal), battante, expansive, et siège d'un souffle systolique. Les pouls d'aval (fémoraux, poplités et distaux) sont
normalement perçus des 2 côtés. L'examen retrouve par ailleurs un souffle systolique (3/10) en regard de l'artère carotide
gauche : la TA est à 16/10 cmHg. Le reste de l'examen est normal. Le malade est en excellent état physiologique.

Quel diagnostic portez-vous sur la seule clinique ?


A - Métastase hépatique du cancer bronchique
B - Anévrisme de l'artère iliaque droite
C - Anévrisme de l'aorte abdominale
D - Compression de l'aorte par une masse abdominale métastatique
E - Dolicho-méga-artère du sujet âgé
Bonne(s) réponse(s) : C

Masse battante, soufflante, expansive, para ombilicale = anévrisme de l'aorte abdominale.


D - Non, car la masse est expansive.
A - Piège grossier.

Dans le doute diagnostique, quel examen complémentaire envisagez-vous en première intention ?


A - Examen doppler des membres inférieurs
B - Echographie abdominale
C - Artériographie de l'aorte et des membres inférieurs par voie fémorale rétrograde
D - Scanner du thorax
E - Aortographie par ponction directe de l'aorte
Bonne(s) réponse(s) : B

Examen rapide, sensible, permettant d'apprécier la thrombose associée.

Dans le bilan, lequel des examens suivants ne vous semble pas indispensable ?
A - Examen doppler des vaisseaux du cou
B - Examen doppler des membres inférieurs
C - Radiographie thoracique
D - Electrocardiogramme
E - Echographie abdominale
Bonne(s) réponse(s) : B

N'a pas d'intérêt, car cliniquement il n'existe aucune anomalie.

Parmi les complications suivantes, Iaquelle est la plus à redouter ici ?


A - Embolie des artères de jambe
B - Hémorragie rétro-péritonéale
C - Péritonite
D - Septicémie
E - Thrombose aortique
Bonne(s) réponse(s) : B

Par rupture d'anévrisme. L'embolie artérielle est également redoutable. La thrombose est quasi toujours associée.

1399
Exclusivement sur DOC - DZ : www.doc-dz.com NADJI 85
RESIDANAT EN POCHE TOME II
Cas Clinique en QCM

Quelle est l'attitude thérapeutique la plus appropriée ?


A - Surveillance clinique et radiologique tous les 6 mois
B - Intervention chirurgicale de principe
C - Mise sous traitement anticoagulant
D - Mise sous traitement anticoagulant et intervention dans 3 mois
E - Mise sous traitement vasodilatateur et antiaggrégant plaquettaire
Bonne(s) réponse(s) : B

D'indication formelle. La rupture menace en permanence.

Monsieur CAM.., 55 ans, ouvrier agricole, consulte pour une masse latérocervicale droite apparue il y a trois mois. Pas de
douleur, mais il signale une anorexie et une asthénie. Il fume plus d'un paquet de cigarettes par jour et avoue boire 2 litres de
vin par jour. Il se plaint de dysphagie sans troubles respiratoires. Antécédents : bronchite chronique non traitée. L'examen
montre :
- Une pharyngite érythémateuse, un état buccodentaire très altéré, et une stase salivaire qui empêche la visualisation de
l'hypopharynx. Par contre, l'examen du larynx révèle une diminution de la mobilité de l'hémilarynx droit, larynx et cordes
vocales rouges.
- Une masse de 4 cm de diamètre indolore au bord antérieur du sterno-cléïdomastoïdien, près de son insertion supérieure,
non battante, sans caractère inflammatoire, bien mobile, et sans modification de la peau en regard.
- Apyrexie
- Le foie déborde de 4 cm
Les examens complémentaires :
- Numération-Formule : pas d'anémie ; hyperleucocytose : 15 000 blancs ; 70 % de polynucléaires
- Vitesse de sédimentation : 1ère heure : 60 mm - 2ème heure : 120 mm
- Macrocytose à 120 microns3
Vous évoquez le diagnostic d'un carcinome des voies aérodigestives supérieures.

Le (les) signe(s) suivant(s) va (vont) vous orienter directement vers le diagnostic :


A - L'accroissement de la vitesse de sédimentation
B - La pharyngolaryngite
C - La stase salivaire
D - L'immobilité laryngée
E - La dysphagie
Bonne(s) réponse(s) : C D E

La stase salivaire et la dysphagie évoquent une obstruction pharyngée, qui, sur ce terrain alcoolo-tabagique fait penser à un
cancer des voies aérodigestives supérieures.
L'immobilité laryngée est fortement suspecte d'envahissement néoplasique dans ce contexte.

Sur les arguments cliniques seuls, quel premier diagnostic évoquez-vous ?


A - Un cancer thyroïdien
B - Un cancer de la base de langue
C - Un cancer de l'hypopharynx
D - Un cancer de l'amygdale palatine
E - Un cancer de la vallécule
Bonne(s) réponse(s) : C

La dysphagie sans trouble respiratoire, avec stase salivaire, et avec une adénopathie cervicale évoquent une origine basse,
hypopharyngée (sinus piriforme).

Parmi les raisons suivantes, quelle (s) est(sont) celle(s) qui pourrai(en)t vous interdire la chirurgie :
A - L'altération trop importante de l'état général
B - Une tumeur de plus d'1 cm de diamètre
C - Une métastase pleuro-pulmonaire
D - Une adénopathie bilatérale
E - Une adénopathie de plus de 5 cm de diamètre
Bonne(s) réponse(s) : A C

L'altération de l'état général peut-être une contre-indication à l'anesthésie générale. La "préparation" à la chirurgie nécessite la
renutrition du malade. L'existence d'une métastase oriente le traitement vers une attitude palliative. Le "contrôle local" n'a ici
plus d'intérêt.
La chirurgie associe une laryngectomie totale et une hypopharyngectomie unilatérale (dans les tumeurs classées T3).
L'évidement ganglionnaire est bilatéral.
La radiothérapie est toujours associée, nécessitant des doses de l'ordre de 65 gys.

1400
Exclusivement sur DOC - DZ : www.doc-dz.com NADJI 85
RESIDANAT EN POCHE TOME II
Cas Clinique en QCM

Parmi les explorations ci-dessous, quelle est celle qui permet d'affirmer le diagnostic ?
A - Transit pharyngo-laryngé lipiodolé
B - Endoscopie et biopsie des voies aérodigestives supérieures
C - Tomographies du larynx
D - Tomodensitométrie cervicale
E - Scintigraphie thyroïdienne
Bonne(s) réponse(s) : B

La certitude ne peut-être qu'histologique (biopsie).

Une femme de 50 ans consulte pour des métrorragies provoquées. Le toucher vaginal et l'examen au spéculum montre une
lésion bourgeonnante et hémorragique du col utérin de 3 cm de diamètre. Le vagin paraît indemne de toute lésion. L'examen
de l'abdomen est normal : il n'y a pas d'adénopathie périphérique. Il n'existe pas de signes urinaires ou digestifs.
La biopsie cervico-utérine confirme le diagnostic de carcinome épidermoïde bien différencié.

Le toucher rectal sous anesthésie générale permettra d'apprécier le(les) élément(s) suivant(s) :
A - L'extension ganglionnaire pelvienne
B - L'état de la cloison recto-vaginale
C - L'atteinte de la cavité utérine
D - L'extension paramétriale
E - La fixation aux parois pelviennes
Bonne(s) réponse(s) : B D E

L'examen doit être fait sous anesthésie générale :


- pour obtenir une bonne décontractation
- pour faciliter l'examen par plusieurs médecins.
L'état de la cloison rectovaginale est appréciée par le toucher bimanuel vaginal et rectal.

Le bilan d'extension rend indispensable la demande de :


A - Dosage de l'antigène carcino embryonnaire
B - Lymphographie
C - Urographie intra-veineuse
D - Echographie pelvienne
E - Cystocopie
Bonne(s) réponse(s) : B C E

Le carcinome épidermoïde du col n'a pas de marqueur connu.


Le bilan systématique pour tout stade comprend :
- lymphographie
- UIV
- cystoscopie.
L'UIV recherche une compression urétérale et son retentissement sur le haut appareil
La Iymphographie recherche un envahissement ganglionnaire pelvien dont le premier relai est iliaque externe (ganglion de
Levoeuf et Godart).

La tumeur est classée T1. Quel traitement retenez-vous parmi les propositions suivantes ?
A - Radiothérapie transcutanée exclusive
B - Colpectomie simple avec curage ganglionnaire
C - Curiethérapie suivie d'une colpohystérectomie avec Iymphadénectomie
D - Curiethérapie simple
E - Hystérectomie simple
Bonne(s) réponse(s) : C

C'est le traitement classique.


La curicthérapie utérovaginale vise à obtenir la stérilisation tumorale. Le vagin est irradié pour diminuer le taux de récidives à
son niveau. La colpohystérectomie a lieu 6 semaines plus tard. Elle sera suivie de radiothérapie externe en cas
d'envahissement ganglionnaire histologique.

1401
Exclusivement sur DOC - DZ : www.doc-dz.com NADJI 85
RESIDANAT EN POCHE TOME II
Cas Clinique en QCM

Cette malade a 90 % de chances environ d'être guérie à 5 ans. Dans le cas contraire, quel est le mode de
récidive le plus probable ?
A - Métastases pulmonaires
B - Récidive pelvienne
C - Métastases osseuses
D - Métastases hépatiques
E - Métastases cérébrales
Bonne(s) réponse(s) : B

Le cancer du col utérin est une maladie à évolution principalement loco-régionale. Les métastases sont rares et tardives (en
dehors des métastases ganglionnaires pelviennes). La récidive survient souvent sur cicatrice vaginale de l'intervention
chirurgicale.

Une malade de 45 ans vient consulter pour une tumeur du quadrant supéro-externe du sein droit découverte 10 jours
auparavant et stable depuis. Cette tumeur indolente de 2,5 cm de diamètre, située à 4 cm en dehors d'un mammelon sain, est
dure, irrégulière, mal limitée, sans adhérence cutanée ni profonde, est unique et l'on perçoit dans le creux axillaire droit 2
petits ganglions souples, élastiques, indolents, parfaitement mobiles, que l'on palpe avec les mêmes caractères dans le creux
axillaire gauche. Sur la mammographie, on constate une opacité de 2 cm de diamètre à bords irréguliers spiculés présentant
en son centre 3 micro-calcifications.

En dehors du cancer du sein, Iaquelle de ces autres affections mammaires aurait pu être envisagée ?
A - Kyste
B - Paget
C - Adénofibrome
D - Tumeur phylloïde
E - Aucune des propositions ci-dessus
Bonne(s) réponse(s) : E

L'âge, la clinique, la mammographie sont fortement évocateurs, nécessitant un bilan à la recherche du cancer avant tout.

Parmi ces examens complémentaires, lequel(lesquels) peut(peuvent) affirmer avec certitude le diagnostic de
cancer du sein.?
A - Echographie
B - Examen histologique de la tumeur
C - Examen histologique d'un ganglion
D - Thermographie
E - Dosage des récepteurs hormonaux
Bonne(s) réponse(s) : B

Seule l'histologie est affirmative, sur la tumeur primitive.

Parmi les données suivantes, quelle(s) est(sont) la(les) plus évocatrice(s) de cancer du sein ?
A - Tumeur indolente
B - Tumeur irrégulière, dure, mal limitée
C - Calcifications à la mammographie
D - Adénopathies souples, élastiques, indolentes
E - Apparition récente
Bonne(s) réponse(s) : B

Valable pour toute tumeur maligne. Les calcifications se voient dans les tumeurs bénignes (macrocalcifications), les
adénopathies métastatiques sont typiquement petites, dures, indolores.

Parmi les examens suivants, le(s) quel(s) est (sont) inutile(s) pour apprécier l'extension de ce cancer ?
A - Echographie hépatique
B - Radio pulmonaire
C - Dosage des récepteurs hormonaux
D - Scintigraphie osseuse
E - Frottis vaginaux
Bonne(s) réponse(s) : C E

Les récepteurs ont un intérêt pronostique et thérapeutique (sensibilité à l'hormonothérapie). Les frottis seraient réalisés, mais
à titre systématique, pour le dépistage du cancer du col utérin, sans rapport avec l'extension de la tumeur du sein.

1402
Exclusivement sur DOC - DZ : www.doc-dz.com NADJI 85
RESIDANAT EN POCHE TOME II
Cas Clinique en QCM

A ce stade du bilan, ce cancer du sein peut être classé :


A - T1 N0
B - T2 N1a
C - T2 N1b
D - T2 N0
E - T3 N1b
Bonne(s) réponse(s) : B

Voir la classification TNM.


T2 : tumeur entre 2 et 5 cm.
N1 a : car on palpe des adénopathies, mais leur aspect clinique est bénin.

Pour ce cancer du sein un traitement conservateur peut être proposé. Il peut être assuré par :
A - Hormonothérapie
B - Radiothérapie seule
C - Chimiothérapie seule
D - Tumorectomie + curage ganglionnaire axillaire + irradiation
E - Tumorectomie seule
Bonne(s) réponse(s) : D

La tumorectomie est généralement pratiquée pour les tumeurs de moins de 3 cm. Le curage axillaire est systématique. La
radiothérapie concerne le sein et la paroi thoracique sous mammaire systématiquement si on trouve un envahissement
ganglionnaire histologique, il faut irradier également : aires ganglionnaires axillaires, sus-claviculaires, mammaire interne
homolatérale.

Une femme de 45 ans non ménopausée consulte en urgence en raison de l'apparition récente d'un état inflammatoire des
quadrants externes du sein droit. A l'examen, la peau de cette région du sein est rouge, chaude, douloureuse : en regard on
retrouve une masse dure mesurant 7 cm dans son plus grand axe, adhérant à la peau, il y a dans l'aisselle homolatérale un
ganglion de 3 cm de diamètre, dur, encore mobile. Le reste de l'examen clinique est normal. La mammographie objective une
opacité spiculée avec quelques microcalcifications et surtout un très important oedème sous cutané ; l'histologie confirme le
diagnostic d'adénocarcinome mammaire devant cette tumeur classée T3 N1b PeV2.

Afin de vérifier l'absence de métastases viscérales et avant de débuter le traitement, lequel(lesquels) des
examens complémentaires suivants devez-vous demander ?
A - Examen tomodensitométrique abdomino-pelvien
B - Scintigraphie osseuse
C - Radiographie du thorax
D - Echographie hépatique
E - Antigène carcino-embryonnaire
Bonne(s) réponse(s) : B C D E

Le cancer du sein est très ostéophile. Les métastases hépatiques et thoraciques sont également fréquentes. L'ACE a un
intérêt dans la surveillance post-thérapeutique.

Chez cette malade, lequel des éléments cliniques suivants vous paraît être le facteur pronostique
majeur ?
A - La taille de la tumeur
B - L'âge
C - La présence d'une adénopathie
D - L'existence de signes inflammatoires
E - L'état hormonal
Bonne(s) réponse(s) : D

Les signes inflammatoires témoignent d'une évolutivité importante, et impose la chimiothérapie première, avant toute chirurgie.

Le traitement initial comporte une polychimiothérapie. Laquelle(lesquelles) des drogues suivantes peu(ven)t y
figurer ?
A - Actinomycine D
B - Cyclophosphamide
C - CDDP
D - D.T.I.C.
E - Adriamycine
Bonne(s) réponse(s) : B E

Les chimiothérapies les plus utilisées dans le cancer du sein sont :


- cyclophosphamide, adriamycine, 5 fluoro-uracile, méthotrexate.

1403
Exclusivement sur DOC - DZ : www.doc-dz.com NADJI 85
RESIDANAT EN POCHE TOME II
Cas Clinique en QCM

Au terme de 6 cures de chimiothérapie, tout signe inflammatoire a disparu, la tumeur mesure encore 5 cm et
l'adénopathie a disparu. Comment envisagez-vous à ce stade la poursuite du traitement ?
A - Hormonothérapie
B - Chimiothérapie d'entretien
C - Association radio-chirurgicale
D - Mammectomie curage
E - Tumorectomie
Bonne(s) réponse(s) : C

La chirurgie et la radiothérapie sont systématiques. Dans ce cas, il faut opérer dès que les signes inflammatoires ont disparu.
Elle associera mammectomie et curage axillaire homolatéral. La radiothérapie suit les modalités habituelles.

A distance du traitement, laquelle des évolutions suivantes vous paraît statistiquement la plus probable ?
A - Récidive pariétale
B - Récidive ganglionnaire axillo-sus-claviculaire
C - Métastases viscérales
D - Leucémie induite
E - Cancer du sein contro-latéral
Bonne(s) réponse(s) : C

L'association radio chirurgicale assure un bon contrôle loco-régional. Le cancer du sein doit être considéré comme une
maladie générale, étant donné la fréquence des micrométastases au moment du diagnostic. Dans le cas présent, le tableau
initial rend très probable l'existence de micrométastases initiales.

Un homme de 40 ans présente un amaigrissement important de 3 kg en 1 mois avec toux et expectorations, crachats
hémoptoïques. La radiographie pulmonaire permet de mettre en évidence une opacité médiastino-hilaire droite. La cytologie
dans l'expectoration confirme qu'il s'agit d'un cancer anaplasique à petites cellules. Une chimiothérapie est mise en route
comprenant une association de Cisplatinum-Adriamycine-Bléomycine une fois par mois.

Parmi les signes suivants, un seul ne témoigne pas de l'envahissement médiastinal :


A - Dysphonie
B - Dysphagie
C - Voix rauque
D - Hoquet
E - Myosis
Bonne(s) réponse(s) : E

N'a rien à voir. Il se voit dans le cancer de l'apex, dans le cadre d'un syndrome de Pancoast Tobias :
- lyse de la 1ère côte
- névralgie C8 D1
- claude Bernard Horner.
L'envahissement médiastinal est très fréquent au cours du cancer anaplasique à petites cellules.

Parmi les signes suivants, un seul ne se trouve pas dans le syndrome cave supérieur :
A - OEdème des membres inférieurs
B - OEdème en pélerine
C - Turgescence des jugulaires
D - Yeux injectés
E - Circulation collatérale thoracique
Bonne(s) réponse(s) : A

Evident. Le syndrome cave supérieur est une indication du traitement d'urgence répondant parfaitement à la chimiothérapie
avec corticothérapie.

Parmi les examens suivants, un seul est inutile pour apprécier l'envahissement régional :
A - Transit oesophagien
B - Lymphographie
C - Scannographie thoracique
D - Radioscopie thoracique
E - Examen des cordes vocales
Bonne(s) réponse(s) : B

Inutile, car parfaitement inexistant au niveau médiastinal.


A C D - Sont systématiques.
E - Permettra de voir l'immobilité de la corde vocale (par paralysie récurrentielle)

1404
Exclusivement sur DOC - DZ : www.doc-dz.com NADJI 85
RESIDANAT EN POCHE TOME II
Cas Clinique en QCM

Dans ce cancer anaplasique à petites cellules, la métastase la plus fréquemment rencontrée est :
A - Osseuse
B - Rénale
C - Cutanée
D - Oculaire
E - Pancréatique
Bonne(s) réponse(s) : A

Justifie une scintigraphie osseuse.


Les métastases cérébrales sont également fréquentes, justifiant un scanner cérébral lors du bilan d'extension.

Parmi les métastases suivantes, quelle est celle qui sera la moins atteinte par cette chimiothérapie
(Cisplatinum®-Adriamycine®-Bléomycine®) ?
A - Osseuse
B - Rénale
C - Cérébrale
D - Hépatique
E - Cutanée
Bonne(s) réponse(s) : C

Du fait de la barrière hémato-encéphalique. Les patients en rémission complète après la chimiothérapie peuvent bénéficier
d'une radiothérapie prophylactique sur l'encéphale (30 Gys en 10 séances).

Parmi les formes histologiques suivantes, quelle est celle qui a le plus mauvais pronostic à stade égal ?
A - Epidermoïde
B - Adénocarcinome
C - Anaplasique à grandes cellules
D - Anaplasique à petites cellules
E - Carcinoïde
Bonne(s) réponse(s) : D

Le taux de survie à 2 ans est de 10 à 20 % dans les formes localisées, la plus faible parmi les cancers bronchiques, et ce,
malgré la chimiosensibilité et la radiosensibilité.

Les cancers anaplasiques à petites cellules traités par chimiothérapie ont un espoir de survie à 2 ans de l'ordre
de :
A - Supérieur à 80 %
B - 70 %
C - 50 %
D - 20 %
E - Inférieur à 5 %
Bonne(s) réponse(s) : E

Les formes disséminées ont une médiane de survie de huit mois environ.

Une femme de 55 ans, non ménopausée consulte pour un nodule du quadrant supéro-externe du sein droit apparu 2 mois
auparavant. L'interrogatoire retrouve la notion d'un cancer du sein chez la grand-mère maternelle, une puberté à l'âge de 10
ans, une première grossesse à l'âge de 32 ans sans allaitement en raison d'un abcès du sein et enfin une cholécystectomie
pour lithiase à 49 ans. Cette malade a régulièrement fumé 1/2 paquet de cigarettes par jour pendant 30 ans.
A l'examen : la tumeur est mobile mais s'accompagne d'une attraction cutanée bien visible à jour frisant ; elle mesure 2,5 cm
dans son plus grand diamètre. A la palpation de l'aisselle homolatérale, on retrouve un ganglion mobile et banal de moins de 1
cm de diamètre ; les autres territoires ganglionnaires sont normaux : il n'existe pas d'hépatomégalie, il n'y a pas de douleurs
osseuses et l'état général est conservé.

Chez cette malade, parmi les antécédents suivants, on retient comme facteur(s) de risque reconnu pour le
cancer du sein ?
A - Puberté précoce-ménopause tardive
B - Antécédent d'abcès du sein
C - Première grossesse tardive
D - Tabagisme
E - Cholécystectomie
Bonne(s) réponse(s) : A C

Les facteurs de risque entrent tous dans le cadre des hyperoestrogénies relatives ou absolues
B - Est faux : ce sont les mastopathies bénignes qui sont un facteur de risque.

1405
Exclusivement sur DOC - DZ : www.doc-dz.com NADJI 85
RESIDANAT EN POCHE TOME II
Cas Clinique en QCM

Dans la classification TNM (UICC - 1978), comment classez-vous ce cancer ?


A - T1 N1 b
B - T2 N1 a
C - T2 N1 b
D - T4 N1 a
E - T4 N1 b
Bonne(s) réponse(s) : B

T1 : inférieur à 2 cm.
T2 : 2 à 5 cm.
T3 : plus de 5 cm.
T4 : extension cutanée ou de la paroi thoracique, (quelle que soit la taille).
N1 : car c'est un ganglion axillaire homolatéral.
N1 a : car il n'est pas suspect d'envahissement (peut-être dû à une lésion, même minime du membre supérieur droit, ou être
simplement réactionnel, sans cellules néoplasiques).

Lequel des examens complémentaires est indispensable pour affirmer la malignité du nodule et autoriser la
mise en route du traitement spécifique ?
A - L'examen cytologique par ponction
B - La mammographie
C - Le dosage de l'antigène carcino-embryonnaire
D - L'examen histopathologique par biopsie
E - La thermographie
Bonne(s) réponse(s) : D

A - Peu sensible.
B E - Ne font qu'orienter le diagnostic.
C - Intérêt dans la surveillance. Non spécifique du cancer du sein. Doit être dosé avant le traitement, puis pendant et après s'il
était élevé initialement.
D - Est le seul examen permettant un diagnostic formel, d'histologie précise.

Compte-tenu des caractères de la tumeur, laquelle des attitudes thérapeutiques suivantes permet
statistiquement d'obtenir un contrôle local satisfaisant avec des séquelles minimes ?
A - Intervention de Patey
B - Tumorectomie et curage limité de l'aisselle suivie d'irradiation
C - Mammectomie simple suivie d'irradiation
D - Irradiation exclusive
E - Intervention de Halsted
Bonne(s) réponse(s) : B

C'est le traitement classique des formes limitées non centrales. Le curage axillaire homolatéral permet d'apprécier l'extension
ganglionnaire, conditionnant la suite du traitement. Un curage limité entraîne peu de risque de Iymphoedème du bras.
L'irradiation est systématique après chirurgie. Elle entraîne un meilleur contrôle loco-régional donc moins de récidives, bien
que n'améliorant pas la survie.
(Si les ganglions sont indemnes, dans un stade limité, l'irradiation comprend le sein et la paroi thoracique. En cas
d'envahissement ganglionnaire, on irradie en plus les aires ganglionnaires, axillaires, sus-claviculaires et mammaires internes
homolatérales).

Parmi les facteurs suivants, lequel (lesquels) vous inciterai(ent) à proposer une chimiothérapie
adjuvante ?
A - Grade II de l'index de Scarff-Bloom et Richardson
B - Adhérence cutanée de la tumeur
C - Positivité des récepteurs hormonaux tumoraux
D - Atteinte ganglionnaire histologique
E - Antécédents familiaux de cancer du sein
Bonne(s) réponse(s) : D

D - Est admis par tous, bien que discuté en fonction du nombre de ganglions atteints.
A - Discutable car les avis divergent amplement.
C - Prescription de tamoxifene.
Cependant, la chimiothérapie du cancer du sein est extrêmement controversée dans ses indications.
Dans les stades localisés (T1 T2), l'indication de chimiothérapie dépend :
- du grade histopronostic de Scarff et Bloom (grade III ?)
- du nombre de ganglions envahis (supérieur à 3 ?).

1406
Exclusivement sur DOC - DZ : www.doc-dz.com NADJI 85
RESIDANAT EN POCHE TOME II
Cas Clinique en QCM
Une femme de 45 ans consulte pour la découverte récente d'une masse dans son sein droit. A l'examen, la tuméfaction siège
à la jonction des deux quadrants externes et mesure 6 cm de diamètre ; la peau en regard est envahie. La palpation retrouve
plusieurs adénopathies axillaires homolatérales dures et mobiles ainsi qu'une adénopathie sus-claviculaire droite de 2 cm de
diamètre. Du côté gauche, l'examen est normal. Il n'existe pas d'hépatomégalie. L'état général est conservé. La
mammographie montre une opacité irrégulière mal limitée avec un épaississement cutané en regard. Il n'y a pas de
microcalcifications. L'examen histologique du prélèvement obtenu par drill-biopsie est en faveur d'un adénocarcinome de
grade histopronostique III et la recherche de récepteurs à la progestérone est négative. La radiographie de thorax est normale.

Quelle et la classification TNM de cette tumeur ?


A - T3 N1
B - T3 N2
C - T3 N3
D - T4 N2
E - T4 N3
Bonne(s) réponse(s) : E

T4 : envahissement cutané ou de la paroi thoracique (sauf le grand pectoral et son aponévrose) quelle que soit la taille de la
tumeur.
N3 : adénopathie sus ou rétro-claviculaire, ou oedème du bras. La détermination est clinique (non histologique).

Lequel (lesquels) des examens suivants doit (doivent) être demandé(s) dans le cadre du bilan d'extension ?
A - Echographie cardiaque
B - Tomodensitométrie cérébrale
C - Tomodensitométrie thoracique
D - Echographie hépatique
E - Scintigraphie osseuse
Bonne(s) réponse(s) : D E

A - Concerne le bilan pré-thérapeutique si une chimiothérapie comprenant des anthracyclines est entreprise (mesure du
coefficient d'éjection systolique). Elle est parfois remplacée par une ventriculographie isotopique voire, plus récemment par la
biopsie endomyocardique.
C - Une radiographie thoracique face et profil est suffisante.
B - En cas de signe d'appel.
D E - Systématiquement, recherche de métastases infracliniques, une scintigraphie positive sera complétée par des
radiographies localisées.

Une polychimiothérapie première associant adriamycine, cyclophosphamide, et fluoro-uracile est instituée.


Quels sont les 3 principaux effets secondaires :
A - Polynévrite
B - Alopécie
C - Nausée et vomissements
D - Surdité
E - Hypoplasie médullaire
Bonne(s) réponse(s) : B C E

A - Surtout avec la Vincristine®.


B - Adriamycine® - Endoxan®.
C - Toute chimiothérapie.
D - Cisplatinum®
E - Adriamycine® Endoxan®.
Ne pas oublier le risque de cystite hémorragique avec le cyclophosphamide, et la cardiotoxicité de l'adriamycine.

Lequel (lesquels) des éléments suivants influence (nt) le pronostic ?


A - Siège externe de la tumeur
B - Négativité des récepteurs à la progestérone
C - Grade histopronostique III
D - Absence de microcalcifications
E - Présence d'une adénopathie sus-claviculaire
Bonne(s) réponse(s) : B C E

B - Le cancer du sein est réputé hormonosensible. L'absence de récepteur privant de l'intérêt de l'hormonothérapie.
C - Classification histopronostique de Scarff et Bloom. Le grade III (noté SBR III) correspondant à un type indifférencié de
mauvais pronostic. Les critères sont cytologiques.
SBR I : bien différencié ; SBR II : moyennement différencié
E - Mauvais pronostic car : début de dissémination, avec risque accru de micrométastases au moment du diagnostic ; de plus,
le curage ganglionnaire ne concerne pas les ganglions sus-claviculaires.
A - Est au contraire de meilleur pronostic qu'une tumeur centrale ou des quadrants internes qui envahissent plus rapidement
le médiastin.

1407
Exclusivement sur DOC - DZ : www.doc-dz.com NADJI 85
RESIDANAT EN POCHE TOME II
Cas Clinique en QCM
Un homme de 24 ans consulte pour l'apparition récente d'une dyspnée d'effort. L'interrogatoire retrouve la notion d'un
amaigrissement de 6 kg en 4 mois. L'examen cardio-pulmonaire est normal. Le reste de l'examen retrouve une masse
abdominale et une tuméfaction du testicule gauche. L'épididyme et le cordon sont normaux. Il existe une gynécomastie
bilatérale. Le toucher rectal est normal.

Quel est le diagnostic le plus probable de la maladie initiale :


A - Cancer bronchique anaplasique à petites cellules
B - Cancer du rein gauche avec métastases pulmonaires
C - Sarcome rétro-péritonéal
D - Lymphome non hodgkinien
E - Cancer non séminomateux du testicule
Bonne(s) réponse(s) : E

La palpation d'une masse testiculaire chez l'homme jeune, doit faire penser au cancer du testicule. La gynécomastie traduit
une sécrétion d'HCG, donc une tumeur non séminomateuse (qui inclut les tumeurs ayant un contingent séminomateux et un
autre contingent) ; ici il s'agit d'un choriocarcinome.
La masse abdominale fait suspecter une atteinte ganglionnaire, qui à gauche est le plus souvent lombo aortique pré-rénale.
La dyspnée d'effort évoque des métastases pulmonaires.

Parmi les examens suivants, le(s)quel(s) doit(doivent) être demandé(s) :


A - Radiographie du thorax
B - Dosage de la béta HCG
C - Dosage de l'ACE
D - Dosage de l'apha-foeto-protéine
E - Echotomographie abdominale
Bonne(s) réponse(s) : A B D E

A - Systématique dans le bilan d'extension (de plus, il existe une dyspnée !). Certains préfèrent actuellement le scanner
thoracique permettant de déceler des petites métastases, notamment des culs-de-sac pleuraux, difficilement visualisables sur
une radiographie standard.
B - Sécrétée par les choriocarcinomes, évoqués ici en raison de la gynécomastie.
C - Sans intérêt.
D - Dont l'augmentation traduirait l'existence d'un contingent tératomateux pouvant être dissocié
E - Remplacée ou associée au scanner abdominal recherchant adénopathies rétropéritonéales et métastases viscérales.
Le bilan est complété par une UIV.

Quelle est le geste indiqué pour affirmer le diagnostic :


A - Laparotomie exploratrice
B - Fibroscopie bronchique avec biopsies
C - Orchidectomie par voie haute
D - Biopsie rénale
E - Biopsie testiculaire transcrotale
Bonne(s) réponse(s) : C

Systématique et immédiate devant toute tuméfaction testiculaire. Seule la "voie haute", c'est à dire inguinale doit être utilisée
permettant la section haute du cordon, et le respect du scrotum. La biopsie transcrotale est à proscrire, du fait du risque de
greffe tumorale scrotale de mauvais pronostic.

Une chimiothérapie incluant du Cisplatinum® est instituée. Le(les) effet(s) secondaire(s) possible(s) est (sont) ?
A - Hépatite
B - Surdité
C - Insuffisance cardiaque
D - Insuffisance rénale
E - Sclérose pulmonaire
Bonne(s) réponse(s) : B D

Ce sont les 2 risques majeurs du cisplatinum.


La surdité est de perception, fonction de la dose cumulée.
Le risque d'insuffisance rénale doit faire pratiquer, pendant l'administration du cisplatinum, une diurèse forcée par perfusion de
sérum glucosé, Lasilix® et Mannitol®.

1408
Exclusivement sur DOC - DZ : www.doc-dz.com NADJI 85
RESIDANAT EN POCHE TOME II
Cas Clinique en QCM

Une fièvre avec frissons survient 10 jours après la fin de la cure de chimiothérapie. Quel examen faut-il
demander en priorité :
A - Prélèvement de gorge
B - Radiographie de thorax
C - Numération-formule sanguine
D - Ionogramme sanguin avec créatininémie
E - Examen cyto-bactériologique des urines
Bonne(s) réponse(s) : C

10 jours correspondent grossièrement au moment où l'aplasie médullaire éventuelle est la plus importante (Nadir). Cette
aplasie nécessiterait la mise en chambre stérile, l'antibiothérapie majeure, parfois des transfusions de leucocytes. Bien sûr,
les prélèvements bactériologiques complètent le bilan.
La NFS permet de distinguer 2 groupes de malades, selon l'aplasie ou non, distinction fondamentale pour le traitement.

Un homme de 72 ans vient consulter pour dysurie et douleurs lombaires. Le toucher rectal montre une augmentation de
volume de la prostate avec un nodule de 1,5 cm au niveau du lobe droit. Les radiographies de la colonne lombaire et du
bassin montrent un aspect dense de L4-L5 et de la région ilio-sacrée. Le taux des phosphatases acides prostatiques est de 45.

Quel est le diagnostic ?


A - Adénome de la prostate
B - Cancer de la prostate stade B
C - Cancer de la prostate stade C
D - Cancer de la prostate stade D1
E - Cancer de la prostate stade D2
Bonne(s) réponse(s) : E

Classification de Whitemore :
- stade A - cancer occulte, découvert sur pièce d'adenomectomie
- stade B 1 - nodule limité à l'intérieur de la glande touchant 1 seul lobe et/ou taille < 1 ,5 cm
B2 - Idem, touchant 2 lobes et/ou > 1,5 cm
- stade C - envahissement locorégional : pelvis, ganglions pelviens
- stade D - métastase
L'augmentation des phosphatases acides prostatiques est contemporaine des métastases osseuses.

A l'heure actuelle, le traitement de ce patient peut faire appel a :


A - OEstrogènes
B - Aminogluthétimide
C - Analogue de la LH-RH
D - Anti-androgènes
E - Progestatifs
Bonne(s) réponse(s) : A B C D E

La réponse E est à mettre entre parenthèse.


Le distilbène®, à la dose de 2-3 mg/jour est le plus utilisé. Cependant, la toxicité, notamment cardiovasculaire, avec risque de
thrombose est un facteur limitant. Son action est constante sur les symptômes urinaires, mais elle n'améliore pas la survie.
Son intérêt est donc palliatif.
Les antiandrogènes (acétate de cyprotérone-flutamide) sont moins toxiques mais moins efficaces que le DES.
Les analogues de la LH-RH donnent actuellement des résultats excellents, de plus en plus utilisé en première intention avant
le distilbène.
Les progestatifs sont parfois utilisés en dernière intention, peu efficaces, ils ont un effet de brève durée.

La possibilité de réponse à un traitement hormonal dans ce cas est de :


A - 10 % à 30 %
B - 40 % à 50 %
C - 50 % à 60 %
D - 70 % à 80 %
E - 90 % à 100 %
Bonne(s) réponse(s) : D

Cependant, il existe à terme un échappement au traitement qui nécessitera une autre hormonothérapie, dont l'efficacité sera
brève.

1409
Exclusivement sur DOC - DZ : www.doc-dz.com NADJI 85
RESIDANAT EN POCHE TOME II
Cas Clinique en QCM

La surveillance du traitement nécessite de disposer du résultat de :


A - Dosage des phosphatases acides prostatiques
B - Dosage de la testostérone
C - Dosage de la LH
D - Dosage de la FSH
E - Dosage des 17 céto-stéroïdes
Bonne(s) réponse(s) : A B C

Le dosage des phosphatases acides prostatiques permet de vérifier l'évolutivité métastatique.


L'oestrogénothérapie diminue le taux de sécrétion de la LH hypophysaire par rétro-controle négatif et diminue la testostérone
libre (celle qui est active).
En fait la surveillance est surtout clinique, vérifiant l'amendement des symptômes et la tolérance cardiovasculaire.

Une jeune femme de 20 ans découvre une adénopathie sus-claviculaire droite. Elle n'a pas d'antécédent connu, pas de
symptômes particuliers : apyrexie, poids stable. Le reste de l'examen est normal. Les radiographies thoraciques montrent un
élargissement du médiastin supérieur et antérieur.
L'hémogramme est normal, la vitesse de sédimentation est à 6 mm à la première heure. Un adénogramme est pratiqué et
montre une prédominance lymphocytaire avec quelques rares cellules volumineuses, à gros noyau bourgeonnant et nucléolé,
ayant les caractères de la cellule de Sternberg.

Parmi les explorations suivantes, la(les)quelle(s) vous semble(nt) indispensable(s) à ce stade des explorations ?
A - Médullogramme
B - Médiastinoscopie
C - Biopsie ganglionaire sus claviculaire
D - Laparotomie exploratrice avec splénectomie
E - Laparotomie exploratrice sans splénectomie
Bonne(s) réponse(s) : C

Le médullogramme est un examen cytologique. La biopsie du ganglion est indispensable pour connaître l'architecture du tissu.
Les autres examens sont évidemment injustifiés à ce stade d'établissement du diagnostic.

Le bilan d'extension anatomique doit en principe comporter un ou plusieurs des examens suivants : le(lesquels)
?
A - Lymphographie bipédieuse
B - Urographie intraveineuse
C - Fibroscopie gastrique
D - Scintigraphie osseuse
E - Immuno-électrophorèse du sérum
Bonne(s) réponse(s) : A

En fait, elle tend à être remplacée par le scanner abdominal. Le bilan, actuellement, comprend :
- examen clinique
- radiographie thoracique
- scanner abdominal et thoracique
- NFS avec plaquettes
- VS - électrophorèse des protides - fibrinémie
- dosage des phosphatases alcalines - transaminases bilirubine - gamma GT.
Tous ces examens sont systématiques. Ponction biopsie osseuse en cas de signes généraux (fièvre, sueurs, amaigrissement)
dans les stades III et IV. Les autres examens seront pratiqués en cas de signes d'appels.

L'(les) examen(s) précédent(s) n'a(ont) pas montré d'anomalie. Vous pouvez en fonction des données dont
vous disposez, classer cette maladie dans l'un des stades suivants : lequel ?
A - I Aa
B - II Aa
C - II Ab
D - II Bb
E - III Aa
Bonne(s) réponse(s) : B

Il existe une atteinte ganglionnaire médiastinale et sus-claviculaire : ce sont deux territoires non contigus du même côté du
diaphragme = stade II.
A = absence d'évolutivité clinique (ni fièvre, ni sueur, ni amaigrissement)
a = absence d'évolutivité biologique (VS<40).

1410
Exclusivement sur DOC - DZ : www.doc-dz.com NADJI 85
RESIDANAT EN POCHE TOME II
Cas Clinique en QCM

Un traitement polychimiothérapique associant Mechlorecthamine®, Oncovin®, Procarbazine® et Prednisone®


(M.O.P.P.) est entrepris. Les complications de ce traitement peuvent comporter :
A - Neuropathie périphérique
B - Vomissements
C - Insuffisance cardiaque
D - Hépatite médicamenteuse
E - Alopécie
Bonne(s) réponse(s) : A B E

La réponse E est à mettre entre parenthèse.


Par ailleurs, hypoplasie médullaire (surveillance hebdomadaire de la NFS) risque de stérilité par toxicité gonadique enfin,
risque leucémigène. L'alopécie est inconstante et transitoire.

Le traitement chimiothérapique est suivi d'une irradiation en mantelet.


Celle-ci doit délivrer :
A - 20 grays en 2 semaines
B - 40 grays en 4 semaines
C - 60 grays en 6 semaines
D - 40 grays en une semaine
E - 40 grays en une seule séance
Bonne(s) réponse(s) : B

L'irradiation en mantelet est réalisée par un faisceau prenant les ganglions cervicaux, sus-claviculaires, axillaires,
médiastinaux, en protégeant le larynx et une partie des poumons. Pendant l'irradiation (qui se fait en étalement classique et <
5 x 2 grays par semaine). La surveillance de la NFS doit être régulière étant donné l'irradiation d'une grande quantité de
moelle hématopoïétique.
Risques tardifs : asialie, fibrose des apex pulmonaires, péricardite, hypothyroïdie, caries, insuffisance coronaire à distance.

Le traitement optimal étant effectué, l'espérance de cette jeune patiente d'être en rémission persistante au bout
de 5 ans est de :
A - 10 %
B - 30 %
C - 50 %
D - 80 %
E - 100 %
Bonne(s) réponse(s) : D

Les stades I et II ont une survie à 5 ans de 90 %.


Facteurs de bon pronostic de stade I-II :
- "couplé" VS/signes généraux favorables ; c'est-à-dire A et Vs < 50 ou B et VS < 30
A et VS < 50 ou B et VS < 30
- stade II avec seulement 2 aires ganglionnaires atteintes, sans atteinte massive du médiastin
- la réponse à la chimiothérapie qui doit être complète (au mieux dès la première cure).

A quelle(s) complication(s) tardive(s) la patiente reste-t-elle exposée du fait du traitement ?


A - Hypothyroïdie
B - Péricardite
C - Zona
D - Leucémie aiguë
E - Diabète
Bonne(s) réponse(s) : A B C D

Complications de la chimiothérapie (MOPP) :


- précoces : hypoplasie médullaire, nausées-vomissements, polynévrite, alopécie
- tardives : leucémies aiguës, stérilité, l'immunodépression est lente à régresser (d'où le risque de zona).
Complications de la radiothérapie (en Mantelet) :
- précoce : asialie, dysphagie, alopécie occipitale transitoire, hypoplasie médullaire
- tardives : sclérose des apex (asymptomatique), asialie, hypothyroïdie, péricardite régressive, baisse de la capacité
respiratoire (régressive en 1 an), syndrome de Lhermitte, spontanément régressif.

1411
Exclusivement sur DOC - DZ : www.doc-dz.com NADJI 85
RESIDANAT EN POCHE TOME II
Cas Clinique en QCM

Un homme de 46 ans, fumeur (36 paquets-année), est hospitalisé pour hémoptysie. La radiographie pulmonaire révèle une
opacité qui vous évoque la possibilité d'un cancer bronchique.

Parmi les caractères suivants, on évoque un cancer primitif devant :


A - Présence d'un bronchogramme aérique
B - Opacité rétractile
C - Opacité inhomogène, non systématisée
D - Miliaire
E - Nodules disséminés dans les deux lobes supérieurs
Bonne(s) réponse(s) : B

L'opacité systématisée rétractile par compression proximale évoque dans ce cadre, un cancer, proximal. La réponse se fait
plutôt par élimination des autres items.

Parmi les examens suivants, quel est celui qui vous semble le plus urgent à réaliser ?
A - Tomographie de face
B - Recherche de bacille de Koch dans l'expectoration
C - Fibroscopie bronchique
D - Dosage de l'antigène carcinoembryonnaire
E - Scanner thoracique
Bonne(s) réponse(s) : C

Toute hémoptysie, surtout chez une femme, doit faire pratiquer une fibroscopie bronchique, permettant des biopsies et un
lavage bronchoalvéolaire.

Chez cet homme de 46 ans présentant une tumeur bourgeonnante de la bronche intermédiaire, quel type
histologique est le plus probable ?
A - Anaplasique à petites cellules
B - Adénocarcinome
C - Carcinoïde
D - Epidermoïde
E - Bronchiolo-alvéolaire
Bonne(s) réponse(s) : D

Le cancer épidermoïde touche presque exclusivement les grosses bronches, sous forme de bourgeonnement.
Le cancer anaplasique à petites cellules est médiastino-pulmonaire, avec nombreuses adénopathies médiastinales. La
radiographie montre souvent un élargissement du médiastin. La fibroscopie trouve une infiltration de la muqueuse, avec
compression extrinsèque.
L'adénocarcinome donne plus souvent une image ronde périphérique unique.

Vous décidez une exérèse chirurgicale qui, en raison de la situation tumorale, ne peut être qu'une
pneumonectomie. Parmi les éléments suivants, le(s) quel (s) contre-indique(nt) formellement ce geste ?
A - Artérite des membres inférieurs
B - Onde Q, séquelles d'infarctus postérieur sur l'électrocardiogramme
C - Adénopathie tumorale sus-claviculaire droite
D - Hépatomégalie homogène avec signes d'intoxication éthylique
E - Fibrillation auriculaire
Bonne(s) réponse(s) : C

ABE.- Sont des problèmes dont il faut tenir compte pour l'anesthésie, mais ne sont pas des contre-indications, si ces lésions
sont stabilisées.
D - Hépatomégalie n'est pas synonyme de métastase hépatique, ni de cirrhose.
C - L'existence d'une adénopathie sus-claviculaire témoigne d'une extension régionale. Le traitement chirurgical dans ces
conditions serait inutile car n'améliorant pas le pronostic.
Contre-indications à la chirurgie :
- tumeur à moins de 2 cm de la carène
- cancer anaplasique à petites cellules
- extension pleurale, costale, péricardique (qui peuvent être des découvertes opératoires)
- métastases ganglionnaires ou viscérales
- état général médiocre
- EFR. altérées (le traitement, en cas de contre-indication chirurgicale, fait appel à la radiothérapie).

1412
Exclusivement sur DOC - DZ : www.doc-dz.com NADJI 85
RESIDANAT EN POCHE TOME II
Cas Clinique en QCM
Un homme de 40 ans sans antécédent pathologique vient consulter pour une augmentation progressive et indolente du
volume de la bourse gauche. A l'examen il n'y a aucun signe inflammatoire sur le scrotum, le poids de la bourse gauche est
manifestement augmenté. Il existe des petits ganglions inguinaux bilatéraux. Le contenu intra-scrotal droit est normal.

En présence de cette tumeur intrascrotale, parmi les signes cliniques suivants, un seul justifie de penser
systématiquement au cancer du testicule :
A - Bourse opaque à la transillumination
B - Adénopathie inguinale
C - Perception des battements de l'artère spermatique
D - Epididyme normal
E - Possibilité de pincer les feuillets de la vaginale
Bonne(s) réponse(s) : D

Une grosse bourse isolée chez un homme jeune, avec palpation normale de l'épididyme doit faire-penser au cancer du
testicule.
Le drainage Iymphatique du testicule est lombo-aortique. Des adénopathies inguinales peuvent s'observer :
- en cas d'envahissement du scrotum
- s'il existe des antécédents chirurgicaux pour hernie inguinale ou pour testicule ectopique.

Indiquez par quel examen on parvient à affirmer le diagnostic de cancer du testicule :


A - Dosage des antigènes carcino-embryonnaires
B - Lymphographie bipédieuse
C - Cytoponction de la tumeur
D - Examen anatomo-pathologique après orchidectomie
E - Recherche des cellules anormales dans le sperme
Bonne(s) réponse(s) : D

La suspicion du diagnostic doit faire pratiquer un abord du testicule par voie haute. Jamais de biopsie transcrotale, ni
d'orchidectomie par voie scrotale du fait du risque de greffe néoplasique scrotale.
La cytologie du sperme n'a aucun intérêt. La Iymphographie ferait partie éventuellement du bilan d'extension (mais remplacée
par le scanner).

Parmi les dosages suivants, indiquez celui(ceux) qui présente(nt) un intérêt dans le cancer du testicule :
A - LDH
B - Gonadotrophines chorioniques Béta (HCG)
C - Alpha foeto-protéine
D - Testostérone
E - Phosphatases acides
Bonne(s) réponse(s) : B C

béta HCG : traduisent l'existence d'un contingent de choriocarcinome. Alphafoetoprotéine : traduit l'existence d'un contingent
de teratocarcinome.
Les associations sont possibles. L'élévation de ces marqueurs signent une tumeur non séminomateuse pure. Les
prélèvements pour recherche de marqueurs sont pratiqués :
- dans le sang, avant orchidectomie
- dans le cordon, pendant l'intervention puis, lors de la surveillance s'ils étaient élevés initialement.

En cas de tumeur séminomateuse pure sans envahissement ganglionnaire décelable sur l'imagerie
radiologique, le traitement comportera :
A - Orchidectomie avec ligature haute du cordon
B - Chimiothérapie
C - Curage ganglionnaire lombo-aortique
D - Irradiation des chaînes ganglionnaires lomboaortiques
E - Corticothérapie
Bonne(s) réponse(s) : A D

A - Est le premier geste, diagnostique et thérapeutique.


D - Le séminome est très radiosensible. Les doses nécessaires (25-35 Gys) sont parfaitement tolérées.
La lymphadénectomie lomboaortique est donc inutile, et source de complications plus fréquentes.

1413
Exclusivement sur DOC - DZ : www.doc-dz.com NADJI 85
RESIDANAT EN POCHE TOME II
Cas Clinique en QCM
Un patient de 60 ans vient consulter pour une dysphagie douloureuse sans régurgitation, s'aggravant depuis un mois. Celle-ci
est haut située, localisée au niveau de la partie gauche du cou et prédomine lors de la déglutition des solides. Il existe
simultanément une otalgie réflexe gauche lors de la déglutition. La voix est enrouée, mais aux dires du malade, la dysphonie
serait ancienne. Ce sujet,éthylo-tabagique, ne présente aucun passé pathologique notable. Il n'y a pas d'amaigrissement.
L'examen de la cavité buccale montre une denture en très mauvais état. La palpation cervicale objective une tuméfaction sous
digastrique gauche, mobile, de 2 cm de diamètre.

Parmi les éléments de ce dossier, l'origine pharyngée de cette dysphagie est évoquée par la constatation de :
A - Dysphagie pour les solides
B - Dysphagie douloureuse
C - Otalgie réflexe
D - Absence de régurgitation
E - Dysphonie ancienne
Bonne(s) réponse(s) : B C D

L'otalgie réflexe est caractéristique de l'origine pharyngée.


La dysphagie des localisations oesophagienne n'est pas douloureuse.
L'absence de régurgitation lors d'une dysphagie est en faveur d'une atteinte haut située.

Les éléments dont vous disposez vous permettent d'évoquer un diagnostic.


Quel est-il parmi les propositions suivantes :
A - Cancer de la corde vocale gauche
B - Cancer des deux cordes vocales
C - Cancer sous-glottique
D - Cancer de l'oesophage thoracique
E - Cancer du sinus piriforme
Bonne(s) réponse(s) : E

Les cancers des cordes vocales donnent des dysphonies isolées, et les adénopathies sont exceptionnelles, lors
d'envahissements importants du larynx.
Le cancer sus-glottique est un cancer du larynx donnant des signes respiratoires avec une voie diminuée d'amplitude. La
dysphagie est le fait d'envahissement postérieur oesophagien.
Ici, le terrain, la dysphagie haute, doivent faire rechercher un cancer pharyngé dont le plus fréquent est le cancer du sinus
piriforme (hypopharynx).

Quel est l'examen le plus simple pour objectiver cette lésion ?


A - Fibroscopie laryngo-trachéale
B - Xérographie du cou de profil
C - Tomographie de face du larynx
D - Pharyngo-laryngoscopie indirecte
E - Echographie
Bonne(s) réponse(s) : D

Le sinus piriforme est visible en laryngoscopie indirecte et peut être biopsié pour un diagnostic histologique indispensable. La
fibroscopie laryngo-trachéale est trop lourde pour faire le diagnostic, elle fera partie du bilan à la recherche d'un 2e cancer.
Les autres examens donnent une imagerie utile au bilan d'extension mais ne font en aucun cas le diagnostic de certitude.

Un examen plus approfondi montre que l'hémilarynx gauche est immobilisé, car infiltré par la tumeur. Quelle
attitude thérapeutique doit on envisager ?
A - Radiothérapie exclusive
B - Pharyngectomie exclusive
C - Pharyngo-laryngectomie totale avec évidement cervical ganglionnaire et radiothérapie post-opératoire
D - Chimiothérapie seule
E - Evidement cervical ganglionnaire et radiothérapie post opératoire
Bonne(s) réponse(s) : C

Dans la classification TNM, il s'agit d'un T3N0M0. La thérapeutique est lourde :


- pharyngo-laryngectomie totale, avec trachéotomie définitive, (nécessitant l'apprentissage d'une voie oesophagienne),
évidement ganglionnaire
- radiothérapie post-opératoire, à des doses élevées (65-70 Gys sur la tumeur primitive) en 6 à 7 semaines. L'irradiation
concerne tous les ganglions cervicaux, les creux sus-claviculaires et la tumeur.
Les problèmes posés lors de l'irradiation sont l'épithélite et la mucite. La cavité buccale doit être remise en état avant la
radiothérapie. (Actuellement, la chimiothérapie pré-opératoire est en cours d'étude).

1414
Exclusivement sur DOC - DZ : www.doc-dz.com NADJI 85
RESIDANAT EN POCHE TOME II
Cas Clinique en QCM
Un homme de 60 ans, sans autre antécédent qu'une intoxication alcoolique nettement ralentie, consulte pour une dysphagie
d'apparition récente. Il se plaint de douleurs lombaires depuis trois mois. La fibroscopie montre une lésion du tiers inférieur de
l'oesophage assez régulière qui n'a pu être franchie. A la biopsie : adénocarcinome. La biologie montre des phosphatases
alcalines à 420 UI/l (normale 180) une gamma GT et une bilirubine strictement normales. Les phosphatases acides et la
calcémie sont également normales.

Pour apprécier l'extension du processus tumoral, il est nécessaire de demander :


A - Transit oesogastroduodénal
B - Scintigraphie oesophagienne
C - Radiographie pulmonaire
D - Echographie hépatique
E - Lymphographie
Bonne(s) réponse(s) : A C D

Le bilan d'extension du cancer de l'oesophage comporte systématiquement :


- TOGD - fibroscopie oesogastrique
- fibroscopie trachéobronchique
- examen ORL - radio pulmonaire
- echographie hépatique ou scanner abdominal (meilleur car visualise d'éventuelles adénopathies coeliaques).

Chez ce malade compte-tenu des données fournies, l'augmentation des phosphatases alcalines est évocatrice
de :
A - Métastase hépatique
B - Alcoolisme chronique
C - Métastase osseuse
D - Cancer de la prostate
E - Hyperparathyroïdie
Bonne(s) réponse(s) : A

Sans commentaire : évident.

Quels sont le(les) examen(s) à demander pour préciser la cause des douleurs lombaires chez ce patient :
A - Radiographie du rachis
B - Scintigraphie osseuse
C - Biopsie de prostate
D - Electrophorèse des protéines
E - Urographie intra-veineuse
Bonne(s) réponse(s) : A B

La scintigraphie permet de visualiser les zones fixantes donc en activité métabolique. Sa positivité est plus précoce que les
images radiologiques.
Les radiographies du rachis centrées sur la zone douloureuse sont systématiques car les troubles peuvent être indépendants
du cancer.

Parmi les méthodes thérapeutiques suivantes non chirurgicales, la dysphagie de ce malade est susceptible
d'être améliorée par :
A - Radiothérapie du bas oesophage
B - Chimiothérapie par cisplatinum
C - Mise en place d'une prothèse oesophagienne sous endoscopie
D - Chimiothérapie par le 5 fluoro-uracile
E - Traitement endoscopique par le laser
Bonne(s) réponse(s) : A B C E

C E - Sont palliatives contre la dysphagie.


B - A montré une efficacité dans le cancer de l'oesophage, sans gain de survie
A - Traitement des malades inopérables.

1415
Exclusivement sur DOC - DZ : www.doc-dz.com NADJI 85
RESIDANAT EN POCHE TOME II
Cas Clinique en QCM
Vous dépistez chez une femme de 65 ans, en bon état général sans autre maladie associée, un carcinome de l'endomètre,
strictement limité au corps utérin.

Parmi les facteurs de risque, vous retenez :


A - Obésité
B - Nulliparité
C - Traitement aux oestrogènes
D - Multiplicité de partenaires
E - Grossesses multiples
Bonne(s) réponse(s) : A C

Voir facteurs de risque, qui sont en fait, tout ce qui augmente l'exposition aux oestrogènes.

La fréquence de ce cancer chez la femme :


A - Est en voie d'augmentation
B - Est en voie de diminution
C - Correspond au 5ème cancer le plus fréquent chez la femme
D - Est constante
E - Correspond au cancer gynécologique le moins fréquent chez la femme
Bonne(s) réponse(s) : A

La fréquence augmente avec l'augmentation de la longévité et du niveau économique.

A quel stade est-elle vue ?


A - Stade 0
B - Stade 1
C - Stade 2
D - Stade 3
E - Stade 4
Bonne(s) réponse(s) : B

T1NoMo.
T1 : tumeur limitée au corps de l'utérus.
T1 a : hystérométrie < 8 cm.
T1 b : hystérométrie > 8 cm.

Le traitement sera :
A - Chimiothérapie pré-opératoire + hystérectomie
B - Radiothérapie + chimiothérapie
C - Hystérectomie avec annexectomie
D - Hystérectomie + chimiothérapie
E - Curetage + hormonothérapie
Bonne(s) réponse(s) : C

Qui sera suivie d'une curithérapie vaginale si la lésion atteint l'isthme.


La radiothérapie post-opératoire n'est pas admise par tous.

L'examen histologique révèle un envahissement des 2/3 internes du myomètre. Faut-il faire un traitement
complémentaire ?
Si oui, lequel ?
A - Non
B - Chimiothérapie
C - Hormonothérapie
D - Chimiothérapie + hormonothérapie
E - Radiothérapie pelvienne
Bonne(s) réponse(s) :

Question annulée.

1416
Exclusivement sur DOC - DZ : www.doc-dz.com NADJI 85
RESIDANAT EN POCHE TOME II
Cas Clinique en QCM

La surveillance sera faite essentiellement par :


A - Radiographie du thorax
B - Examens gynécologiques répétés
C - Echographie hépatique
D - Examens biologiques
E - Scanner pelvien
Bonne(s) réponse(s) : B

C'est une tumeur d'évolution essentiellement loco-régionale. Le scanner pelvien se discute, mais ici, il s'agit d'une petite
lésion, survie supérieure à 80 % à 5 ans.

Cette patiente de 42 ans consulte pour des douleurs lombaires droites irradiant à la fesse et à la face externe de la cuisse. A
l'examen, on note un point douloureux paravertébral en regard de L3 et au niveau de l'épine iliaque postéro-supérieure droite.
Il n'y a pas de signe de Lassègue. Le reste de l'examen est normal. La patiente a été traitée trois ans auparavant pour un
adénocarcinome du sein droit par mastectomie élargie, suivie de radiothérapie.

L'examen le plus sensible pour faire le diagnostic de métastase osseuse est :


A - Scintigraphie du corps entier au Calcium 125
B - Scintigraphie du corps entier aux polyphosphates marquées au Technetium
C - Radiographie du squelette
D - Tomographie du rachis
E - Tomodensitométrie vertébrale
Bonne(s) réponse(s) : B

Evident.

L'examen biologique destiné à apprécier le risque métabolique de la maladie métastatique osseuse est :
A - Calcémie
B - Numération formule
C - Phosphatases acides
D - Antigène carcino-embryonnaire
E - Electrophorèse des protéines
Bonne(s) réponse(s) : A

En effet l'hypercalcémie peut provoquer des signes neurologiques (confusion, coma) et des signes cardiaques
(hyperexcitabilité myocardique).

Le bilan d'extension des métastases dans les autres organes cibles comporte RP (radio pulmonaire) ACE
(Antigène-Carcino-Embryonnaire) :
A - Scintigraphie hépatique + ACE + RP
B - Scintigraphie cérébrale + RP + ACE
C - Echotomographie pelvienne + RP + Scanner hépatique
D - Mammographie du sein restant + RP + ACE
E - Echographie hépatique + RP + ACE
Bonne(s) réponse(s) : E

La scintigraphie hépatique et/ou cérébrale en 1987 sont des propositions peu plausibles.

Au cours d'une castration chirurgicale, une métastase est découverte sur l'ovaire droit et une biopsie confirme la
métastase et la présence de récepteurs hormonaux dans les cellules tumorales. Le traitement hormonal
complémentaire sera :
A - Pas de traitement complémentaire
B - Corticothérapie (2 mg/kg)
C - Tamoxifène
D - Anti-prolactimiques
E - OEstrogènes à faibles doses
Bonne(s) réponse(s) : C

Le Nolvadex® (Tamoxifène) est un antioestrogène agissant par intéraction avec les récepteurs d'oestrédiol.

1417
Exclusivement sur DOC - DZ : www.doc-dz.com NADJI 85
RESIDANAT EN POCHE TOME II
Cas Clinique en QCM

La patiente continue à souffrir de lombalgies un mois après l'intervention, le traitement proposé est alors :
A - Epiphysiolyse
B - Immunothérapie par le BCG
C - Surrénalectomie chimique
D - Radiothérapie sur la colonne lombaire
E - Antalgiques type morphine
Bonne(s) réponse(s) : D

On peut proposer une radiothérapie antalgique centrée sur le ou les vertèbres lombaires douloureuses.

Une chimiothérapie est envisagée. Quelle combinaison de drogues est habituellement utilisée :
A - 5FU - Endoxan® - Cis Platine®
B - Cyclophosphamide - Oncovin® - Bléomycine
C - Adriamyne - 5 Fluoro-Uracile - Cyclophosphamide
D - Nitrosoure - Bleomycine
E - Aucune des propositions ci-dessus
Bonne(s) réponse(s) : C

Il s'agit du classique FAC.

Un homme de 45 ans, consulte pour une sensation de pesanteur au niveau de la bourse droite. L'examen clinique permet de
percevoir un testicule droit tumoral. Une intervention chirurgicale est décidée, et l'examen anatomo-pathologique de la pièce
opératoire est en faveur du diagnostic de séminome.

Dans un cas semblable, la stratégie thérapeutique dépend de :


A - Type histologique de la tumeur
B - Degré d'extension ganglionnaire
C - Etat des marqueurs tumoraux
D - Extension scrotale
E - La voie d'abord chirurgicale
Bonne(s) réponse(s) : A B C

A B - Evident.
C - Ici, l'examen histologique doit être complété par la recherche de marqueurs bétaHCG et alpha foeto-protéine, dans le sang
périphérique et dans le sang du cordon, prélevé lors de l'interventio. L'augmentation franche de l'un des marqueurs signe une
tumeur "non séminome pur", dont le traitement diffère radicalement.

Parmi les attitudes suivantes, quelle est celle qui doit être systématiquement réalisée en premier ?
A - Biopsie testiculaire par voie scrotale
B - Orchidectomie simple
C - Orchidectomie par voie inguinale avec ligature haute du cordon
D - Ponction testiculaire transcutanée
E - Dosage des Bêta HCG et des alpha foetoprotéines
Bonne(s) réponse(s) : E

Le dosage des marqueurs doit être fait avant toute intervention pour suspicion de cancer testiculaire, servant de référence
pour la surveillance ultérieure.

Pour préciser le degré d'extension, il est essentiel de demander :


A - Examen clinique
B - Lymphographie pédieuse
C - Scannographie thoraco-abdominale
D - Scintigraphie osseuse
E - Echographie hépatique
Bonne(s) réponse(s) : A B C E

Bien que le scanner soit de plus en plus à même d'étudier les ganglions abdominaux aussi bien ou mieux que la
Iymphographie. De même, l'échographie du foie et scanner abdominal font double emploi.

1418
Exclusivement sur DOC - DZ : www.doc-dz.com NADJI 85
RESIDANAT EN POCHE TOME II
Cas Clinique en QCM

Le diagnostic de séminome pur ne peut être affirmé qu'après avoir obtenu le résultat de :
A - Examen histologique de la tumeur
B - Dosage des Bêta HCG
C - Dosage de l'antigène carcino-embryonnaire
D - Dosage de l'alpha-foetoprotéine
E - Dosage de la fraction 1 des LDH
Bonne(s) réponse(s) : A B D

En effet, l'examen histologique ne suffit pas, les contingents cellulaires pouvant être de plusieurs natures.
Ainsi, l'augmentation d'un marqueur alors que l'histologie conclue à un séminome signe la présence de cellules non
séminomateuses, et le traitement est celui des tumeurs non séminomateuses.

Si le diagnostic de séminome pur sans extension métastatique est retenu, l'attitude consistera, après
orchidectomie, en l'une des propositions suivantes :
A - Chirurgie ganglionnaire
B - Polychimiothérapie
C - Irradiation iliaque homolatérale et lombo-aortique
D - Hormonothérapie
E - Abstention thérapeutique
Bonne(s) réponse(s) : C

Le séminome est très radiosensible. Des doses de 25 Gys sont suffisantes pour les ganglions iliaques et lombo-aortiques
dans les stades I.

Une malade de 54 ans, sans antécédent personnel, familial ni professionnel, traitée 2 ans auparavant pour un cancer du sein
localisé par association radio-chirurgicale, est admise en urgence pour altération de l'état général avec obnubilation, nausées,
dyspnée et rachialgies d'apparition récente. Il n'existe pas de troubles de la marche ni de troubles sphinctériens. Elle ne
présente pas de céphalées ni d'hyperthermie : le fond d'oeil est normal.

Quel examen paraclinique demandez-vous en urgence ?


A - Dosage de la calcémie
B - Bilan biologique hépatique
C - Radiographie de thorax
D - Radiographies du rachis
E - Vitesse de sédimentation
Bonne(s) réponse(s) : A

Une hypercalcémie est vraisemblablement à l'origine de l'obnubilation.

La radiographie de thorax objective une image en lâcher de ballons. Quel(s) examen(s) pratiquez-vous pour
confirmer l'origine métastatique ?
A - Aucun
B - Fibroscopie bronchique
C - Tomographie pulmonaire
D - Thoracotomie pour exérèse d'un nodule
E - Médiastinoscopie
Bonne(s) réponse(s) : A

Aucun examen n'est nécessaire devant ce lâcher de ballon, n'ayant pas de diagnostic différentiel.

Si l'examen clinique est en faveur d'une compression médullaire confirmée radiologiquement, que proposez-
vous comme traitement en urgence avant la chirurgie ?
A - Chimiothérapie
B - Anti-oestrogènes
C - Corticothérapie à fortes doses
D - Androgènes
E - Progestatifs
Bonne(s) réponse(s) : C

Sans commentaire.

1419
Exclusivement sur DOC - DZ : www.doc-dz.com NADJI 85

S-ar putea să vă placă și